Sie sind auf Seite 1von 2031

A 7 year old with a b - IV ativan is the

Answer Key history of tonic-clonic


seizures has been
benzodiazepine of choice
for treating prolonged
actively seizing for 10 seizure activity. IV
Question 1 See full question minutes. The child benzodiazepines
weighs 22 kg and potentiate the action of
A nurse is teaching the
currently has an the gamma-aminobutyric
parents of a child with cystic intravenous (IV) line of acid (GABA)
fibrosis about proper D5 1/2 NS + 20 meq neurotransmitter,
nutrition. Which instruction KCL/L running at 60 stopping seizure activity.
should the nurse include? ml/hr. Vital signs are a If an IV line is not
temperature of 38 available, rectal Diastat is
You Selected: degrees C, heart rate the benzodiazepine of
of 120, respiratory rate choice. The child does
Encourage a high-calorie, of 28, and oxygen have a low-grade fever;
high-protein diet.
saturation of 92%. however, this is likely
Correct response: Using the SBAR caused by the excessive
(Situation-Background- motor activity. The
Encourage a high-calorie, Assessment- primary goal for the child
high-protein diet. Recommendation) is to stop the seizure in
Explanation: technique for order to reduce
communication, the neurologic damage.
Question 2 See full question nurse calls the primary Benzodiazepines are used
healthcare provider for the initial treatment of
A nurse is providing care for a with a recommendation prolonged seizures. Once
pregnant client. The client for: the seizure has ended, a
asks the nurse how she can loading dose of
best deal with her fatigue. a) Rectal diazepam fosphenytoin or
(Diastat). phenobarbital is given.
The nurse should instruct her
b) IV lorazepam
to: (Ativan).
c) Rectal
You Selected:
acetaminophen
try to get more rest by (Tylenol).
going to bed earlier. d) IV fosphenytoin.

Correct response:

try to get more rest by


going to bed earlier.
Explanation:

Question 3 See full question

Which expected outcome


about nutrition would be
appropriate for a client who
has had a total gastrectomy
for gastric cancer? The client
will:
You Selected:

maintain adequate nutrition


through oral or parenteral
feedings.
Correct response:

maintain adequate nutrition


through oral or parenteral
feedings.
Explanation:

Question 4 See full question

A client with diabetes is


explaining to the nurse how
he cares for the feet at home.
Which statement indicates
the client needs further
instruction on how to care for
the feet properly?
You Selected:

I inspect my feet once a


week for cuts and
redness.
Correct response:

I inspect my feet once a


week for cuts and
redness.
Explanation:

Question 5 See full question

A 7-year-old has had an


appendectomy on November
12. He has had pain for the
last 24 hours. There is a
prescription to administer
acetaminophen with codeine
every 3 to 4 hours as needed.
The nurse is beginning the
shift, and the child is
requesting pain medication.
The nurse reviews the chart
below for pain history. Based
on the information in the
medical record, what should
the nurse do next?
You Selected:

Assess the child again in 1


hour.
Correct response:

Administer the
acetaminophen with
codeine.
Explanation:

Question 1 See full question

Following a precipitous birth,


examination of the client's
vagina reveals a fourth-
degree laceration. Which
intervention is appropriate
when caring for this client?
You Selected:

Instructing the client to use


two or more perineal
pads to cushion the area
Correct response:

Instructing the client about


the importance of
perineal (Kegel) exercises
Explanation:

Question 2 See full question

The nurse administers a tap


water enema to a client.
While the solution is being
infused, the client has
abdominal cramping. What
should the nurse do first?
You Selected:

Clamp the tubing, and


carefully withdraw the
tube.
Correct response:

Temporarily stop the


infusion, and have the
client take deep breaths.
Explanation:

Question 3 See full question

The client has had a


myocardial infarction, and
the nurse has instructed the
client to prevent Valsalvas
maneuver. The nurse
determines the client is
following the instructions
when the client:
You Selected:

avoids holding the breath


during activity.
Correct response:

avoids holding the breath


during activity.
Explanation:

Question 4 See full question

After a radical prostatectomy


for prostate cancer, a client
has an indwelling catheter
removed. The client then
begins to have periods of
incontinence. During the
postoperative period, which
intervention should be
implemented first?
You Selected:

Artificial sphincter use


Correct response:

Kegel exercises
Explanation:

Question 5 See full question

The client with hepatitis A is


experiencing fatigue,
weakness, and a general
feeling of malaise. The client
tires rapidly during morning
care. The most appropriate
goal for this client is to:
You Selected:

learn new self-care skills.


Correct response:

gradually increase activity


tolerance.
Explanation:

1.

2. A 10-month-old child b - although a cool air


has cold symptoms. vaporizer may be
The mother asks how recommended to
she can clear infant's humidify the
nose. Which of the environment, using saline
following would be the nose drops and then a
nurse's best bulb syringe before meals
recommendation? and at nap and bed times
will allow the child to
a. use a cool air breathe more easily.
vaporizer with plain Saline helps to loosen
water secretions and keep the
b. use saline nose mucous membranes
drops and then a bulb moist. The bulb syringe
syringe then gently aids in
c. blow into the child's removing the loosened
mouth to clear the secretions. Blowing into
nose the child's mouth to clear
d. administer a the nose introduces more
nonprescription organisms to the child. A
vasoconstrictive nose nonprescription
spray. vasoconstrictive nasal
spray is not
recommended for infants
because if the spray in
used for longer than 3
days a rebound effect
with increased
inflammation occurs.
3. A 10-month-old child b - In a young child, the
with recurrent otitis eustachian tube is
media is brought to the relatively short, wide, and
clinic for evaluation. To horizontal, promoting
help determine the drainage of secretions
cause of the child's from the nasopharynx
condition, the nurse into the middle ear.
should ask the parents: Therefore, asking if the
child takes a bottle to bed
a) "Does water ever is appropriate because
get into the baby's drinking while lying down
ears during may cause fluids to pool
shampooing?" in the pharyngeal cavity,
b) "Do you give the increasing the risk of
baby a bottle to take to otitis media. Asking if the
bed?" parent noticed earwax, or
c) "Have you noticed a cerumen, in the external
lot of wax in the baby's ear canal is incorrect
ears?" because wax doesn't
d) "Can the baby promote the development
combine two words of otitis media. During
when speaking?" shampooing, water may
become trapped in the
external ear canal by
large amounts of
cerumen, possibly
causing otitis external
(external ear
inflammation) as opposed
to internal ear
inflammation. Asking if
the infant can combine
two words is incorrect
because a 10-month-old
child isn't expected to do
so.
4. A 10-year-old child a) CORRECT ANSWER
diagnosed with acute Taking vital signs every 4
glomerulonephritis is hours and obtaining daily
admitted to the weight
pediatric unit. The Reason: Because major
nurse should ensure complications such as
that which action is a hypertensive
part of the child's care? encephalopathy, acute
renal failure, and cardiac
a) Taking vital signs decompensation can
every 4 hours and occur, monitoring vital
obtaining daily weight signs (including blood
b) Obtaining a blood pressure) is an important
sample for electrolyte measure for a child with
analysis every acute glomerulonephritis.
morning Obtaining daily weight
c) Checking every urine and monitoring intake
specimen for protein and output also provide
and specific gravity evidence of the child's
d) Ensuring that the fluid balance status.
child has accurate Sodium and water
intake and output and restrictions may be
eats a high-protein diet ordered depending on the
severity of the edema
and the extent of
impaired renal function.
Typically, protein intake
remains normal for the
child's age and is only
increased if the child is
losing large amounts of
protein in the urine.
Checking urine
specimens for protein and
specific gravity and daily
monitoring of serum
electrolyte levels may be
done, but their frequency
is determined by the
child's status. These
actions are less important
nursing measures in this
situation.
5. A 10-year-old with b) CORRECT ANSWER
glomerulonephritis Obtain the child's blood
reports a headache and pressure.
blurred vision. The Reason: Hypertension
nurse should occurs with acute
immediately: glomerulonephritis. The
symptoms of headache
a) Put the client to and blurred vision may
bed. indicate an elevated
b) Obtain the child's blood pressure.
blood pressure. Hypertension in acute
c) Notify the glomerulonephritis occurs
physician. due to the inability of the
d) Administer kidneys to remove fluid
acetaminophen and sodium; the fluid is
(Tylenol). reabsorbed, causing fluid
volume excess. The nurse
must verify that these
symptoms are due to
hypertension. Calling the
physician before
confirming the cause of
the symptoms would not
assist the physician in his
treatment. Putting the
client to bed may help
treat an elevated blood
pressure, but first the
nurse must establish that
high blood pressure is the
cause of the symptoms.
Administering Tylenol for
high blood pressure is not
recommended.
6. A 16-year-old d) CORRECT ANSWER "It
academically gifted sounds like you have a
boy is about to real concern about
graduate from high transitioning to college. I
school early, because can refer you to a health
he has completed all care provider for
courses needed to earn assessment and
a diploma. Within the treatment."
last 3 months, he has Reason: The client's
experienced panic concerns are real and
attacks that have serious enough to
forced him to leave warrant assessment by a
classes early and physician rather than
occasionally miss a day being dismissed as trivial.
of school. He is Though he is very
concerned that these intelligent, his
attacks may hinder his intelligence cannot
ability to pursue a overcome his anxiety. In
college degree. What fact, his anxiety is likely
would be the best to interfere with his
response by the school ability to perform in
nurse who has been college if no assessment
helping him deal with and treatment are
his panic attacks? received. Just postponing
college is likely to
a) "It is natural to be increase rather than
worried about going lower the client's anxiety,
into a new because it does not
environment. I am sure address the panic he is
with your abilities you experiencing.
will do well once you
get settled."
b) "You are putting too
much pressure on
yourself. You just need
to relax more and
things will be alright."
c) "It might be best for
you to postpone going
to college. You need to
get these panic attacks
controlled first."
d) "It sounds like you
have a real concern
about transitioning to
college. I can refer you
to a health care
provider for
assessment and
treatment."
7. A 16-year-old d) CORRECT ANSWER
primigravida at 36 Admit the client to a
weeks' gestation who quiet, darkened room.
has had no prenatal Reason: Because of her
care experienced a age and report of a
seizure at work and is seizure, the client is
being transported to probably experiencing
the hospital by eclampsia, a condition in
ambulance. Which of which convulsions occur
the following should in the absence of any
the nurse do upon the underlying cause.
client's arrival? Although the actual cause
is unknown, adolescents
a) Position the client in and women older than 35
a supine position. years are at higher risk.
b) Auscultate breath The client's environment
sounds every 4 hours. should be kept as free of
c) Monitor the vital stimuli as possible. Thus,
signs every 4 hours. the nurse should admit
d) Admit the client to a the client to a quiet,
quiet, darkened room. darkened room. Clients
experiencing eclampsia
should be kept on the left
side to promote placental
perfusion. In some cases,
edema of the lungs
develops after seizures
and is a sign of
cardiovascular failure.
Because the client is at
risk for pulmonary
edema, breath sounds
should be monitored
every 2 hours. Vital signs
should be monitored
frequently, at least every
hour.
8. A 40-year-old executive C - A situational crisis
who was unexpectedly results from a specific
laid off from work 2 event in the life of a
days earlier complains person who is
of fatigue and an overhwlemed by the
inability to cope. He situation and reacts
admits drinking emotional. Fatigue,
excessively over the insomnia, and inability to
previous 48 hours. This make decisions are
behavior is an example common signs and
of: symtpoms. The situations
crisis may precipitate
a. alcoholism behavior that causes a
b. manic episode criss (alcohol or drug
c. situational crisis abuse). There isn't
d. depression enough information to
label this client an
alcoholic. A manic
episodes is characterized
by euphoria and labile
effect. Symptoms of
depression are usually
present for 2 or more
weeks.
9. A 56-year-old client is receiving d - Cough and shortness of breath are
chemotherapy that has the significant symptoms because they may
potential to cause pulmonary indicate decreasing pulmonary function
toxicity. Which of the following secondary to drug toxicity. Decrease in
symptoms indicates a toxic appetite, difficulty in thinking clearly, and
response to the chemotherapy? spasms of the diaphragm may occur as a
result of chemotherapy; however, they are
a) Decrease in appetite. not indicative of pulmonary toxicity.
b) Drowsiness.
c) Spasms of the diaphragm.
d) Cough and shortness of breath.
1 After discussing asthma as a c) CORRECT ANSWER "Although our child's
0. chronic condition, which of the disease is serious, we try not to let it be the
following statements by the father focus of our family."
of a child with asthma best reflects Reason: Positive adjustment to a chronic
the family's positive adjustment to condition requires placing the child's illness in
this aspect of the child's disease? its proper perspective. Children with asthma
need to be treated as normally as possible
a) "We try to keep him happy at all within the scope of the limitations imposed by
costs; otherwise, he has an asthma the illness. They also need to learn how to
attack." manage exacerbations and then resume as
b) "We keep our child away from normal a life as possible. Trying to keep the
other children to help cut down on child happy at all costs is inappropriate and
infections." can lead to the child's never learning how to
c) "Although our child's disease is accept responsibility for behavior and get
serious, we try not to let it be the along with others. Although minimizing the
focus of our family." child's risk for exposure to infections is
d) "I'm afraid that when my child important, the child needs to be with his or
gets older, he won't be able to care her peers to ensure appropriate growth and
for himself like I do." development. Children with a chronic illness
need to be involved in their care so that they
can learn to manage it. Some parents tend to
overprotect their child with a chronic illness.
This overprotectiveness may cause a child to
have an exaggerated feeling of importance or
later, as an adolescent, to rebel against the
overprotectiveness and the parents.
1 After staying several hours with her b - A 9-year-old child should be able to
1. 9-year-old daughter who is tolerate being alone. Frequently asking for
admitted to the hospital with an someone to be in the room indicates a degree
asthma attack, the mother leaves to of psychological distress that, at this age,
attend to her other children. The suggests anxiety. The inability to get
child exhibits continued signs and comfortable is more characteristic of a child
symptoms of respiratory distress. in pain. Inability to answer questions correctly
Which of the following findings may reflect a state of anoxia or a lack of
should lead the nurse to believe the knowledge. Tightness in the chest occurs as a
child is experiencing anxiety? result of bronchial spasms.

a) Not able to get comfortable.


b) Frequent requests for someone
to stay in the room.
c) Inability to remember her exact
address.
d) Verbalization of a feeling of
tightness in her chest.
1 After surgery for an ileal conduit, b - After pelvic surgery, there is an increased
2. the nurse should closely assess chance of thrombophlebitis owing to the pelvic
the client for the occurrence of manipulation that can interfere with circulation
which of the following and promote venous stasis. Peritonitis is a
complications related to pelvic potential complication of any abdominal surgery,
surgery? not just pelvic surgery. Ascites is most frequently
an indication of liver disease. Inguinal hernia
a) Peritonitis. may be caused by an increase in intra-
b) Thrombophlebitis. abdominal pressure or a congenital weakness of
c) Ascites. the abdominal wall; ventral hernia occurs at the
d) Inguinal hernia. site of a previous abdominal incision.
1 An anxious young adult is c) CORRECT ANSWER Recommend a pregnancy
3. brought to the interviewing room test after acknowledging the client's distress.
of a crisis shelter, sobbing and Reason: Before any interventions can occur,
saying that she thinks she is knowing whether the client is pregnant is crucial
pregnant but does not know what in formulating a plan of care. Asking the client
to do. Which of the following about what things she had thought about doing,
nursing interventions is most giving the client some ideas about what to
appropriate at this time? expect next, and questioning the client about
her feelings and possible parental reactions
a) Ask the client about the type would be appropriate after it is determined that
of things that she had thought of the client is pregnant.
doing.
b) Give the client some ideas
about what to expect to happen
next.
c) Recommend a pregnancy test
after acknowledging the client's
distress.
d) Question the client about her
feelings and possible parental
reactions.
1 Based on a client's history of d) CORRECT ANSWER Verbalization of her
4. violence toward others and her feelings in an appropriate manner.
inability to cope with anger, Reason: Verbalizing feelings, especially feelings
which of the following should the of anger, in an appropriate manner is an
nurse use as the most important adaptive method of coping that reduces the
indicator of goal achievement chance that the client will act out these feelings
before discharge? toward others. The client's ability to verbalize
her feelings indicates a change in behavior, a
a) Acknowledgment of her angry crucial indicator of goal achievement. Although
feelings. acknowledging feelings of anger and describing
b) Ability to describe situations situations that precipitate angry feelings are
that provoke angry feelings. important in helping the client reach her goal,
c) Development of a list of how they are not appropriate indicators that she has
she has handled her anger in the changed her behavior. Asking the client to list
past. how she has handled anger in the past is helpful
d) Verbalization of her feelings in if the nurse discusses coping methods with the
an appropriate manner. client. However, based on this client's history,
this would not be helpful because the nurse and
client are already aware of the client's
aggression toward others.
1 Before an incisional c) CORRECT ANSWER The operative incision is near
5. cholecystectomy is the diaphragm.
performed, the nurse Reason: The incisions made for upper abdominal
instructs the client in the surgeries, such as cholecystectomies, are near the
correct use of an incentive diaphragm and make deep breathing painful.
spirometer. Why is incentive Incentive spirometry, which encourages deep
spirometry essential after breathing, is essential to prevent atelectasis after
surgery in the upper surgery. The client is not maintained on bed rest for
abdominal area? several days. The client is encouraged to ambulate
by the first postoperative day, even with drainage
a) The client will be tubes in place. Nasogastric tubes do not inhibit deep
maintained on bed rest for breathing and coughing.
several days.
b) Ambulation is restricted by
the presence of drainage
tubes.
c) The operative incision is
near the diaphragm.
d) The presence of a
nasogastric tube inhibits deep
breathing.
1 Before discharge, which b - the nurse should instruct the client to notify the
6. instruction should a nurse physician if his heart rate is greater than 90 beats/
give to a client receiving minute because cardiac arrhythmias may occur with
digoxin (Lanoxin)? digoxin toxicity. To prevent toxicity, the nurse should
instruct the client never to take an extra dose of
a. "take an extra dose of digoxin if he misses a dose. The nurse should show
digoxin if you miss one the client how to take his pulse and tell him to call
dose." the physician if his pulse rate drops below 60
b. "call the physician if your beats/minute - not 80 beats/minute, which is a
heart rate is above 90 normal pulse rate and doesn't warrant action. The
beats/minute client shouldn't take digoxin with meals; doing so
c. "call the physician if your slows the absorption rate.
pulse drops below 80
beats/minute."
d. "take digoxin with meals."
1 The best indicator that the a - the nurse should judge that learning has
7. client has learned how to give occurred from evidence of a change I the client's
an insulin self-injection behavior. A client who performs a procedure safely
correctly is when the client and correctly demonstrates that he has acquired a
can: skill. Evaluation of this skill acquisition requires
performance of that skill by the client with
a. perform the procedure observation by the nurse. The client must also
safely and correctly demonstrate cognitive understanding, as shown by
b. critique the nurse's the ability to critique the nurse's performance.
performance of the procedure Explaining the steps demonstrates of knowledge at
c. explain all of the steps of the cognitive level only. A post-test does not indicate
the procedure correctly the degree to which the client has learned a
d. correctly answer a post- psychomotor skill.
test about the procedure
1 A child, age 3, is brought to the a - a child with acute epiglottiditis appears
8. emergency department in acutely ill and clinical manifestations may
respiratory distress caused by include drooping (because of difficulty
acute epiglottiditis. Which clinical swallowing), severe sore throat, hoarseness, a
manifestations should the nurse high temperature, and severe inspiratory
expect to assess? stridor. A low grade fever, stridor, and barking
cough that worsens at night are suggestive of
a. severe sore throat, drooling, croup. Pulmonary congestion, productive
and inspiratory strider cough, and fever along with nasal flaring,
b. low grade fever, stridor, and a retractions, chest pain, dyspnea, decreased
barking cough breath sounds, and crackles indicate
c. pulmonary congestion, a pneumococcal pneumonia. A sore throat,
productive cough, and a fever fever, and general malaise point to viral
d. sore throat, a fever, and general pharyngitis.
malaise
1 A child with a poor nutritional c) CORRECT ANSWER Total protein
9. status and weight loss is at risk for Reason: The nurse anticipates the physician
a negative nitrogen balance. To will order a total protein test because negative
help diagnose this problem, the nitrogen balance may result from inadequate
nurse anticipates that the protein intake. Measuring total iron-binding
physician will order which capacity and Hb levels would help detect iron
laboratory test? deficiency anemia, not a negative nitrogen
balance. The sweat test helps diagnose cystic
a) Total iron-binding capacity fibrosis, not a negative nitrogen balance.
b) Hemoglobin (Hb)
c) Total protein
d) Sweat test
2 a client comes to the outpatient b - gonorrhea must be reported to the public
0. department complaining of vaginal health department. Bacterial vaginitis, genital
discharge, dysuria, and genital herpes, and HPV aren't reportable diseases
irritation. Suspecting a sexually
transmitted disease (STD), the
physician orders diagnostic testing
of the vaginal discharge. Which
STD must be reported to the public
health department.

a. bacterial vaginitis
b. gonorrhea
c. genital herpes
d. human papillomavirus (HPV)
2 A client complains of severe b - Asking an open-ended question such as
1. abdominal pain. To elicit as much "Can you describe the pain?" encourages the
information as possible about the client to describe any and all aspects of the
pain, the nurse should ask: pain in his own words. The other options are
likely to elicit less information because they're
a) "Do you have the pain all the more specific and would limit the client's
time?" response.
b) "Can you describe the pain?"
c) "Where does it hurt the most?"
d) "Is the pain stabbing like a
knife?"
2 A client diagnosed with pain d - The nurse should redirect the interaction
2. disorder is talking with the nurse back to fishing or another focus whenever the
about fishing when he suddenly client begins to ruminate about physical
reverts to talking about the pain in symptoms or impairment. Doing so helps the
his arm. Which of the following client talk about topics that are more
should the nurse do next? therapeutic and beneficial to recovery.
Allowing the client to talk about his pain or
a) Allow the client to talk about his asking if he needs additional pain medication
pain. is not therapeutic because it reinforces the
b) Ask the client if he needs more client's need for the symptom. Getting up and
pain medication. leaving the client is not appropriate unless the
c) Get up and leave the client. nurse has set limits previously by saying, "I will
d) Redirect the interaction back to get up and leave if you continue to talk about
fishing. your pain."
2 A client has a herniated disk b - the most common finding in a client with a
3. in the region of the third and herniated lumbar disk is severe lower back pain,
fourth lumbar vertebrae. which radiates to the buttocks, legs, and feet -
Which nursing assessment usually unilaterally. A herniated disk also may cause
finding most supports this sensory and motor loss (such as foot drop) in the
diagnosis? area innervated by the compressed spinal nerve
root. During later stages, it may cause weakness
a. hypoactive bowel sounds and atrophy of leg muscles. The condition doesn't
b. severe lower back pain affect bowel sounds or the arms.
c. sensory deficits in one arm
d. weakness and atrophy of
the arm muscles
2 A client has an episiotomy to c - birth may extend an episiotomy incision to the
4. widen her birth canal. Birth anal sphincter (a third degree laceration) or the anal
extends the incision into the canal (a fourth degree laceration). A first degree
anal sphincter. This laceration involves the fourchette, perineal skin,
complication is called: and vaginal mucous membranes. A second degree
laceration extends to the fasciae and muscle of the
a. a first-degree laceration perineal body.
b. a second-degree laceration
c. a third-degree laceration
d. a fourth-degree laceration
2 A client has just been c) CORRECT ANSWER demonstrate eyedrop
5. diagnosed with early instillation.
glaucoma. During a teaching Reason: Eyedrop instillation is a critical component
session, the nurse should: of self-care for a client with glaucoma. After
demonstrating eyedrop instillation to the client and
a) provide instructions on eye family, the nurse should verify their ability to
patching. perform this measure properly. An eye patch isn't
b) assess the client's visual necessary unless the client has undergone surgery.
acuity. Visual acuity assessment isn't necessary before
c) demonstrate eyedrop discharge. Intraocular lenses aren't implanted in
instillation. clients with glaucoma.
d) teach about intraocular
lens cleaning.
2 A client has refused to take a d - by acknowledging the client's fears, the nurse
6. shower since being admitted 4 can arrange to meet the client's hygiene needs in
days earlier. He tells a nurse, another way. Because these fears are real to the
"there are poison crystals client, providing a demonstration of reality by
hidden in the showerhead. dismantling the shower head wouldn't be effective
They'll kill me if I take a at this time.
shower." Which nursing action
is most appropriate?

a. dismantling the
showerhead and showing the
client that there is nothing in
it
b. explaining that other
clients are complaining about
the client's body odor
c. asking a security officer to
assist in giving the client a
shower
d. accepting these fears and
allowing the client to take a
sponge bath
2 A client has the following a) CORRECT ANSWER Respiratory acidosis
7. arterial blood gas values: pH, Reason: This client has a below-normal (acidic)
7.30; PaO2, 89 mm Hg; PaCO2, blood pH value and an above-normal partial
50 mm Hg; and HCO3-, 26 pressure of arterial carbon dioxide (PaCO2) value,
mEq/L. Based on these values, indicating respiratory acidosis. In respiratory
the nurse should suspect alkalosis, the pH value is above normal and the
which condition? PaCO2 value is below normal. In metabolic acidosis,
the pH and bicarbonate (HCO3-) values are below
a) Respiratory acidosis normal. In metabolic alkalosis, the pH and HCO3-
b) Respiratory alkalosis values are above normal.
c) Metabolic acidosis
d) Metabolic alkalosis
2 A client in the triage area who is at c) CORRECT ANSWER Disseminated
8. 19 weeks' gestation states that she intravascular coagulation.
has not felt her baby move in the Reason: A fetus that has died and is retained
past week and no fetal heart tones in utero places the mother at risk for
are found. While evaluating this disseminated intravascular coagulation (DIC)
client, the nurse identifies her as because the clotting factors within the
being at the highest risk for maternal system are consumed when the
developing which problem? nonviable fetus is retained. The longer the
fetus is retained in utero, the greater the risk
a) Abruptio placentae. of DIC. This client has no risk factors, history,
b) Placenta previa. or signs and symptoms that put her at risk for
c) Disseminated intravascular either abruptio placentae or placenta previa,
coagulation. such as sharp pain and "woody," firm
d) Threatened abortion. consistency of the abdomen (abruption) or
painless bright red vaginal bleeding (previa).
There is no evidence that she is threatening
to abort as she has no complaints of
cramping or vaginal bleeding.
2 A client is prescribed a) CORRECT ANSWER Irregular heartbeat.
9. metaproterenol (Alupent) via a Reason: Irregular heartbeats should be
metered-dose inhaler, two puffs reported promptly to the care provider.
every 4 hours. The nurse instructs Metaproterenol (Alupent) may cause irregular
the client to report adverse effects. heartbeat, tachycardia, or anginal pain
Which of the following are potential because of its adrenergic effect on beta-
adverse effects of metaproterenol? adrenergic receptors in the heart. It is not
recommended for use in clients with known
a) Irregular heartbeat. cardiac disorders. Metaproterenol does not
b) Constipation. cause constipation, pedal edema, or
c) Pedal edema. bradycardia.
d) Decreased pulse rate.
3 A client is scheduled for an a) CORRECT ANSWER 7:30 a.m.
0. excretory urography at 10 a.m. An Reason: It takes up to 2 hours for lidocaine-
order directs the nurse to insert a prilocaine cream (EMLA cream) to anesthetize
saline lock I.V. device at 9:30 a.m.. an insertion site. Therefore, if the insertion is
The client requests a local scheduled for 9:30 a.m., EMLA cream should
anesthetic for the I.V. procedure be applied at 7:30 a.m. The local anesthetic
and the physician orders lidocaine- wouldn't be effective if the nurse
prilocaine cream (EMLA cream). The administered it at the later times.
nurse should apply the cream at:

a) 7:30 a.m.
b) 8:30 a.m.
c) 9 a.m.
d) 9:30 a.m.
3 A client is scheduled for surgery a - The client's statement reveals a Deficient
1. under general anesthesia. The night knowledge related to food restrictions
before surgery, the client tells the associated with general anesthesia. Fear
nurse, "I can't wait to have related to surgery, Risk for impaired skin
breakfast tomorrow." Based on this integrity related to upcoming surgery, and
statement, which nursing diagnosis Ineffective coping related to the stress of
should be the nurse's priority? surgery may be applicable nursing diagnoses
but they aren't related to the client's
a) Deficient knowledge related to statement.
food restrictions associated with
anesthesia
b) Fear related to surgery
c) Risk for impaired skin integrity
related to upcoming surgery
d) Ineffective coping related to the
stress of surgery
3 The client is taking risperidone d - Asocial behavior, anergia, alogia, and
2. (Risperdal) to treat the positive and affective flattening are some of the negative
negative symptoms of symptoms of schizophrenia that may
schizophrenia. Improvement of improve with risperidone therapy. Abnormal
which of the following negative thought form is a positive symptom of
symptoms indicate the drug is schizophrenia. Hallucinations and delusions
effective? are positive symptoms of schizophrenia.
Bizarre behavior is a positive symptom of
a) Abnormal thought form. schizophrenia.
b) Hallucinations and delusions.
c) Bizarre behavior.
d) Asocial behavior and anergia.
3 A client received chemotherapy 24 c) CORRECT ANSWER Wear personal
3. hours ago. Which precautions are protective equipment when handling blood,
necessary when caring for the body fluids, and feces.
client? Reason: Chemotherapy drugs are present in
the waste and body fluids of clients for 48
a) Wear sterile gloves. hours after administration. The nurse should
b) Place incontinence pads in the wear personal protective equipment when
regular trash container. handling blood, body fluids, or feces. Gloves
c) Wear personal protective offer minimal protection against exposure.
equipment when handling blood, The nurse should wear a face shield, gown,
body fluids, and feces. and gloves when exposure to blood or body
d) Provide a urinal or bedpan to fluid is likely. Placing incontinence pads in
decrease the likelihood of soiling the regular trash container and providing a
linens. urinal or bedpan don't protect the nurse
caring for the client.
3 A client was hospitalized for 1 week c) CORRECT ANSWER The depression is
4. with major depression with suicidal improving and the suicidal ideation is
ideation. He is taking venlafaxine lessening.
(Effexor), 75 mg three times a day, Reason: The client's statements about being
and is planning to return to work. in control of his behavior and his or her plans
The nurse asks the client if he is to return to work indicate an improvement in
experiencing thoughts of self-harm. depression and that suicidal ideation,
The client responds, "I hardly think although present, is decreasing. Nothing in
about it anymore and wouldn't do his comments or behavior indicate he is
anything to hurt myself." The nurse decompensating. There is no evidence to
should make which judgment about support an increase or adjustment in the
the client? dose of Effexor or a call to the primary care
provider. Typically, the cognitive components
a) The client is decompensating and of depression are the last symptoms
in need of being readmitted to the eliminated. For the client to be experiencing
hospital. some suicidal ideation in the second week of
b) The client needs an adjustment or psychopharmacologic treatment is not
increase in his dose of unusual.
antidepressant.
c) The depression is improving and
the suicidal ideation is lessening.
d) The presence of suicidal ideation
warrants a telephone call to the
client's primary care provider.
3 A client was talking with her d - At this level of aggression, the client
5. husband by telephone, and then she needs an appropriate physical outlet for the
began swearing at him. The nurse anger. She is beyond writing in a journal.
interrupts the call and offers to talk Urging the client to talk to the nurse now or
with the client. She says, "I can't talk making threats, such as telling her that she
about that bastard right now. I just will be restrained, is inappropriate and could
need to destroy something." Which lead to an escalation of her anger.
of the following should the nurse do
next?

a) Tell her to write her feelings in


her journal.
b) Urge her to talk with the nurse
now.
c) Ask her to calm down or she will
be restrained.
d) Offer her a phone book to
"destroy" while staying with her.
3 A client who has a history of Crohn's c) CORRECT ANSWER Hypokalemia.
6. disease is admitted to the hospital with Reason: Hypokalemia is the most
fever, diarrhea, cramping, abdominal expected laboratory finding owing to the
pain, and weight loss. The nurse should diarrhea. Hypoalbuminemia can also
monitor the client for: occur in Crohn's disease; however, the
client's potassium level is of greater
a) Hyperalbuminemia. importance at this time because a low
b) Thrombocytopenia. potassium level can cause cardiac arrest.
c) Hypokalemia. Anemia is an expected development, but
d) Hypercalcemia. thrombocytopenia is not. Calcium levels
are not affected.
3 A client who has been diagnosed with c) CORRECT ANSWER Hot chocolate.
7. gastroesophageal reflux disease Reason: With GERD, eating substances
(GERD) complains of heartburn. To that decrease lower esophageal sphincter
decrease the heartburn, the nurse pressure causes heartburn. A decrease in
should instruct the client to eliminate the lower esophageal sphincter pressure
which of the following items from the allows gastric contents to reflux into the
diet? lower end of the esophagus. Foods that
can cause a decrease in esophageal
a) Lean beef. sphincter pressure include fatty foods,
b) Air-popped popcorn. chocolate, caffeinated beverages,
c) Hot chocolate. peppermint, and alcohol. A diet high in
d) Raw vegetables. protein and low in fat is recommended for
clients with GERD. Lean beef, popcorn,
and raw vegetables would be acceptable.
3 A client who took an overdose of b) CORRECT ANSWER Question the
8. Tylenol in a suicide attempt is primary health care provider about the
transferred overnight to the psychiatric Tylenol prescription.
inpatient unit from the intensive care Reason: The nurse should question the
unit. The night shift nurse called the Tylenol order because the client
primary health care provider on call to overdosed on Tylenol, and that analgesic
obtain initial prescriptions. The primary would be contraindicated as putting
health care provider prescribes the further stress on the liver. There is no
typical routine medications for clients need to hold the PM Milk of Magnesia or
on this unit: Milk of Magnesia, Maalox Maalox. There is no indication that the
and Tylenol as needed. Prior to client is agitated or needs medication for
implementing the prescriptions, the agitation. There is little likelihood that the
nurse should? client needs an IV after being transferred
out of an intensive care unit, as the client
a) Ask the primary health care provider will be able to take oral fluids.
about holding all the client's PM
prescriptions.
b) Question the primary health care
provider about the Tylenol
prescription.
c) Request a prescription for a
medication to relieve agitation.
d) Suggest the primary health care
provider write a prescription for
intravenous fluids.
3 A client with a bleeding ulcer is a - In early shock, the body attempts to
9. vomiting bright red blood. The nurse meet its perfusion needs through
should assess the client for which of tachycardia, vasoconstriction, and fluid
the following indicators of early shock? conservation. The skin becomes cool and
clammy. Urine output in early shock may
a) Tachycardia. be normal or slightly decreased. The
b) Dry, flushed skin. client may experience increased
c) Increased urine output. restlessness and anxiety from hypoxia,
d) Loss of consciousness. but loss of consciousness is a late sign of
shock.
4 The client with a hearing aid does not d - inadequate amplification can occur
0. seem to be able to hear the nurse. The when a hearing aid is not place properly.
nurse should do which of the following? The certified audiologist is licensed to
dispense hearing aids. The ear mold is the
a. contact the client's audiologist only part of the hearing aid that may be
b. cleanse the hearing aid ear mold in wash frequently; it should be washed
normal saline daily with soap and water. Irrigation of the
c. irrigate the ear canal ear canal is done to remove impacted
d. check the hearing aid's placement cerumen or a foreign body
4 A client with an incomplete small- b) CORRECT ANSWER Advance the tube 2
1. bowel obstruction is to be treated to 4 inches at specified times.
with a Cantor tube. Which of the Reason: Once the intestinal tube has
following measures would most likely passed into the duodenum, it is usually
be included in the client's care once advanced as ordered 2 to 4 inches every
the Cantor tube has passed into the 30 to 60 minutes. This, along with gravity
duodenum? and peristalsis, enables passage of the
tube forward. The client is encouraged to
a) Maintain bed rest with bathroom walk, which also facilitates tube
privileges. progression. A client with an intestinal tube
b) Advance the tube 2 to 4 inches at needs frequent mouth care to stimulate
specified times. saliva secretion, to maintain a healthy oral
c) Avoid frequent mouth care. cavity, and to promote comfort regardless
d) Provide ice chips for the client to of where the tube is placed in the intestine.
suck. Ice chips are contraindicated because
hypotonic fluid will draw extra fluid into an
already distended bowel.
4 A client with bipolar disorder, manic b - The client is demonstrating flight of
2. phase, just sat down to watch ideas, or the rapid, unconnected, and often
television in the lounge. As the nurse illogical progression from one topic to
approaches the lounge area, the client another. Concreteness involves interpreting
states, "The sun is shining. Where is another person's words literally.
my son? I love Lucy. Let's play ball." Depersonalization refers to feelings of
The client is displaying: strangeness concerning the environment
or the self. A neologism is a word made up
a) Concreteness. by a client.
b) Flight of ideas.
c) Depersonalization.
d) Use of neologisms.
4 A client with burns on his groin has d - The nurse should clean the area with a
3. developed blisters. As the client is mild solution such as normal saline, and
bathing, a few blisters break. The best then cover it with a protective dressing.
action for the nurse to take is to: Soap and water and alcohol are too harsh.
The body's first line of defense broke when
a) remove the raised skin because the the blisters opened; removing the skin
blister has already broken. exposes a larger area to the risk of
b) wash the area with soap and water infection.
to disinfect it.
c) apply a weakened alcohol solution
to clean the area.
d) clean the area with normal saline
solution and cover it with a protective
dressing.
4 A client with chronic heart failure is a) CORRECT ANSWER visual disturbances.
4. receiving digoxin (Lanoxin), 0.25 mg Reason: Digoxin toxicity may cause visual
by mouth daily, and furosemide disturbances (such as, flickering flashes of
(Lasix), 20 mg by mouth twice daily. light, colored or halo vision, photophobia,
The nurse instructs the client to notify blurring, diplopia, and scotomata), central
the physician if nausea, vomiting, nervous system abnormalities (such as
diarrhea, or abdominal cramps occur headache, fatigue, lethargy, depression,
because these signs and symptoms irritability and, if profound, seizures,
may signal digoxin toxicity. Digoxin delusions, hallucinations, and memory
toxicity may also cause: loss), and cardiovascular abnormalities
(abnormal heart rate and arrhythmias).
a) visual disturbances. Digoxin toxicity doesn't cause taste and
b) taste and smell alterations. smell alterations. Dry mouth and urine
c) dry mouth and urine retention. retention typically occur with
d) nocturia and sleep disturbances. anticholinergic agents, not inotropic agents
such as digoxin. Nocturia and sleep
disturbances are adverse effects of
furosemide especially if the client takes
the second daily dose in the evening,
which may cause diuresis at night.
4 A client with chronic d) CORRECT ANSWER hallucination.
5. undifferentiated schizophrenia is Reason: Auditory hallucination, in which one
admitted to the psychiatric unit hears voices when no external stimuli exist, is
of a local hospital. During the common in schizophrenic clients. Such behaviors
next several days, the client is as laughing, yelling, and talking to oneself
seen laughing, yelling, and suggest such a hallucination. Delusions, also
talking to himself. This behavior common in schizophrenia, are false beliefs or
is characteristic of: ideas that arise without external stimuli. Clients
with schizophrenia may exhibit looseness of
a) delusion. association, a pattern of thinking and
b) looseness of association. communicating in which ideas aren't clearly
c) illusion. linked to one another. Illusion is a less severe
d) hallucination. perceptual disturbance in which the client
misinterprets actual external stimuli. Illusions are
rarely associated with schizophrenia.
4 A client with inflammatory bowel b - The TPN solution is usually a hypertonic
6. disease is receiving total dextrose solution. The greater the concentration
parenteral nutrition (TPN). The of dextrose in solution, the greater the tonicity.
basic component of the client's Hypertonic dextrose solutions are used to meet
TPN solution is most likely to be: the body's calorie demands in a volume of fluid
that will not overload the cardiovascular system.
a) An isotonic dextrose solution. An isotonic dextrose solution (e.g., 5% dextrose
b) A hypertonic dextrose in water) or a hypotonic dextrose solution will not
solution. provide enough calories to meet metabolic
c) A hypotonic dextrose needs. Colloids are plasma expanders and blood
solution. products and are not used in TPN.
d) A colloidal dextrose solution.
4 A client with pneumonia has a c - frequent linen changes are appropriate for
7. temperature of 102.6F (39.2C), is this client because of the diaphoresis.
diaphoretic, and has a Diaphoresis produces general discomfort. The
productive cough. The nurse client should be kept dry to promote comfort.
should include which of the Position changes need to be done every 2 hours.
following measures in the plan of Nasotracheal suctioning is not indicated with the
care? client's productive cough. Frequent offering of a
bedpan is not indicated by the data provided in
a. position changes every 4 this scenario.
hours
b. nasotracheal suctioning to
clear secretions
c. frequent linen changes
d. frequent offering of a bedpan
4 A client with Rh isoimmunization b - because the neonate is severely ill and needs
8. gives birth to a neonate with an to be placed in the neonatal intensive care unit,
enlarged heart and severe, the client may have a nursing diagnosis of
generalized edema. The neonate 'impaired parenting related to the neonate's
is immediately transferred to the transfer to the neonatal intensive care unit.'
neonatal intensive care unit. (another pertinent nursing diagnosis may be
Which nursing diagnosis is most 'compromised family coping related to lack the
appropriate for the client? opportunity for bonding.) Rh isoimmunization
isn't a socially unacceptable infection. This
a. ineffective parenting related condition causes an excess fluid volume (not
to the neonate's transfer to the deficient) related to cardiac problems. Rh
intensive care unit isoimmunization doesn't lead to loss of the
b. impaired parenting related to neonate by statute.
the neonate's transfer to the
intensive care unit
c. deficient fluid volume related
to severe edema
d. fear related to removal and
loss of the neonate by statute
4 A client with type 1 a - if the nurse administers the client's normal daily
9. diabetes must undergo dose of insulin while he's on nothing-by-mouth status
bowel resection in the before surgery, he'll experience hypoglycemia.
morning. How should the Therefore, the nurse should administer half the daily
nurse proceed while caring insulin dose as ordered. Oral antidiabetic agents aren't
for him on the morning of effective for type 1 diabetes I.V. insulin infusions aren't
surgery? necessary to manage blood glucose levels in clients
undergoing routine surgery.
a. administer half of the
client's typical morning
insulin dose as ordered
b. administer an oral
antidiabeteic agent as
ordered
c. administer an I.V. insulin
infusion as ordered
d. administer the client's
normal daily dose of insulin
as ordered
5 the comatose victim of the a - an unconscious client is best positioned in a lateral
0. car accident is to have a or semiprone position because these positions allow
gastric lavage. Which of the jaw and tongue to fall forward, facilitate drainage of
the following positions secretions, and prevent aspiration. Positioning the
would be most appropriate client supine carries a major risk of airway obstruction
for the client during this from the tongue, vomit, or nasopharyngeal secretions.
procedure? Trendeleburg's position, with the head lower than the
heart, decreases effective lung volume and increases
a. lateral the risk of cerebral edema. The lithotomy position has
b. supine no purpose in this situation.
c. trendelenburg's
d. lithotomy
5 During the first feeding, d) CORRECT ANSWER Clear the neonate's airway with
1. the nurse observes that the suction or gravity.
neonate becomes cyanotic Reason: If a neonate gags on mucus and becomes
after gagging on mucus. cyanotic during the first feeding, the airway is most
Which of the following likely closed. The nurse should clear the airway by
should the nurse do first? gravity (by lowering the infant's head) or suction.
Starting mouth-to-mouth resuscitation is not indicated
a) Start mouth-to-mouth unless the neonate remains cyanotic and lowering his
resuscitation. head or suctioning doesn't clear his airway. Contacting
b) Contact the neonatal the neonatal resuscitation team is not warranted unless
resuscitation team. the infant remains cyanotic even after measures to
c) Raise the neonate's head clear the airway. Raising the neonate's head and
and pat the back gently. patting the back are not appropriate actions for
d) Clear the neonate's removing mucus. Doing so allows the mucus to remain
airway with suction or lodged causing further breathing difficulties.
gravity.
5 A home health nurse who sees a b) CORRECT ANSWER "I should increase my
2. client with diverticulitis is evaluating intake of fresh fruits and vegetables during
teaching about dietary modifications remissions."
necessary to prevent future Reason: A client with diverticulitis needs to
episodes. Which statement by the modify fiber intake to effectively manage the
client indicates effective teaching? disease. During episodes of diverticulitis, he
should follow a low-fiber diet to help
a) "I'll increase my intake of protein minimize bulk in the stools. A client with
during exacerbations." diverticulosis should follow a high-fiber diet.
b) "I should increase my intake of Clients with diverticular disease don't need
fresh fruits and vegetables during to modify their intake of protein and omega-
remissions." 3 fatty acids.
c) "I'll snack on nuts, olives, and
popcorn during flare-ups."
d) "I'll incorporate foods rich in
omega-3 fatty acids into my diet."
5 The major goal of therapy in crisis b - during a period of crisis, the major goal is
3. intervention is to: to resolve the immediate problem, with
hopes of getting the individual to the level of
a. withdraw from the stress functioning that existed before the crisis or
b. resolve the immediate problem to a higher level of functioning. Withdrawing
c. decrease anxiety from stress doesn't address the immediate
d. provide documentation of events problem and isn't therapeutic. The client's
anxiety will decrease after the immediate
problem is resolved. Providing support and
safety are necessary interventions while
working toward accomplishing the goal.
Documentation is necessary for maintaining
accurate records of treatment, but isn't a
major goal.
5 A man of Chinese descent is c) CORRECT ANSWER Restores the balance
4. admitted to the hospital with of energy.
multiple injuries after a motor Reason: Acupuncture, like acumassage and
vehicle accident. His pain is not acupressure, is performed in certain Asian
under control. The client states, "If I cultures to restore the energy balance within
could be with my people, I could the body. Pressure, massage, and fine
receive acupuncture for this pain." needles are applied to energy pathways to
The nurse should understand that help restore the body's balance.
acupuncture in the Asian culture is Acupuncture is not based on a belief in
based on the theory that it: purging evil spirits. Although pain relief
through acupuncture can promote
a) Purges evil spirits. tranquility, acupuncture is performed to
b) Promotes tranquility. restore energy balance. In the Western
c) Restores the balance of energy. world, many researchers think that the gate-
d) Blocks nerve pathways to the control theory of pain may explain the
brain. success of acupuncture, acumassage, and
acupressure.
5 The mother of a client with chronic b - Noncompliance with medications is
5. undifferentiated schizophrenia calls common in the client with chronic
the visiting nurse in the outpatient undifferentiated schizophrenia. The nurse
clinic to report that her daughter has has the responsibility to assess this
not answered the phone in 10 days. situation. Asking the mother if they've
"She was doing so well for months. I argued or if the client is mad at the mother
don't know what's wrong. I'm or telling the mother to go over to the
worried." Which of the following apartment and see what's going on places
responses by the nurse is most the blame and responsibility on the mother
appropriate? and therefore is inappropriate. Telling the
mother not to worry ignores the seriousness
a) "Maybe she's just mad at you. Did of the client's symptoms.
you have an argument?"
b) "She may have stopped taking her
medications. I'll check on her."
c) "Don't worry about this. It
happens sometimes."
d) "Go over to her apartment and
see what's going on."
5 A multigavid client in labor at 38 B - The fetal heart rate of a multipara
6. weeks' gestation has been diagnosed diagnosed with Rh sensitization and
with Rh sensitization and probably probably fetal hydrops and anemia will
fetal hydrops and anemia. When the most likely demonstrate a sinusoidal
nurse observes the fetal heart rate pattern that resembles a sine wave. It has
pattern on the monitor, which of the been hypothesized that this pattern
following patterns is most likely? reflects an absence of autonomic nervous
control over the fetal heart rate resulting
a. early deceleration pattern from severe hypoxia. This client will most
b. sinusoidal pattern likely requires a cesarean delivery to
c. variable deceleration pattern improve the fetal outcome. Early
d. late deceleration pattern decelerations are associated with cord
compression; and late deceleration are
associated with poor placental perfusion
5 The neonoate of a client with type 1 c - lethargy in the neonate may be seen
7. diabetes is at high risk for with hypoglycemia because of a glucose in
hypoglycemia. An initial sign the the nerve cells. Peripheral acrocyanosis is
nurse should recognize as indicating normal in the neonate because of
hypoglycemia in a neonate is: immature capillary function. Tachycardia -
not bradycardia - is seen with
a. peripheral acrocyanosis hypoglycemia. Jaundice isn't a sign of
b. bradycardia hypoglycemia.
c. lethargy
d. jaundice
5 A nurse, a licensed practical nurse c - because it's important to get more
8. (LPN), and a nursing assistant are information about the client with a
caring for a group of clients. The decreased pulse oximetry level, the nurse
nurse asks the nursing assistant to should ask the LPN to obtain vital signs
check the pulse oximetry level of a and administer oxygen as ordered. The
client who underwent laminectomy. nurse must attend to the newly admitted
The nursing assistant reports that the client without delaying treatment to the
pulse oximetry reading is 89%. The client who is already in her care. The nurse
client Kardex contains an order for can effectively do this by delegating tasks
oxygen application at 2 L/min should to an appropriate health team member
the pulse oximetry level fall below such as an LPN. The nurse doesn't need to
92%. The nurse is currently assessing immediately attend to the client with a
a postoperative client who just decreased pulse oximetry level; she may
returned from the postanesthesia wait until she completes the assessment of
unit. How should the nurse proceed? the newly admitted client. The physician
doesn't need to be notified at this time
a. immediately go the client's room because an order for oxygen
and assess vital signs, administer administration is already on record.
oxygen at 2 L/minute, and notify the
physician.
b. ask the nursing assistant to notify
the physician of the low pulse
oximetry level
c. ask the LPN to obtain vital signs
and administer oxygen at 2 L/min to
the client who underwent
laminectomy
d. complete the assessment of the
new client before attending to the
client who underwent laminectomy
5 A nurse assesses a client's respiratory b) CORRECT ANSWER Use of accessory
9. status. Which observation indicates muscles
that the client is having difficulty Reason: The use of accessory muscles for
breathing? respiration indicates the client is having
difficulty breathing. Diaphragmatic and
a) Diaphragmatic breathing pursed-lip breathing are two controlled
b) Use of accessory muscles breathing techniques that help the client
c) Pursed-lip breathing conserve energy.
d) Controlled breathing
6 The nurse has administered b - Aminophylline, a bronchodilator that
0. aminophylline to a client with relaxes smooth muscles in the bronchioles,
emphysema. The medication is is used in the treatment of emphysema to
effective when there is: improve ventilation by dilating the
bronchioles. Aminophylline does not have
a) Relief from spasms of the an effect on the diaphragm or the
diaphragm. medullary respiratory center and does not
b) Relaxation of smooth muscles in promote pulmonary circulation.
the bronchioles.
c) Efficient pulmonary circulation.
d) Stimulation of the medullary
respiratory center.
6 The nurse has discussed b) CORRECT ANSWER "I should not get sexually
1. sexuality issues during the aroused or have any nipple stimulation."
prenatal period with a Reason: This client has already had one episode
primigravida who is at 32 weeks' of preterm labor at 32 weeks' gestation. Sexual
gestation. She has had one intercourse, arousal, and nipple stimulation may
episode of preterm labor. The result in the release of oxytocin which can
nurse determines that the client contribute to continued preterm labor and early
understands the instructions delivery. The client should be advised to refrain
when she says: from these activities until closer to term, which is
6 to 8 weeks later. Telling the client that
a) "I can resume sexual intercourse is acceptable after the bleeding stops
intercourse when the bleeding is incorrect and may lead to early delivery of a
stops." preterm neonate. The client should not have
b) "I should not get sexually intercourse for at least 6 weeks because of the
aroused or have any nipple danger of inducing labor. There is no indication
stimulation." when the client's next prenatal visit is scheduled.
c) "I can resume sexual
intercourse in 1 to 2 weeks."
d) "I should not have sexual
intercourse until my next
prenatal visit."
6 A nurse has received change-of- a) CORRECT ANSWER The entry should include
2. shift-report and is briefly clearer descriptions of the client's mood and
reviewing the documentation behavior.
about a client in the client's Reason: Entries in the medical record should be
medical record. A recent entry precise, descriptive, and objective. An adjective
reads, "Client was upset such as "upset" is unclear and open to many
throughout the morning." How interpretations. As such, the nurse should
could the charting entry be best elaborate on this description so a reader has a
improved? clearer understanding of the client's state of
mind. Stating the apparent reasons that the
a) The entry should include client was "upset" does not resolve the ambiguity
clearer descriptions of the of this descriptor. Cognitive and psychosocial
client's mood and behavior. issues are valid components of the medical
b) The entry should avoid record. Responses and interventions should
mentioning cognitive or normally follow assessment data but the data
psychosocial issues. themselves must first be recorded accurately.
c) The entry should list the
specific reasons that the client
was upset.
d) The entry should specify the
subsequent interventions that
were performed.
6 The nurse is assessing a client at a - by 4 to 6 weeks postpartum, the fundus
3. her postpartum checkup 6 weeks should be deep in the pelvis and the size of a
after a vaginal delivery. The non-pregnant uterus. Subinvolution, caused by
mother is bottle feeding her infection or retained placental fragments, is a
baby. Which client finding problem associated with a uterus that is larger
indicates a problem at this time? than expected at this time. Normal expectations
include a white, thick vaginal discharge, striae
a. firm fundus at the symphysis that are beginning to fade to silver, and breasts
b. white, thick vaginal discharge that are soft without evidence of milk production
c. striae that are silver in color (in a bottle feeding mother).
d. soft breasts without milk
6 A nurse is assessing the b) CORRECT ANSWER Bilateral dependent edema
4. legs of a client who's 36 Reason: As the uterus grows heavier during
weeks pregnant. Which pregnancy, femoral venous pressure rises, leading to
finding should the nurse bilateral dependent edema. Factors interfering with
expect? venous return, such as sitting or standing for long
periods, contribute to edema. Absence of pedal pulses
a) Absent pedal pulses and sluggish capillary refill signal inadequate
b) Bilateral dependent circulation to the legs an unexpected finding during
edema pregnancy. Unilateral calf enlargement, also an
c) Sluggish capillary refill abnormal finding, may indicate thrombosis.
d) Unilateral calf
enlargement
6 A nurse is caring for a client d) CORRECT ANSWER Call the physician immediately.
5. diagnosed with a cerebral Reason: The nurse should notify the physician
aneurysm who reports a immediately because the headache may be an
severe headache. Which indication that the aneurysm is leaking. Sitting with
action should the nurse the client is appropriate but only after the physician
perform? has been notified of the change in the client's
condition. The physician will decide whether or not
a) Sit with the client for a administration of an analgesic is indicated. Informing
few minutes. the nurse manager isn't necessary.
b) Administer an analgesic.
c) Inform the nurse
manager.
d) Call the physician
immediately.
6 A nurse is caring for a client a) CORRECT ANSWER Endotracheal suctioning
6. who has a tracheostomy and Reason: Endotracheal suctioning removes secretions
temperature of 103 F as well as gases from the airway and lowers the
(39.4 C). Which arterial oxygen saturation (SaO2) level. Coughing and
intervention will most likely using an incentive spirometer improve oxygenation
lower the client's arterial and should raise or maintain oxygen saturation.
blood oxygen saturation? Because of superficial vasoconstriction, using a
cooling blanket can lower peripheral oxygen
a) Endotracheal suctioning saturation readings, but SaO2 levels wouldn't be
b) Encouragement of affected.
coughing
c) Use of a cooling blanket
d) Incentive spirometry
6 a nurse is caring for a client d - chest x ray confirms diagnosis by revealing air or
7. who required chest tube fluid in the pleural space. SaO2 values may initially
insertion for a decrease with a pneumothorax but typically return to
pneumothorax. To assess for normal within 24 hours. ABG studies may show
pneumothorax resolution, hypoxemia, possibly with respiratory acidosis and
the nurse can anticipate hypercapnia but these are not necessarily related to a
that the client will require: pneumothorax. Chest auscultation will determine
overall lung status, but it's difficult to determine if the
a. monitoring of arterial best has re-expanded sufficiently.
oxygen saturation (SaO2)
b. arterial blood gas (ABG)
studies
c. chest auscultation
d. chest x ray
6 A nurse is caring for a client d) CORRECT ANSWER The client has normal breath
8. with a diagnosis of Impaired sounds in all lung fields.
gas exchange. Based upon Reason: If the interventions are effective, the client's
this nursing diagnosis, breath sounds should return to normal. The client
which outcome is most should be able to cough effectively and should be
appropriate? encouraged to increase activity, as tolerated. Fluids
should help thin secretions, so fluid intake should be
a) The client maintains a encouraged.
reduced cough effort to
lessen fatigue.
b) The client restricts fluid
intake to prevent
overhydration.
c) The client reduces daily
activities to a minimum.
d) The client has normal
breath sounds in all lung
fields.
6 The nurse is caring for a client with c) CORRECT ANSWER Inspiratory and
9. asthma. The nurse should conduct a expiratory wheezing.
focused assessment to detect which Reason: The hallmark signs of asthma are
of the following? chest tightness, audible wheezing, and
coughing. Inspiratory and expiratory
a) Increased forced expiratory wheezing is the result of
volume. bronchoconstriction. Even between
b) Normal breath sounds. exacerbations, there may be some soft
c) Inspiratory and expiratory wheezing, so a finding of normal breath
wheezing. sounds would be expected in the absence
d) Morning headaches. of asthma. The expected finding is
decreased forced expiratory volume [forced
expiratory flow (FEF) is the flow (or speed)
of air coming out of the lung during the
middle portion of a forced expiration] due to
bronchial constriction. Morning headaches
are found with more advanced cases of
COPD and signal nocturnal hypercapnia or
hypoxemia.
7 A nurse is caring for a client with a - after a myelogram, positioning depends
0. lower back pain who is scheduled for on the dye injected. When a water-soluble
myelography using metrizamide (a dye such as metrizamide in injected, the
water-soluble contrast dye). After the head of the bed is elevated to a 45-degree
test, the nurse should place the client angle to slow the upward dispersion of the
in which position? dye. The prone and supine positions are
contraindicated when a water-soluble
a. head of the bed elevated 45 contrast dye is used. The client should be
degrees positioned supine with the head lower than
b. prone the trunk after an air-contrast study.
c. supine with feet raised
d. supine with the head lower than
the trunk
7 A nurse is caring for a client with a - The client's response indicates that he's
1. poorly managed diabetes mellitus in denial and needs further insight and
who has a serious foot ulcer. When education about his condition. Letting the
she informs him that the physician client know that the nurse has his best
has ordered a wound care nurse to interests in mind helps him accept the
examine his foot, the client asks why wound-care nurse. Although telling the
he should see anyone other than this client that his condition is serious and that
nurse. He states, "It's no big deal. I'll the wound care nurse will see him that day
keep it covered and put antibiotic are true statements, they're much too
ointment on it." What is the nurse's direct and may increase client resistance.
best response? Telling the client he could lose his foot is
inappropriate and isn't therapeutic
a) "We're very concerned about your communication.
foot and we want to provide the best
possible care for you."
b) "This is a big deal and you need to
recognize how serious it is."
c) "This is the physician's
recommendation. The wound care
nurse will see you today."
d) "You could lose your foot if you
don't see the wound care nurse."
7 The nurse is caring for b - The major reasons for afterbirth pains are breast-
2. several mother-baby feeding, high parity, overdistended uterus during
couplets. In planning the pregnancy, and a uterus filled with blood clots.
care for each of the Physiologically, afterbirth pains are caused by
couplets, which mother intermittent contraction and relaxation of the uterus.
would the nurse expect to These contractions are stronger in multigravidas in order
have the most severe to maintain a contracted uterus. The release of oxytocin
afterbirth pains? when breast-feeding also stimulates uterine
contractions. There is no data to suggest any of these
a) G 4, P 1 client who is clients has had an overdistended uterus or currently has
breastfeeding her infant. clots within the uterus. The G 3, P 3 client who is breast-
b) G 3, P 3 client who is feeding has the highest parity of the clients listed, which
breastfeeding her infant. in addition to breast-feedingplaces her most at risk
c) G 2, P 2 cesarean client for afterbirth pains. The G 2, P 2 postcesarean client
who is bottle-feeding her may have cramping but it should be less than the G 3, P
infant. 3 client. The G 3, P 3 client who is bottle-feeding would
d) G 3, P 3 client who is be at risk for afterbirth pains because she has delivered
bottle-feeding her infant. several children, but her choice to bottle-feed reduces
her risk of pain.
7 A nurse is conducting an c - A history of immunocompromised status, such as that
3. initial assessment on a which occurs with liver transplantation, places the client
client with possible at a higher risk for contracting tuberculosis. Other risk
tuberculosis. Which factors include inadequate health care, traveling to
assessment finding countries with high rates of tuberculosis (such as
indicates a risk factor for southeastern Asia, Africa, and Latin America), being a
tuberculosis? health care worker who performs procedures in which
exposure to respiratory secretions is likely, and being
a) The client sees his institutionalized.
physician for a check-up
yearly.
b) The client has never
traveled outside of the
country.
c) The client had a liver
transplant 2 years ago.
d) The client works in a
health care insurance
office.
7 A nurse is developing a nursing c) CORRECT ANSWER Factors influencing
4. diagnosis for a client. Which the client's problem
information should she include? Reason: A nursing diagnosis is a written
statement describing a client's actual or
a) Actions to achieve goals potential health problem. It includes a
b) Expected outcomes specified diagnostic label, factors that
c) Factors influencing the client's influence the client's problem, and any
problem signs or symptoms that help define the
d) Nursing history diagnostic label. Actions to achieve goals
are nursing interventions. Expected
outcomes are measurable behavioral goals
that the nurse develops during the
evaluation step of the nursing process. The
nurse obtains a nursing history during the
assessment step of the nursing process.
7 A nurse is documenting a variance a - an incident report, also termed a
5. that has occurred during the shift, and variance report or occurrence report, is a
this report will be used for quality tool healthcare agencies use to document
improvement to identify high-risk anything out of the ordinary that results in
patterns and potentially initiate in- or has the potential to result in harm to a
services programs. This is an example client, employee, or visitor. These reports
of which type of report? are used for quality improvement and not
for disciplinary action. They are a means of
a. incident report identifying risks and high-risk patterns and
b. nurse's shift report initiating in-service programs to prevent
c. transfer report further problems. A nurse's shift report is
d. telemedicine report. given by a primary nurse to the nurse
replacing him or her by the charge nurse
to the nurse who assumes responsibility
for continuing client care. A transfer report
is a summary of a client's condition and
care when transferring clients from one
unit or institution to another. A
telemedicine report can link healthcare
professional immediately and enable
nurses to receive and give critical
information about clients in a timely
fashion.
7 A nurse is facilitating mandated group a) CORRECT ANSWER from any segment of
6. therapy for clients who have sexually the population.
abused children. Children who are Reason: Victims of childhood sexual abuse
victims of sexual abuse are typically: come from all segments of the population
and from all socioeconomic backgrounds.
a) from any segment of the Most victims know their abuser. Children
population. rarely willingly engage in sexual acts with
b) of low socioeconomic background. adults because they don't have full
c) strangers to the abuser. decision-making capacities.
d) willing to engage in sexual acts
with adults.
7 A nurse is helping a physician insert a c - A nurse who suspects an air embolism
7. subclavian central line. After the should place the client on his left side and
physician has gained access to the in Trendelenburg's position. Doing so
subclavian vein, he connects a 10-ml allows the air to collect in the right atrium
syringe to the catheter and withdraws rather than enter the pulmonary system.
a sample of blood. He then The supine position, high-Fowler's position,
disconnects the syringe from the port. and the shock position are therapeutic for
Suddenly, the client becomes other situations but not for air embolism.
confused, disoriented, and pale. The
nurse suspects an air embolus. She
should:

a) place the client in a supine position


and prepare to perform
cardiopulmonary resuscitation.
b) place the client in high-Fowler's
position and administer supplemental
oxygen.
c) turn the client on his left side and
place the bed in Trendelenburg's
position.
d) position the client in the shock
position with his legs elevated.
7 A nurse is instructing a client b - a client who states that he'll increase his dose
8. with bipolar disorder on proper of lithium if his mood fluctuates requires
use of lithium carbonate additional teaching because increasing the dose
(Eskalith), the drug's adverse of lithium without evaluating the client's
effects, and symptoms of laboratory values can cause serious health
lithium toxicity. Which client problems, such as lithium toxicity, overdose, and
statement indicates that renal failure. Clients taking lithium don't need to
additional teaching is required? limit their sodium intake. A low sodium diet
causes lithium retention. A therapeutic lithium
a. "I can still eat my favorite blood level indicates that the drug concentration
salty foods." has stabilized. The client demonstrates effective
b. "when my moods fluctuate, teaching by stating his lithium levels will be
I'll increase my dose of affected by foods that have a diuretic effect, such
lithium." as watermelon, cantaloupe, grapefruit juice and
c. "a good blood level of the cranberry juice.
drug means the drug
concentration has stabilized."
d. "eating too much watermelon
will affect my lithium level."
7 A nurse is monitoring a client b - The most serious adverse reaction associated
9. receiving tranylcypromine with high doses of MAO inhibitors is hypertensive
sulfate (Parnate). Which serious crisis, which can lead to death. Although not a
adverse reaction can occur with crisis, orthostatic hypotension is also common
high dosages of this monoamine and may lead to syncope with high doses. Muscle
oxidase (MAO) inhibitor? spasticity (not flaccidity) is associated with MAO
inhibitor therapy. Hypoglycemia isn't an adverse
a) Hypotensive episodes reaction of MAO inhibitors.
b) Hypertensive crisis
c) Muscle flaccidity
d) Hypoglycemia
8 A nurse is performing a baseline d) CORRECT ANSWER Overall risk of developing
0. assessment of a client's skin pressure ulcers
integrity. What is the priority Reason: When assessing skin integrity, the overall
assessment parameter? risk potential of developing pressure ulcers takes
priority. Overall risk encompasses existing
a) Family history of pressure pressure ulcers as well as potential areas for
ulcers development of pressure ulcers. Family history
b) Presence of pressure ulcers isn't important when assessing skin integrity.
on the client
c) Potential areas of pressure
ulcer development
d) Overall risk of developing
pressure ulcers
8 A nurse is performing a c) CORRECT ANSWER Moodiness
1. psychosocial assessment on a Reason: Moodiness may occur often during early
14-year-old adolescent. Which adolescence. Frequent anger and combativeness
emotional response is typical are more typical of middle adolescence.
during early adolescence? Cooperativeness typically occurs during late
adolescence.
a) Frequent anger
b) Cooperativeness
c) Moodiness
d) Combativeness
8 A nurse is providing care for a d) CORRECT ANSWER be taught about diet.
2. pregnant client in her second Reason: The client will need to watch her
trimester. Glucose tolerance test overall diet intake to control her blood
results show a blood glucose level glucose level. The client's blood glucose level
of 160 mg/dl. The nurse should should be controlled initially by diet and
anticipate that the client will need exercise, rather than insulin. Oral antidiabetic
to: drugs aren't used in pregnant clients. Urine
glucose levels aren't an accurate indication of
a) start using insulin. blood glucose levels.
b) start taking an oral antidiabetic
drug.
c) monitor her urine for glucose.
d) be taught about diet.
8 The nurse is serving on the hospital a) CORRECT ANSWER Ethical standards are
3. ethics committee which is generally higher than those required by law.
considering the ethics of a proposal Reason: Some behavior that is legally allowed
for the nursing staff to search the might not be considered ethically appropriate.
room of a client diagnosed with Legal and ethical standards are often linked,
substance abuse while he is off the such as in the commandment "Thou shalt not
unit and without his knowledge. kill." Ethical standards are never irrelevant,
Which of the following should be though a client's safety or the safety of others
considered concerning the may pose an ethical dilemma for health care
relationship of ethical and legal personnel. Searching a client's room when
standards of behavior? they are not there is a violation of their
privacy. Room searches can be done with a
a) Ethical standards are generally primary health care provider's order and
higher than those required by law. generally are done with the client present.
b) Ethical standards are equal to
those required by law.
c) Ethical standards bear no
relationship to legal standards for
behavior.
d) Ethical standards are irrelevant
when the health of a client is at
risk.
8 The nurse meets with the client and c) CORRECT ANSWER "It's important for him
4. his wife to discuss depression and to take his medication so that the depression
the client's medication. Which of will not return or get worse."
the following comments by the wife Reason: Improved balance of
would indicate that the nurse's neurotransmitters is achieved with
teaching about disease process and medication. Clients with endogenous
medications has been effective? depression must take antidepressants to
prevent a return or worsening of depressive
a) "His depression is almost cured." symptoms. Depression is a chronic disease
b) "He's intelligent and won't need characterized by periods of remission;
to depend on a pill much longer." however, it is not cured. Depression is not
c) "It's important for him to take his dependent on the client's intelligence to will
medication so that the depression the illness away. Zoloft is not physically
will not return or get worse." addictive.
d) "It's important to watch for
physical dependency on Zoloft."
8 The nurse observes that the right b - when the blink reflex is absent or the eyes
5. eye of an unconscious client does do not close completely, the cornea may
not close completely. Which nursing become dry and irritated. Corneal abrasion
intervention is most appropriate can occur. Taping the eye closed will prevent
injury. Having the client wears eyeglasses or
a. have the client wear eyeglasses cleaning the eyelid will not protect the cornea
at all time from dryness or irritation. Artificial tears
b. lightly tape the eyelid shut instilled once per shift are not frequent
c. instill artificial tears once every enough for preventing dryness.
shift
d. clean the eyelid with a washcloth
every shift
8 A nurse preceptor is working a) CORRECT ANSWER "The action occurs in the
6. with a student nurse who is stomach by increasing the pH of the stomach
administering medications. contents and decreasing pepsin activity."
Which statement by the Reason: The action of an antacid occurs in the
student indicates an stomach. The anions of an antacid combine with the
understanding of the action acidic hydrogen cations secreted by the stomach to
of an antacid? form water, thereby increasing the pH of the stomach
contents. Increasing the pH and decreasing the
a) "The action occurs in the pepsin activity provide symptomatic relief from
stomach by increasing the peptic ulcer disease. Antacids don't work in the large
pH of the stomach contents or small intestine or in the esophagus.
and decreasing pepsin
activity."
b) "The action occurs in the
small intestine, where the
drug coats the lining and
prevents further ulceration."
c) "The action occurs in the
esophagus by increasing
peristalsis and improving
movement of food into the
stomach."
d) "The action occurs in the
large intestine by increasing
electrolyte absorption into
the system that decreases
pepsin absorption."
8 The nurse should instruct the c - children with hyperthyroidism experience
7. family of a child with newly emotional labiality that may strain interpersonal
diagnosed hyperthyroidism relationships. Focusing on one friend's is easier than
to: adapting to group dynamics until the child's condition
improves. Because of their high metabolic rate,
a. keep their home warmer children with hyperthyroidism complain of being too
than usual warm. Bright sunshine may be irritating because of
b. encourage plenty of disease-related ophthalmopathy. Sweating is
outdoor activities common and bathing should be encouraged.
c. promote interactions with
one friend instead of groups
d. limit bathing to prevent
skin irritation
8 Nurses teach infant care and d - Until the infant weighs 20 lb and is 1 year old, he
8. safety classes to assist should ride in a rear-facing car seat.
parents in appropriately
preparing to take their
neonates home. Which
statement about automobile
restraints for infants is
correct?

a) An infant should ride in a


front-facing car seat until he
weighs 20 lb (9.1 kg) and is 1
year old.
b) An infant should ride in a
rear-facing car seat until he
weighs 25 lb (11.3 kg) or is 1
year old.
c) An infant should ride in a
front-facing car seat until he
weighs 30 lb (13.6 kg) or is 2
years old.
d) An infant should ride in a
rear-facing car seat until he
weighs 20 lb and is 1 year
old.
8 A nurse takes informed consent a) CORRECT ANSWER Protects the client's right
9. from a client scheduled for to self-determination in health care decision
abdominal surgery. Which of the making.
following is the most appropriate Reason: Informed consent protects the client's
principle behind informed right to self-determination in health care
consent? decision making. Informed consent helps the
client to refuse a treatment that the client does
a) Protects the client's right to not wish to undergo and helps the client to gain
self-determination in health care in-depth knowledge about the treatment options
decision making. available, but the most important function is to
b) Helps the client refuse encourage shared decision making. Informed
treatment that he or she does not consent does not help the client to make a living
wish to undergo. will.
c) Helps the client to make a
living will regarding future health
care required.
d) Provides the client with in-
depth knowledge about the
treatment options available.
9 The nurse walks into a client's c) CORRECT ANSWER Straighten the client's
0. room to administer the 9:00 a.m. pillow behind his back.
medications and notices that the Reason: The nurse should first help the client
client is in an awkward position in into a position of comfort even though the
bed. What is the nurse's first primary purpose for entering the room was to
action? administer medication. After attending to the
client's basic care needs, the nurse can proceed
a) Ask the client his name. with the proper identification of the client, such
b) Check the client's name band. as asking the client his name and checking his
c) Straighten the client's pillow armband, so that the medication can be
behind his back. administered.
d) Give the client his
medications.
9 On the second postpartum day a d) CORRECT ANSWER afterpains.
1. gravida 6, para 5 complains of Reason: In a multiparous client, decreased
intermittent abdominal cramping. uterine muscle tone causes alternating
The nurse should assess for: relaxation and contraction during uterine
involution, which leads to afterpains. The client's
a) endometritis. symptoms don't suggest endometritis,
b) postpartum hemorrhage. hemorrhage, or subinvolution.
c) subinvolution.
d) afterpains.
9 A parent brings a 5-year-old child b) CORRECT ANSWER Check the website at the
2. to a vaccination clinic to prepare Center for Disease Control and Prevention
for school entry. The nurse notes (CDC).
that the child has not had any Reason: The CDC is the federal body that is
vaccinations since 4 months of ultimately responsible for vaccination
age. To determine the current recommendations for adults and children. A
evidence for best practices for division of the CDC, the Advisory Committee on
scheduling missed vaccinations Immunization Practices, reviews vaccination
the nurse should: evidence and updates recommendation on a
yearly basis. The CDC publishes current
a) Ask the primary care provider. vaccination catch-up schedules that are readily
b) Check the website at the available on their website. The lack of
Center for Disease Control and vaccinations is a strong indicator that the child
Prevention (CDC). probably does not have a primary care provider.
c) Read the vaccine If consulted, the pharmacist would most likely
manufacturer's insert. have to review the CDC guidelines that are
d) Contact the pharmacist. equally available to the nurse. Reading the
manufacturer's inserts for multiple vaccines
would be time consuming and synthesis of the
information could possibly lead to errors.
9 The physician ordered IV c - the nurse should monitor the client's
3. naloxone (Narcan) to reverse respirations closely for 4 to 6 hours because
the respiratory depression naloxone has a shorter duration of action than
from morphine administration. opioids. The client may need repeated doses of
After administration of the naloxone to prevent or treat a recurrence of the
naloxone the nurse should: respiratory depression. Naloxone is usually effective
in a few minutes; however, its effects last only 1 to
a. check respirations in 5 2 hours and ongoing monitoring of the client's
minutes because naxolone is respiratory rate will be necessary. The client's
immediately effective in dosage of morphine will be decreased or a new
relieving respiratory drug will be ordered to prevent another instance of
depression respiratory depression.
b. check respirations in 30
minutes because the effects of
morphine will have worn off by
then
c monitor respirations
frequently for 4 to 6 hours
because the client may need
repeated doses of naloxone
d. monitor respirations each
time the client receives
morphine sulfate 10 mg IM
9 A pregnant client in her third b - Chlorpromazine is a low-potency antipsychotic
4. trimester is started on that is likely to cause sun-sensitive skin. Therefore
chlorpromazine (Thorazine) 25 the client needs instructions about using sunscreen
mg four times daily. Which of with a sun protection factor of 25 or higher.
the following instructions is Typically, chlorpromazine is not associated with an
most important for the nurse increased risk of seizures. Although constipation is
to include in the client's a common adverse effect of this drug, it can be
teaching plan? managed with diet, fluids, and exercise. The drug
does not need to be discontinued. Chlorpromazine
a) "Don't drive because there's is associated with postural hypotension, not
a possibility of seizures hypertension. Additionally, if postural hypotension
occurring." occurs, safety measures, such as changing
b) "Avoid going out in the sun positions slowly and dangling the feet before
without a sunscreen with a sun arising, not stopping the drug, are instituted.
protection factor of 25."
c) "Stop the medication
immediately if constipation
occurs."
d) "Tell your doctor if you
experience an increase in
blood pressure."
9 A primigravid client gives b) CORRECT ANSWER clean and dry the neonate's
5. birth to a full-term girl. perineal area from front to back.
When teaching the client and Reason: When changing a female neonate's diaper,
her partner how to change the caregiver should clean the perineal area from
their neonate's diaper, the front to back to prevent infection and then dry the
nurse should instruct them area thoroughly to minimize skin breakdown. For a
to: male, the caregiver should clean and dry under and
around the scrotum. Because of anatomic factors, a
a) fold a cloth diaper so that female's diaper should have the double thickness
a double thickness covers toward the back. The diaper, not the neonate, should
the front. be positioned properly. Placing a disposable diaper
b) clean and dry the over a cloth diaper isn't necessary. The direction of
neonate's perineal area from urine flow can't be ensured.
front to back.
c) place a disposable diaper
over a cloth diaper to
provide extra protection.
d) position the neonate so
that urine will fall to the
back of the diaper.
9 Prochlorperazine a) CORRECT ANSWER Nausea.
6. (Compazine) is prescribed Reason: Prochlorperazine is administered
postoperatively. The nurse postoperatively to control nausea and vomiting.
should evaluate the drug's Prochlorperazine is also used in psychotherapy
therapeutic effect when the because of its effects on mood and behavior. It is not
client expresses relief from used to treat dizziness, abdominal spasms, or
which of the following? abdominal distention.

a) Nausea.
b) Dizziness.
c) Abdominal spasms.
d) Abdominal distention.
9 To prevent development of d - isoniazid competes for the available vitamin B6 in
7. peripheral neuropathies the body and leaves the client at risk for developing
associated with isoniazid neuropathies related to vitamin deficiency.
administration, the nurse Supplemental vitamin B6 is routinely prescribed to
should teach the client to: address this issue. Avoiding sun exposure is a
preventative measure to lower the risk of skin cancer.
a. avoid excessive sun Following a low-cholesterol diet lowers the
exposure individual's risk of developing atherosclerotic plaque.
b. follow a low-cholesterol Rest is important in maintaining homeostasis but has
diet no real impact on neuropathies.
c. obtain extra rest
d. supplement the diet with
pyridoxine (vitamin B6)
98. Total parenteral nutrition a - TPN is hypertonic, high-calorie, high-protein,
(TPN) is prescribed for a intravenous (IV) fluid that should be provided to
client who has recently had a clients without functional gastrointestinal tract
significant small and large motility, to better meet their metabolic needs and to
bowel resection and is support optimal nutrition and healing. TPN is ordered
currently not taking once daily, based on the client's current electrolyte
anything by mouth. The and fluid balance, and must be handled with strict
nurse should: aseptic technique (because of its high glucose
content, it is a perfect medium for bacterial growth).
a. administer TPN through a Also, because of the high tonicity, TPN must be
nasogastric or gastrostomy administered through a central venous access, not a
tube peripheral IV line. There is no specific need to
b. handle TPN using strict auscultate for bowel sounds to determine whether
aseptic technique TPN can safely be administered
c. auscultate for bowel
sounds prior to
administering TPN
d. designate a peripheral
intravenous (IV) site for TPN
administration
99. When assessing an elderly d) CORRECT ANSWER delayed gastric emptying.
client, the nurse expects to Reason: Aging-related physiologic changes include
find various aging-related delayed gastric emptying, decreased coronary artery
physiologic changes. These blood flow, an increased posterior thoracic curve,
changes include: and increased peripheral resistance.

a) increased coronary artery


blood flow.
b) decreased posterior
thoracic curve.
c) decreased peripheral
resistance.
d) delayed gastric emptying.
10 when caring for a client after a - after a renal biopsy, the client is maintained on
0. a closed renal biopsy, the strict bed rest in a supine position for at least 6 hours
nurse should: to prevent bleeding. If no bleeding occurs, the client
typically resumes general activity after 24 hours.
a. maintain the client on Urine output is monitored, but an indwelling catheter
strict bed rest in a supine is not typically inserted. A pressure dressing is
position for 6 hours applied over the site, but a sandbag is not necessary.
b. insert an indwelling Opioids to control pain would not be anticipated;
catheter to monitor urine local discomfort at a biopsy site can be controlled
output with analgesics.
c. apply a sandbag to the
biopsy site to prevent
bleeding
d. administer IV opioid
medications to promote
comfort
10 When developing a care plan for c) CORRECT ANSWER ensure access to spiritual
1. a client with a do-not- care providers upon the client's request.
resuscitate (DNR) order, a nurse Reason: Ensuring access to spiritual care, if
should: requested by the client, is an appropriate nursing
action. A nurse should continue to administer
a) withhold food and fluids. appropriate doses of pain medication as needed
b) discontinue pain to promote the client's comfort. A health care
medications. provider may not withhold food and fluids unless
c) ensure access to spiritual the client has a living will that specifies this
care providers upon the client's action. A DNR order does not mean that the
request. client does not require nursing care.
d) always make the DNR client
the last in prioritization of
clients.
10 When measuring the fundal b) CORRECT ANSWER At about the level of the
2. height of a primigravid client at client's umbilicus.
20 weeks' gestation, the nurse Reason: Measurement of the client's fundal
will locate the fundal height at height is a gross estimate of fetal gestational
which of the following points? age. At 20 weeks' gestation, the fundal height
should be at about the level of the client's
a) Halfway between the client's umbilicus. The fundus typically is over the
symphysis pubis and umbilicus. symphysis pubis at 12 weeks. A fundal height
b) At about the level of the measurement between these two areas would
client's umbilicus. suggest a fetus with a gestational age between
c) Between the client's 12 and 20 weeks. The fundal height increases
umbilicus and xiphoid process. approximately 1 cm/week after 20 weeks'
d) Near the client's xiphoid gestation. The fundus typically reaches the
process and compressing the xiphoid process at approximately 36 weeks'
diaphragm. gestation. A fundal height between the umbilicus
and the xiphoid process would suggest a fetus
with a gestational age between 20 and 36 weeks.
The fundus then commonly returns to about 4 cm
below the xiphoid owing to lightening at 40
weeks. Additionally, pressure on the diaphragm
occurs late in pregnancy. Therefore, a fundal
height measurement near the xiphoid process
with diaphragmatic compression suggests a fetus
near the gestational age of 36 weeks or older.
10 When obtaining the vital signs c - When ICP increases, Cushing's triad may
3. of a client with multiple develop, which involves decreased heart and
traumatic injuries, a nurse respiratory rates and increased systolic blood
detects bradycardia, bradypnea, pressure. Shock typically causes tachycardia,
and systolic hypertension. The tachypnea, and hypotension. In encephalitis, the
nurse must notify the physician temperature rises and the heart and respiratory
immediately because these rates may increase from the effects of fever on
findings may reflect which the metabolic rate. (If the client doesn't maintain
complication? adequate hydration, hypotension may occur.)
Status epilepticus causes unceasing seizures, not
a) Shock changes in vital signs.
b) Encephalitis
c) Increased intracranial
pressure (ICP)
d) Status epilepticus
10 When planning care for a b) CORRECT ANSWER 30-degree head elevation
4. client with a head injury, Reason: For clients with increased intracranial pressure
which position should the (ICP), the head of the bed should be elevated to 30
nurse include in the care degrees to promote venous outflow. Trendelenburg's
plan to enhance client position is contraindicated because it can raise ICP. Flat
outcomes? or neutral positioning is indicated when elevating the
head of the bed would increase the risk of neck injury
a) Trendelenburg's or airway obstruction. A side-lying position isn't
b) 30-degree head specifically a therapeutic treatment for increased ICP.
elevation
c) Flat
d) Side-lying
10 Which nursing action is a) CORRECT ANSWER Give a fluid bolus of 500 ml.
5. required before a client in Reason: One of the major adverse effects of epidural
labor receives epidural administration is hypotension. Therefore, a 500-ml fluid
anesthesia? bolus is usually administered to prevent hypotension in
the client who wishes to receive an epidural for pain
a) Give a fluid bolus of 500 relief. Assessing maternal reflexes, pupil response, and
ml. gait isn't necessary.
b) Check for maternal
pupil dilation.
c) Assess maternal
reflexes.
d) Assess maternal gait.
10 Which of the following a) CORRECT ANSWER "I will not drink alcohol for at
6. client statements least 1 year."
indicates that the client Reason: It is important that the client understand that
with hepatitis B alcohol should be avoided for at least 1 year after an
understands discharge episode of hepatitis. Sexual intercourse does not need
teaching? to be avoided, but the client should be instructed to use
condoms until the hepatitis B surface antigen
a) "I will not drink alcohol measurement is negative. The client will need to
for at least 1 year." restrict activity until liver function test results are
b) "I must avoid sexual normal; this will not occur within 1 to 2 weeks. Jaundice
intercourse." will subside as the client recovers; it is not a permanent
c) "I should be able to condition.
resume normal activity in
a week or two.
d) "Because hepatitis B is
a chronic disease, I know I
will always be jaundiced."
10 Which of the following c) CORRECT ANSWER Use warm sitz baths.
7. interventions would be Reason: Use of warm sitz baths can help relieve the
most appropriate for the rectal discomfort of hemorrhoids. Fiber in the diet
nurse to recommend to a should be increased to promote regular bowel
client to decrease movements. Laxatives are irritating and should be
discomfort from avoided. Decreasing physical activity will not decrease
hemorrhoids? discomfort.

a) Decrease fiber in the


diet.
b) Take laxatives to
promote bowel
movements.
c) Use warm sitz baths.
d) Decrease physical
activity.
10 Which of the following is d - In glaucoma, peripheral vision is impaired long
8. an early symptom of before central vision is impaired. Hazy, blurred, or
glaucoma? distorted vision is consistent with a diagnosis of
cataracts. Loss of central vision is consistent with senile
a) Hazy vision. macular degeneration but it occurs late in glaucoma.
b) Loss of central vision. Blurred or "sooty" vision is consistent with a diagnosis
c) Blurred or "sooty" of detached retina.
vision.
d) Impaired peripheral
vision.
10 Which of the following is a c) CORRECT ANSWER Maintain intact skin.
9. priority during the first 24 hours Reason: Maintaining intact skin is a priority for
of hospitalization for a the unconscious client. Unconscious clients need
comatose client with suspected to be turned every hour to prevent complications
drug overdose? of immobility, which include pressure ulcers and
stasis pneumonia. The unconscious client cannot
a) Educate regarding drug be educated at this time. Pain is not a concern.
abuse. During the first 24 hours, the unconscious client
b) Minimize pain. will mostly likely be on nothing-by-mouth status.
c) Maintain intact skin.
d) Increase caloric intake.
11 Which of the following c) CORRECT ANSWER Elevated white blood cell
0. laboratory findings are count.
expected when a client has Reason: Because of the inflammatory nature of
diverticulitis? diverticulitis, the nurse would anticipate an
elevated white blood cell count. The remaining
a) Elevated red blood cell laboratory findings are not associated with
count. diverticulitis. Elevated red blood cell counts occur
b) Decreased platelet count. in clients with polycythemia vera or fluid volume
c) Elevated white blood cell deficit. Decreased platelet counts can occur as a
count. result of aplastic anemias or malignant blood
d) Elevated serum blood urea disorders, as an adverse effect of some drugs,
nitrogen concentration. and as a result of some heritable conditions.
Elevated serum blood urea nitrogen
concentration is usually associated with renal
conditions.
11 Which of the following should a) CORRECT ANSWER Moist mucous membranes.
1. the nurse use to determine Reason: The outcome of moist mucous
achievement of the expected membranes indicates adequate hydration and
outcome for an infant with fluid balance, showing that the problem of fluid
severe diarrhea and a nursing volume deficit has been corrected. Although a
diagnosis of Deficient fluid normal bowel movement, ability to tolerate
volume related to passage of intravenous fluids, and an increasing time
profuse amounts of watery interval between bowel movements are all
diarrhea? positive signs, they do not specifically address
the problem of deficient fluid volume.
a) Moist mucous membranes.
b) Passage of a soft, formed
stool.
c) Absence of diarrhea for a 4-
hour period.
d) Ability to tolerate
intravenous fluids well.
11 Which of the following a) CORRECT ANSWER Activity is resumed
2. statements would provide the gradually, and the client can resume her usual
best guide for activity during activities in 5 to 6 weeks.
the rehabilitation period for a Reason: The scarring of the retinal tear needs
client who has been treated for time to heal completely. Therefore, resumption of
retinal detachment? activity should be gradual; the client may resume
her usual activities in 5 to 6 weeks. Successful
a) Activity is resumed gradually, healing should allow the client to return to her
and the client can resume her previous level of functioning.
usual activities in 5 to 6 weeks.
b) Activity level is determined
by the client's tolerance; she
can be as active as she wishes.
c) Activity level will be
restricted for several months,
so she should plan on being
sedentary.
d) Activity level can return to
normal and may include regular
aerobic exercises.
11 Which scenario complies with b - To provide interdisciplinary continuity of
3. Health Insurance Portability and care, nurses must share relevant information
Accountability Act of 1996 during client care conferences. Nurses
(HIPAA) regulations? discussing information in the cafeteria may be
overheard; this indiscretion violates HIPAA
a) Two nurses in the cafeteria are regulations. Looking up laboratory results for a
discussing a client's condition. neighbor is a HIPAA violation, as is discussing a
b) The health care team is client's condition with one's spouse.
discussing a client's care during a
formal care conference.
c) A nurse checks the computer
for the laboratory results of a
neighbor who has been admitted
to another floor.
d) A nurse talks with her spouse
about a client's condition.
11 While assessing a male neonate d - The condition in which the urinary meatus is
4. whose mother desires him to be located on the ventral surface of the penis,
circumcised, the nurse observes termed hypospadias, occurs in 1 of every 500
that the neonate's urinary meatus male infants. Circumcision is delayed until the
appears to be located on the condition is corrected surgically, usually
ventral surface of the penis. The between 6 and 12 months of age. Phimosis is
primary health care provider is an inability to retract the prepuce at an age
notified because the nurse when it should be retractable or by age 3
suspects which of the following? years. Phimosis may necessitate circumcision
or surgical intervention. Hydrocele is a painless
a. phimosis swelling of the scrotum that is common in
b. hydrocele neonates. It is not a contraindication for
c. epispadias circumcision. Epispadias occurs when the
d. hydrospadias urinary meatus is located on the dorsal surface
of the penis. It is extremely rare and is
commonly associated with bladder extrophy.
11 While assessing the fundus of a D - The nurse should place the non-dominant
5. multiparous client on the first hand above the symphysis pubis and the
postpartum day, the nurse dominant hand at the umbilicus to palpate the
performs hand washing and dons fundus. This prevents the uterine inversion and
clean gloves. Which of the trauma, which can be very painful to the client.
following should the nurse do The nurse should ask the client to assume a
next? supine, not side-lying, position with the knees
flexed. The fundus can be palpated in this
a. place the non-dominant hand position and the perineal pads can be
above the symphysis pubis and evaluated for lochia amounts. The fundus
the dominant hand at the should be massaged gently if the fundus feels
umbilicus boggy. Vigorous massaging may fatigue the
b. ask the client to assume a side- uterus and cause it to become firm and then
lying position with the knees boggy again. The nurse should ask the client to
flexed void before fundal evaluation. A full bladder
c. perform massage vigorously at can cause discomfort to the client, the uterus
the level of the umbilicus if the to be deviated to one side, and postpartum
fundus feels boggy hemorrhage.
d. place the client on a bedpan in
case the uterine palpation
stimulates the client to void

Question 1 See full question

The nurse is planning care for a client in restraints. Which nursing intervention is most important when restraining a
this client?

You Selected:

Checking that the restraints have been applied correctly

Correct response:

Checking that the restraints have been applied correctly

Explanation:

Question 2 See full question

A young female client is receiving chemotherapy and mentions to the nurse that she and her husband are using a
diaphragm for birth control. Which information is most important for the nurse to discuss?

You Selected:

infection control

Correct response:

infection control

Explanation:

Question 3 See full question

When planning home care for a 3-year-old child with eczema, what should the nurse teach the mother to remove
from the child's environment at home?

You Selected:

stuffed animals

Correct response:

stuffed animals

Explanation:

Question 4 See full question

A nurse is caring for a client with a fresh postoperative wound following a femoralpopliteal revascularization
procedure. The nurse fails to routinely assess the pedal pulses on the affected leg, and missed the warning sign
that the blood vessel was becoming occluded. The nurse manager is made aware of the complication and the
nurses failure to assess the client properly. What action should be taken by the nurse manager?

You Selected:

Address the nurses omissions as negligent behavior.

Correct response:

Address the nurses omissions as negligent behavior.

Explanation:

Question 5 See full question

A nurse is caring for a client after a hemorrhoidectomy. Which of the following orders would the nurse question on
the medical record?

You Selected:

Low-fiber diet

Correct response:

Low-fiber diet

Explanation:

Question 1 See full question

The nurse is receiving over the telephone a laboratory results report of a neonate's blood glucose level. The nurse
should:

You Selected:

write down the results, read back the results to the caller from the laboratory, and receive confirmation
from the caller that the nurse understands the results.

Correct response:

write down the results, read back the results to the caller from the laboratory, and receive confirmation
from the caller that the nurse understands the results.

Explanation:

Question 2 See full question

A nurse is planning care for an elderly client with cognitive impairment who is still living at home. Which action
should the nurse identify as a priority for safety in planning care for this client?

You Selected:

ensuring the removal of objects in the client's path that may cause him to trip

Correct response:

ensuring the removal of objects in the client's path that may cause him to trip

Explanation:

Question 1 See full question

A nurse is teaching the parents of a child with cystic fibrosis about proper nutrition.
Which instruction should the nurse include?
You Selected:

Encourage a high-calorie, high-protein diet.

Correct response:

Encourage a high-calorie, high-protein diet.

Explanation:

Question 2 See full question

A client is to receive a glycerin suppository. Which nursing action is appropriate


when administering a suppository?

You Selected:

Applying a lubricant to the suppository

Correct response:

Applying a lubricant to the suppository

Explanation:

Question 3 See full question

The breastfeeding mother of a 1-month-old diagnosed with cow's milk sensitivity


asks the nurse what she should do about feeding her infant. Which
recommendation would bemost appropriate?

You Selected:

Continue to breastfeed, but eliminate all milk products from your own diet.

Correct response:

Continue to breastfeed, but eliminate all milk products from your own diet.

Explanation:

Question 4 See full question

When teaching a client with chronic obstructive pulmonary disease to conserve


energy, the nurse should teach the client to lift objects:

You Selected:

while exhaling through pursed lips.


Correct response:

while exhaling through pursed lips.

Explanation:

Question 5 See full question

A nurse is teaching an elderly client about developing good bowel habits. Which
statement by the client indicates to the nurse that additional teaching is required?

You Selected:

"I need to use laxatives regularly to prevent constipation."

Correct response:

"I need to use laxatives regularly to prevent constipation."

Question 1 See full question

For a client with a nursing diagnosis of Insomnia, the nurse should use which
measure to promote sleep?

You Selected:

Playing soft or soothing music

Correct response:

Playing soft or soothing music

Explanation:

Question 2 See full question

A client is recovering from an infected abdominal wound. Which foods should the
nurse encourage the client to eat to support wound healing and recovery from the
infection?

You Selected:

chicken and orange slices

Correct response:

chicken and orange slices


Explanation:

Question 3 See full question

A client has just returned from the postanesthesia care unit after undergoing a
laryngectomy. Which intervention should the nurse include in the plan of care?

You Selected:

Maintain the head of the bed at 30 to 40 degrees.

Correct response:

Maintain the head of the bed at 30 to 40 degrees.

Explanation:

Question 4 See full question

Which measure would be mosteffective for the client to use at home when
managing the discomfort of rhinoplasty 2 days after surgery?

You Selected:

Apply ice compresses.

Correct response:

Apply ice compresses.

Explanation:

Question 5 See full question

A nurse is teaching a client with multiple sclerosis (MS). When teaching the client
how to reduce fatigue, the nurse should tell the client to:

You Selected:

rest in an air-conditioned room.

Correct response:

rest in an air-conditioned room.


Explanation:

Results
COMPLETED IN2m 17s
CORRECTLY ANSWERED1 of 5 questions
Take a Practice Quiz
See your Overall Performance
See your Quiz History

Performance by Client Needs


What's this?
Basic Care and Comfort
48 quizzes taken
3
Your mastery
3
Class average
View performance for all Client Needs

Answer Key
Question 1 See full question

A nurse suspects that a child, age 4, is being neglected physically. To best assess the
child's nutritional status, the nurse should ask the parents which question?

You Selected:

"Do you think your child eats enough?"

Correct response:

"What did your child eat for breakfast?"


Explanation:

Question 2 See full question

When developing a teaching plan for the mother of an infant about introducing
solid foods into the diet, which measure should the nurse expect to include in the
plan to help prevent obesity?

You Selected:

mixing cereal and fruit in a bottle when offering solid food for the first few times

Correct response:

decreasing the amount of formula or breast milk intake as solid food intake increases

Explanation:

Question 3 See full question

Which nursing recommendation ismost appropriate for a client to decrease


discomfort from hemorrhoids?

You Selected:

Use warm sitz baths.

Correct response:

Use warm sitz baths.

Explanation:

Question 4 See full question

A primiparous client is on a regular diet 24 hours postpartum. She is from


Guatemala and speaks only Spanish. The clients mother asks the nurse if she can
bring her daughter some special foods from home. The nurse responds, based on
the understanding about which principle?

You Selected:

This is permissible as long as the foods are nutritious and high in iron.

Correct response:

The mother can bring the daughter any foods that she desires.
Explanation:

Question 5 See full question

The nurse evaluates the client's understanding of nutritional modifications to


manage his hypertension when he states:

You Selected:

"I should eliminate caffeine from my diet to lower my blood pressure."

Correct response:

"Limiting my salt intake to 2 grams per day will improve my blood pressure."

Explanation:

Improve your mastery

Results
COMPLETED IN1m 44s
CORRECTLY ANSWERED2 of 5 questions
Take a Practice Quiz
See your Overall Performance
See your Quiz History

Performance by Client Needs


What's this?
Basic Care and Comfort
50 quizzes taken
3
Your mastery
3
Class average
View performance for all Client Needs
Answer Key
Question 1 See full question

A client who's 7 months pregnant reports severe leg cramps at night. Which nursing
action would be most effective in helping the client cope with these cramps?

You Selected:

Teaching her to dorsiflex her foot during the cramp

Correct response:

Teaching her to dorsiflex her foot during the cramp

Explanation:

Question 2 See full question

Before discharge from the hospital after a myocardial infarction, a client is taught to
exercise by gradually increasing the distance walked. Which vital sign should the
nurse teach the client to monitor to determine whether to increase or decrease the
exercise level?

You Selected:

blood pressure

Correct response:

pulse rate

Explanation:

Question 3 See full question

Sudoriferous glands secrete which type of substance?

You Selected:

Oil

Correct response:

Sweat
Explanation:

Question 4 See full question

Which is the most appropriate nursing intervention for a client with pruritus caused
by medications used to treat cancer?

You Selected:

administration of antihistamines

Correct response:

medicated cool baths

Explanation:

Question 5 See full question

In evaluating a clients response to nutrition therapy, which laboratory test would


be of highest priority to examine?

You Selected:

Albumin level

Correct response:

Albumin level

Explanation:

Improve your

Results
COMPLETED IN1m 42s
CORRECTLY ANSWERED3 of 5 questions
Take a Practice Quiz
See your Overall Performance
See your Quiz History

Performance by Client Needs


What's this?
Basic Care and Comfort
51 quizzes taken
3
Your mastery
3
Class average
View performance for all Client Needs

Answer Key
Question 1 See full question

After nasal surgery, the client expresses concern about how to decrease facial pain
and swelling while recovering at home. Which instruction would be most effective
for decreasing pain and edema?

You Selected:

Apply cold compresses to the area.

Correct response:

Apply cold compresses to the area.

Explanation:

Question 2 See full question

When caring for a client with trigeminal neuralgia, which intervention has the
highest priority?

You Selected:

Providing emotional support while the client adjusts to changes in his physical
appearance

Correct response:

Encouraging the client to bathe with care

Explanation:

Question 3 See full question

A primiparous client is on a regular diet 24 hours postpartum. She is from


Guatemala and speaks only Spanish. The clients mother asks the nurse if she can
bring her daughter some special foods from home. The nurse responds, based on
the understanding about which principle?

You Selected:

This is permissible as long as the foods are nutritious and high in iron.

Correct response:

The mother can bring the daughter any foods that she desires.

Explanation:

Question 4 See full question

The nurse is caring for a 3-year-old child with iron deficiency anemia and providing
dietary instructions to the parents. Which of the following should be a priority for
the nurse to include in the teaching?

You Selected:

Recommending lean meats

Correct response:

Recommending lean meats

Explanation:

Question 5 See full question

Which of the following observations by the nurse would indicate that a client is
unable to tolerate a continuation of a tube feeding?

You Selected:

Formula in the clients mouth during the feeding, and increased cough

Correct response:

Formula in the clients mouth during the feeding, and increased cough

Explanation:

Improve your mastery


Question 1 See full question

During a routine prenatal visit, a pregnant client reports constipation, and the nurse
teaches her how to relieve it. Which statement indicates the client's understanding
of the nurse's instructions?

You Selected:

"I'll increase my intake of unrefined grains."

Correct response:

"I'll increase my intake of unrefined grains."

Explanation:

Question 2 See full question

A nurse is completing the health history for a client who has been taking echinacea
for a head cold. The client asks, "Why is this not helping me feel better?" Which
response by the nurse would be the mostaccurate?

You Selected:

"There is limited information as to the effectiveness of herbal products."

Correct response:

"There is limited information as to the effectiveness of herbal products."

Explanation:

Question 3 See full question

The client newly diagnosed with type 1 diabetes mellitus eats a lot of pasta
products, such as macaroni and spaghetti, and asks if they can be included in the
diet. The nurse should tell the client:

You Selected:

Because you are overweight, it is better to eliminate pasta from your diet.

Correct response:

"Pasta can be a part of your diet. It is included in the bread and cereal exchange.
Explanation:

Question 4 See full question

The nurse teaches a client who has recently been diagnosed with hypertension
about following a low-calorie, low-fat, low-sodium diet. Which menu selection
would bestmeet the client's needs?

You Selected:

baked chicken, an apple, and a slice of white bread

Correct response:

baked chicken, an apple, and a slice of white bread

Explanation:

Question 5 See full question

The nurse is completing an intake and output record for a client who is receiving
continuous bladder irrigation after transurethral resection of the prostate. How
many milliliters of urine should the nurse record as output for her shift if the client
received 1,800 ml of normal saline irrigating solution and the output in the urine
drainage bag is 2,400 ml? Record your answer using a whole number.

Your Response:

2400

Correct response:

600

Explanation:

Question 3 See full question

A client with a cerebellar brain tumor is admitted to an acute care facility. The nurse formulates a nursing diagnosis
of Risk for injury. Which "related-to" phrase should the nurse add to complete the nursing diagnosis statement?

You Selected:

Related to impaired balance

Correct response:

Related to impaired balance


Explanation:

Question 4 See full question

When assessing a hospitalized client diagnosed with Major Depression and Borderline Personality Disorder, the
nurse should ask the client about which of the following first?

You Selected:

Suicidal thoughts.

Correct response:

Suicidal thoughts.

Explanation: Question 1 See full question

A nurse assists in writing a community plan for responding to a bioterrorism threat or attack. When reviewing the
plan, the director of emergency operations should have the nurse correct which intervention?

You Selected:

Clients exposed to anthrax should immediately remove contaminated clothing and place it in the hamper.

Correct response:

Clients exposed to anthrax should immediately remove contaminated clothing and place it in the hamper.

Explanation:

Question 2 See full question

A parent asks the nurse about using a car seat for a toddler who is in a hip spica cast. The nurse should tell the
parent:

You Selected:

"You will need a specially designed car seat for your toddler."

Correct response:

"You will need a specially designed car seat for your toddler."

Explanation:

Question 1 See full question

The nurse should carefully observe a client with internal radium implants for
typical adverse effects associated with radiation therapy to the cervix. These
effects include:
You Selected:

nausea and a foul vaginal discharge.


Correct response:

nausea and a foul vaginal discharge.


Explanation:

Question 2 See full question

A primiparous client who gave birth vaginally 8 hours ago desires to take a
shower. The nurse anticipates remaining nearby the client to assess for
which problem?
You Selected:

fainting
Correct response:

fainting
Explanation:

Question 3 See full question

A nurse on a night shift entered an elderly clients room during a scheduled


check and discovered the client on the floor beside her bed after falling when
trying to ambulate to the washroom. After assessing and assisting the client
back to bed, the nurse has completed an incident report. What is the primary
purpose of this particular type of documentation?
You Selected:

Identifying risks and ensuring future safety for clients.


Correct response:

Identifying risks and ensuring future safety for clients.


Explanation:

Question 4 See full question

What is the nurse expected to do when filing a report about an incident of


finding an elderly client with mild dementia on the floor?
You Selected:

The nurse must file an incident or adverse event report.


Correct response:

The nurse must file an incident or adverse event report.


Explanation:

Question 5 See full question

A nurse is conducting a teaching session with a group of parents on infant


care and safety to assist parents in appropriately preparing to take their
neonates home. Which statement about automobile restraints made by one
of the parents would indicate to the nurse that learning has taken place?
You Selected:

"An infant should ride in a rear-facing car seat until he or she weighs 20 lb (9.1
kg) or is 1 year old."
Correct response:

"An infant should ride in a rear-facing car seat until he or she has reached the
maximum weight allowed by their car seat manufacturer or is 2 years old."
Explanation:

Question 3 See full question

The nurse is discharging a baby with clubfoot who has had a cast applied. The nurse should provide additional
teaching to the parents if they state:

You Selected:

"I should call if I see changes in the color of the toes under the cast."

Correct response:

"I should use a pillow to elevate my child's foot as he sleeps."

Explanation:

Question 4 See full question

A client who was bitten by a wild animal is admitted to an acute care facility for treatment of rabies. Which type of
isolation does this client require?

You Selected:

Contact

Correct response:

Contact

Explanation:

Question 5 See full question

When assessing a hospitalized client diagnosed with Major Depression and Borderline Personality Disorder, the
nurse should ask the client about which of the following first?

You Selected:

Suicidal thoughts.
Correct response:

Suicidal thoughts.

Explanation:

1 A 48-year-old client with cancer has 30 mg.


. been receiving 10 mg of I.V. morphine Correct
while hospitalized. In order to give an Explanation:
equivalent dose of oral morphine, the There is a 1:3 ratio with equianalgesic
nurse should be sure the physician has dosing of I.V. to oral morphine; therefore,
ordered which of the following doses? the physician should order three times
the I.V. dose
a) 10 mg.
b) 40 mg.
c) 30 mg.
d) 25 mg.
2 After administering an I.M. injection, a With one hand, use the needle to scoop
. nurse notices there isn't a sharps- up the cap. Holding the barrel in one
disposal container nearby. Which action hand, carry the syringe to the closest
should the nurse take? sharps-disposal container.
When a sharps-disposal container isn't
nearby, a nurse should use the one-
handed scoop technique to prevent
needle-stick injury while transporting the
needle to a sharps-disposal container.
3 After having a total hip replacement, a Assess the client's understanding of the
. client receives morphine sulfate by PCA pump.
patient-controlled analgesia (PCA) Explanation:
pump. The client says, "This pump The nurse should assess the client's
doesn't help my pain at all." What understanding of the PCA pump because
should the nurse do in response to this the client may not correctly understand
statement? how to use it. If the client can be taught
how to properly use the PCA, other
a) Press the dose delivery button to give measures may not be necessay.
the client an immediate dose of the
drug.
b) Assess the client's understanding of
the PCA pump.
c) Tell the physician that the ordered
dose isn't sufficient for pain control.
d) Push the "Flush" button on the PCA
pump to make sure the I.V. line isn't
infiltrated.
4 At what time should the blood be just before the administration of the next IV dose
. drawn in relation to the Explanation:
administration of the IV dose of To determine how low the gentamicin serum
gentamicin sulfate? level drops between doses, the trough serum
level should be drawn just before the
a) 3 hours before the administration of the next IV dose of gentamicin
administration of the next IV sulfate.
dose
b) 4 hours before the
administration of the next IV
dose
c) just before the administration
of the next IV dose
d) 2 hours before the
administration of the next IV
dose
5 Because of a shortage of IV 17 drops per minute
. infusion pumps, a nurse must The accurate formula used to calculate drip rate
regulate a client's IV by gravity of the IV is volume per hour, divided by infusion
flow. The client has an order for time in minutes, multiplied by the drip factor of
1000 mL of 0.9 NSS to infuse at the tubing.
100 mL/hr. The tubing drip factor
is 10 drops/mL. What is the
appropriate rate for the nurse to
set the IV infusion?
6 Before advising a 24-year-old hypertension
. client desiring oral Correct
contraceptives for family Explanation:
planning, the nurse would assess Before advising a client about oral
the client for which signs and contraceptives, the nurse needs to assess the
symptoms? client for signs and symptoms of hypertension.
Clients who have hypertension, thrombophlebitis,
a) acne vulgaris obesity, or a family history of cerebral or
b) dysmenorrhea cardiovascular accident are poor candidates for
c) anemia oral contraceptives. In addition, women who
d) hypertension smoke, are older than 40 years of age, or have a
history of pulmonary disease should be advised
to use a different method.
7. A client begins taking haloperidol. After a dystonia.
few days, he experiences severe tonic Correct
contractures of muscles in his neck, Explanation:
mouth, and tongue. The nurse should These symptoms describe dystonia,
recognize this as: which commonly occurs after a few
days of treatment with haloperidol.
a) parkinsonism.
b) akathisia.
c) dystonia.
d) psychotic symptoms.
8. A client complains of difficulty swallowing Check for availability of a liquid
when the nurse tries to administer a preparation.
medication in capsule form. What action Explanation:
should the nurse take next? The nurse should find out whether the
medication is available in liquid form.
a) Break the capsule and mix the contents Dissolving or breaking the capsule
with applesauce. may interfere with drug action or
b) Withhold the medication. absorption
c) Check for availability of a liquid
preparation.
d) Dissolve the capsule in a full glass of
water.
9. A client diagnosed with schizophrenia for "I will go back and check the drawer
the last 2 years tells the nurse who has as well as telephone the pharmacy to
brought the morning medications, "That is check about any possible changes in
not my pill! My pill is blue, not green." the medication color."
What should the nurse tell the client? It is important for the nurse to listen
to the client and respect his or her
knowledge about the medication.
1 A client has an ordered intravenous 21
0. infusion that is ordered to infuse at 3000 Correct
mL of D5W in a 24-hour period (drop Explanation:
factor of 10). Calculate the drops per Volume to be infused times the drop
minute. Record your answer using a whole factor, divided by the number of
number. hours, times 60 minutes. 3000 times
10 divided by 24 times 60 equals
30,000 divided by 1440 = 21.
1 A client has a patient a controlled analgesia Inspect the infusion site.
1. (PCA) infusion to manage postoperative Explanation:
pain. In spite of receiving a dose of pain The nurse should first check the
medication, the client rates the pain at 8 on infusion site to be sure the site has
a 0 to 10 pain scale. What should the nurse not infiltrated. Next, the nurse should
do first? check the PCA pump to determine if
it is functioning properly.
a) Notify the health care provider (HCP).
b) Assess vital signs.
c) Inspect the infusion site.
d) Check the patient-controlled analgesia
(PCA) pump function.
1 A client has a patient a controlled analgesia Inspect the infusion site.
2. (PCA) infusion to manage postoperative Correct
pain. In spite of receiving a dose of pain Explanation:
medication, the client rates the pain at 8 on The nurse should first check the
a 0 to 10 pain scale. What should the nurse infusion site to be sure the site has
do first? not infiltrated.

a) Notify the health care provider (HCP).


b) Check the patient-controlled analgesia
(PCA) pump function.
c) Inspect the infusion site.
d) Assess vital signs.
1 A client has developed a hospital- Verify the medication order as written by
3. acquired pneumonia. When preparing the by the health care provider. (HCP).
to administer cephalexin 500 mg, the Return the cefazolin to the pharmacy.
nurse notices that the pharmacy sent Request that cephalexin be sent promptly.
cefazolin. What should the nurse do? Contact the pharmacy and speak to a
Select all that apply. pharmacist.
Correct
a) Request that cephalexin be sent Explanation:
promptly. One of the "five rights" of drug
b) Return the cefazolin to the administration is "right medication."
pharmacy. Cefazolin was not the medication
c) Administer the cefazolin. prescribed. The pharmacist is the
d) Verify the medication order as professional resource and serves as a check
written by the by the health care to ensure that clients receive the right
provider. (HCP). medication. Returning unwanted
e) Contact the pharmacy and speak medications to the pharmacy will decrease
to a pharmacist. the opportunity for a medication error by
the nurse who follows the current nurse. (
1 A client is asking about dietary "Increase your intake of calcium and vitamin
4. modifications to counteract the long- D."
term effects of prednisone. What is Problems associated with long-term
the most appropriate information for corticosteroid therapy include sodium
the nurse to give the client? retention, osteoporosis, and hyperglycemia.
An increase in calcium and vitamin D is
needed to help prevent bone deterioration.
Dietary modifications need to reduce
sodium, maintain high protein levels for
tissue repair, and reduce carbohydrates, as
there is a tendency toward hyperglycemia.
1 A client is brought to the emergency Administer 1 liter 0.9% saline IV.
5. department with abdominal trauma The client is demonstrating vital signs
following an automobile accident. consistent with fluid volume deficit, likely
The vital signs are as follows: heart due to bleeding and/or hypovolemic shock
rate, 132 bpm; respirations, 28 as a result of the automobile accident. The
breaths/min; blood pressure, 84/58 client will need intravenous fluid volume
mm Hg; temperature, 97.0 F (36.1 replacement using an isotonic fluid (e.g.,
C); oxygen saturation 89% on room 0.9% normal saline) to expand or replace
air. Which prescription should the blood volume and normalize vital signs.
nurse implement first?
1 A client is brought to the emergency Administer 1 liter 0.9% saline IV.
6. department with abdominal trauma Correct
following an automobile accident. The Explanation:
vital signs are as follows: heart rate, The client is demonstrating vital signs
132 bpm; respirations, 28 breaths/min; consistent with fluid volume deficit, likely
blood pressure, 84/58 mm Hg; due to bleeding and/or hypovolemic shock
temperature, 97.0 F (36.1 C); oxygen as a result of the automobile accident. The
saturation 89% on room air. Which client will need intravenous fluid volume
prescription should the nurse replacement using an isotonic fluid (e.g.,
implement first? 0.9% normal saline) to expand or replace
blood volume and normalize vital signs.
a) Draw a complete blood count (CBC)
with hematocrit and hemoglobin.
b) Administer 1 liter 0.9% saline IV.
c) Obtain an abdominal x-ray.
d) Insert an indwelling urinary
catheter.
1 A client is prescribed an I.V. solution of 83
7. 1,000 ml to be infused from 0800 to First, determine how many hours the
2000. The nurse will use an infusion infusion needs to run. 0800 to 2000 is 12
pump that delivers in milliliters per hours. Use the following equation to
hour. At what rate should the nurse determine the milliliters per hour: (volume
set the pump to deliver the solution? to infuse/infusion time) = flow rate per
Record your answer using a whole hour; (1,000 ml/12 hours) = 83.3 ml/hour
number (rounded to 83 ml/hour). The pump should
be set to deliver 83 ml/hour.
1 A client is prescribed buspirone 5 mg "Buspirone will help me not to worry so
8. two times a day. Which of the much."
following statements indicates that "I'll be able to focus better."
the client has understood the nurse's Buspirone is not a benzodiazepine but acts
teaching about this drug? Select all as a serotonin agonist. Serotonin is the
that apply. neurotransmitter implicated in depression.
Buspirone reduces symptoms of worry,
apprehension, difficulty with
concentration, and irritability.
1 A client is receiving total parenteral discontinue the current solution, change
9. nutrition (TPN). The nurse notices that the tubing, and hang a new bag of TPN
the bag of TPN solution has been solution.
infusing for 24 hours but has 300 mL Correct
of solution left. The nurse should: Explanation:
IV fluids should not be infused for longer
a) notify the health care provider than 24 hours because of the risk of
(HCP) and obtain prescriptions to alter bacterial growth in the solution. The
the flow rate of the solution. appropriate action for the nurse to take is
b) discontinue the current solution, to discontinue the current TPN solution,
change the tubing, and hang a new change the tubing, and hang a new bag of
bag of TPN solution. solution
c) continue the infusion until the
remaining 300 mL is infused.
d) change the filter on the tubing and
continue with the infusion.
2 A client is scheduled for surgery at 8 Notifying the surgeon that the client
0. a.m.(0800). While completing the hasn't signed the consent form
preoperative checklist, the nurse sees Notifying the surgeon takes priority
that the surgical consent form isn't because the physician must obtain
signed. It's time to administer the informed consent before the client
preoperative analgesic. Which nursing receives drugs that can alter cognition.
action takes the highest priority in this
situation?
2 A client is scheduled to receive a blood Assess the client for chills or low back
1. transfusion. In addition to taking vital pain.
signs and verifying that the unit of Stop the transfusion for reports of
blood cells is checked, what other dyspnea or itching.
assessments/actions would the nurse Explanation:
be responsible for? Select all that Checking for the possibility of transfusion
apply. reactions is an important responsibility.
Chills can be associated with blood
a) Stop the transfusion for reports of contamination, low back pain can be
dyspnea or itching. associated with incompatible blood, and
b) Rapidly transfuse the blood for the dyspnea and skin itching can be
first 15 minutes. associated with an allergic reaction. The
c) Transfuse the blood over 5 hours. transfusion would need to be stopped.
d) Assess the client for chills or low
back pain.
e) Reduce intake of fluids during the
transfusion.
2 A client is taking phenelzine 15 Obtain the client's vital signs
2. mg PO three times a day. The The nurse should first take the client's vital signs
nurse is about to administer the because the client could be experiencing a
next dose when the client tells hypertensive crisis, which requires prompt
the nurse about having a intervention. Signs and symptoms of a
throbbing headache. Which hypertensive crisis include occipital headache, a
action should the nurse do first? stiff or sore neck, nausea, vomiting, sweating,
dilated pupils and photophobia, nosebleed,
tachycardia, bradycardia, and constricting chest
pain.
2 A client reports pain in the right Call the physician to report the finding.
3. heel and is requesting The best response would be to notify the
medication. The nurse assesses physician. The nurse cannot repeat the dose of
the client and administers an analgesia without an order. Massaging the ankle
analgesic. The client experiences and applying moist heat would be inappropriate
no pain relief and states that the for a number of reasons. The client could be
heel pain is worse. What is an developing a deep vein thrombosis, which may
appropriate intervention by the dislodge an embolus. Unrelieved pain indicates
nurse? that an adverse event is developing, and the
physician should be made aware of the situation.
2 A client requests his medication Independent
4. at 9 p.m. (2100) instead of 10 Correct
p.m. (2200) so that he can go to Explanation:
sleep earlier. Which type of Nursing interventions are classified as
nursing intervention is required? independent, interdependent, or dependent.
Altering the drug schedule to coincide with the
a) Interdependent client's daily routine represents an independent
b) Dependent intervention, whereas consulting with the
c) Independent physician and pharmacist to change a client's
d) Intradependent medication because of adverse reactions
represents an interdependent intervention.
Administering an already ordered drug on time is
a dependent intervention.
2 A client's blood pressure is Go to the pharmacy to obtain the drug.
5. elevated at 160/90 mm Hg. The Although the nurse needs to obtain and
physician ordered "clonidine 1 mg administer the medication as soon as possible,
by mouth now." The nurse sent the it is inappropriate for the nurse to go to the
order to pharmacy at 7:10 a.m. pharmacy and request the drug without first
(0710), but the medication still has calling the pharmacy and checking to see
not arrived at 8 a.m.(0800). The whether the medication was delivered. The
nurse should do all except which of drug may have been delivered to several
the following? appropriate spots on the unit, such as the
client's drug bin, the transport system, or the
delivery box. The nurse should assess the
client's blood pressure to determine the
immediacy of the condition for which the
medication was ordered.
2 A client taking clozapine states, "I Report the client's symptoms to the physician
6. think I'm getting the flu. I have a after taking the client's temperature.
fever and feel weak." Which of the The nurse should take the client's temperature
following should the nurse do and report the symptoms to the physician.
next? Flulike symptoms of weakness, malaise, fever,
sore throat, and lethargy may indicate
leukopenia. An elevated temperature could
also indicate an infection.
2 A client who has been taking "You have developed a fungal infection from
7. flunisolide nasal spray, two your medication. It will need to be treated with
inhalations a day, for treatment of an antifungal agent."
asthma has painful, white patches Use of oral inhalant corticosteroids such as
in the mouth. What should the flunisolide can lead to the development of oral
nurse tell the client? thrush, a fungal infection. Once developed,
thrush must be treated by antifungal therapy;
it will not resolve on its own.
2 A client who is experiencing an 31
8. exacerbation of ulcerative colitis is Correct
receiving IV fluids that are to be Explanation:
infused at 125 mL/h. The IV tubing To administer I.V. fluids at 125 mL/h using
delivers 15 gtt/mL. How quickly tubing that has a drip factor of 15 gtt/mL, the
should the nurse infuse the fluids nurse should use the following formula:
in drops per minute to infuse the
fluids at the prescribed rate? 125 mL/60 min 15 gtt/1 mL = 31 gtt/min.
Record your answer using a whole
number.
2 A client who is receiving Lanoxin
9. multiple medications for a Signs of digitalis toxicity include cardiac
myocardial infarction reports dysrhythmias, anorexia, nausea, vomiting, and
severe nausea. Assessments visual disturbances. Cardiac dysrhythmias result
reveal that the heartbeat is from the inhibition, by digitalis, of myocardial Na+
irregular and slow. The nurse and K+.
should recognize these
symptoms as toxic effects of
which medication?
3 A client who is using a patient- "When morphine is used to alleviate severe pain
0. controlled analgesia (PCA) for 2 to 3 days, there is little likelihood of
pump after bowel surgery becoming addicted."
states, "I'm afraid that I'll Correct
become addicted if I use too Explanation:
much morphine." Which would Morphine is a narcotic. Clients need to understand
be the best response by the that when pain is present and morphine is used
nurse? therapeutically, there is less likelihood of
addiction. If morphine is taken in the absence of
a) "Have you had problems with pain, addiction can result. Telling the client that
drug addiction before?" morphine is not addicting is incorrect because,
b) "When morphine is used to although it acknowledges the addictive nature of
alleviate severe pain for 2 to 3 morphine, it does not inform the client regarding
days, there is little likelihood of its effect in pain management.
becoming addicted."
c) "Morphine is not addicting in
these circumstances. Why are
you worried about it?"
d) "You need to take the
morphine to help you rest and
recuperate from the surgery;
you can deal with the addiction
later."
3 A client who weighs 187 lb (85 kg) The physician's order is for the client to
1. has an order to receive enoxaparin 1 receive enoxaparin 1 mg/kg. Therefore, the
mg/kg. This drug is available in a client is to receive 85 mg. The desired dose
concentration of 30 mg/0.3 mL. What in milliliters then can be calculated by using
dose would the nurse administer in the formula of desired dose (D) divided by
milliliters? Record your answer using dose or strength of dose on hand (H) times
two decimal places. volume (V). 85 (mg) 0.3 mL = 25.5 mg/mL
25.5 mg divided by 30 = 0.85 mL.
3 A client who weighs 207 lb (94.1 kg) 141
2. is to receive 1.5 mg/kg of gentamicin 1.5 mg/kg/dose X 94.1 kg = 141.15
sulfate IV three times each day. How mg/dose, which rounds to 141 mg/dose.
many milligrams of medication
should the nurse administer for each
dose? Round to the nearest whole
number.
3 A client with a deep vein thrombosis 360
3. has heparin sodium infusing at 1,500 Correct
units/hour. The concentration of Explanation:
heparin is 25,000 units/500 mL. If the 25,000 u/500 ml = 50 units/ml. 1 ml/50 units
infusion remains at the same rate for x 1500 units/hour = 30 ml/hour x 12 hours =
a full 12 hour shift, how many 360 ml
milliliters of fluid will infuse? Record
your answer using a whole number.
3 Cross-tolerance to a drug is defined one drug reduces response to another drug.
4. as: Cross-tolerance occurs when a drug with a
similar action causes a decreased response
to another drug.
3 An elderly client who experiences reduced drug dosages
5. several adverse drug reactions may In older clients, diminished hepatic and renal
benefit from: function commonly reduces drug
metabolism and excretion. Because adverse
reactions are frequently related to drug
blood level, the client may benefit from
reduced drug dosages.
3 Gentamicin I.V. has been Ototoxicity
6. ordered to treat a client's Ototoxicity is a serious side effect of gentamicin.
infection. The nurse should Tinnitus and dizziness are common; irreversible
monitor the client for: deafness can develop if the onset of ototoxicity is
not detected early. Gentamicin is also known to be
nephrotoxic and hepatotoxic.
3 The health care provider (HCP) Ask the HCP to confirm that the prescription is
7. is calling in a prescription for correct.
ampicillin for a neonate. What Repeat the prescription to the HCP over the
should the nurse do? Select all telephone.
that apply. Write down the prescription.
Correct
a) Ask the nursing supervisor Explanation:
to cosign the telephone The nurse should write down the prescription, read
prescription as transcribed by the prescription back to the HCP, and receive
the nurse. confirmation from the provider that the
b) Ask the HCP to confirm that prescription is correct as understood by the nurse.
the prescription is correct.
c) Write down the prescription.
d) Ask the HCP to come to the
hospital and write the
prescription on the medical
record.
e) Repeat the prescription to
the HCP over the telephone.
3 If a client's central venous Clamp the catheter.
8. catheter accidentally becomes Correct
disconnected, what should a Explanation:
nurse do first? If a central venous catheter becomes
disconnected, the nurse should immediately apply
a) Call the physician. a catheter clamp. If a clamp isn't available, the
b) Apply a dry sterile dressing nurse may place a sterile syringe or catheter plug
to the site. in the catheter hub.
c) Tell the client to take and
hold a deep breath.
d) Clamp the catheter.
3 If a manual end-of-shift count of immediately report the discrepancy to the
9. controlled substances isn't nurse-manager, nursing supervisor, and
correct, the nurse's best action is pharmacy.
to: Explanation:
Reporting a noted discrepancy to the nurse-
a) investigate and correct the manager, nursing supervisor, and pharmacy
discrepancy, if possible, before should be the nurse's first step.
proceeding.
b) immediately report the
discrepancy to the nurse-
manager, nursing supervisor, and
pharmacy.
c) document the discrepancy on a
opioid-inventory form.
d) document the discrepancy on
an incident report.
4 An infusion of lidocaine 2
0. hydrochloride is running at 30 Correct
mL/hour. The dilution is 1,000 Explanation:
mg/250 mL. What dosage is the First, calculate the concentration of mg/mL:
client receiving per minute? 1,000 mg divided by 250 mL equals 4mg/mL.
Record your answer using a
whole number. Next, multiply the number of milligrams per
milliliter by the pump setting in milliliters per
hour: 4 mg/mL x 30 mL/h = 120 mg/h.

Next, divide the milligrams per hour by 60 to


obtain milligrams per minute: 120 mg/h divided
by 60 min/h equals 2 mg/min.
4 Metoclopramide is a medication Gastropathy.
1. prescribed for which of the Correct
following conditions? Explanation:
Metoclopramide is a dopamine antagonist that
a) Cardiomyopathy. stimulates motility of the upper gastrointestinal
b) Nephropathy. (GI) tract, increases lower esophageal sphincter
c) Encephalopathy. tone, and blocks dopamine receptors at the
d) Gastropathy. chemoreceptor trigger zone. It may be used for
delayed gastric emptying secondary to diabetic
gastroparesis.
4 A multiparous client tells the nurse Calcium.
2. that she is using The nurse should instruct the client to
medroxyprogesterone for increase her intake of calcium because
contraception. The nurse should there is a slight increase in the risk of
instruct the client to increase her osteoporosis with this medication. Weight-
intake of which of the following? bearing exercises are also advised. The
drug may also impair glucose tolerance in
women who are at risk for diabetes.
4 A nurse administers cefazolin instead "Reporting the error helps to identify
3. of ceftriaxone to an 8-year-old with system problems to improve client safety."
pneumonia. The client has suffered no Correct
adverse effects. The nurse tells the Explanation:
charge nurse of the incident but fears Client safety is enhanced when the
disciplinary action from reporting the emphasis on medication errors is to
error. The charge nurse should tell the determine the root cause. All errors should
nurse: be reported so systems can identify
patterns that contribute to errors. Here, the
a) "If you do not report the error, I will similar names probably contributed to the
have to." error. The nurse who commits the error
b) "Notify the client's health care knows all the relevant information and is in
provider to see if she wants this the best position to report it.
reported."
c) "This is not a serious mistake, so
reporting it will not affect your
position."
d) "Reporting the error helps to
identify system problems to improve
client safety."
4 The nurse administers theophylline to less difficulty breathing
4. a client. When evaluating the Correct
effectiveness of this medication, the Explanation:
nurse should assess the client for: Theophylline is a bronchodilator that is
administered to relax airways and
a) less difficulty breathing decrease dyspnea.
b) thinning of tenacious, purulent
sputum.
c) suppression of the client's
respiratory infection.
d) decrease in bronchial secretions.
4 The nurse establishes the goal Administering famotidine as ordered
5. of preventing the development Clients with burns are susceptible to the
of a stress ulcer in a burn development of Curling's ulcer, a gastroduodenal
client. Which of the following ulcer that is caused by a generalized stress
interventions would most likely response. The stress response results in increased
contribute to the achievement gastric acid secretion and a decreased production
of this goal? of mucus. Prevention is the best treatment, and
clients are frequently treated prophylactically with
antacids and H2 histamine blockers such as
famotidine (Pepcid).
4 The nurse establishes the goal administering famotidine as ordered
6. of preventing the development Explanation:
of a stress ulcer in a burn Clients with burns are susceptible to the
client. Which would most likely development of Curling's ulcer, a gastroduodenal
contribute to the achievement ulcer that is caused by a generalized stress
of this goal? response. The stress response results in increased
gastric acid secretion and a decreased production
a) implementing relaxation of mucus. Prevention is the best treatment, and
exercises clients are frequently treated prophylactically with
b) providing a soft, bland diet antacids and H2 histamine blockers such as
c) administering famotidine as famotidine.
ordered
d) administering a sedative as
needed
4 The nurse has an order to 0.4
7. administer 1200 mg of an
antibiotic. The drug is
prepared as 6 grams of the
drug in 2 ml of solution. The
nurse should administer how
many ml of the drug? Record
your answer using one decimal
place.
4 A nurse inadvertently gives a The prescriber
8. client a double dose of an Correct
ordered medication. After Explanation:
discovering the error, whom After discovering a medication error, the safety of
should the nurse notify first? the patient is top priority. The nurse should
immediately check the client and observe for any
a) The prescriber adverse effects which may develop. The first
b) The client person the nurse needs to notify is the prescriber,
c) The pharmacist followed by the nursing manager (or the nursing
d) The risk manage supervisor).
4 The nurse is administering a dilute the solution with water, milk, or fruit juice
9. saturated solution of potassium and have the client drink it with a straw.
iodide (SSKI). The nurse should: Correct
Explanation:
a) dilute the solution with water, SSKI should be diluted well in milk, water, juice,
milk, or fruit juice and have the or a carbonated beverage before administration
client drink it with a straw. to help disguise the strong, bitter taste. Also, this
b) pour the solution over ice drug is irritating to mucosa if taken undiluted.
chips.
c) disguise the solution in a
pureed fruit or vegetable.
d) mix the solution with an
antacid.
5 A nurse is administering acetaminophen.
0. sublingual nitroglycerin to a Correct
client. Immediately after Explanation:
administering nitroglycerin, the In the early stages of therapy, nitoglycerin
nurse should expect to commonly causes headache and dizziness.
administer: Acetaminophen usually helps decrease
nitroglycerin-induced headaches.
a) prednisone.
b) alprazolam.
c) insulin.
d) acetaminophen.
5 The nurse is aware that a client Postural or orthostatic hypotension
1. receiving morphine sulfate After the administration of certain
intravenously post-surgical antihypertensives or narcotics, the client's
repair of a hip fracture may neurocirculatory reflexes may have some
exhibit which of the following difficulty adjusting to the force of gravity when
outcomes when getting out of he/she assumes an upright position. Postural or
bed for the first time? orthostatic hypotension may then occur, causing
a temporarily decreased blood supply to the
brain.
5 The nurse is caring for a client Correct response: "I should stop taking
2. with peripheral artery disease clopidogrel if it makes me feel weak and dizzy."
who has recently been Explanation:
prescribed clopidogrel. The Weakness, dizziness, and headache are common
nurse understands that more adverse effects of clopidogrel and the client
teaching is necessary when the should report these to the health care provider
client states: (HCP) if they are problematic; in order to
decrease risk of clot formation, the drug must be
a) "I should stop taking taken regularly and should not be stopped or
clopidogrel if it makes me feel taken intermittently.
weak and dizzy."
b) "The health care provider
prescribed this medicine to make
my platelets less likely to stick
together and help prevent clots
from forming."
c) "I should not be surprised if I
bruise easier or my gums bleed a
little when brushing my teeth."
d) "It does not really matter if I
take this medicine with or
without food, whatever works
best for my stomach."
5 A nurse is developing a teaching tamoxifen.
3. plan for a client who has just Tamoxifen is an estrogen blocker used to treat
been diagnosed with breast premenopausal and postmenopausal breast
cancer. The nurse should include cancer and to prevent breast cancer in certain
information about: women who are at high risk.
5 The nurse is evaluating a One tablespoon equals 15 cc's of medication.
4. parent's understanding of
measuring one tablespoon of
medication in a medicine cup. At
which level on the medicine cup
would the nurse confirm an
appropriate dose?
5 The nurse is instructing the client the levothyroxine before breakfast and the
5. with hypothyroidism who takes other medications 4 hours later.
levothyroxine 100 mcg, digoxin, Correct
and simvastatin. Teaching Explanation:
regarding the use of these Levothyroxine) must be given at the same
medications is effective if the client time each day on an empty stomach,
will take: preferably to 1 hour before breakfast. Other
medications may impair the action of
a) the levothyroxine before levothyroxine absorption; the client should
breakfast and the other separate doses of other medications by 4 to 5
medications 4 hours later. hours.
b) all medications before going to
bed.
c) the levothyroxine with breakfast
and the other medications after
breakfast.
d) all medications together 1 hour
after eating breakfast.
5 The nurse is planning care for Contact the health care provider (HCP) who
6. a client with severe prescribed the medication.
postoperative pain. There is a Correct
prescription for morphine Explanation:
written as "10 mg MSO4" on The nurse should first contact the HCP because the
the medical record. What prescription for the morphine is not complete. The
should the nurse do first? Joint Commission of the United States and the
Institute for Safe Medication Practices Canada
a) Obtain an intravenous recommend not to use MSO4 because it can apply
infusion system. to morphine as well as to magnesium sulfate.
b) Contact the health care There is no mention of an IV system being needed.
provider (HCP) who prescribed The morphine should not be in the medication
the medication. cabinet because the prescription is not complete.
c) Contact the pharmacy
department.
d) Prepare the medication for
administration.
5 A nurse is preparing a 100 units of regular insulin in normal saline
7. continuous insulin infusion for solution
a child with diabetic Continuous insulin infusions use only short-acting
ketoacidosis and a blood regular insulin. Insulin is added to normal saline
glucose level of [800 mg/dl solution and administered until the client's blood
(44.4 mmol/L)]. Which solution glucose level falls.
is the most appropriate at the
beginning of therapy?
5 The nurse is reconstituting a roll the vial gently between her palms
8. powdered medication in a vial. Rolling the vial gently between the palms produces
After adding the solution to heat, which helps dissolve the medication.
the powder, the nurse should:
5 The nurse is reviewing the Clarify with the physician that the spray should
9. following physician's order be given in only one nostril per day.
written for a postmenopausal Calcitonin salmon nasal spray should be
woman: "calcitonin salmon nasal administered in only one nostril per day. Many
spray 200 IU, one spray every preprinted order sheets automatically print
day." What is the appropriate "administer in both nostrils" when a nasal spray
action to be taken by the nurse is ordered. Nurses must be familiar with the
regarding this order? directions for each medication they give before
administering medications.
6 The nurse is to administer 33
0. chloramphenicol 50 mg IV in 100 Correct
mL of dextrose 5% in water over Explanation:
30 minutes. The infusion set The flow rate is determined by the rate of
administers 10 gtt/mL. At what infusion and the number of drops per milliliter of
flow rate (in drops per minute) the fluid being administered: gtt/mL mL/min
should the nurse set the infusion? = IV flow rate (gtt/minute).
Round to the nearest whole
number. Therefore:

10 gtt/mL 100 mL/30 min = 33 gtt/min.


6 The nurse reads the new contact the prescriber for clarification.
1. medications prescriptions for a 4- Correct
year-old child with nephrotic Explanation:
syndrome on the chart as shown. There are many problems with this medication
The nurse should: prescription. The abbreviation QOD is
ambiguous and open to various interpretations.
a) discontinue the prednisolone The abbreviation D/C may be interpreted as
40 mg and give the 30-mg dose "discontinue" or "discharge." The prescriber
today. should have specifically stated when to start
b) contact the prescriber for the lower dose because the nurse could reason
clarification. beginning the medication that day, the next, or
c) check the medication record even the day after that. The only safe thing to
first to see when the last dose of do is call for clarification.
prednisolone was given.
d) start the 30-mg dose
tomorrow.
6 A nurse receives a lithium Within the therapeutic range.
2. level report of l.0 mEq/L (1 For the client who has been receiving lithium
mmol/L) for a client who has therapy for the past 2 months, a maintenance serum
been taking lithium for 2 lithium level of 0.6 to 1.2 mEq/L (0.6 to 1.2 mmol/L)
months. The nurse should is considered therapeutic. A lithium level greater
interpret this level to indicate than 1.2 mEq/L (1.2 mmol/L) suggests toxicity.
which of the following?
6 The nurse should establish Visual acuity.
3. baseline data on a client who Correct
is starting on long-term Explanation:
gentamicin sulfate therapy. Visual acuity is not affected by long-term gentamicin
Which of the following is least sulfate therapy. The nurse should establish baseline
important for assessment data for vestibular, renal, and auditory function
screening in this client? because gentamicin sulfate is ototoxic and causes
renal toxicity.
a) Visual acuity.
b) Auditory function.
c) Vestibular function.
d) Renal function.
6 The nurse should instruct the alcohol
4. client to avoid which drug Correct
while taking metoclopramide Explanation:
hydrochloride? Metoclopramide hydrochloride can cause sedation.
Alcohol and other central nervous system
a) antacids depressants add to this sedation. A client who is
b) anticoagulants taking this drug should be cautioned to avoid driving
c) antihypertensives or performing other hazardous activities for a few
d) alcohol hours after taking the drug.
6 The nurse should teach the International Normalized Ratio (INR) is used to
5. client who is receiving assess effectiveness.
warfarin sodium that: INR is the value used to assess effectiveness of the
warfarin sodium therapy. INR is the prothrombin
time ratio that would be obtained if the
thromboplastin reagent from the World Health
Organization was used for the plasma test. It is now
the recommended method to monitor effectiveness
of warfarin sodium. Generally, the INR for clients
administered warfarin sodium should range from 2
to 3.
6 The physician has prescribed Life-threatening ventricular
6. amiodarone for a client with dysrhythmias.
cardiomyopathy. The nurse should Cardiomyopathy means that the
monitor the client's rhythm to myocardium is weak and irritable.
determine the effectiveness of the Amiodarone is an antiarrhythmic and acts
medication in controlling: directly on the cardiac cell membrane. In
this situation, amiodarone is used to
increase the ventricular fibrillation
threshold. Amiodarone is contraindicated
in sinus node dysfunction, heart block,
and severe bradycardia.
6 A physician orders an infusion of 2,400 120 ml/hour
7. ml of I.V. fluid over 24 hours, with half First, the nurse determines how many
this amount to be infused over the first milliliters (half of the total) to administer
10 hours. During the first 10 hours, a over the first 10 hours: 2,400 ml 2 =
client should receive how many 1,200 ml. Then she determines how
milliliters of I.V. fluid per hour? many of these milliliters to deliver per
hour: 1,200 ml 10 hours = 120
ml/hour.
6 A physician orders normal saline 1.2
8. solution to infuse at a rate of 150 The client is to receive the solution at an
ml/hour for a client admitted with infusion rate of 150 ml/hour. 150 ml X 8
dehydration and pneumonia. How many hours = 1,200 ml, the total volume in
liters of solution will the client receive milliliters the client will receive during an
during an 8-hour shift? Record your 8-hour shift. Next, convert milliliters to
answer using one decimal place. liters by dividing by 1,000. The total
volume in liters of normal saline solution
that the client will receive in 8 hours is
1.2 L.
6 The physician prescribes 20.3
9. acetaminophen 650 mg by mouth every The following formula is used to calculate
4 hours for a client with a temperature drug dosages: dose on hand/quantity on
of 102 F (38.8 C) who has a feeding hand = dose desired/X. In this example,
tube in place. The nurse has the equation is as follows: 160 mg/5 ml =
acetaminophen solution on hand 650 mg/X X = 20.3 ml
containing 160 mg/5 ml. How many
milliliters of solution should the nurse
administer? Record your answer using
one decimal place.
7 A short time after administering pain Carefully review the order and medication
0. medication to a client, the nurse label, then calculate the ordered dose.
returns to the client's room and finds The nurse should always take the time to
the client difficult to arouse. The nurse identify the client, carefully review the
realizes that 25 ml of the liquid medication order, read the medication
medication was administered instead label, and calculate the ordered dose.
of the ordered 25 mg, which is Consistently following these steps helps
contained in 5 ml. How could the nurse prevent medication administration errors.
have prevented this error?
7 The sudden onset of which of the Noisy respirations.
1. following indicates a potentially serious A serious complication of I.V. therapy is
complication for the client receiving an fluid overload. Noisy respirations can
I.V. infusion? develop as a result of pulmonary
congestion. Additional symptoms of fluid
overload include dyspnea, crackles,
hypertension, bounding pulse, and
distended neck veins
7 Three days after surgery, a client The nurse should ask the client about
2. continues to take hydrocodone 7.5 mg having a bowel movement because
and acetaminophen 500 mg for acetaminophen with hydrocodone is an
postoperative pain. What should the opioid, which can be constipating. By the
nurse ask the client before third day, many clients become
administering the pain medication? constipated and are feeling distended,
with sharp, cramping pain due to gas,
which is treated with ambulation, not
more opioids.
7 When a nurse tries to Exploring how the client's feelings affect his/her
3. administer medication, the decision to refuse medication
client refuses it, saying, "I don't Correct
have to take those pills if I Explanation:
don't want to." What By helping the client explore his/her feelings about
intervention by the nurse would his/her change in health status, the nurse can
have the highest priority? determine how these feelings affect his/her
decision to refuse medication. Then the nurse can
a) Explaining the consequences help the client develop new ways to satisfy self-
of not taking the medication, care, esteem, and other needs and, ultimately,
such as a negative outcome participate fully in the therapeutic regimen
b) Exploring how the client's
feelings affect his/her decision
to refuse medication
c) Reporting the client's
comments to the physician and
the treatment team
d) Insisting that the client take
the medication because it is
specifically ordered for the
client
7 When developing a teaching Blood pressure
4. plan for a client taking The incidence of hypertension is three to six times
hormonal contraceptives, a greater in clients using hormonal contraceptives
nurse should ensure that the than in women who don't use these drugs. Age
client knows she must have and duration of the drug's use increase this
which vital sign monitored incidence.
regularly?
7 When preparing the teaching maintaining an adequate sodium intake.
5. plan for a client about lithium Correct
therapy, the nurse should teach Explanation:
the client about: The nurse would teach the client taking lithium
and his family about the importance of
a) increasing sodium in the maintaining adequate sodium intake to prevent
diet. lithium toxicity. Because lithium is a salt, reduced
b) maintaining an adequate sodium intake could result in lithium retention with
sodium intake. subsequent toxicity.
c) buying foods labeled "low in
sodium."
d) discontinuing sodium in the
diet.

Improve your mastery


Question 1 See full question

A client newly admitted to a psychiatric inpatient setting demands a soda from a staff member who tells him to wait
until lunch arrives in 20 minutes. The client becomes angry, pushes over a sofa, throws an end table, and dumps a
potted plant. Which goal should a nurse consider to be of primary importance?

You Selected:

Demonstrating control over aggressive behavior

Correct response:

Demonstrating control over aggressive behavior

Explanation: Question 1 See full question

The nurse is helping a family plan for the discharge of their child, who will be
going home in a spica cast. Which information should be most important for
the nurse to consider?
You Selected:

A 16-year-old sister will care for the child during the day.
Correct response:

The child's bedroom is on the second floor.


Explanation:

Question 2 See full question

A client is admitted to the facility with a productive cough, night sweats, and
a fever. Which action is most important in the initial care plan?
You Selected:

Placing the client in respiratory isolation


Correct response:

Placing the client in respiratory isolation


Explanation:

Question 3 See full question

A read-back procedure has been implemented on a nursing unit to prevent


discrepancies in telephone prescriptions and reports. This procedure should
be implemented when the:
You Selected:

nurse receives a critical lab value via phone or in-person from the lab.
Correct response:

nurse receives a critical lab value via phone or in-person from the lab.
Explanation:

Question 4 See full question

In a client infected with human immunodeficiency virus (HIV) has a low CD4
level. What interventions should the nurse implement as a result of this
finding?
You Selected:

Place the client in reverse isolation.


Correct response:

Place the client in reverse isolation.


Explanation:

Question 5 See full question

A nurse is conducting a teaching session with a group of parents on infant


care and safety to assist parents in appropriately preparing to take their
neonates home. Which statement about automobile restraints made by one
of the parents would indicate to the nurse that learning has taken place?
You Selected:

"An infant should ride in a rear-facing car seat until he or she weighs 20 lb (9.1
kg) or is 1 year old."
Correct response:

"An infant should ride in a rear-facing car seat until he or she has reached
the maximum weight allowed by their car seat manufacturer or is 2 years
old." Question 1 See full question

A physician orders cefoxitin, 1 g in 100 ml of 5% dextrose in water, to be


administered I.V. A nurse determines that the recommended infusion time is 15 to
30 minutes. The available infusion set has a calibration of 10 drops/ml. To infuse
cefoxitin over 30 minutes, which drip rate should the nurse use?

You Selected:

30 drops/minute

Correct response:

33 drops/minute

Explanation:

Question 2 See full question

A client with acute pyelonephritis receives a prescription for co-trimoxazole P.O.


twice daily for 10 days. Which finding best demonstrates that the client has
followed the ordered regimen?

You Selected:

Bacteria are absent on urine culture.

Correct response:

Bacteria are absent on urine culture.


Explanation:

Question 3 See full question

A client has been treated for major depression and is taking antidepressants. He
asks the nurse, "How long do I have to take these pills?" The nurse should tell the
client:

You Selected:

"Antidepressants are prescribed for 6 to 12 months before considering


discontinuation."

Correct response:

"Antidepressants are prescribed for 6 to 12 months before considering


discontinuation."

Explanation:

Question 4 See full question

Twenty-four hours after giving birth to a term neonate, a primipara receives


acetaminophen with codeine for perineal pain. One hour after administering the
medication, which finding should alert the nurse to the development of a possible
side effect?

You Selected:

dizziness

Correct response:

dizziness

Explanation:

Question 5 See full question

The nurse is aware that a client receiving morphine sulfate intravenously post-
surgical repair of a hip fracture may exhibit which of the following outcomes when
getting out of bed for the first time?

You Selected:

Postural or orthostatic hypotension


Correct response:

Postural or orthostatic hypotension

Explanation:

Question 1 See full question

After receiving an oral dose of codeine for an intractable cough, a client asks the
nurse, "How long will it take for this drug to work?" How should the nurse respond?

You Selected:

30 minutes

Correct response:

30 minutes

Explanation:

Question 2 See full question

The nurse is administering packed red blood cells (PRBCs) to a client. The nurse
should first:

You Selected:

stay with the client during the first 15 minutes of infusion.

Correct response:

stay with the client during the first 15 minutes of infusion.

Explanation:

Question 3 See full question

A child with asthma has a heart rate of 160 bpm and a respiratory rate of 36
breaths/minute. The child appears restless and anxious and is given albuterol via
nebulizer. Which finding would indicate that the nebulizer treatment has been
effective?

You Selected:

increase in peak expiratory flow rate


Correct response:

increase in peak expiratory flow rate

Explanation:

Question 4 See full question

The client with heart failure asks the nurse about the reason for taking enalapril
maleate. The nurse should tell the client:

You Selected:

This drug will constrict your blood vessels and keep your blood pressure from getting
too low.

Correct response:

This drug will dilate your blood vessels and lower your blood pressure.

Explanation:

Question 5 See full question

Methylphenidate hydrochloride has been prescribed for a child with attention


deficit hyperactivity disorder. The nurse should make which of the following
statements to the child's parents? Select all that apply.

You Selected:

You will see a positive response to methylphenidate hydrochloride in approximately 8


weeks.
If the symptoms do not improve, the medication may need to be adjusted.
If discontinued, methylphenidate hydrochloride must be tapered off slowly.

Correct response:

If discontinued, methylphenidate hydrochloride must be tapered off slowly.


If the symptoms do not improve, the medication may need to be adjusted.

Explanation:

Question 1 See full question

The nurse notes that a client with acute pancreatitis occasionally experiences
muscle twitching and jerking. How should the nurse interpret the significance of
these symptoms?
You Selected:

The client may be developing hypocalcemia.

Correct response:

The client may be developing hypocalcemia.

Explanation:

Question 2 See full question

A client with type 1 diabetes presents with a decreased level of consciousness and a
fingerstick glucose level of 39 mg/dl (2.2 mmol/L). His family reports that he has
been skipping meals in an effort to lose weight. Which nursing intervention is most
appropriate?

You Selected:

Administering 1 ampule of 50% dextrose solution, per physician's order

Correct response:

Administering 1 ampule of 50% dextrose solution, per physician's order

Explanation:

Question 3 See full question

After completing assessment rounds, which client should the nurse discuss with the
health care provider (HCP) first?

You Selected:

a client with hepatitis whose pulse was 84 bpm and regular and is now 118 bpm and
irregular

Correct response:

a client with hepatitis whose pulse was 84 bpm and regular and is now 118 bpm and
irregular

Explanation:

Question 4 See full question

An infant is receiving rehydration therapy via the intravenous (IV) route for
treatment of dehydration related to diarrhea. The parent informs the nurse she
wants to hold the infant but is afraid of dislodging the IV. Which of the following
interventions should the nurse provide to alleviate this fear and promote bonding?

You Selected:

Apply a stabilization device so that the IV will not dislodge.

Correct response:

Apply a stabilization device so that the IV will not dislodge.

Explanation:

Question 5 See full question

The nurse is caring for an elderly client with a fractured hip who is on bed rest.
Which nursing interventions would be included on the plan of care?

You Selected:

Turn the client every 2 hours, and encourage coughing and deep breathing.

Correct response:

Turn the client every 2 hours, and encourage coughing and deep breathing.

Explanation:

Question 1 See full question

Before preparing a client for surgery, the nurse assists in developing a teaching
plan. What is the primary purpose of preoperative teaching?

You Selected:

To reduce the risk of postoperative complications

Correct response:

To reduce the risk of postoperative complications

Explanation:

Question 2 See full question

As a representative of the treatment team, a nurse is reviewing results of diagnostic


studies with the family of an adolescent with anorexia nervosa. What explanation
should the nurse give the family about the client's abnormal blood urea nitrogen
(BUN) value?

You Selected:

"The BUN is decreased because your daughter has developed hypothyroidism."

Correct response:

"The BUN is elevated because your daughter is dehydrated."

Explanation:

Question 3 See full question

A 27-year-old primigravid client with insulin-dependent diabetes at 34 weeks'


gestation undergoes a nonstress test, the results of which are documented as
reactive. What should the nurse tell the client that the test results indicate?

You Selected:

There is evidence of fetal well-being.

Correct response:

There is evidence of fetal well-being.

Explanation:

Question 4 See full question

A client is scheduled for an arteriogram. The nurse should explain to the client that
the arteriogram will confirm the diagnosis of occlusive arterial disease by:

You Selected:

showing the location of the obstruction and the collateral circulation.

Correct response:

showing the location of the obstruction and the collateral circulation.

Explanation:

Question 5 See full question

After resuming feedings in an infant who has undergone a pyloroplasty, which


action would be most appropriate?
You Selected:

Starting feedings with 5 to 10 ml, slowly increasing amounts as tolerated.

Correct response:

Starting feedings with 5 to 10 ml, slowly increasing amounts as tolerated.

Explanation:

Question 1 See full question

A term neonate's mother is O-negative, and cord studies indicate that the neonate
is A-positive. Which finding indicates that the neonate developed hemolytic
disease?

You Selected:

Signs of kernicterus

Correct response:

Signs of kernicterus

Explanation:

Question 2 See full question

A 27-year-old primigravid client with insulin-dependent diabetes at 34 weeks'


gestation undergoes a nonstress test, the results of which are documented as
reactive. What should the nurse tell the client that the test results indicate?

You Selected:

There is evidence of fetal well-being.

Correct response:

There is evidence of fetal well-being.

Explanation:

Question 3 See full question

A young child who has undergone a tonsillectomy refuses to let the nurse look at
the tonsillar beds to check for bleeding. To assess whether the child is bleeding
from the tonsillar beds, which measure would be most appropriate?
You Selected:

Observe for frequent swallowing.

Correct response:

Observe for frequent swallowing.

Explanation:

Question 4 See full question

A client has undergone an amputation of three toes and a femoral-popliteal bypass.


The nurse should teach the client that after surgery which leg position is
contraindicated while sitting in a chair?

You Selected:

crossing the legs

Correct response:

crossing the legs

Explanation:

Question 5 See full question

The nurse is teaching an adolescent with celiac disease about dietary changes that
will help maintain a healthy lifestyle. Which of the following foods can the nurse
safely recommend as part of the adolescents diet? Select all that apply.

You Selected:

Apples
Potatoes
Corn

Correct response:

Potatoes
Apples
Corn
Explanation:

Question 1 See full question

A client is at the end of her first postpartum day. The nurse is assessing the client's
uterus. Which finding requires further evaluation?

You Selected:

Fundus two fingerbreadths above the umbilicus

Correct response:

Fundus two fingerbreadths above the umbilicus

Explanation:

Question 2 See full question

A client is scheduled for oral cholecystography. Prior to the test, the nurse should:

You Selected:

ask the client about possible allergies to iodine or shellfish.

Correct response:

ask the client about possible allergies to iodine or shellfish.

Explanation:

Question 3 See full question

The nurse is teaching an older adult how to prevent falls. The nurse should tell the
client to:

You Selected:

instruct the client to rise slowly from a supine position.

Correct response:

instruct the client to rise slowly from a supine position.

Explanation:

Question 4 See full question

The nurse's discharge teaching plan for the client with heart failure should
emphasize the importance of:
You Selected:

obtaining daily weights at the same time each day.

Correct response:

obtaining daily weights at the same time each day.

Explanation:

Question 5 See full question

A clients arterial blood gas values are shown. The nurse should monitor the client
for:

You Selected:

metabolic acidosis

Correct response:

metabolic acidosis

Explanation:

Question 1 See full question

A neonate begins to gag and turns a dusky color. What should the nurse do first?

You Selected:

Aspirate the neonate's nose and mouth with a bulb syringe.

Correct response:

Aspirate the neonate's nose and mouth with a bulb syringe.

Explanation:

Question 2 See full question

A 27-year-old primigravid client with insulin-dependent diabetes at 34 weeks'


gestation undergoes a nonstress test, the results of which are documented as
reactive. What should the nurse tell the client that the test results indicate?

You Selected:

There is evidence of fetal well-being.


Correct response:

There is evidence of fetal well-being.

Explanation:

Question 3 See full question

A nurse assessing a client who underwent cardiac catheterization finds the client
lying flat on the bed. His temperature is 99.8 F (37.7 C). His blood pressure is
104/68 mm Hg. His pulse rate is 76 beats/minute. The nurse assesses the limb and
detects weak pulses in the leg distal to the puncture site. Skin on the leg is cool to
the touch. The puncture site is dry, but swollen. What is the most appropriate
action for the nurse to take?

You Selected:

Contact the physician and report the findings.

Correct response:

Contact the physician and report the findings.

Explanation:

Question 4 See full question

A nurse is providing discharge teaching to a client who is immunosuppressed.


Which statement by the client indicates the need for additional teaching?

You Selected:

"I can eat whatever I want as long as it's low in fat."

Correct response:

"I can eat whatever I want as long as it's low in fat."

Explanation:

Question 5 See full question

A child with Down syndrome has an upper respiratory infection (URI). Which of the
following is the nurses best action? Select all that apply.

You Selected:

Providing fluids that the child likes


Consulting a speech therapist
Administering oxygen

Correct response:

Providing fluids that the child likes


Restricting visitation of sick siblings
Ensuring that child is as active as possible

Explanation: Question 1 See full question

A neonate begins to gag and turns a dusky color. What should the nurse do first?

You Selected:

Aspirate the neonate's nose and mouth with a bulb syringe.

Correct response:

Aspirate the neonate's nose and mouth with a bulb syringe.

Explanation:

Question 2 See full question

A 27-year-old primigravid client with insulin-dependent diabetes at 34 weeks'


gestation undergoes a nonstress test, the results of which are documented as
reactive. What should the nurse tell the client that the test results indicate?

You Selected:

There is evidence of fetal well-being.

Correct response:

There is evidence of fetal well-being.

Explanation:

Question 3 See full question

A nurse assessing a client who underwent cardiac catheterization finds the client
lying flat on the bed. His temperature is 99.8 F (37.7 C). His blood pressure is
104/68 mm Hg. His pulse rate is 76 beats/minute. The nurse assesses the limb and
detects weak pulses in the leg distal to the puncture site. Skin on the leg is cool to
the touch. The puncture site is dry, but swollen. What is the most appropriate
action for the nurse to take?
You Selected:

Contact the physician and report the findings.

Correct response:

Contact the physician and report the findings.

Explanation:

Question 4 See full question

A nurse is providing discharge teaching to a client who is immunosuppressed.


Which statement by the client indicates the need for additional teaching?

You Selected:

"I can eat whatever I want as long as it's low in fat."

Correct response:

"I can eat whatever I want as long as it's low in fat."

Explanation:

Question 5 See full question

A child with Down syndrome has an upper respiratory infection (URI). Which of the
following is the nurses best action? Select all that apply.

You Selected:

Providing fluids that the child likes


Consulting a speech therapist
Administering oxygen

Correct response:

Providing fluids that the child likes


Restricting visitation of sick siblings
Ensuring that child is as active as possible

Explanation:
Explanation:

A 2-year-old Suction the tracheostomy.


child with a
tracheostomy Rationale: Diaphoresis, increased heart rate,
suddenly increased respiratory effort, and decreased
becomes oxygen saturation are signs that mucus is
diaphoretic partially occluding the airway. Therefore, the
and has an nurse should suction the tracheostomy first to
increased prevent full occlusion. Turning the child to a
heart rate, an side-lying position won't remove mucus from
increased work the airway. The child may require pain
of breath, and medication after his airway has been cleared if
a decreased his condition warrants it. Chest physiotherapy
oxygen will help drain excess mucus from the lungs but
saturation not from a tracheostomy.
level. What
should the
nurse do first?
2. A 4-year-old child ALL affects all blood-
has recently been forming organs and
diagnosed with systems throughout the
acute lymphocytic body.
leukemia (ALL). Adverse effects of
What information treatment include
about ALL should sleepiness, alopecia,
the nurse provide and stomatitis.
when educating the There's a 95% chance
client's parents? of obtaining remission
Select all that apply. with treatment.
3. A 6-year-old boy is "I want to know more
being treated in the about your situation.
emergency Let's sit and talk."
department for
injuries inflicted by Rationale: The nurse
his stepfather. The needs to obtain more
client's mother says, information before
"This never plans are developed.
happened before. Therefore, asking to
Jim got fired today. know more about the
He got drunk and situation is most
came home in a appropriate. The nurse
tirade. I'm so sorry has no way of
that Jason got hurt, predicting whether
but I don't think it abuse will occur again.
will ever happen Therefore, it is
again." Which of the inappropriate for the
following responses nurse to agree with the
is most appropriate mother, stating that the
initially? abuse probably will not
happen again. Filing
charges and a formal
report may be needed,
but more information is
needed first. These
actions would not be
done without the
mother's understanding
why.
4. A 7-year-old child is Elevated antibody
admitted to the concentration.
hospital with the
medical diagnosis of Rationale: Exactly why
acute rheumatic rheumatic fever follows
fever. Which of the a streptococcal
following laboratory infection is not known,
blood findings but it is theorized that
confirms that the an antigen-antibody
child has had a response occurs to an
streptococcal M protein present in
infection? certain strains of
streptococci. The
antibodies developed
by the body attack
certain tissues such as
in the heart and joints.
Antistreptolysin O titer
findings show elevated
or rising antibody
levels. This blood
finding is the most
reliable evidence of a
streptococcal infection.
5. An 8-year-old child Frequent clearing of
has just returned the throat
from the operating Frequent swallowing
room after having a Bright red vomitus
tonsillectomy. The
nurse is preparing to Rationale: A classic sign
do a postoperative of bleeding after
assessment. The tonsillectomy is
nurse should be frequent swallowing;
alert for which signs this sign occurs
and symptoms of because blood drips
bleeding? Select all down the back of the
that apply. throat, tickling it. Other
signs include frequent
clearing of the throat
and vomiting of bright
red blood. Vomiting of
dark blood may be seen
if the child swallowed
blood during surgery
but doesn't indicate
postoperative bleeding.
Breathing through the
mouth is common
because of dried
secretions in the nares.
Sleeping for long
intervals is normal after
a client receives
sedation and
anesthesia. A pulse rate
of 98 beats/minute is in
the normal range for
this age-group.
6. A 10-year-old boy is Obtain vital signs with a
24 hours post pain score.
appendectomy. He is
awake, alert, and Rationale: The child is
oriented. He tells in pain and needs
the nurse that he is intervention, but before
experiencing pain. the nurse can
He has an order for determine how to
morphine 1 to 2 mg proceed, it is essential
prn for pain. What is to know the client's
the priority nursing pain score to determine
action in managing the appropriate
the child's pain? morphine dose. In
addition, the nurse
cannot evaluate the
effectiveness of the
pain medication if there
is no score prior to
administering the
medication. Changing
the child's position and
administering pain
medication may help to
relieve the child's pain,
but the nurse must first
know the severity of
the pain before
determining the
appropriate
intervention. The nurse
must perform a head-
to-toe assessment;
however, it is not the
priority in managing
the child's pain.
7. A 10-year-old child who is 5' 4" (138 The child needs to use a short-acting
cm) tall with a history of asthma uses inhaled beta2-agonist medication.
an inhaled bronchodilator only when
needed. He takes no other Rationale: The peak flow of 180 L/minute is
medications routinely. His best peak in the yellow zone, or 50% to 80% of the
expiratory flow rate is 270 L/minute. child's personal best. This means that the
The child's current peak flow reading child's asthma is not well controlled, thereby
is 180 L/minute. The nurse interprets necessitating the use of a short-acting
this reading as indicating which of beta2-agonist medication to relieve the
the following? bronchospasm. A peak flow reading greater
than 80% of the child's personal best (in this
case, 220 L/minute or better) would indicate
that the child's asthma is in the green zone
or under good control. A peak flow reading
in the red zone, or less than 50% of the
child's personal best (135 L/minute or less),
would require notification of the health care
provider or a trip to the emergency
department. Cromolyn sodium is not used
for short-term treatment of acute
bronchospasm. It is used as part of a long-
term therapy regimen to help desensitize
mast cells and thereby help to prevent
symptoms.
8. A 20-year-old client diagnosed with Potential for medication non-compliance.
schizophrenia is recovering from his
first psychotic break. Before Rationale: Though disturbed thoughts and
discharge from the hospital, the sensory perceptions would be a concern to
client becomes depressed and the nurse, as well as communication issues,
states, "I don't want this illness. I'm the primary issue for this client in terms of
about to begin my junior year in his comments would be the potential for
college." Which of the following medication noncompliance and relapse.
issues would be most important for Most college students want to be like their
the nurses to address at this time? peers and perceive themselves as capable
and well. These beliefs can lead a young
client with schizophrenia to stop taking
medication which leads to relapse.
9. An 80-year-old client with severe Suggest to the daughter that she discuss
kidney damage is placed on life her father's wishes with her mother.
support and dialysis. Care decisions
are being made by his wife, who is Rationale: The most appropriate action is to
showing signs of early Alzheimer's encourage the daughter to talk to her
disease. The client's daughter arrives mother about the end-of-life issues first to
from out of town with a copy of the reach a consensus or agreement. This is a
client's living will, which states that family decision. Immediately informing the
the client did not want to be on life physician or preparing to remove the client
support. The nurse should: from life support would be premature if the
family is not in agreement. Although a copy
of the living will should be on the client's
chart, it is up to the daughter to show it to
her mother.
1 Acute Coronary Syndrome or MI or M-Morphine
0. sudden onset of chest pain O-Oxygen
treatment N-Nitroglycerin
A- ASPIRIN
1 An adolescent child is admitted to "Our child is just trying to get attention."
1. the nursing unit after an attempted "Our child would not do this again."
suicide. The nurse is discussing the "Our child will be fine in a couple of days."
attempted suicide with the parents.
Which of the following statements by Rationale: Suicide should not be seen just as
the parents indicate to the nurse attention-seeking behavior. It has very
that the parents need more serious consequences and should never be
teaching? Select all that apply. minimized. To believe that such an attempt
might not happen again or that the
adolescent will have resolved the problems
that led to the attempt in a couple of days
shows a lack of understanding of the
seriousness of the situation.
1 After a thoracotomy, the nurse Deep breathing expands the alveoli and
2. instructs the client to perform deep- increases the lung surface available for
breathing exercises. Which of the ventilation.
following is an expected outcome of
these exercises? Rationale: Deep breathing helps prevent
microatelectasis and pneumonitis and also
helps force air and fluid out of the pleural
space into the chest tubes. More than half of
the ventilatory process is accomplished by
the rise and fall of the diaphragm. The
diaphragm is the major muscle of
respiration; deep breathing causes it to
descend, not elevate, thereby increasing the
ventilating surface. Deep breathing
increases blood flow to the lungs; however,
the primary reason for deep breathing is to
expand alveoli and prevent atelectasis. The
remaining lobe naturally hyperinflates to fill
the space created by the resected lobe. This
is an expected phenomenon.
1 After doing well for a period of Relatives are especially grieved when a child
3. time, a child with leukemia does well at first but then declines rapidly.
develops an overwhelming
infection. The child's death is Rationale: It has been found that parents are
imminent. Which of the following more grieved when optimism is followed by
statements offers the nurse the defeat. The nurse should recognize this when
best guide in making plans to planning various ways to help the parents of a
assist the parents in dealing with dying child. It is not necessarily true that
their child's imminent death? knowing about a poor prognosis for years helps
prepare parents for a child's death. Death is
still a shock when it occurs. Trust in health care
personnel is not necessarily destroyed when a
death is untimely if the family views the
personnel as having done all that was possible.
It is not more difficult for parents to accept the
death of an older child than that of a younger
child.
1 After giving birth to an 8-lb (3.6- iron-fortified infant formula
4. kg) girl, a client asks the nurse
what her daughter should receive Rationale: For a bottle-fed neonate, the first
for the first feeding. For a bottle- feeding usually consists of iron-fortified
fed neonate, the first feeding formula. It isn't necessary to start with sterile
usually consists of: water or glucose water.
1 After instructing a primigravid Hydrocephalic infant.
5. client at 38 weeks' gestation
about how preeclampsia can affect Rationale: Congenital anomalies such as
the client and the growing fetus, hydrocephalus are not associated with
the nurse realizes that the client preeclampsia. Conditions such as stillbirth,
needs additional instruction when prematurity, abruptio placentae, intrauterine
she says that preeclampsia can growth restriction, and poor placental perfusion
lead to which of the following? are associated with preeclampsia. Abruptio
placentae occurs because of severe
vasoconstriction. Intrauterine growth restriction
is possible owing to poor placental perfusion.
Poor placental perfusion results from increased
vasoconstriction.
1 After placing an infant with The axillary temperature remains between 97
6. myelomeningocele in an isolette and 98 F (36.1 and 36.7 C).
shortly after birth, which indicator
should the nurse use as the best Rationale: The nurse places the neonate with
way to determine the myelomeningocele in an isolette shortly after
effectiveness of this intervention? birth to help to maintain the infant's
temperature. Because of the defect, the
neonate cannot be bundled in blankets.
Therefore, it may be difficult to prevent cold
stress. The isolette can be maintained at higher
than room temperature, helping to maintain
the temperature of a neonate who cannot be
dressed or bundled. Body temperature
readings, not arterial oxygen levels, are the
best indicator. Typically, an infant loses 5% to
10% of body weight before beginning to regain
the weight.
1 After teaching the parents of a "We'll wrap him in a blanket if he starts
7. child with febrile seizures about shivering."
methods to lower temperature
other than using medication, Rationale: Shivering, the body's defense
which of the following statements against rapid temperature decrease, results in
indicates successful teaching? an increase in body temperature. Therefore the
parents need to take measures to stop the
shivering (and the resulting increase in body
temperature) by increasing the room
temperature or the temperature of the child's
immediate environment (such as with blankets)
until the shivering stops. Then, attempts are
made to lower the temperature more slowly.
Shivering does not necessarily correlate with
being cold. Alcohol, a toxic substance, can be
absorbed through the skin. Its use is to be
avoided.
1 After teaching the parents of an "The nerves at the end of the large colon are
8. infant diagnosed with missing."
Hirschsprung's disease, the nurse
determines that the parents
understand the diagnosis when
the father states which of the
following?
1 Before abdominal surgery for an Inadequate fluid replacement.
9. intestinal obstruction, the nurse
monitors the client's urine output and Rationale: Considering that there is usually
finds that the total output for the 1 L of insensible fluid loss, this client's
past 2 hours was 35 ml. The nurse output exceeds his intake (intake, 2,000 ml;
then assesses the client's total intake output, 2,200 ml), indicating deficient fluid
and output over the last 24 hours and volume. The kidneys are concentrating
notes that he had 2,000 ml of I.V. urine in response to low circulating volume,
fluid for intake, 500 ml of drainage as evidenced by a urine output of less than
from the nasogastric tube, and 700 30 ml/hour. This indicates that increased
ml of urine for a total output of 1,200 fluid replacement is needed. Decreasing
ml. These findings indicate which of urine output can be a sign of decreased
the following? renal function, but the data provided
suggest that the client is dehydrated. Pain
does not affect urine output. There are no
data to suggest that the obstruction has
worsened.
2 A child with hemophilia is brought to Elevate the right knee.
0. the clinic with spontaneous soft
tissue bleeding of the right knee. Rationale: The goal is to decrease the
Immediately on the child's arrival, bleeding. This can be aided by decreasing
what should the nurse do? circulation to the area. Elevating the part
and applying cold decreases circulation to
the area. The child will also receive
cryoprecipitate. Aspirin is contraindicated
for a child with a bleeding disorder because
it increases capillary fragility. The
dependent position will increase bleeding
and swelling, and the goal is to decrease
bleeding. Lack of clotting factors, not lack
of platelets, is the problem in children with
hemophilia.
2 A client admitted for treatment of a "You sound concerned about what's
1. colon tumor, asks, "Do I have cancer?" happening."
Which response by the nurse would
be best?
2 A client has a newly created The client touches the altered body part.
2. colostomy. After participating in a
teaching session with the nurse and Rationale: By touching the altered body
receiving support from the spouse, part, the client recognizes the body change
the client decides to change the and establishes that the change is real.
colostomy pouch unaided. Which Talking about the surgery and making menu
behavior suggests that the client is choices shows the nurse that the teaching
beginning to accept the change in was successful, but does not show
body image? acceptance. Asking the spouse to leave the
room signifies that the client is ashamed of
the change and not coping with it.
2 A client has an ileal conduit. Which of Vinegar.
3. the following solutions will be useful
to help control odor in the urine Rationale: A distilled vinegar solution acts
collecting bag after it has been as a good deodorizing agent after an
cleaned? appliance has been cleaned well with soap
and water. If the client prefers, a
commercial deodorizer may be used. Salt
solution does not deodorize. Ammonia and
bleaching agents may damage the
appliance.
2 A client has a nursing diagnosis of Orthostatic blood pressure changes
4. fluid volume deficit. Which of the
following nursing assessment Rationale: Fluid volume deficit is
findings would support this characterized by hypotension, tachycardia,
diagnosis? increased body temperature, and
weakness. Leathery, pliable skin may not
demonstrate fluid deficit; it may reflect
diabetes. Pitting edema and pedal pulses of
4+ demonstrate localized edema and
potential fluid excess.
2 A client has been placed on long-term Green, leafy vegetables.
5. sulfasalazine therapy for treatment of
ulcerative colitis. The nurse should Rationale: In long-term sulfasalazine
encourage the client to eat which of therapy, the client may develop folic acid
the following foods to help avoid the deficiency. The client can take folic acid
nutrient deficiencies that may supplements, but the nurse should also
develop as a result of this encourage the client to increase the intake
medication? of folic acid in his diet. Green, leafy
vegetables are a good source of folic acid.
Citrus fruits, eggs, and milk products are
not good sources of folic acid.
2 A client in the fourth stage of "You may use the bathroom with my assistance."
6. labor asks to use the bathroom
for the first time since giving Rationale: The nurse should tell the client that
birth. The client has oxytocin she may use the bathroom with the nurse's
infusing. Which response by the assistance. The nurse should assist the client for
nurse is best? the client's first trip to the bathroom after giving
birth because it's common for a client to faint
after birth. Telling the client she must wait until
her vaginal bleeding stops is inappropriate;
vaginal bleeding continues for about 6 weeks
after childbirth. The nurse shouldn't tell the
client she can get up whenever she needs to use
the bathroom; doing so places the client at risk
for injury.
2 A client is admitted for suspected cirrhosis.
7. GI disease. Assessment data
reveal muscle wasting, a Rationale: Muscle wasting, a decrease in chest
decrease in chest and axillary and axillary hair, and increased bleeding
hair, and increased bleeding tendencies are all symptoms of cirrhosis. The
tendency. The nurse suspects the client may also have mild fever, edema,
client has: abdominal pain, and an enlarged liver. Clients
with peptic ulcer disease complain of a dull,
gnawing epigastric pain that's relieved by
eating. Appendicitis is characterized by a
periumbilical pain that moves to the right lower
quadrant and rebound tenderness. Cholelithiasis
is characterized by severe abdominal pain that
presents several hours after a large meal.
2 A client is admitted to the Notify the physician.
8. hospital after vomiting bright red
blood and is diagnosed with a Rationale: The client is experiencing a
bleeding duodenal ulcer. The perforation of the ulcer and the nurse should
client develops a sudden, sharp notify the physician immediately. The body
pain in the mid-epigastric region reacts to perforation of an ulcer by immobilizing
along with a rigid, boardlike the area as much as possible. This results in
abdomen. The nurse should do boardlike abdominal rigidity, usually with
which of the following first? extreme pain. Perforation is a medical
emergency requiring immediate surgical
intervention because peritonitis develops
quickly after perforation. Administering pain
medication is not the first action, although the
nurse later should institute measures to relieve
pain. Elevating the head of the bed will not
minimize the perforation. A nasogastric tube
may be used following surgery
2 A client is being discharged to a Avoid any hip flexion exercises.
9. transitional rehabilitation care Place two pillows between the client's knees.
facility following a hip Place a raised toilet seat in the bathroom.
replacement due to degenerative
arthritis. When reporting to the Rationale: The hip is one of the body's largest
LPN, which nursing actions would joints. In a total hip replacement, the damaged
the orthopedic nurse stress as bone and cartilage are removed and replaced
essential? Select all that apply. with prosthetic components. Until healing
occurs, the legs must be spread outward
(abducted) away from the body by placing
pillows or an abductor foam wedge between the
legs. Adduction of the hip or flexion greater than
90 may dislocate the prosthesis from the joint.
Raising the head of the bed 90 creates
excessive hip flexion. Using a raised toilet seat is
appropriate to avoid bending. The client will be
out of bed for physical therapy once to twice
daily. Keeping the feet elevated is not part of the
hip replacement protocol.
3 A client is diagnosed with Behavior Therapy
0. agoraphobia. Which type of
therapy is most effective for this Rationale: The nurse should suggest behavior
illness? therapy, which is most successful for clients with
phobias. Systematic desensitization, flooding,
exposure, and self-exposure treatments are
most therapeutic for clients with phobias. Self-
exposure treatment is being increasingly used to
avoid frequent therapy sessions. Insight therapy,
exploration of the dynamics of the client's
personality, is not helpful because the process of
anxiety underlies the disorder. Group therapy or
psychoanalysis, which deals with repressed,
intrapsychic conflicts, is not helpful for the client
with phobias because it does not help to
manage the underlying anxiety or disorder.
3 A client is diagnosed with syndrome Restricting fluids to 800 ml/day
1. of inappropriate antidiuretic
hormone (SIADH). Laboratory results Rationale: Excessive release of antidiuretic
reveal serum sodium level 130 mEq/L hormone (ADH) disturbs fluid and electrolyte
and urine specific gravity 1.030. balance in SIADH. The excessive ADH
Which nursing intervention helps causes an inability to excrete dilute urine,
prevent complications associated retention of free water, expansion of
with SIADH? extracellular fluid volume, and
hyponatremia. Symptomatic treatment
begins with restricting fluids to 800 ml/day.
Vasopressin is administered to clients with
diabetes insipidus a condition in which
circulating ADH is deficient. Elevating the
head of the bed decreases vascular return
and decreases atrial-filling pressure, which
increases ADH secretion, thus worsening the
client's condition. The client's sodium is low
and, therefore, shouldn't be restricted.
3 The client is scheduled to have a No special preparation is required for the
2. kidney, ureter, and bladder (KUB) examination.
radiograph. To prepare the client for
this procedure, the nurse should Rationale: A KUB radiographic examination
explain to the client that: ordinarily requires no preparation. It is
usually done while the client lies supine and
does not involve the use of radiopaque
substances.
3 A client presents with blistering Irrigate the wounds with water.
3. wounds caused by an unknown
chemical agent. How should the Rationale: The nurse should begin treatment
nurse intervene? by irrigating the wounds with water.
Delaying treatment until the agent is
identified allows the agent to cause further
tissue damage. Washing the wounds with
soap and water might cause a chemical
reaction that may further damage tissue.
The client may require I.V. fluid; however,
the wounds should be irrigated first.
3 The client who experiences angina Spaghetti with tomato sauce, salad, and
4. has been told to follow a low- coffee.
cholesterol diet. Which of the
following meals would be best? Rationale: Pasta, tomato sauce, salad, and
coffee would be the best selection for the
client following a low-cholesterol diet.
Hamburgers, milkshakes, liver, and fried
foods tend to be high in cholesterol.
3 A client with acute diarrhea is Paregoric 5 ml P.O.
5. requesting an as-needed medication
for loose, watery stools. After Rationale: Paregoric helps decrease
reviewing the physician's orders, peristalsis and diarrhea caused by muscle
which medication should the nurse spasms of the GI tract. Morphine sulfate,
administer? chlorpheniramine polistirex and
hydrocodone polistirex, and alprazolam
aren't indicated for diarrhea.
3 A client with a history of an anterior pulmonary crackles.
6. wall myocardial infarction is being
transferred from the coronary care Rationale: High pulmonary artery wedge
unit (CCU) to the cardiac step-down pressures are diagnostic for left-sided heart
unit (CSU). While giving a report to failure. With left-sided heart failure,
the CSU nurse, the CCU nurse says, pulmonary edema can develop causing
"His pulmonary artery wedge pulmonary crackles. In left-sided heart
pressures have been in the high failure, hypotension may result and urine
normal range." The CSU nurse should output will decline. Dry mucous membranes
be especially observant for: aren't directly associated with elevated
pulmonary artery wedge pressures.
3 A client with a Sengstaken- Affirm airway obstruction by the tube.
7. Blakemore tube has a sudden drop in Deflate the tube by cutting with bedside
SpO2 and increase in respiratory rate scissors.
to 40 breaths/minute. The nurse Remove the tube.
should do which of the following in Apply oxygen via face mask.
order from first to last?
3 The client with breast cancer is "This drug has been found to decrease
8. prescribed tamoxifen 20 mg daily. metastatic breast cancer."
The client states she does not like
taking medicine and asks the nurse if Rationale: Tamoxifen is an antiestrogen drug
the tamoxifen is really worth taking. that has been found to be effective against
The nurse's best response is which of metastatic breast cancer and to improve the
the following? survival rate. The drug causes hot flashes as
an adverse effect.
3 A client with refractory angina up-to-date partial thromboplastin time (PTT)
9. pectoris is scheduled for a result in his record.
percutaneous transluminal
coronary angioplasty (PTCA). The Rationale: Clients undergoing PTCA receive
cardiologist orders an infusion of abciximab because it inhibits platelet
abciximab. Before beginning the aggregation and, thereby, reduces cardiac
infusion, the nurse should ensure ischemic complications. Before abciximab is
the client has: administered, the client should have an up-to-
date PTT result available. The drug isn't
contraindicated in clients with a seizure
history. Abciximab isn't an opioid; therefore,
an opioid antagonist doesn't need to be at the
bedside. Any client with refractory angina
should be on continuous ECG monitoring;
however, monitoring isn't a requirement for
administering abciximab.
4 The client with rheumatoid arthritis "Every person is different. What works for one
0. tells the nurse, "I have a friend who client may not always be effective for
took gold shots and had a another."
wonderful response. Why didn't my
physician let me try that?" Which of Rationale: The nurse's most appropriate
the following responses by the response is one that is therapeutic. The basic
nurse would be most appropriate? principle of therapeutic communication and a
therapeutic relationship is honesty. Therefore,
the nurse needs to explain truthfully that each
client is different and that there are various
forms of arthritis and arthritis treatment. To
state that it is the physician's prerogative to
decide how to treat the client implies that the
client is not a member of his or her own
health care team and is not a participant in his
or her care. The statement also is defensive,
which serves to block any further
communication or questions from the client
about the physician. Asking the client to tell
more about the friend presumes that the
client knows correct and complete
information, which is not a valid assumption to
make. The nurse does not know about the
client's friend and should not make
statements about another client's condition.
Stating that the drug is for cases that are
worse than the client's demonstrates that the
nurse is making assumptions that are not
necessarily valid or appropriate. Also, telling
the client not to worry ignores the underlying
emotions associated with the question, totally
discounting the client's feelings.
4 Crackles heard on lung auscultation Fluid-filled alveoli.
1. indicate which of the following?
Rationale: Crackles are auscultated over fluid-
filled alveoli. Crackles heard on lung
auscultation do not have to be associated with
cyanosis. Bronchospasm and airway narrowing
generally are associated with wheezing
sounds.
4 Cri du chat Example of a disorder caused by chromosomal
2. deletion.
4 Down Syndrome trisomy 21, is an example of a disorder caused
3. by chromosomal translocation.
4 During a home visit with a Puerperal infection.
4. primipara who gave birth 7 days
ago, the client tells the nurse that Rationale: The client is exhibiting signs and
her lochia serosa has been profuse symptoms of puerperal infection, which
and foul-smelling and she has had include profuse foul-smelling lochia, chills,
chills. During palpation of the fever, and a uterus that is larger than
uterus, the client indicates that she expected for the first postpartum day.
is very sore. The nurse should Infection may spread through the lymphatic
further assess the client for: system; antibiotic therapy is necessary. During
normal uterine involution, the lochia becomes
less profuse and should not be foul-smelling. If
the client had retained placental fragments,
lochia rubra, not foul-smelling lochia serosa,
would continue. Uterine atony refers to
relaxation of the uterus and subsequent
failure to contract properly. It may be a result
of retained placental fragments.
4 During clindamycin therapy, a Clostridium difficile
5. nurse monitors a client for
pseudomembranous colitis. This Rationale: Pseudomembranous colitis may
serious adverse reaction to result from a superinfection with C. difficile
clindamycin results from during clindamycin therapy. Clindamycin-
superinfection with which induced pseudomembranous colitis isn't
organism? caused by S. aureus, B. fragilis, or E. coli.
4 During her first prenatal visit, Imbalanced nutrition: Less than body requirements
6. a pregnant client admits to related to limited food intake.
the nurse that she uses
cocaine at least once per day. Rationale: A substance abuser may spend more
Which nursing diagnosis is money on drugs than on food and other basic
most appropriate for this needs, leading to a nursing diagnosis of Imbalanced
client? nutrition: Less than body requirements related to
limited food intake. Activity intolerance might be a
relevant nursing diagnosis if the client were having
trouble sleeping or getting adequate rest; however,
activity intolerance wouldn't be related to decreased
tissue oxygenation in this case. If the client were an
I.V. drug abuser, a diagnosis of Risk for infection
related to I.V. drug use might be appropriate.
Because the question doesn't specify how the client
is using cocaine, a diagnosis of Impaired gas
exchange related to respiratory effects of substance
abuse is inappropriate.
4 The fire alarm sounds on the Close all of the doors on the unit.
7. maternal-neonatal unit at
0200. How can a nurse best Rationale: The nurse should respond quickly by
care for her clients during a closing all of the doors on the unit. This action
fire alarm? prevents the spread of smoke in case of a fire. The
nurse shouldn't begin evacuating the unit until given
notification to do so. The nurse shouldn't ignore the
alarm because fire drills are necessary to prepare
the staff for a fire. The mothers should be awakened
in case evacuation is necessary.
4 Following a total hip Encourage the client to use the overhead trapeze
8. replacement, the nurse to assist with position changes.
should do which of the Use a fracture bedpan when needed by the client.
following? Select all that When the client is in bed, prevent
apply. thromboembolism by encouraging the client to do
toe-pointing exercises.
4 Following the formation of an These findings are normal for a client with an ileal
9. ileal conduit, the nurse notes conduit.
that the client's urinary
drainage appliance contains Rationale: A segment of the terminal ileus is used to
pale yellow urine with large form the conduit that collects urine from the ureters.
amounts of mucus. How Hence, the client with an ileal conduit can be
should the nurse interpret expected to excrete urine that contains mucus from
these data? this intestinal mucous membrane. Mucus production
is not a result of infection or stomal irritation. Mucus
production is not a result of glycosuria. There is no
reason to expect to find glucose in the client's urine.
5 For a client with Increased urine osmolarity
0. hyperglycemia, which
assessment finding best Rationale: In hyperglycemia, urine osmolarity (the
supports a nursing diagnosis measurement of dissolved particles in the urine)
of Deficient fluid volume? increases as glucose particles move into the urine.
The client experiences glucosuria and polyuria,
losing body fluids and experiencing deficient fluid
volume. Cool, clammy skin; jugular vein distention;
and a decreased serum sodium level are signs of
fluid volume excess, the opposite imbalance.
5 For the client who is Using a picture board.
1. experiencing expressive
aphasia, which nursing Rationale: Expressive aphasia is a condition in which
intervention is most helpful in the client understands what is heard or written but
promoting communication? cannot say what he or she wants to say. A
communication or picture board helps the client
communicate with others in that the client can point
to objects or activities that he or she desires.
5 Hemophilia A Example of a disorder caused by an X-linked
2. recessive gene.
5 he nurse notices that a Take the client's blood pressure.
3. client's heart rate decreases
from 63 to 50 beats per The nurse should first assess the client's tolerance
minute on the monitor. The to the drop in heart rate by checking the blood
nurse should first: pressure and level of consciousness and determine
if atropine is needed. If the client is symptomatic,
atropine and transcutaneous pacing are
interventions for symptomatic bradycardia. Once
the client is stable, further physical assessments
can be done.
5 Immediately on return to the Lying on the abdomen with the head turned
4. nursing unit after surgical repair of to the side.
a cleft palate, in which of the
following positions should the nurse Rationale: Immediately after a surgical repair
place the child? of a cleft palate, the child is placed on the
abdomen with the head turned to the side to
lessen the chance of aspiration by allowing
secretions to drain out. Positioning the child
on the back places the child at risk for
aspiration should any regurgitation or
vomiting occur, even in low Fowler's position
with the head to the side or in reverse
Trendelenburg position with the head tilted
forward.
5 In an initial screening for lead Educate parents on ways to reduce lead in
5. poisoning a 2-year-old child is found the environment.
to have a lead level just above 10
mcg/dL (0.48 mol/L). The nurse Rationale: Treatment for children with
should: minimally elevated lead levels should include
family lead education, follow-up testing, and
a social service consultation if needed.
Waiting 6 months for a follow-up screening is
too long because the effects of lead are
irreversible. Oral chelation therapy is not
begun until levels approach 45 mcg/dL (2.2
mol/L). There is no such thing as a "normal"
lead level because there is no beneficial
action in the body.
5 In which areas of the United States Inner-city areas.
6. and Canada is the incidence of
tuberculosis highest? Rationale: Statistics show that of the four
geographic areas described, most cases of
tuberculosis are found in inner-core
residential areas of large cities, where health
and sanitation standards tend to be low.
Substandard housing, poverty, and crowded
living conditions also generally characterize
these city areas and contribute to the spread
of the disease. Farming areas have a low
incidence of tuberculosis. Variations in water
standards and industrial pollution are not
correlated to tuberculosis incidence.
5 Kawasaki disease A condition that causes inflammation in the
7. walls of some blood vessels in the body. One
of the characteristics of children with KD is
irritability. They are often inconsolable.
Placing the child in a quiet environment may
help quiet the child and reduce the workload
of the heart.
5 Laboratory results for a child with a Compensation for hypoxia
8. congenital heart defect with
decreased pulmonary blood flow Rationale:
reveal an elevated hemoglobin (Hb) A congenital heart defect with decreased
level, hematocrit (HCT), and red pulmonary blood flow alters blood flow
blood cell (RBC) count. These data through the heart and lungs, resulting in
suggest which condition? hypoxia. To compensate, the body increases
the oxygen-carrying capacity of RBCs by
increasing RBC production, which causes the
Hb level and Hct to rise. In anemia, the Hb
level and Hct typically decrease. Altered
electrolyte levels and other laboratory values
are better indicators of dehydration. An
elevated Hb level and HCT aren't associated
with jaundice.
5 A man brings his wife to the Obvious neglect of personal hygiene
9. emergency department. He reports Speaking in soft monotone voice
that since the death of their 7- Inconsolable weeping
month-old daughter 8 weeks earlier,
his wife has been neglecting her Rationale: Typically, a depressed client
housework and family, has lost 20 lb exhibits slow movements and fatigue and
(9.1 kg), and has not left the house. poor hygiene/grooming. Such a client also
Which of the following additional has difficulty interacting, speaking in a
assessment findings would indicate monotone voice, and avoiding eye contact.
to the nurse that the client may be In extreme depression, the client may not
experiencing extreme depression? communicate verbally at all, or the client
Select all that apply. may verbalize feelings of anger and lash out
with irritability.
6 Most common site of aneurysm abdominal aorta, just below the renal
0. formation is in the arteries.

Rationale: About 75% of aneurysms occur


in the abdominal aorta, just below the
renal arteries (Debakey type I aneurysms).
Debakey type II aneurysms occur in the
aortic arch around the ascending and
descending aorta, whereas Debakey type
III aneurysms occur in the descending
aorta, beyond the subclavian arteries.
6 A multigravid client will be using "One possible adverse effect is absence of
1. medroxyprogesterone acetate as a a menstrual period."
family planning method. After the
nurse instructs the client about this Rationale: With medroxyprogesterone
method, which of the following client acetate, irregular menstrual cycles and
statements indicates effective amenorrhea are common adverse effects.
teaching? Other adverse effects include weight gain,
breakthrough bleeding, headaches, and
depression. This method requires deep
intramuscular injections every 3 months.
The first injection should occur within 5
days after menses.
6 n an outpatient addiction group, a "I hear how confused and frustrated you
2. recovering client says that before are."
treatment, her husband drank on "It can happen that as one spouse
social occasions. "Now he drinks at becomes sober, the other spouse
home, from the time he comes home deteriorates."
from work until he goes to bed. He "What have you tried to do about your
says that he doesn't like me anymore husband's behaviors?"
and that I expect him to do more work "What do you think you could do to have
on the house and yard. I used to your husband come in for an evaluation?"
ignore that stuff. I don't know what to
do." The nurse would make the
following comments in which order of
priority from first to last?
6 A nurse asks a pregnant client about Learning disability
3. her alcohol use. The client admits she
sometimes has several glasses of wine Rationale: Maternal alcohol use during
with dinner. Her alcohol consumption pregnancy may cause fetal and neonatal
puts her fetus at risk for which central nervous system deficits such as
condition? learning disabilities. It also may lead to
characteristic physical anomalies and
growth restriction. Maternal alcohol use
doesn't cause alcohol addiction in the
fetus or neonate. Anencephaly occurs
when the cranial end of the neural tube
fails to fuse before the 26th day of
gestation; this condition isn't related to
maternal alcohol use. Down syndrome
results from a chromosomal disorder.
6 The nurse assesses the neurologic Makes a walking movement when held
4. system of a newborn. Which of the upright with one foot touching the table
following behaviors would the nurse
interpret as a normal reflex response? Rationale: Newborns who are held in a
vertical position with their feet touching a
hard surface will take a few quick,
alternating steps. This stepping reflex is
more pronounced at birth and typically
disappears between 4 and 8 weeks of age.
When the palm of a newborn's hand is
stimulated, he/she will typically grasp the
object. When the cheek is touched, the
newborn typically will turn the head
toward the stimulation. A newborn's toes
will expand slightly when the sole of the
foot is touched.
6 The nurse establishes the goal of Administering famotidine as ordered.
5. preventing the development of a
stress ulcer in a burn client. Which of Rationale: Clients with burns are
the following interventions would most susceptible to the development of
likely contribute to the achievement of Curling's ulcer, a gastroduodenal ulcer
this goal? that is caused by a generalized stress
response. The stress response results in
increased gastric acid secretion and a
decreased production of mucus.
Prevention is the best treatment, and
clients are frequently treated
prophylactically with antacids and H2
histamine blockers such as famotidine
(Pepcid).
6 The nurse has administered Decreased fetal heart rate variability
6. meperidine to a client in labor.
Which change in the fetal heart Rationale: Possible fetal adverse reactions
rate tracing would the nurse include moderate central nervous system
expect to occur as a result of the depression and decreased fetal heart rate
meperidine administration? variability. Bradycardia, late decelerations, and
increased fetal movement don't occur as a
result of meperidine administration.
6 The nurse in the perioperative area "It's okay to be scared. What is it about cancer
7. is preparing a client for surgery that you're afraid of?"
and notices that the client looks
sad. The client says, "I'm scared of
having cancer. It's so horrible and I
brought it on myself. I should have
quit smoking years ago." What
would be the nurse's best response
to the client?
6 A nurse is assessing a client who "When did the rash start?"
8. has a rash on his chest and upper "Are you allergic to any medications, foods,
arms. Which questions should the or pollen?"
nurse ask in order to gain more "What have you been using to treat the
information about the client's rash? rash?"
Select all that apply. "Have you recently traveled outside the
country?"

Rationale: When assessing a client who has a


rash, the nurse should first find out when the
rash began; this information can identify
where the rash is in the disease process and
assists with the correct diagnosis. The nurse
should also ask about allergies because rashes
related to allergies can occur when a person
changes medications, eats new foods, or
comes into contact with agents in the air, such
as pollen. The nurse needs to find out how the
client has been treating the rash because
treating the rash with topical ointments or
taking oral medications may make the rash
worse. The nurse should ask about recent
travel because travel outside the country
exposes the client to foreign foods and
environments, which can contribute to the
onset of a rash. Although the client's age and
smoking and drinking habits can be important
to know, this information won't provide further
insight to the rash or its cause.
6 A nurse is assigned to a client with Meeting all of the client's physical needs
9. catatonic schizophrenia. Which
intervention should the nurse Rationale: Because a client with catatonic
include in this client's care plan? schizophrenia can't meet his physical needs
independently, the nurse must provide for all
of these needs, including adequate food and
fluid intake, exercise, and elimination.
Although this client is incapable of expressing
concerns, the nurse should try to verbalize the
message his nonverbal behavior conveys.
Lithium is used to treat mania, not catatonic
schizophrenia. Despite the client's mute,
unresponsive state, the nurse should provide
nonthreatening stimulation and should spend
time with him, not leave him alone all the
time. Although aware of the environment, the
client doesn't actively interact with it; the
nurse's support and presence can be
reassuring.
7 A nurse is assisting with the Take a deep breath and hold it.
0. removal a of central venous access
device (CVAD). The nurse should Rationale: The client should be asked to
instruct the client to: perform the Valsalva maneuver (take a deep
breath and hold it) during insertion and
removal of a CVAD. This increases central
venous pressure during the procedure and
helps prevents air embolism. Trendelenburg is
the preferred position for CVAD insertion and
removal. If it is not possible, supine position is
sufficient for CVAD removal. It is not
recommended to position the client on the left
or the right side. The client should hold the
breath, not exhale, which lowers central
venous pressure.
7 The nurse is aware that frequent Pneumonia
1. repositioning in bed will assist in the
prevention of which of the following Rationale: By frequently changing positions
for a client? in bed, the client can prevent the
development of pneumonia, urinary stasis,
and deep vein thrombosis. These
movements promote blood, oxygen, and
fluid circulation throughout the body
systems and prevent stasis. Postural
hypotension can often be associated with
medications and no information is given
about this in the question. Arterial
thrombosis is incorrect because decreased
movement would more likely result in a
venous thrombosis.
7 A nurse is caring for a client who irrigate the NG tube gently with normal
2. underwent a subtotal gastrectomy saline solution if ordered.
24 hours ago. The client has a
nasogastric (NG) tube. The nurse Rationale: The nurse can gently irrigate the
should: tube if ordered, but must be careful not to
reposition it. Repositioning can cause
bleeding. The nurse should apply suction
continuously not every hour. The nurse
shouldn't clamp the NG tube postoperatively
because secretions and gas will accumulate,
stressing the suture line.
7 A nurse is caring for a client with a assess the client for pain.
3. fractured hip. The client is
combative, confused, and trying to Rationale: The nurse should assess the
pull out necessary I.V. lines and an client for possible causes of the behavior,
indwelling urinary catheter. The such as pain. A client should never be left
nurse should: alone while the nurse summons assistance.
All staff members must receive annual
instruction on the use of restraints, and the
nurse should be familiar with the facility's
policy. In most settings, the nurse must have
a physician's order before restraining a
client.
7 A nurse is caring for a client with Clay-colored stools
4. cholelithiasis. Which sign indicates
obstructive jaundice? Rationale: Obstructive jaundice develops
when a stone obstructs the flow of bile in
the common bile duct. When the flow of bile
to the duodenum is blocked, the lack of bile
pigments results in a clay-colored stool. In
obstructive jaundice, urine tends to be dark
amber (not straw-colored) as a result of
soluble bilirubin in the urine. Hematocrit
levels aren't affected by obstructive
jaundice. Because obstructive jaundice
prevents bilirubin from reaching the
intestine (where it's converted to
urobilinogen), the urine contains no
urobilinogen.
7 The nurse is completing a health Pulse 108 bpm, temperature 97.7F
5. history and physical assessment on a (36.5C), distended abdomen, and nausea
client admitted with esophageal
varices and cirrhosis. What signs and Rationale: Increased pulse rate, a distended
symptoms alert the nurse to a abdomen, and nausea signify the possibility
potential internal hemorrhage? of hemorrhage. The other choices are
incorrect.
7 A nurse is evaluating a client's Rinne test
6. auditory function. To compare air
conduction to bone conduction, the Rationale: The Rinne test compares air
nurse should conduct which test? conduction to bone conduction in both ears.
The whispered voice test evaluates low-
pitched sounds, and the watch tick test
assesses high-pitched sounds. Both tests
assess gross hearing. Weber's test evaluates
bone conduction.
7 A nurse is helping a physician insert turn the client on his left side and place the
7. a subclavian central line. After the bed in Trendelenburg's position.
physician has gained access to the
subclavian vein, he connects a 10-ml Rationale: A nurse who suspects an air
syringe to the catheter and embolism should place the client on his left
withdraws a sample of blood. He side and in Trendelenburg's position. Doing
then disconnects the syringe from so allows the air to collect in the right atrium
the port. Suddenly, the client rather than enter the pulmonary system.
becomes confused, disoriented, and The supine position, high-Fowler's position,
pale. The nurse suspects an air and the shock position are therapeutic for
embolus. She should: other situations but not for air embolism.
7 The nurse is managing care of a Anesthesia/pain level assessment every 30
8. primigrada at full term who is minutes.
in active labor. What should be
included in developing the plan Rationale: The nurse should monitor
of care for this client? anesthesia/pain levels every 30 minutes during
active labor to ascertain that this client is
comfortable during the labor process and
particularly during active labor when pain often
accelerates for the client. When in active labor,
oxygen saturation is not monitored unless there is
a specific need, such as heart disease. The client
should not be on her back but wedged to the right
or left side to take the pressure off the vena cava.
When lying on the back, the fetus compresses the
major blood vessels. Vaginal bleeding in active
labor should be monitored every 30 minutes to 1
hour.
7 A nurse is reluctant to provide Good Samaritan laws are designed to protect the
9. care at an accident scene. caregiver in emergency situations.
Which of the following legal
definitions is true regarding the
provision of nursing care?
8 The nurse is reviewing sterile Nasogastric (NG) tube placement
0. procedures with a student
nurse. The nurse understands Rationale: The GI system isn't a sterile system;
that the student requires therefore, NG tube placement doesn't require
additional teaching when the sterile technique. I.V. insertion requires sterile
student identifies which technique because intentional penetration of the
procedure as requiring sterile skin occurs. The urinary system is sterile, so the
technique? nurse must maintain sterility during catheter
placement. Burns have a high risk for infection;
the nurse must maintain sterile technique to
decrease this risk.
8 A nurse is teaching a client "I should limit my fiber intake for 1 to 2 weeks
1. what to expect following a following the procedure."
barium enema. Which client
statement indicates a need for Rationale: There's no need to limit fiber intake
further teaching? after a barium enema. The client may resume his
normal diet. Barium may increase stool
elimination, so there's no need for a laxative after
the procedure. The client should increase fluid
intake to facilitate barium elimination. The client
should report bowel movements so the nurse can
ensure that barium elimination occurs.
8 The nurse is teaching a Pelvic rocking may help relieve lower back pain.
2. pregnant client about exercises Abdominal breathing lifts the abdominal wall off
that may be helpful during of the uterus.
pregnancy. Which points should Kegel exercises help improve vaginal
the nurse include in the contractility and bladder control.
instruction? Select all that
apply.
8 The nurse is teaching the Use the Crede's maneuver to empty the bladder.
3. parents of a child with Encourage frequent emptying of the bladder.
myelomeningocele how to Assure adequate fluid intake.
prevent urinary tract
infections. What should the
care plan include for this child?
Select all that apply.
8 The nurse is teaching the Provide rest periods between activities
4. parents of an 8-year-old child Expect periods when the child will refuse to eat
receiving treatment for cancer. Call the healthcare provider with any concerns
What will the nurse include in
the teaching? Select all that
apply.
8 A nurse observes a physician Discuss the breach of practice with the physician.
5. providing care to an infectious
client without the use of Rationale: The nurse should first discuss the
personal protective equipment. breach of infection control procedures with the
What should the nurse do first? physician and discuss the practices that should be
followed. The other options may be followed
subsequently, but discussing with the physician is
the first step.
8 A nurse observes constant bubbling The system has an air leak.
6. in the water-seal chamber of a closed
chest drainage system. What should Rationale: Constant bubbling in the water-
the nurse conclude? seal chamber indicates a system air leak
and requires immediate intervention. The
client with a pneumothorax will have
intermittent bubbling in the water-seal
chamber. Clients without a pneumothorax
should have no evidence of bubbling in the
chamber. If the tube is obstructed, the fluid
would stop fluctuating in the water-seal
chamber.
8 The nurse should instruct the family Promote interactions with one friend instead
7. of a child with newly diagnosed of groups.
hyperthyroidism to:
Rationale: Children with hyperthyroidism
experience emotional labiality that may
strain interpersonal relationships. Focusing
on one friend is easier than adapting to
group dynamics until the child's condition
improves. Because of their high metabolic
rate, children with hyperthyroidism report
being too warm. Bright sunshine may be
irritating because of disease-related
ophthalmopathy. Sweating is common and
bathing should be encouraged.
8 The nurse teaches a client about Hands.
8. using the crutches, instructing the
client to support the weight primarily Rationale: When using crutches, the client is
on which of the following body areas? taught to support her weight primarily on
the hands. Supporting body weight on the
axillae, elbows, or upper arms must be
avoided to prevent nerve damage from
excessive pressure.
8 The nurse teaches the client to Strengthen the muscles while keeping the
9. perform isometric exercises to joints stationary.
strengthen the leg muscles after
arthroplasty. Isometric exercises are Rationale: An exercise program is
particularly effective for clients with recommended to strengthen muscles after
rheumatoid arthritis because they: arthroplasty. Isometric (or muscle-setting)
exercises strengthen muscles but keep the
joint stationary during the healing process.
Isometric exercise costs little in terms of
time and money but these are not
necessarily primary reasons for using it.
Isometric exercises may help improve a
client's morale by promoting self-care but
this is not the reason for doing them.
Isometric exercise will not help prevent joint
stiffness; the joint is kept stationary.
9 The nurse, while shopping in a local To contract the mother's uterus.
0. department store, hears a
multiparous woman say loudly, "I Rationale: After an emergency birth, the
think the baby's coming." After nurse suggests that the mother begin
asking someone to call 911, the breast-feeding to contract the uterus.
nurse assists the client to give birth Breast-feeding stimulates the natural
to a term neonate. While waiting for production of oxytocin. In a multiparous
the ambulance, the nurse suggests client, uterine atony is a potential
that the mother initiate breast- complication because of the stretching of
feeding, primarily for which of the the uterine fibers following each subsequent
following reasons? pregnancy. Although breast-feeding does
help to begin the parental-infant bonding
process, this is not the primary reason for
the nurse to suggest breast-feeding.
Prevention of neonatal hypothermia is
accomplished by placing blankets on both
the neonate and the mother. Although
colostrum in breast milk provides the
neonate with nutrients and
immunoglobulins, the primary reason for
breast-feeding is to stimulate the natural
production of oxytocin to contract the
uterus.
9 The nurse who uses self-disclosure refocus on the client's experience as quickly
1. should: as possible.

Rationale: The nurse's self-disclosure should


be brief and to the point so that the
interaction can be refocused on the client's
experience. Because the client is the focus
of the nurse-client relationship, discussion
shouldn't dwell on the nurse's experience.
9 A nurse working in the emergency Move toward the door and leave to call the
2. department enters the room of a crisis response team.
client who is agitated and swears at
the nurse. The client stands up and Rationale: The nurse assesses and identifies
moves toward the nurse in an that the nurse's safety is at risk because the
aggressive fashion. What is the most client is agitated and moving aggressively
appropriate action by the nurse to toward the nurse. The nurse needs to leave
address this situation? and obtain help in the form of a crisis
response team. The other options are
incorrect because they do not provide for
the safety of the nurse or the client.
9 A parent is planning to enroll a 9- Soiled diapers are discarded in covered
3. month-old infant in a day-care receptacles.
facility. The parent asks a nurse Disposable papers are used on the diaper-
what to look for as indicators that changing surfaces.
the facility is adhering to good Facilities for hand hygiene are located in
infection control measures. The every classroom.
nurse identifies which as an
indication of meeting proper
infection control standards? Select
all that apply.
9 The pediatric cardiac nurse is 4th Left intercostal space at the nipple
4. assessing the heart sounds of a 3-
year-old child with a mitral valve Rationale: Mitral valve regurgitation or
regurgitation. Which graphic shows insufficiency is a heart disorder where the
the area where the nurse would mitral valve does not close properly when the
assess the site of the insufficiency? heart pumps out blood; thus, causing an
abnormal leaking of blood from the left
ventricle through the mitral valve and into
the left atrium. The mitral valve is heard best
at the 4th left intercostal space at the nipple
line (option A). The aortic valve is heard best
at the 2nd intercostal space to the Right of
the sternum (option B). The pulmonic valve is
heard best at the 2nd intercostal space to the
Left of the sternum (option C). The tricuspid
valve is heard best in the 4th intercostal
space to the Right of the sternum (option D).

A-2nd Right
P-2nd Left
E-3rd
T-4th Right
M-4th/5th Left
9 Phenylketonuria (PKU) Autosomal recessive gene.
5.
PKU is caused by an inborn error of
metabolism. It is an autosomal recessive
disorder that inhibits the conversion of
phenylalanine to tyrosine.
9 A physician orders spironolactone, Loss of 2.2 lb (1 kg) in 24 hours
6. 50 mg by mouth four times daily, for
a client with fluid retention caused Rationale: Daily weight measurement is the
by cirrhosis. Which finding indicates most accurate indicator of fluid status; a loss
that the drug is producing a of 2.2 lb (1 kg) indicates loss of 1 L of fluid.
therapeutic effect? Because spironolactone is a diuretic, weight
loss is the best indicator of its effectiveness.
This client's serum potassium and sodium
levels are normal. A blood pH of 7.25
indicates acidosis, an adverse reaction to
spironolactone.
9 A primigravida at 8 weeks' The volume of amniotic fluid needed for
7. gestation tells the nurse that she testing will be available by 15 weeks.
wants an amniocentesis because
there is a history of Hemophilia A in Rationale: The volume of fluid needed for
her family. The nurse informs the amniocentesis is 15 ml and this is usually
client that she will need to wait available at 15 weeks' gestation. Fetal
until she is 15 weeks gestation for development continues throughout the
the amniocentesis. Which of the prenatal period. Cells necessary for testing
following pr for Hemophilia A are available during the
entire pregnancy but are not accessible by
amniocentesis until 12 weeks' gestation.
Anomalies are not associated with
amniocentesis testing.
9 A public health nurse is teaching a Toddlers often eat one food for many days in
8. group of parents at a community a row.
health center about feeding and
nutrition for toddlers. Which of the Rationale: It is common and not harmful for
following is most important for the toddlers to have food jags, eating one food
nurse to include in the teaching? for days on end. Using dessert as a reward
makes vegetables and other foods seem less
desirable. It is an unreasonable expectation
to let a child choose his or her own food at
this age.
9 The purpose of biofeedback is to Translating the signals of body processes into
9. enable a client to exert control over observable forms
physiologic processes by:
10 A registered nurse (RN) A 2-year-old child who nearly drowned 2 days earlier
0. has been paired with a
licensed practical nurse Rationale: The nurse can delegate care of the near-
(LPN) for the shift. Whose drowning victim to an LPN. Children recover quite
care should the RN quickly from near-drowning experiences; acute care isn't
delegate to the LPN? necessary. The infant who has undergone surgery is still
under the effects of anesthesia and requires close
observation for dehydration, pain, and signs of adverse
reactions. The infant with gastroenteritis also requires
close monitoring for signs of dehydration. The infant
who lost consciousness will need to be monitored most
closely. His status could quickly become very critical.
10 SARS Isolation Airborne and contact precautions
1. precautions
Rationale: SARS, a highly contagious viral respiratory
illness, is spread by close person-to-person contact. The
client should be placed on airborne and contact
precautions to prevent the spread of infection. Droplet
precautions don't require a negative air pressure room
and wouldn't protect the nurse who touches
contaminated items in the client's room. Contact
precautions alone don't provide adequate protection
from airborne particles.
10 Signs and symptoms of Dyspnea
2. left-sided heart failure? Crackles
Tachycardia
Oliguria
10 To follow standard Wearing gloves when administering I.M. medication
3. precautions, the nurse
should carry out which Rationale: To follow standard precautions, caregivers
measure? must place used, uncapped needles and syringes in a
puncture-resistant container; wear gloves when
anticipating contact with a client's blood, body fluid,
mucous membranes, or nonintact skin (such as when
administering an I.M. injection); and wear a gown during
procedures that are likely to generate splashes of blood
or body fluids. Standard precautions don't call for
caregivers to wear a gown or gloves when bathing a
client because this activity isn't likely to cause contact
with blood or body fluids.
10 To prevent "Avoid coffee and alcoholic beverages."
4. gastroesophageal reflux
in a client with hiatal Rationale: To prevent reflux of stomach acid into the
hernia, the nurse should esophagus, the nurse should advise the client to avoid
provide which discharge foods and beverages that increase stomach acid, such
instruction? as coffee and alcohol. The nurse also should teach the
client to avoid lying down after meals, which can
aggravate reflux, and to take antacids after eating. The
client need not limit fluid intake with meals as long as
the fluids aren't gastric irritants.
10 Twenty-four hours after Hirschsprung's disease
5. birth, a neonate hasn't
passed meconium. The Rationale: Failure to pass meconium is an important
nurse suspects which diagnostic indicator for Hirschsprung's disease.
condition? Hirschsprung's disease is a potentially life-threatening
congenital large-bowel disorder characterized by the
absence or marked reduction of parasympathetic
ganglion cells in a segment of the colorectal wall;
narrowing impairs intestinal motility and causes severe,
intractable constipation leading to partial or complete
colonic obstruction. Celiac disease, intussusception, and
abdominal wall defects aren't associated with failure to
pass meconium.
10 Using the Morse Fall Risk 62-year-old client with a history of Parkinson's disease,
6. scale (see exhibit), the admitted for pneumonia and receiving IV antibiotics. The
nurse should initiate client has fallen at home but is able to ambulate with a
highest fall risk cane. During his hospitalization, he has gotten out of
precautions for which of bed without calling for assistance.
the following clients?
Rationale: Using the Morse fall scale, risk factors for this
client include history of falling, secondary diagnosis,
ambulatory aid, IV/heparin lock, weak gait/transfer, and
forgetting limitations (100 points). Client A is also high
risk with a secondary diagnosis, history of falling, IV
access, and confusion but is on bed rest (75 points).
Client B risks include IV access and secondary diagnosis
(35 points). Client D is at risk due to his IV access only
(20 points).
10 What is the most appropriate Attending day therapy three times a week.
7. long-term goal for an outpatient
client with schizophrenia who has Rationale: Attending day therapy three times
been withdrawn from friends and per week is a long-term goal that will show the
family for 3 weeks? most progress in overcoming withdrawal. The
client's calling his mother is a first step in
getting out of a severe withdrawal. Allowing
two friends to visit every day would be
appropriate if the client is successful with
calling his mother once a day. Insufficient
information is presented in the scenario to
indicate that excessive sleep is a problem.
10 What is the most appropriate Deficient fluid volume
8. nursing diagnosis for the client
with acute pancreatitis? Rationale: Clients with acute pancreatitis often
experience deficient fluid volume, which can
lead to hypovolemic shock. Vomiting,
hemorrhage (in hemorrhagic pancreatitis), and
plasma leaking into the peritoneal cavity may
cause the volume deficit. Hypovolemic shock
will cause a decrease in cardiac output.
Gastrointestinal tissue perfusion will be
ineffective if hypovolemic shock occurs, but
this wouldn't be the primary nursing diagnosis.
10 When assessing a client who gave Encouraging increased fluid intake
9. birth 12 hours ago, the nurse
measures an oral temperature of Rationale: During the first postpartum day, mild
99.6 F (37.5 C), a heart rate of dehydration commonly causes a slight
82 beats/minute, a respiratory temperature elevation; the nurse should
rate of 18 breaths/minute, and a encourage fluid intake to counter dehydration.
blood pressure of 116/70 mm Hg. Aspirin is contraindicated in postpartum clients
Which nursing action is most because its anticoagulant effects may increase
appropriate? the risk of hemorrhage. Reassessing vital signs
in 4 hours is sufficient to assess the
effectiveness of hydration measures. The nurse
should request an antibiotic order if the client's
oral temperature exceeds 100.4 F (38 C),
which suggests infection.
11 When assessing for pain in a Observe the child for restlessness.
0. toddler, which of the following
methods should be the most Rationale: Toddlers usually express pain
appropriate? through such behaviors as restlessness, facial
grimaces, irritability, and crying. It is not
particularly helpful to ask toddlers about pain.
In most instances, they would be unable to
understand or describe the nature and location
of their pain because of their lack of verbal and
cognitive skills. However, preschool and older
children have the verbal and cognitive skills to
be able to respond appropriately. Numeric pain
scales are more appropriate for children who
are of school age or older. Changes in vital
signs do occur as a result of pain, but
behavioral changes usually are noticed first.
11 When caring for a 12-month-old tachypnea.
1. infant with dehydration and
metabolic acidosis, the nurse Rationale: The nurse would expect to see
expects to see: tachypnea because the body compensates for
metabolic acidosis via the respiratory system,
which tries to eliminate the buffered acids by
increasing alveolar ventilation through deep,
rapid respirations. Altered WBC and platelet
counts aren't specific signs of metabolic
imbalance.
11 When caring for a client who has Violent behavior.
2. overdosed on phencyclidine
(PCP), the nurse should be Rationale: The nurse must be especially
especially cautious about which cautious when providing care to a client who
of the following client behaviors? has taken PCP because of unpredictable,
violent behavior. The client can appear to be in
a calm state or even in a coma, then become
violent, and then return to a calm or comatose
state. Visual hallucinations, bizarre behavior,
and loud screaming are associated with PCP-
intoxicated clients. However, the unpredictable,
violent behavior presents a major issue of
safety for clients and staff.
11 When caring for a client with restrict the amount of fluid administered.
3. preeclampsia during labor, the
nurse should: Rationale: The volume of fluids administered
during labor to a client with preeclampsia
should be restricted. Clients usually receive
between 60 and 150 ml/hour.
11 When formulating outcomes for the Maintenance of normal body temperature.
4. post-term neonate at discharge,
which of the following would be Rationale: Hypothermia and temperature
most appropriate? instability are primary problems in the post-
term neonate, so maintaining a normal
temperature pattern is the most appropriate
goal. Post-term neonates have little
subcutaneous fat, predisposing them to cold
stress. Establishment of a deep respiratory
pattern is inappropriate because all
neonates tend to breathe in a shallow
manner. A weight gain of 4 ounces (112 g)
may not be feasible because most neonates
lose 5% to 15% of their birth weight during
the first few days of life. All infants should be
assessed for hyperbilirubinemia. Although
polycythemia is common in post-term
infants and may take a while to resolve,
hyperbilirubinemia is not more common in
the post-term neonate than it is in neonates
born at term.
11 When making rounds on the Create a poster presentation on the topic
5. pediatric neurology unit, the nurse with a required post test.
manager notes that when giving IV
medications many of the staff Rationale: A poster presentation is an eye-
nurses are disconnecting the flush catching way to disseminate information
syringe first and then clamping the that can be used to educate nurses on all
intermittent infusion device. The shifts. The addition of the post test will verify
nurse is concerned that the nurses that the poster information has been
do not understand the benefits of received. Because of the large volume of
positive pressure technique and emails the typical employee receives,
turbulence flow flush in preventing information sent this way may be
clots. After discussing the problem overlooked. If several nurses are observed
with the staff educator which not using the most current practice, it is
intervention would be the most quite possible many more do not understand
effective way to improve the nursing it. Thus, a larger scale plan is needed.
practice? Posting an article will not alone assure that
the information is read.
11 When preparing the room for Padding for the side rails.
6. admission of a multigravid client at
36 weeks' gestation diagnosed with Rationale: The client with severe
severe preeclampsia, which of the preeclampsia may develop eclampsia, which
following should the nurse obtain? is characterized by seizures. The client
needs a darkened, quiet room and side rails
with thick padding. This helps decrease the
potential for injury should a seizure occur.
Airways, a suction machine, and oxygen also
should be available. If the client is to
undergo induction of labor, oxytocin infusion
solution can be obtained at a later time.
Tongue blades are not necessary. However,
the emergency cart should be placed nearby
in case the client experiences a seizure. The
ultrasound machine may be used at a later
point to provide information about the fetus.
In many hospitals, the client with severe
preeclampsia is admitted to the labor area,
where she and the fetus can be closely
monitored. The safety of the client and her
fetus is the priority.
11 When providing oral hygiene for an Place the client in a side-lying position.
7. unconscious client, the nurse must
perform which action? Rationale: An unconscious client is at risk for
aspiration. To decrease this risk, the nurse
should place the client in a side-lying
position when performing oral hygiene.
Swabbing the client's lips, teeth, and gums
with lemon glycerin would promote tooth
decay. Cleaning an unconscious client's
tongue with gloved fingers wouldn't be
effective in removing oral secretions or
debris. Placing the client in semi-Fowler's
position would increase the risk of
aspiration.
11 When teaching a parent of a school- Complaints of a stiff neck
8. age child about signs and symptoms
of fever that require immediate Rationale: The nurse should discuss
notification of the physician, which complaints of a stiff neck because fever and
description should the nurse a stiff neck indicate possible meningitis.
include? Burning or pain with urination, fever that
disappears for 24 hours then returns, and a
history of febrile seizures should be
addressed by the physician but can wait
until office hours.
11 When teaching parents about fifth Fifth disease is transmitted by respiratory
9. disease (erythema infectiosum) secretions.
and its transmission, the nurse
should provide which information? Rationale: Fifth disease is transmitted by
respiratory secretions. The transmission mode
for roseola is unknown. Rubella is transmitted
by respiratory secretions, stool, and urine.
Intestinal parasitic conditions, such as
giardiasis and pinworm infection, are
transmitted by stool.
12 When teaching the family of an It adds strength to the cast.
0. older infant who has had a hip
spica cast applied for Rationale: The abduction bar is incorporated
developmental dysplasia of the into the cast to increase the cast's strength
hip, which information should the and maintain the legs in alignment. The bar
nurse include when describing the cannot be removed or adjusted, unless the cast
abduction stabilizer bar? is removed and a new cast is applied. The bar
should never be used to lift or turn the client,
because doing so may weaken the cast.
12 When the nurse is conducting a The client should not have her hip externally
1. preoperative interview with a rotated when she is positioned for the
client who is having a vaginal procedure.
hysterectomy, the client states
that she forgot to tell her doctor Rationale: The nurse should notify the surgery
that she had a total hip department and document the past surgery in
replacement 3 years ago. The the chart in the preoperative notes so that the
nurse communicates this client's hip is not externally rotated and the hip
information to the perioperative dislocated while she is in the lithotomy
nurse because: position. The prosthesis should not be a
problem as long as the perioperative nurse
places the grounding pad away from the
prosthesis site. The perioperative nurse will
inform the rest of the team, but the primary
reason to inform the perioperative nurse is
related to safe positioning of the client. The
surgeon can hand-write an addendum to the
history and initial and date the entry. The
history and physical information can then be
retyped at a later date.
12 Which characteristic is most Ambivalence
2. common among suicidal clients?
Rationale: One of the characteristics most
commonly shared by suicidal persons is
ambivalence, an internal struggle between self-
preserving and self-destructive forces. These
doubts are expressed when a person threatens
or attempts suicide and then tries to get help
to save his life. When the possible
consequences of suicide are discussed, such
persons commonly describe life-related
outcomes such as relief from an unhappy
situation. Many people consider suicide an
alternative to present circumstances, but they
may not have considered the implications of no
longer being alive. A psychotic person may or
may not have suicidal tendencies. Remorse
and anger may be associated with depression
but aren't universally present in suicidal
persons. Frustration isn't specifically associated
with suicidal ideation.
12 Which measure should the nurse Turn the head from side to side when walking.
3. teach the client with adult
macular degeneration (AMD) as a Rationale: To expand the visual field, the
safety precaution? partially sighted client should be taught to turn
the head from side to side when walking.
Neglecting to do so may result in accidents.
This technique helps maximize the use of
remaining sight. A patch does not address the
problem of hemianopsia. Appropriate client
positioning and placement of personal items
will increase the client's ability to cope with the
problem but will not affect safety.
12 Which nursing action is required Give a fluid bolus of 500 ml.
4. before a client in labor receives
an epidural anesthetic? Rationale: One of the major adverse effects of
epidural administration is hypotension.
Therefore, a 500-ml fluid bolus is usually
administered to prevent hypotension in the
client who wishes to receive an epidural for
pain relief. Assessing maternal reflexes, pupil
response, and gait isn't necessary.
12 Which of the following actions Providing warm, humidified oxygen in a warm
5. should the nurse anticipate using environment.
when caring for a term neonate
diagnosed with transient Rationale: Symptoms of transient tachypnea
tachypnea at 2 hours after birth? include respirations as high as 150
breaths/minute, retractions, flaring, and
cyanosis. Treatment is supportive and includes
provision of warm, humidified oxygen in a
warm environment. The nurse should
continuously monitor the neonate's
respirations, color, and behaviors to allow for
early detection and prompt intervention should
problems arise. Feedings are given by gavage
rather than bottle to decrease respiratory
stress. Obtaining extracorporeal membrane
oxygenation equipment is not necessary but
may be used for the neonate diagnosed with
meconium aspiration syndrome.
12 Which of the following Degree of response to pinpricks in the legs and toes.
6. assessments would be
important for the nurse to Rationale: Return of sensation in the toes and legs
make to determine marks recovery from spinal anesthesia. Because the
whether or not a client is client receiving spinal anesthesia is conscious, he will
recovering as expected not ordinarily be disoriented. The client's respiratory
from spinal anesthesia? status is not affected by spinal anesthesia. Capillary
refill time is an indicator of circulatory status, not
neurologic status.
12 Which of the following The neonate is at risk because of multiple factors.
7. best identifies the reason
for assessing a neonate Rationale: ROP, previously called retrolental fibroplasia,
weighing 1,500 g at 32 is associated with multiple risk factors, including high
weeks' gestation for arterial blood oxygen levels, prematurity, and very low
retinopathy of prematurity birth weight (less than 1,500 g). In the early acute
(ROP)? stages of ROP, the neonate's immature retinal vessels
constrict. If vasoconstriction is sustained, vascular
closure follows, and irreversible capillary endothelial
damage occurs. Normal room air is at 21%. Acidosis,
not alkalosis, is commonly seen in preterm neonates,
but this is not related to the development of ROP.
Phototherapy is not related to the development of ROP.
However, during phototherapy, the neonate's eyes
should be constantly covered to prevent damage from
the lights.
12 Which of the following Hyperactivity and speech disorders are common.
8. characteristics should the
nurse teach the mother Rationale: Central nervous system disorders are
about her neonate common in neonates with FAS. Speech and language
diagnosed with fetal disorders and hyperactivity are common manifestations
alcohol syndrome (FAS)? of central nervous system dysfunction. Mild to severe
intellectual disability and feeding problems also are
common. Delayed growth and development is
expected. These neonates feed poorly and commonly
have persistent vomiting until age 6 to 7 months. These
neonates do not have a 70% mortality rate, and there is
no treatment for FAS.
12 Which of the following is Decreased myocardial contractility.
9. characteristic of
cardiogenic shock? Rationale: Cardiogenic shock occurs when myocardial
contractility decreases and cardiac output greatly
decreases. The circulating blood volume is within
normal limits or increased. Infarction is not always the
cause of cardiogenic shock.
13 Which of the following is Quality of breath sounds.
0. significant data to gather Occurence of chest pain.
from a client who has been Color of nail beds.
diagnosed with
pneumonia? Select all that Rationale:
apply. A respiratory assessment, which includes auscultating
breath sounds and assessing the color of the nail beds,
is a priority for clients with pneumonia. Assessing for
the presence of chest pain is also an important
respiratory assessment as chest pain can interfere with
the client's ability to breathe deeply. Auscultating bowel
sounds and assessing for peripheral edema may be
appropriate assessments, but these are not priority
assessments for the client with pneumonia.
131. Which of the following levels K+, 3.2; Cl-, 92; Na+, 120
of the serum electrolytes
potassium (K), chloride (Cl), Rationale: The serum electrolyte values in an
and sodium (Na) should the infant with persistent vomiting reflect
nurse expect to find in an hypokalemia (potassium level of 3.2),
infant with persistent hypochloremia (chloride level of 92), and
vomiting? hyponatremia (sodium level of 120). Chloride
and sodium function together to maintain fluid
and electrolyte balance. With vomiting,
sodium chloride and water are lost in gastric
fluid. As dehydration occurs, potassium moves
into the extracellular fluid. With persistent
vomiting, hypokalemia (from movement of
potassium into the extracellular fluid),
hypochloremia (due to increased losses in
gastric fluid), and hyponatremia (due to
increased losses in gastric fluid) would result.
In option B, the potassium level is almost
normal (normal is 3.5 to 5.5), chloride is
elevated (normal is 98 to 106), and sodium is
normal (normal is 135 to 145). In option C, the
potassium level is normal (normal is 3.5 to
5.5), chloride is decreased (normal is 98 to
106), and sodium is normal (normal is 135 to
145). In option D, the potassium level is
normal (normal is 3.5 to 5.5), chloride is
slightly elevated (normal is 98 to 106), and
sodium is slightly decreased (normal is 135 to
145).
132. Which of the following Fowler's
positions would be
appropriate for a client with Rationale: Ascites can compromise the action
severe ascites? of the diaphragm and increase the client's risk
of respiratory problems. Ascites also greatly
increases the risk of skin breakdown. Frequent
position changes are important, but the
preferred position is Fowler's. Placing the client
in Fowler's position helps facilitate the client's
breathing by relieving pressure on the
diaphragm. The other positions do not relieve
pressure on the diaphragm
133. Which of the following On pillows, with her hand higher than her
positions would be best for a elbow and her elbow higher than her shoulder.
client's right arm when she
returns to her room after a Rationale:
right modified radical Lymph nodes can be removed from the axillary
mastectomy with multiple area when a modified radical mastectomy is
lymph node excisions? done, and each of the nodes is biopsied. To
facilitate drainage from the arm on the
affected side, the client's arm should be
elevated on pillows with her hand higher than
her elbow and her elbow higher than her
shoulder. A sentinel node biopsy procedure is
associated with a decreased risk of
lymphedema because fewer nodes are
excised.
134. Which of the following Raise the arm on the affected side over the
rehabilitative measures should head.
the nurse teach the client who
has undergone chest surgery Rationale: A client who has undergone chest
to prevent shoulder ankylosis? surgery should be taught to raise the arm on
the affected side over the head to help
prevent shoulder ankylosis. This exercise helps
restore normal shoulder movement, prevents
stiffening of the shoulder joint, and improves
muscle tone and power. Turning from side to
side, raising and lowering the head, and
flexing and extending the elbow on the
affected side do not exercise the shoulder
joint.
135. Which of the following "I should avoid alcohol and caffeine."
statements indicates that the
client with a peptic ulcer Rationale: Caffeinated beverages and alcohol
understands the dietary should be avoided because they stimulate
modifications he needs to gastric acid production and irritate gastric
follow at home? mucosa. The client should avoid foods that
cause discomfort; however, there is no need to
follow a soft, bland diet. Eating six small meals
daily is no longer a common treatment for
peptic ulcer disease. Milk in large quantities is
not recommended because it actually
stimulates further production of gastric acid.

Answer Key
Question 1 See full question

While preparing to start a stat I.V. infusion, a nurse notices a broken ground
on the infusion pump's plug. What should the nurse do first?
You Selected:

Obtain another pump from central supply to use for the infusion.
Correct response:

Obtain another pump from central supply to use for the infusion.
Explanation:

Question 2 See full question

The nurse is planning care for a client in restraints. Which nursing


intervention is most important when restraining a this client?
You Selected:

Checking that the restraints have been applied correctly


Correct response:

Checking that the restraints have been applied correctly


Explanation:

Question 3 See full question

In which areas of the United States and Canada is the incidence of


tuberculosis highest?
You Selected:

inner-city areas
Correct response:

inner-city areas
Explanation:

Question 4 See full question

A nurse is walking down the hall in the main corridor of a hospital when the
infant security alert system sounds and a code for an infant abduction is
announced. The first responsibility of the nurse when this situation occurs is
to take which action?
You Selected:

Observe individuals in the area for large bags or oversized coats.


Correct response:

Observe individuals in the area for large bags or oversized coats.


Explanation:

Question 5 See full question

An elderly client admitted with pneumonia and dementia has attempted


several times to pull out the IV and Foley catheter. After trying other options,
the nurse obtains a prescription for bilateral soft wrist restraints. Which
nursing action is most appropriate?
You Selected:

Attach the ties of the restraints to the bedframe.


Correct response:

Attach the ties of the restraints to the bedframe.


Explanation:

Question 2 See full question

The nurse is removing the client's staples from an abdominal incision when the client sneezes and the incision splits
open, exposing the intestines. What should the nurse do first?

You Selected:

Cover the abdominal organs with sterile dressings moistened with sterile normal saline.

Correct response:

Cover the abdominal organs with sterile dressings moistened with sterile normal saline.
Explanation:

Question 3 See full question

The mother of a client who has a radium implant asks why so many nurses are involved in her daughters care. She
states, The doctor said I can be in the room for up to 2 hours each day, but the nurses say they are restricted to 30
minutes. The nurse explains that this variation is based on the fact that nurses:

You Selected:

work with radiation on an ongoing basis, while visitors have infrequent exposure to radiation.

Correct response:

work with radiation on an ongoing basis, while visitors have infrequent exposure to radiation.

Explanation:

Question 4 See full question

The primary goal in the plan of care for the client after cataract removal surgery is to:

You Selected:

promote safety at home.

Correct response:

promote safety at home.

Explanation:

Question 5 See full question

A client is admitted with bacterial meningitis. Which hospital room is the best choice for this client?

You Selected:

An isolation room three doors from the nurses' station

Correct response:

An isolation room three doors from the nurses' station

Explanation:

Question 5 See full question

A 29-year-old multigravida at 37 weeks' gestation is being treated for severe preeclampsia and has magnesium
sulfate infusing at 3 g/h. To maintain safety for this client, the priority intervention is to:

You Selected:

assess reflexes, clonus, visual disturbances, and headache.

Correct response:

assess reflexes, clonus, visual disturb

Question 1 See full question

When changing a sterile surgical dressing, a nurse first must:

You Selected:

wash her hands.


Correct response:

wash her hands.

Explanation:

Question 2 See full question

Which nursing intervention is appropriate for a client with an arm restraint?

You Selected:

Monitoring circulatory status every 2 hours

Correct response:

Monitoring circulatory status every 2 hours

Explanation:

Question 3 See full question

A nurse prepares to transfer a client from a bed to a chair. Which principle demonstrates safe body mechanics?

You Selected:

The nurse uses a rocking motion while helping the client to stand.

Correct response:

The nurse uses a rocking motion while helping the client to stand.

Explanation:

Question 4 See full question

After his spouse has visited, a client begins crying and saying that his spouse is a mean person. When the client
starts pounding on the overbed table and using incomprehensible language, the nurse feels she can't handle the
situation. What should the nurse do at this time?

You Selected:

Use the call system to request assistance.

Correct response:

Use the call system to request assistance.

Explanation:

Question 5 See full question

After an infant undergoes surgical repair of a cleft lip, the physician orders elbow restraints. For this infant, the
postoperative care plan should include which nursing action?

You Selected:

Removing the restraints every 2 hours

Correct response:

Removing the restraints every 2 hours

Explanation:

Question 6 See full question

Which item in the care plan for a toddler with a seizure disorder should a nurse revise?
You Selected:

Arm restraints while asleep

Correct response:

Arm restraints while asleep

Explanation:

Question 7 See full question

A nurse discussing injury prevention with a group of workers at a day-care center is focusing on toddlers. When
discussing this age-group, the nurse should stress that:

You Selected:

accidents are the leading cause of death among toddlers.

Correct response:

accidents are the leading cause of death among toddlers.

Explanation:

Question 8 See full question

A nurse is teaching bicycle safety to a child and his parents. What protective device should the nurse tell the
parents is most important in preventing or lessening the severity of injury related to bicycle crashes?

You Selected:

Helmet

Correct response:

Helmet

Explanation:

Question 9 See full question

A parent of a 9-year-old-child scheduled to have surgery expresses concern about the potential for postoperative
infection. A nurse provides the parent with information about the measures taken to maintain surgical asepsis.
Typical surgical asepsis involves:

You Selected:

performing a preoperative surgical scrub for at least 3 to 5 minutes.

Correct response:

performing a preoperative surgical scrub for at least 3 to 5 minutes.

Explanation:

Question 10 See full question

A nurse notices that a client admitted for treatment of major depression is pacing, agitated, and becoming verbally
aggressive toward other clients. What is the immediate care priority?

You Selected:

Ensuring the safety of this client and other clients on the unit

Correct response:

Ensuring the safety of this client and other clients on the unit
Explanation:

Question 11 See full question

A client in early labor is connected to an external fetal monitor. The physician hasn't noted any restrictions on her
chart. The client tells the nurse that she needs to go to the bathroom frequently and that her partner can help her.
How should the nurse respond?

You Selected:

"Please press the call button. I'll disconnect you from the monitor so you can get out of bed."

Correct response:

"Please press the call button. I'll disconnect you from the monitor so you can get out of bed."

Explanation:

Question 12 See full question

A nurse is preparing to perform a physical examination on a postpartum client. The client asks the nurse why gloves
are necessary for the examination. What is the nurse's best response?

You Selected:

"Gloves are required for standard precautions."

Correct response:

"Gloves are required for standard precautions."

Explanation:

Question 13 See full question

A nurse is preparing to perform a postpartum assessment on a client who gave birth 5 hours ago. Which precaution
should the nurse plan to take for this procedure?

You Selected:

Washing the hands and wearing latex gloves

Correct response:

Washing the hands and wearing latex gloves

Explanation:

Question 14 See full question

An alarm signals, indicating that a neonate's security identification band requires attention. The nurse responds
immediately and finds that the parents removed the identification bands from the neonate. Which action should the
nurse take next?

You Selected:

Compare the information on the neonate's identification bands with that of the mother's, then reattach
the identification bands to one of the neonate's extremities.

Correct response:

Compare the information on the neonate's identification bands with that of the mother's, then reattach
the identification bands to one of the neonate's extremities.
Explanation:

Question 15 See full question

The nurse is receiving over the telephone a laboratory results report of a neonate's blood glucose level. The nurse
should:

You Selected:

write down the results, read back the results to the caller from the laboratory, and receive confirmation
from the caller that the nurse understands the results.

Correct response:

write down the results, read back the results to the caller from the laboratory, and receive confirmation
from the caller that the nurse understands the results.

Explanation:

Question 16 See full question

The nurse is caring for a neonate diagnosed with early onset sepsis and is being treated with intravenous
antibiotics. Which instructions will the nurse include in the parents teaching plan?

You Selected:

Wash hands thoroughly before touching the neonate.

Correct response:

Wash hands thoroughly before touching the neonate.

Explanation:

Question 17 See full question

After the nurse administers haloperidol 5 mg PO to a client with acute mania, the client refuses to lie down on her
bed, runs out on the unit, pushes clients in her vicinity out of the way, and screams threatening remarks to the
staff. What should the nurse do next?

You Selected:

Seclude the client and use restraints if necessary.

Correct response:

Seclude the client and use restraints if necessary.

Explanation:

Question 18 See full question

A client lives in a group home and visits the community mental health center regularly. During one visit with the
nurse, the client states, "The voices are telling me to hurt myself again." Which question by the nurse
is most important to ask?

You Selected:

"Are you going to hurt yourself?"

Correct response:

"Are you going to hurt yourself?"


Explanation:

Question 19 See full question

Bacterial conjunctivitis has affected several children at a local day care center. A nurse should advise which
measure to minimize the risk of infection?

You Selected:

Perform thorough hand washing before and after touching any child in the day care center.

Correct response:

Perform thorough hand washing before and after touching any child in the day care center.

Explanation:

Question 20 See full question

A 14-year-old with rheumatic fever who is on bed rest is receiving an IV infusion of dextrose 5% r administered by
an infusion pump. The nurse should verify the alarm settings on the infusion pump at which times? Select all that
apply.

You Selected:

when the infusion is started


at the beginning of each shift
when the child returns from X-ray

Correct response:

when the infusion is started


at the beginning of each shift
when the child returns from X-ray

Explanation:

Question 21 See full question

The nurse develops a teaching plan for the client about how to prevent the transmission of hepatitis A. Which
discharge instruction is appropriate for the client?

You Selected:

Ask family members to wash their hands frequently.

Correct response:

Ask family members to wash their hands frequently.

Explanation:

Question 22 See full question

Which precautions should the health care team observe when caring for clients with hepatitis A?

You Selected:

wearing gloves when giving direct care

Correct response:

wearing gloves when giving direct care


Explanation:

Question 23 See full question

The nurse is planning care for a client who had surgery for abdominal aortic aneurysm repair 2 days ago. The pain
medication and the use of relaxation and imagery techniques are not relieving the clients pain, and the client
refuses to get out of bed to ambulate as prescribed. The nurse contacts the health care provider (HCP), explains the
situation, and provides information about drug dose, frequency of administration, the clients vital signs, and the
clients score on the pain scale. The nurse requests a prescription for a different, or stronger, pain medication. The
HCP tells the nurse that the current prescription for pain medication is sufficient for this client and that the client
will feel better in several days. The nurse should next:

You Selected:

explain to the HCP that the current pain medication and other strategies are not helping the client and it is
making it difficult for the client to ambulate as prescribed.

Correct response:

explain to the HCP that the current pain medication and other strategies are not helping the client and it is
making it difficult for the client to ambulate as prescribed.

Explanation:

Question 24 See full question

A nurse is planning care for a client with human immunodeficiency virus (HIV). The registered nurse (RN) is
delegating responsibilities to a licensed practical/vocational nurse (LPN/VN). Which statements by the LPN/VN
indicates understanding of HIV transmission? Select all that apply.

You Selected:

"I will wear a gown, mask, and gloves for all client contact."
"I do not need to wear any personal protective equipment because nurses have a low risk of occupational
exposure."
"I will wear a mask if the client has a cough."

Correct response:

"I will wear a mask, gown, and gloves when splashing of body fluids is likely."
"I will wash my hands after client care."

Explanation:

Question 25 See full question

The nurse meets with a client in the outpatient clinic who is suicidal and refuses to sign a no suicide contract.
What should the nurse do next?

You Selected:

Arrange for immediate hospitalization on a locked unit.

Correct response:

Arrange for immediate hospitalization on a locked unit.

Explanation:

Question 26 See full question

After teaching the parents of a 15-month-old child who has undergone cleft palate repair how to use elbow
restraints, which statement by the parents indicates effective teaching?
You Selected:

We will remove the restraints temporarily at least three times a day to check his skin, then put them right
back on.

Correct response:

We will remove the restraints temporarily at least three times a day to check his skin, then put them right
back on.

Explanation:

Question 27 See full question

In caring for the client with hepatitis B, which situation would expose the nurse to the virus?

You Selected:

a blood splash into the nurses eyes

Correct response:

a blood splash into the nurses eyes

Explanation:

Question 28 See full question

A client who was bitten by a wild animal is admitted to an acute care facility for treatment of rabies. Which type of
isolation does this client require?

You Selected:

Contact

Correct response:

Contact

Explanation:

Question 29 See full question

A client reports to a physician's office for intradermal allergy testing. Before testing, the nurse provides client
teaching. Which client statement indicates a need for further education?

You Selected:

"If I notice tingling in my lips or mouth, gargling may help the symptoms."

Correct response:

"If I notice tingling in my lips or mouth, gargling may help the symptoms."

Explanation:

Question 30 See full question

A client with a cocaine dependency is irritable, anxious, highly sensitive to stimuli, and over-reactive to clients and
staff on the unit. Which action is most therapeutic for this client?

You Selected:

providing the client with frequent "time-outs"

Correct response:

providing the client with frequent "time-outs"


Explanation:

Question 31 See full question

A clients blood pressure is elevated at 160/90 mm Hg. The health care provider (HCP) prescribed clonidine 1 mg
by mouth now. The nurse sent the prescription to pharmacy at 0710, but the medication still has not arrived at
0800. The nurse should do all except:

You Selected:

go to the pharmacy to obtain the drug.

Correct response:

go to the pharmacy to obtain the drug.

Explanation:

Question 32 See full question

What is the priority action that a nurse should take after omitting an ordered medication?

You Selected:

Document the omission and the reason for it.

Correct response:

Notify the prescriber.

Explanation:

Question 33 See full question

After administering an I.M. injection, a nurse should:

You Selected:

discard the uncapped needle and syringe in a puncture-proof container.

Correct response:

discard the uncapped needle and syringe in a puncture-proof container.

Explanation:

Question 34 See full question

A 6-month-old on the pediatric floor has a respiratory rate of 68, mild intercostal retractions, and an oxygen
saturation of 89%. The infant has not been feeding well for the last 24 hours and is restless. Using the SBAR
(Situation-Background-Assessment-Recommendation) technique for communication, the nurse calls the health care
provider (HCP) with the recommendation for:

You Selected:

starting oxygen.

Correct response:

starting oxygen.

Explanation:

Question 35 See full question

When teaching about prevention of infection to a client with a long-term venous catheter, the nurse determines that
the client has understood discharge instructions when the client states:
You Selected:

"I will not remove the dressing until I return to the clinic next week."

Correct response:

"My husband will change the dressing three times per week, using sterile technique.

Explanation:

Question 36 See full question

An alert and oriented older adult female with metastatic lung cancer is admitted to the medical-surgical unit for
treatment of heart failure. She was given 80 mg of furosemide in the emergency department. Although the client is
ambulatory, the unlicensed assistive personnel (UAP) are concerned about urinary incontinence because the client
is frail and in a strange environment. The nurse should instruct the UAP to assist with implementing the nursing
plan of care by:

You Selected:

placing a commode at the bedside and instructing the client in its use

Correct response:

placing a commode at the bedside and instructing the client in its use

Explanation: uestion 1 See full question

A nurse suspects that a child, age 4, is being neglected physically. To best


assess the child's nutritional status, the nurse should ask the parents which
question?
You Selected:

"Do you think your child eats enough?"


Correct response:

"What did your child eat for breakfast?"


Explanation:

Question 2 See full question

A nurse is providing care for a pregnant client. The client asks the nurse how
she can best deal with her fatigue. The nurse should instruct her to:
You Selected:

try to get more rest by going to bed earlier.


Correct response:

try to get more rest by going to bed earlier.


Explanation:

Question 3 See full question

A client tells a nurse that she's going to breast-feed her neonate, but she
isn't sure what she should eat. Which client statement requires further
teaching?
You Selected:

"I'll take all the same medications I was taking before my pregnancy."
Correct response:

"I'll take all the same medications I was taking before my pregnancy."
Explanation:

Question 4 See full question

A health care provider prescribes intravenous normal saline solution to be


infused at a rate of 150 ml/hour for a client. How many liter(s) of solution will
the client receive during an 8-hour shift? Record your answer using one
decimal place (For example: 6.2).
Your Response:

1.2
Correct response:

1.2
Explanation:

Question 5 See full question

The nurse is caring for a 7-year-old child who has just returned from the
postoperative unit after surgery. The child is playing in bed with toys. The
childs parents are smiling and state, Isnt it great that our child does not
have any pain? What is the best response by the nurse?
You Selected:

Some children distract themselves with play while in pain.


Correct response:

Some children distract themselves with play while in pain.


Explanation:

During a prenatal visit, a pregnant client with cardiac disease and slight
functional limitations reports increased fatigue. To help combat this problem,
the nurse should advise her to:
You Selected:

divide daily food intake into five or six meals.


Correct response:

divide daily food intake into five or six meals.


Explanation:

Question 2 See full question

A nurse is providing care for a pregnant client. The client asks the nurse how
she can best deal with her fatigue. The nurse should instruct her to:
You Selected:

try to get more rest by going to bed earlier.


Correct response:

try to get more rest by going to bed earlier.


Explanation:

Question 3 See full question

A nurse is instructing the client to do Kegel exercises. What should the nurse
tell the client to do to perform these pelvic floor exercises?
You Selected:

Stop the flow of urine while urinating.


Correct response:

Stop the flow of urine while urinating.


Explanation:

Question 4 See full question

The nurse is preparing a presentation on nutrition to a group of pregnant


adolescents. Which information would be important for the nurse to include
in the teaching plan?
You Selected:

Three or more servings of dairy products meet the calcium requirement.


Correct response:

Three or more servings of dairy products meet the calcium requirement.


Explanation:

Question 5 See full question

A client receiving external radiation to the left thorax to treat lung cancer has
a nursing diagnosis of Risk for impaired skin integrity. Which intervention
should be part of this client's care plan?
You Selected:

Avoiding using deodorant soap on the irradiated areas


Correct response:

Avoiding using deodorant soap on the irradiated areas


Explanation:

Question 1 See full question

When assessing a child for impetigo, the nurse expects which assessment
findings?
You Selected:

Honey-colored, crusted lesions


Correct response:

Honey-colored, crusted lesions


Explanation:

Question 2 See full question

A nurse is instructing the client to do Kegel exercises. What should the nurse
tell the client to do to perform these pelvic floor exercises?
You Selected:

Stop the flow of urine while urinating.


Correct response:

Stop the flow of urine while urinating.


Explanation:

Question 3 See full question

The nurse is teaching a client with stomatitis about mouth care. Which
instruction is most appropriate?
You Selected:

Gargle with antiseptic mouthwash.


Correct response:

Eat a soft, bland diet.


Explanation:

Question 4 See full question

The nurse instructs the client on health maintenance activities to help


control symptoms from a hiatal hernia. Which statement would indicate that
the client has understood the instructions?
You Selected:

"I will avoid lying down after a meal."


Correct response:

"I will avoid lying down after a meal."


Explanation:

Question 5 See full question

Which of the following observations by the nurse would indicate that a client
is unable to tolerate a continuation of a tube feeding?
You Selected:

A passage of flatus pre- and postfeeding


Correct response:

Formula in the clients mouth during the feeding, and increased cough
Explanation:

Question 1 See full question

A nurse is teaching the parents of a child with cystic fibrosis about proper
nutrition. Which instruction should the nurse include?
You Selected:

Encourage a high-calorie, high-protein diet.


Correct response:

Encourage a high-calorie, high-protein diet.


Explanation:

Question 2 See full question

After instruction of a primigravid client at 8 weeks' gestation about measures


to overcome early morning nausea and vomiting, which client statement
indicates the need for additional teaching?
You Selected:

"I will eat two large meals daily with frequent protein snacks."
Correct response:

"I will eat two large meals daily with frequent protein snacks."
Explanation:

Question 3 See full question

The nurse is preparing a presentation on nutrition to a group of pregnant


adolescents. Which information would be important for the nurse to include
in the teaching plan?
You Selected:

Spinach is an excellent source of calcium in the diet.


Correct response:

Three or more servings of dairy products meet the calcium requirement.


Explanation:

Question 4 See full question

The family members of a client who is near death from colon cancer ask the
nurse what to expect if the client becomes dehydrated. What should the
nurse should tell them?
You Selected:

Dehydration is expected during the dying process.


Correct response:

Dehydration is expected during the dying process.


Explanation:

Question 5 See full question

During the first few weeks after a cholecystectomy, the client should follow a
diet that includes:
You Selected:

a decreased intake of fruits, vegetables, whole grains, and nuts, to minimize


pressure within the small intestine.
Correct response:

a limited intake of fat distributed throughout the day so there is not an


excessive amount in the intestine at any one time.
Explanation:

Question 1 See full question

A depressed client in the psychiatric unit hasn't been getting adequate rest
and sleep. To encourage restful sleep at night, the nurse should:
You Selected:

talk with the client for a long time at night to reduce his anxiety.
Correct response:

gently but firmly set limits on how much time the client spends in bed during
the day.
Explanation:

Question 2 See full question

A nurse is assigned to care for a client with anorexia nervosa. During the first
48 hours of treatment, which nursing intervention is most appropriate for this
client?
You Selected:

Giving the client as much time to eat as desired


Correct response:

Providing one-on-one supervision during meals and for 1 hour afterward


Explanation:

Question 3 See full question

The family members of a client who is near death from colon cancer ask the
nurse what to expect if the client becomes dehydrated. What should the
nurse should tell them?
You Selected:

The health care provider (HCP) will make the decision regarding hydration
therapy.
Correct response:

Dehydration is expected during the dying process.


Explanation:

Question 4 See full question

Which is an appropriate nursing goal for the client who has ulcerative colitis?
The client:
You Selected:

maintains a daily record of intake and output.


Correct response:

verbalizes the importance of small, frequent feedings.


Explanation:

Question 5 See full question

The nurse is caring for a client admitted with pyloric stenosis. A nasogastric
tube placed upon admission is on low intermittent suction. Upon review of
the morning's blood work, the nurse observes that the patient's potassium is
below reference range. The nurse should recognize that the patient may be
at risk for what imbalance?
You Selected:

Metabolic acidosis
Correct response:

Metabolic alkalosis
Explanation:

Question 1 See full question

The nurse administers a tap water enema to a client. While the solution is
being infused, the client has abdominal cramping. What should the nurse
do first?
You Selected:

Temporarily stop the infusion, and have the client take deep breaths.
Correct response:

Temporarily stop the infusion, and have the client take deep breaths.
Explanation:

Question 2 See full question

While making a home visit to a multigravida 2 weeks after the birth of viable
twins at 38 weeks gestation, the nurse observes that the client looks pale,
has dark circles around her eyes, and is breastfeeding one of the twins. The
clients apartment is clean, and nothing appears out of place. The client tells
the nurse that she completed three loads of laundry this morning. A priority
need for this client is:
You Selected:

risk for imbalanced nutrition: Less than body requirements related to twin birth.
Correct response:

fatigue related to home maintenance and caring for twins.


Explanation:

Question 3 See full question

A client returned from surgery eight hours ago and has not voided. Which
action should the nurse take first?
You Selected:

Palpate over the synthesis pubis for fullness.


Correct response:

Palpate over the synthesis pubis for fullness.


Explanation:

Question 4 See full question

One day after cataract surgery the client is having discomfort from bright
light. The nurse should advise the client to:
You Selected:

patch the affected eye when in bright light.


Correct response:

use sunglasses that wrap around the side of the face when in bright light.
Explanation:

Question 5 See full question

The nurse is caring for a postoperative client who has not voided since
before surgery. Which is the nurse's most appropriate action?
You Selected:

Palpate for the bladder above the symphysis pubis


Correct response:

Palpate for the bladder above the symphysis pubis


Explanation:

uestion 1 See full question

The nurse uses Montgomery straps primarily so the client is free from:
You Selected:

wandering.
Correct response:

skin breakdown.
Explanation:

Question 2 See full question

As a first step in teaching a woman with a spinal cord injury and quadriplegia
about her sexual health, the nurse assesses her understanding of her current
sexual functioning. Which statement by the client indicates she understands
her current ability?
You Selected:

"I will not be able to have sexual intercourse until the urinary catheter is
removed."
Correct response:

"I can participate in sexual activity but might not experience orgasm."
Explanation:

Question 3 See full question

Which skin preparation would be best to apply around the client's


colostomy?
You Selected:

petroleum jelly
Correct response:

adhesive skin barrier


Explanation:

Question 4 See full question

The nurse is caring for a client in labor. The client wishes to have a
nonmedicated labor and birth. During the early stages of labor, the client
becomes frustrated with the use of music and imagery. Which of the
following would the nurse include in the clients plan of care? Select all that
apply.
You Selected:

Offer the client epidural anesthesia


Administer butorphal IV
Correct response:

Encourage ambulation
Suggest a shower or bath
Offer the use of a yoga ball
Explanation:

Question 5 See full question

The nurse is recording the intake and output for a client with the following:
D5NSS 1,000 ml; urine 450 ml; emesis 125 ml; Jackson Pratt drain #1 35 ml;
Jackson Pratt drain #2 32 ml; and Jackson Pratt drain #3 12 ml. How many
milliliters would the nurse document as the clients output? Record your
answer using a whole number.
Your Response:

1.2
Correct response:

654
Explanation:

uestion 1 See full question

During chemotherapy, a boy, age 10, loses his appetite. When teaching the
parents about his food intake, the nurse should include which instruction?
You Selected:

"Let your child eat any food he wants."


Correct response:

"Let your child eat any food he wants."


Explanation:

Question 2 See full question

A nurse is assigned to care for a client with anorexia nervosa. During the first
48 hours of treatment, which nursing intervention is most appropriate for this
client?
You Selected:

Providing one-on-one supervision during meals and for 1 hour afterward


Correct response:

Providing one-on-one supervision during meals and for 1 hour afterward


Explanation:

Question 3 See full question

Which is a priority goal for the client with chronic obstructive pulmonary
disease (COPD)?
You Selected:

minimizing chest pain


Correct response:

maintaining functional ability


Explanation:

Question 4 See full question

A postpartum woman who gave birth vaginally has unrelenting rectal pain
despite the administration of pain medication. Which action
is most indicated?
You Selected:

preparing a warm sitz bath for the client


Correct response:

assessing the perineum


Explanation:

Question 5 See full question

A nursing assessment for a client with alcohol abuse reveals a disheveled


appearance and a foul body odor. What is the best initial nursing plan that
would assist the clients involvement in personal care?
You Selected:

Bathing and dressing the client each morning until the client is willing to
perform self-care independently
Correct response:

Assisting the client with bathing and dressing by giving clear, simple directions
Explanation:

Answer Key
Question 1 See full question

An adolescent is diagnosed with iron deficiency anemia. After emphasizing


the importance of consuming dietary iron, the nurse asks him to select iron-
rich breakfast items from a sample menu. Which selection demonstrates
knowledge of dietary iron sources?
You Selected:

Ham and eggs


Correct response:

Ham and eggs


Explanation:

Question 2 See full question

As a client progresses through pregnancy, she develops constipation. What is


the primary cause of this problem during pregnancy?
You Selected:

Inadequate fluid intake


Correct response:

Reduced intestinal motility


Explanation:

Question 3 See full question

The nurse notes that a client is too busy investigating the unit and
overseeing the activities of other clients to eat dinner. To help the client
obtain sufficient nourishment, which plan would be best?
You Selected:

Allow her to send out for her favorite foods.


Correct response:

Serve foods that she can carry with her.


Explanation:

Question 4 See full question

During the first few weeks after a cholecystectomy, the client should follow a
diet that includes:
You Selected:

a decreased intake of fruits, vegetables, whole grains, and nuts, to minimize


pressure within the small intestine.
Correct response:

a limited intake of fat distributed throughout the day so there is not an


excessive amount in the intestine at any one time.
Explanation:

Question 5 See full question

A client who underwent surgery had the following intake on the day of
surgery:
Day shift: 500 mL packed blood cells; 236 mL platelets; 750 mL normal saline
solution; 1 L dextrose 5% in normal saline solution
Evening shift: 250 mL normal saline solution; 1 L dextrose 5% in normal
saline solution
Night shift: 1 L dextrose 5% in normal saline solution.
How many milliliters of solution should the nurse document as the client's
24-hour intake? Record your answer using a whole number.
Your Response:

0
Correct response:

4736
Explanation:

Question 1 See full question

An adolescent is diagnosed with iron deficiency anemia. After emphasizing


the importance of consuming dietary iron, the nurse asks him to select iron-
rich breakfast items from a sample menu. Which selection demonstrates
knowledge of dietary iron sources?
You Selected:

Ham and eggs


Correct response:

Ham and eggs


Explanation:

Question 2 See full question

The client with Cushing's disease needs to modify dietary intake to control
symptoms. In addition to increasing protein, which strategy would
be most appropriate?
You Selected:

Restrict potassium.
Correct response:

Restrict sodium.
Explanation:

Question 3 See full question

The nurse is planning to teach a client with chronic obstructive pulmonary


disease how to cough effectively. Which instruction should be included?
You Selected:

Lie flat on the back, splint the thorax, take two deep breaths, and cough.
Correct response:

Take a deep abdominal breath, bend forward, and cough three or four times on
exhalation.
Explanation:

Question 4 See full question

Which is a priority nursing goal for a client with rheumatoid arthritis? The
client will:
You Selected:

verbalize that recovery from rheumatoid arthritis will require several years of
treatment.
Correct response:

demonstrate use of adaptive equipment.


Explanation:

Question 5 See full question

A triple-lumen indwelling urinary catheter is inserted for continuous bladder


irrigation following a transurethral resection of the prostate. In addition to
balloon inflation, the functions of the three lumens include:
You Selected:

continuous inflow and outflow of irrigation solution.


Correct response:

continuous inflow and outflow of irrigation solution.


Explanation:

Question 1 See full question

An adolescent, age 14, is hospitalized for nutritional management and drug


therapy after experiencing an acute episode of ulcerative colitis. Which
nursing intervention is appropriate?
You Selected:

Providing small, frequent meals


Correct response:

Providing small, frequent meals


Explanation:

Question 2 See full question

An adolescent is on the football team and practices in the morning and


afternoon before school starts for the year. The temperature on the field has
been high. The school nurse has been called to the practice field because the
adolescent is now reporting that he has muscle cramps, nausea, and
dizziness. Which action should the school nurse do first?
You Selected:

Move the adolescent to a cool environment.


Correct response:

Move the adolescent to a cool environment.


Explanation:

Question 3 See full question

A client with a history of renal calculi formation is being discharged after


surgery to remove the calculus. What instructions should the nurse include in
the client's discharge teaching plan?
You Selected:

Increase daily fluid intake to at least 2 to 3 L.


Correct response:

Increase daily fluid intake to at least 2 to 3 L.


Explanation:

Question 4 See full question

A client has been unable to void since having abdominal surgery 7 hours
ago. The nurse should first:
You Selected:

encourage the client to increase oral fluid intake.


Correct response:

assist the client up to the toilet to attempt to void.


Explanation:

-Chest pain
=Assessment -Sense of impending doom
Findings/ what do -Tachycardia
you see during -Dyspnea
Pulmonary -Anxiety/restlessness
Embolism= -Decreased breath sounds
-Signs of circulatory collapse
-Decreased SaO2
-Progressive decreasing LOC, caused by shock
17. =Chest Tubes= 1. drain fluid, blood, or
1. Used to..... air
2. Reestablishes...... 2. negative pressure
3. Facilitates...... 3. lung expansion
4. Restores normal.... 4. intrapleural pressure
18. =Nursing care of chest tubes= 1. Check tubing for kinks,
1. Check tubing for_______, check check connections, keep
connections, keep it _______, make it straight, make sure it's
sure it's stabilized against being stabilized against being
_______ dislodged
2. Keep below... 2. level of patient's chest
3. ________________when 3. Clamp briefly
changing drainage apparatuses 4. q shift, dressing
4. Check site.... changes as ordered
Know institution policy if
disconnected
19 =Pneumothorax Etiology & 1.
. Pathophysiology= -Smoking
1. Risk factors: -Family history
-Trauma
-Pulmonary disease
20 =Pneumothorax Treatment= 1. air and fluid from pleural space
. 1. Removal of.... 2. acid-base balance
2. Correction of 3. further damage
3. Minimize 4. Re-expansion of lung
4. Re-....... 5. Chest tube
5.______________ insertion 6. Oxygen
6. give the patient what ? 7. infection control practice
7. Maintain....
21 =Tension Pneumothorax= 1. the pleural space causing high intrapleural pressures and
. 1. A rapid accumulation of air in......... tension on the heart and great vessels.
2. May result from...... 2. open or closed pneumothorax
3. Can also be caused by...... 3. CPR, central line placement, bronchoscopy, barotrauma
4. As pressure increases : what happens to 4. lung collapses and mediastinal shift to the unaffected side
the lungs and mediastinal? 5. Cardiac output
5. ___________________ compromised 6. large bore needle !
6. whats inserted to release the trapped air 7. hemothorax, usually occurring with pneumo. Called
7. There can also be.... hemopneumothorax. Treatment the same, except stop the
bleeding. (from trauma, anticoagulants, malignancy, PE)
22 =To avoid the side effects of corticosteroids, Nonsteroidal anti-inflammatory drugs (NSAIDs)
. which medication classification is used as an Explanation:
alternative to treating inflammatory NSAIDs are used as an alternative in controlling
conditions of the eyes? inflammatory eye conditions and postoperatively to reduce
inflammation. Miotics are used to cause the pupil to constrict.
a) Miotics Mydriatics cause the pupil to dilate. Cycloplegics cause
b) Cycloplegics paralysis of the iris sphincter.
c) Mydriatics
d) Nonsteroidal anti-inflammatory drugs
(NSAIDs)
23 =To avoid the side effects of corticosteroids, Nonsteroidal anti-inflammatory drugs (NSAIDs)
. which medication classification is used as an Correct
alternative to treating inflammatory Explanation:
conditions of the eyes? NSAIDs are used as an alternative in controlling
inflammatory eye conditions and postoperatively to reduce
a) Miotics inflammation. Miotics are used to cause the pupil to constrict.
b) Cycloplegics Mydriatics cause the pupil to dilate. Cycloplegics cause
c) Mydriatics paralysis of the iris sphincter.
d) Nonsteroidal anti-inflammatory drugs
(NSAIDs)
24 1. Atelectasis....what is it? 1. Collapse of alveoli
. 2. what does it Cause ? 2. inadequate lung expansion, localized airway obstruction,
3. what are symptoms ? inadequate surfactant, increased elastic recoil
4. Best treatment is ? 3. dyspnea, cough, increased HR and respirations, increased
5. Management includes ? work of breathing, decreased O2 sat
4. prevention
5. methods to improve ventilation
25 2nd stage syphillis A flu-like illness, a feeling of tiredness
. and loss of appetite, accompanied by
swollen glands (this can last for weeks or
months).
A non-itchy rash covering the whole body
or appearing in patches.
Flat, warty-looking growths on the vulva
in women and around the anus in both
sexes.
White patches on the tongue or roof of
the mouth.
Patchy hair loss.
26 4 stages of labor 1st stage is latent active transition 2nd
. stage is pushing 3rd stage is placenta 4 th
stage is postpartum
27 A 4-year-old typically has a vivid imagination and lacks As the nurse prepares the equipment to be
. concrete thinking abilities. The mother's assistance (C) can used to start an IV on a 4-year-old boy in
provide a stabilizing presence to help soothe the preschooler, the treatment room, he cries continuously.
who may perceive the invasive procedure as mutilating. To What intervention should the nurse
preserve the child's sense of security associated with the implement?
hospital room, it is best to perform difficult or painful A. Take the child back to his room.
procedures in another area (A). (B) may be necessary to B. Recruit others to restrain the child.
prevent injury if the child is unable to cooperate with the C. Ask the mother to be present to soothe
mother's coaxing. (D) is best done before going to the the child.
treatment room when the child feels less threatened. D. Show the child how to manipulate the
Correct Answer: C equipment.
28 A 20-pound box is safely lifted by bending the knees (D), The charge nurse observes an unlicensed
. holding the box close to the center of gravity, and extending the assistive personnel (UAP) bending at the
legs using the quadriceps muscles. (A and B) might be helpful, waist to lift a 20-pound box of medical
but the charge nurse should use this opportunity to reinforce supplies off the treatment room floor.
proper body mechanics techniques. Pushing the box against What instruction should the charge nurse
the wall (C) does not assist with lifting. provide to the UAP?
Correct Answer: D A. Ask another staff member for
assistance.
B. Request that supplies are delivered in
smaller containers.
C. Push the box against the wall to
provide support while lifting.
D. Bend at the knees when lifting heavy
objects.
29 A 23-year-old college athlete is recovering in the postanesthesia "If I'm vomiting, I'll drink lemon-lime
. care unit from a tonsillectomy. After an overnight stay in the soda to keep myself hydrated."
hospital due to increased secretions and vomitting, you deliver Explanation:
his client education and accompanying paperwork. Which of Instruct the client to avoid carbonated
the client's comments, listed below, indicates a need for fluids and fluids high in citrus content.
additional education? Such fluids are caustic to the surgical site
a) "If I'm vomiting, I'll drink lemon-lime soda to keep myself and may traumatize tissue, disrupting the
hydrated." suture line. (less)
b) "I'll sleep on 2 - 3 pillows."
c) "I promise I won't blow my nose."
d) "I'll gargle with weak salt water 3 - 4 times a day.
30 A 25-year-old client was admitted yesterday after a motor vehicle oral temp 102 F
. collision. Neurodiagnostic studies showed a basal skull fracture in
the middle fossa. Assessment on admission revealed both halo and
Battle signs. Which new symptom indicates that the client is likely to
be experiencing a common life-threatening complication associated
with a basal skull fracture?
31 A 26-year-old client is returning for diagnostic followup. Her Holter Sinus tachycardia---->Sinus
. monitor strip reveals a heart rate with normal conduction but with a tachycardia is a dysrhythmia that
rate consistently above 105 beats per minutes. What type of proceeds normally through the
dysrhythmia would you expect the cardiologist to diagnose? conduction pathway but at a faster
than usual rate (100-150 beats/min)
32 A 29-year-old client with severe shortness of breath comes to the b) Airborne and contact precautions
. emergency department. He tells the emergency department staff
that he recently traveled to China for business. Based on his travel
history and presentation, the staff suspects severe acute respiratory
syndrome (SARS). Which isolation precautions should the staff
institute?

a) Droplet precautions
b) Airborne and contact precautions
c) Contact and droplet precautions
d) Contact precautions
33 A 36-year-old mother of six has been recently diagnosed with type 2 Polyphagia
. diabetes. She reports increased hunger and food consumption while
continuing to lose weight. What is the term used to describe this
condition?
34 A 43-year-old homeless, malnourished female client with a history of Magnesium sulfate
. alcoholism is transferred to the ICU. She is placed on telemetry, and
the rhythm strip shown is obtained. The nurse palpates a heart rate
of 160 beats/min, and the client's blood pressure is 90/54. Based on
these findings, which IV medication should the nurse administer?
35 A 55-year-old male client has been admitted to the hospital with a The client smokes 1 to 2 packs of
. medical diagnosis of chronic obstructive pulmonary disease cigarettes per day.
(COPD). Which risk factor is the most significant in the
development of this client's COPD?
36 A 55-year-old male client is admitted to the coronary care unit Elevated CM-MB level
. having suffered an acute myocardial infarction (MI). Within 24
hours of the occurrence, the nurse can expect to find which systemic
sign?
37 A 58-year-old client, who has no health problems, asks the nurse The immunization is administered
. about taking the pneumococcal vaccine (Pneumovax). Which once to older adults or persons with
statement given by the nurse would offer the client accurate a history of chronic illness."
information about this vaccine?
38 A 58-year-old female client tells the nurse that she feels a sense of Advise the client that Pap smear
. loss since she has stopped having menstrual periods. She then states, tests should be continued.
"At least I will no longer have to suffer through those horrible Pap
smear tests every year." Which action should the nurse implement?
39 A 62-year-old woman who lives alone tripped on a rug in her home Osteoporosis resulting from
. and fractured her hip. Which predisposing factor most likely declining hormone levels
contributed to the fracture in the proximal end of her femur?
40 A 63-year-old client with type 2 diabetes mellitus is admitted for Pedal pulses will be weak or absent
. treatment of an ulcer on the heel of the left foot that has not healed in the left foot.
with conventional wound care. The nurse observes that the entire
left foot is darker in color than the right foot. Which additional
symptom should the nurse expect to find?
41 A 64-year-old client reports symptoms consistent with a transient Impaired cerebral circulation
. ischemic attack (TIA) to the physician in the emergency
department. After completing ordered diagnostic tests, the physician
indicates to the client what caused the symptoms that brought him
to the hospital. What is the origin of the client's symptoms?
42 A 64-year-old patient with chronic open-angle Explaining that this is an expected adverse effect
. glaucoma is being taught to self-administer Correct
pilocarpine. After the patient administers the Explanation:
pilocarpine, the patient states that her vision is Pilocarpine, a miotic drug used to treat glaucoma,
blurred. Which nursing action is most achieves its effect by constricting the pupil. Blurred
appropriate? vision lasting 1 to 2 hours after instilling the eyedrops
is an expected adverse effect. The patient may also
a) Suggesting that the patient put on her glasses note difficulty adapting to the dark. Because blurred
b) Explaining that this is an expected adverse vision is an expected adverse effect, the drug doesn't
effect need to be withheld, nor does the physician need to be
c) Treating the patient for an allergic reaction notified. Likewise, the patient doesn't need to be
d) Holding the next dose and notifying the treated for an allergic reaction. Wearing glasses won't
physician alter this temporary adverse effect.
43 A 65-year-old client has come to the emergency Electrocardiogram
. department reporting lightheadedness, chest pain,
and shortness of breath. As you finish your
assessment, the physician enters and orders tests
to ascertain what is causing the client's problems.
In your client education, you explain the tests.
Which test is used to identify cardiac rhythms?
44 A 68-year-old resident at a long-term care facility Lack of free water intake
. lost the ability to swallow following a stroke 4
years ago. She receives nutrition via a PEG tube.
The client remains physically and socially active
and has adapted well to the tube feedings.
Occasionally, the client develops constipation that
requires administration of a laxative to restore
regular bowel function. Which of the following is
the most likely cause of this client's constipation?
45 A 71-year-old patient is admitted with acute D
. respiratory distress related to cor pulmonale.
Which of the following nursing interventions is
most appropriate during admission of this patient?
A) Perform a comprehensive health history with
the patient to review prior respiratory problems.
B) Complete a full physical examination to
determine the effect of the respiratory distress on
other body functions.
C) Delay any physical assessment of the patient
and review with the family the patient's history of
respiratory problems.
D) Perform a physical assessment of the
respiratory system and ask specific questions
related to this episode of respiratory distress.
46 The 73-year-old client with pneumonia should be complications and the audible crackles that may result
. the nurse's priority because of the oxygenation from fluid overload from the I.V. line
47 A 74-year-old male client is admitted to the ICU Add 5 cm positive end-expiratory pressure (PEEP).
. with a diagnosis of respiratory failure secondary to
pneumonia. Currently, he is ventilator-dependent
with settings of tidal volume (VT) 750 ml and
intermittent mandatory ventilation (IMV) rate of
10. ABG results are pH 7.48; PaCO2 30; PaO2 64;
HCO3 25; and FiO2 0.80. Which intervention
should the nurse implement first?
48 A 76-year-old client has a significant history of Examine the client's neck for distended veins, monitor
. congestive heart failure. During his semiannual the client for signs of lethargy or confusion---->During
cardiology examination, for what should you, as a head-to-toe assessment of a client with congestive
his nurse, specifically assess? Choose all correct heart failure, the nurse checks for dyspnea, auscultates
options. apical heart rate and counts radial heart rate, measures
BP, and documents any signs of peripheral edema,
lethargy, or confusion
49 A 77-year-old female client is admitted to the hospital. She is Digitalis (Lanoxin)
. confused and has had no appetite for several days. She has
been nauseated and vomited several times prior to admission.
She is currently complaining of a headache. Her pulse rate is 43
beats/min. The nurse is most concerned about the client's
history related to what medication?
50 80 year old SPINAL anesthesia and 4,000 ml of room cover this client with warm blankets
. temperature isotonic bladder irrigation it is important to because he is at high risk for hypothermia
secondary to age, spinal anesthesia,
51 An 81-year-old male client has emphysema. He lives at home Help the client to determine ways to
. with his cat and manages self-care with no difficulty. When increase his fluid intake.
making a home visit, the nurse notices that this client's tongue
is somewhat cracked and his eyeballs appear sunken into his
head. What nursing intervention is indicated?
52 An 83-year-old client is undergoing lipid profile studies in an LDL sticks to arteries
. effort to determine a proper nutritional balance for his CAD.
In his lipid profile, his LDL is greater than his HDL. Why is
this a risk factor for this client?
53 210- 216
.

54 210 -216
.

55 The "ABCDs" of melanoma are Asymmetry of the lesion, Borders that


. are irregular, Colors that vary in shades,
and increased Diameter
56 The ABCs of caring for clients are airway, breathing, and When caring for an immobile client, what
. circulation. Impaired gas exchange (B) implies that the client is nursing diagnosis has the highest
having trouble with breathing, which has the highest priority of priority?
the nursing diagnoses listed. Though an immobilized client A. Risk for fluid volume deficit.
presents a multitude of nursing care challenges, (A, C, and D) B. Impaired gas exchange.
do not have the priority of (B). C. Risk for impaired skin integrity.
Correct Answer: B D. Altered tissue perfusion.
57 An abdominal CT scan is used to stage the presence of colorectal cancer.
.

58 An abdominal CT scan is used to stage the presence of colorectal cancer.


.

59 Abdominal discomfort secondary to constipation will be bowel movement; an opioid would


. relieved after the client has a contribute to the constipation.
60 Abdominal distention is an early sign of infection hirshsprung and therefore the
. parents need to report it to the physician.
61 abg apply pressure for 5 minuets and 15 minutes if
. anticoagulant therapy in the arm but
femoral artery you keep pressure for 15
minutes
62 The ABI test is a noninvasive test that compares the systolic blood pressure in the arm with
. that of the ankle.
63 The ABI test is a noninvasive test that compares the systolic ...
. blood pressure in the arm with that of the ankle.
64 ABO compatibility is not a necessary requirement for plasma , human leukocyte antigen (HLA)
. but matching of lymphocytes may be
completed to avoid development of anti-
HLA antibodies when multiple platelet
transfusions are necessary.
65 About 12 to 24 hours after a subtotal gastrectomy, gastric brown, which indicates digested blood
. drainage is normally
66 Abrupt withdrawal of endogenous cortisol may lead to severe adrenal insufficiency.
.

67 abundant wbc is neutrophil


.

68 Access is also needed for TPN, preferably via a \ central line.


.

69 Accompanying symptoms of prostate cancer can includ constipation, weight loss, and
. lymphadenopathy.
70 According to states' nurse practice acts, it is the The charge nurse assigns a nursing procedure to a
. responsibility of the nurse to function within the scope new staff nurse who has not previously performed
of competency (D), and in this case safe nursing the procedure. What action is most important for
practice constitutes refusal to perform the procedure the new staff nurse to take?
because of a lack of experience. Although state A. Review the steps in the procedure manual.
mandates, agency policies, and continued education B. Ask another nurse to assist while
and experience identify tasks that are within the scope implementing the procedure.
of nursing practice, nurses should first refuse to C. Follow the agency's policy and procedure.
perform tasks that are beyond their proficiency, and D. Refuse to perform the task that is beyond the
then pursue opportunities to enhance their competency nurse's experience.
(A, B, and C).
Correct Answer: D
71 According to the American Society of Anesthesiology P3; classification P3 pateints are those who have
. Physical Status Classification System, a patient with compensated heart failure, cirrhosis or poorly
severe systemic disease that is not incapacitating is controlled diabetes. Classification P4 pateints
noted to have the physical status classification of which have an incapacitating system disease that is a
of the following? constant threat to life. Classification P1 is healthy.
Classification P2 reflects patient with mild
systemic disease
72 Ace inhibitors are what? -prils
.

73 Acetone in the urine would indicate excessive fat catabolism


.

74 acid ash diet Other foods to avoid on this diet include vegetables except corn and lentils; all fruits
. all except cranberries, plums, and prunes; and any
food containing large amounts of potassium,
sodium, calcium, or magnesium
75 Acknowledging a client's beliefs and customs related to A nurse is becoming increasingly frustrated by
. sickness and health care are valuable components in the family members' efforts to participate in the
the plan of care that prevents conflict between the goals care of a hospitalized client. What action should
of nursing and the client's cultural practices. Cultural the nurse implement to cope with these feelings
sensitivity begins with examining one's own cultural of frustration?
values (B) to compare, recognize, and acknowledge A. Suggest that other cultural practices be
cultural bias. (A and C) do not consider the family's substituted by the family members.
needs to care for the client and are not the best ways to B. Examine one's own culturally based values,
cope with the nurse's frustration. Although (D) may be beliefs, attitudes, and practices.
an option, examining one's cultural differences allows C. Explain to the family that multiple visitors are
the nurse to cope, empathize, and implement culturally exhausting to the client.
specific interventions pertaining to the needs of the D. Allow the situation to continue until a family
client and the family. member's action may harm the client.
Correct Answer: B
76 Acromegaly Evaluation of an adult client reveals Buldging forehead-->Oversecretion of growth
. oversecretion of growth hormone. Which of the hormone in an adult results in acromegaly,
following would the nurse expect to find? manifested by coarse features, a huge lower jaw,
thick lips, thickened tongue, a bulging forehead,
bulbous nose, and large hands and feet. Excessive
urine output, weight loss, and constant thirst are
associated with diabetes insipidus.
77 Actinic keratosis is a premalignant skin lesion.
.

78 Actions of Anti-Tuberculosis Medications 1. inhibit the growth of bacteria


. 1. Bacteriostatic agents, how do they work ? 2. destroy bacteria
2. what do they do to bacteria ?
79 Actions of Anti-Tuberculosis Medications 1. rapidly
. 1. Tubercle bacilli killed..... 2. chemotherapy
2. Combination _____________ minimizes potential for drug 3. sterilizes host tissues
resistance 4. cured with very small likelihood of
3. Sufficient length of treatment does what ? relapse
4. Patient is.....
80 The actions of parathyroid hormone (PTH) are increased in D
. the presence of which vitamin?
81 Active, rather than passive, ROM is best to restore strength The nurse is caring for a client who is weak
. and (B) is an effective schedule. Passive ROM 4 times a day from inactivity because of a 2-week
(A) is not as beneficial for the client as (B). With weights (C), hospitalization. In planning care for the
the client may fatigue quickly and develop muscle soreness. client, the nurse should include which range
ROM is not performed beyond the point of resistance or of motion (ROM) exercises?
pain (D) because of the risk of damage to underlying A. Passive ROM exercises to all joints on all
structures. extremities four times a day.
Correct Answer: B B. Active ROM exercises to both arms and
legs two or three times a day.
C. Active ROM exercises with weights
twice a day with 20 repetitions each.
D. Passive ROM exercises to the point of
resistance and slightly beyond.
82 Acute angle-closure glaucoma produces abrupt changes in the angle of the iris.
.

83 Acute asthmatic attacks are characterized by wheezing.


.

84 Acute bone pain and confusion are associated with aluminum intoxication, another potential
. complication of dialysis.
85 acute respiratory distress syndrome aka shock lung and white lung which is
. increased permeability of alveolar cap lets
the fluid leak out into interstitial space
86 acute respiratory distress syndrome is a non cardiac pulmonary edema
.

87 addision Other early signs and symptoms include mood changes, emotional lability,
. irritability, weight loss, muscle weakness,
fatigue, nausea, and vomiting. Most clients
experience a loss of appetite. Muscles
become weak, not spastic, because of
adrenocortical insufficiency.
LETHARGY
88 addison process is believed to be autoimmune in nature.
.

89 Additional oral cancer risk factors include chronic irritation such as a broken tooth or
. ill-fitting dentures, poor dental hygiene,
overexposure to sun (lip cancer), and
syphilis.
90 Adenosine treats tachyarrhythmias
.

91 Adequate fluid intake of at least 8 glasses a day prevents sulfasalazine therapy.


. crystalluria and stone formation during
92 Adequate protein intake is necessary for improving skin integrity.
.

93. Adequate visualization or palpation of the perineum (A) is essential When teaching a female client to
to ensure correct placement of the catheter. (B) is not necessary to perform intermittent self-
perform self-catheterization. During a self-catheterization, the client catheterization, the nurse should
typically allows the urine to drain into an open collection device, ensure the client's ability to
rather than a drainage bag (C), and uses a straight catheter without perform which action?
a balloon (D). A. Locate the perineum.
Correct Answer: A B. Transfer to a commode.
C. Attach the catheter to a drainage
bag.
D. Manipulate a syringe to inflate
the balloon.
94. Administering serum albumin increases the plasma colloid osmotic pressure,
which causes fluid to flow from the
tissue space into the plasma.
95. The adolescent is most likely experiencing heat exhaustion or heat result from loss of fluids and
collapse, Symptoms include nausea, vomiting,
dizziness, headache, and thirst.
96. An adult client has bacterial conjunctivitis. What should the nurse Use warm saline soaks four times
teach him to do? Select all that apply per day to remove crusting.
Apply topical antibiotic without
touching the tip of the tube to his
eye.
Wash his hands after touching his
eyes.
Avoid touching his eyes.
97. An adult client with cystic fibrosis is admitted to an acute care b) At least 2 hours after a meal
facility with an acute respiratory infection. Prescribed respiratory
treatment includes chest physiotherapy. When should the nurse
perform this procedure?

a) Immediately before a meal


b) At least 2 hours after a meal
c) When bronchospasms occur
d) When secretions have mobilized
98. Adult lice usually bite the scalp behind the ears and
along the back of the neck.
99. An adult resident in a long-term care facility is diagnosed with Discuss the importance of all
hepatitis B. Which intervention should the nurse implement with the employees starting the hepatitis B
staff caring for this client? vaccine series.
100 Adults and children with gonorrhea may develop gonococcal conjunctivitis by
. touching the eyes with
contaminated hands
101 Advance directives are written statements of a A male client with acquired immunodeficiency
. person's wishes regarding medical care, and verbal syndrome (AIDS) develops cryptococcal meningitis
directives may be given to a healthcare provider and tells the nurse he does not want to be resuscitated
with specific instructions in the presence of two if his breathing stops. What action should the nurse
witnesses. To obtain this prescription, the client implement?
should discuss his choice with the healthcare A. Document the client's request in the medical
provider (B). (A) is insufficient to implement the record.
client's request without legal consequences. B. Ask the client if this decision has been discussed
Although (C and D) provide legal protection of the with his healthcare provider.
client's wishes, the present request needs C. Inform the client that a written, notarized advance
additional action. directive, is required to withhold resuscitation efforts.
Correct Answer: B D. Advise the client to designate a person to make
healthcare decisions when the client is unable to do
so.
102 Adverse effects of danazol (Danocrine) include headaches, dizziness, irritability, and decreased
. libido. Masculinization effects, such as deepened
voice, facial hair, and weight gain, also may occur.
103 Adverse effects of pseudoephedrine (Sudafed) are cardiovascular system and through sympathetic
. experienced primarily in the effects on the central nervous system (CNS).
104 . A family or personal history of breast cancer or a HRT hormone replacement therapy
. history of estrogen-dependent dysplasia is an
absolute contraindication for
105 After a bronchoscopy with a lung biopsy, the nurse pneumothorax as well as hemorrhage
. should monitor the client for signs of
106 After admitting a patient to the medical unit with a B
. diagnosis of pneumonia, the nurse will verify that
which of the following physician orders have been
completed before administering a dose of cefotetan
(Cefotan) to the patient?
A) Orthostatic blood pressures
B) Sputum culture and sensitivity
C) Pulmonary function evaluation
D) Serum laboratory studies ordered for am
107 After a fracture, during which stage or phase of remodeling
. bone healing is devitalized tissue removed and new
bone reorganized into its former arrangement
108 After attending a class on reducing cancer risk Switch to skim milk.
. factors, a client selects bran flakes with 2% milk
and orange slices from a breakfast menu. In
evaluating the client's learning, the nurse affirms
that the client has made good choices, and makes
what additional recommendation?
109 After being sick for 3 days, a client with a history Serum potassium level-->The nurse should monitor
. of diabetes mellitus is admitted to the hospital with the client's potassium level because during periods of
diabetic ketoacidosis (DKA). The nurse should acidosis, potassium leaves the cell, causing
evaluate which diagnostic test results to prevent hyperkalemia. As blood glucose levels normalize
arrhythmias? with treatment, potassium reenters the cell, causing
hypokalemia if levels aren't monitored closely.
Hypokalemia places the client at risk for cardiac
arrhythmias such as ventricular tachycardia. DKA has
a lesser affect on serum calcium, sodium, and
chloride levels. Changes in these levels don't
typically cause cardiac arrhythmias.
110. After cleaning the hub with alcohol or povidone- I.V. extension set and restart the infusion.
iodine solution, the nurse must replace the
111. After intracavity radiation, some vaginal bleeding occurs for 1 to 3 months.
112. After removal of nasal packing, the client should water-soluble jelly to the nares to lubricate the nares
be instructed to apply and promote comfort.
113. After supratentorial surgery, the nurse 30 degrees to promote venous outflow through the jugular
should elevate the client's head veins.
114. After the corticotropin-secreting tumor is shouldn't be at risk for hyperglycemia.
removed, the client
115. After the health care provider sees a patient Sputum specimen for Gram stain and culture and sensitivity
hospitalized with a stroke who developed a
fever and adventitious lung sounds, the
following orders are written, Which will the
nurse implement first?
116. After the health care provider sees a patient Sputum specimen for Gram stain and culture and sensitivity
hospitalized with a stroke who developed a
fever and adventitious lung sounds, the
following orders are written. Which will the
nurse implement first?
117. After undergoing a thyroidectomy, a client Calcium gluconate--->Immediate treatment for a client who
develops hypocalcemia and tetany. Which develops hypocalcemia and tetany after thyroidectomy is
electrolyte should the nurse anticipate calcium gluconate. Potassium chloride and sodium
administering? bicarbonate aren't indicated. Sodium phosphorus wouldn't
be given because phosphorus levels are already elevated.
118. Aging and ototoxicity are two causes of sensorineural hearing loss.
119. airborne mtv measles varicella tb
120 Airborne precautions prevent transmission infectious over long distances when suspended in the air
. of infectious agents that remain (e.g. mycobacterium tuberculosis, measles, varicella virus
[chickenpox], and possibly SARS-CoV).
121 air born precaution The preferred placement isolation single-patient room that is equipped with special
. is in an air handling and ventilation.
122 Albuterol (Proventil) is used as the "rescue inhaler" for bronchospasms.
.

123 : All clients exposed to persons with prophylactic isoniazid in daily doses of 300 mg for 6
. tuberculosis should receive months to 1 year to avoid the deleterious effects of the
latent mycobacterium
124 allergic reaction to the dye used during the pruritus and urticaria, which may indicate a mild
. arteriogram anaphylactic reaction Decreased alertness may and dyspnea
(not hypoventilation).
125 Allograft tissue, taken from another person, takes longer to incorporate into the
. recpient'sr body, but there is no second surgical site to heal.
Also, the surgical time and hospital stay may be shorter
when allograft tissue is used.
126 Allograft tissue transplants are not rejected body as with organ transplants, so that it is not necessary to
. by the use drugs to suppress
127 Alpha1 blockers are what? the -osins
. sympatholitics
128 Alpha 1 receptors are responsible for what? vasoconstriction
.

129 alpha-adrenergic blockers. These drugs relax smooth muscle of the bladder neck and prostate, so the
. the urinary symptoms of BPH are reduced in many clients.
130 An alteration in the protective pressure decline in the number of Meissner's and pacinian
. sensation results from a corpuscles.
131 Although all the options are associated with irritability and drowsiness suggests a decrease in hepatic
. hepatitis B, the onset of function.
132 Although an upper GI series might confirm whether the lesion is bleeding.
. the presence of a lesion, it wouldn't
necessarily reveal
133 Although loop diuretics block potassium reabsorption, this isn't a therapeutic action.
.

134 Although steroids should be given during assess the client for it.
. surgery to prevent hypocortisolism, the
nurse should
135 Amantadine, an antiviral agent
.

136 Amantadine and diphenhydramine enhance the effects of anticholinergic agents.


.

137 Amantadine, digoxin, and diphenhydramine can interact cholinergic blocking agent but not through
. with a delayed absorption.
138 Amenorrhea develops in Cushing's syndrome. With successful
. treatment, the client experiences a return of
menstrual flow, not a decline in it.
139 amphotericin B, an antifungal agent,
.

140 An amplitude decrease would support the nurse's suspicion amplitude of the QRS complexes on an
. because fluid surrounding the heart, such as in cardiac ECG.
tamponade, suppresses the
141 Anal excoriation is inevitable with profuse diarrhea, and Sitz baths are comforting and cleansing
. meticulous perianal hygiene is essential.
142 Analysis of behavior patterns using Erikson's framework can The daughter of an older woman who
. identify age-appropriate or arrested development of normal became depressed following the death of
interpersonal skills. Erikson describes the successful her husband asks, "My mother was always
resolution of a developmental crisis in the later years (older well-adjusted until my father died. Will she
than 65-years) to include the achievement of a sense of tend to be sick from now on?" Which
integrity and fulfillment, wisdom, and a willingness to face response is best for the nurse to provide?
one's own mortality and accept the death of others (B). A. She is almost sure to be less able to
Depression is a component of normal grieving, and (A) does adapt than before.
not represent susceptible adaptation to the developmental B. It's highly likely that she will recover
crisis of an older adult, Integrity vs despair. (C and D) are and return to her pre-illness state.
judgmental and not therapeutic. C. If you can interest her in something
Correct Answer: B besides religion, it will help her stay well.
D. Cultural strains contribute to each
woman's tendencies for recurrences of
depression.
143 Anemia is a common problem with multiple gestation clients
.

144 Anesthesia and analgesia can slow the process o labor


.

145 Aneurysm rebleeding occurs most frequently during which First 2 weeks
. timeframe after the initial hemorrhage?
146 Angiotensin-converting enzyme-inhibitor drugs,may help to ventricular remodeling
. prevent
147 Ankle edema seldom follow CABG surgery and may indicate right-
. sided heart failure.
148 Answering questions simply and directly provides A 4-year-old boy who is scheduled for a
. comfort for the preschool-age child and builds confidence tonsillectomy and adenoidectomy asks the
in the health care team (D). (A) uses language (i.e. nurse, Will it hurt to have my tonsils and
'incision') that could create anxiety for the child. Four- adenoids taken out? Which response is best for
year-olds are in the Initiative vs. Guilt stage (Erikson's the nurse to provide?
psychosocial development), and (B) contributes to guilt A. It may hurt a little because of the incision
when the child hurts. (C) is not helpful because the child made in your throat.
may associate being put to sleep with the postoperative B. It won't hurt because you're such a big boy.
throat pain and then become fearful of going to sleep. C. It won't hurt because we put you to sleep.
Correct Answer: D D. It may hurt but we'll give you medicine to
help you feel better.
149 An antacid (Maalox) is prescribed for a client with peptic Maintenance of a gastric pH of 3.5 or above
. ulcer disease. What is the therapeutic action of this
medication which is effective in treating the client's
ulcer?
150 anterior surface of the right ventricle and the anterior
. surface of the left ventricle.

151 antidote for oxytocin is mag sulfate


.
152 ANTITUSSIVES Diphenhydramine (Benadryl)
. Prototype: Dextromethorphan Benzonatate (Tessalon)
Opioid Antitussives
Codeine
Hydrocodone
153 Anxiety stimulates the sympathetic nervous system, epinephrine, angiotensin, and serum proteins
. which results in the secretion of that cause vasoconstriction in the arteries of
the peripheral circulatory system. As a result,
peripheral vascular resistance is increased.
154 Any possible break in surgical asepsis that is identified A healthcare provider is performing a sterile
. when others are unaware should be considered procedure at a client's bedside. Near the end of
contaminated and new sterile supplies added to maintain the procedure, the nurse observes the
the sterile field (D). Reporting the healthcare provider is healthcare provider contaminate a sterile glove
not indicated (A). When sterility is suspect during aseptic and the sterile field. What is the best action for
technique, it should not be questioned (C) but all the nurse to implement?
members of the team should move forward with A. Report the healthcare provider for the
reestablishing a sterile field. Allowing the procedure to violation in aseptic technique.
progress under unsterile conditions (B) places the client B. Allow the completion of the procedure.
at risk for infection and is an act of omission (negligence) C. Ask if the glove and sterile field are
by the nurse and other healthcare team members. contaminated.
Correct Answer: D D. Identify the break in surgical asepsis and
provide another set of sterile supplies.
155 An aortic murmur i loud and rough and is heard over the aortic area.
.

156 Aphasia is more commonly present when the left hemisphere is damaged.
. dominant or
157 Applying pressure against the nose at the inner entering the lacrimal (tear) duct. If the medication
. canthus of the closed eye after administering enters the tear duct, it can enter the nose and pharynx,
eyedrops prevents the medication from where it may be absorbed and cause toxic symptoms
158 An appropriate nursing intervention for a Teach the patient how to cough effectively to bring
. patient with pneumonia with the nursing secretions to the mouth
diagnosis of ineffective airway clearance related
to thick secretions and fatigue would be to:
159 (A) provides the best schedule, because QID A medication is prescribed to be given QID. What
. means four times per day. (B, C, and D) provide schedule should the nurse use to administer this
incorrect dosages. prescription?
Correct Answer: A A. 0800, 1200, 1600, 2000.
B. 800.
C. Every other day at 0800.
D. 0800, 1200, 1600, 2000, 0000, 0400.
160 arsenic exposure After the acute phase bone marrow depression, encephalopathy, and sensory
. neuropathy occur.
161 arsenic exposure Dehydration can lead to shock and death.
.
162 arsenic symptoms Violent vomiting.
. Severe diarrhea.
Abdominal pain.
163 Arteriovenous malformation or AVM is an connection between veins and arteries, usually
. abnormal congenital, congenital malformed blood vessels in the
ventricles
164 as asa
.

165 As a compensatory mechanism for HF, how does Cerebral perfusion pressure drops as a result of low
. the reduced CO and bloodflow affect the brain? CO, and the pp-gland secretes antidiuretic hormone to
increase water reabsorption in the renal tubules,
increasing blood volume, and therefore workload of the
failing heart.
166 As a compensatory mechanism for HF, how does Kidneys sense reduced blood flow and trigger the
. the reduced CO and bloodflow affect the RAAS system to produce aldosterone, which promotes
kidneys? sodium retention and is a potent vasoconstrictor, in
order to raise BP, which increases the workload of the
failing heart.
167 As a compensatory mechanism for HF, how is the SNS is activated due to reduced stroke volume and CO,
. activation of the SNS counterproductive? then it releases Epi and Nepi, which initially increase
HR and contractility. However, that increases the heart's
workload and need for O2.
168 As a result of cholinergic crisis, the muscles stop ACh, leading to flaccid paralysis, respiratory failure,
. responding to the bombardment of increased sweating, salivation, bronchial secretions
along with miosis.
169 Aspiration of a blood return in the lumen of a When preparing to administer an intravenous
. central venous catheter indicates that the catheter medication through a central venous catheter, the
is in place and the medication can be administered. nurse aspirates a blood return in one of the lumens of
The nurse should flush the tubing with the saline the triple lumen catheter. Which action should the
solution, administer the medication (A), then flush nurse implement?
the lumen with saline again. (B and C) are not A. Flush the lumen with the saline solution and
necessary. The aspirated blood can be flushed back administer the medication through the lumen.
through the closed system into the client's B. Determine if a PRN prescription for a
bloodstream, but does not need to be withdrawn thrombolytic agent is listed on the medication record.
(D). C. Clamp the lumen and obtain a syringe of a dilute
Correct Answer: A heparin solution to flush through the tubing.
D. Withdraw the aspirated blood into the syringe and
use a new syringe to administer the medication.
170 Aspirin, magnesium hydroxide, and oral use during pregnancy because these agents may
. antidiabetic agents aren't recommended for cause fetal harm.
171 Assessment Findings with pneumonia 1. Dullness
. 1. __________ with percussion 2. fever greater than 100F (37.8 C)
2. Sudden onset of..... 3. Shaking and chills (bacterial)
3. this happens with (bacterial) pneumonia ? 4. coughing
4. Chest pain aggravated by.... 5. Dyspnea, respiratory grunting, and nasal flaring
5. Dyspnea....along with what ? 6. respiratory rate, tachycardia
6. Increased.... 7. lung sounds, diminished
7. Abnormal...... 8. Purulent
8_________ sputum 9. Anxiety and agitation
9. ___________and ___________
172 Assessment of a client taking lithium reveals dry Continue the lithium and reassure the client that these
. mouth, nausea, thirst, and mild hand tremor. Based temporary side effects will subside.
on an analysis of these findings, which of the
following should the nurse do next?
173 Assessment of visual acuity reveals that the client Myopia
. has blurred vision when looking at distant objects Explanation:
but no difficulty seeing near objects. The nurse Myopia, or nearsightedness, refers to the condition in
documents this as which of the following? which the client can see near objects but has blurred
distant vision. Astigmatism is an irregularity in the
curve of the cornea, which can affect both near and
distant vision. Hyperopia, or farsightedness, refers to
the client's ability to see distant objects clearly, but
sees near objects as blurry. Emmetropia refers to
normal eyesight in which the image focuses precisely
on the retina.
174 Assessment of visual acuity reveals that the client Myopia
. has blurred vision when looking at distant objects Correct
but no difficulty seeing near objects. The nurse Explanation:
documents this as which of the following? Myopia, or nearsightedness, refers to the condition in
which the client can see near objects but has blurred
a) Myopia distant vision. Astigmatism is an irregularity in the
b) Astigmatism curve of the cornea, which can affect both near and
c) Emmetropia distant vision. Hyperopia, or farsightedness, refers to
d) Hyperopia the client's ability to see distant objects clearly, but
sees near objects as blurry. Emmetropia refers to
normal eyesight in which the image focuses precisely
on the retina.
175 As the clinic nurse caring for a client with varicose Demonstrate hoe to apply and remove elastic support
. veins, what is an appropriate nursing action for stockings
this client?
176 As the emergent period ends and capillary fluid in the interstitial compartment will return to the
. permeability returns to normal, the intravascular compartment.
177 Asymptomatic proteinuria is an initial sign of . Microscopic proteinuria
. should be monitored yearly in
all clients with diabetes for
over 5 years.
178 At birth, visual acuity is estimated at approximately 20/100 to 20/150, but it
. improves rapidly during
infancy and toddlerhood.
179 Atomic mass unit (amu) smaller unit of mass
. 1 amu= 1.66x 10(-24)g
180 Atomic mass (unit/location) -Measured in amu (atomic
. mass unit)
-Found using periodic table
181 Atopic dermatitis is a long-term (chronic) skin
. disorder that involves scaly
and itchy rashes
182 ATTENUVAX* (Measles Virus Vaccine Live)
.

183 An audible gurgling sound produced by a dying client is characteristic of A client in hospice care
. ineffective clearance of secretions from the lungs or upper airways, develops audible gurgling
causing a rattling sound as air moves through the accumulated fluid. The sounds on inspiration. Which
nursing priority in this situation is to convey to the family that the client's nursing action has the highest
death is imminent (D). Although culturally sensitive care should be priority?
observed throughout the client's plan of care (A), this is not the priority A. Ensure cultural customs
at this time. Administration of oxygen may be expected care, but a flow are observed.
rate greater than 2 L/minute (B) is not palliative care. (C) may provide B. Increase oxygen flow to
additional information, but is not necessary as death approaches. 4L/minute.
Correct Answer: D C. Auscultate bilateral lung
fields.
D. Inform the family that
death is imminent.
184 autograft donar site A pressure dressing is not needed over the donor site impair healing.
. and ca
185 autograft donar site Occlusive dressings are not used because they do not keep the donor site dry and
. open to the air.
186 autograft donar site Single-layer gauze dressings impregnated with petroleum, scarlet red, or
. biosynthetic dressings may be
used to cover the donor site
as it heals
187 autograft It is important to keep donor site clean, dry, and free of pressure.
.

188 autograph donar Elastic bandages are not used they constrict circulation and can impede healing.
. because
189 autoimmune disorders Other common laboratory Coombs-positive hemolytic anemia,
. findings in these clients include thrombocytopenia, leukopenia, immunoglobulin
excesses or deficiencies, antinuclear antibodies,
antibodies to deoxyribonucleic acid and
ribonucleic acid, rheumatoid factors, elevated
muscle enzymes, and changes in acute phase-
reactive proteins.
190 An automated drug delivery system most effectively the likelihood of medication errors by
. reduces automatically dispensing the drug.
191 Avogadro's number 6.022x10(23) units of that substance
.

192 avoid shoes to stop falls slippers and shoes with deep treads
.

193 Azithromycin is the drug of choice for treatin legionnaires' disease.


.

194 Back muscle strains are common, and moderate strains can cause mild pain and
. stiffness.
195 The bacteria may enter the body through wounds, or they may live in improperly canned
. or preserved food
196 The bag containing platelets needs to be gently rotated to prevent clumping.
.

197 balanced suspension traction The client should be do not press against the footboard. Therefore,
. positioned so that the feet elevating the head of the bed no more than 25
degrees is recommended to keep the client from
moving down in the bed.
198 Bandages for burns may be elasticized and often are occlusive pressure dressing.
. used to form an
199 Basal cell carcinoma presents as lesions that are lightly pigmented. As they
. enlarge, their centers become depressed and their
borders become firm and elevated
200 Based on the WHO pain relief ladder, adjunct A client who has moderate, persistent, chronic
. medications, such as gabapentin (Neurontin), an neuropathic pain due to diabetic neuropathy takes
antiseizure medication, may be used at any step for gabapentin (Neurontin) and ibuprofen (Motrin,
anxiety and pain management, so (A) should be Advil) daily. If Step 2 of the World Health
implemented. Nonopiod analgesics, such as ibuprofen Organization (WHO) pain relief ladder is
(A) and aspirin (C) are Step 1 drugs. Step 2 and 3 prescribed, which drug protocol should be
include opioid narcotics (D), and to maintain freedom implemented?
from pain, drugs should be given around the clock A. Continue gabapentin.
rather than by the client s PRN requests. B. Discontinue ibuprofen.
Correct Answer: A C. Add aspirin to the protocol.
D. Add oral methadone to the protocol.
201 A basilar skull fracture commonly causes only periorbital ecchymosis (raccoon's eyes) and
. postmastoid ecchymosis (Battle sign); however, it
sometimes also causes otorrhea, rhinorrhea, and
loss of cranial nerve I (olfactory nerve) function.
202 Bathing often makes a client feel weak, and if a client What client statement indicates to the nurse that
. is already feeling weak (B), assistance is required the client requires assistance with bathing?
during the bathing process to ensure the client's safety. A. I wasn't able to pack a bag before I left for the
(A and C) do not pose safety issues. Although (D) may hospital.
pose a safety issue, further assessment is needed to B. I don't understand why I'm so weak and tired.
determine if this in fact poses a safety issue for the C. I only bathe every other day.
client. D. I left my eyeglasses at home.
Correct Answer: B
203 Beau's line is a horizontal depression in the nail plate.
. Occurring alone or in multiples, these
depressions result from a temporary
disturbance in nail growth.
204 Because aerobic exercise may increase blood pressure and epistaxis, the client with hypertension
. increased blood pressure can cause should avoid it.
205 Because AFP levels are usually highest at 15 to 18 weeks' gestation, this is the
. optimum time for testing.
206 Because external radiation commonly causes skin irritation, water only and leave the area open to air.
. the nurse should wash the irradiated area with No soaps, deodorants, lotions, or powders
should be applied.
207 Because HF is a progressive disease, how are interventions With quality of life goals
. planned?
208 Because of fluid shifts, weight monitoring is extremely -3 lb gain in 2 days
. important with someone who has chronic HF, what should be -3 to 5lb gain over a week
reported to the HCP?
209 Because of its location near the xiphoid process, th liver is the organ most easily damaged
. from pressure exerted over the xiphoid
process during CPR.
210 Because of the inflammation, a common complication of thrombus formation and potential
. Buerger's disease is occlusion of the vessel. Inflammation of
the immediate and small arteries and
veins is involved in the disease process.
211. Because such lice are tiny (1 to 2 mm) with grayish white bodies, they are hard to see.
However, their bites result in visible
pustular lesions.
212 Because terbutaline can cause tachycardia, the woman should monitor her radial pulse and call the
. be taught to physician for a heart rate greater than 120
beats/minute.
213 Because terbutaline can cause tachycardia, the woman should radial pulse and call the physician for a
. be taught to monitor her heart rate greater than 120 beats/minute.
214 Because the client has a peritoneal catheter in place, blood- abdominal vessels, and the physician
. tinged drainage should not occur. Persistent blood-tinged should be notified. The bleeding is
drainage could indicate damage to the originating in the peritoneal cavity, not
the kidneys.
215 Because the client is anesthetized, the client may not feel the urge to push so bearing-down
. efforts during the second stage of labor
may be less effective.
216 Because the contrast medium used in PTCA acts as an osmotic diuresis with resultant fluid volume
. diuretic, the client may experience deficit after the procedure. Additionally,
potassium levels must be closely
monitored because the client may develop
hypokalemia due to the diuresis.
217 A bed cradle is used to keep the top bedclothes off the client, When making the bed of a client who
. so the nurse should drape the top sheet and covers loosely needs a bed cradle, which action should
over the cradle (D). A client using a bed cradle may still be the nurse include?
able to ambulate independently (A) and does not require A. Teach the client to call for help before
raised side rails (B). (C) causes the nurse to use poor body getting out of bed.
mechanics. B. Keep both the upper and lower side
Correct Answer: D rails in a raised position.
C. Keep the bed in the lowest position
while changing the sheets.
D. Drape the top sheet and covers loosely
over the bed cradle.
218 Bed rest is prescribed for a client with pneumonia during the Decreased cellular demand for oxygen.
. acute phase of the illness. The nurse should determine the
effectiveness of bed rest by assessing the client's:
219 Before amniocentesis, what is done a routine ultrasound
.

220 Before doing anything else, a chemical equation must always BALANCED!!!
. be ...
221 Beginning warfarin concomitantly with heparin can provide a a) 5
. stable INR by which day of heparin treatment? Explanation:
a) 5 Beginning warfarin concomitantly
b) 2 with heparin can provide a stable INR
c) 3 by day 5 of heparin treatment, at
d) 4 which time the heparin maybe
discontinued.
222 Bending, lifting, and the Valsalva maneuver can precipitate hypertensive crises They increase
. transabdominal pressure and may
cause cardiac-stimulating effects
223 Benign conditions that can increase CEA include smoking, infection, inflammatory
. bowel disease, pancreatitis, cirrhosis
of the liver, and some benign tumors
224 Benzonatate is used for cough associated with respiratory conditions and chronic
. pulmonary diseases.
225 The best intervention to reduce the risk for urosepsis (spread of Which nursing intervention is most
. an infectious agent from the urinary tract to systemic beneficial in reducing the risk of
circulation) is removal of the urinary catheter as quickly as urosepsis in a hospitalized client with
possible (D). (A, B, and C) are helpful to reduce the risk of an indwelling urinary catheter?
infection, but are of less priority than (D) in reducing the risk of A. Ensure that the client's perineal area
urosepsis. is cleansed twice a day.
Correct Answer: D B. Maintain accurate documentation of
the fluid intake and output.
C. Encourage frequent ambulation if
allowed or regular turning if on
bedrest.
D. Obtain a prescription for removal of
the catheter as soon as possible.
226 Beta 1 receptors are responsible for what? speeding up HR
.

227 Beta 2 receptors are responsible for what? Vasodilation/ bronchodilation


.

228 Beta blockers are the what? -olols


.

229 Bethanechol (Urecholine), a cholinergic drug, may be used in GERD to increase lower esophageal
. sphincter pressure and facilitate gastric
emptying.
230 Bile green or cloudy white drainage is not expected during the first 12 to 24
. hours after a subtotal gastrectomy.
231 Bile is created in the liver, stored in the gallbladder, and released brown color. A bile duct obstruction
. into the duodenum giving stool its can cause pale colored stools.
232 Bile is not clear and is not green unless it comes in contact with gastric fluid.
.

233 Biliary drainage tubes (T tubes) are placed in the common bile dark yellow-orange
. duct and drain bile, which is
234 Biofeedback involves the use of various monitoring A client with Raynaud's disease asks the nurse about
. devices that help people become more aware and using biofeedback for self-management of symptoms.
able to control their own physiologic responses, What response is best for the nurse to provide?
such as heart rate, body temperature, muscle A. The responses to biofeedback have not been well
tension, and brain waves. (D) is an accurate established and may be a waste of time and money.
statement concerning its use for clients with B. Biofeedback requires extensive training to retrain
Raynaud's disease. (A, B, and C) do not provide voluntary muscles, not involuntary responses.
correct information about biofeedback. C. Although biofeedback is easily learned, it is
Correct Answer: D mostly often used to manage exacerbation of
symptoms.
D. Biofeedback allows the client to control
involuntary responses to promote peripheral
vasodilation.
235 Bleeding and infection are the major complications AML.
. and causes of death for clients with
236 Bleeding is related to the degree of thrombocytopenia, and infection is related to the
. degree of neutropenia.
237 A bleeding ulcer produces black, tarry stools.
.

238 The blood pressure is very labile with these tachycardia, palpitations, angina, or
. activities, and paroxysms may be accompanied by electrocardiographic changes.
239 A blood pressure of 140/90 mm hg is considered to Hypertension--->A BP of 140/90 mm Hg or higher is
. be hypertension. A blood pressure of less than 120/80
mm Hg is considered normal. A BP of 120 to 129/80
to 89 mm Hg is prehypertension. Hypertensive
emergency is a situation in which blood pressure is
severely elevated and there is evidence of actual or
probable target organ damage.
240 Blood-tinged secretions are common for several bronchoscopy, especially if a biopsy was obtained. A
. hours after respiratory rate of 13 breaths/minute is within normal
limits.
241 blood transfusion signs are The nurse should assess for signs of impending shock
. such as diaphoresis. The client would have
hypotension, dysuria, and cool skin.
242 Bloody diarrhea is indicative of cytomegalovirus infection
.

243 A boggy, tender prostate is found with infection (e.g., acute or chronic prostatitis).
.

244 born at 39 weeks' gestation or later signs Extensive rugae on the scrotum and coarse, silky
. scalp hair are typical findings
245 botulism For breathing trouble, A tube may be inserted through the nose or mouth
. into the windpipe to provide an airway for oxygen.
You may need a breathing machine.
246 Botulism is a rare but serious illness caused by Clostridium botulinum bacteria.
.

247 botulism Patients who have trouble swallowing getintravenous fluids. A feeding tube may be inserted.
. may
248 Bradycardia for the first 7 days in the postpartum normal.
. period is
249 Bradycardia, paralytic ileus, and hot and dry skin spinal shock
. typically occur during
250 Breast milk has been found to heal nipples when nipple at the completion of a feeding.
. placed on the
251 Broccoli and brussels sprouts are good sources of ascorbic acid (vitamin C).
.

252 Bronchial breath sounds are loud, high-pitched sounds normally heard next to the
. trachea; discontinuous, they're loudest during
expiration.
253 BRONCHODILATORS Levalbuterol (Xopenex, Xopenex HFA)
. BETA2-ADRENERGIC Oral Beta2-Adrenergic Agonists
AGONISTS Albuterol (generic) tablets or syrup
(Inhaled Short-Acting Agents) Albuterol (VoSpire ER)
Prototype: Albuterol (Proventil, Terbutaline (Brethine)
Ventolin, Accuneb) (Inhaled beta2 agents are preferred)
254 bronchopulmonary dysplasia prolonged hospitalization and permanent assisted ventilation.
. BPD is a chronic illness that may
require
255 bronchopulmonary dysplasia compromised very-low-birth-weight neonates who require oxygen
. typically occurs in therapy and assisted ventilation for treatment of respiratory distress
syndrome.
256 Bronchovesicular breath sounds medium-pitched, continuous sounds that occur during inhalation or
. are exhalation.
257 bucks
.

258 bucks traction the client should turn his body to another position because the bandage may slip.
. not
259 buerger disease Signs and claudication, cyanosis, coldness, and pain at rest.
. symptoms include slowly
developing
260 Buerger's disease is inflammation and fibrosis of arteries, veins, and nerves. White blood
. characterized by cells infiltrate the area and become fibrotic, which results in occlusion of
the vessels.
261 Bullae are elevated, fluid-filled lesions greater than 0.5 cm in diameter; an example
. is a 0.5 blister.
262 burns Adherence to standard mask, eye goggles, gown, and gloves to prevent contamination from the
. precautions requires the nurse to irrigation.
wear a
263 burns Hemoconcentration, not circulatory dehydration as plasma shifts into the extracellular space.
. hemodilution, is caused by
264 The burn should be kept moist to prevent the dressing adhering to the wound.
.

265 The burn should be kept moist to dressing adhering to the wound. Warm, mild soap solutions would be
. prevent the contraindicated because they are irritating to the injured tissue.
266 burns Metabolic acidosis, not alkalosis, commonly develops due to loss of bicarbonate ions.
.

267 burns sodium levels are Hyponatremia because sodium is trapped in edematous fluid.
.

268 burns The irrigation is not painful and sedatives or pain medications are not usually necessary.
.

269 By 4 months, the neonate should turn his eyes and head toward a sound coming from behind
.

270 By day 2 of hospitalization after chest pain. Severe chest pain should not be present on day 2 after and
. an MI, clients are expected to be MI. Day 2 of hospitalization may be too soon for clients to be able to
able to perform personal care identify risk factors for MI or to begin a walking program; however, the
without client may be sitting up in a chair as part of the cardiac rehabilitation
program
271 calculi can form with low fluid intake.
.

272 calculi can form with repeated urinary tract infections, high doses of vitamin C or D,
. immobility, and large doses of calcium.
273 Carcinoembryonic antigen (CEA) is a protein found in many types of
. cells but associated with tumors
and the developing fetus.
274 Cardiac complications, which may occur following resection of an Vagus
. esophageal tumor, are associated with irritation of which nerve at
the time of surgery?
275 Caring for client infected with vancomycin-resistant enterococci contact precautions.
. requires
276 A cassette pump (B) should be used to accurately deliver large The nurse is preparing to give a
. volumes of fluid over longer periods of time with extreme precise, client with dehydration IV fluids
such as ml/hour. A syringe pump (A) is accurate for low-dose delivered at a continuous rate of
continuous infusion of low-dose medication at a basal rate, but not 175 ml/hour. Which infusion
large fluid volume replacement. Volumetric (C) and nonvolumetric device should the nurse use?
(D) controllers count drops/minute to administer fluid volume and A. Portable syringe pump.
are inherently inaccurate because of variation in drop size. B. Cassette infusion pump.
Correct Answer: B C. Volumetric controller.
D. Nonvolumetric controller.
277 Castor oil can initiate premature uterine contractions and
. other adverse effects in pregnant
women.
278 Catheterization isn't routine done in the 4th stage of deliver to
. protect the bladder from trauma.
It's done, for a postpartum
complication of urinary retention.
279 The catheter used for gavage feeding a neonate should be lubricated sterile water before introduction so
. with that if the catheter is inadvertently
introduced into the lungs, serious
damage would not occur.
280 cause for oral cancer Chronic and excessive use of
. alcohol can lead to oral cancer.
Smoking and use of smokeless
tobacco are other significant risk
factors
281 The CEA blood test is not reliable for diagnosing cancer or as a
. screening test for early detection of
cancer
282 CEA is tested in blood. The normal range is <2.5 ng/ml in an adult non-
. smoker and <5.0 ng/ml in a
smoker
283 CEA may be elevated in colorectal cancer but isn't
. considered a confirming test
284 A central venous catheter has been inserted via a jugular vein and a Administer a bolus of normal
. radiograph has confirmed placement of the catheter. A prescription saline solution
has been received for a stat medication, but IV fluids have not yet
been started. What action should the nurse take prior to
administering the prescribed medication?
285 The chancre of syphilis is characteristically a painless, moist ulcer.
.

286 Chancres often disappear even without treatment.


.

287 check for lice behind the ear


.

288 Chest Trauma 1. Blunt or Penetrating


. 1. ________or_____________trauma 2. Sternal, rib
2._____________ fractures 3. Flail chest
3. Flail_______ 4. Pulmonary
4.___________ contusion 5. ***Pneumothorax****
-Spontaneous or simple
5. this can cause it by means of Spontaneous, -Traumatic
simple, -Tension pneumothorax
Traumatic or Tension
289 The chief complications of diltiazem are hypotension, atrioventricular blocks, heart failure, and
. elevated liver enzyme levels. Other reported reactions
include flushing, nocturia, and polyuria, but not renal
failure.
290 chlorothiazide (Diuril). The expected outcome Improved capillary circulation.
. of this drug is:
291 Choice Multiple question - Select all answer Obtains results of a swallow study
. choices that apply. Provides thick liquids
A client is postoperative for a partial Explanation:
laryngectomy following a diagnosed When a client is allowed to eat following a partial
malignancy. The client is to start oral feedings. laryngectomy, a swallow study may be obtained first to
The nurse does the following interventions: determine the client's risk of aspiration. The client is
(Select all that apply.) started with thick liquids because they are easy to
a) Encourages the client to ingest sweet foods swallow. The nurse stays with the client during initial
b) Provides thick liquids feedings to ensure safe ingestion. Solid foods are
c) Orders a regular diet tray introduced as tolerated. The nurse encourages the client
d) Obtains results of a swallow study to avoid sweet foods, which increase salivation and
e) Facilitates privacy while eating suppress appetite.
292 Cholelithiasis is the medical term for gallstone disease.
.

293 Cholinergic adverse effects may include urinary urgency, diarrhea, abdominal cramping,
. hypotension, and increased salivation.
294 A cholinergic blocking agent may cause dry mouth and delay the sublingual absorption of
. nitroglycerin
295 The chordee is corrected when the hypospadias is repaired. Circumcision is performed at the
. same time.
296 Chordee refers to a ventral curvature of the penis that results from a fibrous
. band of tissue that has replaced normal tissue.
297 Chorioamnionitis is an inflammation of the fetal membranes (amnion and
. chorion) due to a bacterial infection.
298 Chorioamnionitis is a serious intrapartum infection that may result in fetal tachycardia and a
. hypotonic labor pattern.
299 Chorioamnionitis the infected amniotic fluid in infection, such as pneumonia, during the neonatal period.
. the fetal lungs may result in a
300 Chronic, excessive acetaminophen use isn't hepatotoxic
. nephrotoxic, although it may be
301 Chronic interstitial fibrosis is associated with adenocarcinoma of the lung
. the development of
302 Circumcision is delayed because the foreskin, reconstruct the urethra.
. which is removed with a circumcision, often is
used to
303 cirrhosis and aspirin Aspirin also should be avoided if esophageal varices are
. present.
304 Cirrhosis Clients are encouraged to eat normal, well-balanced diets and to restrict sodium to
. prevent fluid retention. Protein is not restricted until the
liver actually fails, which is usually late in the disease.
305 (C) is an open-ended question that encourages the client When the nurse enters a client's room to do an
. to discuss personal feelings. (A) devalues the client and initial assessment, the client shouts, "Get out
hinders further communication. Acting defensively and of my room! I'm tired of being bothered!"
asking why questions such as (B) are likely to elicit more How should the nurse respond?
anger and block communication. By deferring to the client A. There is no reason to be so angry.
advocate (D), the nurse fails to even address the client's B. Why do I need to leave your room?
feelings of anger and exasperation. C. What is concerning you this morning?
Correct Answer: C D. Let me call the client advocate for you.
306 The classification of pneumonia as community-acquired Causative agents can be predicted, and
. pneumonia (CAP) or hospital- acquired pneumonia empiric treatment is often effective
(HAP) is clinically useful because
307 A class of beginning nursing students is learning about The heart cannot pump sufficient blood to
. heart failure in their pathophysiology class. What should meet the body's metabolic needs
the students be taught is the reason for heart failure?
308 claudication is pain with walking as seen with peripheral
. arterial disease
309 A client abruptly sits up in bed, reports having difficulty b) Nonrebreather mask
. breathing and has an arterial oxygen saturation of 88%.
Which mode of oxygen delivery would most likely reverse
the manifestations?

a) Simple mask
b) Nonrebreather mask
c) Face tent
d) Nasal cannula
310 A client accidentally splashes chemicals into one eye. The To prevent vision loss
. nurse knows that eye irrigation with plain tap water Correct
should begin immediately and continue for 15 to 20 Explanation:
minutes. What is the primary purpose of this first-aid Prolonged eye irrigation after a chemical burn
treatment? is the most effective way to prevent formation
of permanent scar tissue and thus help prevent
a) To hasten formation of scar tissue vision loss. After a potentially serious eye
b) To eliminate the need for medical care injury, the victim should always seek medical
c) To prevent vision loss care. Eye irrigation isn't considered a stopgap
d) To serve as a stopgap measure until help arrives measure.
311. A client admitted to the facility for treatment for c) "I will stay in isolation for at least 6
tuberculosis receives instructions about the disease. weeks."
Which statement made by the client indicates the need for
further instruction?

a) "I will have to take the medication for up to a year."


b) "This disease may come back later if I am under
stress."
c) "I will stay in isolation for at least 6 weeks."
d) "I will always have a positive test for tuberculosis."
312 A client admitted with acute anxiety has the following d) respiratory alkalosis.
. arterial blood gas (ABG) values: pH, 7.55; partial
pressure of arterial oxygen (PaO2), 90 mm Hg; partial
pressure of arterial carbon dioxide (PaCO2), 27 mm Hg;
and bicarbonate (HCO3-), 24 mEq/L. Based on these
values, the nurse suspects:

a) metabolic acidosis.
b) metabolic alkalosis.
c) respiratory acidosis.
d) respiratory alkalosis.
313 Client census is often used to determine staffing needs. Average daily census
. Which method of obtaining census determination for a
particular unit provides the best formula for determining
long-range staffing patterns?
314 A client comes to the Emergency Department Coronary artery disease
. (ED) complaining of precordial chest pain. In
describing the pain, the client describes it as
pressure with a sudden onset. What disease
process would you suspect in this client?
315 A client complains of leg pain brought on by c) "Practice meticulous foot care."
. walking several blocks a symptom that first Explanation:
arose several weeks earlier. The client's history Intermittent claudication and other chronic peripheral
includes diabetes mellitus and a two-pack-per-day vascular diseases reduce oxygenation to the feet,
cigarette habit for the past 42 years. The physician making them susceptible to injury and poor healing.
diagnoses intermittent claudication and orders Therefore, meticulous foot care is essential. The nurse
pentoxifylline (Trental), 400 mg three times daily should teach the client to bathe his feet in warm water
with meals. Which instruction concerning long- and dry them thoroughly, cut the toenails straight
term care should the nurse provide? across, wear well-fitting shoes, and avoid taking
a) "Reduce your level of exercise." medications without the approval of a physician.
b) "See the physician if complications occur." Because nicotine is a vasoconstrictor, this client
c) "Practice meticulous foot care." should stop smoking, not just consider cutting down.
d) "Consider cutting down on your smoking." Daily walking is beneficial to clients with intermittent
claudication. To evaluate the effectiveness of the
therapeutic regimen, this client should see the
physician regularly, not just when complications
occur.
316 A client complains of leg pain brought on by b) "Practice meticulous foot care."
. walking several blocks a symptom that first Explanation:
arose several weeks earlier. The client's history Intermittent claudication and other chronic peripheral
includes diabetes mellitus and a two-pack-per-day vascular diseases reduce oxygenation to the feet,
cigarette habit for the past 42 years. The physician making them susceptible to injury and poor healing.
diagnoses intermittent claudication and orders Therefore, meticulous foot care is essential. The nurse
pentoxifylline (Trental), 400 mg three times daily should teach the client to bathe his feet in warm water
with meals. Which instruction concerning long- and dry them thoroughly, cut the toenails straight
term care should the nurse provide? across, wear well-fitting shoes, and avoid taking
a) "See the physician if complications occur." medications without the approval of a physician.
b) "Practice meticulous foot care." Because nicotine is a vasoconstrictor, this client
c) "Consider cutting down on your smoking." should stop smoking, not just consider cutting down.
d) "Reduce your level of exercise." Daily walking is beneficial to clients with intermittent
claudication. To evaluate the effectiveness of the
therapeutic regimen, this client should see the
physician regularly, not just when complications
occur.
317 A client diagnosed with angina pectoris complains Support the client to a sitting position.
. of chest pain while ambulating in the hallway.
Which action should the nurse implement first?
318 A client diagnosed with chronic kidney disease hypocalcemia
. (CKD) 2 years ago is regularly treated at a
community hemodialysis facility. In assessing the
client before his scheduled dialysis treatment,
which electrolyte imbalance should the nurse
anticipate?
319 A client has a blockage in the proximal portion of Anticoagulant
. a coronary artery. After learning about treatment
options, the client decides to undergo
percutaneous transluminal coronary angioplasty
(PTCA). During this procedure, the nurse expects
to administer an:
320 A client has a blockage in the proximal a) anticoagulant.
. portion of a coronary artery. After learning Explanation:
about treatment options, the client decides to During PTCA, the client receives heparin, an anticoagulant,
undergo percutaneous transluminal as well as calcium agonists, nitrates, or both, to reduce
coronary angioplasty (PTCA). During this coronary artery spasm. Nurses don't routinely give
procedure, the nurse expects to administer antibiotics during this procedure; however, because the
an: procedure is invasive, the client may receive prophylactic
a) anticonvulsant. antibiotics to reduce the risk of infection. An
b) antihypertensive. antihypertensive may cause hypotension, which should be
c) anticoagulant. avoided during the procedure. An anticonvulsant isn't
d) antibiotic. indicated because this procedure doesn't increase the risk of
seizures.
321 A client has acute angle-closure glaucoma. 1 to 2 days
. The family is concerned that the client may Explanation:
lose sight. The nurse advises that the window Acute angle-closure glaucoma is an emergency. The nurse
in which this problem can progress to refers the client for medical treatment immediately because
permanent sight loss is: vision can be permanently lost in 1 to 2 days.

a) 1 to 2 weeks
b) 1 to 2 months
c) 1 to 2 hours
d) 1 to 2 days
322 A client has been diagnosed with peripheral Participate in regular walking program-->Clients diagnosed
. arterial occlusive disease. Which of the with peripheral arterial occlusive disease should be
following instructions is appropriate for the encouraged to participate in a regular walking program to
nurse to give the client for promoting help develop collateral circulation. They should be advised
circulation to the extremities? to rest if pain develops and to resume activity when pain
subsides. Extremities should be kept in a dependent position
to promote circulation; elevation of the extremities will
decrease circulation. Heating pads should not be used by
anyone with impaired circulation to avoid burns. Massaging
the calf muscles will not decrease pain. Intermittent
claudication subsides with rest.
323 A client has been diagnosed with peripheral a) Participate in a regular walking program.
. arterial occlusive disease. Which of the Explanation:
following instructions is appropriate for the Clients diagnosed with peripheral arterial occlusive disease
nurse to give the client for promoting should be encouraged to participate in a regular walking
circulation to the extremities? program to help develop collateral circulation. They should
a) Participate in a regular walking program. be advised to rest if pain develops and to resume activity
b) Massage the calf muscles if pain occurs. when pain subsides. Extremities should be kept in a
c) Use a heating pad to promote warmth. dependent position to promote circulation; elevation of the
d) Keep the extremities elevated slightly. extremities will decrease circulation. Heating pads should
not be used by anyone with impaired circulation to avoid
burns. Massaging the calf muscles will not decrease pain.
Intermittent claudication subsides with rest.
324 A client has been hospitalized for a) The client has a partial pressure of arterial oxygen (PaO2)
. treatment of acute bacterial pneumonia. value of 90 mm Hg or higher.
Which outcome indicates an
improvement in the client's condition?

a) The client has a partial pressure of


arterial oxygen (PaO2) value of 90 mm
Hg or higher.
b) The client has a partial pressure of
arterial carbon dioxide (PaCO2) value of
65 mm Hg or higher.
c) The client exhibits restlessness and
confusion.
d) The client exhibits bronchial breath
sounds over the affected area.
325 A client has hypoxemia of pulmonary a) Partial pressure of arterial oxygen (PaO2)
. origin. What portion of arterial blood
gas results is most useful in
distinguishing between acute respiratory
distress syndrome and acute respiratory
failure?

a) Partial pressure of arterial oxygen


(PaO2)
b) Partial pressure of arterial carbon
dioxide (PaCO2)
c) pH
d) Bicarbonate (HCO3-)
326 A client has just been diagnosed with demonstrate eyedrop instillation.
. early glaucoma. During a teaching Explanation:
session, the nurse should: Eyedrop instillation is a critical component of self-care for a
client with glaucoma. After demonstrating eyedrop instillation to
the client and family, the nurse should verify their ability to
perform this measure properly. An eye patch isn't necessary
unless the client has undergone surgery. Visual acuity
assessment isn't necessary before discharge. Intraocular lenses
aren't implanted in clients with glaucoma.
327 A client has the following arterial blood d) Metabolic acidosis
. gas (ABG) values: pH, 7.12; partial
pressure of arterial carbon dioxide
(PaCO2), 40 mm Hg; and bicarbonate
(HCO3-), 15 mEq/L. These ABG values
suggest which disorder?

a) Respiratory alkalosis
b) Respiratory acidosis
c) Metabolic alkalosis
d) Metabolic acidosis
328 A client hospitalized for treatment of a d) Light-headedness or paresthesia
. pulmonary embolism develops
respiratory alkalosis. Which clinical
findings commonly accompany
respiratory alkalosis?

a) Nausea or vomiting
b) Abdominal pain or diarrhea
c) Hallucinations or tinnitus
d) Light-headedness or paresthesia
329 A client in the emergency department Administer oxygen, attach a cardiac monitor, take vital
. complains of squeezing substernal pain that signs, and administer sublingual nitroglycerin.---->Cardiac
radiates to the left shoulder and jaw. He also chest pain is caused by myocardial ischemia. Therefore the
complains of nausea, diaphoresis, and nurse should administer supplemental oxygen to increase
shortness of breath. What should the nurse the myocardial oxygen supply, attach a cardiac monitor to
do? help detect life-threatening arrhythmias, and take vital signs
to ensure that the client isn't hypotensive before giving
sublingual nitroglycerin for chest pain. Registration
information may be delayed until the client is stabilized.
Alerting the cardiac catheterization team or the physician
before completing the initial assessment is premature
330 A client in the emergency department is Maintain the client in a supine position to reduce
. bleeding profusely from a gunshot wound to diaphragmatic pressure and visualize the wound
the abdomen. In what position should the
nurse immediately place the client to promote
maintenance of the client's blood pressure
above a systolic pressure of 90 mm Hg?
331 A client is admitted for a revascularization Keeping the involved extremity at or below the body's
. procedure for arteriosclerosis in his left iliac horizontal plane will facilitate tissue perfusion and prevent
artery. To promote circulation in the tissue damage.
extremities, the nurse should
332 A client is admitted for suspected GI disease. Cirrhosis
. Assessment data reveal muscle wasting, a
decrease in chest and axillary hair, and
increased bleeding tendency. The nurse
suspects the client has:
333 A client is admitted to the hospital with a Place the client on NPO status.
. diagnosis of severe acute diverticulitis. Which
nursing intervention has the highest priority
334 A client is admitted with acute osteomyelitis Administering large doses of IV antibiotics as ordered
. that developed after an open fracture of the
right femur. When planning this client's care,
the nurse should anticipate which measure?
335 A client is admitted with a diagnosis of Chronic hypoxemia creates the urge to breathe in COPD.
. chronic obstructive pulmonary disease
(COPD). What is the nurse's rationale for
keeping the client's oxygen administration
level at 3 L/min or less?
336 A client is being assessed for his semiannual Pulmonary congestion--> crackles heard in the bases of the
. examination and you hear crackles bilaterally lungs are signs of pulmonary congestion
in his lungs. Which of the following could be
a cause of crackles in the bases of his lungs
337 A client is being discharged following Follow exposure precautions.
. radioactive seed implantation for prostate
cancer. What is the most important
information for the nurse to provide this
client's family?
338 A client is being treated for glaucoma. What is the rationale Straining at stool increases intraocular
. for the nurse's instruction to maintain regular bowel habits? pressure.
Correct
a) Straining at stool increases intraocular pressure. Explanation:
b) The client's medications may lead to diarrhea. The client should maintain regular bowel
c) The client's medications may cause constriction of all habits because straining at stool can raise
blood vessels, contributing to hemorrhoids. intraocular pressure (IOP). The other
d) Problems with constipation may compound problems answers are distracters for this question.
with lens clarity.
339 A client is chronically short of breath and yet has normal possible hematologic problem.
. lung ventilation, clear lungs, and an arterial oxygen
saturation SaO2 of 96% or better. The client most likely
has:
340 A client is experiencing dryness in the nares while receiving Lubricant jelly
. oxygen via nasal cannula at 4 L/minute. Which medication
should the nurse apply to help alleviate the dryness?
341 A client is hospitalized for repair of an abdominal aortic Constant, intense back pain and falling BP
. aneurysm. The nurse must be alert for signs and symptoms
of aneurysm rupture and thus looks for which of the
following?
342 A client is hospitalized for repair of an abdominal aortic b) Constant, intense back pain and falling
. aneurysm. The nurse must be alert for signs and symptoms blood pressure
of aneurysm rupture and thus looks for which of the Explanation:
following? Indications of a rupturing abdominal aortic
a) Slow heart rate and high blood pressure aneurysm include constant, intense back
b) Constant, intense back pain and falling blood pressure pain; falling blood pressure; and decreasing
c) Constant, intense headache and falling blood pressure hematocrit.
d) Higher than normal blood pressure and falling
hematocrit
343 The client is in the taking hold phase focus on the neonate and learning about and
. fulfilling infant care and needs
344 A client is placed on a mechanical ventilator following a Impaired communication related to
. cerebral hemorrhage, and vecuronium bromide (Norcuron) paralysis of skeletal muscles
0.04 mg/kg every 12 hours IV is prescribed. What is the
priority nursing diagnosis for this client?
345 A client is ready for discharge following creation of an Change the bag when the seal is broken.
. ileostomy. Which instruction should the nurse include in
discharge teaching?
346 A client is receiving an I.V. infusion of mannitol (Osmitrol) Increased urine output
. after undergoing intracranial surgery to remove a brain
tumor. To determine whether this drug is producing its
therapeutic effect, the nurse should consider which finding
most significant?
347 A client is receiving conscious sedation while undergoing c) Oxygen saturation of 90%
. bronchoscopy. Which assessment finding should receive the
nurse's immediate attention?

a) Absent cough and gag reflexes


b) Blood-tinged secretions
c) Oxygen saturation of 90%
d) Respiratory rate of 13 breaths/min
348 A client is receiving long-term treatment with Moon face---->Clients who are receiving long-term
. high-dose corticosteroids. Which of the following high-dose corticosteroid therapy often develop a
would the nurse expect the client to exhibit? cushingoid appearance, manifested by facial fullness
and the characteristic moon face. They also may
exhibit weight gain, peripheral edema, and
hypertension due to sodium and water retention. The
skin is usually thin, and ruddy.
349 A client is receiving moderate sedation while Oxygen saturation of 90%
. undergoing bronchoscopy. Which assessment
finding should the nurse attend to immediately?
350 A client is receiving supplemental oxygen. When c) Partial pressure of arterial oxygen (PaO2)
. determining the effectiveness of oxygen therapy,
which arterial blood gas value is most important?

a) pH
b) Bicarbonate (HCO3-)
c) Partial pressure of arterial oxygen (PaO2)
d) Partial pressure of arterial carbon dioxide
(PaCO2)
351 A client is recovering from an ileostomy that was Increasing fluid intake to prevent dehydration
. performed to treat inflammatory bowel disease.
During discharge teaching, the nurse should stress
the importance of:
352 A client is recovering from gastric surgery. Six small meals daily with 120 ml fluid between
. Toward what goal should the nurse progress the meals
client's enteral intake?
353 A client is returned to his room after a subtotal Tracheostomy set
. thyroidectomy. Which piece of equipment is most
important for the nurse to keep at the client's
bedside?
354 A client is undergoing a complete physical d) chest movements
. examination as a requirement for college. When
checking the client's respiratory status, the nurse
observes respiratory excursion to help assess:

a) lung vibrations
b) vocal sounds
c) breath sounds
d) chest movements
355 A client on an acid-ash diet must avoid milk and milk products because these make the urine
. more alkaline, encouraging bacterial growth.
356 A client reports nasal congestion, sneezing, sore Ineffective airway clearance related to excess mucus
. throat, and coughing up of yellow mucus. The production
nurse assesses the client's temperature as 100.2F. Explanation:
The client states this is the third episode this All the listed nursing diagnoses are appropriate for
season. The highest priority nursing diagnosis is this client. Following Maslow's hierarchy of needs,
a) Acute pain related to upper airway irritation physiological needs are addressed first and, within
b) Deficient fluid volume related to increased fluid physiological needs, airway, breathing, and circulation
needs are the most immediate. Thus, ineffective airway
c) Deficient knowledge related to prevention of clearance is the priority nursing diagnosis.
upper respiratory infections
d) Ineffective airway clearance related to excess
mucus production
357 A client reports to the clinic, stating that she Acute gastritis
. rapidly developed headache, abdominal pain,
nausea, hiccuping, and fatigue about 2 hours ago.
For dinner, she ate buffalo chicken wings and
beer. Which of the following medical conditions is
most consistent with the client's presenting
problems?
358 A client's dietary habits should be determined first by the client's A 73-year-old Hispanic client is
. dietary recall (B) before suggesting protein sources or supplements seen at the community health clinic
(A and C) as options in the client's diet. Although grains and with a history of protein
legumes (D) contain incomplete proteins that reduces the essential malnutrition. What information
amino acid pools inside the cells, the client's cultural preferences should the nurse obtain first?
should be illicited after confirming the client's dietary history. A. Amount of liquid protein
Correct Answer: B supplements consumed daily.
B. Foods and liquids consumed
during the past 24 hours.
C. Usual weekly intake of milk
products and red meats.
D. Grains and legume combinations
used by the client.
359 The client's highest brachial systolic pressure is divided by the left ankle systolic blood pressure to
. get 0.81.
360 The client's highest brachial systolic pressure is divided by the left ankle systolic blood pressure to
. get 0.81
361 The client should receiv normal saline solution through the
. second I.V. site until his blood
glucose level reaches 250 mg/dl.
362 The client should receive a fluid bolus of 500 ml of normal saline solution.
.

363 The client should review the adcance directive with the physician at reversible and temporary, making
. every admission because some conditions may be portions of the advance directive
inappropriate.
364 Clients taking metoclopramide should be instructed to report any involuntary movements of the face,
. eyes, or extremities because adverse
effects of the drug include
extrapyramidal reactions and
parkinsonism-like reactions.
365 A client suspected of having colorectal cancer requires which Sigmoidoscopy
. diagnostic study to confirm the diagnosis?
366 Clients who take only one daily dose of ranitidine are usually at bedtime to inhibit nocturnal
. advised to take it secretion of acid.
367 Clients who take the Ranitidine twice a day are advised to take it in morning and at bedtime.
. the
368 Clients with acute necrotizing pancreatitis should remain \ NPO with early enteral feeding via
. the jejunum to maintain bowel
integrity and immune function. TPN
is considered if enteral feedings are
contraindicated. \
369 Clients with acute necrotizing pancreatitis should remain NPO with early enteral feeding via
. the jejunum to maintain bowel
integrity and immune function.
370 Clients with autoimmune disorders may have either false-positive or false-negative
. serologic tests for syphilis.
371 Clients with cirrhosis should not take acetaminophen (Tylenol), which is
. potentially hepatotoxic.
372 Clients with diverticulosis are encouraged to follow a high-fiber diet. Bran, broccoli, and
. navy beans are foods high in fiber.
373 Clients with stress incontinence are encouraged to avoid caffeine and alcohol, that are
. substances, such a bladder irritants.
374 The client taking dexamethasone needs to know easy bruising, moonface, buffalo hump, and
. the early signs of Cushing's disease, which include osteoporosis.
375 The client was not properly informed of the A signed consent form indicated a client should have
. procedure, and failure to obtain informed consent an electromyogram, but a myelogram was performed
constitutes assault and battery (C). (A) is injury to instead. Though the myelogram revealed the cause of
economics and dignity, such as invasion of privacy the client's back pain, which was subsequently treated,
or defamation of character. This is not an incident the client filed a lawsuit against the nurse and
of failure to respect the client's autonomy (B). An healthcare provider for performing the incorrect
unintentional tort (D) is an act in which the procedure. The court is likely to rule in favor of the
outcome was not expected, such as negligence or plaintiff because these events represent what
malpractice. infraction?
Correct Answer: C A. A quasi-intentional tort because a similar mistake
can happen to anyone.
B. Failure to respect client autonomy to choose based
on intentional tort law.
C. Assault and battery with deliberate intent to deviate
from the consent form.
D. An unintentional tort because the client benefited
from having the myelogram.
376 The client who cannot assign meaning to sound central hearing loss.
. has
377 A client who can stand can safely be assisted to An older client who is able to stand but not to
. pivot and transfer with the use of a transfer belt ambulate receives a prescription to be mobilized into
(D). A mechanical lift (A) is usually used for a a chair as tolerated during each day. What is the best
client who is obese, unable to be weight-bearing, action for the nurse to implement when assisting the
and who is unable to assist. Roller boards (B) client from the bed to the chair?
placed under a sheet are used to facilitate the A. Use a mechanical lift to transfer from the bed to a
transfer of a recumbent client who is being chair.
transferred to and from a stretcher. Lifting a client B. Place a roller board under the client who is sitting
(C) out of bed places the client and nurses at risk on the side of the bed and slide the client to the chair.
for injury and should only be implemented by C. Lift the client out of bed to the chair with another
skilled lift teams. staff member using a coordinated effort on the count
Correct Answer: D of three.
D. Place a transfer belt around the client, assist to
stand, and pivot to a chair that is placed at a right
angle to the bed.
378 A client who is being monitored with telemetry has Perform synchronized cardioversion
. a pattern of uncontrolled atrial fibrillation with a
rapid ventricular response. Based on this finding,
the nurse anticipates administering which
treatment?
379 A client who is free from infection will most likely decreased oxygen requirements versus normal
. have temperature
380 A client who is receiving an ACE inhibitor for Encourage the client to keep taking the drug until seen
. hypertension calls the clinic and reports the recent by the healthcare provider.
onset of a cough to the nurse. What action should
the nurse implement?
381 A client who suffered a stroke has a nursing Repositioning the client every 2 hours helps prevent
. diagnosis of Ineffective airway clearance. The goal secretions from pooling in dependent lung areas.
of care for this client is to mobilize pulmonary Restricting fluids would make secretions thicker and
secretions. Which intervention helps meet this more tenacious, thereby hindering their removal.
goal? Administering oxygen and keeping the head of the
bed at a 30-degree angle might ease respirations and
make them more effective but wouldn't help mobilize
secretions.
382 A client who sustained a pulmonary contusion in a c) Ineffective breathing pattern related to tissue
. motor vehicle accident develops a pulmonary trauma
embolism. Which nursing diagnosis takes priority
with this client?

a) Excess fluid volume related to excess sodium


intake
b) Acute pain related to tissue trauma
c) Ineffective breathing pattern related to tissue
trauma
d) Activity intolerance related to insufficient energy
to carry out activities of daily living
383 A client who underwent total hip replacement Homan's---->A positive Homans' sign, or pain in the
. exhibits a red, painful area on the calf of the calf elicited upon flexion of the ankle with the leg
affected leg. What test validates presence of straight, indicates the presence of a thrombus.
thromboembolism? Testing for Romberg's sign assesses cerebellar
function. Phalen's test assesses carpal tunnel
syndrome. The Rinne test compares air and bone
conduction in both ears to screen for or confirm
hearing loss.
384 A client who underwent total hip replacement a) Homans'
. exhibits a red, painful area on the calf of the Explanation:
affected leg. What test validates presence of A positive Homans' sign, or pain in the calf elicited
thromboembolism? upon flexion of the ankle with the leg straight,
a) Homans' indicates the presence of a thrombus. Testing for
b) Rinne Romberg's sign assesses cerebellar function. Phalen's
c) Romberg's test assesses carpal tunnel syndrome. The Rinne test
d) Phalen's compares air and bone conduction in both ears to
screen for or confirm hearing loss.
385 The client will probably not be able to tolerate a full laparoscopic cholecystectomy surgery.
. meal comfortably the day after
386 The client with acute pancreatitis is prone to respiratory system. Pneumonia, atelectasis, and
. complications associated with the pleural effusion are examples of respiratory
complications that can develop as a result of
pancreatic enzyme exudate.
387 A client with a gastrojejunostomy is beginning to Diarrhea
. take solid food. Which finding would lead the nurse
to suspect that the client is experiencing dumping
syndrome?
388 A client with a history of alcohol abuse comes to the Administering morphine IV as ordered
. emergency department and complains of
abdominal pain. Laboratory studies help confirm a
diagnosis of acute pancreatitis. The client's vital
signs are stable, but the client's pain is worsening
and radiating to his back. Which intervention takes
priority for this client?
389 A client with alcohol-related liver disease is Pentobarbital (Nembutal sodium) 50 mg at bedtime
. admitted to the unit. Which prescription should the for rest
nurse question as possibly inappropriate for this
client?
390 A client with a pulmonary embolus has the b) Administer oxygen by nasal cannula as
. following arterial blood gas (ABG) values: pH, prescribed.
7.49; partial pressure of arterial oxygen (PaO2), 60
mm Hg; partial pressure of arterial carbon dioxide
(PaCO2), 30 mm Hg; bicarbonate (HCO3-) 25
mEq/L. What should the nurse do first?

a) Instruct the client to breathe into a paper bag.


b) Administer oxygen by nasal cannula as
prescribed.
c) Auscultate breath sounds bilaterally every 4
hours.
d) Encourage the client to deep-breathe and cough
every 2 hours.
391 The client with a Sengstaken-Blakemore tube swallow.
. cannot
392 A client with a suspected pulmonary disorder undergoes pulmonary c) Tidal volume
. function tests. To interpret test results accurately, the nurse must be
familiar with the terminology used to describe pulmonary functions.
Which term refers to the volume of air inhaled or exhaled during each
respiratory cycle?

a) Vital capacity
b) Functional residual capacity
c) Tidal volume
d) Maximal voluntary ventilation
393 A client with chronic asthma is admitted to postanesthesia complaining of Call the anesthesia provider
. pain at a level of 8 of 10, with a blood pressure of 124/78, pulse of 88 for a different medication for
beats/min, and respirations of 20 breaths/min. The postanesthesia pain.
recovery prescription is, "Morphine 2 to 4 mg IV push while in recovery
for pain level over 5." What intervention should the nurse implement?
394 The client with chronic bronchitis should exhale through pursed lips to prolong
. exhalation, keep the
bronchioles from collapsing,
and prevent air trapping
395 A client with chronic obstructive pulmonary disease (COPD) is being b) Dyspnea on exertion
. evaluated for a lung transplant. The nurse performs the initial physical c) Barrel chest
assessment. Which signs and symptoms should the nurse expect to find? e) Clubbed fingers and toes

a) Decreased respiratory rate


b) Dyspnea on exertion
c) Barrel chest
d) Shortened expiratory phase
e) Clubbed fingers and toes
f) Fever
396 A client with chronic obstructive pulmonary disease (COPD) is recovering c) atelectasis.
. from a myocardial infarction. Because the client is extremely weak and
can't produce an effective cough, the nurse should monitor closely for:
a) pleural effusion.
b) pulmonary edema.
c) atelectasis.
d) oxygen toxicity.
397 A client with chronic pain is more likely to have adapted physiologically To obtain the most complete
. to vital sign changes, localization or intensity, so pain assessment should assessment data for a client
focus on any interference with daily activities (D), sleep, relationships with chronic pain, which
with others, physical activity, and emotional well-being. Exacerbation of information should the nurse
acute symptoms, such as pain distribution, patterns, intensity, and obtain?
descriptors illicit specific assessment findings, whereas (A, B, and C) are A. Can you describe where
limiting, closed-end questions, and can be answered with a yes, no, or a your pain is the most severe?
number. B. What is your pain
Correct Answer: D intensity on a scale of 1 to
10?
C. Is your pain best described
as aching, throbbing, or
sharp?
D. Which activities during a
routine day are impacted by
your pain?
398 A client with cirrhosis develops increasing pedal edema and ascites. What restrict salts and fluid intakes
. dietary modification is most important for the nurse to teach this client?
399 A client with cirrhosis states that his disease was The year the blood transfusion was received
. caused by a blood transfusion. What information
should the nurse obtain first to provide effective
client teaching?
400 A client with congestive heart failure and atrial Increase oxygen flow via nasal cannula.
. fibrillation develops ventricular ectopy with a pattern
of 8 ectopic beats/min. What action should the nurse
take based on this observation?
401 The client with conjunctivitis can use warm soaks to crusting
. remove
402 A client with COPD is at high risk for development of respiratory infections
.

403 A client with COPD typically has a barrel chest in anteroposterior diameter is larger than the
. which the transverse chest diameter
404 A client with diabetes mellitus is receiving an oral Signs of hypoglycemia
. antidiabetic agent. Which of the following aspects
should the nurse observe when caring for the client?
405 A client with diabetic ketoacidosis was admitted to Change the second I.V. solution to dextrose 5% in
. the intensive care unit 4 hours ago and has these water.
laboratory results: blood glucose level 450 mg/dl,
serum potassium level 2.5 mEq/L, serum sodium level
140 mEq/L, and urine specific gravity 1.025. The
client has two I.V. lines in place with normal saline
solution infusing through both. Over the past 4 hours,
his total urine output has been 50 ml. Which
physician order should the nurse question?
406 A client with genital herpes lesions should avoid all sexual contact to prevent spreading the disease.
.

407 A client with Guillain-Barr syndrome develops d) pH, 7.25; PaCO2 50 mm Hg


. respiratory acidosis as a result of reduced alveolar
ventilation. Which combination of arterial blood gas
(ABG) values confirms respiratory acidosis?

a) pH, 7.5; PaCO2 30 mm Hg


b) pH, 7.40; PaCO2 35 mm Hg
c) pH, 7.35; PaCO2 40 mm Hg
d) pH, 7.25; PaCO2 50 mm Hg
408 A client with hyperparathyroidism declines surgery "Maintain a moderate exercise program."--->The
. and is to receive hormone replacement therapy with nurse should instruct the client to maintain a
estrogen and progesterone. Which instruction is most moderate exercise program. Such a program helps
important for the nurse to include in the client's strengthen bones and prevents the bone loss that
teaching plan? occurs from excess parathyroid hormone. Walking
or swimming provides the most beneficial
exercise. Because of weakened bones, a rigorous
exercise program such as jogging is
contraindicated. Weight loss might be beneficial
but it isn't as important as developing a moderate
exercise program.
409 A client with hypertension has been receiving Administer the dose as prescribed.
. ramipril (Altace) 5 mg PO daily for 2 weeks and is
scheduled to receive a dose at 0900. At 0830 the
client's blood pressure is 120/70. What action should
the nurse take?
410 A client with jaundice has pruritis What can the Take baking soda baths.
. nurse discuss to prevent skin breakdown? Keep nails short and clean.
Rub with knuckles instead of nails.
411. A client with long-standing type 1 diabetes is Deficient knowledge----> The client should inject
admitted to the hospital with unstable angina insulin before, not after, breakfast and dinner 30
pectoris. After the client's condition stabilizes, the minutes before breakfast for the a.m. dose and 30
nurse evaluates the diabetes management regimen. minutes before dinner for the p.m. dose. Therefore,
The nurse learns that the client sees the physician the client has a knowledge deficit regarding when to
every 4 weeks, injects insulin after breakfast and administer insulin. By taking insulin, measuring
dinner, and measures blood glucose before blood glucose levels, and seeing the physician
breakfast and at bedtime. Consequently, the nurse regularly, the client has demonstrated the ability and
should formulate a nursing diagnosis of: willingness to modify his lifestyle as needed to
manage the disease. This behavior eliminates the
nursing diagnoses of Impaired adjustment and
Defensive coping. Because the nurse, not the client,
questioned the client's health practices related to
diabetes management, the nursing diagnosis of
Health-seeking behaviors isn't warranted.
412 A client with metabolic alkalosis may exhibit irritability or nervousness.
.

413 A client with myasthenia gravis is receiving b) Suction the client's artificial airway.
. continuous mechanical ventilation. When the high-
pressure alarm on the ventilator sounds, what
should the nurse do?

a) Check for an apical pulse.


b) Suction the client's artificial airway.
c) Increase the oxygen percentage.
d) Ventilate the client with a handheld mechanical
ventilator.
414 A client with no known history of peripheral c) keep the affected leg level or slightly dependent.
. vascular disease comes to the emergency Explanation:
department complaining of sudden onset of lower While the physician makes treatment decisions, the
leg pain. Inspection and palpation reveal absent nurse should maintain the client on bed rest, keeping
pulses; paresthesia; and a mottled, cyanotic, cold, the affected leg level or slightly dependent (to aid
and cadaverous left calf. While the physician circulation) and protecting it from pressure and other
determines the appropriate therapy, the nurse trauma. Warming the leg with a heating pad (or
should: chilling it with an ice pack) would further
a) elevate the affected leg as high as possible. compromise tissue perfusion and increase injury to
b) place a heating pad around the affected calf. the leg. Elevating the leg would worsen tissue
c) keep the affected leg level or slightly dependent. ischemia. Shaving an ischemic leg could cause
d) shave the affected leg in anticipation of surgery. accidental trauma from cuts or nicks.
415 A client with no known history of peripheral d) keep the affected leg level or slightly dependent.
. vascular disease comes to the emergency Explanation:
department complaining of sudden onset of lower While the physician makes treatment decisions, the
leg pain. Inspection and palpation reveal absent nurse should maintain the client on bed rest, keeping
pulses; paresthesia; and a mottled, cyanotic, cold, the affected leg level or slightly dependent (to aid
and cadaverous left calf. While the physician circulation) and protecting it from pressure and other
determines the appropriate therapy, the nurse trauma. Warming the leg with a heating pad (or
should: chilling it with an ice pack) would further
a) place a heating pad around the affected calf. compromise tissue perfusion and increase injury to
b) elevate the affected leg as high as possible. the leg. Elevating the leg would worsen tissue
c) shave the affected leg in anticipation of surgery. ischemia. Shaving an ischemic leg could cause
d) keep the affected leg level or slightly dependent. accidental trauma from cuts or nicks.
416 A client with non-insulin-dependent diabetes mellitus (NIDDM) takes Regular insulin
. metformin (Glucophage) daily. The client is scheduled for major surgery subcutaneously per sliding
requiring general anesthesia the next day. The nurse anticipates the use of scale
which approach to best manage the client's diabetes while the client is NPO
during the perioperative period?
417 A client with Parkinson disease is taking carbidopa-levodopa (Sinemet). lessing of tremors
. Which observation by the nurse would indicate that the desired outcome of
the medication is being achieved?
418 A client with peptic ulcer disease must begin triple medication therapy. For 10 to 14 days
. how long will the client follow this regimen?
419 The client with pheochromocytoma should be instructed to avoid activities hypertensive crises or
. that precipitate paroxysms, such as The
Valsalva maneuver.
420 A client with primary pulmonary hypertension is being evaluated for a Oxygen
. heart-lung transplant. The nurse asks the client what treatments he's Diuretics
currently receiving for his disease. He's likely to mention which treatments? Vasodilators
Select all that apply
421 A client with primary syphilis is at risk for transmitting the disease t sexual partners if he or she
. is not knowledgeable about
how the disease is spread.
422 A client with respiratory acidosis is admitted to the intensive care unit for a) Shock
. close observation. The nurse should stay alert for which complication
associated with respiratory acidosis?

a) Shock
b) Stroke
c) Seizures
d) Hyperglycemia
423 The client with sensorineural hearing loss has difficulty hearing high-pitched sounds.
.

424 A client with severe and chronic liver disease is showing manifestations Vitamin A
. related to inadequate vitamin intake and metabolism. He reports difficulty
driving at night because he cannot see well. Which of the following vitamins
is most likely deficient for this client?
425 client with severe and chronic liver disease is showing manifestations related Vitamin A
. to inadequate vitamin intake and metabolism. He reports difficulty driving
at night because he cannot see well. Which of the following vitamins is most
likely deficient for this client?
426 A client with status asthmaticus requires endotracheal intubation and Sweating, tremors and
. mechanical ventilation. Twenty-four hours after intubation, the client is tachycardia
started on the insulin infusion protocol. The nurse must monitor the client's
blood glucose levels hourly and watch for which early signs and symptoms
associated with hypoglycemia?
427 A client with suspected severe acute respiratory syndrome (SARS) comes to Institute isolation
. the emergency department. Which physician order should the nurse precautions.
implement first?
428 A client with type 1 diabetes presents with a decreased level of consciousness Adminiistering 1 ampule
. and a fingerstick glucose level of 39 mg/dl. His family reports that he has of 50 % dextrose solution
been skipping meals in an effort to lose weight. Which nursing intervention per physcian order
is most appropriate?
429 A client with unresolved hemothorax is febrile, with chills and sweating. He empyema.
. has a nonproductive cough and chest pain. His chest tube drainage is
turbid. A possible explanation for these findings is:
430 : Clindamycin may enhance the action o neuromuscular blocking
. agents by blocking
neuromuscular
transmission.
431 Clindamycin may enhance the action of neuromuscular blocking agents by blocking neuromuscular
. transmission.
432 Clinical findings for osteoarthritis joint pain, crepitus, Heberden's nodes (bony growths at the
. include distal interphalangeal joints), Bouchard's nodes (growths
involving the proximal interphalangeal joints), and enlarged
joints.
433 Clinical manifestations include severe eye pain, colored halos around lights, and rapid vision
. loss.
434 Clinical Manifestations of Bacterial 1. fever
. pneumonia 2. Productive
=Bacterial = 3. Elevated
1. High _____ 4. Infiltrates
2.____________ cough
3._________ white count
4.____________ on chest X ray
435 Clinical manifestations of hypokalemia irregular pulse, fatigue, muscle weakness, flabby muscles,
. include an decreased reflexes, nausea, vomiting, and ileus.
436 Clinical Manifestations of viral 1. Low grade
. pneumonia 2. Non-productive
=Viral= 3. Normal/low
1.__________ fever 4. Minimal changes
2.________________ cough
3._____________white count
4.__________________ on Chest X ray
437 Clinical Manifestations TB 1. usually asymptomatic
. 1. Early stages........ 2. fatigue, malaise, anorexia, weight loss, low grade fevers,
2. Later....... night sweats
3. Cough becomes frequent 3.
a) white, frothy sputum
a) Produces white..... b) is not common and is usually associated with advanced
b) Hemoptysis..... disease
4.
4. Acute symptoms (generalized flu -High fever
symptoms) -Chills
-Pleuritic pain
-Productive cough
438 Clonidine (Catapres) is a central-acting adrenergic antagonist.
.

439 Clonidine (Catapres) reduces of this drug sympathetic outflow from the central nervous system.
.

440 Clubbing describes an increased angle between the nail plate and nail base.
.

441 Coffee and tea are considered neutral urine pH.


. because they don't alter the
442 Collaborative Care TB 1. Hospitalization....
. 1. _______________ not necessary for Active disease
most patients. 2.
INH, rifampin, pyrazinamide, and Myambutol.
-Active disease Directly observed therapy (DOT)
2. Four drugs are used in initial phase for 3. 4-7 months, prophylaxis 6-12 months
maximum effectiveness....which ones are 4. liver failure
they.

Directly observed therapy (DOT)


3. Drug therapy may last.....
4. Meds may cause what ?
443 Collateral circulation in the heart is a -A person's genetic predisposition for angiogenesis
. result/dependent on which two things? -Chronic ischemia
444 colles fracture
.
445 colles fracture
.

446 colon of hirshsprung disease


.

447 Color Doppler imaging blood flow through the umbilical cord.
. ultrasonography identifie
448 Colorectal polyps are common colon cancer.
. with
449 colostomy care 4 wks post op The client should be encouraged to discuss any concerns about his
. sexuality
450 colostomy care 4 wks post op high-carbohydrate or high-protein diet. Rather, the client will be
. The client will not need to encouraged to maintain a normal diet while avoiding any foods that cause
maintain a odor and flatulence.
451 colostomy for hirshsprung
.

452 A common complication of Pulmonary fibrosis


. many types of environmental
lung diseases is:
453 common complication of gastric irritation and peptic ulcers.
. steroid therapy is
454 Common examples of vesicles blisters and the lesions caused by chickenpox and herpes simplex
. include
455 A common finding of IVH bulging fontanel.
. intraventricular hemorrhaging
is
456 Common side effects of lithium are nausea, dry mouth, diarrhea, thirst, mild
. hand tremor, weight gain, bloating,
insomnia, and light-headedness.
Immediately notifying the physician about
these common side effects is not necessary.
457 community acquired Pneumonia (CAP) 1. community setting or within first 48
. 1. Occurs in hours after hospitalization
2. some organisms that cause it are ? 2.
3. whats Identified in approximately 50% of cases S.Pneumoniae
H. influenzae
Legionella
3. Gram negative rods Identified in
approximately 50% of cases
458 Competent clients have the right to refuse treatment, so the A 35-year-old female client with cancer
. nurse should first ensure that the client is competent (D). (A refuses to allow the nurse to insert an IV for
and C) are not necessary for a competent client to refuse a scheduled chemotherapy treatment, and
treatment. The nurse cannot document (C) until the states that she is ready to go home to die.
healthcare provider is notified of the client's wishes and a What intervention should the nurse initiate?
discharge prescription is obtained. A. Review the client's medical record for an
Correct Answer: D advance directive.
B. Determine if a do-not-resuscitate
prescription has been obtained.
C. Document that the client is being
discharged against medical advice.
D. Evaluate the client's mental status for
competence to refuse treatment.
459 A complication of balloon valvuloplasty is emboli resulting in a stroke.
.

460 complication of deep vein thrombosis is pulmonary tachycardia, fever, hypotension, diaphoresis,
. embolism pallor, shortness of breath, and friction rub.
461 complication of deep vein thrombosis is pulmonary dyspnea, severe chest pain, apprehension,
. embolism cough (possibly accompanied by
hemoptysis),
462 Complications TB Miliary TB
. Miliary TB 1. invade the bloodstream and spread to all
1. Large numbers of organisms do what ? organs
-Acute or chronic symptoms
Pleural effusion and empyema Pleural effusion and empyema
2. Caused by..... 2. bacteria in pleural space
3. Inflammatory reaction with..... 3. pleural exudates of protein-rich fluid
TB pneumonia
TB pneumonia 4. discharging from granulomas into lung or
4. Large amounts of bacilli....... lymph nodes
463 The components of every pain assessment A female client who has breast cancer with metastasis to the
. should include sensory patterns, area, liver and spine is admitted with constant, severe pain despite
intensity, and nature (PAIN) of the pain (A) around-the-clock use of oxycodone (Percodan) and
and are essential in identifying appropriate amitriptyline (Elavil) for pain control at home. During the
therapy for the client's specific type and admission assessment, which information is most important
severity of pain, which may indicate the for the nurse to obtain?
onset of disease progression or A. Sensory pattern, area, intensity, and nature of the pain.
complications. Triggers (B), current drug B. Trigger points identified by palpation and manual pressure
usage (C), and sympathetic responses (D), of painful areas.
such as tachycardia, diaphoresis, and C. Schedule and total dosages of drugs currently used for
elevated blood pressure, are important, but breakthrough pain.
should be obtained after focusing on (A). D. Sympathetic responses consistent with onset of acute
Correct Answer: A pain.
464 % composition= Element molar mass/compound molar mass x 100
.

465 The condition in which the urethral opening hypospadias.


. is on the ventral side of the penis or below
the glans penis is referred to as
466 conductive loss
.

interrupted sound from external ear to junction of staples and


oval window due to wax
467 Considering that there is usually 1 L of insensible fluid loss, this client's output exceeds his intake
. (intake, 2,000 ml; output, 2,200 ml), indicating deficient
fluid volume.
468 Constipation lasting 3 days or longer is doxorubicin (Adriamycin) 10 days ago, she is susceptible to
. unusual in this client and warrants infection and should avoid rectal medications and treatments
immediate action. However, because the
client had chemotherapy with
469 Contact dermatitis is a condition in which the skin becomes red, sore, or inflamed
. after direct contact with a substance.
470 contact isolation The nurse should assemble personal protective equipment and entering the client's roo
. all needed supplies before putting on
471 Contributing Factors to pneumonia 1. move pulmonary secretions
. 1. Inability to do what ? 2. Aspiration pneumonia
2. Aspiration ___________ 3. ETOH intoxication or Cigarette smoking
3. Frequent__________or ___________ 4. cough reflex
4. Depressed what ? 5. immobility
5. Prolonged........ 6. Healthcare providers
6. __________ providers!!
472 Cool, clammy skin occurs in the . hypodynamic or cold phase (later phase). of septic shock
.

473 COPD have CO2 retention and the decreases so if have alarming labs like p02 of 70 pco2 of 66
. respiratory drive is stimulated when the check vitals because you cant give to much oxygen it will
PO2 stop breathing
474 copd Measures that help mobilize secretions include drinking 2 L of fluid daily, practicing controlled
. pursed-lip breathing, and engaging in moderate activity.
475 The correct procedure for wound irrigation 35-ml syringe and 19-French angiocatheter to provide
. includes using irrigation of about 8 pounds of pressure per square inch
to remove necrotic tissue without tissue damage.
476 Coughing during or after meals is a The nurse notes that a client consistently coughs while
. manifestation of dysphagia, or difficulty eating and drinking. Which nursing diagnosis is most
swallowing, which places the client at risk for important for the nurse include in this client's plan of
aspiration (C). Dysphagia can lead to aspiration care?
pneumonia, but the client is not currently A. Ineffective breathing pattern.
exhibiting any symptoms of breathing difficulty B. Impaired gas exchange.
(A) or impaired gas exchange (B). Although (D) C. Risk for aspiration.
may be related to an ineffective cough, the D. Ineffective airway clearance.
client's coughing is an effective response when
solids or liquids are taken orally.
Correct Answer: C
477 creams for scabies The most commonly used cream is permethrin 5%.
. Other creams include benzyl benzoate, sulfur in
petrolatum, and crotamiton. Lindane is rarely used
because of its side effects.
478 crohns
.

479 C (The nursing process is a problem-solving What is the rationale for using the nursing process in
. approach that provides an organized, planning care for clients?
systematic, decision making process to A. As a scientific process to identify nursing diagnoses
effectively address the client's needs and of a clients' healthcare problems.
problems. The nursing process includes an B. To establish nursing theory that incorporates the
organized framework using knowledge, biopsychosocial nature of humans.
judgments, and actions by the nurse as the C. As a tool to organize thinking and clinical decision
client's plan of care is determined, and making about clients' healthcare needs.
encompasses assessment, analysis, planning, D. To promote the management of client care in
implementation, and evaluation of client care collaboration with other healthcare professionals.
(C). (A, B, and D) do not support the basis for
using the nursing process.
Correct Answer: C)
480 CT is comparable not better or worse than to magnetic resonance imaging in evaluating lymph node
. metastasis.
481 C. trachomatis infection in women they can bee asymptomatic,
. but symptoms are yellowish discharge
and painful urination.
482 C. trachomatis infection in women is commonly asymptomatic, may include a yellowish discharge
. but symptoms and painful urination.
483 CT scanning is the standard noninvasive method used in a lung cancer because it can distinguish
. workup for small differences in tissue density and
can detect nodal involvement.
484 Curling's ulcer is an acute peptic ulcer of the duodenum resulting complication from severe burns when
. as a \ reduced plasma volume leads to
sloughing of the gastric mucosa
485 The current availability of many herbal supplements lacks A female client informs the nurse that
. federal regulation, research, control and standardization in the she uses herbal therapies to
manufacture of its purity and dose. Manufacturers that provide supplement her diet and manage
evidence of quality control (C), such as labeling that contains common ailments. What information
scientific generic name, name and address of the manufacturer, should the nurse offer the client about
batch or lot number, date of manufacture, and expiration date, is general use of herbal supplements?
the best information to provide. (A, B, and D) are misleading. A. Most herbs are toxic or
Correct Answer: C carcinogenic and should be used only
when proven effective.
B. There is no evidence that herbs are
safe or effective as compared to
conventional supplements in
maintaining health.
C. Herbs should be obtained from
manufacturers with a history of
quality control of their supplements.
D. Herbal therapies may mask the
symptoms of serious disease, so
frequent medical evaluation is
required during use.
486 cushings Loss of collagen makes the skin weaker and thinner; therefore, the
. client bruises more easily. The nurse
should instruct the client to report any
of these signs to the physician.
487 Cushing syndrome results from a hypersecretion of Monitor blood glucose levels daily.
. glucocorticoids in the adrenal cortex. Based on the clinical
manifestations of Cushing syndrome, which nursing intervention
would be appropriate for a client who is newly diagnosed with
Cushing syndrome?
488 Cutaneous lesions on the palms and soles and alopecia are signs secondary syphilis. Chancres do not
. of bleed sufficiently to alter tissue
perfusion.
489 CYCLOOXYGENASE INHIBITOR Prototype: Celecoxib (Celebrex)
. Second-Generation NSAID (COX-2 Inhibitor)
490 CYCLOOXYGENASE INHIBITORS Other NSAIDs
. First GenerationNonaspirin NSAIDs Fenoprofen (Nalfon)
Prototype: Ibuprofen (Advil, Motrin Flurbiprofen (Ansaid)
Ketoprofen
Naproxen (Aleve, Anaprox, Naprelan,
Naprosyn)
Oxaprozin (Daypro)
Diclofenac (Voltaren, Cataflam)risk
of liver failure
491 cystits
.

492 Cysts, such as sebaceous cysts, are elevated, thick-walled lesions containing fluid or
. semisolid matter
493 cytoscopy is tube into bladder
.

494 cytoscopy normal findings after pink tinged urine


.

495 Daily oral doses of isoniazid and rifampin for 6 active tuberculosis.
. months to 2 years are appropriate for the client
with
496 daunorubicin (DaunoXome) A red, swollen I.V. site indicates possible infiltration.
.

497 Daunorubicin is a vesicant chemotherapeutic agent and can be very


. damaging to tissue if it infiltrates.
498 Daunorubicin The nurse should immediately stop the medication, apply ice to the site, and notify
. the physician.
499 Debilitating anginal pain can be decreased in some A 47-year-old kindergarten teacher diagnosed with
. clients by the administration of beta-blocking asthma 40 years ago
agents such as nadolol (Corgard). Which client
requires the nurse to use extreme caution when
administering Corgard?
500 Decreased pulse pressure reflects Reduced stroke volume
.

501 Decreased RBC production diminishes cellular oxygen, leading to fatigue and weakness.
.

502 Deep palpation may be required to palpate the In assessing a client's femoral pulse, the nurse must
. femoral pulse; and, when palpated, the nurse use deep palpation to feel the pulsation while the
should document the presence and volume of the client is in a supine position. What action should the
pulse (B). The site is best palpated with the client nurse implement?
supine; elevation of the head of the bed requires A. Elevate the head of the bed and attempt to palpate
even deeper palpation (A). The use of deep the site again.
palpation to feel the femoral pulse does not indicate B. Document the presence and volume of the pulse
a problem requiring further assessment, such as palpated.
(C), and does not reflect the presence of edema (D). C. Use a thigh cuff to measure the blood pressure in
Correct Answer: B the leg.
D. Record the presence of pitting edema in the
inguinal area.
503 dental care for endocarditis is manual toothbrush
.

504 development of ARDS are gram-negative septic shock and gastric content
. aspiration. shock cause permabiity so then there is
leakage
505 DEXAMETHASONE is a steroid for cushings
.

506 Dextromethorphan is the most widely used antitussive in the United States because it produces
. few adverse reactions while effectively suppressing a
cough.
507 Diabetes insipidus is caused by a deficiency of antidiuretic hormone, which results in
. excretion of a large volume of dilute urine, urine
specific gravity of less than 1.005 should be reported.
508 Diabetic nephropathy is kidney disease or damage that occurs in people with
. diabetes.
509 The diagnosis of aortic regurgitation (AR) is Echocardiography
. confirmed by which of the following?
510 A diagnosis of diastolic HF is based on what? -presence of pulmonary congestion,
. -pulmonary hypertension,
-ventricular hypertrophy (due to aortic stenosis),
-normal EF,
-backs up into lungs
511. Diagnostic testing TB Skin testing
Skin testing 1. Intradermal administration of tuberculin
1. what kind of injection ? 2. injection site indicates exposure
2. Induration at.....what does it show ? 3. Sensitivity remains for life, and individual should
3. how long does Sensitivity last ? not be tested again
4. Response is less in what kind of person ? 4. immunocompromised patients
5. Reactions greater than or equal to...... 5. 5 mm=positive
Chest x-ray Chest x-ray
6. Cannot...... 6. make diagnosis solely on x-ray
7. Upper lobe infiltrates....... 7. cavitary infiltrates, and lymph node involvement
suggest TB
512 Diagnostic Tests pneumonia CXR
. CBCleukocytosis common in bacterial
Serum lytesespecially for the dehydrated
Sputum gram stain/ culturesbefore treatment if
possible
Blood cultures-for severely ill, can identify sepsis
O2 sats/blood gasescan determine treatment for
hypoxia/hypercapnia
513 Diagnostic Tests -ABGs
. -what are three of them ?
-EKG

-CXR
514 Dialysis equilibrium syndrome causes confusion, a decreasing level of consciousness,
. headache, and seizures.
515 Diaphoresis and increased salivation are not cholinergic crises.
. present in
516 . Diaphragmatic breathing not chest breathing lung expansion.
. increases
517 Diarrhea can lead to fluid volume loss, which is The home health nurse visits an elderly client who
. potentially life-threatening, so the highest priority lives at home with her husband. The client is
is to prevent a fluid volume imbalance (D). experiencing frequent episodes of diarrhea and bowel
Diarrhea and bowel incontinence can also lead to incontinence. Which problem, for which the client is
(A, B, and C), but these are of less potential harm at risk, has the greatest priority when planning the
than a fluid volume deficit. client's care?
Correct Answer: D A. Disturbed sleep pattern.
B. Caregiver role strain.
C. Impaired skin integrity.
D. Fluid volume imbalance.
518 diarrhea has what type of potassium it has low potassium because they are losing
. electrolytes i know you thought since diarrhea has
metabolic acidosis it has low potassium the acidosis
is from loss of bicarbonate acid loss and potassium
loss
519 A diastolic, blowing, aortic insufficiency.
. decrescendo murmur
accompanies
520 dic As blood collects in the dilation and distention, which is reflected in increased abdominal girth. The
. peritoneal cavity, it causes client would be tachycardic and hypotensive. Petechiae reflect bleeding in the
skin.
521 dic sign Pain, redness, warmth, and swelling in the lower leg if blood clots form in the
. deep veins of your leg.
522 dic sign Headaches, speech changes, paralysis (an inability to move), dizziness, and
. trouble speaking and understanding if blood clots form in the blood vessels in
your brain. These signs and symptoms may indicate a stroke.
523 dic sign Chest pain and shortness of breath if blood clots form in the blood vessels in
. your lungs and heart.
524 A dietary modification that Avoiding foods that require a lot of chewing
. helps meet the nutritional
needs of patients with
COPD is
525 diet for cirrhosis A low-protein and high-carbohydrate diet is recommended.
.

526 Diet therapy for patients High-fiber diet


. diagnosed with IBS include
which of the following?
527 Diet therapy for patients High fiber diet
. diagnosed with IBS include
which of the following?
528 Diet therapy for peptic tolerate.
. ulcer disease Most clients
are instructed to follow a
diet that they can
529 difficult case of scabies use Ivermectin is a pill that may be used.
.
530 digestive system
.

531 digixin A nurse usually before administering the daily maintenance dose, about 24 hours after the
. takes a serum sample previous dose.
immediately
532 di has hypernatremia and siadah has hyponatremia
.

533 Direct questions should be used after the A client provides the nurse with information about the
. client's opening narrative to fill in any details reason for seeking care. The nurse realizes that some
that have been left out or during the review of information about past hospitalizations is missing. How
systems to elicit specific facts (D) about past should the nurse obtain this information?
health problems. (A and B) are time A. Solicit information on hospitalization from the
consuming, and may require the client's insurance company.
permission to access information about other B. Look up previous medical records from archived
hospitalizations. (C) may not produce the hospital documents.
specific data needed. C. Ask the client to discuss previous hospitalizations in the
Correct Answer: D last 5 years.
D. Elicit specific facts about past hospitalizations with
direct questions.
534 Disadvantages of CABG? -Sternotomy
. -Longer hospitalization
-longer recovery
535 discharge plan for a client with multiple Carefully test the temperature of bath water.
. sclerosis Avoid hot water bottles and heating pads.
Inspect the skin daily for injury or pressure points.
Wear warm clothing when outside in cold temperatures.
536 Disconnect the syringe and pull the plunger 2cc mark
. back to the Attach the syringe to the end of the feeding tube.
537 di urine osmolarity is low because there is high water less solutes
.

538 Does pneumonia affect ventilation or Both! (lol)


. diffusion?
539 The donor twin may become growth restricted oligohydramnios while the recipient twin may become
. and can have polycythemic with polyhydramnios and develop heart
failure
540 dont use Plain tap water for gavage because may be contaminated. If introduced inadvertently into the
. lungs, it could result in damage or pneumonia.
541 Dopamine is responsible for what? dilating renal blood vessels
.

542 dorsalis pedis artery


.

543 The dorsalis pedis pulse is found on the medial aspect of the dorsal surface of the foot in line with
. the big toe.
544 Drainage during the first 6 to 12 hours bright red blood, but large amounts of blood or excessive
. contains some bloody drainage should be reported to the physician
promptly.
545 A drainage tube is placed in the wound after a modified radical mastectomy to help remove accumulated
. blood and fluid in the area
546 drinking water with copd can cause secretions to be liquidified so it is to
. mobilize and get rid of secretions
547 Droplet transmission occurs when the person coughs or sneezes and releases large
. respiratory droplets into the air. these
droplets are heavy and fall to
surfaces rapidly, usually falling
within 3 feet of the patient.
548 Drug Resistance 1.
. 1. Occurs when treatment is inadequate due to: Non-adherence to prescribed drug
2. Or what else can cause it ? regimen
Malabsorption
Inadequate dosage
Incorrect medication
2.
Occurs when a susceptible host is
infected with drug-resistant
organisms. The growing incidence of
this is an alarming world-wide health
problem.
549 Drugs administered intradermally injected between the skin layers just
. below the surface stratum corneum)
diffuse slowly into the local
microcapillary system.
550 drugs that cause hyperglycemia Prednisone, Lithium may cause
. transient hyperglycemia,
551 A dry gauze dressing not a plastic sheet-type dressing should cover the wet dressing.
.

552 dumping syndrome diet having a diet high in protein and fat
. and low in carbohydrates,
553 The durable power of attorney is a legal document or a form of Which statement best describes
. advance directive that designates another person to voice durable power of attorney for health
healthcare decisions when the client is unable to do so. A durable care?
power of attorney for health directives is legally binding (A). (B, C A. The client signs a document that
and D) do not include the legal parameters that must be designates another person to make
determined by the client in the event the client is unable to make a legally binding healthcare decisions
healthcare decision, which can be changed by the client at any if client is unable to do so.
time. B. The healthcare decisions made by
Correct Answer: A another person designated by the
client are not legally binding.
C. Instructions about actions to be
taken in the event of a client's
terminal or irreversible condition are
not legally binding.
D. Directions regarding care in the
event of a terminal or irreversible
condition must be documented to
ensure that they are legally binding.
554 During administration of a rectal suppository, the client is asked While preparing to insert a rectal
. to take slow deep breaths through the mouth to relax the anal suppository in a male adult client, the
sphincter (D). Bearing down (A) will push the suppository out of nurse observes that the client is
the rectum, so the suppository should not be inserted while the holding his breath while bearing
client is bearing down (B). Further data is needed before down. What action should the nurse
performing an invasive digital exam to check for fecal impaction implement?
(C). A. Advise the client to continue to
Correct Answer: D bear down without holding his breath.
B. Gently insert the lubricated
suppository four inches into the
rectum.
C. Perform a digital exam to
determine if a fecal impaction is
present.
D. Instruct the client to take slow
deep breaths and stop bearing down.
555 During admission of a patient diagnosed with non-small cell A,B,C
. carcinoma of the lung, the nurse questions the patient related to a
history of which of the following risk factors for this type of
cancer (select all that apply)?
A) Asbestos exposure
B) Cigarette smoking
C) Exposure to uranium
D) Chronic interstitial fibrosis
E) Geographic area in which he was born
556 During a health fair, a male client with emphysema tells the nurse Pace your activities and schedule rest
. that he fatigues easily. Assessment reveals marked clubbing of the periods
fingernails and an increased anteroposterior chest diameter.
Which instruction is best to provide the client?
557 During a health-promotion program, the nurse plans to target Women develop lung cancer at a
. women in a discussion of lung cancer prevention because (select younger age than men
all that apply) More women die of lung cancer than
die from breast cancer
Women who smoke are at greater risk
to develop lung cancer than men who
smoke
Women are more likely to develop
small cell carcinoma than men
558 During an acute exacerbation of COPD, the patient is severely Position the patient upright with the
. short of breath and the nurse identifies a nursing diagnosis of elbows resting on the over the bed
ineffective breathing pattern related to obstruction of airflow and table.
anxiety. The best action by the nurse is to
559 During an annual health assessment of a 65 year old clinic patient, Obtain the pneumococcal vaccine this
. the patient tells the nurse he had the pneumonia vaccine when he year with an annual influenza vaccine
was age 59. The nurse advises the patient that the best way for
him to prevent pneumonia now is to
560 During an annual health assessment of a 65 y/o clinic patient, the Obtain the pneumococcal vaccine this
. patient tells the nurse he had the pneumonia vaccine when he was year with an annual influenza vaccine
58. The nurse advises the patient that the best way from him to
prevent pneumonia now is to:
561 During a spinal tap, a sample of cerebrospinal fluid i withdrawn from your spinal canal.
.

562 During assessment of a client admitted for cardiomyopathy, the Decreased cardiac output
. nurse notes the following symptoms: dyspnea on exertion, fatigue,
fluid retention, and nausea. The initial appropriate nursing
diagnosis is which of the following?
563 During assessment of a client in the intensive care unit, the nurse Prepare the client for a pericardial
. notes that the client's breath sounds are clear upon auscultation, tap.
but jugular vein distention and muffled heart sounds are present.
Which intervention should the nurse implement?
564 During change of shift report, the charge nurse reviews the A continuous epidural infusion of
. infusions being received by clients on the oncology unit. The client morphine
receiving which infusion should be assessed first?
565 During discharge teaching for a 65-year-old patient C
. with COPD and pneumonia, which of the following
vaccines should the nurse recommend that this
patient receive?
A) a. Staphylococcus aureus
B) Haemophilus influenzae
C) Pneumococcal
D) Bacille-Calmette-Gurin (BCG)
566 During preoperative teaching for a client who will "You must avoid hyperextending your neck after
. undergo subtotal thyroidectomy, the nurse should surgery" --->To prevent undue pressure on the
include which statement? surgical incision after subtotal thyroidectomy, the
nurse should advise the client to avoid
hyperextending the neck. The client may elevate the
head of the bed as desired and should perform deep
breathing and coughing to help prevent pneumonia.
Subtotal thyroidectomy doesn't affect swallowing
567 During report, the nurse learns that a client with Monitor the client's serum potassium and blood
. tumor lysis syndrome is receiving an IV infusion glucose.
containing insulin. Which assessment should the
nurse complete first?
568 During spinal anesthesia, medication is injected spinal canal to numb the nerves in the lower half of
. into you your body.
569 During the arteriogram, the client reports having Inform the physician, symptoms suggest an allergic
. nausea, tingling, and dyspnea reaction. Treatment may involve administering
oxygen and epinephrine.
570 During the emergent phase of burn management, massive shift of fluid from the blood vessels
. there is a (intravascular compartment) into the tissues
(interstitial compartment).
571 During the intermediate phase of burn care, the diminish as a result of hemodilution, which occurs as
. client's hematocrit should the fluids shift back into the circulating blood volume
from the tissues
572 During the shift report, the charge nurse informs a Continue with shift report and talk to the nurse about
. nurse that she has been assigned to another unit for the incident at a later time.
the day. The nurse begins to sigh deeply and tosses
about her belongings as she prepares to leave,
making it known that she is very unhappy about
being "floated" to the other unit. What is the best
immediate action for the charge nurse to take?
573 dvt The extremity should be kept elevated with heat applied to treat the inflammation and pain
.

574 (D) will help to move and drain respiratory The nurse assesses an immobile, elderly male client
. secretions and prevent pneumonia from occurring, and determines that his blood pressure is 138/60, his
so this intervention has the highest priority. Older temperature is 95.8 F, and his output is 100 ml of
adults often have an increased BP, and a PRN concentrated urine during the last hour. He has wet-
antihypertensive medication is usually prescribed sounding lung sounds, and increased respiratory
for a BP over 140 systolic and 90 diastolic (A). secretions. Based on these assessment findings, what
Older adults often run a lower temperature, nursing action is most important for the nurse to
particularly in the morning, and (B) does not have implement?
the priority of (D). Even though the client has A. Administer a PRN antihypertensive prescription.
adequate output, (C) might be encouraged because B. Provide the client with an additional blanket.
the urine is concentrated, but this intervention does C. Encourage additional fluid intake.
not have the priority of (D). D. Turn the client q2h.
Correct Answer: D
575 Dyshidrotic eczema is a condition in which small, usually itchy blisters
. develop on the hands and feet
576 Dyspnea and cyanosis are associated fluid excess, not CRF.
.

577 Dyspnea and cyanosis are associated with ...


.

578 Early clinical manifestations of cirrhosis are subtle gastrointestinal symptoms, such as anorexia,
. and usually include nausea, vomiting, and changes in bowel patterns.
These changes are caused by the liver's altered
ability to metabolize carbohydrates, proteins, and
fats
579 Early emphasis on rehabilitation is important to complications and to help ensure that the client
. decrease will be able to make the adjustments necessary to
return to an optimal state of health and
independence
580 Eating and sleep are high priorities during this taking in phase
.

581 Elderly clients should not be given meperidine because of the risk of acute confusion
. and seizures in this population.
582 An elderly client with influenza is admitted to an acute b) Pneumonia
. care facility. The nurse monitors the client closely for
complications. What is the most common complication
of influenza?

a) Septicemia
b) Pneumonia
c) Meningitis
d) Pulmonary edema
583 An elderly patient diagnosed with diarrhea is taking Hypokalemia
. digoxin (Lanoxin). Which of the following electrolyte
imbalances should the nurse be alert to?
584 An elevated serum potassium level may lead to a life- palpating the pulse.
. threatening cardiac arrhythmia, which the nurse can
detect immediately by
585 Elevating the legs above the heart or wearing alleviating venous congestion and may worsen
. antiembolism stockings is a strategy for peripheral arterial disease.
586 An emaciated homeless client presents to the Initiate airborne infection precautions.
. emergency department complaining of a productive
cough with blood-tinged sputum and night sweats.
What action is most important for the emergency
department triage nurse to implement for this client?
587 emergent burn phase has what type of acid base? Metabolic acidosis,commonly develops due to
. loss of bicarbonate ions.
588 emergent Fluid shifting into the interstitial space intravascular volume depletion and decreased
. causes perfusion to the kidneys. This would result in an
increase in serum creatinine.
589 emergent Little fluctuation in weight suggests that no fluid retention and the intake is equal to
. there is output.
590 emergent phase of burn Hemoconcentration, not hemodilution, is caused
. by circulatory dehydration as plasma shifts into
the extracellular space.
591 emergent phase of burn potassium is released into the extracellular fluid,
. which leads to hyperkalemia.
592 emergent phase of burn has what type of sodium Hyponatremia is another anticipated electrolyte
. imbalance because sodium is trapped in
edematous
593 Emotional stressors do not cause stress incontinence. It is most commonly caused
. by relaxed pelvic musculature.
594 Emperical formula Simplest formula of a compound
.

595 emphysema If the client has copious secretions and has postural drainage and chest physiotherapy.
. difficulty mobilizing secretions, the nurse should teach
him and his family members how to perform
596 empirical formula simplest formula
.

597 encephalopathy is when liver can not detoxify blood so ammonia builds
. up and go to the brain
598 Endoscopy is then performed to directly visualize the upper GI tract and locate the
. source of bleeding.
599 Epidural anesthesia is associated with a decreased urge to void; therefore, catheterization of a full
. bladder may be necessary.
600 epidural for L & D, To provide the safest care for this check if she can walk not dangling she has
. client the nurse should epidural so see if legs can move
601 Epistaxis, or nosebleed significant blood loss, systemic vertigo, increased pulse, shortness of breath,
. symptoms, such as decreased blood pressure, and pallor, will
occur.
602 Ethambutol (EMB) 1. bacteriostatic
. 1. Action 2. 15-25 mg/kg
2. Dose: ____to ____mg/kg for children and adults 3. oral contraceptives, seizure medications, and
3. Interactions anticoagulants
4. Major adverse effects 4. optic neuritismay cause decreased visual
acuity and/or decreased red-green color
discrimination
603 Etiology/ Patho TB 1. slowly and spreads via the lymphatic system
. 1. Replicates.... 2.
2. If cellular immune system is activated -Tissue granuloma forms
-Contains the bacteria and prevents replication
3. Favorable environments for growth and spread of disease
3.
-Upper lobes of lungs
-Kidneys
-Epiphyses of bone
-Cerebral cortex
-Adrenal glands
604 Even if this is only one incident, the nurse may be A female nurse who sometimes tries to save
. suspected of taking medications on a regular basis and time by putting medications in her uniform
the incident could be interpreted as diversion (A), or pocket to deliver to clients, confides that after
diverting narcotics for her own use, which should be arriving home she found a hydrocodone
reported to the peer review committee and to the State (Vicoden) tablet in her pocket. Which possible
Board of Nursing. (B, C, and D) are also of concern, but outcome of this situation should be the nurse's
(A) is the most serious possible outcome. greatest concern?
Correct Answer: A A. Accused of diversion.
B. Reported for stealing.
C. Reported for a HIPAA violation.
D. Accused of unprofessional conduct.
605 Every shift you work in the hospital unit where you The measured BP reflects the ability of the
. practice nursing, blood pressures are measured as a arteries to stretch and fill with blood, the
component of your policy-scheduled assessments. Much efficiency of the heart as a pump, and the
information can be gleaned from comparing blood volume of circulating blood.
pressure measurements. What does a blood pressure
reading indicate
606 Exhalation should be longer than inhalation to prevent collapse of the
. bronchioles.
607 Exhaling slowly as if trying to blow out a candle is a pursed-lip breathing to facilitate exhalation in
. technique used in clients with chronic obstructive pulmonary
disease.
608 expectorants Robitussin
. Mucinex
Guaifenesin (generic for Mucinex)
609 Experiences with the same type of pain that has The nurse working in the emergency department is
. successfully been relieved makes it easier for a assessing four clients' ability to tolerate pain. Which
client to interpret the pain sensation, and as a client is likely to tolerate a higher level of pain?
result, the client is better prepared to take steps A. A 10-year-old who was burned by a camp fire earlier
to relieve the pain (D). (A, B, and C) are having today.
new experiences with pain. B. A 70-year-old who has a postoperative infection from
Correct Answer: D a surgery one week ago.
C. A 23-year-old woman who sprained her knee while
bicycling.
D. A 55-year-old woman who has had moderate low
back pain for three months.
610 Expressive aphasia is a condition in which the what he or she wants to say
. client understands what is heard or written but
cannot say
611. extra secretion during sex Place a thin piece of gauze over the tracheostom
612 The faceplate opening should be no more than 1/8? to 1/6? larger than the stoma
.

613 Fair skin with a history of sunburn and the melanoma.


. location of the lesion on the leg (the most
common site in women) suggest
614 A family member was taught to suction a client's Inserts the catheter until resistance or coughing occurs
. tracheostomy prior to the client's discharge
from the hospital. Which observation by the
nurse indicates that the family member is
capable of correctly performing the suctioning
technique?
615 Fat necrosis occurring with acute pancreatitis hypocalcemia requiring calcium replacement.
. can cause \
616 Fat necrosis occurring with acute pancreatitis hypocalcemia requiring calcium replacement
. can cause
617 A female client, aged 82, visits the clinic for a A possible adverse effect of blood pressure medicine is
. blood pressure (BP) check. Her hypertension is dizziness when you stand.------>A possible adverse
not well controlled, and a new blood pressure effect of all antihypertensive drugs is postural
medicine is prescribed. What is important for hypotension, which can lead to falls. Teaching should
the nurse to teach this client about her blood include tips for managing syncope and dizziness. You
pressure medicine? would not teach the client to take the medicine on an
empty stomach. (less)
618 A female client is readmitted to the facility with d) Recent pelvic surgery
. a warm, tender, reddened area on her right calf. Explanation:
Which contributing factor should the nurse The client shows signs of deep vein thrombosis (DVT).
recognize as most important? The pelvic area has a rich blood supply, and
a) An active daily walking program thrombophlebitis of the deep veins is associated with
b) A history of diabetes mellitus pelvic surgery. Aspirin, an antiplatelet agent, and an
c) History of increased aspirin use active walking program help decrease the client's risk of
d) Recent pelvic surgery DVT. In general, diabetes mellitus is a contributing
factor associated with peripheral vascular disease.
619 A female client who received a nephrotoxic drug Oliguria
. is admitted with acute renal failure and asks the
nurse if she will need dialysis for the rest of her
life. The nurse should explain which
pathophysiologic consequence that supports the
need for temporary dialysis until acute tubular
necrosis subsides?
620 A female client with a nasogastric tube attached reposition the patient on her side
. to low suction states that she is nauseated. The
nurse assesses that there has been no drainage
through the nasogastric tube in the last 2 hours.
What action should the nurse take first?
621 A female client with diabetes mellitus type 2 has How the client examines her feet
. a plantar foot ulcer. When developing a teaching
plan regarding foot care, what information
should the nurse obtain first from this client?
622 A female practical nurse (PN) who is a valued employee tells the Reassign the PN until the resident
. charge nurse in a long-term facility that she does not want to be can be assessed more completely
assigned to one particular resident. She reports that the male client for reality orientation.
keeps insisting that she is his daughter and begs her to stay in his
room. What is the best managerial decision?
623 A fetus normally move 10 to 12 times per hour.
.

624 a fib Because of the poor emptying of blood from the atrial clot formation around the valves.
. chambers, there is an increased risk for .
625 a fib The clots become dislodged and travel through the
. circulatory system. As a result,
cerebrovascular accident is a
common complication
626 Find the empirical formula of a compound by... 1- Change elements' % to g
. 2- Use molar masses to convert to
mol
3- Divide all by the smallest #
4- Put those #s into the forrmula as
subscripts
if the # ends up being a decimal
(2.5) multiply all #s by 2
627 Fine crackles are present when there is fluid in the lungs
.

628 finish 226


.

629 FIRST-GENERATION H1 ANTAGONISTS Other First-Generation


. Antihistamines Antihistamines
Prototype: Diphenhydramine (Benadryl Brompheniramine
Chlorpheniramine (Chlor-
Trimeton)
Dexchlorpheniramine
Clemastine (Tavist)
Promethazine (Phenergan)
Hydroxyzine (Vistaril)
630 The first heart sound (S1) occurs when the mitral and tricuspid valves close
.

631 The first part of the nursing diagnosis statement is the diagnostic Which statement is an example of
. label and is followed by related to the cause, which should direct the a correctly written nursing
nurse to the appropriate interventions. (D) best fits this criteria. (A diagnosis statement?
and B) contain a medical diagnosis. (C) includes an observable A. Altered tissue perfusion related
cause, but (D) focuses on the client's response, which the nurse can to congestive heart failure.
provide support, reflection, and dialogue. B. Altered urinary elimination
Correct Answer: D related to urinary tract infection.
C. Risk for impaired tissue
integrity related to client's refusal
to turn.
D. Ineffective coping related to
response to positive biopsy test
results.
632 Flail Chest 1. multiple rib fractures, causing
. 1. Caused from what ? unstable thoracic cage
633 The fluid shift, which occurs between the intravascular increased capillary permeability that allows
. and interstitial extracellular compartments, is caused b water, sodium, and protein to shift to the
tissues.
634 A Foley catheter provides accurate output assessment to prerenal acute renal failure that can occur from
. monitor for hypovolemia.
635 folliculitis treatment Treatment may include antibiotics applied to
. the skin (mupirocin) or taken by mouth
(dicloxacillin), or antifungal medications to
control the infection.
636 folliculitis treatment Hot, moist compresses may promote drainage
. of the affected follicles.
637 Following a motor vehicle accident, the nurse assesses Paradoxical chest movement
. the driver for which distinctive sign of flail chest?
638 Following assessment of a patient with pneumonia, the SpO2, of 86%
. nurse identifies a nursing diagnosis of impaired gas
exchange based on the findings of:
639 Following a thoracotomy, the patient has a nursing Medicate the patient with analgesics 20-30 min
. diagnosis of ineffective airway clearance related to before assisting to cough and deep-breathe
inability to cough as a result of pain and positioning. The
best nursing interventionfor this patient is to
640 The following measures are required for Droplet If the patient must leave their room, notify the
. Precaution receiving area and have the patient wear a
surgical mask when possible to minimize the
dispersal of droplets.
641 The following measures are required for Droplet Employees and visitors must wear a surgical
. Precautions mask to enter the room.
642 The following measures are required for Droplet Place the patient in a private room. No special
. Precautions ventilation is required.


643 Foods high in potassium include bran and whole grains; most dried, raw, and
. frozen fruits and vegetables; most milk and
milk products; chocolate, nuts, raisins, coconut,
and strong brewed coffee.
644 Foods that sometimes need to be limited, in order to Raw vegetables
. make it easier to manage your colostomy, include Skins and peels of fruit (fruit flesh is OK)
Dairy products
Very high fiber food such as wheat bran cereals
and breads
Beans, peas, and lentils
Corn and popcorn
Brown and wild rice
Nuts and seeds
Cakes, pies, cookies, and other sweets
High fat and fried food such as fried chicken,
sausage, and other fatty meats
645 For a client with an endotracheal (ET) tube, which a) Auscultating the lungs for bilateral breath
. nursing action is most essential? sounds

a) Auscultating the lungs for bilateral breath sounds


b) Turning the client from side to side every 2 hours
c) Monitoring serial blood gas values every 4 hours
d) Providing frequent oral hygiene
646 . Forcing large quantities of fluid may cause hydronephrosis if urine is prevented from
. flowing past calculi.
647 For clients who have cirrhosis without complications, a high-calorie, high-carbohydrate diet is
. preferred to provide an adequate supply of
nutrients.
648 For enteric wear a mask and must wear a gown only if soiling from fecal matter is likely.
. precautions, the
nurse need not
649 For gallbladder antispasmodic effect on the bile duct and gallbladder. Although the medication
. disease, reduces the production of gastric solutions as well as hypermotility, these aren't the
propantheline has an main reasons for the medication.
650 For maximum 1 hour before or 2 hours after meals. No evidence indicates that antacids interfere
. absorption, the client with the effects of sulfonamides.
should take this drug
at least
651 formula weight used instead of molas mass for ionic compounds (same as molar mass)
.

652 For the client with an auscultating the lungs regularly for bilateral breath sounds to ensure proper tube
. ET tube, the most placement and effective oxygen delivery.
important nursing
action is
653 For the first few depression, fatigue, incisional chest discomfort, dyspnea, and anorexia.
. weeks after CABG
surgery, clients
commonly experience
654 Four-point gait: a slow gait pattern in which one crutch is advanced forward and placed on the floor,
. followed by advancement of the opposite leg; then the remaining crutch is advanced
forward followed by the opposite remaining leg; requires the use of two assistive
devices (crutches or canes); provides maximum stability with three points of support
while one limb is moving.
655 fracture
.
656 fractures
.

657 Frank hematuria anticoagulation and bleeding and heparin overdose. The nurse should discontinue
. indicates excessive the heparin infusion immediately and prepare to administer protamine sulfate, the
antidote for heparin.
658 The frontal lobe personality, judgment, abstract reasoning, social behavior, language expression, and
. influences movement.
659 full thickness burn
.

660 A fungus that enters the skin surface and infection is a dermatophyte.
. causes
661 The gag reflex is governed by the glossopharyngeal nerve, one of the cranial nerves.
.

662 Gastrointestinal hemorrhage occurs in about mechanical ventilation because of the development of
. 25% of clients receiving prolonged stress ulcers.
663 gavage feeding steps Measure the tube distance from the nose
. to the earlobe, then from the earlobe to midway between
the lower tip of the sternum and the belly button. Mark
the tube with the piece of tape or a permanent marker.
664 Gavage (guh-vahj) feeding is a way to provide breastmilk or formula directly to your baby's stomach. A
. tube placed through your baby's nose (called a
Nasogastric or NG tube) carries breast milk/formula to
the stomach.
665 genital herpes Lesions may appear 2 to 12 days after exposure
.

666 Genital herpes simplex lesions typically ar painful, fluid-filled vesicles that ulcerate and heal within
. 1 to 2 weeks
667 gerd The client should no lie down until 2 to 3 hours after a meal. The client should
. sleep with the head of the bed elevated 4 to 6 inches
668 Gloves are most contaminated, so she should first when exiting the room to prevent infection
. remove them transmission.
669 Glucocorticoids should be taken in the morning, not at bedtime.
.

670 Goiter attacks and severe laryngotracheitis are inspiratory stridor only.
. associated with
671 gonnorrhea Some women have no symptoms or vaginal itching and a
. thick, purulent vaginal discharge.
672 gonorrhea The client should avoid sexual completed, and a follow-up culture confirms that the
. intercourse until treatment is infection has been eradicated (which usually takes 4 to 7
days).
673 Good sources of vitamin B12 include meats and dairy products.
.

674 The greater the concentration of dextrose in tonicity.


. solution, the greater the
675 Green, leafy vegetables are good sources of niacin, folate, and carotenoids (precursors of vitamin A)
.

676 Group B Streptococcus is a risk factor for all pregnant women and is not limited to
. those carrying twins.
677 A group of students are studying for an ppyloric sphincter
. examination on the gastrointestinal (GI)
system and are reviewing the structures of the
esophagus and stomach. The students
demonstrate understanding of the material
when they identify which of the following as
the opening between the stomach and
duodenum?
678 Guillain-Barre syndrome causes respiratory Interrupting nerve transmission to respiratory muscles
. problems primarily by:
679 the hallmark symptoms of hyperglycemia are thirst, fruity breath, and glycosuria.
. increased
680 HAP occurs when at least one of three 1. Host defenses are impaired
. conditions exists. What are the three 2. An inoculum of organisms reaches the patient's lower
conditions? respiratory tract and overwhelms host defenses
3. A highly virulent organism is present
681 Healthy middle-aged adults focus on A middle-aged woman who enjoys being a teacher and
. establishing the next generation by nurturing mentor feels that she should pass down her legacy of
and guiding, which is describe by Erikson as knowledge and skills to the younger generation. According
the developmental stage of generativity (A), to Erikson, she is involved in what developmental stage?
and is characteristic of middle adulthood. (B, A. Generativity.
C and D) are not stages of this age group B. Ego integrity.
according to Erickson's psychosocial C. Identification.
developmental theory. D. Valuing wisdom.
Correct Answer: A
682 heart
.

683 heart Injury results from prolonged ischemia ST-segment elevation.


. and is reflected by
684 heat stroke Cool liquids are easier to drink cold liquids.
. than
685 heat stroke Treatment consists of moving the adolescent to a cool environment and giving
. cool liquids.
686 Hemodilution decreased concentration (as after hemorrhage) of cells and
. solids in the blood resulting from gain of fluid
687 Hemoglobin and HCT are typically upper GI bleeding to evaluate the extent of blood loss.
. performed first in clients with \
688 hemophila x linked genetics
.

689 hemorrhoidectomy Positioning in the early stress and pressure on the operative site. The prone and
. postoperative phase should avoid side-lying positions are ideal from a comfort perspective.
690 Hemothorax is a collection of blood in the chest wall and the lung (the pleural cavity).
. space between the
691 he nurse includes frequent oral care in the risk for infection
. plan of care for a client scheduled for an
esophagogastrostomy for esophageal cancer.
This intervention is included in the client's
plan of care to address which nursing
diagnosis?
692 Heparin is administered subcutaneously, intramuscularly. A 25- or 26-gauge, - to 5/8-inch needle is
. never most appropriate for heparin administration.
693 heprin injection Gentle pressure should be applied after the injection, but
. the area must not be massaged.
694 herniated disc Common symptoms include low back pain, numbness or tingling starting in the rear and
. radiating down one leg, or numbness or weakness in the
chest, neck or arm.
695 herniated disc occurs when the inner material of a disc protrudes through the outer layer.
.

696 herpes flu-like symptoms, enlarged lymph nodes, and clusters of vesicles on her
. vagina,vaginal itching and a thick, purulent vaginal discharge.
697 Herpes genitalis flu-like symptoms, enlarged lymph nodes, and clusters of vesicles on her
. vagina
698 Herpes simplex may be passed to fetus transplacentally and, during early pregnancy, may cause
. the spontaneous abortion or premature birth.
699 herpes virus 2 Other signs and second crop of sores, and flu-like symptoms, including fever and
. symptoms during the primary swollen glands.
episode may include a
700 Herpetic keratoconjunctivitis unilateral and causes localized symptoms, such as conjunctivitis ( pink
. usually is eye swelling of conjuctivi) with herpes simplex virus
701 HF is characterized by what 4 -Ventricular dysfunction
. things? -Reduced exercise tolerance
-Diminished quality of life
-Shortened life expectancy
702 hiatal hernia
.

703 hiatal hernia to minimize intra- recline after meals, lift heavy objects, or bend.
. abdominal pressure, the client
shouldn't
704 hiatal hernina To minimize intra- eat frequent, small, bland meals that can pass easily through the
. abdominal pressure and decrease esophagus. Meals should be high in fiber to prevent constipation and
gastric reflux, the client should minimize straining on defecation (which may increase intra-abdominal
pressure from the Valsalva maneuver).
705 High-carbohydrate foods meet acute renal failure
. the body's caloric needs during
706 A high-fiber diet and milk and Crohn's disease because they may promote diarrhea.
. milk products are
contraindicated in clients wit
707 High Risk Indicators of -Chronic illnesses
. pneumonia -Diabetes
-Sickle cell anemia
-CHF
-Resides in ECF
-Over 50 years old & infants 6 to 23 months old
-Weakened immune system
-Long term steroids
-Cancer (chemo)
708 high urine osmalarity means there is high solutes in water means low water
.

709 Hirschsprung's disease is a blockage


. of the

large intestine due to improper muscle movement in the bowel.


710 Hirsutism, not hair loss, is common in Cushing's syndrome; therefore, with successful treatment,
. abnormal hair growth declines.
711. history of breast-conserving surgery, symptoms mean infection so you need to call doctor not elevate on
axillary node dissection, and radiation pillows because the symptoms not from lymph edema or lymph
therapy reports that her arm is red, swellng but from infection
warm to touch, and slightly swollen
712 Home care for hospitalization with It is important for the patient to continue with coughing and deep
. pneumonia breathing exercises for 6 to 8 weeks until all of the infection has
cleared from the lungs.
A patient should seek medical treatment for upper respiratory
infections that persist for more than 7 days.
Increased fluid intake, not caloric intake, is required to liquefy
secretions.
Home O2 is not a requirement unless the patient's oxygenation
saturation is below normal.
713 A home care nurse is caring for a "I'll avoid eating or drinking anything 6 to 8 hours before the test.
. client with complaints of epigastric
discomfort who is scheduled for a
barium swallow. Which statement by
the client indicates an understanding
of the test?
714 The home health nurse is assessing a Document the presence of these assessment findings.
. male client being treated for
Parkinson disease with levodopa-
carbidopa (Sinemet). The nurse
observes that he does not demonstrate
any apparent emotion when speaking
and rarely blinks. Which intervention
should the nurse implement?
715 A home health nurse is seeing an b) Arterial insufficiency
. elderly female client for the first time. Explanation:
During the physical assessment of the Characteristics of arterial insuffiency ulcers include location at the
client's feet, the nurse notes several tips of the toes, extreme painfulness, and circular shape with pale
circular ulcers around the tips of the to black ulcer bases. Ulcers caused by venous insufficiency will be
toes on both feet. The bases of the irregular in shape, minimal pain if superficial (can be painful), and
ulcers are pale, and the client reports usually located around the ankles or the anterier tibial area.
the ulcers to be very painful. From
these assessment findings, the nurse
suspects that the cause of the ulcers is
which of the following?
a) Neither venous nor arterial
b) Arterial insufficiency
c) Trauma
d) Venous insufficiency
716 A home health nurse is seeing an d) Venous insufficiency
. elderly male client for the first time. Explanation:
During the physical assessment of the Symptoms of venous insufficiency include present pedal pulses,
skin on the lower legs, the nurse notes edema, pigmentation in gater area, and a reddish blue color. Ulcers
edema, brown pigmentation in the caused by venous insufficiency will be irregular in shape and
gater area, pedal pulses, and a few usually located around the ankles or the anterior tibial area.
irregularly shaped ulcers around the Characteristics of arterial insufficiency ulcers include location at
ankles. From these findings, the nurse the tips of the toes, great pain, and circular shape with a pale to
knows that the client has a problem black ulcer base.
with peripheral circulation. Which of
the following does the nurse suspect?
a) Trauma
b) Arterial insufficiency
c) Neither venous nor arterial
insufficiency
d) Venous insufficiency
717 A home health nurse is teaching a client with b) Keeping the legs in a neutral or dependent
. peripheral arterial disease ways to improve position
circulation to the lower extremities. The nurse Explanation:
encourages which of the following in teaching? Keeping the legs in a neutral or dependent position
a) Elevation of the legs above the heart assists in delivery of arterial blood from the heart
b) Keeping the legs in a neutral or dependent position to the lower extremities. All the other choices will
c) Application of ace wraps from the toe to below the aid in venous return, but will hinder arterial supply
knees to the lower extremities.
d) Use of antiembolytic stockings
718 A home health nurse is visiting a home care client a) Hypoxia
. with advanced lung cancer. Upon assessing the client,
the nurse discovers wheezing, bradycardia, and a
respiratory rate of 10 breaths/min. These signs are
associated with which condition?

a) Hypoxia
b) Delirium
c) Hyperventilation
d) Semiconsciousness
719 A home health nurse knows that a 70-year-old male Thinning of the skin with loss of elasticity
. client who is convalescing at home following a hip
replacement is at risk for developing decubitus
ulcers. Which physical characteristic of aging
contributes to such a risk?
720 A homeless client with streptococcal pharyngitis is Administer one intramuscular injection of
. being seen in a clinic. The nurse is concerned that the penicillin.
client will not continue treatment after leaving the Explanation:
clinic. Which of the following measures is the highest If a nurse is concerned that a client may not
priority? perform follow-up treatment for streptococcal
a) Provide the client with oral penicillin that will last pharyngitis, the highest priority is to administer
for 5 days. penicillin as a one-time injection dose. Oral
b) Provide emphatic oral instructions for the client. penicillin is as effective and less painful, but the
c) Ask an accompanying homeless friend to monitor client needs to take the full course of treatment to
the client's follow-up. prevent antibiotic-resistant germs from developing.
d) Administer one intramuscular injection of The nurse should provide oral and written
penicillin. instructions for the client, but this is not as high a
priority as administering the penicillin. Having a
homeless friend monitor the client's care does not
ensure that the client will follow therapy.
721 Hopefulness is necessary to sustain a meaningful A client has a nursing diagnosis of, "Spiritual
. existence, even close to death. The nurse should help distress related to a loss of hope, secondary to
the client set short-term goals, and recognize the impending death." What intervention is best for the
achievement of immediate goals (B), such as seeing a nurse to implement when caring for this client?
family member, or listening to music. (A) is too vague A. Help the client to accept the final stage of life.
to be a helpful intervention. (C) does not help the B. Assist and support the client in establishing
client deal with this nursing diagnosis. (D) might be short-term goals.
implemented, but does not have the priority of (B). C. Encourage the client to make future plans, even
Correct Answer: B if they are unrealistic.
D. Instruct the client's family to focus on positive
aspects of the client's life.
722 hospice care Care is provided in the home, independent of
. physician is wrong the doctor still has input in
hospice care
723 Hospital acquired pneumonia (HAP) 1. Nosocomial
. 1. whats another name for it ? 2. more than 48 hours after admission
2. when do Symptoms occur ? -VAP
HAP Occurs when what happens? . Occurs when
3. Host... 3. defenses are impaired
4. Inoculum of ...... 4. organisms reaches the lower tract and
5. Highly....... overwhelms host's defenses
5. virulent organism is present
724 A hospitalized client is found to be Administer 50% glucose intravenously-->The unconscious,
. comatose and hypoglycemic with a blood hypoglycemic client needs immediate treatment with IV
sugar of 50 mg/dL. Which of the following glucose. If the client does not respond quickly and the blood
would the nurse do first? glucose level continues to be low, glucagon, a hormone that
stimulates the liver to release glycogen, or 20 to 50 mL of
50% glucose is prescribed for IV administration. A dose of
1,000 mL D5W over a 12-hour period indicates a lower
strength of glucose and a slow administration rate. Checking
the client's urine for the presence of sugar and acetone is
incorrect because a blood sample is easier to collect and the
blood test is more specific and reliable. An unconscious client
cannot be given a drink. In such a case glucose gel may be
applied in the buccal cavity of the mouth.
725 A hospitalized male client is receiving Turn off the continuous feeding pump.
. nasogastric tube feedings via a small-bore
tube and a continuous pump infusion. He
begins to cough and produces a moderate
amount of white sputum. Which action
should the nurse take first?
726 A hospitalized patient with impaired vision That all furniture remains in the same position
. must get a picture in his or her mind of the Correct
hospital room and where the furniture and Explanation:
the bathroom are in order to move around All articles and furniture must remain in the same positions
independently. What must the nurse throughout the patient's hospitalization.
monitor in the patient's room?

a) That the bathroom floor is always dry


b) That all furniture remains in the same
position
c) That visitors don't leave items on the
bedside table
d) That the patient's slippers stay under
the bed
727 How are unstable and stable angina -Change in usual pattern
. different? -New in onset
-*Occurs at rest
-Has a worsening pattern
-UA is unpredictable and represents a medical emergency
728 How can people who are predisposed to Even though they have nonmodifiable risk factors, they can
. CAD reduce their risk? still lower their risk at developing by reducing their
modifiable risk factors
729 How can we improve gas exchange and -Supplemental oxygen
. oxygenation in a patient with ADHF? -Morphine sulfate
-Noninvasive ventilatory support (BiPAP)
730 How can you decrease afterload in a -IV sodium nitroprusside (Nipride)
. patient with ADHF and why would we -Morphine sulfate
want that? -Nesiritide (Natrecor)

*it will improve CO and decrease pulmonary congestion


731 How can you decrease intravascular -diuretics - furosemide (Lasix)
. volume in a patient with ADHF? -Sometimes they will do ultrafiltration to pull some of the
fluid out
732 How can you decrease the venous return in a -put them in high fowler's
. patient with ADHF? -IV nitro (vasodilate)
733 How can you help determine the target organ Assess their functional health patterns:
. damage in your assessment of a patient with HTN? -Health Perception and Management
-Nutritional metabolic
-Elimination
-Activity exercise
-Sleep rest
-Cognitive-perceptual
-Self perception/self concept
-Role relationship
-Sexuality reproductive
-Coping-stress tolerance
-Value-Belief Pattern
734 How does an MI contribute to systolic HF? loss of contractility
.

735 How does chronic ischemia influence collateral When there is ischemia in a vessel due to an
. circulation? occlusion, your body compensates by creating its
own anastomoses around the occlusion.
736 How does HTN contribute to systolic HF? increase afterload
.

737 How does HTN manifest int he brain? -CVA


. -TIA
738 How does HTN manifest in the eyes? Increased pressure - retinal damage
.

739 How does HTN manifest in the heart? -CAD


. -LVH
-HF
740 How does HTN manifest in the kidneys? Changes in urine output
. nephrosclerosis
741 How does HTN manifest in the peripheral ischemia (PVD)
. vasculature?
742 How does pneumonia normally occur? Normally present flora in a patient whose resistance
. has been altered, or it results from aspiration of flora
present in the oropharynx.
743 How does valve disease contribute to systolic HF? causes mechanical problems in the heart
.

744 How do you improve cardiac function in a patient For patients who don't respond to other pharm
. with ADHF and who is it for? treatments (diuretics, morphine, vasodilators)
Give them positive inotropes (Digoxin, dopamine,
milrinone) dopamine ONLY in hospital setting
*need to monitor blood levels
745 How do you reduce anxiety in patients with ADHF? -Distraction, imagery
. -Sedative medications (e.g., morphine sulfate,
benzodiazepines)
746 How do you treat someone in a HTN crisis? You bring their BP down over a sustained period of
. time (8 hours). You can't drastically reduce their BP
all at one time
747 However, after supratentorial surgery to remove a head flat; typically, the client with such a hematoma
. chronic subdural hematoma, the neurosurgeon may is older and has a less expandable brain
order the nurse to keep the client's
748 However, it is not as direct an indicator as increase urine output.
.

749 How is a HTN crisis defined clinically? BP over 220/140


.

750 How is pain from an MI described? heaviness,


. constriction,
tightness,
burning,
pressure,
crushing
751 How is rejection detected in a heart transplant Endomyocardial biopsies (EMB)
. patient?
752 How is SARS transmitted? Respiratory Droplets and Contact
.

753 How is the appropriate antibiotic determined to be Gram stain results.


. appropriate for pneumonia?
754 How long does it take for full thickness MI? 4-6 hours
.

755 How long does it take for scar tissue to replace 6 weeks - far less compliant afterward
. necrotic tissue after an MI?
756 How long will someone who has had a CABG be in 24-36 hours
. the ICU?
757 How should BP be taken in an assessment? bilaterally, use arm with higher reading for
. subsequent measurement
758 How will you evaluate a patient's management of -Respiratory status
. HF? -Fluid balance
-Activity tolerance
-Anxiety control
-Knowledge of disease process
759 HSV infection is one of a group of disorders that, diagnostic for AIDS
. when diagnosed in the presence of HIV infection,
are considered to be
760 http://nurse-notebook.webnode.com/album/photo-
. gallery-fluids-and-electrolytes/respiratory-acidosis-
jpg2/

761 The human body has 206 bones, which are Short bones
. classified into four categories. Which types of bones
are located in the digits?
762 Human papillomavirus can lead to cervical cancer.
.

763 Humidified air helps to liquefy respiratory secretions, making them easier to
. raise and expectorate
764 Hydralazine acts to lower blood pressure by peripheral dilation without
. interfering with placental circulation.
765 hyperadrenocorticism Cushing's syndrome: a glandular disorder caused by
. excessive cortisol.
766 Hyperbilirubinemia refers to an increase in bilirubin in the blood and is not associated with IVH.
.

767 Hyperglycemia, which develops from glucocorticoid Cushing's syndrome. With successful treatment of
. excess, is a manifestation of the disorder, serum glucose levels decline.
768 hypernatremia
.

769 hypernatremia
.

770 Hyperpigmented lesions are indicators Kaposi's sarcoma.


. of
771 Hypertension is a symptom of Cushing's disease, and muscle mass is decreased.
.

772 Hypertension, not hypotension, is a sign of hypoxia


.

773 Hypertonic dextrose solutions are used t meet the body's calorie demands in a volume of fluid that will
. not overload the cardiovascular system.
774 Hyperventilation is a clinical respiratory alkalosis.
. manifestation of
775 Hypoalbuminemia, a mechanism ascites formation, results in decreased colloid osmotic pressure.
. underlying
776 Hypokalemia is a precipitating factor in hepatic encephalopathy.
.

777 Hypokinesia refers to decreased bodily movement


.

778 Hyposecretion of glucocorticoids, Addison's disease


. aldosterone, and androgens occur with
779 Hypotension, hypothermia, and pulse oximetry values by reducing arterial blood flow
. vasoconstriction may alter
780 Hypotension, not hypertension, would pulmonary embolism.
. suggest a possible
781 Hypothermia may occur as a result of open body wounds
.

782 Hypothyroidism has a metabolic effect skeletal muscle. Muscle injury results, causing the CPK-MM to
. on spill out of the damaged cells and into the bloodstream.
783 Hypovolemic shock from fluid shifts is a acute pancreatitis
. major factor in
784 hyptonic labor are have contraction but there is no effacement of cervix or decent of
. the baby
785 Idiopathic elevated HTN is called what? Essential or Primary HTN
. *there are contributing factors, but no direct cause
786 If a central venous catheter becomes disconnected, the immediately apply a catheter clamp.
. nurse should
787 If a clamp isn't available, the nurse may place a sterile syringe or catheter plug in the catheter
. hub.
788 If a neck or spine injury is suspected, the jaw-thrust maneuver should be used to open
. the client's airway.
789 If a patient has dyspnea, what test will be done if HF is BNP level will tell if it is being caused by HF.
. suspected?
790 If a stent is put in place, what is the patient given? Antiplatelets (Plavix, ASA) because plaque
. can build up on the stent. This is taken until
the intimal layer of the vasculature grows
over the stent

*sometimes drug-eluting stents are put in


place that prevent the overgrowth of the
intimal layer, which is the main cause for
stent restenosis
791 if both parents are carriers habas then 25 chance of child getting it
.

792 if calcium levels are high then parathyroid gland will not cause production of calcium because
. its already high
793 if carrier hbas and has sickle cell hbs then 50 percent chance of child getting it
.

794 if hip is dislocated wiggling toes is not good indicator if it occured


.
795 If infection or blockage caused by calculi is present, a renal colic.
. client can experience sudden severe pain in the flank area,
known as
796 if one parent dose not have sickle sell hba and other has no chance of sickle cell
. hbs then
797 if one parent has hbs and other has hbs then there is 100 percent chance sickle cell
.

798 If resistance is encountered during the initial insertion of The nurse encounters resistance when
. an enema tube, the client should be instructed to relax inserting the tubing into a client's rectum for a
while a small amount of solution runs through the tube tap water enema. What action should the
into the rectum (D) to promote dilation. (A) is unlikely to nurse implement?
resolve the problem. (B) may cause injury. (C) should not A. Withdraw the tube and apply additional
be implemented until other, less invasive actions, such as lubricant to the tube.
(D) have been taken. B. Encourage the client to bear down and
Correct Answer: D continue to insert the tube.
C. Remove the tube and check the client for a
fecal impaction.
D. Ask the client to relax and run a small
amount of fluid into the rectum.
799 if someone has productive cough they do not need to be suctioned
.

800 If someone's medicine makes them drowsy, what Tell them to take their one dose at night to
. consideration can you make to the patient? minimize the side effect
801 If someone's triglycerides are high, what nutritional Reduce or eliminate your alcohol and simple
. recommendation would be wise? sugar consumption (it lowers triglycerides)
802 If spinal fluid leaks through the tiny puncture site, you spinal headache.
. may develop a
803 If the client's elbow is bent, the IV may be unable to The nurse is administering an intermittent
. infuse, resulting in an obstruction alarm, so the nurse infusion of an antibiotic to a client whose
should first attempt to reposition the client's arm to intravenous (IV) access is an antecubital
alleviate any obstruction (B). After other sources of saline lock. After the nurse opens the roller
occlusion are eliminated, the nurse may need to check for clamp on the IV tubing, the alarm on the
a blood return (A), remove the dressing (C), or flush the infusion pump indicates an obstruction. What
saline lock (D) and then resume the intermittent infusion. action should the nurse take first?
Correct Answer: B A. Check for a blood return.
B. Reposition the client's arm.
C. Remove the IV site dressing.
D. Flush the lock with saline.
804 if the kidney is not working in glomnerphritis then it urea and ammonia so then there is buildup of
. cannot excrete ammonia which cause encephalthpy
805 If you don't hear anything, check to make sure that all the other unused openings of the feeding tube are
. closed.
806 illeostomy Eating six small meals a day is not necessary.
.

807 illeostomy The client is usually placed on a regular diet but is encouraged to eat high-fiber,
. high-cellulose foods (e.g., nuts, popcorn, corn,
peas, tomatoes) with caution; these foods may
swell in the intestine and cause an obstruction.
808 Immediately after laparoscopic cholecystectomy drink liquids. A light diet can be resumed the day
. surgery, the client will after surgery.
809 Imperforate anus is a defect that is present from birth opening to the anus is missing or blocked.
. (congenital) in which the
810 Imperforate anus signs are Meconium
.

811. In addition, some spermicides alter the vaginal pH to a strong acidic environment, which is not
conducive to survival of spermatozoa.
812 In a female client, the nurse should advance an 2 to 3 (5 to 7.5 cm) into the urethra
. indwelling urinary catheter
813 In a low anorectal anomaly, the rectum has descended puborectalis muscle.
. normally through the
814 In a male client, the nurse should advance the catheter 6 to 8.
.

815 In an acute asthma attack, diminished or absent inhaled bronchodilators, I.V. corticosteroids and,
. breath sounds can be an ominous sign indicating lack possibly, I.V. theophylline (Theo-Dur).
of air movement in the lungs and impending
respiratory failure. The client requires immediate
intervention with
816 In a patient with a bypass graft, the distal outflow c) 50
. vessel must be at least what percentage patent for the Explanation: The distal outflow vessel must be at
graft to remain patent? least 50% patent for the graft to remain patent.
a) 40
b) 30
c) 50
d) 20
817 In assessing a client diagnosed with primary Potassium
. aldosteronism, the nurse expects the laboratory test
results to indicate a decreased serum level of which
substance?
818 In assessing a client for complications of total Glucose
. parenteral nutrition, it is most important for the nurse
to monitor which lab value regularly?
819 In assessing a client with an arteriovenous shunt who is Notify the healthcare provider of the findings.
. scheduled for dialysis today, the nurse notes the
absence of either a thrill or a bruit at the shunt site.
What action should the nurse take?
820 In assessing an older client with dementia for Observe for tiredness at the end of the day
. sundowning syndrome, what assessment technique is
best for the nurse to use?
821 In assessing a patient with pneumococcal pneumonia, An abrupt onset of fever
. the nurse recognizes that clinical manifestations of this Productive cough with rust-colored sputum
condition include (select all that apply):
822 In a strangulated hernia, the hernia cannot be reduced abdominal cavity.
. back into the
823 An incarcerated hernia refers to a hernia that is irreducible but has not necessarily resulted in an
. obstruction.
824 In caring for a client with acute diverticulitis, which has a rigid hard abdomen and elevated white
. assessment data warrant immediate nursing blood cell count (WBC).
intervention? The client
825 In cholinergic crisis, I.V. edrophonium chloride (Tensilon), a not improve muscle weakness; in
. cholinergic agent, does myasthenic crisis, it does.
826 In chronic bronchitis the diaphragm is flat and weak. Diaphragmatic breathing helps
. to strengthen the diaphragm and
maximizes ventilation.
827 In chronic osteomyelitis, antibiotics are adjunctive therapy in Surgical debridement
. which of the following situations?
828 in chronic renal failure the kidney cant make vitamin d i so then it cant make calcium so then
. then the parathyroid gland begins to
increase production negative feed back
loop
829 Increased AFP levels are associated with neural tube defects, such as spina
. bifida, anencephaly, and encephalocele.
830 Increased atrial contraction or systemic hypertension can result fourth heart sound.
. in a
831 An increased serum albumin level and increase ease of breathing may indirectly imply
. that the administration of albumin is
effective in relieving the ascites.
832 Increased urine output is the best indication that the albumin is having the desired effect.
.

833 The increase in venous pressure results in an increase in capillary hydrostatic pressure, which
. causes a net filtration of fluid out of the
capillaries into the interstitial space,
resulting in edema.
834 Increasing glycosuria is a symptom of poorly managed diabetes.
.

835 Indications for Serevent include only asthma and bronchospasm induced by
. chronic obstructive pulmonary disease.
836 in di the low levels of potassium or high calcium cause the kidneys to not respond to the adh
.

837 An individual is considered obese when his or her BMI is: 30-39; Person's with BMI of less than
. 24 are at risk for poor nutritional status
Persons with BMI of 25 to 29 are
overweight. BMI greater than 40 are
extremely obese
838 In emphysema, the anteroposterior diameter of the chest wall is diminished.
. increased. As a result, the client's breath sounds may be
839 In evaluating care, the nurse should first determine if the In evaluating client care, which action
. expected outcomes of the plan of care were achieved (A). As should the nurse take first?
indicated, the nurse may then review the initial nursing actions A. Determine if the expected outcomes
and the rationales for those actions (B), document successful of care were achieved.
completion of the care plan goals (C), and revise the plan of care B. Review the rationales used as the
(D). basis of nursing actions.
Correct Answer: A C. Document the care plan goals that
were successfully met.
D. Prioritize interventions to be added
to the client's plan of care.
840 In evaluating the effects of lactulose (Cephulac), which outcome Two or three soft stools per day
. would indicate that the drug is performing as intended?
841 An infected chest tube site, lobar pneumonia, and P. carinii fever, chills, and sweating associated
. pneumonia can lead to with infection. However, in this case,
turbid drainage indicates that empyema
has developed.
842 inferior surface of the left ventricle
.

843 inferior wall myocardial infarction T-wave inversion


. (MI). ECG changes associated with ST-segment elevation
an evolving MI? Pathologic Q-wave
844 Inflammation, evidenced by PAIN , swelling, and redness, is one of the early signs of infection
. and needs prompt intervention.
845 Inflammation of a hair follicle is folliculitis
. called
846 inguinal hernia
.

847 An inguinal hernia ( /wnl protrusion of abdominal-cavity contents through the inguinal canal.
. hrni/) is a
848 inh limit foods like . Foods such as cheese, dairy products, alcohol (red wine and beer),
. bananas, raisins, caffeine, and chocolate should be limited.
849 INH, RIF, PZA, and EMB 1. drug susceptibility results are known
. 1. All patients should be started on 2. (first 8 weeks) using 4 drugs
these 4 drugs, until what ? 3. (next 4 months of a 6-month pansensitive regimen) using INH
2. Initiation phase (first___weeks) and RIF
using 4 drugs 4. does not convert within 2 months or if there is cavitation on CXR
3. Continuation phase...... after 2 months
4. Treatment may need to be
extended for patients whose sputum
does not do what ?
850 Initial antibiotic treatment for History and physical examination and characteristics chest
. pneumonia is usually based on radiographic findings
851 In metabolic alkalosis, the body tries carbon dioxide, so there is no need to have the client inhale carbon
. to compensate by conserving dioxide, as would be the case if hyperventilation were occurring.
852 In people with diabetes, the nephrons thicken and slowly become scarred over time. The kidneys begin to
. leak and protein (albumin) passes into the urine
853 In producing urine, the kidneys urea and ammonia
. excrete wastes such as
854 In PVD, decreased blood flow can increased venous pressure.
. result in
855 In severe cases oligohydramnios may amnioinfusion during labor to prevent umbilical cord compression.
. be treated with
856 Inspiratory and expiratory stridor is foreign body obstructing the trachea or mainstem bronchi.
. a low-pitched crowing sound heard
in a client who has a
857 Insufficient secretion of GH causes dwarfism or growth delay.
.

858 Insulin administration causes potassium to enter the cells, which further lowers the serum
. potassium level.
859 Insulin is a required hormone for any client with diabetes pregnant client.
. mellitus, including the
860 In taking health histories of the following individuals, which An alcoholic, tobacco-chewing
. client would have the greatest potential for development of head auctioneer
and neck cancer?
861 Interferons (IFNs) are proteins made and released by host cells in pathogens such as viruses, bacteria,
. response to the presence of parasites or tumor cells
862 intermediate phase of burn care Loss of serum sodium leads to metabolic acidosis, not metabolic
. alkalosis.
863 intermediate phase of burn care Urinary output increases during renal perfusion increases.
. this phase as
864 .Intermittent claudication subsides with rest.
.

865 Intermittent, painless vaginal bleeding is a classic symptom of cervical cancer, but given the client's
. history, bleeding in more likely a
result of the radiation.
866 Intermittent self-catheterization is appropriate for overflow or reflux incontinence, but
. not urge incontinence, because it does
not treat the underlying cause.
867 Intertrigo refers to irritation of opposing skin surfaces
. caused by friction.
868 Intestinal cantour tubes are not irrigated.
.

869 The intestinal lumen and the blood supply to the intestine are acute intestinal obstruction. Without
. obstructed, causing an immediate intervention, necrosis and
gangrene may develop
870 In the early stages of cirrhosis, there is no need to restrict fat, protein, or sodium
.

871 In the intermediate phase of burn care, the client will experience sodium deficits.
. serum
872 In the nursing process, the evaluation component examines the What activity should the nurse use in
. effectiveness of nursing interventions in achieving client the evaluation phase of the nursing
outcomes (D). (A) is an evaluation of client satisfaction, not process?
outcomes. (B) is a written record of the plan of care. Although A. Ask a client to evaluate the nursing
(C) may occur when client outcomes are achieved, evaluation is care provided.
best determined by attainment of measurable client outcomes. B. Document the nursing care plan in
Correct Answer: D the progress notes.
C. Determine whether a client's health
problems have been alleviated.
D. Examine the effectiveness of
nursing interventions toward meeting
client outcomes.
873 In the oliguric phase of acute renal failure, the nurse should Pulmonary edema
. assess the client for:
874 Intractable pain is highly resistant to pain relief measures, so it is A client is admitted to the hospital
. important to promote comfort (A) during all activities. A client with intractable pain. What instruction
with intractable pain may develop drug tolerance and should the nurse provide the
dependence, but (B) is inappropriate for a UAP. Since intractable unlicensed assistive personnel (UAP)
pain is resistant to relief measures, (C) may not be possible. who is preparing to assist this client
Psychogenic pain (D) is a painful sensation that is perceived but with a bed bath?
has no known cause. A. Take measures to promote as much
Correct Answer: A comfort as possible.
B. Report any signs of drug addiction
to the nurse immediately.
C. Wait until the client's pain is gone
before assisting with personal care.
D. This client's pain will be difficult to
manage, since the cause is unknown.
875 intradermally injection Slow allergy testing because rapidly introducing an allergen could cause a
. diffusion is necessary during life-threatening allergic reaction in a sensitive client.
diagnostic
876 intravenous pyelogram
.

877 In which position would a patient Side-lying, knee to chest


. undergoing a lumbar punction be
placed?
878 Ischemia results from inadequate blood supply to the myocardial tissue and is reflected by
. T-wave inversion.
879 is high in fruits and vegetables, DASH diet
. moderate in low-fat dairy products,
and low in animal protein.
880 Isolation for 2 to 4 weeks is active tuberculosis.
. warranted for a client with
881 Isometric, not isotonic, exercises are strengthen muscles
. used to
882 Isoniazid (INH) 1. M. tuberculosis
. 1. Action: bactericidal - highly active 2. 5-15 mg/kg 300 mg QD (max)
against, what ? 3. 10-15 mg/kg
2. Dose: Adult - __to___ mg/kg ____ 4. Phenytoin
mg QD (max) 3. Pediatric - ___to ___ 5. hepatitis, GI distress, rash, peripheral neuropathy, elevated LFT's,
mg/kg mild CNS effects
4. Interactions:
5. Major adverse effects:
883 It is not necessary to keep the stoma all times, although a gauze bib can be used to protect the clothes
. covered at from mucus and to keep irritants from entering the stoma.
884 It may be done before or after exercise.
.

885 It typically results from bacteria ascending into the uterus from the vagina and is associated
. with prolonged labor.
886 I.V. dressing be changed once or twice per week or when it becomes soiled, loose, or wet.
.

887 ivh signs are temperature instability, nystagmus, apnea, bradycardia


.

888 ivh signs are neurologic signs such as hypotonia ( low muscle tone, often
. involving reduced muscle strength. ), lethargy,
889 ivh signs are decreased hematocrit, and increasing hypoxia. Seizures also may
. occur
890 Jack Donohue, a 62-year-old stock broker, attends his annual High LDL level----->LDL levels above
. physical appointment and indicates physical changes since his 100 mg/dl are considered high. The
last examination. He reports chest pain and palpitation during goal is to decrease the LDL level below
and after his morning jogs. Jack's family history reveals 100 mg/dl
includes coronary artery disease. His lipid profile reveals his
LDL level to be 122 mg/dl. Which of the following correctly
states the Jack's condition?
891 jaw thrust position herself at the client's head and
. rest her thumbs on his lower jaw, near
the corners of his mouth. She should
then grasp the angles of his lower jaw
with her fingers and lift the jaw
forward.
892 Karaya and Stomahesive are both effective agents for protecting the skin
. around a colostomy. They keep the skin
healthy and prevent skin irritation from
stoma drainage.
893 ketonuria is a sign of diabetic ketoacidosis.
.

894 Ketosis happens when your body resorts to fat for energy after your stored
. carbohydrates have been burned out
895 The kidneys are concentrating urine in response to low circulating volume, as evidenced
. by a urine output of less than 30
ml/hour. This indicates that increased
fluid replacement is needed.
896 Lactated Ringer's solution replaces lost sodium and corrects metabolic
. acidosis, both of which commonly
occur following a burn.
897 larngetctomy and bath The client is able to take tub baths with
. careful instruction on ways to avoid
slipping, the need to make sure the
water is no more than 6 inches deep,
and other safety measures.
898 Laryngeal stridor is characteristic of respiratory distress from inflammation
. and swelling after bronchoscopy. It
must be reported immediately.
899 laryngectomy client should be encouraged to participate in walking, golfing, and other moderate
. activities such as recreational sports.
900 A late complication of radiation therapy includes c) laryngeal necrosis.
. a) xerostomia. Explanation:
b) dysphasia. Late complications of radiation therapy
c) laryngeal necrosis. include laryngeal necrosis, edema, and
d) pain. fibrosis. Pain, xerostomia, and
dysphasia are not late complications of
radiation therapy..
901 Latent TB Infection Definition
. Definition 1. symptoms or radiographic evidence
1. Presence of the tubercle bacilli without...... of TB disease
Characteristics Characteristics
2. Positive _______result 2. TST
3. Negative _______ result 3. Chest X ray
4.No.... 4. symptoms or physical findings
5. LTBI can be treated with one drug, what is it ? suggestive of disease
5. isoniazid (INH) because the number
of TB bacilli is low (except HIV
positive pts. who need 2 drugs)
902 lateral surface of the left ventricle
.

903 Laurie Sharples, a 45-year-old teacher, is being seen at the Presbyopia


. ophthalmic group where you practice nursing due to a Correct
vision change. She's noticed an inability to see reading Explanation:
material as she had previously, needing to extend her arms Presbyopia is associated with aging and
to make the print readable. What is the term used to results in difficulty with near vision. People
describe her visual condition? with presbyopia hold reading material or
handwork at a distance to see it more clearly.
a) Emmetropia
b) Myopia
c) Presbyopia
d) Hyperopia
904 LBP is commonly associated with overuse or an injury to soft-tissue structures Muscle strain
. the
905 Leads I, aVL, V5, and V6 record electrical events on the lateral surface of the left ventricle.
.

906 Leads I, aVL, V5, and V6 record electrical events on the lateral surface of the left ventricle
.

907 Leads II, III, and aVF record electrical events on the inferior surface of the
. left ventricle.
908 Leads II, III, and aVF record electrical events on the inferior surface of the left ventricle
.

909 Leads V1 and V2 record electrical events on th anterior surface of the right ventricle and the
. anterior surface of the left ventricle.
910 Leads V1 and V2 record electrical events on the anterior surface of the right ventricle and the
. anterior surface of the left ventricle.
911. Leads V3 and V4 record electrical events in the septal region of the
left ventricle.
912 Leads V3 and V4 record electrical events in the septal region of the left ventricle.
.

913 Left untreated a HTN emergency can lead to what? A hypertensive emergency - severe HTN
. with acute impairment of one or more organ
systems (CNS, CV, renal) that can result in
irreversible organ damage.

CAN CAUSE:
-hypertensive encephalopathy
-Cerebral hemorrhage
-ARF
-MI
-HF with pulmonary edema
914 Legionellosis is a pneumonia caused by the bacterium
. Legionella pneumophilia that thrives in water
that is 95 to 115 F (35 to 46 C).
915 legionnaires' disease infection caused by gram neg BACTERIA
.

916 legionnaires' disease recieved from breathing in bacteria from aerosols or ac or


. water
917 legionnaires' disease signs are dry cough, myalgia, gi issues , diarrhea
. pneumonia and heart problems
918 Lethargy puts the posttonsillectomy side-lying position until he's fully awake is best.
. client at risk for aspirating blood from
the surgical wound. Therefore, placing
the client in the \
919 Letting go is the process beginning about 6 weeks postpartum when the mother
. may be preparing to go back to work.
920 The liver is vital in the synthesis of blood products that aid clotting. These products include fresh
. clotting factors, so when it's diseased or frozen plasma containing fibrinogen and cryoprecipitate, which
dysfunctional, as in hepatitis C, bleeding have most of the clotting factors.
occurs. Treatment consists of
administering
921 location breast cancer upper outer quad
.

922 Lomotil, a combination drug containing urine retention, blurred vision, constipation, palpitations,
. atropine, has anticholinergic properties. nervousness, and decreased sweating.
Common side effects include
923 Loop diuretics act on the Na+-K+-2Cl- symporter (cotransporter) in the thick ascending
. limb of the loop of Henle to inhibit sodium and chloride
reabsorption.
924 Loop diuretics block sodium reabsorption in the ascending loop of Henle, which
. promotes water diuresis. They also dilate renal vessels
925 Loss of urine when coughing occurs with stress incontinence
.

926 Lozenges will increase saliva production, aspiration.


. increasing the client's risk of
927 Lubricant jelly is a water-soluble agent Petroleum jelly is combustible; it isn't safe to use with oxygen.
. that the nurse can apply safely during The nurse shouldn't use sterile water or antibiotic ointment to
oxygen therapy to alleviate dryness of the alleviate dryness in the nares.
nares.
928 lung sounds
.

929 Lying flat and drinking fluids are headaches from spinal anesthesia.
. interventions for client's experiencing
930 Lymphatic obstruction is a blockage of lymph vessels that drain fluid from tissues throughout the body
. the and allow immune cells to travel where they are needed.
931 Lymphedema after breast cancer It is caused by the interruption or removal of lymph channels and
. nodes after axillary node dissection. Removal results
in less efficient filtration of lymph fluid and a
pooling of lymph fluid in the tissues on the affected
side.
932 A macrobiotic diet is high in whole-grain cereals, A male client with an infected wound tells the nurse
. vegetables, sea vegetables, beans, and vegetarian that he follows a macrobiotic diet. Which type of
soups, and the client needs essential amino acids to foods should the nurse recommend that the client
provide complete proteins to heal the infected select from the hospital menu?
wound. Although a macrobiotic diet contains no A. Low fat and low sodium foods.
source of animal protein, essential amino acids B. Combination of plant proteins to provide essential
should be obtained by combining plant (incomplete) amino acids.
proteins to provide complete (all essential amino C. Limited complex carbohydrates and fiber.
acids) proteins (B) for anabolic processes. (A, C, D. Increased amount of vitamin C and beta carotene
and D) do not provide the client with food choices rich foods.
consistent with a macrobiotic diet and protein
needs.
Correct Answer: B
933 macular degeneration for Loss of central vision
.

934 Magnesium is normally excreted by the kidneys. magnesium can accumulate and cause severe
. When the kidneys fail, neurologic problems
935 The main goal of nutritional therapy in acute renal protein catabolism.
. failure is to decrease
936 Maintaining a closed urinary drainage system is When assessing a client with an indwelling urinary
. important to prevent infection, so the most catheter, which observation requires the most
immediate priority is to close the clamp (B) to immediate intervention by the nurse?
reduce the risk for ascending microorganisms. If the A. The drainage tubing is secured over the siderail.
drainage tubing is secured over the siderail (A), B. The clamp on the urinary drainage bag is open.
urine will not be able to flow out of the bladder, so C. There are no dependent loops in the drainage
the nurse should next reposition the tubing. (C and tubing.
D) indicate correct care of the urinary drainage D. The urinary drainage bag is attached to the bed
system, so documentation of an intact system is the frame.
last intervention needed.
Correct Answer: B
937 The main use of CEA is as a tumor marker, especially with intestinal cancer
.

938 A major focus of nursing care after transsphenoidal prevention of and monitoring for a CSF leak. CSF
. hypophysectomy is leakage can occur if the patch or incision is
disrupted.
939 The major risk factor for cervical cancer is infection with the human papillomavirus (HPV) that
. is transmitted sexually.
940 A male client has just undergone a laryngectomy just prior to tube feeding
. and has a cuffed tracheostomy tube in place. When
initiating bolus tube feedings postoperatively, when
should the nurse inflate the cuff?
941 A male client has undergone insertion of a states that changes in the pulse and feelings of
. permanent pacemaker. When developing a dizziness are significant changes.
discharge teaching plan, the nurse writes a goal of,
"The client will verbalize symptoms of pacemaker
failure." Which behavior indicates that the goal has
been met? The client
942 A male client is to have an amputation. He is acutely Signs of sepsis--->If the client is acutely ill with a
. ill and diagnosed with a gangrenous limb and gangrenous limb, related fever, disorientation, and
related fever, disorientation, and electrolyte electrolyte imbalances, the nurse should monitor for
imbalances. Which of the following would be most signs of sepsis and circulation in the limb for any
important for the nurse to monitor in this client? changes such as severe pain, color changes, and lack
of peripheral pulses. It is crucial for the nurse to
inform the physician about the problems as they
occur or else the surgery may become an emergency.
Monitoring for signs of nausea and vomiting,
occurrence of allergic reactions, and reduced urine
output, although necessary, is not as crucial for the
client
943 A male client who has never smoked but has had COPD for the Adenocarcinoma
. past 5 years is now being assessed for cancer of the lung. The
nurse knows that he is most likely to develop which type of
lung cancer?
944 A male client with arterial peripheral vascular disease (PVD) Help the client to dangle his legs.
. complains of pain in his feet. Which instruction should the
nurse give to the unlicensed assistive personnel (UAP) to
quickly relieve the client's pain?
945 Many female Muslim clients are very modest and prefer to A male nurse is assigned to care for a
. receive personal care from another female because of their female Muslim client. When the nurse
religious and cultural beliefs. The most culturally sensitive offers to bathe the client, the client
response is for the male nurse to ask a female colleague to requests that a female nurse perform this
perform this task (B). (A and D) are less respectful of the task. How should the male nurse
client's cultural and spiritual preferences. (C) delays the respond?
client's care. A. May I ask your daughter to help you
Correct Answer: B with your personal hygiene?
B. I will ask one of the female nurses to
bathe you.
C. A staff member on the next shift will
help you.
D. I will keep you draped and hand you
the supplies as you need them.
946 Margaret Lawson, a 52-year grocery clerk, has been Increase serum calcium level--->The
. experiencing a decrease in serum calcium. She has undergone parathyroid glands secrete parathormone,
diagnostics and her physician proposes her calcium level which increases the level of calcium in
fluctuation is due to altered parathyroid function. What is the the blood when there is a decrease in the
role of parathormone? serum level
947 A maternal hemoglobin level below 11 g/dl is considered anemia
.

948 Measurements between the 5th and 95th percentiles are considered
. normal
949 mechanical ventilation cause decreased cardiac output
.

950 mechanical ventilation problems Other possible complications include


. incorrect ventilation, oxygen toxicity,
fluid imbalance, decreased cardiac
output, pneumothorax, infection, and
atelectasis.
951 Medical management of cardiac failure, whether it is right- Reducing cardiac workload
. sided or left-sided, demonstrates similar methodology.
Measures such as dietary modifications, lifestyle changes,
medications to reduce dyspnea and relieve anxiety, etc. are all
used with one primary intention. What is the primary goal in
the medical management of heart failure?
952 meds for crohns disease Corticosteroids, such as prednisone,
. reduce the signs and symptoms of
diarrhea, pain, and bleeding by
decreasing inflammation.
953 meds for crohns disease Antidiarrheals, such as diphenoxylate
. (Lomotil), combat diarrhea by decreasing
peristalsis.
954 Members of the Asian culture have high respect While caring for a child and mother from Cambodia,
. for others, especially those in positions of what action should the nurse implement to accommodate
authority. Extended family members need to be the clients' cultural needs?
included in the nursing care plan (A). Southeast A. Speak initially with the oldest family member to show
Asians do not necessarily refuse Western respect.
medications (B). Asians also believe that B. Realize that Southeast Asians may not take Western
touching strangers is not acceptable, medications.
particularly health professionals whom they C. Ask the husband to step out during the mother's pelvic
have not previously known, so the husband examination.
should be allowed to remain with his wife D. Tell the family that planning health care is provided in
during the pelvic exam (C). Provided that the private with the client.
presence of other family members is not
harmful to the client's well-being, (D) is not
correct.
Correct Answer: A
955 men chlyamydia A white/cloudy and watery discharge from the penis that
. may stain underwear; a burning sensation and/or pain
when passing urine; pain and swelling in the testicles.
956 men gonorrhea Painful or frequent urination or urethritis.
. Anal itching, discomfort, bleeding, or discharge.
Sore throat (rare).
Pinkeye (conjunctivitis) (rare).
957 men gonorrhea Irregular menstrual bleeding.
. Lower abdominal (belly) pain.
Fever and general tiredness.
Swollen and painful glands at the opening of the vagina
(Bartholin glands).
Painful sexual intercourse.
Sore throat (rare).
Pinkeye (conjunctivitis) (rare).
958 men gonorrhea Abnormal discharge from the penis (clear or milky at
. first, and then yellow, creamy, and excessive, sometimes
blood-tinged).
959 Meperidine is contraindicated in clients with 2 days and in those for whom large daily doses (more
. acute pain lasting more than than 600 mg) are needed. It would be inappropriate to
urge the client to take the acetaminophen and codeine to
prevent addiction.
960 metabolic
.

961 metabolic
.

962 metabolic acidosis


.

963 metabolic alkalosis


.

964 Metabolic alkalosis can cause to shift into the cells, resulting in a decrease of serum potassium.
. potassium
965 Metabolic alkalosis can cause serum potassium
. potassium to shift into the cells,
resulting in a decrease of
966 Methylxanthine agents inhibit rather adenosine receptors
. than stimulate
967 Metoclopramide hydrochloride esophageal sphincter tone and facilitates gastric emptying; both
. (Reglan) increases actions reduce the incidence of reflux.
968 metoclopramide Other common include diarrhea (not constipation) and nausea. Occasionally
. adverse effects transient hypertension.
969 Metoclopramide (Reglan), which is gastroesophageal reflux disease, acts by stimulating gastric
. prescribed to treat motility and reducing the volume of gastric reflux.
970 Mexiletine, an antiarrhythmic, is used refractory ventricular arrhythmias; it doesn't cause hypoglycemia.
. to treat
971 mi T-wave inversion
. ST-segment elevation
Pathologic Q-wave
972 milk is alkaline
.

973 Mitral insufficiency has a high-pitched, blowing murmur at the apex.


.

974 Mitral stenosis causes a diastolic, rumbling, low-pitched murmur audible at the apex
.

975 Mitral stenosis has a low-pitched rumbling murmur heard at the apex.
.

976 Mitral stenosis, or severe narrowing of the mitral valve, stenotic valve, increasing pressure in the left
. impedes blood flow through the atrium and pulmonary circulation. These
problems may lead to low cardiac output,
pulmonary hypertension, edema, and right-
sided (not left-sided) heart failure.
977 Mobilization and ambulation increase oxygen use, so it is Which client assessment data is most
. most important to assess the client's respiratory rate important for the nurse to consider before
(A)before ambulation to determine tolerance for activity. ambulating a postoperative client?
(B, C, and D) are also important, but are of lower priority A. Respiratory rate.
than (A). B. Wound location.
Correct Answer: A C. Pedal pulses.
D. Pain rating.
978 Moderate peripheral artery disease would yield a score of 0.41 to 0.70.
.

979 Moist heat to the flank area is helpful when renal colic occurs, but it is less necessary as
. pain is lessened.
980 Molar mass (definition/calculating) -Mass (in grams) of 1 mol of substance
. -Molar mass of compound found by adding
atomic masses of each element
981 Mole conversions...1 mol= 6.022 x 10 representative particles
. (Avogadro's #)
22.4 L
Atomic mass in grams
Molar mass of an element or compound
982 Molecular formula Gives composition of the molecules present/
. can be simplified to empirical formula
983 molecular formula gives the composition of the molecules that
. are present
984 Mole (definition) Amount of matter
.

985 More severe strains may cause spasms along with more intense pain and possible
. swelling.
986 morphine works by lowers resistance, reduces cardiac workload,
. and decreases myocardial oxygen demand
987 The most accurate method for determining the presence of pulse oximeter value or arterial blood gas
. hypoxia is to evaluate the values
988 The most beneficial nursing intervention is to use A male client has a nursing diagnosis of
. nonjudgmental reflective listening techniques, to allow the "spiritual distress." What intervention is best
client to feel comfortable expressing his concerns (D). (A for the nurse to implement when caring for
and B) are not therapeutic. The client should be consulted this client?
before implementing (C). A. Use distraction techniques during times of
Correct Answer: D spiritual stress and crisis.
B. Reassure the client that his faith will be
regained with time and support.
C. Consult with the staff chaplain and ask
that the chaplain visit with the client.
D. Use reflective listening techniques when
the client expresses spiritual doubts.
989 The most common cancers that elevate CEA are in the colon and rectum. Others: cancer of the
. pancreas, stomach, breast, lung, and certain
types of thyroid and ovarian cancer.
990 The most common causes of primary adrenocortical autoimmune destruction (70%) and
. insufficiency are tuberculosis (20%).
991 The most common complication after an inguinal hernia repair is the inability to void,
. especially in men.
992 The most common site of hemorrhage is the periventricular subependymal germinal
. matrix, where there is a rich blood supply
and where the capillary walls are thin and
fragile.
993. The most common toxicities from NSAIDs are gastrointestinal disorders
(nausea, epigastric pain, ulcers,
bleeding, diarrhea, and
constipation).
994. Most HTN meds work to influence what? vascular resistance -
determined by the size of the
vessel
995. The most important factor in performing a physical assessment is Which technique is most
following a consistent and systematic technique (C) each time an important for the nurse to
assessment is performed to minimize variation in sequence which may implement when performing a
increase the likelihood of omitting a step or exam of an isolated area. physical assessment?
The method of completing a physical assessment (A, B, and D) may be A. A head-to-toe approach.
at the discretion of the examiner, but a consistent sequence by the B. The medical systems model.
examiner provides a reliable method to ensure thorough review of the C. A consistent, systematic
clients' history, complaints, or body systems. approach.
Correct Answer: C D. An approach related to a
nursing model.
996. Most institutions use tubing especially for platelets instead of tubing blood and blood product
for
997. The mother is commonly unable to consume enough protein, calcium, and iron to
supply her needs and those of
the fetuses.
998. Mouth care should be provided after NG tube removal. Auscultating
and palpating the abdomen
should have been done before
tube removal.
999. The multiple gestation client is at risk for preterm labor because uterine
distention, a major factor
initiating preterm labor, is more
likely with a twin gestation
1000 multiple sclerosis who has an impaired peripheral sensation should hot because they cant feel well
. avoid and cold because of it
constrictive behavior
1001 The murmur in aortic insufficiency is high-pitched and blowing and
. is heard at the third or fourth
intercostal space at the left
sternal border.
1002 Muscle spasms are not seen in hypokalemia.
.

1003 Muscle tears or ruptures cause severe , impede the ability to move or walk, and are usually
. pain accompanied by swelling and bruising.
1004 A nasal cannula can't deliver oxygen 44%.
. concentrations above
1005 A nasal drip pad is not needed after nasal packing.
. removal of
1006 nasal surgery nasal packing Checking the nares for ulcerations is not necessary.
.

1007 nasal surgery, the client has packing in dries the oral mucous membranes. Frequent mouth care is
. the nose Mouth-breathing necessary for comfort and to combat the anorexia associated
with the taste of blood and loss of the sense of smell.
1008 necrotizing pancreatitis TPN is enteral feedings are contraindicated. .
. considered if
1009 A negative pressure room, or an area recirculation is unavoidable.
. that exhausts room air directly outside
or through HEPA filters, should be used
if
1010 A neonate born at 37 weeks' gestation some cartilage in the ear lobes, fine and fuzzy hair, scant to
. will have moderate rugae in the scrotum, and a breast nodule diameter of
4m
1011. Neonates born before 36 weeks' only an anterior transverse crease on the soles of the feet.
gestation will hav
1012 nephrotic syndrome has what type of hypocalcemia.
. calcium
1013 A new client has been admitted with Jugular vein distention--->When the right ventricle cannot
. right-sided heart failure. The nurse effectively pump blood from the ventricle into the pulmonary
knows to look for which of the following artery, the blood backs up into the venous system and causes
assessment findings when assessing this jugular venous distention and congestion in the peripheral
client? tissues and viscera. All the other choices are symptoms of left-
sided heart failure.
1014 New laryngectomy clients may find air- cool and dry at first so they should avoid such environments
. conditioning too
1015 Nitrous oxide is a potent vasodilator because vasodilators decrease afterload, which decreases the
. released by the vascular endothelium in workload of the LV
response to the body's compensatory
mechanisms, why is this a good thing for
patients with HF?
1016 no abnormal regional lymph nodes N0,
.

1017 no evidence of distant metastasis is M0


. classified as
1018 No evidence of primary tumor T0
.

1019 A nonresponsive client has a nasogastric Ausculate lung sounds every 4 hours
. tube to low intermittent suction due to
gastrointestinal bleeding. It is most
important for the nurse to
1020 normal findings in tpn Glycosuria is to be expected during the first few days of therapy
. until the pancreas adjusts by secreting more insulin.
1021 normal findings in tpn A gradual weight gain is to be expected as the client's nutritional
. status improves.
1022 The normal I:E ratio is 1:2, meaning that expiration takes twice as long as inspiration
.

1023 normally liver transforms ammoina to urea which the kidney excrete
.

1024 Normally, pressure in the anterior 10 to 20 mm Hg


. chamber of the eye remains relatively
constant at
1025 Normal oral feedings are resumed as NG tube is removed, usually within 10 days after surgery.
. soon as the
1026 normal ration is transverse is 1 and anterioposterior is 2
.

1027 Normal serum albumin is administered to reduce ascites


.

1028 Normal urine pH is 4.5 to 8; therefore, a urine pH of 3.0 is


. abnormal and requires further
investigation.
1029 The normal uterus is only able to distend to a certain point labor may be initiated.
. and when that point is reached,
1030 Nose drops are not instilled with packing in place.
.

1031 A notched T-wave may indicate pericarditis in an adult client.


.

1032 note question with pt with spinal anesthesia and 4000 ml of note it says patient is getting irrigation it
. isotonic bladder irrigation dosent say that the bag is full
1033 note that steroid use mimics cushing disease and cushings ostoprorosis because takes calcium from
. cause the bone so high calcium
1034 note with htn question also check compliance its leading cause of
. complications
1035 NPH is an example of which type of insulin? Intermediate acting
.

1036 nsaids Voltaren Gel and Flector Patch


. Diclofenac Plus Misoprostol (Arthrotec)
Diflunisal (Dolobid)
Etodolac
Indomethacin (Indocin)
Ketorolac (Toradol), available IM or IV
Mefenamic acid (Ponstel)
Meclofenamate
Meloxicam (Mobic)
Nabumetone
Piroxicam (Feldene)
Sulindac (Clinoril)
Tolmetin (Tolectin
1037 A nurse and a nursing student are caring for a client with "Ask the client to hold the breath while
. pericarditis and perform the physical assessment together. you auscultate; the pericardial friction rub
The client has a pericardial friction rub audible on will continue, while the pleural friction rub
auscultation. When the nurse and student leave the room, will stop.
the student asks how to distinguish a pericardial from a
pleural friction rub. The nurse's best response is which of
the following
1038 The nurse assesses a postoperative client. Oxygen Obtain IV fluids for infusion per protocol.
. is being administered at 2 L/min and a saline lock
is in place. Assessment shows cool, pale, moist
skin. The client is very restless and has scant
urine in the urinary drainage bag. What
intervention should the nurse implement first?
1039 A nurse caring for a client with deep vein d) Chest pain and dyspnea
. thrombosis must be especially alert for
complications such as pulmonary embolism.
Which findings suggest pulmonary embolism?

a) Nonproductive cough and abdominal pain


b) Hypertension and lack of fever
c) Bradypnea and bradycardia
d) Chest pain and dyspnea
1040 The nurse evaluates that discharge teaching for a C
. patient hospitalized with pneumonia has been
most effective when the patient states which of
the following measures to prevent a relapse?
A) "I will seek immediate medical treatment for
any upper respiratory infections."
B) "I will increase my food intake to 2400 calories
a day to keep my immune system well."
C) "I should continue to do deep-breathing and
coughing exercises for at least 6 weeks."
D) "I must use home oxygen therapy for 3
months and then will have a chest x-ray to
reevaluate."
1041 A nurse explains to a client that she will Insulin is absorbed more rapidly at abdominal
. administer his first insulin dose in his abdomen. injection site then others
How does absorption at the abdominal site
compare with absorption at other sites?
1042 The nurse identifies a flail chest in a trauma Paradoxic chest movement occurs during respiration
. patient when:
1043 The nurse initiates neuro checks for a client who Change in level of consciousness
. is at risk for neurologic compromise. Which
manifestation typically provides the first
indication of altered neuro function?
1044 The nurse instructs the unliscensed nursing Use a soft toothbrush to brush the client's teeth after
. personnel (UAP) on how to provide oral hygiene each meal.
for a client who cannot perform this task for
himself. Which of the following techniques should
the nurse tell the UAP to incorporate into the
client's daily care?
1045 A nurse in the free clinic is assessing a patient Both eyes have purulent discharge.
. diagnosed with conjunctivitis who has presented Explanation:
for a follow-up examination. What finding would After effective treatment for conjunctivitis, the
lead the nurse to conclude that the treatment for patient's eye pain should be relieved, preauricular
conjunctivitis was effective? adenopathy should be decreased or completely
resolved, and purulent discharge resolved. Further
a) The patient's eye pain is unrelieved. therapy is needed for the patient who has purulent
b) The patient is photophobic. drainage in both eyes. Photophobia is not associated
c) Preauricular adenopathy is decreased. with conjunctivitis.
d) Both eyes have purulent discharge.
1046 The nurse in the preoperative holding area keeps To minimize oxygen consumption
. a client with gastric bleeding in a dimly lit
environment with one family member present.
What is the primary rationale for these nursing
interventions?
1047 The nurse is administering a nystatin suspension Hold the medication in the mouth for a few minutes
. (Mycostatin) for stomatitis. Which instruction before swallowing it.
will the nurse provide to the client when
administering this medication?
1048 The nurse is admitting a 55-year-old male patient Flashing lights in the visual field
. diagnosed with a retinal detachment in his left eye. Explanation:
While assessing this patient what characteristic Flashing lights in the visual field are a common
symptom would the nurse expect to find? symptom of retinal detachment. Patients may also
report spots or floaters or the sensation of a curtain
a) Sudden eye pain being pulled across the eye. Retinal detachment isn't
b) Loss of color vision associated with eye pain, loss of color vision, or
c) Colored halos around lights colored halos around lights.
d) Flashing lights in the visual field
1049 The nurse is analyzing (B) data to establish an Prior to administering a newly prescribed medication
. individualized nursing diagnosis, such as, "Risk to a client, the nurse reviews the adverse effects of
for injury related to side effects of drugs." This the medication listed in a drug reference guide and
analysis is based on assessment (A) and guides the determines the priority risks to the client. While
planning and implementation (C) of care, such as performing this action, the nurse is engaged in which
the decision to monitor the client frequently. (D) step of the nursing process?
provides the nurse with information about the A. Assessment.
effectiveness of the plan of care. B. Analysis.
Correct Answer: B C. Implementation.
D. Evaluation.
1050 The nurse is assessing a 75-year-old male client for Infection
. symptoms of hyperglycemia. Which symptom of
hyperglycemia is an older adult most likely to
exhibit?
1051 A nurse is assessing a client after a thyroidectomy. Tetany-->Tetany may result if the parathyroid glands
. The assessment reveals muscle twitching and are excised or damaged during thyroid surgery.
tingling, along with numbness in the fingers, toes, Hemorrhage is a potential complication after thyroid
and mouth area. The nurse should suspect which surgery but is characterized by tachycardia,
complication? hypotension, frequent swallowing, feelings of
fullness at the incision site, choking, and bleeding.
Thyroid storm is another term for severe
hyperthyroidism not a complication of
thyroidectomy. Laryngeal nerve damage may occur
postoperatively, but its signs include a hoarse voice
and, possibly, acute airway obstructio
1052 The nurse is assessing a client who comes to the d) Dyspnea and wheezing
. clinic for care. Which findings in this client
suggest bacterial pneumonia?

a) Nonproductive cough and normal temperature


b) Sore throat and abdominal pain
c) Hemoptysis and dysuria
d) Dyspnea and wheezing
1053 The nurse is assessing a client who presents with Serum amylase and lipase levels are twice their
. jaundice. Which assessment finding is the most normal values
significant indication that further follow-up is
needed?
1054 The nurse is assessing a male client with acute While the nurse is taking the client's blood pressure,
. pancreatitis. Which finding requires the most he has a carpal spasm.
immediate intervention by the nurse?
1055 A nurse is assisting an 82-year-old client with Upper torso
. ambulation and is concerned that the client may
fall. What area contains the older person's center
of gravity?
1056 The nurse is awaiting results of cardiac Troponin T and I---->After myocardial injury, these
. biomarkers for a patient with severe chest pain. biomarkers rise early (within 3 to 4 hours), peak in 4
The nurse would identify which cardiac to 24 hours, and remain elevated for 1 to 3 weeks.
biomarker as remaining elevated the longest when These early and prolonged elevations may make very
myocardial damage has occurred early diagnosis of acute myocardial infarction (MI)
possible and allow for late diagnosis in patients who
have delayed seeking care for several days after the
onset of acute MI symptoms. CK-MB returns to
normal within 3 to 4 days. Myoglobin returns to
normal within 24 hours. BNP is not considered a
cardiac biomarker. It is a neurohormone that
responds to volume overload in the heart by acting as
a diuretic and vasodilator.
1057 The nurse is caring for a 73-year-old patient who D
. underwent a left total knee arthroplasty. On the
third postoperative day, the patient complains of
shortness of breath, slight chest pain, and that
"something is wrong." Temperature is 98.4o F,
blood pressure 130/88, respirations 36, and oxygen
saturation 91% on room air. Which of the
following actions should the nurse take first?
A) Notify the physician.
B) Administer a nitroglycerine tablet sublingually.
C) Conduct a thorough assessment of the chest
pain.
D) Sit the patient up in bed as tolerated and apply
oxygen.
1058 The nurse is caring for a 73-year-old patient who C
. underwent a left total knee arthroplasty. On the
third postoperative day, the patient complains of
shortness of breath, slight chest pain, and that
"something is wrong." Temperature is 98.4o F,
blood pressure 130/88, respirations 36, and oxygen
saturation 91% on room air. Which of the
following should the nurse first suspect as the
etiology of this episode?
A) New onset of angina pectoris
B) Septic embolus from the knee joint
C) Pulmonary embolus from deep vein thrombosis
D) Pleural effusion related to positioning in the
operating room
1059 The nurse is caring for a client who is 1 day post Increase the client's oxygen flow rate.
. acute myocardial infarction. The client is receiving
oxygen at 2 L/min via nasal cannula and has a
peripheral saline lock. The nurse notes that the
client is having eight PVCs per minute. Which
intervention should the nurse implement first?
1060 A nurse is caring for a client who is exhibiting Assess the client's level of pain and administer
. signs and symptoms characteristic of a myocardial prescribed analgesics.
infarction (MI). Which statement describes
priorities the nurse should establish while
performing the physical assessment?
1061 A nurse is caring for a client who is exhibiting a) Assess the client's level of pain and administer
. signs and symptoms characteristic of a prescribed analgesics.
myocardial infarction (MI). Which statement Explanation:
describes priorities the nurse should establish The cardinal symptom of MI is persistent, crushing
while performing the physical assessment? substernal pain or pressure. The nurse should first assess
a) Assess the client's level of pain and the client's pain and prepare to administer nitroglycerin
administer prescribed analgesics. or morphine for pain control. The client must be
b) Ensure that the client's family is kept medically stabilized before pulmonary artery
informed of his status. catheterization can be used as a diagnostic procedure.
c) Prepare the client for pulmonary artery Anxiety and a feeling of impending doom are
catheterization. characteristic of MI, but the priority is to stabilize the
d) Assess the client's level of anxiety and client medically. Although the client and his family
provide emotional support. should be kept informed at every step of the recovery
process, this action isn't the priority when treating a
client with a suspected MI.
1062 The nurse is caring for a client who recently b) keeping his airway patent.
. underwent a tracheostomy. The first priority
when caring for a client with a tracheostomy is:

a) helping him communicate.


b) keeping his airway patent.
c) encouraging him to perform activities of
daily living.
d) preventing him from developing an infection.
1063 The nurse is caring for a client with a chest Tidaling of water in water seal chamber
. tube to water seal drainage that was inserted 10
days ago because of a ruptured bullae and
pneumothorax. Which finding should the nurse
report to the healthcare provider before the
chest tube is removed?
1064 The nurse is caring for a client with a fractured Deep, unrelenting pain in the right arm
. right elbow. Which assessment finding has the
highest priority and requires immediate
intervention?
1065 A nurse is caring for a client with an The round head
. intracapsular hip fracture. Identify the area
where this client's fracture occurred
1066 The nurse is caring for a client with atrial "Your atrial chambers may contain blood clots now, so
. fibrillation. The client's symptoms started you must take an anticoagulant for a few weeks before
about 1 week ago, but he is just now seeking the cardioversion."---> Because of the high risk of
medical attention. The client asks the nurse embolization of atrial thrombi, cardioversion of atrial
why he has to wait several weeks before the fibrillation that has lasted longer than 48 hours should
cardioversion takes place. The best answer by be avoided unless the client has received warfarin for at
the nurse is which of the following? least 3 to 4 weeks prior to cardioversion.
1067 The nurse is caring for a client with chest a) Impaired gas exchange
. trauma. Which nursing diagnosis takes highest
priority?

a) Impaired gas exchange


b) Anxiety
c) Decreased cardiac output
d) Ineffective cardiopulmonary tissue perfusion
1068 The nurse is caring for a client with pneumonia. As part of prescribed therapy, d) Eight to ten
. the client must use a bedside incentive spirometer to promote maximal deep
breathing. The nurse checks to make sure the client is using the spirometer
properly. During each waking hour, the client should perform a minimum of how
many sustained, voluntary inflation maneuvers?

a) One to two
b) Three to four
c) Five to seven
d) Eight to ten
1069 The nurse is caring for a critically ill client with cirrhosis of the liver who has a Decreased serum
. nasogastric tube draining bright red blood. The nurse notes that the client's ammonia
serum hemoglobin and hematocrit are decreased. What additional change in lab
data should the nurse expect?
1070 The nurse is caring for an 82-year-old male client who has come to the clinic for a Loss of arterial
. yearly physical. When assessing the client, the nurse notes the blood pressure elasticity
(BP) is 140/93. The nurse knows that in older clients what happens that may
elevate the systolic BP?
1071 A nurse is caring for an elderly female client with osteoporosis. When teaching Bone Fracture
. the client, the nurse should include information about which major
complication?
1072 The nurse is caring for a patient admitted to the hospital with pneumonia. Upon A
. assessment, the nurse notes a temperature of 101.4 F, a productive cough with
yellow sputum, and a respiratory rate of 20. Which of the following nursing
diagnosis is most appropriate based upon this assessment?
A) Hyperthermia related to infectious illness
B) Ineffective thermoregulation related to chilling
C) Ineffective breathing pattern related to pneumonia
D) Ineffective airway clearance related to thick secretions
1073 The nurse is completing an admission interview for a client with Parkinson "Have you ever been
. disease. Which question will provide additional information about manifestations 'frozen' in one spot,
the client is likely to experience? unable to move?
1074 the nurse is conducting an admission history of a client admitted with a fracture. Prednisone
. The nurse recognizes that which of the client's medications placed the client at (Deltasone)
risk for fractures
1075 The nurse is counseling a healthy 30-year-old female client regarding cross country skiing
. osteoporosis prevention. Which activity would be most beneficial in achieving the
client's goal of osteoporosis prevention?
1076 The nurse is developing a teaching plan for a client with asthma. Which teaching c) Take prescribed
. point has the highest priority? medications as
scheduled.
a) Avoid contact with fur-bearing animals.
b) Change filters on heating and air conditioning units frequently.
c) Take prescribed medications as scheduled.
d) Avoid goose down pillows.
1077 The nurse is discussing a treatment plan for mononucleosis with an adolescent. Avoid contact sports
. The nurse emphasizes that the client mus and vigorous
exercise for 2 to 4
weeks
1078 A nurse is discussing pharmacologic Increase in myocardial contractility-->A positive inotrope is a
. therapy used in the treatment of coronary medication that increases myocardial contractility (force of
vascular disease with a nursing student. contraction). Medications that increase the heart rate are
The nurse would be correct in identifying positive chronotropes. Negative chronotropic medications
the use of a positive inotrope as having decrease the heart rate. Negative chronotropes decrease
which of the following functions? myocardial contractility
1079 A nurse is discussing pharmacologic Increase in myocardial contractility--> A positive inotrope is
. therapy used in the treatment of coronary a medication that increases myocardial contractility (force of
vascular disease with a nursing student. contraction). Medications that increase the heart rate are
The nurse would be correct in identifying positive chronotropes. Negative chronotropic medications
the use of a positive inotrope as having decrease the heart rate. Negative chronotropes decrease
which of the following functions? myocardial contractility
1080 A nurse is explaining the action of insulin Beta cells of the pancreas-->The beta cells of the pancreas
. to a client with diabetes mellitus. During secrete insulin. The adenohypophysis, or anterior pituitary
client teaching, the nurse reviews the gland, secretes many hormones, such as growth hormone,
process of insulin secretion in the body. prolactin, thyroid-stimulating hormone, corticotropin,
The nurse is correct when she states that follicle-stimulating hormone, and luteinizing hormone, but
insulin is secreted from the: not insulin. The alpha cells of the pancreas secrete glucagon,
which raises the blood glucose level. The parafollicular cells
of the thyroid secrete the hormone calcitonin, which plays a
role in calcium metabolism.
1081 The nurse is giving preoperative "Let me show you the method of turning I will use after
. instructions to a 14-year-old female client surgery."
scheduled for surgery to correct a spinal
curvature. Which statement by the client
best demonstrates learning has taken
place?
1082 The nurse is interviewing a client who is Review the medication actions and interactions.
. taking interferon-alfa-2a (Roferon-A) and
ribavirin (Virazole) combination therapy
for hepatitis C. The client reports
experiencing overwhelming feelings of
depression. What action should the nurse
implement first?
1083 The nurse is monitoring a client after an Assess vital signs
. above-the-knee amputation and notes that
blood has saturated through the distal part
of the dressing. The nurse should
immediately
1084 The nurse is observing an unlicensed Oral care
. assistive personnel (UAP) who is
performing morning care for a bedfast
client with Huntington disease. Which care
measure is most important for the nurse to
supervise?
1085 The nurse is performing hourly neurologic A unilateral pupil that is dilated and nonreactive to light
. checks for a client with a head injury.
Which new assessment finding warrants
the most immediate intervention by the
nurse?
1086 A nurse is planning care for a client in Decreased cardiac output
. acute addisonian crisis. Which nursing
diagnosis should receive the highest
priority?
1087 The nurse is planning care for a client with Prevent infection
. diabetes mellitus who has gangrene of the
toes to the mid-foot. Which goal should be
included in this client's plan of care?
1088 The nurse is planning the care for a client B.
. who is admitted with the syndrome of Quiet environment
inappropriate antidiuretic hormone
secretion (SIADH). Which interventions
should the nurse include in this client's plan C.
of care? (Select all that apply.) Deep tendon reflex assessments
D.
Neurologic checks

E.
Daily weights
1089 The nurse is preparing a 45-year-old female Attend an ostomy support group within 2 weeks.
. client for discharge from a cancer center
following ileostomy surgery for colon cancer.
Which discharge goal should the nurse
include in this client's discharge plan?
1090 The nurse is preparing a presentation for a Insulin production insuficent
. group of adults at a local community center
about diabetes. Which of the following would
the nurse include as associated with type 2
diabetes?
1091 The nurse is preparing a teaching plan for A teacher whose blood glucose levels average 126 daily
. healthy adults. Which individual is most with oral antidiabetic drugs
likely to maintain optimum health?
1092 The nurse is preparing to examine the Supine with knees flexed
. abdomen of a client complaining of a change
in his bowel pattern. The nurse would place
the client in which position?
1093 The nurse is providing care for a client who Ensure that a set of wrenches are kept in close proximity
. has had a cervical cord injury. Following
reduction of the cervical fracture, a halo vest
is placed to maintain realignment of the
spinal canal. What intervention is needed to
ensure client safety while the halo vest is in
place?
1094 The nurse is providing care to a client Administering the prescribed analgesic-->After an
. following a knee arthroscopy. Which of the arthroscopy, the client's entire leg is elevated without
following would the nurse expect to include flexing the knee. A cold pack is placed over the bulky
in the client's plan of care? dressing covering the site where the arthroscope was
inserted. A prescribed analgesic is administered as
necessary. The client is allowed to resume his or her usual
diet as tolerated.
1095 The nurse is providing care to a client who Signs and symptoms of bleeding
. has had a percutaneous liver biopsy. The
nurse would monitor the client for which of
the following?
1096 A nurse is providing dietary instructions to a Consuming a low carb- high protein diet and avoiding
. client with hypoglycemia. To control fasting--->To control hypoglycemic episodes, the nurse
hypoglycemic episodes, the nurse should should instruct the client to consume a low-carbohydrate,
recommend: high-protein diet, avoid fasting, and avoid simple sugars.
Increasing saturated fat intake and increasing vitamin
supplementation wouldn't help control hypoglycemia.
1097 A nurse is providing instructions for the Sleep with the head of bed elevated.
. client with chronic rhinosinusitis. The nurse Explanation:General nursing interventions for chronic
accurately tells the client: rhinosinusitis include teaching the client how to provide
a) Sleep with the head of bed elevated. self-care. These measures include elevating the head of the
b) Do not perform saline irrigations to the bed to promote sinus drainage. Caffeinated beverages and
nares. alcohol may cause dehydration. Saline irrigations are used
c) You may drink 1 glass of alcohol daily. to eliminate drainage from the sinuses
d) Caffeinated beverages are allowed.
1098 The nurse is proving discharge instruction for a Your family and friends may want to take a CPR class--
. patient with a new arrhythmia. Which of the >Having friends and family learn to take a pulse and
following should the nurse include? perform CPR will help patients to manage their
condition. Antiarrhythmic medication should be taken
on time. Lightheadedness and dizziness are symptoms
which should be reported to the provide
1099 The nurse is receiving report from PACU about If the client's wound is infected
. a client with a Penrose drain who is to be
admitted to the surgical nursing unit. Before
choosing a room for this client, which
information is most important for the nurse to
obtain?
1100. nurse is receiving report from the emergency 40 and 60 years
room regarding a new client being admitted to
the medical-surgical unit with a diagnosis of
peptic ulcer disease. The nurse expects the age
of the client will be between
1101. A nurse is reviewing self-care measures for a "I have my wife look at the soles of my feet every day"
client with peripheral vascular disease. Which
statement indicates proper self-care measures
1102. A nurse is reviewing self-care measures for a a) "I have my wife look at the soles of my feet each
client with peripheral vascular disease. Which day."
statement indicates proper self-care measures? Explanation:
a) "I have my wife look at the soles of my feet A client with peripheral vascular disease should examine
each day." his feet daily for redness, dryness, or cuts. If a client isn't
b) "I like to soak my feet in the hot tub every able to do this examination on his own, then a caregiver
day." or family member should help him. A client with
c) "I stopped smoking and use only chewing peripheral vascular disease should avoid hot tubs
tobacco." because decreased sensation in the feet may make him
d) "I walk only to the mailbox in my bare feet." unable to tell if the water is too hot. The client should
always wear shoes or slippers on his feet when he is out
of bed to help minimize trauma to the feet. Any type of
nicotine, whether it's from cigarettes or smokeless
tobacco, can cause vasoconstriction and further decrease
blood supply to the extremities.
1103. The nurse is reviewing the routine medications An anticholinergic with a side effect of pupillary dilation
taken by a client with chronic angle closure
glaucoma. Which medication prescription
should the nurse question?
1104. The nurse is taking a health history from a Chest pain, weight gain, fatigue-->Chest pain, weight
client admitted with the medical diagnosis of gain, fatigue, dizziness, ascites, and confusion are all
cardiovascular disease (CVD). Identify which symptoms of CVD. Rash, extra-ocular eye movements,
of the following symptoms indicate CVD. ecchymosis, and petechiae are not usually indicative of
CVD.
1105. A nurse is teaching about ischemic stroke Moderate amounts of low-fat dairy products
prevention to a community group and
emphasizes that control of hypertension, which
is the major risk factor for stroke, is key to
prevention. Ways to control hypertension
include the Dietary Approaches to Stop
Hypertension (DASH) diet. This diet includes
which of the following?
1106. A nurse is teaching a client with chronic Use diaphragmatic breathing.
bronchitis about breathing exercises. Which
instruction should the nurse include in the
teaching?
1107. The nurse is teaching a client with chronic bronchitis c) Use diaphragmatic breathing.
about breathing exercises. Which instruction should the
nurse include in the teaching?

a) Make inhalation longer than exhalation.


b) Exhale through an open mouth.
c) Use diaphragmatic breathing.
d) Use chest breathing.
1108. A nurse is teaching a client with gastritis about the need "Caffeine stimulates the central nervous
to avoid the intake of caffeinated beverages. The client system and thus gastric activity and secretions,
asks why this is so important. Which of the following which need to be minimized to promote
explanations from the nurse would be most accurate? recovery."
1109. A nurse is teaching an elderly client about good bowel "I need to use laxatives regularly to prevent
habits. Which statement by the client indicates to the constipation"
nurse that additional teaching is required?
1110. A nurse is teaching a patient with glaucoma how to Conjunctival sac
administer eyedrops to achieve maximum absorption. Correct
Where should the nurse teach the patient to instill the Explanation:
eyedrops? The nurse should instill the eyedrops into the
conjunctival sac, where absorption can best
a) Sclera take place. The pupil permits light to enter the
b) Vitreous humor eye. The sclera maintains the eye's shape and
c) Conjunctival sac size. The vitreous humor maintains the retina's
d) Pupil placement and the shape of the eye.
1111. The nurse is to administer a cyclic feeding through a Elevate the head of bed to 45 degrees
gastric tube. It is most important for the nurse to
1112. The nurse knows that clients taking diuretics must be Presence of a U wave
assessed for the development of hypokalemia, and that
hypokalemia will create changes in the client's normal
ECG tracing. Which ECG change would be an expected
finding in the client with hypokalemia?
1113. The nurse knows that normal lab values expected for an Urinalysis reveals slight protein in the urine
adult may vary in an older client. Which data would the and bacteriuria with pyuria.
nurse expect to find when reviewing laboratory values of
an 80-year-old man who is in good health overall?
1114. The nurse notes that a client who is scheduled for Anorexia and vomiting for the past 2 days
surgery the next morning has an elevated blood urea
nitrogen (BUN) level. Which client condition is most
likely to have contributed to this finding?
1115. The nurse notes that the only ECG for a 55-year-old Call for an ECG to be performed immediately.
male client scheduled for surgery in 2 hours is dated 2
years ago. The client reports that he has a history of
"heart trouble," but has no problems at present.
Hospital protocol requires that those over 50 years of age
have a recent ECG prior to surgery. What nursing action
would be best for the nurse to implement?
1116. The nurse notes tidaling of the water level in the tube Continue to monitor this normal finding
submerged in the water-seal chamber in a patient with
closed chest tube drainage. The nurse should
1117. The nurse observes dry mucous membranes in a client Provide frequent mouth care
who is receiving tube feedings after an oral surgery. The
client also complains of unpleasant tastes and odors.
Which of the following measures should be included in
the client's plan of care?
1118. The nurse observes ventricular fibrillation on telemetry Start cardiopulmonary resuscitation
and upon entering the client's bathroom finds the client
unconscious on the floor. What intervention should the
nurse implement first?
1119. The nurse plans to help an 18-year-old "I'll be back in 30 minutes to help you get out of
developmentally disabled female client ambulate on bed and walk around the room."
the first postoperative day. When the nurse tells her
it is time to get out of bed, the client becomes angry
and yells at the nurse, "Get out of here! I'll get up
when I'm ready." Which response should the nurse
provide?
1120. The nurse prepares to auscultate heart sounds. Explain to the client that the nurse will be listening
Which nursing interventions would be most effective to different areas of the chest and may listen for a
to assist with this procedure? long time, but that does not mean that anything
abnormal is heard
1121. The nurse prepares to perform postural drainage. d)Auscultation
How should the nurse ascertain the best position to
facilitate clearing the lungs?

a) Inspection
b) Chest X-ray
c) Arterial blood gas (ABG) levels
d)Auscultation
1122. A nurse providing education about hypertension to a smoking, physical inactivity, diabetes mellitus
community group is discussing the high risk for
cardiovascular complications. Which of the
following are risk factors for cardiovascular
problems in clients with hypertension? Choose all
that apply
1123. The nurse receives the client's next scheduled bag of return solution to pharmacy
total parental nutrition (TPN) labeled with the
additive NPH insulin. What action should the nurse
implement?
1124. The nurse recognizes that the patient diagnosed with pain 2 to 3 hours after a meal
a duodenal ulcer will likely experience
1125. The nurse requests an order from the physician to equianalgesic dose of morphine
change the dose to an
1126. The nurse's first priority is to notify the family of the In providing care for a terminally ill resident of a
resident's impending death (C). The family may long-term care facility, the nurse determines that
request that hospice care is initiated (A). Reporting the resident is exhibiting signs of impending death
the client's acuity level (B) does not have the priority and has a "do not resuscitate" or DNR status. What
of informing the family of the client's condition. intervention should the nurse implement first?
Once the family is contacted, the nurse can also A. Request hospice care for the client.
contact the chaplain (D). B. Report the client's acuity level to the nursing
Correct Answer: C supervisor.
C. Notify family members of the client's condition.
D. Inform the chaplain that the client's death is
imminent.
1127. The nurse should address the healthcare provider The nurse overhears the healthcare provider
with the written report and discuss why he/she did explaining to the client that the tumor removed was
not tell the client the truth--this may be at the non-malignant and that the client will be fine.
family's request (A). (B, C, and D) may be indicated, However, the nurse has read in the pathology
but first the nurse should confer with the healthcare report that the tumor was malignant and that there
provider to obtain all needed information. is extensive metastasis. Who should the nurse
Correct Answer: A consult with first regarding the situation?
A. Healthcare provider.
B. Client's family.
C. Case manager.
D. Chief of staff.
1128. The nurse should be careful to keep the soiled linens How should the nurse handle linens that are soiled
from contaminating the fresh linens, and should with incontinent feces?
handle the soiled linens like any other dirty linen A. Put the soiled linens in an isolation bag, then
(C). (A, B, and D) are not indicated. place it in the dirty linen hamper.
Correct Answer: C B. Place an isolation hamper in the client's room
and discard the linens in it.
C. Place the soiled linens in a pillow case and
deposit them in the dirty linen hamper.
D. Ask the housekeeping staff to pick up the soiled
linen from the dirty utility room.
1129. A nurse should be prepared to manage a) Renal failure
complications following abdominal aortic aneurysm Explanation:
resection. Which complication is most common? Renal failure commonly occurs if clamping time is
a) Renal failure prolonged, cutting off the blood supply to the
b) Graft occlusion kidneys. Hemorrhage and shock are the most
c) Hemorrhage and shock common complications before abdominal aortic
d) Enteric fistula aneurysm resection, and they occur if the aneurysm
leaks or ruptures. Graft occlusion and enteric fistula
formation are rare complications of abdominal
aortic aneurysm repair.
1130. The nurse should check for leaks in the chest tube There is continuous bubbling in the water-seal
and pleural drainage system when: chamber
1131. The nurse should deal with the issue immediately A nurse observes a student nurse taking a copy of a
and explain that a client's records are the property client's medication administration record. When
of the hospital and cannot be removed (D), even questioned, the student states, Another student is
with the client's permission (C). Next, the clinical scheduled to administer medications for this client
instructor should be notified (B)so that all students tomorrow, so I am going to make a copy to help my
can be educated regarding copying and removing friend prepare for tomorrow's clinical. What
clinical records from the healthcare agency. The response should the nurse provide first?
nursing supervisor (A) should also be alerted to A. Ask the nursing supervisor to meet with the
ensure appropriate supervision of students as well students.
as protection of client information. B. Notify the student's clinical instructor of the
Correct Answer: D situation.
C. Ask the student if permission was obtained from
the client.
D. Explain that the records are hospital property
and may not be removed.
1132. The nurse should document the client's complaints The nurse is completing the plan of care for a client
(A) as subjective data--symptoms only the client can who is admitted for benign prostatic hypertrophy.
describe. (B) should be documented as objective Which data should the nurse document as a
data, which is collected via the nurse's observation. subjective findings?
(C and D) are documented as intervention results. A. Complains of inability to empty bladder.
Correct Answer: A B. Temperature of 99.8 F and pulse of 108.
C. Post-voided residual volume of 750 ml.
D. Specimen collection for culture and sensitivity.
1133. The nurse should expect to hold the insulin infusion potassium replacement has been initiated.
for 30 minutes until the ]
1134. The nurse should first assess what the client desires (C). A single mother of two teenagers, ages 16
(A) is somewhat judgmental and attempts to solve the and 18, was just told that she has advanced
problem for the client without eliciting the client's feelings. cancer. She is devastated by the news, and
Though a referral to the social worker (B) may be expresses her concern about who will care
indicated, the nurse should first offer support. Time is for her children. Which statement by the
likely to help the client cope with this news (D), but the nurse is likely to be most helpful at this
nurse should first provide support and assess what the time?
client wants to see happen with her children. A. Your children are old enough to help you
Correct Answer: C make decisions about their futures.
B. The social worker can tell you about
placement alternatives for your children.
C. Tell me what you would like to see
happen with your children in the future.
D. You have just received bad news, and you
need some time to adjust to it.
1135. The nurse should first provide an immediate comfort An older female client with rheumatoid
measure to address the client's complaint about the linens arthritis is complaining of severe joint pain
and drape the linens over the footboard of the bed (D) that is caused by the weight of the linen on
instead of tucking them under the mattress, which can add her legs. What action should the nurse
pressure perceived by the client as the source of her pain. implement first?
(A, B, and C) may be components of the client's plan of A. Apply flannel pajamas to provide
care, but the nurse should first address the client's warmth.
complaint. B. Administer a PRN dose of ibuprofen.
Correct Answer: D C. Perform range of motion exercises in a
warm tub.
D. Drape the sheets over the footboard of the
bed.
1136. The nurse should first slow the IV flow rate to keep vein The nurses determines a client's IV solution
open (KVO) rate (B) to prevent further risk of fluid is infusing at 250 ml/hr. The prescribed rate
volume overload, then gather additional assessment data, is 125 ml/hr. What action should the nurse
such as when the IV solution was started (A) and the take first?
appearance of the IV insertion site (C) before contacting A. Determine when the IV solution was
the healthcare provider (D) for further instructions. started.
Correct Answer: B B. Slow the IV infusion to keep vein open
rate.
C. Assess the IV insertion site for swelling.
D. Report the finding to the healthcare
provider.
1137. The nurse should immediately assess the blood pressure hypotension from a decrease in preload and
since Nipride and Lasix can cause severe afterload. If the client is hypotensive, the
Nipride dose should be reduced or
discontinued.
1138. The nurse should implement (D), because A client who has been on bedrest for several days
orthostatic hypotension is a common result of now has a prescription to progress activity as
immobilization, causing the client to feel dizzy tolerated. When the nurse assists the client out of
when first getting out of bed following a period of bed for the first time, the client becomes dizzy. What
bedrest. To prevent this problem, it is helpful to action should the nurse implement?
have the body acclimate to a standing position by A. Encourage the client to take several slow, deep
sitting upright for a short period (D) before rising breaths while ambulating.
to a standing position. (A) is unlikely to alleviate B. Help the client to remain standing by the bedside
the dizziness. (B) may result in a loss of until the dizziness is relieved.
consciousness. (C) is not indicated and will increase C. Instruct the client to remain on bedrest until the
the potential for complications associated with healthcare provider is contacted.
prolonged immobility. D. Advise the client to sit on the side of the bed for a
Correct Answer: D few minutes before standing again.
1139. The nurse should instruct a client receiving a 8-oz glasses of fluid daily to maintain a urine output
sulfonamide such as co-trimoxazole to drink at of at least 1,500 ml/day. Otherwise, inadequate urine
least eight output may lead to crystalluria or tubular deposits.
1140. The nurse should instruct the client to take 30 minutes before meals and at bedtime to reduce
Propantheline bromide GI motility, thus relieving spasticity.
1141. The nurse should monitor anesthesia/pain levels active labor to ascertain that this client is
every 30 minutes during comfortable during the labor process and
particularly during active labor when pain often
accelerates for the client.
1142. The nurse should plan to implement (A, B, D, and The nurse is preparing a male client who has an
F). Pre-medicating the client with an analgesic (A) indwelling catheter and an IV infusion to ambulate
reduces the client's pain during mobilization and from the bed to a chair for the first time following
maximizes compliance. To ensure the client's abdominal surgery. What action(s) should the nurse
cooperation and promote independence, the nurse implement prior to assisting the client to the chair?
should inform the client about the plan for moving (Select all that apply.)
to the chair (B) and encourage the client to A. Pre-medicate the client with an analgesic.
participate by pushing the IV pole when walking to B. Inform the client of the plan for moving to the
the chair (D). The nurse should assess the client's chair.
blood pressure (F) prior to mobilization, which can C. Obtain and place a portable commode by the bed.
cause orthostatic hypotension. (C and E) are not D. Ask the client to push the IV pole to the chair.
indicated. E. Clamp the indwelling catheter.
Correct Answer: A, B, D, F F. Assess the client's blood pressure.
1143. A nurse should question the order for morphine biliary spasm. Thus, the preferred opioid analgesic
sulfate because it is believed to cause to treat cholecystitis is meperidine (Demerol).
1144. The nurse should wear a gown, gloves, a mask, and eye protection when
entering the client's room.
1145. The nurse should wrap the client's arms and legs distal to proximal ends and use strict sterile
from the technique throughout the dressing change.
1146. The nurse teaches the patient with a high risk for Walk or perform weight-bearing exercises outdoors
osteoporosis about risk-lowering strategies
including which of the following actions?
1147. The nurse took the correct action and should Before administering a client's medication, the nurse
document the events that occurred in the nurses' assesses a change in the client's condition and
notes (C). (A) did not occur and (B) is not decides to withhold the medication until consulting
indicated. The medication administration record is with the healthcare provider. After consultation with
part of the client's medical record and should be the healthcare provider, the dose of the medication is
placed in the chart, (D) when no longer current. changed and the nurse administers the newly
Correct Answer: C prescribed dose an hour later than the originally
scheduled time. What action should the nurse
implement in response to this situation?
A. Notify the charge nurse that a medication error
occurred.
B. Submit a medication variance report to the
supervisor.
C. Document the events that occurred in the nurses'
notes.
D. Discard the original medication administration
record.
1148. A nurse who suspects an air embolism should place his left side and in Trendelenburg's position. Doing
the client on so allows the air to collect in the right atrium rather
than enter the pulmonary system.
1149. A nurse who works in the OR is required to assess Increased urine output; symptoms of malignant
the patient continuously and protect the patient hyperthermia include tachycardia, tachypnea,
from potential complications. Which of the cyanosis, fever, muscle rigidity, diaphoresis, mottled
following would not be included as a symptom of skin, hypotension, irregulr heart rate, decreased
malignant hyperthermia? urine output and cardiac arrest
1150. The nurse witnesses a baseball player receive a Ability to spontaneously open the eyes before any
blunt trauma to the back of the head with a tactile stimuli are given
softball. What assessment data should the nurse
collect immediately?
1151. A nurse working in a community health setting is A 17-year-old who is sexually active with numerous
performing primary health screenings. Which partners
individual is at highest risk for contracting an HIV
infection?
1152. A nurse working in the clinic is seeing a client who "Hypertension often causes no symptoms."
has just been prescribed a new medication for
hypertension. The client asks why hypertension is
sometimes called the "silent killer." The nurse's
correct response is which of the following?
1153. The nurse would keep the client's head flat after infratentorial, not supratentorial, surgery.
1154. Nursing care for cardiac transplants focused on -Promoting patient adaptation to the transplant
what? Read process
-Monitoring cardiac function
-Managing lifestyle changes
-Providing relevant teaching
1155. The nursing diagnosis of ineffective health The nurse formulates the nursing diagnosis of,
maintenance refers to an inability to identify, "Ineffective health maintenance related to lack of
manage, and/or seek out help to maintain health, motivation" for a client with Type 2 diabetes. Which
and is best exemplified in the client belief or finding supports this nursing diagnosis?
understanding about diet and health maintenance A. Does not check capillary blood glucose as
(D). (A) indicates noncompliance with an action to directed.
be done in the management of diabetes. (B) B. Occasionally forgets to take daily prescribed
represents inattentiveness. (C) reflects knowledge medication.
deficit. C. Cannot identify signs or symptoms of high and
Correct Answer: D low blood glucose.
D. Eats anything and does not think diet makes a
difference in health.
1156. The nursing instructor is teaching their clinical By questioning how many pillows the client
group how to assess a client for congestive heart normally uses for sleep-->The nurse should ask the
failure. How would the instructor teach the client about nocturnal dyspnea by questioning how
students to assess a client with congestive heart many pillows the client normally uses for sleep. This
failure for nocturnal dyspnea? is because being awakened by breathlessness may
prompt the client to use several pillows in bed.
Collecting the client's urine output, observing the
client's diet, or measuring the client's abdominal
girth does not help assess for nocturnal dyspnea
1157. Nursing Management -Promote emergency care
1. what should the nurse promote ?
-Promote measures to maintain adequate chest
expansion

-Promote coping
1158. Nursing Management pneumonia 1. prescribed medications
1. Administer..... 2. Promote infection control measures
2._____________ control measures 3. aspiration pneumonia in a client receiving tube
3. Prevent....... feeding
4. Positioning 4. Positioning
5. Nutrition 5. Nutrition
6. Promote..... 6. fluid intake
7. Promote _________ hygiene 7. bronchial
1159. A nursing measure that should be instituted after a Range-of-motion exercises on the affected upper
pneumonectomy is: extremity
1160. Nursing Process TB 1.
1. Assess for Productive cough
Night sweats
2. Nursing diagnoses Afternoon temperature elevation
Weight loss
3. Goals 2.
Ineffective health maintenance
Activity intolerance
Ineffective breathing pattern
Imbalanced nutrition: Less than body requirements
Noncompliance
3.
Comply with therapeutic regimen
Have no recurrence of disease
Have normal pulmonary function
Take appropriate measures to prevent spread of disease
1161. oa The joint pain occurs with movement and rest. As the disease progresses, pain may also occur at rest.
is relieved by
1162. Obesity is a risk factor for osteoarthritis stress on the joints.
because it places increased
1163. Obesity, stress, high intake of sodium or risk factors for primary hypertension.
saturated fat, and family history are all
1164. An objective should contain four elements: Which statement correctly identifies a written learning
who will perform the activity or acquire the objective for a client with peripheral vascular disease?
desired behavior, the actual behavior that the A. The nurse will provide client instruction for daily foot
learner will exhibit, the condition under care.
which the behavior is to be demonstrated, B. The client will demonstrate proper trimming toenail
and the specific criteria to be used to measure technique.
success. (C) is a concise statement that is a C. Upon discharge, the client will list three ways to protect
learning objective that defines exactly how the feet from injury.
the client will demonstrate mastery of the D. After instruction, the nurse will ensure the client
content. (A, B, and D) lack one or more of understands foot care rationale.
these elements.
Correct Answer: C
1165. The obturator is inserted into the
replacement tracheostomy tube to guide

insertion and is then removed to allow passage of air


through the tube.
1166. Of the following diuretic medications, which Spironolactone (Aldactone)---->Aldactone is known as a
conserves potassium? potassium-sparing diuretic. Lasix causes loss of potassium
from the body. Diuril causes mild hypokalemia. Hygroton
causes mild hypokalemia.
1167. An older client is admitted with a diagnosis of bacterial Confusion and tachycardia
pneumonia. The nurse's assessment of the client will
most likely reveal which sign/symptom?
1168. An older female client with dementia is transferred from "Confusion in an older person often follows
a long-term care unit to an acute care unit. The client's relocation to new surroundings."
children express concern that their mother's confusion is
worsening. How should the nurse respond?
1169. An older male client comes to the geriatric screening Auscultate the client's breath sounds.
clinic complaining of pain in his left calf. The nurse
notices a reddened area on the calf of his right leg that is
warm to the touch and the nurse suspects that the client
may have thrombophlebitis. Which additional
assessment is most important for the nurse to perform?
1170. Oligohydramnios is a condition in pregnancy deficiency of amniotic fluid.
characterized by a
1171. oliguric phase Metabolic acidosis develops because the
kidneys cannot excrete hydrogen ions, and
bicarbonate is used to buffer the hydrogen.
Hypertension may develop as a result of fluid
retention. Hyperkalemia develops as the
kidneys lose the ability to excrete potassium.
1172. On arrival at the intensive care unit, a critically ill client d) Hypotension
suffers respiratory arrest and is placed on mechanical
ventilation. The physician orders pulse oximetry to
monitor the client's arterial oxygen saturation (SaO2)
noninvasively. Which vital sign abnormality may alter
pulse oximetry values?

a) Fever
b) Tachypnea
c) Tachycardia
d) Hypotension
1173. Once a client has been premedicated for surgery with After a client has been premedicated for
any type of sedative, legal informed consent is not surgery with an opioid analgesic, the nurse
possible, so the nurse must notify the surgeon (A). (B, C, discovers that the operative permit has not
and D) are not legally viable options for ensuring been signed. What action should the nurse
informed consent. implement?
Correct Answer: A A. Notify the surgeon that the consent form
has not been signed.
B. Read the consent form to the client before
witnessing the client's signature.
C. Determine if the client's spouse is willing
to sign the consent form.
D. Administer an opioid antagonist prior to
obtaining the client's signature.
1174. On digital rectal examination, key signs of prostate hard prostate, induration of the prostate, and
cancer are a an irregular, hard nodule.
1175. One day after a Billroth II surgery, a male client Apply oxygen at 2 L per nasal cannula.
suddenly grabs his right chest and becomes pale and
diaphoretic. Vital signs are assessed at blood pressure
100/80, pulse 110 beats/min, and respirations 36
breaths/min. What action is most important for the nurse
to take?
1176. One goal of care for a client with PVD is to decrease arteries
anxiety, so as to decrease or prevent vasoconstriction of
the:
1177. One nursing goal for a child with febrile seizures is to maintain the child's temperature at a level low
enough to prevent recurrence of seizures.
1178. : One of the most common adverse effects of the drug tachycardia.
hydralazine (Apresoline) is
1179. o Permanent hypothyroidism is the major The client needs to be educated about the need for lifelong
complication of RAI 131I treatment. thyroid hormone replacement and watch for signs of
o hypothyrodism
1180. Opioid antitussives, such as codeine and treating unruly coughs usually associated with lung cancer
hydrocodone, are reserved for
1181. or a client with advanced chronic obstructive c) Using a high-flow Venturi mask to deliver oxygen as
pulmonary disease (COPD), which nursing prescribed
action best promotes adequate gas
exchange?

a) Encouraging the client to drink three


glasses of fluid daily
b) Keeping the client in semi-Fowler's
position
c) Using a high-flow Venturi mask to deliver
oxygen as prescribed
d) Administering a sedative as prescribed
1182. Oral hygiene is an important aspect of self- laryngectomy client, who is less able to detect mouth odor.
care for the Additionally, the mouth harbors bacteria. Good mouth care
reduces the risk of infection.
1183. Oral steroids can cause gastric irritation and ulcers and should be administered
with meals, if possible, or otherwise with an antacid.
1184. Oropharyngeal candidiasis, or thrush, is the early symptomatic stages of HIV infection.
most common infection associated with the
1185. Orthostatic hypotension occurs most older adults with ISH (isolated systolic HTN)
frequently in which patients?
1186. osteoarthritis Opioid analgesics are not used for osteoarthritic pain
control. Acetaminophen and selected nonsteroidal anti-
inflammatory drugs may be used to achieve pain relief.
1187. osteoarthritis Intra-articular corticosteroid routinely; rather, they are cautiously used during periods of
injections are not used acute joint pain.
1188. Osteoporosis occurs in Cushing's syndrome; therefore, with successful treatment,
bone mineralization increases.
1189. Other AIDS-defining illnesses include Kaposi's sarcoma; cytomegalovirus of the liver, spleen, or
lymph nodes; and Pneumocystis carinii pneumonia.
1190. Other indicators of hypothyroidism are the antithyroid antibodies and elevation of the creatine
presence of phosphokinase (CPK-MM) level.
1191. Other potential complications of mitral mural thrombi, pulmonary hemorrhage, and embolism to
stenosis include vital organs.
1192. Other signs and symptoms of early septic warm and flushed skin fever with restlessness and
shock include confusion; decreased blood pressure with tachypnea and
tachycardia; increased or normal urine output; and nausea
and vomiting or diarrhea.
1193. Other symptoms associated with right upper quadrant tenderness, fever from inflammation
cholelithiasis are or infection, jaundice from elevated serum bilirubin levels,
and nausea or right upper quadrant pain after a fatty meal
1194. Other teaching to decrease community Instruct patient to:
infection: 1. mouth when coughing or sneezing
=Instruct patient to:= 2.mask as instructed
1. Cover.... 3. windows to assure proper ventilation
2. Wear.... 4. work or school until instructed by physician
3. Open..... 5. transportation
4. Do not go to..... 6. visitors
5. Avoid public..... 7. 2-3 weeks .
6. Limit....
7. When sputum cultures negative
after______________ of treatment, patient is
no longer considered infectious.
1195. Other than cholesterol lowering drugs, what Antiplatelet drugs (ASA, Plavix)
is another group of drugs used to treat
atherosclerosis?
1196. Over secretion of ACTH Cushings
1197. Over secretion of ADH SIADH
1198. Over secretion of GH- growth hormone Acromegaly
1199. Over secretion of Prolactin hyperprolactinoma--> abnormal milk production
1200 Over secretion of PTH Hyperparathroidism
.

1201 Oversecretion of the adrenal medulla causes pheochromocytoma


.

1202 Over secretion of TSH Hyper thyroid


.

1203 Overuse of nasal spray containing rhinitis medicamentosa, which is a rebound effect causing
. pseudoephedrine can lead t increased swelling and congestion.
1204 Owing to the massive cellular destruction that potassium is released into the extracellular fluid, which
. occurs in burns, leads to hyperkalemia.
1205 Oxygen therapy is drying to the oral and water-soluble lubricant, such as K-Y jelly, to prevent
. nasal mucosa; therefore, the client should be drying.
encouraged to apply a
1206 Oxytocin is administered as nasal spray before breast-feeding to stimulate lactation.
.

1207 oxytocin is used to treat eclampsia, reduce parenterally


. postpartum bleeding, or treat erythroblastosis
fetalis, the drug is administered
1208 pacemaker care Avoid lifting anything heavier 3 lb.
. than
1209 pacemaker care The client should be lifting the operative-side arm above shoulder level for 1
. instructed to avoid week postinsertion. It takes up to 2 months for the
incision site to heal and full range of motion to return.
1210 pad They should be advised to rest if pain develops and resume activity when pain subsides
.

1211. pad To avoid burns, heating pads should not be used by anyone with impaired
circulation
1212 Pain at McBurney's point lies between the appendicitis.
. umbilicus and right iliac crest and is
associated with
1213 Pain from a kidney stone is considered an emergency situation and requires analgesic intervention.
.

1214 Pain in the calf is common with a diagnosis of deep vein thrombosis.
.

1215 pancreatitis risk factor Excessive alcohol intake and gallstones are the greatest
. risk factors. Abdominal trauma can potentiate
inflammation. Hyperlipidemia is a risk factor for recurrent
pancreatitis.
1216 A pansystolic, blowing, high-pitched murmur mitral insufficiency.
. characterizes
1217 Parasympathetic hyperactivity leading to bradyrhythmia causes vasovagal syncope. That is,
. sudden hypotension secondary to bradyrhythmia leads to cerebral ischemia which, in turn,
leads to syncope.
1218 Parasympathetic reaction can occur as a result of The nurse is digitally removing a fecal impaction
. digital stimulation of the anal sphincter, which should for a client. The nurse should stop the procedure
be stopped if the client experiences a vagal response, and take corrective action if which client reaction
such as bradycardia (B). (A, C, and D) do not warrant is noted?
stopping the procedure. A. Temperature increases from 98.8 to 99.0 F.
Correct Answer: B B. Pulse rate decreases from 78 to 52 beats/min.
C. Respiratory rate increases from 16 to 24
breaths/min.
D. Blood pressure increases from 110/84 to
118/88 mm/Hg.
1219 PARATHYROID GLAND produce parathyroid calcium, phosphorus, and vitamin D levels
. hormone, which controls within the blood and bone.
1220 The parathyroid glands are located in the neck, near thyroid gland.
. or attached to the back side of the
1221 Parathyroid hormone (PTH) has which effects on the Stimulation of calcium reabsorption and
. kidney? phosphate excretion--->PTH stimulates the
kidneys to reabsorb calcium and excrete
phosphate and converts vitamin D to its active
form, 1,25-dihydroxyvitamin D. PTH doesn't
have a role in the metabolism of vitamin E
1222 The parietal lobe interprets and integrate sensations, including pain, temperature, and
. touch; it also interprets size, shape, distance, and
texture.
1223 Paronychia refers to an inflammation of the skinfold at the nail margin
.

1224 The passage of feces through the vagina, not vaginal bleeding, is a sign of rectovaginal fistula.
.

1225 Passive ROM exercise for the hip and knee is provided The nurse is providing passive range of motion
. by supporting the joints of the knee and ankle (D) and (ROM) exercises to the hip and knee for a client
gently moving the limb in a slow, smooth, firm but who is unconscious. After supporting the client's
gentle manner. (A) should be done before the exercises knee with one hand, what action should the nurse
are begun to prevent injury to the nurse and client. (B) take next?
is carried out after both joints are supported. After the A. Raise the bed to a comfortable working level.
knee is bent, then the knee is moved toward the chest B. Bend the client's knee.
to the point of resistance (C) two or three times. C. Move the knee toward the chest as far as it
Correct Answer: D will go.
D. Cradle the client's heel.
1226 Passive ROM exercises and isometric exercises do not risk of osteoporosis.
. provide the bone stress necessary to reduce the
1227 Pathophysiology of pneumonia 1. alveoli activates the inflammatory and
. 1. Spread of microbes in..... immune response
2. Alveoli, interstitial tissue, and bronchioles 2. fluid filled (with fluid or blood) as a result of
become...... inflammation
3. The lungs become...... 3. congested, then air flow and blood flow
decrease (V/Q =silent unit)
-If no complications, healing occurs as the
exudate gets absorbed and macrophages process
debris. Lung function resumes.
1228 A patient diagnosed with active TB 1 week ago is Admitting the patient to an airborne-infection
. admitted to the hospital with symptoms of chest pain. isolation room
Initially, the nurse gives the highest priority to
1229 A patient diagnosed with a pericarditis and pericardial Inability of the ventricles to distend and fill
. effusion. Based on the physiologic mechanisms of adequately---->An increase in pericardial fluid
increased pericardial fluid and its effect on the heart, raises the pressure within the pericardial sac and
which of the following effects would be expected? compresses the heart. This causes increased right
and left ventricular end-diastolic pressures,
decreased venous return, and inability of the
ventricles to distend and fill adequately
1230 A patient diagnosed with class 3 TB 1 week ago is Admitting the patient to an airborne-infection
. admitted to the hospital with symptoms of chest pain. isolation room
Initially, the nurse gives the highest priority to:
1231 Patient Education 1. sputum specimens
. =Prepare patient for continued treatment after 2. tissue when coughing, sneezing, or producing
discharge:= sputum
1. Teach how to obtain and care for what ? 3. sputum-soiled tissues
2. Cover nose and mouth with what ? 4. hand washing and good hygiene
3. Hand washing after handling what ? 5. drug regimen
4. Reinforce importance of..... 6. dosages, frequency, potential adverse effects,
5. Reinforce importance of and evaluate patient's understanding
uninterrupted______________? 7. adequate supply of medications
6. Explain medications what 8. follow-up visit and DOT if indicated
7. Ensure....
8. Arrange for.....
1232 A patient has been declared legally blind. This means Blindness is defined as a best corrected visual
. that the patient has a best corrected visual acuity acuity (BCVA) that can range from 20/400 to no
(BCVA) that does not exceed what in the better eye? light perception (NLP). The clinical definition of
absolute blindness is the absence of light
perception. Legal blindness is a condition of
impaired vision in which a person has a BCVA
that does not exceed 20/200 in the better eye or
whose widest visual field diameter is 20 degrees
or less. This makes options A, C, and D incorrect.
1233 A patient has been receiving high-dose corticosteroids Candidiasis
. and broad-spectrum antibiotics for treatment
secondary to a traumatic injury and infection. The
nurse plans care for the patient knowing that the
patient is most susceptible to:
1234 A patient has bee told he has cataracts in both eyes. Surgical intervention
. The patient wants to know what the treatment options Explanation:
are. What should the nurse tell the patient is the most Surgery is the treatment option of choice when
appropriate treatment option for patients with age- the patient's functional and visual status is
related cataracts that are affecting the patient's ability compromised. No nonsurgical (medications,
to function? eyedrops, eyeglasses) treatment cures cataracts or
prevents age-related cataracts. Studies recently
a) Eyeglasses or magnifying lenses have found no benefit from antioxidant
b) Surgical intervention supplements, vitamins C and E, beta-carotene, or
c) Corticosteroid eye drops selenium. Corticosteroid eyedrops are prescribed
d) Antioxidant supplements, vitamin C and E, beta- for use after cataract surgery, but in fact, increase
carotene, and selenium the risk for cataracts if used long-term or in high
doses. Eyeglasses and magnification may
improve vision in the patient with early stages of
cataracts, but have limitations for the patient with
impaired functioning.
1235 The patient has had biomarkers drawn after b) Troponin
. complaining of chest pain. Which diagnostic of Explanation:
myocardial infarction remains elevated for as long as Troponin remains elevated for a long period,
3 weeks? often as long as 3 weeks, and it therefore can be
a) Total CK used to detect recent myocardial damage.
b) Myoglobin Myoglobin returns to normal in 12 hours. Total
c) Troponin CK returns to normal in 3 days. CK-MB returns
d) CK-MB to normal in 3 to 4 days
1236 A patient is admitted to the hospital with fever, chills, Hyperthermia related to acute infections process
. a productive cough with rusty sputum, and pleuritic
chest pain. Pneumococcal pneumonia is suspected. An
appropriate nursing diagnosis for the patient based on
the patient's manifestations is:
1237 A patient is being discharged home from the New floater in vision
. ambulatory surgical center after cataract Explanation:
surgery. In reviewing the discharge instructions Cataract surgery increases the risk of retinal
with the patient, the nurse instructs the patient detachment and the patient must be instructed to notify
to immediately call the office if the patient the surgeon if new floaters in vision, flashing lights,
experiences what? decrease in vision, pain, or increase in redness occurs.
Slight morning discharge, some redness, and a
scratchy feeling may be expected for a few days after
surgery.
1238 A patient is on a continuous epoprostenol Assess the central line immediately for any obstruction
. infusion pump. The alarm goes off indicating an or accidental clamping of tubing
obstruction in the intravaneous line downstream.
The nurse should:
1239 The patient is on a continuous tube feeding. The Shift
. tube placement should be checked every
1240 A patient receiving chemotherapy for breast Renal impairment
. cancer develops a Cryptococcus infection of the Nausea and vomiting
lungs and is treated with IV amphotericin B. The Malignant hyperthermia reaction
nurse monitors the patient carefully during the
drug's administration with the knowledge that
this drug increases the patient's risk for (select
all that apply)
1241 Patients who are taking beta-adrenergic blocking Worsening angina
. agents should be cautioned not to stop taking
their medications abruptly because which of the
following may occur?
1242 Patients with chronic liver dysfunction have Scurvy
. problems with insufficient vitamin intake. Which
of the following may occur as a result of vitamin
C deficiency?
1243 A patient with a 40-pack-year history of smoking "Screening measures for lung cancer are controversial,
. has recently stopped because of the fear of but we can discuss the advantages and disadvantages
developing lung cancer. The patient asks the of various measures."
nurse what he can do to learn about whether he
develops lung cancer. The best response for the
nurse is,
1244 A patient with active TB continues to have Arrange for directly observed therapy by a responsible
. positive sputum cultures after 6 months of family member or a public health nurse.
treatment because she says she cannot remember
to take the medication all the time. The best
action by the nurse is to
1245 A patient with active TB continues to have Arrange for directly observed therapy by a responsible
. positive sputum cultures ater 6 months of family member of a public health nurse
treatment because she says she cannot remember
to take the medication all the time. The best
action by the nurse is to:
1246 A patient with advanced lung cancer refuses pain Can you tell me what the pain means to you?
. medication saying, "I deserve everything this
cancer can give me." The nurse's best response
to the patient is:
1247 A patient with a lung mass found on chest x-ray Biopsy positive for malignant cells
. is undergoing further testing. The nurse explains
that a diagnosis of lung cancer can be confirmed
by:
1248 patient with breathing problems what should Obtaining vital signs.
. nurse tell uap to do Applying antiembolic stockings.
Keeping the client oriented.
1249 A patient with diabetes mellitus is receiving an Signs of hypoglycemia--->The nurse should observe
. oral antidiabetic agent. The nurse observes for the patient receiving an oral antidiabetic agent for the
which of the following symptoms when caring for signs of hypoglycemia. The time when the reaction
this patient might occur is not predictable and could be from 30 to
60 minutes to several hours after the drug is ingested.
Polyuria, polydipsia, and blurred vision are the
symptoms of diabetes mellitus.
1250 The patient with lung cancer needs to receive Administer both vaccines at the same time in
. influenza vaccine and pneumococcal vaccines. The different arms
nurse will
1251 A patient with pneumonia has a nursing diagnosis of Encourage a fluid intake of at least 3L/day
. ineffective airway clearance related to pain, fatigue,
and thick secretions. An appropriate nursing
intervention for the patient is to:
1252 A patient with pneumonia has a nursing diagnosis of Encourage a fluid intake of at least 3L/day
. ineffective airway clearance related to pain, fatigue
and thick secretions. An appropriate nursing
intervention for the patient is to
1253 A patient with TB has been admitted to the hospital Take all medications for full length of time to
. and is placed in an airborne infection isolation room. prevent multidrug-resistant TB
Which of the following should the patient be taught Wear a standard isolation mask if leaving the
(select all that apply)? airborne infection isolation room
Maintain precautions in airborne infection
isolation room by coughing into a paper tissue
1254 Peak flow numbers should be monitored daily, morning (before taking medication). Peak flow
. usually in the does not need to be monitored after each meal.
1255 The peak incidence of cervical cancer is carcinoma in situ is 20 to 30 years of age in
. African-American and Caucasian women.
1256 Pelvic rocking helps to relieve backache during pregnancy and early labor by
. making the spine more flexible.
1257 People who are lactose-intolerant usually are able to yogurt, cheese, and buttermilk
. tolerate dairy products in which lactose has been
fermented, such as
1258 Percussion Notes and Their Meaning 1. Pleural Effusion or Lobar Pneumonia
. 1. Flat or Dull...... 2. Healthy Lung or Bronchitis
2. Resonant.... 3. Emphysema or Pneumothorax
3. Hyperresonant.......
1259 peripheral nervous system is motor movement and sensory system
.

1260 peritonitis or perforated bowel abdominal rigidity, a cardinal sign of peritonitis


. and perforated bowel
1261 The pharmacology instructor is diagramming the Hypothalamus-->The hypothalamus is the
. nervous and endocrine systems. What organ would coordinating center for the nervous and endocrine
the instructor diagram as the connector between the responses to internal and external stimuli. The
nervous and endocrine systems? pituitary, thyroid, and parathyroid glands all play
an important role in hormones, but do not connect
the nervous and endocrine systems
1262 Physical contact, such as touching the head, in some What action by the nurse demonstrates culturally
. cultures is a sign of respect, whereas in others, it is sensitive care?
strictly forbidden. So asking permission before A. Asks permission before touching a client.
touching a client (A) demonstrates culturally B. Avoids questions about male-female
sensitive care. (B, C, and D) do not demonstrate relationships.
cultural awareness. C. Explains the differences between Western
Correct Answer: A medical care and cultural folk remedies.
D. Applies knowledge of a cultural group unless a
client embraces Western customs.
1263 A physician admits a client to the health care c) Stabilizing heart rate and blood pressure and
. facility for treatment of an abdominal aortic easing anxiety
aneurysm. When planning this client's care, which Explanation:
goal should the nurse keep in mind as she For a client with an aneurysm, nursing interventions
formulates interventions? focus on preventing aneurysm rupture by stabilizing
a) Increasing blood pressure and monitoring fluid heart rate and blood pressure. Easing anxiety also is
intake and output important because anxiety and increased stimulation
b) Decreasing blood pressure and increasing may raise the heart rate and boost blood pressure,
mobility precipitating aneurysm rupture. The client with an
c) Stabilizing heart rate and blood pressure and abdominal aortic aneurysm is typically hypertensive,
easing anxiety so the nurse should take measures to lower blood
d) Increasing blood pressure and reducing pressure, such as administering antihypertensive
mobility agents, as ordered, to prevent aneurysm rupture. To
sustain major organ perfusion, the client should
maintain a mean arterial pressure of at least 60 mm
Hg. Although the nurse must assess each client's
mobility individually, most clients need bed rest
when initially attempting to gain stability.
1264 A physician orders blood coagulation tests to c) Prothrombin time (PT)
. evaluate a client's blood-clotting ability. The nurse Explanation:
knows that such tests are important in assessing PT determines a client's response to oral
clients at risk for thrombi, such as those with a anticoagulant therapy. This test measures the time
history of atrial fibrillation, infective endocarditis, required for a fibrin clot to form in a citrated plasma
prosthetic heart valves, or myocardial infarction. sample following addition of calcium ions and tissue
Which test determines a client's response to oral thromboplastin and compares this time with the
anticoagulant drugs? fibrin-clotting time in a control sample. The
a) Bleeding time physician should adjust anticoagulant dosages as
b) Platelet count needed, to maintain PT at 1.5 to 2.5 times the control
c) Prothrombin time (PT) value. Bleeding time indicates how long it takes for a
d) Partial thromboplastin time (PTT) small puncture wound to stop bleeding. The platelet
count reflects the number of circulating platelets in
venous or arterial blood. PTT determines the
effectiveness of heparin therapy and helps physicians
evaluate bleeding tendencies. Physicians diagnose
appoximately 99% of bleeding disorders on the basis
of PT and PTT values.
1265 Pinching of the tubing used to deliver oxygen high-pitched whistling sound. .
. causes a
1266 Pink-tinged urine and bladder spasms are cystoscopy.
. common after
1267 Pituitary dysfunction can cause Addison's disease, but this is not a primary disease
. process.
1268 PKU is an autosomal recessive disorder involving the absence of
. an enzyme needed to metabolize the essential amino
acid, phenylalanine, to tyrosine.
1269 pku neonate must have ingested sufficient protein, such as breast milk or formula, for at least 24
. hours.
1270 pku testing the infant before that time excessive vomiting, or poor intake can yield false-
. negative results.
1271 pku The infant does not need t fast 4 hours before the test.
.

1272 Placenta accreta, a rare phenomenon, refers to a placenta abnormally adheres to the uterine lining
. condition in which the
1273 Placing a thin piece of gauze over the contain the secretions and yet allow ventilation.
. tracheostomy during sexual activity will help to
1274 Planning & Implementation (TB) 1. strict adherence to guidelines of respiratory
. 1. Hospitalization of the patient with active TB isolation and use of an airborne infection isolation
requires..... room
2. what kind of room should be used ? 2. (negative-pressure room)
3. Wear _______ mask, which can filter..... 3. Wear N 95 maskfilter 95% particulates
4. Wear ______ and _____ 4. gown and gloves
5. Provide client with ______ if necessary to 5. mask
transport
1275 Planning & Implementation (TB) 1. local or state health department
. 1. TB suspects must be reported to.... -This ensures safe and effective treatment after discharge as
2. Upon discharge, patient's care should well as initiates contact investigation for family, friends, co-
continue to be... workers at risk
3. All patients should have monthly what 2. managed by an experienced TB provider
collected 3. sputum collected for smear, culture, and drug
susceptibility
1276 Platelets should be administered as fast as can be tolerated by the client to avoid aggregation.
.

1277 Pleural Conditions 1. inflammation of the parietal and visceral pleura


. 1. Pleurisy... 2. fluid in the pleural space
2. Pleural effusion.... 3. prurulent fluid accumulation in the pleural space
3. Empyema..... 4. accumulation of fluid in alveolar spaces and lung tissue.
4. Pulmonary edema.....
1278 Pneumococcal vaccine 1. For at-risk population: over 65, chronic disease, like
. 1. used in what population ? diabetes, resides in ECF.
2. Important because of.... 2. increasing drug-resistant S. pneumoniae rate.
3. _____to ____% effective 3. 50-80%
4. Can be given at same time as...... 4. influenza vaccine.
5. Given every ____years. Medicare covers it, and other insurances usually do
(somewhat expensive).
5. five years.
1279 Pneumonia 1. Acute or chronic
. 1. _________ or __________ 2. both lungs
2. One or..... 3. bacteria, viruses, or chemical irritants
3. Caused by.... 4. Inflammatory.
4. ____________ process 5. death among elderly
5. 4th leading cause of..... 6. increased age, alcoholism, CHF, malignancies,
6. Risk factors..... immunosuppression
1280 Pneumonia Etiology Bacterial/typical
. -what is it typically caused by ? Viral
Atypical
Opportunistic
Aspirationcan be obstruction, chemical pneumonitis, or
bacterial

CAP
HAP
1281 pneumonia Pharmacology 1. Broad spectrum
. 1. if organism not identified, what meds do 2. Oral in outpatient setting
you use? a) Macrolides recommendedBiaxin, Zithromax
2. what route should meds be given in the 3. IV in hospitalized patient
outpatient setting? b) A quinoloneone of the "floxin's" or a beta-lactam agent
a) give examples of the outpatient meds ? one of the "cef's" (cefuroxime). For MRSA, add
3. what route should meds be given to vancomcin
hospitalized patient
b) give examples of these meds ? -other medications
Antipyretics
Analgesics
Supportive medsdecongestants, cough medicines,
expectorants, inhalers, bronchodilators
1282 Pneumothorax 1. inspiration, gas exchange, or expiration
. 1. Injury to chest wall/lungs that interfere 2. negative
with...... 3. breathing
2. Normal intrapleural pressure is 4. Disruption of the pleura causes air accumulation within
___________ compared to atmospheric pleural space
pressure 5. collapse
3. Pressure difference is stimulant for
________
4. Disruption of the _________ causes air
accumulation within______________
5. A pneumothorax Causes the lung
to_________?
1283 Pneumothorax Clinical Manifestations 1. Dyspnea
. 1..Dyspnea* 2. toward unaffected side with tension
2. Tracheal deviation...... Pneumothora
3. Diminished...... 3. breath sounds with tension Pneumothora
4. Percussion of...... 4. dullness on affected side
5. Unequal..... 5. chest expansion with tension
6. Diminished __________expansionsimple Pneumothora
Pneumothora 6. chest
7. _________often first symptom in simple Pneumothora 7. Pain
8. if severe 8. Tachypnea/tachycardia/air
hunger/diaphoresis/use of accessory
muscles.
1284 Pneumothorax signs and symptoms include sudden, sharp chest pain; tachypnea; and
. tachycardia, absent breath sounds over the
affected lung, anxiety, and restlessness.
Breath sounds are diminished or absent over
the affected side.
1285 Pneumothorax will cause a client to feel extremely short of facilitate ventilation by the unaffected lung.
. breath. Semi- or high- Fowler's position will Positioning the client toward the affected
side does not compromise the remaining,
functional lung.
1286 Pneumovax 23, a polyvalent pneumococcal vaccine, is prevent the pneumococcal sepsis that
. administered prophylactically to sometimes occurs after splenectomy.
1287 PNEUMOVAX 23 is a vaccine indicated for active PNEUMOVAX 23 is approved for use in
. immunization for the ... persons 50 years of age or older and persons
1288 Polycythemia vera early sign Headache and dizziness are early symptoms
. from engorged veins
1289 Polycythemia vera early sign Shortness of breath is an early symptom
. from congested mucous membranes and
ineffective gas exchange.
1290 Polycythemia vera is a bone marrow disease that leads to abnormal increase in the number of blood
. an cells (primarily red blood cells).
1291 Polycythemia vera late sign Pruritus is a late symptom that results from
. abnormal histamine metabolism
1292 Polyhydramnios is a medical condition describing an excess of amniotic
. fluid in the amniotic sac.
1293 Polyhydramnios treated with Antacids may be prescribed to relieve
. heartburn and nausea and amnioreduction,
also known as therapeutic Amniocentesis
1294 Poor peripheral perfusion would cause subnormal SaO2
.

1295 A positive Chvostek's sign is an indication of A client is demonstrating a positive


. hypocalcemia, so the client should be assessed for the Chvostek's sign. What action should the
subjective symptoms of hypocalcemia, such as numbness nurse take?
or tingling of the hands (B) or feet. (A and C) are unrelated A. Observe the client's pupil size and
assessment data. (D) is contraindicated because the client response to light.
is hypocalcemic and needs additional dietary calcium. B. Ask the client about numbness or tingling
Correct Answer: B in the hands.
C. Assess the client's serum potassium level.
D. Restrict dietary intake of calcium-rich
foods.
1296 Possible contact with body secretions, Which client care requires the nurse to wear barrier gloves as
. excretions, or broken skin is an required by the protocol for Standard Precautions?
indication for wearing barrier A. Removing the empty food tray from a client with a urinary
(nonsterile) gloves. Emptying a urine catheter.
drainage bag requires the use of gloves B. Washing and combing the hair of a client with a fractured leg
(D). (A, B, and C) do not require gloves. in traction.
Correct Answer: D C. Administering oral medications to a cooperative client with a
wound infection.
D. Emptying the urinary catheter drainage bag for a client with
Alzheimer's disease.
1297 Posterior nasal packing should be left in 1 to 3 days.
. place for
1298 The posterior tibial pulse is on the
. medial surface of the

ankle just behind the medial malleolus.


1299 postion for buck traction The client can sit up in bed, remaining in the supine position so that an even,
. sustained amount of traction is maintained under the bandage
used in the Buck's traction and bandage doesnt slip
1300 A postoperative client receives a Respiratory rate of 12 per minute with O2 saturation of 85%
. Schedule II opioid analgesic for pain.
Which assessment finding requires the
most immediate intervention by the
nurse?
1301 Postoperatively after a modified radical Accumulated serum and blood in the operative area are removed
. mastectomy, a client has an incisional
drainage tube attached to Hemovac
suction. The nurse determines the
suction is effective when:
1302 Postpericardiotomy syndrome may Pericardial friction rub--->The syndrome is characterized by
. occur in patients who undergo cardiac fever, pericardial pain, pleural pain, dyspnea, pericardial
surgery. The nurse should be alert to effusion, pericardial friction rub, and arthralgia. Leukocytosis
which of the following clinical (elevated WBCs) occurs, along with elevation of the ESR.
manifestations associated with this
syndrome?
1303 Postpericardiotomy syndrome may Pericardial friction rub
. occur in patients who undergo cardiac
surgery. The nurse should be alert to
which of the following clinical
manifestations associated with this
syndrome?
1304 Postpericardiotomy syndrome may b) Pericardial friction rub
. occur in patients who undergo cardiac Explanation:
surgery. The nurse should be alert to The syndrome is characterized by fever, pericardial pain, pleural
which of the following clinical pain, dyspnea, pericardial effusion, pericardial friction rub, and
manifestations associated with this arthralgia. Leukocytosis (elevated WBCs) occurs, along with
syndrome? elevation of the ESR.
a) Decreased white blood cell (WBC)
count
b) Pericardial friction rub
c) Decreased erythrocyte sedimentation
rate (ESR)
d) Hypothermia
1305 Postural drainage may be helpful for affect the nature of secretions.
. respiratory hygiene but will not
1306 Potato chips (A) are high in sodium. Tuna (B) is high in A low-sodium, low-protein diet is prescribed for
. protein. Bacon (C) and crackers (E) are high in sodium. a 45-year-old client with renal insufficiency and
Only (D) is a meal that is in compliance with a low hypertension, who gained 3 pounds in the last
sodium, low protein diet. month. The nurse determines that the client has
Correct Answer: A, B, C, E been noncompliant with the diet, based on
which report from the 24-hour dietary recall?
(Select all that apply.)
A. Snack of potato chips, and diet soda.
B. Lunch of tuna fish sandwich, carrot sticks,
fresh fruit, and coffee.
C. Breakfast of eggs, bacon, toast, and coffee.
D. Dinner of vegetable lasagna, tossed salad,
sherbet, and iced tea.
E. Bedtime snack of crackers and milk.
1307 potent topical corticosteroid cause vasoconstriction, not vasodilation.
.
1308 A potent topical corticosteroid may increase the client's percutaneously, causing the same adverse
. risk for injury because it may be absorbed effects as systemic corticosteroids
1309 The preferred preparation for treating hypothyroidism Levothyroxine (Synthroid)
. includes which of the following?
1310 Prep U Head & Neck dysfunction ...
.

1311. The presence of a U-wave may or may not be apparent ECG; it represents repolarization of the Purkinje
on a normal fibers.
1312 The presence of rectal bleeding is generally a A nurse is preparing to insert a rectal
. contraindication for the insertion of a rectal suppository and observes a small amount of
suppository, so the nurse should withhold the rectal bleeding. What action should the nurse
medication and notify the healthcare provider (C). (A implement?
and D) may cause increased rectal bleeding. Prior to A. Administer the medication as scheduled after
asking the pharmacist for another form of the assessing the client's vital signs.
medication, the nurse must have a new prescription B. Ask the pharmacist to send an alternate form
from the healthcare provider (B). of the prescribed medication to the unit.
Correct Answer: C C. Withhold the administration of the
suppository until contacting the healthcare
provider.
D. Insert the suppository very gently being
careful not to further injure the rectal mucosa.
1313 Preventing edema is an appropriate immediate physical mobility in the immediate and
. postoperative nursing goal, but attaining it does not extended postoperative periods
affect
1314 Prevention of skin breakdown and maintenance of skin Altered protective pressure sensation.
. integrity among older clients is important because they
are at greater risk secondary to:
1315 Primary Addison's disease refers to a problem in the gland itself that results from idiopathic atrophy
. of the glands.
1316 The primary objective in the immediate post operative Maintaining pulmonary ventilation
. period is?
1317 Primary or first intention healing occurs when tissue is cleanly incised and re-approximated
. and healing occurs without complications. The
incisional defect re-epithelizes rapidly and
matrix deposition seals the defect
1318 The primary symptoms of a client who experiences a left-sided weakness, impulsiveness, and poor
. right-sided stroke are judgment.
1319 The primary treatment for cor pulmonale is directed Treating the underlying pulmonary condition
. toward:
1320 The primary treatment for cystic fibrosis is Vigorous and consistent chest physiotherapy.
.

1321 Priority Patients - 1. pulmonary TB


. Candidates for DOT 2. active TB or LTBI
1. Smear-positive ________ TB 3. Psychiatric
2. Previous treatment for..... 4. Children and adolescents
3. __________ illness 5. Substance abuse
4. __________and___________ 6. drug
5. ___________________ (current or 7. HIV
prior) 8. nonadherence to therapy
6. ____________resistance 9. Memory
7. _____ infection
8. Previous......
9. __________ impairment
1322 Procainamide may cause an increased QT intervals
. QRS complexes and
1323 A progressive activity regimen may be pulmonary function after surgical lung resection
. prescribed to increase
1324 A prolonged PR-interval is associated with first-degree atrioventricular block.
.

1325 A prolonged QT interval is significant development of polymorphic ventricular tachycardia, also


. because it can lead to the known as torsades de pointes.
1326 Promoting Adherence Monitor for medication adverse effects that could cause
. patient to discontinue the regimen

Offer solutions to management of unpleasant adverse effects

Implement the use of DOT whenever possible

Provide effective patient education with regular reinforcement


of messages and written materials in the patient's primary
language
1327 The prone position (B) using a small pillow below the What action should the nurse implement to
. diaphragm maintains alignment and provides the best prevent the formation of a sacral ulcer for a
pressure relief over the sacral bony prominence. Using client who is immobile?
protective (restraining) devices (A) is not indicated. A. Maintain in a lateral position using
Raising the head and bed gatch (C) may reduce shearing protective wrist and vest devices.
forces due to sliding down in bed, but it interferes with B. Position prone with a small pillow below
venous return from the legs and places pressure on the the diaphragm.
sacrum, predisposing to ulcer formation. Sitting in a C. Raise the head and knee gatch when lying in
wheelchair (D) places the body weight over the ischial a supine position.
tuberosities and predisposes to a potential pressure D. Transfer into a wheelchair close to the
point. nurse's station for observation.
Correct Answer: B
1328 Propantheline bromide is classified as a GI anticholinergic; the medication inhibits
. muscarinic actions of acetylcholine at
postganglionic parasympathetic neuroeffector
sites.
1329 Propantheline bromide is used to reduce secretions and spasms of the GI tract in clients
. with diverticulitis, a condition characterized by
bowel inflammation and colonic irritability and
spasticity.
1330 Propantheline is an anticholinergic, antispasmodic medication that
. decreases vagal stimulation and pancreatic
secretions
1331 Propantheline is contraindicated in paralytic ileus, therefore the nurse should be
. concerned with the absent bowel sounds.
1332 Propantheline Side effects are urinary retention, constipation, and
. tachycardia.
1333 The proper suctioning technique is to insert the suction 1 to 2 cm, then begin applying intermittent
. catheter until resistance is met, withdraw the catheter suction while withdrawing the catheter. The
suction catheter is inserted approximately 5 to
6 inches. It is not necessary to insert the
catheter as the client exhales. Coughing by a
client does not necessarily indicate when to
begin or stop suctioning.
1334 Protamine sulfate is an antidote to heparin
.

1335 Protein catabolism causes increased levels of urea, phosphate, and potassium.
.

1336 Prothrombin synthesis in the liver requires vitamin K. phytonadione (vitamin K1) to promote
. In cirrhosis, vitamin K is lacking, precluding prothrombin synthesis.
prothrombin synthesis and, in turn, increasing the
client's PT. An increased PT, which indicates clotting
time, increases the risk of bleeding. Therefore, the nurse
should expect to administer
1337 Prune juice is a natural laxative that stimulates On the third postoperative day following
. peristalsis, and warming the prune juice (B) facilitates thoracic surgery, a client reports feeling
peristalsis. (A) is also helpful in promoting peristalsis constipated. Which intervention should the
but is less likely to relieve the client's constipation. (C) nurse implement to promote bowel
reduces discomfort during ambulation, but will not help elimination?
relieve the client's constipation. Defecation is not painful A. Remind the client to turn every two hours
following most surgeries, and many analgesics used while lying in bed.
postoperatively cause constipation, so (D) is B. Provide warm prune juice before the client
contraindicated. goes to bed at night.
Correct Answer: B C. Teach the client to splint the incision while
walking to the bathroom.
D. Administer an analgesic before the client
attempts to defecate.
1338 pseudoephedrine (Sudafed) Common cardiovascular adverse effects tachycardia, hypertension,
. include palpitations, and arrhythmias.
Tachycardia,
1339 pseudoephedrine (Sudafed) The most common CNS adverse effects restlessness, dizziness, tension,
. include anxiety, insomnia, and weakness.
1340 The pt with a fractured left humerus reports dyspnea and chest pain, fat embolism syndrome
. pulese ox 88%, temp 100.2, HR 110, resp 32. the nurse suspects the
client is experiencing
1341 Pulmonary edema can develop during the oliguric phase of acute renal
. failure because of decreased urine
output and fluid retention.
1342 Pulmonary Embolism 1. pulmonary arteries by a
. 1. Obstruction of 1 or more....... thrombus or thrombi
2. Originates in..... 2. the venous system/right side of
the heart
1343 Pulmonary Embolism Diagnostic Studies -ABGs
. -Ventilation-perfusion scan
-Pulmonary angiogram
-Spiral CT
-D-dimer

-Prevention
-Rapid recognition
-Treatment
1344 Pulmonary Embolism Etiology 1. Venous stasis
. Thrombus forms when there is what???? 2. Vessel wall injury
a) whats the most common cause ? 3. Hypercoagulability of the
1. ________________ blood
2. __________________ 4.*** Emboli originating from
3._______________________ DVT most common cause****
4._____________________________________
1345 Pulmonary Embolism Management 1.
. 1. Medications Heparin, LMWH, then warfarin
Fibrinolytic agents, like tPA 1st 3-
=Supportive care= 4 hours)
2. Administer what ? Thrombolytic agents--
3. what position should you place them in streptokinase
4. If patient goes into shock, what kind of med do you give?
5. If patient goes into heart failure, what kind of med do you give ? 2. O2, how depends on ABG's
6.___________ hygiene 3. High Fowler's position
7. Provide_____________support ? 4. vasopressors
8. what can you put on their legs 5. diuretics
9. the patient may need ? 6. Pulmonary
7. emotional
8. Teds/P-Cuffs
9. Surgery
1346 Pulmonary Embolism Risk Factors 1. immobility
. 1. Prolonged ______________ 2. lung disease
2. Chronic....... 3. Heart failure/atrial fibrillation
3. what conditions of the heart can cause it ? 4. Lower extremity
4. what kind of surgeries can cause it ? surgery/abdominal
5. can Pregnancy cause it ? surgery/trauma
6. what bad habit can cause it? 5. yes
7. Most common sites: 6. Smoking
7.
Legs
Pelvic, hepatic, renal veins
Right side of heart
Upper extremities
1347 A pulmonary embolus is suspected in a Spiral (helical) CT scan
. patient with a deep-vein thrombosis who
develops hemoptysis, tachycardia, and
pleuritic chest pain, and diagnostic testing
is scheduled. The nurse plans to teach the
patient about:
1348 pulse wave The progressive increase of pressure radiating through the
. arteries that occurs with each contraction of the left ventricle
of the heart.
1349 pulsus pardaxous an abnormally large decrease in systolic blood pressure and
. pulse wave amplitude during inspiration. The normal fall in
pressure is less than 10 mm Hg.
1350 pursed liped breathing Breathe in normally through your nose for 2 counts (while
. counting to yourself, one, two)."
"Relax your neck and shoulder muscles."
"Pucker your lips as if you were going to whistle."
"Breathe out slowly through pursed lips for 4 counts (while
counting to yourself, one, two, three, four)."
1351 Push 2 cc of air quickly into the stomach You should hear a "whooshing" sound as the air enters the
. while listening over the infant's stomach stomach (see illustration). You should also be able to
with a stethoscope. withdraw the air you pushed in.
1352 Pustules are elevated lesions less than 1 cm in diameter containing purulent material; examples
. include impetigo and acne lesions.
1353 pvd Although heat promotes vasodilation, avoided to reduce the risk of thermal injury secondary to
. use of a heating pad is to be diminished sensation.
1354 pvd and exercise Exercise - exercise may improve arterial blood flow to the
. affected limb. so you need it Exercise is not recommended for
people with severe rest pain, venous ulcers, or gangrene.
1355 pvd Elevating the extremities counteracts the forces of gravity and promotes venous return
. and reduces venous stasis.so its bad
1356 pyloric spincter
.

1357 pyloric spincter


.

1358 Pyloric stenosis involves hypertrophy of the pylorus muscle distal to the stomach and
. obstruction of the gastric outlet resulting in vomiting, metabolic
acidosis, and dehydration.
1359 Pyrazinamide (PZA) 1. bactericidal
. 1. Action: 2. 20-25 mg/kg
2. Dose: ____to _____ mg/kg 3. oral contraceptives, seizure medications and, anticoagulants
children and adults 4. hepatitis, gastrointestinal distress, rash, joint aches, and
3. Interactions: hyperuricemia (gout)
4. Major adverse effects:
1360 quad position 30 degrees
.

1361 Quantification of T-lymphocytes is HIV.


. used to monitor the effectiveness of
treatment for
1362 question about imobilized patient weight bearing excersises remember it does not necessarily mean
. and osteoporosis what should you do just walking
1363 question asked if someone with hep the answer was irritablity because that implies hepatic
. b which is worse fatigue when encephalthypy from liver having to much ammonia
walking or iritability and
drowsniness
1364 question said someone had choose tpn because its parenteral versus jejunostomy which is
. hemicolectomy and was having enteral not as much nutrients
issues with eating what is needed tpn
or jejunostomy
1365 Q-waves may become evident when infarction.
. the injury progresses to
1366 The radiated skin area needs to be clean, dry, and open to air.
. kept
1367 radiation care Clients should use an electric razor, instead of a straight-edge razor, on any skin areas that
. are receiving radiation.
1368 radiation Lotion should be removed from the skin before any treatment and then reapplied after
. the treatment.
1369 radiation The skin should be cleaned mild soap, not harsh antibacterials.
. daily with a
1370 Raloxifene adverse effects is headaches.
. increased
1371 Raloxifene hydrochloride (Evista), bone mineral density without stimulating the endometrium.
. an estrogen receptor modulator,
increases
1372 Raloxifene hydrochloride (Evista) is smoke cigarettes or who have a history of venous thrombosis.
. contraindicated for women who
1373 Raloxifene hydrochloride (Evista) is osteoporosis in postmenopausal women
. useful in preventing
1374 Ranitidine blocks secretion of hydrochloric acid.
.

1375 Ranson's criteria is a clinical severity of acute pancreatitis and prognosis


. predictor scale used to assess the
1376 Rapid filling of the ventricle causes vasodilation that a nurse auscultates as an S3.
.

1377 Rapid volume expansion, IVH.


. hypercarbia, and hypoglycemia
contribute to the development o
1378 Raynaud's disease An adverse effect orthostatic hypotension. The client should report dizziness and low
. of reserpine is blood pressure as it may be necessary to consider stopping the drug
1379 Raynaud's disease The nurse can warm water or placing them next to the body, such as under the
. teach the client to rewarm exposed axilla. It is not realistic to ask this client to change jobs at this time.
extremities by using
1380 Raynaud's disease The nurse should smoking because nicotine is a vasoconstrictor. \
. instruct the client to stop
1381 A recent immigrant from Vietnam is c) Developing a list of people with whom the client has had contact
. diagnosed with pulmonary
tuberculosis (TB). Which
intervention is most important for the
nurse to implement with this client?

a) Client teaching about the cause of


TB
b) Reviewing the risk factors for TB
c) Developing a list of people with
whom the client has had contact
d) Client teaching about the
importance of TB testing
1382 RECOMBIVAX HB is a sterile RECOMBIVAX HB Hepatitis B Vaccine
. suspension for intramuscular
injection. ...
1383 The recommended emergency immersion in cool water or application of clean, cool wet packs.
. treatment for a heat burn is This treatment helps relieve pain and diminishes tissue damage by
cooling the tissue.
1384 The recommended emergency cool water or application of clean, cool wet packs. This treatment
. treatment for a heat burn is helps relieve pain and diminishes tissue damage by cooling the
immersion in tissue.
1385 The recommended procedure for contracting (pulling in) the abdominal muscles and taking a slow,
. teaching clients postoperatively to deep breath through the nose. This breath is held 3 to 5 seconds,
deep breathe includes which facilitates alveolar ventilation by improving the inspiratory
phase of ventilation.
1386 Rectal surgery is accompanied by severe pain resulting from spasms of sphincters and muscles.
. Therefore, controlling pain is a priority goal of nursing care.
1387 Red blood cells should measure 0 to 3 per high-power field; white blood cells, 0 to 4 per high-
. power field.
1388 A reduction in pulmonary artery pulmonary congestion and lung sounds.
. pressures should improve the
1389 The registered nurse is taking shift State your name and role after entering the patient's room.
. report and finds that one of her There are several guidelines to consider when interacting with a
assigned patients is blind. The nurse person who is blind or has low vision. Identify yourself my stating
recalls that the best way to approach your name and role, before touching or making physical contact
a patient who is blind is to what? with the patient. When talking to the person, speak directly at him
or her using a normal tone of voice. There is no need to raise your
voice unless the person asks you to do so. Do not approach or
distract the guide dog unless the owner has given permission or
instructed you to do so.
1390 Reglan does not decrease gastric acid secretion, as do histamine-receptor blockers.
.

1391 Regular exercise for those with alternative small vessels (collateral flow) and the limitation in
. claudication helps open up walking often improves.
1392 Rehabilitation efforts are client's condition is stabilized.
. implemented as soon as the
1393 Removal of the drainage hematoma, abscess formation, and infections
. fluids assists in wound
healing and is intended to
decrease the incidence of
1394 Removing the sheath bradycardia. The nurse should have atropine on hand to increase the client's
. after cardiac heart rate if this occurs
catheterization may cause
a vasovagal response,
including
1395 renal arteriogram
.

1396 renal failure protein breakdown.


. Carbohydrates provide
energy and decrease the
need for
1397 Repositioning the client dependent lung areas.
. every 2 hours helps
prevent secretions from
pooling in
1398 Reserpine is used to treat high blood pressure by lowering so there is risk it could drop too much
.

1399 Resistance to one of the Primary drug resistance; Primary drug resistance to one of the first-line
. first-line antituberculoic antituberculoic agents is people who have not had previous treatment.
agents in people who Secondary or acquired drug resistance is resistance to one or more
have not had previous antituberculoic agents in patients undergoing therapy. Multidrug resistance is
treatment is: resistance to two agents, isoniazid (INH) and rifampin
1400 resp acidosis is build up of acid which can cause heart problems and shock
.
1401 resp alkalosis
.

1402 respiratory
.

1403 respiratory
.

1404 respiratory excursion Assessment of the movement of the chest during respiration
.

1405 Restlessness is an early indicator of hypoxi


.

1406 The result of this shift is hypovolemic shock and edema formation.
.

1407 The resurgence in TB resulting from the Poor compliance with drug therapy in patients with TB
. emergence of multidrug-resistant strains of
Mycobacterium tuberculosis was primarily
the result of
1408 The resurgence in tuberculosis (TB) Poor compliance with drug therapy in patients with TB
. resulting from the emergence of multidrug-
resistan strains of Mycobacterium
tuberculosis was primarily the result of:
1409 Reusing a suction catheter is not consistent aseptic technique.
. with
1410 revascularization procedure for The nurse should avoid placing the affected extremity on a
. arteriosclerosis hard surface, such as a firm mattress, to avoid pressure
ulcers.
1411. Rifampin is used to treat tuberculosis
1412 Rifampin (RIF) 1. bactericidal
. 1. Action: 2.10 mg/kg 600 mg QD (max)
2. Dose: Adult- ___ mg/kg ____ mg QD 3. 10-20 mg/kg
(max) 4. oral contraceptives, seizure medications, anticoagulants,
3. Pediatric - ___to ____ mg/kg methadone, steroids, protease inhibitors (PIs)
4. Interactions: 5. gastrointestinal upset, hepatitis, skin rash, bleeding
5. Major adverse effects: problems,
6. Note: RIF may result in body fluid.... flu-like symptoms
6. discoloration (red/orange) and permanent discoloration
of contact lenses
1413 rifampin (Rifadin) side effects Maintaining follow-up monitoring of liver enzymes.
. Avoiding alcohol intake.
The urine may have an orange color.
1414 right hemecolectomy
.

1415 Right ventricle Pulmonary vein


. Pulmonary artery Left atrium
Arterioles and alveoli
1416 Risk factors for pulmonary embolism Research has demonstrated an increased risk
. of pulmonary embolism in women
associated with obesity, heavy cigarette
smoking, and hypertension. Other risk
factors include immobilization, surgery
within the last 3 months, stroke, history of
DVT, and malignancy.
1417 risk for colorectal cancer crohns and ulcerative
.

1418 The risk of developing chorioamnionitis increases with each vaginal examination including during
. labor
1419 a routine ultrasound is valuable in locating the placenta, locating a
. pool of amniotic fluid, and showing the
physician where to insert the needle.
1420 The S2 results from closing of the aortic and pulmonic valves.
.

1421 A salivary fistula is suspected when there i saliva collecting beneath skin flaps or
. leaking through the suture line or drain site.
1422 Salivary fistula or skin necrosis usually precedes carotid artery rupture
.

1423 Salmeterol (Serevent) is a beta2-agonist, maintenance drug that the asthmatic client
. uses twice daily, every 12 hours.
1424 sars Airborne and contact precaution
.

1425 SARS, a highly contagious viral respiratory illness, is placed on airborne and contact precautions
. spread by
1426 Scabies is an easily spread skin disease caused by a very small species of mite
.

1427 Scatter rugs (C) pose a safety hazard because the client The home health nurse visits an elderly
. can trip on them when ambulating, so this finding has the female client who had a brain attack three
greatest significance in planning this client's care. months ago and is now able to ambulate with
Psychological support of the caregiver (A) is a less acute the assistance of a quad cane. Which
need than that of client safety. The nurse needs to obtain assessment finding has the greatest
more information about (B), but this is not a safety issue. implications for this client's care?
(D) is not a significant increase, and additional assessment A. The husband, who is the caregiver, begins
might provide information about the reason for the to weep when the nurse asks how he is
increase (anxiety, exercise, etc.). doing.
Correct Answer: C B. The client tells the nurse that she does not
have much of an appetite today.
C. The nurse notes that there are numerous
scatter rugs throughout the house.
D. The client's pulse rate is 10 beats higher
than it was at the last visit one week ago.
1428 Seconal 0.1 gram PRN at bedtime is prescribed for rest. 1 tablet
. The scored tablets are labeled grain 1.5 per tablet. How
many tablets should the nurse plan to administer?
1429 Secondary or second intention healing occurs in open wounds. When the wound edges are not
. approximated and it heals with formation of
granulation tissue, contraction and eventual
spontaneous migration of epithelial cells.
1430 Second intention healing occurs in infected wounds or wounds with edges that
. aren't approximated. These wounds are
usually packed with moist dressings
1431 sedentary, obese, middle-aged client is recovering from Aerobic activity.
. a right iliac blood clot. The nurse should develop a Weight control.
discharge plan
1432 The Seldinger maneuver is a method of percutaneous introduction of a catheter into a
. vessel.
1433 The Sengstaken-Blakemore tube has a gastric and an inflated to compress bleeding esophageal
. esophageal balloon that are varices. An inflated esophageal balloon prevents
swallowing. Therefore, the nurse should provide
the client with tissues and encourage him to spit
into the tissues or an emesis basin.
1434 Sensorineural hearing loss (SNHL) is a type of hearing vestibulocochlear nerve (Cranial nerve VIII),
. loss in which the root cause lies in the the inner ear, or central processing centers of the
brain.
1435 sensorineural loos impaired cochlea or 8th cranial nerve failure of
. sound impulses in inner ear or brain
1436 septal region of the left ventricle.
.

1437 Septic shock can be broken down into two different warm (or hyperdynamic) shock and cold (or
. types of shock: hypodynamic) shock.
1438 Serevent can be used to prevent exercise-induced bronchospasms, but it should
. be taken 30 to 60 minutes before exercise. If the
client is taking
1439 Serevent twice daily, it should not be used in additional doses before exercise; twice daily is
. the maximum dosage.
1440 Serous drainage is clear watery plasma, so (C) provides The nurse removes the dressing on a client's
. accurate documentation based on the information heel that is covering a pressure sore one-inch in
provided. Information to stage this pressure score (A) diameter and finds that there is straw-colored
is not provided, and sero-sanguineous drainage is pale drainage seeping from the wound. What
and watery with a combination of plasma and red cells, description of this finding should the nurse
and may be blood-streaked. Exudate (B) is fluid such include in the client's record?
as pus and serum. Purulent drainage (D) is thick, A. Stage 1 pressure sore draining sero-
yellow, green, or brown indicating the presence of dead sanguineous drainage.
or living organisms and white blood cells. B. Pressure sore at bony prominence with
Correct Answer: C exudate noted.
C. One-inch pressure sore draining serous fluid.
D. Pressure sore on heel with a small amount of
purulent drainage.
1441 Serum albumin has a long half-life and is the best long-term A client with chronic renal disease is
. indicator of the body's entry into a catabolic state following admitted to the hospital for evaluation
protein depletion from malnutrition or stress of chronic prior to a surgical procedure. Which
illness (C). While (A) is a good indicator of iron-binding laboratory test indicates the client's protein
capacity in a healthy adult, it is an unreliable measure in the status for the longest length of time?
client with a chronic illness. (B) has a short half-life, and is a A. Transferrin.
sensitive indicator of recent catabolic changes, but it is not B. Prealbumin.
as effective as (C) in indicating long-term protein depletion. C. Serum albumin.
While (D) is a good indicator of a negative nitrogen balance, D. Urine urea nitrogen.
it is not as good an indicator of long-term protein catabolism
as is (C).
Correct Answer: C
1442 Serum CK-MB levels can be detected 4 to 6 hours after the onset of chest pain.
. These levels peak within 12 to 18 hours
and return to normal within 3 to 4 days.
1443 Severe peripheral artery disease would result in a score of 0.00 to 0.40
.

1444 Severe peripheral artery disease would result in a score of 0.00 to 0.40.
.

1445 A severe thunderstorm has moved into a small community, Move clients and visitors into the hallways
. and the tornado warning alarm has been activated at the and close all doors to client rooms.
local hospital. Which action should the charge nurse in the
surgical department implement first?
1446 shoes to prevent falls thin nonslip soles
.

1447 siadh has high urine osmolarity siadh body retains water so there is less
. water being released meaning more solutes
just thing they are inversely related
1448 side effects of clonidine are Dry mouth, impotence, and sleep
. disturbances are possible adverse effects.
1449 A sign of digitalis toxicity is atrial fibrillation, sometimes 100 bpm, nurse is to evaluate the cardiac
. with a heart rate of more than rhythm of the client. Tachycardia can be a
sign of digitalis toxicity.
1450 The signs and symptoms of diabetic ketoacidosis include Kussmaul respirations, fruity breath,
. tachycardia, abdominal pain, nausea,
vomiting, headache, thirst, dry skin, and
dehydration
1451 signs of glomerulonephritis Periorbital edema,hematuria (not green-
. tinged urine), proteinuria, fever, chills,
weakness, pallor, anorexia, nausea, and
vomiting, hypertension ,oliguria or anuria
(not polyuria), headache, reduced visual
acuity, and abdominal or flank pain.
1452 Signs of hip prosthesis dislocation include: acute groin pain in the affected hip,
.

1453 Signs of hypocortisolism include vomiting, increased weakness,


. dehydration, and hypotension.
1454 A SINGLE episode of urinary tract infection is not as likely calculi formation as a routinely low fluid
. to lead to intake.
1455 Six days after a heart-lung transplant, the patient develops a Acute rejection that can be treated with
. low-grade fever and a decreased SpO2 with exercise. The corticosteroids
nurse recognizes that this may indicate:
1456 Skeletal traction is not used to pull weight with a boot.
.

1457 Skin and underlying structures may become anoxic after less than 2 hours of unrelieved pressure.
.

1458 Skin turgor is assessed by pinching the skin and observing When assessing a client with a nursing
. for tenting. This finding confirms the diagnosis of fluid diagnosis of fluid volume deficit, the nurse
volume deficit, so the nurse should continue interventions to notes that the client's skin over the sternum
restore the client's fluid volume (C). Jugular vein distention "tents" when gently pinched. Which action
(A) is a sign of fluid volume overload. High protein snacks should the nurse implement?
(B) will not resolve the fluid volume deficit. Changes in the A. Confirm the finding by further assessing
client's skin integrity are not evident (D). the client for jugular vein distention.
Correct Answer: C B. Offer the client high protein snacks
between regularly scheduled mealtimes.
C. Continue the planned nursing
interventions to restore the client's fluid
volume.
D. Change the plan of care to include a
nursing diagnosis of impaired skin
integrity.
1459 Slow, steady walking is a recommended activity for clients stimulates the development of collateral
. with peripheral vascular disease because it circulation.
1460 A small waxy nodule with pearly borders may indicate a basal cell carcinoma.
.

1461 Smoking and ciliary action Smoking decreases the ciliary action in the
. tracheobronchial tree, resulting in impaired
clearance of respiratory secretions, chronic
cough, and frequent respiratory infections.
1462 A soft toothbrush, Toothette, or gauze pad should be used to stomatitis
. provide oral hygiene at least every 2 hours to promote client
comfort and prevent
1463 so if there is impairment in peripheral sensation problems they have problems with movement and
. that means that sensation like hot and cold
1464 Someone after CABG will have what things on/in/coming -Pulmonary artery catheter for measuring
. out of them after surgery? CO, other hemodynamic parameters
-Intraarterial line for continuous BP
monitoring
-Pleural/mediastinal chest tubes for chest
drainage
-Continuous ECG monitoring to detect
dysrhythmias (esp. atrial dysrhythmias)
-Ventilator (Extubation within 12 hours)
-Epicardial pacing wires (pacemaker) for
emergency pacing of the heart
-Foley
-NG tube for gastric decompression
1465 someone with diarrhea has metabolic acidosis which is associated with
. loss of bicarbonate which means there is
loss of base so then there is more acid
1466 The sounds of sirens announce the arrival of the latest Adrenal glands-->The adrenal medulla
. trauma to address the trauma center where you practice secretes epinephrine and norepinephrine.
nursing. Your heart is pounding, your mouth is dry. What These two hormones are released in
gland is responsible for your physiologic response? response to stress or threat to life. They
facilitate what has been referred to as the
fight-or-flight response
1467 Spermicidal agents work by destroying the spermatozoa before they enter the cervix
.

1468 Spinal fusion, also known as spondylodesis or join two or more vertebrae. Supplementary
. spondylosyndesis, is a surgical technique used to bone tissue, either from the patient
(autograft) or a donor (allograft), is used in
conjunction with the body's natural bone
growth (osteoblastic) processes to fuse the
vertebrae.
1469 Splenomegaly often mononucleosis and is present 2 to 4 weeks after contracting the infection.
. accompanies To prevent splenic rupture, contact sports and vigorous exercise should be
avoided.
1470 A splinter hemorrhage is linear red or brown streak in the nail bed.
.

1471 Spontaneously occurring hives.


. wheals occur in.
1472 Squamous cell carcinoma face, the ears, the dorsa of the hands and forearms, and other sun-damaged
. commonly develops on the areas.
skin of the
1473 squamous cell carcinoma opaque, firm nodules with indistinct borders, scaling, and ulceration.
. Early lesions appear are
1474 stage 1
.

1475 stage 2
.

1476 stage 3
.

1477 stage 4
.

1478 Steroids are used in severe decrease the incidence of bleeding


. flare-ups because they can
1479 Steroid use causes calcium to bone, suppressing parathyroid hormone.
. leave
1480 The stoma does not need to be skin surrounding the stoma needs to be kept clean and dry.
. kept clean and dry; rather the
1481 Stomal stenosis may be suprasternal and intercostal retractions and difficult breathing.
. present when there is
1482 stomatitis Commercial high alcohol content that is irritating to inflamed mucosa.
. mouthwash is contraindicated
because of
1483 stomatitits Lemon-glycerin drying and also can promote bacterial growth.
. swabs should be avoided
because they are
1484 Stool Hematest detects blood, colorectal cancer; however, the test doesn't confirm the diagnosis.
. which is a sign of
1485 straight catheter Prepare the client and equipment, create a sterile field, put on gloves, clean
. the urinary meatus, and insert the catheter until urine flows
1486 Stress incontinence is losing physical activity, such as coughing, sneezing, laughing, or exercise.
. urine without meaning to
during
1487 STRESS TEST Clients need to abstain from eating and drinking for only 4 hours before
. the test.
1488 Stridor occurs as a result of a obstructed larynx or trachea; stridor can be heard without auscultation.
. partially
1489 subdural hematoma is a blood on the surface of the brain.
. collection of
1490 subtotal gastrectomy 24 hours apply suction continuously not every hour.
. ago. The client has a nasogastric
(NG) tube. The nurse should:
1491 suctioning a tracheostomy tube sterile catheter each time the client is suctioned.
. The recommended technique is
to use a
1492 Sulfasalazine can cause gastrointestinal distress and is best taken after meals and in equally
. divided doses.
1493 Sulfasalazine gives alkaline orange-yellow color, but it is not necessary to stop the drug when this
. urine an occurs.
1494 Sulfisoxazole and other antidiabetic agents and may precipitate hypoglycemia.
. sulfonamides are chemically
related to oral
1495 Surgery is required to release strangulation.
. the
1496 Swallowing is regulated by Medulla oblongata
. which area of the central
nervous system (CNS)?
1497 Swelling gradually subsides soon as nasal packing is removed
. over several weeks; the client
can gently clean the nares as
1498 The "swing through" gait is weight on their legs while the crutches are moved one stride ahead.
. often used by paraplegic clients
because it allows them to place
1499 Swing-to gait: both crutches are advanced forward together; weight is shifted onto hands for support and
. both legs are then swung forward to meet the crutches; requires the use
of two crutches or a walker; indicated for individuals with limited use of
both lower extremities and trunk instability.
1500 symptoms of esophageal cancer lump in throat dysphagia, substernal pain, regurgitation of undigested
. include food, foul breath, and hiccups.
1501 syphyllis 1st stage chancres are:
. On the vulva (outside the vagina) or on the cervix (neck of the womb) in
women.
On the penis in men.
Around the anus and mouth (both sexes).
1502 A systolic, harsh, crescendo- aortic stenosis.
. decrescendo murmur occurs
with
1503 Tailor sitting and squatting help perineal muscles in preparation for labor.
. stretch the
1504 The taking in phase is the first period after delivery emphasis on reviewing and reliving the L & D
. process, concern with self and needing to be mothered.
1505 TB 1. Mycobacterium tuberculosis
. 1. Infectious disease caused by.... 2. Lungs, larynx, kidneys, meninges, bones, adrenal glands, lymph
2. Involves what parts of the nodes
body ? 3. HIV infection
3. High rates of TB with.... 4. strains of
4. Multidrug-resistant what? M. tuberculosis
1506 TB Bacteriologic studies
. Bacteriologic studies 1. sputum smears examined for
1. Stained....... acid-fast bacilli
2. Required...... Required for diagnosis *********
3. three separate........ 3. samples on 3 consecutive days required for confirmation
Bacteriologic studies to stop meds
4. On different days, three consecutive Bacteriologic studies
samples are collected from 4.
-Gastric washings
5. QuantiFERON-TB (QFT) -CSF
-Fluid from an abscess or effusion
5. QuantiFERON-TB (QFT)
-New test
-Rapid blood test (few hours)
-Does not replace cultures
1507 TB Etiology and Pathophysiology 1. via airborne droplets when infected person
. 1, how is it Spread? Coughs, Speaks, Sneezes, Sings
=Spread= 2. hands or objects
2. Not by..... 3. rarely causes infection
3. Brief exposure..... 4. close, frequent, or prolonged exposure
4. Transmission requires..... Spread
=Spread= 5. pass down bronchial system and implant themselves on
5. Inhaled bacilli...... bronchioles or alveoli
6. Multiply with..... 6. no initial resistance
1508 tb is airborne spread so if its in the air you can only getting from coughing sneezing
. not from plates plates is not air
1509 TB Without sufficient immune response
. Without sufficient immune response 1. maintained
1. Organism is not___________ 2. primary disease results
2. Active........ 3. Immunosuppressed and diabetic
3. ____________________ & ____________ Dormant TB organisms persist for years
patients are at higher risk for disease 4. ever develop TB
Dormant TB organisms persist for years Reasons for reactivation are not well understood
4. Few...... Classes
Classes 0 = No TB exposure
0, 1, 2, 3, 4,.....name these. 1 = Exposure, no infection
2 = Latent TB, no disease
3 = TB, not clinically active
4 = TB suspected
1510 The temporal lobe controls hearing, language comprehension, and storage and memory
. recall (although memory recall is also stored throughout the
brain)
1511. Terbutaline must be taken every 4 to 6 hours around-the-clock to maintain an effective
serum level that will suppress labor.
1512 Testosterone is an androgen hormone that is protein metabolism as well as maintenance of secondary
. responsible for sexual characteristics; therefore, it is needed by both males
and females.
1513 . The loss of color vision, or achromatopsia, stroke damages the fusiform gyrus. It most often affects
. is a rare symptom that occurs when a only half of the visual field.
1514 theophylline It makes the central respiratory center more sensitive to
. carbon dioxide and stimulates the respiratory drive.
1515 Therapeutic positioning for identifies the best position for the patient assuring stable oxygenation
. adequate breathing status. Research indicates that positioning the patient with the unaffected
lung (good lung) dependent best promotes oxygenation is patients with
unilateral lung disease. For bilateral lung disease, the right lung down has
best ventilation and perfusion. Increasing fluid intake and performing
postural drainage will facilitate airway clearance, but positioning is most
appropriate to enhance oxygenation.
1516 The therapeutic serum 10 to 20 mcg/ml.
. theophylline concentration
ranges from
1517 There are two kinds of contact irritant or allergic.
. dermatitis:
1518 Therefore, an SaO2 of 90% oxygenation, an adverse effect of moderate sedation. The nurse should
. indicates inadequate respond by attempting to arouse the client, assisting the client with deep
breathing, and administering a higher dose of oxygen.
1519 There is no need for the client peristaltic bowel activity should not be affected.
. to remain on nothing-by-mouth
status after laparoscopic
cholecystectomy surgery
because
1520 There is no need for the client bland diet after a laparoscopic cholecystectomy. The client should,
. to stay on a however, avoid excessive fats.
1521 there were 2 question about for the conscious client you can sit them up for the unconscious they
. best position for someone with need to be turned to the side
aspiration risk one was
unconscious one was conscious
1522 Thiazide diuretics, not loop sodium secretion into the distal tubule.
. diuretics, promote
1523 Thickening of the intima and atherosclerosis.
. media of the artery is
characteristic of
1524 Third intention healing wound is allowed to heal open for a few days and then closed as if
. (delayed primary) occurs when primarily. Such wounds are left open initially because of gross
a contamination.
1525 This is achieved by competing thick ascending limb is dependent on sodium and chloride concentrations
. for the Cl- binding site. Because they are also lost in the urine
magnesium and calcium
reabsorption in the
1526 This score is between 0.71 and mild peripheral artery disease.
. 0.90, which suggests .
1527 This score is between 0.71 and mild peripheral artery disease.
. 0.90, which suggests
1528 This vasoconstriction may PVD is the greatest.
. increase pain in the areas
where
1529 Three-point gait both crutches and involved leg are advanced together, then uninvolved
. leg is advanced forward; indicated for use with involvement of one
extremity , e.g. lower extremity fracture.
1530 Thrush is characterized by whitish yellow patches in the mouth
.

1531 Thyroidectomy may lead to


.

hypoparathyroidism if the parathyroid is also removed during


surgery.
1532 The thyroid gland doesn't regulate blood glucose levels; therefore, signs and symptoms of
. hypoglycemia aren't relevant for this client. Dehydration is seen
in diabetes insipidus.
1533 thyrotoxicosis low thyroid
.

1534 TIS, N0, M0 denotes carcinoma in situ, no abnormal regional lymph nodes,
. and no evidence of distant metastasis
1535 To avoid a falsely elevated serum 8 hours after administering oral digoxin and at least 6 hours after
. digoxin level, a nurse shouldn't draw a administering I.V. digoxin
blood sample for at least
1536 To check for arterial insufficiency a) dependent pallor.
. when a client is in a supine position, Explanation:
the nurse should elevate the extremity If arterial insufficiency is present, elevation of the limb would
at a 45-degree angle and then have the yield a pallor from the lack of circulation. Rubor and increased
client sit up. The nurse suspects venous filling time would suggest venous problems secondary to
arterial insufficiency if the assessment venous trapping and incompetent valves.
reveals:
a) dependent pallor.
b) elevational rubor.
c) a 30-second filling time for the veins.
d) no rubor for 10 seconds after the
maneuver.
1537 To compensate for the patient's catecholamines increase cardiac output and myocardial
. profoundly diminished plasma volume, contractility. But these effects won't be strong enough to keep his
blood pressure up.
1538 To decrease the patient's sense of panic Stay with the patient and encourage slow, pursed-lip breathing
. during an acute asthma attack, the
best action of the nurse is to
1539 To determine pulsus paradoxus, the blood pressure in either arm as the client slowly exhales and then
. nurse should measure as the client breathes normally. Unless the client has cardiac
tamponade, the two measurements are usually less than 10 points
apart.
1540 To determine whether a tension Severe respiratory distress and tracheal deviation
. pneumothorax is developing in a
patient with chest trauma, the nurse
assesses the patient for:
1541 To help control pain during coughing Raise the bed to semi-Fowler's position and position the client's
. for a client who has had a pulmonary hands so that the incision is supported anteriorly and posteriorly
lobectomy, the nurse should:
1542 Toilet-training is commonly more Hirschsprung's disease than it is for other children.
. difficult for children who have
undergone surgery for
1543 To irrigate an indwelling urinary catheter, the nurse should first The nurse is preparing to irrigate a
. apply gloves, then draw up the irrigating solution into the client's indwelling urinary catheter
syringe (B). The syringe is then attached to the catheter and the using an open technique. What action
fluid instilled, using aseptic technique (D). Once the irrigating should the nurse take after applying
solution is instilled, the client's catheter should be secured to the gloves?
drainage tubing (C). The urinary drainage bag can be emptied A. Empty the client's urinary drainage
(A) whenever intake and output measurement is indicated, and bag.
the instilled irrigating fluid can be subtracted from the output at B. Draw up the irrigating solution into
that time. the syringe.
Correct Answer: B C. Secure the client's catheter to the
drainage tubing.
D. Use aseptic technique to instill the
irrigating solution.
1544 To maintain enteric precautions, the nurse must wash her hands after touching the
. client or potentially contaminated
articles and before caring for another
client.
1545 To prevent a photosensitivity reaction, the client should avoid co-trimoxazole therapy
. direct sunlight during
1546 To prevent the spread of scabies isolate the client's bed linens until the
. client is no longer infectious
usually 24 hours after treatment
begins, wearing gloves when applying
the pediculicide and during all contact
with the client.
1547 To promote airway clearance in a patient with pneumonia, the A, B, E
. nurse
instructs the patient to do which of the following (select all that
apply)?
A) Maintain adequate fluid intake
B) Splint the chest when coughing
C) Maintain a high Fowler's position
D) Maintain a semi-Fowler's position
E) Instruct patient to cough at end of exhalation
1548 To provide moisture and loosen the necrotic tissue, the eschar A client is admitted with a stage four
. should be covered first with wet to moist dressings (C), which pressure ulcer that has a black,
are discontinued and then a hydrogel alginate can be placed in hardened surface and a light-pink
the prepared wound bed to prevent further damage of wound bed with a malodorous green
granulating any surrounding tissue. Although a hydrogel (A) drainage. Which dressing is best for
liquefies necrotic tissue of slough and rehydrates the wound bed, the nurse to use first?
it does not address wicking the purulent drainage from the A. Hydrogel.
wound. Exudate absorbers (B) provide a moist wound surface, B. Exudate absorber.
absorb exudate, and support debridement, but do not prepare C. Wet to moist dressing.
the wound bed for proper healing. Transparent dressings (D) are D. Transparent adhesive film.
used to protect against contamination and friction while
maintaining a clean moist surface.
Correct Answer: C
1549 To reduce the risk for most occupation lung Using masks and effective ventilation systems to
. diseases, the most important measure promoted reduce exposure to irritants
by the occupational nurse is:
1550 tpa on ulcer Prevent direct trauma to the ulcer.
. Prevent infection.
Reduce pain.
Increase oxygen to the tissues.
1551 tpn complication An elevated temperature can be an indication of an
. infection at the insertion site or in the catheter.
1552 The TPN solution is usually hypertonic dextrose solution.
.

1553 Tracheal breath sounds are harsh, discontinuous sounds heard over the trachea
. during inhalation or exhalation.
1554 The tracheostomy shouldn't be plugged to weaning the client from tracheal support.
. prevent tracheal dilation. This technique is used
when
1555 trach tube surgery after A nasogastric (NG) tube during surgery to allow for enteral feedings
. is usually inserted postoperatively.
1556 Transcutaneous pads should be placed on the third degree heart block.
. client with
1557 transsphenoidal hypophysectomy The nurse temperature, increased white blood cell count,
. should monitor for signs of infection, including rhinorrhea, nuchal rigidity, and persistent headache.
elevated Hypoglycemia and adrenocortical insufficiency may
occur.
1558 treat botulism You will get botulinus antitoxin.
.

1559 Treating Culture-Positive 1. 7 months..... cavitary disease...... 2 months of


. Pulmonary TB treatment *
1. The continuation phase must be extended to 2. INH, Rifapentine
____________for patients with______________ 3. , use daily or thrice-weekly regimens
and when sputum specimens are still positive
after____months of treatment. 4. * 9 months rather than 6 months
2. Use _______________once/week in the
continuation phase for selected patients
3. To prevent acquired resistance in patients
with advanced HIV, do what ?

4. * Total length of treatment


1560 The treatment for keratoconjunctivitis includes: Antibiotic eye drops
. Antibiotic eye ointment
Antiviral eye drops
Cidofovir
Artificial tears
Cold compresses
Corticosteroid eye drops:
Dexamethasone (Ocu-Dex)
Fluorometholone (Flarex, FML Forte)
Prednisolone (Pred Forte)
Rimexolone (Vexol)
1561 treatment for peripheral vascular disease Vasodilation (increasing the diameter of blood vessels)
. Promoting - Vasodilation can be achieved by providing warmth
and preventing long periods of exposure to cold. Never
apply direct heat to the limb, such as with the use of a
heating pad or extremely hot water to reduce the risk of
burns
1562 treatment of chorioamnionitis antibiotics (amoxicillin + gentamicin + metronidazole)
. for the mother, and
quickly delivering the baby
1563 Treatment of symptompatic Atropine--->Treatment of symptomatic bradycardia includes
. bradycardia includes which of the transcutaneous pacing and atropine. Lidocaine may be used in the
following? treatment of ventricular fibrillation. Cardioversion and Adenocard
may be used in patients diagnosed with atrial flutter.
1564 Tremors associated with Parkinson's imbalance between dopamine and acetylcholine
. disease are not psychogenic but are
related to an
1565 trichomonas in women Discomfort with intercourse
.
Itching of the inner thighs

Vaginal discharge (thin, greenish-yellow, frothy or foamy)

Vaginal itching

Vulvar itching or swelling of the labia

Vaginal odor (foul or strong smell)


1566 trichomonias men Burning after urination or ejaculation
.
Itching of urethra

Slight discharge from urethra


1567 Trimethoprim-sulfamethoxazole is diarrhea. Nausea and vomiting are other common adverse effects.
. most likely to cause
1568 The tube allows the suture line to heal adequately, minimizes contamination of the pharyngeal and
. esophageal suture lines, and prevents fluid from leaking through
the wound into the trachea before healing occurs.
1569 Tube feedings do not prevent swallowing or pain upon swallowing.
.
1570 Tuberculosis can be controlled but intense physical or emotional stress increase the likelihood of
. never completely eradicated from the recurrence.
body. Periods of
1571 tubes
.

1572 T-wave inversion, ST-segment hypoxia which occur during an MI


. elevation, and a pathologic Q-wave are
all signs of tissue
1573 Twelve hours after chest tube insertion Assess for kinks or dependent loops in the tubing.
. for hemothorax, the nurse notes that
the client's drainage has decreased
from 50 ml/hr to 5 ml/hr. What is the
best initial action for the nurse to take?
1574 Twin-to-twin transfusion drains blood twin to the second and is a problem that may occur with multiple
. from one gestation.
1575 Two days after surgery to amputate his left lower administer medication as ordered for the reported
. leg, a client states that he has pain in the missing discomfort
extremity, which action by the nurse is most
appropriate
1576 Two days after undergoing a total abdominal b) left calf circumference 1" (2.5 cm) larger than the
. hysterectomy, a client complains of left calf pain. right.
Venography reveals deep vein thrombosis (DVT). Explanation:
When assessing this client, the nurse is likely to Signs of DVT include inflammation and edema in the
detect: affected extremity, causing its circumference to
a) loss of hair on the lower portion of the left leg. exceed that of the opposite extremity. Pallor, coolness,
b) left calf circumference 1" (2.5 cm) larger than decreased pulse, and hair loss in an extremity signal
the right. interrupted arterial blood flow, which doesn't occur in
c) pallor and coolness of the left foot. DVT.
d) a decrease in the left pedal pulse.
1577 Two days after undergoing pelvic surgery, a Raise the head of the bed
. patient develops marked dyspnea and anxiety.
The first action the nurse should take is to:
1578 Two-point gait: one crutch and opposite extremity move together
. followed by the opposite crutch and extremity;
requires use of two assistive devices (canes or
crutches)
1579 Types of Pneumonia Aspiration pneumonia
. =Aspiration pneumonia= 1. loss of consciousness
1. Usually has history of what ? 2. Gag and cough reflexes
2. whats suppressed in these patients ? 3. Tube feeding
3. whats a risk factor for this type ? 4.
4. Name some Forms of aspiration pneumonia -Mechanical obstruction
-Chemical injury
-Bacterial infection
1580 Types of Pneumothorax Injuries 1. external wound
. = Closed (simple)= 2. spontaneous pneumothorax............. small blebs
1. No.... 3. mechanical vent., central line insertion, injury from
2. Most common is _______________________, broken ribs
rupture of ______________ on the visceral 4. woundgunshots, knives, etc
pleura, cause unknown. 5. sucking chest wound
3. Can also be from..... 6. vented dressingtaped on 3 sides, with 4th side left
=Open (traumatic)= open. (called a flutter dressing)
4. Air enters from a.....
5. Often called a....
6. Needs to be covered immediately with a.........
1581 Typically would a patient have JVD in R or L Right
. sided HF?
1582 A typical physical examination finding for a increased vocal fremitus on palpation. Other signs of
. patient with pneumonia is pulmonary consolidation include dullness to
percussion, bronchial breath sounds, and crackles in
the affected area.
1583 Typical signs of cardiogenic shock include low blood pressure, rapid and weak pulse, decreased
. urine output, and signs of diminished blood flow to
the brain, such as confusion and restlessness.
1584 under epidural anesthesia, the client states she let them go you dont have to palpate the bladder
. needs to urinate. The nurse should next: unless the patient is not peeing
1585 Under secretion of ACTH Addisons
.

1586 Under secretion of ADH Diabetes insipidus


.

1587 Under secretion of GH- growth hormone Dwarfism


.

1588 Under secretion of PTH Hypo parathyroidism


.

1589 Under secretion of TSH Hypo thyroid


.
1590 Unilateral hydronephrosis is swelling of one kidney due to a backup of urine
.

1591 Unless the pouch leaks, the client can about 4 to 7 days.
. wear her ileostomy pouch for
1592 The Unna's paste boot becomes rigid What intervention should the nurse include in the plan of care
. after it dries, so it is important to check for a client who is being treated with an Unna's paste boot for
distally for adequate circulation (A). leg ulcers due to chronic venous insufficiency?
Kerlix is often wrapped around the A. Check capillary refill of toes on lower extremity with Unna's
outside of the boot and an ace bandage paste boot.
may be used to cover both, but no B. Apply dressing to wound area before applying the Unna's
bandage should be put under it (B). The paste boot.
Unna's paste boot should be applied C. Wrap the leg from the knee down towards the foot.
from the foot and wrapped towards the D. Remove the Unna's paste boot q8h to assess wound healing.
knee (C). The Unna's paste boot acts as
a sterile dressing, and should not be
removed q8h. Weekly removal is
reasonable (D).
Correct Answer: A
1593 Upon discharge from the hospital, Morphine IV-->Upon patient discharge, there needs to be
. patients diagnosed with a myocardial documentation that the patient was discharged on a statin, an
infarction (MI) must be placed on all of ACE or angiotensin receptor blocking agent (ARB), and
the following medications except: aspirin. Morphine IV is used for these patients to reduce pain
and anxiety. The patient would not be discharged with IV
morphine.
1594 An upper GI series, or barium study, diagnostic method of choice, especially in a client with acute
. usually isn't the active bleeding who's vomiting and unstable
1595 Urethral meatal stenosis, which can meatal ulceration, possibly leading to urinary obstruction. It is
. occur in circumcised infants, results not associated with hypospadias or circumcision.
from
1596 Urine specific gravity normally ranges 1.002 to 1.035, making this client's value normal.
. from
1597 Used to visualize the lower GI tract, the detection of two-thirds of all colorectal cancers
. sigmoidoscopy and proctoscopy aid in
1598 Used to visualize the lower GI tract, detection of two-thirds of all colorectal cancers.
. sigmoidoscopy and proctoscopy aid in
the
1599 The use of ankle-high tennis shoes has
. been found to be most effective in
preventing plantar flexion (footdrop)
because they add support to the foot and
keep it in the correct anatomic position.

1600 The use of histamine2 (H2) blockers paradoxic central nervous system (CNS) stimulation, resulting
. such as cimetidine can cause in ataxia in the elderly. Impaired vision, gait, and thinking may
also occur.
1601 Usually why are women more likely to because angina is usually their precursor symptom, they are less
. die of CAD? likely to report it, which results in more frequent sudden cardiac
death
1602 Uterine atony, or relaxed uterus, may delivery, leading to postpartum hemorrhage.
. occur after
1603 Valves often becom incompetent with PVD.
.

1604 Vasodilation or vasoconstriction produced by an C) keep the client warm.


. external cause will interfere with a nurse's Explanation:
accurate assessment of a client with peripheral The nurse should keep the client covered and expose
vascular disease (PVD). Therefore, the nurse only the portion of the client's body that she's
should: assessing. The nurse should also keep the client warm
a) match the room temperature to the client's by maintaining his room temperature between 68 F
body temperature. and 74 F (20 and 23.3 C). Extreme temperatures
b) maintain room temperature at 78 F (25.6 C). aren't good for clients with PVD. The valves in their
c) keep the client warm. arteries and veins are already insufficient, and
d) keep the client uncovered. exposing them to vast changes in temperature could
affect assessment findings. Keeping the client
uncovered would cause him to become chilled.
Matching the room temperature to the client's body
temperature is inappropriate.
1605 Vasospasm lasting several minutes is Raynaud's disease.
. characteristic of
1606 Vasospastic disorder (Raynaud's disease) is a coldness, pain, and pallor of the fingertips, toes, or tip
. form of intermittent arteriolar vasoconstriction of the nose, and a rebound circulation with redness
that results in and pain.
1607 Ventricular remodeling (or cardiac remodeling) changes in size, shape, structure and physiology of the
. [1] refers to the heart after injury to the myocardium mi
1608 Vesicles are elevated, sharply defined lesions that 0.5 cm in diameter and contain serous fluid.
. are usually less tha
1609 Vesicular breath sounds are soft, low-pitched sounds normally heard over the
. lower lobes of the lung. They're prolonged on
inhalation and shortened on exhalation.
1610 Vibration and percussion of the chest wall may be respiratory hygiene but will not affect the nature of
. helpful for secretions.
1611. Vitamin A is a vitamin that is needed by the retina of the eye
1612 Vitamin B12, also called cobalamin, is a water-soluble vitamin with a key role in the normal
. functioning of the brain and nervous system, and for
the formation of blood.
1613 Vitamin B12 combines with intrinsic factor in the ileum, where it is absorbed into the bloodstream.
. stomach and is then carried to the
1614 Vitamin B12 is normally found in liver, kidney, The nurse is discussing dietary preferences with a
. meat, fish and dairy products. A vegan who client who adheres to a vegan diet. Which dietary
consumes only vegetables without careful dietary supplement should the nurse encourage the client to
planning and supplementation may develop include the dietary plan?
peripheral neuropathy due to a deficiency in A. Fiber.
vitamin B12 (D). (A, B, and C) are commonly B. Folate.
adequate in vegtables and fruits. C. Ascorbic acid.
Correct Answer: D D. Vitamin B12.
1615 Vitamin B12 needs to b injected every month because the ileum has been
. surgically removed.
1616 Vitamin D and calcium are necessary for bone healing
.

1617 Vitamin E is a powerful antioxidant that helps to prevent oxidation of the cell
. membrane.
1618 Vitamins A, C, and K; pyridoxine; riboflavin; and wound healing.
. thiamin are necessary for
1619 A waist-high bed height (A) is a comfortable and What action is most important for the nurse to
. safe working height to maintain the nurse's implement when placing a client in the Sim's
proper body mechanics and prevent back injury. position?
The head should be flat for a Sim's side-lying A. Raise the bed to a waist-high working level.
position, not raised (B). (C) is implemented after B. Elevate the head of the bed 45 degrees.
the client is positioned laterally. (D) brings the C. Place a pillow behind the client's back.
client closer to the nurse when being turned. D. Bring the client to one edge of the bed.
Correct Answer: A
1620 Wallace Guterman, a 36-year-old construction Hyperpituitarism--->Acromegaly is a condition in
. manager, is being seen by a physician in the which GH is oversecreted after the epiphyses of the
primary care group where you practice nursing. long bones have sealed. A client with acromegaly has
He presents with a huge lower jaw, bulging coarse features, a huge lower jaw, thick lips, a
forehead, large hands and feet and frequent thickened tongue, a bulging forehead, a bulbous nose,
headaches. What could be causing his symptoms? and large hands and feet. When the overgrowth is
from a tumor, headaches caused by pressure on the
sella turcica are common.
1621 Warm compresses could also increase fluid accumulation
.

1622 warm shock also makes capillaries more leakage and fluid shifting into tissues and fluid
. permeable causing shifting into tissues and physiologic third spaces.
1623 Warm shock characterized by high cardiac output and low peripheral vascular
. resistance occurs first.
1624 warm shock Vasodilation from the effects of histamine, bradykinins, serotonin, and endorphins
. dramatically decrease total peripheral vascular
resistance
1625 Water does not harm the stoma, so the client does wet.
. not have to worry about getting it
1626 Water-soluble jelly is not recommended for if the catheter is inadvertently inserted into the lungs,
. lubricating a gavage-feeding catheter becaus the jelly could damage the lung tissue or cause
pneumonia
1627 Water-soluble lubricants used during sexual reduced natural vaginal lubrication caused by ovarian
. intercourse can augment dysfunction and decreased circulating estrogen related
to chemotherapy.
1628 Weakness, tingling, and cardiac arrhythmias hyperkalemia, which is associated with renal failure.
. suggest
1629 Weight loss not gain is an indication of colorectal cancer.
.

1630 Wet or damp areas on a sterile field allow What action should the nurse implement when adding
. organisms to wick from the table surface and sterile liquids to a sterile field?
permeate into the sterile area, so the field is A. Use an outdated sterile liquid if the bottle is sealed
considered contaminated if it becomes wet (B). and has not been opened.
Outdated liquids may be contaminated and B. Consider the sterile field contaminated if it
should be discarded, not used (A). The becomes wet during the procedure.
container's cap should be removed, placed facing C. Remove the container cap and lay it with the inside
up, and off the sterile field, not (C). To prevent facing down on the sterile field.
contamination of the sterile field, liquids should D. Hold the container high and pour the solution into
be held close (6 inches) to the receptacle when a receptacle at the back of the sterile field.
pouring to prevent splashing, and the receptacle
should be placed near the front edge to avoid
reaching over or across the sterile field (D).
Correct Answer: B
1631 A wet-to-damp saline dressing should always moist.
. keep the wound
1632 We typically measure by... Count, mass, volume
.
1633 What 6 things are involved in the immediate -Decrease intravascular volume
. collaborative management of ADHF? -Decrease preload
-Decrease afterload
-Improve gas exchange
-Improve cardiac function
-Reduce anxiety
1634 What are 4 things that affect cardiac output? -Preload (right side)
. -Afterload (left side)
-Myocardial contractility
-Heart rate
1635 What are 5 suggestions that we can make to patients -Nutrition
. with HTN? -Drug therapy
-Exercise
-Home BP monitoring
-Quit Smoking
1636 What are assessments for a patient following CABG? -Assessing the patient for bleeding
. (e.g., chest tube drainage, incision sites)
-Monitoring fluid status
-Replacing electrolytes PRN
-Restoring temperature (e.g., warming blankets)
1637 What are causes of hypercalcemia? - malignant neoplastic diseases
. - hyperparathyroidism
- prolonged immoblization
- excessive intake
- immobility
- excessive intake of calcium carbonate antacids
1638 What are causes of hyperkalemia? - renal failure
. - use of potassium supplements
- burns
- crushing injuries
- severe infection
1639 What are causes of hypermagnesium? - renal failure
. - excessive magnesium administration
1640 What are causes of hypernatremia? - hypertonic tube feedings w/o water supplements
. - hyperventilation
- diabetes insipidus
- ingestion of OTC drugs such as Alka-Seltzer
- inhaling large amount of saltwater
- inadequate water ingestion
1641 What are causes of hypocalcemia? - hypoparathyroidism
. - pancreatitis
- renal failure
- steroids and loop diuretics
- inadequate intake
- post-thyroid surgery
1642 What are causes of hypokalemia? - vomiting
. - gastric suction
- prolonged diarrhea
- diuretics and steroids
- inadequate intake
1643 What are causes of hypomagnesium? - alcoholism
. - GI suction
- diarrhea
- intestinal fistuals
- poorly controlled diabetes mellitus
- malabsorption syndrome
1644 What are causes of hyponatremia? - vomiting
. - diuretics
- excessive administration of dextrose and water
IVs
- burns, wound drainage
- excessive water intake
- syndrome of inappropriate anti diuretic hormone
secretion
1645 What are common complications for someone who was -Bleeding and anemia from damage to RBCs
. on bypass? and platelets
-Fluid and electrolyte imbalances
-Hypothermia as blood is cooled as it passes
through the bypass machine
1646 What are complications of heart failure? -Pleural effusions
. -Dysrhythmias (afib most common)
-Left ventricular thrombus (causes emboli)
-Hepatomegaly (Especially RV failure)
-Renal failure
1647 What are diagnostic studies for angina? -Health history/physical examination
. -Laboratory studies
-12-lead ECG
-Chest x-ray
-Echocardiogram
-Exercise stress test
-Cardiac cath/angiography
1648 What are drugs that lower cholesterol? -statins (Lipitor, Zocor)
. -bile sequestrants (Welchol)
-decrease cholesterol absorption (Zetia)
1649 What are HTN issues with older adults? -isolated systolic HTN
. -Ausculatory gap
-white coat HTN
1650 What are lifestyle modifications that are important to -Weight reduction
. suggest to someone suffering from HTN? -DASH eating plan
-Na reduction
-Increase physical activity
-Avoid tabacco
-Reduce stress
-Moderate alcohol consumption (M-2, F-1)
1651 What are modifiable risk factors of CAD? -Hyperlipidemia
. -HTN
-smoking
-sedentary lifestyle
-obesity
-diabetes
1652 What are non modifiable risk factors of CAD? -Age
. -Gender
-Ethnicity
-Family hx
-Genetic inheritance
1653 What are other complications of an MI? -HF
. -Papillary muscle dysfunction
-ventricular aneurysm
-Acute pericarditis
1654 What are overall goals in the nursing planning for a -Decrease in symptoms (e.g., shortness of
. patient with chronic HF? breath, fatigue)
-Decrease in peripheral edema
-Increase in exercise tolerance
***Compliance with the medical regimen
-No complications related to HF
1655 What are risk factors for primary HTN? -Age
. -Smoking
-Diabetes Mellitus
-hyperlipidemia
-Excess Na+
-M Gender
-Family Hx
-Obesity
-Stress
-Sedentary life style
-Ethnicity
-Socioeconomic status
1656 What are side effects that you have to watch out for pretty -**ortho hypo
. much all antihypertensives? -Sexual dysfunction
-Dry mouth
-Frequent urination
1657 What are signs and symptoms of hypercalcemia? - lack of coordination
. - anorexia, nausea, and vomiting
- confusion, decreased level of
consciousness
- personality changes
- dysrhythmias, heart block, cardiac
arrest
1658 What are signs and symptoms of hyperkalemia? - EKG changes: peaked T waves, wide
. QRS complexes
- dysrhythmias, ventricular fibrillation,
heart block
- cardiac arrest
- muscle twitching and weakness
- numbness in hands and feet and around
mouth
- nausea
- diarrhea
1659 What are signs and symptoms of hypermagnesium? - depresses the CNS
. - depresses cardiac impulse transmission
- cardiac arrest
- facial flushing
- muscle weakness
- absent deep tendon reflexes
- paralysis
- shallow reflexes
1660 What are signs and symptoms of hypernatremia? - elevated temp
. - weakness
- disorientation
- irritibility and restlessness
- thirst
- dry, swollen tongue
- sticky mucous mebranes
- hypotension
- tachycardia
1661 What are signs and symptoms of hypocalcemia? - nervous system becomes increasingly
. excitable
- tetany: Trousseau's sign and Chvostek's
sign
- hyperactive reflexes
- confusion
- paresthesias
- irritability
- seizures
1662 What are signs and symptoms of hypokalemia? - anorexia, nausea, vomiting
. - weak peripheral pulses
- muscle weakness, paresthesias, decreased deep
tendon reflexes
- impaired urine concentration
- ventricular dysrhythmias
- increased instance of dig toxicity
- shallow respirations
1663 What are signs and symptoms of hypomagnesium? - increased neuromuscular irritability
. - tremors
- tetany
- hyperactive deep tendon reflexes
- seizures
- dysrhythmias, especially is kypokalemia present
- disorientation
- confusion
1664 What are signs and symptoms of hyponatremia? - nausea
. - muscle cramps
- confusion
- muscular twitching, coma
- seizures
- headache
1665 What are some collaborative problems from -Adverse drug effects
. primary hypertension? -Hypertensive crusis
-Stroke
-MI
1666 What are some collaborative problems with - Shock
. pneumonia? - Respiratory Failure
- Atelectasis
- Pleural Effusion
- Confusion
- Superinfection
1667 What are some strategies for adherence to drug -Empathy to increase patient trust, motivation, and
. regimen? adherence to therapy
-Consider cultural beliefs and individual attitudes
1668 What are the 3 goals in treating primary HTN? -Control BP
. -Reduce CVD risk factors
-Promote Medication adherence
1669 What are the 4 major contributors to primary -Water/Na retention
. HTN? -Stress/increased SNS activity
-Insulin resistance
-Endothelial dysfunction
1670 What are the actions for someone hospitalized with a -IV drugs titrated to MAP
. hypertensive crisis? -Monitor CV and renal function
-Neuro checks
-Determine cause
-Education to avoid future crisis
1671 What are the classifications of drugs used to treat -Diuretics
. HTN? -Adrenergic inhibitors
-Vasodilators
-ACE-inhibitors
-Angiotensin blockers
-Ca Channel blockers
1672 What are the clinical manifestations of acute -The patient usually is anxious, pale, and possibly
. decompensated heart failure (ADHF) (heart cyanotic.
failure in an acute stage)? -The skin is clammy and cold from vasoconstriction
caused by stimulation of the SNS.
-Cough with frothy, blood-tinged sputum
-Breath sounds: Crackles, wheezes, rhonchi
-Tachycardia
-Hypotension or hypertension
1673 What are the clinical manifestations of an MI -Pain - ischemia
. and why? -SNS stimulation (ashen, clammy, and cool skin,
diaphoresis)
-CV problems (BP and HR elevated at first. BP
lowered later due to decreased cardiac output, crackles,
Jugular venous distention,
Abnormal heart sounds, S3 or S4, new murmur)
-N/V- from severe pain
-Fever up to 100.4 as the body's normal response to the
inflammatory process from cell death
1674 What are the clinical manifestations of chronic -Fatigue
. heart failure? -Dyspnea
-Tachycardia
-Edema
-Nocturia
-Skin changes
-Behavior changes
-Chest pain
-Weight changes
1675 What are the clinical manifestations of primary the symptoms are often secondary to target organ
. HTN? disease but can include
-Fatigue/activity intolerance
-Dizziness
-Palpitation/angina
-DOE
1676 What are the diagnostic studies for HF? -History and physical examination
. -Chest x-ray
-ECG
-Lab studies (e.g., cardiac enzymes, BNP)
-Hemodynamic assessment
-Echocardiogram
-Stress testing
-Cardiac catheterization
-Ejection fraction
1677 What are the different stages of atherosclerosis? -fatty streaks
. -Fibrous plaque
-Complicated lesion - most severe
1678 What are the emergent things to do for someone -MONAB
. with ACS? -ECG monitoring
-Monitor VS and pulsox
-Bedrest and limitation of activity for 12-24 hours
-Possible PCI or Fibrinolytic therapy
1679 What are the first line medications for TB? - INH
. - Rifampin
- Pyrazinamide
- Ethambutol
1680 What are the four major types of 1. Community Acquired Pneumonia (CAP)
. pneumonia? 2. Hospital Acquired Pneumonia (HAP)
3. Pneumonia in the Immunocompromised Host
4. Aspiration Pneumonia
1681 What are the goals in treating someone -Treat the underlying cause and contributing factors.
. with chronic HF? -Maximize CO.
-Provide treatment to alleviate symptoms.
-Improve ventricular function.
-Improve quality of life.
-Preserve target organ function.
-Improve mortality and morbidity.
1682 What are the goals of therapy for -Decrease patient symptoms
. patients with ADHF and chronic HF? -Improve LV function
-Reverse ventricular remodeling
-Improve quality of life
-Decrease mortality and morbidity
1683 What are the main symptoms of SARS? - High fever
. - Headache
- Overall discomfort
- Body aches
- Dry cough
- Progressive hypoxemia (leads to pneumonia)
1684 What are the most common -Infection is the primary complication, followed by
. complications of heart transplant? -acute rejection, in the first year after transplantation.
-Malignancy (especially lymphoma) and coronary artery
vasculopathy are major causes of death after the first year.
1685 What are the nursing goals for someone -Relief of pain
. experiencing an MI? Read them -Preservation of myocardium
-Immediate and appropriate treatment
-Effective coping with illness-associated anxiety
-Participation in a rehabilitation plan
-Reduction of risk factors
-Health promotion
1686 What are the qualifications for HTN? Sustained systolic >140
. OR
Sustained diastolic >90
OR
On a HTN medication
1687 What are the risk factors for SCD? -Left ventricular dysfunction (EF 30%)
. -Ventricular dysrhythmias following MI
1688 What are the risk factors for TB? - Close contact with someone who has TB
. - Immunocompromised status
- Substance abuse
- Any person without adequate health care
- Immigrations from countries with a high prevalence of TB
- Living in overcrowded, substandard housing
- Being a health care worker performing high risk activities
1689 What are the signs and symptoms of - Low grade fever
. TB? - Cough
- Night sweats
- Fatique
- Weight loss
1690 What are the three main goals of -Correction of sodium and water retention and volume overload
. drug therapy with someone who -Reduction of cardiac workload
has chronic HF and what drugs -Improvement of myocardial contractility
accomplish this?
Diuretic,
RAAS inhibitors (ACE, ARB, Spironolactone)
Vasodilator (Nitro/hydralazine)
Beta blocker
positive Inotrope
1691 What are the two primary risk -CVD (HTN, CAD, MI)
. factors for HF? -Advancing age

Contributing risk factors:


-Diabetes - **predisposes a person to HF regardless of CAD or HTN
-Tobacco use
-Obesity
-High serum cholesterol
1692 What blood levels need to be serum Digoxin levels
. monitored on a patient taking serum potassium
digoxin and a thiazide or a loop
diuretic? Digoxin and potassium have an inverse relationship. If potassium is
too high, Digoxin will be suptherapeutic. If potassium is too low,
Digoxin levels rise and toxicity can occur leading to fatal
dysrhythmias.
1693 What blood pressure is 120-139/80-89
. prehypertensive?
1694 What causes alterations in Inflammation and injury to the intimal wall
. endothelial lining of vascular
system?
1695 What contributes to HF after an Proinflammatory cytokines are released, which further depresses
. MI or cardiac cell injury? cardiac function by causing hypertrophy, contractile dysfunction, and
myocyte cell death. Over time this leads to a systemic inflammatory
response that affects cardiac and skeletal muscle and is responsible
for the fatigue that accompanies advanced HF
1696 What dietary recommendations A diet rich in protein; For a patient with pleural effusion, a diet rich in
. should a nurse provide a patient protein and calories is pivoral. A charboydrate-dense diet or diets with
with a lung abscess? limited fats are not advisable
1697 What does aspiration pneumonia Pulmonary consequences resulting from the entry of endogenous or
. refer to? exogenous substances into the lower airway.
1698 What does the supportive Fluids, oxygen for hypoxia, antipyretics, antussives, decongestants,
. treatment for pneumonia include? and antihistamines.
1699 What do the coefficients of a The ratio of parts relationship (for example in C (s) + O (g) CO
. balanced chemical equation (g), coefficients= 1 part C needs 1 part O in order to make 1 part
represent? CO.)
1700 What do the inflamed mucosa of Mucopurulent Sputum
. the bronchi produce, in acute
tracheobronchitis?
1701 What happens during a ballon After catheterization (PCI) and location of the blockage, a balloon is
. angioplasty (PTCA)? inflated at the area of blockage to compress the atherosclerotic
plaque, thus dilating the vessel. After this unfractionated or low
molecular weight heparin is given to maintain the open vessel. Often
a stent is put in place to resist vasoconstriction
1702 What happens in the heart after an Ventricular remodeling - Normal myocardium will hypertrophy and
. MI in an attempt to compensate dilate
for the infarcted muscle? *takes a long time
1703 What happens to the aveoli in They become inflammed, producing an exudate that intereres
. pneumonia? with the diffusion of oxygen and carbon dioxide. White blood
cells, mostly neutrophils, also migrate and fill normally air-
containing spaces.
1704 What is ABPM? ambulatory BP monitoring - necessary for those with white coat
. phenomenon. It's a noninvasive system that measures BP at
specific intervals over a 24 hour period
1705 What is acute coronary syndrome When ischemia is prolonged and is not immediately reversible
. (ACS)?
1706 What is acute tracheobronchitis? An acute inflammation of the mucous membranes of the trachea
. and the bronchial tree.
1707 What is afterload? The pressure that the chambers of the heart must overcome in
. order to eject blood out of the heart.
1708 What is a hallmark of systolic Decrease in LV ejection fraction
. dysfunction?
1709 What is an abnormal clinical syndrome heart failure
. involving impaired cardiac pumping or
filling?
1710 What is an MI? Result of sustained ischemia
. (>20 minutes), causing irreversible myocardial cell death
(necrosis)
1711. What is a nonspecific marker of C-Reactive protein (CRP)
inflammation that can be measured
and is elevated in many patients with
CAD?
1712 What is a normal ejection fraction? 60%
.

1713 What is another name for HAP? Nosocomial Pneumonia


.

1714 What is a type of HTN that occurs with pseudohypertension - the veins can't collapse and give a falsely
. atherosclerotic patients? high reading
1715 What is calcium regulated by? - parathyroid hormone
. - vit D
1716 What is Chovstek's sign? tap facial nerve 2 cm anterior to the earlobe just below the
. zygomatic arch, twitching indicates tetany
1717 What is chronic heart failure? It is characterized as progressive worsening of ventricular
. function and chronic neurohormonal activation (compensation)
that result in ventricular remodeling. This process involves
changes in the size, shape, and mechanical performance of the
ventricle.
1718 What is chronic stable angina? Intermittent chest pain that occurs over a long period with the
. same pattern of onset, duration, and intensity of symptoms
1719 What is diastolic heart failure? Inability of the ventricles to relax and fill during diastole,
. resulting in decreased stroke volume and cardiac output
1720 What is important patient education -Medication adherence
. for patient with HF? -Daily weights
-Know when drugs (e.g., digitalis, b-adrenergic blockers) should
be withheld and reported to health care provider. (take pulse for 1
full minute, withhold if pulse below 60)
-Home BP monitoring
-Signs of hypokalemia and hyperkalemia if taking diuretics that
deplete or spare potassium
1721 What is important to monitor on any Potassium levels. All can cause hyperkalemia
. patient receiving an antihypertensive
that affects the RAAS system? (ACE,
ARB, etc.)?
1722 What is lobar pneumonia? Pneumonia that includes a substantial part portion of one or more
. lobes.
1723 What is pain from reversible angina - results when O2 demand is greater than O2 supply
. (temporary) myocardial ischemia?
1724 What is pain related to myocardial microvascular angina - more prevalent in women
. ischemia associated with abnormalities
of the coronary microcirculation?
1725 What is pneumonia? An inflammation of the lung parenchyma caused by
. various microorganisms.
1726 What is preload? the pressure generated in the LV at the end of diastole
.

1727 What is prinzmetal's angina? Pain that often occurs at rest due to a spasm of the
. coronary artery
*not usually precipitated by physical activity
*often happens in the REM cycle, or as a result of
certain substances (tobacco and histamine)
1728 What is pulmonary edema? An acute, life threatening situation in which the lung
. alveoli become filled with serosanguineous fluid as a
result of acute decompensated heart failure (ADHF)
1729 What is pulmonary tuberculosis? An infectious disease that primarily affects the lung
. parenchyma.
1730 What is SARS? A viral respiratory illness cause by a coronavirus,
. called SARS-associated cornavirus.
1731 What is secondary HTN? elevated bp with a specific cause
.

1732 What is silent ischemia? Ischemia that occurs in the absence of any subjective
. symptoms
*likely due to diabetic neuropathy
1733 What is stage 1 and stage 2 HTN? Stage 1= 140-159/90-99
. Stage 2= >160/100
1734 What is sudden cardiac death? Abrupt disruption in cardiac function, resulting in loss
. of CO and cerebral blood flow

Death usually within 1 hour of onset of acute


symptoms
1735 What is systolic heart failure? The inability of the heart to pump blood effectively
.

1736 What is the benefit of a MIDCAB? Minimally invasive and doesn't need a bypass machine
.

1737 What is the best health promotion for CAD? Since clinical manifestations usually don't happen until
. there is a severe cardiac event, identifying high risk
patients and trying to manage their modifiable risk
factors is the best health promotion
1738 What is the cause of most SCD? V-tach
. *occurs less commonly with aortic stenosis
1739 What is the collaborative care of chronic stable Decrease O2 demand and increase O2 supply
. angina? -Nitrates
-beta blockers
-calcium channel blockers
-reduce risk factors
-antiplatelet (ASA)
-cholesterol-lowering drugs
1740 What is the correct location for placement of the Just above the xiphoid process on the lower third of the
. hands for manual chest compressions during sternum
cardiopulmonary resuscitation (CPR) on the
adult client?
1741 What is the correct procedure for performing an From a distance of 8 to 12 inches and slightly to the
. ophthalmoscopic examination on a client's right side, shine the light into the client's pupil.
eye?
1742 What is the definition of bronchopneumonia? Pneumonia that is more distributed in a patchy fashion,
. having originated in one or more localized areas within
the bronchi and extending to the adjacent surrounding
lung parenchyma.
1743 What is the definition of consolidation? Tissue that solidifies as a result of collapsed alveoli or
. pneumonia.
1744 What is the definition of miliary TB? The spread of TB to other parts of the body.
.

1745 What is the definition of pneumonitis? A more general term that describes an inflammatory
. process in the lung tissue that may predispose a patient
to or place a patient at risk for microbrial invasion.
1746 What is the difference between an NSTEMI- partially occluded artery
. NSTEMI and a STEMI? STEMI- fully occluded artery
1747 What is the difference between left Left-sided failure usually manifests as pulmonary congestion and
. and right-sided HF? edema due to the blood backing up on the left side of the heart into
the lungs.

Right-sided failure causes the back up of blood into the right atrium
and therefore venous circulation resulting in peripheral edema.
1748 What is the easiest way a patient can Lifestyle modification
. improve their HTN?
1749 What is the etiology of chronic stable Primary reason for ischemia is insufficient blood flow is narrowing
. angina? of coronary arteries (atherosclerosis)
1750 What is the FITT program and what It's a way to help work out:
. does it mean? Frequency (4-5 days a week)
Intensity (moderate, brisk walking, hiking, biking, swimming)
Type (weight training/cardio)
Time (30 min)
1751 What is the formula for blood Cardiac output multiplied by systemic vascular resistance
. pressure?
1752 What is the goal for someone with decrease afterload
. systolic HF?
1753 What is the leading cause of death Pneumonia
. from infection in Canada?
1754 What is the line of treatment for a -Emergent PCI
. patient with confirmed MI? In most severe cases: CABG
1755 What is the major adverse effect of bleeding
. fibrinolytic therapy?
1756 What is the major cause of coronary atherosclerosis
. artery disease? (CAD)?
1757 What is the major disadvantage of Blurred vision
. ophthalmologic ointments? Explanation:
The major disadvantage of ointments is the blurred vision that
results after application. Therefore options A, B, and C are
incorrect.
1758 What is the major disadvantage of Blurred vision
. ophthalmologic ointments? Correct
Explanation:
a) Hard to administer The major disadvantage of ointments is the blurred vision that
b) Patients don't like them results after application. Therefore options A, B, and C are
c) They ooze out of the eye incorrect.
d) Blurred vision
1759 What is the most common cause of Rheumatic endocarditis
. mitral stenosis
1760 What is the most common cause of Viruses
. pneumonia in children?
1761 What is the most common cause of acute left ventricular failure secondary to CAD.
. pulmonary edema?
1762 What is the most common Dysrhythmia
. complication of an MI?
1763 What is the most common initial MI more than angina
. cardiac event for a man with CAD?
1764 What is the most common initial cardiac angina more than MI
. event for a woman with CAD?
1765 What is the most important nursing straining all urine
. priority for a client who has been admitted
for a possible kidney stone?
1766 What is the most severe type of HF? biventricular HF
.

1767 What is the name of the major tuberculin Mantoux Test


. skin test?
1768 What is the normal pH range for arterial b) 7.35 to 7.45
. blood?

a) 7 to 7.49
b) 7.35 to 7.45
c) 7.50 to 7.60
d) 7.55 to 7.65
1769 What is the order of action for acute -O2
. intervention of an MI? -VS/Pulsox
-ECG
-Pain relief (NTG then MSO4)
-Auscultate heart sounds listening for S3 or S4
1770 What is the pain like with chronic stable -usually lasts 3-5 minutes and abates when the precipitating
. angina? factor is relieved
1771 What is the primary actions of drugs to -reduce systemic vascular resistance
. treat hypertension? -Reduce volume of circulating blood
1772 What is the primary infectious agent of Mycobacterium Tuberculosis
. TB?
1773 What is the process of TB infection? A susceptible person inhales mycobacterium bacilli and
. becomes infected. The bacteria are transmitted through the
airways to the alveoli, where they are deposited and begin to
multiply.
1774 What is Trousseau's sign? inflate BP cuff 10-20 mm Hg above systolic pressure, capral
. spasms w/in 2-5 min indicate tetany
1775 What lab level is checked to determine the brain natriuretic peptide (BNP) would be high because they
. level of HF? are endothelin and aldosterone agonists, and the heart wants
to block their actions as a counterregulatory mechanism.
1776 What labs are usually ordered when Troponin
. someone comes in with suspected ACS? CK-MB
1777 What microorganism is another cause of H. Influenzae
. CAP, and frequently affects elderly people
or those with comorbid illnesses?
1778 What might you discover when assessing - Changed in temperature and pulse.
. your patient diagnosed with pneumonia? - Secretions
- Cough
- Tachypnea and SOB
- Changes in physical assessment, especially inspection and
auscultation of the chest.
- Changes in mental status, fatique, dehydration, and
concomitant heart failure (especially in elderly patients).
1779 What nursing action is necessary for the .
. client with a flail chest? Encourage coughing and deep breathing.
1780 What nutritional recommendations can be -Decrease saturated fat and cholesterol
. made to high risk CAD patients? -Increase complex carbohydrates (fruit, whole grain, veg)
-increase omega-3 fatty acid intake
1781 What occurs if the etiologic agent is not Empiric Antibiotic Therapy
. identified?
1782 What role does endothelin play in ADH, catecholamines, and angiotensin II stimulate the production of
. increasing the workload of a heart endothelin from the vascular endothelial cells. It's a vasoconstrictor
in HF? and also increases the heart's contractility and it hypertrophies the
heart.
1783 What should someone with if they are having pain at rest
. chronic unstable angina be
worried?
1784 What three things are covered -Unstable Angina (UA)
. under ACS? -non-ST elevated MI (NSTEMI)
-ST elevated MI (STEMI)
1785 what to do with scabies Wash underwear, towels, and sleepwear in hot water. Vacuum the
. carpets and upholstered furniture.
1786 What two things does the degree -area of the heart involved
. of altered function depend on? -size of the infarct.
1787 What two things happen as a Dilation
. result of the failing heart's Hypertrophy - result of ventricular remodeling
compensatory mechanisms
1788 What type of angina only occurs nocturnal angina
. at night, but not necessarily while *angina decubitus is anging experienced while lying down
reclined or asleep?
1789 What type of assessment is used PQRST
. when assessing angina? P- precipitating events
Q- quality of pain
R- radiation of pain
S- severity of pain
T- timing
1790 What type of HTN is common in ISH - isolated systolic hypertension
. older adults?
1791 What types of things decrease -anemia
. your oxygen supply? -asthma
-COPD
-hypovolemia
-hypoxemia
-pneumonia
-coronary artery spasm or thrombosis
-dysrhythmia
-HF
-valve disorders
1792 What types of things increase -Anxiety
. your oxygen demand? -HTN
-Hyperthyroidism
-Substance abuse
-exercise
-Aortic stenosis
-Cardiomyopathy
-Dysrhythmia
-Tachycardia
1793 When a building's hot water bacteria thrive; then they may be transmitted via inhalation from air
. plumbing has water at this conditioning, showers, spas, and whirlpools.
temperature, the
1794 When a chest tube becomes disconnected, immediate steps to prevent air from entering the chest cavity,
. the nurse should take which may cause the lung to collapse. when a chest tube is
accidentally disconnected from the drainage tube, the nurse
should either double-clamp the chest tube as close to the
client as possible or place the open end of the tube in a
container of sterile water or saline solution. Then the
physician should be notified.
1795 When a client has one-sided weakness, the unaffected side. strong side
. nurse should place the wheelchair on the
client's
1796 When a client receiving a tube feeding While the nurse is administering a bolus feeding to a client
. begins to vomit, the nurse should first stop via nasogastric tube, the client begins to vomit. What action
the feeding (A) to prevent further vomiting. should the nurse implement first?
(C) should then be implemented to reduce A. Discontinue the administration of the bolus feeding.
the risk of aspiration. After that, (B and D) B. Auscultate the client's breath sounds bilaterally.
can be implemented as indicated. C. Elevate the head of the bed to a high Fowler's position.
Correct Answer: A D. Administer a PRN dose of a prescribed antiemetic.
1797 When a client's ventilation is impaired, the b) Carbon dioxide
. body retains which substance?

a) Sodium bicarbonate
b) Carbon dioxide
c) Nitrous oxide
d) Oxygen
1798 When admitting a 45-year-old female with A, C, D, E
. a diagnosis of pulmonary embolism, the
nurse will assess the patient for which of
the following risk factors (select all that
apply)?
A) Obesity
B) Pneumonia
C) Hypertension
D) Cigarette smoking
E) Recent long distance travel
1799 When a patient with asthma is admitted to Aerosolized albuterol
. the emergency department in severe
respiratory distress, the nurse anticipates
that initial drug treatment will most likely
include administration of
1800 When are Calcium channel blockers When Beta blockers are contraindicated or for Prinzmetal's
. indicated for use in angina? angina
1801 When are the two points that an older adult -When they are hospitalized from their CAD
. might consider a lifestyle change? -When their symptoms are from CAD and not normal aging
1802 When arsenic overexposure occurs, the violent nausea, vomiting, abdominal pain, skin irritation,
. signs and symptoms include severe diarrhea, laryngitis, and bronchitis.
o Dehydration can lead to shock and death.
o After the acute phase, bone marrow depression,
encephalopathy, and sensory neuropathy occur.
1803 When arsenic overexposure occurs, the violent nausea, vomiting, abdominal pain, skin irritation,
. symptoms include severe diarrhea, laryngitis, and bronchitis.
1804 When assigning clients on a medical- An older adult client with pneumonia and viral meningitis
. surgical floor to a registered nurse (RN)
and a practical nurse (PN), it is best for the
charge nurse to assign which client to the
PN?
1805 when breast feeding As much of the the infant's mouth in order to establish a latch that does not
. mother's nipple and areola need to be in cause nipple cracks or fissures which decreasing pain,
cracking and fissures.
1806 When calcium levels are too low, the body increasing production of parathyroid hormone
. responds by
1807 When caring for a client with acute respiratory d) Hypercapnia, hypoventilation, and hypoxemia
. failure, the nurse should expect to focus on resolving
which set of problems?

a) Hypotension, hyperoxemia, and hypercapnia


b) Hyperventilation, hypertension, and hypocapnia
c) Hyperoxemia, hypocapnia, and hyperventilation
d) Hypercapnia, hypoventilation, and hypoxemia
1808 When caring for a client with a diagnosis of aortic a)Level of consciousness, characteristics of pain,
. aneurysm scheduled for surgery, what would be most and signs of hemorrhage or dissection
important for the nurse to monitor? Explanation:
a) Level of consciousness, characteristics of pain, and The nurse monitors BP, pulse, hourly urine
signs of hemorrhage or dissection output, skin color, level of consciousness, and
b) BP, pulse, respirations, and signs of hemorrhage or characteristics of pain for signs of hemorrhage or
dissection dissection. Assessing respirations, nutritional
c) Cultural needs, characteristics of pain, and signs of levels, and cultural needs are important but not
hemorrhage or dissection the most important assessments for the nurse to
d) Intake and output, nutrition level, respirations, and make.
characteristics of pain
1809 When caring for a client with a diagnosis of aortic a) Level of consciousness, characteristics of pain,
. aneurysm scheduled for surgery, what would be most and signs of hemorrhage or dissection
important for the nurse to monitor? Explanation:
a) Level of consciousness, characteristics of pain, and The nurse monitors BP, pulse, hourly urine
signs of hemorrhage or dissection output, skin color, level of consciousness, and
b) Intake and output, nutrition level, respirations, and characteristics of pain for signs of hemorrhage or
characteristics of pain dissection. Assessing respirations, nutritional
c) Cultural needs, characteristics of pain, and signs of levels, and cultural needs are important but not
hemorrhage or dissection the most important assessments for the nurse to
d) BP, pulse, respirations, and signs of hemorrhage or make.
dissection
1810 When caring for a postsurgical patient, the nurse Reinforcing dressing or applying pressure if
. observes that the client has hemorrhaged and is in bleeding is frank
hypovolemic shock. Which of the following nursing
interventions will manage and minimize hemorrhage
and shock?
1811. When caring for the patient with a chest tube, the Stripping or milking the chest tube to promote
nurse should intervene when the nursing assistant is drainage
1812 When death is impending, it is essential for the nurse A 75-year-old client who has a history of end
. to determine who is legally empowered to make stage renal failure and advanced lung cancer,
decisions regarding the use of life-saving measures for recently had a stroke. Two days ago the
the client (D). (A) will be abnormal and will worsen healthcare provider discontinued the client's
without dialysis, so are not of immediate concern. (B) dialysis treatments, stating that death is
may help improve the client's quality of life prior to inevitable, but the client is disoriented and will
death, but is of less immediacy than determining not sign a DNR directive. What is the priority
whether actions should be taken to save a client's life. nursing intervention?
If the nurse remains unable to determine who is A. Review the client's most recent laboratory
empowered to make decisions in this situation, the reports.
nurse may choose to contact the ethics committee (C) B. Refer the client and family members for
for a resolution. hospice care.
Correct Answer: D C. Notify the hospital ethics committee of the
client situation.
D. Determine who is legally empowered to make
decisions.
1813 When describing abdominal hernias to a group of INguinal
. nursing students, the instructor would identify which
type as most common?
1814 When developing a care plan for a client who has The stroke may have impacted the body's
. recently suffered a stroke, a nurse includes the thermoregulation centers.
nursing diagnosis Risk for imbalanced body
temperature. What is the rationale for this diagnosis?
1815 When is a heart transplant indicated? when NOTHING else has worked
.
*Treatment of choice for patients with refractory
end-stage HF, inoperable CAD, and cardiomyopathy
1816 When is BP highest? Early morning
.

1817 When obtaining a health history from a patient at An abrupt onset of fever and chills
. the clinic with suspected CAP, the nurse expects
the patient to report:
1818 When obtaining a health history from a patient Fatigue, low-grade fever, and night sweats
. suspected of having early TB, the nurse asks the
patient about experiencing
1819 When obtaining a health history from a patient Fatigue, low grade fever and night sweats
. suspected of having early TB, the nurse asks the
patient about experiencing
1820 When obtaining the health history from a client light flashes and floaters in front of the eye.
. with retinal detachment, a nurse expects the client Correct
to report: Explanation:
The sudden appearance of light flashes and floaters
a) a recent driving accident while changing lanes. in front of the affected eye is characteristic of retinal
b) headaches, nausea, and redness of the eyes. detachment. Difficulty seeing cars in another driving
c) light flashes and floaters in front of the eye. lane suggests gradual loss of peripheral vision,
d) frequent episodes of double vision. which may indicate glaucoma. Headache, nausea,
and redness of the eyes are signs of acute (angle-
closure) glaucoma. Double vision is common in
clients with cataracts.
1821 When performing a physical examination on a Measure abdominal girth according to a set routine
. client with cirrhosis, a nurse notices that the
client's abdomen is enlarged. Which of the
following interventions should the nurse consider?
1822 When planning appropriate nursing interventions B
. for a patient with metastatic lung cancer and a 60-
pack-per-year history of cigarette smoking, the
nurse recognizes that the smoking has most likely
decreased the patient's underlying respiratory
defenses because of impairment of which of the
following?
A) Cough reflex
B) Mucociliary clearance
C) Reflex bronchoconstriction
D) Ability to filter particles from the air
1823 When postural hypotension occurs, the body A male client with venous incompetence stands up
. attempts to restore arterial pressure by stimulating and his blood pressure subsequently drops. Which
the baro-receptors to increase the heart rate (B), finding should the nurse identify as a compensatory
not decrease it (A). Peripheral vasoconstriction, response?
not dilation (C), of the veins and arterioles occurs A. Bradycardia.
with venous incompetence through the baro- B. Increase in pulse rate.
receptor reflex. A decrease in cardiac output, not C. Peripheral vasodilation.
an increase (D), occurs when orthostatic D. Increase in cardiac output.
hypotension occurs.
Correct Answer: B
1824 When preparing a male client who has had a total Tell the client to carry a medic alert card that
. laryngectomy for discharge, what instruction explains his condition.
would be most important for the nurse to include
in the discharge teaching?
1825 When preparing a patient for a cardiac Methylprednisolone (Solu-Medrol)---->Prior to
. catheterization, the patient states that she has cardiac catheterization, the patient is assessed for
allergies to seafood. Which of the following previous reactions to contrast agents or allergies to
medications may give to her prior to the iodine-containing substances, as some contrast
procedure? agents contain iodine. If allergic reactions are of
concern, antihistamines or methylprednisolone
(Solu-Medrol) may be administered to the patient
before angiography is performed. Lasix, Ativan, and
Dilantin do not counteract allergic reactions.
1826 When preparing for continuous bladder balloon inflation and continuous inflow and outflow of
. irrigation, a triple-lumen indwelling irrigation solution.
urinary catheter is inserted. The three
lumens provide for
1827 When taken in combination with aspirin, hypoglycemia
. glipizide commonly causes
1828 When taking isoniazid, the client should tyramine-rich foods in his diet because these foods and the
. limit drug could interact to cause hypertensio
1829 When teaching a client with peripheral c) Avoid wearing canvas shoes.
. vascular disease about foot care, a nurse Explanation:
should include which instruction? The client should be instructed to avoid wearing canvas
a) Avoid using cornstarch on the feet. shoes. Canvas shoes cause the feet to perspire, and
b) Avoid using a nail clipper to cut toenails. perspiration can cause skin irritation and breakdown. Cotton
c) Avoid wearing canvas shoes. and cornstarch absorb perspiration. The client should be
d) Avoid wearing cotton socks. instructed to cut toenails straight across with nail clippers.
1830 When the client advances both crutches swing through" gait.
. together and follows by lifting both lower
extremities PAST the level of the crutches,
the gait is called a
1831 When the client advances both crutches "swing to" gait.
. together and follows by lifting both lower
extremities to the SAME level as the
crutches, the gait is called a
1832 When the feeding is completed, clear the 3 cc of water. Rinse the syringe and extension tubing with
. tube with water.
1833 When the patient diagnosed with angina d) Unstable
. pectoris complains that he is experiencing Explanation:
chest pain more frequently even at rest, the Unstable angina is also called crescendo or preinfarction
period of pain is longer, and it takes less angina and indicates the need for a change in treatment.
stress for the pain to occur, the nurse Intractable or refractory angina produces severe,
recognizes that the patient is describing incapacitating chest pain that does not respond to
which type of angina? conventional treatment. Variant angina is described as pain
a) Variant at rest with reversible ST-segment elevation and is thought
b) Refractory to be caused by coronary artery vasospasm. Intractable or
c) Intractable refractory angina produces severe, incapacitating chest pain
d) Unstable that does not respond to conventional treatment.
1834 When treating hypertensive emergencies, Continuous IV infusion-->The medications of choice in
. the nurse identifies the most appropriate hypertensive emergencies are best managed through the
route of administration for antihypertensive continuous IV infusion of a short-acting titratable
agents as being which of the following? antihypertensive agent. The nurse avoids the sublingual and
IM routes as their absorption and dynamics are
unpredictable. The oral route would not have as quick an
onset as a continuous IV infusion
1835 When was SARS first reported? In Asia, February 2003.
.

1836 When would a CABG be indicated? -After Failed medical management


. -Presence of left main coronary artery or three-vessel
disease
-Not a candidate for PCI (e.g., lesions are long or difficult to
access)
-Failed PCI with ongoing chest pain
-History of diabetes mellitus
1837 When would fibrinolytics be used? In a hospital setting without a cath lab or where one is too
. far away.
1838 Where does CAP occur? In the community setting or within the first 48 hours of
. hospitilation or institutionalization.
1839 Where is the most common place of MI? Occlusion in the left anterior descending coronary artery
.

1840 Which abnormal lab finding indicates Microalbuminuria


. that a client with diabetes needs further
evaluation for diabetic nephropathy?
1841 Which area of the spinal column is Lower lumbar
. subject to the greatest mechanical stress
and degenerative changes?
1842 Which are the most common vessels Right internal mammary artery
. used for CABG? Radial artery
Saphenous vein
1843 Which assessment finding in a client Fever of 102 F
. with an acute small bowel obstruction
requires the most immediate
intervention by the nurse?
1844 Which assessment finding indicates that Increased ability to solve simple problems
. the expected outcome of administering
donepezil (Aricept) to a client with
Alzheimer disease has been
accomplished?
1845 Which assessment findings would the Leukocytosis and localized bone pain
. nurse expect in a client diagnosed with
osteomyelitis?
1846 Which assessment finding would Examination of portable radiograph taken after the tube was
. confirm the specific location of an inserted
enteral feeding tube?
1847 Which change in lab values indicates to Decrease in hemoglobin
. the nurse that a client with rheumatoid
arthritis may be experiencing an
adverse effect of methotrexate (Mexate)
therapy?
1848 Which client statement indicates a good Limiting my salt intake to 2 grams per day will improve my
. understanding of the nutritional blood pressure."
modifications needed to manage
hypertension?
1849 Which condition most commonly results Atherosclerosis--->Atherosclerosis (plaque formation), is the
. in coronary artery disease? leading cause of CAD. Diabetes mellitus is a risk factor for
CAD, but it isn't the most common cause. Myocardial infarction
is a common result of CAD. Renal failure doesn't cause CAD,
but the two conditions are related.
1850 Which condition should the nurse Infection
. anticipate as a potential problem in a
female client with a neurogenic bladder?
1851 Which consideration is most important Provide a room that can be kept warm.
. when the nurse is assigning a room for a
client being admitted with progressive
systemic sclerosis (scleroderma)?
1852 Which content about self-care should A.
. the nurse include in the teaching plan of Encourage annual physical and Pap smear.
a client who has genital herpes? (Select
all that apply.)
B.
Take antiviral medication as prescribed.

C.
Use condoms to avoid transmission to others.

D.
Warm sitz baths may relieve itching.
1853 Which description of Sudden, stabbing, severe pain over the lip and chin
. symptoms is characteristic of
a client diagnosed with
trigeminal neuralgia (tic
douloureux)?
1854 Which diagnostic is the Echocardiogram--->An echocardiogram is recommended method of
. recommended method of determining whether hypertrophy has occurred. ECG and blood chemistry
determining whether left are part of the routine work up. Renal damage may be suggested by
ventricular hypertrophy has elevations in BUN and creatinine levels.
occurred?
1855 Which diagnostic test does the dual-energy x ray absorption
. nurse expect the client with
psteoporosis to undergo
1856 Which dietary modification is Elimination of coffee
. utilized for a patient
diagnosed with acute
pancreatitis?
1857 Which findings should a nurse Weight gain, decreased appetite, and constipation--->Hashimoto's
. expect to assess in client with thyroiditis, an autoimmune disorder, is the most common cause of
Hashimoto's thyroiditis? hypothyroidism. It's seen most frequently in women older than age 40.
Signs and symptoms include weight gain, decreased appetite; constipation;
lethargy; dry cool skin; brittle nails; coarse hair; muscle cramps; weakness;
and sleep apnea. Weight loss, increased appetite, and hyperdefecation are
characteristic of hyperthyroidism. Weight loss, increased urination, and
increased thirst are characteristic of uncontrolled diabetes mellitus. Weight
gain, increased urination, and purplish-red striae are characteristic of
hypercortisolism.
1858 Which hormone would be Glucagon--->Glucagon is a hormone released by the alpha islet cells of the
. responsible for increasing pancreas that raises blood glucose levels by stimulating glycogenolysis
blood glucose levels by (the breakdown of glycogen into glucose in the liver). Somatostatin is a
stimulating glycogenolysis? hormone secreted by the delta islet cells that helps to maintain a relatively
constant level of blood glucose by inhibiting the release of insulin and
glucagons. Insulin is a hormone released by the beta islet cells that lowers
the level of blood glucose when it rises beyond normal limits.
Cholecystokinin is released from the cells of the small intestine that
stimulates contraction of the gall bladder to release bile when dietary fat is
ingested.
1859 Which instruction should the "Change your tampon frequently."
. nurse teach a female client
about the prevention of toxic
shock syndrome?
1860 Which medication is the drug Atropine
. of choice for sinus
bradycardia?
1861 Which of the following alert the Restlessness and shortness of breath.
. nurse to possible internal bleeding
in a client who has undergone
pulmonary lobectomy 2 days ago?
1862 Which of the following are Trauma from accidents
. assoicated with compartment Surgery
syndrome Casts
Tight bandages
crushing injuries
1863 Which of the following are c) Diminished or absent pulses
. characteristics of arterial Explanation:
insufficiency? A diminished or absent pulse is a characteristic of arterial
a) Aching, cramping pain insufficiency. Venous characteristics include superficial ulcer
b) Superficial ulcer formation, an aching and cramping pain, and presence of pulses.
c) Diminished or absent pulses
d) Pulses are present, may be
difficult to palpate
1864 Which of the following are Kidney, prostate, lung, breast, ovary
. common primary sites of tumors
that metastasize to the bone?
Select all that apply.
1865 Which of the following clients is at Client with blood type O
. highest risk for peptic ulcer
disease?
1866 Which of the following clinical Increased Pulse Rate
. manifestations of hemorrhage is Explanation:
related to carotid artery rupture? The nurse monitors vital signs for changes, particularly increased
a) Shallow respirations pulse rate, decreased blood pressure, and rapid, deep respirations.
b) Dry skin Cold, clammy, pale skin may indicate active blee
c) Increased pulse rate
d) Increased blood pressure
1867 Which of the following clinical C
. manifestations would the nurse
expect to find during assessment
of a patient admitted with
pneumococcal pneumonia?
A) Hyperresonance on percussion
B) Vesicular breath sounds in all
lobes
C) Increased vocal fremitus on
palpation
D) Fine crackles in all lobes on
auscultation
1868 Which of the following diagnostic A computer tomography scan CT-->A computed tomography or
. tests are done to determine magnetic resonance imaging scan is done to detect a suspected
suspected pituitary tumor? pituitary tumor. Radiographs of the chest or abdomen are taken to
detect tumors. Radiographs also determine the size of the organ and
their location. However, measuring blood hormone levels helps
determine the functioning of endocrine glands. A radioimmunoassay
determines the concentration of a substance in plasma.
1869 Which of the following diagnostic b) Air plethysmography
. tests are used to quantify venous Explanation:
reflux and calf muscle pump Air plethysmography is used to quantify venous reflux and calf
ejection? muscle pump action. Contrast phlebography involves injecting a
a) Lymphangiography radiopaque contrast agent into the venous system.
b) Air plethysmography Lymphoscintigraphy is done when a radioactively labeled colloid is
c) Lymphoscintigraphy injected subcutaneously in the second interdigital space. The
d) Contrast phlebography extremity is then exercised to facilitate the uptake of the colloid by
the lymphatic system, and serial images are obtained at present
intervals. Lymphoangiography provides a way of detecting lymph
node involvement resulting from metastatic carcinoma, lymphoma,
or infection in sites that are otherwise inaccessible to the examiner
except by surgery.
1870 Which of the following digestive Amylase
. enzymes aids in the digesting of
starch?
1871 Which of the following exposures Exposure to tobacco smoke
. accounts for the majoirty of cases
withr egard to risk factors for
COPD?
1872 Which of the following features External eye appearance
. should a nurse observe during an Explanation:
ophthalmic assessment? During an ophthalmic assessment, the nurse should examine the
external eye appearance and the pupil responses of the patient. A
qualified examiner determines the internal eye function, the visual
acuity, and the intraocular pressure.

Reference: Timby, B.K., & Smith, N.E. (2010). Introductory


Medical-Surgical Nursing, 10th ed. Philadelphia: Wolters Kluwer
Health/Lippincott Williams & Wilkins, Chapter 42: Caring for
Clients with Eye Disorders, p. 614.
1873 Which of the following health Incorporate physical exercise as tolerated into the daily routine.
. promotion activities should the
nurse include in the discharge
teaching plan for a client with
asthma?
1874 Which of the following hormones Adrenocorticotropic hormone ACTH--->ACTH controls the
. controls secretion of adrenal secretion of adrenal androgens. When secreted in normal amounts,
androgens? the adrenal androgens appear to have little effect, but when secreted
in excess, as in certain inborn enzyme deficiencies, masculinization
may result. The secretion of T3 and T4 by the thyroid gland is
controlled by TSH. Parathormone regulates calcium and
phosphorous metabolism. Calcitonin reduces the plasma level of
calcium by increasing its deposition in bone.
1875 Which of the following instructions To take prophylatic antibiotics
. should a nurse provide a patient
with a history of rheumatic fever
before the patient has any dental
work done
1876 Which of the following interventions sodium restriction may be necessary to promote fluid loss.
. will be most effective in reducing a Restricting fluid intake will not reduce retained fluids
client's fluid volume excess?
1877 Which of the following is accurate a) Reduces circulation to the extremities
. regarding the effects of nicotine and b) Impairs transport and cellular use of oxygen
tobacco smoke on the body? Select c) Causes vasospasm
all that apply. d) Increases blood viscosity
a) Reduces circulation to the Explanation:
extremities Nicotine from tobacco products causes vasospasm and can
b) Impairs transport and cellular dramatically reduce circulation to the extremities. Tobacco smoke
use of oxygen also impairs transport and cellular use of oxygen and increases blood
c) Causes vasospasm viscosity.
d) Increases blood viscosity
e) Decreases blood viscosity
1878 Which of the following is a b) Border regular and well demarcated
. characteristic of an arterial ulcer? Explanation:
a) Brawny edema Characteristics of an arterial ulcer include a border that is regular
b) Border regular and well and demarcated. Brawny edema, ABI > 0.90, and edema that may be
demarcated severe are characteristics of a venous ulcer.
c) Ankle-brachial index (ABI) >
0.90
d) Edema may be severe
1879 Which of the following is a classic Pallor
. sign of hypovolemic shock?
1880 Which of the following is also Unstable angina
. termed preinfarction angina?
1881 Which of the following is a History of smoking
. modifiable risk factor for transient
ischemic attacks and ischemic
strokes?
1882 Which of the following is an early Fatigue---> is an early warning symptom
. warning symptom of acute coronary
syndrome (ACS) and heart failure
(HF)?
1883 Which of the following is an Lengthening
. inaccurate clinical manifestation of
a fracture?
1884 Which of the following is a true Results in a permanent loss of voice; When a patient undergoes a
. statement regarding a total total laryngectomy, there is a permanent loss of the voice. A
laryngectomy? larygnectomy results in permanent tracheostomy and the removal of
the laryngeal structures, and is performed for most advanced State
IV laryngeal cancer
1885 Which of the following is a true The clusters of ulcers take on a cobble stone appearance
. statement regarding regional
enteritis (Crohn's disease)?
1886 Which of the following is important Establish rapport with the patient and family members
. for the nurse to consider during the
preinterview period with an adult?
1887 Which of the following is the Eat soft, easily chewed foods.
. correct advice regarding Correct
food for a patient who Explanation:
underwent a cataract The nurse should advise patients recovering from cataract surgery to eat
surgery? soft, easily chewed foods until healing is complete to avoid tearing from
excessive facial movements. Eating spinach or collard greens two to four
a) Eat spinach or collard times per week reduces the risk of macular degeneration and increasing the
greens two to four times per intake of vitamins A and C is essential for preventing cataracts; however,
week. these have no implications on recovery from cataract surgery.
b) Eat red meat two to four
times per week.
c) Increase intake of vitamins
A and C.
d) Eat soft, easily chewed
foods.
1888 Which of the following is the d) Intermittent claudication
. hallmark symptom for Explanation:
peripheral arterial disease The hallmark symptom of PAD in the lower extremity is intermittent
(PAD) in the lower claudication. This pain may be described as aching or cramping in a muscle
extremity? that occurs with the same degree of exercise or activity and is relieved with
a) Vertigo rest. Acute limb ischemia is a sudden decrease in limb perfusion, which
b) Dizziness produces new or worsening symptoms that may threaten limb viability.
c) Acute limb ischemia Dizziness and vertigo are associated with upper extremity arterial occlusive
d) Intermittent claudication disease.
1889 Which of the following is the Glaucoma
. leading cause of blindness in Explanation:
the United States? Glaucoma is one of the leading causes of irreversible blindness in the world
and is the leading cause of blindness among adults in the United States.
1890 Which of the following is the Glaucoma
. leading cause of blindness in Correct
the United States? Explanation:
Glaucoma is one of the leading causes of irreversible blindness in the world
a) Macular degeneration and is the leading cause of blindness among adults in the United States.
b) Glaucoma
c) Cataracts
d) Retinal detachment
1891 Which of the following is the Osteogenic sarcoma
. most common and most fatal
primary malignant bone
tumor?
1892 Which of the following is the Hemorrhage
. most common complication
associated with peptic ulcer?
1893 Which of the following is the c) Thoracic area
. most common site for a Explanation:
dissecting aneurysm? The thoracic area is the most common site for a dissecting aneurysm. About
a) Cervical area one-third of patients with thoracic aneurysms die of rupture of the
b) Sacral area aneurysm.
c) Thoracic area
d) Lumbar area
1894 Which of the following is the most Inadequate tissue perfusion
. important postoperative assessment
parameter for patients undergoing
cardiac surgery?
1895 Which of the following is the most Removal of tumor
. successful treatment for gastric
cancer?
1896 Which of the following is the priority D
. nursing intervention in helping a
patient expectorate thick lung Although several interventions may help the patient expectorate
secretions? mucus, the highest priority should be on increasing fluid intake,
A) Humidify the oxygen as able which will liquefy the secretions so that the patient can
B) Administer cough suppressant q4hr expectorate them more easily. Humidifying the oxygen is also
C) Teach patient to splint the affected helpful, but is not the primary intervention. Teaching the patient
area to splint the affected area may also be helpful, but does not
D) Increase fluid intake to 3 L/day if liquefy the secretions so that they can be removed.
tolerated
1897 Which of the following medications is d) Alteplase
. considered a thrombolytic? Explanation:
a) Heparin Alteplase is considered a thrombolytic, which lyses and dissolves
b) Lovenox thrombi. Thrombolytic therapy is most effective when given
c) Coumadin within the first 3 days after acute thrombosis. Heparin, Coumadin,
d) Alteplase and Lovenox do not lyse clots.
1898 Which of the following nursing action Elevating the head of the bed-->The nurse should monitor the
. would the nurse include when caring respiratory status and elevate the head of the bed to relieve
for a client with endemic goiter and respiratory symptoms. A high-iodine diet does not relieve
experiencing respiratory symptoms? respiratory distress. Although proper air circulation in the room
and avoiding physical exertion may be important, these actions do
not address the respiratory symptoms.
1899 Which of the following nursing C
. interventions is most appropriate to
enhance oxygenation in a patient with
unilateral malignant lung disease?
A) Positioning patient on right side
B) Maintaining adequate fluid intake
C) Positioning patient with "good lung
down"
D) Performing postural drainage
every 4 hours
1900 Which of the following nursing Report any incident of bloody urine, stools, or both---->The client
. interventions is most appropriate who takes an anticoagulant, such as a low-molecular-weight
when caring for a client with a nursing heparin, is routinely screened for bloody urine, stools, or both.
diagnosis of risk for injury related to
side effects of medication (enoxaparin
[Lovenox])?
1901 Which of the following physical A
. assessment findings in a patient with a
lower respiratory problem best
supports the nursing diagnosis of
ineffective airway clearance?
A) Basilar crackles
B) Respiratory rate of 28
C) Oxygen saturation of 85%
D) Presence of greenish sputum
1902 Which of the following statements The lung is biopsied using a transtracheal method
. describe the management of a patient The use of a home spirometer will help to monitor lung
following lung transplantation (select all function
that apply)? Immunosuppressant therapy usually involves a three-drug
regimen
1903 Which of the following surgical LASIK
. procedures involves flattening the Explanation:
anterior curvature of the cornea by LASIK involves flattening the anterior curvature of the cornea
removing a stromal lamella layer? by removing a stromal lamella or layer. PRK is used to treat
myopia and hyperopia with or without astigmatism.
Keratoconus is cone-shaped deformity of the cornea.
Keratoplasty involves replacing abnormal host tissue with
healthy donor (cadaver) corneal tissue.

Reference: Timby, B.K., & Smith, N.E. (2010). Introductory


Medical-Surgical Nursing, 10th ed. Philadelphia: Wolters
Kluwer Health/Lippincott Williams & Wilkins, Chapter 42:
Caring for Clients with Eye Disorders, p. 607.
1904 Which of the following term is used to Pleural effusion; a pneumothrax is air in the pleural space. A
. describe fluid acccumulating within the hemothorax is blood within the pleural space. Consolidation is
pleural space? lung tissue that has become more solid in nature due to
collapse alveoli or infectious process
1905 Which of the following terms describes Secondary hypertension
. high blood pressure from an identified
cause, such as renal disease?
1906 Which of the following terms is given to Rebound-->Rebound hypertension may precipitate a
. hypertension in which blood pressure hypertensive crisis. Essential or primary hypertension denotes
that is controlled with therapy becomes high blood pressure from an unidentified source. Secondary
uncontrolled (abnormally high) with the hypertension denotes high blood pressure from an identified
discontinuation of therapy? cause, such as renal disease.
1907 Which of the following therapies are for Heart transplant
. patient who have advanced heart failure
(HF) after all other therapies have failed
1908 Which of the following would be an Pain
. inaccurate clinical manifestation of a Explanation:
retinal detachment? Patient may report the sensation of a shade or curtain coming
across the vision of one eye, cobwebs, bright flashing lights, or
a) Pain the sudden onset of a great number of floaters. Patients do no
b) Bright flashing lights complain of pain.
c) Cobwebs
d) Sudden onset of a greater number of
floaters
1909 Which of the following would be included Handing instruments to the surgeon and assistants
. as a responsibility of the scrub nurse?
1910 Which of the following would be most Monitoring stool passage and its color
. important to ensure that a client does not
retain any barium after a barium
swallow?
1911. Which of the following would not be Opacity of the lens
included as a marker of medication Explanation:
effectiveness in glaucoma control? Opacity of the lenses relates to cataract formation. The main
markers of the efficacy of the medication in glaucoma control
a) Visual field are lowering of the IOP to the target pressure, appearance of
b) Appearance of optic nerve head the optic nerve head, and the visual field.
c) Lowering intraocular pressure (IOP)
to target pressure
d) Opacity of the lens
1912 Which of the following would the nurse Exopthalmos--->Exophthalmos that results from enlarged
. expect to find in a client with severe muscle and fatty tissue surrounding the rear and sides of the
hyperthyroidism? eyeball is seen in clients with severe hyperthyroidism. Tetany
is the symptom of acute and sudden hypoparathyroidism.
Buffalo hump and striae are the symptoms of Cushing's
syndrome.
1913 Which sign or symptom suggests c) Increased abdominal and back pain
. that a client's abdominal aortic Explanation:
aneurysm is extending? Pain in the abdomen and back signify that the aneurysm is pressing
a) Elevated blood pressure and downward on the lumbar nerve root and is causing more pain. The
rapid respirations pulse rate would increase with aneurysm extension. Chest pain
b) Decreased pulse rate and blood radiating down the arm would indicate myocardial infarction. Blood
pressure pressure would decrease with aneurysm extension, and the
c) Increased abdominal and back respiratory rate may not be affected.
pain
d) Retrosternal back pain radiating
to the left arm
1914 Which statement best describes the They block sodium reabsorption in the ascending loop and dilate
. therapeutic action of loop renal vessels.---> Loop diuretics block sodium reabsorption in the
diuretics? ascending loop of Henle, which promotes water diuresis. They also
dilate renal vessels. Although loop diuretics block potassium
reabsorption, this isn't a therapeutic action. Thiazide diuretics, not
loop diuretics, promote sodium secretion into the distal tubule.
1915 Which statement best describes the Antibiotics given for a prolonged period are the usual treatment of
. treatment of lung abscess? choice
1916 Which statement reflects the Risk for injury related to syncope and confusion
. highest priority nursing diagnosis
for an older client recently
admitted to the hospital for a new-
onset cardiac dysrhythmia?
1917 Which suggestion would be most Don prper footweat
. important to give a client who has a
mild case of bunions?
1918 Which task can be safely delegated c) Changing the dressing of a client who underwent surgery two days
. to a licensed practical nurse ago.
(LPN)?

a) Teaching a newly diagnosed


diabetic about insulin
administration.
b) Admitting a client who
underwent a thoracotomy to the
nursing unit from the
postanesthesia care unit.
c) Changing the dressing of a client
who underwent surgery two days
ago.
d) Administering an I.V. bolus of
morphine sulfate to a client
experiencing incisional pain
1919 Which two things affect renal fluid RAAS
. volume control? natriuretic peptides
1920 Which type of fat should primarily polyunsaturated fats (vegetable oils, shellfish, walnuts, seeds,
. be consumed? margarine)
1921 Which type of graft is utilized when Xenograft
. a heart valve replacement is made
of tissue from an animal heart
valve?
1922 Which type of lipoprotein contains LDLs - low serum LDLs are desirable
. more cholesterol than any of the
others and is the most closely
correlated with incidence of
atherosclerosis?
1 Which type of sleep apnea is charcterized by lack of Obstructive
9 airflow due to pharyngeal occlusion?
2
3.

1 While ambulating a patient with metastatic lung cancer, C


9 the nurse observes a drop in oxygen saturation from
2 93% to 86%. Which of the following nursing
4. interventions is most appropriate based upon these
findings?
A) Continue with ambulation since this is a normal
response to activity.
B) Obtain a physician's order for arterial blood gas
determinations to verify the oxygen saturation.
C) Obtain a physician's order for supplemental oxygen
to be used during ambulation and other activity.
D) Move the oximetry probe from the finger to the
earlobe for more accurate monitoring during activity.
1 While caring for a patient with primary pulmonary Right ventricular hypertrophy and dilation
9 hypertension, the nurse observes that the patient has
2 exertional dyspnea and chest pain, in addition to
5. fatigue. The nurse knows that these symptoms are
related to:
1 While receiving heparin to treat a pulmonary embolus, Prepare to administer protamine sulfate.
9 a client passes bright red urine. What should the nurse
2 do first?
6.

1 A white male, age 43, with a tentative diagnosis of History of aortic valve replacement
9 infective endocarditis is admitted to an acute care
2 facility. His medical history reveals diabetes mellitus,
7. hypertension, and pernicious anemia; he underwent an
appendectomy 20 years earlier and an aortic valve
replacement 2 years before this admission. Which
history finding is a major risk factor for infective
endocarditis?
1 White pulmonary secretions are normal with deep partial-thickness and full-
9 thickness burns on the face, arms, and chest
2
8.

1 Who is at risk for HTN crisis? patients with hx of HTN who have been
9 undermedicated or who have failed to comply
2 with medications
9.

1 Who is more likely to have white coat hypertension? Older adults


9
3
0.

1 Who is most likely to have CAD? White, middle-aged men


9
3
1.

1 Whole grains are a good source of thiamine.


9
3
2.

1 Why are antihistamines not usually prescribed for acute They may cause excessive drying and make
9 tracheobronchitis? secretions more difficult to expectorate.
3
3.

1 Why does a ventilation-perfusion mismatch occur in the Hypoventilation


9 affected area of the lung during pneumonia?
3
4.

1 Why would someone after an MI be given a stool Because after an MI you are on bedrest and have
9 softener? been on opioids
3
5.

1 With a client in metabolic alkalosis, the nurse should hypokalemia.


9 monitor for
3
6.

1 With a client in metabolic alkalosis, the nurse should hypokalemia


9 monitor for
3
7.

1 With a diagnosis of right rib fracture and closed Semi- to high-Fowler's position, tilted toward the
9 pneumothorax, the client should be placed in: right side.
3
8.

1 Within the physiology of the heart, each chamber has a Left ventricle
9 particular role in maintaining cellular oxygenation.
3 Which chamber of the heart is responsible for pumping
9. blood to all the cells and tissues of the body?
1 Within the physiology of the heart, each chamber has a Left atrium--> The left artrium receives
9 particular role in maintaining cellular oxygenation. oxygenated blood from the lungs
4 Which chamber of the heart is responsible for receiving
0. oxygenated blood from the lungs
1 with terminal ilieum resection surgery vitamin B12 absorbed regardless of the amount of oral intake of
9 tablet cannot be sources of vitamin B12, such as animal protein or
4 vitamin B12 tablets.
1.

1 A woman with a uterus who takes unopposed estrogen endometrial cancer. The addition of progesterone
9 has an increased risk of prevents the formation of endometrial hyperplasia.
4
2.
1 women chlamydia An increase in vaginal discharge caused by an
9 inflamed cervix; the need to urinate more
4 frequently, or pain whilst passing urine; pain
3. during sexual intercourse or bleeding after sex;
lower abdominal pains; irregular menstrual
bleeding.
1 women gonorrhea Painful or frequent urination.
9 Anal itching, discomfort, bleeding, or discharge.
4 Abnormal vaginal discharge.
4. Abnormal vaginal bleeding during or after sex or
between periods.
Genital itching.
1 A wound (regardless of its size) that contains tan, leathery tissue requires evaluation by the
9 wound care nurse. This wound most likely requires
4 debridement before wound healing can take place.
5.

1 Wrapping elastic bandages on dependent areas limits edema formation and bleeding and promotes graft
9 acceptance.
4
6.

1 Written informed consent is required prior to any A male client arrives at the outpatient surgery
9 invasive procedure. The healthcare provider must center for a scheduled needle aspiration of the
4 explain the procedure to the client, but the nurse can knee. He tells the nurse that he has already given
7. witness the client's signature on a consent form (A). (B) verbal consent for the procedure to the healthcare
is not necessary since written consent must be obtained. provider. What action should the nurse implement?
(C) is not correct because written consent has not been A. Witness the client's signature on the consent
obtained. (D) must occur after written consent is form.
obtained. B. Verify the client's consent with the healthcare
Correct Answer: A provider.
C. Notify the healthcare provider that the client is
ready for the procedure.
D. Document that the client has given consent for
the needle aspiration.
1 Yellow, waxy deposits on the lower eyelids, bright red age-related changes to skin.
9 moles on the hands, and areas of dry, scaly skin are
4 normal
8.

1 You are caring for a client with a damaged tricuspid Chordae tendineae
9 valve. You know that the tricuspid valve is held in place
4 by which of the following?
9.

1 You are monitoring the results of laboratory tests Enzymes---->When tissues and cells break down,
9 performed on a client admitted to the cardiac ICU with are damaged, or die, great quantities of certain
5 a diagnosis of myocardial infarction. Which test would enzymes are released into the bloodstream.
0. you expect to show elevated levels? Enzymes can be elevated in response to cardiac or
other organ damage. After an MI, RBCs and
platelets should not be elevated. WBCs would only
be elevated if there was a bacterial infection
present.
1 You are part of a group of nursing students who are Stroke
9 making a presentation on chronic hypertension. What
5 is one subject you would need to include in your
1. presentation as a possible consequence of untreated
chronic hypertension?
1 You are the clinic nurse doing assessments on your Rate, quality, and rhythm
9 clients before they have outpatient diagnostic testing
5 done. What would you document when assessing the
2. client's pulse?
1 You enter your client's room and find them pulseless Immediate defibrillation--->Defibrillation is used
9 and unresponsive. What would be the treatment of during pulseless ventricular tachycardia,
5 choice for this client? ventricular fibrillation, and asystole (cardiac
3. arrest) when no identifiable R wave is present.
A 2- Give acetaminophen (Tylenol).
month- Explanation: Acetaminophen should be given first to decrease the infant's temperature.
old A heart rate of 180 beats/minute is normal in an infant with a fever. Fluid intake is
infant encouraged after the acetaminophen is given to help replace insensible fluid losses.
arrives Carotid massage is an attempt to decrease the heart rate as a vagal maneuver; it won't
in the work in this infant because the source of the increased heart rate is fever. A tepid sponge
emergen bath may be given to help decrease the temperature and calm the infant.
cy
departm
ent with
a heart
rate of
180
beats/mi
nute and
a
temperat
ure of
103.1 F
(39.5 C)
rectally.
Which
intervent
ion is
most
appropri
ate?
2. A 2-year-old Place the toddler in respiratory isolation
child is Explanation: Nurses should take necessary precautions to protect
admitted to themselves and others from possible infection from the bacterial
the pediatric organism causing meningitis. The affected child should immediately be
unit with the placed in respiratory isolation; then the parents can be informed about
diagnosis of the treatment plan. This should be done before laboratory tests are
bacterial performed.
meningitis.
Which
diagnostic
measure
would be
appropriate
for the nurse
to perform
first?
3. A 2-year-old Intercostal retractions
child is Explanation: Clinical manifestations of respiratory distress include
brought to tachypnea, tachycardia, restlessness, dyspnea, and intercostal
the retractions. Fever is a sign of infection. Bradycardia is a late sign of
emergency impending respiratory arrest. Cyanosis, not pallor, is a sign of
department increasing respiratory distress.
with
suspected
croup.
Which data
collection
finding
reflects
increasing
respiratory
distress?
4. A 2-year-old 24 to 72 hours
girl is Explanation:
scheduled to Myringotomy also allows the drainage to be analyzed (by culture and
have a sensitivity testing) so that the infecting organism can be identified and
myringotom appropriate antibiotic therapy prescribed. The incision heals within 24
y. How long to 72 hours. This makes options A, B, and C incorrect.
would the
nurse tell the
parents that
it will take
for the
incision to
heal?
5. A 3-year-old Perform chest physiotherapy as ordered.
client is Encourage coughing and deep breathing.
admitted to Perform postural drainage.
the pediatric Maintain humidification with a cool mist humidifier.
unit with Explanation: Chest physiotherapy and postural drainage work together
pneumonia. to break up congestion and then drain secretions. Coughing and deep
He has a breathing are also effective to remove congestion. A cool mist
productive humidifier helps loosen thick mucous and relax airway passages. Fluids
cough and should be encouraged, not restricted. The child should be placed in
appears to semi-Fowler's to high Fowler's position to facilitate breathing and
have promote optimal lung expansion.
difficulty
breathing.
The parents
tell the nurse
that the
toddler
hasn't been
eating or
drinking
much and
has been
very
inactive.
Which
interventions
to improve
airway
clearance
should be
included by
the nurse in
the care
plan? Select
all that
apply:
6. A 4-year-old It's used to relieve pain and reduce the risk of infection
child had a Explanation: The hematoma is treated with electrocautery to relieve
subungual pain and reduce the risk of infection. Electrocautery doesn't prevent the
hemorrhage loss of the nail. The discoloration seen with subungual hemorrhage is
of the toe from the collection of blood under the nail bed. It isn't permanent and
after a jar doesn't affect nail growth.
fell on his
foot.
Electrocaute
ry is
performed.
Which
teaching
statement
regarding
the rationale
for using
electrocauter
y to treat the
injury is
most
accurate?
7. An 8-year- Fever, muscle weakness, and change in mental status
old client has Correct
tested Explanation: Severe West Nile virus infection (also called West Nile
positive for encephalitis or West Nile meningitis) affects the central nervous system
West Nile and may cause headache, neck stiffness, fever, muscle weakness or
virus paralysis, changes in mental status, and seizures. Such signs and
infection. symptoms as fever, rash, malaise,anorexia, nausea and vomiting, and
The nurse lymphadenopathy suggest the mild form of West Nile virus infection.
suspects the
client has the
severe form
of the disease
when she
recognizes
which signs
and
symptoms?
8. 10 Minutes A nurse has come upon an unresponsive, pulseless victim. She has
placed a 911 call and begins CPR. The nurse understands that if the
patient has not been defibrillated within which time frame, the chance
of survival is close to zero?
9 A 10-year-old child has Diabetes insipidus
. been experiencing Explanation: Polydipsia and polyuria with normal serum glucose are indicative of
insatiable thirst and diabetes insipidus. Interview and laboratory results can determine whether the origin
urinating excessively; is neurogenic or nephrogenic. Type 1 or 2 diabetes mellitus present with an elevated
his serum glucose is serum glucose. A child with hyperthyroidism may present as dehydrated from the
normal. Which excessive sweating and rapid respirations that accompany this hypermetabolic state.
condition is the child
probably
experiencing?
1 A 12-year-old child Respiratory infection
0 diagnosed with Explanation: Respiratory infection can be fatal for children with muscular dystrophy
. muscular dystrophy is due to poor chest expansion and decreased ability to mobilize secretions. Skin
hospitalized secondary integrity, infection of pin sites, and nonunion healing are all causes for concern, but
to a fall. Surgery is not as important as prevention of respiratory infection.
necessary as well as
skeletal traction.
Which complication
would be of greatest
concern to the nursing
staff?
1 A 13-month-old is Give clear liquids in small amounts
1 admitted to the Encourage the child to eat nonsalty soups and broths
. pediatric unit with a Monitor the I.V. solution per the physician's order
diagnosis of Explanation: A child experiencing nausea and vomiting won't be able to tolerate a
gastroenteritis. The regular diet. He should be given sips of clear liquids, and the diet should be advanced
toddler has as tolerated. Unsalted soups and broths are appropriate clear liquids. I.V. fluids should
experienced vomiting be monitored to maintain the fluid status and help to rehydrate the child. Milk
and diarrhea for the shouldn't be given because it can worsen the child's diarrhea. Solid foods may be
past 3 days, and withheld throughout the acute phase, but clear fluids should be encouraged in small
laboratory tests reveal amounts (3 to 4 tablespoons every half hour).
that he's dehydrated.
Which nursing
interventions are
correct to prevent
further dehydration?
Select all that apply:
1 A 13-year-old client at Ventilation
2 the pulmonary clinic Gas exchange
. where you practice
nursing has an
extensive history of
asthma and is seeing
the pulmonologist for
her monthly
appointment. What are
the primary functions
of the lungs? Choose
all correct options.
1 A 14-year-old female Identity
3 client in skeletal Explanation: According to Erikson's theory of personal development, the adolescent is
. traction for treatment in the stage of identity versus role confusion. During this stage, the body is changing
of a fractured femur is as secondary sex characteristics emerge. The adolescent is trying to develop a sense of
expected to be identity, and peer groups take on more importance. When an adolescent is
hospitalized for several hospitalized, she is separated from her peer group and body image may be altered.
weeks. When planning Toddlers are in the developmental stage of autonomy versus shame and doubt.
care, the nurse should Preschool children are in the stage of initiative versus guilt. School-age children are in
take into account the the stage of industry versus inferiority.
client's need to achieve
what developmental
milestone?
1 A 15-year-old client Complete Blood Count with differential
4 has been brought to
. the clinic by their
mother and is
suspected of having an
immune system
disorder. What tests
would you expect to be
ordered for this young
client?
1 A 15-year-old client Encouraging visitation by his friends
5 who sustained a spinal Explanation: Encouraging visitation by friends might best help the adolescent cope
. cord injury is on with prolonged bedrest. Friends are much more important than family to this age-
bedrest. Which group. Providing reading material and video games might be somewhat helpful, but
intervention by the not as helpful as encouraging visits from friends.
nurse might best help
the adolescent cope
with the prolonged
bedrest?
1 An 18-month-old child right upper lung
6 is brought to the
. Emergency
Department by parents
who explain that their
child swallowed a
watch battery.
Radiologic studies
show that the battery
is in the lungs. Which
area of lung is the
battery most likely to
be in?
17 An 18-month-old male child is admitted to the A protuberant abdomen
. pediatric unit with a diagnosis of celiac disease. Explanation: A child with celiac disease has a
What finding would the nurse expect in this child? protuberant abdomen, diarrhea, steatorrhea, and
anorexia, which result in malnutrition. A concave
abdomen, bulges in the groin area, and a palpable
abdominal mass aren't associated with celiac disease.
18 A 21-year-old client with cystic fibrosis develops c) it may induce bronchospasm.
. pneumonia. To decrease the viscosity of
respiratory secretions, the physician prescribes
acetylcysteine (Mucomyst). Before administering
the first dose, the nurse checks the client's history
for asthma. Acetylcysteine must be used
cautiously in a client with asthma because:

a) it's a respiratory depressant.


b) it's a respiratory stimulant.
c) it may induce bronchospasm.
d) it inhibits the cough reflex.
19 A 24-year-old athlete is admitted to the trauma Absorb nutrients
. unit following a motor-vehicle collision. The client
is comatose and has developed ascites as a result
of the accident. You are explaining the client's
condition to his parents. In your education, what
do you indicate is the primary function of the
small intestine?
20 A 29-year-old client with severe shortness of b) Airborne and contact precautions
. breath comes to the emergency department. He
tells the emergency department staff that he
recently traveled to China for business. Based on
his travel history and presentation, the staff
suspects severe acute respiratory syndrome
(SARS). Which isolation precautions should the
staff institute?

a) Droplet precautions
b) Airborne and contact precautions
c) Contact and droplet precautions
d) Contact precautions
21 A 30-year-old client is admitted to the emergency Necrotic tissue through most of the dermis
. department with a deep partial-thickness burn on Explanation: A deep partial-thickness burn causes
his arm after a fire in his workplace. Which signs necrosis of the epidermal and dermal layers. Redness
and symptoms should the nurse expect to see? and pain are characteristics of a superficial injury.
Superficial burns cause slight epidermal damage. With
deep burns, the nerve fibers are destroyed and the client
doesn't feel pain in the affected area. Necrosis through
all skin layers is seen with full-thickness injuries.
22 A 33-year-old female client tells the nurse she has Assess the couple's perception of the problem
. never had an orgasm. She tells the nurse that her Explanation:
partner is upset that he can't meet her needs. Assessing the couple's perception of the problem will
Which nursing intervention is most appropriate? define it and assist the couple and the nurse in
understanding it. A nurse can't make a medical
diagnosis such as sexual aversion disorder. Most
women can be taught to reach orgasm if there's no
underlying medical condition. When assessing the
client, the nurse should be professional and matter-of-
fact; she shouldn't make the client feel inadequate or
defensive
23 A 33-year-old woman with primary pulmonary hypertension is being evaluated a) Oxygen
. for a heart-lung transplant. The nurse asks her what treatments she is currently c) Diuretics
receiving for her disease. She is likely to mention which treatments? d) Vasodilators
a) Oxygen
b) Aminoglycosides
c) Diuretics
d) Vasodilators
e) Antihistamines
f) Sulfonamides
24 A 38-year-old female patient has begun to suffer from rheumatoid arthritis. She Her use of other
. is also being assessed for disorders of the immune system. She works as an aide at drugs.
a facility that cares for children infected with AIDS. Which of the following is the
most important factor related to the patient's assessment?
25 A 38-year-old female patient has begun to suffer from rheumatoid arthritis. She Her use of other drugs
. is also being assessed for disorders of the immune system. She works as an aide at
a facility that cares for children infected with AIDS. Which of the following is the
most important factor related to the patient's assessment?
26 A 44-year-old client has a history of hypertension. As her nurse, you engage her Kidneys
. in client education to make her aware of structures that regulate arterial
pressure. Which of the following structures are a component of that process?
27 A 44-year-old client is in the hospital unit where you practice nursing. From the Proteinuria
. results of a series of diagnostic tests, she has been diagnosed with acute
glomerulonephritis. What would you expect to find as a result of this condition?
28 A 45-year-old waitress with a history of IV drug use also is HIV-positive. She has CD4 count < 200/mm
. been following her antiretroviral medication regimen faithfully and is doing well. indicates a diagnosis
She's attending college to get a social work degree and is focused on a bright of AIDs to be made.
future. In her regular CD counts, what factor will indicate she has progressed
from HIV to AIDS?
29 A 47-year-old male client with unresolved hemothorax is febrile, with chills and b) empyema.
. sweating. He has a nonproductive cough and chest pain. His chest tube drainage
is turbid. A possible explanation for these findings is:

a) lobar pneumonia.
b) empyema.
c) Pneumocystis carinii pneumonia.
d) infected chest tube wound site.
30 A 53-year-old client is seeing the physician today because he has had laryngitis producing sound
. for 2 weeks. After a thorough examination, the doctor orders medications and
instructs the client to follow up in 1 week if his voice has not improved. What is
the primary function of the larynx?
31 A 57-year-old homeless female with a history of 3rd Spacing - Third-spacing describes the
. alcohol abuse has been admitted to your hospital translocation of fluid from the intravascular or
unit. She was admitted with signs and symptoms of intercellular space to tissue compartments, where it
hypovolemia - minus the weight loss. She exhibits a becomes trapped and useless. The client manifests
localized enlargement of her abdomen. What signs and symptoms of hypovolemia with the
condition could she be presenting? exception of weight loss. There may be signs of
localized enlargement of organ cavities (such as the
abdomen) if they fill with fluid, a condition referred
to as ascites.
32 A 57-year-old homeless female with a history of 3rd Spacing - Third-spacing describes the
. alcohol abuse has been admitted to your hospital translocation of fluid from the intravascular or
unit. She was admitted with signs and symptoms of intercellular space to ...(more)
hypovolemia - minus the weight loss. She exhibits a Third-spacing describes the translocation of fluid
localized enlargement of her abdomen. What from the intravascular or intercellular space to tissue
condition could she be presenting? compartments, where it becomes trapped and useless.
The client manifests signs and symptoms of
hypovolemia with the exception of weight loss. There
may be signs of localized enlargement of organ
cavities (such as the abdomen) if they fill with fluid, a
condition referred to as ascites.
33 A 64-year-old client reports symptoms consistent Impaired cerebral circulation
. with a transient ischemic attack (TIA) to the
physician in the emergency department. After
completing ordered diagnostic tests, the physician
indicates to the client what caused the symptoms
that brought him to the hospital. What is the origin
of the client's symptoms?
34 A 64-year-old male client, who leads a sedentary The male client because of his age
. life-style, and a 31-year-old female client, who has a
very stressful and active life-style, require a vaccine
against a particular viral disorder. As the nurse,
you would know that in one of this client's the
vaccine will be less effective. In which client is the
vaccine more likely to be less effective and why?
35 A 67-year-old client is returning for a follow-up Reduce fluid accumulation
. appointment to the primary care group where you Reduce venous pressure
practice nursing. At his last appointment, he
received the diagnosis of portal hypertension and
the physician instituted interventions to begin
treatment of this condition. What is the primary
aim of portal hypertension treatment? Choose all
correct options.
36 A 67-year-old client is returning for a follow-up Reduce venous pressure
. appointment to the primary care group where you Reduce fluid accumulation
practice nursing. At his last appointment, he
received the diagnosis of portal hypertension and
the physician instituted interventions to begin
treatment of this condition. What is the primary
aim of portal hypertension treatment? Choose all
correct options.
37 A 68-year-old resident at a long-term care facility Lack of free water intake
. lost the ability to swallow following a stroke 4 years
ago. She receives nutrition via a PEG tube. The
client remains physically and socially active and has
adapted well to the tube feedings. Occasionally, the
client develops constipation that requires
administration of a laxative to restore regular
bowel function. Which of the following is the most
likely cause of this client's constipation?
38 A 72-year-old client seeks help for chronic decreased abdominal strength.
. constipation. Constipation is a common problem
for elderly clients because of several factors related
to aging, including:
39 A 73-year-old client has been admitted to the Moist, gurgling respirations
. cardiac step-down unit where you practice nursing.
After diagnostics, she was brought to your unit with
acute pulmonary edema. Which of the following
symptoms would you expect to find during your
assessment?
40 An 83-year-old client is undergoing lipid profile LDL sticks to the arteries
. studies in an effort to determine a proper
nutritional balance for his CAD. In his lipid
profile, his LDL is greater than his HDL. Why is
this a risk factor for this client?
41 An 84-year-old woman is to receive 2 units of slow the rate of the transfusion and obtain an order for
. packed red blood cells. During the transfusion of furosemide (Lasix)
the first unit at 125 mL/hour, the client reports
shortness of breath 30 minutes into the process.
The client exhibits the vital signs shown in the
accompanying table. The best nursing intervention
is to:
42 An 89-year-old client lives in a long-term care Gradual changes in position provide time for the heart
. facility where you practice nursing. The client has to increase rate of contraction to resupply oxygen to
a hypertensive history and has fallen several times the brain.
in the past two weeks. As his nurse, why is it Explanation:
important for you to encourage the client to rise It is important for the nurse to encourage the client to
slowly from a sitting or lying position? rise slowly from a sitting or lying position because
gradual changes in position provide time for the heart
to increase its rate of contraction to resupply oxygen to
the brain.
43 153. A nurse is developing a plan of care for a 1. Have the client void immediately before surgery.
. client who is scheduled for surgery. The nurse (The nurse w/assist client w/voiding immediately
w/include which of the following activites in the before surgery so that bladder will be empty. Oral
nuring care plan for the client on the day of hygiene is allowed, but client s/not swallow any water.
surgery? Client usually has restriction of food and fluids for 8
hrs. before surgery rather than 24 hrs. A slight increase
in blood pressure and pulse is common during
preoperative period; this is generally result of anxiety.
44 154. A nurse is caring for a client who is scheduled 3. Ask the cliet to discuss information known about the
. for surgery. The client is concerned about the planned surgery.
surgical procedure. To alleviate the client's fears
and misconceptions about surgery, the nurse
should:
45 155. A nurse is collecting data from a client who is 3. Discontinue the aspirin 48 hrs. before the scheduled
. scheduled for surgery in 1 week in the ambulatory surgery.
care surgical center. The nurse notes that the client Anticoagulants alter normal clotting factors and
has a history of arthritis and has been taking increase the risk of hemorrhage. Aspirin has properties
acetylsalicylic acid (aspirin). The nurse reports the that can alter the clotting mechanism and s/thus be
information to the physician and anticipates that discontinued at least 48 hrs. before surgery.
the physician will prescribe which of the
following?
46 156. A nurse obtains vital signs on a postoperative 2. Continue to monitor vital signs.
. client who just returned to nursing unit. The A slightly lower-than-normal BP and increased pulse
client's BP is 100/60 mm Hg, the pulse 90 rate are common after surgery. Warm blankets are
beats/min., and respiration rate is 20 breaths/min.. applied to maintain the client's body temp. The level of
On basis of these findings,which of the following consciousness can be determined by checking the
nursing actions s/be performed? client's response to light touch and verbal stimuli rather
than by shaking the client. There is no reason to
contact the RN immediately.
47 157. A client arrives to the surgical nursing unit 1. Patency of the airway.
. after surgery. The initial nursing action is to check If the airway is not patent, immediate measures must
the: be taken for the survival of the client. After checking
the client's airway, the nurse would then check the
client's vital signs, and this w/be followed by checking
the dressings, tubes and drains.
48 158. A nurse is monitoring an adult client for 1. A uninary output of 20mL/Hour.
. postoperative complications. Which of the Urine output is maintained at a minimum of at least 30
following w/be the most indicative of a potential mL/hr. for an adult. An ouptut of less than 30mL/hr.
postoperative complication that requires further for each ot 2 consecutive hrs. s/be reported to the
observation? physician. A temp. more than 100F or less than 97F
and a falling systolic blood pressure <90mm Hg are to
be reported. The client's preoperative or baseline blood
pressure is used to make informed postoperative
comparisons. Moderate or light serous drainage from
the surgical site is considered normal.
49 159. A nurse monitors the postoperative 1. Pneumonia
. client frequently for the presence of The most common postoperative respiratory problems are
secretion in the lungs, knowing that atelectasis, pneumonia, and pulmonary emboli. Pneumonia is
accumulated secretions can lead to: the inflammation of lung tissue that causes a productive
cough, dyspnea, and crackles. Pulmonary edema usually
results from L-sided heart failure, and it can be caused by
medications, fluid overload, and smoke inhalation. CO2
retention results from the inability to exhale CO2 in clients
w/conditions such as COPD. Fluid imbalance can be a deficit
or excess related to fluid loss or overload.
50 160. A nurse is caring for a postoperative 4. Secure the drain by curling or folding it and taping it
. client who has a drain inserted into the firmly to the body.
surgical wound. Which of the following Aseptic technique must be used when emptying the drainage
nursing actions w/be inappropriate for the container or changing the dressing to avoid contamination of
care of the drain? the wound. Usualy drainage from the wound is pale, red, and
watery, whereas active bleeding will be bright red in color.
The drain s/be checked for patency to provide an exit for the
fluid or blood to promote healing. The nurse needs to ensure
that drainage flows freely and that there are no kinks in the
drains. Curling or folding the drain prevents the flow of the
drainage.
51 161. A nurse checks the client's surgical 2. The presence of purulent drainage.
. incision for signs of infection. Which of the S/Sx of a wound infection include warm, red, and tender skin
following w/be indicative of a potential around the incision. The client may have fever and chills.
infection? Purulent material may exit from drains or from separated
wound edges. Infection may be caused by poor aseptic
technique or a wound that was contaminated before surgical
exploration; it appears 3-6 days after surgery. Serous drainage
is not indicative of a wound infection. A temp. of 98.8 F is
not an abnormal finding in a postoperative client. The fact
that a client feels cold is not indicative of an infection,
although chills and fever are signs of infection.
52 162. A nurse is checking a client's surgical 2. Apply sterile dressing soaked w/normal saline to the
. incision and notes an increase in the amount wound.
of drainage, a separation of the incision line, Wound dehiscence is the separation of the wound edges at the
and the appearance of underlying tissue. suture line. S/sx include increased drainage and appearance
Which of the following is the intial action? of underlying tissues. It usually occurs as a complication 6-8
days. The client s/be instructed to remain quiet and to avoid
coughing or straining, and he or she s/be positioned to
prevent further stress on the wound. Sterile dressings soaked
w/sterile normal saline s/be used to cover the wound. The
physician needs to be notified.
53 163. A nurse monitors a postoperatve client 1. Increasing restlessness noted in a client is a sign that
. for signs of complications. Which of the requires continuous and lose monitoring, because it could be
following signs w/the nurse determine to be a potential indication of a complication such as hemorhage or
indicative of a potential complication? shock. Neg Homan's sign is normal. + sign indicative of
thrombophlebitis. Faint bowel sounds in all 4 quadrants is
normal. BP 120/70, pulse of 90 relativel normal.
54 164. A nurse is explaining the concept of 4. To allow the surgical team a chance to verbally verify their
. time-out in the perioperative area. The agreement about the client's name, surgical procedure, and
purpose of time out is: the site.
The time-out occurs in the perioperative area after the client
has been prepped and draped. The entire team must verbally
verify their agreement regarding the client's name, the
procedure to be performed, and the surgical site.
55 165. A nurse is explaining the Joint 1. The surgeon marking the area of the operative procedure.
. Commission's universal protocol for The surgeon is responsible for verifying the operative site,
preventing wrong-site, wrong-procedure, and he or she must mark the operative site before the client is
and wrong-person surgery to a group of brought into the operating suite. The client will be asked to
nursing students. The nurse explains that verify the site that requires surgery. The client may refuse to
site marking involves: have the site marked and is asked about marking the site.
56 166. A client who had abdominal surgery 1. Notify RN
. complains of feeling as though something 2. Document the client's complaint.
gave way in the incisional site. The nurse 3. Instruct the client to remain quiet.
removes the dressing and notes the presence 4. Prepare the client for wound closure.
of a loop of bowel protruding through the
incision. Select all nursing interventions that
nurse w/take.:
57 Abcesses? a localized collection of pus in the tissues of the body, often
. accompanied by swelling and inflammation and frequently
caused by bacteria.
58 According to the DASH diet, how many servings of 4 to 5
. vegetables should a person consume per day?
59 Acetylcholine Patients diagnosed with myasthenia gravis have a
. decrease in which of the following receptors?
60 Ada Zontor, a 60-year-old bookkeeper, is a client It's good to know the continual tingling in my fingers
. with the neurological group where you practice and toes is not connected with my nervous system!
nursing. Mrs. Zontor has been exhibiting
neurological symptoms for several weeks and the
neurologist is admitting her to hospital for extensive
testing. Since diagnostics have not yet revealed the
cause of her difficulties, which of her following
comments would indicate the need for further client
education?
61 Adequate hourly urine output for a patient with an 30 mL/hr.
. indwelling urinary catheter is
62 An adolescent, age 16, is brought to the clinic for "Do you like yourself physically?"
. evaluation for a suspected eating disorder. To best Explanation: Role and relationship patterns focus on
evaluate the effects of role and relationship patterns body image and the client's relationship with others,
on the child's nutritional intake, the nurse should which commonly interrelate with food intake.
ask: Questions about activities and food preferences elicit
information about health promotion and health
protection behaviors. Questions about food allergies
elicit information about health and illness patterns.
63 An adolescent is started on valproic acid to treat "A common adverse effect is weight gain."
. seizures. Which statement should be included when Explanation: Weight gain is a common adverse effect
educating the adolescent? of valproic acid. Drowsiness and irritability are
adverse effects more commonly associated with
phenobarbital. Felbamate (Felbatol) more commonly
causes insomnia.
64 An adult client reports that it is taking longer than "Are you regularly taking aspirin?"
. usual for minor cuts and injuries to clot. Which of
the following questions would the nurse most likely
ask the client?
65 An adult client with cystic fibrosis is admitted to an b) At least 2 hours after a meal
. acute care facility with an acute respiratory
infection. Prescribed respiratory treatment includes
chest physiotherapy. When should the nurse
perform this procedure?

a) Immediately before a meal


b) At least 2 hours after a meal
c) When bronchospasms occur
d) When secretions have mobilized
66 After an anterior wall myocardial infarction (MI), Left-sided heart failure
. which problem is indicated by auscultation of Explanation: The left ventricle is responsible for
crackles in the lungs? most of the cardiac output. An anterior wall MI may
result in a decrease in left ventricular function. When
the left ventricle doesn't function properly, resulting
in left-sided heart failure, fluid accumulates in the
interstitial and alveolar spaces in the lungs and
causes crackles. Pulmonic and tricuspid valve
malfunction causes right-sided heart failure.
67 After an anterior wall myocardial infarction (MI), Left-sided heart failure
. which problem is indicated by auscultation of Explanation:
crackles in the lungs? The left ventricle is responsible for most of the
cardiac output. An anterior wall MI may result in a
decrease in left ventricular function. When the left
ventricle doesn't function properly, resulting in left-
sided heart failure, fluid accumulates in the interstitial
and alveolar spaces in the lungs and causes crackles.
Pulmonic and tricuspid valve malfunction causes
right-sided heart failure.
68 After a plane crash, a client is brought to the evaluation of the corneal reflex response.
. emergency department with severe burns and
respiratory difficulty. The nurse helps to secure a
patent airway and attends to the client's immediate
needs, then prepares to perform an initial
neurologic assessment. The nurse should perform
an:
69 After diagnosing a client with pulmonary d) 6 to 12 months
. tuberculosis, the physician tells family members
that they must receive isoniazid (INH [Laniazid])
as prophylaxis against tuberculosis. The client's
teenage daughter asks the nurse how long the drug
must be taken. What is the usual duration of
prophylactic isoniazid therapy?

a) 3 to 5 days
b) 1 to 3 weeks
c) 2 to 4 months
d) 6 to 12 months
70 After receiving an oral dose of codeine for an a) In 30 minutes
. intractable cough, the client asks the nurse, "How
long will it take for this drug to work?" How
should the nurse respond?

a) In 30 minutes
b) In 1 hour
c) In 2.5 hours
d) In 4 hours
71 After receiving the wrong medication, the client's b) Document the incident in the nurses' notes.
. breathing stops. The nurse initiates the code
protocol, and the client is emergently intubated. As
soon as the client's condition stabilizes, the nurse
completes an incident report. What should the
nurse do next?

a) Place the incident report on the client's chart.


b) Document the incident in the nurses' notes.
c) Document in the nurses' notes that an incident
report was completed.
d) Make a copy of the incident report for the client.
72 After reviewing the various primary Angioneurotic edema
. immunodeficiencies, nursing students demonstrate Paroxysmal nocturnal hemoglobinuria
understanding of the information when they
identify which of the following as complement
system deficiencies? Select all that apply.
73 After sustaining a stroke, a client is transferred to Urinary incontinence and right-sided hemiparesis
. the rehabilitation unit. A medical-surgical nurse
reviews the client's residual neurological deficits
with a rehabilitation nurse. Which neurological
deficit places the client at risk for skin breakdown?
74 After teaching a class about the inheritance patterns of Familial Alzheimer's disease
. different conditions, the instructor determines that the Anencephaly
teaching was successful when the students identify which of Osteroarthritis
the following as resulting from multifactorial inheritance? Congenital heart defect
Select all that apply.
75 After teaching a client with immunodeficiency about ways "I should avoid being around other people
. to prevent infection, the nurse determines that teaching was who have an infection."
successful when the client states which of the following?
76 After teaching a group of students about irritable bowel Immodium
. syndrome and antidiarrheal agents, the instructor
determines that the teaching was effective when the
students identify which of the following as an example of an
opiate-related antidiarrheal agent?
77 After teaching a group of students about the various organs Stomach
. of the upper gastrointestinal tract and possible disorders,
the instructor determines that the teaching was successful
when the students identify which of the following structures
as possibly being affected?
78 After teaching nursing students about autosomal-dominant Horizontal transmission is more commonly
. and autosomal-recessive inherited disorders, the instructor seen in families.
determines that the teaching was successful when the class
identifies which of the following as true about autosomal-
dominant inherited conditions?
79 After teaching nursing students about autosomal-dominant Horizontal transmission is typically seen in
. and autosomal-recessive inherited disorders, the instructor families
determines that the teaching was successful when the class
identifies which of the following as true about autosomal-
dominant inherited conditions?
80 After undergoing a left thoracotomy, a client has a chest c) encourage coughing and deep breathing.
. tube in place. When caring for this client, the nurse must:

a) report fluctuations in the water-seal chamber.


b) clamp the chest tube once every shift.
c) encourage coughing and deep breathing.
d) milk the chest tube every 2 hours.
81 After undergoing a liver biopsy, a client should be placed in Right lateral decubitus position
. which position?
82 Agammaglobulinemia is also known as which of the Bruton's Disease
. following?
83 Age-related changes associated with the cardiac system endocardial fibrosis.
. include Explanation:
Age-related changes associated with the
cardiac system include: endocardial fibrosis,
increased size of the left atrium, decreasing
number of SA node cells, and myocardial
thickening
84 Air embolism is a potential complication of IV therapy. The Chest Pain - Manifestations of air embolism
. nurse should be alert to which clinical manifestation include dyspnea and cyanosis; hypotension;
associated with air embolism? weak, rapid pulse; loss of consciousness; and
chest, shoulder, and low back pain. Jaundice
is not associated with air embolism.
85 Albumin is a protein in the plasma portion of the blood. Helps retain fluid in the vascular
. Under normal conditions albumin cannot pass through the compartment.
wall of a capillary. What significance is this for the vascular
compartment?
86 Albumin is important for the maintenance of fluid balance Liver
. within the vascular system. Albumin is produced by which
of the following?
87 All the following items are related to cancer. Which does Diagnostic tests for cancer
. not affect the immune system?
88. Altruism According to Yalom (2005), there are 11 primary
factors through which therapeutic changes occur
in group psychotherapy. Which of the following
factors correlates with learning to give to others?
89. The amount of air inspired and expired with each tidal volume.
breath is called:
90. The anatomy and physiology instructor is explaining a T-cell lymphocytes survey proteins in the body
cell-mediated response to the pre-nursing students. and attack the invading antigens.
What actions would the instructor explain occur in a
cell-mediated response?
91. Anesthesia A state of narcosis, analgesia, relazation and loss
of reflexes.
92. Anesthesiologist and anesthetist roles? - Re assess patient's physical condition
immediately prior to initiating anesthesia & make
- Monitor? Vital signs? (6) sure patient is breathing well!

- Monitor
1) Blood pressure, pulse, respiration ,
electrocardiagram (ECG)
2) Blood oxygen saturation
3) tidal volume
4) Blood and gas level
5) Alveolar gas concentrations
6) Body temperature.
93. Anesthetic agent? A substance such as chemical or gas, used to
unduce anesthesia.
94. Anesthetic agent is administered through (IOI) 1) Intranasal intubation
2) Oral intubation
3) laryngeal mask airway.
95. Aneurysm rebleeding occurs most frequently during First 2 weeks
which timeframe after the initial hemorrhage?
96. Anoxia? . an abnormally low amount of oxygen in the
body tissues
97. Antimanic Lithium was one of the first psychotropic drugs
developed. Lithium is in which of the following
medication classifications?
98. As Americans live longer, relatively rare conditions Weight Gain & Increased breathing effort - Early
are becoming more commonplace - one of which is signs of hypervolemia are elevated BP, increased
hypervolemia. What are early signs of hypervolemia? breathing effort, etc. Eventually, fluid congestion
Choose all correct options. in the lungs leads to moist breath sounds. An
earliest symptom of hypovolemia is thirst.
99. As a nurse practicing within a pediatric medicine Reduce frequency and severity of ear infections
group, you take your role quite seriously in preserving Correct
children's hearing and preventing hearing loss in your Explanation:
clients. What can you do to maintain hearing within Nurses play a pivotal role in preventing hearing
your client base? loss by reducing the severity and frequency of ear
infections among children and advocating for
a) Prevent fevers measures that reduce exposure to loud noise.
b) Increase antibiotic therapy use
c) Reduce frequency and severity of ear infections
d) Distribute earplugs to all clients
100 As a nursing instructor, you realize the importance of T-cell lymphocytes
. your students understanding the role of the immune B-cell lymphocytes
system and its role to protect and defend the body
from potential harm. What type of cells are the
primary participants in immune response? Choose all
correct options.
101 As a nursing instructor, you realize the importance Infectious cells
. of your students understanding the role of the Foreign cells
immune system and its role to protect and defend Cancerous cells
the body from potential harm. What type of cells
are the primary targets of the healthy immune
system? Choose all correct options
102 As a nursing instructor, you realize the importance Foreign cells
. of your students understanding the role of the Cancerous cells
immune system and its role to protect and defend Infectious cells
the body from potential harm. What type of cells
are the primary targets of the healthy immune
system? Choose all correct options.
103 As a patient advocate, the nurse role is to? (4) - Support coping strategies
. - Monitor factors that can cause injury such as
patient's position
- Equipment malfunction
- Environmental hazards.
104 "Asking for help from those who care about us isn't When a client shares that "I will solve my own
. a sign of weakness." problems without asking my family for help," the
nurse reacts most therapeutically when responding.
105 As part of the process of checking the placement of 4
. a nasogastric tube, the nurse checks the pH of the
aspirate. Which pH finding would indicate to the
nurse that the tube is in the stomach?
106 Assertive Community Model Service delivery model that provides comprehensive,
. locally based treatment to people with SPMI.
107 Assisting the client with deep-breathing exercises Deep-breathing exercises are beneficial to promoting
. rest as they help the client to relax. The client's door
should be closed to reduce noise and distractions.
Tea contains caffeine, which acts as a stimulant.
While sedatives may be used occasionally for
assistance with rest, regular use isn't advised because
dependence may develop.
108 At 11 p.m., a client is admitted to the emergency d) albuterol (Proventil).
. department. He has a respiratory rate of 44
breaths/min. He's anxious, and wheezes are audible.
The client is immediately given oxygen by face
mask and methylprednisolone (Depo-medrol) I.V.
At 11:30 p.m., the client's arterial blood oxygen
saturation is 86%, and he's still wheezing. The
nurse should plan to administer:

a) alprazolam (Xanax).
b) propranolol (Inderal).
c) morphine.
d) albuterol (Proventil).
109 At a previous visit, the parents of an infant with Fatty stools
. cystic fibrosis received instruction in the Explanation: Pancreatic enzymes normally aid in
administration of pancrelipase (Pancrease). At a food digestion in the intestine. In a child with cystic
follow-up visit, which finding suggests the need for fibrosis, however, these natural enzymes can't reach
further teaching? the intestine because mucus blocks the pancreatic
duct. Without these enzymes, undigested fats and
proteins produce fatty stools. Treatment with
pancreatic enzymes should result in stools of normal
consistency; noncompliance with the treatment
produces fatty stools. Noncompliance doesn't cause
bloody urine, bloody stools, or glucose in urine.
110. Ataxia is the term that refers to Uncoordinated muscle movement
111. Atrial rate of 300 to 400 Electrocardiogram (ECG) characteristics of atrial
fibrillation include which of the following?
112. At what point does the preoperative period end? When the client is transferred onto the operating
table
113. Atypical antipsychotics work on dopamine-receptor The difference between traditional and atypical
and serotonin-receptor blockade, whereas traditional antipsychotics is that:
antipsychotics work on dopamine-receptor blockade.
114. Austin Holbritter, a six-month-old male, and his elder Eustachian tubes
brother Matthew, a three-year-old male, are being
seen in the pediatric clinic where you practice
nursing. They are being seen by the physician for
their third middle ear infection of this winter season.
The mother reports they develop an upper
respiratory infection and an ear infection seems
quick to follow. What contributes to this event?
115. Avoid sources of electrical interference. Which of the following postimplantation
instructions must a nurse provide a patient with a
permanent pacemaker?
116. Avoid tub baths, but shower as desired. The nurse is providing discharge education for the
client going home after a cardiac catheterization.
Which of the following would be important
information to give this client?
117. Axis II An assigned client's diagnosis is paranoid
personality disorder. Which axis, according to the ,
would the diagnosis be classified?
118. A B-cell deficiency, such as CVID, is a disorder There is a disappearance of germinal centers from
characterized by the following. Choose all that apply. lymphatic tissue.
There is complete lack of antibody production.
Disease onset occurs most often in the second
decade of life.
119. Because of difficulties with hemodialysis, peritoneal WBC count
dialysis is initiated to treat a client's uremia. Which
finding during this procedure signals a significant
problem?
120 Because uteroplacental circulation is compromised in Fetal well-being
. clients with preeclampsia, a nonstress test (NST) is Explanation:
performed to detect which condition? An NST is based on the theory that a healthy fetus
has transient fetal heart rate accelerations with fetal
movement. A fetus with compromised
uteroplacental circulation usually won't have these
accelerations, which indicate a nonreactive NST.
An NST can't detect anemia in a fetus. Serial
ultrasounds will detect IUGR and oligohydramnios
in a fetus.
121 Before administering ephedrine, the nurse assesses d) elderly clients.
. the client's history. Because of ephedrine's central
nervous system (CNS) effects, it is not recommended
for:

a) clients with an acute asthma attack.


b) clients with narcolepsy.
c) clients under age 6.
d) elderly clients.
122 Before a transesophageal echocardiogram, a nurse withhold food and fluids.
. gives a client an oral topical anesthetic spray. When Explanation:
the client returns from the procedure, the nurse Following a transesophageal echocardiogram in
observes that he has no active gag reflex. In response, which the client's throat has been anesthetized, the
the nurse should: nurse should withhold food and fluid until the
client's gag reflex returns. There's no indication
that oral airway placement would be appropriate.
The client should be in the upright position, and
the nurse needn't insert an NG tube.
123 Before seeing a newly assigned client with Extreme Anxiety - Extreme anxiety may lead to
. respiratory alkalosis, a nurse quickly reviews the respiratory alkalosis by causing hyperventilation,
client's medical history. Which condition is a which results in excessive carbon dioxide (CO2) loss.
predisposing factor for respiratory alkalosis? Other conditions that may set the stage for respiratory
alkalosis include fever, heart failure, injury to the
brain's respiratory center, overventilation with a
mechanical ventilator, pulmonary embolism, and early
salicylate intoxication. Type 1 diabetes may lead to
diabetic ketoacidosis; the deep, rapid respirations
occurring in this disorder (Kussmaul's respirations)
don't cause excessive CO2 loss. Myasthenia gravis and
opioid overdose suppress the respiratory drive, causing
CO2 retention, not CO2 loss; this may lead to
respiratory acidosis, not alkalosis.
124 Before weaning a client from a ventilator, which b) Baseline arterial blood gas (ABG) levels
. assessment parameter is most important for the
nurse to review?

a) Fluid intake for the past 24 hours


b) Baseline arterial blood gas (ABG) levels
c) Prior outcomes of weaning
d) Electrocardiogram (ECG) results
125 Below which serum sodium level may convulsions 135 mEq/L - Normal serum concentration level ranges
. or coma can occur? from 135 to 145 mEq/L. When the level dips below
135 mEq/L, there is hyponatremia. Manifestations of
hyponatremia include mental confusion, muscular
weakness, anorexia, restlessness, elevated body
temperature, tachycardia, nausea, vomiting, and
personality changes. Convulsions or coma can occur if
the deficit is severe. Values of 140, 142, and 145
mEq/L are within the normal range.
126 Bence Jones protein Immunoglobulin (protein) fragment found in the urine
. of patients with multiple myeloma
Presence of Bence Jones protein in the urine almost
always confirms multiple myeloma, but absence
doesn't rule it out.
127 Bibasilar crackles A client is returning from the operating room after
. inguinal hernia repair. The nurse notes that he has fluid
volume excess from the operation and is at risk for
left-sided heart failure. Which sign or symptom
indicates left-sided heart failure?
128 A black client with asthma seeks emergency care mucous membranes.
. for acute respiratory distress. Because of this
client's dark skin, the nurse should assess for
cyanosis by inspecting the:
129 Bladder retraining following removal of an instructing the patient to follow a 2 to 3 hour timed
. indwelling catheter begins with voiding schedule.
130 The body has several mechanisms to fight disease, Interferons
. one of which is sending chemical messengers.
Specifically, the messengers released by
lymphocytes, monocytes, and macrophages have
differing roles in the immune response. Which
messenger enables cells to resist viral replication
and slow viral replication?
131 Bone marrow is soft tissue within specific bones Yellow Bone Marrow
. which manufactures blood cells; that is, maintains
a role in hematopoiesis. There are several
structures in the hematopoietic and lymphatic
systems that contribute to the manufacture of
blood cells. Which of the following structures does
not participate in blood cell production?
132 Both the liver and the spleen have a role The spleen removes erythrocytes after 120 days, and the liver
. in erythrocyte metabolism. How would removes severely damaged erythrocytes.
this role best be described?
133 The brain is a complex structure and is The cerebellum
. divided into three parts: the cerebrum,
the cerebellum, and the brain stem. The
cerebrum is divided into two hemispheres
and is further divided into four lobes per
hemisphere. Which section of the brain
controls and coordinates muscle
movements?
134 The calcium level of the blood is Parathyroid hormone (PTH) - The serum calcium level is
. regulated by which mechanism? controlled by PTH and calcitonin. The thyroid hormone,
adrenal gland, or androgens do not regulate the calcium level in
the blood.
135 Cancer of the esophagus is most often Esophagogastroduodenoscopy (EGD) with biopsy and
. diagnosed by which of the following? brushings
136 candidiasis yeast infection; infections occurring on the skin or mucous
. membranes in the warm, moist areas such as the vagina or the
mouth
137 Cardiac complications, which may occur Vagus
. following resection of an esophageal
tumor, are associated with irritation of
which nerve at the time of surgery?
138 The cardinal signs of diabetes insipidus "My infant's fluid intake will be restricted."
. are polyuria and polydipsia. Explanation: The simplest test used to diagnose diabetes
Hypernatremia, not hyponatremia, insipidus is restriction of oral fluids and observation of
occurs with diabetes insipidus. Jaundice consequent changes in urine volume and concentration. A
occurs because of abnormal bilirubin weight loss of 3% to 5% indicates severe dehydration, and the
metabolism, not diabetes insipidus. test should be terminated at this point. This test is done in the
Hyperchloremia, not hypochloremia, hospital, and the infant is watched closely.
occurs with diabetes insipidus.
139 Carry a card identifying yourself as a A client has had a pacemaker inserted and is ready for
. pacemaker recipient. discharge. The nurse is providing education about pacemaker
Avoid large magnetic fields. safety. Which of the following are items that the nurse will be
sure to address? Choose all that apply.
140 Celiac sprue is an example of which Mucosal disorders causing generalized malabsorption
. category of malabsorption?
141 Central venous pressure is measured in Right atrium
. which of the following heart chambers? Explanation:
The pressure in the right atrium is used to assess right
ventricular function and venous blood return to the heart. The
left atrium receives oxygenated blood from the pulmonary
circulation. The left ventricle receives oxygenated blood from
the left atrium. The right ventricle is not the central collecting
chamber of venous circulation.
142 The charge nurse overhears a nurse "It's important for the child to have someone assigned to him
. complaining that she has been assigned to who's familiar with his care."
a toddler diagnosed with tetralogy of Explanation: A toddler with tetralogy of Fallot requires the care
Fallot for the past 3 days and the mother of someone who's familiar with the toddler's condition.
is very demanding. Which response by Providing continuity of care enhances safety and promotes
the charge nurse is best? well- being for this toddler and his parents. Options 2 and 3 are
condescending to the nurse and don't help enhance the nurse's
understanding of the situation. Both responses would further
increase the nurse's anger. Changing the assignment isn't in the
best interest of the toddler.
143 Chest pain, weight gain, fatigue. The nurse is taking a health history from a client admitted with
. the medical diagnosis of cardiovascular disease (CVD).
Identify which of the following symptoms indicate CVD.
144 A child, age 14, is hospitalized for nutritional Providing small, frequent meals
. management and drug therapy after experiencing Explanation: Clients with ulcerative colitis, also
an acute episode of ulcerative colitis. Which known as inflammatory bowel syndrome, tolerate
nursing intervention would be appropriate? small, frequent meals better than a few large meals
daily. Eating large amounts of food may exacerbate the
abdominal distention, cramps, and nausea typically
caused by ulcerative colitis. Frequent meals also
provide the additional calories needed to restore
nutritional balance. This client doesn't lack digestive
enzymes and therefore doesn't need enzyme
supplementation. Antibiotics are contraindicated
because they may interfere with the actions of other
prescribed drugs and because ulcerative colitis isn't
caused by bacteria. High-fiber foods may irritate the
bowel further.
145 A child diagnosed with insulin-dependent 2 p.m. and 5 p.m.
. diabetes mellitus is attending a camp for diabetic Explanation: The action of an intermediate-acting
children. He gives himself regular insulin and insulin such as insulin zinc suspension (Humulin L)
insulin zinc suspension (Humulin L) at 8 a.m. The begins 1 to 3 hours after injection and peaks 6 to 12
nurse should plan to observe him for signs and hours after injection. The child is most at risk for
symptoms of hypoglycemia resulting from the hypoglycemia at the times when the insulin would
effects of the Humulin L insulin between which of peak, in this case beginning between 2 p.m. and 5 p.m.
the following times?
146 A child has just been diagnosed with a primary "Although AIDS is an immune deficiency, your child's
. immune deficiency. The parents state, "Oh, no. condition is different from AIDS."
Our child has AIDS." Which response by the
nurse would be most appropriate?
147 Choice Multiple question - Select all answer Turning the head
. choices that apply. Asking for words to be repeated
Instructing a class of six graders on the Explanation:
importance of protecting their hearing by The nurse observes for signs of hearing impairment
avoiding excessive noise, you list the activities that such as frequently asking that words be repeated.
can destroy hearing. On your list is loud concerts,
loud mP3 player volume, loud headphones, etc.
You also indicate the signs of hearing impairment
so they can help protect their friends. Which of
the following are signs of diminished hearing?
Choose all correct responses.

a) Clear speech
b) Leaning back during conversation
c) Turning the head
d) Asking for words to be repeated
148 Choice Multiple question - Select all answer Turning the head
. choices that apply. Asking for words to be repeated
Instructing a class of six graders on the Correct
importance of protecting their hearing by Explanation:
avoiding excessive noise, you list the activities that The nurse observes for signs of hearing impairment
can destroy hearing. On your list is loud concerts, such as frequently asking that words be repeated.
loud mP3 player volume, loud headphones, etc.
You also indicate the signs of hearing impairment
so they can help protect their friends. Which of
the following are signs of diminished hearing?
Choose all correct responses.

a) Turning the head


b) Leaning back during conversation
c) Clear speech
d) Asking for words to be repeated
149 Chronic illnesses may contribute to Deficiency in circulating lymphocytes
. immune system impairment in various
ways. Renal failure is associated with which
of the following?
150 circadian rhythm the biological clock; regular bodily rhythms that occur on a
. 24-hour cycle
151 Circulating nurse (circulator) Registered nurse who coordinates and documents patient care
. in the operating room.
152 A client, 2 months pregnant, has The client will exhibit uterine growth within the expected
. hyperemesis gravidarum. Which expected norms for gestational age.
outcome is most appropriate for her? Explanation:
For a client with hyperemesis gravidarum, the goal of nursing
care is to achieve optimal fetal growth, which can be
evaluated by monitoring uterine growth through fundal
height assessment. The nurse shouldn't assume that excessive
vomiting signifies that the client doesn't accept the
pregnancy. Clients with hyperemesis gravidarum rarely gain
weight according to the expected pattern. They may be
hospitalized briefly to regulate fluid and electrolyte status,
but they don't require hospitalization for the duration of
pregnancy. In fact, hospitalization may add to the stress of
pregnancy by causing family separation and financial
concerns.
153 A client, 11 weeks pregnant, is admitted to an unknown cause.
. the facility with hyperemesis gravidarum. Explanation:
She tells the nurse she has never known The cause of hyperemesis gravidarum isn't known. However,
anyone who had such severe morning etiologic theories implicate hormonal alterations and allergic
sickness. The nurse understands that or psychosomatic conditions. No evidence suggests that
hyperemesis gravidarum results from: hyperemesis gravidarum results from a neurologic disorder,
inadequate nutrition, or hemolysis of fetal RBCs.
154 A client abruptly sits up in bed, reports b) Nonrebreather mask
. having difficulty breathing and has an
arterial oxygen saturation of 88%. Which
mode of oxygen delivery would most likely
reverse the manifestations?

a) Simple mask
b) Nonrebreather mask
c) Face tent
d) Nasal cannula
155 A client admitted for treatment of a gastric "Continue to take antacids even if your symptoms subside."
. ulcer is being prepared for discharge on
antacid therapy. Discharge teaching should
include which instruction?
156 A client admitted to the facility for treatment c) "I will stay in isolation for at least 6 weeks."
. for tuberculosis receives instructions about
the disease. Which statement made by the
client indicates the need for further
instruction?

a) "I will have to take the medication for up


to a year."
b) "This disease may come back later if I am
under stress."
c) "I will stay in isolation for at least 6
weeks."
d) "I will always have a positive test for
tuberculosis."
157 A client admitted to the hospital for an Ask the client for a copy of the advance directive to place
. abdominal aneurysm repair tells a nurse that on his chart.
he has an advance directive. What action Explanation: Upon admission, a client should be asked if
should the nurse take? he has an advance directive and informed of his right to
create one. If the client has an advance directive, a copy of
the document must be placed in the medical record. It isn't
enough just to note that the client has one. It would be
incorrect to tell the client to give the directive to his lawyer
or to imply that the directive isn't valid when surgery is
being performed.
158 A client admitted with acute anxiety has the respiratory alkalosis. - This client's above-normal pH value
. following arterial blood gas (ABG) values: indicates alkalosis. The below-normal PaCO2 value
pH, 7.55; partial pressure of arterial oxygen indicates acid loss via hyperventilation; this type of acid
(PaO2), 90 mm Hg; partial pressure of loss occurs only in respiratory alkalosis. These ABG values
arterial carbon dioxide (PaCO2), 27 mm Hg; wouldn't occur in metabolic acidosis, respiratory acidosis,
and bicarbonate (HCO3-), 24 mEq/L. Based or metabolic alkalosis.
on these values, the nurse suspects:
159 A client admitted with acute anxiety has the This client's above-normal pH value indicates alkalosis.
. following arterial blood gas (ABG) values: The below-normal PaCO2 value indicates acid...(more)
pH, 7.55; partial pressure of arterial oxygen This client's above-normal pH value indicates alkalosis.
(PaO2), 90 mm Hg; partial pressure of The below-normal PaCO2 value indicates acid loss via
arterial carbon dioxide (PaCO2), 27 mm Hg; hyperventilation; this type of acid loss occurs only in
and bicarbonate (HCO3-), 24 mEq/L. Based respiratory alkalosis. These ABG values wouldn't occur in
on these values, the nurse suspects: metabolic acidosis, respiratory acidosis, or metabolic
alkalosis.
160 A client admitted with acute anxiety has the Respiratory Alkalosis - This client's above-normal pH
. following arterial blood gas (ABG) values: value indicates alkalosis. The below-normal PaCO2 value
pH, 7.55; partial pressure of arterial oxygen indicates acid loss via hyperventilation; this type of acid
(PaO2), 90 mm Hg; partial pressure of loss occurs only in respiratory alkalosis. These ABG values
arterial carbon dioxide (PaCO2), 27 mm Hg; wouldn't occur in metabolic acidosis, respiratory acidosis,
and bicarbonate (HCO3-), 24 mEq/L. Based or metabolic alkalosis.
on these values, the nurse suspects:
161 A client admitted with acute anxiety has the d) respiratory alkalosis.
. following arterial blood gas (ABG) values:
pH, 7.55; partial pressure of arterial oxygen
(PaO2), 90 mm Hg; partial pressure of
arterial carbon dioxide (PaCO2), 27 mm Hg;
and bicarbonate (HCO3-), 24 mEq/L. Based
on these values, the nurse suspects:

a) metabolic acidosis.
b) metabolic alkalosis.
c) respiratory acidosis.
d) respiratory alkalosis.
162 A client admitted with a gunshot wound to Urine output of 250 ml/24 hours
. the abdomen is transferred to the intensive
care unit after an exploratory laparotomy.
I.V. fluid is being infused at 150 ml/hour.
Which assessment finding suggests that the
client is experiencing acute renal failure
(ARF)?
163 A client admitted with multiple traumatic injuries c) adult respiratory distress syndrome (ARDS).
. receives massive fluid resuscitation. Later, the
physician suspects that the client has aspirated
stomach contents. The nurse knows that this client
is at highest risk for:

a) chronic obstructive pulmonary disease


(COPD).
b) bronchial asthma.
c) adult respiratory distress syndrome (ARDS).
d) renal failure.
164 A client and spouse are visiting the clinic. The Hepatic encephalopathy
. client recently experienced a seizure and says she
has been having difficulty writing. Before the
seizure, the client says that for several weeks she
was sleeping late into the day but having
restlessness and insomnia at night. The client's
husband says that he has noticed the client has
been moody and slightly confused. Which of the
following problems is most consistent with the
client's clinical manifestations?
165 A client appears to be breathing faster than Count the rate of respirations.
. during the last assessment. Which of the following
interventions should the nurse perform?
166 A client approaches the nurse and points at the "It seems like the world is pretty scary for you, but
. sky, showing her where the men would be coming you're safe here."
from to get him. Which response is most Explanation:
therapeutic? Explaining that the world is scary but the client is safe
acknowledges the client's fears and feelings, and
offers a sense of security as the nurse tries to
understand the symbolism. She reflects these concerns
to the client, along with reassurance of safety. The first
response validates the delusion, not the feelings and
fears, and doesn't orient the client to reality. The
second response gives false reassurance; because the
nurse isn't sure of the symbolism, she can't make this
promise. The last response rejects the client's feelings
and doesn't address his fears.
167 A client arrives in the emergency department Crackles
. reporting shortness of breath. She has 3+ pitting
edema below the knees, a respiratory rate of 36
breaths per minute, and heaving respirations. The
nurse auscultates the client's lungs to reveal
coarse, moist, high-pitched, and non-continuous
sounds that do not clear with coughing. The nurse
will document these sounds as which type?
168 The client asks the nurse how the spinal "The anesthesiologist will inject the anesthetic into the
. anesthesia will be administered. The best response space around your lower spinal cord."
by the nurse is:
169 A client at 33 weeks' gestation is admitted in The neonate will be delivered with mature lungs.
. preterm labor. She is given betamethasone Explanation: Betamethasone is a corticosteroid that
(Celestone) 12 mg I.M. q 24 hours 2. What is the induces the production of surfactant. The pulmonary
expected outcome of this drug therapy? maturation that results causes the fetal lungs to mature
more rapidly than normal. Because the lungs are
mature, the risk of respiratory distress in the neonate is
lowered but not eliminated. Betamethasone also
decreases the surface tension within the alveoli.
Betamethasone has no influence on contractions or
carrying the fetus to full term. It also does not prevent
infection.
170 A client begins clozapine (Clozaril) therapy after Granulocytopenia
. several other antipsychotic agents fail to relieve Correct
her psychotic symptoms. The nurse instructs her Explanation:
to return for weekly white blood cell (WBC) Clozapine can cause life-threatening neutropenia or
counts to monitor for which adverse reaction? granulocytopenia. To detect this adverse reaction, a
WBC count should be performed weekly. Hepatitis,
infection, and systemic dermatitis aren't adverse
reactions of clozapine therapy.
171 A client being treated for pancreatitis faces the Use incentive spirometry every hour.
. risk of atelectasis. Which of the following
interventions would be important to implement to
minimize this risk?
172 A client being treated for pancreatitis faces the Reposition q 2hrs
. risk of atelectasis. Which of the following
interventions would be important to implement to
minimize this risk?
173 A client comes to the dermatology clinic with Vesicles
. numerous skin lesions. Inspection reveals that the
lesions are elevated, sharply defined, less than 1
cm in diameter, and filled with serous fluid. When
documenting these findings, the nurse should use
which term to describe the client's lesions?
174 A client comes to the Emergency Department Urinary caliculi
. complaining of a sudden onset of sharp, severe
flank pain. During the physical examination, the
client indicates that the pain, which comes in
waves, travels to the suprapubic region. He states,
"I can even feel the pain at the tip of my penis."
Which of the following would the nurse suspect?
175 A client comes to the emergency department Acute pain
. complaining of severe pain in the right flank,
nausea, and vomiting. The physician tentatively
diagnoses right ureterolithiasis (renal calculi).
When planning this client's care, the nurse should
assign the highest priority to which nursing
diagnosis?
176 A client comes to the emergency department a) Erythromycin (Erythrocin)
. complaining of sudden onset of diarrhea, anorexia,
malaise, cough, headache, and recurrent chills.
Based on the client's history and physical findings,
the physician suspects Legionnaires' disease. While
awaiting diagnostic test results, the client is
admitted to the facility and started on antibiotic
therapy. What is the drug of choice for treating
Legionnaires' disease?

a) Erythromycin (Erythrocin)
b) Rifampin (Rifadin)
c) Amantadine (Symmetrel)
d) Amphotericin B (Fungizone)
177 A client comes to the emergency department with Respiratory Alkalosis - Respiratory alkalosis results
. status asthmaticus. His respiratory rate is 48 from alveolar hyperventilation. It's marked by a
breaths/minute, and he is wheezing. An arterial decrease in PaCO2 to less than 35 mm Hg and an
blood gas analysis reveals a pH of 7.52, a partial increase in blood pH over 7.45. Metabolic acidosis is
pressure of arterial carbon dioxide (PaCO2) of 30 marked by a decrease in HCO3? to less than 22
mm Hg, PaO2 of 70 mm Hg, and bicarbonate mEq/L, and a decrease in blood pH to less than 7.35.
(HCO3??') of 26 mEq/L. What disorder is In respiratory acidosis, the pH is less than 7.35 and
indicated by these findings? the PaCO2 is greater than 45 mm Hg. In metabolic
alkalosis, the HCO3? is greater than 26 mEq/L and
the pH is greater than 7.45.
178 A client comes to the emergency department with Respiratory alkalosis - Respiratory alkalosis results
. status asthmaticus. His respiratory rate is 48 from alveolar hyperventilation. It's marked by a
breaths/minute, and he is wheezing. An arterial decrease in PaCO2 to less than 35 mm Hg and an
blood gas analysis reveals a pH of 7.52, a partial increase in blood pH over 7.45. Metabolic acidosis is
pressure of arterial carbon dioxide (PaCO2) of 30 marked by a decrease in HCO3? to less than 22
mm Hg, PaO2 of 70 mm Hg, and bicarbonate mEq/L, and a decrease in blood pH to less than 7.35.
(HCO3??') of 26 mEq/L. What disorder is In respiratory acidosis, the pH is less than 7.35 and
indicated by these findings? the PaCO2 is greater than 45 mm Hg. In metabolic
alkalosis, the HCO3? is greater than 26 mEq/L and
the pH is greater than 7.45.
179 A client comes to the physician's office for Apply sunscreen even on overcast days.
. treatment of severe sunburn. The nurse takes this
opportunity to discuss the importance of
protecting the skin from the sun's damaging rays.
Which instruction would best prevent skin
damage?
180 A client complains of abdominal discomfort and Decreasing the rate of feedings and the concentration
. nausea while receiving tube feedings. Which of the formula
intervention is most appropriate for this problem? Explanation: Complaints of abdominal discomfort
and nausea are common in clients receiving tube
feedings. Decreasing the rate of the feeding and the
concentration of the formula should decrease the
client's discomfort. Feedings are normally given at
room temperature to minimize abdominal cramping,
so this intervention should have already been
performed. To prevent aspiration during feeding, the
head of the client's bed should be elevated at least 30
degrees. Changing tube feeding administration sets
every 24 hours prevents bacterial growth; it doesn't
decrease the client's discomfort.
181 A client, confused and short of breath, is brought to b) Arterial blood
. the emergency department by a family member.
The medical history reveals chronic bronchitis and
hypertension. To learn more about the client's
current respiratory problem, the physician orders a
chest X-ray and arterial blood gas (ABG) analysis.
When reviewing the ABG report, the nurse sees
many abbreviations. What does a lowercase "a" in
an ABG value represent?

a) Acid-base balance
b) Arterial blood
c) Arterial oxygen saturation
d) Alveoli
182 A client diagnosed with acute myelocytic leukemia Closely observe the client's skin for petechiae and
. (AML) has been receiving chemotherapy. During bruising.
the last two cycles of chemotherapy, the client Explanation: The nurse should closely observe the
developed severe thrombocytopenia requiring client's skin for petechiae and bruising, early signs of
multiple platelet transfusions. The client is now thrombocytopenia. Daily platelet counts may not
scheduled to receive a third cycle. How can the reflect the client's condition as quickly as subtle
nurse best detect early signs and symptoms of changes in the client's skin. Performing frequent
thrombocytopenia? cardiovascular assessments and checking the client's
history won't help detect early signs and symptoms
of thrombocytopenia.
183 A client had transurethral prostatectomy for benign Check for the presence of clots, and make sure the
. prostatic hypertrophy. He's currently being treated catheter is draining properly
with a continuous bladder irrigation and is Explanation:
complaining of an increase in severity of bladder Blood clots and blocked outflow of the urine can
spasms. What should the nurse do first for this increase spasms. The irrigation shouldn't be stopped
client? as long as the catheter is draining because clots will
form. A belladonna and opium suppository should be
given to relieve spasms but only after assessment of
the drainage. Oral analgesics should be given if the
spasms are unrelieved by the belladonna and opium
suppository.
184 A client has a blockage in the proximal portion of a Anticoagulant
. coronary artery. After learning about treatment
options, the client decides to undergo percutaneous
transluminal coronary angioplasty (PTCA). During
this procedure, the nurse expects to administer an:
185 The client has a chancre on his lips. The nurse Take measures to prevent spreading to others
. instructs the client to
186 A client has a cheesy white plaque in the mouth. Instruct the client to swish prescribed nystatin
. The plaque looks like milk curds and can be rubbed (Mycostatin) solution for 1 minute.
off. The best nursing intervention is to
187 A client has a circular rash on her leg, accompanied by Take the drug on an empty stomach.
. malaise, fever, headache, and joint aches. Laboratory studies
and physical examination findings confirm that she has Lyme
disease. Her physician prescribes tetracycline hydrochloride
(Achromycin), 500 mg by mouth four times per day. Which
instruction should the nurse give the client about self-
administration of tetracycline?
188 A client has a foot ulcer that hasn't shown signs of Peripheral vascular disease
. improvement over the last several months. What medical
condition is most likely causing the wound healing delay?
189 A client has a gastrointestinal tube that enters the stomach Gastrostomy tube
. through a surgically created opening in the abdominal wall.
The nurse documents this as which of the following?
190 A client has a leukocyte count of 3,000/mm3. How would the Leukopenia
. nurse document the client's condition?
191 A client has a nasogastric tube for continuous tube feeding. The Checks the pH of the gastric contents
. nurse does all the following every shift to verify placement Compares exposed tube length with
(select all options that apply): original measurement
Visually assesses the color of the
aspirate
192 A client has a new order for metoclorpramide (Reglan). The GERD
. nurse identifies that this medication can be safely administered
for which conditon?
193 A client has a new order for metoclorpramide (Reglan). The Extrapyramidal
. nurse identifies that this medication should not be used long
term and only in cases where all other options have been
exhausted. This is because this medication has what type of
potential side effect?
194 A client has a new order for metoclorpramide (Reglan). The Uncontrollable movement of the face
. nurse knows that this medication should not be used long term and limbs
and only in cases where all other options have been exhausted.
This is because this medication has the potential for
extrapyramidal side effects. Extrapyramidal side effects
include which of the following?
195 A client has an increased number of eosinophils. Which of the Allergy
. following disorders would the nurse expect the client to have?
196 A client has an increased number of eosinophils. Which of the Allergy
. following disorders would the nurse expect the client to have? Parasitic infection
Select all that apply.
197 A client has a nursing diagnosis of "ineffective airway Lungs are clear on auscultation.
. clearance" as a result of excessive secretions. An appropriate
outcome for this client would be which of the following?
198 A client has a respiratory rate of 38 breaths/min. What effect Increased arterial pH - Respiratory
. does breathing faster have on arterial pH level? alkalosis is always caused by
hyperventilation, which is a decrease in
plasma carbonic acid concentration. The
pH is elevated above normal as a result
of a low PaCO2.
199 A client has a sucking stab wound to the chest. Which action b) Apply a dressing over the wound and
. should the nurse take first? tape it on three sides.

a) Draw blood for a hematocrit and hemoglobin level.


b) Apply a dressing over the wound and tape it on three sides.
c) Prepare a chest tube insertion tray.
d) Prepare to start an I.V. line.
200 A client has been admitted to the hospital with Serving fluids in large amounts
. signs of dehydration. Which action would be Explanation: Fluids should be served in small amounts at
least beneficial in increasing the client's fluid frequent intervals. It's overwhelming to the client to have
intake? large amounts of fluids to drink. Teaching the client about
the need for fluid increase and including him in the
selection of beverages will enhance compliance. Fluids
should be served at the appropriate temperatures to
increase enjoyment and palatability.
201 A client has been admitted to undergo surgical "I'll receive procedural sedation."
. repair of a torn Achilles' tendon. While meeting
with the anesthesiologist, what statement from
the client would indicate to the nurse that the
anesthesiologist needs to clarify points?
202 A client has been diagnosed with AIDS and The client's immune system cannot mount a response to
. tuberculosis (TB). A nursing student asks the the test.
nurse why the client's skin test for TB is
negative if the client's physician has diagnosed
TB. The nurse's correct reply is which of the
following?
203 A client has been diagnosed with AIDS. Which An interference develops in an already developed
. of the following statements correctly describes immune system.
a secondary immune deficiency?
204 A client has been diagnosed with an intestinal Metabolic Alkalosis - Metabolic alkalosis is a clinical
. obstruction and has a nasogastric tube set to disturbance characterized by a high pH and a high plasma
low continuous suction. Which acid-base biacarbonate concentration. The most common cuase of
disturbance is this client at risk for developing? metabolic alkalosis is vomiting or gastric suction with
loss of hydrogen and choloride ions. Gastric fluid has an
acid pH, and loss of this acidic fluid increases the
alkalinity of body fluids.
205 A client has been hospitalized for treatment of a) The client has a partial pressure of arterial oxygen
. acute bacterial pneumonia. Which outcome (PaO2) value of 90 mm Hg or higher.
indicates an improvement in the client's
condition?

a) The client has a partial pressure of arterial


oxygen (PaO2) value of 90 mm Hg or higher.
b) The client has a partial pressure of arterial
carbon dioxide (PaCO2) value of 65 mm Hg or
higher.
c) The client exhibits restlessness and
confusion.
d) The client exhibits bronchial breath sounds
over the affected area.
206 A client has been receiving intermittent tube Blood glucose level
. feedings for several days at home. The nurse
notes the findings as shown in the
accompanying documentation. The nurse
reports the following as an adverse reaction to
the tube feeding:
207 A client has been receiving radiation therapy to Brush and floss daily
. the lungs and now has erythema, edema, and
pain of the mouth. The nurse instructs the
client to
208 A client has hypercholesterolemia. The nurse Early onset of cardiovascular disease
. understands that which of the following reflects Skin xanthoma
the phenotype of the disease? Select all that Family history of heart disease
apply.
209 A client has hypoxemia of pulmonary a) Partial pressure of arterial oxygen (PaO2)
. origin. What portion of arterial blood
gas results is most useful in
distinguishing between acute respiratory
distress syndrome and acute respiratory
failure?

a) Partial pressure of arterial oxygen


(PaO2)
b) Partial pressure of arterial carbon
dioxide (PaCO2)
c) pH
d) Bicarbonate (HCO3-)
210 A client has the following arterial blood Metabolic Acidosis - This client's pH value is below normal,
. gas (ABG) values: pH, 7.12; partial indicating acidosis. The HCO3- value also is below normal,
pressure of arterial carbon dioxide reflecting an overwhelming accumulation of acids or excessive
(PaCO2), 40 mm Hg; and bicarbonate loss of base, which suggests metabolic acidosis. The PaCO2
(HCO3-), 15 mEq/L. These ABG values value is normal, indicating absence of respiratory compensation.
suggest which disorder? These ABG values eliminate respiratory alkalosis, respiratory
acidosis, and metabolic alkalosis.
211. A client has the following arterial blood d) Metabolic acidosis
gas (ABG) values: pH, 7.12; partial
pressure of arterial carbon dioxide
(PaCO2), 40 mm Hg; and bicarbonate
(HCO3-), 15 mEq/L. These ABG values
suggest which disorder?

a) Respiratory alkalosis
b) Respiratory acidosis
c) Metabolic alkalosis
d) Metabolic acidosis
212 The client has the intake and output 260 mL
. shown in the accompanying chart for an
8-hour shift. What is the positive fluid
balance?
213 A client has undergone a left c) Coughing, breathing deeply, frequent repositioning, and using
. hemicolectomy for bowel cancer. Which an incentive spirometer
activities prevent the occurrence of
postoperative pneumonia in this client?

a) Administering oxygen, coughing,


breathing deeply, and maintaining bed
rest
b) Coughing, breathing deeply,
maintaining bed rest, and using an
incentive spirometer
c) Coughing, breathing deeply, frequent
repositioning, and using an incentive
spirometer
d) Administering pain medications,
frequent repositioning, and limiting fluid
intake
214 A client hospitalized for treatment of a Light-headedness or paresthesia - The client with respiratory
. pulmonary embolism develops alkalosis may complain of light-headedness or paresthesia
respiratory alkalosis. Which clinical (numbness and tingling in the arms and legs). Nausea, vomiting,
findings commonly accompany abdominal pain, and diarrhea may accompany respiratory
respiratory alkalosis? acidosis. Hallucinations and tinnitus rarely are associated with
respiratory alkalosis or any other acid-base imbalance.
215 A client hospitalized for treatment of a d) Light-headedness or paresthesia
. pulmonary embolism develops respiratory
alkalosis. Which clinical findings commonly
accompany respiratory alkalosis?

a) Nausea or vomiting
b) Abdominal pain or diarrhea
c) Hallucinations or tinnitus
d) Light-headedness or paresthesia
216 A client in a clinic setting has just been diagnosed To prevent complications/death by achieving and
. with hypertension. She asks what the end goal is maintaining a blood pressure of 140/90 or less
for treatment. The correct reply from the nurse
is which of the following?
217 A client in acute respiratory distress is brought c) They help prevent cardiac arrhythmias.
. to the emergency department. After
endotracheal (ET) intubation and initiation of
mechanical ventilation, the client is transferred
to the intensive care unit. Before suctioning the
ET tube, the nurse hyperventilates and
hyperoxygenates the client. What is the rationale
for these interventions?

a) They help prevent subcutaneous emphysema.


b) They help prevent pneumothorax.
c) They help prevent cardiac arrhythmias.
d) They help prevent pulmonary edema.
218 A client in a late stage of acquired Risk for injury
. immunodeficiency syndrome (AIDS) shows signs
of AIDS-related dementia. Which nursing
diagnosis takes highest priority?
219 A client in end-stage renal disease is prescribed Assesses the hemoglobin level
. epoetin alfa (Epogen) and oral iron supplements.
Before administering the next dose of epoetin
alfa and oral iron supplement, the nurse
220 A client in labor receives epidural anesthesia. Hypotensive crisis
. The nurse should assess carefully for which Explanation:
adverse reaction to the anesthetic agent? Hypotensive crisis may occur after epidural anesthesia
administration as the anesthetic agent spreads through
the spinal canal and blocks sympathetic innervation.
Other signs and symptoms of hypotensive crisis
associated with epidural anesthesia may include fetal
bradycardia (not tachycardia) and decreased (not
increased) beat-to-beat variability in the FHR. Urine
retention, not renal toxicity, may occur during the
postpartum period.
221 A client in the cardiac unit is undergoing Wheezes
. procedures to determine the extent of his left- Wet lung sounds
sided heart failure. As his nurse, what Explanation:
adventitious lung sounds would you expect to If the left side of the heart fails to pump efficiently,
hear during your auscultation of his lungs to blood backs up into the pulmonary veins and lung
support his diagnosis? Choose all correct tissue. With left-sided congestive heart failure,
options. auscultation reveals a crackling sound and gurgles.
222 A client in the emergency department complains of Administer oxygen, attach a cardiac monitor, take
. squeezing substernal pain that radiates to the left vital signs, and administer sublingual
shoulder and jaw. He also complains of nausea, nitroglycerin.
diaphoresis, and shortness of breath. What should the Explanation:
nurse do? Cardiac chest pain is caused by myocardial
ischemia. Therefore the nurse should administer
supplemental oxygen to increase the myocardial
oxygen supply, attach a cardiac monitor to help
detect life-threatening arrhythmias, and take vital
signs to ensure that the client isn't hypotensive
before giving sublingual nitroglycerin for chest
pain. Registration information may be delayed
until the client is stabilized. Alerting the cardiac
catheterization team or the physician before
completing the initial assessment is premature.
223 A client in the emergency department complains of Administer oxygen, attach a cardiac monitor, take
. squeezing substernal pain that radiates to the left vital signs, and administer sublingual
shoulder and jaw. He also complains of nausea, nitroglycerin.
diaphoresis, and shortness of breath. What should the
nurse do?
224 A client in the emergency department is diagnosed c) Measles
. with a communicable disease. When complications of
the disease are discovered, the client is admitted to the
hospital and placed in respiratory isolation. Which
infection warrants respiratory isolation?

a) Chickenpox
b) Impetigo
c) Measles
d) Cholera
225 A client in the emergency department reports that a Assess lung sounds bilaterally
. piece of meat became stuck in the throat while eating.
The nurse notes the client is anxious with respirations
at 30 breaths/min, frequent swallowing, and little
saliva in the mouth. An esophagogastroscopy with
removal of foreign body is scheduled for today. The
first activity of the nurse is to:
226 A client in the emergency department reports that he Metabolic Alkalosis - A pH over 7.45 with a
. has been vomiting excessively for the past 2 days. His HCO3- level over 26 mEq/L indicates metabolic
arterial blood gas analysis shows a pH of 7.50, partial alkalosis. Metabolic alkalosis is always secondary
pressure of arterial carbon dioxide (PaCO2) of 43 mm to an underlying cause and is marked by decreased
Hg, partial pressure of arterial oxygen (PaO2) of 75 amounts of acid or increased amounts of base
mm Hg, and bicarbonate (HCO3-) of 42 mEq/L. HCO3-. The client isn't experiencing respiratory
Based on these findings, the nurse documents that the alkalosis because the PaCO2 is normal. The client
client is experiencing which type of acid-base isn't experiencing respiratory or metabolic
imbalance? acidosis because the pH is greater than 7.35.
227 A client is actively bleeding from esophageal varices. Vasopressin
. Which of the following medications would the nurse
most expect to be administered to this client?
228 A client is actively bleeding from esophageal varices. Vasopressin (Pitressin)
. Which of the following medications would the nurse
most expect to be administered to this client?
229 A client is admitted for suspected GI disease. Cirrhosis
. Assessment data reveal muscle wasting, a decrease in
chest and axillary hair, and increased bleeding
tendency. The nurse suspects the client has:
230 A client is admitted for treatment of chronic water and sodium retention secondary to a severe
. renal failure (CRF). The nurse knows that this decrease in the glomerular filtration rate.
disorder increases the client's risk of: Explanation: A client with CRF is at risk for fluid
imbalance dehydration if the kidneys fail to
concentrate urine, or fluid retention if the kidneys fail to
produce urine. Electrolyte imbalances associated with
this disorder result from the kidneys' inability to excrete
phosphorus; such imbalances may lead to
hyperphosphatemia with reciprocal hypocalcemia. CRF
may cause metabolic acidosis, not metabolic alkalosis,
secondary to the inability of the kidneys to excrete
hydrogen ions.
231 A client is admitted to a healthcare facility with Platelets break down and migrate to the injured area
. minor lacerations on the leg. The nurse caring
for this client observes swelling in the tissues
surrounding the affected area. A blood clot is
suspected. The nurse should know that which of
the following is the first step when the
formation of a blood clot begins?
232 A client is admitted to the emergency a) Rapidly
. department with an acute asthma attack. The
physician prescribes ephedrine sulfate, 25 mg
subcutaneously (S.C.). How soon should the
ephedrine take effect?

a) Rapidly
b) In 3 minutes
c) In 1 hour
d) In 2 hours
233 A client is admitted to the healthcare facility Elevated urine amylase levels
. suspected of having acute pancreatitis and
undergoes laboratory testing. Which of the
following would the nurse expect to find?
234 A client is admitted to the health care facility Relieving abdominal pain
. with abdominal pain, a low-grade fever,
abdominal distention, and weight loss. The
physician diagnoses acute pancreatitis. What is
the primary goal of nursing care for this client?
235 A client is admitted to the hospital for Test for occult blood
. diagnostic testing to rule out colorectal cancer.
Which intervention should the nurse include on
the plan of care?
236 The client is admitted to the hospital with a Cola colored urine
. diagnosis of acute glomerulonephritis. Which
clinical manifestation would the nurse expect to
find?
237 A client is admitted to the hospital with systolic Pulmonary congestion
. left-sided heart failure. The nurse knows to look
for which of the following assessment findings
for this client?
238 A client is admitted to the psychiatric unit with unpredictable behavior and intense interpersonal
. a diagnosis of borderline personality disorder. relationships.
The nurse expects data collection to reveal: Explanation:
A client with borderline personality disorder displays a
pervasive pattern of unpredictable behavior, mood, and
self-image. Interpersonal relationships may be intense and
unstable, and behavior may be inappropriate and
impulsive. Although the client's impaired ability to form
relationships may affect parenting skills, inability to
function as a responsible parent is more typical of
antisocial personality disorder. Somatic symptoms
characterize avoidant personality disorder. Coldness,
detachment, and lack of tender feelings typify schizoid
and schizotypal personality disorders.
239 A client is admitted to the psychiatric unit with a calming effect from which the client is easily aroused.
. a tentative diagnosis of psychosis. Her Explanation:
physician prescribes the phenothiazine Shortly after phenothiazine administration, a quieting and
thioridazine (Mellaril), 50 mg by mouth three calming effect occurs, but the client is easily aroused,
times per day. Phenothiazines differ from alert, and responsive and has good motor coordination.
central nervous system (CNS) depressants in
their sedative effects by producing:
240 A client is admitted with nausea, vomiting, and Start I.V. fluids with a normal saline solution bolus
. diarrhea. His blood pressure on admission is followed by a maintenance dose.
74/30 mm Hg. The client is oliguric and his
blood urea nitrogen (BUN) and creatinine
levels are elevated. The physician will most
likely write an order for which treatment?
241 A client is beginning highly active 6 weeks
. antiretroviral therapy (HAART). The client
demonstrates an understanding of the need for
follow up when he schedules a return visit for
viral load testing at which time?
242 A client is being prepared to undergo Liver biopsy
. laboratory and diagnostic testing to confirm
the diagnosis of cirrhosis. Which test would the
nurse expect to be used to provide definitive
confirmation of the disorder?
243 A client is being treated for diverticulosis. Drink at least 8 to 10 large glasses of fluid every day.
. Which of the following points should the nurse Do not suppress the urge to defecate.
include in this client's teaching plan? Choose
all correct options.
244 A client is chronically short of breath and yet a possible hematologic problem.
. has normal lung ventilation, clear lungs, and
an arterial oxygen saturation SaO2 of 96% or
better. The client most likely has:
245 A client is diagnosed as having serous otitis media. You have some fluid that has collected in your
. When describing this condition to the client, which middle ear but no infection."
of the following would be most accurate? Correct
Explanation:
a) "You have a common infection in one of the bones Serous otitis media invovles fluid, without evidence
of your face." of active infection, in the middle ear. Recurrent
b) "You have some fluid that has collected in your episodes of acute otitis media leads to chronic otitis
middle ear but no infection." media. An infection of the temporal bone (temporal
c) "Your eardrum has ruptured because of the bone osteomyelitis) is a serious but rare external ear
extreme pressure in your middle ear from the infection called malignant external otitis. Rupturing
infection." of the eardrum refers to tympanic membrane
d) "It has resulted from the several recurrent perforation.
episodes of acute otitis media you've had."
246 A client is diagnosed with a brain tumor. The nurse's Occipital
. assessment reveals that the client has difficulty
interpreting visual stimuli. Based on these findings,
the nurse suspects injury to which lobe of the brain?
247 A client is diagnosed with a hiatal hernia. Which "I'll eat frequent, small, bland meals that are high in
. statement indicates effective client teaching about fiber."
hiatal hernia and its treatment?
248 A client is diagnosed with common variable Pneumocystis jiroveci pneumonia
. immunodeficiency (CVID). When assessing the
client for possible infection, which of the following
would the nurse identify as a least likely cause?
249 A client is diagnosed with common variable Staphylococcus aureus
. immunodeficiency (CVID). Which of the following Streptococcus pneumoniae
would the nurse identify as potential infections for Haemophilus influenzae
this client? Choose all that apply.
250 A client is diagnosed with deep vein thrombosis. Ineffective peripheral tissue perfusion related to
. Which nursing diagnosis should receive highest venous congestion
priority at this time? Explanation: Ineffective peripheral tissue perfusion
related to venous congestion takes highest priority
because venous inflammation and clot formation
impede blood flow in a client with deep vein
thrombosis. Option 1 is incorrect because impaired
gas exchange is related to decreased, not increased,
blood flow. Option 2 is inappropriate because no
evidence suggests that this client has a excessive
fluid volume. Option 3 may be warranted but is
secondary to ineffective tissue perfusion.
251 A client is diagnosed with genital herpes simplex. 3 to 7 days
. Concerned about spread of the virus to others, the
nurse questions the client about recent sexual
activity. What is the average incubation period for
localized genital herpes simplex infection?
252 A client is diagnosed with gonorrhea. When teaching Wash your hands thoroughly to avoid transferring
. the client about this disease, the nurse should the infection to your eyes.
include which instruction?
253 A client is diagnosed with herpes simplex. Which During early pregnancy, herpes simplex infection
. statement about herpes simplex infection is true? may cause spontaneous abortion or premature
delivery.
254 A client is diagnosed with human immunodeficiency Lymphocyte
. virus (HIV). After recovering from the initial shock
of the diagnosis, the client expresses a desire to learn
as much as possible about HIV and acquired
immunodeficiency syndrome (AIDS). When
teaching the client about the immune system, the
nurse states that humoral immunity is provided by
which type of white blood cell?
255 A client is diagnosed with megaloblastic anemia Intrinsic factor
. caused by vitamin B12 deficiency. The physician
begins the client on cyanocobalamin (Betalin-
12), 100 mcg I.M. daily. Which substance
influences vitamin B12 absorption?
256 A client is diagnosed with pericarditis. The nurse Fever, chest discomfort, and elevated erythrocyte
. anticipates that the client may exhibit which sedimentation rate (ESR)
signs and symptoms? Explanation:
The classic signs and symptoms of pericarditis include
fever, positional chest discomfort, nonspecific ST-
segment elevation, elevated ESR, and pericardial
friction rub. All other symptoms may result from acute
renal failure.
257 A client is diagnosed with primary herpes Apply acyclovir ointment to the lesions every 3 hours,
. genitalis. Which instruction should the nurse six times per day for 7 days.
provide?
258 A client is diagnosed with severe combined Preparation for bone marrow transplantation
. immunodficiency (SCID). Which of the following
would the nurse expect to integrate into the
client's plan of care?
259 A client is diagnosed with syndrome of Cerebral Edema - Noncompliance with treatment for
. inappropriate antidiuretic hormone (SIADH). SIADH may lead to water intoxication from fluid
The nurse informs the client that the physician retention caused by excessive antidiuretic hormone.
will order diuretic therapy and restrict fluid and This, in turn, limits water excretion and increases the
sodium intake to treat the disorder. If the client risk for cerebral edema. Hypovolemic shock results
doesn't comply with the recommended from, severe deficient fluid volume; in contrast, SIADH
treatment, which complication may arise? causes excess fluid volume. The major electrolyte
disturbance in SIADH is dilutional hyponatremia, not
hyperkalemia. Because SIADH doesn't alter renal
function, potassium excretion remains normal;
therefore, severe hyperkalemia doesn't occur. Tetany
results from hypocalcemia, an electrolyte disturbance
not associated with SIADH.
260 A client is diagnosed with syndrome of Serum sodium level of 124 mEq/L - In SIADH, the
. inappropriate antidiuretic hormone (SIADH). posterior pituitary gland produces excess antidiuretic
The nurse should anticipate which laboratory hormone (vasopressin), which decreases water excretion
test result? by the kidneys. This, in turn, reduces the serum sodium
level, causing hyponatremia, as indicated by a serum
sodium level of 124 mEq/L. In SIADH, the serum
creatinine level isn't affected by the client's fluid status
and remains within normal limits. A hematocrit of 52%
and a BUN level of 8.6 mg/dl are elevated. Typically,
the hematocrit and BUN level decrease.
261 A client is evaluated for severe pain in the right Acute pain r/t biliary spasms
. upper abdominal quadrant, which is
accompanied by nausea and vomiting. The
physician diagnoses acute cholecystitis and
cholelithiasis. For this client, which nursing
diagnosis takes top priority?
262 A client is evaluated for severe pain in the right Acute pain related to biliary spasms
. upper abdominal quadrant, which is
accompanied by nausea and vomiting. The
physician diagnoses acute cholecystitis and
cholelithiasis. For this client, which nursing
diagnosis takes top priority?
263 A client is examined and found to have pinpoint, Petechiae
. pink-to-purple, nonblanching macular lesions 1 Explanation: Petechiae are small macular lesions 1 to 3
to 3 mm in diameter. Which term best describes mm in diameter. Ecchymosis is a purple-to-brown
these lesions? bruise, macular or papular, that varies in size. A
hematoma is a collection of blood from ruptured blood
vessels that's more than 1 cm in diameter. Purpura are
purple macular lesions larger than 1 cm.
264 A client is given a diagnosis of hepatic Enlarged liver size
. cirrhosis. The client asks the nurse what Ascites
findings led to this determination. Which of Hemorrhoids
the following clinical manifestations would
the nurse correctly identify? Select all that
apply.
265 A client is given a nursing diagnosis of Client demonstrates beginning participation in events and
. social isolation related to withdrawal of activities.
support systems and stigma associated with Client identifies appropriate sources of assistance and
AIDS. Which outcomes would indicate that support.
the nurse's plan of care was effective? Client verbalizes feelings related to the changes imposed by
Select all that apply. the disease.
266 The client is postoperative following a graft Assess the graft for color and temperature
. reconstruction of the neck. It is most
important for the nurse to
267 A client is prescribed an intravenous dose Ensures that epinephrine is available
. of iron dextran. The nurse
268 A client is prescribed metformin lactic acidosis
. (Glucophage) to control type 2 diabetes.
The nurse should monitor for which life-
threatening adverse reaction?
269 A client is prescribed rifampin (Rifadin), c) It's tuberculocidal, destroying the offending bacteria.
. 600 mg P.O. daily. Which statement about
rifampin is true?

a) It's usually given alone.


b) Its exact mechanism of action is
unknown.
c) It's tuberculocidal, destroying the
offending bacteria.
d) It acts primarily against resting bacteria.
270 A client is prescribed tetracycline to treat "Be sure to wear sunscreen when taking this medication"
. peptic ulcer disease. Which of the following
instructions would the nurse give the
client?
271 A client is progressing through the first Bearing-down reflex
. stage of labor. Which finding signals the Explanation:
beginning of the second stage of labor? The second stage of labor is heralded by a bearing-down
reflex with each contraction, increased bloody show, severe
rectal pressure, and rupture of the membranes (if this hasn't
already occurred). Passage of the mucus plug typically occurs
during the latent phase of the first stage of labor. A change in
uterine shape and a gush of dark blood occur during the
placental separation phase of the third stage of labor.
272 A client is readmitted with an exacerbation "I didn't eat anything I shouldn't have; I just ate roast beef on
. of celiac disease 2 weeks after discharge. rye bread."
Which statement by the client indicates the
need for a dietary consult?
273 A client is receiving an I.V. infusion of Increased urine output
. mannitol (Osmitrol) after undergoing
intracranial surgery to remove a brain
tumor. To determine whether this drug is
producing its therapeutic effect, the nurse
should consider which finding most
significant?
274 A client is receiving conscious sedation while undergoing bronchoscopy. c) Oxygen saturation of 90%
. Which assessment finding should receive the nurse's immediate
attention?

a) Absent cough and gag reflexes


b) Blood-tinged secretions
c) Oxygen saturation of 90%
d) Respiratory rate of 13 breaths/min
275 A client is receiving ganciclovir as part of his treatment for Neutropenia
. cytomegalovirus retinitis. The nurse would monitor the results of the
client's laboratory tests for which of the following?
276 A client is receiving intravenous heparin to prevent blood clots. The 24
. order is for heparin 1,200 units per hour. The pharmacy sends 25,000
units of heparin in 500 mL of D5W. At how many mL per hour will the
nurse infuse this solution? Enter the correct number ONLY.
277 A client is receiving nitroglycerin ointment (Nitro-Dur) to treat angina Blood pressure 84/52 mm Hg
. pectoris. The nurse evaluates the therapeutic effectiveness of this drug
by assessing the client's response and checking for adverse effects. Which
vital sign is most likely to reflect an adverse effect of nitroglycerin?
278 A client is receiving parenteral nutrition (PN) through a peripherally No land line; cell phone
. inserted central catheter (PICC) and will be discharged home with PN. available and taken by family
The home health nurse evaluates the home setting and makes a member during working hours
recommendation when noting the following:
279 A client is receiving supplemental oxygen. When determining the c) Partial pressure of arterial
. effectiveness of oxygen therapy, which arterial blood gas value is most oxygen (PaO2)
important?

a) pH
b) Bicarbonate (HCO3-)
c) Partial pressure of arterial oxygen (PaO2)
d) Partial pressure of arterial carbon dioxide (PaCO2)
280 A client is recovering from a neck dissection. What volume of 80-120 mL
. serosanguineous secretions would the nurse expect to drain over the first
24 hours?
281 A client is recovering from an ileostomy that was performed to treat increasing fluid intake to
. inflammatory bowel disease. During discharge teaching, the nurse prevent dehydration.
should stress the importance of:
282 A client is recovering from coronary artery bypass graft (CABG) Decreased cardiac output
. surgery. Which nursing diagnosis takes highest priority at this time? related to depressed
myocardial function, fluid
volume deficit, or impaired
electrical conduction
283 A client is recovering from gastric surgery. Toward what goal should the Six small meals daily with 120
. nurse progress the client's enteral intake? mL fluid between meals
284 A client is returning from the operating room after inguinal hernia Bibasilar crackles
. repair. The nurse notes that he has fluid volume excess from the
operation and is at risk for left-sided heart failure. Which sign or
symptom indicates left-sided heart failure?
285 A client is scheduled for an esophagogastroduodenoscopy (EGD) to Pain and discomfort tolerance
. detect lesions in the gastrointestinal tract. The nurse would observe for
which of the following while assessing the client during the procedure?
286 A client is scheduled for a prostatectomy, and the anesthesiologist plans To prevent cerebrospinal fluid
. to use a spinal (subarachnoid) block during surgery. In the operating (CSF) leakage
room, the nurse positions the client according to the anesthesiologist's
instructions. Why does the client require special positioning for this type
of anesthesia?
287 A client is scheduled for several diagnostic tests to evaluate her Colonoscopy
. gastrointestinal function. After teaching the client about these tests, the
nurse determines that the client has understood the teaching when she
identifies which test as not requiring the use of a contrast medium?
288 A client is scheduled for the following tests: Oral cholecystogram
. barium enema, small bowel series, enteroclysis
enema, and an oral cholecystogram. Which test
would the nurse expect the client to undergo
first?
289 A client is scheduled to receive a 25% dextrose Ensures availability of an infusion pump
. solution of parenteral nutrition. The nurse does Ensures completion of baseline monitoring of the
all of the following. Select all that apply. complete blood count (CBC) and chemistry panel
Places a 1.5-micron filter on the tubing
290 A client is suspected of having an immune system Enzyme-linked immunosorbent assay
. disorder. The physician wants to perform a
diagnostic test to confirm the diagnosis. What test
might the physician order?
291 A client is taking spironolactone (Aldactone) to electrocardiogram (ECG) results. - Although changes
. control her hypertension. Her serum potassium in all these findings are seen in hyperkalemia, ECG
level is 6 mEq/L. For this client, the nurse's results should take priority because changes can
priority should be to assess her: indicate potentially lethal arrhythmias such as
ventricular fibrillation. It wouldn't be appropriate to
assess the client's neuromuscular function, bowel
sounds, or respiratory rate for effects of hyperkalemia.
292 A client is to have an upper GI procedure with The ultrasonography should be scheduled before the
. barium ingestion and abdominal GI procedure.
ultrasonography. While scheduling these
diagnostic tests, the nurse must consider which
factor?
293 A client is to have an upper GI procedure with Ultrasound before GI procedure
. barium ingestion and abdominal
ultrasonography. While scheduling these
diagnostic tests, the nurse must consider which
factor?
294 The client is to receive a unit of packed red blood Verify that the client has signed a written consent form.
. cells. The first intervention of the nurse is to
295 A client is to receive intravenous immunoglobulin 10:30 am and 11:00 am
. (IVIG). The infusion is started at 10 a.m. The
nurse would be alert for signs and symptoms of
an anaphylactic reaction during which time
frame?
296 A client is undergoing a complete physical d) chest movements
. examination as a requirement for college. When
checking the client's respiratory status, the nurse
observes respiratory excursion to help assess:

a) lung vibrations
b) vocal sounds
c) breath sounds
d) chest movements
297 A client is weak and drowsy after a lumbar Position the client flat for at least 3 hours.
. puncture. The nurse caring for the client knows
that what priority nursing intervention should be
provided after a lumbar puncture?
298 A client, newly admitted to the nursing unit, has a Secondary
. primary diagnosis of renal failure. When
assessing the client, the nurse notes a blood
pressure (BP) of 180/100. The nurse knows that
this is what kind of hypertension?
299 A client newly diagnosed with acute lymphocytic 48 hours
. leukemia has a right subclavian central venous
catheter in place. The nurse who's caring for the
client is teaching a graduate nurse about central
venous catheter care. The nurse should instruct
the graduate nurse to change the central venous
catheter dressing every:
300 A client presented with gastrointestinal bleeding "you will need to swallow a capsule"
. 2 days ago and continues to have problems. The
physician has ordered a visualization of the
small intestine via a capsule endoscopy. Which
of the following will the nurse include in the
client education about this procedure?
301 A client presented with gastrointestinal bleeding "You will need to swallow a capsule."
. 2 days ago and continues to have problems. The
physician has ordered a visualization of the
small intestine via a capsule endoscopy. Which
of the following will the nurse include in the
client education about this procedure?
302 A client presents to the emergency department, Metabolic alkalosis and hypokalemia
. reporting that he has been vomiting every 30 to
40 minutes for the past 8 hours. Frequent
vomiting puts this client at risk for which
imbalances?
303 A client presents with anorexia, nausea and Hypercalcemia - More than 99% of the body's calcium is
. vomiting, deep bone pain, and constipation. The found in the skeletal system. Hypercalcemia (greater
following are the client's laboratory values. than 10.2 mg/dL) can be a dangerous imbalance. The
Na + 130 mEq/L client presents with anorexia, nausea and vomiting,
K + 4.6 mEq/L constipation, abdominal pain, bone pain, and confusion.
Cl - 94 mEq/L
Mg ++ 2.8 mg/dL
Ca ++ 13 mg/dL
Which of the following alterations is consistent
with the client's findings?
304 A client presents with fatigue, nausea, vomiting, Hypokalemia - Potassium is the major intracellular
. muscle weakness, and leg cramps. Laboratory electrolyte. Hypocalemia (below 3.5 mEq/L) usually
values are as follows: indicates a deficit in total potassium stores. Potassium
Na + 147 mEq/L deficiency can result in derangements in physiology.
K + 3.0 mEq/L Clinical signs include fatigue, anorexia, nausea,
Cl - 112 mEq/L vomiting, muscles weakness, leg cramps, decreased
Mg ++ 2.3 mg/dL bowel motility, and paresthesias.
Ca ++ 1.5 mg/dL
Which of the following is consistent with the
client's findings?
305 A client presents with fatigue, nausea, vomiting, Hypokalemia - Potassium is the major intracellular
. muscle weakness, and leg cramps. Laboratory electrolyte. Hypocalemia (below 3.5 mEq/L) usually
values are as follows: indicates a d...(more)
Na + 147 mEq/L Potassium is the major intracellular electrolyte.
K + 3.0 mEq/L Hypocalemia (below 3.5 mEq/L) usually indicates a
Cl - 112 mEq/L deficit in total potassium stores. Potassium deficiency
Mg ++ 2.3 mg/dL can result in derangements in physiology. Clinical signs
Ca ++ 1.5 mg/dL include fatigue, anorexia, nausea, vomiting, muscles
Which of the following is consistent with the weakness, leg cramps, decreased bowel motility, and
client's findings? paresthesias.
306 A client presents with muscle weakness, tremors, Hypomagnesemia - Magnesium, the second most
. slow muscle movements, and vertigo. The abundant intracellular cation, plays a role in both
following are the client's laboratory values: carbohydrate and protein metabolism. The most
Na+ 134 mEq/L common cause of this imbalance is loss in the
K+ 3.2 mEq/L gastrointestinal tract. Hypomagnesemia is a value less
Cl- 111 mEq/L than 1.3 mg/dL. Signs and symptoms include muscle
Mg++ 1.1 mg/dL weakness, tremors, irregular movements, tetany,
Ca++ 8.4 mg/dL vertigo, focal seizures, and positive Chvostek's and
Identify which of the following alterations is Trousseau's signs.
consistent with the client's findings.
307 A client realizes that regular use of laxatives has The client's natural bowel function may become
. greatly improved his bowel pattern. However, the sluggish.
nurse cautions this client against the prolonged
use of laxatives for which reason?
308 A client recovering from a pulmonary embolism is b) vitamin K1 (phytonadione).
. receiving warfarin (Coumadin). To counteract a
warfarin overdose, the nurse would administer:

a) heparin.
b) vitamin K1 (phytonadione).
c) vitamin C.
d) protamine sulfate.
309 A client recovering from gastric bypass surgery Notify the surgeon about the tube's removal
. accidentally removes the nasogastric tube. It is
best for the nurse to
310 A client reports an allergy to morphine sulfate, The body produces inappropriate or exaggerated
. which represents an example of a hypersensitivity responses to specific antigens.
reaction. Which of the following statements
correctly describes the process of
hypersensitivity?
311. A client reports to the clinic, stating that she Acute gastritis
rapidly developed headache, abdominal pain,
nausea, hiccuping, and fatigue about 2 hours ago.
For dinner, she ate buffalo chicken wings and
beer. Which of the following medical conditions is
most consistent with the client's presenting
problems?
312 A client's chest X-ray reveals bilateral white-outs, c) increased pulmonary capillary permeability.
. indicating adult respiratory distress syndrome
(ARDS). This syndrome results from:

a) cardiogenic pulmonary edema.


b) respiratory alkalosis.
c) increased pulmonary capillary permeability.
d) renal failure.
313 A client seeks medical attention for dyspnea, chest Decreased CO
. pain, syncope, fatigue, and palpitations. A
thorough physical examination reveals an apical
systolic thrill and heave, along with a fourth heart
sound (S4) and a systolic murmur. Diagnostic
tests reveal that the client has hypertrophic
cardiomyopathy (HCM). Which nursing diagnosis
may be appropriate?
314 A client sees a dermatologist for a skin problem. Irritation of opposing skin surfaces caused by friction
. Later, the nurse reviews the client's chart and
notes that the chief complaint was intertrigo. This
term refers to which condition?
315 A client states that he has 20/40 vision. Which The client can read from 20' (6 m) what a person
. statement about this client's vision is true? with normal vision can read from 40'.
Explanation: The numerator, which is always 20, is
the distance in feet between the vision chart and the
client. The denominator indicates from what
distance an eye with normal vision can read the
chart. Normal vision is defined as 20/20. The other
options are inaccurate.
316 A client suffers adult respiratory distress syndrome a) Kinking of the ventilator tubing
. as a consequence of shock. The client's condition
deteriorates rapidly, and endotracheal (ET)
intubation and mechanical ventilation are initiated.
When the high-pressure alarm on the mechanical
ventilator sounds, the nurse starts to check for the
cause. Which condition triggers the high-pressure
alarm?

a) Kinking of the ventilator tubing


b) A disconnected ventilator tube
c) An ET cuff leak
d) A change in the oxygen concentration without
resetting the oxygen level alarm
317 A client suspected of having HIV has blood drawn ELISA
. for a screening test. What is the first test generally
run to see if a client is, indeed, HIV positive?
318 Clients with recurrent life-threatening You are caring for a client who has been admitted
. tachydysrhythmias. to have a cardioverter defibrillator implanted. You
would know that implanted cardioverter
defibrillators are used in what clients?
319 Clients with Type O blood are at higher risk for Duodenal ulcers
. which of the following GI disorders?
320 A client tells a nurse that her ileoconduit appliance Consult the wound-ostomy nurse.
. won't adhere to her skin. The nurse inspects the site Explanation:
and notes that the area around the stoma is red, The skin is most likely excoriated from urine
moist, and tender to touch. How should the nurse leaking from the appliance. The nurse should
intervene? consult the wound-ostomy nurse, who can suggest
care interventions. Some facilities require a
physician's order to obtain a wound culture. Patting
the skin dry and applying a new appliance won't
address the problem with the appliance. Applying
skin adhesive spray to excoriated skin will further
irritate the skin and increase the client's discomfort.
321 A client transferred to a long-term care facility has a Wound care nurse
. stage II pressure ulcer on her coccyx. Who should the
nurse consult about the care of this client?
322 A client undergoes a total laryngectomy and b) "Family members should continue to talk to the
. tracheostomy formation. On discharge, the nurse client."
should give which instruction to the client and
family?

a) "Clean the tracheostomy tube with alcohol and


water."
b) "Family members should continue to talk to the
client."
c) "Oral intake of fluids should be limited for 1 week
only."
d) "Limit the amount of protein in the diet."
323 A client undergoes a tracheostomy after d) Suction the client, withdraw residual air from the cuff,
. many failed attempts at weaning him from a and reinflate it.
mechanical ventilator. Two days after
tracheostomy, while the client is being
weaned, the nurse detects a mild air leak in
the tracheostomy tube cuff. What should the
nurse do first?

a) Call the physician.


b) Remove the malfunctioning cuff.
c) Add more air to the cuff.
d) Suction the client, withdraw residual air
from the cuff, and reinflate it.
324 A client undergoes extracorporeal shock "Increase your fluid intake to 2 to 3 L per day."
. wave lithotripsy. Before discharge, the nurse Explanation:
should provide which instruction? Increasing fluid intake flushes the renal calculi fragments
through and prevents obstruction of the urinary
system. Measuring temperature every 4 hours isn't needed.
Lithotripsy doesn't require an incision. Hematuria may
occur for a few hours after lithotripsy but then should
disappear.
325 A client undergoes total gastrectomy. Several Notify the physician
. hours after surgery, the nurse notes that the
client's nasogastric (NG) tube has stopped
draining. How should the nurse respond?
326 A client undergoing a complete blood cell Gives its oxygen to cells of the body and picks up carbon
. (CBC) count for the detection of anemia dioxide
wants to know more about hemoglobin.
Which of the following should the nurse
explain to the client as being the main
function of hemoglobin?
327 A client undergoing a diagnostic examination Permit the client to drink only clear liquids.
. for gastrointestinal disorder was given
polyethylene glycol/electrolyte solution as a
part of the test preparation. Which of the
following measures should the nurse take
once the solution is administered?
328 A client undergoing a diagnostic examination Permit to drink only clear liquids
. for gastrointestinal disorder was given
polyethylene glycol/electrolyte solution as a
part of the test preparation. Which of the
following measures should the nurse take
once the solution is administered?
329 A client understands what resources are Before I go home, I'll speak to the home health care nurse to
. available to help him perform wound care at make sure I have the supplies I need.
home when he states the following:
330 A client understands what resources are "Before I go home, I'll speak to the home health care nurse
. available to help him perform wound care at to make sure I have the supplies I need."
home when he states the following: Explanation: The client's acknowledgement that he will
need to speak to the home care nurse about supplies
demonstrates that he is able to perform self-care, and is
familiar with the resources available. The social worker can
help with financial issues, not wound care issues. The home
health care nurse is available for consultation, but she won't
provide all of the client's health care needs. Dressing
changes don't need to be performed in the physician's office.
331 A client visits the nurse complaining of diarrhea every time they Avoid residue, lactose, fat, and
. eat. The client has AIDS and wants to know what they can do to caffeine.
stop having diarrhea. What should the nurse advise?
332 The client vomits during the surgical procedure. The best action Suction the client to remove saliva
. by the nurse is: and gastric secretions.
333 A client who can't tolerate oral feedings begins receiving diaphoresis, vomiting, and diarrhea.
. intermittent enteral feedings. When monitoring for evidence of
intolerance to these feedings, the nurse must remain alert for:
334 A client who had abdominal surgery 4 days ago reports that Applying a sterile, moist dressing
. "something gave way" when he sneezed. The nurse observes a
wound evisceration. Which nursing action is the first priority?
335 A client who has just been diagnosed with myocardial infarction "You sound as though you think
. (MI) begins to cry and tells the nurse that his brother died of a you're going to die."
heart attack last year. Which response by the nurse is most Explanation:
appropriate? The client's questions and concerns
should be acknowledged and
addressed by the nurse after an MI.
The nurse shouldn't give false
reassurance or ignore the client's
immediate concern.
336 A client who has suffered a stroke is too weak to move on his own. turn him frequently.
. To help the client avoid pressure ulcers, the nurse should:
337 A client who has sustained a head injury to the parietal lobe Tactile agnosia
. cannot identify a familiar object by touch. The nurse knows that
this deficit is which of the following?
338 A client who is HIV positive has been prescribed antiretroviral To avoid resistance to the drugs
. drugs. The nurse explains the action of each antiretroviral drug
and develops a schedule for the client's self-administration,
including strong emphasis about rigidly adhering to the dosage,
time and frequency of the administration of the drugs. Why is it
important to adhere to the schedule of drug dosing developed for
this client?
339 A client who is HIV positive is experiencing severe diarrhea. Hypokalemia
. Which laboratory test result would the nurse expect to find?
340 A client who is HIV positive is receiving highly active Disturbed body image related to loss
. antiretroviral therapy (HAART) that includes a protease of fat in the face and arms
inhibitor (PI). The client comes to the clinic for a follow-up visit.
Assessment reveals lipoatrophy of the face and arms. The client
states, "I'm thinking the side effects of the drug are worse than
the disease. Look what's happening to me." The nurse would
most likely identify which nursing diagnosis as the priority?
341 A client who reports increasing difficulty swallowing, weight loss, maintaining a patent airway
. and fatigue is diagnosed with esophageal cancer. Because this
client has difficulty swallowing, the nurse should assign highest
priority to:
342 A client who reports increasing difficulty swallowing, weight loss, Maintaining patent airway
. and fatigue is diagnosed with esophageal cancer. Because this
client has difficulty swallowing, the nurse should assign highest
priority to:
343 A client who's diagnosed with a right subarachnoid hemorrhage With the head of the bed elevated
. should be placed in which position? Explanation:
Elevating the head of the bed
enhances cerebral venous return and
thereby decreases intracranial
pressure (ICP). The other positions
wouldn't decrease ICP.
344 A client who sustained a pulmonary contusion c) Ineffective breathing pattern related to tissue trauma
. in a motor vehicle accident develops a
pulmonary embolism. Which nursing
diagnosis takes priority with this client?

a) Excess fluid volume related to excess sodium


intake
b) Acute pain related to tissue trauma
c) Ineffective breathing pattern related to
tissue trauma
d) Activity intolerance related to insufficient
energy to carry out activities of daily living
345 A client who underwent abdominal surgery Assess patency of the NG tube
. and has a nasogastric (NG) tube in place
begins to complain of abdominal pain that he
describes as "feeling full and uncomfortable."
Which assessment should the nurse perform
first?
346 A client who was bitten by a wild animal is Contact isolation
. admitted to an acute care facility for treatment
of rabies. Which type of isolation does this
client require?
347 A client who was bitten by a wild animal is Contact isolation
. admitted to an acute care facility for treatment Explanation: A client with rabies requires contact isolation
of rabies. Which type of isolation does this because the disease is highly transmissible through close
client require? or direct contact. Rabies isn't transmitted through the air,
eliminating the need for strict isolation, which aims to
prevent transmission of highly contagious or virulent
infections spread by both air and contact. Respiratory
isolation, used to prevent transmission only through the
air, isn't sufficient for a client with rabies. Enteric
isolation is inappropriate because rabies isn't transmitted
through direct or indirect contact with feces.
348 A client who weighs 175 lb (79.4 kg) is a) question the order because the dosage is too low.
. receiving aminophylline (Aminophyllin) (400
mg in 500 ml) at 50 ml/hour. The theophylline
level is reported as 6 mcg/ml. The nurse calls
the physician who instructs the nurse to
change the dosage to 0.45 mg/kg/hour. The
nurse should:

a) question the order because the dosage is too


low.
b) question the order because the dosage is too
high.
c) set the pump at 45 ml/hour.
d) stop the infusion and have the laboratory
repeat the theophylline measurement.
349 A client with achalasia recently underwent Assess lung sounds
. pneumatic dilation. The nurse intervenes after
the procedure by
350 A client with a conductive hearing disorder "Don't fly in an airplane, climb to high altitudes, make
. caused by ankylosis of the stapes in the oval sudden movements, or expose yourself to loud sounds for
window undergoes a stapedectomy to 30 days."
remove the stapes and replace the impaired Explanation:
bone with a prosthesis. After the The nurse should instruct the client to avoid air travel,
stapedectomy, the nurse should provide sudden movements that may cause trauma, and exposure to
which client instruction? loud sounds and pressure changes (such as from high
altitudes) for 30 days after a stapedectomy. Immediately
a) "Don't fly in an airplane, climb to high after surgery, the client should lie flat with the surgical ear
altitudes, make sudden movements, or facing upward; nose blowing is permitted but should be
expose yourself to loud sounds for 30 days." done gently and on one side at a time. The client's first
b) "Lie in bed with your head elevated, and attempt at postoperative ambulation should be supervised to
refrain from blowing your nose for 24 prevent falls caused by vertigo and light-headedness. The
hours." client must avoid shampooing and swimming to keep the
c) "Shampoo your hair every day for 10 days dressing and the ear dry.
to help prevent ear infection."
d) "Try to ambulate independently after
about 24 hours."
351 A client with acute bronchitis is admitted to b) The oxygen tubing is pinched.
. the health care facility and is receiving
supplemental oxygen via nasal cannula.
When monitoring this client, the nurse
suddenly hears a high-pitched whistling
sound. What is the most likely cause of this
sound?

a) The water level in the humidifier reservoir


is too low.
b) The oxygen tubing is pinched.
c) The client has a nasal obstruction.
d) The oxygen concentration is above 44%.
352 A client with acute liver failure exhibits subnormal serum glucose and elevated serum ammonia
. confusion, a declining level of consciousness, levels.
and slowed respirations. The nurse finds him
very difficult to arouse. The diagnostic
information which best explains the client's
behavior is:
353 A client with a disorder of the erythrocytes Transport O2 to and remove CO2 from the tissues
. asks the nurse to explain what role these cells
play. What would the nurse accurately
explain to the client?
354 A client with a disorder of the oral cavity Provide the client with an irrigating solution of baking soda
. cannot tolerate tooth brushing or flossing. and warm water.
Which of the following strategies can the
nurse employ to assist this client?
355 A client with advanced acquired immunodeficiency c) isoniazid, rifampin, ethambutol, and
. syndrome (AIDS) is diagnosed with active pyrazinamide
tuberculosis. Which of the following regimens would
the nurse expect the physician to prescribe?

a) isoniazid (Laniazid) and rifampin (Rifadin)


b) ethambutol (Myambutol), pyrazinamide, and
isoniazid
c) isoniazid, rifampin, ethambutol, and
pyrazinamide
d) ethambutol, ciprofloxacin (Cipro), pyrazinamide,
and streptomycin
356 A client with a feeding tube is to receive medication. Check with pharmacist to see if liquid form is
. The medication supplied is an enteric-coated tablet. available
Which of the following would be most appropriate?
357 A client with a history of chronic renal failure is Hyperkalemia
. admitted with pulmonary edema following a missed Explanation: The kidneys are responsible for
dialysis treatment yesterday. His laboratory results excreting potassium. In renal failure, the kidneys
are serum potassium 6.0 mEg/L, serum sodium 130 can no longer excrete potassium, resulting in
mEg/L, and serum bicarbonate 18 mEg/L. The nurse hyperkalemia. The kidneys are responsible for
interprets that the client has which of the following regulating the acid-base balance; in renal failure,
conditions? acidemia, not alkalemia, would be likely. Generally,
hyponatremia, not hypernatremia, would occur
because of the dilutional effect of water retention.
Hypokalemia is generally seen in clients
undergoing diuresis.
358 A client with a history of type 1 diabetes is admitted c) The pharmacist covering the floor
. to the hospital with community-acquired
pneumonia. The client's blood glucose level in the
emergency care unit was 576 mg/dl. The physician
prescribes an I.V. containing normal saline solution,
an insulin infusion, and I.V. levofloxacin (Levaquin).
The nurse piggybacks the insulin infusion into the
normal saline solution. She questions whether she
can also piggyback the levofloxacin into the same
I.V. line. Which health team member should she
collaborate with to check the compatibility of these
solutions?

a) The physician who prescribed the medications


b) The coworker with 20 years nursing experience
c) The pharmacist covering the floor
d) The infectious disease nurse
359 A client with AIDS comes to the clinic reporting Candidias
. difficulty swallowing. He says, "It hurts so much
when I swallow." Inspection reveals creamy white
patches in the client's mouth. Which of the following
would the nurse suspect?
360 A client with AIDS is exhibiting shortness of breath, Pneumocystis jiroveci
. cough, and fever. The nurse most likely would
suspect infection with which of the following?
361 A client with a nasogastric tube set to low Urine output that decreased from 60 to 40 mL/hr
. intermittent suction is receiving D51/2NS at 100 Heart rate that increased from 82 to 98 beats/min
mL/hr. The nurse has identified a nursing diagnosis within 2 hours
of deficient fluid volume. Which of the following are Fluid output of 2150 mL and total fluid intake of
data that support this diagnosis? Select all that 2000 mL for the past 24 hours
apply.
362 A client with an esophageal stricture is about to Take slow, deep breaths
. undergo esophageal dilatation. As the bougies are
passed down the esophagus, the nurse should
instruct the client to do which action to minimize the
vomiting urge?
363 A client with an esophageal stricture is about to Take long, slow breaths
. undergo esophageal dilatation. As the bougies are
passed down the esophagus, the nurse should
instruct the client to do which action to minimize the
vomiting urge?
364 A client with an intravenous infusion is rubbing Reddended area along the path of the vein & Tender
. his arm. The nurse assesses the site and decides area around the insertion site -Phlebitis is inflammation
to discontinue the current infusion because of of a vein and is characterized by a reddened, warm area
concern that the client has developed phlebitis. around an insertion site or along the path of a vein. The
Which of the following clinical manifestations involved area is also tender and swollen. The nurse
would the nurse assess with phlebitis? Select all assesses infusion sites and determines the proper action
that apply. to take. If indications lead to suspected phlebitis, the
Reddended area along the path of the vein nurse will discontinue the intravenous line and restart
Tender area around the insertion site with a different vessel.
Ecchymosis at the insertion site
Rapid, shallow respirations
Cool area around the insertion site
365 A client with anorexia complains of constipation. Assist client to increase dietary fiber.
. Which of the following nursing measures would
be most effective in helping the client reduce
constipation?
366 A client with a peptic ulcer is diagnosed with "The medications will kill the bacteria and stop the acid
. Heliobacter pylori infection. The nurse is production."
teaching the client about the medications
prescribed, including metronidazole (Flagyl),
omeprazole (Prilosec), and clarithromycin
(Biaxin). Which statement by the client indicates
the best understanding of the medication
regimen?
367 A client with a pilonidal sinus undergoes surgery. Repacking the surgical wound
. Which of the following would the nurse include
in the client's postoperative plan of care?
368 A client with a pneumothorax receives a chest b) There is a leak somewhere in the tubing system.
. tube attached to a Pleur-evac. The nurse notices
that the fluid of the second chamber of the Pleur-
evac isn't bubbling. Which nursing assumption
would be most invalid?

a) The tubing from the client to the chamber is


blocked.
b) There is a leak somewhere in the tubing
system.
c) The client's affected lung has reexpanded.
d) The tubing needs to be cleared of fluid.
369 A client with appendicitis is experiencing Surgery
. excruciating abdominal pain. An abdominal X-
ray film reveals intraperitoneal air. The nurse
should prepare the client for:
370 A client with a pulmonary embolus has the b) Administer oxygen by nasal cannula as prescribed.
. following arterial blood gas (ABG) values: pH,
7.49; partial pressure of arterial oxygen (PaO2),
60 mm Hg; partial pressure of arterial carbon
dioxide (PaCO2), 30 mm Hg; bicarbonate
(HCO3-) 25 mEq/L. What should the nurse do
first?

a) Instruct the client to breathe into a paper


bag.
b) Administer oxygen by nasal cannula as
prescribed.
c) Auscultate breath sounds bilaterally every 4
hours.
d) Encourage the client to deep-breathe and
cough every 2 hours.
371 A client with a recent history of a stroke has The client's arms are fully extended when using the
. been discharged from the rehabilitation walker.
facility with a walker. During the client's Explanation: When using a walker, the client's arms
return visit to the physician's office, the nurse should be slightly bent at the elbow, allowing maximum
assesses his gait. Which finding indicates the support from the arms while ambulating. The weak leg is
need for further teaching about walker use? always moved forward first with the walker to provide the
maximum support. When sitting, the client should always
back up to the chair and feel the chair with his legs before
sitting. The client should use the armrests of the chair for
support because the armrests are more stable than the
walker.
372 A client with a sacral pressure ulcer is limited Coordinate physical therapy with getting the client out of
. to 2 hours of sitting in a chair twice per day. bed for breakfast and dinner; then request bedside
She is scheduled for physical therapy three physical therapy for the third session.
times per day and dressing changes twice per
day. How can a nurse best coordinate this
client's care?
373 A client with a severe staphylococcal infection Ototoxicity
. is receiving the aminoglycoside gentamicin
sulfate (Garamycin) by the I.V. route. The
nurse should monitor the client for which
adverse reaction to this drug?
374 A client with a solar burn of the chest, back, pain management.
. face, and arms is seen in urgent care. The
nurse's primary concern should be:
375 A client with a solar burn of the chest, back, pain management.
. face, and arms is seen in urgent care. The Explanation: With a superficial partial thickness burn such
nurse's primary concern should be: as a solar burn (sunburn), the nurse's main concern is pain
management. Fluid resuscitation and infection become
concerns if the burn extends to the dermal and
subcutaneous skin layers. Body image disturbance is a
concern that has lower priority than pain management.
376 A client with a suspected overdose of an Prepare to assist with ventilation. - Respiratory acidosis is
. unknown drug is admitted to the emergency associated with hypoventilation; in this client,
department. Arterial blood gas values indicate hypoventilation suggests intake of a drug that has
respiratory acidosis. What should the nurse do suppressed the brain's respiratory center. Therefore, the
first? nurse should assume the client has respiratory depression
and should prepare to assist with ventilation. After the
client's respiratory function has been stabilized, the nurse
can safely monitor the heart rhythm, prepare for gastric
lavage, and obtain a urine specimen for drug screening.
377 A client with a suspected pulmonary disorder Tidal Volume
. undergoes pulmonary function tests. To
interpret test results accurately, the nurse
must be familiar with the terminology used to
describe pulmonary functions. Which term
refers to the volume of air inhaled or exhaled
during each respiratory cycle?
378 A client with a suspected pulmonary disorder c) Tidal volume
. undergoes pulmonary function tests. To
interpret test results accurately, the nurse
must be familiar with the terminology used to
describe pulmonary functions. Which term
refers to the volume of air inhaled or exhaled
during each respiratory cycle?

a) Vital capacity
b) Functional residual capacity
c) Tidal volume
d) Maximal voluntary ventilation
379 A client with ataxia-telangiectasia is admitted IV Gamma Globulin Administration
. to the unit. The nurse caring for the client
would expect to see what included in the
treatment regimen?
380 A client with atopic dermatitis is prescribed a potent topical Related to percutaneous absorption of
. corticosteroid. To address a potential client problem associated the topical corticosteroid
with this treatment, the nurse helps formulate the nursing
diagnosis of Risk for injury. To complete the nursing diagnosis
statement, which ""related-to"" phrase should be added?
381 A client with atopic dermatitis is prescribed medication for 24 hours.
. photochemotherapy. The nurse teaches the client about the
importance of protecting the skin from ultraviolet light before
drug administration and stresses the need to protect the eyes.
After administering medication for photochemotherapy, the
client must protect the eyes for:
382 A client with cancer has a neck dissection and laryngectomy. Make a notation in the call light system
. An intervention that the nurse will do is: that the patient cannot speak
383 A client with cancer is being evaluated for possible metastasis. Liver
. Which of the following is a common metastasis site for cancer Explanation: The liver is one of the five
cells? most common cancer metastasis sites.
The others are the lymph nodes, lung,
bone, and brain. The colon, reproductive
tract, and WBCs are occasional
metastasis sites.
384 A client with carcinoma of the head of the pancreas is Packed RBCs
. scheduled for surgery. Which of the following should the nurse
administer to the client before surgery?
385 A client with catatonic schizophrenia is mute, can't perform Assist the client with feeding.
. activities of daily living, and stares out the window for hours. Explanation:
What is the nurse's first priority? According to Maslow's hierarchy of
needs, the need for food is among the
most important. Other needs, in order of
decreasing importance, include hygiene,
safety, and a sense of belonging.
386 A client with chest pain arrives in the emergency department Carvedilol (Coreg)
. and receives nitroglycerin, morphine (Duramorph), oxygen,
and aspirin. The physician diagnoses acute coronary
syndrome. When the client arrives on the unit, his vital signs
are stable and he has no complaints of pain. The nurse reviews
the physician's orders. In addition to the medications already
given, which medication does the nurse expect the physician to
order?
387 A client with chest pain doesn't respond to nitroglycerin. When Within 6 hours
. he's admitted to the emergency department, the health care
team obtains an electrocardiogram and administers I.V.
morphine. The physician also considers administering
alteplase (Activase). This thrombolytic agent must be
administered how soon after onset of myocardial infarction
(MI) symptoms?
388 A client with cholelithiasis has a gallstone lodged in the Yellow sclerae
. common bile duct. When assessing this client, the nurse
expects to note:
389 A client with cholelithiasis has a gallstone lodged in the yellow sclerae.
. common bile duct. When assessing this client, the nurse Explanation: Yellow sclerae may be the
expects to note: first sign of jaundice, which occurs when
the common bile duct is obstructed.
Urine normally is light amber.
Circumoral pallor and black, tarry stools
are signs of hypoxia and GI bleeding,
respectively.
390 A client with chronic bronchitis is admitted to the health Rhonchi
. facility. Auscultation of the lungs reveals low-pitched,
rumbling sounds. Which of the following describes these
sounds?
391 A client with chronic bronchitis is admitted with an Use of accessory muscles to breathe
. exacerbation of symptoms. During the nursing assessment, the Purulent sputum with frequent
nurse will expect which of the following findings? Select all coughing
that apply.
392 A client with chronic obstructive pulmonary disease (COPD) is admitted to the a) instruct the client to
. medical-surgical unit. To help this client maintain a patent airway and achieve drink 2 L of fluid daily.
maximal gas exchange, the nurse should:

a) instruct the client to drink 2 L of fluid daily.


b) maintain the client on bed rest.
c) administer anxiolytics, as prescribed, to control anxiety.
d) administer pain medication as prescribed.
393 A client with chronic obstructive pulmonary disease (COPD) is being evaluated b) Dyspnea on exertion
. for a lung transplant. The nurse performs the initial physical assessment. c) Barrel chest
Which signs and symptoms should the nurse expect to find? e) Clubbed fingers and
toes
a) Decreased respiratory rate
b) Dyspnea on exertion
c) Barrel chest
d) Shortened expiratory phase
e) Clubbed fingers and toes
f) Fever
394 A client with chronic obstructive pulmonary disease (COPD) is recovering c) atelectasis.
. from a myocardial infarction. Because the client is extremely weak and can't
produce an effective cough, the nurse should monitor closely for:

a) pleural effusion.
b) pulmonary edema.
c) atelectasis.
d) oxygen toxicity.
395 A client with chronic obstructive pulmonary disease is admitted to an acute a) 1:2
. care facility because of an acute respiratory infection. When assessing the
client's respiratory rate, the nurse notes an abnormal inspiratory-expiratory
(I:E) ratio of 1:4. What is a normal I:E ratio?

a) 1:2
b) 2:1
c) 1:1
d) 2:2
396 A client with chronic obstructive pulmonary disease presents with respiratory a) Notify the physician
. acidosis and hypoxemia. He tells the nurse that he doesn't want to be placed on immediately so he can
a ventilator. What action should the nurse take? determine client
competency.
a) Notify the physician immediately so he can determine client competency.
b) Have the client sign a do-not-resuscitate (DNR) form.
c) Determine whether the client's family was consulted about his decision.
d) Consult the palliative care group to direct care for the client.
397 A client with chronic obstructive pulmonary disease tells the nurse that he feels d) give the nebulizer
. short of breath. The client's respiratory rate is 36 breaths/min and the nurse treatment herself.
auscultates diffuse wheezes. His arterial oxygen saturation is 84%. The nurse
calls the assigned respiratory therapist to administer a prescribed nebulizer
treatment. The therapist says, "I have several more nebulizer treatments to do
on the unit where I am now. As soon as I'm done, I'll come assess the client."
The nurse's most appropriate action is to:

a) notify the primary physician immediately.


b) stay with the client until the therapist arrives.
c) administer the treatment by metered-dose inhaler.
d) give the nebulizer treatment herself.
398 A client with chronic renal failure (CRF) has developed faulty red blood cell Fatigue and weakness
. (RBC) production. The nurse should monitor this client for:
399 A client with chronic renal failure (CRF) is Blood urea nitrogen (BUN) 100 mg/dl and serum
. admitted to the urology unit. Which diagnostic creatinine 6.5 mg/dl
test results are consistent with CRF?
400 A client with chronic renal failure has a serum Pulse - An elevated serum potassium level may lead to a
. potassium level of 6.8 mEq/L. What should the life-threatening cardiac arrhythmia, which the nurse can
nurse assess first? detect immediately by palpating the pulse. In addition to
assessing the client's pulse, the nurse should place the
client on a cardiac monitor because an arrythmia can occur
suddenly. The client's blood pressure may change, but only
as a result of the arrhythmia. Therefore, the nurse should
assess blood pressure later. The nurse also may delay
assessing respirations and temperature because these aren't
affected by the serum potassium level.
401 A client with chronic schizophrenia who takes Neuroleptic malignant syndrome
. neuroleptic medication is admitted to the Explanation:
psychiatric unit. Nursing assessment reveals The client's signs and symptoms suggest neuroleptic
rigidity, fever, hypertension, tachycardia, malignant syndrome, a life-threatening reaction to
altered consciousness, and diaphoresis. These neuroleptic medication that requires immediate treatment.
findings suggest which life-threatening Tardive dyskinesia causes involuntary movements of the
reaction? tongue, mouth, facial muscles, and arm and leg muscles.
Dystonia is characterized by cramps and rigidity of the
tongue, face, neck, and back muscles. Akathisia causes
restlessness, anxiety, and jitteriness.
402 A client with chronic sinusitis comes to the c) frontal and maxillary sinuses.
. outpatient department complaining of
headache, malaise, and a nonproductive
cough. When examining the client's paranasal
sinuses, the nurse detects tenderness. To
evaluate this finding further, the nurse should
transilluminate the:

a) frontal sinuses only.


b) sphenoidal sinuses only.
c) frontal and maxillary sinuses.
d) sphenoidal and ethmoidal sinuses.
403 A client with cirrhosis has been referred to Discuss meals that include low-fat high-carbohydrate
. hospice care. Assessment data reveal a need to content.
discuss nutrition with the client. What is the
nurse's priority intervention?
404 A client with cirrhosis is at risk for developing Minimize alcohol use
. esophageal varices. Which of the following
instructions should a nurse provide the client
to minimize such risk?
405 A client with complaints of right lower White blood cell (WBC) count 22.8/mm3
. quadrant pain is admitted to the emergency
department. Blood specimens are drawn and
sent to the laboratory. Which laboratory
finding should be reported to the physician
immediately?
406 A client with decreased urine output Nephrotoxic injury secondary to use of contrast media
. refractory to fluid challenges is evaluated for
renal failure. Which condition may cause the
intrinsic (intrarenal) form of acute renal
failure?
407 A client with diabetes begins to have digestive The pancreas secretes digestive enzymes
. problems and is told by the physician that they are a
complication of the diabetes. Which of the following
explanations from the nurse is most accurate?
408 A client with dissociative identity disorder (DID) is To allow staff members to discuss concerns about
. admitted to an inpatient psychiatric unit. A nurse- working with a client with DID
manager asks all staff to attend a meeting. Which is Explanation:
the most likely reason for the meeting? Allowing all staff members to meet together may
prevent them from splitting into groups who
believe the diagnosis is valid and those who don't.
Unless this client shows behaviors harmful to
himself or others, restraints aren't needed. Telling
the staff that no one should refuse to work with the
client or that this client will probably be difficult
sets a negative tone as the staff develops a plan of
care for the client and implements it.
409 A client with dysphagia is being prepared for The client is free from esophagitis and achalasia.
. discharge. Which outcome indicates that the client is
ready for discharge?
410 A client with end-stage acquired immunodeficiency Fluid replacement
. syndrome (AIDS) has profound manifestations of
Cryptosporidium infection caused by the protozoa. In
planning the client's care, the nurse should focus on
his need for:
411. A client with end-stage chronic obstructive b) The client will maintain adequate oxygenation.
pulmonary disease requires bi-level positive airway
pressure (BiPAP). While caring for the client, the
nurse determines that bilateral wrist restraints are
required to prevent compromised care. Which client
care outcome is associated with restraint use in the
client who requires BiPAP?

a) The client will remain infection-free.


b) The client will maintain adequate oxygenation.
c) The client will maintain adequate urine output.
d) The client will remain pain-free.
412 A client with esophageal cancer has difficulty in Give high-protein, semiliquid foods
. swallowing. Which of the following would be
appropriate to help the client achieve improved
nutrition?
413 A client with extreme weakness, pallor, weak "Has he had any recent forceful vomiting?"
. peripheral pulses, and disorientation is admitted to
the emergency department. His wife reports that he
has been "spitting up blood." A Mallory-Weiss tear is
suspected, and the nurse begins taking a client
history from the client's wife. The question by the
nurse that demonstrates her understanding of
Mallory-Weiss tearing is:
414 A client with extreme weakness, pallor, weak "Has your partner had recent forceful vomiting?"
. peripheral pulses, and disorientation is admitted to
the emergency department. His wife reports that he
has been "spitting up blood." A Mallory-Weiss tear is
suspected, and the nurse begins taking a client
history from the client's wife. The question by the
nurse that demonstrates her understanding of
Mallory-Weiss tearing is:
415 A client with generalized anxiety disorder (GAD) is "I want to solve my problems on my own."
. prescribed a benzodiazepine, but the client doesn't Explanation:
want to take the medication. Which explanation for Many clients don't want to take medications because
this behavior would most likely be correct? they believe that using a medication is a sign of
personal weakness and that they can't solve their
problems by themselves. Thinking that the
psychiatrist dislikes them reflects paranoid thinking
that isn't usually seen in clients with GAD. By
waiting several weeks to take the medication, the
client could be denying that the medication is
necessary or beneficial. By focusing on the negative
motives of family members, the client could be
avoiding talking about himself.
416 A client with GERD develops espophagitis. Which Upper endoscopy with biopsy
. diagnostic test would the nurse expect the physician
to order to confirm the diagnosis?
417 A client with Guillain-Barr syndrome develops pH, 7.25; PaCO2 50 mm Hg - In respiratory
. respiratory acidosis as a result of reduced alveolar acidosis, ABG analysis reveals an arterial pH below
ventilation. Which combination of arterial blood 7.35 and partial pressure of arterial carbon dioxide
gas (ABG) values confirms respiratory acidosis? (PaCO2) above 45 mm Hg. Therefore, the
combination of a pH value of 7.25 and a PaCO2
value of 50 mm Hg confirms respiratory acidosis. A
pH value of 7.5 with a PaCO2 value of 30 mm Hg
indicates respiratory alkalosis. A ph value of 7.40
with a PaCO2 value of 35 mm Hg and a pH value of
7.35 with a PaCO2 value of 40 mm Hg represent
normal ABG values, reflecting normal gas exchange
in the lungs.
418 A client with Guillain-Barr syndrome develops d) pH, 7.25; PaCO2 50 mm Hg
. respiratory acidosis as a result of reduced alveolar
ventilation. Which combination of arterial blood
gas (ABG) values confirms respiratory acidosis?

a) pH, 7.5; PaCO2 30 mm Hg


b) pH, 7.40; PaCO2 35 mm Hg
c) pH, 7.35; PaCO2 40 mm Hg
d) pH, 7.25; PaCO2 50 mm Hg
419 A client with herpes zoster is prescribed acyclovir diarrhea.
. (Zovirax), 200 mg by mouth every 4 hours while
awake. The nurse should inform the client that this
drug may cause:
420 A client with human immunodeficiency virus The client is immunodeficient and will not have a
. undergoes intradermal anergy testing using skin response
Candida and mumps antigen. During the 3 days
following the tests, there is no induration or
evidence of reaction at the intradermal injection
sites. The most accurate conclusion the nurse can
make is:
421 A client with left-sided heart failure complains of Acute pulmonary edema
. increasing shortness of breath and is agitated and
coughing up pink-tinged, foamy sputum. The nurse
should recognize these findings as signs and
symptoms of:
422 A client with liver and renal failure has severe Albumin
. ascites. On initial shift rounds, his primary nurse
finds his indwelling urinary catheter collection bag
too full to store more urine. The nurse empties more
than 2,000 ml from the collection bag. One hour
later, she finds the collection bag full again. The
nurse notifies the physician, who suspects that a
bladder rupture is allowing the drainage of
peritoneal fluid. The physician orders a urinalysis
to be obtained immediately. The presence of which
substance is considered abnormal?
423 A client with myasthenia gravis is receiving continuous b) Suction the client's artificial airway.
. mechanical ventilation. When the high-pressure alarm
on the ventilator sounds, what should the nurse do?

a) Check for an apical pulse.


b) Suction the client's artificial airway.
c) Increase the oxygen percentage.
d) Ventilate the client with a handheld mechanical
ventilator.
424 A client with pancreatitis is admitted to the medical Reserving an antecubital site for a peripherally
. intensive care unit. Which nursing intervention is most inserted central catheter (PICC)
appropriate?
425 A client with peptic ulcer disease must begin triple 10-14 days
. medication therapy. For how long will the client follow
this regimen?
426 A client with pneumococcal pneumonia is admitted to a) Inflamed lung tissue
. an acute care facility. The client in the next room is
being treated for mycoplasmal pneumonia. Despite the
different causes of the various types of pneumonia, all of
them share which feature?

a) Inflamed lung tissue


b) Sudden onset
c) Responsiveness to penicillin
d) Elevated white blood cell (WBC) count
427 A client with pneumonia develops respiratory failure c) 0.5
. and has a partial pressure of arterial oxygen of 55 mm
Hg. He's placed on mechanical ventilation with a
fraction of inspired oxygen (FIO2) of 0.9. The nursing
goal should be to reduce the FIO2 to no greater than:

a) 0.21.
b) 0.35.
c) 0.5
d) 0.7
428 The client with polycystic kidney disease asks the nurse, "As the disease progresses, you will most likely
. "Will my kidneys ever function normally again?" The require renal replacement therapy."
best response by the nurse is:
429 A client with psoriasis visits the dermatology clinic. Scale
. When inspecting the affected areas, the nurse expects to
see which type of secondary lesion?
430 A client with renal failure is undergoing continuous Risk for infection
. ambulatory peritoneal dialysis. Which nursing diagnosis
is the most appropriate for this client?
431 A client with respiratory acidosis is admitted to the Shock - Complications of respiratory acidosis
. intensive care unit for close observation. The nurse include shock and cardiac arrest. Stroke and
should stay alert for which complication associated with hyperglycemia aren't associated with respiratory
respiratory acidosis? acidosis. Seizures may complicate respiratory
alkalosis, not respiratory acidosis.
432 A client with respiratory acidosis is admitted to the Shock - Complications of respiratory acidosis
. intensive care unit for close observation. The nurse include shock and cardiac arrest. Stroke and
should stay alert for which complication associated with hyperglycemia aren't associated with respiratory
respiratory acidosis? acidosis. Seizures may complicate respiratory
alkalosis, not respiratory acidosis
433 A client with respiratory acidosis is admitted to the a) Shock
. intensive care unit for close observation. The nurse
should stay alert for which complication associated
with respiratory acidosis?

a) Shock
b) Stroke
c) Seizures
d) Hyperglycemia
434 A client with second- and third-degree burns on the dislodge the autografts.
. arms receives autografts. Two days later, the nurse
finds the client doing arm exercises. The nurse knows
that this client should avoid arm exercise because it
may:
435 A client with severe acute respiratory syndrome c) support the client's decision.
. (SARS) privately informs the nurse that he doesn't
want to be placed on a ventilator if his condition
worsens. The client's wife and children have
repeatedly expressed their desire that everything be
done for the client. The most appropriate action by
the nurse would be to:

a) inform the family of the client's wishes.


b) assure the family that everything possible will be
done.
c) support the client's decision.
d) assure the client that everything possible will be
done.
436 A client with severe and chronic liver disease is Vitamin A
. showing manifestations related to inadequate vitamin
intake and metabolism. He reports difficulty driving
at night because he cannot see well. Which of the
following vitamins is most likely deficient for this
client?
437 A client with severe angina pectoris and troponin.
. electrocardiogram changes is seen by a physician in Explanation:
the emergency department. In terms of serum This client exhibits signs of myocardial infarction
testing, it's most important for the physician to order (MI), and the most accurate serum determinant of
cardiac: an MI is troponin level. Creatine kinase, lactate
dehydrogenase and myoglobin tests can show
evidence of muscle injury, but they're less specific
indicators of myocardial damage than troponin.
438 A client with severe angina pectoris and Troponin
. electrocardiogram changes is seen by a physician in
the emergency department. In terms of serum
testing, it's most important for the physician to order
cardiac:
439 A client with severe combined immunodeficiency is to Immunosuppressive agents
. receive a hematopoietic stem cell transplant. Which
of the following would the nurse expect to be started?
440 A client with severe hypertension states, "I feel fine; Sensory
. I'm not really sick at all." The nurse will teach the
client that the system/organs particularly targeted
for damage by severe hypertension include which of
the following?
441 A client with severe peptic ulcer disease has Hemorrhage
. undergone surgery and is several hours
postoperative. During assessment, the nurse notes
that the client has developed cool skin, tachycardia,
and labored breathing; the client also appears to be
confused. Which of the following complications has
the client most likely developed?
442 A client with suspected HIV has had two positive Western Blot
. enzyme-linked immunosorbent assay tests. What
diagnostic test would be run next?
443 A client with suspected inhalation anthrax is a) Monitor vital signs and oxygen saturation every 15 to
. admitted to the emergency department. Which 30 minutes.
action by the nurse takes the highest priority?

a) Monitor vital signs and oxygen saturation


every 15 to 30 minutes.
b) Suction the client as needed to obtain a
sputum specimen for culture and sensitivity.
c) Assess intake and output and maintain
adequate hydration.
d) Reassure the client that intubation and
mechanical ventilation will be temporary.
444 A client with suspected renal insufficiency is The nephron
. scheduled for a comprehensive diagnostic Explanation:
workup. After the nurse explains the diagnostic The nephron, the functioning unit of the kidney, includes
tests, the client asks which part of the kidney the glomerulus, Bowman's capsule, and tubular system,
"does the work." Which answer is correct? which work together to form urine.
445 A client with urinary tract infection is This medication will relieve your pain
. prescribed phenazopyridine (Pyridium). Which
of the following instructions would the nurse
give the client?
446 A client with viral hepatitis A is being treated wash her hands after touching the client.
. in an acute care facility. Because the client
requires enteric precautions, the nurse should:
447 A Community Health Nurse is giving an Changes in attitude
. informational talk on hearing loss to the local Correct
PTO. What would the nurse tell the attendees Explanation:
that hearing impairment can trigger? Hearing impairment can trigger changes in attitude, the
ability to communicate, the awareness of surroundings,
a) Disturbed image and even the ability to protect oneself, affecting a person's
b) Problems reading quality of life. The scenario does not specify anyone for
c) Overprotectiveness the hearing impaired to be overprotective of, nor does it
d) Changes in attitude specify an image to become disturbed. Hearing
impairment would not cause problems reading.
448 Coordinate internal and external responses What is the function of the thalamus and the
. hypothalamus?
449 Coumadin Which medication is indicated for the patient with atrial
. fibrillation who is at high risk for stroke?
450 Countertransference A nurse therapist finds herself feeling sad after sessions
. with a client. The client's passiveness reminds her of a
family member who led a very unhappy life. What is the
term for this emotional dynamic
451 Covert Which type of cue is being used when the client states,
. "Nothing can help me"?
452 Covert An assigned client's diagnosis is paranoid personality
. disorder. Which axis, according to the , would the
diagnosis be classified?
453 Covert; Covert cues are vague or hidden Which type of cue is being used when the client states,
. messages that need interpretation and "Nothing can help me"?
exploration.
454 Cranial nerve IX is also known as which of the Glossopharyngeal
. following?
455 A critical care nurse is caring for a client with Maintain patient in Semi-Fowlers position
. acute pancreatitis. One potentially severe
complication involves the respiratory system.
Which of the following would be an
appropriate intervention to prevent
complications associated with the respiratory
system?
456 A critical care nurse is caring for a client with Maintain the client in a semi-Fowler's position.
. pancreatitis. One potentially severe complication
involves the respiratory system. Which of the
following would be an appropriate intervention
to prevent complications associated with the
respiratory system?
457 The critical care nurse is giving report on a client Comatose
. they are caring for. The nurse uses the Glasgow
Coma Scale (GCS) to assess the level of
consciousness (LOC) of a female client and
reports to the on-coming nurse that the client has
an LOC of 6. What does an LOC score of 6 in a
client indicate?
458 CULTURAL DIVERSITY- Medications are - Porcine based products (heparine or bovine)
. prohibited for Muslim and Jewish faith?
- Bovine
-Buddhist?
459 A day-treatment program Mrs. Cairns was diagnosed with bipolar I disorder
. several years ago. After occasional inpatient admissions
surrounding manic episodes over the past few years,
she has been receiving outpatient psychiatric services
for the past 12 months. Her care providers, however,
are concerned that these outpatient services are not
meeting her needs, though she does not currently meet
inpatient admission criteria. Mrs. Cairns may benefit
from what?
460 Decreased cardiac output and decreased systolic After evaluating a client for hypertension, a physician
. and diastolic blood pressure orders atenolol (Tenormin), 50 mg P.O. daily. Which
therapeutic effect should atenolol have?
461 Decreased pulse pressure reflects reduced stroke volume.
. Explanation:
Decreased pulse pressure reflects reduced stroke
volume and ejection velocity or obstruction to blood
flow during systole. Increased pulse pressure would
indicate reduced distensibility of the arteries, along
with bradycardia.
462 Decrease myocardial contractility. A patient has a high magnesium level. Identify how
. hypermagnesemia affects cardiac function.
463 The defibrillator won't deliver a shock if the Before using a defibrillator to terminate ventricular
. synchronizer switch is turned on. fibrillation, a nurse should check the synchronizer
switch. Why is this check so important?
464 Deficient fluid volume Clients with acute pancreatitis commonly experience
. deficient fluid volume, which can lead to hypovolemic
shock. The volume deficit may be caused by vomiting,
hemorrhage (in hemorrhagic pancreatitis), and plasma
leaking into the peritoneal cavity. Hypovolemic shock
will cause a decrease in cardiac output. Tissue
perfusion will be ineffective if hypovolemic shock
occurs, but this wouldn't be the primary nursing
diagnosis.
465 Denial. Many families take years to understand that a member
. is mentally ill and to identify the warning signs of
relapse. During this period, they try to normalize
puzzling behaviors. This is called
466 Dennis Morgan, a 49-year-old chef, is a client of All options are correct.
. the primary care group where you practice
nursing. Dennis has been studying up on blood
cell production since the development of his
blood disorder. At each appointment, he tries to
fool you, his nurse, with blood cell trivia. His
latest question: "Which of the following cell types
are produced from pluripotential stem cells?"
Your response is:
467 Despite conventional treatment, a client's psoriasis 10
. has worsened. His physician prescribes
methotrexate (Trexall), 25 mg by mouth as a single
weekly dose. The pharmacy dispenses 2.5 mg scored
tablets. How many tablets should the nurse instruct
the client to consume to achieve the prescribed
dose?
468 The development of a positive HIV antibody test 4 weeks
. following initial infection generally occurs in which
timeframe?
469 Diagnosis of Kaposi's sarcoma (KS) is made by Biopsy
. which of the following?
470 A dialysis client is prescribed erythropoietin 1.1
. (Epogen) to treat anemia associated with end-stage
renal disease. The client weighs 147 lbs. The order is
for Epogen 50 units/kg subcutaneously 3 times per
week. The pharmacy supplied Epogen 3000
units/ml. How many milliliters will the nurse
administer to the client? Round to the nearest tenth.
471 DiGeorge syndrome is an example of which Primary T Cell
. immunodeficiency?
472 The digestion of carbohydrates is aided by Amylase
.

473 dividing the body into sections A frontal or coronal plane runs longitudinally at a
. right angle to a sagittal plane, dividing the body into
anterior and posterior regions. A sagittal plane runs
longitudinally, dividing the body into right and left
regions; if exactly midline, it is called a midsagittal
plane. A transverse plane runs horizontally at a right
angle to the vertical axis, dividing the structure into
superior and inferior regions.
474 Dopamine A nurse is developing a plan of care for a patient
. diagnosed with schizophrenia. The nurse integrates
knowledge of this disorder, identifying which
neurotransmitter as being primarily involved?
475 Dowager's hump Abnormal curvature in the upper thoracic spine.
.

476 During a follow-up visit to the physician, a client phosphorus


. with hyperparathyroidism asks the nurse to explain Explanation: PTH increases the serum calcium level
the physiology of the parathyroid glands. The nurse and decreases the serum phosphate level. PTH
states that these glands produce parathyroid doesn't affect sodium, potassium, or magnesium
hormone (PTH). PTH maintains the balance regulation.
between calcium and:
477 During a prenatal visit, the nurse measures a 19 weeks
. client's fundal height at 19 cm. This measurement Explanation:
indicates that the fetus has reached approximately The fundal height measurement in centimeters
which gestational age? equals the approximate gestational age in weeks,
until week 32. Thus, fundal height at 12 weeks is 12
cm; at 24 weeks, 24 cm; and at 28 weeks, 28 cm.
478 During a pulmonary assessment, the nurse observes Lateral diameter greater than anteroposterior
. the chest for configuration. She identifies the diameter
findings as normal. Which of the following would be
consistent with normal assessment?
479 During a routine examination of a client's Beau's line
. fingernails, the nurse notes a horizontal depression
in each nail plate. When documenting this finding,
the nurse should use which term?
480 During a senior citizen health screening, the nurse Kyphosis
. observes a 75-year-old female with a severely Explanation: Kyphosis refers to an increased
increased thoracic curve, or "humpback". What is thoracic curvature of the spine, or "humpback."
this condition called? Lordosis is an increase in the lumbar curve or
swayback. Scoliosis is a lateral deformity of the
spine. Genus varum is a bow-legged appearance of
the legs.
481 During assessment of a client admitted for Decreased CO
. cardiomyopathy, the nurse notes the following
symptoms: dyspnea on exertion, fatigue, fluid
retention, and nausea. The initial appropriate
nursing diagnosis is which of the following?
482 During a vaginal examination of a client in 0
. labor, the obstetrician determines that the Explanation:
biparietal diameter of the fetal head has When the largest diameter of the presenting part (typically
reached the level of the ischial spines. The the biparietal diameter of the fetal head) is level with the
most accurate documentation of this fetal ischial spines, the fetus is at station 0. A station of -1
station would be: indicates that the fetal head is 1 cm above the ischial
spines. At +1, it's 1 cm below the ischial spines. At +2, it's
2 cm below the ischial spines.
483 During data collection, a client with waxy flexibility.
. schizophrenia leaves his arm in the air after Explanation:
the nurse has taken his blood pressure. His Waxy flexibility is defined as retaining any position that
action shows evidence of: the body has been placed in. Somatic delusions involve a
false belief about the functioning of the body. Neologisms
are invented meaningless words. Nihilistic delusions are
false ideas about self, others, or the world.
484 During inspiration, which of the following b) Diaphragm descends.
. occurs?

a) Lungs recoil.
b) Diaphragm descends.
c) Alveolar pressure is positive.
d) Inspiratory muscles relax.
485 During preoperative teaching for a client who "You must avoid hyperextending your neck after surgery."
. will undergo subtotal thyroidectomy, the Explanation: To prevent undue pressure on the surgical
nurse should include which statement? incision after subtotal thyroidectomy, the nurse should
advise the client to avoid hyperextending the neck. The
client may elevate the head of the bed as desired and
should perform deep breathing and coughing to help
prevent pneumonia. Subtotal thyroidectomy doesn't affect
swallowing.
486 During recovery from a stroke, a client is cranial nerves IX and X.
. given nothing by mouth to help prevent
aspiration. To determine when the client is
ready for a liquid diet, the nurse assesses the
client's swallowing ability once per shift. This
assessment evaluates:
487 During the acute phase of a burn, the nurse Circulatory status
. should assess which of the following?
488 During the admission assessment, a client "You're having a panic attack. I'll stay here with you."
. with a panic disorder begins to hyperventilate Explanation:
and says, "I'm going to die if I don't get out of During a panic attack, the nurse's best approach is to orient
here right now!" What's the nurse's best the client to what's happening and provide reassurance that
response? the client won't be left alone. The client's anxiety level is
likely to increaseand the panic attack is likely to
continueif the client is told to calm down, asked the
reasons for the attack, or is left alone.
489 During the auscultation of heart, what Hypertensive heart disease
. is revealed by an atrial gallop? Explanation:
Auscultation of the heart requires familiarization with normal and
abnormal heart sounds. An extra sound just before S1 is an S4
heart sound, or atrial gallop. An S4 sound often is associated with
hypertensive heart disease. A sound that follows S1 and S2 is
called an S3 heart sound or a ventricular gallop. An S3 heart sound
is often an indication of heart failure in an adult. In addition to
heart sounds, auscultation may reveal other abnormal sounds, such
as murmurs and clicks, caused by turbulent blood flow through
diseased heart valves.
490 During the first few days of recovery ostomy care
. from ostomy surgery for ulcerative
colitis, which aspect should be the first
priority of client care?
491 During the immune response, Effector T-cells
. cytotoxic cells bind to invading cells,
destroy the targeted invader and
release lymphokines to remove the
debris. Which type of T-cell
lymphocyte is cytotoxic?
492 During the insertion of a rigid scope Drop in the client's heart rate
. for bronchoscopy, a client experiences
a vasovagal response. The nurse
should expect:
493 During the insertion of a rigid scope Drop in clients HR
. for bronchoscopy, a client experiences
a vasovagal response. The nurse
should expect:
494 During the surgical procedure, the dantrolene sodium (Dantrium)
. client exhibits tachycardia,
generalized muscle rigidity, and a
temperature of 103F. The nurse
should prepare to administer:
495 During which stage of the immune Proliferation
. response does the circulating
lymphocyte containing the antigenic
message return to the nearest lymph
node?
496 The dynamics of the entire family A mother completes treatment for an addiction to prescription pain
. have and will continue to shift to medications. As part of the mother's therapy, the family
accommodate a change. participates in a family therapy program. According to family
systems theory, this is because of what?
497 Early signs of hypervolemia include increased breathing effort and weight gain - Early signs of
. hypervolemia are weight gain, elevated blood pressure, and
increased breathing effort...(more)
Early signs of hypervolemia are weight gain, elevated blood
pressure, and increased breathing effort. Eventually, fluid
congestion in the lungs leads to moist breath sounds. An earliest
symptom of hypovolemia is thirst
498 Early signs of hypervolemia include Increased breathing effort and weight gain - Early signs of
. hypervolemia are weight gain, elevated blood pressure, and
increased breathing effort. Eventually, fluid congestion in the lungs
leads to moist breath sounds. An earliest symptom of hypovolemia
is thirst
499 Ego A group of nursing students are reviewing information about
. Freud's personality structure. The students demonstrate
understanding of this information when they identify the ability to
form mutually satisfying relationships as a function of which of the
following?
500 An elderly client asks the nurse how to Take a stool softener such as docusate sodium (Colace) daily.
. treat chronic constipation. What is the
best recommendation the nurse can
make?
501 An elderly client is diagnosed with Failure of lymphocytes to recognize mutant cell
. cancer. While reviewing age-related
changes in the immune system, the
nurse identifies which of the following
as having contributed to this client's
condition?
502 An elderly client states, "I don't Exhibiting hemoglobin A1C 8.2
. understand why I have so many caries
in my teeth." The nurse assesses the
following as placing the client at risk:
503 An elderly client takes 40 mg of Lasix Hypokalemia - Hypokalemia (potassium level below 3.5 mEq/L)
. twice a day. Which electrolyte usually indicates a defict in total potassium stores. Potassium-
imbalance is the most serious adverse losing diuretics, such as loop diuretics, can induce hypokalemia.
effect of diuretic use?
504 An elderly client tells the nurse that he Explaining why a bath is important to overall health, and telling
. doesn't want to take a bath. Which the client that she'll return in 30 minutes to help him bathe
action by the nurse is most Explanation: It's important for the client to understand why a bath
appropriate? is important to overall health. Communicating with the client
shows respect and aids compliance. Giving the client a specific
time for the bath allows him time to prepare for the care.
Documenting bath refusal, calling the physician, and contacting
family members are inappropriate before discussing the
importance of the bath with the client and reattempting to provide
care.
505 An elderly client who lives at home Assess the client thoroughly and complete the health history.
. with her daughter is admitted with
unexplained bruises on her arms and
legs. Which action should the nurse
take first?
506 An elderly client with influenza is b) Pneumonia
. admitted to an acute care facility. The
nurse monitors the client closely for
complications. What is the most
common complication of influenza?

a) Septicemia
b) Pneumonia
c) Meningitis
d) Pulmonary edema
507 An elderly patient diagnosed with Hypokalemia
. diarrhea is taking digoxin (Lanoxin).
Which of the following electrolyte
imbalances should the nurse be alert
to?
508 Elderly patients also have a decreased - Potent action!
. plasma proteins, therefore the
anesthetic agent remains free or
unbound. What does this causes?
509 Elective electrical cardioversion Your client has been diagnosed with an atrial dysrhythmia. The
. client has come to the clinic for a follow-up appointment and to
talk with the physician about options to stop this dysrhythmia.
What would be a procedure used to treat this client?
510 ELISA stands for: Enzyme-linked immunoabsorbent assay
.

511. The Emergency Department (ED) nurse is Flush out Excess magnesium - The main objective is to
caring for a client who is known to make flush out excess magnesium. Laxatives contain magnesium,
excessive use of laxatives who is showing and their excessive use may cause hypermagnesemia.
signs of bradycardia. The client is admitted Bradycardia or slow heart rate is one of the signs of this
for hemodialysis. The ED nurse knows that a imbalance. In severe cases, hemodialysis may be necessary.
major goal of managing this client is what? Magnesium sulfate is administered in hypomagnesemia and
not hypermagnesemia. Mechanical ventilation is necessary
only if there is a change in respiratory rate, rhythm, or
depth. The physician may permit the use of magnesium-free
laxatives and the client should follow the recommended
frequency of their use.
512 The Emergency Department (ED) nurse is Bicarbonate - Arterial blood gas (ABG) results are the main
. caring for a client with a possible acid-base tool for measuring blood pH, CO2 content (PaCO2), and
imbalance. The physician has ordered an bicarbonate. An acid-base imbalance may accompany a
arterial blood gas (ABG). What is one of the fluid and electrolyte imbalance. PaO2 and PO2 are not
most important indications of an acid-base indications of acid-base imbalance. Carbonic acid levels are
imbalance that is shown in an ABG? not shown in an ABG.
513 Energizer Which group member attempts to stimulate the group to
. action or decision?
514 enteric precautions Gowns and gloves required, masks not required, protection
. from feces and urine.
515 Epidural anesthesia? - State of narcosis achieved by injecting an anesthetic agent
. into the epidural space of the spinal cord.
516 Establishment or enlargement of state One of the primary reforms accomplished by Dorothea
. hospitals. Lynde Dix was the:
517 Evaluation During the evaluation step of the nursing process, the nurse
. determines whether the goals established in the plan of care
have been achieved and evaluates the success of the plan. If
a goal is unmet or partially met, the nurse reexamines the
data and revises the plan. Data collection involves gathering
relevant information about the patient. Planning involves
setting priorities, establishing goals, and selecting
appropriate interventions. Implementation involves
providing actual nursing care.
518 An example of a curative surgical procedure the excision of a tumor.
. is
519 Examples are control of hemorrhage; repair Emergency surgery
. of trauma, perforated ulcer, intestinal
obstruction; tracheostomy
520 Examples are removal of gallbladder, Urgent surgery
. coronary artery bypass, surgical removal of
a malignant tumor, colon resection,
amputation
521 Examples are tonsillectomy, hernia repair, Elective surgery
. cataract extraction and lens implantation,
hemorrhoidctomy, hip prosthesis, scar
revision, facelift, mammoplasty
522 Excessive probing After reviewing the client's chart, the nurse sets up a time to
. speak with the client. The client has a history of severe
psychological abuse by her mother, who has schizophrenia.
The nurse plans to ask the client about the abuse and how it
has affected her sense of self-esteem. This is an example of
what kind of intervention?
523 Exposure to blood and body fluids? - Double gloving!
.
- What is common in trauma and other types
of surgery?
524 Facilitating the nurse's understanding of When the psychiatric nurse is aware of the cultural beliefs
. how these beliefs affect the client's of a client diagnosed with bipolar disorder, the therapeutic
perception of her disorder. process is most enhanced by
525 Facilitating the nurse's understanding of how When the psychiatric nurse is aware of the cultural
. these beliefs affect the client's perception of her beliefs of a client diagnosed with bipolar disorder, the
disorder therapeutic process is most enhanced by...
526 Failure to capture A client receives a pacemaker to treat a recurring
. arrhythmia. When monitoring the cardiac rhythm strip,
the nurse observes extra pacemaker spikes that don't
precede a beat. Which condition should the nurse
suspect?
527 A family meeting is held with a client who Avoidance of issues that cause conflict
. abuses alcohol. While listening to the family, Explanation:
which unhealthy communication pattern might The interaction pattern of a family with a member who
be identified? abuses alcohol commonly revolves around denying the
problem, avoiding conflict, or rationalizing the
addiction. Health care providers are more likely to use
jargon. The family might have problems setting limits
and expressing disapproval of the client's behavior.
Nonverbal communication usually gives the nurse
insight into family dynamics
528 The Family Nurse Practitioner is assessing a 55- Romberg test
. year-old who came to the clinic complaining of
being "unsteady" on their feet. What would be
a test for equilibrium?
529 The Family Nurse Practitioner is performing Moving the head and chin toward the chest
. the physical examination of a client with a
suspected neurologic disorder. In addition to
assessing other parts of the body, the nurse
should assess for neck rigidity. Which method
should help the nurse assess for neck rigidity
correctly?
530 A female client has undergone a lumbar Encourage a liberal fluid intake for the client.
. puncture for a neurological assessment. The Position the client flat for at least three hours or as
client is put under the post-procedure care of a directed by the physician.
nurse. Which of the following important post-
procedure nursing interventions should be
performed to ensure maximum comfort to the
client? Choose all that apply.
531 A female client with genital herpes simplex is cancer of the cervix.
. being treated in the outpatient department. The
nurse teaches her about measures that may
prevent herpes recurrences and emphasizes the
need for prompt treatment if complications
arise. Genital herpes simplex increases the risk
of:
532 A female patient has undergone a lumbar Encourage a liberal fluid intake for the patient
. puncture for a neurological assessment. The
patient is put under the postprocedure care of a
nurse. Which of the following important
postprocedure nursing interventions should be
performed to ensure maximum comfort to the
patient?
533 A female patient who is 38 years of age has Use of other drugs.
. begun to suffer from rheumatoid arthritis. She
is also being assessed for disorders of the
immune system. She works as an aide at a
facility that cares for children infected with
AIDS. Which of the following is the most
important factor related to the patient's
assessment?
534 A few hours after eating hot and spicy chicken Investigate the initial complaint
. wings, a client presents with lower chest pain.
He wonders if he is having a heart attack. How
should the nurse proceed first?
535 A few hours after eating hot and spicy chicken Further investigate the initial complaint
. wings, a client presents with lower chest pain.
He wonders if he is having a heart attack. How
should the nurse proceed first?
536 Fistula? A fistula is defined as the connection of two body
. cavities.
537 Five days after running out of medication, "You could go through withdrawal symptoms for up to 2
. a client taking clonazepam (Klonopin) weeks."
says to the nurse, "I know I shouldn't Explanation:
have just stopped the drug like that, but Withdrawal symptoms can appear after 1 or 2 weeks because
I'm okay." Which response would be this benzodiazepine has a long half-life. Looking for another
best? problem unrelated to withdrawal isn't the nurse's best strategy.
The act of discontinuing an antianxiety medication doesn't
indicate that a client has learned to cope with stress. Every
client taking medication needs to be monitored for withdrawal
symptoms when the medication is stopped abruptly.
538 A fluid volume deficit can be caused by In hypovolemia only blood volume is low - Dehydration
. either dehydration or hypovolemia. What results when the volume of body fluid is significantly reduced
is the distinction between the two? in both extracellular and...(more)
a) In hypovolemia all fluid compartments Dehydration results when the volume of body fluid is
have decreased volumes. significantly reduced in both extracellular and intracellular
b) In dehydration intracellular fluid compartments. In dehydration, all fluid compartments have
volume is depleted. decreased volumes; in hypovolemia, only blood volume is low.
c) In hypovolemia only blood volume is This makes options A, B, and D incorrect.
low.
d) In dehydration only blood volume is
low.
539 Following a full-thickness (third-degree) range of motion.
. burn of his left arm, a client is treated
with artificial skin. The client understands
postoperative care of artificial skin when
he states that during the first 7 days after
the procedure, he will restrict:
540 The following appears on the medical Glucose level
. record of a male patient receiving
parenteral nutrition:
WBC: 6500/cu mm
Potassium 4.3 mEq/L
Magnesium 2.0 mg/dL
Calcium 8.8 mg/dL
Glucose 190 mg/dL

Which finding would alert the nurse to a


problem?
541 Following a small-bowel resection, a client pressurelike pain.
. develops fever and anemia. The surface
surrounding the surgical wound is warm
to the touch and necrotizing fasciitis is
suspected. Another manifestation that
would most suggest necrotizing fasciitis is:
542 Following a unilateral adrenalectomy, a Muscle Weakness - Muscle weakness, bradycardia, nausea,
. nurse should assess for hyperkalemia as diarrhea, and paresthesia of the hands, feet, tongue, and face
indicated by: are findings associated with hyperkalemia, which is transient
and results from transient hypoaldosteronism when the
adenoma is removed. Tremors, diaphoresis, and constipation
aren't seen in hyperkalemia.
543 Following a unilateral adrenalectomy, a muscle weakness - Muscle weakness, bradycardia, nausea,
. nurse should assess for hyperkalemia as diarrhea, and paresthesia of the hands, feet, tongue, and face
indicated by: are findings associated with hyperkalemia, which is transient
and results from transient hypoaldosteronism when the
adenoma is removed. Tremors, diaphoresis, and constipation
aren't seen in hyperkalemia.
544 Following ingestion of carrots or beets, the Red
. nurse would expect which alteration in
stool color?
545 The following statement appears on a a client outcome.
. client's plan of care: "Client will Explanation: A client outcome is a short- or long-term goal based
ambulate in the hall without assistance on projected nursing interventions. A nursing diagnosis is a
within 4 days." This statement is an statement about a client's actual or potential problem. Subjective
example of: data are information relayed to the nurse by the client. A nursing
intervention is an action the nurse takes in response to a client's
problem.
546 For a child with a Wilms' tumor, which Avoiding abdominal palpation
. preoperative nursing intervention takes Explanation: Because manipulating the abdominal mass may
highest priority? disseminate cancer cells to adjacent and distant sites, the most
important intervention for a child with a Wilms' tumor is to avoid
palpating the abdomen. Restricting oral intake and monitoring
acid-base balance are routine interventions for all preoperative
clients; they have no higher priority in one with a Wilms' tumor.
Isolation isn't required because a Wilms' tumor isn't infectious.
547 For a client with an acute pulmonary b) 22
. embolism, the physician prescribes
heparin (Liquaemin), 25,000 U in 500
ml of dextrose 5% in water (D5W) at
1,100 U/hour. The nurse should
administer how many milliliters per
hour?

a) 8
b) 22
c) 30
d) 50
548 For a client with an endotracheal (ET) a) Auscultating the lungs for bilateral breath sounds
. tube, which nursing action is most
essential?

a) Auscultating the lungs for bilateral


breath sounds
b) Turning the client from side to side
every 2 hours
c) Monitoring serial blood gas values
every 4 hours
d) Providing frequent oral hygiene
549 For a client with cirrhosis, deterioration difficulty in arousal.
. of hepatic function is best indicated by: Explanation: Hepatic encephalopathy, a major complication of
advanced cirrhosis, occurs when the liver no longer can convert
ammonia (a by-product of protein breakdown) into glutamine.
This leads to an increased blood level of ammonia a central
nervous system toxin which causes a decrease in the level of
consciousness. Fatigue, muscle weakness, nausea, anorexia, and
weight gain occur during the early stages of cirrhosis.
550 For an infant who's about to undergo a an arched, side-lying position, avoiding flexion of the neck onto
. lumbar puncture, the nurse should place the chest.
the infant in: Explanation: For a lumbar puncture, the nurse should place the
infant in an arched, side- lying position to maximize the space
between the third and fifth lumbar vertebrae. The nurse's hands
should rest on the back of the infant's shoulders to prevent neck
flexion, which could block the airway and cause respiratory
arrest. The infant should be placed at the edge of the bed or table
during the procedure, and the nurse should speak quietly to calm
the child. A mummy restraint would limit access to the lumbar
area because it involves wrapping the child's trunk and
extremities snugly in a blanket or towel. A prone position
wouldn't cause separation of the vertebral spaces.
551 For a patient with salivary calculi, Lithotripsy
. which of the following procedures uses
shock waves to disintegrate the stone?
552 Foster homes Which type of residential setting may care for one to
. three clients in a family-like atmosphere, including
meals and social activities with the family?
553 Frequently, what is the earliest symptom of left- Dyspnea on exertion
. sided heart failure?
554 From the following profiles of clients, which Mark, suspected of having stones in the gallbladder
. client would be most likely to undergo the
diagnostic test of cholecystography?
555 General anesthesia? - State of narcosis (entire body reflexes is loss)
.

556 Gerontologic considerations: Elderly are at - Pulmonary


. higher risk from anesthesia and cardiovascular - Respond to stress
& ____changes. - Pulmonary Edema

- The aging heart have ___ability to ____to -cerebral ischemia


stress. -thrombosis
- Excessive or rapid administration of IV can -embolism
cause __.
- What happens next? BP will drop and may lead
to ___ ischemia, ___, ___, infarction and anoxia.
557 Giving approval Which of the following is an example of a
. nontherapeutic communication technique?
558 Globulins are proteins contained in plasma. Immunologic agents
. What is their primary function?
559 Gradual, unexplained weight gain. A 63-year-old accountant was admitted to the cardiac
. ICU with full-blown pulmonary edema. After he was
revived, the nurse discusses his symptoms with the
client and his wife. What is a typical, subtle symptom
that communicates right-sided heart failure?
560 A group of 16-and 17-year-old girls are attending Hearing loss may occur with a decibel level in this
. a concert. The music at the concert will be 80 to range.
90 dB. What should the girls be aware of? Correct
Explanation:
a) Hearing loss may occur with a decibel level in Sound louder than 80 dB is perceived by the human ear
this range. to be harsh and can be damaging to the inner ear. Ear
b) Sounds in this decibel level are not perceived protection or plugs do help to minimize the effects of
to be harsh to the ear. high decibel levels.
c) Hearing will not be affected by a decibel level
in this range.
d) Ear plugs will have no affect on decibel levels.
561 A group of nursing students are studying for a Bicarbonate-carbonic acid buffer system - The major
. test over acid-base imbalance. One student asks chemical regulator of plasma pH is the bicarbonate-
another what the major chemical regulator of carbonic acid buffer system.
plasma pH is. What should the second student
respond?
562 A group of nursing students are studying for a Bicarbonate-carbonic acid buffer system - The major
. test over acid-base imbalance. One student asks chemical regulator of plasma pH is the bicarbonate-
another what the major chemical regulator of carbonic acid buffer system. Therefore options A and C
plasma pH is. What should the second student are incorrect. Option D does not exist, it is only a
respond? distractor for this question.
563 A group of students are reviewing material about Epinephrine
. endocrine system function. The students Explanation: The adrenal medulla secretes epinephrine
demonstrate understanding of the information and norepinephrine. The adrenal cortex manufactures
when they identify which of the following as and secretes glucocorticoids, mineralocorticoids, and
secreted by the adrenal medulla? small amounts of androgenic sex hormones. Glucagon
is released by the pancreas.
564 A group of students are reviewing the phases of Oliguria
. acute renal failure. The students demonstrate
understanding of the material when they identify
which of the following as occurring during the
second phase?
565 A group of students is reviewing the Prilosec
. medications that may be used to treat
esophageal reflux. The students
demonstrate understanding of the
information when they identify which of
the following as an example of a proton-
pump inhibitor?
566 Gynecomastia is a common side effect of Spironolactone (Aldactone)
. which of the following diuretics?
567 Halitosis and a sour taste in the mouth are esophageal diverticula.
. clinical manifestations associated most
directly with
568 Hearing aids help with which of the Makes sounds louder
. following problems? Explanation:
A hearing aid makes sounds louder, but it does not improve a
a) Improves discrimination of words patient's ability to discriminate words or understand speech.
b) Makes sounds louder Hearing aids amplify all sounds, including background noise,
c) Improves understanding of speech which may be disturbing to the wearer. It does not improve
d) Improves communication skills communication skills.
569 Heart block Identify which of the following as an age-related change
. associated with conduction system of the heart?
570 herpes zoster infection aka shingles-- a viral disease of spinal ganglia-- is a
. dermatomally distributed skin lesion. Virus invades a spinal
ganglion and is transported along the axon to the skin, where
it produces an infection that causes a sharp burning pain in the
dermatome supplied by the involved nerve. A few days later,
the skin of the dermatome becomes red and vesicular
eruptions appear.
571 Hickman and Groshong are examples of Tunneled central catheters
. which type of central venous access
devices?
572 High doses of this medication can produce Aspirin
. bilateral tinnitus? Correct
Explanation:
a) Aspirin At high doses, aspirin toxicity can produce bilateral tinnitus.
b) Dramamine Antivert and Dramamine is used for nausea and vomiting
c) Promethazine related to motion sickness. Antiemetics such as promethazine
d) Antivert (Phenergan) suppositories help control the nausea and
vomiting and the vertigo because of the antihistamine effect.
573 High LDL Levels Jack Donohue, a 62-year-old stock broker, attends his annual
. physical appointment and indicates physical changes since his
last examination. He reports chest pain and palpitation during
and after his morning jogs. Jack's family history reveals
includes coronary artery disease. His lipid profile reveals his
LDL level to be 122 mg/dl. Which of the following correctly
states the Jack's condition?
574 High or increased compliance occurs in Emphysema
. which disease process?
575 Hippocampus Susan cannot remember anything before her accident
. yesterday. Which brain structure might be injured?
576 Hippocampus part of the limbic system, Susan cannot remember anything before her accident
. controls emotions, memory, and learning. yesterday. Which brain structure might be injured?
It is also thought to mediate feelings of
aggression, sexual impulses, and
submissive behavior.
577 HIV is harbored within which type of cell? Lymphocyte
.

578 A home care nurse is caring for a client with "I'll avoid eating or drinking anything 6 to 8 hours
. complaints of epigastric discomfort who is before the test."
scheduled for a barium swallow. Which statement
by the client indicates an understanding of the
test?
579 A home care nurse is visiting a client with AIDS at Cleaning around the anal area without wearing gloves
. home. During the visit, the nurse observes the
caregiver providing care. Which of the following
would alert the nurse to the need for additional
teaching for the caregiver?
580 A home care nurse visits a client with chronic b) "I make sure my oxygen mask is on tightly so it
. obstructive pulmonary disease who requires won't fall off while I nap."
oxygen. Which statement by the client indicates the
need for additional teaching about home oxygen
use?

a) "I lubricate my lips and nose with K-Y jelly."


b) "I make sure my oxygen mask is on tightly so it
won't fall off while I nap."
c) "I have a 'no smoking' sign posted at my front
entry-way to remind guests not to smoke."
d) "I clean my mask with water after every meal."
581 A home health nurse is visiting a home care client a) Hypoxia
. with advanced lung cancer. Upon assessing the
client, the nurse discovers wheezing, bradycardia,
and a respiratory rate of 10 breaths/min. These
signs are associated with which condition?

a) Hypoxia
b) Delirium
c) Hyperventilation
d) Semiconsciousness
582 The home health nurse sees a client with end-stage a) Decreased oxygen requirements
. chronic obstructive pulmonary disease. An
outcome identified for this client is preventing
infection. Which finding indicates that this
outcome has been met?

a) Decreased oxygen requirements


b) Increased sputum production
c) Decreased activity tolerance
d) Normothermia
583 A home health nurse who sees a client with "I should increase my intake of fresh fruits and
. diverticulitis is evaluating teaching about dietary vegetables during remissions."
modifications necessary to prevent future episodes.
Which statement by the client indicates effective
teaching?
584 The human body is an intricate mechanism which Acids - Acids are substances that release hydrogen
. maintains homeostasis through a multitude of into fluid. The delicate balance of fluid, electrolytes,
chemical reactions. The measureable chemical acids, and bases is ensured by an adequate intake of
levels disclose how well the body is (or is not) water and nutrients, physiologic mechanisms that
functioning. Which of these chemical substances regulate fluid volume, and chemical processes that
release hydrogen into fluid? buffer the blood to keep its pH nearly neutral.
585 Hyperbaric oxygen therapy increases the blood's Compromised skin graft
. capacity to carry and deliver oxygen to
compromised tissues. Which condition would
benefit from hyperbaric oxygen therapy?
586 Hypercarbia? (other name?) An increased concentration of carbon dioxide in the
. - Caused by? blood.
(hypercapnia)

- Hypoventilation
587 Hyperreflexia A 36-year-old patient has been receiving a selective
. serotonin reuptake inhibitor for treatment of depression. She
is exhibiting manifestations of serotonin syndrome. The
nurse should be aware of which of the following symptoms
of this syndrome?
588 Hypoxemia? Insufficient oxygenation of arterial blood/
.

589 hypoxia. Clubbing is a sign of prolonged hypoxia. Causes of clubbing


. include emphysema, chronic bronchitis, lung cancer, and
heart failure. Beau's lines (transverse depressions in the nail
that extend beyond the nail bed) occur with acute illness,
malnutrition, and anemia. Koilonychia (thin, spoon-shaped
nails with lateral edges that tilt upward) is associated with
Raynaud's disease, malnutrition, chronic infections, and
hypochromic anemia. Onycholysis (loosening of the nail
plate with separation from the nail bed) is associated with
hyperthyroidism, psoriasis, contact dermatitis, and
Pseudomonas infections.
590 Ibuprofen (Motrin) has which effect on the Neutropenia
. immune system?
591 If a client's central venous catheter Clamp the catheter
. accidentally becomes disconnected, what
should a nurse do first?
592 If an indwelling catheter is necessary, the performing meticulous perineal care daily with soap and
. nursing interventions that should be water.
implemented to prevent infection include
593 If concern exists about fluid accumulation in Bilateral lower lobes
. a client's lungs, what area of the lungs will
the nurse focus on during assessment?
594 Immunoglobulins (also known as antibodies) (1) neutralizing their toxins; (2) linking them together in a
. promote the destruction of invading cells in process called agglutination and (3) causing them to
various ways, using different mechanisms. precipitate, or become solid. Second, antibodies can
Which of the following mechanisms is used facilitate the destruction of antigens with other mechanisms
by immunoglobulins to destroy pathogenic
antigens?
595 Impassive A client who is schizophrenic is catatonic and has a
. masklike face. Which of the following facial expressions is
being exhibited?
596 In a client who has been burned, which Mafenide acetate (Sulfamylon)
. medication should the nurse expect to use to
prevent infection?
597 In a patient diagnosed with increased Bradypnea
. intracranial pressure (IICP), the nurse
would expect to observe which of the
following respiratory rate or depth?
598 An inappropriate nursing action Moving the client swiftly
. implemented to keep the client safe includes:
599 In a typical spinal cord, it functions as a Between the first and second lumbar vertebrae
. "highway" for sensory and descending
motor neurons - to provide conduction of
impulses to and from the brain. The spinal
cord is surrounded and protected by bony
vertebrae. Where does the spinal cord end?
600 The incidence of hospital-acquired risk manager.
. pressure ulcers on the medical-surgical
unit has increased. A nurse should inform
the:
601 Increased appetite and thirst may Dysfunction of the pancreatic islet cells
. indicate that a client with chronic
pancreatitis has developed diabetes
melitus. Which of the following explains
the cause of this secondary diabetes?
602 An increased risk of falls is dangerous for A patient with vertigo
. any patient. What patient would be at an Explanation:
increased risk of falls? Vertigo is defined as the misperception or illusion of motion
either of the person or the surroundings. A patient suffering
from vertigo will be at an increased risk of falls. This makes
options A, B, and D incorrect.
603 In developing the plan of care for the the client's cultural beliefs
. intraoperative client, the nurse recognizes
that it is essential to consider:
604 Ineffective peripheral tissue perfusion Ineffective peripheral tissue perfusion related to venous
. related to venous congestion congestion takes highest priority because venous inflammation
and clot formation impede blood flow in a client with deep vein
thrombosis. Option 1 is incorrect because impaired gas
exchange is related to decreased, not increased, blood flow.
Option 2 is inappropriate because no evidence suggests that this
client has a excessive fluid volume. Option 3 may be warranted
but is secondary to ineffective tissue perfusion.
605 An infant, age 3 months, undergoes Bulb syringe with tubing
. surgical repair of a cleft lip. After Explanation: An infant with a surgically repaired cleft lip must
surgery, the nurse should use which be fed with a bulb syringe with tubing or Breck feeder to
equipment to feed the infant? prevent sucking or suture line trauma. The other options
wouldn't prevent these actions.
606 An infant, age 8 months, has a tentative Sitting in an infant seat
. diagnosis of congenital heart disease. Explanation: Because the infant's data collection findings
During data collection, the nurse suggest that respiratory distress is developing, the nurse should
measures a heart rate of 170 beats/minute position the infant with the head elevated at a 45-degree angle
and a respiratory rate of 70 to promote maximum chest expansion; an infant seat maintains
breaths/minute. How should the nurse this position. Placing an infant flat on his back or abdomen or
position the infant? in high Fowler's position could increase respiratory distress by
preventing maximal chest expansion.
607 Infants with DiGeorge syndrome have Hypoparathyroidism
. which type of endocrine disorder?
608 Infarction? Death of tissue from deprivation of its blood supply.
.

609 Initially, which diagnostic should be x-ray


. completed following placement of a NG
tube?
610 In order to be effective, percutaneous 60 minutes
. transluminal coronary angioplasty
(PTCA) must be performed within what
time frame, beginning with arrival at the
emergency department after diagnosis of
myocardial infarction?
611. In preparing the client for transfer to the Allow the client to wear dentures.
operating room, which of the following
actions by the nurse is inappropriate?
612 In presenting a workshop on Hemorrhage, sepsis, and anaphylaxis
. parameters of cardiac function, which Explanation:
conditions should a nurse list as those Preload is the volume in the left ventricle at the end of diastole. It's
most likely to lead to a decrease in also referred to as end-diastolic volume. Preload is reduced by any
preload? condition that reduces circulating volume, such as hemorrhage,
sepsis, and anaphylaxis. Hemorrhage reduces circulating volume
by loss of volume from the intravascular space. Sepsis and
anaphylaxis reduce circulating volume by increased capillary
permeability. Diuresis, vasodilation, and third spacing also reduce
preload. Preload increases with fluid overload and heart failure.
613 INSIDE THE OPERATING ROOM: - Bacteria in the air
. Unnecessary personnel and physical
movement may be restricted to
minimize _____.
614 Inspiratory and expiratory stridor c) has aspirated a piece of meat.
. may be heard in a client who:

a) is experiencing an exacerbation of
goiter.
b) is experiencing an acute asthmatic
attack.
c) has aspirated a piece of meat.
d) has severe laryngotracheitis.
615 An instructor asks students 7.5 days
. approximately how long platelets last?
What would the students correctly
identify?
616 The instructor in the anatomy and Albumin
. physiology class is discussing the
components of the blood. What would
the instructor cite as the most
abundant protein in plasma?
617 The instructor in the anatomy and Determines amount of CO2 in the body
. physiology class is talking about
alveolar respiration. What would the
instructor tell the class is the main
purpose of alveolar respiration?
618 The instructor of the pre-nursing To exchange oxygen and CO2 between the atmospheric air and the
. physiology class is explaining blood and between the blood and the cells
respiration to the class. What does the
instructor explain is the main function
of respiration?
619 Instruct the patient to restrict food Which of the following nursing interventions is required to prepare
. and oral intake. a patient with cardiac dysrhythmia for an elective electrical
cardioversion?
620 In the client with burns on the legs, Applying knee splints
. which nursing intervention helps Explanation: Applying knee splints prevents leg contractures by
prevent contractures? holding the joints in a position of function. Elevating the foot of
the bed can't prevent contractures because this action doesn't hold
the joints in a position of function. Hyperextending a body part for
an extended time is inappropriate because it can cause
contractures. Performing shoulder range-of-motion exercises can
prevent contractures in the shoulders, but not in the legs.
621 In the client with burns on the legs, Applying knee splints
. which nursing intervention helps
prevent contractures?
622 In the divisions of the nervous systems, the basic Dendrites
. structure is the neuron. The function of the neuron is
determined by the direction of impulse transmission.
Which part of the neuron is responsible for conducting
impulses to the cell body?
623 In the immediate postoperative period, vital signs are 15 minutes.
. taken at least every:
624 In the interest of public health, the CDC has developed Sexual activity
. HIV Transmission Prevention strategies. The strategies Illegal drugs
address the routes that HIV can be transmitted and
steps that can be taken to reduce or eliminate
transmission. Which categories of risk are addressed by
these strategies? Choose all correct options.
625 Intraosseously fluid admin - good if cannot obtain IV access
. (small animals). Also can be given at a relatively
fast speed. acceptable for dehydration and shock
626 In what location would the nurse palpate for the liver? Right upper quadrant
.

627 In which instance may a surgeon operate without Emergency situations


. informed consent?
628 In which of the following medical conditions would Heart failure
. administering IV normal saline solution be Pulmonary edema
inappropriate? Select all that apply. Renal impairment
Normal saline is not used for heart failure,
pulmonary edema, renal impairment, or sodium
retention. It is used with administration of blood
transfusions and to replace large sodium losses,
as in burn injuries.
629 In which of the following medical conditions would Heart Failure, Pulmonary Edema, Renal
. administering IV normal saline solution be Impairment - Normal saline is not used for heart
inappropriate? Select all that apply. failure, pulmonary edema, renal impairment, or
Severe hemorrhage sodium retention. It is used with administration
Pulmonary edema of blood transfusions and to replace large
Heart failure sodium losses, as in burn injuries.
Burns
Renal Impairment
630 In which position should the patient be placed for a sitting on the edge of bed
. thoracentesis?
631 In which zone of the surgical area are street clothes Unrestricted
. allowed?
632 In women, which of the following types of cancer Breast
. exceeds colorectal cancer?
633 is high in fruits and vegetables, moderate in low-fat DASH diet
. dairy products, and low in animal protein.
634 It involves changing one's values or beliefs. Which of the following is an inaccurate
. depiction of self-awareness?
635 It is important for the nurse to assist a postsurgical avoid aspiration.
. client to sit up and turn his or her head to one side when
vomiting in order to
636 Knowing respiratory physiology is important to Trachea
. understand how the disease process can work within
that system. Which hollow tube transports air from the
laryngeal pharnyx to the bronchi?
637 Laboratory test results confirm that a client's wound is Contact
. infected with methicillin-resistant staphylococcus
aureus. Which type of isolation precautions should the
nurse institute for this client?
638 Lactic acidosis The nurse should monitor the client for signs
. of lactic acidosis, a life- threatening adverse
reaction associated with metformin. Nausea,
vomiting, and megaloblastic anemia are
adverse reactions associated with metformin,
but they aren't considered life-threatening.
639 Lactulose (Cephulac) is administered to a patient Ammonia
. diagnosed with hepatic encephalopathy to reduce which of
the following?
640 Laser risk? -Electrical
.

641 Left atrium Within the physiology of the heart, each


. chamber has a particular role in maintaining
cellular oxygenation. Which chamber of the
heart is responsible for receiving oxygenated
blood from the lungs?
642 Lesions in the temporal lobe may result in which of the Auditory
. following types of agnosia?
643 - Liver and kidney in elderly patients also can't ____. - Metabolized medication and anesthetic
. agents well.
644 Local anesthesia? - Anesthesia injected into the tissues at the
. planned incision site!
645 A longitudinal tear or ulceration in the lining of the anal Anal fissure
. canal is termed a (an)
646 Low blood pressure A patient is receiving anticoagulant therapy.
. The nurse should be alert to potential signs
and symptoms of external or internal
bleeding, as evidenced by which of the
following?
647 Lower doses of anesthetic agents are required in elderly 1) Decreased tissue elasticity in the lung and
. patiens due to (2 things) cardiovascular system.

2) Reduced tissue mass


648 Lower motor neuron lesions cause flaccid muscle paralysis.
.

649 The lower the patient's viral load, The longer the survival time
.

650 Low levels of the neurotransmitter serotonin lead to which Depression


. of the following disease processes?
651 Macrophages attack and destroy foreign substances to the In the lymph node
. body. Where does this action occur?
652 A majority of patients with CVID develop which type of Pernicious
. anemia?
653 The majority of patient with primary immunodeficiency < 20 YO
. are in which age group?
654 A major manifestation of Wiskott-Aldrich syndrome Thrombocytopenia
. includes which of the following?
655 A male client has doubts about performing peritoneal Wear a mask when performing exchanges
. dialysis at home. He informs the nurse about his existing
upper respiratory infection. Which of the following
suggestions can the nurse offer to the client while
performing an at-home peritoneal dialysis?
656 A male client is scheduled for an electroencephalogram Inform the client that he will not experience
. (EEG). When the nurse caring for the client is preparing any electrical shock.
him for the test, the client states that during childhood he
was mildly electrocuted but miraculously lived. Therefore,
he is quite afraid of going through an EEG. In what ways
can the nurse help dispel the client's fear regarding the
test?
657 Malignant hyperthermia? - rare life threatening condition triggered by exposure of
. most anesthetic agents.
- What happens?
- Increase in skeletal muscle oxidative metabolism that
- Can eventually cause? can overwhelm the body's capacity to supply oxygen,
remove carbon dioxide and regulate body temperature.
- How/ why does this happens?
- DEATH

- It is believed to be inherited as an autosomal dominant


disorder!
658 mantoux test Intradermal test to determine tuberculin sensitivity based
. on a positive reaction where the area around the test site
becomes red and swollen
659 "Mental health care services are inadequate and A nurse is preparing a presentation about the current
. fragmented." status of mental health services in the United States.
Which statement would the nurse include as the most
reflective of this status?
660 "Mental health is marked by productivity, A nurse is giving a presentation about preventing mental
. fulfilling relationships, and adaptability." illness to college freshmen. A student asks, "What does
it mean to be mentally healthy?" Which of the following
potential responses by the nurse is best?
661 middle-old Ages 75-84
.
662 Milieu Therapy Involved clients' interactions with one another, including
. practicing interpersonal relationship skills, giving one
another feedback about behavior, and working
cooperatively as a group to solve day-to-day problems.
663 The mode of transmission of hepatitis A virus Fecal-oral
. (HAV) includes which of the following?
664 Moderate sedation? - Conscious sedation -
. - It is used so that patient can still ___by
themselves and respond to ___stimuli and Breath
___commands. Physical
Verbal
665 Monitored anesthesia care? - Moderate sedation
.

666 Morbid obesity is defined as being how many 100 lbs


. pounds over the person's ideal body weight?
667 More than 50% of individuals with this disease Common variable immunodeficiency (CVID)
. develop pernicious anemia:
668 More than 50% of individuals with this disease CVID
. develop pernicious anemia:
669 The most common symptom of esophageal Dysphagia
. disease is
670 A mother and grandmother bring a 3-month-old "We've found that babies can't digest solid food properly
. infant to the well-baby clinic for a routine until they're 4 months old."
checkup. As the nurse weighs the infant, the Explanation: Infants younger than 4 months lack the
grandmother asks, "Shouldn't the baby start enzymes needed to digest complex carbohydrates.
eating solid food? My kids started on cereal Option 1 doesn't address the grandmother's question
when they were 2 weeks old." Which response directly. Option 2 is a clich that may block further
by the nurse would be appropriate? communication with the grandmother. Option 4 is
incorrect because no evidence suggests that introducing
solid food early causes eating disorders.
671 A mother brings her teenage son to the clinic, where Infection at school
. tests show that he has hepatitis A virus (HAV). They Suboptimal sanitary habits
ask the nurse how this could have happened. Which Consumption of sewage-contaminated water or
of the following explanations would the nurse shellfish
correctly identify as possible causes? Select all that Sexual activity
apply.
672 A mother brings her young child to the clinic for an Ten ear infections in the past year
. evaluation of an infection. The mother states, "He's
been taking antibiotics now for more than 2 months
and still doesn't seem any better. It's like he's
always sick." During the history and physical
examination, which of the following would alert the
nurse to suspect a primary immunodeficiency?
673 A mother has brought her child to the clinic for a Moderate diet that is balanced and varied
. wellness check. While talking with the nurse, the
mother asks the nurse to suggest a diet that will
maximize the immune function of her growing
children. What dietary pattern should the nurse
suggest?
674 The mother of a 3-year-old with a Folic acid to 4 mg/day
. myelomeningocele is thinking about having another Explanation: The American Academy of Pediatrics
baby. The nurse should inform the woman that she recommends that a woman who has had a child with
should increase her intake of which acid? a neural tube defect increase her intake of folic acid
to 4 mg per day 1 month before becoming pregnant
and continue this regimen through the first trimester.
A woman who has no family history of neural tube
defects should take 0.4 mg/day. All women of
childbearing age should be encouraged to take a
folic acid supplement because the majority of
pregnancies in the United States are unplanned.
Ascorbic acid hasn't been shown to have any effect
on preventing neural tube defects.
675 The mother of a preschooler recently diagnosed Measure the child's blood glucose level.
. with type 1 diabetes mellitus makes an urgent call Explanation: In a child with type 1 diabetes mellitus,
to the pediatrician's office. She says her child had behavioral changes may signal either hypoglycemia
an uncontrollable temper tantrum while playing or hyperglycemia; measuring the blood glucose level
and now is lethargic and hard to arouse. The nurse is the only way to determine which condition is
should instruct the mother to take which action present. Urine glucose measurement doesn't
first? accurately reflect the current blood glucose level.
Forcing a lethargic child to drink fluids could cause
aspiration. After measuring the child's blood glucose
level, the mother may need to take additional
emergency measures such as administering insulin
or a simple glucose source. If the child doesn't
respond to these measures, the mother may need to
call for emergency help.
676 A mother reports that her 6-year-old girl recently Assess the mother's understanding of UTI and its
. started wetting the bed and running a low-grade causes
fever. A urinalysis is positive for bacteria and Instruct the mother to administer the antibiotic as
protein. A diagnosis of a urinary tract infection prescribedeven if the symptoms diminish
(UTI) is made, and the child is prescribed Discourage taking bubble baths
antibiotics. Which interven-tions are appropriate? Explanation: Assessing the mother's understanding
Select all that apply: of UTI and its causes provides the nurse with a
baseline for teaching. The full course of antibiotics
must be given to eradicate the organism and prevent
recurrence, even if the child's signs and symptoms
de-crease. Bubble baths can irritate the vulva and
urethra and contribute to the development of a UTI.
Fluids should be encouraged, not limited, in order to
prevent urinary stasis and help flush the organism
out of the urinary tract. Instructions should be given
to the child at her level of understanding to help her
better understand the treatment and promote
compliance. The child should wipe from the front to
the back, not back to front, to minimize the risk of
contamination after elimination.
677 Mr. Sam Wallace, a 53-year-old male, is a regular client in Type II Cells
. the respiratory group where you practice nursing. As with
all adults, millions of alveoli form most of the pulmonary
mass. The squamous epithelial cells lining each alveolus
consist of different types of cells. Which type of the alveoli
cells produce surfactant?
678 Muscular rigidity, tremors, and difficulty swallowing. A comprehensive nursing assessment for
. NMS would necessarily include what?
679 A neonate born 18 hours ago with myelomeningocele over Preventing infection
. the lumbosacral region is scheduled for corrective surgery. Explanation: Preventing infection is the
Preoperatively, what is the most important nursing goal? nurse's primary preoperative goal for a
neonate with myelomeningocele. Although
the other options are relevant for this neonate,
they're secondary to preventing infection.
680 A newborn has been diagnosed with DiGeorge syndrome. Hypercalcemia
. Which of the following would the nurse least likely expect
to assess?
681 A new client has been admitted with right-sided heart Jugular vein distention
. failure. The nurse knows to look for which of the following
assessment findings when assessing this client?
682 A new nurse auscultates adventitious breath sounds but is Instructed the client to hold the breath
. not sure what to document and confers with an
experienced nurse. This experienced nurse documents a
pleural friction rub. Which of the following did the
experienced nurse do during her assessment to identify the
rub?
683 A new surgical patient who has undergone a coronary Hypotension
. artery bypass graft (CABG) is receiving opioids for pain
control. The nurse must be alert to adverse effects of
opioids. Which of the following effects would be important
for the nurse to document?
684 A nonresponsive client has a nasogastric tube to low Auscultate lungs every 4 hours
. intermittent suction due to gastrointestinal bleeding. It is
most important for the nurse to
685 A nonverbal client has just finished undergoing a Assess for a cough reflex.
. bronchoscopy procedure and writes that he want to eat
lunch now. Which intervention is necessary for the nurse
to complete at this time?
686 Normally, approximately what percentage of the blood 2%
. pumped by the right ventricle does not perfuse the
alveolar capillaries?
687 Not every structure in the upper airway has a purpose in Pharyngeal tonsils
. respiration. There are some structures whose role is Palantine tonsils
immunological. Which of the following structures protect
against infection? Choose all correct responses.
688 Notify the nursing supervisor and approach the Approaching the person and requesting the client's
. individual. medical record isn't sufficient considering the
confidential health care information. Notifying the
nursing supervisor, then approaching the individual
before informing the client provides the most
appropriate approach to this breech of client
confidentiality. Contacting security might not be
warranted unless the nurse learns the reason the
unauthorized individual was reading the client's chart.
The nurse should also document the incident
according to facility policy.
689 The nurse accompanies a client to an exercise Cardiac catheterization
. stress test. The client can achieve the "target heart Explanation:
rate," but the ECG leads show an ST-segment An elevated ST-segment means an evolving
elevation. The nurse recognizes this as a "positive" myocardial infarction. A cardiac catheterization would
stress test, and will begin to prepare the client for be the logical next step
which of the following procedures?
690 A nurse administered a full strength feeding with Consult physician regarding decreasing to half-
. an increased osmolality through a jejunostomy strength
tube to a client. Immediately following the feeding,
the client expelled a large amount of liquid brown
stool and exhibited a blood pressure of 86/58 and
pulse rate of 112 beats/min. The nurse
691 The nurse administers albuterol (Proventil), as a) Respiratory rate of 22 breaths/minute
. prescribed, to a client with emphysema. Which
finding indicates that the drug is producing a
therapeutic effect?

a) Respiratory rate of 22 breaths/minute


b) Dilated and reactive pupils
c) Urine output of 40 ml/hour
d) Heart rate of 100 beats/minute
692 A nurse and a nursing student are caring for a "Ask the client to hold the breath while you
. client with pericarditis and perform the physical auscultate; the pericardial friction rub will continue,
assessment together. The client has a pericardial while the pleural friction rub will stop."
friction rub audible on auscultation. When the
nurse and student leave the room, the student asks
how to distinguish a pericardial from a pleural
friction rub. The nurse's best response is which of
the following?
693 A nurse and nursing student are caring for a client "The blood will seal the hole in the dura and prevent
. recovering from a lumbar puncture yesterday. The further loss of cerebral spinal fluid."
client reports a headache despite being on bedrest
overnight. The physician plans an epidural blood
patch this morning. The student asks how this will
help the headache. The correct reply from the
nurse is which of the following?
694 The nurse assesses a client for evidence of an increased pulse rate, increased respiratory rate, and
. postpartum hemorrhage during the third stage of decreased blood pressure.
labor. Early signs of this postpartum complication Explanation:
include: An increased pulse rate followed by an increased
respiratory rate and decreased blood pressure may be
the first signs of postpartum hemorrhage and
hypovolemic shock.
695 A nurse assesses a client shortly after living donor Urinary output of 20mL/hr
. kidney transplant surgery. Which postoperative
finding must the nurse report to the physician
immediately?
696 A nurse assesses a client's respiratory status. Use of accessory muscles
. Which observation indicates that the client is
having difficulty breathing?
697 A nurse assesses the abdomen of a newly admitted Asymmetrical upper abdomen
. client. Which finding would necessitate further
investigation?
698 The nurse assessing a client for tracheal d) contralateral side in a hemothorax.
. displacement should know that the trachea will
deviate toward the:

a) contralateral side in a simple pneumothorax.


b) affected side in a hemothorax.
c) affected side in a tension pneumothorax.
d) contralateral side in a hemothorax.
699 A nurse assessing a client who underwent cardiac Contact the physician and report her findings.
. catheterization finds the client lying flat on the Explanation:
bed. His temperature is 99.8 F (37.7 C). His blood The client is probably developing a hematoma at the
pressure is 104/68 mm Hg. His pulse rate is 76 puncture site. The decreased pulses, swelling, and
beats/minute. She detects weak pulses in the leg cool temperature in the leg are all classic signs that
distal to the puncture site. Skin on the leg is cool to blood flow to that extremity is compromised. The
the touch. The puncture site is dry, but swollen. nurse should notify the physician immediately to
What is the most appropriate action for the nurse preserve the blood flow in the client's leg.
to take? Documenting findings and checking the client again
in 1 hour, slowing the I.V. fluid, and encouraging the
client to perform isometric leg exercises aren't
appropriate actions for the nurse to take at this time.
700 The nurse auscultates lung sounds that are harsh pleural friction rub.
. and cracking, sounding like two pieces of leather
being rubbed together. The nurse would be correct
in documenting this finding as
701 A nurse caring for a client with deep vein d) Chest pain and dyspnea
. thrombosis must be especially alert for
complications such as pulmonary embolism. Which
findings suggest pulmonary embolism?

a) Nonproductive cough and abdominal pain


b) Hypertension and lack of fever
c) Bradypnea and bradycardia
d) Chest pain and dyspnea
702 The nurse caring for a client with small-bowel Administering I.V. fluids
. obstruction would plan to implement which Explanation: I.V. infusions containing normal saline
nursing intervention first? solution and potassium should be given first to
maintain fluid and electrolyte balance. For the client's
comfort and to assist in bowel decompression, the
nurse should prepare to insert an NG tube next. A
blood sample is then obtained for laboratory studies
to aid in the diagnosis of bowel obstruction and guide
treatment. Blood studies usually include a complete
blood count, serum electrolyte levels, and blood urea
nitrogen level. Pain medication often is withheld until
obstruction is diagnosed because analgesics can
decrease intestinal motility.
703 The nurse caring for the client with acute renal lanthanum carbonate (Fosrenol)
. failure would question which of the following for
the treatment of hyperkalemia?
704 The nurse-client relationship Two major clinical characteristics affect client
. compliance: the nurse-client relationship and the
therapeutic regimen. The client's drug knowledge,
psychosocial factors, and disease duration and
severity are client characteristics, not clinical ones.
705 A nurse correctly identifies a urine Acidic - Normal urine pH is 4.5 to 8.0; a value of 4.3 reveals
. specimen with a pH of 4.3 as being which acidic urine pH. A pH above 7.0 is considered an alkaline or
type of solution? basic solution. A pH of 7.0 is considered neutral.
706 A nurse detects bilateral crackles when b) They're usually heard on inspiration and sometimes clear
. auscultating a client's lungs. Which with a cough.
statement about crackles is true?
a) They're usually heard on expiration and
may clear with a cough.
b) They're usually heard on inspiration and
sometimes clear with a cough.
c) They're hissing or musical and are
usually heard on inspiration and expiration;
if severe, they may be heard without a
stethoscope.
d) They're creaking and grating and are
usually heard over the problem area on
both inspiration and expiration.
707 The nurse determines that teaching for the "I have learned some relaxation strategies that decrease my
. client with peptic ulcer disease has been stress."
effective when the client states:
708 A nurse discovers that a stat dose of Notify the charge nurse so she can notify the physician of the
. potassium chloride that was prescribed by missed dose.
the physician was never administered. Explanation: An error was made that needs to be addressed
Which action should the nurse take? by notifying the charge nurse. The charge nurse should then
notify the physician to determine if the medication is still
appropriate for the client, and then request the medication
from the pharmacy if it's still needed. The physician might
order a potassium level to see if the dose is sufficient for the
client. It isn't appropriate to ask the client if the medication is
still needed. After the charge nurse and physician have been
notified, the nurse should document the incident according to
facility policy.
709 The nurse documents the presence of a scab Migratory
. on a client's deep wound. The nurse
identifies this as which phase of wound
healing?
710 A nurse educator is preparing to discuss AIDs
. immunodeficiency disorders with a group of
fellow nurses. What would the nurse
identify as the most common secondary
immunodeficiency disorder?
711. A nurse enters a client's room and observes "I coughed that up about 8 hours ago."
a container with sputum. Upon questioning
about the specimen, which of the following
items of information from the client would
necessitate the nurse to obtain a new
specimen?
712 A nurse enters the room of a client who has The client lying in a lateral position, with the head of bed flat
. returned to the unit after having a radical
neck dissection. Which assessment finding
requires immediate intervention?
713 The nurse enters the room of a client who is Diagnosis of peripheral vascular disease
. being monitored with pulse oximetry. Which of
the following factors may alter the oximetry
results?
714 The nurse expects which of the following Dehydration
. assessment findings in the client in the diuretic
phase of acute renal failure?
715 A nurse explains to a client with thyroid disease T3, thyroxine (T4), and calcitonin.
. that the thyroid gland normally produces: Explanation: The thyroid gland normally produces
thyroid hormone (T3 and T4) and calcitonin. The
pituitary gland produces TSH to regulate the thyroid
gland. The hypothalamus gland produces TRH to
regulate the pituitary gland.
716 A nurse has admitted a client suspected of having Edema and inflammation
. acute pancreatitis. The nurse knows that mild
acute pancreatitis is characterized by:
717 The nurse has just admitted a 12-year-old client Client will have a shorter recovery period.
. who is going to have an above-the-knee
amputation of their left leg due to osteosarcoma.
The nurse knows that adequate preoperative
teaching and learning is important for what
reason?

The nurse has just admitted a 12-year-old client


who is going to have an above-the-knee
amputation of their left leg due to osteosarcoma.
The nurse knows that adequate preoperative
teaching and learning is important for what
reason?
718 The nurse identifies a nursing diagnosis of Assist with chest physiotherapy every 4-6 hours
. ineffective airway clearance related to
pneumocystis pneumonia and increased
bronchial secretions for a client with AIDS.
Which of the following would be appropriate for
the nurse to include in the client's plan of care?
719 The nurse inserts a nasoduodenal tube for Abdominal x-ray
. feeding of the client. To check best for placement,
the nurse
720 The nurse in the emergency department is caring External otitis is characterized by pain when the pinna
. for a 4 year-old brought in by his parents with of the ear is pulled.
complaints that the child will not stop crying and Explanation:
pulling at his ear. Based upon information External otitis is an infection of the external ear. Pain
collected by the nurse, which of the following can be elicited when the pinna of the ear is pulled.
statements applies to a diagnosis of external Fever and accompanying upper respiratory infection
otitis? occur more commonly in conjunction with otitis media
(infection of the middle ear). Cotton-tipped applicators
a) External otitis is usually related to an upper can actually cause external otitis so their use should be
respiratory infection. avoided.
b) External otitis is characterized by pain when
the pinna of the ear is pulled.
c) External otitis is usually accompanied by a
high fever in children.
d) External otitis can be prevented by using
cotton-tipped applicators to clean the ear.
721 A nurse is administering lanoxin, which she The force of the contraction related to the status of the
. knows increases contractility as well as cardiac myocardium
output. Contractility refers to which of the
following?
722 The nurse is administering nitroglycerin, which The amount of blood presented to the ventricles just
. he knows decreases preload as well as afterload. before systole
Preload refers to which of the following?
723 The nurse is admitting a client to the unit with Vascular lesions caused by dilated blood vessels
. a diagnosis of ataxia-telangiectasia. A clinical
manifestation is telangiectasia. The nurse
would recognize that the client is exhibiting
telangiectasia when assessing the presence of
what?
724 The nurse is admitting a client who just had a Swallow reflex
. bronchoscopy. Which assessment should be the
nurse's priority?
725 A nurse is assessing a client and obtains the IBD
. following findings: abdominal discomfort, mild
diarrhea, blood pressure of 100/80 mm Hg,
pulse rate of 88 beats/minute, respiratory rate
of 20 breaths/minute, temperature 100 F (37.8
C). The nurse suspects the client will be
diagnosed with:
726 The nurse is assessing a client following "Yes, your abdomen may appear larger as a result of the
. laparoscopy. The client states that his stomach injection of carbon dioxide for visualization."
looks bloated and asks if this is normal. How
will the nurse respond?
727 The nurse is assessing a client for constipation. Review the client's usual pattern of elimination
. Which of the following is the first review that
the nurse should conduct in order to identify
the cause of constipation? Choose the correct
option.
728 The nurse is assessing a client who comes to the d) Dyspnea and wheezing
. clinic for care. Which findings in this client
suggest bacterial pneumonia?

a) Nonproductive cough and normal


temperature
b) Sore throat and abdominal pain
c) Hemoptysis and dysuria
d) Dyspnea and wheezing
729 A nurse is assessing a client with a family A client states he feels a lump in his throat
. history of cancer. Which finding requires
immediate follow-up?
730 A nurse is assessing a client with heart failure. press the right upper abdomen.
. To assess hepatojugular reflux, the nurse Explanation:
should: As the right upper abdomen (the area over the liver) is
compressed for 30 to 40 seconds, the nurse observes the
internal jugular vein. If the internal jugular vein becomes
distended, a client has positive hepatojugular reflux.
Hepatojugular reflux, a sign of right-sided heart failure,
is assessed with the head of the bed at a 45-degree, not
90-degree, angle.
731 The nurse is assessing a client with multiple pathologic bone fractures.
. myeloma. The nurse should keep in mind that Explanation: Clients with multiple myeloma are at risk
clients with multiple myeloma are at risk for: for pathologic bone fractures secondary to diffuse
osteoporosis and osteolytic lesions. Also, clients are at
risk for renal failure secondary to myeloma proteins by
causing renal tubular obstruction. Liver failure and heart
failure aren't usually sequelae of multiple myeloma.
Hypoxemia isn't usually related to multiple myeloma.
732 A nurse is assessing a client with syndrome of Jugular Vein Distension - Jugular vein distention requires
. inappropriate antidiuretic hormone. Which further action because this finding signals vascular fluid
finding requires further action? overload. Tetanic contractions aren't associated with this
disorder, but weight gain and fluid retention from
oliguria are. Polyuria is associated with diabetes
insipidus, which occurs with inadequate production of
antidiuretic hormone.
733 The nurse is assessing a patient for Usual pattern of elimination
. constipation. Which of the following is
the first review that the nurse should
conduct in order to identify the cause
of constipation?
734 The nurse is assessing the lungs of a Crackles at lung bases
. patient diagnosed with pulmonary
edema. Which of the following would
be expected upon auscultation?
735 The nurse is assigned to an adolescent. Fear related to disturbed body image
. Which nursing diagnosis is most Explanation: Fear related to disturbed body image is the most
appropriate for a hospitalized appropriate nursing diagnosis for a hospitalized adolescent because
adolescent? of the adolescent's developmental level and concern for physical
appearance. An adolescent may fear disfigurement resulting from
procedures and treatments. Separation is rarely a major stressor for
the adolescent, eliminating a diagnosis of Anxiety related to
separation from parents. Adolescents may have Fear related to the
unknown but typically ask questions if they want information. A
diagnosis of Ineffective coping related to activity restrictions may
be appropriate for a toddler who has difficulty tolerating activity
restrictions but is an unlikely nursing diagnosis for an adolescent.
736 A nurse is assisting during a lumbar Lateral recumbent, with chin resting on flexed knees
. puncture. How should the nurse
position the client for this procedure?
737 The nurse is assisting in developing a Blood pressure monitoring
. teaching plan for a child with acute Explanation: Because poststreptococcal glomerulonephritis may
poststreptococcal glomerulonephritis. cause severe, life- threatening hypertension, the nurse must teach
What is the most important point to the parents how to monitor the child's blood pressure. Infection
address in this plan? control, nutritional planning, and prevention of streptococcal
pharyngitis are important but are secondary to blood pressure
monitoring.
738 A nurse is assisting with a Ensure that the biopsy equipment is assembled and in order.
. percutaneous liver biopsy. Place the Help the client assume a supine position.
steps involved in care in the correct Place a rolled towel beneath the client's right lower ribs.
sequence from first to last. While the physician inserts the needle, instruct the client to take a
deep breath and hold it to keep the liver as near to the abdominal
wall as possible.
Make sure that the specimen container is labeled and delivered to
the laboratory.
739 A nurse is assisting with newborn Phenylketonuria
. screening. For which condition would Galactosemia
the nurse expect to screen? Select all
that apply.
740 A nurse is assisting with screening of Population screening
. specific ethnic groups to identify
possible cancer-predisposing genes.
The nurse is engaged in which of the
following?
741 The nurse is awaiting results of cardiac Troponin T and I
. biomarkers for a patient with severe chest pain. Explanation:
The nurse would identify which cardiac biomarker After myocardial injury, these biomarkers rise early
as remaining elevated the longest when myocardial (within 3 to 4 hours), peak in 4 to 24 hours, and
damage has occurred? remain elevated for 1 to 3 weeks. These early and
prolonged elevations may make very early diagnosis
of acute myocardial infarction (MI) possible and
allow for late diagnosis in patients who have delayed
seeking care for several days after the onset of acute
MI symptoms. CK-MB returns to normal within 3 to
4 days. Myoglobin returns to normal within 24 hours.
BNP is not considered a cardiac biomarker. It is a
neurohormone that responds to volume overload in
the heart by acting as a diuretic and vasodilator.
742 The nurse is aware that loss of consciousness General
. occurs with which type of anesthesia?
743 The nurse is aware that the amino acid, arginine, Stimulates T cell response
.

744 The nurse is aware that the most prevalent cause Malnutrition
. of immunodeficiency worldwide is
745 The nurse is aware that which of the following Vitamin K
. nutrients promotes normal blood clotting?
746 The nurse is caring for a 10-year-old child with Consulting with the social worker to help the family
. cystic fibrosis. The child's parents tell the nurse find appropriate resources
that they're having difficulty coping with their Explanation: The nurse can help this family by
child's disease. Which action would be most assisting them with finding appropriate financial,
appropriate for the nurse to take? psychological, and social support and by providing
referrals to the local community agencies and the
Cystic Fibrosis Foundation. The child should be
treated as much like a normal child as possible, and
he should be encouraged to make friends with other
children regardless of their physical condition. The
nurse shouldn't encourage the parents not to visit
because the child might feel abandoned.
747 The nurse is caring for a 16-year-old female client no because she isn't sexually active.
. who isn't sexually active. The client asks if she Explanation: A 16-year-old client who isn't sexually
needs a Papanicolaou (Pap) test. The nurse should active doesn't need a Pap test. When a client is
reply: sexually active or reaches age 18, a Pap test should be
performed.
748 A nurse is caring for a 17-year-old girl who's Directly into the superior vena cava
. receiving parenteral nutrition in 25% dextrose Explanation: Solutions that contain more than 12.5%
solution. How should this solution be dextrose are administered through a central venous
administered? access device directly into the superior vena cava by
way of the jugular or subclavian vein. Special tubing
is used that contains an in-line filter to remove
bacteria and particulate material. A superficial vein,
gastrostomy tube, and the oral route are never used
for this type of solution.
749 A nurse is caring for a 30-year-old client diagnosed The high risk for complications if she becomes
. with atrial fibrillation who has just had a mitral pregnant while taking warfarin
valve replacement. The client is being discharged
with prescribed warfarin (Comaudin). She
mentions to you that she relies on the rhythm
method for birth control. What education would be
a priority for the nurse to provide to this client?
750 A nurse is caring for a client admitted with a Respiratory Acidosis - Respiratory acidosis is always
. diagnosis of exacerbation of myasthenia gravis. from inadequate excretion of CO2 with inadequate
Upon assessment of the client, the nurse notes the ventilation, resulting in elevated plasma CO2
client has severely depressed respirations. The concentrations. Respiratory acidosis can occur in
nurse would expect to identify which acid-base diseases that impair respiratory muscles such as
disturbance? myasthenia gravis.
751 A nurse is caring for a client after a lung biopsy. Respiratory rate of 44 breaths/minute
. Which assessment finding requires immediate
intervention?
752 The nurse is caring for a client diagnosed with They can be heard during inspiration and expiration.
. asthma. While performing the shift assessment,
the nurse auscultates breath sounds including
sibilant wheezes, which are continuous musical
sounds. What characteristics describe sibilant
wheezes?
753 The nurse is caring for a client experiencing an b) the airways are so swollen that no air can get through.
. acute asthma attack. The client stops wheezing
and breath sounds aren't audible. This change
occurred because:

a) the attack is over.


b) the airways are so swollen that no air can get
through.
c) the swelling has decreased.
d) crackles have replaced wheezes.
754 A nurse is caring for a client experiencing Heart failure
. dyspnea, dependent edema, hepatomegaly,
crackles, and jugular vein distention. What
condition should the nurse suspect?
755 A nurse is caring for a client in acute renal Hyperkalemia - Hyperkalemia is a common complication
. failure. The nurse should expect hypertonic of acute renal failure. It's life-threatening if immediate
glucose, insulin infusions, and sodium action isn't taken to reverse it. Administering glucose and
bicarbonate to be used to treat: regular insulin, with sodium bicarbonate if necessary, can
temporarily prevent cardiac arrest by moving potassium
into the cells and temporarily reducing serum potassium
levels. Hypernatremia, hypokalemia, and hypercalcemia
don't usually occur with acute renal failure and aren't
treated with glucose, insulin, or sodium bicarbonate.
756 A nurse is caring for a client in the emergency Troponin of 2.3 mcg/L
. department who is complaining of severe
abdominal pain. The client is diagnosed with
acute pancreatitis. Which laboratory value
requires immediate intervention?
757 A nurse is caring for a client receiving warfarin assess the client for bleeding around the gums or in the
. (Coumadin) therapy following a mechanical stool and notify the physician of the laboratory results
valve replacement. The nurse completed the and most recent administration of warfarin.
client's prothrombin time and International
Normalized Ratio (INR) at 7 a.m., before the
morning meal. The client had an INR reading
of 4. The nurse's first priority should be to:
758 The nurse is caring for a client recovering from Serum, which depletes the body's store of
. a major burn. Burns affect the immune system immunoglobulins
by causing a loss of large amounts of which of
the following?
759 A nurse is caring for a client who has a a) Endotracheal suctioning
. tracheostomy and temperature of 103 F (39.4
C). Which intervention will most likely lower
the client's arterial blood oxygen saturation?

a) Endotracheal suctioning
b) Encouragement of coughing
c) Use of cooling blanket
d) Incentive spirometry
760 The nurse is caring for a client who has a c) using the minimal air leak technique with cuff pressure less
. tracheostomy tube and is undergoing than 25 cm H2O.
mechanical ventilation. The nurse can
help prevent tracheal dilation, a
complication of tracheostomy tube
placement, by:

a) suctioning the tracheostomy tube


frequently.
b) using a cuffed tracheostomy tube.
c) using the minimal air leak technique
with cuff pressure less than 25 cm H2O.
d) keeping the tracheostomy tube plugged.
761 The nurse is caring for a client who has Metabolic alkalosis
. bulimia. What is a common metabolic Explanation:
complication associated with bulimia? With repeated emesis, the client with bulimia loses stomach
acids, thus becoming alkalotic. Respiratory pH disturbances
aren't directly related to bulimia.
762 A nurse is caring for a client who has "Clients with diabetes should have a lower blood pressure
. hypertension and diabetes mellitus. The goal. You should strive for 130/80 mm Hg."
client's blood pressure this morning was
150/92 mm Hg. He asks the nurse what his
blood pressure should be. The nurse's
most appropriate response is:
763 The nurse is caring for a client who has progressively deeper breaths followed by shallower breaths
. suffered a severe stroke. During data with apneic periods.
collection, the nurse notices Cheyne- Explanation: Cheyne-Stokes respirations are breaths that
Stokes respirations. Cheyne- Stokes become progressively deeper followed by shallower
respirations are: respirations with apneic periods. Biot's respirations are rapid,
deep breaths with abrupt pauses between each breath, and
equal depth between each breath. Kussmaul's respirations are
rapid, deep breaths without pauses. Tachypnea is abnormally
rapid respirations.
764 A nurse is caring for a client who is at risk Measure intake and output.
. for skin breakdown. To decrease the risk,
the nurse must help ensure that the client
remains adequately hydrated. Which
action can the nurse take to help
determine the client's fluid needs?
765 The nurse is caring for a client who b) keeping his airway patent.
. recently underwent a tracheostomy. The
first priority when caring for a client with
a tracheostomy is:

a) helping him communicate.


b) keeping his airway patent.
c) encouraging him to perform activities of
daily living.
d) preventing him from developing an
infection.
766 The nurse is caring for a client who a chest X-ray.
. required chest tube insertion for a Explanation: Chest X-ray confirms whether the chest tube has
pneumothorax. To assess a client for resolved the pneumothorax. If the chest tube hasn't resolved
pneumothorax resolution after the the pneumothorax, the chest X-ray will reveal air or fluid in
procedure, the nurse can anticipate that the pleural space. SaO2 values may initially decrease with a
he'll require: pneumothorax but typically return to normal in 24 hours. ABG
levels may show hypoxemia, possibly with respiratory
acidosis and hypercapnia not related to a pneumothorax. Chest
auscultation will determine overall lung status, but it's difficult
to determine if the chest is reexpanded sufficiently.
767 A nurse is caring for a client who's The client has a history of diverticulitis.
. ordered continuous ambulatory
peritoneal dialysis (CAPD). Which
finding should lead the nurse to
question the client's suitability for
CAPD?
768 The nurse is caring for a client who Irrigate the NG tube with NS if ordered
. underwent a subtotal gastrectomy 36
hours ago. The client has a
nasogastric (NG) tube. The nurse
knows to do the following with the
NG:
769 A nurse is caring for a client who Drink liquids only between meals
. underwent a subtotal gastrectomy. To
manage dumping syndrome, the
nurse should advise the client to:
770 A nurse is caring for a client with NPO
. active upper GI bleeding. What is the
appropriate diet for this client during
the first 24 hours after admission?
771 The nurse is caring for a client with Increasing fluid intake to 3 L/day
. acute pyelonephritis. Which nursing Explanation: Acute pyelonephritis is a sudden inflammation of the
intervention is most important? interstitial tissue and renal pelvis of one or both kidneys. Infecting
bacteria are normal intestinal and fecal flora that grow readily in
urine. Pyelonephritis may result from procedures that involve the
use of instruments (such as catheterization, cystoscopy, and urologic
surgery) or from hematogenic infection. The most important nursing
intervention is to increase fluid intake to 3 L/day. This helps empty
the bladder of contaminated urine and prevents calculus formation.
Administering a sitz bath would increase the likelihood of fecal
contamination. Using an indwelling urinary catheter could cause
further contamination. Encouraging the client to drink cranberry
juice to acidify urine is helpful but isn't the most important
intervention.
772 The nurse is caring for a client with Increasing fluid intake to 3 L/day
. acute pyelonephritis. Which nursing Explanation:
intervention is most important? Acute pyelonephritis is a sudden inflammation of the interstitial
tissue and renal pelvis of one or both kidneys. Infecting bacteria are
normal intestinal and fecal flora that grow readily in urine.
Pyelonephritis may result from procedures that involve the use of
instruments (such as catheterization, cystoscopy, and urologic
surgery) or from hematogenic infection. The most important nursing
intervention is to increase fluid intake to 3 L/day. This helps empty
the bladder of contaminated urine and prevents calculus formation.
Administering a sitz bath would increase the likelihood of fecal
contamination. Using an indwelling urinary catheter could cause
further contamination. Encouraging the client to drink cranberry
juice to acidify urine is helpful but isn't the most important
intervention.
773 A nurse is caring for a client with a low platelet count. Clotting of blood
. The nurse understands that a low platelet count affects
which of the following?
774 The nurse is caring for a client with an immune system To verify that the client is not hypersensitive to
. disorder. Why should the nurse consult drug references substances in the prescribed medications
when assessing a client with an immune system disorder?
775 A nurse is caring for a client with aortic stenosis whose Exertional dyspnea, orthopnea, pulmonary
. compensatory mechanisms of the heart have begun to edema
fail. The nurse will monitor the client carefully for which
initial symptoms?
776 The nurse is caring for a client with a postoperative The nurse is caring for a client with a
. wound evisceration. Which action should the nurse postoperative wound evisceration. Which
perform first? action should the nurse perform first?
777 The nurse is caring for a client with chest trauma. Which a) Impaired gas exchange
. nursing diagnosis takes highest priority?

a) Impaired gas exchange


b) Anxiety
c) Decreased cardiac output
d) Ineffective cardiopulmonary tissue perfusion
778 A nurse is caring for a client with cirrhosis. The nurse A client's hepatic function is decreasing
. assesses the client at noon and discovers that the client is
difficult to arouse and has an elevated serum ammonia
level. The nurse should suspect which situation?
779 A nurse is caring for a client with cirrhosis. The nurse The client's hepatic function is decreasing.
. assesses the client at noon and discovers that the client is
difficult to arouse and has an elevated serum ammonia
level. The nurse should suspect which situation?
780 The nurse is caring for a client with cirrhosis. Which Purpura and petechiae
. assessment findings indicate that the client has deficient
vitamin K absorption caused by this hepatic disease?
781 A nurse is caring for a client with heart failure. The nurse "I sleep on three pillows each night"
. knows that the client has left-sided heart failure when he
makes which statement?
782 The nurse is caring for a client with hepatitis. Which of Rash
. the following would lead the nurse to suspect that the
client is in the prodromal phase?
783 A nurse is caring for a client with metastatic breast Hypercalcemia - The normal reference range
. cancer who is extremely lethargic and very slow to for serum calcium is 9 to 11 mg/dl. A serum
respond to stimuli. The laboratory report indicates a calcium level of 12 mg/dl clearly indicates
serum calcium level of 12.0 mg/dl, a serum potassium hypercalcemia. The client's other laboratory
level of 3.9 mEq/L, a serum chloride level of 101 mEq/L, findings are within their normal ranges, so the
and a serum sodium level of 140 mEq/L. Based on this client doesn't have hypernatremia,
information, the nurse determines that the client's hypochloremia, or hypokalemia.
symptoms are most likely associated with which
electrolyte imbalance?
784 A nurse is caring for a client with multiple sclerosis. The immune system recognizes one's own
. Client education about the disease process includes which tissues as "foreign."
of the following explanations about the cause of the
disorder?
785 The nurse is caring for a client with pneumonia. As part d) Eight to ten
. of prescribed therapy, the client must use a bedside
incentive spirometer to promote maximal deep breathing.
The nurse checks to make sure the client is using the
spirometer properly. During each waking hour, the client
should perform a minimum of how many sustained,
voluntary inflation maneuvers?

a) One to two
b) Three to four
c) Five to seven
d) Eight to ten
786 The nurse is caring for a client with skin grafts covering wrap elastic bandages distally to proximally on
. third-degree burns on the arms and legs. During dressing dependent areas.
changes, the nurse should be sure to:
787 A nurse is caring for a client with suspected black, tarry stools.
. upper GI bleeding. The nurse should Explanation: As blood from the GI tract passes through the
monitor this client for: intestines, bacterial action causes it to become black.
Hemoptysis involves coughing up blood from the lungs.
Hematuria is blood in the urine. Bright red blood in the stools
indicates bleeding from the lower GI tract.
788 The nurse is caring for a man who has "Having a bowel movement is a spinal reflex requiring intact
. experienced a spinal cord injury. nerve fibers. Yours are not intact."
Throughout his recovery, the client expects
to gain control of his bowels. The nurse's
best response to this client would be which
of the following?
789 The nurse is caring for an 82-year-old male Loss of arterial elasticity
. client who has come to the clinic for a
yearly physical. When assessing the client,
the nurse notes the blood pressure (BP) is
140/93. The nurse knows that in older
clients what happens that may elevate the
systolic BP?
790 A nurse is caring for an adult client with Extracellular Fluid Volume Deficit - Fluid volume deficit
. numerous draining wounds from gunshots. (FVD) occurs when the loss extracellular fluid (ECF) volume
The client's pulse rate has increased from exceeds the intake of fluid. FVD results from loss of body
100 to 130 beats per minute over the last fluids and occurs more rapidly when coupled with decreaesd
hour. The nurse should further assess the fluid intake. A cause of this loss is hemorrhage.
client for which of the following?
791 The nurse is caring for an elderly bedridden Post a turning schedule at the client's bedside.
. adult. To prevent pressure ulcers, which
intervention should the nurse include in the
plan of care?
792 The nurse is caring for an elderly client Impaired ciliary action as a result of exposure to
. with a respiratory infection. While environmental toxins
reviewing age-related changes in the
immune system, the nurse identifies which
of the following as having contributed to
this client's infection?
793 The nurse is caring for an elderly female Bone fracture
. with osteoporosis. When teaching the client, Explanation: Bone fracture is a major complication of
the nurse should include information about osteoporosis that results when loss of calcium and phosphate
which major complication of this condition? increases the fragility of bones. Estrogen deficiencies result
from menopause not osteoporosis. Calcium and vitamin D
supplements may be used to support normal bone
metabolism, but a negative calcium balance isn't a
complication of osteoporosis. Dowager's hump results from
bone fractures. It develops when repeated vertebral fractures
increase spinal curvature.
794 The nurse is caring for an infant with Undescended testes
. hypospadias. Which anomaly would the Explanation: Because undescended testes may also be
nurse assess the infant for that commonly present in hypospadias, the small penis may appear to be an
accompanies this condition? enlarged clitoris. This shouldn't be mistaken for ambiguous
genitalia. If there's any doubt, more tests should be
performed. Hernias don't generally accompany hypospadias.
795 The nurse is caring for a patient diagnosed Hypotension
. with abdominal perforation. Which of the
following is a clinical manifestation of this
disease process?
796 A nurse is caring for a patient experiencing a Tell the client to take deep breaths
. panic attack. Which intervention by the nurse Explanation:
would be most appropriate? During a panic attack a client may experience
symptoms of dizziness, shortness of breath, and
feelings of suffocation. The nurse should remain with
the client and direct what's said toward changing the
physiological response, such as taking deep breaths.
During an attack, the client is unable to talk about
anxious situations and isn't able to address feelings,
especially uncomfortable feelings and frustrations.
While having a panic attack, the client is also unable
to focus on anything other than the symptoms, so the
client won't be able to discuss the cause of the attack
797 The nurse is caring for a patient who is Arabic. Skin color
. The nurse remembers learning that three elements Age
are frequently used to identify diversity. Choose Geographic area
the three from the following list.
798 A nurse is caring for a patient with a Salem sump "It is a vent that prevents backflow of the secretions."
. gastric tube attached to low intermittent suction
for decompression. The patient asks, "What's this
blue part of the tube for?" Which response by the
nurse would be most appropriate?
799 The nurse is caring for a pregnant woman who is Phenylketonuria
. undergoing prenatal screening for genetic
conditions. When developing the client's teaching
plan about the conditions associated with this
screening, which condition would the nurse least
likely include?
800 The nurse is caring for a teenage client involved in cover the opening with sterile petroleum gauze.
. a motor vehicle accident. The client has a chest Explanation: If a chest tube is accidentally removed,
tube in place. If the chest tube is accidentally the nurse should cover the insertion site with sterile
removed, the nurse should immediately: petroleum gauze. The nurse should then observe the
client for respiratory distress, as tension
pneumothorax may develop. If so, the nurse should
remove the gauze to allow air to escape. The nurse
shouldn't reintroduce the tube. Rather, the nurse
should have another staff member call a physician so
another tube can be introduced by the physician under
sterile conditions.
801 The nurse is caring for a wheelchair-bound client. Ring or donut
. Which piece of equipment impedes circulation to
the area it's meant to protect?
802 The nurse is caring for the client following Disturbed body image
. surgery for a urinary diversion. The client refuses
to look at the stoma or participate in its care. The
nurse formulates a nursing diagnosis of:
803 The nurse is caring for the postoperative client in Position the client to maintain a patent airway.
. the postanesthesia care unit. Which of the
following is the priority nursing action?
804 The nurse is changing a dressing and providing Wash her hands thoroughly.
. wound care. Which activity should she perform
first?
805 The nurse is changing the dressing of a client who Dehiscence
. is 4 days postoperative with an abdominal wound.
The nurse has changed this dressing daily since
surgery. Today, the nurse notes increased
serosanguinous drainage, wound edges not
approximated, and a -inch gap at the lower end
of the incision. The nurse concludes which of the
following conditions exists?
806 A nurse is checking laboratory values on a B-type natriuretic peptide (BNP)
. client who has crackles in the lower lobes, 2+ Explanation:
pitting edema, and dyspnea with minimal The client's symptoms suggest heart failure. BNP is a
exertion. Which laboratory value does the neurohormone that's released from the ventricles when the
nurse expect to be abnormal? ventricles experience increased pressure and stretch, such as
in heart failure. A BNP level greater than 51 pg/ml is
commonly associated with mild heart failure. As the BNP
level increases, the severity of heart failure increases.
Potassium levels aren't affected by heart failure. CRP is an
indicator of inflammation. It's used to help predict the risk
of coronary artery disease. There is no indication that the
client has an increased CRP. There is no indication that the
client is experiencing bleeding abnormalities, such as those
seen with an abnormal platelet count.
807 The nurse is collecting data on a client Urine output of 20 ml/hour
. admitted with second- and third-degree
burns on the face, arms, and chest. Which
finding indicates a potential problem?
808 The nurse is collecting data on a geriatric Decreased acetylcholine level
. client with senile dementia. Which Explanation:
neurotransmitter condition is likely to A decreased acetylcholine level has been implicated as a
contribute to this client's cognitive changes?` cause of cognitive changes in healthy geriatric clients and in
the severity of dementia. Choline acetyltransferase, an
enzyme necessary for acetylcholine synthesis, has been
found to be deficient in clients with dementia.
Norepinephrine is associated with aggression, sleep- wake
patterns, and the regulation of physical responses to
emotional stimuli, such as the increased heart and
respiratory rates caused by panic.
809 The nurse is collecting data on whether the Diphtheria, tetanus, and acellular pertussis (DTaP), MMR,
. client has received all recommended inactivated polio virus (IPV), and pneumococcal vaccine
immunizations for his age. Which Explanation: Between ages 4 and 6 the child should receive
immunizations should he have received DTaP, MMR, IPV, and Varicella vaccine. Hepatitis A is
between ages 4 and 6? completed by age 2yrs. MMR alone is incomplete. H.
influenzae, type B immunization is completed by age 15
months.
810 A nurse is completing a head to toe Fingers, hands
. assessment on a patient diagnosed with Explanation:
right-sided heart failure. To assess When right-sided heart failure occurs, blood accumulates in
peripheral edema, which of the following the vessels and backs up in peripheral veins, and the extra
areas should be examined? fluid enters the tissues. Particular areas for examination are
the dependent parts of the body, such as the feet and ankles.
Other prominent areas prone to edema are the fingers,
hands, and over the sacrum. Cyanosis can be detected by
noting color changes in the lips and earlobes.
811. A nurse is completing her annual b) Jaw-thrust
cardiopulmonary resuscitation training. The
class instructor tells her that a client has
fallen off a ladder and is lying on his back;
he is unconscious and isn't breathing. What
maneuver should the nurse use to open his
airway?

a) Head tilt-chin lift


b) Jaw-thrust
c) Heimlich
d) Seldinger
812 The nurse is completing the physical The nurse should not move or manipulate the patient's
. assessment of a patient suspected of a head while assessing for bleeding or swelling.
neurological disorder. The patient reports to
the nurse that he has recently suffered a head
trauma. In such a case, which of the following
precautions should the nurse take for the
patient? Select all that apply.
813 A nurse is concerned that a client may Impaired gas exchange
. develop postoperative atelectasis. Which
nursing diagnosis would be most appropriate
if this complication occurs?
814 A nurse is conducting an initial assessment on The client had a liver transplant 2 years ago. - A history of
. a client with possible tuberculosis. Which immunocompromised status, such as that which occurs
assessment finding indicates a risk factor for with liver transplantation, places the client at a higher risk
tuberculosis? for contracting tuberculosis. Other risk factors include
inadequate health care, traveling to countries with high
rates of tuberculosis (such as southeastern Asia, Africa, and
Latin America), being a health care worker who performs
procedures in which exposure to respiratory secretions is
likely, and being institutionalized.
815 The nurse is deciding whether to report a Incompatibility between the history and the injury
. suspected case of child abuse. Which criterion Explanation: Incompatibility between the history and the
is the most important for the nurse to injury is the most important criterion on which to base the
consider? decision to report suspected child abuse. The other criteria
also may suggest child abuse but are less reliable
indicators.
816 The nurse is developing a plan of care for a Limit foods that are high in sodium.
. client with Meniere's disease and identifies a Correct
nursing diagnosis of excess fluid volume Explanation:
related to fluid retention in the inner ear. Sodium and fluid retention disrupts the delicate balance
Which intervention would be most between the endolymph and perilymph in the inner ear.
appropriate to include in the plan of care? Therefore, many clients can control their symptoms by
adhering to a low-sodium diet. Caffeinated fluids are to be
a) Restrict high-potassium foods. avoided because of their diuretic effect. Diuretics, not
b) Limit foods that are high in sodium. antihistamines, would be prescribed to lower the pressure
c) Administer prescribed antihistamine. in the endolymphatic system. Foods high in potassium
d) Encourage intake of caffeinated fluids. would be encouraged if the client is prescribed a diuretic
that causes potassium loss.
817 A nurse is developing a teaching plan for a Prophylactic medication regimens
. client with an immunodeficiency. Which of Maintenance of a well-balanced diet
the following would the nurse need to Ways to manage stress
emphasize? Select all that apply.
818 The nurse is developing a teaching plan for a c) Take prescribed medications as scheduled.
. client with asthma. Which teaching point has
the highest priority?

a) Avoid contact with fur-bearing animals.


b) Change filters on heating and air
conditioning units frequently.
c) Take prescribed medications as scheduled.
d) Avoid goose down pillows.
819 A nurse is developing a teaching plan for a Stages are applicable to any loss
. terminally ill client and his family about
about the stages of dying and emotional
reactions experienced. The nurse integrates
knowledge of which of the following in the
teaching plan?
820 A nurse is discussing nutrition with a primigravida. "You should consume other non-dairy foods that are
. The client states that she knows that calcium is high in calcium."
important during pregnancy but that she and her Explanation:
family don't consume many milk or dairy products. Food is considered the ideal source of nutrients.
What advice should the nurse give? However, milk and dairy aren't the only food sources
of calcium. While prenatal vitamins are generally
recommended, they don't satisfy all requirements.
The calcium requirement for pregnancy is 1,300
mg/day. Over-the-counter supplements aren't always
safe and should be specifically recommended by the
health care practitioner. While it's true that all fetal
organs are formed by the end of the first trimester,
development continues throughout pregnancy.
821 A nurse is educating a community group about Hyperlipidemia
. coronary artery disease. One member asks about Tobacco use
how to avoid coronary artery disease. Which of the Obesity
following items are considered modifiable risk
factors for coronary artery disease? Choose all that
apply.
822 The nurse is examining a client's scalp for evidence Behind the ears
. of lice. The nurse should pay particular attention to
which part of the scalp?
823 The nurse is explaining the DASH diet to a patient 4 to 5
. diagnosed with hypertension. The patients inquires
about how many servings of fruit per day can be
consumed on the diet. The nurse would be correct
in stating which of the following?
824 The nurse is helping a client ambulate for the first The client's pulse and respiratory rates increased
. time after 3 days of bed rest. Which observation by moderately during ambulation.
the nurse suggests that the client tolerated the Explanation: The pulse and respiratory rates
activity without distress? normally increase during and for a short time after
ambulation, especially if it's the first ambulation after
3 days of bed rest. A normal walking pace is 70 to
100 steps/minute; a much slower pace may indicate
distress. Dizziness, weakness, and profuse
perspiration are definite signs of activity intolerance.
A client who tolerates ambulation well holds the
head erect, gazes straight ahead, and keeps the toes
pointed forward; option 3 describes a client with
activity intolerance.
825 The nurse is helping a patient to correctly perform Keep the dialysis supplies in a clean area, away from
. peritoneal dialysis at home. The nurse must children and pets
educate the patient about the procedure. Which
educational information should the nurse provide
to the patient?
826 A nurse is helping a physician insert a subclavian turn the client on his left side and place the bed in
. central line. After the physician has gained access Trendelenburg's position.
to the subclavian vein, he connects a 10-ml syringe
to the catheter and withdraws a sample of blood.
He then disconnects the syringe from the port.
Suddenly, the client becomes confused, disoriented,
and pale. The nurse suspects an air embolus. She
should:
827 A nurse is helping to plan a teaching session for a "At first, the stoma may bleed slightly when
. client who will be discharged with a colostomy. touched."
Which statement describes a healthy stoma?
828 A nurse is implementing appropriate infection Urine
. control precautions for a client who is positive for
human immunodeficiency virus (HIV). The nurse
demonstrates a need for a review of transmission
routes by identifying which body fluid as a means
of transmission?
829 A nurse is inserting a nasogastric tube for feeding a client. Place in order the Sit the client in an upright
. steps from 1 to 6 for correctly inserting the tube. position
Apply gloves to the nurse's
hands
Measure the length of the
tube that will be inserted
Apply water-soluble
lubricant to the tip of the
tube
Tilt the client's nose
upward
Instruct the client to lower
the head and swallow
830 A nurse is inserting a nasogastric tube in an alert client. During the Inserted into the lungs
. procedure, the client begins to cough constantly and has difficulty breathing.
The nurse suspects the nasogastric tube is
831 A nurse is instructing a client with pressure ulcers about the importance of Protein is essential for
. increasing his protein intake. Why should the nurse encourage protein tissue repair
intake by this client?
832 A nurse is instructing a group of nursing assistants about client care. The Every 2 hours
. nurse tells them to turn clients how often to prevent skin breakdown?
833 A nurse is monitoring a client recovering from moderate sedation that was Oxygen saturation (SaO2)
. administered during a colonoscopy. Which finding requires the nurse's of 85%
immediate attention?
834 A nurse is monitoring a client with peptic ulcer disease. Which of the Tachycardia
. following assessment findings would most likely indicate perforation of the Hypotension
ulcer? Choose all that apply. A rigid, board-like
abdomen
835 A nurse is obtaining a history from a new client in the cardiovascular clinic. Rheumatic fever
. When investigating for childhood diseases and disorders associated with
structural heart disease, which finding should the nurse consider
significant?
836 The nurse is performing a baseline assessment of a client's skin integrity. Overall risk of developing
. Which of the following is a key assessment parameter? pressure ulcers
837 The nurse is performing a neurological assessment of a client who has The inability to tell how a
. sustained damage to the frontal cortex. Which of the following deficits will mouse and a cat are alike
the nurse look for during assessment?
838 A nurse is performing a respiratory assessment on a client with pneumonia. Bronchophony
. She asks the client to say "ninety-nine" several times. Through her
stethoscope, she hears the words clearly over his left lower lobe. What term
should the nurse use to document this finding?
839 A nurse is performing a respiratory assessment on a client with pneumonia. a) Bronchophony
. She asks the client to say "ninety-nine" several times. Through her
stethoscope, she hears the words clearly over his left lower lobe. What term
should the nurse use to document this finding?

a) Bronchophony
b) Tactile fremitus
c) Crepitation
d) Egophony
840 A nurse is performing discharge teaching with a client who had a total "I will have to take
. gastrectomy. Which statement indicates the need for further teaching? vitamin B12 shots up to 1
year after surgery."
841 A nurse is performing health assessment "Food seems to be getting stuck in my throat"
. with a client during an outpatient clinic visit.
The most concerning client statement to the
nurse is:
842 The nurse is performing wound care on a Preparing sterile surgical instruments for the physician to
. client. Which task indicates surgical asepsis? debride the wound
843 The nurse is performing wound care. Which Pouring solution onto a sterile field cloth
. of the following practices violates surgical
asepsis?
844 The nurse is planning care for a client after a b) Encourage the client's communication attempts by
. tracheostomy. One of the client's goals is to allowing him time to select or write words.
overcome verbal communication
impairment. Which of the following
interventions should the nurse include in the
care plan?

a) Make an effort to read the client's lips to


foster communication.
b) Encourage the client's communication
attempts by allowing him time to select or
write words.
c) Answer questions for the client to reduce
his frustration.
d) Avoid using a tracheostomy plug because
it blocks the airway.
845 A nurse is planning care for a client in acute Decreased cardiac output:
. addisonian crisis. Which nursing diagnosis Explanation: An acute addisonian crisis is a life-threatening
should receive the highest priority? event, caused by deficiencies of cortisol and aldosterone.
Glucocorticoid insufficiency causes a decrease in cardiac
output and vascular tone, leading to hypovolemia. The client
becomes tachycardic and hypotensive and may develop
shock and circulatory collapse. The client with Addison's
disease is at risk for infection; however, reducing infection
isn't a priority during an addisonian crisis. Impaired physical
mobility and Imbalanced nutrition: Less than body
requirements are appropriate nursing diagnoses for the client
with Addison's disease, but they aren't priorities in a crisis.
846 A nurse is planning preoperative teaching Noticeable improvement in hearing may not be experienced
. for a client with conductive hearing loss due for up to 6 weeks after surgery.
to otosclerosis. The client is scheduled for a Correct
stapedectomy with insertion of a prosthesis. Explanation:
What information is most crucial to include Stapedectomy is a very successful procedure; approximately
in the client's preoperative teaching? 95% of clients experience restoration of hearing. However,
hearing returns gradually and can continue to improve for
several weeks after surgery. It is important that the client
understand this in order to prevent disappointment, anxiety,
or depression after surgery. While it is important that a
member of the nursing staff assist all postoperative clients
who are getting out of bed for the first time after surgery,
this information is routinely provided and is not specific to
the client undergoing stapedectomy. Availabilty of close-
captioned TV programs is very advantageous; however, it is
not crucial information related to surgical outcomes.
847 The nurse is planning to administer a sodium retain the enema for 30 minutes to allow for sodium
. polystyrene sulfonate (Kayexalate) enema to a exchange; afterward, the client should have diarrhea.
client with a potassium level of 5.9 mEq/L. Correct Explanation:
administration and the effects of this enema would Kayexalate is a sodium exchange resin. Thus the
include having the client: client will gain sodium as potassium is lost in the
bowel. For the exchange to occur, Kayexalate must be
in contact with the bowel for at least 30 minutes.
Sorbitol in the Kayexalate enema causes diarrhea,
which increases potassium loss and decreases the
potential for Kayexalate retention.
848 A nurse is preparing a client for a lumbar Cerebral spinal fluid leakage at the puncture site
. puncture. The client has heard about post-lumbar
puncture headaches and asks what causes them.
The nurse tells the client that these headaches are
caused by which of the following?
849 A nurse is preparing a client for an exercise stress "I won't eat or drink anything after midnight tonight."
. test the following morning. Which client statement Explanation:
indicates a need for additional teaching? The client requires additional teaching if he states that
he'll fast from midnight until the test. Clients need to
abstain from eating and drinking for only 4 hours
before the test. The client should refrain from
smoking for several hours before the test. Although
the physician may direct the client to avoid certain
medications, it's more likely that the physician will
direct the client to take all his normal medications.
The client must sign a consent form before the test.
850 A nurse is preparing a client for lumbar puncture. "Remain prone for 2 to 3 hours."
. The client has heard about post-lumbar puncture
headaches and asks how to avoid having one. The
nurse tells the client that these headches can be
avoided by doing which of the following after the
procedure?
851 A nurse is preparing a client for surgery. During Gallbladder
. preoperative teaching, the client asks where is bile
stored. The nurse knows that bile is stored in the:
852 A nurse is preparing a client with a pleural assist the client to a sitting position on the edge of the
. effusion for a thoracentesis. The nurse should: bed, leaning over the bedside table.
853 A nurse is preparing a client with Crohn's disease Encourage fluid intake
. for a barium enema. What should the nurse do the
day before the test?
854 A nurse is preparing an in-service presentation Attachment
. about human immunodeficiency virus (HIV) for a
group of new graduate nurses. As part of the
presentation, the nurse is planning to describe the
events that occur once HIV enters the host cell.
Which of the following would the nurse describe as
the first step?
855 A nurse is preparing an in-service presentation Most cases are typically diagnosed in infancy
. about primary immunodeficiencies. When
describing these conditions, which of the following
would the nurse need to integrate into the
presentation?
856 A nurse is preparing a presentation for a local A change in bowel habits
. community group of older adults about colon
cancer. Which of the following would the nurse
include as the primary characteristic associated
with this disorder?
857 A nurse is preparing a presentation for a nursing Genetic predisposition as key to risk reduction
. conference about personalized medicine. Which Interaction of genes with the environment
of the following would the nurse include when Individualized approach based on person's and
describing this topic? Select all that apply. disease's genomic profile
858 The nurse is preparing a teaching plan for a Frequent and thorough handwashing
. client with an immunodeficiency. Which of the
following would the nurse emphasize as most
important?
859 A nurse is preparing to administer a 500 mL 10-15 minutes
. bolus tube feeding to a patient. The nurse
anticipates administering this feeding over which
time frame?
860 The nurse is preparing to administer Wash her hands and arrange supplies at the bedside.
. chloramphenicol (Chloromycetin Otic) to a 2- Warm the medication to body temperature.
year-old client with an infection of the external Examine the ear canal for drainage.
auditory canal. The order reads, "2 drops in the Explanation: The nurse should prepare to instill the
right ear three times per day." Which steps eardrops by washing her hands, gathering the supplies,
should the nurse take to administer this and arranging the supplies at the bedside. To avoid
medication? Select all that apply: adverse effects resulting from eardrops that are too cold
(such as vertigo, nausea, and pain), the medication
should be warmed to body temperature in a bowl of
warm water. Temperature of the drops should be tested
by placing a drop on the wrist. Before instilling the
drops, the ear canal should be examined for drainage
that may reduce the medication's effectiveness. Because
the dose is to be given in the right ear, the child should
be placed on his left side with his right ear facing up.
For an infant or a child younger than age 3, gently pull
the auricle down and back because the ear canal is
straighter in children of this age-group.
861 The nurse is preparing to administer intravenous 4 weeks
. immunoglobulin to a client with a primary
immunodeficiency. This is the client's first dose.
The nurse would anticipate administering
another dose in approximately which time?
862 A nurse is preparing to administer saquinavir, Protease inhibitor (PI)
. which is prescribed for a client who is HIV
positive. The nurse integrates knowledge of this
drug, identifying it as which of the following?
863 The nurse is preparing to examine the abdomen Supine with knees flexed
. of a client complaining of a change in his bowel
pattern. The nurse would place the client in
which position?
864 A nurse is preparing to give a client an infusion Shaking chills
. of gamma globulin. The nurse knows to stop the Flank pain
infusion if the client experiences which of the Tightness in the chest
following symptoms? Choose all that apply.
865 A nurse is providing care for a client recovering from Vasomotor symptoms associated with
. gastric bypass surgery. During assessment, the client dumping syndrome
exhibits pallor, perspiration, palpitations, headache, and
feelings of warmth, dizziness, and drowsiness. The client
reports eating 90 minutes ago. The nurse suspects:
866 The nurse is providing care for a client who has a sacral The wound should remain moist from the
. pressure ulcer with a wet-to-dry dressing. Which dressing.
guideline is appropriate for a wet-to-dry dressing?
867 A nurse is providing dietary instructions to a client with a "Maintain a high-carbohydrate, low-fat diet."
. history of pancreatitis. Which instruction is correct?
868 The nurse is providing discharge education for the client Avoid tub baths, but shower as desired.
. going home after a cardiac catheterization. Which of the Explanation:
following would be important information to give this Guidelines for self-care after hospital
client? discharge following a cardiac catheterization
include shower as desired (no tub baths),
avoid bending at the waist and lifting heavy
objects, the physician will indicate when it is
okay to return to work, and notify the
physician right away if you have bleeding,
new bruising, swelling, or pain at the puncture
site.
869 A nurse is providing discharge instructions for a client Ways to obtain supplemental nutrition
. who fell from a bicycle, resulting in a fractured jaw. The
client underwent surgical intervention with rigid fixation.
The nurse includes in the instruction
870 The nurse is providing discharge instructions for a metoclopramide
. slightly overweight client seen in the Emergency
Department for chest pain. The client was diagnosed as
having gastroesophageal reflux disease. The nurse notes
in the client's record that the client is taking
carbidopa/levodopa (Sinemet). The nurse questions the
physician's order for
871 A nurse is providing discharge teaching to a client who is "I can eat whatever I want as long as it's low
. immunosuppressed. Which statement by the client in fat."
indicates the need for additional teaching?
872 The nurse is providing home care instructions to a client protect the graft from direct sunlight.
. who has recently had a skin graft. It's most important
that the client remember to:
873 A nurse is providing in-home hospice care to a 75-year-old Life expectancy of less than 6 months
. client with lung cancer. The nurse determines that the
client is eligible for Medicare hospice benefits based on
which of the following?
874 A nurse is providing postprocedure instructions for a Review instructions with the person assisting
. client who had an esophagogastroduodenoscopy. The the client home
nurse should perform which action?
875 A nurse is providing preoperative teaching to a client Increase respiratory effectiveness
. undergoing a cholecystectomy. Which topic should the
nurse include in her teaching plan?
876 The nurse is reviewing a client's laboratory results and Document the finding as normal.
. notes that her hemoglobin level is 15 g/dL. What action
should the nurse take next?
877 A nurse is reviewing a report of a client's Urine pH of 3.0 - Normal urine pH is 4.5 to 8; therefore, a
. routine urinalysis. Which value requires urine pH of 3.0 is abnormal and requires further investigation.
further investigation? Urine specific gravity normally ranges from 1.010 to 1.025,
making this client's value normal. Normally, urine contains no
protein, glucose, ketones, bilirubin, bacteria, casts, or crystals.
Red blood cells should measure 0 to 3 per high-power field;
white blood cells, 0 to 4 per high-power field. Urine should
be clear, with color ranging from pale yellow to deep amber.
878 A nurse is reviewing the history of a client Recent history of streptococcal infection
. who is suspected of having
glomerulonephritis. Which of the following
would the nurse consider significant?
879 The nurse is reviewing the medication Indocin
. record of a client with acute gastritis.
Which medication, if noted on the client's
record, would the nurse question?
880 The nurse is reviewing the results of a 60
. client's audiometric testing. The nurse Explanation:
determines that the client has a mild A mild hearing loss is identified by a loss of greater than 40
hearing loss based on the result signifying decibels. A loss of up to 15 decibels would be classified as
which loss of decibels? normal hearing. A loss greater than 15 to 25 decibels would
be classified as slight hearing loss. A loss of greater than 40 to
a) 20 55 decibels would be classified as a moderate hearing loss.
b) 80
c) 40
d) 60
881 The nurse is reviewing the results of a Pyuria
. urinalysis on a client with acute
pyelonephritis. Which of the following
would the nurse most likely expect to find?
882 The nurse is revising a client's plan of care. During the evaluation step of the nursing process, the nurse
. During which step of the nursing process determines whether the goals established in the plan of care
does such revision take place? have been achieved and evaluates the success of the plan. If a
goal is unmet or partially met, the nurse reexamines the data
and revises the plan. Data collection involves gathering
relevant information about the patient. Planning involves
setting priorities, establishing goals, and selecting appropriate
interventions. Implementation involves providing actual
nursing care.
883 The nurse is screening donors for blood Reports having a cold 1 month ago that resolved quickly
. donation. The client who is an acceptable
donor for blood is the client who
884 A nurse is taking health history data from a Acetaminophen
. client. Use of which of the following Ketoconazole
medications would especially alert the Valproic acid
nurse to an increased risk of hepatic
dysfunction and disease in this client?
Select all that apply.
885 A nurse is taking health history from a new Exposure to foreign antigens may cause altered immune
. client, which includes asking about a function.
history of blood transfusions. This is
important for which of the following
reasons?
886 A nurse is taking the health history of a newly admitted client and Inotropics
. asks for a list of the client's current medications. Which of the
following medication classifications would NOT place the client at
risk for impaired immune function?
887 A nurse is taking the health history of a newly admitted client. Surgical removal of the appendix
. Which of the following conditions would NOT place the client at
risk for impaired immune function?
888 The nurse is talking with a client who is scheduled for a computed "My doctor will be able to remove
. tomography (CT) colonography. Which client statement would any polyps he finds."
indicate to the nurse that the client needs additional teaching about
this procedure?
889 The nurse is talking with a client who is scheduled for a computed My doctor will be able to remove
. tomography (CT) colonography. Which client statement would any polyps he finds
indicate to the nurse that the client needs additional teaching about
this procedure?
890 A nurse is teaching about ischemic stroke prevention to a Moderate amounts of low-fat dairy
. community group and emphasizes that control of hypertension, products
which is the major risk factor for stroke, is key to prevention. Ways
to control hypertension include the Dietary Approaches to Stop
Hypertension (DASH) diet. This diet includes which of the
following?
891 A nurse is teaching a client about maintaining a healthy heart. The Use alcohol in moderation
. nurse should include which point in her teaching?
892 A nurse is teaching a client who has experienced an episode of acute "My appetite should come back
. gastritis and knows further education is necessary when the client tomorrow"
makes the following statement:
893 A nurse is teaching a client who is having a valuloplasty tomorrow. "A tissue valve is less likely to
. The client asks what the advantage is for having a tissue valve generate blood clots, and so long-
replacement instead of a mechanical valve. The correct answer by term anticoagulation therapy is not
the nurse is which of the following? required."
894 A nurse is teaching a client who receives nitrates for the relief of Lie down or sit in a chair for 5 to 10
. chest pain. Which instruction should the nurse emphasize? minutes after taking the drug.
895 The nurse is teaching a client with a leg ulcer about tissue repair I'll eat plenty of fruits and
. and wound healing. Which of the following statements by the client vegetables
indicates that the teaching has been effective?
896 The nurse is teaching a client with chronic bronchitis about c) Use diaphragmatic breathing.
. breathing exercises. Which instruction should the nurse include in
the teaching?

a) Make inhalation longer than exhalation.


b) Exhale through an open mouth.
c) Use diaphragmatic breathing.
d) Use chest breathing.
897 The nurse is teaching a client with emphysema how to perform a) It helps prevent early airway
. pursed-lip breathing. The client asks the nurse to explain the collapse.
purpose of this breathing technique. Which explanation should the
nurse provide?

a) It helps prevent early airway collapse.


b) It increases inspiratory muscle strength.
c) It decreases use of accessory breathing muscles.
d) It prolongs the inspiratory phase of respiration.
898 A nurse is teaching a client with gastritis about the need to avoid "Caffeine stimulates the central
. the intake of caffeinated beverages. The client asks why this is so nervous system and thus gastric
important. Which of the following explanations from the nurse activity and secretions, which need
would be most accurate? to be minimized to promote
recovery."
899 The nurse is teaching a group of Coordinating a panel of peers who were involved in motor
. adolescents about automobile safety. vehicle accidents
Which is the most effective teaching Explanation: Coordinating a panel of peers to discuss motor
method for this age-group? vehicle accidents and their prevention is more effective for this
age-group. Adolescents are more likely to listen to others their
age who have experienced similar circumstances. Lecturing
about the effects of drugs and alcohol on driving will most
likely be ineffective for this age-group. Adolescents won't be
motivated to read the written materials. Animated videos aren't
age-appropriate and may minimize the importance of the
material.
900 A nurse is teaching an adolescent with "Carry crackers or fruit to eat before or during periods of
. type 1 diabetes about the disease. Which increased activity."
instruction by the nurse about how to Explanation: Hypoglycemia can usually be prevented if an
prevent hypoglycemia would be most adolescent with diabetes eats more food before or during
appropriate for the adolescent? exercise. Because exercise with adolescents isn't commonly
planned, carrying additional carbohydrate foods such as
crackers or fruit is a good preventive measure.
901 The nurse is teaching parents about Make sure all medications are kept in containers with
. accident prevention for a toddler. Which childproof safety caps.
of the following guidelines is most Explanation: All over-the-counter and prescription medications
appropriate? should have childproof safety caps. Poisoning accidents are
common in toddlers, due to the toddler's curiosity and his
increasing mobility and ability to climb. When riding in a car,
a toddler should be strapped into a car seat. Wearing a seat belt
is an appropriate guideline for a school-age child. Never
leaving a child alone on a bed is an appropriate guideline for
parents of infants. Toddlers already have the ability to climb on
and off of beds and other furniture by themselves. Note,
however, that toddlers should never be left unattended on high
surfaces, such as an examining table in a physician's office.
Toddlers should be in a rear-facing convertible car seat; those
who weigh 20 to 40 lb can be placed facing forward.
902 The nurse is to discontinue a nasogastric Flush with 10 mL of water
. tube that had been used for
decompression. The first thing the nurse
does is
903 A nurse is trying to determine if a client Have you received any legal violations related to your drug
. who abuses heroin has any drug-related use?
legal problems. Which assessment Explanation:
question is the best to ask the client? Asking about legal violations related to drug use provides
direct information about drug-related legal problems. When a
spouse becomes aware of a partner's substance abuse, the first
action isn't necessarily to institute legal action. Even if the
client reports to a probation officer, the offense isn't
necessarily a drug-related problem. Asking if the client has a
history of frequent visits with the employee assistance program
manager isn't useful; it assumes any such visit is related to
drug issues.
904 A nurse is working in a pediatric clinic. Active acquired immunity, which lasts many years or a
. After giving a hepatitis B immunization to lifetime
an infant, the mother asks what kind of
protection this provides for her child. The
correct response is which of the following?
905 A nurse is working in a support group for Following safer-sex practices
. clients with acquired immunodeficiency
syndrome (AIDS). Which point is most
important for the nurse to stress?
906 A nurse is working with the family of a client Encouraging the spouse to talk about the difficulties
. who has Alzheimer's disease. The nurse notes involved in caring for a loved one
that the client's spouse is too exhausted to Asking whether friends or church members can help
continue providing care all alone. The adult with errands or provide short periods of relief
children live too far away to provide relief on a Explanation: Many community services exist for
weekly basis. Which nursing interventions would Alzheimer's clients and their families. Encouraging use
be most helpful? Select all that apply. of these resources may make it possible to keep the
Recommending community resources for adult client at home and to alleviate the spouse's exhaustion.
day care and respite care The nurse can also support the caregiver by urging her
to talk about the difficulties she's facing in caring for
her spouse. Friends and church members may be able to
help provide care to the client, allowing the caregiver
time for rest, exercise, or an enjoyable activity.
Arranging a family meeting to tell the children to
participate more would probably be ineffective and
might evoke anger or guilt. Counseling might be
helpful, but it wouldn't alleviate the caregiver's physical
exhaustion or address the client's immediate needs. A
long-term care facility isn't an option until the family is
ready to make that decision.
907 The nurse knows that the client with Vitamin A
. cholelithiasis can have a nutritional deficiency.
The obstruction of bile flow due to cholelitiasis
can interfere with the absorption of
908 A nurse measures the residual gastric volume of 225 mL
. a patient receiving intermittent tube feedings.
The patient's last residual volume was 250 mL.
Which finding would lead the nurse to notify the
physician?
909 A nurse notices that a client who came to the Ask the client basic hygiene questions to determine
. clinic for treatment of anxiety disorder has a how frequently he bathes.
strong body odor. What can the nurse do or say Explanation:
to help this client? The nurse should inquire about the client's basic
hygiene to help determine the cause of his strong body
odor. The nurse can then devise a plan of care based on
the information she obtains. Offering the client an
opportunity to freshen up doesn't address the problem
and might offend him. Preparing the client for his
examination and then leaving the room doesn't address
the hygiene issues. Providing the client with personal
care items also might offend him.
910 The nurse observes a new environmental services d) The employee enters the room wearing a gown,
. employee enter the room of a client with severe gloves, and a mask.
acute respiratory syndrome (SARS). Which
action by the employee requires immediate
intervention by the nurse?

a) The employee wears a gown, gloves, N95


respirator, and eye protection when entering the
room.
b) The employee doesn't remove the stethoscope,
blood pressure cuff, and thermometer that are
kept in the room.
c) The employee removes all personal protective
equipment and washes her hands before leaving
the client's room.
d) The employee enters the room wearing a
gown, gloves, and a mask.
911. The nurse observes constant bubbling in the water- c) The system has an air leak.
seal chamber of a closed chest drainage system. What
should the nurse conclude?

a) The system is functioning normally.


b) The client has a pneumothorax.
c) The system has an air leak.
d) The chest tube is obstructed.
912 The nurse observes dry mucous membranes in a client Provide frequent mouth care
. who is receiving tube feedings after an oral surgery.
The client also complains of unpleasant tastes and
odors. Which of the following measures should be
included in the client's plan of care?
913 The nurse obtains a unit of blood for the client, Refuses to administer the blood
. Donald D. Smith. The name on the label on the unit of
blood reads Donald A. Smith. All the other identifiers
are correct. The nurse
914 A nurse on the medical-surgical unit just received d) The client with petechiae over the chest who's
. report on her client care assignment. Which client complaining of anxiety and shortness of breath
should she assess first?

a) The client with anorexia, weight loss, and night


sweats
b) The client with crackles and fever who is
complaining of pleuritic pain
c) The client who had difficulty sleeping, daytime
fatigue, and morning headache
d) The client with petechiae over the chest who's
complaining of anxiety and shortness of breath
915 A nurse places a client in full leather restraints. How Every 15 minutes
. often must the nurse check the client's circulation? Explanation:
Circulatory as well as skin and nerve damage can
occur quickly. Therefore, circulation should be
assessed at least every 15 minutes. Checking
every hour, 2 hours, or 8 hours isn't often enough
and could result in permanent damage to the
client's extremities.
916 The nurse plans to administer dexamethasone cream In long, even, outward, and downward strokes in
. to a client who has dermatitis over the anterior chest. the direction of hair growth
How should the nurse apply this topical agent?
917 The nurse plays an important role in monitoring and Atelectasis
. managing potential complications in the patient who
has recently undergone a coronary artery bypass
graft (CABG). The nurse should be alert to which of
the following respiratory complications?
918 The nurse prepares to perform postural drainage. d)Auscultation
. How should the nurse ascertain the best position to
facilitate clearing the lungs?

a) Inspection
b) Chest X-ray
c) Arterial blood gas (ABG) levels
d)Auscultation
919 The nurse recognizes older adults require lower doses decreased lean tissue mass
. of anesthetic agents due to:
920 The nurse recognizes that the client most at risk for Client with chronic alcoholism
. mortality associated with surgery is the:
921 The nurse recognizes that the client who takes Respiratory depression
. hydrochlorothiazide (HydroDIURIL) to manage
hypertension is predisposed for which interaction
with anesthesia?
922 The nurse recognizes that the older decreased renal function
. adult is at risk for surgical
complications due to:
923 The nurse recognizes that the older Decreased renal function
. adult is at risk for surgical
complications due to:
924 The nurse recognizes the client has Has small pupils that react to light
. reached stage III of general anesthesia
when the client:
925 A nurse reviews the arterial blood gas Respiratory alkalosis - A client with pneumonia may
. (ABG) values of a client admitted with hyperventilate in an effort to increase oxygen intake.
pneumonia: pH, 7.51; PaCO2, 28 mm Hyperventilation leads to excess carbon dioxide (CO2) loss,
Hg; PaO2, 70 mm Hg; and HCO3--, 24 which causes alkalosis indicated by this client's elevated pH
mEq/L. What do these values indicate? value. With respiratory alkalosis, the kidneys' bicarbonate
(HCO3-) response is delayed, so the client's HCO3- level
remains normal. The below-normal value for the partial pressure
of arterial carbon dioxide (PaCO2) indicates CO2 loss and
signals a respiratory component. Because the HCO3- level is
normal, this imbalance has no metabolic component. Therefore,
the client is experiencing respiratory alkalosis.
926 A nurse reviews the arterial blood gas Respiratory Alkalosis - A client with pneumonia may
. (ABG) values of a client admitted with hyperventilate in an effort to increase oxygen intake.
pneumonia: pH, 7.51; PaCO2, 28 mm Hyperventilation leads to excess carbon dioxide (CO2) loss,
Hg; PaO2, 70 mm Hg; and HCO3--, 24 which causes alkalosis indicated by this client's elevated pH
mEq/L. What do these values indicate? value. With respiratory alkalosis, the kidneys' bicarbonate
(HCO3-) response is delayed, so the client's HCO3- level
remains normal. The below-normal value for the partial pressure
of arterial carbon dioxide (PaCO2) indicates CO2 loss and
signals a respiratory component. Because the HCO3- level is
normal, this imbalance has no metabolic component. Therefore,
the client is experiencing respiratory alkalosis.
927 The nurse's base knowledge of primary Primary immunodeficiencies develop early in life after
. immunodeficiencies includes which of protection from maternal antibodies decreases.
the following statements?
928 The nurse's base knowledge of primary develop early in life after protection from maternal antibodies
. immunodeficiencies includes which of decreases.
the following statements? Primary
immunodeficiencies
929 The nurse sees an unauthorized person Notify the nursing supervisor and approach the individual.
. reading a client's medical record outside Explanation: Approaching the person and requesting the client's
a client's room. Which action should the medical record isn't sufficient considering the confidential health
nurse take? care information. Notifying the nursing supervisor, then
approaching the individual before informing the client provides
the most appropriate approach to this breech of client
confidentiality. Contacting security might not be warranted
unless the nurse learns the reason the unauthorized individual
was reading the client's chart. The nurse should also document
the incident according to facility policy.
930 The nurse should assess for an Serum lipase
. important early indicator of acute
pancreatitis, which is a prolonged and
elevated level of:
931 The nurse should include which fact The virus can be spread through many routes, including sexual
. when teaching an adolescent group contact
about the human immunodeficiency Explanation: HIV can be spread through many routes, including
virus (HIV)? sexual contact and contact with infected blood or other body
fluids. The incidence of HIV in the adolescent population has
increased since 1995, even though more information about the
virus is targeted to reach the adolescent population. Only about
25% of all new HIV infections in the United States occurs in
people younger than age 22.
932 A nurse should teach the client to watch for Dumping syndrome
. which complication of gastric resection? Explanation: Dumping syndrome is a problem that occurs
postprandially after gastric resection because ingested food
rapidly enters the jejunum without proper mixing and without
the normal duodenal digestive processing. Diarrhea, not
constipation, may also be a symptom. Gastric or intestinal
spasms don't occur, but antispasmodics may be given to slow
gastric emptying.
933 The nurse teaches a mother how to provide The child eats finger foods by himself.
. adequate nutrition for her toddler, who has Explanation: The child with cerebral palsy should be
cerebral palsy. Which of the following encouraged to be as independent as possible. Finger foods
observations indicates that teaching has allow the toddler to feed himself. Because spasticity affects
been effective? coordinated chewing and swallowing as well as the ability to
bring food to the mouth, it's difficult for the child with
cerebral palsy to eat neatly. Independence in eating should
take precedence over neatness. The child with cerebral palsy
may require more time to bring food to the mouth; thus,
chewing and swallowing shouldn't be rushed to finish a meal
by a specified time. The child with cerebral palsy may vomit
after eating due to a hyperactive gag reflex. Therefore, the
child should remain in an upright position after eating to
prevent aspiration and choking.
934 The nurse wants to help a client maintain Keeping the client well-hydrated
. healthy skin. Which nursing intervention
will help achieve this goal?
935 A nurse who works in the OR is required to Increased urine output
. assess the patient continuously and protect
the patient from potential complications.
Which of the following would not be
included as a symptom of malignant
hyperthermia?
936 The nurse working in the radiology clinic is Absent distal pulses
. assisting with a pulmonary angiography.
The nurse knows that when monitoring
clients after a pulmonary angiography,
what should the physician be notified
about?
937 The nurse would expect to observe which of Urine that appears dark brown
. the following when assessing a client with
cholelithiasis?
938 Nursing assessment findings reveal that the surgeon
. client is afraid of dying during the surgical
procedure. Which surgical team member
would be most helpful in addressing the
client's concern?
939 A nursing instructor is discussing The complement system
. immunodeficiency disorders with students. B and T lymphocytes
The instructor tells the class that Phagocytic cells
immunodeficiency disorders are caused by
defects or deficiencies in which of the
following? Choose all that apply.
940 A nursing instructor is lecturing to a class about chronic Alcohol consumption and smokig
. pancreatitis. Which of the following does the instructor list as
major causes?
941 A nursing instructor is preparing a class about gastrointestinal Remove gas and fluids from the
. intubation. Which of the following would the instructor include as stomach
reason for this procedure? Select all that apply. Diagnose gastrointestinal motility
disorders
Flush ingested toxins from the
stomach
Administer nutritional substances
942 The nursing instructor is talking with her class about spinal Instruct the client to remain flat for 6
. anesthesia. What would be the nursing care intervention required to 12 hours.
when caring for a client recovering from spinal anesthesia?
943 The nursing instructor is talking with senior nursing students Infection
. about diagnostic procedures used in respiratory diseases. The
instructor discusses thoracentesis, defining it as a procedure
performed for diagnostic purposes or to aspirate accumulated
excess fluid or air from the pleural space. What would the
instructor tell the students purulent fluid indicates?
944 The nursing instructor is teaching their clinical group about Transportation of O2 to the tissues
. laboratory blood tests. What is the major function of and removal of CO2 from the tissues
erythrocytes?
945 A nursing instructor tells the class that review of oral hygiene is "Injury to oral mucosa or tooth decay
. an important component during assessment of the gastrointestinal can lead to difficulty in chewing
system. One of the students questions this statement. Which of the food."
following explanations from the nurse educator is most
appropriate?
946 The nursing priority of care for a client exhibiting signs and Enhance myocardial oxygenation
. symptoms of coronary artery disease should be to:
947 The nursing student asks their instructor what the term is for the pH - The symbol pH refers to the
. amount of hydrogen ions in a solution. What should the instructor amount of hydrogen ions in a
respond? solution; pH can range from 1, which
is highly acidic, to 14, which is
highly basic. All other options are
incorrect.
948 The nursing student has just reviewed material in the course Severe abdominal pain
. textbook regarding pancreatitis. The student knows that a major
symptom of pancreatitis that causes the client to seek medical
care is:
949 A nursing student has learned about many collaborative Encourage bed rest to decrease the
. interventions to achieve pain relief for clients with acute client's metabolic rate.
pancreatitis. Which of the following are appropriate? Choose all Teach the client about the
that apply. correlation between alcohol intake
and pain.
Withhold oral feedings to limit the
release of secretin.
950 A nursing student is assigned to a patient with a mechanical valve "You are at risk of developing an
. replacement. The patient asks the student, "Why do I have to take infection in your heart."
antibiotics before getting my teeth cleaned?" Which response by
the nursing student is most appropriate?
951 A nursing student is caring for a client with end-stage Heart transplant
. cardiomyopathy. The client's spouse asks the student to clarify
one of the last treatment options available that the physician
mentioned. After checking with the primary nurse, the student
would most likely discuss which of the following?
952 A nursing student is preparing a teaching plan for a client with an Chronic diarrhea
. immunodeficiency disorder. The student is going to include the Chronic or recurrent severe
cardinal symptoms in teaching. Which of the following would the infections
student include? Choose all that apply. Poor response to treatment of
infections
953 A nursing student is reviewing an article about genetic disorders -Turner Syndrome
. involving the failure of chromosomes to separate completely, -Down Syndrome
resulting in a cell that contains more than one copy or no copy of
a particular chromosome. The student would identify which
condition as an example of this phenomenon? Select all that
apply.
954 The nursing students are learning about the Lymphoid Tissues
. immune system in their anatomy and physiology
class. What would these students learn is a
component of the immune system?
955 The nursing students are learning about the Lymphoid tissues
. immune system in their anatomy and physiology
class. What would these students learn is a
component of the immune system?
956 Nursing students are reviewing information There remains a conspiracy of silence about dying
. about attitudes related to death and dying. The despite progress in the area.
students demonstrate understanding of the
information when they identify which of the
following as most accurate?
957 Nursing students are reviewing information Kaposi's Sarcoma
. about the different manifestations associated
with AIDS. The students demonstrate
understanding of these manifestations when they
identify which of the following as the most
common HIV-related malignancy?
958 Nursing students are reviewing information IgA deficiency
. about the various types of primary
immunodeficiencies. The students demonstrate
understanding of the material when they identify
which of the following as an example of a
primary immunodeficiency involving B-
lymphocyte dysfunction?
959 Nursing students are reviewing the RNA
. pathophysiology of human immunodeficiency
virus (HIV). They demonstrate understanding of
the information when they state which of the
following as containing the genetic viral
material?
960 Of the following terms, which is used to refer to Bereavement
. the period of time during which mourning a loss
takes place?
961 Olanzapine (Zyprexa) Which of the following is considered an atypical
. antipsychotic?
962 old-old 85 years of age and older
.

963 On a routine visit to the physician, a client with Taking daily walks
. chronic arterial occlusive disease reports Explanation:
stopping smoking after 34 years. To relieve Taking daily walks relieves symptoms of intermittent
symptoms of intermittent claudication, a claudication, although the exact mechanism is unclear.
condition associated with chronic arterial Aerobic exercise may make these symptoms worse.
occlusive disease, the nurse should recommend Clients with chronic arterial occlusive disease must
which additional measure? reduce daily fat intake to 30% or less of total calories.
The client should limit dietary cholesterol because
hyperlipidemia is associated with atherosclerosis, a
known cause of arterial occlusive disease. However,
HDLs have the lowest cholesterol concentration, so this
client should eat, not avoid, foods that raise HDL levels
964 On arrival at the intensive care unit, a critically hypotension
. ill client suffers respiratory arrest and is placed
on mechanical ventilation. The physician orders
pulse oximetry to monitor the client's arterial
oxygen saturation (SaO2) noninvasively. Which
vital sign abnormality may alter pulse oximetry
values?
965 On arrival at the intensive care unit, a critically d) Hypotension
. ill client suffers respiratory arrest and is placed
on mechanical ventilation. The physician orders
pulse oximetry to monitor the client's arterial
oxygen saturation (SaO2) noninvasively. Which
vital sign abnormality may alter pulse oximetry
values?

a) Fever
b) Tachypnea
c) Tachycardia
d) Hypotension
966 On auscultation, which finding suggests a right b) Absence of breath sounds in the right thorax
. pneumothorax?

a) Bilateral inspiratory and expiratory crackles


b) Absence of breath sounds in the right thorax
c) Inspiratory wheezes in the right thorax
d) Bilateral pleural friction rub
967 Once the operating team has assembled in the Review the schedule procedure, site, and client
. room, the circulating nurse calls for a "time
out." What action should the nurse take during
the time out?
968 Oncotic pressure refers to the osmotic pressure exerted by proteins. - Oncotic pressure
. is a pulling pressure exerted by proteins, such as
albumin. Osmolality refers to the number of dissolved
particles contained in a unit of fluid. Osmotic diuresis
occurs when the urine output increases due to excretion
of substances such as glucose. Osmotic pressure is the
amount of pressure needed to stop the flow of water by
osmosis
969 Oncotic pressure refers to the Osmotic pressure exerted by proteins - Oncotic pressure
. is a pulling pressure exerted by proteins, such as
albumin. Osmolality refers to the number of dissolved
particles contained in a unit of fluid. Osmotic diuresis
occurs when the urine output increases due to excretion
of substances such as glucose. Osmotic pressure is the
amount of pressure needed to stop the flow of water by
osmosis.
970 Oncotic pressure refers to the osmotic pressure exerted by proteins. - Oncotic pressure
. is a pulling pressure exerted by proteins, such as
albumin. Osmolality refers to the number of dissolved
particles contained in a unit of fluid. Osmotic diuresis
occurs when the urine output increases due to excretion
of substances such as glucose. Osmotic pressure is the
amount of pressure needed to stop the flow of water by
osmosis.
971 One day after an appendectomy, a 9-year-old The child rates pain at 4 out of 5. Pain medication
. client rates his pain at 4 out of 5 on the pain administered as prescribed.
scale but is playing video games and laughing Explanation: Pain is what the child says it is, and the
with his friend. Which of the following would nurse must document what the child reports. If a child's
the nurse document on the child's chart? behavior appears to differ from the child's rating of pain,
believe the pain rating. A child who uses a passive
coping behavior (such as distraction) may rate pain as
more intense than children who use active coping
behavior (such as crying). Making judgments about pain
based on behavior can result in children being
inadequately medicated for pain.
972 OR? Operating Room!
.

973 or a client with advanced chronic c) Using a high-flow Venturi mask to deliver oxygen as
. obstructive pulmonary disease (COPD), prescribed
which nursing action best promotes
adequate gas exchange?

a) Encouraging the client to drink three


glasses of fluid daily
b) Keeping the client in semi-Fowler's
position
c) Using a high-flow Venturi mask to
deliver oxygen as prescribed
d) Administering a sedative as
prescribed
974 Oral intake is controlled by the thirst Hypothalamus
. center, located in which of the following
cerebral areas?
975 Other than abstinence, what is the only Consistent and correct use of condoms
. proven method of decreasing the risk
for sexual transmission of HIV
infection?
976 Parents of a 6-year-old child tell a Typical absence
. physician that the child has been Explanation: A typical absence seizure has an onset between ages
having periods of unawareness with 4 and 8. It's exhibited by an abrupt loss of consciousness,
short periods of staring. Based on his amnesia, or unawareness characterized by staring and a 3-
history, the child is probably having cycle/second spike and waveform on an EEG. The attack lasts
which type of seizure? from 10 to 30 seconds and may occur as frequently as 50 to 100
times a day. No postictal or confused state follows the attack. A
complex partial seizure most commonly occurs in older children
and adults, causing a brief impairment of consciousness. A
myoclonic seizure occurs in older children and is exhibited by
lightning jerks without loss of consciousness. An abrupt increase
in muscle tone, loss of consciousness, and marked autonomic
signs and symptoms characterize the tonic seizure.
977 The parents of a school-age child with Taking prophylactic drugs before the activity can prevent asthma
. asthma express concern about letting attacks and enable the child to engage in most sports.
the child participate in sports. What Explanation: Although exercise may trigger asthma attacks,
should the nurse tell the parents about taking prophylactic asthma drugs before beginning the activity
the relationship between exercise and can prevent attacks, enabling the child to engage in most sports.
asthma? Asthma attacks may be triggered by various factors, including
allergens, exercise, medications, upper respiratory infections, and
psychological stress. Provided the asthma is under control, most
children can participate in sports and other physical activities; in
fact, they benefit from exercise. Activity restrictions hamper peer
interaction, which is essential to the development of the school-
age child. A child with asthma may tolerate intermittent activities
better than continuous ones.
978 Partial hospitalization programs All of the following are residential treatment settings except
. (PHPs)
979 A patient comes in to get an EIA test Obtaining consent from patient
. done because her physician suspects
AIDS. Which of the following nursing
actions is essential before an EIA test is
performed?
980 A patient complains about chest pain and heavy Assess chest pain and administer prescribed drugs
. breathing when exercising or when stressed. Which of and oxygen
the following is a priority nursing intervention for the
patient diagnosed with coronary artery disease?
981 A patient develops diarrhea secondary to antibiotic dizziness.
. therapy. He is to receive two tablets of diphenoxylate Explanation: The most common adverse effects of
HCl with atropine sulfate (Lomotil) orally as needed diphenoxylate HCl with atropine sulfate are
for each loose stool. The nurse should inform him that drowsiness and dizziness related to the drug's
he may experience chemical similarity to meperidine, an opioid.
Tachycardia is an adverse effect, not bradycardia.
Muscle aches and an increase in appetite are not
adverse effects of the drug.
982 A patient diagnosed with a pulmonary embolism (PE) Dead space
. would be expected to have which type of ventilation-
perfusion?
983 A patient diagnosed with arthritis has been taking Tinnitus
. aspirin and now reports experiencing adverse effects. Correct
What adverse effect indicates that a decrease in dose Explanation:
may be necessary? Tinnitus is a sign of ototoxicity, which can occur
when a patient's dose is too high.
a) Vertigo
b) Otalgia
c) Tinnitus
d) Nystagmus
984 A patient diagnosed with diabetic ketoacidosis would Kussmaul respirations
. be expected to have which type of respiratory
pattern?
985 A patient has a gastric sump tube inserted and 120 mL
. attached to low intermittent suction. The physician
has ordered the tube to be irrigated with 30 mL of
normal saline every 6 hours. When reviewing the
patient's intake and output record for the past 24
hours, the nurse would expect to note that the patient
received how much fluid with the irrigation?
986 A patient has a high magnesium level. Identify how Decreases myocardial contractility
. hypermagnesemia affects cardiac function. Explanation:
Hypermagnesemia can cause depression of
myocardial contractility and excitability heart
block and asystole. Hypomagnesemia predisposes
patient to atrial or ventricular tachycardias.
987 A patient has a high magnesium level. Identify how Decreases myocardial contractility
. hypermagnesemia affects cardiac function. Explanation:
Hypermagnesemia can cause depression of
myocardial contractility and excitability heart
block and asystole. Hypomagnesemia predisposes
patient to atrial or ventricular tachycardias
988 A patient has an elevated serum ammonia level and is Hepatic encephalopathy
. exhibiting mental status changes. The nurse should
suspect which of the following conditions?
989 A patient has an order for arterial blood gases (ABG) Allen Test
. to be drawn? Which of the following tests must be
done prior to the procedure?
990. A patient has been diagnosed with Digoxin (Lanoxin)
congestive heart failure (CHF). The Explanation:
physician has ordered a medication to Contractility is enhanced by circulating catecholamines,
enhance contractility. The nurse would sympathetic neuronal activity, and certain medications, such
expect which medication to be ordered for as Lanoxin. Increased contractility results in increased stroke
the patient? volume. The other medications are classified as platelet-
inhibiting medications.
991. A patient has been prescribed a digitalis Electrolyte and water loss
preparation for heart failure. Which of
the following should you, as her nurse,
closely monitor when caring for this
client?
992. The patient has had biomarkers drawn Troponin
after complaining of chest pain. Which
diagnostic of myocardial infarction
remains elevated for as long as 3 weeks?
993. A patient has undergone a cardiac Avoid heavy lifting for the next 24 hours.
catheterization. He is to be discharged Explanation:
today. What information should the nurse For the next 24 hours, the patient should not bend at the waist,
emphasize during discharge teaching? strain, or life heavy objects. The patient should avoid tub
baths, but shower as desired. The patient should call her the
health care provider if she has any bleeding, swelling, new
bruising, or pain from her procedure puncture site, or a
temperature of 101.5 degrees Fahrenheit or more
994. A patient has undergone a liver biopsy. on the right side
Which of the following postprocedure
positions is appropriate?
995. A patient has undergone a liver biopsy. On the right side
Which of the following postprocedure
positions is appropriate?
996. A patient in the ICU has a central venous Right-sided heart failure
pressure (CVP) line placed. The CVP Explanation:
reading is 10 mm Hg. The nurse would Normal CVP is 2 to 8 mm Hg. A CVP greater that 8 mm Hg
interpret this reading as being related to indicates hypervolemia or right-sided heart failure. A CVP less
which of the following? than 2 mm Hg indicates a reduction in preload or
hypovolemia.
997. A patient is actively hallucinating during Thought content
an assessment. The nurse would be
correct in documenting the hallucination
as a disturbance in which of the
following?
998. A patient is being treated for Drink at least 8 to 10 large glasses of fluid every day
diverticulosis. Which of the following
information should the nurse include in
this patient's teaching plan?
999. A patient is exhibiting digitalis toxicity. Digoxin
Which of the following medications would
the nurse expect to be ordered for this
patient?
1000 The patient is on a continuous tube Shift
. feeding. The tube placement should be
checked every
1001 A patient is receiving anticoagulant Low BP
. therapy. The nurse should be alert to
potential signs and symptoms of external
or internal bleeding, as evidenced by
which of the following?
1002 A patient is receiving continuous tube 30 mL
. feedings via a small bore feeding tube. The
nurse irrigates the tube after administering
medication to maintain patency. Which size
syringe would the nurse use?
1003 A patient is receiving nasogastric tube Excess fluid volume
. feedings. The intake and output record for
the past 24 hours reveals an intake of 3100
mL and an output of 2400 mL. The nurse
identifies which nursing diagnosis as most
likely?
1004 A patient is receiving parenteral nutrition. Hang a solution of dextrose 10% and water until the new
. The current solution is nearing completion, solution is available.
and a new solution is to be hung, but it has
not arrived from the pharmacy. Which
action by the nurse would be most
appropriate?
1005 A patient is to undergo surgery on his Sims (lateral position)
. kidney. The patient would be placed in
which position for the surgery?
1006 Patients diagnosed with esophageal varices Hypotension
. are at risk for hemorrhagic shock. Which of
the following is a sign of potential
hypovolemia?
1007 Patients diagnosed with hypervolemia It increases the client's desire to consume fluid. - The
. should avoid sweet or dry food because: management goal in hypervolemia is to reduce fluid volume.
For this reason, fluid is rationed, and the client is advised to
take limited amount of fluid when thirsty. Sweet or dry food
can increase the client's desire to consume fluid. Sweet or
dry food does not obstruct water elimination nor does it
cause dehydration. Weight regulation is not part of
hypervolemia management except to the extent that it is
achieved on account of fluid reduction.
1008 Patients diagnosed with malabsorption Vitamin K
. syndrome may have vitamin and mineral
deficiency. Patient who easily bleed have
which of the following deficiencies?
1009 Patients who are taking beta-adrenergic Worsening angina
. blocking agents should be cautioned not to
stop taking their medications abruptly
because which of the following may occur?
1010 Patients who have received corticosteroids Adrenal
. preoperatively are at risk for which type of
insufficiency?
1011. Patients with chronic liver dysfunction have Scurvy
problems with insufficient vitamin intake.
Which of the following may occur as a
result of vitamin C deficiency?
1012 A patient undergoing a skin test has been Assure the patient that this is a normal reaction
. intradermally injected with a disease-
specific antigen on the inner forearm. The
patient becomes anxious because the area
begins to swell. Which of the following may
be used to decrease anxiety in this patient?
1013 A patient underwent a continent ileostomy. Every 4 to 6 hours
. Within which timeframe should the patient
expect to empty the reservoir?
1014 A patient with an ileostomy should avoid Enteric coated tablets
. which of the following?
1015 Perfusion refers to blood supply to the The two methods of perfusion are the bronchial and
. lungs, through which the lungs receive pulmonary circulation.
nutrients and oxygen. What are the two
methods of perfusion?
1016 Phagocytic dysfunction is characterized by Increased incidence of bacterial infections
. the following. Choose all that apply. Chronic eczematoid dermatitis
1017 Pharmacologic therapy frequently is used 6-12 months
. to dissolve small gallstones. It takes about
how many months of medication with
UDCA or CDCA for stones to dissolve?
1018 The physician determines that a client has d) Daily doses of isoniazid, 300 mg for 6 months to 1 year
. been exposed to someone with tuberculosis.
The nurse expects the physician to order
which of the following?

a) Daily oral doses of isoniazid (Nydrazid)


and rifampin (Rifadin) for 6 months to 2
years
b) Isolation until 24 hours after
antitubercular therapy begins
c) Nothing, until signs of active disease
arise
d) Daily doses of isoniazid, 300 mg for 6
months to 1 year
1019 A physician has arrived on the floor to Ensures that the client has signed the informed consent form
. perform a bone marrow aspiration. The
nurse first
1020 A physician has ordered a liver biopsy for Low platelet count
. a client whose condition is deteriorating.
Which of the following places the client at
high risk due to her altered liver function
during the biopsy?
1021 A physician has ordered that a client with MRI can view soft tissues and can help stage cancers.
. suspected lung cancer undergo magnetic
resonance imaging (MRI). The nurse
explains the benefits of this study to the
client. Included in teaching would be
which of the following regarding the MRI?
1022 A physician orders an isotonic I.V. solution Lactated Ringer's solution - Lactated Ringer's solution, with
. for a client. Which solution should the an osmolality of approximately 273 mOsm/L, is isotonic. The
nurse plan to administer? nurse shouldn't give half-normal saline solution because it's
hypotonic, with an osmolality of 154 mOsm/L. Giving 5%
dextrose and normal saline solution (with an osmolality of
559 mOsm/L) or 10% dextrose in water (with an osmolality
of 505 mOsm/L) also would be incorrect because these
solutions are hypertonic.
1023 The physician orders a palliative care consult c) Increasing the need for antianxiety agents
. for a client with end-stage chronic obstructive
pulmonary disease who wishes no further
medical intervention. Which step should the
nurse anticipate based on her knowledge of
palliative care?

a) Decreasing administration of pain


medications
b) Reducing oxygen requirements
c) Increasing the need for antianxiety agents
d) Decreasing the use of bronchodilators
1024 The physician orders digoxin (Lanoxin) 0.1 mg 0.25
. orally every morning for a 6-month-old infant Explanation: To convert mg to mcg: 1,000 mcg/1 mg =
with heart failure. Digoxin is available in a 400 X mcg/0.1 mg; X = 100 mcg. To calculate drug dose:
mcg/mL concentration. How many milliliters of Dose on hand/Quantity on hand = Dose desired/X. 400
digoxin should the nurse give? Record your mcg/mL = 100 mcg/X; X = 0.25 mL.
answer using two decimal places.
1025 A physician orders digoxin (Lanoxin) for a Potassium level of 2.8 mEq/L
. client with heart failure. During digoxin
therapy, which laboratory value may predispose
the client to digoxin toxicity?
1026 A physician orders regular insulin 10 units I.V. Hyperkalemia - Administering regular insulin I.V.
. along with 50 ml of dextrose 50% for a client concomitantly with 50 ml of dextrose 50% helps shift
with acute renal failure. What electrolyte potassium from the extracellular fluid into the cell,
imbalance is this client most likely which normalizes serum potassium levels in the client
experiencing? with hyperkalemia. This combination doesn't help
reverse the effects of hypercalcemia, hypernatremia, or
hyperglycemia.
1027 A physician orders spironolactone (Aldactone), Loss of 1 kg in 24 hours
. 50 mg by mouth four times daily, for a client
with fluid retention caused by cirrhosis. Which
finding indicates that the drug is producing a
therapeutic effect?
1028 The physician prescribes a loop diuretic for a Hypokalemia
. client. When administering this drug, the nurse Explanation: A loop diuretic removes water and, along
anticipates that the client may develop which with it, sodium and potassium. This may result in
electrolyte imbalance? hypokalemia, hypovolemia, and hyponatremia.
1029 The physician prescribes an emollient for a Doing so prevents evaporation of water from the
. client with pruritus of recent onset. The client hydrated epidermis.
asks why the emollient should be applied
immediately after a bath or shower. How should
the nurse respond?
1030 The physician prescribes triamcinolone b) Administer the salmeterol and then administer the
. (Azmacort) and salmeterol (Serevent) for a triamcinolone.
client with a history of asthma. What action
should the nurse take when administering these
drugs?

a) Administer the triamcinolone and then


administer
the salmeterol.
b) Administer the salmeterol and then
administer the triamcinolone.
c) Allow the client to choose the order in which
the drugs are administered.
d) Monitor the client's theophylline level before
administering the medications.
1031 A physician suspects that a client has peptic Endoscopy
. ulcer disease. With which of the following
diagnostic procedures would the nurse most
likely prepare to assist?
1032 A physician treating a client in the cardiac Heart
. care unit for atrial arrhythmia orders
metoprolol (Lopressor), 25 mg P.O. two times
per day. Metoprolol inhibits the action of
sympathomimetics at beta1-receptor sites.
Where are these sites mainly located?
1033 Pink frothy sputum may be an indication of pulmonary edema.
.

1034 Place the client in high Fowler's position. A nurse is caring for a client with acute pulmonary edema.
. To immediately promote oxygenation and relieve
dyspnea, the nurse should:
1035 Placing the client in a side-lying position An unconscious client is at risk for aspiration. To decrease
. this risk, the nurse should place the client in a side-lying
position when performing oral hygiene; doing so allows
fluid to drain from the mouth, preventing aspiration.
Swabbing the client's lips, teeth, and gums with lemon
glycerin would promote tooth decay. Cleaning the tongue
with gloved fingers wouldn't be effective in removing oral
secretions or debris in an unconscious client. Placing the
client in semi-Fowler's position would increase the risk of
aspiration.
1036 pleural friction rub creaking or grating sound from roughened, inflamed
. surfaces of the pleura rubbing together, evident during
inspiration, expiration, or both and no change with
coughing; usually uncomfortable, especially on deep
inspiration.
1037 Postoperatively, a patient with a radical neck Fowler's
. dissection should be placed in which position?
1038 Postpericardiotomy syndrome may occur in Pericardial friction rub
. patients who undergo cardiac surgery. The
nurse should be alert to which of the
following clinical manifestations associated
with this syndrome?
1039 Potency Which of the following terms is used to describe the
. amount of the drug needed to achieve the maximum
effect?
1040 Potential adverse effects of surgery and - allergic reaction
. anesthesia? (8) - Cardiac dysrythmia
(Immune? -Electrolyte imbalance
Cardiovascular? - myocardial depression, bradycardia and circulatory
electrolyte, anesthesia, CNS, respiratory?) collapse
- CNS agitation, seizures & respiratory arrest
- Oversedation or undersedation
-Agitation or disorientation
- Hypoxemia or hypercarbia
1041 Potential adverse effects of surgery and - Hypotension
. anesthesia? - Thrombosis
Blood?
1042 Potential adverse effects of surgery and - Laryngeal trauma, oral trauma, broken teeth!
. anesthesia?

Intubation?
1043 Potential adverse effects of surgery and - Hyperthermia (effect of anesthesia)
. anesthesia? - Skin and nerve damage from prolong inappropriate
Temperature? positioning.
Skin? - Electrical shock, laser and burns
Burns? - Drug toxicity, faulty equipment and human error.
Drug?
1044 Prednisone (Deltasone) is prescribed to control b) acute adrenocortical insufficiency.
. inflammation in a client with interstitial lung
disease. During client teaching, the nurse
stresses the importance of taking prednisone
exactly as prescribed and cautions against
discontinuing the drug abruptly. A client who
discontinues prednisone abruptly may
experience:

a) hyperglycemia and glycosuria.


b) acute adrenocortical insufficiency.
c) GI bleeding.
d) restlessness and seizures.
1045 The pre-nursing class is learning about the Parasympathetic
. nervous system in their anatomy class. What
part of the nervous system would the students
learn is responsible for digesting food and
eliminating body waste?
1046 A preschool-age child underwent a Frequent swallowing
. tonsillectomy 4 hours ago. Which data Explanation: Frequent swallowing an attempt to clear
collection finding would make the nurse the throat of trickling blood suggests postoperative
suspect postoperative hemorrhage? hemorrhage. Emesis may be brown or blood-tinged after
a tonsillectomy; only bright red emesis signals
hemorrhage. The child may refuse fluids because of
painful swallowing, not bleeding. Hemorrhage is
associated with an increased, not decreased, heart rate.
1047 Primary Which of the following types of prevention strategies
. would aim to stop mental disorders from ever occurring
and to reduce identified cases of psychiatric disorders
and disabilities within a population?
1048 The primary objective in the immediate maintaining pulmonary ventilation.
. postoperative period is
1049 The primary source of microorganisms for Catheter hub
. catheter-related infections are the skin and
which of the following?
1050 PR interval Which of the following tends to be prolonged on the
. electrocardiogram (ECG) during a first-degree
atrioventricular (AV) block?
1051 A priority nursing intervention for a client with Monitoring respiratory status for signs and symptoms of
. hypervolemia involves which of the following? pulmonary complications. - Hypervolemia, or fluid
volume excess (FVE), refers to an isotonic expansion of
the extracellular fluid. Nursing interventions for FVE
include measuring intake and output, monitoring weight,
assessing breath sounds, monitoring edema, and
promoting rest. The most important intervention in the
list involves monitoring the respiratory status for any
signs of pulmonary congestion. Breath sounds are
assessed at regular intervals.
1052 Promote symptom management Which of the following interventions is appropriate for a
. psychiatric-mental health nurse at the basic level of
practice?
1053 Proteins formed when cells are exposed to viral Interferons
. or foreign agents that are capable of activating
other components of the immune system are
referred to as
1054 Publication of the first psychiatric nursing text, The following events are important in the development
. Nursing Mental Disease, by Harriet Bailey of psychiatric-mental health nursing practice. Which
event occurred first?
1055 Pulmonary artery Which of the follow arteries carries deoxygenated
. blood?
1056 Pulmonary Congestion A client is being assessed for his semiannual
. examination and you hear crackles bilaterally in his
lungs. Which of the following could be a cause of
crackles in the bases of his lungs?
1057 Radioallergosorbent testing (RAST) measures IgE
. which of the following immunoglobulins?
1058 Ralph Wilson, is a 52-year-old client in the Maintain the client on bed rest
. hospital unit where you practice nursing. He is
being treated for myocarditis. Which of the
following nursing interventions should you
perform to reduce cardiac workload in a
client with myocarditis?
1059 Ralph Wilson, is a 52-year-old client in the Maintain pt on bed rest
. hospital unit where you practice nursing. He is
being treated for myocarditis. Which of the
following nursing interventions should you
perform to reduce cardiac workload in a
client with myocarditis?
1060 Rebleeding may occur from a peptic ulcer and Mental confusion
. often warrants surgical interventions. Signs of
bleeding include which of the following?
1061 Rebound hypoglycemia is a complication of Feedings stopped too abruptly
. parental nutrition caused by which of the
following?
1062 A recent abduction of a 2-month-old infant Placing an identification bracelet on the infant and the
. has raised awareness of the need for security parent immediately on admission
plans for hospitals. Which security measure Explanation: The safest way to ensure that the parents or
helps ensure the hospitalized infant's security? legal guardians are who they say they are is to place a
bracelet on both the infant and the parents or guardians at
the time of admission. Limiting visitors isn't necessary.
Locking the door and having visitors call the nurses'
station for admission increases the workload of the
nursing staff. It isn't feasible to place security guards at
the entrances.
1063 A recent immigrant from Vietnam is c) Developing a list of people with whom the client has
. diagnosed with pulmonary tuberculosis (TB). had contact
Which intervention is most important for the
nurse to implement with this client?

a) Client teaching about the cause of TB


b) Reviewing the risk factors for TB
c) Developing a list of people with whom the
client has had contact
d) Client teaching about the importance of TB
testing
1064 Regarding oral cancer, the nurse provides many oral cancers produce no symptoms in the early
. health teaching to inform the patient that stages.
1065 Regional anesthesia? - an anesthetic agent is injected around nerves so that the
. area supplied by these nerves is anesthetized.
(Where is it administered?)
1066 A registered nurse who is responsible for circulating nurse.
. coordinating and documenting patient care in
the operating room is a
1067 Reinforce the dressing and contact the The nurse should reinforce the dressing and notify the
. physician. physician. A saturated dressing might signal postoperative
hemorrhage. Continuing to monitor the client without
notifying the physician delays treatment. The nurse
should also monitor the client's vital signs. The first
postoperative dressing should be changed by the
physician, not the nurse.
1068 "Remember to hold the cane with The nurse should remind the client to hold the cane with the hand on
. the hand on the opposite side of the opposite side of the weak leg. Telling the client that the cane is
your weak leg." temporary offers false reassurance. Safe cane walking requires the
client to hold the cane on the side opposite the disability.
1069 Respite residential care The parents of a young adult diagnosed with schizophrenia are
. providing care for the patient in their home. During a home visit, the
parents state, "It's been so difficult taking care of our son. We need a
break. But he needs constant supervision." Which of the following
would be appropriate for the nurse to suggest?
1070 Responding indirectly to A nurse is engaged in active listening. Which of the following would
. statements the nurse use?
Using open-ended statements
Concentrating on what patient
says
1071 Restlessness Early in shock, hyperactivity of the sympathetic nervous system
. causes increased epinephrine secretion, which typically makes the
client restless, anxious, nervous, and irritable. It also decreases tissue
perfusion to the skin, causing pale, cool, clammy skin. An above-
normal heart rate is a late sign of shock. A urine output of 30 ml/hour
is within normal limits.
1072 Restricted zone? ATTIRE CONSISTS OF SCRUB, CLOTHES, CAPS AND MASK
.
Area in the operating room where scrub attire and surgical masks are
required! Includes operating room and sterile core areas!
1073 Right Ventricle Within the physiology of the heart, each chamber has a particular role
. in maintaining cellular oxygenation. Which chamber of the heart is
responsible for pumping blood to the lungs to be oxygenated?
1074 RNFA? Registered nurse first assistant!
. -responsibilities includes (3) -Handling tissue, suturing & maintaning homeostasis.
1075 Role of the circulating nurse? (5) - Provide appropriate environment
. - Veryfying consent
- Coordinating the team
- Monitors aseptic practices
- Monitor patients and documents specific activities throughout the
operation to ensure patient's safety.
1076 roll the vial gently between the Rolling the vial gently between the palms produces heat, which helps
. palms. dissolve the medication. Doing nothing or inverting the vial wouldn't
help dissolve the medication. Shaking the vial vigorously could cause
the medication to break down, altering its action.
1077 rule of nines method used to calculate the amount of fluid lost as the result of a
. burn; divides the body into 11 areas, each accounting for 9% of the
total body area
1078 A school-age child has a fever, joint "Has your child had strep throat recently?"
. inflammation, and a nonpruritic Explanation: Group A beta-hemolytic streptococcal infection
rash. Knowing that these are signs typically precedes rheumatic fever. An inflammatory disease,
of rheumatic fever, the nurse rheumatic fever affects the heart, joints, and central nervous system.
should ask the parents: It isn't infectious and can't be transmitted from one person to another.
Congenital heart defects don't play a role in the development of
rheumatic fever. H. influenzae vaccine doesn't prevent streptococcal
infection or rheumatic fever.
1079 A school nurse is talking about Engulfment and digestion of foreign material and bacteria
. infection with a high school health
class. What would be the nurse's
best explanation of the process of
phagocytosis?
1080 Scrub role? -registered nurse, licensed practical nure and technicians who scrubs
. and dons sterile surgical attire, supplies and hands instruments to the
surgeon.
1081 A secondary immunodeficiency is characterized It usually occurs as a result of underlying disease
. by the following. Choose all that apply. processes.
It frequently is caused by certain autoimmune
disorders.
It may be caused by certain viruses.
1082 (see full question) "I may stop taking this medication when I feel better."
. Explanation: The client requires additional teaching
A nurse is teaching a client with adrenal because he states that he may stop taking
insufficiency about corticosteroids. Which corticosteroids when he feels better. Corticosteroids
statement by the client indicates a need for should be gradually tapered by the physician. Tapering
additional teaching the corticosteroid allows the adrenal gland to gradually
resume functioning. Corticosteroids increase the risk of
infection and may mask the early signs of infection, so
the client should avoid people who are sick.
Corticosteroids cause muscle wasting in the
extremities, so the client should increase his protein
intake by eating foods such as chicken and dairy
products. Corticosteroids have been linked to
glaucoma and corneal lesions, so the client should visit
his ophthalmologist regularly.
1083 (SELECT ALL THAT APPLY) A 35-year-old (3) Begin an I.V. infusion of lactated Ringer's
. client is brought to the emergency department solution.Administer 6 mg of morphine I.V. (6)
with second- and third-degree burns over 15% Administer tetanus prophylaxis, as ordered.
of his body. His admission vital signs are: blood
pressure 100/50 mm Hg, heart rate 130
beats/minute, and respiratory rate 26
breaths/minute. Which nursing interventions are
appropriate for this client?
1084 (SELECT ALL THAT APPLY) A 42-year-old (1) Severe, deep pain around the thorax (2) Red,
. client comes to the clinic and is diagnosed with nodular skin lesions around the thorax (3) Fever (4)
shingles. Which findings confirm this diagnosis? Malaise
1085 (SELECT ALL THAT APPLY) A client who was (1) Reposition the client every 2 hours. (2) Perform
. transferred from a long-term care facility is range-of-motion exercises. Encourage the client to eat
admitted with dehydration and pneumonia. a well-balanced diet.
Which nursing interventions can help prevent
pressure ulcer formation in this client?
1086 (SELECT ALL THAT APPLY) Which "(2) Take tetracycline on an empty stomach. (4)
. instructions should be included in the teaching Maintain the prescribed treatment because it is more
plan of a 19-year-old client with acne vulgaris likely to improve acne than a strict diet and fanatic
who's prescribed tretinoin (Retin-A), benzoyl scrubbing with soap and water.
peroxide, and tetracycline (Achromycin)?
1087 (SELECT ALL THAT APPLY) Which nursing (1) Clean the skin with warm water and a mild
. interventions are effective in preventing pressure cleaning agent; then apply a moisturizer. (4) Turn and
ulcers? reposition the client every 1 to 2 hours unless
contraindicated. (6) Use pillows to position the client
and increase his comfort.
1088 Semirestricted zone? ATTIRE CONSISTS OF SCRUB, CLOTHES AND
. CAPS

- Area where scrub attire etc is required, may include


areas where surgical instruments are processed.
1089 Services that promote the patient's reintegration A patient is referred to a psychosocial rehabilitation
. into the community. program. When explaining this type of care to the
patient, the nurse would emphasize which of the
following?
1090 Services that promote the patient's reintegration A patient is referred to a psychosocial rehabilitation
. into the community program. When explaining this type of care to the
patient, the nurse would emphasize which of the
following?
1091 Sevelamer hydrochloride (Renagel) has been prescribed 6
. for a client with chronic renal failure. The physician has
prescribed Renagel 800 mg orally three times per day
with meals to treat the client's hyperphosphatemia. The
medication is available in 400 mg tablets. How many
tablets per day will the nurse administer to the client?
1092 Several children at a daycare center have been infected Hand washing after diaper changes
. with hepatitis A virus. Which instruction by the nurse Explanation: Children in daycare centers are
would reduce the risk of hepatitis A to other children and at risk of hepatitis A infection which is
staff members? transmitted via fecal-oral route due to poor
hand hygiene practices and poor sanitation.
Isolation of sick children, use of masks during
contact, and sterilization of all eating utensils
would not be useful in breaking the chain of
infection.
1093 Sinoatrial (SA) node, atrioventricular (AV) node, bundle The nurse is teaching a beginning EKG class
. of His, right and left bundle branches, and the Purkinje to staff nurses. As the nurse begins to discuss
fibers the the parts of the EKG complex, one of the
students asks what the normal order of
conduction through the heart is. The correct
response would be which of the following?
1094 A slightly obese client with a history of allergy-induced a) resonant sounds.
. asthma, hypertension, and mitral valve prolapse is
admitted to an acute care facility for elective surgery. The
nurse obtains a complete history and performs a
thorough physical examination, paying special attention
to the cardiovascular and respiratory systems. When
percussing the client's chest wall, the nurse expects to
elicit:

a) resonant sounds.
b) hyperresonant sounds.
c) dull sounds.
d) flat sounds.
1095 Sodium Admission lab values on a patient admitted
. with congestive heart failure are as follows:
potassium 3.4 mEq/L; sodium 148 mEq/L;
calcium 9.8 mg/dL; and magnesium 1.5
mEq/L. Which lab value is abnormal?
1096 A son brings his father into the clinic, stating that his Cyanosis
. father's color has changed to bluish around the mouth.
The father is confused, with a respiratory rate of 28
breaths per minute and scattered crackles throughout.
The son states this condition just occurred within the last
hour. Which of the following factors indicates that the
client's condition has lasted for more than 1 hour?
1097 Spinal anesthesia? (Where?) - Subarachnoid space of the spinal cord.
.

1098 stethescope; diaphragm; bell and palpation The diaphragm of a stethoscope detects high-
. pitched sounds best; the bell detects low-
pitched sounds best. Palpation detects thrills
best.
1099 A student nurse is caring for a client who is severely "The cells are denied adequate oxygen
. anemic. The instructor asks the student how anemia because most of the oxygen in the body is
affects the transport of oxygen to the cells. What would transported by the hemoglobin in red blood
be the student's best answer? cells."
1100. A student nurse is preparing a plan of care for a client Impaired nutrition: less than body
with chronic pancreatitis. What nursing diagnosis related requirements
to the care of a client with chronic pancreatitis is the
priority?
1101. A student nurse is working with a client who is diagnosed Regular breathing where the rate and depth
with head trauma. The nurse has documented Cheyne- increase, then decrease
Stokes respirations. The student would expect to see
which of the following?
1102. Students are reviewing information from the Centers Nonlatex lambskin condoms are highly effective
for Disease Control and Prevention (CDC) for a class in preventing HIV infection
presentation about preventing the transmission of
HIV transmission. Which of the following would the
students be least likely to include in their
presentation?
1103. Students are reviewing the concepts of phenotype Mutations in low-density lipoprotein (LDL)
and genotype as they apply to hypercholesterolemia. receptors
The students demonstrate an understanding by Disruption in an apolipoprotein gene
identifying which of the following as characteristic of
the genotype of this disease? Select all that apply.
1104. Subjective family burden that occurs in many The Lawson family has been caring for Randy,
families who have a mentally ill loved one. their 35-year-old son with schizophrenia, for about
15 years. They report that they often are fearful
that Randy will become psychotic and hurt
someone in public. They are sad because they
remember that when Randy was in high school, he
was a star student and athlete, and they enjoyed
watching him play football. These feelings of the
family can best be described as
1105. Sudoriferous glands secrete which type of substance? Sweat
1106. The surgical area is divided into 3 1) Unrestricted zone
2) Semirestricted zone
3) Restricted zone
1107. Surgical asepsis? - Absence of microorganism in the surgical
environment to reduce risk of infection.
1108. The surgical client has been intubated and general Stage IV
anesthesia has been administered. The client exhibits
cyanosis, shallow respirations, and a weak, thready
pulse. The nurse recognizes that the client is in which
stage of general anesthesia?
1109. Surgical safety checklist: 1) Identity
2) Safety
1) Patient's ___ 3) Known or unknown?
2) Anesthesia ___ 4) oximeter
3) Allergies?__or___ 5) aspiration
4) Pulse ____ on patient and functioning? 6) Loss!
5) Difficult aiway and ___risk?
6) Risk of blood ___?
1110. The surgical team: Consist of? Patient, anesthesiologist or anesthetist, surgeon,
nurses &surgical technologist/ assistants!
1111. Susan Hopkins, a 32-year-old administrative Indwelling catheter
assistant, is being seen by a physician with the Decreased fluid intake
urology practice where you practice nursing. She has
a history of neurogenic bladder and uses a
permanent, indwelling catheter to facilitate urine
elimination. What contributes to the likelihood of
developing urinary tract or bladder infections?
Choose all correct options.
1112. Symptoms associated with pyloric obstruction Diarrhea
include all of the following except:
1113. Symptoms of progressive gastric cancer include Bloating after meals
which of the following?
1114. T-cell and B-cell lymphocytes are the primary T-cell and B-cell lymphocytes distinguish harmful
participants in the immune response. What do they substances and ignore those natural and unique to
do? a person.
1115. T-cell deficiency occurs when which of the following Thymus
glands fails to develop normally during
embryogenesis?
1116. T-cells can be either regulator T cells or effector Fighting infection
T cells. Regulator T cells are made up of helper
and suppressor cells. What function are helper
T-cells important in?
1117. A teenager is brought to the facility by friends Respiratory system
after accidentally ingesting gasoline while Explanation: The primary concern with petroleum
siphoning it from a car. Based on the nurse's distillate ingestion is its effect on the respiratory system.
knowledge of petroleum distillates, which Aspiration or absorption of petroleum distillates can
system would be most affected? cause severe chemical pneumonitis and impaired gas
exchange. The GI, neurologic, and cardiovascular
systems may be affected if the petroleum contains
additives such as pesticides.
1118. A teenager with heart failure prescribed a cardiac glycoside
digoxin (Lanoxin) asks the nurse, "What's the Explanation: Digoxin is a cardiac glycoside. It decreases
drug supposed to do?" The nurse responds to the workload of the heart and improves myocardial
the teenager based on the understanding that function. ACE inhibitors cause vasodilation and
this drug is classified as: increase sodium excretion. Diuretics help remove
excess fluid. Vasodilators enhance cardiac output by
decreasing afterload.
1119. The term for a reddened circumscribed lesion Chancre
that ulcerates and becomes crusted and is a
primary lesion of syphilis is a
1120. The term that describes the percentage of Penetrance
individuals known to carry the gene for a trait
and who actually manifest the condition is
1121. The term used to define the balance between Viral Set Point
the amount of HIV in the body and the immune
response is
1122. The term used to define the balance between Viral set point
the amount of HIV in the body and the immune
response is
1123. Thallium-201 Your client is going to have a stress test. What
radionuclide would most likely be used to diagnose
ischemic heart disease during this test?
1124. That all behavior is meaningful and can be An adolescent has a history of self-mutilation. The nurse
understood from the person's perspective questions the client about her behavior. The nurse's
questioning reflects which principle?
1125. "The only difference is the rate, which will be A nursing student is caring for one of the nurse's
below 60 bpm in sinus bradycardia." assigned cardiac clients. The student asks, "How can I
tell the difference between sinus rhythm and sinus
bradycardia when I look at the EKG strip" The best
reply by the nurse is which of the following?
1126. "Theory provides the focus for my nursing care A nurse shows an understanding of the impact of
of depressed clients." nursing theory on nursing practice when stating
1127. There are many ethical issues in the care of Disclosure of the patient's condition
clients with HIV or HIV/AIDS. What is an
ethical issue healthcare providers deal with
when caring for clients with HIV/AIDS?
1128. They require rephrasing of unclear questions. Which of the following is an inaccurate depiction of a
concrete question?
1129. This example of cholesterol gallstones (left side 75%
of picture) is the result of decreased bile acid
synthesis and increased cholesterol synthesis in
the liver, which in turn, form stones.
Cholesterol stones account for what percentage
of cases of gallbladder disease in the United
States?
1130. To avoid recording an erroneously low inflate the cuff at least another 30 mm Hg after the radial
systolic blood pressure because of failure to pulse becomes impalpable.
recognize an auscultatory gap, the nurse Explanation: The nurse should wrap an appropriately sized
should: cuff around the client's upper arm and then place the
diaphragm of the stethoscope over the brachial artery. The
nurse should then rapidly inflate the cuff until she can no
longer palpate or auscultate the pulse and continue inflating
until the pressure rises another 30 mm Hg. The other options
aren't appropriate measures.
1131. To compensate for decreased fluid volume Tachycardia - Fluid volume deficit, or hypovolemia, occurs
(hypovolemia), the nurse can anticipate when the loss of extracellular fluid exceeds the intake...
which response by the body? (more)
Fluid volume deficit, or hypovolemia, occurs when the loss
of extracellular fluid exceeds the intake of fluid. Clinical
signs include oliguia, rapid heart rate, vasoconstriction, cool
and clammy skin, and muscle weakness. The nurse monitors
for rapid, weak pulse and orthostatic hypotension.
1132. To compensate for decreased fluid volume Tachycardia - Fluid volume deficit, or hypovolemia, occurs
(hypovolemia), the nurse can anticipate when the loss of extracellular fluid exceeds the intake of
which response by the body? fluid. Clinical signs include oliguia, rapid heart rate,
vasoconstriction, cool and clammy skin, and muscle
weakness. The nurse monitors for rapid, weak pulse and
orthostatic hypotension.
1133. A toddler is brought to the emergency Intraosseously
department in cardiac arrest. The Explanation: The physician can safely administer emergency
physician tries three times to insert an I.V. medications, such as sodium bicarbonate, calcium, glucose,
catheter but is unsuccessful. By which crystalloids, colloids, blood, dopamine, epinephrine, and
alternate route can the physician dobutamine by the intraosseous route if the I.V. route is
administer emergency medications? inaccessible. Emergency medications shouldn't be
administered by the sublingual, topical, or subcutaneous
routes.
1134. A toddler is brought to the emergency a barium enema.
department with sudden onset of Explanation: A barium enema commonly is used to confirm
abdominal pain, vomiting, and stools that and correct intussusception. Performing a suprapubic
look like red currant jelly. To confirm aspiration or inserting an NG tube or an indwelling urinary
intussusception, the suspected cause of catheter wouldn't help diagnose or treat this disorder.
these findings, the nurse expects the
physician to order:
1135. To ensure patency of central venous line Daily, when not in use
ports, diluted heparin flushes are used in
which of the following situations?
1136. To evaluate a client for hypoxia, the Arterial blood gas (ABG) analysis - Red blood cell count,
physician is most likely to order which sputum culture, total hemoglobin, and ABG analysis all help
laboratory test? evaluate a client with respiratory problems. However, ABG
analysis is the only test that evaluates gas exchange in the
lungs, providing information about the client's oxygenation
status.
1137. To evaluate a client's cerebellar function, a "Do you have any problems with balance?"
nurse should ask:
1138. To give a comprehensive picture of client What is the purpose of the five-axis system used in the fourth
functioning edition of the Diagnostic and Statistical Manual of Mental
Disorders, text revision (DSM-IV-TR)?
1139. To implant an intracranial pressure Dura mater
monitor, what membranes will the surgeon Arachnoid
need to penetrate? Choose all correct Pia mater
responses.
1140. To minimize their dependency on The Community Mental Health Centers Act mandates that
institutionalized care. communities make psychiatric emergency care available to
its population. The benefit of this mandate to the chronically
mentally ill is..
1141. The tongue In many cases, the muscles controlling the tongue relax,
causing the tongue to obstruct the airway. When this occurs,
the nurse should use the head-tilt, chin-lift maneuver to cause
the tongue to fall back in place. If a neck injury is suspected,
the jaw- thrust maneuver must be performed. A foreign
object, saliva or mucus, and edema are less common sources
of airway obstruction in an unconscious adult.
1142. To prevent gastroesophageal reflux in a client with Avoid coffee and alcoholic beverages
hiatal hernia, the nurse should provide which
discharge instruction?
1143. To treat a client with acne vulgaris, the physician is Tretinoin (retinoic acid [Retin-A])
most likely to prescribe which topical agent for nightly
application?
1144. Translocation is a term used to describe the general Osmosis - Osmosis is the movement of water
movement of fluid and chemicals within body fluids. through a semipermeable membrane, one that
In every client's body, fluid-electrolyte balance is allows some but not all substances in a solution
maintained through the process of translocation. What to pass through, from a dilute area to a more
specific process allows water to pass through a concentrated area.
membrane from a dilute to a more concentrated area?
1145. A trauma victim in the intensive care unit has a a) Decreased cardiac output
tension pneumothorax. Which signs or symptoms are d) Hypotension
associated with a tension pneumothorax? e) Tracheal deviation to the opposite side

a) Decreased cardiac output


b) Flattened neck veins
c) Tracheal deviation to the affected side
d) Hypotension
e) Tracheal deviation to the opposite side
f) Bradypnea
1146. A treatment option for SCID includes Stem cell transplant
1147. Troponin T and I The nurse is awaiting results of cardiac
biomarkers for a patient with severe chest pain.
The nurse would identify which cardiac
biomarker as remaining elevated the longest
when myocardial damage has occurred?
1148. Two days after undergoing a total abdominal left calf circumference 1" (2.5 cm) larger than
hysterectomy, a client complains of left calf pain, and the right.
venography reveals deep vein thrombosis (DVT). Explanation: Signs of DVT include inflammation
When checking this client, the nurse is most likely to and edema in the affected extremity, causing its
detect: circumference to exceed that of the opposite
extremity. Pallor, coolness, decreased pulse, and
hair loss in an extremity signal interrupted
arterial blood flow, which doesn't occur in DVT.
1149. The two types of inherited B-cell deficiencies result Mature B-cells
from lack of differentiation of B cells. These types Plasma cells
result from which two of the following deficiencies?
Choose the two that apply.
1150. Tyramine Patients taking monoamine oxidase inhibitors
(MAOIs) for depression must be placed on a diet
that is low in which of the following?
1151. Undifferentiated cells that migrate to the thymus T lymphocytes
gland develop into which of the following?
1152. An unemployed woman, age 24, seeks help Lack of self-esteem, strong dependency needs, and
because she feels depressed and abandoned and impulsive behavior
doesn't know what to do with her life. Last Explanation:
week, her boyfriend broke up with her after she Borderline personality disorder is characterized by lack
drove his car into a tree after an argument. The of self-esteem, strong dependency needs, and impulsive
client's initial diagnosis is borderline behavior. Instability in interpersonal relationships,
personality disorder. Which nursing mood, and poor self-image also are common. The client
observations support this diagnosis? typically can't tolerate being alone and expresses
feelings of emptiness or boredom. Flat affect, social
withdrawal, and unusual dress are characteristic of
schizoid personality disorder. Suspiciousness,
hypervigilance, and emotional coldness are seen in
paranoid personality disorders. In antisocial personality
disorder, clients are usually insensitive to others and act
out sexually; they may also be violent.
1153. Unintentional retention of object will cause ___ Wound infection
___.
- abcesses
What can form? - Fistula between organs
What may develop?
1154. Unrestricted zone? STREET CLOTHES ARE ALLOWED

in the operating room that interfaces with other


departments, including patien't reception and holding
area!
1155. Upon discharge from the hospital, patients Morphine IV
diagnosed with a myocardial infarction (MI)
must be placed on all of the following
medications except:
1156. Upon palpation of the sinus area, what would No sensation during palpation
the nurse identify as a normal finding?
1157. Upon receiving the dinner tray for a client Hot roast beef sandwich with gravy
admitted with acute gallbladder inflammation,
the nurse will question which of the following
foods on the tray?
1158. Use team-building exercises. The nurse has begun group counseling sessions for
several hospitalized patients in the psychiatric facility.
Which of the following would be most effective for the
nurse to do to promote group cohesiveness?
1159. Vomiting results in which of the following acid- Metabolic alkalosis
base imbalances?
1160. Wallace Guterman, a 36-year-old construction Hyperpituitarism:
manager, is being seen by a physician in the
primary care group where you practice Explanation: Acromegaly is a condition in which GH is
nursing. He presents with a huge lower jaw, oversecreted after the epiphyses of the long bones have
bulging forehead, large hands and feet and sealed. A client with acromegaly has coarse features, a
frequent headaches. What could be causing his huge lower jaw, thick lips, a thickened tongue, a bulging
symptoms? forehead, a bulbous nose, and large hands and feet.
When the overgrowth is from a tumor, headaches caused
by pressure on the sella turcica are common.
1161. Weighing the client daily at the same time each A client is at risk for excess fluid volume. Which
day. nursing intervention ensures the most accurate
monitoring of the client's fluid status?
1162. Welfare of the patient A psychiatric nurse working in an inpatient facility
Legal welfare of the nurse understands the importance of accurate documentation
Legal welfare of the institution and explains to a new graduate that documentation is
done for which reasons?
1163. What are antigens? Protein markers on cells.
1164. What are antigens? Protein markers on cells
1165. What chemical is released by cytotoxic T cells? Lymphokine
1166. What factor in plasma can be transformed Fibrinogen
from a liquid to a solid?
1167. What is hematopoiesis? The manufacture and development of blood cells
1168. What is the difference between Ventilation is the movement of air in and out of the respiratory
respiration and ventilation? tract.
1169. What is the function of the thymus Programs T lymphocytes to become regulator or effector T cells.
gland?
1170. What is the major purpose of Prevent aspiration
withholding food and fluid before
surgery?
1171. What is the most appropriate nursing Deficient fluid volume
diagnosis for the client with acute
pancreatitis?
1172. What is the most important "Remain supine for the time specified by the physician."
postoperative instruction a nurse must
give to a client who has just returned
from the operating room after receiving
a subarachnoid block?
1173. What is the normal pH range for b) 7.35 to 7.45
arterial blood?

a) 7 to 7.49
b) 7.35 to 7.45
c) 7.50 to 7.60
d) 7.55 to 7.65
1174. What is the term for the concentration Azotemia
of urea and other nitrogenous wastes in
the blood?
1175. What kind of solution is used to clean - Antiseptic
patient's skin?
1176. What method of communication can be Using gestures
used with a hearing-impaired patient? Correct
Explanation:
a) Talking into the more-impaired ear Strategies such as talking into the less-impaired ear and using
b) Using gestures gestures and facial expressions can help. Therefore options A, C,
c) Grimacing and D are incorrect
d) Talking loudly
1177. What organ is considered lymphoid Spleen
tissue?
1178. What part of the brain controls and Cerebellum
coordinates muscle movement?
1179. What safety actions does the nurse Ensure that no patient care equipment containing metal enters the
need to take for a patient on oxygen room where the MRI is located.
therapy who is undergoing magnetic
resonance imaging (MRI)?
1180. What's the best way for a nurse to Left side-lying
position a 3-year-old child with right Explanation: The child with right lower lobe pneumonia should
lower lobe pneumonia? be placed on his left side. This places the unaffected left lung in a
position that allows gravity to promote blood flow though the
healthy lung tissue and improve gas exchange. Placing the child
on his right side, back, or stomach doesn't promote circulation to
the unaffected lung.
1181. When a central venous catheter Remove the dressing, clean the site, and apply a new dressing.
dressing becomes moist or loose, what
should a nurse do first?
1182. When a client's ventilation is impaired, the b) Carbon dioxide
body retains which substance?

a) Sodium bicarbonate
b) Carbon dioxide
c) Nitrous oxide
d) Oxygen
1183. When administering intravenous gamma Flank Pain
globulin infusion, the nurse recognizes that
which of the following complaints, if reported
by the client, may indicate an adverse effect
of the infusion?
1184. When an attenuated toxin is administered to Artificially acquired active immunity
a client, the B lymphocytes create memory
cells that recognize the antigen if it invades
the body at a future time. What kind of
immunity is this?
1185. When assessing a client during a routine Chancre sore of the oral soft tissues
checkup, the nurse reviews the history and
notes that the client had aphthous stomatitis
at the time of the last visit. Aphthous
stomatitis is best described as:
1186. When assessing a client, which adaptation Orthopnea
indicates the presence of respiratory distress?
1187. When assessing a client who reports recent "The pain occurred while I was mowing the lawn."
chest pain, the nurse obtains a thorough
history. Which client statement most strongly
suggests angina pectoris?
1188. When assessing a client who reports recent The pain occurred while I was mowing the lawn
chest pain, the nurse obtains a thorough
history. Which client statement most strongly
suggests angina pectoris?
1189. When assessing a client with glaucoma, the Complaints of halos around lights
nurse expects which finding? Explanation:
Glaucoma is largely asymptomatic. Symptoms that occur
can include loss of peripheral vision or blind spots,
reddened sclera, firm globe, decreased accommodation,
halos around lights, and occasional eye pain. Normal
intraocular pressure is 10 to 21 mm Hg.
1190. When assessing a client with partial thickness Hoarseness of the voice
burns over 60% of the body, which finding
should the nurse report immediately?
1191. When assessing for signs and symptoms A client undergoing cancer treatment
related to hematopoietic and lymphatic
systems, what details should the nurse ask
about further?
1192. When assessing whether a patient is a Health status
candidate for home parenteral nutrition, Family support
which of the following would be important to Motivation for learning
address? Select all that apply. Telephone access
1193. When assisting in developing a plan of care Preschool age
for a hospitalized child, the nurse knows that Explanation: School-age children are most likely to view
children in which age-group are most likely to illness as a punishment for misdeeds. Separation anxiety,
view illness as a punishment for misdeeds? although seen in all age-groups, is most common in older
infants. Fear of death is typical of older children and
adolescents.
1194. When assisting the patient to interpret a his body has not produced antibodies to the AIDS virus.
negative HIV test result, the nurse informs
the patient that the results mean that
1195. When assisting the patient to interpret a negative Antibodies to HIV are not present in his blood.
HIV test result, the nurse informs the patient that
the results mean which of the following?
1196. When assisting with developing a plan of care for a maintaining the client's fluid, electrolyte, and acid-
client recovering from a serious thermal burn, the base balance.
nurse knows that the most important immediate
goal of therapy is:
1197. When bowel sounds are heard about every 15 Normal
seconds, the nurse would record that the bowel
sounds are
1198. When caring for a client who is a Mormon, the Beer
nurse notices something on the lunch tray that
should be removed or substituted out of respect for
the client's religion. Which of the following would
be an item to remove?
1199. When caring for a client with acute respiratory d) Hypercapnia, hypoventilation, and hypoxemia
failure, the nurse should expect to focus on
resolving which set of problems?

a) Hypotension, hyperoxemia, and hypercapnia


b) Hyperventilation, hypertension, and
hypocapnia
c) Hyperoxemia, hypocapnia, and hyperventilation
d) Hypercapnia, hypoventilation, and hypoxemia
1200 When caring for a client with an acute Assess abdomen and vitals
. exacerbation of a peptic ulcer, the nurse finds the
client doubled up in bed with severe pain to his
right shoulder. The intial appropriate action by the
nurse is to
1201 When caring for a client with cirrhosis, which of Change in mental status
. the following should a nurse notify immediately? Signs of GI bleeding
Choose all correct options.
1202 When caring for a client with hepatitis B, the nurse Irritability and drowsiness
. should monitor closely for the development of
which finding associated with a decrease in hepatic
function?
1203 When caring for a client with severe impetigo, the Administering systemic antibiotics as prescribed
. nurse should expect which intervention in the plan
of care?
1204 When caring for a patient with cirrhosis, which of Change in mental status
. the following symptoms should the nurse report
immediately?
1205 When caring for the patient with acute Tea-colored urine
. glomerulonephritis, which of the following
assessment findings should the nurse anticipate?
1206 When checking a client's I.V. insertion site, the Check the tubing for kinks and reposition the client's
. nurse notes normal color and temperature at the wrist and elbow.
site and no swelling. However, the I.V. solutions Explanation: The nurse should first check for
haven't infused at the ordered rate; the flow rate is common causes of a decreased I.V. flow rate, such as
slow even with the roller clamp wide open. When kinks in the tubing and poor positioning of the
the nurse lowers the I.V. fluid bag, no blood affected arm. The nurse should discontinue the I.V.
returns to the tubing. What should the nurse do infusion only if other measures fail to solve the
first? problem. Irrigating I.V. tubing may dislodge clots, if
present. Elevating the I.V. fluid bag may help if no
kinks are found and if repositioning doesn't resolve
the problem.
1207 When collecting data on a child with juvenile Goiter
. hypothyroidism, the nurse expects which finding? Explanation: Juvenile hypothyroidism results in
goiter, weight gain, sleepiness, and a slow heart rate.
It doesn't cause weight loss, insomnia, or
tachycardia.
1208 When collecting data on a client who has just been wear gloves when providing care and isolate the
. admitted to the medical-surgical unit, the nurse client's bed linens until the client is no longer
discovers scabies. To prevent scabies infection in infectious.
other clients, the nurse should:
1209 When counseling parents of a neonate with Duration of the condition before treatment
. congenital hypothyroidism, the nurse Explanation: The severity of the intellectual deficit is related
understands that the severity of the to the degree of hypothyroidism and the duration of the
intellectual deficit is related to which condition before treatment. Cranial malformations don't
parameter? affect the severity of the intellectual deficit, nor does the
degree of hypothermia as it relates to hypothyroidism. It isn't
the specific T4 level at diagnosis that affects the intellect but
how long the child has been hospitalized.
1210 When describing the role of the pancreas to Triglycerides
. a client with a pancreatic dysfunction, the
nurse would identify which substance as
being acted on by pancreatic lipase?
1211. When developing a care plan for a client The stroke may have impacted the body's thermoregulation
who has recently suffered a stroke, a nurse centers.
includes the nursing diagnosis Risk for
imbalanced body temperature. What is the
rationale for this diagnosis?
1212 When do most perinatal HIV infections After exposure during delivery
. occur?
1213 When evaluating arterial blood gases pH 7.48 - Metabolic alkalosis is a clinical disturbance
. (ABGs), which value is consistent with characterized by a high pH and high plasma bicarbonate
metabolic alkalosis? concentration. The HCO value is below normal. The PaCO
value and the oxygen saturation level are within a normal
range.
1214 When evaluating a severely depressed a preoccupation with death.
. adolescent, the nurse knows that one Explanation: An adolescent who demonstrates a
indicator of a high risk for suicide is: preoccupation with death (such as by talking about death
frequently) should be considered at high risk for suicide.
Although depression, excessive sleepiness, and a history of
cocaine use may occur in suicidal adolescents, they also
occur in adolescents who aren't suicidal.
1215 When examining the abdomen of a client Inspection
. with complaints of nausea and vomiting,
which of the following would the nurse do
first?
1216 When gastric analysis testing reveals excess Duodenal ulcer
. secretion of gastric acid, which of the
following medical diagnoses is supported?
1217 When interviewing a client with internal Rectal Bleeding
. hemorrhoids, which of the following would
the nurse expect the client to report?
1218 When interviewing a client with internal Rectal bleeding
. hemorrhoids, which of the following would
the nurse expect the client to report?
1219 When is the ideal time to discuss Preadmission visit
. preoperative teaching
1220 When monitoring a patient who has 130/80 mmHg
. hypertension and chronic kidney disease,
the target pressure for this individual
should be less than which blood pressure
reading?
1221 When obtaining the vital signs of a client Increased intracranial pressure (ICP)
. with multiple traumatic injuries, the nurse Explanation:
detects bradycardia, bradypnea, and Decreased heart and respiratory rates and increased systolic
systolic hypertension. The nurse must blood pressure reflect Cushing's triad, which may develop
notify the physician immediately because when ICP increases. Shock typically causes tachycardia,
these findings may reflect which tachypnea, and hypotension. In encephalitis, the temperature
complication? rises and the heart and respiratory rates may increase from
the effects of fever on the metabolic rate. If the client doesn't
maintain adequate hydration, hypotension may occur. Status
epilepticus causes unceasing seizures, not changes in vital
signs.
1222 When performing a physical examination Measuring abdominal girth
. on a client with cirrhosis, a nurse notices
that the client's abdomen is enlarged.
Which of the following interventions should
the nurse consider?
1223 When performing a physical examination Measure abdominal girth according to a set routine.
. on a client with cirrhosis, a nurse notices
that the client's abdomen is enlarged.
Which of the following interventions should
the nurse consider?
1224 When preparing a client for hemodialysis, Check for thrill or bruit over the access site.
. which of the following would be most
important for the nurse to do?
1225 When preparing a client for magnetic "I really don't like to be in small, enclosed spaces."
. resonance imaging (MRI) of the abdomen,
which statement would indicate the need to
notify the physician?
1226 When preparing a client for surgery to Obstruction of the appendix reduces arterial flow, leading to
. treat appendicitis, the nurse formulates a ischemia, inflammation, and rupture of the appendix.
nursing diagnosis of Risk for infection
related to inflammation, perforation, and
surgery. What is the rationale for choosing
this nursing diagnosis?
1227 When preparing a client with acquired Avoid sharing things such as toothbrushes and razors
. immunodeficiency syndrome (AIDS) for
discharge to home, the nurse should be sure
to include which instruction?
1228 When preparing to feed an infant with Burp the infant frequently
. pyloric stenosis, which intervention is Explanation: Infants with pyloric stenosis usually swallow a
important? lot of air from sucking on their hands and fingers because of
their intense hunger (feedings aren't easily tolerated).
Burping often lessens gastric distention and increases the
likelihood the infant will retain the feeding. Feedings are
given slowly with the infant lying in a semiupright position.
Parental participation should be encouraged and allowed to
the extent possible. Record the type, amount, and character
of the vomit as well as its relation to the feeding. The
amount of feeding volume lost is usually refed.
1229 When reviewing the history of a client with Heredity
. Crohn disease, which factor would the Explanation: Although the definitive cause of Crohn disease
nurse associate with this disorder? is unknown, it's thought to be associated with infectious,
immune or psychological, factors. Because it has a higher
incidence in siblings, it may have a genetic cause.
Constipation isn't linked to Crohn disease. On the contrary,
Crohn disease causes bouts of diarrhea. Diet may contribute
to exacerbations of Crohn disease but isn't considered a
cause. A lack of exercise isn't considered a cause of Crohn
disease.
1230 When reviewing the history of a client with History of pancreatitis
. pancreatic cancer, the nurse would identify
which of the following as a possible risk
factor?
1231 When reviewing the laboratory test Low levels of IgM
. results of a client with X-linked Absent B cells
agammaglobulinemia, which of the
following would be most likely? Select all
that apply.
1232 When talking with 10- and 11-year-old The children will be curious about the physical aspects of
. children about death, the nurse should death
incorporate which guides? Select all that The children will know that death is inevitable and
apply: irreversible
The children will be influenced by the attitudes of the adults
in their lives
Explanation: School-age children are curious about the
physical aspects of death and may wonder what happens to the
body. By age 9 or 10, most children know that death is
universal, inevitable, and irreversible. Their cognitive abilities
are advanced and they respond well to logical explanations.
They should be encouraged to ask questions. Because the
adults in their environment influence their attitudes towards
death, they should be encouraged to include children in the
family rituals and be prepared to answer questions that may
seem shocking. Teaching about death should begin early in
childhood. Comparing death to sleep can be frightening for
children and cause them to fear falling asleep.
1233 When teaching a client with intertrigo Never apply cornstarch to the affected areas.
. about prescribed skin care measures, the
nurse should include which important
instruction to prevent Candida albicans
overgrowth?
1234 When the balloon on the distal tip of a pulmonary artery wedge pressure.
. pulmonary artery catheter is inflated and Explanation:
a pressure is measured, the measurement When the balloon is inflated, the tip of the catheter floats into
obtained is referred to as the smaller branches of the pulmonary artery until it can no longer
be passed and the pressure is recorded, reflecting left atrial
pressure and left ventricular end-diastolic pressure. Central
venous pressure is measured in the right atrium. Pulmonary
artery pressure is measured when the balloon tip is not
inflated. Cardiac output is determined through thermodilution
involving injection of fluid into the pulmonary artery catheter.
1235 When the nurse notes that the post Inadequate fluid volume
. cardiac surgery patient demonstrates low
urine output (< 25 mL/hr) with high
specific gravity (> 1.025), the nurse
suspects:
1236 When the parents of an infant diagnosed Withhold the medication and call the primary health care
. with hypothyroidism have been taught to provider
count the pulse, which intervention Explanation: If parents have been taught to count the pulse of
should the nurse teach them in case they an infant diagnosed with hypothyroidism, they should be
obtain a high pulse rate? instructed to withhold the dose and consult their primary
health care provider if the pulse rate is above a certain value.
1237 "When you exercise, try to avoid doing so A young client with a new diagnosis of bipolar I disorder is
. at the hottest times of the day." scheduled to begin therapy with lithium. Which of the
following instructions should the nurse provide to this client?
1238 Which action by the nurse displays client Ensuring client privacy by pulling the curtain closed
. advocacy during a skin assessment?
1239 Which action should a nurse take Institute isolation precautions according to facility policy.
. first when admitting a client with
herpes zoster infection?
1240 Which action should a nurse take Institute isolation precautions according to facility policy.
. first when admitting a client with Explanation: The nurse should first institute isolation precautions to
herpes zoster infection? prevent the spread of the herpes zoster infection. After isolation
precautions are in effect, the nurse can instruct the client to wear light
clothing and provide a tepid bath to promote client comfort. The nurse
should also caution the client against scratching the lesions because
that might cause infection and scarring.
1241 Which adverse effect can be Orange body secretions
. expected by the parents of a 2- Explanation: Rifampin and its metabolites will turn urine, feces,
year-old child who has been sputum, tears, and sweat an orange color. This isn't a serious adverse
started on rifampin after testing effect. Rifampin may also cause GI upset, headache, drowsiness,
positive for tuberculosis? dizziness, vision disturbances, and fever. Liver enzyme and bilirubin
levels increase because of hepatic metabolism of the drug. Parents
should be taught the signs and symptoms of hepatitis and
hyperbilirubinemia such as jaundice of the sclera or skin.
1242 Which assessment finding would Barrel-shaped chest
. be most consistent with advanced
emphysema?
1243 Which assessment finding would Vascular Lesions
. the nurse expect to document for
a client with ataxia-telangiectasis?
1244 Which blood test confirms the ELISA
. presence of antibodies to HIV?
1245 Which category of HIV disease A
. correlates with asymptomatic
AIDS?
1246 Which cells are converted to red Myeloid stem cells
. blood cells, white blood cells, and
platelets?
1247 Which cells are white blood cells Lymphocytes
. with immune functions?
1248 Which cells have the lifespan of Platelets
. approximately 7.5 days and one-
third of their population remains
in the spleen (unless needed to
fight significant bleeding)?
1249 Which cells have the major Erythrocytes
. function of transporting O2 to
and removing CO2 from tissues?
1250 Which characteristics would Loose associations, grandiose delusions, and auditory hallucinations
. the nurse expect to see in the Explanation:
client with schizophrenia? Loose associations, grandiose delusions, and auditory hallucinations are all
characteristic of the classic schizophrenic client. These clients aren't able to
care for their physical appearance. They frequently hear voices telling
them to do something either to themselves or to others. Additionally, they
verbally ramble from one topic to the next. Periods of hyperactivity and
irritability alternating with depression are characteristic of bipolar disorder.
Delusions of jealousy and persecution, paranoia, and mistrust are
characteristics of paranoid disorders. Sadness, apathy, feelings of
worthlessness, anorexia, and weight loss are characteristics of depression.
1251 Which client requires presents with a rigid, boardlike abdomen.
. immediate nursing
intervention? The client who:
1252 Which clinical finding should Uremia
. a nurse look for in a client
with chronic renal failure?
1253 Which condition most Atherosclerosis
. commonly results in coronary Explanation: Atherosclerosis, or plaque formation, is the leading cause of
artery disease (CAD)? CAD. Diabetes mellitus is a risk factor for CAD but isn't the most common
cause. Renal failure doesn't cause CAD, but the two conditions are related.
MI is commonly a result of CAD.
1254 Which condition most Atherosclerosis
. commonly results in coronary
artery disease (CAD)?
1255 Which conduction block Paravertebral anesthesia
. produces anesthesia of the
nerves supplying the chest,
abdominal wall, and
extremities?
1256 Which diagnostic is more PET scan
. accurate in detecting
malignancies than a CT scan?
1257 Which diagnostic produces PET Scan
. images of the body by
detecting the radiation
emitted from radioactive
substances?
1258 Which diagnostic test Viral Load
. measures HIV RNA in the
plasma?
1259 Which diet plan is Increase caloric content per ounce
. recommended for an infant Explanation: Formulas with increased caloric content are given to meet
with heart failure? the greater caloric requirements from the overworked heart and labored
breathing. Fluid restriction and low-sodium formulas aren't
recommended. An infant's nutritional needs depend on fluid. Daily
weights at the same time of the day on the same scale before feedings are
recommended to follow trends in nutritional stability and diuresis. Low-
sodium formulas may cause hyponatremia.
1260 Which drug is most commonly Dopamine (Intropin)
. used to treat cardiogenic
shock?
1261 Which drug will the physician Flumazenil (Romazicon)
. most likely prescribe for the Explanation:
client admitted with a Flumazenil reverses the sedative effects of benzodiazepines such as
lorazepam (Ativan) overdose? lorazepam. The antidote for acetaminophen toxicity is acetylcysteine. It
enhances conversion of toxic metabolites to nontoxic metabolites.
Deferoxamine mesylate is the antidote for iron intoxication. Succimer is
an antidote for lead poisoning.
1262 Which factor in a client's Use of corticosteroids
. history indicates she's at risk Explanation: Small numbers of the fungus Candida albicans commonly
for candidiasis? inhabit the vagina. Because corticosteroids decrease host defense, they
increase the risk of candidiasis. Pregnancy, not nulliparity, increases the
risk of candidiasis. Candidiasis is rare before menarche and after
menopause. The use of hormonal contraceptives, not spermicidal jelly,
increases the risk of candidiasis.
1263 Which factor will most likely Inefficient liver function
. decrease drug metabolism Explanation: Inefficient liver function will most likely decrease drug
during infancy? metabolism during infancy. As the liver matures during the 1st year of
life, drug metabolism improves. Decreased glomerular filtration and
increased tubular secretion may affect drug excretion rather than
metabolism; reduced protein-binding ability may affect drug distribution
but not metabolism.
1264 Which finding is an early Painless, intermittent hematuria
. indicator of bladder cancer? Explanation: As cancer cells destroy normal bladder tissue, bleeding
occurs and causes painless, intermittent hematuria. (Pain is a late
symptom of bladder cancer.) The other options aren't associated with
bladder cancer. Occasional polyuria may occur with diabetes or increased
alcohol or caffeine intake. Nocturia commonly accompanies benign
prostatic hypertrophy. Dysuria may indicate a urinary tract infection
(UTI).
1265 Which finding should a nurse Platelet count of 115,000/mm3
. identify as requiring further
investigation?
1266 Which findings indicate that a Confusion and seizures - Classic signs of water intoxication include
. client has developed water confusion and seizures, both of which are caused by cerebral edema.
intoxication secondary to Weight gain will also occur. Sunken eyeballs, thirst, and increased BUN
treatment for diabetes levels indicate fluid volume deficit. Spasticity, flaccidity, and tetany are
insipidus? unrelated to water intoxication.
1267 Which findings indicate that a Confusion and Seizures - Classic signs of water intoxication include
. client has developed water confusion and seizures, both of which are caused by cerebral edema.
intoxication secondary to Weight gain will also occur. Sunken eyeballs, thirst, and increased BUN
treatment for diabetes levels indicate fluid volume deficit. Spasticity, flaccidity, and tetany are
insipidus? unrelated to water intoxication.
1268 Which health care profession Physician
. has the ultimate responsibility
to provide appropriate
information regarding a
nonemergent surgery?
1269 Which intervention by a nurse might help Placing an alternating-current mattress on the client's bed
. prevent pressure ulcers?
1270 Which intervention has the highest priority Keeping the skin clean and dry without using harsh soaps
. when providing skin care to a bedridden
client?
1271 Which intervention is most appropriate for a Instruct the client to breathe into a paper bag. - The ABG
. client with an arterial blood gas (ABG) of results reveal respiratory alkalosis. The best intervention to
pH 7.5, a partial pressure of arterial carbon raise the PaCO2 level would be to have the client breathe
dioxide (PaCO2) of 26 mm Hg, oxygen (O2) into a paper bag. Administering a decongestant, offering
saturation of 96%, bicarbonate (HCO3-) of fluids frequently, and administering supplemental oxygen
24 mEq/L, and a PaO2 of 94 mm Hg? wouldn't raise the lowered PaCO2 level.
1272 Which intervention should the nurse Allow the client to void
. implement in the client scheduled for Explanation:
aminocentesis? Before amniocentesis, the client should void to empty the
bladder, reducing the risk of bladder perforation. The client
doesn't need to drink fluids before amniocentesis nor does
she need to fast. The client should be placed in a supine
position for the procedure.
1273 Which lobe of the brain is responsible for Frontal
. concentration and abstract thought?
1274 Which lobe of the brain is responsible for Parietal
. spatial relationships?
1275 Which medication is considered safe during Insulin
. pregnancy? Explanation:
Insulin is a required hormone for any client with diabetes
mellitus, including the pregnant client. Aspirin, magnesium
hydroxide, and oral antidiabetic agents aren't recommended
for use during pregnancy because these agents may cause
fetal harm.
1276 Which method is most reliable for Check the hospital identification bracelet.
. confirming a preschooler's identity before Explanation: The only safe method for identifying the child
administering a medication? is to check the identification band for the client's name and
medical record number and then compare that information
with the medication record. Children sometimes exchange
beds during play, so checking the name on the bed isn't
reliable. Infants are unable to give their names, toddlers or
preschoolers may admit to any name, and school-age
children may deny their identities in an attempt to avoid the
medication. Parents aren't always at the bedside, so they
shouldn't be relied on for identification
1277 Which New York Heart Association IV
. classification of heart failure has a poor
prognosis and includes symptoms of cardiac
insufficiency at rest?
1278 Which nursing assessment finding indicates Fever
. that the client who has undergone renal
transplant has not met expected outcomes?
1279 Which nursing diagnosis is most important Risk for perioperative positioning injury related to
. for the client who is undergoing a surgical positioning in the OR
procedure expected to last several hours?
1280 Which nursing diagnosis is most relevant in Risk for infection
. the first 12 hours of life for a neonate born Explanation: All of these nursing diagnoses are important
with a myelomeningocele? for a child with a myelomeningocele. However, during the
first 12 hours of life, the most life-threatening event would
be an infection. The other diagnoses will be addressed as
the child develops
1281 Which nursing diagnosis should the Anxiety related to ineffective coping with surgical concerns
. nurse plan to address first in the
client upon arrival in the
intraoperative setting?
1282 Which nursing diagnosis takes Risk for injury related to vertigo
. highest priority for a client Correct
admitted for evaluation for Explanation:
Mnire's disease? Vertigo, the chief finding in Mnire's disease, is a severe, rotational
whirling sensation that typically causes the client to fall when
a) Risk for deficient fluid volume attempting to stand or walk. Because client safety is paramount, the
related to vomiting nursing diagnosis of Risk for injury related to vertigo takes priority.
b) Imbalanced nutrition: Less than Vertigo doesn't cause pain. Although nausea and vomiting may lead
body requirements related to to inadequate nutrition and fluid loss, these problems are secondary
nausea and vomiting to client safety.
c) Acute pain related to vertigo
d) Risk for injury related to vertigo
1283 Which nursing intervention takes Instructing the client to report any itching, swelling, or dyspnea
. highest priority when caring for a Explanation: Because administration of blood or blood products may
client who's receiving a blood cause serious adverse effects such as allergic reactions, the nurse
transfusion? must monitor the client for these effects. Signs and symptoms of life-
threatening allergic reactions include itching, swelling, and dyspnea.
Although the nurse should inform the client of the duration of the
transfusion and should document its administration, these actions are
less critical to the client's immediate health. The nurse should
monitor vital signs 5 minutes after the transfusion is started, again in
15 minutes, and then at least hourly depending on the client's
condition.
1284 Which nursing objective is most Early identification
. important when working with Explanation: The most important nursing objective is early
neonates who are suspected of identification of the disorder. Nurses caring for neonates must be
having congenital hypothyroidism? certain that screening is performed, especially in neonates who are
preterm, discharged early, or born at home. Promoting bonding,
allowing rooming in, and encouraging fluid intake are all important
but are less important than early identification.
1285 Which of the following actions by Discarding an object that comes in contact with the 1-inch border
. the nurse is appropriate?
1286 Which of the following adverse Respiratory or urinary system infections
. effects should the nurse closely
monitor in a patient who takes
immunosuppressive drugs?
1287 Which of the following appears to Diet
. be a significant factor in the
development of gastric cancer?
1288 Which of the following are Prozac
. antidepressants used in the treatment Tofranil
of AIDS? Select all that apply Norpramin
1289 Which of the following areas is the Back
. most suitable area of the body for skin
testing
1290 Which of the following are functions of Lubrication
. saliva? Select all that apply. Digestion
Protection against harmful bacteria
1291 Which of the following are modes of Seminal fluid
. transmission for HIV? Select all that Vaginal secretions
apply. Blood
Amniotic fluid
1292 Which of the following are sympathetic Dilated pupils
. effects of the nervous system?
1293 Which of the following are the Breathing - Loss of fluid from sweat or diaphoresis is referred to
. insensible mechanisms of fluid loss? as insensible loss because it is unnoticeable and immeasurable.
Losses from urination and bowel elimination are measurable.
1294 Which of the following arterial blood Serum bicarbonate of 28 mEq/L - Evaluation of arterial blood
. gas results would be consistent with gases reveals a pH greater than 7.45 and a serum bicarbonate
metabolic alkalosis? concentration greater than 26 mEq/L.
1295 Which of the following assessment Ataxia
. should be completed if suspecting
immune dysfunction in the
neurosensory system?
1296 Which of the following causes should Glomerulonephritis
. the nurse suspect in a client is
diagnosed with intrarenal failure?
1297 Which of the following cells are capable B lymphocytes
. of differentiating into plasma cells?
1298 Which of the following cell types are B lymphocytes
. involved in humoral immunity?
1299 Which of the following cerebral lobes Temporal
. contains the auditory receptive areas?
1300 Which of the following cerebral lobes is Frontal
. the largest and controls abstract
thought?
1301 Which of the following cerebrospinal 22 mm Hg
. fluid (CSF) pressure values would be
indicative of increased intracranial
pressure (ICP)?
1302 Which of the following clients would be A client with type I diabetes
. most susceptible to experiencing Explanation: The most common need for urine testing is the test
ketoacidosis? for ketones if a client's blood glucose level is consistently high.
Because only clients with type 1 diabetes are susceptible to
diabetic ketoacidosis, these clients learn to test their urine for
ketones if their blood glucose readings exceed 240 mg/dL.
1303 Which of the following clinical Tachycardia (heart rate above 150 beats per minute)
. manifestations is often the earliest sign
of malignant hyperthermia?
1304 Which of the following clinical Difficulty in breathing
. manifestations should a nurse monitor
for during a pulmonary angiography,
which indicates an allergic reaction to
the contrast medium?
1305 Which of the following Hemorrhage and shock
. complications is most common Explanation: Hemorrhage and shock are the most common complications
after an abdominal aortic after abdominal aortic aneurysm resection. Renal failure can occur as a
aneurysm resection? result of shock or from injury to the renal arteries during surgery. Graft
occlusion and enteric fistula formation are rare complications of
abdominal aortic aneurysm repair.
1306 Which of the following Hypomagnesemia - If there is a unilateral spasm of facial muscles when
. conditions does the nurse need the nurse taps over the facial muscle, it is known as Chvostek's sign,
to confirm when he or she taps which is a sign of hypocalcemia and hypomagnesemia. The additional
the facial nerve of a client who symptom of dysphagia reinforces the possibility of hypomagnesemia
has dysphagia? rather than hypocalcemia. A positive Chvostek's sign does not apply to
hypercalcemia, hypervolemia, or hypermagnesemia.
1307 Which of the following cranial Oculomotor
. nerves is responsible for
muscles that move the eye and
lid?
1308 Which of the following Otosclerosis
. describes a condition Explanation:
characterized by abnormal Otosclerosis is more common in females than males and is frequently
spongy bone formation around hereditary. A middle ear effusion is denoted by fluid in the middle ear
the stapes? without evidence of infection. Chronic otitis media is defined as repeated
episodes of acute otitis media causing irreversible tissue damage and
a) Otitis externa persistent tympanic membrane perforation. Otitis externa refers to
b) Otosclerosis inflammation of the external auditory canal.
c) Middle ear effusion
d) Chronic otitis media
1309 Which of the following Autograft
. describes a valve used in
replacement surgery that is
made from the patient's own
heart valve?
1310 Which of the following PET
. diagnostics is used to identify
malignant cells associated with
esophageal cancer?
1311. Which of the following A computed tomography scan
diagnostic tests are done to Explanation: A computed tomography or magnetic resonance imaging
determine suspected pituitary scan is done to detect a suspected pituitary tumor
tumor?
1312 Which of the following Potassium - Potassium is a major cation that affects cardiac muscle
. electrolytes is a major cation in functioning. Chloride is an anion. Bicarbonate is an anion. Phosphate is
body fluid? an anion.
1313 Which of the following Sodium - Sodium is the primary determinant of ECF osmolality. Sodium
. electrolytes is the primary plays a major role in controlling water distribution throughout the body
determinant of extracellular because it does not easily cross the intracellular wall membrane and
fluid (ECF) osmolality? because of its abundance and high concentration in the body. Potassium,
calcium, and magnesium are not primary determinants of ECF
osmolality.
1314 Which of the following enzymes Lipase
. aids in the digestion of fats?
1315 Which of the following Glasgow 7
. Coma Scale scores indicates
coma?
1316 Which of the following has not Past substance abuse
. been implicated as a factor for
noncompliance with
antiretroviral treatment?
1317 Which of the following immunoglobulins IgG
. assumes a major role in blood-borne and
tissue infections?
1318 Which of the following in a client's health Splenoectomy
. history would the nurse recognize as
potentially compromising the client's blood
cell volume?
1319 Which of the following indicates that a Herpes simplex ulcer persisting for 2 months
. client with HIV has developed AIDS?
1320 Which of the following instructions, Wear soft plastic ear plugs.
. regarding swimming, should the nurse give Correct
to the client who is recovering from otitis Explanation:
externa? Choose the correct option. The nurse should advise the client to wear soft plastic ear
plugs to prevent trapping water in the ear while swimming.
a) Avoid cold water.
b) Insert a loose cotton pledget in the
external ear.
c) Wear soft plastic ear plugs.
d) Wear a scarf.
1321 Which of the following interventions would Cover the client with a light blanket.
. be most appropriate for a client who has
undergone surgery for a liver disorder and
has started shivering?
1322 Which of the following is a age-related Decreased antibody production
. change associated with the immune system?
1323 Which of the following is a center for Lymph node
. immune cell proliferation?
1324 Which of the following is a change that Anemia
. occurs in chronic glomerulonephritis?
1325 Which of the following is a characteristic of JVD (Jugular vein distention)
. right-sided heart failure?
1326 Which of the following is a characteristic of Increased BUN
. the intrarenal category of acute renal
failure?
1327 Which of the following is a clinical Clinical manifestations of FVE include distended neck
. manifestation of fluid volume excess (FVE)? veins, crackles in the lung fields, shortness of breath,
Select all that apply. increased blood pressure, and tachycardia.
1328 Which of the following is a correct route of Oral - Potassium may be administered through the oral
. administration for potassium? route. Potassium is never administered by IV push or
intramuscularly to avoid replacing potassium too quickly.
Potassium is not administered subcutaneously.
1329 Which of the following is a factor affecting Free Water Loss - Free water loss is a factor increasing
. an increase in serum osmolality? serum osmolality. Diuretic use, overhydration, and
hyponatremia are factors decreasing serum osmolality.
1330 Which of the following is a factor affecting Syndrome of inappropriate antidiuretic hormone release
. an increase in urine osmolality? (SIADH) - Factors increasing urine osmolality include
SIADH, fluid volume deficit, acidosis, and congestive heart
failure. Myocardial infarction typically is not a factor that
increases urine osmolality.
1331 Which of the following is a factor that GI Bleeding - Factors that increase BUN include GI
. increases blood urea nitrogen (BUN)? bleeding, dehydration, increased protein intake, and fever.
1332 Which of the following is a function of Reduces bone resorption, Increases urinary excretion of
. calcitonin? Select all that apply. calcium, Increases deposition of calcium in bones -
Calcitonin reduces bones resorption, increasing deposition
of calcium and phosphorous in the bones, and increases
urinary excretion of calcium and phosphate.
1333 Which of the following is a gerontological Increase in fibrous material
. consideration associated with the
pancreas?
1334 Which of the following is a key diagnostic BNP
. indicator of heart failure (HF)?
1335 Which of the following is a late sign of Cyanosis
. hypoxia?
1336 Which of the following is also known as a Durable power of attorney for healthcare
. proxy directive?
1337 Which of the following is also termed Unstable angina
. preinfarction angina?
1338 Which of the following is a modifiable History of smoking
. risk factor for transient ischemic attacks
and ischemic strokes?
1339 Which of the following is an accurate Chronic irritation of the esophagus is a known risk factor
. statement regarding cancer of the
esophagus?
1340 Which of the following is an age-related Increased thickness of the alveolar membranes
. change associated with the lung?
1341 Which of the following is an age-related Decreased size of the airway
. change associated with the respiratory
system?
1342 Which of the following is an age-related Weakened gag reflex
. change of the gastrointestinal system?
1343 Which of the following is a nasoenteric Dobhoff
. feeding tube?
1344 Which of the following is an early Headache
. manifestation of HIV encephalopathy?
1345 Which of the following is an early Fatigue
. warning symptom of acute coronary Explanation:
syndrome (ACS) and heart failure (HF)? Fatigue is an early warning symptom of ACS, heart failure, and
valvular disease. Other signs and symptoms of cardiovascular
disease are hypotension, change in level of consciousness, and
weight gain.
1346 Which of the following is an enzyme Pepsin
. secreted by the gastric mucosa?
1347 Which of the following is an enzyme that Ptyalin
. begins the digestion of starches?
1348 Which of the following is an Nystagmus
. involuntary rhythmic movement Correct
of the eyes that is also associated Explanation:
with vestibular dysfunction? Nystagmus is an involuntary rhythmic movement of the eyes;
pathologically it is an ocular disorder but is also associated with
a) Tinnitus vestibular dysfunction. Nystagmus can be horizontal, vertical, or
b) Vertigo rotary, and can be caused by a disorder in the central or peripheral
c) Presbycusis nervous system. Vertigo is defined as the misperception or illusion of
d) Nystagmus motion of the person or their surroundings. Tinnitus is ringing in the
ears. Presbycusis is a progressive hearing loss.
1349 Which of the following is a Pulse Oximetry
. noninvasive method of
continuously monitoring the
oxygen saturation of hemoglobin
(SaO2)?
1350 Which of the following is a Gender
. nonmodifiable risk factor for
coronary artery disease (CAD)?
1351 Which of the following is a Increased peristalsis
. parasympathetic response in the
GI tract?
1352 Which of the following is a Opsonization
. process in which the antigen-
antibody molecule is coated with
a sticky substance that facilitates
phagocytosis?
1353 Which of the following is Decreased renal function
. associated with impaired
immunity in the aging patient?
1354 Which of the following is a term Mourning
. that refers to individual, family,
group, and cultural expressions of
grief and associated behaviors?
1355 Which of the following is a term Phagocytosis
. used to describe the process of
ingestion and digestion of bacteria
by cells?
1356 Which of the following is a true Air is drawn through the trachea and bronchi into the alveoli during
. statement regarding air pressure inspiration.
variances?
1357 Which of the following is a true The clusters of ulcers take on a cobblestone appearance.
. statement regarding regional
enteritis (Crohn's disease)?
1358 Which of the following is a true Initiates the parasympathetic response
. statement regarding the role of Explanation:
baroreceptors? During elevations of blood pressure, the baroreceptors increase their
rate of discharge. This initiates parasympathetic activity and inhibits
sympathetic response, lowering the heart rate and blood pressure.
1359 Which of the following is caused by improper Pneumothorax
. catheter placement and inadvertent puncture
of the pleura?
1360 Which of the following is clinical Clay colored stools
. manifestation of cholelithiasis?
1361 Which of the following is considered a bulk- Metamucil
. forming laxative?
1362 Which of the following is considered a Dulcolax
. stimulant laxative?
1363 Which of the following is inconsistent as a Hypotension
. condition related to metabolic syndrome?
1364 Which of the following is one of the first Increasing difficulty is swallowing
. clinical manifestations of esophageal cancer?
1365 Which of the following is one of the primary Diarrhea
. symptoms of Irritable Bowel Syndrome
(IBS)?
1366 Which of the following is the analgesic of Morphine sulfate
. choice for acute MI?
1367 Which of the following is the first barrier Female condom
. method that can be controlled by the
woman?
1368 Which of the following is the greatest risk if Rapid onset of drug resistance
. the client does not take non-nucleoside
reverse transcriptase inhibitors (NNRTI) as
prescribed? Choose the correct option.
1369 Which of the following is the hallmark of the Proteinuria
. diagnosis of nephrotic syndrome?
1370 Which of the following is the major cause of Pancreatic necrosis
. morbidity and mortality in patients with
acute pancreatitis?
1371 Which of the following is the most common Medications
. cause of anaphylaxis?
1372 Which of the following is the most common Alcoholism - Alcoholism is currently the most common
. cause of symptomatic hypomagnesemia? cause of symptomatic hypomagnesemia. IV drug use,
sedentary lifestyle, and burns are not the most common
causes of hypomagnesemia.
1373 Which of the following is the most common Alcoholism - Alcoholism is currently the most common
. cause of symptomatic hypomagnesemia in cause of symptomatic hypomagnesemia in the United
the United States? States. Any disruption in small bowel function, as in
intestinal resection or inflammatory bowel disease, can
lead to hypomagnesemia.
1374 Which of the following is the most common Change in bowel habits
. presenting symptom of colon cancer?
1375 Which of the following is the most common Rectal bleeding
. symptom of a polyp?
1376 Which of the following is the most common Nausea
. symptom of gastrointestinal (GI) problems in
general?
1377 Which of the following is the most effective Vaccine
. strategy to prevent hepatitis B infection?
1378 Which of the following is the most important Inadequate tissue perfusion
. postoperative assessment parameter for
patients undergoing cardiac surgery?
1379 Which of the following is the most severe Anaphylaxis
. form of hypersensitivity reaction?
1380 Which of the following is the most Removal of the tumor
. successful treatment for gastric
cancer?
1381 Which of the following is the Absorption
. primary function of the small
intestine?
1382 Which of the following is the Difficulty swallowing
. primary symptom of achalasia?
1383 Which of the following is the Surgery
. treatment of choice for acoustic Correct
neuromas? Explanation:
Surgical removal of acoustic tumors is the treatment of choice
a) Surgery because these tumors do not respond well to radiation or
b) Chemotherapy chemotherapy. There would be no need for palliation.
c) Palliation
d) Radiation
1384 Which of the following is true Less control of rectal sphincter
. statement regarding older patients,
considering the age-related effects
on their GI system?
1385 Which of the following is usually the CD4 Counts
. most important consideration in
decisions to initiate antiretroviral
therapy?
1386 Which of the following laboratory Increased direct bilirubin
. test results would the nurse associate
with obstructive jaundice?
1387 Which of the following may occur Increased Intracranial Pressure (ICP) - If respiratory acidosis is
. with respiratory acidosis? severe, intracranial pressure may increase, resulting in papilledema
and dilated conjunctival blood vessels. Increased blood pressure,
increased pulse, and decreased mental alertness occur with
respiratory acidosis.
1388 Which of the following medication NSAIDs in large doses
. classifications are known to inhibit
prostaglandin synthesis or release?
1389 Which of the following medications Vasopressin
. are used to decrease portal pressure,
halting bleeding of esophageal
varices?
1390 Which of the following medications Over-the-counter (OTC) allergy medications
. are usually withheld for 48 to 96 Corticosteroids
hours before skin testing? Select all Antihistamines
that apply.
1391 Which of the following medications Protamine sulfate
. is an antidote to heparin?
1392 Which of the following medications Lasix
. is categorized as a loop diuretic?
1393 Which of the following medications Omeprazole
. is classified as a proton pump
inhibitor (PPI)?
1394 Which of the following medications Plavix
. is given to patients diagnosed with
angina and is allergic to aspirin?
1395 Which of the following medications plavix
. is given to patients diagnosed with
angina and is allergic to aspirin?
1396 Which of the following medications requires the nurse to Enteric coated tablets
. contact the pharmacist in consultation when the patient
receives all oral medications by feeding tube?
1397 Which of the following medications reverses digitalis Digoxin immune FAB (Digibind)
. toxicity?
1398 Which of the following medications, used in the treatment Metoclopramide (Reglan)
. of GERD, accelerate gastric emptying?
1399 Which of the following medications, used in the treatment Reglan
. of GERD, accelerate gastric emptying?
1400 Which of the following medications would the nurse expect Aldactone
. the physician to order for a client with cirrhosis who
develops portal hypertension?
1401 Which of the following medications would the nurse expect attacurium (Tracrium)
. to be used to facilitate intubation of the client?
1402 Which of the following mouth conditions are associated Karposi's Sarcoma
. with HIV infection?
1403 Which of the following neurotransmitters are deficient in Acetylcholine
. myasthenia gravis?
1404 Which of the following nursing actions is most important in Strain the urine carefully for stone
. caring for the client following lithotripsy? fragments.
1405 Which of the following nursing interventions is most Report any incident of bloody urine, stools,
. appropriate when caring for a client with a nursing or both.
diagnosis of risk for injury related to side effects of Explanation:
medication (enoxaparin [Lovenox])? The client who takes an anticoagulant, such
as a low-molecular-weight heparin, is
routinely screened for bloody urine, stools,
or both.
1406 Which of the following nursing interventions should a Maintain on bedrest
. nurse perform to reduce cardiac workload in a patient
diagnosed with myocarditis?
1407 Which of the following nursing interventions should a Check for dependent edema regularly
. nurse perform when a patient with cardiomyopathy
receives a diuretic?
1408 Which of the following nutritional deficiencies may delay Lack of vitamin C
. wound healing?
1409 Which of the following options should the nurse encourage Liquids
. to replace fluid and electrolyte losses in a patient with
AIDS?
1410 Which of the following precautions should the nurse take Have the client sit in a wheelchair when
. when a client is at risk of injury secondary to the vertigo moving
and probable imbalance? Correct
Explanation:
a) Recommend that the client keep his/her eyes close The nurse should have the client sit in a
b) Restrict the client from looking at one place wheelchair when moving him or her.
c) Allow the client to move the head slowly
d) Have the client sit in a wheelchair when moving
1411. Which of the following protective responses begin with the Humoral
B lymphocytes?
1412 Which of the following reports from the client during the Experiences prolonged bleeding from an
. health history would lead the nurse to suspect that the obvious injury.
client has a disorder of the hematopoietic or lymphatic Has unexplained blood loss, as in rectal
system? Select all that apply. bleeding, nosebleeds, bleeding gums, or
vomiting blood.
Feels fatigued with normal activities.
1413 Which of the following represents a responsibility of the Preparing sutures
. scrub nurse?
1414 Which of the following results in decreased gas exchange in The alveolar walls contain fewer
. older adults? capillaries.
1415 Which of the following sets of clinical data would allow the Vital signs within normal limits; absence of
. nurse to conclude that the nursing actions taken to prevent chills and cough
postoperative pneumonia have been effective?
1416 Which of the following should be incorporated into the Hourly leg exercises
. patient teaching plan to prevent deep vein thrombosis?
1417 Which of the following should not be Let masks hang around the neck
. allowed with regards to the wearing of
masks in the operating room?
1418 Which of the following signs would the Dark, tarry stools
. nurse recognize as signs of Bleeding of the gums
thrombocytopenia? Select all that apply. Oozing from injection sites
1419 Which of the following solutions is 0.45% NaCl - Half-strength saline is hypotonic. Lactated
. hypotonic? Ringer's solution is isotonic. Normal saline (0.9% NaCl) is
isotonic. A solution that is 5% NaCl is hypertonic.
1420 Which of the following statements The stem cell is known as a precursor cell that continually
. accurately reflects current stem cell replenishes the body's entire supply of both red and white cells.
research?
1421 Which of the following statements is not Anticoagulation is necessary
. accurate regarding an autograft?
1422 Which of the following surgical Gastric banding
. procedures for obesity utilizes a
prosthetic device to restrict oral intake?
1423 Which of the following terms describes Asterixis
. the involuntary flapping movements of
the hands associated with metabolic liver
dysfunction?
1424 Which of the following terms is an Duchenne MD
. example of an X-linked recessive
condition?
1425 Which of the following terms is used to Sialolithiasis
. describe stone formation in a salivary
gland, usually the submandibular gland?
1426 Which of the following terms is used to Orthopnea
. describe the inability to breathe easily
except in an upright position?
1427 Which of the following terms is used to Grief
. describe the personal feelings that
accompany an anticipated or actual
loss?
1428 Which of the following terms refers to a Stem Cell
. primitive cell, capable of self-replication
and differentiation?
1429 Which of the following terms refers to Tympanoplasty
. surgical repair of the tympanic Correct
membrane? Explanation:
Tympanoplasty may be necessary to repair a scarred eardrum. A
a) Myringotomy tympanotomy is an incision into the tympanic membrane. A
b) Tympanoplasty myringotomy is an incision into the tympanic membrane. An
c) Tympanotomy ossiculoplasty is a surgical reconstruction of the middle ear
d) Ossiculoplasty bones to restore hearing.
1430 Which of the following terms refers to Tympanoplasty
. surgical repair of the tympanic Explanation:
membrane? Tympanoplasty may be necessary to repair a scarred eardrum. A
tympanotomy is an incision into the tympanic membrane. A
a) Tympanoplasty myringotomy is an incision into the tympanic membrane. An
b) Ossiculoplasty ossiculoplasty is a surgical reconstruction of the middle ear
c) Myringotomy bones to restore hearing.
d) Tympanotomy
1431 Which of the following terms refers to Ataxia
. the inability to coordinate muscle
movements, resulting difficulty walking?
1432 Which of the following terms refer to a Electromyogram
. method of recording, in graphic form, the
electrical activity of the muscle?
1433 Which of the following the are early Pain
. manifestations of liver cancer? Select all Continuous aching in the back
that apply.
1434 Which of the following therapies are for Heart transplant
. patient who have advanced heart failure
(HF) after all other therapies have failed?
1435 Which of the following ulcers is associated Curling's Ulcer
. with extensive burn injury?
1436 Which of the following venous access Non-tunneled catheter
. devices can be used for less than 6 weeks in
patients requiring parenteral nutrition?
1437 Which of the following venous access Nontunneled catheter
. devices can be used for less than 6 weeks in
patients requiring parenteral nutrition?
1438 Which of the following will the nurse Sudden, sustained abdominal pain
. observe as symptoms of perforation in a Abdominal distention
client with intestinal obstruction? Choose
all that apply.
1439 Which of the following will the nurse Sudden, sustained abdominal pain
. observe as symptoms of perforation in a
patient with intestinal obstruction?
1440 Which of the following would a nurse be Genetics
. least likely to identify as a cause of
secondary immunodeficiency?
1441 Which of the following would be a factor Acidosis
. that may decrease myocardial Explanation:
contractility? Contractility is depressed by hypoxemia, acidosis, and
certain medications, such as beta-adrenergic blocking
medications. Contractility is enhanced by sympathetic
neuronal activity, and certain medications, such as Lanoxin.
1442 Which of the following would be an Determination of atrial thrombi
. indication for a transesophageal Explanation:
echocardiography (TEE)? The TEE is an important diagnostic tool for determining if atrial or
ventricular thrombi are present in patients with heart failure, valvular
heart disease, and arrhythmias. The electrocardiogram (ECG) is a
graphic recording of the electrical activity of the heart. Stress testing
is used to evaluate the response of the cardiovascular system to
increased demands for oxygen and nutrients. Thallium is used with
exercise or pharmacologic stress testing to assess changes in
myocardial perfusion at rest and after exercise.
1443 Which of the following would be an Insertion of nasogastric tube
. intervention for a patient with a
chemical burn to the esophagus?
1444 Which of the following would be an Insert NG tube
. intervention for a patient with a
chemical burn to the esophagus?
1445 Which of the following would be Acute gallbladder infection
. considered an urgent surgical
procedure?
1446 Which of the following would be The inheritance of SCID can be autosomal dominant.
. inaccurate information pertaining
to SCID?
1447 Which of the following would be Push fluids
. inconsistent as a lifestyle change
directive for the patient diagnosed
with heart failure?
1448 Which of the following would be Inability to speak
. inconsistent as criterion of
extubation in the patient who has
undergone a coronary artery
bypass graft (CABG)?
1449 Which of the following would be Ensure that the client rests.
. most appropriate for a client who is
experiencing biliary colic?
1450 Which of the following would be Provide physical and emotional support
. the highest priority, immediate
nursing intervention for a client
just diagnosed with acute gastritis?
1451 Which of the following would be patient ambulatory aids
. the least important issue
concerning safety for the
perioperative team prior to
proceeding to the operating room?
1452 Which of the following would be the Patient ambulatory aids
. least important issue concerning
safety for the perioperative team
prior to proceeding to the operating
room?
1453 Which of the following would Signs of dizziness and confusion
. indicate to the nurse that a client
who has received three cleansing
enemas in preparation for a barium
enema is experiencing dehydration?
1454 Which of the following would the Slowed healing
. nurse expect of an elderly client's
skin?
1455 Which of the following would the Levothyroxine sodium:
. nurse expect the physician to order Explanation: Hypothyroidism is treated with thyroid replacement
for a client with hypothyroidism? therapy, in the form of dessicated thyroid extract or a synthetic
product, such as levothyroxine sodium (Synthroid) or liothyronine
sodium (Cytomel). Methimazole and propylthiouracil are
antithyroid agents used to treat hyperthyroidism. Propranolol is a
beta blocker that can be used to treat hyperthyroidism.
1456 Which of the following would the Asterixis
. nurse expect to assess in a client with
hepatic encephalopathy?
1457 Which of the following would the Stratum germinativum
. nurse identify as the deepest layer of
the epidermis?
1458 Which outcome indicates effective The client reports engaging in a regular exercise regimen.
. client teaching to prevent
constipation?
1459 Which outcome is most appropriate The client will obtain appropriate mental health services
. for a teenager who's irritable, hasn't Explanation:
slept well in 6 months, and has Mental health services can protect the client and offer the best
dropped out of social activities? means of regaining mental health. The client could reestablish a
healthy sleeping pattern without addressing underlying issues. The
parents' worrying is unrelated to the child's immediate need for
help. The child's behavior suggests the need for professional
service, not disciplinary measures.
1460 Which principle of the All behavior has meaning.
. psychoanalytic model is particularly Explanation:
useful to psychiatric nurses? The principle that all behavior has meaning is of particular
importance to the psychiatric nurse. It serves as the basis for the
nurse's assessment and analysis of the client's behavior, which
reflects the client's needs. Psychoanalytic theory also proposes that
the first 6 years of a person's life determine personality; these early
influences are difficult, if not impossible, to counteract. However,
this assumption is less useful to the nurse in planning interventions
that meet the client's current needs. Reinforcement as a means of
perpetuating behavior is associated with behavioral theory, not the
psychoanalytic model. Incongruence between verbal and nonverbal
communications is a part of communications theory.
1461 Which pulse should the nurse Carotid
. palpate during rapid assessment of
an unconscious adult?
1462 Which signs and symptoms Fever, chest discomfort, and elevated erythrocyte sedimentation rate
. accompany a diagnosis of (ESR)
pericarditis?
1463 Which stage of anesthesia is III
. termed surgical anesthesia?
1464 Which stage of HIV infection is CDC category A: HIV asymptomatic
. indicated when the results are
more than 500 CD4+
lymphocytes/mm?
1465 Which stage of surgical anesthesia II
. is also known as excitement?
1466 Which stage of the immune Response stage
. response occurs when the
differentiated lymphocytes
function in either a humoral or a
cellular capacity?
1467 Which statement best describes an Goals that the client should reach as a result of planned nursing
. expected outcome? interventions
Explanation: Expected outcomes are realistic, measurable goals that
include target dates for when the goals will be achieved. They're
devised by the nursing staff with input from the client. The goals are
attained by following planned nursing interventions.
1468 Which statement is correct about The psychological conflict is repressed
. conversion disorders? Explanation:
In conversion disorders, the client isn't conscious of intentionally
producing symptoms that can't be self-controlled. The symptoms are
characterized by one or more neurologic symptoms. Understanding
the principles and conflicts behind the symptoms can prove helpful
during a client's therapy.
1469 Which statement reflects "Client's skin is moist and cool."
. appropriate documentation in the Explanation: Documentation should include data that the nurse
medical record of a hospitalized obtains using only observations that are heard, seen, smelled, or felt.
client? The nurse should record findings or observations precisely and
accurately. Documentation of a leg ulcer should include its exact size
and location. Documenting observed client behaviors or conversations
is appropriate, but drawing conclusions about a client's feelings is not.
Stating that the client had a good day doesn't provide precise enough
information to be useful.
1470 Which symptom, when observed Increasing oliguria
. in laboring clients with gestational Explanation:
hypertension, would most likely Renal plasma flow and glomerular filtration are decreased in
indicate a worsening condition? gestational hypertension, so increasing oliguria indicates a worsening
condition. Blood pressure increases as a result of increased peripheral
resistance. Increasing (not decreasing) edema would suggest a
worsening condition. Trace levels to +1 proteinuria are acceptable;
higher levels would indicate a worsening condition.
1471 Which task can a licensed Turning a client every 2 hours
. practical nurse (LPN) safely
delegate to a nursing assistant?
1472 Which task can be safely delegated to c) Changing the dressing of a client who underwent surgery two
. a licensed practical nurse (LPN)? days ago.

a) Teaching a newly diagnosed


diabetic about insulin
administration.
b) Admitting a client who underwent
a thoracotomy to the nursing unit
from the postanesthesia care unit.
c) Changing the dressing of a client
who underwent surgery two days
ago.
d) Administering an I.V. bolus of
morphine sulfate to a client
experiencing incisional pain
1473 Which term means a lack of one or Hypogammaglobulinemia
. more of the five immunoglobulins?
1474 Which terms refers to the Presbycusis
. progressive hearing loss associated Correct
with aging? Explanation:
Both middle and inner ear age-related changes result in hearing
a) Otalgia loss. Exostoses refer to small, hard, bony protrusions in the lower
b) Sensorineural hearing loss posterior bony portion of the ear canal. Otalgia refers to a sensation
c) Presbycusis of fullness or pain in the ear. Sensorineural hearing loss is loss of
d) Exostoses hearing related to damage of the end organ for hearing and/or
cranial nerve VIII.
1475 Which type of cells destroys antigens Null Cells
. already coated with antibody?
1476 Which type of deficiency results in Folic Acid
. macrocytic anemia?
1477 Which type of graft is utilized when a Xenograft
. heart valve replacement is made of
tissue from an animal heart valve?
1478 Which type of immunity becomes Naturally acquired active immunity
. active as a result of the infection of a
specific microorganism?
1479 Which type of incontinence refers to Stress
. involuntary loss of urine through an
intact urethra as a result of a sudden
increase in intra-abdominal
pressure?
1480 Which type of jaundice seen in adults Hemolytic
. is the result of increased destruction
of red blood cells?
1481 Which type of leukocyte contains Basophils
. histamine and is an important part
of hypersensitivity reactions?
1482 Which type of phagocytic disorder Hyperimmunoglobulinemia E
. (formerly known as Job syndrome)
occurs when white blood cells cannot
initiate an inflammatory response to
infectious organisms?
1483 Which type of positioning should be Upright at the edge of the bed
. utilized for a patient undergoing a
paracentesis?
1484 Which type of white blood cell Neutrophil
. (WBC) is the most numerous?
1485 Which ventilation-perfusion ratio is exhibited by acute respiratory distress Silent Unit
. syndrome (ARDS)?
1486 Which ventilation-perfusion ratio is exhibited by a pulmonary emboli? Dead Space
.

1487 Which zone of the surgical area only allows for attire in the form of scrub Semirestricted zone
. clothes and caps?
1488 While assessing a client, a nurse notes a stage I pressure ulcer on the Document the size, extent,
. client's left hip. How should the nurse report this finding? and location of the wound
in the client's medical
record
1489 While assessing a patient with pericarditis, the nurse cannot auscultate a Ask the patient to lean
. friction rub. Which action should the nurse implement? forward and listen again.
1490 While assessing for tactile fremitus, the nurse palpates almost no Emphysema
. vibration. Which of the following conditions in this client's history will
account for this finding?
1491 While auscultating the lungs of a client with asthma, the nurse hears a wheezes
. continuous, high-pitched whistling sound on expiration. The nurse will
document this sound as which of the following?
1492 While conducting a physical examination of a client, which of the Ecchymoses
. following skin findings would alert the nurse to the possibility of liver Jaundice
problems? Select all that apply. Petechiae
1493 While conducting the physical examination during assessment of the Deviation from the midline
. respiratory system, which of the following does a nurse assess by
inspecting and palpating the trachea?
1494 While hospitalized, a client accidentally injures his finger and begins to Platelets
. bleed. What substance does the nurse recognize as naturally rushing to the
site of injury before any other action takes place?
1495 While in a skilled nursing facility, a client contracted scabies, which is All family members will
. diagnosed the day after discharge. The client is living at her daughter's need to be treated.
home, where six other family members are living. During her visit to the
clinic, she asks a staff nurse, "What should my family do?" The most
accurate response from the nurse is:
1496 While reviewing the health history of a 72- Routine use of quinine for management of leg cramps
. year-old client experiencing hearing loss the Correct
nurse notes the patient has had no trauma or Explanation:
loss of balance. What data is likely to be Long-term, regular use of quinine for management of leg
linked to the client's hearing deficit? cramps is associated with loss of hearing acuity. Radiation
therapy for cancer should not affect hearing; however,
a) Previous perforation of the eardrum as a hearing can be significantly compromised by
result of a high dive chemotherapy. Allergy to hair products may be associated
b) Routine use of quinine for management of with otitis externa; however, it is not linked to hearing loss.
leg cramps An ear drum that perforates spontaneously due to the
c) Recent completion of radiation therapy for sudden drop in altitude associated with a high dive usually
treatment of thyroid cancer heals well and is not likely to become infected. Recurrent
d) Allergy to hair coloring and hair spray otitis media with perforation can affect hearing as a result
of chronic inflammation of the ossicles in the middle ear.
1497 While taking the health history of a newly Biofeedback, relaxation, hypnosis.
. admitted client, the nurse reviews general
lifestyle behaviors. Which of the following
would have a positive effect on the immune
system?
1498 While visiting the pediatric clinic with her 2 Active acquired immunity, because the person's own body
. year old, a mother picks up a brochure about develops defenses
immunizations and asks about active and
passive acquired immunity to childhood
diseases. The nurse explains that
immunizations are which of the following
and why?
1499 Why are antacids administered regularly, To maintain gastric pH at 3.0-3.5
. rather than as needed, in peptic ulcer
disease?
1500 Why is it important for a nurse to provide Manage decreased energy levels
. required information and appropriate
explanations of diagnostic procedures to
patients with respiratory disorders?
1501 Why should the nurse encourage a client Chewing may cause discomfort
. with otitis externa to eat soft foods? Choose Correct
the correct option. Explanation:
The nurse encourages a client with otitis externa to eat soft
a) Chewing may lead to further foods or consume nourishing liquids because chewing may
complications, such as otitis media cause discomfort.
b) Chewing may cause discomfort
c) Chewy foods, such as red meat, may react
with the prescribed analgesics and antibiotics
d) Chewing may cause excessive drainage
1502 Within our brains, cerebrospinal fluid (CSF) Subarachnoid space
. is manufactured in the ventricles and
constantly circulates around the brain and
spinal cord. The CSF functions as a cushion
to protect structures and maintain relatively
consistent intracranial pressure. Where does
CSF circulate?
1503 Within the physiology of the heart, each Right atrium
. chamber has a particular role in maintaining Explanation: The right atrium receives deoxygenated
cellular oxygenation. Which chamber of the blood from the venous system.
heart is responsible for receiving
deoxygenated blood from the venous system?
1504 Within the physiology of the heart, each Right atrium
. chamber has a particular role in maintaining Explanation:
cellular oxygenation. Which chamber of the The right atrium receives deoxygenated blood from the
heart is responsible for receiving venous system.
deoxygenated blood from the venous system?
1505 A woman whose husband was recently advising her to begin prophylactic therapy with isoniazid
. diagnosed with active pulmonary tuberculosis (INH).
(TB) is a tuberculin skin test converter but Explanation: Individuals who are tuberculin skin test
doesn't show signs of active tuberculosis. converters should begin a 6-month regimen of an
Management of her care would include: antitubercular drug such as INH, and they should never
have another skin test. After an individual has a positive
tuberculin skin test, subsequent skin tests will cause
severe skin reactions but won't provide new information
about the client's TB status. The client doesn't have active
TB, so she can't transmit, or spread, the bacteria.
Therefore, she shouldn't be quarantined or asked for
information about recent contacts.
1506 A woman whose husband was recently d) advising her to begin prophylactic therapy with
. diagnosed with active pulmonary tuberculosis isoniazid (INH).
(TB) is a tuberculin skin test converter.
Management of her care would include:

a) scheduling her for annual tuberculin skin


testing.
b) placing her in quarantine until sputum
cultures are negative.
c) gathering a list of persons with whom she
has had recent contact.
d) advising her to begin prophylactic therapy
with isoniazid (INH).
1507 Working hard to memorize the functions of All options are correct
. the cranial nerves is a typical part of nursing
school. Not only is it important to correlate the
proper nerve number and name, but including
the proper function makes this task quite a
challenge! Which cranial nerves are enabling
you to read this question?
1508 You are caring for a 72-year-old client who has Dehydration - The most common fluid imbalance in older
. been admitted to your unit for a fluid volume adults is dehydration. Because of reduced thirst sensation
imbalance. You know which of the following is that often accompanies aging, older adults tend to drink
the most common fluid imbalance in older less water. Use of diuretic medications, laxatives, or
adults? enemas may also deplete fluid volume in older adults.
Chronic fluid volume deficit can lead to other problems
such as electrolyte imbalances.
1509 You are caring for a 72-year-old client who has Dehydration - The most common fluid imbalance in older
. been admitted to your unit for a fluid volume adults is dehydration. Because of reduced thirst sensation
imbalance. You know which of the following is that often accompanies aging, older adults tend to drink
the most common fluid imbalance in older less water. Use of diuretic medications, laxatives, or
adults? enemas may also deplete fluid volume in older adults.
Chronic fluid volume deficit can lead to other problems
such as electrolyte imbalances. Therefore, options A, C,
and D are incorrect.
1510 You are caring for a client admitted with "Conditions such as chronic bronchitis cause thickening
. chronic bronchitis. The client is having of the bronchial mucosa so it makes it harder to breathe."
difficulty breathing, and the family asks you
what causes this difficulty. What would be
your best response?
1511. You are caring for a client that has been Hypocalcemia - Hypocalcemia or low serum calcium
admitted with a possible clotting disorder. levels can affect clotting. Therefore, in this condition the
The client is complaining of excessive nurse should take extra care to check for bruising or
bleeding and bruising without cause. You bleeding. There is no such risk in dehydration,
know that you should take extra care to check hypokalemia, or hypomagnesemia
for signs of bruising or bleeding in what
condition?
1512 You are caring for a client who has a Side effects of drugs
. diagnosis of HIV. Part of this client's teaching
plan is educating the client about his or her
medications. What is essential for the nurse to
include in the teaching of this client regarding
medications?
1513 You are caring for a client who is in A puncture at the radial artery
. respiratory distress. The physician orders
arterial blood gases (ABGs) to determine
various factors related to blood oxygenation.
What site can ABGs be obtained from?
1514 You are caring for a client who is undergoing Support the client during a bone marrow aspiration and
. bone marrow aspiration to determine their monitor the status.
blood cell formation status. What nursing
intervention should you provide to your client
after the test?
1515 You are caring for a client with a damaged Chordae tendineae
. tricuspid valve. You know that the tricuspid Explanation:
valve is held in place by which of the Attached to the mitral and tricuspid valves are cordlike
following?Chordae tendineae structures known as chordae tendineae, which in turn
attach to papillary muscles, two major muscular
projections from the ventricles. Options B, C, and D are
distractors for the question.
1516 You are caring for a new client on your unit Generalized - There may be generalized edema in all the
. who is third-spacing fluid. You know to assess interstitial spaces, which sometimes is called brawny
for what type of edema? edema or anasarca. Options B and D are not part of the
process of third-spacing fluid. Option C is a distractor for
this question.
1517 You are caring for three clients who have the Client A has a higher WBC count than normal, client B
. following blood count values: Client A has has a normal hemoglobin count, and client C has a normal
24,500 white blood cells (WBCs), client B has platelet count.
13.4 g/dL hemoglobin, and client C has a
250,000/mm3 platelet count. Which statement
correctly describes the condition of each
client?
1518 You are making rounds on your clients. You Administer Sodium Bicarbonate IV - When the client
. find one of your clients struggling to breathe, makes frantic efforts to breathe, breathes slowly, or stops
appears confused, has tachycardia, and the breathing, and has tachycardia, and the skin appears dusky
skin appears dusky. What should you do to (cyanosis), the condition is likely to be acute respiratory
restore normal pH if ventilation efforts are acidosis. The accumulation of CO2 leads to behavioral
not very successful? changes, including confusion. Excess carbonic acid pulls
pH below 7.35. The nurse should administer sodium
bicarbonate IV to balance the acid and bring the pH to a
normal level. Bronchodilators may be useful in chronic
respiratory acidosis but not in the acute version. Potassium
(needed in hypokalemia) and magnesium sulfate (needed
in hypomagnesemia) have no role in acute respiratory
acidosis
1 You are performing pulmonary function In the standing position
5 studies on clients in the clinic. What
1 position do you know a client should be in
9. to have maximum lung capacities and
volumes?
1 You are presenting an educational Older adults are more susceptible to infections and
5 workshop for a local community group. malignancies.
2 You have been asked to speak about older
0. adults and their health. What is the most
important piece of information to include
in this presentation?
1 You are studying for a physiology test They respond to changes in CO2 levels and hydrogen ion
5 about the respiratory system. What should concentrations (pH) in the cerebrospinal fluid.
2 you know about central chemoreceptors in
1. the medulla?
1 You are the hospice nurse caring for a "The fibrosis of the lungs makes the lungs stiff, which makes it
5 client with pulmonary fibrosis who wants harder to breathe.
2 to die at home. The client is having
2. difficulty breathing. The family asks why it
is so hard for the client to breathe. What
would be the nurse's best response?
1 You are the nurse working in an "Many people have diagnostic or short therapeutic surgical
5 ambulatory surgery center. A teenage son procedures."
2 of your clients ask you why so many people
3. have surgery. What would be your best
reply?
1 You are the triage nurse in a walk-in clinic Drug therapy
5 when a diabetic client visits the clinic and
2 asks you to take her blood pressure (BP).
4. The measurements are 150/90 mm Hg.
Which of the following would the nurse
expect as the treatment to normalize the
client's BP?
1 young-old 65-74 years of age
5
2
5.

1 young-old to old-old and frail elderly A 76-year-old client with no debilitating conditions belongs to
5 explaination the middle-old geriatric population. The young-old geriatric
2 population ranges in age from 65 to 74; the middle-old from 75
6. to 84; and the old-old from 85 and older. Within each of these
three subgroups is another group, the frail elderly, which
includes all individuals older than age 65 who have one or more
debilitating conditions.
1 You notify the physician that your client is Start IV fluids and blood products - This is done by
5 third-spacing fluid. What orders would you administering IV solutionssometimes at rapid ratesand
2 expect the physician to give you? blood products, such as albumin, to restore colloidal osmotic
7. pressure. The restriction of fluids; the administration of
diuretics and the increase of sodium in the diet are not orders
the physician would be expected to give for a client is third-
spacing fluids.
1 Your client, a 2-year-old male, is scheduled Reconstructive
5 to have surgery related to his cleft palate.
2 You will be preparing this client for which
8. type of surgery?
1 Your client is taking medications that Signs of leukopenia and thrombocytopenia
5 depress the hematopoietic system. What
2 signs should you closely monitor in this
9. client?
1 Your client's lab values are sodium 166 Metabolic Acidsosis - The anion gap is the difference between
5 mEq/L, potassium 5.0 mEq/L, chloride 115 sodium and potassium cations and the sum of chloride and
3 mEq/L, and bicarbonate 35 mEq/L. What bicarbonate anions. An anion gap that exceeds 16 mEq/L
0. condition is this client likely to have, indicates metabolic acidosis. In this case, the anion gap is (166
judging by anion gap? + 5) minus (115 + 35), yielding 21 mEq/L, which suggests
metabolic acidosis. Anion gap is not used to check for
respiratory alkalosis, metabolic alkalosis, or respiratory acidosis
A 49-year-old Monoamine oxidase (MAO) inhibitors
painter who
recently
fractured his
tibia worries
about his
finances
because he
can't work. To
treat his
anxiety, his
physician
prescribes
buspirone
(BuSpar), 5
mg by mouth
three times
per day.
During
buspirone
therapy, the
client should
avoid which
of the
following
drugs?
2. A 59-year-old client "4. sedatives reduce excitement; hypnotics induce sleep.
is scheduled for
cardiac
catheterization the
next morning. His
physician
prescribed
secobarbital
sodium (Seconal),
100 mg by mouth
at bedtime, for
sedation. Before
administering the
drug, the nurse
should know that:
3. "After months of 2. Desensitization 3. Alprazolam (Xanax) therapy 4. Paroxetine
coaxing by her (Paxil) therapy
husband, a client
comes to the
mental health
clinic. She reports
that she suffers
from an
overwhelming fear
of leaving her
house. This
overwhelming fear
has caused the
client to lose her
job and is
beginning to take a
toll on her
marriage. The
physician diagnoses
the client with
agoraphobia.
Which treatment
options are
effective in treating
this disorder?
4. After seeking help 1. Exploring the meaning of the traumatic event with the client
at an outpatient
mental health
clinic, a client who
was raped while
walking her dog is
diagnosed with
posttraumatic
stress disorder
(PTSD). Three
months later, the
client returns to the
clinic, complaining
of fear, loss of
control, and
helpless feelings.
Which nursing
intervention is
most appropriate
for this client?
5. Because 1. Avoid mixing antianxiety agents with alcohol or other central
antianxiety agents nervous system (CNS) depressants
such as lorazepam
(Ativan) can
potentiate the
effects of other
drugs, the nurse
should incorporate
which instruction
in her teaching
plan?
6. Before eating a "Systematically decrease the number of repetitions of rituals and
meal, a client with the amount of time spent performing them.
obsessive-
compulsive
disorder (OCD)
must wash his
hands for 18
minutes, comb his
hair 444 strokes,
and switch the
bathroom light on
and off 44 times.
What is the most
appropriate goal of
care for this client?
7 A client admitted to the psychiatric "3. escort the client to a quiet area and suggest using a relaxation
. unit for treatment of repeated panic exercise that he's been taught.
attacks comes to the nurses' station
in obvious distress. After observing
that the client is short of breath,
dizzy, trembling, and nauseated, the
nurse should first:
8 A client admitted to the unit is 3. Increased heart rate
. visibly anxious. When collecting
data on the client, the nurse would
expect to see which cardiovascular
effect produced by the sympathetic
nervous system?
9 A client, age 40, is admitted for a "In case anything goes wrong? What are your thoughts and feelings
. surgical biopsy of a suspicious lump right now?"
in her left breast. When the nurse
comes to take her to surgery, she is
tearfully finishing a letter to her two
children. She tells the nurse, "I want
to leave this for my children in case
anything goes wrong today." Which
response by the nurse would be most
therapeutic?
1 A client arrives on the psychiatric "3. Risk for injury
0 unit exhibiting extreme excitement,
. disorientation, incoherent speech,
agitation, frantic and aimless
physical activity, and grandiose
delusion. Which nursing diagnosis
takes highest priority for the client
at this time?
1 A client comes to the emergency staying with the client until the attack subsides
1 department while experiencing a
. panic attack. The nurse should
respond to a client having a panic
attack by:
1 A client diagnosed as having panic 2. To help the client function effectively in her environment
2 disorder with agoraphobia is
. admitted to the inpatient psychiatric
unit. Until her admission, she had
been a virtual prisoner in her home
for 5 weeks, afraid to go outside even
to buy food. When planning care for
this client, what is the nurse's overall
goal?
1 A client enters the crisis unit 3. Diphenhydramine (Benadryl)
3 complaining of increased stress from
. her studies as a medical student. She
states that she has been increasingly
anxious for the past month. Her
physician prescribes alprazolam
(Xanax), 25 mg by mouth three
times per day, along with
professional counseling. Before
administering alprazolam, the nurse
reviews the client's medication
history. Which drug can produce
additive effects when given
concomitantly with alprazolam?
1 A client in a psychiatric facility is 2. Consult a pharmacist to see if these symptoms are adverse effects of
4 prescribed escitalopram (Lexapro) the drug.
. for anxiety. She tells the nurse that
she has been having "weird
dreams" and feelings of wanting to
"end it all." What action should the
nurse take?
1 A client is admitted to an inpatient "repetitive thoughts and recurring, irresistible impulses.
5 psychiatric unit for treatment of
. obsessive-compulsive symptoms.
Obsessive-compulsive disorder
(OCD) is associated with:
1 A client is admitted to the acute Administering a sedative as prescribed
6 psychiatric care unit after 2 weeks
. of increasingly erratic behavior.
The client has been sleeping poorly,
has lost 8 lb (3.6 kg), is poorly
groomed, exhibits hyperactivity,
and loudly denies the need for
hospitalization. Which nursing
intervention takes priority for this
client?
1 A client is admitted to the 1. helping the client identify and verbalize feelings about the incident.
7 psychiatric unit with a diagnosis of
. conversion disorder. Since
witnessing the beating of his wife at
gunpoint, he has been unable to
move his arms, complaining that
they are paralyzed. When planning
the client's care, the nurse should
focus on:
1 A client is diagnosed with obsessive- 2. Giving the client adequate time to perform rituals
8 compulsive disorder. Which
. intervention should the nurse
include when assisting with
development of the plan of care?
1 A client is undergoing treatment for 3. 6 months
9 an anxiety disorder. Such a disorder
. is considered chronic and
generalized when excessive anxiety
and worry about two or more life
circumstances exist for at least:
2 A client tells the nurse that she has 2. Panic disorder
0 an overwhelming fear of having a
. heart attack. This client is most
likely suffering from which
disorder?
2 A client who has been diagnosed " ""Your health information is confidential, and I can't talk to anyone
1 with a sexually transmitted disease about it without your permission.""
. (STD) asks that this information
not be shared with her family
members. Which of the following
responses from the nurse would be
appropriate?
22 A client who lost her home and dog in an earthquake tells the admitting "3. posttraumatic stress
. nurse at the community health center that she finds it harder and harder to disorder (PTSD).
"feel anything." She says she can't concentrate on the simplest tasks, fears
losing control, and thinks about the earthquake incessantly. She becomes
extremely anxious whenever the earthquake is mentioned and must leave
the room if people talk about it. The nurse suspects that she has:
23 A client who recently developed paralysis of the arms is diagnosed with Exercising the client's arms
. conversion disorder after tests fail to uncover a physical cause for the regularly
paralysis. Which intervention should the nurse include in the plan of care?
24 A client with a conversion disorder reports blindness, and ophthalmologic 2. having been forced to
. examinations reveal that no physiologic disorder is causing progressive watch a loved one's torture.
vision loss. The most likely source of this client's reported blindness is:
25 A client with agoraphobia has been symptom-free for 4 months. Classic 2. severe anxiety and fear.
. signs and symptoms of phobias include:
26 A client with a history of drug and alcohol abuse is concerned that the 1. "Your personal health
. hospital will divulge her history to her employer without her knowledge. information can't be
What response by the nurse would be appropriate? disclosed to your employer
without your permission."
27 A client with borderline personality disorder tells the nurse, "You're the 2. ""I'll have to discuss your
. only nurse who really understands me. The others are mean." The client request with the team. Can
then asks the nurse for an extra dose of antianxiety medication because of we talk about how you're
increased anxiety. How should the nurse respond? feeling right now?""
28 A client with obsessive-compulsive disorder and ritualistic behavior must 3. setting consistent limits
. brush the hair back from his forehead 15 times before carrying out any on the ritualistic behavior if
activity. The nurse notices that the client's hair is thinning and the skin on it harms the client or others.
the forehead is irritated possible effects of this ritual. When planning the
client's care, the nurse should assign highest priority to:
29 A client with obsessive-compulsive disorder may use reaction 1. The client assumes an attitude
. formation as a defense mechanism to cope with anxiety and stress. that is the opposite of an impulse
What typically occurs in reaction formation? that the client harbors.
30 A client with obsessive-compulsive disorder tells the nurse that he reduce anxiety.
. must check the lock on his apartment door 25 times before leaving
for an appointment. The nurse knows that this behavior represents
the client's attempt to:
31 During alprazolam (Xanax) therapy, the nurse should be alert for 1. Ataxia
. which dose-related adverse reaction?
32 During a panic attack, a client hyperventilates, becomes unable to 4. Accompany the client to his
. speak, and reports symptoms that mimic those of a heart attack. room; remain there and provide
Which nursing intervention would be best? instructions in short, simple
statements.
33 During a panic attack, a client runs to the nurse and reports 1. Assist the client to breathe
. breathing difficulty, chest pain, and palpitations. The client is pale deeply into a paper bag
with his mouth wide open and eyebrows raised. What should the
nurse do first?
34 During a shift report, the nurse learns that she will be providing care 2. antianxiety drugs.
. for a client who's vulnerable to panic attacks. Treatment for panic
attacks includes behavioral therapy, supportive psychotherapy, and
medication such as
35 During the admission data collection, a client with a panic disorder 4. ""You're having a panic attack.
. begins to hyperventilate and says, "I'm going to die if I don't get out I'll stay here with you
of here right now!" What is the nurse's best response?
36 During the client-teaching session, which instruction should the "Inform the physician if you
. nurse give to a client receiving alprazolam (Xanax)? become pregnant or intend to do
so."
37 Initial interventions for the client with acute anxiety include: 2. encouraging the client to
. verbalize feelings and concerns.
38 Lorazepam (Ativan) is often given along with a neuroleptic agent, 1. To reduce anxiety and potentiate
. such as haloperidol (Haldol). What is the purpose of administering the sedative action of the
the drugs together? neuroleptic
39 The nurse discovers that a client with obsessive-compulsive disorder (OCD) 4. increased anxiety.
. is attempting to resist the compulsion. Based on this finding, the nurse
should look for signs of:
40 A nurse has been providing care to the same group of clients for 4 2. Voice her concerns about
. consecutive days. On day 5, she sees that her assignment has changed, and continuity of care with the
she is concerned about the continuity of care for these clients. What should charge nurse.
the nurse do?
41 The nurse in a psychiatric inpatient unit is caring for a client with "1. avoid caffeine.
. obsessive-compulsive disorder. As part of the client's treatment, the
psychiatrist orders lorazepam (Ativan), 1 mg by mouth three times per day.
During lorazepam therapy, the nurse should remind the client to:
42 The nurse is caring for a client experiencing an anxiety attack. Appropriate 3. staying with the client and
. nursing interventions include: speaking in short sentences.
43 The nurse is caring for a client with panic disorder who has difficulty "2. Encouraging the use of
. sleeping. Which nursing intervention would best help the client achieve relaxation exercises
healthy long-term sleeping habits?
44 The nurse is caring for a Vietnam War veteran with a history of explosive 1. "Many people who have
. anger, unemployment, and depression since being discharged from the been in your situation
service. The client reports feeling ashamed of being "weak" and of letting experience similar emotions
past experiences control thoughts and actions in the present. What is the and behaviors."
nurse's best response?
45 The nurse is collecting data on a client suffering from stress and anxiety. A 2. diarrhea
. common physiological response to stress and anxiety is:
46 The nurse is formulating a short-term goal for a client suffering from a "2. participate in a daily
. severe obsessive-compulsive disorder (OCD). An appropriately stated exercise group.
short-term goal is that after 1 week, the client will:
47 A nurse notices that a client who came to the clinic for "2. Ask the client basic hygiene
. treatment of anxiety disorder has a strong body odor. What can questions to determine how frequently
the nurse do or say to help this client? he bathes.
48 The nurse notices that a client with obsessive-compulsive 1. I saw you change clothes several
. disorder dresses and undresses numerous times each day. times today. That must be very tiring.
Which comment by the nurse would be therapeutic?
49 The nurse notices that a client with obsessive-compulsive "1. By designating times during which
. disorder washes his hands for long periods each day. How the client can focus on the behavior
should the nurse respond to this compulsive behavior?
50 A nurse observes a medical student walk into a client's room 3. Explain to the client that she has the
. and begin questioning her about her current health status. The right to refuse to answer questions asked
client appears reluctant to respond. How should the nurse by the medical student.
intervene?
51 A nurse on the psychiatric unit realizes that she typically fails to "2. Evaluate her current practice and
. administer medications according to schedule. What's the best devise an improvement plan.
way for the nurse to improve her medication administration
practice?
52 The nurse refers a client with severe anxiety to a psychiatrist for 1. Buspirone (BuSpar), 5 mg orally three
. medication evaluation. The physician is most likely to prescribe times per day "
which psychotropic drug regimen for this client?
53 The physician orders a new medication for a client with "Do you have any concerns about taking
. generalized anxiety disorder. During medication teaching, the medication?"
which statement or question by the nurse would be most
appropriate?
54 A physician's order states to administer lorazepam (Ativan), 20 2. Clarify the order with the prescribing
. mg by mouth twice per day, to treat anxiety. How should the physician because the amount prescribed
nurse proceed? exceeds the recommended dose.
5 (SELECT ALL THAT APPLY) After being examined by the " 1. Recurrent, intrusive recollections or
5 forensic nurse in the emergency department, a rape victim is nightmares 3. Sleep disturbances 6.
. prepared for discharge. Due to the nature of the attack, this Difficulty concentrating "
client is at risk for posttraumatic stress disorder (PTSD).
Which symptoms are associated with PTSD?
5 (SELECT ALL THAT APPLY) After receiving a referral from 2. Support the use of appropriate defense
6 the occupational health nurse, a client comes to the mental mechanisms. 4. Explore the patterns leading
. health clinic with a suspected diagnosis of obsessive- to the compulsive behavior. 6. Encourage
compulsive disorder. The client explains that his compulsion activities, such as listening to music."
to wash his hands is interfering with his job. Which
interventions are appropriate when caring for a client with
this disorder?
5 (SELECT ALL THAT APPLY) A physician prescribes "1. Avoid hazardous activities that require
7 clomipramine (Anafranil) for a client diagnosed with alertness or good coordination until adverse
. obsessive-compulsive disorder (OCD). What instructions central nervous system (CNS) effects are
should the nurse include when teaching the client about this known. 2. Avoid alcohol and other
medication? depressants. 3. Use saliva substitutes or
sugarless candy or gum to relieve dry
mouth. "
5 (SELECT ALL THAT APPLY) A registered nurse caring for a 4. Observe the client for overt signs of
8 client with generalized anxiety disorder identifies a nursing anxiety. 5. Help the client connect anxiety
. diagnosis of Anxiety. The short-term goal identified is: The with uncomfortable physical, emotional, or
client will identify his physical, emotional, and behavioral behavioral responses. 6. Introduce the client
responses to anxiety. Which nursing interventions will help to new strategies for coping with anxiety,
the client achieve this goal? such as relaxation techniques and exercise.
5 (SELECT AL THAT APPLY) A 54-year-old client diagnosed 1. Biofeedback 2. Buspirone 3. Relaxtion
9 with generalized anxiety disorder is admitted to the facility. technique
. Which therapeutic modalities are typically used to treat this
disorder?
Abnor Vesicoureteral Reflux (VUR)
mal
retrogr
ade
flow of
bladde
r urine
into the
ureters
.
2. An acute renal disease Hemolytic Uremic Syndrome (HUS)
caused by bacteria,
chemicals or viruses.
Toxin damaged
glomeruli lining swells
and occludes with clots.
3. After a surgical repair B. To provide an alternative urinary elimination route.
of a hypospadias, a 12
month old child returns
to the nursing unit with
an intravenous line, a
urethral catheter, and a
suprapubic catheter in
place. Which of the
following would the
nurse explain to the
parents is the primary
purpose for the
suprapubic catheter?
A. To ensure an
accurate measurement
of urine output.
B. To provide an
alternative urinary
elimination route.
C. To provide an entry
port for bladder
irrigation.
D. To allow assessment
for blood clots in the
urine.
4. Can children with yes
Vesicoureteral Reflux
(VUR) outgrow the
condition?
5. Children less than __ Children less than 2 years may be unable to have bladder control due
______ may be unable to immature nerve function
to have bladder control
due to immature nerve
function
6. A clinical state that Nephrotic Syndrome
includes massive
proteinuria,
hypoalbuminemia,
hyperlipidemia and
edema. Increased
glomerular
permeability to plasma
protein
7. Do you you lose red no
blood cells in Nephrotic
Syndrome?
8. How is Vesicoureteral Diagnosed with VCUG
Reflux (VUR)
diagnosed?
9. How is Vesicoureteral Others treated by prophylactic antibiotics, polymer injection, or
Reflux (VUR) treated? reconstructive surgery (ureteral reimplantation)
1 How much does bladder of Bladder of infant holds 20-50 ml.
0 infant hold? Adult 700 ml.
.
Adult?
1 Immune complex disease that Acute Postinfectious Glomeronephritis
1 occurs after a pneumococcal,
. streptococcal or viral
infection. Immune complexes
deposit in the glomerular
membrane. Decreased
filtration
1 Infants and Toddlers tend to Infants and Toddlers tend to have non- classic symptoms such as fever, poor
2 have what types of symptoms feeding and vomiting.
. that indicate urinary tract
infection?
1 In what age Hemolytic Most common < 5 yrs
3 Uremic Syndrome (HUS)
. most common?

1 ________ less efficient at Kidneys less efficient at regulating fluid and electrolyte balance and acid base
4 regulating fluid and balance in infants and children.
. electrolyte balance and acid
base balance in infants and
children.
1 Location of the urethral Epispadius
5 opening is on the dorsal of the
. shaft.

1 Location of the urethral Hypospadius


6 opening is on the ventral side
. of the shaft.
1 The mother of a preschooler B. Elevating the head of the child's bed
7 with nephrotic syndrome asks
. the nurse what can be done
about the child's swollen eyes.
Which of the following would
the nurse suggest?
A. Applying cool compresses
to the child's eyes
B. Elevating the head of the
child's bed
C. Applying eye drops every 8
hours
D. Limiting the child's
television watching
1 A school-age boy is admitted D.Playing a card game with another child the same age.
8 for treatment of a urinary
. tract infection. Which of the
following activities would be
best for a diversional activity?

A.Putting a puzzle together


with his mother.
B.Playing a video game with a
4 year old.
C.Watching a movie with his
sister.
D.Playing a card game with
another child the same age.
1 A school age child with acute B. Ice chips
9 post streptococcal
. glomerulonephritis who has a
fluid restriction of 1,000 ml/day,
which of the following fluids
would the nurse consider as
most appropriate for the child's
condition and effective for
preventing excessive thirst?

A. Diet cola
B. Ice chips
C. Lemonade
D. Water
2 The toddler with nephrotic B. Keep the child away from others with an infection.
0 syndrome responds to
. treatment and is ready to go
home. When helping the family
plan for home care, which of
the following instructions
would the nurse include in the
teaching?
A. Administer pain medication
as needed.
B. Keep the child away from
others with an infection.
C. Notify the physician if there
is an increase in the child's
urine output.
D. Administer acetaminophen
daily
2 The urinalysis of a toddler with B. Increased glomerular permeability
1 nephrotic syndrome reveals +4
. for protein. The nurse
interprets this result as
indicating which of the
following

A. Decreased secretion of
aldosterone
B. Increased glomerular
permeability
C. Inhibited tubular
reabsorption of sodium and
water
D. Loss of red blood cells
2 What age is Nephrotic Most common between 2 and 7 years.
2 Syndrome most common?
.

2 What are general symptoms of polyuria, oliguria, pallor, dizziness, fatigue, headache nausea, hypertension,
3 renal failure? edema, hyperkalemia, hyponatremia and hypocalcaemia. Delayed growth.
.

2 What are the symptoms of Acute Symptoms - Edema, anorexia, cola colored urine, oliguria, pale,
4 Postinfectious Glomeronephritis? elevated BP, hematuria, proteinuria. Possible vomiting, HA or
. abdominal discomfort
2 What are the symptoms of Hemolytic anemia, vomiting, renal injury,
5 Uremic Syndrome (HUS)? thrombocytopenia, oliguria,
. high BUN & creatinine,
petechiae and CNS symptoms.
2 What can Vesicoureteral Reflux (VUR) do to Can cause renal scarring and hydronephrosis.
6 the kidneys?
.

2 What happens to RBCs and platelets in RBCs become damaged and platelets
7 patients with Hemolytic Uremic Syndrome are drawn to the area of damage
. (HUS)
?
2 What is post op care for Post-op Care - Maintain Stint, double diaper, limit activity,
8 Hypospadius/Epispadius? hydration, hourly I and O, anticholenergics for spasms, pain
. meds.
2 What is the nursing care for Nephrotic Nursing care includes administering corticosteroids, low salt
9 Syndrome? diet, possible diuretics and fluid restrictions, skin care, strict I
. and O
3 What is the nursing management for Acute Nursing - Daily weights, Intake and Output, Possible fluid
0 Postinfectious Glomeronephritis? restrictions and low salt diet
.

3 What is the treatment for Hemolytic Uremic Tx- dialysis, blood transfusions, fluid restrictions, monitor vs
1 Syndrome (HUS)? and neuro symptoms
.

3 What is the treatment for renal failure? Management- Fluid therapy, diuretics, Diet low in NA, K and
2 Phos. Ace inhibitors, Epogen
. prevent infection, dialysis, electrolytes
3 What is typically the first sign of Nephrotic periorbital edema, that is often mistaked for an allergy
3 Syndrome?
.

3 What type of kidney failure mightonic chronic kidney failure


4 Polycystic Kidney Disease, reflux, congenital
. anomalies cause?

3 What type of renal failure might HUS, acute renal failure


5 poisoning or obstructive disorders cause?
.

1 A 3-way indwelling urinary continuous inflow and outflow of irrigation solution.


. catheter is inserted for
continuous bladder irrigation
following a transurethral
resection of the prostate. In
addition to inflating the balloon,
the functions of the three lumens
include:
2 A 25-year-old client comes to the Collect forensic evidence.
. emergency department with her
clothes torn. She has visible cuts,
bruises, and profuse vaginal
bleeding. A nurse suspects that
this client has been raped. What
should the nurse do?
3 A 25-year-old female client seeks Phenazopyridine (Pyridium)
. care for a possible urinary tract
infection (UTI). Her symptoms
include burning on urination and
frequent, urgent voiding of small
amounts of urine. She's placed on
trimethoprim-sulfamethoxazole
(Bactrim) to treat the possible
infection. Another medication is
prescribed to decrease the client's
pain and frequency. Which of the
following is most likely the
second medication prescribed?
4 After having a transurethral The client reports bladder spasms and the urge to void.
. resection of the prostate (TURP),
a client returns to the unit with a
three-way indwelling urinary
catheter and continuous closed
bladder irrigation. Which finding
suggests that the client's catheter
is occluded?
5 Because of difficulties with White blood cell (WBC) count of 20,000/mm3
. hemodialysis, peritoneal dialysis
is initiated to treat a client's
uremia. Which finding signals a
significant problem during this
procedure?
6 A charge nurse in a long-term The charge nurse performs treatments and supervises staff, the LPN
. care facility is planning the administers medications and assists with care, and the nursing assistants
nursing assignments for the provide direct client care.
oncoming shift. Her staff consists
of four nursing assistants and a
licensed practical nurse (LPN).
How should she divide nursing
care among the staff to
adequately ensure safe, effective
care?
7 A client admitted with a gunshot Urine output of 400 ml/24 hours
. wound to the abdomen is
transferred to the intensive care
unit after an exploratory
laparotomy. Which data
collection finding suggests that
the client is experiencing acute
renal failure (ARF)?
8 A client comes to the emergency Acute pain
. department complaining of severe
pain in the right flank, nausea, and
vomiting. The physician tentatively
diagnoses right ureterolithiasis
(renal calculi). When planning this
client's care, the nurse should assign
highest priority to which nursing
diagnosis?
9 A client comes to the emergency Kidney
. department complaining of sudden
onset of sharp, severe pain in the
lumbar region, which radiates
around the side and toward the
bladder. The client also reports
nausea and vomiting and appears
pale, diaphoretic, and anxious. The
physician tentatively diagnoses renal
calculi and orders flat-plate
abdominal X-rays. Renal calculi can
form anywhere in the urinary tract.
What is their most common
formation site?
1 A client comes to the outpatient Gonorrhea
0 department complaining of vaginal
. discharge, dysuria, and genital
irritation. Suspecting a sexually
transmitted disease (STD), the
physician orders diagnostic tests of
the vaginal discharge. Which STD
must be reported to the public
health department?
1 A client develops acute renal failure hyperkalemia.
1 (ARF) after receiving an I.V.
. nephrotoxic antibiotic. Because the
client's 24-hour urine output totals
240 ml, the nurse suspects that the
client is at risk for:
1 A client diagnosed with a sexually Explain that although she is a friend of the client, the nurse can't violate
2 transmitted disease has been feeling client confidentiality.
. poorly. A friend of the client's who is
employed by the hospital asks a
nurse why her friend is hospitalized.
How should the nurse respond?
1 A client diagnosed with renal calculi Perform nonpharmacologic pain interventions.
3 is experiencing severe pain despite
. having received pain medication. A
nurse pages a physician. Which
intervention can the nurse perform
while awaiting the physician's
response?
1 A client in the short-procedure unit check the client's pedal pulses frequently.
4 is recovering from renal
. angiography in which a femoral
puncture site was used. When
providing postprocedure care, the
nurse should:
1 A client is admitted for treatment water and sodium retention secondary to a severe decrease in the
5 of chronic renal failure (CRF). The glomerular filtration rate.
. nurse knows that this disorder
increases the client's risk of:
1 A client is admitted for treatment generalized edema, especially of the face and periorbital area.
6 of glomerulonephritis. On initial
. assessment, the nurse detects one
of the classic signs of acute
glomerulonephritis of sudden
onset. Such signs include:
1 A client is admitted with a drug toxicity.
7 diagnosis of acute renal failure.
. The nurse should monitor closely
for:
1 A client is frustrated and Assessing present elimination patterns
8 embarrassed by urinary
. incontinence. Which of the
following measures should the
nurse include in a bladder
retraining program?
1 A client receiving total parental start after a known voiding that empties the bladder.
9 nutrition is prescribed a 24-hour
. urine test. When initiating a 24-
hour urine specimen, the collection
time should:
2 A client reports experiencing Cottage cheese-like discharge
0 vulvar pruritus. Which finding
. may indicate that the client has an
infection caused by Candida
albicans?
2 A client requires hemodialysis. Cardiac glycosides
1 Which of the following drugs
. should be withheld before this
procedure?
2 A client tells a nurse that her Consult the wound-ostomy nurse.
2 ileoconduit appliance won't adhere
. to her skin. The nurse inspects the
site and notes that the area around
the stoma is red, moist, and tender
to touch. How should the nurse
intervene?
2 A client undergoes extracorporeal "Increase your fluid intake to 2 to 3 L per day."
3 shock wave lithotripsy. Before
. discharge, the nurse should
provide which instruction?
2 The client underwent a Use aseptic technique when irrigating the catheter.
4 transurethral resection of the
. prostate gland 24 hours ago and
has a continuous bladder
irrigation. Which of the following
nursing interventions is
appropriate?
2 A client who has been treated for "Make sure to include carbohydrates in your diet."
5 chronic renal failure (CRF) is
. ready for discharge. The nurse
should reinforce which dietary
instruction?
2 A client who returned from a Sit in a warm sitz bath.
6 cystoscopic examination complains
. of pain while attempting to void.
Which intervention should a nurse
suggest to ease the client's pain
while attempting to void?
27 A client who returns to the surgical floor after undergoing Check the client's medical record
. transurethral resection of the prostate complains of pain. Which action for postoperative orders.
should the nurse take first?
28 A client with acute pyelonephritis is prescribed co-trimoxazole Bacteria are absent on urine
. (Septra). Which finding best demonstrates that the client has followed culture.
the prescribed regimen?
29 A client with a genitourinary problem is being assessed in the The left kidney usually is slightly
. emergency department. When palpating the client's kidneys, the nurse higher than the right one.
should keep which anatomical fact in mind?
30 A client with a history of chronic cystitis comes to the outpatient clinic Milk
. with signs and symptoms of this disorder. To prevent cystitis from
recurring, the nurse recommends maintaining an acid-ash diet to
acidify the urine, thereby decreasing the rate of bacterial
multiplication. On an acid-ash diet, the client must restrict which
beverage?
31 A client with a history of heart failure is found to have a cystocele. Stress urinary incontinence
. When planning care for this client, the nurse is likely to formulate
which nursing diagnosis?
32 A client with an indwelling urinary catheter is suspected of having a wiping the self-sealing aspiration
. urinary tract infection. The nurse should collect a urine specimen for port or stopcock with antiseptic
culture and sensitivity by: solution and aspirating urine with
a sterile needle and a sterile
syringe.
33 A client with a suspected diagnosis of renal cancer is ordered to Notify a physician that the client
. undergo a renal biopsy to confirm the diagnosis. The client informs a refuses to give consent.
nurse that she will not sign the informed consent form. Which action
should the nurse take?
34 A client with a urinary tract infection is prescribed co-trimoxazole "Drink at least eight 8-oz glasses
. (trimethoprim-sulfamethoxazole). The nurse should provide which of fluid daily."
medication instruction?
35 A client with benign prostatic hyperplasia (BPH) doesn't respond to medical Transurethral resection
. treatment and is admitted to the facility for surgical intervention. Before of the prostate (TURP)
providing preoperative and postoperative instructions to the client, the nurse
asks the surgeon which prostatectomy procedure will be done. What is the most
widely used procedure for treatment of BPH?
36 A client with bladder cancer has had his bladder removed and an ileal conduit The pouch faceplate
. created for urine diversion. While changing this client's pouch, the nurse doesn't fit the stoma.
observes that the area around the stoma is red, weeping, and painful. What
should the nurse conclude?
37 A client with chronic renal failure (CRF) is admitted to the urology unit. Which Blood urea nitrogen
. diagnostic test results are consistent with CRF? (BUN) 100 mg/dl and
serum creatinine 6.5
mg/dl
38 A client with chronic renal failure (CRF) is receiving a hemodialysis treatment. weight loss.
. After hemodialysis, the nurse knows that the client is most likely to experience:
39 A client with chronic renal failure has a serum potassium level of 6.8 mEq/L. Cardiac rhythm
. What should the nurse assess first?
40 A client with chronic renal failure must restrict her fluid intake to 500 ml daily. Allow her to have a
. Despite having reached the limit, the client is insisting that she have more fluid. piece of hard candy.
Which intervention by a nurse is appropriate?
41 A client with chronic renal failure who receives hemodialysis three times weekly epoetin alfa (Epogen)
. has a hemoglobin (Hb) level of 7 g/dl. The most therapeutic pharmacologic
intervention would be to administer:
42 A client with decreased urine output refractory to fluid challenges is evaluated Nephrotoxic injury
. for renal failure. Which condition may cause the intrinsic (intrarenal) form of secondary to use of
acute renal failure? contrast media
43 A client with dysuria is prescribed phenazopyridine (Pyridium). The nurse appear orange.
. should advise the client that his urine will:
44 A client with renal cancer who has not yet been informed of his diagnosis asks "It must be difficult for
. the nurse what his test results showed. How should the nurse respond? you not to know the
results of your test."
45 A client with renal dysfunction of acute onset comes to the emergency recent streptococcal infection.
. department complaining of fatigue, oliguria, and coffee-colored urine.
When obtaining the client's history to check for significant findings,
the nurse should ask about:
46 A client with renal failure is undergoing continuous ambulatory Risk for infection
. peritoneal dialysis. Which nursing diagnosis is most appropriate for
this client?
47 A client with suspected renal insufficiency is scheduled for a The nephron
. comprehensive diagnostic workup. After the nurse explains the
diagnostic tests, the client asks which part of the kidney "does the
work." Which answer is correct?
48 Discharge teaching has been performed for a client who is being The client holds the drainage bag
. discharged with an indwelling urinary catheter. Which action by the below the level of the bladder.
client indicates that the teaching was successful?
49 During a routine examination, the nurse notes that the client seems stimulating or hindering
. unusually anxious. Anxiety can affect the genitourinary system by: micturition.
50 During rounds, a client admitted with gross hematuria asks the nurse Provide privacy for the
. about the physician's diagnosis. To facilitate effective communication, conversation.
what should the nurse do?
51 A female client has just been diagnosed with condylomata acuminata This condition puts her at a higher
. (genital warts). What information is appropriate to tell this client? risk for cervical cancer; therefore,
she should have a Papanicolaou
(Pap) smear annually.
52 A female client with a history of four urinary tract infections (UTIs) in Wipe the perineal area from front
. the past 3 months comes to the urology clinic complaining of burning to back.
and urinary urgency and frequency. A physician makes the diagnosis
of UTI. Which instruction should the nurse give the client to help
prevent recurring infections?
53 For a client in the oliguric phase of acute renal failure (ARF), which Limiting fluid intake
. nursing intervention is most important?
54 Four days after undergoing a right nephrectomy, a client develops a Discard the dressing in a
. methicillin-resistant Staphylococcus aureus infection in the surgical biohazard bag located in the
incision. A physician orders contact isolation and dressing changes 3 times designated receptacle inside
daily. How should the soiled dressing be handled during dressing changes? the client's room.
55 The nurse correctly identifies a urine sample with a pH of 5.2 as being Acidic
. which type of solution?
56 The nurse is caring for a client in acute renal failure. The nurse should hyperkalemia
. expect hypertonic glucose, insulin infusions, and sodium bicarbonate to be
used to treat:
57 The nurse is caring for a client who had a stroke. Which nursing Encouraging intake of at
. intervention promotes urinary continence? least 2 L of fluid daily
58 The nurse is caring for a client with acute pyelonephritis. Which nursing Increasing fluid intake to 3
. intervention is most important? L/day
59 The nurse is caring for a male client with gonorrhea. The client asks how wear a condom every time
. he can reduce his risk of contracting another sexually transmitted disease he has sexual intercourse.
(STD). The nurse should instruct the client to:
60 The nurse is caring for a patient with acute renal failure. Rank in (2) Initial insult, (3) Oliguric
. chronological order the phases of acute renal failure. Use all the options. Phase, (4) Diuretic Phase,
(1) recovery phase
61 The nurse is collecting data on a male client diagnosed with gonorrhea. Foul-smelling discharge
. Which symptom likely prompted the client to seek medical attention? from the penis
62 The nurse is completing an intake and output record for a client who is 600
. receiving continuous bladder irrigation after transurethral resection of the
prostate. How many milliliters of urine should the nurse record as output
for her shift if the client received 1,800 ml of normal saline irrigating
solution and the output in the urine drainage bag is 2,400 ml?
63 The nurse is inserting a urinary catheter into a client who is extremely breathe deeply.
. anxious about the procedure. The nurse can facilitate the insertion by
asking the client to:
64 The nurse is monitoring the fluid intake and output of a female client Maintaining a closed indwelling
. recovering from an exploratory laparotomy. Which nursing urinary catheter system and
intervention would help the client avoid a urinary tract infection (UTI)? securing the catheter to the leg
65 The nurse is planning a group teaching session on the topic of urinary Notify the physician if urinary
. tract infection (UTI) prevention. Which point would the nurse want to urgency, burning, frequency, or
include? difficulty occurs.
66 The nurse is planning to administer a sodium polystyrene sulfonate retain the enema for 30 minutes
. (Kayexalate) enema to a client with a potassium level of 5.9 mEq/L. to allow for sodium exchange;
Correct administration and the effects of this enema would include afterward, the client should have
having the client: diarrhea.
67 The nurse is providing postprocedure care for a client who underwent notify the physician about
. percutaneous lithotripsy. In this procedure, an ultrasonic probe inserted cloudy or foul-smelling urine.
through a nephrostomy tube into the renal pelvis generates ultra-high-
frequency sound waves to shatter renal calculi. The nurse should
instruct the client to:
68 The nurse is reviewing a client's fluid intake and output record. Fluid Fluid intake should be
. intake and urine output should relate in which way? approximately equal to the urine
output.
69 The nurse is reviewing the report of a client's routine urinalysis. Which Urine pH of 3.0
. value should the nurse consider abnormal?
70 The nurse is teaching a client with genital herpes. Education for this the importance of informing his
. client should include an explanation of: partner of the disease.
71 The nurse just received the shift report on her group of clients. Based A client who underwent a right
. on the information she received, which client should she assess first? nephrectomy yesterday and is
complaining of pain
72 A nurse-manager on the urology unit tells the staff that supplies have Compare charge slips for
. been disappearing at an alarming rate. A staff nurse has been assigned supplies used against the
to monitor supply use. Which method can best help the nurse monitor inventory left in the supply
supply use? room every 24 hours.
73 A nurse reviews a client's medical record and notes that a physician Verify the client's identity.
. ordered an indwelling urinary catheter due to client's urine retention.
Which action should the nurse perform first?
74 A nurse's neighbor complains of severe right flank pain. She explains Advise the neighbor to seek
. that it began during the night, but she was able to take acetaminophen medical attention.
(Tylenol) and return to bed. When she awoke, the pain increased in
intensity. How should the nurse intervene?
75 The nurse suspects that a client with a temperature of 103.6 F (39.8 C) Urinary tract infection (UTI)
. and an elevated white blood cell count is in the initial stage of sepsis.
What is the most common cause of sepsis in hospitalized clients?
76 A physician informs a client that her renal calculus is small enough that Encourage the client to consume
. she should be able to pass it without surgical intervention. Which action 3 to 4 liters of fluid a day.
should the nurse take to help the client pass the renal calculus?
77 The physician prescribes norfloxacin (Noroxin), 400 mg by mouth twice 7 to 10 days.
. daily, for a client with a urinary tract infection (UTI). The client asks
the nurse how long to continue taking the drug. For an uncomplicated
UTI, the usual duration of norfloxacin therapy is:
78 The red blood cell (RBC) production in a client with chronic renal fatigue and weakness.
. failure (CRF) has decreased. The nurse should monitor this client for:
79 (SELECT ALL THAT APPLY) The nurse is collecting data on a client (2) "I need to urinate
. who has a urinary tract infection (UTI). Which statements should the frequently.", (3) "It burns when I
nurse expect the client to make? urinate.", (5) "I need to urinate
urgently."
8 (SELECT ALL THAT APPLY) The nurse is teaching a (2), "Discard the first voiding and record the
0 client how to collect a 24-hour urine specimen for time.", (4) "Refrigerate the urine sample or keep it
. creatinine clearance. Which of the following directions on ice.", (5) "At the end of 24 hours, void and
should the nurse give the client? save the urine."
8 A stepfather brings a child with a fever to the emergency Report the suspected abuse according to facility
1 department. The child is crying, calling for her mother, policy.
. and attempting to get out of the stepfather's arms. Upon
inspection, the nurse notes that the child's underpants
are stained with a bloodlike substance. Which action
should the nurse take?
8 To treat a urinary tract infection (UTI), a client is Diarrhea
2 prescribed sulfamethoxazole (Gantanol), 2 g by mouth
. initially, and then 1 g by mouth three times daily. The
nurse should teach the client that sulfamethoxazole is
most likely to cause which adverse effect?
8 Two staff nurses on the urology unit are responsible for Poll the staff to find out their preferences.
3 the unit schedule. The holidays are nearing, and many
. staff members would like to take vacation days. Which
method might fairly solve the holiday staffing problem?
8 When a client returns from the operating room after assesses function of the remaining kidney.
4 undergoing a left nephrectomy, a nurse must make sure
. that urine is draining through the client's indwelling
urinary catheter. This assessment is important for this
client because it:
8 When a client with an indwelling urinary catheter insists When a client with an indwelling urinary catheter
5 on walking to the hospital lobby to visit with family insists on walking to the hospital lobby to visit
. members, the nurse teaches him how to do this without with family members, the nurse teaches him how
compromising the catheter. Which client action indicates to do this without compromising the catheter.
an accurate understanding of this information? Which client action indicates an accurate
understanding of this information?
1. After 1. Answer B. Reports of bladder spasms and the urge to void suggest that a blood clot
having may be occluding the catheter. After TURP, urine normally appears red to pink, and
transureth normal saline irrigant usually is infused at a rate of 40 to 60 drops/minute or according
ral to facility protocol. The amount of returned fluid (1,200 ml) should correspond to the
resection amount of instilled fluid, plus the client's urine output (1,000 ml + 200 ml), which
of the reflects catheter patency.
prostate
(TURP), a
Mr. Locke
returns to
the unit
with a
three-way
indwelling
urinary
catheter
and
continuous
closed
bladder
irrigation.
Which
finding
suggests
that the
client's
catheter is
occluded?
a. The
urine in
the
drainage
bag
appears
red to
pink.
b. The
client
reports
bladder
spasms
and the
urge to
void.
c. The
normal
saline
irrigant is
infusing at
a rate of 50
drops/min
ute.
d. About
1,000 ml of
irrigant
have been
instilled;
1,200 ml of
drainage
have been
returned.
2. 1. The nurse 1. Answer B. When the urethra is ruptured, a hematoma or collection of
is aware blood separates the two sections of urethra. This may feel like a boggy
that the mass on rectal examination. Because of the rupture and hematoma, the
following prostate becomes high riding. A palpable prostate gland usually
findings indicates a nonurethral injury. Absent sphincter tone would refer to a
would be spinal cord injury. The presence of blood would probably correlate
further with GI bleeding or a colon injury.
evidence of
a urethral
injury in a
male client
during
rectal
examination
?
a. A low-
riding
prostate
b. The
presence of
a boggy
mass
c. Absent
sphincter
tone
d. A positive
Hemoccult
3. 2. Nurse 2. Answer B. When inserting a urinary catheter, facilitate insertion by
Myrna is asking the client to breathe deeply. Doing this will relax the urinary
inserting a sphincter. Initiating a stream of urine isn't recommended during
urinary catheter insertion. Turning to the side or holding the labia or penis
catheter won't ease insertion, and doing so may contaminate the sterile field. ...
into a client
who is
extremely
anxious
about the
procedure.
The nurse
can
facilitate the
insertion by
asking the
client to:
a. initiate a
stream of
urine.
b. breathe
deeply.
c. turn to
the side.
d. hold the
labia or
shaft of
penis.
4. 2. When a 2. Answer B. To maintain effective drainage, the client should keep the
female drainage bag below the bladder; this allows the urine to flow by gravity
client with from the bladder to the drainage bag. The client shouldn't lay the
an drainage bag on the floor because it could become grossly
indwelling contaminated. The client shouldn't clamp the catheter drainage tubing
urinary because this impedes the flow of urine. To promote drainage, the client
(Foley) may loop the drainage tubing above not below its point of entry
catheter into the drainage bag.
insists on
walking to
the hospital
lobby to
visit with
family
members,
nurse Rose
teaches how
to do this
without
compromisi
ng the
catheter.
Which
client action
indicates an
accurate
understandi
ng of this
information
?
a. The client
sets the
drainage
bag on the
floor while
sitting
down.
b. The client
keeps the
drainage
bag below
the bladder
at all times.
c. The client
clamps the
catheter
drainage
tubing while
visiting with
the family.
d. The client
loops the
drainage
tubing
below its
point of
entry into
the
drainage
bag.
5 3. A female adult client 3. Answer D. ARF, characterized by abrupt loss of kidney function, commonly
. admitted with a gunshot causes oliguria, which is demonstrated by a urine output of 400 ml/24 hours. A
wound to the abdomen is serum creatinine level of 1.2 mg/dl isn't diagnostic of ARF. A BUN level of 22
transferred to the intensive mg/dl or a temperature of 100.2 F (37.8 C) wouldn't result from this
care unit after an exploratory disorder. ...
laparotomy. Which assessment
finding suggests that the client
is experiencing acute renal
failure (ARF)?
a. Blood urea nitrogen (BUN)
level of 22 mg/dl
b. Serum creatinine level of 1.2
mg/dl
c. Serum creatinine level of 1.2
mg/dl
d. Urine output of 400 ml/24
hours
6 3. A female client has just been 3. Answer A. Women with condylomata acuminata are at risk for cancer of the
. diagnosed with condylomata cervix and vulva. Yearly Pap smears are very important for early detection.
acuminata (genital warts). Because condylomata acuminata is a virus, there is no permanent cure.
What information is Because condylomata acuminata can occur on the vulva, a condom won't
appropriate to tell this client? protect sexual partners. HPV can be transmitted to other parts of the body,
a. This condition puts her at a such as the mouth, oropharynx, and larynx.
higher risk for cervical cancer;
therefore, she should have a
Papanicolaou (Pap) smear
annually.
b. The most common
treatment is metronidazole
(Flagyl), which should
eradicate the problem within 7
to 10 days.
c. The potential for
transmission to her sexual
partner will be eliminated if
condoms are used every time
they have sexual intercourse.
d. The human papillomavirus
(HPV), which causes
condylomata acuminata, can't
be transmitted during oral sex.
7 4. A 55-year old client with 4. Answer A. TURP is the most widely used procedure for prostate gland
. benign prostatic hyperplasia removal. Because it requires no incision, TURP is especially suitable for men
doesn't respond to medical with relatively minor prostatic enlargements and for those who are poor
treatment and is admitted to surgical risks. Suprapubic prostatectomy, retropubic prostatectomy, and
the facility for prostate gland transurethral laser incision of the prostate are less common procedures; they
removal. Before providing all require an incision. ...
preoperative and
postoperative instructions to
the client, nurse Gail asks the
surgeon which prostatectomy
procedure will be done. What
is the most widely used
procedure for prostate gland
removal?
a. Transurethral resection of
the prostate (TURP)
b. Suprapubic prostatectomy
c. Retropubic prostatectomy
d. Transurethral laser incision
of the prostate
8 4. A male client with bladder 4. Answer B. If the pouch faceplate doesn't fit the stoma properly, the skin
. cancer has had the bladder around the stoma will be exposed to continuous urine flow from the stoma,
removed and an ileal conduit causing excoriation and red, weeping, and painful skin. A lubricant shouldn't
created for urine diversion. be used because it would prevent the pouch from adhering to the skin. When
While changing this client's properly applied, a skin barrier prevents skin excoriation. Stoma dilation isn't
pouch, the nurse observes that performed with an ileal conduit, although it may be done with a colostomy if
the area around the stoma is ordered.
red, weeping, and painful.
What should nurse Katrina
conclude?
a. The skin wasn't lubricated
before the pouch was
applihttp://quizlet.com/create_
set/ed.
b. The pouch faceplate doesn't
fit the stoma.
c. A skin barrier was applied
properly.
d. Stoma dilation wasn't
performed.
9 5. A female client with 5. Answer B. Fluid depletion before excretory urography is contraindicated in
. suspected renal dysfunction is clients with multiple myeloma, severe diabetes mellitus, and uric acid
scheduled for excretory nephropathy conditions that can seriously compromise renal function in
urography. Nurse July reviews fluid-depleted clients with reduced renal perfusion. If these clients must
the history for conditions that undergo excretory urography, they should be well hydrated before the test.
may warrant changes in client Cystic fibrosis, gout, and myasthenia gravis don't necessitate changes in client
preparation. Normally, a client preparation for excretory urography. ...
should be mildly hypovolemic
(fluid depleted) before
excretory urography. Which
history finding would call for
the client to be well hydrated
instead?
a. Cystic fibrosis
b. Multiple myeloma
c. Gout
d. Myasthenia gravis
10 5. The nurse is aware that the following 5. Answer C. Pyelonephritis is diagnosed by the presence of
. laboratory values supports a diagnosis of leukocytosis, hematuria, pyuria, and bacteriuria. The client
pyelonephritis? exhibits fever, chills, and flank pain. Because there is often a
a. Myoglobinuria septic picture, the WBC count is more likely to be high rather
b. Ketonuria than low, as indicated in option D. Ketonuria indicates a
c. Pyuria diabetic state.
d. Low white blood cell (WBC) count
11. 6. A female client with chronic renal failure 6. Answer B. Because CRF causes loss of renal function, the
(CRF) is receiving a hemodialysis treatment. client with this disorder retains fluid. Hemodialysis removes
After hemodialysis, nurse Sarah knows that this fluid, causing weight loss. Hematuria is unlikely to
the client is most likely to experience: follow hemodialysis because the client with CRF usually
a. hematuria. forms little or no urine. Hemodialysis doesn't increase urine
b. weight loss. output because it doesn't correct the loss of kidney function,
c. increased urine output. which severely decreases urine production in this disorder.
d. increased blood pressure. By removing fluids, hemodialysis decreases rather than
increases the blood pressure.
12 6. Nurse Kim is caring for a client who had a 6. Answer A. By encouraging a daily fluid intake of at least 2
. cerebrovascular accident (CVA). Which L, the nurse helps fill the client's bladder, thereby promoting
nursing intervention promotes urinary bladder retraining by stimulating the urge to void. The nurse
continence? shouldn't give the client soda before bedtime; soda acts as a
a. Encouraging intake of at least 2 L of fluid diuretic and may make the client incontinent. The nurse
daily should take the client to the bathroom or offer the bedpan at
b. Giving the client a glass of soda before least every 2 hours throughout the day; twice per day is
bedtime insufficient. Consultation with a dietitian won't address the
c. Taking the client to the bathroom twice problem of urinary incontinence.
per day
d. Consulting with a dietitian
13 7. Nurse Lea is assessing a male client 7. Answer D. Symptoms of gonorrhea in men include
. diagnosed with gonorrhea. Which symptom purulent, foul-smelling drainage from the penis and painful
most likely prompted the client to seek urination. Rashes on the palms of the hands and soles of the
medical attention? feet are symptoms of the secondary stage of syphilis.
a. Rashes on the palms of the hands and Cauliflower-like warts on the penis are a sign of human
soles of the feet papillomavirus. Painful red papules on the shaft of the penis
b. Cauliflower-like warts on the penis may be a sign of the first stage of genital herpes.
c. Painful red papules on the shaft of the
penis
d. Foul-smelling discharge from the penis
14 7. When examining a female client's 7. Answer B. Percussion over the costovertebral angle
. genitourinary system, nurse Sally assesses normally produces a dull, thudding sound, which is soft to
for tenderness at the costovertebral angle by moderately loud with a moderate pitch and duration. This
placing the left hand over this area and sound occurs over less dense, mostly fluid-filled matter, such
striking it with the right fist. Normally, this as the kidneys, liver, and spleen. In contrast, a flat sound
percussion technique produces which sound? occurs over highly dense matter such as muscle;
a. A flat sound hyperresonance occurs over the air-filled, overinflated lungs
b. A dull sound of a client with pulmonary emphysema or the lungs of a child
c. Hyperresonance (because of a thin chest wall); and tympany occurs over
d. Tympany enclosed structures containing air, such as the stomach and
bowel.
15 8. A male client with chronic renal 8. Answer D. An elevated serum potassium level may lead to a
. failure has a serum potassium level of life-threatening cardiac arrhythmia, which the nurse can detect
6.8 mEq/L. What should nurse Olivia immediately by palpating the pulse. The client's blood pressure
assess first? may change, but only as a result of the arrhythmia. Therefore, the
a. Blood pressure nurse should assess blood pressure later. The nurse also can delay
b. Respirations assessing respirations and temperature because these aren't
c. Temperature affected by the serum potassium level.
d. Pulse
16 8. Nurse Agnes is reviewing the report of 8. Answer B. Normal urine pH is 4.5 to 8; therefore, a urine pH of
. a client's routine urinalysis. Which value 3.0 is abnormal. Urine specific gravity normally ranges from
should the nurse consider abnormal? 1.002 to 1.035, making this client's value normal. Normally, urine
a. Specific gravity of 1.03 contains no protein, glucose, ketones, bilirubin, bacteria, casts, or
b. Urine pH of 3.0 crystals. Red blood cells should measure 0 to 3 per high-power
c. Absence of protein field; white blood cells, 0 to 4 per high-power field. Urine should
d. Absence of glucose be clear, its color ranging from pale yellow to deep amber.
17 9. A male client is scheduled for a renal 9. Answer A. The renal clearance test determines the kidneys'
. clearance test. Nurse Maureen should ability to remove a substance from the plasma in 1 minute. It
explain that this test is done to assess the doesn't measure the kidneys' ability to remove a substance over a
kidneys' ability to remove a substance longer period.
from the plasma in:
a. 1 minute.
b. 30 minutes.
c. 1 hour.
d. 24 hours.
18 9. Nurse Hazel is ware that the following 9. Answer C. Infection can occur with renal calculi from urine
. is an appropriate nursing diagnosis for a stasis caused by obstruction. Options A and D aren't appropriate
client with renal calculi? for this diagnosis, and retention of urine usually occurs, rather
a. Ineffective tissue perfusion than incontinence.
b. Functional urinary incontinence
c. Risk for infection
d. Decreased cardiac output
19 10. A male client develops acute renal 10. Answer A. As urine output decreases, the serum potassium
. failure (ARF) after receiving I.V. level rises; if it rises sufficiently, hyperkalemia may occur,
therapy with a nephrotoxic antibiotic. possibly triggering a cardiac arrhythmia. Hyperkalemia doesn't
Because the client's 24-hour urine cause paresthesia (sensations of numbness and tingling).
output totals 240 ml, nurse Andy Dehydration doesn't occur during this oliguric phase of ARF,
suspects that the client is at risk for: although typically it does arise during the diuretic phase. In a
a. cardiac arrhythmia. client with ARF, pruritus results from increased phosphates and
b. paresthesia. isn't associated with hyperkalemia.
c. dehydration.
d. pruritus.
20 10. A male client in the short-procedure 10. Answer C. After renal angiography involving a femoral
. unit is recovering from renal angiography puncture site, the nurse should check the client's pedal pulses
in which a femoral puncture site was used. frequently to detect reduced circulation to the feet caused by
When providing postprocedure care, the vascular injury. The nurse also should monitor vital signs for
nurse should: evidence of internal hemorrhage and should observe the
a. keep the client's knee on the affected side puncture site frequently for fresh bleeding. The client should
bent for 6 hours. be kept on bed rest for several hours so the puncture site can
b. apply pressure to the puncture site for 30 seal completely. Keeping the client's knee bent is unnecessary.
minutes. By the time the client returns to the short-procedure unit,
c. check the client's pedal pulses frequently. manual pressure over the puncture site is no longer needed
d. remove the dressing on the puncture site because a pressure dressing is in place. The nurse shouldn't
after vital signs stabilize. remove this dressing for several hours and only if
instructed to do so.
21 11. A female client is admitted for 11. Answer A. A client with CRF is at risk for fluid imbalance
. treatment of chronic renal failure (CRF). dehydration if the kidneys fail to concentrate urine, or fluid
Nurse Juliet knows that this disorder retention if the kidneys fail to produce urine. Electrolyte
increases the client's risk of: imbalances associated with this disorder result from the
a. water and sodium retention secondary to kidneys' inability to excrete phosphorus; such imbalances may
a severe decrease in the glomerular lead to hyperphosphatemia with reciprocal hypocalcemia.
filtration rate. CRF may cause metabolic acidosis, not metabolic alkalosis,
b. a decreased serum phosphate level secondary to inability of the kidneys to excrete hydrogen ions.
secondary to kidney failure.
c. an increased serum calcium level
secondary to kidney failure.
d. metabolic alkalosis secondary to
retention of hydrogen ions.
22 11. After undergoing transurethral 11. Answer C. Although postoperative pain is expected, the
. resection of the prostate to treat benign nurse should make sure that other factors, such as an
prostatic hyperplasia, a male client returns obstructed irrigation catheter, aren't the cause of the pain. After
to the room with continuous bladder assessing catheter patency, the nurse should administer an
irrigation. On the first day after surgery, analgesic, such as meperidine, as prescribed. Increasing the
the client reports bladder pain. What I.V. flow rate may worsen the pain. Notifying the physician
should nurse Andrew do first? isn't necessary unless the pain is severe or unrelieved by the
a. Increase the I.V. flow rate. prescribed medication.
b. Notify the physician immediately.
c. Assess the irrigation catheter for patency
and drainage.
d. Administer meperidine (Demerol), 50 mg
I.M., as prescribed.
23 12. Because of difficulties with 12. Answer D. An increased WBC count indicates infection,
. hemodialysis, peritoneal dialysis is probably resulting from peritonitis, which may have been caused by
initiated to treat a female client's insertion of the peritoneal catheter into the peritoneal cavity.
uremia. Which finding signals a Peritonitis can cause the peritoneal membrane to lose its ability to
significant problem during this filter solutes; therefore, peritoneal dialysis would no longer be a
procedure? treatment option for this client. Hyperglycemia occurs during
a. Potassium level of 3.5 mEq/L peritoneal dialysis because of the high glucose content of the
b. Hematocrit (HCT) of 35% dialysate; it's readily treatable with sliding-scale insulin. A
c. Blood glucose level of 200 mg/dl potassium level of 3.5 mEq/L can be treated by adding potassium to
d. White blood cell (WBC) count of the dialysate solution. An HCT of 35% is lower than normal.
20,000/mm3 However, in this client, the value isn't abnormally low because of the
daily blood samplings. A lower HCT is common in clients with
chronic renal failure because of the lack of erythropoietin.
24 12. When performing a scrotal 12. Answer D. A nurse who discovers a nodule, swelling, or other
. examination, nurse Paul finds a abnormal finding during a scrotal examination should
nodule. What should the nurse do transilluminate the scrotum by darkening the room and shining a
next? flashlight through the scrotum behind the mass. A scrotum filled
a. Notify the physician. with serous fluid transilluminates as a red glow; a more solid lesion,
b. Change the client's position and such as a hematoma or mass, doesn't transilluminate and may appear
repeat the examination. as a dark shadow. Although the nurse should notify the physician of
c. Perform a rectal examination. the abnormal finding, performing transillumination first provides
d. Transilluminate the scrotum. additional information. The nurse can't uncover more information
about a scrotal mass by changing the client's position and repeating
the examination or by performing a rectal examination.
25 13. A male client who has been 13. Answer C. In a client with CRF, unrestricted intake of sodium,
. treated for chronic renal failure protein, potassium, and fluid may lead to a dangerous accumulation
(CRF) is ready for discharge. Nurse of electrolytes and protein metabolic products, such as amino acids
Bea should reinforce which dietary and ammonia. Therefore, the client must limit intake of sodium;
instruction? meat, which is high in protein; bananas, which are high in
a. "Be sure to eat meat at every potassium; and fluid, because the failing kidneys can't secrete
meal." adequate urine. Salt substitutes are high in potassium and should be
b. "Monitor your fruit intake, and eat avoided. Extra carbohydrates are needed to prevent protein
plenty of bananas." catabolism.
c. "Increase your carbohydrate
intake."
d. "Drink plenty of fluids, and use a
salt substitute."
26 13. For a male client in the oliguric phase of 13. Answer C. During the oliguric phase of ARF, urine
. acute renal failure (ARF), which nursing output decreases markedly, possibly leading to fluid
intervention is most important? overload. Limiting oral and I.V. fluid intake can prevent
a. Encouraging coughing and deep breathing fluid overload and its complications, such as heart failure
b. Promoting carbohydrate intake and pulmonary edema. Encouraging coughing and deep
c. Limiting fluid intake breathing is important for clients with various respiratory
d. Providing pain-relief measures disorders. Promoting carbohydrate intake may be helpful in
ARF but doesn't take precedence over fluid limitation.
Controlling pain isn't important because ARF rarely causes
pain.
27 14. A female client requires hemodialysis. 14. Answer D. Cardiac glycosides such as digoxin should be
. Which of the following drugs should be withheld before hemodialysis. Hypokalemia is one of the
withheld before this procedure? electrolyte shifts that occur during dialysis, and a
a. Phosphate binders hypokalemic client is at risk for arrhythmias secondary to
b. Insulin digitalis toxicity. Phosphate binders and insulin can be
c. Antibiotics administered because they aren't removed from the blood by
d. Cardiac glycosides dialysis. Some antibiotics are removed by dialysis and
should be administered after the procedure to ensure their
therapeutic effects. The nurse should check a formulary to
determine whether a particular antibiotic should be
administered before or after dialysis.
28 14. Nurse Wayne is aware that the following 14. Answer B. Urinary incontinence isn't a normal part of
. statements describing urinary incontinence aging nor is it a disease. It may be caused by confusion,
in the elderly is true? dehydration, fecal impaction, restricted mobility, or other
a. Urinary incontinence is a normal part of causes. Certain medications, including diuretics, hypnotics,
aging. sedatives, anticholinergics, and antihypertensives, may
b. Urinary incontinence isn't a disease. trigger urinary incontinence. Most clients with urinary
c. Urinary incontinence in the elderly can't incontinence can be treated; some can be cured.
be treated.
d. Urinary incontinence is a disease.
29 15. A client comes to the outpatient 15. Answer B. Gonorrhea must be reported to the public
. department complaining of vaginal health department. Chlamydia, genital herpes, and human
discharge, dysuria, and genital irritation. papillomavirus infection aren't reportable diseases.
Suspecting a sexually transmitted disease
(STD), Dr. Smith orders diagnostic tests of
the vaginal discharge. Which STD must be
reported to the public health department?
a. Chlamydia
b. Gonorrhea
c. Genital herpes
d. Human papillomavirus infection
30 15. The client underwent a transurethral 15. Answer D. If the catheter is blocked by blood clots, it
. resection of the prostate gland 24 hours ago may be irrigated according to physician's orders or facility
and is on continuous bladder irrigation. protocol. The nurse should use sterile technique to reduce
Nurse Yoly is aware that the following the risk of infection. Urinating around the catheter can cause
nursing interventions is appropriate? painful bladder spasms. Encourage the client to drink fluids
a. Tell the client to try to urinate around the to dilute the urine and maintain urine output. The catheter
catheter to remove blood clots. remains in place for 2 to 4 days after surgery and is only
b. Restrict fluids to prevent the client's removed with a physician's order.
bladder from becoming distended.
c. Prepare to remove the catheter.
d. Use aseptic technique when irrigating the
catheter.
31 16. A female client with a urinary tract infection is 16. Answer B. When receiving a sulfonamide such as
. prescribed co-trimoxazole (trimethoprim- co-trimoxazole, the client should drink at least eight
sulfamethoxazole). Nurse Don should provide which 8-oz glasses of fluid daily to maintain a urine output
medication instruction? of at least 1,500 ml/day. Otherwise, inadequate urine
a. "Take the medication with food." output may lead to crystalluria or tubular deposits.
b. "Drink at least eight 8-oz glasses of fluid daily." For maximum absorption, the client should take this
c. "Avoid taking antacids during co-trimoxazole drug at least 1 hour before or 2 hours after meals. No
therapy." evidence indicates that antacids interfere with the
d. "Don't be afraid to go out in the sun." effects of sulfonamides. To prevent a photosensitivity
reaction, the client should avoid direct sunlight
during co-trimoxazole therapy.
32 16. A male client with acute pyelonephritis receives 16. Answer C. Co-trimoxazole is a sulfonamide
. a prescription for co-trimoxazole (Septra) P.O. twice antibiotic used to treat urinary tract infections.
daily for 10 days. Which finding best demonstrates Therefore, absence of bacteria on urine culture
that the client has followed the prescribed regimen? indicates that the drug has achieved its desired effect.
a. Urine output increases to 2,000 ml/day. Although flank pain may decrease as the infection
b. Flank and abdominal discomfort decrease. resolves, this isn't a reliable indicator of the drug's
c. Bacteria are absent on urine culture. effectiveness. Co-trimoxazole doesn't affect urine
d. The red blood cell (RBC) count is normal. output or the RBC count.
33 17. A 26-year-old female client seeks care for a 17. Answer D. Phenazopyridine may be prescribed in
. possible infection. Her symptoms include burning conjunction with an antibiotic for painful bladder
on urination and frequent, urgent voiding of small infections to promote comfort. Because of its local
amounts of urine. She's placed on trimethoprim- anesthetic action on the urinary mucosa,
sulfamethoxazole (Bactrim) to treat possible phenazopyridine specifically relieves bladder pain.
infection. Another medication is prescribed to Nitrofurantoin is a urinary antiseptic with no
decrease the pain and frequency. Which of the analgesic properties. While ibuprofen and
following is the most likely medication prescribed? acetaminophen with codeine are analgesics, they
a. nitrofurantoin (Macrodantin) don't exert a direct effect on the urinary mucosa.
b. ibuprofen (Motrin)
c. acetaminophen with codeine
d. phenazopyridine (Pyridium)
34 17. A male client is admitted for treatment of 17. Answer A. Generalized edema, especially of the
. glomerulonephritis. On initial assessment, Nurse face and periorbital area, is a classic sign of acute
Rose detects one of the classic signs of acute glomerulonephritis of sudden onset. Other classic
glomerulonephritis of sudden onset. Such signs signs and symptoms of this disorder include
include: hematuria (not green-tinged urine), proteinuria, fever,
a. generalized edema, especially of the face and chills, weakness, pallor, anorexia, nausea, and
periorbital area. vomiting. The client also may have moderate to
b. green-tinged urine. severe hypertension (not hypotension), oliguria or
c. moderate to severe hypotension. anuria (not polyuria), headache, reduced visual
d. polyuria. acuity, and abdominal or flank pain.
35 18. A client reports experiencing vulvar pruritus. 18. Answer A. The symptoms of C. albicans include
. Which assessment factor may indicate that the itching and a scant white discharge that has the
client has an infection caused by Candida albicans? consistency of cottage cheese. Yellow-green
a. Cottage cheese-like discharge discharge is a sign of Trichomonas vaginalis. Gray-
b. Yellow-green discharge white discharge and a fishy odor are signs of
c. Gray-white discharge Gardnerella vaginalis.
d. Discharge with a fishy odor
36 18. A triple-lumen indwelling urinary 18. Answer A. When preparing for continuous bladder
. catheter is inserted for continuous bladder irrigation, a triple-lumen indwelling urinary catheter is
irrigation following a transurethral resection inserted. The three lumens provide for balloon inflation and
of the prostate. In addition to balloon continuous inflow and outflow of irrigation solution.
inflation, the nurse is aware that the
functions of the three lumens include:
a. Continuous inflow and outflow of
irrigation solution.
b. Intermittent inflow and continuous
outflow of irrigation solution.
c. Continuous inflow and intermittent
outflow of irrigation solution.
d. Intermittent flow of irrigation solution
and prevention of hemorrhage.
37 19. A 24-year old female client has just been 19. Answer A. Women with condylomata acuminata are at
. diagnosed with condylomata acuminata risk for cancer of the cervix and vulva. Yearly Pap smears are
(genital warts). What information is very important for early detection. Because condylomata
appropriate to tell this client? acuminata is a virus, there is no permanent cure. Because
a. This condition puts her at a higher risk for condylomata acuminata can occur on the vulva, a condom
cervical cancer; therefore, she should have a won't protect sexual partners. HPV can be transmitted to
Papanicolaou (Pap) smear annually. other parts of the body, such as the mouth, oropharynx, and
b. The most common treatment is larynx.
metronidazole (Flagyl), which should
eradicate the problem within 7 to 10 days.
c. The potential for transmission to her
sexual partner will be eliminated if condoms
are used every time they have sexual
intercourse.
d. The human papillomavirus (HPV), which
causes condylomata acuminata, can't be
transmitted during oral sex.
38 19. Nurse Claudine is reviewing a client's 19. Answer B. Normally, fluid intake is approximately equal
. fluid intake and output record. Fluid intake to the urine output. Any other relationship signals an
and urine output should relate in which abnormality. For example, fluid intake that is double the
way? urine output indicates fluid retention; fluid intake that is half
a. Fluid intake should be double the urine the urine output indicates dehydration. Normally, fluid intake
output. isn't inversely proportional to the urine output.
b. Fluid intake should be approximately
equal to the urine output.
c. Fluid intake should be half the urine
output.
d. Fluid intake should be inversely
proportional to the urine output.
39 20. After trying to conceive for a year, a 20. Answer C. Mumps is the most significant childhood
. couple consults an infertility specialist. When infectious disease affecting male fertility. Chickenpox,
obtaining a history from the husband, nurse measles, and scarlet fever don't affect male fertility.
Jenny inquires about childhood infectious
diseases. Which childhood infectious disease
most significantly affects male fertility?
a. Chickenpox
b. Measles
c. Mumps
d. Scarlet fever
40 20. Nurse Vic is monitoring the fluid intake and 19. Answer A. Women with condylomata acuminata
. output of a female client recovering from an are at risk for cancer of the cervix and vulva. Yearly
exploratory laparotomy. Which nursing Pap smears are very important for early detection.
intervention would help the client avoid a urinary Because condylomata acuminata is a virus, there is
tract infection (UTI)? no permanent cure. Because condylomata acuminata
a. Maintaining a closed indwelling urinary catheter can occur on the vulva, a condom won't protect
system and securing the catheter to the leg sexual partners. HPV can be transmitted to other
b. Limiting fluid intake to 1 L/day parts of the body, such as the mouth, oropharynx, and
c. Encouraging the client to use a feminine larynx.
deodorant after bathing
d. Encouraging the client to douche once a day after
removal of the indwelling urinary catheter
41 21. A male client comes to the emergency 21. Answer A. The most common site of renal calculi
. department complaining of sudden onset of sharp, formation is the kidney. Calculi may travel down the
severe pain in the lumbar region, which radiates urinary tract with or without causing damage and
around the side and toward the bladder. The client may lodge anywhere along the tract or may stay
also reports nausea and vomiting and appears pale, within the kidney. The ureter, bladder, and urethra are
diaphoretic, and anxious. The physician tentatively less common sites of renal calculi formation.
diagnoses renal calculi and orders flat-plate
abdominal X-rays. Renal calculi can form anywhere
in the urinary tract. What is their most common
formation site?
a. Kidney
b. Ureter
c. Bladder
d. Urethra
42 21. Nurse Eve is caring for a client who had a 21. Answer A. By encouraging a daily fluid intake of
. cerebrovascular accident (CVA). Which nursing at least 2 L, the nurse helps fill the client's bladder,
intervention promotes urinary continence? thereby promoting bladder retraining by stimulating
a. Encouraging intake of at least 2 L of fluid daily the urge to void. The nurse shouldn't give the client
b. Giving the client a glass of soda before bedtime soda before bedtime; soda acts as a diuretic and may
c. Taking the client to the bathroom twice per day make the client incontinent. The nurse should take
d. Consulting with a dietitian the client to the bathroom or offer the bedpan at least
every 2 hours throughout the day; twice per day is
insufficient. Consultation with a dietitian won't
address the problem of urinary incontinence.
43 22. A female client with acute renal failure is 22. Answer A. Dialysis equilibrium syndrome causes
. undergoing dialysis for the first time. The nurse in confusion, a decreasing level of consciousness,
charge monitors the client closely for dialysis headache, and seizures. These findings, which may
equilibrium syndrome, a complication that is most last several days, probably result from a relative
common during the first few dialysis sessions. excess of interstitial or intracellular solutes caused by
Typically, dialysis equilibrium syndrome causes: rapid solute removal from the blood. The resultant
a. confusion, headache, and seizures. organ swelling interferes with normal physiologic
b. acute bone pain and confusion. functions. To prevent this syndrome, many dialysis
c. weakness, tingling, and cardiac arrhythmias. centers keep first-time sessions short and use a
d. hypotension, tachycardia, and tachypnea. reduced blood flow rate. Acute bone pain and
confusion are associated with aluminum intoxication,
another potential complication of dialysis. Weakness,
tingling, and cardiac arrhythmias suggest
hyperkalemia, which is associated with renal failure.
Hypotension, tachycardia, and tachypnea signal
hemorrhage, another dialysis complication.
44 22. A female client with an indwelling 22. Answer B. Most catheters have a self-sealing port for
. urinary catheter is suspected of having a obtaining a urine specimen. Antiseptic solution is used to
urinary tract infection. Nurse Angel should reduce the risk of introducing microorganisms into the
collect a urine specimen for culture and catheter. Tubing shouldn't be disconnected from the urinary
sensitivity by: catheter. Any break in the closed urine drainage system may
a. disconnecting the tubing from the urinary allow the entry of microorganisms. Urine in urine drainage
catheter and letting the urine flow into a bags may not be fresh and may contain bacteria, giving false
sterile container. test results. When there is no urine in the tubing, the catheter
b. wiping the self-sealing aspiration port may be clamped for no more than 30 minutes to allow urine
with antiseptic solution and aspirating urine to collect.
with a sterile needle.
c. draining urine from the drainage bag into
a sterile container.
d. clamping the tubing for 60 minutes and
inserting a sterile needle into the tubing
above the clamp to aspirate urine.
45 23. Dr. Marquez prescribes norfloxacin 23. Answer B. For an uncomplicated UTI, norfloxacin
. (Noroxin), 400 mg P.O. twice daily, for a therapy usually lasts 7 to 10 days. Taking the drug for less
client with a urinary tract infection (UTI). than 7 days wouldn't eradicate such an infection. Taking it for
The client asks the nurse how long to more than 10 days isn't necessary. Only a client with a
continue taking the drug. For an complicated UTI must take norfloxacin for 10 to 21 days.
uncomplicated UTI, the usual duration of
norfloxacin therapy is:
a. 3 to 5 days.
b. 7 to 10 days.
c. 12 to 14 days.
d. 10 to 21 days.
46 23. Nurse Grace is assessing a male client 23. Answer D. Symptoms of gonorrhea in men include
. diagnosed with gonorrhea. Which symptom purulent, foul-smelling drainage from the penis and painful
most likely prompted the client to seek urination. Rashes on the palms of the hands and soles of the
medical attention? feet are symptoms of the secondary stage of syphilis.
a. Rashes on the palms of the hands and Cauliflower-like warts on the penis are a sign of human
soles of the feet papillomavirus. Painful red papules on the shaft of the penis
b. Cauliflower-like warts on the penis may be a sign of the first stage of genital herpes.
c. Painful red papules on the shaft of the
penis
d. Foul-smelling discharge from the penis
47 24. Nurse Ethel is planning to administer a 24. Answer A. Kayexalate is a sodium exchange resin. Thus
. sodium polystyrene sulfonate (Kayexalate) the client will gain sodium as potassium is lost in the bowel.
enema to a client with a potassium level of For the exchange to occur, Kayexalate must be in contact with
5.9 mEq/L. Correct administration and the the bowel for at least 30 minutes. Sorbitol in the Kayexalate
effects of this enema would include having enema causes diarrhea, which increases potassium loss and
the client: decreases the potential for Kayexalate retention.
a. retain the enema for 30 minutes to allow
for sodium exchange; afterward, the client
should have diarrhea.
b. retain the enema for 30 minutes to allow
for glucose exchange; afterward, the client
should have diarrhea.
c. retain the enema for 60 minutes to allow
for sodium exchange; diarrhea isn't
necessary to reduce the potassium level.
d. retain the enema for 60 minutes to allow
for glucose exchange; diarrhea isn't
necessary to reduce the potassium level.
48 24. Nurse Joy is providing postprocedure care 24. Answer C. The client should report the presence of
. for a client who underwent percutaneous foul-smelling or cloudy urine. Unless contraindicated, the
lithotripsy. In this procedure, an ultrasonic client should be instructed to drink large quantities of
probe inserted through a nephrostomy tube fluid each day to flush the kidneys. Sandlike debris is
into the renal pelvis generates ultra-high- normal due to residual stone products. Hematuria is
frequency sound waves to shatter renal calculi. common after lithotripsy.
The nurse should instruct the client to:
a. limit oral fluid intake for 1 to 2 weeks.
b. report the presence of fine, sandlike particles
through the nephrostomy tube.
c. notify the physician about cloudy or foul-
smelling urine.
d. report bright pink urine within 24 hours
after the procedure.
49 25. A client is frustrated and embarrassed by 25. Answer D. The guidelines for initiating bladder
. urinary incontinence. Which of the following retraining include assessing the client's intake patterns,
measures should nurse Bea include in a bladder voiding patterns, and reasons for each accidental voiding.
retraining program? Lowering the client's fluid intake won't reduce or prevent
a. Establishing a predetermined fluid intake incontinence. The client should actually be encouraged to
pattern for the client drink 1.5 to 2 L of water per day. A voiding schedule
b. Encouraging the client to increase the time should be established after assessment.
between voidings
c. Restricting fluid intake to reduce the need to
void
d. Assessing present elimination patterns
50 25. When caring for a male client with acute 25. Answer A. Because acetaminophen is metabolized in
. renal failure (ARF), Nurse Fatima expects to the liver, its dosage and dosing schedule need not be
adjust the dosage or dosing schedule of certain adjusted for a client with ARF. In contrast, the dosages
drugs. Which of the following drugs would not and schedules for gentamicin and ticarcillin, which are
require such adjustment? metabolized and excreted by the kidney, should be
a. acetaminophen (Tylenol) adjusted. Because cyclosporine may cause nephrotoxicity,
b. gentamicin sulfate (Garamycin) the nurse must monitor both the dosage and blood drug
c. cyclosporine (Sandimmune) level in a client receiving this drug.
d. ticarcillin disodium (Ticar)
51 ACUTE RENAL - is a syndrome in which there is an abrupt loss of renal function that may occur over
. FAILURE several hours or days, characterized by Uremia. The most common cause of Acute Renal
Function is hypotension and hypovolemia.

HYPERKALEMIA is a common complication of acute renal failure. It is life threatening if


not reversed. The administration of glucose and regular insulin can prevent cardiac arrest
caused by hypekalemia.

A sudden but reversible loss of kidney function.


Acute Renal Failure

IT IS BASICALLY A RAPID DECREASE IN RENAL FUNCTION.

CAUSES A COLLECTION OF METABOLIC WASTE IN THE BODY

Types of Acute Renal Failure

PRERENAL
INTRARENAL
POST RENAL

Phases of Acute Renal Failure


ONSET PHASE
OLIGURIC PHASE
DIURETIC PHASE

- the patient usually has diuresis of up to 10 L/day.

RECOVERY PHASE

Laboratory Tests:

BUN
Creatinine clearance
Serum creatinine
Serum potassium
Urinalysis

Treatment

DIET: High carbohydrates and low in protien, salt, and potassium.


ANTIBIOTICS

*** limit sodium, potassium, and phosphorus intake.

Dialysis may be necessary.


52 BUN level 10-20 mg/dl
.

53 Calcium 9- 11 mg/ dl
.

54 CHRONIC RENAL STAGES OF CHRONIC RENAL FAILURE


. FAILURE
Stage 1 Diminished Renal Reserve

- renal function is reduced, but no accumulation of metablic wastes occurs.


- the healthier kidney compensates for the diseased kidney
- The ability to concentrate urine is decreased
Results in nocturia and polyuria

- Stage 2 Renal Insufficiency

- metabolic waste begin to accumulate in the blood, because affected nephrons


can no
longer compensate.
- responsiveness to diuretics is decreased, resulting in Oliguria and edema
Stage 3 End Stage Renal Disease.

- excessive amount of metabolic wastes such as


urea and creatinine accumulate in the blood.
- kidney is unable to maintain homeostasis
- treatment is by dialysis

Metabolic Changes
- Urea and Creatinine
- Sodium
- Pottasium
- Acid Base Balance
- Calcium and Phosporus

Cardiac Changes
- Hypertension
- Hyperlipidemia
- Heart Failure
- Uremic Pericarditis

INTERVENTIONS
- It is important to monitor renal, respiratory and cardiovascular status and the
fluid balance.

REMEMBER FOR NCLEX


- Patient with Chronic Renal Failure would have

UREMIA, ANEMIA AND ACIDOSIS

DIALYSIS:

Peritoneal Dialysis:

Complication: can include Peritonitis

Hemodialysis:

Pt. may use external shunt or surgically constructed internal arterivnous


fistula (long-term)
55 Creatinine level 0.5- 1.5 mg/dl
.

56 Glomerulonephritis A NON BACTERIAL INFLAMMATION OF THE KIDNEY'S


. GLUMEROLUS.
Usually a result of an antigen antibody response to a beta hemolytic
streptococci
(this means that the patient usually had strep throat 2-3 weeks prior).

SERUM CREATININE LEVEL


BLOOD UREA NITROGEN BUN

ASSESSMENT
- Monitor the patient for dark color, cloudy appearance and foul odor urine.
- WBC blood count will be above 10,000/ mm3
- Blood culture will be positive for presence of bacteria.
- Azotemia= presence of nitrogenous waste products in the blood.

Wilms Tumor (Nephroblastoma)


- a common intraabdominal tumors of the childhood.
- usually associated with genitourinary anomalies.

Assessment:
- patient will have a swelling or mass within the abdomen.
- the patient will have hypertension, hematuria, pallor and anorexia

NEVER AND DO NOT palpate the abdomen of the patient.

Urine Culture and Sensitiviy

CREATININE CLEARANCE TEST

CREATININE CLEARANCE TEST, EVALUATES HOW WELL THE


KIDNEYS REMOVE CREATININE
BLOOD IS DRAWN AFTER TAKING THE URINE SAMPLE.

Bladder Ultrasonography
Intravenous Pyelography

AN X RAY PROCEDURE
USES A RADIOPAQUE DYE
ASSESS FOR ALLERGIES

Renal Angiography
- an IV injection of radiopaque dye is inserted into the renal artery

- INJECTION OF A RADIOPAQUE DYE


- OBTAIN INFORMED CONSENT.
- ASSESS FOR ALLERGIES
- WITHOLD FOOD AFTER MIDNIGHT
- INFORM PT. ABOUT THROAT IRRITATION

Uric Acid Test


URIC ACID TEST IS A 24 HR. URINE COLLECTION TO DIAGNOSE
FOR GOUT.

Interventions:

Manage Pain- (give analgesics)


Administer Antibiotics
Increase the patients's fluid intake (2-3 L/day)
57 Hypercalcemia (increased Renal calculi, coma, arrythmias, decreased reflex
. calcium)
58 Hyperkalemia Irritability, nausea and vomiting
. (increased potassium) diarrhea
59 Hypernatremia Tachycardia, dry mucus membrane
. (increased sodium) decreased urinary output
60 Hypokalcemia osteoporosis, fractures, muscle spasms
. (decreased calcium) tetany, n & v, vomiting.
61 Hypokalemia (decreased Anorexia, nausea, vomiting
. potassium) abdominal distention
62 Hyponatremia Muscle weaknes, Headaches
. (decreased sodium) Fatigue, confusion, vomiting, coma
63 NEPHROTIC - There is an increased glomerular permeability allowing larger molecules to pass
. SYNDROME through the membrane into the urine.

Main assessment in patients is Severe Protienuria (>3.5 G OF PROTIEN IN 24


HOURS)

HYPERLIPIDEMA
EDEMA
HYPERTENSION

HYPOALBUMINEMIA

- a condition of increased glomerular permeability that allows larger molecules to


pass through the membrane into the urine and be removed from the blood.
- This process causes massive loss of protein into the urine, edema formation and
decreased
plasma albumin levels. Many agents and disorders are possible cause of NS.

PATIENT ASSESSMENT:
- main feauture is severe protienuria (>3.5 grams of protien in 24 hours)
- Clients also have hypoalbuminemia (serum albumin <3 g/dl)
- hyperlipidemia, lipiduria, edema and hypertension

KEY FEAUTURES
- massive protienuria (protien in urine)
- hypoalbuminemia
- edema
- lipiduria
- hyperlipidemia

TREATMENT
- treatment varies depending upon what process is causing the disorder.
- Immunologic processes may improve with suppressive therapy using steroids
and catatonic agents.

PROVIDE BED REST


DIET: MODERATE PROTIEN, LOW SODIUM LOW SATURATED FAT.

MEDICATIONS:
- Ace Inhibitors= to decrease protienuria
- Cholesterol lowering agents- decrease hyperlipidemia
- Diuretics- to control edema
64 POLYCYSTIC - A cyst develops in the nephron (kidneys)
. KIDNEY DISEASE - It is an inherited disease of the kidneys.
-IN THE DOMINANT FORM- ONLY FEW NEPHRONS HAVE CYSTS.
-IN THE RECESSIVE FORM-100% OF NEPHRONS HAVE CYSTS FROM
BIRTH.
- Cysts in kidneys would look like a cluster of grapes
- Patients would have hypertension. (Due to Renin Angiotensin System)
MANIFESTATIONS IN THE PATIENT:
-PAIN IS THE FIRST MANIFESTATION
-FLANK PAIN IS DULL, SHARP OR INTERMITTENT
-DULL ACHING PAIN- is caused by increase kidney size, from infection from the
cyst.
-SHARP INTERMITTENT PAIN- is when a cyst is rupture or when a stone is
present.
-BERRY ANEURYSM
- can occur (bleeding into brain from ruptured intracranial vascular cysts)
- causes severe headaches, with or without vision changes (pay extra attention
to patients with severe headaches, since it can be a sign of a ruptured cyst).
DIAGNOSTIC TESTS:
- renal sonogrophy
- computed tomography
- MRI
INTERVENTIONS:
-BE VERY CAUTIOUS IN USING NSAIDS, BECAUSE IT CAN CAUSE
BLEEDING
-ANTIHYPERTENSIVES AND DIURETIC AGENTS (ACE INHIBITOR,
CALCIUM CHANNEL BLOCKER, BETA BLOCKER)IS USED FOR
HYPERTENSION.
65 Pyelonephritis AN INFECTIOUS DISEASE CAUSING INFLAMMATION OF THE KIDNEY ORGAN
. AND TISSUE.
E. Coli- has been the main organism that causes most cases

Symptoms:

- Chills
- Flank Pain
- Fever
- Fatigue
- Fever

Treatment:

Antibiotics

Commonly used antibiotics include the following:

Amoxicillin
Cephalosporin

- Cephalosporins are very similar to penicillins.

Most commonly used Cephalosporins include:


Keflex, Ceprozil (Cefzil), Cefadroxil (Duricef)
Most Cephalosporins starts with a (cef or cep)*

Other Antibiotics includes:

Levofloxacin and ciprofloxacin


Sulfa drugs such as sulfisoxazole/trimethoprim

Levofloxacin- a type of fluoroquinolones

Side Effects (Levofloxacin):

dizziness
sudden pain on joints
diarrhea
confusion
hallucinations
66 RENAL CELL - A cancer that originates in the kidneys.
. CARCINOMA - Children will develop a kind of kidney cancer called Wilms' tumor.

Assessment:
- Pink, red or cola colored urine
- Weight loss
- Fatigue
- Intermittent fever

Risks
- Sex: Male
- Smoking
- Obesity
- Hypertension

Tests and procedures:


- Urineanalysis
- computerized tomography (CT)
- magnetic resonance imaging (MRI)
- Kidney Biopsy (main diagnostic tests)

Treatments:
Surgery
- Removing the affected kidney (nephrectomy)

Chemotherapy

Blocking blood flow to the tumor (embolization).


Treatment to freeze cancer cells (cryoablation).
Biological therapy (immunotherapy) uses your body's immune system to fight
cancer.

Manage Pain with Narcotic Analgesics or NSAID's.


67 RENAL Two Types of Dialysis:
. DIALYSIS - Hemodialysis
- Peritoneal Dialysis

Continous Renal Replacement Therapy (CRRT)


This type of therapy is an alternative to other types of dialysis.
More focused on treating acute renal failure.

HEMODIALYSIS

- Used for Renal Failure


- Toxic wastes are removed from the blood through surgically created access site.
- Removes excess fluids and waste products and restores Electrolye balance.
- The blood through an artificial semi permeable membrane.
- starting the therapy depends on client symptoms not on the creatinine clearance.

TYPES OF ACCESS ROUTE:

AV Fistula
Internal anastomosis of an artery to an adjacent vein.
Take about 4-6 weeks to be ready for use.
External AV Shunt
One canula is insterted into an artery and another into a vein.
Dialysate- is made from clear water and chemicals and is free of any waste products or drugs.

Anticoagulation
- to prevent blood clots from forming within the dialyzer or the blood tubing, anticoagulation
is needed during HD treatments. Heparin is the most common used drug, to prevent clot from
forming.

Complications of Hemodialysis

Disequilibrium Syndrome, and Viral Infections.


Dialysis Disequilibrium Syndrome
- may develop during hemodialysis or after hem dialysis has been completed.
- the cause is thought to be due to rapid decrease in fluid volume and blood urea nitrogen
(BUN) levels during HD.
- The change in urea levels can cause cerebral edema and increased intracranial pressure

Caring for Patient undergoing Hemodialysis:


- weight the client before and after dialysis
- know the client's dry weight
- discuss with the physician whether any of the ct's meds. Is withheld until after dialysis
- measure blood pressure, pulse rate, respirations and temperature.
- Take note that patient is at risk for fluid volume excess or deficit.

SIGNS OF FLUID OVERLOAD

hypertension
tachycardia
dyspnea
crackles in lungs
distended neck veins
high CVP

PERITONEAL DIALYSIS

The client's peritoneal membrane will be used as a semipermeable dialyzing membrane.


Hypertonic dialyzing solution is instilled through a catheter that is being inserted into the
patient's peritoneal cavity. Through the process of diffusion the excess amount of concentration
of uremic toxins and wastes in the blood moves across the peritoneal membrane and into the
dialysis solution. After a certain time, the dialysis solution (along with the toxins and waste
with it) is being drained. The client will be trained to do this procedure.

- occurs in the peritoneal cavity


- Peritoneal Dialysis is slower than Hemodialysis, however, and more time is
needed to achieve the same effect.
- at times, a client may use PD until a new arteriovenous (AV) fistula matures.
- PD is also the treatment of choice for adults because its more flexible.

Types Of Peritoneal Dialysis

Continous Ambulatory Peritoneal Dialysis


The dialysate is instilled into the abdomen and left in place 4-8 hrs.
When it is time to drain, the bag is rolled & fluid is drained by gravity.
6 Renal Two types of donors:
8 Transplantati Living donors
. on Cadaveric donors

The transplanted kidney will be placed in the illiac fossa.

- is usually a treatment for end stage renal disease.


- the donors maybe living or a cadaver.
- It is important for the tissue type to be matched
- position of the kidney: Anterioir Illiac Fossa
- the posibility of organ rejection can occur:

Types of Rejection
Hyperacute
Acute
Chronic

Acute Rejection
- Usually less than 1 yr, after transplant.
- increased blood urea levels
- weight gain, edema
- increased blood pressure

Chronic rejection
- usually more that 1 year
- hypertension
- protienuria

Major Complications:

Rejection (Signs of Rejection include: Malaise, Fever, Anemia & Graft Tenderness.
Infection
Hypertension
6 Urinary URINARY CALCULI is also called urilothiasis or KIDNEY STONES, and it can results from
9 Calculi/ anything from immobility, cancer, increased intake of Vitamin D, or an overactivity of the
. Urilithiasis Parathyroid gland.
Kidney stones is made up of:

calcium
magnesium
phosphorus
oxalate.

- High urine acidity or alkalinity contributes to stone formation.


SYMPTOMS (Urinary Calculi)

Flank pain
fever
nausea and vomiting
changes in the urinary output.

*** There will be an increase in RBC's, WBC's and bacteria.

DIAGNOSTIC TESTS for urinary calculi would include x-ray, blood tests and a 24 hour urine
test.

INTERVENTIONS:

Non Surgical Management:

- Incudes the use of laser to break the stone fragments

(Extracorporeal shock wave lithotripsy (ESWL).

patient is given a local anesthetic .


pt. is placed in a water bath or on a soft cushion.
shock waves are transmitted through the stones inside the kidney.
shock waves cause the calculi to break up into smaller pieces.

- It is very important to strain the urine after the ESWL Procedure.

* Encourage fluid intake after the procedure.


* Assess for any hemorrhage

Discharge Teaching (DIET) *Know for NCLEX:

If Uric Acid Stone:

Alkaline Ash & Low Purine Diet (limit wine, cheese & meat)
Give Allopurinol as prescribed

If Calcium Stone:
Calium Restricted Diet (Limit Dairy Foods)

Surgical Interventions:
- Uretherolithotomy

- Nephrolithotomy

Removal of renal calculi using a nephroscope.


A 32-year- A) Renal circulation
old flight A renal angiogram (renal arteriogram) provides details of the arterial supply to the
attendant is kidneys, specifically the location and number of renal arteries (multiple vessels to the
undergoing kidney are not unusual) and the patency of each renal artery.
diagnostics
due to a
significant
drop in
renal
output. The
physician
has
scheduled
an
angiography
and you are
in the midst
of
completing
client
education
about the
procedure.
The client
asks what
the
angiography
will reveal.
What is
your
response, as
her nurse?
a) Renal
circulation
b) Urine
production
c) Kidney
function
d) Kidney
structure
2. A 57-year- C) Third spacing
old Third-spacing describes the translocation of fluid from the intravascular
homeless or intercellular space to tissue compartments, where it becomes trapped
female with and useless. The client manifests signs and symptoms of hypovolemia
a history of with the exception of weight loss. There may be signs of localized
alcohol enlargement of organ cavities (such as the abdomen) if they fill with
abuse has fluid, a condition referred to as ascites.
been
admitted to
your
hospital
unit. She
was
admitted
with signs
and
symptoms
of
hypovolemi
a - minus
the weight
loss. She
exhibits a
localized
enlargement
of her
abdomen.
What
condition
could she be
presenting?
a)
Hypovolemi
a
b) Pitting
edema
c) Third-
spacing
d) Anasarca
3. A 64-year- B) No, sodium intake should be restricted
old client is The symptoms and the high level of serum sodium suggest
brought in hypernatremia, (excess of sodium). It is necessary to restrict sodium
to the clinic intake. Salt tablets and sodium chloride IV can only worsen this
with thirsty, condition but may be required in hyponatremia (sodium deficit).
dry, sticky Hypotonic solution IV may be a part of the treatment but not along with
mucous the salt tablets.
membranes,
decreased
urine
output,
fever, a
rough
tongue, and
lethargy.
Serum
sodium level
is above 145
mEq/L.
Should the
nurse start
salt tablets
when caring
for this
client?
a) No, start
with the
sodium
chloride IV.
b) No,
sodium
intake
should be
restricted.
c) Yes, this
will correct
the sodium
deficit.
d) Yes,
along with
the
hypotonic
IV.
4 Absorption or urothelium
. leakage of urine
wastes out of the
urinary system is
prevented by the
cellular
characteristics of
the _______.
5 After teaching a D) It is appropriate to warm the dialysate in a microwave
. group of students Explanation: The dialysate should be warmed in a commercial warmer and never in a
about how to microwave oven. Strict aseptic technique is essential. The infusion clamp is opened during
perform peritoneal the infusion and clamped after the infusion. When the dwell time is done, the drain clamp
dialysis, which is opened and the fluid is allowed to drain by gravity into the drainage bag.
statement would
indicate to the
instructor that the
students need
additional
teaching?
a) "The effluent
should be allowed
to drain by
gravity."
b) "It is important
to use strict aseptic
technique."
c) "The infusion
clamp should be
open during
infusion."
d) "It is
appropriate to
warm the dialysate
in a microwave."
6 After teaching a group of D) Decreased pelvic muscle tone due to multiple pregnancies
. students about the types Stress incontinence is due to decreased pelvic muscle tone, which is associated with
of urinary incontinence multiple pregnancies, obstetric injuries, obesity, menopause, or pelvic disease.
and possible causes, the Transient incontinence is due to increased urine production related to metabolic
instructor determines that conditions. Urge incontinence is due to bladder irritation related to urinary tract
the student have infections, bladder tumors, radiation therapy, enlarged prostate, or neurologic
understood the material dysfunction. Overflow incontinence is due to obstruction from fecal impaction or
when they identify which enlarged prostate.
of the following as a cause
of stress incontinence?
a) Obstruction due to
fecal impaction or
enlarged prostate
b) Bladder irritation
related to urinary tract
infections
c) Increased urine
production due to
metabolic conditions
d) Decreased pelvic
muscle tone due to
multiple pregnancies
7 After undergoing A) It's an abornmal finding that requires further assessment.
. retropubic prostatectomy, The drop in urine output to less than 30 ml/hour is abnormal and requires further
a client returns to his assessment. The reduction in urine output may be caused by an obstruction in the
room. The client is on urinary catheter tubing or deficient fluid volume from blood loss. The client's
nothing-by-mouth status nothing-by-mouth status isn't the cause of the low urine output because the client is
and has an I.V. infusing in receiving I.V. fluid to compensate for the lack of oral intake. Ambulation promotes
his right forearm at a rate urination; however, the client should produce at least 30 ml of urine/hour.
of 100 ml/hour. The client
also has an indwelling
urinary catheter that's
draining light pink urine.
While assessing the client,
the nurse notes that his
urine output is red and
has dropped to 15 ml and
10 ml for the last 2
consecutive hours. How
can the nurse best explain
this drop in urine output?
a) It's an abnormal
finding that requires
further assessment.
b) It's a normal finding
caused by blood loss
during surgery.
c) It's an abnormal
finding that will correct
itself when the client
ambulates.
d) It's a normal finding
associated with the
client's nothing-by-mouth
status.
8 An age related change D) decreased ability to concentrate urine
. in the kidney that leads The decreased ability to concentrate urine results in an increased volume of dilute
to nocturia in an older urine, which does not maintain the usual diurnal elimination pattern. A decrease in
adult is bladder capacity also contributes to nocturia, but decreased bladder muscle tone
a) decreased renal mass results in urinary retention. Decreased renal mass decreases renal reserve, but
b) decreased detrusor function is generally adequate under normal circumstances.
muscle tone
c) decreased ability to
conserve sodium
d) decreased ability to
concentrate urine
9 Below which serum B) 135 mEq/L
. sodium level may Normal serum concentration level ranges from 135 to 145 mEq/L. When the level
convulsions or coma dips below 135 mEq/L, there is hyponatremia. Manifestations of hyponatremia
can occur? include mental confusion, muscular weakness, anorexia, restlessness, elevated body
a) 140 mEq/L temperature, tachycardia, nausea, vomiting, and personality changes. Convulsions or
b) 135 mEq/L coma can occur if the deficit is severe. Values of 140, 142, and 145 mEq/L are
c) 142 mEq/L within the normal range.
d) 145 mEq/L
10 The calcium level of the blood is regulated by C) PTH
. which mechanism? The serum calcium level is controlled by PTH and
a) Androgens calcitonin. The thyroid hormone, adrenal gland, or
b) Adrenal gland androgens do not regulate the calcium level in the
c) Parathyroid hormone (PTH) blood.
d) Thyroid hormone (TH)
11. A client admitted with a gunshot wound to the A) Urine output of 250 ml/24 hours
abdomen is transferred to the intensive care unit ARF, characterized by abrupt loss of kidney function,
after an exploratory laparotomy. I.V. fluid is commonly causes oliguria, which is characterized by a
being infused at 150 ml/hour. Which assessment urine output of 250 ml/24 hours. A serum creatinine
finding suggests that the client is experiencing level of 1.2 mg/dl isn't diagnostic of ARF. A BUN level
acute renal failure (ARF)? of 22 mg/dl or a temperature of 100.2 F (37.8 C)
a) Urine output of 250 ml/24 hours wouldn't result from this disorder.
b) Temperature of 100.2 F (37.8 C)
c) Serum creatinine level of 1.2 mg/dl
d) Blood urea nitrogen (BUN) level of 22 mg/dl
12 The client asks the nurse about the functions of A) Vitamin D synthesis
. the kidney. Which should the nurse include when B) Secretion of prostaglandins
responding to the client? Select all that apply. E) Regulation of blood pressure
a) Vitamin D synthesis Explanation: Functions of the kidney include secretion
b) Secretion of prostaglandins of prostaglandins, regulation of blood pressure, and
c) Vitamin B production synthesis of aldosterone and vitamin D. The pancreas
d) Secretion of insulin secretes insulin. The body does not produce Vitamin B.
e) Regulation of blood pressure
13 A client comes to the emergency department with B) Respiratory Alkalosis
. status asthmaticus. His respiratory rate is 48 Respiratory alkalosis results from alveolar
breaths/minute, and he is wheezing. An arterial hyperventilation. It's marked by a decrease in PaCO2
blood gas analysis reveals a pH of 7.52, a partial to less than 35 mm Hg and an increase in blood pH
pressure of arterial carbon dioxide (PaCO2) of 30 over 7.45. Metabolic acidosis is marked by a decrease
mm Hg, PaO2 of 70 mm Hg, and bicarbonate in HCO3? to less than 22 mEq/L, and a decrease in
(HCO3??') of 26 mEq/L. What disorder is blood pH to less than 7.35. In respiratory acidosis, the
indicated by these findings? pH is less than 7.35 and the PaCO2 is greater than 45
a) Metabolic acidosis mm Hg. In metabolic alkalosis, the HCO3? is greater
b) Respiratory alkalosis than 26 mEq/L and the pH is greater than 7.45.
c) Metabolic alkalosis
d) Respiratory acidosis
14 A client develops decreased renal function and D) Creatinine clearance
. requires a change in antibiotic dosage. On which The physician should base changes to antibiotic
factor should the physician base the dosage dosages on creatinine clearance test results, which
change? gauge the kidney's glomerular filtration rate; this factor
a) Therapeutic index is important because most drugs are excreted at least
b) GI absorption rate partially by the kidneys. The GI absorption rate,
c) Liver function studies therapeutic index, and liver function studies don't help
d) Creatinine clearance determine dosage change in a client with decreased
renal function.
15 A client has a serum calcium level of 7.2 mg/dl. A) Trossaeu's sign
. During the physical examination, the nurse This client's serum calcium level indicates
expects to assess: hypocalcemia, an electrolyte imbalance that causes
a) Trousseau's sign. Trousseau's sign (carpopedal spasm induced by
b) Hegar's sign. inflating the blood pressure cuff above systolic
c) Homans' sign. pressure). Homans' sign (pain on dorsiflexion of the
d) Goodell's sign. foot) indicates deep vein thrombosis. Hegar's sign
(softening of the uterine isthmus) and Goodell's sign
(cervical softening) are probable signs of pregnancy.
16 A client has the following arterial blood gas (ABG) B) Metabolic acidosis
. values: pH, 7.12; partial pressure of arterial carbon This client's pH value is below normal, indicating
dioxide (PaCO2), 40 mm Hg; and bicarbonate acidosis. The HCO3- value also is below normal,
(HCO3-), 15 mEq/L. These ABG values suggest reflecting an overwhelming accumulation of acids or
which disorder? excessive loss of base, which suggests metabolic
a) Metabolic alkalosis acidosis. The PaCO2 value is normal, indicating
b) Metabolic acidosis absence of respiratory compensation. These ABG
c) Respiratory acidosis values eliminate respiratory alkalosis, respiratory
d) Respiratory alkalosis acidosis, and metabolic alkalosis.
17 A client hospitalized for treatment of a pulmonary C) Light-headedness or paresthesia
. embolism develops respiratory alkalosis. Which The client with respiratory alkalosis may complain of
clinical findings commonly accompany respiratory light-headedness or paresthesia (numbness and
alkalosis? tingling in the arms and legs). Nausea, vomiting,
a) Nausea or vomiting abdominal pain, and diarrhea may accompany
b) Hallucinations or tinnitus respiratory acidosis. Hallucinations and tinnitus rarely
c) Light-headedness or paresthesia are associated with respiratory alkalosis or any other
d) Abdominal pain or diarrhea acid-base imbalance.
18 A client in the emergency department reports that C) Metabolic Alkalosis
. he has been vomiting excessively for the past 2 A pH over 7.45 with a HCO3- level over 26 mEq/L
days. His arterial blood gas analysis shows a pH of indicates metabolic alkalosis. Metabolic alkalosis is
7.50, partial pressure of arterial carbon dioxide always secondary to an underlying cause and is
(PaCO2) of 43 mm Hg, partial pressure of arterial marked by decreased amounts of acid or increased
oxygen (PaO2) of 75 mm Hg, and bicarbonate amounts of base HCO3-. The client isn't experiencing
(HCO3-) of 42 mEq/L. Based on these findings, the respiratory alkalosis because the PaCO2 is normal.
nurse documents that the client is experiencing The client isn't experiencing respiratory or metabolic
which type of acid-base imbalance? acidosis because the pH is greater than 7.35.
a) Respiratory acidosis
b) Respiratory alkalosis
c) Metabolic alkalosis
d) Metabolic acidosis
19 A client is admitted for C) water and sodium retention secondary to a severe decrease in the
. treatment of chronic renal glomerular filtration rate.
failure (CRF). The nurse Explanation: The client with CRF is at risk for fluid imbalance
knows that this disorder dehydration if the kidneys fail to concentrate urine, or fluid retention if the
increases the client's risk of: kidneys fail to produce urine. Electrolyte imbalances associated with this
a) a decreased serum disorder result from the kidneys' inability to excrete phosphorus; such
phosphate level secondary to imbalances may lead to hyperphosphatemia with reciprocal hypocalcemia.
kidney failure. CRF may cause metabolic acidosis, not metabolic alkalosis, secondary to
b) an increased serum inability of the kidneys to excrete hydrogen ions.
calcium level secondary to
kidney failure.
c) water and sodium
retention secondary to a
severe decrease in the
glomerular filtration rate.
d) metabolic alkalosis
secondary to retention of
hydrogen ions.
20 A client is admitted with nausea, vomiting, A) Start IV fluids with normal saline solution bolus followed
. and diarrhea. His blood pressure on by a maintenance dose.
admission is 74/30 mm Hg. The client is Explanation: The client is in prerenal failure caused by
oliguric and his blood urea nitrogen (BUN) hypovolemia. I.V. fluids should be given with a bolus of
and creatinine levels are elevated. The normal saline solution followed by maintenance I.V. therapy.
physician will most likely write an order for This treatment should rehydrate the client, causing his blood
which treatment? pressure to rise, his urine output to increase, and the BUN
a) Start I.V. fluids with a normal saline and creatinine levels to normalize. The client wouldn't be
solution bolus followed by a maintenance able to tolerate oral fluids because of the nausea, vomiting,
dose. and diarrhea. The client isn't fluid-overloaded so his urine
b) Administer furosemide (Lasix) 20 mg I.V. output won't increase with furosemide, which would actually
c) Encourage oral fluids. worsen the client's condition. The client doesn't require
d) Start hemodialysis after a temporary dialysis because the oliguria and elevated BUN and
access is obtained. creatinine levels are caused by dehydration.
21 A client is frustrated and embarrassed D) Assessing present voiding patterns
. by urinary incontinence. Which Explanation: The guidelines for initiating bladder retraining include
measure should the nurse include in a assessing the client's present intake patterns, voiding patterns, and
bladder retraining program? reasons for each accidental voiding. Lowering the client's fluid
a) Restricting fluid intake to reduce intake won't reduce or prevent incontinence. The client should be
the need to void encouraged to drink 1.5 to 2 L of water per day. A voiding schedule
b) Establishing a predetermined fluid should be established after assessment.
intake pattern for the client
c) Encouraging the client to increase
the time between voidings
d) Assessing present voiding patterns
22 A client is scheduled for a creatinine C) 1 minute
. clearance test. The nurse should Explanation: The creatinine clearance test determines the kidneys'
explain that this test is done to assess ability to remove a substance from the plasma in 1 minute. It doesn't
the kidneys' ability to remove a measure the kidneys' ability to remove a substance over a longer
substance from the plasma in: period.
a) 1 hour.
b) 24 hours.
c) 1 minute.
d) 30 minutes.
23 A client is taking spironolactone A) ECG results
. (Aldactone) to control her hypertension. Although changes in all these findings are seen in
Her serum potassium level is 6 mEq/L. hyperkalemia, ECG results should take priority because
For this client, the nurse's priority should changes can indicate potentially lethal arrhythmias such as
be to assess her: ventricular fibrillation. It wouldn't be appropriate to assess the
a) electrocardiogram (ECG) results. client's neuromuscular function, bowel sounds, or respiratory
b) neuromuscular function. rate for effects of hyperkalemia.
c) bowel sounds.
d) respiratory rate.
24 A client presents at the testing center for C) Do you have any allergies?
. an intravenous pyelogram. What question Explanation: Many contrast dyes contain iodine. Therefore, it is
should the nurse ask to ensure the safety essential for the nurse to determine whether the client has any
of the client? allergies, especially to iodine, shellfish, and other seafood.
a) "Have you any artificial joints?"
b) "Do you have a pacemaker?"
c) "Do you have any allergies?"
d) "Who has come with you today?"
25 The client presents with nausea and B) Ureteral colic
. vomiting, absent bowel sounds, and
colicky flank pain. The nurse interprets
these findings as consistent with:
a) Urethritis
b) Ureteral colic
c) Interstitial cystitis
d) Acute prostatitis
26 A client undergoes extracorporeal B) Increase your fluid intake to 2 to 3 L per day
. shock wave lithotripsy. Before The nurse should instruct the client to increase his fluid intake.
discharge, the nurse should provide Increasing fluid intake flushes the renal calculi fragments through
which instruction? and prevents obstruction of the urinary system. Measuring
a) "Be aware that your urine will be temperature every 4 hours isn't needed. Lithotripsy doesn't require an
cherry-red for 5 to 7 days." incision. Hematuria may occur for a few hours after lithotripsy but
b) "Increase your fluid intake to 2 to should then disappear.
3 L per day."
c) "Apply an antibacterial dressing
to the incision daily."
d) "Take your temperature every 4
hours."
27 A client with a genitourinary B) The left kidney usually is slightly higher than the right one
. problem is being examined in the The left kidney usually is slightly higher than the right one. An
emergency department. When adrenal gland lies atop each kidney. The average kidney measures
palpating the client's kidneys, the approximately 11 cm (4??) long, 5 to 5.8 cm (2? to 2?) wide, and
nurse should keep in mind which 2.5 cm (1?) thick. The kidneys are located retroperitoneally, in the
anatomic fact? posterior aspect of the abdomen, on either side of the vertebral
a) The kidneys are situated just column. They lie between the 12th thoracic and 3rd lumbar vertebrae.
above the adrenal glands.
b) The left kidney usually is slightly
higher than the right one.
c) The kidneys lie between the 10th
and 12th thoracic vertebrae.
d) The average kidney is
approximately 5 cm (2?) long and 2
to 3 cm (? to 1??) wide.
28 A client with a suspected overdose of D) Prepare to assist with ventilation
. an unknown drug is admitted to the Respiratory acidosis is associated with hypoventilation; in this
emergency department. Arterial client, hypoventilation suggests intake of a drug that has suppressed
blood gas values indicate respiratory the brain's respiratory center. Therefore, the nurse should assume the
acidosis. What should the nurse do client has respiratory depression and should prepare to assist with
first? ventilation. After the client's respiratory function has been stabilized,
a) Prepare for gastric lavage. the nurse can safely monitor the heart rhythm, prepare for gastric
b) Monitor the client's heart rhythm. lavage, and obtain a urine specimen for drug screening.
c) Obtain a urine specimen for drug
screening.
d) Prepare to assist with ventilation.
29 A client with Guillain-Barr B) oh, 7.25; PaCO2 50 mm Hg
. syndrome develops respiratory In respiratory acidosis, ABG analysis reveals an arterial pH below
acidosis as a result of reduced 7.35 and partial pressure of arterial carbon dioxide (PaCO2) above
alveolar ventilation. Which 45 mm Hg. Therefore, the combination of a pH value of 7.25 and a
combination of arterial blood gas PaCO2 value of 50 mm Hg confirms respiratory acidosis. A pH
(ABG) values confirms respiratory value of 7.5 with a PaCO2 value of 30 mm Hg indicates respiratory
acidosis? alkalosis. A ph value of 7.40 with a PaCO2 value of 35 mm Hg and
a) pH, 7.25; PaCO2 50 mm Hg a pH value of 7.35 with a PaCO2 value of 40 mm Hg represent
b) pH, 7.35; PaCO2 40 mm Hg normal ABG values, reflecting normal gas exchange in the lungs.
c) pH, 7.40; PaCO2 35 mm Hg
d) pH, 7.5; PaCO2 30 mm Hg
30 A client with pancreatic cancer has the following A) Potassium
. blood chemistry profile: Glucose, fasting: 204 The nurse should identify potassium: 2.2 mEq/L as
mg/dl; blood urea nitrogen (BUN): 12 mg/dl; critical because a normal potassium level is 3.8 to 5.5
Creatinine: 0.9 mg/dl; Sodium: 136 mEq/L; mEq/L. Severe hypokalemia can cause cardiac and
Potassium: 2.2 mEq/L; Chloride: 99 mEq/L; respiratory arrest, possibly leading to death.
CO2: 33 mEq/L. Which result should the nurse Hypokalemia also depresses the release of insulin and
identify as critical and report immediately? results in glucose intolerance. The glucose level is above
a) Potassium normal (normal is 75 to 110 mg/dl) and the chloride
b) Sodium level is a bit low (normal is 100 to 110 mEq/L).
c) Chloride Although these levels should be reported, neither is life-
d) CO2 threatening. The BUN (normal is 8 to 26 mg/dl) and
creatinine (normal is 0.8 to 1.4 mg/dl) are within normal
range.
31 A client with renal failure is undergoing C) Risk for infection
. continuous ambulatory peritoneal dialysis. Explanation: The peritoneal dialysis catheter and regular
Which nursing diagnosis is the most appropriate exchanges of the dialysis bag provide a direct portal for
for this client? bacteria to enter the body. If the client experiences
a) Impaired urinary elimination repeated peritoneal infections, continuous ambulatory
b) Toileting self-care deficit peritoneal dialysis may no longer be effective in clearing
c) Risk for infection waste products. Impaired urinary elimination, Toileting
d) Activity intolerance self-care deficit, and Activity intolerance may be
pertinent but are secondary to the risk of infection.
32 A client with urinary incontinence a) "Make sure to eat enough fiber to prevent constipation."
. asks the nurse for suggestions Explanation: Suggestions to manage urinary incontinence include
about managing this condition. avoiding constipation such as eating adequate fiber and drinking
Which suggestion would be most adequate amounts of fluid. Scented powders, lotions, or sprays should
appropriate? be avoided because they can intensify the urine odor, irritate the skin,
or cause a skin infection. Stimulants such as caffeine, alcohol, and
a) "Make sure to eat enough fiber aspartame should be avoided. The client should void regularly,
to prevent constipation." approximately every 2 to 3 hours to ensure bladder emptying.
b) "Try drinking coffee throughout
the day."
c) "Use scented powders to disguise
any odor."
d) "Limit the number of times you
urinate during the day."
33 A client with urinary tract infection C) This medication will relieve your pain
. is prescribed phenazopyridine Phenazopyridine (Pyridium) is a urinary analgesic agent used for the
(Pyridium). Which of the following treatment of burning and pain associated with UTIs.
instructions would the nurse give
the client?
a) "This medication will prevent
re-infection."
b) "This medication should be
taken at bedtime."
c) "This medication will relieve
your pain."
d) "This will kill the organism
causing the infection."
34 A clinical situation in which the a) Hypoexmia
. increased release of erythropoietin Erythropoietin is released when the oxygen tension of the renal blood
would be expected is: supply is low and stimulates production of red blood cells in the bone
marrow. Hypotension causes activation of the renin-angiotensin-
a) hypoexmia aldosterone system, as well as release of ADH. Hyperkalemis
b) hypotension stimulates release of aldosterone from the adrenal cortex, and fluid
c) hyperkalemia overload does not directly stimulate factors affecting the kidney.
d) fluid overload
35 During physical assessment of the c) percusses the kidney with a firm blow at the posterior costovertebral
. urinary system, the nurse angle
a) auscultates the lower abdominal To assess for kidney tenderness, the nurse strikes the fist of one hand
quadrants for fluid sounds over the dorsum of the other hand at the posterior costovertebral angle.
b) palpates an empty bladder at the The upper abdominal quadrants and costovertebral angles are
level of the symphysis pubis auscultated for vascular bruits in the renal vessels and aorta, and an
c) percusses the kidney with a firm empty bladder is not palpable. The kidneys are palpated through the
blow at the posterior costovertebral abdomen, with the patient supine.
angle
d) positions the patient prone to
palpate the kidneys with a
posterior approach
36 An elderly client takes 40 mg of C) Hypokalemia
. Lasix twice a day. Which Hypokalemia (potassium level below 3.5 mEq/L) usually indicates a
electrolyte imbalance is the most defict in total potassium stores. Potassium-losing diuretics, such as
serious adverse effect of diuretic loop diuretics, can induce hypokalemia.
use?
a) Hypophosphatemia
b) Hypernatremia
c) Hypokalemia
d) Hyperkalemia
37 Following a unilateral C) Muscle weakness
. adrenalectomy, a nurse should assess Muscle weakness, bradycardia, nausea, diarrhea, and paresthesia of
for hyperkalemia as indicated by: the hands, feet, tongue, and face are findings associated with
a) diaphoresis. hyperkalemia, which is transient and results from transient
b) tremors. hypoaldosteronism when the adenoma is removed. Tremors,
c) muscle weakness. diaphoresis, and constipation aren't seen in hyperkalemia.
d) constipation.
38 The following catheterization C) Permanent drainage with a urethral catheter
. procedures are used to treat clients Permanent drainage with a urethral catheter carries the greatest risk. It
with urinary retention. Which may also increase the risk for bladder stones, renal diseases, bladder
procedure would the nurse identify infections, and urosepsis, a severe systemic infection by
as carrying the greatest risk to the microorganisms in the urinary tract invading the bloodstream. Clean
client? intermittent catheterization has the fewest complications and is the
a) Clean intermittent catheterization preferred treatment for urinary retention. The Cred voiding
b) Suprapubic cystostomy tube procedure is used in the case of clients who have lost control over
c) Permanent drainage with a their nervous systems, secondary to injury or disease.
urethral catheter
d) Cred voiding procedure
39 A group of nursing students are studying for a test B) Bicarbonate-carbonic acid buffer system
. over acid-base imbalance. One student asks The major chemical regulator of plasma pH is the
another what the major chemical regulator of bicarbonate-carbonic acid buffer system. Therefore
plasma pH is. What should the second student options A and C are incorrect. Option D does not
respond? exist, it is only a distractor for this question.
a) Renin-angiotensin-aldosterone system
b) Bicarbonate-carbonic acid buffer system
c) Sodium-potassium pump
d) ADH-ANP buffer system
40 A group of students are reviewing information A) Urinary retention
. about disorders of the bladder and urethra. The Urinary retention and urinary incontinence are
students demonstrate understanding of the voiding dysfunctions, temporary or permanent
material when they identify which of the following alterations in the ability to urinate normally. Cystitis
as a voiding dysfunction? is an infectious disorder. Bladder stones and urethral
a) Urinary retention stricture are obstructive disorders.
b) Cystitis
c) Bladder stones
d) Urethral stricture
41 A history of infection specifically caused by A) Acute glomerulonephritis
. group A Acute glomerulonephritis is also associated with varicella
beta-hemolytic streptococci is associated with zoster virus, hepatitis B, and Epstein-Barr virus. Acute renal
which of the following disorders? failure is associated with hypoperfusion to the kidney,
a) Acute glomerulonephritis parenchymal damage to the glomeruli or tubules, and
b) Acute renal failure obstruction at a point distal to the kidney. Chronic renal
c) Nephrotic syndrome failure may be caused by systemic disease, hereditary
d) Chronic renal failure lesions, medications, toxic agents, infections, and
medications. Nephrotic syndrome is caused by disorders
such as chronic glomerulonephritis, systemic lupus
erythematosus, multiple myeloma, and renal vein
thrombosis.
42 A male client has doubts about performing D) Wear a mask when performing exchanges
. peritoneal dialysis at home. He informs the The nurse should advise the client to wear a mask while
nurse about his existing upper respiratory performing exchanges. This prevents contamination of the
infection. Which of the following suggestions dialysis catheter and tubing, and is usually advised to clients
can the nurse offer to the client while with upper respiratory infection. Auscultation of the lungs
performing an at-home peritoneal dialysis? will not prevent contamination of the catheter or tubing. The
a) Perform deep-breathing exercises client may also be advised to perform deep-breathing
vigorously. exercises to promote optimal lung expansion, but this will
b) Avoid carrying heavy items. not prevent contamination. Clients with a fistula or graft in
c) Auscultate the lungs frequently. the arm should be advised against carrying heavy items.
d) Wear a mask when performing exchanges.
43 The most common presenting B) Change in cognitive functioning
. objective symptoms of a urinary The most common objective finding is a change in cognitive
tract infection in older adults, functioning, especially in those with dementia, because these patients
especially in those with dementia, usually exhibit even more profound cognitive changes with the onset of
include? a UTI. Incontinence, hematuria, and back pain are not the most
a) Hematuria common presenting objective symptoms.
b) Change in cognitive functioning
c) Back pain
d) Incontinence
44 The normal serum value for C) 3.5-5.5 mEq/L
. potassium is Serum potassium must be within normal limits to prevent cardiac
a) 96 to 106 mEq/L. dysrhythmia. Normal serum sodium is 135 to 145 mEq/L. Normal
b) 135 to 145 mEq/L. serum chloride is 96 to 106 mEq/L. Normal total serum calcium is 8.5
c) 3.5 to 5.5 mEq/L. to 10
d) 8.5 to 10.5 mg/dL.
45 A nurse assesses a client shortly D) Urine output of 20 ml/hour
. after living donor kidney Explanation: Because kidney transplantation carries the risk of
transplant surgery. Which transplant rejection, infection, and other serious complications, the
postoperative finding must the nurse should monitor the client's urinary function closely. A decrease
nurse report to the physician from the normal urine output of 30 ml/hour is significant and warrants
immediately? immediate physician notification. A serum potassium level of 4.9
a) Serum sodium level of 135 mEq/L, a serum sodium level of 135 mEq/L, and a temperature of
mEq/L 99.2 F are normal assessment findings.
b) Serum potassium level of 4.9
mEq/L
c) Temperature of 99.2 F (37.3 C)
d) Urine output of 20 ml/hour
46 A nurse correctly identifies a urine specimen with B) Acidic
. a pH of 4.3 as being which type of solution? Normal urine pH is 4.5 to 8.0; a value of 4.3 reveals
a) Alkaline acidic urine pH. A pH above 7.0 is considered an
b) Acidic alkaline or basic solution. A pH of 7.0 is considered
c) Basic neutral.
d) Neutral
47 The nurse expects which of the following A) Dehydration
. assessment findings in the client in the diuretic The diuretic phase of acute renal failure is characterized
phase of acute renal failure? by increased urine output, hypotension, and
a) Dehydration dehydration.
b) Crackles
c) Hypertension
d) Hyperkalemia
48 A nurse has been asked to speak to a local A) Need to urinate after engaging in sexual intercourse
. women's group about preventing cystitis. Which Measures to prevent cystitis include voiding after
of the following would the nurse include in the sexual intercourse, wearing cotton underwear, urinating
presentation? every 2 to 3 hours while awake, and taking showers
a) Need to urinate after engaging in sexual instead of tub baths.
intercourse
b) Need to wear underwear made from synthetic
material
c) Importance of urinating every 4 to 6 hours
while awake
d) Suggestion to take tub baths instead of showers
49 The nurse is caring for a client who is B) Find and correct the cause of tachypnea
. exhibiting symptoms of tachypnea Tachypnea or rapid breathing may result from various reasons
and circumoral paresthesias. What including acute anxiety, high fever, thyrotoxicosis, early salicylate
should be the nurse's first course of poisoning, hypoxemia, or mechanical ventilation. The rapid
action? breathing expels more CO2 than necessary. This causes a deficit in
a) Stop mechanical ventilation. carbonic acid, leading to respiratory alkalosis. Circumoral
b) Find and correct the cause of paresthesia is one of the symptoms. The first course of action is to
tachypnea. detect and treat the cause of tachypnea. The nurse has to maintain
c) Administer cardiopulmonary mechanical ventilation if the client is dependent on it. CPR
resuscitation (CPR). administration is required only if the client's condition needs it.
d) Give a dose of aspirin. Aspirin is not advised as early aspirin poisoning may be a cause of
the tachypnea.
50 The nurse is caring for a client who is D) My urine will be eliminated through a stoma
. scheduled for the creation of an ileal An ileal conduit is a non-continent urinary diversion whereby the
conduit. Which statement by the client ureters drain into an isolated section of ileum. A stoma is created at
provides evidence that client teaching one end of the ileum, exiting through the abdominal wall.
was effective?
a) "My urine will be eliminated with
my feces."
b) "A catheter will drain urine
directly from my kidney."
c) "I will not need to worry about
being incontinent of urine."
d) "My urine will be eliminated
through a stoma."
51 A nurse is caring for a client with metastatic breast cancer D) Hypercalcemia
. who is extremely lethargic and very slow to respond to The normal reference range for serum calcium
stimuli. The laboratory report indicates a serum calcium is 9 to 11 mg/dl. A serum calcium level of 12
level of 12.0 mg/dl, a serum potassium level of 3.9 mEq/L, mg/dl clearly indicates hypercalcemia. The
a serum chloride level of 101 mEq/L, and a serum sodium client's other laboratory findings are within
level of 140 mEq/L. Based on this information, the nurse their normal ranges, so the client doesn't have
determines that the client's symptoms are most likely hypernatremia, hypochloremia, or
associated with which electrolyte imbalance? hypokalemia.
a) Hypocalcemia
b) Hyperkalemia
c) Hypokalemia
d) Hypercalcemia
52 The nurse is caring for the client following surgery for a A) Disturbed body image
. urinary diversion. The client refuses to look at the stoma The client is exhibiting defining characteristics
or participate in its care. The nurse formulates a nursing of disturbed body image.
diagnosis of:

a) Disturbed body image


b) Situational low self esteem
c) Anticipatory grieving
d) Deficient knowledge: stoma care
53 The nurse is preparing an education program on risk D) Hypotension
. factors for kidney disorders. Which of the following risk Hypertension, not hypotension, is a risk factor
factors would be inappropriate for the nurse to include in for kidney disease.
the teaching program?
a) Pregnancy
b) Diabetes mellitus
c) Neuromuscular disorders
d) Hypotension
54 A nurse is reviewing the history and A) Painless hematuria
. physical examination of a client with a Explanation: The most common first symptom of a malignant
suspected malignant tumor of the tumor of the bladder is painless hematuria. Additional early
bladder. Which finding would the nurse symptoms include UTI with symptoms such as fever, dysuria,
identify as the most common initial urgency, and frequency. Later symptoms are related to
symptom? metastases and include pelvic pain, urinary retention (if the
a) Urinary retention tumor blocks the bladder outlet), and urinary frequency from the
b) Painless hematuria tumor occupying bladder space.
c) Fever
d) Frequency
55 A nurse is reviewing the history of a client C) Recent hx of streptococcal infection
. who is suspected of having Explanation: Glomerulonephritis can occur as a result of
glomerulonephritis. Which of the infections from group A beta-hemolytic streptococcal infections,
following would the nurse consider bacterial endocarditis, or viral infections such as hepatitis B or C
significant? or human immunodeficiency virus (HIV). A history of
a) History of hyperparathyroidism hyperparathyroidism or osteoporosis would place the client at
b) History of osteoporosis risk for developing renal calculi. A history of pyelonephritis
c) Recent history of streptococcal would increase the client's risk for chronic pyelonephritis.
infection
d) Previous episode of acute
pyelonephritis
56 A nurse is reviewing the laboratory test D) Increased serum creatinine
. results of a client with renal disease. In clients with renal disease, the serum creatinine level would be
Which of the following would the nurse increased. The BUN also would be increased, serum albumin
expect to find? would be decreased, and potassium would likely be increased.
a) Decreased blood urea nitrogen (BUN)
b) Decreased potassium
c) Increased serum albumin
d) Increased serum creatinine
57 The nurse is to check residual urine A) Catheterize the client immediately after the client voids
. amounts for a client experiencing urinary Explanation: To obtain accurate residual volumes, it is important
retention. Which of the following would that clients void first and that catheterization occur immediately
be most important? after the attempt. The nurse should record both the volume
a) Catheterize the client immediately voided (even if it is zero) and the volume obtained by
after the client voids. catheterization. Intermittent catheterizations are performed based
b) Check for residual after the client on a schedule, usually 3 to 4 times per day. Residual urine refers
reports the urge to void. to the amount remaining in the bladder after voiding. It is
c) Set up a routine schedule of every 4 essential that the client voids.
hours to check for residual urine.
d) Record the volume of urine obtained.
58 The nurse observes the color of the B) Urinary tract tests
. client's urine which appears pale blue- It is important to inquire about drugs because some drugs may
green. The nurse obtains a drug history affect the outcome of urinary tract tests as well as the color and
from the client based on the odor of the urine. Dietary intake may affect urine characteristics
understanding that drugs used by the as well as urinary tract disorders and their management. Drugs
client may affect which of the following? do not directly affect the size of the urinary bladder or the
a) Size of the urinary bladder amount of urine produced.
b) Urinary tract tests
c) Urine specific gravity
d) Amount of urine produced
59 A nurse reviews the arterial blood C) Respiratory Alkalosis
. gas (ABG) values of a client admitted A client with pneumonia may hyperventilate in an effort to increase
with pneumonia: pH, 7.51; PaCO2, oxygen intake. Hyperventilation leads to excess carbon dioxide
28 mm Hg; PaO2, 70 mm Hg; and (CO2) loss, which causes alkalosis indicated by this client's
HCO3--, 24 mEq/L. What do these elevated pH value. With respiratory alkalosis, the kidneys'
values indicate? bicarbonate (HCO3-) response is delayed, so the client's HCO3- level
a) Metabolic alkalosis remains normal. The below-normal value for the partial pressure of
b) Metabolic acidosis arterial carbon dioxide (PaCO2) indicates CO2 loss and signals a
c) Respiratory alkalosis respiratory component. Because the HCO3- level is normal, this
d) Respiratory acidosis imbalance has no metabolic component. Therefore, the client is
experiencing respiratory alkalosis.
60 A nurse, when caring for a client, B) Excessive fluid intake
. notes that the specific gravity of the Excess fluid intake results in low specific gravity of urine. Excessive
client's urine is low. What could have fluid intake will result in formation of dilute urine. When the urine is
lead to the low specific gravity of diluted, it results in low specific gravity of urine. Frequent vomiting,
urine? repeated diarrhea, and urine retention will result in high specific
a) Repeated diarrhea gravity of urine.
b) Excess fluid intake
c) Frequent vomiting
d) Urine retention
61 A nursing instructor is reviewing with the class -Plasma filtered through glomerulus
. the steps in urine formation. Place in the correct -Filtrate enters Bowman's capsule
order from first to last the sequence the -Filtrate moves through tubular system of the nephron
instructor would present. and is either reabsorped or excreted
-Filtrate enters Bowman's capsule -Formed urine drains from the collecting tubules, into
-Plasma filtered through glomerulus the renal pelvis, and down each ureter to the bladder
-Formed urine drains from the collecting
tubules, into the renal pelvis, and down each
ureter to the bladder
-Filtrate moves through tubular system of the
nephron and is either reabsorped or excreted
62 Nursing management of the client with a urinary B) Discouraging caffeine intake
. tract infection should include: Strategies for preventing urinary tract infection include
a) Teaching the client to douche daily proper perineal hygiene, increased fluid intake, avoiding
b) Discouraging caffeine intake urinary tract irritants (including caffeine), and
c) Administering morphine sulfate establishing a frequent voiding regimen.
d) Instructing the client to limit fluid intake
63 Patients diagnosed with hypervolemia should avoid D) It increases the client's desire to consume fluid
. sweet or dry food because: The management goal in hypervolemia is to reduce
a) It obstructs water elimination. fluid volume. For this reason, fluid is rationed, and the
b) It can cause dehydration. client is advised to take limited amount of fluid when
c) It can lead to weight gain. thirsty. Sweet or dry food can increase the client's
d) It increases the client's desire to consume fluid. desire to consume fluid. Sweet or dry food does not
obstruct water elimination nor does it cause
dehydration. Weight regulation is not part of
hypervolemia management except to the extent that it
is achieved on account of fluid reduction.
64 A patient with an obstruction of the renal artery A) Increase Renin Release
. causing renal ischemia exhibits HTN. One factor Renin is released in resonse to decreased B/P, renal
that may contribute to HTN: ischemia, eosinophil chemotactic factor (ECF)
depletion, and other factors affecting blood suppy to
a) increase renin release the kidney. It is they catalyst of the renin-angiotensin-
b) increased ADH secretion aldosterone system, which raises B/P when stimulated.
c) decreased aldosterone secretion ADH is secreted by the posterior pituitary in response
d) increased synthesis and release of to serum hyperosmolality and low blood volume.
prostaglandins Aldosterone is secreted within the renin-angiotensin
II, and kidney prostaglandins lower B/P by causing
vasodilation.
65 A physician orders cystoscopy and random B) hematuria
. biopsies of the bladder for a client who reports Intravesical instillation of BCG commonly causes
painless hematuria. Test results reveal carcinoma hematuria. Other common adverse effects of BCG
in situ in several bladder regions. To treat bladder include urinary frequency and dysuria. Less
cancer, the client will have a series of intravesical commonly, BCG causes cystitis, urinary urgency,
instillations of bacillus Calmette-Gurin (BCG), urinary incontinence, urinary tract infection,
administered 1 week apart. When teaching the abdominal cramps or pain, decreased bladder capacity,
client about BCG, the nurse should mention that tissue in urine, local infection, renal toxicity, and
this drug commonly causes: genital pain. BCG isn't associated with renal calculi,
a) delayed ejaculation. delayed ejaculation, or impotence.
b) hematuria.
c) impotence.
d) renal calculi.
66 A physician orders regular insulin 10 units I.V. C) Hyperkalemia
. along with 50 ml of dextrose 50% for a client Administering regular insulin I.V. concomitantly with 50
with acute renal failure. What electrolyte ml of dextrose 50% helps shift potassium from the
imbalance is this client most likely experiencing? extracellular fluid into the cell, which normalizes serum
a) Hyperglycemia potassium levels in the client with hyperkalemia. This
b) Hypercalcemia combination doesn't help reverse the effects of
c) Hyperkalemia hypercalcemia, hypernatremia, or hyperglycemia.
d) Hypernatremia
67 A priority nursing intervention for a client with A) Monitoring respiratory status for signs ans symptoms
. hypervolemia involves which of the following? of pulmonary complications
a) Monitoring respiratory status for signs and Hypervolemia, or fluid volume excess (FVE), refers to
symptoms of pulmonary complications. an isotonic expansion of the extracellular fluid. Nursing
b) Establishing I.V. access with a large-bore interventions for FVE include measuring intake and
catheter. output, monitoring weight, assessing breath sounds,
c) Encouraging the client to consume sodium- monitoring edema, and promoting rest. The most
free fluids. important intervention in the list involves monitoring the
d) Drawing a blood sample for typing and respiratory status for any signs of pulmonary congestion.
crossmatching. Breath sounds are assessed at regular intervals.
68 Retention of which electrolyte is the most life- A) Potassium
. threatening effect of renal failure? Retention of potassium is the most life-threatening effect
a) Potassium of renal failure.
b) Calcium
c) Phosphorous
d) Sodium
69 Russell Thompkins, a 77-year-old retired male, A) One liter
. visits your general practice office twice monthly A 2-lb weight gain in 24 hours indicates that the client is
to maintain control of his congestive heart retaining 1L of fluid.
failure. He measures his weight daily and phones
it to your office for his medical record. In a 24-
hour period, how much fluid is Russell retaining
if his weight increases by two pounds?
a) One liter
b) 1250 ml
c) 1500 ml
d) 500 ml
70 The sites where urinary stones are most likely to Ureteropelvic junction and Ureterovesical junction
. obstruct the urinary system are at the
__________ and the ____________.
71 Susan Young, a 57-year-old financial officer, has C) Angiography
. been exhibiting signs and symptoms which lead Angiography provides the details of the arterial supply to
her urologist to suspect the adequacy of her the kidneys, specifically the number and location of renal
urinary function. Beginning with the least arteries. Radiography shows the size and position of the
invasive tests, which of the following would you kidneys, ureters, and bladder. A CT scan is useful in
expect the physician to prescribe to assess identifying calculi, congenital abnormalities, obstruction,
kidney function? Choose all correct options. infections, and polycystic diseases. Cystoscopy is used
a) Blood urea nitrogen (BUN) level for providing a visual examination of the internal
b) Creatinine clearance bladder.
c) Angiography
d) All options are correct
72 To assess circulating oxygen levels, the 2001 A) Hemoglobin
. Kidney Disease Outcomes Quality Initiative: Explanation: Although hematocrit has always been the
Management of Anemia Guidelines recommends blood test of choice to assess for anemia, the 2001
the use of which of the following diagnostic Kidney Disease Outcomes Quality Initiative:
tests? Management of Anemia Guidelines, recommend that
a) Hemoglobin anemia be quantified using hemoglobin rather than
b) Hematocrit hematocrit measurements. Hemoglobin is recommended
c) Arterial blood gases as it is more accurate in the assessment of circulating
d) Serum iron levels oxygen than hematocrit. Serum iron levels measure iron
storage in the body. Arterial blood gases assess the
adequacy of oxygenation, ventilation, and acid-base
status.
73 To compensate for decreased fluid B) Tachycardia
. volume (hypovolemia), the nurse can Fluid volume deficit, or hypovolemia, occurs when the loss of
anticipate which response by the extracellular fluid exceeds the intake of fluid. Clinical signs include
body? oliguia, rapid heart rate, vasoconstriction, cool and clammy skin,
a) Bradycardia and muscle weakness. The nurse monitors for rapid, weak pulse and
b) Tachycardia orthostatic hypotension.
c) Increased urine output
d) Vasodilation
74 To evaluate a client for hypoxia, the D) ABGs
. physician is most likely to order Red blood cell count, sputum culture, total hemoglobin, and ABG
which laboratory test? analysis all help evaluate a client with respiratory problems.
a) Red blood cell count However, ABG analysis is the only test that evaluates gas exchange
b) Sputum culture in the lungs, providing information about the client's oxygenation
c) Total hemoglobin status.
d) Arterial blood gas (ABG) analysis
75 Total bladder capacity ranges from 600-1000mL
. _________ mL to _________ mL.
76 True or False? False
. Atrial Natriuretic Factor (ANF) is Atrial Natriuretic Factor (ANF) is secreted by the right atruim when
secreted by the right atruim when atrial blood pressure is low, and it inhibits the action of high
atrial blood pressure is low, and it antidiuretic hormone (ADH) or renin or angiotensin II
inhibits the action of aldosterone.
77 True or False? True
. GFR is primarily dependent on
adequate blood flow and adequate
hydrostatic pressure.
78 True or False? True
. Increased permeability in the
glomerulus causes loss of proteins into
the urine.
79 True or False? True
. Prostaglandin synthesis by the
kidneys causes vasodilation and
increased renal blood flow.
80 True or False? False
. The primary function of the kidney Primary function of kidney is to regulate the volume and composition
is to excrete nitogeneous waste of extracellular fluids
products.
81 True or False? True
. Water is the primary substance
reabsorbed in the collecting duct?
82 A urinalysis of a urine specimen B) bacteria
. that is not processed within 1 hour bacteria in warm urine specimens multiply rapidly, and false or
may result in erroneous unreliable bacterial counts may occur with old urine. Glucose, specific
measurement of gravity, and WBCs do not change in urine specimens, but pH becomes
a) glucose more alkaline, RBCs are hemolyzed, and casts may disintegrate.
b) bacteria
c) specific gravity
d) white blood cells
83 The volume of urine in the bladder 200-250 mL
. that usually causes the urge to
urinate is ____ mL.
84 When a client's ventilation A) Carbon dioxide (CO2)
. is impaired, the body When ventilation is impaired, the body retains CO2 because the carbonic acid
retains which substance? level increases in the blood. Sodium bicarbonate is used to treat acidosis.
a) Carbon dioxide (CO2) Nitrous oxide, which has analgesic and anesthetic properties, commonly is
b) Oxygen administered before minor surgical procedures. When ventilation is impaired,
c) Sodium bicarbonate the body doesn't retain oxygen. Instead, the tissues use oxygen and CO2 results.
d) Nitrous oxide
85 When evaluating arterial A) PaCO 36
. blood gases (ABGs), which Metabolic alkalosis is a clinical disturbance characterized by a high pH and high
value is consistent with plasma bicarbonate concentration. The HCO value is below normal. The PaCO
metabolic alkalosis? value and the oxygen saturation level are within a normal range
a) PaCO 36
b) HCO 21 mEq/L
c) O saturation 95%
d) pH 7.48
86 When preparing a client for A) Check for thrill or bruit over the access site. When preparing a client for
. hemodialysis, which of the hemodialysis, the nurse would need to check for a thrill or bruit over the
following would be most vascular access site to ensure patency. Inspecting the catheter insertion site
important for the nurse to for infection, adding the prescribed drug to the dialysate, and warming the
do? solution to body temperature would be necessary when preparing a client for
a) Check for thrill or bruit peritoneal dialysis.
over the access site.
b) Warm the solution to body
temperature.
c) Inspect the catheter
insertion site for infection.
d) Add the prescribed drug to
the dialysate.
87 Which conditions lead to A) Thoracic skeletal change
. chronic respiratory acidosis Poor respiratory exchange as the result of chronic lung disease, inactivity, or
in older adults? thoracic skeletal changes may lead to chronic respiratory acidosis. Decreased
a) Thoracic skeletal change renal function in older adults can cause an inability to concentrate urine and
b) Overuse of sodium is usually associated with fluid and electrolyte imbalance. A poor appetite,
bicarbonate erratic meal patterns, inability to prepare nutritious meals, or financial
c) Decreased renal function circumstances may influence nutritional status, resulting in imbalances of
d) Erratic meal patterns electrolytes. Overuse of sodium bicarbonate may lead to metabolic alkalosis.
88 Which is the correct term for B) Renal Clearance
. the ability of the kidneys to Explanation: Renal clearance refers to the ability of the kidneys to clear
clear solutes from the plasma? solutes from the plasma. GFR is the volume of plasma filtered at the
a) Glomerular filtration rate glomerulus into the kidney tubules each minute. Specific gravity reflects the
(GFR) weight of particles dissolved in the urine. Tubular secretion is the
b) Renal clearance movement of a substance from the kidney tubule into the blood in the
c) Specific gravity peritubular capillaries or vasa recta.
d) Tubular secretion
89 Which nursing assessment b) Fever
. finding indicates that the client Fever is an indicator of infection or transplant rejection.
who has undergone renal
transplant has not met
expected outcomes?
a) Weight loss
b) Fever
c) Absence of pain
d) Diuresis
90 Which of the following are the D) Breathing
. insensible mechanisms of fluid Loss of fluid from sweat or diaphoresis is referred to as insensible loss
loss? because it is unnoticeable and immeasurable. Losses from urination and
a) Bowel elimination bowel elimination are measurable.
b) Urination
c) Nausea
d) Breathing
91 Which of the following A) Potassium
. electrolytes is a major cation in Potassium is a major cation that affects cardiac muscle functioning.
body fluid? Chloride is an anion. Bicarbonate is an anion. Phosphate is an anion.
a) Potassium
b) Bicarbonate
c) Chloride
d) Phosphate
92 Which of the following is a D) Pink color
. characteristic of a normal stoma? Explanation: Characteristics of a normal stoma include a pink and
a) Painful moist appearance. It is insensitive to pain because it has no nerve
b) No bleeding when cleansing stoma endings. The area is vascular and may bleed when cleaned.
c) Dry in appearance
d) Pink color
93 Which of the following is a correct B) Oral
. route of administration for Potassium may be administered through the oral route. Potassium is
potassium? never administered by IV push or intramuscularly to avoid replacing
a) IV (intravenous) push potassium too quickly. Potassium is not administered subcutaneously.
b) Oral
c) Intramuscular
d) Subcutaneous
94 Which of the following is a factor C) Immunocompromise
. contributing to UTI in older adults? Factors that contribute to urinary tract infection in older adults
a) Low incidence of chronic illness include immunocompromise, high incidence of chronic illness,
b) Sporadic use of antimicrobial immobility, and frequent use of antimicrobial agents.
agents
c) Immunocompromise
d) Active lifestyle
95 Which of the following is considered B) 0.9% Normal Saline
. an isotonic solution? An isotonic solution is 0.9% normal saline (NaCl). Dextran in NS is a
a) 3% NaCl colloid solution, 0.45% normal saline is a hypotonic solution, and 3%
b) 0.9% normal saline NaCl is a hypertonic solution.
c) Dextran in NS
d) 0.45% normal saline
96 Which of the following is the most A) Alcoholism
. common cause of symptomatic Alcoholism is currently the most common cause of symptomatic
hypomagnesemia? hypomagnesemia. IV drug use, sedentary lifestyle, and burns are not
a) Alcoholism the most common causes of hypomagnesemia.
b) IV drug use
c) Sedentary lifestyle
d) Burns
97. Which of the following solutions is A) 0.45% NaCl
hypotonic? Half-strength saline is hypotonic. Lactated Ringer's solution is
a) 0.45% NaCl isotonic. Normal saline (0.9% NaCl) is isotonic. A solution that is
b) 5% NaCl 5% NaCl is hypertonic.
c) 0.9% NaCl
d) Lactated Ringer's solution
98. Which of the following terms is used to A) Pyleonephritis
refer to inflammation of the renal Pyelonephritis is an upper urinary tract inflammation, which may
pelvis? be acute or chronic. Cystitis is inflammation of the urinary
a) Pyelonephritis bladder. Urethritis is inflammation of the urethra. Interstitial
b) Interstitial nephritis nephritis is inflammation of the kidney.
c) Urethritis
d) Cystitis
99. Which of the following urine specific a) 1.002
gravity values would indicate to the A urine specific gravity of 1.002 is low, indicating dluite urine
nurse that the patient is receiving and the excretion of excess fluid. Fluid overload, diuretics, or lack
excessive IV fluid therapy? of ADH can cause dilute urine. Normal urine specific gravity
a) 1.002 indicates concentrated urine that would be seen in dehydration.
b) 1.010
c) 1.025
d) 1.030
100 Which of the following would be a D) Hypoventilation
. potential cause of respiratory acidosis? Respiratory acidosis is always due to inadequate excretion of CO,
a) Vomiting with inadequate ventilation, resulting in elevated plasma CO
b) Hyperventilation concentration, which causes increased levels of carbonic acid. In
c) Diarrhea addition to an elevated PaCO, hypoventilation usually causes a
d) Hypoventilation decrease in PaO.
101 Which of the following would be included in D) Drink liberal amounts of fluids
. a teaching plan for a patient diagnosed with Patients diagnosed with a UTI should drink liberal amounts
a urinary tract infection? of fluids. They should void every 2 to 3 hours. Coffee and tea
a) Drink coffee or tea to increase diuresis are urinary irritants. The patient should shower instead of
b) Use tub baths as opposed to showers bathe in a tub because bacteria in the bath water may enter
c) Void every 4 to 6 hours the urethra.
d) Drink liberal amount of fluids
102 Which of the following would the nurse D) Increased serum creatinine level
. expect to find when reviewing the Explanation: In renal failure, laboratory blood tests reveal
laboratory test results of a client with renal elevations in BUN, creatinine, potassium, magnesium, and
failure? phosphorus. Calcium levels are low. The RBC count,
a) Increased red blood cell count hematocrit, and hemoglobin are decreased.
b) Decreased serum potassium level
c) Increased serum calcium level
d) Increased serum creatinine level
103 Which set of arterial blood gas (ABG) C)pH 7.49, PaCO2 30 mm Hg, PaO2 89 mm Hg, and HCO3-
. results requires further investigation? 18 mEq/LThe ABG results pH 7.49, PaCO2 30 mm Hg,
a) pH 7.35, PaCO2 40 mm Hg, PaO2 91 mm PaO2 89 mm Hg, and HCO3- 18 mEq/L indicate respiratory
Hg, and HCO3- 22 mEq/L alkalosis.
b) pH 7.44, PaCO2 43 mm Hg, PaO2 99 mm The pH level is increased, and the HCO3- and PaCO2 levels
Hg, and HCO3- 26 mEq/L are decreased. Normal values are pH 7.35 to 7.45; PaCO2 35
c) pH 7.49, PaCO2 30 mm Hg, PaO2 89 mm to 45 mm Hg; HCO3- 22 to 26 mEq/L.
Hg, and HCO3- 18 mEq/L
d) pH 7.38, partial pressure of arterial
carbon dioxide (PaCO2) 36 mm Hg, partial
pressure of arterial oxygen (PaO2) 95 mm
Hg, bicarbonate (HCO3-) 24 mEq/L
104 Which type of incontinency refers to D) Iatrogenic
. the involuntary loss of urine due to Explanation: Iatrogenic incontinence is the involuntary loss of
medications? urine due to medications. Reflex incontinence is the involuntary
a) Overflow loss of urine due to hyperreflexia in the absence of normal
b) Urge sensations usually associated with voiding. Urge incontinence is
c) Reflex the involuntary loss of urine associated with a strong urge to void
d) Iatrogenic that cannot be suppressed. Overflow incontinence is the
involuntary loss of urine associated with overdistention of the
bladder.
105 Which type of medication may be A) Anticholinergic agent
. used in the treatment of a patient with Anticholinergic agents are considered first-line medications for
incontinence to inhibit contraction of urge incontinence. Estrogen decreases obstruction to urine flow by
the bladder? restoring the mucosal, vascular, and muscular integrity of the
a) Anticholinergic agent urethra. Tricyclic antidepressants decrease bladder contractions as
b) Over-the-counter decongestant well as increase bladder neck resistance. Stress incontinence may
c) Tricyclic antidepressants be treated using pseudoephedrine and phenylpropanolamine,
d) Estrogen hormone ingredients found in over-the-counter decongestants.
106 While caring for a 77-year old woman c) vague abdominal pain and disorientation
. who has a urinary catheter, the nurse The usual classic symptoms of UTI are often absent in older adults,
monitors the patient for the who tend to experience nonlocalized abdominal pain rather than
development of a UTI. The clinical dysuria and suprapubic pain. They may also experience cognitive
manifestations the patient is most impairment characterized by confusion or decreased level of
likely to experience include: consciousness.
a) cloudy urine and fever
b) urethral burning and blood urine
c) vague abdominal pain and
disorientation
d) suprapubic pain and slight decline
in body temperature
1 A woman with no hx of UTIs who is d) be treated empirically with TMP-SMX Bactrium for 3 days
0 experiencing urgency, frequency, and Unless a patient has a hx of recurrent UTIs, TMP-SMX or
7 dysuria comes to the clinic, where a nitrofurantoin is usually used to empirically treat an initial UTI
. dipstick and microscopic urinalysis without a culture and sensitivity test. Asymptomatic bacteriuria does
indicate bacteriuria. The nurse not justify tc, but symptomatic UTIs should always be treated.
anticipates that the patient will
a) need to have a blood specimen
drawn for a CBC and kidney function
test
b) not be treated with medications
unless she develops a fever, chills, or
flank pain
c) be requested to obtain a clean-catch
midstream urine specimen for culture
and sensitivity
d) be treated empirically with TMP-
SMX Bactrim for 3 days.
1 You are caring for a 72-year-old client B) Dehydration
0 who has been admitted to your unit for The most common fluid imbalance in older adults is dehydration.
8 a fluid volume imbalance. You know Because of reduced thirst sensation that often accompanies aging,
. which of the following is the most older adults tend to drink less water. Use of diuretic medications,
common fluid imbalance in older laxatives, or enemas may also deplete fluid volume in older adults.
adults? Chronic fluid volume deficit can lead to other problems such as
a) Hypovolemia electrolyte imbalances. Therefore, options A, C, and D are incorrect.
b) Dehydration
c) Hypervolemia
d) Fluid volume excess
After an 2.
intraveno After an IVP, the nurse should encourage fluids to decrease the risk of renal
us complications caused by the contrast agent. There is no need to place the client on bed
pyelogra rest or administer a laxative. An IVP would not cause hematuria.
m (IVP),
the nurse
should
anticipate
incorpora
ting
which of
the
following
measures
into the
client's
plan of
care?
1.
Maintaini
ng bed
rest.
2.
Encourag
ing
adequate
fluid
intake.
3.
Assessing
for
hematuri
a.
4.
Administe
ring a
laxative.
2. Allopurinol 2.
(Zyloprim), Allopurinol (Zyloprim) is used to treat renal calculi composed of uric
200 mg/ acid. Adverse effects of allopurinol include drowsiness, maculopapular
day, is rash, anemia, abdominal pain, nausea, vomiting, and bone marrow
prescribed depression. Clients should be instructed to report rashes and unusual
for the bleeding or bruising. Retinopathy, nasal congestion, and dizziness are not
client with adverse effects of allopurinol.
renal calculi
to take at
home. The
nurse
should
teach the
client about
which of the
following
adverse
effects of
this
medication?

1.
Retinopathy
.
2.
Maculopap
ular rash.
3. Nasal
congestion.
4. Dizziness.
3 Because a client's 1.
. renal stone was found Because a high-purine diet contributes to the formation of uric acid, a low-purine diet
to be composed of uric is advocated. An alkaline-ash diet is also advocated because uric acid crystals are more
acid, a low-purine, likely to develop in acid urine. Foods that may be eaten as desired in a low-purine diet
alkaline-ash diet was include milk, all fruits, tomatoes, cereals, and corn. Foods allowed on an alkaline-ash
ordered. diet include milk, fruits (except cranberries, plums, and prunes), and vegetables
Incorporation of (especially legumes and green vegetables). Gravy, chicken, and liver are high in
which of the following purine.
food items into the
home diet would
indicate that the client
understands the
necessary diet
modifications?
1. Milk, apples,
tomatoes, and corn.
2. Eggs, spinach, dried
peas, and gravy.
3. Salmon, chicken,
caviar, and asparagus.
4. Grapes, corn,
cereals, and liver.
4 A client has a ureteral 2.
. catheter in place after The ureteral catheter should drain freely without bleeding at the site. The catheter is
renal surgery. A rarely irrigated, and any irrigation would be done by the physician. The catheter is
priority nursing action never clamped. The client's total urine output (ureteral catheter plus voiding or
for care of the ureteral indwelling urinary catheter output) should be 30 mL/ hour.
catheter would be to:
1. Irrigate the catheter
with 30 mL of normal
saline every 8 hours.
2. Ensure that the
catheter is draining
freely.
3. Clamp the catheter
every 2 hours for 30
minutes.
4. Ensure that the
catheter drains at least
30 mL/ hour.
5 A client has been 1, 2, 5.
. prescribed Common adverse effects of allopurinol (Zyloprim) include gastrointestinal distress, such
allopurinol as anorexia, nausea, vomiting, and diarrhea. A rash is another potential adverse effect. A
(Zyloprim) for renal potentially life-threatening adverse effect is bone marrow depression. Constipation and
calculi that are flushed skin are not associated with this drug.
caused by high uric
acid levels. Which
of the following
indicate the client is
experiencing
adverse effect( s) of
this drug? Select all
that apply.
1. Nausea.
2. Rash.
3. Constipation.
4. Flushed skin.
5. Bone marrow
depression.
6 A client has renal 4.
. colic due to renal If infection or blockage caused by calculi is present, a client can experience sudden
lithiasis. What is the severe pain in the flank area, known as renal colic. Pain from a kidney stone is considered
nurse's first priority an emergency situation and requires analgesic intervention. Withholding fluids will make
in managing care urine more concentrated and stones more difficult to pass naturally. Forcing large
for this client? quantities of fluid may cause hydronephrosis if urine is prevented from flowing past
1. Do not allow the calculi. Straining urine for small stones is important, but does not take priority over pain
client to ingest management.
fluids.
2. Encourage the
client to drink at
least 500 mL of
water each hour.
3. Request the
central supply
department to send
supplies for
straining urine.
4. Administer an
opioid analgesic as
prescribed.
7 A client is admitted to the hospital with a 3.
. diagnosis of renal calculi. The client is The priority nursing goal for this client is to alleviate the pain,
experiencing severe flank pain and which can be excruciating. Prevention of urinary tract
nausea; the temperature is 100.6 F (38.1 complications and alleviation of nausea are appropriate
C). Which of the following would be a throughout the client's hospitalization, but relief of the severe
priority outcome for this client? pain is a priority. The client is at little risk for fluid and
1. Prevention of urinary tract electrolyte imbalance.
complications.
2. Alleviation of nausea.
3. Alleviation of pain.
4. Maintenance of fluid and electrolyte
balance.
8 The client is scheduled for an intravenous 3.
. pyelogram (IVP) to determine the location A client scheduled for an IVP should be assessed for allergies to
of the renal calculi. Which of the following iodine and shellfish. Clients with such allergies may be allergic
measures would be most important for the to the IVP dye and be at risk for an anaphylactic reaction.
nurse to include in pretest preparation? Adequate fluid intake is important after the examination.
1. Ensuring adequate fluid intake on the Bladder spasms are not common during an IVP. Bowel
day of the test. preparation is important before an IVP to allow visualization of
2. Preparing the client for the possibility of the ureters and bladder, but checking for allergies is most
bladder spasms during the test. important.
3. Checking the client's history for allergy
to iodine.
4. Determining when the client last had a
bowel movement.
9. The client is scheduled to have a 4.
kidney, ureter, and bladder (KUB) A KUB radiographic examination ordinarily requires no preparation.
radiograph. To prepare the client for It is usually done while the client lies supine and does not involve the
this procedure, the nurse should use of radiopaque substances.
explain to the client that:
1. Fluid and food will be withheld the
morning of the examination.
2. A tranquilizer will be given before
the examination.
3. An enema will be given before the
examination.
4. No special preparation is required
for the examination.
10 A client who has been diagnosed with 2.
. renal calculi reports that the pain is Intermittent pain that is less colicky indicates that the calculi may be
intermittent and less colicky. Which moving along the urinary tract. Fluids should be encouraged to
of the following nursing actions is promote movement, and the urine should be strained to detect
most important at this time? passage of the stone. Hematuria is to be expected from the irritation
1. Report hematuria to the of the stone. Analgesics should be administered when the client
physician. needs them, not routinely. Moist heat to the flank area is helpful
2. Strain the urine carefully. when renal colic occurs, but it is less necessary as pain is lessened.
3. Administer meperidine (Demerol)
every 3 hours.
4. Apply warm compresses to the
flank area.
1 A client with a history of renal 1.
1 calculi formation is being A high daily fluid intake is essential for all clients who are at risk for
. discharged after surgery to remove calculi formation because it prevents urinary stasis and concentration,
the calculus. What instructions which can cause crystallization. Depending on the composition of the
should the nurse include in the stone, the client also may be instructed to institute specific dietary
client's discharge teaching plan? measures aimed at preventing stone formation. Clients may need to limit
1. Increase daily fluid intake to at purine, calcium, or oxalate. Urine may need to be either alkaline or acid.
least 2 to 3 L. There is no need to strain urine regularly.
2. Strain urine at home regularly.
3. Eliminate dairy products from
the diet.
4. Follow measures to alkalinize the
urine.
1 In addition to nausea and severe 2.
2 flank pain, a female client with The pain associated with renal colic due to calculi is commonly referred
. renal calculi has pain in the groin to the groin and bladder in female clients and to the testicles in male
and bladder. The nurse should clients. Nausea, vomiting, abdominal cramping, and diarrhea may also
assess the client further for signs be present. Nephritis or urine retention is an unlikely cause of the
of: referred pain. The type of pain described in this situation is unlikely to
1. Nephritis. be caused by additional stone formation.
2. Referred pain.
3. Urine retention.
4. Additional stone formation.
Acute When evaluating a client for complications of acute pancreatitis, the nurse would observe
pancreatiti for:
s can
cause
decreased
urine
output,
which
results
from the
renal
failure
that
sometimes
accompani
es this
condition
2. Albumin A client with liver and renal failure has severe ascites. On initial shift rounds,
is an his primary nurse finds his indwelling urinary catheter collection bag too full to
abnorma store more urine. The nurse empties more than 2,000 ml from the collection
l finding bag. One hour later, she finds the collection bag full again. The nurse notifies
in a the physician, who suspects that a bladder rupture is allowing the drainage of
routine peritoneal fluid. The physician orders a urinalysis to be obtained immediately.
urine If the physician's suspicion is correct, the urine will abnormally contain
specime
n
3. alcohol The nurse is teaching a group of middle-aged men about peptic ulcers. When
abuse discussing risk factors for peptic ulcers, the nurse should mention:
and
smoking.
4. Alteratio Alterations in hepatic blood flow resulting from a drug interaction also can
ns in affect:
hepatic
blood
flow
resulting
from a
drug
interacti
on can
affect
metaboli
sm and
excretion
5. assess The physician orders a Bernstein test (which is performed by inserting a
for acid nasogastric [NG] tube and aspirating gastric contents) for a client who
perfusio complains of chest pain. When teaching the client about this test, the nurse
n of the explains that it's done to:
esophage
al
mucosa.
6. Atrophy A client, age 82, is admitted to an acute care facility for treatment of an acute
of the flare-up of a chronic GI condition. In addition to assessing the client for
gastric complications of the current illness, the nurse monitors for age-related changes
mucosa in the GI tract. Which age-related change increases the risk of anemia?
7 Atropine decreases One hour before a client is to undergo abdominal surgery, the physician orders atropine,
. salivation and 0.6 mg I.M. The client asks the nurse why this drug must be administered. How should
gastric secretions the nurse respond?
8 Avoid aspirin and A client with a peptic ulcer is about to begin a therapeutic regimen that includes a bland
. products that diet, antacids, and famotidine (Pepcid). Before the client is discharged, the nurse should
contain aspirin." provide which instruction?
9 Avoid coffee and To prevent gastroesophageal reflux in a client with hiatal hernia, the nurse should
. alcoholic beverages." provide which discharge instruction?
1 The client diagnosed Which infection control equipment is necessary for the client diagnosed with
0 with Clostridium Clostridium difficile diarrhea?
. difficile diarrhea
requires contact
isolation. Contact
isolation precautions
require the use of
glove and a gown if
soiling is likely.
1 The client has the The nurse is administering medications to a client diagnosed with hepatitis B. When the
1 right to refuse any nurse hands the client his medications, the client says, "I would rather not take that pill
. medical treatment, or any others. I know there is no cure for hepatitis B." The nurse recognizes that the
regardless of the client is expressing feelings of hopelessness about his diagnosis. Which response by the
consequences. The nurse respects the client's rights concerning medication administration?
client is displaying
hopelessness over the
diagnosis; therefore,
the nurse should
encourage the client
to discuss these
feelings using
therapeutic
conversation.
1 The client is free A client with dysphagia is being prepared for discharge. Which outcome indicates that
2 from esophagitis and the client is ready for discharge?
. achalasia.
1 A client who experiences The nurse is caring for a client who underwent a subtotal gastrectomy. To
3 dumping syndrome after a manage dumping syndrome, the nurse should advise the client to:
. subtotal gastrectomy should
be advised to (Ans. ingest
liquids between meals)
rather than with meals.
Taking fluids between meals
allows for adequate
hydration, reduces the
amount of bulk ingested with
meals, and aids in the
prevention of rapid gastric
emptying.
1 A client who's dizzy and A 68-year-old male is being admitted to the hospital with abdominal pain,
4 anemic is at risk for injury anemia, and bloody stools. He complains of feeling weak and dizzy. He has
. because of his weakened rectal pressure and needs to urinate and move his bowels. The nurse should help
state. Assisting him with the him:
bedpan would best meet his
needs at this time without
risking his safety. The client
may fall if walking to the
bathroom, left alone to
urinate, or trying to stand
up.
1 collect the specimen in a The physician orders a stool culture to help diagnose a client with prolonged
5 sterile container. diarrhea. The nurse who obtains the stool specimen should:
.

1 Corticosteroid therapy and A 28-year-old client is admitted with inflammatory bowel syndrome (Crohn's
6 Antidiarrheal medications disease). Which therapies should the nurse expect to be part of the care plan?
.

1 Cyanosis, coughing, and Which findings are common in neonates born with esophageal atresia?
7 choking occur when fluid
. from the blind pouch is
aspirated into the trachea
1 Daily weight measurement is The physician prescribes spironolactone (Aldactone), 50 mg by mouth four
8 the most accurate indicator times daily, for a client with fluid retention caused by cirrhosis. Which finding
. of fluid status; a loss of 2.2 lb indicates that the drug is producing a therapeutic effect?
(1 kg) indicates loss of 1 L of
fluid. Because spironolactone
is a diuretic,
19 decrease the intestinal bacteria count. For a client who must undergo colon surgery, the physician orders
. preoperative cleansing enemas and neomycin sulfate (Mycifradin).
The rationale for neomycin use in this client is to:
20 destroys the odor-proof seal. The nurse is caring for a client with a colostomy. The client tells the
. nurse that he makes small pin holes in the drainage bag to help
relieve gas. The nurse should teach him that this action:
21 Diarrhea causes a bicarbonate deficit. In a client with enteritis and frequent diarrhea, the nurse should
. With loss of the relative alkalinity of anticipate an acid-base imbalance of:
the lower GI tract, the relative acidity
of the upper GI tract predominates
leading to (Ans.metabolic acidosis).
22 docusate sodium (Colace) Which medication should the nurse expect to administer to a client
. with constipation?
23 Duodenal ulcers are more common in A client is undergoing an extensive diagnostic workup for a
. people with type O blood, suggesting a suspected GI problem. The nurse discovers that the client has a
genetic basis family history of ulcer disease. Which blood type also is a risk
factor for duodenal ulcers?
24 Evaluate the client's understanding of As the nurse completes the admission assessment of a client
. the procedure admitted for gastric bypass surgery, the client states, "Finally! I'll be
thin and able to eat without much concern." How should the nurse
intervene?
25 The Hemovac drain isn't compressed; The physician calls the nurse for an update on his client who
. instead it's fully expanded. underwent abdominal surgery five hours ago. The physician asks
the nurse for the total amount of drainage collected in the Hemovac
since surgery. The nurse reports that according to documentation,
no drainage has been recorded. When the nurse finishes on the
telephone, she goes to assess the client. Which assessment finding
explains the absence of drainage?
26 A hepatic disorder, such as cirrhosis, may disrupt the liver's The nurse is caring for a client with
. normal use of vitamin K to produce prothrombin (a clotting cirrhosis. Which assessment findings
factor). Consequently, the nurse should monitor the client for indicate that the client has deficient
signs of bleeding, including purpura and petechiae. vitamin K absorption caused by this
hepatic disease?
27 Hepatic encephalopathy, a major complication of advanced For a client with cirrhosis, deterioration
. cirrhosis, occurs when the liver no longer can convert ammonia of hepatic function is best indicated by:
(a by-product of protein breakdown) into glutamine. This leads
to an increased blood level of ammonia a central nervous
system toxin which causes a decrease in the level of
consciousness. Fatigue, muscle weakness, nausea, anorexia, and
weight gain occur during the early stages of cirrhosis. Ans.
difficulty in arousal.
28 The hepatitis A virus is transmitted by the fecal-oral route, The nurse is developing a care plan for a
. primarily through ingestion of contaminated food or liquids. client with hepatitis A. What is the main
route of transmission of this hepatitis
virus?
29 hyperglycemia. A client with severe inflammatory bowel
. disease is receiving total parenteral
nutrition (TPN). When administering
TPN, the nurse must take care to
maintain the prescribed flow rate because
giving TPN too rapidly may cause:
30 Hypovolemic shock from fluid shifts is a major factor in acute Which condition is most likely to have a
. pancreatitis. nursing diagnosis of fluid volume deficit?
3 If abdominal distention is accompanied by A client had a nephrectomy 2 days ago and is now
1 nausea, the nurse must first auscultate bowel complaining of abdominal pressure and nausea. The first
. sounds nursing action should be to:
3 inhibits contraction of the bile duct and A client with cholecystitis is receiving propantheline
2 gallbladder. bromide. The client is given this medication because it:
.

3 Irritability and drowsiness When caring for a client with hepatitis B, the nurse should
3 monitor closely for the development of which finding
. associated with a decrease in hepatic function?
3 It has a short duration of action. A client takes 30 ml of magnesium hydroxide and
4 aluminum hydroxide with simethicone (Maalox TC) P.O.
. 1 hour and 3 hours after each meal and at bedtime for
treatment of a duodenal ulcer. Why does the client take
this antacid so frequently?
3 maintaining fluid balance. When planning care for a client with a small-bowel
5 obstruction, the nurse should consider the primary goal to
. be:
3 metallic taste. A client with amebiasis, an intestinal infection, is
6 prescribed metronidazole (Flagyl). When teaching the
. client about adverse reactions to this drug, the nurse
should mention:
3 Monitoring the client's weight every day A client with pancreatitis has been receiving total
7 parenteral nutrition (TPN) for the past week. Which
. nursing intervention helps determine if TPN is providing
adequate nutrition?
3 Nothing by mouth The nurse is caring for a client with active upper GI
8 bleeding. What is the appropriate diet for this client
. during the first 24 hours after admission?
3 Obstruction of the appendix A client with severe abdominal pain is being evaluated for
9 appendicitis. What is the most common cause of
. appendicitis?
4 Paregoric starts to act within 1 hour after A client with acute diarrhea is prescribed paregoric, 5 ml
0 administration. Onset of action isn't as rapid as 5 by mouth up to four times daily, until acute diarrhea
. or 20 minutes or as slow as 2 to 4 hours. subsides. The client asks the nurse how soon the
medication will start to work after the first dose is taken.
How should the nurse respond?
4 Positioning the client on the side with the knees When caring for a client with acute pancreatitis, the nurse
1 flexed should use which comfort measure?
.

4 The predominant clinical feature of acute A client is admitted to the health care facility with
2 pancreatitis is abdominal pain, which usually abdominal pain, a low-grade fever, abdominal distention,
. reaches peak intensity several hours after onset of and weight loss. The physician diagnoses acute
the illness. Therefore, relieving abdominal pain is pancreatitis. What is the primary goal of nursing care for
the nurse's primary goal this client?
4 A serum potassium level of 3 mEq/L is below A client with nausea, vomiting, and abdominal cramps
3 normal, indicating hypokalemia. Because and distention is admitted to the health care facility.
. hypokalemia may cause cardiac arrhythmias and Which test result is most significant?
asystole, it's the most significant finding.
4 Take long, slow breaths A client with an esophageal stricture is about to undergo
4 esophageal dilatation. As the bougies are passed down the
. esophagus, the nurse should instruct the client to do
which of the following to minimize the vomiting urge?
4 This drug should be injected into a large muscle A client is scheduled to undergo a left hemicolectomy for
5 mass. colorectal cancer. The physician prescribes phenobarbital
. (Luminal), 100 mg I.M. 60 minutes before surgery, for
sedation. Which statement accurately describes
administration of phenobarbital?
4 Ulcerative colitis A client comes to the emergency department complaining
6 of acute GI distress. When obtaining the client's history,
. the nurse inquires about the family history. Which
disorder has a familial basis?
4 vitamin B12. A client is admitted to the hospital with an exacerbation
7 of his chronic gastritis. When assessing his nutritional
. status, the nurse should expect a deficiency in:
4 Withholding all oral intake, as ordered, to Which nursing action is most appropriate for a client
8 decrease pancreatic secretions hospitalized with acute pancreatitis?
.

4. phenazopyridine (Pyridium)
A 25-year-old Phenazopyridine may be prescribed in conjunction with an antibiotic for painful
female client bladder infections to promote comfort. Because of its local anesthetic action on the
seeks care for urinary mucosa, phenazopyridine specifically relieves bladder pain. Nitrofurantoin is
a possible a urinary antiseptic with no analgesic properties. Although ibuprofen and
infection. Her acetaminophen with codeine are analgesics, they don't exert a direct effect on the
symptoms urinary mucosa.
include
burning on
urination and
frequent,
urgent
voiding of
small
amounts of
urine. She's
placed on
trimethoprim
-
sulfamethoxa
zole
(Bactrim) to
treat possible
infection.
Another
medication is
prescribed to
decrease the
pain and
frequency.
Which is the
most likely
medication
prescribed
for the pain?

1.
nitrofurantoi
n
(Macrodantin
)
2. ibuprofen
(Motrin)
3.
acetaminophe
n with
codeine
4.
phenazopyrid
ine
(Pyridium)
2. A 28-year- 4. Corticosteroid therapy
old client 5. Antidiarrheal medications
is Corticosteroids, such as prednisone, reduce the signs and symptoms of
admitted diarrhea, pain, and bleeding by decreasing inflammation.
with Antidiarrheals, such as diphenoxylate (Lomotil), combat diarrhea by
inflammat decreasing peristalsis. Lactulose is used to treat chronic constipation
ory bowel and would aggravate the symptoms. A high-fiber diet and milk and
syndrome milk products are contraindicated in clients with Crohn's disease
(Crohn's because they may promote diarrhea.
disease).
Which
therapies
should the
nurse
expect to
be part of
the care
plan?

1.
Lactulose
therapy
2. High-
fiber diet
3. High-
protein
milkshake
s
4.
Corticoste
roid
therapy
5.
Antidiarr
heal
medicatio
ns
3. A 32-year- 1. surgery.
old male The client should be prepared for surgery because his signs and
client with symptoms indicate bowel perforation. Appendicitis is the most
appendicit common cause of bowel perforation in the United States. Because
is is perforation can lead to peritonitis and sepsis, surgery wouldn't be
experienci delayed to perform any other intervention. Also, none of the other
ng procedures are necessary at this point.
excruciati
ng
abdominal
pain. An
abdominal
X-ray film
reveals
intraperit
oneal air.
The nurse
should
prepare
the client
for:

1. surgery.
2.
colonosco
py.
3.
nasogastri
c tube
insertion.
4. barium
enema.
4. A 37-year- 3. Support the client in changing the ostomy appliance, but realize he
old man may not like it.
with Teaching the client to inspect the stoma daily does not directly address
ulcerative the client's self-image.
colitis has Providing privacy is appropriate unless the client gives permission for
a new the family to observe and they will be participating in the care of the
ileostomy. stoma.
To Answer 3 is correct because this behavior conveys to the client that
promote a the nurse understands and accepts some of his feelings about the new
positive ostomy.
self-image If the nurse expresses distaste, verbally or nonverbally, this reinforces
for the the distastefulness of the ostomy to the client.
client, the
nurse will:

1. Teach
the client
to inspect
the stoma
daily.
2. Invite
the
client's
wife and
two sons
to observe
while the
client
changes
the
ostomy
appliance.
3. Support
the client
in
changing
the
ostomy
appliance,
but realize
he may
not like it.
4.
Acknowle
dge the
presence
of odor
during
ostomy
appliance
changes
by holding
her
breath.
5. A 68-year- 3. onto the bedpan.
old male is A client who's dizzy and anemic is at risk for injury because of his
being weakened state. Assisting him with the bedpan would best meet his
admitted needs at this time without risking his safety. The client may fall if
to the walking to the bathroom, left alone to urinate, or trying to stand up.
hospital
with
abdominal
pain,
anemia,
and
bloody
stools. He
complains
of feeling
weak and
dizzy. He
has rectal
pressure
and needs
to urinate
and move
his
bowels.
The nurse
should
help him:

1. to the
bathroom.

2. to the
bedside
commode.

3. onto the
bedpan.
4. to a
standing
position so
he can
urinate.
6 A 72-year-old client 2. Decreased abdominal strength
. seeks help for chronic Decreased abdominal strength, muscle tone of the intestinal wall, and motility all
constipation. This is a contribute to chronic constipation in the elderly. A decrease in hydrochloric acid
common problem for causes a decrease in absorption of iron and B12, whereas an increase in intestinal
elderly clients due to bacteria actually causes diarrhea.
several factors related to
aging. Which is one such
factor?

1. Increased intestinal
motility
2. Decreased abdominal
strength
3. Increased intestinal
bacteria
4. Decreased production
of hydrochloric acid
7 A 75-year-old client with 1. Blood urea nitrogen
. renal insufficiency is Blood urea nitrogen and creatinine levels should be closely monitored to detect
admitted to the hospital elevations caused by nephrotoxicity. Sodium level should be routinely monitored in
with pneumonia. He's all hospitalized clients. Alkaline phosphatase helps evaluate liver function. The WBC
being treated with count should be monitored to evaluate the effectiveness of the antibiotic; it doesn't
gentamicin help evaluate kidney function.
(Garamycin), which can
be nephrotoxic. Which
laboratory value should
be closely monitored?

1. Blood urea nitrogen


2. Sodium level
3. Alkaline phosphatase
4. White blood cell
(WBC) count
8 An adult is admitted 4. High pitched bowel sounds in the right quadrants; hypoactive bowel sounds in the
. with probable large left quadrants.
bowel obstruction. The
nurse auscultates the
abdomen. Which
finding, if present, is
consistent with an
intestinal obstruction?

1. Hyperactive bowel
sounds in all 4
quadrants.
2. Hypoactive bowel
sounds in all 4
quadrants.
3. Coffee grounds
emesis and tarry stools.
4. High pitched bowel
sounds in the right
quadrants; hypoactive
bowel sounds in the left
quadrants.
9 An adult male is 2. Using a scrotal support.
. admitted to the due to scrotal edema
emergency department
with a strangulated
inguinal hernia.
Emergency surgery that
involves reduction of the
hernia, resection of a
portion of the bowel,
and repair of the
abdominal wall is
performed. What is
most likely to be
included in the
immediate
postoperative care?

1. Encouraging him to
cough and deep breathe
every 2 hours.
2. Using a scrotal
support.
3. Frequently offering
him oral fluids.
4. Inserting an
indwelling urinary
catheter.
1 After admission for 2. 15 to 30 minutes
0 acute appendicitis, a Orally administered pentazocine has an onset of action of 15 to 30 minutes, reaches
. client undergoes an peak concentration in less than 1 hour, and has a duration of 3 to 4 hours.
appendectomy. He
complains of moderate
postsurgical pain for
which the physician
prescribes pentazocine
(Talwin), 50 mg by
mouth every 4 hours.
How soon after
administration of this
drug can the nurse
expect the client to feel
relief?

1. Less than 15 minutes


2. 15 to 30 minutes
3. 30 to 60 minutes
4. 1 to 2 hours
11. After checking the client's chart for possible 3. A monoamine oxidase (MAO) inhibitor
contraindications, the nurse is administering MAO inhibitors increase the effects of meperidine
meperidine (Demerol), 50 mg I.M., to a client with and can cause rigidity, hypotension, and excitation.
pain after an appendectomy. Which type of drug The client shouldn't receive meperidine within 14
therapy would contraindicate the use of days after administration of an MAO inhibitor.
meperidine? Antibiotics, antiemetics, and loop diuretics don't
cause significant drug interactions when administered
1. An antibiotic concurrently with meperidine.
2. An antiemetic
3. A monoamine oxidase (MAO) inhibitor
4. A loop diuretic
12 After having transurethral resection of the prostate 2. The client reports bladder spasms and the urge to
. (TURP), a client returns to the unit with a three- void.
way indwelling urinary catheter and continuous Reports of bladder spasms and the urge to void
closed bladder irrigation. Which finding suggests suggest that a blood clot may be occluding the
that the client's catheter is occluded? catheter. After TURP, urine normally appears red to
pink, and normal saline irrigant usually is infused at a
1. The urine in the drainage bag appears red to rate of 40 to 60 drops/min or according to facility
pink. protocol. The amount of returned fluid (1,200 ml)
2. The client reports bladder spasms and the urge to should correspond to the amount of instilled fluid,
void. plus the client's urine output (1,000 ml + 200 ml),
3. The normal saline irrigant is infusing at a rate of which reflects catheter patency.
50 drops/min.
4. About 1,000 ml of irrigant have been instilled;
1,200 ml of drainage have been returned.
13 After undergoing a total cystectomy and urinary 1. "I'll have to wear an external collection pouch for
. diversion, a client has a Kock pouch (continent the rest of my life."
internal reservoir). Which statement by the client An internal collection pouch, such as the Kock
indicates a need for further teaching? pouch, allows the client to perform self-
catheterization for ileal drainage. This pouch is an
1. "I'll have to wear an external collection pouch for internal reservoir, eliminating the need for an external
the rest of my life." collection pouch. A well-balanced diet is essential for
2. "I should eat foods from all the food groups." healing; the client need not include or exclude
3. "I'll need to drink at least eight glasses of water a particular foods. The client should drink at least eight
day." glasses of fluid daily to prevent calculi formation and
4. "I'll have to catheterize my pouch every 2 urinary tract infection (UTI). Intervals between pouch
hours." drainings should be increased gradually until the
pouch is emptied two to four times daily.
14 After undergoing renal arteriogram, in which the 1. Assess peripheral pulses in the left leg.
. left groin was accessed, the client complains of left The nurse should begin by assessing peripheral pulses
calf pain. Which intervention should the nurse in the left leg to determine if blood flow was
perform first? interrupted by the procedure. The client may also
have thrombophlebitis. Cool compresses aren't used
1. Assess peripheral pulses in the left leg. to relieve pain and inflammation in thrombophlebitis.
2. Place cool compresses on the calf. The leg should remain straight after the procedure.
3. Exercise the leg and foot. Calf pain isn't a symptom of anaphylaxis.
4. Assess for anaphylaxis.
15 After undergoing retropubic prostatectomy, a client 3. It's an abnormal finding that requires further
. returns to his room. The client is on nothing-by- assessment.
mouth status and has an I.V. infusing in his right The drop in urine output to less than 30 ml/hour is
forearm at a rate of 100 ml/hour. The client also has abnormal and requires further assessment. The
an indwelling urinary catheter that is draining light reduction in urine output may be caused by an
pink urine. While assessing the client, the nurse obstruction in the urinary catheter tubing or deficient
notes that his urine output is red and has dropped fluid volume from blood loss. The client's nothing-
to 15 ml and 10 ml for the last two consecutive by-mouth status isn't the cause of the low urine
hours. How can the nurse best explain this drop in output because the client is receiving I.V. fluid to
urine output? compensate for the lack of oral intake. Ambulation
promotes urination; however, the client should
1. It's a normal finding caused by blood loss during produce at least 30 ml of urine/hour.
surgery.
2. It's a normal finding associated with the client's
nothing-by-mouth status.
3. It's an abnormal finding that requires further
assessment.
4. It's an abnormal finding that will correct itself
when the client ambulates.
16 After undergoing transurethral resection of 3. Assess the irrigation catheter for patency and drainage.
. the prostate to treat benign prostatic Although postoperative pain is expected, the nurse should
hyperplasia, a client returns to the room make sure that other factors, such as an obstructed irrigation
with continuous bladder irrigation. On the catheter, aren't the cause of the pain. After assessing catheter
first day after surgery, the client reports patency, the nurse should administer an analgesic, such as
bladder pain. What should the nurse do morphine sulfate, as prescribed. Increasing the I.V. flow rate
first? may worsen the pain. Notifying the physician isn't necessary
unless the pain is severe or unrelieved by the prescribed
1. Increase the I.V. flow rate. medication.
2. Notify the physician immediately.
3. Assess the irrigation catheter for patency
and drainage.
4. Administer morphine sulfate, 2 mg I.V.,
as prescribed.
17 Appendicitis Inflammation of the appendix. When the appendix becomes
. inflamed or infected, rupture may occur within a matter of
hours, leading to peritonitis and sepsis.
18 Assessment finding for chronic a. Frequently diagnosed incidentally when a client is being
. pyelonephritis evaluated for hypertension
b. Poor urine-concentrating ability
c. Pyuria
d. Azotemia
e. Proteinuria
19 Assessment findings for acute a. Fever and chills
. pyelonephritis b. Nausea
c. Flank pain on the affected side
d. Costovertebral angle tenderness
e. Headache
f. Dysuria
g. Frequency and urgency
h. Cloudy, bloody, or foul-smelling urine
i. Increased white blood cells in the urine
20 Assessment findings for appendicitis 1. Pain in the periumbilical area that descends to the right
. lower quadrant
2. Abdominal pain that is most intense at McBurney's point
3. Rebound tenderness and abdominal rigidity
4. Low-grade fever
5. Elevated white blood cell count
6. Anorexia, nausea, and vomiting
7. Client in side-lying position, with abdominal guarding and
legs flexed
8. Constipation or diarrhea
21 Assessment findings for BPH 1. Diminished size and force of urinary stream (early sign of
. BPH)
2. Urinary urgency and frequency
3. Nocturia
4. Inability to start (hesitancy) or continue a urinary stream
5. Feelings of incomplete bladder emptying
6. Postvoid dribbling from overflow incontinence (later sign)
7. Urinary retention and bladder distention
8. Hematuria
9. Urinary stasis
10. Dysuria and bladder pain
11. UTIs
22 Assessment findings for 1. Fever
. Chron's disease 2. Cramp-like and colicky pain after meals
3. Diarrhea (semisolid), which may contain mucus and pus (non-bloody)
4. Abdominal distention
5. Anorexia, nausea, and vomiting
6. Weight loss
7. Anemia
8. Dehydration
9. Electrolyte imbalances
10. Malnutrition (may be worse than that seen in ulcerative colitis)
23 Assessment findings for 1. Blood in stool (most common manifestation)
. colon cancer 2. Anorexia, vomiting, and weight loss
3. Anemia
4. Abnormal stools a. Ascending colon tumor: Diarrhea b. Descending colon
tumor: Constipation or some diarrhea, or flat, ribbon-like stool caused by a
partial obstruction c. Rectal tumor: Alternating constipation and diarrhea
5. Guarding or abdominal distention, abdominal mass (late sign)
6. Cachexia (late sign)
7. Masses noted on barium enema, colonoscopy, CT scan, sigmoidoscopy
24 Assessment findings for 1. Left lower quadrant abdominal pain that increases with coughing, straining, or
. diverticulitis lifting
2. Elevated temperature
3. Nausea and vomiting
4. Flatulence
5. Cramp-like pain
6. Abdominal distention and tenderness
7. Palpable, tender rectal mass may be present.
8. Blood in the stools
25 Assessment findings for 1. Gross hematuria
. glomerulonephritis 2. Dark, smoky, cola-colored or red-brown urine
3. Proteinuria that produces a persistent and excessive foam in the urine
4. Urinary debris
5. Moderately elevated to high urine specific gravity
6. Low urinary pH 7. Urinalysis shows large numbers of erythrocytes
8. Oliguria or anuria
9. Headache
10. Chills and fever
11. Fatigue and weakness
12. Anorexia, nausea, and vomiting
13. Pallor
14. Edema in the face, periorbital area, feet, or generalized
15. Shortness of breath, ascites, pleural effusion, and CHF
16. Abdominal or flank pain
17. Hypertension
18. Reduced visual acuity
19. Increased blood urea nitrogen and serum creatinine levels
20. Increased antistreptolysin O titer (used to diagnose disorders caused by
streptococcal infections)
26 Assessment findings for 1. Bright red bleeding with defecation
. hemorrhoids 2. Rectal pain
3. Rectal itching
27 Assessment findings for nausea
. intestinal obstruction abdominal distention
obstipation
high pitched bowel sounds
temp > 100

SBO
projectile vomiting
pain relieved with vomiting

LBO
persistent colicky pain
no vomiting
orange brown stool
28 Assessment findings for 1. Renal colic, which originates in the lumbar region and radiates around
. nephrolithiasis the side and down to the testicles in men and to the bladder in women
2. Ureteral colic, which radiates toward the genitalia and thighs
3. Sharp, severe pain of sudden onset
4. Dull, aching pain in the kidney
5. Nausea and vomiting, pallor, and diaphoresis during acute pain
6. Urinary frequency, with alternating retention
7. Signs of a urinary tract infection
8. Low-grade fever
9. High numbers of red blood cells, white blood cells, and bacteria noted in
the urinalysis report
10. Gross hematuria
29 Assessment findings for 1. Often asymptomatic until the ages of 30 to 40 years
. polycystic kidney disease 2. Flank, lumbar , or abdominal pain that worsens with activity and is
relieved when lying
3. Fever and chills
4. Recurrent urinary tract infections
5. Hematuria, proteinuria, pyuria
6. Calculi
7. Hypertension
8. Palpable abdominal masses and enlarged kidneys
30 Assessment findings for 1. Anorexia
. ulcerative colitis 2. Weight loss
3. Malaise
4. Abdominal tenderness and cramping
5. Severe diarrhea that may contain blood and mucus
6. Malnutrition, dehydration, and electrolyte imbalances 7. Anemia
8. Vitamin K deficiency
31 Assessment findings for UTIs 1. Frequency and urgency
. 2. Burning on urination
3. Voiding in small amounts
4. Inability to void
5. Incomplete emptying of the bladder
6. Lower abdominal discomfort or back discomfort
7. Cloudy, dark, foul-smelling urine
8. Hematuria
9. Bladder spasms
10. Malaise, chills, fever
11. Nausea and vomiting
12. WBC count greater than 100,000 cells/mm3 on urinalysis
13. An elevated specific gravity and pH may be noted on urinalysis.
32 Because of difficulties with 2. White blood cell (WBC) count of 20,000/mm3
. hemodialysis, peritoneal dialysis is An increased WBC count indicates infection, probably resulting from
initiated to treat a client's uremia. peritonitis, which may have been caused by insertion of the peritoneal
Which finding signals a significant catheter into the peritoneal cavity. Peritonitis can cause the peritoneal
problem during this procedure? membrane to lose its ability to filter solutes; therefore, peritoneal
dialysis would no longer be a treatment option for this client.
1. Blood glucose level of 200 mg/dl Hyperglycemia occurs during peritoneal dialysis because of the high
2. White blood cell (WBC) count of glucose content of the dialysate; it's readily treatable with sliding-scale
20,000/mm3 insulin. A potassium level of 3.5 mEq/L can be treated by adding
3. Potassium level of 3.5 mEq/L potassium to the dialysate solution. An HCT of 35% is lower than
4. Hematocrit (HCT) of 35% normal. However, in this client, the value isn't abnormally low because
of the daily blood samplings. A lower HCT is common in clients with
chronic renal failure because of the lack of erythropoietin.
33 Benign prostatic hypertrophy a slow enlargement of the prostate gland, with hypertrophy and
. (benign prostatic hyperplasia; hyperplasia of normal tissue. Enlargement compresses the urethra,
BPH) resulting in partial or complete obstruction. Usually occurs in men
older than 50 years.
34 Causes for cystectomy with cancer of bladder
. urinary diversion neurogenic bladder
congenital anomalies
trauma
chronic infection
35 Causes of constipation chronic disorders - IBS, diverticular
. drug induced - antacids, antidepressants, anticholingergics, barium
sulfate, iron
endocrine - hypothyroidism, diabetes
scleroderma
neurogenic - megacolon, MS, parkinson's
36 Causes of diarrhea decreased fluid absorption - laxative abuse, mucosal damage (Chron's,
. radiation, colitis, ischemic bowel dis.)
increased fluid secretion - infectious bacteria endotoxins, antibiotics,
foods w/ sorbitol, hormonal, adenoma of pancreas
motility disturbances - IBS, gastrectomy
37 Causes of fecal incontinence motor - muscle contraction
. sensory - dementia, stroke, spinal cord injury, degenerative disease
38 Causes of mechanical intestinal adhesions following surgery (50%)
. obstruction hernia (15%)
cancer (15%)
volvulus
diverticular disease
39 Causes of non-mechanical neuromuscular
. intestinal obstruction vascular disorders (emboli to mesenteric artery)
post abdominal surgery
inflammatory response (pancreatitis, appendicitis)
electrolyte imbalance
spinal fracture
40 Causes of peritonitis blood born infection
. cirrhosis of the liver
perforation/rupture of bowel/appendix
pancreatitis
peritoneal dialysis
abdominal surgery
41 Chron's disease An inflammatory disease that can occur anywhere in the
. gastrointestinal tract but most often affects the terminal
ileum and leads to thickening and scarring, a narrowed
lumen, fistulas, ulcerations, and abscesses. Characterized
by remissions and exacerbations.
42 A client, age 82, is admitted to an acute care 1. Atrophy of the gastric mucosa
. facility for treatment of an acute flare-up of a Atrophy of the gastric mucosa reduces hydrochloric acid
chronic GI condition. In addition to assessing secretion; this, in turn, impairs absorption of iron and
the client for complications of the current illness, vitamin B12, increasing the risk of anemia as a person
the nurse monitors for age-related changes in ages. A decrease in hydrochloric acid increases, not
the GI tract. Which age-related change increases decreases, intestinal flora; as a result, the client is at
the risk of anemia? increased risk for infection, not anemia. A reduction, not
increase, in bile secretion may lead to malabsorption of
1. Atrophy of the gastric mucosa fats and fat-soluble vitamins. Dulling of nerve impulses
2. Decrease in intestinal flora associated with aging increases the risk of constipation,
3. Increase in bile secretion not anemia.
4. Dulling of nerve impulses
43 The client arrives at the emergency department 3. Trauma to the bladder or abdomen
. with complaints of low abdominal pain and Bladder trauma or injury should be considered or
hematuria. The client is afebrile. The nurse next suspected in the client with low abdominal pain and
assesses the client to determine a history of: hematuria. Glomerulonephritis and pyelonephritis would
be accompanied by fever and are thus not applicable to
1. Pyelonephritis the client described in this question. Renal cancer would
2. Glomerulonephritis not cause pain that is felt in the low abdomen; rather pain
3. Trauma to the bladder or abdomen would be in the flank area.
4. Renal cancer in the client's family
44 A client asks a nurse how soon after bowel 1. "By 72 hours you should start to pass gas."
. surgery normal bowel function will return. The The bowel should be functioning by 72 hours.
best response by the nurse would be: There may be faint bowel sounds at 48 hours but the
bowel will not be fully functional.
1. "By 72 hours you should start to pass gas." At 12 hours the effects of general anesthesia on the
2. Around 48 hours, if there are no bowel are still present.
complications."
3. "Some function will return by 12 hours."
4. "You'll pass gas by 24 hours."
45 A client comes to the emergency department 3. Ulcerative colitis
. complaining of acute GI distress. When Ulcerative colitis is more common in people who have
obtaining the client's history, the nurse inquires family members with the disease. (The same is true of
about the family history. Which disorder has a some types of GI cancers, ulcers, and Crohn's disease.)
familial basis? Hepatitis, iron deficiency anemia, and chronic peritonitis
are acquired disorders that don't run in families.
1. Hepatitis
2. Iron deficiency anemia
3. Ulcerative colitis
4. Chronic peritonitis
46 A client comes to the emergency department 1. Acute pain
. complaining of severe pain in the right flank, Ureterolithiasis typically causes such acute, severe
nausea, and vomiting. The physician tentatively pain that the client can't rest and becomes increasingly
diagnoses right ureterolithiasis (renal calculi). anxious. Therefore, the nursing diagnosis of Acute
When planning this client's care, the nurse should pain takes highest priority. Diagnoses of Risk for
assign highest priority to which nursing diagnosis? infection and Impaired urinary elimination are
appropriate when the client's pain is controlled. A
1. Acute pain diagnosis of Imbalanced nutrition: Less than body
2. Risk for infection requirements isn't pertinent at this time.
3. Impaired urinary elimination
4. Imbalanced nutrition: Less than body
requirements
47 A client comes to the emergency department 1. Kidney
. complaining of sudden onset of sharp, severe pain The most common site of renal calculi formation is
in the lumbar region, which radiates around the the kidney. Calculi may travel down the urinary tract
side and toward the bladder. The client also reports with or without causing damage and may lodge
nausea and vomiting and appears pale, diaphoretic, anywhere along the tract or may stay within the
and anxious. The physician tentatively diagnoses kidney. The ureter, bladder, and urethra are less
renal calculi and orders flat-plate abdominal X- common sites of renal calculi formation.
rays. Renal calculi can form anywhere in the
urinary tract. What is their most common
formation site?

1. Kidney
2. Ureter
3. Bladder
4. Urethra
48 A client complains of not having had a bowel 3. Ensure maximum fluid intake (3,000mL/day)
. movement since being admitted 2 days previously Enema may not be necessary and requires a doctor's
for multiple fractures of both lower legs. The client order.
is on bedrest and skeletal traction. Which The bedpan requires a great deal of exertion when the
intervention would be the most appropriate client is not expressing the urge to defecate.
nursing action? The best early intervention is to increase fluid intake
b/ constipation is common when activity is decreased
1. Administer an enema. or usual routines have been interrupted.
2. Put the client on the bedpan every 2 hours. It would be impossible to exercise extremities that
3. Ensure maximum fluid intake (3,000mL/day) have unhealed fractures.
4. Perform range-of-motion exercises to all
extremities.
49 A client develops decreased renal function and 3. Creatinine clearance
. requires a change in antibiotic dosage. On which The physician orders tests for creatinine clearance to
factor would the physician base the dosage gauge the kidney's glomerular filtration rate; this is
change? important because most drugs are excreted at least
partially by the kidneys. The GI absorption rate,
1. GI absorption rate therapeutic index, and liver function studies don't help
2. Therapeutic index determine dosage change in a client with decreased
3. Creatinine clearance renal function.
4. Liver function studies
50 A client had a nephrectomy 2 days ago and is now 1. auscultate bowel sounds.
. complaining of abdominal pressure and nausea. If abdominal distention is accompanied by nausea, the
The first nursing action should be to: nurse must first auscultate bowel sounds. If bowel
sounds are absent, the nurse should suspect gastric or
1. auscultate bowel sounds. small intestine dilation and these findings must be
2. palpate the abdomen. reported to the physician. Palpation should be avoided
3. change the client's position. postoperatively with abdominal distention. If
4. insert a rectal tube. peristalsis is absent, changing positions and inserting
a rectal tube won't relieve the client's discomfort.
51 The client had a new colostomy created 2 1. This is a normal, expected event.
. days earlier and is beginning to pass As peristalsis returns following creation of a colostomy, the
malodorous flatus from the stoma. What is client begins to pass malodorous flatus. This indicates
the correct interpretation by the nurse? returning bowel function and is an expected event. Within 72
hours of surgery, the client should begin passing stool via the
1. This is a normal, expected event. colostomy. Options 2, 3, and 4 are incorrect.
2. The client is experiencing early signs of
ischemic bowel.
3. The client should not have the nasogastric
tube removed.
4. This indicates inadequate preoperative
bowel preparation.
52 A client had a positive fecal occult blood test 1. Orthostatic blood pressure and pulse.
. during a health screening. The nursing 3. Description of stool volume, color, and consistency.
assessment would include: 4. Diet history of raw meat consumption.
Select all the apply. 5. Regular use of aspirin or NSAIDs.
6. Vitamin C, 250-mg tablets, before the test.
1. Orthostatic blood pressure and pulse.
2. STAT request for hemoglobin and Inadequate volume from a significant blood loss will result
hematocrit. in a drop of the systolic blood pressure greater than 25 mm
3. Description of stool volume, color, and Hg and the diastolic value greater than 20 mm Hg, and an
consistency. increase in pulse rate of 30 bpm when the client goes from
4. Diet history of raw meat consumption. flat to stitting/standing.
5. Regular use of aspirin or NSAIDs. Ordering lab tests is not part of nursing assessment.
6. Vitamin C, 250-mg tablets, before the test. Nursing assessment would include a description of the
characteristics of the stool.
Eating red meat before the test may result in a false positive
finding.
It should be known if the client has a history of frequent and
high doses of drugs known to cause GI irritation and
bleeding.
Taking Vitamin C within 48 hours of a fecal occult blood test
will cause a false positive reading.
53 A client has a colostomy following surgery 3. Complaint of pain with light touch.
. for colon cancer. Which assessment finding The stoma will be 1 to 2 cm above the skin.
would require immediate action by a nurse? Some bleeding is normal.
There are no nerves in the mucous membranes, there should
1. Stoma raised 2 cm from the abdominal be no pain when touching. Pain would possibly indicate a
wall. problem internally.
2. Bleeding was noted from the stoma during
care.
3. Complaint of pain with light touch.
4. Stoma was pinkish-red in color.
54 A client has a newly created colostomy. After 4. The client touches the altered body part.
. participating in counseling with the nurse By touching the altered body part, the client recognizes the
and receiving support from the spouse, the body change and establishes that the change is real. Closing
client decides to change the colostomy pouch the eyes, not looking at the abdomen when the colostomy is
unaided. Which behavior suggests that the exposed, or avoiding talking about the surgery reflects
client is beginning to accept the change in denial, instead of acceptance of the change. Asking the
body image? spouse to leave the room signifies that the client is ashamed
of the change and not coping with it.
1. The client closes the eyes when the
abdomen is exposed.
2. The client avoids talking about the recent
surgery.
3. The client asks the spouse to leave the
room.
4. The client touches the altered body part.
55 The client has a NG tube connected to low 2. Irrigate the tube with 30 mL of water.
. continuous suction for abdominal Problem solving should be done first. Inserting a new NG
decompression. The nurse notes that gastric tube would cause the client unnecessary discomfort.
fluid in the suction tubing is not moving and The mos likely cause of the problem is that the NG tube is
the client's abdomen is becoming distended. plugged with gastric contents of has adhered to the gastric
The nurse's best action is to: mucosa and is no longer draining. Irrigating the tube should
clear any obstructions and free the tube from the gastric
1. Pull out the NG tube and insert a new mucosa.
one. Having the client take a few deep breaths will not solve the
2. Irrigate the tube with 30 mL of water. problem.
3. Tell the client to take a few deep breaths. Turning the suction higher may cause additional trauma to
4. Turn the suction higher. the gastric mucosa.
56 A client has just had a hemorrhoidectomy. 3. Apply and maintain ice packs over the dressing until the
. What nursing intervention is appropriate packing is removed.
for this client? Nursing interventions after a hemorrhoidectomy are aimed at
management of pain and avoidance of bleeding. An ice pack
1. Instruct the client to limit fluid intake to will increase comfort and decrease bleeding. Options 1, 2,
avoid urinary retention. and 4 are incorrect interventions.
2. Instruct the client to eat low-fiber foods to
decrease the bulk of the stool.
3. Apply and maintain ice packs over the
dressing until the packing is removed.
4. Help the client to a Fowler's position to
place pressure on the rectal area and
decrease bleeding.
57 The client has just had surgery to create an 4. Fluid and electrolyte imbalance
. ileostomy. The nurse assesses the client in A frequent complication that occurs following ileostomy is
the immediate postoperative period for fluid and electrolyte imbalance. The client requires constant
which most frequent complication of this monitoring of intake and output to prevent this from
type of surgery? occurring. Losses require replacement by intravenous
infusion until the client can tolerate a diet orally. Intestinal
1. Folate deficiency obstruction is a less frequent complication. Fat malabsorption
2. Malabsorption of fat and folate deficiency are complications that could occur later
3. Intestinal obstruction in the postoperative period.
4. Fluid and electrolyte imbalance
58 A client has undergone a colon resection. 2. place saline-soaked sterile dressings on the wound.
. While turning him, wound dehiscence with The nurse should first place saline-soaked sterile dressings on
evisceration occurs. The nurse's first the open wound to prevent tissue drying and possible
response is to: infection. Then the nurse should call the physician and take
the client's vital signs. The dehiscence needs to be surgically
1. call the physician. closed, so the nurse should never try to close it.
2. place saline-soaked sterile dressings on
the wound.
3. take a blood pressure and pulse.
4. pull the dehiscence closed.
59 A client in the short-procedure unit is recovering 3. check the client's pedal pulses frequently.
. from renal angiography in which a femoral After renal angiography involving a femoral puncture
puncture site was used. When providing site, the nurse should check the client's pedal pulses
postprocedure care, the nurse should: frequently to detect reduced circulation to the feet
caused by vascular injury. The nurse also should
1. keep the client's knee on the affected side bent monitor vital signs for evidence of internal
for 6 hours. hemorrhage and should observe the puncture site
2. apply pressure to the puncture site for 30 frequently for fresh bleeding. The client should be
minutes. kept on bed rest for several hours so the puncture site
3. check the client's pedal pulses frequently. can seal completely. Keeping the client's knee bent is
4. remove the dressing on the puncture site after unnecessary. By the time the client returns to the
vital signs stabilize. short-procedure unit, manual pressure over the
puncture site is no longer needed because a pressure
dressing is in place. The nurse shouldn't remove this
dressing for several hours and only if instructed to
do so.
60 A client is admitted for treatment of 1. generalized edema, especially of the face and
. glomerulonephritis. On initial assessment, the periorbital area.
nurse detects one of the classic signs of acute Generalized edema, especially of the face and
glomerulonephritis of sudden onset. Such signs periorbital area, is a classic sign of acute
include: glomerulonephritis of sudden onset. Other classic
signs and symptoms of this disorder include
1. generalized edema, especially of the face and hematuria (not green-tinged urine), proteinuria, fever,
periorbital area. chills, weakness, pallor, anorexia, nausea, and
2. green-tinged urine. vomiting. The client also may have moderate to
3. moderate to severe hypotension. severe hypertension (not hypotension), oliguria or
4. polyuria. anuria (not polyuria), headache, reduced visual acuity,
and abdominal or flank pain.
61 The client is admitted to the emergency department 2. Shoulder
. following a motor vehicle accident. The client was Bladder trauma or injury is characterized by lower
wearing a lap seat belt when the accident occurred abdominal pain that may radiate to one of the
and now the client has hematuria and lower shoulders due to phrenic nerve irritation. Bladder
abdominal pain. To assess further whether the pain injury pain does not radiate to the umbilicus,
is caused by bladder trauma, the nurse asks the costovertebral angle, or hip.
client if the pain is referred to which of the
following areas?

1. Hip
2. Shoulder
3. Umbilicus
4. Costovertebral angle
62 The client is admitted to the hospital with a 4. Blood pressure, 100/ 50 mm Hg; pulse, 130 beats/
. diagnosis of benign prostatic hyperplasia, and a min
transurethral resection of the prostate is Frank bleeding (arterial or venous) may occur during
performed. Four hours after surgery, the nurse the first day after surgery. Some hematuria is usual for
takes the client's vital signs and empties the urinary several days after surgery. A urinary output of 200 mL
drainage bag. Which of the following assessment more than intake is adequate. Bladder spasms are
findings would indicate the need to notify the expected to occur following surgery. A rapid pulse
physician? with a low blood pressure is a potential sign of
excessive blood loss. The physician should be
1. Red bloody urine notified.
2. Pain related to bladder spasms
3. Urinary output of 200 mL higher than intake
4. Blood pressure, 100/ 50 mm Hg; pulse, 130 beats/
min
63 A client is admitted with nausea, vomiting, and 4. Start I.V. fluids with a normal saline solution bolus
. diarrhea. His blood pressure on admission is followed by a maintenance dose.
74/30 mm Hg. The client is oliguric and his blood The client is in prerenal failure caused by hypovolemia.
urea nitrogen (BUN) and creatinine levels are I.V. fluids should be given with a bolus of normal saline
elevated. The physician will most likely write an solution followed by maintenance I.V. therapy. This
order for which treatment? treatment should rehydrate the client, causing his blood
pressure to rise, his urine output to increase, and the
1. Force oral fluids. BUN and creatinine levels to normalize. The client
2. Administer furosemide (Lasix) 20 mg I.V. wouldn't be able to tolerate oral fluids because of the
3. Start hemodialysis after a temporary access is nausea, vomiting, and diarrhea. The client isn't fluid-
obtained. overloaded so his urine output won't increase with
4. Start I.V. fluids with a normal saline solution furosemide, which would actually worsen the client's
bolus followed by a maintenance dose. condition. The client doesn't require dialysis because the
oliguria and elevated BUN and creatinine levels are
caused by dehydration.
64 A client is diagnosed with Crohn's disease after 2. "I appreciate your concern, but I can't give out any
. undergoing two weeks of testing. The client's information."
boss calls the medical-surgical floor requesting to The nurse may not release any confidential information
speak with the nurse manager. He expresses to unauthorized individuals, such as the client's boss.
concern over the client and explains that he must Options 1, 3, and 4 breech client confidentiality.
know the client's diagnosis for insurance
purposes. Which response by the nurse is best?

1. "Sure, I understand how demanding


insurance companies can be."
2. "I appreciate your concern, but I can't give
out any information."
3. "Why don't you come in, and we can further
discuss this issue."
4. "He has been diagnosed with Crohn's
Disease."
65 A client is frustrated and embarrassed by 4. Assessing present elimination patterns
. urinary incontinence. Which measure should the The guidelines for initiating bladder retraining include
nurse include in a bladder retraining program? assessing the client's intake patterns, voiding patterns,
and reasons for each accidental voiding. Lowering the
1. Establishing a predetermined fluid intake client's fluid intake won't reduce or prevent
pattern for the client incontinence. The client should actually be encouraged
2. Encouraging the client to increase the time to drink 1.5 to 2 L of water per day. A voiding schedule
between voidings should be established after assessment.
3. Restricting fluid intake to reduce the need to
void
4. Assessing present elimination patterns
66 The client is prescribed continuous bladder 1. Evaluating patency of the drainage lumen
. irrigation at a rate of 60 gtt/minute. The nurse The nurse should evaluate patency of the drainage
hangs a 2 L bag of sterile solution with tubing on tubing before leaving the client's room. If the lumen is
a three-legged I.V. pole. She then attaches the obstructed, the solution infuses into the bladder but isn't
tubing to the client's three-way urinary catheter, eliminated through the drainage tubing, a situation that
adjusts the flow rate, and leaves the room. Which may cause client injury. Balancing the pole is
important procedural step did the nurse fail to important; however, the nurse would have had to
follow? address this issue immediately after hanging the 2 L
bag. Using an I.V. pump isn't necessary for continuous
1. Evaluating patency of the drainage lumen bladder irrigation. Unless specifically ordered,
2. Counter-balancing the I.V. pole obtaining a urine specimen before beginning
3. Attaching the infusion set to an infusion pump continuous bladder irrigation isn't necessary.
4. Collecting a urine specimen before beginning
irrigation
67 A client is recovering from an ileostomy that was 1. increasing fluid intake to prevent dehydration.
. performed to treat inflammatory bowel disease. Because stool forms in the large intestine, an ileostomy
During discharge teaching, the nurse should stress typically drains liquid waste. To avoid fluid loss
the importance of: through ileostomy drainage, the nurse should instruct
the client to increase fluid intake. The nurse should
1. increasing fluid intake to prevent dehydration. teach the client to wear a collection appliance at all
2. wearing an appliance pouch only at bedtime. times because ileostomy drainage is incontinent, to
3. consuming a low-protein, high-fiber diet. avoid high-fiber foods because they may irritate the
4. taking only enteric-coated medications. intestines, and to avoid enteric-coated medications
because the body can't absorb them after an ileostomy.
68 A client is recovering from a small-bowel 2. 10 to 15 minutes
. resection. To relieve pain, the physician prescribes Meperidine's onset of action is 10 to 15 minutes. It
meperidine (Demerol), 75 mg I.M. every 4 hours. peaks between 30 and 60 minutes and has a duration of
How soon after administration should action of 2 to 4 hours.
meperidine's onset of action occur?

1. 5 to 10 minutes
2. 10 to 15 minutes
3. 30 to 60 minutes
4. 2 to 4 hours
69 A client is scheduled for a renal arteriogram. 3. Pruritus
. When the nurse checks the chart for allergies to The nurse should be alert for urticaria and pruritus,
shellfish or iodine, the nurse finds no allergies which may indicate a mild anaphylactic reaction to the
recorded. The client is unable to provide the arteriogram dye. Decreased (not increased) alertness
information. During the procedure, the nurse may occur as well as dyspnea (not hypoventilation).
should be alert for which finding that may Unusually smooth skin isn't a sign of anaphylaxis.
indicate an allergic reaction to the dye used
during the arteriogram.

1. Increased alertness
2. Hypoventilation
3. Pruritus
4. Unusually smooth skin
70 A client is scheduled for a renal clearance test. 1. 1 minute.
. The nurse should explain that this test is done to The renal clearance test determines the kidneys' ability
assess the kidneys' ability to remove a substance to remove a substance from the plasma in 1 minute. It
from the plasma in: doesn't measure the kidneys' ability to remove a
substance over a longer period.
1. 1 minute.
2. 30 minutes.
3. 1 hour.
4. 24 hours.
71 A client is scheduled for bowel resection with 2. Related to the presence of bacteria at the surgical
. anastomosis involving the large intestine. Because of site
the surgical site, the nurse formulates the nursing The large intestine normally contains bacteria
diagnosis of Risk for infection. To complete the because its alkaline environment permits growth of
nursing diagnosis statement, the nurse should add organisms that putrefy and break down remaining
which "related-to" phrase? proteins and indigestible residue. These organisms
include Escherichia coli, Aerobacter aerogenes,
1. Related to major surgery required by bowel Clostridium perfringens, and Lactobacillus. Although
resection bowel resection with anastomosis is considered
2. Related to the presence of bacteria at the surgical major surgery, it poses no greater risk of infection
site than any other type of major surgery. Malnutrition
3. Related to malnutrition secondary to bowel seldom follows bowel resection with anastomosis
resection with anastomosis because nutritional absorption (except for some
4. Related to the presence of a nasogastric (NG) tube water, sodium, and chloride) is completed in the
postoperatively small intestine. An NG tube is placed through a
natural opening, not a wound, and therefore doesn't
increase the client's risk of infection.
72 The client is scheduled for urinary diversion surgery 4. Client educator
. to treat bladder cancer. Before surgery, the health The nurse should consult the client educator to help
care team consisting of a nurse, dietician, social the client with his fears and concerns. Providing the
worker, enterostomal therapist, surgeon, client client with information can greatly allay the client's
educator, and mental health worker meet with the fears. The social worker can provide the client with
client. After the meeting, the client states, "My life services he may need after discharge. The dietician
won't ever be the same. What am I going to do?" can help with dietary concerns but can't provide help
Which health team member should the nurse with direct concerns about the surgery.
consult to help with the client's concerns?

1. Social worker
2. Surgeon
3. Dietician
4. Client educator
73 A client is scheduled to undergo a left 4. This drug should be injected into a large muscle
. hemicolectomy for colorectal cancer. The physician mass.
prescribes phenobarbital (Luminal), 100 mg I.M. 60 Phenobarbital should be injected into a large muscle
minutes before surgery, for sedation. Which mass. The onset of action by the I.M. route is 10 to
statement accurately describes administration of 30 minutes. Barbiturates are involved in many drug
phenobarbital? interactions, so the drug shouldn't be mixed or given
with other medications. The drug solution should be
1. The onset of action for I.M. administration is 30 used within 30 minutes after opening to minimize
to 60 minutes. deterioration.
2. This drug can be mixed and given with other
medications.
3. This drug should be used within 24 hours after
opening.
4. This drug should be injected into a large muscle
mass.
74 A client is scheduled to undergo an exploratory 4. Initiating intravenous therapy, as ordered
. laparoscopy. The registered nurse asks the licensed The registered nurse must confirm that the LPN has
practical nurse (LPN) to prepare the client for specialized I.V. training before asking her to begin
surgery. The registered nurse must confirm that the I.V. therapy for this client. Initiating I.V. therapy is
LPN has specialized training before delegating beyond the usual scope of practice for an LPN.
which task? Options 1, 2, and 3 are within the scope of LPN
practice and don't require additional training.

1. Weighing the client


2. Teaching the client coughing and deep breathing
exercises
3. Teaching the client how to collect a urine
specimen
4. Initiating intravenous therapy, as ordered
75 A client is scheduled to undergo surgical creation of 2. help the client cope with the anxiety associated
. an ileal conduit. The primary nurse educates the with changes in body image.
client about surgery and the postoperative period. Many clients who undergo surgery for creation of an
The nurse informs the client that many members of ileal conduit experience anxiety associated with
the health care team (including a mental health changes in body image. The mental health practitioner
practitioner) will see him. A mental health can help with client cope these feelings of anxiety.
practitioner should be involved in the client's care Mental health practitioners don't evaluate whether the
to: client is a surgical candidate. None of the evidence
suggests that urinary diversion surgery, such as
1. assess whether the client is a good candidate for creation of an ileal conduit, places the client at risk for
surgery. suicide. Although evaluating the need for mental
2. help the client cope with the anxiety associated health intervention is always important, this client
with changes in body image. displays no behavioral changes that suggest
3. assess suicidal risk postoperatively. intervention is necessary at this time.
4. evaluate the client's need for mental health
intervention.
76 A client receiving total parental nutrition is 2. start after a known voiding.
. prescribed a 24-hour urine test. When initiating a When initiating a 24-hour urine specimen, have the
24-hour urine specimen, the collection time should: client void, then start the timing. The collection
should start on an empty bladder. The exact time the
1. start with the first voiding. test starts isn't important but it's commonly started in
2. start after a known voiding. the morning.
3. always be with the first morning urine.
4. always be the evening's last void as the last
sample.
77 A client returns to the medical-surgical unit after 1. Dopamine (Intropin), 3 mcg/kg/min
. coronary artery bypass graft surgery, which was This client is at high risk for acute prerenal failure
complicated by prolonged cardiopulmonary bypass secondary to decreased renal perfusion during
and hypotension. After 3 hours in the unit, the surgery. To dilate the renal arteries and help prevent
client's condition stabilizes. However, the urine renal shutdown, the physician is likely to prescribe
output has decreased despite adequate filling dopamine at a low flow rate (2 to 5 mcg/kg/min).
pressures. The nurse expects the physician to add Although this drug has mixed dopaminergic and beta
which drug, at which flow rate, to the client's activity when given at 5 to 10 mcg/kg/min, the client
regimen? is stabilized and thus doesn't need the beta effects
from the higher flow rate or the sympathomimetic
1. Dopamine (Intropin), 3 mcg/kg/min effects of epinephrine. The dopaminergic effects of
2. Epinephrine, 2 mcg/kg/min dopamine increase renal perfusion, contractility, and
3. Dopamine (Intropin), 8 mcg/kg/min vasodilation. Stimulation of beta receptors causes beta
4. Epinephrine, 4 mcg/kg/min effects namely, increases in the heart rate,
myocardial contraction force, and cardiac conduction.
78 The client returns to the postanesthesia 2. Obtain an order for dextrose solution IV.
. care unit following GI surgery. It is too soon for any indication of wound infection. The slight
Postoperative laboratory values include: elevation in the WBC count is most likely related to the reason
O2 saturation 99%, WBC 11,500, Hct 35, for the GI surgery.
Na+ 139, K+ 3.9, glucose 51. The first The client's serum glucose level is dangerously low and will
priority of the nurse would be to: interfere with accurately assessing the level of consciousness as
the client is waking up from anesthesia. Glucose is needed for
1. Assess the postoperative wound for normal brain functioning. The lowest normal for serum glucose
infection. is 70 mg/dL.
2. Obtain an order for dextrose solution The blood is well saturated with oxygen.
IV. The potassium level is low normal.
3. Increase nasal O2 to improve oxygen
delivery.
4. Decrease amount of K+ in IV to avoid
arrhythmias.
79 A client undergoes extracorporeal shock 2. "Increase your fluid intake to 2 to 3 L per day."
. wave lithotripsy. Before discharge, the Increasing fluid intake flushes the renal calculi fragments
nurse should provide which instruction? through and prevents obstruction of the urinary system.
Measuring temperature every 4 hours isn't needed. Lithotripsy
1. "Take your temperature every 4 doesn't require an incision. Hematuria may occur for a few
hours." hours after lithotripsy but should then disappear.
2. "Increase your fluid intake to 2 to 3 L
per day."
3. "Apply an antibacterial dressing to the
incision daily."
4. "Be aware that your urine will be
cherry-red for 5 to 7 days."
80 The client understands post-ileostomy 2. "I empty my bag before it is half full."
. care when the client states: The bag should be emptied before it gets "too heavy."
Emptying the bag when it is about half full will prevent
1. "I empty my bag when it gets too problems with the bag falling off from the weight of the
heavy." effluent.
2. "I empty my bag before it is half full." Emptying the bag when it is only one-third full could be
3. "I empty my bag before it is one-third expensive because they will need to buy supplies more
full." frequently.
4. I don't need to wear a bag all the time." A client with an ileostomy needs to wear a bag all the time.
81 The client underwent a transurethral 4. Use aseptic technique when irrigating the catheter.
. resection of the prostate gland 24 hours If the catheter is blocked by blood clots, it may be irrigated
ago and is on continuous bladder according to physician's orders or facility protocol. The nurse
irrigation. Which nursing intervention is should use sterile technique to reduce the risk of infection.
appropriate? Urinating around the catheter can cause painful bladder spasms.
Encourage the client to drink fluids to dilute the urine and
1. Tell the client to try to urinate around maintain urine output. The catheter remains in place for 2 to 4
the catheter to remove blood clots. days after surgery and is only removed with a physician's order.
2. Restrict fluids to prevent the client's
bladder from becoming distended.
3. Prepare to remove the catheter.
4. Use aseptic technique when irrigating
the catheter.
82 A client who has been treated for diverticulitis is 3. 30 minutes before meals and at bedtime
. being discharged on oral propantheline bromide Propantheline bromide is used to reduce secretions and
(Pro-Banthine). The nurse should instruct the spasms of the GI tract in clients with diverticulitis, a
client to take the drug at which times? condition characterized by bowel inflammation and
colonic irritability and spasticity. The nurse should
1. With meals and at bedtime instruct the client to take the drug 30 minutes before
2. Immediately before meals and at bedtime meals and at bedtime to reduce GI motility, thus
3. 30 minutes before meals and at bedtime relieving spasticity. Taking it with a meal, immediately
4. 1 hour after meals and at bedtime before a meal, or 1 hour after a meal would interfere
with the drug's action and absorption, thereby reducing
its effectiveness.
83 A client who underwent abdominal surgery who 3. Assess patency of the NG tube.
. has a nasogastric (NG) tube in place begins to When an NG tube is no longer patent, stomach
complain of abdominal pain that he describes as contents collect in the stomach giving the client a
"feeling full and uncomfortable." Which sensation of fullness. The nurse should begin by
assessment should the nurse perform first? assessing patency of the NG tube. The nurse can
measure abdominal girth, auscultate bowels, and assess
1. Measure abdominal girth. vital signs, but she should check NG tube patency first
2. Auscultate bowel sounds. to help relieve the client's discomfort.
3. Assess patency of the NG tube.
4. Assess vital signs.
84 A client with acute diarrhea is prescribed 3. "Within 1 hour"
. paregoric, 5 ml by mouth up to four times daily, Paregoric starts to act within 1 hour after
until acute diarrhea subsides. The client asks the administration. Onset of action isn't as rapid as 5 or 20
nurse how soon the medication will start to work minutes or as slow as 2 to 4 hours.
after the first dose is taken. How should the nurse
respond?
1. "Within 5 minutes"
2. "Within 20 minutes"
3. "Within 1 hour"
4. "Within 2 to 4 hours"
85 A client with acute pyelonephritis receives a 3. Bacteria are absent on urine culture.Co-trimoxazole
. prescription for co-trimoxazole (Septra) P.O. twice is a sulfonamide antibiotic used to treat urinary tract
daily for 10 days. Which finding best demonstrates infections. Therefore, absence of bacteria on urine
that the client has followed the prescribed culture indicates that the drug has achieved its desired
regimen? effect. Although flank pain may decrease as the
infection resolves, this isn't a reliable indicator of the
1. Urine output increases to 2,000 ml/day. drug's effectiveness. Co-trimoxazole doesn't affect
2. Flank and abdominal discomfort decrease. urine output or the RBC count.
3. Bacteria are absent on urine culture.
4. The red blood cell (RBC) count is normal.
86 A client with a history of chronic cystitis comes to 4. Milk
. the outpatient clinic with signs and symptoms of A client on an acid-ash diet must avoid milk and milk
this disorder. To prevent cystitis from recurring, products because these make the urine more alkaline,
the nurse recommends maintaining an acid-ash encouraging bacterial growth. Other foods to avoid on
diet to acidify the urine, thereby decreasing the this diet include all vegetables except corn and lentils;
rate of bacterial multiplication. On an acid-ash all fruits except cranberries, plums, and prunes; and
diet, the client must restrict which beverage? any food containing large amounts of potassium,
sodium, calcium, or magnesium. Cranberry and prune
1. Cranberry juice juice are encouraged because they acidify the urine.
2. Coffee Coffee and tea are considered neutral because they
3. Prune juice don't alter the urine pH.
4. Milk
87 A client with a history of severe abdominal 1. Observing for signs and symptoms of shock.
. cramping arrives at the emergency department The client has experienced some gastric bleeding; the
vomiting a moderate amount of blood. The nurse's client should be observed closely for symptoms of
primary concern should be: shock.
Paging the physician is secondary to the close
1. Observing for signs and symptoms of shock. monitoring of the client's vital signs for symptoms of
2. Immediately paging the physician on call. shock.
3. Filling out the appropriate assessment tool. Completing the appropriate assessment tool may be
4. Immediately administering CPR. done later, when the client is under less stress.
CPR is not indicated, the client is not experiencing
cardiac or respiratory failure.
88 A client with amebiasis, an intestinal infection, is 1. metallic taste.
. prescribed metronidazole (Flagyl). When teaching Metronidazole commonly causes a metallic taste. Other
the client about adverse reactions to this drug, the adverse reactions include nausea, anorexia, headache,
nurse should mention: and dry mouth. The drug isn't associated with tinnitus,
blurred vision, or loss of smell.
1. metallic taste.
2. tinnitus.
3. blurred vision.
4. loss of smell.
89 The client with a new ileostomy has received 2. Baked chicken, mashed potato, cooked carrots, angel
. discharge teaching. Which dinner menu selected food cake.
by the client indicates understanding of an Whole grains, corn, peas, nuts, and raw vegetables are
appropriate diet for a new ileostomy? considered high-fiber foods and may cause blockages.
Garlic may produce odor.
1. Baked chicken, whole-grain biscuit, corn on the
cob, canned peaches.
2. Baked chicken, mashed potato, cooked carrots,
angel food cake.
3. Ham, mashed potato, salad with raw carrots,
canned peaches.
4. Roast beef, pasta with butter and garlic, split
pea soup, 2 chocolate chip cookies with walnuts.
90 A client with an indwelling urinary catheter is 2. wiping the self-sealing aspiration port with antiseptic
. suspected of having a urinary tract infection. solution and aspirating urine with a sterile needle.
The nurse should collect a urine specimen for Most catheters have a self-sealing port for obtaining a
culture and sensitivity by: urine specimen. Antiseptic solution is used to reduce the
risk of introducing microorganisms into the catheter.
1. disconnecting the tubing from the urinary Tubing shouldn't be disconnected from the urinary
catheter and letting the urine flow into a sterile catheter. Any break in the closed urine drainage system
container. may allow the entry of microorganisms. Urine in urine
2. wiping the self-sealing aspiration port with drainage bags may not be fresh and may contain
antiseptic solution and aspirating urine with a bacteria, giving false test results. When there is no urine
sterile needle. in the tubing, the catheter may be clamped for no more
3. draining urine from the drainage bag into a than 30 minutes to allow urine to collect.
sterile container.
4. clamping the tubing for 60 minutes and
inserting a sterile needle into the tubing above
the clamp to aspirate urine.
91 A client with a retroperitoneal abscess is 1. Hearing
. receiving gentamicin (Garamycin). Which signs 2. Urine output
should the nurse monitor? 4. Blood urea nitrogen (BUN) and creatinine levels
Adverse reactions to gentamicin include ototoxicity and
1. Hearing nephrotoxicity. The nurse must monitor the client's
2. Urine output hearing and instruct him to report any hearing loss or
3. Hematocrit (HCT) tinnitus. Signs of nephrotoxicity include decreased urine
4. Blood urea nitrogen (BUN) and creatinine output and elevated BUN and creatinine levels.
levels Gentamicin doesn't affect the serum calcium level or
5. Serum calcium level HCT.
92 A client with benign prostatic hyperplasia 1. Transurethral resection of the prostate (TURP)
. doesn't respond to medical treatment and is TURP is the most widely used procedure for prostate
admitted to the facility for prostate gland gland removal. Because it requires no incision, TURP is
removal. Before providing preoperative and especially suitable for men with relatively minor
postoperative instructions to the client, the nurse prostatic enlargements and for those who are poor
asks the surgeon which prostatectomy procedure surgical risks. Suprapubic prostatectomy, retropubic
will be done. What is the most widely used prostatectomy, and transurethral laser incision of the
procedure for prostate gland removal? prostate are less common procedures; they all require an
incision.
1. Transurethral resection of the prostate
(TURP)
2. Suprapubic prostatectomy
3. Retropubic prostatectomy
4. Transurethral laser incision of the prostate
93 A client with benign prostatic hypertrophy 3. "Herbal therapy such as saw palmetto may improve
. (BPH) asks a nurse what can be done to manage urinary flow."
the symptoms. The correct response by the nurse Dietary changes such as decreasing fats and increasing
would be: fruits may reduce symptoms. Reducing meat and dairy
intake decreases hormone stimulation.
1. "There is really nothing that can be done Zinc has been shown to delay the development of BPH,
except surgical removal of the prostate." but not reverse or manage the symptoms.
2. "Increase the amount of the zinc in your diet." Saw palmetto has been shown to improve urine flow bu
3. "Herbal therapy such as saw palmetto may blocking the ability of dihydrotestosterone (DHT) to
improve urinary flow." stimulate prostate cell growth.
4. "Vitamin E will reduce the size of the Vitamin E has not been shown to reduce the size.
prostate."
94 A client with bladder cancer has had the 2. The pouch faceplate doesn't fit the stoma.
. bladder removed and an ileal conduit If the pouch faceplate doesn't fit the stoma properly, the skin
created for urine diversion. While changing around the stoma will be exposed to continuous urine flow
this client's pouch, the nurse observes that from the stoma, causing excoriation and red, weeping, and
the area around the stoma is red, weeping, painful skin. A lubricant shouldn't be used because it would
and painful. What should the nurse prevent the pouch from adhering to the skin. When properly
conclude? applied, a skin barrier prevents skin excoriation. Stoma
dilation isn't performed with an ileal conduit, although it may
1. The skin wasn't lubricated before the be done with a colostomy if ordered.
pouch was applied.
2. The pouch faceplate doesn't fit the stoma.
3. A skin barrier was applied properly.
4. Stoma dilation wasn't performed.
95 A client with complaints of right, lower 2. White blood cell (WBC) count 22.8/mm3
. quadrant pain is admitted to the emergency The nurse should report the elevated WBC count, which is
department. Blood specimens are drawn evident in option 2. This finding, which is a sign of infection,
and sent to the laboratory. Which indicates that the client's appendix might have ruptured.
laboratory finding should be reported to the Hematocrit of 42%, serum potassium of 4.2 mEq/L, and
physician immediately? serum sodium of 135 mEq/L are within normal limits.
Alterations in these levels aren't indicative of appendicitis.
1. Hematocrit 42%
2. White blood cell (WBC) count 22.8/mm3
3. Serum potassium 4.2 mEq/L
4. Serum sodium 135 mEq/L
96 A client with heart failure admitted to an 4. Stress urinary incontinence
. acute care facility and is found to have a Stress urinary incontinence is a urinary problem associated
cystocele. When planning care for this client, with cystocele herniation of the bladder into the birth
the nurse is most likely to formulate which canal. Other problems associated with this disorder include
nursing diagnosis? urinary frequency, urinary urgency, urinary tract infection
(UTI), and difficulty emptying the bladder. Total
1. Total urinary incontinence incontinence, functional incontinence, and reflex
2. Functional urinary incontinence incontinence usually result from neurovascular dysfunction,
3. Reflex urinary incontinence not cystocele.
4. Stress urinary incontinence
97 A client with inflammatory bowel disease 1. Blood supply to the stoma has been interrupted.
. undergoes an ileostomy. On the first day An ileostomy stoma forms as the ileum is brought through
after surgery, the nurse notes that the the abdominal wall to the surface skin, creating an artificial
client's stoma appears dusky. How should opening for waste elimination. The stoma should appear
the nurse interpret this finding? cherry red, indicating adequate arterial perfusion. A dusky
stoma suggests decreased perfusion, which may result from
1. Blood supply to the stoma has been interruption of the stoma's blood supply and may lead to
interrupted. tissue damage or necrosis. A dusky stoma isn't a normal
2. This is a normal finding 1 day after finding. Adjusting the ostomy bag wouldn't affect stoma
surgery. color, which depends on blood supply to the area. An
3. The ostomy bag should be adjusted. intestinal obstruction also wouldn't change stoma color.
4. An intestinal obstruction has occurred.
98. A client with mild diarrhea, fever, and 1. Diarrhea is the most common sign of IBD.
abdominal discomfort is being evaluated IBD is a collective term for several GI inflammatory diseases
for inflammatory bowel disease (IBD). with unknown causes. The most prominent sign of IBD is mild
Which statement about IBD is true? diarrhea, which sometimes is accompanied by fever and
abdominal discomfort. The pathophysiology of ulcerative colitis
1. Diarrhea is the most common sign of involves vascular congestion, hemorrhage, and edema
IBD. usually affecting the rectum and left colon. Although abscesses
2. Transmural inflammation with fistula may occur in IBD, they result from buildup of lymphocytes and
formation occurs in ulcerative colitis, one cellular debris in crypts, which may serve as abscess sites. Only
form of IBD. about 3% of clients with a long history of Crohn's disease
3. Abscesses may occur in IBD as poor develop bowel cancer.
nutrition causes breakdown of cells in the
GI tract.
4. Bowel cancer is common in clients with
a history of Crohn's disease, one form of
IBD.
99. A client with nausea, vomiting, and 4. Serum potassium level of 3 mEq/L
abdominal cramps and distention is A serum potassium level of 3 mEq/L is below normal,
admitted to the health care facility. indicating hypokalemia. Because hypokalemia may cause
Which test result is most significant? cardiac arrhythmias and asystole, it's the most significant
finding. In a client with a potential fluid volume imbalance,
1. Blood urea nitrogen (BUN) level of 29 such as from vomiting, the other options are expected and none
mg/dl is as life-threatening as hypokalemia. A BUN level of 29 mg/dl
2. Serum sodium level of 132 mEq/L indicates slight dehydration, probably caused by vomiting. A
3. Urine specific gravity of 1.025 serum sodium level of 132 mEq/L is slightly below normal but
4. Serum potassium level of 3 mEq/L not life-threatening. A urine specific gravity of 1.025 is normal.
100 A client with renal dysfunction of acute 2. recent streptococcal infection.
. onset comes to the emergency A skin or upper respiratory infection of streptococcal origin
department complaining of fatigue, may lead to acute glomerulonephritis. Other infections that may
oliguria, and coffee-colored urine. When be linked to renal dysfunction include infectious
obtaining the client's history to check for mononucleosis, mumps, measles, and cytomegalovirus.
significant findings, the nurse should ask Chronic, excessive acetaminophen use isn't nephrotoxic,
about: although it may be hepatotoxic. Childhood asthma and a family
history of pernicious anemia aren't significant history findings
1. chronic, excessive acetaminophen use. for a client with renal dysfunction.
2. recent streptococcal infection.
3. childhood asthma.
4. family history of pernicious anemia.
101 A client with severe abdominal pain is 2. Obstruction of the appendix
. being evaluated for appendicitis. What is Appendicitis most commonly results from obstruction of the
the most common cause of appendicitis? appendix, which may lead to rupture. A high-fat diet or
duodenal ulcer doesn't cause appendicitis; however, a client
1. Rupture of the appendix may require dietary restrictions after an appendectomy.
2. Obstruction of the appendix
3. A high-fat diet
4. A duodenal ulcer
102 A client with severe inflammatory bowel 1. hyperglycemia.
. disease is receiving total parenteral Hyperglycemia may occur if TPN is administered too rapidly,
nutrition (TPN). When administering exceeding the client's glucose metabolism rate. With
TPN, the nurse must take care to hyperglycemia, the renal threshold for glucose reabsorption is
maintain the prescribed flow rate because exceeded and osmotic diuresis occurs, leading to dehydration
giving TPN too rapidly may cause: and electrolyte depletion. Although air embolism may occur
during TPN administration, this problem results from faulty
1. hyperglycemia. catheter insertion, not overly rapid administration. TPN may
2. air embolism. cause diarrhea, not constipation, especially if administered too
3. constipation. rapidly. Dumping syndrome results from food moving through
4. dumping syndrome. the GI tract too quickly; because TPN is given I.V., it can't
cause dumping syndrome.
103 A client with suspected renal insufficiency 3. The nephron
. is scheduled for a comprehensive The nephron is the functioning unit of the kidney. The
diagnostic workup. After the nurse glomerulus, Bowman's capsule, and tubular system are
explains the diagnostic tests, the client components of the nephron.
asks which part of the kidney "does the
work." Which answer is correct?

1. The glomerulus
2. Bowman's capsule
3. The nephron
4. The tubular system
104 A client with ulcerative colitis is receiving 2. Reduce peristaltic activity.
. methaneline bromide (Banthine). This
nurse knows the primary reason this
client is receiving methaneline bromide is
to:

1. Supress inflammation of the bowel.


2. Reduce peristaltic activity.
3. Neutralize acid in the gastrointestinal
tract.
4, Increase bowel tone.
105 Colostomy irrigation obtain colostomy irrigation set (cone tip)
. expel air from tubing
position client near toilet
instil 500-1,000 mL luke warm water
hold irrigation bag 18 - 24 inches above the stoma (shoulder
height)
lubricate cone and insert into stoma
5 - 10 min in
35 -45 min out
stop irrigation if sever cramping
106 Complications of bowel obstruction hypovolemic shock - due to necrosis and rupture of bowel -
. fluid shifts into peritoneum

fluid/electrolyte and acid/base imbalance - metabolic alkalosis


high in bowel - metabolic acidosis lower in bowel

dehydration - from vomiting

strangulation and gangrene of bowel


107 Complications of UTIs
. BPH Hydronephrosis
Pyelonephritis
108 Complications of scar tissue with narrowing & SBO
. Chron's disease fistulas between bowel and urinary tract
bowel perforation/peritonitis
fat malabsorption deficiencies (A,D,E,K)
gluten intolerance
cholelitiasis & nephrolithiasis (due to oxalate absorption)
109 Complications of valsalva maneuver & syncope
. constipation divertulosis
obstipation
bowel perforation
110. Complications of dehydration
diarrhea electrolyte imbalance
111. Complications of peritonitis
diverticulitis abscess & fistula formation
bowel obstruction
urethral obstruction
lower GI bleed
112. Complications of hypovolemic shock
peritonitis septicemia
paralytic ileus
organ failure
113. Complications of #1 hemorrhage
prostate surgery infection
bladder spasms
urinary retention
erectile dysfunction
114. Continuous A three-way (lumen) irrigation is used to decrease bleeding and to keep the bladder free
Bladder Irrigation from clotsone lumen is for inflating the balloon (30 mL); one lumen is for instillation
(inflow); one lumen is for outflow.
115. Creatinine The creatinine clearance test evaluates how well the kidneys remove creatinine from
clearance test the blood.
The test includes obtaining a blood sample and timed urine specimens. Blood is drawn
when the urine specimen collection is complete. The urine specimen for the creatinine
clearance is usually collected for 24 hours, but shorter periods such as 8 or 12 hours
could be prescribed.
116. Crohn's disease A chronic progressive disease of the liver characterized by diffuse degeneration and
destruction of hepatocytes. Repeated destruction of hepatic cells causes the formation
of scar tissue.
117. The day after colostomy surgery an adult 2. Say "It must be difficult to have this kind of surgery."
says to the nurse, "I know the doctor did not
really do a colostomy." The nurse
understands that he is in an early stage of
adjustment to his diagnosis and surgery.
What nursing action is indicated at this time?

1. Agree with him until he is ready to accept


his colostomy.
2. Say "It must be difficult to have this kind
of surgery."
3. Force him to look at his colostomy.
4. Ask the surgeon to explain the surgery to
the client.
118. Diagnostics for peritonitis CBC
serum electrolytes
x-ray, CT, ultrasound
paracentesis
119. Diagnostic tests for diarrhea blood (CBC, LFTs, iron/folate)
stool culture (WBCs, blood, ova & parasites)
endoscopy
GI series w/ barium swallow
120 Diet for fecal incontinence high fiber
. increased fluid
121 diverticulitis Inflammation of one or more diverticula from penetration
. of fecal matter through the thin-walled diverticula, resulting
in local abscess formation. A perforated diverticulum can
progress to intra-abdominal perforation with generalized
peritonitis.
122 diverticulosis Outpouching or herniations of the intestinal mucosa that
. can occur in any part of the intestine but is most common in
the sigmoid colon.
123 During a client-teaching session, which 4. "Drink 8 to 13 8-oz glasses (2 to 3 L) of fluid daily."
. instruction should the nurse give to a client The nurse should tell the client to drink 8 to 13 8-oz glasses
receiving kaolin and pectin (Kaopectate) for of fluid daily to replace fluids lost through diarrhea. Kaolin
treatment of diarrhea? and pectin mixtures should be taken after each loose bowel
movement for up to eight doses daily. The client should
1. "Take the medication after every other avoid self-medication for longer than 48 hours. The client
loose bowel movement, up to five doses per should consult a physician if diarrhea persists longer than
day." 48 hours despite treatment.
2. "Avoid taking this medication for more
than 4 days."
3. "Consult the physician if you have more
than six bowel movements in 1 day."
4. "Drink 8 to 13 8-oz glasses (2 to 3 L) of
fluid daily."
124 During clindamycin (Cleocin) therapy, the 4. Clostridium difficile
. nurse monitors a client for Pseudomembranous colitis may result from a
pseudomembranous colitis. This serious superinfection with C. difficile during clindamycin therapy.
adverse reaction to clindamycin results from Clindamycin-induced pseudomembranous colitis isn't
superinfection with which organism? caused by S. aureus, B. fragilis, or E. coli.

1. Staphylococcus aureus
2. Bacteroides fragilis
3. Escherichia coli
4. Clostridium difficile
125 During preparation for bowel surgery, a client 4. vitamin K
. receives an antibiotic to reduce intestinal bacteria. Intestinal bacteria synthesize such nutritional
Antibiotic therapy may interfere with synthesis of substances as vitamin K, thiamine, riboflavin,
which vitamin and may lead to vitamin B12, folic acid, biotin, and nicotinic acid.
hypoprothrombinemia? Therefore, antibiotic therapy may interfere with
synthesis of these substances, including vitamin K.
1. vitamin A Intestinal bacteria don't synthesize vitamins A, D,
2. vitamin D or E.
3. vitamin E
4. vitamin K
126 During rectal examination, which finding would be 2. The presence of a boggy mass
. further evidence of a urethral injury? When the urethra is ruptured, a hematoma or
collection of blood separates the two sections of
1. A low-riding prostate urethra. This may feel like a boggy mass on rectal
2. The presence of a boggy mass examination. Because of the rupture and
3. Absent sphincter tone hematoma, the prostate becomes high riding. A
4. A positive Hemoccult palpable prostate gland usually indicates a
nonurethral injury. Absent sphincter tone would
refer to a spinal cord injury. The presence of blood
would probably correlate with GI bleeding or a
colon injury.
127 An elderly client continues to have fecal incontinence 4. Fecal impaction.
. with 6 to 7 small brown liquid stools each day. The Inadequate roughage would not cause diarrhea.
client eats a soft diet and does not receive any stool Fecal impaction is likely causing diarrhea. Fiber
softeners or laxatives. The client's primary form of can be added to a soft diet to prevent constipation.
activity is sitting in the wheelchair for 2 hours twice a Inactivity contributes to the potential for
day. What is the correct explanation for the frequent constipation and fecal impaction, but the
diarrhea stools? impaction is causing the liquid stools.
Pressure on the colonic mucosa causes seepage of
1. Inadequate roughage in the diet. liquid stool around the area of impaction.
2. Inactivity from sedentary lifestyle.
3. Gastrointestinal virus.
4. Fecal impaction.
128 Extracorporeal shock wave lithotripsy (ESWL) A noninvasive mechanical procedure for breaking
. up stones located in the kidney or upper ureter so
that they can pass spontaneously or be removed by
other methods No incision is made and no drains
are placed; a stent may be placed to facilitate
passing stone fragments. Fluoroscopy is used to
visualize the stone and ultrasonic waves are
delivered to the area of the stone to disintegrate it.
The stones are passed in the urine within a few
days.
129 The female client is admitted to the emergency 1. Notify the physician.
. department following a fall from a horse and the The presence of blood at the urinary meatus may
physician prescribes insertion of a Foley catheter. indicate urethral trauma or disruption . The nurse
While preparing for the procedure, the nurse notes notifies the physician, knowing that the client
blood at the urinary meatus. The nurse should: should not be catheterized until the cause of the
bleeding is determined by diagnostic testing.
1. Notify the physician. Therefore options 2, 3, and 4 are incorrect.
2. Use a small-sized of catheter.
3. Administer pain medication before inserting the
catheter.
4. Use extra povidone-iodine solution in cleansing the
meatus.
130 A female client reports to the nurse that she 3. stress incontinence.
. experiences a loss of urine when she jogs. The Stress incontinence is a small loss of urine with activities
nurse's assessment reveals no nocturia, that increase intra-abdominal pressure, such as running,
burning, discomfort when voiding, or urine laughing, sneezing, jumping, coughing, and bending.
leakage before reaching the bathroom. The These symptoms occur only in the daytime. Functional
nurse explains to the client that this type of incontinence is the inability of a usually continent client to
problem is called: reach the toilet in time to avoid unintentional loss of urine.
Reflex incontinence is an involuntary loss of urine at
1. functional incontinence. predictable intervals when a specific bladder volume is
2. reflex incontinence. reached. Total incontinence occurs when a client
3. stress incontinence. experiences a continuous and unpredictable loss of urine.
4. total incontinence.
131 Following an earthquake, a client who was 4. orthostatic hypotension.
. rescued from a collapsed building is seen in Bleeding is a volume-loss problem, which causes a drop in
the emergency department. He has blunt blood pressure. As the bleeding persists and the body's
trauma to the thorax and abdomen. The ability to compensate declines, orthostatic hypotension
nursing observation that most suggests the becomes evident. A prolonged PTT and a history of
client is bleeding is: warfarin usage are causes of bleeding but aren't evidence
of bleeding. As bleeding persists and the client's level of
1. a prolonged partial thromboplastin time consciousness declines, breathing will become more
(PTT). shallow and breath sounds will diminish; however, this is
2. a recent history of warfarin (Coumadin) a late and unreliable manifestation of bleeding.
usage.
3. diminished breath sounds.
4. orthostatic hypotension.
132 Following an ileostomy, when should the 2. In the operating room.
. drainage appliance be applied to the stoma? Drainage from the ileostomy contains secretions that are
rich in digestive enzymes and highly irritating to the skin.
1. 24 hours later, when edema has subsided. Protection of the skin from the effects of these enzymes is
2. In the operating room. begun at once. Skin exposed to these enzymes even for a
3. After the ileostomy begins to function. short time becomes reddened, painful, and excoriated.
4. When the client is able to begin self-care
procedures.
133 Following perineal surgery, a client is at risk 1. Insert a continuous indwelling catheter per order.
. for a wound infection related to incontinence. An indwelling catheter diverts urine away from the
The correct management of this problem is to: operative site, reducing risk of wound infection.

1. Insert a continuous indwelling catheter per


order.
2. Assist to the toilet and protect the skin with
cream.
3. Limit oral fluid intake.
4. Give a loop diuretic, such as furosemide, as
ordered.
134 Following surgery for a ruptured appendix, 2. Promote drainage and prevent subdiaphragmatic
. the nurse should place the client in a semi- abscesses.
Fowler's position primarily to: Although this position will help aerate the lungs by
allowing for maximum expansion, it is not he primary
1. Fully aerate the lungs. reason for this position.
2. Promote drainage and prevent This position will no help splint the incision.
subdiaphragmatic abscesses. This position will not help facilitate movement or
3. Splint the wound. necessarily reduce complications.
4. Facilitate movement and reduce
complications from immobility.
135 For a client who must undergo colon 2. decrease the intestinal bacteria count.
. surgery, the physician orders The antibiotic neomycin sulfate (Mycifradin) is prescribed to
preoperative cleansing enemas and decrease the bacterial count and reduce the risk of fecal
neomycin sulfate (Mycifradin). The contamination during surgery. After surgery, the physician may
rationale for neomycin use in this client prescribe an antiemetic not an antibiotic to control
is to: postoperative nausea and vomiting. Antibiotics decrease the
intestinal bacteria count, not increase it. They have no relation
1. control postoperative nausea and to megacolon development. To prevent this complication, the
vomiting. client should avoid opioid analgesics, such as morphine
2. decrease the intestinal bacteria count. (Duramorph), which can decrease intestinal motility and
3. increase the intestinal bacteria count. contribute to megacolon.
4. prevent the development of
megacolon.
136 Functional incontinence Lower urinary tract function is intact but other factors, such as
. severe cognitive impairment (eg, Alzheimer's dementia), make it
difficult for the patient to identify the need to void or physical
impairments make it difficult or impossible for the patient to
reach the toilet in time for voiding.
137 Functions of the kidney 1. Maintain acid-base balance
. 2. Excrete end products of body metabolism
3. Control fluid and electrolyte balance
4. Excrete bacterial toxins, water-soluble drugs, and drug
metabolites
5. Secrete renin to regulate the blood pressure and
erythropoietin to stimulate the bone marrow to produce red
blood cells.
6. Synthesize vitamin D for calcium absorption and regulation
of the parathyroid hormones.
138 Glomerulonephritis Term that includes a variety of disorders, most of which are
. caused by an immunological reaction. Results in proliferative
and inflammatory changes within the glomerular structure.
Destruction, inflammation, and sclerosis of the glomeruli of
both kidneys occur. The inflammation of the glomeruli results
from an antigen-antibody reaction produced from an infection or
autoimmune process elsewhere in the body. Loss of kidney
function occurs.
139 The graduate nurse and her preceptor 1. The newly admitted client with acute abdominal pain
. are establishing priorities for their The graduate nurse and her preceptor should assess the new
morning assessments. Which client admission with acute abdominal pain first because he just
should they assess first? arrived on the floor and might be unstable. Next, they should
change the abdominal dressing for the postoperative client or
1. The newly admitted client with acute measure feeding tube residual in the client with continuous tube
abdominal pain feedings. These tasks are of equal importance. They should
2. The client who underwent surgery assess the sleeping client who received pain medication 1 hour
three days ago and who now requires a ago last because he just received relief from his pain and is able
dressing change to sleep.
3. The client receiving continuous tube
feedings who needs the tube-feeding
residual checked
4. The sleeping client who received pain
medication 1 hour ago
140 Hernia abnormal protrusion of an organ through the structures normally
. containing it
141 The home care nurse is making a visit with a 3. Ask the neighbor to come back in 20 minutes.
. client who had a double barrel colostomy created The home care nurse should ask the neighbor to come
after bowel surgery. While the nurse is changing back in 20 minutes when the visit is over. Client
the client's appliance there is a knock on the door. privacy is a priority even in the home care setting.
The nurse answers the door. The client's next- Options 1 and 2 violate client privacy and
door neighbor wants to visit with the client. confidentiality. Option 4 is inappropriate because the
Which intervention by the nurse is most client didn't request help from the neighbor.
appropriate?

1. Allow the neighbor to enter.


2. Have the neighbor wait in the next room until
the appliance is applied.
3. Ask the neighbor to come back in 20 minutes.
4. Suggest that the neighbor come in and learn
how to apply the appliance in case the client
needs help.
142 Iatrogenic incontinence the involuntary loss of urine due to extrinsic medical
. factors, predominantly medications
143 If a bowel obstruction occurs from inflammatory 1. Increased RLQ; decreased LLQ.
. bowel disease at the transverse colon, a nurse will Initially peristalsis will increase in the ascending colon
initially hear bowel sounds that are: (RLQ) in an attempt to clear the blockage. There will
be no peristalsis distal to the obstruction.
1. Increased RLQ; decreased LLQ.
2. Decreased RLQ; decreased LLQ.
3. Absent RLQ; absent LLQ.
4. Decreased RLQ; increased LLQ.
144 ileal conduit Ureters are implanted into a segment of the ileum, with
. the formation of an abdominal stoma. The urine flows
into the conduit and is propelled continuously out
through the stoma by peristalsis. The client is required
to wear an appliance over the stoma to collect the
urine.
Complications include obstruction, pyelonephritis,
leakage at the anastomosis site, stenosis,
hydronephrosis, calculi, skin irritation and ulceration,
and stomal defects.
145 Ileoanal reservoir surgical intervention for ulcerative colitis
. Creation of an ileoanal reservoir is a two-stage
procedure that involves the excision of the rectal
mucosa , an abdominal colectomy, construction of a
reservoir to the anal canal, and a temporary loop
ileostomy.
The ileostomy is closed in 3 to 4 months after the
capacity of the reservoir is increased and has had time
to heal.
146 In a client with enteritis and frequent diarrhea, 3. metabolic acidosis.
. the nurse should anticipate an acid-base Diarrhea causes a bicarbonate deficit. With loss of the
imbalance of: relative alkalinity of the lower GI tract, the relative
acidity of the upper GI tract predominates leading to
1. respiratory acidosis. metabolic acidosis. Loss of acid, which occurs with
2. respiratory alkalosis. severe vomiting, may lead to metabolic alkalosis.
3. metabolic acidosis. Diarrhea doesn't lead to respiratory acid-base
4. metabolic alkalosis. imbalances, such as respiratory acidosis and
respiratory alkalosis.
147 Inflammatory bowel disease a chronic recurrent inflammation of the intestinal tract
. that has exacerbations and remissions.
Ulcerative colitis
Chron's disease
148 Interventions for NPO
. appendicitis Administer fluids intravenously to prevent dehydration. Monitor for changes in
level of pain.
Monitor for signs of ruptured appendix and peritonitis
Position the client in a right side-lying or low to semi-Fowler's position to
promote comfort.
Monitor bowel sounds.
Apply ice packs to the abdomen for 20 to 30 minutes every hour as prescribed.
(Avoid heat - heat can cause rupture)
Administer antibiotics as prescribed.
Pain meds after dx is made.
Schedule appendectomy.
149 Interventions for antimicrobials
. Chron's disease corticosteroids
immunosuppressive agents
flagyl
B12 injections
high calorie, high protein, nitrogen, low fat, low residue diet - no dairy
If for surgery then NPO and TPN
surgery only done if not responsive to other therapy - high incidence of
recurrence - not a cure
150 Interventions for laxatives (use cautiously)
. constipation enemas (oil retention - softens feces, hypotonic, phosphate)
high fiber food
increased fluids (at least 3,000mL/day)
151 Interventions for 1. Provide bedrest during the acute phase.
. diverticulitis 2. Maintain NPO status or provide clear liquids during the acute phase as
prescribed.
3. Introduce a fiber-containing diet gradually, when the inflammation has
resolved.
4. Administer antibiotics, analgesics, and anticholinergics to reduce bowel
spasms as prescribed.
5. Instruct the client to refrain from lifting, straining, coughing, or bending to
avoid increased intra-abdominal pressure.
6. Monitor for perforation, hemorrhage, fistulas, and abscesses.
7. Instruct the client to increase fluid intake to 2500 to 3000 mL daily, unless
contraindicated.
8. Instruct the client to eat soft high-fiber foods, such as whole grains; the client
should avoid high-fiber foods when inflammation occurs because these foods
will irritate the mucosa further.
9. Instruct the client to avoid gas-forming foods or foods containing indigestible
roughage, seeds, or nuts because these food substances become trapped in
diverticula and cause inflammation.
10. Instruct the client to consume a small amount of bran daily and to take bulk-
forming laxatives as prescribed to increase stool mass.
152 Interventions for 1. Monitor vital signs, especially for hypertension and temperature elevations.
. glomerulonephritis 2. Monitor intake and output and urine characteristics closely.
3. Monitor daily weight.
4. Monitor for edema.
5. Monitor for fluid overload, ascites, pulmonary edema, and CHF.
6. Restrict fluid intake as prescribed.
7. Provide a high-calorie, low-protein, low-sodium, and low-potassium diet to
prevent worsening azotemia, fluid retention, and hyperkalemia.
8. Provide bed rest and limit activity.
9. Administer diuretics, antihypertensives, and antibiotics as prescribed.
10. Monitor for signs of renal failure, cardiac failure, and hypertensive
encephalopathy.
11. Instruct the client to report signs of bloody urine, headache, or edema.
12. Instruct the client to obtain treatment for infections, especially sore throats,
skin lesions, and upper respiratory infections.
153 Interventions for 1. Apply cold packs to the anal-rectal area followed by sitz baths as prescribed.
. hemorrhoids 2. Apply witch hazel soaks and topical anesthetics as prescribed.
3. Encourage a high-fiber diet and fluids to promote bowel movements without
straining.
4. Administer stool softeners as prescribed.
154 Interventions for hernias truss
. scrotal support
avoid coughing
no heavy lifting
surgery
155 Interventions for NPO
. intestinal obstruction NG tube (irrigate with normal saline)
Intestinal tube
enema
IV fluids (NS with K+)
TPN
surgery (preop neomycin enema)
156 Interventions for 1. Monitor vital signs, especially the temperature, for signs of infection.
. nephrolithiasis 2. Monitor intake and output.
3. Assess for fever, chills, and infection.
4. Monitor for nausea, vomiting, and diarrhea.
5. Encourage fluid intake up to 3000 mL/ day, unless contraindicated, to facilitate
the passage of the stone and prevent infection.
6. Administer fluids intravenously as prescribed if unable to take fluids orally or
in adequate amounts to increase the flow of urine and facilitate passage of the
stone.
7. Provide warm baths and heat to the flank area (massage therapy should be
avoided).
8. Administer analgesics at regularly scheduled intervals as prescribed to relieve
pain.
9. Assess the client's response to pain medication.
10. Assist the client in performing relaxation techniques to assist in relieving
pain.
11. Encourage client ambulation, if stable, to promote the passage of the stone.
12. Turn and reposition the immobilized client to promote passage of the stone.
13. Instruct the client in the diet restrictions specific to the stone composition if
prescribed.
14. Prepare the client for surgical procedures if prescribed
157 Interventions for antibiotics
. peritonitis NG tube
IV fluids
analgesics
surgery
158 Interventions for 1. Monitor vital signs, especially for elevated temperature.
. pyelonephritis 2. Encourage fluid intake up to 3000 mL /day to reduce fever and prevent
dehydration.
3. Monitor intake and output (ensure that output is a minimum of 1500 mL/ 24
hr).
4. Monitor weight.
5. Encourage adequate rest.
6. Instruct the client in a high-calorie, low-protein diet.
7. Provide warm, moist compresses to the flank area to help relieve pain.
8. Encourage the client to take warm baths for pain relief.
9. Administer analgesics, antipyretics, antibiotics, urinary antiseptics, and
antiemetics as prescribed.
159 Interventions for 1. Acute phase: Maintain NPO status and administer fluids and electrolytes
. ulcerative colitis intravenously or via parenteral nutrition as prescribed.
2. Restrict the client's activity to reduce intestinal activity.
3. Monitor bowel sounds and for abdominal tenderness and cramping.
4. Monitor stools, noting color, consistency, and the presence or absence of blood.
5. Monitor for bowel perforation, peritonitis, and hemorrhage.
6. Following the acute phase, the diet progresses from clear liquids to a low-fiber diet as
tolerated.
7. Instruct the client about diet; usually a low-fiber, high-protein diet with vitamins and
iron supplements are prescribed.
8. Instruct the client to avoid gas-forming foods, milk products, and foods such as whole
wheat grains, nuts, raw fruits and vegetables, pepper, alcohol, and caffeine-containing
products.
9. Instruct the client to avoid smoking.
10. Administer medications as prescribed , which may include a combination of
medications such as salicylate compounds , corticosteroids, immunosuppressants, and
antidiarrheals.
160 Interventions for 1. Before administering prescribed antibiotics, obtain a urine specimen for culture and
. UTIs sensitivity, if prescribed, to identify bacterial growth.
2. Encourage the client to increase fluids up to 3000 mL/ day, especially if the client is
taking a sulfonamide; sulfonamides can form crystals in concentrated urine.
3. Administer prescribed medications , which may include analgesics, antiseptics,
antispasmodics, antibiotics, and antimicrobials.
4. Maintain an acid urine pH (5.5); instruct the client about foods to consume to maintain
acidic urine.
5. Provide heat to the abdomen or sitz baths for complaints of discomfort.
6. Note that if the client is prescribed an aminoglycoside, sulfonamide, or nitrofurantoin
(Macrodantin), the actions of these medications are decreased by acidic urine.
7. Use sterile technique when inserting a urinary catheter.
8. Maintain closed urinary drainage systems for the client with an indwelling catheter
and avoid elevating the urinary drainage bag above the level of the bladder. 9. Provide
meticulous perineal care for the client with an indwelling catheter.
10. Discourage caffeine products such as coffee, tea, and cola.
11. Client education a. Avoid alcohol. b. Take medications as prescribed. c. Take
antibiotics on schedule and complete the entire course of medications as prescribed,
which may be 10 to 14 days. d. Repeat the urine culture following treatment. e. Prevent
recurrence of cystitis
161 Intravenous An x-ray procedure in which an intravenous injection of a radiopaque dye is used to
. pyelography visualize and identify abnormalities in the renal system.
Obtain an informed consent.
Assess the client for allergies to iodine, seafood, and radiopaque dyes.
Withhold food and fluids after midnight on the night before the test.
Monitor vital signs.
Monitor urinary output.
162 irreducible hernia cannot be replaced into cavity
.

163 Kock ileostomy An intraabdominal pouch constructed from the terminal ileum. The pouch is connected
. (continent to the stoma with a nipple-like valve constructed from a portion of the ileum. The stoma
ileostomy) is flush with the skin.
164 Kock ileostomy surgical intervention for ulcerative colitis
. (continent An intra-abdominal pouch that stores the feces and is constructed from the terminal
ileostomy) ileum.
The pouch is connected to the stoma with a nipple-like valve constructed from a portion
of the ileum ; the stoma is flush with the skin.
A catheter is used to empty the pouch, and a small dressing or adhesive bandage is worn
over the stoma between emptyings.
165 Kock pouch A continent internal ileal reservoir created from a segment of the ileum and ascending
. colon. The ureters are implanted into the side of the reservoir, and a special nipple valve
is constructed to attach the reservoir to the skin.
Postoperatively, the client will have a Foley catheter in place to drain urine continuously
until the pouch has healed. The catheter is irrigated gently with normal saline to prevent
obstruction from mucus or clots. Following removal of the catheter, the client is
instructed in how to self-catheterize and to drain the reservoir at 4- to 6-hour intervals
166 KUB (kidneys, ureters, and bladder) An x-ray of the urinary system and adjacent structures to detect
. radiography urinary calculi.
167 The male client has a tentative 4. Dysuria and penile discharge
. diagnosis of urethritis. The nurse Urethritis in the male client often results from chlamydial
assesses the client for which of the infection and is characterized by dysuria, which is accompanied by
following manifestations of the a clear to mucopurulent discharge. Because this disorder often
disorder? coexists with gonorrhea, diagnostic tests are done for both and
include culture and rapid assays.
1. Hematuria and pyuria
2. Dysuria and proteinuria
3. Hematuria and urgency
4. Dysuria and penile discharge
168 Mixed urinary incontinence encompasses several types of urinary incontinence, is involuntary
. leakage associated with urgency and also with exertion, effort,
sneezing, or coughing
169 The most important postoperative 2. Maintain skin-protective barrier.
. nursing concern for a client following The drainage from an ileostomy will always be liquid.
an ileostomy is to: The first concern is the stoma and the condition of the skin around
the stoma. The drainage from the stoma is made up of digestive
1. Check for the presence of diarrhea. enzymes. The pouch opening should be no more than 1/8 inch
2. Maintain skin-protective barrier. larger than the stoma.
3. Allow the client to observe Irrigation is not done with an ileostomy.
irrigation. The pouch is empties, not changed, when it is 1/3 full. It is
4. Change the stoma pouch daily when changed every 5-7 days, usually before eating, when the stoma is
full. least active.
170 The mouth care measure that should 2. Sucking on ice chips to relieve dryness.
. be used with caution by the nurse The client should be cautioned to limit the number of ice chips he
when a client has a nasogastric tube is: or she sucks on. The NG tube will remove not only the increased
water ingested from the melted chips but also essential
1. Regularly brushing teeth and tongue electrolytes.
with soft brush.
2. Sucking on ice chips to relieve
dryness.
3. Occasionally rinsing mouth with a
nonastringent substance and
massaging gums.
4. Application of lemon juice and
glycerine swabs to the lips.
171 Nephrectomy Performed for extensive kidney damage, renal infection, severe
. obstruction from stones or tumors, and prevention of stone
recurrence.
172 Nephrosclerosis sclerosis of the small arteries and arterioles of the kidney
.
usually caused by vascular changes due to malignant hypertension
and arteriosclerosis

treated with antihypertensives


173 Nephrostomy tube temporary - used to preserve renal function
. inserted into pelvis of kidney
174 The nurse asks a client who had 2. Encourage the client to ambulate at least three times per day.
. abdominal surgery 3 days ago if he has The nurse should encourage the client to ambulate at least three
moved his bowels since surgery. The times per day. Ambulating stimulates peristalsis, which helps the
client states, "I haven't moved my bowels to move. It isn't appropriate to apply heat to a surgical
bowels, but I am passing gas." How wound. Moreover, heat application can't be initiated without a
should the nurse intervene? physician order. A tap water enema is typically administered as a
last resort after other methods fail. A physician's order is needed
1. Apply moist heat to the client's with a tap water enema as well. Notifying the physician isn't
abdomen. necessary at this point because the client is exhibiting bowel
2. Encourage the client to ambulate at function by passing flatus.
least three times per day.
3. Administer a tap water enema.
4. Notify the physician.
175 The nurse can quickly assess volume 2. Taking the client's blood pressure first supine, then sitting,
. depletion in a client with ulcerative noting any changes.
colitits by: Urine output and specific gravity are better measures of the
adequacy of fluid volume replacement than of fluid volume
1. Measuring the quantity and specific depletion.
gravity of the client's urine output. Postural blood pressure readings are an excellent mode for
2. Taking the client's blood pressure assessing volume depletion. If the systolic blood pressure
first supine, then sitting, noting any decreases more than 10 mm Hg and there is a concurrent increase
changes. in pulse rate, a volume depletion problem is indicated.
3. Comparing the client's present
weight with the weight on a previous
admission.
4. Administering the oral water test.
176 The nurse caring for a client with small- 4. Administering I.V. fluids
. bowel obstruction would plan to I.V. infusions containing normal saline solution and potassium
implement which nursing intervention should be given first to maintain fluid and electrolyte balance.
first? For the client's comfort and to assist in bowel decompression, the
nurse should prepare to insert an NG tube next. A blood sample is
1. Administering pain medication then obtained for laboratory studies to aid in the diagnosis of
2. Obtaining a blood sample for bowel obstruction and guide treatment. Blood studies usually
laboratory studies include a complete blood count, serum electrolyte levels, and
3. Preparing to insert a nasogastric blood urea nitrogen level. Pain medication often is withheld until
(NG) tube obstruction is diagnosed because analgesics can decrease
4. Administering I.V. fluids intestinal motility.
177 The nurse correctly identifies a urine 3. Acidic
. sample with a pH of 4.3 as being which Normal urine pH is 4.5 to 8.0; a value of 4.3 reveals acidic urine
type of solution? pH. A pH above 7.0 is considered an alkaline or basic solution. A
pH of 7.0 is considered neutral.
1. Neutral
2. Alkaline
3. Acidic
4. Basic
178 The nurse explains to a client that 2. When drainage leaks through the seal.
. the colostomy appliance for a Every day is too often and can damage the skin from pulling the
double-barrel colostomy should be appliance off so often.
changed: The appliance should be changes every 2-3 days or as soon as there
is leakage. Drainage can excoriate the skin; therefore a new
1. Every day. appliance needs to be applied as soon as leakage appears.
2. When drainage leaks through the 1 week is too long for the skin to go without being examined and
seal. cleansed.
3. Once a week.
4. At a time selected by the visiting
nurse.
179 The nurse explains to the client that 2. Insertion of a fiberoptic scope that allows for direct visual
. a flecible sigmoidoscopy involves: examination of the anal canal, rectum, and sigmoid colon.

1. Instillation of a radiopaque
contrast medium into the lower GI
tract.
2. Insertion of a fiberoptic scope that
allows for direct visual examination
of the anal canal, rectum, and
sigmoid colon.
3. Insertion of a fiberoptic scope that
allows for direct visualization of the
sigmoid colon, transverse colon and
ileocecal valve.
4. Surgical removal of polyps and
biopsy of suspicious GI mucosa.
180 The nurse is assessing a client who 2. Inspection, auscultation, percussion, and palpation
. complains of abdominal pain, The correct sequence for abdominal examination is inspection,
nausea, and diarrhea. When auscultation, percussion, and palpation. This sequence differs from
examining the client's abdomen, that used for other body regions (inspection, palpation, percussion,
which sequence should the nurse and auscultation) because palpation and percussion increase
use? intestinal activity, altering bowel sounds. Therefore, the nurse
shouldn't palpate or percuss the abdomen before auscultating.
1. Inspection, palpation, percussion, Assessment of any body system or region starts with inspection;
and auscultation therefore, auscultating or palpating the abdomen first would be
2. Inspection, auscultation, incorrect.
percussion, and palpation
3. Auscultation, inspection,
percussion, and palpation
4. Palpation, auscultation,
percussion, and inspection
181 The nurse is assessing a client who is 2. Increased urine output
. receiving total parenteral nutrition Glucose supplies most of the calories in TPN; if the glucose
(TPN). Which finding suggests that the infusion rate exceeds the client's rate of glucose metabolism,
client has developed hyperglycemia? hyperglycemia arises. When the renal threshold for glucose
reabsorption is exceeded, osmotic diuresis occurs, causing an
1. Cheyne-Stokes respirations increased urine output. A decreased appetite and diaphoresis
2. Increased urine output suggest hypoglycemia. Cheyne-Stokes respirations are
3. Decreased appetite characterized by a period of apnea lasting 10 to 60 seconds,
4. Diaphoresis followed by gradually increasing depth and frequency of
respirations. Cheyne-Stokes respirations typically occur with
cerebral depression or heart failure.
182 The nurse is assessing for stoma prolapse 1. Protruding stoma
. in a client with a colostomy. What should A prolapsed stoma is one in which the bowel protrudes through
the nurse observe if stoma prolapse the stoma. A stoma retraction is characterized by sinking of the
occurs? stoma. Ischemia of the stoma would be associated with a dusky
or bluish color. A stoma with a narrowed opening at the level of
1. Protruding stoma the skin or fascia is said to be stenosed.
2. Sunken and hidden stoma
3. Narrowed and flattened stoma
4. Dark- and bluish-colored stoma
183 The nurse is assessing the colostomy of a 2. The passage of flatus
. client who has had an abdominal Following abdominal perineal resection, the nurse would expect
perineal resection for a bowel tumor. the colostomy to begin to function within 72 hours after
Which of the following assessment surgery, although it may take up to 5 days . The nurse should
findings indicates that the colostomy is assess for a return of peristalsis, listen for bowel sounds, and
beginning to function? check for the passage of flatus . Absent bowel sounds would not
indicate the return of peristalsis. The client would remain NPO
1. Absent bowel sounds until bowel sounds return and the colostomy is functioning.
2. The passage of flatus Bloody drainage is not expected from a colostomy.
3. The client's ability to tolerate food
4. Bloody drainage from the colostomy
184 The nurse is assessing the stoma of a 4. A red and moist stoma
. client following a ureterostomy. Which of Following ureterostomy, the stoma should be red and moist. A
the following should the nurse expect to pale stoma may indicate an inadequate amount of vascular
note? supply. A dry stoma may indicate a body fluid deficit. Any sign
of darkness or duskiness in the stoma may indicate a loss of
1. A dry stoma vascular supply and must be reported immediately or necrosis
2. A pale stoma can occur.
3. A dark-colored stoma
4. A red and moist stoma
185 The nurse is caring for a client who had a 1. Encouraging intake of at least 2 L of fluid daily
. stroke. Which nursing intervention By encouraging a daily fluid intake of at least 2 L, the nurse
promotes urinary continence? helps fill the client's bladder, thereby promoting bladder
retraining by stimulating the urge to void. The nurse shouldn't
1. Encouraging intake of at least 2 L of give the client soda before bedtime; soda acts as a diuretic and
fluid daily may make the client incontinent. The nurse should take the
2. Giving the client a glass of soda before client to the bathroom or offer the bedpan at least every 2 hours
bedtime throughout the day; twice per day is insufficient. Consultation
3. Taking the client to the bathroom twice with a dietitian won't address the problem of urinary
per day incontinence.
4. Consulting with a dietitian
186 The nurse is caring for a client with a 1. destroys the odor-proof seal.
. colostomy. The client tells the nurse that Any hole, no matter how small, will destroy the odor-proof
he makes small pin holes in the drainage seal of a drainage bag. Removing the bag or unclamping it is
bag to help relieve gas. The nurse should the only appropriate method for relieving gas.
teach him that this action:

1. destroys the odor-proof seal.


2. won't affect the colostomy system.
3. is appropriate for relieving the gas in a
colostomy system.
4. destroys the moisture-barrier seal.
187 The nurse is caring for a client with acute 2. Increasing fluid intake to 3 L/day
. pyelonephritis. Which nursing Acute pyelonephritis is a sudden inflammation of the
intervention is most important? interstitial tissue and renal pelvis of one or both kidneys.
Infecting bacteria are normal intestinal and fecal flora that
1. Administering a sitz bath twice per day grow readily in urine. Pyelonephritis may result from
2. Increasing fluid intake to 3 L/day procedures that involve the use of instruments (such as
3. Using an indwelling urinary catheter to catheterization, cystoscopy, and urologic surgery) or from
measure urine output accurately hematogenic infection. The most important nursing
4. Encouraging the client to drink intervention is to increase fluid intake to 3 L/day. This helps
cranberry juice to acidify the urine empty the bladder of contaminated urine and prevents calculus
formation. Administering a sitz bath would increase the
likelihood of fecal contamination. Using an indwelling urinary
catheter could cause further contamination. Encouraging the
client to drink cranberry juice to acidify urine is helpful but
isn't the most important interaction.
188 The nurse is caring for a client with acute 4. Hyperkalemia
. renal failure. The nurse should expect that Hyperkalemia is a common complication of acute renal
hypertonic glucose, insulin infusions, and failure. The administration of glucose and regular insulin
sodium bicarbonate will be used to treat infusions, with sodium bicarbonate if necessary, can
what complication of acute renal failure? temporarily prevent cardiac arrest by moving potassium into
the cells and temporarily reducing potassium levels. This
1. Hypokalemia treatment isn't used to treat hyperphosphatemia or
2. Hyperphosphatemia hypophosphatemia.
3. Hypophosphatemia
4. Hyperkalemia
189 The nurse is developing a discharge plan for 1. Risk for fluid volume deficit.
. a client with a new ileostomy. The client is 4. Risk for impaired skin integrity.
32 years old, mentally competent, lives with 6. Altered sexuality patterns.
her husband of 3 years, and works outside An ileostomy produced liquid stool. The client's daily fluid
her home. Which nursing diagnoses are intake should be 2 to 3 liters to prevent dehydration.
important to include in the discharge plan? Fecal drainage or a poorly fitting ostomy appliance can
Select all that apply. irritate and erode the stoma and surrounding the skin.
A client with an ileostomy is more likely at risk for nutrition
1. Risk for fluid volume deficit. less than body requirements.
2. Risk for fluid volume excess. The client with an ileostomy is often concerned about loss of
3. Risk for constipation. sexual appeal or leakage of fecal material during sexual
4. Risk for impaired skin integrity. activity.
5. Altered nutrition: more than body
requirements.
6. Altered sexuality patterns.
190 The nurse is doing preoperative teaching 3. "I will need to drain the pouch regularly with a catheter."
. with the client who is about to undergo A Kock pouch is a continent ileostomy. As the ileostomy
creation of a Kock pouch. The nurse begins to function , the client drains it every 3 to 4 hours and
interprets that the client has the best then decreases the draining to about three times a day, or as
understanding of the nature of the surgery if needed when full. The client does not need to wear a
the client makes which statement? drainage bag but should wear an absorbent dressing to
absorb mucous drainage from the stoma. Ileostomy drainage
1. "I will be able to pass stool by the rectum is liquid. The client would be able to pass stool only from
eventually." the rectum if an ileal-anal pouch or anastomosis were
2. "The drainage from this type of ostomy created. This type of operation is a two-stage procedure.
will be formed."
3. "I will need to drain the pouch regularly
with a catheter."
4. "I will need to wear a drainage bag for
the rest of my life."
191 The nurse is inserting a urinary catheter 2. breathe deeply.
. into a client who is extremely anxious about When inserting a urinary catheter, facilitate insertion by
the procedure. The nurse can facilitate the asking the client to breathe deeply. Doing this will relax the
insertion by asking the client to: urinary sphincter. Initiating a stream of urine isn't
recommended during catheter insertion. Turning to the side
1. initiate a stream of urine. or holding the labia or penis won't ease insertion, and doing
2. breathe deeply. so may contaminate the sterile field.
3. turn to the side.
4. hold the labia or shaft of the penis.
192 The nurse is irrigating a colostomy when the 3. Pinch the tubing the interrupt the flow of solution.
. client complains of cramping. What is the
most appropriate initial action for the
nurse?

1. Increase the flow of solution.


2. Ask the client to turn to his other side.
3. Pinch the tubing the interrupt the flow of
solution.
4. Remove the tube from the colostomy.
193 The nurse is monitoring a client admitted to the 1. Notify the physician.
. hospital with a diagnosis of appendicitis who is Based on the signs and symptoms presented in the
scheduled for surgery in 2 hours. The client begins question, the nurse should suspect peritonitis and
to complain of increased abdominal pain and notify the physician. Administering pain medication is
begins to vomit. On assessment, the nurse notes not an appropriate intervention. Heat should never be
that the abdomen is distended and bowel sounds applied to the abdomen of a client with suspected
are diminished. Which is the appropriate nursing appendicitis because of the risk of rupture. Scheduling
intervention? surgical time is not within the scope of nursing
practice, although the physician probably would
1. Notify the physician. perform the surgery earlier than the prescheduled
2. Administer the prescribed pain medication. time.
3. Call and ask the operating room team to
perform the surgery as soon as possible.
4. Reposition the client and apply a heating pad on
warm setting to the client's abdomen.
194 The nurse is monitoring a client receiving 2. Anticholinergic drugs
. paregoric to treat diarrhea for drug interactions. Paregoric has an additive effect of constipation when
Which drugs can produce additive constipation used with anticholinergic drugs. Antiarrhythmics,
when given with an opium preparation? anticoagulants, and antihypertensives aren't known to
interact with paregoric.
1. Antiarrhythmic drugs
2. Anticholinergic drugs
3. Anticoagulant drugs
4. Antihypertensive drugs
195 A nurse is observing a client for possible 2. Localized or diffuse pain.
. complications of postoperative peritonitis. Which 3. Abdominal rigidity.
manifestations are most indicative of peritonitis? 4. Shallow respirations.
Select all that apply. Peristalsis is diminished and paralytic ileus may
develop.
1. Hyperactive bowel sounds. Pain is often diffuse initially and then localized.
2. Localized or diffuse pain. The abdominal muscles are rigid and the abdomen is
3. Abdominal rigidity. tender and distended.
4. Shallow respirations. With abdominal tenderness and distension, the client
5. Temperature over 102. will not breathe as deeply.
Temperature is usually 100 - 101.
196 The nurse is performing an assessment on a client 2. hypoactive.
. who has developed a paralytic ileus. The client's If a paralytic ileus occurs, bowel sounds will be
bowel sounds will be: hypoactive or absent. Hyperactive bowel sounds may
signify hunger, intestinal obstruction, or diarrhea.
1. hyperactive. High-pitched sounds may signify a dilated bowel. A
2. hypoactive. blowing sound may be a bruit from a partially
3. high-pitched. obstructed abdominal aorta.
4. blowing.
197 The nurse is planning a group teaching session 3. Notify the physician if urinary urgency, burning,
. on the topic of urinary tract infection (UTI) frequency, or difficulty occurs.
prevention. Which point would the nurse want Urgency, burning, frequency, and difficulty urinating are
to include? all common symptoms of a UTI. The client should notify
his physician so that a microscopic urinalysis can be done
1. Limit fluid intake to reduce the need to and appropriate treatment can be initiated. The client
urinate. should be instructed to drink 2 to 3 L of fluid per day to
2. Take medication prescribed for a UTI until dilute the urine and reduce irritation of the bladder
the symptoms subside. mucosa. The full amount of antibiotics prescribed for
3. Notify the physician if urinary urgency, UTIs must be taken despite the fact that the symptoms
burning, frequency, or difficulty occurs. may have subsided. This will help to prevent recurrences
4. Wear only nylon underwear to reduce the of UTI. Women are told to avoid scented toilet tissue and
chance of irritation. bubble baths and to wear cotton underwear, not nylon, to
reduce the chance of irritation.
198 The nurse is planning to administer a sodium 1. retain the enema for 30 minutes to allow for sodium
. polystyrene sulfonate (Kayexalate) enema to a exchange; afterward, the client should have diarrhea.
client with a potassium level of 6.2 mEq/L. Kayexalate is a sodium-exchange resin. Thus, the client
Correct administration and the effects of this will gain sodium as potassium is lost in the bowel. For the
enema would include having the client: exchange to occur, Kayexalate must be in contact with the
bowel for at least 30 minutes. Sorbitol in the Kayexalate
1. retain the enema for 30 minutes to allow for enema causes diarrhea, which increases potassium loss
sodium exchange; afterward, the client should and decreases the potential for Kayexalate retention.
have diarrhea.
2. retain the enema for 30 minutes to allow for
glucose exchange; afterward, the client should
have diarrhea.
3. retain the enema for 60 minutes to allow for
sodium exchange; diarrhea isn't necessary to
reduce the potassium level.
4. retain the enema for 60 minutes to allow for
glucose exchange; diarrhea isn't necessary to
reduce the potassium level.
199 The nurse is providing postprocedure care for 3. notify the physician about cloudy or foul-smelling
. a client who underwent percutaneous urine.
lithotripsy. In this procedure, an ultrasonic The client should report the presence of foul-smelling or
probe inserted through a nephrostomy tube cloudy urine. Unless contraindicated, the client should be
into the renal pelvis generates ultra-high- instructed to drink large quantities of fluid each day to
frequency sound waves to shatter renal calculi. flush the kidneys. Sandlike debris is normal because of
The nurse should instruct the client to: residual stone products. Hematuria is common after
lithotripsy.
1. limit oral fluid intake for 1 to 2 weeks.
2. report the presence of fine, sandlike
particles through the nephrostomy tube.
3. notify the physician about cloudy or foul-
smelling urine.
4. report bright pink urine within 24 hours
after the procedure.
200 The nurse is reviewing a client's fluid 2. Fluid intake should be approximately equal to the urine
. intake and output record. Fluid intake output.
and urine output should relate in which Normally, fluid intake is approximately equal to the urine
way? output. Any other relationship signals an abnormality. For
example, fluid intake that is double the urine output indicates
1. Fluid intake should be double the urine fluid retention; fluid intake that is half the urine output
output. indicates dehydration. Normally, fluid intake isn't inversely
2. Fluid intake should be approximately proportional to the urine output.
equal to the urine output.
3. Fluid intake should be half the urine
output.
4. Fluid intake should be inversely
proportional to the urine output.
201 The nurse is reviewing the client's record 2. Elevated creatinine level
. and notes that the physician has Measuring the creatinine level is a frequently used laboratory
documented that the client has a renal test to determine renal function. The creatinine level increases
disorder. On review of the laboratory when at least 50% of renal function is lost. A decreased
results, the nurse most likely would expect hemoglobin level and red blood cell count may be noted if
to note which of the following? bleeding from the urinary tract occurs or if erythropoietic
function by the kidney is impaired. An increased white blood
1. Decreased hemoglobin level cell count is most likely to be noted in renal disease.
2. Elevated creatinine level
3. Decreased red blood cell count
4. Decreased white blood cell count
202 The nurse is reviewing the history of a 2. Hematuria
. client with bladder cancer. The nurse The most common symptom in clients with cancer of the
expects to note documentation of which bladder is hematuria. The client also may experience irritative
most common symptom of this type of voiding symptoms such as frequency, urgency, and dysuria, and
cancer? these symptoms often are associated with carcinoma in situ.

1. Dysuria
2. Hematuria
3. Urgency on urination
4. Frequency of urination
203 The nurse is reviewing the record of a 1. Diarrhea
. client with Crohn's disease. Which stool Crohn's disease is characterized by nonbloody diarrhea of
characteristic should the nurse expect to usually not more than four to five stools daily. Over time, the
note documented in the client's record? diarrhea episodes increase in frequency, duration, and severity.
Options 2, 3, and 4 are not characteristics of Crohn's disease.
1. Diarrhea
2. Chronic constipation
3. Constipation alternating with diarrhea
4. Stool constantly oozing from the
rectum
204 The nurse is reviewing the report of a 2. Urine pH of 3.0
. client's routine urinalysis. Which value Normal urine pH is 4.5 to 8; therefore, a urine pH of 3.0 is
should the nurse consider abnormal? abnormal. Urine specific gravity normally ranges from 1.002 to
1.035, making this client's value normal. Normally, urine
1. Specific gravity of 1.03 contains no protein, glucose, ketones, bilirubin, bacteria, casts,
2. Urine pH of 3.0 or crystals. Red blood cells should measure 0 to 3 per high-
3. Absence of protein power field; white blood cells, 0 to 4 per high-power field.
4. Absence of glucose Urine should be clear, with color ranging from pale yellow to
deep amber.
205 The nurse is taking the history of a client 4. Decreased force in the stream of urine
. who has had benign prostatic hyperplasia Decreased force in the stream of urine is an early sign of
in the past. To determine whether the benign prostatic hyperplasia. The stream later becomes weak
client currently is experiencing difficulty, and dribbling. The client then may develop hematuria,
the nurse asks the client about the frequency, urgency, urge incontinence, and nocturia. If
presence of which early symptom? untreated, complete obstruction and urinary retention can
occur.
1. Nocturia
2. Urinary retention
3. Urge incontinence
4. Decreased force in the stream of urine
206 The nurse is teaching a client about the 1. Age younger than 50 years
. risk factors associated with colorectal Colorectal cancer risk factors include age older than 50 years,
cancer. The nurse determines that further a family history of the disease, colorectal polyps, and chronic
teaching related to colorectal cancer is inflammatory bowel disease.
necessary if the client identifies which of
the following as an associated risk factor?
1. Age younger than 50 years
2. History of colorectal polyps
3. Family history of colorectal cancer
4. Chronic inflammatory bowel disease
207 The nurse is teaching a client how to 1. Hanging the irrigation bag 24 to 36 (60 to 90 cm) above
. irrigate his stoma. Which action indicates the stoma
that the client needs more teaching? An irrigation bag should be elevated 18 to 24 (40 to 60 cm)
above the stoma. Typically, adults use 500 to 1,000 ml of water
1. Hanging the irrigation bag 24 to 36 at a temperature no higher than 105 F (41 C) to irrigate a
(60 to 90 cm) above the stoma colostomy. If cramping occurs during irrigation, irrigation
2. Filling the irrigation bag with 500 to should be stopped and the client should take deep breaths until
1,000 ml of lukewarm water the cramping stops. Irrigation can then be resumed. Hand
3. Stopping irrigation for cramps and washing reduces the spread of microorganisms.
clamping the tubing until cramps pass
4. Washing hands with soap and water
when finished
208 The nurse is teaching a client with 2. small intestine.
. malabsorption syndrome about the The small intestine absorbs products of digestion, completes
disorder and its treatment. The client asks food digestion, and secretes hormones that help control the
which part of the GI tract absorbs food. secretion of bile, pancreatic juice, and intestinal secretions.
The nurse tells the client that products of The stomach stores, mixes, and liquefies the food bolus into
digestion are absorbed mainly in the: chyme and controls food passage into the duodenum; it doesn't
absorb products of digestion. Although the large intestine
1. stomach. completes the absorption of water, chloride, and sodium, it
2. small intestine. plays no part in absorbing food. The rectum is the portion of
3. large intestine. the large intestine that forms and expels feces from the body;
4. rectum. its functions don't include absorption.
209 The nurse is teaching an elderly client 2. "I need to use laxatives regularly to prevent constipation."
. about good bowel habits. Which The elderly client should be taught to gradually eliminate the use of
statement by the client would indicate laxatives. Point out that using laxatives to promote regular bowel
to the nurse that additional teaching movements may have the opposite effect. A high-fiber diet, ample
is required? amounts of fluids, and regular exercise promote good bowel health.

1. "I should eat a fiber-rich diet with


raw, leafy vegetables, unpeeled fruit,
and whole grain bread."
2. "I need to use laxatives regularly to
prevent constipation."
3. "I need to drink 2 to 3 liters of
fluid every day."
4. "I should exercise four times per
week."
210 The nurse knows the colostomy 2. 2 to 3 days postoperatively.
. begins functioning: Ileostomies begin to drain immediately.
The stoma will begin to secrete mucus within 48 hours, and the
1. Immediately. proximal loop should begin to drain fecal material within 72 hours.
2. 2 to 3 days postoperatively.
3. 1 week postoperatively.
4. 2 weeks postoperatively.
211. The nurse must provide total 1. cover the catheter insertion site with an occlusive dressing.
parenteral nutrition (TPN) to a client TPN increases the client's risk of infection because the catheter
through a triple-lumen central line. insertion site creates a port of entry for bacteria. To reduce the risk
To prevent complications of TPN, the of infection, the nurse should cover the insertion site with an
nurse should: occlusive dressing, which is airtight. Because the insertion site is an
open wound, the nurse should use sterile technique when changing
1. cover the catheter insertion site the dressing. TPN doesn't necessitate placement of an indwelling
with an occlusive dressing. urinary catheter or bed rest.
2. use clean technique when changing
the dressing.
3. insert an indwelling urinary
catheter.
4. keep the client on complete bed
rest.
212 The nurse should know that the 3. Loose, bloody.
. drainage is normal 4 days after a Normal bowel function and soft-formed stool usually do not occur
sigmoid colostomy when the material until around the seventh day following surgery.
is:

1. Green liquid.
2. Solid formed.
3. Loose, bloody.
4. Semiformed.
213 The nurse suspects wound 1. Tell the client to remain quiet and not to cough.
. dehiscence, and lifts the edges of the The client should remain quiet in a low Fowler's or horizontal
client's dressings. The nurse notes position. They should be cautioned not to cough so as note to
that the wound edges are entirely extrutde any intestines by increasing intra-abdominal pressures. The
separated. What is the next nursing physician should be notified next. Remain with the client,
action? reassuring them, monitoring vital signs, and having others bring
equipment such as IV setup, nasogastric tube, and suction
1. Tell the client to remain quiet and equipment.
not to cough. The client should be kept NPO.
2. Offer the client a warm drink to The dressing should be left in place to prevent evisceration.
promote relaxation.
3. Position the cient in a chair with
the feet elevated.
4. Apply a Scultetus bandage.
214 The nurse would position a client with 1. Semi-Fowler's.
. ruptured appendix in: The client is placed in a semi-Fowler's position to promote the
flow of drainage to the pelvic region, where a localized abscess
1. Semi-Fowler's. can be frained or resolved by the body's normal defenses. The
2. Trendelenburg. elevated position also keeps the infection from spreading upward
3. Left Sims'. in the peritoneal cavity.
4. Dorsal recumbent.
215 Nursing assessment of a client with 3. severe abdominal pain with direct palpation or rebound
. peritonitis (acute or chronic tenderness.
inflammation of the peritoneum) Peritonitis decreases intestinal motility and causes intestinal
reveals hypotension, tachycardia, and distention. A classic sign of peritonitis is a sudden, diffuse, severe
signs and symptoms of dehydration. abdominal pain that intensifies in the area of the underlying
The nurse also expects to find: causative disorder (such as appendicitis, diverticulitis, ulcerative
colitis, or a strangulated obstruction). The client also has direct or
1. tenderness and pain in the right rebound tenderness. Tenderness and pain in the right upper
upper abdominal quadrant. abdominal quadrant suggest cholecystitis. Jaundice and vomiting
2. jaundice and vomiting. are signs of cirrhosis of the liver. Rectal bleeding or a change in
3. severe abdominal pain with direct bowel habits may indicate colorectal cancer.
palpation or rebound tenderness.
4. rectal bleeding and a change in bowel
habits.
216 Nursing care of nephrostomy tube urine output is measured separately from each tube q 1-2 hrs
. ensure tube does not get kinked, clamped, layed on etc.
do not irrigate without order - 5-10cc sterile saline - strict aseptic
technique
daily weights
BUN/creatinine
217 One day after surgery for intestinal 2. Record this expected finding.
. resection, a client has no bowel sounds. Paralytic ileus and the absence of bowel sounds is expected on
Which action should a nurse take? the first postop day.

1. Take the vital signs and notify the


physician.
2. Record this expected finding.
3. Check rectally for impacted stool.
4. Perform abdominal massage.
218 One hour before a client is to undergo 1. "Atropine decreases salivation and gastric secretions."
. abdominal surgery, the physician When used as preanesthesia medications, atropine and other
orders atropine, 0.6 mg I.M. The client cholinergic blocking agents reduce salivation and gastric
asks the nurse why this drug must be secretions, thus helping to prevent aspiration of secretions during
administered. How should the nurse surgery. Atropine increases the heart rate and cardiac
respond? contractility, decreases bronchial secretions, and causes
bronchodilation. No evidence indicates that the drug enhances the
1. "Atropine decreases salivation and effect of anesthetic agents.
gastric secretions."
2. "Atropine controls the heart rate and
blood pressure."
3. "Atropine improves ventilation by
increasing the respiratory rate."
4. "Atropine enhances the effect of
anesthetic agents."
219 Paralytic ileus is described by the nurse as: 3. Absent, diminished, or uncoordinated autonomic
. stimulation of peristalsis.
1. Edema of the intestinal mucosa.
2. Acute dilation of the colon.
3. Absent, diminished, or uncoordinated autonomic
stimulation of peristalsis.
4. High-pitched, tinkling bowel sounds over the
area of obstruction.
220 Patient teaching after prostate surgery no intercourse or heavy lifting x 6 weeks
. avoid sitting/walking for prolonged periods of time
drink at least 1-2 L of fluid daily
urinate q 2-3 hrs
kegels
incontinent briefs for dribbling
yearly rectal exams
221 Peritonitis an inflammatory process in the peritoneum that
. results in extracellular fluid shifts
222 Pharmacological therapy for urinary incontinence Anticholinergic agents inhibit bladder contraction
. and are considered first-line medications for urge
incontinence.
Several tricyclic antidepressant medications can also
decrease bladder contractions as well as increase
bladder neck resistance.
Pseudoephedrine sulfate (Sudafed), which acts on
alpha-adrenergic receptors, causing urinary retention,
may be used to treat stress incontinence; it needs to
be used with caution in men with prostatic
hyperplasia.
Hormone therapy (eg, estrogen) taken orally,
transdermally, or topically was once the treatment of
choice for urinary incontinence in postmenopausal
women because it restores the mucosal, vascular, and
muscular integrity of the urethra.
223 Pharmacological treatment for BPH Proscar - long term - blocks testosterone formation
. causing regression of hyoerplastic tissue and
decreases the size of the prostate

Minipress, Hytrin, Flomax - relax smooth muscle and


increase urinary flow
224 The physician calls the nurse for an update on his 3. The Hemovac drain isn't compressed; instead it's
. client who underwent abdominal surgery five fully expanded.
hours ago. The physician asks the nurse for the The Hemovac must be compressed to establish
total amount of drainage collected in the Hemovac suction. If the Hemovac is allowed to fully expand
since surgery. The nurse reports that according to suction is no longer present causing the drain to
documentation, no drainage has been recorded. malfunction. The client who requires major
When the nurse finishes on the telephone, she goes abdominal surgery typically produces abdominal
to assess the client. Which assessment finding drainage despite the client's position. An NG tube
explains the absence of drainage? drains stomach contents, not incisional contents.
Therefore, the NG tube drainage of 400 ml is normal
1. The client has been lying on his side for two in this client and is not related to the absence of
hours with the drain positioned upward. Hemovac drainage. Dry drainage on the dressing
2. The client has a nasogastric (NG) tube in place indicates leakage from the incision; it isn't related to
that drained 400 ml. the Hemovac drainage.
3. The Hemovac drain isn't compressed; instead
it's fully expanded.
4. There is a moderate amount of dry drainage on
the outside of the dressing.
225 The physician enters a computer order for the nurse to 2. inappropriate because irrigation requires
. irrigate a client's nephrostomy tube every four hours to strict sterile technique.
maintain patency. The nurse irrigates the tube using sterile Irritating a nephrostomy tube requires strict
technique. After irrigating the tube, the nurse decides that sterile technique; therefore, reusing the
she can safely use the same irrigation set for her 8-hour irrigation set (even if covered by a sterile
shift if she covers the set with a paper, sterile drape. This drape) is inappropriate. Bacteria can
action by the nurse is: proliferate inside the syringe and irrigation
container. Although this procedure checks
1. appropriate because the irrigation just checks for patency, it requires sterile technique to
patency. prevent the introduction of bacteria into the
2. inappropriate because irrigation requires strict sterile kidney. The material in which the sterile
technique. drape is made is irrelevant because a sterile
3. appropriate because the irrigation set will only be used drape doesn't deter bacterial growth in the
during an 8-hour period. irrigation equipment.
4. inappropriate because the sterile drape must be cloth,
not paper.
226 The physician orders a stool culture to help diagnose a 3. collect the specimen in a sterile container.
. client with prolonged diarrhea. The nurse who obtains the The nurse should collect the stool specimen
stool specimen should: using sterile technique and a sterile stool
container. The stool may be collected for 3
1. take the specimen to the laboratory immediately. consecutive days; no follow-up care is
2. apply a solution to the stool specimen. needed. Although a stool culture should be
3. collect the specimen in a sterile container. taken to the laboratory as soon as possible, it
4. store the specimen on ice. need not be delivered immediately (unlike
stool being examined for ova and parasites).
Applying a solution to a stool specimen
would contaminate it; this procedure is done
when testing stool for occult blood, not
organisms. The nurse shouldn't store a stool
culture on ice because the abrupt temperature
change could kill the organisms.
227 The physician prescribes a single dose of 4. Call the hospital pharmacist and question
. trimethoprim/sulfamethoxazole (Bactrim) by mouth for a the medication supplied.
client diagnosed with an uncomplicated urinary tract The nurse should call the hospital pharmacy
infection (UTI). The pharmacy sends three unit-dose and question the medication supplied. The
tablets. The nurse verifies the physician's order. What hospital pharmacist should be able to tell the
should the nurse do next? nurse whether three tablets are necessary for
the single dose or whether a dispensing error
1. Administer the three tablets as the single dose. occurred. It isn't clear whether the three
2. Call the physician to verify the order. tablets are the single dose because they were
3. Give one tablet, three times per day. packaged as a unit-dose. The physician order
4. Call the hospital pharmacist and question the was clearly written, so clarifying the order
medication supplied. with the physician isn't necessary.
Administering the tablets without
clarification might cause a medication error.
228 Polycystic kidney disease A cystic formation and hypertrophy of the
. kidneys, which leads to cystic rupture,
infection, formation of scar tissue, and
damaged nephrons
There is no specific treatment to arrest the
progress of the destructive cysts. The
ultimate result of this disease is renal failure.
229 PostOp colostomy a. If a pouch system is not in place, apply a petroleum jelly gauze over the stoma to
. keep it moist, covered with a dry sterile dressing; place a pouch system on the
stoma as soon as possible.
b. Monitor the pouch system for proper fit and signs of leakage; empty the pouch
when one-third full.
c. Monitor the stoma for size, unusual bleeding, color changes, or necrotic tissue.
d. Note that the normal stoma color is red or pink, indicating high vascularity.
e. Note that a pale pink stoma indicates low hemoglobin and hematocrit levels.
f. Assess the functioning of the colostomy.
g. Expect that stool will be liquid postoperatively but will become more solid,
depending on the area of the colostomy.
h. Expect liquid stool from an ascending colon colostomy, loose to semiformed
stool from a transverse colon colostomy, or close to normal stool from a
descending colon colostomy.
i. Fecal matter should not be allowed to remain on the skin.
j. Administer analgesics and antibiotics as prescribed.
k. Irrigate perineal wound if present and if prescribed, and monitor for signs of
infection; provide comfort measures for perineal itching and pain.
l. Instruct the client to avoid foods that cause excessive gas formation and odor.
m. Instruct the client in stoma care and irrigations as prescribed.
n. Instruct the client on how to resume normal activities, including work, travel,
and sexual intercourse, as prescribed; provide psychosocial support.
230 PostOp ileostomy a. Healthy stoma is red; a color change to dark blue or black should be reported to
. the physician.
b. Postoperative drainage will be dark green and progress to yellow as the client
begins to eat.
c. Stool is liquid.
d. Risk for dehydration and electrolyte imbalance exists.
e. Do not administer medications such as suppositories through an ileostomy.
231 PostOp interventions 1. Assist the client to a prone or side-lying position to prevent bleeding.
. for rectal surgery 2. Maintain ice packs over the dressing as prescribed until the packing is removed
by the physician.
3. Monitor for urinary retention.
4. Administer stool softeners as prescribed.
5. Instruct the client to increase fluids and high-fiber foods.
6. Instruct the client to limit sitting to short periods of time.
7. Instruct the client in the use of sitz baths three or four times a day as prescribed.
232 PostOp Interventions for 1. Monitor vital signs.
. urinary diversion 2. Assess incision site.
3. Assess stoma (should be red and moist) every hour for the first 24 hours.
4. Monitor for edema in the stoma, which may be present in the immediate
postoperative period.
5. Notify the physician if the stoma appears dark and dusky (indicates necrosis).
6. Monitor for prolapse or retraction of the stoma.
7. Assess bowel function; monitor for expected return of peristalsis in 3 to 4
days.
8. Maintain NPO status as prescribed until bowel sounds return.
9. Monitor continuous urine flow (30 to 60 mL/ hr). 10. Notify the physician if
the urine output is less than 30 mL/ hr or if no urine output occurs for more than
15 minutes.
11. Ureteral stents or catheters, if present, may be in place for 2 to 3 weeks or
until healing occurs; maintain stability with catheters to prevent dislodgment.
12. Monitor for hematuria.
13. Monitor for signs of peritonitis.
14. Monitor for bladder distention following a partial cystectomy.
15. Monitor for shock, hemorrhage, thrombophlebitis, and lower extremity
lymphedema after a radical cystectomy.
16. Monitor the urinary drainage pouch for leaks, and check skin integrity.
17. Monitor the pH of the urine (do not place the dipstick in the stoma) because
strongly alkaline urine can cause skin irritation and facilitate crystal formation.
18. Instruct the client regarding the potential for urinary tract infection or the
development of calculi.
19. Instruct the client to assess the skin for irritation, monitor the urinary
drainage pouch, and report any leakage.
20. Encourage the client to express feelings about changes in body image,
embarrassment, and sexual dysfunction.
233 PostOp prostate surgery 1. Monitor vital signs.
. 2. Monitor urinary output and urine for hemorrhage or clots.
3. Increase fluids to 2400 to 3000 mL/ day, unless contraindicated.
4. Monitor for arterial bleeding as evidenced by bright red urine with numerous
clots; if it occurs, increase CBI and notify the physician immediately.
5. Monitor for venous bleeding as evidenced by burgundy-colored urine output;
if it occurs, inform the physician, who may apply traction on the catheter.
6. Monitor hemoglobin and hematocrit levels.
7. Expect red to light pink urine for 24 hours, turning to amber in 3 days.
8. Ambulate the client as early as possible and as soon as urine begins to clear in
color.
9. Inform the client that a continuous feeling of an urge to void is normal.
10. Instruct the client to avoid attempts to void around the catheter because this
will cause bladder spasms.
11. Administer antibiotics, analgesics, stool softeners, and antispasmodics as
prescribed.
12. Monitor the three-way Foley catheter, which usually has a 30- to 45-mL
retention balloon.
13. Maintain CBI with sterile bladder irrigation solution as prescribed to keep
the catheter free of obstruction and maintain the urine pink in color.
234 The purpose of giving neomycin before 3. Reduce the incidence of wound infections by decreasing the
. ileostomy surgery is to: number of intestinal organisms.
Neomycin is not effective in reducing postop atelectasis.
1. Decrease the incidence of The ability of the body to ward off infection is a result of the
postoperative atelectasis due to decrease in intestinal organisms.
decreased depth of respirations. Neomycin is administered postoperatively because it is a poorly
2. Increase the effectiveness of the absorbed antibiotic and therefore is effective in reducing the
body's immunologic response number of intestinal organisms that may cause infection of the
following surgical trauma. suture line.
3. Reduce the incidence of wound Bladder atony is generally due to decreased parasympathetic
infections by decreasing the number of outflow and bladder tone secondary to anesthesia.
intestinal organisms.
4. Prevent postoperative bladder atony
due to catheterization.
235 Pyelonephritis An inflammation of the renal pelvis and the parenchyma
. commonly caused by bacterial invasion.
Acute pyelonephritis often occurs after bacterial contamination of
the urethra or following an invasive procedure of the urinary tract.
Chronic pyelonephritis most commonly occurs following chronic
urinary flow obstruction with reflux. Escherichia coli is the most
common causative bacterial organism.
236 Questions to ask a patient with History of travel?
. diarrhea Diet?
Medications?
Previous surgery?
Family history?
Personal contacts?
237 reducible hernia can be manually replaced into cavity
.

238 The registered nurse and nursing 3. A 45-year-old client diagnosed with renal calculi who must
. assistant are caring for a group of ambulate four times daily and drink plenty of fluids.
clients. Which client's care can safely The care of the client in option 3 can safely be delegated to the
be delegated to the nursing assistant? nursing assistant. The client in option 1 had surgery 12 hours ago;
therefore, the registered nurse should care for the client because
1. A 35-year-old client who underwent the client requires close assessment. The client in option 2 also
surgery 12 hours ago and has a requires careful assessment by the registered nurse because the
suprapubic catheter in place that is client's diabetes mellitus is uncontrolled. In addition, the registered
draining burgundy colored urine nurse should care for the client in option 4 because the client
2. A 63-year-old client with requires neurological assessment, which isn't within the scope of
uncontrolled diabetes mellitus who practice for the nursing assistant.
underwent radical suprapubic
prostatectomy 1 day ago and has an
indwelling urinary catheter draining
yellow urine with clots
3. A 45-year-old client diagnosed with
renal calculi who must ambulate four
times daily and drink plenty of fluids.
4. A 19-year-old client who requires
neurological assessment every four
hours after sustaining a spinal cord
injury in a motor vehicle accident that
left him with paraplegia
239 Renal angiography An injection of a radiopaque dye through a catheter inserted into the femoral artery to
. examine the renal blood vessels and renal arterial supply

Assess and mark the peripheral pulses.


Maintain bed rest and apply a sandbag or other device that will provide pressure to
prevent bleeding, if prescribed, at the insertion site for 4 to 8 hours.
Instruct the client to maintain a supine position with the leg straight (the head of the
bed should not be elevated greater than 20 degrees for 8 hours, (or as prescribed).
Assess the temperature, color, movement, and sensation (CMS) of the toes of the
involved extremity with each vital sign check.
Inspect the catheter insertion site for bleeding or swelling with each vital sign check.
240 Renal biopsy Insertion of a needle into the kidney to obtain a sample of tissue for examination;
. usually done percutaneously

Preprocedure interventions:
Assess vital signs.
Assess baseline coagulation studies; notify the physician if abnormal results are
noted.
Obtain an informed consent.
NPO

Postprocedure interventions:
Monitor vital signs, especially for hypotension and tachycardia, which could indicate
bleeding.
Provide pressure to the biopsy site for 30 minutes.
Monitor the hemoglobin and hematocrit levels for decreases, which could indicate
bleeding.
Place the client in the supine position and on bed rest for 8 hours as prescribed.
Check the biopsy site and under the client for bleeding.
Encourage fluid intake of 1500 to 2000 mL as prescribed.
Observe the urine for gross and microscopic bleeding.
Instruct the client to avoid heavy lifting and strenuous activity for 2 weeks.
Instruct the client to notify the physician if either a temperature greater than 100 F
or hematuria occurs after the first 24 hours postprocedure.
241 Risk factors for 1. Family history of stone formation
. nephrolithiasis 2. Diet high in calcium, vitamin D, protein, oxalate, purines, or alkali
3. Obstruction and urinary stasis
4. Dehydration
5. Use of diuretics, which can cause volume depletion
6. Urinary tract infections and prolonged urinary catheterization
7. Immobilization
8. Hypercalcemia and hyperparathyroidism
9. Elevated uric acid level, such as in gout
242 Signs of Bowel Guarding of the abdomen
. Perforation and Increased fever and chills
Peritonitis Pallor
Progressive abdominal distention and abdominal pain
Restlessness
Tachycardia and tachypnea
243 The spouse of a client who had a transurethral 4. "The rate of irrigant should be slowed when the
. resection of the prostate (TURP) 24 hours ago is drainage is pale pink."
upset because the irrigation seems to be increasing The nurse cannot stop the irrigation without an
the client's pain. Which is the best response by a order.
nurse? The nurse does not have the authority to add
anesthetic.
1. "I will stop the bladder irrigation and the pain The purpose of the irrigant is to prevent clots. A
should subside." pale pink would indicate that bleeding is
2. "I will add a local anesthetic to the irrigant to diminishing.
treat the pain."
3. "Perhaps you should go home and get some rest."
4. "The rate of irrigant should be slowed when the
drainage is pale pink."
244 strangulated hernia irreducible with obstruction to blood supply -
. surgical emergency
245 Stress incontinence the involuntary loss of urine through an intact
. urethra as a result of sneezing, coughing, or
changing position
246 Surgical interventions for ulcerative colitis Total proctocolectomy with permanent ileostomy
. Kock ileostomy (continent ileostomy)
Ileoanal reservoir
247 Total proctocolectomy with permanent ileostomy surgical intervention for ulcerative colitis
. The procedure is curative and involves the removal
of the entire colon.
The end of the terminal ileum forms the stoma,
which is located in the right lower quadrant.
248 Transurethral resection of the prostate (TURP) Removal of benign prostatic tissue surrounding the
. urethra with use of a resectoscope introduced
through the urethra; there is little risk of impotence
and it is most commonly used for BPH.
249 Treatment for polycystic kidney disease nephrectomy
. dialysis
250 A triple-lumen indwelling urinary catheter is 1. continuous inflow and outflow of irrigation
. inserted for continuous bladder irrigation following solution.
a transurethral resection of the prostate. In addition When preparing for continuous bladder irrigation, a
to balloon inflation, the functions of the three lumens triple-lumen indwelling urinary catheter is inserted.
include: The three lumens provide for balloon inflation and
continuous inflow and outflow of irrigation
1. continuous inflow and outflow of irrigation solution.
solution.
2. intermittent inflow and continuous outflow of
irrigation solution.
3. continuous inflow and intermittent outflow of
irrigation solution.
4. intermittent flow of irrigation solution and
prevention of hemorrhage.
251 Two weeks after being diagnosed with a streptococcal 1. Place the client on bed rest.
. infection, a client develops fatigue, a low-grade fever, and The nurse immediately must enforce bed rest
shortness of breath. The nurse auscultates bilateral for a client with glomerulonephritis to ensure
crackles and observes neck vein distention. Urinalysis a complete recovery and help prevent
reveals red and white blood cells and protein. After the complications. Depending on disease severity,
physician diagnoses poststreptococcal glomerulonephritis, the client may require fluid, sodium,
the client is admitted to the medical-surgical unit. Which potassium, and protein restrictions. Because of
immediate action should the nurse take? the risk of altered urinary elimination related
to oliguria, this client may require
1. Place the client on bed rest. hemodialysis or plasmapheresis for several
2. Provide a high-protein, fluid-restricted diet. weeks until renal function improves; however,
3. Prepare to assist with insertion of a Tenckhoff catheter a Tenckhoff catheter is used in peritoneal
for hemodialysis. dialysis, not hemodialysis. Although comfort
4. Place the client on a sheepskin, and monitor for measures such as placing the client on a
increasing edema. sheepskin are important, they don't take
precedence.
252 ulcerative colitis Ulcerative and inflammatory disease of the
. bowel that results in poor absorption of
nutrients. Acute ulcerative colitis results in
vascular congestion, hemorrhage, edema, and
ulceration of the bowel mucosa. Chronic
ulcerative colitis causes muscular hypertrophy,
fat deposits, and fibrous tissue with bowel
thickening, shortening, and narrowing.
253 Ureterolithotomy/pyelolithotomy and nephrolithotomy An open surgical procedure performed if
. lithotripsy is not effective for removal of a
stone in the ureter, renal pelvis, or kidney.
254 Urge incontinence the involuntary loss of urine associated with a
. strong urge to void that cannot be suppressed.
The patient is aware of the need to void but is
unable to reach a toilet in time
255 What is the primary nursing diagnosis for a client with a 1. Deficient fluid volume
. bowel obstruction? Feces, fluid, and gas accumulate above a
bowel obstruction. Then the absorption of
1. Deficient fluid volume fluids decreases and gastric secretions
2. Deficient knowledge increase. This process leads to a loss of fluids
3. Acute pain and electrolytes in circulation. Options 2, 3,
4. Ineffective tissue perfusion and 4 are applicable but not the primary
nursing diagnosis.
256 What nursing action best facilitates the passage of the NG 2. Positioning the client on the right side for 2
. tube from the stomach though the pylorous and into the hours after insertion.
duodenum?

1. Gently advancing the tube 1 to 4 inches at the regular


time intervals.
2. Positioning the client on the right side for 2 hours after
insertion.
3. Maintaining strict bedrest and avoiding all unnecessary
movement.
4. Positioning the client in a flat supine position.
257 What should be the first nursing action if an NG tube is 3. Determine tube placement.
. not draining? Placement must be confirmed before
irrigating.
1. Irrigate the tube. The lack of drainage may mean the tube is no
2. Reposition the client. longer in the correct location.
3. Determine tube placement. Placement must be determined before
4. Remove the tube and reinsert. removal.
258 What should the nurse include in health 4. Maintain adequate fluid intake.
. teaching for a client who has an ileostomy? High-fiber foods should be avoided.
Laxatives are not needed; drainage is liquid. Increased
1. Eat high-fiber foods. peristalsis may contribute to FVD and discomfort.
2. Take laxatives for any blockage. Certain raw fruits and vegetables have high cellulose
3. Eat raw fruits and vegetables. content and should be avoided because they could cause
4. Maintain adequate fluid intake. blockage.
Due to lack of reabsorption of fluid from the large intestine,
the client is at risk for FVD.
259 When a client with an indwelling urinary 2. The client keeps the drainage bag below the bladder at all
. catheter insists on walking to the hospital times.
lobby to visit with family members, the nurse To maintain effective drainage, the client should keep the
teaches him how to do this without drainage bag below the bladder; this allows the urine to
compromising the catheter. Which client flow by gravity from the bladder to the drainage bag. The
action indicates an accurate understanding of client shouldn't lay the drainage bag on the floor because it
this information? could become grossly contaminated. The client shouldn't
clamp the catheter drainage tubing because this impedes the
1. The client sets the drainage bag on the flow of urine. To promote drainage, the client may loop the
floor while sitting down. drainage tubing above not below its point of entry
2. The client keeps the drainage bag below into the drainage bag.
the bladder at all times.
3. The client clamps the catheter drainage
tubing while visiting with the family.
4. The client loops the drainage tubing below
its point of entry into the drainage bag.
260 When planning care for a client with a small- 3. maintaining fluid balance.
. bowel obstruction, the nurse should consider Because a client with a small-bowel obstruction can't
the primary goal to be: tolerate oral intake, fluid volume deficit may occur and can
be life-threatening. Therefore, maintaining fluid balance is
1. reporting pain relief. the primary goal. The other options are secondary; pain
2. maintaining body weight. relief and maintaining body weight don't reflect life-
3. maintaining fluid balance. threatening conditions, and the client's normal bowel
4. reestablishing a normal bowel pattern. pattern can be reestablished after fluid volume is stabilized.
261 When preparing a client, age 50, for surgery 2. Obstruction of the appendix reduces arterial flow, leading
. to treat appendicitis, the nurse formulates a to ischemia, inflammation, and rupture of the appendix.
nursing diagnosis of Risk for infection A client with appendicitis is at risk for infection related to
related to inflammation, perforation, and inflammation, perforation, and surgery because obstruction
surgery. What is the rationale for choosing of the appendix causes mucus fluid to build up, increasing
this nursing diagnosis? pressure in the appendix and compressing venous outflow
drainage. The pressure continues to rise with venous
1. Obstruction of the appendix may increase obstruction; arterial blood flow then decreases, leading to
venous drainage and cause the appendix to ischemia from lack of perfusion. Inflammation and
rupture. bacterial growth follow, and swelling continues to raise
2. Obstruction of the appendix reduces pressure within the appendix, resulting in gangrene and
arterial flow, leading to ischemia, rupture. Geriatric, not middle-aged, clients are especially
inflammation, and rupture of the appendix. susceptible to appendix rupture.
3. The appendix may develop gangrene and
rupture, especially in a middle-aged client.
4. Infection of the appendix diminishes
necrotic arterial blood flow and increases
venous drainage.
262 When preparing a client for a 1. Administer an enema as ordered.
. hemorrhoidectomy, the nurse When preparing a client for a hemorrhoidectomy, the nurse should
should take which action? administer an enema, as ordered, and record the results. After surgery,
the client may require antibiotics and analgesics.
1. Administer an enema as ordered.
2. Administer oral antibiotics as
prescribed.
3. Administer topical antibiotics as
prescribed.
4. Administer analgesics as
prescribed.
263 Which conditions are functions of 2. Water reabsorption and urine concentration
. antidiuretic hormone (ADH)? ADH stimulates the renal tubules to reabsorb water, thereby
concentrating urine. Aldosterone is responsible for sodium
1. Sodium absorption and reabsorption and potassium excretion by the kidneys.
potassium excretion
2. Water reabsorption and urine
concentration
3. Water reabsorption and urine
dilution
4. Sodium reabsorption and
potassium retention
264 Which food should the nurse advise 1. Carbonated drinks.
. a client with a colostomy to avoid? Carbonated drinks, cabbage, sauerkraut, and nuts tend to increase
flatulence, and most clients feel uncomfortable passing flatus into the
1. Carbonated drinks. colostomy bag, because it causes the bag to inflate. Onions, cheese,
2. Fresh-cooked green beans. and fish may cause odorous drainage.
3. Liver and bacon.
4. Cooked cereals.
265 Which infection control equipment 1. Gloves
. is necessary for the client diagnosed The client diagnosed with Clostridium difficile diarrhea requires
with Clostridium difficile diarrhea? contact isolation. Contact isolation precautions require the use of
glove and a gown if soiling is likely. A mask, face shield, and N-95
1. Gloves respirator aren't necessary to maintain contact isolation.
2. Mask
3. Face shield
4. N-95 respirator
266 Which laboratory test is the most 2. Creatinine clearance
. accurate indicator of a client's renal Creatinine clearance is the most accurate indicator of a client's renal
function? function because it closely correlates with the kidney's glomerular
filtration rate and tubular excretion ability. Results from the other
1. Blood urea nitrogen options may be influenced by various conditions and aren't specific to
2. Creatinine clearance renal disease.
3. Serum creatinine
4. Urinalysis
2 Which laboratory value 3. Pyuria
6 supports a diagnosis of Pyelonephritis is diagnosed by the presence of leukocytosis, hematuria,
7 pyelonephritis? pyuria, and bacteriuria. The client exhibits fever, chills, and flank pain.
. Because there is often a septic picture, the WBC count is more likely to be
1. Myoglobinuria high rather than low, as indicated in option 4. Ketonuria indicates a diabetic
2. Ketonuria state.
3. Pyuria
4. Low white blood cell (WBC)
count
2 Which medication should the 4. docusate sodium (Colace)
6 nurse expect to administer to a Docusate sodium, a laxative, is used to treat constipation. It softens the stool
8 client with constipation? by stimulating the secretion of intestinal fluid into the stool. Lorazepam, an
. antianxiety agent, has no laxative effect. Administering loperamide, an
1. lorazepam (Ativan) antidiarrheal agent, could cause the constipation to worsen. Flurbiprofen is a
2. loperamide (Imodium) nonsteroidal anti-inflammatory agent with no laxative effect.
3. flurbiprofen (Ansaid)
4. docusate sodium (Colace)
2 Which of the following is an 3. Risk for infection
6 appropriate nursing diagnosis Infection can occur with renal calculi from urine stasis caused by obstruction.
9 for a client with renal calculi? Options 1 and 4 aren't appropriate for this client, and retention of urine,
. rather than incontinence, usually occurs.
1. Ineffective renal tissue
perfusion
2. Functional urinary
incontinence
3. Risk for infection
4. Decreased cardiac output
2 Which outcome indicates 4. The client reports engaging in a regular exercise regimen.
7 effective client teaching to A regular exercise regimen promotes peristalsis and contributes to regular
0 prevent constipation? bowel elimination patterns. A low-fiber diet, a sedentary lifestyle, and limited
. water intake would predispose the client to constipation.
1. The client verbalizes
consumption of low-fiber
foods.
2. The client maintains a
sedentary lifestyle.
3. The client limits water
intake to three glasses per day.
4. The client reports engaging
in a regular exercise regimen.
2 Which statement best describes 3. They block sodium reabsorption in the ascending loop and dilate renal
7 the therapeutic action of loop vessels.
1 diuretics? Loop diuretics block sodium reabsorption in the ascending loop of Henle,
. which promotes water diuresis. They also dilate renal vessels. Loop diuretics
1. They block reabsorption of block potassium reabsorption, but this isn't a therapeutic effect. Thiazide
potassium on the collecting diuretics promote sodium secretion into the distal tubule.
tubule.
2. They promote sodium
secretion into the distal tubule.
3. They block sodium
reabsorption in the ascending
loop and dilate renal vessels.
4. They promote potassium
secretion into the distal tubule
and constrict renal vessels.
2 Which statement by the nurse 3. It has two stomas. A proximal loop discharges feces, and a distal loop
7 correctly describes a double- discharges mucus.
2 barrel colostomy? Number 1 is an example of an ascending colostomy.
. 1. It is the least common type of Number 2 is a transverse loop colostomy.
colostomy, and it discharges Number 4 is a descending colostomy.
liquid or unformed stool.
2. A single loop of the
transverse colon is exteriorized
and supported by a glass rod.
There are two openings, a
proximal loop and distal loop.
3. It has two stomas. A
proximal loop discharges feces,
and a distal loop discharges
mucus.
4. It is most often permanent
and is done to treat disorders of
the sigmoid colon.
2 Which statement describing 2. Urinary incontinence isn't a disease.
7 urinary incontinence in the Urinary incontinence isn't a normal part of aging nor is it a disease. It may
3 elderly is true? be caused by confusion, dehydration, fecal impaction, restricted mobility, or
. other causes. Certain medications, including diuretics, hypnotics, sedatives,
1. Urinary incontinence is a anticholinergics, and antihypertensives, may trigger urinary incontinence.
normal part of aging. Most clients with urinary incontinence can be treated; some can be cured.
2. Urinary incontinence isn't a
disease.
3. Urinary incontinence in the
elderly can't be treated.
4. Urinary incontinence is a
disease.
After Left-sided heart failure
an Explanation: The left ventricle is responsible for most of the cardiac output. An anterior
anterio wall MI may result in a decrease in left ventricular function. When the left ventricle
r wall doesn't function properly, resulting in left-sided heart failure, fluid accumulates in the
myocar interstitial and alveolar spaces in the lungs and causes crackles. Pulmonic and tricuspid
dial valve malfunction causes right-sided heart failure.
infarcti
on
(MI),
which
proble
m is
indicat
ed by
auscult
ation of
crackle
s in the
lungs?
2. Assisting the Deep-breathing exercises are beneficial to promoting rest as they help the
client with client to relax. The client's door should be closed to reduce noise and
deep- distractions. Tea contains caffeine, which acts as a stimulant. While
breathing sedatives may be used occasionally for assistance with rest, regular use
exercises isn't advised because dependence may develop.
3. candidiasis yeast infection; infections occurring on the skin or mucous membranes in
the warm, moist areas such as the vagina or the mouth
4. A child 2 p.m. and 5 p.m.
diagnosed Explanation: The action of an intermediate-acting insulin such as insulin
with insulin- zinc suspension (Humulin L) begins 1 to 3 hours after injection and peaks
dependent 6 to 12 hours after injection. The child is most at risk for hypoglycemia at
diabetes the times when the insulin would peak, in this case beginning between 2
mellitus is p.m. and 5 p.m.
attending a
camp for
diabetic
children. He
gives himself
regular
insulin and
insulin zinc
suspension
(Humulin L)
at 8 a.m. The
nurse should
plan to
observe him
for signs and
symptoms of
hypoglycemia
resulting
from the
effects of the
Humulin L
insulin
between
which of the
following
times?
5. circadian the biological clock; regular bodily rhythms that occur on a 24-hour cycle
rhythm
6. A client Ask the client for a copy of the advance directive to place on his chart.
admitted to Explanation: Upon admission, a client should be asked if he has an
the hospital advance directive and informed of his right to create one. If the client has
for an an advance directive, a copy of the document must be placed in the
abdominal medical record. It isn't enough just to note that the client has one. It would
aneurysm be incorrect to tell the client to give the directive to his lawyer or to imply
repair tells a that the directive isn't valid when surgery is being performed.
nurse that he
has an
advance
directive.
What action
should the
nurse take?
7. A client at 33 The neonate will be delivered with mature lungs.
weeks' Explanation: Betamethasone is a corticosteroid that induces the
gestation is production of surfactant. The pulmonary maturation that results causes
admitted in the fetal lungs to mature more rapidly than normal. Because the lungs are
preterm mature, the risk of respiratory distress in the neonate is lowered but not
labor. She is eliminated. Betamethasone also decreases the surface tension within the
given alveoli. Betamethasone has no influence on contractions or carrying the
betamethason fetus to full term. It also does not prevent infection.
e (Celestone)
12 mg I.M. q
24 hours 2.
What is the
expected
outcome of
this drug
therapy?
8. A client has Serving fluids in large amounts
been Explanation: Fluids should be served in small amounts at frequent
admitted to intervals. It's overwhelming to the client to have large amounts of fluids
the hospital to drink. Teaching the client about the need for fluid increase and
with signs of including him in the selection of beverages will enhance compliance.
dehydration. Fluids should be served at the appropriate temperatures to increase
Which action enjoyment and palatability.
would be
least
beneficial in
increasing
the client's
fluid intake?
9. A client is water and sodium retention secondary to a severe decrease in the
admitted for glomerular filtration rate.
treatment of Explanation: A client with CRF is at risk for fluid imbalance
chronic renal dehydration if the kidneys fail to concentrate urine, or fluid retention if
failure the kidneys fail to produce urine. Electrolyte imbalances associated with
(CRF). The this disorder result from the kidneys' inability to excrete phosphorus; such
nurse knows imbalances may lead to hyperphosphatemia with reciprocal
that this hypocalcemia. CRF may cause metabolic acidosis, not metabolic
disorder alkalosis, secondary to the inability of the kidneys to excrete hydrogen
increases the ions.
client's risk
of:
1 A client is diagnosed Ineffective peripheral tissue perfusion related to venous congestion
0 with deep vein Explanation: Ineffective peripheral tissue perfusion related to venous congestion takes
. thrombosis. Which highest priority because venous inflammation and clot formation impede blood flow
nursing diagnosis in a client with deep vein thrombosis. Option 1 is incorrect because impaired gas
should receive highest exchange is related to decreased, not increased, blood flow. Option 2 is inappropriate
priority at this time? because no evidence suggests that this client has a excessive fluid volume. Option 3
may be warranted but is secondary to ineffective tissue perfusion.
1 A client is prescribed lactic acidosis
1 metformin
. (Glucophage) to control
type 2 diabetes. The
nurse should monitor
for which life-
threatening adverse
reaction?
1 A client states that he The client can read from 20' (6 m) what a person with normal vision can read from
2 has 20/40 vision. Which 40'.
. statement about this Explanation: The numerator, which is always 20, is the distance in feet between the
client's vision is true? vision chart and the client. The denominator indicates from what distance an eye with
normal vision can read the chart. Normal vision is defined as 20/20. The other options
are inaccurate.
1 A client with a history Hyperkalemia
3 of chronic renal failure Explanation: The kidneys are responsible for excreting potassium. In renal failure, the
. is admitted with kidneys can no longer excrete potassium, resulting in hyperkalemia. The kidneys are
pulmonary edema responsible for regulating the acid-base balance; in renal failure, acidemia, not
following a missed alkalemia, would be likely. Generally, hyponatremia, not hypernatremia, would occur
dialysis treatment because of the dilutional effect of water retention. Hypokalemia is generally seen in
yesterday. His clients undergoing diuresis.
laboratory results are
serum potassium 6.0
mEg/L, serum sodium
130 mEg/L, and serum
bicarbonate 18 mEg/L.
The nurse interprets
that the client has
which of the following
conditions?
1 A client with a recent The client's arms are fully extended when using the walker.
4 history of a stroke has Explanation: When using a walker, the client's arms should be slightly bent at the
. been discharged from elbow, allowing maximum support from the arms while ambulating. The weak leg is
the rehabilitation always moved forward first with the walker to provide the maximum support. When
facility with a walker. sitting, the client should always back up to the chair and feel the chair with his legs
During the client's before sitting. The client should use the armrests of the chair for support because the
return visit to the armrests are more stable than the walker.
physician's office, the
nurse assesses his gait.
Which finding indicates
the need for further
teaching about walker
use?
1 Deficient fluid volume Clients with acute pancreatitis commonly experience deficient fluid volume, which
5 can lead to hypovolemic shock. The volume deficit may be caused by vomiting,
. hemorrhage (in hemorrhagic pancreatitis), and plasma leaking into the peritoneal
cavity. Hypovolemic shock will cause a decrease in cardiac output. Tissue perfusion
will be ineffective if hypovolemic shock occurs, but this wouldn't be the primary
nursing diagnosis.
1 dividing the body into A frontal or coronal plane runs longitudinally at a right angle to a sagittal plane,
6 sections dividing the body into anterior and posterior regions. A sagittal plane runs
. longitudinally, dividing the body into right and left regions; if exactly midline, it is
called a midsagittal plane. A transverse plane runs horizontally at a right angle to the
vertical axis, dividing the structure into superior and inferior regions.
1 An elderly client tells Explaining why a bath is important to overall health, and telling the client that she'll
7 the nurse that he return in 30 minutes to help him bathe
. doesn't want to take a Explanation: It's important for the client to understand why a bath is important to
bath. Which action by overall health. Communicating with the client shows respect and aids compliance.
the nurse is most Giving the client a specific time for the bath allows him time to prepare for the care.
appropriate? Documenting bath refusal, calling the physician, and contacting family members are
inappropriate before discussing the importance of the bath with the client and
reattempting to provide care.
18 Evaluation During the evaluation step of the nursing process, the nurse determines
. whether the goals established in the plan of care have been achieved
and evaluates the success of the plan. If a goal is unmet or partially
met, the nurse reexamines the data and revises the plan. Data collection
involves gathering relevant information about the patient. Planning
involves setting priorities, establishing goals, and selecting appropriate
interventions. Implementation involves providing actual nursing care.
19 The following statement appears on a client outcome.
. a client's plan of care: "Client will Explanation: A client outcome is a short- or long-term goal based on
ambulate in the hall without projected nursing interventions. A nursing diagnosis is a statement
assistance within 4 days." This about a client's actual or potential problem. Subjective data are
statement is an example of: information relayed to the nurse by the client. A nursing intervention is
an action the nurse takes in response to a client's problem.
20 hypoxia. Clubbing is a sign of prolonged hypoxia. Causes of clubbing include
. emphysema, chronic bronchitis, lung cancer, and heart failure. Beau's
lines (transverse depressions in the nail that extend beyond the nail
bed) occur with acute illness, malnutrition, and anemia. Koilonychia
(thin, spoon-shaped nails with lateral edges that tilt upward) is
associated with Raynaud's disease, malnutrition, chronic infections,
and hypochromic anemia. Onycholysis (loosening of the nail plate
with separation from the nail bed) is associated with hyperthyroidism,
psoriasis, contact dermatitis, and Pseudomonas infections.
21 Ineffective peripheral tissue Ineffective peripheral tissue perfusion related to venous congestion
. perfusion related to venous takes highest priority because venous inflammation and clot formation
congestion impede blood flow in a client with deep vein thrombosis. Option 1 is
incorrect because impaired gas exchange is related to decreased, not
increased, blood flow. Option 2 is inappropriate because no evidence
suggests that this client has a excessive fluid volume. Option 3 may be
warranted but is secondary to ineffective tissue perfusion.
22 Lactic acidosis The nurse should monitor the client for signs of lactic acidosis, a life-
. threatening adverse reaction associated with metformin. Nausea,
vomiting, and megaloblastic anemia are adverse reactions associated
with metformin, but they aren't considered life-threatening.
23 middle-old Ages 75-84
.

24 Notify the nursing supervisor and Approaching the person and requesting the client's medical record isn't
. approach the individual. sufficient considering the confidential health care information.
Notifying the nursing supervisor, then approaching the individual
before informing the client provides the most appropriate approach to
this breech of client confidentiality. Contacting security might not be
warranted unless the nurse learns the reason the unauthorized
individual was reading the client's chart. The nurse should also
document the incident according to facility policy.
25 The nurse-client relationship Two major clinical characteristics affect client compliance: the nurse-
. client relationship and the therapeutic regimen. The client's drug
knowledge, psychosocial factors, and disease duration and severity are
client characteristics, not clinical ones.
26 A nurse discovers that a stat dose of Notify the charge nurse so she can notify the physician of the missed
. potassium chloride that was dose.
prescribed by the physician was Explanation: An error was made that needs to be addressed by
never administered. Which action notifying the charge nurse. The charge nurse should then notify the
should the nurse take? physician to determine if the medication is still appropriate for the
client, and then request the medication from the pharmacy if it's still
needed. The physician might order a potassium level to see if the dose
is sufficient for the client. It isn't appropriate to ask the client if the
medication is still needed. After the charge nurse and physician have
been notified, the nurse should document the incident according to
facility policy.
27 The nurse is caring for a client who progressively deeper breaths followed by shallower breaths with
. has suffered a severe stroke. apneic periods.
During data collection, the nurse Explanation: Cheyne-Stokes respirations are breaths that become
notices Cheyne-Stokes respirations. progressively deeper followed by shallower respirations with apneic
Cheyne- Stokes respirations are: periods. Biot's respirations are rapid, deep breaths with abrupt pauses
between each breath, and equal depth between each breath. Kussmaul's
respirations are rapid, deep breaths without pauses. Tachypnea is
abnormally rapid respirations.
28 The nurse is caring for a client who required a chest X-ray.
. chest tube insertion for a pneumothorax. To Explanation: Chest X-ray confirms whether the chest
assess a client for pneumothorax resolution after tube has resolved the pneumothorax. If the chest tube
the procedure, the nurse can anticipate that he'll hasn't resolved the pneumothorax, the chest X-ray will
require: reveal air or fluid in the pleural space. SaO2 values may
initially decrease with a pneumothorax but typically
return to normal in 24 hours. ABG levels may show
hypoxemia, possibly with respiratory acidosis and
hypercapnia not related to a pneumothorax. Chest
auscultation will determine overall lung status, but it's
difficult to determine if the chest is reexpanded
sufficiently.
29 The nurse is caring for a client with acute Increasing fluid intake to 3 L/day
. pyelonephritis. Which nursing intervention is Explanation: Acute pyelonephritis is a sudden
most important? inflammation of the interstitial tissue and renal pelvis of
one or both kidneys. Infecting bacteria are normal
intestinal and fecal flora that grow readily in urine.
Pyelonephritis may result from procedures that involve
the use of instruments (such as catheterization,
cystoscopy, and urologic surgery) or from hematogenic
infection. The most important nursing intervention is to
increase fluid intake to 3 L/day. This helps empty the
bladder of contaminated urine and prevents calculus
formation. Administering a sitz bath would increase the
likelihood of fecal contamination. Using an indwelling
urinary catheter could cause further contamination.
Encouraging the client to drink cranberry juice to acidify
urine is helpful but isn't the most important intervention.
30 The nurse is helping a client ambulate for the The client's pulse and respiratory rates increased
. first time after 3 days of bed rest. Which moderately during ambulation.
observation by the nurse suggests that the client Explanation: The pulse and respiratory rates normally
tolerated the activity without distress? increase during and for a short time after ambulation,
especially if it's the first ambulation after 3 days of bed
rest. A normal walking pace is 70 to 100 steps/minute; a
much slower pace may indicate distress. Dizziness,
weakness, and profuse perspiration are definite signs of
activity intolerance. A client who tolerates ambulation
well holds the head erect, gazes straight ahead, and keeps
the toes pointed forward; option 3 describes a client with
activity intolerance.
31 The nurse is revising a client's plan of care. During the evaluation step of the nursing process, the
. During which step of the nursing process does nurse determines whether the goals established in the
such revision take place? plan of care have been achieved and evaluates the
success of the plan. If a goal is unmet or partially met,
the nurse reexamines the data and revises the plan. Data
collection involves gathering relevant information about
the patient. Planning involves setting priorities,
establishing goals, and selecting appropriate
interventions. Implementation involves providing actual
nursing care.
32 A nurse is working with the family of a client Encouraging the spouse to talk about the difficulties
. who has Alzheimer's disease. The nurse notes involved in caring for a loved one
that the client's spouse is too exhausted to Asking whether friends or church members can help
continue providing care all alone. The adult with errands or provide short periods of relief
children live too far away to provide relief on a Explanation: Many community services exist for
weekly basis. Which nursing interventions Alzheimer's clients and their families. Encouraging use
would be most helpful? Select all that apply. of these resources may make it possible to keep the client
Recommending community resources for adult at home and to alleviate the spouse's exhaustion. The
day care and respite care nurse can also support the caregiver by urging her to talk
about the difficulties she's facing in caring for her spouse.
Friends and church members may be able to help provide
care to the client, allowing the caregiver time for rest,
exercise, or an enjoyable activity. Arranging a family
meeting to tell the children to participate more would
probably be ineffective and might evoke anger or guilt.
Counseling might be helpful, but it wouldn't alleviate the
caregiver's physical exhaustion or address the client's
immediate needs. A long-term care facility isn't an option
until the family is ready to make that decision.
33 The nurse sees an unauthorized person reading Notify the nursing supervisor and approach the
. a client's medical record outside a client's room. individual.
Which action should the nurse take? Explanation: Approaching the person and requesting the
client's medical record isn't sufficient considering the
confidential health care information. Notifying the
nursing supervisor, then approaching the individual
before informing the client provides the most appropriate
approach to this breech of client confidentiality.
Contacting security might not be warranted unless the
nurse learns the reason the unauthorized individual was
reading the client's chart. The nurse should also
document the incident according to facility policy.
34 old-old 85 years of age and older
.

3 A patient develops diarrhea secondary to dizziness.


5 antibiotic therapy. He is to receive two tablets of Explanation: The most common adverse effects of
. diphenoxylate HCl with atropine sulfate (Lomotil) diphenoxylate HCl with atropine sulfate are drowsiness
orally as needed for each loose stool. The nurse and dizziness related to the drug's chemical similarity to
should inform him that he may experience meperidine, an opioid. Tachycardia is an adverse effect,
not bradycardia. Muscle aches and an increase in appetite
are not adverse effects of the drug.
3 The physician prescribes a loop diuretic for a Hypokalemia
6 client. When administering this drug, the nurse Explanation: A loop diuretic removes water and, along
. anticipates that the client may develop which with it, sodium and potassium. This may result in
electrolyte imbalance? hypokalemia, hypovolemia, and hyponatremia.
3 Placing the client in a side-lying position An unconscious client is at risk for aspiration. To
7 decrease this risk, the nurse should place the client in a
. side-lying position when performing oral hygiene; doing
so allows fluid to drain from the mouth, preventing
aspiration. Swabbing the client's lips, teeth, and gums
with lemon glycerin would promote tooth decay.
Cleaning the tongue with gloved fingers wouldn't be
effective in removing oral secretions or debris in an
unconscious client. Placing the client in semi-Fowler's
position would increase the risk of aspiration.
3 Reinforce the dressing and contact the physician. The nurse should reinforce the dressing and notify the
8 physician. A saturated dressing might signal
. postoperative hemorrhage. Continuing to monitor the
client without notifying the physician delays treatment.
The nurse should also monitor the client's vital signs. The
first postoperative dressing should be changed by the
physician, not the nurse.
3 "Remember to hold the cane with the hand on the The nurse should remind the client to hold the cane with
9 opposite side of your weak leg." the hand on the opposite side of the weak leg. Telling the
. client that the cane is temporary offers false reassurance.
Safe cane walking requires the client to hold the cane on
the side opposite the disability.
4 Restlessness Early in shock, hyperactivity of the sympathetic nervous
0 system causes increased epinephrine secretion, which
. typically makes the client restless, anxious, nervous, and
irritable. It also decreases tissue perfusion to the skin,
causing pale, cool, clammy skin. An above-normal heart
rate is a late sign of shock. A urine output of 30 ml/hour
is within normal limits.
4 roll the vial gently between the palms. Rolling the vial gently between the palms produces heat,
1 which helps dissolve the medication. Doing nothing or
. inverting the vial wouldn't help dissolve the medication.
Shaking the vial vigorously could cause the medication to
break down, altering its action.
4 stethescope; diaphragm; bell and palpation The diaphragm of a stethoscope detects high-pitched
2 sounds best; the bell detects low-pitched sounds best.
. Palpation detects thrills best.
4 To avoid recording an erroneously low systolic inflate the cuff at least another 30 mm Hg after the radial
3 blood pressure because of failure to recognize an pulse becomes impalpable.
. auscultatory gap, the nurse should: Explanation: The nurse should wrap an appropriately
sized cuff around the client's upper arm and then place
the diaphragm of the stethoscope over the brachial artery.
The nurse should then rapidly inflate the cuff until she
can no longer palpate or auscultate the pulse and continue
inflating until the pressure rises another 30 mm Hg. The
other options aren't appropriate measures.
4 The tongue In many cases, the muscles controlling the tongue relax,
4 causing the tongue to obstruct the airway. When this
. occurs, the nurse should use the head-tilt, chin-lift
maneuver to cause the tongue to fall back in place. If a
neck injury is suspected, the jaw- thrust maneuver must
be performed. A foreign object, saliva or mucus, and
edema are less common sources of airway obstruction in
an unconscious adult.
4 Two days after undergoing a total abdominal left calf circumference 1" (2.5 cm) larger than the right.
5 hysterectomy, a client complains of left calf pain, Explanation: Signs of DVT include inflammation and
. and venography reveals deep vein thrombosis edema in the affected extremity, causing its
(DVT). When checking this client, the nurse is circumference to exceed that of the opposite extremity.
most likely to detect: Pallor, coolness, decreased pulse, and hair loss in an
extremity signal interrupted arterial blood flow, which
doesn't occur in DVT.
4 When checking a client's I.V. insertion site, the Check the tubing for kinks and reposition the client's
6 nurse notes normal color and temperature at the wrist and elbow.
. site and no swelling. However, the I.V. solutions Explanation: The nurse should first check for common
haven't infused at the ordered rate; the flow rate is causes of a decreased I.V. flow rate, such as kinks in the
slow even with the roller clamp wide open. When tubing and poor positioning of the affected arm. The
the nurse lowers the I.V. fluid bag, no blood nurse should discontinue the I.V. infusion only if other
returns to the tubing. What should the nurse do measures fail to solve the problem. Irrigating I.V. tubing
first? may dislodge clots, if present. Elevating the I.V. fluid
bag may help if no kinks are found and if repositioning
doesn't resolve the problem.
4 When reviewing the history of a client with Crohn Heredity
7 disease, which factor would the nurse associate Explanation: Although the definitive cause of Crohn
. with this disorder? disease is unknown, it's thought to be associated with
infectious, immune or psychological, factors. Because it
has a higher incidence in siblings, it may have a genetic
cause. Constipation isn't linked to Crohn disease. On the
contrary, Crohn disease causes bouts of diarrhea. Diet
may contribute to exacerbations of Crohn disease but
isn't considered a cause. A lack of exercise isn't
considered a cause of Crohn disease.
4 Which condition most commonly results in Atherosclerosis
8 coronary artery disease (CAD)? Explanation: Atherosclerosis, or plaque formation, is the
. leading cause of CAD. Diabetes mellitus is a risk factor
for CAD but isn't the most common cause. Renal failure
doesn't cause CAD, but the two conditions are related.
MI is commonly a result of CAD.
4 Which factor in a client's history indicates she's at Use of corticosteroids
9 risk for candidiasis? Explanation: Small numbers of the fungus Candida
. albicans commonly inhabit the vagina. Because
corticosteroids decrease host defense, they increase the
risk of candidiasis. Pregnancy, not nulliparity, increases
the risk of candidiasis. Candidiasis is rare before
menarche and after menopause. The use of hormonal
contraceptives, not spermicidal jelly, increases the risk of
candidiasis.
5 Which nursing intervention takes highest priority Instructing the client to report any itching, swelling, or
0 when caring for a client who's receiving a blood dyspnea
. transfusion? Explanation: Because administration of blood or blood
products may cause serious adverse effects such as
allergic reactions, the nurse must monitor the client for
these effects. Signs and symptoms of life-threatening
allergic reactions include itching, swelling, and dyspnea.
Although the nurse should inform the client of the
duration of the transfusion and should document its
administration, these actions are less critical to the
client's immediate health. The nurse should monitor vital
signs 5 minutes after the transfusion is started, again in
15 minutes, and then at least hourly depending on the
client's condition.
5 Which of the following clients would be most A client with type I diabetes
1 susceptible to experiencing ketoacidosis? Explanation: The most common need for urine testing is
. the test for ketones if a client's blood glucose level is
consistently high. Because only clients with type 1
diabetes are susceptible to diabetic ketoacidosis, these
clients learn to test their urine for ketones if their blood
glucose readings exceed 240 mg/dL.
5 Which statement best describes an expected Goals that the client should reach as a result of planned
2 outcome? nursing interventions
. Explanation: Expected outcomes are realistic,
measurable goals that include target dates for when the
goals will be achieved. They're devised by the nursing
staff with input from the client. The goals are attained by
following planned nursing interventions.
5 Which statement reflects appropriate "Client's skin is moist and cool."
3 documentation in the medical record of a Explanation: Documentation should include data that the
. hospitalized client? nurse obtains using only observations that are heard,
seen, smelled, or felt. The nurse should record findings
or observations precisely and accurately. Documentation
of a leg ulcer should include its exact size and location.
Documenting observed client behaviors or conversations
is appropriate, but drawing conclusions about a client's
feelings is not. Stating that the client had a good day
doesn't provide precise enough information to be useful.

Step 1: Choose layout


Table

Glossary

Small

Large

3" x 5" Index card

Step 2: Customize options


Alphabetize

Flip terms and definitions

7 duplicate terms removed


Step 3: Open and print the PDF file
Open PDF
Want to print a test? Click here.

pr
Stu
at
28
1.

Question 5 See full question

The nurse is recording the intake and output for a client with the following:
D5NSS 1,000 ml; urine 450 ml; emesis 125 ml; Jackson Pratt drain #1 35 ml;
Jackson Pratt drain #2 32 ml; and Jackson Pratt drain #3 12 ml. How many
milliliters would the nurse document as the clients output? Record your
answer using a whole number.
Your Response:

0
Correct response:

654
Explanation:

Question 1 See full question

A nurse is caring for a client with an endotracheal tube who receives enteral
feedings through a feeding tube. Before each tube feeding, the nurse checks
for tube placement in the stomach as well as residual volume. The purpose
of the nurse's actions is to prevent:
You Selected:

diarrhea.
Correct response:

aspiration.
Explanation:

Question 2 See full question

The nurse is teaching the mother of a newly diagnosed diabetic child about
the principles of the diabetic diet. Which statement by the mother indicates
effective teaching?
You Selected:

"Snacks are used to keep blood glucose at acceptable levels during times when
the insulin level peaks."
Correct response:

"Snacks are used to keep blood glucose at acceptable levels during times when
the insulin level peaks."
Explanation:

Question 3 See full question

After cataract removal surgery, the nurse teaches the client about activities
that can be done at home. Which activity would be contraindicated?
You Selected:

bending over the sink to wash the face


Correct response:

bending over the sink to wash the face


Explanation:

Question 4 See full question

An appropriate nursing intervention for a client with fatigue related to cancer


treatment includes teaching the client to:
You Selected:

increase fluid intake.


Correct response:

conserve energy by prioritizing activities.


Explanation:

Question 5 See full question

A child has a nasogastric (NG) tube inserted by the nurse to administer a


continuous feeding. Which of the following actions should the nurse take
before starting the NG feeding on the child? Select all that apply.
You Selected:

Verify the physicians order.


Correct response:

Verify the physicians order.


Check placement of the NG tube.
Assess for bowel sounds.
Explanation:

Question 1 See full question

A depressed client in the psychiatric unit hasn't been getting adequate rest
and sleep. To encourage restful sleep at night, the nurse should:
You Selected:

gently but firmly set limits on how much time the client spends in bed during
the day.
Correct response:

gently but firmly set limits on how much time the client spends in bed during
the day.
Explanation:

Question 2 See full question

A nurse is instructing the client to do Kegel exercises. What should the nurse
tell the client to do to perform these pelvic floor exercises?
You Selected:

Tighten her stomach muscles.


Correct response:

Stop the flow of urine while urinating.


Explanation:

Question 3 See full question

The nurse is developing a plan of care for a client who has joint stiffness due
to rheumatoid arthritis. Which measure will be the most effective in relieving
stiffness?
You Selected:

a warm shower before performing activities of daily living


Correct response:

a warm shower before performing activities of daily living


Explanation:

Question 4 See full question

A client with Addisons disease has fluid and electrolyte loss due to
inadequate fluid intake and to fluid loss secondary to inadequate adrenal
hormone secretion. As the clients oral intake increases, which fluids would
be most appropriate?
You Selected:

water and eggnog


Correct response:

chicken broth and juice


Explanation:

Question 5 See full question

A client returned from surgery eight hours ago and has not voided. Which
action should the nurse take first?
You Selected:

Catheterize the client with a straight catheter.


Correct response:

Palpate over the synthesis pubis for fullness.


Explanation:

Question 1 See full question

A nurse is teaching the parents of a child with cystic fibrosis about proper
nutrition. Which instruction should the nurse include?
You Selected:

Encourage a high-calorie, high-protein diet.


Correct response:

Encourage a high-calorie, high-protein diet.


Explanation:

Question 2 See full question

A pregnant client asks the nurse whether she can take castor oil for her
constipation. How should the nurse respond?
You Selected:

"No, it can initiate premature uterine contractions."


Correct response:

"No, it can initiate premature uterine contractions."


Explanation:

Question 3 See full question

A client is ordered continuous bladder irrigation at a rate of 60 gtt/minute.


The nurse hangs a 2 L bag of sterile solution with tubing on a three-legged
I.V. pole. She then attaches the tubing to the client's three-way urinary
catheter, adjusts the flow rate, and leaves the room. Which important
procedural step did the nurse fail to follow?
You Selected:

Collecting a urine specimen before beginning irrigation


Correct response:

Evaluating patency of the drainage lumen


Explanation:

Question 4 See full question

While the nurse is caring for a primiparous client with cephalopelvic


disproportion 4 hours after a cesarean birth, the client requests assistance in
breastfeeding. To promote maximum maternal comfort, which position would
be most appropriate for the nurse to suggest?
You Selected:

cradle hold
Correct response:

football hold
Explanation:

Question 5 See full question

A 10-year-old boy is 24 hours post appendectomy. He is awake, alert, and


oriented. He tells the nurse that he is experiencing pain. He has a
prescription for morphine 1 to 2 mg PRN for pain. What is
the priority nursing action in managing the child's pain?
You Selected:

Obtain vital signs with a pain score.


Correct response:

Obtain vital signs with a pain score.


Explanation:

Question 1 See full question

A nurse has just removed an I.V. catheter from a client's arm because fluid
has infiltrated the arm. The physician orders warm soaks for the area. Based
on the principles of heat and cold application, the nurse should:
You Selected:

question the order because heat increases edema.


Correct response:

remove the warm compress for at least 15 minutes after each 20-minute
application.
Explanation:

Question 2 See full question

A client with rheumatoid arthritis tells the nurse, I know it is important to


exercise my joints so that I will not lose mobility, but my joints are so stiff
and painful that exercising is difficult. Which response by the nurse would
be most appropriate?
You Selected:

Tell the health care provider about your symptoms. Maybe your analgesic
medication can be increased.
Correct response:

"Take a warm tub bath or shower before exercising. This may help with your
discomfort."
Explanation:

Question 3 See full question

The nurse notes that a client is too busy investigating the unit and
overseeing the activities of other clients to eat dinner. To help the client
obtain sufficient nourishment, which plan would be best?
You Selected:

Serve food in small, attractively arranged portions.


Correct response:

Serve foods that she can carry with her.


Explanation:

Question 4 See full question

A nurse is providing dietary instructions to a client with hypoglycemia. To


control hypoglycemic episodes, the nurse should recommend:
You Selected:

eating a candy bar if light-headedness occurs.


Correct response:

consuming a low-carbohydrate, high-protein diet and avoiding fasting.


Explanation:

Question 5 See full question

A female client reports to a nurse that she experiences a loss of urine when
she jogs. The nurse's assessment reveals no nocturia, burning, discomfort
when voiding, or urine leakage before reaching the bathroom. The nurse
explains to the client that this type of problem is called:
You Selected:

functional incontinence.
Correct response:

stress incontinence.
Explanation:

Question 37 See full question

The nurse sees a client walking in the hallway who begins to have a seizure. What should the nurse do in order of
priority from first to last? All options must be used.

You Selected:

Ease the client to the floor.


Maintain a patent airway.
Obtain vital signs.
Record the seizure activity observed.

Correct response:

Ease the client to the floor.


Maintain a patent airway.
Obtain vital signs.
Record the seizure activity observed.
Explanation:

Question 38 See full question

A nurse assesses a client with psychotic symptoms and determines that the client likely poses a safety threat and
needs vest restraints. The client is adamantly opposed to this. What would be the best nursing action?

You Selected:

Contact the physician and obtain necessary orders.

Correct response:

Contact the physician and obtain necessary orders.

Explanation:

Question 39 See full question

A 13-year-old is having surgery to repair a fractured left femur. As a part of the preoperative safety checklist, what
should the nurse do?

You Selected:

Verify that the site, side, and level are marked.

Correct response:

Verify that the site, side, and level are marked.

Explanation:

Question 40 See full question

The nurse is caring for a client after surgery. The surgeon has written resume pre-op meds as an order on a
clients chart. What should the nurse do next?

You Selected:

Contact the surgeon for clarification because this is not a complete order.

Correct response:

Contact the surgeon for clarification because this is not a complete order.

The health education nurse reinforces instructions to a group of


Explanation:

clients regarding measures that will assist in preventing skin cancer. Which
instructions should the nurse reinforce to the client?
Use sunscreen when participating in outdoor activities.
Wear a hat, opaque clothing, and sunglasses when in the sun.
Examine your body monthly for any lesions that may be suspicious.
The nurse is assisting with developing a plan of care for the client with
multiple myeloma. Which is a priority nursing intervention for this client?
Encouraging fluids
The nurse is assisting with conducting a health-promotion program to
community members regarding testicular cancer. The nurse determines the
need for further teaching if a community member states that which is a
sign/symptom of testicular cancer?
Alopecia
The nurse is reviewing the laboratory results of a client with leukemia who
has received a regimen of chemotherapy. Which laboratory finding would
provide information about the massive cell destruction that occurs with the
chemotherapy?
Increased uric acid level
The client is receiving external radiation to the neck for cancer of the larynx.
The nurse monitors the client knowing that which is the most likely
side/adverse effect of the external radiation?
Sore throat
The nurse is reinforcing instructions to a client receiving external radiation
therapy. The nurse determines that the client needs further teaching if the
client states an intention to take which action?
Apply pressure on the radiated area to prevent bleeding
The nurse is caring for a client with an internal radiation implant. The nurse
should observe which principle?
Pregnant women are not allowed into the client's room
The nurse provides skin care instructions to the client who is receiving
external radiation therapy. Which statement by the client indicates the need
for further teaching?
"I will limit sun exposure to 1 hour daily."
The client is hospitalized for the insertion of an internal cervical radiation
implant. While giving care, the nurse finds the radiation implant in the bed.
Which is the immediate nursing action?
Pick up the implant with long-handled forceps and place into a lead
container.
The nurse is assisting with developing a plan of care for a client who is
experiencing hematological toxicity as a result of chemotherapy. The nurse
should suggest including which in the plan of care?
Restricting fresh fruits and vegetables in the diet
The client with carcinoma of the lung develops the syndrome of
inappropriate antidiuretic hormone (SIADH) as a complication of the cancer.
The nurse anticipates that which may be prescribed to treat this
complication?
Radiation, Chemotherapy, Serum sodium blood levels, Medication that is
antagonistic to antidiuretic hormone (ADH)
The client is admitted to the hospital with a diagnosis of suspected Hodgkin's
disease. Which finding should the nurse most likely expect to find
documented in the client's record?
Enlarged lymph nodes
When reviewing the health care record of a client with ovarian cancer, the
nurse recognizes which sign/symptom as being a typical manifestation of the
disease?
Abdominal distention
The nurse is caring for a client after a mastectomy. Which finding would
indicate that the client is experiencing a complication related to the surgery?
Arm edema on the operative side
The nurse is reinforcing discharge instructions to a client with cancer of the
prostate after a prostatectomy. The nurse should reinforce which discharge
instruction?
Avoid lifting objects heavier than 20 pounds for at least 6 weeks.
The nurse is reviewing the laboratory results of a client who is receiving
chemotherapy and notes that the platelet count is 10,000 cells/mm3. On the
basis of this laboratory value, the nurse should collect which data as a
priority?
Level of consciousness
The nurse reinforces instructions to the client about breast self-examination
(BSE). The nurse instructs the client to lie down and examine the left breast.
Which is the correct area for placing a pillow when examining the left breast?
Under the left shoulder
The nurse is caring for a client dying of ovarian cancer. During care, the
client states, "If I can just live long enough to attend my daughter's
graduation, I'll be ready to die." Which phase of coping is this client
experiencing?
Bargaining
The nurse is reinforcing instructions to a community group regarding the
risks and causes of bladder cancer. The nurse determines that there is a
need for further teaching if a member of the community group makes which
statement regarding this type of cancer
It most often occurs in women
The nurse is collecting data from a client with a history of bladder cancer.
Which sign/symptom is the client most likely to report?
Hematuria
The nurse is caring for a client after a mastectomy. Which nursing
intervention should assist with preventing lymphedema of the affected arm?
Elevating the affected arm on a pillow above heart level
The nurse is reinforcing instructions to a client on performing a testicular
self-examination (TSE). Which instruction should the nurse provide to the
client?
The best time for the examination is after a shower
The nurse is assisting with conducting a health-promotion program at a local
school. The nurse determines that there is a need for further teaching if a
student identifies which as a risk factor associated with cancer?
Low-fat and high-fiber diets
A client with cancer is receiving chemotherapy and develops
thrombocytopenia. Which intervention is a priority in the nursing plan of
care?
Monitor the client for bleeding
The nurse is reinforcing instructions to a group of female clients about breast
self-examination (BSE). When should the nurse instruct the client to perform
this examination?
One week after menstruation begins
A client who has been diagnosed with multiple myeloma asks the nurse
about the diagnosis. The nurse bases the response on which characteristic of
the disorder?
Malignant proliferation of plasma cells and tumors within the bone
The nurse is reviewing the laboratory results of a client who has been
diagnosed with multiple myeloma. Which finding should the nurse expect to
note with this diagnosis?
Increased calcium level
A gastrectomy is performed on a client with gastric cancer. In the immediate
postoperative period the nurse notes bloody drainage from the nasogastric
(NG) tube. Which action should the nurse take?
Continue to monitor the drainage
The nurse is reviewing the medical history of a client admitted to the hospital
with a diagnosis of colorectal cancer. The nurse understands that which
information documented in the medical history is an unassociated risk factor
of this type of cancer?
Regular consumption of a high-fiber diet
A client is tentatively diagnosed with ovarian cancer. The nurse gathers data
about which late symptom of this disease?
Pelvic pain, anemia, and ascites
A client with ovarian cancer is scheduled to receive chemotherapy with
cisplatin. The nurse assisting in caring for the client reviews the plan of care,
expecting to note which intervention?
Encourage fluids.
A client receiving chemotherapy asks the nurse, "What will I do when my hair
starts to fall out?" Which action by the nurse is appropriate?
Encourage her to select a wig
A client has just been told by the health care provider about her diagnosis of
breast cancer. The client responds, "Oh no, does this mean I'm going to die?"
The nurse interprets which response as the client's initial reaction?
Fear
The nurse should monitor for which laboratory result as indicating an adverse
reaction in the client with endometrial cancer who is receiving
chemotherapy?
Platelet count 20,000 cells/mm3
The nurse determines that a client with which history is most at risk for
endometrial cancer?
Estrogen replacement therapy
A client with endometrial cancer is receiving doxorubicin (Adriamycin), an
antineoplastic agent. The nurse should specifically collect data about which
criterion?
Hematological laboratory values
The nurse is caring for a client with cancer receiving chemotherapy who has
developed stomatitis. The nurse plans to give mouth care by using oral care
agents and devices that meet which additional criterion?
Care will be based on the severity of stomatitis
A client with cancer has undergone a total abdominal hysterectomy and has
a Foley catheter in place. The nurse should expect to note which type of
urinary drainage immediately following this surgery?
Blood tinged
A client with lung cancer receiving chemotherapy tells the nurse that the
food on the meal tray tastes "funny." Which is the appropriate nursing
intervention?
Provide oral hygiene care frequently.
The nurse is reviewing the record of a client admitted to the hospital for
treatment of bladder cancer. Which risk factor related to this type of cancer
should the nurse likely note in the client's record?
Drinks coffee and smokes cigarettes
The nurse is obtaining data from a client admitted with a diagnosis of
bladder cancer. Which question should the nurse ask the client to determine
if the client experienced the common symptom associated with this type of
cancer?
"Do you notice any blood in the urine?"
The nurse is preparing a client for an intravesical instillation of an alkylating
chemotherapeutic agent into the bladder for the treatment of bladder
cancer. The nurse provides instructions to the client regarding the procedure.
Which client statement indicates an understanding of this procedure?
"After the instillation is done, I will need to change position every 15 minutes
from side to side."
The nurse is developing a plan of care for a client following a radical
mastectomy and includes measures that will assist in preventing
lymphedema of the affected arm. The nurse should include which action to
prevent this complication?
Elevate the affected arm on a pillow
The nurse is assisting in caring for a client with an inoperable lung tumor and
helps develop a plan of care by addressing complications related to the
disorder. The nurse includes monitoring for the early signs of vena cava
syndrome in the plan. Which early sign of this oncological emergency should
the nurse include monitoring for in the plan of care?
Edema of the face and eyes
The nurse has reinforced discharge instructions regarding home care to a
client following a prostatectomy for cancer of the prostate. Which statement
by the client indicates an understanding of the instructions?
"I should not lift anything over 20 pounds."
The nurse is assisting in caring for a client receiving chemotherapy. On
review of the morning laboratory results, the nurse notes that the white
blood cell count is extremely low, and the client is immediately placed on
neutropenic precautions. The client's breakfast tray arrives, and the nurse
inspects the meal and prepares to bring the tray into the client's room.
Which action should the nurse take before bringing the meal to the client?
Remove the fresh orange from the breakfast tray.
The nurse is assisting in caring for a client with a diagnosis of bladder cancer
who recently received chemotherapy. The nurse receives a telephone call
from the laboratory who reports that the client's platelet count is
20,000/mm3. Based on this laboratory value, the nurse revises the plan of
care and suggests including which intervention?
Monitor skin for the presence of petechiae.
The nurse is reviewing the record of a client with a diagnosis of cervical
cancer. Which should the nurse expect to note in the client's record related
to a risk factor associated with this type of cancer?
History of human papillomavirus
The nurse is collecting data from a client who is admitted to the hospital for
diagnostic studies to rule out the presence of Hodgkin's disease. Which
question should the nurse ask the client to elicit information specifically
related to this disease?
"Have you noticed any swollen lymph nodes?"
The nurse is collecting data from a client suspected of having ovarian cancer.
Which question should the nurse ask the client to elicit information
specifically related to this disorder?
"Does your abdomen feel as though it is swollen?"
The nurse is reinforcing instructions to a client scheduled for conization in 1
week for the treatment of microinvasive cervical cancer. The procedure has
been explained by the health care provider, and the nurse is reviewing the
complications associated with the procedure. The nurse determines that the
client needs further teaching if the client states that which is a complication
of this procedure?
Ovarian perforation
The nurse is reviewing the laboratory results of a client with bladder cancer
and bone metastasis and notes that the calcium level is 15 mg/dL. The nurse
should take which appropriate action?
Notify the health care provider
The nurse is reinforcing client education regarding symptoms of testicular
cancer. The nurse encourages the client to report which symptoms as being
associated with testicular cancer?
A grainy mass palpated in a testicle, and An enlargement of one of the testes
The nurse is assisting in planning care for a client with Hodgkin's disease
who is neutropenic as a result of radiation and chemotherapy. Which actions
would be included in the client's plan of care?
Monitor white blood cell counts daily., Ensure meticulous hand washing
before caring for the client., Ask visitors with respiratory infection symptoms
to not visit the client.
The nurse is caring for a client who has undergone pelvic exenteration. In
addressing psychosocial issues related to the surgery, which statement by
the nurse should be therapeutic?
How do you feel about your body?"
A nursing student is assisting in caring for a client with a lung tumor; the
client will be having a pneumonectomy. The nursing instructor reviews the
postoperative plan of care developed by the student and suggests deleting
which item from the plan?
Monitoring the closed chest tube drainage system
The nurse is monitoring a client with a diagnosis of cancer for signs and
symptoms related to vena cava syndrome. The nurse understands that which
is an early sign of this oncological emergency?
Periorbital edema
The nurse when inspecting the stoma of a client following an ureterostomy 6
hours ago, notes that the stoma appears pale in color. Which interpretation
does the nurse make based on this finding?
The vascular supply to the stoma is insufficient.
The nurse is assisting in developing a postoperative plan of care for a client
following a mastectomy. Which interventions will be included in the plan of
care?
Place the affected arm on a pillow., Check the incision for signs of infection.,
Monitor and measure drainage in the collection device
The nurse is caring for a client with metastatic lung cancer. The client was
medicated 2 hours ago and now reports a new and sudden sharp pain in the
back. The nurse appropriately interprets this finding as possibly indicating
which complication?
Spinal cord compression
The nurse is assisting in providing a session to community members about
the risks associated with laryngeal cancer. Which statement by a client
indicates an understanding of the risk factors?
"Exposure to airborne carcinogens can cause this type of cancer."
The nurse is assisting in the care of a client diagnosed with multiple
myeloma who has been prescribed an intravenous solution. Which finding
would indicate a positive response to this treatment?
Creatinine of 1 mg/dL
A client who has just been told by the health care provider that she has
breast cancer responds by stating, "Oh, no, this has to be a big mistake." The
nurse interprets the client's initial response as which type of reaction?
Denial
A client is receiving chemotherapy that carries a risk of phototoxicity as an
adverse effect. Which finding indicates that the client experienced this side
effect?
Erythema
The nurse reviews the care plan of a client with cancer and notes that the
client has a problem with adequate food intake related to side effects of
therapy. In order to enhance appetite and nutrition, the nurse should offer
which advice to the client?
Avoid strong-smelling foods.
The nurse reviews the care plan of a client with cancer undergoing
chemotherapy. The nurse notes that the client has a concern about her
appearance as a result of alopecia. The nurse plans to tell the client which
information about hair loss and regrowth to assist the client in coping with
this possible change?
Regrown hair may have a different color and texture.
The nurse is assisting in preparing a teaching plan of care for a client being
discharged from the hospital following surgery for testicular cancer. Which
instruction should the nurse suggest to include in the plan?
"An elevation in temperature should be reported to the health care provider."
A client is receiving radiation therapy to the brain because of a diagnosis of a
brain tumor. Which side/adverse effect does the nurse expect the client is
likely to experience?
Nausea and vomiting
The nurse's teaching plan for a client with a family history of breast cancer
should include which important item?
Teaching the breast self-exam technique to be done every month
The nurse caring for a client following a radical neck dissection and creation
of a tracheostomy performed for laryngeal cancer is reinforcing discharge
instructions to the client. Which statement by the client indicates the need
for further teaching regarding care of the stoma?
"I need to use an air conditioner to provide cool air to assist in breathing."
A client is diagnosed as having a bowel tumor, and several diagnostic tests
are prescribed. The nurse understands that which test will confirm the
diagnosis of malignancy?
Biopsy of the tumor
The nurse is preparing a client with a bowel tumor for surgery. The health
care provider has informed the client that the surgery is palliative in the
treatment of the tumor. Which rationale is the reason to perform this type of
surgery?
To reduce pain
A cervical radiation implant is placed in the client for treatment of cervical
cancer. Which activity would the nurse most likely expect to note in the
health care provider's prescriptions?
Bed rest
A client who has been receiving radiation therapy for bladder cancer tells the
nurse that it feels as if she is voiding through the vagina. The nurse
interprets that the client may be experiencing which?
The development of a vesicovaginal fistula
A nursing instructor asks a nursing student about the characteristics of
Hodgkin's disease. The instructor determines that the student needs to read
about the characteristics of this disease if the student states that which is an
associated characteristic?
Occurs most often in older adults
A client with liver cancer who is receiving chemotherapy tells the nurse that
some foods on the meal tray taste bitter. The nurse should try to limit which
food that is most likely to have this taste for the client?
Beef
The nurse is reinforcing instructions to a group of high school males in a
health class about how to perform a testicular self-examination (TSE). The
nurse should make which statement?
"Do the examination after a warm bath or shower."
The nurse working in an obstetrical-gynecological health care provider's
office is instructing a small group of female clients about breast self-
examination (BSE). Which instructions should the nurse reinforce?
BSE should be performed 1 week after menstruation begins.
The nurse is teaching a local women's church group about the risks of
cervical cancer. The nurse determines that there is a need for further
teaching if a group member states that which is a risk factor?
Intercourse with circumcised males
The nurse discusses the risk factors associated with gastric cancer as part of
a health promotion program. The nurse determines that there is a need for
further teaching if a member attending the program states that which factor
is a risk?
High meat and carbohydrate consumption
The nurse is reinforcing instructions to a group of adults about the seven
warning signs of cancer. The nurse determines that a member of the group
needs further teaching if the member states which sign/symptom is a
warning sign?
Absence or decreased frequency of menses
A client with cancer is at risk for experiencing vena cava syndrome. The
nurse should monitor this client for which early sign of this oncological
emergency?
Periorbital edema
The nurse is reinforcing instructions to the client who is about to begin
external radiation therapy on how to maintain optimal skin integrity during
therapy. The nurse determines that there is a need for further teaching if the
client states that he will do which action?
Apply tight dressings over the area to prevent bleeding.
The nurse is orienting a new nurse to the care of a client who has an internal
radiation implant. The nurse includes which instructions in discussions with
the new nurse?
Pregnant women are not allowed in the client's room.
The nurse answers the call bell of a client who had insertion of an internal
cervical radiation implant. The client states that the implant fell out, and the
nurse sees it lying in the bed after moving back the sheet. Which action
should the nurse take?
Use long-handled forceps to place the implant in a lead container
A client with cancer develops white, doughy patches on the mucous
membranes of the oral cavity. Which action should the nurse take when
noting this?
Report these symptoms, which are consistent with candidiasis
The nurse is caring for a client in the oncology unit who has developed
stomatitis during chemotherapy. The nurse should plan which measure to
treat this complication?
Rinse the mouth with dilute baking soda or saline solution
The nurse is reviewing the health record of a client with laryngeal cancer.
The nurse should expect to note which most common risk factor for this type
of cancer documented in the record?
Cigarette smoking

Question 41 See full question

A restraint order is implemented for a client who is restless and combative due to alcohol intoxication. What is the
most appropriate nursing intervention for this client?

You Selected:

Check the extremities for circulation based on hospital protocols.

Correct response:

Check the extremities for circulation based on hospital protocols.

Explanation:

Question 42 See full question

A client is scheduled for a renal arteriogram. No allergies are recorded in the client's medical record, and the client
is unable to provide allergy information. During the arteriogram, the nurse should be alert for which assessment
finding that may indicate an allergic reaction to the dye used?

You Selected:

Pruritus

Correct response:

Pruritus

Explanation:

Question 43 See full question

A nursing assistant is caring for a client with Clostridium difficile diarrhea and asks the charge nurse, "How can I
keep from catching this from the client?" Which interventions does the charge nurse remind the nursing assistant to
employ? Select all that apply.

You Selected:

Wash hands before and after care with this client.


Place the client on protective isolation.
Provide disposable utensils and dishes for the client.
Use contact isolation procedures when caring for the client.

Correct response:

Wash hands before and after care with this client.


Use contact isolation procedures when caring for the client.
Explanation:

Question 44 See full question

A client is admitted to the emergency department with sneezing and coughing. The client is in the triage area,
waiting to be seen by a health care provider (HCP). To prevent spread of infection to others in the area and to the
health care staff, the nurse should:

You Selected:

give the client a surgical mask to wear.

Correct response:

give the client a surgical mask to wear.

Explanation:

Question 1 See full question

A mother of a 5-year-old child who was admitted to the hospital has a Protection from Abuse order for the child
against his father. A copy of the order is kept on the pediatric medical surgical unit where the child is being treated.
The order prohibits the father from having any contact with the child. One night, the father approaches the nurse at
the nurses' station, politely but insistently demanding to see his child, and refusing to leave until he does so. What
should the nurse do first?

You Selected:

Notify hospital security or the local authorities.

Correct response:

Notify hospital security or the local authorities.

Explanation:

Question 2 See full question

The nurse is reconciling the medications with a client who is being discharged. Which information indicates there is
a "discrepancy"?

You Selected:

There is lack of congruence between a clients home medication list and current medication prescriptions.

Correct response:

There is lack of congruence between a clients home medication list and current medication prescriptions.

Explanation:

Question 3 See full question

A client in the postanesthesia care unit is being actively rewarmed with an external warming device. How often
should the nurse monitor the client's body temperature?

You Selected:

every 15 minutes

Correct response:

every 15 minutes

Explanation:

Question 4 See full question

Which action by the nursing assistant would require immediate intervention by the nurse?
You Selected:

Restraining a school-age child at risk for self-harm because the nursing assistant had to leave the room

Correct response:

Restraining a school-age child at risk for self-harm because the nursing assistant had to leave the room

Explanation:

Question 5 See full question

A client has an indwelling urinary catheter and is prescribed physical therapy. As the client is being placed in a
wheelchair, which action by the assistant would need further clarification by the nurse?

You Selected:

The catheter bag is placed upon the clients lap for safe transport.

Correct response:

The catheter bag is placed upon the clients lap for safe transport.

Explanation:

Question 45 See full question

A client has an indwelling urinary catheter and is prescribed physical therapy. As the client is being placed in a
wheelchair, which action by the assistant would need further clarification by the nurse?

You Selected:

The catheter bag is placed upon the clients lap for safe transport.

Correct response:

The catheter bag is placed upon the clients lap for safe transport.

Explanation:

Question 46 See full question

Which nursing action best addresses the outcome: The client will be free from falls?

You Selected:

Encourage use of grab bars and railings in the bathroom and halls

Correct response:

Encourage use of grab bars and railings in the bathroom and halls

Explanation:

Question 47 See full question

A nurse is teaching a new mother how to prevent burns in the home. Which statement by the mother indicates
more teaching is required?

You Selected:

I will heat my infants formula in the microwave.

Correct response:

I will heat my infants formula in the microwave.


Explanation:

Question 48 See full question

A client is admitted with an infectious wound. Contact precautions are initiated. To help the client cope with staff
using isolation procedures, which nursing action is most helpful?

You Selected:

Discuss the rationale for contact precautions

Correct response:

Discuss the rationale for contact precautions

Explanation:

Question 49 See full question

Which nursing action is most beneficial to prevent fungal infections in hospitalized clients?

You Selected:

Dry all skin folds thoroughly

Correct response:

Dry all skin folds thoroughly

Explanation:

Question 50 See full question

The nurse is planning care with an older adult who is at risk for falling because of postural hypotension. Which
intervention will be most effective in preventing falls in this client?

You Selected:

Instruct the client to sit, obtain balance, dangle legs, and rise slowly.

Correct response:

Instruct the client to sit, obtain balance, dangle legs, and rise slowly.

Explanation:

Question 1 See full question

Which nursing intervention is appropriate for a client with an arm restraint?

You Selected:

Applying the restraint loosely to prevent pressure on the skin

Correct response:

Monitoring circulatory status every 2 hours

Explanation:

Question 2 See full question

The staff of an outpatient clinic has formed a task force to develop new procedures for swift, safe evacuation of the
unit. The new procedures haven't been reviewed, approved, or shared with all personnel. When a nurse-manager
receives word of a bomb threat, the task force members push for evacuating the unit using the new procedures.
Which action should the nurse-manager take?
You Selected:

Tell staff members to assemble in the staff lounge, where she will quickly gather opinions about
evacuation procedures before deciding what to do.

Correct response:

Determine that the procedures currently in place must be followed and direct staff to follow them without
question.

Explanation:

Question 3 See full question

Entering a client's room, a nurse on the maternity unit sees a mother slapping the face of a crying neonate. Which
action should the nurse take in this situation?

You Selected:

Return the neonate to the nursery, inform the physician so he can thoroughly examine the neonate for
injuries, and notify social services for assistance.

Correct response:

Return the neonate to the nursery, inform the physician so he can thoroughly examine the neonate for
injuries, and notify social services for assistance.

Explanation:

Question 4 See full question

After an infant undergoes surgical repair of a cleft lip, the physician orders elbow restraints. For this infant, the
postoperative care plan should include which nursing action?

You Selected:

Keeping the restraints on both arms only while the child is awake

Correct response:

Removing the restraints every 2 hours

Explanation:

Question 5 See full question

A 3-month-old infant with meningococcal meningitis has just been admitted to the pediatric unit. Which nursing
intervention has the highest priority?

You Selected:

orienting the parents to the pediatric unit

Correct response:

instituting droplet precautions

Explanation:

Question 6 See full question

A nurse is teaching parents about accident prevention for a toddler. Which guideline is most appropriate?

You Selected:

Never leave a toddler unattended on a bed.

Correct response:

Make sure all medications are kept in containers with childproof safety caps.
Explanation:

Question 7 See full question

What should a nurse do to ensure a safe hospital environment for a toddler?

You Selected:

Pad the crib rails.

Correct response:

Move the equipment out of reach.

Explanation:

Question 8 See full question

Which item in the care plan for a toddler with a seizure disorder should a nurse revise?

You Selected:

Oxygen mask and bag system at bedside

Correct response:

Arm restraints while asleep

Explanation:

Question 9 See full question

A nurse is teaching parents how to reduce the spread of impetigo. The nurse should encourage parents to:

You Selected:

teach children to cover mouths and noses when they sneeze.

Correct response:

teach children the importance of proper hand washing.

Explanation:

Question 10 See full question

Which use of restraints in a school-age child should the nurse question?

You Selected:

To substitute for observation

Correct response:

To substitute for observation

Explanation:

Question 11 See full question

A client is transferred from the emergency department to the locked psychiatric unit after attempting suicide by
taking 200 acetaminophen tablets. The client is now awake and alert but refuses to speak with the nurse. In this
situation, the nurse's first priority is to:

You Selected:

try to communicate with the client in writing.


Correct response:

ensure safety by initiating suicide precautions.

Explanation:

Question 12 See full question

A client refuses his evening dose of haloperidol and then becomes extremely agitated in the day room while other
clients are watching television. He begins cursing and throwing furniture. The nurse's first action is to:

You Selected:

remove all other clients from the day room.

Correct response:

remove all other clients from the day room.

Explanation:

Question 13 See full question

The nurse is receiving over the telephone a laboratory results report of a neonate's blood glucose level. The nurse
should:

You Selected:

request that the laboratory send the results by e-mail to transfer to the client's medical record.

Correct response:

write down the results, read back the results to the caller from the laboratory, and receive confirmation
from the caller that the nurse understands the results.

Explanation:

Question 14 See full question

After the nurse administers haloperidol 5 mg PO to a client with acute mania, the client refuses to lie down on her
bed, runs out on the unit, pushes clients in her vicinity out of the way, and screams threatening remarks to the
staff. What should the nurse do next?

You Selected:

Follow the client and ask her to calm down.

Correct response:

Seclude the client and use restraints if necessary.

Explanation:

Question 15 See full question

A 7-year-old child is admitted to the hospital with acute rheumatic fever with chorea-like movements. Which eating
utensil should the nurse remove from the meal tray?

You Selected:

fork

Correct response:

fork
Explanation:

Question 16 See full question

Which statement by the parent of an infant with a repaired upper lumbar myelomeningocele indicates that the
parent understands the nurses teaching at the time of discharge?

You Selected:

"I will call the health care provider if his urine has a funny smell."

Correct response:

"I will call the health care provider if his urine has a funny smell."

Explanation:

Question 17 See full question

The nurse in the emergency department is administering a prescription for 20 mg intravenous furosemide, which is
to be given immediately. The nurse scans the clients identification band and the medication barcode. The
medication administration system does not verify that furosemide is prescribed for this client; however, the
furosemide is prepared in the accurate unit dose for intravenous infusion. What should the nurse do next?

You Selected:

Contact the pharmacist immediately to check the order and the barcode label for accuracy.

Correct response:

Contact the pharmacist immediately to check the order and the barcode label for accuracy.

Explanation:

Question 18 See full question

An alert and oriented elderly client is admitted to the hospital for treatment of cellulitis of the left shoulder after an
arthroscopy. Which fall prevention strategy is most appropriate for this client?

You Selected:

Place the client in a room with a camera monitor.

Correct response:

Use a nightlight in the bathroom.

Explanation:

Question 19 See full question

When changing a wet-to-dry dressing covering a surgical wound, what should the nurse do?

You Selected:

Pack the wet dressing tightly into the wound.

Correct response:

Cover the wet packing with a dry sterile dressing.

Explanation:

Question 20 See full question

A client with cervical cancer is undergoing internal radium implant therapy. A lead-lined container and a pair of long
forceps are kept in the client's hospital room for:
You Selected:

storage of the radiation dose.

Correct response:

handling of the dislodged radiation source.

Explanation:

Question 21 See full question

The nurse is preparing a community education program about preventing hepatitis B infection. Which information
should be incorporated into the teaching plan?

You Selected:

Frequent ingestion of alcohol can predispose an individual to development of hepatitis B.

Correct response:

The use of a condom is advised for sexual intercourse.

Explanation:

Question 22 See full question

Which topic would be most important to include when teaching the parents how to promote overall toddler
development?

You Selected:

Language is the most important achievement.

Correct response:

Safety is a priority concern for this age-group.

Explanation:

Question 23 See full question

In caring for the client with hepatitis B, which situation would expose the nurse to the virus?

You Selected:

touching the clients arm with ungloved hands while taking a blood pressure

Correct response:

a blood splash into the nurses eyes

Explanation:

Question 24 See full question

The nurse is instructing the unlicensed assistive personnel (UAP) on how to position the wheelchair to assist a client
with left-sided weakness transfer from the bed to a wheelchair using a transfer belt. Which statement by the UAP
tells the nurse that the UAP has understood the instructions for placing the wheelchair?

You Selected:

I will place the wheelchair behind the client.

Correct response:

The wheelchair should be placed on the right side of the bed.


Explanation:

Question 25 See full question

A nurse checks the synchronizer switch before using a defibrillator to terminate ventricular fibrillation. Why is this
check so important?

You Selected:

The shock must be synchronized with the client's T wave.

Correct response:

The defibrillator will not deliver a shock if the synchronizer switch is turned on.

Explanation:

Question 26 See full question

A client with chronic obstructive pulmonary disease (COPD) is intubated and placed on continuous mechanical
ventilation. Which equipment is most important for the nurse to keep at this client's bedside?

You Selected:

Oxygen analyzer

Correct response:

Manual resuscitation bag

Explanation:

Question 27 See full question

Which client is at highest risk for developing a hospital-acquired infection?

You Selected:

A client with Crohn's disease

Correct response:

A client with an indwelling urinary catheter

Explanation:

Question 28 See full question

A nurse is caring for a client with a history of falls. The nurse's first priority when caring for a client at risk for falls
is:

You Selected:

keeping the bed in the lowest possible position.

Correct response:

keeping the bed in the lowest possible position.

Explanation:

Question 29 See full question

A nursing instructor is instructing group of new nursing students. The instructor reviews that surgical asepsis will be
used for which of the following procedures?

You Selected:

Instilling eye drops


Correct response:

I.V. catheter insertion

Explanation:

Question 30 See full question

A 29-year-old multigravida at 37 weeks' gestation is being treated for severe preeclampsia and has magnesium
sulfate infusing at 3 g/h. To maintain safety for this client, the priority intervention is to:

You Selected:

assess reflexes, clonus, visual disturbances, and headache.

Correct response:

assess reflexes, clonus, visual disturbances, and headache.

Explanation:

Question 31 See full question

A parent tells the nurse that their 6-year-old child has severe nosebleeds. To manage the nosebleed, the nurse
should tell the parent to:

You Selected:

help the child lie on the stomach and collect the blood on a clean towel.

Correct response:

place the child in a sitting position with the neck bent forward and apply firm pressure on the nasal
septum.

Explanation:

Question 32 See full question

A school nurse interviews the parent of a middle school student who is exhibiting behavioral problem, including
substance abuse, following a siblings suicide. The parent says I am a single parent who has to work hard to
support my family, and now I have lost my only son and my daughter is acting out and making me crazy! I just
cannot take all this stress! Which concern regarding this family has top priority at this time?

You Selected:

the adolescent's anger

Correct response:

potential suicidal thoughts/plans of both family members

Explanation:

Question 33 See full question

While making rounds, the nurse finds a client with COPD sitting in a wheelchair, slumped over a lunch tray. After
determining the client is unresponsive and calling for help, the nurse's first action should be to:

You Selected:

push the "code blue" (emergency response) button.

Correct response:

open the client's airway.


Explanation:

Question 34 See full question

A client is admitted to the Emergency Department with a full thickness burn to the right arm. Upon assessment, the
arm is edematous, fingers are mottled, and radial pulse is now absent. The client states that the pain is 8 on a scale
of 1 to 10. The nurse should:

You Selected:

instruct the client to exercise his fingers and wrist.

Correct response:

call the health care provider (HCP) to report the loss of the radial pulse.

Explanation:

Question 35 See full question

A soldier on his second tour of duty was notified of the date that he will be redeployed. As this date approaches, he
is showing signs of excess anxiety and irritability and inability to sleep at night because of nightmares of IED
(improvised explosive devices) tragedies, all leading to poor work performance. His commanding officer refers him
to the base hospital for an evaluation. What should the nurse do in order of priority from first to last? All options
must be used.

You Selected:

Remove any weapons and dangerous items in his possession.


Acknowledge any injustices/unfairness related to his experiences, and offer empathy and support.
Remind him that any feelings and problems he is having are typical in his current situation.
Ask him to talk about his upsetting experiences.

Correct response:

Remove any weapons and dangerous items in his possession.


Remind him that any feelings and problems he is having are typical in his current situation.
Acknowledge any injustices/unfairness related to his experiences, and offer empathy and support.
Ask him to talk about his upsetting experiences.

Explanation:

Question 36 See full question

A nurse assesses a client with psychotic symptoms and determines that the client likely poses a safety threat and
needs vest restraints. The client is adamantly opposed to this. What would be the best nursing action?

You Selected:

Restrain the client with vest restraints.

Correct response:

Contact the physician and obtain necessary orders.

Explanation:

Question 37 See full question

The nurse has just completed a clients home visit and has scheduled another clients visit immediately after. Which
of the following measures should the nurse take to minimize risks of infection during home visits? Select all that
apply.

You Selected:

Wear gloves at all times while performing a home visit.


Correct response:

Perform hand hygiene before and after client contact.


Implement standard precautions during home visits.

Explanation:

Question 38 See full question

Over the past few weeks, a client in a long-term care facility has become increasingly unsteady. The nurses are
worried that the client will climb out of bed and fall. Which of the following measures does not comply with a least
restraint policy?

You Selected:

Providing a bed that is low to the floor

Correct response:

Raising all side rails while the client is in bed

Explanation:

Question 39 See full question

The nurse is caring for a client after surgery. The surgeon has written resume pre-op meds as an order on a
clients chart. What should the nurse do next?

You Selected:

Contact the surgeon for clarification because this is not a complete order.

Correct response:

Contact the surgeon for clarification because this is not a complete order.

Explanation:

Question 40 See full question

A staff nurse is caring for a child with a urinary tract infection. The nurse is 1 hour late administering the childs
prescribed antibiotic therapy and pain medication. The charge nurse challenges the staff nurse about the lateness
of the medications. The staff nurse responds, Its no big deal; at least the child got the medication. What is the
best course of action for the charge nurse to take?

You Selected:

Speak to the pharmacy about re-timing the childs medications.

Correct response:

Speak to the unit manager and fill out a medication error report.

Explanation: Question 1 See full question

A parent asks the nurse about using a car seat for a toddler who is in a hip spica cast. The nurse should tell the
parent:

You Selected:

"You will need a specially designed car seat for your toddler."

Correct response:

"You will need a specially designed car seat for your toddler."
Explanation:

Question 2 See full question

The nurse is admitting a 4-year old with a possible meningococcal infection. Which type of isolation is indicated?

You Selected:

droplet precautions

Correct response:

droplet precautions

Explanation:

Question 3 See full question

A hospitalized 5-year-old is pulseless, and after verifying the child is not breathing, the nurse begins chest
compressions. The nurse should apply pressure:

You Selected:

on the lower sternum with the heel of one hand.

Correct response:

on the lower sternum with the heel of one hand.

Explanation:

Question 4 See full question

The children of an elderly male client who has suffered an ischemic stroke have informed the nurse that an herbalist
will be coming to their fathers bedside tomorrow to make recommendations for his care. Which of the following
considerations should the nurse prioritize in light of the practitioners planned visit?

You Selected:

Ensuring any complementary therapies are safe when combined with his prescribed therapy.

Correct response:

Ensuring any complementary therapies are safe when combined with his prescribed therapy.

Explanation:

Question 5 See full question

A staff nurse is caring for a child with a urinary tract infection. The nurse is 1 hour late administering the childs
prescribed antibiotic therapy and pain medication. The charge nurse challenges the staff nurse about the lateness
of the medications. The staff nurse responds, Its no big deal; at least the child got the medication. What is the
best course of action for the charge nurse to take?

You Selected:

Speak to the unit manager and fill out a medication error report.

Correct response:

Speak to the unit manager and fill out a medication error report.

Explanation:

Question 1 See full question

A physician orders hourly urine output measurement for a postoperative client with an indwelling catheter. The
nurse records the following amounts of output for 2 consecutive hours: 8 a.m. (0800): 50 ml; 9 a.m. (0900): 60 ml.
Based on these amounts, which action should the nurse take?
You Selected:

Notify the physician.

Correct response:

Continue to monitor and record hourly urine output.

Explanation:

Question 2 See full question

A client has left-sided paralysis. The nurse should document this condition as left-sided:

You Selected:

hemiplegia.

Correct response:

hemiplegia.

Explanation:

Question 3 See full question

For a client with a nursing diagnosis of Insomnia, the nurse should use which measure to promote sleep?

You Selected:

Increasing the client's activity 2 hours before bedtime

Correct response:

Playing soft or soothing music

Explanation:

Question 4 See full question

A client is to receive a glycerin suppository. Which nursing action is appropriate when administering a suppository?

You Selected:

Removing the suppository from the refrigerator 30 minutes before insertion

Correct response:

Applying a lubricant to the suppository

Explanation:

Question 5 See full question

A client complains of severe abdominal pain. To elicit as much information as possible about the pain, the nurse
should ask:

You Selected:

"Is the pain stabbing like a knife?"

Correct response:

"Can you describe the pain?"

Explanation:

Question 6 See full question

A nurse suspects that a child, age 4, is being neglected physically. To best assess the child's nutritional status, the
nurse should ask the parents which question?
You Selected:

"What did your child eat for breakfast?"

Correct response:

"What did your child eat for breakfast?"

Explanation:

Question 7 See full question

Parents of a 4-year-old child with acute leukemia ask a nurse to explain the concept of complementary therapy. The
nurse should tell the parents that:

You Selected:

there's no research that indicates that complementary therapies are effective.

Correct response:

complementary therapy is an alternative to conventional medical therapies.

Explanation:

Question 8 See full question

When performing a physical examination on an anxious client, a nurse should expect to find which effect produced
by the parasympathetic nervous system?

You Selected:

Constipation

Correct response:

Hyperactive bowel sounds

Explanation:

Question 9 See full question

A depressed client in the psychiatric unit hasn't been getting adequate rest and sleep. To encourage restful sleep at
night, the nurse should:

You Selected:

encourage environmental stimulation during the evening.

Correct response:

gently but firmly set limits on how much time the client spends in bed during the day.

Explanation:

Question 10 See full question

A client is in the manic phase of bipolar disorder. To help the client maintain adequate nutrition, the nurse should
plan to:

You Selected:

let the client choose his favorite foods.

Correct response:

offer finger foods and sandwiches.


Explanation:

Question 11 See full question

As a client progresses through pregnancy, she develops constipation. What is the primary cause of this problem
during pregnancy?

You Selected:

Decreased appetite

Correct response:

Reduced intestinal motility

Explanation:

Question 12 See full question

During a routine prenatal visit, a pregnant client reports heartburn. To minimize her discomfort, the nurse should
include which suggestion in the care plan?

You Selected:

Eat small, frequent meals.

Correct response:

Eat small, frequent meals.

Explanation:

Question 13 See full question

A client is recovering from an infected abdominal wound. Which foods should the nurse encourage the client to eat
to support wound healing and recovery from the infection?

You Selected:

chicken and orange slices

Correct response:

chicken and orange slices

Explanation:

Question 14 See full question

A nurse is planning care for a 12-year-old with rheumatic fever. The nurse should teach the parents to:

You Selected:

observe the child closely.

Correct response:

provide for adequate periods of rest between activities.

Explanation:

Question 15 See full question

When teaching the parent of an infant with Hirschsprung's disease who received a temporary colostomy about the
types of foods the infant will be able to eat, which diet would the nurse recommend?

You Selected:

regular diet
Correct response:

regular diet

Explanation:

Question 16 See full question

The breastfeeding mother of a 1-month-old diagnosed with cow's milk sensitivity asks the nurse what she should do
about feeding her infant. Which recommendation would be most appropriate?

You Selected:

Limit breastfeeding to once per day, and begin feeding an iron-fortified formula.

Correct response:

Continue to breastfeed, but eliminate all milk products from your own diet.

Explanation:

Question 17 See full question

An adolescent is on the football team and practices in the morning and afternoon before school starts for the year.
The temperature on the field has been high. The school nurse has been called to the practice field because the
adolescent is now reporting that he has muscle cramps, nausea, and dizziness. Which action should the school
nurse do first?

You Selected:

Administer cold water with ice cubes.

Correct response:

Move the adolescent to a cool environment.

Explanation:

Question 18 See full question

The client will have an abdominal hysterectomy tomorrow. Which information will be most important for the nurse
to give to the client prior to admission to the hospital?

You Selected:

what to wear to the hospital

Correct response:

what she can eat and drink before admission

Explanation:

Question 19 See full question

A client with a history of renal calculi formation is being discharged after surgery to remove the calculus. What
instructions should the nurse include in the client's discharge teaching plan?

You Selected:

Strain urine at home regularly.

Correct response:

Increase daily fluid intake to at least 2 to 3 L.


Explanation:

Question 20 See full question

A client has just returned from the postanesthesia care unit after undergoing a laryngectomy. Which intervention
should the nurse include in the plan of care?

You Selected:

Teach the client how to use esophageal speech.

Correct response:

Maintain the head of the bed at 30 to 40 degrees.

Explanation:

Question 21 See full question

Which is a priority goal for the client with chronic obstructive pulmonary disease (COPD)?

You Selected:

minimizing chest pain

Correct response:

maintaining functional ability

Explanation:

Question 22 See full question

The client who had a permanent pacemaker implanted 2 days earlier is being discharged from the hospital. The
nurse knows that the client understands the discharge plan when the client:

You Selected:

states a need for bed rest for 1 week after discharge.

Correct response:

verbalizes safety precautions needed to prevent pacemaker malfunction.

Explanation:

Question 23 See full question

When explaining to a pregnant client about the need to take supplemental vitamins with iron during her pregnancy,
the nurse should instruct the client to take the iron with which liquid to promote maximum absorption?

You Selected:

orange juice

Correct response:

orange juice

Explanation:

Question 24 See full question

The nurse notes that a client is too busy investigating the unit and overseeing the activities of other clients to eat
dinner. To help the client obtain sufficient nourishment, which plan would be best?

You Selected:

Serve foods that she can carry with her.


Correct response:

Serve foods that she can carry with her.

Explanation:

Question 25 See full question

A preschool child immobilized in a hip spica cast has trouble breathing after meals. Which action would be best?

You Selected:

Give the child a laxative after meals.

Correct response:

Offer the child small feedings several times a day.

Explanation:

Question 26 See full question

The nurse is preparing the client with heart failure to go home. The nurse should instruct the client to:

You Selected:

monitor urine output daily.

Correct response:

monitor weight daily.

Explanation:

Question 27 See full question

Which nursing recommendation is most appropriate for a client to decrease discomfort from hemorrhoids?

You Selected:

Decrease physical activity.

Correct response:

Use warm sitz baths.

Explanation:

Question 28 See full question

A client is learning about caring for an ileostomy. Which statement would indicate that the client understands how
to care for the ileostomy pouch?

You Selected:

"I will empty my pouch when it is about one-third full.

Correct response:

"I will empty my pouch when it is about one-third full.

Explanation:

Question 29 See full question

When helping the client who has had a cerebrovascular accident (CVA) learn self-care skills, the nurse should:

You Selected:

teach the client to put on clothing on the affected side first.


Correct response:

teach the client to put on clothing on the affected side first.

Explanation:

Question 30 See full question

An elderly client asks the nurse how to treat chronic constipation. What is the best recommendation the nurse can
make?

You Selected:

Administer a phospho-soda enema when necessary.

Correct response:

Take a stool softener such as docusate sodium daily.

Explanation:

Question 31 See full question

A client with right sided hemiparesis has limited mobility. Which action should the nurse include in the plan of care
to help maintain skin integrity?

You Selected:

Perform passive range-of-motion (ROM) exercises.

Correct response:

Turn him regularly.

Explanation:

Question 32 See full question

A nurse is caring for a client who has limited mobility and requires a wheelchair. The nurse has concern for
circulation problems when which device is used?

You Selected:

Ring or donut

Correct response:

Ring or donut

Explanation:

Question 33 See full question

A nurse is providing dietary instructions to a client with hypoglycemia. To control hypoglycemic episodes, the nurse
should recommend:

You Selected:

increasing saturated fat intake and fasting in the afternoon.

Correct response:

consuming a low-carbohydrate, high-protein diet and avoiding fasting.

Explanation:

Question 34 See full question

After a radical prostatectomy for prostate cancer, a client has an indwelling catheter removed. The client then
begins to have periods of incontinence. During the postoperative period, which intervention should be implemented
first?
You Selected:

Kegel exercises

Correct response:

Kegel exercises

Explanation:

Question 35 See full question

After having transurethral resection of the prostate (TURP), a client returns to the unit with a three-way indwelling
urinary catheter and continuous closed bladder irrigation. Which finding suggests that the client's catheter is
occluded?

You Selected:

The client reports bladder spasms and the urge to void.

Correct response:

The client reports bladder spasms and the urge to void.

Explanation:

Question 36 See full question

A female client reports to a nurse that she experiences a loss of urine when she jogs. The nurse's assessment
reveals no nocturia, burning, discomfort when voiding, or urine leakage before reaching the bathroom. The nurse
explains to the client that this type of problem is called:

You Selected:

reflex incontinence.

Correct response:

stress incontinence.

Explanation:

Question 37 See full question

A client is receiving morphine sulfate by a patient-controlled analgesia (PCA) system after a left lower lobectomy
about 4 hours ago. The client reports moderately severe pain in the left thorax that worsens when coughing. The
nurse's first course of action is to:

You Selected:

encourage the client to take deep breathes and expectorate the mucous that is stimulating the cough.

Correct response:

assess the pain systematically with the hospital-approved pain scale.

Explanation:

Question 38 See full question

A client is admitted with severe abdominal pains and the diagnosis of acute pancreatitis. The nurse should develop
a plan of care during the acute phase of pancreatitis that will involve interventions to manage:

You Selected:

risk for injury.

Correct response:

severe pain.
Explanation:

Question 39 See full question

The client with hepatitis A is experiencing fatigue, weakness, and a general feeling of malaise. The client tires
rapidly during morning care. The most appropriate goal for this client is to:

You Selected:

gradually increase activity tolerance.

Correct response:

gradually increase activity tolerance.

Explanation:

Question 40 See full question

Which goal is a priority for the diabetic client who is taking insulin and has nausea and vomiting from a viral illness
or influenza?

You Selected:

relieving pain

Correct response:

obtaining adequate food intake

Explanation:

Question 41 See full question

A client with Addisons disease has fluid and electrolyte loss due to inadequate fluid intake and to fluid loss
secondary to inadequate adrenal hormone secretion. As the clients oral intake increases, which fluids would
be most appropriate?

You Selected:

milk and diet soda

Correct response:

chicken broth and juice

Explanation:

Question 42 See full question

An appropriate nursing intervention for a client with fatigue related to cancer treatment includes teaching the client
to:

You Selected:

minimize naps or periods of rest during the day.

Correct response:

conserve energy by prioritizing activities.

Explanation:

Question 43 See full question

A nurse observes an LPN measuring a clients urine output from an indwelling catheter drainage bag. Which
observation by the nurse ensures that the clients urine has been measured accurately?

You Selected:

The LPN pours the urine into a paper cup that holds approximately 250 mL.
Correct response:

The LPN pours the urine into a graduated measuring container.

Explanation:

Question 44 See full question

A client returned from surgery eight hours ago and has not voided. Which action should the nurse take first?

You Selected:

Call the physician to report the client's condition.

Correct response:

Palpate over the synthesis pubis for fullness.

Explanation:

Question 45 See full question

The nurse is teaching a client with multiple sclerosis about prevention of urinary tract infection (UTI) and renal
calculi. Which of the following nutrition recommendations by the nurse would be the most likely to reduce the risk of
these conditions?

You Selected:

Eat foods containing vitamins C, D, and E, and drink at least 2 L of fluid a day.

Correct response:

Increase fluids (2500 mL/day) and maintain urine acidity by drinking cranberry juice.

Explanation:

Question 46 See full question

A nursing assessment for a client with alcohol abuse reveals a disheveled appearance and a foul body odor. What is
the best initial nursing plan that would assist the clients involvement in personal care?

You Selected:

Bathing and dressing the client each morning until the client is willing to perform self-care independently

Correct response:

Assisting the client with bathing and dressing by giving clear, simple directions

Explanation:

Question 47 See full question

An 15-month-old child is recovering from surgery to remove Wilms' tumor. The nurse is performing a postoperative
pain assessment. Which of the following parameters indicates that the child may be experiencing pain? Select all
that apply.

You Selected:

Crying

Correct response:

Crying
Increasing heart rate
Touching the painful area
Explanation:

Question 48 See full question

The nurse is caring for a postoperative client who has not voided since before surgery. Which is the nurse's most
appropriate action?

You Selected:

Palpate for the bladder above the symphysis pubis

Correct response:

Palpate for the bladder above the symphysis pubis

Explanation:

Question 49 See full question

A client with Tourette syndrome is seen in an outpatient clinic. The client has multiple tics occurring several times
per day. The nurse notices that the client has a difficult time completing tasks such as activities of daily living
(ADLs). In which of the following ways can the nurse best help this client?

You Selected:

Complete the ADLs for the client

Correct response:

Break down tasks into small achievable steps

Explanation:

Question 50 See full question

The nurse is planning care for a client admitted for vascular dementia. Which action is most appropriate in
assisting the client with activities of daily living?

You Selected:

Inform client that if morning care is not completed by 0830 hours, the UAP will complete it.

Correct response:

Encourage client to complete as many activities as possible, and provide ample time to complete them.

Explanation:

Question 1 See full question

As a nurse helps a client ambulate, the client says, "I had trouble sleeping last night." Which action should the
nurse take first?

You Selected:

Finding out whether the client is taking medication that may impede sleep

Correct response:

Gathering more information about the client's sleep problem

Explanation:

Question 2 See full question

A nurse has been teaching a client about a high-protein diet. The teaching is successful if the client identifies which
meal as high in protein?

You Selected:

Spaghetti with cream sauce, broccoli, and tea


Correct response:

Baked beans, hamburger, and milk

Explanation:

Question 3 See full question

For a client with a nursing diagnosis of Insomnia, the nurse should use which measure to promote sleep?

You Selected:

Increasing the client's activity 2 hours before bedtime

Correct response:

Playing soft or soothing music

Explanation:

Question 4 See full question

The mother of an 11-month-old infant reports to the nurse that her infant sleeps much less than other children. The
mother asks the nurse whether her infant is getting sufficient sleep. What should be the nurse's initial response?

You Selected:

Ask the mother for more information about the infant's sleep patterns.

Correct response:

Ask the mother for more information about the infant's sleep patterns.

Explanation:

Question 5 See full question

Which relaxation strategy would be effective for a school-age child to use during a painful procedure?

You Selected:

Having the child keep his eyes shut at all times

Correct response:

Having the child take a deep breath and blow it out until told to stop

Explanation:

Question 6 See full question

A nurse is caring for a client with bulimia nervosa. Strict management of the client's dietary intake is necessary.
Which intervention is the most important?

You Selected:

Fill out the client's menu and make sure she eats at least half of what is on her tray.

Correct response:

Serve the client's menu choices in a supervised area and observe her 1 hour after each meal.

Explanation:

Question 7 See full question

As a client progresses through pregnancy, she develops constipation. What is the primary cause of this problem
during pregnancy?
You Selected:

Decreased appetite

Correct response:

Reduced intestinal motility

Explanation:

Question 8 See full question

A client who's 2 months pregnant complains of urinary frequency and says she gets up several times at night to go
to the bathroom. She denies other urinary symptoms. How should the nurse intervene?

You Selected:

Refer the client to a urologist for further investigation.

Correct response:

Explain that urinary frequency is expected during the first trimester.

Explanation:

Question 9 See full question

A nurse is caring for a 16-year-old pregnant adolescent. The client is taking an iron supplement. What should this
client drink to increase the absorption of iron?

You Selected:

A cup of hot tea.

Correct response:

A glass of orange juice.

Explanation:

Question 10 See full question

A pregnant client complains of nausea every morning and again before meals. As a result of the nausea, she's been
unable to eat enough and has lost weight. Which nonpharmacologic intervention should the nurse recommend?

You Selected:

Eating three large meals per day

Correct response:

Keeping crackers at the bedside to eat before getting out of bed

Explanation:

Question 11 See full question

A client asks about complementary therapies for relief of discomfort related to pregnancy. Which comfort measure
mentioned by the client indicates a need for further teaching?

You Selected:

Herbal remedies

Correct response:

Herbal remedies
Explanation:

Question 12 See full question

A client in labor asks the nurse about Reiki, an alternative therapy that she's heard may be useful during the
intrapartum period. The nurse tells the client that Reiki is based on the principle of:

You Selected:

vigorous massage.

Correct response:

energy from light touch.

Explanation:

Question 13 See full question

A client returns to the postnatal ward with her 3-week-old infant. Which statement by the client would prompt the
nurse to document "Imbalanced nutrition less than body requirements related to inadequate intake"?

You Selected:

"The baby does not burp after a feeding."

Correct response:

The baby does not exhibit a steady weight gain."

Explanation:

Question 14 See full question

Before discharge from the hospital after a myocardial infarction, a client is taught to exercise by gradually
increasing the distance walked. Which vital sign should the nurse teach the client to monitor to determine whether
to increase or decrease the exercise level?

You Selected:

blood pressure

Correct response:

pulse rate

Explanation:

Question 15 See full question

A client is recovering from an infected abdominal wound. Which foods should the nurse encourage the client to eat
to support wound healing and recovery from the infection?

You Selected:

cheese omelet and bacon

Correct response:

chicken and orange slices

Explanation:

Question 16 See full question

A 6-year-old child is admitted for an appendectomy. What is the most appropriate way for the nurse to prepare the
child for surgery?

You Selected:

Explain how to use a patient-controlled analgesia (PCA) pump for pain control.
Correct response:

Permit the child to play with the blood pressure cuff, electrocardiogram (ECG) pads, and a face mask.

Explanation:

Question 17 See full question

The breastfeeding mother of a 1-month-old diagnosed with cow's milk sensitivity asks the nurse what she should do
about feeding her infant. Which recommendation would be most appropriate?

You Selected:

Continue to breastfeed, but eliminate all milk products from your own diet.

Correct response:

Continue to breastfeed, but eliminate all milk products from your own diet.

Explanation:

Question 18 See full question

A preschooler with a fractured femur of the left leg in traction tells the nurse that his leg hurts. It is too early for
pain medication. The nurse should:

You Selected:

reposition the pulleys so the traction is looser.

Correct response:

assess the feet for signs of neurovascular impairment.

Explanation:

Question 19 See full question

The nurse is preparing to administer a preoperative medication that includes a sedative to a client who is having
abdominal surgery. The nurse should first:

You Selected:

ensure that the operative area has been shaved.

Correct response:

have the client empty the bladder.

Explanation:

Question 20 See full question

A client with rheumatoid arthritis tells the nurse, I know it is important to exercise my joints so that I will not lose
mobility, but my joints are so stiff and painful that exercising is difficult. Which response by the nurse would
be most appropriate?

You Selected:

"Stiffness and pain are part of the disease. Learn to cope by focusing on activities you enjoy."

Correct response:

"Take a warm tub bath or shower before exercising. This may help with your discomfort."
Explanation:

Question 21 See full question

As a first step in teaching a woman with a spinal cord injury and quadriplegia about her sexual health, the nurse
assesses her understanding of her current sexual functioning. Which statement by the client indicates she
understands her current ability?

You Selected:

"I should be able to participate in sexual activity, but I will be infertile."

Correct response:

"I can participate in sexual activity but might not experience orgasm."

Explanation:

Question 22 See full question

The client with diabetes mellitus says, "If I could just avoid what you call carbohydrates in my diet, I guess I would
be okay." The nurse should base the response to this comment on the knowledge that diabetes affects metabolism
of which nutrients?

You Selected:

protein and carbohydrates only

Correct response:

proteins, fats, and carbohydrates

Explanation:

Question 23 See full question

A client has just returned from the postanesthesia care unit after undergoing a laryngectomy. Which intervention
should the nurse include in the plan of care?

You Selected:

Teach the client how to use esophageal speech.

Correct response:

Maintain the head of the bed at 30 to 40 degrees.

Explanation:

Question 24 See full question

Which is a priority goal for the client with chronic obstructive pulmonary disease (COPD)?

You Selected:

increasing carbon dioxide levels in the blood

Correct response:

maintaining functional ability

Explanation:

Question 25 See full question

A client develops chronic pancreatitis. What would be the appropriate home diet for a client with chronic
pancreatitis?

You Selected:

a low-protein, high-fiber diet distributed over four to five moderate-sized meals daily
Correct response:

A low-fat, bland diet distributed over five to six small meals daily

Explanation:

Question 26 See full question

When developing a long term care plan for the client with multiple sclerosis, the nurse should teach the client to
prevent:

You Selected:

dry mouth.

Correct response:

contractures.

Explanation:

Question 27 See full question

Which measure would be most effective for the client to use at home when managing the discomfort of rhinoplasty
2 days after surgery?

You Selected:

Lie in a prone position.

Correct response:

Apply ice compresses.

Explanation:

Question 28 See full question

A client receiving external radiation to the left thorax to treat lung cancer has a nursing diagnosis of Risk for
impaired skin integrity. Which intervention should be part of this client's care plan?

You Selected:

Avoiding using deodorant soap on the irradiated areas

Correct response:

Avoiding using deodorant soap on the irradiated areas

Explanation:

Question 29 See full question

A female client reports to a nurse that she experiences a loss of urine when she jogs. The nurse's assessment
reveals no nocturia, burning, discomfort when voiding, or urine leakage before reaching the bathroom. The nurse
explains to the client that this type of problem is called:

You Selected:

functional incontinence.

Correct response:

stress incontinence.
Explanation:

Question 30 See full question

While the nurse is caring for a primiparous client with cephalopelvic disproportion 4 hours after a cesarean birth,
the client requests assistance in breastfeeding. To promote maximum maternal comfort, which position would
be most appropriate for the nurse to suggest?

You Selected:

cross-cradle hold

Correct response:

football hold

Explanation:

Question 31 See full question

A postpartum woman who gave birth vaginally has unrelenting rectal pain despite the administration of pain
medication. Which action is most indicated?

You Selected:

preparing a warm sitz bath for the client

Correct response:

assessing the perineum

Explanation:

Question 32 See full question

While making a home visit to a multigravida 2 weeks after the birth of viable twins at 38 weeks gestation, the
nurse observes that the client looks pale, has dark circles around her eyes, and is breastfeeding one of the twins.
The clients apartment is clean, and nothing appears out of place. The client tells the nurse that she completed
three loads of laundry this morning. A priority need for this client is:

You Selected:

anxiety related to inability to cope with twins who are breast-feeding.

Correct response:

fatigue related to home maintenance and caring for twins.

Explanation:

Question 33 See full question

Which intervention should the nurse suggest to a parent to relieve itching in a child with chicken pox?

You Selected:

soft towels moistened with hydrogen peroxide

Correct response:

oatmeal preparation baths

Explanation:

Question 34 See full question

Which assessment is most appropriate for determining the correct placement of an endotracheal tube in a
mechanically ventilated client?
You Selected:

verifying the amount of cuff inflation

Correct response:

auscultating breath sounds bilaterally

Explanation:

Question 35 See full question

While changing the clients colostomy bag and dressing, the nurse determines that the client is ready to participate
in self-care when the client:

You Selected:

talks about the news on the television.

Correct response:

asks about the supplies used during the dressing change.

Explanation:

Question 36 See full question

When explaining hospice care to a client, the nurse should tell the client:

You Selected:

"Hospice care uses a team approach to direct hospice activity."

Correct response:

"Clients and their families are the focus of care."

Explanation:

Question 37 See full question

A nurse is assessing an immobile client and notes an area of sacral skin is reddened, but not broken. The reddened
area continues to blanch and refill with fingertip pressure. The most appropriate nursing action at this time is to:

You Selected:

apply a moist-to-moist dressing, being careful to pack just the wound bed.

Correct response:

reposition the client off the reddened skin and reassess in a few hours.

Explanation:

Question 38 See full question

A typically developing preschool child is experiencing pain after an appendectomy. Which data collection tool is
the most appropriate for the nurse use to assess the pain?

You Selected:

visual analog scale

Correct response:

FACES Pain Rating Scale


Explanation:

Question 39 See full question

The nurse teaches a client who has recently been diagnosed with hypertension about following a low-calorie, low-
fat, low-sodium diet. Which menu selection would best meet the client's needs?

You Selected:

mixed green salad with blue cheese dressing, crackers, and cold cuts

Correct response:

baked chicken, an apple, and a slice of white bread

Explanation:

Question 40 See full question

A client with Addisons disease has fluid and electrolyte loss due to inadequate fluid intake and to fluid loss
secondary to inadequate adrenal hormone secretion. As the clients oral intake increases, which fluids would
be most appropriate?

You Selected:

chicken broth and juice

Correct response:

chicken broth and juice

Explanation:

Question 41 See full question

Which is an expected outcome for a client with Parkinson's disease who has had a pallidotomy?

You Selected:

reduced emotional stress

Correct response:

improved functional ability

Explanation:

Question 42 See full question

A client is ordered to receive a sodium phosphate enema for relief of constipation. Proper administration of the
enema includes which steps? Select all that apply.

You Selected:

Washing hands and putting on gloves.

Correct response:

Assisting the client into Sims' position.


Washing hands and putting on gloves.
Encouraging the client to retain the solution for 5 to 15 minutes.

Explanation:

Question 43 See full question

What role will the nurse play in transferring a client to a long-term care facility?
You Selected:

Provide a verbal report to the nurse at the long-term care facility on the client, the hospital care, and the
client's current condition.

Correct response:

Provide a verbal report to the nurse at the long-term care facility on the client, the hospital care, and the
client's current condition.

Explanation:

Question 44 See full question

A child has a nasogastric (NG) tube inserted by the nurse to administer a continuous feeding. Which of the following
actions should the nurse take before starting the NG feeding on the child? Select all that apply.

You Selected:

Check placement of the NG tube.

Correct response:

Verify the physicians order.


Check placement of the NG tube.
Assess for bowel sounds.

Explanation:

Question 45 See full question

A nursing assessment for a client with alcohol abuse reveals a disheveled appearance and a foul body odor. What is
the best initial nursing plan that would assist the clients involvement in personal care?

You Selected:

Drawing up a schedule and making certain that it is adhered to

Correct response:

Assisting the client with bathing and dressing by giving clear, simple directions

Explanation:

Question 46 See full question

Which of the following is a common method of evaluating the urine output for newborns, infants, and toddlers who
are not potty trained.

You Selected:

Weighing the child before and after feeds

Correct response:

Weighing the diaper before and after micturition

Explanation: Question 1 See full question

For a client with anorexia nervosa, which goal takes the highest priority?
You Selected:

The client will establish adequate daily nutritional intake.


Correct response:

The client will establish adequate daily nutritional intake.


Explanation:

Question 2 See full question

Before placement of a ventriculoperitoneal shunt for hydrocephalus, an


infant is irritable, lethargic, and difficult to feed. To maintain the infant's
nutritional status, which action would be most appropriate?
You Selected:

Give the infant small, frequent feedings.


Correct response:

Give the infant small, frequent feedings.


Explanation:

Question 3 See full question

The nurse should assess the client with hypothyroidism for:


You Selected:

decreased activity due to fatigue.


Correct response:

decreased activity due to fatigue.


Explanation:

Question 4 See full question

A client with rheumatoid arthritis has increasing fatigue and is unable to


manage all of the usual activities. The nurse should:
You Selected:

encourage the client to alternate periods of rest and activity throughout the
day.
Correct response:

encourage the client to alternate periods of rest and activity throughout the
day.
Explanation:

Question 5 See full question

Which of the following is a common method of evaluating the urine output


for newborns, infants, and toddlers who are not potty trained.
You Selected:

Weighing the diaper before and after micturition


Correct response:

Weighing the diaper before and after micturition


Explanation:

Question 47 See full question

An 15-month-old child is recovering from surgery to remove Wilms' tumor. The nurse is performing a postoperative
pain assessment. Which of the following parameters indicates that the child may be experiencing pain? Select all
that apply.

You Selected:

Smiling

Correct response:

Crying
Increasing heart rate
Touching the painful area

Explanation:

Question 48 See full question

The nurse is caring for a postoperative client who has not voided since before surgery. Which is the nurse's most
appropriate action?

You Selected:

Initiate hourly intake and output measurement

Correct response:

Palpate for the bladder above the symphysis pubis

Explanation:

Question 49 See full question

The nurse is caring for a 5 year-old that had surgery 12 hours ago. The child tells the nurse that she does not have
pain, but a few minutes later tells her parent that she does. Which would the nurse consider when interpreting this?

You Selected:

Inconsistency in pain reporting suggests that pain is not present.

Correct response:

Children may be experiencing pain even though they deny it to the nurse.

Explanation:

Question 50 See full question

The nurse is planning care for a client who has been experiencing a manic episode for 6 days and is unable to sit
still long enough to eat meals. Which choice will best meet the clients nutritional needs at this time?

You Selected:

Offer a green salad topped with chicken pieces.


Correct response:

Offer a peanut butter sandwich.

Explanation:

Question 1 See full question

When planning pain control for a client with terminal gastric cancer, a nurse should consider that:

You Selected:

a client who can fall asleep isn't in pain.

Correct response:

clients with terminal cancer may develop tolerance to opioids.

Explanation:

Question 2 See full question

A nurse suspects that a child, age 4, is being neglected physically. To best assess the child's nutritional status, the
nurse should ask the parents which question?

You Selected:

"Is your child a picky eater?"

Correct response:

"What did your child eat for breakfast?"

Explanation:

Question 3 See full question

An adolescent is diagnosed with iron deficiency anemia. After emphasizing the importance of consuming dietary
iron, the nurse asks him to select iron-rich breakfast items from a sample menu. Which selection demonstrates
knowledge of dietary iron sources?

You Selected:

Pancakes and a banana

Correct response:

Ham and eggs

Explanation:

Question 4 See full question

An adolescent is being nursed with a skeletal traction for a fractured femur. Which is the most appropriate nursing
intervention for this client?

You Selected:

Ensure that the rope knots catch on the pulley.

Correct response:

Assess pin sites every shift and as needed.

Explanation:

Question 5 See full question

A depressed client in the psychiatric unit hasn't been getting adequate rest and sleep. To encourage restful sleep at
night, the nurse should:
You Selected:

encourage the client to take an antianxiety agent as needed at bedtime.

Correct response:

gently but firmly set limits on how much time the client spends in bed during the day.

Explanation:

Question 6 See full question

A client is in the manic phase of bipolar disorder. To help the client maintain adequate nutrition, the nurse should
plan to:

You Selected:

let the client choose his favorite foods.

Correct response:

offer finger foods and sandwiches.

Explanation:

Question 7 See full question

During a prenatal visit, a pregnant client with cardiac disease and slight functional limitations reports increased
fatigue. To help combat this problem, the nurse should advise her to:

You Selected:

eat three well-balanced meals per day.

Correct response:

divide daily food intake into five or six meals.

Explanation:

Question 8 See full question

A nurse is reviewing a client's fluid intake and output record. Fluid intake and urine output should relate in which
way?

You Selected:

Fluid intake should be about equal to the urine output.

Correct response:

Fluid intake should be about equal to the urine output.

Explanation:

Question 9 See full question

When developing a plan of care to manage a clients pain from cancer, what should the nurse plan to do?

You Selected:

Administer pain medication as soon as the client requests it.

Correct response:

Individualize the pain medication regimen for the client.


Explanation:

Question 10 See full question

The nurse uses Montgomery straps primarily so the client is free from:

You Selected:

falls.

Correct response:

skin breakdown.

Explanation:

Question 11 See full question

A 6-year-old child is admitted for an appendectomy. What is the most appropriate way for the nurse to prepare the
child for surgery?

You Selected:

Show the child a visual analog scale (VAS) based on a scale from 0 to 10.

Correct response:

Permit the child to play with the blood pressure cuff, electrocardiogram (ECG) pads, and a face mask.

Explanation:

Question 12 See full question

The breastfeeding mother of a 1-month-old diagnosed with cow's milk sensitivity asks the nurse what she should do
about feeding her infant. Which recommendation would be most appropriate?

You Selected:

Continue to breastfeed, but eliminate all milk products from your own diet.

Correct response:

Continue to breastfeed, but eliminate all milk products from your own diet.

Explanation:

Question 13 See full question

A preschooler with a fractured femur of the left leg in traction tells the nurse that his leg hurts. It is too early for
pain medication. The nurse should:

You Selected:

place a pillow under the child's buttocks to provide support.

Correct response:

assess the feet for signs of neurovascular impairment.

Explanation:

Question 14 See full question

A client in a hospice program has increasing pain. The nurse and client collaborate to schedule analgesics to
provide:

You Selected:

an analgesia-free period so that the client can carry out daily hygienic activities.
Correct response:

around-the-clock routine administration of analgesics for continuous pain relief.

Explanation:

Question 15 See full question

When planning diet teaching for the client with a colostomy, the nurse should develop a plan that emphasizes
which dietary instruction?

You Selected:

Foods containing roughage should not be eaten.

Correct response:

Clients should experiment to find the diet that is best for them.

Explanation:

Question 16 See full question

A client has just returned from the postanesthesia care unit after undergoing a laryngectomy. Which intervention
should the nurse include in the plan of care?

You Selected:

Teach the client how to use esophageal speech.

Correct response:

Maintain the head of the bed at 30 to 40 degrees.

Explanation:

Question 17 See full question

The nurse is preparing the client with heart failure to go home. The nurse should instruct the client to:

You Selected:

monitor weight daily.

Correct response:

monitor weight daily.

Explanation:

Question 18 See full question

Which nursing recommendation is most appropriate for a client to decrease discomfort from hemorrhoids?

You Selected:

Decrease physical activity.

Correct response:

Use warm sitz baths.

Explanation:

Question 19 See full question

The nurse gives a pamphlet that describes Kegel exercises to a client with stress incontinence. Which statement
indicates that the client has understood the instructions contained in the pamphlet?
You Selected:

I should perform these exercises every evening.

Correct response:

I can do these exercises sitting up, lying down, or standing.

Explanation:

Question 20 See full question

Which measure would be most effective for the client to use at home when managing the discomfort of rhinoplasty
2 days after surgery?

You Selected:

Apply ice compresses.

Correct response:

Apply ice compresses.

Explanation:

Question 21 See full question

Which is an appropriate nursing goal for the client who has ulcerative colitis? The client:

You Selected:

uses a heating pad to decrease abdominal cramping.

Correct response:

verbalizes the importance of small, frequent feedings.

Explanation:

Question 22 See full question

A nurse is teaching an elderly client about developing good bowel habits. Which statement by the client indicates to
the nurse that additional teaching is required?

You Selected:

"I need to use laxatives regularly to prevent constipation."

Correct response:

"I need to use laxatives regularly to prevent constipation."

Explanation:

Question 23 See full question

A nurse asks a client who had abdominal surgery 3 days ago if he has moved his bowels since surgery. The client
states, "I haven't moved my bowels, but I am passing gas." How should the nurse intervene?

You Selected:

Administer a tap water enema.

Correct response:

Encourage the client to ambulate at least three times per day.


Explanation:

Question 24 See full question

A nurse is assisting a client with range-of-motion exercises. The nurse moves the client's leg out and away from the
midline of the body. What movement does the nurse document?

You Selected:

Adduction

Correct response:

Abduction

Explanation:

Question 25 See full question

After having transurethral resection of the prostate (TURP), a client returns to the unit with a three-way indwelling
urinary catheter and continuous closed bladder irrigation. Which finding suggests that the client's catheter is
occluded?

You Selected:

The urine in the drainage bag appears red to pink.

Correct response:

The client reports bladder spasms and the urge to void.

Explanation:

Question 26 See full question

A client is frustrated and embarrassed by urinary incontinence. Which measure should the nurse include in a
bladder retraining program?

You Selected:

Encouraging the client to increase the time between voidings

Correct response:

Assessing present voiding patterns

Explanation:

Question 27 See full question

A female client reports to a nurse that she experiences a loss of urine when she jogs. The nurse's assessment
reveals no nocturia, burning, discomfort when voiding, or urine leakage before reaching the bathroom. The nurse
explains to the client that this type of problem is called:

You Selected:

stress incontinence.

Correct response:

stress incontinence.

Explanation:

Question 28 See full question

The nurse is caring for a full-term, nonmedicated, primiparous client who is in the transition stage of labor. The
client is writhing in pain and saying, Help me, help me! Her last vaginal exam 1 hour ago showed that she was 8
cm dilated, +1 station, and in what appeared to be a comfortable position. What does the nurse anticipate as the
highest priority intervention in caring for this client?
You Selected:

Perform a vaginal examination to determine if the client is fully dilated.

Correct response:

Perform a vaginal examination to determine if the client is fully dilated.

Explanation:

Question 29 See full question

While making a home visit to a multigravida 2 weeks after the birth of viable twins at 38 weeks gestation, the
nurse observes that the client looks pale, has dark circles around her eyes, and is breastfeeding one of the twins.
The clients apartment is clean, and nothing appears out of place. The client tells the nurse that she completed
three loads of laundry this morning. A priority need for this client is:

You Selected:

fatigue related to home maintenance and caring for twins.

Correct response:

fatigue related to home maintenance and caring for twins.

Explanation:

Question 30 See full question

The nurse observes a client with a history of panic attacks is hyperventilating. The nurse should:

You Selected:

tell the client to take several deep, slow breaths and exhale normally.

Correct response:

have the client breathe into a paper bag.

Explanation:

Question 31 See full question

A client is recovering from a gastric resection for peptic ulcer disease. Which outcome indicates that the goal of
adequate nutritional intake is being achieved 3 weeks following surgery? The client:

You Selected:

increases food intake and tolerance gradually.

Correct response:

increases food intake and tolerance gradually.

Explanation:

Question 32 See full question

The nurse instructs the client on health maintenance activities to help control symptoms from a hiatal hernia. Which
statement would indicate that the client has understood the instructions?

You Selected:

"I will avoid lying down after a meal."

Correct response:

"I will avoid lying down after a meal."


Explanation:

Question 33 See full question

The nurse teaches a client who has recently been diagnosed with hypertension about following a low-calorie, low-
fat, low-sodium diet. Which menu selection would best meet the client's needs?

You Selected:

ham sandwich on rye bread and an orange

Correct response:

baked chicken, an apple, and a slice of white bread

Explanation:

Question 34 See full question

Which goal is a priority for the diabetic client who is taking insulin and has nausea and vomiting from a viral illness
or influenza?

You Selected:

relieving pain

Correct response:

obtaining adequate food intake

Explanation:

Question 35 See full question

A client with Addisons disease has fluid and electrolyte loss due to inadequate fluid intake and to fluid loss
secondary to inadequate adrenal hormone secretion. As the clients oral intake increases, which fluids would
be most appropriate?

You Selected:

chicken broth and juice

Correct response:

chicken broth and juice

Explanation:

Question 36 See full question

A child has a nasogastric (NG) tube inserted by the nurse to administer a continuous feeding. Which of the following
actions should the nurse take before starting the NG feeding on the child? Select all that apply.

You Selected:

Check placement of the NG tube.

Correct response:

Verify the physicians order.


Check placement of the NG tube.
Assess for bowel sounds.

Explanation:

Question 37 See full question

The nurse is caring for a client with bipolar disorder who was recently admitted to an inpatient unit and is
experiencing a manic episode. What is a priority nursing intervention for this client?
You Selected:

Closely monitor the clients eating and sleeping habits.

Correct response:

Closely monitor the clients eating and sleeping habits.

Explanation:

Question 38 See full question

A nurse observes an LPN measuring a clients urine output from an indwelling catheter drainage bag. Which
observation by the nurse ensures that the clients urine has been measured accurately?

You Selected:

The LPN uses the measuring markings on the Foley drainage bag.

Correct response:

The LPN pours the urine into a graduated measuring container.

Explanation:

Question 39 See full question

The nurse evaluates the client's understanding of nutritional modifications to manage his hypertension when he
states:

You Selected:

"Limiting my salt intake to 2 grams per day will improve my blood pressure."

Correct response:

"Limiting my salt intake to 2 grams per day will improve my blood pressure."

Explanation:

Question 40 See full question

The nurse is teaching a client with multiple sclerosis about prevention of urinary tract infection (UTI) and renal
calculi. Which of the following nutrition recommendations by the nurse would be the most likely to reduce the risk of
these conditions?

You Selected:

Eat foods and ingest fluids that will cause the urine to be less acidic.

Correct response:

Increase fluids (2500 mL/day) and maintain urine acidity by drinking cranberry juice.

Explanation:

Question 41 See full question

Because of symptoms experienced after a cerebrovascular accident (CVA), the nurse discovers that a client needs
assistance using utensils while eating. What would the nurse do to support this activity of care?

You Selected:

Request that the clients food be pureed by dietary staff.

Correct response:

Encourage participation in the feeding process to the best of the client's abilities.
Explanation:

Question 42 See full question

A nurse is providing an education in-service about low-residue diets to a group of clients with colitis in a public
health clinic. Which of the following diet choices would show that teaching has been effective?

You Selected:

Stewed chicken, baked potatoes with butter, strained peas, white bread, plain cake, and milk

Correct response:

Lean roast beef, buttered white rice with egg slices, white bread with butter and jelly, and tea with sugar

Explanation:

Question 43 See full question

Which of the following observations by the nurse would indicate that a client is unable to tolerate a continuation of
a tube feeding?

You Selected:

Formula in the clients mouth during the feeding, and increased cough

Correct response:

Formula in the clients mouth during the feeding, and increased cough

Explanation:

Question 44 See full question

A 7-year-old has had an appendectomy on November 12. He has had pain for the last 24 hours. There is a
prescription to administer acetaminophen with codeine every 3 to 4 hours as needed. The nurse is beginning the
shift, and the child is requesting pain medication. The nurse reviews the chart below for pain history. Based on the
information in the medical record, what should the nurse do next?

You Selected:

Distract the child by giving him breakfast.

Correct response:

Administer the acetaminophen with codeine.

Explanation: Question 1 See full question

The nurse is developing a discharge plan for a client who has had a
myocardial infarction. Planning for discharge for this client should begin:
You Selected:

on admission to the hospital.


Correct response:

on admission to the hospital.


Explanation:

Question 2 See full question

Which approach would be most effective when the nurse is communicating


with a client who has a hearing impairment?
You Selected:

Stand in front of the client and speak slowly and clearly.


Correct response:

Stand in front of the client and speak slowly and clearly.


Explanation:

Question 3 See full question

Which assessment is most appropriate for determining the correct


placement of an endotracheal tube in a mechanically ventilated client?
You Selected:

auscultating breath sounds bilaterally


Correct response:

auscultating breath sounds bilaterally


Explanation:

Question 4 See full question

Which of the following is a common method of evaluating the urine output


for newborns, infants, and toddlers who are not potty trained.
You Selected:

Weighing the diaper before and after micturition


Correct response:

Weighing the diaper before and after micturition


Explanation:

Question 5 See full question

A 7-year-old has had an appendectomy on November 12. He has had pain for
the last 24 hours. There is a prescription to administer acetaminophen with
codeine every 3 to 4 hours as needed. The nurse is beginning the shift, and
the child is requesting pain medication. The nurse reviews the chart below
for pain history. Based on the information in the medical record, what should
the nurse do next?
You Selected:

Administer the acetaminophen with codeine.


Correct response:

Administer the acetaminophen with codeine.


Explanation:

Question 45 See full question

An 15-month-old child is recovering from surgery to remove Wilms' tumor. The nurse is performing a postoperative
pain assessment. Which of the following parameters indicates that the child may be experiencing pain? Select all
that apply.

You Selected:

Increasing interest in play

Correct response:

Crying
Increasing heart rate
Touching the painful area

Explanation:

Question 1 See full question

The nurse is developing a discharge plan for a client who has had a
myocardial infarction. Planning for discharge for this client should begin:
You Selected:

on admission to the hospital.


Correct response:

on admission to the hospital.


Explanation:

Question 2 See full question

In developing a teaching plan for the client with a hiatal hernia, the nurse's
assessment of which work-related factors would be most useful?
You Selected:

body mechanics used in lifting


Correct response:

body mechanics used in lifting


Explanation:

Question 3 See full question

A client with jaundice has pruritus and areas of irritation from scratching.
What measures can the nurse suggest the client use to prevent skin
breakdown? Select all that apply.
You Selected:

Add baking soda to the water in a tub bath.


Rub the skin when it itches with knuckles instead of nails.
Keep nails short and clean.
Correct response:

Add baking soda to the water in a tub bath.


Keep nails short and clean.
Rub the skin when it itches with knuckles instead of nails.
Explanation:

Question 4 See full question

The nurse teaches a client who has recently been diagnosed with
hypertension about following a low-calorie, low-fat, low-sodium diet. Which
menu selection would best meet the client's needs?
You Selected:

baked chicken, an apple, and a slice of white bread


Correct response:

baked chicken, an apple, and a slice of white bread


Explanation:

Question 5 See full question

An 15-month-old child is recovering from surgery to remove Wilms' tumor.


The nurse is performing a postoperative pain assessment. Which of the
following parameters indicates that the child may be experiencing pain?
Select all that apply.
You Selected:

Increasing heart rate


Touching the painful area
Crying
Correct response:

Crying
Increasing heart rate
Touching the painful area
Explanation:

Question 1 See full question

A client hasn't voided since before surgery, which took place 8 hours ago.
When assessing the client, a nurse will:
You Selected:

palpate the bladder above the symphysis pubis.


Correct response:

palpate the bladder above the symphysis pubis.


Explanation:

Question 2 See full question

Which statement would provide the best guide for activity during the
rehabilitation period for a client who has been treated for retinal
detachment?
You Selected:

Activity is resumed gradually; the client can resume usual activities in 5 to 6


weeks.
Correct response:

Activity is resumed gradually; the client can resume usual activities in 5 to 6


weeks.
Explanation:

Question 3 See full question

The client who had a permanent pacemaker implanted 2 days earlier is being
discharged from the hospital. The nurse knows that the client understands
the discharge plan when the client:
You Selected:

verbalizes safety precautions needed to prevent pacemaker malfunction.


Correct response:

verbalizes safety precautions needed to prevent pacemaker malfunction.


Explanation:

Question 4 See full question

Which is an expected outcome for a client with Parkinson's disease who has
had a pallidotomy?
You Selected:

improved functional ability


Correct response:

improved functional ability


Explanation:

Question 5 See full question

Upon repositioning an immobile client, the nurse notes redness with


blanching over a bony prominence. What is the most probable cause?
You Selected:

Deep tissue damage


Correct response:

The reactive hyperemia is likely transient.


Explanation:

If the area blanches white and the erythema returns when the finger is
removed, the reactive hyperemia is likely transient. The other choices are
indicative of further infections or deep tissue damage.
s
1 A 60-year-old male client comes into the 1. Although obtaining the ECG, chest
. emergency department with a radio-graph, and blood work are all
complaint of crushing substernal chest important, the nurse's priority action
pain that radiates to his shoulder and should be to relieve the crushing chest
left arm. The admitting diagnosis is pain. Therefore, administering morphine
acute myo-cardial infarction (MI). sulfate is the priority action.
Immediate admission orders include
oxygen by nasal cannula at 4 L/minute,
blood work, a chest radiograph, a 12-
lead electrocardiogram (ECG), and 2 mg
of morphine sulfate given I.V. The nurse
should first:
1. Administer the morphine.
2. Obtain a 12-lead ECG.
3. Obtain the blood work.
4. Order the chest radiograph
2 A 65-year-old client is admitted to the 3. The client is having symptoms of a
. emergency department with a fractured myo-cardial infarction. The fi rst action is
hip. The client has chest pain and to prevent platelet formation and block
shortness of breath. The health care prostaglandin synthe-sis. The
provider orders nitroglycerin tablets. nitroglycerin tablet will be absorbed
Which should the nurse instruct the fastest if the client chews the tablet.
client to do? 1. Put the tablet under the
tongue until it is absorbed.
2. Swallow the tablet with 120 mL of
water.
3. Chew the tablet until it is dissolved.
4. Place the tablet between his cheek
and gums.
3 A 68-year-old female client on day 2 1. Further assessment is needed in this
. after hip surgery has no cardiac history situation. It is premature to initiate other
but reports having chest heaviness. The actions until further data have been
first nursing action should be to: gathered. Inquiring about the onset,
1. Inquire about the onset, duration, duration, location, severity, and
severity, and precipitating factors of the precipitating factors of the chest
heaviness. heaviness will provide pertinent
2. Administer oxygen via nasal cannula. information to convey to the physician.
3. Offer pain medication for the chest
heaviness.
4. Inform the physician of the chest
heaviness.
4 A 69-year-old female has a history of 1. It is a priority to assess blood pressure
. heart failure. She is admitted to the first because people with pulmonary
emergency department with heart edema typically experience severe
failure complicated by pulmonary hypertension that requires early
edema. On admission of this client, intervention. The client probably does not
which of the following should the nurse have skin breakdown on admission;
assess first? however, when the client is stable, the
1. Blood pressure. nurse should inspect the skin. Potassium
2. Skin breakdown. levels are not the first priority. The nurse
3. Serum potassium level. should monitor urine output after the
4. Urine output client is stable.
5 After the administration of t-PA, the 1. Although monitoring the 12-lead ECG
. assessment priority is to: and monitoring breath sounds are
1. Observe the client for chest pain. important, observing the client for chest
2. Monitor for fever. pain is the nursing assessment priority
3. Monitor the 12-lead because closure of the previously
electrocardiogram (ECG) every 4 hours. obstructed coronary artery may recur.
4. Monitor breath sounds. Clients who receive t-PA frequently
receive heparin to prevent closure of the
artery after administration of t-PA. Careful
assessment for signs of bleeding and
monitoring of partial thromboplastin time
are essential to detect complications.
Administration of t-PA should not cause
fever.
6 Alteplase recombinant, or tissue 4. The thrombolytic agent t-PA,
. plasminogen activator (t-PA), a administered intravenously, lyses the clot
thrombolytic enzyme, is administered blocking the coronary artery. The drug is
during the first 6 hours after onset of most effective when adminis-tered within
myocardial infarction (MI) to: the fi rst 6 hours after onset of MI. The
1. Control chest pain. drug does not reduce coronary artery
2. Reduce coronary artery vasospasm. vasospasm; nitrates are used to promote
3. Control the arrhythmias associated vasodilation. Arrhyth-mias are managed
with MI. by antiarrhythmic drugs. Surgical
4. Revascularize the blocked coronary approaches are used to open the
artery. coronary artery and reestablish a blood
supply to the area.
7. As an initial step in treating a client 3. Nitroglycerin produces peripheral
with angina, the physician prescribes vasodi-lation, which reduces myocardial
nitroglycerin tab-lets, 0.3 mg given oxygen consump-tion and demand.
sublingually. This drug's principal Vasodilation in coronary arteries and
effects are produced by: collateral vessels may also increase blood
1. Antispasmodic effects on the fl ow to the ischemic areas of the heart.
pericardium. Nitroglycerin decreases myocardial
2. Causing an increased myocardial oxygen demand. Nitroglycerin does not
oxygen demand. have an effect on pericardial spasticity or
3. Vasodilation of peripheral conductivity in the myocardium.
vasculature.
4. Improved conductivity in the
myocardium.
8. A client has a history of heart failure 2. Early symptoms of digoxin toxicity
and has been taking several include anorexia, nausea, and vomiting.
medications, including furosemide Visual disturbances can also occur,
(Lasix), digoxin (Lanoxin) and including double or blurred vision and
potassium chloride. The client has visual halos. Hypokalemia is a common
nausea, blurred vision, headache, and cause of digoxin toxicity associated with
weakness. The nurse notes that the arrhythmias because low serum
client is confused. The telemetry strip potassium can enhance ectopic
shows first-degree atrioventricular pacemaker activity. Although vomiting
block. The nurse should assess the can lead to fl uid defi cit, given the
client for signs of which condition? client's history, the vomiting is likely due
1. Hyperkalemia. to the adverse effects of digoxin toxic-ity.
2. Digoxin toxicity. Pulmonary edema is manifested by
3. Fluid deficit. dyspnea and coughing.
4. Pulmonary edema.
9. A client has a throbbing headache when 1. Headache is a common side effect of
nitroglycerin is taken for angina. The nitro-glycerin that can be alleviated with
nurse should instruct the client that: aspirin, acetaminophen or ibuprofen. The
1. Acetaminophen (Tylenol) or sublingual nitroglycerin needs to be
Ibuprofen (Advil) can be taken for this absorbed in the mouth, which will be
common side effect. disrupted with drinking. Lying fl at will
2. Nitroglycerin should be avoided if increase blood flow to the head and may
the client is experiencing this serious increase pain and exacerbate other
side effect. symptoms, such as shortness of breath.
3. Taking the nitroglycerin with a few
glasses of water will reduce the
problem.
4. The client should lie in a supine
position to alleviate the headache.
1 A client has chest pain rated at 8 on a 4. Nursing management for a client with
0. 10 point visual analog scale. The 12- a myocardial infarction should focus on
lead electrocardiogram reveals ST pain manage-ment and decreasing
elevation in the inferior leads and myocardial oxygen demand. Fluid status
Troponin levels are elevated. What is should be closely monitored. Client
the highest priority for nursing education should begin once the client is
management of this client at this time? stable and amenable to teaching.
1. Monitor daily weights and urine Visitation should be based on client
output. comfort and maintaining a calm
2. Permit unrestricted visitation by environment.
family and friends.
3. Provide client education on
medications and diet.
4. Reduce pain and myocardial oxygen
demand.
1 A client is admitted with a myocardial 1. An S3 heart sound occurs early in
1. infarction and new onset atrial diastole as the mitral and tricuspid valves
fibrillation. While auscultating the open and blood rushes into the
heart, the nurse notes an irregular ventricles.
heart rate and hears an extra heart
sound at the apex after the S2 that
remains constant throughout the
respiratory cycle. The nurse should
document these findings as:
1. Heart rate irregular with S3.
2. Heart rate irregular with S4.
3. Heart rate irregular with aortic
regurgitation.
4. Heart rate irregular with mitral
stenosis.
1 Clients with heart failure are 2. Characteristics of atrial fi brillation include pulse
2. prone to atrial fibrillation. rate greater than 100 bpm, totally irregular rhythm,
During physical assessment, and no defi nite P waves on the ECG. During
the nurse should suspect atrial assessment, the nurse is likely to note the irregular
fibrillation when palpation of rate and should report it to the physician. A weak,
the radial pulse reveals: thready pulse is characteristic of a client in shock.
1. Two regular beats followed Two regular beats followed by an irregular beat may
by one irregular beat. indicate a premature ventricular contraction.
2. An irregular pulse rhythm.
3. Pulse rate below 60 bpm.
4. A weak, thready pulse.
1 The client who experiences 3. Pasta, tomato sauce, salad, and coffee would be
3. angina has been told to follow the best selection for the client following a low-
a low-cholesterol diet. Which cholesterol diet. Hamburgers, milkshakes, liver, and
of the following meals should fried foods tend to be high in cholesterol.
the nurse tell the client would
be best on her low-cholesterol
diet?
1. Hamburger, salad, and
milkshake.
2. Baked liver, green beans,
and coffee.
3. Spaghetti with tomato
sauce, salad, and coffee.
4. Fried chicken, green beans,
and skim milk
1 A client with acute chest pain 1, 4, 5. Morphine sulfate acts as an analgesic and
4. is receiving I.V. morphine sedative. It also reduces myocardial oxygen con-
sulfate. Which of the following sumption, blood pressure, and heart rate. Morphine
results are intended effects of also reduces anxiety and fear due to its sedative
morphine in this client? Select effects and by slowing the heart rate. It can
all that apply. depress respirations; however, such an effect may
1. Reduces myocardial oxygen lead to hypoxia, which should be avoided in the
consumption. treatment of chest pain. Angiotensin-converting
2. Promotes reduction in enzyme-inhibitor drugs, not morphine, may help to
respiratory rate. prevent ventricular remodeling.
3. Prevents ventricular
remodeling.
4. Reduces blood pressure and
heart rate.
5. Reduces anxiety and fear.
1 A client with angina has been 3. The client taking nifedipine should inspect the
5. taking nifedipine. The nurse gums daily to monitor for gingival hyperplasia. This
should teach the client to: is an uncommon adverse effect but one that
1. Monitor blood pressure requires monitoring and intervention if it occurs.
monthly. The client taking nifedipine might be taught to
2. Perform daily weights. monitor blood pressure, but more often than
3. Inspect gums daily. monthly. These clients would not generally need to
4. Limit intake of green leafy perform daily weights or limit intake of green leafy
vegetables. vegetables.
1 A client with chest pain is 2. Nitroglycerin is a vasodilator that will lower blood
6. prescribed intravenous pressure. The client is having chest pain and the ST
nitroglycerin (Tridil). Which elevation indicates injury to the myocardium, which
assessment is of greatest may benefit from nitroglycerin. The potassium and
concern for the nurse initiating heart rate are within normal range.
the nitro-glycerin drip?
1. Serum potassium is 3.5
mEq/L.
2. Blood pressure is 88/46.
3. ST elevation is present on
the electrocardiogram.
4. Heart rate is 61.
1 A client with chronic heart failure has 3. Coumadin is an anticoagulant, which is
7. atrial fibrillation and a left ventricular used in the treatment of atrial fi brillation
ejection fraction of 15%. The client is and decreased left ventricular ejection
taking warfarin (Coumadin). The fraction (less than 20%) to prevent
expected outcome of this drug is to: thrombus formation and release of emboli
1. Decrease circulatory overload. into the circulation. The client may also
2. Improve the myocardial workload. take other medication as needed to
3. Prevent thrombus formation. manage the heart failure. Coumadin does
4. Regulate cardiac rhythm. not reduce circulatory load or improve
myocardial workload. Coumadin does not
affect cardiac rhythm.
1 A client with heart failure is receiving 2. Digoxin is a cardiac glycoside with posi-
8. digoxin intravenously. The nurse tive inotropic activity. This inotropic
should determine the effectiveness of activity causes increased strength of
the drug by assessing which of the myocardial contractions and thereby
following? increases output of blood from the left
1. Dilated coronary arteries. ventricle. Digoxin does not dilate coronary
2. Increased myocardial contractility. arteries. Although digoxin can be used to
3. Decreased cardiac arrhythmias. 4. treat arrhythmias and does decrease the
Decreased electrical conductivity in electrical conductivity of the myocardium,
the heart. these are not primary reasons for its use in
clients with heart failure and pulmonary
edema.
1 Contraindications to the 2. A history of cerebral hemorrhage is a
9. administration of tissue plasminogen contraindication to administration of t-PA
activator (t-PA) include which of the because the risk of hemorrhage may be
following? further increased. Age greater than 60
1. Age greater than 60 years. years, history of heart failure, and
2. History of cerebral hemorrhage. cigarette smoking are not
3. History of heart failure. contraindications.
4. Cigarette smoking.
2 During the previous few months, a 56- 3. Nitroglycerin may be used prophylacti-
0. year-old woman felt brief twinges of cally before stressful physical activities
chest pain while working in her garden such as stair-climbing to help the client
and has had frequent episodes of remain pain free. Visiting her friend early
indigestion. She comes to the hospital in the day would have no impact on
after experiencing severe anterior decreasing pain episodes. Resting before
chest pain while raking leaves. Her or after an activity is not as likely to help
evaluation confirms a diagnosis of prevent an activity-related pain episode.
stable angina pectoris. After
stabilization and treatment, the client
is discharged from the hospital. At her
follow-up appointment, she is
discouraged because she is
experiencing pain with increasing
frequency. She states that she visits
an invalid friend twice a week and now
cannot walk up the second flight of
steps to the friend's apartment
without pain. Which of the following
measures that the nurse could suggest
would most likely help the client
prevent this problem?
1. Visit her friend early in the day.
2. Rest for at least an hour before
climbing the stairs.
3. Take a nitroglycerin tablet before
climbing the stairs.
4. Lie down once she reaches the
friend's apartment.
2 If a client displays risk factors for 3. A basic principle of behavior
1. coronary artery disease, such as modification is that behavior that is
smoking cigarettes, eating a diet high learned and continued is behavior that has
in saturated fat, or leading a been rewarded. Other reinforcement
sedentary lifestyle, techniques of techniques have not been found to be as
behavior modification may be used to effective as reward.
help the client change the behavior.
The nurse can best reinforce new
adaptive behaviors by:
1. Explaining how the old behavior
leads to poor health.
2. Withholding praise until the new
behavior is well established.
3. Rewarding the client whenever the
acceptable behavior is performed.
4. Instilling mild fear into the client to
extinguish the behavior.
2 In which of the following positions 3. Sitting almost upright in bed with the feet
2. should the nurse place a client and legs resting on the mattress decreases
with suspected heart failure? venous return to the heart, thus reducing
1. Semi-sitting (low Fowler's myocardial work-load. Also, the sitting position
position). allows maximum space for lung expansion. Low
2. Lying on the right side (Sims' Fowler's position would be used if the client
position). could not tolerate high Fowler's position for
3. Sitting almost upright (high some reason. Lying on the right side would not
Fowler's position). be a good position for the client in heart failure.
4. Lying on the back with the head The client in heart failure would not tolerate the
lowered (Trendelenburg's Trendelenburg's position.
position).
2 The major goal of therapy for a 1. Increasing cardiac output is the main goal of
3. client with heart failure and therapy for the client with heart failure or
pulmonary edema should be to: pulmo-nary edema. Pulmonary edema is an
1. Increase cardiac output. acute medical emergency requiring immediate
2. Improve respiratory status. intervention. Respi-ratory status and comfort
3. Decrease peripheral edema. will be improved when cardiac output increases
4. Enhance comfort. to an acceptable level. Peripheral edema is not
typically associated with pulmonary edema.
2 The nurse has completed an 2. A low urine output and confusion are signs of
4. assessment on a client with a decreased tissue perfusion. Orthopnea is a sign
decreased cardiac output. Which of left-sided heart failure. Crackles, edema and
fi nd-ings should receive the weight gain should be monitored closely, but
highest priority? the levels are not as high a priority. With atrial fi
1. BP 110/62, atrial fi brillation brillation there is a loss of atrial kick, but the
with HR 82, bibasilar crackles. blood pressure and heart rate are stable.
2. Confusion, urine output 15 mL
over the last 2 hours, orthopnea.
3. SpO2 92 on 2 liters nasal
cannula, respirations 20, 1+
edema of lower extremities.
4. Weight gain of 1 kg in 3 days,
BP 130/80, mild dyspnea with
exercise.
2 The nurse is admitting a 68-year- 1. The ankle edema suggests fl uid volume
5. old male to the medical floor. The overload. The nurse should assess respiratory
echocardiogram report revealed rate, lung sounds, and SpO2 to identify any
left ventricular enlargement. The signs of respiratory symptoms of heart failure
nurse notes 2+ pitting edema in requiring immediate attention. The nurse can
the ankles when getting the client then draw blood for laboratory studies, insert
into bed. Based on this finding, the Foley catheter, and weigh the client.
what should the nurse do first?
1. Assess respiratory status.
2. Draw blood for laboratory
studies.
3. Insert a Foley catheter.
4. Weigh the client.
2 A nurse is assessing a client with 1, 3, 5. When the heart begins to fail, the body
6. heart failure. The nurse should activates three major compensatory systems:
assess the client based on which ventricular hypertrophy, the renin-angiotensin-
compensatory mechanisms that aldosterone system, and sympathetic nervous
are activated in the presence of stimulation. Parasympathetic stimulation and
heart failure? Select all that jugular venous distention are not compensatory
apply. mechanisms associated with heart failure.
1. Ventricular hypertrophy.
2. Parasympathetic nervous
stimulation.
3. Renin-angiotensin-aldosterone
system.
4. Jugular venous distention.
5. Sympathetic nervous
stimulation
2 The nurse is assessing clients at a 4. The woman who is 65 years old, over-weight
7. health fair. Which client is at and has an elevated LDL is at greatest risk.
greatest risk for coronary artery Total cholesterol > 200, LDL > 100, HDL < 40
disease? in men, HDL < 50 in women, men 45 years and
1. A 32-year-old female with mitral older, women 55 years and older, smoking and
valve pro-lapse who quit smoking obesity increase the risk of CAD. Atorvastatin is
10 years ago. a medica-tion to reduce LDL and decrease risk
2. A 43-year-old male with a family of CAD. The combination of postmenopausal,
history of CAD and cholesterol obesity, and high LDL cholesterol places this
level of 158. client at greatest risk.
3. A 56-year-old male with an HDL
of 60 who takes atorvastatin
(Lipitor).
4. A 65-year-old female who is
obese with an LDL of 188.
2 The nurse is caring for a client 1. Infarction of the papillary muscles is a
8. diagnosed with an anterior potential complication of an MI causing
myocardial infarction 2 days ago. ineffective closure of the mitral valve during
Upon assessment, the nurse systole. Mitral regurgitation results when the
identifies a new systolic murmur left ventricle con-tracts and blood flows
at the apex. The nurse should backward into the left atrium, which is heard at
first: the fifth intercostal space, left midclavicular
1. Assess for changes in vital line. The murmur worsens during expiration
signs. and in the supine or left-side position. Vital sign
2. Draw an arterial blood gas. changes will reflect the severity of the sudden
3. Evaluate heart sounds with the drop in cardiac output: decrease in blood
client leaning forward. pressure, increase in heart rate, and increase in
4. Obtain a 12 Lead respirations. A 12-lead ECG views the electrical
electrocardiogram. activity of the heart; an echocardiogram views
valve function.
2 The nurse is tracking data on a 4. The goals of managing clients outside of the
9. group of clients with heart failure hospital are for the clients to maintain health
who have been discharged from and prevent readmission, thus interventions,
the hospital and are being such as monitoring and teaching appear to
followed at a clinic. Which of the have contributed to the low readmission rate in
following data indicate that this group of clients. Although it is important
nursing interventions of that clients do not gain weight, view
monitoring and teaching have educational material and continue to take their
been effective? medication, the primary indicator of
1. 90 percent of clients have not effectiveness of the program is the lack of re-
gained weight. hospitalization.
2. 75 percent of the clients viewed
the educational DVD.
3. 80 percent of the clients
reported that they are taking their
medications.
4. 5 percent of the clients required
hospitalization in the last 90 days.
3 The nurse notices that a client's 4. The nurse should fi rst assess the client's tol-
0. heart rate decreases from 63 to 50 erance to the drop in heart rate by checking
beats per minute on the monitor. the blood pressure and level of consciousness
The nurse should first: and determine if Atropine is needed. If the
1. Administer Atropine 0.5 mg I.V. client is symptomatic, Atropine and
push. transcutaneous pacing are interven-tions for
2. Auscultate for abnormal heart symptomatic bradycardia. Once the client is
sounds. stable, further physical assessments can be
3. Prepare for transcutaneous done.
pacing.
4. Take the client's blood pressure.
3 The nurse receives 4. Detection of myoglobin is one diagnostic tool to
1. emergency laboratory determine whether myocardial damage has occurred.
results for a client with Myoglobin is generally detected about 1 hour after a
chest pain and immediately heart attack is experienced and peaks within 4 to 6
informs the physician. An hours after infarction. Myoglobin does not help
increased myoglobin level diagnose cancer, hypertension, or liver disease.
suggests which of the
following?
1. Cancer.
2. Hypertension.
3. Liver disease.
4. Myocardial damage.
3 The nurse's discharge 3. Heart failure is a complex and chronic condition.
2. teaching plan for the client Education should focus on health promotion and
with heart failure should preventive care in the home environment. Signs and
stress the importance of symptoms can be monitored by the client. Instructing
which of the following? the client to obtain daily weights at the same time
1. Maintaining a high-fiber each day is very important. The client should be told
diet. to call the physician if there has been a weight gain of
2. Walking 2 miles every 2 lb. or more. This may indicate fluid overload, and
day. treatment can be prescribed early and on an
3. Obtaining daily weights outpatient basis, rather than waiting until the
at the same time each day. symptoms become life-threatening.
4. Remaining sedentary for
most of the day.
3 The nurse should be 4. A low serum potassium level (hypokalemia)
3. especially alert for signs predisposes the client to digoxin toxicity. Because
and symptoms of digoxin potassium inhibits cardiac excit-ability, a low serum
toxicity if serum levels potassium level would mean that the client would be
indicate that the client has prone to increased cardiac excitability. Sodium,
a: glucose, and calcium levels do not affect digoxin or
1. Low sodium level. contribute to digoxin toxicity.
2. High glucose level.
3. High calcium level.
4. Low potassium level.
3 The nurse should teach the 3. Colored vision and seeing yellow spots are
4. client that signs of digoxin symptoms of digoxin toxicity. Abdominal pain,
toxicity include which of the anorexia, nausea, and vomiting are other common
following? symptoms of digoxin toxicity. Additional signs of
1. Rash over the chest and toxicity include arrhythmias, such as atrial fi brilla-tion
back. or bradycardia. Rash, increased appetite, and elevated
2. Increased appetite. blood pressure are not associated with digoxin toxicity.
3. Visual disturbances such
as seeing yellow spots
4. Elevated BP.
3 An older, sedentary adult 1. In older adults who are less active and do not
5. may not respond to exercise the heart muscle, atrophy can result. Disuse
emotional or physical stress or deconditioning can lead to abnormal changes in the
as well as a younger myocardium of the older adult. As a result, under
individual because of: sudden emotional or physical stress, the left ventricle
1. Left ventricular atrophy. is less able to respond to the increased demands on
2. Irregular heartbeats. the myocardial muscle. Decreased car-diac output,
3. Peripheral vascular cardiac hypertrophy, and heart failure are examples of
occlusion. the chronic conditions that may develop in response to
4. Pacemaker placement. inactivity, rather than in response to the aging
process.
3 The physician orders continuous 1. I.V. nitroglycerin infusion requires an infusion
6. I.V. nitro-glycerin infusion for the pump for precise control of the medica-tion.
client with myocardial infarction. Blood pressure monitoring would be done with a
Essential nursing actions include continuous system, and more frequently than
which of the following? every 4 hours. Hourly urine outputs are not
1. Obtaining an infusion pump for always required. Obtaining serum potassium
the medication. levels is not associated with nitroglycerin
2. Monitoring blood pressure infusion.
every 4 hours.
3. Monitoring urine output
hourly.
4. Obtaining serum potassium
levels daily.
3 The physician refers the client 2. Cardiac catheterization is done in clients with
7. with unstable angina for a angina primarily to assess the extent and
cardiac catheterization. The severity of the coronary artery blockage. A
nurse explains to the client that decision about medical management,
this procedure is being used in angioplasty, or coronary artery bypass surgery
this specific case to: will be based on the catheterization results.
1. Open and dilate blocked
coronary arteries.
2. Assess the extent of arterial
blockage.
3. Bypass obstructed vessels.
4. Assess the functional
adequacy of the valves and heart
muscle.
3 Sublingual nitroglycerin tablets 3. The correct protocol for nitroglycerin use
8. begin to work within 1 to 2 involves immediate administration, with
minutes. How should the nurse subsequent doses taken at 5-minute intervals as
instruct the client to use the drug needed, for a total dose of three tablets.
when chest pain occurs? Sublingual nitroglycerin appears in the
1. Take one tablet every 2 to 5 bloodstream within 2 to 3 minutes and is
minutes until the pain stops. metabolized within about 10 minutes.
2. Take one tablet and rest for 10
minutes. Call the physician if
pain persists after 10 minutes.
3. Take one tablet, then an
additional tablet every 5 minutes
for a total of three tablets. Call
the physician if pain persists
after three tablets.
4. Take one tablet. If pain
persists after 5 minutes, take
two tablets. If pain still persists 5
minutes later, call the physician.
3 When administering a Thrombolytic drugs are administered within the
9. thrombolytic drug to the client fi rst 6 hours after onset of an MI to lyse clots
experiencing a myocardial and reduce the extent of myocardial damage.
infarction (MI), the nurse
explains that the purpose of the
drug is to:
1. Help keep him well hydrated.
2. Dissolve clots that he may
have.
3. Prevent kidney failure.
4. Treat potential cardiac
arrhythmias
4 When monitoring a client who is 1. Cardiac arrhythmias are commonly observed
0. receiving tissue plasminogen with administration of t-PA. Cardiac arrhythmias
activator (t-PA), the nurse should are associated with reperfusion of the cardiac
have resuscitation equipment tissue. Hypotension is commonly observed with
available because reperfusion of administra-tion of t-PA. Seizures and
the cardiac tissue can result in hypothermia are not gener-ally associated with
which of the following? reperfusion of the cardiac tissue.
1. Cardiac arrhythmias.
2. Hypertension.
3. Seizure.
4. Hypothermia.
4 When teaching a client with heart 1, 2, 4. The client stating that he would call the
1. failure about preventing physician with increasing shortness of breath,
complications and future weight gain over 2 lb in 1 day, and having to
hospitalizations, which problems sleep sitting up, indicates that he has
stated by the client as reasons to understood the teaching because these signs
call the physician would indicate and symptoms suggest worsening of the client's
to the nurse that the client has heart failure. Although the client will most likely
understood the teaching? Select be placed on a sodium-restricted diet, the client
all that apply. would not need to notify the physician if he or
1. Becoming increasingly short of she had consumed a high-sodium breakfast.
breath at rest. Instead the client would need to be alert for
2. Weight gain of 2 lb or more in 1 possible signs and symptoms of worsening
day. heart failure and work to reduce sodium intake
3. High intake of sodium for for the rest of that day and in the future.
breakfast.
4. Having to sleep sitting up in a
reclining chair.
5. Weight loss of 2 lb in 1 day.
4 When teaching the client with 4. An MI interferes with or blocks blood
2. myocardial infarction (MI), the circulation to the heart muscle. Decreased
nurse explains that the pain blood supply to the heart muscle causes
associated with MI is caused by: ischemia, or poor myocardial oxygenation.
1. Left ventricular overload. Diminished oxygenation or lack of oxygen to
2. Impending circulatory collapse. the cardiac muscle results in ischemic pain or
3. Extracellular electrolyte angina.
imbalances.
4. Insufficient oxygen reaching
the heart muscle.
4 Which activity would be 4. Unlicensed personnel are able to measure
3. appropriate to delegate to and record intake and output. The nurse is
unlicensed personnel for a client respon-sible for client teaching, physical
diagnosed with a myocardial assessments, and evaluating the information
infarction who is stable? collected on the client.
1. Evaluate the lung sounds.
2. Help the client identify risk
factors for CAD.
3. Provide teaching on a 2 g
sodium diet.
4. Record the intake and output.
4 Which of the following is an . By day 2 of hospitalization after an MI, cli-ents
4. expected out-come for a client on are expected to be able to perform personal
the second day of hospitalization care without chest pain. Severe chest pain
after a myocardial infarction (MI)? should not be present on day 2 after and MI.
The client: Day 2 of hospitaliza-tion may be too soon for
1. Has severe chest pain. clients to be able to identify risk factors for MI
2. Can identify risk factors for MI. or to begin a walking program; however, the
3. Agrees to participate in a client may be sitting up in a chair as part of the
cardiac rehabilitation walking cardiac rehabilitation program.
program.
4. Can perform personal self-care
activities with-out pain.
4 Which of the following is not a 2. Late onset of puberty is not generally con-
5. risk factor for the development of sidered to be a risk factor for the development
atherosclerosis? of ath-erosclerosis. Risk factors for
1. Family history of early heart atherosclerosis include family history of
attack. atherosclerosis, cigarette smoking,
2. Late onset of puberty. hypertension, high blood cholesterol level, male
3. Total blood cholesterol level gen-der, diabetes mellitus, obesity, and
greater than 220 mg/dL. physical inactivity.
4. Elevated fasting blood glucose
concentration.
4 Which of the following 1, 2, 3, 5. A decrease in cardiac output occurs from a
6. nursing diagnoses decreased stroke volume with impaired contractility in
would be appropriate systolic heart failure. This impairs peripheral and renal
for a client with systolic perfusion. The impaired perfusion and impaired
heart failure? Select all oxygenation cause the symptoms of activity intolerance.
that apply. The decreased systolic function causes an increase in
1. Ineffective peripheral residual volume and pressure in the left ventricle. A
tissue perfusion related retrograde buildup of pressure from the left ventricle to left
to a decreased stroke atria increases hydrostatic pressure in the pulmonary
volume. vasculature. This causes a leakage of fluid into the
2. Activity intolerance interstitial tissue of the lungs resulting in pulmonary
related to impaired gas symptoms. With diastolic heart failure, there is impaired
exchange and ventricular filling due to a rigid ventricle and reduced
perfusion. ventricular relaxation.
3. Dyspnea related to
pulmonary congestion
and impaired gas
exchange.
4. Decreased cardiac
output related to
impaired cardiac filling.
5. Impaired renal
perfusion related to a
decreased cardiac
output.
4 Which of the following 2. Recommended dietary principles in the acute phase of
7. reflects the principle on MI include avoiding large meals because small, easily
which a client's diet will digested foods are better tolerated.
most likely be based
during the acute phase
of myocardial
infarction?
1. Liquids as desired.
2. Small, easily digested
meals.
3. Three regular meals
per day.
4. Nothing by mouth.

Question 46 See full question

The nurse is caring for a client in labor. The client wishes to have a nonmedicated labor and birth. During the
early stages of labor, the client becomes frustrated with the use of music and imagery. Which of the following would
the nurse include in the clients plan of care? Select all that apply.

You Selected:

Offer the use of a yoga ball

Correct response:

Encourage ambulation
Suggest a shower or bath
Offer the use of a yoga ball

Explanation: Question 1 See full question

A nurse is reviewing a pregnant client's nutritional status. To determine


whether she has an adequate intake of vitamin A, the nurse should assess
her diet for consumption of:
You Selected:

milk.
Correct response:

milk.
Explanation:

Question 2 See full question

A nurse is teaching a client with multiple sclerosis (MS). When teaching the
client how to reduce fatigue, the nurse should tell the client to:
You Selected:

rest in an air-conditioned room.


Correct response:

rest in an air-conditioned room.


Explanation:

Question 3 See full question

Which is an expected outcome for a client with Parkinson's disease who has
had a pallidotomy?
You Selected:

improved functional ability


Correct response:

improved functional ability


Explanation:

Question 4 See full question

The nurse is caring for a postoperative client who has not voided since
before surgery. Which is the nurse's most appropriate action?
You Selected:

Palpate for the bladder above the symphysis pubis


Correct response:

Palpate for the bladder above the symphysis pubis


Explanation:

Question 5 See full question

The nurse is caring for a client in labor. The client wishes to have a
nonmedicated labor and birth. During the early stages of labor, the client
becomes frustrated with the use of music and imagery. Which of the
following would the nurse include in the clients plan of care? Select all that
apply.
You Selected:

Offer the use of a yoga ball


Encourage ambulation
Suggest a shower or bath
Correct response:

Encourage ambulation
Suggest a shower or bath
Offer the use of a yoga ball
Explanation:

Give acetaminophen (Tylenol).


A 2-month-old Explanation: Acetaminophen should be given first
infant arrives to decrease the infant's temperature. A heart rate
in the of 180 beats/minute is normal in an infant with a
emergency fever. Fluid intake is encouraged after the
department acetaminophen is given to help replace insensible
with a heart fluid losses. Carotid massage is an attempt to
rate of 180 decrease the heart rate as a vagal maneuver; it
beats/minute won't work in this infant because the source of
and a the increased heart rate is fever. A tepid sponge
temperature bath may be given to help decrease the
of 103.1 F temperature and calm the infant.
(39.5 C)
rectally.
Which
intervention
is most
appropriate?
2. A 2-year-old child is Place the toddler in
admitted to the respiratory isolation
pediatric unit with Explanation: Nurses
the diagnosis of should take necessary
bacterial precautions to protect
meningitis. Which themselves and others
diagnostic measure from possible infection
would be from the bacterial
appropriate for the organism causing
nurse to perform meningitis. The affected
first? child should immediately
be placed in respiratory
isolation; then the
parents can be informed
about the treatment
plan. This should be
done before laboratory
tests are performed.
3. A 2-year-old child is Intercostal retractions
brought to the Explanation: Clinical
emergency manifestations of
department with respiratory distress
suspected croup. include tachypnea,
Which data tachycardia,
collection finding restlessness, dyspnea,
reflects increasing and intercostal
respiratory retractions. Fever is a
distress? sign of infection.
Bradycardia is a late
sign of impending
respiratory arrest.
Cyanosis, not pallor, is a
sign of increasing
respiratory distress.
4. A 3-year-old client is Perform chest
admitted to the physiotherapy as
pediatric unit with ordered.
pneumonia. He has Encourage coughing
a productive cough and deep breathing.
and appears to have Perform postural
difficulty breathing. drainage.
The parents tell the Maintain
nurse that the humidification with a
toddler hasn't been cool mist humidifier.
eating or drinking Explanation: Chest
much and has been physiotherapy and
very inactive. Which postural drainage work
interventions to together to break up
improve airway congestion and then
clearance should be drain secretions.
included by the Coughing and deep
nurse in the care breathing are also
plan? Select all that effective to remove
apply: congestion. A cool mist
humidifier helps loosen
thick mucous and relax
airway passages. Fluids
should be encouraged,
not restricted. The child
should be placed in
semi-Fowler's to high
Fowler's position to
facilitate breathing and
promote optimal lung
expansion.
5. A 4-year-old child It's used to relieve pain
had a subungual and reduce the risk of
hemorrhage of the infection
toe after a jar fell Explanation: The
on his foot. hematoma is treated
Electrocautery is with electrocautery to
performed. Which relieve pain and reduce
teaching statement the risk of infection.
regarding the Electrocautery doesn't
rationale for using prevent the loss of the
electrocautery to nail. The discoloration
treat the injury is seen with subungual
most accurate? hemorrhage is from the
collection of blood under
the nail bed. It isn't
permanent and doesn't
affect nail growth.
6. An 8-year-old client Fever, muscle weakness,
has tested positive and change in mental
for West Nile virus status
infection. The nurse Correct
suspects the client Explanation: Severe
has the severe form West Nile virus infection
of the disease when (also called West Nile
she recognizes encephalitis or West Nile
which signs and meningitis) affects the
symptoms? central nervous system
and may cause
headache, neck
stiffness, fever, muscle
weakness or paralysis,
changes in mental
status, and seizures.
Such signs and
symptoms as fever,
rash, malaise,anorexia,
nausea and vomiting,
and lymphadenopathy
suggest the mild form of
West Nile virus infection.
7. A 10-year-old child Diabetes insipidus
has been Explanation: Polydipsia
experiencing and polyuria with normal
insatiable thirst and serum glucose are
urinating indicative of diabetes
excessively; his insipidus. Interview and
serum glucose is laboratory results can
normal. Which determine whether the
condition is the origin is neurogenic or
child probably nephrogenic. Type 1 or 2
experiencing? diabetes mellitus
present with an elevated
serum glucose. A child
with hyperthyroidism
may present as
dehydrated from the
excessive sweating and
rapid respirations that
accompany this
hypermetabolic state.
8. A 12-year-old child diagnosed with Respiratory infection
muscular dystrophy is hospitalized Explanation: Respiratory infection can be fatal
secondary to a fall. Surgery is for children with muscular dystrophy due to
necessary as well as skeletal poor chest expansion and decreased ability to
traction. Which complication would mobilize secretions. Skin integrity, infection of
be of greatest concern to the pin sites, and nonunion healing are all causes
nursing staff? for concern, but not as important as
prevention of respiratory infection.
9. A 13-month-old is admitted to the Give clear liquids in small amounts
pediatric unit with a diagnosis of Encourage the child to eat nonsalty soups
gastroenteritis. The toddler has and broths
experienced vomiting and diarrhea Monitor the I.V. solution per the physician's
for the past 3 days, and laboratory order
tests reveal that he's dehydrated. Explanation: A child experiencing nausea and
Which nursing interventions are vomiting won't be able to tolerate a regular
correct to prevent further diet. He should be given sips of clear liquids,
dehydration? Select all that apply: and the diet should be advanced as tolerated.
Unsalted soups and broths are appropriate
clear liquids. I.V. fluids should be monitored to
maintain the fluid status and help to
rehydrate the child. Milk shouldn't be given
because it can worsen the child's diarrhea.
Solid foods may be withheld throughout the
acute phase, but clear fluids should be
encouraged in small amounts (3 to 4
tablespoons every half hour).
1 A 14-year-old female client in Identity
0. skeletal traction for treatment of a Explanation: According to Erikson's theory of
fractured femur is expected to be personal development, the adolescent is in
hospitalized for several weeks. the stage of identity versus role confusion.
When planning care, the nurse During this stage, the body is changing as
should take into account the secondary sex characteristics emerge. The
client's need to achieve what adolescent is trying to develop a sense of
developmental milestone? identity, and peer groups take on more
importance. When an adolescent is
hospitalized, she is separated from her peer
group and body image may be altered.
Toddlers are in the developmental stage of
autonomy versus shame and doubt. Preschool
children are in the stage of initiative versus
guilt. School-age children are in the stage of
industry versus inferiority.
1 A 15-year-old client who sustained Encouraging visitation by his friends
1. a spinal cord injury is on bedrest. Explanation: Encouraging visitation by friends
Which intervention by the nurse might best help the adolescent cope with
might best help the adolescent prolonged bedrest. Friends are much more
cope with the prolonged bedrest? important than family to this age-group.
Providing reading material and video games
might be somewhat helpful, but not as helpful
as encouraging visits from friends.
1 An 18-month-old male child is A protuberant abdomen
2. admitted to the pediatric unit with Explanation: A child with celiac disease has a
a diagnosis of celiac disease. What protuberant abdomen, diarrhea, steatorrhea,
finding would the nurse expect in and anorexia, which result in malnutrition. A
this child? concave abdomen, bulges in the groin area,
and a palpable abdominal mass aren't
associated with celiac disease.
1 A 30-year-old client is admitted to Necrotic tissue through most of the dermis
3. the emergency department with a Explanation: A deep partial-thickness burn
deep partial-thickness burn on his causes necrosis of the epidermal and dermal
arm after a fire in his workplace. layers. Redness and pain are characteristics of
Which signs and symptoms should a superficial injury. Superficial burns cause
the nurse expect to see? slight epidermal damage. With deep burns,
the nerve fibers are destroyed and the client
doesn't feel pain in the affected area. Necrosis
through all skin layers is seen with full-
thickness injuries.
1 An adolescent, age 16, is brought "Do you like yourself physically?"
4. to the clinic for evaluation for a Explanation: Role and relationship patterns
suspected eating disorder. To best focus on body image and the client's
evaluate the effects of role and relationship with others, which commonly
relationship patterns on the child's interrelate with food intake. Questions about
nutritional intake, the nurse should activities and food preferences elicit
ask: information about health promotion and
health protection behaviors. Questions about
food allergies elicit information about health
and illness patterns.
1 An adolescent is started on valproic "A common adverse effect is weight gain."
5. acid to treat seizures. Which Explanation: Weight gain is a common
statement should be included when adverse effect of valproic acid. Drowsiness
educating the adolescent? and irritability are adverse effects more
commonly associated with phenobarbital.
Felbamate (Felbatol) more commonly causes
insomnia.
1 At a previous visit, the parents of an Fatty stools
6. infant with cystic fibrosis received Explanation: Pancreatic enzymes normally
instruction in the administration of aid in food digestion in the intestine. In a
pancrelipase (Pancrease). At a follow- child with cystic fibrosis, however, these
up visit, which finding suggests the natural enzymes can't reach the intestine
need for further teaching? because mucus blocks the pancreatic duct.
Without these enzymes, undigested fats
and proteins produce fatty stools.
Treatment with pancreatic enzymes should
result in stools of normal consistency;
noncompliance with the treatment
produces fatty stools. Noncompliance
doesn't cause bloody urine, bloody stools,
or glucose in urine.
1 Bence Jones protein Immunoglobulin (protein) fragment found
7. in the urine of patients with multiple
myeloma
Presence of Bence Jones protein in the
urine almost always confirms multiple
myeloma, but absence doesn't rule it out.
1 The cardinal signs of diabetes "My infant's fluid intake will be restricted."
8. insipidus are polyuria and polydipsia. Explanation: The simplest test used to
Hypernatremia, not hyponatremia, diagnose diabetes insipidus is restriction of
occurs with diabetes insipidus. oral fluids and observation of consequent
Jaundice occurs because of abnormal changes in urine volume and
bilirubin metabolism, not diabetes concentration. A weight loss of 3% to 5%
insipidus. Hyperchloremia, not indicates severe dehydration, and the test
hypochloremia, occurs with diabetes should be terminated at this point. This
insipidus. test is done in the hospital, and the infant
is watched closely.
1 The charge nurse overhears a nurse "It's important for the child to have
9. complaining that she has been someone assigned to him who's familiar
assigned to a toddler diagnosed with with his care."
tetralogy of Fallot for the past 3 days Explanation: A toddler with tetralogy of
and the mother is very demanding. Fallot requires the care of someone who's
Which response by the charge nurse is familiar with the toddler's condition.
best? Providing continuity of care enhances
safety and promotes well- being for this
toddler and his parents. Options 2 and 3
are condescending to the nurse and don't
help enhance the nurse's understanding of
the situation. Both responses would further
increase the nurse's anger. Changing the
assignment isn't in the best interest of the
toddler.
2 A child, age 14, is hospitalized for Providing small, frequent meals
0. nutritional management and drug Explanation: Clients with ulcerative colitis,
therapy after experiencing an acute also known as inflammatory bowel
episode of ulcerative colitis. Which syndrome, tolerate small, frequent meals
nursing intervention would be better than a few large meals daily. Eating
appropriate? large amounts of food may exacerbate the
abdominal distention, cramps, and nausea
typically caused by ulcerative colitis.
Frequent meals also provide the additional
calories needed to restore nutritional
balance. This client doesn't lack digestive
enzymes and therefore doesn't need
enzyme supplementation. Antibiotics are
contraindicated because they may
interfere with the actions of other
prescribed drugs and because ulcerative
colitis isn't caused by bacteria. High-fiber
foods may irritate the bowel further.
2 A client complains of abdominal Decreasing the rate of feedings and the
1. discomfort and nausea while receiving concentration of the formula
tube feedings. Which intervention is Explanation: Complaints of abdominal
most appropriate for this problem? discomfort and nausea are common in
clients receiving tube feedings. Decreasing
the rate of the feeding and the
concentration of the formula should
decrease the client's discomfort. Feedings
are normally given at room temperature to
minimize abdominal cramping, so this
intervention should have already been
performed. To prevent aspiration during
feeding, the head of the client's bed should
be elevated at least 30 degrees. Changing
tube feeding administration sets every 24
hours prevents bacterial growth; it doesn't
decrease the client's discomfort.
2 A client diagnosed with acute Closely observe the client's skin for
2. myelocytic leukemia (AML) has been petechiae and bruising.
receiving chemotherapy. During the Explanation: The nurse should closely
last two cycles of chemotherapy, the observe the client's skin for petechiae and
client developed severe bruising, early signs of thrombocytopenia.
thrombocytopenia requiring multiple Daily platelet counts may not reflect the
platelet transfusions. The client is client's condition as quickly as subtle
now scheduled to receive a third changes in the client's skin. Performing
cycle. How can the nurse best detect frequent cardiovascular assessments and
early signs and symptoms of checking the client's history won't help
thrombocytopenia? detect early signs and symptoms of
thrombocytopenia.
2 A client is examined and Petechiae
3. found to have pinpoint, Explanation: Petechiae are small macular lesions 1 to 3
pink-to-purple, mm in diameter. Ecchymosis is a purple-to-brown bruise,
nonblanching macular macular or papular, that varies in size. A hematoma is a
lesions 1 to 3 mm in collection of blood from ruptured blood vessels that's
diameter. Which term best more than 1 cm in diameter. Purpura are purple macular
describes these lesions? lesions larger than 1 cm.
2 A client understands what "Before I go home, I'll speak to the home health care
4. resources are available to nurse to make sure I have the supplies I need."
help him perform wound Explanation: The client's acknowledgement that he will
care at home when he need to speak to the home care nurse about supplies
states the following: demonstrates that he is able to perform self-care, and is
familiar with the resources available. The social worker
can help with financial issues, not wound care issues.
The home health care nurse is available for consultation,
but she won't provide all of the client's health care
needs. Dressing changes don't need to be performed in
the physician's office.
2 A client who was bitten by Contact isolation
5. a wild animal is admitted Explanation: A client with rabies requires contact
to an acute care facility isolation because the disease is highly transmissible
for treatment of rabies. through close or direct contact. Rabies isn't transmitted
Which type of isolation through the air, eliminating the need for strict isolation,
does this client require? which aims to prevent transmission of highly contagious
or virulent infections spread by both air and contact.
Respiratory isolation, used to prevent transmission only
through the air, isn't sufficient for a client with rabies.
Enteric isolation is inappropriate because rabies isn't
transmitted through direct or indirect contact with feces.
2 A client with a solar burn pain management.
6. of the chest, back, face, Explanation: With a superficial partial thickness burn
and arms is seen in urgent such as a solar burn (sunburn), the nurse's main
care. The nurse's primary concern is pain management. Fluid resuscitation and
concern should be: infection become concerns if the burn extends to the
dermal and subcutaneous skin layers. Body image
disturbance is a concern that has lower priority than
pain management.
2 A client with cancer is Liver
7. being evaluated for Explanation: The liver is one of the five most common
possible metastasis. cancer metastasis sites. The others are the lymph
Which of the following is a nodes, lung, bone, and brain. The colon, reproductive
common metastasis site tract, and WBCs are occasional metastasis sites.
for cancer cells?
2 A client with cholelithiasis yellow sclerae.
8. has a gallstone lodged in Explanation: Yellow sclerae may be the first sign of
the common bile duct. jaundice, which occurs when the common bile duct is
When assessing this obstructed. Urine normally is light amber. Circumoral
client, the nurse expects pallor and black, tarry stools are signs of hypoxia and GI
to note: bleeding, respectively.
2 Dowager's hump Abnormal curvature in the upper thoracic spine.
9.

3 During a senior citizen Kyphosis


0. health screening, the Explanation: Kyphosis refers to an increased thoracic
nurse observes a 75-year- curvature of the spine, or "humpback." Lordosis is an
old female with a severely increase in the lumbar curve or swayback. Scoliosis is a
increased thoracic curve, lateral deformity of the spine. Genus varum is a bow-
or "humpback". What is legged appearance of the legs.
this condition called?
3 During the first few days ostomy care
1. of recovery from ostomy
surgery for ulcerative
colitis, which aspect
should be the first priority
of client care?
3 enteric precautions Gowns and gloves required, masks not required,
2. protection from feces and urine.
3 For a child with a Wilms' tumor, Avoiding abdominal palpation
3. which preoperative nursing Explanation: Because manipulating the abdominal
intervention takes highest mass may disseminate cancer cells to adjacent
priority? and distant sites, the most important intervention
for a child with a Wilms' tumor is to avoid
palpating the abdomen. Restricting oral intake
and monitoring acid-base balance are routine
interventions for all preoperative clients; they
have no higher priority in one with a Wilms'
tumor. Isolation isn't required because a Wilms'
tumor isn't infectious.
3 For a client with cirrhosis, difficulty in arousal.
4. deterioration of hepatic function Explanation: Hepatic encephalopathy, a major
is best indicated by: complication of advanced cirrhosis, occurs when
the liver no longer can convert ammonia (a by-
product of protein breakdown) into glutamine.
This leads to an increased blood level of ammonia
a central nervous system toxin which causes
a decrease in the level of consciousness. Fatigue,
muscle weakness, nausea, anorexia, and weight
gain occur during the early stages of cirrhosis.
3 For an infant who's about to an arched, side-lying position, avoiding flexion of
5. undergo a lumbar puncture, the the neck onto the chest.
nurse should place the infant in: Explanation: For a lumbar puncture, the nurse
should place the infant in an arched, side- lying
position to maximize the space between the third
and fifth lumbar vertebrae. The nurse's hands
should rest on the back of the infant's shoulders
to prevent neck flexion, which could block the
airway and cause respiratory arrest. The infant
should be placed at the edge of the bed or table
during the procedure, and the nurse should speak
quietly to calm the child. A mummy restraint
would limit access to the lumbar area because it
involves wrapping the child's trunk and
extremities snugly in a blanket or towel. A prone
position wouldn't cause separation of the
vertebral spaces.
3 herpes zoster infection aka shingles-- a viral disease of spinal ganglia-- is
6. a dermatomally distributed skin lesion. Virus
invades a spinal ganglion and is transported
along the axon to the skin, where it produces an
infection that causes a sharp burning pain in the
dermatome supplied by the involved nerve. A few
days later, the skin of the dermatome becomes
red and vesicular eruptions appear.
3 An infant, age 3 months, Bulb syringe with tubing
7. undergoes surgical repair of a Explanation: An infant with a surgically repaired
cleft lip. After surgery, the cleft lip must be fed with a bulb syringe with
nurse should use which tubing or Breck feeder to prevent sucking or
equipment to feed the infant? suture line trauma. The other options wouldn't
prevent these actions.
3 An infant, age 8 months, has a Sitting in an infant seat
8. tentative diagnosis of Explanation: Because the infant's data collection
congenital heart disease. During findings suggest that respiratory distress is
data collection, the nurse developing, the nurse should position the infant
measures a heart rate of 170 with the head elevated at a 45-degree angle to
beats/minute and a respiratory promote maximum chest expansion; an infant
rate of 70 breaths/minute. How seat maintains this position. Placing an infant flat
should the nurse position the on his back or abdomen or in high Fowler's
infant? position could increase respiratory distress by
preventing maximal chest expansion.
3 In the client with burns on the Applying knee splints
9. legs, which nursing intervention Explanation: Applying knee splints prevents leg
helps prevent contractures? contractures by holding the joints in a position of
function. Elevating the foot of the bed can't
prevent contractures because this action doesn't
hold the joints in a position of function.
Hyperextending a body part for an extended time
is inappropriate because it can cause
contractures. Performing shoulder range-of-
motion exercises can prevent contractures in the
shoulders, but not in the legs.
4 Intraosseously fluid admin - good if cannot obtain IV access
0. (small animals). Also can be given at a relatively
fast speed. acceptable for dehydration and shock
4 mantoux test Intradermal test to determine tuberculin
1. sensitivity based on a positive reaction where the
area around the test site becomes red and
swollen
4 A mother and grandmother bring a "We've found that babies can't digest solid
2. 3-month-old infant to the well-baby food properly until they're 4 months old."
clinic for a routine checkup. As the Explanation: Infants younger than 4 months
nurse weighs the infant, the lack the enzymes needed to digest complex
grandmother asks, "Shouldn't the carbohydrates. Option 1 doesn't address the
baby start eating solid food? My grandmother's question directly. Option 2 is a
kids started on cereal when they clich that may block further communication
were 2 weeks old." Which response with the grandmother. Option 4 is incorrect
by the nurse would be appropriate? because no evidence suggests that
introducing solid food early causes eating
disorders.
4 The mother of a 3-year-old with a Folic acid to 4 mg/day
3. myelomeningocele is thinking about Explanation: The American Academy of
having another baby. The nurse Pediatrics recommends that a woman who
should inform the woman that she has had a child with a neural tube defect
should increase her intake of which increase her intake of folic acid to 4 mg per
acid? day 1 month before becoming pregnant and
continue this regimen through the first
trimester. A woman who has no family history
of neural tube defects should take 0.4
mg/day. All women of childbearing age should
be encouraged to take a folic acid supplement
because the majority of pregnancies in the
United States are unplanned. Ascorbic acid
hasn't been shown to have any effect on
preventing neural tube defects.
4 The mother of a preschooler Measure the child's blood glucose level.
4. recently diagnosed with type 1 Explanation: In a child with type 1 diabetes
diabetes mellitus makes an urgent mellitus, behavioral changes may signal
call to the pediatrician's office. She either hypoglycemia or hyperglycemia;
says her child had an uncontrollable measuring the blood glucose level is the only
temper tantrum while playing and way to determine which condition is present.
now is lethargic and hard to arouse. Urine glucose measurement doesn't
The nurse should instruct the accurately reflect the current blood glucose
mother to take which action first? level. Forcing a lethargic child to drink fluids
could cause aspiration. After measuring the
child's blood glucose level, the mother may
need to take additional emergency measures
such as administering insulin or a simple
glucose source. If the child doesn't respond to
these measures, the mother may need to call
for emergency help.
4 A mother reports that her 6-year- Assess the mother's understanding of UTI
5. old girl recently started wetting the and its causes
bed and running a low-grade fever. Instruct the mother to administer the
A urinalysis is positive for bacteria antibiotic as prescribedeven if the
and protein. A diagnosis of a symptoms diminish
urinary tract infection (UTI) is Discourage taking bubble baths
made, and the child is prescribed Explanation: Assessing the mother's
antibiotics. Which interven-tions understanding of UTI and its causes provides
are appropriate? Select all that the nurse with a baseline for teaching. The
apply: full course of antibiotics must be given to
eradicate the organism and prevent
recurrence, even if the child's signs and
symptoms de-crease. Bubble baths can
irritate the vulva and urethra and contribute
to the development of a UTI. Fluids should be
encouraged, not limited, in order to prevent
urinary stasis and help flush the organism out
of the urinary tract. Instructions should be
given to the child at her level of
understanding to help her better understand
the treatment and promote compliance. The
child should wipe from the front to the back,
not back to front, to minimize the risk of
contamination after elimination.
4 A neonate born 18 hours ago with Preventing infection
6. myelomeningocele over the Explanation: Preventing infection is the
lumbosacral region is scheduled for nurse's primary preoperative goal for a
corrective surgery. Preoperatively, neonate with myelomeningocele. Although
what is the most important nursing the other options are relevant for this
goal? neonate, they're secondary to preventing
infection.
4 The nurse caring for a client with Administering I.V. fluids
7. small-bowel obstruction would plan Explanation: I.V. infusions containing normal
to implement which nursing saline solution and potassium should be given
intervention first? first to maintain fluid and electrolyte balance.
For the client's comfort and to assist in bowel
decompression, the nurse should prepare to
insert an NG tube next. A blood sample is
then obtained for laboratory studies to aid in
the diagnosis of bowel obstruction and guide
treatment. Blood studies usually include a
complete blood count, serum electrolyte
levels, and blood urea nitrogen level. Pain
medication often is withheld until obstruction
is diagnosed because analgesics can
decrease intestinal motility.
4 The nurse is assessing a client pathologic bone fractures.
8. with multiple myeloma. The Explanation: Clients with multiple myeloma are at
nurse should keep in mind that risk for pathologic bone fractures secondary to
clients with multiple myeloma diffuse osteoporosis and osteolytic lesions. Also,
are at risk for: clients are at risk for renal failure secondary to
myeloma proteins by causing renal tubular
obstruction. Liver failure and heart failure aren't
usually sequelae of multiple myeloma. Hypoxemia
isn't usually related to multiple myeloma.
4 The nurse is assigned to an Fear related to disturbed body image
9. adolescent. Which nursing Explanation: Fear related to disturbed body image
diagnosis is most appropriate is the most appropriate nursing diagnosis for a
for a hospitalized adolescent? hospitalized adolescent because of the
adolescent's developmental level and concern for
physical appearance. An adolescent may fear
disfigurement resulting from procedures and
treatments. Separation is rarely a major stressor
for the adolescent, eliminating a diagnosis of
Anxiety related to separation from parents.
Adolescents may have Fear related to the
unknown but typically ask questions if they want
information. A diagnosis of Ineffective coping
related to activity restrictions may be appropriate
for a toddler who has difficulty tolerating activity
restrictions but is an unlikely nursing diagnosis for
an adolescent.
5 The nurse is assisting in Blood pressure monitoring
0. developing a teaching plan for a Explanation: Because poststreptococcal
child with acute glomerulonephritis may cause severe, life-
poststreptococcal threatening hypertension, the nurse must teach
glomerulonephritis. What is the the parents how to monitor the child's blood
most important point to address pressure. Infection control, nutritional planning,
in this plan? and prevention of streptococcal pharyngitis are
important but are secondary to blood pressure
monitoring.
5 The nurse is caring for a 10- Consulting with the social worker to help the
1. year-old child with cystic family find appropriate resources
fibrosis. The child's parents tell Explanation: The nurse can help this family by
the nurse that they're having assisting them with finding appropriate financial,
difficulty coping with their psychological, and social support and by
child's disease. Which action providing referrals to the local community
would be most appropriate for agencies and the Cystic Fibrosis Foundation. The
the nurse to take? child should be treated as much like a normal
child as possible, and he should be encouraged to
make friends with other children regardless of
their physical condition. The nurse shouldn't
encourage the parents not to visit because the
child might feel abandoned.
5 The nurse is caring for a 16- no because she isn't sexually active.
2. year-old female client who isn't Explanation: A 16-year-old client who isn't
sexually active. The client asks sexually active doesn't need a Pap test. When a
if she needs a Papanicolaou client is sexually active or reaches age 18, a Pap
(Pap) test. The nurse should test should be performed.
reply:
5 A nurse is caring for a 17-year- Directly into the superior vena cava
3. old girl who's receiving Explanation: Solutions that contain more than
parenteral nutrition in 25% 12.5% dextrose are administered through a
dextrose solution. How should central venous access device directly into the
this solution be administered? superior vena cava by way of the jugular or
subclavian vein. Special tubing is used that
contains an in-line filter to remove bacteria and
particulate material. A superficial vein,
gastrostomy tube, and the oral route are never
used for this type of solution.
5 A nurse is caring for a client black, tarry stools.
4. with suspected upper GI Explanation: As blood from the GI tract passes
bleeding. The nurse should through the intestines, bacterial action causes it
monitor this client for: to become black. Hemoptysis involves coughing
up blood from the lungs. Hematuria is blood in the
urine. Bright red blood in the stools indicates
bleeding from the lower GI tract.
5 The nurse is caring for an Bone fracture
5. elderly female with Explanation: Bone fracture is a major
osteoporosis. When teaching complication of osteoporosis that results when
the client, the nurse should loss of calcium and phosphate increases the
include information about which fragility of bones. Estrogen deficiencies result
major complication of this from menopause not osteoporosis. Calcium and
condition? vitamin D supplements may be used to support
normal bone metabolism, but a negative calcium
balance isn't a complication of osteoporosis.
Dowager's hump results from bone fractures. It
develops when repeated vertebral fractures
increase spinal curvature.
5 The nurse is caring for an infant Undescended testes
6. with hypospadias. Which anomaly Explanation: Because undescended testes
would the nurse assess the infant may also be present in hypospadias, the
for that commonly accompanies this small penis may appear to be an enlarged
condition? clitoris. This shouldn't be mistaken for
ambiguous genitalia. If there's any doubt,
more tests should be performed. Hernias
don't generally accompany hypospadias.
5 The nurse is caring for a teenage cover the opening with sterile petroleum
7. client involved in a motor vehicle gauze.
accident. The client has a chest Explanation: If a chest tube is accidentally
tube in place. If the chest tube is removed, the nurse should cover the insertion
accidentally removed, the nurse site with sterile petroleum gauze. The nurse
should immediately: should then observe the client for respiratory
distress, as tension pneumothorax may
develop. If so, the nurse should remove the
gauze to allow air to escape. The nurse
shouldn't reintroduce the tube. Rather, the
nurse should have another staff member call
a physician so another tube can be
introduced by the physician under sterile
conditions.
5 The nurse is collecting data on Diphtheria, tetanus, and acellular pertussis
8. whether the client has received all (DTaP), MMR, inactivated polio virus (IPV), and
recommended immunizations for his pneumococcal vaccine
age. Which immunizations should Explanation: Between ages 4 and 6 the child
he have received between ages 4 should receive DTaP, MMR, IPV, and Varicella
and 6? vaccine. Hepatitis A is completed by age 2yrs.
MMR alone is incomplete. H. influenzae, type
B immunization is completed by age 15
months.
5 The nurse is deciding whether to Incompatibility between the history and the
9. report a suspected case of child injury
abuse. Which criterion is the most Explanation: Incompatibility between the
important for the nurse to consider? history and the injury is the most important
criterion on which to base the decision to
report suspected child abuse. The other
criteria also may suggest child abuse but are
less reliable indicators.
6 The nurse is helping a client The client's pulse and respiratory rates
0. ambulate for the first time after 3 increased moderately during ambulation.
days of bed rest. Which observation Explanation: The pulse and respiratory rates
by the nurse suggests that the normally increase during and for a short time
client tolerated the activity without after ambulation, especially if it's the first
distress? ambulation after 3 days of bed rest. A normal
walking pace is 70 to 100 steps/minute; a
much slower pace may indicate distress.
Dizziness, weakness, and profuse perspiration
are definite signs of activity intolerance. A
client who tolerates ambulation well holds the
head erect, gazes straight ahead, and keeps
the toes pointed forward; option 3 describes a
client with activity intolerance.
6 The nurse is preparing to Wash her hands and arrange supplies at the
1. administer chloramphenicol bedside.
(Chloromycetin Otic) to a 2-year-old Warm the medication to body temperature.
client with an infection of the Examine the ear canal for drainage.
external auditory canal. The order Explanation: The nurse should prepare to
reads, "2 drops in the right ear instill the eardrops by washing her hands,
three times per day." Which steps gathering the supplies, and arranging the
should the nurse take to administer supplies at the bedside. To avoid adverse
this medication? Select all that effects resulting from eardrops that are too
apply: cold (such as vertigo, nausea, and pain), the
medication should be warmed to body
temperature in a bowl of warm water.
Temperature of the drops should be tested by
placing a drop on the wrist. Before instilling
the drops, the ear canal should be examined
for drainage that may reduce the
medication's effectiveness. Because the dose
is to be given in the right ear, the child should
be placed on his left side with his right ear
facing up. For an infant or a child younger
than age 3, gently pull the auricle down and
back because the ear canal is straighter in
children of this age-group.
6 The nurse is teaching a group of Coordinating a panel of peers who were
2. adolescents about automobile involved in motor vehicle accidents
safety. Which is the most effective Explanation: Coordinating a panel of peers to
teaching method for this age- discuss motor vehicle accidents and their
group? prevention is more effective for this age-
group. Adolescents are more likely to listen to
others their age who have experienced
similar circumstances. Lecturing about the
effects of drugs and alcohol on driving will
most likely be ineffective for this age-group.
Adolescents won't be motivated to read the
written materials. Animated videos aren't
age-appropriate and may minimize the
importance of the material.
6 A nurse is teaching an adolescent "Carry crackers or fruit to eat before or during
3. with type 1 diabetes about the periods of increased activity."
disease. Which instruction by the Explanation: Hypoglycemia can usually be
nurse about how to prevent prevented if an adolescent with diabetes eats
hypoglycemia would be most more food before or during exercise. Because
appropriate for the adolescent? exercise with adolescents isn't commonly
planned, carrying additional carbohydrate
foods such as crackers or fruit is a good
preventive measure.
6 The nurse is teaching parents Make sure all medications are kept in containers
4. about accident prevention for a with childproof safety caps.
toddler. Which of the following Explanation: All over-the-counter and prescription
guidelines is most appropriate? medications should have childproof safety caps.
Poisoning accidents are common in toddlers, due
to the toddler's curiosity and his increasing
mobility and ability to climb. When riding in a car,
a toddler should be strapped into a car seat.
Wearing a seat belt is an appropriate guideline
for a school-age child. Never leaving a child alone
on a bed is an appropriate guideline for parents
of infants. Toddlers already have the ability to
climb on and off of beds and other furniture by
themselves. Note, however, that toddlers should
never be left unattended on high surfaces, such
as an examining table in a physician's office.
Toddlers should be in a rear-facing convertible car
seat; those who weigh 20 to 40 lb can be placed
facing forward.
6 The nurse should include which The virus can be spread through many routes,
5. fact when teaching an including sexual contact
adolescent group about the Explanation: HIV can be spread through many
human immunodeficiency virus routes, including sexual contact and contact with
(HIV)? infected blood or other body fluids. The incidence
of HIV in the adolescent population has increased
since 1995, even though more information about
the virus is targeted to reach the adolescent
population. Only about 25% of all new HIV
infections in the United States occurs in people
younger than age 22.
6 A nurse should teach the client Dumping syndrome
6. to watch for which complication Explanation: Dumping syndrome is a problem
of gastric resection? that occurs postprandially after gastric resection
because ingested food rapidly enters the jejunum
without proper mixing and without the normal
duodenal digestive processing. Diarrhea, not
constipation, may also be a symptom. Gastric or
intestinal spasms don't occur, but antispasmodics
may be given to slow gastric emptying.
6 The nurse teaches a mother how The child eats finger foods by himself.
7. to provide adequate nutrition for Explanation: The child with cerebral palsy should
her toddler, who has cerebral be encouraged to be as independent as possible.
palsy. Which of the following Finger foods allow the toddler to feed himself.
observations indicates that Because spasticity affects coordinated chewing
teaching has been effective? and swallowing as well as the ability to bring food
to the mouth, it's difficult for the child with
cerebral palsy to eat neatly. Independence in
eating should take precedence over neatness.
The child with cerebral palsy may require more
time to bring food to the mouth; thus, chewing
and swallowing shouldn't be rushed to finish a
meal by a specified time. The child with cerebral
palsy may vomit after eating due to a
hyperactive gag reflex. Therefore, the child
should remain in an upright position after eating
to prevent aspiration and choking.
6 One day after an appendectomy, The child rates pain at 4 out of 5. Pain medication
8. a 9-year-old client rates his pain administered as prescribed.
at 4 out of 5 on the pain scale Explanation: Pain is what the child says it is, and
but is playing video games and the nurse must document what the child reports.
laughing with his friend. Which If a child's behavior appears to differ from the
of the following would the nurse child's rating of pain, believe the pain rating. A
document on the child's chart? child who uses a passive coping behavior (such
as distraction) may rate pain as more intense
than children who use active coping behavior
(such as crying). Making judgments about pain
based on behavior can result in children being
inadequately medicated for pain.
6 Parents of a 6-year-old child tell Typical absence
9. a physician that the child has Explanation: A typical absence seizure has an
been having periods of onset between ages 4 and 8. It's exhibited by an
unawareness with short periods abrupt loss of consciousness, amnesia, or
of staring. Based on his history, unawareness characterized by staring and a 3-
the child is probably having cycle/second spike and waveform on an EEG. The
which type of seizure? attack lasts from 10 to 30 seconds and may occur
as frequently as 50 to 100 times a day. No
postictal or confused state follows the attack. A
complex partial seizure most commonly occurs in
older children and adults, causing a brief
impairment of consciousness. A myoclonic
seizure occurs in older children and is exhibited
by lightning jerks without loss of consciousness.
An abrupt increase in muscle tone, loss of
consciousness, and marked autonomic signs and
symptoms characterize the tonic seizure.
7 The parents of a school-age child Taking prophylactic drugs before the activity can
0. with asthma express concern prevent asthma attacks and enable the child to
about letting the child engage in most sports.
participate in sports. What Explanation: Although exercise may trigger
should the nurse tell the parents asthma attacks, taking prophylactic asthma
about the relationship between drugs before beginning the activity can prevent
exercise and asthma? attacks, enabling the child to engage in most
sports. Asthma attacks may be triggered by
various factors, including allergens, exercise,
medications, upper respiratory infections, and
psychological stress. Provided the asthma is
under control, most children can participate in
sports and other physical activities; in fact, they
benefit from exercise. Activity restrictions hamper
peer interaction, which is essential to the
development of the school-age child. A child with
asthma may tolerate intermittent activities better
than continuous ones.
7 The physician orders digoxin 0.25
1. (Lanoxin) 0.1 mg orally every Explanation: To convert mg to mcg: 1,000 mcg/1
morning for a 6-month-old infant mg = X mcg/0.1 mg; X = 100 mcg. To calculate
with heart failure. Digoxin is drug dose: Dose on hand/Quantity on hand =
available in a 400 mcg/mL Dose desired/X. 400 mcg/mL = 100 mcg/X; X =
concentration. How many 0.25 mL.
milliliters of digoxin should the
nurse give? Record your answer
using two decimal places.
7 pleural friction rub creaking or grating sound from roughened,
2. inflamed surfaces of the pleura rubbing
together, evident during inspiration, expiration,
or both and no change with coughing; usually
uncomfortable, especially on deep inspiration.
7 A preschool-age child underwent Frequent swallowing
3. a tonsillectomy 4 hours ago. Explanation: Frequent swallowing an attempt
Which data collection finding to clear the throat of trickling blood suggests
would make the nurse suspect postoperative hemorrhage. Emesis may be
postoperative hemorrhage? brown or blood-tinged after a tonsillectomy; only
bright red emesis signals hemorrhage. The child
may refuse fluids because of painful swallowing,
not bleeding. Hemorrhage is associated with an
increased, not decreased, heart rate.
7 A recent abduction of a 2-month- Placing an identification bracelet on the infant
4. old infant has raised awareness and the parent immediately on admission
of the need for security plans for Explanation: The safest way to ensure that the
hospitals. Which security parents or legal guardians are who they say
measure helps ensure the they are is to place a bracelet on both the infant
hospitalized infant's security? and the parents or guardians at the time of
admission. Limiting visitors isn't necessary.
Locking the door and having visitors call the
nurses' station for admission increases the
workload of the nursing staff. It isn't feasible to
place security guards at the entrances.
7 rule of nines method used to calculate the amount of fluid
5. lost as the result of a burn; divides the body into
11 areas, each accounting for 9% of the total
body area
7 A school-age child has a fever, "Has your child had strep throat recently?"
6. joint inflammation, and a Explanation: Group A beta-hemolytic
nonpruritic rash. Knowing that streptococcal infection typically precedes
these are signs of rheumatic rheumatic fever. An inflammatory disease,
fever, the nurse should ask the rheumatic fever affects the heart, joints, and
parents: central nervous system. It isn't infectious and
can't be transmitted from one person to another.
Congenital heart defects don't play a role in the
development of rheumatic fever. H. influenzae
vaccine doesn't prevent streptococcal infection
or rheumatic fever.
7 Several children at a daycare Hand washing after diaper changes
7. center have been infected with Explanation: Children in daycare centers are at
hepatitis A virus. Which risk of hepatitis A infection which is transmitted
instruction by the nurse would via fecal-oral route due to poor hand hygiene
reduce the risk of hepatitis A to practices and poor sanitation. Isolation of sick
other children and staff children, use of masks during contact, and
members? sterilization of all eating utensils would not be
useful in breaking the chain of infection.
7 A teenager is brought to the Respiratory system
8. facility by friends after Explanation: The primary concern with
accidentally ingesting gasoline petroleum distillate ingestion is its effect on the
while siphoning it from a car. respiratory system. Aspiration or absorption of
Based on the nurse's knowledge petroleum distillates can cause severe chemical
of petroleum distillates, which pneumonitis and impaired gas exchange. The
system would be most affected? GI, neurologic, and cardiovascular systems may
be affected if the petroleum contains additives
such as pesticides.
7 A teenager with heart failure a cardiac glycoside
9. prescribed digoxin (Lanoxin) asks Explanation: Digoxin is a cardiac glycoside. It
the nurse, "What's the drug decreases the workload of the heart and
supposed to do?" The nurse improves myocardial function. ACE inhibitors
responds to the teenager based cause vasodilation and increase sodium
on the understanding that this excretion. Diuretics help remove excess fluid.
drug is classified as: Vasodilators enhance cardiac output by
decreasing afterload.
8 A toddler is brought to the Intraosseously
0. emergency department in cardiac Explanation: The physician can safely administer
arrest. The physician tries three emergency medications, such as sodium
times to insert an I.V. catheter bicarbonate, calcium, glucose, crystalloids,
but is unsuccessful. By which colloids, blood, dopamine, epinephrine, and
alternate route can the physician dobutamine by the intraosseous route if the I.V.
administer emergency route is inaccessible. Emergency medications
medications? shouldn't be administered by the sublingual,
topical, or subcutaneous routes.
81. A toddler is brought to the a barium enema.
emergency department with sudden Explanation: A barium enema
onset of abdominal pain, vomiting, commonly is used to confirm and
and stools that look like red currant correct intussusception. Performing a
jelly. To confirm intussusception, the suprapubic aspiration or inserting an
suspected cause of these findings, NG tube or an indwelling urinary
the nurse expects the physician to catheter wouldn't help diagnose or treat
order: this disorder.
82. What's the best way for a nurse to Left side-lying
position a 3-year-old child with right Explanation: The child with right lower
lower lobe pneumonia? lobe pneumonia should be placed on his
left side. This places the unaffected left
lung in a position that allows gravity to
promote blood flow though the healthy
lung tissue and improve gas exchange.
Placing the child on his right side, back,
or stomach doesn't promote circulation
to the unaffected lung.
83. When assisting in developing a plan Preschool age
of care for a hospitalized child, the Explanation: School-age children are
nurse knows that children in which most likely to view illness as a
age-group are most likely to view punishment for misdeeds. Separation
illness as a punishment for misdeeds? anxiety, although seen in all age-
groups, is most common in older
infants. Fear of death is typical of older
children and adolescents.
84. When collecting data on a child with Goiter
juvenile hypothyroidism, the nurse Explanation: Juvenile hypothyroidism
expects which finding? results in goiter, weight gain,
sleepiness, and a slow heart rate. It
doesn't cause weight loss, insomnia, or
tachycardia.
85. When counseling parents of a Duration of the condition before
neonate with congenital treatment
hypothyroidism, the nurse Explanation: The severity of the
understands that the severity of the intellectual deficit is related to the
intellectual deficit is related to which degree of hypothyroidism and the
parameter? duration of the condition before
treatment. Cranial malformations don't
affect the severity of the intellectual
deficit, nor does the degree of
hypothermia as it relates to
hypothyroidism. It isn't the specific T4
level at diagnosis that affects the
intellect but how long the child has
been hospitalized.
86. When evaluating a severely a preoccupation with death.
depressed adolescent, the nurse Explanation: An adolescent who
knows that one indicator of a high demonstrates a preoccupation with
risk for suicide is: death (such as by talking about death
frequently) should be considered at
high risk for suicide. Although
depression, excessive sleepiness, and a
history of cocaine use may occur in
suicidal adolescents, they also occur in
adolescents who aren't suicidal.
87. When preparing to feed an infant Burp the infant frequently
with pyloric stenosis, which Explanation: Infants with pyloric
intervention is important? stenosis usually swallow a lot of air
from sucking on their hands and fingers
because of their intense hunger
(feedings aren't easily tolerated).
Burping often lessens gastric distention
and increases the likelihood the infant
will retain the feeding. Feedings are
given slowly with the infant lying in a
semiupright position. Parental
participation should be encouraged and
allowed to the extent possible. Record
the type, amount, and character of the
vomit as well as its relation to the
feeding. The amount of feeding volume
lost is usually refed.
88. When talking with 10- and 11-year- The children will be curious about the
old children about death, the nurse physical aspects of death
should incorporate which guides? The children will know that death is
Select all that apply: inevitable and irreversible
The children will be influenced by the
attitudes of the adults in their lives
Explanation: School-age children are
curious about the physical aspects of
death and may wonder what happens
to the body. By age 9 or 10, most
children know that death is universal,
inevitable, and irreversible. Their
cognitive abilities are advanced and
they respond well to logical
explanations. They should be
encouraged to ask questions. Because
the adults in their environment
influence their attitudes towards death,
they should be encouraged to include
children in the family rituals and be
prepared to answer questions that may
seem shocking. Teaching about death
should begin early in childhood.
Comparing death to sleep can be
frightening for children and cause them
to fear falling asleep.
89. When the parents of an infant Withhold the medication and call the
diagnosed with hypothyroidism primary health care provider
have been taught to count the Explanation: If parents have been taught
pulse, which intervention should to count the pulse of an infant diagnosed
the nurse teach them in case they with hypothyroidism, they should be
obtain a high pulse rate? instructed to withhold the dose and
consult their primary health care provider
if the pulse rate is above a certain value.
90. Which action should a nurse take Institute isolation precautions according
first when admitting a client with to facility policy.
herpes zoster infection? Explanation: The nurse should first
institute isolation precautions to prevent
the spread of the herpes zoster infection.
After isolation precautions are in effect,
the nurse can instruct the client to wear
light clothing and provide a tepid bath to
promote client comfort. The nurse should
also caution the client against scratching
the lesions because that might cause
infection and scarring.
91. Which adverse effect can be Orange body secretions
expected by the parents of a 2- Explanation: Rifampin and its metabolites
year-old child who has been started will turn urine, feces, sputum, tears, and
on rifampin after testing positive sweat an orange color. This isn't a serious
for tuberculosis? adverse effect. Rifampin may also cause
GI upset, headache, drowsiness,
dizziness, vision disturbances, and fever.
Liver enzyme and bilirubin levels increase
because of hepatic metabolism of the
drug. Parents should be taught the signs
and symptoms of hepatitis and
hyperbilirubinemia such as jaundice of
the sclera or skin.
92. Which diet plan is recommended for Increase caloric content per ounce
an infant with heart failure? Explanation: Formulas with increased
caloric content are given to meet the
greater caloric requirements from the
overworked heart and labored breathing.
Fluid restriction and low-sodium formulas
aren't recommended. An infant's
nutritional needs depend on fluid. Daily
weights at the same time of the day on
the same scale before feedings are
recommended to follow trends in
nutritional stability and diuresis. Low-
sodium formulas may cause
hyponatremia.
93. Which factor will most likely Inefficient liver function
decrease drug metabolism during Explanation: Inefficient liver function will
infancy? most likely decrease drug metabolism
during infancy. As the liver matures
during the 1st year of life, drug
metabolism improves. Decreased
glomerular filtration and increased tubular
secretion may affect drug excretion rather
than metabolism; reduced protein-binding
ability may affect drug distribution but
not metabolism.
94. Which finding is an early indicator Painless, intermittent hematuria
of bladder cancer? Explanation: As cancer cells destroy
normal bladder tissue, bleeding occurs
and causes painless, intermittent
hematuria. (Pain is a late symptom of
bladder cancer.) The other options aren't
associated with bladder cancer.
Occasional polyuria may occur with
diabetes or increased alcohol or caffeine
intake. Nocturia commonly accompanies
benign prostatic hypertrophy. Dysuria
may indicate a urinary tract infection
(UTI).
95. Which method is most reliable for Check the hospital identification bracelet.
confirming a preschooler's identity Explanation: The only safe method for
before administering a medication? identifying the child is to check the
identification band for the client's name
and medical record number and then
compare that information with the
medication record. Children sometimes
exchange beds during play, so checking
the name on the bed isn't reliable. Infants
are unable to give their names, toddlers
or preschoolers may admit to any name,
and school-age children may deny their
identities in an attempt to avoid the
medication. Parents aren't always at the
bedside, so they shouldn't be relied on for
identification
96. Which nursing diagnosis is most Risk for infection
relevant in the first 12 hours of life Explanation: All of these nursing
for a neonate born with a diagnoses are important for a child with a
myelomeningocele? myelomeningocele. However, during the
first 12 hours of life, the most life-
threatening event would be an infection.
The other diagnoses will be addressed as
the child develops
97. Which nursing objective is most Early identification
important when working with Explanation: The most important nursing
neonates who are suspected of objective is early identification of the
having congenital hypothyroidism? disorder. Nurses caring for neonates must
be certain that screening is performed,
especially in neonates who are preterm,
discharged early, or born at home.
Promoting bonding, allowing rooming in,
and encouraging fluid intake are all
important but are less important than
early identification.

Question 1 See full question

A nurse is assigned to care for a client with anorexia nervosa. During the first
48 hours of treatment, which nursing intervention is most appropriate for this
client?
You Selected:

Providing one-on-one supervision during meals and for 1 hour afterward


Correct response:

Providing one-on-one supervision during meals and for 1 hour afterward


Explanation:

Question 2 See full question

Which statement would provide the best guide for activity during the
rehabilitation period for a client who has been treated for retinal
detachment?
You Selected:

Activity is resumed gradually; the client can resume usual activities in 5 to 6


weeks.
Correct response:

Activity is resumed gradually; the client can resume usual activities in 5 to 6


weeks.
Explanation:

Question 3 See full question

Which measure would the nurse expect to include in the teaching plan for a
multiparous client who gave birth 24 hours ago and is receiving intravenous
antibiotic therapy for cystitis?
You Selected:

emptying the bladder every 2 to 4 hours while awake


Correct response:

emptying the bladder every 2 to 4 hours while awake


Explanation:

Question 4 See full question

A nurse is providing nutritional teaching for a client with a family history of


colon cancer. Which food choice by the client demonstrates an
understanding of the correct diet to follow?
You Selected:

Egg salad on rye bread


Correct response:

Vegetarian chili
Explanation:

Question 5 See full question

The nurse is teaching the mother of a preschool-aged child with celiac


disease about a gluten-free diet. The nurse determines that the mother
understands the diet if she tells the nurse she will prepare:
You Selected:

eggs and orange juice


Correct response:

eggs and orange juice


Explanation:

Question 1 See full question

The nurse has administered aminophylline to a client with emphysema. The


medication is effective when there is:
You Selected:

relaxation of smooth muscles in the bronchioles.


Correct response:

relaxation of smooth muscles in the bronchioles.


Explanation:

Question 2 See full question

The nurse should teach the client that signs of digoxin toxicity include:
You Selected:

visual disturbances such as seeing yellow spots.


Correct response:

visual disturbances such as seeing yellow spots.


Explanation:

Question 3 See full question

A clients wife states, I do not think lithium is helping my husband. He has


been taking it for 2 days now, and he is still so hyper and thinks we are rich.
Which response by the nurse would be most accurate?
You Selected:

It takes 1 to 2 weeks for the drug to build up in the blood to be effective.


Correct response:

It takes 1 to 2 weeks for the drug to build up in the blood to be effective.


Explanation:

Question 4 See full question

After completing client teaching on use of patient-controlled analgesia (PCA),


the nurse determines that the client understands when the client states:
You Selected:

"The machine will give me only the prescribed amount of pain medication
even if I push the button too soon."
Correct response:

"The machine will give me only the prescribed amount of pain medication
even if I push the button too soon."
Explanation:

Question 5 See full question

A client with an intravenous line in place states having pain at the insertion
site. Assessment of the site reveals a vein that is red, warm, and hard. Which
actions would the nurse take? Select all that apply.
You Selected:

Assess the client for skin sloughing.


Restart the infusion distal to the discontinued intravenous site.
Document the assessment, nursing actions taken, and the clients response.
Discontinue the infusion at the affected site.
Correct response:

Discontinue the infusion at the affected site.


Apply warm soaks to the intravenous site.
Document the assessment, nursing actions taken, and the clients response.
Explanation:

Redness, warmth, pain, and a hard, cordlike vein at the intravenous catheter
insertion site suggest that the client has phlebitis. The nurse would
discontinue the intravenous infusion and insert a new catheter proximal to or
above the discontinued site or in the other arm. Applying warm soaks to the
site reduces inflammation. The nurse would document the assessment of the
intravenous. site, the actions taken, and clients response to the situation.
Slowing the infusion rate would not reduce the phlebitis. Restarting the
infusion at a site distal to the phlebitis may contribute to the inflammation.
Skin sloughing is not a symptom of phlebitis; it is associated with
extravasation of certain toxic medications.

Question 47 See full question

A client with Tourette syndrome is seen in an outpatient clinic. The client has multiple tics occurring several times
per day. The nurse notices that the client has a difficult time completing tasks such as activities of daily living
(ADLs). In which of the following ways can the nurse best help this client?

You Selected:

Break down tasks into small achievable steps

Correct response:

Break down tasks into small achievable steps

Explanation: Question 1 See full question

After instructing a 20-year-old nulligravid client about adverse effects of oral


contraceptives, the nurse determines that further instruction is needed when
the client states which as an adverse effect?
You Selected:

ovarian cancer
Correct response:

ovarian cancer
Explanation:

Question 2 See full question

A home care nurse visits a client diagnosed with atrial fibrillation who is
ordered warfarin. The nurse teaches the client about warfarin therapy. Which
statement by the client indicates the need for further teaching?
You Selected:

"I'll eat four servings of fresh, dark green vegetables every day."
Correct response:

"I'll eat four servings of fresh, dark green vegetables every day."
Explanation:

Question 3 See full question

Total parenteral nutrition (TPN) is prescribed for a client who has recently had
a significant small and large bowel resection and is currently not taking
anything by mouth. The nurse should:
You Selected:

administer TPN through a nasogastric or gastrostomy tube.


Correct response:

handle TPN using strict aseptic technique.


Explanation:

Question 4 See full question

The nurse notes grapefruit juice on the breakfast tray of a client taking
repaglinide. The nurse should:
You Selected:

remove the grapefruit juice from the client's tray and bring another juice of
the client's preference.
Correct response:

remove the grapefruit juice from the client's tray and bring another juice of
the client's preference.
Explanation:

Question 5 See full question

After a 3-month trial of dietary therapy, a client with type 2 diabetes still has
blood glucose levels above 180 mg/dl (9.99mmol/L). The physician adds
glyburide, 2.5 mg P.O. daily, to the treatment regimen. The nurse should
instruct the client to take the glyburide:
You Selected:

at breakfast.
Correct response:

at breakfast.
Explanation:

Question 1 See full question

The husband of a client who was diagnosed 6 years ago with Alzheimers
disease approaches the nurse and says, I am so excited that my wife is
starting to use donepezil for her illness. The nurse should tell the husband:
You Selected:

the medication is effective mostly in the early stages of the illness.


Correct response:

the medication is effective mostly in the early stages of the illness.


Explanation:

Question 2 See full question

Nonsteroidal anti-inflammatory drugs (NSAIDs) are commonly used in the


treatment of musculoskeletal conditions. It is important for the nurse to
remind the client to:
You Selected:

take NSAIDs with food.


Correct response:

take NSAIDs with food.


Explanation:

Question 3 See full question

The nurse is working on discharge plans with a client who is diagnosed with
intermittent explosive disorder, characterized by sudden angry outbursts.
The nurse determines that the client is ready for discharge when he makes
which comment?
You Selected:

"I will be taking valproic acid and propranolol to help stay in control."
Correct response:

"I will be taking valproic acid and propranolol to help stay in control."
Explanation:

Question 4 See full question

A client experiences initial indications of excitation after having an IV infusion


of lidocaine hydrochloride started. The nurse should further assess the client
when the client reports having:
You Selected:

tinnitus.
Correct response:

tinnitus.
Explanation:

Question 5 See full question

The wife of a 67-year-old client who has been taking imipramine for 3 days
asks the nurse why her husband is not better. The nurse should tell the wife:
You Selected:

It takes 2 to 4 weeks before the full therapeutic effects are experienced.


Correct response:

It takes 2 to 4 weeks before the full therapeutic effects are experienced.


Explanation:

Discard the uncapped needle in a puncture


After proof container.
administering
an I.M
injection, a
nurse should:
2. After knee replacement "Avoid driving a car while
surgery, patient taking this medication."
discharged with
acetaminophen &
codeine tablets, 30 mg,
for pain. The nurse
should include which
instruction during
discharge?
3. After receiving an I.M. Apply a warm compress to
injection, client dilate the blood vessels. (Heat
complains of burning at increases blood flow in area,
injection site. Which which increases medication
nursing action would be absorption)
most appropriate at this
time?
4. After reconstituting a Strength of the medication
multidose vial of
medication, a nurse
writes date and time of
reconstitution on vial
label. What else should
the nurse write on the
label?
5. Before administering Normal saline solution
packed red blood cells,
nurse must flush a
client's I.V. line. Which
solution should nurse
use to flush the line?
6. Beta-adrenergic Work by blocking beta
blockers receptors in myocardium,
reducing response to
catecholamines & sympathetic
nerve stimulation. Protect
myocardium, helping reduce
risk of another infarction by
decreasing myocardial oxygen
demand.
7. Calcium channel Reduce the hearts workload by
blockers decreasing the heart rate.
8. Child with type 1 Hypokalemia (Insulin
diabetes develops administration causes glucose
diabetic ketoacidosis & & potassium to move into the
receives a continuous cells)
insulin infusion. Which
condition represents the
greatest risk to child?
9. Client admitted into emergency department after Bind with the ingested drug
intentionally taking overdose of amitriptyline so that the body does not
(Elavil). A nurse knows giving activated charcoal absorb it.
will:
1 Client comes to emergency department after Administering activated
0. taking an overdose of amitriptyline (Evavil). charcoal every 4 hours for
Immediate care for this client should include: 24 hours. (Charcoal binds
with amtriptyline and
inactivates it)
1 Client discharged with prescription for an Within 6 months
1. analgesic that is a controlled substance. During
discharge teaching, nurse should explain that
client must fill the prescription how soon after the
date on which the physician wrote it?
1 Client is in the bathroom when nurse enters to give Return to the client's room a
2. prescribed medications. What should the nurse do? few minutes later & remain
there until the client take
the medication.
1 A client is scheduled for surgery at 8 a.m. While Notify the surgeon that the
3. completing the preoperative checklist, nurse sees client has not signed the
the surgical consent form has not been signed. It's consent form.
time to administer the preoperative analgesic.
Which nursing action takes highest priority in this
situation?
1 Client is to receive a glycerin suppository. When 4" (10cm) - Far enough to
4. administering suppository, nurse should insert it pass the internal anal
how far into client's rectum? sphincter.
1 Client is to receive a glycerin suppository. Which Applying a lubricant to the
5. nursing action is appropriate when administering a suppository.
suppository?
1 Client ordered heparin 6,000 units every 12 hours. 0.6 ml
6. Pharmacy dispenses a vial containing 10,000
units/ml. How many ml of heparin should nurse
administer?
1 Client taking hormonal contraceptives, nurse Blood pressure
7. should ensure the client knows she must have
which vial sign monitored regularly?
1 Client with a deficient fluid volume receiving Dark amber urine
8. an I.V. infusion of dextrose 5% in water &
lactated Ringer's solution at 125 mL/hour,
Which assessment finding indicated need
for additional fluids?
1 Dopamine Is a vasconstrictor used to treat
9. hypotension.
2 A drug must enter the bloodstream before it I.V.
0. can act within the body. Which parental
administration route places drug directly
into the circulation, requiring no
absorption?
2 Drug package reads "(Demerol), 50mg/ml" 0.6 ml
1. How many ml should a nurse give a client
for a 30 mg dose?
2 For client who takes over-the-counter drugs Whether the client knows the drug
2. regularly, nurse should ascertain" dosages & administration schedules.
2 If a manual end-of-shift count of controlled Immediately report the discrepancy
3. substances isn't correct, the nurses best to the nurse-manager, nursing
action is to: supervisor, & pharmacy.
2 The main advantage of using a floor stock A nurse can implement medication
4. system? orders quickly.
2 The maximum transfusion time for a unit of 4 hours ( Beyond four hours of
5. packed red blood cells (RBCs) is: transfusion there is increased risk
for bacterial contamination of the
blood)
2 Most common cause of medication errors Deficient knowledge
6. among non-institutionalized elderly clients?
2 Nitrates Reduce myocardial oxygen
7. consumption by decreasing left
ventricular end-diastolic pressure
(preload) & systemic vascular
resistance (afterload).
2 Nurse caring for a client taking an oral Avoid foods high in vitamin K. (K can
8. anticoagulant. The nurse should teach the interfere with anti-coagulation.)
client to:
2 Nurse caring for a patient with central Tachypnea (Signs of infection
9. venous catheter. Which assessment finding tachypnea, dizziness, & lethargy)
would indicate possible infection?
3 A nurse has a order to administer iron dextran Use the Z-track technique
0. (INFeD) 50 mg I.M. injection. When carrying out
this order, the nurse should:
3 Nurse is administering two drugs to a client at the Synergism
1. same time. The nurse knows the most probable
reason for giving the drugs together is:
3 Nurse is calculating proper dosage of medication Body surface area in relation
2. for a child. What parameter should influence this to weight
calculation?
3 Nurse is reconstituting a powdered medication in Roll the vial gently between
3. vial. After adding the solution to the powder, the her palms
nurse should:
3 Nurse is to administer I.M. injection into a client's Lying supine
4. left Vastus lateralis muscle. How should nurse
position patient?
3 Nurse is to administer several oral medications to Terll the client the name and
5. a client at the same time. Which nursing action or use of each
instruction or action is appropriate in this medication before
situation? administering it.
3 Nurse is to give a client a 325-mg aspirin Withhold the suppository &
6. suppository. The client has diarrhea & is in the notify the client's physician.
bathroom. Best nursing approach at this time
would be:
3 A nurse must verify a client's identity before Check the clients
7. administering medication. The safest way to identification band.
verify identity is to:
3 A nurse notes client's I.V. insertion site is red, Discontinue the I.V. infusion.
8. swollen, & warm to the touch. Which action
should nurse take first?
3 Nurse regularly inspects a client's IV site to Elevating the limb, applying
9. ensure patency & prevent extravasation during warm compresses, &
dopamine (Intropin) therapy. What is the administering phentolamine
treatment for dopamine extravasation? (Regitine) as ordered.
4 Opioids Reduce myocardial oxygen
0. demand, promote
vasodilation, & decrease
anxiety.
4 Order digoxin (Lanoxin) 0.125 mg by mouth every 0.5
1. morning for client with heart failure. Pharmacy
dispenses tablets that contain 0.25 mg each. How
many tablets should the nurse administer in each
dose?
4 Order heparin, 7,500 units administered 3/4 ml
2. subcutaneously every 12 hours. The vial reads 10,000
units per milliliter. Nurse should anticipate giving how
much heparin for each dose?
4 Patient being discharged after surgery. After providing Educator
3. medication teaching, nurse ask patient to repeat
instructions. This approach is an example of which
professional role?
4 Physican orders an infusion of 2,400 ml of I.V. fluid 120 ml/hour
4. over 24 hours, with half this amount to be infused over
first 10 hours. During the first 10 hours, client should
receive how many ml of I.V. fluid per hour?
4 Physican writes order for client that says: "Digoxin . "Digoxin 0.125 mg P.O.
5. 125 mg P.O. once daily." To prevent a dosage error, once daily."
how should nurse transcribe the order onto medication
administration record.
4 Physician orders 1 liter dextrose 5% in water at 150 150/60 min= 2.5
6. ml/hr. The drip factor of the I.V. tubing is 15 gtt/ml. 2.5x15= 37.5
What is the drip rate for this I.V. infusion in drops per 37.5 gtt/minute
minute?
4 Physician orders activated charcoal for client who Has audible bowel
7. intentionally took overdose of hydrocodone (Vicodin). sounds.
Before administering the drug, nurse should ensure
that client:
4 Physician orders amipicillin (Omnipen), 500 mg by A standing order.
8. mouth every 6 hours. This medication order is an
example of:
4 Physician orders an infusion of whole blood for a Staying with the patient
9. patient. When planning car, nurse should include which for 15 minutes after
intervention? starting the infusion.
5 physician orders dextrose 5% in water, 1000ml to be 125/60 minutes=x/1
0. infused over 8 hours. I.V. tubing delivers 15 drops/ml.. minute
the nurse should run the I.V. infusion at a rate of: 60min=125=2.1
ml/minute
32 drops/minute
5 A physician orders morphine, 3mg I.V. every 2 hours as 0.75
1. needed, to control pain. Insert reads "Morphine,
4mg/ml." How many milliliter of morphine should client
receive?
5 Physician orders nitroglycerin, 5mg by mouth twice a Two
2. day. The drug is dispensed in 2.5 mg tablets. How
many tablets will the nurse administer with each dose?
5 Physician orders normal saline 150x8 = 1,200 ml
3 infused at rate of 150 ml/hour. 1.2 liters
. How many liters will the client
receive during an 8 hour shift.
5 Progesterone Hormone used to treat amenorrhea or
4 dysfunctional uterine bleeding.
.

5 Tamoxifen Estrogen blocker used to treat premenopausal &


5 postmenopausal breast cancer & to prevent
. breast cancer in certain women who are at high
risk.
5 What statement by student "Nurses who are pregnant must wear gloves
6 nurse demonstrates further during administration of cytotoxic drugs."
. instruction about cytotoxic (Pregnant nurses cannot administer cytotoxic
drugs is needed? drugs because exposure may be associated with
teratogenic effects)
5 When a central venous catheter Remove the dressing, clean the site, & apply a
7 becomes moist or loose, what new dressing. (dressing should be changed every
. should a nurse do first? 72 hours or when it becomes soiled, moist, or
loose.
5 When administering an I.M. Using the one-handed needle-recapping
8 injection, which action puts the technique after administering all injections.
. nurse at risk for a needle-stick
injury?
5 When administering an 15 degree
9 intradermal injection the
. appropriate angle is:
6 When administering I.M. With one hand, use the needle to scoop up the
0 injection, nurse notices there is cap. Holding the barrel in one hand, carry the
. not a sharps-disposal container syringe to the nearest sharps disposal container.
handy. Which action should the
nurse take?
6 When giving an I.M. injection, 90 degrees
1 nurse should insert the needle
. into the muscle at an angle of:
6 When giving a subcutaneous 45 or 90 degree angle
2 injection the nurse may use
. what angle?
A 4-month-old Strategy: Answers are implementations.
infant who had a Determine the outcome of each answer choice.
temperature of Is it desired?
103F (39.4C)
following the last (1) withholding the diphtheria and tetanus
DTaP (diphtheria, vaccine is not indicated
tetanus, and
pertussis) (2) child would still receive the pertussis, which
vaccine is seen would probably cause another febrile reaction
in the clinic for
another (3) correctfever over 103F (39.4C) in first 48
immunization hours after DTaP is a valid contraindication for
administration. pertussis vaccine
Prior to the
nurse's (4) would be correct if just the DT were given
administering
the DTaP, which
of the following
should be the
nurse's priority?

1. Withhold the
immunization.
2. Give half the
dose in this
injection.
3. Consult the
physician about
giving pediatric
DT (diphtheria
and tetanus).
4. Instruct the
parents to give
acetaminophen
following
administration of
the full dose of
DTaP.
2. An 8-year-old Strategy: "MOST important"
child is brought indicates a priority question. All
to the physician's answer choices are assessments.
office by his Determine why you would
mother. The perform each assessment and
mother is how it relates to the situation.
concerned
because the boy (1) check of grasp strength is a
has a fever, nonspecific neurological check
vomited twice,
and slept all day (2) normal response plantar
yesterday with flexion of toes (negative
the curtains Babinski); dorsiflexion of great
closed. The child toe and fanning of other toes
complains of (positive Babinski) abnormal in
headache and child older than 2; indicates CNS
nausea and has a disease, not indicated in this
temperature of situation
103F (39.3C).
The nurse (3) correctBrudzinski's reflex;
observes the positive response (flexion of the
child has a hips and knees) indicates
petechial rash on meningeal irritation
the trunk of the
body. Which of (4) Romberg's sign; nonspecific;
the following assesses equilibrium and
assessments is cerebellar functions
MOST important
for the nurse to
perform?

1. Grasp the
child's hands, and
ask him to
squeeze the
nurse's hands.
2. Stroke the
plantar surface of
the child's foot
with a reflex
hammer.
3. Gently flex the
child's head and
neck onto the
chest.
4. Have the child
stand with his
eyes closed, his
arms at his sides,
and his feet and
knees close
together.
3. An 11-month-old Strategy: Answers are a mix of
baby is having assessments and
trouble gaining implementations. Is validation
weight after required? Yes.
discharge from
the hospital. (1) correctassessment; will
Which of the provide the most information
following actions
by the nurse is (2) assessment; may or may not
BEST? secure an accurate picture

1. Observe the (3) assessment; weight should be


child at mealtime. obtained more often or on each
2. Inquire about visit
the child's eating
patterns. (4) implementation; need to
3. Weigh the baby assess before determining
each month. appropriate interventions
4. Attempt to
feed the baby for
the mother.
4 A 12-year-old client is admitted Strategy: All answers are implementations.
. to the pediatric unit in vaso- Determine the outcome of each answer choice. Is
occlusive crisis from sickle cell it desired?
anemia. As the nurse prepares
the plan of care, which of the (1) appropriate orders for this client
following orders should the
nurse question? (2) appropriate orders for this client

1. Bedrest with bathroom (3) correctadequate hydration must be


privileges. maintained to prevent sickling and clumping of the
2. Two liters oxygen via nasal affected cells
cannula.
3. Maintain IV at keep-open (4) appropriate orders for this client
rate.
4. Administer analgesics as
ordered.
5 A 16-year-old is brought by her Strategy: Remember the principles of therapeutic
. parents to the outpatient clinic communication.
for treatment of pelvic
inflammatory disease (PID). (1) correctuses therapeutic technique of
While the nurse obtains a reflecting; validates feelings without placing value
history, the client says bitterly, judgment or giving approval or disapproval
"My parents are mean and don't
really care about me." Which of (2) negates client's feelings, blocks communication
the following responses by the
nurse is BEST? (3) negates client's feelings, blocks communication

1. "You feel your parents don't (4) yes/no question


care about you?"
2. "Your parents brought you to
the clinic, didn't they?"
3. "I am sure that your parents
have your best interests at
heart."
4. "Did you have a disagreement
with your parents?"
6 A 20-year-old, gravida 1, para 0 Strategy: Determine how each answer choice
. woman comes to the clinic for relates to determining the EDC.
her first routine prenatal exam.
During the physical assessment, (1) probable sign of pregnancy
the client informs the nurse that
she is unsure of the date of her (2) probable sign of pregnancy
last menstrual period. Which of
the following assessments (3) presumptive sign of pregnancy
would best assist the nurse in
determining her expected date (4) correctfetal heartbeat can be heard at 12
of confinement (EDC)? weeks; is a positive sign of pregnancy

1. The presence of Hegar's sign.


2. A positive pregnancy test.
3. The presence of quickening.
4. Auscultation of the fetal
heartbeat.
7 A 41-year-old woman was Strategy: Answers are a mix of assessments and
. brought to the emergency room implementations. Does this situation require
by two police officers after she validation? No. Determine the best
had been standing barefoot in implementation.
the rain for more than two
hours. The police officers report (1) assessment; should not be first action
that the woman had to be
restrained after she resisted (2) correctimplementation; major priority of the
and became agitated. The nurse is to provide and maintain safety for the
intake nurse's FIRST action client who is unable to provide for herself; safe
should be which of the environment will generate trust and rapport; will
following? decrease resistance to doing preliminary physical
exam, which includes orienting client and doing a
1. Complete a physical mental status exam
examination.
2. Maintain a safe environment. (3) assessment; should not be first action
3. Ascertain the client's mental
status. (4) implementation; should not be first action
4. Orient the client to place and
time.
8. A 74-year-old man is brought by his daughter Strategy: Remember therapeutic
to the emergency room. When asked his name, communication.
he is unable to remember it and appears to be
disheveled, restless, and confused. His (1) closed statement; doesn't
daughter says that she has been caring for him encourage discussion of feelings
at home for the last year, but he "ran away"
after they had an argument about his (2) minimizes feelings and
deteriorating personal hygiene. She found him problems; nontherapeutic
several hours later sitting in the street. She
confides to the nurse that she feels horrible (3) correctresponds to feeling
about yelling at her father. Which of the tone, encourages discussion of
following is the BEST response by the nurse? feelings
1. "We all do things that we are sorry for later." (4) passing the buck; doesn't
2. "Don't feel guilty because he is confused." respond to feeling tone
3. "Your father's illness must be difficult for
both of you."
4. "The social worker will be able to help you
with this problem."
9. A 76-year-old woman has a medical history Strategy: All answers are
that includes hypertension with cardiac assessments. Determine how
involvement. A public health nurse visits this each relates to the situation.
client regularly and on each visit records her Remember the "comma, comma,
vital signs. Which of the following findings and" rule.
should the nurse expect for this client?
(1) temperature, pulse, and
1. Temperature 99.5F (37.5C), pulse 110, respirations too high for the
respirations 32, blood pressure 140/80. elderly
2. Temperature 98.6F (37C), pulse 78,
respirations 16, blood pressure 120/80. (2) would expect with younger
3. Temperature 99.8F (37.7C), pulse 90, client without history of
respirations 20, blood pressure 150/90. hypertension
4. Temperature 96.8F (36C), pulse 80,
respirations 20, blood pressure 160/90. (3) temperature and pulse
elevated; not expected with
elderly clients

(4) correcttemperature is
usually lower due to decrease in
BMR; pulse and respirations
normal; BP expected with history
of hypertension
1 An adolescent is admitted for insertion of a Strategy: All answers are
0. Harrington rod due to scoliosis. In preparation implementations. Determine the
for the immediate postoperative care, the outcome of each answer. Is it
nurse should include which of the following in desired?
a teaching plan for this client?
(1) correctclients must be
1. Take 10 deep breaths every 2 hours. monitored closely for the first 48-
2. Get on the bedpan by lifting the hips. 72 hours for respiratory
3. Soft diet as tolerated. problems; bowel and urinary
4. Elevate legs 10 times every 4 hours. problems need to be assessed
along with neurological problems
in the extremities

(2) client will have a catheter

(3) client may have a nasogastric


tube connected to low suction
(4) not appropriate for the
situation
1 An adolescent is admitted to the hospital Strategy: "MOST concerned" indicates a
1. after sustaining a concussion because of complication.
an auto accident. The nurse is MOST
concerned if which of the following is (1) correctincreased systolic pressure
observed? and widening pulse pressure indicate
increased intracranial pressure
1. The patient's blood pressure changes
from 130/88 to 150/74. (2) normal finding
2. The patient's pupils are equal and
react to light and accommodation. (3) alteration in level of consciousness
3. The patient has difficulty remembering is symptom of cerebral edema, but
what happened just before the accident. some amnesia for events immediately
4. The patient has a urinary output of before accident is expected because of
120 ml from 5 PM until 7 PM. trauma

(4) 30 ml/hour normal finding


1 An adolescent is seen in the emergency Strategy: Answers are a mix of
2. room for an overdose of acetylsalicylic assessments and implementations. Is
acid (Aspirin). Which of the following assessment required? Yes. Is there an
actions by the nurse is BEST? appropriate assessment? Yes.

1. Determine when the client took the (1) correctcharcoal, if given within
aspirin. two hours, will absorb particles of
2. Initiate an intravenous infusion, and salicylate
administer protamine sulfate.
3. Administer vitamin K (2) antidote for heparin
(AquaMEPHYTON).
4. Obtain an arterial blood gas, and (3) antidote for warfarin (Coumadin)
request respiratory therapy to begin
respiratory support. (4) may be necessary later, but current
need is to evaluate response to
charcoal
1 After abdominal surgery, a client Strategy: All answers are
3. complains of gas pains in her abdomen. It implementations. Determine the
is MOST important for the nurse to take outcome of each answer choice. Is it
which of the following actions? desired?

1. Offer the client fresh fruits and (1) should not be encouraged until the
vegetables. bowel sounds have returned and client
2. Ambulate the client frequently. is able to eat; will help prevent
3. Teach the client how to splint the constipation but will not prevent gas
abdomen during activity. pains
4. Position the client on her right side.
(2) correctambulation promotes the
return of peristalsis and facilitates
expulsion of flatus

(3) does nothing to increase peristalsis,


which is needed after surgery

(4) does nothing to increase peristalsis,


which is needed after surgery
1 After termination of preterm labor, the Strategy: Determine the significance of
4. nurse confirms the ability of a client to each assessment and how it relates to
monitor herself at home for fetal well- fetal well-being.
being if she can do which of the
following? (1) might indicate the onset of
premature labor
1. Count uterine contractions.
2. Measure her urine output. (2) relates to maternal well-being
3. Count fetal kicks.
4. Weigh herself daily. (3) correctensures that the fetus is
moving and makes the mother aware of
the importance of monitoring fetal
movement daily

(4) relates to maternal well-being


1 After the anesthesiologist administers an Strategy: Answers are a mix of
5. epidural to a woman in labor, which of the assessments and implementations.
following nursing actions has the HIGHEST Does this situation require
priority? assessment? Yes. Is there an
appropriate assessment? Yes.
1. Decrease IV fluids.
2. Assess the fetal heart monitor. (1) implementation; client must be
3. Place the mother on her right side. well hydrated before and after the
4. Obtain the blood pressure. procedure

(2) assessment; may be done as


ongoing management but is not a
priority

(3) implementation; laboring mother


would be placed on left side to
promote uterine perfusion

(4) correctassessment; side effect


of an epidural is hypotension from
the vasodilation that occurs
1 At a health-screening clinic, an adult male Strategy: Answer choices are a mix
6. client's total plasma cholesterol level is 200 of assessments and
mg/dL. Which of the following actions by the implementations. Does this situation
nurse is BEST? require assessment? Yes.

1. Refer the client to a physician for (1) implementation; levels higher


appropriate medication. than 250 mg/dL may require
2. Refer the client to the dietitian. medication if diet therapy is not
3. Obtain a diet history. effective
4. Recheck the cholesterol level in two
years. (2) implementation; passing the
buck

(3) correctassessment; total


cholesterol level for an adult male
should be under 200 mg/dL; higher
levels require a low-fat diet; obtain
diet history before instructing on a
low-fat diet

(4) assessment; blood level should


be checked earlier than two years
1 A child is admitted to the pediatric unit with Strategy: "MOST appropriate"
7. a suspected diagnosis of Haemophilus indicates that two answer choices
influenzae meningitis. As the nurse explains will be similar.
care to the parents, they ask how long their
child will need to be in a room by herself. (1) not accurate; may depend on
Which response by the nurse is MOST negative lumbar puncture
appropriate?
(2) nontherapeutic, dismissive tone
1. "It depends on the results of her blood
counts." (3) correcttreated with penicillin;
2. "Patients like her are usually in isolation IV fluids and isolation for 24 hours
for a couple of days or so." after the start of antibiotic therapy
3. "Isolation can usually be stopped 24 to prevent respiratory transmission
hours after the start of antibiotic therapy."
4. "When your child has been afebrile for 48 (4) inaccurate; don't use medical
hours, we will move her." terms without explanation
1 A client at 16 weeks' gestation has a Strategy: Answers are implementations.
8. blood sample drawn for rubella Determine the outcome of each answer
antibody screening. The test results choice. Is it desired?
reveal a low titer. When discussing the
results with the client, the nurse should (1) active immunization should not be
take which of the following actions? administered

1. Arrange for the client to have an (2) should not be done in this situation
MMR immunization immediately.
2. Explain to the client that the results (3) should not be done in this situation
are expected and nothing needs to be
done. (4) correctwith a low rubella titer, the
3. Explore options with the client about client is at risk for developing rubella;
whether to terminate the pregnancy. immediately after delivery, within early
4. Encourage the client to receive the postpartum period, she needs to receive
rubella immunization immediately after an immunization
delivery.
1 A client at 38 weeks' gestation is Strategy: Answers are a mix of
9. admitted in active labor. The nursing assessments and implementations. Does
assessment reveals a decrease in the this situation require validation? No.
client's blood pressure to 90/50, and Determine the outcome of each
the fetal heart rate (FHR) is 130 and implementation.
regular. Which of the following nursing
actions is MOST important? (1) may be necessary, but answer choice
4 should be done first
1. Contact the physician.
2. Elevate the head of the bed. (2) does not address the problem of low
3. Check the client's blood pressure and BP
FHR every 30 minutes.
4. Place the client on her left side. (3) validation not required; problem
needs to be addressed immediately

(4) correctdecrease in blood pressure is


most likely due to pressure on the inferior
vena cava, which occurs in the supine
position (vena-caval syndrome);
positioning client on her side will relieve
pressure so that BP will increase
2 A client awakens during the night with Strategy: All answers are
0. dyspnea, severe anxiety, jugular vein implementations. Determine the outcome
distention (JVD), and frothy pink of each answer choice. Is it desired?
sputum. After the nurse begins oxygen
at 4 liters per nasal cannula, which of (1) would increase fluids to the lungs
the following actions is MOST
appropriate? (2) correctnext priority is to notify the
physician; signs indicate pulmonary
1. Place two pillows behind the head, edema
and elevate the legs.
2. Notify the physician about the (3) would increase fluids to the lungs
change in the client's condition.
3. Increase IV fluids to liquefy the (4) the nurse should stay with the client
secretions. for reassurance
4. Dim the lights, and provide privacy.
2 A client begins taking haloperidol Strategy: All answers are
1. (Haldol) 5 mg tid. It is MOST important implementations. Determine the outcome
for the nurse to share which of the of each answer. Is it desired?
following with the client?
(1) appropriate for a monoamine oxidase
1. "Do not eat aged cheese, beer, or red (MAO) inhibitor
wine."
2. "Rise slowly when standing." (2) correctside effect of Haldol is
3. "Suck on hard candy." hypotension; moving slowly to a standing
4. "Avoid pretzels, potato chips, and position will decrease the problem with
carbonated beverages." orthostatic hypotension

(3) medication does not have


anticholinergic effects

(4) salt does not have any effect on the


medication
2 A client comes to the clinic for a glycosylated Strategy: Answers are a mix of
2. hemoglobin assay (HbA1c). The result is 6%. assessments and
The nurse should take which of the following implementations. Does this
actions? situation require validation? No.
Determine the outcome of each
1. Document the findings in the chart. answer choice.
2. Call the physician about orders to adjust
the insulin dosage. (1) correctresults normal,
3. Give the client 15 g of carbohydrates. indicates good control of diabetes
4. Ask the client to list the foods he has eaten
in the last 24 hours. (2) no adjustments need to be
made

(3) does not reflect hypoglycemia

(4) no adjustment needs to be


made in diet; result is not altered
by intake day before test
2 A client complains of hearing loss. While the Strategy: Answers are a mix of
3. nurse is irrigating the client's ear to remove assessments and
cerumen for better observation of the implementations. Are the
tympanic membrane, the client complains of assessments correct? No.
dizziness. Which of the following actions Determine the outcome of the
should the nurse take FIRST? implementations.

1. Notify the physician immediately. (1) unnecessary


2. Monitor for changes in intracranial
pressure. (2) assessment; client is not
3. Warm the irrigant, and resume the experiencing increased
procedure. intracranial pressure
4. Explore the canal with a cotton applicator.
(3) correctwater that is too cool
can elicit dizziness when it comes
into contact with the tympanic
membrane

(4) assessment; could compact


the cerumen against the
membrane; is never
recommended
2 A client diagnosed with acute lymphocytic Strategy: Think about each
4. leukemia is admitted with shortness of breath, answer choice.
anemia, and tachycardia. The MOST
appropriately stated nursing diagnosis for this (1) incorrectly stated
client is which of the following?
(2) correctleukemia causes a
1. Altered protection, immunosuppression: decrease in all blood components;
leukemia. a gas exchange problem results
2. Impaired gas exchange related to from depletion of oxygen-carrying
decreased RBCs. red cells
3. Risk for infection related to altered immune
system. (3) appropriately stated and
4. Risk of injury related to decreased relates to leukemia but is not
platelets. supported by the assessment
data in the question

(4) appropriately stated and


relates to leukemia but is not
supported by the assessment
data in the question
2 A client diagnosed with AIDS is admitted Strategy: All answer choices are
5. to the medical unit with complaints of implementations. Determine the
fatigue, a persistent dry cough, and outcome of each answer choice. Is it
dyspnea on exertion. Vital signs include desired?
BP 136/88, temperature 104F (40C),
pulse 95, respirations 22. Which of the (1) correctwill reduce fever and
following actions by the nurse is BEST? provide for comfort; head of bed
elevated because of respiratory distress
1. Administer a tepid sponge bath with
the patient in semi-Fowler's position. (2) needs fluids for hydration because of
2. Limit oral intake to a maximum of fever; fluids should be encouraged to
2,000 ml of fluid per day. 3,000 ml/day
3. Encourage the patient to perform
passive ROM four times a day. (3) activities should be moderate and
4. Suction the patient every four hours paced to reduce shortness of breath and
to maintain a patent airway. exhaustion; not best choice
(4) suctioning is performed PRN;
unnecessary suctioning would irritate
and increase secretions
2 A client diagnosed with a severe thought Strategy: Answers are implementations
6. disturbance has not been taking his Determine the outcome of each answer
medication and appears to be choice. Is it desired?
hallucinating more actively. The client
claims that the medicine makes him (1) correctmedication dose
drowsy during the day. Which of the noncompliance is often associated with
following actions by the nurse is BEST? negative side effects and a multiple-
dosing daily schedule; when client has
1. Ask the physician to schedule the only one daily dose at bedtime, is easier
client's entire dose at bedtime. to remember to take medication; other
2. Tell the client that he is getting sicker advantage is that sedative effects of the
and must take his medicine. drug peak while client is sleeping
3. Teach the client about the side effects
of the medication. (2) does not offer concrete solutions and
4. Ask the family to talk to the client may encourage the client to act more
about this problem. childlike

(3) does not deal with the side effects of


the medication

(4) passing the buck


2 A client diagnosed with chronic Strategy: All answers are
7. alcoholism who occasionally uses implementations. Determine the
marijuana and cocaine is attending a outcome of each answer choice. Is it
second group therapy meeting. The desired?
client comments, "I am having difficulty
sitting still. Am I bothering some of the (1) correctclient is experiencing some
group members? Maybe I should stop mild anxiety related to detoxification as
coming to these group meetings." Which well as participation in group process;
of the following nursing actions is MOST needs reinforcement and
appropriate? encouragement to continue attending
the group meetings and to share
1. Encourage the client to share his feelings
problem with the group members.
2. Remove the client from the group and (2) is a result of anxiety
assess his needs.
3. Recognize that this is manipulative (3) is not manipulative behavior but
behavior and encourage the client to result of anxiety
remain in the group.
4. Tell the client not to concern himself (4) is a result of anxiety
about the group members and to
continue in the group.
2 A client diagnosed with chronic Strategy: Select the incorrect
8. obstructive pulmonary disease (COPD) medication. Think about the action of
is admitted with an acute exacerbation. each drug.
The client's vital signs are B/P 162/100,
pulse 78, respirations 30 and labored (1) drug of choice for acute asthma
with wheezing. The nurse should
question which of the following orders? (2) broad spectrum antibiotic, not
contraindicated
1. Theophylline (Somophyllin) 0.7
mg/kg/h IV. (3) blocks parasympathetic stimulation
2. Tetracycline hydrochloride (Sumycin) and decreases mucus; used with asthma
250 mg IM qd.
3. Ipratropium bromide (Atrovent) (4) correctbeta-blocker that blocks
inhaler 2 inhalations qid. beta adrenergic impulses to the
4. Propranolol hydrochloride (Inderal) bronchial tree that cause bronchodilation
40 mg PO bid. resulting in increased
bronchoconstriction
2 A client diagnosed with end-stage Strategy: All answers are
9. metastatic cancer of the breast is implementations. Determine the
admitted to the hospital. It is MOST outcome of each answer. Is it desired?
important for the nurse to take which of
the following actions? (1) decreases oxygen levels, is
uncomfortable and unnecessary
1. Suction the patient frequently.
2. Provide an air mattress. (2) equipment is not most important
3. Turn the patient every two hours.
4. Give the patient frequent baths. (3) correctprevents complications such
as skin breakdown

(4) will dry out her skin and cause


chilling
3 A client diagnosed with peripheral Strategy: All answers are
0. vascular disease (PVD) states that he implementations. Determine the
experiences leg pain frequently when outcome of each answer choice. Is it
walking, and he asks the nurse in the desired?
clinic what he should do. The nurse
should advise him to take which of the (1) decreases arterial flow to legs
following actions?
(2) decreased sensitivity to pain may
1. Lie down with his feet elevated above result in burns
his heart when he experiences pain.
2. Apply a heating pad to his legs for 15 (3) correctexercise increases collateral
minutes before walking. circulation, should be encouraged
3. Walk until he experiences pain, then
rest, and then resume walking. (4) stretching will not reduce pain due to
4. Perform stretching exercises 20 intermittent claudication
minutes before starting to walk.
3 A client diagnosed with tertiary syphilis Strategy: Answers are a mix of
1. is admitted to a nursing unit. The client assessments and implementations. Does
exhibits signs of marked dementia and this situation require assessment? Yes.
disorientation. Which of the following
actions should the nurse take FIRST? (1) implementation; should not be
assumed that the client will be able to
1. Place the nurse call bell within reach. use the call light appropriately
2. Frequently observe the client's
behavior. (2) correctassessment; placing the
3. Apply a vest-type restraint. client on frequent observation status
4. Provide an around-the-clock sitter. would be the first action to ensure the
client's safety

(3) implementation; should never be the


first option exercised by a professional
nurse; current standards require not only
a physician's order but a time limit, the
exact type of restraint to be used, and
the specific rationale

(4) implementation; is not usually a


nursing responsibility
3 A client had a mitral valve replacement Strategy: All answers are
2. three days ago. It is MOST important for implementations. Determine the
the nurse to take which of the following outcome of each answer choice. Is it
actions? desired?

1. Maintain the client in the supine (1) client is maintained in semi-Fowler's


position to prevent tension on the position
mediastinal suture line.
2. Encourage deep breathing but (2) coughing and deep breathing should
discourage coughing because of be encouraged
increased central venous pressure.
3. Decrease fluids to prevent fluid (3) fluids are encouraged unless there is
retention and development of evidence of cardiac failure
congestive heart failure.
4. Encourage early activity to promote (4) correctpostoperative open heart
ventilation and improve quality of clients should be encouraged to be out
circulation. of bed and ambulating as soon as
possible, frequently one to two days
after surgery
3 A client had a thoracotomy three hours Strategy: All answers are
3. ago. For the past two hours, there has implementations. Determine the
been 100 ml/hour of bloody chest outcome of each answer. Is it desired?
drainage. Which of the following actions
should the nurse take FIRST? (1) may be appropriate after the
physician is notified
1. Increase the IV fluid rate.
2. Administer oxygen at 5 L/minute per (2) may be appropriate after the
oxygen mask. physician is notified
3. Elevate the head of the bed.
4. Advise the physician of the amount of (3) may be appropriate after the
drainage. physician is notified

(4) correctchest drainage of 100


ml/hour is abnormal; physician should
be notified
3 A client has a cataract removed from the Strategy: Answers are all
4. left eye. Which of the following is an implementations. Determine the
important nursing intervention in the outcome of each answer choice. Is it
immediate postoperative period? desired?

1. Position the client on the right side (1) correctshould be positioned on


with the head slightly elevated. back or unaffected side to prevent
2. Place the client on the left side to trauma to surgical eye
protect the eye.
3. Perform sensory neurological checks (2) should be positioned on unaffected
every two hours. side
4. Maintain complete bedrest for the
first 48 hours. (3) unnecessary for cataract clients

(4) unnecessary for cataract clients


3 A client has an order for aminophylline Strategy: Determine the significance of
5. (Truphylline) PO. The nurse should each answer choice and how it relates to
withhold the medication and notify the aminophylline.
physician if the client makes which of
the following statements? (1) aminoglycosides are antibiotics

1. "I am allergic to neomycin." (2) correctmay decrease metabolism


2. "I am taking propranolol (Inderal)." and lead to toxicity
3. "I have trouble breathing when I
exercise." (3) medication is given to treat airway
4. "I have had several urinary tract problems
infections."
(4) urinary tract infections are not a
concern
3 A client has an order for hydrochlorothiazide Strategy: All answers are
6. (HydroDIURIL) 50 mg qd. The nurse knows implementations. Determine the
that teaching is successful if the client states outcome of each answer. Is it
which of the following? desired?

1. "I should not operate heavy machinery." (1) medication does not cause
2. "I should drink five glasses of liquid per drowsiness
day."
3. "This medication will cause my urine to (2) there are no specific restrictions
turn orange." on fluid at this time
4. "I should eat dried apricots each day."
(3) does not occur

(4) correctcontinued use of this


diuretic may cause a loss of
potassium; dietary intake of foods
such as bananas or dried apricots,
which are high in potassium,
should be encouraged
3 A client has been placed on phenelzine Strategy: All answers are
7. sulfate (Nardil) 11 mg PO daily to assist in implementations. Determine the
treating depression. The nurse determines outcome of each answer choice. Is
that teaching is effective if the client makes it desired?
which of the following statements?
(1) correctmedication is an MAO
1. "I will call my physician and stop taking inhibitor; hypertensive crisis may
the medication if I begin to have severe be precipitated by foods containing
headaches." tyramine; client should be taught
2. "I can drink wine, but I should avoid to report problems associated with
alcoholic beverages that contain high levels hypertension
of alcohol."
3. "I know I am going to feel better in a (2) wine is contraindicated; it
couple of days. I am so glad that I finally got contains tyramine
some medication."
4. "I can take the over-the-counter (OTC) cold (3) takes up to two weeks for the
medications that contain pseudoephedrine." medication to be effective

(4) cold medications with


pseudoephedrine are
contraindicated due to possible
hypertensive problems
3 A client has been receiving chlorpromazine Strategy: All answers are
8. (Thorazine) 400 mg/day for four weeks. He implementations. Determine the
experiences an oral temperature of 105F outcome of each answer. Is it
(40.5C), severe rigidity, oculogyric crisis, desired?
and severe hypertension. It is MOST
important for the nurse to take which of the (1) bromocriptine (Parlodel) or
following actions? dantrolene (Dantrium) is used for
CNS toxicity
1. Administer PRN benztropine mesylate
(Cogentin) immediately. (2) correctclient is experiencing
2. Hold the chlorpromazine, and notify the neuroleptic malignant syndrome;
medical staff stat. fatal in about 15-20% of cases; is
3. Place the client in isolation on bedrest in toxic effect of antipsychotic
semi-Fowler's position. medication
4. Administer acetaminophen 500 mg, and
place the client on a cooling mattress. (3) isolation is unnecessary

(4) is not most important; cooling


blanket is used for fever, IV fluids
for hydration, airway if necessary,
frequent monitoring of vital signs
3 A client has received cimetidine Strategy: All answers are
9. (Tagamet) 300 mg qid for several implementations. Determine the outcome
weeks. During an office visit, the of each answer choice. Is it desired?
physician gives the client an additional
prescription for aluminum hydroxide (1) antacids interfere with absorption of
(Amphojel) 600 mg qid. Which of the Tagamet; administration should be
following instructions, if given by the separated by one hour
nurse, is BEST?
(2) Amphojel should be given one hour
1. Take the Tagamet and Amphojel after meals and Tagamet with meals
together after meals and hs for
combined effect. (3) Tagamet should be given with meals
2. Take the Amphojel with meals and and Amphojel one hour after meals
before bed, and take the Tagamet one
hour after meals and before bed. (4) correctgive Tagamet with meals
3. Take the Tagamet two hours before (causes more consistent therapeutic
meals and before bed, and take the effect) and hs; antacids interfere with
Amphojel two hours after meals and at absorption; separate administration by
bedtime. one hour, give Amphojel one hour after
4. Take the Tagamet with meals and one meals and hs (separate administration by
hour before bed, and take the Amphojel one hour)
two hours after meals and hs.
4 A client has recently been placed on Strategy: Determine the nutrients
0. warfarin (Coumadin) for transient contained in each answer and how they
ischemic attacks (TIAs). The nurse is relate to Coumadin.
MOST concerned if the patient makes
which of the following statements? (1) high in potassium; would have no
effect on the medication
1. "I eat cantaloupe and bananas every
day." (2) high in sodium; would have no effect
2. "I can eat potato chips and dill on the medication
pickles."
3. "I eat strawberries and oranges (3) high in vitamin C; would have no
every day." effect on the medication
4. "I have to eat more green salads and
pork." (4) correctingestion of large quantities
of foods high in vitamin K content may
antagonize the anticoagulant effect of
warfarin
4 A client has taken levothyroxine sodium Strategy: Think about each answer choice
1. (Synthroid) 0.4 mg daily for 4 days. and how it relates to Synthroid.
Which of the following findings should
cause the nurse to recommend a (1) correctsuggests overdosage of
change in the client's medication? thyroid hormone replacement therapy

1. The client develops nervousness and (2) symptom of hypothyroidism, the


difficulty sleeping. reason for giving this medication
2. The client states that she has no
energy and is "just tired." (3) symptom of hypothyroidism, the
3. The client has coarse hair and skin. reason for giving this medication
4. The client has a persistent weight
gain. (4) symptom of hypothyroidism, the
reason for giving this medication
4 A client is admitted to the emergency Strategy: All answers are
2. room with complaints of crushing chest implementations. Determine the
pain, shortness of breath, and left arm outcome of each answer choice. Is it
pain. Which of the following actions, if desired?
taken by the nurse, is BEST?
(1) would be implemented after answer
1. Administer oxygen. choice 3 was implemented
2. Place in a semi-Fowler's position.
3. Administer morphine sulfate. (2) the position should be high Fowler's
4. Administer lidocaine.
(3) correctthe priority is to decrease
the pain; this will decrease the metabolic
needs, which will result in a decrease in
the cardiac demands; morphine reduces
preload and afterload pressures,
increasing cardiac output

(4) there is no indication of ventricular


tachycardia
4 A client is diagnosed with a tumor of the Strategy: "MOST important" indicates
3. pituitary gland and has a that this is a priority question. Determine
transsphenoidal hypophysectomy. The what each assessment measures and
nurse plans care for the patient two how it relates to the situation.
days after surgery. It is MOST important
for the nurse to monitor which of the (1) not affected by surgery
following?
(2) controlled by the medulla, not the
1. Complete blood count (CBC). pituitary
2. Temperature.
3. Specific gravity of urine. (3) correctlack of ADH from pituitary
4. Intracranial pressure. will cause diabetes insipidus and diuresis
with very low specific gravity

(4) surgery performed through nose;


does not affect cerebral pressure
4 A client is diagnosed with myasthenia Strategy: "Need for further teaching"
4. gravis, and the nurse instructs the client indicates you are looking for an incorrect
about the disease. Which of the statement.
following statements, if made by the
client to the nurse, indicates the need (1) should be avoided
for further teaching?
(2) may cause infection
1. "I should not drink alcoholic
beverages." (3) emotional extremes can cause
2. "I should not go places that are exacerbations
crowded."
3. "I should try to stay calm." (4) correctshould avoid heat (sauna,
4. "I should use my hot tub daily." hot tubs, sunbathing)
4 A client is diagnosed with otosclerosis Strategy: Determine how each answer
5. and is admitted for a stapedectomy. It is relates to otosclerosis.
MOST important for the nurse to ask
which of the following questions? (1) describes ruptured tympanic
membrane, not relevant to otosclerosis
1. "Have you noticed fluid draining from
your left ear?" (2) hearing impairment may begin in the
2. "Have you had problems hearing for early adult years, but not at birth
your entire life?"
3. "Did you require speech therapy (3) speech is not affected by hearing loss
when you were a child?"
4. "When did you notice that your (4) correctotosclerosis occurs gradually
hearing was impaired?" over many years; often client is not
aware of it until the impairment is
significant
4 A client is given an aminophylline Strategy: All answers are
6. (Somophyllin) capsule four hours too early. implementations. Determine the
This incident is discovered 30 minutes after outcome of each answer choice. Is it
administration of the medication. The desired?
nurse should take which of the following
actions? (1) correctdocumenting the error
on an incident report, assessing for
1. Document the event on an incident side effects, and notifying the
report form, and notify the physician. physician are the most appropriate
2. Change the time for the next medication actions for the nurse to take
administration.
3. Assess for bradycardia and lethargy, and (2) unsafe, incorrect nursing
notify the physician. intervention
4. Skip the next dose of the medication.
(3) tachycardia, anxiety will occur

(4) unsafe, incorrect nursing


intervention
4 A client is scheduled for a myelogram Strategy: All answers are
7. because of complaints of severe lumbar implementations. Determine the
pain. Which of the following nursing outcome of each answer choice. Is it
interventions is MOST important for this desired?
procedure?
(1) will be NPO for four to six hours,
1. Inform the client about being NPO prior but not most important
to the test.
2. Encourage ambulation after the test. (2) bedrest may be maintained for as
3. Encourage fluids prior to and after the long as 24 hours
test.
4. Instruct the client to remain prone for 24 (3) correctfluids promote excretion
hours. of the dye and also facilitate
formation of spinal fluid

(4) should have the head elevated at


30 for 24 hours
4 A client is seen in the physician's office for Strategy: "MOST beneficial" indicates
8. follow-up after treatment for renal calculi. priority. All answers are
The nurse discusses methods to prevent a implementations. Determine the
recurrence of the problem. Which of the outcome of each answer choice. Is it
following instructions by the nurse is MOST desired?
beneficial?
(1) correctprevention program:
1. "Drink at least 3,000 ml of fluid a day." diet, medications, fluids 3,000 to
2. "Reduce the amount of dairy products 4,000 ml/day
and eggs in your diet."
3. "Increase the amount of whole grains (2) only helpful if you know
and vegetables that you eat." composition of stones; information
4. "Avoid foods that contain tyramine, such not presented in question
as wine and cheese."
(3) dietary changes helpful if
composition of stones known

(4) should be avoided if taking MAO


inhibitor medications such as
Marplan to treat depression
4 A client is to receive peritoneal dialysis Strategy: "ESSENTIAL" indicates a priority
9. through a catheter inserted through a question. Answers are a mix of
trocar. Which of the following nursing assessments and implementations. Is
interventions is ESSENTIAL for the assessment required? Yes. What is the
nurse to perform? best assessment?

1. Maintain the client in a supine (1) implementation; on strict bedrest


position during the procedure. because of trocar but may be in semi-
2. Weigh the client during the Fowler's position to prevent pressure of
procedure and again 24 hours later. fluid on diaphragm
3. Change the dwell time according to
the client's tolerance during the (2) assessment; should obtain weight,
procedure. pulse, and BP before procedure and again
4. Check the client's BP and apical and after
radial pulses before the procedure.
(3) implementation; dwell time is
prescribed by physician

(4) correctassessment; should obtain


baseline vital signs
5 The client is to receive regional Strategy: Answers are a mix of
0. anesthesia (spinal anesthesia) during assessments and implementations. Do the
surgery. Which of the following is an assessments make sense? No.
important nursing implication
regarding this anesthesia? (1) correctimplementation; important
that the client be well hydrated to prevent
1. Adequately hydrate the client. hypotensive problems after the spinal
2. NPO client for at least 12 hours. anesthesia is initiated
3. Assess the client for any allergies to
Betadine or iodine preparations. (2) implementation; unnecessary for client
4. Determine the specific gravity of the to be NPO for 12 hours
urine.
(3) assessment; unnecessary, as iodine
dyes are not used

(4) assessment; irrelevant to the


procedure
5 A client is treated for deep vein Strategy: "MOST concerned" indicates a
1. thrombosis with IV unfractionated complication.
heparin. The nurse is MOST concerned
if which of the following is observed? (1) incorrect

1. Increased anxiety. (2) incorrect


2. Decreased heart rate.
3. Increased activated partial (3) desired response to therapy
thromboplastin time (aPTT).
4. Decreased level of consciousness. (4) correctmajor side effect is bleeding;
decrease in level of consciousness
indicates intracranial bleeding
5 A client is treated for rheumatoid Strategy: Determine the significance of
2. arthritis. Which of the following each answer choice and how it relates to
findings should assume the HIGHEST rheumatoid arthritis.
priority for the nurse when planning
the client's care? (1) expected symptom of disease

1. The client has subcutaneous nodules (2) correctsign of inadequate


on the right and left forearms. management; should be treated
2. The client has a slight contracture of immediately to prevent further damage
the right wrist.
3. The client has mild erythema of the (3) redness expected symptom of disease
finger joints.
4. The client has an area of ecchymosis (4) may be result of mild trauma, not
approximately 3 mm in diameter on highest priority
right forearm.
5 A client on continuous mechanical ventilation Strategy: Determine how the
3. desires to go home. In order to determine assessment relates to home care.
the client's ability for home care, the nurse
should take which of the following actions? (1) correctto ensure safety and to
provide client with quality care at
1. Assess the ability of others in the home to home, assessing ability of others in
be trained to provide appropriate care for home is critical before proceeding
the client. with efforts to discharge the client
2. Confer with the client's physician, and
discuss the feasibility of the client's request. (2) should occur, but ensuring that
3. Assess the number of people in the home someone can care for the client
and the adequacy of space to care for the should occur before consulting with
client. the physician
4. Examine the client's reasons for wanting
to go home, and discuss the implications of (3) may be appropriate after the
home care. home situation is evaluated

(4) may occur but first determine if


someone can care for the client
5 A client receives prochlorperazine maleate Strategy: "MOST concerned"
4. (Compazine) 10 mg IM before repair of a indicates a complication.
hernia under general anesthesia. The nurse
is MOST concerned if which of the following (1) normal to replace fluids lost
is observed six hours after surgery? during surgery

1. An IV of 0.9% NaCl is infusing at 100 (2) normal due to general


ml/hour. anesthesia
2. The patient is sleepy but able to be
aroused. (3) correcturine retention is side
3. The patient has not voided since surgery. effect of medication and is caused
4. There is a moderate amount of by general anesthesia
serosanguineous drainage on the abdominal
dressing. (4) some drainage expected
5 A client receives thiethylperazine maleate Strategy: All answers are
5. (Torecan) 10 mg IM after surgery for repair of implementations. Determine the
a hernia. The ordered activity is up ad lib. outcome of each answer choice. Is
One half hour after administration of the it desired?
medication, the patient has to void. The
nurse should take which of the following (1) should stay in bed one hour
actions? after getting medication because of
possible orthostatic hypotension
1. Accompany the patient to the bathroom.
2. Place the patient on the bedpan. (2) correctnecessary for patient
3. Obtain a bedside commode for the patient safety
to use.
4. Obtain an order to catheterize the patient. (3) should stay in bed one hour
after getting medication because of
possible orthostatic hypotension

(4) unnecessary invasive procedure


5 A client recently admitted to labor and Strategy: All answers are
6. delivery states that she is having severe implementations. Determine the
discomfort with contractions. The nursing outcome of each answer choice. Is it
assessment reveals that the client is 3 cm desired?
dilated. The nurse assists the client
through guided imagery. Ten minutes later (1) increase in discomfort and
the client is more agitated. The nurse agitation indicates that pain
should take which of the following actions? management through guided
imagery is not successful
1. Reteach the exercise.
2. Reposition the client. (2) correctincrease in discomfort
3. Turn on the television. and agitation indicates that pain
4. Ambulate the client. management through guided
imagery is not successful, so it
should be discontinued and another
nursing intervention implemented,
such as repositioning the client

(3) might distract the client but


would not be as effective as answer
choice 2

(4) would increase the pain


5 A client returns to the floor following a Strategy: Answers are a mix of
7. bronchoscopy. The client complains of assessments and implementations.
thirst and requests ice chips. The physician Does this situation require
left an order for the patient to resume a assessment? Yes. Is there an
regular diet. The nurse should take which appropriate assessment? Yes.
of the following actions?
(1) correctassessment; local
1. Touch the back of the client's throat with anesthesia sprayed on throat may
a tongue depressor. interfere with swallowing
2. Observe the client while he sucks on a
few ice chips. (2) assessment; need to check gag
3. Provide clear fluids to the client and reflex first
advance to soft foods.
4. Assess the client's tissue turgor and (3) implementation; need to check
intake and output. gag reflex, may choke or aspirate

(4) assessment; does not address


need to check gag reflex
5 A client returns to the recovery room at the Strategy: "BEST" indicates a priority
8. outpatient surgery center after cataract question. All answers are
surgery. The nurse notes that the IV site in implementations. Determine the
the client's left hand appears reddened and outcome of each answer choice. Is it
warm. Which of the following actions, if desired?
performed by the nurse, is BEST?
(1) phlebitis can by caused by
1. Call the physician to obtain an order to insertion technique, medication, or
remove the IV cannula. break in aseptic technique; nurse can
2. Apply cool compresses, and continue to remove cannula without physician
assess the IV insertion site. order
3. Stop the IV infusion, and remove the IV
cannula. (2) should remove and replace IV in
4. Apply antibiotic ointment to the site, and another site
change the IV dressing.
(3) correctif IV site is infected, IV
should be removed and restarted

(4) discontinue IV and restart in


another site
5 A client undergoes a total laryngectomy Strategy: "Teaching was effective"
9. because of carcinoma. The nurse instructs indicates a correct action.
the client and spouse how to suction the
laryngectomy tube. Which of the following (1) used for oral suctioning of
actions indicates to the nurse that teaching mouth
is effective?
(2) correcthyperoxygenates and
1. The man selects a Yankauer tonsil tip prevents anoxia
catheter to suction the laryngectomy tube.
2. The man takes several deep breaths (3) apply suction only as catheter is
before the suction catheter is inserted. withdrawn
3. The man applies suction as he introduces
the sterile catheter into the stoma. (4) suction laryngectomy tube and
4. The wife suctions the man's mouth and then mouth
then the laryngectomy tube.
6 A client undergoes peritoneal dialysis at Strategy: "FIRST" indicates priority.
0. home. The home care nurse notices the fluid
outflow is inadequate. Which of the following (1) correctfacilitates drainage
actions should the nurse take FIRST?
(2) second action
1. Turn the client from side to side.
2. Check for kinks in the tubing. (3) all clamps should be open
3. Close the clamp to the drainage tubing for
one half hour, and then reopen. (4) not best first action; done
4. Milk the drainage tubing firmly every 20 carefully as needed if fibrin clot has
minutes. formed
6 A client visits the rape-crisis clinic one week Strategy: All answers are
1. after being assaulted. The client is currently implementations. Determine the
taking alprazolam (Xanax) 0.25 mg PO q 6 outcome of each answer. Is it
hours for anxiety. Which of the following desired?
statements, if made by the client to the
nurse, reflects a correct understanding of (1) indicates a need for further
this medication? medication teaching

1. "I can take it whenever I feel upset." (2) indicates a need for further
2. "I should not take this with anything but medication teaching
water."
3. "I guess I need to stop drinking white (3) correctantianxiety, should not
wine." be taken with alcoholic beverages
4. "This medication will help me forget and
go on." (4) indicates a need for further
medication teaching
6 A client who had an appendectomy four days Strategy: All answers are
2. ago complains of severe abdominal pain. implementations. Determine the
During the initial assessment he states, "I outcome of each answer choice. Is
have had two almost-black stools today." it desired?
Which of the following nursing actions is
MOST important? (1) requires a physician's order and
would probably be ordered after the
1. Start an IV with D5W at 125 ml/hour. nurse notifies the physician of the
2. Insert a nasogastric tube. findings
3. Notify the physician.
4. Obtain a stool specimen. (2) requires a physician's order and
would probably be ordered after the
nurse notifies the physician of the
findings

(3) correctdevelopment of black,


tarry stool in the presence of
abdominal pain could represent
gastrointestinal bleeding; should be
reported to physician as soon as
possible

(4) requires a physician's order and


would probably be ordered after the
nurse notifies the physician of the
findings
6 A client who is terminal is on a unit with limited Strategy: Answers are
3. visiting hours that restrict children younger than implementations. Determine
12 years of age from visiting. Which nursing the outcome of each answer.
action has the HIGHEST priority? Is it desired?

1. Explain the visiting hours to the client's family. (1) does not address the
2. Propose a policy change to the medical and client's needs
nursing staff.
3. Allow flexibility with family members' (2) not highest priority
visitation.
4. Encourage the family to call the unit between (3) correctrole of the nurse
visiting hours. is to function as client
advocate; is important to
individualize care with all
clients

(4) does not address the


client's needs
6 A client with sudden onset of deep vein Strategy: "Question orders"
4. thrombosis (DVT) is started on IV unfractionated indicates an incorrect order.
heparin. Which of the following orders should the
nurse question? (1) appropriate therapy

1. Warm, moist packs to the affected leg. (2) appropriate therapy


2. Elevate the foot of the bed 6 inches.
3. Commode privileges without weight bearing. (3) correcton bedrest for 5
4. Elastic stockings on unaffected leg. to 7 days; keep affected
extremity elevated

(4) appropriate therapy


6 A client with urinary frequency, burning, and a Strategy: All answers are
5. temperature of 102F (38.8C) is instructed by implementations. Determine
the nurse to collect a urine specimen for culture the outcome of each answer
and sensitivity. The nurse knows that teaching is choice. Is it desired?
successful if the client states which of the
following? (1) unnecessary for the
collection
1. "I will call the lab before I collect my urine."
2. "I will drink several glasses of water before I (2) unnecessary for the
collect my urine." collection
3. "I will collect the specimen using an aseptic
technique." (3) correctaseptic technique
4. "I will discard my first voiding in the morning." decreases the possibility of
contamination with organisms

(4) unnecessary for the


collection
6 The clinic nurse returns a phone call from a client Strategy: All answers are
6. diagnosed with type 1 diabetes who has been implementations. Determine
vomiting for 24 hours. It is MOST important for the outcome of each answer
the nurse to instruct the client to take which of choice. Is it desired?
the following actions?
(1) diabetic should always
1. Take half of the regular insulin dose. take the regular dose of
2. Attempt to maintain the regular diabetic diet. insulin or alter it only
3. Limit intake of sweets and sugar. according to serial glucose
4. Drink liquids as often as possible. checks

(2) client is not tolerating PO


foods

(3) sweets can be used as


calories in this situation

(4) correctdiabetic
ketoacidosis is frequently
associated with dehydration;
fluids should be encouraged
6 The daughter of an elderly client Strategy: Think about what the words
7. diagnosed with Alzheimer's disease mean. How do they relate to the
provides care for her parent in her home. caregiver?
The nurse knows that which of the
following observations MOST likely (1) may be impossible for the daughter
represents caregiver burnout? to do alone

1. The daughter fails to get her mother (2) correctcluttered environment may
into a wheelchair daily. represent depression and burnout
2. The home environment is extremely
cluttered at each visit. (3) may reveal the limited time the
3. The daughter is always in a housecoat daughter has to take of herself
at the time of the nurse's visits.
4. The daughter's husband is seen (4) is very healthy and desirable
assisting with his mother-in-law's care.
6 During administration of oral medications Strategy: All answers are
8. to an elderly, confused client, the client implementations. Determine the
states, "These pills look funny. They outcome of each answer. Is it desired?
belong to the lady down the hall." Which
of the following is the BEST response by (1) unsafe action
the nurse?
(2) unsafe action
1. "Your physician has ordered new
medications for you. They will help you (3) unsafe action
get well."
2. "Remember yesterday when I brought (4) correcteven confused client
your medications? They look the same." should have his/her medications
3. "I'll explain why you are receiving rechecked when there is any possibility
these medications." of an error; always observe the six
4. "I'll be back after I check your rights of medication administration
medications again."
6 During a health history, a teenaged girl Strategy: Answers are a mix of
9. tells the nurse, "I have no appetite, and assessments and implementations.
I've lost 4 lb this week." It is MOST Does this situation require validation?
important for the nurse to take which of Yes.
the following actions?
(1) passing the buck; no reason to
1. Notify the physician. contact the physician
2. Weigh the client.
3. Continue with the interview. (2) will be done as part of physical
4. Examine the abdomen. assessment; complete the interview

(3) correctcomplete the health history


interview before beginning physical
assessment

(4) part of the physical assessment


7 During a nonstress test (NST), the nurse Strategy: Answers are implementations.
0. observes several late decelerations. Determine the outcome of each answer
Which of the following nursing actions is choice. Is it desired?
MOST appropriate?
(1) does not resolve the immediate
1. Reposition the client on her right side. problem
2. Notify the physician for further
evaluation. (2) correctappearance of any
3. Document these results in the nurse's decelerations of the fetal heart rate
notes. (FHR) during NST should be
4. Stop the oxytocin (Pitocin) immediately evaluated by the physician
immediately.
(3) does not resolve the immediate
problem

(4) incorrect for this test; oxytocin


(Pitocin) is not used for the nonstress
test
7 During a well-baby checkup, the nurse Strategy: MOST concerned" indicates a
1. evaluates the reflexes of a 6-month-old complication.
child. The nurse is MOST concerned if
which of the following is observed? (1) disappears at approximately 1 year of
age
1. Presence of a positive Babinski
reflex. (2) correctextrusion reflex disappears
2. Extrusion reflex when feeding. between 3 and 4 months of age
3. Able to grasp objects voluntarily.
4. Rolls from abdomen to back at will. (3) normal occurrence at this age level

(4) normal occurrence at this age level


7 During the first trimester of pregnancy, Strategy: Answers are a mix of
2. a client experiences hyperemesis, which assessments and implementations. Is the
results in a decrease in weight, poor assessment appropriate? No. Determine
skin turgor, and a chloride deficiency. the outcome of the implementations.
Which of the following nursing orders
should the nurse implement FIRST? (1) correctoffering fluids at frequent
intervals will increase hydration status
1. Offer fluids every hour as tolerated.
2. Complete an intake and output record (2) contains a plan to evaluate the status
every 4 hours. of hydration
3. Start an IV immediately upon
admission. (3) contains a plan formulated from the
4. Perform a weight check every physician's orders; the question refers to
morning. nursing orders

(4) assessment; is an appropriate action


for evaluating the client, but it does not
meet hydration needs
7 During the initial prenatal visit, the Strategy: All answers are
3. physician orders an iron supplement to implementations. Determine the
be taken throughout the client's outcome of each answer choice. Is it
pregnancy. It is MOST important for the desired?
nurse to include which of the following
instructions?" (1) correctvitamin C facilitates
absorption of iron
1. "The medication should be taken with
orange juice." (2) antacids will decrease absorption of
2. "Take the medication with antacids to the iron
decrease gastric distress."
3. "Drinking 8 oz of water will enhance (3) client needs increased fluids; fluids
absorption of the medication." will not affect absorption
4. "Notify the physician if your stools
become dark or loose." (4) stools will turn dark, but there is no
need to notify the physician
7 An elderly client constantly comes to Strategy: All answers are
4. the nurse's station with varying implementations. Determine the
complaints and requests. Which of the outcome of each answer choice. Is it
following actions by the nurse is BEST? desired?

1. Speak to the client only when the (1) can escalate his feelings of
client calls the nurse. abandonment
2. Address and manage each specific
complaint and request. (2) not the best action and would not
3. Redirect the client to other activity. serve to reduce the behavior
4. Interact with the client at consistent
intervals. (3) can escalate his feelings of
abandonment

(4) correctclient is probably fearful of


being abandoned; interacting with the
client at consistent intervals when he is
not complaining will begin to reduce the
calling, requesting, and complaining
behaviors
7 An elderly client has had a subtotal gastrectomy. Strategy: Think about each
5. The client received meperidine (Demerol) 75 mg answer choice and how it
and hydroxyzine hydrochloride (Vistaril) 50 mg relates to the medication.
IM. The nurse is MOST concerned if which of the
following was observed? (1) inaccurate

1. Tachypnea. (2) expected finding


2. Lethargy.
3. Hypertension. (3) inaccurate
4. Disorientation.
(4) correctelderly are prone
to paradoxical reactions and
can become agitated and
disoriented
7 An elderly client is diagnosed with a vitamin K Strategy: Answers are a mix of
6. deficiency because of dietary malabsorption. assessments and
Which of the following is an appropriate nursing implementations. Does this
intervention for this client? situation require assessment?
Yes.
1. Encourage the client to remain in bed.
2. Carefully check the client's arm after taking (1) implementation; remaining
the blood pressure. in bed does not decrease the
3. Increase dietary intake of fruits and fiber. potential for bleeding
4. Observe the client for signs of angina or
cardiac dysrhythmia. (2) correctassessment;
observe for bruising of the arm
after taking a blood pressure
reading

(3) implementation; does not


affect absorption of vitamin K

(4) assessment; inappropriate


for vitamin K deficiency
7 An elderly client is very confused and Strategy: Remember Maslow.
7. disoriented when admitted to the hospital from
a long-term care facility. Which of the following (1) not the highest priority
is a priority nursing assessment?
(2) not the highest priority
1. Determine the client's level of mobility when
walking. (3) not the highest priority
2. Evaluate the client's teeth, and determine an
appropriate diet. (4) correctpresence of
3. Determine if a family member can remain at hypoxia needs to be addressed
the bedside. immediately; hypoxia
4. Assess the respiratory status, and evaluate contributes to the confusion
for hypoxia.
7 An elderly patient diagnosed with Alzheimer's Strategy: "MOST important"
8. disease frequently wanders down the halls of the indicates a priority question.
extended care facility and displays restless All answers are
agitation. The physician orders a vest restraint. implementations. Determine
When the nurse takes the restraint to the room, the outcome of each answer
the patient refuses to put it on. It is MOST choice. Is it desired?
important for the nurse to take which of the
following actions? (1) correctas long as
behavior is not unsafe, nurse
1. Take the restraint away, and check the patient should try other methods to
frequently. engage patient in activities to
2. Notify the physician immediately that the reduce wandering
patient refused the restraint.
3. Ask a coworker to hold the patient and gently (2) not first or most important
apply the restraint. action of nurse; patient is
4. Exchange the vest restraint for wrist primary responsibility
restraints.
(3) should not forcibly restrain
patient
(4) will make agitation worse;
shouldn't restrain patients
against their will
7 An elderly patient undergoes a colonoscopy. Strategy: Determine how each
9. During the postprocedure period, it is MOST answer choice relates to a
important for the nurse to monitor for which of colonoscopy.
the following?
(1) spinal anesthesia not used
1. Patient's ability to move the legs.
2. Fluid and electrolyte balance. (2) correctbowel prep and
3. Characteristics of the patient's stool. NPO status puts patient at
4. Level of pain the patient experiences. high risk for imbalances

(3) will not have stools


immediately after procedure
because of bowel preparation

(4) not a painful procedure


8 The family members of an 85-year-old report to Strategy: Remember
0. the nurse that they suspect that their father is therapeutic communication.
masturbating. Which of the following responses
by the nurse is BEST? (1) inappropriate

1. "I understand your concern because this is not (2) inappropriate


a normal part of aging."
2. "Don't worry because I think that he will stop (3) correctmasturbation is
soon." an activity performed by some
3. "This is considered a normal behavior for elderly men
men."
4. "The best thing you can do is talk to your (4) would embarrass the
father about this behavior." father and cause him to have
feelings of guilt and anxiety
8 A father brings his 15-month-old son to the Strategy: "BEST" indicates priority.
1. well-baby clinic for a routine checkup. The
father confides to the nurse that he is (1) child usually begins walking
concerned that his son still crawls and does 12-15 months of age, no reason
not walk. Which of the following responses, if for referral
made by the nurse to the father, is BEST?
(2) does not recognize behavior as
1. "I will refer you to a pediatric specialist if normal
he doesn't start walking soon."
2. "Have you noticed any signs of paralysis or (3) children will walk when they
weakness in your son?" are ready
3. "Try standing him on his feet several times
a day." (4) correctchildren are
4. "Children frequently set their own pace for individuals
development."
8 The father of a 1-day-old son works the Strategy: Answers are
2. evening shift (3 PM to 11 PM) at another implementation. Determine the
hospital. Which of the following plans is a outcome of each answer. Is it
priority to meet the needs of this father? desired?

1. Encourage the father to call his wife after (1) inflexible


work.
2. Instruct the father about visiting policy and (2) inflexible
suggest AM visitation.
3. Adjust visiting hours to meet the new (3) correctrole of nurse is to be
parents' needs. a family and client advocate; this
4. Present a change of visiting hours to the provides individualized care
appropriate hospital committee.
(4) not a priority, although it may
be an appropriate long-range goal
8 Four days after a client has an abdominal Strategy: Determine how each
3. perineal resection, which of the following answer choice relates to a
signs is MOST important for the nurse to complication after surgery.
report to the physician?
(1) expected after this type of
1. Moderate amount of serosanguineous surgery
drainage on the abdominal dressing.
2. Nausea, vomiting, and increased abdominal (2) correctsymptoms indicate
distention. paralytic ileus and should be
3. Moderate amount of yellow-green reported to the physician
nasogastric drainage and decreased urine
output. (3) nasogastric drainage is
4. Urinary output via Foley catheter 120 ml expected after this type of
over a 4-hour period. surgery, but amount of urinary
output is not expected to change
significantly

(4) normal output via Foley


8 Four days ago the physician prescribed Strategy: Determine how each
4. lithium carbonate (Lithobid) 600 mg tid for a answer relates to lithium.
client. The client returns to the outpatient
clinic for evaluation. What teaching regarding (1) should check for ankle edema;
the medication is MOST important for the lithium causes polyuria (increase
nurse to reinforce with the client and the in urinary output) and dehydration
client's spouse?
(2) should check weight daily and
1. The client should check for ankle swelling have lithium levels checked 8-12
and decreased urinary output. hours after first dose, 2-3 times a
2. The client should keep a log of the time of week the first month, then weekly
day medication is taken and the way he is to monthly while on maintenance
feeling.
3. The client should call the clinic if tremors, (3) correct50% will develop
muscular weakness, or ataxia develops. dose-related tremors; signs of
4. Because of the medication, the client toxicity are diarrhea, vomiting,
should be experiencing remission of the drowsiness, muscular weakness,
symptoms. ataxia

(4) takes one to two weeks before


patient will see results
8 The home care nurse makes a follow-up visit to a Strategy: Think about what
5. client recently diagnosed with AIDS. Which of the the words mean.
following activities, if performed by the client,
indicates that the nurse's teaching has been (1) should use a soft
effective? toothbrush three to four
times/day to avoid injury to
1. The patient uses a firm toothbrush once a day oral mucosa
to brush her teeth.
2. The patient eats a large lunch at noon and a (2) small frequent meals
small dinner at 6 PM. recommended to aid
3. The patient changes the litter in her cat's litter digestion
box every day.
4. The patient takes docusate sodium (Colace) 300 (3) AIDS patients shouldn't
mg once a day. handle pet excreta

(4) correctbowel programs,


stool softeners, and laxatives
reduce intestinal stasis and
bacterial overgrowth
8 The home care nurse visits a client is taking Strategy: Determine how
6. carbidopa-levodopa (Sinemet) for tremors, each answer choice relates to
shuffling gait, and rigidity. Which of the following Sinemet.
statements, if made by the home health aide to
the nurse, indicates that the medication is (1) not result of the
effective? medication

1. "The client's weight increased by 2 lb." (2) not result of the


2. "The client is getting over an upper respiratory medication
infection."
3. "There is an increase in the fine motor (3) is unrealistic
tremors."
4. "The client seems to be more ambulatory." (4) correctis no cure for
these symptoms, but
carbidopa-levodopa
(Sinemet) does reduce the
rigidity and tremors, which
facilitates mobility for the
client
8 The home care nurse visits a client taking Strategy: The topic of the
7. diazepam (Valium) to help deal with the anxiety question is not clearly stated.
experienced due to nightmares and flashbacks Read answer choices for
about the war. During a visit, the home care nurse clues.
observes the client is ataxic, confused, has
slurred speech, and complains of dizziness. Which (1) does not deal with the
of the following nursing actions is MOST physiologic problem
appropriate?
(2) inaccurate
1. Give the client a relaxation tape, and send the
client to a quiet room. (3) would only compound the
2. Sit quietly with the client because the client is problem because sleeping
having a flashback. pills are cross-tolerant with
3. Recommend to the physician that the client be diazepam
given medication for sleep.
4. Recommend that the physician evaluate the (4) correctclient is
client's excessive use of the drug diazepam. displaying symptoms of
intoxication caused by
excessive use of the drug
diazepam, an indication of
potential dependence
8 The home care nurse visits a client with Strategy: "Teaching is effective"
8. a diagnosis of Addison's disease. The indicates a correct statement.
nurse determines that teaching is
effective when the client makes which of (1) correctif steroids are taken at night,
the following statements? they may cause sleeplessness

1. "I'll take hydrocortisone (Cortef) in (2) dosage has to be regulated according


the morning." to the amount of stress
2. "I'm glad that I will not have to
change my dose of hydrocortisone (3) client with Addison's disease has
(Cortef)." hyperkalemia
3. "I'll increase my potassium by eating
more bananas." (4) steroids cause fluid retention, which
4. "This medicine probably won't affect can increase the blood pressure
my blood pressure."
8 The home care nurse visits a young Strategy: "Further teaching is needed"
9. adult with a diagnosis of hepatitis A. indicates you are looking for an incorrect
Which of the following statements, if response.
made by the client to the nurse,
indicates that further teaching is (1) because hepatitis A is spread by the
needed? oral-rectal route, it is important to
protect others by practicing good hand-
1. "I have been very careful to wash my washing techniques and avoiding
hands after I go to the bathroom." contact with items that will be placed in
2. "I have had to take Tylenol several others' mouths
times this week for this sinus infection I
have." (2) correctclient should be cautioned
3. "I have been very careful not to about taking any drugs not approved by
handle my child's toys or eating the health care provider; may become
utensils." dangerous because of the liver's inability
4. "My husband has been preparing all to detoxify and excrete them
of the meals since I've been sick."
(3) because hepatitis A is spread by the
oral-rectal route, it is important to
protect others by practicing good hand-
washing techniques and avoiding
contact with items that will be placed in
others' mouths

(4) because hepatitis A is spread by the


oral-rectal route, it is important to
protect others by practicing good hand-
washing techniques and avoiding
contact with items that will be placed in
others' mouths
9 The home health nurse makes home Strategy: All answers are
0. visits to follow the progress of a 2-year- implementations. Determine the
old diagnosed with tetralogy of Fallot. outcome of each answer choice. Is it
When the nurse visits the home, the desired?
child is diaphoretic and short of breath.
What should the nurse do FIRST? (1) not first intervention

1. Give the boy oxygen at 2 liters via (2) correctknee-chest position


nasal cannula. treatment for cyanotic spells, enhances
2. Place the boy in the knee-chest or systemic venous return, dilates right
squatting position. ventricle, decreases the obstruction
3. Administer morphine 0.1 mg/kg SQ to
the boy. (3) not done first
4. Lay the boy in bed flat with his head
elevated. (4) should use knee-chest position
9 A husband and wife meet at the mental Strategy: Remember therapeutic
1. health clinic to make an appointment for communication.
family therapy. Suddenly, the wife begins to
sob loudly. As the nurse approaches, the (1) nontherapeutic; emphasis is
husband says, "I guess we just don't get placed on wife, not the situation
along." Which of the following responses by
the nurse is MOST appropriate? (2) nontherapeutic; closes off
communication
1. "Your wife seems to be upset by the
situation." (3) nontherapeutic; appears to
2. "Perhaps you should both go home now." blame the husband for precipitating
3. "Try to think about what precipitated her the wife's behavior, would cause
crying." him to react defensively
4. "The situation is difficult for both of you."
(4) correcttherapeutic; avoids
blaming, focuses on feelings of both
husband and wife
9 In caring for a client with dementia, the Strategy: Don't read into the
2. nurse should give highest priority to which question.
of the following goals?
(1) would take priority if the client
1. Keep the client alive. was experiencing delirium
2. Ensure that the client has an adequate
fluid intake. (2) no information indicating intake
3. Return the client to a functional role in is a problem
the community.
4. Maintain an optimal level of functioning. (3) inappropriate because the client
with dementia is not able to function
at a higher level

(4) correctdementia is
characterized by severe, prolonged
impairment, which is often
irreversible; main focus of care is to
keep client as healthy as possible
for as long as possible
9 The infant of a diabetic mother has a blood Strategy: Determine the action of
3. glucose of 90 mg/dL and a total serum each drug and how it relates to the
calcium level of 7.0 mg/dL. The nurse should lab values.
anticipate that which of the following
medications would be administered IV? (1) would be given for blood sugar
problems
1. Insulin.
2. Glucose. (2) would be given for blood sugar
3. Phenobarbital. problems
4. Calcium gluconate.
(3) not appropriate for a neonate

(4) correcthypocalcemia causes


tetany; calcium gluconate will
replace the calcium
9 A LPN/LVN contacts the nurse to say that Strategy: The topic of the question
4. she has shingles on her back. Which of the is unstated. Read answer choices for
following statements by the nurse is BEST? clues.

1. "You can't take care of clients for 14 (1) staff with localized lesions can
days." care for non-high-risk clients
2. "Come to work as scheduled."
3. "You can't care for clients until the lesions (2) correctable to care for non-
are crusted." high-risk clients; cover lesions
4. "Please contact your physician."
(3) can't care for
immunosuppressed clients until
lesions have crusted

(4) passing the buck


9 A man is hospitalized with a fractured pelvis Strategy: Remember
5. following an automobile accident. A female nurse therapeutic communication.
is administering routine morning care when the
patient tells the nurse that he finds her extremely (1) minimizes comment and
attractive and would like her to come back to visit asks "why", which is
him that evening after work. Which of the nontherapeutic
following responses by the nurse is BEST?
(2) correctuses technique
1. "I appreciate the compliment you paid me, but of acknowledging and
why did you ask me that?" redirecting
2. "That kind of interaction is not appropriate.
Why don't you finish shaving?" (3) observing and clarifying
3. "It is interesting that you feel that way. Do I not appropriate in this
remind you of someone?" situation
4. "You seem to be attracted to me, but I don't
feel the same way." (4) reflecting and rejection
not appropriate in this
situation
9 The mother of a child diagnosed with type 1 Strategy: Answers are a mix
6. diabetes calls to discuss the child's self- of assessments and
monitoring blood glucose (SMBG) home readings. implementations. Does this
The child is being tightly regulated with a situation require validation?
combination of NPH and regular insulin before Yes.
breakfast and supper. The past two mornings, the
blood sugar readings were 220 mg/dL and 210 (1) implementation; is
mg/dL. The nurse should advise the mother to showing hyperglycemia,
take which of the following actions? should try to find reason and
prevent problem
1. Continue the child's medication regime.
2. Check the blood sugar during the night. (2) correctassessment; may
3. Give the NPH insulin later in the evening. be having rebound
4. Serve the bedtime snack earlier in the evening. hyperglycemia (Somogyi
effect) following
hypoglycemia episode while
sleeping
(3) implementation; assumes
that blood sugar is elevated
all night; don't take action
until cause determined

(4) implementation; must


first determine cause of
elevated readings; physician
could increase amount of
bedtime snack to prevent
Somogyi effect
9 A mother tells the nurse the 7-year-old sibling of Strategy: Answers are a mix
7. her child diagnosed with cystic fibrosis is having of assessments and
difficulty in school, fights frequently with implementations. Does this
playmates, and throws toys. Which of the situation require assessment?
following is the BEST response by the nurse? Yes. Is there an appropriate
assessment? Yes.
1. "Did your child have these behaviors before his
sister was diagnosed?" (1) correctassess for a
2. "That is typical of 7-year-olds." cause of the disruptive
3. "Spend time with each child daily and it will behavior
stop."
4. "He is jealous of the attention his sister is (2) dismisses parent's
receiving." concern

(3) implementation; nurse


must assess behavior prior to
his sibling's illness

(4) nurse must assess


behavior prior to his sibling's
illness
98. The nurse admits a client with possible Strategy: Answers are a mix of
Haemophilus influenzae meningitis. It is assessments and implementations.
MOST important for the nurse to take Is the assessment appropriate? No.
which of the following actions? Determine the outcome of the
implementations.
1. Place the client on airborne precautions
for 24 hours. (1) implementation; clients with
2. Perform neurological checks every four meningitis are placed on droplet
to six hours. precautions for at least 24 hours
3. Dim the lights in the room, and minimize
environmental stimuli. (2) assessment; although these
4. Encourage PO fluids during the day to assessments are important, they
decrease fever. should be done more frequently
(3) correctimplementation; will
minimize the likelihood of seizures

(4) implementation; many clients will


be on fluid restriction due to
increased intracranial pressure
99. The nurse assesses a child diagnosed with Strategy: Determine the significance
cystic fibrosis. The nurse is MOST of each answer choice and how it
concerned if which of the following is relates to cystic fibrosis.
observed?
(1) correctis indicative of
1. The child is expectorating thick, yellow pneumonia
mucus.
2. There is increased mucus production (2) increased mucus drainage is the
with postural drainage. purpose of postural drainage
3. Exertional dyspnea increases during the
day. (3) is not unusual for a child with
4. The child complains about difficulty cystic fibrosis
breathing.
(4) is not unusual for a child with
cystic fibrosis
10 The nurse assesses the daily lab reports Strategy: Determine the significance
0. for a patient with a long history of cirrhosis of each assessment and how it
with acute hepatic encephalopathy. Which relates to hepatic encephalopathy.
of the following findings indicates to the
nurse that the patient is improving? (1) normal FBS 70-120 mg/dL,
indicates glucose metabolism;
1. The patient's fasting blood sugar usually altered with diabetes
decreased from 100 to 90 mg/dL.
2. The patient's prothrombin time (PT) (2) normal PT 11-15 seconds,
increased from 20 to 25 seconds. indicates blood coagulation; usually
3. The patient's ammonia level decreased altered with cirrhosis or
from 160 to 120 g/dL. anticoagulant medications, would
4. The patient's AST (SGOT) increased from decrease if improving
24 to 30 Units.
(3) correctindicates a decrease in
ammonia; normal ammonia 80-110
g/dL

(4) normal 4-20 Units, indicates liver


damage; usually altered with acute
pancreatitis and cirrhosis, indicates
increased hepatic cell damage
10 The nurse assesses the emotional Strategy: Think about what information is
1. support available to a client who is being asked for in each assessment
starving herself. Which of the question. Does it fit the situation?
following questions is MOST
important for the nurse to ask in the (1) important question to ask during the
assessment interview? assessment but is not directly related to
the issue of emotional support
1. "What do you consider your ideal
weight to be?" (2) important question to ask during the
2. "How does your eating pattern assessment but is not directly related to
change when you are around other the issue of emotional support
people?"
3. "What happens at home when you (3) correctis the question most likely to
express opinions that are different get at the ability of the parents to support
from those of your parents?" the emotional needs of their children as
4. "What do you think about your separate people
present weight?"
(4) important question to ask during the
assessment but is not directly related to
the issue of emotional support
10 The nurse believes a coworker is Strategy: All answers are assessment.
2. diverting narcotics for personal Determine how each relates to the
consumption. The nurse approaches situation.
the nurse manager to report the
suspicions. Which of the following (1) correctreport objective information
statements by the nurse is BEST? that can be verified; clues to possible
substance abuse by staff include memory
1. "After my coworker has been on lapses, frequent absences from the floor,
duty, the patients often need increased number of clients reporting
repeated doses of pain medication. I unrelieved pain or insomnia
have seen her/him sleeping on duty
three times." (2) subjective observation
2. "I saw my coworker downtown
after work. She/he was acting really (3) subjective observation
strange, like she/he didn't even
recognize me." (4) "hanging around with drug dealers" is
3. "I think my coworker is stealing subjective
narcotics because she/he is always
acting euphoric and seems high."
4. "My coworker is hanging around
with drug dealers, and I think I saw
tracks on her/his arms."
10 The nurse cares for a 4-year-old child Strategy: "Nurse would be reassured"
3. diagnosed with a closed head injury. indicates a positive outcome.
The nurse is reassured by which of
the following observations? (1) correctbeing able to state one's name
demonstrates orientation to person,
1. The child is able to state his name positive sign with head trauma
when asked who he is.
2. The child reaches for a stuffed (2) an act that even a disoriented child
animal brought from home. may perform
3. The child maintains himself in
opisthotonos. (3) indicates meningeal irritation
4. The child withdraws from mildly
painful stimuli. (4) may be observed in an unconscious
client
10 The nurse cares for a 23-year-old Strategy: Determine how each answer
4. college student from China awaiting relates to pain.
arthroscopic knee surgery. Which of
the following observations suggests (1) would expect diaphoresis
to the nurse that the client is
anxious? (2) correctnurse should assess for
nonverbal cues such as increased
1. The client's skin is cool and dry. restlessness, aggressive behaviors
2. The client shifts his position in bed
frequently. (3) would expect increase in pulse and BP
3. The client's pulse is 74 and BP is with anxiety
104/66.
4. The client answers questions (4) does not indicate anxiety
appropriately.
10 The nurse cares for a child after a Strategy: Determine the significance of
5. tonsillectomy. The nurse is MOST each answer choice and how it relates
concerned if which of the following is to a tonsillectomy.
observed?
(1) normal rate for an 8-year-old
1. Heart rate of 88 beats per minute.
2. Expectorating bright red secretions. (2) correctsecretions that are bright
3. Thirty milliliters of dark brown red indicate hemorrhage
secretions.
4. Infrequent swallowing. (3) would be expected because of the
surgical procedure

(4) would be expected after the


surgery because of discomfort
10 The nurse cares for a client after a Strategy: "MOST concerned" indicates
6. thyroidectomy. The nurse is MOST a complication.
concerned if which of the following is
observed? (1) correctpositive Trousseau's sign,
indicates tetany; surgery may damage
1. Tension and muscle spasm of the hand parathyroid glands and cause a
when a blood pressure cuff is applied to decrease in serum calcium
the arm and inflated.
2. Absence of facial movement when the (2) negative Chvostek's sign, positive
muscles of the facial nerve or branches sign indicates low serum calcium
of the nerve are tapped.
3. Pain in the neck when pulling self to a (3) expected after surgery in neck area
sitting position or with sudden head
movements. (4) expected; may be caused by blood
4. Blood pressure readings that remain loss during surgery
10 points below the preoperative
readings.
10 The nurse cares for a client after dental Strategy: Determine how each
7. surgery. The dentist prescribes ibuprofen statement relates to ibuprofen.
(Motrin IB) 600 mg PO. The nurse is
MOST concerned if the client makes (1) correctside effects include
which of the following statements? epigastric distress, nausea, occult
blood loss, peptic ulceration; use
1. "I was treated for a peptic ulcer two cautiously with history of previous
years ago." gastrointestinal disorders
2. "I had a transurethral resection of the
prostate (TURP) last year." (2) medication not contraindicated
3. "I attend Weight Watchers."
4. "I have been having problems with (3) medication not contraindicated
gout."
(4) medication not contraindicated
10 The nurse cares for a client diagnosed Strategy: Answers are a mix of
8. with a CVA with right-sided paralysis. It assessments and implementations. Is
is MOST appropriate for the nurse to the assessment appropriate? Yes.
take which of the following actions?
(1) not required unless complications
1. Insert a Foley catheter. occur
2. Assist the client to ambulate three
times per day. (2) need to assess first
3. Determine if assistance is needed with
feeding. (3) correctdifficulty eating causes the
4. Position the client on the right side. CVA client severe anxiety

(4) positioning stipulations are not


required unless complications occur
10 The nurse cares for a client Strategy: All answers are implementations.
9. diagnosed with dissociative Determine the outcome of each answer
disorder. Which of the following is choice. Is it desired?
a priority nursing action?
(1) reasonable nursing goal that must be
1. Assist the client to understand developed within a multidisciplinary approach
the relationship between anxiety in collaboration with the client after safe
and dissociation. relationship is established
2. Assess the client's level of and
reason for memory loss. (2) reasonable nursing goal that must be
3. Assist the client to incorporate developed within a multidisciplinary approach
the dissociated material into in collaboration with the client after safe
conscious memories. relationship is established
4. Establish an honest,
nonjudgmental, and safe (3) reasonable nursing goal that must be
relationship with the client. developed within a multidisciplinary approach
in collaboration with the client after safe
relationship is established

(4) correctclient is protecting himself/herself


from intense levels of anxiety that are beyond
his/her coping abilities; client needs a safe,
nonthreatening approach because he/she
typically exhibits difficulty in establishing
trusting relationships; if the client begins to
feel "rushed," his/her anxiety level may rise,
producing a potential for further dissociation
11 The nurse cares for a client Strategy: Determine what the problem is in
0. immediately after an abdominal the stem (shock). Determine how each answer
aortic aneurysm repair. Vital signs relates to shock.
are blood pressure 100/70, pulse
120, respirations 24, urine output (1) not a priority
75 ml during the past three hours.
Which of the following is a priority (2) an EKG will not determine cause of
nursing action(s) for this client? tachycardia

1. Weigh the client. (3) rate of IV fluids will most likely be


2. Obtain an EKG. increased
3. Decrease the rate of the IV
fluids, and start nasal oxygen. (4) correctclient is at increased risk for
4. Maintain bedrest, and evaluate development of hypovolemic shock; vital
for a decrease in CVP readings. signs and urine output correlate with the early
signs of shock; the nurse should compare the
CVP with previous readings
11 The nurse cares for a client in Strategy: All answers are implementations.
1. active labor. As labor progresses, Determine the outcome of each answer. Is it
the client becomes irritable and desired?
complains of feeling increasingly
uncomfortable. The nurse notes (1) unnecessary, situation as described is
that the client is 8 cm dilated. normal
Which of these actions should the
nurse take FIRST? (2) correctirritability and discomfort
expected occurrences
1. Notify the physician of the
patient's complaints. (3) not first action; medications may depress
2. Coach the patient in proper infant's respirations after delivery; shouldn't
breathing and relaxation give medication during transition
techniques.
3. Administer the standing order (4) situation as described is normal, not result
for meperidine (Demerol). of placement of fetal monitor
4. Reposition the fetal monitor to
allow the patient to change
positions.
11 The nurse cares for a client in Strategy: Think about the action of each
2. alcohol withdrawal. The nurse medication.
expects the physician to order
which of the following oral (1) antidepressant
medications to assist the client in
decreasing the severity of the (2) antidepressant
symptoms?
(3) antipsychotic medication
1. Amitriptyline (Elavil).
2. Trazodone (Desyrel). (4) correctantianxiety medication is
3. Fluphenazine (Prolixin). pharmacologically similar to alcohol; is used
4. Chlordiazepoxide (Librium). effectively as a substitute for alcohol in
decreasing doses to comfortably and safely
withdraw a client from alcohol dependence
11 The nurse cares for a client in Strategy: Think about how each answer
3. hypovolemic shock. Which of the choice relates to adequate fluid volume.
following indicates a therapeutic
response to volume replacement? (1) correctprimary objective of fluid
replacement is to perfuse vital organs;
1. Urine output increases to 40 ml/hour. increase in urine output to a normal
2. Blood glucose of 180 mg/dL, serum range indicates that kidneys are
potassium of 4.0 mEq/L. adequately perfused; other major
3. CVP of 5 cm water, pupils equal and organs are being perfused also
reactive.
4. Pulse rate of 110 with no (2) does not give any indication of
dysrhythmias. adequate fluid replacement

(3) CVP is an indicator of fluid balance;


CVP of 5 cm water is in the low range
and does not indicate adequate tissue
perfusion

(4) client is tachycardiac; absence of


dysrhythmias does not indicate tissue
perfusion
11 The nurse cares for a client in the clinic. Strategy: Determine the cause of each
4. The physician's orders read: "sulindac answer and how it relates to Clinoril.
(Clinoril) 200 mg PO bid for 14 days."
The nurse should instruct the client to (1) side effect but not most important
report which of the following symptoms
IMMEDIATELY to the physician? (2) not side effect
1. Nervousness. (3) correctshould notify physician if
2. Photophobia. easy bruising or prolonged bleeding
3. Ecchymosis of the extremities. occurs
4. Slight edema of the feet.
(4) does cause sodium retention, but
not most important
11 The nurse cares for a client receiving Strategy: Determine how each
5. chemotherapy. The client has a WBC assessment relates to a low white
count of 1,200/mm3. Which of the count.
following nursing actions should the
nurse take FIRST? (1) correctimportant to monitor for
infection, which would be evidenced by
1. Check temperature q4h. an elevated temperature in a client with
2. Monitor urine output. a low WBC
3. Assess for bleeding gums.
4. Obtain an order for blood cultures. (2) important because of problems of
increased uric acid excretion from
chemotherapeutic drugs but should not
be done first

(3) would be associated with a low


platelet count

(4) would be done if the temperature


were elevated to determine the type of
organism involved
11 The nurse cares for a client receiving Strategy: Answers are a mix of
6. total parenteral nutrition. Lab values are assessments and implementations.
glucose 72 mg/dL, chloride 98 mEq/L, Does this situation require validation?
sodium 138 mEq/L, potassium 3.0 mEq/L. No. Determine the outcome of the
Which of the following nursing actions is implementations.
MOST appropriate?
(1) does not address the problem of
1. Discontinue the TPN administration. hypokalemia; will cause hypoglycemia
2. Notify the physician.
3. Administer IV glucose. (2) correctnormal plasma potassium
4. Check the client's vital signs. level is 3.5-5.0 mEq/L; this client is low
and needs replacement

(3) does not address the problem of


hypokalemia

(4) situation does not require validation


11 The nurse cares for a client receiving Strategy: Determine how each answer
7. treatment for hypoparathyroidism. The relates to hypoparathyroidism.
nurse determines that treatment is
successful if which of the following is (1) important to monitor but is not top
observed? priority

1. The client's output is 1,500 ml of (2) confusion and decreased memory are
clear, straw-colored urine. symptoms of hypercalcemia
2. The client is unable to state his
name. (3) correcttetany is major sign of
3. The client denies numbness and hypoparathyroidism
tingling.
4. The client loses 3 lb in 1 week. (4) most frequently observed with
hyperparathyroidism
11 The nurse cares for a client recovering Strategy: Determine what type of diet is
8. from lower bowel surgery. The nurse required. Select the menu that reflects
determines that teaching is successful the diet.
if the client selects which of the
following menus? (1) contains a high-residue food

1. Milk, green beans, whole-wheat (2) contains a high-residue food


bread.
2. Creamed chicken soup, broccoli, (3) correctlow-residue diet will leave a
pudding. relatively small amount of residue, or
3. Baked chicken, buttered rice, plain indigestible material, in the colon; all
gelatin. meats, fish, and poultry must be broiled
4. Cabbage salad, fried chicken, or baked
applesauce.
(4) contains a high-residue food
11 The nurse cares for a client recovering Strategy: All answers are
9. from streptococcal pneumonia who has implementations. Determine the outcome
a chest x-ray that reveals a higher of each answer choice. Is it desired?
degree of atelectasis in the right lower
lobe. Which of the following nursing (1) would not be as effective as answer
interventions is MOST appropriate? choice 3

1. Instruct the client to take deep (2) would actually decrease thoracic
breaths more frequently. expansion of the chest wall on the right
2. Reposition the client every hour to side
the right side.
3. Increase the frequency of incentive (3) correctincentive spirometry is a
spirometry. quantifiable method of assessing
4. Change respiratory treatment to respiratory effort with deep-breathing
every two hours. exercises; increasing the frequency would
be a sound nursing decision in an effort to
improve the client's pulmonary status

(4) is not the best way to increase


respiratory function
12 The nurse cares for a client scheduled Strategy: Answers are a mix of
0. for surgery. Immediately before assessments and implementations. Is the
transporting the client to the surgical assessment appropriate? Yes.
area, the nurse should take which of
the following actions? (1) assessment; vital signs are checked
prior to surgery but not necessarily at the
1. Check the client's vital signs. time the client is transferred
2. Check the client's identification
bracelet. (2) correctassessment; verifying the
3. Ask the client to sign the operative identity of the client must be completed
permit. before anything else
4. Administer the preoperative
medications. (3) implementation; must be done when
the client is not rushed and has time to
read and ask any questions regarding the
permit before surgery is scheduled

(4) implementation; if the preoperative


medication is not administered prior to
this time, it will not achieve its maximum
effectiveness
12 The nurse cares for a client three Strategy: All answers are implementations.
1. days after an above-knee amputation Determine the outcome of each answer
(AKA). The client complains about choice. Is it desired?
phantom limb pain in the lower leg.
Which of the following nursing (1) majority of clients feel pain for several
responses is BEST? months; minimizes what client is feeling

1. "It should improve within a year." (2) passing the buck


2. "I'll call the physician."
3. "Keep your leg on this pillow." (3) contraindicated after 24 hours because
4. "Staying active will help decrease of possibility of causing contractures
the episodes."
(4) correctactivity helps reduce frequency
and degree of phantom pain
12 The nurse cares for a client who had Strategy: Priority question. Remember
2. a cholecystectomy. Which of the Maslow and the ABCs.
following observations is MOST
important for the nurse to report to (1) psychosocial; not a priority
the next shift?
(2) physical; expected finding after surgery
1. Resting after receiving IM pain due to decrease in peristalsis from
medication. anesthetic agents
2. No bowel sounds present.
3. IV infusing at 100 ml/h. (3) physical; not a priority
4. Breath sounds decreased in both
lower lobes. (4) correctphysical; incision for a
cholecystectomy is high on the abdominal
wall, which inhibits ventilatory movement;
decreased breath sounds might indicate a
complication of pneumonia
12 The nurse cares for a client with a Strategy: All answers are implementation.
3. complete heart block. The nurse Determine the outcome of each answer. Is
should question which of the it desired?
following orders?
(1) correctin complete heart block, the AV
1. Administer lidocaine (Xylocaine) node blocks all impulses from the SA node,
50 mg IV push for PVCs in excess of so the atria and ventricles beat
six per minute. independently; because lidocaine
2. Administer atropine sulfate suppresses ventricular irritability, it may
(Atropine) 0.05 mg IV for diminish the existing ventricular response;
symptomatic bradycardia. cardiac depressants are contraindicated in
3. Anticipate scheduling the client for the presence of complete heart block
a temporary pacemaker if the pulse
continues to decrease. (2) appropriate treatment
4. Mix 10 ml of 1:5,000 solution of
isoproterenol (Isuprel) in 500 ml D5W (3) appropriate treatment
for sustained bradycardia below 30.
(4) appropriate treatment
12 The nurse cares for a client with a Strategy: All answers are implementations.
4. long leg cast on the right leg. The Determine the outcome of each answer. Is
nurse notes that the right foot is pale it desired?
and cool to the touch, and the client
continues to complain of pain even (1) inappropriate response to the
though an analgesic was symptoms observed
administered 45 minutes ago. What
is the FIRST action the nurse should (2) although no time frame for
take? administration is specified, it is not likely
that the analgesic was ordered q 45
1. Apply a heating pad to the client's minutes; is only palliative
right toes.
2. Repeat the dose of the analgesic (3) is the action that probably will be done
stat. by the physician
3. Remove the cast immediately.
4. Notify the physician immediately. (4) correctsymptoms of compartmental
syndrome; document observations and
secure physician's intervention immediately
12 The nurse cares for a client with a radium Strategy: Answers are a mix of
5. implant. It is MOST important for the nurse to assessments and
take which of the following actions? implementations. Is the
assessment appropriate? No.
1. Evaluate the position of the applicator Determine the outcome of each
every two hours. implementation.
2. Place the client on a low-residue diet to
decrease bowel movements. (1) assessment; should be
3. Encourage the use of the bedside commode checked every eight hours
every one to two hours.
4. Decrease fluid intake to decrease radiation (2) correctimplementation;
in the bladder. bowel movements can dislodge
radium implant; this diet will
decrease amount of stool and
number of bowel movements

(3) implementation; client is on


strict bedrest

(4) implementation; decreasing


fluids will not alter exposure to
radiation; client should have a
high fluid intake
12 The nurse cares for a client with a three- Strategy: All answers are
6. chamber water-seal drainage system (Pleur- implementations. Determine the
evac). While the nurse assists the client from outcome of each answer choice.
the bed to the chair, the drainage tubing Is it desired?
becomes disconnected from the Pleur-evac.
The nurse should take which of the following (1) correctprevents air from re-
actions? entering the pleural space

1. Insert the tubing in a container of sterile (2) clamping tube alters the
saline solution. pressure in the pleural space
2. Cut the tubing 2 in from the end, and clamp
securely. (3) must maintain sterility of
3. Reconnect the tubing to the Pleur-evac equipment
container.
4. Connect the tubing to a new Pleur-evac (4) need to do something while
container. additional equipment is obtained
12 The nurse cares for a neonate diagnosed with Strategy: Determine how each
7. an infection. The nurse is MOST concerned if answer relates to neonatal
which of the following is observed? infection.

1. Heart rate of 150 bpm. (1) is within the normal range


2. Axillary temperature of 96F (35.5C).
3. Weight increase of 4 oz. (2) is not significant
4. Respiratory rate of 65 at rest.
(3) neonates normally experience
a 5-10% gain of weight within the
first few days of life
(4) correctnormal respiratory
rate of a neonate is 30-50;
tachypnea is a sign of sepsis or
hypoxia in a neonate
12 The nurse cares for a patient admitted to the Strategy: Determine what each
8. unit three days ago with deep partial thickness assessment is measuring and
and full thickness burns over 30% of the body. It how it relates to burns.
is MOST important for the nurse to report which
of the following observations to the next shift? (1) indicates adequate fluid
resuscitation
1. CVP reading of 12 cm water pressure.
2. General muscle weakness and lethargy. (2) correctmuscle weakness
3. Heart rate of 100 beats per minute. and lethargy are signs of
4. Systolic blood pressure of 105. hypokalemia, which can occur
on the third day after a burn;
hypokalemia is caused by
diuresis

(3) indicates adequate fluid


resuscitation

(4) indicates adequate fluid


resuscitation
12 The nurse cares for a patient after a Strategy: Determine the
9. craniotomy. The patient's history reveals breast significance of each answer
cancer with metastatic lesions to the brain, and choice and how it relates to a
the patient has received chemotherapy for one craniotomy.
month. Postoperatively, the nurse is MOST
concerned if which of the following is observed? (1) indicates either dehydration
or infection
1. Urine is foul smelling, and the urine specific
gravity is 1.035. (2) intake is normal
2. The client's 24-hour fluid intake is 3,000 ml.
3. The client's 24-hour urinary output is 4,000 (3) correctindicates surgically
ml. induced diabetes insipidus;
4. The client has diarrhea and excoriation of the increased urine output with
anal area. pale-colored urine and low
specific gravity

(4) is expected with a client in


chemotherapy but is not as
high a priority as answer choice
3
13 The nurse cares for a patient after a surgical Strategy: Determine how each
0. procedure. The patient is a 23-year-old answer choice relates to pain.
exchange student from Japan. Which of the
following observations by the nurse suggests (1) would expect BP and pulse
the patient is experiencing pain? to increase in response to pain

1. The patient's pulse is 74, BP 104/66. (2) no relationship with


2. The patient's dressing has a small amount of drainage from incision and pain
serosanguinous drainage.
3. The patient repeatedly rubs his hands (3) correctnurse should
together. assess for nonverbal cues to
4. The patient's skin is cool and dry. pain, such as increased
confusion, restlessness,
aggressive behaviors

(4) would expect patient to be


diaphoretic in response to pain
13 The nurse cares for a patient following Strategy: Answers are a mix of
1. an appendectomy. Several hours after assessments and implementations.
surgery, the nurse finds the knee gatch Does this situation require assessment?
on the patient's bed elevated. The Yes.
patient says it feels better with the
knee gatch elevated. The nurse should (1) correctpatient indirectly indicates
take which of the following actions? that he has pain; assess before
implementing action
1. Check to see when the patient last
received pain medication. (2) contraindicated, would compress
2. Lower the knee gatch, and place two blood vessels behind knee
pillows behind his knees.
3. Check to make sure that the knee (3) knee gatch should not be used,
gatch is not elevated more than 20. would compromise circulation and
4. Help the patient turn on the side, and predispose to thrombus formation
support the back with blankets.
(4) assessment should be done before
implementation
13 The nurse cares for a patient following a Strategy: Remember the ABCs.
2. right adrenalectomy. During the
immediate postoperative period, it is (1) severity of this complication is not as
MOST important for the nurse to life-threatening as that of shock
observe for which of the following?
(2) severity of this complication is not as
1. Fluid and electrolyte imbalance. life-threatening as that of shock
2. Temperature fluctuation.
3. Respiratory atelectasis. (3) severity of this complication is not as
4. Blood pressure alteration. life-threatening as that of shock

(4) correctdecrease in blood pressure


may indicate shock
13 The nurse cares for a patient one day Strategy: All answers are
3. after a thoracotomy. The patient is implementations. Determine the
receiving 40% humidified oxygen. outcome of each answer choice. Is it
Arterial blood gas (ABG) results are desired?
PaO2 90 mm Hg, PaCO2 49 mm Hg, pH
7.30, HCO3 26 mEq/L. Which of the (1) correctclient is experiencing
following nursing actions is BEST? respiratory acidosis from decreased
ventilation; increasing quality of
1. Position in high Fowler's, and ventilation by removing secretions may
encourage coughing and deep resolve the problem
breathing; evaluate airway patency.
2. Place in prone position, and request (2) is used for chronic airway problems
respiratory therapy to perform postural
drainage and percussion therapy. (3) oxygen levels are within normal
3. Call the physician to advise about the range; need to take action to improve
arterial blood gas report; anticipate ventilation before notifying the
increase in oxygen percentage. physician
4. Administer antianxiety agent, and
assist the patient with a rebreathing (4) treatment for respiratory alkalosis
device to increase oxygen levels.
13 The nurse cares for a patient three days Strategy: Answers are a mix of
4. after a spinal cord injury at the level of assessments and implementations.
T5. The patient complains of a pounding Does this situation require assessment?
headache, and the nurse notes profuse Yes. Is the assessment appropriate? Yes.
sweating on the patient's forehead.
Which of the following actions should (1) autonomic dysreflexia may be
the nurse take FIRST? precipitated by a full bladder; life-
threatening complication characterized
1. Determine the patency of the Foley by severe hypertension; must be
catheter. treated promptly to prevent a stroke,
2. Place ice packs on the neck and head. but this is the second action that the
3. Elevate the head of the bed. nurse should take
4. Apply a rigid cervical collar.
(2) client will need to be kept warm

(3) correctelevate head of bed to


lower blood pressure

(4) does not address the immediate


situation
13 The nurse cares for a patient who is Strategy: Answers are a mix of
5. lethargic but responsive to verbal assessments and implementations.
commands. The nurse now observes that the Does this situation require
client is responding to noxious stimuli by validation? No. Determine the
withdrawing. Which of the following actions outcome of the implementations.
by the nurse is MOST appropriate?
(1) validation not required
1. Reassess the client in one hour.
2. Notify the physician. (2) correctwithdrawing from pain
3. Place the client in Trendelenburg's is a sign of deterioration in client's
position. condition; physician should be
4. Contact the family. notified

(3) will increase the cranial


pressure

(4) physician should be notified


immediately
13 The nurse cares for a patient with a head Strategy: All answers are
6. injury. Appropriate nursing interventions for implementations. Determine the
minimizing the risk of increasing intracranial outcome of each answer choice. Is
pressure include which of the following? it desired?

1. Maintain a liquid diet, perform frequent (1) tracheal suctioning should be


tracheal suctioning, and turn the client done only as necessary because it
every two hours. will increase intracranial pressure
2. Keep the head of the bed flat, turn the
client every two hours, and perform (2) tracheal suctioning should be
nasotracheal suctioning every hour. done only as necessary because it
3. Maintain the head of the bed elevated at will increase intracranial pressure
90, keep the room dark and quiet, and
place the call light within easy reach. (3) is appropriate only if client is
4. Keep the patient's head from flexing or awake and alert
rotating, elevate the head of the bed 30,
and avoid frequent suctioning. (4) correctobjective is to increase
venous return and decrease
cerebral edema
13 The nurse cares for a teenaged boy in Buck's Strategy: Answers are a mix of
7. traction. It is MOST important for the nurse assessments and implementations.
to take which of the following actions? Is the assessment appropriate?
Yes.
1. Check the pin sites for bleeding or
infection. (1) Buck's traction is a type of skin
2. Apply topical or antibiotic ointment as traction; there are no pins
ordered.
3. Assess that the elastic bandages are not (2) Buck's traction is a type of skin
too loose or too tight. traction; there is no need for
4. Remove the bandages daily to lubricate topical ointment
the skin.
(3) correctnurse needs to assess
the client to make sure circulation
is not being compromised
(4) skin is not lubricated under the
bandages
13 The nurse cares for a woman with Strategy: "MOST concerned"
8. pregnancy-induced hypertension (PIH) indicates a complication.
treated with magnesium sulfate. The nurse
is MOST concerned if which of the following (1) correctis metabolized and
is observed? excreted by the kidneys; decrease
in the urine output can lead to
1. Urine output decreased from 70 to 30 toxicity
ml/hour.
2. Respiratory rate increased from 14 to (2) is a normal finding
18/minute.
3. Hypertonic patellar reflexes. (3) suggests magnesium deficiency
4. Blood pressure increased from 150/90 to
170/100. (4) suggests magnesium deficiency
13 The nurse cares for clients at a student Strategy: Determine how each
9. health clinic at a large university. Which of answer choice relates to cocaine
the following signs and symptoms should abuse.
cause the nurse to suspect cocaine abuse in
a college student? (1) suggests viral infection or
allergic rhinitis
1. Frequent sneezing, complaints of a sore
throat, and a temperature of 100F (37.8C). (2) could indicate gastrointestinal
2. Diarrhea, vomiting, and abdominal pain. problem or substance withdrawal
3. Fatigue, dilated pupils, and anorexia.
4. Complaints of insomnia, rhinorrhea, and (3) could indicate any type of
facial pain. substance abuse or other illness

(4) correctassociated with cocaine


use by inhalation; nose is most
common route for administration
14 The nurse cares for clients in a Strategy: Answers are a mix of
0. rehabilitation facility. The nursing team assessments and implementations.
reports a client recovering from a hip Does this situation require
fracture has repeatedly "transferred herself assessment? Yes. Is there an
to the floor." Which of the following actions, appropriate assessment? Yes.
if taken by the nurse, is BEST?
(1) implementation; assumes that
1. Place the call light within the client's client can't reach the call light
reach.
2. Remove the footrests from the (2) implementation; assumes that
wheelchair. client is tripping on the footrest
3. Observe the client rise from a sitting to a
standing position. (3) correctassessment; nurse can
4. Place a Posey vest restraint on the client. determine if client is safe to
perform this activity

(4) implementation; must exhaust


all other interventions before
restraining client
14 The nurse cares for clients in a Strategy: Remember the steps of
1. rehabilitation facility. To support the client the nursing process.
learning self-care, the nurse should take
which of the following actions FIRST? (1) appropriate; but first establish
goal
1. Provide instructions to complete an
activity. (2) appropriate; but first establish
2. Observe client progress with an activity. goal
3. Establish the goal of the activity with the
client. (3) correctclient commitment to
4. Allow the client to complete as much self- completing or learning techniques
care as desired. for self-care is supported by
participation in goal-setting

(4) appropriate; assess and plan


before implementing
14 The nurse cares for clients in the Strategy: All answers are
2. outpatient clinic. After instilling implementations. Determine the
atropine sulfate (Isopto Atropine) outcome of each answer choice. Is it
eyedrops, the nurse should instruct the desired?
client to take which of the following
actions? (1) correctholding pressure on the
inner canthus decreases the amount of
1. Hold pressure on the inner canthus medication absorbed systemically
for one minute.
2. Keep the eyes opened, and blink (2) not relevant to this medication
frequently to disperse the medication.
3. Roll the eyes in all directions to (3) not relevant to this medication
enhance the action of the medication.
4. Close the eyes tightly to prevent (4) not relevant to this medication
leakage of the medication.
14 The nurse cares for clients in the Strategy: "MOST concerned" indicates
3. prenatal clinic. The nurse is MOST you are looking for a complication.
concerned if a diabetic client in the
third trimester makes which of the (1) correctplacenta produces
following statements? hormones that make the cells insulin-
resistant; as pregnancy progresses,
1. "I am taking less insulin now than I these hormones increase; if insulin
did two months ago." requirement is decreased, this indicates
2. "I am eating a large bedtime snack." that the placenta is not functioning
3. "I walk 15 minutes after lunch every appropriately
day."
4. "I check my blood sugar two hours (2) recommended to prevent
after each meal." hypoglycemia and starvation ketosis
during the night

(3) best time for exercise is after meals


when blood glucose is rising

(4) postprandial measurements done to


prevent hyperglycemia; also check
blood sugars before meals and at
bedtime
14 The nurse cares for clients in the same- Strategy: Determine the significance of
4. day surgery unit. It is MOST important each statement and how it relates to
for the nurse to further investigate surgery.
which of the following client
statements? (1) correctsome appetite suppressants
resemble amphetamines and may cause
1. "I take an herbal dietary supplement palpitation and hypertension; notify the
to lose weight." physician
2. "I had acute glomerulonephritis 10
years ago." (2) nephritis usually caused by beta-
3. "I perform yoga and walk on my hemolytic strep elsewhere in the body;
treadmill every day." not priority
4. "I took my blood pressure medication
3 hours ago." (3) good health habits

(4) medication for cardiac disease and


hypertension may be taken at least 2
hours before surgery with small sips of
water
14 The nurse cares for patients on the Strategy: Think about how the diseases
5. pediatric unit. An 8-year-old patient are transmitted.
with deep partial thickness and full
thickness burns on the right thigh is (1) infectious disease
admitted. The nurse should assign the
new patient with which of the following (2) correctpatient not infectious;
roommates? lowest risk of cross-contamination

1. A 2-year-old with chickenpox. (3) requires contact precautions; could


2. A 4-year-old with asthma. cause infection in the burn patient
3. A 9-year-old with acute diarrhea.
4. A 10-year-old with methicillin- (4) could cause infection in the burn
resistant Staphylococcus aureus patient
(MRSA).
14 The nurse completes discharge teaching Strategy: Topic of question is
6. for a new mother who is breast-feeding unstated. Read answer choices for
her newborn. Which of the following clues.
statements, if made by the mother to the
nurse, indicates the need for further (1) could be used and does not
teaching? indicate a need for further teaching

1. "I will go to my physician and get fitted (2) could be used and does not
for a diaphragm." indicate a need for further teaching
2. "I will ask my husband to use a
condom." (3) correctthe Pill (oral
3. "I will get a prescription from my contraceptive) suppresses production
physician for the Pill." of breast milk; while breast-feeding,
4. "I will practice abstinence during my another method of contraception
fertile time." should be used

(4) could be used and does not


indicate a need for further teaching
14 The nurse conducts a class at a senior Strategy: "MOST concerned" indicates
7. citizen center on the changes associated something wrong. Think about the
with aging. The nurse is MOST concerned answer choices and how each relates
if a client states which of the following? to aging.

1. "I seem to get colds more often now (1) normal change associated with
than I did years ago." aging process
2. "I'm about an inch shorter now than I
was when I was working." (2) normal change associated with
3. "I don't mind cooking, but eating aging process; occurs due to collapse
doesn't appeal to me much anymore." of vertebral column
4. "I've been sleeping with fewer blankets
over me lately." (3) normal change associated with
aging process; may be due to
depression

(4) correctusually becomes


intolerant to cold
14 The nurse determines which of the Strategy: All answers are
8. following diversional activities is most implementations. Determine the
appropriate for a 10-year-old female client outcome of each answer choice. Is it
recovering from a sickle cell crisis? desired?

1. Walking in the hall 20 minutes twice a (1) will not conserve her much-
day. needed energy
2. Watching the cartoon channel all day.
3. Collecting pictures of favorite stars (2) an isolating activity
from magazines.
4. Putting together large-pieced wooden (3) correctcollecting is an activity
puzzles. that is important to school-aged
children

(4) appropriate for preschool-aged


children
14 The nurse discusses symptoms of the Strategy: "Need for further teaching"
9. onset of labor with a 26-year-old indicates you are looking for an
primipara. Which of the following incorrect response.
statements, if made by the client to the
nurse, indicates the need for further (1) correctusually movement
teaching? decreases with the onset of labor

1. "I will note an increase in fetal (2) indicates rupture of membranes,


movement." symptom of labor
2. "I may feel a gush of fluid run down my
legs." (3) bloody show is symptom of labor
3. "I may see some blood in my vaginal
discharge." (4) symptom of labor
4. "I may experience a low backache."
15 The nurse encounters a client Strategy: Remember therapeutic
0. diagnosed with psychosis coming out communication.
of the room nude. Which of the
following responses by the nurse is (1) inappropriate behavior must be
BEST? explained to client

1. "Come with me. You need to get (2) don't ask "why" questions
dressed."
2. "Why are you coming into the (3) correctidentifies inappropriate
hallway undressed?" behavior and tells client what change must
3. "Being naked in the hallway is take place
inappropriate. Return to your room to
get dressed." (4) yes/no question
4. "Do I need to get a male nurse to
help you get dressed?"
15 The nurse enters the room and Strategy: Answers are a mix of
1. discovers that the client has slurred assessments and implementations. Does
speech, right-sided paralysis, and this situation require assessment? Yes. Is
unequal pupils. Which of the there an appropriate assessment? Yes.
following actions should the nurse
take FIRST? (1) physician will need to be notified after
the nurse completes assessment of vital
1. Call the physician. signs
2. Assess the respiratory status.
3. Determine the level of (2) correctassessing the respiratory
consciousness. status and ensuring the client has an open
4. Perform a complete neurological airway is the appropriate next step
evaluation.
(3) would need to be determined but is not
most appropriate next step

(4) would need to be done but is not most


appropriate next step
15 The nurse evaluates care for a client Strategy: "MOST concerned" indicates an
2. diagnosed with depression. The nurse incorrect action.
is MOST concerned if which of the
following is observed? (1) therapeutic intervention to help the
client learn how to create an environment
1. The LPN/LVN teaches the client conducive to sleep
deep breathing and relaxation
techniques. (2) therapeutic intervention to help the
2. The staff allows the client to client learn how to create an environment
verbalize thoughts when he tries to conducive to sleep
sleep.
3. The staff encourages the client to (3) therapeutic intervention to help the
express his feelings more clearly. client learn how to create an environment
4. The LPN/LVN administers conducive to sleep
flurazepam hydrochloride (Dalmane)
15 mg hs. (4) correctmedication that produces
dependence should be a last resort; used
only if other nursing measures and
antidepressant medications have not
worked and the client is exhausted
15 The nurse evaluates care given to Strategy: "Nurse would intervene"
3. clients by the home health aide. The indicates an incorrect action.
nurse should intervene in which of
the following situations? (1) appropriate action; standard,
unchanging procedure
1. The nursing assistant walks a
client 15 ft with a walker. (2) appropriate action; standard,
2. The nursing assistant uses a spoon unchanging procedure
to feed a client with blindness.
3. The nursing assistant administers (3) correctcare not within the scope of a
a client's medication. nursing assistant
4. The nursing assistant performs
catheter care for a client. (4) appropriate action
15 The nurse evaluates care in the long-term care Strategy: Determine the
4. facility. Which of the following provides the best outcome of each answer
evidence that the nursing intervention to deal choice. Is it desired?
with a client's self-care deficit in relation to
feeding is effective? (1) correctconcrete measure
of the client's eating patterns
1. The client eats at least one-half of all meals indicates adequate intake of a
and drinks a minimum of 2,000 ml/day. well-balanced diet
2. The client's dentures have been replaced, and
he is able to chew. (2) doesn't ensure that client
3. The client will eat without verbalizing is eating
suspicions when a particular nurse sits with him.
4. The client appears to have increased energy (3) indicates that the client is
to complete grooming activities. still experiencing distorted
thinking about the foods he is
to eat

(4) may not be an accurate


measure of adequate nutrition
15 The nurse explains the use of transcutaneous Strategy: "Further teaching"
5. electrical nerve stimulation (TENS) to a client indicates an incorrect
diagnosed with sciatica. Which of the following response.
actions, if performed by the patient, indicates to
the nurse that further teaching is necessary? (1) gel is used; should rotate
sites to prevent irritation of
1. The client applies a conducting gel before skin
applying the electrodes.
2. The client places the electrodes on the side of (2) correctshould be over,
the body opposite from the painful area. above, or below the painful
3. The client turns up the voltage until she/he area
feels a prickly "pins and needles" sensation.
4. The client adjusts the voltage based on the (3) uses battery-operated
relief of pain she/he experiences. device to deliver small
currents to skin and
underlying tissues

(4) used for localized pain,


such as low back pain
15 The nurse finds a client unresponsive and Strategy: Answers are all
6. making funny sounds. The client's arms and legs implementations. Determine
are stiff and jerking, and there is no verbal the outcome of each answer.
response. Which of the following actions should Is it desired?
the nurse take FIRST?
(1) may cause fracture
1. Open the client's mouth, and place a tongue
blade between the teeth. (2) impossible or
2. Position the client on his back, open the inappropriate to carry out
airway, and assess respiratory status. during a seizure
3. Remain with the client, and prevent him from
injuring himself or falling out of bed. (3) correctnurse should
4. Restrain the client's extremities, and remain with client to prevent
determine the neurological status. injury
(4) action is appropriate after
a seizure
15 The nurse in a small town is called to a neighbor's Strategy: Answers are a mix
7. house in the middle of a blizzard. The neighbor of assessments and
woman states that she is at 39 weeks' gestation implementations. Does this
with her second baby and has been having situation require
contractions for several hours. The woman has assessment? Yes. Is there an
been unable to obtain assistance because the appropriate assessment? Yes.
roads are impassable. The nurse assists with the
delivery of the infant. Once the head is delivered, (1) implementation; should
it is MOST important for the nurse to take which bear down with contractions
of the following actions? but not most important

1. Instruct the woman to bear down and push. (2) implementation; head will
2. Turn the infant's head in a clockwise direction. rotate without assistance to
3. Check the infant's neck for the umbilical cord. right or left, should not be
4. Ask the woman to pant through her mouth. turned

(3) correctassessment;
infant could become anoxic if
cord is around neck

(4) implementation; should


bear down with contractions
15 The nurse instructs a client about how to Strategy: All answers are
8. understand and deal with hallucinations. Which of implementations. Determine
the following indicates to the nurse that teaching the outcome of each answer
is successful? choice. Is it desired?

1. The client reports that he is feeling anxious and (1) correctclient is


requests his radio and headphones in anticipation connecting his voices with
of "the voices." feeling of anxiety and is
2. The client sleeps during the day and avoids taking action to get rid of the
going to his assigned activities. voices
3. The client requests PRN medication between
the two regularly scheduled doses. (2) gives no indication of
4. The client reports that he is angry and wishes learning a positive behavior
to leave the hospital immediately.
(3) does not indicate any
level of understanding of the
cause of the symptoms

(4) does not indicate any


level of understanding of the
cause of the symptoms
15 The nurse in the newborn nursery receives Strategy: Eliminate the stable
9. report from the previous shift. Which of patients.
the following infants should the nurse see
FIRST? (1) correctinfant has tachycardia;
normal resting rate is 120-160 bpm;
1. A 2-day-old infant, lying quietly alert, requires further investigation
heart rate of 185 bpm.
2. A 1-day-old infant, crying, and the (2) crying causes increased
anterior fontanel is bulging. intracranial pressure, causes fontanel
3. A 12-hour-old infant held by the mother, to bulge
respirations 45 and irregular.
4. A 5-hour-old infant, sleeping, hands and (3) normal respiratory rate is 30-50
feet are blue bilaterally. bpm with apneic episodes

(4) acrocyanosis is normal for two to


six hours postdelivery due to poor
peripheral circulation
16 The nurse in the outpatient clinic plans Strategy: All answers are
0. care for an older client with left-sided implementation. Determine the
weakness due to a cerebral vascular outcome of each answer. Is it desired?
accident (CVA). The client has a history of
hypertension and osteoporosis. It is MOST (1) diet should have adequate
important for the nurse to encourage the calcium, should increase intake in
client to increase which of the following? middle age to protect against skeletal
demineralization; not most important
1. Calcium in the daily diet.
2. Vitamin D in the daily diet. (2) adequate serum levels of vitamin
3. Time of exposure to sunlight. D needed for calcium to be absorbed
4. Activities that involve weight bearing. from gastrointestinal tract, should
increase intake in middle age to
protect against skeletal
demineralization; not most important

(3) vitamin D is synthesized in the


skin with exposure to sunshine; not
most important for this patient

(4) correctweight bearing and


exercise primary ways to develop
high-density bones, decrease bone
reabsorption and stimulate bone
formation; would also help maintain
mobility with left-sided weakness
16 The nurse in the pediatrician's office Strategy: "MOST appropriate" indicates
1. instructs the parents of a toddler discrimination is required to answer the
about a scheduled magnetic question.
resonance imaging (MRI). The nurse
tells the parents the child should be (1) chloral hydrate causes paradoxical
sedated using chloral hydrate excitation in children; administer at facility
(Noctec) prior to the MRI. The parents where MRI is scheduled
ask if they can administer the
medication at home so that the (2) correctchloral hydrate, a sedative,
toddler will be asleep when they can have the opposite effect on a toddler,
arrive at the hospital. Which of the causing excitability; child should be
following responses by the nurse is continuously monitored after medication is
MOST appropriate? administered

1. "I will ask the physician if it is (3) safety of toddler is most important
permissible."
2. "The medication should be (4) medication should not be given at
administered at the hospital." home prior to coming to facility
3. "The child should be awake when
arriving at the hospital."
4. "Are you sure you can handle your
sedated toddler?"
16 The nurse is caring for a client in a Strategy: All answers are implementations.
2. manic phase of bipolar disorder. It is Determine the outcome of each answer. Is
MOST important for the nurse to offer it desired?
which of the following meals?
(1) correctclients with mania need
1. Tuna salad sandwich and orange nutritious finger foods; foods contain
slices. protein, carbohydrates, vitamin C, and
2. Bologna sandwich and french fries. fiber
3. Milkshake and banana.
4. Fried chicken and tossed salad. (2) finger foods but little nutritive value

(3) finger foods but not as balanced

(4) too difficult to eat in manic phase


16 The nurse is the leader of a group of Strategy: Determine the outcome of each
3. developmentally disabled adults. The answer choice. Think about what the
nurse instructs the group members to words mean.
ignore another client whenever the
client interrupts others who are (1) correctnurse leader will be able to
speaking. To evaluate the progress of measure improvement by counting the
this intervention, the nurse should number of times the client succeeds in
take which of the following actions? controlling his interruptions when others
are speaking; tokens can be awarded for
1. Measure improvement by counting successes and then exchanged for
the number of times the client privileges; tokens would not be given for
succeeds. continued interruptions; belonging to a
2. Measure improvement by counting group and being allowed to go to group
the number of interruptions. sessions can be a reward for the members;
3. Assess the ability of the group to power of group helps decrease socially
control the client's interruptions. unacceptable behavior
4. Count the number of tokens and
earned privileges given for (2) count the successes, not failures
interruptions.
(3) goal is for client to control his/her
behavior

(4) reward positive behavior


16 The nurse knows that which of the Strategy: Think about each answer and the
4. following information in a client's relationship to suicide.
history places the client at greatest
risk for suicide? (1) correctincludes two factors that
increase risk: the previous suicide
1. Two previous suicide attempts and attempts plus the increase in alcohol use
increased use of alcohol.
2. Verbal threats without a specific (2) shows less risk because there is no
plan. specific plan
3. Choice of a method that includes
going to sleep and not waking up. (3) includes a vague plan without a lethal
4. A specific plan but with method
ambivalence verbalized.
(4) shows less risk because the client is
ambivalent
16 The nurse learns that a staff member Strategy: Answers are a mix of
5. providing care to a client diagnosed with assessments and implementations. Is
cytomegalovirus is in early pregnancy. validation required? No. Determine
Which of the following actions, if taken by the outcome of each implementation.
the nurse, is BEST?
(1) no reason to reassign the staff
1. Reassign the pregnant staff member to member
care for other patients.
2. Instruct the staff member to contact (2) passing the buck
her physician.
3. Ask the staff member how she is feeling (3) will discuss nurse's feeling about
about her pregnancy. caring for high-risk clients but is not
4. Ensure that the staff member follows the priority
standard precautions.
(4) correctmake pregnant personnel
aware of the risks; use standard
precautions, do not reassign
16 The nurse monitors a client diagnosed Strategy: "MOST concerned" indicates
6. with cholecystitis. The nurse is MOST a complication.
concerned if which of the following is
observed? (1) symptom of cholecystitis; does not
necessarily indicate a complication
1. Nausea.
2. Frequent belching. (2) symptom of cholecystitis; does not
3. Jaundice. necessarily indicate a complication
4. Right upper abdominal pain.
(3) correctjaundice indicates a
possible stone in the bile duct causing
obstruction

(4) symptom of cholecystitis and does


not necessarily indicate a
complication
16 The nurse observes a student nurse Strategy: "Require an intervention"
7. administer a Dulcolax suppository. Which indicates an incorrect action.
of the following actions, if performed by
the student nurse, requires an (1) correctmay cause rectal muscles
intervention by the nurse? to tighten, should breathe through
mouth to relax
1. The client is instructed to hold his
breath during insertion of the suppository. (2) acts by direct stimulation of
2. The suppository is positioned to touch mucosa, prevents suppository from
the wall of the patient's rectum. being embedded in stool
3. The suppository is inserted 3 to 4
inches into the patient's rectum. (3) placement is correct
4. Lubricant is applied to the tip of the
suppository before insertion. (4) lubricant eases insertion by
preventing friction
16 The nurse observes care given to a client Strategy: "Should intervene"
8. who vigorously follows several rituals daily, indicates that you are looking for
including frequent hand washing. The incorrect behavior. Don't lose the
client's hands are now reddened and question.
sensitive to touch. The nurse should
intervene if which of the following is (1) appropriate nursing action
observed?
(2) appropriate nursing action
1. The staff administers special skin care to
the client. (3) correctwill only increase the
2. The staff gives positive reinforcement for client's anxiety and need for the
nonritualistic behavior. rituals; limits must be gradually
3. The staff limits the amount of time the instituted
client may use to wash hands.
4. The staff protects the client from ridicule (4) appropriate nursing action
by other clients on the unit.
16 The nurse observes the psychiatric staff Strategy: "The nurse would
9. interact with a client exhibiting intervene" means you are looking
manipulative behavior. The nurse should for an incorrect response.
intervene in which of the following
situations? (1) appropriate and effective
strategy for intervening with a
1. The staff discusses with the client the manipulative client
consequences of the manipulative behavior.
2. The staff collaborates to establish limits (2) appropriate and effective
on the manipulative behavior. strategy for intervening with a
3. The staff clarifies the consequences of manipulative client
the client's manipulative behavior.
4. The staff decreases demands placed on (3) appropriate and effective
the client that trigger the manipulative strategy for intervening with a
behavior. manipulative client

(4) correctcan foster a sense of


entitlement along with
underfunctioning; establishing
realistic, achievable goals and
activities is necessary to build self-
esteem
17 The nurse obtains a health history from a Strategy: Determine how each
0. client in the medical clinic. The client answer relates to an ulcer.
states, "I think I have an ulcer." Which of
the following responses by the nurse is (1) correctpeptic ulcer pain is
BEST? often referred to as a "boring pain in
the back" or a "burning, gnawing"
1. "Do you have a burning pain in the feeling in the midepigastric area
epigastric region?"
2. "Do you have sharp pain in your lower (2) may indicate intestinal
abdomen?" perforation
3. "Do you have right shoulder pain with
vomiting?" (3) often associated with
4. "Do you have heartburn when you lie gallbladder disease or with irritation
down?" of the diaphragm, most often
caused by free air in abdominal
cavity (a postoperative
complication)

(4) describes indigestion or possible


hiatal hernia
17 The nurse obtains a health history from a Strategy: Determine the significance
1. client taking phenytoin sodium (Dilantin). of each answer and how it relates to
It is MOST important for the nurse to Dilantin.
report which of the following client
statements to the physician? (1) not relevant to this medication

1. "I've had several 'blackouts' in the past (2) not relevant to this medication
year."
2. "My mother has seizures, and this (3) correctphenytoin sodium
medication does not work for her." (Dilantin) is in pregnancy risk
3. "I don't know when I had my last category D; physician should be
menstrual period." notified of the possibility of a
4. "I took this medicine several years ago pregnancy
but stopped when my urine turned pink."
(4) pink urine is a normal occurrence
when taking Dilantin
17 The nurse obtains a health history from Strategy: Determine how each
2. the mother of a child diagnosed with answer choice relates to failure to
failure to thrive. Which of the following thrive.
assessments provides the MOST pertinent
data to the nurse? (1) correctphysical; provides the
most pertinent data in assessing
1. Weight and height. actual growth
2. Urine output.
3. Type of feedings. (2) physical; indicates hydration
4. Mother/child interactions. status but not most important
assessment

(3) physical; but not most important


assessment

(4) psychosocial; physical takes


priority
17 The nurse on postpartum prepares four Strategy: Eliminate the most stable
3. clients for discharge. It is MOST important patients.
for the nurse to refer which of the
following clients for home care? (1) stable situation, no indication of
problems with mother or baby
1. A 15-year-old who vaginally delivered a
7-lb male 2 days ago. (2) stable situation, no indication of
2. An 18-year-old multipara who delivered problems with mother or baby
a 9-lb female by cesarean section 2 days
ago. (3) stable patient, cramping due to
3. A 20-year-old multipara who delivered 1 uterine contractions
day ago and is complaining of cramping.
4. A 22-year-old who delivered by cesarean (4) correctunstable patient,
section and is complaining of burning on indicates urinary tract infection;
urination. requires follow-up
17 The nurse overhears a conversation in the Strategy: Answers are
4. cafeteria between two nurses regarding a implementations. Determine the
client's home situation. Which of the outcome of each answer choice. Is it
following actions is the MOST appropriate? desired?

1. Report the incident to the nurse (1) may occur, but the situation
manager. requires immediate action that the
2. Join the conversation with the nurses. manager may not be able to provide
3. Suggest that the nurses continue their
conversation in private. (2) does not resolve the problem in a
4. Ignore the incident because the nurse is positive manner
not involved.
(3) correctclient's confidentiality is
being violated; it is nurse's
responsibility to intervene to protect
the client

(4) does not resolve the problem in a


positive manner
17 The nurse performs a diet history on a client Strategy: "Further teaching is
5. diagnosed with AIDS hospitalized for a necessary" indicates an
cytomegalovirus (CMV) infection of the incorrect response.
gastrointestinal tract that has resulted in
diarrhea. Which of the following food choices (1) low residue, contains
indicates a need for the nurse to do further protein
teaching?
(2) low residue, high calorie
1. A cup of beef bouillon, steamed white rice, and
strawberry gelatin. (3) low residue, contains
2. Clear chicken broth, two slices of white toast, protein
and a serving of applesauce.
3. A cup of apple juice, cottage cheese, and three (4) correctcontain caffeine,
unsalted crackers. roughage, and dairy products
4. Plain tea, a fresh fruit salad, and chocolate ice
cream.
17 The nurse performs a health screening at a senior Strategy: All answers are
6. citizen facility. A client has been taking oral iron implementations. Determine
supplements for a month and complains of the outcome of each answer
constipation. The nurse should adapt a diet plan choice. Is it desired?
to include which of the following?
(1) correctcontains foods
1. Oatmeal, green beans, and celery. highest in fiber (green
2. Strawberries and mushrooms. vegetables and grains) to
3. Grits, orange juice, and cheddar cheese. assist in counteracting
4. Pasta, buttermilk, and bananas. constipation

(2) does not have as high a


fiber content

(3) does not have as high a


fiber content
(4) does not have as high a
fiber content
17 The nurse performs a home care visit for the Strategy: Think about what
7. family of a toddler. The nurse is MOST concerned the words mean?
if which of the following is observed?
(1) common assessment
1. A bruise on the toddler's knee. during the toddler years
2. The toddler cries and is fearful when the
parents leave. (2) expected behavior for
3. The toddler's immunizations are not up to date. toddlers
4. The toddler throws a temper tantrum during an
injection. (3) correctmost likely
indicates a lack of concern
for child's well-being and is a
sign of poor quality of home
care

(4) expected behavior for


toddlers
17 The nurse performs a home visit on a Strategy: Determine the outcome of
8. client diagnosed with progressive each answer choice.
multiple sclerosis (MS). The physician
orders cyclophosphamide (Cytoxan) and (1) correctclients receiving
adrenocorticotropic hormone (ACTH). It cyclophosphamide usually develop
is MOST important for the nurse to take alopecia four to five weeks after starting
which of the following actions initially? treatment

1. Advise the client to purchase a wig or (2) should increase fluid intake to
a hairpiece. combat the side effect of hemorrhagic
2. Instruct the client to decrease fluid cystitis
intake.
3. Test the client's serum glucose (3) involves adverse effect of ACTH and
concentration. should be monitored after treatment is
4. Observe for indications of initiated
gastrointestinal bleeding.
(4) involves adverse effect of ACTH and
should be monitored after treatment is
initiated
17 The nurse performs an ice massage for a Strategy: "MOST concerned" indicates a
9. client in chronic pain. The nurse is MOST complication.
concerned if which of the following is
observed? (1) indicates inflammation

1. Redness or inflammation of the (2) correctsite should be observed


tissue. every five minutes for signs of tissue
2. Mottling or graying of the tissue. intolerance, including blanching,
3. The client states that she feels a mottling, or graying
burning and tingling sensation in the
area. (3) usually indicates ischemia or
4. The client states that she feels a sensorineural impairment
numbness and a cold sensation in the
area. (4) expected outcome of numbness,
which would lead to decreased pain
perception
18 The nurse performs dietary teaching for Strategy: Determine the outcome of
0. a client diagnosed with asymptomatic each statement. Is it desired?
diverticular disease. The nurse
determines further teaching is required (1) correcttomatoes have seeds that
if the client states which of the may block the neck of a diverticulum
following?
(2) encouraged for a high-fiber content
1. "I'm glad that I can eat the tomatoes to add bulk to stools
from my garden."
2. "I eat baby carrots as a snack almost (3) encouraged for a high-fiber content
every day." to add bulk to stools
3. "I mix several different kinds of
lettuce for my evening salad." (4) encouraged to eat a diet high in
4. "I only eat whole-wheat bread for my cellulose and hemicellulose types of
lunch sandwich." fiber, found in wheat bran and whole-
wheat bread
18 The nurse performs diet teaching for a Strategy: Type of diet needed by the
1. client with a spinal cord injury at S3. client is unstated. Determine what type
Which of the following meals, if chosen of diet is required and select the
by the client, indicates to the nurse that appropriate menu.
teaching is effective?
(1) should have high-fiber, low-fat diet;
1. Cheeseburger with tomato and onion. this diet is high in fat
2. Spaghetti with meat sauce and green
beans. (2) correcthigh-fiber diet is an
3. Tuna fish sandwich with orange juice. important part of bowel program; fiber
4. Grilled cheese sandwich and helps prevent the complication of
chocolate pudding. constipation; includes whole-grain
foods, bran, fresh and dried fruits;
increased fiber will facilitate defecation,
especially with reduction in fat intake

(3) should increase intake of fiber foods


and decrease intake of fat

(4) should have high-fiber, low-fat diet;


this is a high-fat diet
18 The nurse performs health screening at Strategy: Focus on the question.
2. a shelter for the homeless. Which of the
following nursing observations most (1) indicates the possibility of diabetic
likely indicates the need for teaching acidosis
about personal hygiene?
(2) may be a result of poor fat
1. Fruity breath odor. absorption
2. Foul-smelling stools.
3. Vaginal itching. (3) could be the result of antibiotic
4. Red, swollen gums. therapy and subsequent yeast infection

(4) correctred, swollen gums can


indicate pyorrhea, which is caused by
improper cleaning and poor mouth
hygiene
18 The nurse performs hypertension Strategy: All answers are
3. screening at the local grocery store. It is implementations. Determine the
MOST important for the nurse to outcome of each answer choice. Is it
complete which of the following tasks? desired?

1. Use a blood pressure cuff that (1) unnecessary


overlaps the arm at least 4 inches.
2. Support the client's arm above the (2) arm should be supported at the
level of the heart. level of the heart
3. Take two readings at least five
minutes apart. (3) correctrecognition of adult
4. Take the blood pressure after the hypertension should be done after two
client has exercised for 10 minutes. readings taken at least five minutes
apart

(4) unnecessary
18 The nurse performs teaching for a client Strategy: "Teaching was successful"
4. being discharged on dexamethasone indicates a correct response.
(Decadron) 0.75 mg PO daily. The nurse
determines teaching successful if the (1) contains incorrect information about
client makes which of the following the administration of steroids
statements?
(2) correctoral steroids have
1. "I will take my medication with orange ulcerogenic properties and need to be
juice in the morning." administered with meals; if ordered
2. "I will take my medication with daily, administer in morning
breakfast."
3. "I will take my medication three hours (3) contains incorrect information about
after eating." steroid administration
4. "I will take my medication before I eat
breakfast." (4) contains incorrect information about
steroid administration
18 The nurse performs teaching for a client Strategy: Determine how each answer
5. receiving isoniazid (INH) 300 mg PO relates to INH.
daily. The nurse identifies that teaching
is successful if the client states which of (1) untrue statement
the following?
(2) untrue statement
1. "My urine will turn brown."
2. "I will take this medication for two (3) untrue statement
weeks."
3. "I shouldn't take any other medication (4) correctalcohol consumption while
while taking this drug." on INH therapy has been reported to
4. "I should not drink any alcoholic increase isoniazid-related hepatitis;
beverages." clients should be cautioned to restrict
consumption of alcohol
18 The nurse performs teaching on a Strategy: Answers are all implementations.
6. client diagnosed with Bell's palsy. It Determine the outcome of each answer
is MOST important for the nurse to choice. Is it desired?
include which of the following
instructions? (1) correctparalysis of the eyelid allows
the cornea to dry; patch can be used to
1. Use artificial tears four times per keep the eyelid closed to prevent damage;
day. drops and/or ointments are used to reduce
2. Wear sunglasses at all times. chance of corneal damage
3. Avoid sudden movements of the
head. (2) no problem with intolerance to light
4. Change the pillowcase daily.
(3) is for clients with increased intraocular
pressure

(4) Bell's palsy is not contagious


18 The nurse plans care for a client who Strategy: Answers are implementations.
7. had surgery for an ileal conduit 2 Determine the outcome of each answer. Is it
days ago. It is MOST important for desired?
the nurse to take which of the
following actions? (1) soap and water should be used to clean
the skin, not an antiseptic solution
1. Remove the appliance regularly,
and clean the skin with antiseptic (2) correctprimary preventative measure
solution. to prevent urine from contacting the skin
2. Apply a close-fitting drainage bag
to the stoma. (3) would hinder the application of the bag
3. Massage the skin around the for urine collection
stoma with an emollient.
4. Expose the area around the stoma (4) unnecessary; would not help prevent
to air twice a day. skin breakdown
18 The nurse plans care for a client with Strategy: "Should intervene" indicates a
8. Graves' disease. The nurse should wrong action.
intervene if the client drinks which of
the following fluids? (1) not limited for Graves' disease

1. Milk. (2) not limited for Graves' disease


2. Apples.
3. Orange juice. (3) not limited for Graves' disease
4. Tea.
(4) correctstimulant that would increase
metabolic rate
18 The nurse prepares a client for a Strategy: "MOST important" indicates
9. laparoscopic cholecystectomy for discrimination is required to answer the
treatment of cholelithiasis. It is question.
MOST important for the nurse to ask
which of the following questions? (1) not the priority question; information
required for health promotion
1. "Tell me about your sleep
patterns." (2) correctclient usually discharged the
2. "Who is going to help you at home day of surgery or the next day; ensure that
during the next couple of days?" client has help at home for first 24-48 hours
3. "Have you noticed an intolerance
to fatty foods?" (3) should maintain nutritious diet and
4. "Have you had difficulty avoid excessive fats; more important to
maintaining your weight?" determine if client has help at home

(4) due to fat intolerance, may have lost


weight; if obese, physician may recommend
a weight loss program
19 The nurse prepares a client for an intravenous Strategy: "Teaching has been
0. pyelogram (IVP). Which of the following effective" indicates you are
statements, if made by the client to the nurse, looking for a correct response.
indicates teaching is effective?
(1) these symptoms
1. "I may feel a fluttery sensation when the experienced during cardiac
catheter is inserted." catheterization as catheter is
2. "The test may cause spasms and shooting passed into left ventricle, not
pains in my back." seen with IVP
3. "I may experience a hot feeling, and my skin
may become flushed." (2) does not occur during IVP
4. "I may become light-headed and have a desire
to cough." (3) correctmay be
accompanied by nausea
caused by dye injection

(4) not associated with IVP


19 The nurse prepares a client for insertion of a Strategy: All answers are
1. subclavian triple lumen catheter to be used for implementations. Determine
administration of total parenteral nutrition the outcome of each answer
(TPN). The nurse should position the client in choice. Is it desired?
which of the following positions?
(1) would not promote dilation
1. High-Fowler's position with the client's head in of blood vessels involved
a neutral position.
2. Semi-Fowler's position with the client's head (2) would not promote dilation
extended. of blood vessels involved
3. Supine with the client's head low and turned
away from the insertion site. (3) correctproduces dilation
4. Left lateral with the client's head turned of neck and shoulder vessels,
toward the insertion site. making entry easier and
preventing air embolus

(4) not best position; client


should turn head away from
insertion site
19 The nurse prepares to discharge a client after an Strategy: All answers are
2. abdominal cholecystectomy for treatment of implementations. Determine
cholelithiasis. The client will go home with a T the outcome of each answer
tube in place. Which of the following statements, choice. Is it desired?
if made by the client to the nurse, indicates a
need for further teaching? (1) maintains personal
hygiene
1. "It will be great to finally get home, take a
shower, and wash my hair." (2) increase in bile drainage is
2. "If the amount of drainage increases over the sign of obstruction, should
next several days, I should call my physician." continue to decrease
3. "I can resume swimming laps three times a
week at my health club." (3) correctshould avoid
4. "I will check the skin around the tube once a strenuous exercise and do not
day to see how it is doing." immerse T tube in water

(4) should check for signs of


inflammation
19 The nurse prepares to examine the Strategy: All answers are
3. client's thyroid gland. Which of the implementations. Determine why you
following statements, if made by the would perform each and determine
nurse, is BEST? the outcome.

1. "Would you like a Band-Aid?" (1) not needed, not an invasive


2. "Here is a glass of water." procedure
3. "I will be using this tape measure."
4. "Please use this specimen cup." (2) correctdrinking water facilitates
swallowing during examination of
thyroid

(3) don't measure with a tape measure

(4) no specimen is obtained


19 The nurse prepares to insert a Foley Strategy: All answers are
4. catheter into a patient. It is MOST implementations. Determine the
important for the nurse to take which of outcome of each answer choice. Is it
the following actions? desired?

1. Place all supplies close to the edge of (1) would break sterile technique
the table.
2. Keep the field holding the supplies in (2) correctrepresents the best
front of the nurse. technique for a sterile field
3. Set up the field below the nurse's waist
level. (3) would break sterile technique
4. Add only clean supplies to the field.
(4) supplies should be sterile, would
break sterile technique
19 The nurse questions the family of a Strategy: Think about the cause of
5. patient admitted with hyperglycemic each answer choice and how it relates
hyperosmolar nonketotic syndrome to the situation.
(HHNK). The nurse should expect which of
the following information to be contained (1) usually seen with type 2 diabetes
in the patient's history?
(2) ketosis usually not seen with this
1. The patient was diagnosed with type 1 condition
diabetes four years ago.
2. The patient has a history of 3+ ketones (3) no relationship to weight or
in his urine. smoking
3. The patient is 20 lb overweight and
smokes a pack of cigarettes a day. (4) correctseen after 50 years old;
4. The patient is 66 years old and takes age-related changes in thirst
propranolol (Inderal) 20 mg PO tid. perception result in dehydration and
decrease in urine-concentrating
abilities of the kidney
19 The nurse reading an EKG rhythm strip Strategy: Do the math.
6. determines that there are 8 QRS
complexes in 30 large squares for a 6- (1) inaccurate
second strip. The nurse calculates the
heart rate to be which of the following? (2) inaccurate

1. 60. (3) correct30 large squares on the


2. 70. EKG paper represent 6 seconds;
3. 80. multiply the number of QRS
4. 120. complexes found in 30 large squares
by 10 (8 10 = 80 beats per minute)

(4) inaccurate
19 The nurse receives report about four Strategy: Determine who is the least
7. pregnant women in active labor who have stable client.
been admitted to the labor and delivery
unit. Which of the following women should (1) nullipara women usually have a
the nurse see FIRST? longer second stage than multipara
women
1. A 22-year-old nullipara at term, cervical
dilation of 10 cm, 100% effaced, fetus (2) delivery is not imminent
presenting as left occiput posterior with
short-term variability of the FHT at three to (3) correcttransition phase of labor
five beats. and delivery quick for many
2. A 27-year-old nullipara at 38 weeks' multipara women
gestation has a cervical dilation of 2 cm,
fetus in transverse lie with baseline FHT of (4) labor has not progressed very far
155 bpm.
3. A 32-year-old multipara at term, cervical
dilation of 8 cm, fetus in a vertex
presentation with the presenting part at +2
station.
4. A 34-year-old multipara at 37 weeks'
gestation has intact amniotic membranes,
cervical dilation of 3 cm, and fetus in a
frank breech presentation with the
presenting part at 0 station.
19 The nurse reviews client care Strategy: "Requires additional
8. documentation. Which of the following instruction" indicates incorrect
statements BEST indicates to the nurse charting.
that the staff requires additional
instruction about documentation? (1) correctdocumentation is
subjective
1. "Patient is very sad about the death of
his daughter." (2) quotes patient; correct
2. "Patient states, 'I just can't get over my documentation is complete and
daughter's death.'" objective
3. "Patient frequently verbalizes about his
daughter's death." (3) objective observation
4. "Patient presents a sad face, stooped
posture, and tear-streaked eyes." (4) objective observation
19 The nurse reviews the record for a patient Strategy: All answers are
9. with a chest tube attached to a Pleur-evac implementations. Determine the
system. The nurse evaluates which of the outcome of each answer choice. Is it
following nursing actions is appropriate? desired?
1. "Chest tube was clamped." (1) chest tube should not be
2. "Pleur-evac next to bed." clamped
3. "Suction decreased to 15 cm."
4. "Chest tube disconnected from the Pleur- (2) correctPleur-evac should be
evac." maintained below the level of the
chest to prevent back flow of fluid
and air into the pleural space

(3) there is no reason to decrease


the level of suction

(4) do not disconnect a chest tube


20 The nurse screens an 8-month-old girl in a Strategy: "MOST concerned"
0. well-baby clinic. The nurse is MOST indicates you are looking for
concerned if the infant's mother makes something wrong.
which of the following statements?
(1) correctweight should double by
1. "My daughter has almost doubled her 5 months of age
birth weight."
2. "When I walk in the room, my child (2) begins to recognize parents at 6
smiles at me." months of age
3. "When she is around her grandpa, my
child cries." (3) begins to fear strangers at 6
4. "My daughter can't quite say 'mama' months, increases until 9 months of
yet." age

(4) begins to say "dada" and


"mama" with meaning at 10 months
of age
20 The nurse sees clients in a pediatric Strategy: All answers are
1. clinic. The nurse receives a phone call implementations. Determine the
from the mother of a 3-year-old saying outcome of each answer choice. Is it
that her child has vomited several times desired?
today. Which of the following
instructions by the nurse is BEST? (1) is part of a full liquid diet

1. "Offer your child some ice cream." (2) correctclear liquids should be
2. "Give your child some apple juice." offered first; as child tolerates these
3. "Offer your child orange juice." fluids, then full liquids may be offered
4. "Make some pudding for your child."
(3) is part of a full liquid diet

(4) is part of a full liquid diet


20 The nurse supervises a nurse's aide Strategy: "Require an intervention"
2. administer a soapsuds enema to a indicates incorrect behavior.
patient prior to abdominal surgery.
Which of the following actions, if (1) correctshould be 12-18 inches;
performed by the aide, requires an too high causes rapid distention and
intervention by the nurse? pressure in intestine causing rapid
expulsion of solution, poor defecation,
1. The aide holds the irrigation set 30 and damage to mucous membranes
inches above the patient's rectum.
2. The aide inserts the irrigation tube 3 (2) should insert 3-4 inches
inches into the patient's rectum.
3. The aide positions the patient in Sims' (3) places descending colon at lowest
position. point
4. The aide warms the water to 105F
(40C). (4) should be slightly higher than body
temperature
20 The nurse supervises a nurse's aide Strategy: "Nurse should intervene"
3. transferring a client from the bed to the indicates an incorrect action.
chair after a right total hip replacement.
The nurse should intervene if which of (1) helps prevent orthostatic
the following is observed? hypotension and dizziness

1. The nurse's aide helps the woman to a (2) right way to place chair
sitting position.
2. The nurse's aide positions the chair at (3) correctshould stand on affected
a 90 angle to the bed. side
3. The nurse's aide stands on the same
side of the bed as the patient's (4) right way to move patient
unaffected side.
4. The nurse's aide pivots the patient on
the unaffected leg.
20 The nurse supervises a student nurse Strategy: Determine the outcome of
4. administer a tube feeding to a client each answer choice.
with a tracheostomy. The nurse should
intervene if which of the following is (1) correctto minimize the risk for
observed? aspirations, the client should be
maintained in semi-Fowler's position
1. The student nurse places the client in
a supine position. (2) appropriate intervention
2. The student nurse aspirates and
returns the residual stomach contents. (3) appropriate intervention
3. The student nurse checks the pH of
the gastric content. (4) appropriate intervention
4. The student nurse checks the bowel
sounds for five minutes in each quadrant
20 The nurse supervises care of a group of Strategy: Remember growth and
5. children in a day care facility. The nurse development milestones.
should intervene in which of the
following situations? (1) correcttask too advanced for a
preschooler
1. A 4-year-old is given paper to write to
a pen pal. (2) appropriate for this age group
2. A 7-year-old is playing with an electric
train set. (3) helps cognitive development of
3. A 9-year-old is performing magic child
tricks for his friends.
4. A 12-year-old discusses collecting (4) appropriate for this age group
canned goods for the holidays.
20 A nursing student with a history of breast Strategy: Topic of the question is
6. cancer reports to the nurse on the unit that the unstated. Read answer choices
nursing student has just developed shingles on to determine topic.
her trunk. Which of the following actions by the
nurse is BEST? (1) passing the buck; care of
clients determined by the RN
1. Suggest that the nursing student contact her
physician. (2) can't care for any clients
2. Assign the nursing student to clients that until lesions have crusted
are not high risk.
3. Inform the nursing student that she cannot (3) correctbecause student is
care for clients. immunocompromised, restrict
4. Restrict the nursing student from performing from patient contact until
invasive procedures. lesions have crusted

(4) restricted from any patient


contact
20 A nursing team consists of an RN, an LPN/LVN, Strategy: Think about the skill
7. and a nursing assistant. The nurse should level involved in each patient's
assign which of the following patients to the care.
LPN/LVN?
(1) correctstable patient with
1. A 72-year-old patient with diabetes requiring an expected outcome
a dressing change for a stasis ulcer.
2. A 55-year-old patient with terminal cancer (2) requires nursing judgment;
being transferred to hospice home care. RN is the appropriate caregiver
3. A 42-year-old patient with cancer of the
bone complaining of pain. (3) requires assessment; RN is
4. A 23-year-old patient with a fracture of the the appropriate caregiver
right leg asking to use the urinal.
(4) standard unchanging
procedure; assign to the nursing
assistant
20 The nursing team consists of an RN who has Strategy: The RN cares for
8. been practicing for 6 months, an LPN/LVN who clients that require assessment,
has been practicing for 15 years, and a nursing teaching, and nursing judgment.
assistant who has been caring for clients for 3
years. The RN should care for which of the (1) care can be assigned to the
following clients? nursing assistant; standard,
unchanging procedure
1. A client 1 day postoperative after an internal
fixation of a fractured left femur. (2) medication can be given by
2. A client receiving diltiazem (Cardizem) and the LPN
phenytoin (Dilantin).
3. A client ordered to receive 2 U of packed (3) correctrequires the
cells prior to an upper endoscopy procedure. assessment and teaching skills
4. A client admitted yesterday with exhaustion of RN
and a diagnosis of acute bipolar disorder.
(4) offer food and fluids, assign
to LPN
20 The nursing team includes three RNs, Strategy: RN cannot delegate clients that
9. one LPN/LVN, and one nursing require assessment, teaching, or nursing
assistant. The nurse should consider judgment.
the assignments appropriate if the
nursing assistant is assigned to (1) correctstable client with standard,
which of the following clients? unchanging procedures

1. A client with an appendectomy. (2) requires assessment; RN should care for


2. A client with infectious meningitis. this client
3. An immunosuppressed client.
4. A client who had a radical (3) requires skills of RN
mastectomy.
(4) requires assessment and teaching
21 One hour after receiving 7 U of Strategy: Answers are a mix of
0. regular insulin, the client presents assessments and implementations. Does
with diaphoresis, pallor, and this situation require validation? No.
tachycardia. Which of the following Determine the outcome of each
actions should the nurse take FIRST? implementation.

1. Notify the physician. (1) action should be taken prior to notifying


2. Call the lab for a blood glucose the physician
level.
3. Offer the client milk and crackers. (2) does not require validation,
4. Administer glucagon. implementation required

(3) correctonset of action for regular


insulin is 30-60 minutes; assessment
indicates a problem with hypoglycemia;
foods such as milk and crackers should be
given if blood sugar is around 40-60 mg/dL;
if orange juice or simple sugar is given, it
should be followed with a meal or with
protein intake
(4) unnecessary, unless client is
unresponsive
21 On the morning after surgery to Strategy: Answers are a mix of
1. repair a fractured hip, the nurse finds assessments and implementations. Does
an older client struggling to get out this situation require assessment? Yes. Is
of bed. The client tells the nurse, "I there an appropriate assessment? Yes.
have to clean the kitchen now."
Which of the following actions, if (1) correctassessment; fat embolism is
taken by the nurse, is MOST common with fractures of long bones,
appropriate? results in pulmonary or cerebral emboli,
interferes with adequate circulation;
1. Obtain blood gas studies. confusion is first symptom
2. Instruct the client to remain in
bed. (2) need to assess first
3. Take the client's blood pressure.
4. Ask the family to remain with the (3) assessment; need to obtain ABG, start
client. oxygen

(4) implementation; doesn't address the fat


embolism
21 An order has been received to obtain Strategy: Determine the action of each
2. a stool specimen and test for occult drug and how it relates to a stool specimen.
blood. The nurse is MOST concerned
if the client makes which of the (1) correctiron supplements can cause
following statements? color of stool to resemble melena

1. "I take Feosol every day." (2) opiate narcotic; would have little effect
2. "My physician prescribed Vicodin." on stool specimen reliability
3. "I've been taking Lomotil."
4. "I sometimes take Motrin." (3) antidiarrheal; would have little effect on
stool specimen reliability

(4) nonsteroidal anti-inflammatory drug


(NSAID); would have little effect on stool
specimen reliability
21 The parents of a child just diagnosed with a Strategy: Answers are
3. chronic illness share with the nurse that they implementations. Determine the
are concerned about the sibling's sudden outcome of each answer choice. Is
change in behavior. Which of the following is it desired?
the BEST response by the nurse?
(1) correcttotal family
1. "Her brother is feeling left out right now, participation is accomplished
but we plan to include him in his sister's when you include the sibling
care."
2. "Her brother is just feeling left out right (2) may be appropriate but does
now, but he will start acting normal soon." not help the family adjust to a
3. "Her brother is worried about her and is child with a chronic illness
just reacting to his fear."
4. "Her brother is going through a normal (3) may be appropriate but does
developmental stage." not help the family adjust to a
child with a chronic illness

(4) inaccurate
21 The parents of a newborn diagnosed with a Strategy: Determine what the
4. meningocele have been grieving the loss of words mean.
their perfect child. After three days of
grieving, the progress in their emotional (1) correctthis comment
status is indicated to the nurse by which of indicates a desire to begin
the following comments? stroking and cuddling this baby;
this must happen before parents
1. "When will it be safe for us to hold our can provide physical care
baby?"
2. "We would rather that you feed our baby." (2) indicates a fear or a sense of
3. "What did we do to cause this problem?" insecurity with feedings
4. "When do you anticipate our baby going
home?" (3) indicates feelings of guilt

(4) is not specific to the question


21 A patient on suicide precautions asks for a Strategy: Remember therapeutic
5. razor to shave her legs. When the nurse tells communication.
the patient that she must remain with the
patient, the patient responds, "Don't you (1) true statement but not the
trust me?" Which of the following responses most therapeutic
by the nurse is BEST?
(2) passing the buck
1. "It is against hospital policy to allow
patients on suicide precautions to have (3) yes/no question
razors unsupervised."
2. "I trust you, but your physician said a (4) correctprovides patient with
nurse has to watch you if you want to shave factual information in a caring
your legs." manner
3. "Wouldn't you rather wait until you are
feeling better before you try to shave your
legs?"
4. "You have been having thoughts about
wanting to hurt yourself recently, so I'll stay
with you."
21 A patient receives gentamycin (Garamycin) 500 Strategy: "MOST important"
6. mg q 8 hours IV for a Pseudomonas infection of indicates a priority question.
the leg. When the nurse walks into the patient's All answers are assessments.
room, the patient is sitting in a chair with his Determine why you would
back to the door, looking out the window. When make each assessment and
the patient does not respond to the nurse's how it relates to the situation.
greeting, the nurse touches him on the shoulder.
The patient jumps and acts startled. Which of the (1) assessment; does not
following actions, if performed by the nurse, is address reason he was
MOST important? nonresponsive

1. Ask the patient what he is thinking. (2) assessment; does not


2. Monitor the color and sensation in the recognize that assessment
patient's leg. needed because of the
3. Obtain the patient's temperature, pulse, and medication he is receiving
blood pressure.
4. Check the patient for tinnitus and hearing (3) assessment; not relevant
loss. for issues raised in question

(4) correctassessment;
ototoxicity is serious adverse
effect of aminoglycosides
such as gentamycin
21 A permanent demand pacemaker, set at a rate of Strategy: Determine how each
7. 72, is implanted in a client for persistent third- answer relates to a
degree block. The nurse is MOST concerned if pacemaker.
which of the following is observed?
(1) does not indicate
1. Pulse rate 88 and irregular. malfunction of the pacemaker
2. Apical pulse rate regular at 68.
3. Blood pressure 110/88, pulse at 78. (2) correctany time the
4. Skin warm and dry to touch. pulse rate drops below the
preset rate on the pacemaker,
the pacer is malfunctioning;
the pulse should be
maintained at a minimal rate
set on the pacemaker

(3) do not indicate


malfunction of the pacemaker

(4) may be an early sign of


infection at the site
21 The physician adds cholestyramine (Questran) 4 (1) Questran interferes with
8. gm PO ac and hs to the medication regimen for absorption of Digoxin and
an older client. The client is also taking digoxin Esidrix
(Lanoxin) 0.125 mg PO qd and
hydrochlorothiazide (Esidrix) 25 mg PO qd. The (2) Questran interferes with
nurse assists the client to set up a medication absorption of Digoxin and
schedule. Which of the following medication Esidrix; take Esidrix in early
schedules is BEST? afternoon to prevent nocturia

1. Give the 3 meds at 8am, and Questran also (3) correctQuestran


at12,5 and hs interferes with absorption of
2. Digoxin- Give Lanoxin and
3. Esidrex at 7am, And give
4. Questran at 8,12,5, and HS.

(4) Questran interferes with


absorption of Digoxin
21 The physician orders an analgesic to be Strategy: Answers are
9. administered to a woman in labor who is 9 cm implementations. Determine the
dilated and is having contractions every 3 outcome of each answer choice.
minutes, lasting for 50 seconds. Which of the Is it desired?
following nursing actions is MOST important?
(1) client should not receive
1. Identify the client prior to administering analgesic
medication.
2. Calculate the amount of medicine to be (2) not appropriate for this
administered. situation
3. Hold the medication, and document the
order in nursing notes. (3) does not address the
4. Notify the physician regarding the status of immediate problem
the contractions.
(4) correctinformation indicates
that woman is in transition
phase; analgesics should not be
administered during transition
phase
22 The physician orders an arterial blood gas Strategy: All answers are
0. (ABG) for a client receiving oxygen at 6 implementations. Determine the
L/minute. Results show: pH 7.37, HCO3 26 outcome of each answer choice.
mEq/L, PaCO2 42 mm Hg, PaO2 90 mm Hg. The Is it desired?
nurse should take which of the following
actions? (1) oxygen level normal

1. Increase the rate of oxygen flow that the (2) unnecessary, results normal
patient is receiving.
2. Elevate the head of the bed. (3) correctresults normal,
3. Document the results in the chart. should be recorded
4. Instruct the patient to cough and deep
breathe. (4) unnecessary, results normal
22 The physician prescribes ciprofloxacin (Cipro) Strategy: All answers are
1. for a client. Which of the following implementations. Determine the
instructions is MOST important for the nurse outcome of each answer. Is it
to include when instructing the client about desired?
this medication?
(1) correctprevents crystalluria
1. "Drink plenty of fluids." and stone formation
2. "You may take this medication with your
multivitamin." (2) do not take within 6 hours
3. "Eliminate dairy products from your diet." before ciprofloxacin
4. "Take this medication with meals."
(3) do not take with milk or
yogurt alone, decreases the
absorption of ciprofloxacin; can
ingest dietary sources of calcium

(4) may take with meals if gastric


irritation occurs
22 The physician prescribes estrogen (Premarin) Strategy: "Further teaching is
2. daily for a middle-aged woman. Which of the necessary" indicates that wrong
following statements, if made by the client to information.
the nurse, indicates further teaching is
necessary? (1) common side effect

1. "There may be a change in my libido (2) common side effect


because of this medication."
2. "I may have a change in my weight while (3) causes dryness of eyes
taking this medication."
3. "I may have some difficulty wearing my (4) correctshould continue to
contact lenses because of the medication." perform monthly self-breast
4. "It is unnecessary for me to perform routine exams
self-breast exams while I am taking this
medication."
22 The physician prescribes hydrochlorothiazide Strategy: Determine what
3. (Oretic) 50 mg PO daily for a client. The client also nutrients are being lost
takes dexamethasone (Decaspray) 2 sprays in each due to the medication.
nostril bid. The nurse should encourage the client Determine which foods are
to increase the intake of which of the following highest in potassium.
foods?
(1) no need to increase
1. Chicken and low-fat meats. protein
2. Dairy products and eggs.
3. Whole grain breads and fresh vegetables. (2) no need to increase
4. Citrus fruits and green, leafy vegetables. calcium intake

(3) no need to increase


intake of roughage

(4) correctneed to
increase intake of
potassium-rich foods
because of potassium loss
from medications
22 Polyethylene glycol-electrolyte solution (GoLYTELY) Strategy: "Need for further
4. is ordered for a client before a colonoscopy. The instruction" indicates you
physician's office nurse explains to the client how to are looking for an incorrect
take the solution. Which of the following statement
statements, if made by the client, indicates the
need for further instruction? (1) true statement

1. "I need to drink 4 liters of the solution." (2) correctcan cause


2. "If I drink it ice cold, it won't taste as bad." hypothermia due to large
3. "Once I finish drinking the solution, I can drink quantity of solution
only water." ingested
4. "I can use tap water to reconstitute the powder."
(3) true statement

(4) true statement


22 A primipara is admitted in early labor, and her Strategy: Determine how
5. membranes rupture. Which of the following each answer choice relates
assessments by the nurse is MOST important? to the rupture of
membranes.
1. Determine the pH of the amniotic fluid.
2. Evaluate the mother's blood pressure. (1) amniotic fluid is
3. Check the monitor for decelerations. important to check to
4. Assess for a prolapsed cord. differentiate it from urine;
pH will be acidic if it is
urine

(2) mother's blood


pressure is not affected by
rupture of the membranes

(3) nurse should look for


variable decelerations if
cord is prolapsed

(4) correctinitial
assessment is to check for
a prolapsed cord
22 Prior to a nurse discharging an infant home with Strategy: "Need for further
6. the parents, which of the following statements, if teaching" indicates that you
made by the mother to the nurse, indicates a need are looking for an incorrect
for further teaching about newborn care? response.

1. "I will notify my physician about absence of (1) correctis normal for a
breathing for 10 seconds." neonate; apnea lasting
2. "I will notify my physician about more than one longer than 15 seconds
episode of projectile vomiting." should be reported
3. "I will notify my physician if my baby's
temperature is greater than 101F (38.3C)." (2) does not indicate a need
4. "I will rock and cuddle my infant frequently to for further teaching
promote a sense of trust."
(3) does not indicate a need
for further teaching

(4) does not indicate a need


for further teaching
22 Prior to helping a client out of bed on the first day Strategy: All answers are
7. after an anterior cervical fusion, the nurse should implementations.
take which of the following actions? Determine the outcome of
each answer choice. Is it
1. Remove the client's cervical collar. desired?
2. Raise the head of the bed.
3. Position the client supine at the edge of the bed. (1) contraindicated; collar
4. Ask the client to fold both arms across his chest. offers additional support for
the neck

(2) correctraising head of


bed decreases the effort for
the client and the nurse

(3) inconvenient and would


cause undue strain on the
client

(4) does not allow the client


to assist in the transfer
22 A psychiatric patient admitted involuntarily asks Strategy: All answers are
8. the nurse to mail a letter to the President. The implementations.
patient states that the letter will make the Determine the outcome of
President regret his actions to prevent each answer. Is it desired?
homosexuals from serving in the military. Which of
the following responses by the nurse is BEST? (1) psychiatric patients do
not forfeit their civil rights
1. Accept the letter and place it in the patient's
medical record. (2) patient has the right to
2. Read the patient's letter and decide if it is send and receive unopened
appropriate to mail. mail
3. Call the patient's psychiatrist and inform him of
the letter. (3) has the right to mail the
4. Discourage the patient from sending the letter, letter
but mail it if patient insists.
(4) correctretains the
right to communicate with
elected officials
22 The public health nurse cares for a Strategy: "MOST concerned" indicates a
9. child diagnosed with impetigo. The complication.
nurse is MOST concerned if which of
the following is observed? (1) describes a fungal infection

1. White patches on the buccal (2) can be caused by many other factors
mucosa.
2. Hearing loss. (3) can be caused by many other factors
3. Respiratory wheezes.
4. Periorbital edema. (4) correctindicative of poststreptococcal
glomerulonephritis, a possible complication
of impetigo
23 A school-aged child informs the Strategy: Answers are a mix of assessments
0. school nurse that his right knee and implementations. Does this situation
"doesn't feel right." Which of the require assessment? Yes. Is there an
following actions should the nurse appropriate assessment? Yes.
take FIRST?
(1) will not help determine if the knee is
1. Instruct the child to extend the edematous
right leg.
2. Put both of the child's legs (2) inspection first step of physical
through range of motion. assessment
3. Advise the child to soak the right
knee in warm water. (3) implementation; need to assess to
4. Compare the appearance of the determine the problem
right knee with the left knee.
(4) correctshould compare corresponding
joints for symmetry and to determine normal
parameters
23 The school nurse interviews an Strategy: "MOST concerned" indicates
1. adolescent. The nurse is MOST something wrong.
concerned if the adolescent states
which of the following? (1) fatigue with increased activity is normal

1. "I am so busy all the time, and at (2) occasional awkwardness seen with
the end of the day, I am tired." growth spurts
2. "Once in a while, I fall over my
feet when I am just walking (3) correctshould have increased sweat
around." production due to hormonal changes
3. "I'm glad I don't get as sweaty as
my friends when I work out." (4) preoccupation with physical appearance
4. "It is important that I wear normal
clothes that are similar to what my
friends wear."
23 The school nurse notes that one of Strategy: Answers are a mix of assessments
2. the children has a copious watery and implementations. Does this situation
discharge from the left eye, and the require validation? No. Determine the
eye is red. Which of the following outcome of the interventions.
actions, if taken by the nurse, is
BEST? (1) correctextreme tearing, redness, and
foreign body sensation are symptoms of viral
1. Contact the child's parents to conjunctivitis; highly contagious; children
pick up the child. restricted from school until symptoms have
2. Instruct the child to use a clean resolved, 3 to 7 days
tissue each time he wipes his eye.
3. Contact the child's physician. (2) appropriate action; more important to
4. Obtain the child's temperature. prevent child from spreading infection to
other children

(3) passing the buck

(4) no reason to check temperature, should


remove child from environment
23 Several hours after an oxytocin (Pitocin) Strategy: All answers are
3. infusion is started, the client's implementations. Determine the
contractions are sustained over two outcome of each answer choice. Is it
minutes. Which of these nursing actions desired?
is MOST important?
(1) correctsustained contractions
1. Discontinue the IV Pitocin. can lead to a ruptured uterus and/or
2. Administer oxygen. fetal distress
3. Reposition the client.
4. Decrease the IV Pitocin rate. (2) important if fetal distress is
apparent

(3) important if fetal distress is


apparent

(4) not appropriate for situation; client


would continue to receive Pitocin
23 A spica cast is applied to a 14-month-old Strategy: Answers are a mix of
4. with developmental dysplasia of the hips. assessments and implementations.
Which of the following is MOST Does this situation require
appropriate for the nurse to teach the assessment? Yes. Is there an
parents? appropriate assessment? Yes.

1. Change diapers frequently to prevent (1) implementation; appropriate


cast soiling.
2. Inspect the skin around the cast. (2) correctassessment; assess for
3. Turn the client by using the abduction skin breakdown; change client's
stabilizer bar. position frequently to prevent skin
4. Keep small toys out of the client's breakdown
reach.
(3) implementation; inappropriate

(4) implementation; appropriate but is


not higher priority than answer choice
2
23 A staff member working in the newborn Strategy: Answers are a mix of
5. nursery complains to the nurse that even assessments and implementations.
though he doesn't feel bad, he has been Does this situation require validation?"
having loose stools for the last couple of No. Determine the outcome of each
days. Which of the following responses by implementation.
the nurse is BEST?
(1) good hand washing is essential to
1. "Make sure you wash your hands after preventing the spread of infection
going to the bathroom."
2. "Are you drinking plenty of fluids?" (2) yes/no question; increase fluid to
3. "Describe to me how you are feeling." prevent fluid volume deficit
4. "I'm going to reassign you to the
orthopedics." (3) staff member already stated that
she/he feels okay

(4) correctrestrict from care of


newborn, infants, or
immunocompromised patients
23 To best evaluate home compliance Strategy: Topic of question is unstated. Read
6. with metoclopramide (Reglan) for answer choices for clues.
a 3-month-old, the nurse should
take which of the following (1) baby may have reflux after a home visit
actions?
(2) may reflect feeding retention; subjective
1. Observe the mother feeding information is not as reliable as objective
the infant. measurements
2. Ask the mother about the
infant's retention of feedings. (3) may reflect feeding retention; subjective
3. Ask the mother how many wet information is not as reliable as objective
diapers the baby has each day. measurements
4. Weigh the baby, and compare
to baseline weight. (4) correctis most accurate indicator of
feeding retention
23 To coordinate community Strategy: All answers are implementations.
7. placement for a client diagnosed Determine the outcome of each answer choice.
with schizophrenia and alcoholism Is it desired?
who is homeless, the nurse
should take which of the following (1) need to include client in decision-making
actions?
(2) correctis important that multidisciplinary
1. Collaborate with members of team discusses and collaborates with the client
the client's family to explore on all discharge placements; client will need
placement options. support and assistance in making decisions
2. Collaborate with the health about discharge and residential living
care team and the client to arrangements
schedule a predischarge visit to a
residential placement facility. (3) if the nurse visits independently of client,
3. Visit the placement facility that can diminish the client's sense of self-
alone to make an independent worth and decision-making
decision about the facility, and
report to the client and family. (4) reviewing rules with client prematurely can
4. Review with the client specific inhibit opportunity to explore feelings about
rules of the facility. this decision
23 When doing an admission Strategy: Determine how each assessment
8. assessment for a client diagnosed relates to herpes zoster.
with herpes zoster (shingles), it is
important for the nurse to (1) herpes zoster is caused by a virus; it is not
determine which of the following? an allergic reaction

1. When the client developed this (2) herpes zoster is not caused by a food
allergic reaction and how long it allergy
has lasted.
2. If the client has eaten any new (3) herpes simplex, not herpes zoster, is
foods within the past 24 hours. related to fever blisters and canker sores
3. If the client has a history of
fever blisters or canker sores. (4) correctclose relationship between the
4. If the client comes in contact virus that causes herpes zoster (shingles) and
with anyone with chickenpox. chickenpox virus
23 When using palpation techniques Strategy: All answers are implementations.
9. during the physical assessment of Determine the outcome of each answer choice.
an adult female with abdominal Is it desired?
pain, which of the following
actions should the nurse take (1) holding a deep breath is done during
FIRST? palpation of the liver

1. Instruct the client to take a (2) correctbreathing slowly will enhance


deep breath and hold it. relaxation of the abdominal muscles
2. Inform the client to breathe
slowly. (3) bimanual palpation shouldn't be used for a
3. Use bimanual palpation client with abdominal pain
technique.
4. Apply light palpation in the (4) prior to the abdominal palpation, instruct
area. client to breathe slowly because client likely to
protect the abdomen when in pain
24 When working with an adolescent Strategy: Answers are implementations.
0. diagnosed with hypertension and Determine the outcome of each answer. Is
obesity, it is MOST important for the it desired?
nurse to make which of the following
suggestions? (1) properly supervised physical activity is
desirable, not to be avoided
1. Avoid participating in organized
sports. (2) correctexcellent means of obtaining
2. Join an adolescent weight information and support for the client
reduction support group.
3. Limit socialization with friends of (3) isolation from peers should be avoided
normal weight.
4. Adhere to a 1,000-calorie low-fat (4) does not supply enough calories for an
diet. adolescent
24 Which nursing action is MOST Strategy: All answers are implementations.
1. appropriate after intubating a Determine the outcome of each answer. Is
postoperative client who had a it desired?
respiratory arrest?
(1) inappropriate action
1. Soak the intubation equipment in
concentrated Betadine solution. (2) correctsterilization of equipment after
2. Place the intubation blade in a bag, exposure to body fluids of a client is
and arrange for gas sterilization. protocol
3. Soak the intubation blade in Cidex
solution. (3) inappropriate action
4. Wash the equipment with soap and
water and allow to air-dry. (4) inappropriate action
24 Which nursing action is MOST Strategy: Answers are a mix of
2. appropriate when an infant is assessments and implementations. Is this
admitted for fever, poor feeding, a situation that requires validation? No.
irritability, and a bulging fontanel? Determine the outcome of each
implementation.
1. Perform neurological checks every
four hours. (1) assessment; not a priority
2. Place the client on droplet
precautions. (2) correctimplementation; classic signs
3. Monitor the client's urine output of meningitis; client should be isolated
closely. from other clients
4. Encourage fluid intake.
(3) assessment; not a priority

(4) implementation; inappropriate for this


situation
24 Which of the following actions, if Strategy: All answers are implementations.
3. performed by the nurse, is considered Determine the outcome of each answer
negligence? choice. Is it desired?

1. Inserting a 16 Fr NG tube and (1) appropriate procedure; verify


aspirating 15 ml of gastric contents. placement by checking pH of gastric
2. Administering Demerol IM to a aspirate
patient prior to his using the
incentive spirometer. (2) reduces the pain, enables the patient
3. Administering ferrous sulfate to take a deep breath
(Feosol) 325 mg with coffee.
4. Initially administering blood at 5 (3) correctdo not give together, may
ml/minute for 15 minutes. impair iron absorption; give with orange
juice

(4) appropriate procedure, start blood with


normal saline and 19-gauge needle
24 Which of the following actions Strategy: Answers are all implementations.
4. should the nurse instruct the Determine the outcome of each answer. Is it
client to complete FIRST to desired?
establish a normal urinary
pattern? (1) client should start voiding every two hours and
gradually progress to three to four hours
1. Urinate every two hours.
2. Record each time the client (2) second thing to do
urinates.
3. Keep a record of daily fluid (3) correctclient needs to know how much and
intake. when he ingests fluid
4. Stay near a bathroom.
(4) appropriate but not the first thing to do
24 Which of the following Strategy: Think about each answer. Does the
5. indicates that a client is behavior indicate trust?
beginning to develop a
trusting relationship with the (1) delusional system is indication of anxiety,
nurse? delusions will increase with greater anxiety; trust
of nurse is not related to an explanation of client's
1. The client describes delusions
delusions to the nurse.
2. The client can describe (2) correctclient who is suspicious and delusional
his/her feelings to the nurse. begins to demonstrate trusting behaviors when
3. The nurse feels more she/he shares feelings with the nurse
comfortable with the client.
4. The client reports feeling (3) nurse's response can be an indication of
less anxious. transference/countertransference issues; is not
indicative of client beginning to enter a trusting
relationship
(4) is beneficial that the client's anxiety level is
becoming less intense; will facilitate development
of a trusting relationship
24 Which of the following Strategy: Answers are implementations. Determine
6. indicates to the nurse the need the outcome of each answer choice. Is it desired?
for further teaching for a
postoperative client using the (1) correctincentive spirometry is designed to
incentive spirometer? promote lung expansion by encouraging sustained
maximal inspirations
Select all that apply:
(2) benefits the postoperative client during use of
1. The client exhales with the spirometry
spirometer in his mouth.
2. The client inhales with the (3) benefits the postoperative client during use of
spirometer in his mouth. spirometry
3. The client splints his
incision before using the (4) benefits the postoperative client during use of
spirometer. spirometry
4. The client raises the head of
his bed before using the (5) correctbreath should not exceed 10/minute to
spirometer. 12/minute
5. The client breath rate is
20/minute while using the (6) correctshould hold breath at end of maximal
spirometer. inspiration
6. The client exhales and holds
his breath for two to three
seconds.
24 Which of the following Strategy: Answers are implementations. Determine
7. interventions should be the the outcome of each answer choice. Is it desired?
priority during the nursing
care of a 2-month-old infant (1) would lead to further deprivation
after surgery?
(2) would lead to further deprivation
1. Minimize stimuli for the
infant. (3) correcttactile stimulation is imperative for an
2. Restrain all of the infant's infant's normal emotional development; after the
extremities. trauma of surgery, sensory deprivation can cause
3. Encourage the parents to failure to thrive
stroke the infant.
4. Demonstrate to the parents (4) does not address the emotional needs of the
how they can assist with their infant
infant's care.
24 Which of the following Strategy: Determine how each answer choice relates
8. questions BEST aids the to orientation.
nurse in assessing the
orientation of a client on the (1) some well-oriented people do not know the
psychiatric unit? answer to this question, depending upon their age,
educational level, etc.
1. "Who is the president of
the United States?" (2) irrelevant
2. "Do you remember my
name?" (3) correctis a specific question related to
3. "What is your name?" orientation of person
4. "What time is it?"
(4) without consulting a watch or clock, most well-
oriented people cannot answer this question
24 Which of the following Strategy: Determine the outcome of each answer
9. statements by an adult client choice. Is it desired?
indicates to the nurse the
need for further teaching (1) possible for many clients to go without a
regarding care of a sigmoid collection bag by performing routine irrigations
colostomy?
(2) correctirrigation of sigmoid colostomy is not
1. "I hope to be able to go necessary more than once a day and sometimes
without a pouch soon." every two or three days, if at all
2. "I'm irrigating my
colostomy after each meal." (3) normal reactions
3. "My stoma is looking
better all the time." (4) normal reactions
4. "It's not hard to change
my pouch every several
days."
25 Which of the following Strategy: Think about what the words mean.
0. statements by the client
indicates to the nurse that (1) indicates that the client believes her anxiety is
the client has an accurate really exhaustion rather than stress
understanding of the cause
of anxiety? (2) common but shows inaccurate client
understanding of the source of the anxiety
1. "When I get overly tired
from working too hard, I (3) common but shows inaccurate client
begin to have severe understanding of the source of the anxiety
headaches and nausea."
2. "I'm losing my mind. I (4) correctanxiety is often expressed as physical
can't think straight." symptoms and can be triggered by situations in
3. "My chest pounds and I adult life that reawaken feelings of anger or anxiety
can't catch my breath. I must unresolved from childhood
be having a heart attack."
4. "Now that my mother has
died, I've been thinking a lot
about the way she abused
me. I feel very tense and
sick."
25 Which of the following Strategy: Answers are a mix of assessments and
1. statements, if made by a implementations. Determine whether it is
client diagnosed with appropriate to assess or implement.
Buerger's disease to the
nurse, indicates that (1) implementation; pain (cramps in feet and
teaching is effective? intermittent claudication) is outstanding symptom of
disease; pain control goal of treatment
1. "I should avoid taking
analgesics if I become (2) implementation; goal of medical management is
uncomfortable." to prevent progression of disease
2. "The medication I am
taking will prevent this (3) correctassessment; check for ulcer formation
disease from recurring." and gangrene; disease involves recurring
3. "I should inspect my inflammation of arteries and veins in upper and
fingers and toes every day." lower extremities, results in thrombus and occlusion,
4. "I should keep track of seen in men 20-35 years old; smoking is a causative
how much fluid I drink during factor; pain at rest and coldness major symptoms
the day."
(4) assessment; fluids are not restricted or monitored
25 Which statement, if made by the Strategy: "Need for further teaching"
2. client diagnosed with Cushing's indicates an incorrect response.
syndrome, indicates to the nurse
the need for further teaching? (1) exercise program should begin gradually

1. "I realize I'll have to gradually (2) client may develop hypertension related
begin an exercise program." to sodium and water retention
2. "I'm going to have to keep a close
eye on my blood pressure." (3) correctstatement indicates that the
3. "I'm not really worried about client does not realize there is an increased
getting pneumonia this winter." susceptibility to infections
4. "I'll be eating foods low in
carbohydrates and salt." (4) diet should be low carbohydrate, low
sodium, and high protein
25 While a client is receiving TPN, it is Strategy: "MOST important" indicates a
3. MOST important for the nurse to priority question.
monitor which of the following?
(1) most common complications involve fluid
1. Vital signs and level of and electrolytes
consciousness.
2. Arterial blood gases and liver (2) abnormalities in liver function may occur,
enzymes. but most common complications involve
3. Serum glucose and electrolytes. fluid and electrolytes
4. Skin turgor and daily weights.
(3) correcthyperglycemia can cause
diuresis and excessive fluid loss; should
check fingerstick blood sugar every 6 hours,
check serum electrolytes (sodium,
potassium, calcium, magnesium,
phosphates) several times a week

(4) not most important; should assess skin


turgor to check for dehydration and weigh
daily
25 While checking the patency of a Strategy: All answers are implementations.
4. Salem sump tube, the nurse finds Determine the outcome of each answer
stomach contents draining from the choice. Is it desired?
air vent. Which of the following
nursing actions is MOST (1) important not to put fluids through the
appropriate? air vent

1. Insert water through the air vent. (2) tube should not be withdrawn
2. Pull the sump tube back 2-3
inches. (3) correctclearing the air vent with air will
3. Insert 30 ml of air through the air re-establish proper suction in the Salem
vent. sump tube
4. Insert a new nasogastric tube.
(4) unnecessary
25 A woman diagnosed with hepatitis B Strategy: "MOST important" indicates a
5. is scheduled for an abdominal priority question. Determine what each value
hysterectomy. It is MOST important is measuring and how it relates to hepatitis.
for the nurse to check which of the
following lab results before the (1) hepatitis does not affect potassium levels
patient goes to surgery?
(2) hepatitis does not affect sodium levels
1. Potassium.
2. Sodium. (3) correctdeficiency of clotting factors can
3. Prothrombin time. increase risk of hemorrhage during surgery
4. Hemoglobin.
(4) hepatitis does not alter hemoglobin
levels significantly
25 A woman is in active labor with her first Strategy: "MOST" indicates there may be
6. child when her membranes rupture. She more than one attractive answer.
voices a concern to the nurse that she is
afraid of having a "dry labor." Which of (1) amniotic fluid cushions fetus, allows
the following responses by the nurse is freedom of movement for
MOST appropriate? musculoskeletal development, facilitates
symmetrical growth, maintains constant
1. "The amniotic fluid provides only body temperature, is a source of oral
minimal lubrication for the labor fluids, and collects wastes
process."
2. "The amniotic sac may impede the (2) correctsometimes done to assist or
progress of labor and is often ruptured induce labor
artificially."
3. "Labor is only slightly more difficult (3) does not make labor more difficult
with early rupture of the amniotic sac."
4. "Because there is limited amniotic (4) no additional fluids will be supplied
fluid, additional fluids will be supplied."
25 A young adult comes to the outpatient Strategy: All answer choices are
7. clinic with complaints of vaginal itching. implementations. Determine the
Which of the following outcome of each answer choice. Is it
recommendations, if given to the client desired?
by the nurse, is MOST appropriate?
(1) contains bacilli that naturally exist in
1. "Supplement your diet with yogurt gastrointestinal tract, no effect on
and dairy products." vaginal pH
2. "Douche with an over-the-counter
preparation." (2) may alleviate discomfort of vaginal
3. "Wash the area with soap and water discharge but would disrupt normal pH of
several times a day." vagina
4. "Wear underwear that is lined with a
cotton crotch." (3) this frequency would cause dryness
and increase itching in the area

(4) correctmore absorbent; allows for


better circulation of air to body;
dampness aggravates itching
25 A young adult is in a motor vehicle Strategy: Determine which situation is
8. accident and has been admitted to an most unstable.
emergency department. Which of the
following observations indicates to the (1) correctdisoriented client with
nurse that the client should not be left irregular vital signs represents a grave
alone? safety risk

1. Disorientation and irregular vital (2) may increase the need for nursing
signs. interaction/assessment but does not
2. Irregular vital signs and hostility. require the nurse to stay with client all
3. Rapid respirations and agitation. the time
4. Elevated vital signs and
apprehension. (3) may increase the need for nursing
interaction/assessment but does not
require the nurse to stay with client all
the time

(4) may increase the need for nursing


interaction/assessment but does not
require the nurse to stay with client all
the time
25 A young Hispanic client who speaks Strategy: Answers are implementations.
9. little English is admitted to a medical- Determine the outcome of each answer.
surgical unit with an increased Is it desired?
temperature. Prior to the nurse
performing a physical assessment, (1) less effective
which of the following is the MOST
appropriate nursing action? (2) less effective

1. Attempt to prepare the client with (3) correctstaff who speak other
hand signals. languages are usually noted by nursing
2. Show the client pictures of the administration for instances where a
physical exam process. translator is the best option
3. Contact an employee who speaks
Spanish to translate. (4) less effective
4. Speak slowly to explain the physical
assessment.
A 3-month-old (1) does not indicate any immediate problem;
infant is as pressure increases, pupils may become
experiencing dilated
increased
intracranial (2) correctsign of increased intracranial
pressure (ICP). pressure
Which of the
following (3) does not reflect complication of increased
assessment intracranial pressure
findings should
the nurse (4) does not reflect complication of increased
report to the intracranial pressure
physician?

1. Pinpoint
pupils.
2. High-pitched
cry.
3. Decrease in
blood pressure.
4. Absence of
reflexes.
2. A 3-month-old infant (1) correct3-month-old infant
is placed in traction can grasp a rattle
for developmental
dysplasia of the (2) not as good as answer choice
hips. Which of the (1)
following toys is
appropriate for the (3) designed for an older child
nurse to offer the
infant during (4) not as good as answer choice
hospitalization? (1)

1. A rattle.
2. A stuffed animal.
3. Colorful blocks.
4. A tape playing
nursery rhymes.
3. A 4-week-old infant Strategy: Determine how each
with symptoms of statement relates to pyloric
pyloric stenosis is stenosis.
brought to the
outpatient clinic by (1) not expected with pyloric
his mother. Which of stenosis, suggestive of blood in
the following stool
statements does the
nurse expect the (2) sucking problems not
mother to make expected with pyloric stenosis
about her son's
symptoms? (3) correctbecomes lethargic,
dehydrated, and malnourished
1. "My son's bowel
movements have (4) would expect emesis to
turned black and contain milk or formula, should
sticky." not be bile-colored
2. "I really have to
encourage my son
to suck the bottle."
3. "My son is fussy
and seems hungry
all the time."
4. "My son spits up
green liquid after
feeding."
4. A 5-year-old child is (1) would be very difficult to
scheduled for a prepare a 5-year-old child for a
lumbar puncture totally foreign procedure with
(LP). Which of the only words
following nursing
actions BEST (2) may be frightening without
prepares the child additional preparation
for the procedure?
(3) correctexcellent method to
1. Explain the use with a child because it
procedure in detail. incorporates actually "feeling"
2. Show a video of many aspects of the procedure
the procedure. as they are explained
3. Do a mock run-
through of the (4) child probably doesn't know
procedure. enough to ask many questions
4. Answer all
questions simply
and honestly.
5 A 10-year-old child weighing 50 lb (23.6 (1) correctimplementation, amount is
. kg) returns from surgery for a skin graft excessive for child and there are no
to the left leg. The patient has an IV of electrolytes in fluid
D5W infusing into the left arm. The
physician's orders read: "D5W 2,000 (2) implementation, may have serious
cc/24 h." It is MOST important for the electrolyte disturbances before
nurse to take which of the following discrepancies are seen in I and O
actions?
(3) implementation, rate is correct for
1. Call the physician to clarify the IV fluid amount of fluid ordered, but amount is
order. excessive for child and fluid is
2. Keep accurate records of the patient's inappropriate
intake and output.
3. Set the controller on the IV pump to (4) assessment, should not administer
infuse at 84 gtt/min. fluids as ordered because they are
4. Monitor the patient for fluid and inappropriate in amount and content
electrolyte balance.
6 A 20-year-old primipara attends a class (1) use of creams not recommended,
. for women who plan to breast feed. To could cause breast tissues to become
prepare for breast feeding, the nurse tender, sebaceous glands keep skin
should encourage the women to do which pliable
of the following?
(2) doesn't prepare breasts for feeding
1. Apply moisturizer to the breasts every
day after bathing. (3) correctprepares nipples for
2. Expose the breasts to air every day for stretching action of sucking during
20 minutes. breast feeding, soap avoided to prevent
3. Wash breasts with water and rub with drying
a towel every day.
4. Massage the breasts to increase (4) could cause breast tissues to become
circulation twice daily. tender
7 A 20-year-old woman calls the outpatient (1) sperm doesn't resemble atypical cells
. clinic to schedule her first Papanicolaou that the test is designed to find
smear. The nurse should recommend
which of the following to the client? (2) correctdouching would affect
appearance of cells in vaginal smear,
1. Avoid intercourse for 48 hours before would make test inaccurate
the examination.
2. Avoid douching for 24 hours before her (3) will concentrate urine but won't
appointment. affect Pap smear
3. Withhold all foods and fluids 12 hours
before the appointment. (4) part of routine GYN exam, but not
4. Save her first voided urine specimen related to Pap smear
the morning of her appointment.
8 An abdominal wound irrigation with a Strategy: Answers are all
. normal saline solution is ordered for a client. implementations. Determine the
To perform this procedure, the nurse should outcome of each answer choice. Is it
take which of the following actions? desired?

1. Warm the irrigating solution to 110.0F (1) too warm, should be room
(43.3C). temperature or 90-95F (32.2-
2. Establish a sterile field that includes the 35.0C)
irrigating equipment.
3. Direct the irrigating solution at the outer (2) correctrequires strict aseptic
edges of the wound, then the center of the technique
wound.
4. Aspirate the irrigating fluid with a syringe (3) may cause new microorganisms
to prevent accumulation in the wound. to be flushed into wound

(4) fluid should drain by gravity


9 A client is diagnosed with a flaccid bladder (1) promotes alkaline urine; should
. following a spinal cord injury. The nurse also avoid citrus juices, excessive
teaches the client about dietary changes. amounts of milk, and carbonated
Which of the following beverages, if selected beverages
by the client, indicates to the nurse that (2)(5)(6) correctpromotes
teaching is effective? acidic urine, minimizes risk of
Select all that apply.* urinary tract infection and stone
1. Lemonade. formation; also use cranberry,
2. Prune juice. tomato juice, bouillon
3. Milk. (3) excessive amounts of milk
4. Orange juice. promote alkaline urine
5. Cranberry juice. (4) promotes alkaline urine; should
6. Tomato juice. also avoid citrus juices, excessive
amounts of milk, and carbonated
beverages
1 An adult client with newly diagnosed type 1 (1) hyperglycemia symptoms
0. diabetes is being seen by the home health nurse. are hot dry skin, rapid, deep
The physician has placed the client on an 1,800- respirations (Kussmaul),
calorie ADA diet, ordered the client to self- lethargic, polyuria, polydipsia,
administer 15 units of NPH insulin each day polyphagia, glycosuria,
before breakfast, and check his blood sugar qid. nausea, and vomiting
When the nurse visits the client at 5 PM, the
nurse discovers that the client has not eaten (2) NPH insulin is
since noon and has just returned from jogging. intermediate-acting, onset 3-4
The client's vital signs are: BP 110/80, pulse hours, peak 8-16 hours,
120/min, respirations 18/min, and temperature duration 18-26 hours
98.2F (36.8C). The nurse anticipates the client's
blood sugar to be which of the following? (3) normal blood sugar 70-110
mg/dL
1. 250 mg/dL.
2. 160 mg/dL. (4) correcthypoglycemia
3. 90 mg/dL. symptoms are cool, clammy
4. 50 mg/dL. skin, diaphoresis,
nervousness, weakness,
hunger, confusion, headache,
slurred speech, coma
1 After receiving report, which of the following (1) correctIV fluids are
1. patients should the nurse see FIRST? critical to reduce clotting and
pain
1. A patient in sickle-cell crisis with an infiltrated
IV. (2) no indication patient is
2. A patient with leukemia who has received 0.5 unstable
unit of packed cells.
3. A patient scheduled for a bronchoscopy. (3) stable patient
4. A patient complaining of a leaky colostomy
bag. (4) stable patient
1 At approximately 6 PM, the nurse begins to open Strategy: All answers are
2. the nurses' notes for the evening shift. The last implementations. Determine
entry is noted for 1 PM, and there is no signature. the outcome of each answer
Which of the following responses by the nurse is choice. Is it desired?
MOST appropriate?
(1) blank lines should never
1. Leave approximately three or four lines for the be left in the nurses' notes
day nurse to enter the day information and sign
the chart. (2) nurse should chart only
2. Review with the client the activities after 1 PM the care that s/he has
and enter what are determined to be the administered
activities after 1 PM.
3. Begin charting on the next line below the last (3) correctday nurse can
entry and make a note for the day nurse to make make a "late entry" to add
a late entry to complete the chart. any additional information
4. Do not enter anything until the day nurse has
been notified of the problem and returns to the (4) unnecessary
unit to complete charting.
1 A brace is ordered for a young teen with (1) bed-boards maintain proper
3. scoliosis. The nurse determines teaching is vertebral alignment but can't correct
effective if the client makes which of the lateral curvature of scoliosis
following statements?
(2) diet should be high-calorie due to
1. "I will have my parents put bed-boards age of child and growth
on my bed." requirements; diet doesn't affect
2. "I should decrease my caloric intake." curvature of the spine
3. "I should only take showers."
4. "I will hold on the rail when going down (3) either tub bathing or a shower is
the stairs." permitted

(4) correctprevents falls, should


also avoid slippery surfaces
1 A charge nurse develops assignments for (1) should be in a private room away
4. the evening shift. The nurse notes that a from roommate with infection
client with a tracheostomy with purulent
drainage and a pending culture and (2) should be in a private room away
sensitivity (C&S;) is sharing a room with a from roommate with infection
client diagnosed with neutropenia. Which of
the following actions by the charge nurse is (3) should be cared for by different
MOST appropriate? nurses

1. Assign an experienced nurse to care for (4) correctinfection in a


both clients in the same room. neutropenic individual may cause
2. Assign each client a separate nurse. morbidity and fatality; place the
3. Place the client diagnosed with neutropenic client in a private room;
neutropenia in a private room and assign limit and screen visitors and hospital
the same nurse to care for both clients. staff with potentially communicable
4. Place the client diagnosed with illnesses
neutropenia in a private room and assign
different nurses to care for each client.
1 The charge nurse notes a young child is (1) hereditary dysfunction of
5. placed on droplet precautions. The charge exocrine glands causing obstruction
nurse identifies that the nurse cares for because of flow of thick mucus,
which of the following clients? standard precautions

1. A child with cystic fibrosis. (2) inflammation of tonsils, standard


2. A child with tonsillitis. precautions
3. A child with bronchitis.
4. A child with pertussis. (3) inflammation of large airway,
standard precautions

(4) correctdroplet precautions


required, private room, maintain
spatial separation of 3 feet between
patient and visitors
1 A child comes to the school nurse with a Strategy: All answers are
6. honey-colored crusted lesion below the implementations. Determine the
right nostril. Which of the following actions outcome of each answer choice. Is it
should the nurse take FIRST? desired?

1. Remove the scab. (1) notify parents first; loosen scab


2. Apply a wet cloth to the lesion. with Burrow's solution compress;
3. Notify the child's parents. gently remove, topical ointment
4. Contact the health department.
(2) notify parents first; treated with
systemic antibiotics, antibacterial
soap

(3) correctdescribes impetigo,


highly infectious superficial bacterial
infection; notify parents so they can
contact the physician

(4) unnecessary to report impetigo to


the health department
1 A client admitted with a diagnosis of Strategy: Determine how each answer choice
7. metastatic cancer has been relates to the lab values
receiving chemotherapy for 3
months. The client's lab values (1) will increase due to decreased
include RBC 3.8 million/ mm3, WBC oxygenation caused by anemia; normal RBC
2,000/ mm3, Hgb 9.3 g/dL, platelets male: 4.3-5.9 million/mm3; female: 3.5-5.5
50,000/ mm3. Which of the following million/mm3; decreased with anemia, causes
nursing diagnoses is MOST heart rate and respirations to increase;
appropriate for this patient? normal WBC 4,500-11,000/mm3; decreased
(leukopenia) causes susceptibility to
1. Decreased cardiac output. infection; normal Hgb: male 13.5-17.5 g/dL,
2. Ineffective thermoregulation. female 12-16 g/dL; decreased with anemia
3. Risk for injury.
4. Ineffective airway clearance. (2) no change in temperature

(3) correctdue to low platelet count, normal


platelets 150,000-400,000/ mm3, decrease
causes problems with blood clotting

(4) no information about airway problems


1 A client begins doxepin Strategy: Think about the cause of each
8. hydrochloride (Sinequan) 75 mg PO assessment and how it relates to Sinequan.
tid. The nurse should recommend a
change in the client's therapy if (1) not relevant to this medication
which of the following occurs?
(2) correctdoxepin HCL (Sinequan) is an
1. The client refuses to speak and antidepressant; signs of overdosage include
sits quietly in the room. excitability and tremors
2. The client becomes excitable and
develops tremors. (3) not relevant to this medication
3. The client refuses to eat
breakfast. (4) not relevant to this medication
4. The client sleeps 18 hours a day.
1 A client comes to the clinic (1) vital signs are objective data
9. complaining of severe facial pain. To
collect subjective data from the (2) correctsubjective data is collected in
client, it is MOST important for the the health history or interview
nurse to take which of the following
actions? (3) objective data

1. Obtain the client's vital signs. (4) implementation, complete assessment to


2. Interview the client. determine the problem
3. Inspect the face for grimacing.
4. Administer pain medication.
2 Client comes to the clinic for Strategy: All answers are implementations.
0. hepatitis B vaccine and asks if more Determine the outcome of each answer
than one injection is necessary. choice. Is it desired?
Which of the following responses by (1) Yearly doses are given for flu shots, not
the nurse is BEST? for hepatitis B vaccine
1. "A booster shot is required (2) Correcthepatitis B vaccine is
yearly." repeated at 1 and 6 months
2. "Additional injections are given at (3) Schedule for infant immunizations for IPV
one and six months." and DPT
3. "Repeat doses are given at two (4) Inaccurate
and four months."
4. "Revaccination is not required."
2 A client diagnosed with Addison's (1) not as important as answer choice 4
1. disease is admitted with pneumonia.
The nurse suggests salted broth for (2) not a correct statement for this condition
lunch. The appropriateness of this
decision is based on which of the (3) steroid replacement increases sodium
following statements about retention
Addison's disease?
(4) correctwith decrease in aldosterone,
1. The client requires increased there is an increased excretion of sodium;
sodium intake to prevent sodium intake should be increased
hypotension.
2. A decrease in sodium intake may
lead to seizures.
3. Steroid replacement causes rapid
loss of sodium.
4. Sodium intake should be
increased during periods of stress.
2 The client had an aortic aneurysm resection 2 Strategy: Remember the "comma,
2. days ago. A complete blood count reveals a comma, and" rule. Each part of
decreased red blood cell count. The nursing the answer choice must be correct
assessment is MOST likely to reveal which of for the answer to be correct.
the following?
(1) correctcharacteristic of most
1. Fatigue, pallor, and exertional dyspnea. types of anemia; result of tissue
2. Nausea, vomiting, and diarrhea. hypoxia secondary to inadequate
3. Vertigo, dizziness, and shortness of breath. red blood cells
4. Malaise, flushing, and tachycardia.
(2) indicates GI problems

(3) vertigo not an indication of


anemia

(4) flushing not an indication of


anemia
2 A client has a bovine graft inserted into the Strategy: Determine the outcome
3. left arm for hemodialysis. During the of each answer choice.
immediate postoperative period, which of the
following actions, if performed by the nurse, (1) IVs should not be started in
is BEST? the grafted arm

1. Restart the IV above the level of the graft. (2) correctBP should always be
2. Take blood pressures on the right arm. taken on the opposite arm from
3. Elevate the left arm above the level of the the graft
heart.
4. Check the radial pulse on the left arm q4h. (3) unnecessary

(4) important to assess circulation


in extremity; priority is to prevent
complication
2 A client has an order for furosemide (Lasix) 40 (1) needle gauge is too large
4. mg IV push via a heparin lock. Which of the
following nursing actions is MOST (2) correctfurosemide (Lasix)
appropriate? given IV push should be
administered slowly over 1-2
1. Use a 16- to 18-gauge 1-inch needle for minutes
administration.
2. Administer the medication over 1-2 (3) lock is flushed with heparin
minutes. after administration of the
3. One mL of 1:1,000 heparin flush should be medication
administered before the medication.
4. A primary IV should be started prior to (4) unnecessary
medication administration.
2 A client has a right total hip replacement. The (1) massage may cause emboli
5. client returns from surgery with an IV of
0.45% NaCl infusing into the left forearm at (2) would cause external pressure
100 mL/h. It is MOST important for the nurse on the popliteal space, hip should
to take which of the following actions? not be flexed beyond 90
1. Massage the client's legs to increase (3) correctuse of antiembolic
circulation. hose and/or sequential
2. Elevate the knee gatch to reduce stress on compression devices decreases
the suture line. venous stasis and reduces risk of
3. Apply thigh-high TED hose to promote thrombus formation
venous return.
4. Decrease fluid intake to 1,200 mL to (4) adequate fluid intake (1,500
prevent circulatory overload. mL) prevents dehydration
2 A client has a subclavian triple lumen (1) should be reported to the
6. catheter used for administration of total physician to see if patency can be
parenteral nutrition (TPN). The physician re-established before it is labeled
orders all lumens be flushed with a diluted as clotted off
heparin solution BID. When the nurse
attempts to flush the distal lumen, resistance (2) force should never be used to
is met. The nurse should take which of the irrigate the catheter
following actions?
(3) blood should not be aspirated
1. Clamp off the lumen and label it as "clotted from the catheter
off."
2. Gradually increase the pressure on the (4) correctstreptokinase may be
irrigating solution. used to dissolve clot; if
3. Aspirate blood from the lumen to restore unsuccessful, lumen is labeled as
patency. clotted off
4. Secure the lumen with a Luer-Lock cap and
notify the physician.
2 A client has surgery for cancer of the colon, (1) not appropriate for a client
7. and a colostomy is performed. Before after a colostomy
discharge, the client states that he will no
longer be able to swim. Which of the (2) not appropriate for a client
following responses by the nurse is BEST? after a colostomy

1. "You should begin looking for other areas (3) not appropriate for a client
of interest." after a colostomy
2. "You will have to wear a watertight
dressing over the stoma." (4) correctall activities that the
3. "You cannot go into water that covers the client participated in before the
stoma area." colostomy may be resumed after
4. "You may resume all previous activities." appropriate healing of the stoma
or incisions
2 A client is admitted with a diagnosis of a (1) correctimmobility is a leading
8. fractured right hip. The doctor writes an cause of problems with Buck's
order for Buck's traction. Which of the traction; important to turn client to
following actions, if taken by the nurse, is unaffected side
MOST important?
(2) head of the bed should be
1. Turn the client every 2 hours to the elevated 15-20 because the
unaffected side. supine position can increase
2. Maintain the client in a supine position. problems with immobility
3. Encourage the client to use a bedside
commode. (3) client is on strict bedrest
4. Place a footboard on the bed.
(4) would interfere with the
traction
2 A client is admitted with a diagnosis of Strategy: Answers are all
9. trigeminal neuralgia (tic douloureux) implementations. Determine the
involving the maxillary branch of the affected outcome of each answer choice. Is
nerve. When performing client teaching, it is it desired?
MOST important for the nurse to include
which of the following instructions? (1) unnecessary, does not occur
with this condition
1. "Report an increase in blurred vision."
2. "Eat soft, warm foods." (2) correctintense facial pain
3. "Change positions slowly." experienced along nerve tract is
4. "Chew food on the affected side." characteristic of this condition;
nursing care should be directed
toward preventing stimuli to the
area and decreasing pain

(3) intervention for Mnire's


disease

(4) chewing food on unaffected


side less likely to trigger an attack
3 A client is scheduled for a cardiac Strategy: "Understands teaching"
0. catheterization and the nurse teaches indicates that you are looking for a true
the client about the procedure. Which of statement.
the following statements, if made by the
client to the nurse, indicates an (1) may feel burning sensation when
understanding of the teaching? dye injected

1. "I'm going to feel cold during the (2) on bedrest 8 to 12 h after procedure
procedure." with pressure dressing applied over
2. "I can get up and walk to the catheter insertion site
bathroom immediately after the
procedure." (3) correctperipheral pulses checked
3. "The nurse will be checking my foot every 15 min for 1 h, then every 30 min
pulses after the procedure." for 2 h, then every 4 h
4. "I won't be able to eat for 24 hours
before the procedure." (4) NPO midnight before procedure
3 A client is scheduled to have a (1) correctlow-calcium diet is
1. parathyroidectomy. The nurse is MOST recommended preoperatively
concerned if the client is observed
eating quantities of food from which of (2) diet should be high in phosphorus
the following food groups? and low in calcium

1. Milk products. (3) diet should be high in phosphorus


2. Green vegetables. and low in calcium
3. Seafood.
4. Poultry products. (4) poultry is allowed in the diet
3 A client is transferred to the neurology (1) does not encourage verbal
2. unit after developing right-sided communication
paralysis and aphasia. The nurse should
include which of the following in the (2) inappropriate for the situation
client's plan of care?
(3) correctwill decrease tension and
1. Encourage client to shake head in anxiety; client may understand some of
response to questions. the incoming communication if it is kept
2. Speak in a loud voice during simple; speech may be relearned with
interactions. appropriate support and interventions
3. Speak using phrases and short
sentences. (4) inappropriate for the situation
4. Encourage the use of radio to
stimulate the client.
3 A client received thrombolytic therapy, (1) assessment, unnecessary
3. and the physician orders meperidine
(Demerol) IM for pain. Before (2) correctimplementation,
administering the injection, the nurse complications of thrombolytic therapy
should take which of the following include bleeding, which can occur with
actions? intramuscular injections; nurse should
confer with the physician about the
1. Confirm that all lab work has been appropriateness of the order
completed.
2. Verify the order with the physician. (3) assessment, PTT should be
3. Check the client's PTT. monitored, but this is not a priority
4. Determine that all of the thrombolytic action
agent has infused.
(4) implementation, unnecessary
3 A client taking chlorpromazine (1) possible side effect of antipsychotic
4. (Thorazine) should be instructed to medications, but client can be
notify the nurse immediately if the client instructed on measures to take at home
experiences which of the following? to resolve this problem

1. Dry mouth and nasal stuffiness. (2) possible side effect of antipsychotic
2. Increased sensitivity to heat. medications, but client can be
3. Difficulty urinating. instructed on measures to take at home
4. Weight gain and constipation. to resolve this problem

(3) correctis an anticholinergic


reaction that may become a severe
health problem unless treated

(4) possible side effect of antipsychotic


medications, but client can be
instructed on measures to take at home
to resolve this problem
3 A client tested positive for the tuberculosis (1) correctinitial indications of
5. antibody and was placed on isoniazid (INH) 4 hepatic dysfunction
weeks ago. The nurse observes the client in the
outpatient clinic. The nurse is MOST concerned if (2) seen with pancreatic
which of the following is observed? problems

1. Fatigue and dark urine. (3) seen with renal problems


2. Malaise and glucosuria.
3. Proteinuria and lethargy. (4) is not seen with liver
4. Diluted urine and epigastric distress. problems
3 A client undergoes an appendectomy and the (1) needs high-protein diet to
6. nurse performs discharge teaching. The nurse maintain anabolic state, diet
determines that teaching is effective if the client should contain adequate
states which of the following? carbohydrates and be low in fat

1. "I shall eat a diet low in protein, high in (2) correctsupplemental


carbohydrates, low in fats." vitamin C, iron, and
2. "I shall eat a diet high in protein, high in multivitamins aid in wound
calories, high in vitamin C." healing and formation of RBCs
3. "I shall eat a diet high in protein, low in
calories, low in fat." (3) needs high calories to
4. "I shall eat a diet low in protein, low in promote wound healing
carbohydrates, high in vitamin D."
(4) needs high protein and
high-calorie diet to maintain
anabolic state
3 A client undergoes peritoneal dialysis. The Inflow and intake are recorded
7. physician orders 2 liters to be instilled with a separately. The difference
dwell time of 40 minutes. The nurse measures between inflow and outflow is
the outflow and finds it to be 1,800 mL. During considered intake.
the nurse's shift, the client drinks 700 mL of
fluids and voids 400 mL. Record the client's
intake in milliliters.

Your Response: 2700


Correct Response: 900
3 A client with chronic pain due to cancer receives 1) decreases the amount of
8. meperidine (Demerol) 100 mg PO q4h PRN for pain medication
pain without much relief. Which of the following
changes in narcotic pain management is the (2) decreases the amount of
MOST valid suggestion for the nurse to make to pain medication
the physician?
(3) correctaround-the-clock
1. Decrease medication to twice a day. (ATC) administration of
2. Decrease medication to every 6 h PRN. analgesics is more effective in
3. Administer medication every 4 h around the maintaining blood levels to
clock. alleviate the pain associated
4. Administer medication every 2 h PRN. with cancer

(4) might be too frequent an


interval to administer the
medication
3 A college student has a Mantoux test performed (1) correctwill perform chest
9. at the college health clinic and the result is x-ray
positive. The clinic nurse should take which of
the following actions? (2) premature action,
insufficient information
1. Refer the student to an appropriate center for
further testing. (3) true if active disease
2. Restrict the student's activity until his confirmed, premature action
parents can be notified.
3. Notify the local Public Health Department. (4) premature action,
4. Place the student in an isolation room in the insufficient information
college infirmary.
4 The daughter of a patient diagnosed with (1) closed statement, focus is on
0. cancer asks the nurse, "Do you believe in the nurse and not the client
euthanasia?" Which of the following
responses by the nurse is BEST? (2) focus is on the nurse and not
the client
1. "I think that each person has to decide
this issue for herself." (3) correctopen-ended question,
2. "My religion is opposed to euthanasia." allows client to verbalize
3. "What are your thoughts about
euthanasia?" (4) yes/no question
4. "Did you see the TV program about
euthanasia last night?"
4 An elderly adult is admitted to a medical unit (1) gown unnecessary, trash should
1. with shortness of breath and is diagnosed be left inside room
with an upper respiratory infection (URI).
The client is placed on droplet precautions. (2) wash hands then remove mask,
The nurse administers oral medications to so microbes aren't transferred from
the client. As the nurse leaves the room, the hands to face
nurse should take which of the following
actions? (3) correcthands should be
washed before removing mask to
1. Wash hands, remove the gown and mask, prevent transfer of microbes to
and throw the trash in a container outside of face
the room.
2. Remove the mask, wash hands, and throw (4) gown unnecessary
the trash in a container inside the room.
3. Wash hands, remove the mask, and throw
the trash in a container inside the room.
4. Remove the gown and gloves, wash hands,
remove the mask, and throw the trash in a
container inside the room.
4 An elderly female client is frantically yelling (1) yes/no question, attempt to
2. for the nurse to come into the room. The reason or argue with the client will
nurse enters the room as the client states, only entrench her more firmly into
"See it? It's the devil!" Which of the following this distortion
responses by the nurse is BEST?
(2) attempt to reason or argue with
1. "The devil is here?" the client will only entrench her
2. "Show me where the devil appeared to more firmly into this distortion
you."
3. "I don't see the devil, but I understand (3) correctnurse should not
that he is real to you." reinforce client's hallucinatory
4. "The devil is not here; your mind is playing experiences; direct challenge to
tricks on you." client's belief about sensory-
perceptual intake will only increase
mistrust and conflict between
nurse and client

(4) argumentative, attempt to


reason or argue with the client will
only entrench her more firmly into
this distortion
4 An elderly man is admitted to an inpatient (1) important, but secondary to
3. psychiatric unit with an initial diagnosis of safety issues
psychotic depression. The INITIAL nursing
priority includes which of the following? (2) important, but secondary to
safety issues
1. Clarify perceptual distortions.
2. Establish reality orientation. (3) correctinitial nursing priority
3. Ensure client and milieu safety. for all psychiatric patients is to
4. Increase self-esteem. ensure their safety and the safety
of all members of the milieu

(4) important, but secondary to


safety issues
4 A family member of a client who has sustained (1) incorrect regarding electrical
4. an electrical burn states, "I don't understand burns
why my brother has been here a week. The
burn does not look that bad." Which of the (2) not the most accurate
following responses by the nurse is BEST? statement

1. "Electrical burns are more prone to (3) is true in the immediate post-
infection." burn phase, not a week later
2. "Electrical burns are always much worse
than they look on the outside." (4) correctelectrical burn
3. "Cardiac monitoring is important because injuries are typically more
electrical burns affect cardiac function." injurious to underlying tissue,
4. "Electrical burns can be deceptive because such as nerve and vascular
underlying tissue is also damaged." tissue, which require complex and
timely treatment
4 The home care nurse instructs the spouse of a (1) correctcontraindicated,
5. client about how to perform a wet-to-dry remove dry so wound debris and
abdominal dressing for the client because of necrotic tissue are removed with
an infected abdominal incision. The nurse old dressing
should intervene if which of the following is
observed? (2) purpose of wet-to-dry dressing
is to dbride incision; wetting
1. The client's spouse wets the old dressing dressing before removal defeats
with sterile saline before removing it. purpose of dressing
2. The client's spouse covers the wound with
wet, sterile 4 4s. (3) irrigation of wound sometimes
3. The client's spouse irrigates the wound with used
hydrogen peroxide using a bulb syringe.
4. The client's spouse uses Montgomery straps (4) adhesive is attached to skin
to secure the dressing. and laced to secure dressing,
used when frequent dressing
changes are anticipated
4 The home care nurse visits a client reporting (1) provides information
6. episodes of vomiting for 3 days. The client has regarding the fluid volume level,
a low-grade temperature and complains about but is not the best action for
feeling lethargic. Which of the following evaluation
nursing actions is MOST appropriate to
evaluate for fluid volume deficit? (2) correctdaily weight is the
best way to evaluate for fluid
1. Obtain a urinalysis for casts and specific volume deficit
gravity.
2. Determine client's weight and assess gain (3) provides information
or loss. regarding the fluid volume level,
3. Ask client to provide a 24-hour intake and but is not the best action for
output record. evaluation
4. Determine the quality of the client's skin
turgor. (4) provides information
regarding the fluid volume level
but is not the best action
4 The lab reports a lecithin/sphingomyelin (L/S) (1) no longer necessary, as the
7. ratio of 3:1 for a client who has been on results indicate sufficient lung
bedrest 48 hours in an unsuccessful attempt maturity for safe delivery
to arrest premature labor at 33 weeks'
gestation. Based on this result, the nurse (2) although the lungs are mature
anticipates which of the following? enough for safe delivery, client
would either be allowed to
1. Administration of ritodrine hydrochloride progress naturally to a vaginal
(Yutopar). delivery or would be sectioned,
2. Initiation of an oxytocin (Pitocin) drip. but not induced
3. Delivery of the infant by cesarean section.
4. Continuation of bedrest until otherwise (3) correctbecause the lungs
indicated. are adequately mature, there is
no need to attempt to postpone
labor; delivery by cesarean
section is generally preferred for
preterm infants

(4) is no longer necessary with


adequately mature lungs
4 A middle-aged adult is seen in the Strategy: "MOST important" indicates
8. emergency department for discrimination is required to answer the
complaints of severe right-flank question.
pain. The client is 20 pounds
overweight, lives a sedentary (1) should force fluids to 3,000 mL/day to
lifestyle, and was treated for renal assist client to pass stone
calculi 4 years ago. Which of the
following actions, if performed by (2) not most important, used to identify
the nurse, is MOST important? infection

1. Ensure that the client has (3) not most important, analgesics given to
nothing to eat or drink. reduce discomfort
2. Obtain a "clean-catch" urine
specimen for analysis. (4) correctwill document passage of stone
3. Provide warm packs to relieve and allow composition to be analyzed
discomfort.
4. Measure and strain the client's
urine.
4 A middle-aged female client begins (1) may be used for social phobia or social
9. outpatient therapy sessions for anxiety disorder
management of a phobic disorder.
The nurse identifies which of the (2) may benefit from cognitive-behavioral
following interventions is MOST therapy
effective to reduce the client
symptoms? (3) correctphobic disorders are learned
responses; learned responses can be
1. Antianxiety medication. unlearned through certain techniques, such as
2. Group psychotherapy. behavior modification; systematic
3. Systematic desensitization. desensitization is a form of behavior
4. Biofeedback. modification; is a strategy used in conjunction
with deep muscle relaxation to decrease the
extreme response to anxiety-producing
situations as they are gradually exposed; then
exposure is increased; goal is to eradicate the
phobic response by replacing it with the
relaxation response

(4) one learns to control the autonomic


nervous system; is usually more useful for
reducing stress associated with physiologically
based disorders
5 The mother of an 8-month-old Strategy: Remember therapeutic
0. infant prepares to take her child communication.
home after treatment for bacterial
meningitis. The mother confides to (1) nontherapeutic, diminishes person's
the nurse that she is afraid that concerns and feelings
her child will have brain damage as
a result of his illness. Which of the (2) nontherapeutic to discuss statistics with
following is the BEST response by patients, wrong emphasis for discussion
the nurse?
(3) correctif treated early, good prognosis;
1. "Trust your doctors. They are may be complications and long-term effects
excellent pediatricians and will (seizure disorders, hydrocephalus, impaired
know what to look for." intelligence, visual and hearing defects);
2. "There is a 20% incidence of therapeutic response
residual brain damage after this
type of illness, but the odds are in (4) nontherapeutic, interprets person's feelings
your favor."
3. "It is an unlikely possibility, but
if your child doesn't develop
normally, your pediatrician will
help you with any problems."
4. "You feel guilty about your son's
illness, and that's understandable.
You will feel better after you get
home."
5 The nurse assesses a client (1) loss of peripheral vision occurs with
1. diagnosed with a detached retina. glaucoma; loss of acuity occurs with cataracts
Which of the following
observations supports this (2) occurs with ocular infections
diagnosis?
(3) has no correlation with detached retina
1. Loss of acuity in the peripheral
visual field. (4) correctbright flashes of light and client
2. Increased lacrimation, blurred stating that portion of visual field is dark are
vision. classic symptoms
3. Conjunctivitis, dilated pupils
bilaterally.
4. Photophobia, loss of a portion of
the visual field.
5 The nurse assesses a pregnant client with (1) correctmost common cause of
2. a diagnosis of mitral stenosis and heart mitral valve problems is a history of
failure (HF). The nurse identifies that rheumatic fever with a subsequent
which of the following in the client's complication of carditis, which affects
history has a direct correlation with the the valve
current problem?
(2) does not contribute to mitral valve
1. History of rheumatic fever 4 years ago. disease
2. Presence of ventricular septal defect as
an infant. (3) does not contribute to mitral valve
3. Heart disease in both the maternal and disease
the paternal families.
4. Persistent ear infections and mastoiditis (4) does not contribute to mitral valve
as a child. disease
5 The nurse assists a patient from the bed to Strategy: All answers are
3. the chair for the first time after a right implementations. Determine the
total hip replacement. It is MOST outcome of each answer choice. Is it
important for the nurse to take which of desired?
the following actions?
(1) should not bear weight on affected
1. Assist the patient to stand on the right side, dislocation may occur
leg and pivot to a low soft chair, keeping
her hips straight. (2) correctprevents dislocation
2. Assist the patient to stand on the left
leg and pivot to a straight-backed chair, (3) no weightbearing on affected leg,
flexing her hips slightly. dislocation may occur
3. Ask the patient to bear weight equally
on both legs, bend at the waist, and sit in (4) no weightbearing on affected leg,
a low soft chair. dislocation may occur
4. Assist the patient to stand on both legs
and take a few steps to a straight-backed
chair.
5 A nurse begins a therapeutic relationship (1) helpful data; priority is to
4. with a client diagnosed with generalized determine in what situations the
anxiety disorder. It is MOST important for client becomes anxious
the nurse to obtain which of following
information? (2) helpful data; priority is to
determine in what situations the
1. What the client's priorities are. client becomes anxious
2. How the client views herself.
3. In what situations the client gets (3) correctwill provide necessary
anxious. information in baseline assessment of
4. If anyone in the client's family has had client's anxiety
mental problems.
(4) helpful data but not priority
5 The nurse cares for a 2-month-old infant (1) may not be necessary if
5. diagnosed with reflux. Which of the positioning is effective
following nursing actions is MOST
appropriate? (2) inappropriate

1. Hold the next feeding. (3) client's feedings should be


2. Teach the mother CPR. changed to small-volume, frequent
3. Maintain a normal feeding schedule. feedings
4. Elevate the head of the bed.
(4) correctinfant with reflux should
be maintained in an upright position;
head of the bed should be raised at a
30 angle
5 The nurse cares for a child diagnosed with a Strategy: Answers are a mix of
6. fractured right femur. The child is in balanced assessments and
suspension traction with a Thomas splint and implementations. Does this
Pearson attachment. When the nurse checks the situation require assessment?
patient, the nurse finds the weights on the floor, No. Determine the outcome of
and the child's feet touching the foot of the bed. each answer choice.
Which of the following actions by the nurse is
MOST appropriate? (1) release of weights would
change pull of traction, weight
1. Release the traction weights and reposition should never be released
the patient in bed.
2. Pull on the traction weights while two nurse's (2) pulling on traction weights
aides pull the girl up in bed. would alter proper pull on
3. Steady the traction and ask the child to bend fracture
the left leg and push up in bed.
4. Assess the patient's right leg for proper (3) correctpermits patient to
position and alignment. reposition self and re-establish
pull of traction weights

(4) would not re-establish


proper pull of traction
5 The nurse cares for a client admitted 4 days ago Strategy: Answers are a mix of
7. for treatment of alcohol dependence. The nurse assessments and
notes the client has slurred speech, ataxia, and implementations. Is this a
uncoordinated movements, and complains of a situation that requires
headache. Which of the following actions should validation? Yes.
the nurse take FIRST?
(1) will not provide the data
1. Observe the client for 8 hours to collect that a physical assessment
additional data. would; may be a medical
2. Perform a complete physical assessment. emergency requiring an
3. Collect a urine specimen for a drug screen. immediate intervention
4. Encourage the client to talk about whatever is
bothering him. (2) correctbest way to
identify possible physical
complications of alcohol
dependence is through a
complete physical assessment

(3) should be done after the


physical assessment is
completed

(4) inaccurate because the


symptoms are most likely
caused by physical and not
psychological stressors
5 The nurse cares for a client admitted with a (1) not the most common
8. diagnosis of myocardial infarction (MI) 36 hours occurrence
ago. An appropriate nursing diagnosis is "Risk for
alteration in cardiac output" related to which of (2) not the most common
the following? occurrence

1. Mitral valve collapse. (3) correctmost common


2. Endocarditis. complication following a
3. Ventricular dysrhythmias. myocardial infarction is
4. Hypertensive crisis. dysrhythmia, with ventricular
types being the most serious

(4) client would most probably


experience a decrease rather
than an increase in blood
pressure
5 The nurse cares for a client after an (1) normal process, ileostomies are
9. ileostomy. The nurse is MOST concerned if not irrigated
which of the following is observed?
(2) correctimportant to report
1. The ileostomy functions without daily these findings to the physician;
irrigations. may indicate an obstruction or
2. The stoma appears to be tight, and there is stoma stricture
a decreased amount of stool.
3. A small amount of mucus is seen around (3) anal area is not functional but
the anal area. some mucus may be seen
4. There is weight gain of 5 lb over a 3-week
period of time. (4) should not concern nurse
6 The nurse cares for a client after delivering (1) correctaccepts feelings and
0. an 8 lb, 4 oz girl with diagnosed talipes gives correct information, serial
equinovarus. The woman confides to the casting is used to treat infant
nurse, "I feel so bad that my baby is
abnormal." Which of the following responses (2) doesn't accept person's
by the nurse is BEST? feelings, nontherapeutic

1. "It's understandable that you feel this way, (3) prematurely interprets person's
but there are treatments to correct your feelings as guilt, nontherapeutic
baby's problem."
2. "Your baby is not really abnormal. Her feet (4) nontherapeutic to tell person
just look different because of the way the how to feel
muscles pull."
3. "You have nothing to feel guilty about. The
abnormality is not your fault."
4. "Don't feel bad. Your baby's abnormality
can be corrected surgically."
6 The nurse cares for a client diagnosed with Strategy: All answers are
1. Cushing's syndrome. Which of the following implementations. Determine the
nursing actions is the priority? outcome of each answer choice. Is
it desired?
1. Implement measures to prevent skin
breakdown. (1) clients are susceptible to skin
2. Plan measures to prevent infections. breakdown and infections
3. Teach the client signs and symptoms of
hyperglycemia. (2) clients are susceptible to skin
4. Instigate measures to prevent fluid breakdown and infections
overload.
(3) impaired glucose tolerance
often leads to hyperglycemia, but
is not highest priority

(4) correctrespirations are the


first priority; clients with Cushing's
syndrome are prone to fluid
overload and CHF due to sodium
and water retention
6 The nurse cares for a client diagnosed with Strategy: Think about each answer
2. deep vein thrombosis (thrombophlebitis) of choice.
the left leg. Which of the following is an
appropriate nursing goal for the client? (1) correctimportant to prevent
the complication of pulmonary
1. Decrease inflammatory response in the embolism in clients at high risk
affected extremity and prevent embolus
formation. (2) relates to arterial disease
2. Increase peripheral circulation and
oxygenation of the affected extremity. (3) surgery is not anticipated for
3. Prepare the client and family for this client
anticipated vascular surgery on the affected
extremity. (4) preventing embolism is the
4. Prevent hypoxia associated with the first priority
development of a pulmonary embolus.
6 The nurse cares for a client diagnosed with (1) treatment for acute strain
3. rheumatoid arthritis. The plan of care should or fracture
include which of the following?
(2) joints need extension and
1. Cold packs, immobilization, and hand splints. rotation in addition to flexion to
2. Maintain flexion of the joints and proper body maintain full range of motion
mechanics.
3. Analgesics, physical therapy, and a soft (3) medications used are anti-
mattress on the bed. inflammatory in addition to
4. Heat, range-of-motion exercises, and weight analgesics, a firm mattress
reduction. should be used

(4) correctgoal is to prevent


contractures and minimize
deformity with a balance of
rest and activity
6 The nurse cares for a client diagnosed with Strategy: Determine how each
4. schizophrenia. Which of the following answer choice relates to
statements is MOST descriptive of the affect of a schizophrenia.
patient with schizophrenia?
(1) not indicative of
1. The client answers all questions with one schizophrenia
word.
2. The client laughs while talking about being (2) correctinappropriate
raped. affect, expression of feelings
3. The client exhibits no energy or interest in bizarre for situation
tasks.
4. The client cries while talking about mother's (3) describes depression
death.
(4) appropriate response
6 The nurse cares for a client diagnosed with (1) rejects the client
5. schizophrenia who has become increasingly
withdrawn to the point of mutism. It is MOST (2) correctnurse should
important for the nurse to take which of the maintain contact with client
following actions? but not make demands to
communicate or participate in
1. Ignore the client until he is ready to respond. activities
2. Sit with the client for brief periods of time.
3. Read to the client in a quiet area of the unit. (3) not going to benefit this
4. Encourage the client to play dominos with the client
group.
(4) not going to benefit this
client
6 The nurse cares for a client diagnosed with (1) may feel sleepy due to
6. sickling crisis. The nurse instructs the client medication
about how to use patient-controlled analgesia
(PCA). The nurse determines teaching is (2) preset dose administered
effective if the client states which of the with preset lock-out times
following?
(3) correctitching is a
1. "If I start feeling drowsy, I should notify the common side effect of
nurse." narcotics used in PCA pain
2. "This button will give me enough to kill the management
pain whenever I want it."
3. "If I start itching, I need to call you." (4) indicates a need for further
4. "This medicine will help me feel no pain." teaching or clarification
6 The nurse cares for a client in her third trimester Strategy: Think about the
7. of pregnancy. The nurse is MOST concerned by cause of each symptom and
which of the following assessments? how it relates to pregnancy.

1. The client complains of epigastric pain. (1) correctis usually


2. The client complains of shortness of breath. indicative of an impending
3. The client states she has increased rectal convulsion
pressure.
4. The client has gained of 33 pounds during her (2) expected observation
pregnancy.
(3) expected observation

(4) is important to address, but


is not as high a priority as
answer choice 1
6 The nurse cares for a client receiving (1) appropriate action; grapefruit
8. atorvastatin (Lipitor). It is MOST important juice decreases the enzyme that
for the nurse to report which of the breaks down atorvastatin
following client statements to the physician?
(2) appropriate action
1. "I no longer drink grapefruit juice."
2. "I have my liver enzymes checked (3) not contraindicated
regularly."
3. "I take a daily multivitamin." (4) correctpropranolol decreases
4. "I take propranolol (Inderal)." the effectiveness of atorvastatin
6 The nurse cares for a client recently (1) performed as part of
9. diagnosed with AIDS. The nurse identifies assessment, does not address
the following nursing diagnosis: risk for patient's limited ability to respond
infection. Which of the following to possible infection
interventions by the nurse is BEST?
(2) correctimplementation,
1. Inspect the skin daily for signs of decreases exposure to
breakdown. microorganisms
2. Limit the number of health care personnel
caring for the patient. (3) implementation, done with all
3. Use standard precautions when patients to protect health care
administering parenteral medications. workers
4. Monitor the patient's vital signs q4h.
(4) performed as part of ongoing
assessment
7 Nurse cares for a client with a long history Strategy: All answers
0. of alcohol and drug dependence. It is MOST implementations. Determine
important for the nurse to include which of outcome of each answer choice. Is it
the following as part of the discharge desired?
planning? (1) May be of some help, but will
1. Refer to a social service agency for not directly provide support
assistance with housing. necessary to maintain sobriety
2. Refer to an aftercare center in the (2) May be of some help, but will
community. not directly provide support
3. Encourage participation Alcoholics necessary to maintain sobriety
Anonymous (AA) meetings with a sponsor. (3) Correctself-help groups
4. Ask the client to obtain a prescription for have greatest success rate as
an antidepressant medication. sustained support system in the
community
(4) Is information to indicate client
depressed
7 The nurse cares for an 8-lb, 8-oz newborn. "MOST concerned" indicates a
1. The infant's history indicates the mother complication.
was given magnesium sulfate IV 4 g in 250
mL D5W several hours before delivery. The (1) normal temperature 98.6F
nurse is MOST concerned if which of the (37.0C), magnesium sulfate does
following was observed? not affect temperature

1. Temperature 97.6F (36.5C). (2) normal pulse 120-140 bpm,


2. Apical pulse 140 bpm. magnesium sulfate does not affect
3. Respirations 18/min. cardiac system of infant
4. BP 80/50.Strategy:
(3) correctmagnesium sulfate can
cause slowing of respirations and
hyporeflexia; normal respirations
30-60/min
(4) normal BP 60/40-80/50,
magnesium sulfate does not affect
BP
7 The nurse cares for a patient 36 hours after (1) correctcould indicate
2. a traditional cholecystectomy. The nurse is peritonitis or wound infection
MOST concerned if which of the following is
observed? (2) expected drainage, usually 500-
1000 mL/day initially, will gradually
1. The patient complains of severe decrease
abdominal pain in the right upper quadrant.
2. 500 mL of greenish-brown fluid drained (3) some nausea expected
from the T-tube in the last 24 hours
3. The patient has received an antiemetic (4) results within normal limits,
twice since surgery. normal Hgb: male 13.5-17.5 g/dL,
4. Lab tests indicate an Hgb of 14 g/dL, Hct female 12-16 g/dL, normal Hct:
of 44%, and WBC of 6,000/mm3 male 41-53%, female 36-46%,
normal WBC 5,000-10,000/mm3
7 The nurse cares for a patient admitted with low (1) NSAID (nonsteroidal anti-
3. back pain. The history indicates that the patient inflammatory drug) used for
has hemophilia A. The nurse should question short-term management of
which of the following orders? pain

1. Ketorolac tromethamine (Toradol). (2) analgesic used for


2. Codeine phosphate (Paveral). moderate to severe pain
3. Oxycodone terephthalate (Percodan).
4. Hydromorphone hydrochloride (Dilaudid). (3) correctcontraindicated
for persons with bleeding
disorders, contains aspirin

(4) narcotic analgesic used for


moderate to severe pain
7 The nurse cares for a patient following surgery (1) correctwill see dyspnea,
4. for a coronary artery bypass graft (CABG). Which cough, edema, hemoptysis
of the following symptoms would the nurse
expect to see if the patient was in the early (2) will initially increase and
stages of circulatory overload? then fall due to congestive
heart failure, doesn't fluctuate
1. Change in the character of respirations.
2. Fluctuation in the blood pressure. (3) reflects body's general
3. Reduced tissue turgor. hydration status, mainly
4. Increase in body temperature. shows dramatic changes with
dehydration

(4) would indicate infectious,


inflammatory process, skin
temperature will fall with
circulatory overload
7 The nurse cares for a patient hospitalized with an (1) symptom of acute asthma
5. acute asthma attack. The nurse is MOST attack, doesn't indicate
concerned if which of the following is observed? deterioration of status

1. The patient becomes more diaphoretic. (2) expected with acute


2. The patient's respirations increase from 14 to asthmatic attack, doesn't
16 per minute. indicate deterioration of status
3. The patient's pulse increases from 86 to 100
beats per minute. (3) correctpulse increase is
4. The patient shows increasing pallor. due to decrease in
oxygenation of tissues

(4) subjective symptom,


unreliable indicator of
deterioration of status
7 The nurse cares for a patient recovering from Strategy: All answers are
6. abdominal surgery. During ambulation, the implementations. Determine
patient complains about a dull ache in the left the outcome of each answer
leg. Which of the following actions should the choice. Is it desired?
nurse take FIRST?
(1) correctpromotes venous
1. Place the patient on bedrest with extremity return and decreases venous
elevated. pressure, relieving pain and
2. Place a pillow under the patient's knee. edema
3. Encourage patient to ambulate more
frequently. (2) obstructs venous flow,
4. Obtain thigh-high compression stockings. increasing chance for
thrombus formation

(3) can cause pulmonary


emboli, should be on bedrest 5
to 7 days

(4) used to prevent deep vein


thrombosis, should be on
bedrest initially
7 The nurse cares for a patient who experienced a (1) should be investigated
7. thermal injury 2 weeks ago. The nurse is MOST further, but alone do not
concerned if which of the following is observed? represent significant
compromise
1. Increased heart rate and elevated blood pressure.
2. Temperature of 100.6F (38.1C) and decreased (2) should be investigated
respiratory rate. further, but alone do not
3. Increased heart rate and decreased respiratory represent significant
rate. compromise
4. Increased respiratory rate and decreased blood
pressure. (3) should be investigated
further, but alone do not
represent significant
compromise

(4) correctmay indicate


burn wound sepsis, a life-
threatening complication
of thermal injury
7 The nurse cares for a patient with a three-chamber (1) milking is done only
8. water-seal drainage system (Pleur-evac). When the with order of physician to
nurse checks the patient, the nurse notices that the clear obstruction due to
fluid in the water-seal chamber does not fluctuate. clots, fluid is clear
Which of the following actions by the nurse is BEST?
(2) correctfluctuations
1. Milk the tube gently toward the collection stop with re-expansion of
chamber. lung, x-ray will confirm
2. Anticipate the need for a chest x-ray.
3. Add water to the water seal chamber to re- (3) should be kept at level
establish the system. of 2 mL to maintain
4. Clamp the chest tube and call the physician. negative pressure

(4) only clamp tube when


checking for air leaks or
changing equipment
7 The nurse cares for a postoperative patient. Four (1) implies bladder
9. hours after surgery, the patient voids 200 mL of distention and urinary
urine with a specific gravity of 1.019. The nurse retention, 200 mL divided
should take which of the following actions? by 6 hours = more than 30
mL/h
1. Palpate the patient's lower abdomen for
distention. (2) doesn't recognize
2. Encourage an increased intake of oral fluids. amount and specific
3. Record the time and the amount of urine. gravity as normal in this
4. Encourage the patient to void again in 2 hours. situation

(3) correctamount and


specific gravity normal
(1.010-1.030)

(4) doesn't recognize


amount and specific
gravity as normal in this
situation
8 The nurse cares for clients in the hospital. Which Strategy: All answers are
0. of the following nursing activities BEST promotes implementations. Determine
rest for an elderly hospitalized client? the outcome of each answer
choice. Is it desired?
1. Place a clock at the bedside.
2. Restrict visitors so that the client is alone (1) does not promote rest
during the evening.
3. Tell the client how to call for help if needed. (2) does not promote rest
4. Postpone explanation of further tests that the
client will need. (3) correctelderly client who
feels isolated and unable to
obtain help if needed cannot
rest properly

(4) elderly client will rest


better if s/he understands
what is going on with his/her
health care
8 The nurse cares for clients in the outpatient Strategy: Eliminate the two
1. clinic. The nurse returns to the desk and finds most stable clients. Use the
four phone messages. Which of the following ABCs to determine the most
messages should the nurse return FIRST? unstable client.

1. A client with cold symptoms has an oral (1) elevated temperature


temperature of 103F (39.4C). indicates infection; determine
2. A client with stage II decubitus ulcer reports the underlying cause,
that the dressing has come off. encourage fluids
3. A client is nauseated and has vomited 6 times
in the previous 24 hours. (2) stable client
4. A client complains of leg pain after walking
half a mile. (3) correctassess amount,
character, symptoms of fluid
volume deficit

(4) stable client, complaint


indicates intermittent
claudication
8 The nurse changes the dressing on a woman who Strategy: Remember
2. had a mastectomy 2 days ago. After the nurse therapeutic communication.
removes the old dressing, the client turns her
head away. Which of the following is the BEST (1) correctstates observation
response by the nurse?
(2) doesn't help patient
1. "I notice that you turn your head away as if confront feelings
you don't want to look at your incision."
2. "It's good that you turn your head away while I (3) doesn't deal with
am doing this sterile procedure." avoidance behavior
3. "Your incision looks like it's healing nicely."
4. "Why don't you look at the incision while I (4) nontherapeutic to ask why,
have the old dressing off?" causes patient to be defensive
8 The nurse enters the room of a 17-year-old (1) correctshows bonding
3. mother breast feeding her 6-lb, 7-oz infant girl. behavior of eye-to-eye
Which of the following observations, if made by contact, proceeds to touching
the nurse, BEST indicates that mother-infant and holding
bonding is taking place successfully?
(2) shows distance between
1. The mother is looking into her infant's eyes as mother and infant
she feeds her.
2. The mother and infant are laying side-by-side (3) doesn't involve
in the bed. communication between
3. The mother appears to be relaxed and is mother and infant
reading a book on childcare.
4. The mother interrupts feeding the infant to (4) shows distance between
talk to her roommate. mother and infant
8 The nurse has just received report from the Strategy: Determine the least
4. previous shift. Which of the following clients stable client.
should the nurse see FIRST?
(1) indicates peripheral edema,
1. A client with chronic renal failure treatment includes fluid and sodium
complaining of swollen fingers and ankle restrictions
edema.
2. A client 1 day postoperative after (2) stable client
abdominal surgery with dried blood on the
abdominal dressing. (3) correctindicates
3. A client diagnosed with type 1 diabetes hypoglycemia; symptoms include
mellitus who states, "I have this quivering tachycardia, cold and clammy skin,
feeling in my abdomen." weakness and pallor; check blood
4. A client on high doses of antibiotics for a sugar, offer milk
resistant infection complaining of diarrhea.
(4) common sequelae of antibiotic
therapy, monitor fluid and
electrolytes, check for skin
breakdown
8 The nurse identifies which of the following Strategy: Determine how each
5. clients as being at HIGHEST risk of answer choice relates to pulmonary
developing pulmonary embolus? embolism.

1. A 19-year-old 4 days' postpartum (1) not at risk for pulmonary


diagnosed with a placenta previa at 28 embolism
weeks' gestation.
2. A 22-year-old client diagnosed with (2) at high risk for bleeding
leukemia with a platelet count of
120,000/mm3, hemoglobin 9.0 g/dL. (3) correctobesity, immobility,
3. A 40-year-old man who is obese and and pooling of blood in the pelvic
diagnosed with multiple pelvic fractures due cavity contribute to development of
to a motor vehicle accident 2 days ago. pulmonary emboli
4. A 65-year-old woman who had a fractured
hip repaired 10 days ago and is currently (4) client does not have a high risk
receiving daily physical therapy. for pulmonary emboli
8 The nurse in a long-term care facility reviews (1) appropriate charting of normal
6. the nurse's notes in a client's chart. The urine
nurse is MOST concerned by which of the
following entries? (2) correctblanching or hyperemia
that does not disappear in a short
1. "Foley catheter draining clear urine and time is a warning sign of pressure
the pH is 6.5." ulcers
2. "The client's skin is blanched over the
scapular areas." (3) although the charting is not
3. "Vital signs are within normal limits." objective, blanching of the skin
4. "The client drinks three glasses of orange takes priority because it indicates a
juice every day." problem

(4) appropriate charting


8 The nurse instructs a client with newly (1) too concentrated a
7. diagnosed type 1 diabetes how to treat carbohydrate, will cause
hypoglycemia at home. The nurse should hyperglycemia
instruct the client to do which of the
following actions if symptoms of (2) correctwill correct
hypoglycemia occur? hypoglycemia and stabilize blood
sugar
1. Eat a candy bar.
2. Drink 1/2 cup fruit juice followed by a (3) treatment for hyperglycemia
protein snack.
3. Inject 10 units of Humulin R. (4) used if person becomes
4. Inject glucagon. unconscious
8 The nurse instructs the client about a low- (1) canned foods contain increased
8. sodium, low-cholesterol diet. The nurse salt, and milk contains cholesterol
determines the client teaching is effective if
the client selects which of the following (2) breads contain sodium, and
menus? dairy products and beef contain
cholesterol
1. Canned vegetable soup, applesauce, and
hot chocolate. (3) correctfresh fruits and
2. Cheeseburger, french fries, and skim milk. vegetables are low sodium,
3. Tomato and lettuce salad, roasted chicken, roasted chicken is low cholesterol
and lemonade.
4. Tuna fish sandwich, cottage cheese, and a (4) bread and carbonated
cola. beverages contain sodium
8 The nurse in the outpatient clinic teaches a (1) correctstand slightly behind
9. young adult with a sprained right ankle to patient on strong side
walk with a cane. While teaching the client to
use the cane, how should the nurse be (2) incorrect positioning
positioned?
(3) use a gait belt to assist patient,
1. Standing on the client's left side and don't place hands on patient's
slightly behind the client. arms
2. Standing on the client's right with one
hand on the client's waist. (4) stand slightly behind patient on
3. Standing directly in front of the woman strong side
with both hands on the client's arms.
4. Standing in front of the client on the right
side.
9 The nurse is called to the room of a patient 4 Strategy: Answers are all
0. days after abdominal surgery. The patient implementations. Determine the
had been coughing and said he "felt outcome of each answer choice. Is
something give." The nurse observes that the it desired?
edges of the incision have separated, and a
small loop of the bowel protrudes through (1) semi-Fowler's; too high, puts
the incision. The nurse should position the pressure on abdominal area
patient in which of the following positions?
(2) Trendelenburg position;
1. Head of the bed elevated 30. impedes respiratory excursion
2. Head of the bed tilted down.
3. Head of the bed elevated 15. (3) correctlow Fowler's; reduces
4. Head of the bed elevated 90. stress on suture line, may be
placed supine with hips and knees
bent

(4) high Fowler's; too high, puts


pressure on abdominal area
9 The nurse is caring for a client in the ICU. Strategy: Think about each answer
1. Hemodynamic monitoring is accomplished by choice.
way of a Swan-Ganz catheter. The nurse is
aware that this type of monitoring will (1) not a function of this catheter,
provide which of the following information? and does not reflect hemodynamic
monitoring
1. Measures the circulatory volume in the
coronary arteries. (2) correctCVP readings measure
2. Indirectly measures the pressure in the the pressure in the right ventricle,
ventricles. the Swan-Ganz catheter measures
3. Analyzes the adequacy of pulmonary the pulmonary artery wedge
circulation. pressure, which is an indirect
4. Directly measures the adequacy of carbon reading of the pressure in the left
dioxide exchange. ventricle
(3) not a function of this catheter,
and does not reflect hemodynamic
monitoring

(4) not a function of this catheter,


and does not reflect hemodynamic
monitoring
9 The nurse is caring for an elderly client (1) correctusually use sliding scale
2. diagnosed with type 1 diabetes. The client with regular insulin based on blood
is scheduled for cataract surgery under glucose readings
general anesthesia at 9 AM. The client
usually receives 30 units of NPH and 10 (2) may cause hypoglycemia
units of regular insulin each morning at 7 because client will be NPO when
AM. At 7 AM the morning of surgery, the NPH peaks, NPH intermediate-acting
nurse expects to take which of the following insulin, onset 1-2 hours, peaks 6-12
actions? hours, duration 18-26 hours; regular
insulin short-acting, onset 0.5-1
1. Hold the morning dose of NPH and hour, peaks 2-4 hours, duration 6-8
regular insulin and monitor the blood hours
glucose.
2. Give half the morning dose of NPH insulin (3) client may become
together with the regular insulin and hypoglycemic because NPH will
monitor the blood glucose when the client peak when client is NPO
returns from surgery.
3. Give the full dose of NPH and regular (4) may cause hypoglycemia during
insulin and monitor the blood glucose every surgery
2-4 hours.
4. Give the full dose of regular insulin but
hold the NPH insulin and monitor the blood
glucose until the client goes to surgery.
9 The nurse is caring for a woman completing (1) correctentering transition
3. the first stage of labor. The woman's phase of first stage of labor, slow
husband is at her side and has been shallow breaths needed (pant
coaching her according to exercises they breathing)
learned in childbirth classes. Suddenly the
woman begins to shake and screams, "I (2) doesn't address issue of
can't stand this anymore!" The nurse should breathing pattern needed during
encourage the husband to take which of the transition phase of labor
following actions?
(3) used in conjunction with
1. Instruct his wife to use shallow controlled breathing for Lamaze
respirations during the contractions.
2. Offer his wife ice chips or sips of water to (4) needs support and coaching of
distract her from the pain. husband during transition phase of
3. Stroke his wife's abdomen between labor
contractions.
4. Review with his wife the breathing
pattern needed at each stage of labor.
9 A nurse is the first on the scene of a motor (1) correctimplementation, in an
4. vehicle accident. The victim has sucking open pneumothorax, air enters the
sounds with respirations at a chest wound pleural cavity through an open
site and tracheal deviation toward the wound; placing a sterile dressing
uninjured side. Which of the following loosely over the wound allows air to
actions should the nurse take FIRST? escape but not re-enter the pleural
space
1. Loosely cover the wound, preferably with
a sterile dressing. (2) implementation, would prevent
2. Place a sandbag over the wound. air from escaping
3. Monitor chest wound drainage.
4. Place a firm, airtight, sterile dressing (3) assessment, chest tube has not
over the wound. yet been inserted

(4) implementation, would prevent


air from escaping
9 The nurse makes patient assignments on a (1) correctstable patient with
5. medical/surgical unit. The staff includes one expected outcome
RN, one RN pulled from the pediatric floor,
an LPN/LVN, and a nursing assistant. Which (2) requires frequent assessment for
of the following patients should be assigned hemorrhage, instruct client to avoid
to the RN from the pediatric floor? sneezing, coughing, or straining at
stool
1. A client 1 day postoperative after an
appendectomy. (3) requires frequent monitoring due
2. A client who had a detached retina to hemorrhage
surgically repaired 4 hours ago.
3. A client with a Sengstaken-Blakemore (4) requires assessment and
tube in place. teaching
4. A client 2 days postoperative after a
laminectomy with spinal fusion.
9 The nurse makes rounds on the Strategy: Answers are a mix of
6. postpartum unit. The nurse notes that assessments and implementations. Does
a client's uterus is relaxed. The nurse this situation require validation? No.
should take which of the following Determine the outcome of each
actions? implementation.

1. Put the infant to the woman's breast. (1) correctimplementation, causes


2. Encourage the woman to drink warm natural surge of oxytocin that results in
oral fluids. contraction of uterus
3. Check the woman's pulse and
respirations. (2) implementation, has no effect on
4. Continue to monitor the firmness of contraction of uterus
the uterus.
(3) assessment, not best action, situation
does not suggest that patient is in shock

(4) assessment, needs manual massage


or release of natural oxytocin to contract
uterus
9 The nurse monitors a client in active (1) normal frequency and duration
7. labor who is receiving oxytocin
(Pitocin) 1 mU/min IV. The nurse should (2) normal frequency and duration
stop the infusion if which of the
following is observed? (3) correctcontractions should be less
frequent (longer than 2-minute intervals)
1. The contractions occur at 3-minute and should be of shorter duration (less
intervals and last more than 60 than 90 seconds); allows for longer
seconds. resting time between contractions
2. The contractions occur at 2.5-minute
intervals and last more than 90 (4) normal frequency and duration
seconds.
3. The contractions occur at 2-minute
intervals and last more than 90
seconds.
4. The contractions occur at 2-minute
intervals and last more than 60
seconds.
9 Nurse notes that one of the staff (1) Correctextreme tearing,
8. members caring for clients has a redness, foreign body sensation are
watery discharge from the right eye symptoms of viral conjunctivitis;
and the eye appears red. Which of the highly contagious; infected
following actions, if taken by the nurse, employees cannot work until
is BEST? symptoms have resolved in 3 to 7
1. Send the staff member home. days
2. Assess the staff member's (2) Restrict from patient contact and the
compliance with standard precautions. patient's environment
3. Assign the staff member only to (3) Restrict from patient contact and the
clients with chronic diseases. patient's environment
4. Reassign the staff member to clean (4) Cannot work
the supply closet.
9 The nurse observes a staff member (1) correctindicates Candida, standard
9. enter the room of a client wearing a precautions required
scrub suit. The nurse determines that
the staff member is using the proper (2) requires airborne precautions
precautions if the staff member cares
for which of the following clients? (3) requires droplet precautions

1. A client diagnosed with cancer (4) abscess with no dressing requires


complaining of a sore mouth. contact precaution
2. A client diagnosed with tuberculosis
requiring administration of Rifampin.
3. A client diagnosed with rubella
requiring an IM injection.
4. A client diagnosed with a draining
abscess that is not covered with a
dressing.
10 The nurse observes a student nurse check the (1) appropriate action; if client
0. placement of a nasogastric (NG) tube prior to has for at least 4 hours, pH of
administering an intermittent feeding. Which gastric aspirate is 1 to 4
of the following actions, if performed by the
student nurse, requires an intervention by the (2) correctair injected to
nurse? lungs, pharynx or esophagus
may transmit similar sound
Select all that apply.
(3) acceptable action
1. The student nurse checks the pH of the
contents aspirated from the NG tube. (4) appropriate action; enables
2. The student nurse positions a stethoscope easier aspiration of fluid
on the upper abdomen and listens as air is
introduced into the NG tube. (5) correctnot considered
3. The student nurse uses a large-barreled acceptable procedure; if tube
syringe to aspirate for stomach contents. placed in lungs, may cause
4. The student nurse flushes the NG tube with bubbling
30 ml of air before aspirating fluid.
5. The student nurse places the end of the NG
tube in a cup of water and watches for bubble
formation.
10 The nurse observes late deceleration of the (1) correctwill decrease
1. fetal heart rate while the client is receiving contractions and thus possibly
oxytocin (Pitocin) IV to stimulate labor. Which remove uterine pressure to the
of the following actions should the nurse take fetus, which is possibly cause of
FIRST? deceleration

1. Discontinue the infusion. (2) may help the deceleration,


2. Turn client to the left side. but is not a priority
3. Change the fluids to Ringers lactate.
4. Increase the IV flow rate. (3) will have no influence on the
fetal heart rate

(4) will have no influence on the


fetal heart rate
10 The nurse observes the following patients in (1) crying demonstrates
2. the emergency department (ED). Which of the adequate airway, not life-
following patients should the nurse see FIRST? threatening
1. 8-month-old infant crying loudly with facial (2) not life-threatening
ecchymosis.
2. 12-year-old boy with a possible fractured (3) correctpossibility of
ankle. internal bleeding, life-
3. 34-year-old man with a distended abdomen threatening situation
and splenomegaly.
4. 44-year-old woman with possible whiplash (4) not life-threatening
from an automobile accident.
10 The nurse obtains a history on a client with (1) correctsigns and
3. hyperthyroidism. The nurse should report symptoms of hyperthyroidism
which of the following assessments to the are related to an increased
physician? metabolic rate

1. Anxiety with extreme nervousness. (2) related to a decreased


2. Slow, sluggish pulse. metabolic rate
3. Cool, clammy skin.
4. Husky, slow speech. (3) related to a decreased
metabolic rate

(4) related to a decreased


metabolic rate
10 The nurse performs screening at the local (1) usual finding for the older
4. senior citizens' facility. The nurse is MOST adult
concerned if which of the following is
observed? (2) usual finding for the older
adult
1. A 69-year-old man has a slightly elevated
systolic blood pressure. (3) correctventricular gallop is
2. The nurse has difficulty palpating an apical the earliest sign of HF
pulse on a 74-year-old woman.
3. The nurse auscultates an S3 ventricular (4) may be normal in all age
gallop on a 78-year-old woman. groups
4. An 81-year-old man has a temperature of
98.2F (36.7C).
10 The nurse plans care for a client (1) not highest priority initially, usually
5. immediately after a cesarean section. not seen until 48-72 hours after
Which of the following nursing goals is surgery
MOST important?
(2) correcthemorrhage and shock
1. Prevent infection. are the most life-threatening
2. Prevent fluid and electrolyte conditions that occur after surgery
imbalances.
3. Provide for pain management. (3) not highest priority initially, not
4. Prevent hazards of immobility. life-threatening

(4) not highest priority initially, not


life-threatening
10 The nurse prepares a child diagnosed (1) needed for lifetime to prevent
6. with Addison's disease for discharge. The recurrence of adrenal insufficiency
child's mother asks how long her
daughter must continue receiving (2) needed for lifetime to prevent
replacement therapy. Which of the recurrence of adrenal insufficiency
following responses by the nurse is BEST?
(3) needed for lifetime to prevent
1. "For approximately 6 months." recurrence of adrenal insufficiency
2. "For approximately 1 year."
3. "Until she reaches puberty." (4) correctdisease is caused by
4. "For the rest of her life." deficiency in glucocorticoids, will
always need corticosteroids and
mineralocorticoids
10 The nurse prepares a client for a Strategy: Answers are a mix of
7. herniorrhaphy. It is MOST important for assessments and implementations. Is
the nurse to take which of the following the assessment appropriate 1 hour
actions 1 hour before surgery? before surgery? No. Determine the
outcome of each implementation.
1. Administer an enema.
2. Confirm that the consent form has been (1) should be done earlier than 1 hour
signed. before surgery
3. Perform a preoperative shave and
scrub. (2) correctsurgical consent should
4. Evaluate for food or medication be rechecked before going to surgery
allergies.
(3) should be done earlier than 1 hour
before surgery

(4) assessment; should be done


earlier than 1 hour before surgery
10 The nurse prepares a client for a liver (1) incorrect positioning for procedure
8. biopsy. The nurse should position the
client in which of the following positions? (2) positioned on right side with small
pillow under puncture site for 3 hours
1. Prone with the head turned to the side. after procedure
2. On the right side with the head slightly
elevated. (3) correctelevates the ribs to allow
3. Supine with arms raised above the access to the liver, needle is inserted
head. between two of the lower ribs or
4. On the left side with the bed flat. below the right rib cage

(4) incorrect positioning for procedure


10 The nurse prepares a client for a Strategy: Answers are all
9. paracentesis. It is MOST important for implementations. Determine the outcome
the nurse to take which of the of each answer choice. Is it desired?
following actions?
(1) does not need to be NPO
1. Keep the client NPO 12 hours before
the procedure. (2) correctprevents puncture of bladder
2. Ask the client to void just before
the procedure. (3) bowel preparation unnecessary
3. Initiate a bowel preparation
program 24 hours before the (4) would make it more difficult to drain
procedure. fluid; patient should be positioned sitting
4. Place the client supine during the upright at side of bed with feet supported
procedure.
11 The nurse prepares to suction a client (1) use suction 90-120 mm Hg and #12 or
0. with a new tracheostomy in the #14 suction catheter
postanesthesia recovery room. Which
of the following actions, if performed (2) use a twirling motion to remove
by the nurse, indicates a break in catheter while applying suction
proper technique?
(3) correctbreak in sterile procedure,
1. The nurse sets the suction source at suction mouth after trachea
120 mm Hg and obtains a #14 French
suction catheter. (4) hyperoxygenates client to prevent
2. The nurse inserts the suction hypoxia from procedure
catheter until resistance is met, and
then applies intermittent suction as
the catheter is withdrawn.
3. The nurse suctions the client's
mouth prior to suctioning the
tracheostomy to ensure a patent
airway.
4. The nurse administers oxygen to
the client using an Ambu bag attached
to 100% oxygen prior to suctioning.
11 The nurse receives a bedside report (1) correctclient confidentiality is being
1. from another nurse. The nurse giving violated, nurse should intervene to
the report begins to talk about protect client
another client. Which action by the
nurse receiving the report is MOST (2) does not provide for client
appropriate? confidentiality

1. Ask the nurse to report on this (3) does not provide for client
client only. confidentiality
2. Ask the nurse to lower his/her voice.
3. Ask the nurse to move to another (4) does not provide for client
part of the room. confidentiality
4. Ask the nurse to clarify which client
s/he is reporting on.
11 The nurse reviews histories in the (1) incompatibility only seen with Rh-
2. prenatal clinic. The nurse identifies negative woman
which of the following pregnant
women is MOST likely to have an Rh- (2) correctRh-positive dominant, fetus
incompatibility problem? will be Rh-positive, Rh antibodies from the
mother will break down fetus's blood cells
1. An Rh-positive woman pregnant for
the third time who conceived with an (3) incompatibility only seen with Rh-
Rh-negative man. The woman has negative woman
never received RhoGAM.
2. An Rh-negative woman who (4) infant would be Rh-negative like
conceived with an Rh-positive man. parents, so there would be no
The woman has Rh antibodies. incompatibility
3. An Rh-positive woman who
previously aborted a fetus at 12
weeks' gestation and did not receive
RhoGAM. The woman currently
conceived with an Rh-positive man.
4. An Rh-negative woman who never
received RhoGAM. The woman
currently conceived with an Rh-
negative man.
11 The nurse supervises a student nurse The nurse supervises a student nurse
3. administer a tube feeding to a client administer a tube feeding to a client via a
via a Levin tube. Which of the Levin tube. Which of the following actions,
following actions, if performed by the if performed by the student nurse,
student nurse, indicates a proper indicates a proper understanding of the
understanding of the procedure? procedure?

1. The Levin tube remains unclamped 1. The Levin tube remains unclamped for
for 30 min after the feeding. 30 min after the feeding.
2. Sterile equipment is used to 2. Sterile equipment is used to administer
administer the feeding. the feeding.
3. The amount of the feeding is varied 3. The amount of the feeding is varied
according to the patient's tolerance. according to the patient's tolerance.
4. The tube feeding is given at room 4. The tube feeding is given at room
temperature. temperature.
11 The nurse supervises a student Strategy: "Requires an intervention" indicates
4. nurse teach the client about a that you are looking for an incorrect behavior.
newly prescribed medication.
Which of the following actions, if (1) correctlack of attending behaviors are
observed by the nurse, requires an always a barrier to learning
intervention?
(2) appropriate teaching strategy
1. The student nurse glances at the
clock when instructing the client. (3) appropriate teaching strategy
2. The student nurse uses
culturally appropriate language (4) appropriate teaching strategy
and teaching materials.
3. The student nurse begins
instructions to the client
discussing information that
concerns the client.
4. The student nurse chooses a
time for teaching when there are
no visitors.
11 The nurse talks to a mother in the Strategy: The question is unstated. Read the
5. emergency department (ED) answers to determine the topic of the
immediately after her son's death question. Answers contain both assessments
from sudden infant death and implementations. Is assessment required
syndrome (SIDS). Which of the at this time? No. Determine the outcome of
following actions by the nurse is each implementation.
BEST?
(1) assessment, does not help with current
1. Ask the mother if she has other loss
children at home.
2. Explain the cause of SIDS. (2) implementation, too soon, should allow to
3. Allow the mother to cry and talk vent feelings and experience grief
about her son.
4. Determine how the infant was (3) correctimplementation, needs to go
positioned in bed. through the grieving process

(4) assessment, may make her feel guiltier,


inappropriate at this time
11 The nurse teaches a group of Boy (1) assessment, should be done to check for
6. Scouts how to prevent Lyme ticks that transmit disease; pay particular
disease. Which of the following attention to arms, legs, and hairline
statements, if made by one of the
Boy Scouts to the nurse, indicates (2) protects exposed skin from ticks
that further teaching is necessary?
(3) correctshould not be crushed, remove
1. "When I go on a long hike, I tick with tweezers or fingers and flush down
should check any exposed skin for toilet; burning a tick could spread infection
insects every 4 hours."
2. "When I hike in the woods, I (4) protects exposed skin from ticks, avoid
should wear long pants, socks, and heavily wooded areas
a long-sleeved shirt."
3. "I should remove any ticks by
crushing them firmly against the
skin."
4. "I should reapply insect
repellant every couple of hours
when hiking."
11 The nursing care plan for a 5-year- (1) an increase in sleep could indicate a
7. old child with a closed head injury complication with intracranial pressure
should contain which of the
following? (2) correctearly signs of increased
intracranial pressure are alterations in
1. Encourage child to sleep and orientation
decrease stimuli in the room.
2. Assess orientation to person, (3) negative Babinski is normal
place, and time every hour.
3. Notify the physician regarding a (4) ignores assessment of a potential
negative Babinski reflex. complication; fluid would not be increased for
4. Increase fluid intake to maintain a child with a closed head injury
adequate urinary output.
11 Nursing staff plans use behavior modification Strategy: Determine what is being
8. techniques for an elderly woman who assessed in each answer choice
constantly screams. Which of following and how it relates to screaming.
nursing assessments is necessary to (1) Important because activities of
establish a successful program? daily living can contribute to the
1. Monitor the client's ability to complete her targeted behavior of screaming;
activities of daily living (ADL). assessing only the area of ADLs
2. Assess the client's levels of pain and does not provide comprehensive
correlate it with her response to analgesia. data for developing a behavior
3. Observe the client's behavior at regular management program
intervals to obtain baseline information (2) Important because activities of
related to her screaming. pain can contribute to the
4. Ask the client why she is screaming and targeted behavior of screaming;
document it on her nursing assessment assessing only the area of pain
record. does not provide comprehensive
data for developing a behavior
management program
(3) Correctto design an
effective behavior
modification program,
accurate baseline data must
first be collected about the
target behavior in relation to
frequency, amount, time, and
precipitating factors
(4) Client may be unable to state
why she is screaming; asking
"why" questions is nontherapeutic
11 The nursing team consists of an RN, two (1) stable patient with an
9. LPN/LVNs, and a nursing assistant. The RN expected outcome, assigned to
should care for which of the following the LPN/LVN
clients?
(2) standard, unchanging
1. An infant 2 days postoperative after repair procedure, assign to the nursing
of cleft lip requiring a tube feeding. assistant
2. A preschool child 3 days postoperative
after surgical removal of Wilms' tumor (3) stable patient with an
requiring a bath. expected outcome, assign to the
3. A school-aged child diagnosed with LPN/LVN
osteomyelitis requiring a dressing change.
4. A teenager with a head injury, Glasgow (4) correctGlasgow coma scale
coma scale is 5, requiring personal care. of 5 indicates coma, client
requires frequent assessment
12 An older client comes to the outpatient clinic Strategy: "MOST important"
0. for a routine health screening. The nurse indicates priority. Each answer
learns the client is a retired teacher who lives choice is an implementation.
alone on a limited income. A history indicates Determine the outcome of each
the client drinks about 1,500 mL a day and answer choice. Is it desired?
the client's diet consists primarily of
starches. It is MOST important for the nurse (1) correctprotein needed to
to encourage the client to take which of the slow down degeneration process
following actions? of aging

1. Increase protein intake. (2) necessary, but not most


2. Increase intake of vitamins. important
3. Reduce caloric intake.
4. Reduce fluid intake. (3) necessary, but not most
important

(4) should maintain oral intake


12 An older client has an order for digoxin Strategy: Answers are a mix of
1. (Lanoxin) 0.25 mg PO daily. The nurse assessments and
reviews the following information: apical implementations. Is the
pulse 68/min, respirations 16/min, plasma assessment appropriate? No.
digoxin level 2 ng/mL. Which of the following Determine the outcome of the
actions by the nurse is BEST? implementations.

1. Give the medication on time. (1) medication should be withheld


2. Withhold the medication; notify the
physician. (2) correcttherapeutic plasma
3. Administer epinephrine 1:1,000 stat. level of digoxin is 0.5-2.0 ng/mL
4. Check the client's blood pressure.
(3) not a correct action

(4) assessment, does not address


the issue of the elevated blood
level of digoxin
12 On a home health visit, an elderly client tells (1) jumps ahead to solutions
2. the nurse, "This neighborhood has really without adequately defining the
gone down. I feel like a prisoner in my own problem
home with all the trouble out there." Which
of the following nursing responses by the (2) empathetic response, but does
nurse is BEST? not obtain more information from
the client or encourage the client
1. "Have you and your neighbors formed a to continue
Neighborhood Watch?"
2. "It must be very difficult for you to live in (3) false reassurance
this neighborhood."
3. "I see a lot of police cars, so you should be (4) correctassessing the basis
pretty safe." for client's fears and encouraging
4. "Tell me what has happened to make you client to talk about them is the
feel that you are not safe." first positive step
12 A patient complains of pain after an (1) correctwould reduce stress
3. appendectomy. After administering an on suture line and provide for
analgesic, the nurse should take which of the comfort
following actions?
(2) would put pressure on
1. Elevate the head of the bed 30-45. popliteal space, would restrict
2. Place a pillow behind the patient's knees. circulation and increase risk of
3. Elevate the knee gatch on the bed 30. thrombophlebitis
4. Position the client supine with a small
pillow under the head. (3) would put pressure on
popliteal space, would restrict
circulation and increase risk of
thrombophlebitis

(4) does not reduce stress on


suture line
12 A patient is returned to the room at 10 AM (1) should turn, cough, and deep-
4. following laparoscopic gall bladder surgery. breathe patient every 2 hours to
The nurse plans to get the patient out of bed prevent postoperative
for the first time at 6 PM. In preparation for complications, but would not help
this activity, the nurse should take which of with ambulation
the following actions?
(2) correctreduction of pain will
1. Ask the patient to cough and deep-breathe allow patient to cooperate with
at 4 PM. activities designed to reduce
2. Offer pain medication to the patient at postoperative complications such
5:30 PM. as ambulation
3. Turn the patient from side to side at noon
and 4 PM. (3) should turn patient every 2
4. Encourage the patient to use the incentive hours to prevent postoperative
spirometer. complications, but would not help
with ambulation
(4) used to promote complete
lung expansion and prevent
respiratory complications
following surgery, but would not
help with ambulation
12 A patient is returned to the room following an Strategy: Determine how each
5. appendectomy. The nurse notices a large answer choice relates to an
amount of serosanguineous drainage on the appendectomy.
dressing. It is MOST important for the nurse to
obtain an answer to which of the following (1) doesn't indicate
questions? understanding that drainage
may be normal after this surgery
1. "Were there any intraoperative
complications?" (2) first dressing usually changed
2. "Has the dressing been changed?" by physician
3. "Why didn't the recovery room nurse report
any drainage?" (3) doesn't indicate
4. "Was a tissue drain placed during surgery?" understanding that drainage
may be normal after this surgery

(4) correctdrain is frequently


placed during surgery to prevent
accumulation in wound, dressing
should be reinforced
12 A patient received meperidine (Demerol) 75 (1) should ambulate patient
6. mg IM 2 hours ago for complaints of pain. The safely to prevent hazards of
patient turns on the call light and tells the immobility
nurse he has to go to the bathroom. The
physician ordered bathroom privileges. The (2) correctside effects of
nurse should take which of the following medication include decreased
actions? BP, orthostatic hypotension,
bradycardia
1. Obtain a bedside commode for the patient's
use and provide privacy. (3) easier for patient to use
2. Help the patient to sit on the side of the bed bathroom than to use bedpan
before proceeding to the bathroom.
3. Provide a bedpan for the patient's use and (4) an additional nurse not
pull the curtains. necessary, before ambulating
4. Ask two nurses to assist the patient to the should sit on side of bed to allow
bathroom. body to adjust to change in
position
12 The physician orders indomethacin (Indocin) Strategy: "MOST important"
7. 25 mg PO bid for a client. It is MOST important indicates discrimination is
for the nurse to make which of the following required to answer the question.
statements?
(1) correctreduces GI upset
1. "Take this medication with food."
2. "Take this medication one hour before (2) risk of GI upset
meals."
3. "Take this medication one hour after meals." (3) should be given with food
4. "Take this medication with orange juice."
(4) risk of GI upset
12 The physician orders meperidine (Demerol) 50 Strategy: Answers are a mix of
8. mg IM every 3-4 h PRN for pain for a client. assessments and
The client asks the nurse for the medication at implementations. Does this
bedtime. Before administering the pain situation require assessment?
medication, the nurse should take which of the Yes. Is there an appropriate
following actions? assessment? Yes.

1. Determine if the pain is psychological. (1) should assess patient first


2. Read the client's chart to see if the client
has a history of addiction. (2) not highest priority, should
3. Try several other comfort and pain relief assess patient first
measures.
4. Ask the client about the location, character, (3) need to assess before
and intensity of the pain. implementing action

(4) correctassessment first


step in nursing process
12 The physician prescribes cimetidine Strategy: "Further teaching" indicates
9. (Tagamet) 300 mg PO qid for an elderly incorrect information
client. The nurse instructs the client about
the medication. Which of the following (1) taking with meals ensures
statements, if made by the client, consistent therapeutic effect
indicates further teaching is needed?
(2) common side effect, usually
1. "I'll take this pill with meals and before subsides
bed."
2. "I may experience mild diarrhea for a (3) correctno change in stool color
while."
3. "My stools may change color while I'm (4) side effect seen with medication
on this medication."
4. "I should call my doctor if I get an acne-
like rash."
13 A preschooler is brought to the emergency (1) dizziness not seen with aspirin
0. department after ingesting a bottle of overdose
baby aspirin. The nurse should observe
the preschooler for which of the following (2) nosebleed may occur, but not
signs and symptoms? paralysis

1. Nausea and vertigo. (3) may see hyperventilation with use


2. Epistaxis and paralysis. of aspirin, does not affect heart
3. Dysrhythmia and hypoventilation. rhythm
4. Tinnitus and gastric distress.
(4) correctsymptoms of overdose
13 Prior to a caesarean section delivery, a Strategy: Determine how each
1. client is treated for abruptio placenta. The answer choice relates to DIC.
nurse cares for the woman during the
postpartum period. Which of the following (1) may indicate hemorrhage or
symptoms is suggestive of disseminated sepsis
intravascular coagulation (DIC)?
(2) results normal, DIC would be
1. The client's vital signs are: BP 90/58, reflected in clotting studies (PT, PTT)
temperature 101.0F (38.3C), pulse
112/min, respirations 18/min. (3) nonspecific, could be related to
2. The client's laboratory results are Hgb anesthesia or pain medication
13 g/dL, HCT 40%, WBC 7,000/ mm3.
3. The client is nauseated, lethargic, and (4) correctDIC is an acquired
has vomited three times. clotting disorder from
4. There is oozing blood from the overstimulation, prolonged oozing
venipuncture site and abdominal incision. from sites of minor trauma first
symptom
13 A teenager comes to the clinic complaining Strategy: All answers are
2. of fatigue, a sore throat, and flu-like implementations. Determine the
symptoms for the previous 2 weeks. outcome of each answer choice. Is it
Physical exam reveals enlarged lymph desired?
nodes and temperature of 100.3F
(37.9C). Which of the following (1) mononucleosis is spread by direct
statements by the nurse is BEST? contact

1. "Cover your mouth and nose when you (2) no reason to be isolated
sneeze or cough."
2. "Eat in a separate room away from your (3) correctsymptoms indicate
family." mononucleosis, spread by direct
3. "Don't share your drinking glass or contact; advise family to avoid
silverware with anybody." contact with cups and silverware for
4. "Stay in your room until all of your about 3 months
symptoms are gone."
(4) clients with mononucleosis are
not isolated
13 The nurse plans care for a client returning Strategy: Topic of the question
3. from surgery after a bowel resection with unstated. Read choices to determine
an IV of 0.9 % NaCl infusing at 100 mL/h topic. "BEST" indicates priority
into the left wrist. Which of the following question. Answers implementations.
actions, if performed by the nurse, is Determine outcome of each answer
BEST? choice. Is it desired?
1. Change the IV tubing each time a new (1) Unnecessary, changed every 48
IV solution is hung. to 72 h
2. Cleanse the IV site with an alcohol swab (2) Should move swab in a circular
using long strokes. motion outward
3. Limit manipulation of the cannula at the (3) Correctwill prevent
IV insertion site. dislodgment of needle
4. Adjust the drop rate to keep the total (4) Should give IV at rate ordered by
volume of IV fluids on schedule. physician, don't play "catch-up" with
fluids
13 The nurse prepares a patient for an 8:00 (1) Correctclient given general
4. AM outpatient electroconvulsive (ECT) anesthesia for ECT; NPO after
treatment. Which of the following midnight
questions is MOST important for the nurse (2) not relevant to ECT
to ask? (3) not most important
1. "Did you have anything to eat or drink (4) memory loss is an expected
before you came in today?" outcome
2. "Have you had any headaches since
your last treatment?"
3. "Who came with you to the hospital
today?"
4. "Have you had much memory loss since
you began your treatments?"
13 To assist a parent to provide appropriate (1) correctchild is going through
5. foods for a 3-year-old, the nurse identifies autonomy versus shame and doubt
which of the following as the HIGHEST stage; finger foods allow child the
priority? necessary independence for this
stage
1. Provide the child with finger foods.
2. Allow the child to eat her favorite foods. (2) child may eat food without
3. Encourage a diet higher in protein than appropriate nutrients
in other nutrients.
4. Limit the number of snacks during the (3) inappropriate for a 3-year-old
day. child

(4) inappropriate for a 3-year-old


child
13 When caring for an elderly client with a Strategy: All answers are
6. depressed affect, which of the following implementations. Determine the
nursing actions is MOST appropriate to outcome of each answer choice. Is it
help the client to complete activities of desired?
daily living?
(1) will not increase the client's
1. Medicate the client before the activities independence and may interfere with
begin. the client's self-esteem
2. Develop a written schedule of activities,
allowing extra time. (2) correctwritten schedule with
3. Assist the client with grooming built-in extra time will allow client to
activities so it doesn't take as long. understand what is expected and will
4. Provide frequent forceful direction to allow client to participate at a slower
keep the client focused. pace

(3) will not increase the client's


independence; allow extra time for
care

(4) will not increase the client's


independence and may interfere with
the client's self-esteem
13 Which of the following activities (1) psychosocial, speaks to his
7. documented by the recreational therapist psychosocial status, but is not an
following a community reorientation indication for discharge
outing for a client with paraplegia
indicates to the nurse the client's (2) psychosocial, addresses social
readiness for discharge? skills, but is not an indication for
discharge
1. The client states that he enjoyed being
outside the hospital environment. (3) physical, not pertinent for a
2. The client participated in a structured paraplegic
team sport by keeping score.
3. The client independently ordered his (4) correctphysical, these skills are
meal and fed himself. requisite for discharge
4. The client is independent in transfers
and wheelchair mobility.
13 Which of the following assessments is (1) correctduring a crisis such as
8. priority when documenting the nursing hospitalization, children are able to
history of a 2-year-old child? establish a sense of security through
consistency of the rituals and
1. The child's rituals and routines at home. routines from home
2. The child's understanding of
hospitalization. (2) important, but not as critical to
3. The child's ability to be separated from the planning of the child's hospital
the parents. care
4. The parent's methods for dealing with
the child's temper tantrums. (3) important, but not as critical to
the planning of the child's hospital
care

(4) important, but not as critical to


the planning of the child's hospital
care
13 Which of the following guidelines Strategy: Remember growth and development.
9. is appropriate for the nurse to give
a mother concerning the (1) correctnormal for developmental stage,
developmental stage of her 7-year- beginning to show independence from parents
old daughter?
(2) nightmares are frequently experienced at
1. The child's periods of shyness this age
are to be expected.
2. Nightmares are not (3) should be encouraged to be independent,
characteristic of this age and not responsible for sibling, inappropriate for
should be investigated. this age group
3. The child should be encouraged
to care for her younger sister. (4) should allow child to be increasingly
4. Punishment may be necessary independent without punishment
for acts of independence.
14 Which of the following nursing Strategy: All answers are implementations.
0. actions is the priority for an infant Determine the outcome of each answer
admitted with a positive stool choice. Is it desired?
culture for Salmonella?
(1) may be appropriate, but is not a priority
1. Change diet to clear liquids. over answer choice 3, which will prevent
2. Initiate intravenous fluids. transmission
3. Maintain contact precautions.
4. Apply cloth diapers. (2) may be appropriate, but is not a priority
over answer choice 3, which will prevent
transmission

(3) correctprevents transmission of this


bacterium to other individuals

(4) may be appropriate, but is not a priority


over answer choice 3, which will prevent
transmission
14 Which of the following nursing (1) medication is never added to the infant's
1. approaches is MOST appropriate to formula feeding
use while administering an oral
medication to a 4-month-old (2) correctis a convenient method for
infant? administering medications to an infant

1. Place the medication in 45 mL of (3) medication is never added to the infant's


formula. formula feeding
2. Place the medication in an
empty nipple and allow the infant (4) infant is never placed in a reclining position
to suck. during procedure due to potential for
3. Place the medication in a full aspiration
bottle of formula.
4. Administer the medication using
a plastic syringe with the infant in
the reclining position.
14 Which of the following nursing (1) describes hallucinations
2. observations documented in the
client's chart MOST clearly (2) describes altered thought processes
indicates the client's mood?
(3) correctgives data that reflect client's
1. "Client states, 'I see snakes feelings, tone, and behavior associated with
climbing on the walls at all times those feelings, as well as content area of
of the day.'" conversation that evoked that mood
2. "Unable to sustain a train of
thought for long periods of time (4) describes disorientation
during history-taking."
3. "Clenches her fists and shouts in
an angry tone of voice when asked
about family problems."
4. "Is unaware of where she is,
what day and year it is, or what
time it is."
14 Which of the following (1) both parents are carriers of the abnormal
3. statements, if made to the nurse, gene
indicates parental understanding
about the cause of their (2) both parents are carriers of the abnormal
newborn's diagnosis of cystic gene
fibrosis (CF)?
(3) there is a 25% chance of passing the
1. "The gene came from my disease on to any of their offspring
husband's side of the family."
2. "The gene came from my wife's (4) correctcystic fibrosis is inherited by an
side of the family." autosomal recessive trait
3. "There is a 50% chance that our
next child will have the disease."
4. "Both of us carry a recessive
trait for cystic fibrosis."
14 Which of the following strategies Strategy: Answers are a mix of assessments
4. is MOST therapeutic as the nurse and implementations. Is the assessment
tries to analyze a bulimic client's appropriate? No. Determine the outcome of
eating habits and the each implementation.
circumstances that precipitate the
client's eating problems? (1) assessment, should be done after a
food/thought/feelings/actions journal
1. Observe family communication
patterns at a "monitored (2) implementation, should be done after a
mealtime." food/thought/feelings/actions journal
2. Distract the client at mealtime.
3. Assign the client a (3) correctimplementation, nurse is trying to
food/thought/feelings/actions analyze and understand what triggers the
journal. client's binging and purging activities, so
4. Assign the client to write a therapeutic nursing intervention of assigning a
"lifeline" in relation to eating thought/feelings/actions (T/F/A) journal relating
behaviors. to client's eating behaviors will be most helpful
to the nurse and therapeutic to the client; after
this information is gained and reviewed,
collaboration by the nurse and client on other
strategies such as delay and distraction
techniques, stress reduction, and developing a
"lifeline" in relation to eating behaviors will
further benefit the client

(4) implementation, should be done after a


food/thought/feelings/actions journal
14 While a 2-day-old infant is in (1) not appropriate or true regarding this
5. surgery for repair of spina bifida, condition
the infant's mother expresses
concern to the nurse because the (2) not appropriate or true regarding this
doctor told her the infant would condition
be confined to a wheelchair. Which
of the following statements, if (3) not appropriate or true regarding this
made by the nurse, is BEST? condition

1. "Physical therapy can restore (4) correctspinal nerves that are destroyed by
the function to affected muscles." the myelomeningocele cannot be corrected;
2. "Orthopedic devices will allow nothing can return function to portions of the
your child to strengthen lower body that are innervated by the spinal nerves
extremity muscles." below the site of the myelomeningocele
3. "Corrective surgery will return
function to the affected muscles."
4. "The corrective surgery will not
change your child's physical
disability."
1. A white Answer C. Obesity is a risk factor for CVA. Other risk factors
female client is include a history of ischemic episodes, cardiovascular
admitted to an disease, diabetes mellitus, atherosclerosis of the cranial
acute care vessels, hypertension, polycythemia, smoking,
facility with a hypercholesterolemia, oral contraceptive use, emotional
diagnosis of stress, family history of CVA, and advancing age. The
cerebrovascula client's race, sex, and bronchial asthma aren't risk factors
r accident for CVA.
(CVA). Her
history reveals
bronchial
asthma,
exogenous
obesity, and
iron deficiency
anemia. Which
history finding
is a risk factor
for CVA?
a. Caucasian
race
b. Female sex
c. Obesity
d. Bronchial
asthma
2. 2 While hospitalized a client is very
worried aboiut business activities.
The client spends a great deal opf
time on the phone and with
collegues instead of resting. Which
principle of need therapy applies
to this client?
1. his higher level need cannot be
met unless the lower level
physicological need is met
2. His lower level physiological
needs are being deferred while
higher need are addressed.
3. The higher need takes
precedence and the lower need no
longer must be met.
4. It is necessary for someone else
to meet his higher level needs so
he can focus on the lower level
needs.
3. 2. The Answer B. Fatigue is a common
nurse is symptom in clients with multiple
teaching sclerosis. Lowering the body
a female temperature by resting in an air-
client conditioned room may relieve
with fatigue; however, extreme cold
multiple should be avoided. A hot bath or
sclerosis. shower can increase body
When temperature, producing fatigue.
teaching Muscle relaxants, prescribed to
the client reduce spasticity, can cause
how to drowsiness and fatigue. Planning
reduce for frequent rest periods and naps
fatigue, can relieve fatigue. Other
the nurse measures to reduce fatigue in the
should client with multiple sclerosis
tell the include treating depression, using
client to: occupational therapy to learn
a. take a energy conservation techniques,
hot bath. and reducing spasticity.
b. rest in
an air-
condition
ed room
c.
increase
the dose
of muscle
relaxants.
d. avoid
naps
during
the day
4. 3 A nurse is planning a workshop on
health promotion for older adults.
Which topic will be included?
1. prevention of falls
2. cardiovascular risk factors
3. adequate sleep
4. how to stop smoking
5. 3 Which one of the followin is an
example of the emotional
component of wellness?
1. the client chooses healthy foods
2. a new father decides to take
parenting classes
3. A client expressess frustration
with her partner's substance abuse
4. A widow with no family decides
to join a bowling league
6. 3 Using maslow's framework which
statement charecterizes the
highest level of need?
1. Nurse my pain is severe. . . is it
time for my shot?
2. I felt welcomed when i first
joined the group and i look forward
to the monthly meetings
3. Im very proud of recieving the
employee of the month award
4. There have been home breakins
with burglary in our neighborhood.
we are thinking of moving.
7 3. A male client is having Answer D. Protecting the client from injury is the
. a tonic-clonic seizures. immediate priority during a seizure. Elevating the head of
What should the nurse do the bed would have no effect on the client's condition or
first? safety. Restraining the client's arms and legs could cause
a. Elevate the head of the injury. Placing a tongue blade or other object in the
bed. client's mouth could damage the teeth
b. Restrain the client's
arms and legs.
c. Place a tongue blade in
the client's mouth.
d. Take measures to
prevent injury.
8 3 kinds of ICP monitoring -Intraventricular catheter (ventriculostomy)
. systems -Subacrachnoid screw or bolt
-Epidural or subdural sensor
9 4 Based on the life changes index which individual would
. have an increased possibility of illness in the near future?
1. A 25 year old man who recently married his high
school sweetheart
2. 1 35 year old man fired from his job
3. a 40 year old woman beginning a nursing program
4. 50 year old woman whos husband died a month ago
1 4 which is the best response by the nurse if the client fails
0 to follow the information or teaching provided?
. 1. give up because the client doesnt want to change
2. develop a tough approach
3. guide the client to create a plan of action
4. Remind the client of previous successes.
1 4. A female client with 4. Answer A. The nurse should inform the client that the
1 Guillain-Barr syndrome paralysis that accompanies Guillain-Barr syndrome is
. has paralysis affecting the only temporary. Return of motor function begins
respiratory muscles and proximally and extends distally in the legs.
requires mechanical
ventilation. When the
client asks the nurse
about the paralysis, how
should the nurse
respond?
a. "You may have difficulty
believing this, but the
paralysis caused by this
disease is temporary."
b. "You'll have to accept
the fact that you're
permanently paralyzed.
However, you won't have
any sensory loss."
c. "It must be hard to
accept the permanency of
your paralysis."
d. "You'll first regain use
of your legs and then your
arms."
1 5. The nurse is working on 5. Answer A. The client who has had spinal surgery, such
2 a surgical floor. The nurse as laminectomy, must be logrolled to keep the spinal
. must logroll a male client column straight when turning. The client who has had a
following a: thoracotomy or cystectomy may turn himself or may be
a. laminectomy. assisted into a comfortable position. Under normal
b. thoracotomy. circumstances, hemorrhoidectomy is an outpatient
c. hemorrhoidectomy. procedure, and the client may resume normal activities
d. cystectomy. immediately after surgery.
1 6. A female client with a Answer B. Because CT commonly involves use of a
3. suspected brain tumor is contrast agent, the nurse should determine
scheduled for computed whether the client is allergic to iodine, contrast
tomography (CT). What should dyes, or shellfish. Neck immobilization is necessary
the nurse do when preparing only if the client has a suspected spinal cord injury.
the client for this test? Placing a cap over the client's head may lead to
a. Immobilize the neck before misinterpretation of test results; instead, the hair
the client is moved onto a should be combed smoothly. The physician orders a
stretcher. sedative only if the client can't be expected to
b. Determine whether the remain still during the CT scan.
client is allergic to iodine,
contrast dyes, or shellfish.
c. Place a cap over the client's
head.
d. Administer a sedative as
ordered.
1 7. During a routine physical 7. Answer B. To prevent the attached muscle from
4. examination to assess a male contracting, the nurse should support the joint
client's deep tendon reflexes, where the tendon is being tested. The nurse should
the nurse should make sure to: use the flat, not pointed, end of the reflex hammer
a. use the pointed end of the when striking the Achilles tendon. (The pointed end
reflex hammer when striking is used to strike over small areas, such as the
the Achilles tendon. thumb placed over the biceps tendon.) Tapping the
b. support the joint where the tendon slowly and softly wouldn't provoke a deep
tendon is being tested. tendon reflex response. The nurse should hold the
c. tap the tendon slowly and reflex hammer loosely, not tightly, between the
softly thumb and fingers so it can swing in an arc.
d. hold the reflex hammer
tightly.
1 8. A female client is admitted 8. Answer D. Because the client is disoriented and
5. in a disoriented and restless restless, the most important nursing diagnosis is
state after sustaining a risk for injury. Although the other options may be
concussion during a car appropriate, they're secondary because they don't
accident. Which nursing immediately affect the client's health or safety.
diagnosis takes highest
priority in this client's plan of
care?
a. Disturbed sensory
perception (visual)
b. Self-care deficient:
Dressing/grooming
c. Impaired verbal
communication
d. Risk for injury
1 9. A female client with 9. Answer B. This comment best supports a nursing
6. amyotrophic lateral sclerosis diagnosis of Powerlessness because ALS may lead
(ALS) tells the nurse, to locked-in syndrome, characterized by an active
"Sometimes I feel so and functioning mind locked in a body that can't
frustrated. I can't do anything perform even simple daily tasks. Although Anxiety
without help!" This comment and Risk for disuse syndrome may be diagnoses
best supports which nursing associated with ALS, the client's comment
diagnosis? specifically refers to an inability to act
a. Anxiety autonomously. A diagnosis of Ineffective denial
b. Powerlessness would be indicated if the client didn't seem to
c. Ineffective denial perceive the personal relevance of symptoms or
d. Risk for disuse syndrome danger.
1 10. For a male client with 10. Answer C. The goal of treatment is to
7. suspected increased intracranial prevent acidemia by eliminating carbon dioxide.
pressure (ICP), a most That is because an acid environment in the
appropriate respiratory goal is to: brain causes cerebral vessels to dilate and
a. prevent respiratory alkalosis. therefore increases ICP. Preventing respiratory
b. lower arterial pH. alkalosis and lowering arterial pH may bring
c. promote carbon dioxide about acidosis, an undesirable condition in this
elimination. case. It isn't necessary to maintain a PaO2 as
d. maintain partial pressure of high as 80 mm Hg; 60 mm Hg will adequately
arterial oxygen (PaO2) above 80 oxygenate most clients.
mm Hg
1 11. Nurse Maureen witnesses a 11. Answer C. If a neck injury is suspected, the
8. neighbor's husband sustain a fall jaw thrust maneuver is used to open the airway.
from the roof of his house. The The head tilt-chin lift maneuver produces
nurse rushes to the victim and hyperextension of the neck and could cause
determines the need to opens the complications if a neck injury is present. A
airway in this victim by using flexed position is an inappropriate position for
which method? opening the airway.
a. Flexed position
b. Head tilt-chin lift
c. Jaw thrust maneuver
d. Modified head tilt-chin lift
1 12. The nurse is assessing the 12. Answer B. Motor testing in the unconscious
9. motor function of an unconscious client can be done only by testing response to
male client. The nurse would plan painful stimuli. Nail bed pressure tests a basic
to use which plan to use which of peripheral response. Cerebral responses to pain
the following to test the client's are tested using sternal rub, placing upward
peripheral response to pain? pressure on the orbital rim, or squeezing the
a. Sternal rub clavicle or sternocleidomastoid muscle.
b. Nail bed pressure
c. Pressure on the orbital rim
d. Squeezing of the
sternocleidomastoid muscle
2 13. A female client admitted to 13. Answer C. The client having a magnetic
0. the hospital with a neurological resonance imaging scan has all metallic objects
problem asks the nurse whether removed because of the magnetic field
magnetic resonance imaging may generated by the device. A careful history is
be done. The nurse interprets obtained to determine whether any metal
that the client may be ineligible objects are inside the client, such as orthopedic
for this diagnostic procedure hardware, pacemakers, artificial heart valves,
based on the client's history of: aneurysm clips, or intrauterine devices. These
a. Hypertension may heat up, become dislodged, or malfunction
b. Heart failure during this procedure. The client may be
c. Prosthetic valve replacement ineligible if significant risk exists.
d. Chronic obstructive pulmonary
disorder
2 14. A male client is having a 14. Answer D. The client undergoing lumbar
1. lumbar puncture performed. The puncture is positioned lying on the side, with the
nurse would plan to place the legs pulled up to the abdomen and the head
client in which position? bent down onto the chest. This position helps
a. Side-lying, with a pillow under open the spaces between the vertebrae.
the hip
b. Prone, with a pillow under the
abdomen
c. Prone, in slight-Trendelenburg's
position
d. Side-lying, with the legs pulled
up and head bent down onto
chest.
2 15. The nurse is positioning the 15. Answer B. The head of the client with
2. female client with increased increased intracranial pressure should be
intracranial pressure. Which of positioned so the head is in a neutral midline
the following positions would the position. The nurse should avoid flexing or
nurse avoid? extending the client's neck or turning the head
a. Head mildline side to side. The head of the bed should be
b. Head turned to the side raised to 30 to 45 degrees. Use of proper
c. Neck in neutral position positions promotes venous drainage from the
d. Head of bed elevated 30 to 45 cranium to keep intracranial pressure down.
degrees
2 16. A female client has clear 16. Answer D. Leakage of cerebrospinal fluid (CSF)
3. fluid leaking from the nose from the ears or nose may accompany basilar skull
following a basilar skull fracture. CSF can be distinguished from other body
fracture. The nurse assesses fluids because the drainage will separate into bloody
that this is cerebrospinal and yellow concentric rings on dressing material,
fluid if the fluid: called a halo sign. The fluid also tests positive for
a. Is clear and tests negative glucose.
for glucose
b. Is grossly bloody in
appearance and has a pH of 6
c. Clumps together on the
dressing and has a pH of 7
d. Separates into concentric
rings and test positive of
glucose
2 17. A male client with a 17. Answer D. The most frequent cause of autonomic
4. spinal cord injury is prone to dysreflexia is a distended bladder. Straight
experiencing automatic catheterization should be done every 4 to 6 hours,
dysreflexia. The nurse would and foley catheters should be checked frequently to
avoid which of the following prevent kinks in the tubing. Constipation and fecal
measures to minimize the impaction are other causes, so maintaining bowel
risk of recurrence? regularity is important. Other causes include
a. Strict adherence to a stimulation of the skin from tactile, thermal, or
bowel retraining program painful stimuli. The nurse administers care to
b. Keeping the linen wrinkle- minimize risk in these areas.
free under the client
c. Preventing unnecessary
pressure on the lower limbs
d. Limiting bladder
catheterization to once every
12 hours
2 18. The nurse is caring for 18. Answer B. Nursing actions during a seizure
5. the male client who begins to include providing for privacy, loosening restrictive
experience seizure activity clothing, removing the pillow and raising side rails in
while in bed. Which of the the bed, and placing the client on one side with the
following actions by the head flexed forward, if possible, to allow the tongue
nurse would be to fall forward and facilitate drainage. The limbs are
contraindicated? never restrained because the strong muscle
a. Loosening restrictive contractions could cause the client harm. If the client
clothing is not in bed when seizure activity begins, the nurse
b. Restraining the client's lowers the client to the floor, if possible, protects the
limbs head from injury, and moves furniture that may
c. Removing the pillow and injure the client. Other aspects of care are as
raising padded side rails described for the client who is in bed.
d. Positioning the client to
side, if possible, with the
head flexed forward
2 19. The nurse is assigned to care for a 19. Answer B. Hemiparesis is a weakness
6. female client with complete right- of one side of the body that may occur
sided hemiparesis. The nurse plans after a stroke. Complete hemiparesis is
care knowing that this condition: weakness of the face and tongue, arm,
a. The client has complete bilateral and leg on one side. Complete bilateral
paralysis of the arms and legs. paralysis does not occur in this condition.
b. The client has weakness on the The client with right-sided hemiparesis has
right side of the body, including the weakness of the right arm and leg and
face and tongue. needs assistance with feeding, bathing,
c. The client has lost the ability to and ambulating.
move the right arm but is able to walk
independently.
d. The client has lost the ability to
move the right arm but is able to walk
independently.
2 20. The client with a brain attack 20. Answer A. Before the client with
7. (stroke) has residual dysphagia. When dysphagia is started on a diet, the gag and
a diet order is initiated, the nurse swallow reflexes must have returned. The
avoids doing which of the following? client is assisted with meals as needed
a. Giving the client thin liquids and is given ample time to chew and
b. Thickening liquids to the swallow. Food is placed on the unaffected
consistency of oatmeal side of the mouth. Liquids are thickened to
c. Placing food on the unaffected side avoid aspiration.
of the mouth
d. Allowing plenty of time for chewing
and swallowing
2 A 20-year-old who hit his head while 1. Trauma is one of the primary causes of
8. playing football has a tonic-clonic brain damage and seizure activity in
seizure. Upon awakening from the adults. Other common causes of seizure
seizure, the client asks the nurse, activity in adults include neoplasms,
"What caused me to have a seizure? withdrawal from drugs and alcohol, and
I've never had one before." Which vascular disease. Given the history of
cause should the nurse include in the head injury, electrolyte imbalance is not
response as a primary cause of tonic- the cause of the seizure. There is no
clonic seizures in adults older than age information to indicate that the seizure is
20? 1. Head trauma. 2. Electrolyte related to a congenital defect. Epilepsy is
imbalance. 3. Congenital defect. 4. usually diagnosed in younger clients.
Epilepsy.
2 21. The nurse is assessing the 21. Answer D. Clients are evaluated as
9. adaptation of the female client to coping successfully with lifestyle changes
changes in functional status after a after a brain attack (stroke) if they make
brain attack (stroke). The nurse appropriate lifestyle alterations, use the
assesses that the client is adapting assistance of others, and have appropriate
most successfully if the client: social interactions. Options A, B, and C are
a. Gets angry with family if they not adaptive behaviors.
interrupt a task
b. Experiences bouts of depression
and irritability
c. Has difficulty with using modified
feeding utensils
d. Consistently uses adaptive
equipment in dressing self
3 A 21-year-old female client takes 1, 2, 3. The nurse should assess the
0. clonazepam (Klonopin). What should number and type of seizures the client has
the nurse ask this client about? Select experienced since starting clonazepam
all that apply. 1. Seizure activity. 2. monotherapy for seizure control. The
Pregnancy status. 3. Alcohol use. 4. nurse should also determine if the client
Cigarette smoking. 5. Intake of might be pregnant because clonazepam
caffeine and sugary drinks. crosses the placental barrier. The nurse
should also ask about the client's use of
alcohol because alcohol potentiates the
action of clonazepam. Although the nurse
may want to check on the client's diet or
use of cigarettes for health maintenance
and promotion, such information is not
specifically related to clonazepam therapy.
3 22. Nurse Kristine is trying to 22. Answer C. Clients with aphasia after
1. communicate with a client with brain brain attack (stroke) often fatigue easily
attack (stroke) and aphasia. Which of and have a short attention span. General
the following actions by the nurse guidelines when trying to communicate
would be least helpful to the client? with the aphasic client include speaking
a. Speaking to the client at a slower more slowly and allowing adequate
rate response time, listening to and watching
b. Allowing plenty of time for the attempts to communicate, and trying to
client to respond put the client at ease with a caring and
c. Completing the sentences that the understanding manner. The nurse would
client cannot finish avoid shouting (because the client is not
d. Looking directly at the client during deaf), appearing rushed for a response,
attempts at speech and letting family members provide all the
responses for the client.
3 23. A female client has 23. Answer C. Myasthenic crisis often is caused by
2. experienced an episode of undermedication and responds to the
myasthenic crisis. The nurse administration of cholinergic medications, such as
would assess whether the client neostigmine (Prostigmin) and pyridostigmine
has precipitating factors such (Mestinon). Cholinergic crisis (the opposite
as: problem) is caused by excess medication and
a. Getting too little exercise responds to withholding of medications. Too little
b. Taking excess medication exercise and fatty food intake are incorrect.
c. Omitting doses of medication Overexertion and overeating possibly could
d. Increasing intake of fatty trigger myasthenic crisis.
foods
3 24. The nurse is teaching the 24. Answer D. Clients with myasthenia gravis are
3. female client with myasthenia taught to space out activities over the day to
gravis about the prevention of conserve energy and restore muscle strength.
myasthenic and cholinergic Taking medications correctly to maintain blood
crises. The nurse tells the client levels that are not too low or too high is
that this is most effectively important. Muscle-strengthening exercises are not
done by: helpful and can fatigue the client. Overeating is a
a. Eating large, well-balanced cause of exacerbation of symptoms, as is
meals exposure to heat, crowds, erratic sleep habits,
b. Doing muscle-strengthening and emotional stress.
exercises
c. Doing all chores early in the
day while less fatigued
d. Taking medications on time to
maintain therapeutic blood
levels
3 A 24-year-old patient is observing respiratory rate and effort.
4. hospitalized with the onset of
Guillain-Barr syndrome. During rational: The most serious complication of
this phase of the patient's Guillain-Barr syndrome is respiratory failure, and
illness, the most essential the nurse should monitor respiratory function
assessment for the nurse to continuously. The other assessments also will be
carry out is ______________ included in nursing care, but they are not as
important as respiratory assessment.
3 25. A male client with Bell's 25. Answer A. Bell's palsy is a one-sided facial
5. palsy asks the nurse what has paralysis from compression of the facial nerve.
caused this problem. The The exact cause is unknown, but may include
nurse's response is based on an vascular ischemia, infection, exposure to viruses
understanding that the cause is: such as herpes zoster or herpes simplex,
a. Unknown, but possibly autoimmune disease, or a combination of these
includes ischemia, viral factors.
infection, or an autoimmune
problem
b. Unknown, but possibly
includes long-term tissue
malnutrition and cellular
hypoxia
c. Primary genetic in origin,
triggered by exposure to
meningitis
d. Primarily genetic in origin,
triggered by exposure to
neurotoxins
3 26. The nurse has given the 26. Answer A. Prevention of muscle atrophy with
6. male client with Bell's palsy Bell's palsy is accomplished with facial massage,
instructions on preserving facial exercises, and electrical stimulation of the
muscle tone in the face and nerves. Exposure to cold or drafts is avoided.
preventing denervation. The Local application of heat to the face may improve
nurse determines that the client blood flow and provide comfort.
needs additional information if
the client states that he or she
will:
a. Exposure to cold and drafts
b. Massage the face with a
gentle upward motion
c. Perform facial exercises
d. Wrinkle the forehead, blow
out the cheeks, and whistle
3 A 26-year-old patient with a T3 Multiple options are available to maintain
7. spinal cord injury asks the sexuality after spinal cord injury.
nurse about whether he will be
able to be sexually active. rational: Although sexuality will be changed by the
Which initial response by the patient's spinal cord injury, there are options for
nurse is best? expression of sexuality and for fertility. The other
information also is correct, but the choices will
depend on the degrees of injury and the patient's
individual feelings about sexuality.
3 27. Female client is admitted to 27. Answer D. Guillain-Barr syndrome is a clinical
8. the hospital with a diagnosis of syndrome of unknown origin that involves cranial
Guillain-Barre syndrome. The and peripheral nerves. Many clients report a
nurse inquires during the history of respiratory or gastrointestinal infection
nursing admission interview if in the 1 to 4 weeks before the onset of
the client has history of: neurological deficits. Occasionally, the syndrome
a. Seizures or trauma to the can be triggered by vaccination or surgery.
brain
b. Meningitis during the last 5
years
c. Back injury or trauma to the
spinal cord
d. Respiratory or
gastrointestinal infection
during the previous month.
3 28. A female client with 28. Answer C. The client with Guillain-Barr
9. Guillian-Barre syndrome has syndrome experiences fear and anxiety from the
ascending paralysis and is ascending paralysis and sudden onset of the
intubated and receiving disorder. The nurse can alleviate these fears by
mechanical ventilation. Which providing accurate information about the client's
of the following strategies condition, giving expert care and positive
would the nurse incorporate in feedback to the client, and encouraging relaxation
the plan of care to help the and distraction. The family can become involved
client cope with this illness? with selected care activities and provide diversion
a. Giving client full control over for the client as well.
care decisions and restricting
visitors
b. Providing positive feedback
and encouraging active range
of motion
c. Providing information, giving
positive feedback, and
encouraging relaxation
d. Providing intravaneously
administered sedatives,
reducing distractions and
limiting visitors
4 A 28-year-old woman who has "MS symptoms may be worse after the
0. multiple sclerosis (MS) asks the pregnancy."
nurse about risks associated
with pregnancy. Which rational: During the postpartum period, women
response by the nurse is with MS are at greater risk for exacerbation of
accurate? symptoms. There is no increased risk for
congenital defects in infants born of mothers with
MS. Symptoms of MS may improve during
pregnancy. Onset of labor is not affected by MS.
4 29. A male client has an 29. Answer D. Cranial nerve II is the optic nerve,
1. impairment of cranial nerve II. which governs vision. The nurse can provide
Specific to this impairment, the safety for the visually impaired client by clearing
nurse would plan to do which of the path of obstacles when ambulating. Testing
the following to ensure client to the shower water temperature would be useful if
ensure client safety? there were an impairment of peripheral nerves.
a. Speak loudly to the client Speaking loudly may help overcome a deficit of
b. Test the temperature of the cranial nerve VIII (vestibulocochlear). Cranial
shower water nerve VII (facial) and IX (glossopharyngeal) control
c. Check the temperature of the taste from the anterior two thirds and posterior
food on the delivery tray. third of the tongue, respectively.
d. Provide a clear path for
ambulation without obstacles
4 30. A female client has a 30. Answer B. The limbic system is responsible
2. neurological deficit involving the for feelings (affect) and emotions. Calculation
limbic system. Specific to this ability and knowledge of current events relates
type of deficit, the nurse would to function of the frontal lobe. The cerebral
document which of the following hemispheres, with specific regional functions,
information related to the control orientation. Recall of recent events is
client's behavior. controlled by the hippocampus.
a. Is disoriented to person,
place, and time
b. Affect is flat, with periods of
emotional lability
c. Cannot recall what was eaten
for breakfast today
d. Demonstrate inability to add
and subtract; does not know who
is president
4 A 32-year-old patient has a Applying intermittent pneumatic compression
3. stroke resulting from a ruptured stockings
aneurysm and subarachnoid
hemorrhage. Which intervention rational: The patient with a subarachnoid
will be included in the care plan? hemorrhage usually has minimal activity to
prevent cerebral vasospasm or further bleeding
and is at risk for venous thromboemboism (VTE).
Activities such as coughing and sitting up that
might increase intracranial pressure (ICP) or
decrease cerebral blood flow are avoided.
Because there is no indication that the patient is
unconscious, an oropharyngeal airway is
inappropriate.
4 A 42-year-old patient who was availability of genetic testing to determine the
4. adopted at birth is diagnosed HD risk for the patient's children.
with early Huntington's disease
(HD). When teaching the patient, rational: Genetic testing is available to determine
spouse, and children about this whether an asymptomatic individual has the HD
disorder, the nurse will provide gene. The patient and family should be informed
information about the of the benefits and problems associated with
______________ genetic testing. Sinemet will increase symptoms
of HD given that HD involves an increase in
dopamine. Antibiotic therapy will not reduce the
risk for aspiration. There are no effective
treatments or lifestyle changes that delay the
progression of symptoms in HD.
4 A 58-year-old client complaining Vitamin A
5. of difficulty driving at night
states that the "lights bother my
eyes." The client wears
corrective glasses. The nurse
would suspect that the client is
experiencing a deficiency in
which of the following vitamins?
4 A 58-year-old patient who began 1) Administer oxygen to keep O2 saturation
6. experiencing right-sided arm and >95%
leg weakness is admitted to the 2) Use National Institute of Health Stroke Scale to
emergency department. In which assess patient
order will the nurse implement 3) Obtain CT scan without contrast.
these actions included in the 4) Infuse tissue plasminogen activator (tPA).
stroke protocol?
rational: The initial actions should be those that
help with airway, breathing, and circulation.
Baseline neurologic assessments should be done
next. A CT scan will be needed to rule out
hemorrhagic stroke before tPA can be
administered.
4 A 62-year-old patient is brought "What did you have for breakfast?"
7. to the clinic by a family member
who is concerned about the rational: This question tests the patient's recent
patient's inability to solve memory, which is decreased early in Alzheimer's
common problems. To obtain disease (AD) or dementia. Asking the patient
information about the patient's about birthplace tests for remote memory, which
current mental status, which is intact in the early stages. Questions about the
question should the nurse ask patient's emotions and self-image are helpful in
the patient? assessing emotional status, but they are not as
helpful in assessing mental state.
4 65. Which goal is the most realistic 4. Helping the client function at his or her
8. and appropriate for a client diagnosed best is most appropriate and realistic.
with Parkinson's disease? 1. To cure There is no known cure for Parkinson's
the disease. 2. To stop progression of disease. Parkinson's disease progresses in
the disease. 3. To begin preparations severity, and there is no known way to
for terminal care. 4. To maintain stop its progression. Many clients live for
optimal body function. years with the disease, however, and it
would not be appropriate to start planning
terminal care at this time.
4 A 69-year-old client is admitted and 2.
9. diagnosed with delirium. Later in the Loss of orientation, especially for time and
day, he tries to get out of the locked place, is common in delirium. The nurse
unit. He yells, "Unlock this door. I've should orient the client by telling him the
got to go see my doctor. I just can't time, date, place, and who the client is
miss my monthly Friday appointment." with. Taking the client to his room and
Which of the following responses by telling him why the door is locked does not
the nurse is most appropriate? address his disorientation. Telling the
1. "Please come away from the door. client to eat before going to the doctor
I'll show you your room." reinforces his disorientation.
2. "It's Tuesday and you are in the
hospital. I'm Anne, a nurse."
3. "The door is locked to keep you from
getting lost."
4. "I want you to come eat your lunch
before you go the doctor."
5 A 70-year-old client with a diagnosis of Promoting weight-bearing exercises
0. left-sided stroke is admitted to the
facility. To prevent the development of
disuse osteoporosis, which of the
following objectives is appropriate?
5 A 72-year-old patient is diagnosed loss of both recent and long-term memory.
1. with moderate dementia as a result of
multiple strokes. During assessment of rational: Loss of both recent and long-term
the patient, the nurse would expect to memory is characteristic of moderate
find _____________ dementia. Patients with dementia have
frequent nighttime awakening. Dementia
is progressive, and the patient's ability to
perform tasks would not have periods of
improvement. Difficulty eating and
swallowing is characteristic of severe
dementia.
5 A 75-year-old client who was admitted Make arrangements for the client to
2. to the hospital with a stroke informs receive information about advance
the nurse that he doesn't want to be directives.
kept alive with machines. He wants to
make sure that everyone knows his
wishes. Which action should the nurse
take?
5 A 78-year-old Alzheimer's client is wander.
3. being treated for malnutrition and
dehydration. The nurse decides to
place him closer to the nurses' station
because of his tendency to:
5 An 83-year-old woman is admitted to 1, 2, 3.
4. the unit after being examined in the The client does need rest and it is true
emergency department (ED) and that there is no specific medicine for
diagnosed with delirium. After the delirium, but it is crucial to identify and
admission interviews with the client treat the underlying causes of delirium.
and her grandson, the nurse explains Other tests will be based on the results of
that there will be more laboratory already completed tests. Although some
tests and X-rays done that day. The medications may be prescribed to help the
grandson says, "She has already been client with her behaviors, this is not the
stuck several times and had a brain primary basis for medication orders.
scan or something. Just give her some Because the underlying medical causes of
medicine and let her rest." The nurse delirium could be fatal, treatment must be
should tell the grandson which of the initiated as soon as possible. It is not the
following? Select all that apply. nurse's role to determine medications for
1. "I agree she needs to rest, but there this client. Postponing tests until the next
is no one specific medicine for your day is inappropriate.
grandmother's condition."
2. "The doctor will look at the results
of those tests in the ED and decide
what other tests are needed."
3. "Delirium commonly results from
underlying medical causes that we
need to identify and correct."
4. "Tell me about your grandmother's
behaviors and maybe I could figure out
what medicine she needs."
5. "I'll ask the doctor to postpone more
tests until tomorrow."
5 An 85-year-old client has impaired 1. Obtaining an amplified telephone; The
5. hearing. When creating the care plan, amplified telephone helps with hearing
which intervention should have the and provides a means for
highest priority? communicating with others.

1. Obtaining an amplified telephone. Option 2 refers to a tactile impairment.


2. Teaching the importance of changing Option 3 relates to a visual impairment,
his position. and option 4 an olfactory impairment.
3. Providing reading material with large
print.
4. Checking expiration dates on food
packages
5 A 90-year-old client diagnosed with 4.
6. major depression is suddenly The client is showing symptoms of
experiencing sleep disturbances, delirium, a common outcome of UTI in
inability to focus, poor recent memory, older adults. The nurse can request a
altered perceptions, and disorientation transfer to a medical unit for acute
to time and place. Lab results indicate medical intervention. The client's
the client has a urinary tract infection symptoms are not just due to a
and dehydration. After explaining the worsening of the depression. There are
situation and giving the background and not indications that the client needs
assessment data, the nurse should make restraints or a transfer to a nursing
which of the following recommendations home at this point.
to the client's physician?
1. An order to place the client in
restraints.
2. A reevaluation of the client's mental
status.
3. The transfer of the client to a medical
unit.
4. A transfer of the client to a nursing
home.
5 a A major goal of treament for the patient
7. with AD is to:
a. maintain patient safety
b. maintain or increase body weight
c. return to a higher level of self care
d. enhance functional ability over time
5 a,b,c,d,e Social effects of a chronic neurologic
8. disease include (select all that apply)
a. divorce
b. job loss
c. depression
d. role changes.
e. loss of self esteem
5 Absence seizures Sudden interruption of activity
9. associated with unresponsiveness and
blank stare, sometimes with eyelid
fluttering
-Often precipitated by hyperventilation
-Tx: ethosuximide (initial), valproic acid
(long-term)
6 Advantages very broad spectrum (good if you don't
0. know what spec type of seizure)
good for migraines
6 Advantages Advantages:
1. easy to load (PO or IV)
once a day dosing
low cost
in use since 1938 (no surprises)
6 Advantages and Disadvantages Adv: sustained release forms available
2.
Disadv: drug interactions (note:
erythromycin reduces carbamazepine
clearance - potential toxicity)
6 Advantages and DIsadvantages of broad spectrum
3. Topirimate doesn't alter other AEDs
weight loss

Disadv - slow to load; unresponsive


6 Advantages and Disadv of broad spectrum
4. Levetiracitam no drug-drug interactions
relatively rapid titration

Disadv - expensive
6 Advantages of Gabapentoin no drug interactions
5. non-hepatic metabolism
easy to load orally
very safe, well tolerated
6 Advantages of Lamotragine broad spectrum
6. seems to be very potent
less sedating than other AEDs
6 Advantages of pregabalin no drug interactions
7. non-hepatic metabolism
easy to load orally
very safe, well tolerated; effective at lower
doses than gabapentin
linear kinetics
6 Adverse Effects weight gain
8. tremor
alopecia
thrombocytopenia
pancreatitis
hyperammonemia
6 Adverse Effects behavioral problems
9. rare psychosis
7 Adverse effects of Topirimate metabolic acidosis (carbonic anhydrase
0. inhibitor)
kidney stones
cognitive slowing, word-finding difficulty
glaucoma (rare)
7 After a 25-year-old patient has develop a plan to increase the patient's
1. returned home following independence in consultation with the patient
rehabilitation for a spinal cord and the spouse.
injury, the home care nurse notes
that the spouse is performing many rational: The best action by the nurse will be
of the activities that the patient to involve all the parties in developing an
had been managing during optimal plan of care. Because family members
rehabilitation. The most who will be assisting with the patient's
appropriate action by the nurse at ongoing care need to feel that their input is
this time is to _____________ important, telling the spouse that the patient
can perform activities independently is not the
best choice. Reminding the patient about the
importance of independence may not change
the behaviors of the spouse. Supporting the
activities of the spouse will lead to ongoing
dependency by the patient.
7 After a motor vehicle accident, a Flat, except for logrolling as needed
2. client is admitted to the medical-
surgical unit with a cervical collar
in place. The cervical spinal X-rays
haven't been read, so the nurse
doesn't know whether the client
has a cervical spinal injury. Until
such an injury is ruled out, the
nurse should restrict this client to
which position?
7 After an eye examination, a client instilling one drop of pilocarpine 0.25% into
3. is diagnosed with open-angle both eyes four times daily.
glaucoma. The physician's
prescription says "pilocarpine
ophthalmic solution (Pilocar),
0.25% 1 gtt both eyes q.i.d." Based
on this prescription, the nurse
should teach the client or a family
member to administer the drug by:
7 After a plane crash, a client is brought to evaluation of the corneal reflex
4. the emergency department with severe response.
burns and respiratory difficulty. The
nurse helps to secure a patent airway,
attends to the client's immediate needs,
and then prepares to perform a
neurologic assessment. Because the
client is unstable and in critical condition,
this examination must be brief but should
include:
7 After a stroke, a 75-year-old client is Elevating the head of the bed to 30
5. admitted to a health care facility. The degrees
client has left-sided weakness and an
absent gag reflex. He's incontinent and
has a tarry stool. His blood pressure is
90/50 mm Hg, and his hemoglobin is 10 g.
Which action is a priority for this client?
7 after change of shift report which pt 28 yr old w fx complaining cast is tight
6. needs to be assessed first
7 After having a craniectomy and left perform range-of-motion (ROM)
7. anterior fossae incision, a patient has a exercises every 4 hours.
nursing diagnosis of impaired physical
mobility related to decreased level of rational: ROM exercises will help to
consciousness and weakness. An prevent the complications of
appropriate nursing intervention is to immobility. Patients with anterior
______________ craniotomies are positioned with the
head elevated. The patient with a
craniectomy should not be turned to
the operative side. When the patient is
weak, clustering nursing activities may
lead to more fatigue and weakness.
7 After noting that a patient with a head Check the nasal drainage for glucose.
8. injury has clear nasal drainage, which
action should the nurse take? rational: Clear nasal drainage in a
patient with a head injury suggests a
dural tear and cerebrospinal fluid (CSF)
leakage. If the drainage is CSF, it will
test positive for glucose. Fluid leaking
from the nose will have normal nasal
flora, so culture and sensitivity will not
be useful. Blowing the nose is avoided
to prevent CSF leakage.
7 After receiving change-of-shift report on A patient with right-sided weakness
9. the following four patients, which patient who has an infusion of tPA prescribed
should the nurse see first?
rational: tPA needs to be infused within
the first few hours after stroke
symptoms start in order to be effective
in minimizing brain injury. The other
medications also should be given as
quickly as possible, but timing of the
medications is not as critical.
8 After receiving two of nine ANS: C
0. electroconvulsive therapy (ECT) The client has the right to terminate
treatments, a client states, "I can't even treatment. This nursing reply
remember eating breakfast, so I want to acknowledges this right but focuses on
stop the ECT treatments." Which is the the client's concerns so that the nurse
most appropriate nursing reply? can provide needed information.

A. "After you begin the course of


treatments, you must complete all of
them."
B. "You'll need to talk with your doctor
about what you're thinking."
C. "It is within your right to discontinue
the treatments, but let's talk about your
concerns."
D. "Memory loss is a rare side effect of
the treatment. I don't think it should be a
concern."
8 After striking his head on a tree while Perform a lumbar puncture.
1. falling from a ladder, a young man is
admitted to the emergency department.
He's unconscious and his pupils are
nonreactive. Which intervention would be
the most dangerous for the client?
8 After suctioning, the nurse Assure that the patient's neck is not in a flexed
2. notes that the intracranial position.
pressure for a patient with a
traumatic head injury has rational: Since suctioning will cause a transient
increased from 14 to 16 mm increase in intracranial pressure, the nurse should
Hg. Which action should the initially check for other factors that might be
nurse take first? contributing to the increase and observe the patient
for a few minutes. Documentation is needed, but this
is not the first action. There is no need to notify the
health care provider about this expected reaction to
suctioning. Propofol is used to control patient anxiety
or agitation; there is no indication that anxiety has
contributed to the increase in intracranial pressure.
8 After teaching a patient "I will try to lie down someplace dark and quiet when
3. about management of the headaches begin."
migraine headaches, the
nurse determines that the rational: It is recommended that the patient with a
teaching has been effective migraine rest in a dark, quiet area. Topiramate
when the patient says, (Topamax) is used to prevent migraines and must be
_____________ taken for several months to determine effectiveness.
Aspirin or other nonsteroidal anti-inflammatory
medications can be taken with the triptans. Alcohol
may precipitate migraine headaches.
8 After the emergency A patient whose right pupil is 10 mm and
4. department nurse has unresponsive to light
received a status report on
the following patients who rational: The dilated and nonresponsive pupil may
have been admitted with indicate an intracerebral hemorrhage and increased
head injuries, which patient intracranial pressure. The other patients are not at
should the nurse assess immediate risk for complications such as herniation.
first?
8 Agnosia any of many types of loss of neurological function
5. associated with interpretation of sensory information
8 a. idiopathic generalized Childhood Absence Epilepsy
6. epilepsy (2 exs) Juvenile Absence Epilepsy
8 Akenesia Difficulty initiating movement
7.

8 An alert 80-year-old client is 3. Disturbed sensory perception; The transfer to a


8. transferred to a long-term different setting can change the amount or pattern of
care facility. On the second incoming stimuli, and the client may have a
night he becomes confused diminished, exaggerated, distorted, or impaired
and agitated. What is the response to such stimuli.
most appropriate nursing
diagnosis? Disturbed Thought Processes is applied when
cognitive abilities (e.g., dementia) interfere with the
1. Chronic Confusion ability to interpret stimuli accurately. Options 1 & 2
2. Impaired Memory offer no evidence to support chronic confusion or
3. Disturbed Sensory impaired memory.
Perception
4. Disturbed Thought
Processes
8 ALS progressive degeneration of motor system
9.
muscle weakness leading to flaccid quadraplegia and
death
9 Amputation DESCRIPTION: Surgical removal of a diseased part or
0. organ.
Causes for amputation include the following:
Peripheral vascular disease, 80% (75% ar diabetics).
Trauma.
Congenital deformities.
Malignant tumors
Infection.
Amputation necessitates major lifestyle and body-image
adjustments.
NURSING ASSESSMENT: Prior to amputation, symptoms of
peripheral vascular disease include:
Cool extremity.
Absent peripheral pulses.
Hair loss on affected extremity.
Necrotic tissue or wounds: blue or blue-gray, turning black.
Drainage possible with or without odor.
Leathery skin on affected extremity.
Decrease of pain sensation in affected extremity.
Inadequate circulation is determined by: Arteriogram and
Doppler flow studies.
NURSING PLANS AND INTERVENTIONS:Provide wound care:
Mark dressing for bleeding, and check marking at least
every 8 hours.
Measure suction drainage every shift.
Change dressing as needed (physician usually performs
initial dressing change): ***large tourniqiet at bedside
for frank hemorrhage***
Maintain aseptic technique.
Observe wound color and warmth.
Observe for wound healing.
Monitor for signs of infection: fever, tachycardia, redness of
incision area.
Maintain proper body alignment in and out of bed.
Position client to relieve edema and spasms at residual limb
(stump) site.
**Elevate stump for the first 24 hours postop**
Do not continually elevate stump after 48 hrs postop. (can
cause contracture).
Keep stump in extended position, and turn client to prone
position three times a day to prevent hip flexion
contracture.
Be aware that phantom pain is real; it will eventually
diappear, and it responds to pain meds.
Handle affected ody part gently and with smooth
movements.
Provide passive ROM until client is able to perform active
ROM. Collaborate with rehab team members for mobility
improvement.
Encourage independence in self-care, allowing sufficient
time for client to complete care and to have input into care.
9 Amputation Hesi Hint The residual limb or stump should be elevated on one
1. #1 pillow. If the residual limb is elevated too high, the elevation
can cause a contracture.
9 Anticholenergics- Medication that helps control tremors and rigidity. Monitor
2. benzotropine for signs of dry mouth, constipation, and urinary retention
(cogentin) & artane
9 ANTI-EPILEPTIC DRUGS ...
3.

9 Antivirals Medications that stimulate release of dopamine and


4. prevent re-uptake. Monitor for signs of swollen ankles and
discoloration of the skin.
9 Apraxia impaired ability to carry out motor activities despite intact
5. motor function
9 Aricept (donepezil) is a cholinesterase inhibitor block cholinesterase the
6. enzyme responsible for breakdown of ach in the synaptic
cleft.
97. Aspirin is ordered for a patient who is The patient states, "My symptoms
admitted with a possible stroke. Which started with a terrible headache."
information obtained during the
admission assessment indicates that the rational: A sudden onset headache is
nurse should consult with the health typical of a subarachnoid hemorrhage,
care provider before giving the aspirin? and aspirin is contraindicated. Atrial
fibrillation, dysphasia, and transient
ischemic attack (TIA) are not
contraindications to aspirin use, so the
nurse can administer the aspirin.
98. Astrocytoma The most common type of Glioma.
Earliest S&S: headache. Also, seizures,
memory loss, weakness, visual
symptoms, personality changes.
99. Atonic seizure Sudden decrease in muscle tone -> loss
of postural control -> patient may fall
("drop attack")
10 At what time of day should the nurse .2. Demanding physical activity should
0. encourage a client with Parkinson's be performed during the peak action of
disease to schedule the most demanding drug therapy. Clients should be
physical activities to minimize the encouraged to maintain independence
effects of hypokinesia? 1. Early in the in self-care activities to the greatest
morning, when the client's energy level extent possible. Although some clients
is high. 2. To coincide with the peak may have more energy in the morning
action of drug therapy. 3. Immediately or after rest, tremors are managed with
after a rest period. 4. When family drug therapy.
members will be available.
10 Audiometry confirms a client's chronic conductive hearing loss.
1. progressive hearing loss. Further
investigation reveals ankylosis of the
stapes in the oval window, a condition
that prevents sound transmission. This
type of hearing loss is called a:
10 An auto mechanic accidentally has decreasing leukocyte infiltration at the
2. battery acid splashed in his eyes. His site of ocular inflammation.
coworkers irrigate his eyes with water
for 20 minutes, and then take him to the
emergency department of a nearby
health care facility. He receives
emergency care for corneal injury. The
physician prescribes dexamethasone
(Maxidex Ophthalmic Suspension), two
drops of 0.1% solution to be instilled
initially into the conjunctival sacs of
both eyes every hour; and polymyxin B
sulfate (Neosporin Ophthalmic), 0.5%
ointment to be placed in the conjunctival
sacs of both eyes every 3 hours.
Dexamethasone exerts its therapeutic
effect by:
10 Autonomic Nervous System Fight or flight response
3.

10 B The early stage of AD is characterized


4. by
a. no noticeable change in behavior
b. memory problems and mild
confusion
c. increased time spent sleeping on or
in bed.
d. Incontinence, agitation, and
wandering behavior
10 B Which of the following statements accurately
5. describes mild cognitive impairment. (select all
that apply)
a. always progresses to AD
b. Caused by variety of factors and may
progress to AD
c. Should be aggressively treated with
acetylcholinesterase drugs
d. Caused by vascular infarcts that if treated will
delay progression to AD
c. Patient is usually not aware that there is a
problem with his or her memory.
10 babinksi reflex abnormal: dorsiflexion of great toe and fanning
6.

10 Before inserting an otoscope in C. pull the pinna up and back to straighten the
7. an adult client, which of the auditory canal
following maneuvers should the
examiner perform

A. irrigate the ear canal to make


sure the tympanic membrane is
visible

B. have the client blow his or her


nose to make sure pressure is
equalized between the external
ear and the middle ear.

C. pull the pinna up and back to


straighten the auditory canal

D. tilt the client's head toward


the side being examined
10 bells palsy unilateral facial palsy CN VII
8. exercise face, lubicate eyes, chew on
unaffected side
10 Benign childhood epilepsy Benign "rolandic" epilepsy
9. w/centrotemporal spikes -Simple partial seizures with abnormal
movements and sensation of the mouth,
sometimes the hand, with excessive salivation
and drooling
-May have secondary generalization
-Typically present at age 5-10 yrs, almost
always resolve by age 18
11 Benign Rolandic Epilepsy - 5 to 10 y/o
0. - hemifacial twitching and drooling esp at night
- rx = AED (if needed) and resolves by teen
years
11 Bradykenesia Slowness of movement
1.

11 Brain Stem Major sensory and motor pathway for impulses


2. running to and from the cerebrum. Regulates
body functions (resp, auditory, visual, gag,
swallowing, coughing)
11 Brain Stem includes Midbrain, pons, and medulla
3.

11 bromocriptine (Parlodel) & Medication that activate release of dopamine.


4. pramipezole (Mirapex) Work in conjunction with dopaminergics for
better results. Monitor orthostatic hypertension,
dyskinesia, and hallucinations.
11 C Vascular dementia is associated with
5. a. transient ischemic attacks
b. bacterial or viral infection of neuronal tissue
c. cognitive changes secondary to cerebral
eschemia
d. abrupt changes in cognitive function that are
irreversibe
11 C A 65 Year old woman was just diagnosed with
6. parkinson's disease the priority nursing
intervention is:
a. searching the internet for educational videos
b. evaluating the home for environmental safety
c. promoting physical exercise and a well
balanced diet
d. Designing an exercise program to strengthen
and stretch specific muscles.
11 c A nurse and a primary care provider inform a client
7. that chemotherapy is recommended for a diagnosis
of cancer. Which nursing action is most
representative of the concept of holism?
a. Offer to come to the client's home to provide
needed physical care
b. Contact the client's spiritual advisor
c. Inquire how this will affect other aspects of the
client's life
d. Provide the client with information about how to
join a support group
1. ch 16
11 Carbamazepine (Tegretol) Approved for:
8. -Focal, generalized tonic-clonic seizures
Side effects:
-Ataxia, nystagmus, diplopia, hepatotoxicity, rash
-Agranulocytosis/aplastic anemia (BM
suppression), teratogenic, CYP inducer,
SIADH -> hyponatremia
11 Carbamezapine ...
9.

12 Carbidopa Medication that mimics dopamine action, increases


0. efficacy of dopamine
12 The care plan for a patient Check capillary blood glucose level every 6 hours.
1. who has increased intracranial
pressure and a rational: Experienced NAP can obtain capillary
ventriculostomy includes the blood glucose levels when they have been trained
following nursing actions. and evaluated in the skill. Monitoring and
Which action can the nurse documentation of cerebrospinal fluid (CSF) color
delegate to nursing assistive and intracranial pressure (ICP) require RN-level
personnel (NAP) who regularly education and scope of practice. Although
work in the intensive care repositioning patients is frequently delegated to
unit? NAP, repositioning a patient with a ventriculostomy
is complex and should be done by the RN.
12 caring for a client w an infection
2. external fixator on the lower
leg for a fractured tibia,
complication
12 Carpal tunnel ...
3.

12 Cataract Hesi Hint #1 The lens of the eye is responsible for projecting
4. light onto the retina so that images can be
discerned. Without the lens, which becomes
opaque with cataracts, light cannot be filtered and
vision is blurred.
12 Cataract Hesi Hint #2 When the cataract is removed, the lens is gone,
5. making prevention of falls important. When the
lens is replaced with an implant, vision is better.
12 CATARACTS DESCRIPTION: Condition characterized by opacity of
6. the lens. Aging accounts for 95% of cataracts. The
remaining 5% result from trauma, toxic substances,
or systemic diseases or are congenital. Safety
precatuions may reduce the incidence of traumatic
cataracts. Surgical removal is done when vision
impairment interferes with ADL's. Intraocular lens
implants may be used. Most operations are
performed under local anesthesia on an outpatient
basis.
NURSING ASSESSMENT:Early signs: Blurred vision and
decreased color perception. Late signs include:
Diplopia (double vision). Reduced visual acuity,
progressing to blindness. Clouded pupil, progressing
to a milky-white appearance.
DX Tests: Ophthalmoscope. Slit-lamp biomicroscope.
NURSING PLANS AND INTERVENTIONS:
Preoperative: Demonstrate and request a return
demonstration of eye medication instillation from
client or family member.
Develop a postop teaching plan that includes:
Warning not to rub or put pressure on eye.
Teach that glasses or shaded lens should be worn
during waking hours. An eye shield should be worn
during sleeping hours.
Teach to avoid lifting objects over 15lbs, bending,
straining, coughing, or any other activity that can
increase IOP.
Teach to use a stool softener to prevent straining at
stool.
Teach to avoid lying on operative side.
Teach the need to keep water from getting into eye
while showering or washing hair.
Teach to observe and report signs of increased IOP
and infection (eg. pain, changes in vital signs).
Tylenol should control postop pain.
**S/S***
12 Cerebellum Major motor and sensory pathway. Controls smooth,
7. coordinated muscle mvmts and helps maintain
equilibrium
12 cerebellum smooth muscle, muscle tone, posture, equilibrium
8.

12 Cerebral Angiogram -assesses blood flow; involves catheter insertion, dye


9. injection and sequential x-ray
-know if pt has allergy to shellfish, iodine or has blood
clotting disorder; these allergies require different
contrast media
-observe for catheter site bleeding 8-12hrs
postprocedure
13 Cerebral Arteriography is a form of medical imaging that visualizes the
0. arterial and venous supply of the brain. It was
pioneered by Dr Egas Moniz in 1927, and is now the
gold standard for detecting vascular problems of the
brain.
13 Cerebrum; Cerebral Lobes complex problem solving, value judgements,
1. control language, emotions, visual image interpretation,
touch, pressure, and temp
13 The charge nurse observes The staff nurse suctions the patient every 2 hours.
2. an inexperienced staff nurse
who is caring for a patient rational: Suctioning increases intracranial pressure
who has had a craniotomy and is done only when the patient's respiratory
for a brain tumor. Which condition indicates it is needed. The other actions by
action by the inexperienced the staff nurse are appropriate.
nurse requires the charge
nurse to intervene?
13 charge nurse you are 72 yr old with pagets disease returned from surgery
3. making assignments, pulled tk replacement
from pacu for the day
13 cheyne stokes rhythmic with apnea
4.
metabolic cause or damage to basal ganglia
13 Childhood absence epilepsy Absence seizures that begin in children ages 4-8 y.o.
5. and usually resolve before adulthood
13 Childhood Absence Epilepsy - 4 to 8 y/o
6. - age range - staring off for a few sec then
- clinical manifestation (and EEG) back to "normal"; can elicit this via
- rx and prognosis hyperventilation during P/E
- AEDs - best is ethosusimide then
valproic acid
- 2/3 recover from this and stop rx
13 A chronically depressed and suicidal client is ANS: A
7. admitted to a psychiatric unit. The client is ECT is an intervention for major
scheduled for electroconvulsive therapy depression that often includes
(ECT). During the course of ECT treatments, a suicidal ideations as a symptom.
nurse should recognize the continued need Continued suicide assessment is
for which critical intervention? needed because mood
improvement due to ECT may
A. Suicide assessment must continue cause the client to act on suicidal
throughout the ECT course of treatment. ideations.
B. Antidepressant medications are
contraindicated throughout the ECT course of
treatment.
C. Discourage expressions of hopelessness
throughout the ECT course of treatment.
D. Encourage high-caloric diet throughout
the ECT course of treatment.
13 Classification of Epilepsies (name them) idiopathic generalized epilepsy
8. idiopathic localized epilepsy
symptomatic generalized epilepsy
(does it exist?)
symptomatic localized epilepsy
13 Classifications of Epilepsies 1. Idiopathic vs. symptomatic (of
9. known or suspected CNS
pathology)
2. Generalized vs. localization-
related (focal) - re. most of the
seizures the person experiences
Result: 4 categories
14 Classifications of focal seizures 1. Local - stay in that same focus
0. 2. Regional - spreads to other
areas of that region (eg from legs
to arms in that side of the body)
3. Bilateral (more diffuse
spreading) - spreads to both sides
of body
14 A client accidentally splashes chemicals into To prevent vision loss
1. his eye. The nurse knows that eye irrigation
with plain tap water should begin
immediately and continue for 15 to 20
minutes. What is the primary purpose of this
first-aid treatment?
14 A client admitted to an acute care facility Lidocaine (Xylocaine)
2. after a car accident develops signs and
symptoms of increased intracranial pressure
(ICP). The client is intubated and placed on
mechanical ventilation to help reduce ICP. To
prevent a further rise in ICP caused by
suctioning, the nurse anticipates
administering which drug endotracheally
before suctioning?
14 A client admitted with a cerebral contusion is Risk for injury related to
3. confused, disoriented, and restless. Which neurologic deficit
nursing diagnosis takes highest priority?
14 A client, age 21, is admitted with bacterial An isolation room close to the
4. meningitis. Which hospital room would be nurses' station
the appropriate choice for this client?
14 A client arrives at the emergency department Edrophonium (Tensilon)
5. complaining of extreme muscle weakness
after minimal effort. The physician suspects
myasthenia gravis. Which drug will be used
to test for this disease?
14 A client arrives in the emergency 3. Studies show that clients who receive
6. department with an ischemic stroke recombinant t-PA treatment within 3
and receives tissue plasminogen hours after the onset of a stroke have
activator (t-PA) administration. The better outcomes. The time from the onset
nurse should first: 1. Ask what of a stroke to t-PA treatment is critical. A
medications the client is taking. 2. complete health assessment and history
Complete a history and health is not possible when a client is receiving
assessment. 3. Identify the time of emergency care. Upcoming surgical
onset of the stroke. 4. Determine if procedures may need to be delayed
the client is scheduled for any surgical because of the administration of t-PA,
procedures. which is a priority in the immediate
treatment of the current stroke. While the
nurse should identify which medications
the client is taking, it is more important to
know the time of the onset of the stroke
to determine the course of action for
administering t-PA.
14 The client asks the nurse, "How does constricts cerebral blood vessels.
7. ergotamine (Ergostat) relieve migraine
headaches?" The nurse should respond
that it:
14 A client comes to the clinic for an The tonometer will register the force
8. ophthalmologic screening, which will required to indent or flatten the corneal
include measurement of intraocular apex.
pressure with a tonometer. When
teaching the client about the test, the
nurse should cover which point?
14 A client comes to the emergency Meningeal irritation
9. department complaining of headache,
malaise, chills, fever, and a stiff neck.
Vital sign assessment reveals a
temperature elevation, increased
heart and respiratory rates, and
normal blood pressure. On physical
examination, the nurse notes
confusion, a petechial rash, nuchal
rigidity, Brudzinski's sign, and Kernig's
sign. What does Brudzinski's sign
indicate?
15 A client complains of periorbital Cholinergic blocker
0. aching, tearing, blurred vision, and
photophobia in her right eye.
Ophthalmologic examination reveals a
small, irregular, nonreactive pupil a
condition resulting from acute iris
inflammation (iritis). As part of the
client's therapeutic regimen, the
physician prescribes atropine sulfate
(Atropisol), two drops of 0.5% solution
in the right eye twice daily. Atropine
sulfate belongs to which drug
classification?
15 A client complains of vertigo. The Inner ear
1. nurse anticipates that the client may
have a problem with which portion of
the ear?
15 A client diagnosed with a brain tumor Assist the client to a side-lying position on
2. experiences a generalized seizure the floor, and protect her with linens.
while sitting in a chair. How should the
nurse intervene first?
15 A client diagnosed with dementia 2.
3. wanders the halls of the locked Elderly clients have increased risk for falls
nursing unit during the day. To ensure due to balance problems, medication use,
the client's safety while walking in the and decreased eyesight. Haldol may
halls, the nurse should do which of the cause extrapyramidal side effects (EPSE)
following? which increase the risk for falls. The client
1. Administer PRN haloperidol (Haldol) is not agitated, so restraints are not
to decrease the need to walk. indicated. Ativan may increase fall risk
2. Assess the client's gait for and cause paradoxical excitement.
steadiness.
3. Restrain the client in a geriatric
chair.
4. Administer PRN lorazepam (Ativan)
to provide sedation.
15 A client experienced bradycardia during ANS: C
4. electroconvulsive therapy (ECT) Vagal stimulation induced by ECT may
treatment. A nurse assigns a nursing cause a client to experience
diagnosis of decreased cardiac output bradycardia. Adequate tissue perfusion
R/T vagal stimulation occurring during would be a realistic expectation when
ECT. Which outcome would the nurse normal cardiac output is restored.
expect the client to achieve?

A. The client will verbalize an


understanding of the need for moving
slowly after treatment.
B. The client will maintain an oxygen
saturation level of 88% 1 hour after
treatment.
C. The client will continue adequate
tissue perfusion 1 hour after treatment.
D. The client will verbalize an
understanding of common side effects
of ECT.
15 A client has a history of painful, treatment of spasticity associated with
5. continuous muscle spasms. He has spinal cord lesions.
taken several skeletal muscle relaxants
without experiencing relief. His
physician prescribes diazepam (Valium),
2 mg by mouth twice daily. In addition
to being used to relieve painful muscle
spasms, diazepam also is recommended
for:
15 A client has an exacerbation of multiple 1 to 2 weeks
6. sclerosis accompanied by leg spasticity.
The physician prescribes dantrolene
sodium (Dantrium), 25 mg by mouth
daily. How soon after administration
can the nurse expect to see a
significant reduction in spasticity?
15 A client has been in the critical care 1, 2, 4.
7. unit for 3 days following a severe The abnormal stimuli of the critical care
myocardial infarction. Although he is unit can aggravate the symptoms of
medically stable, he has begun to have delirium. Arguing with hallucinations is
fluctuating episodes of consciousness, inappropriate. When a client has illogical
illogical thinking, and anxiety. He is thinking, gently presenting reality is
picking at the air to "catch these baby appropriate. Dementia is not the likely
angels flying around my head." While cause of the client's symptoms. The
waiting for medical and psychiatric client is experiencing delirium, not
consults, the nurse must intervene with dementia.
the client's needs. Which of the
following needs have the highest
priority? Select all that apply.
1. Decreasing as much "foreign" stimuli
as possible.
2. Avoiding challenging the client's
perceptions about "baby angels."
3. Orienting the client about his medical
condition.
4. Gently presenting reality as needed.
5. Calling the client's family to report
his onset of dementia.
15 A client has had multiple sclerosis (MS) 2
8. for 15 years and has received various
drug therapies. What is the primary
reason why the nurse has found it
difficult to evaluate the effectiveness of
the drugs that the client has used? 1.
The client exhibits intolerance to many
drugs. 2. The client experiences
spontaneous remissions from time to
time. 3. The client requires multiple
drugs simultaneously. 4. The client
endures long periods of exacerbation
before the illness responds to a
particular drug.
15 A client has just been diagnosed with demonstrate eyedrop instillation.
9. early glaucoma. During a teaching
session, the nurse should:
16 The client has some equipment that is 2. Explain the sounds in the
0. noisy, and the roommate also has environment; Moving the client is not
equipment that makes noise, and the wise, as he and his roommate obviously
room is close to a noisy nursing station, need to be watched closely. Telling them
where they can be watched a little to ignore the noise or playing music to
closer. Which of the following cover it is not as helpful as explaining
interventions by the nurse would be the sounds in the environment. When
best for the client as well as reduce the clients understand the meaning of the
risk of sensory overload? sounds, the stimuli are frequently less
confusing and more easily ignored.

1. Move the client away from the


nurses' station area.
2. Explain the sounds in the
environment.
3. Tell the client to ignore the sounds.
4. Play the client's favorite music louder
than the sounds.
16 A client in a nursing home is II
1. diagnosed with Alzheimer's
disease. He exhibits the following
symptoms: difficulty with recent
and remote memory, irritability,
depression, restlessness, difficulty
swallowing, and occasional
incontinence. This client is in what
stage of Alzheimer's disease?
16 A client injured in a train hypoxia.
2. derailment is admitted to an acute
care facility with a suspected
dysfunction of the lower brain
stem. The nurse should monitor
this client closely for:
16 A client injures his spinal cord in a C4
3. diving accident. The nurse knows
that the client will be unable to
breathe spontaneously if the injury
site is above which vertebral level?
16 The client in the early stage of 1.
4. Alzheimer's disease and his adult The son's statements regarding his father's
son attend an appointment at the recalling past events is typical for family
community mental health center. members of clients in the early stage of
While conversing with the nurse, Alzheimer's disease, when recent memory is
the son states, "I'm tired of hearing impaired. Telling the son to be more
about how things were 30 years accepting is critical and not an attempt to
ago. Why does Dad always talk educate. Understanding the client's level of
about the past?" The nurse should anxiety is unrelated to the memory loss of
tell the son: Alzheimer's disease. The client cannot stop
1. "Your dad lost his short-term reminiscing at will.
memory, but he still has his long-
term memory."
2. "You need to be more accepting
of your dad's behavior."
3. "I want you to understand your
dad's level of anxiety."
4. "Telling your dad that you are
tired of hearing about the past will
help him stop."
16 A client in the emergency Helicopod
5. department has a suspected
neurologic disorder. To assess gait,
the nurse asks the client to take a
few steps; with each step, the
client's feet make a half circle. To
document the client's gait, the
nurse should use which term?
16 A client is admitted in a disoriented Risk for injury
6. and restless state after sustaining
a concussion from a car accident.
Which nursing diagnosis takes
highest priority in this client's plan
of care?
16 A client is admitted to an acute Muscle contraction is palpable and visible.
7. care facility for treatment of a brain
tumor. When reviewing the chart,
the nurse notes that the client's
extremity muscle strength is rated
1/5. What does this mean?
16 A client is admitted to the Prepare to assist with ventilation.
8. emergency department with a
suspected overdose of an unknown
drug. Arterial blood gas values
indicate respiratory acidosis. What
should the nurse do first?
16 A client is admitted to the facility vertigo, tinnitus, and hearing loss.
9. for investigation of balance and
coordination problems, including
possible Mnire's disease. When
assessing this client, the nurse
expects to note:
17 A client is admitted with a cervical Ineffective breathing pattern
0. spine injury sustained during a
diving accident. When planning this
client's care, the nurse should
assign highest priority to which
nursing diagnosis?
17 A client is at risk for sensory 2, 4, & 5. Decreased attention span,
1. deprivation. Which of the following irritability, crying, and depression.
clinical signs would the nurse observe?
Select all that apply. Options 1 & 3 are clinical signs of
sensory overload.
1. Sleeplessness
2. Decreased attention span
3. Irritability
4. Excessive sleeping
5. Crying, depression
17 A client is being switched from levodopa 3
2. (l-dopa) to carbidopa-levodopa
(Sinemet). The nurse should monitor for
which of the following possible
complications during medication
changes and dosage adjustment? 1.
Euphoria. 2. Jaundice. 3. Vital sign
fluctuation. 4. Signs and symptoms of
diabetes.
17 A client is color blind. The nurse cones.
3. understands that this client has a
problem with:
17 A client is diagnosed with a conductive on the affected side by bone conduction.
4. hearing loss. When performing Weber's
test, the nurse expects that this client
will hear sound:
17 A client is exhibiting signs and 3. Keep the room organized and clean; A
5. symptoms of acute confusion/delirium. disorganized, cluttered environment
Which strategy should the nurse increases confusion.
implement to promote a therapeutic
environment? Keeping the room well-lit during waking
hours (option 1) promotes adequate
1. Keep lights in the room dimmed sleep at night. It is important to
during the day to decrease stimulation. eliminate unnecessary noise (option 2).
2. Keep the environmental noise level Client does not meet the standard
high to increase stimulation. criteria for restraint application (option
3. Keep the room organized and clean. 4).
4. Use restraints for client safety.
17 A client is experiencing agnosia as a 3.
6. result of vascular dementia. She is Agnosia is the lack of recognition of
staring at dinner and utensils without objects and their purpose. The nurse
trying to eat. Which intervention should should inform the client about the fork
the nurse attempt first? and what to do with it. Feeding the
1. Pick up the fork and feed the client client does not address the agnosia or
slowly. give the client specific directions. It
2. Say, "It's time for you to start eating should only be attempted if identifying
your dinner." the fork and explaining what to do with
3. Hand the fork to the client and say, it is ineffective. Waiting for the family to
"Use this fork to eat your green beans." care for the client is not appropriate
4. Save the client's dinner until her unless identifying the fork and
family comes in to feed her. explaining or feeding the client are not
successful.
17 A client is experiencing mood swings 3. A client who has brain damage may
7. after a stroke and often has episodes of be emotionally labile and may cry or
tearfulness that are distressing to the laugh for no explainable reason. Crying
family. Which is the best technique for is best dealt with by attempting to
the nurse to instruct family members to divert the client's attention. Ignoring the
try when the client experiences a crying behavior will not affect the mood swing
episode? 1. Sit quietly with the client or the crying and may increase the
until the episode is over. 2. Ignore the client's sense of isolation. Telling the
behavior. 3. Attempt to divert the client to stop is inappropriate.
client's attention. 4. Tell the client that
this behavior is unacceptable.
17 A client is having a tonic-clonic seizure. Take measures to prevent injury.
8. What should the nurse do first?
17 A client is hospitalized with Guillain- Even, unlabored respirations
9. Barr syndrome. Which data collection
finding is most significant?
18 A client is receiving an I.V. infusion of Increased urine output
0. mannitol (Osmitrol) after undergoing
intracranial surgery to remove a brain
tumor. To determine whether this drug
is producing its therapeutic effect, the
nurse should consider which finding the
most significant?
18 A client is scheduled for an EEG "Avoid stimulants and alcohol for 24 to 48
1. after having a seizure for the first hours before the test."
time. Client preparation for this
test should include which
instruction?
18 A client is scheduled for an initial ANS: B
2. treatment of electroconvulsive The most common side effect of ECT is
therapy (ECT). Which information temporary amnesia following the ECT
should a nurse include when procedure.
teaching about the potential side
effects of this procedure?

A. "You may experience transient


tangential thinking."
B. "You may experience some
memory deficit surrounding the
ECT."
C. "You may experience avolution
for the remainder of the day."
D. "You may experience a higher
risk for subsequent seizures."
18 A client is scheduled for Succinylcholine (Anectine)
3. electroconvulsive therapy (ECT).
Before ECT begins, the nurse
expects which neuromuscular
blocking agent to be administered?
18 A client is sitting in a chair and carefully move him to a flat surface and turn
4. begins having a tonic-clonic him on his side.
seizure. The most appropriate
nursing response is to:
18 A client is suspected of having electromyography (EMG).
5. amyotrophic lateral sclerosis
(ALS). To help confirm this
disorder, the nurse prepares the
client for various diagnostic tests.
The nurse expects the physician to
order:
18 A client is thrown from an C5
6. automobile during a collision. The
nurse knows that the client will be
able to maintain gross arm
movements and diaphragmatic
breathing if the injury occurs at
what vertebral level?
18 client is to begin drug for you will need to undergo treatment with iv
7. osteomyelitis: what is included in antibiotics for several weeks
educating client?
18 A client is transferred to the Administering a stool softener as prescribed
8. intensive care unit after
evacuation of a subdural
hematoma. Which nursing
intervention would reduce the
client's risk of increased
intracranial pressure (ICP)?
18 A client recovering from a stroke nonfluent aphasia.
9. has right-sided hemiplegia and
telegraphic speech and often
seems frustrated and agitated,
especially when trying to
communicate. However, the chart
indicates that the client's auditory
and reading comprehension are
intact. The nurse suspects that the
client has:
19 Clients admitted into the 4. Sensory overload; Sensoristasis is time of
0. emergency department may optimum arousal, not too much or too little.
experience behavior changes due Sensory reception is the process of receiving
to: internal and external data. This is partially
correct, in that the client does receive data
1. Sensoristasis that may result in behavior changes.
2. Sensory reception However, Answer 4 is a better answer in that
3. Stereognosis it more directly addresses the situation
4. Sensory overload presented. Stereognosis is the awareness of
an object's size, shape, and texture.
19 A client scheduled for ANS: A
1. electroconvulsive therapy (ECT) at 9:00 A client who is scheduled for ECT
a.m. is discovered eating breakfast at treatments is given nothing by mouth
8:00 a.m. Based on this observation, (NPO) for a minimum of 6 to 8 hours
which is the most appropriate nursing before treatment.
action?

A. The nurse notifies the client's


physician of the situation and cancels
the ECT.
B. The nurse removes the breakfast
tray and assists the client to the ECT
treatment room.
C. The nurse allows the client to finish
breakfast and reschedules ECT for
10:00 a.m.
D. The nurse increases the client's fluid
intake to facilitate the digestive
process.
19 A client states, "My doctor has told me ANS: B
2. I am a candidate for electroconvulsive Most clients require an average of 6 to 12
therapy (ECT). Where will the ECT treatments, but some may require up
treatment take place and how much to 20 treatments. Treatments are usually
time would this entail?" Which is the administered every other day, three
most accurate nursing reply? times per week. Treatments are
performed on either an inpatient or
A. "Clients typically receive ECT in outpatient basis depending on the need
their hospital room, daily for 1 month." for client monitoring.
B. "Clients typically receive 6 to 12 ECT
treatments, three times a week in an
outpatient setting."
C. "Clients typically receive an
unlimited number of treatments in the
hospital treatment room."
D. "Clients typically receive two to
three treatments in either an
outpatient or inpatient setting."
19 A client states that she is afraid she 2. Specific motor vehicle regulations and
3. will not be able to drive again because restrictions for people who experience
of her seizures. Which response by the seizures vary locally. Most commonly,
nurse would be best? 1. A person with evidence that the seizures are under
a history of seizures can drive only medical control is required before the
during daytime hours. 2. A person with person is given permission to drive. Time
evidence that the seizures are under of day is not a consideration when
medical control can drive. 3. A person determining driving restrictions related to
with evidence that seizures occur no seizures. The amount of time a person
more often than every 12 months can has been seizure-free is a consideration
drive. 4. A person with a history of for lifting driving restrictions; however,
seizures can drive if he carries a the time frame is usually 2 years. It is
medical identification card. recommended, not required, that a
person who is subject to seizures carry a
card or wear an identification bracelet
describing the illness to facilitate quick
identification in the event of an
emergency.
19 A client undergoes a craniotomy with Elevated 30 degrees
4. supratentorial surgery to remove a
brain tumor. On the first postoperative
day, the nurse notes the absence of a
bone flap at the operative site. How
should the nurse position the client's
head?
19 A client undergoes cerebral Hemiplegia, seizures, and decreased
5. angiography to evaluate for neurologic level of consciousness (LOC)
deficits. Afterward, the nurse checks
frequently for signs and symptoms of
complications associated with this
procedure. Which findings indicate
spasm or occlusion of a cerebral vessel
by a clot?
19 A client who experienced a severe Notify a physician of the findings.
6. stroke develops a fever and a cough
that produces thick, yellow sputum. A
nurse observes sediment in the client's
urine in the indwelling urinary catheter
tubing. Based on these findings, which
action should the nurse take?
19 A client who experienced a stroke has Make sure a tonsil suction device is
7. left-sided facial droop. During mouth readily available while providing mouth
care, the client begins to cough care.
violently. What should the nurse do?
19 A client who experienced a stroke Client's ability to climb the stairs while
8. that left her with residual right-sided using a walker
weakness was just discharged to go
home. The client lives in a two-story
house in which the bathroom is
located on the second floor. A home
health care nurse is visiting the
client for the first time. Which issue
should the nurse address during this
visit?
19 A client who has been severely raccoon eyes and Battle's sign.
9. beaten is admitted to the emergency
department. The nurse suspects a
basilar skull fracture after assessing:
20 The client who has had a stroke with 4, 5. When offering emotional support to a
0. residual physical handicaps becomes client who is discouraged and has a
discouraged by his physical negative self-concept because of physical
appearance. What approach to the handicaps, the nurse should approach the
client is best for the nurse to use to client with encouragement and patience.
help the client overcome his The client should be praised when he or she
negative self-concept? Select all that shows progress in efforts to overcome
apply. 1. Helpfulness. 2. Charity. 3. handicaps. An attitude of helpfulness and
Firmness. 4. Encouragement. 5. sympathy allows the client to assume a role
Patience. of someone not ordinary, someone who is
not like others. Regardless of the handicap,
the client still feels the same on the inside
and has the same innate needs for his or
her growth and developmental age-group.
An attitude of charity tends to make the
client feel like a "charity case" or like
someone who is given something free
because of his "condition." The client feels
unequal to his peers or unable to fulfill the
role relationships that were obtained before
the stroke. An approach using firmness is
inappropriate because it implies that the
client can do better if he just tries harder
and leaves no room for softness in the
approach to overcoming a negative self-
concept.
20 A client who is learning about ANS: C
1. electroconvulsive therapy (ECT) Clients are given medical clearance for ECT.
treatment asks a nurse "Isn't this This decreases the risk of injury from the
treatment dangerous?" Which is the treatment.
most appropriate nursing reply?

A. "No, this treatment is side-effect


free."
B. "There can be temporary paralysis
but full functioning returns within 3
hours of treatment."
C. "There are some risks, but a
thorough examination will determine
your candidacy for ECT."
D. "Transient ischemic attacks (TIA)
can occur but are rare."
20 A client who recently experienced a Notify the physician.
2. stroke tells the nurse that he has
double vision. Which nursing
intervention is the most appropriate?
20 A client who's paralyzed on the left side has been The client uses a mirror
3. receiving physical therapy and attending teaching to inspect his skin.
sessions about safety. Which behavior indicates that
the client accurately understands safety measures
related to paralysis?
20 A client who's receiving phenytoin (Dilantin) to control 10 to 20 mcg/ml
4. seizures is admitted to the health care facility for
observation. The physician orders measurement of the
client's serum phenytoin level. Which serum phenytoin
level is therapeutic?
20 A client who sustained a closed head injury in a motor "What has the physician
5. vehicle accident is diagnosed as brain dead by a explained about the
neurosurgeon. The physician has scheduled a meeting client's prognosis?"
with the client's family about discontinuing life
support. Before the meeting, a family member asks
the nurse her opinion about life support. Which
response by the nurse is appropriate?
20 A client who sustained a closed head injury in a Fasten the restraint to
6. skating accident pulls out his feeding tube, I.V. the bed frame using a
catheter, and indwelling urinary catheter. To ensure quick-release knot.
this client's safety, a physician prescribes restraints.
Which action should a nurse take when using
restraints?
20 a client who sustained a crush injury to right lower leg notify the hcp- this is
7. c/o numbness and tingling of the affected extremity. signs of compartment
right leg appears pale and pedal pulse is weak. syndrome
20 A client who sustained an L1 to L2 spinal cord injury in "What has your
8. a construction accident asks a nurse if he'll ever be physician told you
able to walk again. Which response by the nurse is about your ability to
appropriate? walk again?"
20 A client who was diagnosed with multiple sclerosis 3 "It's important for us to
9. years ago now presents with lower extremity have this information.
weakness and heaviness. During the admission You should review the
process, the client presents her advance directive, document with your
which states that she doesn't want intubation, physician at every
mechanical ventilation, or tube feedings should her admission."
condition deteriorate. How should the nurse respond?
21 A client who was trapped inside a car for hours after a Midbrain
0. head-on collision is rushed to the emergency
department with multiple injuries. During the
neurologic examination, the client responds to painful
stimuli with decerebrate posturing. This finding
indicates damage to which part of the brain?
21 A client with a conductive hearing disorder caused by "Don't fly in an airplane,
1. ankylosis of the stapes in the oval window undergoes climb to high altitudes,
a stapedectomy to remove the stapes and replace the make sudden
impaired bone with a prosthesis. After the movements, or expose
stapedectomy, the nurse should provide which client yourself to loud sounds
instruction? for 30 days."
21 A client with a hemorrhagic stroke is slightly agitated, 4,2,1,3
2. heart rate is 118, respirations are 22, bilateral rhonchi
are auscultated, SpO2 is 94%, blood pressure is 144/
88, and oral secretions are noted. What order of
interventions should the nurse follow when suctioning
the client to prevent increased intracranial pressure
(ICP) and maintain adequate cerebral perfusion? 1.
Suction the airway 2.hyperoxygenate 3.suction the
mouth 4. Provide sedation
21 A client with Alzheimer's disease is admitted for hip Risk for caregiver role
3. surgery after falling and fracturing the right hip. The strain related to
spouse tells the nurse of feeling guilty for letting the increased client care
accident happen and reports not sleeping well because needs
the spouse has been getting up at night and doing odd
things. Which nursing diagnosis is most appropriate
for the client's spouse?
21 The client with Alzheimer's 3.
4. disease may have delusions The nurse needs to present reality without
about being harmed by staff and arguing with the delusions. Therefore, stating
others. When the client expresses that the client is in the hospital and the nurses
fear of being killed by staff, are trying to help is most appropriate. The
which of the following responses client doesn't recognize the delusion or why it
is most appropriate? exists. Telling the client that the staff likes him
1. "What makes you think we too much to want to kill him is inappropriate
want to kill you?" because the client believes the delusions and
2. "We like you too much to want doesn't know that they are false beliefs. It also
to kill you." restates the word, kill, which may reinforce the
3. "You are in the hospital. We client's delusions. Telling the client not to be
are nurses trying to help you." silly is condescending and disparaging and
4. "Oh, don't be so silly. No one therefore inappropriate.
wants to kill you here."
21 A client with amyotrophic lateral Powerlessness
5. sclerosis (ALS) tells the nurse,
"Sometimes I feel so frustrated. I
can't do anything without help!"
This comment best supports
which nursing diagnosis?
21 A client with an inflammatory increased intraocular pressure.
6. ophthalmic disorder has been
receiving a -inch ribbon of
corticosteroid ointment in the
lower conjunctival sac four times
per day as directed. The client
reports a headache and blurred
vision. The nurse suspects that
these symptoms represent:
21 A client with a spinal cord injury increase the frequency of the catheterizations.
7. and subsequent urine retention
receives intermittent
catheterization every 4 hours.
The average catheterized urine
volume has been 550 ml. The
nurse should plan to:
21 A client with a suspected brain Determine whether the client is allergic to
8. tumor is scheduled for computed iodine, contrast dyes, or shellfish.
tomography (CT). What should
the nurse do when preparing the
client for this test?
21 A client with a tentative a positive edrophonium (Tensilon) test.
9. diagnosis of myasthenia gravis is
admitted for a diagnostic workup.
Myasthenia gravis is confirmed
by:
22 A client with cognitive deficits is ANS: C
0. extremely suicidal. The client has A client who is experiencing cognitive deficits
not responded to antidepressants cannot give informed consent that is required
and the treatment team is prior to ECT treatment. A court proceeding
considering electroconvulsive could determine the client's level of
therapy (ECT). What client competency and, if necessary, appoint a
information would determine the guardian.
feasibility of this treatment option?

A. Because the client is extremely


suicidal, ECT is an appropriate
option.
B. Because antidepressant
medications have been ineffective,
ECT is a good alternative.
C. Because informed consent is
required for ECT, cognitive deficits
preclude this option.
D. Because of the client's cognitive
deficits, a signed consent is
waived.
22 The client with dementia states to 1.
1. the nurse, "I know you. You're Because of the client's short-term memory
Margaret, the girl who lives down impairment, the nurse gently corrects the
the street from me." Which of the client by stating her name and who she is.
following responses by the nurse is This approach decreases anxiety,
most therapeutic? embarrassment, and shame and maintains
1. "Mrs. Jones, I'm Rachel, a nurse the client's self-esteem. Telling the client that
here at the hospital." 2. "Now Mrs. she knows who the nurse is or that she forgot
Jones, you know who I am." can elicit feelings of embarrassment and
3. "Mrs. Jones, I told you already, shame. Saying, "I told you already" sounds
I'm Rachel and I don't live down the condescending, as if blaming the client for
street." not remembering.
4. "I think you forgot that I'm
Rachel, Mrs. Jones."
22 A client with dementia who prefers 3.
2. to stay in his room has been The tape and television are competing, even
brought to the dayroom. After 10 conflicting, stimuli. Crime events portrayed
minutes, the client becomes on television could be misperceived as a real
agitated and retreats to his room threat to the client. A low number of clients
again. The nurse decides to assess and the presence of a few staff members
the conditions in the dayroom. quietly working are less intense stimuli for
Which is the most likely occurrence the client and not likely to be disturbing.
that is disturbing to this client?
1. There is only one other client in
the dayroom; the rest are in a
group session in another room.
2. There are three staff members
and one physician in the nurse's
station working on charting.
3. A relaxation tape is playing in
one corner of the room, and a
television airing a special on crime
is playing in the opposite corner.
4. A housekeeping staff member is
washing off the countertops in the
kitchen, which is on the far side of
the dayroom.
22 A client with early dementia 2.
3. exhibits disturbances in her mental Clients with chronic cognitive disorders
awareness and orientation to experience defects in memory orientation
reality. The nurse should expect to and intellectual functions, such as judgment
assess a loss of ability in which of and discrimination. Loss of other abilities,
the following other areas? such as speech, endurance, and balance, is
1. Speech. less typical.
2. Judgment.
3. Endurance.
4. Balance.
22 A client with epilepsy is having a place the client on his side, remove
4. seizure. During the active seizure dangerous objects, and protect his head.
phase, the nurse should:
22 A client with Guillain-Barr "The paralysis caused by this disease is
5. syndrome has paralysis affecting temporary."
the respiratory muscles and
requires mechanical ventilation.
What should the nurse tell the
client about the paralysis?
22 A client with hypertension comes to the clinic for a Vertigo
6. routine checkup. Because hypertension is a risk
factor for cerebral hemorrhage, the nurse questions
the client closely about warning signs and
symptoms of hemorrhage. Which complaint is a
possible indicator of cerebral hemorrhage in this
client?
22 A client with idiopathic seizure disorder is being "Schedule follow-up visits
7. discharged with a prescription for phenytoin with your physician for
(Dilantin). Client teaching about this drug should blood tests."
include which instruction?
22 A client with impaired vision is admitted to the 1, 3, & 4. Identify yourself
8. hospital. Which interventions are most appropriate by name, stay in the
to meet the client's needs? Select all that apply. client's field of vision, and
explain the sounds in the
1. Identify yourself by name. environment.
2. Decrease background noise before speaking.
3. Stay in the client's field of vision. Options 2 and 5 relate to
4. Explain the sounds in the environment. interventions for a client
5. Keep your voice at the same level throughout the with a hearing impairment.
conversation.
22 A client with multiple sclerosis (MS) is experiencing 4
9. bowel incontinence and is starting a bowel
retraining program. Which strategy is
inappropriate? 1. Eating a diet high in fiber. 2.
Setting a regular time for elimination. 3. Using an
elevated toilet seat. 4. Limiting fluid intake to 1,000
mL/ day.
23 A client with multiple sclerosis (MS) is receiving 3
0. baclofen (Lioresal). The nurse determines that the
drug is effective when it achieves which of the
following? 1. Induces sleep. 2. Stimulates the
client's appetite. 3. Relieves muscular spasticity. 4.
Reduces the urine bacterial count.
23 A client with multiple sclerosis (MS) lives with her 2
1. daughter and 3-year-old granddaughter. The
daughter asks the nurse what she can do at home
to help her mother. Which of the following
measures would be most beneficial? 1.
Psychotherapy. 2. Regular exercise. 3. Day care for
the granddaughter. 4. Weekly visits by another
person with MS.
23 A client with multiple sclerosis who is unable to "I'm sorry you haven't
2. bathe herself complains that other staff members been bathed. I'm available
haven't been bathing her. How should the nurse to bathe you now."
respond to this client's complaint?
23 A client with Parkinson's disease asks the nurse to 1
3. explain to his nephew "what the doctor said the
pallidotomy would do." The nurse's best response
includes stating that the main goal for the client
after pallidotomy is improved: 1. Functional ability.
2. Emotional stress. 3. Alertness. 4. Appetite.
23 A client with Parkinson's disease is prescribed 2. Levodopa is prescribed
4. levodopa (l-dopa) therapy. Improvement in which of to decrease severe muscle
the following indicates effective therapy? 1. Mood. rigidity. Levodopa does not
2. Muscle rigidity. 3. Appetite. 4. Alertness. improve mood, appetite, or
alertness in a client with
Parkinson's disease.
23 A client with Parkinson's disease needs a long time 2
5. to complete her morning hygiene, but she becomes
annoyed when the nurse offers assistance and
refuses all help. Which action is the nurse's best
initial response in this situation? 1. Tell the client
firmly that she needs assistance and help her with
her care. 2. Praise the client for her desire to be
independent and give her extra time and
encouragement. 3. Tell the client that she is being
unrealistic about her abilities and must accept the
fact that she needs help. 4. Suggest to the client
that if she insists on self-care, she should at least
modify her routine.
23 A client with Parkinson's disease visits the "I take the medication at
6. physician's office for a routine checkup. The nurse bedtime."
notes that the client takes benztropine (Cogentin),
0.5 mg by mouth daily, and asks when the client
takes the drug each day. Which response indicates
that the client understands when to take
benztropine?
23 A client with quadriplegia is in spinal shock. What Absence of reflexes along
7. should the nurse expect? with flaccid extremities
23 A client with respiratory complications of Suction machine with
8. multiple sclerosis (MS) is admitted to the catheters
medical-surgical unit. Which equipment is most
important for the nurse to keep at the client's
bedside?
23 A client with seizure disorder comes to the Excessive gum tissue growth
9. physician's office for a routine checkup. Knowing
that the client takes phenytoin (Dilantin) to
control seizures, the nurse assesses for which
common adverse drug reaction?
24 A client with weakness and tingling in both legs Lung auscultation and
0. is admitted to the medical-surgical unit with a measurement of vital
tentative diagnosis of Guillain-Barr syndrome. In capacity and tidal volume
this syndrome, polyneuritis leads to progressive
motor, sensory, and cranial nerve dysfunction. On
admission, which assessment is most important
for this client?
24 Clonazepam (Klonopin) Approved for:
1. -Absence (failed
ethosuximide)
-Myoclonic
-Akinetic
Side effects:
-Sedation, cognitive
dysfunction, ataxia
24 cognex used in treatment of mild to
2. moderate dementia
associated with Alzheimer's
disease. Increases level f ACH
in the CNS by inhibiting its
breakdown
24 Coma A state of being unaware of
3. one's surroundings and being
unable to react or respond to
people, places, or things.
24 A common age-related change in auditory acuity Presbycusis
4. is called:
24 COMMON TYPES OF BURST: Characterized by multiple pieces of bone; often
5. FRACTURES occurs at bone ends or in vertebrae.
COMMINUTED: More than one fracture line; more than two
bone fragments; fragments may be splintered or crushed.
COMPLETE: Break across the entire section of bone,
dividing it into distinct fragments; often displaced.
DISPLACED:Fragments out of normal position at fracture
site.
INCOMPLETE:Fracture occurs theough only one cortex of
the bone; usually nondisplaced.
LINEAR: Fracture line is intact; fracture line is intact;
fracture is caused by minor to moderate force applied
directly to the bone.
LONGITUDINAL:Fracture line extends in the direction of
the bone's longitudinal axis.
NONDISPLACED: Fragments aligned at fracture site.
OBLIQUE: Break occurs at an angle across the bone.
Occurs at approximately 45 deg angle across the
longitudinal axis of the bone.
SPIRAL: Break twists around the bone.Fracture line results
from twisting force.
STELLATE:Fracture lines radiate from one central point.
TRANSVERSE: Break occurs across the bone. Fracture line
occurs at a 90 deg angle to longitudinal axis of bone.
AVULSION:Bone fragments are torn away from the body of
the bone at the site of attachment of a ligament or
tendon.
COMPRESSION:Bone buckles an deventually cracks as the
result of unusual loading force applied to its longitudinal
axis.
GREENSTICK: One side of a bone is broken; the other side
is bent.
COLLES':Fracture within the last inch of the distal radius;
distal fragment is displaced in a position of dorsal and
medial deviation.
POTT'S: fracture of the distal fibula, seriously disrupting
the tibiofibular articulation; a piece of the medial
malleolus may be chipped off as a result of rupture of the
internal lateral ligament.
IMPACTED: Telescoped fracture, with one fragment driven
into another.
24 The community health Immunize adolescents and college freshman against
6. nurse is developing a Neisseria meningitides.
program to decrease the
incidence of meningitis rational: The Neisseria meningitides vaccination is
in adolescents and recommended for children ages 11 and 12, unvaccinated
young adults. Which teens entering high school, and college freshmen. Hand
nursing action is most washing may help decrease the spread of bacteria, but it
important? is not as effective as immunization. Vaccination with
Haemophilus influenzae is for infants and toddlers.
Because adolescents and young adults are in school or
the workplace, avoiding crowds is not realistic.
24 Compare and contrast Meningitis: inflammation of the meninges of the brain and
7. four inflammatory spinal cord.
neurological conditions.
Enchephalitis: an acute inflammation of the white and
gray matter of the brain.

Brain abscess: collection of purulent material within the


brain.
24 Compare and contrast ...
8. the different types of
seizures in children.
24 Compare the symptoms ...
9. of PDD, Autism and
Down syndrome.
25 Complex partial seizure Focal seizure in which consciousness IS impaired
0. -Common feature: motionless stare during which patient
does not respond to external stimuli and expresses
automatisms
-Patient does not remember seizure
25 Computed Tomography -cross sectional images; contrast optional
1. (CT) scan -know if pt has allergy to shellfish or iodine: these
allergies require different contrast media
-Assess renal fxn (BUN); contrast excreted renally
25 COMT- entacapone Medication that decreases breakdown of levadopa making
2. (Comtan) more available to brain for dopamine. Monitor for
dyskinesia, hyperkinesia, diarrhea. Dark urine is a normal
finding.
25 Conductive DESCRIPTION: Hearing loss in which sound does not travel well
3. Hearing Loss to the sound organs of the inner ear. The volume of sound is
less, but the sound remains clear. If volume is raised, hearing is
normal. Hearing loss is the most common disability in the U.S. It
usually results from cerumen impaction or middle ear disorders.
25 A consensual light C. the right pupil constricts when a light is shone into the left
4. reflex is present pupil
when which of the
following occurs?

A. the right pupil


dilates when a
light is shone on
the left pupil

B. the left pupil


dilates
immediately after
the light is
removed from the
left pupil

C. the right pupil


constricts when a
light is shone into
the left pupil

D. the left pupil


constricts after the
light is removed
from the right
pupil
25 corneal reflex abnormal: loss of blink (CN V)
5.

25 Corticosteroids: DRUGS: Hydrocortisone; Prednisone; Dexamethasone


6. INDICATIONS: Hormone replacement; Severe Rheumatoid
Arthritis; Autoimmune disorders. decrease the bodies
inflammatory response, makes client more prone to
infection but mask the signs of inflammation.
ADVERSE REACTIONS: Emotional lability, personality changes.
Impaired wound healing, bruise easily. Skin fragility. Abnormal
fat deposition. Hyperglycemia, bs increase. Hirsutism, moon
face. Osteoporosis remember steroids leech calcium from
the bones.
NURSING IMPLICATIONS: Wean slowly (administer high dose
then taper off).; careful monitoring is required during withdrawal
(NO cold turkey). Monitor serum potassium (normal: 3.5-5),
glucose (normal: 70-100) (can become diabetic), and sodium
(normal:136-145). Weigh daily, report weight gain of more than
5lb per week this could indicate water retention. Administer
with antiulcer drugs or food (zantac, pepsid,
tagament) ulcerogenic. Use care to prevent injuries. Monitor
bp and pulse closely, these can increase bp. remember to
replace calcium, supplement.
25 Coup-contrecoup ..., These injuries to the cerebrum occur when a blow to the
7. head caused the brain to shift towards the area of impact and
injure itself by hitting the inner surface of the skull, and then
rebounding in the opposite direction and injuring itself again by
hitting the skull on the opposite side of the skull where the
original blow was delivered.
25 CPP normals > 70-80mmHg
8.

25 Cranial Nerve I Olfactory - sensory


9.

26 Cranial Nerve II Optic Nerve - sensory


0.

26 Cranial Nerve III Oculomotor - motor: Assess pupil size and light reflex
1.

26 Cranial Nerve IV Trochlear - motor: eye movement


2.

26 Cranial Nerve IX Glossopharyngeal


3. -motor: depress tongue, say "Ahh"; uvula moves, gag
reflex, voice sounds smooth
26 Cranial Nerve V Trigeminal -mixed
4. - motor: clenching teeth
-sensory: assess ability to taste
26 Cranial Nerve VI Abducens - motor: muscles that move the eye
5.

26 Cranial Nerve VII Facial


6. -motor: symmetry and mobility of face
-sensory: ability to taste
26 Cranial Nerve VIII Acoustic or Vestibulocochlear
7.

26 Cranial Nerve X Vagus


8.

26 Cranial Nerve XI Spinal Accessory


9. -motor: patient rotate head and shrug shoulder against
resistance
27 Cranial Nerve XII Hypoglossal
0. -motor: assess tongue control
27 CT Scan a series of x-ray photographs taken from different
1. angles and combined by computer into a composite
representation of a slice through the body.
27 The culturally sensitive 2. May be accustomed to, and need, high stimulation
2. nurse will realize which of level; An individual's culture often determines the
the following about a amount of stimulation that the client considers normal
client from a large active or usual. A person raised in a large active Latino family
Latino family who is put may be accustomed to more stimulation than an only
into isolation for a child raised in a European American family. A decrease
communicable disease? in sensory stimulation could result in sensory
deprivation or culture shock.
1. The number of visitors
greatly needs to be 2. May
be accustomed to, and
need, high stimulation
level
3. Is a likely candidate for
sensory overload
4. Will need more personal
space than other clients
27 D Which patient is most at risk for developing delirium?
3. a. a 50 year old woman with cholecystitis
b. a 19 year old man with a fractured femur
c. A 42 year old woman having an elective
hysterectomy
d. A 78 year old man admitted to the medical unit with
complications related to heart failure
27 D Dementia is defines as a
4. a. syndrome that results only in memory loss
b. disease associated with abrupt changes in behavior
c. disease that is always due to reduced blood flow to
the brain
d. syndrome characterized by cognitive dysfunction and
loss of memory
27 D The nurse assesses that an 87 year old woman with
5. alzheimers disease is continually rubbing, flexing, and
kicking out her legs throughout the day. THe night shift
reports that this same behavior escalates at night,
preventing her from obtaining her required sleep. The
next step the nurse should take is to:
a. ask the physician for a daytime sedative for the
patient
b. request soft restraints to prevent her from falling out
of her bed.
c. ask the physician for a nightime sleep medication for
the patient
d. assess the patient more closely, suspecting a
disorder such as restless legs syndrome
27 D The clinical diagnosis of dementia is based on
6. a. CT or MRS
b. brain biopsy
c. electroencephalogram
d. patient history and cognitive assessment
27 Damage to which Temporal lobe
7. area of the brain
results in
receptive
aphasia?
27 Decerebrate -"extensor posturing";
8. posturing -abduction of arms, elbow and wrist extension
27 decerebrate posturing in which the neck is extended with jaw clenched; arms
9. posturing are pronated, extended, and close to the sides; legs are extended
straight out; more ominous sign of brain stem damage. Most
Severe.
28 Decorticate -"flexor posturing" or "mummy baby" (think Egyptian mummy
0. posturing preservation)
-adduction of arms (arms fold to chest); flexion of elbows and
wrists
28 decorticate characterized by upper extremities flexed at the elbows and held
1. posturing closely to the body and lower extremities that are externally
rotated and extended. occurs when the brainstem is not inhibited
by the motor function of the cerebral cortex.
28 Deep brain Brain pacemaker. Delivers shocks to the brain and helps stop
2. stimulation signals that cause PD
28 Define aura; Aura = subjective; at onset of seizure but w/o clinical signs of
3. define seizure
automatism
Automatism = coordinated, stereotyped, involuntary motor
activity (lip smacking, hand wringing, verbalization of short,
stereotyped phrases)
28 Define a disorder involving the relationship between nerves and
4. neuromuscular muscles, and especially the weakening or dysfunction of muscles.
disorder.
28 Define "pill Bradykinesia: since the extrapyramidal system regulates posture
5. rolling" and and skeletal muscle tone, a result is the characteristic of
"bradykinesia". bradykinesia of Parkinson's. It is a slowness of movement.
Slowness in the execution of movement, not initiation (like
akinesia). "Stone face".

Pill Rolling: The Parkinson's tremor tends to more often affect the
hands and causes a movement sometimes referred to as "pill
rolling". This "pill rolling" 'tremor' involves the uncontrolled
movement of the thumb and finger(s) in a back and forth motion.
This may also appear as the thumb and fingers are rubbing
together, hence the term "pill rolling" movement. These tremors
are usually rhythmic and may occur between 4 to 5 cycles per
second. It may only affect one side of the body, or one hand, but
as the disease progresses, the tremor may become more
generalized affecting many parts of the body.
28 Define seizure vs. Seizure = altered behavior or sensorium due to excessive or
6. epilepsy hypersynchronous discharge of neurons

Epilepsy = predisposition to generate seizures + occurrence of at


least 1 seizure
28 Describe a ...
7. nursing care for
the patient with a
brain tumor.
28 Describe assessment Nuchal rigidity: neck stiffness
8. findings: nuchal
rigidity, photophobia, Photophobia: intolerance of bright light
opisthotonus,
Kerning's sign, and Opisthotnus: A type of spasm in which the head and heels
Brudzinksi's sign. arch backward in extreme hyperextension and the body
forms a reverse bow
Kerning's sign: is positive when the leg is bent at the hip
and knee at 90 degree angles, and subsequent extension in
the knee is painful

Brudzinski's sign: is the appearance of involuntary lifting of


the legs when lifting a patient's head.
28 Describe a therapeutic make schedule of the client's daily activities, label drawers
9. environment for a containing client's clothes and label rooms, use
patient with communication techniques to the client's level of ability,
Alzheimer's disease. when the client is agitated re-direct attention, if pt wanders
they need a MedicAlert, schedule rest periods or quite times
throughout the day, set boundaries by placing red or yellow
tape on the floor, assign the same caregivers as much as
possible, music/art therapy, orient to person place and time
if needed.
29 Describe common X-rays
0. diagnostic tests for CT Scans
the patient with MRI
neurological
manifestations. Cerebral angiography: contrast material is injected and an
combined X-ray and fluroscopy is performed.

Myelography: X-ray of spinal cord and canal after contrast


media is injected.

PET: radioactive agent is injected and CT measures


metabolic activity of the brain.

Ultrasound

Carotid duplex study: sound waves identify blood flow


velocity to determine the presence of occlusive vascular
disease.

EEG

EMG: needles inserted in muscles to record electrical


activity.

Evoked potentials: electrodes are placed on scalp and skin


to record the visual or auditory stimulus along sensory
pathways
29 Describe common The warning signs of a TIA are exactly the same as for a
1. symptoms of a TIA. stroke.
Sudden numbness or weakness of the face, arm or leg,
especially on one side of the body

Sudden confusion, trouble speaking or understanding

Sudden trouble seeing in one or both eyes

Sudden trouble walking, dizziness, loss of balance or


coordination

Sudden, severe headache with no known cause


29 Describe four types spastic (most common): the cortex is affected resulting in the
2. of CP. child having a scissor-like gait where one foot crosses in front
of the other foot. other s&s: underdeveloped limbs, increased
deep tendon reflexes, contractures, involuntary muscle
contraction and relaxation, flexion.
athetoid: the basal ganglia are affected resulting in
uncoordinated involuntary motion. other s&s: uncontrolled
involuntary movements, drooling writhing, all extremities
move with voluntary movement, difficulty swallowing, facial
grimacing.
ataxic: the cerebellum is affected resulting in poor balance and
difficulty with muscle coordination. other s&s: wide-based gait,
unsteadiness, clumsiness, poor balance, unnatural muscle
coordination.
29 Describe IQ related ...
3. to MR.
29 Describe nursing Acute phase: Aggressive respiratory therapy. Above C5 injury
4. and interdisciplinary are intubated/ventilator. Intermittent positive-pressure
care for a patient breathing (IPPB) are used to prevent atelectasis. Foley
with a spinal cord catheter, surgery/immobilization, tracheotomy if long-term
injury. ventilation is needed. Parentreal nutrition and fluids until the
GI tract starts functioning. A diet high in protein and fiber.
Bowel program during spinal shock: manual disipaction and
small-volume enemas. PT and OT therapy: passive ROM and
then aggressive rehab long term.
Chronic phase: orthostatic hypotension prevention, dietary
management (weight gain likely), skin care/turning, respiratory
management.
29 Describe nursing Be aware that phantom pain is real and will eventually
5. care for the client disappear. Administer pain meds; phantom pain responds to
who is experiencing meds.
phantom pain after
amputation.
29 Describe nursing 1.Reposition the client on a regular schedule as dictated by
6. care to assist the individual situation.
patient with Allow proper blood circulation, prevents venous stasis and
mobility, gait, formation of decubitus ulcers
strength, and motor 2.Place patient on moderate high back rest position with head
skills. at the midline
allows greater lung expansion and prevent compression on
the diaphragm from prolong bed rest.
3.Support body part especially the affected side using pillows
or rolls
Prevention from developing pressure ulcers particularly on
bony prominences
4.Keep body aligned and place extremities in proper position
Proper positioning and turning maintains joint function and
prevents contractures.
5.Perform active range of motion on unaffected extremities
and passive range of motion exercises on affected extremities
every 4 hours.
Active range of motion exercise improves muscle strength
while passive range of motion exercise improves joint mobility
6.Encourage patient to perform certain movements according
to ones capability such as moving left upper and lower
extremities, moving tongue, and moving head.
To maintain strength and integrity of the functioning body
parts.
7. Raise the siderails and provide a responsible watcher.
weakness and loss of body coordination are at risk for fall or
accidents.
8.Provide enteral feeding via NGT
Provision of nutrition for metabolic and energy demand.
9. Perform regular skin care. (e.i sponge bath,apply lotion)
Maintains skin integrity and decreases risk for skin
breakdown.
10.Schedule activities with adequate rest periods
To reduce fatigue and decrease energy demand
11. Provide a positive atmosphere while acknowledging ones
difficulty.
Helps minimize frustration and rechannel energy.
29 Describe nursing hold next dose immediately, call PHP if bleeding will not stop
7. measures in after 10min of pressure if external, be prepared to administer
response to antidote, FFP, or other drugs as ordered by PHP.
unexpected
negative response
(bleeding).
29 Describe postop Elevate stump for first 24 hours. Do not elevate stump after 48
8. residual lib care hours. Keep stump in extended position, and turn client to
after amputation prone position three times a day to prevent flexion contracture.
for the first 48
hours?
29 Describe Alzheimer's effects cranial nerves, especially #19.
9. predisposing
factors linked to patho
Alzheimer's disease. 1. loss of nerve cells 2. reduce brain size 3. presence of
neurofibrillary tangles 4. neuritic plaques by amyloid protein.

Aging. One out of eight people over age 65 has Alzheimer's.


Nearly half of people over age 85 have the disease.

Family history and genetics


Another risk factor is family history. Research has shown that
those who have a parent, brother or sister with Alzheimer's are
two to three times more likely to develop the disease.

There appears to be a strong link between serious head injury


and future risk of Alzheimer's. It's important to protect your
head by buckling your seat belt, wearing your helmet when
participating in sports and "fall-proofing" your home.

Some evidence suggests that strategies for general healthy


aging may also help reduce the risk of developing Alzheimer's.
These measures include controlling blood pressure, weight and
cholesterol levels; exercising both body and mind; eating a
balanced diet; and staying socially active.

Scientists don't know yet exactly how Alzheimer's and diabetes


are connected, but they do know that excess blood sugar or
insulin can harm the brain in several ways:

Diabetes raises the risk of heart disease and stroke, which hurt
the heart and blood vessels. Damaged blood vessels in the
brain may contribute to Alzheimer's disease.
The brain depends on many different chemicals, which may be
unbalanced by too much insulin. Some of these changes may
help trigger Alzheimer's disease.
High blood sugar causes inflammation. This may damage brain
cells and help Alzheimer's to develop.
30 Describe seven LOC: EARLY IICP: restleness, irritability, LATE IICP: coma, no
0. signs and symptoms response to stimuli
of increased
intracranial Pupils: EARLY IICP: equal round and reactive to light. LATE IICP:
pressure. sluggish response, progressing to fixed response, pupils may
dilate only on one side.

Vision: EARLY IICP: decreased visual acuity, blurred vision. LATE


IICP: unable to assess

Motor Function: EARLY IICP: weakness in on extremity or side.


LATE IICP: decorticate or decebrate posturing.

Speech: EARLY IICP: difficulty speaking. LATE IICP: cannot


assess due to decrease in LOC.

Blood Pressure: EARLY IICP: elevated blood pressure. LATE IICP:


Cushing's Triad, increased systolic BP, wideining pulse
pressure, bradycardia

Pulse: EARLY IICP: slighty elevated. LATE IICP: widening pulse.

Respiration: EARLY IICP: rate may increase. LATE IICP:


decreased respiratory rate or cheyne-stokes breathing.

Temperature: EARLY IICP: may be decreased or increased. LATE


IICP: significantly elevated.

Other sx: EARLY IICP: headaches worse on rising in the morning


and with position changes, LATE IICP: Continual headache,
projectile vomitiing. Loss of pupil, corneal, gag, and swallowing
reflexes.
30 Describe s&s of a depends on the location of the injury and other unique factors.
1. spinal cord injury. general complications include hypotonia, autonomic dysreflexia,
spinal shock, orthostatic hypotension, bradycardia, DVT,
pressure ulcers, pain, limited chest expansion, pneumonia,
stress ulcers GI, urinary incontinence, neurogenic bladder, UTIs,
impotence, decreased vaginal lubrication, join contractures,
muscle spasms, muscle atrophy, pathologic fractures,
hypercalcemia

spinal shock: temporary loss of reflex activity below the level of


spinal cord injury, this usually happens 30-60 min after a
complete SCI. There is loss of motor function, sensation, spinal
reflexes, and autonomic function. other manifestations include
bradycardia, hypotension, loss of sweating and temp control,
bowel/bladder dysfunction, flaccid paralysis, loss of ability to
perspire. could last from days to months before reflex activity
returns.
within the first year of injury even, the patient is at risk for
spinal shock whenever they are moved.
30 Describe strategies Ischemic Stroke Modifiers:
2. and approaches to Hypertension (Because HBP damages arteries throughout the
prevent a CVA. body, it is critical to keep your blood pressure within acceptable
ranges to protect your brain from this often disabling or fatal
event.)
Atrial fibrillation
Hyperlipidemia
Diabetes mellitus (associated with accelerated
atherogenesis)
Smoking
Asymptomatic carotid stenosis
Obesity
Excessive alcohol consumption

Hemorrhagic Stroke Modifiers:


Primary prevention of hemorrhagic stroke is the best ap- proach
and includes managing hypertension and ameliorat- ing other
significant risk factors. Control of hypertension, especially in
people older than 55 years of age, reduces the risk of
hemorrhagic stroke. Additional risk factors are increased age,
male gender, and excessive alcohol intake. Stroke risk
screenings provide an ideal opportunity to lower hemorrhagic
stroke risk by identifying high- risk individuals or groups and
educating patients and the community about recognition and
prevention.
30 Describe symptoms Irritability
3. that may indicate a Restlessness
change in LOC. Personality changes
Short-term memory changes
Disorientation to place, time, and person
30 Describe a daily routine that they can count on, everything familiar to
4. techniques to them has one place inside of their room, encourage as much
promote self-care as possible, demonstrate use of equipment, modify
independence for clothing with Velcro and lay out daily clothing, encourage
the patient with "finger foods" during meals.
impaired cognition.
30 Describe the Eating Devices
5. adaptive Nonskid mats to stabilize plates
equipment Plate guards to prevent food from being pushed
available for off plate
patient care. Wide-grip utensils to accommodate a weak grasp

Bathing and Grooming Devices


Long-handled bath sponge
Grab bars, nonskid mats, handheld shower heads
Electric razors with head at 90 degrees to handle
Shower and tub seats, stationary or on wheels

Toileting Aids
Raised toilet seat
Grab bars next to toilet

Dressing Aids
Velcro closures
Elastic shoelaces
Long-handled shoe horn

Mobility Aids
Canes, walkers, wheelchairs
Transfer devices such as transfer boards and belts
30 Describe the Prothrombin time (PT) and the international normalized ratio
6. criteria for (INR) are used to monitor the pts response to warfarin therapy.
determining The daily dose is based on these labs. Therapeutic range of the
dosage of PT is 1.2 to 1.5 times the control value (11-13 seconds, think
anticoagulant "pre teen"). INR should be maintained between 2 and 3.
drugs. (PT, INR,
APTT) The The Activated Partial Thromboplastin Time (APTT)
determines the overall capacity of the blood to clot for pts on
heparin. 1.5-2.5 the control value (25-45 seconds, think "prime
teaching time"). The APTT needs to be drawn q6hrs, heparin has
a short half-life and so the amount can vary greatly within a
short period of time.

If the numbers are too low, they are at risk for clots. If too high,
then they are at risk for bleeding. There is a narrow therapeutic
range for anticoagulants.
30 Describe the Aura: bright light
7. phases of a tonic-
clonic seizure. Tonic phase: muscles are rigid with the arms extended and jaws
clenched

Clonic phase: movements are jerky as the muscles alternately


contract and relax

Postictal phase: the pt is unconscious for 30 minutes and then


regains conciousness slowly
30 Describe the role Nurses work in diverse community settings to provide primary
8. of the nurse in the nursing and health care across the lifespan. Traditionally
community setting. community nurses meet a continuum-of-health needs that
range from the management of specific disease/s to broader
community development and public health promotion needs.
Health promotion and intervention consciously centre on the
client who is viewed holistically; thus, care also considers the
social conditions and relationships that affect an individual or a
population's health status. In recent years the community
nurse's role has begun to shift, directing more attention to the
provision of disease recovery nursing care for transitioning
clients as they move out of the hospital environment and into
the community context. Additionally, the community nurse's
role has become more focused on the provision of early
intervention measures to prevent exacerbations or
complications for clients living with chronic illness/conditions to
prevent unnecessary hospital (re)admission.
30 Describe the use of Purpose: To reduce unnecessary utilization of hospital resources,
9. clinical to give the most efficient care possible (because time is of the
pathway/care map essence).
to guide the care Clinical pathways are multidisciplinary plans (or blueprint for a
of the patient with plan of care) of best clinical practice for specified groups of
a CVA. patients with a particular diagnosis that aid in the coordination
and delivery of high quality care.
31 Describe two Anti-platelet drugs. These medications make your platelets, one
0. classifications of of the circulating blood cell types, less likely to stick together.
drugs used to treat When blood vessels are injured, sticky platelets begin to form
a TIA. clots, a process completed by clotting proteins in blood plasma.

Anticoagulants. They affect clotting-system proteins instead of


platelet function.
31 DETACHED RETINA DESCRIPTION:Hole or tear in, or separation of the sensory retina
1. from, the pigmented epithelium.
It can be result of increasing age, severe myopia, eye trauma,
retiopathy (diabetic), cataract or glaucoma surgery, family or
personal hx.
Resealing is done by surgery. Cryotherapy (freezing).
Photocoagulation (laser). Diathermy (heat). Scleral buckling
(most often used).
NURSING PLANS AND INTERVENTIONS: The client may be on
bed rest.
Place eye patch over affected eye.
Administer meds to inhibit accommodation and constriction;
cycloplegics (mydriatics and homatropine) are given to dilate
pupil before surgery.
Administer meds for potop pain: Tylenol, Demerol, oxycodone.
If gas bubble is used, position client so bubble can rise against
area to be reattached.
****S/S***
31 Differential 1. syncope - warning of lightheadedness (eg sudden emotion)
2. Diagnosis for but no other manifestations
Seizures 2. movement disorders - usually ongoing and variable duration;
NOT STEREOTYPED
3. Sleep disorders - occur in spec phases of sleep
4. TIAs - usually negative sx
5. Migraines - longer duration, often w/ headache
6. Non-epileptic spells (less stereotyped, more variable than
seizures, bilateral movements, asynchronous limb movements;
side to side head turning)
31 Differentiate Multiple Sclerosis (MS) starts in ages 20-50 usually, in females
3. between MS, more than males. It is due to a demylization of the myelin
Myasthenia, and sheaths of neuron cells in the CNS.
ALS.
Symptoms include extreme fatigue, dizziness, muscle
twitching/spasms, numbness, tingling, loss of concentration,
sensory and/or visual and/or speech impairment., depression.

Myasthenia Gravis starts in ages 20-30 usually, and in females


more than males. Autoantibodies from the thymus gland
directed at acetylcholine receptor sites impair transmission of
impulses across the myoneural junction. This reduces the
number of receptor sites. The difference (from MS) is that M.
Gravis does not affect the CNS, but instad the nerve-muscle
communication point of the PNS.

Symptoms include at first diplopia (double vision) and ptosis


(dooping of eyelids), and often are accompanied by facial
muscle weakness, speech and swallowing impairment, and
generalized weakness of the muscles. It is purely a motor
disorder and has no effect on sensation or coordination

Amyotrophic Lateral Sclerosis (ALS) is a fatal disease of known


cause. Death usually occurs as a result of infection, respiratory
failure, or aspiration with an avg. time from onset of 3 years.
There is a loss of motor neurons in the brain and spinal cord,
which decreases function of all smooth and skeletal muscles.
The muscles eventually atrophy.

Symptoms depend on the location of the affected motor


neurons, because spefic neurons activate specific muscle fibers
Chief complaints are fatigue, progressive muscle weakness,
craps, fasciculations (twitching), and incoordination.
31 Differentiate RF occurs bilaterally, Osteoarthritis occurs asymmetrically.
4. between RF and
osteoarthritis in
terms of joint
involvement.
31 Differentiate Decorticate posture: an abnormal posturing in which a person
5. decorticate is stiff with bent arms, clenched fists, and legs held out
posturing; straight. The arms are bent in toward the body and the wrists
decerebrate and fingers are bent and held on the chest. This type of
posturing, and flaccid posturing is a sign of severe damage in the brain
response.
Decerebrate posture: an abnormal body posture that involves
the arms and legs being held straight out, the toes being
pointed downward, and the head and neck being arched
backwards. The muscles are tightened and held rigidly. This
type of posturing usually means there has been severe
damage to the brain.

Flaccid response: quality of lack of tone of muscular or


vascular organ or tissue.
31 Differentiate kinds of Partial seizures: simple partial seizures: uncontrolled jerking
6. seizures by types and movements of a finger hand, foot, leg, or the face (jacksonian
symptoms. march).

Complete partial seizures: repititive non-purposeful actions


(lip-smacking)

Generalized seizures: absence seizures: blank stare, blinking


of the eyes, eyelid fluttering.

Tonic-clonic seizures: sudden onset, most common seizure


31 Disadvantages short half-life
7. may be less effective than other AEDs
expense
31 Disadvantages Complex kinetics - as increase dose, blood level rises
8. gradually until takes off (see graph)
Many drug interactions
31 Disadvantages hepatic enzyme (CP450) inhibitor
9. drug interactions
32 Disadvantages of saturable absorption
0. Gabapentoin short half-life
may be less effective than other AEDs
expense
32 Disadvantages of slow to load (especially in patients already taking
1. Lamotragine valproate)
expensive
not good drug if need to get pt on seizure med ASAP (but
can introduce later)

May induce bad rash


32 Discuss alternative Face the patient and establish eye contact.
2. communication Speak in a normal manner and tone.
methods with an Use short phrases, "yes" and "no" questions, and pause
aphasic patient. between phrases to allow the patient time to understand
what is being
said.
Limit conversation to practical and concrete matters.
Use gestures, pictures, objects, and writing.
As the patient uses and handles an object, say what the
object is. It helps to match the words with the object or
action.
Be consistent in using the same words and gestures
each time you give instructions or ask a question.
Keep extraneous noises and sounds to a minimum. Too
much background noise can distract the patient or make it
difficult to sort out the message being spoken.
Ask them to nod the head or blink their eyes, provide pad
and pencil, magic slate, flash cards, computerized talking
board, and/or pictures boards to help with communication.
32 Discuss assessment Ask family what the patient's baseline is
3. methods used to
identify changes in Assess LOC, and orientation, if pt is unconcious use the
patient neurological Glascow Coma Scale
status?
Assess for numbness and tingling in extremeties

Determine if pt has difficulty with sensory functions

Assess strength of hands grip and movement of extremities

Assess pupils using PEERLA

Obtain past medical hx


32 Discuss a teaching Teach patient/parent about:
4. plan for the child with
a head injury. Dizziness, nausea vomiting, when to call HCP

Visual disturbances; blurring, pupils

Headaches

LOC - Keep patient oriented, check pt at least every hour

Avoiding contact sports


32 Discuss at risk ...
5. population for PDD,
Autism and Down
syndrome.
32 Discuss common MVAs, falls, violent assaults, sports injuries, IEDs at war.
6. causes of a head the cause is what influences the kind of head injury they
injury. have.
32 Discuss common drugs Cholinesterase inhibitor drugs, such as Cognex (40-80mh
7. used in the treatment 4x/day, admin 1hr before or 2hr after meal), Aricept (5-
of Alzheimer's disease. 10mg/day bedtime), and Exelon (1.5-6mg b.i.d), Reminyl
(4-12mg b.i.d), and Namada (5-10mg b.i.d) block the
breakdown of acetylcholine. Slows cognitive decline.
Monitor ALT levels with cognex, elevated levels may
indicate hepatoxicty Adverse Reactions: N/V/D, HA,
confusion, upset stomach.

SSRIs such as Prozac treat depression. Risperdal or


Seroquel is used to control behavioral symptoms.
32 Discuss community ...
8. resources for home
health care, meals,
equipment, respite
care, social services,
professional or lay
support, and shelters.
32 Discuss components of Ginko Biloba seems to improve memory. Antioxidants such
9. a nutritional plan for a as Vit-C, Vit-E, and coenzyme 10 may slow progression.
patient with Huperzine A, a traditional Chinese medicine, acts as an
Alzheimer's disease. acetylcholinesterase inhibitor, encourage fluids and fiber.
33 Discuss conditions that Head Injury
0. result in increased ICP. Hematoma
CVA
Tumors
Infections
33 Discuss drugs used to treat Sodium Luminal (Phenobarbital)
1. seizures by: name, action, Diphenylhdantin (Dilantin)
adverse reactions and special Mephenytoin (Mesantoin)
precautions. Valproic Acid (Depakene)
Carbamazine (Tegretol)
33 Discuss educational challenges in ...
2. the care of the MR.
33 Discuss factors that may ...
3. contribute to brain tumor
formation.
33 Discuss functional abilities c1-c3 = no movement or sensation below the
4. related to area of spinal cord neck; ventilator-dependent
injury.
c4 = movement and sensation of head and
neck; some partial function of the diaphragm

c5= controls head, neck, and shoulders; flexes


elbow

c6 = uses shoulder, extends wrist

c7-c8 = extends elbow, flexes wrist, some use


of fingers

T1-T5 = has full hand and finger control, full


use of thoracic muscles

T6-T10 = controls abdominal muscles, has


good balance

T11-L5 = flexes and abducts the hips; flexes


and extends the knee

S1-S5 = full control of legs; progressive bowel,


bladder, and sexual function
33 Discuss Glasgow Coma scale. The Glascow Coma Scale provides a quick
5. guide for assesing LOC, It measures how well
the pt responds to eye opening and verbal and
motor responses.
33 Discuss health promotion ...
6. techniques and teaching to be
done for a patient and family with
a seizure disorder.
33 Discuss health promotion ...
7. techniques for a patient taking
anticonvulsants.
33 Discuss local and national ...
8. resources available to patients
with Alzheimer's disease.
33 Discuss Exaggerated unopposed autonomic response to noxious stimuli
9. manifestations of for individuals with SCI at or above T6 (as low as T8). Nursing
autonomic Interventions: bowel/skin care regimen, flushing catheter daily,
dysreflexia and monitor for distention, I&O, monitor VS for indicators of AD
nursing care to such as hypertension, pounding HA, bradycardia, blurred
prevent or relieve vision, nausea, nasal congestion, flushing and sweating above
symptoms. the level of injury. If AD is suspected, raise head 90 degrees to
lower BP. Monitor BP q3-5min during hypertensive episode.
Assess for the cause, implement measures for removing the
noxious stimulus. Could be: Bladder distension, bowel
constipation/impaction, skin problems (pressure, infection,
injury, heat, pain, cold).
34 Discuss nursing ROM exercises to prevent contractures, use special appliances
0. care for the child to help the child perform ADLs, provide protective head gear
with CP. and bed pads to prevent injury, provide a high-calorie diet
because the child will have a high metabolism rate due to high
motor function, explain the disorder and treatment to the
family and that efforts should be made to ensure that the child
reaches the optimal developmental level possible.
34 Discuss nursing Frequent assesment of vital signs
1. care for the child
with ICP. Careful assement of neurological status

Maintaining patent airway

Maintaing fluid and electrolyte balance

Assesing for s/s of bleeding

Parental education and support

Elevate HOB 30 degrees, keep head still

Use logrolling

Nutrition

ROM - Mobility

Avoid vaso Vagus stimulation

Monitor lab values

Assess for s/s of infection


34 Discuss nursing Preventing injury
2. care of patients
with neurological Monitor for decreased cerebral tissue perfusion.
infection or
inflammatory Preventing increased temperature
disorder.
Reducing headache

Decreases enviornmental stimulation


34 Discuss nursing ...
3. care of the child
with MR.
34 Discuss nursing care of Assess ability to swallow, chew, and taste
4. the patient with
neuromuscular disorder. Assess weight daily

Assess bowel sounds

Assess/monitor changes in vital signs

Assess respiratory rate, character, and use of accessory


muscle

Administer oxygen as ordered

Administer medications as ordered

Teach patient about disease process


34 Discuss nursing care to encourage the client to use the unaffected arm, teach
5. promote independence family/client to put clothing on the affected extremity first
with ADL's. and then dress the unaffected extremity, consult with
occupational therapist to teach the client how to use
assistive devices for eating, hygiene, and dressing.
34 Discuss nursing ...
6. implications regarding
lab values for the
patient prescribed
anticonvulsants.
34 Discuss nursing Loosening of clothing around the neck
7. interventions for a
patient having a seizure. Turn client to side

Suction at bedside

O2 as ordered

Record symptoms during seizure

Pad side rails

Bed in low position

Fall pads on floor


34 Discuss nursing Full Vital Signs
8. responsibilities of
neurological vital signs. Assess LOC, and orientation, if pt is unconcious use the
Glascow Coma Scale

Assess strength of hands grip and movement of


extremities

Assess pupils using PEERLA


34 Discuss prenatal, ...
9. perinatal, and postnatal
causes of MR.
35 Discuss prenatal, May be caused prenatally by the mother contracting
0. perinatal, postnatal, rubella or other infection, malnutrition, abnormal
causes of CP. attachment of the placenta, toxemia, radiation, or
medication.
Perinatally, it may be caused by a difficult birth,
prolapsed umbilical cord, or multiple births.
Postnatally, an infant might develop it as a result of
trauma and result in prolonged anorexia or decreased
circulation to the brain.
35 Discuss safety measures ...
1. for a patient in status
epilepticus.
35 Discuss techniques ...
2. used to evaluate the
credibility and
usefulness of health
related information.
35 Discuss the adaptions ...
3. the nurse makes to
provide care in the
home environment.
35 Discuss the age A child advocacy team or child protective services should be
4. related (pediatric) contacted if child abuse is suspected, the mechanism of
nursing care and injury is unknown or unexplained, or the history is
interdisciplinary care inconsistent.
for a head injury.
NB shock is rarely due to isolated head injury except in young
children and in patients with medullary injuries or large scalp
lacerations.

Pediatric head injury has unique issues that make patient


management and outcome different from that of adult head
injury. Age related aspects will determine a greater or lesser
degree of craniocervical junction injuries (disproportionate
cranial size to trunk in infancy and early childhood). Other
factors are potential underlying congenital anomalies,
physiological factors (cerebrovascular reactivity and blood
flow), differing support systems needed from that of adults
for neuro imaging and specialized medical, nursing and allied
health care support. Pediatric rehabilitation and educational
needs and goals are different to that of adult head injury.

intubating a child is harder than intubating and adult

The physical exam is frequently normal

CT scan = significant radiation exposure

children sometimes cannot talk but frequently vomit due to


stress (instead of head injury)

Brain is less myelinated, results in greater sensitivity to


shearing forces

Cranial bones thinner, resulting in greater transmission of a


single force to brain

Non-fused sutures makes skull easily deformable

Children (particularly < 24 months old) are at increased


risk of cerebral hypo-perfusion after TBI
35 Discuss the common atherosclerosis of large cerebral arteries (thrombotic) a-fib,
5. causes of a CVA. CHF, endocarditis, rheumatic heart disease, mitral valve
disease (embolic) HTN (hemorrhagic)
risk factors: male, over 65 years of age, african american,
hypertension, DM, obesity, a-fib, atherosclerosis, smoking,
high cholesterol diet, excessive use of alcohol,
cocaine/heroin, oral contraceptives.
Another common cause of intracerebral hemorrhage in the
elderly is cerebral amyloid angiopathy, which involves
damage caused by the deposit of beta-amyloid protein in the
small and medium-sized blood vessels of the brain
35 Discuss the common Sinemet: Dopaminergics - carbidoma-levodopa mixture.
6. drugs, side effects, and Levodopa is converted to doapaimine in the brain and
precautions when carbidopa prevents levodopa from being destroyed.
administering drugs Comtan is used in adjunct to Sinemet sometimes.
prescribed for extra- Enhances
pyramidal disorders.
Tasmar: last resort

Dry mouth/difficulty swallowing, anorexia, nausea,


diskinesia vomiting, abdm pain and constipation,
increased hand tremor, headache and dizziness.

Caution should be used in combination with opioids,


antacids, anticonvulsantsm and tricyclic
antidepressants.

Choreiform movements and dystonic movements are


the most adverse reaction to levodopa.
35 Discuss the community Muscular Dystrophy Association (MDA)
7. resources available to the
patient with a Outpatient Therapy
neuromuscular disorder.
Support Groups
35 Discuss the differences ...
8. among home care
agencies.
35 Discuss the discharge- A thorough review of the dosage regimen, possible
9. teaching plan for adverse drug reactions, and early signs of bleeding
anticoagulants (and anti- tendencies help the patient cooperate with the
platelets). prescribed therapy. Teach:
-Follow the dosage schedule prescribed by the PHCP,
and report any signs of active bleeding immediately.
(gums bleeding, bruising, bloody stools, black and tarry
stools, vomit that is bright red or looks like coffee
grounds). If these are found, d/c the next dose and
contact your PHCP immediately.
-The INR will be monitored periodically. Keep all
appointments, because dosage changes may be
necessary.
-Do not take or stop taking other drugs except on the
advice of the PHCP.
-Inform your dentist and other PHCP of therapy.
-Take the drug at the same time each day.
-Do not change brands of anticoagulants without
consulting a physician or pharmacist.
-Avoid alcohol unless use has been approved by the
primary health care provider.
-Be aware of foods high in vit-K, such as leafy green
vegetables, beans, broccoli, cabbage, cauliflower,
cheese, fish, and yogurt. Maintaining a healthy diet
including these foods may help maintain a consistent
INR value.
-Keep in mind that anti-platelet drugs can lower all
blood counts, including the WBC count. Patients may be
at greater risk for infection during the first 3 months of
treatment.
-Use a soft toothbrush.
-Use an electric razor when possible.
-Wear or carry medical identification.
36 Discuss the etiology of This is a disorder that involves a sudden episode of
0. seizures. abnormal, uncontrolled dis- charge of the electrical
activity of the neurons within the brain. The patient
may experience a variety of symptoms depending on
the type of seizure and the cause.
36 Discuss the importance of ...
1. early intervention for the
child with PDD, Autism
and Down syndrome.
36 Discuss the local and psychological support, respite care, meals on wheels,
2. national community sources for special adaptive equipment, support
resources available for groups, social services
patients for home care.
36 Discuss the local and ...
3. national community
resources available for
patients with seizure
disorders.
36 Discuss the nursing drugs - osmotic/loop diuretics, elevate HOB 30 degrees,
4. care for a patient midline position, o2 as ordered, avoid hip flexion and
experiencing abdominal distention (stool softeners as ordered), monitor
increased temp q2hrs for hyperthermia (no rectal temps), reduce
intracranial presure. stimulation of environment, turn client gently, limit fluid over
24hr period.
barbiturates is used to induce coma, reduces (glucose)
metabolism to decrease continued damage to the brain
surgical interventions include burr holes (to evacuate
hematoma or remove blood clot), craniotomy (relieves
pressure of brain tumor), and a brain flap may be removed (to
allow room for the brain to expand). post-op care is important,
especially relating to IICP and respiratory function.

For Head Injuries: tetanus immunization status should be


checked and updated, especially when lacerations or
contaminated wounds are present. Anticonvulsants may be
needed to control or provide prophylaxis for seizure activity.
Nonsteroidal anti-inflammatory drugs (NSAIDs) may be used
for minor pain control. Beta-blockers can be prescribed for
patients with trauma-induced migraines.

hypotension is a indicative of morbidity


36 Discuss the nursing Nursing care focuses on assisting the client and caregiver to
5. care for patients maintain the highest quality of life.
with Alzheimer's Diags: Disturbed Thought Process, Self-Care Deficits, Caregiver
disease. Role Strain
36 Discuss the nursing Prevent IICP, and avoid the complications of IICP (ie;
6. care for the patient ineffective breathing patterns, cerebral edema, IICP, coma,
with increased ICP. brain herniation)
36 Discuss the nursing ...
7. care of an individual
with a seizure
disorder.
36 Discuss the nursing ...
8. care of the newborn
and child with Down
syndrome.
36 Discuss the nursing osmotic diuretics (Mannitol) expel large amount of h2o and
9. implications for electrolytes - may have to switch to loop diuretic.
medications ordered Corticosteroids reduce inflammation. Zantac, Protonix, or
for a patients with a antacids are given to prevent GI irritation. Antemetics are used
head injury. to prevent vomiting. Anticonvulsants (Dilantin, Valium,
phenobarbital). Barbituates are given to induce coma, last
resort, reduces metabolism and slows brain death.

Nursing Implications:
37 Discuss the nursing Corticosteroids: reduces damage and improves functional
0. implications for recovery by protecting the neuromembrane from further
medications ordered destruction. Monitor for increased infection rate,
for patients with a hyperglycemia, GI bleeding.
spinal cord injury.
May also use osmotic diuretics, analgesics, antacids,
anticoagulants, stool softeners, vasopressors. Histamine H2-
receptor antagonists (ranitidine) are used to prevent stress-
related gastric ulcers.

Antispasmodics: baclofen, diazepam, dantrolene. they are


used to control muscle spasm and pain associated with acute
or chronic musculoskeletal conditions. they are not always
effective in controlling spasticity resulting from cerebral or
spinal cord conditions. assess the client's spasticity and
involuntary movements. give with food to decrease GI
symptoms. monitor for drowsiness and dizziness.
37 Discuss the nursing Monitor neuro status for changes, monitor respiratory status
1. interventions are for for changes, encourage self-care, allow patient extra time,
the patient with encourage exercise; assist with passive ROM if necessary,
Parkinson's disease. weigh patient; I&O; explain importance of following med
schedule as well as effects of medication wearing off; reduce
falls at home.
37 Discuss the nursing I&O, q2hr offer bedpan or urinal, maintain skin integrity in the
2. measures in a perineal area, promote daily intake of 2L, but limit intake at
bowel/bladder night, high-fiber diet, offer the bedpan/urinal at the same
training program. times each day, stool softeners as ordered, increase physical
mobility as tolerated (increases peristalsis)
37 Discuss the Precautions: in pts with fever, heart failure, diarrhea,
3. precautions to teach diabetes, malignancy, HTN, renal/hepatic disease,
when instructing a psychoses, depression, or spinal procedures.
patient on
anticoagulants. Interactions: aspirin, acetaminophen, NSAIDS, penicillin,
aminoglycosides, tetracyclines, cephalosporins, beta
blockers, loop diuretics, oral contraceptives, vitamin-K,
barbiturates

Contraindicated: hemorrhagic diseases, TB, leukemia,


uncontrolled HTN, GI ulcers, recent surgery of eye or CNS,
aneurysm. Use during pregnancy can cause fetal death. May
be contraindicated with a hypersensitivity to pork products.
37 Discuss the primary akinesia (inability to initiate movement) and akathisia
4. characteristics of (inability to remain motionless), dystonia. relating to the
extra-pyramidal part of the nervous system that affects body posture and
disorders. promotes smooth and uninterrupted movement of various
muscle groups.
37 Discuss the teaching Actions of medication
5. plan for the patient
with an extra- Continued support and counseling
pyramidal disorder.
37 Discuss the use of anti Anti platelet drugs prevent thrombus formation in the
6. platelet drugs. arterial system (as opposed to anticoagulants, that prevent
thrombosis in the venous system). they work by decreasing
the platelet's ability to stick together in the blood, thus
forming a clot. Often prescribed prophylactically to pts with
a-fib for risk of embolic strokes, but have no other warning
signs or indicators of future stroke.

Compared with antiplatelet therapy, oral anticoagulation


significantly reduces stroke at an average follow-up of one
to three years, but does not reduce mortality. Applied to all-
comers with atrial fibrillation, aspirin reduces stroke by 20
percent, whereas warfarin (Coumadin) reduces it by 65
percent. But SEVERE Intracranial or extracranial hemorrhage
is more common with anticoagulation and must be weighed
against its therapeutic benefit.
37 Disoriented Not oriented to time, place, or person.
7.

37 Distinguish the left hemisphere lesion: right hemiplegia, right visual field
8. characteristics of right deficits, aphasia both expressive and receptive, agrahia -
and left hemiplegia. difficulty writing, alexia - reading problems, aware of
deficits, impaired intellectual ability, no memory deficits, no
hearing deficits, deficits in the right visual field as reading,
problems and inability to discriminate words and letters,
behavior slow cautious and disorganized, anxious when
attempting new task, depression, sense of guilt, quick anger
and frustration, feeling of worthlessness, worries over the
future

right hemisphere lesion: left hemiplegia, left visual deficits,


disoriented to time place and person, cannot recognize
faces, spatial - perception deficits, neglect of left side,
patient unaware of paralyzed side, loss of depth perception,
impulsive - easily distracted, unaware of neurological
deficits, confabulates, euphoric impaired sense of humor,
constantly smiles, denies illness, poor judgement,
overestimates ability, loss of ability to hear tonal variations
37 Do not abruptly ...
9. discontinue use of the
antiparkinsonism
drugs. Can cause
malignant-like
syndrome.
38 Doppler Ultrasound a study that uses sound for detection of blood flow within
0. the vessels; used to assess intermittent claudication, deep
vein thrombosis, and other blood flow abnormalities
38 Dorsal Spinal Nerves Sensory
1. Function
38 Do you want bleeding YES, yes because it means that there is a good supply of
2. when a wound is being blood and healing is taking place.
debrided?
38 During a course of 12 ANS: A, B, C, E
3. electroconvulsive therapy (ECT) Because of the post-ECT thought
treatments, an anxious client alterations of confusion and memory loss,
diagnosed with major depression the client is anxious, accident prone, and
refuses to bathe or attend group has socially isolated self. Altered sensory
therapy. At this time, which of the perception is related to psychotic
following nursing diagnoses should be thoughts of a sensory nature such as
assigned to this client? (Select all that hallucinations, and because this client is
apply.) diagnosed with major depression, not
schizophrenia, altered sensory perception
A. Anxiety R/T post-ECT confusion and would not be anticipated.
memory loss
B. Risk for injury R/T post-ECT
confusion and memory loss
C. Disturbed thought processes R/T
post-ECT confusion and memory loss
D. Altered sensory perception R/T
post-ECT confusion and memory loss
E. Social isolation R/T post-ECT
confusion and memory loss
38 During a home visit to an elderly client 2, 3, 4.
4. with mild dementia, the client's A set routine and brief exercises help
daughter reports that she has one decrease daytime sleeping. Decreasing
major problem with her mother. She caffeine and fluids and promoting
says, "She sleeps most of the day and relaxation at bedtime promote nighttime
is up most of the night. I can't get a sleeping. A strong sleep medicine for an
decent night's sleep anymore." Which elderly client is contraindicated due to
suggestions should the nurse make to changes in metabolism, increased
the daughter? Select all that apply. adverse effects, and the risk of falls.
1. Ask the client's physician for a Using caffeinated beverages may
strong sleep medicine. 2. Establish a stimulate metabolism but can also have
set routine for rising, hygiene, meals, long-lasting adverse effects and may
short rest periods, and bedtime. prevent sleep at bedtime.
3. Engage the client in simple, brief
exercises or a short walk when she
gets drowsy during the day.
4. Promote relaxation before bedtime
with a warm bath or relaxing music.
5. Have the daughter encourage the
use of caffeinated beverages during
the day to keep her mother awake.
38 during discharge a client with i take ibprofen every morning as soon as i
5. oteoporosis which statement needs get up-dont take it on empty stomach!
more teaching? ulcerogenic drug
38 During recovery from a stroke, a client cranial nerves IX and X.
6. is given nothing by mouth, to help
prevent aspiration. To determine when
the client is ready for a liquid diet, the
nurse assesses the client's swallowing
ability once each shift. This
assessment evaluates:
38 During the course of a busy shift, a Notify the physician of the documentation
7. nurse fails to document that a client's omission.
ventricular drain had an output of 150
ml. Assuming that the drain was no
longer draining cerebrospinal fluid, the
physician removes the drain. When the
nurse arrives for work the next
morning, she learns that the client
became agitated during the night and
his blood pressure became elevated.
What action should the nurse take?
38 During the first 24 hours after 3. Control of blood pressure is critical
8. thrombolytic treatment for an ischemic during the first 24 hours after treatment
stroke, the primary goal is to control because an intracerebral hemorrhage is
the client's: 1. Pulse. 2. Respirations. the major adverse effect of thrombolytic
3. Blood pressure. 4. Temperature. therapy. Vital signs are monitored, and
blood pressure is maintained as identified
by the physician and specific to the
client's ischemic tissue needs and risk of
bleeding from treatment. The other vital
signs are important, but the priority is to
monitor blood pressure.
38 During the morning change-of-shift report Keep blinds open during the daytime
9. at the long-term care facility, the nurse hours.
learns that the patient with dementia has
had sundowning. Which nursing action rational: The most likely cause of
should the nurse take while caring for the sundowning is a disruption in
patient? circadian rhythms and keeping the
patient active and in daylight will help
to reestablish a more normal circadian
pattern. Moving the patient to a
different room might increase
confusion. Taking a nap will interfere
with nighttime sleep. Hourly
orientation will not be helpful in a
patient with memory difficulties.
39 Dysarthria the inability to use speech that is
0. distinct and connected because of a
loss of muscle control after damage to
the peripheral or central nervous
system
39 Dyskenesia Difficulty moving
1.

39 Dysphagia condition in which swallowing is


2. difficult or painful
39 An elderly woman's husband died. When 4.
3. her brother arrives for the funeral, he Delirium is commonly due to a
notices her short-term memory problems medical condition such as a UTI in the
and occasional disorientation. A few elderly. Delirium often involves
weeks later, she calls him to say that her memory problems, disorientation, and
husband just died. She says, "I didn't hallucinations. It develops rather
know he was so sick. Why did he die quickly. There is not enough data to
now?" She also complains of not sleeping, suggest Alzheimer's disease
urinary frequency and burning, and especially given the quick onset of
seeing rats in the kitchen. A home care symptoms. Delayed grieving and
nurse is sent to evaluate her situation and adjusting to being alone are unlikely
finds the woman reclusive and passive, to cause hallucinations.
but pleasant. The nurse calls the woman's
primary care physician to discuss the
client's situation and background, and
give his assessment and
recommendations. The nurse concludes
that the woman:
1. Is experiencing the onset of Alzheimer's
disease.
2. Is having trouble adjusting to living
alone without her husband.
3. Is having delayed grieving related to
her Alzheimer's disease.
4. Is experiencing delirium and a urinary
tract infection.
39 Electroencephalography (EEG) -assesses electrical activity of brain
4. noninvasively; detection of seizures,
behavioural changes, sleep disorders;
electrodes placed on head
-review meds to be taken by pt with
provider
-shampoo hair--no gels, oils, sprays
-pt to be sleep deprived if possible to
increase intracranial stress
39 An elementary teacher who has just been "Most patients with epilepsy are well
5. diagnosed with epilepsy after having a controlled with antiseizure
generalized tonic-clonic seizure tells the medications."
nurse, "I cannot teach anymore, it will be
too upsetting if I have a seizure at work." rational: The nurse should inform the
Which response by the nurse is best? patient that most patients with
seizure disorders are controlled with
medication. The other information
may be necessary if the patient
seizures persist after treatment with
antiseizure medications is
implemented.
39 Emotional Lability excessive emotional reposnisveness
6. characterized by unstable and rapidly
changing emotions
39 encephalitis inflammation of brain and meninges
7.
acyclovir
39 epidural hematoma quick, arterial bleed
8.

39 Epidural or subdural a fiber optic sensor put into epidural space via burr hole.
9. sensor Uses light sensors to measure ICP. Does not penetrate dura.
40 Epilepsia partialis Persistent focal motor seizure activity (i.e. focal motor
0. continua status epilepticus)
-Distal hand and foot muscles are most often affected
-Active or passive movement of limb may exacerbate
activity
40 Epilepsy Brain disorder characterized by enduring predisposition to
1. generate seizures + actual occurrence of at least one
seizure
40 Epilepsy syndromes 1. Symptomatic, localized
2. -Post-stroke epilepsy, mesial temporal sclerosis
2. Symptomatic, generalized
-Lennox-Gastaut syndrome
3. Idiopathic, localized
-Benign childhood epilepsy
4. Idiopathic, generalized
-Childhood absence epilepsy
-Juvenile myoclonic epilepsy
40 Ethosuximide Approved for:
3. (Zarontin) -Absence
Side effects:
-Nausea, sedation, BM suppression, rash
40 Examine health Teaching prevention to avoid head injury is key.
4. promotion techniques
and available
resources for the
patient with a head
injury.
40 Examine safety ...
5. measures for the child
with seizures.
40 Ex: Pathogenesis in - mossy fibers of hippocampus (key to episodic memory)
6. mesial temporal sprout collateral fibers to facilitate episodic memory so
sclerosis hippo is prone to re-organization of processes being
disrupted
- seizures themselves may induce collateral sprouting so
the more you seize, the more you seize
40 Explain interventions place client in upright positon for meals and 30 minutes
7. to prevent patient afterward. tild head slightly forward. do not feed client who
aspiration and assist does not have functioning gag reflex or has altered LOC.
with feeding a patient provide oral care before meals. serve thickened liquids and
with a swallowing pureed or soft food and place foods on unaffected side of
disorder. mouth. limit distractions at meal time. have suction
equipment available during mealtimes.
40 Explain ketogenic diet. ...
8.

40 Explain measures used ...


9. to keep populations
healthy.
41 Explain nursing ...
0. responsibilities
included in the referral
process.
41 Explain the difference While transient ischemic attack (TIA) is often labeled "mini-
1. between a TIA and a stroke," it is more accurately characterized as a "warning
CVA. stroke," a warning you should take very seriously.

TIA is caused by a clot; the only difference between a stroke


and TIA is that with TIA the blockage is transient
(temporary). TIA symptoms occur rapidly and last a
relatively short time. Most TIAs last less than five minutes;
the average is about a minute. Unlike a stroke, when a TIA
is over, there's no permanent injury to the brain.
41 Explain the Medicaid: U.S government sponsored program for low-income
2. differences individuals and families to pay the cost of health care. Medicaid
between Medicare beneficiaries are low income families and individuals. Covers a
and Medicaid wider range than Medicare: hospitalization, x-rays, laboratory
reimbursement. services, midwife services, clinic treatment, pediatrics care,
family planning, nursing services and in-home nursing facilities
for 21+ years, medical and surgical dental care.
In some states Medicaid beneficiaries are required to pay the
provider a small fee (co-payment) of up to $30 per month for
medical services. May require payment of deductibles and co-
pay for certain services provided. Program is run by individual
states so the type of coverage and policies may vary between
states. But generally, patients usually pay no (or very little) part
of costs for covered medical expenses.

Medicare: U.S government sponsored health care program for


people above 65 years of age, people under 65 with certain
disabilities and all people with end stage renal disease. Medicare
beneficiaries are senior citizens over the age of 65, end stage
renal disease, and disabled eligible to receive social security
benefits. Divided in to Part A which covers hospital care, Part B
which covers medical insurance and Part D covers prescription
drugs.
May require payment of deductibles and co-pay for certain
services provided., Medicare reserves the right to refuse to pay
for treatments it deems unnecessary. Small monthly premiums
are required for non-hospital coverage. Federally run so the
program and coverage is uniform throughout the country. Run
by the Health Care Financing Administration.
41 Explain the Helping to determine what kind of stroke it is, and acting
3. emergency care appropriately. If ischemic, determine if pt is candidate for
for a patient thrombolytic therapy. If hemorrhagic stroke, measures to reduce
experiencing a bleeding and IICP should be taken.
CVA.
Ischemic: (If non-thrombolytic therapy is needed)
Interventions during this period include measures to reduce ICP,
such as administering an osmotic diuretic (eg, mannitol),
maintaining the partial pres- sure of carbon dioxide (PaCO2)
within the range of 30 to 35 mm Hg, and positioning to avoid
hypoxia. Other treatment measures include the following:
Elevation of the head of the bed to promote venous drainage
and to lower increased ICP
Possible hemicraniectomy for increased ICP from brain edema
in a very large stroke
Intubation with an endotracheal tube to establish a patent
airway, if necessary
Continuous hemodynamic monitoring (the goals for blood
pressure remain controversial for a patient who has not received
thrombolytic therapy; antihyperten- sive treatment may be
withheld unless the systolic blood pressure exceeds 220 mm Hg
or the diastolic blood pressure exceeds 120 mm Hg)
Neurologic assessment to determine if the stroke is evolving
and if other acute complications are devel- oping; such
complications may include seizures, bleeding from
anticoagulation, or medication-
induced bradycardia, which can result in hypotension and
subsequent decreases in cardiac output and cere- bral perfusion
pressure

During the acute phase, a neurologic flow sheet is main- tained


to provide data about the following important mea- sures of the
patient's clinical status:
Change in level of consciousness or responsiveness as
evidenced by movement, resistance to changes of po- sition,
and response to stimulation; orientation to time, place, and
person
Presence or absence of voluntary or involuntary movements of
the extremities; muscle tone; body pos- ture; and position of the
head
Stiffness or flaccidity of the neck
Eye opening, comparative size of pupils and pupillary
reactions to light, and ocular position
Color of the face and extremities; temperature and
moisture of the skin
Quality and rates of pulse and respiration; arterial
blood gas values as indicated, body temperature, and
arterial pressure
Ability to speak
Volume of fluids ingested or administered; volume of
urine excreted each 24 hours
Presence of bleeding
Maintenance of blood pressure within the desired pa-
rameters
41 Explain the etiology Parkinson's Disease: chronic progressive degenerative
4. of each extra- neurologic disease that alters motor coordination.
pyramidal disorder.
Myasthenia Gravis: chronic autoimmune disorder.

MS: chronic degenerative disease that damages the myelin


sheath aurrounding the axons of the CNS

Huntington's disease: progressive neurologic disease.

ALS: rapidly progressive, fatal neurologic disease.


41 Explain the Frequent vitals allows the LPN to report and changes in the
5. importance of vitals immediately to HCP. It also allows nurse to identify the
frequent neuro vital types of interventions the patient may need.
signs in the early
phase of neurological
injury.
41 Explain the nursing -Monitor labs, monitor VS, monitor for signs of bleeding,
6. care for the patient reduce risk factors such as shaving (electric), etc.
receiving -Patients should be given the NPSA booklet (see guidance and
anticoagulant resources)
therapy. -On discharge, nurses should ensure patients know their drug
dosage and arrange follow-up care
-There is no evidence to suggest grapefruit juice should be
avoided but cranberry juice can affect INR results. Foods rich
in vitamin K can affect INR results if eaten in large quantities
-Almost any drug can interact with oral anticoagulants,
including herbal remedies. Most increase the effect but some
reduce it. The INR should be closely monitored when a new
drug is started or dose altered
-Patients must know to seek medical attention for injuries,
particularly head injuries, due to haemorrhage risk
41 Explain the nursing The priority of care during the initial period is preserving
7. care of the patient functional brain cells and preventing acute complications.
with a CVA. Once the client's condition is stable, problems of physical
mobility, communication, sensory-perceptual deficits, bowel
and urine eliminations, and swallowing present the major
nursing challenges.

Diags: Ineffective Tissue Perfusion: Cerebral, Risk for


Ineffective Airway Clearance, Impaired Physical Mobility,
Impaired Verbal Communication, Disturbed Sensory
Perception, Impaired Urinary Elimination and Constipation,
Impaired Swallowing, Self-Care Deficit
41 Explain the surgical Angioplasty
8. treatment for a TIA. In selected cases, a procedure called carotid angioplasty, or
stenting, is an option. This procedure involves using a
balloon-like device to open a clogged artery and placing a
small wire tube (stent) into the artery to keep it open.

If you have a moderately or severely narrowed neck (carotid)


artery, your doctor may suggest carotid endarterectomy
(end-ahr-tur-EK-tuh-me). This preventive surgery clears
carotid arteries of fatty deposits (atherosclerotic plaques)
before another TIA or stroke can occur. An incision is made to
open the artery, the plaques are removed, and the artery is
closed.

Carotid endarterectomy is often not done until several


months after a TIA, but a large study showed that people
benefit most from the surgery if it is done within 2 weeks of a
TIA. Delaying surgery longer than 2 weeks increases the risk
for stroke, because a person is more likely to have a stroke in
the first few days and weeks after a TIA.

Each person must carefully weigh the benefits and risks of


surgery and compare them with the benefits and risks of
using medicine to reduce the risk of TIA or stroke. The
success of either treatment will depend on the amount of
blockage you have and which medicine you use. Risks of
surgery depend on your age, your health status, the skill and
experience of the surgeon, and the experience of the medical
center where the surgery is done.
41 Explain the teaching ...
9. plan for a patient
taking
anticonvulsants.
42 Explain the use of Bite stick
0. emergency equipment for Suction
patients with a seizure O2
disorder.
42 Explain the use of thrombolytic drugs dissolve blood clots that have
1. thrombolytic drugs in the already formed within the walls of a blood vessel. is
treatment of a CVA. prescribed after an ischemic stroke has occurred, within
3 hours of onset this therapy is given. 0.9mg/kg, 10%
given IV bolus over one minute, the rest given over 60
minutes.
criteria for receiving thrombolytic drugs:
Age 18 years or older
Clinical diagnosis of ischemic stroke
Time of onset of stroke known and is 3 hours or less
Systolic blood pressure <185 mm Hg; diastolic
<110 mm Hg
Not a minor stroke or rapidly resolving stroke
No seizure at onset of stroke
Not taking warfarin (Coumadin)
Prothrombin time <15 seconds or INR <1.7
Not receiving heparin during the past 48 hours with
elevated partial thromboplastin time
Platelet count >100,000/mm3
No prior intracranial hemorrhage, neoplasm,
arteriovenous malformation, or aneurysm
No major surgical procedures within 14 days
No stroke, serious head injury, or intracranial surgery
within 3 months
No gastrointestinal or urinary bleeding within 21 days

assess q 15min for first hour, then every 15-30 minutes


for the next 8 hours, then at least q4hrs.
bleeding and IICP are side effects to monitor for.

draw CBC before starting thrombolytics


patients is critical and cared for in ICU for 48 hours
used as soon as possible after formulation of clot
42 Expressive Aphasia damage to Broca's area can cause this condition in
2. which person cannot talk, though understand speech
42 EYE TRAUMA DESCRIPTION: Injury to the eye sustained as the result
3. of sharp or blunt trauma, chemicals or heat. Permanent
visual impairment can occur. Every eye injury should be
considered an emergency. Protective eye shields in
hazardous work environments and during athletic sports
may prevent injuries.
NURSING PLANS AND INTERVENTIONS:Position the client
according to the type of injury; a sitting position
decreases IOP. Remove conjunctival foreign bodies
unless embedded. *Never attempt to remove a
penetrating or embedded object. Do not apply
pressure.*
Apply cold compresses to eye contusion (black eye).
After chemical injuries, irrigate the eye with copious
amounts of water. Administer eye meds as prescribed.
Explain that an eye patch may be applied to rest the
eye. Reading and watching tv may be restricted for 3-5
days.
Explain that a sudden increase in eye pain should be
reported.
42 Family members would Cranial nerve V
4. like to bring in a birthday
cake for a client with
nerve damage. What
cranial nerve needs to be
functioning so the client
can chew?
42 The family of a client, diagnosed with 2, 4, 5.
5. Alzheimer's disease, wants to keep the Motion and sound detectors, a Medical
client at home. They say that they have Alert bracelet, and door alarms are all
the most difficulty in managing his appropriate interventions for wandering.
wandering. The nurse should instruct Sleep medications do not prevent
the family to do which of the following? wandering before and after the client is
(Select all that apply). asleep and may have negative effects.
1. Ask the physician for a sleeping Having a relative sit with the client is
medication. usually an unrealistic burden.
2. Install motion and sound detectors.
3. Have a relative sit with the client all
night.
4. Have the client wear a Medical Alert
bracelet.
5. Install door alarms and high door
locks.
42 The family of a patient who has had a Correct Answer: 2
6. brain attack (CVA) asks if the patient Rationale: Therapeutic communication
will ever talk again. The nurse should do is needed. It is important to allow hope
which of the following? but be honest by not promising
1. Explain that the patient's speech will progress, since no one knows how much
return to normal with time. the patient will improve. Progress may
2. Explain that it is difficult to know how depend on the extent and the areas
far the patient will progress. affected. The nurse does not know that
3. Tell the family that nurses cannot speech will return in time. It is not
discuss such issues. Tell them to ask the therapeutic to tell the family to ask the
physician. physician, and it does not display a
4. Tell the family what they see today is professional, caring attitude.
all they can expect.
42 Focal (partial) seizure First clinical and EEG changes indicate
7. initial activation of a system of neurons
limited to one part of one cerebral
hemisphere
-Tx: carbamazepine, phenytoin,
oxcarbazepine, topiramate, valproic
acid
42 Following a head injury, an unconscious Allow the family to stay with the patient
8. 32-year-old patient is admitted to the and briefly explain all procedures to
emergency department (ED). The them.
patient's spouse and children stay at
the patient's side and constantly ask rational: The need for information about
about the treatment being given. What the diagnosis and care is very high in
action is best for the nurse to take? family members of acutely ill patients,
and the nurse should allow the family to
observe care and explain the
procedures. A pastor or counseling
service can offer some support, but
research supports information as being
more effective. Asking the family to stay
in the waiting room will increase their
anxiety.
42 Following a thymectomy, a patient with Notify the patient's health care provider.
9. myasthenia gravis receives the usual
dose of pyridostigmine (Mestinon). An rational: The patient's history and
hour later, the patient complains of symptoms indicate a possible
nausea and severe abdominal cramps. cholinergic crisis. The health care
Which action should the nurse take provider should be notified immediately,
first? and it is likely that atropine will be
prescribed. The other actions will be
appropriate if the patient is not
experiencing a cholinergic crisis.
43 For a client who has had a stroke, which Attaching braces or splints to each foot
0. nursing intervention can help prevent and leg
contractures in the lower legs?
43 For a client with a head injury whose 30-degree head elevation.
1. neck has been stabilized, the preferred
bed position is:
43 For a client with suspected increased promote carbon dioxide elimination.
2. intracranial pressure (ICP), the most
appropriate respiratory goal is to:
43 For breakfast on the morning a client is 2. Beverages containing caffeine, such
3. to have an electroencephalogram (EEG), as coffee, tea, and cola drinks, are
the client is served a soft-boiled egg, withheld before an EEG because of the
toast with butter and marmalade, stimulating effects of the caffeine on the
orange juice, and coffee. Which of the brain waves. A meal should not be
following should the nurse do? 1. omitted before an EEG because low
Remove all the food. 2. Remove the blood sugar could alter brain wave
coffee. 3. Remove the toast, butter, and patterns; the client can have the entire
marmalade only. 4. Substitute vegetable meal except for the coffee.
juice for the orange juice.
43 Formula for CPP (cerebral CPP= MAP - ICP
4. perfusion pressure) where MAP is mean arterial pressure and ICP is
intracranial pressure
43 Formula for MAP (mean arterial MAP = [(2 x diastolic) + systolic] / 3
5. pressure)
43 For the client who is 2. Expressive aphasia is a condition in which the
6. experiencing expressive client understands what is heard or written but
aphasia, which nursing cannot say what he or she wants to say. A
intervention is most helpful in communication or picture board helps the client
promoting communication? 1. communicate with others in that the client can
Speaking loudly. 2. Using a point to objects or activities that he or she
picture board. 3. Writing desires.
directions so client can read
them. 4. Speaking in short
sentences.
43 Fracture DESCRIPTION: Any break in the continuity of the
7. bone. Fractures are described by the type and
extent of the break. Fractures are caused by a
direct blow, crushing force, a sudden twisting
motion, or a disease such as cancer or
osteoporosis. Comple fracture breaks across the
entire cross section of the bone. Incomplete
fracture breaks only part of the bone. Closed
fracture there is no break in the skin. Open
fracture has broken bone that protrudes theough
skin or mucous membranes and are much more
prone to infection. *update tetanus toxoid,
prophylactic antibiotics*.
NURSING ASSESSMENT:Signs and symptoms of
fracture include:
Pain, swelling, tenderness.
Deformity, loss of functional ability.
Discoloration, bleeding at the site throguh an
open wound.
Crepitus-crackling sound between two broken
bones.
Fracture is evident on radiograph.
Therapeutic management is based on: reduction
of the fracture. Maintenance of realignment by
immobilization. *Don't dislodge hematoma*.
Restoration of function.
Crutches: there should be 2-3 finger widths
between the axilla and the top of the crutch. A
three-point gait is most common. The client
advances both crutches and the impaired leg at
the same time. The client then swings the
uninvolved leg ahead to the crutches.
Cane: It is placed on the unaffected side. The top
of the cane should be at the level of the greater
trochanter.
Walker: Strength of upper extremity and
unaffected leg is assessed and improved with
exercises, if necessary so that upper body is
strong enough to use walker. Client lifts and
advances the walker and steps forward.
43 Fracture Hesi Hint #1 Questions focus on safety precautions. Improper
8. use of assistive devices can be very risky. When
using a nonwheeled walker, the client should lift
an dmove the walker forward and then take a step
into it. The client should avoid scooting the walker
or shuffling forward into it; these movements take
more energy and provide less stability than does
a single movement.
43 Fracture Hesi Hint #3 The risk for the development of a fat embolism, a
9. syndrome in which fat globules migrate into the
bloodstream and combine with platelets to form
emboli, is greatest in the first 36 hours after a
fracture. It is more common in clients with
multiple fractures, fractures of long bones, and
fractures of the pelvis. The initial symptom of a fat
embolism is confusion due to hypoxemia. Assess
for respiratory distress, restlessness, irritability,
fever, and petechiae. If an embolus is suspected,
notify the physician stat, draw blood gases,
administer O2, and assist with endotracheal
intubation. ***Imobilization and stabilization
of the long bones can help prevent fat
emboli***
44 Fracture Hesi Hint #4 In clients with hip fractures, thromboembolism is
0. the most common complication. Prevention
includes passive ROM exercises, use of ted hose,
elevation of the foot of the bed 25 degrees to
increase venous return, and low-dose
anticoagulation therapy (lovenox IM or xarelto
PO). ***hip fx compare effected to unaffected
side- it will be shorter and externally
rotated*
44 Fracture Hesi Hint #5 Clients with fractures or edema in or casts on the
1. extremities need frequent neurovascular
assessment distal to the injury. Skin, color, temp,
sensation, capillary refill, mobility, pain, and
pulses should be assessed. (CMS)
44 Fracture Hesi Hint #6 Assess the 6 P's of neurovascular functioning: pain,
2. paresthesia, pulse, pallor, paralysis, pressure.
44 Fractures Hesi Hint What type of fracture is more difficult to heal: an
3. #2 extracapsular fracture (below the neck of the femur) or an
intracapsular fracture (in the neck of the femur)? The blood
supply enters the femur below the neck of the femur.
Therefore, an intracapsular fracture heals with greater
difficulty, and there is a greater likelihood that necrosis will
occur because the fracture is cut off from the blood supply.
44 Frontal Lobe Concentration, abstract thought, information
4. storage/memory, motor function, speech motor function,
affect, judgement, personality, inhibitions.
44 Full consciousness Alert; oriented to person, time, place; understands verbal and
5. written words
44 Gabapentin Approved for:
6. (Neurontin) -Focal
Side effects:
-Sedation, ataxia, *weight gain, peripheral edema
44 Gabapentoin First in the newer generation of drugs;
7. Not as great as the others (efficacy) but no DDIs and less S/E;
often use in pt w/ transplants; facing S/E
44 gag reflex abnormal: loss (CN IX, X)
8.

44 General health Exercising regularly


9. promotion and illness
prevention measures
the nurse may
recommend to older
adults include
45 Generalized tonic- First clinical and EEG changes indicate initial involvement of
0. clonic seizure both hemispheres
-Sustained muscular contraction and rhythmic jerking of
muscles
-Upward eye deviation, pupillary dilatation, "epileptic cry",
may have urinary incontinence
-Patient is confused or sleepy afterwards with aching and
stiffness
-Tx: phenytoin, valproic acid, lamotrigine, levetiracetam,
topiramate
45 Glasgow Coma Scale Best response for each:
1. (GCS) categories -Eye (4 points max)
-Verbal (5 points max)
-Motor (6 points max)
45 Glasgow Coma Scale -13-15 minor head trauma
2. (GCS) scoring -9-12 moderate head trauma
-<8 severe head trauma
45 Glaucoma 2 Types: Chronic open-angle glaucoma and Acute closed angle
3. glaucoma.
DESCRIPTION: Condition characterized by increased intraocular
pressure (IOP). Glaucoma involves gradual, painless vision loss
(peripheral lost). Glaucoma may lead to blindness if untreated.
Glaucoma usually occurs bilaterally in those who have a family hx of
the condition. Aqueous fluid is inadequately drained from the eye. It is
generally asymptomatic, especially in early stages. It tends to be dx
during routine visual examinations. It cannot be cured but can be
treated with success pharacologically and surgically.
NURSING ASSESSMENT: Early signs: Decreased accommodation or
ability to focus.
Late signs: Loss of peripheral vision. Seeing halos ar ound lights.
Decreased visual acuity not correctable with glasses. Headache or eye
pain that may be so severe as to cause n/v. **acute closed-angle
glaucoma-surgical emergency**
DX Tests: Tonometer, used to measure IOP. Electronic tonometer, used
to detect drainage of aqueous humor. Gonioscopy, used to obtain a
direct visualization of the lens.
RISK FACTORS: Family Hx of glaucoma. Family Hx of diabetes. Hx of
previous ocular problems. Medication use, glaucoma is a side effect of
many meds eg. antihistamines, anticholinergics. It can also result from
the interaction of meds.
NURSING PLANS AND INTERVENTIONS: Administer eye drops as
prescribed.
Orient client to surroundings. Avoid nonverbal communication that
requires visual acuity.
Develop a teaching plan that includes the following: Careful adherence
to eye-drop regimen can prevent blindness.
Vision already lost cannot be restored.
Eye drops are needed for the rest of life.
Proper eye-drop instillation technique: Wash hands and external eye.
Tilt head back slightly. Instill drop into lower lid, without touching the
lid with the tip of the dropper. Release the lid, and sponage excess fluid
from lid and cheek. Close eye gently, and leave closed 3-5 minutes.
Apply gentle pressure on inner canthus to decrease systemic
absorption.
Safety measures to prevent injuries: Remove throw rugs. Adjust
lighting to meet needs.
Avoid activites that may increase IOP: Emotional upsets. Exertion like
pushing, heavy lifting, shoveling. Coughing severely or excessive
sneezing (get medical attention before upper respiratory infection
worsens.) Wearing constrictive clothing eg tight collar or tie, tight belt
or girdle. Straining at stool and constipation.
45 Glaucoma Glaucoma is often painless and symptom free. It is usually picked up as
4. Hesi Hint #1 part of a regular eye examination.
45 Glaucoma Eye drops are used to cause pupil constrictions *avoid mydriatics*
5. Hesi Hint #2 because movement of the muscles to constrict the pupil also allows
aqueous humor to flow out, thereby decreasing the pressure in the
eye. Pilocarpine is commonly used. Caution client that vision may be
blurred for 1-2 hours after administration of pilocarpine and that
adaptation to dark environments is difficult because of pupillary
constriction (the desired effect of the drug.)
45 Glaucoma There is an increased incidence of glaucoma in older adult population.
6. Hesi Hint #3 Older clients are prone to problems associated with constipation.
Therefore, the nurse should assess these clients for constipation and
postop complications associated with constipation and should
implement a plan of care directed at prevention of and, if necessary,
treatment for constipation. *fiber, fluids, exercise*
45 Glioma 42% of all brain tumors are this type.
7. 77% of malignant tumors are this type.
45 Global Nonfluent aphasia w/ impaired comprehension. Both Broca's and
8. Aphasia Wernicke's areas affected.
45 Gout: s/s, ...
9. meds,
manifests,
dietary
restrictions
46 guillan barre acute infection of cranial and peripheral nerves
0. syndrome
gradual progressive weakness of the upper extremities and facial
muscles
46 A health care provider has ordered 3, 4, 5. The nurse should contact the
1. carbidopa-levodopa (Sinemet) four health care provider before administering
times per day for a client with Sinemet because this medication can
Parkinson's disease. The client states cause further symptoms of depression.
that he wants "to end it all now that Suicide threats in clients with chronic
the Parkinson's disease has illness should be taken seriously. The
progressed." What should the nurse nurse should also determine if the client is
do? Select all that apply. 1. Explain on an MAO inhibitor because concurrent
that the new prescription for Sinemet use with Sinemet can cause a
will treat his depression. 2. Encourage hypertensive crisis. Sinemet is not a
the client to discuss his feelings as treatment for depression. Having the
the Sinemet is being administered. 3. client discuss his feelings is appropriate
Contact the health care provider when the prescription is finalized.
before administering the Sinemet. 4.
Determine if the client is on
antidepressants or monoamine
oxidase (MAO) inhibitors. 5.
Determine if the client is at risk for
suicide.
46 The health care provider is The patient has a history of a recent acute
2. considering the use of sumatriptan myocardial infarction.
(Imitrex) for a patient with migraine
headaches. Which information rational: The triptans cause coronary
obtained by the nurse is most artery vasoconstriction and should be
important to report to the health care avoided in patients with coronary artery
provider? disease. The other information will be
reported to the health care provider, but
none of it is an indication that sumatriptan
would be an inappropriate treatment.
46 The healthcare provider orders 2.5 mg Correct Answer: 0.25
3. IV of morphine sulfate (Morphine) to
be administered to a patient with a
ruptured interverterbral disk. The
nurse has a 1 milliliter (mL) syringe
containing 10 mg of morphine sulfate.
How many milliliters of morphine
sulfate does the nurse need to
withdraw from the syringe?
46 The healthcare provider orders 15 mg Correct Answer: 1.25
4. IV of ketorolac (Toradol) for a patient
who has recently undergone a spinal
fusion. The nurse has a 5 milliliter
(mL) ampule containing 60 mg of
ketorolac. How many milliliters of
ketorolac does the nurse need to
withdraw from the syringe?
46 The health care provider prescribes to call the health care provider if stools
5. clopidogrel (Plavix) for a patient with are tarry.
cerebral atherosclerosis. When
teaching about the new medication, rational: Plavix inhibits platelet function
the nurse will tell the patient and increases the risk for gastrointestinal
______________ bleeding, so patients should be advised to
notify the health care provider about any
signs of bleeding. The medication does
not lower blood pressure, decrease plaque
formation, or dissolve clots.
46 The health care provider prescribes Give magnesium citrate 8 oz now.
6. these interventions for a patient with
possible botulism poisoning. Which rational: Magnesium is contraindicated
one will the nurse question? because it may worsen the neuromuscular
blockade. The other orders are appropriate
for the patient.
46 The health care provider recommends "The carotid endarterectomy involves
7. a carotid endarterectomy for a patient surgical removal of plaque from an artery
with carotid atherosclerosis and a in the neck."
history of transient ischemic attacks
(TIAs). The patient asks the nurse to rational: In a carotid endarterectomy, the
describe the procedure. Which carotid artery is incised and the plaque is
response by the nurse is appropriate? removed. The response beginning, "The
diseased portion of the artery in the brain
is removed" describes an arterial graft
procedure. The answer beginning, "A
catheter with a deflated balloon is
positioned at the narrow area" describes
an angioplasty. The final response
beginning, "A wire is threaded through the
artery" describes the MERCI procedure.
46 Hearing Loss Hesi Hint #1 The ear consists of three parts: the external ear, the
8. middle ear, and the inner ear. Inner ear disorders, or
disorders of the sensory fibers going to the CNS, often
are neurogenic in nature and may not be helped with
a hearing aid. External and middle ear problems,
(conductive) may result from infections, trauma or
wax buildup. These types of disorders are treated
more successfully with hearing aids.
46 Hearing Loss Hesi Hint #2 Questions often focus on communicating with older
9. adults who are hearing impaired. Speak in a low-
pitched voice, slowly and distinclty. Stand in front of
the person, with the light source behind the client.
Use visual aids if available.
47 Hemianopsia Blidness or defective vision in half of the visual field
0. in one or both eyes, usually due to stroke, brain
tumor, or trauma.
47 A home health nurse visits a Advise the client to discard the drug because it may
1. client who's taking have undergone chemical changes or become
pilocarpine, a miotic agent, contaminated.
to treat glaucoma. The
nurse notes that the client's
pilocarpine solution is
cloudy. What should the
nurse do first?
47 Homonymous Hemianopia Loss of vision in half of the visual field on the same
2. side of both eyes
47 A hospitalized 24-year-old Start the ordered PRN oxygen at 6 L/min.
3. patient with a history of
cluster headache awakens rational: Acute treatment for cluster headache is
during the night with a administration of 100% oxygen at 6 to 8 L/min. If the
severe stabbing headache. patient obtains relief with the oxygen, there is no
Which action should the immediate need to notify the health care provider.
nurse take first? Cluster headaches last only 60 to 90 minutes, so oral
pain medications have minimal effect. Hot packs are
helpful for tension headaches but are not as likely to
reduce pain associated with a cluster headache.
47 A hospitalized client is 4. "It may seem like a train station sometimes, but
4. disoriented and believes she this is Hogwarts."; Option 4 is the only response that
is in a train station. Which helps orient the client and treats the client with
response from the nurse is respect.
the most appropriate?

1. "You wouldn't be getting


a bath at the train station."
2. "Let's finish your bath
before the train arrives."
3. "Don't you know where
you are?"
4. "It may seem like a train
station sometimes, but this
is Hogwarts."
47 A hospitalized patient acetaminophen (Tylenol)
5. complains of a moderate
bilateral headache that rational: The patient's symptoms are consistent with
radiates from the base of a tension headache, and initial therapy usually
the skull. Which of these involves a nonopioid analgesic such as
prescribed PRN medications acetaminophen, sometimes combined with a sedative
should the nurse administer or muscle relaxant. Lorazepam may be used in
initially? conjunction with acetaminophen but would not be
appropriate as the initial monotherapy. Morphine
sulfate and butalbital and aspirin would be more
appropriate for a headache that did not respond to a
nonopioid analgesic.
47 A hospitalized patient has Correct Answer: 3
6. become unresponsive. The Rationale: In any unconscious patient, the airway
left side of the body is flaccid. must be protected. Assessment of the current
The attending physician airway and breathing status is of highest priority
believes the patient may have and will continue to be. Blood pressure and output
had a stroke (CVA). What is monitoring as well as ensuring appropriate level of
the nurse's priority care are important interventions, but assessment of
intervention? the patient's ability to maintain an airway is the
1. Move the patient to the most vital.
critical care unit.
2. Assess blood pressure.
3. Assess the airway and
breathing.
4. Observe urinary output.
47 A hospitalized patient with a Correct Answer: 4
7. C7 cord injury begins to yell "I Rationale: Spinal shock is a condition almost half
can't feel my legs anymore." the people with acute spinal injury experience. It is
Which is the most appropriate characterized by a temporary loss of reflex function
action by the nurse? below level of injury, and includes the following
1. Remind the patient of her symptomatology: flaccid paralysis of skeletal
injury and try to comfort her. muscles, loss of sensation below the injury, and
2. Call the healthcare possibly bowel and bladder dysfunction and loss of
provider and get an order for ability to perspire below the injury level. In this
radiologic evaluation. case, the nurse should explain to the patient what
3. Prepare the patient for is happening.
surgery, as her condition is
worsening.
4. Explain to the patient that
this could be a common,
temporary problem.
47 How can someone protect Direct contact with saliva, sputum, or mucous of an
8. themselves from viral infected person. Wash hands well. And mosquitos
meningitis can carry some virus. Protect from bites.
47 How can someone with a C1- Blow stick/mouth stick on wheelchair that allows
9. C4 injury maintain some them to control the chair.
independence?
48 How can you keep your Have airway open, clear harmful objects, assess
0. patient safe during seizure? respirations and then pulse. Do not restrain patient
during seizure. Do not try to open clenched jaw. Do
not move unless it is unsafe not to move. Do not
suction until motor activity stops.
48 How does GBS progress? Begins in lower extremities and ascends bilaterally
1. starting with weakness, then ataxia, then bilateral
paresthesia progressing in paralysis.
48 How do you describe a Blurred, but vision can return to normal.
2. CATARACT vision?
48 How do you describe a Described as a halo or tunnel vision. It never
3. GLAUCOMA vision? returns to normal
48 How do you describe a A blind spot / black spot in the center of the vision
4. MACULAR DEGENERATION field. Peripheral vision is retained.
vision?
48 How do you describe a A curtain in front of or partially over your eye.
5. RETINAL DETACHMENT vision?
48 How is multiple sclerosis With steroids when active. Patient then becomes
6. treated? immunocompromised.
48 how is skeletal traction Skeletal traction is maintained by pins or wires
7. applied? applied to the distal fragment of the fracture.
48 How is the diagnosis of Since both are present with the same symptoms,
8. myasthenia gravis made administer TENSILON to differentiate one from the
versus cholinergic crisis? other. If the symptoms improve, then it is
myasthenia gravis. A lack of medication,
PROSTIGMIN, would then be the problem. So,
increase medication. If the symptoms get worse,
then it is a cholinergic crisis. Too much medication
is the problem with increased Prostigmin. So
provide an antidote which is ATROPIN.
48 How is viral meningitis Not treated with medicine. Rest, plenty of fluids,
9. treated? and medication to reduce fever
49 How long should a patient At least 22 hours a day to aid in wound healing.
0. wear a negative pressure
system?
49 How many cervical spinal Eat breakfast at 8 in the morning = 8 spinal
1. nerves are there? What do nerves
they control? Controls: diaphragm C1-4, chest, wall muscles,
arms and shoulders.
49 How many lumbar and sacral Eat dinner at 5 in the afternoon = 5 lumbar spinal
2. spinal nerves are there? What nerves and 5 sacral spinal nerves.
do they control? Controls: lower body, bowel and bladder
49 How many thoracic spinal Eat lunch at noon = 12 thoracic spinal nerves
3. nerves are there? What do Controls: upper body and gastrointestinal function
they control?
49 How often are hearing aid On a fixed schedule every 6 months
4. batteries changed?
49 How should a wound be Using millimeters or centimeters, measure length
5. correctly measured? from 12 to 6 o'clock and width from 3 to 9 o'clock.
Measure wound depth, distance from surface to
deepest point. Carefully insert sterile 6" cotton
tipped applicator into the deepest portion of
wound and look for tunneling. Don't use subjective
terms like, small, large, et. al.
49 How should someone intervene To avoid contaminating other patients move
6. if someone presents with signs patient to a private room; wearing gloves, mask,
of bacterial meningitis? gown in presence - contact precautions. Prepare to
assist with endotracheal intubation and arrange
seizure precuations. Keep room dark and quiet
49 How should the nurse position Laterally, with knees drawn up to the abdomen
7. a client for a lumbar puncture? and chin touching the chest
49 How should you assist with 1) Obtain informed consent
8. lumbar puncture? 2) Have patient empty bladder
3) Explain that she will be injected with a local
anesthetic and she may feel pressure during the
procedure.
4) Place patient in lateral recumbent position with
knees flexed toward abdomen. Help patient to
remain in that position.
5) When procedure is over, change to dorsal
recumbent and monitor
6) Encourage fluids to reduce headache.
7) You should also label the tubes removed from
the body. Discard 1st and 2nd tubes, 3rd tube
goes to lab because it does not have any
contamination.
49 How should you clean a Pipe cleaner, toothpick to remove wax
9. hearing aid?
50 The husband of a client with 1.
0. Alzheimer's disease that was When compared with other similar medications,
diagnosed 6 years ago donepezil (Aricept) has fewer adverse effects.
approaches the nurse and Donepezil is effective primarily in the early stages
says, "I'm so excited that my of the disease. The drug helps to slow the
wife is starting to use progression of the disease if started in the early
donepezil (Aricept) for her stages. After the client has been diagnosed for 6
illness." The nurse should tell years, improvement to the level seen 6 years ago
the husband: is highly unlikely. Data are not available to support
1. The medication is effective the drug's effectiveness for clients in the terminal
mostly in the early stages of phase of the disease.
the illness.
2. The adverse effects of the
drug are numerous.
3. The client will attain a
functional level of that of 6
years ago.
4. Effectiveness in the terminal
phase of the illness is
scientifically proven.
50 hypothalamus stress, hormones, PNS, SNS,
1.

50 ICP HOB 30-40


2. avoid tredelenburg, hip and neck flexion, valsalva
avoid morphine

MEDS
mannitol!!!!!
anticonvuslants
antipyretics
muscle relaxants (dantrium, demerol, thorazine to stop
shivering)
BP meds
corticosteroids
IV fluids

surgery: shunt
50 Identify common The most frequently used anti-platelet medication is aspirin.
3. drugs in the Aspirin is also the least expensive treatment with the fewest
treatment of TIA. potential side effects.
An alternative to aspirin is the anti-platelet drug clopidogrel
(Plavix).
Also maybe Aggrenox, a combination of low-dose aspirin and
the anti-platelet drug dipyridamole, to reduce blood clotting.
The way dipyridamole works is slightly different from aspirin.
Ticlid is used when there is an aspirin allergy, are are used
with aspirin in order to avoid clots from forming on coronary
stents.
Persantine is also an antiplatelet used.

Anticoagulant drugs include heparin and warfarin (Coumadin).


Heparin is used short term and warfarin over a longer term.
These drugs require careful monitoring. If atrial fibrillation is
present, may prescribe another type of anticoagulant,
dabigatran (Pradaxa). Think +10 to differentiate PTT from PT.
(C+O+U+M+A+D+I+N + 2. PT.) Vitamin K is the antidote for
Coumadin, and Protamine is the antidote for heparin. Heparin
should only be used parenterally.
Lovenox is used instead of Heparin during pregnancy (does
not pass the placenta) and is more long term than Heparin,
although not as long-term as Coumadin unless in a LTC. It is a
type of Heparin.
All are used prophylactically for DVTs, PEs.
50 Identify community physical therapy will demonstrate assistive devices, social
4. care and resources services can arrange referral to home health agency, transfer
to assist patient and to rehab center, or job retraining program.
family with chronic
and long-term
disabilities.
50 Identify pain-relief Warm, moist heat (compresses, baths, showers); diversionary
5. interventions for activities (imaging, distraction, self-hypnosis, biofeedback);
clients with arthritis. and meds.
50 Identify specific ABC+R - Avonex, Betaseron, Copaxone. All for relapsing
6. medications and and/or reducing lesions.
usage for various
sign/symptoms of
MS.
50 Identify support Teach about psychological support, respite care, community
7. systems available to resources such as home health agency, meals on wheels,
patient and family. elder care, sources for special adaptive equipment, support
groups and stroke clubs.
50 Identify terms describing level of Full: alert, oriented to time place, and
8. consciousness (LOC). person, pt fully understands written and
spoken words.

Confusion: unable to think rapidly and


clearly; easily bewildered (confused)
with short attention span and poor
memory.

Disorientation: disorientened to time,


place, and person.

Obtundation: appears drowsy and


lethargic; responds to verbal and tactile
stimuli but quickly drifts back to sleep.

Stupor: generally unresponsive; may


withdraw purposefully with vigorous or
painful stimuli.

Coma: unarousable, does not stir or


moan in response to stimuli
50 Identify the categories of drugs NSAIDs, of which salicylates are the
9. commonly used to threat arthritis. cornerstone of treatment, and
corticosteroids (used when arthritic
symptoms are severe).
51 Identify two types of hearing loss. Conductive (transmission of sound to
0. inner ear is blocked) and sensorineural
(damage to 8th cranial nerve).
51 IDIOPATHIC LOCALIZATION RELATED ...
1. EPILEPSY
51 If a client experienced a stroke that body temperature control.
2. damaged the hypothalamus, the nurse
would anticipate that the client has
problems with:
51 If a patient with CVA is getting dressed Dress the weak side first
3. which side should they address first?
51 If someone with GBS has voiding Get a vent in place because diaphragm
4. problems, what should you be is going to be affected next.
concerned about?
51 If sound lateralizes to one ear when C. sound is heard LOUDER in one ear
5. performing the Weber test, which of the
following occurring?

A. sound is heard LONGER in one ear


than the other.

B. higher FREQUENCIES of sound are


heard better in one ear

C. sound is heard LOUDER in one ear

D. electrical impulses are amplified in


one ear
51 If the negative pressure system is Count the sponges as you remove
6. already in use, what should you do
when changing dressings?
51 If the unit alarm on a negative pressure Check the patients skin folds and drape
7. system indicates a lack of seal what around the tubing assembly or curved
should you do? body areas.
51 If the wound using a negative pressure Consider requesting a pressure
8. system appears dry what should you reduction. If very wet, consider increase
do? in pressure.
51 Immediately after an initial ANS: A
9. electroconvulsive therapy (ECT) Immediately after electroconvulsive
treatment a client states, "I'm not therapy a nurse should monitor pulse,
hungry and just want to stay in bed respirations, and blood pressure every 15
and sleep." Based on this information, minutes for the first hour, during which
which is the most appropriate nursing time the client should remain in bed.
intervention?

A. Allow the client to remain in bed.


B. Encourage the client to join the
milieu to promote socialization.
C. Obtain a physician's order for
parenteral nutrition.
D. Involve the client in physical
activities to stimulate circulation.
52 Immediately after electroconvulsive ANS: A
0. therapy, in which position should a The nurse should place a client who has
nurse place the client? received electroconvulsive therapy on his
or her side to prevent aspiration. After the
A. On his or her side to prevent treatment, most clients will awaken within
aspiration 10 to 15 minutes and will be confused and
B. In semi-Fowler's position to disoriented. Some clients will sleep for 1 to
promote oxygenation 2 hours. All clients require close
C. In Trendelenburg's position to observation following treatment.
promote blood flow to vital organs
D. In prone position to prevent airway
blockage
52 In a client with amyotrophic lateral Increased restlessness
1. sclerosis and respiratory distress,
which finding is the earliest sign of
reduced oxygenation?
52 In addition to developing over a 1.
2. period of hours or days, the nurse Fluctuating symptoms are characteristic of
should assess delirium as delirium. The failure to identify objects
distinguishable by which of the despite intact sensory functions,
following characteristics? significant impairment in social or
1. Disturbances in cognition and occupational functioning over time, and
consciousness that fluctuate during memory impairment to the degree
the day.
2. The failure to identify objects
despite intact sensory functions.
3. Significant impairment in social or
occupational functioning over time.
4. Memory impairment to the degree
of being called amnesia.
52 An industrial nurse is conducting a Correct Answer: 2,3,1,4
3. class to teach methods to prevent Rationale: In teaching prevention of back
back pain. What is the correct of steps injuries the nurse would incorporate
for lifting heavy objects? principles of proper body mechanics,
Choice 1. Spread the feet apart to which are work as close to the object as
broaden the base of support. possible, spread feet apart, use large leg
Choice 2. Use large leg muscles to muscles for leverage. Sometimes rolling or
push when lifting. pushing will enable movement of a heavy
Choice 3. Stand as closely as possible object.
to the object to be moved.
Choice 4. Rolling or pushing the obect
insrtead of lifting.
52 Infantile spasms Sudden, rapid flexion of neck and truck,
4. adduction of shoulders and outstretched
arms, variable flexion of lower extremities
-Tx: vigabatrin
52 In planning care for the client who has 1. The primary reason for the nursing
5. had a stroke, the nurse should obtain assessment of a client's functional status
a history of the client's functional before and after a stroke is to guide the
status before the stroke because? 1. plan. The assessment does not help to
The rehabilitation plan will be guided predict how far the rehabilitation team can
by it. 2. Functional status before the help the client to recover from the residual
stroke will help predict outcomes. 3. It effects of the stroke, only what plans can
will help the client recognize his help a client who has moved from one
physical limitations. 4. The client can functional level to another. The nursing
be expected to regain much of his assessment of the client's functional
functioning. status is not a motivating factor.
52 In planning for the discharge of a 1, 2, 3, 5, 6.
6. client with a cognitive disorder, it It is important for a caregiver to have support
is important to assess the client's for herself as well as be able to provide
caregiver support system. Which adequate safety, supervision, and medical care
aspects are the most crucial to to the client. The caregiver must also have
assess? Select all that apply. realistic expectations of the client, given his
1. Availability of resources for abilities and limitations. Reminiscing and
caregiver support. engaging the client in games is desirable but
2. Ability to provide the level of not crucial to care.
care and supervision needed by
the client.
3. Willingness to transport the
client to medical and psychiatric
services.
4. Interest in engaging the
cognitively disordered family
member in reminiscence and
games.
5. Willingness to install door
alarms and make other safety
changes.
6. Understanding the client's
abilities and limitations.
52 In responding to visceral stimuli, 3. Awareness of a full stomach; Visceral refers
7. the client would be most likely to to organs that may produce stimuli that make a
experience which of the person aware of them, e.g., a full stomach.
following?

1. Being aware train is coming


because of hearing whistle
2. Being aware of which foot is
forward when walking
3. Awareness of a full stomach
4. Being aware of an unpleasant
smell
52 An internal corneal reflex would A. a bilateral blink
8. be demonstrated by which of the
following? reflect to protect the eyes from damage by
foreign objects
A. a bilateral blink

B. a bilateral pupillary
constriction

C. a bilateral horizontal deviation


of the eye

D. bilateral tearing
52 In the Snellen chart, the B. distance that a person stands away from the
9. NUMERATOR represents the chart

A. distance a "normal" person the denominator is the distance a "normal"


could read a particular line person could read a particular line

B. distance that a person stands


away from the chart

C. % of the # of letters a person


can read on a line

D. # of feet that a person has to


move forward to read a line
53 Intracranial Pressure Monitoring -continuous monitoring of intracranial pressure
0. (ICP) by an invasive transducer
-rarely used; usually for comatose pts; GCS <8
-Nursing Priority: prevent infections from
occuring.
53 intraventricular catheter a fluid filled catheter inserted in to lateral
1. (ventriculostomy) vetricles via burr hole in skull. Cath attached to
CSF drainage system & transducer monitor
53 In which order will the nurse perform 1) Immobilize the patient's head, neck,
2. the following actions when caring for and spine.
a patient with possible C6 spinal cord 2) Administer O2 using a non-rebreather
trauma who is admitted to the mask.
emergency department? 3) Monitor cardiac rhythm and blood
pressure.
4) Infuse normal saline at 150 mL/hr.
5) Transfer the patient to radiology for
spinal computed tomography (CT).

rational: The first action should be to


prevent further injury by stabilizing the
patient's spinal cord. Maintenance of
oxygenation by administration of 100% O2
is the second priority. Because neurogenic
shock is a possible complication,
monitoring of heart rhythm and BP are
indicated, followed by infusing normal
saline for volume replacement. A CT scan
to determine the extent and level of injury
is needed once initial assessment and
stabilization are accomplished.
53 Is it serious? Yes, medical Emergency!! 20% mortality.
3.

53 Is periorbital edema present with NO


4. glaucoma?
53 Is someone with aphasia still YES
5. considered competent?
53 Is someone with Meniere's disease a NO, they can be discharged because they
6. priority disease? won't die from the condition.
53 It is the night before a client is to 4. The client will be asked to hold the head
7. have a computed tomography (CT) very still during the examination, which
scan of the head without contrast. lasts about 30 to 60 minutes. In some
The nurse should tell the client? 1. instances, food and fluids may be withheld
"You must shampoo your hair tonight for 4 to 6 hours before the procedure if a
to remove all oil and dirt." 2. "You contrast medium is used because the
may drink fluids until midnight, but radiopaque substance sometimes causes
after that drink nothing until the scan nausea. There is no special preparation for
is completed." 3. "You will have some a CT scan, so a shampoo the night before
hair shaved to attach the small is not required. The client may drink fluids
electrode to your scalp." 4. "You will until 4 hours before the scan is scheduled.
need to hold your head very still Electrodes are not used for a CT scan, nor
during the examination." is the head shaved.
53 Jacksonian march Primarily motor seizure that begins in one
8. focus but then spreads throughout the
cortex
-Often secondarily generalizes -> loss of
consciousness and possible GTCS
-Most often starts in the hand; face may
be involved early
53 Joint DESCRIPTION: Surgical procedure in which a mechanical device,
9. Replacement designed to act as a joint, is used to replace a diseased joint.
(common for The most commonly replaced joints: Hip; Knee; Shoulder; Finger
RF clients) Accurate fitting is essential.
Client must have healthy bone stock for adequate healing.
Infection is the concern postoperatively.
NURSING ASSESSMENT: Joint pathology: 1.Osteoarthritis
2. Rheumatoid arthritis
3. Fracture
Pain not relieved by medication.
Poor ROM in the affected joint.
NURSING INTERVENTIONS:Provide postoperative care for wound and
joint.
Monitor incision site: assess for bleeding and drainage; assess suture
line for erythema and edema; assess suction drainage apparatus for
proper functioning; assess for signs of infection.
Monitor functioning of extremity: check circulation, sensation, and
movement of extremity disal to replacement; provide proper
alignment of affected extremity. Client will return from the operating
room with alignment for the initial postoperative period; Provide
abductor appliance (hip replacement) or continuous passive motion
(cpm) device if indicated; monitor I&O every shift, including suction
drainage.
Encourage fluid intake of 3L per day. Enourage client to perform self-
care activities at maximal level. Coordinate rehabilitation: work closely
with health care team to increase client's mobility gradually.
Get client out of bed as soon as possible.
Keep client out of bed as much as possible.
Keep abductor pillow in place while client is in bed (hip replacement).
Use elevated toilet seat and chairs with high seats for those who have
had hip or knee replacements (prevents dislocation).
Do not flex hip more than 90 degrees (hip replacement)
Provide discharge planning that includes rehabilitation on an
outpatient basis as prescribed.
54 Joint Orthopedic wounds have a tendency to ooze more than other wounds.
0. replacement A suction drainage device usually accompanies the client to the
Hesi Hint #1 postoperative floor. Check drainage often. *200-400 ml in a 24 hr
time frame is normal*
54 Joint Questions about joint replacement focus on complications. A big
1. Replacement problem after joint replacement is infection.
Hesi Hint #2
54 Joint Fractures of bone predispose the client to anemia, especially if long
2. Replacement bones are involved. Check H&H every 3-4 days to monitor
Hesi Hint #3 erythropoiesis. Iron can be given PO with meals. (watch for
constipation)
54 Joint After hip replacement, instruct the client not to lift the leg upward from
3. Replacement a lying position or to elevate the knee when sitting. This upward
Hesi Hint #4 motion can pop the prosthesis out of the socket.
54 Joint Hazards of Immobility:
4. Replacement Immobile clients are prone to complications: skin integrity problems;
Hesi Hint #5 formation of urinary calculi (client's milk intake may be limited); and
venous thrombosis (client may be on prophylactic anticoagulants).
54 JRA HH#1 Corticosteroids are used in the short term in low doses during
5. exacerbations. Long term use is avoided because of side effects and
their adverse effects on growth.
54 Juevenile DESCRIPTION: Chronic inflammatory disorder of the joint
6. Rheumatoid synovium. Single or multiple joints may be affected. It may also
Arthritis (JRA) or have a systemic presentation. It occurs between ages 2-5 and
Juevenille 9-12.
Idiopathic Arthritis NURSING ASSESSMENT: Joint swelling and stiffness (usually
(JIA) large joints). Painful joints. Generalized symptoms: fever,
malaise and rash. Periods of exacerbations and remissions.
Varying severity: mild and self-limited or severe and disabling.
Lab data: latex fixation test (usually negative) and elevated
ESR. Poorest prognosis: Positive RF and Polyarticular systemic
onset.
NURSING PLANS AND INTERVENTIONS: Plan home program of
prescribed exercise, splinting, and activity.
Assist in identifying adaptations in routine (eg. Velcro fasteners,
frequent rest periods).
Support the maintaining of school schedule and activites
appropriate for age.
Teach about med regimen: combination drugs are used.
Nonsteroidal antiinflammatory drugs: aspirin, Tolmetin sodium,
Ibprofen, Naproxen. Antirheumatic drugs-gold salts.
corticosteroids-prednisone. Cytotoxic drugs-methotrexate.
Teach child and family about side effects and toxic effects of
prescribed drugs.
Inform child and family that the optimum antiinflammatory
effects of drugs may take a month to achieve.
Encourage periodic eye exams for early detection of iridocyclitis
so to prevent vision loss.
encourage family to allow child's independence.
54 Juvenile Absence - 12 to 18 year olds
7. Epilepsy - starts w/ myoclonic jerks
- rx = lifelong valproic acid (lots of potential side effects)
54 Juvenile myoclonic Myoclonic jerks of shoulders and arms that usually occur after
8. epilepsy awakening
-Typical age of onset: 12-18 y.o.
-Requires lifelong Tx w/AEDs (esp. levetiracetam)
54 Ketogenic diet most children w/ difficult seizures
9. - which pts - increases ketones and acids in the bloodstream
- what it is - compliance is tough
- disadvantages
55 Kyphosis, a change Changes in the configuration of the spine that affect the lungs
0. in the and thorax
musculoskeletal
system, leads to:
55 Lamotragine Initially, led to big rash (starting pt on it too quickly) but if start
1. pt on it slowly, less rash; careful of DDIs;
Very effective
55 Lamotrigine Approved for:
2. (Lamictal) -Focal (when converting from another AED)
-Generalized tonic-clonic
-Lenox-Gastaut
Side effects:
-Stevens Johnson syndrome, dizziness, sedation, diplopia,
headache, nausea
55 left brain stroke parallyzed right
3. speech problems
discrimination probs
caution
aware
impaired language, math
55 Left-sided CVA altered intellectual ability, slow cautious behavior, Aphasia -
4. difficulty speaking, understanding speech, numbers, reading.
55 Lennox-Gastaut Manifests between ages 2-8 yrs, triad of:
5. syndrome 1. Mental retardation
2. Diffuse slow spike and wave pattern on EEG
3. Multiple types of generalized seizures
-Pts. commonly have status epilepticus
55 Levetiracetam Approved for:
6. (Keppra) -Focal
-Generalized tonic-clonic
-Juvenile myoclonic epilepsy
Side effects:
-Somnolence, dizziness, anxiety, psychiatric manifestations
55 Levetiracitam Few S/E and no DDIs so good drug esp not sure the type of seizure;
7. drug of choice although can cause behavioral problems (affect, etc)
55 Levodopa Medication that crosses blood brain barrier. Goes into the brain and
8. converts to dopamine.
55 List common the most common general symptoms:
9. signs and Impulsive behavior
symptoms of Loss of memory
head injury. Impaired perception
Personality changes
Loss of taste and smell
Diminished concentration
Hearing and balance disorders
Cognitive fatigue
Concussion
Coma
Epilepsy

open head injury: open wound on head, no nerves receptors so


patient might not even realize the extent of injuries. Most open
head injuries expose the brain to the outside environment,
leaving victims extremely susceptible to infection (meningitis).

closed head injury:


Loss of consciousness
Dilated pupils
Respiratory issues
Convulsions
Headache
Dizziness
Nausea and vomiting
Cerebrospinal fluid leaking from nose or ears
Speech and language problems
Vision issues

scalp injury:

concussion: immediate loss of consciousness for <5min.


drowsiness, confusion, dizziness, HA, blurred or double vision.

contusion: varies with the size and location of injury. initial loss of
consciousness;if LOC remmains altered, client may become
combative. During unconsciousness, lies motionless; has pale,
clammy skin; faint pulse; hypotension; shallow resps; altered
motor responses.

epidural hematoma: brief loss of consciousness followed by a


short period of alterntess. the client rapidly progresses into coma
with decorticate or decerebate posturing, ipsilateral pupil dilation,
and seizures.

subdural hematoma:
acute - rapid deterioration from drowsiness and confusion to
coma, ipsilateral pupil dilation and contralateral hemiparesis
subacute - appear 48 hours - 2 weeks later; alert period followed
by slow progression to coma
chronic - develops within weeks/months after initial injury. slowed
thinking, confusion, drowsiness; may progress to pupil changes
and motor deficits

intracerebral hematoma: decreased LOC; pupil changes and


motor deficits.
56 List normal Warm extremity, brisk capillary refill, free movement, normal
0. findings in a sensation of the affected extremity, and equal pulses.
neurovascular
assessment.
56 List the common signs and Muscle rigidity
1. symptoms of Parkinson's
diseases. Pin rolling

Bradykinesia

Stooped posture, and shuffling gait

Difficulty swallowing
56 List three nursing interventions Passive ROM exercises, elastic stockings, and
2. for the prevention of elevation of foot of bed 25 degrees to increase
thromboembolism in venous return.
immobilized clients with
musculoskeletal problems.
56 List three of the most common Hip, knee, finger
3. joints that are replaced?
56 List three problems associated Venous thrombosis, urinary calculi, skin integrity
4. with immobility. problems.
56 List three symptoms which may Seizures, poor sucking, difficulty feeding.
5. be present in CP.
56 List two community resources ...
6. for individuals and families of
persons with CP.
56 List two community resources ...
7. for individuals and families of
persons with MR.
56 A long-term care patient with assess for factors that might be causing
8. moderate dementia develops discomfort.
increased restlessness and
agitation. The nurse's initial rational: Increased motor activity in a patient
action should be to with dementia is frequently the patient's only
way of responding to factors like pain, so the
nurse's initial action should be to assess the
patient for any precipitating factors.
Administration of sedative drugs may be
indicated, but this should not be done until
assessment for precipitating factors has been
completed and any of these factors have been
addressed. Reorientation is unlikely to be helpful
for the patient with moderate dementia.
Assigning a nursing assistant to stay with the
patient also may be necessary, but any physical
changes that may be causing the agitation
should be addressed first.
56 Lumbar Puncture removal by centesis of fluid from the
9. subarachnoid space of the lumbar region of the
spinal cord for diagnostic or therapeutic
purposes
57 Lumbar puncture -CSF sample withdrawn from spinal canal
0. -
57 Lumbar puncture -CSF sample withdrawn from spinal canal
1.

57 A lumbar puncture (LP) is done Correct Answer: 2


2. on a patient to rule out a spinal Rationale: Froin's syndrome is seen with spinal
cord tumor. The cerebrospinal cord tumors. A lumbar puncture, x-rays, CT
fluid (CSF) is xanthochromic, has scans, MRI, and myelogram are all common tests
increased protein, no cells, and that are used to diagnose a spinal cord tumor.
clots immediately. What Glasgow's syndrome, cord tumor syndrome, and
syndrome do these findings reflex syndrome are not terms associated with
describe? the symptoms of spinal cord tumor described.
1. Glasgow's syndrome
2. Froin's syndrome
3. cord tumor syndrome
4. reflex syndrome
57 Magnetic resonance imaging A) Surgery
3. (MRI) has revealed the presence Surgical removal is the preferred treatment for
of a brain tumor in a patient. The brain tumors.
nurse would recognize the
patient's likely need for which of
the following treatment
modalities?
A) Surgery
B) Chemotherapy
C) Radiation therapy
D) Pharmacologic treatment
57 Managing Epilepsy w/ AEDs 1. Increase dose until no seizures and DON'T
4. (algorithm) stop just because serum drug level is high -
"normal" ranges are population averages, not
nec individuals

2. If seizures not controlled on maximum


tolerated dose, add a 2nd AED
2a. If seizure control achieved: start to taper the
first AED
2b. If patient still having seizures: taper one of
the two AEDs and start a third

3. If seizures not controlled after trying several


AEDs at adequate doses, then
a. re-assess diagnosis
b. consider surgical management
57 Managing status epilepticus a. ABC (airway/breathing/circulation)
5. b. IV lorazepam, 0.1 mg/kg (a benzodiazepine)
c. IV fosphenytoin (pre-cursor of phenytoin) (if
available; otherwise, IV phenytoin)
d. If still seizing: more fosphenytoin
e. If still seizing: add another agent - may need
to start thinking about inducing coma
57 Medullablastoma Most common malignant brain tumor in
6. children. Occur in the cerebellum.
57 medulla - stem hr, respirations, blood vessula diameter,
7. sneeze, swallow, cough, vomit
57 Mesial Temporal Sclerosis - Simple partial or complex partial szs w/ focus
8. in TEMPORAL lobe then bilateral, often among
those w/ history of febrile seizures

Rx = may respond to meds, may need surgery


57 Mesial temporal sclerosis Most common cause of intractable complex
9. partial seizures in adults
-Due to hippocampal neuron atrophy and gliosis
-Tx: temporal lobectomy
58 midbrain - stem motor coordination, visual, auditory
0.

58 Mode of Action - operate on synapses that control electrical


1. activity of the pre or post-synaptic neuron (see
visual - no detail at all!!)

- decrease excitation via targeting of various ion


channels (depends on the drug) eg. Na channel
blockers; GABA enhancers; Glutamate blockers
58 More about pharmacokinetics Complex kinetics - from right below therapeutic
2. and complex kinetics of level then shoots up way beyond) - could make
phenytoin someone v. toxic quickly serum level starts to
rise exponentially after reaching a certain dose
(different dose for different people)
58 MRI a technique that uses magnetic fields and radio
3. waves to produce computer-generated images
that distinguish among different types of soft
tissue; allows us to see structures within the
brain
58 mutliple sclerosis chronic degnerative demyelinization of
4. neurons
ages 20-40
remissions, exacerbations

baclofen, valium for spasms


bethanechol for urinary retention
tergretol for paresthesia
steroids for edema and minimizing
exacerbations
immunoupressives for stabilization of
process
ditropan to decrease bladder spasms
inderal, klonopin for cerebellar ataxia
58 Myoclonic seizures Nonrhythmic, rapid, jerking movements
5. that can be local or widespread
-Some patients may exhibit diffuse,
severe nonepileptic myoclonus after
anoxic brain injury
-Tx: valproic acid (DOC)
58 Neglect Syndrome The result of certain right parietal lobe
6. lesions that leave a patient completely
inattentive to stimuli to her left, including
the left side of her own body.
58 The neurologic unit has identified a Creating a spreadsheet on which nursing
7. 30% occurrence of pressure ulcers in staff should document repositioning of
clients admitted with the diagnosis of clients admitted with a stroke
stroke. Which of the following actions
should be included in the unit's
performance improvement plan?
58 A new medication regimen is ordered 4
8. for a client with Parkinson's disease.
At which time should the nurse make
certain that the medication is taken?
1. At bedtime. 2. All at one time. 3.
Two hours before mealtime. 4. At the
time scheduled.
58 Normal ICP 10-15mmHg
9.

59 NSAIDs AKA Prostoglandin antagonists DRUGS: Aspirin; Motrin; Indomethacin (RF,


0. Gout); Toradol; Naproxen
INDICATIONS: Used as antiinflammatory;
antipyretic; analgesic; Can be used with
other agents, may alternate with
narcotics.
ADVERSE REACTIONS: GI irritation, (slow
GI bleeds shown by H&H); n/v,
constipation. Elevated liver enzymes.
Prolonged coagulation time; Tinnitus;
thrombocytopenia; fluid retention;
Nephrotoxicity; blood dyscrasias.
NURSING IMPLICATIONS:
1.Teach to take with food or milk to reduce
GI symptoms.
2. Therapeutic serum salicylates level 20-
25 mg%
3. Teach to watch for signs of bleeding.
4. Teach to avoid alcohol.
5. Teach to observe for Tinnitus (aspirin
toxicity).
6. Administer corticosteroids for severe
rheumatoid arthritis.
7.NSAIDs reduce the effect of ACE
inhibitors in hypertensive clients.
8. Encourage routine appointments to
check liver/renal labs and CBC.
59 Number of cervical nerves 8
1.

59 Number of lumbar nerves 5


2.

59 Number of thoracic nerves 12


3.

59 A nurse administers ordered ANS: D


4. preoperative glycopyrrolate Glycopyrrolate (Robinul) is the standard
(Robinul) 30 minutes prior to a preoperative medication given prior to ECT
client's electroconvulsive therapy treatments to decrease secretions and prevent
(ECT) treatment. What is the aspiration.
rationale for administering this
medication?

A. Robinul decreases anxiety


during the ECT procedure.
B. Robinul induces an unconscious
state to prevent pain during the
ECT procedure.
C. Robinul prevents severe muscle
contractions during the ECT
procedure.
D. Robinul decreases secretions
to prevent aspiration during the
ECT procedure.
59 A nurse administers pure oxygen ANS: C
5. to a client during and after The nurse administers 100% oxygen during
electroconvulsive therapy and after electroconvulsive therapy to prevent
treatment. What is the nurse's anoxia due to medication-induced paralysis of
rationale for this procedure? respiratory muscles. Electroconvulsive therapy
is the induction of a grand mal seizure through
A. To prevent increased the application of electrical current to the
intracranial pressure resulting brain.
from anoxia
B. To prevent hypotension,
bradycardia, and bradypnea due
to electrical stimulation
C. To prevent anoxia due to
medication-induced paralysis of
respiratory muscles
D. To prevent blocked airway
resulting from seizure activity
59 The nurse advises a woman 3. Assess factors that mainly cause hearing
6. considering pregnancy of the impairments in baby; Women who are
importance of being tested for considering pregnancy should be advised of
syphilis and rubella. What is most the importance of testing for syphilis and
likely the reason the nurse is rubella, which can cause hearing impairments
offering this advice? in newborns.

1. Suspicion that a client has high


risk of sexual and drug behaviors
2. Assess factors that mainly
cause visual impairments in baby
3. Assess factors that mainly
cause hearing impairments in
baby
4. The health history assessment
findings
59 The nurse assesses a patient for D) Resistance of flexion of the neck
7. signs of meningeal irritation and Nuchal rigidity is a clinical manifestation of
observes her for nuchal rigidity. meningitis. During assessment, the patient will
Which of the following indicates resist passive flexion of the neck by the health
the presence of this sign of care provider.
meningeal irritation?
A) Tonic spasms of the legs
B) Curling in a fetal position
C) Arching of the neck and back
D) Resistance to flexion of the
neck
59 The nurse assesses normal pupils Equal
8. in a client who had a craniotomy,
and then writes "PERRLA" in the
nurse's notes, along with other
findings. What does the "E" stand
for in this acronym?
59 The nurse determines that the son of 4.
9. a client with Alzheimer's disease The statement about expecting that the old
needs further education about the Dad would be back conveys a lack of
disease when he makes which of the acceptance of the irreversible nature of the
following statements? disease. The statement about not realizing
1. "I didn't realize the deterioration that the deterioration would be so
would be so incapacitating." incapacitating is based in reality. The
2. "The Alzheimer's support group statement about the Alzheimer's group is
has so much good information." based in reality and demonstrates the son's
3. "I get tired of the same old stories, involvement with managing the disease.
but I know it's important for Dad." Stating that reminiscing is important
4. "I woke up this morning expecting reflects a realistic interpretation on the
that my old Dad would be back." son's part.
60 The nurse develops a teaching plan 3. The primary focus is on maintaining a
0. for a client newly diagnosed with safe environment because the client with
Parkinson's disease. Which of the Parkinson's disease usually has a
following topics that the nurse plans propulsive gait, characterized by a
to discuss is the most important? 1. tendency to take increasingly quicker steps
Maintaining a balanced nutritional while walking. This type of gait commonly
diet. 2. Enhancing the immune causes the client to fall or to have trouble
system. 3. Maintaining a safe stopping. The client should maintain a
environment. 4. Engaging in balanced diet, enhance the immune
diversional activity. system, and enjoy diversional activities;
60 A nurse discovers that a client who is A fat embolism, which is characterized by
1. in traction for a long bone fracture hypoxemia, respiratory distress, irritability,
has a slight fever, is sob and is restlessness, fever, and petechiae.
restless. What does the client most
likely have?
60 The nurse discusses the possibility of 3.
2. a client's attending day treatment for The best rationale for day treatment for the
clients with early Alzheimer's client with Alzheimer's disease is the
disease. Which of the following is the enhancement of social interactions. More
best rationale for encouraging day daily structure, excellent staff, and allowing
treatment? caregivers more time for themselves are all
1. The client would have more positive aspects, but they are less focused
structure to his day. on the client's needs.
2. Staff are excellent in the
treatment they offer clients.
3. The client would benefit from
increased social interaction.
4. The family would have more time
to engage in their daily activities.
60 The nurse expects that management oral administration of low dose aspirin
3. of the patient who experiences a therapy.
brief episode of tinnitus, diplopia,
and dysarthria with no residual rational: The patient's symptoms are
effects will include ______________ consistent with transient ischemic attack
(TIA), and drugs that inhibit platelet
aggregation are prescribed after a TIA to
prevent stroke. Continuous heparin infusion
is not routinely used after TIA or with acute
ischemic stroke. The patient's symptoms
are not consistent with a cerebral
aneurysm. tPA is used only for acute
ischemic stroke, not for TIA.
60 The nurse formulates a nursing Hypothalamus
4. diagnosis of Risk for imbalanced
body temperature for a client who
suffers a stroke after surgery. The
expected outcomes incorporate
assessment of the client's
temperature to detect abnormalities.
The thermoregulatory centers are
located in which part of the brain?
60 The nurse identifies the nursing Assist the patient to eat with the left hand.
5. diagnosis of imbalanced nutrition:
less than body requirements related rational: Because the nursing diagnosis
to impaired self-feeding ability for a indicates that the patient's imbalanced
patient with right-sided hemiplegia. nutrition is related to the right-sided
Which intervention should be hemiplegia, the appropriate interventions
included in the plan of care? will focus on teaching the patient to use the
left hand for self-feeding. The other
interventions are appropriate for patients
with other etiologies for the imbalanced
nutrition.
60 The nurse identifies the nursing ask simple questions that the patient can answer
6. diagnosis of impaired verbal with "yes" or "no."
communication for a patient
with expressive aphasia. An rational: Communication will be facilitated and
appropriate nursing less frustrating to the patient when questions that
intervention to help the patient require a "yes" or "no" response are used. When
communicate is to the language areas of the brain are injured, the
_______________ patient might not be able to read or recite words,
which will frustrate the patient without improving
communication. Expressive aphasia is caused by
damage to the language areas of the brain, not by
the areas that control the motor aspects of
speech. The nurse should allow time for the
patient to respond.
60 The nurse is administering Schedule the medication before meals.
7. neostigmine to a client with
myasthenia gravis. Which
nursing intervention should the
nurse implement?
60 The nurse is admitting a person 1. Disorientation is a normal reaction to sudden
8. who has had a sudden loss of blindness; Sudden loss of eyesight can result in
eyesight. On assessing this disorientation. With gradual loss of sensory
client, the nurse finds that the function, individuals often develop behaviors to
client is disoriented. The nurse compensate for loss, whereas with sudden loss,
will most suspect which of the the compensatory behavior often takes days or
following about the weeks to develop.
disorientation?

1. Disorientation is a normal
reaction to sudden blindness.
2. Compensatory behavior to
eyesight loss includes
disorientation.
3. Client will compensate for
the eyesight loss within 48
hours.
4. Disorientation is a symptom
of the cause of sudden eyesight
loss.
60 The nurse is assessing a client 1. The nurse should expect a client in the postictal
9. in the postictal phase of phase to experience drowsiness to somnolence
generalized tonic-clonic because exhaustion results from the abnormal
seizure. The nurse should spontaneous neuron firing and tonic-clonic motor
determine if the client has? 1. response. An inability to move a muscle part is
Drowsiness. 2. Inability to not expected after a tonic-clonic seizure because
move. 3. Paresthesia. 4. a lack of motor function would be related to a
Hypotension. complication, such as a lesion, tumor, or stroke, in
the correlating brain tissue. A change in sensation
would not be expected because this would
indicate a complication such as an injury to the
peripheral nerve pathway to the corresponding
part from the central nervous system.
Hypotension is not typically a problem after a
seizure.
61 The nurse is assessing a The patient has new onset weakness of both legs.
0. patient who is being evaluated
for a possible spinal cord rational: The new onset of symptoms indicates
tumor. Which finding by the cord compression, an emergency that requires
nurse requires the most rapid treatment to avoid permanent loss of
immediate action? function. The other patient assessments also
indicate a need for nursing action but do not
require intervention as rapidly as the new onset
weakness.
61 The nurse is assessing for sensory 1. Identifying taste c. Visual
1. function. Match the assessment tool 2. Stereognosis e. Tactile
to the specific sense it will be 3. Snellen chart b. Gustatory
testing. 4. Identifying aromas d. Olfactory
5. Tuning fork a. Hearing
1. Identifying taste a. Hearing
2. Stereognosis b. Gustatory
3. Snellen chart c. Visual
4. Identifying aromas d. Olfactory
5. Tuning fork e. Tactile
61 The nurse is attempting to draw 1.
2. blood from a woman with a Explaining why blood is being taken
diagnosis of delirium who was responds to the client's concerns or fears
admitted last evening. The client about what is happening to her. Threatening
yells out, "Stop; leave me alone. more pain or promising to explain later
What are you trying to do to me? ignores or postpones meeting the client's
What's happening to me?" Which need for information. The client's
response by the nurse is most statements do not reflect loss of self control
appropriate? requiring medication intervention.
1. "The tests of your blood will help
us figure out what is happening to
you."
2. "Please hold still so I don't have
to stick you a second time."
3. "After I get your blood, I'll get
some medicine to help you calm
down."
4. "I'll tell you everything after I get
your blood tests to the laboratory."
61 The nurse is caring for a client Call the physician immediately.
3. diagnosed with a cerebral aneurysm,
who reports a severe headache.
Which action should the nurse
perform first?
61 The nurse is caring for a client in a Elevate the head of the bed to 30 degrees.
4. coma who has suffered a closed
head injury. What intervention
should the nurse implement to
prevent increases in intracranial
pressure (ICP)?
61 The nurse is caring for a client who Urine retention or incontinence
5. underwent a lumbar laminectomy 2
days ago. Which finding should the
nurse consider abnormal?
61 The nurse is caring for a client with keep the client in one position to decrease
6. an acute bleeding cerebral bleeding.
aneurysm. The nurse should take all
of the following steps except:
61 The nurse is caring for a client with Establishing an intermittent catheterization
7. L1-L2 paraplegia who is undergoing routine every 4 hours
rehabilitation. Which goal is
appropriate?
61 A nurse is caring for a group of Arrange an escort for a client who needs to
8. clients on the neurologic unit. Which go to the physical therapy department.
task should the nurse perform first?
61 The nurse is caring for an elderly Alzheimer's disease.
9. client who exhibits signs of
dementia. The most common cause
of dementia in an elderly client is:
62 The nurse is caring for a patient A,B,C,E
0. admitted for evaluation and Brain tumors can manifest themselves in a
surgical removal of a brain tumor. wide variety of symptoms depending on
The nurse will plan interventions location, including vision loss and focal
for this patient based on neurologic deficits. Tumors that put pressure on
knowledge that brain tumors can the pituitary can lead to dysfunction of the
lead to which of the following gland. As the tumor grows, clinical
complications (select all that manifestations of increased intracranial
apply)? pressure (ICP) and cerebral edema can appear.
A) Vision loss The parathyroid gland is not regulated by the
B) Cerebral edema cerebral cortex or the pituitary gland.
C) Pituitary dysfunction
D) Parathyroid dysfunction
E) Focal neurologic deficits
62 The nurse is caring for a patient B) Bradycardia
1. admitted with a subdural Changes in vital signs indicative of increased
hematoma following a motor intracranial pressure are known as Cushing's
vehicle accident. Which of the triad, which consists of increasing systolic
following changes in vital signs pressure with a widening pulse pressure,
would the nurse interpret as a bradycardia with a full and bounding pulse, and
manifestation of increased irregular respirations.
intracranial pressure?
A) Tachypnea
B) Bradycardia
C) Hypotension
D) Narrowing pulse pressure
62 The nurse is caring for a patient The patient has difficulty talking.
2. with carotid artery narrowing who
has just returned after having left rational: Small emboli can occur during carotid
carotid artery angioplasty and artery angioplasty and stenting, and the
stenting. Which assessment aphasia indicates a possible stroke during the
information is of most concern to procedure. Slightly elevated pulse rate and
the nurse? blood pressure are not unusual because of
anxiety associated with the procedure. Fine
crackles at the lung bases may indicate
atelectasis caused by immobility during the
procedure; the nurse should have the patient
take some deep breaths.
62 The nurse is caring for a patient Correct Answer: 3
3. with increased intracranial Rationale: Suctioning further increases
pressure (IICP). The nurse realizes intracranial pressure; therefore, suctioning
that some nursing actions are should be done to maintain a patent airway but
contraindicated with IICP. Which not as a matter of routine. Maintaining patient
nursing action should be avoided? comfort by frequent repositioning as well as
1. Reposition the patient every keeping the head elevated 30 degrees will help
two hours. to prevent (or even reduce) IICP. Keeping the
2. Position the patient with the patient properly oxygenated may also help to
head elevated 30 degrees. control ICP.
3. Suction the airway every two
hours per standing orders.
4. Provide continuous oxygen as
ordered.
62 The nurse is collecting data on a Vision changes
4. 38-year-old client diagnosed with
multiple sclerosis. Which of the
following symptoms would the
nurse expect to find?
62 The nurse is collecting data on a Decreased acetylcholine level
5. geriatric client with senile
dementia. Which
neurotransmitter condition is
likely to contribute to this client's
cognitive changes?
62 The nurse is educating a patient and Correct Answer: 2
6. the family about different types of Rationale: A halo device will allow the
stabilization devices. Which patient to be mobile since it does not
statement by the patient indicates require weights like the Gardner-Wells
that the patient understands the tongs. The patient's pain level is not
benefit of using a halo fixation device dependant on the type of stabilization
instead of Gardner-Wells tongs? device used. The patient does not have a
1. "I will have less pain if I use the great risk of infection with the Garnder-
halo device." Wells tongs; both devices require pins to be
2. "The halo device will allow me to inserted into the skull. The time required
get out of bed." for stabilization is not dependant on the
3. "I am less likely to get an infection type of stabilization device used.
with the halo device."
4. "The halo device does not have to
stay in place as long."
62 The nurse is making a home visit with 1.
7. a client diagnosed with Alzheimer's Although all of the side effects listed are
disease. The client recently started possible with Ativan, paradoxical
on lorazepam (Ativan) due to excitement is cause for immediate
increased anxiety. The nurse is discontinuation of the medication.
cautioning the family about the use (Paradoxical excitement is the opposite
of lorazepam (Ativan). The nurse reaction to Ativan than is expected.) The
should instruct the family to report other side effects tend to be minor and
which of the following significant side usually are transient.
effects to the health care provider?
1. Paradoxical excitement.
2. Headache.
3. Slowing of reflexes.
4. Fatigue.
62 The nurse is monitoring a client for Tachycardia
8. adverse reactions to atropine sulfate
(Atropine Care) eyedrops. Systemic
absorption of atropine sulfate
through the conjunctiva can cause
which adverse reaction?
62 The nurse is monitoring a client for Muscle weakness
9. adverse reactions to dantrolene
(Dantrium). Which adverse reaction is
most common?
63 The nurse is monitoring a client for diminished responsiveness.
0. increasing intracranial pressure (ICP).
Early signs of increased ICP include:
63 The nurse is observing a client with 90
1. cerebral edema for evidence of
increasing intracranial pressure. She
monitors his blood pressure for signs
of widening pulse pressure. His
current blood pressure is 170/80 mm
Hg. What is the client's pulse
pressure?
63 The nurse is performing a mental Cerebral function
2. status examination on a client
diagnosed with a subdural
hematoma. This test assesses which
of the following functions?
63 The nurse is planning care for a client Provide close supervision because of the
3. who suffered a stroke in the right client's impulsiveness and poor judgment.
hemisphere of his brain. What should
the nurse do?
63 The nurse is planning the care of a 1, 2, 4. Placing a pillow in the axilla so the
4. hemiplegic client to prevent joint arm is away from the body keeps the arm
deformities of the arm and hand. abducted and prevents skin from touching
Which of the following positions are skin to avoid skin breakdown. Placing a
appropriate? 1. Placing a pillow in the pillow under the slightly flexed arm so the
axilla so the arm is away from the hand is higher than the elbow prevents
body. 2. Inserting a pillow under the dependent edema. Positioning a hand cone
slightly flexed arm so the hand is (not a rolled washcloth) in the hand
higher than the elbow. 3. prevents hand contractures. Immobilization
Immobilizing the extremity in a sling. of the extremity may cause a painful
4. Positioning a hand cone in the shoulder-hand syndrome. Flexion
hand so the fingers are barely flexed. contractures of the hand, wrist, and elbow
5. Keeping the arm at the side using can result from immobility of the weak or
a pillow. paralyzed extremity. It is better to extend
the arms to prevent contractures
63 The nurse is preparing a client for a "Are you allergic to seafood or iodine?"
5. computed tomography (CT) scan, which
requires infusion of radiopaque dye.
Which question is important for the
nurse to ask?
63 The nurse is preparing a client with 2
6. multiple sclerosis (MS) for discharge
from the hospital to home. The nurse
should tell the client: 1. "You will need
to accept the necessity for a quiet and
inactive lifestyle." 2. "Keep active, use
stress reduction strategies, and avoid
fatigue." 3. "Follow good health habits
to change the course of the disease." 4.
"Practice using the mechanical aids
that you will need when future
disabilities arise."
63 The nurse is preparing to administer 7.5
7. carbamazepine (Tegretol) oral
suspension, 150 mg by mouth. The
pharmacy has dispensed
carbamazepine suspension 100 mg/5
ml. How many milliliters of
carbamazepine should the nurse
administer to the client?
63 The nurse is providing care for a B, D, E
8. patient who has been admitted to the The three dimensions of the GCS are eye
hospital with a head injury and who opening, best verbal response, and best
requires regular neurologic vital signs. motor response.
Which of the following assessments will
be components of the patient's score on
the Glasgow Coma Scale (GCS) (select
all that apply)?
A) Judgment
B) Eye opening
C) Abstract reasoning
D) Best verbal response
E) Best motor response
F) Cranial nerve function
63 The nurse is reviewing the care plan of 3
9. a client with Multiple Sclerosis. Which
of the following nursing diagnoses
should receive further validation? 1.
Impaired mobility related to spasticity
and fatigue. 2. Risk for falls related to
muscle weakness and sensory loss. 3.
Risk for seizures related to muscle
tremors and loss of myelin. 4. Impaired
skin integrity related bowel and bladder
incontinence.
64 The nurse is teaching a client about 2, 3, 5. The maximum dosage of warfarin
0. taking prophylactic warfarin sodium sodium (Coumadin) is not achieved until
(Coumadin). Which statement indicates 3 to 4 days after starting the medication,
that the client understands how to take and the effects of the drug continue for 4
the drug? Select all that apply. 1. "The to 5 days after discontinuing the
drug's action peaks in 2 hours." 2. medication. The client should have his
"Maximum dosage is not achieved until blood levels tested periodically to make
3 to 4 days after starting the sure that the desired level is maintained.
medication." 3. "Effects of the drug Warfarin has a peak action of 9 hours.
continue for 4 to 5 days after Vitamin K is the antidote for warfarin;
discontinuing the medication." 4. protamine sulfate is the antidote for
"Protamine sulfate is the antidote for heparin.
warfarin." 5. "I should have my blood
levels tested periodically."
64 The nurse is teaching a client and his Gamma aminobutyric acid (GABA)
1. family about baclofen (Lioresal)
therapy. Baclofen is an analogue of
which neurotransmitter?
64 The nurse is teaching a client to 3. An aura is a premonition of an
2. recognize an aura. The nurse should impending seizure. Auras usually are of a
instruct the client to note: 1. A postictal sensory nature (e.g., an olfactory, visual,
state of amnesia. 2. An hallucination gustatory, or auditory sensation); some
that occurs during a seizure. 3. A may be of a psychic nature. Evaluating
symptom that occurs just before a an aura may help identify the area of the
seizure. 4. A feeling of relaxation as the brain from which the seizure originates.
seizure begins to subside. Auras occur before a seizure, not during
or after (postictal). They are not similar
to hallucinations or amnesia or related to
relaxation.
64 A nurse is teaching a client who had a 4. To expand the visual field, the partially
3. stroke about ways to adapt to a visual sighted client should be taught to turn the
disability. Which does the nurse head from side to side when walking.
identify as the primary safety Neglecting to do so may result in
precaution to use? 1. Wear a patch accidents. This technique helps maximize
over one eye. 2. Place personal items the use of remaining sight. Covering an
on the sighted side. 3. Lie in bed with eye with a patch will limit the field of
the unaffected side toward the door. vision. Personal items can be placed within
4. Turn the head from side to side sight and reach, but most accidents occur
when walking. from tripping over items that cannot be
seen. It may help the client to see the
door, but walking presents the primary
safety hazard.
64 The nurse is teaching a client who has destruction of acetylcholine receptors.
4. facial muscle weakness and has
recently been diagnosed with
myasthenia gravis. The nurse should
teach the client that myasthenia
gravis is caused by:
64 The nurse is teaching a client with a his upper body to the wheelchair first.
5. T4 spinal cord injury and paralysis of
the lower extremities how to transfer
from the bed to a wheelchair. The
nurse should instruct the client to
move:
64 The nurse is teaching a client with 2,3,4,5
6. bladder dysfunction from multiple
sclerosis (MS) about bladder training
at home. Which instructions should
the nurse include in the teaching
plan? Select all that apply. 1. Restrict
fluids to 1,000 mL/ 24 hours. 2. Drink
400 to 500 mL with each meal. 3.
Drink fluids midmorning,
midafternoon, and late afternoon. 4.
Attempt to void at least every 2
hours. 5. Use intermittent
catheterization as needed.
64 The nurse is teaching a client with rest in a room set at a comfortable
7. multiple sclerosis. When teaching the temperature.
client how to reduce fatigue, the
nurse should tell the client to:
64 A nurse is teaching a wellness class Correct Answer: 3
8. and is covering the warning signs of Rationale: Warning signs of stroke include
stroke. A patient asks, "What is the sudden weakness, paralysis, loss of
most important thing for me to speech, confusion, dizziness,
remember?" Which is an appropriate unsteadiness, and loss of balance the key
response by the nurse? word is sudden. Family history and past
1. "Know your family history." medical history can be indicators for risk,
2. "Keep a list of your medications." but they are not warning signs of stroke.
3. "Be alert for sudden weakness or Gradual onset of symptoms is not
numbness." indicative of a stroke.
4. "Call 911 if you notice a gradual
onset of paralysis or confusion."
64 The nurse is teaching regarding risk Correct Answer: 4
9. factors for stroke (CVA). The greatest Rationale: Hypertension is the greatest
risk factor is which of the following? risk factor for stroke, and should be
1. diabetes controlled. Diabetes, heart disease, and
2. heart disease renal insufficiency can all lead to stroke,
3. renal insufficiency however hypertension is the greatest risk.
4. hypertension
65 The nurse is working on a surgical laminectomy.
0. floor. The nurse must logroll a client
following a:
65 The nurse must be alert to Correct Answer: 2,3,4
1. complications in the patient who has Rationale: Headache is a sign of a
suffered a ruptured intracranial probable rebleed. Hydrocephalus,
aneurysm. The nurse should assess rebleeding, and vasospasm are the three
the patient for signs of which of the major complications that a nurse must
following? anticipate following a ruptured intracranial
Select all that apply. aneurysm. Stiff neck is a manifestation of
1. headache intracranial aneurysm, not a complication.
2. hydrocephalus
3. rebleeding
4. vasospasm
5. stiff neck
65 The nurse observes a client in a group 3.
2. who is reminiscing about his past. Reminiscing can help reduce depression in
Which effect should the nurse expect an elderly client and lessens feelings of
reminiscing to have on the client's isolation and loneliness. Reminiscing
functioning in the hospital? encourages a focus on positive memories
1. Increase the client's confusion and and accomplishments as well as shared
disorientation. memories with other clients. An increase in
2. Cause the client to become sad. confusion and disorientation is most likely
3. Decrease the client's feelings of the result of other cognitive and situational
isolation and loneliness. factors, such as loss of short-term
4. Keep the client from participating memory, not reminiscing. The client will
in therapeutic activities. not likely become sad because reminiscing
helps the client connect with positive
memories. Keeping the client from
participating in therapeutic activities is
less likely with reminiscing.
65 The nurse observes that a client's 2. Voluntary and purposeful movements
3. upper arm tremors disappear as he often temporarily decrease or stop the
unbuttons his shirt. Which statement tremors associated with Parkinson's
best guides the nurse's analysis of disease. In some clients, however, tremors
this observation about the client's may increase with voluntary effort.
tremors? 1. The tremors are probably Tremors associated with Parkinson's
psychological and can be controlled at disease are not psychogenic but are
will. 2. The tremors sometimes related to an imbalance between
disappear with purposeful and dopamine and acetylcholine. Tremors
voluntary movements. 3. The tremors cannot be reduced by distracting the
disappear when the client's attention client.
is diverted by some activity. 4. There
is no explanation for the observation;
it is probably a chance occurrence.
65 The nurse observes that a comatose dysfunction in the brain stem.
4. client's response to painful stimuli is
decerebrate posturing. The client
exhibits extended and pronated arms,
flexed wrists with palms facing
backward, and rigid legs extended
with plantar flexion. Decerebrate
posturing as a response to pain
indicates:
65 The nurse obtains all of the following The patient's blood pressure (BP) is usually
5. information about a 65-year-old about 180/90 mm Hg.
patient in the clinic. When developing
a plan to decrease stroke risk, which rational: Hypertension is the single most
risk factor is most important for the important modifiable risk factor and this
nurse to address? patient's hypertension is at the stage 2
level. People who drink more than 1 (for
women) or 2 (for men) alcoholic beverages
a day may increase risk for hypertension.
Physical inactivity and obesity contribute
to stroke risk but not so much as
hypertension.
65 The nurse obtains these assessment Urine output of 800 mL in the last hour
6. findings for a patient who has a head
injury. Which finding should be rational: The high urine output indicates
reported rapidly to the health care that diabetes insipidus may be developing
provider? and interventions to prevent dehydration
need to be rapidly implemented. The other
data do not indicate a need for any change
in therapy.
65 A nurse on the Geropsychiatric unit 2.
7. receives a call from the son of a The two medicines are commonly given
recently discharged client. He reports together. Neither medicine will improve
that his father just got a prescription dementia, but may slow the progression.
for memantine (Namenda) to take "on Neither medicine is more effective than
top of his donepezil (Aricept)." The the other; they act differently in the brain.
son then asks, "Why does he have to Both medicines have a half-life of 60 or
take extra medicines?" The nurse more hours.
should tell the son:
1. "Maybe the Aricept alone isn't
improving his dementia fast enough
or well enough."
2. "Namenda and Aricept are
commonly used together to slow the
progression of dementia."
3. "Namenda is more effective than
Aricept. Your father will be tapered
off the Aricept."
4. "Aricept has a short half-life and
Namenda has a long half-life. They
work well together."
65 A nurse on the neurologic unit A client who sustained a fall on the
8. evaluates her client care assignment previous shift and is attempting to get out
after receiving the shift report. Which of bed
client in her assignment should she
attend to first?
65 The nurse on the neurologic unit A client who requires minimal bathing
9. must provide care for four clients assistance and ambulates with a walker
who require different levels of care. independently
Which client should the nurse assist
first with morning care?
66 The nurse realizes that the goal of Correct Answer: 3
0. surgery for a patient with a Rationale: The tumor can exert pressure on
secondary metastatic spinal cord the spinal cord, which interferes with
tumor is function. In the case of secondary metastatic
1. complete removal of the tumor spinal tumor (which means a second site of
and affected spinal cord tissue. cancer) and the metastasis (spread of
2. eradication of the tumor with cancer) the patient outcome may be limited
excision and drainage. to preventing compression on the spinal cord
3. tumor excision to reduce cord and not totally removing the cancerous
compression. lesion. Complete removal along with affected
4. exploration to visualize the spinal tissue or eradication by excision and
tumor and obtain a biopsy. drainage would not be likely due to the
secondary nature of the tumor and the
resulting disability. Biopsy can be
accomplished without direct visualization.
66 The nurse receives a physician's 31
1. order to administer 1,000 ml of
normal saline solution I.V. over 8
hours to a client who recently had a
stroke. What should the drip rate
be if the drop factor of the tubing is
15 gtt/ml?
66 The nurse receives a verbal report visual deficits.
2. that a patient has an occlusion of
the left posterior cerebral artery. rational: Visual disturbances are expected
The nurse will anticipate that the with posterior cerebral artery occlusion.
patient may have ___________________ Aphasia occurs with middle cerebral artery
involvement. Cognitive deficits and changes
in judgment are more typical of anterior
cerebral artery occlusion.
66 The nurse recognizes that the most Correct Answer: 1
3. common type of brain attack (CVA) Rationale: Eighty percent of all strokes are
is related to which of the following? caused by ischemia. Hemorrhagic strokes are
1. ischemia less common than ischemic strokes.
2. hemorrhage Headache and vomiting may be symptoms
3. headache associated with CVA, but not common
4. vomiting causes.
66 Nurses can increase environmental 3. Establishing a routine identified with each
4. stimuli for clients with sensory meal; Regular meaningful stimuli will benefit
deficit by: the client. The radio can provide meaningful
or meaningless stimuli. The nurse must
1. Keeping the radio on throughout carefully choose programming based on the
the day to provide auditory client's preferences and expose the client to
stimulation that programming only at appropriate times.
2. Keeping the bathroom light on at Listening to the radio constantly can
night to avoid complete darkness introduce meaningless stimuli that confuse
3. Establishing a routine identified the client. A 24-hour light may actually keep
with each meal clients awake, leading to sleep deprivation.
4. Ensuring the client's safety Safety is a priority diagnosis but is not an
intervention to provide environmental
stimuli.
66 The nurse sees a client walking in 3,1,4,2
5. the hallway who begins to have a
seizure. The nurse should do which
of the following in priority order? 1.
Maintain patent airway 2.record the
seizure activity observed 3.ease the
client to the floor 4. Obtain vital
signs
66 Nurses have the responsibility Have a reduced ability to respond to stress
6. to dispel myths and replace
stereotypes of older adults with
accurate information. The nurse
knows that most older adults:
66 A nurse should recognize that ANS: A, B, C
7. electroconvulsive therapy (ECT) ECT has been shown to be effective in the
would potentially improve the treatment of severe depression; acute mania;
symptoms of clients with which and acute schizophrenia, particularly if it is
of the following Axis I accompanied by catatonic or affective
diagnoses? (Select all that (depression or mania) symptomatology. ECT has
apply.) also been tried with other disorders, such as
obsessive-compulsive disorder (OCD) and
A. Major depressive disorder anxiety disorders, but little evidence exists to
B. Bipolar disorder: manic phase support its efficacy in the treatment of these
C. Schizoaffective disorder conditions
D. Obsessive-compulsive anxiety
disorder
E. Body dysmorphic disorder
66 The nurse understands that Correct Answer: 2
8. when the spinal cord is injured, Rationale: Within 24 hours necrosis of both gray
ischemia results and edema and white matter begins if ischemia has been
occurs. How should the nurse prolonged and the function of nerves passing
explain to the patient the reason through the injured area is lost. Because the
that the extent of injury cannot edema extends above and below the area
be determined for several days affected, the extent of injury cannot be
to a week? determined until after the edema is controlled.
1. "Tissue repair does not begin Neurons do not regenerate, and the edema is
for 72 hours." the factor that limits the ability to predict extent
2. "The edema extends the level of injury.
of injury for two cord segments
above and below the affected
level."
3. "Neurons need time to
regenerate so stating the injury
early is not predictive of how
the patient progresses."
4. "Necrosis of gray and white
matter does not occur until days
after the injury."
66 The nurse witnesses a patient assess the patient for a possible head injury.
9. with a seizure disorder as the
patient suddenly jerks the arms rational: The patient who has had a myoclonic
and legs, falls to the floor, and seizure and fall is at risk for head injury and
regains consciousness should be evaluated and treated for this possible
immediately. It will be most complication first. Documentation of the seizure,
important for the nurse to notification of the seizure, and administration of
_________________ antiseizure medications also are appropriate
actions, but the initial action should be
assessment for injury.
67 The nursing diagnosis Risk for 2. Uses a wheelchair due to paraplegia. Because
0. Impaired Skin Integrity related of the paraplegia (paralysis of lower body), the
to sensory-perceptual client is unable to feel discomfort. The client will
disturbance would best fit a be taught to lift self using chair arms every 10
client who: minutes if possible.

1. Cut a foot by stepping on


broken glass.
2. Uses a wheelchair due to
paraplegia.
3. Wears glasses because of
poor vision.
4. Is legally blind and smokes in
bed.
67 A nursing instructor is teaching ANS: B
1. about electroconvulsive therapy Electroconvulsive therapy is the induction of a
(ECT). Which student statement grand mal seizure through the application of
indicates that learning has electrical current to the brain for the purpose of
occurred? decreasing depression.

A. "During ECT a state of


euphoria is induced."
B. "ECT induces a grand mal
seizure."
C. "During ECT a state of
catatonia is induced."
D. "ECT induces a petit mal
seizure."
67 A nursing instructor is teaching ANS: C
2. about the medications given prior In order to render a client unconscious during
to and during electroconvulsive the ECT procedure, an anesthesiologist
therapy (ECT) treatments. Which administers intravenously, a short-acting
student statement indicates that anesthetic like thiopental sodium (Pentothal).
learning has occurred?

A. "Atropine (Atro-Pen) is
administered to paralyze skeletal
muscles during ECT."
B. "Succinylcholine chloride
(Anectine) decreases secretions to
prevent aspiration."
C. "Thiopental sodium (Pentothal)
is a short-acting anesthesia to
render the client unconscious."
D. "Glycopyrrolate (Robinul) is
given to prevent severe muscle
contractions during seizure."
67 Nursing Plans and Interventions: On entering room, announce your presence
3. The Blind Client clearly and identify yourself; address client
by name.
Never touch client unless he or she knows
you are there.
On admission, orient client thoroughly to
surroundings; Demonstrate use of the call
bell; Walk client around the room and
acquaint them with all objects, chairs, bed,
tv, telephone, ect.
Guide client when walking; Walk ahead of
client, and place their hand in the bend of
your elbow; Describe where you are walking,
note whether passageway is narrowing or
you are approaching stairs, curb, or incline.
Always raise side rails for newly sightless
persons.
Assist with meal enjoyment by describing
food and its placement in terms of the face of
a clock.
When administering meds, inform client of
number of pills and give only a half glass of
water to avoid spills.
67 Nursing staff are trying to provide 1.
4. for the safety of an elderly female Using a bed alarm enables the staff to
client with moderate dementia. She respond immediately if the client tries to get
is wandering at night and has out of bed. Sleeping in a chair at the nurse's
trouble keeping her balance. She station interferes with the client's restful
has fallen twice but has had no sleep and privacy. Using all four bedrails is
resulting injuries. The nurse considered a restraint and unsafe practice. It
should: is not appropriate to expect a family member
1. Move the client to a room near to stay all night with the client.
the nurse's station and install a bed
alarm.
2. Have the client sleep in a
reclining chair across from the
nurse's station.
3. Help the client to bed and raise
all four bedrails.
4. Ask a family member to stay with
the client at night.
67 A nursing student is observing an ANS: B
5. electroconvulsive therapy (ECT) A blood pressure cuff is placed on the lower
treatment. The student notices a leg and inflated above systolic pressure
blood pressure cuff on the client's before injection of succinylcholine. This is to
lower leg. The student questions ensure that seizure activity can be observed
the instructor about the cuff and timed in this one limb that is unaffected
placement. Which is the most by the paralytic agent.
accurate instructor reply?

A. "The cuff has to be placed on the


leg because both arms are used for
intravenous fluids."
B. "The cuff functions to prevent
succinylcholine from reaching the
foot."
C. "The cuff position gives a more
accurate blood pressure reading
during the treatment."
D. "The cuff is placed on the leg so
that arms can easily be restrained
during seizure."
67 observe the nursing assistant replace the pts splint in hyperextension
6. performing all of these
interventions for the pt with CTS
67 Occipital Lobe Visual interpretation and memory.
7.

67 Of the following, which Correct Answer: 3,4


8. groups are the most at risk Rationale: Military personnel living on a base and
for bacterial meningitis? young adults living in close proximity (such as college
Select all that apply. students living in a dormitory) are at a greater risk of
1. older adults contracting bacterial meningitis. The other populations
2. pregnant women are at lower risk.
3. military recruits
4. college students
5. low-income
67 The older adult is at risk for Instruct the older adult to use a night light in the
9. falls for various reasons. To bathroom
help prevent falls the nurse
may:
68 An older adult recently Delirium
0. diagnosed with a urinary
tract infection displays a
sudden onset of confusion.
She most likely is
experiencing
68 Older adults are at Have reduced kidney functioning
1. increased risk for drug
toxicity because they:
68 Older adults experience a Frequency and opportunities for sexual activity may
2. change in sexual activity. decline.
Which best explains this
change?
68 On patients with spinal C1-C4 because the diaphragm is paralyzed.
3. cord injuries, what areas
must be placed on vents?
68 On the 5th postoperative Auscultate the abdomen for bowel sounds.
4. day, a client who
underwent spinal fusion
begins to complain of
nausea and has an episode
of vomiting. How should a
nurse intervene?
68 The oropharynx is C. tonsillar pillars
5. separated from the mouth
by which of the following?

A. Frenulum

B. posterior pharyngeal
wall

C. tonsillar pillars

D. dorsum of the tongue


68 Osteoarthritis (OA) AKA DESCRIPTION: Noninflammatory arthritis. OA is
6. Degenerative Joint Disease characterized by a degeneration of cartilage, a wear-
(DJD) and-tear process. It usually affects on or two joints. It
occurs asymmetrically. Obesity and overuse are
predisposing factors.
NURSING ASSESSMENT:Joint pain that increases with
activity and improves with rest. Morning stiffness.
Asymmetry of affected joints. *Crepitus (grating
sound in the joint).* Limited movement. Visible joint
abnormalities indicated on radiographs. Joint
enlargement and bony nodules.
NURSING INTERVENTIONS:(same as RF)
Instruct in weight-reduction diet.
Remind client that excessive use of the involved joint
aggravates pain and may accelerate degeneration.
Teach client to: Use correct posture and body
mechanics.
Sleep with rolled terry cloth towel under cervical spine
if neck pain is a problem.
Relieve pain in fingers and hands by wearing stretch
gloves at night.
Keep joints in functional position.
Tylenol or NSAIDs
68 Osteoporosis DESCRIPTION: Metabolic disease in which bone
7. demineralization results in decreased density
and subsequent fractures. Many fractures in
older adults occur as result of osteoporosis and
often occur prior to the client's falling rather
than as the result of a fall *pathologic
fracture. The cause of osteoporosis is
unknown. Postmenopausal women are at
highest risk estrogen keeps calcium in
bone*
NURSING ASSESSMENT: Classic dowager's
hump, or kyphosis of the dorsal spine. Loss of
height, often 2-3 inches. Back pain, often
radiating around the trunk. Pathologic fractures,
often occurring in the distal end of the radius
and the upper third of the femur. Compression
fracture of spine- assess ability to void and
defecate.
NURSING PLANS AND INTERVENTIONS: Create a
hazard-free environment *safety first*. Keep
bed in low position. Encourage client to wear
shoes or nonskid slippers when out of bed.
Encourage environmental safety: adequate
lighting, keep floor clear, discourage use of
throw rugs, slean spills promptly, keep side rails
up at all times. Provide assistance with
ambulation: client may need walker or cane.
Client may need standby assistance when
initially getting out of bed or chair. Teach regular
exercise program. ROM exercise several times a
day, ambulation several times a day, use of
proper body mechanics. Provide diet that is high
in protein, calcium, and vitamin D; discourage
use of alcohol and caffeine. Preventive
measures for females: HRT has been used as a
primary prevention straegy for reducing bone
loss in the postmenopausal woman. Recent
studies demonstrated that HRT may increase a
woman's risk of breast cancer, cardiovascular
disease, and stroke. If using HRT the benefits
should outweigh the risks.
Take prescribed meds to prevent further loss of
bone mineral density (BMD).
*Bisposphonates: inhibits osteoclast-
mediated bone resorption, thereby
increasing BMD. Common side effects are
anorexia, weight loss, and gastritis.
Instruct the client to take with full glass of
water, take 30 minutes before food or
other meds and remain upright for at least
30 minutes after taking*.
Fosamax, Bonefos, Actonel.
Selective estrogen receptor modulator: to
mimmic the effect of estrogen on bone by
reducing bone resorption without stimulating
the tissues of the breast or uterus. The most
common side effects are leg cramps, hot
flashes, vaginal dryness. Evista
High calium and vitamin D intake beginning in
early adulthood. Calcium supplementation after
menopause (Tums are an excellent source of
calcium).
Weight-bearing exercise.
Osteopenia is defined as a bone loss that is
more than normal and has a T-score less than or
equal to a range of -1 to -2.5 but is not yet at
the level for a dx of osteoporosis.
68 Osteoporosis Hesi Hint #1 Postmenopausal, thin white women are at
8. highest risk for development of osteoporosis.
Encourage exercise, a diet high in calcium and
supplemental calcium. Tums are an excellent
source of calcium, but they are also high in
sodium, so hypertensive or edematous
individuals should seek another souce of
supplemental calcium.
68 Osteoporosis Hesi Hint #2 The main cause of fractures in older adults,
9. especially women is osteoporosis. The main
fracture sites seem to be hip, vertebral bodies,
and Colles fracture of the forearm.
69 OTHER OPTIONS ...
0.

69 PARA T1 - L4
1.

69 Parasympathetic peace
2.

69 The parents of a client who Social worker


3. sustained a closed head injury in
a motor vehicle accident voice
their concerns about the
distance and cost of the
rehabilitation center chosen for
their son. Which health care
team member can help the
parents with their questions and
concerns?
69 Parietal Lobe Sensory lobe: person's awareness of body
4. position in space, size and shape discrimination,
right-left orientation.
69 parkinsons depletion of dopamine
5.

69 Parkinson's Disease Dopamine deficiency occurs in basil ganglia. the


6. dopamine releasing pathway that connects the
substantia nigra to the corpus stratum
69 Partial/Focal Seizure - motor
7. Manifestations - sensory eg visual hallucinations; burnt rubber
smell
- autonomic
- emotional/psychic - fear, deja vue
- simple partial or complex partial (simple partial
= affects consciousness)
69 Patency of the nostrils is B. sniff inward through one nairs while the other
8. assessed by having the is occluded
individual:

A. breathe through both nares


rapidly

B. sniff inward through one


nairs while the other is
occluded

C. blow of through each naris


while the other is occluded

D. observe each naris for flaring


during each inhalation and
exhalation
69 Pathogenesis 1. excessive neuronal excitability
9. 2. synapse connections between
hypersynchronous neurons

- consider contributions of voltage gated ion


channels in this process (AEDs act on these ion
channels)
- consider balance of GABAergic neurons
(inhibitory pathways) and excitatory (esp
Glutaminergic) neurons (excitatory pathways)
70 pathologic fx results from minimal trauma to a bone weakened
0. by disease
70 A patient admitted to the administer an intradermal test dose.
1. emergency department is
diagnosed with botulism, and rational: To prevent allergic reactions, an
an order for botulinum antitoxin intradermal test dose of the antitoxin should be
is received. Before administered. Although temperature, allergy
administering the antitoxin, it is history, and symptom assessment and
most important for the nurse to documentation are appropriate, these
_______________ assessments will not affect the decision to
administer the antitoxin.
70 A patient admitted with Swap the nasopharyngeal mucosa for cultures.
2. bacterial meningitis and a
temperature of 102 F (38.8 C) rational: Antibiotic therapy should be instituted
has orders for all of these rapidly in bacterial meningitis, but cultures must
collaborative interventions. be done before antibiotics are started. As soon as
Which action should the nurse the cultures are done, the antibiotic should be
take first? started. Hypothermia therapy and
acetaminophen administration are appropriate
but can be started after the other actions are
implemented.
70 A patient arrives at an urgent administration of the tetanus-diphtheria (Td)
3. care center with a deep booster.
puncture wound after stepping
on a nail that was lying on the rational: If the patient has not been immunized
ground. The patient reports within 5 years, administration of the Td booster is
having had a tetanus booster 7 indicated because the wound is deep. Immune
years ago. The nurse will globulin administration is given by the IM route if
anticipate ______________ the patient has no previous immunization.
Administration of a series of immunization is not
indicated. TIG is not indicated for this patient,
and a test dose is not needed for immune
globulin.
70 A patient found in a tonic- Atonic
4. clonic seizure reports
afterward that the seizure rational: The initial symptoms of a partial seizure
was preceded by numbness involve clinical manifestations that are localized to
and tingling of the arm. The a particular part of the body or brain. Symptoms of
nurse knows that this finding an absence seizure are staring and a brief loss of
indicates what type of consciousness. In an atonic seizure, the patient
seizure? loses muscle tone and (typically) falls to the
ground. Myoclonic seizures are characterized by a
sudden jerk of the body or extremities.
70 A patient has a new The patient's blood pressure is 90/46 mm Hg.
5. prescription for bromocriptine
(Parlodel) to control rational: Hypotension is an adverse effect of
symptoms of Parkinson's bromocriptine, and the nurse should check with the
disease. Which information health care provider before giving the medication.
obtained by the nurse may Diarrhea, cough, and deep vein thrombosis are not
indicate a need for a decrease associated with bromocriptine use.
in the dose?
70 A patient has an incomplete Positioning the patient's right leg when turning the
6. right spinal cord lesion at the patient
level of T7, resulting in
Brown-Squard syndrome. rational: The patient with Brown-Squard syndrome
Which nursing action should has loss of motor function on the ipsilateral side
be included in the plan of and will require the nurse to move the right leg.
care? Pain sensation will be lost on the patient's left leg.
Left arm weakness will not be a problem for a
patient with a T7 injury. The patient will retain
position sense for the left leg.
70 A patient has a stroke risk for injury related to denial of deficits and
7. affecting the right hemisphere impulsiveness.
of the brain. Based on
knowledge of the effects of rational: Right-sided brain damage typically causes
right brain damage, the nurse denial of any deficits and poor impulse control,
establishes a nursing leading to risk for injury when the patient attempts
diagnosis of _______________ activities such as transferring from a bed to a chair.
Right-sided brain damage causes left hemiplegia.
Left-sided brain damage typically causes language
deficits. Left-sided brain damage is associated with
depression and distress about the disability.
70 A patient has a systemic BP of Report the BP and ICP to the health care provider.
8. 108/51 mm Hg and an
intracranial pressure (ICP) of rational: The patient's cerebral perfusion pressure
14 mm Hg. Which action is 56 mm Hg, below the normal of 60 to 100 mm Hg
should the nurse take first? and approaching the level of ischemia and neuronal
death. Immediate changes in the patient's therapy
such as fluid infusion or vasopressor administration
are needed to improve the cerebral perfusion
pressure. Adjustments in the head elevation should
only be done after consulting with the health care
provider. Continued monitoring and documentation
also will be done, but they are not the first actions
that the nurse should take.
70 A patient has a tonic-clonic Time and observe and record the details of the
9. seizure while the nurse is in seizure and postictal state.
the patient's room. Which
action should the nurse take? rational: Because diagnosis and treatment of
seizures frequently are based on the description of
the seizure, recording the length and details of the
seizure is important. Insertion of an oral airway and
restraining the patient during the seizure are
contraindicated. The nurse may need to move the
patient to decrease the risk of injury during the
seizure.
71 A patient has manifestations Correct Answer: 2,5
0. of autonomic dysreflexia. Rationale: Autonomic dysreflexia can be caused by
Which of these assessments kinked catheter tubing allowing the bladder to
would indicate a possible become full, triggering massive vasoconstriction
cause for this condition? below the injury site, producing the manifestations
Select all that apply. of this process. Acute symptoms of autonomic
1. hypertension dysreflexia, including a sustained elevated blood
2. kinked catheter tubing pressure, may indicate fecal impaction. The other
3. respiratory wheezes and answers will not cause autonomic dysreflexia.
stridor
4. diarrhea
5. fecal impaction
71 A patient has the nursing diagnosis Correct Answer: 1
1. of Impaired Swallowing and Rationale: Keeping the patient upright for a
complains of frequent heartburn. time after the meal will help prevent food
What is the most appropriate action from being regurgitated back into the
by the nurse? esophagus. The position of the patient
1. Assist the patient in maintaining a during the meals as well as teaching the
sitting position for 30 minutes after "chin tuck" technique will assist with the
the meal. swallowing mechanism, but will not help
2. Teach the patient the "chin tuck" with regurgitation. Pocketing food does not
technique when swallowing. cause regurgitation.
3. Check the patient's mouth for
pocketing of food.
4. Assist the patient to a 90-degree
sitting position, or as high as
tolerated, during meals.
71 A patient hospitalized with a known Correct Answer: 3
2. AV malformation begins to complain Rationale: An AV malformation is a cluster
of a headache and becomes of vessels, usually located in the midline
disorientated. Which is the most cerebral artery, that, if ruptured, becomes a
appropriate action by the nurse? surgical emergency to cut the blood flow to
1. Recommend to the family the vessels or the patient will bleed out into
members that they start to look for a the brain. Symptoms of rupture include
long-term care facility. headache,,change in level of
2. Prepare to give aspirin or a "clot consciousness,, nausea and vomiting, and
buster." neurological deficits symptoms that mimic
3. Prepare the patient for surgery. any brain bleed. Giving medication to affect
4. Document the changes and coagulation will only make the bleeding
monitor closely. worse. Recommending long-term care and
merely documenting the changes are not
appropriate interventions for a medical
emergency.
71 A patient is admitted to the hospital aspirin (Ecotrin).
3. with dysphasia and right-sided
weakness that resolves in a few rational: Following a TIA, patients typically
hours. The nurse will anticipate are started on medications such as aspirin
teaching the patient about to inhibit platelet function and decrease
____________ stroke risk. tPA is used for acute ischemic
stroke. Coumadin is usually used for
patients with atrial fibrillation. Nimodipine is
used to prevent cerebral vasospasm after a
subarachnoid hemorrhage.
71 The patient is admitted with injuries Correct Answer: 2
4. that were sustained in a fall. During Rationale: Spinal shock is common in acute
the nurse's first assessment upon spinal cord injuries. In addition to the signs
admission, the findings are: blood and symptoms mentioned, the additional
pressure 90/60 (as compared to sign of absence of the cremasteric reflex is
136/66 in the emergency associated with spinal shock. Lack of
department), flaccid paralysis on the respiratory effort is generally associated
right, absent bowel sounds, zero with high cervical injury. The findings
urine output, and palpation of a describe paralysis that would be associated
distended bladder. These signs are with spinal shock in an spinal injured
consistent with which of the patient. The likely cause of these findings is
following? not hypovolemia, but rather spinal shock.
1. paralysis
2. spinal shock
3. high cervical injury
4. temporary hypovolemia
71 A patient is admitted with signs of a Correct Answer: 4
5. stroke (CVA). On admission, vital Rationale: Rising systolic blood pressure,
signs were blood pressure 128/70, falling pulse, and a pupil that has become
pulse 68, and respirations 20. Two sluggish suggest increasing intracranial
hours later the patient is not awake, pressure (IICP). This is an emergency
has a blood pressure of 170/70, pulse situation that requires notification of the
52, and the left pupil is now slower physician. This is an emergency situation
than the right pupil in reacting to that requires intervention as the patient's
light. These findings suggest which condition is becoming more unstable. Brain
of the following? death is diagnosed by lack of brain waves
1. impending brain death and inability to maintain vital function.
2. decreasing intracranial pressure
3. stabilization of the patient's
condition
4. increased intracranial pressure
71 A patient is brought to the Obtain oxygen saturation.
6. emergency department (ED) by
ambulance after being found rational: Airway patency and breathing are
unconscious on the bathroom floor the most vital functions and should be
by the spouse. Which action will the assessed first. The neurologic assessments
nurse take first? should be accomplished next and the
health and medication history last.
71 A patient is placed in Correct Answer: 3,4,5
7. ventilator support with Rationale: Maintaining fluids at KVO is inappropriate
the diagnosis of botulism since this patient will be placed on NPO (nothing by
and failure to thrive. mouth) status while ventilated. It is important that the
Which nursing actions patient receive adequate fluids for hydration and
would be most nutrition since nothing will be consumed by mouth. The
appropriate for this patient's bowel sounds need to be assessed more often
patient? than once a shift (every one to two hours while in the
Select all that apply. ICU) since the patient is at risk for a paralytic ileus.
1. maintaining Physical therapy will be beneficial for maintaining ROM
intravenous fluids at KVO (range of motion) while the patient is immobile from
(keep vein open) ventilation and sedation. The nurse must closely monitor
2. assessing bowel the patient's calorie intake to determine nutritional
sounds once a shift needs while NPO. Any time a patient is on maintenance
3. referring the patient intravenous fluids urinary output must be monitored
for a physical therapy closely. Additionally, this particular patient is at risk for
consult urinary retention.
4. recording the patient's
ongoing calorie count
5. assessing the patient's
urinary output every hour
71 A patient is recovering Correct Answer: 1
8. following a carotid Rationale: Take a blood pressure reading manually to
endarterectomy. The blood check technique, compare the results to the last
pressure has risen this several blood pressures recorded, and call the
morning to 168/60. The nurse physician to report this blood pressure. Physicians
should do which of the typically have a range for maintaining the blood
following? pressure following carotid endarterectomy, with
1. Recheck the blood standing orders for higher or lower blood pressures.
pressure and make sure the If the blood pressure becomes higher, it is a danger
correct size cuff was used. and should be reported to the physician and
Then compare the trend of documented in the patient record along with orders
blood pressure readings and received. Although the skill of the staff is important,
call the physician now. it is a priority to notify the physician of the blood
2. Recheck the blood pressure reading so that treatment can begin.
pressure every hour and Antihypertensives may be ordered and administered
report this change to the p.r.n., but physician notification after verification of
physician when he or she the reading is the priority, so that further evaluation
makes rounds the next time. can occur.
3. Record the blood pressure
and find out who took this
reading. Have that staff
member demonstrate his or
her blood pressure procedure
and offer tips to obtain more
accurate readings.
4. Check the standing orders
and see if there is a
medication ordered p.r.n. for
lowering blood pressure. If
so, administer it and
document the action.
71 A patient is seen in the antiparkinsonian drugs.
9. health clinic with symptoms
of a stooped posture, rational: The diagnosis of Parkinson's is made when
shuffling gait, and pill rolling- two of the three characteristic signs of tremor,
type tremor. The nurse will rigidity, and bradykinesia are present. The
anticipate teaching the confirmation of the diagnosis is made on the basis of
patient about _______________ improvement when antiparkinsonian drugs are
administered. This patient has symptoms of tremor
and bradykinesia; the next anticipated step will be
treatment with medications. MRI and EEG are not
useful in diagnosing Parkinson's disease, and
corticosteroid therapy is not used to treat it.
72 A patient seen at the health Ask the patient to keep a headache diary.
0. clinic with a severe migraine
headache tells the nurse rational: The initial nursing action should be further
about having four similar assessment of the precipitating causes of the
headaches in the last 3 headaches, quality, and location of pain, etc. Stress
months. Which initial action reduction, muscle relaxation, and the triptan drugs
should the nurse take? may be helpful, but more assessment is needed first.
72 Patients with problems with 9 glossopharyngeal , 10 vagus, and 12
1. which cranial nerves should not hypoglossal
be fed by a nursing assistant?
72 Patient Teaching consists of: (7) Taking medications at prescribed times
2. Good skin care to avoid skin ulcers
Staying physically active (dance, yoga, pilates,
biking)
Proper ambulation (assistive devices)
Proper positioning (avoid ulcers)
High-Fiber diet & adequate fluid intake (Prevent
constipation)
Proper feeding techniques (avoid aspiration)
72 A patient tells the nurse about Discuss the need to stop taking the
3. using acetaminophen (Tylenol) acetaminophen.
several times every day for
recurrent bilateral headaches. rational: The headache description suggests that
Which action will the nurse plan the patient is experiencing medication overuse
to take first? headache. The initial action will be withdrawal of
the medication. The other actions may be
needed if headaches persist.
72 A patient was diagnosed with a Correct Answer: 1,2,5
4. left cerebral hemorrhage. Which Rationale: The left cerebral hemisphere is
topics are most appropriate for responsible for the language center, calculation
the nurse to include in patient skills, and thinking/reasoning abilities. Reading
and family teaching? and speaking could be compromised if there is
Select all that apply. left-sided brain damage. The patient also might
1. how to use a sign board display overcautious behavior and might be slow
2. transfer techniques to respond or complete activities. Transfer
3. information about impulse techniques would apply regardless of the side
control involved. Impulse control problems can arise
4. time adjustment to complete with right-sided involvement.
activities
5. safety precautions for
transferring
72 A patient who has a head injury Provide discharge instructions about monitoring
5. is diagnosed with a concussion. neurologic status.
Which action will the nurse plan
to take? rational: A patient with a minor head trauma is
usually discharged with instructions about
neurologic monitoring and the need to return if
neurologic status deteriorates. MRI, hospital
admission, or surgery are not indicated in a
patient with a concussion.
72 A patient who has a history of a Explain that the aspirin is ordered to decrease
6. transient ischemic attack (TIA) stroke risk.
has an order for aspirin 160 mg
daily. When the nurse is rational: Aspirin is ordered to prevent stroke in
administering the medications, patients who have experienced TIAs.
the patient says, "I don't need Documentation of the patient's refusal to take
the aspirin today. I don't have the medication is an inadequate response by the
any aches or pains." Which nurse. There is no need to clarify the order with
action should the nurse take? the health care provider. The aspirin is not
ordered to prevent aches and pains.
72 A patient who has bacterial Encourage family members to remain at the
7. meningitis is disoriented and bedside.
anxious. Which nursing action
will be included in the plan of rational: Patients with meningitis and
care? disorientation will be calmed by the presence of
someone familiar at the bedside. Restraints
should be avoided because they increase
agitation and anxiety. The patient requires
frequent assessment for complications; the use
of touch and a soothing voice will decrease
anxiety for most patients. The patient will have
photophobia, so the light should be dim.
72 A patient who has had a stroke assist the patient into a chair.
8. has a new order to attempt oral
feedings. The nurse should rational: The patient should be as upright as
assess the gag reflex and then possible before attempting feeding to make
_____________ swallowing easier and decrease aspiration risk.
To assess swallowing ability, the nurse should
initially offer water or ice to the patient. Pureed
diets are not recommended because the texture
is too smooth. The patient may have a poor
appetite, but the oral feeding should be
attempted regardless.
72 A patient who has had a The patient's blood pressure is 90/50 mm Hg.
9. subarachnoid hemorrhage is
being cared for in the intensive rational: To prevent cerebral vasospasm and
care unit. Which information maintain cerebral perfusion, blood pressure
about the patient is most needs to be maintained at a level higher than
important to communicate to the 90 mm Hg systolic after a subarachnoid
health care provider? hemorrhage. A low BP or drop in BP indicates a
need to administer fluids and/or vasopressors
to increase the BP. An ongoing headache, RBCs
in the CSF, and a stiff neck are all typical
clinical manifestations of a subarachnoid
hemorrhage and do not need to be rapidly
communicated to the health care provider.
73 A patient who has numbness and IV infusion of immunoglobulin (Sandoglobulin).
0. weakness of both feet is
hospitalized with Guillain-Barr rational: Because the Guillain-Barr syndrome
syndrome. The nurse will is in the earliest stages (as evidenced by the
anticipate the need to teach the symptoms), use of high-dose immunoglobulin
patient about _____________ is appropriate to reduce the extent and length
of symptoms. Mechanical ventilation and tube
feedings may be used later in the progression
of the syndrome but are not needed now.
Corticosteroid use is not helpful in reducing the
duration or symptoms of the syndrome.
73 A patient who has right-sided Disabled family coping related to inadequate
1. weakness after a stroke is understanding by patient's spouse
attempting to use the left hand
for feeding and other activities. rational: The information supports the
The patient's wife insists on diagnosis of disabled family coping because
feeding and dressing him, telling the wife does not understand the rehabilitation
the nurse, "I just don't like to see program. There are no data supporting low
him struggle." Which nursing self-esteem, and the patient is attempting
diagnosis is most appropriate for independence. The data do not support an
the patient? interruption in family processes because this
may be a typical pattern for the couple. There
is no indication that the patient has impaired
nutrition.
73 A patient who is hospitalized with The patient was oriented and alert when
2. pneumonia is disoriented and admitted.
confused 2 days after admission.
Which information obtained by rational: The onset of delirium occurs acutely.
the nurse about the patient The degree of disorientation does not
indicates that the patient is differentiate between delirium and dementia.
experiencing delirium rather than Increasing confusion for several years is
dementia? consistent with dementia. Fragmented and
incoherent speech may occur with either
delirium or dementia.
73 A patient who is seen in the diphenhydramine (Benadryl)
3. outpatient clinic complains of
restless legs syndrome. Which of rational: Antihistamines can aggravate restless
the following over-the-counter legs syndrome. The other medications will not
medications that the patient is contribute to the restless legs syndrome.
taking routinely should the nurse
discuss with the patient?
73 A patient who is suspected of Prepare the patient for immediate craniotomy.
4. having an epidural hematoma is
admitted to the emergency rational: The principal treatment for epidural
department. Which action will the hematoma is rapid surgery to remove the
nurse plan to take? hematoma and prevent herniation. If
intracranial pressure (ICP) is elevated after
surgery, furosemide or high-dose barbiturate
therapy may be needed, but these will not be
of benefit unless the hematoma is removed.
Minimal blood loss occurs with head injuries,
and transfusion is usually not necessary.
73 A patient whose status is post- Correct Answer: 1
5. stroke (CVA) has severe right- Rationale: Proper use of the cane is essential to
sided weakness. Physical fall prevention. The patient should hold the cane
therapy recommends a quad in the left hand. The patient should move the
cane. Which of the following is cane forward first, then the right leg, and then
proper use of the cane by the the left leg.
patient?
1. The patient holds the cane in
the left hand. The patient
moves the cane forward first,
then the right leg, and then the
left leg.
2. The cane is held in either
hand and moved forward at the
same time as the left leg. Then
the patient drags the right leg
forward.
3. The patient holds the cane in
the right hand for support. The
patient moves the cane forward
first, then the left leg, and then
the right leg.
4. The patient holds the cane in
the left hand. The patient
moves the left leg forward first,
then moves the cane and the
right leg forward together.
73 A patient who sustained a Ask for the patient's input into the plan for care.
6. spinal cord injury a week ago
becomes angry, telling the rational: The patient is demonstrating behaviors
nurse "I want to be transferred consistent with the anger phase of the mourning
to a hospital where the nurses process, and the nurse should allow expression of
know what they are doing!" anger and seek the patient's input into care.
Which reaction by the nurse is Expression of anger is appropriate at this stage
best? and should be tolerated by the nurse.
Reassurance about the competency of the staff
will not be helpful in responding to the patient's
anger. Ignoring the patient's comments will
increase the patient's anger and sense of
helplessness.
73 A patient with a head injury has Blood pressure 156/60, pulse 55, respirations 12
7. admission vital signs of blood
pressure 128/68, pulse 110, and rational: Systolic hypertension with widening
respirations 26. Which of these pulse pressure, bradycardia, and respiratory
vital signs, if taken 1 hour after changes represent Cushing's triad and indicate
admission, will be of most that the intracranial pressure (ICP) has increased,
concern to the nurse? and brain herniation may be imminent unless
immediate action is taken to reduce ICP. The
other vital signs may indicate the need for
changes in treatment, but they are not indicative
of an immediately life-threatening process.
73 A patient with a head injury 11
8. opens the eyes to verbal
stimulation, curses when rational: The patient has a score of 3 for eye
stimulated, and does not opening, 3 for best verbal response, and 5 for
respond to a verbal command to best motor response.
move but attempts to remove a
painful stimulus. The nurse
records the patient's Glasgow
Coma Scale score as _______
73 A patient with a history of a T2 Check the blood pressure (BP).
9. spinal cord injury tells the
nurse, "I feel awful today. My rational: The BP should be assessed immediately
head is throbbing, and I feel in a patient with an injury at the T6 level or
sick to my stomach." Which higher who complains of a headache to
action should the nurse take determine whether autonomic dysreflexia is
first? occurring. Notification of the patient's health care
provider is appropriate after the BP is obtained.
Administration of an antiemetic is indicated after
autonomic dysreflexia is ruled out as the cause of
the nausea. The nurse may assess for a fecal
impaction, but this should be done after checking
the BP and lidocaine jelly should be used to
prevent further increases in the BP.
74 A patient with a history of tissue plasminogen activator (tPA) infusion.
0. several transient ischemic
attacks (TIAs) arrives in the rational: The patient's history and clinical
emergency department with manifestations suggest an acute ischemic
hemiparesis and dysarthria that stroke and a patient who is seen within 4.5
started 2 hours previously. The hours of stroke onset is likely to receive tPA
nurse anticipates the need to (after screening with a CT scan). Heparin
prepare the patient for __________ administration in the emergency phase is not
indicated. Emergent carotid transluminal
angioplasty or endarterectomy is not indicated
for the patient who is having an acute ischemic
stroke.
74 A patient with a left-sided brain teach the family that emotional outbursts are
1. stroke suddenly bursts into tears common after strokes.
when family members visit. The
nurse should _____________ rational: Patients who have left-sided brain
stroke are prone to emotional outbursts, which
are not necessarily related to the emotional
state of the patient. Depression after a stroke is
common, but the suddenness of the patient's
outburst suggests that depression is not the
major cause of the behavior. The family should
stay with the patient. The crying is not within
the patient's control and asking the patient to
stop will lead to embarrassment.
74 A patient with amyotrophic Assist with active range of motion.
2. lateral sclerosis (ALS) is
hospitalized with pneumonia. rational: ALS causes progressive muscle
Which nursing action will be weakness, but assisting the patient to perform
included in the plan of care? active ROM will help to maintain strength as
long as possible. Psychotic symptoms such as
agitation and paranoia are not associated with
ALS. Cognitive function is not affected by ALS,
and the patient's ability to understand
procedures will not be impaired. Muscle
relaxants will further increase muscle weakness
and depress respirations.
74 A patient with a neck fracture at hypotension, bradycardia, and warm
3. the C5 level is admitted to the extremities.
intensive care unit. During initial
assessment of the patient, the rational: Neurogenic shock is characterized by
nurse recognizes the presence of hypotension, bradycardia, and vasodilation
neurogenic shock on finding leading to warm skin temperature. Spasticity
________________ and hyperactive reflexes do not occur at this
stage of spinal cord injury. Lack of movement
and sensation indicate spinal cord injury, but
not neurogenic shock.
74 A patient with a spinal cord injury Correct Answer: 2
4. at the T1 level complains of a Rationale: Autonomic dysreflexia occurs in
severe headache and an "anxious patients with injury at level T6 or higher, and is
feeling." Which is the most a life-threatening situation that will require
appropriate initial reaction by the immediate intervention or the patient will die.
nurse? The most common cause is an overextended
1. Try to calm the patient and bladder or bowel. Symptoms include
make the environment soothing. hypertension, headache, diaphoresis,
2. Assess for a full bladder. bradycardia, visual changes, anxiety, and
3. Notify the healthcare provider. nausea. A calm, soothing environment is fine,
4. Prepare the patient for though not what the patient needs in this case.
diagnostic radiography. The nurse should recognize this as an
emergency and proceed accordingly. Once the
assessment has been completed, the findings
will need to be communicated to the healthcare
provider.
74 A patient with a spinal cord injury Correct Answer: 1
5. is recovering from spinal shock. Rationale: Be attuned to the prevention of a
The nurse realizes that the distended bladder when caring for spinal cord
patient should not develop a full injury (SCI) patients in order to prevent this
bladder because what emergency chain of events that lead to autonomic
condition can occur if it is not dysreflexia. Track urinary output carefully.
corrected quickly? Routine use of bladder scanning can help
1. autonomic dysreflexia prevent the occurrence. Other causes of
2. autonomic crisis autonomic dysreflexia are impacted stool and
3. autonomic shutdown skin pressure. Autonomic crisis, autonomic
4. autonomic failure shutdown, and autonomic failure are not terms
used to describe common complications of
spinal injury associated with bladder distension.
74 A patient with a spinal cord Correct Answer: 4
6. injury (SCI) has complete Rationale: Quadriplegia describes complete
paralysis of the upper paralysis of the upper extremities and complete
extremities and complete paralysis of the lower part of the body.
paralysis of the lower part of the Hemiplegia describes paralysis on one side of
body. The nurse should use the body. Paresthesia does not indicate
which medical term to paralysis. Paraplegia is paralysis of the lower
adequately describe this in body.
documentation?
1. hemiplegia
2. paresthesia
3. paraplegia
4. quadriplegia
74 A patient with a spinal cord Correct Answer: 2,4,5
7. injury (SCI) is admitted to the Rationale: The healthcare provider is
unit and placed in traction. responsible for initial applying of the traction
Which of the following actions is device. The weights on the traction device must
the nurse responsible for when not be changed without the order of a
caring for this patient? healthcare provider. When caring for a patient in
Select all that apply. traction, the nurse is responsible for assessment
1. modifying the traction weights and care of the skin due to the increased risk of
as needed skin breakdown. The patient in traction is likely
2. assessing the patient's skin to experience pain and the nurse is responsible
integrity for assessing this pain and administering the
3. applying the traction upon appropriate analgesic as ordered. Passive range
admission of motion helps prevent contractures; this is
4. administering pain medication often performed by a physical therapist or a
5. providing passive range of nurse.
motion
74 A patient with a spinal cord Correct Answer: 2
8. injury was given IV Decadron Rationale: A common side effect of
(dexamethasone) after arriving corticosteroids is hyperglycemia. Stress as well
in the emergency department. as the medication could cause this person to
The patient also has a history of have periods of elevated blood sugars.
hypoglycemia. During the
hospital stay, the nurse would
expect to see which of the
following?
1. increased episodes of
hypoglycemia
2. possible episodes of
hyperglycemia
3. no change in the patient's
glycemic parameters
4. both hyper- and hypoglycemic
episodes
74 A patient with a stroke Difficulty in understanding commands
9. experiences right-sided arm and
leg paralysis and facial drooping rational: Right-sided paralysis indicates a left-
on the right side. When brain stroke, which will lead to difficulty with
admitting the patient, which comprehension and use of language. The left-
clinical manifestation will the side reflexes are likely to be intact. Impulsive
nurse expect to find? behavior and neglect are more likely with a
right-side stroke.
75 A patient with a stroke has Risk for aspiration related to inability to protect
0. progressive development of airway
neurologic deficits with
increasing weakness and rational: Protection of the airway is the priority
decreased level of consciousness of nursing care for a patient having an acute
(LOC). Which nursing diagnosis stroke. The other diagnoses also are
has the highest priority for the appropriate, but interventions to prevent
patient? aspiration are the priority at this time.
75 A patient with a suspected A) Halo sign on the nasal drip pad
1. closed head injury has bloody When drainage containing both CSF and blood is
nasal drainage. The nurse allowed to drip onto a white pad, within a few
suspects that this patient has minutes the blood will coalesce into the center, and
a cerebrospinal fluid (CSF) a yellowish ring of CSF will encircle the blood, giving
leak when observing which of a halo effect. The presence of glucose would be
the following? unreliable for determining the presence of CSF
A) A halo sign on the nasal because blood also contains glucose.
drip pad
B) Decreased blood pressure
and urinary output
C) A positive reading for
glucose on a Test-tape strip
D) Clear nasal drainage along
with the bloody discharge
75 A patient with a T1 spinal full function of the patient's arms will be retained.
2. cord injury is admitted to the
intensive care unit. The nurse rational: The patient with a T1 injury can expect to
will teach the patient and retain full motor and sensory function of the arms.
family that ______________ Use of only the shoulders is associated with cervical
spine injury. Loss of respiratory function occurs with
cervical spine injuries. Bradycardia is associated
with injuries above the T6 level.
75 A patient with Bell's palsy respect the patient's desire and arrange for privacy
3. refuses to eat while others at mealtimes.
are present because of
embarrassment about rational: The patient's desire for privacy should be
drooling. The best response respected to encourage adequate nutrition and
by the nurse to the patient's reduce patient embarrassment. Liquid supplements
behavior is to _______________ will reduce the patient's enjoyment of the taste of
food. It would be inappropriate for the nurse to
discuss the patient's embarrassment with visitors
unless the patient wishes to share this information.
Chewing on the unaffected side of the mouth will
enhance nutrition and enjoyment of food but will
not decrease the drooling.
75 A patient with left-sided Check the respiratory rate.
4. hemiparesis arrives by
ambulance to the emergency rational: The initial nursing action should be to
department. Which action assess the airway and take any needed actions to
should the nurse take first? ensure a patent airway. The other activities should
take place quickly after the ABCs (airway, breathing,
circulation) are completed.
75 A patient with mild dementia Having the patient's spouse administer the
5. has a new prescription for medication
donepezil (Aricept). Which
nursing action will be most rational: Because the patient with mild dementia
effective in ensuring will have difficulty with learning new skills and
compliance with the forgetfulness, the most appropriate nursing action is
medication? to have someone else administer the drug. The
other nursing actions will not be as effective in
ensuring that the patient takes the medications.
75 A patient with multiple Teach the patient how to use the Cred method.
6. sclerosis (MS) has urinary
retention caused by a flaccid rational: The Cred method can be used to improve
bladder. Which action will the bladder emptying. Decreasing fluid intake will not
nurse plan to take? improve bladder emptying and may increase risk for
urinary tract infection (UTI) and dehydration. The
use of incontinence briefs and frequent toileting will
not improve bladder emptying.
75 A patient with multiple How to draw up and administer injections of the
7. sclerosis (MS) is to begin medication
treatment with glatiramer
acetate (Copaxone). Which rational: Copaxone is administered by self-injection.
information will the nurse Oral contraceptives are an appropriate choice for
include in patient teaching? birth control. There is no need to avoid driving or
drink large fluid volumes when taking glatiramer.
75 A patient with paraplegia Teach the patient how to self-catheterize.
8. resulting from a T10 spinal cord
injury has a neurogenic reflex rational: Because the patient's bladder is spastic
bladder. Which action will the and will empty in response to overstretching of
nurse include in the plan of the bladder wall, the most appropriate method is
care? to avoid incontinence by emptying the bladder at
regular intervals through intermittent
catheterization. Assisting the patient to the toilet
will not be helpful because the bladder will not
empty. The Cred method is more appropriate for
a bladder that is flaccid, such as occurs with a
reflexic neurogenic bladder. Catheterization after
voiding will not resolve the patient's incontinence.
75 A patient with Parkinson's Suggest that the patient rock from side to side to
9. disease has a nursing diagnosis initiate leg movement.
of impaired physical mobility
related to bradykinesia. Which rational: Rocking the body from side to side
action will the nurse include in stimulates balance and improves mobility. The
the plan of care? patient will be encouraged to continue exercising
because this will maintain functional abilities.
Maintaining a wide base of support will help with
balance. The patient should lift the feet and avoid
a shuffling gait.
76 A patient with Parkinson's Imbalanced nutrition: less than body
0. disease has decreased tongue requirements
mobility and an inability to
move the facial muscles. Which rational: The data about the patient indicate that
nursing diagnosis is of highest poor nutrition will be a concern because of
priority? decreased swallowing. The other diagnoses also
may be appropriate for a patient with Parkinson's
disease, but the data do not indicate they are
current problems for this patient.
76 A patient with Parkinson's -Use an elevated toilet seat
1. disease is admitted to the -Cut patient's food into small pieces
hospital for treatment of an -Place an arm chair at the patient's bedside
acute infection. Which nursing
interventions will be included in rational: Since the patient with Parkinson's has
the plan of care: difficulty chewing, food should be cut into small
pieces. An armchair should be used when the
patient is seated so that the patient can use the
arms to assist with getting up from the chair. An
elevated toilet seat will facilitate getting on and
off the toilet. High protein foods will decrease the
effectiveness of L-dopa. Parkinson's is a steadily
progressive disease without acute exacerbations.
76 A patient with possible cerebral Administer 5% hypertonic saline intravenously.
2. edema has a serum sodium
level of 115 mEq/L (115 rational: The patient's low sodium indicates that
mmol/L) and a decreasing level hyponatremia may be causing the cerebral
of consciousness (LOC) and edema, and the nurse's first action should be to
complains of a headache. Which correct the low sodium level. Acetaminophen
of these prescribed (Tylenol) will have minimal effect on the headache
interventions should the nurse because it is caused by cerebral edema and
implement first? increased intra-cranial pressure (ICP). Drawing
ABGs and obtaining a CT scan may add some
useful information, but the low sodium level may
lead to seizures unless it is addressed quickly.
76 A patient with right-sided Noncontrast computed tomography (CT) scan
3. weakness that started 90
minutes earlier is admitted to rational: Rapid screening with a noncontrast CT
the emergency department and scan is needed before administration of tissue
all these diagnostic tests are plasminogen activator (tPA), which must be given
ordered. Which test should be within 4.5 hours of the onset of clinical
done first? manifestations of the stroke. The sooner the tPA is
given, the smaller the area of brain injury. The
other diagnostic tests give information about
possible causes of the stroke and do not need to
be completed as urgently as the CT scan.
76 A patient with sudden-onset The patient has atrial fibrillation and takes
4. right-sided weakness has a CT warfarin (Coumadin).
scan and is diagnosed with an
intracerebral hemorrhage. rational: The use of warfarin will have contributed
Which information about the to the intracerebral bleeding and remains a risk
patient is most important to factor for further bleeding. Administration of
communicate to the health care vitamin K is needed to reverse the effects of the
provider? warfarin, especially if the patient is to have
surgery to correct the bleeding. The history of
hypertension is a risk factor for the patient but
has no immediate effect on the patient's care.
The BP of 144/90 indicates the need for ongoing
monitoring but not for any immediate change in
therapy. Slurred speech is consistent with a left-
sided stroke, and no change in therapy is
indicated.
76 Pediatric fractures DESCRIPTION: Traumatic injury to bone. Fractures that occur in
5. the epiphyseal plate (growth plate) may affect growth of the
limb.
ASSESSMENT: General condition: visible bone fragments. Pain,
swelling, contusions. Child guarding or protecting the
extremity.
Possibility of being able to use fractured extremity due to
intact periosteum.
NURSING PLANS AND INTERVENTIONS:Obtain baseline data,
and frequently perform neurovascular assessments.
Report abnormal assessment promptly! Compartment
syndrome may occur; it results in permanent damage to the
nerves and vasculature of the injured extremity due to
compression.
Maintain traction if prescribed. Note bed position, type of
traction, weights, pulleys, pins, pin sites, adhesive strips, ace
wraps, splints and casts.
Skin traction: force is applied to skin.
Buck extension traction: lower extremity, legs extended, no
hip flexion.
Dunlop traction: two lines of pull on the arm.
Russell traction: two lines of pull on the lower extremity, one
perpendicular, one longitudinal.
Bryant traction: both lower extremities flexed 90 degrees at
hips (rarely used because extreme elevation of lower
extremities causes decreased peripheral circulation).
Skeletal traction: pin or wire applies pull directly to the distal
bone fragment. 90 degree traction: flexion of hip and knee;
lowr extremity is in a boot cast, can also be used on upper
extremities. Dunlop traction may be used as skeletal traction.
Maintain child in proper body alignment; restrain if necessary.
Monitor for problems of immobility.
Provide age-appropriate play and toys.
Prepare child for cast application; use age-appropriate terms
when exlpaining procedures.
Provide routine cast care following application; petal cast
edges.
Teach home cast care to family: neurvascular assessment of
casted extremity; not to get cast wet; not to place anything
under cast; keep small objects, toys, and food out of cast.
Teach family to modify diapering and toileting to prevent cast
soilage.
Teach that in the presence of a hip spica, family may use a
Bradford frame under a small child to help with toileting; they
must not use abduction bar to turn child.
Teach to seek follow-up care with HCP.
76 Pedi HH #1 Fractures in older children are common because they fall
6. during play and are involved in MVAs. Spiral fractures (caused
by twisting) and fx in infants may be related to child abuse.
Fractures involving the epiphyseal plate can have serious
consequences in terms of the growth of the affected limb.
76 Pedi HH #2 Skin traction for fx reduction should not be removed unless
7. HCP prescibes its removal.
76 Pedi HH #3 Pin sites can be sources of infection. Monitor for signs of
8. infection. Cleanse adn dress pin sites as prescribed.
76 Pedi HH #4 Skeletal disorders affect the infant's or child's physical
9. mobility and typical questions focus on appropriate toys and
activities for the child who is confined to bed rest and is
immobilized.
77 Peripheral 1. Risk for injury; The nurse determines what effects
0. neuropathy and peripheral neuropathy will have on the client. Swallowing
paresthesias takes place in the posterior pharynx and esophagus. This is
become the etiology centrally located; unrelated to peripheral neuropathy. Fluid
for other nursing overload is related to excess intake relative to output, or
diagnoses. An organ failure such as heart failure, not sensory perception. It is
example of such a unlikely paresthesias would cause social isolation.
diagnosis is:

1. Risk for injury


2. Impaired
swallowing
3. Fluid volume
overload
4. Social isolation
77 PET Scan a visual display of brain activity that detects where a
1. radioactive form of glucose goes while the brain performs a
given task
77 Phenobarbital Approved for:
2. -Focal
-Generalized tonic-clonic
Side effects:
-Sedation, dizziness, ataxia,
osteopenia, behavior disturbance
in children
77 Phenytoin (Dilantin) Approved for:
3. -Focal
-Generalized tonic-clonic
Side effects:
-CYP inducer
-Teratogenic (fetal hydantoin
syndrome)
-Nystagmus, ataxia, diplopia,
sedation
-SLE-like syndrome ("purple glove
syndrome")
-Gum hyperplasia
-Hirsutism
-Megaloblastic anemia,
generalized lymphadenopathy
77 Phenytoin - key characteristics ...
4.

77 The physician determines that a client's Otosclerosis


5. chronic, progressive hearing loss results from
excess bone formation around the oval
window, which impedes normal stapes
movement and prevents sound transmission.
What is the clinical term for this correctable
middle ear disorder?
77 A physician diagnoses a client with Intestinal obstruction
6. myasthenia gravis and prescribes
pyridostigmine (Mestinon), 60 mg by mouth
every 3 hours. Before administering this
anticholinesterase agent, the nurse reviews
the client's history. Which preexisting
condition would contraindicate the use of
pyridostigmine?
77 The physician orders measurement of the 4 hours
7. serum acetaminophen level of a client
admitted with a suspected overdose of this
drug. To ensure an accurate result, the nurse
should wait how long after acetaminophen
(Tylenol) ingestion before drawing the blood
sample?
77 The physician orders risperidone (Risperdal) 3.
8. for a client with Alzheimer's disease. The Antipsychotics are most effective
nurse anticipates administering this with agitation and assaultiveness.
medication to help decrease which of the Antipsychotics have little effect
following behaviors? on sleep disturbances,
1. Sleep disturbances. concomitant depression, or
2. Concomitant depression. confusion and withdrawal.
3. Agitation and assaultiveness.
4. Confusion and withdrawal.
77 The physician prescribes diazepam (Valium), It should be administered no
9. 10 mg I.V., for a client experiencing status faster than 5 mg/minute in an
epilepticus. Which statement about I.V. adult.
diazepam is true?
78 The physician prescribes mannitol (Osmitrol) Warm the solution in hot water to
0. I.V. stat for a client who develops increased dissolve the crystals.
intracranial pressure after a head injury.
While preparing to administer mannitol, the
nurse notices crystals in the solution. What
should the nurse do?
78 The physician suspects myasthenia gravis in a thymus gland hyperplasia.
1. client with chronic fatigue, muscle weakness,
and ptosis. Myasthenia gravis is associated
with:
78 pons - stem respiratory
2.

78 A post-stroke patient is going Correct Answer: 2


3. home on oral Coumadin Rationale: Warfarin is a vitamin K antagonist.
(warfarin). During discharge Green, leafy vegetables contain vitamin K, and will
teaching, which statement by therefore interfere with the therapeutic effects of
the patient reflects an the drug. Bruising is a common side effect, and the
understanding of the effects drug should not be stopped unless by prescriber
of this medication? order. Low-fat foods do not interfere with warfarin
1. "I will stop taking this therapy, which is not prescribed to affect the blood
medicine if I notice any pressure.
bruising."
2. "I will not eat spinach while
I'm taking this medicine."
3. "It will be OK for me to eat
anything, as long as it is low
fat."
4. "I'll check my blood
pressure frequently while
taking this medication."
78 Pregabalin (Lyrica) Approved for:
4. -Focal
Side effects:
-Weight gain, edema, somnolence, dizziness, ataxia,
tremor
78 Pregabalin (Lyrica) Same risks and benefits as Gabapentoin though bit
5. more S/E
78 Pregnancy, pre-pregnancy Use an AED
6. and AEDs -try to use only one,
-at lowest effective dose;
-follow free serum drug levels (monitor levels
during pregnancy)
-try to keep them constant;
-try to avoid valproate (most teratogenic)

Risk of AED use during pregnancy << risk of


seizures on fetus during pregnancy

Pre-pregnancy -- consider extra folate if taking


AEDs (which often interrupt folic acid pathway)
78 Process of Diagnosing 1) first seizure, unprovoked:
7. Seizures and Recommending - ID cause (labs eg. CBC, electrolytes, UA, toxicity
Mgmt screen, BUN, Glu). Cause found, then treat it
- No cause ID --> no AED (b/c 50% will go on to
have seizures, but 50% will not)
- F/u tests (MRI, etc - missed anything/)
- NO DRIVING

2) second seizure
- AED (b/c most likely to continue having seizures)
- which AED?
78 Prodromal manifestations Correct Answer: 1,2,4
8. prior to an intracranial Rationale: Often intracranial aneurysms are
aneurysm rupture could be asymptomatic until rupture but patients can
recognized by the nurse as complain of headache and eye pain, and have
which of the following? visual deficits and a dilated pupil. Nausea and
Select all that apply. vomiting and stiff neck are not usually associated
1. visual deficits with the prodromal manifestations of an intracranial
2. headache aneurysm, but may occur with leaking or rupture.
3. mild nausea
4. dilated pupil
5. stiff neck
78 QUAD C1 - C8
9.

79 A quadriplegic client is Muscle spasms with paraplegia or quadriplegia from


0. prescribed baclofen spinal cord lesions
(Lioresal), 5 mg by mouth
three times daily. What is the
principal indication for
baclofen?
79 Quirks / Adverse gingival hyperplasia (overgrowth)
1. Effects hirsutism
rash (occasionally serious)
osteoporosis (vit D metabolism issues)
cerebellar damage with long use
79 Receptive Aphasia aphasia characterized by fluent but meaningless speech and
2. severe impairment of the ability to understand spoken or
written words
79 Recognize changes glasgow's coma scale, A&O x3, widening pulse pressure,
3. in neurological abnormal body posturing, cushing's triad, cranial nerve checks,
status. confusion, hallucinations, out of control emotions

diagnostic tests for head injuries


blood glucose, ABGs, tox screen, creatinine, BUN, liver function
tests, CBC + diff, CT, MRI, LP, cerebral angiography, xray of the
brain
will be able to determine where the injury is, how big it is
with an LP, encourage fluid intake - CSF reproduces after 24h
hours
when an LP is done, there is a space in the spinal column. pat
may complain of HA because the air from that space naturally
goes upwards, in this case towards the head.
to test to see if leakage is CSF, check for glucose - see halo on
gauze.
79 Recognize patient's preventions: allow the client time to grieve or to express denial,
4. response to SC depression, and anger over the changes in social, financial, and
injury (anger, grief, personal roles - the patient needs time to adjust to lifestyle
hopeless or changes. provide accurate information based on the physician's
suicidal) . prognosis. include family and significant others to treat the
client as normally as possible. refer the client and family to
support groups.
79 Recognize signs STAGE 1: 2-4 years after onset
5. and symptoms of short term memeory loss; forgets locations and names of
client's impaired objects
cognition. attempts to cover up memory loss
has difficulty learning new information or making decisions
decreased attention span
can be angry or depressed
antidementia meds are trying to prolong this stage.

STAGE 2: 2-10 years after end of stage 1


unable to remember names of family members and gets lost in
familiar locations
easily agitated and irritable
has difficulty using objects; reading, writing, and speaking
cannot follow a conversation
personal hygiene declines
unable to make decisions (choose clothing)
walks and unsteady gait, head down, shoulders bent, shuffles
exhibits "sundowning" and wandering behavior

STAGE 3: 1-3 years after stage 2


cannot recognize self or others
inability to communicate
has delusions and hallucinations
bowel and bladder incontinence
79 Recognize the The middle-aged adult family member may become the care-
6. patient/family's taker for an older parent. An older adult may be unable to care
ability to adapt to for a spouse who has had a stroke. They may have to accept
role changes. placement of the spouse into an LTC.

Emphasize that physical function may continue to improve for


up to 3 months, and speech may continue to improve even
longer.
79 The red light reflex is caused by the B. reflection of light off the inner retina
7.
A. refraction of light off the
conjunctiva

B. reflection of light off the inner


retina

C. reflection of light off the choroids


layer

D. condenstaion of light as it passes


through the aqueous humor.
79 Regular oral hygiene is essential for 1. A helpless client should be positioned on
8. the client who has had a stroke. the side, not on the back, with the head on
Which of the following nursing a small pillow. A lateral position helps
measures is not appropriate when secretions escape from the throat and
providing oral hygiene? 1. Placing the mouth, minimizing the risk of aspiration. It
client on the back with a small pillow may be necessary to suction the client if
under the head. 2. Keeping portable he aspirates. Suction equipment should be
suctioning equipment at the bedside. nearby. It is safe to use a padded tongue
3. Opening the client's mouth with a blade, and the client should receive oral
padded tongue blade. 4. Cleaning the care, including brushing with a toothbrush.
client's mouth and teeth with a
toothbrush.
79 Reinforce teaching to patient/family Talk about changing modifiers to prevent
9. healthy lifestyle choices. future TIAs or CVAs: controlling
hypertension (stress, meds), reinforcing
the benefits of anticoagulant therapy and
other info about medications, low-
cholesterol and low-fat diet to reduce
arteriosclerosis, etc.
when to seek medical care; complications
such as aspiration, pneumonia, UTI, skin
breakdown; safety measures to prevent
falls; psychologic support.
80 Requirements for Epilepsy Surgery 1. Spells must be epileptic
0. 2. Seizures must be focal (partial)
a. All seizures must have the same focus
3. The focus must be in a region of brain
that can be safely removed
80 respiratory issues (ataxic, cluster, damage to pons, medulla
1. apneustic)
80 RF Hesi Hint #1 A client comes to the clinic complaining of
2. morning stiffness, weight loss, and swelling
of both hands and wrists. Rheumatoid
arthritis is suspected. Which methods of
assessment might the nurse use, and
which methods should not be used?
Use: Inspection, Palpation, Stregnth testing
Do NOT Use:ROM, this activity promotes
pain because ROM is limited.
80 RF Hesi Hint #2 In the joint, the normal cartilage becomes
3. soft, fissures and pitting occur, and the
cartilage thins. Spurs form and
inflammation sets in. The result is
deformity marked by:
Immobility
Pain
Muscle spasm
The prescribed treatment regimen is
corticosteroids for the inflammation;
splinting, immobilization, and rest for the
joint deformity; and NSAIDS for pain.
80 RF Hesi Hint #3 Synovial tissues line the bones of the
4. joints. Inflammation of this lining causes
destruction of tissue and bone. Early
detection of rheumatoid arthritis can
decrease the amount of bone and joint
destruction. Often the disease goes into
remission. Decreasing the amount of bone
and joint destruction reduces the amount
of disability.
80 RF Hesi What activity recommendations should the nurse provide a client with
5. Hint #4 rheumatoid arthritis?
Do not exercise painful, swollen joints.
Do not exercise any joint ot the point of pain.
Perform exercises slowly and smoothly; avoid jerky movements.
80 Rheumato DESCRIPTION: Chronic, systematic, progressive deterioration of the
6. id connective tissue (synovioum) of the joints; characterized by
Arthritis inflammation. The exact cause is unknown but it is classified as an
immune complex disorder, autoimmune. Joint involvement is bilateral and
symmetrical. Severe cases may require joint replacement.
NURSING ASSESSMENT: Fatigue; Generalized weakness; Weight loss;
Anorexia; Morning stiffness; Bilateral inflammation of joints with:
decreased ROM, joint pain, warmth, edema, erythema (rash). Joint
deformity.
DIAGNOSIS: Elevated erythrocyte sedimentation rate (ESR); Positive
rheumatoid factor (RF). Presence of antinuclear antibody, positive (ANA).
Joint-space narrowing indicated by arthroscopic examination, (provides
joint visualization). *Spongy and boggy joints* Abnormal synovial fluid
(fluid in joint) indicated by arthrocentesis. C-reactive protein (CRP)
indicated by active inflammation.
NANDA: Chronic pain r/t..... * antidepressants usually ordered.*
NURSING INTERVENTIONS:
A. Pain relief measures:
1. Use moist heat. Warm, moist compresses, Whirlpool baths, Hot shower
in the morning.
2. Use diversionary activities. Imaging, Distraction, Self-hypnosis,
Biofeedback.
3. Administer meds and teach client about meds.
B. Provide periods of rest after periods of activity:
1. Encourage self-care to maximal level.
2. Allow adequate time for the client to perform activities.
3. Perform activities during time of day when client feels most energetic.
C. Encourage the client to avoid overexertion and to maintain proper
posture and joint position.
D. Encourage use of assistive devices to promote funtional ADL's:
1. Elevated toilet seat.
2. Shower chair.
3. Cane, walker, wheelchair.
4. Reachers.
5. Adaptive clothing and shoes with velcro closures.
6. Straight-backed chair with elevated seat. (remember you can build up a
chair with pillows if needed).
Develop a teaching plan to include the following:
1.Medication regimen.
2. Need for routine follow-up for evaluation of possible side effects.
3. ROM and stretching exercies tailored to specific client needs.
4. Safety tips and precaustions about equipment use and environment.
**Early diagnosis is better because DMAR's can be given to
prevent joint deformity**
80 right brain paralyzed left
7. stroke spatial defects
short attention
impulsivity
denial
impaired time, judgement
80 The right hand of a client with multiple 4
8. sclerosis trembles severely whenever
she attempts a voluntary action. She
spills her coffee twice at lunch and
cannot get her dress fastened securely.
Which is the best legal documentation
in nurses' notes of the chart for this
client assessment? 1. "Has an intention
tremor of the right hand." 2. "Right-
hand tremor worsens with purposeful
acts." 3. "Needs assistance with
dressing and eating due to severe
trembling and clumsiness." 4. "Slight
shaking of right hand increases to
severe tremor when client tries to
button her clothes or drink from a
cup."
80 Right-sided CVA Spatial-perceptual deficits, increased
9. distractibility, impulsive behavior and
poor judgement, lack of awareness of
deficits
81 Rinne Test to assess bone conduction, place base
0. against mastoid bone and note how long
it takes until the patient can no longer
hear the sound

air conduction, place next to auditory


canal
81 The roof of the mouth is divided into 2 A. hard and soft palates
1. parts known as the:

A. hard and soft palates

B. anterior and posterior palates

C. frontal and ethmoid arches

D. superior lingual and uvular arches


81 A school nurse is called after a student Correct Answer: 3
2. falls down a flight of stairs. The Rationale: Guidelines for emergency care
student is breathing, but are avoiding flexing, extending, or
unconsciousness. After calling the rotating the neck; immobilizing the neck;
ambulance, which is the most securing the head; maintaining the
appropriate action by the nurse? patient in the supine position; and
1. Protect the patient's neck and head transferring from the stretcher with
from any movement. backboard in place to the hospital bed.
2. Place the patient on his side to This patient is unconscious, and the
prevent aspiration. nurse must protect the neck from any (or
3. Immobilize the neck,,securing the any further) damage. If the patient
head. vomits, the nurse should utilize the log-
4. Try to rouse the patient by gently roll technique to turn the patient while
shaking his shoulders. keeping the head, neck, and spine in
alignment. Rousing the patient by
shaking could cause damage to the
spinal cord.
81 S/E weight gain
3. peripheral edema
81 (SELECT ALL THAT APPLY) A client is (2) Turn the client on his right side., (5)
4. admitted to the medical-surgical unit Apply a soft collar to keep the client's
after undergoing intracranial surgery neck in a neutral position.
to remove a tumor from the left
cerebral hemisphere. Which nursing
interventions are appropriate for the
client's postoperative care?
81 (SELECT ALL THAT APPLY) A client who (2) Wrist pronation, (3) Stiff extension of
5. had a massive stroke exhibits the arms and legs, (4) Plantar flexion of
decerebrate posture. What are the the feet (5) Opisthotonos
characteristics of this posture?
81 (SELECT ALL THAT APPLY) A client with (1) Assist the client to the floor., (2) Turn
6. a history of epilepsy is admitted to the the client to his side., (3) Place a pillow
medical-surgical unit. While assisting under the client's head.
the client from the bathroom, the
nurse observes the start of a tonic-
clonic seizure. Which nursing
interventions are appropriate for this
client?
81 (SELECT ALL THAT APPLY) A client with (1) Monitor for skin rash., (3) Perform
7. tonic-clonic seizure disorder is being good oral hygiene, including daily
discharged with a prescription for brushing and flossing., (4) Periodic follow-
phenytoin (Dilantin). Which up blood work is necessary., (5) Report to
instructions about phenytoin should the physician problems with walking and
the nurse give this client? coordination, slurred speech, or nausea.
81 (SELECT ALL THAT APPLY) (3) Furnish the client's environment with familiar
8. The nurse is assigned to possessions., (4) Assist the client with activities of
care for a client with early daily living (ADLs) as necessary., (5) Assign tasks in
stage Alzheimer's disease. simple steps.
Which nursing
interventions should be
included in the client's care
plan?
81 (SELECT ALL THAT APPLY) (1) Visual disturbances, (3) Balance problems, (5)
9. The nurse is planning care Mood disorders
for a client with multiple
sclerosis. Which problems
should the nurse expect the
client to experience?
82 (SELECT ALL THAT APPLY) (2) "I'll try to chew my food on the unaffected side.",
0. The nurse is teaching a (4) "Drinking fluids at room temperature should
client with trigeminal reduce pain.", (5) "If brushing my teeth is too painful,
neuralgia how to minimize I'll try to rinse my mouth instead."
pain episodes. Which
comments by the client
indicate that he
understands the
instructions?
82 Semicomatose Can be aroused by extreme or repeated stimuli
1.

82 Sensorineural Hearing Loss DESCRIPTION: Form of hearing loss in which sound


2. passes properly through the outer and middle ear but
is distorted by a defect in the inner ear. It involves
perceptual loss, usually progressive and bilateral. It
involves damage to the 8th cranial nerve.
(vestibulocochlear). It is detected easily by the use of
a tuning fork.
Common causes: Infections.
Ototoxic drugs-aspirin, lasix, aminoglycasides,
vancomycin
Trauma
Neuromas
Noise
Aging process- presbycusis
NURSING ASSESSMENT: Inability to hear a whisper
from 1-2 feet away. Inability to respond if nurse covers
mouth when talking, indicating that client is lip
reading. Inability to hear a watch tick 5 inches from
ear. Shouting in conversation. Straining to hear.
Turning head to favor one ear. Answering questions
incorrectly or inappropriately. Raising volume of radio
or tv.
NURING PLANS AND INTERVENTIONS:The nurse should
do the following to enhance therapeutic
communication with the hearing impaired:
Prior to starting conversation, reduce distraction as
much as possible.
Turn the tv or radio down or off, close the door, or
move to a quieter location.
Devote full attention to the conversation; do not try to
do two things at once.
Look and listen during the conversation.
Begin with casual topics, and progress to more critical
issues slowly.
Do not switch topics abruptly.
If you do not understand, let the client know.
If the client is a lip reader, face them directly.
Speak slowly and distinctly; determine whether you
are being understood.
Allow adequate time for the conversation to take
place; try to avoid hurried conversations.
Use active listening techniques.
Be sure to inform the health care staff of the clients
hearing loss.
Helpful aids may include a d telephone amplifier,
earphone attachments for the radio and tv, and lights
or buzzers that indicate the doorbell is ringing, located
in the most commonly used rooms of the house.
82 Sensoristasis Is the term used to describe the state in which a
3. person is in optimal arousal.
82 Sensory deficit defects in the function of one or more of the senses,
4. resulting in visual, auditory, or olfactory impairments.
82 Sensory deprivation condition resulting from decreased sensory input or
5. input that is monotonous, unpatterned, or
meaningless.
82 Sensory overload condition resulting from excessive sensory input to
6. which the brain is unable to meaningfully respond
82 Sensory perception involves the conscious organization and translation
7. of the data or stimuli into meaningful information
82 Sensory reception the process of receiving data about the internal or
8. external environment through the senses
82 S/E of all AEDs a. sleepiness
9. b. nausea
c. dizziness
d. diplopia
e. ataxia
f. headache
g. rash (**)
h. TERATOGENICITY! (esp valproic acid)
83 Several weeks after a stroke, Assist the patient onto the bedside commode every
0. a patient has urinary 2 hours.
incontinence resulting from
an impaired awareness of rational: Developing a regular voiding schedule will
bladder fullness. For an prevent incontinence and may increase patient
effective bladder training awareness of a full bladder. A 1200 mL fluid
program, which nursing restriction may lead to dehydration. Intermittent
intervention will be best to catheterization and use of a condom catheter are
include in the plan of care? appropriate in the acute phase of stroke but should
not be considered solutions for long-term
management because of the risks for urinary tract
infection (UTI) and skin breakdown.
83 Sexuality is recognized as a All older adults, whether healthy or frail, need to
1. factor in the care of older express sexual feelings
adults, thus:
83 Shortly after admission to an In 10 to 15 minutes
2. acute care facility, a client
with a seizure disorder
develops status epilepticus.
The physician orders
diazepam (Valium), 10 mg I.V.
stat. How soon can the nurse
administer a second dose of
diazepam, if needed and
prescribed?
83 Should everyone young and YES
3. old wear sunglasses when in
the sun?
83 Signs and Symptoms of PD classic triad (tremors, rigidity, kinesia)
4. (13) bradykinesia
akinisea
dyskenesia
stupor
mask like face, poor blink reflex, wide open eye
unsteady on feet
shuffling gait
cognitive impairment
drooling
loss of postural control
slowed, monotonous speech
dysphagia
83 Simple partial seizure Focal seizure in which consciousness is NOT
5. impaired
83 Sinemet This drug is given to Parkinson's Disease patients
6. early in the disease course. It is very effective for
the management of akinetic symptoms. Tremor and
rigidity may also respond to this drug. After a few
years of therapy the effectiveness of sinemet wears
off and other drugs are prescribed.
83 skin around pin site is swollen notify the hcp-indicates osteomalitis
7. red and crusty with dried
drainage
83 Somatic Nervous System Voluntary control of body movements
8.

83 Somnolent Extreme drowsiness but will respond to


9. stimuli
84 The son of an elderly client who has 4.
0. cognitive impairments approaches Expecting the social worker to make the
the nurse and says, "I'm so upset. decision indicates that the son is avoiding
The physician says I have 4 days to participating in decisions about his father.
decide on where my dad is going to The other responses convey that the son
live." The nurse responds to the son's understands the importance of a careful
concerns, gives him a list of types of decision, the availability of resources, and
living arrangements, and discusses the ability to make new plans if needed.
the needs, abilities, and limitations of
the client. The nurse should intervene
further if the son makes which
comment?
1. "Boy, I have a lot to think about
before I see the social worker
tomorrow."
2. "I think I can handle most of Dad's
needs with the help of some home
health care."
3. "I'm so afraid of making the wrong
decision, but I can move him later if I
need to."
4. "I want the social worker to make
this decision so Dad won't blame me."
84 Spinal cord Houses the reflex arc for actions such as
1. the knee-jerk reflex
84 The spouse of a male patient with -Offer ideas for ways to distract or redirect
2. early stage Alzheimer's disease (AD) the patient.
tells the nurse, "I am just exhausted -Educate the spouse about the availability
from the constant worry. I don't know of adult day care as a respite.
what to do." Which action is best for -Ask the spouse what she knows and has
the nurse to take next: considered about dementia care options.
rational: The stress of being a caregiver
can be managed with a multicomponent
approach. This includes respite care,
learning ways to manage challenging
behaviors, and further assessment of what
the spouse may already have considered.
The patient is in the early stages and does
not need long-term placement. Antianxiety
medications may be appropriate but other
measures should be tried first.
84 Stage 1 Normal function; no memory problems
3.

84 Stage 2 Very mild cognitive decline; may be normal


4. age-related changes or very early signs of
AD; forgetfulness, especially of everyday
objects (eyeglasses or wallet; no memory
problems evident to provider, friends, or
coworkers
84 Stage 3 Mild cognitive decline; problems w/memory
5. or concentration may be measurable in
clinical testing or during a detailed medical
interview; mild cognitive deficit, including
losing or misplacing important objects;
decreased ability to plan; short-term
memory loss noticeable to close relatives;
decreased attention span; difficulty
remembering words or names; difficulty in
social or work situations
84 Stage 4 Moderate cognitive decline; mild or early-
6. stage AD; medical interview will detect
clear-cut decline; personality changes -
appearing withdrawn or subdued,
especially in social or mentally challenging
situations; obvious memory loss; limited
knowledge & memory of recent occasions,
current events, or personal history
84 Stage 5 Moderately severe cognitive decline;
7. moderate or mid-stage AD; increasing
cognitive deficits emerge; inability to recall
important details such as address,
telephone number or schools attended, but
memory of information about self & family
remains intact; disorientation & confusion
as to time & place
84 Stage 6 Severe cognitive decline; moderately severe or mid-
8. stage AD; memory difficulties continue to worsen
loss of awareness of recent events & surroundings;
may recall own name, but unable to recall personal
history; significant personality changes are evident
(delusions, hallucinations, & compulsive behaviors);
wandering behavior; requires assistance w/usual
daily activities such as dressing, toiling, & other
grooming; normal sleep/wake cycle is disrupted;
increased episodes of urinary & fecal incontinence
84 Stage 7 Very severe cognitive decline; severe or late-stage
9. AD; ability to respond to environment, speak, &
control movement is lost; unrecognizable speech;
general urinary incontinence; inability to eat w/out
assistance & impaired swallowing; gradual loss of all
ability to move extremities (ataxia)
85 Status epilepticus Most common causes: stroke, meningitis,
0. management encephalitis, hypoxic-ischemic encephalopathy
1. Ensure ABCs
2. Get labs and start normal saline
3. IV lorazepam, then phenytoin/fosphenytoin
4. If seizures persist -> additional IV fosphenytoin ->
IV midazolam -> IV phenobarbital -> induce
barbiturate coma
85 Steroid Hesi Hint** *Many people take steroids for a variety of
1. conditions, questions often focus on the need
to teach clients the importance of following
the prescribed regimen precisely. Clients
should be cautioned against stopping the
medications suddenly and should be informed
that it is necessary to taper off the dosage
when taking steroids.*
85 Subarachnoid screw or bolt a hollow bolt put into subarachnoid space via burr
2. hole in front of skull and connected to fluid filled
transducer. LEVEL transducer to approximate
location of lateral ventricles.
85 subdural hematoma slow, venous bleed
3.

85 suffered a fx femr pt appears confused-fracture of a long bone early


4. manifestiation of fat emboli
85 Sympathetic stress
5.

85 SYMPTOMATIC ...
6. LOCALIZATION-RELATED
EPILEPSY
85 Symptoms of ICP severe headache, deteriorating LOC, restlessness,
7. irritability, dilated or pinpoint pupils, slow rxn time,
altered breathing pattern, deteriorating motor
function, abnormal posturing (decorticate,
decerebrate, flaccidity)
85 Tapering Medications a. in some patients who have gone for a length of
8. time with no seizures, the seizure focus resolves; no
test can reliably determine if this has happened, so
patients must recognize it is a gamble, but often one
worth taking (with physician supervision)
85 Taste buds atrophy and lose Salty, sour, and bitter tastes
9. sensitivity, and appetite may
decrease. The older adult is
less able to discern:
86 Temporal Lobe Auditory receptive areas, plays a role in memory of
0. sound and understanding language and music.
86 The term motor apraxia 2.
1. relates to a decline in motor Highly conditioned motor skills, such as brushing the
patterns essential for teeth, may be retained by the client who has
complex motor tasks. dementia and motor apraxia. Balancing a checkbook
However, the client with involves calculations, a complex skill that is lost with
severe dementia may be able severe dementia Confabulation is fabrication of
to perform which of the details to fill a memory gap. This is more common
following actions? when the client is aware of a memory problem, not
1. Balance a checkbook when dementia is severe. Finding keys is a memory
accurately. factor, not a motor function.
2. Brush the teeth when
handed a toothbrush.
3. Use confabulation when
telling a story.
4. Find misplaced car keys.
86 thalamus pain
2.

86 . The nurse is teaching the family of a 2. A client with dysphagia (difficulty


3. client with dysphagia about swallowing) commonly has the most
decreasing the risk of aspiration difficulty ingesting thin liquids, which are
while eating. Which of the following easily aspirated. Liquids should be
strategies is not appropriate? 1. thickened to avoid aspiration. Maintaining
Maintaining an upright position. 2. an upright position while eating is
Restricting the diet to liquids until appropriate because it minimizes the risk
swallowing improves. 3. Introducing of aspiration. Introducing foods on the
foods on the unaffected side of the unaffected side allows the client to have
mouth. 4. Keeping distractions to a better control over the food bolus. The
minimum. client should concentrate on chewing and
swallowing; therefore, distractions should
be avoided.
86 Thoughts on pathogenesis in Thalamocortical circuits going "haywire"
4. generalized epilepsy eg. seizure due to cocaine use (hits
throughout the brain)
86 The three common conditions Delirium, depression, and dementia
5. affecting cognition in older adults
are:
86 To determine whether a new patient's Use the Confusion Assessment Method tool
6. confusion is caused by dementia or to assess the patient.
delirium, which action should the
nurse take? rational: The Confusion Assessment
Method tool has been extensively tested in
assessing delirium. The other actions will
be helpful in determining cognitive
function or risk factors for dementia or
delirium, but they will not be useful in
differentiating between dementia and
delirium.
86 To encourage adequate nutritional stay with the client and encourage him to
7. intake for a client with Alzheimer's eat.
disease, the nurse should:
86 To enhance effectiveness in teaching Speak in clear, low-pitched tones
8. the older adult, the nurse should:
86 To evaluate a client's cranial nerve gag reflex.
9. function, the nurse should assess:
87 To evaluate the effectiveness of IV Leg strength and sensation
0. methylprednisolone (Solu-Medrol)
given to a patient with a T4 spinal rational: The purpose of
cord injury, which information is most methylprednisolone administration is to
important for the nurse to obtain? help preserve motor function and
sensation. Therefore the nurse will assess
this patient for lower extremity function.
The other data also will be collected by the
nurse, but they do not reflect the
effectiveness of the methylprednisolone.
87 To help reduce confusion of the older Make telephone calls to friends or family
1. adult at night, the nurse may members to let the older adult hear
reassuring voices.
87 To meet the psychosocial needs of Use appropriate therapeutic touch
2. the older adult, the nurse may:
87 Topiramate (Topamax) Approved for:
3. -Focal
-Generalized tonic-clonic
-Lennox-Gastaut
Side effects:
-Sedation, kidney stones, mental dulling,
weight loss
87 Topirimate Can cause acidosis so increased risk of
4. kidney stones; wt loss (popular) BUT can
cause memory loss esp words
87 To protect a patient from injury assign a nursing assistant to stay with the
5. during an episode of delirium, the patient and offer frequent reorientation.
most appropriate action by the nurse
is to ______________ rational: The priority goal is to protect the
patient from harm, and a staff member will
be most experienced in providing safe
care. Visits by family members are helpful
in reorienting the patient, but families
should not be responsible for protecting
patients from injury. Antipsychotic
medications may be ordered, but only if
other measures are not effective because
these medications have multiple side
effects. Restraints are sometimes used but
tend to increase agitation and
disorientation.
87 Transfer data for a client brought by 1, 2, 5.
6. ambulance to the hospital's Confused clients need fewer choices,
psychiatric unit from a nursing acceptance as a person, and step-by-step
home indicate that the client has directions. Allowing the client to do as he
become increasingly confused and wishes can lead to substandard care and the
disoriented. The client's behavior is risk of harm. Acting nonchalantly conveys a
found to be the result of cerebral lack of caring.
arteriosclerosis. Which of the
following behaviors of the nursing
staff should positively influence the
client's behavior? Select all that
apply.
1. Limiting the client's choices.
2. Accepting the client as he is.
3. Allowing the client to do as he
wishes.
4. Acting nonchalantly.
5. Explaining to the client what he
needs to do step-by-step.
87 Treatment of Glaucoma Parasympathomimetics (mimics
7. parasympathetic- rest/relaxation syndrome):
Pilocarpine: this drug enhances papillary
constriction they are myotic drops. Adverse
reactions: Bronchospasm. N/V, diarrhea.
Blurred vision, twitching eyelids, eye pain
with focusing. Nursing Implications: Use
cautiously with pregnancy, asthma,
hypertension. Teach proper drop instillation
technique. Need for ongoing use of the drug
at precribed intervals.** Blurred vision
tends to decrease with regular use of
this drug.**
Beta-Adrenergic Receptor Blocking Agents:
Timolol/ Carteolol: Inhibits formation of
aqueous humor. Adverse Reactions: Side
effects are insignificant. Hypotension.
Nursing Implications: use cautiously with-
hypersensitivity, Asthma, Second or third-
degree heart block, HF, Congenital
glaucoma, Pregnancy. Teach proper drop
instillation technique. Need for ongoing use
of the drug at prescribed intervals. Blurred
vision tends to decrease with regular use of
this drug.
Carbonic Anhydrase Inhibitors: Diamox-PO:
reduces aqueous humor production. Adverse
Reactions: numbness, tingling of hands and
feet. Nausea and Malaise. Nursing
Implications: Administer orally or IV. Produces
diuresis. Assess for metabolic acidosis.
Prostaglandin Antagonists: Lumigan: lowers
IOP of gluacoma by increasing outflow of
aqueous humor. Adverse Reactions: Local
irritation. Foreign-body sensation. Increased
brown pigmentation of iris. Increased eyelash
growth.
87 trigeminal neuralgia CN V disorder
8. severe face pain along nerve (lips, gum,
nose, cheek)
avoid hot, cold foods/fluids

valium
dilantin
tegretol
baclofen
87 Tuning fork hearing tests measure A. air/bone conduction
9. hearing by which of the following
mechanisms?
A. air/bone conduction
B. bone/vestibular conduction
C. air/water conduction
D. bone/water conduction
88 Two types of seizures (define) 1. Focal - initial activation in 1 part of brain
0. (EEG and clinical)
2. Generalized - initial involvement of both
hemispheres
88 Types of generalized seizures Absence (formerly called "petit mal"): staring
1. for a few seconds and unresponsive
Myoclonic: quick jerks
Clonic: rhythmic jerking (eg. dorsiflex the
ankle and it jerks)
Tonic: stiffening (sustained posture)
Tonic-clonic: stiffening jerking (start tonic
eg. pt w/ seizures in the hospitalist shadowing)
Atonic: brief loss of muscle tone
88 An unconscious patient has a Keep the head of the bed elevated to 30
2. nursing diagnosis of ineffective degrees.
cerebral tissue perfusion related
to cerebral tissue swelling. Which rational: The patient with increased intracranial
nursing intervention will be pressure (ICP) should be maintained in the
included in the plan of care? head-up position to help reduce ICP. Flexion of
the hips and knees increases abdominal
pressure, which increases ICP. Because the
stimulation associated with nursing
interventions increases ICP, clustering
interventions will progressively elevate ICP.
Coughing increases intrathoracic pressure and
ICP.
88 An unconscious patient receiving Correct Answer: 1,2,5
3. emergency care following an Rationale: In the emergency setting, all
automobile crash accident has a patients who have sustained a trauma to the
possible spinal cord injury. What head or spine, or are unconscious should be
guidelines for emergency care treated as though they have a spinal cord
will be followed? injury. Immobilizing the neck, maintaining a
Select all that apply. supine position and securing the patient's head
1. Immobilize the neck using to prevent movement are all basic guidelines of
rolled towels or a cervical collar. emergency care. Placement on the ventilator
2. The patient will be placed in a and raising the head of the bed will be
supine position considered after admittance to the hospital.
3. The patient will be placed on a
ventilator.
4. The head of the bed will be
elevated.
5. The patient's head will be
secured with a belt or tape
secured to the stretcher.
88 An unconscious patient with a 72 mm Hg
4. traumatic head injury has a blood (The formula for calculation of cerebral
pressure of 126/72 mm Hg, and perfusion pressure is [(Systolic pressure +
an intracranial pressure of 18 mm Diastolic blood pressure 2)/3] = intracranial
Hg. The nurse will calculate the pressure.)
cerebral perfusion pressure as
____________________.
88 Vagus Nerve Stimulation Stimulator placed in the body below clavicle
5. - what it is and lead connected to vagus nerve -->
- effectiveness provides intermittent stimulation --> battery
lasts 6 to 10 years

Effectiveness / Outcome
-30-50% of patients with reduction in seizure
frequency
-4-10% seizure free
-About equal to AED efficacy, but no systemic
side-effects
88 Valproic acid Partial, generalized seizure
6.
Broad spectrum AED that is also good for h/a
but has s/e related to weight gain, tremors,
alopecia
88 Valproic acid (Depakote, Approved for:
7. Depacon) -Focal
-Absence (long-term)
Side effects:
-Tremor, weight gain, nausea, sedation,
hepatotoxicity, thrombocytopenia, hair loss,
pancreatitis, neural tube defects
88 Ventral Spinal Nerves Function Motor
8.

88 Vigabatrin (Sabril) Approved for:


9. -Infantile spasms
Side effects:
-Irreversible visual field constriction, headache, somnolence,
dizziness, ataxia, weight gain
89 Visual acuity declines The ability of the eyes to accommodate for close, detailed
0. with age. Presbyopia work
is a progressive
decline in:
89 Weber Test place the base of the fork on the midline of the patients skull
1.

89 West syndrome Generalized seizure disorder of infants


2. -Recurrent spasms, EEG pattern of hypsarrhythmia,
retardation
-Associated with tuberous sclerosis
-Tx: ACTH
89 What 3 kinds of 1) Occupational therapy, activities of daily living
3. therapy will someone 2) Physical therapy, ambulation
with a post CVA 3) Speech therapy, speech and swallowing
receive?
89 What are CVA risk Smoking, obesity, increased salt intake, sedentary lifestyle,
4. factors that are increased stress, and birth control pills.
reversible?
89 What are Used for alternative ways to handle ADLs (dress in slip on
5. occupational clothes and shoes, no buttons). Also, how to use utensils to
therapist used for in eat.
PD patients
89 What are physical Teach safety! Patient learns to walk again when shuffling
6. therapist used for in (draw imaginary life and have them follow it. Life entire foot
PD patients and do "u-turn"). Exercise very important, helps them gain
control again (swimming, biking, yoga, dance).
89 What are symptoms 1) Sudden weakness or numbness of the face, arm and leg,
7. of stroke? usually on one side of the body.
2) Difficulty talking or understanding speech
3) Dimness or loss of vision on one eye
4) Unexplained dizziness and unsteadiness
5) Falls
6) Sudden severe headache
89 What are the 4 areas 1. Cervical
8. of the spinal cord? 2. Thoracic
3. Lumbar
4. Sacral
89 What are the adverse Gingival hyperplasia, GI disturbances, heptoxicity, ataxia,
9. reactions from hypocalcemia, and decrease in vitamin D absorption
medication?
90 What are the causes 1) Atherosclerosis
0. of CVA? 2) Thrombosis
3) Embolism
4) Cerebral hemorrhage (due to trauma or tissue damage)
90 What are the Parasympathomimetic for pupillary constriction; beta-
1. classifications of the adrenergic receptor-blocking agents to inhibit formation of
commonly prescribed aqueous humor; carbonic anhydrase inhibitors to reduce
eye drops for aqueous humor production; and prostaglandin agonists to
glaucoma? increase aqueous humor outflow.
90 What are the common GI irritation, tinnitus, thrombocytopenia, mild liver enzyme
2. side effects of elevation.
salicylates?
90 What are the five 1- unilateral shaking or tremor of one limb
3. stages of 2- Bilateral limb involvement occurs making walking and
involvement in PD balance difficult
patients? 3-Physical movements slow down significantly, affecting
walking more
4-Tremors may decrease but akinesia and rigidity make daily
to day tasks difficult
5-client unable to stand or walk, is dependent for all care and
90 What are the goals of Locate site of bleeding and repair the damage. Also
4. treatment for a manage complications (hydrocephalus, hyponatremia,
subarachnoid seizures, and increased ICP)
hemorrhage?
90 What are the immediate Notify physician stat, draw blood gases, administer O2
5. nursing actions if fat according to blood gas results, assist with endotracheal
embolization is suspected intubation and treatment of respiratory failure.
in a client with a fracture
or other orthopedic
condition?
90 What are the non- Sex, age, race and heredity
6. reversible risk factors for
CVA?
90 What are the nursing 1) Airway and oxygenation
7. goals related to CVA care? 2) Decreased ICP
3) Nutrition
4) Preserve function
5) Safety
6) Rehabilitation
7) Education
90 What are the people with Respiratory Distress
8. myasthenia gravis always
at risk for?
90 What are the proper steps 1. Wash hands
9. for obtaining a wound 2. Use disposable gloves
drainage specimen? 3. Remove old dressing
4. Apply sterile gloves
5. Assess wound appearance (color, odor, et. al)
6. Irrigate wound with NaCl solution
7. Absorb excess with sterile gauze
8. Remove culture tube
9. Gently roll swab over granulation tissue
10. Replace in tube
11. Crush ampule of medium in tube.
12. Remove gloves
13. Apply sterile gloves
14. Dress wound
15. Label specimen
16. Arrange transport to laboratory
91 What are the risk factors HTN, smoking, alcohol, or cocaine use and pregnancy
0. for a subarachnoid
hemorrhage?
91 What are the risk factors Familial over age 40, diabetes, and history of ocular
1. for GLAUCOMA? problems.
91 What are the risk factors More common among the 20-50 age group, associated
2. for Guillain Barre with swine flu immunizations, frequently preceded by
Syndrome? mild respiratory or intestinal infection.
91 What are the risk factors caucasians
3. for PD (7) more common in men
average age onset 60
Age 40-70
Genetics
Exposure to toxins
91 What are the signs and Flu-like symptoms, HA, nausea, decreased appetite,
4. symptoms of bacterial aching joints, slight fever, vomiting, stiff neck, positive
(meningococcal) Brudzinski's sign, involuntary flexion of knee/hip when
meningitis? neck flexed, and positive Kernig's sign, inability to
extend leg from 90 degress flexion of hip when supine
because it causes pain. Brudzinski and Kernigs are
always abnormal
91 What are the signs and Abnormal neurovascular assessment: cold extremity,
5. symptoms of severe pain, inability to move the extremity, and poor
compartment syndrome? capillary refill.
91 What are the signs and 1. vertigo
6. symptoms of MENIERE's 2. tinnitus
disease? 3. unilateral nerve deafness
91 What are the signs and 1. tinnitus
7. symptoms of multiple 2. decreased hearing
sclerosis? 3. urinary retention
4. spastic bladder
5. constipation
6. nystagmus
7. diplopia
8. blurred vision
9. dysarthria
10. dysphagia
11. numbness
12. tingling
13. weakness
14. paralysis
15. muscle spasticity
16. ataxia
17. vertigo
91 What are the signs and 1. bobblehead
8. symptoms of MYASTHENIA 2. ptosis
GRAVIS? 3. diplopia
4. slow speech
5. frowning
6. decreased tongue movement
7. drooling
8. pupils slowly react
9. increased frowning
10. decreased chewing
91 What are the signs and 1. Mask like blank expression
9. symptoms of Parkinson's 2. pill rolling
Disease? 3. Shuffling gait
4. propulsive gait
5. tremors
6. bradykinesia
7. loss of ability to swallow
8. decreased blinking
9. muscle rigidity
92 What are the signs and Myopia, nearsightedness, seeing flashing spots, recent
0. symptoms of risk factors intraocular eye surgery, direct eye trauma, peripheral
for retinal detachment? retinal holes, or history of retinal degeneration.
92 What are the steps for 1) Clean wound with Saline solution
1. applying a negative 2) Clean and ry area around wound
pressure system? 3) Trim sponge to fit entire wound bed but not
overflow
4) Fill wound with sponge
5) Apply adherent ,transparent drape over wound to
secure sponge, cut hole in drape over wound to secure
sponge, cut hole in drape about 2 cm.
6) Insert end of tubing and secure
92 What are the symptoms 1. Exploding headache
2. of subarachnoid 2. Loss of consciousness
hemorrhage? 3. Seizure
4. Sluggish pupil reaction
5. Photophobia
6. Elevated blood pressure, heart rate, respiration
7. One sided weakness
8. Lethargic
92 What are the three types 1) Flexor (Decorticate - toward core)
3. of abnormal posturing? 2) Extenser (Decerebrate) and
3) Flaccid
92 What are the two types of 1. Primary open-angle = chronic
4. GLAUCOMA? 2. Closed angle = acute and painful
92 What care is indicated for Prescribed exercise to maintain mobility; splinting of
5. a child with juvenile affected joints; and teaching about medication
rheumatoid arthritis? management and side effects of drugs.
92 What causes tremors? The globus thalamus and the hypothalamus become
6. over active and diminish dopamine and motor
movement that cause tremors
92 What discharge Check clid's circulation. Keep cast dry. Do not place
7. instructions should be anything under cast. Prevent cast soilage during
included concerning a toileting or diapering. Do not turn child using an
child with a spica cast? abductor bar.
92 What does hemiplegia One half of the body is paralyzed. Cranial nerves are
8. mean? What are the involved so at risk for aspiration.
considerations?
92 What does paraplegia Paralysis from waist down. Patient can insert own
9. mean? What are the suppository and roll hands over bladder to facilitate.
considerations?
93 What does quadriplegia Paralysis from shoulders down. Broken cervical area, C2-
0. mean? What are the 4. Need vent because diaphragm innervation is lost.
considerations? Bowel care by RN is necessary.
93 What drug is commonly LEVODOPA. Watch for hypotension and place TED hose
1. taken to help Parkinson's on your patient to prevent further complications.
symptoms? What are the
Considerations?
93 What drug is used by PROCARDIA to decrease BP. Prick and squeeze under
2. someone experiencing tongue. Works quickly and raise head of bed.
autonomic dysreflexia
symptoms at home?
93 What drugs help decrease 1. DIAMOX - decreases production of fluid
3. the fluid in the eyeball 2. MANNITOL - draws fluid out of the eyeball
and therefore the
intraocular pressure?
93 What drug should never ATROPINE
4. be given to someone with
Glaucoma?
93 What if you end up with a Put a dressing over both eyes and go to the ER. If
5. foreign body in your eye? chemical, then you should flush eyes first then head to
And what if it were a the ER.
chemical?
93 What is a cataract? A condition in which the lens of the eye becomes
6. opaque or cloudy and making vision difficult. It is usually
a slow gradual loss of vision in the affected eye. They
are common in elderly but may also be caused by some
drugs like corticosteroids.
93 What is AMYOTROPHIC Also known as Lou Gehrig's Disease, it is a
7. LATERAL SCLEROSIS? What is progressive neurological disease characterized by
important to know about it? neuron death resulting in muscle weakness and
eventually paralysis. The patient will die when
respiratory paralysis reaches diaphragm so
discuss advance directives and make sure to
check gag reflex before feeding this patient.
93 What is an appropriate Since flexed, roll washcloth and place between
8. intervention to provide safety hands to prevent fingernails from embedding into
in someone with decorticate palms because tension is so tight.
posturing?
93 What is an aura? It is a foreknowledge that a seizure is going to
9. occur
94 What is a normal deep tendon +2; 0 = absent; and +4 = clonic
0. reflex?
94 What is a normal eye pressure? 22 mm Hg, measured by TONOMETRY
1. What device is used?
94 What is a priority nursing 2. It is crucial to monitor the pupil size and
2. assessment in the first 24 pupillary response to indicate changes around the
hours after admission of the cranial nerves. The cholesterol level is not a
client with a thrombotic priority assessment, although it may be an
stroke? 1. Cholesterol level. 2. assessment to be addressed for long-term healthy
Pupil size and pupillary lifestyle rehabilitation. Bowel sounds need to be
response. 3. Bowel sounds. 4. assessed because an ileus or constipation can
Echocardiogram. develop, but this is not a priority in the first 24
hours, when the primary concerns are cerebral
hemorrhage and increased intracranial pressure.
An echocardiogram is not needed for the client
with a thrombotic stroke without heart problems.
94 What is autonomic dysreflexia? Also known as hyperreflexia, it is a potentially life
3. What is the cause? What are threatening condition involving exaggeration of
the signs and symptoms? the sympathetic response to stimulation. The
condition occurs in people with spinal cord injuries
at T-6 or higher. It is triggered by a sustained
stimuli such as restrictive clothing, pressure areas,
FULL BLADDER, UTI or FECAL IMPACTION.
Signs and symptoms above the level of injury:
1) flushed face
2) increased blood pressure 200/100
3) headache
4) distended neck veins
5) decreased heart rate
6) increased sweating, vasodilation

Signs and symptoms below level of injury:


1. Pale
2. Cool
3. No sweating, vasoconstriction
94 What is BELLS PALSY? What Bell's palsy is a form of facial paralysis resulting
4. are the considerations? from a dysfunction of the cranial nerve VII (the
facial nerve) causing an inability to control facial
muscles on the affected side. The biggest
complications are pain and an eyelid that won't
shut. To protect the eyes from corneal abrasions,
use drops and eye patches at night or tape shut to
protect.
94 What is characteristic of a left Right sided weakness and aphasia
5. hemisphere CVA?
94 What is characteristic of a right Left sided weakness and perceptual deficits
6. hemisphere CVA? (vulnerable to accidents)
94 What is characteristic of a Full thickness skin loss down through the dermis
7. Stage III pressure ulcer? which may include subcutaneous tissue, eschar =
dead tissue
94 What is characteristic of a Epidermis is broken, lesion is superficial and there
8. Stage II pressure ulcer? is partial thickness skin loss, blister
94 What is characteristic of a Nonblanchable erythema, redness, that remains
9. Stage I pressure ulcer? red 30 minutes after pressure has been relieved.
Epidermis remains intact.
95 What is characteristic Full thickness skin loss extending into supportive
0. of a Stage IV pressure structures, such as muscle, tendon, and bone.
ulcer?
95 What is characteristic Gradual loss of peripheral vision - tunnel. Generally,
1. of open angle PAINLESS, and blindness can result if not treated
glaucoma?
95 What is compartment Damage to nerves and vasculature of an extremity due to
2. syndrome? compression.
95 What is DILANTIN used Seizure prevention
3. for?
95 What is ELASE? An enzyme that dissolves eschar. Treatment is effective
4. since good blood supplies an area of healing
95 What is GLAUCOMA? A condition characterized by increased intraocular pressure
5. and progressive vision loss.
95 What is Guillain Barre An autoimmune disease of the nervous system due to
6. Syndrome? damage of myelin sheath around the nerves., progresses
rapidly or over 2-3 weeks, characterized by muscle
weakness or symmetrical paralysis. Pig Head. Also,
Landry's paralysis, an acute polyneuropathy affecting the
PNS. The most typical symptoms cause change in sensation
or pain, as well as dysfunction of the ANS. It can cause
complications, in particularly in the respiratory muscles if
the ANS is involved. It is usually triggered by an infection.
95 What is HOMONYMOUS A pupillary abnormality associated with CVA where vision is
7. HEMIANOPSIA? lost on the same side of each eye so when driving must
turn head to see (CN II is affected)
95 What is important to It is when bones from the inner ear are removed and
8. know about replaced with artificial ones. Swelling and noise reduction is
STAPEDECTOMY? normal for 1 year. Sleep on non-surgical side and sneeze
with mouth open
95 What is important to DO NOT TAKE, because it causes birth defects.
9. teach pregnant women
about DILANTIN?
96 What is infratentorial 1) It is an incision made at nape of neck.
0. cranial surgical 2) Maintain neck in straight alignment and position patient
approach? What are on either side, not on back
the interventions
involved?
96 What is MENIERE's It is called 'my ears disease'. An idiopathic chronic
1. Disease? condition associated with water in the ear.
96 What is motor aphasia? Also known as EXPRESSIVE APHASIA, Broca's area, it is the
2. inability to speak or write. However, patient can
comprehend the spoken or written form of communication.
96 What is multiple It is a progressive neurological disease with an onset
3. sclerosis? among those who are at their 20's through 40's. It has a
hereditary link and occurs most commonly among women.
It involves the hardening of multiple nerves, and is
aggravated by stress. It limits changes.
96 What is myasthenia It is an autoimmune neuromuscular disease that affects the
4. gravis? motor cranial nerves. It is the GRAVE MUSCLE WEAKNESS.
Exacerbation and remissions are parts of the disease which
tend to be progressive over time.
96 What is negative The wound is filled with a sponge and a vacuum is applied
5. pressure wound care? via a closed system to remove stagnant fluid and
decompress interstitial space.
96 What is PARKINSON'S It is a progressive neurological disease with a slow onset that
6. DISEASE? usually occurs after age 50, rarely occurring in the black
population, and leading to a respiratory death. The motor
symptoms of Parkinson's disease result from the death of
dopamine-generating cells in the substantia nigra, a region of
the midbrain; the cause of this cell death is unknown.
96 What is retinal A small horseshoe shaped tear that allows sub retinal fluid to
7. detachment? leak between the retinal pigment epithelium and the layer of
rods and cones. This leakage splits the retinas outer layer,
pulling it away from its blood supply.
96 What is sensory Also known as RECEPTIVE APHASIA, a patient cannot
8. aphasia? understand oral or written forms of communication.
96 What is speech When a patient is at risk for aspiration, swallow education,
9. therapy used for in dysphagia diet
patients with PD
97 What is status Defn: Recurrent seizures that aren't stopping; If seizure
0. epilepticus? persists for more than 5 minutes, likely to continue - this is
status epilepticus
Most seizures resolve spontaneously in 45-90 seconds; no
acute treatment necessary (except to turn patient on side if
possible to reduce aspiration risk) - DON'T put tongue blade
or anything else in mouth
97 What is 1) It is an incision behind the hairline
1. supratentorial cranial 2) Main head of bed at 30-45 degrees with neck in neutral
surgical approach? alignment.
What are the 3) Position patient on either side or back
interventions
involved?
97 What is the cause of Rupture of a blood vessel or an intracranial aneurysm,
2. subarachnoid typically at circle of Willis or major cranial artery branches
hemorrhage? and commonly after head trauma
97 What is the correct 1. Tell the patient each time you enter the room and say your
3. way to help a patient name. 2. Always explain what you are going to do
who is visually 3. When walking, ask which side they prefer you on, then
impaired? offer your arm for them to grasp
4. Position self on half step beside and ahead of them
97 What is the correct Functioning vs. affected side CVA. Place wheel chair on the
4. way to help someone patient's strong side, lock the wheels, elevate the head of
with a CVA get out of bed, and stand on the patient's strong side to assist
bed?
97 What is the correct 1. Check order
5. way to instill 2. Warm medication to body temperature
eardrops in a 3. Wash your hands and wear clean gloves
sequential order? 4. Check patient ID and then explain the process
5. Have patient lie on side with affected ear up
6. Straighten ear canal by pulling auricle up and back
7. Gently clean drainage with tissue paper or cotton tipped
applicator.
8. Support hand hold dropper against head
9. Hold dropper 1/2 inch above ear, aim at canal and
administer. Administer water based drops before oil based
10. Gently massage ear to move meds.
11. Lie on side for 5-10 minutes and tuck cotton ball into ear
12. After 15 minutes, remove and dry ear.
97 What is the correct 1. Thumb of nondominant hand raise upper eyelid.
6. way to reinsert an 2. Dominant hand grasp the eyeball so that indented part is
artificial eye? facing the patient's nose and slide it in as far as possible.
3. Depress the lower lid and pull forward to cover prosthesis.
97 What is the correct 1. Using your dominant hand, raise the upper eyelid with
7. way to remove an thumb.
artificial eye? 2. Cup nondominant hand under clients lower lid
3. Apply slight pressure with index finger between brow and
artificial eye and remove
97 What is the eighth Vestibular cochlear. Sensory. Balance, hearing acuity.
8. cranial nerve? Normal
response?
97 What is the eleventh, cranial Spinal accessory. Motor. Push chin against hand
9. nerve? Normal response? and shoulder shrug
98 What is the expected outcome 3. Thrombolytic enzyme agents are used for
0. of thrombolytic drug therapy for clients with a thrombotic stroke to dissolve
stroke? 1. Increased vascular emboli, thus reestablishing cerebral perfusion.
permeability. 2. They do not increase vascular permeability,
Vasoconstriction. 3. Dissolved cause vasoconstriction, or prevent further
emboli. 4. Prevention of hemorrhage.
hemorrhage.
98 What is the fifth? Trigeminal. Signs and manifestation. clench teeth
1.

98 What is the first cranial nerve? Olfactory. Sensory. Should be able to identify
2. Normal response? familiar odors.
98 What is the fourth cranial Trochlear. Motor. Same as three: EOM,
3. nerve? Normal response constriction
98 What is the function of It cushions the brain and spinal cord.
4. cerebrospinal fluid (CSF)?
98 What is the hallmark symptom SEVERE HEADACHE THAT COMES SUDDENLY
5. of subarachnoid hemorrhage?
98 What is the medical treatment DILANTIN or TEGRETOL. For surgical, local nerve
6. for TRIGEMINAL NEURALGIA? blocks or slow nerve transmission to decrease
What is the surgical treatment? pain.
98 What is the mortality rate 40% mortality rate
7. associated with subarachnoid
hemorrhage?
98 What is the ninth cranial nerve? Glossopharyngeal. S/M. Identify taste, gag reflex,
8. Normal response? uvula rise with "ahh"
98 What is the only treatment for Surgical removal
9. cataracts?
99 What is the primary goal 1. The primary goal of physical therapy and
0. collaboratively established by nursing interventions is to maintain joint
the client with Parkinson's flexibility and muscle strength. Parkinson's
disease, nurse, and physical disease involves a degeneration of dopamine-
therapist? 1. To maintain joint producing neurons; therefore, it would be an
flexibility. 2. To build muscle unrealistic goal to attempt to build muscles or
strength. 3. To improve muscle increase endurance. The decrease in dopamine
endurance. 4. To reduce ataxia. neurotransmitters results in ataxia secondary to
extrapyramidal motor system effects. Attempts
to reduce ataxia through physical therapy would
not be effective.
99 What is the priority nursing 3. A priority for the client in the postictal phase
1. intervention in the postictal (after a seizure) is to assess the client's
phase of a seizure? 1. Reorient breathing pattern for effective rate, rhythm, and
the client to time, person, and depth. The nurse should apply oxygen and
place. 2. Determine the client's ventilation to the client as appropriate. Other
level of sleepiness. 3. Assess interventions, to be completed after the airway
the client's breathing pattern. 4. has been established, include reorientation of the
Position the client comfortably. client to time, person, and place. Determining
the client's level of sleepiness is useful, but it is
not a priority. Positioning the client comfortably
promotes rest but is of less importance than
ascertaining that the airway is patent.
99 What is the priority nursing Administer or teach client to take drugs with food
2. intervention used with clients or milk.
taking NSAIDs?
99 What is the proper way to 1. Get your patient's attention
3. converse with hearing impaired 2. Ask permission to turn off television and
patients? reduce noise
3. Face patient at eye level
4. Make sure light is on face
5. If patient has hearing aid, make sure it is in
and turned on and functioning properly.
6. Use simple sentences, speak slowly and avoid
shouting.
7. Supplement with gestures
8. Avoid jokes, slang and ask for oral feedback to
assess understanding.
99 What is the proper way to give IV Push: (not compatible with IV solutions), give
4. DILANTIN? closest insertion site, flush/push/flush
995. What is the purpose of a When used as an adjunctive therapy it can reduce the
TENS system? amount of pain meds a patient needs for relief. It is
battery powered and allows the patient to administer
therapy as needed.
996. What is the second Optic. Sensory. Snellen chart and light reflex
cranial nerve? Normal
response?
997. What is the seventh Facial. S/M. Smile, frown, puff cheeks, saliva presence
cranial nerve? Normal
response?
998. What is the short term Short term is TENSILON. Long term treatment is
treatment for diagnosis PROSTIGMIN, an airway medication and MESTINON.
of myasthenia gravis?
And for long term?
999. What is the sixth cranial Abducens. Motor. Look to the right and left
nerve? Normal
response?
100 What is the tenth cranial Vagus. S/M. Same as IX (gag, uvula rise, taste) and draw
0. nerve? Normal line toward umbilicus
response?
100 What is the therapeutic "Dial at Ten" 10-20 = therapeutic level
1. level of DILANTIN?
100 What is the third cranial Oculomotor. Motor. EOM, penlight = constriction
2. nerve? Normal
response?
100 What is the treatment 1. Decreased sodium in diet
3. for Meniere's disease? 2. Stop smoking
3. Benadryl, atropine, Dramamine and
4. Lasix to decrease water in ear.
100 What is the twelfth Hypoglossal. Motor. Move tongue side to side and against
4. cranial nerve? Normal tongue depressor.
response?
100 What is transphenoidal 1) It is an incision made beneath upper lip to gain access
5. cranial surgical into nasal cavity
approach? What are the 2) Instruct patient to avoid blowing his nose and keep
interventions involved? head of bed elevated to promote venous drainage from
surgical site
100 What is TRIGEMINAL A cranial nerve disorder affecting sensory branches of
6. NEURALGIA? What are the trigeminal nerve (CN V). Lukewarm food, chew on
the considerations? unaffected side, eye care, tearing, blinking, oral hygiene,
increased protein, calories, room temperature and avoid
touching client.
100 What is true about the There is no single theory that explains aging.
7. theories of aging?
100 What items come in the 1) Wound vacuum unit
8. negative pressure sterile 2) Sterile foam sponge kit
kit? 3) Measuring tool
4) NaCl
5) Skin prep
6) Gloves and PPE
7) A pad, towel and gauze pads
100 What kind of death does Respiratory. Watch for hypoxia, restlessness, and
9. a person with multiple agitation.
sclerosis normally die
from?
101 What kind of diet should High protein and high Vitamin C, e.g. custard and
0. someone with a post strawberries, to assist with pressure ulcer prevention or
CVA be on? healing.
101 What kind of shoes Shoes with 1 thread so they can slide through lie. No
1. should you recommend tennis shoes or leather bottom shoes. Slippers are good.
for a patient with
Parkinson's Disease?
101 What kinds of food or Spinach, bananas, fish and pepper. Also, Vitamin B6,
2. vitamins should because they all deactivate LEVODOPA, the precursor to
someone with the neurotransmitters dopamine, norepinephrine
Parkinson's avoid? Why? (noradrenaline), and epinephrine (adrenaline) collectively
known as catecholamines
101 What lid is the correct way to 1. Offer tissue paper. Make sure patient has
3. administer Eyedrops? What is the no contact lenses.
sequential order? 2. Do hand hygiene, wear gloves.
3. Clean eyes with gauze or moistened
cotton balls
4. Ask patient to tilt head back
5. Turn head slightly to treated side
6. Pull lower lid down with thumb.
7. Place bottle 1/2 to 3/4 inches from above
the conjunctival sac
8. Administer dose, ask to gently close eyes
and move them while closed to help
distribute medication.
9. Do not squeeze, shut or rub eyes.
10. Press on inner canthus for about 30
seconds to slow drainage of medication.
11. Use tissue paper to remove excess
medication.
12. Give water drops before oil and leave
for 5 minutes between eyedrop
administration.
101 What makes someone with Exercise and Infection
4. myasthenia gravis worse?
101 What measures should the nurse Possible estrogen replacement after
5. encourage female clients to take to menopause; high calcium and vitamin D
prevent osteoporosis? intake beginning in early adulthood;
calcium supplements after menopause; and
weight-bearing exercise.
101 What nursing assessments should 4. During a seizure, the nurse should note
6. be documented at the beginning of movement of the client's head and eyes
the ictal phase of a seizure? 1. Heart and muscle rigidity, especially when the
rate, respirations, pulse oximeter, seizure first begins, to obtain clues about
and blood pressure. 2. Last dose of the location of the trigger focus in the
anticonvulsant and circumstances at brain. Other important assessments would
the time. 3. Type of visual, auditory, include noting the progression and duration
and olfactory aura the client of the seizure, respiratory status, loss of
experienced. 4. Movement of the consciousness, pupil size, and incontinence
head and eyes and muscle rigidity. of urine and stool. It is typically not possible
to assess the client's pulse and blood
pressure during a tonic-clonic seizure
because the muscle contractions make
assessment difficult to impossible. The last
dose of anticonvulsant medication can be
evaluated later. The nurse should focus on
maintaining an open airway, preventing
injury to the client, and assessing the onset
and progression of the seizure to determine
the type of brain activity involved. The type
of aura should be assessed in the preictal
phase of the seizure.
101 What nursing interventions should 1) Stand in front of client
7. be implemented for someone with 2) Speak clearly, slowly.
aphasia? 3) Do not shout or speak loudly. They can
hear.
4) Be patient and give client time to
respond
5) Use nonverbal communication, e.g.
touche, smile
6) Assist client with motor aphasia to
practice simple words,
7) Listen carefully
8) Provide simple directions
9) Involve family in practice
10) Show picture cards to help convey a
message
101 What occurs in the post-ictal period? Patient sleeps for several hours after
8. seizure. Do not call physician because this
is expected.
101 What population has the highest risk Black males (most risk), white males, black
9. for stroke? females, then white females (least risk).
102 What should a diet for someone with Increased protein and vitamin
0. a pressure ulcer consist of? C:strawberries, custard, orange juice and
tomato juice
102 What should be done on a post 1. Position face down because gas was
1. retinal detachment surgery? placed in eye
2. Apply ice compresses and provide
analgesics as needed
3. No lifting of anything heavier than 20
lbs.
4. No rubbing of eyes
5. No coughing or suctioning
6. Needs 2 eye patches at night. 2 words =
2 patches
102 What should someone with a CVA do Keep joints fully extended without bends.
2. to prevent contractures?
102 What should the nurse do when Apply pressure on the inner canthus to
3. administering pilocarpine (Pilocar)? prevent systemic absorption.
102 What should you discuss with your Why use it, how it works, and how
4. patient regarding negative pressure frequently dressings need to be changed.
wound care? Explain that dressing changes will be
uncomfortable but that the discomfort
should end as soon as negative pressure is
applied.
102 What should you document for a The time it occurred and lasted and what
5. seizure occurrence? parts of the body were affected.
102 What should you do for excessive Wash and dry skin and apply protective
6. skin moisture? sealant
102 What should you do if a pregnant Make sure the patient is side lying and not
7. woman starts thrashing in bed? handrails up first because she may aspirate
first
102 What should you do immediately for Administer oxygen, place IV line, do an
8. someone suspected of subarachnoid EKG, once preliminary diagnosis is made:
hemorrhage? prepare patient for cerebral angiogram to
locate site.
102 What should you make sure to Advanced directives because patient will
9. discuss with someone who has eventually end up on vent and the use of a
multiple sclerosis? peak flow meter.
103 What should you teach a patient No straining for bowel movement, no
0. about managing GLAUCOMA? vomiting or sneezing
103 What should you teach a patient on Good oral hygiene and nutrition are
1. DILANTIN? important
103 What should you teach your patient How to apply the electrodes properly. Do
2. about TENS therapy? not apply over eyes, carotid sinus nerves,
or broken/irritated skin and how to operate,
avoid using while sleeping, not without
Doctor's approval.
103 What should you teach your patient They should wear sunglasses when in the
3. for post op care after cataract sun and avoid sneezing or vomiting. If they
surgery? have pain in the eye, they should call the
physician and they should wear 1 eye patch
over the affected eye at night. One word =
one patch.
103 What should you use to clean an Hydrogen peroxide and normal saline.
4. artificial eye?
103 What should you watch for with Used as an anticonvulsant for seizure
5. TEGETROL? prevention. Monitor CBC d/t bone marrow
suppression and watch for infection
103 What things should you question for No suctioning every 2 hours just as needed,
6. a patient with increased ICP if no grouping of activities because patient
ordered? Head of bed elevated 30- needs rest periods, and no ice mattress
45 degrees, suctioning every 2 because shivering can cause increase ICP.
hours, lateral positioning, group Keep Head and neck straight to allow for
activities to allow downtime and ice CSF movement.
mattress for comfort?
103 What things should you question for No suctioning every 2 hours just as needed,
7. a patient with increased ICP if no grouping of activities because patient
ordered? Head of bed elevated 30- needs rest periods, and no ice mattress
45 degrees, suctioning every 2 because shivering can cause increase ICP.
hours, lateral positioning, group
activities to allow downtime and ice
mattress for comfort?
103 What treatment is done for seizure 1) Anticonvulsants: Phebobarbital,
8. patients? Carbamazepine (Tegretol) or Phenytoin
(Dilantin).
2) Evaluate consciousness, safety, avoid
alcohol.
3) Reduce activities that stimulate and
reduce stimuli; no strobe lights because it is
repetitive.
4) Reorient client after seizure.
103 What type of precautions should be Standard precautions
9. used for applying a negative
pressure system?
104 When administering a mental status "I don't know."
0. examination to a patient, the nurse
suspects depression when the rational: Answers such as "I don't know" are
patient responds with ______________ more typical of depression. The response
"Who are those people over there?" is more
typical of the distraction seen in a patient
with delirium. The remaining two answers
are more typical of a patient with dementia.
104 When administering a mental choose a place without distracting
1. status examination to a patient environmental stimuli.
with delirium, the nurse should
_____________ rational: Because overstimulation by
environmental factors can distract the patient
from the task of answering the nurse's
questions, these stimuli should be avoided.
The nurse will not wait to give the examination
because action to correct the delirium should
occur as soon as possible. Reorienting the
patient is not appropriate during the
examination. Antianxiety medications may
increase the patient's delirium.
104 When admitting a patient who judgment changes
2. has a tumor of the right frontal
lobe, the nurse would expect to rational: The frontal lobes control intellectual
find __________ activities such as judgment. Speech is
controlled in the parietal lobe. Weakness and
hemiplegia occur on the contralateral side from
the tumor. Swallowing is controlled by the
brainstem.
104 When admitting a patient with a The patient takes warfarin (Coumadin) daily.
3. possible coup-contracoup injury
after a car accident to the rational: The use of anticoagulants increases
emergency department, the the risk for intracranial hemorrhage and should
nurse obtains the following be immediately reported. The other
information. Which finding is information would not be unusual in a patient
most important to report to the with a head injury who had just arrived to the
health care provider? ED.
104 When a patient is experiencing a unilateral eyelid swelling.
4. cluster headache, the nurse will
plan to assess for _______________ rational: Unilateral eye edema, tearing, and
ptosis are characteristic of cluster headaches.
Nuchal rigidity suggests meningeal irritation,
such as occurs with meningitis. Although
nausea and vomiting may occur with migraine
headaches, projectile vomiting is more
consistent with increases in intracranial
pressure (ICP). Unilateral sharp, stabbing pain,
rather than throbbing pain, is characteristic of
cluster headaches.
104 When a patient's intracranial Oral temperature 101.6 F
5. pressure (ICP) is being monitored
with an intraventricular catheter, rational: Infection is a serious consideration
which information obtained by with ICP monitoring, especially with
the nurse is most important to intraventricular catheters. The temperature
communicate to the health care indicates the need for antibiotics or removal of
provider? the monitor. The ICP, arterial pressure, and
apical pulse are all borderline high but require
only ongoing monitoring at this time.
104 When assessing a patient with Place the patient in a room close to the nurses'
6. Alzheimer's disease (AD) who is station.
being admitted to a long-term
care facility, the nurse learns that rational: Patients at risk for problems with
the patient has had several safety require close supervision. Placing the
episodes of wandering away from patient near the nurse's station will allow
home. Which nursing action will nursing staff to observe the patient more
the nurse include in the plan of closely. The use of "why" questions is
care? frustrating for patients with AD because they
are unable to understand clearly or verbalize
the reason for wandering behaviors. Because
of the patient's short-term memory loss,
reorientation will not help prevent wandering
behavior. Because the patient had wandering
behavior at home, familiar objects will not
prevent wandering.
104 When assessing a patient with a Titrate labetolol (Normodyne) drip to keep BP
7. possible stroke, the nurse finds less than 140/90 mm Hg.
that the patient's aphasia started
3.5 hours previously and the rational: Because elevated BP may be a
blood pressure is 170/92 mm Hg. protective response to maintain cerebral
Which of these orders by the perfusion, antihypertensive therapy is
health care provider should the recommended only if mean arterial pressure
nurse question? (MAP) is >130 mm Hg or systolic pressure is
>220 mm Hg. Fluid intake should be 1500 to
2000 mL daily to maintain cerebral blood flow.
The head of the bed should be elevated to at
least 30 degrees, unless the patient has
symptoms of poor tissue perfusion. tPA may be
administered if the patient meets the other
criteria for tPA use.
104 When assessing a patient with The patient's blood pressure is 86/42 mm Hg.
8. bacterial meningitis, the nurse
obtains the following data. Which rational: Shock is a serious complication of
finding should be reported meningitis, and the patient's low blood
immediately to the health care pressure indicates the need for interventions
provider? such as fluids or vasopressors. Nuchal rigidity
and a positive Kernig's sign are expected with
bacterial meningitis. The nurse should
intervene to lower the temperature, but this is
not as life threatening as the hypotension.
104 When assessing a patient with triggers that lead to facial pain.
9. newly diagnosed trigeminal
neuralgia, the nurse will ask rational: The major clinical manifestation of
the patient about _______________ trigeminal neuralgia is severe facial pain that is
triggered by cutaneous stimulation of the nerve.
Ptosis, loss of taste, and facial weakness are not
characteristics of trigeminal neuralgia.
105 When assessing the older Increase in the time it takes for the heart rate to
0. adult, the nurse should know return to baseline after exercise
which findings represent
common physiological changes
associated with aging and
which are abnormal findings. A
normal and common
physiological change is:
105 When assessing the older Engaging in more introspective, self-focused
1. adult, the nurse should review activities
the client's achievement of
developmental tasks. For the
older adult, these may include
all of the following except:
105 When caring for a client with a Rising blood pressure and bradycardia
2. head injury, the nurse must
stay alert for signs and
symptoms of increased
intracranial pressure (ICP).
Which cardiovascular findings
are late indicators of increased
ICP?
105 When caring for a client with Test the nasal drainage for glucose.
3. head trauma, the nurse notes a
small amount of clear, watery
fluid oozing from the client's
nose. What should the nurse
do?
105 When caring for a client with elevate the head of the bed 90 degrees during
4. the nursing diagnosis Impaired meals.
swallowing related to
neuromuscular impairment, the
nurse should:
105 When caring for a patient Correct Answer: 1
5. admitted post-stroke (CVA) Rationale: The side-lying position is the safest
who has altered consciousness, position to allow adequate drainage of fluids
the nurse should place the without aspiration.
patient in which position?
1. side-lying
2. supine
3. prone
4. semi-Fowler's
105 When caring for a patient who -Urinary catheter care
6. experienced a T1 spinal cord -Continuous cardiac monitoring
transsection 2 days ago, which -Avoidance of cool room temperature
collaborative and nursing -Administration of H2 receptor blockers
actions will the nurse include in
the plan of care: rational: The patient is at risk for bradycardia and
poikilothermia caused by sympathetic nervous
system dysfunction and should have continuous
cardiac monitoring and maintenance of a
relatively warm room temperature.
Gastrointestinal (GI) motility is decreased initially
and NG suctioning is indicated. To avoid bladder
distention, a urinary retention catheter is used
during this acute phase. Stress ulcers are a
common complication but can be avoided
through the use of the H2 receptor blockers such
as famotidine.
105 When caring for a patient who place the hands on the epigastric area and push
7. had a C8 spinal cord injury 10 upward when the patient coughs.
days ago and has a weak cough
effort and loose-sounding rational: Since the cough effort is poor, the initial
secretions, the initial action should be to use assisted coughing
intervention by the nurse should techniques to improve the ability to mobilize
be to ____________ secretions. Administration of oxygen will
improve oxygenation, but the data do not
indicate hypoxemia. The use of the spirometer
may improve respiratory status, but the
patient's ability to take deep breaths is limited
by the loss of intercostal muscle function.
Suctioning may be needed if the patient is
unable to expel secretions by coughing but
should not be the nurse's first action.
105 When caring for a patient who The patient has continuous drooling of saliva.
8. has Guillain-Barr syndrome,
which assessment data obtained rational: Drooling indicates decreased ability to
by the nurse will require the swallow, which places the patient at risk for
most immediate action? aspiration and requires rapid nursing and
collaborative actions such as suctioning and
possible endotracheal intubation. The foot pain
should be treated with appropriate analgesics,
and the BP requires ongoing monitoring, but
these actions are not as urgently needed as
maintenance of respiratory function. Absence of
the reflexes should be documented, but this is a
common finding in Guillain-Barr syndrome.
105 When caring for a patient who The patient is more difficult to arouse.
9. has had a head injury, which
assessment information rational: The change in level of consciousness
requires the most rapid action (LOC) is an indicator of increased intracranial
by the nurse? pressure (ICP) and suggests that action by the
nurse is needed to prevent complications. The
change in BP should be monitored but is not an
indicator of a need for immediate nursing action.
Headache is not unusual in a patient after a
head injury. A slightly irregular apical pulse is
not unusual.
106 When caring for a patient who Assessment of respiratory rate and depth
0. was admitted 24 hours
previously with a C5 spinal cord rational: Edema around the area of injury may
injury, which nursing action has lead to damage above the C4 level, so the
the highest priority? highest priority is assessment of the patient's
respiratory function. The other actions also are
appropriate but are not as important as
assessment of respiratory effort.
106 When caring for a patient with Place objects needed for activities of daily living
1. left-sided homonymous on the patient's right side.
hemianopsia resulting from a
stroke, which intervention rational: During the acute period, the nurse
should the nurse include in the should place objects on the patient's unaffected
plan of care during the acute side. Since there is a visual defect in the left half
period of the stroke? of each eye, an eye patch is not appropriate.
The patient should be approached from the right
side. The visual deficit may not resolve,
although the patient can learn to compensate
for the defect.
106 When caring for the client 3.
2. diagnosed with delirium, which Polypharmacy is much more common in the
condition is the most important elderly. Drug interactions increase the incidence
for the nurse to investigate? of intoxication from prescribed medications,
1. Cancer of any kind. especially with combinations of analgesics,
2. Impaired hearing. digoxin, diuretics, and anticholinergics. With
3. Prescription drug drug intoxication, the onset of the delirium
intoxication. typically is quick. Although cancer, impaired
4. Heart failure. hearing, and heart failure could lead to delirium
in the elderly, the onset would be more gradual.
106 When caring for the older adult, Treat the client as an individual with a unique
3. it is important to: history of his or her own
106 When communicating with a 2. The nurse should encourage the client to
4. client who has aphasia, which of write messages or use alternative forms of
the following nursing communication to avoid frustration. Presenting
interventions is not one thought at a time decreases stimuli that
appropriate? 1. Present one may distract the client, as does speaking in a
thought at a time. 2. Encourage normal volume and tone. The nurse should ask
the client not to write the client to "show me" and should encourage
messages. 3. Speak with normal the use of gestures to assist in getting the
volume. 4. Make use of message across with minimal frustration and
gestures. exhaustion for the client.
106 When communicating with a use short, simple sentences.
5. client who has sensory
(receptive) aphasia, the nurse
should:
106 When communicating with the 3.
6. client who is experiencing Competing and excessive stimuli lead to sensory
dementia and exhibiting overload and confusion. Therefore, the nurse
decreased attention and should first eliminate any distracting stimuli. After
increased confusion, which of this is accomplished, then using touch and
the following interventions rephrasing questions are appropriate. Going for a
should the nurse employ as walk while talking has little benefit on attention
the first step? and confusion.
1. Using gentle touch to
convey empathy.
2. Rephrasing questions the
client doesn't understand.
3. Eliminating distracting
stimuli such as turning off the
television.
4. Asking the client to go for a
walk while talking.
106 When comparing air D. air conduction is normally 2 times as long as
7. conduction vs. bone bone conduction
conduction, which is expected
to occur?

A. bone conduction is
normally 2 times as long as
air conduction

B. bone conduction and air


conduction are equal

C. air conduction is normally 3


times as long as bone
conduction

D. air conduction is normally


2 times as long as bone
conduction
106 When dealing with safety in a Clear away all rugs from the home, make
8. PD patient, what should one environment clutter free, and use electric shaver.
tell them?
106 When developing a plan of Remind the patient frequently about being in the
9. care for a hospitalized patient hospital.
with moderate dementia,
which intervention will the rational: The patient with moderate dementia will
nurse include? have problems with short- and long-term memory
and will need reminding about the hospitalization.
The other interventions would be used for a patient
with severe dementia, who would have difficulty
with swallowing, self-care, and immobility.
107 When developing the plan of 1.
0. care for a client with Considerable assistance is associated with
Alzheimer's disease who is moderate impairment when the client cannot make
experiencing moderate decisions but can follow directions. Managing
impairment, which of the medications is needed even in mild impairment.
following types of care should Constant care is needed in the terminal phase,
the nurse expect to include? when the client cannot follow directions.
1. Prompting and guiding Supervision of shaving is appropriate with mild
activities of daily living. impairment that is, when the client still has
2. Managing a medication motor function but lacks judgment about safety
schedule. issues.
3. Constant supervision and
total care.
4. Supervision of risky
activities such as shaving.
107 When do symptoms occur? When the affected brain cells can no longer
1. perform their normal inhibitory function within the
CNS
107 When evaluating a patient question the patient about social activities with
2. with trigeminal neuralgia family and friends.
who has had a glycerol
rhizotomy, the nurse will rational: Because withdrawal from social activities is
______________ a common manifestation of trigeminal neuralgia,
asking about social activities will help in evaluating
whether the patient's symptoms have improved.
Glycerol rhizotomy does not damage the corneal
reflex or motor functions of the trigeminal nerve, so
there is no need to use an eye shield, do facial
exercises, or take precautions with chewing.
107 When examining the set of D. corner of the eye to the occiput
3. the ear, the top of the pinna
should match an imaginary
line drawn from the

A. tip of the lateral point of


the eyebrow to the occiput

B. tip of the nose to the


crown of the head.

C. tip of the corner of the


mouth to the crown of the
head

D. corner of the eye to the


occiput
107 When family members ask "The monitoring system helps show whether blood
4. the nurse about the purpose flow to the brain is adequate."
of the ventriculostomy
system being used for rational: Short and simple explanations should be
intracranial pressure given to patients and family members. The other
monitoring for a patient, explanations are either too complicated to be easily
which response by the nurse understood or may increase the family member's
is best? anxiety.
107 When helping the families of 1.
5. clients with Alzheimer's The vulgar or sexual behaviors are commonly
disease cope with vulgar or expressions of anger or more sensual needs that
sexual behaviors, which of can be addressed directly. Therefore, the families
the following suggestions is should be encouraged to ignore the behaviors but
most helpful? attempt to identify their purpose. Then the purpose
1. Ignore the behaviors, but can be addressed, possibly leading to a decrease in
try to identify the underlying the behaviors. Because of impaired cognitive
need for the behaviors. function, the client is not likely to be able to process
2. Give feedback on the the inappropriateness of the behaviors if given
inappropriateness of the feedback. Likewise, anger management strategies
behaviors. would be ineffective because the client would
3. Employ anger probably be unable to process the
management strategies. inappropriateness of the behaviors. Risperidone
4. Administer the prescribed (Risperdal) may decrease agitation, but it does not
risperidone (Risperdal). improve social behaviors.
107 When is someone with 24 hours after 1st antibiotic dose. Standard
6. bacterial meningitis no precuations after this point are okay.
longer contagious?
107 When might a deep tendon reflex Pregnant woman or alcoholic on magnesium
7. be less than 2? sulfate
107 When might a deep tendon reflex Seizure patient
8. be more than 2?
107 When obtaining a health history inquire about any urinary tract problems.
9. and physical assessment for a
patient with possible multiple rational: Urinary tract problems with
sclerosis (MS), the nurse should incontinence or retention are common
_____________ symptoms of MS. Chest pain and skin rashes
are not symptoms of MS. A decrease in libido
is common with MS.
108 When obtaining the health history light flashes and floaters in front of the eye.
0. from a client with retinal
detachment, the nurse expects the
client to report:
108 When obtaining the vital signs of a Increased intracranial pressure (ICP)
1. client with multiple traumatic
injuries, the nurse detects
bradycardia, bradypnea, and
systolic hypertension. The nurse
must notify the physician
immediately because these
findings may reflect which
complication?
108 When preparing to admit a patient -Siderail pads
2. who has been treated for status -Oxygen mask
epilepticus in the emergency -Suction tubing
department, which equipment
should the nurse have available in rational: The patient is at risk for further
the room: seizures, and oxygen and suctioning may be
needed after any seizures to clear the airway
and maximize oxygenation. The bed's side
rails should be padded to minimize the risk
for patient injury during a seizure. Insertion
of a nasogastric (NG) tube is not indicated
because the airway problem is not caused by
vomiting or abdominal distention. Use of
tongue blades during a seizure is
contraindicated.
108 When preparing to teach a client 2. Anticonvulsant drug therapy should never
3. about phenytoin sodium (Dilantin) be stopped suddenly; doing so can lead to
therapy, the nurse should urge the life-threatening status epilepticus. Phenytoin
client not to stop the drug sodium does not carry a risk of physical
suddenly because: 1. Physical dependency or lead to hypoglycemia.
dependency on the drug develops Phenytoin has antiarrhythmic properties, and
over time. 2. Status epilepticus discontinuation does not cause heart block.
may develop. 3. A hypoglycemic
reaction develops. 4. Heart block is
likely to develop.
108 When providing discharge teaching "Avoid hot baths and showers."
4. for a client with multiple sclerosis
(MS), the nurse should include
which instruction?
108 When providing family education 4.
5. for those who have a relative with Change increases stress. Therefore, the most
Alzheimer's disease about important and relevant suggestion is to
minimizing stress, which of the maintain consistency in the client's
following suggestions is most environment, routine, and caregivers.
relevant? Although rest periods are important, going to
1. Allow the client to go to bed four bed interferes with the sleep-wake cycle.
to five times during the day. Rest in a recliner chair is more useful. Testing
2. Test the cognitive functioning of cognitive functioning and reality orientation
the client several times a day. are not likely to be successful and may
3. Provide reality orientation even increase stress if memory loss is severe.
if the memory loss is severe.
4. Maintain consistency in
environment, routine, and
caregivers.
108 When scheduling ANS: A
6. electroconvulsive therapy A client who is experiencing suicidal ideations is in
(ECT), which client should need of an immediate intervention to prevent self-
the nurse prioritize? harm and must be prioritized when the nurse
schedules ECT.
A. A client in bed in a fetal
position who is
experiencing active suicidal
ideations
B. A client with an irritable
mood and exhibiting angry
outbursts
C. A client experiencing
command hallucinations
and delusions of reference
D. A client experiencing
manic episodes of bipolar
disorder
108 When self-administering 2 to 3 minutes
7. atropine (Atropisol), the
nurse should instruct the
client to wait how long
between instilling the first
drop and instilling the
second drop?
108 When should negative If a wound is difficult to manage or traditional
8. pressure wound care be methods are not working for healing.
used?
108 When should you not use a On a wound with an infection, necrosis, malignancy,
9. negative pressure system? fistulas, osteomyelitis, or if can see arteries/veins in
wound
109 When teaching a client "Be aware that your urine may appear darker than
0. about levodopa and usual."
carbidopa (Sinemet)
therapy for Parkinson's
disease, the nurse should
include which instruction?
109 When teaching a patient perform physically demanding activities in the
1. with myasthenia gravis morning.
(MG) about management of
the disease, the nurse rational: Muscles are generally strongest in the
advises the patient to morning, and activities involving muscle activity
______________ should be scheduled then. Plasmapheresis is not
routinely scheduled but is used for myasthenia crisis
or for situations in which corticosteroid therapy
should be discontinued. There is no decrease in
sensation with MG, and muscle atrophy does not
occur because muscles are used during part of the
day.
109 When teaching patients "Call the doctor if pain or herpes lesions occur near
2. who are at risk for Bell's the ear."
palsy because of previous
herpes simplex infection, rational: Pain or herpes lesions near the ear may
which information should indicate the onset of Bell's palsy and rapid
the nurse include? corticosteroid treatment may reduce the duration of
Bell's palsy symptoms. Antiviral therapy for herpes
simplex does not reduce the risk for Bell's palsy.
Corticosteroid therapy will be most effective in
reducing symptoms if started before paralysis is
complete but will still be somewhat effective when
started later. Facial exercises do not prevent Bell's
palsy.
109 When teaching the children a diagnosis of AD can be made only when other
3. of a patient who is being causes of dementia have been ruled out.
evaluated for Alzheimer's
disease (AD) about the rational: The diagnosis of AD is one of exclusion. Age
disorder, the nurse explains is the most important risk factor for development of
that AD. Drugs can slow the deterioration but do not
dramatically reverse the effects of AD. Brain atrophy
is a common finding in AD, but it can occur in other
diseases as well and does not confirm an AD
diagnosis.
109 When the home health RN is Place medications in the home medication
4. planning care for a patient with a organizer.
seizure disorder, which nursing
action can be delegated to an rational: LPN/LVN education includes
LPN/LVN? administration of medications. The other
activities require RN education and scope of
practice.
109 When the nurse applies a painful decorticate posturing.
5. stimulus to the nail beds of an
unconscious patient, the patient rational: Internal rotation, adduction, and
responds with internal rotation, flexion of the arms in an unconscious patient
adduction, and flexion of the arms. is documented as decorticate posturing.
The nurse documents this as Extension of the arms and legs is decerebrate
_____________ posturing. Because the flexion is generalized,
it does not indicate localization of pain or
flexion withdrawal.
109 When the nurse is assessing a Observe respiratory effort.
6. patient with myasthenia gravis,
which action will be most rational: Because respiratory insufficiency
important to take? may be life threatening, it will be most
important to monitor respiratory function.
The other data also will be assessed but are
not as critical.
109 When the nurse is developing a push a manual wheelchair on flat, smooth
7. rehabilitation plan for a patient surfaces.
with a C6 spinal cord injury, an
appropriate patient goal is that the rational: The patient with a C6 injury will be
patient will be able to _____________ able to use the hands to push a wheelchair
on flat, smooth surfaces. Because flexion of
the thumb and fingers is minimal, the patient
will not be able to grasp a wheelchair during
transfer, drive a car with powered hand
controls, or turn independently in bed.
109 When the nurse talks with a client 4
8. with multiple sclerosis who has
slurred speech, which nursing
intervention is contraindicated? 1.
Encouraging the client to speak
slowly. 2. Encouraging the client to
speak distinctly. 3. Asking the
client to repeat indistinguishable
words. 4. Asking the client to
speak louder when tired.
109 Where do ulcers start? At bone and work their way through the skin
9.

110 Which action should the nurse take Examine the mouth and teeth thoroughly.
0. when assessing a patient with
trigeminal neuralgia? rational: Oral hygiene is frequently neglected
because of fear of triggering facial pain.
Having the patient clench the facial muscles
will not be useful because the sensory
branches of the nerve are affected by
trigeminal neuralgia. Light touch and
palpation may be triggers for pain and should
be avoided.
110 Which action will the nurse include Assess intake and output and dietary intake.
1. in the plan of care when caring for
a patient who is experiencing rational: The patient with an acute episode of
trigeminal neuralgia? trigeminal neuralgia may be unwilling to eat
or drink, so assessment of nutritional and
hydration status is important. Because
stimulation by touch is the precipitating
factor for pain, relaxation of the facial
muscles will not improve symptoms.
Application of ice is likely to precipitate pain.
The patient will not want to engage in
conversation, which may precipitate attacks.
110 Which action will the nurse in the Schedule the patient for more frequent
2. outpatient clinic include in the plan appointments.
of care for a patient with mild
cognitive impairment (MCI)? rational: Ongoing monitoring is
recommended for patients with MCI. MCI
does not interfere with activities of daily
living, acetylcholinesterase drugs are not
used for MCI, and an assisted living facility is
not indicated for MCI.
110 Which action will the nurse take Inspect the oral mucosa.
3. when evaluating a patient who is
taking phenytoin (Dilantin) for rational: Phenytoin can cause gingival
adverse effects of the medication? hyperplasia, but does not affect bowel tones,
lung sounds, or pupil reaction to light.
110 Which assessment finding in a Temperature of 101.5 F (38.6 C)
4. patient who was admitted the
previous day with a basilar rational: Patients who have basilar skull fractures
skull fracture is most are at risk for meningitis, so the elevated
important to report to the temperature should be reported to the health care
health care provider? provider. The other findings are typical of a
patient with a basilar skull fracture.
110 Which assessment information Short-term memory
5. will the nurse collect to
determine whether a patient is rational: Decreased short-term memory is one
developing postconcussion indication of postconcussion syndrome. The other
syndrome? data may be assessed but are not indications of
postconcussion syndrome.
110 Which assessment test results ANS: A, B, D, E
6. should a nurse evaluate and A nurse should evaluate electrocardiogram
report in the process of graphic records, pulmonary function study results,
clearing a client for complete blood count, and urinalysis results and
electroconvulsive therapy report any abnormalities to the client's physician.
(ECT)? (Select all that apply.) The client must be medically cleared prior to ECT
treatment.
A. Electrocardiogram graphic
records
B. Pulmonary function study
results
C. Electroencephalogram
analysis
D. Complete blood count
values
E. Urinalysis results
110 Which client is at greatest risk 4. An 80-year-old client admitted for emergency
7. for experiencing sensory surgery; A sudden, unexpected admission for
overload? surgery may involve many experiences (e.g., lab
work, x-rays, signing of forms) while the client is
1. A 40-year-old client in in pain or discomfort. The time for orientation will
isolation with no family. thus be lessened. After surgery, the client may be
2. A 28-year-old quadriplegic in pain and possibly in a critical care setting.
client in a private room.
3. A 16-year-old listening to Options 1 and 2 would more likely be at risk for
loud music. sensory deprivation. Option 3 is considered a
4. An 80-year-old client normal activity for most teenagers.
admitted for emergency
surgery.
110 Which client is most likely to 2. A deaf 88-year-old single client with
8. experience sensory deprivation? +4 edema who lives in an upstairs
apartment; Sensory stimulation comes
1. A blind 93-year-old bedridden from our senses, environment, and
resident of a nursing home presence of meaningful data. Although
2. A deaf 88-year-old single client with the client has no sight and is unable to
+4 edema who lives in an upstairs get out of bed, she is still capable and
apartment likely to receive sensory stimulation. She
3. A child with genetic anomalies, may converse with staff and other
abandoned in infancy, cared for in a residents, feel the touch of bathing, and
special needs foster home, who taste a variety of foods. There is a
attends preschool three times a week potential for sensory deprivation related
4. A premature infant transferred to a to abandonment and the presence of
Neonatal Intensive Care Unit anomalies. Since the child is being cared
for in a special needs foster home, and
attends preschool, one can reasonably
assume that the child receives some
stimulation. Premature infants in
Neonatal Intensive Care Units often
suffer from sensory overload.
110 Which clinical manifestation is a typical 3. A common adverse effect of long-term
9. reaction to long-term phenytoin phenytoin therapy is an overgrowth of
sodium (Dilantin) therapy? 1. Weight gingival tissues. Problems may be
gain. 2. Insomnia. 3. Excessive growth minimized with good oral hygiene, but in
of gum tissue. 4. Deteriorating some cases, overgrown tissues must be
eyesight. removed surgically. Phenytoin does not
cause weight gain, insomnia, or
deteriorating eyesight.
111 which clinical manifest on a leg from the bony prominences are excoriated;
0. which a cast has just been removed is (restricted motion, smaller, atrophy, skin
abnormal finding peeling wrinkled and dry are all normal
findings)
111 Which food-related behaviors are 2. Homonymous hemianopia is blindness
1. expected in a client who has had a in half of the visual field; therefore, the
stroke that has left him with client would see only half of his plate.
homonymous hemianopia? 1. Increased Eating only the food on half of the plate
preference for foods high in salt. 2. results from an inability to coordinate
Eating food on only half of the plate. 3. visual images and spatial relationships.
Forgetting the names of foods. 4. There may be an increased preference
Inability to swallow liquids. for foods high in salt after a stroke, but
this would not be related to
homonymous hemianopia. Forgetting
the names of foods would be aphasia,
which involves a cerebral cortex lesion.
Being unable to swallow liquids is
dysphagia, which involves motor
pathways of cranial nerves IX and X,
including the lower brain stem.
111 Which information about a patient who Uncontrolled head movement
2. is being treated with
carbidopa/levodopa (Sinemet) for rational: Dyskinesia is an adverse effect
Parkinson's disease is most important of the Sinemet, indicating a need for a
for the nurse to report to the health change in medication or decrease in
care provider? dose. The other findings are typical with
Parkinson's disease.
111 Which information about a patient who Pressure of oxygen in brain tissue
3. is hospitalized after a traumatic brain (PbtO2) is 14 mm Hg
injury requires the most rapid action by
the nurse? rational: The PbtO2 should be 20 to 40
mm Hg. Lower levels indicate brain
ischemia. An intracranial pressure (ICP)
of 15 mm Hg is at the upper limit of
normal. CSF is produced at a rate of 20
to 30 mL/hour. The reason for the sinus
tachycardia should be investigated, but
the elevated heart rate is not as
concerning as the decrease in PbtO2.
111 Which information about a patient with The patient has an increased creatinine
4. MS indicates that the nurse should level.
consult with the health care provider
before giving the prescribed dose of rational: Fampridine should not be given
fampridine (Ampyra)? to patients with impaired renal function.
The other information will not impact on
whether the fampridine should be
administered.
111 Which intervention is most effective in 4. Carbamazepine (Tegretol) is an
5. minimizing the risk of seizure activity anticonvulsant that helps prevent
in a client who is undergoing further seizures. Bed rest, sedation
diagnostic studies after having (phenobarbital), and providing privacy
experienced several episodes of do not minimize the risk of seizures.
seizures? 1. Maintain the client on bed
rest. 2. Administer butabarbital sodium
(phenobarbital) 30 mg P.O., three times
per day. 3. Close the door to the room
to minimize stimulation. 4. Administer
carbamazepine (Tegretol) 200 mg P.O.,
twice per day.
111 Which intervention should the nurse 3
6. suggest to help a client with multiple
sclerosis avoid episodes of urinary
incontinence? 1. Limit fluid intake to
1,000 mL/ day. 2. Insert an indwelling
urinary catheter. 3. Establish a
regular voiding schedule. 4.
Administer prophylactic antibiotics,
as ordered.
111 Which intervention will the nurse Maintain a consistent daily routine for the
7. include in the plan of care for a patient's care.
patient who has late-stage
Alzheimer's disease (AD)? rational: Providing a consistent routine will
decrease anxiety and confusion for the
patient. In late-stage AD, the patient will
not remember events from the past.
Reorientation to time and place will not be
helpful to the patient with late-stage AD,
and the patient will not be able to read.
111 Which nursing action will the home Teach the purpose of a prescribed bowel
8. health nurse include in the plan of program.
care for a patient with paraplegia in
order to prevent autonomic rational: Fecal impaction is a common
dysreflexia? stimulus for autonomic dysreflexia. The
other actions may be included in the plan
of care but will not reduce the risk for
autonomic dysreflexia.
111 Which nursing diagnosis takes Risk for injury related to vertigo
9. highest priority for a client admitted
for evaluation for Mnire's disease?
112 Which nursing diagnosis takes Ineffective airway clearance
0. highest priority for a client with
Parkinson's crisis?
112 Which nursing intervention can Monitoring the patency of an indwelling
1. prevent a client from experiencing urinary catheter
autonomic dysreflexia?
112 Which nursing intervention has been 3. The use of ankle-high tennis shoes has
2. found to be the most effective means been found to be most effective in
of preventing plantar flexion in a preventing plantar flexion (footdrop)
client who has had a stroke with because they add support to the foot and
residual paralysis? 1. Place the keep it in the correct anatomic position.
client's feet against a firm footboard. Footboards stimulate spasms and are not
2. Reposition the client every 2 hours. routinely recommended. Regular
3. Have the client wear ankle-high repositioning and range-of-motion
tennis shoes at intervals throughout exercises are important interventions, but
the day. 4. Massage the client's feet the client's foot needs to be in the correct
and ankles regularly. anatomic position to prevent
overextension of the muscle and tendon.
Massaging does not prevent plantar
flexion and, if rigorous, could release
emboli.
112 Which of the following conditions ANS: A, B, D
3. would place a client at risk for injury Severe osteoporosis, acute and chronic
during electroconvulsive therapy pulmonary disorders, and a recent history
(ECT) treatments? (Select all that of cardiovascular accident (CVA) can
apply.) render clients at high risk for injury during
electroconvulsive therapy.
A. Severe osteoporosis
B. Acute and chronic pulmonary
disorders
C. Hypothyroidism
D. Recent cardiovascular accident
E. Prostatic hypertrophy
112 Which of the following interventions 2. Place liquid deodorant on a gauze near
4. would most help reduce olfactory the clean, covered wound; The odor from
stimuli for a client who is hospitalized a draining wound can be minimized by
with a draining wound and is sensory keeping the dressing dry and clean and
overloaded? applying a liquid deodorant on a gauze
near the wound.
1. Use strong disinfectants to clean
the wound.
2. Place liquid deodorant on a gauze
near the clean, covered wound.
3. Spray strong floral room deodorizer
in room to mask wound odor.
4. Use strong disinfectant on
everything possible in room.
112 Which of the following is an initial 2. The first sign of Parkinson's disease is
5. sign of Parkinson's disease? 1. usually tremors. The client commonly is the
Rigidity. 2. Tremor. 3. first to notice this sign because the tremors
Bradykinesia. 4. Akinesia. may be minimal at first. Rigidity is the second
sign, and brady-kinesia is the third sign.
Akinesia is a later stage of bradykinesia.
112 Which of the following is a priority 2.
6. to include in the plan of care for a Because the client is experiencing difficulty
client with Alzheimer's disease processing and completing complex tasks, the
who is experiencing difficulty priority is to provide the client with only one
processing and completing step at a time, thereby breaking the task up
complex tasks? into simple steps, ones that the client can
1. Repeating the directions until process. Repeating the directions until the
the client follows them. client follows them or demonstrating how to
2. Asking the client to do one step do the task is still too overwhelming to the
of the task at a time. client because of the multiple steps involved.
3. Demonstrating for the client Although maintaining structure and routine is
how to do the task. important, it is unrelated to task completion.
4. Maintaining routine and
structure for the client.
112 Which of the following is a 3.
7. realistic short-term goal to be In approximately 2 to 3 days, the client should
accomplished in 2 to 3 days for a be able to regain orientation and thus become
client with delirium? oriented to time and place. Being able to
1. Explain the experience of explain the experience of having delirium is
having delirium. something that the client is expected to
2. Resume a normal sleep-wake achieve later in the course of the illness, but
cycle. ultimately before discharge. Resuming a
3. Regain orientation to time and normal sleep-wake cycle and establishing
place. normal bowel and bladder function probably
4. Establish normal bowel and will take longer, depending on how long it
bladder function. takes to resolve the underlying condition.
112 Which of the following is a true C. they're not all present in the newborn but
8. statement about the paranasal grow and develop with the child, reaching full
sinuses? development after puberty

A. they're fully developed in


newborns and shrink in size
during puberty relative to the
growth of the skill of the bones

B. they're totally absent in


newborn and rapidly develop
during puberty

C. they're not all present in the


newborn but grow and develop
with the child, reaching full
development after puberty.
112 Which of the following is 3. Temperatures are not assessed orally with a
9. contraindicated for a client with glass thermometer because the thermometer
seizure precautions? 1. could break and cause injury if a seizure
Encouraging him to perform his occurred. The client can perform personal
own personal hygiene. 2. Allowing hygiene. There is no clinical reason to
him to wear his own clothing. 3. discourage the client from wearing his own
Assessing oral temperature with a clothes. As long as there are no other
glass thermometer. 4. limitations, the client should be encouraged to
Encouraging him to be out of bed. be out of bed.
113 Which of the following is essential 2.
0. when caring for a client who is The most critical aspect when caring for the
experiencing delirium? client with delirium is to institute measures to
1. Controlling behavioral correct the underlying causative condition or
symptoms with low-dose illness. Controlling behavioral symptoms with
psychotropics. low-dose psychotropics, manipulating the
2. Identifying the underlying environment, and decreasing or discontinuing
causative condition or illness. 3. all medications may be dangerous to the
Manipulating the environment to client's health.
increase orientation.
4. Decreasing or discontinuing all
previously prescribed
medications.
113 Which of the following is not a Osteoporosis
1. normal physiological change
associated with aging?
113 Which of the following is not a 3
2. realistic outcome to establish
with a client who has multiple
sclerosis (MS)? The client will:
1. Develop joint mobility. 2.
Develop muscle strength. 3.
Develop cognition. 4. Develop
mood elevation.
113 Which of the following is not a 2
3. typical clinical manifestation of
multiple sclerosis (MS)? 1.
Double vision. 2. Sudden
bursts of energy. 3. Weakness
in the extremities. 4. Muscle
tremors.
113 Which of the following is NOT D. tonsils
4. considered a border for the
oral cavity?

A. lips
B. cheeks
C. tongue
D. tonsils
113 Which of the following is the Correct Answer: 3
5. priority nursing diagnosis for a Rationale: Ineffective Airway Clearance is the
patient diagnosed with a spinal priority nursing diagnosis for this patient. The
cord injury? nurse utilizes the ABCs (airway, breathing,
1. Fluid Volume Deficit circulation) to determine priority. With Ineffective
2. Impaired Physical Mobility Airway Clearance, the patient is at risk for
3. Ineffective Airway Clearance aspiration and therefore, impaired gas exchange.
4. Altered Tissue Perfusion Fluid Volume Deficit is the nurse's next priority
(circulation), and then Altered Tissue Perfusion. If
the patient does not have enough volume to
circulate, then tissue perfusion cannot be
adequately addressed. The last priority for this
patient is Impaired Physical Mobility.
113 Which of the following is the Correct Answer: 2
6. priority nursing diagnosis for Rationale: The priority nursing diagnosis for a
the patient who has undergone patient who has undergone a spinal fusion is
surgery for a spinal fusion? Impaired Mobility, due to the assessment of the
1. Acute Pain ABCs (airway, circulation, breathing). Impaired
2. Impaired Mobility mobility can affect the patient's circulation,
3. Risk for Infection therefore affecting tissue perfusion and causing a
4. Risk for Injury risk for skin breakdown. Acute Pain is the next
priority since it is an active diagnosis. Diagnoses
with "risk for" do not take priority over active
diagnoses.
113 Which of the following nursing Correct Answer: 4
7. actions is appropriate for Rationale: A patient who has undergone a
preventing skin breakdown in laminectomy needs to be turned by log rolling to
a patient who has recently prevent pressure on the area of surgery. An air
undergone a laminectomy? mattress will help prevent skin breakdown but the
1. Provide the patient with an patient still needs to be turned frequently. Placing
air mattress. pillows under the patient can help take pressure
2. Place pillows under patient off of one side but the patient still needs to
to help patient turn. change positions often. Teaching the patient to
3. Teach the patient to grasp grasp the side rail will cause the spine to twist,
the side rail to turn. which needs to be avoided.
4. Use the log roll to turn the
patient to the side.
113 Which of the following nursing A) Monitor fluid and electrolyte status
8. actions should be implemented in the astutely
care of a patient who is experiencing Fluid and electrolyte disturbances can
increased intracranial pressure (ICP)? have an adverse effect on ICP and must
A) Monitor fluid and electrolyte status be monitored vigilantly. The head of the
astutely. patient's bed should be kept at 30
B) Position the patient in a high degrees in most circumstances, and
Fowler's position. physical restraints are not applied unless
C) Administer vasoconstrictors to absolutely necessary. Vasoconstrictors are
maintain cerebral perfusion. not typically administered in the
D) Maintain physical restraints to treatment of ICP.
prevent episodes of agitation.
113 Which of the following questions 1. "Water?"; A simple, clearly spoken, one-
9. would be easiest for a client with a word question is less confusing and easier
hearing deficit to understand? to understand than more complex
phrases. Simple is more easily heard than
1. "Water?" complex. The more words there are in a
2. "Would you like a drink of water?" sentence, the more likely it is that some
3. "Want a drink?" will not be understood, causing a
4. "Are you thirsty?" distortion in meaning.
114 Which of the following should the 1.
0. nurse expect to include as a priority Identifying oneself and making sure that
in the plan of care for a client with the nurse has the client's attention
delirium based on the nurse's addresses the difficulties with focusing,
understanding about the orientation, and maintaining attention.
disturbances in orientation Eliminating daytime napping is unrealistic
associated with this disorder? 1. until the cause of the delirium is
Identifying self and making sure that determined and the client's ability to
the nurse has the client's attention. focus and maintain attention improves.
2. Eliminating the client's napping in Engaging the client in reminiscing and
the daytime as much as possible. avoiding arguing are also unrealistic at
3. Engaging the client in reminiscing this time.
with relatives or visitors.
4. Avoiding arguing with a suspicious
client about his perceptions of reality.
114 Which of the following should the 1,3,4,5
1. nurse include in the discharge plan
for a client with multiple sclerosis
who has an impaired peripheral
sensation? Select all that apply. 1.
Carefully test the temperature of
bath water. 2. Avoid kitchen activities
because of the risk of injury. 3. Avoid
hot water bottles and heating pads.
4. Inspect the skin daily for injury or
pressure points. 5. Wear warm
clothing when outside in cold
temperatures.
114 Which of the following should the 2. Gabapentin (Neurontin) may impair
2. nurse include in the teaching plan for vision. Changes in vision, concentration,
a client with seizures who is going or coordination should be reported to the
home with a prescription for physician. Gabapentin should not be
gabapentin (Neurontin)? 1. Take all stopped abruptly because of the potential
the medication until it is gone. 2. for status epilepticus; this is a medication
Notify the physician if vision changes that must be tapered off. Gabapentin is to
occur. 3. Store gabapentin in the be stored at room temperature and out of
refrigerator. 4. Take gabapentin with direct light. It should not be taken with
an antacid to protect against ulcers. antacids.
114 Which of the following techniques 2. Sliding a client on a sheet causes
3. does the nurse avoid when changing friction and is to be avoided. Friction
a client's position in bed if the client injures skin and predisposes to pressure
has hemiparalysis? 1. Rolling the ulcer formation. Rolling the client is an
client onto the side. 2. Sliding the acceptable method to use when changing
client to move up in bed. 3. Lifting positions as long as the client is
the client when moving the client up maintained in anatomically neutral
in bed. 4. Having the client help lift positions and her limbs are properly
off the bed using a trapeze. supported. The client may be lifted as
long as the nurse has assistance and uses
proper body mechanics to avoid injury to
himself or herself or the client. Having the
client help lift herself off the bed with a
trapeze is an acceptable means to move a
client without causing friction burns or
skin breakdown.
114 Which of the following will the nurse 4. A generalized tonic-clonic seizure
4. observe in the client in the ictal involves both a tonic phase and a clonic
phase of a generalized tonic-clonic phase. The tonic phase consists of loss of
seizure? 1. Jerking in one extremity consciousness, dilated pupils, and muscular
that spreads gradually to adjacent stiffening or contraction, which lasts about
areas. 2. Vacant staring and 20 to 30 seconds. The clonic phase involves
abruptly ceasing all activity. 3. Facial repetitive movements. The seizure ends
grimaces, patting motions, and lip with confusion, drowsiness, and resumption
smacking. 4. Loss of consciousness, of respiration. A partial seizure starts in one
body stiffening, and violent muscle region of the cortex and may stay focused
contractions. or spread (e.g., jerking in the extremity
spreading to other areas of the body). An
absence seizure usually occurs in children
and involves a vacant stare with a brief loss
of consciousness that often goes unnoticed.
A complex partial seizure involves facial
grimacing with patting and smacking.
114 which of the follow should be curtain-like shadow across visual field-
5. immediately reported to hcp detached retina surgical emergency
114 Which of these nursing actions for a Passive range of motion to extremities q8hr
6. patient with Guillain-Barr syndrome
is most appropriate for the nurse to rational: Assisting a patient with movement
delegate to an experienced nursing is included in nursing assistant education
assistant? and scope of practice. Administration of
tube feedings, administration of ordered
medications, and assessment are skills
requiring more education and scope of
practice, and the RN should perform these
skills.
114 Which of these nursing actions Administer the prescribed clopidogrel
7. included in the care of a patient who (Plavix).
has been experiencing stroke
symptoms for 60 minutes can the rational: Administration of oral medications
nurse delegate to an LPN/LVN? is included in LPN education and scope of
practice. The other actions require more
education and scope of practice and should
be done by the RN.
114 Which of these patients is most A 44-year-old receiving IV antibiotics for
8. appropriate for the intensive care meningococcal meningitis
unit (ICU) charge nurse to assign to
an RN who has floated from the rational: An RN who works on a medical
medical unit? unit will be familiar with administration of
IV antibiotics and with meningitis. The
postcraniotomy patient, patient with an ICP
monitor, and the patient on a ventilator
should be assigned to an RN familiar with
the care of critically ill patients.
114 Which of these prescribed Administer lorazepam (Ativan) 4 mg IV.
9. interventions will the nurse
implement first for a hospitalized rational: To prevent ongoing seizures, the
patient who is experiencing nurse should administer rapidly acting
continuous tonic-clonic seizures? antiseizure medications such as the
benzodiazepines. A CT scan is appropriate,
but prevention of any seizure activity
during the CT scan is necessary. Phenytoin
also will be administered, but it is not
rapidly acting. Patients who are
experiencing tonic-clonic seizures are
nonresponsive, although the nurse should
assess LOC after the seizure.
115 Which parameter is best for the Intracranial pressure
0. nurse to monitor to determine
whether the prescribed IV mannitol rational: Mannitol is an osmotic diuretic and
(Osmitrol) has been effective for an will reduce cerebral edema and intracranial
unconscious patient? pressure. It may initially reduce hematocrit
and increase blood pressure, but these are
not the best parameters for evaluation of
the effectiveness of the drug. Oxygen
saturation will not directly improve as a
result of mannitol administration.
115 Which patient is at highest risk for a Correct Answer: 1
1. spinal cord injury? Rationale: The three major risk factors for
1. 18-year-old male with a prior spinal cord injuries (SCI) are age (young
arrest for driving while intoxicated adults), gender (higher incidence in males),
(DWI) and alcohol or drug abuse. Females tend to
2. 20-year-old female with a history engage in less risk-taking behavior than
of substance abuse young men.
3. 50-year-old female with
osteoporosis
4. 35-year-old male who coaches a
soccer team
115 Which statement about older adults The number of older adults is rising
2. in the United States is correct? because of the increase in the average life
span and the aging of the baby boom
generation
115 Which statement by a client with a 2. Toxic effects of topiramate (Topamax)
3. seizure disorder taking topiramate include nephrolithiasis, and clients are
(Topamax) indicates the client has encouraged to drink 6 to 8 glasses of water
understood the nurse's instruction? a day to dilute the urine and flush the renal
1. "I will take the medicine before tubules to avoid stone formation. Topiramate
going to bed." 2. "I will drink 6 to 8 is taken in divided doses because it
glasses of water a day." 3. "I will produces drowsiness. Although eating fresh
eat plenty of fresh fruits." 4. "I will fruits is desirable from a nutritional
take the medicine with a meal or standpoint, this is not related to the
snack." topiramate. The drug does not have to be
taken with meals.
115 Which statement by a patient who "I am going to drive home and go to bed."
4. is being discharged from the
emergency department (ED) after a rational: Following a head injury, the patient
head injury indicates a need for should avoid operating heavy machinery.
intervention by the nurse? Retrograde amnesia is common after a
concussion. The patient can take
acetaminophen for headache and should
return if symptoms of increased intracranial
pressure such as dizziness or nausea occur.
115 Which statement indicates the 2. "I can't hear the doorbell."; This client
5. client needs a sensory aid in the could use an assistive device that flashes a
home? light when the doorbell rings.

1. "I tripped over that throw rug


again."
2. "I can't hear the doorbell."
3. "My eyesight is good if I wear my
glasses."
4. "I can hear the TV if I turn it up
high."
115 Which two factors contribute to the The aging of the baby boom generation and
6. projected increase in the number of the growth of the population segment over
older adults? age 85
115 Which two procedures are used to Thalmotomy (Thalamus)
7. control tremors when medications Pallidotomy (Globus Thalamus)
are not working?
115 While admitting a patient with a Insert nasogastric tube.
8. basal skull fracture, the nurse
notes clear drainage from the rational: Rhinorrhea may indicate a dural
patient's nose. Which of these tear with cerebrospinal fluid (CSF) leakage,
admission orders should the nurse and insertion of a nasogastric tube will
question? increase the risk for infections such as
meningitis. Turning the patient, elevating the
head, and applying cold pack are
appropriate orders.
115 While assessing a client diagnosed 2.
9. with dementia, the nurse notes that Vulgar language is common in clients with
her husband is concerned about dementia when they are having trouble
what he should do when she uses communicating about a topic. Ignoring the
vulgar language with him. The vulgarity and distracting her is appropriate.
nurse should: Telling the client she is rude or to stop
1. Tell her that she is very rude. swearing will have no lasting effect and may
2. Ignore the vulgarity and distract cause agitation. Just leaving the room is
her. abandonment that the client will not
3. Tell her to stop swearing understand.
immediately.
4. Say nothing and leave the room.
116 While bathing a client who Ensure the client's privacy, put up the side
0. sustained a stroke, a nurse is asked rail, and explain that she'll return shortly.
by a coworker to assist with
repositioning another client. What
should the nurse do?
116 While caring for a patient who has The nursing assistant goes into the patient's
1. just been admitted with room without a mask.
meningococcal meningitis, the RN
observes all of the following. Which rational: Meningococcal meningitis is spread
one requires action by the RN? by respiratory secretions, so it is important
to maintain respiratory isolation as well as
standard precautions. Because the patient
may be confused and weak, bedrails should
be elevated at both the food and head of the
bed. Low light levels in the room decrease
pain caused by photophobia. Nutrition is an
important aspect of care in a patient with
meningitis.
11 While caring for the patient with Correct Answer: 3
62. spinal cord injury (SCI), the nurse Rationale: Autonomic dysreflexia is an
elevates the head of the bed, removes emergency that requires immediate
compression stockings, and continues assessment and intervention to prevent
to assess vital signs every two to complications of extremely high blood
three minutes while searching for the pressure. Additional nursing assistance will
cause in order to prevent loss of be needed and a colleague needs to reach
consciousness or death. By practicing the physician stat.
these interventions, the nurse is
avoiding the most dangerous
complication of autonomic
dysreflexia, which is which of the
following?
1. hypoxia
2. bradycardia
3. elevated blood pressure
4. tachycardia
11 While educating the daughter of a 4.
63. client with dementia about the illness, Telling the client that she is wrong and
the daughter complains to the nurse then telling her what is right is
that her mother distorts things. The argumentative and challenging. Arguing
nurse understands that the daughter with or challenging distortions is least
needs further teaching about effective because it increases
dementia when she makes which defensiveness. Telling the client about
statement? reality indicates awareness of the issues
1. "I tell her reality, such as, 'That and is appropriate. Acknowledging that
noise is the wind in the trees.'" misperceptions are part of the disease
2. "I understand the misperceptions indicates an understanding of the disease
are part of the disease." and an awareness of the issues. Turning off
3. "I turn off the radio when we're in the radio helps to limit environmental
another room." stimuli and indicates an awareness of the
4. "I tell her she is wrong and then I issues.
tell her what's right."
11 While performing a history, the nurse 2. Mental status; Mental status is assessed
64. assesses sensory perceptions such as: while performing a history. Kinesthetic
perception, deep tendon reflexes, and
1. Kinesthetic perception cranial nerves are assessed during the
2. Mental status physical exam, not the history.
3. Deep tendon reflexes
4. Cranial nerves
11 A white female client is admitted to Obesity
65. an acute care facility with a diagnosis
of stroke. Her history reveals
bronchial asthma, exogenous obesity,
and iron deficiency anemia. Which
history finding is a risk factor for
stroke?
11 Who does ALL the care of patients RN or LPN
66. with spinal cord injuries
11 Who identifies hearing? Audiologist
67.

11 Who is eligible for epilepsy surgery? About half of patients with partial seizures
68. (what %) who don't respond to AEDs are eligible for
surgery.
11 Who is retinal detachment more Myopic, nearsighted whites and men who
69. common in? suffered traumatic injuries.
11 Who may have some risk posed if Pregnant, demand type cardiac
70. using TENS? pacemaker, bladder stimulator, metal
plates or pins, diabetes, or heart disease.
11 Why are fractures of the epiphyseal Fractures of the epiphyseal plate (growth
71. plate a special concern? plate) may affect the growth of the limb.
11 Why should the patient with Because the unaffected side is painful.
72. trigeminal neuralgia chew on the Patient would lose weight. The outcome
unaffected side? What is the should be weight gain as a result of
outcome? chewing on unaffected side and eating a
high calorie, high protein diet like custard,
milk and eggs.
11 Write four nursing interventions for Care of the blind: announce presence
73. the care of the blind person and four clearly, call by name, orient carefully to
nursing interventions for the care of surroundings, guide by walking in front of
the deaf person. client with their hand in your elbow.
Care of deaf: reduce distraction before
beginning conversation, look and listen to
client, give client full attention if they are a
lip reader, face client directly.

Question 1 See full question

A nurse is to administer several oral medications to a client at the same


time. Which nursing instruction or action is appropriate in this situation?
You Selected:

Tell the client the name and action or use of each medication before
administering it.
Correct response:

Tell the client the name and action or use of each medication before
administering it.
Explanation:

Question 2 See full question

Which instruction should the nurse include in the teaching plan for a client
with seizures who is going home with a prescription for gabapentin?
You Selected:

Notify the health care provider (HCP) if vision changes occur.


Correct response:

Notify the health care provider (HCP) if vision changes occur.


Explanation:

Question 3 See full question

A nurse is administering vitamin K to a neonate following birth. The


medication comes in a concentration of 2 mg/ml, and the ordered dose is 0.5
mg to be given subcutaneously. How many milliliters would the nurse
administer? Record your answer using two decimal places.
Your Response:

1
Correct response:

0.25
Explanation:

Question 4 See full question

During an assessment of a neonate born at 33 weeks' gestation, a nurse


finds and reports a heart murmur. An echocardiogram reveals patent ductus
arteriosis, for which the neonate received indomethacin. An expected
outcome after the administration of indomethacin to a neonate with patent
ductus arteriosis is:
You Selected:

closure of a patent ductus arteriosus.


Correct response:

closure of a patent ductus arteriosus.


Explanation:

Question 5 See full question

A client who is using a patient-controlled analgesia (PCA) pump after bowel


surgery states, Im afraid that Ill become addicted if I use too much
morphine. Which would be the best response by the nurse?
You Selected:

Have you had problems with drug addiction before?


Correct response:

When morphine is used to alleviate severe pain for 2 to 3 days, there is


little likelihood of becoming addicted.
Explanation:

1 Acidosis a condition where cells tends to make blood relatively too acidic
. * respiratory acidosis
* metabolic acidosis
2 active
. transport
s

sodium -potassium pump, moves sodium out of the cell and potassium into
the cell, keeping ICF lower in sodium and higher in potassium than the ECF
3 alkalosis
.

condition, blood relatively too basic


* Respiratory alkalosis
* Metabolic alkalosis
4 Anion
. gap

Difference between the concentrations of serum cations and anions:


determined by measuring the concentrations of sodium cations and chloride
and bicarbonate anions.
(Na+ - [Cl- + HCO3-]) = 8-12 mEq/L
5. Anions

Chloride (Cl-)
bicarbonate HCO-3)
6. Arterial blood A measurement of oxygen and carbon dioxide in the blood to monitor
gase a patient's acid-base balance
7. Autologous Using one's own blood...
transfusion
8. buffer

pairs of chemicals that work together to maintain normal pH of body


fluids
* Free H+ ions, a buffer takes them up so they no longer are free.

* Too few, a buffer can release H+ ions to prevent an acid-balance


imbalance
9. cations

an ion that has a positive charge


1 cations sodium
0. potassium
calcium
magnesium
1 colloid
1. osmotic
pressure

Pressure exerted by plasma proteins in blood vessel's plasma that pulls


water into the circulatory system. Reduced plasma proteins can cause
fluid buildup in tissues (edema)
1 Colloids
2.

do not freely diffuse across a semi permeable membrane, hence keepign


the fluid within the intravascular space. IE ALBUMIN, heastarch
pentasatarch, dextran
1 colloids
3. (e.g. blood
or albumin )

IV fluids that contain large molecules that don't pass through


semipermeable membranes. When infused, they remain in the
intravascular compartment and expand intravascular volume by drawing
fluid from extravascular spaces via their higher oncotic pressure.
1 Crystalloids
4.

This is used for fluid replacement or maintenance fluids. Examples: D5W


(sugar water) & normal saline (0.9%)
15. dehydration

hypernatremia may occur in combination with ECV deficit


* A serious reduction in the body's water content
16. electrolytes

Sodium (+) = 135 - 145


potassium (+) = 3.5 - 5.0
chloride (-) = 98 - 106
calcium (+) = 8.4 - 10.5
magnesium (-) = 1.5 - 2.5
bicarbonate (-) = 22 -26 (arterial) 24-30 (venous)
phosphate (-) = 2.7 4.5
17. extracellular fluid

outside cells
* Intravascular plasma (3 L)
* Interstitial fluid 11 L
1 Extracellular
8. fluid volume
deficit =
hypovolemia

insufficient isotonic fluid in the extracellular compartments


* output of isotonic fluid exceeds intake of sodium-containing fluid
1 Extracellular
9. fluid volume
excess =
hypervolemia

too much isotonic fluid in the extracellular compartment


* intake of sodium-containing isotonic fluid has exceeded fluid output
2 extravasation
0.

A nurse is administering daunorubicin (Daunoxome) through a


peripheral I.V. line when the client complains of burning at the
insertion site. The nurse notes no blood return from the catheter and
redness at the I.V. site. The client is most likely experiencing which
complication?
2 filtration
1.

fluid move into and out capillaries (between the vascular and interstitial
compartments
* net effect of four forces
2 fluid
2.

water that contains dissolved or suspended substances such as


glucose, mineral salts, and proteins
2 hydrostatic
3. pressure

The movement of fluid across the arterial end of capillary membranes


into the interstitial fluid surrounding the capillary.
2 hyernatremi
4. a

water deficit = hypertonic condition


body fluids too concentrated
when interstitil fluid is hypertonic water leaves cells by osmosis, cell
shrivel
SS: cerebral dysfunction
25 hypercalcemia
.

shift of calcium from bones into ECF and decreased calcium output
* PT with cancer
* decreases neuromuscular excitability (lethargy
26 hypermagnese
. mia

abnormally high magnesium concentration in the blood (laxative,


antacids
* end-stage renal dis. cause it
- SS decreased tendon reflexes being most common
27 hypertonic
.

solution is more concentrated than blood


28 hypocalcemia
.

low calcium concentration in the blood


* acute pancreatitis calcium binds to undigested fat in their feces and
is excreted
* neuromuscular excitability
29 hypokalemia
.

shift of potassium from the ECF into cells, and increase output
diarrhea
vomiting
potassium wasting diuretics
30 hypomagnes
. emia

shift of plasma magnesium to its inactive bound from, and increase


magnesium output
31. hyponatremi
a

water excess or water intoxication hypotonic condition


water enter cells by osmosis causing the cells to swell
*cerebral dysfunction occur when brain cells swell
32. hypotonic

solution is more dilute than the blood


33. hypovolemia

decreased vascular volume and often is used when discussing ECV


deficit
3 infiltration
4.

IV catheter become dislodged or a vein ruptures and IV fluids


inadvertently enter subcutaneous tissue around the veinipuncutre site
3 interstitial
5. fluid
located between cells
* part of the ECF
* 11 L
3 intracellular
6. fluid

inside cells
* 28 L or 2/3 of the total body water
3 intravascula
7. r fluid

Fluid contained within vascular channels (about one-twentieth of the


body weight)
3 ions
8.

charged particles when dissolved in water


cations
Anions
3 isotonic
9.

fluid with the same concentration of nonpermanent particles such as


normal blood
4 kidneys metabolic acids
0. excrete
4 lungs carbonic acid
1. excrete
4 major bicarbonate (HCO-3) buffers metabolic acids (20 bicarbonate: 1 carbonic
2. buffer in acid.
the ECF
4 metabolic
3. acidosis

increase of metabolic acid or decrease of base (bicarbonate) kidneys are


unable to excrete enough metabolic acids, which accumulate in the blood,
or bicarbonate is removed from the body directly as with diarrhea
4 Metaboli
4 c
. alkalosis

increase of bicarbonate or decrease of metabilic acid which increase blood


bicarbonate by releasing it from its buffering function
4 oncotic
5 pressure
.

AKA colloid osmotic pressure


inward-pulling force caused by blood proteins that helps move fluid from
interstitial area back into capillaries
4 osmolalit
6 y
.

The concentration of a solution determined by the number of dissolved


particles per kilogram of water; controls water movement and distribution in
the body fluid compartments.
4 osmosis
7
.

Diffusion of water through a selectively permeable membrane from a high


concentration to a low concentration.

Question 1 See full question

When teaching a client how to take a sublingual tablet, the nurse should
instruct the client to place the tablet:
You Selected:

on the floor of the mouth.


Correct response:

on the floor of the mouth.


Explanation:

Question 2 See full question

While a client is taking alprazolam, which food should the nurse instruct the
client to avoid?
You Selected:

alcohol
Correct response:

alcohol
Explanation:

Question 3 See full question

At an outpatient visit 3 months after discharge from the hospital, a client


says he has stopped his olanzapine even though it controls his symptoms of
schizophrenia better than other medications. "I have gained 20 lb (9.1 kg)
already. I cannot stand anymore." Which response by the nurse
is most appropriate?
You Selected:

"I can help you with a diet and exercise plan to keep your weight down."
Correct response:

"I can help you with a diet and exercise plan to keep your weight down."
Explanation:

Question 4 See full question

A nurse is administering daunorubicin through a peripheral I.V. line when the


client complains of burning at the insertion site. The nurse notes no blood
return from the catheter and redness at the I.V. site. The client is most likely
experiencing which complication?
You Selected:

Extravasation
Correct response:

Extravasation
Explanation:

Question 5 See full question

A clients diagnosis of pneumonia requires treatment with antibiotics. The


corresponding order in the clients chart should be written as:
You Selected:

Moxifloxacin 400 mg daily.


Correct response:

Moxifloxacin 400 mg daily.


Explanation:

A 60-year-old male 1. Although obtaining the ECG, chest


client comes into the radio-graph, and blood work are all
emergency important, the nurse's priority action
department with a should be to relieve the crushing chest
complaint of crushing pain. Therefore, administering morphine
substernal chest pain sulfate is the priority action.
that radiates to his
shoulder and left arm.
The admitting
diagnosis is acute
myo-cardial infarction
(MI). Immediate
admission orders
include oxygen by
nasal cannula at 4
L/minute, blood work,
a chest radiograph, a
12-lead
electrocardiogram
(ECG), and 2 mg of
morphine sulfate
given I.V. The nurse
should first:
1. Administer the
morphine.
2. Obtain a 12-lead
ECG.
3. Obtain the blood
work.
4. Order the chest
radiograph
2. A 65-year-old client 3. The client is having
is admitted to the symptoms of a myo-
emergency cardial infarction. The
department with a fi rst action is to
fractured hip. The prevent platelet
client has chest pain formation and block
and shortness of prostaglandin synthe-
breath. The health sis. The nitroglycerin
care provider orders tablet will be
nitroglycerin absorbed fastest if
tablets. Which the client chews the
should the nurse tablet.
instruct the client to
do? 1. Put the tablet
under the tongue
until it is absorbed.
2. Swallow the
tablet with 120 mL
of water.
3. Chew the tablet
until it is dissolved.
4. Place the tablet
between his cheek
and gums.
3. A 68-year-old female 1. Further assessment
client on day 2 after is needed in this
hip surgery has no situation. It is
cardiac history but premature to initiate
reports having chest other actions until
heaviness. The first further data have
nursing action been gathered.
should be to: Inquiring about the
1. Inquire about the onset, duration,
onset, duration, location, severity, and
severity, and precipitating factors
precipitating factors of the chest
of the heaviness. heaviness will provide
2. Administer pertinent information
oxygen via nasal to convey to the
cannula. physician.
3. Offer pain
medication for the
chest heaviness.
4. Inform the
physician of the
chest heaviness.
4. A 69-year-old female 1. It is a priority to
has a history of assess blood pressure
heart failure. She is first because people
admitted to the with pulmonary
emergency edema typically
department with experience severe
heart failure hypertension that
complicated by requires early
pulmonary edema. intervention. The
On admission of this client probably does
client, which of the not have skin
following should the breakdown on
nurse assess first? admission; however,
1. Blood pressure. when the client is
2. Skin breakdown. stable, the nurse
3. Serum potassium should inspect the
level. skin. Potassium levels
4. Urine output are not the first
priority. The nurse
should monitor urine
output after the client
is stable.
5. After the 1. Although
administration of t- monitoring the 12-
PA, the assessment lead ECG and
priority is to: monitoring breath
1. Observe the client sounds are important,
for chest pain. observing the client
2. Monitor for fever. for chest pain is the
3. Monitor the 12- nursing assessment
lead priority because
electrocardiogram closure of the
(ECG) every 4 previously obstructed
hours. coronary artery may
4. Monitor breath recur. Clients who
sounds. receive t-PA
frequently receive
heparin to prevent
closure of the artery
after administration
of t-PA. Careful
assessment for signs
of bleeding and
monitoring of partial
thromboplastin time
are essential to
detect complications.
Administration of t-PA
should not cause
fever.
6. Alteplase 4. The thrombolytic
recombinant, or agent t-PA,
tissue plasminogen administered
activator (t-PA), a intravenously, lyses
thrombolytic the clot blocking the
enzyme, is coronary artery. The
administered during drug is most effective
the first 6 hours when adminis-tered
after onset of within the fi rst 6
myocardial hours after onset of
infarction (MI) to: MI. The drug does not
1. Control chest reduce coronary
pain. artery vasospasm;
2. Reduce coronary nitrates are used to
artery vasospasm. promote vasodilation.
3. Control the Arrhyth-mias are
arrhythmias managed by
associated with MI. antiarrhythmic drugs.
4. Revascularize the Surgical approaches
blocked coronary are used to open the
artery. coronary artery and
reestablish a blood
supply to the area.
7. As an initial step in treating a client 3. Nitroglycerin produces peripheral
with angina, the physician prescribes vasodi-lation, which reduces myocardial
nitroglycerin tab-lets, 0.3 mg given oxygen consump-tion and demand.
sublingually. This drug's principal Vasodilation in coronary arteries and
effects are produced by: collateral vessels may also increase blood
1. Antispasmodic effects on the fl ow to the ischemic areas of the heart.
pericardium. Nitroglycerin decreases myocardial
2. Causing an increased myocardial oxygen demand. Nitroglycerin does not
oxygen demand. have an effect on pericardial spasticity or
3. Vasodilation of peripheral conductivity in the myocardium.
vasculature.
4. Improved conductivity in the
myocardium.
8. A client has a history of heart failure 2. Early symptoms of digoxin toxicity
and has been taking several include anorexia, nausea, and vomiting.
medications, including furosemide Visual disturbances can also occur,
(Lasix), digoxin (Lanoxin) and including double or blurred vision and
potassium chloride. The client has visual halos. Hypokalemia is a common
nausea, blurred vision, headache, and cause of digoxin toxicity associated with
weakness. The nurse notes that the arrhythmias because low serum
client is confused. The telemetry strip potassium can enhance ectopic
shows first-degree atrioventricular pacemaker activity. Although vomiting
block. The nurse should assess the can lead to fl uid defi cit, given the
client for signs of which condition? client's history, the vomiting is likely due
1. Hyperkalemia. to the adverse effects of digoxin toxic-ity.
2. Digoxin toxicity. Pulmonary edema is manifested by
3. Fluid deficit. dyspnea and coughing.
4. Pulmonary edema.
9. A client has a throbbing headache when 1. Headache is a common side effect of
nitroglycerin is taken for angina. The nitro-glycerin that can be alleviated with
nurse should instruct the client that: aspirin, acetaminophen or ibuprofen. The
1. Acetaminophen (Tylenol) or sublingual nitroglycerin needs to be
Ibuprofen (Advil) can be taken for this absorbed in the mouth, which will be
common side effect. disrupted with drinking. Lying fl at will
2. Nitroglycerin should be avoided if increase blood flow to the head and may
the client is experiencing this serious increase pain and exacerbate other
side effect. symptoms, such as shortness of breath.
3. Taking the nitroglycerin with a few
glasses of water will reduce the
problem.
4. The client should lie in a supine
position to alleviate the headache.
1 A client has chest pain rated at 8 on a 4. Nursing management for a client with
0. 10 point visual analog scale. The 12- a myocardial infarction should focus on
lead electrocardiogram reveals ST pain manage-ment and decreasing
elevation in the inferior leads and myocardial oxygen demand. Fluid status
Troponin levels are elevated. What is should be closely monitored. Client
the highest priority for nursing education should begin once the client is
management of this client at this time? stable and amenable to teaching.
1. Monitor daily weights and urine Visitation should be based on client
output. comfort and maintaining a calm
2. Permit unrestricted visitation by environment.
family and friends.
3. Provide client education on
medications and diet.
4. Reduce pain and myocardial oxygen
demand.
1 A client is admitted with a myocardial 1. An S3 heart sound occurs early in
1. infarction and new onset atrial diastole as the mitral and tricuspid valves
fibrillation. While auscultating the open and blood rushes into the
heart, the nurse notes an irregular ventricles.
heart rate and hears an extra heart
sound at the apex after the S2 that
remains constant throughout the
respiratory cycle. The nurse should
document these findings as:
1. Heart rate irregular with S3.
2. Heart rate irregular with S4.
3. Heart rate irregular with aortic
regurgitation.
4. Heart rate irregular with mitral
stenosis.
1 Clients with heart failure are 2. Characteristics of atrial fi brillation include pulse
2. prone to atrial fibrillation. rate greater than 100 bpm, totally irregular rhythm,
During physical assessment, and no defi nite P waves on the ECG. During
the nurse should suspect atrial assessment, the nurse is likely to note the irregular
fibrillation when palpation of rate and should report it to the physician. A weak,
the radial pulse reveals: thready pulse is characteristic of a client in shock.
1. Two regular beats followed Two regular beats followed by an irregular beat may
by one irregular beat. indicate a premature ventricular contraction.
2. An irregular pulse rhythm.
3. Pulse rate below 60 bpm.
4. A weak, thready pulse.
1 The client who experiences 3. Pasta, tomato sauce, salad, and coffee would be
3. angina has been told to follow the best selection for the client following a low-
a low-cholesterol diet. Which cholesterol diet. Hamburgers, milkshakes, liver, and
of the following meals should fried foods tend to be high in cholesterol.
the nurse tell the client would
be best on her low-cholesterol
diet?
1. Hamburger, salad, and
milkshake.
2. Baked liver, green beans,
and coffee.
3. Spaghetti with tomato
sauce, salad, and coffee.
4. Fried chicken, green beans,
and skim milk
1 A client with acute chest pain 1, 4, 5. Morphine sulfate acts as an analgesic and
4. is receiving I.V. morphine sedative. It also reduces myocardial oxygen con-
sulfate. Which of the following sumption, blood pressure, and heart rate. Morphine
results are intended effects of also reduces anxiety and fear due to its sedative
morphine in this client? Select effects and by slowing the heart rate. It can
all that apply. depress respirations; however, such an effect may
1. Reduces myocardial oxygen lead to hypoxia, which should be avoided in the
consumption. treatment of chest pain. Angiotensin-converting
2. Promotes reduction in enzyme-inhibitor drugs, not morphine, may help to
respiratory rate. prevent ventricular remodeling.
3. Prevents ventricular
remodeling.
4. Reduces blood pressure and
heart rate.
5. Reduces anxiety and fear.
1 A client with angina has been 3. The client taking nifedipine should inspect the
5. taking nifedipine. The nurse gums daily to monitor for gingival hyperplasia. This
should teach the client to: is an uncommon adverse effect but one that
1. Monitor blood pressure requires monitoring and intervention if it occurs.
monthly. The client taking nifedipine might be taught to
2. Perform daily weights. monitor blood pressure, but more often than
3. Inspect gums daily. monthly. These clients would not generally need to
4. Limit intake of green leafy perform daily weights or limit intake of green leafy
vegetables. vegetables.
1 A client with chest pain is 2. Nitroglycerin is a vasodilator that will lower blood
6. prescribed intravenous pressure. The client is having chest pain and the ST
nitroglycerin (Tridil). Which elevation indicates injury to the myocardium, which
assessment is of greatest may benefit from nitroglycerin. The potassium and
concern for the nurse initiating heart rate are within normal range.
the nitro-glycerin drip?
1. Serum potassium is 3.5
mEq/L.
2. Blood pressure is 88/46.
3. ST elevation is present on
the electrocardiogram.
4. Heart rate is 61.
1 A client with chronic heart failure has 3. Coumadin is an anticoagulant, which is
7. atrial fibrillation and a left ventricular used in the treatment of atrial fi brillation
ejection fraction of 15%. The client is and decreased left ventricular ejection
taking warfarin (Coumadin). The fraction (less than 20%) to prevent
expected outcome of this drug is to: thrombus formation and release of emboli
1. Decrease circulatory overload. into the circulation. The client may also
2. Improve the myocardial workload. take other medication as needed to
3. Prevent thrombus formation. manage the heart failure. Coumadin does
4. Regulate cardiac rhythm. not reduce circulatory load or improve
myocardial workload. Coumadin does not
affect cardiac rhythm.
1 A client with heart failure is receiving 2. Digoxin is a cardiac glycoside with posi-
8. digoxin intravenously. The nurse tive inotropic activity. This inotropic
should determine the effectiveness of activity causes increased strength of
the drug by assessing which of the myocardial contractions and thereby
following? increases output of blood from the left
1. Dilated coronary arteries. ventricle. Digoxin does not dilate coronary
2. Increased myocardial contractility. arteries. Although digoxin can be used to
3. Decreased cardiac arrhythmias. 4. treat arrhythmias and does decrease the
Decreased electrical conductivity in electrical conductivity of the myocardium,
the heart. these are not primary reasons for its use in
clients with heart failure and pulmonary
edema.
1 Contraindications to the 2. A history of cerebral hemorrhage is a
9. administration of tissue plasminogen contraindication to administration of t-PA
activator (t-PA) include which of the because the risk of hemorrhage may be
following? further increased. Age greater than 60
1. Age greater than 60 years. years, history of heart failure, and
2. History of cerebral hemorrhage. cigarette smoking are not
3. History of heart failure. contraindications.
4. Cigarette smoking.
2 During the previous few months, a 56- 3. Nitroglycerin may be used prophylacti-
0. year-old woman felt brief twinges of cally before stressful physical activities
chest pain while working in her garden such as stair-climbing to help the client
and has had frequent episodes of remain pain free. Visiting her friend early
indigestion. She comes to the hospital in the day would have no impact on
after experiencing severe anterior decreasing pain episodes. Resting before
chest pain while raking leaves. Her or after an activity is not as likely to help
evaluation confirms a diagnosis of prevent an activity-related pain episode.
stable angina pectoris. After
stabilization and treatment, the client
is discharged from the hospital. At her
follow-up appointment, she is
discouraged because she is
experiencing pain with increasing
frequency. She states that she visits
an invalid friend twice a week and now
cannot walk up the second flight of
steps to the friend's apartment
without pain. Which of the following
measures that the nurse could suggest
would most likely help the client
prevent this problem?
1. Visit her friend early in the day.
2. Rest for at least an hour before
climbing the stairs.
3. Take a nitroglycerin tablet before
climbing the stairs.
4. Lie down once she reaches the
friend's apartment.
2 If a client displays risk factors for 3. A basic principle of behavior
1. coronary artery disease, such as modification is that behavior that is
smoking cigarettes, eating a diet high learned and continued is behavior that has
in saturated fat, or leading a been rewarded. Other reinforcement
sedentary lifestyle, techniques of techniques have not been found to be as
behavior modification may be used to effective as reward.
help the client change the behavior.
The nurse can best reinforce new
adaptive behaviors by:
1. Explaining how the old behavior
leads to poor health.
2. Withholding praise until the new
behavior is well established.
3. Rewarding the client whenever the
acceptable behavior is performed.
4. Instilling mild fear into the client to
extinguish the behavior.
2 In which of the following positions 3. Sitting almost upright in bed with the feet
2. should the nurse place a client and legs resting on the mattress decreases
with suspected heart failure? venous return to the heart, thus reducing
1. Semi-sitting (low Fowler's myocardial work-load. Also, the sitting position
position). allows maximum space for lung expansion. Low
2. Lying on the right side (Sims' Fowler's position would be used if the client
position). could not tolerate high Fowler's position for
3. Sitting almost upright (high some reason. Lying on the right side would not
Fowler's position). be a good position for the client in heart failure.
4. Lying on the back with the head The client in heart failure would not tolerate the
lowered (Trendelenburg's Trendelenburg's position.
position).
2 The major goal of therapy for a 1. Increasing cardiac output is the main goal of
3. client with heart failure and therapy for the client with heart failure or
pulmonary edema should be to: pulmo-nary edema. Pulmonary edema is an
1. Increase cardiac output. acute medical emergency requiring immediate
2. Improve respiratory status. intervention. Respi-ratory status and comfort
3. Decrease peripheral edema. will be improved when cardiac output increases
4. Enhance comfort. to an acceptable level. Peripheral edema is not
typically associated with pulmonary edema.
2 The nurse has completed an 2. A low urine output and confusion are signs of
4. assessment on a client with a decreased tissue perfusion. Orthopnea is a sign
decreased cardiac output. Which of left-sided heart failure. Crackles, edema and
fi nd-ings should receive the weight gain should be monitored closely, but
highest priority? the levels are not as high a priority. With atrial fi
1. BP 110/62, atrial fi brillation brillation there is a loss of atrial kick, but the
with HR 82, bibasilar crackles. blood pressure and heart rate are stable.
2. Confusion, urine output 15 mL
over the last 2 hours, orthopnea.
3. SpO2 92 on 2 liters nasal
cannula, respirations 20, 1+
edema of lower extremities.
4. Weight gain of 1 kg in 3 days,
BP 130/80, mild dyspnea with
exercise.
2 The nurse is admitting a 68-year- 1. The ankle edema suggests fl uid volume
5. old male to the medical floor. The overload. The nurse should assess respiratory
echocardiogram report revealed rate, lung sounds, and SpO2 to identify any
left ventricular enlargement. The signs of respiratory symptoms of heart failure
nurse notes 2+ pitting edema in requiring immediate attention. The nurse can
the ankles when getting the client then draw blood for laboratory studies, insert
into bed. Based on this finding, the Foley catheter, and weigh the client.
what should the nurse do first?
1. Assess respiratory status.
2. Draw blood for laboratory
studies.
3. Insert a Foley catheter.
4. Weigh the client.
2 A nurse is assessing a client with 1, 3, 5. When the heart begins to fail, the body
6. heart failure. The nurse should activates three major compensatory systems:
assess the client based on which ventricular hypertrophy, the renin-angiotensin-
compensatory mechanisms that aldosterone system, and sympathetic nervous
are activated in the presence of stimulation. Parasympathetic stimulation and
heart failure? Select all that jugular venous distention are not compensatory
apply. mechanisms associated with heart failure.
1. Ventricular hypertrophy.
2. Parasympathetic nervous
stimulation.
3. Renin-angiotensin-aldosterone
system.
4. Jugular venous distention.
5. Sympathetic nervous
stimulation
2 The nurse is assessing clients at a 4. The woman who is 65 years old, over-weight
7. health fair. Which client is at and has an elevated LDL is at greatest risk.
greatest risk for coronary artery Total cholesterol > 200, LDL > 100, HDL < 40
disease? in men, HDL < 50 in women, men 45 years and
1. A 32-year-old female with mitral older, women 55 years and older, smoking and
valve pro-lapse who quit smoking obesity increase the risk of CAD. Atorvastatin is
10 years ago. a medica-tion to reduce LDL and decrease risk
2. A 43-year-old male with a family of CAD. The combination of postmenopausal,
history of CAD and cholesterol obesity, and high LDL cholesterol places this
level of 158. client at greatest risk.
3. A 56-year-old male with an HDL
of 60 who takes atorvastatin
(Lipitor).
4. A 65-year-old female who is
obese with an LDL of 188.
2 The nurse is caring for a client 1. Infarction of the papillary muscles is a
8. diagnosed with an anterior potential complication of an MI causing
myocardial infarction 2 days ago. ineffective closure of the mitral valve during
Upon assessment, the nurse systole. Mitral regurgitation results when the
identifies a new systolic murmur left ventricle con-tracts and blood flows
at the apex. The nurse should backward into the left atrium, which is heard at
first: the fifth intercostal space, left midclavicular
1. Assess for changes in vital line. The murmur worsens during expiration
signs. and in the supine or left-side position. Vital sign
2. Draw an arterial blood gas. changes will reflect the severity of the sudden
3. Evaluate heart sounds with the drop in cardiac output: decrease in blood
client leaning forward. pressure, increase in heart rate, and increase in
4. Obtain a 12 Lead respirations. A 12-lead ECG views the electrical
electrocardiogram. activity of the heart; an echocardiogram views
valve function.
2 The nurse is tracking data on a 4. The goals of managing clients outside of the
9. group of clients with heart failure hospital are for the clients to maintain health
who have been discharged from and prevent readmission, thus interventions,
the hospital and are being such as monitoring and teaching appear to
followed at a clinic. Which of the have contributed to the low readmission rate in
following data indicate that this group of clients. Although it is important
nursing interventions of that clients do not gain weight, view
monitoring and teaching have educational material and continue to take their
been effective? medication, the primary indicator of
1. 90 percent of clients have not effectiveness of the program is the lack of re-
gained weight. hospitalization.
2. 75 percent of the clients viewed
the educational DVD.
3. 80 percent of the clients
reported that they are taking their
medications.
4. 5 percent of the clients required
hospitalization in the last 90 days.
3 The nurse notices that a client's 4. The nurse should fi rst assess the client's tol-
0. heart rate decreases from 63 to 50 erance to the drop in heart rate by checking
beats per minute on the monitor. the blood pressure and level of consciousness
The nurse should first: and determine if Atropine is needed. If the
1. Administer Atropine 0.5 mg I.V. client is symptomatic, Atropine and
push. transcutaneous pacing are interven-tions for
2. Auscultate for abnormal heart symptomatic bradycardia. Once the client is
sounds. stable, further physical assessments can be
3. Prepare for transcutaneous done.
pacing.
4. Take the client's blood pressure.
3 The nurse receives 4. Detection of myoglobin is one diagnostic tool to
1. emergency laboratory determine whether myocardial damage has occurred.
results for a client with Myoglobin is generally detected about 1 hour after a
chest pain and immediately heart attack is experienced and peaks within 4 to 6
informs the physician. An hours after infarction. Myoglobin does not help
increased myoglobin level diagnose cancer, hypertension, or liver disease.
suggests which of the
following?
1. Cancer.
2. Hypertension.
3. Liver disease.
4. Myocardial damage.
3 The nurse's discharge 3. Heart failure is a complex and chronic condition.
2. teaching plan for the client Education should focus on health promotion and
with heart failure should preventive care in the home environment. Signs and
stress the importance of symptoms can be monitored by the client. Instructing
which of the following? the client to obtain daily weights at the same time
1. Maintaining a high-fiber each day is very important. The client should be told
diet. to call the physician if there has been a weight gain of
2. Walking 2 miles every 2 lb. or more. This may indicate fluid overload, and
day. treatment can be prescribed early and on an
3. Obtaining daily weights outpatient basis, rather than waiting until the
at the same time each day. symptoms become life-threatening.
4. Remaining sedentary for
most of the day.
3 The nurse should be 4. A low serum potassium level (hypokalemia)
3. especially alert for signs predisposes the client to digoxin toxicity. Because
and symptoms of digoxin potassium inhibits cardiac excit-ability, a low serum
toxicity if serum levels potassium level would mean that the client would be
indicate that the client has prone to increased cardiac excitability. Sodium,
a: glucose, and calcium levels do not affect digoxin or
1. Low sodium level. contribute to digoxin toxicity.
2. High glucose level.
3. High calcium level.
4. Low potassium level.
3 The nurse should teach the 3. Colored vision and seeing yellow spots are
4. client that signs of digoxin symptoms of digoxin toxicity. Abdominal pain,
toxicity include which of the anorexia, nausea, and vomiting are other common
following? symptoms of digoxin toxicity. Additional signs of
1. Rash over the chest and toxicity include arrhythmias, such as atrial fi brilla-tion
back. or bradycardia. Rash, increased appetite, and elevated
2. Increased appetite. blood pressure are not associated with digoxin toxicity.
3. Visual disturbances such
as seeing yellow spots
4. Elevated BP.
3 An older, sedentary adult 1. In older adults who are less active and do not
5. may not respond to exercise the heart muscle, atrophy can result. Disuse
emotional or physical stress or deconditioning can lead to abnormal changes in the
as well as a younger myocardium of the older adult. As a result, under
individual because of: sudden emotional or physical stress, the left ventricle
1. Left ventricular atrophy. is less able to respond to the increased demands on
2. Irregular heartbeats. the myocardial muscle. Decreased car-diac output,
3. Peripheral vascular cardiac hypertrophy, and heart failure are examples of
occlusion. the chronic conditions that may develop in response to
4. Pacemaker placement. inactivity, rather than in response to the aging
process.
3 The physician orders continuous 1. I.V. nitroglycerin infusion requires an infusion
6. I.V. nitro-glycerin infusion for the pump for precise control of the medica-tion.
client with myocardial infarction. Blood pressure monitoring would be done with a
Essential nursing actions include continuous system, and more frequently than
which of the following? every 4 hours. Hourly urine outputs are not
1. Obtaining an infusion pump for always required. Obtaining serum potassium
the medication. levels is not associated with nitroglycerin
2. Monitoring blood pressure infusion.
every 4 hours.
3. Monitoring urine output
hourly.
4. Obtaining serum potassium
levels daily.
3 The physician refers the client 2. Cardiac catheterization is done in clients with
7. with unstable angina for a angina primarily to assess the extent and
cardiac catheterization. The severity of the coronary artery blockage. A
nurse explains to the client that decision about medical management,
this procedure is being used in angioplasty, or coronary artery bypass surgery
this specific case to: will be based on the catheterization results.
1. Open and dilate blocked
coronary arteries.
2. Assess the extent of arterial
blockage.
3. Bypass obstructed vessels.
4. Assess the functional
adequacy of the valves and heart
muscle.
3 Sublingual nitroglycerin tablets 3. The correct protocol for nitroglycerin use
8. begin to work within 1 to 2 involves immediate administration, with
minutes. How should the nurse subsequent doses taken at 5-minute intervals as
instruct the client to use the drug needed, for a total dose of three tablets.
when chest pain occurs? Sublingual nitroglycerin appears in the
1. Take one tablet every 2 to 5 bloodstream within 2 to 3 minutes and is
minutes until the pain stops. metabolized within about 10 minutes.
2. Take one tablet and rest for 10
minutes. Call the physician if
pain persists after 10 minutes.
3. Take one tablet, then an
additional tablet every 5 minutes
for a total of three tablets. Call
the physician if pain persists
after three tablets.
4. Take one tablet. If pain
persists after 5 minutes, take
two tablets. If pain still persists 5
minutes later, call the physician.
3 When administering a Thrombolytic drugs are administered within the
9. thrombolytic drug to the client fi rst 6 hours after onset of an MI to lyse clots
experiencing a myocardial and reduce the extent of myocardial damage.
infarction (MI), the nurse
explains that the purpose of the
drug is to:
1. Help keep him well hydrated.
2. Dissolve clots that he may
have.
3. Prevent kidney failure.
4. Treat potential cardiac
arrhythmias
4 When monitoring a client who is 1. Cardiac arrhythmias are commonly observed
0. receiving tissue plasminogen with administration of t-PA. Cardiac arrhythmias
activator (t-PA), the nurse should are associated with reperfusion of the cardiac
have resuscitation equipment tissue. Hypotension is commonly observed with
available because reperfusion of administra-tion of t-PA. Seizures and
the cardiac tissue can result in hypothermia are not gener-ally associated with
which of the following? reperfusion of the cardiac tissue.
1. Cardiac arrhythmias.
2. Hypertension.
3. Seizure.
4. Hypothermia.
4 When teaching a client with heart 1, 2, 4. The client stating that he would call the
1 failure about preventing physician with increasing shortness of breath,
. complications and future weight gain over 2 lb in 1 day, and having to
hospitalizations, which problems sleep sitting up, indicates that he has
stated by the client as reasons to understood the teaching because these signs
call the physician would indicate and symptoms suggest worsening of the client's
to the nurse that the client has heart failure. Although the client will most likely
understood the teaching? Select be placed on a sodium-restricted diet, the client
all that apply. would not need to notify the physician if he or
1. Becoming increasingly short of she had consumed a high-sodium breakfast.
breath at rest. Instead the client would need to be alert for
2. Weight gain of 2 lb or more in 1 possible signs and symptoms of worsening heart
day. failure and work to reduce sodium intake for the
3. High intake of sodium for rest of that day and in the future.
breakfast.
4. Having to sleep sitting up in a
reclining chair.
5. Weight loss of 2 lb in 1 day.
4 When teaching the client with 4. An MI interferes with or blocks blood
2 myocardial infarction (MI), the circulation to the heart muscle. Decreased blood
. nurse explains that the pain supply to the heart muscle causes ischemia, or
associated with MI is caused by: poor myocardial oxygenation. Diminished
1. Left ventricular overload. oxygenation or lack of oxygen to the cardiac
2. Impending circulatory muscle results in ischemic pain or angina.
collapse.
3. Extracellular electrolyte
imbalances.
4. Insufficient oxygen reaching
the heart muscle.
4 Which activity would be 4. Unlicensed personnel are able to measure and
3 appropriate to delegate to record intake and output. The nurse is respon-
. unlicensed personnel for a client sible for client teaching, physical assessments,
diagnosed with a myocardial and evaluating the information collected on the
infarction who is stable? client.
1. Evaluate the lung sounds.
2. Help the client identify risk
factors for CAD.
3. Provide teaching on a 2 g
sodium diet.
4. Record the intake and output.
4 Which of the following is an . By day 2 of hospitalization after an MI, cli-ents
4 expected out-come for a client on are expected to be able to perform personal care
. the second day of hospitalization without chest pain. Severe chest pain should not
after a myocardial infarction (MI)? be present on day 2 after and MI. Day 2 of
The client: hospitaliza-tion may be too soon for clients to be
1. Has severe chest pain. able to identify risk factors for MI or to begin a
2. Can identify risk factors for MI. walking program; however, the client may be
3. Agrees to participate in a sitting up in a chair as part of the cardiac
cardiac rehabilitation walking rehabilitation program.
program.
4. Can perform personal self-care
activities with-out pain.
4 Which of the following is not a 2. Late onset of puberty is not generally con-
5 risk factor for the development of sidered to be a risk factor for the development of
. atherosclerosis? ath-erosclerosis. Risk factors for atherosclerosis
1. Family history of early heart include family history of atherosclerosis,
attack. cigarette smoking, hypertension, high blood
2. Late onset of puberty. cholesterol level, male gen-der, diabetes
3. Total blood cholesterol level mellitus, obesity, and physical inactivity.
greater than 220 mg/dL.
4. Elevated fasting blood glucose
concentration.
4 Which of the following nursing 1, 2, 3, 5. A decrease in cardiac output occurs
6 diagnoses would be appropriate from a decreased stroke volume with impaired
. for a client with systolic heart contractility in systolic heart failure. This impairs
failure? Select all that apply. peripheral and renal perfusion. The impaired
1. Ineffective peripheral tissue perfusion and impaired oxygenation cause the
perfusion related to a decreased symptoms of activity intolerance. The decreased
stroke volume. systolic function causes an increase in residual
2. Activity intolerance related to volume and pressure in the left ventricle. A
impaired gas exchange and retrograde buildup of pressure from the left
perfusion. ventricle to left atria increases hydrostatic
3. Dyspnea related to pulmonary pressure in the pulmonary vasculature. This
congestion and impaired gas causes a leakage of fluid into the interstitial
exchange. tissue of the lungs resulting in pulmonary
4. Decreased cardiac output symptoms. With diastolic heart failure, there is
related to impaired cardiac impaired ventricular filling due to a rigid ventricle
filling. and reduced ventricular relaxation.
5. Impaired renal perfusion
related to a decreased cardiac
output.
4 Which of the following reflects 2. Recommended dietary principles in the acute
7 the principle on which a client's phase of MI include avoiding large meals because
. diet will most likely be based small, easily digested foods are better tolerated.
during the acute phase of
myocardial infarction?
1. Liquids as desired.
2. Small, easily digested meals.
3. Three regular meals per day.
4. Nothing by mouth.

Question 1 See full question

The nurse is assisting another member of the health care team who is
placing a peripherally inserted catheter in a 10-year-old with peritonitis from
a ruptured appendix. The family is present in the treatment room to support
the child. The nurse observes the other team member has contaminated a
sterile glove. The nurse should:
You Selected:

tell the team member the glove is contaminated.


Correct response:

tell the team member the glove is contaminated.


Explanation:

Question 2 See full question

After a plaster cast has been applied to the arm of a child with a fractured
right humerus, the nurse completes discharge teaching. The nurse should
evaluate the teaching as successful when the mother agrees to seek medical
advice if the child experiences which symptom?
You Selected:

inability to extend the fingers on the right hand


Correct response:

inability to extend the fingers on the right hand


Explanation:

Question 3 See full question

The nurse develops a teaching plan for a client newly diagnosed with
Parkinsons disease. Which topic is most important to include in the plan?
You Selected:

maintaining a balanced nutritional diet


Correct response:

maintaining a safe environment


Explanation:

Question 4 See full question

Which symptom should the nurse teach the client with unstable angina to
report immediately to the health care provider (HCP)?
You Selected:

a change in the pattern of the chest pain


Correct response:

a change in the pattern of the chest pain


Explanation:

Question 5 See full question

A client admitted to the hospital with peptic ulcer disease tells the nurse
about having black, tarry stools. The nurse should:
You Selected:

encourage the client to increase fluid intake.


Correct response:

report the finding to the health care provider (HCP).


Explanation:

Question 48 See full question

A nurse is caring for a 14-year-old client in skeletal traction to the left leg. The client is reporting pain on the 0 to 10
pain scale of 8. Which action would the nurse take first?

You Selected:

Medicate for pain.

Correct response:

Realign the client in bed.

Explanation:

Question 49 See full question

A client receiving chemotherapy is nauseated and has lost 15 pounds (6.8 kg) in one month. Which nutritional
instruction would the nurse include in the plan of care?

You Selected:

Eat hot or cold foods

Correct response:

Eat frequent but small meals

Explanation:

Question 50 See full question

The nurse is planning care for a client who has been experiencing a manic episode for 6 days and is unable to sit
still long enough to eat meals. Which choice will best meet the clients nutritional needs at this time?

You Selected:

Offer a peanut butter sandwich.

Correct response:

Offer a peanut butter sandwich.

Explanation:

Question 1 See full question

A client rates the pain level of a migraine an 8 on a scale of 1-10. How would the nurse administer the medication
to give the client the quickest relief?

You Selected:

Sublingual

Correct response:

Intravenous (IV)

Explanation:

Question 2 See full question

A nurse is to give a client a 325-mg aspirin suppository. The client has diarrhea and is in the bathroom. The best
nursing approach at this time would be to:
You Selected:

substitute 325-mg aspirin by mouth.

Correct response:

withhold the suppository and notify the client's physician.

Explanation:

Question 3 See full question

A client receiving morphine for long-term pain management develops tolerance. Tolerance is defined as:

You Selected:

an ability to take the same drug for extended periods.

Correct response:

a diminished response to a drug so that more medication is required to achieve the same effect.

Explanation:

Question 4 See full question

A 2-year-old child with a low blood level of the immunosuppressive drug cyclosporine comes to a liver transplant
clinic for her appointment. The mother says the child hasn't been vomiting and hasn't had diarrhea, but she admits
that her daughter doesn't like taking the liquid medication. Which statement by the nurse is most appropriate?

You Selected:

"Offer the medication diluted with chocolate milk or orange juice to make it more palatable."

Correct response:

"Offer the medication diluted with chocolate milk or orange juice to make it more palatable."

Explanation:

Question 5 See full question

Teaching for women of childbearing years who are receiving antipsychotic medications includes which statement?

You Selected:

This medication may result in heightened libido.

Correct response:

Continue previous contraceptive use even if you're experiencing amenorrhea.

Explanation:

Question 6 See full question

Which client statement indicates the need for additional teaching about benzodiazepines?

You Selected:

"Diazepam will help my tight muscles feel better."

Correct response:

"I can stop taking the drug anytime I want."

Explanation:

Question 7 See full question

The antidote for heparin is:


You Selected:

vitamin K.

Correct response:

protamine sulfate.

Explanation:

Question 8 See full question

When administering blood, the nurse must check the name on the label of the blood with the name on the client's:

You Selected:

Wristband with a family member present.

Correct response:

Wristband in the presence of another nurse.

Explanation:

Question 9 See full question

During an emergency, a physician has asked for I.V. calcium to treat a client with hypocalcemia. The nurse should:

You Selected:

Hand the physician calcium gluconate for I.V. use.

Correct response:

Check with the physician for his complete order.

Explanation:

Question 10 See full question

The mother of an older infant reports stopping the prescribed iron supplements after 2 weeks of treatment. Which
response by the nurse is most appropriate?

You Selected:

"You need to continue the iron for several more weeks."

Correct response:

"You need to continue the iron for several more weeks."

Explanation:

Question 11 See full question

The health care provider (HCP) has prescribed nitroglycerin to a client with angina. The client also has closed-angle
glaucoma. The nurse contacts the HCP to discuss the potential for:

You Selected:

increased intraocular pressure.

Correct response:

increased intraocular pressure.


Explanation:

Question 12 See full question

A client receives an IV dose of gentamicin sulfate. How long after the completion of the dose should the peak serum
concentration level be measured?

You Selected:

10 minutes

Correct response:

30 minutes

Explanation:

Question 13 See full question

The nurse is caring for a child receiving a blood transfusion. The child becomes flushed and is wheezing. What
should the nurse do first?

You Selected:

Take the child's vital signs.

Correct response:

Switch the transfusion to normal saline solution.

Explanation:

Question 14 See full question

Clients who are receiving total parenteral nutrition (TPN) are at risk for development of which complication?

You Selected:

pulmonary hypertension

Correct response:

fluid imbalances

Explanation:

Question 15 See full question

A 75-year-old client who has been taking furosemide regularly for 4 months tells the nurse about having trouble
hearing. What should the nurse do?

You Selected:

Report the hearing loss to the health care provider (HCP).

Correct response:

Report the hearing loss to the health care provider (HCP).

Explanation:

Question 16 See full question

The nurse learns that a client who is scheduled for a tonsillectomy has been taking 40 mg of oral prednisone daily
for the last week for poison ivy on the leg. What should the nurse do first?

You Selected:

Send the client to surgery.


Correct response:

Notify the anesthesiologist of the prednisone administration.

Explanation:

Question 17 See full question

A client had a total abdominal hysterectomy and bilateral oophorectomy for ovarian carcinoma yesterday. She
received 2 mg of morphine sulfate I.V. by patient-controlled analgesia (PCA) 10 minutes ago. The nurse was
assisting her from the bed to a chair when the client felt dizzy and fell into the chair. The nurse should:

You Selected:

take the client's blood pressure.

Correct response:

take the client's blood pressure.

Explanation:

Question 18 See full question

A 52-year-old male was discharged from the hospital for cancer-related pain. His pain appeared to be well controlled
on the IV morphine. He was switched to oral morphine when discharged 2 days ago. He now reports his pain as an 8
on a 10-point scale and wants the IV morphine. Which explanation is the most likely for the clients reports of
inadequate pain control?

You Selected:

He is physically dependent on the IV morphine.

Correct response:

He is undermedicated on the oral opioid.

Explanation:

Question 19 See full question

The nurse is assessing the clients understanding of the use of medications. Which medication may cause a
complication with the treatment plan of a client with diabetes?

You Selected:

angiotensin-converting enzyme (ACE) inhibitors

Correct response:

steroids

Explanation:

Question 20 See full question

A client is receiving total parenteral nutrition (TPN) solution. The nurse should assess a clients ability to metabolize
the TPN solution adequately by monitoring the client for which sign?

You Selected:

hypertension

Correct response:

hyperglycemia
Explanation:

Question 21 See full question

When teaching a client about propranolol hydrochloride, the nurse should base the information on the knowledge
that propranolol:

You Selected:

increases norepinephrine secretion and thus decreases blood pressure and heart rate.

Correct response:

blocks beta-adrenergic stimulation and thus causes decreased heart rate, myocardial contractility, and
conduction.

Explanation:

Question 22 See full question

A client is receiving magnesium sulfate at 3 g/h intravenously. The bag of 1,000 mL normal saline contains 20 g of
magnesium sulfate. How many mL/hour should the nurse set the IV pump rate in order to deliver 3 g/h? Record your
answer using a whole number.

Your Response:

55

Correct response:

150

Explanation:

Question 23 See full question

When preparing a teaching plan for a client who is to receive a rubella vaccine during the postpartum period, the
nurse should include which information?

You Selected:

The vaccine prevents a future fetus from developing congenital anomalies.

Correct response:

Pregnancy should be avoided for 4 weeks after the immunization.

Explanation:

Question 24 See full question

The client with Alzheimer's disease has been prescribed donepezil 5 mg at bedtime. Which instruction should the
nurse give to the client's daughter?

You Selected:

Avoid suddenly stopping the medication.

Correct response:

Avoid suddenly stopping the medication.

Explanation:

Question 25 See full question

A 20-year-old client visiting the clinic requests the use of oral contraceptives. When reviewing the clients history,
which finding would alert the nurse to a possible contraindication to using these agents?
You Selected:

ulcerative colitis

Correct response:

thrombophlebitis

Explanation:

Question 26 See full question

When developing a teaching plan for the parents of a 1 1/2-month-old infant about how to administer levothyroxine,
what should the nurse suggest as most appropriate for dissolving and mixing the medication?

You Selected:

small amount of formula or breast milk

Correct response:

small amount of formula or breast milk

Explanation:

Question 27 See full question

In teaching a client with tuberculosis about self-care at home, which directive has the highest priority?

You Selected:

Take medications as prescribed.

Correct response:

Take medications as prescribed.

Explanation:

Question 28 See full question

After 5 days of hospitalization, a client who is receiving morphine sulfate for pain control asks for pain medication
with increasing frequency and exhibits increased anxiety and restlessness. The vital signs are within normal ranges.
What is a possible cause of this behavior?

You Selected:

The client is addicted to the morphine.

Correct response:

The client has developed tolerance to the dose of morphine.

Explanation:

Question 29 See full question

The nurse has given a client a nitroglycerin tablet sublingually for angina. Which vital signs should be assessed
following administration of nitroglycerin?

You Selected:

pulse rate

Correct response:

blood pressure
Explanation:

Question 30 See full question

When a central venous catheter dressing becomes moist or loose, what should a nurse do first?

You Selected:

Draw a circle around the moist spot and note the date and time.

Correct response:

Remove the dressing, clean the site, and apply a new dressing.

Explanation:

Question 31 See full question

The nurse is caring for a client in the intensive care unit. Which drug is most commonly used to treat cardiogenic
shock?

You Selected:

Dopamine

Correct response:

Dopamine

Explanation:

Question 32 See full question

A few minutes after beginning a blood transfusion, a nurse notes that a client has chills, dyspnea, and urticaria. The
nurse reports this to the physician immediately because the client probably is experiencing which problem?

You Selected:

A hemolytic reaction to Rh-incompatible blood

Correct response:

A hemolytic allergic reaction caused by an antigen reaction

Explanation:

Question 33 See full question

A nurse is teaching a client with type 1 diabetes how to treat adverse reactions to insulin. To reverse hypoglycemia,
the client ideally should ingest an oral carbohydrate. However, this treatment isn't always possible or safe.
Therefore, the nurse should advise the client to keep which alternate treatment on hand?

You Selected:

Epinephrine

Correct response:

Glucagon

Explanation:

Question 34 See full question

For a client with hyperthyroidism, treatment is most likely to include:

You Selected:

a thyroid hormone antagonist.


Correct response:

a thyroid hormone antagonist.

Explanation:

Question 35 See full question

A client is admitted to an acute care facility after an episode of status epilepticus. After the client is stabilized,
which factor is most beneficial in determining the potential cause of the episode?

You Selected:

Recent stress level

Correct response:

Compliance with the prescribed medication regimen

Explanation:

Question 36 See full question

A client develops decreased renal function and requires a change in antibiotic dosage. On which factor should the
physician base the dosage change?

You Selected:

Liver function studies

Correct response:

Creatinine clearance

Explanation:

Question 37 See full question

Assessment of a client taking lithium reveals dry mouth, nausea, thirst, and mild hand tremor. Based on an analysis
of these findings, which of the following should the nurse do next?

You Selected:

Continue the lithium, and immediately notify the health care provider (HCP) about the assessment findings.

Correct response:

Continue the lithium, and reassure the client that these temporary side effects will subside.

Explanation:

Question 38 See full question

Prophylactic heparin therapy is prescribed to treat thrombophlebitis in a multiparous client who gave birth 24 hours
ago. After instructing the client about the medication, the nurse determines that the client understands the
instructions when she states which as the purpose of the drug?

You Selected:

to prevent further blood clot formation

Correct response:

to prevent further blood clot formation

Explanation:

Question 39 See full question

A full-term client is admitted for an induction of labor. The health care provider (HCP) has assigned a Bishop score of
10. Which drug would the nurse anticipate administering to this client?
You Selected:

dinoprostone 10 mg

Correct response:

oxytocin 30 units in 500 ml D5W

Explanation:

Question 40 See full question

A toddler with croup is given a racemic epinephrine treatment because of increasing respiratory distress. The nurse
evaluates the treatment as being effective when the childs:

You Selected:

heart rate is 100 bpm.

Correct response:

retractions are less severe.

Explanation:

Question 41 See full question

A client with depression states, "I am still feeling nauseous after I take venlafaxine. Maybe I need something else."
The nurse should tell the client to:

You Selected:

take venlafaxine before bedtime.

Correct response:

take the medication at mealtime.

Explanation:

Question 42 See full question

Which statement made by an adolescent who has just begun taking an antidepressant would indicate the need for
further teaching?

You Selected:

"After a week of taking my antidepressant, I can sleep a little better6 hours or so each night."

Correct response:

"Now that I have been taking my antidepressant for 1 week, I am going to feel better about myself."

Explanation:

Question 43 See full question

A client is brought to the emergency department unconscious. An empty bottle of aspirin was found in the car, and
a drug overdose is suspected. Which medication should the nurse have available for further emergency treatment?

You Selected:

vitamin K

Correct response:

activated charcoal powder


Explanation:

Question 44 See full question

A client who has been taking flunisolide nasal spray, two inhalations a day, for treatment of asthma has painful,
white patches in the mouth. What should the nurse tell the client?

You Selected:

"Be sure to brush your teeth and floss daily. Good oral hygiene will treat this problem."

Correct response:

"You have developed a fungal infection from your medication. It will need to be treated with an antifungal
agent."

Explanation:

Question 45 See full question

The health care provider prescribes sulfasalazine for the client with ulcerative colitis. Which instruction should the
nurse give the client about taking this medication?

You Selected:

Take the total dose at bedtime.

Correct response:

Take it with a full glass (240 mL) of water.

Explanation:

Question 46 See full question

The nurse receives a physician's order to administer 1,000 mL of intravenous (IV) normal saline solution over 8
hours to a client who recently had a stroke. What should the drip rate be if the drop factor of the tubing is 15
gtt/mL? Record your answer using a whole number.

Your Response:

33

Correct response:

31

Explanation:

Question 47 See full question

A child with iron deficiency anemia was prescribed ferrous sulfate. Which statement by the parent would indicate a
need for further instruction on proper administration?

You Selected:

I encourage my child to drink lots of fluids.

Correct response:

I mix the medication in milk to make it taste better.

Explanation:

Question 48 See full question

The nurse has completed client instruction about lorazepam. Which of the following client statements would
indicate that the client understands?
You Selected:

My anxiety will be eliminated if I take this medication as prescribed.

Correct response:

This medication will help me relax so that I can focus on problem solving.

Explanation:

Question 49 See full question

A 74-year-old client receiving fluphenazine decanoate therapy develops pseudoparkinsonism, and is ordered
amantadine hydrochloride. With the addition of this medication, the client reports feeling dizzy when standing.
Which response by the nurse is best?

You Selected:

Do you have any slurred speech or weakness in one extremity?

Correct response:

When you change positions, do so slowly.

Explanation:

Question 50 See full question

The nurse is collaborating with the health care provider (HCP) to develop a care plan to help control chronic pain in
a client with cancer who is receiving hospice home care. Which plan is most appropriate for preventing and
reducing the clients pain?

You Selected:

Encourage the client to avoid intravenous pain medication until the condition has reached the terminal stage.

Correct response:

Administer analgesics on a regular basis with administration of additional analgesics for breakthrough pain.

Explanation:

Question 1 See full question

A physician orders codeine, grain every 4 hours, for a client experiencing pain. How many milligrams of codeine
should the nurse administer?

You Selected:

15 mg

Correct response:

30 mg

Explanation:

Question 2 See full question

A client begins taking haloperidol. After a few days, he experiences severe tonic contractures of muscles in his
neck, mouth, and tongue. The nurse should recognize this as:

You Selected:

dystonia.

Correct response:

dystonia.
Explanation:

Question 3 See full question

A physician writes a medication order for meperidine 500 mg. The nurse's appropriate action would be to:

You Selected:

clarify the order with the physician.

Correct response:

clarify the order with the physician.

Explanation:

Question 4 See full question

A 2-year-old child with a low blood level of the immunosuppressive drug cyclosporine comes to a liver transplant
clinic for her appointment. The mother says the child hasn't been vomiting and hasn't had diarrhea, but she admits
that her daughter doesn't like taking the liquid medication. Which statement by the nurse is most appropriate?

You Selected:

"Offer the medication diluted with chocolate milk or orange juice to make it more palatable."

Correct response:

"Offer the medication diluted with chocolate milk or orange juice to make it more palatable."

Explanation:

Question 5 See full question

How should a nurse prepare a suspension before administration?

You Selected:

By diluting it with normal saline solution

Correct response:

By shaking it so that all the drug particles are dispersed uniformly

Explanation:

Question 6 See full question

A client is admitted to the local psychiatric facility with bipolar disorder in the manic phase. The physician decides
to start the client on lithium carbonate therapy. One week after this therapy starts, the nurse notes that the client's
serum lithium level is 1 mEq/L. What should the nurse do?

You Selected:

Continue to administer the medication as ordered.

Correct response:

Continue to administer the medication as ordered.

Explanation:

Question 7 See full question

During labor, a primigravid client receives an epidural anesthetic, and the nurse assists in monitoring maternal and
fetal status. Which finding suggests an adverse reaction to the anesthesia?

You Selected:

Maternal hypotension
Correct response:

Maternal hypotension

Explanation:

Question 8 See full question

A nurse is teaching a group of clients about birth control methods. When providing instruction about subdermal
contraceptive implants, the nurse should cite which feature as the main advantage of this method?

You Selected:

The implants cost less over the long term than other contraceptive methods.

Correct response:

The implants provide effective, continuous contraception that isn't user dependent.

Explanation:

Question 9 See full question

A client with primary diabetes insipidus is ready for discharge on desmopressin (DDAVP). Which instruction should
the nurse provide?

You Selected:

"You won't need to monitor your fluid intake and output after you start taking desmopressin."

Correct response:

"You may not be able to use desmopressin nasally if you have nasal discharge or blockage."

Explanation:

Question 10 See full question

After an eye examination, a client is diagnosed with open-angle glaucoma. The physician orders pilocarpine
ophthalmic solution, 0.25% gtt i, OU q.i.d. Based on this prescription, the nurse should teach the client or a family
member to administer the drug by:

You Selected:

instilling one drop of pilocarpine 0.25% into both eyes daily.

Correct response:

instilling one drop of pilocarpine 0.25% into both eyes four times daily.

Explanation:

Question 11 See full question

A client diagnosed with schizophrenia is being switched to risperidone long-acting injection. He is told that he will
remain on his oral dose of risperidone daily for approximately 1 month. The client says, "I did not have to take pills
when I was on fluphenazine shots in the past." The nurse should tell the client:

You Selected:

"Your health care provider did not believe you would take both the pills and fluphenazine injections."

Correct response:

"Risperidone long-acting injection initially takes a little longer to reach the ideal blood level."

Explanation:

Question 12 See full question

The antidote for heparin is:


You Selected:

thrombin.

Correct response:

protamine sulfate.

Explanation:

Question 13 See full question

The nurse should dispose of a used needle and syringe by:

You Selected:

Separating the needle and syringe and placing both in the precaution container in the client's room.

Correct response:

Placing uncapped, used needles and syringes immediately in the universal precaution container in the client's
room.

Explanation:

Question 14 See full question

A woman is taking oral contraceptives. The nurse teaches the client that medications that may interfere with oral
contraceptive efficacy include:

You Selected:

antibiotics.

Correct response:

antibiotics.

Explanation:

Question 15 See full question

A client takes isosorbide dinitrate as an antianginal medication. Which statement indicates that the client
understands the adverse effects of the drug?

You Selected:

"I will need to change positions slowly so I will not get dizzy."

Correct response:

"I will need to change positions slowly so I will not get dizzy."

Explanation:

Question 16 See full question

The nurse is preparing to start an IV infusion. Before inserting the needle into a vein, the nurse should apply a
tourniquet to the clients arm to:

You Selected:

stabilize the veins.

Correct response:

distend the veins.


Explanation:

Question 17 See full question

Nursing responsibilities for the client with a patient-controlled analgesia (PCA) system include:

You Selected:

instructing the client to continue pressing the system's button whenever pain occurs.

Correct response:

documenting the client's response to pain medication.

Explanation:

Question 18 See full question

Which medication should be available to provide emergency treatment if a client develops tetany after a subtotal
thyroidectomy?

You Selected:

echothiophate iodide

Correct response:

calcium gluconate

Explanation:

Question 19 See full question

A client with diabetes is taking insulin lispro injections. The nurse should advise the client to eat:

You Selected:

within 10 to 15 minutes after the injection.

Correct response:

within 10 to 15 minutes after the injection.

Explanation:

Question 20 See full question

A client is receiving pentoxifylline for intermittent claudication. The nurse should determine the effectiveness of the
drug by asking if the client:

You Selected:

is urinating more frequently.

Correct response:

has less pain in the legs.

Explanation:

Question 21 See full question

A client with deep vein thrombosis has been receiving warfarin for 2 months. The client is to go to an anticoagulant
monitoring laboratory every 3 weeks. The last visit to the laboratory was 2 weeks ago. The client reports bleeding
gums, increased bruising, and dark stools. What should the nurse should instruct the client to do?

You Selected:

Return to laboratory for analysis of prothrombin times.


Correct response:

Return to laboratory for analysis of prothrombin times.

Explanation: Question 1 See full question

What is the main advantage of using a floor stock system?

You Selected:

The system reinforces accurate calculations.

Correct response:

A nurse can implement medication orders quickly.

Explanation:

Question 2 See full question

Which toxic adverse reaction should the nurse monitor for in a toddler taking
digoxin?

You Selected:

Tachycardia

Correct response:

Nausea and vomiting

Explanation:

Question 3 See full question

A nurse is teaching a client about hormonal contraceptive therapy. If a client misses


three or more pills in a row, the nurse should instruct the client to:

You Selected:

discard the pack, use an alternative contraceptive method until her period begins, and
start a new pack on the regular schedule.

Correct response:

discard the pack, use an alternative contraceptive method until her period begins, and
start a new pack on the regular schedule.
Explanation:

Question 4 See full question

A client is receiving pentoxifylline for intermittent claudication. The nurse should


determine the effectiveness of the drug by asking if the client:

You Selected:

has less pain in the legs.

Correct response:

has less pain in the legs.

Explanation:

Question 5 See full question

The nurse is evaluating a client with hyperthyroidism who is taking propylthiouracil


(PTU) 100 mg/day in three divided doses for maintenance therapy. Which
statement from the client indicates the drug is effective?

You Selected:

"I am able to sleep and rest at night."

Correct response:

"I am able to sleep and rest at night."

Explanation:

Question 1 See full question

While the nurse is caring for a neonate at 32 weeks' gestation in an isolette with
continuous oxygen administration, the neonate's mother asks why the neonate's
oxygen is humidified. The nurse should tell the mother?

You Selected:

"Oxygen is drying to the mucous membranes unless it is humidified."

Correct response:

"Oxygen is drying to the mucous membranes unless it is humidified."


Explanation:

Question 2 See full question

A client who is experiencing alcohol withdrawal exhibits tremors, diaphoresis, and


hyperactivity. Blood pressure is 190/87 mm Hg, and pulse is 92 bpm. Which
medication should the nurse expect to administer?

You Selected:

lorazepam

Correct response:

lorazepam

Explanation:

Question 3 See full question

A clients serum ammonia level is elevated, and the health care provider (HCP)
prescribes 30 mL of lactulose. Which effect is common for this drug?

You Selected:

increased bowel movements

Correct response:

increased bowel movements

Explanation:

Question 4 See full question

Which is a priority assessment for the client in shock who is receiving an IV infusion
of packed red blood cells and normal saline solution?

You Selected:

anaphylactic reaction

Correct response:

anaphylactic reaction
Explanation:

Question 5 See full question

A client with marked oliguria is ordered a test dose of 0.2 g/kg of 15% mannitol
solution intravenously over 5 minutes. The client weighs 132 lb. How many grams
would the nurse administer? Record your answer as a whole number.

Your Response:

26

Correct response:

12

Explanation:

First, convert the clients weight from pounds to kilograms:

132 lb 2.2 lb/kg = 60 kg.

Then, to calculate the number of grams to administer, multiply the ordered number
of grams by the clients weight in kilograms:

0.2g/kg X 60 kg = 12 g.

Question 1 See full question

A physician writes a medication order for meperidine 500 mg. The nurse's
appropriate action would be to:

You Selected:

clarify the order with the physician.

Correct response:

clarify the order with the physician.


Explanation:

Question 2 See full question

A client will be discharged on lithium carbonate 600 mg three times daily. When
teaching the client and his family about lithium therapy, the nurse determines that
teaching has been effective if the client and family state that they will notify the
prescribing health care provider (HCP) immediately which symptoms? Select all that
apply.

You Selected:

nausea
vomiting
muscle weakness

Correct response:

muscle weakness
vertigo
vomiting

Explanation:

Question 3 See full question

A 3-year-old child is to receive 500 ml of dextrose 5% in normal saline (D 5NSS)


solution over 8 hours. At what rate (in milliliters/hour) would the nurse set the
infusion pump? Record your answer using one decimal place.

Your Response:

62.5

Correct response:

62.5

Explanation:

Question 4 See full question

An expected outcome of theophylline ethylenediamine when administered to a


client with chronic obstructive pulmonary disease is:
You Selected:

relax bronchial smooth muscle.

Correct response:

relax bronchial smooth muscle.

Explanation:

Question 5 See full question

A client whose symptoms of schizophrenia are under control with olanzapine, and
who is functioning at home and in her part-time employment, states that she is
very concerned about her 20-lb (9.1-kg) weight gain since she started taking the
medication 6 months ago. The nurse should:

You Selected:

suggest that the client talk with her healthcare provider about changing to another
antipsychotic.

Correct response:

discuss nutrition, daily diet, and exercise with the client.

Explanation:

Question 1 See full question

A client with deep vein thrombosis has been receiving warfarin for 2 months. The
client is to go to an anticoagulant monitoring laboratory every 3 weeks. The last visit
to the laboratory was 2 weeks ago. The client reports bleeding gums, increased
bruising, and dark stools. What should the nurse should instruct the client to do?

You Selected:

Return to laboratory for analysis of prothrombin times.

Correct response:

Return to laboratory for analysis of prothrombin times.


Explanation:

Question 2 See full question

A client is to receive 1,000 mL of lactated Ringers (LR) over 10 hours. The drip factor
is 15 drops (gtts)/mL. How many gtts per minute should the client receive?

You Selected:

25 gtts/min

Correct response:

25 gtts/min

Explanation:

Question 3 See full question

An elderly male client has been taking doxazosin 2 mg daily for 4 weeks for
treatment of benign prostatic hypertrophy. The client reports feeling dizzy. The
nurse should first:

You Selected:

take his blood pressure lying, standing, and sitting.

Correct response:

take his blood pressure lying, standing, and sitting.

Explanation:

Question 4 See full question

Metoprolol is added to the pharmacologic therapy of a woman with diabetes


diagnosed with stage 2 hypertension and initially treated with furosemide and
ramipril. An expected therapeutic effect is:

You Selected:

decrease in heart rate.

Correct response:

decrease in heart rate.


Explanation:

Question 5 See full question

The nurse understands that assessment of blood pressure in clients receiving


antipsychotic drugs is important. What is a reason for this assessment?

You Selected:

Orthostatic hypotension is a common side effect.

Correct response:

Orthostatic hypotension is a common side effect.

Explanation:

Question 1 See full question

A physician orders prednisone to control inflammation in a client with interstitial


lung disease. During client teaching, the nurse stresses the importance of taking
prednisone exactly as ordered and cautions against discontinuing the drug
abruptly. A client who discontinues prednisone abruptly may experience:

You Selected:

acute adrenocortical insufficiency.

Correct response:

acute adrenocortical insufficiency.

Explanation:

Question 2 See full question

The nurse is reviewing laboratory reports for a client who is taking allopurinol.
Which finding indicates that the drug has had a therapeutic effect?

You Selected:

decreased serum uric acid level

Correct response:

decreased serum uric acid level


Explanation:

Question 3 See full question

The nurse is preparing a teaching plan about increased exercise for a female client
who is receiving long-term corticosteroid therapy. What type of exercise
is most appropriate for this client?

You Selected:

stretching

Correct response:

walking

Explanation:

Question 4 See full question

The nurse is reviewing the following physicians order written for a postmenopausal
woman: calcitonin salmon nasal spray 200 IU, one spray every day. What is the
appropriate action to be taken by the nurse regarding this order?

You Selected:

Clarify with the physician that the spray should be given in only one nostril per day.

Correct response:

Clarify with the physician that the spray should be given in only one nostril per day.

Explanation:

Question 5 See full question

During a home visit, the nurse assesses a client who is taking hydrochlorothiazide
and lisinopril for the treatment of hypertension. Which finding would indicate the
nurse should inform the health care provider of a possible need to change
medication therapy?

You Selected:

Potassium level is 4.1 mEq/L.

Correct response:

Client has a persistent cough.


Explanation:

Question 22 See full question

A primigravida in active labor is about 10 days postterm. The client desires a pudendal block anesthetic before
childbirth. After the nurse explains this type of anesthesia to the client, which location if identified by the client as
the area of relief would indicate to the nurse that the teaching was effective?

You Selected:

abdomen

Correct response:

perineum

Explanation:

Question 23 See full question

After undergoing small-bowel resection, a client is prescribed metronidazole 500 mg intravenously. The mixed
solution is 100 ml. The nurse is to administer the drug over 30 minutes. The drop factor of the available intravenous
tubing is 15 gtt/ml. What is the drip rate in drops per minute? Record your answer using a whole number. (For
example: 62)

Your Response:

Correct response:

50

Explanation:

Question 24 See full question

After the nurse has taught the client who is being discharged on lithium about the drug, which client statement
would indicate that the teaching has been successful?

You Selected:

"If I forget a dose, I can double the dose the next time I take it."

Correct response:

I will call my health care provider right away for any vomiting, severe hand tremors, or muscle weakness.

Explanation:

Question 25 See full question

The client with Alzheimer's disease has been prescribed donepezil 5 mg at bedtime. Which instruction should the
nurse give to the client's daughter?

You Selected:

Avoid suddenly stopping the medication.

Correct response:

Avoid suddenly stopping the medication.

Explanation:

Question 26 See full question

The nurse has administered promethazine intravenously to a client in active labor. The drug has had the desired
effect when the nurse notes:
You Selected:

increased contraction strength.

Correct response:

decreased nausea and vomiting.

Explanation:

Question 27 See full question

A 30-year-old multiparous client has been prescribed oral contraceptives as a method of birth control. The nurse
instructs the client that decreased effectiveness may occur if the client is prescribed which drug?

You Selected:

omeprazole

Correct response:

ampicillin

Explanation:

Question 28 See full question

A client who has been experiencing angina has a new prescription for nitroglycerin. The nurse should instruct the
client to report having which potential side effect of nitroglycerin?

You Selected:

hypertension

Correct response:

headache

Explanation:

Question 29 See full question

A physician orders several drugs for a client with hemorrhagic stroke. Which drug order should the nurse question?

You Selected:

Dexamethasone

Correct response:

Heparin sodium

Explanation:

Question 30 See full question

A physician orders spironolactone, 50 mg by mouth four times daily, for a client with fluid retention caused by
cirrhosis. Which finding indicates that the drug is producing a therapeutic effect?

You Selected:

Loss of 2.2 lb (1 kg) in 24 hours

Correct response:

Loss of 2.2 lb (1 kg) in 24 hours


Explanation:

Question 31 See full question

A client is receiving an I.V. infusion of mannitol after undergoing intracranial surgery to remove a brain tumor. To
determine whether this drug is producing its therapeutic effect, the nurse should consider which finding most
significant?

You Selected:

Decreased heart rate

Correct response:

Increased urine output

Explanation:

Question 32 See full question

The health care provider (HCP) prescribes an intramuscular injection of vitamin K for a term neonate. The nurse
explains to the mother that this medication is used to prevent which problem?

You Selected:

hyperbilirubinemia

Correct response:

hemorrhage

Explanation:

Question 33 See full question

The client with depression has been hospitalized for 3 days on the psychiatric unit. This is the second hospitalization
during the past year. The healthcare provider prescribes tranylcypromine sulfate because the client did not respond
positively to a tricyclic antidepressant. If the clients diet includes foods containing tyramine, the nurse should teach
the client about which possible reaction?

You Selected:

heart block

Correct response:

hypertensive crisis

Explanation:

Question 34 See full question

An 80-year-old client is admitted with nausea and vomiting. The client has a history of heart failure and is being
treated with digoxin. The client has been nauseated for a week and began vomiting 2 days ago. Laboratory values
indicate hypokalemia. Because of these clinical findings, the nurse should assess the client carefully for:

You Selected:

chronic renal failure.

Correct response:

digoxin toxicity.

Explanation:

Question 35 See full question

The health care provider prescribes sulfasalazine for the client with ulcerative colitis. Which instruction should the
nurse give the client about taking this medication?
You Selected:

Stop taking it if urine turns orange-yellow.

Correct response:

Take it with a full glass (240 mL) of water.

Explanation:

Question 36 See full question

A physician orders digoxin for a client with heart failure. During digoxin therapy, which laboratory value may
predispose the client to digoxin toxicity?

You Selected:

Calcium level of 7.5 mg/dl (0.4 mmol/L)

Correct response:

Potassium level of 3.1 mEq/L (3.1 mmol/L)

Explanation:

Question 37 See full question

The nurse must administer ferrous sulfate to an infant who weighs 8 lb 13 oz (4 kg). The dosage prescribed is 6
mg/kg/day to be given in three doses. What would be the correct amount to be administered for each dose? Record
your answer using a whole number.

Your Response:

Correct response:

Explanation:

Question 38 See full question

The physicians order reads "digoxin 0.075 mg." The pharmacy packaging contains three digoxin tablets labeled as
0.25 mg each. The packaging states to administer all 3 tablets to the client. What should the nurse do next?

You Selected:

Contact the pharmacist because the dose is too low.

Correct response:

Contact the pharmacist because the dose is too high.

Explanation:

Question 39 See full question

The nurse is caring for a child who has an order for ferrous sulfate. The nurse instructs the childs parents to
administer the ferrous sulfate with a citrus juice. The parents ask why they need to do this. Which response by the
nurse is the best explanation for administering ferrous sulfate with a citrus juice?

You Selected:

The citrus juice helps with the absorption of ferrous sulfate.

Correct response:

The citrus juice helps with the absorption of ferrous sulfate.


Explanation:

Question 40 See full question

A client who is 1 day postoperative is using a morphine patient-controlled analgesia (PCA) pump. The client is
confused and disoriented. What is the priority intervention by the nurse?

You Selected:

Check for shortness of breath, signifying a pulmonary embolism.

Correct response:

Check respiratory rate and depth as well as oxygen saturation levels.

Explanation:

Question 41 See full question

A client with ulcerative colitis is scheduled for a bowel resection. The client is receiving parenteral nutrition prior to
surgery. Which of the following is the best explanation for the nurse to give the client about the need for parenteral
nutrition?

You Selected:

The client cannot absorb nutrients through the colon.

Correct response:

The client has lost 15% of body weight and has prolonged diarrhea.

Explanation:

Question 42 See full question

A client is receiving parenteral nutrition through a central venous catheter. As the nurse is changing the dressing at
the catheter site, the client asks why this type of catheter is being used instead of a regular peripheral IV. Which is
the best response by the nurse to explain the use of the central venous catheter?

You Selected:

The nutrients that are being administered are too concentrated for a peripheral IV.

Correct response:

The nutrients that are being administered are too concentrated for a peripheral IV.

Explanation:

Question 43 See full question

Which of the following results would indicate that levothyroxine sodium is effectively resolving the symptoms of a
client with hypothyroidism?

You Selected:

Improved appetite, weight gain, and sleeping fewer hours

Correct response:

Increased energy, weight loss, and a higher temperature and pulse rate

Explanation:

Question 44 See full question

The nurse is reconciling the prescriptions for a client diagnosed recently with pulmonary tuberculosis who is being
admitted to the hospital for a total hip replacement. (See accompanying medication prescription sheet). The client
asks if it is necessary to take all of these medications while in the hospital. The nurse should:
You Selected:

ask the pharmacist to check for drug interactions between the rifampin and isoniazid.

Correct response:

tell the client that it is important to continue to take the medications because the combination of drugs
prevents bacterial resistance.

Explanation:

Question 45 See full question

The nurse is to administer midazolam 2.5 mg. The medication is available in a 5 mg/mL vial. How many mL should
the nurse administer? Record your answer using one decimal point.

Your Response:

Correct response:

0.5

Explanation:

Question 46 See full question

The nurse is caring for an adolescent client that sustained a head injury in a motor vehicle crash. The client begins
to experience extreme thirst and excretes 4 L of urine in a 24-hour period with a specific gravity of 1.002. What
pharmacological intervention does the nurse anticipate performing?

You Selected:

Administration of demeclocycline

Correct response:

Administration of desmopressin

Explanation:

Question 47 See full question

Which technique is correct when the nurse administers a subcutaneous injection?

You Selected:

Insert the needle at a 45-degree angle to the skin.

Correct response:

Insert the needle at a 45-degree angle to the skin.

Explanation:

Question 48 See full question

A nurse is administering an IV antineoplastic agent when the client says, My arm is burning by the IV site. What
should the nurse do first?

You Selected:

Call the primary care provider to report the incident.

Correct response:

Stop infusing the medication.


Explanation:

Question 49 See full question

Eardrops have been prescribed to be instilled in the adult clients left ear to soften cerumen. To position the client,
what should the nurse do?

You Selected:

Pull the auricle lobe up and back.

Correct response:

Pull the auricle lobe up and back.

Explanation:

Question 50 See full question

The health care provider prescribes furosemide 40 mg intravenous push daily. The medication comes in a vial of 50
mg/mL. Mark on the syringe the dosage of medication the nurse would give.

You Selected:

Your selection and the correct area, market by the green box.

Explanation:

Question 1 See full question

A nurse is instructing a client with asthma on the use of an inhaler with a spacer. The client asks what the purpose
of the spacer is. The nurse's best response is:

You Selected:

"You should ask your physician to explain the purpose of the spacer."

Correct response:

"The spacer traps medicine from the inhaler, then breaks up and slows down the medication particles, so you
get more medication."

Explanation:

Question 2 See full question

A child has just received a dose of theophylline I.V. for asthma. What assessment finding should the nurse expect?

You Selected:

Decreased pulmonary wheezing

Correct response:

Decreased pulmonary wheezing


Explanation:

Question 3 See full question

While shopping at a mall, a woman experiences an episode of extreme terror accompanied by anxiety, tachycardia,
trembling, and fear of going crazy. A friend drives her to the emergency department, where a physician rules out
physiologic causes and refers her to the psychiatric resident on call. To control the client's anxiety, the nurse caring
for this client expects the resident to order:

You Selected:

bupropion.

Correct response:

lorazepam.

Explanation:

Question 4 See full question

A client diagnosed with major depression has started taking amitriptyline hydrochloride, a tricyclic antidepressant.
What is a common adverse effect of this drug?

You Selected:

Weight loss

Correct response:

Dry mouth

Explanation:

Question 5 See full question

A nurse is caring for a woman receiving a lumbar epidural anesthetic block to control labor pain. What should the
nurse do to prevent hypotension?

You Selected:

Ensure adequate hydration before the anesthetic is administered.

Correct response:

Ensure adequate hydration before the anesthetic is administered.

Explanation:

Question 6 See full question

After the nurse teaches a client with bipolar disorder about lithium therapy, which client statement indicates the
need for additional teaching?

You Selected:

"It is important to keep using a regular amount of salt in my diet."

Correct response:

"It is okay to double my next dose of lithium if I forget a dose."

Explanation:

Question 7 See full question

Which client statement indicates the need for additional teaching about benzodiazepines?

You Selected:

"Diazepam can make me drowsy, so I should not drive for a while."


Correct response:

"I can stop taking the drug anytime I want."

Explanation:

Question 8 See full question

The nurse should dispose of a used needle and syringe by:

You Selected:

Separating the needle and syringe and placing both in the precaution container in the client's room.

Correct response:

Placing uncapped, used needles and syringes immediately in the universal precaution container in the client's
room.

Explanation:

Question 9 See full question

The health care provider (HCP) has prescribed nitroglycerin to a client with angina. The client also has closed-angle
glaucoma. The nurse contacts the HCP to discuss the potential for:

You Selected:

hypotension.

Correct response:

increased intraocular pressure.

Explanation:

Question 10 See full question

A client receives an IV dose of gentamicin sulfate. How long after the completion of the dose should the peak serum
concentration level be measured?

You Selected:

20 minutes

Correct response:

30 minutes

Explanation:

Question 11 See full question

Which clinical manifestation is a typical reaction to long-term phenytoin sodium therapy?

You Selected:

excessive growth of gum tissue

Correct response:

excessive growth of gum tissue

Explanation:

Question 12 See full question

The nurse is reviewing laboratory reports for a client who is taking allopurinol. Which finding indicates that the drug
has had a therapeutic effect?
You Selected:

increased serum calcium level

Correct response:

decreased serum uric acid level

Explanation:

Question 13 See full question

The nurse is administering a saturated solution of potassium iodide (SSKI). The nurse should:

You Selected:

dilute the solution with water, milk, or fruit juice and have the client drink it with a straw.

Correct response:

dilute the solution with water, milk, or fruit juice and have the client drink it with a straw.

Explanation:

Question 14 See full question

Vasopressin is administered to the client with diabetes insipidus because it:

You Selected:

increases release of insulin from the pancreas.

Correct response:

increases tubular reabsorption of water.

Explanation:

Question 15 See full question

A client is to be discharged with a prescription for lactulose. The nurse teaches the client and the clients spouse
how to administer this medication. Which statement would indicate that the client has understood the information?

You Selected:

"I will take it with a laxative."

Correct response:

"I will mix it with apple juice."

Explanation:

Question 16 See full question

When caring for the client who is receiving an aminoglycoside antibiotic, the nurse should monitor which laboratory
value?

You Selected:

serum creatinine

Correct response:

serum creatinine
Explanation:

Question 17 See full question

Alteplase recombinant, or tissue plasminogen activator (t-PA), a thrombolytic enzyme, is administered during the
first 6 hours after onset of myocardial infarction (MI) to:

You Selected:

control the arrhythmias associated with MI.

Correct response:

revascularize the blocked coronary artery.

Explanation:

Question 18 See full question

As an initial step in treating a client with angina, the health care provider (HCP) prescribes nitroglycerin tablets, 0.3
mg given sublingually. This drugs principal effects are produced by:

You Selected:

improved conductivity in the myocardium.

Correct response:

vasodilation of peripheral vasculature.

Explanation:

Question 19 See full question

When preparing a teaching plan for a client who is to receive a rubella vaccine during the postpartum period, the
nurse should include which information?

You Selected:

Pregnancy should be avoided for 4 weeks after the immunization.

Correct response:

Pregnancy should be avoided for 4 weeks after the immunization.

Explanation:

Question 20 See full question

A 44-lb (20-kg) preschooler is being treated for inflammation. The physician orders 0.2 mg/kg/day of
dexamethasone by mouth to be administered every 6 hours. The elixir comes in a strength of 0.5 mg/5 ml. How
many teaspoons of dexamethasone should the nurse give this client per dose? Record your answer using a whole
number.

Your Response:

Correct response:

Explanation:

Question 21 See full question

A client is admitted with a diagnosis of diabetic ketoacidosis. An insulin drip is initiated with 50 units of insulin in
100 ml of normal saline solution administered via an infusion pump set at 10 ml/hour. The nurse determines that
the client is receiving how many units of insulin each hour? Record your answer using a whole number.
Your Response:

Correct response:

Explanation:

Question 22 See full question

A 20-year-old client visiting the clinic requests the use of oral contraceptives. When reviewing the clients history,
which finding would alert the nurse to a possible contraindication to using these agents?

You Selected:

urinary tract infections

Correct response:

thrombophlebitis

Explanation:

Question 23 See full question

Twenty-four hours after giving birth to a term neonate, a primipara receives acetaminophen with codeine for
perineal pain. One hour after administering the medication, which finding should alert the nurse to the development
of a possible side effect?

You Selected:

urinary frequency

Correct response:

dizziness

Explanation:

Question 24 See full question

A primigravid client at 32 weeks gestation with ruptured membranes is prescribed to receive betamethasone 12
mg intramuscularly for two doses 24 hours apart. When teaching the client about the medication, what should the
nurse include as the purpose of this drug?

You Selected:

to prevent potential infection

Correct response:

to accelerate fetal lung maturity

Explanation:

Question 25 See full question

In teaching a client with tuberculosis about self-care at home, which directive has the highest priority?

You Selected:

Take medications as prescribed.

Correct response:

Take medications as prescribed.


Explanation:

Question 26 See full question

When administering atropine sulfate preoperatively to a client scheduled for lung surgery, the nurse should tell the
client?

You Selected:

This medicine will make you drowsy.

Correct response:

This medicine will make your mouth feel dry.

Explanation:

Question 27 See full question

The nurse is teaching a client with osteoporosis about taking alendronate sodium. The nurse emphasizes that the
client is to take the medication:

You Selected:

with a full glass of juice and then rest for 30 minutes.

Correct response:

with a full glass of water and remain upright for 30 minutes.

Explanation:

Question 28 See full question

The nurse has administered mannitol IV. Which is a priority assessment for the nurse to make after administering
this drug?

You Selected:

Monitor urine output.

Correct response:

Monitor urine output.

Explanation:

Question 29 See full question

The nurse has given a client a nitroglycerin tablet sublingually for angina. Which vital signs should be assessed
following administration of nitroglycerin?

You Selected:

blood pressure

Correct response:

blood pressure

Explanation:

Question 30 See full question

A nurse is preparing a continuous insulin infusion for a child with diabetic ketoacidosis and a blood glucose level of
[800 mg/dl (44.4 mmol/L)]. Which solution is the most appropriate at the beginning of therapy?

You Selected:

100 units of regular insulin in normal saline solution


Correct response:

100 units of regular insulin in normal saline solution

Explanation:

Question 31 See full question

A client with chest pain doesn't respond to nitroglycerin. When he's admitted to the emergency department, the
health care team obtains an electrocardiogram and administers I.V. morphine. The physician also considers
administering alteplase. This thrombolytic agent must be administered how soon after onset of myocardial
infarction (MI) symptoms?

You Selected:

Within 24 to 48 hours

Correct response:

Within 6 hours

Explanation:

Question 32 See full question

For a client with hyperthyroidism, treatment is most likely to include:

You Selected:

a synthetic thyroid hormone.

Correct response:

a thyroid hormone antagonist.

Explanation:

Question 33 See full question

A nurse should expect to administer which medication to a client with gout?

You Selected:

Colchicine

Correct response:

Colchicine

Explanation:

Question 34 See full question

The primary health care provider (HCP) prescribes intravenous magnesium sulfate for a primigravid client at 38
weeks' gestation diagnosed with severe preeclampsia. Which medication would be most important for the nurse to
have readily available?

You Selected:

phenytoin

Correct response:

calcium gluconate

Explanation:

Question 35 See full question

The primary health care provider (HCP) prescribes whole blood replacement for a multigravid client with abruptio
placentae. Before administering the intravenous blood product, the nurse should first:
You Selected:

validate client information and the blood product with another nurse.

Correct response:

validate client information and the blood product with another nurse.

Explanation:

Question 36 See full question

Which statement by the client indicates an understanding of teaching regarding use of corticosteroids during
preterm labor?

You Selected:

"I will be taking corticosteroids until my baby's due date so that he will have the best chance of doing well."

Correct response:

The corticosteroids may help my babys lungs mature.

Explanation:

Question 37 See full question

A client comes to the mental health clinic 2 days after being discharged from the hospital. The client was given a 1-
week supply of clozapine. Which client statement indicates an accurate understanding of the nurse's teaching
about this medication?

You Selected:

I need to call my health care provider in 2 weeks for a checkup.

Correct response:

"I need to keep my appointment here at the clinic this week for a blood test."

Explanation:

Question 38 See full question

The nurse has an order to administer 1200 mg of an antibiotic. The drug is prepared as 6 grams of the drug in 2 ml
of solution. The nurse should administer how many ml of the drug? Record your answer using one decimal place.

Your Response:

Correct response:

0.4

Question 39 See full question

The nurse teaches a client taking desmopressin nasal spray about how to manage treatment. The nurse determines
that the client needs additional instruction when the client says:

You Selected:

"I should report any signs of respiratory infection."

Correct response:

"I should use the same nostril each time I take the medicine."
Explanation:

Question 40 See full question

A client who weighs 187 lb (85 kg) has an order to receive enoxaparin 1 mg/kg. This drug is available in a
concentration of 30 mg/0.3 mL. What dose would the nurse administer in milliliters? Record your answer using two
decimal places.

Your Response:

Correct response:

0.85

Explanation:

Question 41 See full question

The nurse is using the Z-track method of intramuscular (IM) injection to administer iron dextran to a client with iron-
deficiency anemia. Which techniques should the nurse use to give this injection? Select all that apply.

You Selected:

After removing the needle, massage the injection site.

Correct response:

Confirm the client's identity before administering the iron dextran.


Change the needle after drawing up the iron dextran.
Before inserting the needle, displace the skin laterally by pulling it away from the injection site.
Inject the iron dextran after aspirating for a blood return.

Explanation:

Question 42 See full question

The nurse has an order to administer 2 oz of lactulose to a client who has cirrhosis. How many milliliters of lactulose
should the nurse administer? Record your answer using a whole number.

Your Response:

Correct response:

60

Explanation:

Question 43 See full question

A nurse teaches a client experiencing heartburn to take 1.5 oz of aluminum hydroxide when symptoms appear. How
many milliliters should the client take? Record your answer using a whole number.

Your Response:

Correct response:

45

Explanation:

Question 44 See full question

After the nurse has administered droperidol, care is taken to move the client slowly based on the knowledge of
droperidols effect on the:
You Selected:

cardiovascular system.

Correct response:

cardiovascular system.

Explanation:

Question 45 See full question

A physician orders digoxin for a client with heart failure. During digoxin therapy, which laboratory value may
predispose the client to digoxin toxicity?

You Selected:

Magnesium level of 2.5 mg/dl (0.1 mmol/L)

Correct response:

Potassium level of 3.1 mEq/L (3.1 mmol/L)

Explanation:

Question 46 See full question

Two weeks before a client is scheduled for an ileostomy, the nurse should instruct the client to:

You Selected:

abstain from having sex.

Correct response:

stop taking drugs that will interfere with clotting (aspirin, ibuprofen).

Explanation:

Question 47 See full question

The nurse is caring for a client who is 1 day post total hip replacement. The client has patient-controlled analgesia
(PCA) but is reporting pain. Which of the following actions would be most important for the nurse to take in relation
to the clients pain management?

You Selected:

Explain the use of PCA to the client and confirm that the client understands.

Correct response:

Assess the clients pain to determine whether the PCA dosage is adequate.

Explanation:

Question 48 See full question

The nurse is to administer 1,200 mg of an antibiotic. The drug is prepared with 6 g of the drug in 2 mL of solution.
The nurse should administer how many milliliters of the drug? Record your answer using one decimal place.

Your Response:

Correct response:

0.4
Explanation:

Question 49 See full question

A client who is receiving doxorubicin should have a plan of care for reducing the risk for which of the following
complications? Select all that apply.

You Selected:

Cardiac toxicity

Correct response:

Cardiac toxicity
Pulmonary toxicity

Explanation:

Question 50 See full question

The health care provider (HCP) has prescribed insulin detemir for a client with type 2 diabetes requiring insulin. The
nurse should tell the client:

You Selected:

You may refill the detemir insulin pen.

Correct response:

You do not mix insulin detemir; the solution is clear.

Explanation:

Question 1 See full question

A nurse has been teaching a client how to use an incentive spirometer that he must use at home for several days
after discharge. Which client action indicates an accurate understanding of the technique?

You Selected:

The client takes slow, deep breaths to elevate the spirometer ball.

Correct response:

The client takes slow, deep breaths to elevate the spirometer ball.

Explanation:

Question 2 See full question

For the last 6 days, a 7-month-old infant has been receiving amoxicillin trihydrate to treat an ear infection. Now the
parents report redness in the diaper area and small, red patches on the infant's inner thighs and buttocks. After
diagnosing Candida albicans, the physician orders topical nystatin to be applied to the perineum four times daily.
The nurse should focus her assessment on:

You Selected:

the infant's fontanels.

Correct response:

the inside of the infant's mouth.

Explanation:

Question 3 See full question

A dehydrated infant is receiving I.V. therapy. The mother tells the nurse she wants to hold her infant but is afraid
this might cause the I.V. line to become dislodged. What should the nurse do?
You Selected:

Advise the mother to let the infant lie quietly in bed.

Correct response:

Show the mother how to hold the infant properly.

Explanation:

Question 4 See full question

When preparing a client for electroconvulsive therapy (ECT), the nurse should make sure that:

You Selected:

the client has undergone a thorough medical evaluation.

Correct response:

the client has undergone a thorough medical evaluation.

Explanation:

Question 5 See full question

During a physical examination, a client who is 32 weeks pregnant becomes pale, dizzy, and light-headed while
supine. Which action should the nurse immediately take?

You Selected:

Listen to fetal heart tones.

Correct response:

Turn the client on her left side.

Explanation:

Question 6 See full question

A nurse is caring for a client whose membranes ruptured prematurely 12 hours ago. When assessing this client, the
nurse's highest priority is to evaluate:

You Selected:

maternal vital signs and fetal heart rate (FHR).

Correct response:

maternal vital signs and fetal heart rate (FHR).

Explanation:

Question 7 See full question

A client, 7 months pregnant, is admitted to the unit with abdominal pain and bright red vaginal bleeding. Which
action should the nurse take first?

You Selected:

Administer I.V. oxytocin, as ordered, to stimulate uterine contractions and prevent further hemorrhage.

Correct response:

Place the client on her left side and start supplemental oxygen, as ordered, to maximize fetal oxygenation.
Explanation:

Question 8 See full question

A certified nurse-midwife places a neonate under the radiant heat unit for the nurse's initial assessment. The initial
assessment includes heart rate 110 beats/minute and an irregular respiratory effort. The neonate is moving all
extremities and his body is pink. He also has a vigorous cry. The nurse notes copious amounts of clear mucus
present both orally and nasally. Based on these assessment findings, what should the nurse do next?

You Selected:

Assign an Apgar score of 9, place the neonate in modified Trendelenburg's position, and suction the neonate's
nose and oropharynx.

Correct response:

Assign an Apgar score of 9, place the neonate in modified Trendelenburg's position, and suction the neonate's
nose and oropharynx.

Explanation:

Question 9 See full question

When teaching a primiparous client who used cocaine during pregnancy how to comfort her fussy neonate, the
nurse can advise the mother to:

You Selected:

tightly swaddle the neonate.

Correct response:

tightly swaddle the neonate.

Explanation:

Question 10 See full question

When developing the plan of care for a multigravid client with class III heart disease, the nurse should expect to
assess the client frequently for which problem?

You Selected:

tachycardia

Correct response:

tachycardia

Explanation:

Question 11 See full question

After instructing a female client about the radioimmunoassay pregnancy test, the nurse determines that the client
understands the instructions when the client states that which hormone is evaluated by this test?

You Selected:

follicle-stimulating hormone

Correct response:

human chorionic gonadotropin (hCG)


Explanation:

Question 12 See full question

A loading dose of digoxin is given to a client newly diagnosed with atrial fibrillation. The nurse begins instructing the
client about the medication and the importance of monitoring his heart rate. An expected outcome of this
instruction is:

You Selected:

verbalization of the need for the medication.

Correct response:

a return demonstration of palpating the radial pulse.

Explanation:

Question 13 See full question

The nurse is teaching a client who is taking insulin about the signs of diabetic ketoacidosis, which include:

You Selected:

excessive hunger.

Correct response:

Kussmaul's respirations.

Explanation:

Question 14 See full question

A young adult has been bitten by a human, and the skin on the forearm is broken. The client's last tetanus shot was
about 8 years ago. The nurse should prepare the client for:

You Selected:

an injection of tetanus toxoid.

Correct response:

an injection of tetanus toxoid.

Explanation:

Question 15 See full question

A 10-month-old child has cold symptoms. The mother asks how she can clear the infants nose. What would be the
nurses best recommendation?

You Selected:

Blow into the child's mouth to clear the infant's nose.

Correct response:

Use saline nose drops and then a bulb syringe.

Explanation:

Question 16 See full question

A child with sickle cell crisis is being discharged. As part of discharge teaching to prevent further crisis, what should
the nurse advise the parent to do?

You Selected:

Weigh the child every day.


Correct response:

Encourage the child to drink lots of liquids.

Explanation:

Question 17 See full question

Which finding would alert the nurse to suspect that a child with severe gastroenteritis who has been receiving
intravenous therapy for the past several hours may be developing circulatory overload?

You Selected:

auscultation of moist crackles

Correct response:

auscultation of moist crackles

Explanation:

Question 18 See full question

The father of a preschool-age child with a tentative diagnosis of juvenile idiopathic arthritis (JIA) asks about a test to
definitively diagnose JIA. The nurse's response is based on knowledge of what information?

You Selected:

An increased erythrocyte sedimentation rate is diagnostic.

Correct response:

No specific laboratory test is diagnostic.

Explanation:

Question 19 See full question

The surgeon prescribes cefazolin 1 g to be given IV at 0730 when the clients surgery is scheduled at 0800. What is
the primary reason to start the antibiotic exactly at 0730?

You Selected:

The peak and titer levels are needed for antibiotic therapy.

Correct response:

The antibiotic is most effective in preventing infection if it is given 30 to 60 minutes before the operative
incision is made.

Explanation:

Question 20 See full question

The initial postoperative assessment is completed on a client who had an arthroscopy of the knee. Which
information is not necessary to obtain every 15 minutes during the first postoperative hour?

You Selected:

pain rating of the operative site

Correct response:

urine output

Explanation:

Question 21 See full question

A client with suspected gastric cancer undergoes an endoscopy of the stomach. Which assessment made after the
procedure would indicate the development of a potential complication?
You Selected:

The client displays signs of sedation.

Correct response:

The client experiences a sudden increase in temperature.

Explanation:

Question 22 See full question

After surgery for gastric cancer, a client is scheduled to undergo radiation therapy. The nurse should include which
information in the teaching plan?

You Selected:

management of alopecia

Correct response:

nutritional intake

Explanation:

Question 23 See full question

While caring for a just born female term neonate, the nurse observes that the neonates clitoris is enlarged and
there is some fusion of the posterior labia majora. The nurse should notify the health care provider because these
findings are associated with which problem?

You Selected:

Turners syndrome

Correct response:

ambiguous genitalia

Explanation:

Question 24 See full question

After resuming feedings in an infant who has undergone a pyloroplasty, which action would be most appropriate?

You Selected:

Offer 45 mL of an oral electrolyte solution initially.

Correct response:

Starting feedings with 5 to 10 ml, slowly increasing amounts as tolerated.

Explanation:

Question 25 See full question

A 23-month-old child pulls a pan of hot water off the stove and spills it onto her chest and arms. Her mother is right
there when it happens. What should the mother do immediately?

You Selected:

Place the child in a bathtub of cool water.

Correct response:

Place the child in a bathtub of cool water.


Explanation:

Question 26 See full question

A client who is having an abdominal perineal resection with permanent colostomy asks, Where will my colostomy
be placed? The nurse should tell the client:

You Selected:

A permanent colostomy is usually located on the left side of the abdomen.

Correct response:

A permanent colostomy is usually located on the left side of the abdomen.

Explanation:

Question 27 See full question

When the client who has had a hip replacement is lying on the side, the nurse should place pillows or an abductor
splint between the legs to prevent:

You Selected:

hyperextension of the knees.

Correct response:

adduction of the hip joint.

Explanation:

Question 28 See full question

A nurse assessing a client who underwent cardiac catheterization finds the client lying flat on the bed. His
temperature is 99.8 F (37.7 C). His blood pressure is 104/68 mm Hg. His pulse rate is 76 beats/minute. The nurse
assesses the limb and detects weak pulses in the leg distal to the puncture site. Skin on the leg is cool to the touch.
The puncture site is dry, but swollen. What is the most appropriate action for the nurse to take?

You Selected:

Document the findings and recheck the client in 1 hour.

Correct response:

Contact the physician and report the findings.

Explanation:

Question 29 See full question

A client with Crohn's disease is scheduled for a barium enema. What should the plan of care include today to
prepare for the test tomorrow?

You Selected:

Encourage plenty of fluids.

Correct response:

Encourage plenty of fluids.

Explanation:

Question 30 See full question

A nurse is caring for a client with type 1 diabetes who exhibits confusion, light-headedness, and aberrant behavior.
The client is conscious. The nurse should first administer:
You Selected:

15 to 20 g of a fast-acting carbohydrate such as orange juice.

Correct response:

15 to 20 g of a fast-acting carbohydrate such as orange juice.

Explanation:

Question 31 See full question

A client with type 1 diabetes has a highly elevated glycosylated hemoglobin (Hb) test result. In discussing the result
with the client, the nurse is most accurate in stating:

You Selected:

"It looks like you aren't following the ordered diabetic diet."

Correct response:

"It tells us about your sugar control for the last 3 months."

Explanation:

Question 32 See full question

A client undergoes a total hip replacement. Which statement made by the client indicates to the nurse that the
client requires further teaching?

You Selected:

"I'll need to keep several pillows between my legs at night."

Correct response:

"I don't know if I'll be able to get off that low toilet seat at home by myself."

Explanation:

Question 33 See full question

Which statement indicates a client understands teaching about the purified protein derivative (PPD) test for
tuberculosis?

You Selected:

"If the test area turns red that means I have tuberculosis."

Correct response:

"Because I had a previous reaction to the test, this time I need to get a chest X-ray."

Explanation:

Question 34 See full question

The best method to remove cerumen from a client's ear involves:

You Selected:

using a cerumen curette.

Correct response:

irrigating the ear gently.


Explanation:

Question 35 See full question

A primigravid client at 38 weeks gestation diagnosed with mild preeclampsia calls the clinic nurse to say she has
had a continuous headache for the past 2 days accompanied by nausea. The client does not want to take aspirin.
The nurse should tell the client:

You Selected:

"I think the health care provider should see you today. Can you come to the clinic this morning?"

Correct response:

"I think the health care provider should see you today. Can you come to the clinic this morning?"

Explanation:

Question 36 See full question

While performing a complete assessment of a term neonate, which finding would alert the nurse to notify the health
care provider (HCP)?

You Selected:

red reflex in the eyes

Correct response:

expiratory grunt

Explanation:

Question 37 See full question

After teaching the parent of an infant who has had a surgical repair for a cleft lip about the use of elbow restraints
at home, the nurse determines that the teaching has been successful when the parent makes which statement?

You Selected:

"We will keep the restraints on continuously except when checking the skin under them for redness."

Correct response:

"We will keep the restraints on continuously except when checking the skin under them for redness."

Explanation:

Question 38 See full question

A nurse is caring for a 3-year-old client with a neuroblastoma who has been receiving chemotherapy for the last 4
weeks. His laboratory test results indicate a Hgb of 12.5 g/dL (125 g/L), HCT of 36.8% (0.37), WBC of 2000 mm 3 (2 X
109/L), and platelet count of 150,000/L (150 X 109/L). Based on the child's values, what is the highest priority
nursing intervention?

You Selected:

Encourage mouth care with a soft toothbrush.

Correct response:

Encourage meticulous handwashing by the client and visitors.

Explanation:

Question 39 See full question

An infant is born with facial abnormalities, growth retardation, and vision abnormalities. These abnormalities are
likely caused by maternal:
You Selected:

alcohol consumption.

Correct response:

alcohol consumption.

Explanation:

Question 40 See full question

A client with diabetes is explaining to the nurse how to care for the feet at home. Which statement indicates that
the client understands proper foot care?

You Selected:

"It is okay to go barefoot in the house."

Correct response:

"It is important to dry my feet carefully after my bath."

Explanation:

Question 41 See full question

The nurse has just received morning change-of-shift report on four clients. In what order from first to last should the
nurse perform the actions? All options must be used.

You Selected:

Notify the health care provider (HCP) about a client who has a serum potassium level of 6.2.
Assess the client who has been vomiting according to the report from the night nurse.
Begin discharge paperwork for a client who is eager to go home.
Discuss the plan for the day with the unlicensed assistive personnel (UAP), delegating duties as appropriate.

Correct response:

Notify the health care provider (HCP) about a client who has a serum potassium level of 6.2.
Assess the client who has been vomiting according to the report from the night nurse.
Discuss the plan for the day with the unlicensed assistive personnel (UAP), delegating duties as appropriate.
Begin discharge paperwork for a client who is eager to go home.

Explanation:

Question 42 See full question

A client with type I diabetes mellitus is scheduled to have surgery. The client has been nothing-by-mouth (NPO)
since midnight. In the morning, the nurse notices the clients daily insulin has not been prescribed. Which action
should the nurse do first?

You Selected:

Obtain the client's blood glucose at the bedside.

Correct response:

Obtain the client's blood glucose at the bedside.

Explanation:

Question 43 See full question

Which goal is most important when developing a long-term care plan for a child with hemophilia?

You Selected:

Increase the parent's and child's knowledge about hemophilia.


Correct response:

Prevent injury during each stage of development.

Explanation:

Question 44 See full question

The nurse is assessing an adolescent 1 hour after admission for a head injury. The nurse identifies that there have
been changes since the baseline assessment, including apnea, bradycardia, and a widening pulse pressure. What is
the primary reason for the nurse to notify the physician?

You Selected:

Too much pain medication can cause the changes observed by the nurse.

Correct response:

The changes suggest that the adolescents intracranial pressure is increasing.

Explanation:

Question 45 See full question

The nurse is caring for a 4-month-old infant who is scheduled for surgical removal of a cloudy lens and insertion of
an intraocular lens. Following surgery, which of the following is the highest priority of care for the nurse?

You Selected:

To control pain

Correct response:

To prevent vomiting

Explanation:

Question 46 See full question

A client with a nasogastric (NG) tube who is 2 days postoperative bowel resection is reporting increased abdominal
pain and nausea. Which assessments or actions by the nurse would be most appropriate?

You Selected:

Explain that nausea is common because the NG tube irritates the gag reflex.

Correct response:

Check the patency and amount of drainage from the NG tube.

Explanation:

Question 47 See full question

The student nurse is caring for a client with a suspected respiratory infection. Which of the following statements by
the nursing student indicates to the instructor that the student will facilitate the best time to collect this specimen?

You Selected:

"I will collect the specimen before bedtime."

Correct response:

"I will instruct the client to give the specimen in the morning, as soon as the client awakens."

Explanation:

Question 48 See full question

The nurse is aware that, in addition to the rule of nines, which is the most important assessment priority when
assessing a client with facial burns?
You Selected:

Determining oxygen saturation levels

Correct response:

Checking for airway patency

Explanation:

Question 49 See full question

A nurse is discussing nutrition and weight control with clients during a class about diabetes. Which statement best
reflects the purpose of nutritional management of diabetes?

You Selected:

To meet energy needs by eating all foods that keep blood glucose within a relatively normal range

Correct response:

To maintain blood glucose levels as close as possible to the normal range to reduce the risk for long-term
complications"

Explanation:

Question 50 See full question

The nurse is reviewing the electrocardiogram of a client who has elevated ST segments visible in leads II, III, and
aVf. Which is the nurses best action?

You Selected:

Document the finding in the medical record

Correct response:

Notify the healthcare provider

Explanation:

Question 1 See full question

For a child with a circumferential chest burn, what is the most important factor for the nurse to assess?

You Selected:

Breathing pattern

Correct response:

Breathing pattern

Explanation:

Question 2 See full question

A nurse is caring for an 8-year-old child with acute asthma exacerbation. Which situation would be of greatest
concern to the nurse?

You Selected:

Recent blood gas analysis indicates an oxygen saturation of 95%.

Correct response:

Before a respiratory therapy treatment, wheezing isn't heard on auscultation.


Explanation:

Question 3 See full question

A nurse is developing a care plan for a client who has undergone electroconvulsive therapy (ECT). The nurse should
include which intervention?

You Selected:

Monitoring the client's vital signs every hour for 4 hours

Correct response:

Reorienting the client to time and place

Explanation:

Question 4 See full question

Nursing care for a client after electroconvulsive therapy (ECT) should include:

You Selected:

bed rest for the first 8 hours after a treatment.

Correct response:

assessment of short-term memory loss.

Explanation:

Question 5 See full question

A physician orders electroconvulsive therapy (ECT) for a severely depressed client who fails to respond to drug
therapy. When teaching the client and family about his treatment, the nurse should include which point about ECT?

You Selected:

ECT will induce a seizure.

Correct response:

ECT will induce a seizure.

Explanation:

Question 6 See full question

A client recently admitted to the hospital with sharp, substernal chest pain suddenly reports palpitations. The client
ultimately admits to using cocaine 1 hour before admission. The nurse should immediately assess the client's:

You Selected:

level of consciousness.

Correct response:

pulse rate and character.

Explanation:

Question 7 See full question

During routine prenatal screening, a nurse tells a client that her blood sample will be used for alpha fetoprotein
(AFP) testing. Which statement best describes what AFP testing indicates?

You Selected:

"This test will show if you have gestational diabetes."


Correct response:

"This test will screen for spina bifida, Down syndrome, or other genetic defects."

Explanation:

Question 8 See full question

The correct landmark for obtaining an apical pulse is the:

You Selected:

left fifth intercostal space, midaxillary line.

Correct response:

left fifth intercostal space, midclavicular line.

Explanation:

Question 9 See full question

After completing discharge instructions for a primiparous client who is bottle-feeding her term neonate, the nurse
determines that the mother understands the instructions when the mother says that she should contact the
pediatrician if the neonate exhibits which sign or symptom?

You Selected:

ability to fall asleep easily after each feeding

Correct response:

passage of a liquid stool with a watery ring

Explanation:

Question 10 See full question

When teaching a primiparous client who used cocaine during pregnancy how to comfort her fussy neonate, the
nurse can advise the mother to:

You Selected:

touch the baby only when he is crying.

Correct response:

tightly swaddle the neonate.

Explanation:

Question 11 See full question

A 27-year-old primigravid client with insulin-dependent diabetes at 34 weeks' gestation undergoes a nonstress test,
the results of which are documented as reactive. What should the nurse tell the client that the test results indicate?

You Selected:

The nonstress test should be repeated.

Correct response:

There is evidence of fetal well-being.

Explanation:

Question 12 See full question

A primigravid client with class II heart disease who is visiting the clinic at 8 weeks' gestation tells the nurse that she
has been maintaining a low-sodium, 1,800-calorie diet. Which instruction should the nurse give the client?
You Selected:

Avoid folic acid supplements to prevent megaloblastic anemia.

Correct response:

Increase caloric intake to 2,200 calories daily to promote fetal growth.

Explanation:

Question 13 See full question

Which nursing action is contraindicated for the client who is experiencing severe symptoms of alcohol withdrawal?

You Selected:

monitoring intake and output

Correct response:

helping the client walk

Explanation:

Question 14 See full question

Which statement indicates that the client with a peptic ulcer understands the dietary modifications to follow at
home?

You Selected:

"It is important to eat six small meals a day."

Correct response:

"I should avoid alcohol and caffeine."

Explanation:

Question 15 See full question

The nurse assesses a client with diverticulitis and suspects peritonitis when which of the following symptoms is
noted?

You Selected:

Hyperactive bowel sounds.

Correct response:

Rigid abdominal wall.

Explanation:

Question 16 See full question

The most appropriate toys to give to a 5-month-old infant are:

You Selected:

stuffed animals.

Correct response:

soft, washable toys.


Explanation:

Question 17 See full question

The nurse is teaching the parents of a child with myelomeningocele how to prevent urinary tract infections. What
should the care plan include for this child? Select all that apply.

You Selected:

Use the Crede's maneuver to empty the bladder.

Correct response:

Use the Crede's maneuver to empty the bladder.


Encourage frequent emptying of the bladder.
Assure adequate fluid intake.

Explanation:

Question 18 See full question

Which nursing intervention is most important in preventing postoperative complications?

You Selected:

early ambulation

Correct response:

early ambulation

Explanation:

Question 19 See full question

The initial postoperative assessment is completed on a client who had an arthroscopy of the knee. Which
information is not necessary to obtain every 15 minutes during the first postoperative hour?

You Selected:

neurovascular check distal to the operative site

Correct response:

urine output

Explanation:

Question 20 See full question

Which client statement identifies a knowledge deficit about cast care?

You Selected:

"I will apply ice for 10 minutes to control edema for the first 24 hours."

Correct response:

"I can pull out cast padding to scratch inside the cast."

Explanation:

Question 21 See full question

After surgery for gastric cancer, a client is scheduled to undergo radiation therapy. The nurse should include which
information in the teaching plan?

You Selected:

access to community resources


Correct response:

nutritional intake

Explanation:

Question 22 See full question

When performing the Heimlich maneuver on a conscious adult victim, the rescuer delivers inward and upward
thrusts specifically:

You Selected:

above the umbilicus.

Correct response:

below the xiphoid process and above the umbilicus.

Explanation:

Question 23 See full question

A pregnant woman states that she frequently ingests laundry starch. The nurse should assess the client for:

You Selected:

muscle spasms.

Correct response:

anemia.

Explanation:

Question 24 See full question

A nurse is caring for a client who underwent surgical repair of a detached retina in the right eye. Which nursing
interventions would the nurse perform postoperatively? Select all that apply.

You Selected:

Discourage the client from bending down.

Correct response:

Approach the client from the left side.


Discourage the client from bending down.
Orient the client to his environment.
Administer a stool softener.

Explanation:

Question 25 See full question

When preparing for the discharge of a neonate who has undergone corrective surgery for tracheoesophageal fistula,
the nurse teaches the parents about the need for long-term health care because their child has a high probability of
developing which complication?

You Selected:

esophageal stricture

Correct response:

esophageal stricture
Explanation:

Question 26 See full question

What is the primary reason that the nurse inserts an indwelling urinary (Foley) catheter in a child with severe
burns?

You Selected:

to prevent urinary retention

Correct response:

to measure urine output accurately

Explanation:

Question 27 See full question

On admission, the clients arterial blood gas (ABG) values were: pH, 7.20; PaO 2, 64 mm Hg (8.5 kPa); PaCO2, 60 mm
Hg (8 kPa); and HCO3-, 22 mEq/L (22 mmol/L). A chest tube is inserted, and oxygen at 4 L/minute is started. Thirty
minutes later, repeat blood gas values are: pH, 7.30; PaO 2, 76 mm Hg (10.1 kPa); PaCO2, 50 mm Hg (6.7 kPa); and
HCO3-, 22 mEq/L (22 mmol/L). This change would indicate:

You Selected:

impending respiratory failure.

Correct response:

improving respiratory status.

Explanation:

Question 28 See full question

A client is scheduled to undergo percutaneous transluminal coronary angioplasty (PTCA). Which statement by the
nurse best explains the procedure to the client?

You Selected:

PTCA involves passing a catheter through the coronary arteries to find blocked arteries.

Correct response:

PTCA involves opening a blocked artery with an inflatable balloon located on the end of a catheter.

Explanation:

Question 29 See full question

A client has been diagnosed with cirrhosis. When obtaining a health history, the nurse should specifically determine
if the client takes?

You Selected:

Neomycin sulfate.

Correct response:

Acetaminophen.

Explanation:

Question 30 See full question

While palpating a client's right upper quadrant (RUQ), the nurse would expect to find which structure?

You Selected:

Spleen
Correct response:

Liver

Explanation:

Question 31 See full question

A client undergoes a total hip replacement. Which statement made by the client indicates to the nurse that the
client requires further teaching?

You Selected:

"I don't know if I'll be able to get off that low toilet seat at home by myself."

Correct response:

"I don't know if I'll be able to get off that low toilet seat at home by myself."

Explanation:

Question 32 See full question

After a plane crash, a client is brought to the emergency department with severe burns and respiratory difficulty.
The nurse helps to secure a patent airway and attends to the client's immediate needs, then prepares to perform an
initial neurologic assessment. The nurse should perform an:

You Selected:

examination of the fundus of the eye.

Correct response:

evaluation of the corneal reflex response.

Explanation:

Question 33 See full question

Which statement indicates a client understands teaching about the purified protein derivative (PPD) test for
tuberculosis?

You Selected:

"I will avoid contact with my family until I am done with the test."

Correct response:

"Because I had a previous reaction to the test, this time I need to get a chest X-ray."

Explanation:

Question 34 See full question

A nurse is caring for a client who has a tracheostomy and temperature of 103 F (39.4 C). Which intervention will
most likely lower the client's arterial blood oxygen saturation?

You Selected:

Use of a cooling blanket

Correct response:

Endotracheal suctioning
Explanation:

Question 35 See full question

A client with chronic renal failure is admitted with a heart rate of 122 beats/minute, a respiratory rate of 32
breaths/minute, a blood pressure of 190/110 mm Hg, jugular vein distention, and bibasilar crackles. Which nursing
diagnosis takes highest priority for this client?

You Selected:

Excess fluid volume

Correct response:

Excess fluid volume

Explanation:

Question 36 See full question

A client has returned from surgery during which the jaws were wired as treatment for a fractured mandible. The
client is in stable condition. The nurse is instructing the unliscensed nursing personnel (UAP) on how to properly
position the client. Which instructions about positioning would be appropriate for the nurse to give the UAP?

You Selected:

Do not reposition the client without the assistance of a registered nurse (RN).

Correct response:

Keep the client in a side-lying position with the head slightly elevated.

Explanation:

Question 37 See full question

The nurse should teach a client that a normal local tissue response to radiation following surgery for breast cancer
is:

You Selected:

sloughing of two layers of skin.

Correct response:

redness of the surface tissue.

Explanation:

Question 38 See full question

Which action is not appropriate when providing oral hygiene for a client who has had a stroke?

You Selected:

opening the client's mouth with a padded tongue blade

Correct response:

placing the client on the back with a small pillow under the head

Explanation:

Question 39 See full question

The nurse has just received morning change-of-shift report on four clients. In what order from first to last should the
nurse perform the actions? All options must be used.

You Selected:

Begin discharge paperwork for a client who is eager to go home.


Assess the client who has been vomiting according to the report from the night nurse.
Discuss the plan for the day with the unlicensed assistive personnel (UAP), delegating duties as appropriate.
Notify the health care provider (HCP) about a client who has a serum potassium level of 6.2.

Correct response:

Notify the health care provider (HCP) about a client who has a serum potassium level of 6.2.
Assess the client who has been vomiting according to the report from the night nurse.
Discuss the plan for the day with the unlicensed assistive personnel (UAP), delegating duties as appropriate.
Begin discharge paperwork for a client who is eager to go home.

Explanation:

Question 40 See full question

A nurse is assessing the left lower extremity of a client with type 2 insulin-requiring diabetes and cellulitis. What
should the nurse do?

You Selected:

Massage the left leg with alcohol to stimulate circulation.

Correct response:

Instruct the client to elevate the left leg when sitting in the chair.

Explanation:

Question 41 See full question

A priority for nursing care for an older adult who has pruritus, is continuously scratching the affected areas, and
demonstrates agitation and anxiety regarding the itching is:

You Selected:

instructing the client not to scratch.

Correct response:

preventing infection.

Explanation:

Question 42 See full question

Which serum electrolytes findings should the nurse expect to find in an infant with persistent vomiting?

You Selected:

K+, 5.5; Cl-, 110; Na+, 130

Correct response:

K+, 3.2; Cl-, 92; Na+, 120

Explanation:

Question 43 See full question

A client has had an exacerbation of ulcerative colitis with cramping and diarrhea persisting longer than 1 week. The
nurse should assess the client for which complication?

You Selected:

heart failure

Correct response:

hypokalemia
Explanation:

Question 44 See full question

A 12-year-old child is scheduled for surgery to repair a fractured tibia. One hour prior to surgery, the nurse assesses
that the child is febrile. What is the best action for the nurse to take?

You Selected:

Administer an antipyretic.

Correct response:

Inform the surgeon.

Explanation:

Question 45 See full question

A client is receiving intravenous fluids and upon assessment presents with increased pulse, increased respirations,
and jugular vein distension. What is the priority action by the nurse?

You Selected:

Administer oxygen and encourage the client to breathe deeply.

Correct response:

Slow the intravenous rate and notify the physician.

Explanation:

Question 46 See full question

A client with a history of hypertension has been prescribed a new antihypertensive medication and is reporting
dizziness. Which of the following is the best way for the nurse to assess blood pressure?

You Selected:

Taking blood pressure on the left arm and again in 5 minutes on the right arm

Correct response:

In the supine, sitting, and standing positions

Explanation:

Question 47 See full question

A client taking furosemide and digoxin for exacerbation of heart failure reports weakness and heart fluttering. What
would be the priority action by the nurse?

You Selected:

Offer the client clear instructions about avoiding foods that contain caffeine.

Correct response:

Investigate the symptoms further with the client and suggest contacting the physician.

Explanation:

Question 48 See full question

A client has been prescribed diuretic therapy for hypertension. It has been causing frequent urination at night and
now the client is refusing to take the morning dose of furosemide. What would be the best response by the nurse?

You Selected:

Tell the client that the extra fluid will be gone and urination will not be as frequent.
Correct response:

Reinforce the reason for the medication. Respect the decision if the client still refuses the medication, and
chart the refusal.

Explanation:

Question 49 See full question

A nurse places electrodes on a collapsed individual who was visiting a hospitalized family member, the monitor
exhibits the following. Which interventions would the nurse do first?

You Selected:

Confirm the rhythm with a 12-lead ECG.

Correct response:

Assess the clients airway, breathing, and circulation.

Explanation:

Question 50 See full question

A medication nurse is preparing to administer 9 a.m. medications to a client with liver cancer. Which consideration
is the nurses highest priority?

You Selected:

Metabolism of the medication

Correct response:

Metabolism of the medication

Explanation:

Question 1 See full question

Which sentence correctly describes the prone position?

You Selected:

The body is supine.

Correct response:

The body is facedown.

Explanation:

Question 2 See full question

A nurse is preparing a child, age 4, for cardiac catheterization. Which explanation of the procedure is appropriate?

You Selected:

"The special medicine will feel warm when it's put in the tubing."

Correct response:

"The special medicine will feel warm when it's put in the tubing."

Explanation:

Question 3 See full question

A client, age 87, with major depression undergoes a sixth electroconvulsive therapy (ECT) treatment. When
assessing the client immediately after ECT, the nurse expects to find:
You Selected:

transitory short- and long-term memory loss and confusion.

Correct response:

transitory short- and long-term memory loss and confusion.

Explanation:

Question 4 See full question

A nurse is assessing a client who has just been admitted to the emergency department. Which signs suggest an
overdose of an antianxiety agent?

You Selected:

Slurred speech, dyspnea, and impaired coordination

Correct response:

Slurred speech, dyspnea, and impaired coordination

Explanation:

Question 5 See full question

A client is admitted to the psychiatric clinic for treatment of anorexia nervosa. At the beginning of the client's
hospitalization, the most important nursing action is to:

You Selected:

monitor the client's vital signs, serum electrolyte levels, and acid-base balance.

Correct response:

monitor the client's vital signs, serum electrolyte levels, and acid-base balance.

Explanation:

Question 6 See full question

A client at term arrives in the labor unit experiencing contractions every 4 minutes. After a brief assessment, she's
admitted and an electric fetal monitor is applied. Which finding alerts the nurse to an increased risk for fetal
distress?

You Selected:

Total weight gain of 30 lb (13.6 kg)

Correct response:

Blood pressure of 146/90 mm Hg

Explanation:

Question 7 See full question

Which behavior should cause the nurse to suspect that a client's labor is moving quickly and that the physician
should be notified?

You Selected:

Episodes of nausea and vomiting

Correct response:

An increased sense of rectal pressure


Explanation:

Question 8 See full question

A term neonate's mother is O-negative, and cord studies indicate that the neonate is A-positive. Which finding
indicates that the neonate developed hemolytic disease?

You Selected:

Weight loss of less than 10%

Correct response:

Signs of kernicterus

Explanation:

Question 9 See full question

After completing discharge instructions for a primiparous client who is bottle-feeding her term neonate, the nurse
determines that the mother understands the instructions when the mother says that she should contact the
pediatrician if the neonate exhibits which sign or symptom?

You Selected:

ability to fall asleep easily after each feeding

Correct response:

passage of a liquid stool with a watery ring

Explanation:

Question 10 See full question

A loading dose of digoxin is given to a client newly diagnosed with atrial fibrillation. The nurse begins instructing the
client about the medication and the importance of monitoring his heart rate. An expected outcome of this
instruction is:

You Selected:

a return demonstration of how to take the medication.

Correct response:

a return demonstration of palpating the radial pulse.

Explanation:

Question 11 See full question

A client who had a transurethral resection of the prostate (TURP) 1 day earlier has a three-way Foley catheter
inserted for continuous bladder irrigation. Which of the following statements best explains why continuous
irrigation is used after TURP?

You Selected:

To prevent bladder distention.

Correct response:

To keep the catheter free from clot obstruction.

Explanation:

Question 12 See full question

A client reports having pain in the casted left arm that is unrelieved by pain medication. The nurse assesses the
arm and notes that the fingers are swollen and difficult to separate. What should the nurse do first?
You Selected:

Apply an ice bag to the fingers to relieve pain.

Correct response:

Call the health care provider (HCP) to report swelling and pain.

Explanation:

Question 13 See full question

Which finding should first alert the nurse that a child is hemorrhaging after a tonsillectomy?

You Selected:

mouth breathing

Correct response:

frequent swallowing

Explanation:

Question 14 See full question

When giving a client a tube feeding, the nurse should:

You Selected:

verify position of the tube before beginning feeding.

Correct response:

verify position of the tube before beginning feeding.

Explanation:

Question 15 See full question

When developing the plan of care for a toddler who has taken an acetaminophen overdose, which intervention
should the nurse expect to include as part of the initial treatment?

You Selected:

frequent blood level determinations

Correct response:

gastric lavage

Explanation:

Question 16 See full question

The nurse should teach the mother of a child who has a new cast for a fractured radius to do which intervention for
the first few days at home?

You Selected:

Administer acetaminophen every 8 to 12 hours for discomfort.

Correct response:

Have the child refrain from strenuous activities.


Explanation:

Question 17 See full question

A client has a leg immobilized in traction. Which observation by the nurse indicates that the client understands
actions to take to prevent muscle atrophy?

You Selected:

The client adducts the affected leg every 2 hours.

Correct response:

The client performs isometric exercises to the affected extremity three times per day.

Explanation:

Question 18 See full question

Which is an expected outcome for a client with peptic ulcer disease? The client will:

You Selected:

eliminate engaging in contact sports.

Correct response:

explain the rationale for eliminating alcohol from the diet.

Explanation:

Question 19 See full question

A client with bacterial pneumonia is to be started on IV antibiotics. Which diagnostic tests must be completed
before antibiotic therapy begins?

You Selected:

urinalysis

Correct response:

sputum culture

Explanation:

Question 20 See full question

The nurse is developing a care plan for a client who has leukemia. What instructions should the nurse include in the
plan? Select all that apply.

You Selected:

Avoid crowds.

Correct response:

Monitor temperature and report elevation.


Recognize signs and symptoms of infection.
Avoid crowds.
Maintain integrity of skin and mucous membranes.

Explanation:

Question 21 See full question

A nurse is helping a suspected choking victim. The nurse should perform the Heimlich maneuver when the victim:
You Selected:

cannot speak due to airway obstruction.

Correct response:

cannot speak due to airway obstruction.

Explanation:

Question 22 See full question

A client with eclampsia begins to experience a seizure. Which intervention should the nurse do immediately?

You Selected:

Insert a padded tongue blade into the mouth.

Correct response:

Maintain a patent airway.

Explanation:

Question 23 See full question

A young child who has undergone a tonsillectomy refuses to let the nurse look at the tonsillar beds to check for
bleeding. To assess whether the child is bleeding from the tonsillar beds, which measure would
be most appropriate?

You Selected:

Assess capillary refill.

Correct response:

Observe for frequent swallowing.

Explanation:

Question 24 See full question

When preparing a client for a scheduled colonoscopy, the nurse should tell the client that this procedure will
involve:

You Selected:

placing the client on a full-liquid diet 48 hours before the procedure.

Correct response:

cleansing the bowel with laxatives or enemas.

Explanation:

Question 25 See full question

A client is scheduled to undergo percutaneous transluminal coronary angioplasty (PTCA). Which statement by the
nurse best explains the procedure to the client?

You Selected:

PTCA involves passing a catheter through the coronary arteries to find blocked arteries.

Correct response:

PTCA involves opening a blocked artery with an inflatable balloon located on the end of a catheter.
Explanation:

Question 26 See full question

A client is scheduled for oral cholecystography. Prior to the test, the nurse should:

You Selected:

have the client drink 1,000 mL of water.

Correct response:

ask the client about possible allergies to iodine or shellfish.

Explanation:

Question 27 See full question

Which instruction is the most important to give a client who has recently had a skin graft?

You Selected:

Protect the graft from direct sunlight.

Correct response:

Protect the graft from direct sunlight.

Explanation:

Question 28 See full question

A nurse is teaching a client with adrenal insufficiency about corticosteroids. Which statement by the client indicates
a need for additional teaching?

You Selected:

"I may stop taking this medication when I feel better."

Correct response:

"I may stop taking this medication when I feel better."

Explanation:

Question 29 See full question

A client with hyperparathyroidism declines surgery and is to receive hormone replacement therapy with estrogen
and progesterone. Which instruction is most important for the nurse to include in the client's teaching plan?

You Selected:

"Lose weight."

Correct response:

"Maintain a moderate exercise program."

Explanation:

Question 30 See full question

A client is transferred to the intensive care unit after evacuation of a subdural hematoma. Which nursing
intervention reduces the client's risk of increased intracranial pressure (ICP)?

You Selected:

Administering a stool softener as ordered


Correct response:

Administering a stool softener as ordered

Explanation:

Question 31 See full question

A nurse is caring for a client admitted to the unit with a seizure disorder. The client seems upset and asks the nurse,
"What will they do to me? I'm scared of the tests and of what they'll find out." The nurse should focus her teaching
plans on which diagnostic tests?

You Selected:

Electrocardiography, TEE, prothrombin time (PT), and International Normalized Ratio (INR)

Correct response:

EEG, blood cultures, and neuroimaging studies

Explanation:

Question 32 See full question

Which statement indicates a client understands teaching about the purified protein derivative (PPD) test for
tuberculosis?

You Selected:

"If the test area turns red that means I have tuberculosis."

Correct response:

"Because I had a previous reaction to the test, this time I need to get a chest X-ray."

Explanation:

Question 33 See full question

The nurse is planning care for a client on complete bed rest. The plan of care should include all except:

You Selected:

maintaining the client in the supine position.

Correct response:

maintaining the client in the supine position.

Explanation:

Question 34 See full question

A primiparous client 3 days postpartum is to be discharged on heparin therapy. After teaching her about possible
adverse effects of heparin therapy, the nurse determines that the client needs further instruction when she states
that the adverse effects include which symptom?

You Selected:

petechiae

Correct response:

slow pulse

Explanation:

Question 35 See full question

Twelve hours after a vaginal birth with epidural anesthesia, the nurse palpates the fundus of a primiparous client
and finds it to be firm, above the umbilicus, and deviated to the right. What should the nurse do next?
You Selected:

Document this as a normal finding in the clients record.

Correct response:

Encourage the client to ambulate to the bathroom and void.

Explanation:

Question 36 See full question

A nurse is caring for a 3-year-old client with a neuroblastoma who has been receiving chemotherapy for the last 4
weeks. His laboratory test results indicate a Hgb of 12.5 g/dL (125 g/L), HCT of 36.8% (0.37), WBC of 2000 mm 3 (2 X
109/L), and platelet count of 150,000/L (150 X 109/L). Based on the child's values, what is the highest priority
nursing intervention?

You Selected:

Encourage mouth care with a soft toothbrush.

Correct response:

Encourage meticulous handwashing by the client and visitors.

Explanation:

Question 37 See full question

Which goal is most important when developing a long-term care plan for a child with hemophilia?

You Selected:

Improve the child's self-esteem during bleeding episodes.

Correct response:

Prevent injury during each stage of development.

Explanation:

Question 38 See full question

A client had surgery for a deviated nasal septum. Which finding would indicate that bleeding is occurring even if the
nasal drip pad remains dry and intact?

You Selected:

increased pain

Correct response:

repeated swallowing

Explanation:

Question 39 See full question

The nurse is observing a client with cerebral edema for evidence of increasing intracranial pressure and monitors
the blood pressure for signs of widening pulse pressure. The client's current blood pressure is 170/80 mm Hg. What
is the client's pulse pressure? Record your answer using a whole number.

Your Response:

Correct response:

90
Explanation:

Question 40 See full question

Following a total hip replacement, the nurse should position the client by:

You Selected:

elevating both feet on two pillows.

Correct response:

keeping the extremity in slight abduction using an abduction splint or pillows placed between the thighs.

Explanation:

Question 41 See full question

Which assessment findings would the nurse expect to find in the postoperative client experiencing fat embolism
syndrome?

You Selected:

Column D

Correct response:

Column B

Explanation:

Question 42 See full question

The nurse is offering further education to a client about the management of COPD. Which of the following outcomes
would indicate the teaching has been effective? Select all that apply.

You Selected:

Smoking cessation will be maintained.

Correct response:

The client can demonstrate pursed-lip breathing and coughing exercises.


Smoking cessation will be maintained.
Follow-up physician appointments will be made.

Explanation:

Question 43 See full question

The nurse is caring for an unconscious client recovering from anesthesia. Knowing the most common reason for
airway obstruction, which position should the nurse place the client in?

You Selected:

Reverse trendelenburg

Correct response:

Left lateral recumbent


Explanation:

Question 44 See full question

A 5-year-old child is brought to the emergency department after injuries sustained in a motor vehicle accident. The
child is diagnosed with a cervical spinal cord injury. Which assessment data would the nurse consider as most
significant when assessing for signs of cervical spinal cord swelling?

You Selected:

Urinary retention

Correct response:

Changes in respiration

Explanation:

Question 45 See full question

The nurse is assessing a client who has been admitted with impaired arterial circulation in the lower extremities due
to diabetes mellitus. Which of the following would be expected findings?

You Selected:

Edema and coolness in the ankles and feet

Correct response:

Absence of dorsalis pedis pulse, coolness, and decreased sensation in the feet

Explanation:

Question 46 See full question

Which is the best positioning for a client who has a fractured spine as a result of a diving accident?

You Selected:

Prone with head to the side

Correct response:

Supine with the head midline

Explanation:

Question 47 See full question

The nurse is preparing a client for a cardiac catheterization. Which of the following client statements would the
nurse need to report to the healthcare provider immediately?

You Selected:

I took my metformin this morning.

Correct response:

I took my metformin this morning.

Explanation:

Question 48 See full question

A client is diagnosed with diabetic ketoacidosis. Which of the following findings would the nurse anticipate?

You Selected:

Potassium 4.5 mEq/L


Correct response:

Arterial pH 7.33

Explanation:

Question 49 See full question

The nurse is caring for an elderly client with a fractured hip who is on bed rest. Which nursing interventions would
be included on the plan of care?

You Selected:

Encourage coughing and deep breathing, and limit fluid intake.

Correct response:

Turn the client every 2 hours, and encourage coughing and deep breathing.

Explanation:

Question 50 See full question

The nurse is caring for the following infant after surgery. Which short term goal is the priority?

You Selected:

The infant will continue breastfeeding 3 to 5 times daily.

Correct response:

The infant will remain infection free in the postoperative period.

Explanation:

Question 41 See full question

The nurse is caring for an elderly patient who needs help with ADLs. Which of the following is most important for
the nurse to understand when implementing care in order to avoid injury?

You Selected:

Bending and twisting while providing care may cause injury.

Correct response:

Bending and twisting while providing care may cause injury.

Explanation:

Question 42 See full question

A nurse recognizes that a client with tuberculosis needs further teaching when the client states:

You Selected:

"I'll have to take these medications for 9 to 12 months."

Correct response:

"It will be necessary for the people I work with to take medication."

Explanation:

Question 43 See full question

A client is scheduled for a renal arteriogram. No allergies are recorded in the client's medical record, and the client
is unable to provide allergy information. During the arteriogram, the nurse should be alert for which assessment
finding that may indicate an allergic reaction to the dye used?
You Selected:

Pruritus

Correct response:

Pruritus

Explanation:

Question 44 See full question

The nurse is inserting a nasogastric tube in an infant to administer feedings. In the accompanying figure, indicate
the location for the correct placement of the distal end of the tube.

You Selected:

Your selection and the correct area, market by the green box.

Explanation:

Question 45 See full question

A nurse is supervising a student during medication administration to a client. Which of the following action by the
student would cause the nurse to intervene during the med pass at the bedside?

You Selected:

First asks the client's name.

Correct response:

Check the room number and the client's name on the bed.

Explanation:

Question 46 See full question

During a bedside shift report, the nurse finds that the client is receiving the wrong IV solution. Which action by the
nurse is indicated?

You Selected:

Report the off-going nurse to the board of nursing

Correct response:

Write up an incident report describing the error


Explanation:

Question 47 See full question

A client has an indwelling urinary catheter and is prescribed physical therapy. As the client is being placed in a
wheelchair, which action by the assistant would need further clarification by the nurse?

You Selected:

The catheter drainage bag is placed on the lower side of the wheelchair.

Correct response:

The catheter bag is placed upon the clients lap for safe transport.

Explanation:

Question 48 See full question

Which nursing action best addresses the outcome: The client will be free from falls?

You Selected:

Install a monitoring system to help the client in an emergency situation.

Correct response:

Encourage use of grab bars and railings in the bathroom and halls

Explanation:

Question 49 See full question

A nurse is teaching a new mother how to prevent burns in the home. Which statement by the mother indicates
more teaching is required?

You Selected:

I will not hold my infant while drinking coffee.

Correct response:

I will heat my infants formula in the microwave.

Explanation:

Question 50 See full question

The nurse is conducting a routine risk assessment at a prenatal visit. Which question would be the best to screen
for intimate partner violence?

You Selected:

Does your partner have an arrest record?

Correct response:

How safe do you feel in your home?

Explanation:

A nurse is performing a sterile dressing change. Which action contaminates the sterile field?

You Selected:

Opening the outermost flap of a sterile package away from the body
Correct response:

Pouring solution onto a sterile field cloth

Explanation:

Question 2 See full question

Entering a client's room, a nurse on the maternity unit sees a mother slapping the face of a crying neonate. Which
action should the nurse take in this situation?

You Selected:

Confront the mother by asking her what she's doing and why.

Correct response:

Return the neonate to the nursery, inform the physician so he can thoroughly examine the neonate for
injuries, and notify social services for assistance.

Explanation:

Question 3 See full question

A nurse assists in writing a community plan for responding to a bioterrorism threat or attack. When reviewing the
plan, the director of emergency operations should have the nurse correct which intervention?

You Selected:

Clients exposed to anthrax should immediately remove contaminated clothing and place it in the hamper.

Correct response:

Clients exposed to anthrax should immediately remove contaminated clothing and place it in the hamper.

Explanation:

Question 4 See full question

After an infant undergoes surgical repair of a cleft lip, the physician orders elbow restraints. For this infant, the
postoperative care plan should include which nursing action?

You Selected:

Removing the restraints while the infant is asleep

Correct response:

Removing the restraints every 2 hours

Explanation:

Question 5 See full question

A 3-month-old infant with meningococcal meningitis has just been admitted to the pediatric unit. Which nursing
intervention has the highest priority?

You Selected:

instituting droplet precautions

Correct response:

instituting droplet precautions

Explanation:

Question 6 See full question

A nurse is teaching parents about accident prevention for a toddler. Which guideline is most appropriate?
You Selected:

Never leave a toddler unattended on a bed.

Correct response:

Make sure all medications are kept in containers with childproof safety caps.

Explanation:

Question 7 See full question

A nurse discussing injury prevention with a group of workers at a day-care center is focusing on toddlers. When
discussing this age-group, the nurse should stress that:

You Selected:

toddlers will always chase a ball that rolls into the street.

Correct response:

accidents are the leading cause of death among toddlers.

Explanation:

Question 8 See full question

A 15-year-old adolescent confides in the nurse that he has been contemplating suicide. He says he has developed a
specific plan to carry it out and pleads with the nurse not to tell anyone. What is the nurse's best response?

You Selected:

"We can keep this between you and me, but promise me you won't try anything."

Correct response:

"For your protection, I can't keep this secret. After I notify the physician, we will need to involve your
family. We want you to be safe."

Explanation:

Question 9 See full question

Emergency restraints or seclusion may be implemented without a physician's order under which condition?

You Selected:

When a licensed practitioner will do a face-to-face assessment within 1 hour

Correct response:

When a licensed practitioner will do a face-to-face assessment within 1 hour

Explanation:

Question 10 See full question

A nurse must restrain a client to ensure the safety of other clients. When using restraints, which principle is a
priority?

You Selected:

Have three staff members present, one to restrain each side of the client's body and one for the head.

Correct response:

Use an organized, efficient team approach to apply and secure the restraints.
Explanation:

Question 11 See full question

A client in early labor is connected to an external fetal monitor. The physician hasn't noted any restrictions on her
chart. The client tells the nurse that she needs to go to the bathroom frequently and that her partner can help her.
How should the nurse respond?

You Selected:

"Because you're connected to the monitor, you can't get out of bed. You'll need to use the bedpan."

Correct response:

"Please press the call button. I'll disconnect you from the monitor so you can get out of bed."

Explanation:

Question 12 See full question

A nurse is preparing to perform a postpartum assessment on a client who gave birth 5 hours ago. Which precaution
should the nurse plan to take for this procedure?

You Selected:

Washing the hands

Correct response:

Washing the hands and wearing latex gloves

Explanation:

Question 13 See full question

Which situations should a supervisor consider in making assignments for nurses in the neonatal unit?

You Selected:

A nurse with young children shouldn't care for a neonate with erythema toxicum.

Correct response:

A pregnant nurse shouldn't care for a neonate with cytomegalovirus (CMV).

Explanation:

Question 14 See full question

The nurse is receiving over the telephone a laboratory results report of a neonate's blood glucose level. The nurse
should:

You Selected:

indicate to the caller that the nurse cannot receive verbal results from laboratory tests for neonates, and
ask the laboratory to bring the written results to the nursery.

Correct response:

write down the results, read back the results to the caller from the laboratory, and receive confirmation
from the caller that the nurse understands the results.

Explanation:

Question 15 See full question

Whichis an initial clinical manifestation of gonorrhea in men?


You Selected:

urethral discharge

Correct response:

urethral discharge

Explanation:

Question 16 See full question

An older infant who has been injured in an automobile accident is to wear a splint on the injured leg. The mother
reports that the infant has become mobile even while wearing the splint. What should the nurse advise the mother
to do?

You Selected:

Keep the infant in the splint at night, removing it during the day.

Correct response:

Remove any unsafe items from the area in which the infant is mobile.

Explanation:

Question 17 See full question

A 14-year-old with rheumatic fever who is on bed rest is receiving an IV infusion of dextrose 5% r administered by
an infusion pump. The nurse should verify the alarm settings on the infusion pump at which times? Select all that
apply.

You Selected:

when the infusion is started

Correct response:

when the infusion is started


at the beginning of each shift
when the child returns from X-ray

Explanation:

Question 18 See full question

A diagnosis of hemophilia A is confirmed in an infant. Which of the instructions should the nurse provide the parents
as the infant becomes more mobile and starts to crawl?

You Selected:

Check the color of the child's urine every day.

Correct response:

Sew thick padding into the elbows and knees of the child's clothing.

Explanation:

Question 19 See full question

When developing the teaching plan for the mother of a 2-year-old child diagnosed with scabies, what information
should the nurse expect to include?

You Selected:

The floors of the house should be cleaned with a damp mop.


Correct response:

The entire family should be treated.

Explanation:

Question 20 See full question

A client with cervical cancer is undergoing internal radium implant therapy. A lead-lined container and a pair of long
forceps are kept in the client's hospital room for:

You Selected:

disposal of the client's eating utensils.

Correct response:

handling of the dislodged radiation source.

Explanation:

Question 21 See full question

The nurse understands that the client who is undergoing induction therapy for leukemia needs additional instruction
when the client makes which statement?

You Selected:

"I cannot wait to get home to my cat!"

Correct response:

"I cannot wait to get home to my cat!"

Explanation:

Question 22 See full question

After teaching the parents of a 15-month-old child who has undergone cleft palate repair how to use elbow
restraints, which statement by the parents indicates effective teaching?

You Selected:

The restraints should be taped directly to our childs arms so that they will stay in one place.

Correct response:

We will remove the restraints temporarily at least three times a day to check his skin, then put them right
back on.

Explanation:

Question 23 See full question

When planning home care for a 3-year-old child with eczema, what should the nurse teach the mother to remove
from the child's environment at home?

You Selected:

plastic figures

Correct response:

stuffed animals
Explanation:

Question 24 See full question

The nurse is instructing the unlicensed assistive personnel (UAP) on how to position the wheelchair to assist a client
with left-sided weakness transfer from the bed to a wheelchair using a transfer belt. Which statement by the UAP
tells the nurse that the UAP has understood the instructions for placing the wheelchair?

You Selected:

The wheelchair should be placed at the head of the bed.

Correct response:

The wheelchair should be placed on the right side of the bed.

Explanation:

Question 25 See full question

A client with a cerebellar brain tumor is admitted to an acute care facility. The nurse formulates a nursing diagnosis
of Risk for injury. Which "related-to" phrase should the nurse add to complete the nursing diagnosis statement?

You Selected:

Related to difficulty swallowing

Correct response:

Related to impaired balance

Explanation:

Question 26 See full question

A client is being admitted to the hospital with abdominal pain, anemia, and bloody stools. He complains of feeling
weak and dizzy. He has rectal pressure and needs to urinate and move his bowels. The nurse should help him:

You Selected:

to the bathroom.

Correct response:

onto the bedpan.

Explanation:

Question 27 See full question

A client reports to a physician's office for intradermal allergy testing. Before testing, the nurse provides client
teaching. Which client statement indicates a need for further education?

You Selected:

"If I notice tingling in my lips or mouth, gargling may help the symptoms."

Correct response:

"If I notice tingling in my lips or mouth, gargling may help the symptoms."

Explanation:

Question 28 See full question

Which statement by a staff nurse on the orthopedic floor indicates the need for further staff education?
You Selected:

"The client is receiving physical therapy twice per day, so he doesn't need a continuous passive motion
device."

Correct response:

"The client is receiving physical therapy twice per day, so he doesn't need a continuous passive motion
device."

Explanation:

Question 29 See full question

A client with chronic obstructive pulmonary disease (COPD) is intubated and placed on continuous mechanical
ventilation. Which equipment is most important for the nurse to keep at this client's bedside?

You Selected:

Oxygen analyzer

Correct response:

Manual resuscitation bag

Explanation:

Question 30 See full question

A clients blood pressure is elevated at 160/90 mm Hg. The health care provider (HCP) prescribed clonidine 1 mg
by mouth now. The nurse sent the prescription to pharmacy at 0710, but the medication still has not arrived at
0800. The nurse should do all except:

You Selected:

go to the pharmacy to obtain the drug.

Correct response:

go to the pharmacy to obtain the drug.

Explanation:

Question 31 See full question

After administering an I.M. injection, a nurse should:

You Selected:

recap the needle and discard the needle and syringe in a puncture-proof container.

Correct response:

discard the uncapped needle and syringe in a puncture-proof container.

Explanation:

Question 32 See full question

A nurse is caring for a client with a history of falls. The nurse's first priority when caring for a client at risk for falls
is:

You Selected:

keeping the bedpan available so that the client doesn't have to get out of bed.

Correct response:

keeping the bed in the lowest possible position.


Explanation:

Question 33 See full question

A multigravid client is admitted at 4-cm dilation and is requesting pain medication. The nurse gives the client
nalbuphine 15 mg. Within five minutes, the client tells the nurse she feels like she needs to have a bowel
movement. The nurse should first:

You Selected:

have naloxone hydrochloride available in the birthing room.

Correct response:

complete a vaginal examination to determine dilation, effacement, and station.

Explanation:

Question 34 See full question

A parent tells the nurse that their 6-year-old child has severe nosebleeds. To manage the nosebleed, the nurse
should tell the parent to:

You Selected:

help the child lie on the stomach and collect the blood on a clean towel.

Correct response:

place the child in a sitting position with the neck bent forward and apply firm pressure on the nasal
septum.

Explanation:

Question 35 See full question

The nurse is caring for a client who is confused about time and place. The client has intravenous fluid infusing. The
nurse attempts to reorient the client, but the client remains unable to demonstrate appropriate use of the call light.
In order to maintain client safety, what should the nurse do first?

You Selected:

Increase the frequency of client observation.

Correct response:

Increase the frequency of client observation.

Explanation:

Question 36 See full question

A client is admitted to the Emergency Department with a full thickness burn to the right arm. Upon assessment, the
arm is edematous, fingers are mottled, and radial pulse is now absent. The client states that the pain is 8 on a scale
of 1 to 10. The nurse should:

You Selected:

continue to assess the arm every hour for any additional changes.

Correct response:

call the health care provider (HCP) to report the loss of the radial pulse.
Explanation:

Question 37 See full question

A nurse assesses a client with psychotic symptoms and determines that the client likely poses a safety threat and
needs vest restraints. The client is adamantly opposed to this. What would be the best nursing action?

You Selected:

Ask a family member to come in to supervise the client.

Correct response:

Contact the physician and obtain necessary orders.

Explanation:

Question 38 See full question

A client living in a long-term care facility has become increasingly unsteady when out of bed. The nurse is worried
that the client is going to climb out of bed and fall. The facility has a least restraint policy for the clients. Which of
the following actions should the nurse take to best ensure the safety of the client while complying with policy?

You Selected:

Instruct the client on use of the call bell.

Correct response:

Provide a bed that is low to the floor.

Explanation:

Question 39 See full question

A nurse who is 6 months pregnant is assigned to a client with a diagnosis of HIV. The nurse tells the manager that
she is unable to care for the client because it would be a risk to her baby. Which of the following is the most
appropriate statement by the manager?

You Selected:

I will ask that you be transferred to another unit while you are pregnant so there is no risk to you or your
baby.

Correct response:

You will be OK if you follow standard precautions and use protective equipment to avoid contact with
blood and body fluids when providing care.

Explanation:

Question 40 See full question

A nurse administers digoxin 0.125 mg to a client at 1400 instead of the prescribed dose of digoxin 0.25 mg. Which
of the following statements should the nurse record in the medical record?

You Selected:

At 1400, wrong dose of digoxin given due to heavy workload.

Correct response:

Digoxin 0.125 mg given at 1400 instead of prescribed dose of 0.25 mg.


Explanation:

Question 41 See full question

The nurse is caring for a toddler who is visually impaired. What is the most important action for the nurse to take to
ensure the safety of the child?

You Selected:

Request that the parents stay with the child.

Correct response:

Maintain a tidy environment around the child.

Explanation:

Question 42 See full question

The nurse is caring for an elderly patient who needs help with ADLs. Which of the following is most important for
the nurse to understand when implementing care in order to avoid injury?

You Selected:

A client's level of consciousness and ability to cooperate are not important factors during transfer.

Correct response:

Bending and twisting while providing care may cause injury.

Explanation:

Question 43 See full question

A nurse recognizes that a client with tuberculosis needs further teaching when the client states:

You Selected:

"I'll have to take these medications for 9 to 12 months."

Correct response:

"It will be necessary for the people I work with to take medication."

Explanation:

Question 44 See full question

Which of the following is the priority action the nurse should take when finding medications at a clients bedside?

You Selected:

Leave the medications and seek the nurse who left them in the room.

Correct response:

Remove the medications from the room and discard them into an appropriate disposal bin.

Explanation:

Question 45 See full question

A nurse is caring for a client who is undergoing chemotherapy. Current laboratory values are noted on the medical
record. Which action would be most appropriate for the nurse to implement?

You Selected:

contacting the health care provider (HCP) for a prescription for hematopoietic factors such as
erythropoietin
Correct response:

washing hands before and after entering the room

Explanation:

Question 46 See full question

The nurse is inserting a nasogastric tube in an infant to administer feedings. In the accompanying figure, indicate
the location for the correct placement of the distal end of the tube.

You Selected:

Your selection and the correct area, market by the green box.

Explanation:

Question 47 See full question

Which action by the nursing assistant would require immediate intervention by the nurse?

You Selected:

Restraining a school-age child at risk for self-harm because the nursing assistant had to leave the room

Correct response:

Restraining a school-age child at risk for self-harm because the nursing assistant had to leave the room

Explanation:

Question 48 See full question

A nurse-manager identifies fall prevention as a unit priority. Which of the following actions can the nurses
implement to meet these goals? Select all that apply.

You Selected:

Use bed alarms to remind clients to call for help getting up

Correct response:

Use bed alarms to remind clients to call for help getting up


Maintain a clear path to client bathrooms
Make hourly rounds to client rooms
Explanation:

Question 49 See full question

A client is admitted with an infectious wound. Contact precautions are initiated. To help the client cope with staff
using isolation procedures, which nursing action is most helpful?

You Selected:

Speak to the client from the doorway unless needing close contact

Correct response:

Discuss the rationale for contact precautions

Explanation:

Question 50 See full question

The nurse is planning care with an older adult who is at risk for falling because of postural hypotension. Which
intervention will be most effective in preventing falls in this client?

You Selected:

Complete a fall diary.

Correct response:

Instruct the client to sit, obtain balance, dangle legs, and rise slowly.

Explanation:

Answer Key

Question 1 See full question

A nurse is performing a sterile dressing change. Which action contaminates the sterile field?

You Selected:

Holding sterile objects above the waist

Correct response:

Pouring solution onto a sterile field cloth

Explanation:

Question 2 See full question

A nurse is assigned to a client with a cardiac disorder. The nurse should question an order to monitor the client's
body temperature by which route?

You Selected:

Rectal

Correct response:

Rectal

Explanation:

Question 3 See full question

The staff of an outpatient clinic has formed a task force to develop new procedures for swift, safe evacuation of the
unit. The new procedures haven't been reviewed, approved, or shared with all personnel. When a nurse-manager
receives word of a bomb threat, the task force members push for evacuating the unit using the new procedures.
Which action should the nurse-manager take?

You Selected:

Tell staff members to use whatever procedures they feel are best.

Correct response:

Determine that the procedures currently in place must be followed and direct staff to follow them without
question.

Explanation:

Question 4 See full question

A nurse gives a client the wrong medication. After assessing the client, the nurse completes an incident report.
Which statement describes what will happen next?

You Selected:

The incident will be documented in the nurse's personnel file.

Correct response:

The incident report will provide a basis for promoting quality care and risk management.

Explanation:

Question 5 See full question

A physician has ordered a heating pad for an elderly client's lower back pain. Which item would be most important
for a nurse to assess before applying the heating pad?

You Selected:

Client's level of consciousness

Correct response:

Client's level of consciousness

Explanation:

Question 6 See full question

After his spouse has visited, a client begins crying and saying that his spouse is a mean person. When the client
starts pounding on the overbed table and using incomprehensible language, the nurse feels she can't handle the
situation. What should the nurse do at this time?

You Selected:

Use the call system to request assistance.

Correct response:

Use the call system to request assistance.

Explanation:

Question 7 See full question

Which of the following objects poses the most serious safety threat to a 2-year-old child in the hospital?

You Selected:

Mobile hanging over the crib


Correct response:

Side rails in the halfway position

Explanation:

Question 8 See full question

What should a nurse do to ensure a safe hospital environment for a toddler?

You Selected:

Place the child in a youth bed.

Correct response:

Move the equipment out of reach.

Explanation:

Question 9 See full question

A nurse discussing injury prevention with a group of workers at a day-care center is focusing on toddlers. When
discussing this age-group, the nurse should stress that:

You Selected:

the risk for homicide is highest among toddlers.

Correct response:

accidents are the leading cause of death among toddlers.

Explanation:

Question 10 See full question

A mother tells the nurse that her preschool-aged daughter with spina bifida sneezes and gets a rash when playing
with brightly colored balloons, and that recently she had an allergic reaction after eating kiwi fruit and bananas.
Based on the mother's report, the nurse suspects that the child may have an allergy to:

You Selected:

Bananas.

Correct response:

Latex.

Explanation:

Question 11 See full question

A 15-year-old adolescent confides in the nurse that he has been contemplating suicide. He says he has developed a
specific plan to carry it out and pleads with the nurse not to tell anyone. What is the nurse's best response?

You Selected:

"We can keep this between you and me, but promise me you won't try anything."

Correct response:

"For your protection, I can't keep this secret. After I notify the physician, we will need to involve your
family. We want you to be safe."
Explanation:

Question 12 See full question

When planning care for a client who has ingested phencyclidine (PCP), the nurse's highest priority should be
meeting the:

You Selected:

client's physical needs.

Correct response:

client's safety needs.

Explanation:

Question 13 See full question

A nurse is preparing to perform a postpartum assessment on a client who gave birth 5 hours ago. Which precaution
should the nurse plan to take for this procedure?

You Selected:

Washing the hands

Correct response:

Washing the hands and wearing latex gloves

Explanation:

Question 14 See full question

An emergency department nurse is awaiting the arrival of multiple persons exposed to botulism at the local
shopping mall. What should the nurse do?

You Selected:

Separate those exposed to botulism from those who were not exposed.

Correct response:

Activate the facility's emergency disaster plan.

Explanation:

Question 15 See full question

While reviewing the day's charts, a nurse who's been under a great deal of personal stress realizes that she forgot
to administer insulin to client with diabetes mellitus. She's made numerous errors in the past few weeks and is now
afraid her job is in jeopardy. What is her best course of action?

You Selected:

Administer the medication immediately and chart it as given on time.

Correct response:

Report the error, complete the proper paperwork, and meet with the unit manager.

Explanation:

Question 16 See full question

Which activity should the nurse recommend to the client on an inpatient unit when thoughts of suicide occur?

You Selected:

playing a card game with other clients


Correct response:

talking with the nurse

Explanation:

Question 17 See full question

Nursing staff are trying to provide for the safety of an elderly client with moderate dementia. The client is
wandering at night and has trouble keeping her balance. She has fallen twice but has had no resulting injuries. The
nurse should:

You Selected:

ask a family member to stay with the client at night.

Correct response:

move the client to a room near the nurse's station and install a bed alarm.

Explanation:

Question 18 See full question

Whichis an initial clinical manifestation of gonorrhea in men?

You Selected:

urethral discharge

Correct response:

urethral discharge

Explanation:

Question 19 See full question

Bacterial conjunctivitis has affected several children at a local day care center. A nurse should advise which
measure to minimize the risk of infection?

You Selected:

Restrict the infected children from returning for 48 hours after treatment.

Correct response:

Perform thorough hand washing before and after touching any child in the day care center.

Explanation:

Question 20 See full question

A diagnosis of hemophilia A is confirmed in an infant. Which of the instructions should the nurse provide the parents
as the infant becomes more mobile and starts to crawl?

You Selected:

Expect the eruption of the primary teeth to produce moderate to severe bleeding.

Correct response:

Sew thick padding into the elbows and knees of the child's clothing.
Explanation:

Question 21 See full question

A 3-year-old child receiving chemotherapy after surgery for a Wilms' tumor has developed neutropenia. The parent
is trying to encourage the child to eat by bringing extra foods to the room. Which food would not be appropriate for
this child?

You Selected:

fresh strawberries

Correct response:

fresh strawberries

Explanation:

Question 22 See full question

The nurse in the emergency department is administering a prescription for 20 mg intravenous furosemide, which is
to be given immediately. The nurse scans the clients identification band and the medication barcode. The
medication administration system does not verify that furosemide is prescribed for this client; however, the
furosemide is prepared in the accurate unit dose for intravenous infusion. What should the nurse do next?

You Selected:

Contact the pharmacist immediately to check the order and the barcode label for accuracy.

Correct response:

Contact the pharmacist immediately to check the order and the barcode label for accuracy.

Explanation:

Question 23 See full question

To ensure safety for a hospitalized blind client, the nurse should:

You Selected:

orient the client to the room environment.

Correct response:

orient the client to the room environment.

Explanation:

Question 24 See full question

The nurse should use which type of precautions for a client being admitted to the hospital with suspected
tuberculosis?

You Selected:

contact precautions

Correct response:

airborne precautions

Explanation:

Question 25 See full question

The nurse understands that the client who is undergoing induction therapy for leukemia needs additional instruction
when the client makes which statement?
You Selected:

"I will use warm saline gargle instead of brushing my teeth."

Correct response:

"I cannot wait to get home to my cat!"

Explanation:

Question 26 See full question

After teaching the parents of a 15-month-old child who has undergone cleft palate repair how to use elbow
restraints, which statement by the parents indicates effective teaching?

You Selected:

The restraints should be taped directly to our childs arms so that they will stay in one place.

Correct response:

We will remove the restraints temporarily at least three times a day to check his skin, then put them right
back on.

Explanation:

Question 27 See full question

In caring for the client with hepatitis B, which situation would expose the nurse to the virus?

You Selected:

a blood splash into the nurses eyes

Correct response:

a blood splash into the nurses eyes

Explanation:

Question 28 See full question

Before assisting a client to ambulate after surgery, the nurse helps the client to dangle the feet over the side of the
bed. Which action will best prepare the client to dangle the feet over the side of the bed?

You Selected:

Position the client on his or her side for 5 minutes.

Correct response:

Place the client in a high Fowlers position.

Explanation:

Question 29 See full question

A client with a cerebellar brain tumor is admitted to an acute care facility. The nurse formulates a nursing diagnosis
of Risk for injury. Which "related-to" phrase should the nurse add to complete the nursing diagnosis statement?

You Selected:

Related to psychomotor seizures

Correct response:

Related to impaired balance


Explanation:

Question 30 See full question

A client who was bitten by a wild animal is admitted to an acute care facility for treatment of rabies. Which type of
isolation does this client require?

You Selected:

Enteric

Correct response:

Contact

Explanation:

Question 31 See full question

A client is preparing for discharge from the emergency department after sustaining an ankle sprain. The client is
instructed to avoid weight bearing on the affected leg and is given crutches. After instruction, the client
demonstrates proper crutch use in the hallway. What additional information is most important to know before
discharging the client?

You Selected:

Whether the client drives a car with a stick shift

Correct response:

Whether the client needs to navigate stairs routinely at home

Explanation:

Question 32 See full question

A client with a suspected brain tumor is scheduled for a computed tomography (CT) scan. What should the nurse do
when preparing the client for this test?

You Selected:

Administer a sedative as ordered.

Correct response:

Determine whether the client is allergic to iodine, contrast dyes, or shellfish.

Explanation:

Question 33 See full question

A client with suspected severe acute respiratory syndrome (SARS) comes to the emergency department. Which
physician order should the nurse implement first?

You Selected:

Obtain a nasopharyngeal specimen for reverse-transcription polymerase chain reaction testing.

Correct response:

Institute isolation precautions.

Explanation:

Question 34 See full question

Which client is at highest risk for developing a hospital-acquired infection?


You Selected:

A client with an indwelling urinary catheter

Correct response:

A client with an indwelling urinary catheter

Explanation:

Question 35 See full question

After administering an I.M. injection, a nurse should:

You Selected:

recap the needle and discard the needle and syringe in any medical waste container.

Correct response:

discard the uncapped needle and syringe in a puncture-proof container.

Explanation:

Question 36 See full question

An alert and oriented older adult female with metastatic lung cancer is admitted to the medical-surgical unit for
treatment of heart failure. She was given 80 mg of furosemide in the emergency department. Although the client is
ambulatory, the unlicensed assistive personnel (UAP) are concerned about urinary incontinence because the client
is frail and in a strange environment. The nurse should instruct the UAP to assist with implementing the nursing
plan of care by:

You Selected:

padding the bed with extra absorbent linens

Correct response:

placing a commode at the bedside and instructing the client in its use

Explanation:

Question 1 See full question

The fire alarm sounds on the maternal-neonatal unit at 0200. How can a nurse best care for her clients during a fire
alarm?

You Selected:

Close all of the doors on the unit.

Correct response:

Close all of the doors on the unit.

Explanation:

Question 2 See full question

The risk for injury during an attack of Mnire's disease is high. The nurse should instruct the client to take which
immediate action when experiencing vertigo?

You Selected:

"Place your head between your knees."

Correct response:

"Assume a reclining or flat position."


Explanation:

Question 3 See full question

When changing a wet-to-dry dressing covering a surgical wound, what should the nurse do?

You Selected:

Cover the wet packing with a dry sterile dressing.

Correct response:

Cover the wet packing with a dry sterile dressing.

Explanation:

Question 4 See full question

While making rounds, the nurse enters a clients room and finds the client on the floor between the bed and the
bathroom. In which order of priority from first to last should the nurse take the actions? All options must be used.

You Selected:

Assess the client's current condition and vital signs.


If no acute injury, get help, and carefully assist the client back to bed.
Document as required by the facility.
Notify the clients health care provider (HCP) and family.

Correct response:

Assess the client's current condition and vital signs.


If no acute injury, get help, and carefully assist the client back to bed.
Notify the clients health care provider (HCP) and family.
Document as required by the facility.

Explanation:

Question 5 See full question

A nurse working in the emergency department is concerned that a client, who is in police custody, is handcuffed to
the stretcher. The nurse asks the police officer to remove the cuffs, but the officer refuses. What should be the next
action by the nurse?

You Selected:

Call the supervisor and report the officers decision to keep the cuffs on.

Correct response:

Continue to assess the client, allowing the officer to assume responsibility for the restraint.

Explanation:

Question 37 See full question

A nurse discovers that a hospitalized client with stage 4 esophageal cancer and major depression has a gun in the
home. What is the best nursing intervention to help the client remain safe after discharge?

You Selected:

Have the client promise to use the gun only for home protection.

Correct response:

Talk with the health care provider (HCP) about requiring gun removal as a condition of discharge.
Explanation: Question 1 See full question

An older infant who has been injured in an automobile accident is to wear a splint on the injured leg. The mother
reports that the infant has become mobile even while wearing the splint. What should the nurse advise the mother
to do?

You Selected:

Remove any unsafe items from the area in which the infant is mobile.

Correct response:

Remove any unsafe items from the area in which the infant is mobile.

Explanation:

Question 2 See full question

A client with chronic obstructive pulmonary disease (COPD) is intubated and placed on continuous mechanical
ventilation. Which equipment is most important for the nurse to keep at this client's bedside?

You Selected:

Manual resuscitation bag

Correct response:

Manual resuscitation bag

Explanation:

Question 3 See full question

The nurse assesses a client to be at risk for self-mutilation and implements a safety contract with the client. Which
client behavior indicates that the contract is working?

You Selected:

The client notifies staff when anxiety is increasing.

Correct response:

The client notifies staff when anxiety is increasing.

Explanation:

Question 4 See full question

When preparing the room for admission of a multigravid client at 36 weeks gestation diagnosed with severe
preeclampsia, which item is most important for the nurse to obtain?

You Selected:

padding for the side rails

Correct response:

padding for the side rails

Explanation:

Question 5 See full question

A charge nurse observes two nurses using inappropriate technique when starting an I.V. on a child. The charge
nurse should first:

You Selected:

talk with the nurses about proper technique and the risk of infection resulting from improper technique.
Correct response:

talk with the nurses about proper technique and the risk of infection resulting from improper technique.

Explanation:

Question 38 See full question

A 13-year-old is having surgery to repair a fractured left femur. As a part of the preoperative safety checklist, what
should the nurse do?

You Selected:

Ask the teen to point to the surgery site.

Correct response:

Verify that the site, side, and level are marked.

Explanation:

Question 39 See full question

The nurse is caring for a client after surgery. The surgeon has written resume pre-op meds as an order on a
clients chart. What should the nurse do next?

You Selected:

Transcribe the preoperative medication orders the surgeon has ordered.

Correct response:

Contact the surgeon for clarification because this is not a complete order.

Explanation:

Question 40 See full question

The nurse manager of a surgical unit observes a nurse providing colostomy care to a client without using any
personal protective equipment (PPE). What is the most appropriate response by the nurse manager in relation to
the use of PPE?

You Selected:

You should be aware that PPE is used when caring for any client in the hospital.

Correct response:

PPE should be used when you risk exposure to blood or bodily fluids.

Explanation:

Question 41 See full question

A nurse practitioner (NP) orders an antibiotic to which the client is allergic. The nurse preparing the medication
notices the allergy alert and contacts the NP by phone. The NP does not return the call and the first dose is due to
be given. Which of the following actions by the nurse is the best solution to this situation?

You Selected:

Hold the medication until speaking with the NP.

Correct response:

Hold the medication until speaking with the NP.


Explanation:

Question 42 See full question

A staff nurse is caring for a child with a urinary tract infection. The nurse is 1 hour late administering the childs
prescribed antibiotic therapy and pain medication. The charge nurse challenges the staff nurse about the lateness
of the medications. The staff nurse responds, Its no big deal; at least the child got the medication. What is the
best course of action for the charge nurse to take?

You Selected:

Apologize to the parents for the staff nurses behavior.

Correct response:

Speak to the unit manager and fill out a medication error report.

Explanation:

Question 43 See full question

The nurse is caring for a toddler who is visually impaired. What is the most important action for the nurse to take to
ensure the safety of the child?

You Selected:

Avoid startling the child by limiting excess noise.

Correct response:

Maintain a tidy environment around the child.

Explanation:

Question 44 See full question

The nurse is caring for an immune compromised client with a fungal infection of the scalp. What recommendation
should the nurse make to prevent future problems?

You Selected:

Wash hair with a dandruff-preventing shampoo.

Correct response:

Avoid sharing combs and brushes.

Explanation:

Question 45 See full question

A client is scheduled for a renal arteriogram. No allergies are recorded in the client's medical record, and the client
is unable to provide allergy information. During the arteriogram, the nurse should be alert for which assessment
finding that may indicate an allergic reaction to the dye used?

You Selected:

Nausea

Correct response:

Pruritus

Explanation:

Question 46 See full question

The nurse is inserting a nasogastric tube in an infant to administer feedings. In the accompanying figure, indicate
the location for the correct placement of the distal end of the tube.
You Selected:

Your selection and the correct area, market by the green box.

Explanation:

Question 47 See full question

A nurse is caring for a client with acquired immunodeficiency syndrome (AIDS). To adhere to standard precautions,
the nurse should wear gloves when:

You Selected:

delivering the client's food tray.

Correct response:

providing mouth care.

Explanation:

Question 48 See full question

The nurse is caring for a client recently diagnosed with hepatitis C. In reviewing the clients history, what
information will be most helpful as the nurse develops a teaching plan? The client:

You Selected:

has a history of exercise-induced asthma.

Correct response:

has a known history of sexually transmitted disease.

Explanation:

Question 49 See full question

A nurse is teaching a new mother how to prevent burns in the home. Which statement by the mother indicates
more teaching is required?

You Selected:

I will keep loose appliance cords tied up on the counter.

Correct response:

I will heat my infants formula in the microwave.


Explanation:

Question 50 See full question

The nurse is teaching the family of a client diagnosed with leukemia about ways to prevent infection. Which
instruction has the most impact?

You Selected:

Ingesting a plant-based diet

Correct response:

Maintaining an intact skin integrity

Explanation:

Question 1 See full question

When moving a client in bed, the nurse can ensure proper body mechanics by:

You Selected:

standing with her feet apart.

Correct response:

standing with her feet apart.

Explanation:

Question 2 See full question

After an instructor has posted assignments, a person claiming to be a nursing student arrives on a unit and asks a
nurse for access to the medication records of a client to whom she's assigned. The student's only identification (ID)
is a laboratory coat with the school's name on it. What is the nurse's most appropriate response?

You Selected:

Ask the student to provide a photo ID for comparison with the names on the assignment sheet.

Correct response:

Ask the student to provide a photo ID for comparison with the names on the assignment sheet.

Explanation:

Question 3 See full question

A nurse is teaching the parents of a young child how to handle suspected poisoning. If the child ingests poison, the
parents should first:

You Selected:

call the poison control center.

Correct response:

call the poison control center.

Explanation:

Question 4 See full question

When developing a teaching plan for parents of toddlers about poisonous substances, the nurse should emphasize
which safety points? Select all that apply.
You Selected:

Following any poisoning, the parents should call the Poison Control Center for instructions for appropriate
treatment.
All poisonous substances should be kept out of the reach of children and stored in a locked cabinet if
necessary.
The difference between pediatric and adult dosages of medicines is significant, and adult dosages given to
children can have serious, harmful effects.
Toddlers should be adequately supervised at all times.

Correct response:

Toddlers should be adequately supervised at all times.


All poisonous substances should be kept out of the reach of children and stored in a locked cabinet if
necessary.
The difference between pediatric and adult dosages of medicines is significant, and adult dosages given to
children can have serious, harmful effects.
Following any poisoning, the parents should call the Poison Control Center for instructions for appropriate
treatment.

Explanation:

Question 5 See full question

When preparing a client with acquired immunodeficiency syndrome (AIDS) for discharge to home, the nurse should
be sure to include which instruction?

You Selected:

"Put on disposable gloves before bathing."

Correct response:

"Avoid sharing such articles as toothbrushes and razors."

Explanation:

Question 6 See full question

A client is to have a below-the-knee amputation. Prior to surgery, the circulating nurse in the operating room
should:

You Selected:

initiate a time-out.

Correct response:

initiate a time-out.

Explanation:

Question 7 See full question

During rounds, a nurse finds that a client with hemiplegia has fallen from the bed because the nursing assistant
failed to raise the side rails after giving a back massage. The nurse assists the client to the bed and assesses for
injury. As per agency policies, the nurse fills out an incident report. Which of the following activities should the
nurse perform after finishing the incident report?

You Selected:

Include the time and date of the incident.

Correct response:

Include the time and date of the incident.


Explanation:

Question 8 See full question

A nurse is presenting an in-service on the topic of preventing urinary tract infections in young girls. The nurse talks
about evidence-based practice and teaching young girls to wipe from front to back. Another nurse interrupts,
stating, I havent seen any research to justify wiping front to back. It really makes no difference. What is the most
appropriate response by the nurse presenting the in-service?

You Selected:

Current professional research indicates that this technique is critical in preventing infections.

Correct response:

Current professional research indicates that this technique is critical in preventing infections.

Explanation:

Question 9 See full question

A nurse is caring for a client after a hemorrhoidectomy. Which of the following orders would the nurse question on
the medical record?

You Selected:

Stool softener daily

Correct response:

Low-fiber diet

Explanation:

Question 10 See full question

The nurse is caring for a client with a nasogastric tube and in mitt restraints. Which nursing action is required every
one to two hours?

You Selected:

Remove restraints and assess skin and circulation.

Correct response:

Remove restraints and assess skin and circulation.

Explanation:

Question 1 See full question

A nurse is teaching accident prevention to the parents of a toddler. Which instruction is appropriate for the nurse to
tell the parents?

You Selected:

Place locks on cabinets containing toxic substances.

Correct response:

Place locks on cabinets containing toxic substances.

Explanation:

Question 2 See full question

A nurse is teaching the parents of a young child how to handle suspected poisoning. If the child ingests poison, the
parents should first:
You Selected:

administer ipecac syrup.

Correct response:

call the poison control center.

Explanation:

Question 3 See full question

A nurse is caring for a client returning from cardiac catheterization. The nurse helps transfer the client back to bed.
Which transfer technique uses appropriate ergonomic principles?

You Selected:

The nurse raises the bed for transfer, maintains a wide base of support during transfer, and lowers the
bed before leaving the room.

Correct response:

The nurse raises the bed for transfer, maintains a wide base of support during transfer, and lowers the
bed before leaving the room.

Explanation:

Question 4 See full question

When a nurse removes an I.V. from an client with acquired immunodeficiency syndrome (AIDS), blood splashes into
her eyes. What should the nurse do next?

You Selected:

Rinse her eyes with water, report the incident, and go to Employee Health.

Correct response:

Rinse her eyes with water, report the incident, and go to Employee Health.

Explanation:

Question 5 See full question

A client with stage II Alzheimer's disease is admitted to the short stay unit after cardiac catheterization that
involved a femoral puncture. The client is reminded to keep his leg straight. A knee immobilizer is applied, but the
client repeatedly attempts to remove it. The nurse is responsible for three other clients who underwent cardiac
catheterization. What's the best step the nurse can take?

You Selected:

Ask the staffing coordinator to assign a nursing assistant to sit with the client.

Correct response:

Ask the staffing coordinator to assign a nursing assistant to sit with the client.

Explanation:

Question 6 See full question

The client has various sensory impairments associated with type 1 diabetes. The nurse determines that the client
needs further instruction when the client says:

You Selected:

"I will avoid kitchen activities."


Correct response:

"I will avoid kitchen activities."

Explanation:

Question 7 See full question

The nurse is reconciling the medications with a client who is being discharged. Which information indicates there is
a "discrepancy"?

You Selected:

There is lack of congruence between a clients home medication list and current medication prescriptions.

Correct response:

There is lack of congruence between a clients home medication list and current medication prescriptions.

Explanation:

Question 8 See full question

The nurse is caring for a client with influenza. The most effective way to decrease the spread of microorganisms is:

You Selected:

washing the hands frequently.

Correct response:

washing the hands frequently.

Explanation:

Question 9 See full question

The family cannot go with the surgical client past the doors that separate the public from the restricted area of the
operating room suite. These measures are designed to:

You Selected:

provide for an aseptic environment to prevent infection.

Correct response:

provide for an aseptic environment to prevent infection.

Explanation:

Question 10 See full question

A nurse inadvertently transcribes a clients medication order that was written as Ampicillin 250 mg four times a
day" as Ampicillin 2500 mg four times a day. The nurse gives two doses as transcribed to the client. Another nurse
gives one dose before the pharmacist questions the reorder of the medication. What should the two nurses do in
this situation?

You Selected:

Both nurses must acknowledge making the medication error.

Correct response:

Both nurses must acknowledge making the medication error.


Explanation:

Question 1 See full question

A 10-year-old child presents to the emergency department with dehydration. A physician orders 1 L of normal saline
solution be administered at a rate of 60 ml/hour. While preparing the infusion, a nurse notices that the I.V. pump's
safety inspection sticker has expired. Which action should the nurse take next?

You Selected:

Take the pump out of commission and locate a pump with a valid inspection sticker.

Correct response:

Take the pump out of commission and locate a pump with a valid inspection sticker.

Explanation:

Question 2 See full question

A client refuses his evening dose of haloperidol and then becomes extremely agitated in the day room while other
clients are watching television. He begins cursing and throwing furniture. The nurse's first action is to:

You Selected:

remove all other clients from the day room.

Correct response:

remove all other clients from the day room.

Explanation:

Question 3 See full question

A client arrives on the psychiatric unit exhibiting extreme excitement, disorientation, incoherent speech, agitation,
frantic and aimless physical activity, and grandiose delusion. Which nursing diagnosis takes highest priority for this
client at this time?

You Selected:

Risk for injury

Correct response:

Risk for injury

Explanation:

Question 4 See full question

Which finding best indicates that a nursing assistant has an understanding of blood glucose meter use?

You Selected:

Demonstrating correct technique

Correct response:

Demonstrating correct technique

Explanation:

Question 5 See full question

The nurse has provided an in-service presentation to ancillary staff about standard precautions on the birthing unit.
The nurse determines that one of the staff members needs further instructions when the nurse makes which
observation?
You Selected:

wearing of sterile gloves to bathe a neonate at 2 hours of age

Correct response:

wearing of sterile gloves to bathe a neonate at 2 hours of age

Explanation:

Question 6 See full question

A client with recurrent, endogenous depression has been hospitalized on the psychiatric unit for 3 days. He exhibits
psychomotor retardation, anhedonia, indecision, and suicidal thoughts. Which goal of nursing care should
have highest priority?

You Selected:

Use measures to protect the client from harming himself.

Correct response:

Use measures to protect the client from harming himself.

Explanation:

Question 7 See full question

A 29-year-old multigravida at 37 weeks' gestation is being treated for severe preeclampsia and has magnesium
sulfate infusing at 3 g/h. To maintain safety for this client, the priority intervention is to:

You Selected:

assess reflexes, clonus, visual disturbances, and headache.

Correct response:

assess reflexes, clonus, visual disturbances, and headache.

Explanation:

Question 8 See full question

The family cannot go with the surgical client past the doors that separate the public from the restricted area of the
operating room suite. These measures are designed to:

You Selected:

provide for an aseptic environment to prevent infection.

Correct response:

provide for an aseptic environment to prevent infection.

Explanation:

Question 9 See full question

A client has a history of macular degeneration. While in the hospital, the priority nursing goal will be to:

You Selected:

promote a safe, effective care environment.

Correct response:

promote a safe, effective care environment.


Explanation:

Question 10 See full question

A school nurse is conducting a seminar for parents of preschool children on the prevention of head injuries. What is
the most appropriate information for the nurse to give the parents?

You Selected:

Children should always wear helmets when riding bicycles.

Correct response:

Children should always wear helmets when riding bicycles.

Explanation:

Question 1 See full question

Which technique is most effective in preventing nosocomial infection transmission when caring for a preschooler?

You Selected:

Hand washing

Correct response:

Hand washing

Explanation: Question 1 See full question

When checking a client's medication profile, a nurse notes that the client is receiving a drug contraindicated for
clients with glaucoma. The nurse knows that this client, who has a history of glaucoma, has been taking the
medication for the past 3 days. What should the nurse do first?

You Selected:

Hold the medication and report the information to the physician to ensure client safety.

Correct response:

Hold the medication and report the information to the physician to ensure client safety.

Explanation:

Question 2 See full question

An infant requires cardiorespiratory monitoring. A nurse must locate and clean the necessary equipment, move it
into the infant's room, and secure it to the bedside wall-mounting device. Which principles should a nurse use to
complete this task safely?

You Selected:

Principles of infection control and ergonomics

Correct response:

Principles of infection control and ergonomics

Explanation:

Question 3 See full question

A client is transferred from the emergency department to the locked psychiatric unit after attempting suicide by
taking 200 acetaminophen tablets. The client is now awake and alert but refuses to speak with the nurse. In this
situation, the nurse's first priority is to:
You Selected:

ensure safety by initiating suicide precautions.

Correct response:

ensure safety by initiating suicide precautions.

Explanation:

Question 4 See full question

After teaching the parents of a 15-month-old child who has undergone cleft palate repair how to use elbow
restraints, which statement by the parents indicates effective teaching?

You Selected:

We will remove the restraints temporarily at least three times a day to check his skin, then put them right
back on.

Correct response:

We will remove the restraints temporarily at least three times a day to check his skin, then put them right
back on.

Explanation:

Question 5 See full question

The nurse is instructing the unlicensed assistive personnel (UAP) on how to position the wheelchair to assist a client
with left-sided weakness transfer from the bed to a wheelchair using a transfer belt. Which statement by the UAP
tells the nurse that the UAP has understood the instructions for placing the wheelchair?

You Selected:

The wheelchair should be placed on the right side of the bed.

Correct response:

The wheelchair should be placed on the right side of the bed.

Explanation:

Question 6 See full question

When the client is involuntarily committed to a hospital because he is assessed as being dangerous to himself or
others, which client rights are lost?

You Selected:

the right to leave the hospital against medical advice

Correct response:

the right to leave the hospital against medical advice

Explanation:

Question 7 See full question

A nurse records a clients finger stick blood glucose level and gives 2 units of regular insulin as ordered. At the next
scheduled blood glucose assessment, the nurse realizes that he/she previously tested and administered the insulin
to the wrong client. What is the nurses priority action related to this incident?

You Selected:

Assess both clients and call the appropriate physicians to notify them of the errors.

Correct response:

Assess both clients and call the appropriate physicians to notify them of the errors.
Explanation:

Question 8 See full question

A nurse practitioner (NP) orders an antibiotic to which the client is allergic. The nurse preparing the medication
notices the allergy alert and contacts the NP by phone. The NP does not return the call and the first dose is due to
be given. Which of the following actions by the nurse is the best solution to this situation?

You Selected:

Hold the medication until speaking with the NP.

Correct response:

Hold the medication until speaking with the NP.

Explanation:

Question 9 See full question

Which action by the nursing assistant would require immediate intervention by the nurse?

You Selected:

Restraining a school-age child at risk for self-harm because the nursing assistant had to leave the room

Correct response:

Restraining a school-age child at risk for self-harm because the nursing assistant had to leave the room

Explanation:

Question 10 See full question

The nurse is caring for a client recently diagnosed with hepatitis C. In reviewing the clients history, what
information will be most helpful as the nurse develops a teaching plan? The client:

You Selected:

has a known history of sexually transmitted disease.

Correct response:

has a known history of sexually transmitted disease.

Explanation:

Question 1 See full question

A client lives in a group home and visits the community mental health center regularly. During one visit with the
nurse, the client states, "The voices are telling me to hurt myself again." Which question by the nurse
is most important to ask?

You Selected:

"Are you going to hurt yourself?"

Correct response:

"Are you going to hurt yourself?"

Explanation:

Question 2 See full question

Which dietary strategy best meets the nutritional needs of a client with acquired immunodeficiency syndrome
(AIDS)?

You Selected:

Tell the client to prepare food in advance and leave it out to eat small amounts throughout the day.
Correct response:

Instruct the client to cook foods thoroughly and adhere to safe food-handling practices.

Explanation:

Question 3 See full question

When developing a teaching plan for parents of toddlers about poisonous substances, the nurse should emphasize
which safety points? Select all that apply.

You Selected:

Following any poisoning, the parents should call the Poison Control Center for instructions for appropriate
treatment.
All poisonous substances should be kept out of the reach of children and stored in a locked cabinet if
necessary.
The difference between pediatric and adult dosages of medicines is significant, and adult dosages given to
children can have serious, harmful effects.
Toddlers should be adequately supervised at all times.

Correct response:

Toddlers should be adequately supervised at all times.


All poisonous substances should be kept out of the reach of children and stored in a locked cabinet if
necessary.
The difference between pediatric and adult dosages of medicines is significant, and adult dosages given to
children can have serious, harmful effects.
Following any poisoning, the parents should call the Poison Control Center for instructions for appropriate
treatment.

Explanation:

Question 4 See full question

The nurse is administering an intramuscular injection to an infant. Indicate the appropriate site for this injection.

You Selected:

Your selection and the correct area, market by the green box.
Explanation:

Question 5 See full question

The nurse assesses a client to be at risk for self-mutilation and implements a safety contract with the client. Which
client behavior indicates that the contract is working?

You Selected:

The client notifies staff when anxiety is increasing.

Correct response:

The client notifies staff when anxiety is increasing.

Explanation:

Question 6 See full question

A client has a history of macular degeneration. While in the hospital, the priority nursing goal will be to:

You Selected:

promote a safe, effective care environment.

Correct response:

promote a safe, effective care environment.

Explanation:

Question 7 See full question

The nurse gives a client 0.25 mg of digoxin instead of the prescribed dose of 0.125 mg. What action should the
nurse take after realizing the mistake?

You Selected:

Assess the client and notify the physician.

Correct response:

Assess the client and notify the physician.

Explanation:

Question 8 See full question

A nurse practitioner (NP) orders an antibiotic to which the client is allergic. The nurse preparing the medication
notices the allergy alert and contacts the NP by phone. The NP does not return the call and the first dose is due to
be given. Which of the following actions by the nurse is the best solution to this situation?

You Selected:

Hold the medication until speaking with the NP.

Correct response:

Hold the medication until speaking with the NP.

Explanation:

Question 9 See full question

The nurse is caring for a client admitted for pneumonia with a history of hypertension and heart failure. The client
has reported at least one fall in the last 3 months. The client may ambulate with assistance, has a saline lock in
place, and has demonstrated appropriate use of the call light to request assistance. Using the Morse Fall Scale (see
chart), what is this clients total score and risk level?
You Selected:

60, high risk

Correct response:

60, high risk

Explanation:

Question 10 See full question

The family of a client, stung by a bee, is rushed the client to the emergency room. The client is experiencing hives
and redness at the site. Upon arrival, the client states, I feel a lump in my throat and I am sweating. I cant
breathe! I think I am going to die! The nurse anticipates which emergency treatment next?

You Selected:

Administer an injection of epinephrine stat.

Correct response:

Administer an injection of epinephrine stat.

Explanation:

Question 2 See full question

A nurse must restrain a client to ensure the safety of other clients. When using restraints, which principle is a
priority?

You Selected:

Have three staff members present, one to restrain each side of the client's body and one for the head.

Correct response:

Use an organized, efficient team approach to apply and secure the restraints.

Explanation:

Question 3 See full question

After the initial phase of the burn injury, the client's plan of care will focus primarily on:

You Selected:

preventing infection.

Correct response:

preventing infection.

Explanation:

Question 4 See full question

The charge nurse on a hematology/oncology unit is reviewing the policy for using abbreviations with the staff. The
charge nurse should emphasize which information about why dangerous abbreviations need to be eliminated?
Select all that apply.

You Selected:

to ensure efficient and accurate communication


to ensure client safety
to prevent medication errors
Correct response:

to ensure efficient and accurate communication


to prevent medication errors
to ensure client safety

Explanation:

Question 5 See full question

An unlicensed assistive personnel (UAP) is taking care of a child in the arm restraint shown in the figure. To provide
care for this child, the nurse should instruct the UAP to:

You Selected:

unpin the restraint and perform range-of-motion exercises.

Correct response:

leave the restraint in its current position.

Explanation:

Question 6 See full question

A client is preparing for discharge from the emergency department after sustaining an ankle sprain. The client is
instructed to avoid weight bearing on the affected leg and is given crutches. After instruction, the client
demonstrates proper crutch use in the hallway. What additional information is most important to know before
discharging the client?

You Selected:

Whether the client needs to navigate stairs routinely at home

Correct response:

Whether the client needs to navigate stairs routinely at home

Explanation:

Question 7 See full question

A nurse is caring for a client following a tonsillectomy and fails to routinely assess the back of the clients throat for
signs of bleeding. The nurse manager reviews the clients chart and notices the omission of the assessments. Which
of the following is the best response to the nurse regarding the missing assessments?

You Selected:

Failure to complete these assessments constitutes negligent behavior.

Correct response:

Failure to complete these assessments constitutes negligent behavior.

Explanation:

Question 8 See full question

A nurse reports to the hospital occupational health nurse (OHN) that he/she was splashed with blood during the
resuscitation of an HIV-positive client. The nurse asks the OHN when he/she will know whether he/she is positive or
negative for HIV infection. Which of the following is the most appropriate response by the OHN?

You Selected:

Accurate results will be obtained by testing at 3 months and again at 6 months.

Correct response:

Accurate results will be obtained by testing at 3 months and again at 6 months.


Explanation:

Question 9 See full question

A client is admitted to the healthcare facility with active tuberculosis (TB). The nurse should include which
intervention in the care plan?

You Selected:

Putting on an individually-fitted mask when entering the client's room

Correct response:

Putting on an individually-fitted mask when entering the client's room

Explanation:

Question 10 See full question

The nurse is applying a hand mitt restraint for a client with pruritus (see figure). The nurse should first:

You Selected:

verify the prescription to use the restraint.

Correct response:

verify the prescription to use the restraint.

Explanation:

Question 1 See full question

When teaching school-age children important injury prevention strategies, the nurse must use creativity to gain
cooperation because children tend not to comply with:

You Selected:

learning to swim.

Correct response:

wearing safety apparel (helmets, knee pads, elbow pads).

Explanation:

Question 2 See full question

A 3-year-old child receiving chemotherapy after surgery for a Wilms' tumor has developed neutropenia. The parent
is trying to encourage the child to eat by bringing extra foods to the room. Which food would not be appropriate for
this child?

You Selected:

fresh strawberries

Correct response:

fresh strawberries

Explanation:

Question 3 See full question

Which topic would be most important to include when teaching the parents how to promote overall toddler
development?

You Selected:

Discipline is critical to appropriate development.


Correct response:

Safety is a priority concern for this age-group.

Explanation:

Question 4 See full question

The nurse is instructing the unlicensed assistive personnel (UAP) on how to position the wheelchair to assist a client
with left-sided weakness transfer from the bed to a wheelchair using a transfer belt. Which statement by the UAP
tells the nurse that the UAP has understood the instructions for placing the wheelchair?

You Selected:

The wheelchair should be placed at the head of the bed.

Correct response:

The wheelchair should be placed on the right side of the bed.

Explanation:

Question 5 See full question

The nurse from the nursery is bringing a newborn to a mothers room. The nurse took care of the mother yesterday
and knows the mother and baby well. The nurse should implement which action to ensure the safest transition of
the infant to the mother?

You Selected:

Complete the hospital identification procedure with mother and infant.

Correct response:

Complete the hospital identification procedure with mother and infant.

Explanation:

Question 6 See full question

Which infections require contact precautions? Select all that apply:

You Selected:

Pertussis
Measles

Correct response:

Clostridium difficile
Methicillin-resistant staphylococcus aureus

Explanation:

Question 7 See full question

Over the past few weeks, a client in a long-term care facility has become increasingly unsteady. The nurses are
worried that the client will climb out of bed and fall. Which of the following measures does not comply with a least
restraint policy?

You Selected:

Raising all side rails while the client is in bed

Correct response:

Raising all side rails while the client is in bed


Explanation:

Question 8 See full question

After the discharge of a client from a surgical unit, the housekeeper brings a blue pill to the nurse. The pill was
found in the sheets when the linens were removed from the clients bed. The nurse reviews the clients medication
administration record, which shows that the client received this medication at 0800. What would be the nurses
priority action?

You Selected:

Complete an incident form and notify the physician.

Correct response:

Complete an incident form and notify the physician.

Explanation:

Question 9 See full question

A nurse realizes that data has been entered on the wrong client's written health record. Which of the following steps
should the nurse take to correct this documentation error?

You Selected:

Use liquid paper to cover over the incorrect entry.

Correct response:

Put a line through the entry, leaving the content visible, and initialize.

Explanation:

Question 10 See full question

A nurse is presenting an in-service on the topic of preventing urinary tract infections in young girls. The nurse talks
about evidence-based practice and teaching young girls to wipe from front to back. Another nurse interrupts,
stating, I havent seen any research to justify wiping front to back. It really makes no difference. What is the most
appropriate response by the nurse presenting the in-service?

You Selected:

Our community newspaper just ran an article supporting this technique.

Correct response:

Current professional research indicates that this technique is critical in preventing infections.

Explanation:

Class average
View performance for all Client Needs

Answer Key

Question 1 See full question

Entering a client's room, a nurse on the maternity unit sees a mother slapping the face of a crying neonate. Which
action should the nurse take in this situation?

You Selected:

Return the neonate to the nursery, inform the physician so he can thoroughly examine the neonate for injuries,
and notify social services for assistance.
Correct response:

Return the neonate to the nursery, inform the physician so he can thoroughly examine the neonate for injuries,
and notify social services for assistance.

Explanation:

Question 2 See full question

When discharging a 5-month-old infant from the hospital, the nurse checks to see whether the parent's car restraint
system for the infant is appropriate. Which restraint system would be safest?

You Selected:

A front-facing convertible car seat in the middle of the back seat

Correct response:

A rear-facing infant safety seat in the middle of the back seat

Explanation:

Question 3 See full question

A client diagnosed with schizophrenia for the last 2 years tells the nurse who has brought the morning medications,
"That is not my pill! My pill is blue, not green." What should the nurse tell the client?

You Selected:

"Go ahead and take it. You can trust me. I am watching out for your safety and well being."

Correct response:

"I will go back and check the drawer as well as telephone the pharmacy to check about any possible changes
in the medication color."

Explanation:

Question 4 See full question

The nurse understands that the client who is undergoing induction therapy for leukemia needs additional instruction
when the client makes which statement?

You Selected:

"I cannot wait to get home to my cat!"

Correct response:

"I cannot wait to get home to my cat!"

Explanation:

Question 5 See full question

The charge nurse on a hematology/oncology unit is reviewing the policy for using abbreviations with the staff. The
charge nurse should emphasize which information about why dangerous abbreviations need to be eliminated?
Select all that apply.

You Selected:

to ensure efficient and accurate communication


to prevent medication errors

Correct response:

to ensure efficient and accurate communication


to prevent medication errors
to ensure client safety
Explanation:

Question 6 See full question

A nurse is teaching a safety class for parents of preschoolers. Which injuries should the nurse include as common
among preschoolers? Select all that apply.

You Selected:

Pedestrian accidents
Drowning

Correct response:

Automobile accidents
Drowning
Pedestrian accidents
Fire

Explanation:

Question 7 See full question

A client with suspected severe acute respiratory syndrome (SARS) comes to the emergency department. Which
physician order should the nurse implement first?

You Selected:

Institute isolation precautions.

Correct response:

Institute isolation precautions.

Explanation:

Question 8 See full question

After administering an I.M. injection, a nurse should:

You Selected:

recap the needle and discard the needle and syringe in a puncture-proof container.

Correct response:

discard the uncapped needle and syringe in a puncture-proof container.

Explanation:

Question 9 See full question

The nurse is caring for a toddler who is visually impaired. What is the most important action for the nurse to take to
ensure the safety of the child?

You Selected:

Request that the parents stay with the child.

Correct response:

Maintain a tidy environment around the child.

Explanation:

Question 10 See full question

The nurse has provided an in-service presentation to ancillary staff about standard precautions on the birthing unit.
The nurse determines that one of the staff members needs further instructions when the nurse observes which
action?
You Selected:

wearing of sterile gloves to bathe a neonate at 2 hours of age

Correct response:

wearing of sterile gloves to bathe a neonate at 2 hours of age

Explanation:

Answer Key

Question 1 See full question

A physician orders chest physiotherapy for a client with pulmonary congestion. When should the nurse plan to
perform chest physiotherapy?

You Selected:

When the nurse has time

Correct response:

Before meals

Explanation:

Question 2 See full question

The nurse is receiving over the telephone a laboratory results report of a neonate's blood glucose level. The nurse
should:

You Selected:

request that the laboratory send the results by e-mail to transfer to the client's medical record.

Correct response:

write down the results, read back the results to the caller from the laboratory, and receive confirmation from
the caller that the nurse understands the results.

Explanation:

Question 3 See full question

A 3-year-old child receiving chemotherapy after surgery for a Wilms' tumor has developed neutropenia. The parent
is trying to encourage the child to eat by bringing extra foods to the room. Which food would not be appropriate for
this child?

You Selected:

fudge

Correct response:

fresh strawberries

Explanation:

Question 4 See full question

A client with cervical cancer is undergoing internal radium implant therapy. A lead-lined container and a pair of long
forceps are kept in the client's hospital room for:

You Selected:

storage of the radiation dose.


Correct response:

handling of the dislodged radiation source.

Explanation:

Question 5 See full question

A client is admitted to the Emergency Department with a full thickness burn to the right arm. Upon assessment, the
arm is edematous, fingers are mottled, and radial pulse is now absent. The client states that the pain is 8 on a scale
of 1 to 10. The nurse should:

You Selected:

call the health care provider (HCP) to report the loss of the radial pulse.

Correct response:

call the health care provider (HCP) to report the loss of the radial pulse.

Explanation:

Question 6 See full question

The nurse sees a client walking in the hallway who begins to have a seizure. What should the nurse do in order of
priority from first to last? All options must be used.

You Selected:

Maintain a patent airway.


Ease the client to the floor.
Obtain vital signs.
Record the seizure activity observed.

Correct response:

Ease the client to the floor.


Maintain a patent airway.
Obtain vital signs.
Record the seizure activity observed.

Explanation:

Question 7 See full question

A nurse assesses a client with psychotic symptoms and determines that the client likely poses a safety threat and
needs vest restraints. The client is adamantly opposed to this. What would be the best nursing action?

You Selected:

Apply wrist restraints instead of vest restraints.

Correct response:

Contact the physician and obtain necessary orders.

Explanation:

Question 8 See full question

A nurse inadvertently transcribes a clients medication order that was written as Ampicillin 250 mg four times a
day" as Ampicillin 2500 mg four times a day. The nurse gives two doses as transcribed to the client. Another nurse
gives one dose before the pharmacist questions the reorder of the medication. What should the two nurses do in
this situation?

You Selected:

Adjust the medication administration record to reflect the correct dose only.
Correct response:

Both nurses must acknowledge making the medication error.

Explanation:

Question 9 See full question

A nurse administers digoxin 0.125 mg to a client at 1400 instead of the prescribed dose of digoxin 0.25 mg. Which
of the following statements should the nurse record in the medical record?

You Selected:

Nurse accidentally gave digoxin 0.125 mg to the client at 1400.

Correct response:

Digoxin 0.125 mg given at 1400 instead of prescribed dose of 0.25 mg.

Explanation:

Question 10 See full question

Which is the correct knot used to secure a restraint correctly to the bed frame?

You Selected:

Correct response:

Explanation:

Question 1 See full question

A nurse gives a client the wrong medication. After assessing the client, the nurse completes an incident report.
Which statement describes what will happen next?
You Selected:

The incident will be documented in the nurse's personnel file.

Correct response:

The incident report will provide a basis for promoting quality care and risk management.

Explanation:

Question 2 See full question

A 3-month-old infant with meningococcal meningitis has just been admitted to the pediatric unit. Which nursing
intervention has the highest priority?

You Selected:

instituting droplet precautions

Correct response:

instituting droplet precautions

Explanation:

Question 3 See full question

The nurse's best explanation for why the severely neutropenic client is placed in reverse isolation is that reverse
isolation helps prevent the spread of organisms:

You Selected:

by using special techniques to handle the client's linens and personal items.

Correct response:

to the client from sources outside the client's environment.

Explanation:

Question 4 See full question

When developing a seminar on injury prevention to be presented to a group of parents of children from 2 to 18
years, the nurse should place priority on discussing the use of which measure?

You Selected:

child restraints in automobiles

Correct response:

child restraints in automobiles

Explanation:

Question 5 See full question

When planning home care for a 3-year-old child with eczema, what should the nurse teach the mother to remove
from the child's environment at home?

You Selected:

stuffed animals

Correct response:

stuffed animals
Explanation: Question 1 See full question

A nurse is teaching the parents of a child with cystic fibrosis about proper nutrition. Which instruction should the
nurse include?

You Selected:

Encourage a high-calorie, high-protein diet.

Correct response:

Encourage a high-calorie, high-protein diet.

Explanation:

Question 2 See full question

A client is to receive a glycerin suppository. Which nursing action is appropriate when administering a suppository?

You Selected:

Removing the suppository from the refrigerator 30 minutes before insertion

Correct response:

Applying a lubricant to the suppository

Explanation:

Question 3 See full question

The mother of an 11-month-old infant reports to the nurse that her infant sleeps much less than other children. The
mother asks the nurse whether her infant is getting sufficient sleep. What should be the nurse's initial response?

You Selected:

Reassure the mother that each infant's sleep needs are individual.

Correct response:

Ask the mother for more information about the infant's sleep patterns.

Explanation:

Question 4 See full question

During chemotherapy, a boy, age 10, loses his appetite. When teaching the parents about his food intake, the nurse
should include which instruction?

You Selected:

"Offer dry toast and crackers."

Correct response:

"Let your child eat any food he wants."

Explanation:

Question 5 See full question

A client is in the eighth month of pregnancy. To enhance cardiac output and renal function, the nurse should advise
her to use which body position?

You Selected:

Left lateral

Correct response:

Left lateral
Explanation:

Question 6 See full question

As a client progresses through pregnancy, she develops constipation. What is the primary cause of this problem
during pregnancy?

You Selected:

Decreased appetite

Correct response:

Reduced intestinal motility

Explanation:

Question 7 See full question

During a prenatal visit, a pregnant client with cardiac disease and slight functional limitations reports increased
fatigue. To help combat this problem, the nurse should advise her to:

You Selected:

exercise 1 hour before each meal.

Correct response:

divide daily food intake into five or six meals.

Explanation:

Question 8 See full question

A nurse is reviewing a client's fluid intake and output record. Fluid intake and urine output should relate in which
way?

You Selected:

Fluid intake should be inversely proportional to the urine output.

Correct response:

Fluid intake should be about equal to the urine output.

Explanation:

Question 9 See full question

After instruction of a primigravid client at 8 weeks' gestation about measures to overcome early morning nausea
and vomiting, which client statement indicates the need for additional teaching?

You Selected:

"I will snack on a small amount of carbohydrates throughout the day."

Correct response:

"I will eat two large meals daily with frequent protein snacks."

Explanation:

Question 10 See full question

A primigravid client at 32 weeks' gestation is enrolled in a breast-feeding class. Which statements indicate that the
client understands the breast-feeding education? Select all that apply.

You Selected:

"If I breast-feed, my uterus will return to prepregnancy size more quickly."


Correct response:

"I can hold my baby several different ways during feedings."


"If I breast-feed, my uterus will return to prepregnancy size more quickly."
"I need to feed my baby when I see feeding cues and not wait until she is crying."

Explanation:

Question 11 See full question

A primigravid client at 36 weeks' gestation tells the nurse that she has been experiencing insomnia for the past 2
weeks. Which suggestion would be most helpful?

You Selected:

Exercise for 30 minutes just before bedtime.

Correct response:

Practice relaxation techniques before bedtime.

Explanation:

Question 12 See full question

A client is recovering from an infected abdominal wound. Which foods should the nurse encourage the client to eat
to support wound healing and recovery from the infection?

You Selected:

gelatin salad and tea

Correct response:

chicken and orange slices

Explanation:

Question 13 See full question

Which client would benefit from the application of warm moist heat?

You Selected:

a client with a recently sprained joint

Correct response:

a client with low back pain

Explanation:

Question 14 See full question

When assessing pain in a client from Mexico, the nurse should understand the implications of which statement from
the client about the pain experience?

You Selected:

I have got to see a health care provider right away.

Correct response:

"Enduring pain is a part of God's will."

Explanation:

Question 15 See full question

A nurse is planning care for a 12-year-old with rheumatic fever. The nurse should teach the parents to:
You Selected:

provide for adequate periods of rest between activities.

Correct response:

provide for adequate periods of rest between activities.

Explanation:

Question 16 See full question

The breastfeeding mother of a 1-month-old diagnosed with cow's milk sensitivity asks the nurse what she should do
about feeding her infant. Which recommendation would be most appropriate?

You Selected:

Change to a soy-based formula exclusively, and begin solid foods.

Correct response:

Continue to breastfeed, but eliminate all milk products from your own diet.

Explanation:

Question 17 See full question

An adolescent is on the football team and practices in the morning and afternoon before school starts for the year.
The temperature on the field has been high. The school nurse has been called to the practice field because the
adolescent is now reporting that he has muscle cramps, nausea, and dizziness. Which action should the school
nurse do first?

You Selected:

Administer cold water with ice cubes.

Correct response:

Move the adolescent to a cool environment.

Explanation:

Question 18 See full question

A preschooler with a fractured femur of the left leg in traction tells the nurse that his leg hurts. It is too early for
pain medication. The nurse should:

You Selected:

remove the weight from the left leg.

Correct response:

assess the feet for signs of neurovascular impairment.

Explanation:

Question 19 See full question

The nurse is preparing to administer a preoperative medication that includes a sedative to a client who is having
abdominal surgery. The nurse should first:

You Selected:

have the client empty the bladder.

Correct response:

have the client empty the bladder.


Explanation:

Question 20 See full question

After teaching the client with rheumatoid arthritis about measures to conserve energy in activities of daily living
involving the small joints, which activity observed by the nurse indicates the need for additional teaching?

You Selected:

carrying a laundry basket with clinched fingers and fists

Correct response:

carrying a laundry basket with clinched fingers and fists

Explanation:

Question 21 See full question

As a first step in teaching a woman with a spinal cord injury and quadriplegia about her sexual health, the nurse
assesses her understanding of her current sexual functioning. Which statement by the client indicates she
understands her current ability?

You Selected:

"I cannot have sexual intercourse because it causes hypertension, but other sexual activity is okay."

Correct response:

"I can participate in sexual activity but might not experience orgasm."

Explanation:

Question 22 See full question

A nurse is completing the health history for a client who has been taking echinacea for a head cold. The client asks,
"Why is this not helping me feel better?" Which response by the nurse would be the most accurate?

You Selected:

"Combining herbal products with prescription antiviral medications is sure to help you."

Correct response:

"There is limited information as to the effectiveness of herbal products."

Explanation:

Question 23 See full question

A client has just returned from the postanesthesia care unit after undergoing a laryngectomy. Which intervention
should the nurse include in the plan of care?

You Selected:

Maintain the head of the bed at 30 to 40 degrees.

Correct response:

Maintain the head of the bed at 30 to 40 degrees.

Explanation:

Question 24 See full question

The nurse is planning to teach a client with chronic obstructive pulmonary disease how to cough effectively. Which
instruction should be included?

You Selected:

Assume a side-lying position, extend the arm over the head, and alternate deep breathing with coughing.
Correct response:

Take a deep abdominal breath, bend forward, and cough three or four times on exhalation.

Explanation:

Question 25 See full question

The client who had a permanent pacemaker implanted 2 days earlier is being discharged from the hospital. The
nurse knows that the client understands the discharge plan when the client:

You Selected:

explains signs and symptoms of myocardial infarction (MI).

Correct response:

verbalizes safety precautions needed to prevent pacemaker malfunction.

Explanation:

Question 26 See full question

The nurse is preparing the client with heart failure to go home. The nurse should instruct the client to:

You Selected:

monitor urine output daily.

Correct response:

monitor weight daily.

Explanation:

Question 27 See full question

When helping the client who has had a cerebrovascular accident (CVA) learn self-care skills, the nurse should:

You Selected:

advise the client to ask for help when dressing.

Correct response:

teach the client to put on clothing on the affected side first.

Explanation:

Question 28 See full question

For a client with osteoporosis, the nurse should provide which dietary instruction?

You Selected:

"Decrease your intake of red meat."

Correct response:

"Eat more dairy products to increase your calcium intake."

Explanation:

Question 29 See full question

A client in the surgical intensive care unit has skeletal tongs in place to stabilize a cervical fracture. Protocol
dictates that pin care should be performed each shift. When providing pin care for the client, which finding should
the nurse report to the physician?
You Selected:

A small amount of yellow drainage at the left pin insertion site

Correct response:

A small amount of yellow drainage at the left pin insertion site

Explanation:

Question 30 See full question

After a radical prostatectomy for prostate cancer, a client has an indwelling catheter removed. The client then
begins to have periods of incontinence. During the postoperative period, which intervention should be implemented
first?

You Selected:

Fluid restriction

Correct response:

Kegel exercises

Explanation:

Question 31 See full question

A female client reports to a nurse that she experiences a loss of urine when she jogs. The nurse's assessment
reveals no nocturia, burning, discomfort when voiding, or urine leakage before reaching the bathroom. The nurse
explains to the client that this type of problem is called:

You Selected:

stress incontinence.

Correct response:

stress incontinence.

Explanation:

Question 32 See full question

A primiparous client is on a regular diet 24 hours postpartum. She is from Guatemala and speaks only Spanish. The
clients mother asks the nurse if she can bring her daughter some special foods from home. The nurse responds,
based on the understanding about which principle?

You Selected:

The mother can bring the daughter any foods that she desires.

Correct response:

The mother can bring the daughter any foods that she desires.

Explanation:

Question 33 See full question

Which intervention should the nurse suggest to a parent to relieve itching in a child with chicken pox?

You Selected:

cool compresses moistened with a weak salt solution

Correct response:

oatmeal preparation baths


Explanation:

Question 34 See full question

The nurse observes a client with a history of panic attacks is hyperventilating. The nurse should:

You Selected:

tell the client to take several deep, slow breaths and exhale normally.

Correct response:

have the client breathe into a paper bag.

Explanation:

Question 35 See full question

A client with diabetes is explaining to the nurse how he cares for the feet at home. Which statement indicates the
client needs further instruction on how to care for the feet properly?

You Selected:

I am not allowed to use a heating pad on my feet.

Correct response:

I inspect my feet once a week for cuts and redness.

Explanation:

Question 36 See full question

The nurse is irrigating a clients colostomy. The client has abdominal cramping after receiving about 100 mL of the
irrigating solution. The nurse should first:

You Selected:

massage the abdomen gently.

Correct response:

stop the flow of solution.

Explanation:

Question 37 See full question

Which assessment is most appropriate for determining the correct placement of an endotracheal tube in a
mechanically ventilated client?

You Selected:

verifying the amount of cuff inflation

Correct response:

auscultating breath sounds bilaterally

Explanation:

Question 38 See full question

The nurse should assess the client with hypothyroidism for:

You Selected:

corneal abrasion due to inability to close the eyelid.


Correct response:

decreased activity due to fatigue.

Explanation:

Question 39 See full question

A nurse is assessing an immobile client and notes an area of sacral skin is reddened, but not broken. The reddened
area continues to blanch and refill with fingertip pressure. The most appropriate nursing action at this time is to:

You Selected:

apply a moist-to-moist dressing, being careful to pack just the wound bed.

Correct response:

reposition the client off the reddened skin and reassess in a few hours.

Explanation:

Question 40 See full question

Which skin preparation would be best to apply around the client's colostomy?

You Selected:

antiseptic cream

Correct response:

adhesive skin barrier

Explanation:

Question 41 See full question

Which goal is a priority for the diabetic client who is taking insulin and has nausea and vomiting from a viral illness
or influenza?

You Selected:

managing own health

Correct response:

obtaining adequate food intake

Explanation:

Question 42 See full question

The nurse is completing an intake and output record for a client who is receiving continuous bladder irrigation after
transurethral resection of the prostate. How many milliliters of urine should the nurse record as output for her shift
if the client received 1,800 ml of normal saline irrigating solution and the output in the urine drainage bag is 2,400
ml? Record your answer using a whole number.

Your Response:

Correct response:

600

Explanation:

Question 43 See full question

A client returned from surgery eight hours ago and has not voided. Which action should the nurse take first?
You Selected:

Catheterize the client with a straight catheter.

Correct response:

Palpate over the synthesis pubis for fullness.

Explanation:

Question 44 See full question

Because of symptoms experienced after a cerebrovascular accident (CVA), the nurse discovers that a client needs
assistance using utensils while eating. What would the nurse do to support this activity of care?

You Selected:

Encourage participation in the feeding process to the best of the client's abilities.

Correct response:

Encourage participation in the feeding process to the best of the client's abilities.

Explanation:

Question 45 See full question

Which of the following observations by the nurse would indicate that a client is unable to tolerate a continuation of
a tube feeding?

You Selected:

Intermittent epigastric tenderness

Correct response:

Formula in the clients mouth during the feeding, and increased cough

Explanation:

Question 46 See full question

A 7-year-old has had an appendectomy on November 12. He has had pain for the last 24 hours. There is a
prescription to administer acetaminophen with codeine every 3 to 4 hours as needed. The nurse is beginning the
shift, and the child is requesting pain medication. The nurse reviews the chart below for pain history. Based on the
information in the medical record, what should the nurse do next?

You Selected:

Administer the acetaminophen with codeine.

Correct response:

Administer the acetaminophen with codeine.

Explanation:

Question 47 See full question

The nurse is caring for a client in labor. The client wishes to have a nonmedicated labor and birth. During the
early stages of labor, the client becomes frustrated with the use of music and imagery. Which of the following would
the nurse include in the clients plan of care? Select all that apply.

You Selected:

Offer the use of a yoga ball

Correct response:

Encourage ambulation
Suggest a shower or bath
Offer the use of a yoga ball

Explanation:

Question 48 See full question

The nurse is giving prenatal instructions to a 32-year-old primagravida. Which nutritional instructions would the
nurse review? Select all that apply.

You Selected:

Intake of all minerals, especially iron, would be increased.

Correct response:

Caloric intake would be increased by 300 cal/day.


Protein intake would be increased to more than 30 g/day.
Folic acid intake would be increased to 800 mg/day.
Intake of all minerals, especially iron, would be increased.

Explanation:

Question 49 See full question

A client with iron deficiency anemia is having trouble selecting food from the hospital menu. Which foods should the
nurse suggest to meet the clients need for iron? Select all that apply.

You Selected:

tea

Correct response:

eggs
brown rice
dark green vegetables

Explanation:

Question 1 See full question

When planning pain control for a client with terminal gastric cancer, a nurse
should consider that:
You Selected:

clients with terminal cancer may develop tolerance to opioids.


Correct response:

clients with terminal cancer may develop tolerance to opioids.


Explanation:

Question 2 See full question

The client with a lumbar laminectomy asks to be turned onto the side. The
nurse should:
You Selected:

get another nurse to help logroll the client into position.


Correct response:

get another nurse to help logroll the client into position.


Explanation:

Question 3 See full question

The nurse is teaching a client with multiple sclerosis about prevention of


urinary tract infection (UTI) and renal calculi. Which of the following nutrition
recommendations by the nurse would be the most likely to reduce the risk of
these conditions?
You Selected:

Increase fluids (2500 mL/day) and maintain urine acidity by drinking cranberry
juice.
Correct response:

Increase fluids (2500 mL/day) and maintain urine acidity by drinking cranberry
juice.
Explanation:

Question 4 See full question

The nurse is recording the intake and output for a client with the following:
D5NSS 1,000 ml; urine 450 ml; emesis 125 ml; Jackson Pratt drain #1 35 ml;
Jackson Pratt drain #2 32 ml; and Jackson Pratt drain #3 12 ml. How many
milliliters would the nurse document as the clients output? Record your
answer using a whole number.
Your Response:

654
Correct response:

654
Explanation:

Question 5 See full question

A client with iron deficiency anemia is having trouble selecting food from the
hospital menu. Which foods should the nurse suggest to meet the clients
need for iron? Select all that apply.
You Selected:

dark green vegetables


eggs
brown rice
Correct response:

eggs
brown rice
dark green vegetables
Explanation:

Question 50 See full question

The nurse is teaching the mother of a preschool-aged child with celiac disease about a gluten-free diet. The nurse
determines that the mother understands the diet if she tells the nurse she will prepare:

You Selected:

wheat toast and grape jelly

Correct response:

eggs and orange juice

Explanation:

Question 1 See full question

When planning pain control for a client with terminal gastric cancer, a nurse should consider that:

You Selected:

only low doses of opioids are safe; higher doses may cause respiratory depression.

Correct response:

clients with terminal cancer may develop tolerance to opioids.

Explanation:

Question 2 See full question

A client states, "I have abdominal pain." Which assessment question would best determine the client's need for pain
medication?

You Selected:

"What does the pain feel like?"

Correct response:

"What does the pain feel like?"

Explanation:

Question 3 See full question

A client complains of severe abdominal pain. To elicit as much information as possible about the pain, the nurse
should ask:
You Selected:

"Is the pain stabbing like a knife?"

Correct response:

"Can you describe the pain?"

Explanation:

Question 4 See full question

A client with schizophrenia is mute, can't perform activities of daily living, and stares out the window for hours.
What is the nurse's first priority?

You Selected:

Assist the client with showering.

Correct response:

Assist the client with feeding.

Explanation:

Question 5 See full question

Which instruction should a nurse give to a client who's 26 weeks pregnant and complains of constipation?

You Selected:

Encourage her to increase her intake of roughage and to drink at least six glasses of water per day.

Correct response:

Encourage her to increase her intake of roughage and to drink at least six glasses of water per day.

Explanation:

Question 6 See full question

An elderly client with Alzheimer's disease begins supplemental tube feedings through a gastrostomy tube to
provide adequate calorie intake. The nurse's priority should be the potential for:

You Selected:

hyperglycemia.

Correct response:

aspiration.

Explanation:

Question 7 See full questionA nurse is caring for a client with an endotracheal tube who receives enteral
feedings through a feeding tube. Before each tube feeding, the nurse checks for tube placement in the stomach as
well as residual volume. The purpose of the nurse's actions is to prevent:

You Selected:

diarrhea.

Correct response:

aspiration.
Explanation:

Question 8 See full question

A nurse is reviewing a client's fluid intake and output record. Fluid intake and urine output should relate in which
way?

You Selected:

Fluid intake should be inversely proportional to the urine output.

Correct response:

Fluid intake should be about equal to the urine output.

Explanation:

Question 9 See full question

A primigravid client at 36 weeks' gestation tells the nurse that she has been experiencing insomnia for the past 2
weeks. Which suggestion would be most helpful?

You Selected:

Drink a small glass of wine with dinner.

Correct response:

Practice relaxation techniques before bedtime.

Explanation:

Question 10 See full question

Before discharge from the hospital after a myocardial infarction, a client is taught to exercise by gradually
increasing the distance walked. Which vital sign should the nurse teach the client to monitor to determine whether
to increase or decrease the exercise level?

You Selected:

blood pressure

Correct response:

pulse rate

Explanation:

Question 11 See full question

The nurse is developing a plan of care for a client who has joint stiffness due to rheumatoid arthritis. Which
measure will be the most effective in relieving stiffness?

You Selected:

cold compresses to joints for 30 minutes to relieve stiffness

Correct response:

a warm shower before performing activities of daily living

Explanation:

Question 12 See full question

Which client would benefit from the application of warm moist heat?

You Selected:

a client with a suspected malignancy


Correct response:

a client with low back pain

Explanation:

Question 13 See full question

When assessing pain in a client from Mexico, the nurse should understand the implications of which statement from
the client about the pain experience?

You Selected:

"I cannot go on in pain like this any longer."

Correct response:

"Enduring pain is a part of God's will."

Explanation:

Question 14 See full question

A nurse is planning care for a 12-year-old with rheumatic fever. The nurse should teach the parents to:

You Selected:

observe the child closely.

Correct response:

provide for adequate periods of rest between activities.

Explanation:

Question 15 See full question

The breastfeeding mother of a 1-month-old diagnosed with cow's milk sensitivity asks the nurse what she should do
about feeding her infant. Which recommendation would be most appropriate?

You Selected:

Change to a soy-based formula exclusively, and begin solid foods.

Correct response:

Continue to breastfeed, but eliminate all milk products from your own diet.

Explanation:

Question 16 See full question

A preschooler with a fractured femur of the left leg in traction tells the nurse that his leg hurts. It is too early for
pain medication. The nurse should:

You Selected:

assess the feet for signs of neurovascular impairment.

Correct response:

assess the feet for signs of neurovascular impairment.

Explanation:

Question 17 See full question

After teaching the client with rheumatoid arthritis about measures to conserve energy in activities of daily living
involving the small joints, which activity observed by the nurse indicates the need for additional teaching?
You Selected:

pushing with palms when rising from a chair

Correct response:

carrying a laundry basket with clinched fingers and fists

Explanation:

Question 18 See full question

The client with a spinal cord injury asks the nurse why the dietitian has recommended to decrease the total daily
intake of calcium. Which response by the nurse would provide the most accurate information?

You Selected:

"Excessive intake of dairy products makes constipation more common."

Correct response:

"Lack of weight bearing causes demineralization of the long bones."

Explanation:

Question 19 See full question

As a first step in teaching a woman with a spinal cord injury and quadriplegia about her sexual health, the nurse
assesses her understanding of her current sexual functioning. Which statement by the client indicates she
understands her current ability?

You Selected:

"I will not be able to have sexual intercourse until the urinary catheter is removed."

Correct response:

"I can participate in sexual activity but might not experience orgasm."

Explanation:

Question 20 See full question

A nurse is caring for a client who recently had a bowel resection. The client has a hemoglobin level of 8 g/dl and
HCT of 30%. Dextrose 5% in half-normal saline solution (D 5NS) is infusing through a triple-lumen central catheter
at 125 ml/hour. The healthcare provider s orders include:

gentamicin 80 mg intravenous piggyback in 50 ml D5W over 30 minutes


ranitidine (Zantac) 50 mg intravenous in 50 ml D5W piggyback over 30 minutes
one unit of 250 ml of packed red blood cells (RBCs) over 3 hours
nasogastric tube flushes with 30 ml of normal saline solution every 2 hours

How many milliliters would the nurse document as the total intake for the 8-hour shift? Record your answer as a
whole number.

Your Response:

Correct response:

1470

Explanation:

Question 21 See full question

The parents of an ill child are concerned because the child is not eating well. Which strategies are appropriate to
encourage the child to eat? Select all that apply.
You Selected:

Let the child substitute items on the tray for other nutritious foods.

Correct response:

Allow the child to choose meals from an acceptable list of foods.


Let the child substitute items on the tray for other nutritious foods.
Ask the child to say why he or she is not eating.

Explanation:

Question 22 See full question

The nurse is evaluating the outcome of therapy for a client with osteoarthritis. Which outcome indicates the goals of
therapy have been met?

You Selected:

feels better than on hospital admission

Correct response:

joint range of motion improved

Explanation:

Question 23 See full question

The nurse is preparing the client with heart failure to go home. The nurse should instruct the client to:

You Selected:

monitor urine output daily.

Correct response:

monitor weight daily.

Explanation:

Question 24 See full question

During the first few weeks after a cholecystectomy, the client should follow a diet that includes:

You Selected:

a limited intake of fat distributed throughout the day so there is not an excessive amount in the intestine at
any one time.

Correct response:

a limited intake of fat distributed throughout the day so there is not an excessive amount in the intestine at
any one time.

Explanation:

Question 25 See full question

Immediately after nasogastric (NG) tube removal the nurse should:

You Selected:

provide orange sherbet.

Correct response:

provide the client with mouth care.


Explanation:

Question 26 See full question

The nurse gives a pamphlet that describes Kegel exercises to a client with stress incontinence. Which statement
indicates that the client has understood the instructions contained in the pamphlet?

You Selected:

I can do these exercises sitting up, lying down, or standing.

Correct response:

I can do these exercises sitting up, lying down, or standing.

Explanation:

Question 27 See full question

Because of religious beliefs, a client, who is an Orthodox Jew, refuses to eat hospital food. Hospital policy
discourages food from outside the hospital. The nurse should next:

You Selected:

encourage the clients family to bring food for the client because of the special circumstances.

Correct response:

discuss the situation and possible courses of action with the dietitian and the client.

Explanation:

Question 28 See full question

A nurse is teaching an elderly client about developing good bowel habits. Which statement by the client indicates to
the nurse that additional teaching is required?

You Selected:

"I will eat raw, green-leafy vegetables, unpeeled fruit, and whole grain bread."

Correct response:

"I need to use laxatives regularly to prevent constipation."

Explanation:

Question 29 See full question

A nurse asks a client who had abdominal surgery 3 days ago if he has moved his bowels since surgery. The client
states, "I haven't moved my bowels, but I am passing gas." How should the nurse intervene?

You Selected:

Encourage the client to ambulate at least three times per day.

Correct response:

Encourage the client to ambulate at least three times per day.

Explanation:

Question 30 See full question

A client with right sided hemiparesis has limited mobility. Which action should the nurse include in the plan of care
to help maintain skin integrity?

You Selected:

Encourage fluid intake.


Correct response:

Turn him regularly.

Explanation:

Question 31 See full question

A client is ordered continuous bladder irrigation at a rate of 60 gtt/minute. The nurse hangs a 2 L bag of sterile
solution with tubing on a three-legged I.V. pole. She then attaches the tubing to the client's three-way urinary
catheter, adjusts the flow rate, and leaves the room. Which important procedural step did the nurse fail to follow?

You Selected:

Attaching the infusion set to an infusion pump

Correct response:

Evaluating patency of the drainage lumen

Explanation:

Question 32 See full question

A postpartum woman who gave birth vaginally has unrelenting rectal pain despite the administration of pain
medication. Which action is most indicated?

You Selected:

reassuring the client that such pain is normal after vaginal birth

Correct response:

assessing the perineum

Explanation:

Question 33 See full question

A client is recovering from a gastric resection for peptic ulcer disease. Which outcome indicates that the goal of
adequate nutritional intake is being achieved 3 weeks following surgery? The client:

You Selected:

drinks 2,000 mL/day of water.

Correct response:

increases food intake and tolerance gradually.

Explanation:

Question 34 See full question

The client with a lumbar laminectomy asks to be turned onto the side. The nurse should:

You Selected:

ask the client to help by using an overhead trapeze to turn.

Correct response:

get another nurse to help logroll the client into position.

Explanation: Question 1 See full question

An adolescent is diagnosed with iron deficiency anemia. After emphasizing


the importance of consuming dietary iron, the nurse asks him to select iron-
rich breakfast items from a sample menu. Which selection demonstrates
knowledge of dietary iron sources?
You Selected:

Ham and eggs


Correct response:

Ham and eggs


Explanation:

Question 2 See full question

A nurse is reviewing a client's fluid intake and output record. Fluid intake and
urine output should relate in which way?
You Selected:

Fluid intake should be about equal to the urine output.


Correct response:

Fluid intake should be about equal to the urine output.


Explanation:

Question 3 See full question

In providing discharge teaching for the client after a modified radical


mastectomy, the nurse should instruct the client to avoid:
You Selected:

taking a late-evening swim.


Correct response:

working in her rose garden.


Explanation:

Question 4 See full question

A nurse is teaching an elderly client about developing good bowel habits.


Which statement by the client indicates to the nurse that additional teaching
is required?
You Selected:

"I need to use laxatives regularly to prevent constipation."


Correct response:

"I need to use laxatives regularly to prevent constipation."


Explanation:

Question 5 See full question

While making a home visit to a multigravida 2 weeks after the birth of viable
twins at 38 weeks gestation, the nurse observes that the client looks pale,
has dark circles around her eyes, and is breastfeeding one of the twins. The
clients apartment is clean, and nothing appears out of place. The client tells
the nurse that she completed three loads of laundry this morning. A priority
need for this client is:
You Selected:

fatigue related to home maintenance and caring for twins.


Correct response:

fatigue related to home maintenance and caring for twins.


Explanation:

Question 1 See full question

As a client progresses through pregnancy, she develops constipation. What is


the primary cause of this problem during pregnancy?
You Selected:

Reduced intestinal motility


Correct response:

Reduced intestinal motility


Explanation:

Question 2 See full question

Which instruction should a nurse give to a client who's 26 weeks pregnant


and complains of constipation?
You Selected:

Encourage her to increase her intake of roughage and to drink at least six
glasses of water per day.
Correct response:

Encourage her to increase her intake of roughage and to drink at least six
glasses of water per day.
Explanation:

Question 3 See full question

Because of religious beliefs, a client, who is an Orthodox Jew, refuses to eat


hospital food. Hospital policy discourages food from outside the hospital. The
nurse should next:
You Selected:

discuss the situation and possible courses of action with the dietitian and the
client.
Correct response:

discuss the situation and possible courses of action with the dietitian and the
client.
Explanation:

Question 4 See full question

Which is an appropriate nursing goal for the client who has ulcerative colitis?
The client:
You Selected:

verbalizes the importance of small, frequent feedings.


Correct response:

verbalizes the importance of small, frequent feedings.


Explanation:

Question 5 See full question

The client with a lumbar laminectomy asks to be turned onto the side. The
nurse should:
You Selected:

get another nurse to help logroll the client into position.


Correct response:

get another nurse to help logroll the client into position.


Explanation:

1 The admitting nurse is assessing a client Dyspnea and hypoxemia


. with COPD. The nurse auscultates Correct
diminished breath sounds, which signify
changes in the airway. These changes These changes in the airway require that
indicate to the nurse to monitor the the nurse monitor the patient for
patient for what? dyspnea and hypoxemia. Clubbing of the
fingers is not a sign of COPD. Cyanosis is
a) Dyspnea and hypoxemia a sign of hypoxemia.

b) Bronchospasm and clubbing of the


fingers

c) Clubbing of the fingers and cyanosis

d) Hypoxemia and clubbing of the


fingers
2 After reviewing the pharmacological Increase oxygen distribution
. treatment for pulmonary diseases, the Reduce airway obstruction
nursing student knows that Alter smooth muscle tone
bronchodilators relieve bronchospasm in
three ways. Choose the correct three of Bronchodilators relieve bronchospasm by
the following options. altering smooth muscle tone and reduce
airway obstruction by allowing increased
a) Increase oxygen distribution oxygen distribution throughout the lungs
and improving alveolar ventilation.
b) Reduce airway obstruction

c) Alter smooth muscle tone

d) Decrease alveolar ventilation


3 At 11 p.m., a client is admitted to the albuterol (Proventil).
. emergency department. He has a
respiratory rate of 44 breaths/minute. The client is hypoxemic because of
He's anxious, and wheezes are audible. bronchoconstriction as evidenced by
The client is immediately given oxygen wheezes and a subnormal arterial oxygen
by face mask and methylprednisolone saturation level. The client's greatest need
(Depo-medrol) I.V. At 11:30 p.m., the is bronchodilation, which can be
client's arterial blood oxygen accomplished by administering
saturation is 86%, and he's still bronchodilators. Albuterol is a beta2
wheezing. The nurse should plan to adrenergic agonist, which causes dilation
administer: of the bronchioles. It's given by
nebulization or metered-dose inhalation
a) albuterol (Proventil). and may be given as often as every 30 to
60 minutes until relief is accomplished.
b) propranolol (Inderal). Alprazolam is an anxiolytic and central
nervous system depressant, which could
c) morphine. suppress the client's breathing. Propranolol
is contraindicated in a client who's
d) alprazolam (Xanax). wheezing because it's a beta2 adrenergic
antagonist. Morphine is a respiratory
center depressant and is contraindicated in
this situation.
4 A client being seen in the emergency Oxygen therapy through a non-rebreather
. department has labored respirations. mask
Auscultation reveals inspiratory and
expiratory wheezes. Oxygen saturation The description is consistent with status
is 86%. The client was nonresponsive to asthmaticus. The client has not responded
an albuterol (Ventolin) inhaler and to treatment. Oxygen saturation is low. As
intravenous methylprednisolone (Solu- oxygenation is the priority per Maslow's
Medrol). The nurse administers the hierarchy of needs, oxygen therapy would
following prescribed treatment first: be supplied first. Then, the nurse would
initiate intravenous fluids and magnesium
a) Oxygen therapy through a non- sulfate. Last, the nurse would encourage
rebreather mask the client to drink fluids to prevent
dehydration
b) Normal saline 0.9% at 100 mL/hr
intravenously

c) Intravenous magnesium sulfate

d) Oral fluid of at least 2500 mL/day


5 A client has a history of chronic Lung sounds
. obstructive pulmonary disease (COPD).
Following a coughing episode, the A client with COPD is at risk for developing
client reports sudden and unrelieved pneumothorax. The description given is
shortness of breath. Which of the consistent with possible pneumothorax.
following is the most important for the Though the nurse will assess all the data,
nurse to assess? auscultating the lung sounds will provide
the nurse with the information if the client
a) Skin color has a pneumothorax.

b) Heart rate

c) Respiratory rate

d) Lung sounds
6 A client has intermittent Inhaled albuterol (Ventolin)
. asthma attacks. Which of the
following therapies does the For intermittent asthma, the preferred treatment is
nurse teach the client to use with an inhaled short-acting beta2-agonist. The other
at home when experiencing treatments are for persistent asthma.
an asthma attack?

a) Oxygen therapy

b) Inhaled cromolyn sodium


(Intal)

c) Oral theophylline (Theo-


Dur)

d) Inhaled albuterol
(Ventolin)
7 A client is admitted to a Impaired gas exchange related to airflow obstruction
. health care facility for
treatment of chronic A patent airway and an adequate breathing pattern
obstructive pulmonary are the top priority for any client, making Impaired gas
disease. Which nursing exchange related to airflow obstruction the most
diagnosis is most important important nursing diagnosis. Although Activity
for this client? intolerance, Anxiety, and Risk for infection may also
apply to this client, they aren't as important as
a) Anxiety related to actual Impaired gas exchange.
threat to health status

b) Impaired gas exchange


related to airflow obstruction

c) Risk for infection related


to retained secretions

d) Activity intolerance
related to fatigue
8 A client is being seen in the emergency Oxygen through nasal cannula at 2
. department for exacerbation of chronic L/minute
obstructive pulmonary disease (COPD).
The first action of the nurse is to All options listed are treatments that
administer which of the following may be used for a client with an
prescribed treatments? exacerbation of COPD. The first line of
treatment is oxygen therapy.
a) Vancomycin 1 gram intravenously over
1 hour

b) Oxygen through nasal cannula at 2


L/minute

c) Intravenous methylprednisolone (Solu-


Medrol) 120 mg

d) Ipratropium bromide (Alupent) by


metered-dose inhaler
9 A client is being sent home with oxygen Smoking or a flame is dangerous near
. therapy. The nurse instructs that oxygen.
a) The client should raise the flow of The nurse should cautions the client
oxygen if shortness of breath increases. against smoking or using a flame near
oxygen. Oxygen is not addictive. Clients
b) The client will not be able to travel can travel with portable oxygen
with oxygen. systems. Teaching also includes the
proper flow of oxygen.
c) Oxygen is addictive and its use must be
decreased.

d) Smoking or a flame is dangerous near


oxygen.
1 A client is diagnosed with a Anxiety
0. chronic respiratory disorder.
After assessing the client's In a client with a respiratory disorder, anxiety
knowledge of the disorder, the worsens such problems as dyspnea and
nurse prepares a teaching bronchospasm. Therefore, Anxiety is a likely nursing
plan. This teaching plan is diagnosis. This client may have inadequate
most likely to include which nutrition, making Imbalanced nutrition: More than
nursing diagnosis? body requirements an unlikely nursing diagnosis.
Impaired swallowing may occur in a client with an
a) Anxiety acute respiratory disorder, such as upper airway
obstruction, but not in one with a chronic respiratory
b) Unilateral neglect disorder. Unilateral neglect may be an appropriate
nursing diagnosis when neurologic illness or trauma
c) Impaired swallowing causes a lack of awareness of a body part; however,
this diagnosis doesn't occur in a chronic respiratory
d) Imbalanced nutrition: More disorder.
than body requirements
1 A client with chronic Weigh yourself daily and report a gain of 2 lb in 1 day."
1. obstructive pulmonary
disease (COPD) and cor The nurse should instruct the client to weigh himself
pulmonale is being daily and report a gain of 2 lb in 1 day. COPD causes
prepared for discharge. pulmonary hypertension, leading to right-sided heart
The nurse should provide failure or cor pulmonale. The resultant venous congestion
which instruction? causes dependent edema. A weight gain may further
stress the respiratory system and worsen the client's
a) "Limit yourself to condition. The nurse should also instruct the client to eat
smoking only 2 cigarettes a low-sodium diet to avoid fluid retention and engage in
per day." moderate exercise to avoid muscle atrophy. The client
shouldn't smoke at all.
b) "Weigh yourself daily
and report a gain of 2 lb
in 1 day."

c) "Maintain bed rest."


d) "Eat a high-sodium
diet."
1 A client with chronic instruct the client to drink at least 2 L of fluid daily.
2. obstructive pulmonary
disease (COPD) is admitted Mobilizing secretions is crucial to maintaining a patent
to the medical-surgical unit. airway and maximizing gas exchange in the client
To help this client maintain with COPD. Measures that help mobilize secretions
a patent airway and achieve include drinking 2 L of fluid daily, practicing controlled
maximal gas exchange, the pursed-lip breathing, and engaging in moderate
nurse should: activity. Anxiolytics rarely are recommended for the
client with COPD because they may cause sedation
a) maintain the client on and subsequent infection from inadequate
bed rest. mobilization of secretions. Because COPD rarely
causes pain, pain medication isn't indicated
b) instruct the client to
drink at least 2 L of fluid
daily.

c) administer anxiolytics, as
ordered, to control anxiety.

d) administer pain
medication as ordered.
1 A client with chronic atelectasis.
3. obstructive pulmonary
disease (COPD) is recovering In a client with COPD, an ineffective cough impedes
from a myocardial infarction. secretion removal. This, in turn, causes mucus
Because the client is plugging, which leads to localized airway obstruction
extremely weak and can't a known cause of atelectasis. An ineffective cough
produce an effective cough, doesn't cause pleural effusion (fluid accumulation in
the nurse should monitor the pleural space). Pulmonary edema usually results
closely for: from left-sided heart failure, not an ineffective cough.
Although many noncardiac conditions may cause
a) oxygen toxicity. pulmonary edema, an ineffective cough isn't one of
them. Oxygen toxicity results from prolonged
b) atelectasis. administration of high oxygen concentrations, not an
ineffective cough.
c) pleural effusion.

d) pulmonary edema.
1 A client with chronic obstructive "Delay self-care activities for 1 hour."
4. pulmonary disease (COPD)
reports increased shortness of Some clients with COPD have shortness of
breath and fatigue for 1 hour breath and fatigue in the morning on arising as a
after awakening in the morning. result of bronchial secretions. Planning self-care
Which of the following activities around this time may be better
statements by the nurse would tolerated by the client, such as delaying
best help with the client's activities until the client is less short of breath or
shortness of breath and fatigue? fatigued. The client raising the arms over the
head may increase dyspnea and fatigue. Sitting
a) "Drink fluids upon arising from in a chair when bathing or dressing will aid in
bed." dyspnea and fatigue but does not address the
situation upon arising. Drinking fluids will assist
b) "Raise your arms over your in liquifying secretions which, thus, will aid in
head." breathing, but again does not address the
situation in the morning.
c) "Sit in a chair whenever doing
an activity."

d) "Delay self-care activities for 1


hour."
1 The clinic nurse is caring for a The most important risk factor for COPD is
5. patient who has just been cigarette smoking."
diagnosed with chronic obstructive
pulmonary disease (COPD). The The most important risk factor for COPD is
patient asks the nurse what he cigarette smoking. Nutrition, exercise, and
could have done to minimize the exposure to dust and pollen are not risk
risk of contracting this disease. factors for COPD.
What would be the nurse's best
answer?

a) "The most important risk factor


for COPD is exposure to dust and
pollen."

b) "The most important risk factor


for COPD is inadequate nutrition."

c) "The most important risk factor


for COPD is regular exercise."

d) "The most important risk factor


for COPD is cigarette smoking."
1 For a client with chronic obstructive Teaching the client how to perform controlled
6. pulmonary disease, which nursing coughing
intervention helps maintain a patent
airway? Controlled coughing helps maintain a patent
airway by helping to mobilize and remove
a) Teaching the client how to secretions. A moderate fluid intake (usually 2
perform controlled coughing L or more daily) and moderate activity help
liquefy and mobilize secretions. Bed rest and
b) Restricting fluid intake to 1,000 sedatives may limit the client's ability to
ml/day maintain a patent airway, causing a high risk
of infection from pooled secretions
c) Enforcing absolute bed rest

d) Administering ordered sedatives


regularly and in large amounts
1 In chronic obstructive Respiratory acidosis
7. pulmonary disease (COPD),
decreased carbon dioxide Increased carbon dioxide tension in arterial blood
elimination results in increased leads to respiratory acidosis and chronic
carbon dioxide tension in respiratory failure. In acute illness, worsening
arterial blood, leading to which hypercapnia can lead to acute respiratory failure.
of the following acid-base The other acid-base imbalances would not
imbalances? correlate with COPD.

a) Respiratory acidosis

b) Metabolic acidosis

c) Metabolic alkalosis

d) Respiratory alkalosis
1 A nurse administers albuterol Respiratory rate of 22 breaths/minute
8. (Proventil), as ordered, to a
client with emphysema. Which In a client with emphysema, albuterol is used as a
finding indicates that the drug bronchodilator. A respiratory rate of 22
is producing a therapeutic breaths/minute indicates that the drug has
effect? achieved its therapeutic effect because fewer
respirations are required to achieve oxygenation.
a) Urine output of 40 ml/hour Albuterol has no effect on pupil reaction or urine
output. It may cause a change in the heart rate,
b) Heart rate of 100 but this is an adverse, not therapeutic, effect.
beats/minute

c) Respiratory rate of 22
breaths/minute

d) Dilated and reactive pupils


=
1 The nurse at the beginning An 86 year old with COPD who arrived on the floor 30
9. of the evening shift receives minutes ago and is a direct admit from the doctor's
a report at 1900 on the office
following patients. Which
patient would the nurse On the patient's arrival at the emergency department,
assess first? the first line of treatment is supplemental oxygen
therapy and rapid assessment to determine if the
a) A 74 year old with chronic exacerbation is life-threatening. Pulse oximetry is
bronchitis who has BP helpful in assessing response to therapy but does not
128/58, HR 104, and R 26 assess PaCO2 levels. The fluids will not run out during
the very beginning of the shift. The vital signs listed
b) A 62 year old with are normal findings for patients with COPD
emphysema who has 300 mL
of intravenous fluid
remaining

c) An 86 year old with COPD


who arrived on the floor 30
minutes ago and is a direct
admit from the doctor's
office

d) An 85 year old with COPD


with wheezing and an O2
saturation of 89% on 2 L of
oxygen
2 A nurse consulting with a High-protein
0. nutrition specialist knows
it's important to consider a Breathing is more difficult for clients with COPD, and
special diet for a client with increased metabolic demand puts them at risk for
chronic obstructive nutritional deficiencies. These clients must have a
pulmonary disease (COPD). high intake of protein for increased calorie
Which diet is appropriate for consumption. Full liquids, 1,800-calorie ADA, and low-
this client? fat diets aren't appropriate for a client with COPD

a) Low-fat

b) Full-liquid

c) High-protein

d) 1,800-calorie ADA
2 A nurse has established a Has wheezes in the right lung lobes
1. nursing diagnosis of
ineffective airway Of the data listed, wheezing, an adventitious lung
clearance. The datum that sound, is the best datum that supports the diagnosis of
best supports this ineffective airway clearance. An increased respiratory
diagnosis is that the client rate and a report of dyspnea are also defining
characteristics of this nursing diagnosis. They could
a) Has wheezes in the support other nursing diagnoses, as would inability to
right lung lobes perform activities of daily living.
b) Reports shortness of
breath

c) Cannot perform
activities of daily living

d) Has a respiratory rate


of 28 breaths/minute
2 The nurse has instructed Exhales hard and fast with a single blow
2. the client to use a peak
flow meter. The nurse To use a peak flow meter, the client stands. Then the
evaluates client learning client takes a deep breath and exhales hard and fast
as satisfactory when the with a single blow. The client repeats this twice and
client records a "personal best" in an asthma diary

a) Sits in a straight-back
chair and leans forward

b) Inhales deeply and


holds the breath

c) Records in a diary the


number achieved after one
breath

d) Exhales hard and fast


with a single blow
2 A nurse is assisting with a subclavian diminished or absent breath sounds on
3. vein central line insertion when the the affected side.
client's oxygen saturation drops
rapidly. He complains of shortness of In the case of a pneumothorax,
breath and becomes tachypneic. The auscultating for breath sounds will reveal
nurse suspects the client has absent or diminished breath sounds on
developed a pneumothorax. Further the affected side. Paradoxical chest wall
assessment findings supporting the movements occur in flail chest conditions.
presence of a pneumothorax include: Tracheal deviation occurs in a tension
pneumothorax. Muffled or distant heart
a) paradoxical chest wall movement sounds occur in cardiac tamponade.
with respirations.

b) tracheal deviation to the unaffected


side.

c) diminished or absent breath sounds


on the affected side.
d) muffled or distant heart sounds.
2 A nurse is caring for a client the airways are so swollen that no air can
4. experiencing an acute asthma attack. get through.
The client stops wheezing and breath
sounds aren't audible. This change During an acute asthma attack, wheezing
occurred because: may stop and breath sounds become
inaudible because the airways are so
a) the swelling has decreased. swollen that air can't get through. If the
attack is over and swelling has decreased,
b) the airways are so swollen that no there would be no more wheezing and
air can get through. less emergent concern. Crackles don't
replace wheezes during an acute asthma
c) the attack is over. attack.

d) crackles have replaced wheezes.


2 The nurse is caring for a Use diaphragmatic breathing
5. client with COPD. It is time to
do discharge teaching with Inspiratory muscle training and breathing retraining
this client. The nurse teaches may help improve breathing patterns. Training in
the client about breathing diaphragmatic breathing reduces the respiratory
exercises. What should the rate, increases alveolar ventilation, and, sometimes,
nurse include in the helps expel as much air as possible during
teaching? expiration. Pursed-lip breathing helps slow
expiration, prevents collapse of small airways, and
a) Use diaphragmatic controls the rate and depth of respiration. It also
breathing promotes relaxation, which allows patients to gain
control of dyspnea and reduce feelings of panic.
b) Exhale through an open Diaphragmatic breathing, not chest breathing,
mouth increases lung expansion

c) Make inhalation longer


than exhalation

d) Use chest breathing


2 A nurse is developing a teaching Take ordered medications as scheduled.
6. plan for a client with asthma.
Which teaching point has the Although avoiding contact with fur-bearing
highest priority? animals, changing filters on heating and air
conditioning units frequently, and avoiding
a) Change filters on heating and air goose down pillows are all appropriate
conditioning units frequently. measures for clients with asthma, taking
ordered medications on time is the most
b) Avoid contact with fur-bearing important measure in preventing asthma
animals. attacks.

c) Take ordered medications as


scheduled.

d) Avoid goose down pillows.


2 The nurse should be alert for a Atelectasis
7. complication of bronchiectasis that
results from a combination of Retention of secretions and subsequent
retained secretions and obstruction ultimately cause the aveoli distal
obstruction that leads to the to the obstruction to collapse (atelectasis).
collapse of alveoli. This
complication is known as

a) Atelectasis

b) Pleurisy

c) Emphysema

d) Pneumonia
2 The nursing instructor is teaching A barrel chest
8. a class of level I nursing students
how to do a physical assessment In COPD patients with a primary
on a patient with lung disease and emphysematous component, chronic
chronic hyperinflation of the lungs. hyperinflation leads to the barrel chest thorax
What would a nurse most likely configuration. The nurse most likely would not
assess in this type of patient? assess dry, flaky skin; large, drooping eyes, or
long, thin fingers.
a) Large, drooping eyes

b) Long, thin fingers

c) Dry, flaky skin

d) A barrel chest
2 Nursing management of the Avoid going outdoors if the pollen count is high
9. client with COPD involves a
great deal of teaching. Which It is important to caution the patient to avoid going
self-care activity would it be outdoors if the pollen count is high or if there is
important for you to teach a significant air pollution, because of the risk of
client with COPD to do to bronchospasm. Encouraging the patient to get
avoid bronchospasm? immunizations is an action to prevent infection.
Monitoring the patient for signs and symptoms of
a) Avoid going outdoors if the respiratory infection is an action the nurse does, not
pollen count is high something the nurse teaches the client to do.
Assessing the client is not teaching a self-care
b) Assess the client for technique.
familiarity with potential side
effects of prescribed
medications

c) Encourage the patient to


be immunized against
influenza and S. pneumoniae

d) Monitor signs and


symptoms of respiratory
infection
3 A nursing student is taking a Inflamed airways that obstruct airflow
0. pathophysiology examination. Mucus secretions that block airways
Which of the following factors Overinflated alveoli that impair gas exchange
would the student correctly
identify as contributing to the Because of chronic inflammation and the body's
underlying pathophysiology of attempts to repair it, changes and narrowing
chronic obstructive pulmonary occur in the airways. In the proximal airways,
disease (COPD)? Choose all that changes include increased numbers of goblet
apply. cells and enlarged submucosal glands, both of
which lead to hypersecretion of mucus. In the
a) Inflamed airways that obstruct peripheral airways, inflammation causes
airflow thickening of the airway wall, peribronchial
fibrosis, exudate in the airway, and overall
b) Decreased numbers of goblet airway narrowing.
cells

c) Mucus secretions that block


airways

d) Overinflated alveoli that impair


gas exchange

e) Dry airways that obstruct


airflow
3 The nursing student recalls that Mucus secretions block airways.
1. the underlying pathophysiology Overinflated alveoli impair gas exchange.
of chronic obstructive pulmonary Inflamed airways obstruct airflow.
disease (COPD) includes the
following components: (Select all Because of the chronic inflammation and the
that apply.) body's attempts to repair it, changes and
narrowing occur in the airways. In the peripheral
a) Mucus secretions block airways, inflammation causes thickening of the
airways. airway wall, peribronchial fibrosis, exudate in
the airway, and overall airway narrowing
b) Dry airways obstruct airflow. (obstructive bronchiolitis). The airways are
actually moist, not dry. In the proximal airways,
c) Overinflated alveoli impair gas changes include increased goblet cells and
exchange. enlarged submucosal glands, both of which lead
to hypersecretion of mucus.
d) Inflamed airways obstruct
airflow.
3 A physician orders a beta2 Albuterol
2. adrenergic-agonist agent
(bronchodilator) that is short- Short-acting beta2-adrenergic agonists include
acting and administered only by albuterol, levalbuterol, and pirbuterol. They are
inhaler. The nurse knows this the medications of choice for relief of acute
would probably be symptoms and prevention of exercise-induced
asthma. They are used to relax smooth muscle
a) Isuprel

b) Foradil

c) Atrovent

d) Albuterol
3 A student nurse prepares to care Bronchodilators
3. for a patient with bronchiectasis.
The student nurse should Bronchodilators, which may be prescribed for
anticipate that what would be patients who also have reactive airway disease,
ordered for this patient? may also assist with secretion management.
Antihypertensives, potassium supplements, and
a) Diuretics diuretics would not be routinely administered to
patients with bronchiectasis.
b) Bronchodilators

c) Potassium supplements

d) Antihypertensives
3 When caring for a client with Cognitive changes
4. COPD, the nurse knows it is
important to monitor what?
The nurse monitors for cognitive changes
a) Cognitive changes (personality and behavioral changes, memory
impairment), increasing dyspnea, tachypnea, and
b) Bradycardia tachycardia, which may indicate increasing
hypoxemia and impending respiratory failure.
c) Increasing hyperpnea

d) Support systems
3 When developing a Smoking cessation
5. preventative plan of care for a
patient at risk for developing The most important risk factor for the development
chronic obstructive pulmonary of COPD is cigarette smoking. The effects of
disease (COPD), which of the cigarette smoke are complex and lead to the
following should be development of COPD in approximately 15% to 20%
incorporated? of smokers. Tobacco smoke irritates the airways
and, in susceptible individuals, results in mucus
a) Smoking cessation hypersecretion and airway inflammation.

b) Weight reduction

c) Cancer prevention

d) Cholesterol management
3 Which of the following would Hypoventilation
6. be a potential cause of
respiratory acidosis? Respiratory acidosis is always due to inadequate
excretion of CO, with inadequate ventilation,
a) Diarrhea resulting in elevated plasma CO concentration,
which causes increased levels of carbonic acid. In
b) Vomiting addition to an elevated PaCO, hypoventilation
usually causes a decrease in PaO
c) Hyperventilation

d) Hypoventilation
3 Which vaccine should a nurse Influenza
7. encourage a client with
chronic obstructive pulmonary Clients with COPD are more susceptible to
disease (COPD) to receive? respiratory infections, so they should be
encouraged to receive the influenza and
a) Hepatitis B pneumococcal vaccines. Clients with COPD aren't at
high risk for varicella or hepatitis B. The HPV
b) Human papilloma virus vaccine is to guard against cervical cancer and is
(HPV) recommended only for women ages 9 to 26.

c) Influenza

d) Varicella
3 You are a pediatric nurse Pets
8. practitioner caring for a child who
has just been diagnosed with Common causative agents that may trigger an
asthma. You provide the parents asthma attack are as follows: dust, dust mites,
with information that includes pets, soap, certain foods, molds, and pollens.
potential causative agents for an Watching television, hot showers, and
asthmatic reaction. What would rainstorms are not triggers for asthma attacks.
this information include as a
potential causative agent for an
asthma attack?
a) Pets

b) Rainstorm

c) Hot shower

d) Watching television
3 You are developing the teaching Chronic inhalation of indoor toxins causes lung
9. portion of a care plan for a patient damage
with COPD. What would be an
important component for you to Other environmental risk factors for COPD
emphasize? include prolonged and intense exposure to
occupational dusts and chemicals, indoor air
a) ADLs should be completed in pollution, and outdoor air pollution. Smoking
the waking hours cessation should be taught to all patients who
are currently smoking. Minor respiratory
b) Chronic inhalation of indoor infections that are of no consequence to the
toxins causes lung damage person with normal lungs can produce fatal
disturbances in the lungs of the person with
c) Minor respiratory infections are emphysema. Activities of daily living (ADLs)
not treated should be paced throughout day to permit
patients to perform these without excessive
d) Smoking one-half of a pack distress.
weekly is allowable

Question 35 See full question

The nurse is teaching the client who is receiving chemotherapy and the family how to manage possible nausea and
vomiting at home. The nurse should include information about:

You Selected:

eating three normal meals a day.

Correct response:

eating frequent, small meals throughout the day.

Explanation:

Question 36 See full question

A nurse is assessing an immobile client and notes an area of sacral skin is reddened, but not broken. The reddened
area continues to blanch and refill with fingertip pressure. The most appropriate nursing action at this time is to:

You Selected:

complete and document a Braden skin breakdown risk score for the client.

Correct response:

reposition the client off the reddened skin and reassess in a few hours.
Explanation:

Question 37 See full question

A typically developing preschool child is experiencing pain after an appendectomy. Which data collection tool is
the most appropriate for the nurse use to assess the pain?

You Selected:

FACES Pain Rating Scale

Correct response:

FACES Pain Rating Scale

Explanation:

Question 38 See full question

A client with rheumatoid arthritis has increasing fatigue and is unable to manage all of the usual activities. The
nurse should:

You Selected:

administer opioids to promote pain relief and rest.

Correct response:

encourage the client to alternate periods of rest and activity throughout the day.

Explanation:

Question 39 See full question

The client with hepatitis A is experiencing fatigue, weakness, and a general feeling of malaise. The client tires
rapidly during morning care. The most appropriate goal for this client is to:

You Selected:

gradually increase activity tolerance.

Correct response:

gradually increase activity tolerance.

Explanation: Question 1 See full question

A nurse is teaching the parents of a child with cystic fibrosis about proper
nutrition. Which instruction should the nurse include?
You Selected:

Encourage a high-calorie, high-protein diet.


Correct response:

Encourage a high-calorie, high-protein diet.


Explanation:

Question 2 See full question

A nurse is providing teaching to a postpartum client who has decided to


breast-feed her neonate. She has questions regarding her nutritional intake
and wants to know how many extra calories she should eat. What number of
additional calories should the nurse instruct the client to eat per day? Record
your answer using a whole number.
Your Response:

6
Correct response:

500
Explanation:

Question 3 See full question

The client who is in Bucks traction is constipated. A plan of care that


incorporates which breakfast would be most helpful in reestablishing a
normal bowel routine?
You Selected:

an orange, raisin bran and milk, and wheat toast with butter
Correct response:

an orange, raisin bran and milk, and wheat toast with butter
Explanation:

Question 4 See full question

A nurse consulting with a nutrition specialist knows it's important to consider


a special diet for a client with chronic obstructive pulmonary disease (COPD).
Which diet is appropriate for this client?
You Selected:

High-protein
Correct response:

High-protein
Explanation:

Question 5 See full question

The client with hepatitis A is experiencing fatigue, weakness, and a general


feeling of malaise. The client tires rapidly during morning care.
The most appropriate goal for this client is to:
You Selected:

gradually increase activity tolerance.


Correct response:

gradually increase activity tolerance.


Explanation:

Improve your mastery

1. +3 edema The nurse is explaining how to assess


edema to the nursing students working
on the antepartum unit. Which score
indicates edema of lower extremities,
face, hands, and sacral area?
2. A 3 yr old boy was seen in the clinic by importance of taking all of the prescribed
the pediatrician and diagnosed with amoxicillin
pneumonia. Amoxicillin for 10 days was
prescribed, with a followup visit in 2
weeks. What need to be taught to the
patient
3. 4.2-5.4 RBC norm
4. 5,000 to 10,000 WBC norm
5. 6.5-lb infant after a 2-hour labor. A postpartum client would be at increased
risk for postpartum hemorrhage if she
delivered a:
6. 10-13 lbs how much of the 25-35 total pounds
gained in first 20 weeks
7. 12-16 g/dL normal hemoglobin
8. An 18-year-old pregnant woman tells 4. should be referred to community
the nurse that she's concerned that she resources available for pregnant women
may not be able to take care of herself
during her pregnancy. She states that
prenatal care is expensive and her job
doesn't provide insurance. The nurse
should recognize that the client:
9. 20-36 weeks what is considered pre-term birth
1 A 20-year-old female's pregnancy is "2. telling her such feelings are normal in
0. confirmed at a clinic. She says her the beginning of pregnancy.
husband will be excited, but she is
concerned because she herself isn't
excited. She fears this may mean she'll
be a bad mother. The nurse should
respond by:
1 24-28 weeks when do you check mother glucose for
1. gestational diabetes
1 A 30-year-old primiparous client at 34 3. ""These streaks are called striae
2. weeks' gestation comes to the prenatal gravidarum, or stretch marks; they'll grow
facility concerned about the reddish lighter after delivery.""
streaks she has increasingly developed
on her breasts and abdomen. She asks
what these skin changes are and
whether they're permanent. What
should the nurse tell her?
1 37-47% normal hematocrit
3.

1 37 weeks what is considered a term birth


4.

1 A 48-year-old whose father died of When reviewing patient laboratory


5. metastatic prostate cancer results, the nurse in the clinic notes
elevated prostate specific antigen (PSA)
levels in the following four patients.
Which patient's PSA result is most
important to report to the health care
provider?
1 A 52-year-old client states, "My ANS: B
6. husband is upset because I don't enjoy The nurse should assess the client's last
sex as much as I used to." Which menstrual cycle to determine if the client
priority client data should a nurse is experiencing the onset of menopause.
initially collect? Menopause usually occurs around the age
of 50. The decrease in estrogen can result
A. History of hysterectomy in multiple symptoms including a
B. Date of last menstrual cycle decrease in biological drives and sexual
C. Use of birth control methods activity.
D. History of thought disorder
1 A 66-year-old man who has a painful The nurse working in a health clinic
7. erection that has lasted over 7 hours receives calls from all these patients.
Which patient should be seen by the
health care provider first?
1 100.8 F on the second and third Which temperature indicates the
8. postpartum days presence of postpartum infection?
1 120-160 bpm normal fetal heart rate beginning at 8
9. weeks with doppler
2 150,000 to 400,000 platelet counts
0.

2 500 mL in the first 24 hours after vaginal Early postpartum hemorrhage is defined
1. delivery. as a blood loss greater than:
2 Abdominal palpation Which intrapartal assessment should be
2. avoided when caring for the woman with
hemolysis, elevated liver enzymes, and
low platelet count (HELLP) syndrome?
2 aborted if pregnancy end in spontaneous or
3. therapeutic __ prior to 20 weeks
2 ABORTION Elective
4. Spontaneous-vaginal bleeding
w/passage of clots and tissue thru
vagina.
Low uterine cramping and contractions
Hemmorrhage and shock
Interventions: Bedrest, vital signs,
monitor cramping and bleeding,
count perineal pads to evaluate blood
loss (save expelled tissues and clots);
maintain IV fluids, monitor for shock;
prepare client for D & C; Rh immune
globulin given to appropriate Rh-
women.
2 about one pound per week how much weight gain per week after
5. week 20
2 abruptio placentae, placentae previa, what are three possible causes of
6. bloody show vaginal bleeding during pregnancy
2 Absence of deep tendon reflexes Which assessment finding would
7. convince the nurse to "hold" the next
dose of magnesium sulfate?
2 acceptance, vitals, weight, nutrition, what do you assess for during
8. glucose or protein in urine collaborative management during
pregnancy
2 "A client admitted with preterm labor is " 3. Respiratory rate less than 12
9. prescribed magnesium sulfate to halt breaths/minute 4. Extreme muscle
contractions. The nurse should monitor weakness Palpitations 6. Hot flashes"
the client for which adverse reactions to
the drug?
3 "A client is scheduled for amniocentesis. " 1. Ask the client to void. 4. Assess fetal
0. What should the nurse do to prepare the heart rate. 6. Monitor maternal vital
client for the procedure? signs. "
3 "A client who is 14 weeks pregnant "1. Progesterone
1. mentions that she has been having
difficulty moving her bowels since she
became pregnant. Which hormones are
responsible for this common discomfort
during pregnancy?
3 additional 300 calories a day a healthy pregnant woman will need an
2. additional - calories per day
3 Adequate prenatal care What is most helpful in preventing
3. premature birth?
3 administer calcium gluconate. A woman taking magnesium sulfate has
4. respiratory rate of 10 breaths/min. In
addition to discontinuing the
medication, the nurse should:
3 An adolescent in her first trimester of 1. Review the hazards of smoking with
5. pregnancy continues to smoke the client and suggest a smoking-
cigarettes. The client tells the nurse that cessation class.
she'd like to quit but she doesn't want to
gain any more weight. What should the
nurse do for this client?
3 adolescent mother these mothers need increased caloric
6. intake for both the meternal and fetal
growth
3 An adolescent who's 14 weeks pregnant comes to the . "Have you and your
7. clinic for a prenatal examination. During the physician discussed your
examination, the client says to the nurse, "I'm still options?"
not sure whether I want to keep my baby." Which
response by the nurse is best?
3 AFP alpha-fetoprotein test measured at 16-18
8. weeks gestation if serum
increased indicates
neural tube defects.
3 After an amniotomy, which client goal should take the 2. The client will maintain
9. highest priority? adequate fetal tissue
perfusion
4 After delivering an 8 lb (3.6 kg) girl, a client asks the sterile water.
0. nurse what her daughter should receive for the first
feeding. For a bottle-fed neonate, the first feeding
usually consists of:
4 After determining that a pregnant client is Rh- 4. To detect maternal
1. negative, the physician orders an indirect Coombs' antibodies against fetal
test. What is the purpose of performing this test in a Rh-positive factor
pregnant client?
4 After developing severe hydramnios, a primigravid "2. Amniocentesis to
2. client exhibits dyspnea, along with edema of the legs temporarily relieve
and vulva. Which procedure should the nurse expect discomfort
her to undergo and why?
4 After receiving large doses of an ovulatory stimulant 2. demonstrating signs of
3. such as menotropins (Pergonal), a client comes in for hyperstimulation
her office visit. Data collection reveals the following syndrome.
findings: Weight gain of 6 lb (3 kg), ascites, and pedal
edema. These findings indicate that the client is:
4 After undergoing prenatal blood testing, a client 4. Follow facility policy for
4. learns that she tested positive for human documenting and
immunodeficiency virus (HIV). What should the nurse communicating HIV
do with this information? status
4 AIDS HIV causitive factor for AIDS
5. A.. Description
AZT (Zidovudine) for prevention of maternal-fetal HIV transmission;
administered orally after 14 wks gestation, IV during labor, Syrup to
neonate after birth for 6 weeks.
B. Transmission - genital secretions from infected person; perinatal
exposure of infant to infected maternal secretion thru birth process or
breast-feeding
C. Mother managed as high risk - vulnerable to infections
D. Diagnosis
1. Tests to determine - ELISA (enzyme-linked immunosorbent assay);
WB (Western blot) and IFA (indirect fluorescent antibody)
2. Two ELISA test w/same blood sample; if reactive follow-up test of WB
or IFA
3. + WB or IFA - confirmatory for HIV
4. + ELISA that fails to be confirmed by WB or IFA not considered
negative; repeat in 3 to 6 months.
E. Interventions
1. Prenatal period-Prevent opportunistic infections; avoid
amniocentesis and fetal scalp sampling (perinatal transmission)
2. Intrapartum period-a. If fetus not exposed to HIV in utero - highest
risk during DELIVERY thru birth canal. b. avoid use of scalp electrodes
c. avoid episiotomy (maternal blood) d. avoid admin of oxytocin
(vaginal tears-blood) e. Place heavy absorbent pads under mother's
hips-absorb amniotic fluid and maternal blood f. promptly remove
neonate from mother's blood after delivery g. Suction infant promptly.
h. Admin AZT IV to mother during labor and delivery
3. Postpartum period-a. Monitor for sings of infection b. Mother in
protective isolation if immunosuppressed c. Restrict breast-feeding D.
Instruct mother to monitor for signs of infection, report any sign if they
occur
4 Alpha- 1. Assesss quantity of fetal serum proteins; if AFP level is elevated, it is
6. fetoprotein associated w/open neural tube and abdominal wall defects.
(AFP) 2. Can detect spina bifida and Down syndrome
screening 3. Interventions
a. Explain AFP level is determined by a single maternal blood sample
drawn at 15-18 weeks' gestation.
b. If level is elevated an gestation is less than 18 wks, 2nd sample
drawn
c. Ultrasonography performed if AFP level is elevated to rule out fetal
abnormalities and multiple gestations.
4 AMA (against You notice that your patient has his call light on. As you enter the room,
7. medical the patient is fully dressed and ready to leave. He begins yelling at you
advice) to take his IV out because he is going home. When a patient leaves the
hospital without a physician's order, it is considered to be leaving:
4 Amniocentesi 1. Aspiration of amniotic fluid done at 16 weeks or thereafter.
8. s 2. May reveal genetic disorders, metabolic defects, lung maturity , and
sex
3. Risks
a. Maternal hemorrhage
b. Infection
c. Rh isoimmunization
d. Abruptio placentai
e. Amniotic fluid emboli
f. Premature rupture of membranes
4. Interventions
a. Obtain informed consent
b. If less than 20 wks, woman must have full bladder to support uterus;
if more than 20 wks, woman must have empty bladder to minimize
chance of puncture.
c. Prepare client for ultrasonography, which is performed to locate
placenta and avoid puncture.
d. Obtain baseline vital signs and fetal heart rate; monitor q 15 min.
e. Position client supine during exam and on left side after
f. Instruct client that if chills, fever, leakage of fluid at needle-insertion
site, decreased fetal movement, uterine contractions, or cramping
occur, she is to notify doc.
4 anemia, exhaustion, failure to The visiting nurse must be aware that women who
9. attach to her infant, have had a postpartum hemorrhage are subject to
postpartum infection. a variety of complications after discharge from the
hospital. These include:
5 anticonvulsant. A woman with preeclampsia is being treated with
0. bed rest and intravenous magnesium sulfate. The
drug classification of this medication is:
5 Anxiety can be decreased in explaining all procedures before they are done
1. both the family and the child
who has cancer by
5 Any activity could increase the What activity guidelines should be included when
2. risk of recurrence of labor teaching a client about home care for preterm
contractions. labor?
5 An appropriate-for-gestational- between the 10th and the 99th percentiles for age.
3. age neonate should weigh:
5 An appropriate intervention for increased fluids
4. a child with bronchiolitis is
5 An appropriate nursing action avoid examination of the pharynx
5. when a child is suspected of
having epiglottistis is to
5 An appropriate nursing monitor the childs response to analgesics
6. intervention for the child
admitted to the hospital in
sickle cell crisis would be to
5 Approximately how much time 2. 7 days
7. is required for the blastocyst
to reach the uterus for
implantation?
5 "Are you using any recreational A 22-year-old man tells the nurse at the health
8. drugs or drinking a lot of clinic that he has recently had some problems with
alcohol?" erectile dysfunction. When assessing for possible
etiologic factors, which question should the nurse
ask first?
5 arise slowly, avoid standing for manage faintness during late prego
9. long and check H&H
6 As a client progresses through 4. Reduced intestinal motility
0. pregnancy, she develops
constipation. What is the
primary cause of this problem
during pregnancy?
6 Ask the patient about any A 53-year-old man tells the nurse he has been
1. prescription drugs he is taking. having increasing problems with erectile
dysfunction (ED) for several years but is now
interested in using Viagra (sildenafil). Which action
should the nurse take first?
6 Ask the patient if he has any A 32-year-old man who is being admitted for a
2. questions or concerns about unilateral orchiectomy for testicular cancer does
the diagnosis and treatment. not talk to his wife and speaks to the nurse only to
answer the admission questions. Which action is
best for the nurse to take?
6 As part of the respiratory neonates are obligate nose breathers.
3. assessment, the nurse
observes the neonate's nares
for patency and mucus. The
information obtained from this
assessment is important
because:
6 assess fetal heart rate (FHR) The priority nursing intervention when admitting a
4. and maternal vital signs. pregnant woman who has experienced a bleeding
episode is to:
6 Assess the fundus for firmness. A woman delivered a 9-lb, 10-oz baby 1 hour ago.
5. When you arrive to perform her 15-minute
assessment, she tells you that she "feels all wet
underneath." You discover that both pads are
completely saturated and that she is lying in a 6-
inch-diameter puddle of blood. What is your first
action?
6 assess weight gain, location of A 17-year-old primigravida has gained 4 lb since
6. edema, and urine for protein. her last prenatal visit. Her blood pressure is 140/92
mm Hg. The most important nursing action is to:
6 Assist the client in performing Which nursing measure would be appropriate to
7. leg exercises every 2 hours. prevent thrombophlebitis in the recovery period
after a cesarean birth?
6 Assist with the plan of care and recommend As an LPN, you know that
8. revisions as necessary. your role in the nursing
process when a patient is
admitted is to:
6 At 15 weeks' gestation, a client is scheduled for a "1. Family history of spina
9. serum alpha-fetoprotein (AFP) test. Which bifida in a sister
maternal history finding would indicate a need for
this test?
7 At 32 weeks' gestation, a client is admitted to the 1. edema.
0. facility with a diagnosis of pregnancy-induced
hypertension (PIH). Based on this diagnosis, the
nurse expects assessment to reveal:
7 At birth, a neonate weighs 7 lb, 3 oz. When Risk for injury related to
1. assessing the neonate 1 day later, the nurse hyperbilirubinemia
obtains a weight of 7 lb and an axillary
temperature of 98 F (36.7 C) and notes that the
sclerae are slightly yellow. The neonate has been
breast-feeding once every 2 to 3 hours. Based on
these findings, the nurse should expect which
nursing diagnosis to be added to the plan of care?
7 At what age are the following appropriate 4 years
2. milestones: balance on one foot for 5 seconds,
walk heel to toe, catch a ball, throw a ball
overhand, skip and hop on one foot, use scissors,
lace shoes, copy a square, and add three parts to
a stick figure?
7 At what age are the following appropriate 3 years
3. milestones: have a vocabulary of about 900 words
and can use complete sentences of 3-4 words,
asks many questions, and begins to sing songs?
7 At what age are the following appropriate 5 years
4. milestones: have a vocabulary of approximately
2,100 words and can use sentences with 6-8
words, asks the meaning of words, has many
questions, can name the days of the week and the
months of the year, enjoys group play with similar
or identical activities but without organization or
rules (associative play)?
7 At what age are the following appropriate 3 years
5. milestones: pedal a tricycle, jump in place, broad
jump, balance on one foot, walk up and down
steps using alternating feet, build a tower of 9-10
cubes, copy a circle, put facial features on a circle,
and feed and dress himself?
7 At what age are the following appropriate 5 years
6. milestones: skip and hop on alternate feet, throw
and catch a ball, jump rope, jump from a height of
12 inches, balance on alternate feet with eyes
closed, tie shoelaces, use scissors, begin to print a
few letters or numbers, copy a diamond and
triangle, and draw a stick figure with seven to
nine parts?
7 At what age are the following appropriate 4 years
7. milestones: understands time in relation to daily
events, prizes independence, takes pride in his
accomplishments, enjoys entertaining others,
shares family secrets with outsiders, and
commonly has an imaginary friend, is egocentric
(unable to envision situations from perspectives
other than his own)?
7 At what gestational age would a primigravida 3. 18 to 22 weeks
8. expect to feel quickening?
7 The average height increase for the preschooler is 2-3 inches; legs (rather than
9. _______ per year which is mostly due to an the trunk)
elongation of the __________.
8 The average weight gain for the preschooler is 5 lbs
0. ________ per year.
8 avoid constipation, apply topical anesthetics, how to manage hemorrhoids
1. ointments or ice paks, sitz bath or warm soak, during late prego
reinsert into rectum
8 avoid strong odors, avoid drinking while eating, to manage NV during early
2. avoid spicy or greasy foods, pregnancy - drink carbonated
beveraes, eat crackers or
toast before getting out of
bed, small frequent meals
but avoid -
8 B12 all vegetarian prego need
3. this vitamin
8 A baby born 2 hours ago has just arrived in the Drying him thoroughly after a
4. nursery. Which nursing measure will prevent the bath
neonate from losing heat due to evaporation?
85. bacterial endocarditis. Antiinfective prophylaxis is indicated for
the pregnant woman with a history of
mitral valve stenosis related to rheumatic
heart disease because the woman is at
risk of developing:
86. Bed rest What order should the nurse expect for a
client admitted with a threatened
abortion?
87. biparietal diameter and femur length how will the ultrasound help assess health
care provider of growth rate
88. Bladder irrigation prevents When teaching a patient who is scheduled
obstruction of the catheter after for a transurethral resection of the
surgery. prostate (TURP) about continuous bladder
irrigation, which information will the nurse
include?
89. bladder is full when is it best to have an abdominal
ultrasound
90. blood glucose of 25 mg/dL. In caring for the postterm infant,
thermoregulation can be a concern,
especially in the infant who also has a:
91. BP drops what does the BP do in the first and
second trimester
92. bradley birthing method husband to coach
relaxation through abdominal breathing
and exercise
93. By what age do children realize that 10
death is final and permanent
94. Call the admitting nurse at the facility Your patient is being transferred to a long-
and give a brief summary of the term care facility for rehabilitation. As the
patient's medical diagnosis, treatment nurse, what will you need to do in order to
care plan, and medications. provide continuity of care for this patient?
95. The caloric requirements for a client of 1. 300 kcal
normal weight increase by how much
during pregnancy?
96. Can a quadriplegic deliver vaginally? Yes!
97. Can a woman use tampons after birth NO!!!! bc they are at risk for toxic shock
for lochia? syndrome
98. Can you use the pediatric pads on an NO. however, you can use adult pads on a
adult for a defibrillator? child.
99. Cardiac output increases 30-40% what does the cardia output levels do with
an increased pulse of 10-15 bpm during
prego
10 cesarean delivery. Birth for the nulliparous woman with a
0. fetus in a breech presentation is usually
by:
10 chadwicks sign estrogen causes bluish color of cervix
1. during pregnancy
10 cheese or yogurt if lactose intolerated these two dairy
2. products may be better tolerated
10 A child appears apathetic and weak. kwashiorkor
3. His growth is below normal normal for
his age. There is a white streak in the
child's hair. The nurse recognizes
these signs as characteristic of
10 Children with failure to thrive fall 3rd
4. below the ___ percentile in weight and
height on growth charts
10 children with hemophilia should aviod salicylates
5.

10 Children with Hodgkin's disease painless cervical neck lump


6. usually present with
10 Children with intussusception may currant jelly stools
7. have bowel movements containing
blood and mucus and no feces
10 Children with sickle cell trait Will no develop the disease
8.

10 A childs arterial blood gas results are metabolic acidosis


9. pH 7.30, PaCO2-36,HCO3-21. The
nurse determines the child is
experiencing which acid-base
imbalance
11 chloasma, linea negra and striae what three integumentary systems are
0. gravidarum affected during prego
11 Chorionic villus sampling (CVS) 1. Physician aspirates
1. small sample of chorionic
villus tissue at 8 to 12
weeks
2. CVS performed to
detect genetic
abnormalities
3. Interventions
a. Obtain informed
consent
b. Instruct client to drink
water to fill bladder
before procedure to aid
in positioning uterus for
catheter insertion
c. Instruct client to report
bleeding, infection, or
leakage of fluid at
insertion site after
procedure.
d. Rh - woman may be
given Rho(D) immune
globulin (RhoGAM),
because CVS increases
risk of Rh sensitization.
11 A client, 2 months pregnant, has hyperemesis "4. The client will exhibit
2. gravidarum. Which expected outcome is most uterine growth within the
appropriate for her? expected norms for
gestational age.
11 A client, 7 months pregnant, is admitted to the unit 1. Place the client on her
3. with abdominal pain and bright red vaginal bleeding. left side and start
Which action should the nurse take? supplemental oxygen, as
ordered, to maximize
fetal oxygenation
11 A client, 7 months pregnant, is receiving the tocolytic 2. pulmonary edema.
4. agent terbutaline (Bricanyl), 17.5 mcg/minute I.V., to
halt uterine contractions. She also takes prednisone
(Orasone), 5 mg by mouth twice per day, to control
asthma. To detect an adverse interaction between
these drugs, the nurse should monitor the client for:
11 A client, 8 weeks pregnant, has a history of lactose 3. Add lactase
5. intolerance. To prevent a nutritional deficiency as a replacement drops to
result of lactose intolerance, the nurse teaches her milk at least 24 hours
about lactase replacement. Which teaching point is before drinking it.
appropriate?
11 A client, 11 weeks pregnant, is admitted to the 3. an unknown cause.
6. facility with hyperemesis gravidarum. She tells the
nurse she has never known anyone who had such
severe morning sickness. The nurse understands that
hyperemesis gravidarum results from:
11 A client, 30 weeks pregnant, is scheduled for a 3. The fetus isn't in
7. biophysical profile (BPP) to evaluate the health of her distress at this time.
fetus. Her BPP score is 8. What does this score
indicate?
11 A client asks how long she and her husband can "4. ""As long as you wish,
8. safely continue sexual activity during pregnancy. if the pregnancy is
How should the nurse respond? normal""
11 A client at 28 weeks' gestation is complaining of 4. promote fetal lung
9. contractions. Following admission and hydration, the maturity.
physician writes an order for the nurse to give 12 mg
of betamethasone I.M. This medication is given to:
12 A client calls to schedule a pregnancy test. The nurse "1. Human chorionic
0. knows that most pregnancy tests measure which gonadotropin (hCG)
hormone?
12 A client comes to her prenatal examination 2. Instructing the client
1. complaining of increasing episodes of leg cramping. to gently stretch her legs
Which intervention by the nurse is most appropriate? to relieve the cramping
12 A client comes to the clinic for a routine prenatal 2. Offer to assist the
2. examination. The nurse notices that the client is client with basic care
disheveled and unkempt. The client explains, "I have needs while waiting for
no energy to take care of myself and I really don't the physician.
care." How should the nurse intervene?
12 A client confides that she's estranged from the Discussing with the client
3. abusive father of her infant. Which nursing whether she'd like to use
intervention would ensure client confidentiality? another name while
hospitalized
12 A client gives birth to a neonate prematurely, at 28 respiration.
4. weeks' gestation. To obtain the neonate's Apgar
score, the nurse assesses the neonate's:
12 A client has come to the clinic for her first prenatal 4. ""Consult with your
5. visit. The nurse should include which statement health care provider before
about using drugs safely during pregnancy in her taking any medications.""
teaching?
12 A client has just expelled a hydatidiform mole. 2. ""I can see that you're
6. She's visibly upset over the loss and wants to know upset; however, you must
when she can try to become pregnant again. Which wait at least 1 year before
of the following would be the nurse's best becoming pregnant
response? again.""
12 A client in her 15th week of pregnancy has 2. Deficient fluid volume
7. presented with abdominal cramping and vaginal
bleeding for the past 8 hours. She has passed
several clots. What is the primary nursing diagnosis
for this client?
12 A client in her first trimester of pregnancy "1. ""Needs for rest and
8. complains that she's always tired. Which response sleep typically increase
by the nurse is best? during the first trimester of
pregnancy.""
12 A client in her second trimester of pregnancy has 4. "The monitor helps you
9. been diagnosed with gestational diabetes. While see how your blood sugar
instructing the client about home glucose is being controlled during
monitoring the client states, "Why should I use this your pregnancy to help
if the diabetes is going to resolve after I have my prevent complications."
baby anyway?" Which response by the nurse is
best?
13 A client in the 13th week of pregnancy develops 4. Ketones in urine
0. hyperemesis gravidarum. Which laboratory finding
indicates the need for intervention?
13 A client in the first trimester of pregnancy comes to 2. ""You're feeling
1. the facility for a routine prenatal visit. She tells the ambivalent, which is
nurse she doesn't know whether she's ready to normal during the first
have a baby, even though this was a planned trimester.""
pregnancy. Which response should the nurse offer?
13 A client in the first trimester of pregnancy joins a 3. Warning signs of
2. childbirth education class. During this trimester, the complications
class is most likely to cover which physiologic
aspect of pregnancy?
13 A client in transition complains to the nurse that ask the physician to step
3. the physician was verbally abusive and "rough out of the room and then
during a vaginal exam." Just then, the physician discuss with him the need
reappears and asks the nurse for a sterile glove for to transfer care to another
another vaginal check. The nurse's first priority physician.
should be to:
13 A client is 2 months pregnant. Which factor should 1. Support from her
4. the nurse anticipate as most likely to affect her partner
psychosocial transition during pregnancy?
13 A client is 8 weeks pregnant. Which teaching topic 2. Common discomforts of
5. is most appropriate at this time? pregnancy
13 A client is admitted for an amniocentesis. Initial 1. Administer RhoGAM.
6. data collection findings include the following: 16
weeks pregnant, vital signs within normal limits,
hemoglobin 12.2 gm, hematocrit 35%, and type O-
negative blood. Which action would be most
important to include in the client's plan of care
after the amniocentesis has been completed?
13 A client is admitted to the facility in preterm labor. "4. terbutaline (Brethine).
7. To halt her uterine contractions, the nurse expects
the physician to prescribe:
13 A client is admitted to the facility with a suspected 1. a history of pelvic
8. ectopic pregnancy. When reviewing the client's inflammatory disease.
health history for risk factors for this abnormal
condition, the nurse expects to find:
13 A client is at her ideal weight when she conceives. 4. ""You should gain 24 to
9. During a prenatal visit 2 months later, she asks the 32 lb.""
nurse how much weight she should gain during
pregnancy. What is the nurse's best response?
14 A client is concerned that her 2-day-old, breast- the neonate latches onto
0. feeding neonate isn't gaining weight. The nurse the areola and swallows
should teach the client that breast-feeding is audibly.
effective if:
14 A client is diagnosed with female sexual ANS: C
1. aversion disorder. In addition to systematic The nurse should identify that
desensitization techniques, which medication therapy of amoxapine
medication therapy could accompany this could complement systematic
intervention? desensitization techniques.
Amoxapine is a heterocyclic
A. Quetiapine (Seroquel) antidepressant that can assist in
B. Phenelzine (Nardil) reduction of anxiety.
C. Amoxapine (Asendin)
D. Carbamazepine (Tegretol)
14 A client is diagnosed with sexual aversion ANS: A
2. disorder. Which symptom of this disorder The nurse should recognize that
should the nurse correctly pair with an this sexual aversion disorder is
appropriate therapeutic intervention? characterized by an avoidance of
genital sexual contact. Sexual
A. Avoidance of all genital sexual contact aversion implies anxiety, fear, or
treated by systematic desensitization disgust in sexual situations. Sexual
B. Avoidance of all genital sexual contact aversion can be treated by
treated by medicating with tadalafil (Cialis) systematic desensitization.
C. Anorgasmia treated by vardenafil
(Levitra)
D. Anorgasmia treated by sensate focus
exercises
14 A client is expecting her second child in 6 "Each pregnancy has a unique
3. months. During the psychosocial psychosocial meaning."
assessment, she says, "I've been through
this before. Why are you asking me these
questions?" What is an appropriate response
by the nurse?
14 A client is in the 8th month of pregnancy. To "2. Left lateral
4. enhance cardiac output and renal function,
the nurse should advise her to use which
body position?
14 A client is in the 38th week of her first 1. Review premonitory signs of
5. pregnancy. She calls the prenatal facility to labor with the client.
report occasional tightening sensations in
the lower abdomen and pressure on the
bladder from the fetus, which she says
seems lower than usual. The nurse should
take which action?
14 A client is in the last trimester of pregnancy. "1. blurred vision.
6. The nurse should instruct her to notify her
obstetrician immediately if she notices:
14 A client is receiving terbutaline (Brethine) to "3. ""Assess the insertion site for
7. stop preterm labor. The physician plans to signs of infection.""
discharge the client in the morning with a
terbutaline infusion pump. Which instruction
should the nurse include when teaching the
client about the drug?
14 A client is scheduled for amniocentesis. 1. Ask her to void.
8. When preparing her for the procedure, the
nurse should complete which of the
following tasks?
14 A client just had twins. Twin "A" weighs Placing the twins in the same crib
9. 2,500 g (5 lb, 8 oz), and Twin "B" weighs only so the larger baby can keep the
1,900 (4 lb, 3 oz). In addition to routine smaller baby warm
nursing care, the physician has ordered that
Twin "B" be kept in an isolette to help
maintain his temperature. The nurse might
suggest which of the following interventions
instead of using an isolette to maintain the
baby's temperature?
15 A client makes a routine visit to the prenatal "2. grapelike clusters.
0. clinic. Although she believes that she is 14
weeks pregnant, the size of her uterus
approximates that in an 18- to 20-week
pregnancy. The physician diagnoses
gestational trophoblastic disease and orders
ultrasonography. The nurse expects
ultrasonography to reveal:
15 A client, now 37 weeks pregnant, calls the "2. Explain that these are expected
1. clinic because she's concerned about being problems for the latter stages of
short of breath and is unable to sleep unless pregnancy.
she places three pillows under her head.
After listening to her concerns, the nurse
should take which action?
15 A client plans to bottle-feed her full-term 40 to 60 ml of formula every 2 to 4
2. neonate. What is the normal feeding pattern hours
for a full-term neonate during the 24 hours
after delivery?
15 A client plans to breast-feed her healthy, the neonate will be responsive and
3. full-term neonate. The nurse encourages her eager to suck at this time.
to start breast-feeding within 30 minutes of
the neonate's birth because:
15 A client's gestational diabetes is poorly controlled Provide frequent early
4. throughout her pregnancy. She goes into labor at 38 feedings with formula.
weeks and delivers a baby boy. Which priority
intervention should be included in the plan of care for
the baby during his first 24 hours?
15 A client's membranes rupture during the 36th week of 2. delivery.
5. pregnancy. Eighteen hours later, the nurse measures
the client's temperature at 101.8 F (38.8 C). After
initiating prescribed antibiotic therapy, the nurse
should prepare the client for:
15 A client tells the nurse that she doesn't want to sign Discussing the purpose
6. the hepatitis B vaccination consent form because she of the vaccine and
heard that, "vaccinations can cause autism." What's providing the client
the most appropriate nursing interaction? with written
information
15 A client treated with terbutaline (Brethine) for preterm "1. Report a heart rate
7. labor is ready for discharge. Which instruction should greater than 120
the nurse include in the discharge teaching plan? beats/minute to the
physician.
15 A client who admits she uses heroin gives birth to a Evaluation for signs of
8. neonate at 32 weeks' gestation. Which neonatal drug withdrawal
assessment is most important for the nurse to
perform?
15 A client who's 2 months pregnant complains of urinary "4. Explain that urinary
9. frequency and says she gets up several times at night frequency is expected
to go to the bathroom. She doesn't have other urinary during the first
symptoms. How should the nurse intervene? trimester.
16 A client who's 4 months pregnant asks the nurse how 3. ""Walk briskly for 10
0. much and what type of exercise she should get during to 15 minutes daily,
pregnancy. How should the nurse counsel her? and gradually increase
this time.""
16 A client who's 4 weeks pregnant comes to the clinic for 4. The client consumes
1. her first prenatal visit. When obtaining her health no alcohol.
history, the nurse explores her use of drugs, alcohol,
and cigarettes. Which client behavior identifies a safe
level of alcohol intake for this client?
16 A client who's 5 weeks pregnant reports nausea and 2. 9 to 12 weeks'
2. vomiting. The nurse reassures the client that these gestation.
symptoms probably will subside by:
16 A client who's 7 months pregnant reports severe leg "3. Teaching her to
3. cramps at night. Which nursing action would be most dorsiflex her foot
effective in helping her cope with these cramps? during the cramp
16 A client who's 7 weeks pregnant comes to the clinic for 3. intrauterine growth
4. her first prenatal visit. She reports smoking 20 to 25 retardation.
cigarettes per day. When planning the client's care, the
nurse anticipates informing her that if she doesn't stop
smoking, her fetus will develop:
16 A client who's 12 weeks pregnant attends a class on 3. have audible heart
5. fetal development as part of a childbirth education sounds.
program. The nurse says that at 16 weeks' gestation,
the client's fetus will likely:
16 A client who's 12 weeks pregnant is complaining of 1. Risk for deficient
6. severe left lower quadrant pain and vaginal spotting. fluid volume.
She's admitted for treatment of an ectopic pregnancy.
Of the following nursing diagnoses, the nurse should
give the highest priority to:
16 A client who's 24 weeks pregnant and diagnosed with 3. ""The community
7. preeclampsia is sent home with orders for bed rest and health nurse will check
a referral for home health visits by a community health me and my baby and
nurse. Which comment made by the client should talk with my
indicate to the nurse that the client understands the physician.""
purpose of home health visits?
16 A client who's 24 weeks pregnant has sickle cell "2. Dehydration
8. anemia. When preparing the plan of care, the nurse
should identify which factor as a potential trigger for a
sickle cell crisis during pregnancy?
16 A client who's 30 weeks pregnant has a corrected atrial 2. Increased plasma
9. septal defect and minor functional limitations. Which volume
pregnancy-related physiologic change places her at
greatest risk for more severe cardiac problems?
17 A client who's 32 weeks pregnant is hospitalized with "2. ""Abstain from
0. preterm labor. After preterm labor is arrested, she's sexual intercourse
discharged with a prescription for oral terbutaline unless you use a
(Brethine). Which instruction should the nurse provide condom.""
during discharge teaching?
17 A client who's 37 weeks pregnant comes to the "What changes have you made
1. clinic for a prenatal checkup. To evaluate the at home to get ready for the
client's preparation for parenting, the nurse baby?"
might ask which question?
17 A client who's planning a pregnancy asks the 4. ""Folic acid supplements
2. nurse about ways to promote a healthy improve pregnancy outcomes by
pregnancy. Which of the following would be preventing certain
the nurse's best response? complications.""
17 A client who's pregnant with her second child 1. Braxton Hicks contractions
3. comes to the clinic complaining of a pulling
and tightening sensation over her pubic bone
every 15 minutes. She reports no vaginal fluid
leakage. Because she has just entered her
36th week of pregnancy, she's apprehensive
about her symptoms. Vaginal examination
discloses a closed, thick, posterior cervix.
These findings suggest that the client is
experiencing
17 A client with human immunodeficiency virus hepatosplenomegaly.
4. (HIV) infection gives birth to an HIV-positive
neonate. When assessing the neonate, the
nurse is likely to detect:
17 A client with hyperemesis gravidarum is on a "3. tea and gelatin dessert.
5. clear liquid diet. The nurse should serve this
client:
17 A client with hyperemesis gravidarum is on a "3. Tea and gelatin 4. Ginger ale
6. clear liquid diet. Which foods would be and apple juice 5. Cranberry
appropriate for the nurse to serve? juice and chicken broth "
17 A client with pregnancy-induced hypertension 3. Proteinuria, headaches, and
7. (PIH) probably exhibits which of the following double vision
symptoms?
17 A client with pregnancy-induced hypertension 2. To prevent seizures
8. (PIH) receives magnesium sulfate, 4 g in 50%
solution I.V. over 20 minutes. What is the
purpose of administering magnesium sulfate
to this client?
17 A client with type 1 diabetes mellitus has just 3. ""Insulin requirements usually
9. learned she's pregnant. The nurse is teaching decrease during the first
her about insulin requirements during trimester.""
pregnancy. Which guideline should the nurse
provide?
18 Cloudy amniotic fluid with strong odor A pregnant woman with
0. premature rupture of
membranes is at higher risk for
postpartum infection. Which
assessment data indicate a
potential infection?
18 A cold or dehydrated baby= A dead baby
1.

18 colostrum milk production in breasts


2.

18 A common childhood disease that can have chickenpox


3. devastating effect on an immunosuppressed
child
18 compression of the umbilical cord is more The fetus in a breech
4. likely. presentation is often born by
cesarean delivery because:
18 A congenital heart defect that results in tetralogy of Fallot
5. decreased pulmonary blood flow is
18 Contact with blood; Contact with skin disease; As nurses today, we have to care
6. Improper care of the body; Lack of sleep for patients from many different
cultures and backgrounds. Select
all of the following ideas/beliefs
from the Japanese culture about
how illness is caused.
18 continuity of care from hospital to home. The services of a transition
7. specialist for patient discharge
often leads to an increase in:
18 contractions abruptly stop during labor. The nurse should suspect
8. uterine rupture if:
18 Contraction Description
9. stress test *Assess placental oxygenation and function
*Determines fetal ability to tolerate labor
*Determies fetal well-being
*Fetus is exposed to the stressor of contractions to assess the
adequacy of placental perfusion under simulated labor conditions
*Performed if nonstress test is abnormal

Interventions
*The external fetal monitor is applied to the mother, and a 20-30 min
baseline strip is recorded.
*The uterus is stimulated to contract, either by the administration of
a dilute dose of oxytocin (Pitocin) or by having the mother use nipple
stimulation, until 3 palpable contractions w/duration of 40 sec. or
more during a 10 min. period have been achieved.
*Frequent maternal blood pressure reading are obtained, and the
mother is monitored closely while increasing doses of oxytocin are
given.
Results
*Negative Contraction Stress Test
*Represented by no late decelerations of the FHR

*Positive Contraction Stress Test (Abnormal )


*Represented by late decelerations of FHR w/50% or more of
contractions in absence of hyperstimulation of the uterus.

Equivocal
*Contains decelerations but w/less than 50% of contractions, or
uterine activity shows a hyperstimulated uterus.

Unsatisfactory
*Adequate uterine contractions cannot be achieved or FHR tracing is
not sufficient quality for adequate interpretation.
19 cool air how to manage nasal stuffiness and nasal bleeding during early
0. vaporizer prego
19 Cracked, What data would alert the nurse that the neonate is postmature?
1. peeling skin
19 cytomegalovir respiratory droplet infection transmitted from day care, mentally
2. us CMV handicapped are at high risk may cause fetal death, jaundice,
hydrocephaly, deafness
19 Danger Signs Severe vomiting
3. of Pregnancy Chills
Fever
Burning on urination
Diarrhea
Abdominal cramping or vaginal bleeding
Sudden discharge of fluid from Vagina before 37 wks.
Severe backache or flank pain
Change in fetal movements
Visual disturbances
Swelling of face or fingers or over sacrum
Headaches
Epigastric or abdominal pain
Muscular irritability or seizures
Glycosuria or other signs of diabetes mellitus
19 D&C If nonsurgical treatment for subinvolution is ineffective, which
4. surgical procedure is appropriate to correct the cause of this
condition?
19 decreased The nurse understands that a laboratory finding indicative of DIC is:
5. fibrinogen.
19 Define apnea Apnea is an unexplained episode of cessation of
6. breathing for 20 seconds or longer, hypotonia, and
cyanosis or pallor.
19 Degree of glycemic control Which factor is most important in diminishing
7. before and during the maternal/fetal/neonatal complications in the pregnant
pregnancy woman with diabetes?
19 Dehydration stimulates Why is adequate hydration important when uterine
8. secretion from the activity occurs before pregnancy is at term?
posterior pituitary.
19 Delivery of the fetus What is the only known cure for preeclampsia?
9.

20 Describe associative play no group goal; often follows a leader


0. PRESCHOOL age (4 year)
20 Describe cooperative play organized, rules, leader/follower relationship est
1.
SCHOOL-AGE
20 Describe games and ages 8-12
2. hobbies phase?
20 Describe parallel play plays alongside, not with, another; characteristic of
3. TODDLERS, but can occur in other age groups
20 Describe phases of Protest- cries/screams for parents; inconsolable by
4. separation anxiety. others

Despair- crying ends; less active; disinterested in


food/play; clutches security obj if available

Denial- appears adjusted; evidences interest in


environment; ignores paraent when he/she returns;
resigned, not contented
20 Describe solitary play plays alone, but enjoys presence of others, interest
5. centered on own activity

occurs during infancy


20 Describe the exploratory holding toys, ages 0-7 yr
6. phase?
20 Describe toys as adults imitation, age 1-7 yr
7. tools phase?
20 Determining cervical What routine nursing assessment is contraindicated in
8. dilation and effacement the client admitted with suspected placenta previa?
20 diagnostic what signs can only be associated with pregnancy
9.

21 DIAGNOSTIC TESTS 1. Outlines and identifies fetal and maternal structures


0. Ultrasonography 2. Helps confirm gestational age and estimated date
of delivery.
3. May be done abdominally or transvaginally during
pregnancy.
4. Interventions
a. If an abdominal ultrasound is being performed, the
woman may need to drink water to fill bladder before
procedure to obtain a better image of the fetus
b. If a transvaginal ultrasound is being perfromed, a
lubricated probe is inserted into the vagina.
c. Inform client that the test presents no known risks
to client or fetus..
21 The diagnostic test that bone marrow aspiration
1. confirms a diagnosis of
leukemia is
21 diastasis recti separation of abdominal muscle after uterus enlarges
2.

21 Digoxin(Lanoxin) is 100
3. withheld if the pulse of a
newborn is below ____ bpm
21 DISCOMFORTS OF Nausea and vomiting
4. PREGNANCY 1. Occur during first trimester
2. Caused by elevated hCG levels and changes in carbohydrate
metabolism.
3. Interventions
a. Eating dry crackers before arising
b. Avoiding brushing teeth immediately after arising.
c. Eating small, frequent, low-fat meals during the day.
d. Drinking liquids between meals rather than at meals.
e. Avoiding fried foods and spicy foods.
f. Acupressure (some types may require a prescription)
g. Herbal remedies, only if approved by a physician or nurse-
midwife.

Syncope
1. Usually occurs during 1st trimester; supine hypotension
occurs, particularly during the 2nd and 3rd trimesters.
2. May be hormonally triggered or caused by increased blood
volume, anemia, fatigue, sudden position changes, or lying
supine.
3. Interventions
a. Sitting w/feet elevated
b. Changing positions slowly
c. Changing to lateral recumbent position to relieve pressure of
uterus on inferior vena cava.
d. Increasing fluid intake

Urinary urgency and frequency


1. Usually occurs during 1st and 3rd trimesters.
2. Caused by pressure of uterus on bladder.
3. Interventions
a. Drinking 2 quarts of fluid during day
b. Limiting fluid intake during evening.
c. Voiding at regular intrvals.
d. Sleeping on side at night.
e. Wearing perineal pads, if necessary.
f. Performing Kegel excercises

Breast tenderness and tingling


1. Can occur from 1st through 3rd trimesters.
2. Caused by increased levels of estrogen and progesterone.
3. Interventions
a. Encouraging use of a supportive bra w/nonelastic straps
b. Avoiding use of soap on nipples and areolae to prevent
drying.

Increased vaginal discharge


1. Can occur from 1st - 3rd rimesters
2. Caused by hyperplasia of vaginal mucosa and increased
mucus production.
3. Interventions
a. Wearing cotton underwear
b. Avoiding douching
c. Using proper cleansing and hygiene techniques.
d. Advising client to consult physician or nurse-midwife if
infection is suspected.

Nasal stuffiness or nosebleeds


1. Occurs during 1st-3rd trimesters
2. Occurs as result of increased estrogen that causes swelling
of nasal tissues and dryness
3. Interventions
a. Encouraging use of humidifier.
b. Avoiding use of nasal sprays or antihistamines.

Fatigue
1. Occurs usually during 1st-3rd trimesters
2. Is usually result of hormonal changes
3. Interventions.
a. Arranging frequent rest periods throughout day.
b. Using correct body mechanics
c. Engaging in regular exercise
d. Performing muscle relaxation and strengthening exercises
for legs and hip joints.
e. Avoiding eating and drinking foods that contain stimulants
throughtout pregnancy.

Heartburn
1. Occurs during 2-3rd trimesters.
2. Results from increased progesterone levels,decreased GI
motility, esophageal reflux, and the displacement of the
stomach by enlarging uterus.
3. Interventions
a. Eating small, frequent meals and avoiding fatty and spicy
food
b. Sitting upright for 30 min. after a meal.
c. Drinking milk between meals.
d. Performing tailor-sitting exercises (sitting cross-legged)
e. Taking antacids only if recommended by the physician or
nurse-midwife.

Ankle edema
1. Usually occurs during 2-3rd trimesters
2. Occurs as a result of vasodilation, venous stasis, and
increased venous pressure below the uterus.
3. Interventions
a. Elevating legs during day
b. Sleeping on left side
c. Wearing supportive stockings
d. Avoiding sitting or standing in one position for long periods.

Varicose veins
1. Usually occur during 2-3rd trimesters
2. Occur because of weakening walls of veins or valves and
venous congestion.
3. Interventions
a. Wearing support hose
b. Elevating feet when sitting
c. Lying w/feet and hips elevated.
d. Avoiding long periods of standing or sitting
e. Moving about while standing to improve circulation
f. Avoiding leg crossing
g. Avoiding constricting articles of clothing

Headaches
1. Usually occur during 2-3rd trimesters.
2. Occur as a result of changes in blood volume and vascular
tone.
3. Interventions
a. Changing position slowly.
b. Applying a cool cloth to forehead
c. Eating a small snack
d. Using acetaminophen (Tylenol) only if prescribed by the
physician or nurse-midwife.

Hemorrhoids
1. Usually occur during 2-3rd trimester
2. Occur as a result of increased venous pressure and/or
contstipation.
3. Interventions
a. Soaking in a warm sitz bath
b. Sitting on a soft pillow
c. Eating high-fiber foods and avoiding constipation
d. Drinking sufficient fluids
e. Increasing exercise, such as walking
f. Applying ointments, suppositories,s or compresses as
prescribed by the physician or nurse-midwife.

Constipation
1. Usually occurs during 2-3rd trimesters.
2. Occurs as result of decreased intestinal motility,
displacement of intestines, and taking iron supplements.
3. Interventions
a. Eating high-fiber foods
b. Drinking sufficient fluids
c. Exercising regularly
d. Avoiding laxatives and enemas unless their use is approved
by the physician or nurse-midwife.

Backache
1. 2-3rd trimester
2. Occurs as a result of exaggerated lumbosacral curve, which
is cause by enlarged uterus.
3. Interventions
a. Encouraging rest
b. Using correct body mechanics and improving posture.
c. Wearing low-heeled shoes
d. Performing pelvic rocking and abdominal breathing exercises
e. Sleeping ona firm mattress

Leg cramps
1. Usually occur during 2-3 trimester
2. Occur as result of an altered calcium-phosphorus balance,
pressure of the uterus on nerves, or fatigue
3. Interventions
a. Getting regular exercise, especially walking
b. Dorsiflexing foot of affected leg
c. Increasing calcium intake
Shortness of breath
1. 2-3rd trimester
2. Occurs as reult of pressure on diaphragm
3. Interventions
a. Allowing frequent rest periods and avoiding overexertion
b. Sleeping w/head elevated or on side
c. Performing tailor-sitting exercises.
21 Does GFR increase or decrease in cases of decreases, the patient retains
5. AGN? fluids and thus should eat a diet
of low protein, salt, and K
21 During a bath, a neonate has a nursing Support the neonate's head and
6. diagnosis of Risk for injury related to slippage back with the forearm.
while bathing. Which intervention best
addresses this nursing diagnosis?
21 During a health-teaching session, a pregnant 1. 7 days after fertilization
7. client asks the nurse how soon the fertilized
ovum becomes implanted in the endometrium.
Which answer should the nurse supply?
21 During a home-care visit with a pregnant client, 1. Explain that extension cords
8. the nurse notices extension cords crossing can pose a danger to the client
between rooms. How should the nurse when she's walking.
intervene?
21 During a physical examination, a client who's 1. Turn the client on her left side
9. 32 weeks pregnant becomes pale, dizzy, and
light-headed while supine. Which action should
the nurse immediately take
22 "During a prenatal screening of a client with 1. Still birth 3. Pregnancy-
0. diabetes, the nurse should keep in mind that induced hypertension 5.
the client is at increased risk for which Spontaneous abortion
complications?
22 During a prenatal visit, a pregnant client with "4. divide daily food intake into
1. cardiac disease and slight functional limitations five or six meals.
reports increased fatigue. To help combat this
problem, the nurse should advise her to:
22 During a prenatal visit, the nurse measures a 2. 19 weeks
2. client's fundal height at 19 cm. This
measurement indicates that the fetus has
reached approximately which gestational age?
22 During a routine assessment, a pregnant client 3. Discuss the client's diet,
3. tells the nurse that she hasn't had a bowel focusing on her fiber and water
movement for "close to a week." What should intake
the nurse do to help this client
22 During a routine prenatal visit, a pregnant "3. ""I'll increase my intake of
4. client reports constipation, and the nurse unrefined grains.""
teaches her how to relieve it. Which client
statement indicates an accurate understanding
of the nurse's instructions?
22 During a routine prenatal visit, a pregnant 1. Eat small, frequent meals
5. client reports heartburn. To minimize her
discomfort, the nurse should include which
suggestion in the plan of care
22 During each prenatal checkup, the nurse 1. Evaluating the client for
6. obtains the client's weight and blood pressure edema
and measures fundal height. What is another
essential part of each prenatal checkup?
22 During her first prenatal visit, a client 1. Ask the client how she feels
7. expresses concern about gaining weight. Which about gaining weight and
of the following actions should be the nurse's provide instructions about
next step? expected weight gain and diet.
22 During her first prenatal visit, a pregnant client "3. Imbalanced nutrition: Less
8. admits to the nurse that she uses cocaine at than body requirements related
least once per day. Which nursing diagnosis is to limited food intake
most appropriate for this client?
22 During her fourth clinic visit, a client who's 5 . "No, because the live viral
9. months pregnant tells the nurse she was vaccine is contraindicated
exposed to rubella during the past week and during pregnancy."
asks whether she can be immunized now. How
should the nurse respond?
23 During hospitalization Effective discharge requires
0. careful planning and continuing
assessment of a patient's
needs:
23 During neonatal resuscitation immediately 60
1. after delivery, chest compressions should be
initiated when the heart rate falls below how
many beats per minute?
23 During routine preconception counseling, a 1. ""8 days after conception""
2. client asks how early a pregnancy can be
diagnosed. What is the nurse's best response?
23 During the first 3 months, which hormone is "1. Human chorionic
3. responsible for maintaining pregnancy? gonadotropin (hCG)
23 During the sixth month of pregnancy, a client 2. Eustachian tube
4. reports intermittent earaches and a constant vascularization
feeling of fullness in the ears. What is the likely
cause of these symptoms?
23 Early detection of an ectopic "1. Abdominal pain, vaginal bleeding, and a
5. pregnancy is paramount in positive pregnancy test
preventing a life-threatening
rupture. Which symptoms should
alert the nurse to the possibility
of an ectopic pregnancy?
23 eat 6 saltine crackers. When the pregnant diabetic experiences
6. hypoglycemia while hospitalized, the nurse
should have the client:
23 eating her meals and snacks on a To manage her diabetes appropriately and
7. fixed schedule. ensure a good fetal outcome, the pregnant
diabetic will need to alter her diet by:
23 eat small frequent meals, avoid how to manage heartburn during late prego
8. spicy/greasy foods, dont lie
down after eating use low
sodium antacids
23 eclampsia. Methergine (an oxytocic drug) increases the
9. blood pressure. The nurse should question the
order to administer Methergine to the woman
with a history of:
24 elevate legs, wear hose, avoid manage varicose veins in late prego
0. crossing legs, avoid restrictive
clothing and standing for long
24 encourage the parents to touch An infant girl is preterm and on a respirator with
1. her. intravenous lines and much equipment around
her when her parents come to visit for the first
time. It is important for the nurse to:
24 encouraging urination about An important independent nursing action to
2. every 1 to 2 hours. promote normal progress in labor is:
24 Endocrine system. Basal metablic rate rises.
3.

24 ensure continuity of care. A 45-year-old patient has been recently


4. diagnosed and hospitalized for type 1 diabetes
mellitus. The multidisciplinary health care team
has been preparing for her dismissal. The
purpose of discharge planning is to:
24 ensuring the patient has an During the registration process, the admission
5. allergy band. clerk is responsible for:
24 epigastric pain besides visual disturbance, edema, or HA what
6. els is a danger sign to preeclampsia
24 erythroblastosis or what two complications may be identified from a
7. hyperbilirubinemia blood type and Rh stats during first prenatal
visit
24 estrogen hormone stimulates uterine development to
8. support fetal grwoth and ducts in breasts for
lacatation`
24 estrogen what hormone causes increased pigmentation
9. during prego
25 estrogen and progesterone what hormones cause breasts to increase in size
0. and number of glands
25 "Even though my test is positive, A woman has tested human immunodeficiency
1. my baby might not be affected." virus (HIV) positive and has now discovered that
she is pregnant. Which statement indicates that
she understands the risks of this diagnosis?
25 every 2 weeks when are prenatal visist be done between 28-36
2. weeks
25 every 4 weeks during first 28 when are prenatal visits wo be done in first 28
3. weeks gestation weeks gestation
25 every week until delivery when are prenatal visits done after 36 weeks
4.

25 explain the risks of leaving and A patient is determined to leave the hospital. His
5. request that the patient sign a physician is not aware of his intent, nor is it in
paper accepting responsibility his best interests to be discharged at this time.
for problems that may occur. When a patient chooses to leave a health care
facility without a physician's written order, the
nurse should:
25 facilitates drainage of lochia. The client who is being treated for endometritis
6. is placed in Fowler's position because
it:facilitates drainage of lochia.
25 False, On the contrary, the Should a postpartum complication such as
7. unusual activity of the hospital hemorrhage occur, the nursing staff will spring
staff may make the mother and into action to ensure that client safety needs are
her family very anxious. met. This level of activity is very reassuring to
both the new mother and her family members
as they can see that the client is receiving the
best of care.
25 false, This is often referred to as The nurse has been caring for a primiparous
8. the "turtle sign" and is an client who is suspected of carrying a
indication of shoulder dystocia. macrosomic infant. Pushing appears to have
been effective so far; however, as soon as the
head is born, it retracts against the perineum
much like a turtle's head drawing into its shell.
In evaluating the labor progress so far, the
nurse is aware that this is normal with large
infants and extra pushing efforts by the
mother may be necessary.
25 A famous pregnant client comes to 2. Inform the media that you can't comment
9. the physician's office for a routine about whether the person is being seen in the
prenatal examination. While the office
client is in the office, the media
arrives asking for information
about the client. What should the
nurse do?
26 A female client on an inpatient ANS: C
0. unit enters the day area for The most appropriate intervention by the
visiting hours dressed in a see- nurse is to lead the client back to her room
through blouse and wearing no and assist her to choose appropriate clothing.
undergarments. Which The client could be exhibiting signs of
intervention should be a nurse's exhibitionism which is characterized by urges
first priority? to expose oneself to unsuspecting strangers.

A. Discuss with the client the


inappropriateness of her attire.
B. Avoid addressing her attention-
seeking behavior.
C. Lead the client back to her
room and assist her to choose
appropriate clothing.
D. Restrict client to room until
visiting hours are over.
26 A female neonate born by elective Respiratory distress due to lack of contractions
1. cesarean section to a 25-year-old
mother weighs 3,265 g (7 lb, 3 oz).
The nurse places the neonate
under the warmer unit. In addition
to routine assessments, the nurse
should closely monitor this
neonate for which of the
following?
26 Fern test 1. A microscopic slide test to determine the
2. presence of amniotic fluid leakage.
2. With use of sterile technique, a specimen is
obtained rom the external os of cervix and
vaginal pool and examined on a slide under a
microscope.
3 A fernlike pattern that results from the salts
of amniotic fluid indicates the presence of
amniotic flid
4. Interventions
a. Place client in dorsal lithotomy position.
b. Instruct client to cough; this causes fluid to
leak from uterus if membranes are ruptured.
26 Fetal heart rate, maternal pulse, "You should come into the office and let the
3. and blood pressure doctor check you."
26 fetoscope FHR assessed by this equipment starting at 16
4. weeks
26 fetus is Rh positive. Rh incompatibility can occur if the woman is
5. Rh negative and her:
26 FHR auscultated this can occur as early as 8 weeks gestation
6. using doppler but normally heard 10-12 weeks
26 first trimester and obese when is transvaginal ultrasound completed
7.

26 first trimester and third trimester gravid uterus presses on bladder causing
8. urniary frequency in this trimester
26 folic acid B6 vitamin that reduce risk of birth defects
9.

27 forcing fluids to at least 3000 Nursing measures that help prevent


0. mL/day. postpartum urinary tract infection include:
27 For how long should a baby face <1 yr
1. the back in a car seat?
27 A full-term neonate is diagnosed wide or bulging fontanels.
2. with hydrocephalus. Data
collection is likely to reveal:
27 Fundal height What finding on a prenatal visit at 10 weeks might suggest a
3. measurement of hydatidiform mole?
18 cm
27 fundal height, what are three assessment findings for fetus
4. movment and FHR
27 gardasil vaccine what is the prevention for HPV
5.

27 Gastrointestinal 1. Nausea and vomiting may occur as result of secretion of


6. (GI) System human chorionic gonadotropin (hCG), which subsides by the 3rd
month.
2. Increase in body's nutritional requirements (vitamins, minerals,
calores, protein, and iron)
3. Constipation as a result of decreased GI motility or pressure of
the uterus.
4. Chages in appetite, flatulence, and heartburn as result of
decreased GI motility and slow emtying of stomach.
5. Alteration in taste and smell may occur.
6. Hemorrhoids as a result of increased venous pressure.
7. Altered gallbladder function that can lead to gallstone
formation caused by an increase in progesterone.
27 G&D 2 month posterior fontanell closes
7. diminished tonic neck and Moro reflex (gone by 3 mo)
able to turn from back to side
eyes begin to follow moving obj
social smile first appears
2-3 months posterior fontanell closure
able to lift head 45 degrees
27 G&D 3 months can bring obj to mouth at will; head held erect, steady; smiles
8. w/mother's presence; audibly laughs
27 G&D 4 months ABSENT tonic neck reflex; drooling; Moro reflex gone
9.
4-5 mo start solid foods
28 G&D 5 months BIRTH WEIGHT doubles
0. takes obj presented to him/her
28 G&D 6 months avg weight gain 4 oz per week; teething may begin; can turn
1. from back --> stomach; early ability to distinguish and recognize
strangers [DON'T leave infant alone]; can sit up with support
28 G&D 7 months sits for short periods using hands for support; grasps toys
2. w/hand; abrupt mood shifts
28 G&D 8 months stranger anxiety and reverts to phases
3.

28 G&D 9 months can SIT ALONE, responds to parental anger, PINCHER GRASP;
4. responds to verbal commands like "no-no"; count 2 cubes
28 G&D 10 months crawls well; pulls self to standing position w/spt (think crib
5. safety), brings one hand together, vocalizes 1-2 words
28 G&D 11 months erect standing posture w/spt
6.

28 G&D 12 months tripled birth weight; needs help w/walking;eats w/fingers; can say
7. two words in addition to mama and dada

can build 2 block tower

shows interest in pleasing parents like hugging mom when asked


28 G&D at 1 month head sags, early crawling movement, "floppy"
8.

28 gestational age fundal height is a measure from symphysis pubis to top of uterine
9. fundus shows this age
29 GFR increases during second trimester this function increasesand remains
0. 50% elevated until delivery in renal function
29 A girl neonate is admitted to the check the neonate's blood glucose level.
1. nursery following a long and
difficult labor. Admission vital signs
are temperature 96.5 F (35.8 C),
heart rate 168 beats/minute, and
respiratory rate 64 breaths/minute.
After placing the neonate under
the radiant heater, the nurse's next
action should be to:
29 glucose what is found in urine to diagnose diabetes
2. mellitus during prego
29 glucose tolerance test GTT 50 g oral glucose load is given after one hour
3. levels found greater than 130-140 mg/dL then
followed by a 3 hour 100 g to diagnose
gestational diabetes
29 Glycosuria Which clinical sign is not included in the
4. classic symptoms of preeclampsia?
29 gonorrhea what is the STI that if not treated to neonate
5. with eye drops the infant could be blind
29 goodells sign estrogen causes softening of cervix is this
6. sign during pregnancy
29 gravida number of times woman is pregnant
7.

29 greater surface area in proportion In comparison with the term infant, the
8. to weight. preterm infant has:
29 greater than 95 what is an abnormal BS after fasting for 8
9. hours
30 The greatest concern of a nurse injuries that might initiate bleeding
0. caring for a child with ITP is
30 greet the patient by name. The nurse is admitting a patient to the
1. nursing unit. The nurse's first action is to:
30 Grunting Of all the signs seen in infants with
2. respiratory distress syndrome, which sign is
especially indicative of the syndrome?
30 Gynecoid Which pelvic shape is most conducive to
3. vaginal labor and delivery?
30 Hard, boardlike abdomen What condition would indicate concealed
4. hemorrhage in an abruptio placentae?
30 Have the patient sign a Consent for A patient is being admitted to the surgical
5. Treatment floor. Before the patient goes to surgery, the
Admissions department will:
30 have the patient sign consent for A patient is being admitted to the hospital for
6. treatment. stabilization of her heart condition. Before
arriving on the nursing unit, the admissions
department will:
30 "Have you been taking any over- A patient with benign prostatic hyperplasia
7. the-counter (OTC) medications (BPH) with mild obstruction tells the nurse,
recently?" "My symptoms have gotten a lot worse this
week." Which response by the nurse is most
appropriate?
30 hCG hormones stimulates progesterone and
8. estrogen production and cause NV in first
trimester
30 The head seems large compared What will the nurse note when assessing an
9. with the rest of the body. infant with asymmetric intrauterine growth
restriction?
31 "Hello, Ms. Green, my name is ?" You are assisting the RN with a newly
0. admitted female patient. What is the most
appropriate way to address the patient?
31 hematoma The nurse is in the process of assessing the
1. comfort level of her postpartum client. Excess
bleeding is not obvious; however, the new
mother complains of deep, severe pelvic pain.
The registered nurse (RN) has noted both skin
and vital sign changes. This client may have
formed a ____________________.
31 hemorrhage is the major concern. A 32-year-old primigravida is admitted with a
2. diagnosis of ectopic pregnancy. Nursing care
is based on the knowledge that:
31 hemorrhoids constipation and increase pressure on blood vessel in
3. rectum can lead to this
31 Hep A infection spread by droplets associated with poor
4. handwashing after defecation
31 Hep B most common fetus infection from blood and genital
5. secretions during labor and delivery.
31 Herpes STI from vescular lesions on genitalia fetus is at high
6. risk from primary infection of mother if lesion in birth
canal, may need a cesarean delivery
31 he should continue to use When performing discharge teaching for a patient who
7. other methods of birth has undergone a vasectomy in the health care
control for 6 weeks. provider's office, the nurse instructs the patient that
______________
31 Hgb what is tested for iron-deficiency anemia
8.

31 H&H, blood type, Rh, what type of tests are given to pregnant female
9. irregular antibody,
rubella, TB, renal
function, UA, STI, Pap,
offer HIV
32 H&H drop a little what happens to H&H levels during pregnancy
0.

32 his interest in sexual The health care provider prescribes finasteride (Proscar)
1. activity may decrease for a 56-year-old patient who has benign prostatic
while he is taking the hyperplasia (BPH). When teaching the patient about the
medication. drug, the nurse informs him that ____________
32 hot flashes. Leuprolide (Lupron) is prescribed for a patient with
2. cancer of the prostate. In teaching the patient about
this drug, the nurse informs the patient that side effects
may include
32 How do a child's bones Children's bones are more flexible and porous than
3. compare to those of an those of adults; in fact, fractures are very rare before
adults? age 1.
32 How does muscular proceeds head-to-toe (cephalocaudal), trunk-to-
4. coordination and control periphery (proximosdistal), gross-to-fine
happen?
32 How does the nurse Stroking the neonate's cheek
5. assess the rooting reflex
of a neonate?
32 How do you know how One gram of diaper weight equals one milliliter of urine.
6. much a baby is peeing, Children with urine output less than one
and how much should milliliter/kilogram/hour should be closely monitored for
they be peeing? possible renal failure.
32 How is methergine used It is used to control bleeding by promoting increased
7. after birth? contractions and stiffening of the uterus muscles.
32 How is osteoporosis With diet (calcium and vitamin D), exercise, and
8. treated? medication if needed.
32 How long should a child 12 yrs
9. remain seated in the
backseat?
33 How should you position a Fowlers or high fowlers
0. child to reduce the
periorbital edema
associated with nephrotic
syndrome?
33 How should your patient Do not elevate or lower head: maintain complete bed
1. that you suspect is going rest in flat position or with legs slightly raised to
into (or is in) shock be increase venous return (modified trendelenburg)
positioned?
33 how to care for an A patient with symptomatic benign prostatic hyperplasia
2. indwelling urinary (BPH) is scheduled for photovaporization of the prostate
catheter. (PVP) at an outpatient surgical center. The nurse will
plan to teach the patient ______________
33 How to introduce foods? only introduce one food at a time for each 2 week
3. period

start with least allergenic foods from 6 mo (iron-fortified


rice cereal) progressing to more allergic foods (egg,
orange juice)

cereal --> fruit -->vegetables --> potatoes --> meat -->


egg --> orange juice
33 How/When should To prevent GI side effects take with 8 ounces of water
4. Fosamax be taken? first thing in the morning. Stay sitting up and NPO for at
least half an hour before eating or taking other drugs.
33 How will a premature (before Soles are smoother, creases extend less than 2/3
5. 37 weeks) infants heel creases of the way from the toes to the heel.
vary from normal?
33 How will a premature (before Fewer shallow rugae on the scrotum
6. 37 weeks) infants male
genitalia vary from normal?
33 HPV human papillmavirus STI genital warts fetus can acquire during birth
7. process
33 Hydatidiform mole A woman is admitted with vaginal bleeding at
8. approximately 10 weeks of gestation. Her fundal
height is 13 cm. What potential problem should be
investigated?
33 hyperemesis gravidarum persistent vomiting during pregnancy that may
9. cause dehydration
34 Hypertension is identified in a a blood pressure measurement is scheduled in 4
0. 10yr old child during routine weeks
screening, the nurse should
expect which plan of care to be
implemented initially?
34 Hypoglycemia Which major neonatal complication is carefully
1. monitored after the birth of the infant of a diabetic
mother?
34 hypoxia in utero. While caring for the postterm infant, the nurse
2. recognizes that the elevated hematocrit level
most likely results from:
34 If a newborn coughs, chokes, tracheoesophageal fistula (note the 3 C's - cough,
3. and turns blue with feeding, choke, and cyanosis).
what should the nurse
suspect?
34 If a person is choking and 1. activate the EMS system
4. becomes unresponsive, 2. lower the client to the ground
describe the sequence of the 3. begin with compressions (don't check for a
rescuers actions. pulse)
*if you were not certain the person was choking,
you would just immediately begin CPR
4. Open the victim's mouth wide and look for the
object before giving breaths; remove object only if
able to remove without further pushing down
airway
34 If a woman during her 1st Investigate further bc back pain during the 1st
5. trimester is experiencing back trimester isn't normal and she may be
pain what should be done? experiencing an ectopic pregnancy. Other
symptoms associated with ectopic pregnancy
include increased heart rate and pulse from
hemorrhage.
34 If a woman in her 3rd trimester Nothing bc leg cramps in this trimester are
6. of pregnancy is experiencing normal. (NOT DVT)
leg cramps what should be
done?
34 If a womens last menstrual February 28th!
7. period is 5/21 when is her
estimated due date?
34 If the power goes out and a YES!!!! Bc they may get cool rapidly without the
8. baby is under a bili-light are light. Cold baby=dead baby
they a priority patient?
34 If you see signs of Check sugar and give glucose h20 (don't need
9. hypoglycemia in a newborn anymore evidence)
what should you do?
35 "I have notified the doctor that A laboring client in the latent phase is
0. you are having a lot of experiencing uncoordinated, irregular contractions
discomfort. Let me rub your of low intensity. How should the nurse respond to
back and see if that helps." complaints of constant cramping pain?
35 "I'll put my support stockings Which statement by a postpartal woman indicates
1. on every morning before that further teaching is not needed regarding
rising." thrombus formation?
35 Immediate nursing care of a immobilization of the area of pain
2. child with hemophilia who has
hemarthrois includes
35 Incomplete abortion at 10 In what situation would a dilation and curettage
3. weeks (D&C) be indicated?
35 Incomplete uterine relaxation Which factor is most likely to result in fetal
4. hypoxia during a dysfunctional labor?
35 increase daytime fluid intake, how to avoid urinary frequency during early
5. decrease night time fluid pregnancy
intake
35 increased hCG (human chrionic during the first trimester this hormone increases
6. gonadotropin) and can cause NV
35 increase fluid and fiber, how to manage constipation during late prego
7. exercise, stool softeners
35 ineffective role performance A 46-year-old man who has erectile dysfunction
8. related to effects of ED. (ED) tells the nurse that he decided to seek
treatment because his wife "is losing patience
with the situation." The most appropriate
nursing diagnosis for the patient is ___________
35 An infant with a congenital heart difficult feeding
9. abnormality would most likely
experience
36 An infant with tetralogy of Fallot Knee-chest
0. becomes hypercyanotic. the
nurse would place the infant in
the ________________ position.
36 Infertility in a 25-year-old couple 3. the couple's inability to conceive after 1 year
1. is defined as: of unprotected attempts
36 Initiating early and frequent Which measure may prevent mastitis in the
2. feedings breastfeeding mother?
36 Insert a urinary retention A patient with benign prostatic hyperplasia
3. catheter. (BPH) is admitted to the hospital with urinary
retention and new onset elevations in the blood
urea nitrogen (BUN) and creatinine. Which of
these prescribed therapies should the nurse
implement first?
36 inspect the placenta after The nurse knows that a measure for preventing
4. delivery. late postpartum hemorrhage is to:
36 instruct her to avoid strenuous When teaching the pregnant woman with class
5. activity. II heart disease, the nurse should:
36 In the course of an assessment ANS: A
6. interview, a female client reveals The nurse should initially self-assess personal
a history of bisexual orientation. attitudes toward homosexuality. The nurse must
Which action should the nurse be able to recognize the potential for negative
initially implement when working feelings compromising client care.
with this client? Unconditional acceptance of each individual is
an essential component of compassionate
A. Self-assess personal attitudes nursing.
toward homosexuality.
B. Review client's possible
childhood sexual abuse history.
C. Encourage discussion of
aversion to heterosexual
relationships.
D. Explore client's family history
of homosexuality.
36 intraagency transfer. A patient has been transferred out of the ICU to
7. a medical unit. A nurse has been assigned to
complete the transfer. This type of transfer is
called a:
36 introduce self and roommates. A 36-year-old schoolteacher is admitted for
8. observation and various diagnostic tests. The
initial nursing action in her admission process is
to:
36 In what babies is the moro/startle Crack/drug babies
9. reflex exaggerated?
37 In what situation do you give a ONLY for an infant or child when two rescuers
0. ratio (between chest are present to participate in CPR. All other
compressions and ventilation) of situations are 30:2!
15:2 vs. 30:2?
37 "I should call the doctor if I have Following discharge teaching for a patient who
1. any incontinence at home." has had a transurethral resection of the
prostate (TURP) for benign prostatic hyperplasia
(BPH), the nurse determines that additional
instruction is needed when the patient says,
_______________
37 is unable to coordinate sucking The preterm infant who should receive gavage
2. and swallowing. feedings instead of a bottle is the one who:
37 it could make respiratory distress The most important reason to protect the
3. syndrome worse. preterm infant from cold stress is that:
37 It is recommended that iron 12months
4. fortified formula be given to
infants through the age of
37 It's difficult to awaken a neonate normal progression into the sleep cycle.
5. 3 hours after birth. The nurse
recognizes that this behavior
indicates:
37 Just after delivery, the nurse measures a Rewarm the neonate
6. neonate's axillary temperature at 94.1 F (34.5 gradually.
C). What should the nurse do?
37 keeps the alveoli open during expiration. Decreased surfactant
7. production in the preterm
lung is a problem because
surfactant:
37 Keep underpads and linens as dry as possible What actions of the nurse
8. prevent infections in the labor
and delivery area?
37 ketones this product found in urine
9. may indicate diabetes
mellitus and hyperglycemia
38 Kick counts (fetal movement counting) 1. Mother sits quietly or lies
0. down on her L side and
counts fetal kicks for a period
o time, as instructed.
2. Instruct client to notify
physician if fewer than 10
kicks in a 12 hr. period or as
instructed by doc.
38 kitzenger birthing method of sensory
1. memory Stanislavsky method
to teach relaxation
38 The labor and delivery unit has 10 clients in The stable clients in the early
2. varying stages of labor. Staffing for the upcoming stages of labor should be
shift consists of four registered nurses (RNs) and assigned to the LPN.
one licensed practical nurse (LPN). Which client
care assignment is best?
38 LABORATORY A. Blood type and Rh factor
3. TESTS 1. ABO typing is performed to determine woman's blood type.
2. Rh typing done to determine presence or absence of Rh antigen
(Rh positive or Rh negative)
3. If client if Rh- and has a negative antibody screen, client will need
repeat antibody screens and should receive Rh immune globulin at 28
weeks' gestation.

B. Rubella titer
1. If client has negative titer, indicates susceptibility to rubella virus;
client should receive appropriate immunization postpartum.
2. Client must be using effective birth contol at time of immunization,
she must be counseled to not become pregnant for 1 to 3 months
after immunization (as specified by health care provider) and to avoid
contact w/anyone who is immunocompromised.
3. If rubella vaccine is administered at same time as Rh immune
globulin, may not be effective.

C. Hemoglobin and hematocrit levels


1. Hemoglbin and hematocrit levels will drop during gestation as
result of increased plasma volume.
2. A decrease in hemoglobin level below 10g/dL or a decrease in
hematocrit level below 30g/dL indicates anemia.

D. Papanicolaou (Pap) smear: Done during initial prenatal examination


to screen for cervical neoplasia.

E. Sexually transmitted infections


Gonorrhea
Syphilis
Herpes virus
Chlamydia

F. Sickle cell screening


1. Indicated for clients who are at risk for sickle cell disease.
2. A + test result may indicate a need for further screening.

G. Tuberculin skin test


1. Health care provider may prefer to perform this skin test after
delivery.
2. A + skin test indicates need for a chest radiograph (using
abdominal lead shield) to rule out active disease; in a pregnant client,
a chest radiograph will not be performed until after 20 weeks'
gestation (after fetal organs are formed)
3. Those who convert to positive may be referred for treatment
w/medication after delivery.

H. Hepatitis B surface antigen


1. Recommended for all women because of prevalence of sidease in
general population
2. Vaccination for hepatitis B antigen may be specifically indicated for
following:
a. Health care workers
b. Clients born in Asia, Africa, Haiti, or Pacific islands
c. Clients w/previously undiagnosed jaundice or chromic liver disease
d. IV drug abuses
e. Clients w/tattos
f. Clients w/histories of blood transfusions
g. Clients w/histories of multiple episodes of sexually transmitted
infections
h. Clients who have been previousl rejected as blood donors
i. Clients w/histories of dialysis or renal transplantation
j. Clients from households having hepatitis B-infected members or
hemodialysis clients.

I. Urinalysis and urine culture


1. A urine specimen for glucose and protein determinations s/b
obtained at every prenatal visit.
2. Glycosuria is a common result of decreased renal threshold that
occurs during pregnancy
3. If glycosuria persists, this may indicate infection
4. White blood cells in the urine may indicate infection
5. Ketonuria may result from insufficient food intake or vomiting
6. Protein levels of 2+ to 4+ in urine may indicate infection or
gestatinal hypertension.
38 lacerations of the genital tract. A steady trickle of bright red blood from the
4. vagina in the presence of a firm fundus
suggests:
38 Lack of subcutaneous fat What is a characteristic of the postterm infant
5. who weighs 7 lb, 12 oz?
38 lactase this may be added to milk or chewed before
6. ingesting milk to help breakdown lactose
38 Lactose intolerance * if consumed - abdominal distention,
7. discomfort, nausea, vomiting, cramps, loose
stools
* need to consume calcium other than diary
*Cheese, yogurt; cooked forms-custards,
fermented dairy products
*Lactaste may be prescribed before ingesting
milk products.
38 lamaze birthing method to educate about developing
8. fetus and exercises to decrease fear and pain
38 Left occiput anterior = Easy labor
9.

39 Left occiput posterior= Difficult/painful labor


0.

39 The licensed practical nurse is Bottle-feeding a 24-hour-old neonate


1. delegating responsibilities to a
certified nurse's aide on a busy
postpartum unit. Which task can
be appropriately delegated to
the nurse's aide?
39 Lie supine and relax. Which technique is least effective for the
2. woman with persistent occiput posterior
position?
39 Local tenderness, heat, and The mother-baby nurse must be able to
3. swelling recognize what sign of thrombophlebitis?
39 lordosis curvature of lumbar spine increases to
4. complensate for weight of gravid uterus
39 A low-risk client who's 6 weeks "1. establishing a schedule of prenatal visits.
5. pregnant comes to the clinic for
her first prenatal visit. At this
time, the nurse should assign
highest priority to:
39 Manually instill and then After a transurethral resection of the prostate
6. withdraw 50 mL of saline into the (TURP), a patient with continuous bladder
catheter. irrigation complains of painful bladder spasms.
The nurse observes a decrease in urine output
and clots in the urine. Which action should the
nurse take first?
39 marginal A placenta previa in which the placental edge
7. just reaches the internal os is called:
39 maternal physical exam vital signs, HW, thyroid, heart and breath
8. sounds, reproductive organs, uterus size, pelvis
exam are part of these
39 McDonalds method method use uterine size to indicate gestaional
9. age by measuring cm distane from symphysis
pubis to top of uterine fundus. fundal hieght
40 Meconium aspiration syndrome is Vocal cord examination using a laryngoscope
0. suspected in a neonate. What's
the most accurate diagnostic tool
used to confirm the diagnosis?
40 method of discharge. Nursing documentation at discharge should
1. include a:
40 mid-stream clean catch specimen what kind of urinalysis specimen is collected for
2. pregnancy
40 miscarriage, hydrocephaly, CNS what can happen to fetus with toxoplasmosis
3. lesions
40 missed abortion. An abortion in which the fetus dies but is
4. retained in the uterus is called:
40 Mitral valve prolapse What form of heart disease in women of
5. childbearing years usually has a benign effect
on pregnancy?
40 Moments after birth, a neonate of 32 keep the neonate's head in the "sniff"
6. weeks' gestation develops asphyxia. As position.
the neonatal team starts resuscitation,
the nurse must:
40 more or less calories nutritional status of underweight or
7. over weight will need this nutrient
respectivley
40 A mother is concerned that her neonate, "I understand your concern, but he
8. who was delivered without complications has stored nutrients before birth just
at 38 weeks, isn't eating enough and will for this reason."
lose too much weight. The mother states,
"He only breast-feeds for about 3 minutes
on one side." Which instruction should
the nurse provide to this mother?
40 The mother of a 3-day-old, breast-fed "It's normal for breast-fed infants to
9. infant expresses concern that her infant pass three or more loose, yellow
has had two recent diapers that stools per day."
contained a lot of loose, yellowish stool.
Which explanation by the nurse is best?
41 A multiparous woman at 39 weeks of Which client situation presents the
0. gestation who is expecting twins greatest risk for the occurrence of
hypotonic dysfunction during labor?
41 multiple count as one pregnancy if mother has twins she is gravida 1
1. para 2
41 Musulosketetal system 1. Postural changes occur as
2. increased weight of uterus causes a
forward pull of bony pelvis.
2. Ligament laxity and softening of
connective tissue are seen throughout
body as a result of an increase in
estrogen and relaxin.
41 Naegeles rule rule for EDB first day of last menstrual
3. period subtract 3 months and add 7
days
41 A neonate begins to gag and turns a Aspirate the neonate's nose and
4. dusky color. What should the nurse do mouth with a bulb syringe.
first?
41 A neonate born 30 hours after rupture of Instruct the family to keep the infant
5. membranes has an axillary temperature in the mother's room until culture
of 100.8 F (38.2 C). Which intervention results are available.
should the nurse employ?
41 A neonate must receive an eye By positioning the neonate so that the
6. preparation to prevent ophthalmia head remains still
neonatorum. How should the nurse
administer this preparation?
41 The neonate of a client with type 1 lethargy
7. diabetes mellitus is at high risk for
hypoglycemia. An initial sign the nurse
should recognize as indicating
hypoglycemia in a neonate is:
41 A neonate receives an Apgar score at 1 how well the neonate tolerated labor.
8. and 5 minutes of age. The 1-minute Apgar
score is a good indication of:
41 A neonate's pulse rate drops below 60 2. Gently shake the neonates
9. beats/minute. How should the nurse shoulders, 3. Call for Help, 1. Place the
intervene? Rank in chronological order. neonate on a firm, flat surface, 4. Us
Use all the options. the hand-tilt-chin-lift method, 5.
Assess breathing, and then give 2
slow breaths 6. Give compressions
42 never appropriate. A 90-year-old great-grandfather has
0. been hospitalized with pneumonia. It
is necessary to reorient him to his
surroundings periodically. The nurse
assisting him with his morning care
remembers that to call an older male
patient "Gramps" is:
42 a newborns total body weight is about 77%
1. _______water
42 A newly married woman comes to a ANS: A
2. gynecology clinic reporting anorexia, The nurse should expect the physician to
insomnia, and extreme dyspareunia implement a thorough physical to include a
that have affected her intimate gynecological examination to assess for any
relationship. What initial physiological causes of the client's
intervention should the nurse expect symptoms. Dyspareunia is recurrent or
a physician to implement? persistent genital pain associated with
sexual intercourse.
A. A thorough physical to include
gynecological examination
B. Referral to a sex therapist
C. Assessment of sexual history and
previous satisfaction with sexual
relationships
D. Referral to the recreational
therapist for relaxation therapy
42 A newly pregnant woman tells the "2. take the vitamin on a full stomach.
3. nurse that she hasn't been taking
her prenatal vitamins because they
make her nauseous. In addition to
telling the client how important
taking the vitamins are, the nurse
should advise her to:
42 A new mother is concerned because "Breast milk is ideal for your baby, so his
4. her breast-feeding neonate wants to stomach will digest it quickly, which
"nurse all the time." Which of the requires more feedings."
following responses is most
appropriate to address the mother's
concerns regarding her neonate's
breast-feeding behavior?
42 The night before discharge, a client "It's difficult to be a working parent, but
5. expresses guilt that she'll have to having a nanny will provide your baby with
return to work in 3 weeks and leave a consistent caregiver while you're gone."
her infant with a nanny. The client
asks the nurse for an opinion about
using a nanny. What should the
nurse say first?
42 nitrates and leukocytes what is found in urine to diagnose UTI
6.

42 Nitrazine test 1. A Nitrazine test strip is used to detect


7. presence of amniotic fluid in vaginal
secretions.
2. Vaginal secretions have a pH of 4.5 to
5.5; they do not affect yellow color of
Nitrazine strip or swab
3. Amniotic fluid has a pH of 7.0 to 7.5 and
turn yellow Nitrazine strip or swab a blue
color.
4. Interventions
a. Place client in dorsal lithotomy position
b. Touch test tape to fluid
c. Asses test tape for a blue-green, blue-
gray, or deep blue color, which indicates
that membranes are probably ruptured.
42 Nonstress test *Performed to assess placental function and
8. oxygenation
*Determines fetal well-being
*Evaluates fetal heart rate (FHR) in response to
fetal movement

Interventions:
*An external ultrasound transducer and
tocodynamometer are applied to the mother, and
a tracing of at least 20 minutes' duration is
obtained so that FHR and uterine activity can be
observed.
*Obtain a baseline BP reading, and monitor BP
frequently.
*Position mother in L lateral position to avoid vena
cava compression.
*Mother may be asked to press a button every
time she feels fetal movement. Monitor records a
mark at each point of fetal movement, and this is
used as a reference point to assess FHR response.
Results
*Reactive Nonstress Test (normal, negative)
*"Reactive" indicates a health fetus.
*The result requires 2 or more FHR accelerations
of at least 15 beats per minute and lasting at least
15 seconds from beginning of acceleration to end,
in association w/fetal movement, during a 20 min.
period.

*Nonreactive Nonstress Test (Abnormal)


*No accelerations or accelerations of less than 15
beats per minute or lasting less than 15 seconds
in duration during a 40-minute observation.

*Unsatisfactory
*Cannot be interpreted because of poor quality of
FHR tracing
42 notify the physician. A multiparous woman is admitted to the
9. postpartum unit after a rapid labor and birth of a
4000 g infant. Her fundus is boggy, lochia is
heavy, and vital signs are unchanged. The nurse
has the woman void and massages her fundus,
but her fundus remains difficult to find, and the
rubra lochia remains heavy. The nurse should:
43 Notify the physician of any What instructions should be included in the
0. increase in the amount of discharge teaching plan to assist the client in
lochia or a return to bright red recognizing early signs of complications?
bleeding.
43 notify the physician promptly. A woman who had two previous cesarean births is
1. in active labor, when she suddenly complains of
pain between her scapulae. The nurse's priority
action should be to:
43 The nurse assesses a 1-day-old Nasal flaring
2. neonate. Which finding
indicates that the neonate is
experiencing respiratory
distress?
43 The nurse caring for a 3-day- Large, immature liver
3. old neonate notices that he
looks slightly jaundiced.
Although not a normal finding,
it's an expected finding of
physiologic jaundice and is
caused by which of the
following?
43 The nurse checks a client for "4. Abdominal pain
4. signs and symptoms of ectopic
pregnancy. What is the most
common finding associated
with this antepartum
complication?
43 The nurse determines a parent High calorie, high protein
5. understands diet teaching for a
child with cystic fibrosis when
she states the child should eat
which type of diet
43 The nurse determines that a client 4. ""Return to the facility if your contractions
6. is in false labor. After obtaining become more intense.""
discharge orders from the nurse-
midwife, the nurse provides
discharge teaching to the client.
Which instruction is appropriate at
this time?
43 The nurse doing a newborn failure to pass meconium
7. assessment knows the earliest
sign of Hirschsprung's disease is
43 The nurse has a client at 30 weeks' 4. Discourage breast-feeding because HIV can
8. gestation who has tested positive be transmitted through breast milk.
for the human immunodeficiency
virus (HIV). What should the nurse
tell the client when she says that
she wants to breast-feed her
baby?
43 The nurse has been teaching a new Burping the baby frequently
9. mother how to feed her infant son
who was born with a cleft lip and
palate. Which action by the mother
would indicate that the teaching
has been successful?
44 A nurse in a prenatal clinic is 4. Hypertension, edema, proteinuria
0. assessing a 28-year-old who's 24
weeks pregnant. Which findings
would lead this nurse to suspect
that the client has mild
preeclampsia?
44 The nurse is about to give a full- Bathe the neonate only after his vital signs
1. term neonate his first bath. How have stabilized.
should the nurse proceed?
44 The nurse is assessing a 1-day-old Small size for gestational age
2. neonate whose mother smoked 1
pack of cigarettes daily during
pregnancy. Which finding is most
common in neonates whose
mothers smoked during
pregnancy?
44 A nurse is assessing a client ANS: B
3. diagnosed with pedophilia. What The nurse should identify that pedophilia is a
would differentiate this sexual sexual disorder in which individuals partake in
disorder from a sexual inappropriate sexual behaviors. Sexual
dysfunction? dysfunction involves impairment in normal
sexual response. Pedophilia involves having
A. Symptoms of sexual dysfunction sexual urges, behaviors, or sexually arousing
include inappropriate sexual fantasies involving sexual activity with a
behaviors, whereas symptoms of a prepubescent child.
sexual disorder include
impairment in normal sexual
response.
B. Symptoms of a sexual disorder
include inappropriate sexual
behaviors, whereas symptoms of
sexual dysfunction include
impairment in normal sexual
response.
C. Sexual dysfunction can be
caused by increased levels of
circulating androgens, whereas
levels of circulating androgens do
not affect sexual disorders.
D. Sexual disorders can be caused
by decreased levels of circulating
androgens, whereas levels of
circulating androgens do not affect
sexual dysfunction.
44 The nurse is assessing a neonate. Upturned nose
4. Health history findings indicate
that the mother drank 3 oz (89 ml)
or more of alcohol per day
throughout pregnancy. Which
characteristic should the nurse
expect to find in the neonate?
44 The nurse is assessing a neonate. Enlarged breast tissue
5. When maternal estrogen has been
transferred to the fetus, which
sign will the nurse see in the
neonate?
44 The nurse is assigned to care for four neonates. A 4-hour-old, 10-lb, 7-oz
6. Which neonate should she assess first? (4,734 g) boy delivered
vaginally
44 The nurse is assigned to care for two mothers and Observe standard
7. their infants. One mother tested positive for group B precautions and place
streptococcus infection and her infant has been the infant of the infected
running a low temperature of 97.4 F (36.3 C). Which mother in a warmer
precaution should the nurse take while waiting for inside the mother's
the physician to evaluate the infant? room.
44 The nurse is assisting in planning care for a 16-year- 1. Iron deficiency anemia
8. old client in the prenatal clinic. Adolescents are
prone to which complication during pregnancy?
44 The nurse is caring for a 16-year-old pregnant client. 4. A glass of orange juice
9. The client is taking an iron supplement. What should
this client drink to increase the absorption of iron?
45 The nurse is caring for a client after evacuation of a 4. use birth control for at
0. hydatidiform mole. The nurse should tell the client least 1 year.
to:
45 The nurse is caring for a client in her 34th week of 3. ""I can lie in any
1. pregnancy who wears an external monitor. Which comfortable position, but
statement by the client indicates an understanding of I should stay off my
the nurse's teaching? back.""
45 The nurse is caring for a client in the first 4 weeks of 3. Breast sensitivity
2. pregnancy. The nurse should expect to collect which
findings?
45 The nurse is caring for a client whose membranes 2. maternal vital signs
3. ruptured prematurely 12 hours ago. When collecting and fetal heart rate
data on this client, the nurse's highest priority is to (FHR).
evaluate:
45 The nurse is caring for a client who's on ritodrine 2. Crackles
4. (Yutopar) therapy to halt premature labor. What
condition indicates an adverse reaction to ritodrine
therapy?
45 The nurse is caring for a client with hyperemesis "4. On admission to the
5. gravidarum who will need close monitoring at home. facility
When should the nurse begin discharge planning?
45 The nurse is caring for a neonate with a preventing infection.
6. myelomeningocele. The priority nursing care of a
neonate with a myelomeningocele is primarily
directed toward:
45 The nurse is checking the legs of a client who's 36 2. Bilateral dependent
7. weeks pregnant. Which finding should the nurse edema
expect?
45 The nurse is collecting data on a baby boy born 3 Unequal gluteal folds
8. hours ago. Which finding would make the nurse
suspect a congenital hip dislocation?
45 The nurse is collecting data on a client who is 2. Abnormally high
9. believed to be pregnant. Which signs or symptoms human chorionic
indicate a hydatidiform mole? gonadotropin (hCG)
levels
46 The nurse is collecting data on a pregnant woman in "1. assess the client's
0. the clinic. In the course of the data collection, the readiness to stop.
nurse learns that this woman smokes one pack of
cigarettes per day. The first step the nurse should
take to help the woman stop smoking is to:
46 A nurse is counseling a client diagnosed ANS: B
1. with transgenderism. Which The nurse should identify that clients
characteristic would differentiate this diagnosed with transgenderism are
disorder from transvestic fetishism? dissatisfied with their gender, whereas
clients diagnosed with transvestic
A. Clients diagnosed with transvestic fetishism are not. Both clients
fetishism are dissatisfied with their diagnosed with transgenderism and
gender, whereas clients diagnosed with transvestic fetishism may participate in
transgenderism are not. cross-dressing.
B. Clients diagnosed with
transgenderism are dissatisfied with
their gender, whereas clients diagnosed
with transvestic fetishism are not.
C. Clients diagnosed with
transgenderism never engage in cross-
dressing, whereas clients diagnosed
with transvestic fetishism do.
D. Clients diagnosed with transvestic
fetishism never engage in cross-
dressing, whereas clients diagnosed
with transgenderism do.
46 The nurse is developing a plan of care 3. Promoting adequate hydration
2. for a client in her 34th week of gestation
who's experiencing preterm labor. What
nonpharmacologic intervention should
the plan include to halt premature
labor?
46 The nurse is developing a teaching plan 2. Between 16 and 22 weeks' gestation
3. for a client who's 2 months pregnant.
The nurse should tell the client that she
can expect to feel the fetus move at
which time?
46 The nurse is discussing posture with a 1. This position impedes blood flow to
4. client who's 18 weeks pregnant. Why the fetus.
should the nurse caution her to avoid
the supine position?
46 A nurse is obtaining a medication 3. Fifth week
5. history from a client who suspects she's
pregnant. At which week of gestation
does placental transport of substances
to and from the fetus begin?
46 The nurse is obtaining a prenatal history 2. ""Do you have any cats at home?""
6. from a client who's 8 weeks pregnant. To
help determine whether the client is at
risk for a TORCH infection, the nurse
should ask:
46 The nurse is preparing for the discharge Deficient knowledge related to apneic
7. of a neonate born 7 weeks premature. episodes
The neonate has had several apneic
episodes and will need a home apnea
monitor but will require no other
specialized care. Which nursing
diagnosis is most appropriate for the
neonate's parents?
46 A nurse is preparing to perform A 2-day-old baby who has been breast-
8. phenylketonuria (PKU) testing. Which fed
baby is ready for the nurse to test?
46 The nurse is providing care for a 3. ""Nutrition is important because
9. pregnant 16-year-old client. The client depriving your baby of nutrients can
says that she's concerned she may gain cause developmental and growth
too much weight and wants to start problems.""
dieting. The nurse should respond by
saying:
47 The nurse is providing care for a 4. be taught about diet.
0. pregnant client in her second trimester.
Glucose tolerance test results show a
blood glucose level of 160 mg/dl. The
nurse should anticipate that the client
will need to:
47 The nurse is providing care for a 1. the delivery may need to be induced
1. pregnant client with gestational early.
diabetes. The client asks the nurse if her
gestational diabetes will affect her
delivery. The nurse should know that:
47 The nurse is providing care for a 2. try to get more rest by going to bed
2. pregnant woman. The woman asks the earlier.
nurse how she can effectively deal with
her fatigue. The nurse should instruct
her to:
47 The nurse is providing dietary teaching 4. spinach and beef.
3. to a pregnant client. To help meet the
client's iron needs, the nurse should
advise her to eat:
47 The nurse is providing instruction to a 1. Fundal height of approximately 18
4. woman who is 18 weeks pregnant. Which cm 2. Quickening "
findings are expected at this time?
47 The nurse is recording an Apgar score for heart rate, respiratory effort, reflex
5. a neonate. The nurse should assess: irritability, and color.
47 The nurse is reviewing a client's prenatal "2. The client has a child with cystic
6. history. Which finding indicates a genetic fibrosis.
risk factor?
47 The nurse is reviewing a pregnant client's 4. dairy products"
7. nutritional status. To determine whether
she has an adequate intake of vitamin A,
the nurse should check her diet for
consumption of:
47 The nurse is teaching a client who's 28 2. ""I need to use insulin each day.""
8. weeks pregnant and has gestational
diabetes how to control her blood glucose
levels. Diet therapy alone has been
unsuccessful in controlling this client's
blood glucose levels, so she has started
insulin therapy. The nurse should consider
the teaching effective when the client
says:
47 The nurse is teaching a pregnant client 2. They're usually felt in the abdomen.
9. how to distinguish prelabor contractions
from true labor contractions. Which
statement about prelabor contractions is
accurate?
48 The nurse is teaching the mother of an results from exposure of an antigen
0. infant about the importance of through immunization or disease
immunizations. The nurse should teach contact.
her that active immunity:
48 A nurse is working with a client diagnosed ANS: C
1. with pedophilia. Which client outcome is During the first week of
appropriate for the nurse to expect during hospitalization, identifying triggers
the first week of hospitalization? that lead to inappropriate behaviors is
an appropriate outcome for a client
A. The client will verbalize an diagnosed with pedophilia. Pedophilia
understanding of the importance of involves intense sexual urges,
follow-up care. behaviors, or fantasies involving
B. The client will implement several sexual activity with a prepubescent
relapse-prevention strategies. child.
C. The client will identify triggers that
lead to inappropriate behaviors.
D. The client will attend aversion therapy
groups.
48 The nurse measuring an infants blood Coarctation of the aorta
2. pressure finds it is higher in the arms
than the legs. The finding is associated
with which congenital heart defect
48 The nurse observes a child who had a Notify the dr
3. tonsillectomy a few hours earlier is
swallowing frequently. What is the
appropriate action for the nurse to take?
48 The nurse observes many cuts and bruises 3. ""It's our responsibility to maintain
4. on the back, arms, and legs of a pregnant client confidentiality and to make sure
client. The client tells the nurse, "I was our clients are safe. I'd be glad to
cleaning and a box of supplies fell on me." listen if there's anything you want to
Which response by the nurse is most talk about.""
appropriate?
48 The nurse obtains the antepartum history 2. Her consumption of six to eight
5. of a client who's 6 weeks pregnant. Which cans of beer on weekends
finding should the nurse discuss with the
client first
48 The nurse places a child with croup in and Decreased mucosal swelling
6. environment of high humidity for which
effect?
48 The nurse places a neonate with decrease the serum unconjugated
7. hyperbilirubinemia under a phototherapy bilirubin level.
lamp, covering the eyes and gonads for
protection. The nurse knows that the goal
of phototherapy is to:
48 The nurse prepares a client who's 28 "1. Having the client drink orange
8. weeks pregnant for a nonstress test juice
(NST). Which intervention is likely to
stimulate fetal movements during this
test?
48 The nurse prepares to 25G, 5/8" needle
9. administer an I.M. injection of
prophylactic vitamin K to a
normal, full-term neonate.
Which needle should the nurse
use?
49 A nurse's responsibility when a monitor serum electrolyte levels
0. child is receiving diuretics is to
49 The nurse taking a history Projectile vomiting
1. form parents of an infant with
pyloric stenosis would expect
them to report the infant
experienced which sign
49 The nurse uses nitrazine paper 2. Blue
2. to determine whether a
pregnant client's membranes
have ruptured. If the
membranes have ruptured, the
paper will turn which color?
49 The nurse was giving a Tracheoesophageal atresia
3. newborn her first feeding
when the baby started
coughing and choking. This is
indicative of which condition
49 the nurse would expect the barking
4. parent of an infant with croup
to describe the infant's cough
as
49 The nurse would explain to likely to put everything in their mouths
5. parents that infants are more
susceptible to accidental
ingestion of foreign bodies
because they are
49 The nurse wraps an infant in Place the infant under a warmer and notify the
6. two blankets and places a hat registered nurse.
on his head. His axillary
temperature is 97.2 F (36.2
C). After 30 minutes, his
axillary temperature is 97.4 F
(36.3 C). How should the
licensed practical nurse (LPN)
intervene?
49 Nursing care of an adolescent allowing the adolescent to make some choices
7. with cancer who is refusing to
cooperate with treatment
should include
49 A nursing instructor is ANS: B, C, E
8. teaching about the various Categories of paraphilia include voyeurism
categories of paraphilia. Which (observing unsuspecting people who are naked,
of the following categories are dressing, or engaged in sexual activity),
correctly matched with frotteurism (touching or rubbing against a
expected behaviors? (Select all nonconsenting person), fetishism (using nonliving
that apply.) objects in sexual ways), and pedophilia (recurrent
sexual urges involving sexual activity with a
A. Exhibitionism: Mary models prepubescent child). Exhibitionism is a paraphilia
lingerie for a company that but involves the urge to show one's genitals to
specializes in home parties. unsuspecting strangers.
B. Voyeurism: John is arrested
for peering in a neighbor's
bathroom window.
C. Frotteurism: Peter enjoys
subway rush-hour female
contact that results in arousal.
D. Pedophilia: George can
experience an orgasm by
holding and feeling shoes.
E. Fetishism: Henry
masturbates into his wife's silk
panties.
49 nurture both the infant and The neonatal intensive care unit (NICU)
9. the parents to optimize environment should:
neonatal outcomes.
50 Nutrition *Average expected weight gain - 25 to 35
0. lbs.
*Increase of 300 calories per day during
pregnancy.
*Calories needs greater in last 2 trimesters
*Increase of 500 calores per day during
lactation.
*Diet rich in folic acid - prevents neural tube
defects.
*no alcohol
50 NV, epigastric discomfort, abd s/s of lactos intolerance from insufficient
1. cramping, abd distention, loose lactase enzyme
stools
50 obtaining vital signs. When a patient arrives on the nursing unit,
2. the LPN is probably responsible for:
50 odor of the lochia. If the nurse suspects a uterine infection in
3. the postpartum client, she should assess
the:
50 One minute after birth, a neonate Stimulate breathing by rubbing the
4. has an Apgar score of 7. What neonate's back.
should the nurse do?
50 On her second visit to the prenatal 4. Accepting the biological fact of pregnancy
5. facility, a client states, "I guess I
really am pregnant. I've missed two
periods now." Based on this
statement, the nurse determines
that the client has accomplished
which psychological task of
pregnancy?
50 on his admission to the hospital. An 84-year-old patient has been hospitalized
6. for 6 days with a diagnosis of a stroke. The
nurse knows planning for the patient's
dismissal should begin:
50 oral glucose tolerance test OGTT test procedure of high carb diet for 3 days
7. then 8 hour fasting with glucose levels
assessed then 100 g oral glucose and
checked again at 1,2,3 hours for gestational
diabetes
50 oral methylergonovine maleate The nurse should expect medical
8. (Methergine) for 48 hours. intervention for subinvolution to include:
50 The organ damaged by liver
9. acetaminophen poising is the
51 the organisms that cause mastitis A mother with mastitis is concerned about
0. are not passed to the milk. breastfeeding while she has an active
infection. The nurse should explain that:
51 pain The NICU nurse begins her shift by
1. assessing one of the preterm infants
assigned to her care. The infant's color is
pale, his O2 saturation has decreased, and
he is grimacing. This infant is displaying
common signs of ____________________.
51 pain management. When caring for a patient who has been
2. diagnosed with orchitis, the nurse will plan
to provide teaching about __________________
51 pallor, dizzy, clammy skin what are s/s of postural hypotension
3.

51 palpate the infant's clavicles. After a birth complicated by a shoulder


4. dystocia, the infant's Apgar scores were 7 at
1 minute and 9 at 5 minutes. The infant is
now crying vigorously. The nurse in the
birthing room should:palpate the infant's
clavicles.
51 para number of births after 20 weeks gestation
5.
51 A parent asks the nurse about the "Nighttime feedings provide the infant with
6. practice of adding rice cereal at age adequate fluid, and are typically needed
3 months to a bedtime bottle to until at least age 4 months."
"help the baby sleep through the
night." Which response accurately
addresses the parent's concern?
51 Parents of a neonate born with making sure the parents are well informed
7. severe congenital anomalies have about their infant's condition and that
requested that the staff institute a they've made an informed decision.
do-not-resituate (DNR) order. While
working with this family, the nurse
applies the ethical principle of
autonomy by:
51 The patient has had a gradual Which assessment information about a 62-year-
8. decrease in the force of his old man is most important for the nurse to
urinary stream. report to the health care provider when the
patient is asking for a prescription for
testosterone replacement therapy?
51 the patient is a human being A 52-year-old patient is being transferred to the
9. deserving dignity, courtesy, and recovery room from the surgical unit following
respect. extensive surgery as the result of trauma from
an automobile accident. The nurse assigned to
complete his care remembers that when
admitting, transferring, or discharging a
patient:
52 pelvic floor muscle exercises. Following a radical retropubic prostatectomy for
0. prostate cancer, the patient is incontinent of
urine. The nurse will plan to teach the patient
_______________
52 pelvic pain, abdominal pain, Throughout the world the rate of ectopic
1. vaginal spotting or light bleeding, pregnancy has increased dramatically over the
missed period. past 20 years. This is believed to be due
primarily to scarring of the fallopian tubes as a
result of pelvic infection, inflammation, or
surgery. The nurse who suspects that a client
has early signs of ectopic pregnancy should be
observing her for symptoms such as:
52 pH decreases with ketones and what happens to urine during pregnancy
2. glucose
52 Physically, the preschooler is built adult; slender; agile
3. more like a(n) (toddler/adult) and
is usually _________ and_______.
52 Physical therapy During discharge planning, which of the
4. following disciplines of the health care team
assists in rehabilitating and restoring a
patient's musculoskeletal function to its highest
potential?
52 PHYSIOLOGICAL MATERNAL 1. Circulating blood volume increases
5. CHANGES 2. Heart is elevated upward and to L because of
Cardiovascular system displacement of diaphragm as uterus enlarges.
3. Pulse may increase about 10 beats per
minute; blood pressure may slightly decline
during 2nd trimester
4. Increase in body's demand for iron;
physiological anemia may occur.
5. Sodium and water retention may occur,
which can lead to weight gain.
52 placental hormones are Glucose metabolism is profoundly affected
6. antagonistic to insulin, resulting during pregnancy because:
in insulin resistance.
52 Plan for retesting during the third A woman has a history of drug use and is
7. trimester. screened for hepatitis B during the first
trimester. What is an appropriate action?
52 plan nap during day and early how to avoid fatigue during early pregnancy
8. bedtime
52 position on left side how do you correct postural hypotension
9.

53 positive signs diagnostic sign of pregnancy fetal heartbeat,


0. fetal movement felt by examiner, fetus on
ultrasound
53 possible fecal contamination of A 64-year-old has a perineal radical
1. the surgical wound. prostatectomy for prostatic cancer. In the
immediate postoperative period, the nurse
establishes the nursing diagnosis of risk for
infection related to _______________
53 possible infection. A white blood cell (WBC) count of 28,000
2. cells/mm3 on the morning of the first
postpartum day indicates:
53 Postpartum blues Which condition is a transient, self-limiting
3. mood disorder that affects new mothers after
childbirth?
53 A postpartum client is receiving Avoid over-the-counter preparations containing
4. anticoagulant therapy for deep aspirin.
vein thrombophlebitis. Discharge
teaching should include which
instruction?
53 A postpartum client plans to exhibits a steady weight gain.
5. breast-feed her first child, a full-
term neonate. She asks the
nurse, "How will I know if my
baby is getting enough to eat?"
The nurse informs her that
nutritional intake is adequate if
the neonate:
53 preeclampsia this may be indicated if
6. protein is found in urine of
+1
53 the pregnancy is less than 20 weeks. Spontaneous termination
7. of a pregnancy is
considered to be an
abortion if:
53 A pregnant client arrives in the emergency 3. To relieve pressure on
8. department and states, "My baby is coming." The the umbilical cord
nurse sees a portion of the umbilical cord
protruding from the vagina. Why should the nurse
apply manual pressure to the baby's head?
53 A pregnant client asks how she can best prepare 4. ""Involve your son in
9. her 3-year-old son for the upcoming birth of a planning and preparing for
sibling. The nurse should make which suggestion? a sibling.""
54 A pregnant client asks questions about labor and "2. ""What did the
0. delivery. During the nurse's explanations, the client physician tell you about
states, "Is it true that a lot of other people are the delivery room?""
going to be in the room while I'm giving birth? I was
expecting privacy." Which response would be
appropriate for the nurse to make to this client?
54 A pregnant client asks the nurse about the 1. 1%
1. percentage of congenital anomalies caused by drug
exposure. How should the nurse respond?
54 A pregnant client asks the nurse whether she can "No, it can initiate
2. take castor oil for her constipation. How should the premature uterine
nurse respond? contractions."
54 A pregnant client comes to the clinic after missing 2. "You should mention the
3. several scheduled prenatal appointments. During homeopathic specialist to
the initial assessment, the client states, "I haven't your physician so he can
been coming to some of my appointments because I help devise the best care
go to a homeopathic specialist who takes great care plan for you."
of me." Which response by the nurse is best?
54 A pregnant client comes to the facility for her first 2. Danger signs during
4. prenatal visit. When providing teaching, the nurse pregnancy
should be sure to cover which topic?
54 A pregnant client concerned about gaining weight "4. ""I'll ask the dietitian to
5. during pregnancy questions the nurse about dietary speak with you about
intake. Which response by the nurse is best? normal weight gain during
pregnancy.""
54 A pregnant client in her second trimester reveals provide her with the
6. that she feels very anxious because of a lack of information and teach her
knowledge about giving birth. Which intervention the skills she'll need to
by the nurse is most appropriate for this client? understand and cope
during
54 A pregnant client in her second trimester visits the 2. 1 lb (0.45 kg)
7. health care practitioner for a regular prenatal
checkup. The nurse weighs the client. She then
compares the client's current and previous weights.
During the second trimester, how much weight
should the client gain per week?
54 A pregnant client in her third trimester asks why 3. This symptom is normal
8. she needs to urinate frequently, as she did during and results from the fetus
the first trimester. What should the nurse tell her? exerting pressure on the
bladder.
54 A pregnant client is brought to the emergency 3. ""The only way to safely
9. department after being an unrestrained driver in a secure yourself in a car is
motor vehicle accident. When questioned about to use a seatbelt.""
seatbelt use, the client states that she thought a
seatbelt would harm her baby. Which response by
the nurse is best?
55 A pregnant client is prescribed an iron supplement. 3. Eat a diet rich in vitamin
0. Which point should the nurse include when teaching C to promote iron
the client about taking the supplement? absorption.
55 A pregnant client's last menstrual period began on 4. July 19
1. October 12. The nurse calculates the estimated date
of delivery (EDD) as:
55 A pregnant client tells the nurse that she doesn't 2. Explaining that other
2. like milk and can't possibly drink three to four dairy products, such as
glasses per day as recommended by her physician. cheese and yogurt, may be
Which intervention by the nurse is best? substituted for milk
55 A pregnant client tells the nurse that she dreads 2. "Have you asked the
3. coming for her prenatal examinations. She states, physician why he feels it's
"When I see the physician, he constantly reminds important for him to say
me that so many women keep weight on after that to you?"
having children; it really depresses me." How
should the nurse respond?
55 A pregnant client who's 2. Insulin acting as a growth hormone on the fetus
4. diabetic is at risk for having
a large-for-gestational-age
infant because of which of
the following?
55 pregnant uterus press on postural hypotension during prego occurs due to
5. pelvic and femoral vessels limiting blood return to heart because
55 premature labor or abruptio what are two ppossible causes of abdominal pain
6. placentae during pregnancy
55 PRENATAL VISITS *Every 4 wks for first 28-32 weeks' gestation
7. *Every 2 wks from 32 to 36 weeks' gestation
*Every wk for 36 to 40 weeks
55 The Preschool age child is 3-5 years
8. in which age range?
55 presence of abdominal pain. The primary symptom present in abruptio placentae
9. that distinguishes it from placenta previa is:
56 Presenting part at a station What factor found in maternal history should alert the
0. of -3 nurse to the potential for a prolapsed umbilical cord?
56 presumptive signs subjective signs may not be associated with
1. pregnancy - amenorrhea, NV, fatigue, urinary
requency, breast change and quickening
56 preterm birth, hepititis hep A and B effects the fetus with these risks
2. intrauterine death
56 Prevent hypoglycemia by What nursing action is especially important for the
3. early and frequent SGA newborn?
feedings.
56 Previous birth of large Which factor is known to increase the risk of
4. infant gestational diabetes mellitus?
56 A primigravid client gives clean and dry the neonate's perineal area from front
5. birth to a full-term girl. to back.
When teaching the client
and her husband how to
change their neonate's
diaper, the nurse should
instruct them to:
56 probable signs objective signs may not be associated with pregnancy
6. - hegar's sign, mcdonalds sign, enlarged abdomen,
pigmentation, striae, ballottement, positive pregnancy
test, palpation of fetal outline
56 problems with Infants born between 34 0/7 and 36 6/7 weeks of
7. thermoregulation, gestation are called late preterm infants because they
hyperbilirubinemia, sepsis have many needs similar to those of preterm infants.
Because they are more stable than early preterm
infants, they may receive care that is much like that
of a full-term baby. The mother-baby or nursery nurse
knows that these babies are at increased risk for:
56 progesterone maintains endometrium , decreases uterin
8. contractility, breast ancini, relaxes smooth muscles
56 prostaglandins lipids found in female reproductive system, contribute
9. to onset of labor
57 prostate specific antigen A 51-year-old man is scheduled for an annual physical
0. (PSA) testing. exam at the outpatient clinic. The nurse will plan to
teach the patient about the purpose of
_______________________
57 protein, folate, prenatal what is added to the diet for prego
1. vitamin, minerals
57 protien what is found in urine to diagnose preeclamspia in
2. pregnancy
57 A psychiatric nursing instructor is ANS: D
3. teaching about the psychological The instructor should identify the need for
effects of the diagnosis of a further instruction if a student states that
sexually transmitted disease antibiotics can cure all STDs. STDs refer to
(STD). Which student statement infections that are contracted primarily
indicates that further instruction is through sexual activities or intimate contact.
needed? An example of an incurable STD is HIV. STDs
are at epidemic levels in the United States.
A. "STDs carry strong connotations
of illicit sex and considerable
social stigma."
B. "STDs can cause insanity."
C. "The diagnosis of HIV can
generate hopelessness and
helplessness."
D. "Antibiotics administered in the
early stages can cure all STDs."
57 PSYCHOLOGICAL MATERNAL Ambivalence
4. CHANGES 1. May occur even when the pregnancy is
planned.
2. Mother may experience a dependence-
independene conflict and ambivalence related
to role changes, changes in body image, and
fear of unknown.
3. Partner may experience ambivalence
related to assuming new role, increased
financial responsibilities, mother's physical
changes, and having to share attention with
child.

Acceptance
1. Factors that may be related to acceptance
of pregnancy are woman's readiness for
experience and her identification w/the
motherhood role.
2. Commonly occurs at 20 weeks, when
quickening is likely to occur.

Emotional lability
1. Frequency in change of emotional states or
extremes in emotional states caused by
hormone changes.
2. These emotional changes are common, but
the mother may believe that these changes
are abnormal.

Body image changes: perception of her image


during pregnancy occurs gradually and may
be either positive or negative; often
influenced by her partner.

Relationship w/fetus.
1. Woman may daydream to prepare for
motherhood and think about maternal
qualities she would like to possess.
2. Woman first accepts biological fact that she
is pregnant.
3. Woman next accepts growing fetus as
distinct from herself and as a person to
nurture.
4. Finally, woman prepares realistically for
birth and parenting of child.
57 put an undershirt on the infant in To maintain optimal thermoregulation for the
5. the incubator. premature infant, the nurse should:
57 quickening the feeling of fetal movement by mother
6. occurs between 16-18 weeks, is less accurate
EDB
57 quickening, colostrum production what is instructed to mother concerning
7. physical changes and danger signs physical changes of pregnancy
57 RBC increases what happens to RBC during pregnancy
8.

57 A recently widowed client reports a ANS: B


9. fear of intimacy due to an inability Based on the client's symptoms, the nurse
to achieve and sustain an erection. should prioritize the nursing diagnosis of
He has become isolative, has sexual dysfunction R/T dysfunctional grieving
difficulty sleeping, and has recently AEB inability to experience orgasm. The
lost weight. Which correctly written nurse should assess the client's mood and
nursing diagnosis should be level of energy because depression and
prioritized for this client? fatigue can decrease desire for participation
in sexual activity.
A. Risk for situational low self-
esteem AEB inability to achieve an
erection
B. Sexual dysfunction R/T
dysfunctional grieving AEB inability
to experience orgasm
C. Social isolation R/T low self-
esteem AEB refusing to engage in
dating activities
D. Disturbed body image R/T penile
flaccidity AEB client statements
58 Recurrent pelvic infections What data on a client's health history would
0. place her at risk for an ectopic pregnancy?
58 relaxin hormone made by corpus luteum decreases
1. uterine contractility softens cervix
58 relax smooth muscles progesterone levels do this causing
2. decreased peristalsis causing bloating,
reflux, constipation and worsens when gravid
uterus presses on intestines
58 Renal System 1. Frequency of urination occurs during the
3. first and third trimesters as a result of
pressure of enlarging uterus on bladder,
which causes decreased bladder capacity.
2. Decreased bladder tone is caused by
hormonal changes.
3. Renal function increases
4. Renal threshold for glucose may be
reduce.
58 Reposition the mother with her hips Which nursing action must be initiated first
4. higher than her head. when evidence of prolapsed cord is found?
58 Reproductive system 1. Uterus
5. a. Enlarges from 60 to 1000g.
b. Irregular contractions occur.
2. Cervix
a. Becomes shorter, more elastic, and larger
in diameter.
b. Endocervical glands secrete a thick mucus
plug, which is expelled from the canal when
dilation begins
c. Increased vascularization causes a
softening and a violet discoloration
(Chadwick's sign)
3. Ovaries: Cease ovum production
4. Vagina
a. Hypertrophy and thickening of muscle
occurs.
b. Increase in vaginal secretions occurs;
secretions are usually thick, white, and
acidic.
5. Breasts
a. Breast size increases
b. Increase in blood flow to the breasts;
swelling, tingling, and tenderness are
common.
c. Nipples ecome more pronounced and the
areolae become darder, with an increase in
ductal growth.
d. Colostrum may leak from breasts.
58 Respiratory system 1. Oxygen consumption increases as a result
6. of an increase in metabolic rate and tissue
mass.
2. Diaphragm is elevated as result of
inlarged uterus.
3. Respiratory rate remains unchanged.
4. Shortness of breath may be experienced.
58 Restore circulating blood volume by What action should be initiated to limit
7. increasing the intravenous infusion hypovolemic shock when uterine inversion
rate. occurs?
58 Retinopathy of prematurity (ROP) In caring for the preterm infant, what
8. complication is thought to be a result of high
arterial blood oxygen level?
58 Risk for infection Because of the premature infant's decreased
9. immune functioning, what nursing diagnosis
would the nurse include in a plan of care for
a premature infant?
59 Rubella For which of the infectious diseases can a
0. woman be immunized?
59 rubella droplet infection that may affect child with
1. miscarriage, congential anomalies and death
59 Safety and security A young child is being admitted to an acute
2. care setting. He has never been hospitalized
before. As the nurse enters the room, the
child begins to cry and cling to his mother.
According to Maslow's hierarchy of needs,
the child is exhibiting the need for:
59 (SELECT ALL THAT APPLY) The nurse 1. Keep the diaper below the cord. 4 Only
3. is demonstrating cord care to a sponge-bathe the neonate until the cord falls
mother of a neonate. Which actions off. 5 Clean the length of the cord with
should the nurse teach the mother alcohol several times daily.
to perform?
59 (SELECT ALL THAT APPLY) The nurse 3. The neonate's toes don't curl downward
4. is performing a neurologic when his soles are stroked. 4. The neonate
assessment on a 1-day-old neonate. doesn't respond when the nurse claps her
Which of the following findings hands above him. 6. The neonate displays
would indicate possible asphyxia in weak, ineffective sucking.
utero?
59 (SELECT ALL THAT APPLY) The nurse 1. Jaundice 2. Edema. 3. Erythema 4. Pain at
5. is planning to administer an the injection site
injection of Vitamin K
(AquaMEPHYTON) to a neonate.
After administering the drug, the
nurse should monitor the neonate
for which adverse effects?
59 (SELECT ALL THAT APPLY) The nurse (1) Clean the eyes before administration (2)
6. must administer erythromycin Put on gloves.(4) Open the eyes by putting a
ophthalmic ointment 0.5% to a thumb and finger at the corner of each lid
neonate born 30 minutes ago. How and gently pressing on the periorbital ridges.
should the nurse proceed? (5) Spread the ointment from the inner
canthus of the eye to the outer canthus. (6)
Wipe excess ointment from the eyes after
waiting 1 minute.
59 (SELECT ALL THAT APPLY) Which of 1. The neonate must void before being
7. the following instructions should the discharged home. 2. Apply petroleum jelly to
nurse provide on discharge from the the glans of the penis with each diaper
facility to the parents of a neonate change. 5 The circumcision requires care for
who has been circumcised? 2 to 4 days after discharge.
59 serologic test how is toxoplasmosis identified in pregnant
8. female
59 Sexual intercourse and Which information will the nurse include
9. masturbation will help relieve when teaching a patient who has a diagnosis
symptoms. of chronic prostatitis?
60 She must report any chest Which instructions are most important to
0. discomfort or productive cough. include in a teaching plan for a client in
early pregnancy who has class I heart
disease?
60 she should be assessed for signs of A 28-year-old primigravida is admitted to the
1. dehydration and starvation. antepartum unit with a diagnosis of
hyperemesis gravidarum. Nursing care is
based on the knowledge that:
60 Should a baby's eyes be covered for NO. The bili-light requires the eyes to be
2. both the bili-light and the fiber optic covered but the fiber optic blanket does not.
blanket?
60 Should you attempt visualization Never attempt to directly visualize epiglottis
3. of the epiglottis for a child with with tongue depressor on child with any of the
croup? croup syndromes (laryngitis, tracheitis,
epiglottitis) because it can trigger
laryngospasm.
60 Signs and symptoms that might petechiae and purpura
4. indicate that a child has
idiopathic thrombocytopenic
purpura include
60 Sit face to face with the patient, As a nurse caring for an older adult, you know
5. conversing slowly and clearly. that being hospitalized can be confusing and
upsetting. Which intervention would be most
appropriate when caring for this patient?
60 Six hours after birth, a neonate is Cool isolette walls
6. transferred to the nursery, where
the nurse intervenes to prevent
hypothermia. What is a common
source of radiant heat loss?
60 Skin 1. Dark streak down midline of abdomen may
7. appear (linea nigra)
2. Cholasma (mask of pregnancy), which is a
blotchy, brownish hyperpigmentation, may
occur over forehead, cheeks, and nose.
3. Reddish-purple stretch marks (striae) may
occur on abdomen, breasts, thighs, and upper
arms.
60 strength of the urinary stream. To determine the severity of the symptoms for a
8. patient with benign prostatic hyperplasia (BPH)
the nurse will ask the patient about
_________________
60 subjective and objective signs what signs may or may not be associated with
9. pregnancy.
61 Teach that ED is not a common A patient who has been recently diagnosed with
0. complication following a TURP. benign prostatic hyperplasia (BPH) tells the
nurse that he does not want to have a
transurethral resection of the prostate (TURP)
because he is afraid it might affect his ability to
have intercourse. Which action should the nurse
take?
61 Teach the parents signs of Overstimulation may cause increased oxygen
1. overstimulation, such as turning use in a preterm infant. Which nursing
the face away or stiffening and intervention helps to avoid this problem?
extending the extremities and
fingers.
61 Terbutaline A woman in labor at 34 weeks of gestation is
2. hospitalized and treated with intravenous
magnesium sulfate for 18 to 20 hours. When
the magnesium sulfate is discontinued, which
oral drug will probably be prescribed for at-
home continuation of the tocolytic effect?
61 Testicular self-examination Which information will the nurse plan to include
3. should be done in a warm area. when teaching a 19-year-old to perform
testicular self-examination?
61 "The nurse is caring for a 16- " 3. Avoid taking the supplement with milk. 4.
4. year-old pregnant client who is Avoid taking the supplement with antacids. 5.
taking an iron supplement. Which Avoid chewing the extended-release form of the
instruction should the nurse drug"
include when teaching the
adolescent about ferrous sulfate?
61 "The nurse is teaching a pregnant "1. Pelvic rocking may help relieve lower back
5. client about exercises that may pain. 2. Abdominal breathing lifts the abdominal
be helpful during pregnancy. wall off of the uterus. 3. Kegel exercises help
Which points should the nurse improve vaginal contractility and bladder
include in her instruction? control.
61 They are below the 10th hich is true about newborns classified as small
6. percentile on gestational growth for gestational age (SGA)?
charts.
61 They are prone to hypoglycemia, Which statement is true about large for
7. polycythemia, and birth injuries. gestational age (LGA) infants?
61 Three successive temperature What data would alert the nurse caring for an
8. measurements were 97 F, 96 F, SGA infant that additional calories may be
and 97 F. needed?
61 To ensure that the breast-feeding Provide feeding on demand.
9. neonate's weight loss remains
within the expected parameter of
5% to 10%, the nurse should
initially establish which of the
following types of feeding
schedules?
62 To minimize the amount of a drug take the medication immediately after breast-
0. received by an infant through feeding.
breast-feeding, the nurse should
tell the mother to:
62 Total placenta previa Which maternal condition always necessitates
1. delivery by cesarean section?
62 toxoplasmosis infection in mother assoc. with consuming
2. undercooked meat or poor handwashing after
handling cat liter and may pass it to fetus
through placenta causing flu symptoms
62 transrectal ultrasonography A patient has an enlarged prostate detected
3. (TRUS). by digital rectal examination (DRE) and an
elevated prostate specific antigen (PSA) level.
The nurse will anticipate that the patient will
need teaching about ______________
62 Trunk-curve reflex? Fingers down babies spine should curve like a
4. cat.
62 undescended testicles. When obtaining a focused health history for a
5. patient with possible testicular cancer, the
nurse will ask the patient about any history of
________________
62 An unmarried, unemployed young Determining whether the temperature in the
6. mother is being discharged with client's home is kept around 70 F (21.1 C)
her infant, accompanied by a during the day
friend. After referrals to social
services and home health care,
which assessment is most
important to make?
62 varicocele. A couple is seen at the infertility clinic
7. because they have not been able to conceive.
When performing a focused examination to
determine any possible causes for infertility,
the nurse will check the man for the presence
of __________________
62 varies depending on the stage of Nursing intervention for the pregnant diabetic
8. gestation. is based on the knowledge that the need for
insulin:
62 Vegan Refrains from eating animal products.
9.
Protein - from whole grains, legumes, seeds,
nuts, and vegetables to provide essential
amino acids.
63 visual disturbance, edema hands what are three danger signs of HTN or
0. and face, severe HA preeclampsia
63 water retention increased sex hormones and decreased serum
1. protein causes this in prego
63 WBC increase slightly what happens to WBC during pregnancy
2.
63 WBC or nitrates these may be found in urine to show UTI may
3. show risk of pre-term labor
63 well-fitting supportive bra how to manage breast tenderness during
4. early pregnancy
63 What activities should people with High impact sports. Water aerobic and
5. osteoporosis avoid? swimming is great!!!
63 What are age approps toys for child imiative of adult patterns and roles.
6. preschoolers aged 3-5?
offer playground materials, housekeeping toys
(play kitchen), coloring books, tricycles
w/helmet
63 What are age approp toys for construction toys, sewing tools, table games,
7. school-age kids 6-12? sports, repair building, chores, mechanical
activities
63 What are age-approp toys for push-pull toys, low rocking horses, dolls,
8. toddlers, 12-36 mo? stuffed animals

like toys that move b/c they are big explorers


63 What are appropriate toys for birth mobiles 8-10 in from face
9. - 2 months
64 What are approps toys for 2-4 mo? rattle, cradle gym
0.

64 What are approps toys brightly colored toys (sm enough to grasp, large enough
1. for 4-6 mo? for safety)
64 What are approps toys Large toys with bright colors, movable parts, and
2. for 6-9 mo? noisemakers
64 What are approps toys books w/large pics, large push-pull toys, teddy bears
3. for 9-12 mo?
64 What are causes of HTN, trauma, diabetes, cocaine use, alcohol abuse,
4. abruptio placentae? cigarette smoking, PROM, or an abnormally short umbilical
cord.
64 What are Contractions (increase in frequency, duration, and
5. characteristics of TRUE intensity), cervix is dilated and effaced, the presenting
LABOR? part is descending into the birth canal, a mucous plug is
present, when walking pain intensifies, and membranes
may rupture.
64 WHAT ARE EARLY HEAD COMPRESSION (NOT OMINOUS=NO INTERVENTION
6. DECELERATIONS REQUIRED).
INDICATIVE OF?
64 What are important ASSESS for hypotension at least every 5 mins after block is
7. considerations when an started, YOU MUST HAVE AN IV, AND IF HYPOTENSION OR
epidural block is being SIGNS OF IMPAIRED PLACENTAL PERFUSION OCCUR=
administered? INCREASE THE RATE OF IV FLUID/ TURN WOMAN ON HER
LEFT SIDE/ AND ADMINISTER O2 VIA FACE MASK (8-10 L).
64 WHAT ARE LATE HR DECREASES AFTER PEAK OF CONTRACTION AND
8. DECELERATIONS? RECOVERS AFTER CONTRACTION ENDS. (BEING LATE IS
BAD).
64 WHAT ARE LATE FETAL STRESS AND HYPOXIA OR DEFICIENT PLACENTAL
9. DECELERATIONS PERFUSION (CHANGE MATERNAL POSITION AND
INDICATIVE OF? DISCONTINUE OXYTOCIN (PITOCIN)).
INTERVENTIONS?
65 What are potential enuresis, encopresis, head lice
0. probs for school age
kids?
65 What are risk factors Post maturity, cold stress, and maternal diabetes.
1. for hypoglycemia in a
newborn?
65 What are signs of (40-60 mg/dL =normal in infant) Jitteriness, low
2. hypoglycemia in a temperature, and lethargy.
newborn?
65 What are some common full or bulging fontanelles, macrocephaly (large head),
3. findings among infants poor feeding, vomiting, and irritability.
with increased ICP?
65 What are some common Alendronate (FOSAMAX), Ibandronate (Boniva),
4. osteoporosis Risedronate (Actonel), Teriparatide (Forteo), and
medications? Raloxifene (Evista)
65 What are some findings oliguria
5. associated with acute edema (periorbital and peripheral)
glomerulonephritis? hematuria ("smoky" or "tea-colored" urine)
mild hypertension
lethargy
moderate proteinuria
loss of appetite
65 What are S&S of Hot flashes, night sweats, sensitive and moody, increase in
6. menopause? vaginal bleeding during menstrual period times, and
anemia.
65 What are the #1 cause MVA, scalding and posioning
7. of death for toddlers?
65 What are the 3 Rubra (dark red, 1st 3 days), Serosa (pinkish, sero-sanguinous, 3-
8. types of lochia 10 days), and alba (creamy or yellowish after the 10th day and
and when do may last a week or two).
they occur?
65 What are the 4 First (stage of cervical dilation), second (stage of expulsion), third
9. stages of labor? (placental stage), and fourth (maternal homeostatic stabilization).
66 What are the 4 Evaporation (moisture on skin), CONDUCTION (CONTACT WITH
0. types of heat COLD OBJECTS-SCALE OR COLD HANDS), Convection (drafts from
loss in an infant? open doors or air conditioning), and Radiation (by cold surfaces,
so from baby to crib and out window).
66 What are the Position head slightly lower than rest of the body, compress bulb
1. correct steps for before inserting into mouth, insert into side of mouth, release
using a bulb slowly, empty, then suction the nose AFTER the mouth
syringe?
66 What are the COLOR, HR, REFLEX IRRITABILITY, MUSCLE TONE, RESPIRATORY
2. different criteria EFFORT.
for APGAR
SCORING?
66 What are the T =Tetralogy of fallot
3. four "t's" in T =Truncus arteriosus
terms of cyanotic T =Transportation of the great vessels
defects in T =Tricuspid atresia
pediatrics?
66 What are the Crush fractures especially at T8 and below. Hip and Colles' (distal
4. most common fracture of radius in forearm) fractures are MOST COMMON.
osteoporosis
associated
injuries?
66 What are the Fetal anoxia or death, and maternal shock, hemorrhage, or death.
5. risks of abruptio (an emergency c-section is usually indicated).
placentae?
66 What are the PITOCIN= pressure elevated, intake and output bc holding fluid,
6. side effects of TETANIC CONTRACTIONS, oxygen decrease in fetus, cardiac
pitocin? arrhythmia, irregularity in fetal heart rate, and N&V. Imagine Pitty
Pitocin in a row boat looking into a PIT watching the Tetanic sink
into the ocean (ocin).
66 What are the Sick to stomach, cramps, pallor, slight fever, bright red vaginal
7. S&S of a bleeding, excruciating shooting pain in right shoulder which
ruptured ectopic worsens with inhaling, tachycardia, and hypotension. (if states last
pregnancy? menstrual period 8 weeks ago with spotting can suspect)
66 What are the BURP= Blood pressure decreased, urine output decreased (<30),
8. S&S of MgS04 respirations <12, and patella reflex absent. D/C mag sulfate if see
toxicity? these signs.
Treatment?
66 What are the Hypertension, Edema, Proteinuria, Midepigastic pain d/t liver
9. S&S of severe edema, increased ICP (cerebral), visual changes, vomiting, and
preeclampsia? oliguria.
67 What are the BUBBLE HE= BREAST (mastitis), UTERUS (fundus should be firm
0. steps of a and midline several fingers above umbilicus about the size of a
postpartum GRAPEFRUIT), BLADDER (distension, diuresis during 1st 2 days
assessment? post), BOWEL (laxative need 3rd day should be normal), LOCHIA
(no odor & appropriate type), EPIOSIOTOMY (infection & healing),
HOMANS SIGN (thrombophlebitis), AND EMOTIONAL (Support).
67 What are the Hypertension (sys increase of 30 and diastolic of 15), Edema
1. symptoms of (generalized & periorbital, gain 1lb/wk in 3rd T), and Proteinuria
mild (1gm/24 hrs, 1+ protein).
preeclampsia?
67 What are the three different 0 (POOR), 1, and 2 (BETTER). LOWEST SCORE IS A
2. ratings for APGAR CRITERIA? 0 AND HIGHEST IS 10.
67 What are the three parts of Latent (<4cm, if give drugs at this time will stop
3. the first stage of labor? the labor), Active (Give drugs), and Transitional
(>8cm)
67 What are the three types of Nonstress test, contraction stress test, and the
4. antepartum fetal surveillance? biophysical profile. The goal is to prevent perinatal
morbidity and mortality, determine fetal health,
and guide interventions.
67 What are the three types of Epidural block, subarachnoid block, and general
5. Pharmacologic intrapartum anesthesia.
pain management methods?
67 What are the treatments for a Bili-light or fiber optic blanket, monitor skin temp,
6. baby with elevated bilirubin? check for dehydration, and increased formula or
breast feeding to increase excretion of bilirubin in
stool.
67 What are the two back Dorsal Kyphosis (hump back) and cervical lordosis
7. alterations that osteoporosis (Lower L-shaped curve to back)
can cause?
67 WHAT ARE VARIABLE TRANSIENT DECREASE IN HR ANY TIME DURING
8. DECELERATIONS? CONTRACTION.
67 WHAT ARE VARIABLE CORD COMPRESSION (CHANGE MATERNAL
9. DECELERATIONS INDICATIVE POSITION-KNEE CHEST/ASS IN AIR). IF CORD IS
OF? INTERVENTIONS? PROTRUDING NEVER TOUCH.
68 What can elevated levels of OPEN NEURAL TUBE DEFECTS, ABDOMINAL WALL
0. AFP indicate? DEFECTS (OMPHALOCELE, GASTROSCHISIS).
68 What can low levels of AFP CHROMOSOMAL TRISOMIES (DOWNS SYNDROME).
1. indicate?
68 What can meconium after 36 Megacolon with absence of peristalsis.
2. hours indicate?
68 What causes preeclampsia to The presence of seizures.
3. become eclampsia?
68 What causes respiratory Not enough surfactant in the lungs
4. distress in infants?
68 What change in hearing will a Locates sound accurately
5. baby develop by 7 months?
68 What characterizes the Definition: massive proteinuria, hypoalbuminemia,
6. presentation of nephrotic hyperlipemia and edema
syndrome?
68 What classifies as eclampsia? Pregnancy induced hypertension after a seizure.
7.

68 What defines an "infant"? 1 year or less. 1 year->puberty=child


8.

68 What differences will a post They are old so wrinkly with sugar problems.
9. term baby have?
69 What discomforts are common Morning sickness, fatigue, and urinary frequency.
0. during the 1st trimester of
pregnancy?
69 What discomforts are common Heartburn and constipation (no laxatives!)
1. during the 2nd trimester of
pregnancy?
69 What discomforts are common Urinary frequency (end of 3rd), heartburn,
2. during the 3rd trimester of constipation, backache, insomnia, supine
pregnancy? hypotensive syndrome (left side lying), and leg
cramps.
69 What does a nonreactive Nonreactive, nonstress, not good. It means that the
3. nonstress test indicate? baby probably can't handle a vaginal birth (ex: cord
around baby's neck=variable decelerations and
emergency c-section).
69 What does a reactive Reactive Response is Real good. Heart rate is
4. nonstress test indicate? elevated in response to stress of activity. Baby can
handle vaginal delivery.
69 What does the 1st stage of Begins with onset of regular contractions and ends
5. labor (cervical dilation stage) with complete dilation.
begin and end with?
69 What does the 2nd stage of Begins with complete cervical dilation and ends
6. labor (expulsion stage) begin with delivery of the fetus.
and end with?
69 What does the 3rd stage of Begins immediately after fetus is born and ends
7. labor (placental stage) begin when the placenta is delivered.
and end with?
69 What does the 4th Begins after the delivery of the placenta and continues for
8. stage of labor one hour after delivery.
(maternal
homeopathic
stabilization stage)
begin and end with?
69 What does the Types of shock are classified according to etiology: CHANS
9. acronym "CHANS" C ardiogenic - caused by inability of the heart to pump blood
refer to? effectively (due to heart attack or heart failure)
H ypovolemic - caused by inadequate blood volume (due to
bleeding or dehydration)
A naphylactic - caused by allergic reaction
N eurogenic - caused by damage to nervous system (due to
extreme emotional upset due to personal tragedy or disaster)
S eptic - caused by systemic infection
70 What do the Resemble copious amounts of fine white frothy bubbles of
0. secretions produced mucus in the mouth and nose. Neonates with TEF develop
by an infant with a these secretions, which recur despite suctioning. They may
transesophogeal also develop rattling respiration and episodes of coughing,
fistula look like? choking, and cyanosis.
70 What drug can be Betamethasone. It takes 24 hours to start working and may
1. used to stimulate increase mom's blood sugar.
fetal lung maturity in
a fetus?
Considerations?
70 What general safety Car seats should always be in the center of the backseat of
2. precautions should the car, always keep on hand on baby during a bath, never
be implemented with shake or jiggle your baby vigorously or throw him/her in the
infants? air, always store medications and household products out of
reach, and never leave your baby unattended on surfaces
above the floor such as changing tables (SAFEST PLACE FOR
BABY IS UNDER LEGS WHILE COOKING BC CAN'T ROLL OFF
OF ANYTHING), no chains/crosses around babies neck
(Hispanics-tape cross to HOB or chart but not wear), and NO
METAL TOYS.
70 What has to be Absence of periods for 12 months in a row. If both ovaries are
3. existent for it to be removed=instant menopause.
considered sterile?
70 What hormone do HcG human chorionic gonadatropin
4. home pregnancy
tests look for
indicating
pregnancy?
70 What info would be Identify early signs of an asthma attack
5. included in a
teaching plan for a
child with asthma
70 What interventions Prehydration, displace uterus to left, observe BP and fetal HR,
6. should be completed and observe for bladder distension.
with antepartal pain
management?
70 What is a baby at risk Kernicterus (brain damage)
7. for if bilirubin levels
become too elevated?
70 What is a biophysical It is a prenatal ultrasound of fetal well being often done with
8. profile? a non-stress test. SINCE IT IS AN ULTRASOUND A FULL
BLADDER IS REQUIRED.
70 What is a painless bleeding around the 7th month
9. characteristic sign of
placenta previa?
71 What is a common Watery stools
0. adverse effect of
phototherapy?
71 What is a common Calcium supplement, Fosomax or Actonel, and Estrogen.
1. med regimen for a
person with
osteoporosis?
71 What is a condition Extrusion reflex has disappeared (4-5 months). Babies don't
2. needed before chew!
introduction of solid
foods is allowed?
71 What is a cremateric Touch the inner thigh of a boy and his scrotum will elevate
3. reflex? (+ for life).
71 What is a hydatidiform It is a rare mass or growth that forms inside the uterus at
4. mole? Treatment? the beginning of pregnancy. It is a presumptive sign of
pregnancy. A D&C will be performed and HcG levels will be
taken every month for a year to determine the tissue is
gone. The patient shouldn't get pregnant again for another
year and they are at increased risk for Ca. (Chemo is given
for HcG levels that indicate choriocarcinoma).
71 What is ALPHA- Alpha-fetoprotein is the predominant protein in fetal plasma.
5. FETOPROTEIN It is excreted into the amniotic fluid thru fetal urine. Some
SCREENING? cross into maternal circulation so AFP CAN BE MEASURED IN
BOTH THE MATERNAL SERUM AND IN THE AMNIOTIC FLUID.
Abnormal concentrations are associated with fetal
anomalies. It is optimally done during 15 and 18 weeks of
gestation. If abnormal ultrasound follows and if still
unexplained amniocentesis is next.
71 What is a Surgical procedure in which a small incision is created in the
6. Myringotomy? eardrum to relieve pressure caused by excessive build-up of
fluid or to drain pul from the middle ear.
71 What is an acme? A peak of something
7.

71 WHAT IS AN EARLY HR SLOWS BEFORE PEAK OF CONTRACTION AND RECOVERS


8. DECELERATION? BY END OF CONTRACTION. (BEING EARLY IS GOOD)
71 What is an important MUST HAVE AN IV and care for hypotension or signs of
9. consideration when a impaired placental perfusion. (a subarachnoid block is done
subarachnoid block is just before birth so it doesn't provide pain relief for
being administered? labor).AND IF HYPOTENSION OR SIGNS OF IMPAIRED
PLACENTAL PERFUSION OCCUR= INCREASE THE RATE OF IV
FLUID/ TURN WOMAN ON HER LEFT SIDE/ AND ADMINISTER
O2 VIA FACE MASK (8-10 L).
72 What is a normal 25-35 lbs. is normal for entire pregnancy. Gain 3 lbs. during
0. weight gain during 1st trimester, 0.7 lb/week or half of expected wt gain during
pregnancy? By 2nd trimester, and 1 lb/wk during the 3rd trimester. Calories
trimester? Calorie should be increased by 300/day while pregnant and 500
increase? while nursing.
72 What is beneficial It is a simple blood test and is least invasive.
1. about maternal serum
AFP evaluation?
72 What is characteristic Tender abdomen c pain/bleeding
2. of abruptio placentae?
72 What is The villi are made up of cells that reflect the genetic makeup of
3. CHORIONIC VILLI the fetus. The samples are used to diagnose abnormalities (it
SAMPLING? doesn't assess for neural tube defects bc those require amniotic
fluid). It is recommended only for women who are at risk for fetal
anomalies. Use is restricted bc of reported complications
(spotting, bleeding, uterine cramping, and fetal loss). Essentially
taking a piece of the baby so try to avoid if possible.
72 What is <120 bpm, indicative of fetal hypoxia or stress and maternal
4. considered fetal hypotension after epidural initiation (place client on left side,
bradycardia? increase fluids, and stop pitocin).
Indicative of?
Intervention?
72 What is >160 bpm, indicative of maternal or fetal infection or fetal
5. considered hypoxia (an ominous sign).
tachycardia for a
fetus? Indicative
of what?
72 What is a blood product; given for blood transfusions for those lacking
6. cryoprecipitate? clotting factors
72 What is decreased A smooth baseline. The baseline should vary by 10 to 15 beats
7. variability? every minute. The loss of short term variability can be ominous
where as long term loss (20 to 40 mins) is probably not
significant and can indicate a sleep cycle.
72 What is decreased Fetal sleep cycle, depressant drugs, hypoxia, or CNS anomalies.
8. variability
indicative of?
72 What is done for Purse string procedure
9. an incompetent
cervix?
73 What is done for Since they experience polyuria, polydypsia, and polyphagia ICE
0. GDM patients CHIPS would be good and frequent blood sugar checks.
during any
trimester?
73 What is enuresis? Definition: intentional or involuntary passage of urine in a child
1. who is beyond age when bladder control should be normally
controlled (occurs twice a week for 3 months in child older than
5)
73 What is expected walks alone, builds 2 block tower, throws obj, grasps spoor,
2. for a 15 month names commonplace obj
old?
73 What is expected copies a circle, builds a bridge w/3 cubes, less negative/dec
3. of a 3 yr old? temper tantrums, learns from experience, rides tricycle, walks
backwards & downstairs, undresses without help, 900 word
vocab USES SENTENCES, may invent an imaginary friend
73 What is given to Magnesium sulfate
4. prevent or treat
PIH seizures?
73 What is hemolytic an acute disorder shows the following "triad":
5. uremic syndrome? 1 hemolytic anemia
2 thrombocytopenia
3 acute renal failure
73 What is A disorder of the abdomen that occurs when all or part of the
6. Hirschprung's large intestine has no nerves and therefore cannot function. The
disease??? affected segment of the colon cannot contract and therefore
pass stool through the colon, creating an obstruction.
73 What is Leopolds It is used to determine the baby's position and know where to
7. maneuver? place the stethescope.
73 What is Nagel's rule? It is used to determine an estimate of the due date. It says
8. to count back 3 months and add 7 days from your last
menstrual period. Watch for a post due date question (it
may ask about post not the actual estimated due date).
73 What is pitocin used To induce labor.
9. for?
74 What is the antidote Calcium gluconate
0. for MgS04?
74 What is the best liquid Apple Juice
1. for the nurse to give to
a child who has had a
tonsillectomy?
74 What is the best feed smaller amounts more frequently
2. method of feeding an
infant in congestive
heart failure from a
large ventricular septal
defect
74 What is the best test The lecithin/sphingomyelin (L/S) ratio is best known for
3. for determining fetal estimating fetal lung maturity.
lung maturity?
74 What is the child's Infant/toddler (0-2 yrs)= loss of caretaker (bottle/milk),
4. concept of death over early childhood (2-7 yrs)= reversible/temporary "sleeping",
time? Middle childhood/school age (7-12)= irreversible but
doesn't apply to them, and Adolescence (12+)=
understand death is inevitable and irreversible but seems
like a distant event.
74 What is the correct Head slightly lower than the rest of the body
5. position for using a (trendelenburg=dependent)
bulb syringe on an
infant?
74 What is the Denver II screen kids from birth to 6 yr in 4 skill areas:
6. test?
1. personal-social
2. fine motor
3. language
4. gross motor
74 What is the difference adult: give 1 breath every 5 to 6 seconds (approximately 10
7. in terms of recovery to 12 breaths/minute) and recheck pulse every 2 minutes
breaths for an adult vs.
an infant? child and infant: give 1 breath every 3 to 5 seconds
(approximately 12 to 20 breaths/minute) and recheck pulse
every 2 minutes
74 What is the major aspiration--- watch out for hot dogs, candy
8. cause of death in
infants? also falls, car seats and bursn
74 What is the most ventricular septal defect
9. common congenital
heart defect occurring
in children
75 What is the palmar Hypothyroidism (cretinism) will sleep 23 hours.
0. grasp used to check
for?
75 What is the plantar When foot is touched toes spread out and foot twists
1. grasp (Babinski) inward. Gone by 9-12 months.
reflex?
75 What is the primary 3. Deficient fluid volume
2. nursing diagnosis for a
client with a ruptured
ectopic pregnancy?
75 What is the proper way Put finger in the corner of the mouth to break suction.
3. to help the baby DON'T pull or can cause trauma to breast.
release suction on the
breast?
75 What is the purpose of To observe the RESPONSE of the fetal heart rate to the
4. a nonstress test? stress of activity.
75 What is the red reflex? Red pupils when picture is taken.
5.

75 What is the Bed Rest, increased protein diet, BP meds, monitor for seizures,
6. treatment for and MgS04 if needed.
pregnancy induced
HTN (PIH)?
75 What is the triad of Edema, increased BP, and Proteinuria
7. symptoms in
hypertensive
disorders?
75 What is TRIPLE It is when two other markers, human chorionic gonadotropin
8. MARKER (hCG) and unconjugated estriol have been added to an AFP
SCREENING? evaluation. This triple marker screening has been found to
increase the detection of trisomy 18 and 21. Maternal serum
samples are taken between 15 and 22 weeks gestation. If all
three markers are low= positive result (amniocentesis would
follow).
75 What is Truncus rare type of heart disease that occurs at birth (congenital heart
9. arteriosus? disease), in which a single blood vessel (truncus arteriosus)
comes out of the right and left ventricles, instead of the normal
two vessels
76 What is typical of a climbs and jumps well, laces shoes, brushes teeth, 1500 word
0. 4 yr old? vocab, skips and hops on one foot, throws overhead
76 What is typical of a runs well, jumps rope, 2100 word vocab, begins cooperative
1. 5 year old? play, gender specific paly (dress up, imitation), skips on
alterante feet
76 What is Wilms rare type of kidney cancer; a nephroblastoma
2. tumor?
76 What key "4. Overcoming fears she may have about the unknown, loss of
3. psychosocial tasks control, and death
must a woman
accomplish during
the third trimester?
76 What kind of diet no added salt but high in potassium and protein
4. should patients
with nephrotic
syndrome be on?
76 What kind of stool Within 48 hours after birth transitional stool is passed
5. follows meconium (meconium combined with greenish/brown milk stools).
and when?
76 WHAT KIND OF MECONIUM (THICK, BLACK/GREEN, TENACIOUS).
6. STOOL IS PASSED
WITHIN 36 HOURS
OF BIRTH?
76 What kind of stool Milk stools (yellow, green-yellow).
7. occurs 4-5 days
after birth?
76 What L/S ratio At 30 weeks the sphingomyelin plateaus but the lecithin
8. indicates adequate continues to rise. A ratio greater than 2:1 generally indicates
maturity? that surfactant is adequate and the lungs are mature.
76 What might cause Increased brown fat present
9. babies to not shiver
when cold?
77 What needs to be Blood (ABO) bc the patient will lose an increased amount of
0. available when a blood during delivery.
patient with
preeclampsia
delivers and why?
77 What needs to be Observe for bleeding with FIRST URINATION, Change the
1. taught to new dressing with vaseline gauze only, NO ICE on penis, and apply
parents about diaper loosely to prevent irritation.
circumcision care?
77 What needs to be Keep area clean and dry, diaper below cord to prevent
2. taught to new irritation, cleanse with alcohol, signs of infection are redness,
parents about cord drainage, swelling, and odor.
care?
77 What needs to be taught to Encourage parents to hold/sing to their infant,
3. new parents about preterm promote skin to skin contact, feedings promote
bonding? bonding, and make sure focused when bonding (ex:
NO TV while breast feeding.
77 What pregnancy Ectopic pregnancy, abortion, incompetent cervix,
4. complications are of concern and hyperemesis gravidarum)
in the 1st trimester?
77 What pregnancy Placenta previa, abruptio placenta, and
5. complications are of concern hypertensive disorders.
in the 3rd trimester?
77 What preventative measures No sodium intake, I&O, and elevate extremities.
6. are taken for hypertensive
disorders during pregnancy?
77 What reflexes is a baby born Moro, palmar grasp, plantar grasp (babinski),
7. with? sucking, rooting, stepping/walking, placing, tonic
neck (fencing), and swimming.
77 Whats common for an 18 anterior fontanelle usually closed
8. month old?
walks backwards, climbs stairs

scribbles

builds 3 block tower

thumb sucking

temper tantrum
77 What should a baby As little clothes as possible (diaper) and eye
9. undergoing bili light therapy coverings.
be wearing?
78 What should an umbilical cord AVA (two arteries and a vein) if patient doesn't have
0. contain? AVA then they are priority.
78 What should a pregnant Expose to air after a feeding
1. woman be taught to do to
toughen nipples?
78 What should a pregnant Legumes
2. woman who is constipated be
told to eat?
78 What should a woman do if Apply ice pack to breasts and wear a tight bra
3. not breast feeding their baby
to prevent engorgement?
78 What should a woman who is Apply heating pads to the breasts
4. having trouble producing milk
do?
78 What should be avoided if you Carbonated beverages, excess salt, red meat,
5. have osteoporosis? excess caffeine, and excess alcohol bc can slow
body's calcium absorption.
78 What should be done 1st? Place in left side lying position and administer 4 L
6. nasal cannula.
78 What should be done if a Nothing it is an expected outcome and means that
7. woman is cramping after the drug is working.
being given methergine?
78 What should be done if STOP THE PITOCIN. If contractions are lasting that
8. contractions are lasting 1 1/2 long then the baby isn't getting oxygen during that
mins or if side effects are time.
presenting?
78 What should be done Notify MD, insert a large bore IV, infuse NaCl at
9. immediately for a women with 150/hr, ask about food incase surgery, blood and
a ruptured ectopic urine tests, vital signs, morphine, and surgery prep.
pregnancy?
79 What should be done in Keep her NPO for surgery, administer supplemental
0. preparation for an emergency oxygen, and start a large bore IV line.
c-section as a result of an
abruptio placentae?
79 What should be done when Document contractions: counting from beginning of
1. pitocin is being administered? contraction to beginning of the next and add to the
rest of the period.
79 What should be looked out for Pain, redness, swelling in the legs, unexplained
2. when taking Evista (selective uterine bleeding, or breast abnormalities.
estrogen receptor
modulator)?
79 What should be Sit or lie down if the drug causes a fast heartbeat,
3. looked out for when lightheadedness, or dizziness.
taking Forteo (a
parathyroid
hormone)?
79 What should be Trouble swallowing, chest pain, or severe/worsening of heart
4. looked out for when burn.
taking Fosamax,
Actonel, or Boniva
(bisphosphates)?
79 What should be All caregivers should learn how to use the home apnea
5. taught to a family monitor and CPR!!! That the alarm is set for apnea >20
preparing to take seconds, and that EXTENSION CORDS/ ADAPTERS/ CHEATER
their infant home on PLUGS ARE NOT TO BE USED WITH THE MONITOR (or any
a home apnea medical equipment).
monitor?
79 What should be Continue to breast feed on both breasts.
6. taught to a woman
with mastitis?
79 What should be Squatting or pelvic rocking
7. taught to pregnant
women who want to
avoid an episiotomy?
79 What should be told Intercourse must stop after discharge and should not be
8. to a woman who asks reinstated for 6 weeks.
about intercourse one
day after giving birth?
79 What should be "Too much amniotic fluid". There is an increased risk for
9. watched with hemorrhage.
hydramnios?
80 What should you do Place sheets or newspapers under mother, don gloves, WITH
0. in the case of an ONE HAND PRESS A CLEAN CLOTH AGAINST THE PERINEUM
imminent (right now) FOR SUPPORT AND INSTRUCT HER TO PANT. AS THE BABY'S
vaginal birth? HEAD EMERGES, PLACE THE OTHER HAND ON IT GENTLY BUT
FIRMLY TO PREVENT AN EXPLOSIVE DELIVERY (CAN CAUSE
TEARING AND RAPID PRESSURE CHANGES IN BABYS SKULL)
AND SUPPORT THE CHILD!!!!!!. Check umbilical cord, place
hands on either side of head and deliver body, after
delivered and the baby is crying dry him/her thoroughly,
including head, and wrap him in towel or blanket (heat loss is
dangerous). If she can start breast feeding immediately it will
stimulate oxytocin release and uterine contractions to help
deliver the placenta and reduce bleeding.
80 What sign will the Their lips will flange
1. baby show that they
are correctly latched
to the breast?
80 What S&S will a pregnant Tight and itchy=(hematoma)
2. women who had a forceps
assisted birth present with?
80 Whats typical for a 10-12 yr uses telephone, develops interest for opposite sex,
3. old? loves conversation, raises pets
80 Whats typical for a 24 early efforts at jumping, builds 5-6 block tower; 300
4. month old? word vocabulary; obeys easy commands
80 Whats typical for a 30 walks on tiptoe, builds 7-8 block tower, stands on one
5. month old? foot, has sphincter control for toilet training
80 Whats typical of a 6 yr old? self-centered, show off, rude; extreme sensitivity to
6. criticism, begins losing temp teeth, perm teeth start to
come in, ties knots
80 Whats typical of a 7 year team games/sports/organizations, develops concept of
7. old? time, prefers playing with same sex
80 Whats typical of a 9 yr old Skillful manual work possible, conflict btw adult
8. authorities and peer group
80 Whats typical of an 8 yr actively seeks out friends, eye development complete,
9. old? writing replaces printing
81 What things can be done to Wear cotton clothes, wear natural fibers, dress in
0. reduce hot flashes? layers, herb: black cohosh (helps decrease), avoiding
dairy product and meat (they increase HFs).
81 What things place someone Woman, thin, white or Asian, family Hx, menopause b4
1. at risk for developing age 45, smoking, drink more than 2 drinks per day,
osteoporosis? not getting enough exercise, excess coffee and
carbonated beverages, and lack of calcium and
vitamin D.
81 What treatment should be Surgical treatment and IV FLUID
2. done for ectopic
pregnancy?
81 What type of pregnancy Hyperemesis Gravidarum
3. complication patient could
be assigned to a non-OB
nurse?
81 What types of smells/tastes Sweet smells and tastes is prefered
4. do babies prefer?
81 What usually causes the The most common form of croup is acute
5. croup, and how do we treat laryngotracheobronchitis or viral croup, which is an
it? infection of both the upper and lower respiratory
tracts. The classic "barky" harsh cough, stridor, and
fever are treated with antipyretics and cool air/mist.
81 What vaccines should Tetanus (q 10 yrs), Influenza (q yr @50), Pneumonia
6. adults be getting? (1x @65), Shingles (1x @60), MMR (2 doses between
19 & 49), and Varicella (2 doses between 19 & 49 if no
immunity).
81 What will a woman on Sleepy bc it is a nervous system depressant.
7. MgS04 feel like?
81 What will a woman with Sharp abdominal pains and vaginal spotting (or large
8. abruptio placentae present amounts of vaginal bleeding), tender and rigid
with? abdomen, pale, anxiousness, diaphoretic, tachycardia,
increased respirations, and increased temperature.
81 What will happen to a It will open slowly or stay folded in very premature
9. preterm (before 37 weeks) infants.
infants ear if folded
forward?
82 What will lab tests show HELLP= Hemolysis, Elevated Liver function tests, and
0. with severe preeclampsia? Low Platelet count.
82 What would be needed if Narcan for the baby bc will show effects of drug.
1. mom is given demerol and
ends up delivering the baby
an hour later?
82 What would be the initial discontinue the transfusion
2. nursing action when a child
receiving a transfusion of
packed red blood cells
complains of chills and
back pain
82 What would the findings in Findings in newborns include failure to take liquids,
3. a newborn with constipation, and bile-stained vomitus.
Hirschprung's disease be?
82 When a child has pinworms, andy family member with symptoms should be treated
4. the nurse should know that
82 When an infant is receiving digoxin (Lanoxin), the nausea and vomiting
5. nurse would be alert to which finding as a sign of
toxicity?
82 When are pain drugs given during pregnancy? ACTIVE phase only. If
6. given during transitional
phase they can effect the
baby.
82 When assessing a male neonate, the nurse notices She should report the
7. that the urinary meatus is located on the ventral finding as hypospadias.
surface of the penis. How should the nurse report
this finding?
82 When assessing a neonate 1 hour after delivery, the Hypothermia related to
8. nurse measures an axillary temperature of 95.6 F heat loss
(35.3 C), an apical pulse of 110 beats/minute, and a
respiratory rate of 64 breaths/minute. Which nursing
diagnosis takes highest priority at this time?
82 When assessing the neonate of a client who used irritability and poor
9. heroin during her pregnancy, the nurse expects to sucking.
find:
83 When caring for a child on steriod therapy, it is develops a fever
0. important to seek immediate medical attention if the
child
83 When caring for a neonate, what is the most Practicing meticulous
1. important step the nurse can take to prevent and hand washing
control infection?
83 When collecting data on a client during her first 2. ""I support your
2. prenatal visit, the nurse discovers that the client had commitment; however,
a reduction mammoplasty. The mother indicates she you may have to
wants to breast-feed. What information should the supplement each feeding
nurse give to this mother regarding breast-feeding with formula.
success?
83 When collecting data on a pregnant client with "4. Glycosuria
3. diabetes mellitus, the nurse stays alert for signs and
symptoms of a vaginal or urinary tract infection
(UTI). Which condition makes this client more
susceptible to such infections?
83 When determining maternal and fetal well-being, 2. Fetal heart rate and
4. which of the following data collection findings is activity
most important?
83 When does a child respond to his/her own name? 6-8 months
5.

83 When does a child take deliberate steps? 9-10 months


6.

83 When does preeclampsia usually occur? Third trimester


7.

83 When does separation anxiety happen? 8 mo


8.

83 When evaluating a client's knowledge of symptoms . "If I have blurred or


9. to report during her pregnancy, which statement double vision, I should call
would indicate to the nurse that the client the clinic immediately."
understands the information given to her?
84 When evaluating a pregnant client's fundal height, 4. From the symphysis
0. the nurse should measure in which way? pubis notch to the highest
level of the fundus
84 When is Rhogam received? 1st trimester or 72 hours
1. after childbirth
84 When is vision in a baby fully developed? At seven months
2.

84 When planning care for a ANS: D


3. client diagnosed with female The expected outcome of senate focus exercises is
sexual arousal disorder, what to reduce goal-oriented demands of intercourse.
should a nurse document as Senate focus exercises consist of touching and
an expected outcome of being touched by another with attention focused
senate focus exercises? on the physical sensations encountered. Erotic
contact is gradually increased, leading to the
A. To initiate immediate possibility of sexual intercourse. The reduction in
orgasm demands reduces performance pressures and
B. To reduce anxiety by anxiety associated with possible failure.
eliminating physical touch
C. To focus on touching
breasts and genitals
D. To reduce goal-oriented
demands of intercourse
84 When providing health 2. Between 23 and 27 weeks' gestation
4. teaching to a primigravid
client, the nurse tells the
client that she's likely to first
experience Braxton Hicks
contractions. When do these
contractions typically begin?
84 When questioned, a pregnant 4. Learning disability
5. client admits she sometimes
has several glasses of wine
with dinner. Her alcohol
consumption puts her fetus at
risk for which condition?
84 When should a client who's 3. At about 28 weeks' gestation and again within
6. Rh(D)-negative and D- 72 hours after delivery
negative and who hasn't
already formed Rh antibodies
receive Rh(D) human
immunoglobulin (RhIg) to
prevent isoimmunization?
84 When teaching a group of 4. in the first third of the fallopian tube.
7. pregnant adolescents about
reproduction and conception,
the nurse is correct when
stating that fertilization
occurs:
84 When using a Pavlik harness Flexed and abducted
8. on a neonate with
developmental dysplasia of
the hip, the nurse should
ensure that the affected hip is
in which position?
84 When will a baby show a By third month
9. preference for mothers voice
and turn head in the direction
of sound?
85 Which action should the nurse establish that child is voiding
0. take before adding potassium
to a child's IV
85 Which approach might best encourage the mother to participate in the child's
1. support maternal attachment care
when caring for a child with
failure to thrive
85 Which diagnostic test permits Endoscopy
2. visualization of the upper GI
tract
85 Which drug will the physician "4. Phosphorated carbohydrate solution (Emetrol)
3. probably order to treat a
pregnant client who is
experiencing morning
sickness?
85 Which finding in a neonate Bradycardia
4. suggests hypothermia?
85 Which finding Weight loss, then return to birth weight
5. is considered
normal in the
neonate during
the first few
days after
birth?
85 Which findings "3. Fetal heartbeat and fetal movement on palpation
6. would be
considered
positive signs
of pregnancy?
85 Which info Place prone with the head elevated after feeding
7. would the
nurse give to
parents of an
infant with
gastroesophag
eal reflex
disease
85 Which Dry and dress the neonate in clothing with a hat, and swaddle him
8. intervention snuggly in blankets.
should the
nurse perform
as soon as
possible when
caring for a 21-
week-old
anencephalic
neonate?
85 Which Covering the neonate's head with a cap
9. intervention
takes priority
when caring
for a neonate
immediately
after birth?
86 Which 3. Insulin
0. medication is
considered
safe during
pregnancy?
86 Which med is cromolyn sodium
1. not useful
when a child is
experiencing
an asthma
attack
86 Which neonate A bottle-fed, 7-lb, 2-oz (3.2-kg) neonate who produces two stools
2. is at greatest and wets four diapers per day
risk for the
nursing
diagnosis
Imbalanced
nutrition: Less
than body
requirements
related to poor
sucking?
86 Which of the ANS: A, B, E
3. following The nurse should identify that experimenting with masturbation and
characteristics homosexual play and not wanting to undress in front of others are
should a nurse characteristics that are normal in the development of human
identify as sexuality in an 11-year-old child. Interest in the opposite sex usually
"normal" in the increases during this age, and children often become self-conscious
development of about their bodies.
human
sexuality for an
11-year-old
child? (Select
all that apply.)

A. The child
experiments
with
masturbation.
B. The child
may
experience
homosexual
play.
C. The child
shows little
interest in the
opposite sex.
D. The child
shows little
concern about
physical
attractiveness.
E. The child is
unlikely to
want to
undress in
front of others.
86 Which of the Slight yellowish hue to the skin
4. following data
collection
findings would
the nurse
interpret as
abnormal for a
term male
neonate who's
1 hour old?
86 Which of the A neonate born at less than 37 weeks' gestation regardless of
5. following weight
describes a
preterm
neonate?
86 Which of the A neonate who's in good condition
6. following
describes how
the nurse
interprets a
neonate's
Apgar score of
8 at 5 minutes?
86 Which of the The neonate lacks intestinal flora to make the vitamin.
7. following
explanations
describes the
rationale for
administering
vitamin K to
every neonate?
86 Which of the 4. Production of maternal antibodies
8. following
functions would
the nurse
expect to be
unrelated to the
placenta?
86 Which of the Most neonates are alert immediately following birth and are ready
9. following is the to nurse.
primary reason
for putting
breast-feeding
neonates to the
breast
immediately
after delivery?
87 Which of the 110 to 130 calories per kg
0. following
measurements
reflects normal
calorie intake
for a neonate?
87 "Which of the " 1. Caloric intake should be increased 300 cal/day. 2. Protein intake
1. following should be increased by more than 30 g/day. 5. Intake of all
nutritional minerals, especially iron, should be increased. "
instructions
should the
nurse provide to
a 32-year-old
primigravida?
87 Which of the Immature central nervous system (CNS)
2. following
options is a
contributory
factor to
thermoregulatio
n in a preterm
neonate?
87 Which of the Intracranial bleeding
3. following
presents the
greatest risk to
the risk to the
child with
hemophilia
87 Which of the 1. Amenorrhea and quickening
4. following would
the nurse
expect to find
as presumptive
signs of
pregnancy?
87 Which sign Nasal flaring
5. indicates
respiratory
distress in a
neonate?
87 Which signs and itching, irritability, and restlessness
6. symptoms are
characteristic of
pinworms
87 Which 1. Both estrogen and progesterone levels are rising.
7. statement
accurately
describes
estrogen and
progesterone
levels during
the 16th week
of pregnancy?
87 While caring for Do nothing because this is normal.
8. a healthy
female neonate,
the nurse
notices red
stains on the
diaper after the
baby voids.
Which of the
following should
the nurse do
next?
87 While caring for 4. Level of emotional maturity
9. pregnant
adolescents,
the nurse
should develop
a plan of care
that
incorporates
which health
concern?
88 While Pointing out that an infant car seat is safest and arranging for them
0. discharging a to rent one
neonate, the
nurse notices
that the parents
have placed the
infant in a child
car seat. Which
action takes
priority?
88 While receiving This is a normal adverse effect of phototherapy.
1. phototherapy, a
neonate begins
to have
frequent, loose,
watery, green
stools and is
very irritable.
The nurse
interprets this
as which of the
following?
88 Who is most The woman with the 11 lb. baby.
2. likely to die
from
hemorrhage
during labor, a
women having
an 11 lb. baby
or a woman
delivering
sextuplets?
88 whole grain strict vegetarians need combinations of proteins for adequate
3. foods and amounts
legumes, or
nuts and
legumes, or
whole grain and
nuts
88 Why is an It is done during the 2nd (before 30 wks) to identify chromosomal
4. amniocentesis abnormalities. It is done during the 3rd (after 30 wks) to determine
done during the fetal lung maturity to diagnose fetal hemolytic disease (Rh
2nd trimester? incompatible).
During the 3rd?
88 Why is To prevent seizures with the complication of pregnancy induced
5. MAGNESIUM hypertension (PIH).
SULFATE given
to pregnant
women?
88 wish the patient The patient is being discharged. The nurse should:
6. well.
88 The woman A woman who delivered her third child yesterday has just learned
7. must make that her two school-age children have contracted chickenpox. What
arrangements should the nurse tell her?
to stay
somewhere
other than her
home until the
children are no
longer
contagious.
88 A woman who's 1. recognize these as normal early pregnancy signs and symptoms.
8. 10 weeks
pregnant tells
the nurse that
she's worried
about her
fatigue and
frequent
urination. The
nurse should:
1 A 58-year-old client complaining of difficulty driving at Vitamin A
. night states that the "lights bother my eyes." The client
wears corrective glasses. The nurse would suspect that
the client is experiencing a deficiency in which of the
following vitamins?
2 A 70-year-old client with a diagnosis of left-sided stroke is Promoting weight-
. admitted to the facility. To prevent the development of bearing exercises
disuse osteoporosis, which of the following objectives is
appropriate?
3 A 75-year-old client who was admitted to the hospital with Make arrangements
. a stroke informs the nurse that he doesn't want to be kept for the client to
alive with machines. He wants to make sure that everyone receive information
knows his wishes. Which action should the nurse take? about advance
directives.
4 A 78-year-old Alzheimer's client is being treated for wander.
. malnutrition and dehydration. The nurse decides to place
him closer to the nurses' station because of his tendency
to:
5 After a motor vehicle accident, a client is admitted to the Flat, except for
. medical-surgical unit with a cervical collar in place. The logrolling as needed
cervical spinal X-rays haven't been read, so the nurse
doesn't know whether the client has a cervical spinal
injury. Until such an injury is ruled out, the nurse should
restrict this client to which position?
6 After an eye examination, a client is diagnosed with open- instilling one drop of
. angle glaucoma. The physician's prescription says pilocarpine 0.25%
"pilocarpine ophthalmic solution (Pilocar), 0.25% 1 gtt into both eyes four
both eyes q.i.d." Based on this prescription, the nurse times daily.
should teach the client or a family member to administer
the drug by:
7 After a plane crash, a client is brought to the emergency evaluation of the
. department with severe burns and respiratory difficulty. corneal reflex
The nurse helps to secure a patent airway, attends to the response.
client's immediate needs, and then prepares to perform a
neurologic assessment. Because the client is unstable and
in critical condition, this examination must be brief but
should include:
8 After a stroke, a 75-year-old client is admitted to a health Elevating the head
. care facility. The client has left-sided weakness and an of the bed to 30
absent gag reflex. He's incontinent and has a tarry stool. degrees
His blood pressure is 90/50 mm Hg, and his hemoglobin is
10 g. Which action is a priority for this client?
9. After striking his head on a tree while falling from a ladder, Perform a lumbar
a young man is admitted to the emergency department. puncture.
He's unconscious and his pupils are nonreactive. Which
intervention would be the most dangerous for the client?
1 Audiometry confirms a client's chronic progressive hearing conductive
0. loss. Further investigation reveals ankylosis of the stapes in hearing loss.
the oval window, a condition that prevents sound
transmission. This type of hearing loss is called a:
1 An auto mechanic accidentally has battery acid splashed in decreasing
1. his eyes. His coworkers irrigate his eyes with water for 20 leukocyte
minutes, and then take him to the emergency department infiltration at the
of a nearby health care facility. He receives emergency care site of ocular
for corneal injury. The physician prescribes dexamethasone inflammation.
(Maxidex Ophthalmic Suspension), two drops of 0.1%
solution to be instilled initially into the conjunctival sacs of
both eyes every hour; and polymyxin B sulfate (Neosporin
Ophthalmic), 0.5% ointment to be placed in the conjunctival
sacs of both eyes every 3 hours. Dexamethasone exerts its
therapeutic effect by:
1 A client accidentally splashes chemicals into his eye. The To prevent vision
2. nurse knows that eye irrigation with plain tap water should loss
begin immediately and continue for 15 to 20 minutes. What
is the primary purpose of this first-aid treatment?
1 A client admitted to an acute care facility after a car Lidocaine
3. accident develops signs and symptoms of increased (Xylocaine)
intracranial pressure (ICP). The client is intubated and
placed on mechanical ventilation to help reduce ICP. To
prevent a further rise in ICP caused by suctioning, the nurse
anticipates administering which drug endotracheally before
suctioning?
1 A client admitted with a cerebral contusion is confused, Risk for injury
4. disoriented, and restless. Which nursing diagnosis takes related to
highest priority? neurologic deficit
1 A client, age 21, is admitted with bacterial meningitis. An isolation room
5. Which hospital room would be the appropriate choice for close to the
this client? nurses' station
1 A client arrives at the emergency department complaining Edrophonium
6. of extreme muscle weakness after minimal effort. The (Tensilon)
physician suspects myasthenia gravis. Which drug will be
used to test for this disease?
1 The client asks the nurse, "How does ergotamine (Ergostat) constricts cerebral
7. relieve migraine headaches?" The nurse should respond blood vessels.
that it:
1 A client comes to the clinic for an ophthalmologic The tonometer will
8. screening, which will include measurement of intraocular register the force
pressure with a tonometer. When teaching the client about required to indent
the test, the nurse should cover which point? or flatten the
corneal apex.
1 A client comes to the emergency department complaining Meningeal irritation
9. of headache, malaise, chills, fever, and a stiff neck. Vital
sign assessment reveals a temperature elevation,
increased heart and respiratory rates, and normal blood
pressure. On physical examination, the nurse notes
confusion, a petechial rash, nuchal rigidity, Brudzinski's
sign, and Kernig's sign. What does Brudzinski's sign
indicate?
2 A client complains of periorbital aching, tearing, blurred Cholinergic blocker
0. vision, and photophobia in her right eye. Ophthalmologic
examination reveals a small, irregular, nonreactive pupil
a condition resulting from acute iris inflammation (iritis).
As part of the client's therapeutic regimen, the physician
prescribes atropine sulfate (Atropisol), two drops of 0.5%
solution in the right eye twice daily. Atropine sulfate
belongs to which drug classification?
2 A client complains of vertigo. The nurse anticipates that Inner ear
1. the client may have a problem with which portion of the
ear?
2 A client diagnosed with a brain tumor experiences a Assist the client to
2. generalized seizure while sitting in a chair. How should the a side-lying
nurse intervene first? position on the
floor, and protect
her with linens.
2 A client has a history of painful, continuous muscle spasms. treatment of
3. He has taken several skeletal muscle relaxants without spasticity
experiencing relief. His physician prescribes diazepam associated with
(Valium), 2 mg by mouth twice daily. In addition to being spinal cord lesions.
used to relieve painful muscle spasms, diazepam also is
recommended for:
2 A client has an exacerbation of multiple sclerosis accompanied 1 to 2 weeks
4. by leg spasticity. The physician prescribes dantrolene sodium
(Dantrium), 25 mg by mouth daily. How soon after
administration can the nurse expect to see a significant
reduction in spasticity?
2 A client has just been diagnosed with early glaucoma. During a demonstrate
5. teaching session, the nurse should: eyedrop
instillation.
2 A client in a nursing home is diagnosed with Alzheimer's II
6. disease. He exhibits the following symptoms: difficulty with
recent and remote memory, irritability, depression,
restlessness, difficulty swallowing, and occasional
incontinence. This client is in what stage of Alzheimer's
disease?
2 A client injured in a train derailment is admitted to an acute hypoxia.
7. care facility with a suspected dysfunction of the lower brain
stem. The nurse should monitor this client closely for:
2 A client injures his spinal cord in a diving accident. The nurse C4
8. knows that the client will be unable to breathe spontaneously
if the injury site is above which vertebral level?
2 A client in the emergency department has a suspected Helicopod
9. neurologic disorder. To assess gait, the nurse asks the client to
take a few steps; with each step, the client's feet make a half
circle. To document the client's gait, the nurse should use
which term?
3 A client is admitted in a disoriented and restless state after Risk for injury
0. sustaining a concussion from a car accident. Which nursing
diagnosis takes highest priority in this client's plan of care?
3 A client is admitted to an acute care facility for treatment of a Muscle
1. brain tumor. When reviewing the chart, the nurse notes that contraction is
the client's extremity muscle strength is rated 1/5. What does palpable and
this mean? visible.
3 A client is admitted to the emergency department with a Prepare to
2. suspected overdose of an unknown drug. Arterial blood gas assist with
values indicate respiratory acidosis. What should the nurse do ventilation.
first?
3 A client is admitted to the facility for investigation of balance vertigo,
3. and coordination problems, including possible Mnire's tinnitus, and
disease. When assessing this client, the nurse expects to note: hearing loss.
3 A client is admitted with a cervical spine injury Ineffective breathing
4. sustained during a diving accident. When planning this pattern
client's care, the nurse should assign highest priority to
which nursing diagnosis?
3 A client is color blind. The nurse understands that this cones.
5. client has a problem with:
3 A client is diagnosed with a conductive hearing loss. on the affected side by
6. When performing Weber's test, the nurse expects that bone conduction.
this client will hear sound:
3 A client is having a tonic-clonic seizure. What should Take measures to
7. the nurse do first? prevent injury.
3 A client is hospitalized with Guillain-Barr syndrome. Even, unlabored
8. Which data collection finding is most significant? respirations
3 A client is receiving an I.V. infusion of mannitol Increased urine output
9. (Osmitrol) after undergoing intracranial surgery to
remove a brain tumor. To determine whether this drug
is producing its therapeutic effect, the nurse should
consider which finding the most significant?
4 A client is scheduled for an EEG after having a seizure "Avoid stimulants and
0. for the first time. Client preparation for this test should alcohol for 24 to 48
include which instruction? hours before the test."
4 A client is scheduled for electroconvulsive therapy Succinylcholine
1. (ECT). Before ECT begins, the nurse expects which (Anectine)
neuromuscular blocking agent to be administered?
4 A client is sitting in a chair and begins having a tonic- carefully move him to a
2. clonic seizure. The most appropriate nursing response flat surface and turn
is to: him on his side.
4 A client is suspected of having amyotrophic lateral electromyography
3. sclerosis (ALS). To help confirm this disorder, the nurse (EMG).
prepares the client for various diagnostic tests. The
nurse expects the physician to order:
4 A client is thrown from an automobile during a collision. C5
4. The nurse knows that the client will be able to maintain
gross arm movements and diaphragmatic breathing if
the injury occurs at what vertebral level?
4 A client is transferred to the intensive care unit after Administering a stool
5. evacuation of a subdural hematoma. Which nursing softener as
intervention would reduce the client's risk of increased prescribed
intracranial pressure (ICP)?
4 A client recovering from a stroke has right-sided nonfluent aphasia.
6. hemiplegia and telegraphic speech and often seems
frustrated and agitated, especially when trying to
communicate. However, the chart indicates that the
client's auditory and reading comprehension are intact.
The nurse suspects that the client has:
4 A client undergoes a craniotomy with supratentorial Elevated 30 degrees
7. surgery to remove a brain tumor. On the first
postoperative day, the nurse notes the absence of a bone
flap at the operative site. How should the nurse position
the client's head?
4 A client undergoes cerebral angiography to evaluate for Hemiplegia, seizures,
8. neurologic deficits. Afterward, the nurse checks and decreased level
frequently for signs and symptoms of complications of consciousness
associated with this procedure. Which findings indicate (LOC)
spasm or occlusion of a cerebral vessel by a clot?
4 A client who experienced a severe stroke develops a Notify a physician of
9. fever and a cough that produces thick, yellow sputum. A the findings.
nurse observes sediment in the client's urine in the
indwelling urinary catheter tubing. Based on these
findings, which action should the nurse take?
5 A client who experienced a stroke has left-sided facial Make sure a tonsil
0. droop. During mouth care, the client begins to cough suction device is
violently. What should the nurse do? readily available while
providing mouth care.
5 A client who experienced a stroke that left her with Client's ability to
1. residual right-sided weakness was just discharged to go climb the stairs while
home. The client lives in a two-story house in which the using a walker
bathroom is located on the second floor. A home health
care nurse is visiting the client for the first time. Which
issue should the nurse address during this visit?
5 A client who has been severely beaten is admitted to the raccoon eyes and
2. emergency department. The nurse suspects a basilar Battle's sign.
skull fracture after assessing:
5 A client who recently experienced a stroke tells the Notify the physician.
3. nurse that he has double vision. Which nursing
intervention is the most appropriate?
5 A client who's paralyzed on the left side has been The client uses a mirror
4. receiving physical therapy and attending teaching to inspect his skin.
sessions about safety. Which behavior indicates that
the client accurately understands safety measures
related to paralysis?
5 A client who's receiving phenytoin (Dilantin) to control 10 to 20 mcg/ml
5. seizures is admitted to the health care facility for
observation. The physician orders measurement of the
client's serum phenytoin level. Which serum phenytoin
level is therapeutic?
5 A client who sustained a closed head injury in a motor "What has the physician
6. vehicle accident is diagnosed as brain dead by a explained about the
neurosurgeon. The physician has scheduled a meeting client's prognosis?"
with the client's family about discontinuing life
support. Before the meeting, a family member asks
the nurse her opinion about life support. Which
response by the nurse is appropriate?
5 A client who sustained a closed head injury in a Fasten the restraint to
7. skating accident pulls out his feeding tube, I.V. the bed frame using a
catheter, and indwelling urinary catheter. To ensure quick-release knot.
this client's safety, a physician prescribes restraints.
Which action should a nurse take when using
restraints?
5 A client who sustained an L1 to L2 spinal cord injury in "What has your
8. a construction accident asks a nurse if he'll ever be physician told you about
able to walk again. Which response by the nurse is your ability to walk
appropriate? again?"
5 A client who was diagnosed with multiple sclerosis 3 "It's important for us to
9. years ago now presents with lower extremity have this information.
weakness and heaviness. During the admission You should review the
process, the client presents her advance directive, document with your
which states that she doesn't want intubation, physician at every
mechanical ventilation, or tube feedings should her admission."
condition deteriorate. How should the nurse respond?
6 A client who was trapped inside a car for hours after a Midbrain
0. head-on collision is rushed to the emergency
department with multiple injuries. During the
neurologic examination, the client responds to painful
stimuli with decerebrate posturing. This finding
indicates damage to which part of the brain?
6 A client with a conductive hearing disorder caused "Don't fly in an airplane,
1. by ankylosis of the stapes in the oval window climb to high altitudes,
undergoes a stapedectomy to remove the stapes make sudden movements,
and replace the impaired bone with a prosthesis. or expose yourself to loud
After the stapedectomy, the nurse should provide sounds for 30 days."
which client instruction?
6 A client with Alzheimer's disease is admitted for hip Risk for caregiver role
2. surgery after falling and fracturing the right hip. The strain related to increased
spouse tells the nurse of feeling guilty for letting the client care needs
accident happen and reports not sleeping well
because the spouse has been getting up at night
and doing odd things. Which nursing diagnosis is
most appropriate for the client's spouse?
6 A client with amyotrophic lateral sclerosis (ALS) tells Powerlessness
3. the nurse, "Sometimes I feel so frustrated. I can't do
anything without help!" This comment best supports
which nursing diagnosis?
6 A client with an inflammatory ophthalmic disorder increased intraocular
4. has been receiving a -inch ribbon of corticosteroid pressure.
ointment in the lower conjunctival sac four times per
day as directed. The client reports a headache and
blurred vision. The nurse suspects that these
symptoms represent:
6 A client with a spinal cord injury and subsequent increase the frequency of
5. urine retention receives intermittent catheterization the catheterizations.
every 4 hours. The average catheterized urine
volume has been 550 ml. The nurse should plan to:
6 A client with a suspected brain tumor is scheduled Determine whether the
6. for computed tomography (CT). What should the client is allergic to iodine,
nurse do when preparing the client for this test? contrast dyes, or shellfish.
6 A client with a tentative diagnosis of myasthenia a positive edrophonium
7. gravis is admitted for a diagnostic workup. (Tensilon) test.
Myasthenia gravis is confirmed by:
6 A client with epilepsy is having a seizure. During the place the client on his side,
8. active seizure phase, the nurse should: remove dangerous objects,
and protect his head.
6 A client with Guillain-Barr syndrome has paralysis "The paralysis
9. affecting the respiratory muscles and requires mechanical caused by this
ventilation. What should the nurse tell the client about the disease is
paralysis? temporary."
7 A client with hypertension comes to the clinic for a routine Vertigo
0. checkup. Because hypertension is a risk factor for cerebral
hemorrhage, the nurse questions the client closely about
warning signs and symptoms of hemorrhage. Which
complaint is a possible indicator of cerebral hemorrhage in
this client?
7 A client with idiopathic seizure disorder is being "Schedule follow-up
1. discharged with a prescription for phenytoin (Dilantin). visits with your
Client teaching about this drug should include which physician for blood
instruction? tests."
7 A client with multiple sclerosis who is unable to bathe "I'm sorry you
2. herself complains that other staff members haven't been haven't been
bathing her. How should the nurse respond to this client's bathed. I'm
complaint? available to bathe
you now."
7 A client with Parkinson's disease visits the physician's "I take the
3. office for a routine checkup. The nurse notes that the medication at
client takes benztropine (Cogentin), 0.5 mg by mouth bedtime."
daily, and asks when the client takes the drug each day.
Which response indicates that the client understands
when to take benztropine?
7 A client with quadriplegia is in spinal shock. What should Absence of reflexes
4. the nurse expect? along with flaccid
extremities
7 A client with respiratory complications of multiple Suction machine
5. sclerosis (MS) is admitted to the medical-surgical unit. with catheters
Which equipment is most important for the nurse to keep
at the client's bedside?
7 A client with seizure disorder comes to the physician's Excessive gum
6. office for a routine checkup. Knowing that the client takes tissue growth
phenytoin (Dilantin) to control seizures, the nurse
assesses for which common adverse drug reaction?
7 A client with weakness and tingling in both legs is Lung auscultation
7. admitted to the medical-surgical unit with a tentative and measurement
diagnosis of Guillain-Barr syndrome. In this syndrome, of vital capacity and
polyneuritis leads to progressive motor, sensory, and tidal volume
cranial nerve dysfunction. On admission, which
assessment is most important for this client?
7 Damage to which area of the brain results in receptive Temporal lobe
8. aphasia?
7 During recovery from a stroke, a client is given nothing cranial nerves IX and X.
9. by mouth, to help prevent aspiration. To determine
when the client is ready for a liquid diet, the nurse
assesses the client's swallowing ability once each shift.
This assessment evaluates:
8 During the course of a busy shift, a nurse fails to Notify the physician of
0. document that a client's ventricular drain had an the documentation
output of 150 ml. Assuming that the drain was no omission.
longer draining cerebrospinal fluid, the physician
removes the drain. When the nurse arrives for work
the next morning, she learns that the client became
agitated during the night and his blood pressure
became elevated. What action should the nurse take?
8 Family members would like to bring in a birthday cake Cranial nerve V
1. for a client with nerve damage. What cranial nerve
needs to be functioning so the client can chew?
8 For a client who has had a stroke, which nursing Attaching braces or
2. intervention can help prevent contractures in the splints to each foot and
lower legs? leg
8 For a client with a head injury whose neck has been 30-degree head
3. stabilized, the preferred bed position is: elevation.
8 For a client with suspected increased intracranial promote carbon dioxide
4. pressure (ICP), the most appropriate respiratory goal elimination.
is to:
8 A home health nurse visits a client who's taking Advise the client to
5. pilocarpine, a miotic agent, to treat glaucoma. The discard the drug
nurse notes that the client's pilocarpine solution is because it may have
cloudy. What should the nurse do first? undergone chemical
changes or become
contaminated.
8 How should the nurse position a client for a lumbar Laterally, with knees
6. puncture? drawn up to the
abdomen and chin
touching the chest
8 If a client experienced a stroke that damaged the body temperature
7. hypothalamus, the nurse would anticipate that the control.
client has problems with:
8 In a client with amyotrophic lateral sclerosis and Increased restlessness
8. respiratory distress, which finding is the earliest sign
of reduced oxygenation?
8 The neurologic unit has identified a 30% occurrence Creating a spreadsheet
9. of pressure ulcers in clients admitted with the on which nursing staff
diagnosis of stroke. Which of the following actions should document
should be included in the unit's performance repositioning of clients
improvement plan? admitted with a stroke
9 The nurse assesses normal pupils in a client who had Equal
0. a craniotomy, and then writes "PERRLA" in the
nurse's notes, along with other findings. What does
the "E" stand for in this acronym?
9 The nurse formulates a nursing diagnosis of Risk for Hypothalamus
1. imbalanced body temperature for a client who suffers
a stroke after surgery. The expected outcomes
incorporate assessment of the client's temperature to
detect abnormalities. The thermoregulatory centers
are located in which part of the brain?
9 The nurse is administering neostigmine to a client Schedule the medication
2. with myasthenia gravis. Which nursing intervention before meals.
should the nurse implement?
9 The nurse is caring for a client diagnosed with a Call the physician
3. cerebral aneurysm, who reports a severe headache. immediately.
Which action should the nurse perform first?
9 The nurse is caring for a client in a coma who has Elevate the head of the
4. suffered a closed head injury. What intervention bed to 30 degrees.
should the nurse implement to prevent increases in
intracranial pressure (ICP)?
9 The nurse is caring for a client who underwent a Urine retention or
5. lumbar laminectomy 2 days ago. Which finding should incontinence
the nurse consider abnormal?
9 The nurse is caring for a client with an acute bleeding keep the client in one
6. cerebral aneurysm. The nurse should take all of the position to decrease
following steps except: bleeding.
9 The nurse is caring for a client with L1-L2 paraplegia Establishing an
7. who is undergoing rehabilitation. Which goal is intermittent
appropriate? catheterization routine
every 4 hours
98. A nurse is caring for a group of clients on the Arrange an escort for a
neurologic unit. Which task should the nurse perform client who needs to go
first? to the physical therapy
department.
99. The nurse is caring for an elderly client who exhibits Alzheimer's disease.
signs of dementia. The most common cause of
dementia in an elderly client is:
10 The nurse is collecting data on a 38-year-old client Vision changes
0. diagnosed with multiple sclerosis. Which of the
following symptoms would the nurse expect to find?
10 The nurse is collecting data on a geriatric client with Decreased acetylcholine
1. senile dementia. Which neurotransmitter condition is level
likely to contribute to this client's cognitive changes?
10 The nurse is monitoring a client for adverse reactions Tachycardia
2. to atropine sulfate (Atropine Care) eyedrops.
Systemic absorption of atropine sulfate through the
conjunctiva can cause which adverse reaction?
10 The nurse is monitoring a client for adverse reactions Muscle weakness
3. to dantrolene (Dantrium). Which adverse reaction is
most common?
10 The nurse is monitoring a client for increasing diminished
4. intracranial pressure (ICP). Early signs of increased responsiveness.
ICP include:
10 The nurse is observing a client with cerebral edema 90
5. for evidence of increasing intracranial pressure. She
monitors his blood pressure for signs of widening
pulse pressure. His current blood pressure is 170/80
mm Hg. What is the client's pulse pressure?
10 The nurse is performing a mental status examination Cerebral function
6. on a client diagnosed with a subdural hematoma. This
test assesses which of the following functions?
10 The nurse is planning care for a client who suffered a Provide close
7. stroke in the right hemisphere of his brain. What supervision because of
should the nurse do? the client's
impulsiveness and poor
judgment.
10 The nurse is preparing a client for a computed "Are you allergic to
8. tomography (CT) scan, which requires infusion of seafood or iodine?"
radiopaque dye. Which question is important for the
nurse to ask?
10 The nurse is preparing to administer carbamazepine 7.5
9. (Tegretol) oral suspension, 150 mg by mouth. The
pharmacy has dispensed carbamazepine suspension
100 mg/5 ml. How many milliliters of carbamazepine
should the nurse administer to the client?
11 The nurse is teaching a client and his family about Gamma aminobutyric
0. baclofen (Lioresal) therapy. Baclofen is an analogue of acid (GABA)
which neurotransmitter?
11 The nurse is teaching a client who has facial muscle destruction of
1. weakness and has recently been diagnosed with acetylcholine receptors.
myasthenia gravis. The nurse should teach the client
that myasthenia gravis is caused by:
11 The nurse is teaching a client with a T4 spinal cord his upper body to the
2. injury and paralysis of the lower extremities how to wheelchair first.
transfer from the bed to a wheelchair. The nurse
should instruct the client to move:
11 The nurse is teaching a client with multiple sclerosis. rest in a room set at a
3. When teaching the client how to reduce fatigue, the comfortable
nurse should tell the client to: temperature.
11 The nurse is working on a surgical floor. The nurse laminectomy.
4. must logroll a client following a:
11 The nurse observes that a comatose client's response dysfunction in the brain
5. to painful stimuli is decerebrate posturing. The client stem.
exhibits extended and pronated arms, flexed wrists
with palms facing backward, and rigid legs extended
with plantar flexion. Decerebrate posturing as a
response to pain indicates:
11 A nurse on the neurologic unit evaluates her client A client who sustained
6. care assignment after receiving the shift report. Which a fall on the previous
client in her assignment should she attend to first? shift and is attempting
to get out of bed
11 The nurse on the neurologic unit must provide care for A client who requires
7. four clients who require different levels of care. Which minimal bathing
client should the nurse assist first with morning care? assistance and
ambulates with a
walker independently
11 The nurse receives a physician's order to administer 31
8. 1,000 ml of normal saline solution I.V. over 8 hours to
a client who recently had a stroke. What should the
drip rate be if the drop factor of the tubing is 15
gtt/ml?
11 On the 5th postoperative day, a client who underwent Auscultate the
9. spinal fusion begins to complain of nausea and has an abdomen for bowel
episode of vomiting. How should a nurse intervene? sounds.
12 The parents of a client who sustained a closed head injury Social worker
0. in a motor vehicle accident voice their concerns about the
distance and cost of the rehabilitation center chosen for
their son. Which health care team member can help the
parents with their questions and concerns?
12 The physician determines that a client's chronic, Otosclerosis
1. progressive hearing loss results from excess bone
formation around the oval window, which impedes normal
stapes movement and prevents sound transmission. What
is the clinical term for this correctable middle ear disorder?
12 A physician diagnoses a client with myasthenia gravis and Intestinal
2. prescribes pyridostigmine (Mestinon), 60 mg by mouth obstruction
every 3 hours. Before administering this anticholinesterase
agent, the nurse reviews the client's history. Which
preexisting condition would contraindicate the use of
pyridostigmine?
12 The physician orders measurement of the serum 4 hours
3. acetaminophen level of a client admitted with a suspected
overdose of this drug. To ensure an accurate result, the
nurse should wait how long after acetaminophen (Tylenol)
ingestion before drawing the blood sample?
12 The physician prescribes diazepam (Valium), 10 mg I.V., for It should be
4. a client experiencing status epilepticus. Which statement administered no
about I.V. diazepam is true? faster than 5
mg/minute in an
adult.
12 The physician prescribes mannitol (Osmitrol) I.V. stat for a Warm the solution
5. client who develops increased intracranial pressure after a in hot water to
head injury. While preparing to administer mannitol, the dissolve the
nurse notices crystals in the solution. What should the crystals.
nurse do?
12 The physician suspects myasthenia gravis in a client with thymus gland
6. chronic fatigue, muscle weakness, and ptosis. Myasthenia hyperplasia.
gravis is associated with:
12 A quadriplegic client is prescribed Muscle spasms with paraplegia or
7. baclofen (Lioresal), 5 mg by mouth three quadriplegia from spinal cord lesions
times daily. What is the principal
indication for baclofen?
12 (SELECT ALL THAT APPLY) A client is (2) Turn the client on his right side., (5)
8. admitted to the medical-surgical unit Apply a soft collar to keep the client's
after undergoing intracranial surgery to neck in a neutral position.
remove a tumor from the left cerebral
hemisphere. Which nursing interventions
are appropriate for the client's
postoperative care?
12 (SELECT ALL THAT APPLY) A client who (2) Wrist pronation, (3) Stiff extension
9. had a massive stroke exhibits of the arms and legs, (4) Plantar flexion
decerebrate posture. What are the of the feet (5) Opisthotonos
characteristics of this posture?
13 (SELECT ALL THAT APPLY) A client with a (1) Assist the client to the floor., (2)
0. history of epilepsy is admitted to the Turn the client to his side., (3) Place a
medical-surgical unit. While assisting the pillow under the client's head.
client from the bathroom, the nurse
observes the start of a tonic-clonic
seizure. Which nursing interventions are
appropriate for this client?
13 (SELECT ALL THAT APPLY) A client with (1) Monitor for skin rash., (3) Perform
1. tonic-clonic seizure disorder is being good oral hygiene, including daily
discharged with a prescription for brushing and flossing., (4) Periodic
phenytoin (Dilantin). Which instructions follow-up blood work is necessary., (5)
about phenytoin should the nurse give Report to the physician problems with
this client? walking and coordination, slurred
speech, or nausea.
13 (SELECT ALL THAT APPLY) The nurse is (3) Furnish the client's environment
2. assigned to care for a client with early with familiar possessions., (4) Assist
stage Alzheimer's disease. Which the client with activities of daily living
nursing interventions should be included (ADLs) as necessary., (5) Assign tasks
in the client's care plan? in simple steps.
13 (SELECT ALL THAT APPLY) The nurse is (1) Visual disturbances, (3) Balance
3. planning care for a client with multiple problems, (5) Mood disorders
sclerosis. Which problems should the
nurse expect the client to experience?
13 (SELECT ALL THAT APPLY) The nurse is (2) "I'll try to chew my food on the
4. teaching a client with trigeminal neuralgia unaffected side.", (4) "Drinking
how to minimize pain episodes. Which fluids at room temperature should
comments by the client indicate that he reduce pain.", (5) "If brushing my
understands the instructions? teeth is too painful, I'll try to rinse
my mouth instead."
13 Shortly after admission to an acute care In 10 to 15 minutes
5. facility, a client with a seizure disorder
develops status epilepticus. The physician
orders diazepam (Valium), 10 mg I.V. stat.
How soon can the nurse administer a second
dose of diazepam, if needed and prescribed?
13 To encourage adequate nutritional intake for stay with the client and encourage
6. a client with Alzheimer's disease, the nurse him to eat.
should:
13 To evaluate a client's cranial nerve function, gag reflex.
7. the nurse should assess:
13 What is the function of cerebrospinal fluid It cushions the brain and spinal
8. (CSF)? cord.
13 What should the nurse do when Apply pressure on the inner
9. administering pilocarpine (Pilocar)? canthus to prevent systemic
absorption.
14 When caring for a client with a head injury, Rising blood pressure and
0. the nurse must stay alert for signs and bradycardia
symptoms of increased intracranial pressure
(ICP). Which cardiovascular findings are late
indicators of increased ICP?
14 When caring for a client with head trauma, Test the nasal drainage for glucose.
1. the nurse notes a small amount of clear,
watery fluid oozing from the client's nose.
What should the nurse do?
14 When caring for a client with the nursing elevate the head of the bed 90
2. diagnosis Impaired swallowing related to degrees during meals.
neuromuscular impairment, the nurse
should:
14 When communicating with a client who has use short, simple sentences.
3. sensory (receptive) aphasia, the nurse
should:
14 When obtaining the health history from a light flashes and floaters in front of
4. client with retinal detachment, the nurse the eye.
expects the client to report:

Question 40 See full question

A nurse is caring for an infant who is to be administered an enema. What spiritually oriented interventions could the
nurse follow with newborns and infants?

You Selected:

Tell the infant that it will be over within a minute.

Correct response:

Encourage parents to be present during the treatment.

Explanation:

Question 41 See full question

A client has extreme fatigue and is malnourished, and laboratory tests reveal a hemoglobin level of 8.5 g/dL (85
g/L). The nurse should specifically ask the client about the intake of food high in which nutrients?

You Selected:

thiamine, riboflavin, and niacin

Correct response:

vitamins B6 and B12, folate, iron, and copper

Explanation:

Question 42 See full question

The nurse is caring for a client with bipolar disorder who was recently admitted to an inpatient unit and is
experiencing a manic episode. What is a priority nursing intervention for this client?

You Selected:

Encourage the client to keep a journal about feelings and emotions.

Correct response:

Closely monitor the clients eating and sleeping habits.

Explanation:

Question 43 See full question

A nurse observes an LPN measuring a clients urine output from an indwelling catheter drainage bag. Which
observation by the nurse ensures that the clients urine has been measured accurately?

You Selected:

The LPN pours the urine into a paper cup that holds approximately 250 mL.

Correct response:

The LPN pours the urine into a graduated measuring container.


Explanation:

Question 44 See full question

The nurse will be assisting with the cast removal from the leg of a 5-year-old child. Which of the following is most
important for the nurse to warn the parents about before the procedure?

You Selected:

The loud noise could damage the childs hearing.

Correct response:

The child may be afraid of the cast cutter.

Explanation:

Question 45 See full question

A client returned from surgery eight hours ago and has not voided. Which action should the nurse take first?

You Selected:

Palpate over the synthesis pubis for fullness.

Correct response:

Palpate over the synthesis pubis for fullness.

Explanation:

Question 46 See full question

A client who is in rehabilitation following a cerebrovascular accident (or brain attack) is experiencing total
hemiplegia of the dominant right side. The nurse finds that the client needs assistance with eating to ensure
optimum nutrition. Which of the following actions is most important for the nurse to take to facilitate rehabilitation
with eating?

You Selected:

Request a diet of thickened liquids that can be taken through a straw.

Correct response:

Assist the client in learning to eat with the left hand.

Explanation:

Question 47 See full question

The nurse is teaching a client with multiple sclerosis about prevention of urinary tract infection (UTI) and renal
calculi. Which of the following nutrition recommendations by the nurse would be the most likely to reduce the risk of
these conditions?

You Selected:

Eat foods and ingest fluids that will cause the urine to be less acidic.

Correct response:

Increase fluids (2500 mL/day) and maintain urine acidity by drinking cranberry juice.

Explanation:

Question 48 See full question

Which statement would be appropriate for a nurse documenting a stage 1 pressure ulcer found on a client who is
immobilized?
You Selected:

The clients subcutaneous tissue is visible with a blood blistered wound bed.

Correct response:

The clients skin is intact with non-blanchable redness of a localized area over a bony prominence.

Explanation:

Question 49 See full question

A 7-year-old has had an appendectomy on November 12. He has had pain for the last 24 hours. There is a
prescription to administer acetaminophen with codeine every 3 to 4 hours as needed. The nurse is beginning the
shift, and the child is requesting pain medication. The nurse reviews the chart below for pain history. Based on the
information in the medical record, what should the nurse do next?

You Selected:

Assess the child again in 1 hour.

Correct response:

Administer the acetaminophen with codeine.

Explanation:

Question 50 See full question

The nurse is caring for a 5 year-old that had surgery 12 hours ago. The child tells the nurse that she does not have
pain, but a few minutes later tells her parent that she does. Which would the nurse consider when interpreting this?

You Selected:

Inconsistency in pain reporting suggests that pain is not present.

Correct response:

Children may be experiencing pain even though they deny it to the nurse.

Explanation:

Question 1 See full question

As a nurse helps a client ambulate, the client says, "I had trouble sleeping last night." Which action should the
nurse take first?

You Selected:

Recommending warm milk or a warm shower at bedtime

Correct response:

Gathering more information about the client's sleep problem

Explanation:

Question 2 See full question

A physician orders hourly urine output measurement for a postoperative client with an indwelling catheter. The
nurse records the following amounts of output for 2 consecutive hours: 8 a.m. (0800): 50 ml; 9 a.m. (0900): 60 ml.
Based on these amounts, which action should the nurse take?

You Selected:

Increase the I.V. fluid infusion rate.

Correct response:

Continue to monitor and record hourly urine output.


Explanation:

Question 3 See full question

A client hasn't voided since before surgery, which took place 8 hours ago. When assessing the client, a nurse will:

You Selected:

be unable to palpate the bladder.

Correct response:

palpate the bladder above the symphysis pubis.

Explanation:

Question 4 See full question

A client states, "I have abdominal pain." Which assessment question would best determine the client's need for pain
medication?

You Selected:

"How does the pain medication make you feel?"

Correct response:

"What does the pain feel like?"

Explanation:

Question 5 See full question

A client complains of severe abdominal pain. To elicit as much information as possible about the pain, the nurse
should ask:

You Selected:

"Do you have the pain all the time?"

Correct response:

"Can you describe the pain?"

Explanation:

Question 6 See full question

A nurse suspects that a child, age 4, is being neglected physically. To best assess the child's nutritional status, the
nurse should ask the parents which question?

You Selected:

"What did your child eat for breakfast?"

Correct response:

"What did your child eat for breakfast?"

Explanation:

Question 7 See full question

An adolescent is being nursed with a skeletal traction for a fractured femur. Which is the most appropriate nursing
intervention for this client?

You Selected:

Add and remove weights at the adolescent's request.


Correct response:

Assess pin sites every shift and as needed.

Explanation:

Question 8 See full question

A nurse is caring for a client with bulimia nervosa. Strict management of the client's dietary intake is necessary.
Which intervention is the most important?

You Selected:

Fill out the client's menu and make sure she eats at least half of what is on her tray.

Correct response:

Serve the client's menu choices in a supervised area and observe her 1 hour after each meal.

Explanation:

Question 9 See full question

During a prenatal visit, a pregnant client with cardiac disease and slight functional limitations reports increased
fatigue. To help combat this problem, the nurse should advise her to:

You Selected:

take a vitamin and mineral supplement.

Correct response:

divide daily food intake into five or six meals.

Explanation:

Question 10 See full question

A woman who's 10 weeks pregnant tells the nurse that she's worried about her fatigue and frequent urination. The
nurse should:

You Selected:

tell her that she may be excessively worried.

Correct response:

recognize these as normal early pregnancy signs and symptoms.

Explanation:

Question 11 See full question

A nurse is caring for a 16-year-old pregnant adolescent. The client is taking an iron supplement. What should this
client drink to increase the absorption of iron?

You Selected:

A liquid antacid.

Correct response:

A glass of orange juice.

Explanation:

Question 12 See full question

Which instruction should a nurse give to a client who's 26 weeks pregnant and complains of constipation?
You Selected:

Encourage her to increase her intake of roughage and to drink at least six glasses of water per day.

Correct response:

Encourage her to increase her intake of roughage and to drink at least six glasses of water per day.

Explanation:

Question 13 See full question

A pregnant client complains of nausea every morning and again before meals. As a result of the nausea, she's been
unable to eat enough and has lost weight. Which nonpharmacologic intervention should the nurse recommend?

You Selected:

Drinking liquids with dry foods

Correct response:

Keeping crackers at the bedside to eat before getting out of bed

Explanation:

Question 14 See full question

A postpartum client decides to bottle-feed her neonate. Which client statement indicates the need for further
teaching about preventing engorgement?

You Selected:

"I shouldn't express milk manually."

Correct response:

"Taking hot showers can help reduce engorgement."

Explanation:

Question 15 See full question

After a vaginal birth, a postpartum client complains of perineal discomfort when sitting. A nurse provides teaching
on how to promote comfort. Which statement by the client indicates an understanding of how to promote comfort?

You Selected:

"I should place a pillow behind my back."

Correct response:

"I should contract my buttocks before sitting or rising."

Explanation:

Question 16 See full question

A nurse is instructing the client to do Kegel exercises. What should the nurse tell the client to do to perform these
pelvic floor exercises?

You Selected:

Do pelvic squats.

Correct response:

Stop the flow of urine while urinating.


Explanation:

Question 17 See full question

A client returns to the postnatal ward with her 3-week-old infant. Which statement by the client would prompt the
nurse to document "Imbalanced nutrition less than body requirements related to inadequate intake"?

You Selected:

The baby does not exhibit a steady weight gain."

Correct response:

The baby does not exhibit a steady weight gain."

Explanation:

Question 18 See full question

A nurse is reviewing a client's fluid intake and output record. Fluid intake and urine output should relate in which
way?

You Selected:

Fluid intake should be inversely proportional to the urine output.

Correct response:

Fluid intake should be about equal to the urine output.

Explanation: Question 1 See full question

A toddler with a ventricular septal defect is receiving digoxin to treat heart failure.
Which assessment finding should be the nurse's priority concern?

You Selected:

Tachycardia

Correct response:

Bradycardia

Explanation:

Question 2 See full question

A client is discharged after an aortic aneurysm repair with a synthetic graft to


replace part of the aorta. The nurse should instruct the client to notify the health
care provider (HCP) before having:

You Selected:

major dental work.

Correct response:

major dental work.


Explanation:

Question 3 See full question

A client experiencing alcohol withdrawal exhibits tremors, diaphoresis, and


hyperactivity. Blood pressure is 190/87 mm Hg, and pulse is 92 bpm. Which
medication should the nurse expect to administer?

You Selected:

lorazepam

Correct response:

lorazepam

Explanation:

Question 4 See full question

The nurse is caring for a client with an order for an intravenous infusion of dextrose
with 5% normal saline at 1500 mL over 8 hrs. The drip administration is set at 10
drops/mL. How fast will the IV infuse (drops/minute)? Record your answer using a
whole number.

Your Response:

188

Correct response:

31

Explanation:

Question 5 See full question

The nurse is caring for an infant diagnosed with thrush. Which instruction would
the nurse give to a clients mother who will be administering nystatin oral solution?

You Selected:

Administer the drug right after meals by swabbing the mouth.

Correct response:

Administer the drug right after meals by swabbing the mouth.


Explanation:

Question 1 See full question

A child is being discharged with albuterol nebulizer treatments. The nurse should
instruct the parents to watch for:

You Selected:

tachycardia.

Correct response:

tachycardia.

Explanation:

Question 2 See full question

A client is discharged after an aortic aneurysm repair with a synthetic graft to


replace part of the aorta. The nurse should instruct the client to notify the health
care provider (HCP) before having:

You Selected:

major dental work.

Correct response:

major dental work.

Explanation:

Question 3 See full question

A client is receiving magnesium sulfate at 3 g/h intravenously. The bag of 1,000 mL


normal saline contains 20 g of magnesium sulfate. How many mL/hour should the
nurse set the IV pump rate in order to deliver 3 g/h? Record your answer using a
whole number.

Your Response:

Correct response:

150
Explanation:

Question 4 See full question

After a dose-response test, the client with an overdose of barbiturates receives


pentobarbital sodium at a nonintoxicating maintenance level for 2 days and at
decreasing dosages thereafter. This regimen is effective in the client does not
develop:

You Selected:

seizures.

Correct response:

seizures.

Explanation:

Question 5 See full question

A 25-year-old client taking hydroxychloroquine for rheumatoid arthritis reports


difficulty seeing out of the left eye. What does this finding indicate?

You Selected:

possible retinal degeneration

Correct response:

possible retinal degeneration

Explanation:

Answer Key
Question 1 See full question

Which principle should a nurse consider when administering pain medication to a


client?

You Selected:

Sustained-release oral formulations should be given around the clock, if possible, for
control of chronic pain.
Correct response:

Sustained-release oral formulations should be given around the clock, if possible, for
control of chronic pain.

Explanation:

Question 2 See full question

A client is admitted to the local psychiatric facility with bipolar disorder in the manic
phase. The physician decides to start the client on lithium carbonate therapy. One
week after this therapy starts, the nurse notes that the client's serum lithium level is
1 mEq/L. What should the nurse do?

You Selected:

Continue to administer the medication as ordered.

Correct response:

Continue to administer the medication as ordered.

Explanation:

Question 3 See full question

A registered nurse on the oncology floor is busy with another client, so she
delegates care of a client to her coworker, a licensed practical nurse (LPN). The
client that the LPN begins caring for requires a three-hour chemotherapy infusion.
Which statement is in accordance with the Nurse Practice Act?

You Selected:

A chemotherapy certified RN must begin the chemotherapy, then the LPN may monitor
the client.

Correct response:

A chemotherapy certified RN must begin the chemotherapy, then the LPN may monitor
the client.
Explanation:

Question 4 See full question

A client is to be discharged with a prescription for an analgesic that is a controlled


substance. Which comment by the client indicates to the nurse that further
teaching is needed?

You Selected:

"I know I can titrate the dose according to the pain level."

Correct response:

"I know I can titrate the dose according to the pain level."

Explanation:

Question 5 See full question

Which of the following actions should the nurse take prior to administering an oral
medication to an infant? Select all that apply.

You Selected:

Verifying that it is the correct dose


Ensuring that it is the correct medication
Verifying the infant's name

Correct response:

Ensuring that it is the correct medication


Verifying that it is the correct dose
Verifying the infant's name

Explanation:

Question 1 See full question

A client has a nasogastric (NG) tube. How should the nurse administer oral
medication to this client?

You Selected:

Crush the tablets and prepare a liquid form; then insert the liquid into the NG tube.
Correct response:

Crush the tablets and prepare a liquid form; then insert the liquid into the NG tube.

Explanation:

Question 2 See full question

A child is being discharged with albuterol nebulizer treatments. The nurse should
instruct the parents to watch for:

You Selected:

tachycardia.

Correct response:

tachycardia.

Explanation:

Question 3 See full question

A client has been placed on long-term sulfasalazine therapy for treatment of


ulcerative colitis. The nurse should encourage the client to eat which foods to help
avoid the nutrient deficiencies that may develop as a result of this medication?

You Selected:

green, leafy vegetables

Correct response:

green, leafy vegetables

Explanation:

Question 4 See full question

Which statement indicates that the client needs further teaching about taking
medication to control cancer pain?

You Selected:

"I should skip doses periodically so I do not get hooked on my drugs.

Correct response:

"I should skip doses periodically so I do not get hooked on my drugs.


Explanation:

Question 5 See full question

A nurse is ordered to administer ampicillin 125 mg intramuscularly every 6 hours to


a 10-kg child with a respiratory tract infection. The drug label reads, The
recommended dose for a client weighing less than 40 kg is 25 to 50 mg/kg/day
intramuscularly or intravenously in equally divided doses at 6- to 8-hour intervals.
The drug concentration is 125 mg/5 ml. Which nursing interventions are
appropriate at this time? Select all that apply.

You Selected:

Assess the client for allergies to penicillin.


Administer the medication because it is within the dosing recommendations.
Question the physician about the order because it is more than the recommended
dosage.

Correct response:

Assess the client for allergies to penicillin.


Administer the medication because it is within the dosing recommendations.
Obtain a sputum culture, as ordered, before administering the first dose of the
medication.

Explanation:

Question 19 See full question

For the client who has difficulty falling asleep at night because of withdrawal symptoms from alcohol, which are
abating, which nursing intervention is likely to be most effective?

You Selected:

advising the client to take multiple short naps during the day until symptoms improve

Correct response:

teaching the client relaxation exercises to use before bedtime

Explanation:

Question 20 See full question

Before discharge from the hospital after a myocardial infarction, a client is taught to exercise by gradually
increasing the distance walked. Which vital sign should the nurse teach the client to monitor to determine whether
to increase or decrease the exercise level?

You Selected:

body temperature
Correct response:

pulse rate

Explanation:

Question 21 See full question

A 6-year-old child is admitted for an appendectomy. What is the most appropriate way for the nurse to prepare the
child for surgery?

You Selected:

Permit the child to play with the blood pressure cuff, electrocardiogram (ECG) pads, and a face mask.

Correct response:

Permit the child to play with the blood pressure cuff, electrocardiogram (ECG) pads, and a face mask.

Explanation:

Question 22 See full question

A nurse is planning care for a 12-year-old with rheumatic fever. The nurse should teach the parents to:

You Selected:

observe the child closely.

Correct response:

provide for adequate periods of rest between activities.

Explanation:

Question 23 See full question

Which statements by the mother of a toddler should lead the nurse to suspect that the child is at risk for iron
deficiency anemia? Select all that apply.

You Selected:

"He refuses to eat more than two different kinds of vegetables."

Correct response:

"He drinks over 4 glasses of milk per day."


"I cannot keep enough apple juice in the house; he must drink over 10 oz (300 mL) per day."

Explanation:

Question 24 See full question

An adolescent is on the football team and practices in the morning and afternoon before school starts for the year.
The temperature on the field has been high. The school nurse has been called to the practice field because the
adolescent is now reporting that he has muscle cramps, nausea, and dizziness. Which action should the school
nurse do first?

You Selected:

Move the adolescent to a cool environment.

Correct response:

Move the adolescent to a cool environment.


Explanation:

Question 25 See full question

The nurse is preparing to administer a preoperative medication that includes a sedative to a client who is having
abdominal surgery. The nurse should first:

You Selected:

make sure the client is covered with a warm blanket.

Correct response:

have the client empty the bladder.

Explanation:

Question 26 See full question

The nurse is instructing the client with chronic renal failure to maintain adequate nutritional intake. Which diet
would be most appropriate?

You Selected:

high-calcium, high-potassium, high-protein

Correct response:

low-protein, low-sodium, low-potassium

Explanation:

Question 27 See full question

The nurse is evaluating the outcome of therapy for a client with osteoarthritis. Which outcome indicates the goals of
therapy have been met?

You Selected:

joint degeneration arrested

Correct response:

joint range of motion improved

Explanation:

Question 28 See full question

The nurse is preparing the client with heart failure to go home. The nurse should instruct the client to:

You Selected:

monitor weight daily.

Correct response:

monitor weight daily.

Explanation:

Question 29 See full question

When developing a long term care plan for the client with multiple sclerosis, the nurse should teach the client to
prevent:

You Selected:

ascites.
Correct response:

contractures.

Explanation:

Question 30 See full question

To prevent back injury, the nurse should instruct the client to:

You Selected:

sleep on a soft mattress.

Correct response:

avoid prolonged sitting and standing.

Explanation: Question 1 See full question

Which relaxation strategy would be effective for a school-age child to use


during a painful procedure?
You Selected:

Having the child take a deep breath and blow it out until told to stop
Correct response:

Having the child take a deep breath and blow it out until told to stop
Explanation:

Question 2 See full question

After instruction of a primigravid client at 8 weeks' gestation about measures


to overcome early morning nausea and vomiting, which client statement
indicates the need for additional teaching?
You Selected:

"I will eat two large meals daily with frequent protein snacks."
Correct response:

"I will eat two large meals daily with frequent protein snacks."
Explanation:

Question 3 See full question

The nurse is instructing the client with chronic renal failure to maintain
adequate nutritional intake. Which diet would be most appropriate?
You Selected:

low-protein, low-sodium, low-potassium


Correct response:

low-protein, low-sodium, low-potassium


Explanation:

Question 4 See full question

The client with diabetes mellitus says, "If I could just avoid what you call
carbohydrates in my diet, I guess I would be okay." The nurse should base
the response to this comment on the knowledge that diabetes affects
metabolism of which nutrients?
You Selected:

proteins, fats, and carbohydrates


Correct response:

proteins, fats, and carbohydrates


Explanation:

Question 5 See full question

To prevent back injury, the nurse should instruct the client to:
You Selected:

avoid prolonged sitting and standing.


Correct response:

avoid prolonged sitting and standing.


Explanation:

Question 1 See full question

When planning pain control for a client with terminal gastric cancer, a nurse
should consider that:
You Selected:

clients with terminal cancer may develop tolerance to opioids.


Correct response:

clients with terminal cancer may develop tolerance to opioids.


Explanation:

Question 2 See full question

During chemotherapy, a boy, age 10, loses his appetite. When teaching the
parents about his food intake, the nurse should include which instruction?
You Selected:

"Let your child eat any food he wants."


Correct response:

"Let your child eat any food he wants."


Explanation:

Question 3 See full question

A nurse is caring for a client with an endotracheal tube who receives enteral
feedings through a feeding tube. Before each tube feeding, the nurse checks
for tube placement in the stomach as well as residual volume. The purpose
of the nurse's actions is to prevent:
You Selected:

aspiration.
Correct response:

aspiration.
Explanation:

Question 4 See full question

A client is being discharged with nasal packing in place. The nurse should
instruct the client to:
You Selected:

perform frequent mouth care.


Correct response:

perform frequent mouth care.


Explanation:

Question 5 See full question

A client who suffered blunt chest trauma in a motor vehicle accident


complains of chest pain, which is exacerbated by deep inspiration. On
auscultation, the nurse detects a pericardial friction rub a classic sign of
acute pericarditis. The physician confirms acute pericarditis and begins
appropriate medical intervention. To relieve chest pain associated with
pericarditis, which position should the nurse encourage the client to assume?
You Selected:

Leaning forward while sitting


Correct response:

Leaning forward while sitting

Explanation: Question 1 See full question

A nurse is teaching the parents of a child with cystic fibrosis about proper
nutrition. Which instruction should the nurse include?
You Selected:

Encourage a high-calorie, high-protein diet.


Correct response:

Encourage a high-calorie, high-protein diet.


Explanation:

Question 2 See full question

After instruction of a primigravid client at 8 weeks' gestation about measures


to overcome early morning nausea and vomiting, which client statement
indicates the need for additional teaching?
You Selected:

"I will eat two large meals daily with frequent protein snacks."
Correct response:

"I will eat two large meals daily with frequent protein snacks."
Explanation:

Question 3 See full question

The nurse is developing a plan of care for a client who has joint stiffness due
to rheumatoid arthritis. Which measure will be the most effective in relieving
stiffness?
You Selected:

a warm shower before performing activities of daily living


Correct response:

a warm shower before performing activities of daily living


Explanation:

Question 4 See full question

A health care provider prescribes intravenous normal saline solution to be


infused at a rate of 150 ml/hour for a client. How many liter(s) of solution will
the client receive during an 8-hour shift? Record your answer using one
decimal place (For example: 6.2).
Your Response:

1.2
Correct response:

1.2
Explanation:

Question 5 See full question

A client who suffered blunt chest trauma in a motor vehicle accident


complains of chest pain, which is exacerbated by deep inspiration. On
auscultation, the nurse detects a pericardial friction rub a classic sign of
acute pericarditis. The physician confirms acute pericarditis and begins
appropriate medical intervention. To relieve chest pain associated with
pericarditis, which position should the nurse encourage the client to assume?
You Selected:

Leaning forward while sitting


Correct response:

Leaning forward while sitting


Explanation:

Improve your mastery


TAKE A PRACTICE QUIZ

Question 1 See full question

A client is admitted with multiple pressure ulcers. When developing the


client's diet plan, the nurse should include:
You Selected:

ground beef patties.


Correct response:

ground beef patties.


Explanation:

Question 2 See full question

A client complains of abdominal discomfort and nausea while receiving tube


feedings. Which intervention is most appropriate for this problem?
You Selected:

Stop the feedings and check for residual volume.


Correct response:

Stop the feedings and check for residual volume.


Explanation:

Question 3 See full question

A nurse who is preparing to boost a client up in bed instructs the client to use
the overbed trapeze. Which risk factor for pressure ulcer development is the
nurse reducing by instructing the client to move in this manner?
You Selected:

Shearing forces
Correct response:

Shearing forces
Explanation:

Question 4 See full question

A nurse suspects that a child, age 4, is being neglected physically. To best


assess the child's nutritional status, the nurse should ask the parents which
question?
You Selected:

"Do you think your child eats enough?"


Correct response:

"What did your child eat for breakfast?"


Explanation:

Question 5 See full question

During chemotherapy, a boy, age 10, loses his appetite. When teaching the
parents about his food intake, the nurse should include which instruction?
You Selected:

"Let your child eat any food he wants."


Correct response:

"Let your child eat any food he wants."


Explanation:

Question 6 See full question

Which relaxation strategy would be effective for a school-age child to use


during a painful procedure?
You Selected:

Having the child take a deep breath and blow it out until told to stop
Correct response:

Having the child take a deep breath and blow it out until told to stop
Explanation:

Question 7 See full question

An adolescent, age 14, is hospitalized for nutritional management and drug


therapy after experiencing an acute episode of ulcerative colitis. Which
nursing intervention is appropriate?
You Selected:

Administering antibiotics with meals as ordered


Correct response:

Providing small, frequent meals


Explanation:

Question 8 See full question

A client has refused to take a shower since being admitted 4 days earlier. He
tells a nurse, "There are poison crystals hidden in the showerhead. They'll kill
me if I take a shower." Which nursing action is most appropriate?
You Selected:

Asking a security officer to assist in giving the client a shower


Correct response:

Accepting these fears and allowing the client to take a sponge bath
Explanation:

Question 9 See full question

A nurse is assigned to care for a client with anorexia nervosa. During the first
48 hours of treatment, which nursing intervention is most appropriate for this
client?
You Selected:

Trying to persuade the client to eat and thus restore nutritional balance
Correct response:

Providing one-on-one supervision during meals and for 1 hour afterward


Explanation:

Question 10 See full question

A client who's 7 months pregnant reports severe leg cramps at night. Which
nursing action would be most effective in helping the client cope with these
cramps?
You Selected:

Teaching her to dorsiflex her foot during the cramp


Correct response:

Teaching her to dorsiflex her foot during the cramp


Explanation:

Question 11 See full question

A woman who's 10 weeks pregnant tells the nurse that she's worried about
her fatigue and frequent urination. The nurse should:
You Selected:

recognize these as normal early pregnancy signs and symptoms.


Correct response:

recognize these as normal early pregnancy signs and symptoms.


Explanation:

Question 12 See full question

After instruction of a primigravid client at 8 weeks' gestation about measures


to overcome early morning nausea and vomiting, which client statement
indicates the need for additional teaching?
You Selected:

"I will eat dry crackers or toast before arising in the morning."
Correct response:

"I will eat two large meals daily with frequent protein snacks."
Explanation:

Question 13 See full question

The nurse administers a tap water enema to a client. While the solution is
being infused, the client has abdominal cramping. What should the nurse
do first?
You Selected:

Temporarily stop the infusion, and have the client take deep breaths.
Correct response:

Temporarily stop the infusion, and have the client take deep breaths.
Explanation:

Question 14 See full question

When teaching the parent of an infant with Hirschsprung's disease who


received a temporary colostomy about the types of foods the infant will be
able to eat, which diet would the nurse recommend?
You Selected:

low-fat diet
Correct response:

regular diet
Explanation:

Question 15 See full question

During the acute stage of meningitis, a 3-year-old child is restless and


irritable. Which intervention would be most appropriate to institute?
You Selected:

allowing the child to play in the bathtub


Correct response:

keeping extraneous noise to a minimum


Explanation:

Question 16 See full question

An adolescent is on the football team and practices in the morning and


afternoon before school starts for the year. The temperature on the field has
been high. The school nurse has been called to the practice field because the
adolescent is now reporting that he has muscle cramps, nausea, and
dizziness. Which action should the school nurse do first?
You Selected:

Have the adolescent go to the swimming pool.


Correct response:

Move the adolescent to a cool environment.


Explanation:

Question 17 See full question

A 5-year-old child with burns on the trunk and arms has no appetite. The
nurse and parent develop a plan of care to stimulate the child's appetite.
Which suggestion made by the parent would indicate the need for additional
teaching?
You Selected:

serving smaller and more frequent meals


Correct response:

withholding dessert and treats unless meals are eaten


Explanation:

Question 18 See full question

In discussing home care with a client after transurethral resection of the


prostate (TURP), the nurse should teach the male client that dribbling of
urine:
You Selected:

is an abnormal sign that requires intervention.


Correct response:

can persist for several months.


Explanation:

Question 19 See full question

A client has an ileal conduit. Which solution will be useful to help control odor
in the urine collecting bag after it has been cleaned?
You Selected:

ammonia
Correct response:

vinegar
Explanation:

Question 20 See full question

Which diet would be most appropriate for the client with ulcerative colitis?
You Selected:

low-sodium, high-carbohydrate
Correct response:

high-protein, low-residue
Explanation:

Question 21 See full question

A client is being discharged with nasal packing in place. The nurse should
instruct the client to:
You Selected:

use normal saline nose drops daily.


Correct response:

perform frequent mouth care.


Explanation:

Question 22 See full question

The client with tuberculosis is to be discharged home with nursing follow-up.


Which aspect of nursing care will have the highest priority?
You Selected:

teaching the client about the disease and its treatment


Correct response:

teaching the client about the disease and its treatment


Explanation:

Question 23 See full question

Which diet would be most appropriate for a client with chronic obstructive
pulmonary disease (COPD)?
You Selected:

high-calorie, high-protein diet


Correct response:

high-calorie, high-protein diet


Explanation:

Question 24 See full question

The client who had a permanent pacemaker implanted 2 days earlier is being
discharged from the hospital. The nurse knows that the client understands
the discharge plan when the client:
You Selected:

selects a low-cholesterol diet to control coronary artery disease.


Correct response:

verbalizes safety precautions needed to prevent pacemaker malfunction.


Explanation:

Question 25 See full question

During the first few weeks after a cholecystectomy, the client should follow a
diet that includes:
You Selected:

a decreased intake of fruits, vegetables, whole grains, and nuts, to minimize


pressure within the small intestine.
Correct response:

a limited intake of fat distributed throughout the day so there is not an


excessive amount in the intestine at any one time.
Explanation:

Question 26 See full question

Which is an appropriate nursing goal for the client who has ulcerative colitis?
The client:
You Selected:

uses a heating pad to decrease abdominal cramping.


Correct response:

verbalizes the importance of small, frequent feedings.


Explanation:

Question 27 See full question

An elderly client asks the nurse how to treat chronic constipation. What is the
best recommendation the nurse can make?
You Selected:

Take a stool softener such as docusate sodium daily.


Correct response:

Take a stool softener such as docusate sodium daily.


Explanation:

Question 28 See full question

Sudoriferous glands secrete which type of substance?


You Selected:

Sweat
Correct response:

Sweat
Explanation:

Question 29 See full question

A diet plan is developed for a client with gouty arthritis. The nurse should
advise the client to limit his intake of:
You Selected:

green vegetables.
Correct response:

organ meats.
Explanation:

Question 30 See full question

A client has just been diagnosed with early glaucoma. During a teaching
session, the nurse should:
You Selected:

demonstrate eyedrop instillation.


Correct response:

demonstrate eyedrop instillation.


Explanation:

Question 31 See full question

A client who recently experienced a stroke tells the nurse that he has double
vision. Which nursing intervention is the most appropriate?
You Selected:

Alternatively patch one eye every 2 hours.


Correct response:

Alternatively patch one eye every 2 hours.


Explanation:

Question 32 See full question

After a radical prostatectomy for prostate cancer, a client has an indwelling


catheter removed. The client then begins to have periods of incontinence.
During the postoperative period, which intervention should be implemented
first?
You Selected:

Kegel exercises
Correct response:

Kegel exercises
Explanation:

Question 33 See full question

A client is frustrated and embarrassed by urinary incontinence. Which


measure should the nurse include in a bladder retraining program?
You Selected:

Restricting fluid intake to reduce the need to void


Correct response:

Assessing present voiding patterns


Explanation:

Question 34 See full question

A female client reports to a nurse that she experiences a loss of urine when
she jogs. The nurse's assessment reveals no nocturia, burning, discomfort
when voiding, or urine leakage before reaching the bathroom. The nurse
explains to the client that this type of problem is called:
You Selected:

total incontinence.
Correct response:

stress incontinence.
Explanation:

Question 35 See full question

The client newly diagnosed with type 1 diabetes mellitus eats a lot of pasta
products, such as macaroni and spaghetti, and asks if they can be included
in the diet. The nurse should tell the client:
You Selected:

Eating pasta can cause hyperglycemia, so it is better to eliminate it.


Correct response:

"Pasta can be a part of your diet. It is included in the bread and cereal
exchange.
Explanation:

Question 36 See full question

The nurse has been instructing the client about how to prepare meals that
are low in fat. Which of these comments would indicate the client needs
additional teaching?
You Selected:

Ill eat water-packed tuna.


Correct response:

I will eat more liver with onions.


Explanation:

Question 37 See full question

The client attends two sessions with the dietitian to learn about diet
modifications to minimize gastroesophageal reflux. The teaching would be
considered successful if the client decreases the intake of which foods?
You Selected:

high-calcium foods
Correct response:

Fats

Question 1 See full question

A client states, "I have abdominal pain." Which assessment question would
best determine the client's need for pain medication?
You Selected:

"What does the pain feel like?"


Correct response:

"What does the pain feel like?"


Explanation:

Question 2 See full question

After 2 days of breast-feeding, a postpartum client reports nipple soreness.


Which client statement indicates an understanding of measures to help
relieve nipple soreness?
You Selected:

"I will apply warm compresses to my nipples just before feedings."


Correct response:

"I should lubricate my nipples with expressed milk before feedings."


Explanation:

Question 3 See full question

The client with tuberculosis is to be discharged home with nursing follow-up.


Which aspect of nursing care will have the highest priority?
You Selected:

teaching the client about the disease and its treatment


Correct response:

teaching the client about the disease and its treatment


Explanation:

Question 4 See full question

Which health-promoting activity should the nurse teach the client who
recently underwent a laryngectomy?
You Selected:

Cleanse the mouth three times a day.


Correct response:

Cleanse the mouth three times a day.


Explanation:

Question 5 See full question

The client attends two sessions with the dietitian to learn about diet
modifications to minimize gastroesophageal reflux. The teaching would be
considered successful if the client decreases the intake of which foods?
You Selected:

fats
Correct response:

fats
Explanation:

Explanation:

Question 38 See full question

An adult is dying from metastatic lung cancer, and all treatments have been
discontinued. The clients breathing pattern is labored, with gurgling sounds.
The clients spouse asks the nurse, Can you do something to help with the
breathing? Which is the nurses best response in this situation?
You Selected:

Explain to the spouse that it is standard practice not to suction clients when
treatments have been discontinued.
Correct response:

Reposition the client, elevate the head of the bed, and provide a cool compress.
Explanation:

Question 39 See full question

A client who was transferred from a long-term care facility is admitted with
dehydration and pneumonia. Which nursing interventions can help prevent
pressure ulcer formation in this client? Select all that apply.
You Selected:

Perform range-of-motion exercises.


Correct response:

Reposition the client every 2 hours.


Perform range-of-motion exercises.
Encourage the client to eat a well-balanced diet.
Explanation:

Question 40 See full question

A nurse is caring for an infant who is to be administered an enema. What


spiritually oriented interventions could the nurse follow with newborns and
infants?
You Selected:

Ask a child specialist to be present during treatment.


Correct response:

Encourage parents to be present during the treatment.


Explanation:

Question 41 See full question

The nurse evaluates the client's understanding of nutritional modifications to


manage his hypertension when he states:
You Selected:

"Limiting my salt intake to 2 grams per day will improve my blood pressure."
Correct response:

"Limiting my salt intake to 2 grams per day will improve my blood pressure."
Explanation:

Question 42 See full question

A diabetic client with peripheral vascular disease is ordered to wear knee-


high elastic compression stockings continuously until discharge. Which of the
following would be the priority for this client after the stockings are applied?
You Selected:

Remove elastic stockings once per day and observe lower extremities.
Correct response:

Remove elastic stockings once per day and observe lower extremities.
Explanation:

Question 43 See full question

A nursing assessment for a client with alcohol abuse reveals a disheveled


appearance and a foul body odor. What is the best initial nursing plan that
would assist the clients involvement in personal care?
You Selected:

Bathing and dressing the client each morning until the client is willing to
perform self-care independently
Correct response:

Assisting the client with bathing and dressing by giving clear, simple directions
Explanation:

Question 44 See full question

A client with diabetes mellitus has had declining renal function over the past
several years. Which diet regimen should the nurse recommend to the client
on days between dialysis?
You Selected:

A low-protein diet with an unlimited amount of water


Correct response:

A low-protein diet with a prescribed amount of water


Explanation:

Question 45 See full question

A nurse is caring for a group of pediatric clients. The nurse understands that
which age group would most likely identify their pain as punishment for past
behavior?
You Selected:

School age children (age 6 11 years)


Correct response:

Preschool or toddler (age 25 years)


Explanation:

Question 46 See full question

A frail elderly client with a hip fracture is to use an alternating air pressure
mattress at home to prevent pressure ulcers while recovering from surgery.
The nurse is assisting the clients family to place the mattress (see image).
What should the nurse instruct the family to do?
You Selected:

Make the bed with the bed sheet on top of the pressure mattress.
Correct response:

Make the bed with the bed sheet on top of the pressure mattress.
Explanation:

Question 47 See full question

A nurse is caring for a client who has been hospitalized with schizophrenia.
The client has had this disorder for 8 years and is now displaying regression,
increased disorganization and inappropriate social interactions. Which
nursing intervention will best help this client meet self-care needs?
You Selected:

Instruct client to bathe and dress by 0900.


Correct response:

Provide client with assistance in hygiene, grooming, and dressing.


Explanation:

Question 48 See full question

A postpartum client has a nursing diagnosis of risk for impaired urinary


elimination related to loss of bladder sensation after childbirth. Which of the
following priorities outcome criteria should the client achieve?
You Selected:

Client voids more than 30 mL/hour without urinary retention beginning 1 hour
after birth.
Correct response:

Client voids more than 30 mL/hour without urinary retention beginning 1 hour
after birth.
Explanation:

Question 49 See full question

A nurse is providing nutritional teaching for a client with a family history of


colon cancer. Which food choice by the client demonstrates an
understanding of the correct diet to follow?
You Selected:

Egg salad on rye bread


Correct response:

Vegetarian chili
Explanation:

Question 50 See full question

The nurse is instructing a client about skin care while receiving radiation
therapy to the chest. What should the nurse instruct the client to do?
You Selected:

Shave the chest to prevent contamination from chest hair.


Correct response:

Wash the area with tepid water and mild soap.


Explanation:

Question 1 See full question

A nurse is caring for an elderly client with a pressure ulcer on the sacrum.
When teaching the client about dietary intake, which foods should the nurse
emphasize?
You Selected:

Legumes and cheese


Correct response:

Lean meats and low-fat milk


Explanation:

Question 2 See full question

A nurse has been teaching a client about a high-protein diet. The teaching is
successful if the client identifies which meal as high in protein?
You Selected:

Chicken cutlet, spinach, and soda


Correct response:

Baked beans, hamburger, and milk


Explanation:

Question 3 See full question

For the past 24 hours, a client with dry skin and dry mucous membranes has
had a urine output of 600 ml and a fluid intake of 800 ml. The client's urine is
dark amber. These assessments indicate which nursing diagnosis?
You Selected:

Deficient fluid volume


Correct response:

Deficient fluid volume


Explanation:

Question 4 See full question

A client states, "I have abdominal pain." Which assessment question would
best determine the client's need for pain medication?
You Selected:

"Are you having pain?"


Correct response:

"What does the pain feel like?"


Explanation:

Question 5 See full question

Parents of a 4-year-old child with acute leukemia ask a nurse to explain the
concept of complementary therapy. The nurse should tell the parents that:
You Selected:

complementary therapy wouldn't help their child.


Correct response:

complementary therapy is an alternative to conventional medical therapies.


Explanation:

Question 6 See full question

An adolescent is diagnosed with iron deficiency anemia. After emphasizing


the importance of consuming dietary iron, the nurse asks him to select iron-
rich breakfast items from a sample menu. Which selection demonstrates
knowledge of dietary iron sources?
You Selected:

Ham and eggs


Correct response:

Ham and eggs


Explanation:

Question 7 See full question

A client in a catatonic state is admitted to the inpatient unit. The client is


emaciated, stares blankly into space, and does not respond to verbal or
tactile stimuli. In formulating nursing care interventions, the nurse should
give priority to:
You Selected:

observing and evaluating the client's nutritional needs.


Correct response:

observing and evaluating the client's nutritional needs.


Explanation:

Question 8 See full question

A client is in the eighth month of pregnancy. To enhance cardiac output and


renal function, the nurse should advise her to use which body position?
You Selected:

Semi-Fowler's
Correct response:

Left lateral
Explanation:

Question 9 See full question

As a client progresses through pregnancy, she develops constipation. What is


the primary cause of this problem during pregnancy?
You Selected:

Decreased appetite
Correct response:

Reduced intestinal motility


Explanation:

Question 10 See full question

During a prenatal visit, a pregnant client with cardiac disease and slight
functional limitations reports increased fatigue. To help combat this problem,
the nurse should advise her to:
You Selected:

eat three well-balanced meals per day.


Correct response:

divide daily food intake into five or six meals.


Explanation:

Question 11 See full question

A nurse is caring for a client during the first postpartum day. The client asks
the nurse how to relieve pain from her episiotomy. What should the nurse
instruct the woman to do?
You Selected:

Apply an ice pack to her perineum.


Correct response:

Apply an ice pack to her perineum.


Explanation:

Question 12 See full question

A nurse is planning care for a 12-year-old with rheumatic fever. The nurse
should teach the parents to:
You Selected:

provide for adequate periods of rest between activities.


Correct response:

provide for adequate periods of rest between activities.


Explanation:

Question 13 See full question

The parents of an ill child are concerned because the child is not eating
well. Which strategies are appropriate to encourage the child to eat? Select
all that apply.
You Selected:

Ask the child to say why he or she is not eating.


Correct response:

Allow the child to choose meals from an acceptable list of foods.


Let the child substitute items on the tray for other nutritious foods.
Ask the child to say why he or she is not eating.
Explanation:

Question 14 See full question

After teaching the parent of a child with a spica cast about skin care, which
parental action would indicate the need for additional teaching?
You Selected:

application of powder to the skin under the cast


Correct response:

application of powder to the skin under the cast


Explanation:

Question 15 See full question

The nurse is teaching the mother of a newly diagnosed diabetic child about
the principles of the diabetic diet. Which statement by the mother indicates
effective teaching?
You Selected:

"Most children find it difficult to eat all the calories required by their diets in
three main meals."
Correct response:

"Snacks are used to keep blood glucose at acceptable levels during times when
the insulin level peaks."
Explanation:

Question 16 See full question

A client develops chronic pancreatitis. What would be the appropriate home


diet for a client with chronic pancreatitis?
You Selected:

A low-fat, bland diet distributed over five to six small meals daily
Correct response:

A low-fat, bland diet distributed over five to six small meals daily
Explanation:

Question 17 See full question

When developing a long term care plan for the client with multiple sclerosis,
the nurse should teach the client to prevent:
You Selected:

fluid overload.
Correct response:

contractures.
Explanation:

Question 18 See full question

After cataract removal surgery, the nurse teaches the client about activities
that can be done at home. Which activity would be contraindicated?
You Selected:

walking down the hall unassisted


Correct response:

bending over the sink to wash the face


Explanation:

Question 19 See full question

Which measure would be most effective for the client to use at home when
managing the discomfort of rhinoplasty 2 days after surgery?
You Selected:

Lie in a prone position.


Correct response:

Apply ice compresses.


Explanation:

Question 20 See full question

Which health-promoting activity should the nurse teach the client who
recently underwent a laryngectomy?
You Selected:

Cleanse the mouth three times a day.


Correct response:

Cleanse the mouth three times a day.


Explanation:

Question 21 See full question

A nurse is assigned to care for a client with a tracheostomy tube. How can
the nurse communicate with this client?
You Selected:

By placing the call button under the client's pillow


Correct response:

By supplying a magic slate or similar device


Explanation:

Question 22 See full question

A client receiving external radiation to the left thorax to treat lung cancer has
a nursing diagnosis of Risk for impaired skin integrity. Which intervention
should be part of this client's care plan?
You Selected:

Avoiding using deodorant soap on the irradiated areas


Correct response:

Avoiding using deodorant soap on the irradiated areas


Explanation:

Question 23 See full question

A nurse is providing dietary instructions to a client with a history of


pancreatitis. Which of the following instructions would be most appropriate?
You Selected:

Maintain a low-carbohydrate, low-fat diet.


Correct response:

Maintain a high-carbohydrate, low-fat diet.


Explanation:

Question 24 See full question

A nurse is caring for a client who has limited mobility and requires a
wheelchair. The nurse has concern for circulation problems when which
device is used?
You Selected:

Gel flotation pad


Correct response:

Ring or donut
Explanation:

Question 25 See full question

A nurse is caring for a client placed in traction to treat a fractured femur.


Which nursing intervention has the highest priority?
You Selected:

Keeping the client from sliding to the foot of the bed


Correct response:

Assessing the extremity for neurovascular integrity


Explanation:

Question 26 See full question

A client in the surgical intensive care unit has skeletal tongs in place to
stabilize a cervical fracture. Protocol dictates that pin care should be
performed each shift. When providing pin care for the client, which finding
should the nurse report to the physician?
You Selected:

A small amount of yellow drainage at the left pin insertion site


Correct response:

A small amount of yellow drainage at the left pin insertion site


Explanation:

Question 27 See full question

A nurse consulting with a nutrition specialist knows it's important to consider


a special diet for a client with chronic obstructive pulmonary disease (COPD).
Which diet is appropriate for this client?
You Selected:

Full-liquid
Correct response:

High-protein
Explanation:

Question 28 See full question

A client is ordered continuous bladder irrigation at a rate of 60 gtt/minute.


The nurse hangs a 2 L bag of sterile solution with tubing on a three-legged
I.V. pole. She then attaches the tubing to the client's three-way urinary
catheter, adjusts the flow rate, and leaves the room. Which important
procedural step did the nurse fail to follow?
You Selected:

Attaching the infusion set to an infusion pump


Correct response:

Evaluating patency of the drainage lumen


Explanation:

Question 29 See full question

Which measure would the nurse expect to include in the teaching plan for a
multiparous client who gave birth 24 hours ago and is receiving intravenous
antibiotic therapy for cystitis?
You Selected:

limiting fluid intake to 1 L daily to prevent overload


Correct response:

emptying the bladder every 2 to 4 hours while awake


Explanation:

Question 30 See full question

In evaluating a clients response to nutrition therapy, which laboratory test


would be of highest priority to examine?
You Selected:

Lymphocyte count
Correct response:

Albumin level
Explanation:

Question 31 See full question

A physician orders a bland, full-liquid diet for a client. Which response, if


made by the client, would indicate to the nurse that the client has
understood the nurses dietary teaching?
You Selected:

"I can have oatmeal, custard, and tea."


Correct response:

"Today I can have apple juice, chicken broth, and vanilla ice cream."
Explanation:

Question 32 See full question

While the nurse is caring for a primiparous client with cephalopelvic


disproportion 4 hours after a cesarean birth, the client requests assistance in
breastfeeding. To promote maximum maternal comfort, which position would
be most appropriate for the nurse to suggest?
You Selected:

cradle hold
Correct response:

football hold
Explanation:

Question 33 See full question

A child has chickenpox. The father asks how to care for the lesions. The
nurse should advise that the child:
You Selected:

remove lesions' crusts as they form.


Correct response:

take an antihistamine and use calamine lotion on the closed lesions.


Explanation:

Question 34 See full question

A 10-year-old boy is 24 hours post appendectomy. He is awake, alert, and


oriented. He tells the nurse that he is experiencing pain. He has a
prescription for morphine 1 to 2 mg PRN for pain. What is
the priority nursing action in managing the child's pain?
You Selected:

Change the child's position in bed.


Correct response:

Obtain vital signs with a pain score.


Explanation:

Question 35 See full question

The nurse has been instructing the client about how to prepare meals that
are low in fat. Which of these comments would indicate the client needs
additional teaching?
You Selected:

I will eat more liver with onions.


Correct response:

I will eat more liver with onions.


Explanation:

Question 36 See full question

A client who has skeletal traction to stabilize a fractured femur has not had a
bowel movement for 2 days. The nurse should:
You Selected:

increase the clients fluid intake to 3,000 mL/day.


Correct response:

increase the clients fluid intake to 3,000 mL/day.


Explanation:

Question 37 See full question

Which skin preparation would be best to apply around the client's


colostomy?
You Selected:

cornstarch
Correct response:

adhesive skin barrier


Explanation:

Question 38 See full question

The client with hepatitis A is experiencing fatigue, weakness, and a general


feeling of malaise. The client tires rapidly during morning care.
The most appropriate goal for this client is to:
You Selected:

adapt to new levels of energy.


Correct response:

gradually increase activity tolerance.


Explanation:

Question 39 See full question

A client with rheumatoid arthritis states, I cannot do my household chores


without becoming tired. My knees hurt whenever I walk. Which goal for this
client should take priority?
You Selected:

Adapt self-care skills.


Correct response:

Conserve energy.
Explanation:

Question 40 See full question

The nurse is teaching a client with trigeminal neuralgia how to minimize pain
episodes. Which comments by the client indicate an understanding of the
instructions? Select all that apply.
You Selected:

"I can wash my face with cold water."


Correct response:

"I'll try to chew my food on the unaffected side."


"Drinking fluids at room temperature should reduce pain."
"If brushing my teeth is too painful, I'll try to rinse my mouth instead."
Explanation:

Question 41 See full question

A client returned from surgery eight hours ago and has not voided. Which
action should the nurse take first?
You Selected:

Tell the client to bear down and try to void.


Correct response:

Palpate over the synthesis pubis for fullness.


Explanation:

Question 42 See full question

The nurse is caring for a client with a nasogastric tube who is receiving
intermittent tube feedings by gravity every 4 hours. The nurse aspirates 75
mL of residual prior to the next feeding. What action should the nurse take
next?
You Selected:

Administer an amount of water equivalent to the feeding.


Correct response:

Return the residual and begin the feeding.


Explanation:

Question 43 See full question

A client has been using Chinese herbs and acupuncture to maintain health.
What is the best response by the nurse when asked if this practice could be
continued during recuperation from a long illness?
You Selected:

Lets discuss your desire to integrate these practices with the physician and
advocate on your behalf.
Correct response:

Lets discuss your desire to integrate these practices with the physician and
advocate on your behalf.
Explanation:

Question 44 See full question

The nurse is caring for a postoperative client who has not voided since
before surgery. Which is the nurse's most appropriate action?
You Selected:

Force fluids to encourage voiding


Correct response:

Palpate for the bladder above the symphysis pubis


Explanation:

Question 45 See full question

The nurse is caring for a client in labor. The client wishes to have a
nonmedicated labor and birth. During the early stages of labor, the client
becomes frustrated with the use of music and imagery. Which of the
following would the nurse include in the clients plan of care? Select all that
apply.
You Selected:

Offer the use of a yoga ball


Correct response:

Encourage ambulation
Suggest a shower or bath
Offer the use of a yoga ball
Explanation:

Question 46 See full question

The nurse is recording the intake and output for a client with the following:
D5NSS 1,000 ml; urine 450 ml; emesis 125 ml; Jackson Pratt drain #1 35 ml;
Jackson Pratt drain #2 32 ml; and Jackson Pratt drain #3 12 ml. How many
milliliters would the nurse document as the clients output? Record your
answer using a whole number.
Your Response:

0
Correct response:

654
Explanation:

Question 47 See full question

A client requires behavioral therapies to decrease or eliminate urinary


incontinence. Which procedures would the nurse expect to include in the
teaching plan for this client? Select all that apply.
You Selected:

Kegel exercises.
Correct response:

Kegel exercises.
Scheduled voiding.
Biofeedback.
Explanation:

Question 48 See full question

When providing discharge teaching for a client with uric acid calculi, the
nurse would include an instruction to avoid which type of diet?
You Selected:

Low calcium
Correct response:

High purine
Explanation:

Question 49 See full question

When developing a teaching plan for a client who is 8 weeks pregnant, which
of the following foods would the nurse suggest to meet the client's need for
increased folic acid? Select all that apply.
You Selected:

spinach
Correct response:

spinach
beans
Explanation:

Question 50 See full question

The nurse is caring for an older adult with mild dementia admitted with heart
failure. What nursing care will be helpful for this client in reducing potential
confusion related to hospitalization and change in routine? Select all that
apply.
You Selected:

Perform necessary procedures quickly.


Correct response:

Reorient frequently to time, place and situation.


Arrange for familiar pictures or special items at bedside.
Spend time with the client, establishing a trusting relationship.
Explanation:

Improve your mastery


TAKE A PRACTICE QUIZ

Question 31 See full question

Which is an appropriate nursing goal for the client who has ulcerative colitis? The client:

You Selected:

verbalizes the importance of small, frequent feedings.

Correct response:

verbalizes the importance of small, frequent feedings.

Explanation:

Question 32 See full question

A nurse is instructing a client about using antiembolism stockings. Antiembolism stockings help prevent deep vein
thrombosis (DVT) by:

You Selected:

elevating the extremity to prevent pooling of blood.

Correct response:

forcing blood into the deep venous system.

Explanation:

Question 33 See full question

A nurse is teaching an elderly client about developing good bowel habits. Which statement by the client indicates to
the nurse that additional teaching is required?
You Selected:

"I need to use laxatives regularly to prevent constipation."

Correct response:

"I need to use laxatives regularly to prevent constipation."

Explanation:

Question 34 See full question

Sudoriferous glands secrete which type of substance?

You Selected:

Oil

Correct response:

Sweat

Explanation:

Question 35 See full question

To encourage adequate nutritional intake for a client with Alzheimer's disease, a nurse should:

You Selected:

give the client privacy during meals.

Correct response:

stay with the client and encourage him to eat.

Explanation:

Question 36 See full question

When a client with an indwelling urinary catheter wants to walk to the hospital lobby to visit with family members,
the nurse teaches him how to do this without compromising the catheter. Which client action indicates an accurate
understanding of this information?

You Selected:

The client clamps the catheter drainage tubing while visiting with the family.

Correct response:

The client keeps the drainage bag below the bladder at all times.

Explanation:

Question 37 See full question

A postpartum woman who gave birth vaginally has unrelenting rectal pain despite the administration of pain
medication. Which action is most indicated?

You Selected:

preparing a warm sitz bath for the client

Correct response:

assessing the perineum


Explanation:

Question 38 See full question

While making a home visit to a multigravida 2 weeks after the birth of viable twins at 38 weeks gestation, the
nurse observes that the client looks pale, has dark circles around her eyes, and is breastfeeding one of the twins.
The clients apartment is clean, and nothing appears out of place. The client tells the nurse that she completed
three loads of laundry this morning. A priority need for this client is:

You Selected:

possible anemia related to large volume of blood loss and twin birth.

Correct response:

fatigue related to home maintenance and caring for twins.

Explanation:

Question 39 See full question

A client with diabetes is explaining to the nurse how he cares for the feet at home. Which statement indicates the
client needs further instruction on how to care for the feet properly?

You Selected:

I am not allowed to use a heating pad on my feet.

Correct response:

I inspect my feet once a week for cuts and redness.

Explanation:

Question 40 See full question

The client with a lumbar laminectomy asks to be turned onto the side. The nurse should:

You Selected:

ask the client to help by using an overhead trapeze to turn.

Correct response:

get another nurse to help logroll the client into position.

Explanation:

Question 41 See full question

A client with Addisons disease has fluid and electrolyte loss due to inadequate fluid intake and to fluid loss
secondary to inadequate adrenal hormone secretion. As the clients oral intake increases, which fluids would
be most appropriate?

You Selected:

milk and diet soda

Correct response:

chicken broth and juice

Explanation:

Question 42 See full question

The nurse is teaching a pregnant client about exercises that may be helpful during pregnancy. Which points should
the nurse include in the instruction? Select all that apply.
You Selected:

Pelvic rocking may help relieve lower back pain.

Correct response:

Pelvic rocking may help relieve lower back pain.


Abdominal breathing lifts the abdominal wall off of the uterus.
Kegel exercises help improve vaginal contractility and bladder control.

Explanation:

Question 43 See full question

A nurse is caring for an infant who is to be administered an enema. What spiritually oriented interventions could the
nurse follow with newborns and infants?

You Selected:

Tell the infant that it will be over within a minute.

Correct response:

Encourage parents to be present during the treatment.

Explanation:

Question 44 See full question

A mentally incapacitated client is scheduled for surgery. Considering the principle of autonomy, who should give the
consent for surgery?

You Selected:

Attending nurse.

Correct response:

Surrogate decision maker.

Explanation:

Question 45 See full question

A client has extreme fatigue and is malnourished, and laboratory tests reveal a hemoglobin level of 8.5 g/dL (85
g/L). The nurse should specifically ask the client about the intake of food high in which nutrients?

You Selected:

thiamine, riboflavin, and niacin

Correct response:

vitamins B6 and B12, folate, iron, and copper

Explanation:

Question 46 See full question

The nurse is caring for a client with bipolar disorder who was recently admitted to an inpatient unit and is
experiencing a manic episode. What is a priority nursing intervention for this client?

You Selected:

Base permission for family visits on the client's attendance at therapy groups.

Correct response:

Closely monitor the clients eating and sleeping habits.


Explanation:

Question 47 See full question

The nurse is teaching a client with multiple sclerosis about prevention of urinary tract infection (UTI) and renal
calculi. Which of the following nutrition recommendations by the nurse would be the most likely to reduce the risk of
these conditions?

You Selected:

Drink a large amount of fluids, especially milk products, and eat a diet that includes multiple sources of
vitamin D.

Correct response:

Increase fluids (2500 mL/day) and maintain urine acidity by drinking cranberry juice.

Explanation:

Question 48 See full question

Because of symptoms experienced after a cerebrovascular accident (CVA), the nurse discovers that a client needs
assistance using utensils while eating. What would the nurse do to support this activity of care?

You Selected:

Have the family feed the client at every meal to reduce staffing limitations.

Correct response:

Encourage participation in the feeding process to the best of the client's abilities.

Explanation:

Question 49 See full question

A nursing assessment for a client with alcohol abuse reveals a disheveled appearance and a foul body odor. What is
the best initial nursing plan that would assist the clients involvement in personal care?

You Selected:

Devising a bathing and dressing schedule for each morning

Correct response:

Assisting the client with bathing and dressing by giving clear, simple directions

Explanation:

Question 50 See full question

A nurse is caring for a client who is 3 days postpartum and breastfeeding her baby girl. The following assessment is
made by the nurse: episiotomy area: red and edematous; breasts: firm and tender on palpation; fundus: firm 2
finger breaths below umbilicus. What nursing actions are indicated? Select all that apply.

You Selected:

Suggest the client apply cool compress to breasts.

Correct response:

Suggest the client apply cool compress to breasts.


Ask the client how often the baby feeds.

Explanation:

Question 1 See full question

A nurse notes that a client's I.V. insertion site is red, swollen, and warm to the touch. Which action should the nurse
take first?
You Selected:

Discontinue the I.V. infusion.

Correct response:

Discontinue the I.V. infusion.

Explanation:

Question 2 See full question

The physician orders dextrose 5% in water, 1,000 ml to be infused over 8 hours. The I.V. tubing delivers 15
drops/ml. The nurse should run the I.V. infusion at a rate of:

You Selected:

32 drops/minute.

Correct response:

32 drops/minute.

Explanation:

Question 3 See full question

A nurse is preparing to administer a sustained-release capsule to a client. Which is an appropriate nursing


intervention?

You Selected:

Opening the capsule, shaking the contents into water, and administering it to the client

Correct response:

Administering the capsule whole with a glass of water

Explanation:

Question 4 See full question

A physician orders heparin, 7,500 units, to be administered subcutaneously every 12 hours. The vial reads 10,000
units per milliliter. The nurse should anticipate giving how much heparin for each dose?

You Selected:

1 ml

Correct response:

ml

Explanation:

Question 5 See full question

A nurse regularly inspects a client's I.V. site to ensure patency and prevent extravasation during dopamine therapy.
What is the treatment for dopamine extravasation?

You Selected:

Asking the physician to make an incision and allowing the affected area to drain

Correct response:

Elevating the affected limb, applying warm compresses, and administering phentolamine as ordered
Explanation:

Question 6 See full question

A physician orders digoxin elixir for a toddler with heart failure. Immediately before administering this drug, the
nurse must check the toddler's:

You Selected:

urine output.

Correct response:

apical pulse.

Explanation:

Question 7 See full question

A toddler develops acute otitis media and is ordered cefpodoxime proxetil 5 mg/kg P.O. every 12 hours. If the child
weighs 22 lb (10 kg), how many milligrams will the nurse administer with each dose?

You Selected:

220 mg

Correct response:

50 mg

Explanation:

Question 8 See full question

An agitated and incoherent client comes to the emergency department with complaints of visual and auditory
hallucinations. The history reveals that this client was hospitalized for schizophrenia from ages 20 to 21. The
physician orders haloperidol, 5 mg I.M. The nurse understands that this drug is used in this client to treat:

You Selected:

tardive dyskinesia.

Correct response:

psychosis.

Explanation:

Question 9 See full question

The nurse is checking the client's chart for possible contraindications, before administering meperidine, 50 mg I.M.,
to a client with pain after an appendectomy. The nurse should hold the meperidine when she sees an order for what
type of drug?

You Selected:

An antibiotic

Correct response:

A monoamine oxidase (MAO) inhibitor

Explanation:

Question 10 See full question

A woman is taking oral contraceptives. The nurse teaches the client to report which complication?

You Selected:

breakthrough bleeding
Correct response:

severe calf pain

Explanation:

Question 11 See full question

A female client is treated for trichomoniasis with metronidazole. The nurse instructs the client that:

You Selected:

she should avoid alcohol during treatment and for 24 hours after completion of the drug.

Correct response:

she should avoid alcohol during treatment and for 24 hours after completion of the drug.

Explanation:

Question 12 See full question

A client with asthma has been prescribed beclomethasone via metered-dose inhaler. To determine if the client has
been rinsing the mouth after each use of the inhaler, the nurse should inspect the client's mouth for:

You Selected:

ulceration

Correct response:

oral candidiasis.

Explanation:

Question 13 See full question

A client has been taking furosemide for 2 days. The nurse should assess the client for:

You Selected:

an elevated blood urea nitrogen (BUN) level.

Correct response:

a decreased potassium level.

Explanation:

Question 14 See full question

The nurse teaches the client with chronic cancer pain about optimal pain control. Which recommendation
is most effective for pain control?

You Selected:

Take prescribed analgesics on an around-the-clock schedule to prevent recurrent pain.

Correct response:

Take prescribed analgesics on an around-the-clock schedule to prevent recurrent pain.

Explanation:

Question 15 See full question

Which statement indicates that the client with osteoarthritis understands the effects of capsaicin cream?

You Selected:

"I also use the same cream when I get a cut or a burn."
Correct response:

"I always wash my hands right after I apply the cream."

Explanation:

Question 16 See full question

Immediately after a lumbar laminectomy, the nurse administers ondansetron hydrochloride to the client as
prescribed. The nurse determines that the drug is effective when which sign is controlled?

You Selected:

nausea

Correct response:

nausea

Explanation:

Question 17 See full question

A client is to receive belladonna and opium suppositories, as needed, postoperatively after transurethral resection
of the prostate (TURP). The nurse should give the client these drugs when he demonstrates signs of:

You Selected:

pain from bladder spasms.

Correct response:

pain from bladder spasms.

Explanation:

Question 18 See full question

The nurse is reviewing laboratory reports for a client who is taking allopurinol. Which finding indicates that the drug
has had a therapeutic effect?

You Selected:

increased serum calcium level

Correct response:

decreased serum uric acid level

Explanation:

Question 19 See full question

The best indicator that the client has learned how to give an insulin self-injection correctly is when the client can:

You Selected:

perform the procedure safely and correctly.

Correct response:

perform the procedure safely and correctly.

Explanation:

Question 20 See full question

As an initial step in treating a client with angina, the health care provider (HCP) prescribes nitroglycerin tablets, 0.3
mg given sublingually. This drugs principal effects are produced by:
You Selected:

vasodilation of peripheral vasculature.

Correct response:

vasodilation of peripheral vasculature.

Explanation:

Question 21 See full question

A nurse is administering vitamin K to a neonate following birth. The medication comes in a concentration of 2
mg/ml, and the ordered dose is 0.5 mg to be given subcutaneously. How many milliliters would the nurse
administer? Record your answer using two decimal places.

Your Response:

Correct response:

0.25

Explanation:

Question 22 See full question

A client has been treated for major depression and is taking antidepressants. He asks the nurse, "How long do I
have to take these pills?" The nurse should tell the client:

You Selected:

"Antidepressants are prescribed for 6 to 12 months before considering discontinuation."

Correct response:

"Antidepressants are prescribed for 6 to 12 months before considering discontinuation."

Explanation:

Question 23 See full question

How many tablets should the nurse give each day to a child who is prescribed to receive mercaptopurine 75 mg/day
orally, when the pharmacy supplies mercaptopurine packaged in 50-mg tablets for oral administration? Record your
answer using one decimal place.

Your Response:

Correct response:

1.5

Explanation:

Question 24 See full question

Which measure should the nurse include in the care plan for a child who is receiving high-dose methotrexate
therapy?

You Selected:

Keep the child in a fasting state.

Correct response:

Obtain a white blood cell (WBC) count.


Explanation:

Question 25 See full question

The nurse should assess older adults for which serious adverse effects of ibuprofen?

You Selected:

impaired renal function

Correct response:

impaired renal function

Explanation:

Question 26 See full question

During the first 48 to 72 hours of fluid resuscitation therapy after a major burn injury, the nurse should
monitor hourly which information that will be used to determine the IV infusion rate?

You Selected:

body weight

Correct response:

urine output

Explanation:

Question 27 See full question

The client is taking triamcinolone acetonide inhalant to treat bronchial asthma. The nurse should assess the client
for:

You Selected:

oral candidiasis.

Correct response:

oral candidiasis.

Explanation:

Question 28 See full question

A client is diagnosed with megaloblastic anemia caused by vitamin B 12 deficiency. The physician begins the client on
cyanocobalamin (Betalin-12), 100 mcg I.M. daily. Which substance influences vitamin B 12absorption?

You Selected:

Hydrochloric acid

Correct response:

Intrinsic factor

Explanation:

Question 29 See full question

A client has an exacerbation of multiple sclerosis. The physician orders dantrolene, 25 mg P.O. daily. Which
assessment finding indicates the medication is effective?

You Selected:

Increased ability to sleep


Correct response:

Reduced muscle spasticity

Explanation:

Question 30 See full question

The nurse is preparing to administer 0.1 mg of digoxin intravenously. Digoxin comes in a concentration of 0.5 mg/2
ml. How many milliliters should the nurse administer? Record your answer using one decimal place.

Your Response:

Correct response:

0.4

Explanation:

Question 31 See full question

A child is to receive dexamethasone intravenously at the ordered dosage of 7.6 mg. The drug concentration in the
vial is 4 mg/ml. The nurse should administer?

Your Response:

Correct response:

1.9

Explanation:

Question 32 See full question

The primary health care provider (HCP) prescribes intravenous magnesium sulfate for a primigravid client at 38
weeks' gestation diagnosed with severe preeclampsia. Which medication would be most important for the nurse to
have readily available?

You Selected:

phenytoin

Correct response:

calcium gluconate

Explanation:

Question 33 See full question

Which statement by the client indicates an understanding of teaching regarding use of corticosteroids during
preterm labor?

You Selected:

"I will be taking corticosteroids until my baby's due date so that he will have the best chance of doing well."

Correct response:

The corticosteroids may help my babys lungs mature.

Explanation:

Question 34 See full question

A full-term client is admitted for an induction of labor. The health care provider (HCP) has assigned a Bishop score of
10. Which drug would the nurse anticipate administering to this client?
You Selected:

misoprostol 50 mcg

Correct response:

oxytocin 30 units in 500 ml D5W

Explanation:

Question 35 See full question

The primary care provider prescribes digoxin 0.15 mg by mouth daily, for a child. The pharmacy supplies the
digoxin in liquid form at a concentration of 0.05 mg/mL. How much of the medication should the nurse administer at
each dose? Record your answer using a whole number.

Your Response:

Correct response:

Explanation:

Question 36 See full question

The nurse is evaluating a client with hyperthyroidism who is taking propylthiouracil (PTU) 100 mg/day in three
divided doses for maintenance therapy. Which statement from the client indicates the drug is effective?

You Selected:

"I do perspire throughout the entire day."

Correct response:

"I am able to sleep and rest at night."

Explanation:

Question 37 See full question

The nurse notes grapefruit juice on the breakfast tray of a client taking repaglinide. The nurse should:

You Selected:

contact the manager of the Food and Nutrition Department.

Correct response:

remove the grapefruit juice from the client's tray and bring another juice of the client's preference.

Explanation:

Question 38 See full question

A physician orders phenytoin 150 mg by mouth twice per day for a child. The dosage strength of the oral
suspension on hand is 30 mg/5 mL. How many milliliters of suspension should the nurse administer with each dose?
Record your answer using a whole number.

Your Response:

Correct response:

25
Explanation:

Question 39 See full question

The nurse is justified in assessing for sexual dysfunction among male clients who are taking:

You Selected:

Anti-hypertensives.

Correct response:

Anti-hypertensives.

Explanation:

Question 40 See full question

A client who is 1 day postoperative is using a morphine patient-controlled analgesia (PCA) pump. The client is
confused and disoriented. What is the priority intervention by the nurse?

You Selected:

Check for shortness of breath, signifying a pulmonary embolism.

Correct response:

Check respiratory rate and depth as well as oxygen saturation levels.

Explanation:

Question 41 See full question

A client is receiving lithium carbonate for a bipolar disorder. The nurse is aware that early signs of lithium toxicity
include which of the following?

You Selected:

Diarrhea

Correct response:

Diarrhea

Explanation:

Question 42 See full question

The nurse is aware that further teaching is needed for a client receiving alprazolam by the following statement?

You Selected:

"If I drink early, having of couple glasses of wine should be fine."

Correct response:

"If I drink early, having of couple glasses of wine should be fine."

Explanation:

Question 43 See full question

Which statement about concurrent administration of ticarcillin disodium and clavulanate potassium and gentamicin
is correct?

You Selected:

The nurse should notify the physician regarding incompatibility of the drugs and request an alternative.
Correct response:

The nurse should separate the doses by at least 1 hour to prevent inactivation of gentamicin.

Explanation:

Question 44 See full question

The nurse is caring for a client with a serum sodium level of 128 mEq/L. Which orders for intravenous fluids should
the nurse should question?

You Selected:

Dextrose 5% in half-normal saline solution D5.45

Correct response:

Dextrose 5% in water (D5W)

Explanation:

Question 45 See full question

A 74-year-old client receiving fluphenazine decanoate therapy develops pseudoparkinsonism, and is ordered
amantadine hydrochloride. With the addition of this medication, the client reports feeling dizzy when standing.
Which response by the nurse is best?

You Selected:

I will talk to your doctor about taking you off of one of these medications.

Correct response:

When you change positions, do so slowly.

Explanation:

Question 46 See full question

In which of the following parts of the body should the nurse administer an intramuscular injection to a 6-month-old
infant?

You Selected:

Two finger breadths below the acromion process on the lateral side of the arm

Correct response:

The lateral middle third of the thigh between the greater trochanter and the knee

Explanation:

Question 47 See full question

A nurse is preparing to administer phenytoin to a 99-lb (45 kg) client with a seizure disorder. The medication
administration record documents phenytoin 5 mg/kg/day to be administered in three divided doses. How many
milligrams of phenytoin should be administered in the first dose? Record your answer as a whole number.

Your Response:

Correct response:

75
Explanation:

Question 48 See full question

A client with acute lymphocytic leukemia is receiving vincristine. Prior to infusing the drug, the nurse administers
diphenhydramine. The nurse should inform the client that the expected outcome of using diphenhydramine in this
situation is to:

You Selected:

reduce anxiety associated with the vincristine infusion.

Correct response:

decrease incidence of a reaction to the vincristine.

Explanation:

Question 49 See full question

The nurse is educating a client who insists that the newly prescribed imipramine is not working for her feelings of
depression. When evaluating the clients statement, which question is most important to ask first?

You Selected:

What is the dosage of medication that you are prescribed?

Correct response:

How long have you been taking the medication?

Explanation:

Question 50 See full question

The nurse is caring for an adolescent with diabetes who admits to consuming many simple sugars and
carbohydrates at a graduation party. The parents brought the client to the emergency room with unusual behavior.
The serum glucose level was 375 mg/dL (20.8 mmol/L).

The health care provider provided a coverage schedule:

150 to 200 mg/dL (8.3 to 11.1 mmol/L)2 units of Humulin R.


201 to 250 mg/dL (11.1 to 13.9 mmol/L)4 units of Humulin R.
251 to 300 mg/dL (13.9 to 16.7 mmol/L)6 units of Humulin R.
301 to 350 mg/dL (16.7 to 19.4 mmol/L)8 units of Humulin R.
351 to 399 mg/dL (19.5 to 22.1 mmol/L)10 units of Humulin R.
Over 400 mg/dL 22.2 (mmol/L)call the health care provider.

Select the line on the low-dose insulin syringe corresponding to the amount of insulin that should be drawn up.
You Selected:

Your selection and the correct area, market by the green box.

Explanation:

Question 1 See full question

A physician orders cefoxitin, 1 g in 100 ml of 5% dextrose in water, to be administered I.V. A nurse determines that
the recommended infusion time is 15 to 30 minutes. The available infusion set has a calibration of 10 drops/ml. To
infuse cefoxitin over 30 minutes, which drip rate should the nurse use?

You Selected:

30 drops/minute

Correct response:

33 drops/minute

Explanation:

Question 2 See full question

A client has a nasogastric (NG) tube. How should the nurse administer oral medication to this client?
You Selected:

Crush the tablets and wash the powder down the NG tube, using a syringe filled with saline solution.

Correct response:

Crush the tablets and prepare a liquid form; then insert the liquid into the NG tube.

Explanation:

Question 3 See full question

The nurse is reconstituting a powdered medication in a vial. After adding the solution to the powder, the nurse
should:

You Selected:

invert the vial and let it stand for 2 to 3 minutes.

Correct response:

roll the vial gently between her palms.

Explanation:

Question 4 See full question

After knee replacement surgery, a client is being discharged with acetaminophen with codeine 30 mg tablets for
pain. During discharge preparation, the nurse should include which instruction?

You Selected:

"Decrease your fluid intake to two glasses daily."

Correct response:

"Avoid driving a car while taking this medication."

Explanation:

Question 5 See full question

A physician orders the following preoperative medications to be administered to a client by the I.M. route:
meperidine, 50 mg; hydroxyzine pamoate, 25 mg; and glycopyrrolate, 0.3 mg. The medications are dispensed as
follows: meperidine, 100 mg/ml; hydroxyzine pamoate, 100 mg/2 ml; and glycopyrrolate, 0.2 mg/ml. How many
milliliters in total should the nurse administer?

You Selected:

2.5 ml

Correct response:

2.5 ml

Explanation:

Question 6 See full question

A nurse has an order to administer iron dextran 50 mg I.M. injection. When carrying out this order, the nurse should:

You Selected:

pull the skin laterally toward the injection site.

Correct response:

use the Z-track technique.


Explanation:

Question 7 See full question

Why would a nurse be interested in a client's dietary history when administering drugs?

You Selected:

The number of calories a client consumes can alter a drug's metabolism.

Correct response:

Dietary intake can alter the effectiveness of some drugs.

Explanation:

Question 8 See full question

A child with diabetes insipidus receives desmopressin acetate. When evaluating for therapeutic effectiveness, the
nurse should interpret which finding as a positive response to this drug?

You Selected:

Decreased urine output

Correct response:

Decreased urine output

Explanation:

Question 9 See full question

A client who has received a new prescription for oral contraceptives asks the nurse how to take them. Which
symptom should the nurse instruct the client to report to her primary caregiver?

You Selected:

Breakthrough bleeding within first 3 months of use

Correct response:

Blurred vision and headache

Explanation:

Question 10 See full question

A client with a severe staphylococcal infection is receiving the aminoglycoside gentamicin sulfate by the I.V. route.
The nurse should assess the client for which adverse reaction?

You Selected:

Seizures

Correct response:

Ototoxicity

Explanation:

Question 11 See full question

A client with rheumatoid arthritis is being discharged with a prescription for aspirin, 600 mg P.O. every 6 hours.
Which statement by the client indicates understanding of the adverse effects of the medication?

You Selected:

"I'll call my physician if I have ringing in the ears."


Correct response:

"I'll call my physician if I have ringing in the ears."

Explanation:

Question 12 See full question

A client is receiving paroxetine 20 mg every morning. After taking the first three doses, the client tells the nurse
that the medication upsets his stomach. What instruction should the nurse give to the client?

You Selected:

"Take the medication with 4 oz (120 mL) of orange juice."

Correct response:

"Take the medication with some food."

Explanation:

Question 13 See full question

The nurse hands the medication cup to a client who is psychotic and exhibiting concrete thinking, and tells the
client to take his medicine. The client takes the cup, holds it in his hand, and stares at it. What should the nurse
do next?

You Selected:

Ask another staff member to stay with the client until he takes the medication.

Correct response:

Tell the client to put the medicine in his mouth and swallow it with some water.

Explanation:

Question 14 See full question

Which information should the nurse include when teaching the family and a client who was prescribed benztropine,
1 mg PO twice daily, about the drug therapy?

You Selected:

The drug can be used with over-the-counter cough and cold preparations.

Correct response:

The client should not discontinue taking the drug abruptly.

Explanation:

Question 15 See full question

The parents of a child on sulfamethoxazole and trimethoprim for a urinary tract infection report that the child has a
red, blistery rash. The nurse should tell the parents to:

You Selected:

apply lotion to the affected areas.

Correct response:

discontinue the medicine and come for immediate further evaluation.

Explanation:

Question 16 See full question

A 75-year-old client who has been taking furosemide regularly for 4 months tells the nurse about having trouble
hearing. What should the nurse do?
You Selected:

Tell the client that the hearing loss is only temporary; when the body adjusts to the furosemide, hearing will
improve.

Correct response:

Report the hearing loss to the health care provider (HCP).

Explanation:

Question 17 See full question

An older adult takes two 81-mg aspirin tablets daily to prevent a heart attack. The client reports having a constant
"ringing" in both ears. How should the nurse respond to the client's comment?

You Selected:

Schedule the client for audiometric testing.

Correct response:

Explain to the client that the "ringing" may be related to the aspirin.

Explanation:

Question 18 See full question

The nurse assesses a client who has just received morphine sulfate. The client's blood pressure is 90/50 mm Hg;
pulse rate, 58 bpm; respiration rate, 4 breaths/min. The nurse should check the client's chart for a prescription to
administer:

You Selected:

naloxone hydrochloride.

Correct response:

naloxone hydrochloride.

Explanation:

Question 19 See full question

At which time should the nurse instruct the client to take ibuprofen, prescribed for left hip pain secondary to
osteoarthritis, to minimize gastric mucosal irritation?

You Selected:

on an empty stomach

Correct response:

immediately after a meal

Explanation:

Question 20 See full question

A client is to be discharged with a prescription for lactulose. The nurse teaches the client and the clients spouse
how to administer this medication. Which statement would indicate that the client has understood the information?

You Selected:

"I will mix it with apple juice."

Correct response:

"I will mix it with apple juice."


Explanation:

Question 21 See full question

The nurse should caution sexually active female clients taking isoniazid (INH) that the drug:

You Selected:

decreases the effectiveness of hormonal contraceptives.

Correct response:

decreases the effectiveness of hormonal contraceptives.

Explanation:

Question 22 See full question

The client with peripheral vascular disease has been prescribed diltiazem. The nurse should determine the
effectiveness of this medication by assessing the client for:

You Selected:

sedation.

Correct response:

vasodilation.

Explanation:

Question 23 See full question

Which is an expected outcome when a client is receiving an IV administration of furosemide?

You Selected:

decreased pain

Correct response:

increased urine output

Explanation:

Question 24 See full question

When teaching a client about propranolol hydrochloride, the nurse should base the information on the knowledge
that propranolol:

You Selected:

is an angiotensin-converting enzyme inhibitor that reduces blood pressure by blocking the conversion of
angiotensin I to angiotensin II.

Correct response:

blocks beta-adrenergic stimulation and thus causes decreased heart rate, myocardial contractility, and
conduction.

Explanation:

Question 25 See full question

When preparing a teaching plan for a client who is to receive a rubella vaccine during the postpartum period, the
nurse should include which information?

You Selected:

The vaccine prevents a future fetus from developing congenital anomalies.


Correct response:

Pregnancy should be avoided for 4 weeks after the immunization.

Explanation:

Question 26 See full question

A cardiologist prescribes digoxin 125 mcg by mouth every morning for a client diagnosed with heart failure. The
pharmacy dispenses tablets that contain 0.25 mg each. How many tablet(s) would the nurse administer in each
dose? Record your answer using one decimal place. (For example: 6.2)

Your Response:

Correct response:

0.5

Explanation:

Question 27 See full question

A neonate weighing 1,870 g with a respiratory rate of 46 breaths/minute, a pulse rate of 175 bpm, and a serum pH
of 7.11 has received sodium bicarbonate intravenously. The drug has been effective if the neonate:

You Selected:

is not dehydrated.

Correct response:

does not go into metabolic acidosis.

Explanation:

Question 28 See full question

How many tablets should the nurse give each day to a child who is prescribed to receive mercaptopurine 75 mg/day
orally, when the pharmacy supplies mercaptopurine packaged in 50-mg tablets for oral administration? Record your
answer using one decimal place.

Your Response:

Correct response:

1.5

Explanation:

Question 29 See full question

When administering an IM injection to a neonate, which muscle should the nurse consider as the best injection site?

You Selected:

deltoid

Correct response:

vastus lateralis

Explanation:

Question 30 See full question

A child is to receive IV fluids at a rate of 95 mL/h. The tubing for the infusion delivers 10 drops/mL. At which rate
should the nurse infuse the solution?
You Selected:

20 drops/min

Correct response:

16 drops/min

Explanation:

Question 31 See full question

In teaching a client with tuberculosis about self-care at home, which directive has the highest priority?

You Selected:

Take medications as prescribed.

Correct response:

Take medications as prescribed.

Explanation:

Question 32 See full question

The client with a urinary tract infection is given a prescription for trimethoprim. Which statement indicates that the
client understands how to take the medication?

You Selected:

I will take the pills until the symptoms go away and then reduce the dose to one pill a day.

Correct response:

"I will take all the pills and then return to my doctor.

Explanation:

Question 33 See full question

The nurse has a prescription to administer ampicillin 250 mg IM. After reconstituting the ampicillin with sterile water
for injection, the solution available is 500 mg/mL. How many milliliters should the nurse administer? Record your
answer using one decimal place.

Your Response:

Correct response:

0.5

Explanation:

Question 34 See full question

When a central venous catheter dressing becomes moist or loose, what should a nurse do first?

You Selected:

Remove the catheter, check for catheter integrity, and send the tip for culture.

Correct response:

Remove the dressing, clean the site, and apply a new dressing.
Explanation:

Question 35 See full question

A client is taking spironolactone to control her hypertension. Her serum potassium level is [6 mEq/L (56mmol/L)].
For this client, the nurse's priority should be to assess her:

You Selected:

electrocardiogram (ECG) results.

Correct response:

electrocardiogram (ECG) results.

Explanation:

Question 36 See full question

The primary healthcare provider (HCP) orders 1,000 mL of Ringer's Lactate intravenously over an 8-hour period for
a 29-year-old primigravid client at 16 weeks gestation with hyperemesis. The drip factor is 12 gtts/mL. The nurse
should administer the IV infusion at how many drops per minute? Record your answer as a whole number.

Your Response:

Correct response:

25

Explanation:

Question 37 See full question

A client with depression states, "I am still feeling nauseous after I take venlafaxine. Maybe I need something else."
The nurse should tell the client to:

You Selected:

take venlafaxine before bedtime.

Correct response:

take the medication at mealtime.

Explanation:

Question 38 See full question

A clients wife states, I do not think lithium is helping my husband. He has been taking it for 2 days now, and he is
still so hyper and thinks we are rich. Which response by the nurse would be most accurate?

You Selected:

It takes 1 to 2 weeks for the drug to build up in the blood to be effective.

Correct response:

It takes 1 to 2 weeks for the drug to build up in the blood to be effective.

Explanation:

Question 39 See full question

A client is taking phenelzine 15 mg PO three times a day. The nurse is about to administer the next dose when the
client tells the nurse about having a throbbing headache. Which action should the nurse do first?

You Selected:

Give the client an analgesic prescribed PRN.


Correct response:

Obtain the client's vital signs.

Explanation:

Question 40 See full question

A nulliparous client has been given a prescription for oral contraceptives. The nurse should instruct the client to
report which sign to the health care provider (HCP) immediately?

You Selected:

blurred vision

Correct response:

blurred vision

Explanation:

Question 41 See full question

The health care provider has prescribed penicillin for a client admitted to the hospital for treatment of pneumonia.
Prior to administering the first dose of penicillin, the nurse should ask the client:

You Selected:

"Do you have any cardiac history?"

Correct response:

"Have you had a previous allergy to penicillin?"

Explanation:

Question 42 See full question

A 7-year-old has been diagnosed with bacterial meningitis. Who should receive chemoprophylaxis?

You Selected:

all household contacts and close contacts

Correct response:

all household contacts and close contacts

Explanation:

Question 43 See full question

The healthcare provider prescribes venlafaxinefor the client. The nurse explains the purpose of the medication to
the client. The client asks the nurse, If I start taking the pills, will I have to take them the rest of my life? Which
would be the nurses most accurate and therapeutic reply?

You Selected:

"I would hope not!"

Correct response:

"After your symptoms decrease, the need for medication will be reevaluated."

Explanation:

Question 44 See full question

A clients diagnosis of pneumonia requires treatment with antibiotics. The corresponding order in the clients chart
should be written as:
You Selected:

Moxifloxacin 400 mg Q.D.

Correct response:

Moxifloxacin 400 mg daily.

Explanation:

Question 45 See full question

The nurse ascertains that there is a discrepancy in the records of use of a controlled substance for a client who is
taking large doses of narcotic pain medication. What should the nurse do next?

You Selected:

Contact the hospital's administration or legal department.

Correct response:

Notify the nursing supervisor of the clinical unit.

Explanation:

Question 46 See full question

The nurse is educating the parents of a 2-year-old child regarding immunizations. When the parents ask where the
injections will be given the nurse answers that the most appropriate site for an intramuscular injection for a child
this age is the:

You Selected:

vastus lateralis muscle.

Correct response:

vastus lateralis muscle.

Explanation:

Question 47 See full question

Which category of medications would the nurse expect to administer for a client with myasthenia gravis?

You Selected:

Cholinesterase inhibitors and corticosteroids

Correct response:

Cholinesterase inhibitors and corticosteroids

Explanation:

Question 48 See full question

A client with an intravenous (IV) site is experiencing pain. The nurse understands that pain with infusion is a sign of
which of the following?

You Selected:

Fibrin sheath occlusion

Correct response:

Catheter position at the insertion site due to movement


Explanation:

Question 49 See full question

The nurse is to administer midazolam 2.5 mg. The medication is available in a 5 mg/mL vial. How many mL should
the nurse administer? Record your answer using one decimal point.

Your Response:

Correct response:

0.5

Explanation:

Question 50 See full question

The nurse is caring for a client with blood pressure of 210/94 mm Hg. The health care provider prescribes Vasotec
20 mg b.i.d. Which nursing action is best when administering a new blood pressure medication to a client?

You Selected:

Inform the client about the new medication and provide a handout.

Correct response:

Inform the client about the new medication, including its name, use, and the reason for the change in
medication.

Explanation:

Question 1 See full question

A nurse has been teaching a client how to use an incentive spirometer that he must use at home for several days
after discharge. Which client action indicates an accurate understanding of the technique?

You Selected:

The client tilts the spirometer down when using it.

Correct response:

The client takes slow, deep breaths to elevate the spirometer ball.

Explanation:

Question 2 See full question

A client with burns on his groin has developed blisters. As the client is bathing, a few blisters break. The best action
for the nurse to take is to:

You Selected:

clean the area with normal saline solution and cover it with a protective dressing.

Correct response:

clean the area with normal saline solution and cover it with a protective dressing.

Explanation:

Question 3 See full question

How should a nurse position a 4-month-old infant when administering an oral medication?

You Selected:

Held on the nurse's lap


Correct response:

Held in the bottle-feeding position

Explanation:

Question 4 See full question

Parents bring their infant to the clinic, seeking treatment for vomiting and diarrhea that has lasted for 2 days. On
assessment, the nurse detects dry mucous membranes and lethargy. What other finding suggests a fluid volume
deficit?

You Selected:

A sunken fontanel

Correct response:

A sunken fontanel

Explanation:

Question 5 See full question

A toddler with hemophilia is hospitalized with multiple injuries after falling off a sliding board. X-rays reveal no bone
fractures. When caring for the child, what is the nurse's highest priority?

You Selected:

Frequently assessing the child's level of consciousness (LOC)

Correct response:

Frequently assessing the child's level of consciousness (LOC)

Explanation:

Question 6 See full question

A child, age 2, with a history of recurrent ear infections is brought to the clinic with a fever and irritability. To elicit
the most pertinent information about the child's ear problems, the nurse should ask the parent:

You Selected:

"Does your child have any hearing problems?"

Correct response:

"Does your child tug at either ear?"

Explanation:

Question 7 See full question

A child with a full-thickness burn is scheduled for debridement using hydrotherapy. Before hydrotherapy begins, the
nurse should:

You Selected:

provide nutritional supplements.

Correct response:

implement pain control measures.


Explanation:

Question 8 See full question

Parents of a preschooler with chickenpox ask the nurse about measures to make their child comfortable. The nurse
instructs the parents to avoid administering aspirin or any other product that contains salicylates. When given to
children with chickenpox, aspirin has been linked to which disorder?

You Selected:

Reye's syndrome

Correct response:

Reye's syndrome

Explanation:

Question 9 See full question

A nurse is assessing a 15-year-old girl who has lost 30 lb (13.6 kg) over 3 months. What other finding is the nurse
likely to assess?

You Selected:

dysphagia.

Correct response:

amenorrhea.

Explanation:

Question 10 See full question

Which foods are contraindicated for a client taking tranylcypromine?

You Selected:

Oranges and vodka

Correct response:

Chicken livers, Chianti wine, and beer

Explanation:

Question 11 See full question

A client with major depression is taking tranylcypromine sulfate, a monoamine oxidase (MAO) inhibitor. The nurse
understands that additional teaching is needed when the client expresses he ate which food?

You Selected:

Whole grain bread

Correct response:

Aged cheese

Explanation:

Question 12 See full question

A nurse is assessing a client who has just been admitted to the emergency department. Which signs suggest an
overdose of an antianxiety agent?

You Selected:

Suspiciousness, dilated pupils, and increased blood pressure


Correct response:

Slurred speech, dyspnea, and impaired coordination

Explanation:

Question 13 See full question

A nurse assesses a client during the third stage of labor. Which assessment findings indicate that the client is
experiencing postpartum hemorrhage?

You Selected:

Heart rate 50 beats/minute, respiratory rate 8 breaths/minute, blood pressure 150/100 mm Hg

Correct response:

Heart rate 120 beats/minute, respiratory rate 28 breaths/minute, blood pressure 80/40 mm Hg

Explanation:

Question 14 See full question

What assessment data of a laboring woman would require further intervention by the nurse?

You Selected:

Fetal heart rate (FHR) 150 beats/minute

Correct response:

Maternal heart rate 125 beats/minute

Explanation:

Question 15 See full question

After completing discharge instructions for a primiparous client who is bottle-feeding her term neonate, the nurse
determines that the mother understands the instructions when the mother says that she should contact the
pediatrician if the neonate exhibits which sign or symptom?

You Selected:

passage of a liquid stool with a watery ring

Correct response:

passage of a liquid stool with a watery ring

Explanation:

Question 16 See full question

After the birth of a viable neonate, a 20-year-old primiparous client comments to her mother and the nurse about
the baby. Which comment would the nurse interpret as a possible sign of potential maternal-infant bonding
problems?

You Selected:

"I think my mother should give him the first feeding."

Correct response:

"I think my mother should give him the first feeding."


Explanation:

Question 17 See full question

A multigravid client who stands for long periods while working in a factory visits the prenatal clinic at 35 weeks'
gestation, stating, "The varicose veins in my legs have really been bothering me lately." Which instruction would
be most helpful?

You Selected:

Avoid support hose that reach above the leg varicosities.

Correct response:

Take frequent rest periods with the legs elevated above the hips.

Explanation:

Question 18 See full question

Which client statement indicates an understanding of the risk of alcohol relapse?

You Selected:

"If someone tells me I'm about to relapse, I will be sure to do something about it."

Correct response:

"Stopping Alcoholics Anonymous (AA) and not expressing feelings can lead to relapse."

Explanation:

Question 19 See full question

Which baseline laboratory data should be established before a client is started on tissue plasminogen activator or
alteplase recombinant?

You Selected:

blood glucose level

Correct response:

hemoglobin level, hematocrit, and platelet count

Explanation:

Question 20 See full question

A 70-year-old, previously well client asks the nurse, "I notice I have tremors. Is this just normal for my age?" What
should the nurse tell the client?

You Selected:

You should report this to the health care provider because it may indicate a problem.

Correct response:

You should report this to the health care provider because it may indicate a problem.

Explanation:

Question 21 See full question

The results of which serologic test should the nurse have on the medical record before a client is started on tissue
plasminogen activator or alteplase recombinant?

You Selected:

partial thromboplastin time


Correct response:

partial thromboplastin time

Explanation:

Question 22 See full question

A 10-month-old child has cold symptoms. The mother asks how she can clear the infants nose. What would be the
nurses best recommendation?

You Selected:

Blow into the child's mouth to clear the infant's nose.

Correct response:

Use saline nose drops and then a bulb syringe.

Explanation:

Question 23 See full question

A parent brings a 3-month-old infant to the clinic, reporting that the infant has a cold, is having trouble breathing,
and just does not seem to be acting right. Which action should the nurse take first?

You Selected:

Obtain more information from the father.

Correct response:

Assess the infant's oxygen saturation.

Explanation:

Question 24 See full question

A parent asks, "How should I bathe my baby now that he has had surgery for his inguinal hernia?" Which instruction
should the nurse give the mother?

You Selected:

"Use sterile sponges to cleanse the inguinal incision."

Correct response:

"Give him a sponge bath daily for 1 week."

Explanation:

Question 25 See full question

A client has a leg immobilized in traction. Which observation by the nurse indicates that the client understands
actions to take to prevent muscle atrophy?

You Selected:

The client asks the nurse to add a 5-lb (2.3-kg) weight to the traction for 30 minutes per day.

Correct response:

The client performs isometric exercises to the affected extremity three times per day.

Explanation:

Question 26 See full question

As the nurse assists the postoperative client out of bed, the client reports having gas pains in the abdomen. To
reduce this discomfort, what should the nurse do?
You Selected:

Encourage the client to drink carbonated liquids.

Correct response:

Encourage the client to ambulate.

Explanation:

Question 27 See full question

A nurse is helping a suspected choking victim. The nurse should perform the Heimlich maneuver when the victim:

You Selected:

cannot speak due to airway obstruction.

Correct response:

cannot speak due to airway obstruction.

Explanation:

Question 28 See full question

Immediately after the return of an 18-month-old child to his room following insertion of a ventriculoperitoneal shunt,
the nurse should first:

You Selected:

position him on the side opposite the shunt site.

Correct response:

position him on the side opposite the shunt site.

Explanation:

Question 29 See full question

The expected outcome of withholding food and fluids from a client who will receive general anesthesia is to help
prevent:

You Selected:

pressure on the diaphragm with poor lung expansion during surgery.

Correct response:

vomiting and possible aspiration of vomitus during surgery.

Explanation:

Question 30 See full question

The nurse monitors IV replacement therapy for a client with a nasogastric (NG) tube attached to low suction in order
to:

You Selected:

maintain fluid and electrolyte balance.

Correct response:

maintain fluid and electrolyte balance.


Explanation:

Question 31 See full question

A client comes to the physician's office for a follow-up visit 4 weeks after suffering a myocardial infarction (MI). The
nurse takes this opportunity to evaluate the client's knowledge of the ordered cardiac rehabilitation program. Which
evaluation statement suggests that the client needs more instruction?

You Selected:

"Client verbalizes an understanding of the need to seek emergency help if his heart rate increases markedly
while at rest."

Correct response:

"Client walks 4 miles (6.4 kilometers) in 1 hour every day."

Explanation:

Question 32 See full question

A client receiving chemotherapy has a nursing diagnosis of Deficient diversional activity related to decreased
energy. Which client statement indicates an accurate understanding of appropriate ways to deal with this deficit?

You Selected:

"I'll bowl with my team after discharge."

Correct response:

"I'll play card games with my friends."

Explanation:

Question 33 See full question

A client is diagnosed with a hiatal hernia. Which statement indicates effective client teaching about hiatal hernia
and its treatment?

You Selected:

"I'll eat three large meals every day without any food restrictions."

Correct response:

"I'll eat frequent, small, bland meals that are high in fiber."

Explanation:

Question 34 See full question

A client is scheduled for a prostatectomy, and the anesthetist plans to use a spinal (subarachnoid) block during
surgery. In the operating room, the nurse positions the client according to the anesthetist's instructions. Why does
the client require special positioning for this type of anesthesia?

You Selected:

To prevent cerebrospinal fluid (CSF) leakage

Correct response:

To prevent cerebrospinal fluid (CSF) leakage

Explanation:

Question 35 See full question

The nurse is planning care for a client on complete bed rest. The plan of care should include all except:
You Selected:

use of thromboembolic disease support (TED) hose.

Correct response:

maintaining the client in the supine position.

Explanation:

Question 36 See full question

A 3-month-old infant just had a cleft lip and palate repair. To prevent trauma to the operative site, the nurse should:

You Selected:

place the infant's arms in soft elbow restraints.

Correct response:

place the infant's arms in soft elbow restraints.

Explanation:

Question 37 See full question

A client with chronic renal failure is admitted with a heart rate of 122 beats/minute, a respiratory rate of 32
breaths/minute, a blood pressure of 190/110 mm Hg, jugular vein distention, and bibasilar crackles. Which nursing
diagnosis takes highest priority for this client?

You Selected:

Toileting self-care deficit

Correct response:

Excess fluid volume

Explanation:

Question 38 See full question

Assessment of a term neonate at 2 hours after birth reveals a heart rate of less than 100 bpm, periods of apnea
approximately 25 to 30 seconds in length, and mild cyanosis around the mouth. The nurse notifies the health care
provider (HCP) based on the interpretation that these findings may lead to which condition?

You Selected:

intraventricular hemorrhage

Correct response:

respiratory arrest

Explanation:

Question 39 See full question

The most reliable early indicator of infection in a client who is neutropenic is:

You Selected:

tachycardia.

Correct response:

fever.
Explanation:

Question 40 See full question

A clients 1200 blood glucose was inaccurately documented as 310 mg/dL (17.2 mmol/L) instead of 130 mg/dL (7.2
mmol/L). This error was not noticed until 1300. The nurse administered the sliding scale insulin for a blood glucose
of 310 mg/dL (17.2 mmol/L). What should the nurse do first?

You Selected:

Notify the health care provider (HCP).

Correct response:

Assess for hypoglycemia.

Explanation:

Question 41 See full question

A client is admitted to the psychiatric unit following a suicide attempt. The client has suffered identity theft through
the Internet and states, My savings, checking, and retirement accounts are empty. I have nothing left to pay my
bills or buy food and medicines. The only thing left is to die. After 1 week, the nurse would conclude that the client
has been helped upon hearing which statements? Select all that apply.

You Selected:

*I filed identity theft claims with the bank, my retirement account, and the government authorities.

Correct response:

"I realize that I still can get monthly public assistance benefits."
*I filed identity theft claims with the bank, my retirement account, and the government authorities.
"With all the help I got here, I think I may be able to survive after all."

Explanation:

Question 42 See full question

The nurse provides teaching on postoperative wound care to a client being discharged from a surgical unit. Which of
the following statements documented by the nurse indicates that the client understood the teaching?

You Selected:

Client verbalized to the nurse the steps to follow if wound becomes red and warm.

Correct response:

Client verbalized to the nurse the steps to follow if wound becomes red and warm.

Explanation:

Question 43 See full question

A nurse is caring for a client who had a prostatectomy for prostate cancer. The nurse is reviewing the client's vital
signs and intake and output as documented by a nursing assistant. Which documented finding requires immediate
action?

You Selected:

Intake and output

Correct response:

Intake and output


Explanation:

Question 44 See full question

A clients arterial blood gas values are shown. The nurse should monitor the client for:

You Selected:

respiratory acidosis

Correct response:

metabolic acidosis

Explanation:

Question 45 See full question

The nurse is monitoring a client admitted with a myocardial infarction (MI) who is at risk for cardiogenic shock. The
nurse should report which change on the clients chart to the health care provider (HCP)?

You Selected:

blood pressure

Correct response:

urine output

Explanation:

Question 46 See full question

An 18-year-old client is seen in the emergency department following a fall from a horse. After vigorously cleaning a
large, dirty laceration on his/her leg, a nurse dresses the wound. The client has received the full tetanus-toxoid
immunization regimen at 11 years old. How should the nurse proceed with this client's care?

You Selected:

Collaborate with the practitioner for administering a dose of tetanus vaccine.

Correct response:

Collaborate with the practitioner for administering a dose of tetanus vaccine.

Explanation:

Question 47 See full question

The nurse is caring for a client in labor. The nurse notes variable decelerations on the fetal monitor strip. Which of
the following interventions should the nurse include in the clients plan of care?

You Selected:

Administering terbutaline subcutaneously

Correct response:

Repositioning the client on her side

Explanation:

Question 48 See full question

A nurse is caring for a client who is receiving chemotherapy for lung cancer. During the hand-off report, the nurse
from the previous shift states that the client has been placed on neutropenic precautions. Which laboratory value
supports this nursing action?

You Selected:

A red blood cell count of 3.5 million/mm3


Correct response:

A white blood cell count of 2200/mm3

Explanation:

Question 49 See full question

A 35-year-old client who is 28 weeks pregnant is admitted for testing. After reading the nursing notes, which
rationale best explains why a pregnant client would lie on her left side when resting or sleeping in the later stages
of pregnancy?

You Selected:

To prevent development of fetal anomalies.

Correct response:

To prevent compression of the vena cava.

Explanation:

Question 50 See full question

The nurse is assessing a client who is 4 hours postpartum. Based on the findings documented by the nurse, which
action is most appropriate at this time?

You Selected:

Straight-catheterize the client for half of her urine volume.

Correct response:

Ask the client to empty her bladder.

Explanation:

Question 1 See full question

Parents bring their infant to the clinic, seeking treatment for vomiting and diarrhea that has lasted for 2 days. On
assessment, the nurse detects dry mucous membranes and lethargy. What other finding suggests a fluid volume
deficit?

You Selected:

Decreased pulse rate

Correct response:

A sunken fontanel

Explanation:

Question 2 See full question

A nurse is preparing a child, age 4, for cardiac catheterization. Which explanation of the procedure is appropriate?

You Selected:

"You must sleep the whole time that the test is being done."

Correct response:

"The special medicine will feel warm when it's put in the tubing."
Explanation:

Question 3 See full question

Parents of a preschooler with chickenpox ask the nurse about measures to make their child comfortable. The nurse
instructs the parents to avoid administering aspirin or any other product that contains salicylates. When given to
children with chickenpox, aspirin has been linked to which disorder?

You Selected:

Guillain-Barr syndrome

Correct response:

Reye's syndrome

Explanation:

Question 4 See full question

A 13-year-old adolescent may have appendicitis. Which finding is a reliable indicator of appendicitis?

You Selected:

A history of vomiting and diarrhea, if present

Correct response:

The severity, location, and movement of pain

Explanation:

Question 5 See full question

A client with major depression is taking tranylcypromine sulfate, a monoamine oxidase (MAO) inhibitor. The nurse
understands that additional teaching is needed when the client expresses he ate which food?

You Selected:

Whole grain bread

Correct response:

Aged cheese

Explanation:

Question 6 See full question

A client recently admitted to the hospital with sharp, substernal chest pain suddenly reports palpitations. The client
ultimately admits to using cocaine 1 hour before admission. The nurse should immediately assess the client's:

You Selected:

anxiety level.

Correct response:

pulse rate and character.

Explanation:

Question 7 See full question

A client, 30 weeks pregnant, is scheduled for a biophysical profile (BPP) to evaluate the health of her fetus. Her BPP
score is 8. What does this score indicate?

You Selected:

The client should repeat the test in 24 hours.


Correct response:

The fetus isn't in distress at this time.

Explanation:

Question 8 See full question

A pregnant adolescent admitted with premature uterine contractions was successfully treated with I.V. fluids. She is
eager to return to high school to take a math test. The nurse's discharge examination reveals vaginal blood pooling
under the adolescent's buttocks that's painless to the client. Which action should the nurse take?

You Selected:

Encourage developmental growth by wishing the client luck on her math test.

Correct response:

Stop the discharge process and notify the physician immediately.

Explanation:

Question 9 See full question

A primigravid client is admitted to the labor and delivery area, where the nurse evaluates her. Which assessment
finding may indicate the need for cesarean birth?

You Selected:

Fetal prematurity

Correct response:

Umbilical cord prolapse

Explanation:

Question 10 See full question

A nurse is caring for a client with mild active bleeding from placenta previa. Which assessment factor indicates that
an emergency cesarean birth may be necessary?

You Selected:

Decreased amount of vaginal bleeding

Correct response:

Fetal heart rate of 80 beats/minute

Explanation:

Question 11 See full question

When assessing the fetal heart rate tracing, a nurse becomes concerned about the fetal heart rate pattern. In
response to the loss of variability, the nurse repositions the client to her left side and administers oxygen. These
actions are likely to improve:

You Selected:

fetal hypoxia.

Correct response:

fetal hypoxia.
Explanation:

Question 12 See full question

A nurse needs to obtain a good monitor tracing on a client in labor The client lies in a supine position. Suddenly, she
complains of feeling light-headed and becomes diaphoretic. Which action should the nurse perform first?

You Selected:

Increase the I.V. fluids to correct the client's dehydration.

Correct response:

Reposition the client to her left side.

Explanation:

Question 13 See full question

A home care nurse is making the initial home visit to a client with lung cancer who had a peripherally inserted
central catheter placed during hospitalization for an upper respiratory infection. During the visit, the nurse must
administer an antibiotic, teach the client how to care for the catheter, and provide information about when to notify
the home care agency and physician. When the nurse arrives at the client's home, the client's face is flushed and
he complains of feeling tired. Which actions should the nurse take first?

You Selected:

Administer the antibiotic, obtain vital signs, assess breath sounds, and then begin the teaching session.

Correct response:

Obtain the client's vital signs and assess breath sounds.

Explanation:

Question 14 See full question

The nurse is caring for a neonate at 38 weeks gestation when the nurse observes marked peristaltic waves on the
neonates abdomen. After this observation, the neonate exhibits projectile vomiting. The nurse notifies the health
care provider (HCP) because these signs are indicative of which problem?

You Selected:

pyloric stenosis

Correct response:

pyloric stenosis

Explanation:

Question 15 See full question

When developing the plan of care for a multigravid client with class III heart disease, the nurse should expect to
assess the client frequently for which problem?

You Selected:

tachycardia

Correct response:

tachycardia

Explanation:

Question 16 See full question

When teaching a primigravid client at 24 weeks' gestation about the diagnostic tests to determine fetal well-being,
which information should the nurse include?
You Selected:

A fetal biophysical profile involves assessments of breathing movements, body movements, tone, amniotic
fluid volume, and fetal heart rate reactivity.

Correct response:

A fetal biophysical profile involves assessments of breathing movements, body movements, tone, amniotic
fluid volume, and fetal heart rate reactivity.

Explanation:

Question 17 See full question

A client whose condition remains stable after a myocardial infarction gradually increases activity. To determine
whether the activity is appropriate for the client the nurse should assess the client for:

You Selected:

edema.

Correct response:

dyspnea.

Explanation:

Question 18 See full question

A clients burn wounds are being cleaned twice a day in a hydrotherapy tub. Which intervention should be included
in the plan of care before a hydrotherapy treatment is initiated?

You Selected:

Apply a topical antibiotic cream to burns to prevent infection.

Correct response:

Administer pain medication 30 minutes before therapy to help manage pain.

Explanation:

Question 19 See full question

When assessing for oxygenation in a client with dark skin, the nurse should examine the client's:

You Selected:

buccal mucosa.

Correct response:

buccal mucosa.

Explanation:

Question 20 See full question

When monitoring a client who is receiving tissue plasminogen activator (t-PA), the nurse should have resuscitation
equipment available because reperfusion of the cardiac tissue can result in:

You Selected:

seizure.

Correct response:

cardiac arrhythmias.
Explanation:

Question 21 See full question

A client with eclampsia begins to experience a seizure. Which intervention should the nurse do immediately?

You Selected:

Pad the side rails.

Correct response:

Maintain a patent airway.

Explanation:

Question 22 See full question

When obtaining a blood sample to screen a neonate for phenylketonuria (PKU), the nurse should obtain the sample
from the:

You Selected:

brachial artery.

Correct response:

heel.

Explanation:

Question 23 See full question

A parent of a child with hemophilia states that she worries whenever the child has a bump or cut. The nurse should
explain that after the area is cleansed, the wound should be treated by applying which measure?

You Selected:

a tourniquet above the injured area

Correct response:

gentle pressure

Explanation:

Question 24 See full question

Immediately following endoscopy of the upper gastrointestinal tract, it is most important for the nurse to assess
for:

You Selected:

bowel sounds.

Correct response:

return of the gag reflex.

Explanation:

Question 25 See full question

A client is scheduled for oral cholecystography. Prior to the test, the nurse should:

You Selected:

administer an intravenous contrast agent the evening before the test.


Correct response:

ask the client about possible allergies to iodine or shellfish.

Explanation:

Question 26 See full question

In a client with chronic bronchitis, which sign would lead the nurse to suspect right-sided heart failure?

You Selected:

Cyanosis of the lips

Correct response:

Leg edema

Explanation:

Question 27 See full question

A client who underwent total hip replacement exhibits a red, painful area on the calf of the affected leg. What test
validates presence of thromboembolism?

You Selected:

Homans'

Correct response:

Homans'

Explanation:

Question 28 See full question

The ABCD method offers one way to assess skin lesions for possible skin cancer. What does the A stand for?

You Selected:

Assessment

Correct response:

Asymmetry

Explanation:

Question 29 See full question

A nurse is assessing a client who reports abdominal pain, nausea, and diarrhea. The nurse knows that palpating the
abdomen first would:

You Selected:

be an appropriate intervention.

Correct response:

alter abdominal sounds from baseline.

Explanation:

Question 30 See full question

A client tells the nurse that she has been working hard for the past 3 months to control her type 2 diabetes with diet
and exercise. To determine the effectiveness of the client's efforts, the nurse should check:
You Selected:

fasting blood glucose level.

Correct response:

glycosylated hemoglobin level.

Explanation:

Question 31 See full question

A client is scheduled for a laminectomy to repair a herniated intervertebral disk. When developing the postoperative
care plan, the nurse should include which action?

You Selected:

Keeping a pillow under the client's knees at all times

Correct response:

Turning the client from side to side, using the logroll technique

Explanation:

Question 32 See full question

Which nursing intervention is essential in caring for a client with compartment syndrome?

You Selected:

Keeping the affected extremity below the level of the heart

Correct response:

Removing all external sources of pressure, such as clothing and jewelry

Explanation:

Question 33 See full question

A client was undergoing conservative treatment for a herniated nucleus pulposus, at L5 S1, which was diagnosed
by magnetic resonance imaging. Because of increasing neurological symptoms, the client undergoes lumbar
laminectomy. The nurse should take which step during the immediate postoperative period?

You Selected:

Have the client sit up in a chair as much as possible.

Correct response:

Logroll the client from side to side.

Explanation:

Question 34 See full question

A client with chronic renal failure is admitted with a heart rate of 122 beats/minute, a respiratory rate of 32
breaths/minute, a blood pressure of 190/110 mm Hg, jugular vein distention, and bibasilar crackles. Which nursing
diagnosis takes highest priority for this client?

You Selected:

Excess fluid volume

Correct response:

Excess fluid volume


Explanation:

Question 35 See full question

A client with known coronary artery disease reports intermittent chest pain, usually on exertion. The physician
diagnoses angina pectoris and orders sublingual nitroglycerin to treat acute angina episodes. When teaching the
client about nitroglycerin administration, which instruction should the nurse provide?

You Selected:

"Replace leftover sublingual nitroglycerin tablets every 9 months to make sure your pills are fresh."

Correct response:

"Be sure to take safety precautions because nitroglycerin may cause dizziness when you stand up."

Explanation:

Question 36 See full question

The nurse is working in an ambulatory obstetrics setting. What are emphasized client safety procedures for this
setting? Select all that apply.

You Selected:

conduct preprocedure verification asking for client name and procedure to be performed

Correct response:

handwashing or antiseptic use when entering and leaving a room


use of two client identifiers when initiating contact with a client
conduct preprocedure verification asking for client name and procedure to be performed

Explanation:

Question 37 See full question

The primary care provider prescribes cefepime 250 mg every 6 hours for a child weighing 25 kg who had infected
burns. The normal dosage for this antibiotic and condition is 20 to 50 mg/kg per 24 hours. Which actions would
be most appropriate?

You Selected:

Give the dose recommended by the pharmacy reference material.

Correct response:

Carry out the prescription because the prescribed dose is acceptable.

Explanation:

Question 38 See full question

A client who has had AIDS for years is being treated for a serious episode of pneumonia. A psychiatric nurse consult
was arranged after the client stated that he was tired of being in and out of the hospital. I am not coming in here
anymore. I have other options. The nurse would evaluate the psychiatric nurse consult as helpful if the client
makes which statements?

You Selected:

"Nobody wants me to commit suicide."

Correct response:

"I realize that I really do have more time to enjoy my friends and family."
Explanation:

Question 39 See full question

An elderly client with type 2 diabetes had hyperglycemic hyperosmolar syndrome (HHS). The nurse should monitor
the infusion for too rapid correction of the blood glucose in order to prevent:

You Selected:

a major vascular accident.

Correct response:

cerebral edema.

Explanation:

Question 40 See full question

The most reliable early indicator of infection in a client who is neutropenic is:

You Selected:

dyspnea.

Correct response:

fever.

Explanation:

Question 41 See full question

A priority for nursing care for an older adult who has pruritus, is continuously scratching the affected areas, and
demonstrates agitation and anxiety regarding the itching is:

You Selected:

preventing infection.

Correct response:

preventing infection.

Explanation:

Question 42 See full question

The nurse provides teaching on postoperative wound care to a client being discharged from a surgical unit. Which of
the following statements documented by the nurse indicates that the client understood the teaching?

You Selected:

Client told to come back to the hospital if wound is warm, red, and draining.

Correct response:

Client verbalized to the nurse the steps to follow if wound becomes red and warm.

Explanation:

Question 43 See full question

The student nurse is caring for a client with a suspected respiratory infection. Which of the following statements by
the nursing student indicates to the instructor that the student will facilitate the best time to collect this specimen?

You Selected:

"I will collect the specimen early in the evening when secretions settle in the lungs."
Correct response:

"I will instruct the client to give the specimen in the morning, as soon as the client awakens."

Explanation:

Question 44 See full question

A client with a subdural hematoma needs a feeding tube inserted due to inadequate swallowing ability. How would
the nurse best explain this to the family?

You Selected:

Nutrients are needed; however, eating and drinking without control of the swallowing reflex can result in
aspirational pneumonia.

Correct response:

Nutrients are needed; however, eating and drinking without control of the swallowing reflex can result in
aspirational pneumonia.

Explanation:

Question 45 See full question

During a panic attack, a client runs to the nurse and reports experiencing difficulty breathing, chest pain, and
palpitations. The client is pale, with his mouth wide open and his eyebrows raised. What should the nurse do first?

You Selected:

Assist the client to breathe deeply into a paper bag.

Correct response:

Assist the client to breathe deeply into a paper bag.

Explanation:

Question 46 See full question

A client with heart failure has assessment findings of jugular vein distension (JVD) when lying flat in bed. Which of
the following is the best nursing intervention?

You Selected:

Notify the healthcare provider

Correct response:

Elevate the head of the bed to 30 to 45 degrees and reassess JVD

Explanation:

Question 47 See full question

The charge nurse is reviewing the laboratory results of a child admitted with nephrotic syndrome with a nurse new
to the pediatric unit. The nurse is aware that teaching is required when the new nurse states that an expected
finding in nephrotic syndrome is:

You Selected:

proteinuria.

Correct response:

hyperalbuminemia.
Explanation:

Question 48 See full question

The nurse is caring for a client who sustained a head injury during a football game. The nurse is completing the
following examination. Which documentation by the nurse provides normal results of this examination?

You Selected:

The clients pupils are equal and reactive to light and accommodation.

Correct response:

The clients pupils are equal and reactive to light and accommodation.

Explanation:

Question 49 See full question

A client in the first stage of labor is being monitored using an external fetal monitor. After the nurse reviews the
monitoring strip from the clients chart (shown above), into which of the following positions would the nurse assist
the client?

You Selected:

Left lateral

Correct response:

Left lateral

Explanation:

Question 50 See full question

The nurse working in a long term care facility notes changes in the client of confusion and change in vital signs.
Upon consulting with the health care provider, lab work and a urine culture via an indwelling catheter is ordered.
Identify the location the nurse would access the urine from the catheter.

You Selected:

Your selection and the correct area, market by the green box.
Explanation:

Question 6 See full question

A 24-hour-old, full-term neonate is showing signs of possible sepsis. The nurse is assisting the health care provider
(HCP) with a lumbar puncture on this neonate. What should the nurse do to assist in this procedure? Select all that
apply.

You Selected:

Hold the neonate steady in the correct position.


Obtain a serum glucose level.

Correct response:

Hold the neonate steady in the correct position.


Ensure a patent airway.
Maintain a sterile field.

Explanation:

Question 7 See full question

A nurse assesses a client with psychotic symptoms and determines that the client likely poses a safety threat and
needs vest restraints. The client is adamantly opposed to this. What would be the best nursing action?

You Selected:

Contact the physician and obtain necessary orders.

Correct response:

Contact the physician and obtain necessary orders.

Explanation:

Question 8 See full question

Which prescription is entered correctly on the medical record?

You Selected:

fentanyl 50 mcg given IV every 2 hours as needed for pain greater than 6/10

Correct response:

fentanyl 50 mcg given IV every 2 hours as needed for pain greater than 6/10

Explanation:

Question 9 See full question

A nurse inadvertently transcribes a clients medication order that was written as Ampicillin 250 mg four times a
day" as Ampicillin 2500 mg four times a day. The nurse gives two doses as transcribed to the client. Another nurse
gives one dose before the pharmacist questions the reorder of the medication. What should the two nurses do in
this situation?

You Selected:

Adjust the medication administration record to reflect the correct dose only.

Correct response:

Both nurses must acknowledge making the medication error.


Explanation:

Question 10 See full question

The nurse is applying a hand mitt restraint for a client with pruritus (see figure). The nurse should first:

You Selected:

place the mitt on top of the hand.

Correct response:

verify the prescription to use the restraint.

Explanation: Question 1 See full question

When moving a client in bed, the nurse can ensure proper body mechanics by:

You Selected:

standing with her feet apart.

Correct response:

standing with her feet apart.

Explanation:

Question 2 See full question

A nurse gives a client the wrong medication. After assessing the client, the nurse completes an incident report.
Which statement describes what will happen next?

You Selected:

The facility will report the incident to the state board of nursing for disciplinary action.

Correct response:

The incident report will provide a basis for promoting quality care and risk management.

Explanation:

Question 3 See full question

A physician has ordered a heating pad for an elderly client's lower back pain. Which item would be most important
for a nurse to assess before applying the heating pad?

You Selected:

Client's level of consciousness

Correct response:

Client's level of consciousness

Explanation:

Question 4 See full question

A nurse assists in writing a community plan for responding to a bioterrorism threat or attack. When reviewing the
plan, the director of emergency operations should have the nurse correct which intervention?

You Selected:

Clients should be instructed to wash thoroughly with soap and water.

Correct response:

Clients exposed to anthrax should immediately remove contaminated clothing and place it in the hamper.
Explanation:

Question 5 See full question

When checking a client's medication profile, a nurse notes that the client is receiving a drug contraindicated for
clients with glaucoma. The nurse knows that this client, who has a history of glaucoma, has been taking the
medication for the past 3 days. What should the nurse do first?

You Selected:

Continue to give the medication because the client has been taking it for 3 days.

Correct response:

Hold the medication and report the information to the physician to ensure client safety.

Explanation:

Question 6 See full question

While performing rounds, a nurse finds that a client is receiving the wrong I.V. solution. The nurse's initial response
should be to:

You Selected:

slow the I.V. flow rate and hang the appropriate solution.

Correct response:

slow the I.V. flow rate and hang the appropriate solution.

Explanation:

Question 7 See full question

A 3-month-old infant with meningococcal meningitis has just been admitted to the pediatric unit. Which nursing
intervention has the highest priority?

You Selected:

orienting the parents to the pediatric unit

Correct response:

instituting droplet precautions

Explanation:

Question 8 See full question

Which of the following objects poses the most serious safety threat to a 2-year-old child in the hospital?

You Selected:

Stuffed teddy bear in the crib

Correct response:

Side rails in the halfway position

Explanation:

Question 9 See full question

What should a nurse do to ensure a safe hospital environment for a toddler?

You Selected:

Pad the crib rails.


Correct response:

Move the equipment out of reach.

Explanation:

Question 10 See full question

A child, age 3, who tests positive for the human immunodeficiency virus (HIV) is placed in foster care. The foster
parents ask the nurse how to prevent HIV transmission to other family members. How should the nurse respond?

You Selected:

"Wear gloves when you're likely to come into contact with the child's blood or body fluids."

Correct response:

"Wear gloves when you're likely to come into contact with the child's blood or body fluids."

Explanation:

Question 11 See full question

A nurse is teaching parents how to reduce the spread of impetigo. The nurse should encourage parents to:

You Selected:

have their children immunized against impetigo.

Correct response:

teach children the importance of proper hand washing.

Explanation:

Question 12 See full question

As a client is being released from restraints, he says, "I'll never get that angry and lose it again. Those restraints
were the worst things that ever happened to me." Which response by the nurse is most appropriate?

You Selected:

"Do you really mean what you just said?"

Correct response:

"I'd like to talk with you about your experience."

Explanation:

Question 13 See full question

A nurse is assigned to care for a recently admitted client who has attempted suicide. What should the nurse do?

You Selected:

Remind all staff members to check on the client frequently.

Correct response:

Search the client's belongings and his room carefully for items that could be used to attempt suicide.

Explanation:

Question 14 See full question

When planning care for a client who has ingested phencyclidine (PCP), the nurse's highest priority should be
meeting the:
You Selected:

client's safety needs.

Correct response:

client's safety needs.

Explanation:

Question 15 See full question

A client newly admitted to a psychiatric inpatient setting demands a soda from a staff member who tells him to wait
until lunch arrives in 20 minutes. The client becomes angry, pushes over a sofa, throws an end table, and dumps a
potted plant. Which goal should a nurse consider to be of primary importance?

You Selected:

Talking with the client's family about his angry feelings

Correct response:

Demonstrating control over aggressive behavior

Explanation:

Question 16 See full question

Which action is the best precaution against transmission of infection?

You Selected:

Strict isolation for a neonate whose mother has cytomegalovirus (CMV) infection

Correct response:

Eye prophylaxis with antibiotics for a neonate whose mother has gonorrhea infection

Explanation:

Question 17 See full question

A nurse preparing to discharge a child with leukemia observes a family member who has a cold sharing a meal with
the child. How should the nurse approach the situation?

You Selected:

Post isolation signs on the child's door and carefully assess the health status of all visitors.

Correct response:

Offer a face mask to the person with the cold and use this as an opportunity for further teaching.

Explanation:

Question 18 See full question

A nurse has just been trained in how to use and care for a new blood glucose monitor. Which nursing intervention
demonstrates proper use of a blood glucose monitor?

You Selected:

Starting the timer on the machine while gathering supplies

Correct response:

Calibrating the machine after installing a new battery


Explanation:

Question 19 See full question

While reviewing the day's charts, a nurse who's been under a great deal of personal stress realizes that she forgot
to administer insulin to client with diabetes mellitus. She's made numerous errors in the past few weeks and is now
afraid her job is in jeopardy. What is her best course of action?

You Selected:

Administer the medication immediately and chart it as given on time.

Correct response:

Report the error, complete the proper paperwork, and meet with the unit manager.

Explanation:

Question 20 See full question

The nurse is receiving over the telephone a laboratory results report of a neonate's blood glucose level. The nurse
should:

You Selected:

write down the results, read back the results to the caller from the laboratory, and receive confirmation from
the caller that the nurse understands the results.

Correct response:

write down the results, read back the results to the caller from the laboratory, and receive confirmation from
the caller that the nurse understands the results.

Explanation:

Question 21 See full question

The nurse is caring for a neonate diagnosed with early onset sepsis and is being treated with intravenous
antibiotics. Which instructions will the nurse include in the parents teaching plan?

You Selected:

Wear protective gear near the isolation incubator.

Correct response:

Wash hands thoroughly before touching the neonate.

Explanation:

Question 22 See full question

The nurse is administering prednisone to a preschool child with nephrosis. What should the nurse do to ensure that
the nurse has identified the child correctly? Select all that apply.

You Selected:

Ask the client to state the first name.


Compare the room number on the bed with the number on the client's identification band.

Correct response:

Check the child's identification band against the medical record number.
Verify the date of birth from the medical record with the date of birth on the client's identification band.
Explanation:

Question 23 See full question

When developing a teaching plan for parents of toddlers about poisonous substances, the nurse should emphasize
which safety points? Select all that apply.

You Selected:

Following any poisoning, the parents should call the Poison Control Center for instructions for appropriate
treatment.

Correct response:

Toddlers should be adequately supervised at all times.


All poisonous substances should be kept out of the reach of children and stored in a locked cabinet if
necessary.
The difference between pediatric and adult dosages of medicines is significant, and adult dosages given to
children can have serious, harmful effects.
Following any poisoning, the parents should call the Poison Control Center for instructions for appropriate
treatment.

Explanation:

Question 24 See full question

When teaching parent workshops about measures to prevent lead poisoning in children, which preventive measure
should the nurse include as the most effective?

You Selected:

educating the public on the importance of good nutrition

Correct response:

educating the public on common sources of lead

Explanation:

Question 25 See full question

A 17-year-old female with severe nodular acne is considering treatment with isotretinoin. Prior to beginning the
medication, the nurse explains that the client will be required to:

You Selected:

enroll in a risk management plan.

Correct response:

enroll in a risk management plan.

Explanation:

Question 26 See full question

The nurse is preparing a community education program about preventing hepatitis B infection. Which information
should be incorporated into the teaching plan?

You Selected:

The use of a condom is advised for sexual intercourse.

Correct response:

The use of a condom is advised for sexual intercourse.


Explanation:

Question 27 See full question

The nurse should teach clients that the most common route of transmitting tubercle bacilli from person to person is
through contaminated:

You Selected:

dust particles.

Correct response:

droplet nuclei.

Explanation:

Question 28 See full question

The nurse should use which type of precautions for a client being admitted to the hospital with suspected
tuberculosis?

You Selected:

contact precautions

Correct response:

airborne precautions

Explanation:

Question 29 See full question

A client is to have a transfusion of packed red blood cells from a designated donor. The client asks if any diseases
can be transmitted by this donor. The nurse should inform the client that which diseases can be transmitted by a
designated donor? Select all that apply.

You Selected:

cytomegalovirus (CMV)

Correct response:

Epstein-Barr virus
human immunodeficiency virus (HIV)
cytomegalovirus (CMV)

Explanation:

Question 30 See full question

The nurse is planning care for a client who had surgery for abdominal aortic aneurysm repair 2 days ago. The pain
medication and the use of relaxation and imagery techniques are not relieving the clients pain, and the client
refuses to get out of bed to ambulate as prescribed. The nurse contacts the health care provider (HCP), explains the
situation, and provides information about drug dose, frequency of administration, the clients vital signs, and the
clients score on the pain scale. The nurse requests a prescription for a different, or stronger, pain medication. The
HCP tells the nurse that the current prescription for pain medication is sufficient for this client and that the client
will feel better in several days. The nurse should next:

You Selected:

explain to the HCP that the current pain medication and other strategies are not helping the client and it is
making it difficult for the client to ambulate as prescribed.

Correct response:

explain to the HCP that the current pain medication and other strategies are not helping the client and it is
making it difficult for the client to ambulate as prescribed.
Explanation:

Question 31 See full question

A client with a cerebellar brain tumor is admitted to an acute care facility. The nurse formulates a nursing diagnosis
of Risk for injury. Which "related-to" phrase should the nurse add to complete the nursing diagnosis statement?

You Selected:

Related to impaired balance

Correct response:

Related to impaired balance

Explanation:

Question 32 See full question

A client with a suspected brain tumor is scheduled for a computed tomography (CT) scan. What should the nurse do
when preparing the client for this test?

You Selected:

Place a cap over the client's head.

Correct response:

Determine whether the client is allergic to iodine, contrast dyes, or shellfish.

Explanation:

Question 33 See full question

A 29-year-old multigravida at 37 weeks' gestation is being treated for severe preeclampsia and has magnesium
sulfate infusing at 3 g/h. To maintain safety for this client, the priority intervention is to:

You Selected:

maintain continuous fetal monitoring.

Correct response:

assess reflexes, clonus, visual disturbances, and headache.

Explanation:

Question 34 See full question

A 6-month-old on the pediatric floor has a respiratory rate of 68, mild intercostal retractions, and an oxygen
saturation of 89%. The infant has not been feeding well for the last 24 hours and is restless. Using the SBAR
(Situation-Background-Assessment-Recommendation) technique for communication, the nurse calls the health care
provider (HCP) with the recommendation for:

You Selected:

starting oxygen.

Correct response:

starting oxygen.

Explanation:

Question 35 See full question

The nurse is caring for a client who is confused about time and place. The client has intravenous fluid infusing. The
nurse attempts to reorient the client, but the client remains unable to demonstrate appropriate use of the call light.
In order to maintain client safety, what should the nurse do first?
You Selected:

Contact the health care provider (HCP), and request a prescription for soft wrist restraints.

Correct response:

Increase the frequency of client observation.

Explanation:

Question 36 See full question

A client is admitted to the Emergency Department with a full thickness burn to the right arm. Upon assessment, the
arm is edematous, fingers are mottled, and radial pulse is now absent. The client states that the pain is 8 on a scale
of 1 to 10. The nurse should:

You Selected:

call the health care provider (HCP) to report the loss of the radial pulse.

Correct response:

call the health care provider (HCP) to report the loss of the radial pulse.

Explanation:

Question 37 See full question

The nurse in the emergency department reports there is a possibility of having had direct contact with blood of a
client who is suspected of having HIV/AIDS. The nurse requests that the client have a blood test. Consent for human
immunodeficiency virus (HIV) testing can only be completed when which circumstances are present? Select all that
apply.

You Selected:

Testing is done on blood collected anonymously in an epidemiologic survey.

Correct response:

An emergency medical provider has been exposed to the client's blood or body fluids.
Testing is prescribed by a health care provider (HCP) under emergency circumstances.
Testing is ordered by a court, based on evidence that the client poses a threat to others.
Testing is done on blood collected anonymously in an epidemiologic survey.

Explanation:

Question 38 See full question

A 26-year-old is being treated for delirium due to acute alcohol intoxication. The client is restless, does not want to
stay seated, and has a staggering gait. What should the nurse do first?

You Selected:

Place the client in a chair with a waist restraint.

Correct response:

Provide one-to-one supervision of the client until detoxification treatment can begin.

Explanation:

Question 39 See full question

A nurse on a night shift entered an elderly clients room during a scheduled check and discovered the client on the
floor beside her bed after falling when trying to ambulate to the washroom. After assessing and assisting the client
back to bed, the nurse has completed an incident report. What is the primary purpose of this particular type of
documentation?
You Selected:

Protecting the nurse and the hospital from litigation.

Correct response:

Identifying risks and ensuring future safety for clients.

Explanation:

Question 40 See full question

A nurse assesses a client with psychotic symptoms and determines that the client likely poses a safety threat and
needs vest restraints. The client is adamantly opposed to this. What would be the best nursing action?

You Selected:

Apply wrist restraints instead of vest restraints.

Correct response:

Contact the physician and obtain necessary orders.

Explanation:

Question 41 See full question

The nurse manager of a surgical unit observes a nurse providing colostomy care to a client without using any
personal protective equipment (PPE). What is the most appropriate response by the nurse manager in relation to
the use of PPE?

You Selected:

In the future, have the physician write an order for PPE for clients with colostomies.

Correct response:

PPE should be used when you risk exposure to blood or bodily fluids.

Explanation:

Question 42 See full question

The nurse is caring for a 5-year-old child who has a history of multiple admissions for fractures and cuts. The
mother explains that the child fractured the femur by falling, but does not give any further details. The child
indicates that the mothers boyfriend was present when the injury occurred, and the childs recollection of the event
conflicts with the mothers explanation. What is the nurses immediate responsibility?

You Selected:

Call the police department and report abuse.

Correct response:

Keep the child safe and assess for abuse.

Explanation:

Question 43 See full question

The nurse is caring for a toddler who is visually impaired. What is the most important action for the nurse to take to
ensure the safety of the child?

You Selected:

Use visual aids to facilitate communication.

Correct response:

Maintain a tidy environment around the child.


Explanation:

Question 44 See full question

A nurse notices the smell of marijuana on a nursing colleague upon return from lunch break. The colleague is
having difficulty drawing up a dose of insulin, appears uncoordinated, and is unaware that the needle has been
contaminated. What is the best action for the nurse to take?

You Selected:

Reassign the responsibilities, and inform the colleague that the unit manager will be notified if it occurs again.

Correct response:

Stop the colleague from drawing up the insulin. Notify the supervisor about the incident, and document the
observations.

Explanation:

Question 45 See full question

A nurse recognizes that a client with tuberculosis needs further teaching when the client states:

You Selected:

"I'll need to have scheduled laboratory tests while I'm on the medication."

Correct response:

"It will be necessary for the people I work with to take medication."

Explanation:

Question 46 See full question

Which of the following is the priority action the nurse should take when finding medications at a clients bedside?

You Selected:

Leave the medications, as the client will take them after breakfast.

Correct response:

Remove the medications from the room and discard them into an appropriate disposal bin.

Explanation:

Question 47 See full question

The nurse is inserting a nasogastric tube in an infant to administer feedings. In the accompanying figure, indicate
the location for the correct placement of the distal end of the tube.
You Selected:

Your selection and the correct area, market by the green box.

Explanation:

Question 48 See full question

The nurse is applying a hand mitt restraint for a client with pruritus (see figure). The nurse should first:

You Selected:

secure the mitt with ties around the wrist tied to the bed frame.

Correct response:

verify the prescription to use the restraint.

Explanation:

Question 49 See full question

A client is diagnosed with esophageal cancer and presents with difficulty swallowing. Which intervention should
receive the highest priority?

You Selected:

Helping the client cope with impending body image changes

Correct response:

Establishing aspiration precautions

Explanation:

Question 50 See full question

Which nursing action best addresses the outcome: The client will be free from falls?

You Selected:

Install a monitoring system to help the client in an emergency situation.

Correct response:

Encourage use of grab bars and railings in the bathroom and halls
Explanation:

Improve your mastery


Question 1 See full question

A nurse is assigned to a client with a cardiac disorder. The nurse should question an order to monitor the client's
body temperature by which route?

You Selected:

Rectal

Correct response:

Rectal

Explanation:

Question 2 See full question

Which of the following objects poses the most serious safety threat to a 2-year-old child in the hospital?

You Selected:

Crayons and paper

Correct response:

Side rails in the halfway position

Explanation:

Question 3 See full question

A nurse is teaching parents about accident prevention for a toddler. Which guideline is most appropriate?

You Selected:

Teach rules of the road for bicycle safety.

Correct response:

Make sure all medications are kept in containers with childproof safety caps.

Explanation:

Question 4 See full question

A nurse discussing injury prevention with a group of workers at a day-care center is focusing on toddlers. When
discussing this age-group, the nurse should stress that:

You Selected:

toddlers can distinguish right from wrong.

Correct response:

accidents are the leading cause of death among toddlers.

Explanation:

Question 5 See full question

Which use of restraints in a school-age child should the nurse question?

You Selected:

To substitute for observation


Correct response:

To substitute for observation

Explanation:

Question 6 See full question

Mental health laws in each state specify when restraints may be used and which type of restraints may be used.
Most laws stipulate that restraints may be used:

You Selected:

only with the client's consent.

Correct response:

if a client poses a present danger to himself or others.

Explanation:

Question 7 See full question

A client arrives on the psychiatric unit exhibiting extreme excitement, disorientation, incoherent speech, agitation,
frantic and aimless physical activity, and grandiose delusion. Which nursing diagnosis takes highest priority for this
client at this time?

You Selected:

Risk for injury

Correct response:

Risk for injury

Explanation:

Question 8 See full question

A nurse notices that a client admitted for treatment of major depression is pacing, agitated, and becoming verbally
aggressive toward other clients. What is the immediate care priority?

You Selected:

Removing the other clients from the area until this client settles down

Correct response:

Ensuring the safety of this client and other clients on the unit

Explanation:

Question 9 See full question

In the emergency department, a client with facial lacerations states that her husband beat her with a shoe. After
her lacerations are repaired, the client waits to be seen by the crisis intake nurse, who will evaluate the continued
threat of violence her husband represents. Suddenly the client's husband arrives, shouting that he wants to "finish
the job." What is the first priority of the nurse who witnesses this scene?

You Selected:

Telling the client's husband that he must leave at once

Correct response:

Calling a security guard and another staff member for assistance


Explanation:

Question 10 See full question

When planning care for a client who has ingested phencyclidine (PCP), the nurse's highest priority should be
meeting the:

You Selected:

client's psychosocial needs.

Correct response:

client's safety needs.

Explanation:

Question 11 See full question

An alarm signals, indicating that a neonate's security identification band requires attention. The nurse responds
immediately and finds that the parents removed the identification bands from the neonate. Which action should the
nurse take next?

You Selected:

Reprimand the parents for allowing the identification bands to come off.

Correct response:

Compare the information on the neonate's identification bands with that of the mother's, then reattach the
identification bands to one of the neonate's extremities.

Explanation:

Question 12 See full question

The nurse is caring for a neonate diagnosed with early onset sepsis and is being treated with intravenous
antibiotics. Which instructions will the nurse include in the parents teaching plan?

You Selected:

Wear protective gear near the isolation incubator.

Correct response:

Wash hands thoroughly before touching the neonate.

Explanation:

Question 13 See full question

A client diagnosed with schizophrenia for the last 2 years tells the nurse who has brought the morning medications,
"That is not my pill! My pill is blue, not green." What should the nurse tell the client?

You Selected:

"Do not worry; your medication is generic, and sometimes the manufacturers change the color of the pills
without letting us know."

Correct response:

"I will go back and check the drawer as well as telephone the pharmacy to check about any possible changes
in the medication color."
Explanation:

Question 14 See full question

The nurse is meeting weekly with an adolescent recently diagnosed with depression to monitor progress with
therapy and antidepressant medication. The nurse should be most concerned when the client reports what
information?

You Selected:

She received a low score on her last history test.

Correct response:

An acquaintance hanged herself two days ago.

Explanation:

Question 15 See full question

When developing the teaching plan for the mother of a 2-year-old child diagnosed with scabies, what information
should the nurse expect to include?

You Selected:

The entire family should be treated.

Correct response:

The entire family should be treated.

Explanation:

Question 16 See full question

A client with cervical cancer is undergoing internal radium implant therapy. A lead-lined container and a pair of long
forceps are kept in the client's hospital room for:

You Selected:

handling of the dislodged radiation source.

Correct response:

handling of the dislodged radiation source.

Explanation:

Question 17 See full question

A nurse is providing wound care to a client 1 day following an appendectomy. A drain was inserted into the
incisional site during surgery. When providing wound care, the nurse should:

You Selected:

gently irrigate the drain to remove exudate.

Correct response:

clean the area around the drain moving away from the drain.

Explanation:

Question 18 See full question

Which family member exposed to tuberculosis would be at highest risk for contracting the disease?

You Selected:

45-year-old mother
Correct response:

76-year-old grandmother

Explanation:

Question 19 See full question

The nurse's best explanation for why the severely neutropenic client is placed in reverse isolation is that reverse
isolation helps prevent the spread of organisms:

You Selected:

to the client from sources outside the client's environment.

Correct response:

to the client from sources outside the client's environment.

Explanation:

Question 20 See full question

In caring for the client with hepatitis B, which situation would expose the nurse to the virus?

You Selected:

disposing of syringes and needles without recapping

Correct response:

a blood splash into the nurses eyes

Explanation:

Question 21 See full question

A client with a cerebellar brain tumor is admitted to an acute care facility. The nurse formulates a nursing diagnosis
of Risk for injury. Which "related-to" phrase should the nurse add to complete the nursing diagnosis statement?

You Selected:

Related to psychomotor seizures

Correct response:

Related to impaired balance

Explanation:

Question 22 See full question

A client with suspected severe acute respiratory syndrome (SARS) comes to the emergency department. Which
physician order should the nurse implement first?

You Selected:

Obtain a nasopharyngeal specimen for reverse-transcription polymerase chain reaction testing.

Correct response:

Institute isolation precautions.

Explanation:

Question 23 See full question

After administering an I.M. injection, a nurse should:


You Selected:

discard the uncapped needle and syringe in a puncture-proof container.

Correct response:

discard the uncapped needle and syringe in a puncture-proof container.

Explanation:

Question 24 See full question

A nursing instructor is instructing group of new nursing students. The instructor reviews that surgical asepsis will be
used for which of the following procedures?

You Selected:

Nasogastric tube irrigation

Correct response:

I.V. catheter insertion

Explanation:

Question 25 See full question

A 29-year-old multigravida at 37 weeks' gestation is being treated for severe preeclampsia and has magnesium
sulfate infusing at 3 g/h. To maintain safety for this client, the priority intervention is to:

You Selected:

maintain continuous fetal monitoring.

Correct response:

assess reflexes, clonus, visual disturbances, and headache.

Explanation:

Question 26 See full question

A multigravid client is admitted at 4-cm dilation and is requesting pain medication. The nurse gives the client
nalbuphine 15 mg. Within five minutes, the client tells the nurse she feels like she needs to have a bowel
movement. The nurse should first:

You Selected:

complete a vaginal examination to determine dilation, effacement, and station.

Correct response:

complete a vaginal examination to determine dilation, effacement, and station.

Explanation:

Question 27 See full question

A 6-month-old on the pediatric floor has a respiratory rate of 68, mild intercostal retractions, and an oxygen
saturation of 89%. The infant has not been feeding well for the last 24 hours and is restless. Using the SBAR
(Situation-Background-Assessment-Recommendation) technique for communication, the nurse calls the health care
provider (HCP) with the recommendation for:

You Selected:

starting oxygen.

Correct response:

starting oxygen.
Explanation:

Question 28 See full question

One evening the client takes the nurse aside and whispers, Do not tell anybody, but I am going to call in a bomb
threat to this hospital tonight. Which action is the priority?

You Selected:

warning the client that his telephone privileges will be taken away if he abuses them

Correct response:

explaining to the client that this information will have to be shared immediately with the staff and the health
care provider (HCP)

Explanation:

Question 29 See full question

A young client is being admitted to the psychiatric unit after her obstetricians staff suspected she was experiencing
a postpartum psychosis. Her husband said she was doing fine for 2 weeks after the birth of the baby, except for
pain from the C-section and trouble sleeping. These symptoms subsided over the next 4 weeks. Three days ago,
however, the client started having anxiety, irritability, vomiting, diarrhea, and delirium, resulting in her inability to
care for the baby. The husband says, I saw that my bottles of alprazolam and oxycodone were empty even though I
have not been taking them. What should the nurse do in order of priority from first to last? All options must be
used.

You Selected:

Immediately place the client on withdrawal precautions.


Call the health care provider (HCP) for prescriptions for appropriate treatment for opiate and benzodiazepine
withdrawal.
Confirm with the client that she has in fact been using her husband's medications.
Assess the client for prior and current use of any other substances.

Correct response:

Confirm with the client that she has in fact been using her husband's medications.
Assess the client for prior and current use of any other substances.
Immediately place the client on withdrawal precautions.
Call the health care provider (HCP) for prescriptions for appropriate treatment for opiate and benzodiazepine
withdrawal.

Explanation:

Question 30 See full question

While making rounds, the nurse enters a clients room and finds the client on the floor between the bed and the
bathroom. The nurse should first:

You Selected:

assist the client back to bed.

Correct response:

assess the clients current condition and vital signs.

Explanation:

Question 31 See full question

An alert and oriented older adult female with metastatic lung cancer is admitted to the medical-surgical unit for
treatment of heart failure. She was given 80 mg of furosemide in the emergency department. Although the client is
ambulatory, the unlicensed assistive personnel (UAP) are concerned about urinary incontinence because the client
is frail and in a strange environment. The nurse should instruct the UAP to assist with implementing the nursing
plan of care by:

You Selected:

padding the bed with extra absorbent linens

Correct response:

placing a commode at the bedside and instructing the client in its use

Explanation:

Question 32 See full question

A client is to have a below-the-knee amputation. Prior to surgery, the circulating nurse in the operating room
should:

You Selected:

initiate a time-out.

Correct response:

initiate a time-out.

Explanation:

Question 33 See full question

While making rounds, a nurse observes that a clients primary bag of intravenous (IV) solution is light yellow. The
label on the IV bag says the solution is D5W. What should the nurse do first?

You Selected:

Hang a new bag of D5W, and complete an incident report.

Correct response:

Hang a new bag of D5W, and complete an incident report.

Explanation:

Question 34 See full question

The nurse in the emergency department reports there is a possibility of having had direct contact with blood of a
client who is suspected of having HIV/AIDS. The nurse requests that the client have a blood test. Consent for human
immunodeficiency virus (HIV) testing can only be completed when which circumstances are present? Select all that
apply.

You Selected:

A health care provider (HCP) who is taking care of a client suspected of having HIV/AIDS requests a blood test.

Correct response:

An emergency medical provider has been exposed to the client's blood or body fluids.
Testing is prescribed by a health care provider (HCP) under emergency circumstances.
Testing is ordered by a court, based on evidence that the client poses a threat to others.
Testing is done on blood collected anonymously in an epidemiologic survey.

Explanation:

Question 35 See full question

During rounds, a nurse finds that a client with hemiplegia has fallen from the bed because the nursing assistant
failed to raise the side rails after giving a back massage. The nurse assists the client to the bed and assesses for
injury. As per agency policies, the nurse fills out an incident report. Which of the following activities should the
nurse perform after finishing the incident report?
You Selected:

Mention the name of the nursing assistant in the client records.

Correct response:

Include the time and date of the incident.

Explanation:

Question 36 See full question

A nurse assesses a client with psychotic symptoms and determines that the client likely poses a safety threat and
needs vest restraints. The client is adamantly opposed to this. What would be the best nursing action?

You Selected:

Contact the physician and obtain necessary orders.

Correct response:

Contact the physician and obtain necessary orders.

Explanation:

Question 37 See full question

Which prescription is entered correctly on the medical record?

You Selected:

fentanyl 50 mcg given IV every 2 hours as needed for pain greater than 6/10

Correct response:

fentanyl 50 mcg given IV every 2 hours as needed for pain greater than 6/10

Explanation:

Question 38 See full question

A 13-year-old is having surgery to repair a fractured left femur. As a part of the preoperative safety checklist, what
should the nurse do?

You Selected:

Ask the teen to point to the surgery site.

Correct response:

Verify that the site, side, and level are marked.

Explanation:

Question 39 See full question

When the client is involuntarily committed to a hospital because he is assessed as being dangerous to himself or
others, which client rights are lost?

You Selected:

the right to send and receive uncensored mail

Correct response:

the right to leave the hospital against medical advice


Explanation:

Question 40 See full question

A nurse inadvertently transcribes a clients medication order that was written as Ampicillin 250 mg four times a
day" as Ampicillin 2500 mg four times a day. The nurse gives two doses as transcribed to the client. Another nurse
gives one dose before the pharmacist questions the reorder of the medication. What should the two nurses do in
this situation?

You Selected:

Both nurses must acknowledge making the medication error.

Correct response:

Both nurses must acknowledge making the medication error.

Explanation:

Question 41 See full question

A nurse is caring for a confused client and develops a plan of care based on a least restraint policy. Which of the
following interventions would be most appropriate for the nurse to implement based on this policy?

You Selected:

Vest-type restraint

Correct response:

Alarm-activating wrist bracelet

Explanation:

Question 42 See full question

While hospitalized, a child develops a Clostridium difficile infection. The nurse can anticipate adding which of the
following types of precautions for this client?

You Selected:

Contact precautions

Correct response:

Contact precautions

Explanation:

Question 43 See full question

The nurse is inserting a nasogastric tube in an infant to administer feedings. In the accompanying figure, indicate
the location for the correct placement of the distal end of the tube.
You Selected:

Your selection and the correct area, market by the green box.

Explanation:

Question 44 See full question

The nurse is applying a hand mitt restraint for a client with pruritus (see figure). The nurse should first:

You Selected:

verify the prescription to use the restraint.

Correct response:

verify the prescription to use the restraint.

Explanation:

Question 45 See full question

A client is diagnosed with esophageal cancer and presents with difficulty swallowing. Which intervention should
receive the highest priority?

You Selected:

Helping the client cope with impending body image changes

Correct response:

Establishing aspiration precautions

Explanation:

Question 46 See full question

The nurse is developing a care plan for a client who has had radiation therapy for Hodgkins lymphoma. What is the
primary goal of care for this client?

You Selected:

Avoid depression.

Correct response:

Prevent infection.
Explanation:

Question 47 See full question

The nurse is caring for a client recently diagnosed with hepatitis C. In reviewing the clients history, what
information will be most helpful as the nurse develops a teaching plan? The client:

You Selected:

has a known history of sexually transmitted disease.

Correct response:

has a known history of sexually transmitted disease.

Explanation:

Question 48 See full question

The nurse is teaching the family of a client diagnosed with leukemia about ways to prevent infection. Which
instruction has the most impact?

You Selected:

Maintaining an intact skin integrity

Correct response:

Maintaining an intact skin integrity

Explanation:

Question 49 See full question

The nurse is conducting a routine risk assessment at a prenatal visit. Which question would be the best to screen
for intimate partner violence?

You Selected:

How safe do you feel in your home?

Correct response:

How safe do you feel in your home?

Explanation:

Question 50 See full question

A 40-year-old client is admitted to the hospital with a diagnosis of acute cholecystitis. The nurse should contact the
health care provider (HCP) to question which prescription?

You Selected:

IV fluid therapy of normal saline solution to be infused at 100 mL/h until further prescriptions.

Correct response:

Administer meperidine hydrochloride 50 mg IM every 4 hours as needed for severe abdominal pain.

Explanation:

Question 1 See full question

A physician orders chest physiotherapy for a client with pulmonary congestion. When should the nurse plan to
perform chest physiotherapy?

You Selected:

When the client has time


Correct response:

Before meals

Explanation:

Question 2 See full question

When changing a sterile surgical dressing, a nurse first must:

You Selected:

wash her hands.

Correct response:

wash her hands.

Explanation:

Question 3 See full question

The staff of an outpatient clinic has formed a task force to develop new procedures for swift, safe evacuation of the
unit. The new procedures haven't been reviewed, approved, or shared with all personnel. When a nurse-manager
receives word of a bomb threat, the task force members push for evacuating the unit using the new procedures.
Which action should the nurse-manager take?

You Selected:

Tell staff members to use whatever procedures they feel are best.

Correct response:

Determine that the procedures currently in place must be followed and direct staff to follow them without
question.

Explanation:

Question 4 See full question

A nurse observes a 10-month-old infant chewing on the security alarm attached to his identification bracelet. The
nurse should:

You Selected:

distract the infant with a more appropriate toy.

Correct response:

distract the infant with a more appropriate toy.

Explanation:

Question 5 See full question

A nurse is teaching parents about accident prevention for a toddler. Which guideline is most appropriate?

You Selected:

Always make the toddler wear a seat belt when riding in a car.

Correct response:

Make sure all medications are kept in containers with childproof safety caps.

Explanation:

Question 6 See full question

A nurse discussing injury prevention with a group of workers at a day-care center is focusing on toddlers. When
discussing this age-group, the nurse should stress that:
You Selected:

toddlers can distinguish right from wrong.

Correct response:

accidents are the leading cause of death among toddlers.

Explanation:

Question 7 See full question

A nurse is teaching parents how to reduce the spread of impetigo. The nurse should encourage parents to:

You Selected:

have their children immunized against impetigo.

Correct response:

teach children the importance of proper hand washing.

Explanation:

Question 8 See full question

A 10-year-old child presents to the emergency department with dehydration. A physician orders 1 L of normal saline
solution be administered at a rate of 60 ml/hour. While preparing the infusion, a nurse notices that the I.V. pump's
safety inspection sticker has expired. Which action should the nurse take next?

You Selected:

After starting the fluids, contact the maintenance department and request a pump inspection.

Correct response:

Take the pump out of commission and locate a pump with a valid inspection sticker.

Explanation:

Question 9 See full question

A nurse is assigned to care for a recently admitted client who has attempted suicide. What should the nurse do?

You Selected:

Remind all staff members to check on the client frequently.

Correct response:

Search the client's belongings and his room carefully for items that could be used to attempt suicide.

Explanation:

Question 10 See full question

A nurse notices that a client admitted for treatment of major depression is pacing, agitated, and becoming verbally
aggressive toward other clients. What is the immediate care priority?

You Selected:

Isolating the agitated client and offering sedation to calm his behavior

Correct response:

Ensuring the safety of this client and other clients on the unit
Explanation:

Question 11 See full question

Which point should a nurse include when teaching mothers about preventing childhood falls?

You Selected:

Place the infant in an infant rocker when he's awake.

Correct response:

Never leave the infant alone on an elevated surface.

Explanation:

Question 12 See full question

An alarm signals, indicating that a neonate's security identification band requires attention. The nurse responds
immediately and finds that the parents removed the identification bands from the neonate. Which action should the
nurse take next?

You Selected:

Obtain the neonate's footprints and compare them with the footprints obtained at birth.

Correct response:

Compare the information on the neonate's identification bands with that of the mother's, then reattach the
identification bands to one of the neonate's extremities.

Explanation:

Question 13 See full question

While reviewing the day's charts, a nurse who's been under a great deal of personal stress realizes that she forgot
to administer insulin to client with diabetes mellitus. She's made numerous errors in the past few weeks and is now
afraid her job is in jeopardy. What is her best course of action?

You Selected:

Report the error and request a private meeting with the unit manager.

Correct response:

Report the error, complete the proper paperwork, and meet with the unit manager.

Explanation:

Question 14 See full question

The nurse is reconciling the medications with a client who is being discharged. Which information indicates there is
a "discrepancy"?

You Selected:

There is lack of congruence between a clients home medication list and current medication prescriptions.

Correct response:

There is lack of congruence between a clients home medication list and current medication prescriptions.

Explanation:

Question 15 See full question

Whichis an initial clinical manifestation of gonorrhea in men?

You Selected:

penile lesion
Correct response:

urethral discharge

Explanation:

Question 16 See full question

While performing daily peritoneal dialysis and catheter exit site care with the mother of a child with chronic renal
failure, which information would be an important step to emphasize to the mother?

You Selected:

Pull on the catheter to hold taut while cleaning the skin.

Correct response:

Examinine the site for signs of infection while cleaning the area.

Explanation:

Question 17 See full question

The mother of an 8-year-old with diabetes tells the nurse that she does not want the school to know about her
daughter's condition. The nurse should reply:

You Selected:

"What is it that concerns you about having the school know about your daughter's condition?"

Correct response:

"What is it that concerns you about having the school know about your daughter's condition?"

Explanation:

Question 18 See full question

The nurse in the emergency department is administering a prescription for 20 mg intravenous furosemide, which is
to be given immediately. The nurse scans the clients identification band and the medication barcode. The
medication administration system does not verify that furosemide is prescribed for this client; however, the
furosemide is prepared in the accurate unit dose for intravenous infusion. What should the nurse do next?

You Selected:

Report the problem to the information technology team to have the barcode system recalibrated.

Correct response:

Contact the pharmacist immediately to check the order and the barcode label for accuracy.

Explanation:

Question 19 See full question

To ensure safety for a hospitalized blind client, the nurse should:

You Selected:

require that the client stays in bed until the nurse can assist.

Correct response:

orient the client to the room environment.

Explanation:

Question 20 See full question

The nurse is admitting a client with glaucoma. The client brings prescribed eyedrops from home and insists on using
them in the hospital. The nurse should:
You Selected:

place the eyedrops in the hospital medication drawer and administer as labeled on the bottle.

Correct response:

explain to the client that the health care provider (HCP) will write a prescription for the eyedrops to be used at
the hospital.

Explanation:

Question 21 See full question

When changing a wet-to-dry dressing covering a surgical wound, what should the nurse do?

You Selected:

Cover the wet packing with a dry sterile dressing.

Correct response:

Cover the wet packing with a dry sterile dressing.

Explanation:

Question 22 See full question

A client with cervical cancer is undergoing internal radium implant therapy. A lead-lined container and a pair of long
forceps are kept in the client's hospital room for:

You Selected:

handling of the dislodged radiation source.

Correct response:

handling of the dislodged radiation source.

Explanation:

Question 23 See full question

The nurse should teach clients that the most common route of transmitting tubercle bacilli from person to person is
through contaminated:

You Selected:

eating utensils.

Correct response:

droplet nuclei.

Explanation:

Question 24 See full question

Which family member exposed to tuberculosis would be at highest risk for contracting the disease?

You Selected:

8-year-old son

Correct response:

76-year-old grandmother
Explanation:

Question 25 See full question

The nurse understands that the client who is undergoing induction therapy for leukemia needs additional instruction
when the client makes which statement?

You Selected:

"I cannot wait to get home to my cat!"

Correct response:

"I cannot wait to get home to my cat!"

Explanation:

Question 26 See full question

The nurse's best explanation for why the severely neutropenic client is placed in reverse isolation is that reverse
isolation helps prevent the spread of organisms:

You Selected:

by using special techniques to handle the client's linens and personal items.

Correct response:

to the client from sources outside the client's environment.

Explanation:

Question 27 See full question

A client is admitted to the facility with a productive cough, night sweats, and a fever. Which action is most
important in the initial care plan?

You Selected:

Wearing gloves during all client contact

Correct response:

Placing the client in respiratory isolation

Explanation:

Question 28 See full question

A 6-month-old on the pediatric floor has a respiratory rate of 68, mild intercostal retractions, and an oxygen
saturation of 89%. The infant has not been feeding well for the last 24 hours and is restless. Using the SBAR
(Situation-Background-Assessment-Recommendation) technique for communication, the nurse calls the health care
provider (HCP) with the recommendation for:

You Selected:

transferring the child to pediatric intensive care.

Correct response:

starting oxygen.

Explanation:

Question 29 See full question

A suicidal client is placed in the seclusion room and given lorazepam because she tried to harm herself by banging
her head against the wall. After 10 minutes, the client starts to bang her head against the wall in the seclusion
room. What action should the nurse take next?
You Selected:

Call the health care provider (HCP) for additional medication prescriptions.

Correct response:

Place the client in restraints.

Explanation:

Question 30 See full question

A 26-year-old is being treated for delirium due to acute alcohol intoxication. The client is restless, does not want to
stay seated, and has a staggering gait. What should the nurse do first?

You Selected:

Ask the client to sit in a chair next to the nurses' station.

Correct response:

Provide one-to-one supervision of the client until detoxification treatment can begin.

Explanation:

Question 31 See full question

When completing the preoperative checklist on the nursing unit, the nurse discovers an allergy that the client has
not reported. What should the nurse do first?

You Selected:

Contact the scrub nurse in the operating room.

Correct response:

Inform the anesthesiologist.

Explanation:

Question 32 See full question

A nurse on a night shift entered an elderly clients room during a scheduled check and discovered the client on the
floor beside her bed after falling when trying to ambulate to the washroom. After assessing and assisting the client
back to bed, the nurse has completed an incident report. What is the primary purpose of this particular type of
documentation?

You Selected:

Identifying risks and ensuring future safety for clients.

Correct response:

Identifying risks and ensuring future safety for clients.

Explanation:

Question 33 See full question

During rounds, a nurse finds that a client with hemiplegia has fallen from the bed because the nursing assistant
failed to raise the side rails after giving a back massage. The nurse assists the client to the bed and assesses for
injury. As per agency policies, the nurse fills out an incident report. Which of the following activities should the
nurse perform after finishing the incident report?

You Selected:

Include the time and date of the incident.


Correct response:

Include the time and date of the incident.

Explanation:

Question 34 See full question

A nurse assesses a client with psychotic symptoms and determines that the client likely poses a safety threat and
needs vest restraints. The client is adamantly opposed to this. What would be the best nursing action?

You Selected:

Restrain the client with vest restraints.

Correct response:

Contact the physician and obtain necessary orders.

Explanation:

Question 35 See full question

Over the past few weeks, a client in a long-term care facility has become increasingly unsteady. The nurses are
worried that the client will climb out of bed and fall. Which of the following measures does not comply with a least
restraint policy?

You Selected:

Raising one bed rail to offer stabilization when standing

Correct response:

Raising all side rails while the client is in bed

Explanation:

Question 36 See full question

When the client is involuntarily committed to a hospital because he is assessed as being dangerous to himself or
others, which client rights are lost?

You Selected:

the right to leave the hospital against medical advice

Correct response:

the right to leave the hospital against medical advice

Explanation:

Question 37 See full question

A client has been placed in an isolation room, and family members have stated that access to the client seems
restricted. Which of the following actions would be appropriate for the nurse to take to address this situation? Select
all that apply.

You Selected:

A communication plan for the family and client

Correct response:

A communication plan for the family and client


A thorough explanation of the isolation procedures
Acknowledgement of the family's concerns
Explanation:

Question 38 See full question

A nurse practitioner (NP) orders an antibiotic to which the client is allergic. The nurse preparing the medication
notices the allergy alert and contacts the NP by phone. The NP does not return the call and the first dose is due to
be given. Which of the following actions by the nurse is the best solution to this situation?

You Selected:

Hold the medication until speaking with the NP.

Correct response:

Hold the medication until speaking with the NP.

Explanation:

Question 39 See full question

A nurse reports to the hospital occupational health nurse (OHN) that he/she was splashed with blood during the
resuscitation of an HIV-positive client. The nurse asks the OHN when he/she will know whether he/she is positive or
negative for HIV infection. Which of the following is the most appropriate response by the OHN?

You Selected:

We will test you in 4 weeks, and then we will have a definitive answer.

Correct response:

Accurate results will be obtained by testing at 3 months and again at 6 months.

Explanation:

Question 40 See full question

The treatment team plans to place a client in full leather restraints. What is the best care for this client?

You Selected:

Remove the leather restraints every 60 minutes

Correct response:

Remove the leather restraints every 10-15 minutes

Explanation:

Question 41 See full question

A nurse is caring for a client with a fresh postoperative wound following a femoralpopliteal revascularization
procedure. The nurse fails to routinely assess the pedal pulses on the affected leg, and missed the warning sign
that the blood vessel was becoming occluded. The nurse manager is made aware of the complication and the
nurses failure to assess the client properly. What action should be taken by the nurse manager?

You Selected:

Address the nurses omissions as negligent behavior.

Correct response:

Address the nurses omissions as negligent behavior.

Explanation:

Question 42 See full question

A nurse notices the smell of marijuana on a nursing colleague upon return from lunch break. The colleague is
having difficulty drawing up a dose of insulin, appears uncoordinated, and is unaware that the needle has been
contaminated. What is the best action for the nurse to take?
You Selected:

Take the syringe and insulin vials, draw up the insulin, and instruct the colleague to focus more clearly when
giving the injection.

Correct response:

Stop the colleague from drawing up the insulin. Notify the supervisor about the incident, and document the
observations.

Explanation:

Question 43 See full question

A nurse recognizes that a client with tuberculosis needs further teaching when the client states:

You Selected:

"It will be necessary for the people I work with to take medication."

Correct response:

"It will be necessary for the people I work with to take medication."

Explanation:

Question 44 See full question

A nurse is caring for a client with acquired immunodeficiency syndrome (AIDS). To adhere to standard precautions,
the nurse should wear gloves when:

You Selected:

taking a blood pressure.

Correct response:

providing mouth care.

Explanation:

Question 45 See full question

A client is diagnosed with esophageal cancer and presents with difficulty swallowing. Which intervention should
receive the highest priority?

You Selected:

Ensuring adequate nutrition given the client's recent weight loss of 20 lbs (9.07 kg)

Correct response:

Establishing aspiration precautions

Explanation:

Question 46 See full question

A parent of a 9-year-old child who is scheduled to have surgery expresses concern about the potential for a
postoperative infection. Which of the following information would be most important for the nurse to tell the parent?

You Selected:

Do not bring fresh flowers or fruit to the room after surgery.

Correct response:

All visitors should wash their hands before they leave or enter the room.
Explanation:

Question 47 See full question

A charge nurse is completing client assignments for the nursing staff on the pediatric unit. Which client would the
nurse refrain from assigning to a pregnant staff member?

You Selected:

A 3-month-old with Roseola.

Correct response:

An 8-year-old with Rubella.

Explanation:

Question 48 See full question

The nurse is developing a care plan for a client who has had radiation therapy for Hodgkins lymphoma. What is the
primary goal of care for this client?

You Selected:

Maintain fluid balance.

Correct response:

Prevent infection.

Explanation:

Question 49 See full question

The nurse is caring for a client recently diagnosed with hepatitis C. In reviewing the clients history, what
information will be most helpful as the nurse develops a teaching plan? The client:

You Selected:

has a known history of sexually transmitted disease.

Correct response:

has a known history of sexually transmitted disease.

Explanation:

Question 50 See full question

A client is admitted with an infectious wound. Contact precautions are initiated. To help the client cope with staff
using isolation procedures, which nursing action is most helpful?

You Selected:

Discuss the rationale for contact precautions

Correct response:

Discuss the rationale for contact precautions

Explanation:

Question 1 See full question

A physician has ordered a heating pad for an elderly client's lower back pain. Which item would be most important
for a nurse to assess before applying the heating pad?

You Selected:

Client's risk for falls


Correct response:

Client's level of consciousness

Explanation:

Question 2 See full question

After an infant undergoes surgical repair of a cleft lip, the physician orders elbow restraints. For this infant, the
postoperative care plan should include which nursing action?

You Selected:

Using the restraints until the infant recovers fully from anesthesia

Correct response:

Removing the restraints every 2 hours

Explanation:

Question 3 See full question

A 3-month-old infant with meningococcal meningitis has just been admitted to the pediatric unit. Which nursing
intervention has the highest priority?

You Selected:

obtaining history information from the parents

Correct response:

instituting droplet precautions

Explanation:

Question 4 See full question

Which of the following objects poses the most serious safety threat to a 2-year-old child in the hospital?

You Selected:

Side rails in the halfway position

Correct response:

Side rails in the halfway position

Explanation:

Question 5 See full question

A nurse is teaching parents about accident prevention for a toddler. Which guideline is most appropriate?

You Selected:

Make sure all medications are kept in containers with childproof safety caps.

Correct response:

Make sure all medications are kept in containers with childproof safety caps.

Explanation:

Question 6 See full question

A client found sitting on the floor of the bathroom in the day treatment clinic has moderate lacerations on both
wrists. Surrounded by broken glass, she sits staring blankly at the lacerations. What is the most important action for
the nurse to take next to the client?
You Selected:

Approach the client slowly while speaking in a calm voice, calling her by her name, and telling her that the
nurse is there to help her.

Correct response:

Approach the client slowly while speaking in a calm voice, calling her by her name, and telling her that the
nurse is there to help her.

Explanation:

Question 7 See full question

Which instruction should a nurse include in a home-safety teaching plan for a pregnant client?

You Selected:

Place a nonskid mat on the floor of the tub or shower.

Correct response:

Place a nonskid mat on the floor of the tub or shower.

Explanation:

Question 8 See full question

Which instruction should a nurse include in an injury-prevention plan for a pregnant client?

You Selected:

"Wear your seat belt across your tummy."

Correct response:

"Take rest periods during the day."

Explanation:

Question 9 See full question

A nurse is preparing to perform a postpartum assessment on a client who gave birth 5 hours ago. Which precaution
should the nurse plan to take for this procedure?

You Selected:

Washing the hands

Correct response:

Washing the hands and wearing latex gloves

Explanation:

Question 10 See full question

Which point should a nurse include when teaching mothers about preventing childhood falls?

You Selected:

Use small pillows in the crib to prevent the infant from rolling onto the stomach.

Correct response:

Never leave the infant alone on an elevated surface.

Explanation:

Question 11 See full question

Which action is the best precaution against transmission of infection?


You Selected:

Strict isolation for a neonate whose mother has human immunodeficiency virus (HIV)

Correct response:

Eye prophylaxis with antibiotics for a neonate whose mother has gonorrhea infection

Explanation:

Question 12 See full question

A nurse preparing to discharge a child with leukemia observes a family member who has a cold sharing a meal with
the child. How should the nurse approach the situation?

You Selected:

Tell family members to be careful to avoid the child if they're sick.

Correct response:

Offer a face mask to the person with the cold and use this as an opportunity for further teaching.

Explanation:

Question 13 See full question

While reviewing the day's charts, a nurse who's been under a great deal of personal stress realizes that she forgot
to administer insulin to client with diabetes mellitus. She's made numerous errors in the past few weeks and is now
afraid her job is in jeopardy. What is her best course of action?

You Selected:

Contact the physician and follow his instructions.

Correct response:

Report the error, complete the proper paperwork, and meet with the unit manager.

Explanation:

Question 14 See full question

A physician enters a computer order for a nurse to irrigate a client's nephrostomy tube every 4 hours to maintain
patency. The nurse irrigates the tube using sterile technique. After irrigating the tube, the nurse decides that she
can safely use the same irrigation set for her 8-hour shift if she covers the set with a paper, sterile drape. This
action by the nurse is:

You Selected:

appropriate because the irrigation just checks for patency.

Correct response:

inappropriate because irrigation requires strict sterile technique.

Explanation:

Question 15 See full question

A client lives in a group home and visits the community mental health center regularly. During one visit with the
nurse, the client states, "The voices are telling me to hurt myself again." Which question by the nurse
is most important to ask?

You Selected:

"How long have you heard the voices?"


Correct response:

"Are you going to hurt yourself?"

Explanation:

Question 16 See full question

A nurse is planning care for an elderly client with cognitive impairment who is still living at home. Which action
should the nurse identify as a priority for safety in planning care for this client?

You Selected:

putting the client's favorite belongings in a safe place so that he will not lose them

Correct response:

ensuring the removal of objects in the client's path that may cause him to trip

Explanation:

Question 17 See full question

Which dietary strategy best meets the nutritional needs of a client with acquired immunodeficiency syndrome
(AIDS)?

You Selected:

Tell the client to prepare food in advance and leave it out to eat small amounts throughout the day.

Correct response:

Instruct the client to cook foods thoroughly and adhere to safe food-handling practices.

Explanation:

Question 18 See full question

A 14-year-old with rheumatic fever who is on bed rest is receiving an IV infusion of dextrose 5% r administered by
an infusion pump. The nurse should verify the alarm settings on the infusion pump at which times? Select all that
apply.

You Selected:

when the child returns from X-ray

Correct response:

when the infusion is started


at the beginning of each shift
when the child returns from X-ray

Explanation:

Question 19 See full question

When developing the teaching plan for the mother of a 2-year-old child diagnosed with scabies, what information
should the nurse expect to include?

You Selected:

The floors of the house should be cleaned with a damp mop.

Correct response:

The entire family should be treated.


Explanation:

Question 20 See full question

A suspected outbreak of anthrax has been transmitted by skin exposure. A client is admitted to the emergency
department with lesions on the hands. The physician prescribes antibiotics and sends the client home. What should
the nurse instruct the client to do? Select all that apply.

You Selected:

Expect the skin lesions to clear up within 1 to 2 weeks.

Correct response:

Take the prescribed antibiotics for 60 days.


Expect the skin lesions to clear up within 1 to 2 weeks.

Explanation:

Question 21 See full question

When changing a wet-to-dry dressing covering a surgical wound, what should the nurse do?

You Selected:

Cover the wet packing with a dry sterile dressing.

Correct response:

Cover the wet packing with a dry sterile dressing.

Explanation:

Question 22 See full question

The mother of a client who has a radium implant asks why so many nurses are involved in her daughters care. She
states, The doctor said I can be in the room for up to 2 hours each day, but the nurses say they are restricted to 30
minutes. The nurse explains that this variation is based on the fact that nurses:

You Selected:

are at greater risk from the radiation because they are younger than the mother.

Correct response:

work with radiation on an ongoing basis, while visitors have infrequent exposure to radiation.

Explanation:

Question 23 See full question

The nurse should use which type of precautions for a client being admitted to the hospital with suspected
tuberculosis?

You Selected:

airborne precautions

Correct response:

airborne precautions

Explanation:

Question 24 See full question

The nurse understands that the client who is undergoing induction therapy for leukemia needs additional instruction
when the client makes which statement?
You Selected:

"I must report a temperature of 100 F (37.7 C)."

Correct response:

"I cannot wait to get home to my cat!"

Explanation:

Question 25 See full question

In caring for the client with hepatitis B, which situation would expose the nurse to the virus?

You Selected:

contact with fecal material

Correct response:

a blood splash into the nurses eyes

Explanation:

Question 26 See full question

The nurse is instructing the unlicensed assistive personnel (UAP) on how to position the wheelchair to assist a client
with left-sided weakness transfer from the bed to a wheelchair using a transfer belt. Which statement by the UAP
tells the nurse that the UAP has understood the instructions for placing the wheelchair?

You Selected:

The wheelchair should be placed at the head of the bed.

Correct response:

The wheelchair should be placed on the right side of the bed.

Explanation:

Question 27 See full question

The nurse is administering an intramuscular injection to an infant. Indicate the appropriate site for this injection.
You Selected:

Your selection and the correct area, market by the green box.

Explanation:

Question 28 See full question

Before inserting a nasogastric (NG) tube in an adult client, the nurse estimates the length of tubing to insert.
Identify the point on the illustration where the nurse would end the measurement.

You Selected:

Your selection and the correct area, market by the green box.
Explanation:

Question 29 See full question

A nurse is giving discharge teaching to a client with an eye injury. Which statement about preventing eye injuries
should the nurse include?

You Selected:

"Make sure you stand next to, not in front, of a moving lawn mower."

Correct response:

"Direct all spray nozzles away from your face before spraying."

Explanation:

Question 30 See full question

Which client is at highest risk for developing a hospital-acquired infection?

You Selected:

A client who's taking prednisone

Correct response:

A client with an indwelling urinary catheter

Explanation:

Question 31 See full question

When creating an educational program about safety, what information should the nurse include about sexual
predators? Select all that apply.

You Selected:

Child molesters resort to molestation because they have bad childhoods, so understanding that can help them
decrease their molesting.

Correct response:

Child molesters pick children or teens over which they have some authority, making it easier for them to
manipulate the child with special favors or attention.
Child molesters gain the childs trust before making sexual advances so the child feels obligated to comply
with sex.
Child molesters often choose children whose parents must work long hours, making the extra attention initially
welcomed by the child.
Child molesters maintain the secrecy of their actions by making threats if offering attention and favors fail or if
the child is close to revealing the secret.

Explanation:

Question 32 See full question

The nurse assesses a client to be at risk for self-mutilation and implements a safety contract with the client. Which
client behavior indicates that the contract is working?

You Selected:

The client withdraws to his room when feeling overwhelmed.

Correct response:

The client notifies staff when anxiety is increasing.


Explanation:

Question 33 See full question

A client has a history of macular degeneration. While in the hospital, the priority nursing goal will be to:

You Selected:

improve vision.

Correct response:

promote a safe, effective care environment.

Explanation:

Question 34 See full question

A 26-year-old is being treated for delirium due to acute alcohol intoxication. The client is restless, does not want to
stay seated, and has a staggering gait. What should the nurse do first?

You Selected:

Provide one-to-one supervision of the client until detoxification treatment can begin.

Correct response:

Provide one-to-one supervision of the client until detoxification treatment can begin.

Explanation:

Question 35 See full question

Over the past few weeks, a client in a long-term care facility has become increasingly unsteady. The nurses are
worried that the client will climb out of bed and fall. Which of the following measures does not comply with a least
restraint policy?

You Selected:

Providing a bed that is low to the floor

Correct response:

Raising all side rails while the client is in bed

Explanation:

Question 36 See full question

Which prescription is entered correctly on the medical record?

You Selected:

fentanyl 50 mcg given IV every 2 hours as needed for pain greater than 6/10

Correct response:

fentanyl 50 mcg given IV every 2 hours as needed for pain greater than 6/10

Explanation:

Question 37 See full question

When the client is involuntarily committed to a hospital because he is assessed as being dangerous to himself or
others, which client rights are lost?

You Selected:

the right to send and receive uncensored mail


Correct response:

the right to leave the hospital against medical advice

Explanation:

Question 38 See full question

The nurse is caring for an immune compromised client with a fungal infection of the scalp. What recommendation
should the nurse make to prevent future problems?

You Selected:

Allow hair to air dry after shampooing.

Correct response:

Avoid sharing combs and brushes.

Explanation:

Question 39 See full question

A restraint order is implemented for a client who is restless and combative due to alcohol intoxication. What is the
most appropriate nursing intervention for this client?

You Selected:

Secure the restraints to the bed with knots to ensure the client cannot undo them.

Correct response:

Check the extremities for circulation based on hospital protocols.

Explanation:

Question 40 See full question

While hospitalized, a child develops a Clostridium difficile infection. The nurse can anticipate adding which of the
following types of precautions for this client?

You Selected:

Airborne precautions

Correct response:

Contact precautions

Explanation:

Question 41 See full question

The nurse is administering a subcutaneous injection (see accompanying figure). After releasing the skin, prior to
injecting the medication, the needle pulls out of the skin. The nurse should:

You Selected:

using a new needle, syringe, and medication, stretch the skin taut and administer the medication.

Correct response:

discard the needle, attach a new needle to the syringe, and administer the medication.

Explanation:

Question 42 See full question

Which action by the nursing assistant would require immediate intervention by the nurse?
You Selected:

Assisting a preschool-age child in the bathroom with the door closed

Correct response:

Restraining a school-age child at risk for self-harm because the nursing assistant had to leave the room

Explanation:

Question 43 See full question

A nurse-manager identifies fall prevention as a unit priority. Which of the following actions can the nurses
implement to meet these goals? Select all that apply.

You Selected:

Apply soft waist restraint to confused clients

Correct response:

Use bed alarms to remind clients to call for help getting up


Maintain a clear path to client bathrooms
Make hourly rounds to client rooms

Explanation:

Question 44 See full question

A charge nurse is completing client assignments for the nursing staff on the pediatric unit. Which client would the
nurse refrain from assigning to a pregnant staff member?

You Selected:

A 2-year-old with Kawasakis disease.

Correct response:

An 8-year-old with Rubella.

Explanation:

Question 45 See full question

A client has an indwelling urinary catheter and is prescribed physical therapy. As the client is being placed in a
wheelchair, which action by the assistant would need further clarification by the nurse?

You Selected:

The assistant brings a container to drain the urine from the bag.

Correct response:

The catheter bag is placed upon the clients lap for safe transport.

Explanation:

Question 46 See full question

Which nursing action best addresses the outcome: The client will be free from falls?

You Selected:

Place emergency contacts telephone number in a prominent place.

Correct response:

Encourage use of grab bars and railings in the bathroom and halls
Explanation:

Question 47 See full question

A client is admitted with an infectious wound. Contact precautions are initiated. To help the client cope with staff
using isolation procedures, which nursing action is most helpful?

You Selected:

Don gloves when providing all client care

Correct response:

Discuss the rationale for contact precautions

Explanation:

Question 48 See full question

The nurse is teaching the family of a client diagnosed with leukemia about ways to prevent infection. Which
instruction has the most impact?

You Selected:

Covering the clients mouth when coughing

Correct response:

Maintaining an intact skin integrity

Explanation:

Question 49 See full question

The nurse has provided an in-service presentation to ancillary staff about standard precautions on the birthing unit.
The nurse determines that one of the staff members needs further instructions when the nurse observes which
action?

You Selected:

use of protective goggles during a caesarean birth

Correct response:

wearing of sterile gloves to bathe a neonate at 2 hours of age

Explanation:

Question 50 See full question

The nurse is conducting a routine risk assessment at a prenatal visit. Which question would be the best to screen
for intimate partner violence?

You Selected:

How safe do you feel in your home?

Correct response:

How safe do you feel in your home?

Explanation:

When leaving the room of a client in strict isolation, the nurse should remove which protective equipment first?

You Selected:

Mask
Correct response:

Gloves

Explanation:

Question 2 See full question

A client has a soft wrist-safety device. Which assessment finding should the nurse investigate further?

You Selected:

A palpable radial pulse

Correct response:

Cool, pale fingers

Explanation:

Question 3 See full question

A nurse gives a client the wrong medication. After assessing the client, the nurse completes an incident report.
Which statement describes what will happen next?

You Selected:

The facility will report the incident to the state board of nursing for disciplinary action.

Correct response:

The incident report will provide a basis for promoting quality care and risk management.

Explanation:

Question 4 See full question

While performing rounds, a nurse finds that a client is receiving the wrong I.V. solution. The nurse's initial response
should be to:

You Selected:

remove the I.V. catheter and call the physician.

Correct response:

slow the I.V. flow rate and hang the appropriate solution.

Explanation:

Question 5 See full question

After an instructor has posted assignments, a person claiming to be a nursing student arrives on a unit and asks a
nurse for access to the medication records of a client to whom she's assigned. The student's only identification (ID)
is a laboratory coat with the school's name on it. What is the nurse's most appropriate response?

You Selected:

Allow the student supervised access to the client's medication record.

Correct response:

Ask the student to provide a photo ID for comparison with the names on the assignment sheet.

Explanation:

Question 6 See full question

A 3-month-old infant with meningococcal meningitis has just been admitted to the pediatric unit. Which nursing
intervention has the highest priority?
You Selected:

instituting droplet precautions

Correct response:

instituting droplet precautions

Explanation:

Question 7 See full question

A mother tells the nurse that her preschool-aged daughter with spina bifida sneezes and gets a rash when playing
with brightly colored balloons, and that recently she had an allergic reaction after eating kiwi fruit and bananas.
Based on the mother's report, the nurse suspects that the child may have an allergy to:

You Selected:

Latex.

Correct response:

Latex.

Explanation:

Question 8 See full question

Which use of restraints in a school-age child should the nurse question?

You Selected:

To substitute for observation

Correct response:

To substitute for observation

Explanation:

Question 9 See full question

A 10-year-old child presents to the emergency department with dehydration. A physician orders 1 L of normal saline
solution be administered at a rate of 60 ml/hour. While preparing the infusion, a nurse notices that the I.V. pump's
safety inspection sticker has expired. Which action should the nurse take next?

You Selected:

Begin the infusion of the fluids while looking for a pump with a valid inspection sticker.

Correct response:

Take the pump out of commission and locate a pump with a valid inspection sticker.

Explanation:

Question 10 See full question

A 15-year-old adolescent confides in the nurse that he has been contemplating suicide. He says he has developed a
specific plan to carry it out and pleads with the nurse not to tell anyone. What is the nurse's best response?

You Selected:

"I will need to notify the local authorities of your intentions."

Correct response:

"For your protection, I can't keep this secret. After I notify the physician, we will need to involve your family. We
want you to be safe."
Explanation:

Question 11 See full question

A client refuses his evening dose of haloperidol and then becomes extremely agitated in the day room while other
clients are watching television. He begins cursing and throwing furniture. The nurse's first action is to:

You Selected:

check the client's medical record for an order for an as-needed dose of medication for agitation.

Correct response:

remove all other clients from the day room.

Explanation:

Question 12 See full question

A nurse is assigned to care for a recently admitted client who has attempted suicide. What should the nurse do?

You Selected:

Express trust that the client won't harm himself while in the facility.

Correct response:

Search the client's belongings and his room carefully for items that could be used to attempt suicide.

Explanation:

Question 13 See full question

A client found sitting on the floor of the bathroom in the day treatment clinic has moderate lacerations on both
wrists. Surrounded by broken glass, she sits staring blankly at the lacerations. What is the most important action for
the nurse to take next to the client?

You Selected:

Sit quietly next to her.

Correct response:

Approach the client slowly while speaking in a calm voice, calling her by her name, and telling her that the
nurse is there to help her.

Explanation:

Question 14 See full question

In the emergency department, a client with facial lacerations states that her husband beat her with a shoe. After
her lacerations are repaired, the client waits to be seen by the crisis intake nurse, who will evaluate the continued
threat of violence her husband represents. Suddenly the client's husband arrives, shouting that he wants to "finish
the job." What is the first priority of the nurse who witnesses this scene?

You Selected:

Calling a security guard and another staff member for assistance

Correct response:

Calling a security guard and another staff member for assistance

Explanation:

Question 15 See full question

A client in early labor is connected to an external fetal monitor. The physician hasn't noted any restrictions on her
chart. The client tells the nurse that she needs to go to the bathroom frequently and that her partner can help her.
How should the nurse respond?
You Selected:

"I'll insert a urinary catheter; then you won't need to get out of bed."

Correct response:

"Please press the call button. I'll disconnect you from the monitor so you can get out of bed."

Explanation:

Question 16 See full question

A nurse is orienting a new nurse to the labor and delivery unit. Which action by the new nurse regarding a
neonate's security requires intervention by the preceptor?

You Selected:

Allowing volunteers to return neonates to the nursery

Correct response:

Allowing volunteers to return neonates to the nursery

Explanation:

Question 17 See full question

An alarm signals, indicating that a neonate's security identification band requires attention. The nurse responds
immediately and finds that the parents removed the identification bands from the neonate. Which action should the
nurse take next?

You Selected:

Compare the information on the neonate's identification bands with that of the mother's, then reattach the
identification bands to one of the neonate's extremities.

Correct response:

Compare the information on the neonate's identification bands with that of the mother's, then reattach the
identification bands to one of the neonate's extremities.

Explanation:

Question 18 See full question

A nurse preparing to discharge a child with leukemia observes a family member who has a cold sharing a meal with
the child. How should the nurse approach the situation?

You Selected:

Offer a face mask to the person with the cold and use this as an opportunity for further teaching.

Correct response:

Offer a face mask to the person with the cold and use this as an opportunity for further teaching.

Explanation:

Question 19 See full question

When a nurse removes an I.V. from an client with acquired immunodeficiency syndrome (AIDS), blood splashes into
her eyes. What should the nurse do next?

You Selected:

Rinse her eyes with water, record the incident on the client's chart, and see Employee Health.

Correct response:

Rinse her eyes with water, report the incident, and go to Employee Health.
Explanation:

Question 20 See full question

Whichis an initial clinical manifestation of gonorrhea in men?

You Selected:

impotence

Correct response:

urethral discharge

Explanation:

Question 21 See full question

When a client has a tearing of tissue with irregular wound edges, the nurse should document this as:

You Selected:

laceration.

Correct response:

laceration.

Explanation:

Question 22 See full question

A 14-year-old with rheumatic fever who is on bed rest is receiving an IV infusion of dextrose 5% r administered by
an infusion pump. The nurse should verify the alarm settings on the infusion pump at which times? Select all that
apply.

You Selected:

when the infusion is started

Correct response:

when the infusion is started


at the beginning of each shift
when the child returns from X-ray

Explanation:

Question 23 See full question

An alert and oriented elderly client is admitted to the hospital for treatment of cellulitis of the left shoulder after an
arthroscopy. Which fall prevention strategy is most appropriate for this client?

You Selected:

Keep all the lights on in the room at all times.

Correct response:

Use a nightlight in the bathroom.

Explanation:

Question 24 See full question

The nurse develops a teaching plan for the client about how to prevent the transmission of hepatitis A. Which
discharge instruction is appropriate for the client?
You Selected:

Ask family members to wash their hands frequently.

Correct response:

Ask family members to wash their hands frequently.

Explanation:

Question 25 See full question

Which precautions should the health care team observe when caring for clients with hepatitis A?

You Selected:

wearing a mask when providing care

Correct response:

wearing gloves when giving direct care

Explanation:

Question 26 See full question

A nurse is caring for a client who has just returned from surgery to treat a fractured mandible. The jaws are wired.
Which items should always be available at this clients bedside? Select all that apply.

You Selected:

nasogastric tube

Correct response:

wire cutters
suction equipment

Explanation:

Question 27 See full question

The nurse should teach clients that the most common route of transmitting tubercle bacilli from person to person is
through contaminated:

You Selected:

water.

Correct response:

droplet nuclei.

Explanation:

Question 28 See full question

The nurse should use which type of precautions for a client being admitted to the hospital with suspected
tuberculosis?

You Selected:

hand hygeine

Correct response:

airborne precautions
Explanation:

Question 29 See full question

The nurse is instructing the unlicensed assistive personnel (UAP) on how to position the wheelchair to assist a client
with left-sided weakness transfer from the bed to a wheelchair using a transfer belt. Which statement by the UAP
tells the nurse that the UAP has understood the instructions for placing the wheelchair?

You Selected:

The wheelchair should be placed at the head of the bed.

Correct response:

The wheelchair should be placed on the right side of the bed.

Explanation:

Question 30 See full question

A client with a suspected brain tumor is scheduled for a computed tomography (CT) scan. What should the nurse do
when preparing the client for this test?

You Selected:

Determine whether the client is allergic to iodine, contrast dyes, or shellfish.

Correct response:

Determine whether the client is allergic to iodine, contrast dyes, or shellfish.

Explanation:

Question 31 See full question

A nurse is giving discharge teaching to a client with an eye injury. Which statement about preventing eye injuries
should the nurse include?

You Selected:

"When working in a workshop, you don't need to wear safety goggles unless you're the person using the
tools."

Correct response:

"Direct all spray nozzles away from your face before spraying."

Explanation:

Question 32 See full question

A client admitted with bacterial meningitis must be transported to the radiology department for a repeat computed
tomography scan of the head. His level of consciousness is decreased, and he requires nasopharyngeal suctioning
before transport. Which infection control measures are best when caring for this client?

You Selected:

Put on gloves, a mask, and eye protection.

Correct response:

Put on gloves, a mask, and eye protection during suctioning, and then apply a mask to the client's face for
transport.

Explanation:

Question 33 See full question

A client with chronic obstructive pulmonary disease (COPD) is intubated and placed on continuous mechanical
ventilation. Which equipment is most important for the nurse to keep at this client's bedside?
You Selected:

Manual resuscitation bag

Correct response:

Manual resuscitation bag

Explanation:

Question 34 See full question

After administering an I.M. injection, a nurse should:

You Selected:

recap the needle and discard the needle and syringe in a puncture-proof container.

Correct response:

discard the uncapped needle and syringe in a puncture-proof container.

Explanation:

Question 35 See full question

A 6-month-old on the pediatric floor has a respiratory rate of 68, mild intercostal retractions, and an oxygen
saturation of 89%. The infant has not been feeding well for the last 24 hours and is restless. Using the SBAR
(Situation-Background-Assessment-Recommendation) technique for communication, the nurse calls the health care
provider (HCP) with the recommendation for:

You Selected:

providing sedation.

Correct response:

starting oxygen.

Explanation:

Question 36 See full question

The nurse is caring for a client who is confused about time and place. The client has intravenous fluid infusing. The
nurse attempts to reorient the client, but the client remains unable to demonstrate appropriate use of the call light.
In order to maintain client safety, what should the nurse do first?

You Selected:

Increase the frequency of client observation.

Correct response:

Increase the frequency of client observation.

Explanation:

Question 37 See full question

An alert and oriented older adult female with metastatic lung cancer is admitted to the medical-surgical unit for
treatment of heart failure. She was given 80 mg of furosemide in the emergency department. Although the client is
ambulatory, the unlicensed assistive personnel (UAP) are concerned about urinary incontinence because the client
is frail and in a strange environment. The nurse should instruct the UAP to assist with implementing the nursing
plan of care by:

You Selected:

requesting an indwelling urinary catheter to avoid incontinence


Correct response:

placing a commode at the bedside and instructing the client in its use

Explanation:

Question 38 See full question

A client has a history of macular degeneration. While in the hospital, the priority nursing goal will be to:

You Selected:

provide education regarding community services for clients with adult macular degeneration (AMD).

Correct response:

promote a safe, effective care environment.

Explanation:

Question 39 See full question

The nurse sees a client walking in the hallway who begins to have a seizure. What should the nurse do in order of
priority from first to last? All options must be used.

You Selected:

Ease the client to the floor.


Maintain a patent airway.
Record the seizure activity observed.
Obtain vital signs.

Correct response:

Ease the client to the floor.


Maintain a patent airway.
Obtain vital signs.
Record the seizure activity observed.

Explanation:

Question 40 See full question

A nurse is documenting a variance that has occurred during the shift, and this report will be used for quality
improvement to identify high-risk patterns and potentially initiate in-service programs. This is an example of which
type of report?

You Selected:

Incident report.

Correct response:

Incident report.

Explanation:

Question 41 See full question

After the discharge of a client from a surgical unit, the housekeeper brings a blue pill to the nurse. The pill was
found in the sheets when the linens were removed from the clients bed. The nurse reviews the clients medication
administration record, which shows that the client received this medication at 0800. What would be the nurses
priority action?

You Selected:

Advise the housekeeper to throw the pill in the garbage.


Correct response:

Complete an incident form and notify the physician.

Explanation:

Question 42 See full question

A nurse practitioner (NP) orders an antibiotic to which the client is allergic. The nurse preparing the medication
notices the allergy alert and contacts the NP by phone. The NP does not return the call and the first dose is due to
be given. Which of the following actions by the nurse is the best solution to this situation?

You Selected:

Ask if the client is really allergic to the medication.

Correct response:

Hold the medication until speaking with the NP.

Explanation:

Question 43 See full question

A nurse administers digoxin 0.125 mg to a client at 1400 instead of the prescribed dose of digoxin 0.25 mg. Which
of the following statements should the nurse record in the medical record?

You Selected:

Digoxin 0.125 mg given at 1400 instead of prescribed dose of 0.25 mg.

Correct response:

Digoxin 0.125 mg given at 1400 instead of prescribed dose of 0.25 mg.

Explanation:

Question 44 See full question

The nurse is caring for a 5-year-old child who has a history of multiple admissions for fractures and cuts. The
mother explains that the child fractured the femur by falling, but does not give any further details. The child
indicates that the mothers boyfriend was present when the injury occurred, and the childs recollection of the event
conflicts with the mothers explanation. What is the nurses immediate responsibility?

You Selected:

Restrict family who are visiting the child.

Correct response:

Keep the child safe and assess for abuse.

Explanation:

Question 45 See full question

A nurse is supervising a student during medication administration to a client. Which of the following action by the
student would cause the nurse to intervene during the med pass at the bedside?

You Selected:

First asks the client's name.

Correct response:

Check the room number and the client's name on the bed.
Explanation:

Question 46 See full question

A nurse-manager identifies fall prevention as a unit priority. Which of the following actions can the nurses
implement to meet these goals? Select all that apply.

You Selected:

Maintain a clear path to client bathrooms

Correct response:

Use bed alarms to remind clients to call for help getting up


Maintain a clear path to client bathrooms
Make hourly rounds to client rooms

Explanation:

Question 47 See full question

A charge nurse is completing client assignments for the nursing staff on the pediatric unit. Which client would the
nurse refrain from assigning to a pregnant staff member?

You Selected:

A 2-year-old with Kawasakis disease.

Correct response:

An 8-year-old with Rubella.

Explanation:

Question 48 See full question

A client has an indwelling urinary catheter and is prescribed physical therapy. As the client is being placed in a
wheelchair, which action by the assistant would need further clarification by the nurse?

You Selected:

The assistant brings a container to drain the urine from the bag.

Correct response:

The catheter bag is placed upon the clients lap for safe transport.

Explanation:

Question 49 See full question

Which nursing action best addresses the outcome: The client will be free from falls?

You Selected:

Install a monitoring system to help the client in an emergency situation.

Correct response:

Encourage use of grab bars and railings in the bathroom and halls

Explanation:

Question 50 See full question

The nurse has provided an in-service presentation to ancillary staff about standard precautions on the birthing unit.
The nurse determines that one of the staff members needs further instructions when the nurse observes which
action?
You Selected:

disposal of used scalpel blades in a puncture-resistant container

Correct response:

wearing of sterile gloves to bathe a neonate at 2 hours of age

Explanation:

Question 1 See full question

A client had a repair of a thoracoabdominal aneurysm 2 days ago. Which finding should the nurse consider
unexpected and report to the health care provider (HCP) immediately?

You Selected:

adominal pain at 5 on a scale of 0 to 10 for the last 2 days

Correct response:

weakness and numbness in the lower extremities

Explanation:

Question 2 See full question

A client found sitting on the floor of the bathroom in the day treatment clinic has moderate lacerations on both
wrists. Surrounded by broken glass, she sits staring blankly at the lacerations. What is the most important action for
the nurse to take next to the client?

You Selected:

Approach the client slowly while speaking in a calm voice, calling her by her name, and telling her that the
nurse is there to help her.

Correct response:

Approach the client slowly while speaking in a calm voice, calling her by her name, and telling her that the
nurse is there to help her.

Explanation:

Question 3 See full question

Which of the following objects poses the most serious safety threat to a 2-year-old child in the hospital?

You Selected:

Crayons and paper

Correct response:

Side rails in the halfway position

Explanation:

Question 4 See full question

When developing the teaching plan for the mother of a 2-year-old child diagnosed with scabies, what information
should the nurse expect to include?

You Selected:

Itching should cease in a few days.

Correct response:

The entire family should be treated.


Explanation:

Question 5 See full question

Which prescription is entered correctly on the medical record?

You Selected:

60.0 mg ketorolac tromethamine given IM for c/o pain

Correct response:

fentanyl 50 mcg given IV every 2 hours as needed for pain greater than 6/10

Explanation:

Question 6 See full question

A charge nurse is completing client assignments for the nursing staff on the pediatric unit. Which client would the
nurse refrain from assigning to a pregnant staff member?

You Selected:

A 2-year-old with Kawasakis disease.

Correct response:

An 8-year-old with Rubella.

Explanation:

Question 7 See full question

A nurse is preparing to administer an I.V. containing dextrose 10% in normal saline solution to a 6-month-old
infant. The nurse should select which tubing to safely administer the solution?

You Selected:

I.V. tubing with a volume-control chamber

Correct response:

I.V. tubing with a volume-control chamber

Explanation:

Question 8 See full question

A nursing instructor is instructing group of new nursing students. The instructor reviews that surgical asepsis will be
used for which of the following procedures?

You Selected:

I.V. catheter insertion

Correct response:

I.V. catheter insertion

Explanation:

Question 9 See full question

The nurse assists the client to the operating room table and supervises the operating room technician preparing the
sterile field. Which action, completed by the surgical technician, indicates to the nurse that a sterile field has been
contaminated?

You Selected:

Wetness in the sterile cloth on top of the nonsterile table has been noted.
Correct response:

Wetness in the sterile cloth on top of the nonsterile table has been noted.

Explanation:

Question 10 See full question

The nurse is inserting a nasogastric tube in an infant to administer feedings. In the accompanying figure, indicate
the location for the correct placement of the distal end of the tube.

You Selected:

Your selection and the correct area, market by the green box.

Explanation:

Question 11 See full question

A client refuses his evening dose of haloperidol and then becomes extremely agitated in the day room while other
clients are watching television. He begins cursing and throwing furniture. The nurse's first action is to:

You Selected:

call the physician and report the behavior.

Correct response:

remove all other clients from the day room.

Explanation:

Question 12 See full question

A nurse is documenting a variance that has occurred during the shift, and this report will be used for quality
improvement to identify high-risk patterns and potentially initiate in-service programs. This is an example of which
type of report?

You Selected:

Incident report.

Correct response:

Incident report.
Explanation:

Question 13 See full question

As a client is being released from restraints, he says, "I'll never get that angry and lose it again. Those restraints
were the worst things that ever happened to me." Which response by the nurse is most appropriate?

You Selected:

"Someday this experience won't bother you like it does now."

Correct response:

"I'd like to talk with you about your experience."

Explanation:

Question 14 See full question

A nurse is preparing to perform a postpartum assessment on a client who gave birth 5 hours ago. Which precaution
should the nurse plan to take for this procedure?

You Selected:

Washing the hands and wearing latex gloves and a barrier gown

Correct response:

Washing the hands and wearing latex gloves

Explanation:

Question 15 See full question

Which instruction should a nurse include in a home-safety teaching plan for a pregnant client?

You Selected:

It's OK to clean your cat's litter box.

Correct response:

Place a nonskid mat on the floor of the tub or shower.

Explanation:

Question 16 See full question

While making rounds, the nurse enters a clients room and finds the client on the floor between the bed and the
bathroom. The nurse should first:

You Selected:

assist the client back to bed.

Correct response:

assess the clients current condition and vital signs.

Explanation:

Question 17 See full question

A client in early labor is connected to an external fetal monitor. The physician hasn't noted any restrictions on her
chart. The client tells the nurse that she needs to go to the bathroom frequently and that her partner can help her.
How should the nurse respond?

You Selected:

"I'll insert a urinary catheter; then you won't need to get out of bed."
Correct response:

"Please press the call button. I'll disconnect you from the monitor so you can get out of bed."

Explanation:

Question 18 See full question

When planning care for a client who has ingested phencyclidine (PCP), the nurse's highest priority should be
meeting the:

You Selected:

client's physical needs.

Correct response:

client's safety needs.

Explanation:

Question 19 See full question

A nurse working in a physicians office observes a physician sneeze into his/her hand as he/she is walking from one
examination room to another. The physician does not wash his/her hands before entering the room to examine the
next client. What is the nurses first priority?

You Selected:

Tell the physician to wash his/her hands before examining the client.

Correct response:

Tell the physician to wash his/her hands before examining the client.

Explanation:

Question 20 See full question

A restraint order is implemented for a client who is restless and combative due to alcohol intoxication. What is the
most appropriate nursing intervention for this client?

You Selected:

Check the extremities for circulation based on hospital protocols.

Correct response:

Check the extremities for circulation based on hospital protocols.

Explanation:

Question 21 See full question

The nurse is teaching the family of a client diagnosed with leukemia about ways to prevent infection. Which
instruction has the most impact?

You Selected:

Bathing the client daily

Correct response:

Maintaining an intact skin integrity

Explanation:

Question 22 See full question

An alert and oriented elderly client is admitted to the hospital for treatment of cellulitis of the left shoulder after an
arthroscopy. Which fall prevention strategy is most appropriate for this client?
You Selected:

Keep all four side rails up at all times.

Correct response:

Use a nightlight in the bathroom.

Explanation:

Question 23 See full question

A 26-year-old is being treated for delirium due to acute alcohol intoxication. The client is restless, does not want to
stay seated, and has a staggering gait. What should the nurse do first?

You Selected:

Provide one-to-one supervision of the client until detoxification treatment can begin.

Correct response:

Provide one-to-one supervision of the client until detoxification treatment can begin.

Explanation:

Question 24 See full question

The nurse from the nursery is bringing a newborn to a mothers room. The nurse took care of the mother yesterday
and knows the mother and baby well. The nurse should implement which action to ensure the safest transition of
the infant to the mother?

You Selected:

Check the crib to determine if there are enough diapers and formula.

Correct response:

Complete the hospital identification procedure with mother and infant.

Explanation:

Question 25 See full question

After the discharge of a client from a surgical unit, the housekeeper brings a blue pill to the nurse. The pill was
found in the sheets when the linens were removed from the clients bed. The nurse reviews the clients medication
administration record, which shows that the client received this medication at 0800. What would be the nurses
priority action?

You Selected:

Tell the housekeeper not to worry if this happens in the future.

Correct response:

Complete an incident form and notify the physician.

Explanation:

Question 26 See full question

When moving a client in bed, the nurse can ensure proper body mechanics by:

You Selected:

lifting the client to the proper position.

Correct response:

standing with her feet apart.


Explanation:

Question 27 See full question

Which instruction should a nurse include in an injury-prevention plan for a pregnant client?

You Selected:

"Position the steering wheel toward your abdomen."

Correct response:

"Take rest periods during the day."

Explanation:

Question 28 See full question

A client admitted with bacterial meningitis must be transported to the radiology department for a repeat computed
tomography scan of the head. His level of consciousness is decreased, and he requires nasopharyngeal suctioning
before transport. Which infection control measures are best when caring for this client?

You Selected:

Take no special precautions for this client.

Correct response:

Put on gloves, a mask, and eye protection during suctioning, and then apply a mask to the client's face for
transport.

Explanation:

Question 29 See full question

A 3-month-old infant with meningococcal meningitis has just been admitted to the pediatric unit. Which nursing
intervention has the highest priority?

You Selected:

instituting droplet precautions

Correct response:

instituting droplet precautions

Explanation:

Question 30 See full question

A nurse should question an order for a heating pad for a client who has:

You Selected:

a reddened abscess.

Correct response:

active bleeding.

Explanation:

Question 31 See full question

The nurse is caring for a client with Clostridium difficile infection. Upon entering the room, which of the following
steps should the nurse take?

You Selected:

Wear a face mask and goggles.


Correct response:

Put on an isolation gown and gloves.

Explanation:

Question 32 See full question

A client has received numerous different antibiotics and now is experiencing diarrhea. The health care provider
(HCP) has prescribed a transmission-based precaution. The nurse should institute:

You Selected:

needlestick precautions.

Correct response:

contact precautions.

Explanation:

Question 33 See full question

The nurse is conducting a routine risk assessment at a prenatal visit. Which question would be the best to screen
for intimate partner violence?

You Selected:

Does your partner have an arrest record?

Correct response:

How safe do you feel in your home?

Explanation:

Question 34 See full question

A 15-year-old adolescent confides in the nurse that he has been contemplating suicide. He says he has developed a
specific plan to carry it out and pleads with the nurse not to tell anyone. What is the nurse's best response?

You Selected:

"We can keep this between you and me, but promise me you won't try anything."

Correct response:

"For your protection, I can't keep this secret. After I notify the physician, we will need to involve your family. We
want you to be safe."

Explanation:

Question 35 See full question

When changing a wet-to-dry dressing covering a surgical wound, what should the nurse do?

You Selected:

Pack the wet dressing tightly into the wound.

Correct response:

Cover the wet packing with a dry sterile dressing.

Explanation:

Question 36 See full question

The nurse is caring for a neonate diagnosed with early onset sepsis and is being treated with intravenous
antibiotics. Which instructions will the nurse include in the parents teaching plan?
You Selected:

Visit but do not touch the neonate.

Correct response:

Wash hands thoroughly before touching the neonate.

Explanation:

Question 37 See full question

A nurse is teaching parents how to reduce the spread of impetigo. The nurse should encourage parents to:

You Selected:

teach children the importance of proper hand washing.

Correct response:

teach children the importance of proper hand washing.

Explanation:

Question 38 See full question

A nurse is caring for a group of toddlers in a large urban hospital. When considering providing care, which clients
require contact precautions? Select all that apply.

You Selected:

A toddler with a multidrug-resistant organism.

Correct response:

A toddler with scabies.


A toddler with a multidrug-resistant organism.

Explanation:

Question 39 See full question

A nurse is leading a group of parents of toddlers in a discussion on home safety. The nurse should emphasize which
fact?

You Selected:

A toddler's risk of injury is the same as that of an adult.

Correct response:

Most toddler deaths are accidental.

Explanation:

Question 40 See full question

A nurse-manager identifies fall prevention as a unit priority. Which of the following actions can the nurses
implement to meet these goals? Select all that apply.

You Selected:

Apply soft waist restraint to confused clients

Correct response:

Use bed alarms to remind clients to call for help getting up


Maintain a clear path to client bathrooms
Make hourly rounds to client rooms
Explanation:

Question 41 See full question

Four clients in a critical care unit have been diagnosed with Psuedomonas aeruginosa. The Infection Prevention and
Control Department has determined that this is probably a nosocomial infection. Select the most appropriate
intervention by the nurse. The nurse should:

You Selected:

initiate transmission-based precautions.

Correct response:

ensure that staff members do not have artificial fingernails.

Explanation:

Question 42 See full question

While caring for the neonate of a human immunodeficiency virus-positive mother, the nurse prepares to administer
a prescribed vitamin K intramuscular injection at 1 hour after birth. Which action should the nurse do first?

You Selected:

Place the neonate under a radiant warmer.

Correct response:

Bathe the neonate.

Explanation:

Question 43 See full question

A physician enters a computer order for a nurse to irrigate a client's nephrostomy tube every 4 hours to maintain
patency. The nurse irrigates the tube using sterile technique. After irrigating the tube, the nurse decides that she
can safely use the same irrigation set for her 8-hour shift if she covers the set with a paper, sterile drape. This
action by the nurse is:

You Selected:

appropriate because the irrigation just checks for patency.

Correct response:

inappropriate because irrigation requires strict sterile technique.

Explanation:

Question 44 See full question

A nurse notices that a client admitted for treatment of major depression is pacing, agitated, and becoming verbally
aggressive toward other clients. What is the immediate care priority?

You Selected:

Removing the other clients from the area until this client settles down

Correct response:

Ensuring the safety of this client and other clients on the unit

Explanation:

Question 45 See full question

When planning home care for a 3-year-old child with eczema, what should the nurse teach the mother to remove
from the child's environment at home?
You Selected:

wooden blocks

Correct response:

stuffed animals

Explanation:

Question 46 See full question

A client arrives on the psychiatric unit exhibiting extreme excitement, disorientation, incoherent speech, agitation,
frantic and aimless physical activity, and grandiose delusion. Which nursing diagnosis takes highest priority for this
client at this time?

You Selected:

Risk for injury

Correct response:

Risk for injury

Explanation:

Question 47 See full question

To ensure safety for a hospitalized blind client, the nurse should:

You Selected:

keep the side rails up when the client is alone.

Correct response:

orient the client to the room environment.

Explanation:

Question 48 See full question

A client develops a facial rash and urticaria after receiving penicillin. Which laboratory value does the nurse expect
to be elevated?

You Selected:

IgB

Correct response:

IgE

Explanation:

Question 49 See full question

During rounds, a nurse finds that a client with hemiplegia has fallen from the bed because the nursing assistant
failed to raise the side rails after giving a back massage. The nurse assists the client to the bed and assesses for
injury. As per agency policies, the nurse fills out an incident report. Which of the following activities should the
nurse perform after finishing the incident report?

You Selected:

Attach a copy to the client's records.

Correct response:

Include the time and date of the incident.


Explanation:

Question 50 See full question

A physician has ordered a heating pad for an elderly client's lower back pain. Which item would be most important
for a nurse to assess before applying the heating pad?

You Selected:

Client's nutritional status

Correct response:

Client's level of consciousness

Explanation:

Question 1 See full question

Which technique is most effective in preventing nosocomial infection transmission when caring for a preschooler?

You Selected:

Hand washing

Correct response:

Hand washing

Explanation:

Question 2 See full question

Which use of restraints in a school-age child should the nurse question?

You Selected:

To substitute for observation

Correct response:

To substitute for observation

Explanation:

Question 3 See full question

Which instruction should a nurse include in a home-safety teaching plan for a pregnant client?

You Selected:

It's OK to clean your cat's litter box.

Correct response:

Place a nonskid mat on the floor of the tub or shower.

Explanation:

Question 4 See full question

Which instruction should a nurse include in an injury-prevention plan for a pregnant client?

You Selected:

"Take rest periods during the day."

Correct response:

"Take rest periods during the day."


Explanation:

Question 5 See full question

Which type of restraint is best for the nurse to use for a child in the immediate postoperative period after cleft
palate repair?

You Selected:

elbow restraints

Correct response:

elbow restraints

Explanation:

Question 6 See full question

When teaching parent workshops about measures to prevent lead poisoning in children, which preventive measure
should the nurse include as the most effective?

You Selected:

condemning of old housing developments

Correct response:

educating the public on common sources of lead

Explanation:

Question 7 See full question

In caring for the client with hepatitis B, which situation would expose the nurse to the virus?

You Selected:

contact with fecal material

Correct response:

a blood splash into the nurses eyes

Explanation:

Question 8 See full question

A school nurse interviews the parent of a middle school student who is exhibiting behavioral problem, including
substance abuse, following a siblings suicide. The parent says I am a single parent who has to work hard to
support my family, and now I have lost my only son and my daughter is acting out and making me crazy! I just
cannot take all this stress! Which concern regarding this family has top priority at this time?

You Selected:

parent's ability to emotionally support the adolescent in this crisis

Correct response:

potential suicidal thoughts/plans of both family members

Explanation:

Question 9 See full question

The nurse manager of a surgical unit observes a nurse providing colostomy care to a client without using any
personal protective equipment (PPE). What is the most appropriate response by the nurse manager in relation to
the use of PPE?
You Selected:

PPE should be used when you risk exposure to blood or bodily fluids.

Correct response:

PPE should be used when you risk exposure to blood or bodily fluids.

Explanation:

Question 10 See full question

A client has an indwelling urinary catheter and is prescribed physical therapy. As the client is being placed in a
wheelchair, which action by the assistant would need further clarification by the nurse?

You Selected:

The catheter bag is placed upon the clients lap for safe transport.

Correct response:

The catheter bag is placed upon the clients lap for safe transport.

Explanation:

Question 1 See full question

A physician has ordered a heating pad for an elderly client's lower back pain. Which item would be most important
for a nurse to assess before applying the heating pad?

You Selected:

Client's vital signs and breath sounds

Correct response:

Client's level of consciousness

Explanation:

Question 2 See full question

Which instruction should a nurse include in an injury-prevention plan for a pregnant client?

You Selected:

"Wear your seat belt across your tummy."

Correct response:

"Take rest periods during the day."

Explanation:

Question 3 See full question

A physician enters a computer order for a nurse to irrigate a client's nephrostomy tube every 4 hours to maintain
patency. The nurse irrigates the tube using sterile technique. After irrigating the tube, the nurse decides that she
can safely use the same irrigation set for her 8-hour shift if she covers the set with a paper, sterile drape. This
action by the nurse is:

You Selected:

appropriate because the irrigation set will be used only during an 8-hour period.

Correct response:

inappropriate because irrigation requires strict sterile technique.


Explanation:

Question 4 See full question

The nurse is reconciling the medications with a client who is being discharged. Which information indicates there is
a "discrepancy"?

You Selected:

There is agreement between the clients home medication list and current medication prescriptions.

Correct response:

There is lack of congruence between a clients home medication list and current medication prescriptions.

Explanation:

Question 5 See full question

A nurse discovers that a hospitalized client with stage 4 esophageal cancer and major depression has a gun in the
home. What is the best nursing intervention to help the client remain safe after discharge?

You Selected:

Have the client promise to use the gun only for home protection.

Correct response:

Talk with the health care provider (HCP) about requiring gun removal as a condition of discharge.

Explanation:

Question 6 See full question

A 26-year-old is being treated for delirium due to acute alcohol intoxication. The client is restless, does not want to
stay seated, and has a staggering gait. What should the nurse do first?

You Selected:

Ask the client to sit in a chair next to the nurses' station.

Correct response:

Provide one-to-one supervision of the client until detoxification treatment can begin.

Explanation:

Question 7 See full question

A nurse notices the smell of marijuana on a nursing colleague upon return from lunch break. The colleague is
having difficulty drawing up a dose of insulin, appears uncoordinated, and is unaware that the needle has been
contaminated. What is the best action for the nurse to take?

You Selected:

Stop the colleague from drawing up the insulin. Notify the supervisor about the incident, and document the
observations.

Correct response:

Stop the colleague from drawing up the insulin. Notify the supervisor about the incident, and document the
observations.

Explanation:

Question 8 See full question

A nurse-manager identifies fall prevention as a unit priority. Which of the following actions can the nurses
implement to meet these goals? Select all that apply.
You Selected:

Use bed alarms to remind clients to call for help getting up


Maintain a clear path to client bathrooms

Correct response:

Use bed alarms to remind clients to call for help getting up


Maintain a clear path to client bathrooms
Make hourly rounds to client rooms

Explanation:

Question 9 See full question

The nurse is teaching the family of a client diagnosed with leukemia about ways to prevent infection. Which
instruction has the most impact?

You Selected:

Ingesting a plant-based diet

Correct response:

Maintaining an intact skin integrity

Explanation:

Question 10 See full question

The nurse is planning care with an older adult who is at risk for falling because of postural hypotension. Which
intervention will be most effective in preventing falls in this client?

You Selected:

Complete a fall diary.

Correct response:

Instruct the client to sit, obtain balance, dangle legs, and rise slowly.

Explanation:

Question 1 See full question

When changing a sterile surgical dressing, a nurse first must:

You Selected:

wash her hands.

Correct response:

wash her hands.

Explanation:

Question 2 See full question

A nurse is orienting a new nurse to the labor and delivery unit. Which action by the new nurse regarding a
neonate's security requires intervention by the preceptor?

You Selected:

Positioning a rooming-in neonate's bassinet toward the center of room rather than near the door to the hallway

Correct response:

Allowing volunteers to return neonates to the nursery


Explanation:

Question 3 See full question

A nurse preparing to discharge a child with leukemia observes a family member who has a cold sharing a meal with
the child. How should the nurse approach the situation?

You Selected:

Tell family members to be careful to avoid the child if they're sick.

Correct response:

Offer a face mask to the person with the cold and use this as an opportunity for further teaching.

Explanation:

Question 4 See full question

When teaching parent workshops about measures to prevent lead poisoning in children, which preventive measure
should the nurse include as the most effective?

You Selected:

educating the public on common sources of lead

Correct response:

educating the public on common sources of lead

Explanation:

Question 5 See full question

A client with suspected severe acute respiratory syndrome (SARS) comes to the emergency department. Which
physician order should the nurse implement first?

You Selected:

Obtain a nasopharyngeal specimen for reverse-transcription polymerase chain reaction testing.

Correct response:

Institute isolation precautions.

Explanation:

Question 6 See full question

A client has been placed in an isolation room, and family members have stated that access to the client seems
restricted. Which of the following actions would be appropriate for the nurse to take to address this situation? Select
all that apply.

You Selected:

A communication plan for the family and client

Correct response:

A communication plan for the family and client


A thorough explanation of the isolation procedures
Acknowledgement of the family's concerns

Explanation:

Question 7 See full question

While hospitalized, a child develops a Clostridium difficile infection. The nurse can anticipate adding which of the
following types of precautions for this client?
You Selected:

Droplet precautions

Correct response:

Contact precautions

Explanation:

Question 8 See full question

A client on a mental health unit becomes increasing agitated and barricades himself in a corner room holding
another client hostage. Verbal exchanges indicate an escalation in client desperation. Which nursing actions would
be taken at this time? Select all that apply.

You Selected:

Speak to the client in an authoritarian manner.


Discretely notify security to assist.

Correct response:

Identify one nurse to interact with the client.


Direct other clients away from the area.
Discretely notify security to assist.
Identify with the clients position and reason for agitation.

Explanation:

Question 9 See full question

The nurse notices that a cart being used to transport a client has a nonfunctioning clasp on the safety belt. The
nurse should:

You Selected:

use a draw sheet to secure the client during transport.

Correct response:

request that the transporter bring a different cart with a functional clasp.

Explanation:

Question 10 See full question

The nurse is teaching the family of a client diagnosed with leukemia about ways to prevent infection. Which
instruction has the most impact?

You Selected:

Maintaining an intact skin integrity

Correct response:

Maintaining an intact skin integrity

Explanation:

Class average
View performance for all Client Needs

Answer Key

Question 1 See full question

A nurse is performing a sterile dressing change. Which action contaminates the sterile field?
You Selected:

Pouring solution onto a sterile field cloth

Correct response:

Pouring solution onto a sterile field cloth

Explanation:

Question 2 See full question

Which item in the care plan for a toddler with a seizure disorder should a nurse revise?

You Selected:

Padded side rails

Correct response:

Arm restraints while asleep

Explanation:

Question 3 See full question

A nurse is supervising a new nurse who is preparing to perform wound care for a client whose abdominal wound is
infected with vancomycin-resistant enterococci. The supervising nurse should make sure that the new nurse:

You Selected:

assembles all wound care supplies before entering the client's room.

Correct response:

wears a gown, gloves, a mask, and eye protection when entering the client's room.

Explanation:

Question 4 See full question

A nurse is planning care for an elderly client with cognitive impairment who is still living at home. Which action
should the nurse identify as a priority for safety in planning care for this client?

You Selected:

having two people accompany the client whenever the client is up and about

Correct response:

ensuring the removal of objects in the client's path that may cause him to trip

Explanation:

Question 5 See full question

A client who was bitten by a wild animal is admitted to an acute care facility for treatment of rabies. Which type of
isolation does this client require?

You Selected:

Strict

Correct response:

Contact
Explanation:

Question 6 See full question

The nurse manager of a surgical unit observes a nurse providing colostomy care to a client without using any
personal protective equipment (PPE). What is the most appropriate response by the nurse manager in relation to
the use of PPE?

You Selected:

If youre not using PPE, you need to be careful not to touch any of the drainage.

Correct response:

PPE should be used when you risk exposure to blood or bodily fluids.

Explanation:

Question 7 See full question

The nurse is performing a surgical dressing change and drops a sterile gauze on the bedside table outside of the
sterile dressing trays field. What would be the most appropriate action of the nurse?

You Selected:

Turn the gauze inside out so that the contaminated part is not next to the wound.

Correct response:

Discard the gauze and use another sterile piece.

Explanation:

Question 8 See full question

A client with hepatitis B is visiting with her sister when the client's IV catheter dislodges and bleeds onto the surface
of the bedside table. Which of the following, if observed, would cause the nurse to intervene?

You Selected:

The nursing assistant uses tissue to blot up the blood.

Correct response:

The nursing assistant uses tissue to blot up the blood.

Explanation:

Question 9 See full question

A nurse recognizes that a client with tuberculosis needs further teaching when the client states:

You Selected:

"It will be necessary for the people I work with to take medication."

Correct response:

"It will be necessary for the people I work with to take medication."

Explanation:

Question 10 See full question

A client returns to the nursing unit, after being discharged, demanding acetaminophen with codeine. The client is
advised that he/she is no longer being treated on the unit and this medication cannot be administered. The client
states, I know where you park your cars, and youd better watch out when you leave here tonight. What is the
next appropriate step that the nurse should take?
You Selected:

Bring the client to the emergency department immediately.

Correct response:

Call the police.

Explanation:

Question 1 See full question

A nurse is assigned to a client with a cardiac disorder. The nurse should question an order to monitor the client's
body temperature by which route?

You Selected:

Rectal

Correct response:

Rectal

Explanation:

Question 2 See full question

A physician has ordered a heating pad for an elderly client's lower back pain. Which item would be most important
for a nurse to assess before applying the heating pad?

You Selected:

Client's vital signs and breath sounds

Correct response:

Client's level of consciousness

Explanation:

Question 3 See full question

One evening the client takes the nurse aside and whispers, Do not tell anybody, but I am going to call in a bomb
threat to this hospital tonight. Which action is the priority?

You Selected:

notifying the proper authorities after saying nothing until the client has actually completed the call

Correct response:

explaining to the client that this information will have to be shared immediately with the staff and the health
care provider (HCP)

Explanation:

Question 4 See full question

A client was brought to the unit and admitted involuntarily. During visiting the next day, the clients brother
demands that the client be released immediately. The brother says he might have to hurt staff if the unit door is not
opened. In which order of priority from first to last should the nursing actions be implemented? All options must be
used.

You Selected:

Quietly ask the other clients and visitors to move to another area of the unit with a staff member.
Call security officers to the unit for the protection of all on the unit.
Ask the client's brother to leave the unit quietly when he repeats his demands.
Calmly restate to the client and his brother that the client cannot be released without a health care providers
(HCPs) prescription.
Correct response:

Calmly restate to the client and his brother that the client cannot be released without a health care providers
(HCPs) prescription.
Ask the client's brother to leave the unit quietly when he repeats his demands.
Quietly ask the other clients and visitors to move to another area of the unit with a staff member.
Call security officers to the unit for the protection of all on the unit.

Explanation:

Question 5 See full question

While making rounds, the nurse enters a clients room and finds the client on the floor between the bed and the
bathroom. The nurse should first:

You Selected:

ask what the client was doing out of bed.

Correct response:

assess the clients current condition and vital signs.

Explanation:

Question 6 See full question

When the client is involuntarily committed to a hospital because he is assessed as being dangerous to himself or
others, which client rights are lost?

You Selected:

the right to send and receive uncensored mail

Correct response:

the right to leave the hospital against medical advice

Explanation:

Question 7 See full question

A client returns to the nursing unit, after being discharged, demanding acetaminophen with codeine. The client is
advised that he/she is no longer being treated on the unit and this medication cannot be administered. The client
states, I know where you park your cars, and youd better watch out when you leave here tonight. What is the
next appropriate step that the nurse should take?

You Selected:

Bring the client to the emergency department immediately.

Correct response:

Call the police.

Explanation:

Question 8 See full question

A nurse-manager identifies fall prevention as a unit priority. Which of the following actions can the nurses
implement to meet these goals? Select all that apply.

You Selected:

Close doors to client rooms at night

Correct response:

Use bed alarms to remind clients to call for help getting up


Maintain a clear path to client bathrooms
Make hourly rounds to client rooms

Explanation:

Question 9 See full question

A client had a liver biopsy 1 hour ago. The nurse should first:

You Selected:

apply packing to the biopsy site.

Correct response:

auscultate lung sounds.

Explanation:

Question 10 See full question

A client has an indwelling urinary catheter and is prescribed physical therapy. As the client is being placed in a
wheelchair, which action by the assistant would need further clarification by the nurse?

You Selected:

The assistant brings a container to drain the urine from the bag.

Correct response:

The catheter bag is placed upon the clients lap for safe transport.

Explanation:

Question 1 See full question

What should a nurse do to ensure a safe hospital environment for a toddler?

You Selected:

Move the equipment out of reach.

Correct response:

Move the equipment out of reach.

Explanation:

Question 2 See full question

A child, age 3, who tests positive for the human immunodeficiency virus (HIV) is placed in foster care. The foster
parents ask the nurse how to prevent HIV transmission to other family members. How should the nurse respond?

You Selected:

"Wear gloves when you're likely to come into contact with the child's blood or body fluids."

Correct response:

"Wear gloves when you're likely to come into contact with the child's blood or body fluids."

Explanation:

Question 3 See full question

The nurse should use which type of precautions for a client being admitted to the hospital with suspected
tuberculosis?

You Selected:

droplet precautions
Correct response:

airborne precautions

Explanation:

Question 4 See full question

Which client is at highest risk for developing a hospital-acquired infection?

You Selected:

A client with Crohn's disease

Correct response:

A client with an indwelling urinary catheter

Explanation:

Question 5 See full question

When creating an educational program about safety, what information should the nurse include about sexual
predators? Select all that apply.

You Selected:

Child molesters gain the childs trust before making sexual advances so the child feels obligated to comply
with sex.
Child molesters often choose children whose parents must work long hours, making the extra attention initially
welcomed by the child.
Child molesters pick children or teens over which they have some authority, making it easier for them to
manipulate the child with special favors or attention.

Correct response:

Child molesters pick children or teens over which they have some authority, making it easier for them to
manipulate the child with special favors or attention.
Child molesters gain the childs trust before making sexual advances so the child feels obligated to comply
with sex.
Child molesters often choose children whose parents must work long hours, making the extra attention initially
welcomed by the child.
Child molesters maintain the secrecy of their actions by making threats if offering attention and favors fail or if
the child is close to revealing the secret.

Explanation:

Question 6 See full question

The nurse is caring for a client who is confused about time and place. The client has intravenous fluid infusing. The
nurse attempts to reorient the client, but the client remains unable to demonstrate appropriate use of the call light.
In order to maintain client safety, what should the nurse do first?

You Selected:

Ask family members to stay with the client.

Correct response:

Increase the frequency of client observation.

Explanation:

Question 7 See full question

A nurse is scheduled to perform an initial home visit to a new client who is beginning home intravenous therapy. As
the nurse is getting out of her car and beginning to approach the clients building, a group of men begin following
and jeering at her. Which of the following is the nurses best response to this situation?
You Selected:

Call out to attract attention from bystanders.

Correct response:

Leave the area in her car, provided she can get to it safely.

Explanation:

Question 8 See full question

A school nurse is conducting a seminar for parents of preschool children on the prevention of head injuries. What is
the most appropriate information for the nurse to give the parents?

You Selected:

Children should not do any athletic events until they are older.

Correct response:

Children should always wear helmets when riding bicycles.

Explanation:

Question 9 See full question

A nurse notices the smell of marijuana on a nursing colleague upon return from lunch break. The colleague is
having difficulty drawing up a dose of insulin, appears uncoordinated, and is unaware that the needle has been
contaminated. What is the best action for the nurse to take?

You Selected:

Stop the colleague from drawing up the insulin. Notify the supervisor about the incident, and document the
observations.

Correct response:

Stop the colleague from drawing up the insulin. Notify the supervisor about the incident, and document the
observations.

Explanation:

Question 10 See full question

A client is admitted with an infectious wound. Contact precautions are initiated. To help the client cope with staff
using isolation procedures, which nursing action is most helpful?

You Selected:

Put stickers on the face mask to increase conversation

Correct response:

Discuss the rationale for contact precautions

Explanation:

Question 1 See full question

A physician orders chest physiotherapy for a client with pulmonary congestion. When should the nurse plan to
perform chest physiotherapy?

You Selected:

When the client has time

Correct response:

Before meals
Explanation:

Question 2 See full question

When changing a sterile surgical dressing, a nurse first must:

You Selected:

wash her hands.

Correct response:

wash her hands.

Explanation:

Question 3 See full question

A nurse implements a health care facility's disaster plan. Which action should she perform first?

You Selected:

Identify a command center at which activities are coordinated.

Correct response:

Identify a command center at which activities are coordinated.

Explanation:

Question 4 See full question

When checking a client's medication profile, a nurse notes that the client is receiving a drug contraindicated for
clients with glaucoma. The nurse knows that this client, who has a history of glaucoma, has been taking the
medication for the past 3 days. What should the nurse do first?

You Selected:

Find out whether there are extenuating reasons for giving the drug to this client.

Correct response:

Hold the medication and report the information to the physician to ensure client safety.

Explanation:

Question 5 See full question

When administering an I.M. injection, which action puts the nurse at risk for a needle-stick injury?

You Selected:

Planning safe handling and disposal of needles before initiating a procedure

Correct response:

Using the one-handed needle-recapping technique after administering all injections

Explanation:

Question 6 See full question

When developing the teaching plan for a client who uses a walker, which principle should a nurse consider?

You Selected:

If one leg is weaker than the other, the walker and the stronger leg should move, together, approximately 6
ahead of the body. The client's weight is supported by his weaker leg.
Correct response:

When maximum support is required, the walker should be moved ahead approximately 6 (15 cm) while both
legs support the client's weight.

Explanation:

Question 7 See full question

A 3-month-old infant with meningococcal meningitis has just been admitted to the pediatric unit. Which nursing
intervention has the highest priority?

You Selected:

obtaining history information from the parents

Correct response:

instituting droplet precautions

Explanation:

Question 8 See full question

A nurse is teaching parents about accident prevention for a toddler. Which guideline is most appropriate?

You Selected:

Never leave a toddler unattended on a bed.

Correct response:

Make sure all medications are kept in containers with childproof safety caps.

Explanation:

Question 9 See full question

What should a nurse do to ensure a safe hospital environment for a toddler?

You Selected:

Place the child in a youth bed.

Correct response:

Move the equipment out of reach.

Explanation:

Question 10 See full question

A child, age 3, who tests positive for the human immunodeficiency virus (HIV) is placed in foster care. The foster
parents ask the nurse how to prevent HIV transmission to other family members. How should the nurse respond?

You Selected:

"Make sure the child uses disposable plates and utensils."

Correct response:

"Wear gloves when you're likely to come into contact with the child's blood or body fluids."

Explanation:

Question 11 See full question

In the emergency department, a client with facial lacerations states that her husband beat her with a shoe. After
her lacerations are repaired, the client waits to be seen by the crisis intake nurse, who will evaluate the continued
threat of violence her husband represents. Suddenly the client's husband arrives, shouting that he wants to "finish
the job." What is the first priority of the nurse who witnesses this scene?
You Selected:

Telling the client's husband that he must leave at once

Correct response:

Calling a security guard and another staff member for assistance

Explanation:

Question 12 See full question

A nurse must restrain a client to ensure the safety of other clients. When using restraints, which principle is a
priority?

You Selected:

Secure restraints to the bed with knots to prevent the client from escaping.

Correct response:

Use an organized, efficient team approach to apply and secure the restraints.

Explanation:

Question 13 See full question

Which instruction should a nurse include in a home-safety teaching plan for a pregnant client?

You Selected:

Place a nonskid mat on the floor of the tub or shower.

Correct response:

Place a nonskid mat on the floor of the tub or shower.

Explanation:

Question 14 See full question

Which point should a nurse include when teaching mothers about preventing childhood falls?

You Selected:

Use small pillows in the crib to prevent the infant from rolling onto the stomach.

Correct response:

Never leave the infant alone on an elevated surface.

Explanation:

Question 15 See full question

A client with chronic progressive multiple sclerosis is learning to use a walker. What instruction will best ensure the
client's safety?

You Selected:

"Use a walker with wheels to help you move forward."

Correct response:

"Place the walker directly in front of you and step into it as you move it forward."
Explanation:

Question 16 See full question

While reviewing the day's charts, a nurse who's been under a great deal of personal stress realizes that she forgot
to administer insulin to client with diabetes mellitus. She's made numerous errors in the past few weeks and is now
afraid her job is in jeopardy. What is her best course of action?

You Selected:

Report the error, complete the proper paperwork, and meet with the unit manager.

Correct response:

Report the error, complete the proper paperwork, and meet with the unit manager.

Explanation:

Question 17 See full question

Whichis an initial clinical manifestation of gonorrhea in men?

You Selected:

urethral discharge

Correct response:

urethral discharge

Explanation:

Question 18 See full question

The mother of an 8-year-old with diabetes tells the nurse that she does not want the school to know about her
daughter's condition. The nurse should reply:

You Selected:

"In order to keep your daughter safe, it is necessary for all adults in the school to know her condition."

Correct response:

"What is it that concerns you about having the school know about your daughter's condition?"

Explanation:

Question 19 See full question

An alert and oriented elderly client is admitted to the hospital for treatment of cellulitis of the left shoulder after an
arthroscopy. Which fall prevention strategy is most appropriate for this client?

You Selected:

Use a nightlight in the bathroom.

Correct response:

Use a nightlight in the bathroom.

Explanation:

Question 20 See full question

When changing a wet-to-dry dressing covering a surgical wound, what should the nurse do?

You Selected:

Pack the wet dressing tightly into the wound.


Correct response:

Cover the wet packing with a dry sterile dressing.

Explanation:

Question 21 See full question

The nurse should use which type of precautions for a client being admitted to the hospital with suspected
tuberculosis?

You Selected:

contact precautions

Correct response:

airborne precautions

Explanation:

Question 22 See full question

A client had a repair of a thoracoabdominal aneurysm 2 days ago. Which finding should the nurse consider
unexpected and report to the health care provider (HCP) immediately?

You Selected:

adominal pain at 5 on a scale of 0 to 10 for the last 2 days

Correct response:

weakness and numbness in the lower extremities

Explanation:

Question 23 See full question

After teaching the parents of a 15-month-old child who has undergone cleft palate repair how to use elbow
restraints, which statement by the parents indicates effective teaching?

You Selected:

We will remove the restraints temporarily at least three times a day to check his skin, then put them right
back on.

Correct response:

We will remove the restraints temporarily at least three times a day to check his skin, then put them right
back on.

Explanation:

Question 24 See full question

In caring for the client with hepatitis B, which situation would expose the nurse to the virus?

You Selected:

a blood splash into the nurses eyes

Correct response:

a blood splash into the nurses eyes


Explanation:

Question 25 See full question

The nurse develops a health education program about preventing the transmission of hepatitis B. The nurse
evaluates that the teaching has been effective when the participants identify which activities to be high risk for
acquiring hepatitis B?

You Selected:

ingestion of large amounts of acetaminophen

Correct response:

sharing needles for drug use

Explanation:

Question 26 See full question

The nurse is instructing the unlicensed assistive personnel (UAP) how to care for a client who is receiving
chemotherapy. What self-care precautions should the nurse tell the UAP to take when caring for the client?

You Selected:

Gowns, mask, and gloves are required for any contact with the client.

Correct response:

Wear a double set of gloves when handling clients fluids.

Explanation:

Question 27 See full question

A client with viral hepatitis A is being treated in an acute care facility. Because the client requires enteric
precautions, the nurse should:

You Selected:

wash her hands after touching the client.

Correct response:

wash her hands after touching the client.

Explanation:

Question 28 See full question

A nurse is giving discharge teaching to a client with an eye injury. Which statement about preventing eye injuries
should the nurse include?

You Selected:

"Use a lighter or matches if you need to look at a car battery at night."

Correct response:

"Direct all spray nozzles away from your face before spraying."

Explanation:

Question 29 See full question

A 6-month-old on the pediatric floor has a respiratory rate of 68, mild intercostal retractions, and an oxygen
saturation of 89%. The infant has not been feeding well for the last 24 hours and is restless. Using the SBAR
(Situation-Background-Assessment-Recommendation) technique for communication, the nurse calls the health care
provider (HCP) with the recommendation for:
You Selected:

transferring the child to pediatric intensive care.

Correct response:

starting oxygen.

Explanation:

Question 30 See full question

An alert and oriented older adult female with metastatic lung cancer is admitted to the medical-surgical unit for
treatment of heart failure. She was given 80 mg of furosemide in the emergency department. Although the client is
ambulatory, the unlicensed assistive personnel (UAP) are concerned about urinary incontinence because the client
is frail and in a strange environment. The nurse should instruct the UAP to assist with implementing the nursing
plan of care by:

You Selected:

requesting an indwelling urinary catheter to avoid incontinence

Correct response:

placing a commode at the bedside and instructing the client in its use

Explanation:

Question 31 See full question

A 26-year-old is being treated for delirium due to acute alcohol intoxication. The client is restless, does not want to
stay seated, and has a staggering gait. What should the nurse do first?

You Selected:

Provide one-to-one supervision of the client until detoxification treatment can begin.

Correct response:

Provide one-to-one supervision of the client until detoxification treatment can begin.

Explanation:

Question 32 See full question

A nurse on a night shift entered an elderly clients room during a scheduled check and discovered the client on the
floor beside her bed after falling when trying to ambulate to the washroom. After assessing and assisting the client
back to bed, the nurse has completed an incident report. What is the primary purpose of this particular type of
documentation?

You Selected:

Protecting the nurse and the hospital from litigation.

Correct response:

Identifying risks and ensuring future safety for clients.

Explanation:

Question 33 See full question

A 13-year-old is having surgery to repair a fractured left femur. As a part of the preoperative safety checklist, what
should the nurse do?

You Selected:

Ask the teen to point to the surgery site.


Correct response:

Verify that the site, side, and level are marked.

Explanation:

Question 34 See full question

A nurse working in a physicians office observes a physician sneeze into his/her hand as he/she is walking from one
examination room to another. The physician does not wash his/her hands before entering the room to examine the
next client. What is the nurses first priority?

You Selected:

Tell the physician to wash his/her hands before examining the client.

Correct response:

Tell the physician to wash his/her hands before examining the client.

Explanation:

Question 35 See full question

A nurse practitioner (NP) orders an antibiotic to which the client is allergic. The nurse preparing the medication
notices the allergy alert and contacts the NP by phone. The NP does not return the call and the first dose is due to
be given. Which of the following actions by the nurse is the best solution to this situation?

You Selected:

Give the medication as ordered by the NP.

Correct response:

Hold the medication until speaking with the NP.

Explanation:

Question 36 See full question

A nurse is caring for a confused client and develops a plan of care based on a least restraint policy. Which of the
following interventions would be most appropriate for the nurse to implement based on this policy?

You Selected:

Chair with locking lap tray

Correct response:

Alarm-activating wrist bracelet

Explanation:

Question 37 See full question

A nurse administers digoxin 0.125 mg to a client at 1400 instead of the prescribed dose of digoxin 0.25 mg. Which
of the following statements should the nurse record in the medical record?

You Selected:

Digoxin 0.125 mg given at 1400 instead of prescribed dose of 0.25 mg.

Correct response:

Digoxin 0.125 mg given at 1400 instead of prescribed dose of 0.25 mg.


Explanation:

Question 38 See full question

A restraint order is implemented for a client who is restless and combative due to alcohol intoxication. What is the
most appropriate nursing intervention for this client?

You Selected:

Secure the restraints to the bed with knots to ensure the client cannot undo them.

Correct response:

Check the extremities for circulation based on hospital protocols.

Explanation:

Question 39 See full question

While hospitalized, a child develops a Clostridium difficile infection. The nurse can anticipate adding which of the
following types of precautions for this client?

You Selected:

Droplet precautions

Correct response:

Contact precautions

Explanation:

Question 40 See full question

A nurse is caring for a client with acquired immunodeficiency syndrome (AIDS). To adhere to standard precautions,
the nurse should wear gloves when:

You Selected:

delivering the client's food tray.

Correct response:

providing mouth care.

Explanation:

Question 41 See full question

A nurse is supervising a student during medication administration to a client. Which of the following action by the
student would cause the nurse to intervene during the med pass at the bedside?

You Selected:

First asks the client's name.

Correct response:

Check the room number and the client's name on the bed.

Explanation:

Question 42 See full question

Which of the following clients will the nurse place in reverse isolation? Select all that apply.

You Selected:

Client with a white blood cell count (WBC) of 600 L with a granulocyte count of 100 L
Client with human immunodeficiency virus (HIV)
Correct response:

Client with a white blood cell count (WBC) of 600 L with a granulocyte count of 100 L
Client with a burn injury involving > 30% of the total body surface area (TBSA)

Explanation:

Question 43 See full question

A nurse is caring for an infant who requires intravenous therapy. The nurse notes that the only available IV pump is
in a toddlers room. In which order should the nurse complete the following actions?

1. Remove pump from toddler's room


2. Clean the pump
3. Take pump into infants room
4. Use the pump

You Selected:

2, 1, 3, 4

Correct response:

1, 2, 3, 4

Explanation:

Question 44 See full question

A charge nurse is completing client assignments for the nursing staff on the pediatric unit. Which client would the
nurse refrain from assigning to a pregnant staff member?

You Selected:

A 3-month-old with Roseola.

Correct response:

An 8-year-old with Rubella.

Explanation:

Question 45 See full question

A client has anemia resulting from bleeding from ulcerative colitis and is to receive two units of packed red blood
cells (PRBCs). The client is receiving an infusion of total parenteral nutrition (TPN). In preparing to administer the
PRBCs, what should the nurse do to ensure client comfort and safety?

You Selected:

Administer PRBCs in the same IV as the TPN.

Correct response:

Start an IV infusion of normal saline.

Explanation:

Question 46 See full question

The nurse is caring for a client recently diagnosed with hepatitis C. In reviewing the clients history, what
information will be most helpful as the nurse develops a teaching plan? The client:

You Selected:

is a scientist and is frequently exposed to multiple chemicals.

Correct response:

has a known history of sexually transmitted disease.


Explanation:

Question 47 See full question

A client has an indwelling urinary catheter and is prescribed physical therapy. As the client is being placed in a
wheelchair, which action by the assistant would need further clarification by the nurse?

You Selected:

The assistant brings a container to drain the urine from the bag.

Correct response:

The catheter bag is placed upon the clients lap for safe transport.

Explanation:

Question 48 See full question

The nurse is conducting a routine risk assessment at a prenatal visit. Which question would be the best to screen
for intimate partner violence?

You Selected:

How safe do you feel in your home?

Correct response:

How safe do you feel in your home?

Explanation:

Question 49 See full question

The nurse is planning care with an older adult who is at risk for falling because of postural hypotension. Which
intervention will be most effective in preventing falls in this client?

You Selected:

Attach a sensor to the client that will alarm when client attempts to get up.

Correct response:

Instruct the client to sit, obtain balance, dangle legs, and rise slowly.

Explanation:

Question 50 See full question

The nurse is conducting a routine risk assessment at a prenatal visit. Which question would be the best to screen
for intimate partner violence?

You Selected:

How safe do you feel in your home?

Correct response:

How safe do you feel in your home?

Explanation:
Question 1 See full question

A client had a repair of a thoracoabdominal aneurysm 2 days ago. Which finding should the nurse consider
unexpected and report to the health care provider (HCP) immediately?

You Selected:

adominal pain at 5 on a scale of 0 to 10 for the last 2 days

Correct response:

weakness and numbness in the lower extremities

Explanation:

Question 2 See full question

A client found sitting on the floor of the bathroom in the day treatment clinic has moderate lacerations on both
wrists. Surrounded by broken glass, she sits staring blankly at the lacerations. What is the most important action for
the nurse to take next to the client?

You Selected:

Approach the client slowly while speaking in a calm voice, calling her by her name, and telling her that the nurse is
there to help her.

Correct response:

Approach the client slowly while speaking in a calm voice, calling her by her name, and telling her that the nurse is
there to help her.

Explanation:

Question 3 See full question

Which of the following objects poses the most serious safety threat to a 2-year-old child in the hospital?

You Selected:

Crayons and paper

Correct response:

Side rails in the halfway position

Explanation:

Question 4 See full question


When developing the teaching plan for the mother of a 2-year-old child diagnosed with scabies, what information
should the nurse expect to include?

You Selected:

Itching should cease in a few days.

Correct response:

The entire family should be treated.

Explanation:

Question 5 See full question

Which prescription is entered correctly on the medical record?

You Selected:

60.0 mg ketorolac tromethamine given IM for c/o pain

Correct response:

fentanyl 50 mcg given IV every 2 hours as needed for pain greater than 6/10

Explanation:

Question 6 See full question

A charge nurse is completing client assignments for the nursing staff on the pediatric unit. Which client would the
nurse refrain from assigning to a pregnant staff member?

You Selected:

A 2-year-old with Kawasakis disease.

Correct response:

An 8-year-old with Rubella.

Explanation:

Question 7 See full question

A nurse is preparing to administer an I.V. containing dextrose 10% in normal saline solution to a 6-month-old
infant. The nurse should select which tubing to safely administer the solution?

You Selected:

I.V. tubing with a volume-control chamber


Correct response:

I.V. tubing with a volume-control chamber

Explanation:

Question 8 See full question

A nursing instructor is instructing group of new nursing students. The instructor reviews that surgical asepsis will be
used for which of the following procedures?

You Selected:

I.V. catheter insertion

Correct response:

I.V. catheter insertion

Explanation:

Question 9 See full question

The nurse assists the client to the operating room table and supervises the operating room technician preparing the
sterile field. Which action, completed by the surgical technician, indicates to the nurse that a sterile field has been
contaminated?

You Selected:

Wetness in the sterile cloth on top of the nonsterile table has been noted.

Correct response:

Wetness in the sterile cloth on top of the nonsterile table has been noted.

Explanation:

Question 10 See full question

The nurse is inserting a nasogastric tube in an infant to administer feedings. In the accompanying figure, indicate
the location for the correct placement of the distal end of the tube.

You Selected:

Your selection and the correct area, market by the green box.

Explanation:

Question 11 See full question

A client refuses his evening dose of haloperidol and then becomes extremely agitated in the day room while other
clients are watching television. He begins cursing and throwing furniture. The nurse's first action is to:

You Selected:

call the physician and report the behavior.

Correct response:

remove all other clients from the day room.

Explanation:

Question 12 See full question

A nurse is documenting a variance that has occurred during the shift, and this report will be used for quality
improvement to identify high-risk patterns and potentially initiate in-service programs. This is an example of which
type of report?

You Selected:

Incident report.

Correct response:

Incident report.

Explanation:

Question 13 See full question


As a client is being released from restraints, he says, "I'll never get that angry and lose it again. Those restraints
were the worst things that ever happened to me." Which response by the nurse is most appropriate?

You Selected:

"Someday this experience won't bother you like it does now."

Correct response:

"I'd like to talk with you about your experience."

Explanation:

Question 14 See full question

A nurse is preparing to perform a postpartum assessment on a client who gave birth 5 hours ago. Which precaution
should the nurse plan to take for this procedure?

You Selected:

Washing the hands and wearing latex gloves and a barrier gown

Correct response:

Washing the hands and wearing latex gloves

Explanation:

Question 15 See full question

Which instruction should a nurse include in a home-safety teaching plan for a pregnant client?

You Selected:

It's OK to clean your cat's litter box.

Correct response:

Place a nonskid mat on the floor of the tub or shower.

Explanation:

Question 16 See full question

While making rounds, the nurse enters a clients room and finds the client on the floor between the bed and the
bathroom. The nurse should first:

You Selected:

assist the client back to bed.


Correct response:

assess the clients current condition and vital signs.

Explanation:

Question 17 See full question

A client in early labor is connected to an external fetal monitor. The physician hasn't noted any restrictions on her
chart. The client tells the nurse that she needs to go to the bathroom frequently and that her partner can help her.
How should the nurse respond?

You Selected:

"I'll insert a urinary catheter; then you won't need to get out of bed."

Correct response:

"Please press the call button. I'll disconnect you from the monitor so you can get out of bed."

Explanation:

Question 18 See full question

When planning care for a client who has ingested phencyclidine (PCP), the nurse's highest priority should be
meeting the:

You Selected:

client's physical needs.

Correct response:

client's safety needs.

Explanation:

Question 19 See full question

A nurse working in a physicians office observes a physician sneeze into his/her hand as he/she is walking from one
examination room to another. The physician does not wash his/her hands before entering the room to examine the
next client. What is the nurses first priority?

You Selected:

Tell the physician to wash his/her hands before examining the client.

Correct response:

Tell the physician to wash his/her hands before examining the client.
Explanation:

Question 20 See full question

A restraint order is implemented for a client who is restless and combative due to alcohol intoxication. What is the
most appropriate nursing intervention for this client?

You Selected:

Check the extremities for circulation based on hospital protocols.

Correct response:

Check the extremities for circulation based on hospital protocols.

Explanation:

Question 21 See full question

The nurse is teaching the family of a client diagnosed with leukemia about ways to prevent infection. Which
instruction has the most impact?

You Selected:

Bathing the client daily

Correct response:

Maintaining an intact skin integrity

Explanation:

Question 22 See full question

An alert and oriented elderly client is admitted to the hospital for treatment of cellulitis of the left shoulder after an
arthroscopy. Which fall prevention strategy is most appropriate for this client?

You Selected:

Keep all four side rails up at all times.

Correct response:

Use a nightlight in the bathroom.

Explanation:

Question 23 See full question

A 26-year-old is being treated for delirium due to acute alcohol intoxication. The client is restless, does not want to
stay seated, and has a staggering gait. What should the nurse do first?
You Selected:

Provide one-to-one supervision of the client until detoxification treatment can begin.

Correct response:

Provide one-to-one supervision of the client until detoxification treatment can begin.

Explanation:

Question 24 See full question

The nurse from the nursery is bringing a newborn to a mothers room. The nurse took care of the mother yesterday
and knows the mother and baby well. The nurse should implement which action to ensure the safest transition of
the infant to the mother?

You Selected:

Check the crib to determine if there are enough diapers and formula.

Correct response:

Complete the hospital identification procedure with mother and infant.

Explanation:

Question 25 See full question

After the discharge of a client from a surgical unit, the housekeeper brings a blue pill to the nurse. The pill was
found in the sheets when the linens were removed from the clients bed. The nurse reviews the clients medication
administration record, which shows that the client received this medication at 0800. What would be the nurses
priority action?

You Selected:

Tell the housekeeper not to worry if this happens in the future.

Correct response:

Complete an incident form and notify the physician.

Explanation:

Question 26 See full question

When moving a client in bed, the nurse can ensure proper body mechanics by:

You Selected:

lifting the client to the proper position.


Correct response:

standing with her feet apart.

Explanation:

Question 27 See full question

Which instruction should a nurse include in an injury-prevention plan for a pregnant client?

You Selected:

"Position the steering wheel toward your abdomen."

Correct response:

"Take rest periods during the day."

Explanation:

Question 28 See full question

A client admitted with bacterial meningitis must be transported to the radiology department for a repeat computed
tomography scan of the head. His level of consciousness is decreased, and he requires nasopharyngeal suctioning
before transport. Which infection control measures are best when caring for this client?

You Selected:

Take no special precautions for this client.

Correct response:

Put on gloves, a mask, and eye protection during suctioning, and then apply a mask to the client's face for
transport.

Explanation:

Question 29 See full question

A 3-month-old infant with meningococcal meningitis has just been admitted to the pediatric unit. Which nursing
intervention has the highest priority?

You Selected:

instituting droplet precautions

Correct response:

instituting droplet precautions

Explanation:
Question 30 See full question

A nurse should question an order for a heating pad for a client who has:

You Selected:

a reddened abscess.

Correct response:

active bleeding.

Explanation:

Question 31 See full question

The nurse is caring for a client with Clostridium difficile infection. Upon entering the room, which of the following
steps should the nurse take?

You Selected:

Wear a face mask and goggles.

Correct response:

Put on an isolation gown and gloves.

Explanation:

Question 32 See full question

A client has received numerous different antibiotics and now is experiencing diarrhea. The health care provider
(HCP) has prescribed a transmission-based precaution. The nurse should institute:

You Selected:

needlestick precautions.

Correct response:

contact precautions.

Explanation:

Question 33 See full question

The nurse is conducting a routine risk assessment at a prenatal visit. Which question would be the best to screen
for intimate partner violence?

You Selected:
Does your partner have an arrest record?

Correct response:

How safe do you feel in your home?

Explanation:

Question 34 See full question

A 15-year-old adolescent confides in the nurse that he has been contemplating suicide. He says he has developed a
specific plan to carry it out and pleads with the nurse not to tell anyone. What is the nurse's best response?

You Selected:

"We can keep this between you and me, but promise me you won't try anything."

Correct response:

"For your protection, I can't keep this secret. After I notify the physician, we will need to involve your family. We
want you to be safe."

Explanation:

Question 35 See full question

When changing a wet-to-dry dressing covering a surgical wound, what should the nurse do?

You Selected:

Pack the wet dressing tightly into the wound.

Correct response:

Cover the wet packing with a dry sterile dressing.

Explanation:

Question 36 See full question

The nurse is caring for a neonate diagnosed with early onset sepsis and is being treated with intravenous
antibiotics. Which instructions will the nurse include in the parents teaching plan?

You Selected:

Visit but do not touch the neonate.

Correct response:

Wash hands thoroughly before touching the neonate.


Explanation:

Question 37 See full question

A nurse is teaching parents how to reduce the spread of impetigo. The nurse should encourage parents to:

You Selected:

teach children the importance of proper hand washing.

Correct response:

teach children the importance of proper hand washing.

Explanation:

Question 38 See full question

A nurse is caring for a group of toddlers in a large urban hospital. When considering providing care, which clients
require contact precautions? Select all that apply.

You Selected:

A toddler with a multidrug-resistant organism.

Correct response:

A toddler with scabies.


A toddler with a multidrug-resistant organism.

Explanation:

Question 39 See full question

A nurse is leading a group of parents of toddlers in a discussion on home safety. The nurse should emphasize which
fact?

You Selected:

A toddler's risk of injury is the same as that of an adult.

Correct response:

Most toddler deaths are accidental.

Explanation:

Question 40 See full question

A nurse-manager identifies fall prevention as a unit priority. Which of the following actions can the nurses
implement to meet these goals? Select all that apply.
You Selected:

Apply soft waist restraint to confused clients

Correct response:

Use bed alarms to remind clients to call for help getting up


Maintain a clear path to client bathrooms
Make hourly rounds to client rooms

Explanation:

Question 41 See full question

Four clients in a critical care unit have been diagnosed with Psuedomonas aeruginosa. The Infection Prevention and
Control Department has determined that this is probably a nosocomial infection. Select the most appropriate
intervention by the nurse. The nurse should:

You Selected:

initiate transmission-based precautions.

Correct response:

ensure that staff members do not have artificial fingernails.

Explanation:

Question 42 See full question

While caring for the neonate of a human immunodeficiency virus-positive mother, the nurse prepares to administer
a prescribed vitamin K intramuscular injection at 1 hour after birth. Which action should the nurse do first?

You Selected:

Place the neonate under a radiant warmer.

Correct response:

Bathe the neonate.

Explanation:

Question 43 See full question

A physician enters a computer order for a nurse to irrigate a client's nephrostomy tube every 4 hours to maintain
patency. The nurse irrigates the tube using sterile technique. After irrigating the tube, the nurse decides that she
can safely use the same irrigation set for her 8-hour shift if she covers the set with a paper, sterile drape. This
action by the nurse is:

You Selected:
appropriate because the irrigation just checks for patency.

Correct response:

inappropriate because irrigation requires strict sterile technique.

Explanation:

Question 44 See full question

A nurse notices that a client admitted for treatment of major depression is pacing, agitated, and becoming verbally
aggressive toward other clients. What is the immediate care priority?

You Selected:

Removing the other clients from the area until this client settles down

Correct response:

Ensuring the safety of this client and other clients on the unit

Explanation:

Question 45 See full question

When planning home care for a 3-year-old child with eczema, what should the nurse teach the mother to remove
from the child's environment at home?

You Selected:

wooden blocks

Correct response:

stuffed animals

Explanation:

Question 46 See full question

A client arrives on the psychiatric unit exhibiting extreme excitement, disorientation, incoherent speech, agitation,
frantic and aimless physical activity, and grandiose delusion. Which nursing diagnosis takes highest priority for this
client at this time?

You Selected:

Risk for injury

Correct response:

Risk for injury


Explanation:

Question 47 See full question

To ensure safety for a hospitalized blind client, the nurse should:

You Selected:

keep the side rails up when the client is alone.

Correct response:

orient the client to the room environment.

Explanation:

Question 48 See full question

A client develops a facial rash and urticaria after receiving penicillin. Which laboratory value does the nurse expect
to be elevated?

You Selected:

IgB

Correct response:

IgE

Explanation:

Question 49 See full question

During rounds, a nurse finds that a client with hemiplegia has fallen from the bed because the nursing assistant
failed to raise the side rails after giving a back massage. The nurse assists the client to the bed and assesses for
injury. As per agency policies, the nurse fills out an incident report. Which of the following activities should the
nurse perform after finishing the incident report?

You Selected:

Attach a copy to the client's records.

Correct response:

Include the time and date of the incident.

Explanation:

Question 50 See full question


A physician has ordered a heating pad for an elderly client's lower back pain. Which item would be most important
for a nurse to assess before applying the heating pad?

You Selected:

Client's nutritional status

Correct response:

Client's level of consciousness

Explanation:

Answer Key

Question 1 See full question


A nurse notices that a client with obsessive-compulsive disorder washes his hands for long periods each day. How
should the nurse respond to this compulsive behavior?

You Selected:
By setting aside times during which the client can focus on the behavior

Correct response:
By setting aside times during which the client can focus on the behavior

Explanation:

Question 2 See full question


Which client statement indicates the need for additional teaching about benzodiazepines?

You Selected:
"I can stop taking the drug anytime I want."

Correct response:
"I can stop taking the drug anytime I want."

Explanation:

Question 3 See full question


A client describes anxiety attacks that usually occur shortly after work when he is preparing his evening meal.
Which question would be most appropriate for the nurse to ask the client first in an effort to learn how he can be
helped?

You Selected:
"What are you thinking about before you start to prepare supper?"

Correct response:
"What are you thinking about before you start to prepare supper?"
Explanation:

Question 4 See full question


A shy 12-year-old girl who must change school systems just before she begins junior high school begins cutting her
arms to relieve the stress that she feels about leaving long-standing friends, having to develop new friendships, and
meeting high academic standards in her new school. After she has been cutting for a few weeks, her parent
discovers the injuries and takes her to a psychiatrist mental health provider who prescribes a therapeutic group at
the local mental health center and medication to help decrease her anxiety. Which findings indicate that the girl had
made appropriate progress toward recovery? Select all that apply.

You Selected:
The girl indicates that she had joined three clubs at school and agreed to be an officer in one of them.
The girl says she has developed a friendship with a girl in her class and one in her therapy group.
The girl wears short-sleeved and/or sleeveless tops when the weather is warm.

Correct response:
The girl says she has developed a friendship with a girl in her class and one in her therapy group.
The girl wears short-sleeved and/or sleeveless tops when the weather is warm.
The girl's grades are good, and her hours of study are not excessive.

Explanation:

Answer Key

Question 1 See full question


While shopping at a mall, a woman experiences an episode of extreme terror accompanied by anxiety, tachycardia,
trembling, and fear of going crazy. A friend drives her to the emergency department, where a physician rules out
physiologic causes and refers her to the psychiatric resident on call. To control the client's anxiety, the nurse caring
for this client expects the resident to order:

You Selected:
lorazepam.

Correct response:
lorazepam.

Explanation:

Question 2 See full question


A client is diagnosed with obsessive-compulsive disorder. Which intervention should the nurse include when
developing the care plan for this client?

You Selected:
Giving the client time to perform rituals

Correct response:
Giving the client time to perform rituals

Explanation:

Question 3 See full question


A woman has become increasingly afraid to ride in elevators. While in an elevator one morning, she experiences
shortness of breath, palpitations, dizziness, and trembling. A physician can find no physiological basis for these
symptoms and refers her to a psychiatric clinical nurse specialist for outpatient counseling sessions. Which type of
therapy is most likely to reduce the client's anxiety level?

You Selected:
Systematic desensitization

Correct response:
Systematic desensitization

Explanation:

Question 4 See full question


The nurse observes a client with a history of panic attacks is hyperventilating. The nurse should:

You Selected:
have the client breathe into a paper bag.

Correct response:
have the client breathe into a paper bag.

Explanation:

Question 5 See full question


A client is anxious following a robbery. The client is worried about identity theft and states, I could lose everything.
I cannot stand the fears I have. I reported everything, but I still cannot eat or sleep. Which intervention should the
nurse implement first?

You Selected:
Listen empathetically while the client discusses the fears.

Correct response:
Listen empathetically while the client discusses the fears.

Explanation:

Answer Key

Question 1 See full question


An 18-year-old is highly dependent on her parents and fears leaving home to attend college. Shortly before the fall
semester starts, she complains that her legs are paralyzed and is rushed to the emergency department. When
physical examination rules out a physical cause for her paralysis, the physician admits the woman to the psychiatric
unit, where she is diagnosed with functional neurologic symptom disorder. She asks the nurse, "Why has this
happened to me?" What is the nurse's best response?

You Selected:
"Your problem is real but, there is no physical basis for it. We'll work on what is going on in your life to find out why
it's happened."

Correct response:
"Your problem is real but, there is no physical basis for it. We'll work on what is going on in your life to find out why
it's happened."
Explanation:

Question 2 See full question


A 40-year-old client is admitted for a surgical biopsy of a suspicious lump in her left breast. When the nurse arrives
to take the client to surgery, she is tearfully completing a letter to her two children. She tells the nurse, "I want to
leave this for my children in case anything goes wrong today." Which response by the nurse would be most
therapeutic?

You Selected:
"In case anything goes wrong? What are your thoughts and feelings right now?"

Correct response:
"In case anything goes wrong? What are your thoughts and feelings right now?"

Explanation:

Question 3 See full question


A client arrives on the psychiatric unit exhibiting extreme excitement, disorientation, incoherent speech, agitation,
frantic and aimless physical activity, and grandiose delusion. Which nursing diagnosis takes highest priority for this
client at this time?

You Selected:
Risk for injury

Correct response:
Risk for injury

Explanation:

Question 4 See full question


After learning that a roommate is HIV-positive, a client asks a nurse about moving to another room on the
psychiatric unit because he no longer feels "safe." What should the nurse do first?

You Selected:
Ask the client to describe his fears.

Correct response:
Ask the client to describe his fears.

Explanation:

Question 5 See full question


During a shift report, a nurse learns that she will be providing care for a client who's vulnerable to panic attack.
Treatment for panic attacks includes behavioral therapy, supportive psychotherapy, and medication such as:

You Selected:
antianxiety drugs.

Correct response:
antianxiety drugs.
Explanation:

Question 6 See full question


While a client is taking alprazolam, which food should the nurse instruct the client to avoid?

You Selected:
alcohol

Correct response:
alcohol

Explanation:

Question 7 See full question


What should the nurse teach a client with generalized anxiety disorder to help the client cope with anxiety?

You Selected:
cognitive and behavioral strategies

Correct response:
cognitive and behavioral strategies

Explanation:

Question 8 See full question


The client with obsessive-compulsive disorder eats slowly and is always the last to finish lunch, which makes it
difficult for the group to start at 1300. Which approach would be the best plan of action for this problem?

You Selected:
Arrange for the client to start eating earlier than the others.

Correct response:
Arrange for the client to start eating earlier than the others.

Explanation:

Question 9 See full question


A client is admitted to the psychiatric unit with a diagnosis of functional neurologic symptom disorder. Since
witnessing a beating at gunpoint, the client is paralyzed. Which action should the nurse initially focus on when
planning this client's care?

You Selected:
Helping the client identify and verbalize his/her feelings about the incident

Correct response:
Helping the client identify and verbalize his/her feelings about the incident
Explanation:

Answer Key

Question 1 See full question


A client diagnosed as having panic disorder with agoraphobia is admitted to the inpatient psychiatric unit. Until her
admission, this client had been a virtual prisoner in her home for 5 weeks, afraid to go outside even to buy food.
When planning care for this client, what is the nurse's overall goal?

You Selected:
To help the client function effectively in her environment

Correct response:
To help the client function effectively in her environment

Explanation:

Question 2 See full question


A nurse in a psychiatric inpatient unit is caring for a client with generalized anxiety disorder. As part of the client's
treatment, the psychiatrist orders lorazepam, 1 mg by mouth three times per day. During lorazepam therapy, the
nurse should remind the client to:

You Selected:
avoid caffeine.

Correct response:
avoid caffeine.

Explanation:

Question 3 See full question


A client is scheduled for cardiac catheterization the next morning. His physician ordered temazepam, 30 mg by
mouth at bedtime, for sedation. Before administering the drug, the nurse should know that:

You Selected:
sedatives reduce excitement; hypnotics induce sleep.

Correct response:
sedatives reduce excitement; hypnotics induce sleep.

Explanation:

Question 4 See full question


A nurse is interviewing a client with posttraumatic stress disorder (PTSD) when a loud, booming noise from a
passing car's radio rattles the windows. The client jumps onto a chair, wide-eyed and frantic. Which statement by
the nurse is the most therapeutic response?

You Selected:
"What kinds of feelings are you experiencing?"

Correct response:
"What kinds of feelings are you experiencing?"
Explanation:

Question 5 See full question


A client is taking diazepam while establishing a therapeutic dose of antidepressants for generalized anxiety
disorder. Which instruction should the nurse give to this client? Select all that apply.

You Selected:
to consult with his health care provider (HCP) before he stops taking the drug
not to use alcohol while taking the drug
to stop taking the drug if he experiences swelling of the lips and face and difficulty breathing

Correct response:
to consult with his health care provider (HCP) before he stops taking the drug
not to use alcohol while taking the drug
to stop taking the drug if he experiences swelling of the lips and face and difficulty breathing

Explanation:

Question 6 See full question


A client with posttraumatic stress disorder needs to find new housing and wants to wait for a month before setting
another appointment to see the nurse. How should the nurse interpret this action?

You Selected:
a method of avoidance

Correct response:
a necessary break in treatment

Explanation:

Question 7 See full question


An elderly client hospitalized 4 days ago for treatment of acute respiratory distress has become confused and
disoriented. The client has been picking invisible items off blankets and has been yelling at the daughter who is not
in the room. The family tells the nurse that the client has been treated for anxiety with alprazolam for years, but
alprazolam is not on the current medication list. Which safety measures should be implemented? Select all that
apply.

You Selected:
A prescription should be obtained to help with the hallucinations.
The client will be placed on withdrawal precautions and treatment started immediately.
The client's medical and mental status will be evaluated frequently and treated as needed.

Correct response:
The client will be placed on withdrawal precautions and treatment started immediately.
A prescription should be obtained to help with the hallucinations.
The client's medical and mental status will be evaluated frequently and treated as needed.

Explanation:

Question 8 See full question


A registered nurse caring for a client with generalized anxiety disorder identifies a nursing diagnosis of Anxiety. A
short-term goal is established as follows: The client will identify physical, emotional, and behavioral responses to
anxiety. Which nursing interventions will help the client achieve this goal? Select all that apply.
You Selected:
Help the client connect anxiety with uncomfortable physical, emotional, or behavioral responses.
Observe the client for overt signs of anxiety.
Introduce the client to new strategies for coping with anxiety, such as relaxation techniques and exercise.

Correct response:
Observe the client for overt signs of anxiety.
Help the client connect anxiety with uncomfortable physical, emotional, or behavioral responses.
Introduce the client to new strategies for coping with anxiety, such as relaxation techniques and exercise.

Explanation:

Question 9 See full question


In a mental health interview, a client who has returned from military service reports feeling ashamed of being
weak and of letting past experiences control thoughts and actions in the present. What is the nurses best
response?

You Selected:
Many people whove been in your situation experience similar emotions and behaviors.

Correct response:
Many people whove been in your situation experience similar emotions and behaviors.

Explanation:

Question 10 See full question


A client is admitted to the psychiatric unit with a diagnosis of functional neurologic symptom disorder. Since
witnessing a beating at gunpoint, the client is paralyzed. Which action should the nurse initially focus on when
planning this client's care?

You Selected:
Helping the client identify and verbalize his/her feelings about the incident

Correct response:
Helping the client identify and verbalize his/her feelings about the incident

Explanation:
Question 1 See full question
A client diagnosed as having panic disorder with agoraphobia is admitted to the inpatient psychiatric unit. Until her
admission, this client had been a virtual prisoner in her home for 5 weeks, afraid to go outside even to buy food.
When planning care for this client, what is the nurse's overall goal?
You Selected:
To help the client function effectively in her environment
Correct response:
To help the client function effectively in her environment
Explanation:
Question 2 See full question
A client is diagnosed with obsessive-compulsive disorder. Which intervention should the nurse include when
developing the care plan for this client?
You Selected:
Giving the client time to perform rituals
Correct response:
Giving the client time to perform rituals
Explanation:
Question 3 See full question
When performing a physical examination on an anxious client, a nurse should expect to find which effect produced
by the parasympathetic nervous system?
You Selected:
Hyperactive bowel sounds
Correct response:
Hyperactive bowel sounds
Explanation:
Question 4 See full question
A client is taking diazepam while establishing a therapeutic dose of antidepressants for generalized anxiety
disorder. Which instruction should the nurse give to this client? Select all that apply.
You Selected:
not to use alcohol while taking the drug
to consult with his health care provider (HCP) before he stops taking the drug
to stop taking the drug if he experiences swelling of the lips and face and difficulty breathing
Correct response:
to consult with his health care provider (HCP) before he stops taking the drug
not to use alcohol while taking the drug
to stop taking the drug if he experiences swelling of the lips and face and difficulty breathing
Explanation:
Question 5 See full question
The client states he washes his feet endlessly because they "are so dirty that I canot put on my socks and shoes."
The nurse recognizes the client is using ritualistic behavior primarily to relieve discomfort associated with which
feeling?
You Selected:
intolerable anxiety
Correct response:
intolerable anxiety
Explanation:
Question 6 See full question
A client describes anxiety attacks that usually occur shortly after work when he is preparing his evening meal.
Which question would be most appropriate for the nurse to ask the client first in an effort to learn how he can be
helped?
You Selected:
"What are you thinking about before you start to prepare supper?"
Correct response:
"What are you thinking about before you start to prepare supper?"
Explanation:
Question 7 See full question
A client moves in with her family after her boyfriend of 4 weeks told her to leave. She is admitted to the subacute
unit after reporting feeling empty and lonely, being unable to sleep, and eating very little for the last week. Her
arms are scarred from frequent self-mutilation. What should the nurse do in order of priority from first to last? All
options must be used.
You Selected:
Monitor for suicide and self-mutilation.
Monitor sleeping and eating behaviors.
Discuss the issues of loneliness and emptiness.
Discuss her housing options for after discharge.
Correct response:
Monitor for suicide and self-mutilation.
Monitor sleeping and eating behaviors.
Discuss the issues of loneliness and emptiness.
Discuss her housing options for after discharge.
Explanation:
Question 8 See full question
A client commonly jumps when spoken to and reports feeling uneasy. The client says, It is as though something
bad is going to happen. In which order, from first to last, should the nursing actions be done? All options must be
used.
You Selected:
Reduce environmental stimuli.
Discuss the clients feelings in more depth.
Ask the client to deep breathe for 2 minutes.
Teach problem-solving strategies.
Correct response:
Reduce environmental stimuli.
Ask the client to deep breathe for 2 minutes.
Discuss the clients feelings in more depth.
Teach problem-solving strategies.
Explanation:
Question 9 See full question
After being examined by a forensic nurse in the emergency department, a rape victim is prepared for discharge.
Because of the nature of the attack, this client is at risk for post-traumatic stress disorder (PTSD). Which symptoms
are associated with PTSD? Select all that apply.
You Selected:
Sleep disturbances
Recurrent, intrusive recollections or nightmares
Flight of ideas
Correct response:
Recurrent, intrusive recollections or nightmares
Sleep disturbances
Difficulty concentrating
Explanation:
Question 10 See full question
An 8-year-old child, diagnosed with obsessive-compulsive disorder, is admitted by the nurse to a psychiatric facility.
During the admission assessment, which behaviors would be characterized as compulsions? Select all that apply.
You Selected:
Repeatedly washing the hands.
Routinely climbing up and down a flight of stairs three times before leaving the house.
Checking and rechecking that the television is turned off before going to school.
Correct response:
Checking and rechecking that the television is turned off before going to school.
Repeatedly washing the hands.
Routinely climbing up and down a flight of stairs three times before leaving the house.
Explanation:

Question 10 See full question


An 8-year-old child, diagnosed with obsessive-compulsive disorder, is admitted by the nurse to a psychiatric facility.
During the admission assessment, which behaviors would be characterized as compulsions? Select all that apply.

You Selected:
Checking and rechecking that the television is turned off before going to school.
Repeatedly washing the hands.
Routinely climbing up and down a flight of stairs three times before leaving the house.

Correct response:
Checking and rechecking that the television is turned off before going to school.
Repeatedly washing the hands.
Routinely climbing up and down a flight of stairs three times before leaving the house.

Explanation:
Question 1 See full question
A nurse refers a client with severe anxiety to a psychiatrist for medication evaluation. The physician is most likely to
order which psychotropic drug regimen on a short-term basis?
You Selected:
Alprazolam, 0.25 mg orally every 8 hours
Correct response:
Alprazolam, 0.25 mg orally every 8 hours
Explanation:
Question 2 See full question
A client periodically has acute panic attacks. These attacks are unpredictable and have no apparent association
with a specific object or situation. During an acute panic attack, the client may experience:
You Selected:
a decreased perceptual field.
Correct response:
a decreased perceptual field.
Explanation:
Question 3 See full question
A client was admitted to the inpatient unit 3 days ago with a flat affect, psychomotor retardation, anorexia,
hopelessness, and suicidal ideation. The health care provider (HCP) prescribed 75 mg of venlafaxine extended
release to be given every morning. The client interacted minimally with the staff and spent most of the day in his
room. As the nurse enters the unit at the beginning of the evening shift, the client is smiling and cheerful and
appears to be relaxed. What should the nurse interpret as the most likely cause of the clients behavior?
You Selected:
The client's sudden improvement calls for close observation by the staff.
Correct response:
The client's sudden improvement calls for close observation by the staff.
Explanation:
Question 4 See full question
The client rushes out of the day room where he has been watching television with other clients. He is
hyperventilating and flushed and his fists are clenched. He states to the nurse, That bastard! He is just like Tom. I
almost hit him. What would be the nurses best response?
You Selected:
"You are angry, and you did well to leave the situation. Let us walk up and down the hall while you tell me about it.
Correct response:
"You are angry, and you did well to leave the situation. Let us walk up and down the hall while you tell me about it.
Explanation:
Question 5 See full question
An abused woman tells the nurse that her 8-year-old daughter refuses to go to school because she is afraid her
mother will not be home when she returns. What is the most therapeutic response for the nurse to make?
You Selected:
"Children often feel responsible for trouble in the family. Have you talked with her about what she is afraid might
happen?"
Correct response:
She is aware of the trouble in the family and is worried about what might happen. Would you like to have her talk
to the child therapist here? I think it would be helpful.
Explanation:
Question 6 See full question
A client is diagnosed with agoraphobia without panic disorder. Which type of therapy would most the nurse expect
to see included in the plan of care?
You Selected:
behavior therapy
Correct response:
behavior therapy
Explanation:
Question 7 See full question
A 16-year-old boy who is academically gifted is about to graduate from high school early since he has completed all
courses needed to earn a diploma. Within the last 3 months he has begun to experience panic attacks that have
forced him to leave classes early and occasionally miss a day of school. He is concerned that these attacks may
hinder his ability to pursue a college degree. What would be the best response by the school nurse who has been
helping him deal with his panic attacks?
You Selected:
"It sounds like you have a real concern about transitioning to college. I can refer you to a health care provider for
assessment and treatment."
Correct response:
"It sounds like you have a real concern about transitioning to college. I can refer you to a health care provider for
assessment and treatment."
Explanation:
Question 8 See full question
A client at an outpatient psychiatric clinic has been experiencing anxiety. The nurse would like to suggest activities
for the client to do in his/her spare time. What would be an appropriate activity for the nurse to suggest to the
client? Select all that apply.
You Selected:
Stretching exercises
Daily walks
Taking up a hobby
Correct response:
Taking up a hobby
Daily walks
Stretching exercises
Explanation:
Question 9 See full question
The nurse is assessing a client who has just experienced a crisis. The nurse should first assess this client for which
behavior?
You Selected:
level of anxiety
Correct response:
level of anxiety
Explanation:
Question 10 See full question
A client comes to the emergency department while experiencing a panic attack. What action by the nurse is most
appropriate?
You Selected:
Staying with the client until the attack subsides
Correct response:
Staying with the client until the attack subsides
Explanation:

Answer Key

Question 1 See full question


A nurse notices that a client with obsessive-compulsive disorder washes his hands for long periods each day. How
should the nurse respond to this compulsive behavior?

You Selected:
By setting aside times during which the client can focus on the behavior

Correct response:
By setting aside times during which the client can focus on the behavior

Explanation:

Question 2 See full question


A nurse is caring for a client with obsessive-compulsive disorder (OCD) with rituals of washing hands for 18 minutes,
combing hair 444 strokes, and switching the bathroom light on and off 44 times prior to meals. What is the most
appropriate long-term goal of care for this client?

You Selected:
Systematically decrease the number of repetitions of rituals and the amount of time the client spends performing
them.

Correct response:
Systematically decrease the number of repetitions of rituals and the amount of time the client spends performing
them.
Explanation:

Question 3 See full question


The nurse is caring for a client with a panic attack. Which nursing intervention is most helpful for this client?

You Selected:
Stay with the client and remaining calm, confident, and reassuring

Correct response:
Stay with the client and remaining calm, confident, and reassuring

Explanation:

Question 4 See full question


The nurse is caring for a client with social anxiety disorder. Which statement by the client is of concern for the
nurse?

You Selected:
"It helps me to have one or two drinks at lunch."

Correct response:
"It helps me to have one or two drinks at lunch."

Explanation:

Question 5 See full question


The nurse has completed client instruction about lorazepam. Which of the following client statements would
indicate that the client understands?

You Selected:
This medication will help me relax so that I can focus on problem solving.

Correct response:
This medication will help me relax so that I can focus on problem solving.

Explanation:

Answer Key

Question 1 See full question


A woman has become increasingly afraid to ride in elevators. While in an elevator one morning, she experiences
shortness of breath, palpitations, dizziness, and trembling. A physician can find no physiological basis for these
symptoms and refers her to a psychiatric clinical nurse specialist for outpatient counseling sessions. Which type of
therapy is most likely to reduce the client's anxiety level?

You Selected:
Systematic desensitization

Correct response:
Systematic desensitization
Explanation:

Question 2 See full question


A client is admitted to the acute psychiatric care unit after 2 weeks of increasingly erratic behavior. He has been
sleeping poorly, has lost 8 lb (3.6 kg), is poorly groomed, exhibits hyperactivity, and loudly denies the need for
hospitalization. Which nursing intervention takes priority for this client?

You Selected:
Decreasing environmental stimulation

Correct response:
Decreasing environmental stimulation

Explanation:

Question 3 See full question


A nurse is caring for a veteran with a history of explosive anger, unemployment, and depression since being
discharged from the service. The client reports feeling ashamed of being "weak" and of letting past experiences
control his thoughts and actions in the present. What is the nurse's best response?

You Selected:
"Many people who've been in your situation experience similar emotions and behaviors."

Correct response:
"Many people who've been in your situation experience similar emotions and behaviors."

Explanation:

Question 4 See full question


A nurse is assessing a client suffering from stress and anxiety. The most common physiologic response to stress and
anxiety is:

You Selected:
diarrhea.

Correct response:
diarrhea.

Explanation:

Question 5 See full question


A client taking paroxetine 40 mg PO every morning tells the nurse that her mouth feels like cotton. Which
statement by the client necessitates further assessment by the nurse?

You Selected:
"I am drinking 12 glasses of water every day."

Correct response:
"I am drinking 12 glasses of water every day."
Explanation:

Question 6 See full question


The client, who is a veteran and has posttraumatic stress disorder, tells the nurse about the horror and mass
destruction of war. He states, I killed all of those people for nothing. Which response by the nurse is appropriate?

You Selected:
"You did what you had to do at that time."

Correct response:
"You did what you had to do at that time."

Explanation:

Question 7 See full question


A client with obsessive-compulsive disorder reveals that he was late for his appointment because of my dumb
habit. I have to take off my socks and put them back on 41 times! I cannot stop until I do it just right. The nurse
interprets the clients behavior as most likely representing an effort to obtain:

You Selected:
relief from anxiety.

Correct response:
relief from anxiety.

Explanation:

Question 8 See full question


Which nursing action would be therapeutic for the client being admitted to the unit with panic disorder? Select all
that apply.

You Selected:
Support the client's attempts to discuss feelings.
Reassure the client of safety.
Respect the client's personal space.

Correct response:
Support the client's attempts to discuss feelings.
Respect the client's personal space.
Reassure the client of safety.

Explanation:

Question 9 See full question


A client reports that before he leaves home to go anywhere, he counts the money in his wallet as many as 12 times.
The nurse judges this behavior to indicate which client need?

You Selected:
the need to channel emotions unacceptable to him with an acceptable activity

Correct response:
the need to channel emotions unacceptable to him with an acceptable activity
Explanation:

Question 10 See full question


After months of coaxing by her husband, a client is brought by him to the mental health clinic and reports that she
suffers from an overwhelming fear of leaving the house, which has caused her to lose her job and is taking a toll on
her marriage. The physician diagnoses the client with agoraphobia. Which treatment options would the nurse be
aware of as effective in treating this disorder? Select all that apply.

You Selected:
Electroconvulsive therapy
Alprazolam therapy
Desensitization

Correct response:
Desensitization
Alprazolam therapy
Paroxetine therapy

Explanation:

The development of friendships and good grades with moderate amounts of study are positive signs since friends
and grades in the new school were sources of stress and anxiety for the girl. The ability to wear clothes appropriate
to the weather rather than hiding her arms is a sign she is no longer injuring her arms. Joining three clubs and being
an officer in one of them is unlikely and would probably be an additional source of stress for the girl as would be
pushing herself to extraordinary academic achievement to secure a place in college when she has just entered
junior high.

Question 5 See full question


The nurse is caring for a client with social anxiety disorder. Which statement by the client is of concern for the
nurse?

You Selected:
"It helps me to have one or two drinks at lunch."

Correct response:
"It helps me to have one or two drinks at lunch."

Explanation:
Question 1 See full question
A client with bipolar disorder is taking lithium carbonate 300 mg t.i.d. His lithium level is 2.7 mEq/L. In assessing the
client at his clinic visit, the nurse finds no evidence of lithium toxicity. The first assessment question the nurse
should ask before ordering another blood test is:
You Selected:
when the client took his last dose of lithium.
Correct response:
when the client took his last dose of lithium.
Explanation:
Question 2 See full question
Which client statement indicates that the client has gained insight into his use of the defense mechanism of
displacement?
You Selected:
"Now when I am mad at my wife, I talk to her instead of taking it out on the kids."
Correct response:
"Now when I am mad at my wife, I talk to her instead of taking it out on the kids."
Explanation:
Question 3 See full question
The nurse is teaching an unlicensed assistive personnel (UAP) about the care of clients with self-mutilation. Which
statement by the UAP would indicate teaching about self-mutilation has been effective?
You Selected:
"It is a way to express anger and rage."
Correct response:
"It is a way to express anger and rage."
Explanation:
Question 4 See full question
The nurse is caring for a client who has been physically abused. Which statement by the nurse expresses empathy
for this client?
You Selected:
I am so sad to see you going through so much pain.
Correct response:
It must be difficult what you have been going through.
Explanation:
Empathy is a persons ability to understand what another person is going through and be objective at the same
time. The nurse does not carry those feelings or that situation with them as in sympathy but is still able to relate to
the person well. It must be difficult what you have been going through is such an example. It gives the client an
opening to express any feelings regarding the abuse. Our staff will do the best they can to make you feel
comfortable is a stereotypical response that does not empathize with the client. Do you have questions about
what is happening? is a closed question and also a stereotypical question that nurses often ask when no other
statement is known to them. I am so sad to see you going through so much pain is an example of a sympathetic
response because the nurse is showing feelings of sadness over the clients situation.
Question 5 See full question
A 74-year-old client receiving fluphenazine decanoate therapy develops pseudoparkinsonism, and is ordered
amantadine hydrochloride. With the addition of this medication, the client reports feeling dizzy when standing.
Which response by the nurse is best?
You Selected:
When you change positions, do so slowly.
Correct response:
When you change positions, do so slowly.
Explanation:
Question 1 See full question
A nurse is leading group therapy with psychiatric clients. During the working phase of the group, what should the
nurse do?
You Selected:
Encourage group cohesiveness.
Correct response:
Encourage group cohesiveness.
Explanation:
Question 2 See full question
Which finding indicates that a client who has been raped will have future adjustment problems and need additional
counseling?
You Selected:
Her parents show shame and suspicion about her part in the rape.
Correct response:
Her parents show shame and suspicion about her part in the rape.
Explanation:
Question 3 See full question
In closed or locked units, the nurse judges the milieu as therapeutic when priorities are given to:
You Selected:
safety, structure, and support.
Correct response:
safety, structure, and support.
Explanation:
Question 4 See full question
A client states the following to the nurse: I am a failure, and I wish I had died. Which of the following statements
by the nurse demonstrates a therapeutic response?
You Selected:
You feel like a failure; would you like to talk more about the way you feel?
Correct response:
You feel like a failure; would you like to talk more about the way you feel?
Explanation:
Question 5 See full question
When working with clients in crisis, the nurse must be aware that crisis intervention differs from other forms of
therapeutic intervention in that crisis intervention focuses on which concern(s)?
You Selected:
Determining immediate problems, as perceived by the client, with the short-term goal of problem solving
Correct response:
Determining immediate problems, as perceived by the client, with the short-term goal of problem solving
Explanation:
Question 1 See full question
During the mental status examination, a client may be asked to explain such proverbs as "Don't cry over spilled
milk." The purpose of this is to evaluate the client's ability to think:
You Selected:
abstractly.
Correct response:
abstractly.
Explanation:
Question 2 See full question
A nurse documents, "The client described her husband's abuse in an emotionless tone and with a flat facial
expression." This statement describes the client's:
You Selected:
affect.
Correct response:
affect.
Explanation:
Question 3 See full question
The basis for building a strong, therapeutic nurse-client relationship begins with a nurse's:
You Selected:
sincere desire to help others.
Correct response:
self-awareness and understanding.
Explanation:
The nurse must be aware of herself and understand personal feelings before she can understand and help others.
Although wanting to help others, accepting others, and being knowledgeable of psychiatric nursing are desirable
traits, self-awareness and understanding are the basis of a therapeutic nurse-client relationship.
Question 4 See full question
A nurse is using drawing, puppetry, and other forms of play therapy while treating a terminally ill, school-age child.
The purpose of these techniques is to help the child:
You Selected:
express feelings that he can't articulate.
Correct response:
express feelings that he can't articulate.
Explanation:
Question 5 See full question
A nurse is developing a care plan for a client who has undergone electroconvulsive therapy (ECT). The nurse should
include which intervention?
You Selected:
Monitoring the client's vital signs every hour for 4 hours
Correct response:
Reorienting the client to time and place
Explanation:
Confusion and temporary memory loss are the most common adverse effects of ECT. A nurse should continually
reorient a client to time and place as he wakes up from the procedure. Following ECT, the nurse should monitor the
client's vital signs every 15 minutes for the first hour. The nurse should position the client on his side after the
procedure to reduce the risk of aspiration. The client should remain on bed rest until he's fully awake and oriented.
Question 6 See full question
A client in an acute care center lacerates her wrists. She has a history of conflicts and acting out and asks the
nurse, "I did a good job, didn't I?" Which response by the nurse is best?
You Selected:
"What were you feeling before you hurt yourself?"
Correct response:
"What were you feeling before you hurt yourself?"
Explanation:
Question 7 See full question
A nurse is obtaining a history from a client. The client reports that he is a waiter. When asked about his work
environment, the client says, "If customers confront me for not being attentive enough, I just spit on their food."
The nurse suspects the client is prone to which type of behavior?
You Selected:
Passive-aggressive
Correct response:
Passive-aggressive
Explanation:
Question 8 See full question
A client in a group therapy setting is very demanding. He repeatedly interrupts others and monopolizes most of the
group time. The nurse's best response would be:
You Selected:
"Will you briefly summarize your point? Others also need time."
Correct response:
"Will you briefly summarize your point? Others also need time."
Explanation:
Question 9 See full question
On admission to the mental health unit, a client tells the nurse she's afraid to leave the house for fear of criticism.
She informs the nurse, "My nose is so big. I know everyone is looking at me and making fun of me. I had plastic
surgery and it still looks awful!" These symptoms are an indication of:
You Selected:
body dysmorphic disorder.
Correct response:
body dysmorphic disorder.
Explanation:
Question 10 See full question
Assertive behavior involves:
You Selected:
standing up for one's rights while respecting the rights of others.
Correct response:
standing up for one's rights while respecting the rights of others.
Explanation:
Question 11 See full question
The nurse is planning an education for new nurses on psychiatric units. Which topic should be given priority?
You Selected:
Breach of confidentiality
Correct response:
Breach of confidentiality
Explanation:
Question 12 See full question
The charge nurse in an acute care setting assigns a client who is on one-on-one suicide precautions to a psychiatric
aide. This assignment is considered:
You Selected:
reasonable nursing practice because one-on-one requires the total attention of a staff member.
Correct response:
reasonable nursing practice because one-on-one requires the total attention of a staff member.
Explanation:
Question 13 See full question
In group therapy, a client angrily speaks up and responds to a peer, "You're always whining, and I'm getting tired of
listening to you! Here is the world's smallest violin playing for you." Which role is the client playing?
You Selected:
Aggressor
Correct response:
Aggressor
Explanation:
Question 14 See full question
Parents tell a nurse that they have not met their goal of home management of their son with schizoaffective
disorder. They report that the client poses a threat to their safety. Based on this information, what recommendation
should the nurse make?
You Selected:
Evaluate the client for voluntary admission to a mental health facility.
Correct response:
Evaluate the client for voluntary admission to a mental health facility.
Explanation:
Question 15 See full question
A client with a diagnosis of bipolar disorder is energetic, impulsive, and verbalizes loudly in the community room. To
prevent injury while complying with the principle of the least-restrictive environment, which action should the nurse
take to prevent escalation of the client's mood?
You Selected:
Try to channel the client's energy into appropriate activities.
Correct response:
Try to channel the client's energy into appropriate activities.
Explanation:
Question 16 See full question
During an assessment interview, a depressed 15-year-old girl states that she "can't sleep at night." The nurse
begins to explore factors that might contribute to this situation by asking if the girl is sexually active. The girl
changes the subject. What should the nurse suspect based on the client's response to the assessment question?
You Selected:
Sexual abuse
Correct response:
Sexual abuse
Explanation:
Question 17 See full question
The client approaches various staff with numerous requests and needs to the point of disrupting the staff's work
with other clients. The nurse meets with the staff to decide on a consistent, therapeutic approach for this client.
Which approach will be most effective?
You Selected:
having the client discuss needs with the staff person assigned
Correct response:
having the client discuss needs with the staff person assigned
Explanation:
Question 18 See full question
A married female client has been referred to the mental health center because she is depressed. The nurse notices
bruises on her upper arms and asks about them. After denying any problems, the client starts to cry and says, He
did not really mean to hurt me, but I hate for the kids to see this. I am so worried about them. What is the most
crucial information for the nurse to determine?
You Selected:
the potential of immediate danger to the client and her children
Correct response:
the potential of immediate danger to the client and her children
Explanation:
Question 19 See full question
After months of counseling, a client abused by her husband tells the nurse that she has decided to stop treatment.
There has been no abuse during this time, and she feels better able to cope with the needs of her husband and
children. In discussing this decision with the client, the nurse should:
You Selected:
find out more about the client's rationale for her decision to stop treatment.
Correct response:
find out more about the client's rationale for her decision to stop treatment.
Explanation:
Question 20 See full question
A nurse is teaching the families of clients with chronic mental illnesses about causes of relapse and
rehospitalization. What should the nurse include as the primary cause?
You Selected:
noncompliance with medications
Correct response:
noncompliance with medications
Explanation:
Question 21 See full question
In addition to teaching assertiveness and problem-solving skills when helping the client cope effectively with stress
and anxiety, the nurse should also address the client's ability to:
You Selected:
use conflict resolution skills.
Correct response:
use conflict resolution skills.
Explanation:
Question 22 See full question
When planning the care of a client experiencing aggression, the nurse incorporates the principle of "least restrictive
alternative," meaning that less restrictive interventions must be tried before more restrictive measures are
employed. Which measure should the nurse consider to be the most restrictive?
You Selected:
haloperidol given intramuscularly
Correct response:
haloperidol given intramuscularly
Explanation:
Question 23 See full question
The nurse manager on a psychiatric unit is reviewing the outcomes of staff participation in an aggression
management program. What indicator would the nurse used to evaluate the effectiveness of such a program?
You Selected:
a reduction in the total number of restraint procedures
Correct response:
a reduction in the total number of restraint procedures
Explanation:
Question 24 See full question
During an appointment with the nurse, a client says, I could hate God for that flood. The nurse responds, Oh, do
not feel that way. We are making progress in these sessions. The nurses statement demonstrates a failure to do
what?
You Selected:
Look for meaning in what the client says.
Correct response:
Look for meaning in what the client says.
Explanation:
Question 25 See full question
The nurse is teaching a group of unlicensed assistive personnel (UAP) about providing care to clients with
depression. Which approach by one of the UAPs indicates an understanding of the most effective approach to a
depressed client?
You Selected:
empathetic
Correct response:
empathetic
Explanation:
Question 26 See full question
A 6-year-old client is diagnosed with attention deficit hyperactivity disorder (ADHD). When asking this client to
complete a task, what techniques should the nurse use to communicate most effectively with him?
You Selected:
Obtain eye contact before speaking, use simple language, and have him repeat what was said. Praise him if he
completes the task.
Correct response:
Obtain eye contact before speaking, use simple language, and have him repeat what was said. Praise him if he
completes the task.
Explanation:
Question 27 See full question
What client behaviors would be most important for the nurse to consider in deciding to institute suicide precautions
because of high-risk behavior?
You Selected:
The client recently attempted suicide with a lethal method.
Correct response:
The client recently attempted suicide with a lethal method.
Explanation:
Question 28 See full question
The client was admitted to the psychiatric unit yesterday evening. In the morning, the client approaches the nurse
and states, The psychiatrist all of you nurses are conspiring against me. I have been warned and I know it is true.
You know what I mean. Which response by the nurse would be most therapeutic?
You Selected:
"You must feel very frightened. You are safe here."
Correct response:
"You must feel very frightened. You are safe here."
Explanation:
Question 29 See full question
When upset, the client curls into a fetal position in bed. The nurse judges the client to be exhibiting which
condition?
You Selected:
regression
Correct response:
regression
Explanation:
Question 30 See full question
As the nurse helps the client prepare for discharge, the client says, You know, I have been in lots of hospitals, and I
know when I am sick enough to be there. I am not that sick now. You do not need to worry about me. What would
be the most therapeutic response by the nurse?
You Selected:
"We are concerned about you. How can we help you before you leave?"
Correct response:
"We are concerned about you. How can we help you before you leave?"
Explanation:
Question 31 See full question
A client asks the nurse to help make out a will. The nurse should tell the client:
You Selected:
I am not a lawyer, but I will do what I can for you.
Correct response:
You need to consult an attorney because I am not trained in such matters. Is there a family lawyer you can call?
Explanation:
A will is an important legal document. It is best to have one prepared with the help of an attorney. It would be
unwise to help the client or to seek another nurses help because a nurse is not a lawyer. Asking the client to delay
preparing the will just avoids the problem.
Question 32 See full question
A client who has been physically abused by her spouse agrees to meet with the nurse. Before the nurse terminates
the meeting with the client, the nurse should:
You Selected:
give the client the telephone numbers of a shelter or a safe house and the crisis line.
Correct response:
give the client the telephone numbers of a shelter or a safe house and the crisis line.
Explanation:
Question 33 See full question
During the conversation with the nurse, a victim of physical abuse says, Let me try to explain why I stay with my
husband. Which response would the nurse find inconsistent with the profile of a battered partner?
You Selected:
I am not sure I could get a job that pays even minimum wage.
Correct response:
"The abuse adds spice to our relationship."
Explanation:
Saying that abuse adds spice suggests the woman actually enjoys the violent relationship and is inconsistent with
the profile of victim of battery.

Women are conditioned to be responsible for the familys well-being. This is often a motivation for a battered
woman to stay in an abusive relationship.

The victim believes that she can save the relationship and that her partner will change. Feelings of guilt surrounding
issues such as these often influence an abused womans decisions about staying with her partner.

A womans lack of job skills and financial resources may cause her to stay. Many women are injured or killed when
they try to leave a violent relationship.

Question 34 See full question


A client asks the nurse for medication because she is feeling nervous. Within a therapeutic milieu, what should the
nurse do initially?
You Selected:
Talk with the client about her feelings.
Correct response:
Talk with the client about her feelings.
Explanation:
Question 35 See full question
The nurse meets with a client in the outpatient clinic who is suicidal and refuses to sign a no suicide contract.
What should the nurse do next?
You Selected:
Arrange for immediate hospitalization on a locked unit.
Correct response:
Arrange for immediate hospitalization on a locked unit.
Explanation:
Question 36 See full question
The nurse is teaching a group of unlicensed personnel new to psychiatry about balance in a therapeutic milieu.
Which statement by a member of the group indicates the need for further teaching?
You Selected:
Controlling clients helps them feel more comfortable.
Correct response:
Controlling clients helps them feel more comfortable.
Explanation:
Question 37 See full question
Which physiologic response should the nurse expect as unlikely to occur when a client is angry?
You Selected:
decreased blood pressure
Correct response:
decreased blood pressure
Explanation:
Question 38 See full question
The client diagnosed with conversion disorder has a paralyzed arm. A staff member states, I would just tell the
client her arm is paralyzed because she had an affair and neglected her babys care to the point where the baby
had to be hospitalized for dehydration. Which response by the nurse is best?
You Selected:
"Pushing insight will increase the client's anxiety and the need for physical symptoms."
Correct response:
"Pushing insight will increase the client's anxiety and the need for physical symptoms."
Explanation:
Question 39 See full question
An adolescent girl is brought to the hospital emergency department in a state of unconsciousness after having
swallowed a bottle of pain pills 45 minutes earlier. The pills are identified as oxycodone. A suicide note is found
that asks for forgiveness. Which measure should the nurse be prepared to carry out when this client is admitted?
You Selected:
giving naloxone IV
Correct response:
giving naloxone IV
Explanation:
Question 40 See full question
The client is laughing and telling jokes to a group of clients. Suddenly, the client is crying and talking about a death
in the family. A moment later, the client is laughing and joking again. The nurse should:
You Selected:
Ask the client to come to a quiet area to talk to the nurse individually.
Correct response:
Ask the client to come to a quiet area to talk to the nurse individually.
Explanation:
Question 41 See full question
An elderly woman experiences short-term memory problems and occasional disorientation a few weeks after her
husbands death. She also is not sleeping, has urinary frequency and burning, and sees rats in the kitchen. The
home care nurse calls the womans health care provider (HCP) to discuss the clients situation and background,
assess, and give recommendations. The nurse concludes that the woman:
You Selected:
is experiencing delirium and a urinary tract infection (UTI).
Correct response:
is experiencing delirium and a urinary tract infection (UTI).
Explanation:
Question 42 See full question
A client diagnosed with paranoid personality disorder is being admitted on an involuntary 24-hour hold after a
physical altercation with a police officer who was investigating the clients threatening phone calls to his neighbors.
He states that his neighbors are spying on him for the government, saying, I want them to stop and leave me
alone. Now they have you nurses and doctors involved in their conspiracy. Which nursing approaches are most
appropriate? Select all that apply.
You Selected:
Approach the client in a professional, matter-of-fact manner.
Develop trust consistently with the client.
Gently present reality to counteract the client's paranoid beliefs.
Avoid intrusive interactions with the client.
Correct response:
Approach the client in a professional, matter-of-fact manner.
Avoid intrusive interactions with the client.
Develop trust consistently with the client.
Do not pressure the client to attend any groups.
Explanation:
A professional, matter-of-fact approach and developing trust are the most effective with this client. A friendly
approach, intrusiveness, and attempting to counteract the clients beliefs will increase the clients paranoia; he will
present more false beliefs to prove he is right about the conspiracy. Placing the client in group settings may be
counterproductive because questions and emotionality from peers, as well as confrontations with reality, will
increase the clients anxiety.v
Question 43 See full question
The healthcare provider prescribes venlafaxinefor the client. The nurse explains the purpose of the medication to
the client. The client asks the nurse, If I start taking the pills, will I have to take them the rest of my life? Which
would be the nurses most accurate and therapeutic reply?
You Selected:
"The medication prescribed is safe and routine."
Correct response:
"After your symptoms decrease, the need for medication will be reevaluated."
Explanation:
Telling the client that the need for medication will be reevaluated after symptoms decrease provides the most
complete information about both the current and future treatment plans and answers the question asked by the
client.

The other responses dismiss or ignore the clients concerns. Two provide no answer to the clients question.
Question 44 See full question
The nurse is working with a client with depression in a mental health clinic. During the interaction, the nurse uses
the technique of self-disclosure. In order for this technique to be therapeutic, which of the following steps must be a
priority for the nurse?
You Selected:
Ensuring relevance to, and quickly refocusing upon, the clients experience
Correct response:
Ensuring relevance to, and quickly refocusing upon, the clients experience
Explanation:
Question 45 See full question
The nurse is performing an assessment on a client with a history of a dysfunctional family. Which findings should
the nurse anticipate? Select all that apply.
You Selected:
Direction and attention
Abuse and neglect
Unhealthy personal boundaries
Correct response:
Unhealthy personal boundaries
Abuse and neglect
Explanation:
Healthy boundaries setting limits are established in childhood when parents provide consistent, supportive limits,
and attention. In a dysfunctional family, the parents are unable to give the support, attention, care, discipline, and
direction that children need in order to develop into mature adults. Often they are abused, emotionally or otherwise,
or neglected. This leads to a poor self-concept and role confusion, the basis for unhealthy personal boundaries.
Question 46 See full question
The nurse is admitting a client diagnosed with depression. Which statements by the nurse should be made in the
orientation phase of the nurseclient relationship? Select all that apply.
You Selected:
I will not be sharing any information with other family without your permission.
We will be meeting every day at 10:00 am for 15 minutes.
Tell me what brought you here today.
Correct response:
I will not be sharing any information with other family without your permission.
We will be meeting every day at 10:00 am for 15 minutes.
Tell me what brought you here today.
Explanation:
Question 47 See full question
The nurse has been teaching a client about depression. Which statement indicates insight into the clients
diagnosis?
You Selected:
I believe that my sadness affects my work and feelings.
Correct response:
I believe that my sadness affects my work and feelings.
Explanation:
Question 48 See full question
A nurse is working with an adolescent who has reported low self-esteem. When developing a plan of care, the nurse
considers the adolescent's psychosocial needs. Which of the following questions will best assist the nurse in
assessing the adolescents psychosocial development?
You Selected:
How did you come to understand your feelings about yourself?
Correct response:
How did you come to understand your feelings about yourself?
Explanation:
Question 49 See full question
A client has been involuntarily committed to a hospital because he has been assessed as being dangerous to self or
others. The client has lost which right?
You Selected:
the right to leave the hospital against medical advice
Correct response:
the right to leave the hospital against medical advice
Explanation:
Question 50 See full question
A client arrives at the emergency department confused and disoriented. The client does not have anyone else
present to provide information. Which route of communication used by the nurse is most effective?
You Selected:
Speaking slowly and enunciating words
Correct response:
Therapeutic touch
Explanation:
Option one is the only option which does not require being oriented. Therapeutic touch can calm the client while the
nurse is assessing client symptoms. The client is not hard of hearing to need to speak slowly and enunciate or need
a loud tone. Writing down the questions requires that the client is oriented enough to answer them.
Improve your mastery
TAKE A PRACTICE QUIZ
Question 1 See full question
A client stalks a man she met briefly 3 years earlier. She believes he loves her and eventually will marry her and
she has been sending him cards and gifts. When she violates a restraining order he has obtained, a judge orders
her to undergo a 10-day psychiatric evaluation. What is the most probable psychiatric diagnosis for this client?
You Selected:
Delusional disorder erotomanic type
Correct response:
Delusional disorder erotomanic type
Explanation:
Question 2 See full question
Erikson described the psychosocial tasks of the developing person in his theoretical model. He proposed that the
primary developmental task of the young adult (ages 18 to 25) is:
You Selected:
intimacy versus isolation.
Correct response:
intimacy versus isolation.
Explanation:
Question 3 See full question
What is a generally accepted criterion of mental health?
You Selected:
Self-acceptance
Correct response:
Self-acceptance
Explanation:
Question 4 See full question
Which term refers to the primary unconscious defense mechanism that blocks intense, anxiety-producing situations
from a person's conscious awareness?
You Selected:
Repression
Correct response:
Repression
Explanation:
Question 5 See full question
A nurse must assess a client's judgment to determine his mental status. To best accomplish this, the nurse should
have the client:
You Selected:
discuss hypothetical ethical situations.
Correct response:
discuss hypothetical ethical situations.
Explanation:
Question 6 See full question
The nurse is admitting a client with Borderline Personality Disorder. When planning care for this client, the nurse
should give priority to which item?
You Selected:
Safety
Correct response:
Safety
Explanation:
Question 7 See full question
A client diagnosed with antisocial personality disorder asks the nurse if he can have an additional smoke break
because he's anxious. Which response by the nurse is best?
You Selected:
"Clients are permitted to smoke at designated times. You have to follow the rules."
Correct response:
"Clients are permitted to smoke at designated times. You have to follow the rules."
Explanation:
Question 8 See full question
Which statement accurately describes therapeutic communication?
You Selected:
Avoiding advice, judgment, false reassurance, and approval
Correct response:
Avoiding advice, judgment, false reassurance, and approval
Explanation:
Question 9 See full question
A nurse is caring for a client diagnosed with body dysmorphic disorder. When the client verbalizes disapproval of
her own physical features, the nurse should:
You Selected:
encourage the client to talk about her fears and stressful life situations.
Correct response:
encourage the client to talk about her fears and stressful life situations.
Explanation:
Question 10 See full question
Nursing care for a client after electroconvulsive therapy (ECT) should include:
You Selected:
assessment of short-term memory loss.
Correct response:
assessment of short-term memory loss.
Explanation:
Question 11 See full question
On the second day of hospitalization, a client is discussing with the nurse his concerns about unhealthy family
relationships. During a nurse-client interaction, the client changes the subject to a job situation. The nurse
responds, "Let's go back to what we were just talking about." What therapeutic communication technique did the
nurse use?
You Selected:
Focusing
Correct response:
Focusing
Explanation:
Question 12 See full question
As a client is being released from restraints, he says, "I'll never get that angry and lose it again. Those restraints
were the worst things that ever happened to me." Which response by the nurse is most appropriate?
You Selected:
"I'd like to talk with you about your experience."
Correct response:
"I'd like to talk with you about your experience."
Explanation:
Question 13 See full question
The widow of a client who successfully completed suicide tearfully says, "I feel guilty because I am so angry at him
for killing himself. It must have been what he wanted." After assisting the widow with dealing with her feelings,
which intervention is most helpful?
You Selected:
Suggest she receive individual therapy by the nurse.
Correct response:
Refer her to a group for survivors of suicide.
Explanation:
Question 14 See full question
A client lives in a group home and visits the community mental health center regularly. During one visit with the
nurse, the client states, "The voices are telling me to hurt myself again." Which question by the nurse is most
important to ask?
You Selected:
"Are you going to hurt yourself?"
Correct response:
"Are you going to hurt yourself?"
Explanation:
Question 15 See full question
The client tells the nurse at the outpatient clinic that she does not need to attend groups because she is "not a
regular like these other people here." The nurse should respond to the client by saying:
You Selected:
"You say you are not a regular here, but you are experiencing what others are experiencing."
Correct response:
"You say you are not a regular here, but you are experiencing what others are experiencing."
Explanation:
Question 16 See full question
One of the myths about sexual abuse of young children is that it usually involves physically violent acts. Which
behavior is more likely to be used by the abusers?
You Selected:
coercion as a result of the trusting relationship
Correct response:
coercion as a result of the trusting relationship
Explanation:
Question 17 See full question
When working with a group of adult survivors of childhood sexual abuse, dealing with anger and rage is a major
focus. Which strategy should the nurse expect to be successful? Select all that apply.
You Selected:
writing letters to the abusers but not sending them
keeping a journal of memories and feelings
writing letters to the adults who did not protect them but not sending them
using a foam bat while symbolically confronting the abuser
Correct response:
using a foam bat while symbolically confronting the abuser
keeping a journal of memories and feelings
writing letters to the abusers but not sending them
writing letters to the adults who did not protect them but not sending them
Explanation:
Question 18 See full question
A client who is suspicious of others, including staff, is brought to the hospital wearing a wrinkled dress with stains
on the front. Assessment also reveals a flat affect, confusion, and slow movements. Which goal should the nurse
identify as the initial priority when planning this client's care?
You Selected:
helping the client feel safe and accepted
Correct response:
helping the client feel safe and accepted
Explanation:
Question 19 See full question
A 79-year-old woman is brought to the outpatient clinic by her daughter for a routine medication evaluation. The
daughter reports that her mother is quite stable and has no adverse effects from the risperidone she is taking. Then
the daughter says, I just think my mother could be even better if she was on a larger dosage. My son takes 1 mg of
risperidone every day and my mother is only on 0.5 mg. What is the most helpful response by the nurse?
You Selected:
"Older clients generally need a lesser dose than younger people."
Correct response:
"Older clients generally need a lesser dose than younger people."
Explanation:
Question 20 See full question
After teaching a group of students who are volunteering for a local crisis hotline, the nurse judges that further
education about crisis and intervention is needed when a student makes which statement?
You Selected:
"Most people in crisis will be calling the line once every day for at least a year."
Correct response:
"Most people in crisis will be calling the line once every day for at least a year."
Explanation:
Question 21 See full question
The nurse understands that with the right help at the right time, a client can successfully resolve a crisis and
function better than before the crisis, based primarily on which factor?
You Selected:
acquisition of new coping skills
Correct response:
acquisition of new coping skills
Explanation:
Question 22 See full question
As an angry client becomes more agitated while talking about his problems, the nurse decides to ask for staff
assistance in taking control of the situation when the client demonstrates which behavior?
You Selected:
picking up a pool cue stick and telling the nurse to get out of his way
Correct response:
picking up a pool cue stick and telling the nurse to get out of his way
Explanation:
Question 23 See full question
When a client is about to lose control, the extra staff who come to help commonly stay at a distance from the client
unless asked to move closer by the nurse who is talking to the client. What best explains the primary rationale for
staying at a distance initially?
You Selected:
The client is likely to perceive others as being closer than they are and feel threatened.
Correct response:
The client is likely to perceive others as being closer than they are and feel threatened.
Explanation:
Question 24 See full question
Despite education and role-play practice of restraint procedures, a staff member is injured when actually restraining
a client. When helping the uninjured staff deal with the incident, the nurse should address which factor?
You Selected:
The emotional responses may be similar to those of other crime victims.
Correct response:
The emotional responses may be similar to those of other crime victims.
Explanation:
Question 25 See full question
When preparing to present a community program about women who are victims of physical abuse, which should the
nurse stress about the incidence of battering?
You Selected:
Battering is a major cause of injury to women.
Correct response:
Battering is a major cause of injury to women.
Explanation:
Question 26 See full question
A client suspected of being a victim of abuse returns to the emergency department and, sobbing, tells the nurse, I
guess you really know that my husband beats me and that is why I have bruises all over my body. I do not know
what to do. I am afraid he will kill me one of these times. Which response best demonstrates that the nurse
recognizes the clients needs at this time?
You Selected:
"We can begin by discussing various options open to you."
Correct response:
"We can begin by discussing various options open to you."
Explanation:
Question 27 See full question
A nurse has been working with a battered woman who is being discharged and returning home with her husband.
The nurse says, All this work with her has been useless. She is just going back to him as usual. Which statement
by a nursing colleague would be most helpful to this nurse?
You Selected:
"Her reasons for staying are complex. She can leave only when she is ready and can be safe."
Correct response:
"Her reasons for staying are complex. She can leave only when she is ready and can be safe."
Explanation:
Question 28 See full question
When providing a therapeutic milieu for clients, which intervention would be most appropriate?
You Selected:
Promote optimal functioning of an individual or group.
Correct response:
Promote optimal functioning of an individual or group.
Explanation:
Question 29 See full question
The client has tearfully described her negative feelings about herself to the nurse during their last three
interactions. Which goal would be most appropriate for the nurse to include in the plan of care at this time? The
client will:
You Selected:
verbalize three things she likes about herself
Correct response:
verbalize three things she likes about herself
Explanation:
Question 30 See full question
A client is being admitted to the psychiatric unit. She responds to some of the nurse's questions with one-word
answers. Her eyes are downcast and her movements are very slow. Later that morning, the nurse approaches the
client and asks how she feels about being in the hospital. The client does not respond verbally and continues to
gaze at the floor. Which action should the nurse take first?
You Selected:
Spend time sitting in silence with the client.
Correct response:
Spend time sitting in silence with the client.
Explanation:
Question 31 See full question
The nurse notes that a client is too busy investigating the unit and overseeing the activities of other clients to eat
dinner. To help the client obtain sufficient nourishment, which plan would be best?
You Selected:
Serve foods that she can carry with her.
Correct response:
Serve foods that she can carry with her.
Explanation:
Question 32 See full question
A 5-year-old child exhibits signs of extreme restlessness, short attention span, and impulsiveness. Which
intervention by the nurse would be therapeutic for this child?
You Selected:
Increase the child's sensory stimulation.
Correct response:
Define behaviors that are acceptable and behaviors that are not permitted.
Explanation:
Question 33 See full question
The client is admitted to the hospital because of threatening, aggressive behavior toward the clients family. Which
factor is most important for the nurse to consider when assessing the angry clients potential for violence?
You Selected:
the client's past history of violent behavior
Correct response:
the client's past history of violent behavior
Explanation:
Question 34 See full question
A nurse is planning interventions for a victim of physical abuse. On what principle should the nurse base the plan?
You Selected:
Assessing the client's level of danger is a prerequisite to intervention.
Correct response:
Assessing the client's level of danger is a prerequisite to intervention.
Explanation:
Question 35 See full question
After talking with the nurse, a client admits to being physically abused by her husband. She says that she has never
called the police because her husband has threatened to kill her if she does. She says, I do not want to get him
into trouble, because he is the father of my children. I do not know what to do! Which nursing intervention would
be most therapeutic at this time?
You Selected:
Express concern for the client's safety.
Correct response:
Express concern for the client's safety.
Explanation:
Question 36 See full question
Two days after a client's wife and child were found dead in a flood, the client returns to the crisis center and says he
thinks it would be better to "end it all right now and join my wife and kid, wherever they are." The nurse has already
determined that the client has no history of psychiatric problems. What should the nurse consider this client's risk
for suicide to be?
You Selected:
The risk is high; the clients suicide threat can be considered a call for help and should be taken seriously.
Correct response:
The risk is high; the clients suicide threat can be considered a call for help and should be taken seriously.
Explanation:
Question 37 See full question
A client with a chronic mental illness who does not always take her medications is separated from her husband and
receives public assistance funds. She lives with her mother and older sister and manages her own medication. The
clients mother is in poor health and also receives public assistance benefits. The clients sister works outside the
home, and the clients father is dead. Which issue should the nurse address first?
You Selected:
medication compliance
Correct response:
medication compliance
Explanation:
Question 38 See full question
An experienced nurse is precepting a new nurse in a psychiatric emergency room and is discussing criteria for
involuntary commitment. Which client would signal to the experienced nurse that the new nurse understands the
criteria?
You Selected:
A man who threatens to kill his wife of 38 years
Correct response:
A man who threatens to kill his wife of 38 years
Explanation:
Question 39 See full question
The family members of the victims of a three-car accident have arrived at the emergency department. The wife of
one accident victim is sitting away from the others and crying. Which action by the nurse would be best?
You Selected:
Sit next to the wife and offer her some tissues.
Correct response:
Sit next to the wife and offer her some tissues.
Explanation:
Question 40 See full question
A suicidal client is placed in the seclusion room and given lorazepam because she tried to harm herself by banging
her head against the wall. After 10 minutes, the client starts to bang her head against the wall in the seclusion
room. What action should the nurse take next?
You Selected:
Place the client in restraints.
Correct response:
Place the client in restraints.
Explanation:
Question 41 See full question
An elderly client who has been diagnosed with delusional disorder for many years is exhibiting early symptoms of
dementia. His daughter lives with him to help him manage daily activities, and he attends a day care program for
seniors during the week while she works. A nurse at the day care center hears him say, If my neighbor puts up a
fence, I will blow him away with my shotgun. He has never respected my property line, and I have had it! Which
action should the nurse take?
You Selected:
Report the comment to the neighbor, the daughter, and the police since there is the potential for a criminal act.
Correct response:
Report the comment to the neighbor, the daughter, and the police since there is the potential for a criminal act.
Explanation:
Question 42 See full question
A client who has experienced the loss of her husband through divorce, the loss of her job and apartment, and the
development of drug dependency is suffering situational low self-esteem. Which outcome is most appropriate
initially?
You Selected:
The client will discuss her feelings related to her losses.
Correct response:
The client will discuss her feelings related to her losses.
Explanation:
Question 43 See full question
The health care provider (HCP) refers a client diagnosed with somatization disorder to the outpatient clinic because
of problems with nausea. The clients past symptoms involved back pain, chest pain, and problems with urination.
The client tells the nurse that the nausea began when his wife asked him for a divorce. Which intervention is most
appropriate?
You Selected:
directing the client to describe his feelings about his impending divorce
Correct response:
directing the client to describe his feelings about his impending divorce
Explanation:
Question 44 See full question
A 35-year-old man was experiencing martial discord with his wife of 4 years. When his wife walked out, he became
angry, throwing things and breaking dishes. A friend talked him into seeking help at the local mental health center.
Which question should the nurse ask initially to begin to assess this man's immediate problem?
You Selected:
"What led you to come in for help today?"
Correct response:
"What led you to come in for help today?"
Explanation:
Question 45 See full question
A 26-year-old is being treated for delirium due to acute alcohol intoxication. The client is restless, does not want to
stay seated, and has a staggering gait. What should the nurse do first?
You Selected:
Provide one-to-one supervision of the client until detoxification treatment can begin.
Correct response:
Provide one-to-one supervision of the client until detoxification treatment can begin.
Explanation:
Question 46 See full question
A client is admitted to the psychiatric unit following a suicide attempt. The client has suffered identity theft through
the Internet and states, My savings, checking, and retirement accounts are empty. I have nothing left to pay my
bills or buy food and medicines. The only thing left is to die. After 1 week, the nurse would conclude that the client
has been helped upon hearing which statements? Select all that apply.
You Selected:
"I realize that I still can get monthly public assistance benefits."
"With all the help I got here, I think I may be able to survive after all."
*I filed identity theft claims with the bank, my retirement account, and the government authorities.
Correct response:
"I realize that I still can get monthly public assistance benefits."
*I filed identity theft claims with the bank, my retirement account, and the government authorities.
"With all the help I got here, I think I may be able to survive after all."
Explanation:
Question 47 See full question
The nurse is performing an assessment on a client with an avoidant personality. Which findings should the nurse
anticipate?
You Selected:
Hypersensitivity to negative evaluation and fear of criticism
Correct response:
Hypersensitivity to negative evaluation and fear of criticism
Explanation:
Question 48 See full question
During therapy, a client on the mental health unit is restless and is starting to make sarcastic remarks to others in
the therapy session. The nurse responds by saying, you look angry. Which of the following communication
techniques is the nurse using?
You Selected:
Making observations
Correct response:
Making observations
Explanation:
Question 49 See full question
The nurse is caring for a client who has been physically abused. Which statement by the nurse expresses empathy
for this client?
You Selected:
It must be difficult what you have been going through.
Correct response:
It must be difficult what you have been going through.
Explanation:
Question 50 See full question
While listening to a taped-report at shift change, one of the other team members remarks that My mother lives
near this client, and his yard is always full of junk. What should the nurse assigned to provide care to this client do
in this situation?
You Selected:
Ask the team member what the purpose was in sharing the information.
Correct response:
Ask the team member what the purpose was in sharing the information.
Explanation:
Improve your mastery
Question 1 See full question
A nurse may use self-disclosure with a client if:
You Selected:
it achieves a specific therapeutic goal.
Correct response:
it achieves a specific therapeutic goal.
Explanation:
Question 2 See full question
A client changes topics quickly while relating past psychiatric history. This client's pattern of thinking is called:
You Selected:
flight of ideas.
Correct response:
flight of ideas.
Explanation:
Question 3 See full question
Which action demonstrates the role of the psychiatric nurse in primary prevention?
You Selected:
Providing sexual education classes for adolescents
Correct response:
Providing sexual education classes for adolescents
Explanation:
Question 4 See full question
A client with schizophrenia started risperidone 2 weeks ago. Today, he tells the nurse he feels like he has the flu.
The nurse's assessment reveals the following: temperature 104.4 F (40.2 C), respirations 24 breaths/minute,
blood pressure 130/102 mm Hg, pulse rate 120 beats/minute. The nurse also notes muscle stiffness and pain,
excessive sweating and salivation, and changes in mental status. The nurse suspects the client is experiencing:
You Selected:
neuroleptic malignant syndrome.
Correct response:
neuroleptic malignant syndrome.
Explanation:
Question 5 See full question
A client with antisocial personality disorder smokes in prohibited areas and refuses to follow other unit and facility
rules. The client persuades others to do his laundry and other personal chores, splits the staff, and will work only
with certain nurses. The care plan for this client should focus primarily on:
You Selected:
consistently enforcing unit rules and facility policy.
Correct response:
consistently enforcing unit rules and facility policy.
Explanation:
Question 6 See full question
An agitated client demands to see her chart so she can read what has been written about her. Which statement is
the nurse's best response to the client?
You Selected:
"You have the right to see your chart. Please discuss your wish with your physician."
Correct response:
"You have the right to see your chart. Please discuss your wish with your physician."
Explanation:
Question 7 See full question
Which task may be delegated to a nursing assistant (unregulated care provider) in an acute care mental health
setting?
You Selected:
Checking for sharp objects
Correct response:
Checking for sharp objects
Explanation:
Question 8 See full question
An obese client has returned to the unit after receiving electroconvulsive therapy (ECT). A nurse requests
assistance in moving the client from the stretcher to the bed. Which direction should the nurse give to a nurse who
volunteers to help?
You Selected:
"Obtain the sliding board or two other people to assist us."
Correct response:
"Obtain the sliding board or two other people to assist us."
Explanation:
Question 9 See full question
A nurse is instructing a client with bipolar disorder on proper use of lithium carbonate, the drug's adverse effects,
and symptoms of lithium toxicity. Which client statement indicates that additional teaching is required?
You Selected:
"When my moods fluctuate, I'll increase my dose of lithium."
Correct response:
"When my moods fluctuate, I'll increase my dose of lithium."
Explanation:
Question 10 See full question
Which outcome should the nurse include in the initial plan of care for a client who is exhibiting psychomotor
retardation, withdrawal, minimal eye contact, and unresponsiveness to the nurses questions?
You Selected:
The client will interact with the nurse.
Correct response:
The client will interact with the nurse.
Explanation:
Question 11 See full question
Which reaction to learning about a diagnosis of being HIV positive would put the client at the greatest need of
intervention by the nurse?
You Selected:
A person who says, "I have found a solution for this mess."
Correct response:
A person who says, "I have found a solution for this mess."
Explanation:
Question 12 See full question
A client diagnosed with pain disorder is talking with the nurse about fishing when he suddenly reverts to talking
about the pain in his arm. What should the nurse do next?
You Selected:
Redirect the interaction back to fishing.
Correct response:
Redirect the interaction back to fishing.
Explanation:
Question 13 See full question
Which statement indicates to the nurse that the client is progressing toward recovery from a somatoform disorder?
You Selected:
"I understand my pain will feel worse when I am worried about my divorce."
Correct response:
"I understand my pain will feel worse when I am worried about my divorce."
Explanation:
Question 14 See full question
When planning the care for a client who is being abused, which measure is most important to include?
You Selected:
helping the client develop a safety plan
Correct response:
helping the client develop a safety plan
Explanation:
Question 15 See full question
A client who is neatly dressed and clutching a leather briefcase tightly in his arms scans the adult inpatient unit on
his arrival at the hospital and backs away from the window. The client requests that the nurse move away from the
window. The nurse recognizes that doing as the client requested is contraindicated for which reason?
You Selected:
The action indicates nonverbal agreement with the client's false ideas.
Correct response:
The action indicates nonverbal agreement with the client's false ideas.
Explanation:
Question 16 See full question
The nurse observes a client in a group who is reminiscing about his past. Which effect should the nurse expect
reminiscing to have on the client's functioning in the hospital?
You Selected:
Decrease the client's feelings of isolation and loneliness.
Correct response:
Decrease the client's feelings of isolation and loneliness.
Explanation:
Question 17 See full question
When assessing an aggressive client, which behavior warrants the nurse's prompt reporting and use of safety
precautions?
You Selected:
naming another client as his adversary
Correct response:
naming another client as his adversary
Explanation:
Question 18 See full question
The nurse manager of a psychiatric unit notices that one of the nurses commonly avoids a 75-year-old client's
company. Which factor should the nurse manager identify as being the most likely cause of this nurse's discomfort
with older clients?
You Selected:
recent experiences with her mother's elderly friends
Correct response:
fears and conflicts about aging
Explanation:
Question 19 See full question
Which client statement indicates that the client has gained insight into his use of the defense mechanism of
displacement?
You Selected:
"Now when I am mad at my wife, I talk to her instead of taking it out on the kids."
Correct response:
"Now when I am mad at my wife, I talk to her instead of taking it out on the kids."
Explanation:
Question 20 See full question
A client is admitted to the hospital because of threatening, aggressive behavior toward his family. In the first group
meeting after the client is admitted, another client sits near the nurse and says loudly, "I'm sitting here because I'm
afraid of Ted. He's so big, and I heard him talk about hitting people." The nurse should say to the client:
You Selected:
"It's frightening to have new people on the unit. We're here to talk about things like being afraid."
Correct response:
"It's frightening to have new people on the unit. We're here to talk about things like being afraid."
Explanation:
Question 21 See full question
The treatment team recommends that a client take an assertiveness training class offered in the hospital. Which
behavior indicates that the client is becoming more assertive?
You Selected:
The client asks his roommate to put away his dirty clothes after telling the roommate that this bothers him.
Correct response:
The client asks his roommate to put away his dirty clothes after telling the roommate that this bothers him.
Explanation:
Question 22 See full question
A woman who was raped in her home was brought to the emergency department by her husband. After being
interviewed by the police, the husband talks to the nurse. I do not know why she did not keep the doors locked like
I told her. I cannot believe she has had sex with another man now. The nurse should respond by saying:
You Selected:
Let us talk about how you feel. Maybe it would help to talk to other men who have been through this.
Correct response:
Let us talk about how you feel. Maybe it would help to talk to other men who have been through this.
Explanation:
Question 23 See full question
Assessment of a client who has just been admitted to the inpatient psychiatric unit reveals an unshaven face,
noticeable body odor, visible spots on the shirt and pants, slow movements, gazing at the floor, and a flat affect.
Which of the following should the nurse interpret as indicating psychomotor retardation?
You Selected:
Slow movements.
Correct response:
Slow movements.
Explanation:
Question 24 See full question
In the emergency department, a client reveals to the nurse a lethal plan for committing suicide and agrees to a
voluntary admission to the psychiatric unit. Which information would the nurse discuss with the client to answer the
question How long do I have to stay here? Select all that apply.
You Selected:
You may leave the hospital at any time unless youre suicidal or homicidal or unable to meet your basic needs.
Lets talk more after the health care team has assessed you.
Because you have stated that you want to hurt yourself, you must be safe before being discharged.
Correct response:
You may leave the hospital at any time unless youre suicidal or homicidal or unable to meet your basic needs.
Lets talk more after the health care team has assessed you.
Because you have stated that you want to hurt yourself, you must be safe before being discharged.
Explanation:
Question 25 See full question
Which of the following client behaviors indicates the nurse-client relationship is in the working phase?
You Selected:
The client tries to summarize his or her progress in the relationship.
Correct response:
The client makes an effort to describe his or her problems in detail.
Explanation:
Question 26 See full question
When a client expresses feelings of unworthiness, the nurse should respond by saying:
;
You Selected:
"As you begin to feel better, your feelings of unworthiness will begin to disappear."
Correct response:
"As you begin to feel better, your feelings of unworthiness will begin to disappear."
Explanation:
Question 27 See full question
The nurse correctly judges that the danger of a suicide attempt is greatest with which client behavior?
You Selected:
at the point of deepest despair
Correct response:
increase in energy level
Explanation:
Question 28 See full question
The client goes to her room and slams the door immediately after the first family therapy session. Later she tells
the nurse, I am so mad. The therapist did not let me tell my side of the story. He just agreed with everything my
parents said. Which nursing action would be most therapeutic in this situation?
You Selected:
Allow the client to continue to ventilate her feelings to the nurse.
Correct response:
Redirect the client to the therapist to tell him how she feels.
Explanation:
Question 29 See full question
A client scans the adult inpatient unit on arrival at the hospital. The client is neatly dressed and clutches a leather
briefcase. The client refuses to let the nurse touch the briefcase to check it for valuables or contraband. Which
action by the nurse would be best?
You Selected:
Tell the client that he must follow hospital policy if he wishes to stay.
Correct response:
Ask the client to open the briefcase and describe its contents.
Explanation:
Question 30 See full question
As the nurse stands near the window in the clients room, the client shouts, Come away from the window! They will
see you! Which response by the nurse would be best?
You Selected:
"You have no reason to be afraid."
Correct response:
"Who are 'they?"
Explanation:
Question 31 See full question
A client suddenly behaves in an impulsive, hyperactive, unpredictable manner. Which approach would be best for
the nurse to use first if the client becomes violent?
You Selected:
Get help to handle the situation safely.
Correct response:
Get help to handle the situation safely.
Explanation:
Question 32 See full question
Which approach by the nurse would most likely foster a therapeutic relationship with a client who tries to
manipulate people?
You Selected:
consistency
Correct response:
consistency
Explanation:
Question 33 See full question
The client, who is dying from acquired immunodeficiency syndrome (AIDS), is admitted to the inpatient psychiatric
unit because he attempted suicide. His close friend recently died from AIDS. The client states to the nurse, What is
the use of living? My time is running out. What is the nurses best response?
You Selected:
You are in a lot of pain. What are you feeling?
Correct response:
You are in a lot of pain. What are you feeling?
Explanation:
Question 34 See full question
When working with a client who has a mental illness and the clients family, which approach will be most effective?
You Selected:
Convey warmth and acceptance to each family member.
Correct response:
Convey warmth and acceptance to each family member.
Explanation:
Question 35 See full question
Which statement is the best wording of a no-harm, no-suicide contract?
You Selected:
"I will not accidentally or purposely kill myself during the next 24 hours."
Correct response:
"I will not accidentally or purposely kill myself during the next 24 hours."
Explanation:
Question 36 See full question
The decision is made to involuntarily admit a client to a psychiatric hospital on an emergency detention. The nurse
explains the involuntary hospitalization process to the client. Which of the following statements made by the nurse
would not be accurate about the involuntary admission process?
You Selected:
"You cannot have any visitors while you're here involuntarily."
Correct response:
"You cannot have any visitors while you're here involuntarily."
Explanation:
Question 37 See full question
A client is admitted to the inpatient psychiatric unit. He is unshaven, has body odor, and has spots on his shirt and
pants. He moves slowly, gazes at the floor, and has a flat affect. When assessing the client on admission, the nurse
should first ask the client:
You Selected:
if he is thinking about hurting himself.
Correct response:
if he is thinking about hurting himself.
Explanation:
Question 38 See full question
A successful real estate agent brought to the clinic after being arrested for harassing and stalking his ex-wife denies
any other symptoms or problems except anger about being arrested. The ex-wife reports to the police, He is fine
except for this irrational belief that we will remarry. When collaborating with the health care provider (HCP) about a
plan of care, which intervention would be most effective for the client at this time?
You Selected:
referral to an outpatient therapist
Correct response:
referral to an outpatient therapist
Explanation:
Question 39 See full question
A client has been diagnosed with Avoidant Personality Disorder. He reports loneliness, but has fears about making
friends. He also reports anxiety about being rejected by others. In a long-term treatment plan, in what order, from
first to last, should the nurse list goals for the client? All options must be used.
You Selected:
Talk with the client about his self-esteem and his fears.
Teach the client anxiety management and social skills.
Ask the client to join one of his chosen activities with the nurse and two other clients.
Help the client make a list of small group activities at the center he would find interesting.
Correct response:
Talk with the client about his self-esteem and his fears.
Teach the client anxiety management and social skills.
Help the client make a list of small group activities at the center he would find interesting.
Ask the client to join one of his chosen activities with the nurse and two other clients.
Explanation:
Question 40 See full question
A client who has had AIDS for years is being treated for a serious episode of pneumonia. A psychiatric nurse consult
was arranged after the client stated that he was tired of being in and out of the hospital. I am not coming in here
anymore. I have other options. The nurse would evaluate the psychiatric nurse consult as helpful if the client
makes which statements?
You Selected:
"I realize that I really do have more time to enjoy my friends and family."
Correct response:
"I realize that I really do have more time to enjoy my friends and family."
Explanation:
Question 41 See full question
The nurse working at the site of a severe flood sees a woman, standing in knee-deep water, staring at an empty lot.
The woman states, I keep thinking that this is a nightmare and that I will wake up and see that my house is still
there. Which crisis intervention strategies are most needed at this time? Select all that apply.
You Selected:
Refer her to the shelter for dry clothes and food.
Assess her for risk of suicide and other signs of decompensation.
Determine if any of her family are injured or missing.
Tell her that groups are being formed at the shelter for flood survivors.
Ask the client about any physical injuries she may have.
Allow the client to talk about her fears, anger, and other feelings.
Correct response:
Ask the client about any physical injuries she may have.
Determine if any of her family are injured or missing.
Allow the client to talk about her fears, anger, and other feelings.
Assess her for risk of suicide and other signs of decompensation.
Explanation:
Question 42 See full question
A client is brought to the emergency department (ED) by a friend who states that the client recently ran out of his
lorazepam and has been having a grand mal seizure for the last 10 minutes. The nurse observes that the client is
still seizing. What should the nurse do in order of priority from first to last? All options must be used.
You Selected:
Monitor the clients safety, and place seizure pads on the cart rails.
Page the ED health care provider (HCP) and prepare to give diazepam intravenously.
Ask the friend about the clients medical history and current medications.
Record the time, duration, and nature of the seizures.
Correct response:
Page the ED health care provider (HCP) and prepare to give diazepam intravenously.
Monitor the clients safety, and place seizure pads on the cart rails.
Record the time, duration, and nature of the seizures.
Ask the friend about the clients medical history and current medications.
Explanation:
Question 43 See full question
A nurse assesses a client with psychotic symptoms and determines that the client likely poses a safety threat and
needs vest restraints. The client is adamantly opposed to this. What would be the best nursing action?
You Selected:
Contact the physician and obtain necessary orders.
Correct response:
Contact the physician and obtain necessary orders.
Explanation:
Question 44 See full question
Detention center staff asked for a mental health evaluation of a 21-year-old woman after she stabbed herself with a
fork and woke from nightmares in fits of rage. The evaluation revealed that she was kidnapped and held from ages
8 to 16 by a convicted child pornographer. She said she never contacted her family after her release from captivity.
In what order of priority from first to last should the nurse implement the steps? All options must be used.
You Selected:
Initiate suicide precautions and a no-harm contract.
Encourage safe verbalizations of her emotions, especially anger.
Offer empathy and support, and be nonjudgmental and honest with her.
Ask the client if she wishes to contact her family while hospitalized.
Correct response:
Initiate suicide precautions and a no-harm contract.
Offer empathy and support, and be nonjudgmental and honest with her.
Encourage safe verbalizations of her emotions, especially anger.
Ask the client if she wishes to contact her family while hospitalized.
Explanation:
Question 45 See full question
A nurse overhears a second nurse making plans to meet a hospitalized client for a drink after the client has been
discharged. Which of the following is the best action for the first nurse to take?
You Selected:
Report the conversation to the nurse manager.
Correct response:
Discuss the conversation directly with the other nurse.
Explanation:
Question 46 See full question
The nurse is performing an assessment on a client after her third electroconvulsive therapy (ECT). Which finding
should she anticipate most frequently?
You Selected:
Short-term memory loss
Correct response:
Short-term memory loss
Explanation:
Question 47 See full question
A nurse is caring for a client who has been hospitalized with schizophrenia. The client has had this disorder for 8
years and is now displaying regression, increased disorganization and inappropriate social interactions. Which
nursing intervention will best help this client meet self-care needs?
You Selected:
Provide complete bathing and grooming tasks for client.
Correct response:
Provide client with assistance in hygiene, grooming, and dressing.
Explanation:
Question 48 See full question
The nurse is performing an admission interview when the client attempts to shift the session focus to the nurse by
asking personal questions. Which statement by the nurse is most appropriate?
You Selected:
I have a family. Tell me about you and your family.
Correct response:
I have a family. Tell me about you and your family.
Explanation:
Question 49 See full question
A nurse is teaching a client stress management. Which of the following techniques would be considered adaptive
coping skills? Select all that apply.
You Selected:
Set realistic goals for each day
Practice relaxation techniques
Maintain control of my life
Balance sleep, rest, and exercise
Correct response:
Set realistic goals for each day
Practice relaxation techniques
Balance sleep, rest, and exercise
Explanation:
Question 50 See full question
During a unit meeting attended by clients and staff, several clients are criticizing their primary nurses. These clients
have also been intimidating two other clients who have recently been admitted to the unit, and now the new clients
have stopped sharing their opinions during the meeting. What is the first action for the nurse to take?
You Selected:
Ask the clients criticizing their nurses to suggest some possible solutions for the practices they are criticizing.
Correct response:
Ask the clients criticizing their nurses to suggest some possible solutions for the practices they are criticizing.
Explanation:
Improve your mastery
TAKE A PRACTICE QUIZ
Question 1 See full question
Which commonly administered psychiatric medication is prescribed in individualized dosages according to the blood
levels of the drug?
You Selected:
Lithium carbonate
Correct response:
Lithium carbonate
Explanation:
Question 2 See full question
A nurse is developing a care plan for a client who has undergone electroconvulsive therapy (ECT). The nurse should
include which intervention?
You Selected:
Reorienting the client to time and place
Correct response:
Reorienting the client to time and place
Explanation:
Question 3 See full question
A client becomes angry and belligerent toward the nurse after speaking on the phone with his mother. The nurse
recognizes this as what defense mechanism?
You Selected:
Displacement
Correct response:
Displacement
Explanation:
Question 4 See full question
When teaching an unlicensed assistive personnel (UAP) new to the unit about the principles for the care of a client
diagnosed with a personality disorder, the nurse should explain that:
You Selected:
the clients are accepted although their behavior may not be.
Correct response:
the clients are accepted although their behavior may not be.
Explanation:
Question 5 See full question
A client is to be discharged from an alcohol rehabilitation program. What should the nurse emphasize in the
discharge plan as a priority?
You Selected:
follow-up care
Correct response:
follow-up care
Explanation:
Question 6 See full question
A client with suspected abuse describes her husband as a good man who works hard and provides well for his
family. She does not work outside the home and states that she is proud to be a wife and mother just like her own
mother. The nurse interprets the family pattern described by the client as best illustrating which characteristic of
abusive families?
You Selected:
role stereotyping
Correct response:
role stereotyping
Explanation:
Question 7 See full question
Based on a client's history of violence toward others and inability to cope with anger, what should the nurse use as
the most important indicator of goal achievement before discharge?
You Selected:
verbalization of feelings in an appropriate manner
Correct response:
verbalization of feelings in an appropriate manner
Explanation:
Question 8 See full question
Which finding indicates that a client who has been raped will have future adjustment problems and need additional
counseling?
You Selected:
Her parents show shame and suspicion about her part in the rape.
Correct response:
Her parents show shame and suspicion about her part in the rape.
Explanation:
Question 9 See full question
During the conversation with the nurse, a victim of physical abuse says, Let me try to explain why I stay with my
husband. Which response would the nurse find inconsistent with the profile of a battered partner?
You Selected:
"The abuse adds spice to our relationship."
Correct response:
"The abuse adds spice to our relationship."
Explanation:
Question 10 See full question
A newly admitted client describes her mission in life as one of saving her son by eliminating the "provocative sluts"
of the world. There are several attractive young women on the unit. What should the nurse do first?
You Selected:
Ask the client to inform the staff if she has negative thoughts about other clients.
Correct response:
Ask the client to inform the staff if she has negative thoughts about other clients.
Explanation:
Question 11 See full question
A nurse working at an outpatient mental health center primarily with chronically mentally ill clients receives a
telephone call from the mother of a client who lives at home. The mother reports that the client has not been taking
her medication and now is refusing to go to the work center where she has worked for the past year. What should
the nurse do first?
You Selected:
Ask to speak to the client directly on the phone.
Correct response:
Ask to speak to the client directly on the phone.
Explanation:
Question 12 See full question
A client is irritable and hostile. He becomes agitated and verbally lashes out when his personal needs are not
immediately met by the staff. When the clients request for a pass is refused by the healthcare provider, he utters a
stream of profanities. Which statement best describes the clients behavior?
You Selected:
The client's anger is not intended personally.
Correct response:
The client's anger is not intended personally.
Explanation:
Question 13 See full question
A client has been diagnosed with Avoidant Personality Disorder. He reports loneliness, but has fears about making
friends. He also reports anxiety about being rejected by others. In a long-term treatment plan, in what order, from
first to last, should the nurse list goals for the client? All options must be used.
You Selected:
Talk with the client about his self-esteem and his fears.
Teach the client anxiety management and social skills.
Help the client make a list of small group activities at the center he would find interesting.
Ask the client to join one of his chosen activities with the nurse and two other clients.
Correct response:
Talk with the client about his self-esteem and his fears.
Teach the client anxiety management and social skills.
Help the client make a list of small group activities at the center he would find interesting.
Ask the client to join one of his chosen activities with the nurse and two other clients.
Explanation:
Question 14 See full question
A client is being admitted to a psychiatric outpatient program for counseling for his ongoing emotional symptoms.
He is asked to rate the severity of his depression, anxiety, and anger. He states, I do not have any anger any more.
I lost my temper once and nearly hurt my wife. I never got angry again. In which order of priority from first to last
should the principles related to anger be shared with this client? All options must be used.
You Selected:
"Anger is a natural emotion occurring in all human relationships."
"Unexpressed anger has a negative effect on the human body and mind."
"Holding your anger inside contributes to your depression."
"You can learn effective ways to discuss anger with others and still maintain control."
Correct response:
"Anger is a natural emotion occurring in all human relationships."
"Unexpressed anger has a negative effect on the human body and mind."
"Holding your anger inside contributes to your depression."
"You can learn effective ways to discuss anger with others and still maintain control."
Explanation:
Question 15 See full question
A 19-year-old male with cystic fibrosis (CF) is hospitalized for a serious lung infection and is in need of a lung
transplant. However, he has a rare blood type that complicates the process of obtaining a donor organ. He has also
been diagnosed with bipolar disorder and treated successfully since mid-adolescence with medication and therapy.
The client requests to see a chaplain to help him make plans for a funeral and donation of his body to science after
death. How should the nurse interpret the clients request?
You Selected:
It is a signal of the depressive side of his bipolar disorder, and he should be checked for suicidal thoughts/plans.
Correct response:
It is a signal of the client's growing awareness that he is likely to have a shortened lifespan and should be
supported by unit staff.
Explanation:
Question 16 See full question
A nurse in a psychiatric care unit finds that a client with psychosis has become violent and has struck another client
in the unit. What action should the nurse take in this case?
You Selected:
Restrain the client, as he is harmful to the other clients.
Correct response:
Restrain the client, as he is harmful to the other clients.
Explanation:
Question 17 See full question
Detention center staff asked for a mental health evaluation of a 21-year-old woman after she stabbed herself with a
fork and woke from nightmares in fits of rage. The evaluation revealed that she was kidnapped and held from ages
8 to 16 by a convicted child pornographer. She said she never contacted her family after her release from captivity.
In what order of priority from first to last should the nurse implement the steps? All options must be used.
You Selected:
Initiate suicide precautions and a no-harm contract.
Offer empathy and support, and be nonjudgmental and honest with her.
Encourage safe verbalizations of her emotions, especially anger.
Ask the client if she wishes to contact her family while hospitalized.
Correct response:
Initiate suicide precautions and a no-harm contract.
Offer empathy and support, and be nonjudgmental and honest with her.
Encourage safe verbalizations of her emotions, especially anger.
Ask the client if she wishes to contact her family while hospitalized.
Explanation:
Question 18 See full question
The nurse is performing an assessment on a client with a history of a dysfunctional family. Which findings should
the nurse anticipate? Select all that apply.
You Selected:
Abuse and neglect
Unhealthy personal boundaries
Correct response:
Unhealthy personal boundaries
Abuse and neglect
Explanation:
Question 19 See full question
Which client outcome best indicates effective mental health care coordination when the nurse uses a client-
centered approach?
You Selected:
The client is compliant with the treatment plan.
Correct response:
Preferred client outcomes vary from client to client.
Explanation:
Question 20 See full question
A client who is taking olanzapine states he is being poisoned and refuses to take his scheduled medication. The
nurse states, If you do not take your medication, you will be put into seclusion. The nurses statement is an
example of which legal concept?
You Selected:
assault
Correct response:
assault
Explanation:
Question 1 See full question
A client with severe and persistent depression can't decide if he'll undergo electroconvulsive therapy (ECT). His
family asks a nurse to convince him that this treatment modality would be beneficial. In educating the family about
the client's situation, what statement about client rights should the nurse make?
You Selected:
"The client, treatment team, and family must meet to discuss this treatment option."
Correct response:
"The client, treatment team, and family must meet to discuss this treatment option."
Explanation:
Question 2 See full question
When assessing a client for suicidal risk, which method of suicide should the nurse identify as most lethal?
You Selected:
use of a gun
Correct response:
use of a gun
Explanation:
Question 3 See full question
A client is being discharged after 3 days of hospitalization for a suicide attempt that followed the receipt of a
divorce notice. Which client finding indicates to the nurse that the client is ready for discharge?
You Selected:
Has a list of support persons and community resources.
Correct response:
Has a list of support persons and community resources.
Explanation:
Question 4 See full question
The client is suspicious of staff members and other clients. To help establish a therapeutic relationship with the
client, which plan would be best?
You Selected:
Allow the client to initiate conversations when he feels ready for them.
Correct response:
Spend brief intervals with the client each day.
Explanation:
Question 5 See full question
A client is brought to the mental health center by a police officer for an evaluation because she has been bothering
other people when she eats in the hotel restaurant. She denies this, will not give her name, and holds tightly to her
purse. She refuses to talk to anyone except to say, You have no right to keep me here. I have money, and I can
take care of myself. Which factor would be most relevant to a decision about this clients disposition?
You Selected:
She is not known to the mental health center.
Correct response:
She seems able to care for herself.
Explanation:
Question 6 See full question
The family of an older adult wants their mother to have counseling for depression. During the initial nursing
assessment, the client denies the need for counseling. Which comment by the client supports the fact that the
client may not need counseling?
You Selected:
Since I have gotten over the death of my husband, I have had more energy and been more active than before he
died.
Correct response:
Since I have gotten over the death of my husband, I have had more energy and been more active than before he
died.
Explanation:
Question 7 See full question
A nurse is assessing a client at a mental health clinic who threatens suicide and describes having a plan. Which of
the following should the nurse recognize as the priority goal for the client?
You Selected:
Obtaining admission to an acute care facility
Correct response:
Working with the client to resolve the immediate crisis
Explanation:
Question 8 See full question
A man brings his wife to the emergency department. He reports that since the death of their 7-month-old daughter
8 weeks earlier, his wife has been neglecting her housework and family, has lost 20 lb (9.1 kg), and has not left the
house. Which of the following additional assessment findings would indicate to the nurse that the client may be
experiencing extreme depression? Select all that apply.
You Selected:
Inconsolable weeping
Speaking in soft monotone voice
Obvious neglect of personal hygiene
Correct response:
Obvious neglect of personal hygiene
Speaking in soft monotone voice
Inconsolable weeping
Explanation:
Question 9 See full question
A client with depression has not responded to drug therapy. At a team conference, staff members recommend
electroconvulsive therapy (ECT). Which statement should the nurse add when explaining the procedure to the
client?
You Selected:
Your healthcare provider has decided electroconvulsive therapy is the best course of action for you.
Correct response:
This treatment has been proven to be effective, and we expect a positive outcome.
Explanation:
Question 10 See full question
What should be charted by the nurse when the client has an involuntary commitment or formal admission status?
You Selected:
The clients receipt of information about status and rights should be charted.
Correct response:
The clients receipt of information about status and rights should be charted.
Explanation:
Question 1 See full question
A nurse is assessing a client with bipolar disorder. Findings include coarse hand tremors, muscle twitching, and
mental confusion. These findings suggest:
You Selected:
lithium toxicity.
Correct response:
lithium toxicity.
Explanation:
Question 2 See full question
The nurse correctly judges that the danger of a suicide attempt is greatest with which client behavior?
You Selected:
increase in energy level
Correct response:
increase in energy level
Explanation:
Question 3 See full question
A nurse is counseling a married woman who has two children under 4 years of age and is a victim of spousal abuse.
Before the client leaves the clinic, what is the most important thing the nurse should do?
You Selected:
Help the client develop a safety plan.
Correct response:
Help the client develop a safety plan.
Explanation:
Question 4 See full question
A female client is admitted to a mental health unit with a diagnosis of depression and is participating in group
sessions. She asks a male nurse if he is married or has a girlfriend. What is the best response by the nurse to
maintain a therapeutic relationship?
You Selected:
Im curious about your question, but I want to know how you are feeling today.
Correct response:
Im curious about your question, but I want to know how you are feeling today.
Explanation:
Question 5 See full question
A nurse is caring for a client who has been hospitalized with schizophrenia. The client has had this disorder for 8
years and is now displaying regression, increased disorganization and inappropriate social interactions. Which
nursing intervention will best help this client meet self-care needs?
You Selected:
Provide client with assistance in hygiene, grooming, and dressing.
Correct response:
Provide client with assistance in hygiene, grooming, and dressing.
Explanation:

Question 1 See full question


When should a nurse introduce information about the end of the nurse-client relationship?

You Selected:
During the orientation phase

Correct response:
During the orientation phase
Explanation:

Question 2 See full question


When preparing to use seclusion as an alternative to restraint for a client who has not yet lost control, the nurse
expects to use a room with limited furniture and no access to dangerous articles. What should the nurse also
consider as critical for the safety of the client?

You Selected:
a security window in the door or a room camera

Correct response:
a security window in the door or a room camera

Explanation:

Question 3 See full question


According to hospital protocol, after a client is restrained, the staff meet and discuss the restraint situation. In
addition to sharing feelings and offering support, what should the nurse identify as the long-term goal for the
debriefing?

You Selected:
improving the staff's use of restraint procedures

Correct response:
improving the staff's use of restraint procedures

Explanation:

Question 4 See full question


A client in an inpatient psychiatric unit tells the nurse, Im going to divorce my no-good husband. I hope he rots in
hell. But I miss him so bad. I love him. When is he going to come get me out of here? The nurse interprets the
clients statements as indicative of which condition?

You Selected:
ambivalence

Correct response:
ambivalence

Explanation:

Question 5 See full question


A nurse makes a home visit to a client who was discharged from a psychiatric hospital. The client is irritable and
walks about her room slowly and gloomily. After 10 minutes, the nurse prepares to leave, but the client plucks at
the nurses sleeve and quickly asks for help rearranging her belongings. She also anxiously makes inconsequential
remarks to keep the nurse with her. In view of the fact that the client has previously made a suicidal gesture, which
intervention by the nurse should be a priority at this time?

You Selected:
Ask the client frankly if she has suicidal thoughts or plans.

Correct response:
Ask the client frankly if she has suicidal thoughts or plans.
Explanation:

Question 6 See full question


A mute client begins to express herself verbally on occasion. Which nursing action should be credited with helping a
mute client express herself verbally?

You Selected:
making open-ended statements followed with silence

Correct response:
making open-ended statements followed with silence

Explanation:

Question 7 See full question


A nurse is assessing a client at a mental health clinic who threatens suicide and describes having a plan. Which of
the following should the nurse recognize as the priority goal for the client?

You Selected:
Working with the client to resolve the immediate crisis

Correct response:
Working with the client to resolve the immediate crisis

Explanation:

Question 8 See full question


A nurse is teaching self-esteem to a client. Which statements by the client would indicate understanding of the
concept? Select all that apply.

You Selected:
"Living in a critical environment is not good for me."
"I need to have consistent limits."
"I need to have healthy boundaries."

Correct response:
"I need to have consistent limits."
"Living in a critical environment is not good for me."
"I need to have healthy boundaries."

Explanation:

Question 9 See full question


A 74-year-old client receiving fluphenazine decanoate therapy develops pseudoparkinsonism, and is ordered
amantadine hydrochloride. With the addition of this medication, the client reports feeling dizzy when standing.
Which response by the nurse is best?

You Selected:
When you change positions, do so slowly.

Correct response:
When you change positions, do so slowly.
Explanation:

Question 10 See full question


A nurse hears a client state, I have had it with this marriage. It would be so much easier to just hire someone to kill
my husband! What action should the nurse take?

You Selected:
An assessment of the clients response to treatment must be performed.

Correct response:
The nurse must start the process to warn the clients husband.

Explanation:

Question 1 See full question


A nurse is assessing a client who has just been admitted to the emergency department. Which signs suggest an
overdose of an antianxiety agent?
You Selected:
Slurred speech, dyspnea, and impaired coordination
Correct response:
Slurred speech, dyspnea, and impaired coordination
Explanation:
Question 2 See full question
A client has been taking imipramine, 125 mg by mouth daily, for 1 week. Now the client reports wanting to stop
taking the medication because he still feels depressed. At this time, what is the nurse's best response?
You Selected:
"Because imipramine must build to a therapeutic level, it may take 2 to 3 weeks to reduce depression."
Correct response:
"Because imipramine must build to a therapeutic level, it may take 2 to 3 weeks to reduce depression."
Explanation:
Question 3 See full question
After a period of unsuccessful treatment with amitriptyline, a woman diagnosed with depression is switched to
tranylcypromine. Which statement by the client indicates the client understands the side effects of
tranylcypromine?
You Selected:
"I must refrain from eating aged cheese or yeast products."
Correct response:
"I must refrain from eating aged cheese or yeast products."
Explanation:
Question 4 See full question
A client diagnosed with major depression spends most of the day lying in bed with the sheet pulled over his head.
Which approach by the nurse is most therapeutic?
You Selected:
Initiate contact with the client frequently.
Correct response:
Initiate contact with the client frequently.
Explanation:
Question 5 See full question
The client with bipolar disorder, manic phase, appears at the nurse's station wearing a transparent shirt, miniskirt,
high heels, 10 bracelets, and eight necklaces. Her makeup is overdone and she is not wearing underwear. The nurse
should:
You Selected:
Escort the client to her room and assist with choosing appropriate attire.
Correct response:
Escort the client to her room and assist with choosing appropriate attire.
Explanation:
Question 6 See full question
The wife of a client with bipolar disorder, manic phase, states to the nurse, "He is acting so crazy. What did he do to
get this way?" The nurse bases the response on which understanding of this disorder?
You Selected:
It is the result of an imbalance of chemicals in the brain.
Correct response:
It is the result of an imbalance of chemicals in the brain.
Explanation:
Question 7 See full question
While assessing a client diagnosed with dementia, the nurse notes that her husband is concerned about what he
should do when she uses vulgar language with him. The nurse should:
You Selected:
say nothing and leave the room.
Correct response:
ignore the vulgarity and distract her.
Explanation:
Question 8 See full question
The health care provider (HCP) prescribes a serum lithium level tomorrow for a client with bipolar disorder, manic
phase, who has been receiving lithium 300 mg PO three times daily for the past 5 days. At what time should the
nurse plan to have the blood specimen obtained?
You Selected:
before breakfast
Correct response:
before breakfast
Explanation:
Question 9 See full question
The family of a client diagnosed with Alzheimer's disease wants to keep the client at home. They say that they have
the most difficulty in managing his wandering. What should the nurse instruct the family to do? Select all that apply.
You Selected:
Install door alarms and high door locks.
Install motion and sound detectors.
Have a relative sit with the client all night.
Have the client wear a Medical Alert bracelet.
Correct response:
Install motion and sound detectors.
Have the client wear a Medical Alert bracelet.
Install door alarms and high door locks.
Explanation:
Question 10 See full question
A newly admitted client is extremely hostile toward a staff member without apparent reason. According to Freudian
theory, the nurse would suspect that the client is exhibiting which phenomena?
You Selected:
Transference
Correct response:
Transference
Explanation:
Question 1 See full question
A client with major depression is frequently irritable, abrasive, and uncooperative and refuses to participate in
group activities. When working with this client, the nurse should use which approach?
You Selected:
Firmness
Correct response:
Firmness
Explanation:
Question 2 See full question
A nurse is assigned to care for a recently admitted client who has attempted suicide. What should the nurse do?
You Selected:
Search the client's belongings and his room carefully for items that could be used to attempt suicide.
Correct response:
Search the client's belongings and his room carefully for items that could be used to attempt suicide.
Explanation:
Question 3 See full question
A client with a diagnosis of major depression is ordered clonazepam for agitation in addition to an antidepressant.
Client teaching should include which statement?
You Selected:
Clonazepam may have a slight depressant effect.
Correct response:
Clonazepam may have a slight depressant effect.
Explanation:
Question 4 See full question
A nurse is caring for a severely depressed client who is barely functioning. The priority nursing goal for this client
would be to:
You Selected:
assess for and maintain adequate nutrition and hydration.
Correct response:
assess for and maintain adequate nutrition and hydration.
Explanation:
Question 5 See full question
A client was admitted to the inpatient unit 3 days ago with a flat affect, psychomotor retardation, anorexia,
hopelessness, and suicidal ideation. The health care provider (HCP) prescribed 75 mg of venlafaxine extended
release to be given every morning. The client interacted minimally with the staff and spent most of the day in his
room. As the nurse enters the unit at the beginning of the evening shift, the client is smiling and cheerful and
appears to be relaxed. What should the nurse interpret as the most likely cause of the clients behavior?
You Selected:
The client's sudden improvement calls for close observation by the staff.
Correct response:
The client's sudden improvement calls for close observation by the staff.
Explanation:
Question 6 See full question
A client with bipolar disorder, manic phase, is scheduled for a chest radiograph. Before taking the client to the
radiology department, the nurse should:
You Selected:
call security to be on standby for possible problems.
Correct response:
explain the procedure in simple terms.
Explanation:
Question 7 See full question
The client with Alzheimer's disease has been prescribed donepezil 5 mg at bedtime. Which instruction should the
nurse give to the client's daughter?
You Selected:
Avoid suddenly stopping the medication.
Correct response:
Avoid suddenly stopping the medication.
Explanation:
Question 8 See full question
The nurse attempts to interact with a client who barely responds with yes or no. The client states, "Do not bother
me. I want to die." The nurse should:
You Selected:
sit with the client for 10 minutes.
Correct response:
sit with the client for 10 minutes.
Explanation:
Question 9 See full question
The nurse is caring for a client with bipolar disorder who was recently admitted to an inpatient unit and is
experiencing a manic episode. What is a priority nursing intervention for this client?
You Selected:
Closely monitor the clients eating and sleeping habits.
Correct response:
Closely monitor the clients eating and sleeping habits.
Explanation:
Question 10 See full question
In assessing an adolescent client at an outpatient clinic, the nurse is able to recognize that depression in
adolescents is often which of the following?
You Selected:
Often masked by aggressive behaviors
Correct response:
Often masked by aggressive behaviors
Explanation:
Question 1 See full question
A client in the manic phase of bipolar disorder constantly belittles other clients and demands special favors from
the nurses. Which nursing intervention is most appropriate for this client?
You Selected:
Set limits with consequences for belittling or demanding behavior.
Correct response:
Set limits with consequences for belittling or demanding behavior.
Explanation:
Question 2 See full question
During an interaction with a nurse, a client with bipolar disorder states that she doesn't have anything to contribute
to the art therapy group. On exploration of the client's concerns, the nurse recognizes the client's pattern of
withdrawal and nonparticipation in situations requiring her to communicate with others. Which nursing diagnosis is
appropriate for this client?
You Selected:
Impaired social interaction
Correct response:
Impaired social interaction
Explanation:
Question 3 See full question
During a group session, a client who is depressed tells the group that he lost his job. Which response by the nurse is
best?
You Selected:
"It must have been very upsetting for you."
Correct response:
"It must have been very upsetting for you."
Explanation:
Question 4 See full question
A grandson calls the crisis center expressing concern about his grandmother, who lost her husband a month ago.
He states, She has been in bed for a week and is not eating or showering. She told me that she did not want to kill
herself, but it is not like her to do nothing for herself. She will not even talk to me when I visit her. The nurse
encourages the grandson to bring his grandmother to the center for evaluation based on which reason?
You Selected:
The behaviors may reflect passive suicidal thoughts.
Correct response:
The behaviors may reflect passive suicidal thoughts.
Explanation:
Question 5 See full question
The nurse is counseling a client regarding treatment of the client's newly diagnosed depression. The nurse
emphasizes that full benefit from antidepressant therapy usually takes how long?
You Selected:
2 to 4 weeks
Correct response:
2 to 4 weeks
Explanation:
Question 6 See full question
A 72-year-old female client is brought by ambulance to the hospitals psychiatric unit from a nursing home where
she has been a client for 3 months. Transfer data indicate that she has become increasingly confused and
disoriented. In which way should the hospital admission process be modified for the client?
You Selected:
Allow her sufficient extra time in which to gain an understanding of what is happening to her.
Correct response:
Allow her sufficient extra time in which to gain an understanding of what is happening to her.
Explanation:
Question 7 See full question
The health care provider (HCP) prescribes mirtazapine 30 mg PO at bedtime for a client diagnosed with depression.
The nurse should:
You Selected:
question the HCP's prescription.
Correct response:
give the medication as prescribed.
Explanation:
Question 8 See full question
The client with acute mania has been admitted to the inpatient unit voluntarily. The nurse approaches the client
with medication to be taken orally as prescribed by the health care provider (HCP). The client states, "I do not need
that stuff." Which response by the nurse is best?
You Selected:
"The medication will help you feel calmer."
Correct response:
"The medication will help you feel calmer."
Explanation:
Question 9 See full question
The nurse is admitting a client with a history of bipolar mania. Which of the following assessment findings is the
priority when developing a plan of care?
You Selected:
Hyperactivity, ignoring eating, and sleeping
Correct response:
Hyperactivity, ignoring eating, and sleeping
Explanation:
Question 10 See full question
A couple informs the nurse that they have been having some problems in the bedroom. What is the most
appropriate response by the nurse?
You Selected:
"I would like to hear your concerns."
Correct response:
"I would like to hear your concerns."
Explanation:
Question 1 See full question
A client with acute mania exhibits euphoria, pressured speech, and flight of ideas. The client has been talking to the
nurse nonstop for 5 minutes and lunch has arrived on the unit. What should the nurse do next?
You Selected:
Do not interrupt the client, but wait for him to finish talking.
Correct response:
Excuse oneself while telling the client to come to the dining room for lunch.
Explanation:
The nurse would request to be excused, showing respect and regard for the client, while telling the client to come to
the dining room for lunch. Acutely manic clients need clear, concise comments and directions. Telling the client that
he needs to stop talking because it is lunchtime is disrespectful and does not give the client directions for what he
needs to do. Using the familiar skill of waiting without interrupting until the person pauses would not be effective
with the very talkative, manic client. Walking away and approaching the client after a few minutes before the food
gets cold is not helpful because the client would probably continue talking.
Question 2 See full question
After the nurse administers haloperidol 5 mg PO to a client with acute mania, the client refuses to lie down on her
bed, runs out on the unit, pushes clients in her vicinity out of the way, and screams threatening remarks to the
staff. What should the nurse do next?
You Selected:
Seclude the client and use restraints if necessary.
Correct response:
Seclude the client and use restraints if necessary.
Explanation:
Question 3 See full question
During the discharge planning teaching process, a client who has been prescribed tranylocypromine states that he
enjoys a beer or two in the evenings. Which of the following is the nurse's most appropriate response?
You Selected:
Beer contains tyramine, which must be avoided when on this medication.
Correct response:
Beer contains tyramine, which must be avoided when on this medication.
Explanation:
Question 4 See full question
A young adult client with severe depression and suicide ideation has been started on the selective seratonin
reuptake inhibitor (SSRI) sertraline. Which client statement would indicate the client needs further teaching about
seratraline?
You Selected:
Being on sertraline will significantly decrease the chances that I might hurt myself.
Correct response:
Being on sertraline will significantly decrease the chances that I might hurt myself.
Explanation:
Question 5 See full question
After teaching a client about lorazepam, which client statement indicates the need for further teaching? Select all
that apply.
You Selected:
I can take lorazepam with food if I get nauseous.
I can adjust the dosage when I feel more anxious.
I can stop taking lorazepam immediately if I need to.
Correct response:
I can adjust the dosage when I feel more anxious.
I can stop taking lorazepam immediately if I need to.
Explanation:
Lorazepam, a benzodiazepine, is used as an antianxiety agent and depresses the central nervous system (CNS).
Benzodiazepines cause physical dependence and tolerance and should never be stopped abruptly because
withdrawal symptoms can occur. Slow tapering is required to minimize withdrawal symptoms.

The client should not adjust the dosage when feeling anxious because of tolerance and the possibility of overdose.

Common CNS adverse effects are drowsiness, fatigue, and incoordination. Other adverse effects such as dry mouth
can be helped by rinsing the mouth and using sugarless gum and candy.

The drug can be taken with food if the client experiences nausea.

The use of alcohol and other CNS depressants can further CNS depression.

Answer Key

Question 1 See full question


A depressed client in the psychiatric unit hasn't been getting adequate rest and sleep. To encourage restful sleep at
night, the nurse should:

You Selected:
gently but firmly set limits on how much time the client spends in bed during the day.

Correct response:
gently but firmly set limits on how much time the client spends in bed during the day.

Explanation:

Question 2 See full question


Which food should the nurse tell the client to avoid while taking phenelzine?

You Selected:
hamburger
Correct response:
salami

Explanation:
Phenelzine is a monoamine oxidase inhibitor (MAOI). MAOIs block the enzyme monoamine oxidase, which is
involved in the decomposition and inactivation of norepinephrine, serotonin, dopamine, and tyramine (a precursor
to the previously stated neurotransmitters). Foods high in tyraminethose that are fermented, pickled, aged, or
smokedmust be avoided because, when they are ingested in combination with MAOIs, a hypertensive crisis
occurs. Some examples include salami, bologna, dried fish, sour cream, yogurt, aged cheese, bananas, pickled
herring, caffeinated beverages, chocolate, licorice, beer, red wine, and alcohol-free beer.

Question 3 See full question


Which behavior if exhibited by a client with a depressive disorder should lead the nurse to determine that the client
is ready for discharge?

You Selected:
verbalization of feeling in control of self and situations

Correct response:
verbalization of feeling in control of self and situations

Explanation:

Question 4 See full question


When developing a teaching plan for a client about the medications prescribed for depression, which component is
most important for the nurse to include?

You Selected:
dosage regulation and adjustment

Correct response:
management of common adverse effects

Explanation:
Compliance with medication therapy is crucial for the client with depression. Medication noncompliance is the
primary cause of relapse among psychiatric clients. Therefore, the nurse needs to teach the client about managing
common adverse effects to promote compliance with medication. Teaching the client about the medications
pharmacokinetics may help the client to understand the reason for the drug. However, teaching about how to
manage common adverse effects to promote compliance is crucial. Current research about the medication is more
important to the nurse than to the client. Teaching about dosage regulation and adjustment of medication may be
helpful, but typically the HCP, not the client, is the person in charge of this aspect.

Question 5 See full question


A client with bipolar disorder, mania, has flight of ideas and grandiosity and becomes easily agitated. To prevent
harmful behaviors, which of the following should the nurse do initially?

You Selected:
Tell the client to seek out staff when feeling agitated.

Correct response:
Tell the client to seek out staff when feeling agitated.
Explanation:

Question 6 See full question


The nurse is caring for a client who is unconscious following an attempted suicide by drug overdose. When speaking
with the clients distraught wife, what should the nurse do first?

You Selected:
Encourage the wife to express her feelings and concerns, and listen carefully.

Correct response:
Encourage the wife to express her feelings and concerns, and listen carefully.

Explanation:

Question 7 See full question


The unlicensed assistive personnel (UAP) approaches the nurse and states, The client does not know what caused
him to be so depressed. He must not want to tell me because he does not trust me yet. In responding to this staff
member, which statement by the nurse will help the UAP understand the clients illness?

You Selected:
Endogenous depression is biochemical and is not caused by an outside stressor or problem. The client cannot tell
you why he is depressed because he really does not know.

Correct response:
Endogenous depression is biochemical and is not caused by an outside stressor or problem. The client cannot tell
you why he is depressed because he really does not know.

Explanation:

Question 8 See full question


The client with a cognitive disorder tells the nurse, "Everyone is after me. They want to kill me." The nurse should
respond:

You Selected:
"You are frightened. This is a hospital and these people are staff members. You are safe here."

Correct response:
"You are frightened. This is a hospital and these people are staff members. You are safe here."

Explanation:

Question 9 See full question


A client with a cocaine dependency is irritable, anxious, highly sensitive to stimuli, and over-reactive to clients and
staff on the unit. Which action is most therapeutic for this client?

You Selected:
providing the client with frequent "time-outs"

Correct response:
providing the client with frequent "time-outs"
Explanation:

Question 10 See full question


A client taking disulfiram during alcohol rehabilitation therapy reports to the nurse that he has a mild cold and plans
to use a cough medicine. Which statement made by the client indicates understanding of the nurse's teaching?

You Selected:
"Small doses of cough syrup might make me crave alcohol."

Correct response:
"I may experience vomiting and an upset stomach if I take cough medicine while taking this medicine."

Explanation:
Disulfiram provokes a violent reaction in the presence of alcohol; the client may not realize that cough medicine
may contain an alcohol base. This medication combination won't cause depression. Because the cold is minor,
there's no need for the client to talk with his physician.

Answer Key

Question 1 See full question


A client with major depression is frequently irritable, abrasive, and uncooperative and refuses to participate in
group activities. When working with this client, the nurse should use which approach?

You Selected:
Firmness

Correct response:
Firmness

Explanation:

Question 2 See full question


A nurse is assigned to care for a recently admitted client who has attempted suicide. What should the nurse do?

You Selected:
Search the client's belongings and his room carefully for items that could be used to attempt suicide.

Correct response:
Search the client's belongings and his room carefully for items that could be used to attempt suicide.

Explanation:

Question 3 See full question


A client with a diagnosis of major depression is ordered clonazepam for agitation in addition to an antidepressant.
Client teaching should include which statement?

You Selected:
Clonazepam may have a slight depressant effect.

Correct response:
Clonazepam may have a slight depressant effect.
Explanation:

Question 4 See full question


A nurse is caring for a severely depressed client who is barely functioning. The priority nursing goal for this client
would be to:

You Selected:
assess for and maintain adequate nutrition and hydration.

Correct response:
assess for and maintain adequate nutrition and hydration.

Explanation:

Question 5 See full question


A client was admitted to the inpatient unit 3 days ago with a flat affect, psychomotor retardation, anorexia,
hopelessness, and suicidal ideation. The health care provider (HCP) prescribed 75 mg of venlafaxine extended
release to be given every morning. The client interacted minimally with the staff and spent most of the day in his
room. As the nurse enters the unit at the beginning of the evening shift, the client is smiling and cheerful and
appears to be relaxed. What should the nurse interpret as the most likely cause of the clients behavior?

You Selected:
The client's sudden improvement calls for close observation by the staff.

Correct response:
The client's sudden improvement calls for close observation by the staff.

Explanation:

Question 6 See full question


A client with bipolar disorder, manic phase, is scheduled for a chest radiograph. Before taking the client to the
radiology department, the nurse should:

You Selected:
call security to be on standby for possible problems.

Correct response:
explain the procedure in simple terms.

Explanation:
The nurse needs to explain the procedure in simple terms because the client in a manic phase has difficulty
concentrating, is easily distracted, and can misinterpret what the nurse states. Giving a thorough explanation of the
procedure is not helpful and can confuse the client. Calling security to be on standby is inappropriate. If the nurse
judges that the client might elope or become agitated, the nurse should schedule the appointment for another time.
Canceling the appointment until the client can go unescorted is impractical and may not follow unit or hospital
policy and the clients treatment plan.

Question 7 See full question


The client with Alzheimer's disease has been prescribed donepezil 5 mg at bedtime. Which instruction should the
nurse give to the client's daughter?

You Selected:
Avoid suddenly stopping the medication.
Correct response:
Avoid suddenly stopping the medication.

Explanation:

Question 8 See full question


The nurse attempts to interact with a client who barely responds with yes or no. The client states, "Do not bother
me. I want to die." The nurse should:

You Selected:
sit with the client for 10 minutes.

Correct response:
sit with the client for 10 minutes.

Explanation:

Question 9 See full question


The nurse is caring for a client with bipolar disorder who was recently admitted to an inpatient unit and is
experiencing a manic episode. What is a priority nursing intervention for this client?

You Selected:
Closely monitor the clients eating and sleeping habits.

Correct response:
Closely monitor the clients eating and sleeping habits.

Explanation:

Question 10 See full question


In assessing an adolescent client at an outpatient clinic, the nurse is able to recognize that depression in
adolescents is often which of the following?

You Selected:
Often masked by aggressive behaviors

Correct response:
Often masked by aggressive behaviors

Explanation:

Answer Key

Question 1 See full question


A nurse is caring for a client with bipolar disorder. The care plan for a client in a manic state would include:

You Selected:
offering high-calorie meals and strongly encouraging the client to finish all his food.

Correct response:
listening attentively to the client's requests with a neutral attitude, and avoiding power struggles.
Explanation:
The nurse should listen to the client's requests, express willingness to seriously consider each request. The nurse
should encourage the client to take short daytime naps because he expends so much energy. High-calorie finger
foods should be offered to supplement the client's diet if he can't remain seated long enough to eat a complete
meal. The client shouldn't be forced to stay seated at the table to finish a meal. The nurse should set limits in a
calm, clear, and self-confident tone of voice.

Question 2 See full question


A nurse notices that a depressed client who has been taking amitriptyline hydrochloride for 2 weeks has become
very outgoing, cheerful, and talkative. The nurse suspects that the client:

You Selected:
may be experiencing increased energy and is at increased risk for suicide.

Correct response:
may be experiencing increased energy and is at increased risk for suicide.

Explanation:

Question 3 See full question


Assessment of suicidal risk in children and adolescents requires the nurse to know what information?

You Selected:
The risk of suicide increases during adolescence, with those who have recently suffered a loss, abuse, or family
discord being most at risk.

Correct response:
The risk of suicide increases during adolescence, with those who have recently suffered a loss, abuse, or family
discord being most at risk.

Explanation:

Question 4 See full question


A client with agnosia as a result of vascular dementia is staring at dinner and utensils without trying to eat. Which
intervention should the nurse attempt first?

You Selected:
Hand the fork to the client and say, "Use this fork to eat your green beans."

Correct response:
Hand the fork to the client and say, "Use this fork to eat your green beans."

Explanation:

Question 5 See full question


The nurse is counseling a client regarding treatment of the client's newly diagnosed depression. The nurse
emphasizes that full benefit from antidepressant therapy usually takes how long?

You Selected:
2 to 4 weeks

Correct response:
2 to 4 weeks
Explanation:

Question 6 See full question


A client with bipolar disorder is monopolizing the use of the telephone by making several calls each day, interfering
with the ability of other clients to use the telephone. The nurse should:

You Selected:
limit the amount of calls the client can make each day.

Correct response:
limit the amount of calls the client can make each day.

Explanation:

Question 7 See full question


A client has just been admitted with acute delirium of unknown etiology. The client's daughter states that she is
worried about her mom because she has never been this sick before. Which would be the most helpful statement to
make to the daughter?

You Selected:
The health care provider will prescribe tests to find out what is causing her condition.

Correct response:
The health care provider will prescribe tests to find out what is causing her condition.

Explanation:

Question 8 See full question


The client, who is taking fluoxetine 20 mg at bedtime, tell the nurse the drug is interfering with his sleep. The nurse
should conclude that:

You Selected:
the client should take fluoxetine in the morning.

Correct response:
the client should take fluoxetine in the morning.

Explanation:

Question 9 See full question


A client has been taking 30 mg of duloxetine hydrochloride twice daily for 2 months because of depression and
vague aches and pains. While interacting with the nurse, the client discloses a pattern of drinking a 6-pack of beer
daily for the past 10 years to help with sleep. What should the nurse do first?

You Selected:
Report the client's beer consumption to the health care provider (HCP).

Correct response:
Report the client's beer consumption to the health care provider (HCP).
Explanation:

Question 10 See full question


A client attends a follow-up visit to a clinic after being diagnosed with atypical depression. The practitioner
prescribed tranylcypromine sulfate, 10 mg by mouth twice a day during the last visit 14 days ago. Which of the
following would be the priority action by the nurse for this client?

You Selected:
Screen the client for new, worsened, or increased depression

Correct response:
Screen the client for new, worsened, or increased depression

Explanation:

Improve your mastery


TAKE A PRACTICE QUIZ

Answer Key

Question 1 See full question


An adolescent who is depressed and whose parents report as having difficulty in school is brought to the community
mental health center to be evaluated. Which additional problem would the nurse expect the client to have?

You Selected:
Anxiety disorder

Correct response:
Behavioral difficulties

Explanation:
Adolescents with depression tend to demonstrate severe irritability and behavioral problems. Anxiety disorder more
commonly affects small children. Cognitive impairment is typically associated with delirium and dementia.
Compulsive behaviors are more likely in a client with an anxiety disorder, specifically obsessive-compulsive
disorder.

Question 2 See full question


A nurse is frustrated by her inability to make much progress establishing a therapeutic relationship with a client
with bipolar disorder. Her most professional response would be to:

You Selected:
keep trying to talk with the client even though the nurse is frustrated.

Correct response:
discuss the situation with a more experienced peer.

Explanation:
A collaborative approach is always a better way to address challenging situations; additional input may provide
insight to help the nurse provide more effective client care. Asking to be reassigned and suggesting that another
nurse might provide more effective care are avoidant responses that do not address the underlying issues. At this
time, there is no indication that a medication reevaluation is necessary.
Question 3 See full question
A client with depression and suicidal ideation voices feelings of self-doubt and powerlessness and is very dependent
on the nurse for most aspects of her care. According to Erikson's stages of growth and development, the nurse
determines the client to be manifesting problems in which stage?

You Selected:
autonomy versus shame/doubt

Correct response:
autonomy versus shame/doubt

Explanation:

Question 4 See full question


The daughter of a client with Alzheimer's disease tells the nurse that her mother thinks someone is stealing her
things. Which response by the nurse would be most helpful?

You Selected:
We asked the health care provider to evaluate your mother for paranoid delusions, which are common in people
with Alzheimers disease.

Correct response:
We asked the health care provider to evaluate your mother for paranoid delusions, which are common in people
with Alzheimers disease.

Explanation:

Question 5 See full question


An elderly client who has been diagnosed with delusional disorder for many years is exhibiting early symptoms of
dementia. His daughter lives with him to help him manage daily activities, and he attends a day care program for
seniors during the week while she works. A nurse at the day care center hears him say, If my neighbor puts up a
fence, I will blow him away with my shotgun. He has never respected my property line, and I have had it! Which
action should the nurse take?

You Selected:
Report the comment to the neighbor, the daughter, and the police since there is the potential for a criminal act.

Correct response:
Report the comment to the neighbor, the daughter, and the police since there is the potential for a criminal act.

Explanation:

Answer Key

Question 1 See full question


A client comes to the emergency department after being attacked and sexually assaulted. What is the most
accurate nursing diagnosis for this client?

You Selected:
Rape-trauma syndrome

Correct response:
Rape-trauma syndrome
Explanation:

Question 2 See full question


Which nursing intervention is most appropriate if a client develops orthostatic hypotension while taking
amitriptyline hydrochloride?

You Selected:
Advising the client to sit up for 1 minute before getting out of bed

Correct response:
Advising the client to sit up for 1 minute before getting out of bed

Explanation:

Question 3 See full question


A client was found unconscious on the floor of his bathroom with self-inflicted wrist lacerations. An ambulance was
called and the client was transferred to the inpatient psychiatric unit for observation and treatment with
antidepressants. Which nursing intervention is most appropriate?

You Selected:
Continue suicide precautions.

Correct response:
Continue suicide precautions.

Explanation:

Question 4 See full question


Which behavior if exhibited by a client with a depressive disorder should lead the nurse to determine that the client
is ready for discharge?

You Selected:
verbalization of feeling in control of self and situations

Correct response:
verbalization of feeling in control of self and situations

Explanation:

Question 5 See full question


A client has been taking fluoxetine, 40 mg daily at 0900 for 1 week. The client states he feels nervous and has had
diarrhea. The nurse determines the clients symptoms are:

You Selected:
important, probably suggesting a decrease in dosage or change to another medication.

Correct response:
important, probably suggesting a decrease in dosage or change to another medication.
Explanation:

Answer Key

Question 1 See full question


A client chronically complains of being unappreciated and misunderstood by others. He is argumentative and sullen.
He always blames others for his failure to complete work assignments. He expresses feelings of envy toward people
he perceives as more fortunate. He voices exaggerated complaints of personal misfortune. The client most likely
suffers from which personality disorder?

You Selected:
Avoidant personality disorder

Correct response:
Passive-aggressive personality

Explanation:
The client with passive-aggressive personality disorder displays a pervasive pattern of negative attitudes, chronic
complaints, and passive resistance to demands for adequate social and occupational performance. The client with a
dependent personality is unable to make everyday decisions and allows others to make important decisions for him.
In addition, the client with a dependent personality commonly volunteers to do things that are unpleasant so that
others will like him. The avoidant personality displays a pervasive pattern of social discomfort, fear of negative
evaluation, and timidity. The client with obsessive-compulsive disorder displays a pervasive pattern of perfectionism
and inflexibility.

Question 2 See full question


The nurse is caring for a client whose somatic symptom disorder is characterized by frequent descriptions of pain.
What statement is true of this client's pain?

You Selected:
The pain is real to the client, even though the pain may not have an organic etiology.

Correct response:
The pain is real to the client, even though the pain may not have an organic etiology.

Explanation:

Question 3 See full question


A client with acute mania is to receive lithium carbonate 600 mg PO three times daily and 2 mg of haloperidol PO at
bedtime. The nurse should:

You Selected:
request a decreased dosage of lithium.

Correct response:
give the medications as prescribed.

Explanation:
Lithium commonly is combined with an antipsychotic agent, such as haloperidol, or a benzodiazepine such as
lorazepam. Antipsychotic agents, such as haloperidol, are prescribed to produce a neuroleptic effect until the
lithium, produces a clinical response. After a clinical response is achieved, the antipsychotic agent usually is
discontinued. Additionally, the dosages of each drug listed are appropriate. Therefore, the nurse would administer
the drugs as prescribed.
Question 4 See full question
The nurse understands that the client with severe dementia and motor apraxia may be able to perform which
action?

You Selected:
Brush the teeth when handed a toothbrush.

Correct response:
Brush the teeth when handed a toothbrush.

Explanation:

Question 5 See full question


The nurse is assessing a 38-year-old client at risk for suicide. Which of the following are significant assessment data
when determining whether a client will require hospitalization? Select all that apply.

You Selected:
Having an organized plan
A description of command hallucinations

Correct response:
Being intoxicated with alcohol
Having an organized plan
A description of command hallucinations

Explanation:
The age of the client does not pose a risk because even though certain populations are at higher risk for suicide,
the age of a client does not indicate that hospitalization is recommended. The substance abuse as a minor also
does not impact this clients judgment at this time. A detailed plan to commit suicide places the client at a high risk,
and hospitalization is indicated. Clients who are experiencing altered perceptions are at risk and may need to be in
a protective environment. The use of alcohol or drugs can impair the client's judgment, and risk for suicide is higher.

Answer Key

Question 1 See full question


When caring for an adolescent diagnosed with depression, the nurse should remember that depression manifests
differently in adolescents than it does in adults. In an adolescent, signs and symptoms of depression are likely to
include:

You Selected:
truancy, a change of friends, social withdrawal, and oppositional behavior.

Correct response:
truancy, a change of friends, social withdrawal, and oppositional behavior.

Explanation:

Question 2 See full question


Which characteristic is most common among suicidal clients?

You Selected:
Ambivalence
Correct response:
Ambivalence

Explanation:

Question 3 See full question


Two family members are visiting their father who is experiencing acute delirium. They are upset that their father is
so disoriented. He knows who we are, but that is about it. We do not know what to say to him. What should the
nurse tell the family? Select all that apply.

You Selected:
"Answer his questions simply, honestly, slowly, and clearly."
"Correct him when he is hearing and seeing things that are not there."
"Occasionally remind him of the time, day, and place when he does not remember."
"Include him in your conversation, instead of talking about him while he is present."

Correct response:
"Answer his questions simply, honestly, slowly, and clearly."
"Occasionally remind him of the time, day, and place when he does not remember."
"Include him in your conversation, instead of talking about him while he is present."

Explanation:
Clear communication is crucial for a client with delirium. The family must include the client in all conversations and
keep him oriented to time and place. It is inappropriate to argue with a clients hallucinations because they are real
to the client. Speaking more loudly will not help this client hear more distinctly and may increase the clients
confusion.

Question 4 See full question


Nurses should be aware of their own feelings about clients, and the difficulty of maintaining effective relationships
with depressed clients experiencing suicidal ideation because of which behaviors?

You Selected:
pessimism, which arouses frustration and anger in others

Correct response:
pessimism, which arouses frustration and anger in others

Explanation:

Question 5 See full question


A client exhibits confusion and severe memory loss. At 1130, he tells the nurse that he is going to work and
proceeds to walk toward the door. The nurse should:

You Selected:
divert his attention toward the dining room where lunch is being served.

Correct response:
divert his attention toward the dining room where lunch is being served.

Explanation:

Question 1: A client with obsessive-compulsive disorder reveals that he was late for his
(see full question) appointment because of my dumb habit. I have to take off my socks and put
them back on 41 times! I cannot stop until I do it just right. The nurse interprets
the clients behavior as most likely representing an effort to obtain:
You selected: relief from anxiety.
Correct
Explanation: A client who is exhibiting compulsive behavior is attempting to control his anxiety.
The compulsive behavior is performed to relieve discomfort and to bind or
neutralize anxiety. The client must perform the ritual to avoid an extreme increase
in tension or anxiety even though the client is aware that the actions are absurd.
The repetitive behavior is not an attempt to control thoughts; the obsession or
thinking component cannot be controlled. It is not an attention-seeking
mechanism or an attempt to express hostility. (less)

Question 2: The nurse notes that a client with acute pancreatitis occasionally experiences
(see full question) muscle twitching and jerking. How should the nurse interpret the significance of
these symptoms?
You selected: The client may be developing hypocalcemia.
Correct
Explanation: Hypocalcemia develops in severe cases of acute pancreatitis. The exact cause is
unknown. Signs and symptoms of hypocalcemia include jerking and muscle
twitching, numbness of fingers and lips, and irritability. Meperidine may cause
tremors or seizures as an adverse effect, but not muscle twitching. Muscle
twitching is not caused by a nutritional deficit, nor does it indicate that the client
needs a muscle relaxant. (less)

Question 3: Twenty minutes after a transfusion of packed red blood cells is initiated, a client
(see full question) reports shivering, headache, and lower back pain. The vital signs show a normal
temperature and increased pulse and respiratory rate. What should be the first
nursing actions?
You selected: Stop the transfusion, continue with saline infusion, and notify the physician
regarding a suspected hemolytic reaction.
Correct
Explanation: Hemolytic reaction is one of the most severe blood reactions, so prompt action to
stop the transfusion is very important, followed by ensuring the IV access is
preserved.

Question 4: What therapeutic outcome does the nurse expect for a client who has received a
(see full question) premedication of glycopyrrolate?
You selected: decreased secretions
Correct
Explanation: Glycopyrrolate is an anticholinergic given for its ability to reduce oral and
respiratory secretions before general anesthesia. Increased heart rate and
respiratory rate would be adverse effects of the drug. Amnesia should not be an
effect of the drug. (less)

Question 5: A client will receive IV midazolam hydrochloride during surgery. Which finding
(see full question) indicates a therapeutic effect?
You selected: amnesia
Correct
Explanation: Midazolam hydrochloride causes antegrade amnesia or decreased ability to
remember events that occurred around the time of sedation. Nausea, mild
agitation, and blurred vision are adverse effects of midazolam. (less)

Question 6: Which statements by the mother of a toddler should lead the nurse to suspect that
(see full question) the child is at risk for iron deficiency anemia? Select all that apply.
You selected: "He does not like meat, but he will eat small amounts of it."
"He sleeps 12 hours every night and takes a 2-hour nap."
"He refuses to eat more than two different kinds of vegetables."
Incorrect
Correct response: "He drinks over 4 glasses of milk per day."
"I cannot keep enough apple juice in the house; he must drink over 10 oz (300
mL) per day."
Explanation: Toddlers should have between two and three servings milk per day and no more
than 6 oz (180 mL) of juice per day. If they have more than that, then they are
probably not eating enough other foods, including iron-rich foods that have the
needed nutrients. Food preferences vary among children. It is acceptable for the
child to refuse foods as long as the diet is balanced and contains adequate
calories. The child is obtaining a normal amount of sleep. (less)
Question 7: A client has received an overdose of sympathomimetic agents. The nurse should
(see full question) assess the client for which late signs of an overdose? Select all that apply.
You selected: hypotension
seizures
bradycardia
Incorrect
Correct response: hypotension
seizures
profound pyrexia
Explanation: As the homeostatic responses begin to decompensate, late clinical manifestations
from a large overdose of sympathomimetic agents include loss of function of the
hypothalamus such as temperature regulation, leading to profound pyrexia, and
ectopic brain activity leading to seizures. Hypotension is a late sign that occurs as
the vascular system collapses. Hypertension, an earlier sign, precedes
hypotension. Tachycardia occurs as a reflex to hypotension, a late sign. (less)

Question 8: The nurse is caring for a child who sustained a spinal cord injury in a motor vehicle
(see full question) accident. The childs body temperature fluctuates markedly, and the parents
question why this is occurring. What is the most accurate response for the nurse
to give to the parents?
You selected: The childs sympathetic nervous system was damaged in the accident.
Correct
Explanation: A common cause of temperature fluctuation in clients with spinal cord injury is
damage to the sympathetic nervous system. Infections will induce a fever, but
temperature will not fluctuate markedly. (less)

Question 9: While assessing a child experiencing respiratory distress, the nurse notes
(see full question) subcostal retractions. Which graphic highlights the area where subcostal
retractions are seen?
You selected:

Correct
Explanation: Subcostal retractions are retractions seen below the lower costal margin of the rib
cage. Option B highlights the area where subcostal retractions are seen. Option A
shows the areas where intercostal retractions would be seen. Option C shows the
area for suprasternal retraction. Option D shows the areas for clavicular
retractions. (less)

Question 10: The nurse should assess older adults for which serious adverse effects of
(see full question) ibuprofen?
You selected: impaired renal function
Correct
Explanation: Renal function may already be compromised in the elderly, and ibuprofen can
further impair renal or liver function. Nonsteroidal anti-inflammatory drugs can
also cause nephrosis, cirrhosis, and heart failure in elderly persons.

Rebound headaches are not a serious adverse effect of ibuprofen.

Neuropathy and hypoglycemia are not adverse effects of ibuprofen.

(less)

Answer Key
Question 1: What should the nurse explain to the pregnant client about the importance
(see full question) of the fetal stage of development?
You selected: There is additional growth and development of the organs and body
systems.
Correct
Explanation: The fetal stage is from the beginning of the ninth week after fertilization
and continues until birth. At this time, the developing human is called a
fetus. During the fetal stage, ... (more)

Reference: Ricci, S.S., Kyle, T., & Carman, S. Maternity and Pediatric Nursing, 2nd ed.
Philadelphia: Wolters Kluwer Health/Lippincott Williams & Wilkins, 2013,
Chapter 10: Fetal Development and Genetics, p. 315.
Chapter 10: Fetal Development and Genetics - Page 315

Question 2: The nurse is caring for a client and her partner who are considering a
(see full question) future pregnancy. The client reports her last two pregnancies ended in
stillbirth related to an underlying genetic disorder. What response by the
nurse is most appropriate?
You selected: Consultation with a genetic counselor before you become pregnant would
likely be beneficial.
Correct
Explanation: Prepregnancy screening with a genetic counselor would be helpful to the
client who has a history of fetal loss as a result of a genetic disorder. The
screening would allow the ... (more)

Reference: Ricci, S.S., Kyle, T., & Carman, S. Maternity and Pediatric Nursing, 2nd ed.
Philadelphia: Wolters Kluwer Health/Lippincott Williams & Wilkins, 2013,
Chapter 10: Fetal Development and Genetics, p. 326.
Chapter 10: Fetal Development and Genetics - Page 326

Question 3: When providing education on pre-conceptual considerations to a female


(see full question) client and her spouse, the nurse knows that teratogens during the
embryonic stage pose the greatest risk to the fetus. The nurse describes
this stage as occurring during which time frame?
You selected: the entire pregnancy
Incorrect
Correct response: weeks 2 to 8
Explanation: The embryonic stage produces the greatest risk of damaging effects to the
fetus because the cells are rapidly dividing and differentiating into specific
body structures. The embryo ... (more)

Reference: Ricci, S.S., Kyle, T., & Carman, S. Maternity and Pediatric Nursing, 2nd ed.
Philadelphia: Wolters Kluwer Health/Lippincott Williams & Wilkins, 2013,
Chapter 10: Fetal Development and Genetics, p. 308.
Chapter 10: Fetal Development and Genetics - Page 308

Question 4: The nurse is caring for a child with Down syndrome (trisomy 21). This is an
(see full question) example of which type of inheritance?
You selected: Mendelian recessive
Incorrect
Correct response: chromosome nondisjunction
Explanation: Down syndrome occurs when an ovum or sperm cell does not divide
evenly, permitting an extra 21st chromosome to cross to a new cell.

Reference: Ricci, S.S., Kyle, T., & Carman, S. Maternity and Pediatric Nursing, 2nd ed.
Philadelphia: Wolters Kluwer Health/Lippincott Williams & Wilkins, 2013,
Chapter 10: Fetal Development and Genetics, p. 325.
Chapter 10: Fetal Development and Genetics - Page 325

Question 5: A pregnant client has heard about Down syndrome and wants to know
(see full question) about the risk factors associated with it. What would the nurse include as a
risk factor?
You selected: family history of condition
Incorrect
Correct response: advanced maternal age
Explanation: Advanced maternal age is one the most important factors that increases
the risk of an infant being born with Down syndrome. Down syndrome is
not associated with advanced paternal a ... (more)
Reference: Ricci, S.S., Kyle, T., & Carman, S. Maternity and Pediatric Nursing, 2nd ed.
Philadelphia: Wolters Kluwer Health/Lippincott Williams & Wilkins, 2013,
Chapter 10: Fetal Development and Genetics, p. 326.
Chapter 10: Fetal Development and Genetics - Page 326

Question 6: While talking with a pregnant woman who has undergone genetic testing,
(see full question) the woman informs the nurse that her baby will be born with Down
syndrome. The nurse understands that Down syndrome is an example of a:
You selected: trisomy numeric abnormality.
Correct
Explanation: Down syndrome is an example of a chromosomal abnormality involving the
number of chromosomes (trisomy numeric abnormality), in particular
chromosome 21, in which the individual has ... (more)

Question 7: A couple wants to start a family. They are concerned that their child will be
(see full question) at risk for cystic fibrosis because they each have a cousin with cystic
fibrosis. They are seeing a nurse practitioner for preconceptual counseling.
What would the nurse practitioner tell them about cystic fibrosis?
You selected: It is an autosomal dominant disorder.
Incorrect
Correct response: It is an autosomal recessive disorder.
Explanation: Cystic fibrosis is autosomal recessive. Nurses also consider other issues
when assessing the risk for genetic conditions in couples and families. For
example, when obtaining a prec ... (more)

Reference: Ricci, S.S., Kyle, T., & Carman, S. Maternity and Pediatric Nursing, 2nd ed.
Philadelphia: Wolters Kluwer Health/Lippincott Williams & Wilkins, 2013,
Chapter 10: Fetal Development and Genetics, pp. 322-323.
Chapter 10: Fetal Development and Genetics - Page 322

Question 8: After teaching a class on the various structures formed by the embryonic
(see full question) membranes, the nurse determines that the teaching was successful when
the class identifies which structure as being formed by the mesoderm?
You selected: bones
Correct
Explanation: The endoderm forms the structures of the respiratory system. The
mesoderm forms the structures of the skeletal system. The ectoderm forms
the structures of the special senses. The ... (more)

Reference: Ricci, S.S., Kyle, T., & Carman, S. Maternity and Pediatric Nursing, 2nd ed.
Philadelphia: Wolters Kluwer Health/Lippincott Williams & Wilkins, 2013,
Chapter 10: Fetal Development and Genetics, p. 311.
Chapter 10: Fetal Development and Genetics - Page 311

Question 9: A 33-year-old pregnant client asks the nurse about testing for birth defects
(see full question) that are safe for both her and her fetus. Which test would the nurse state
as being safe and noninvasive?
You selected: ultrasound
Correct
Explanation: The nurse would state that an ultrasound is a noninvasive test that is
completely safe for both mother and child. Amniocentesis, CVS, and
percutaneous umbilical cord sampling are i ... (more)

Reference: Ricci, S.S., Kyle, T., & Carman, S. Maternity and Pediatric Nursing, 2nd ed.
Philadelphia: Wolters Kluwer Health/Lippincott Williams & Wilkins, 2013,
Chapter 10: Fetal Development and Genetics, p. 329.
Chapter 10: Fetal Development and Genetics - Page 329

Question 10: Down syndrome may occur because of a translocation defect. This means
(see full question) the:
You selected: additional chromosome was inherited because it was attached to a normal
chromosome.
Correct
Explanation: A translocation defect causes Down syndrome when a 21st chromosome is
attached to another chromosome, so dysjunction results in an abnormal
distribution of chromosomes.
Reference: Ricci, S.S., Kyle, T., & Carman, S. Maternity and Pediatric Nursing, 2nd ed.
Philadelphia: Wolters Kluwer Health/Lippincott Williams & Wilkins, 2013,
Chapter 10: Fetal Development and Genetics, p. 325.
Chapter 10: Fetal Development and Genetics - Page 325
stion 1: Amniotic fluid is produced throughout the pregnancy by the fetal
(see full question) membranes. Amniotic fluid has four major functions. What is one of these
functions?
You selected: physical protection
Correct
Explanation: Amniotic fluid serves four main functions for the fetus: physical protection,
temperature regulation, provision of unrestricted movement, and
symmetrical growth.

Reference: Ricci, S.S., Kyle, T., & Carman, S. Maternity and Pediatric Nursing, 2nd ed.
Philadelphia: Wolters Kluwer Health/Lippincott Williams & Wilkins, 2013,
Chapter 10: Fetal Development and Genetics, p. 311.
Chapter 10: Fetal Development and Genetics - Page 311

Question 2: Which structure supplies the fetus with nutrients?


(see full question)
You selected: placenta
Correct
Explanation: The placenta is a flat, round structure attached to the fetus by the
umbilical cord. Blood is passed through the umbilical cord, and the
placenta is the organ responsible for suppl ... (more)

Reference: Ricci, S.S., Kyle, T., & Carman, S. Maternity and Pediatric Nursing, 2nd ed.
Philadelphia: Wolters Kluwer Health/Lippincott Williams & Wilkins, 2013,
Chapter 10: Fetal Development and Genetics, pp. 311-312.
Chapter 10: Fetal Development and Genetics - Page 311

Question 3: Fetal circulation differs from the circulatory path of the newborn infant. In
(see full question) utero the fetus has a hole connecting the right and left atria of the heart.
This allows oxygenated blood to quickly pass to the major organs of the
body. What is this hole called?
You selected: foramen ovale
Correct
Explanation: The foramen ovale is a hole that connects the right and left atria so the
majority of oxygenated blood can quickly pass into the left side of the fetal
heart and go to the brain an ... (more)

Reference: Ricci, S.S., Kyle, T., & Carman, S. Maternity and Pediatric Nursing, 2nd ed.
Philadelphia: Wolters Kluwer Health/Lippincott Williams & Wilkins, 2013,
Chapter 10: Fetal Development and Genetics, p. 315.
Chapter 10: Fetal Development and Genetics - Page 315

Question 4: The nurse explains to a pregnant woman that the germ layers that develop
(see full question) in the embryo and become different organs and tissues consist of which
layers? Select all that apply.
You selected: endoderm
ectoderm
mesoderm
Correct
Explanation: The three germ layers in the embryo are the ectoderm, mesoderm, and
endoderm. The others are layers of the skin.

Reference: Ricci, S.S., Kyle, T., & Carman, S. Maternity and Pediatric Nursing, 2nd ed.
Philadelphia: Wolters Kluwer Health/Lippincott Williams & Wilkins, 2013,
Chapter 10: Fetal Development and Genetics, p. 311.
Chapter 10: Fetal Development and Genetics - Page 311

Question 5: The nurse is developing a presentation for a community group of young


(see full question) adults discussing fetal development and pregnancy. The nurse would
identify that the sex of offspring is determined at the time of:
You selected: formation of morula.
Incorrect
Correct response: fertilization.
Explanation: Sex determination occurs at the time of fertilization. Meiosis refers to cell
division resulting in the formation of an ovum or sperm with half the
number of chromosomes. The morula develops after a series of four
cleavages following the formation of the zygote. Oogenesis refers to the
development of a mature ovum, which has half the number of
chromosomes. (less)

Question 6: Genetics-related health care is basic to the holistic practice of nursing.


(see full question) What should nursing practice in genetics include?
You selected: gathering relevant family and medical history information
Correct
Explanation: The nurse's role in genetic counseling is to provide information, collect
relevant data, offer support, and coordinate resources. The other answers
are incorrect because the nurse does not provide advice or influence the
client to make choices. The nurse also does not identify genetic markers.
(less)

Reference: Ricci, S.S., Kyle, T., & Carman, S. Maternity and Pediatric Nursing, 2nd ed.
Philadelphia: Wolters Kluwer Health/Lippincott Williams & Wilkins, 2013,
Chapter 10: Fetal Development and Genetics, p. 327.
Chapter 10: Fetal Development and Genetics - Page 327

Question 7: Pregnancy tests analyze for the presence of human chorionic gonadotropin
(see full question) (HCG) hormone. The nurse understands that the function of this hormone
is to:
You selected: prolong the life of the corpus luteum.
Correct
Explanation: The corpus luteum is responsible for producing progesterone until this
function is assumed by the placenta. HCG is a fail-safe mechanism to
prolong the life of the corpus luteum and ensure progesterone production.
Somatomammotropin is a growth hormone. The uterine growth is in
response to estrogen and other growth factors. Oxygen transport occurs
via blood flow from the mother. (less)

Reference: Ricci, S.S., Kyle, T., & Carman, S. Maternity and Pediatric Nursing, 2nd ed.
Philadelphia: Wolters Kluwer Health/Lippincott Williams & Wilkins, 2013,
Chapter 10: Fetal Development and Genetics, p. 314.
Chapter 10: Fetal Development and Genetics - Page 314

Question 8: A number of inherited diseases can be detected in utero by amniocentesis.


(see full question) Which disease can be detected by this method?
You selected: trisomy 21
Correct
Explanation: Karyotyping for chromosomal defects can be carried out using
amniocentesis.
Reference: Ricci, S.S., Kyle, T., & Carman, S. Maternity and Pediatric Nursing, 2nd ed.
Philadelphia: Wolters Kluwer Health/Lippincott Williams & Wilkins, 2013,
Chapter 10: Fetal Development and Genetics, p. 316.
Chapter 10: Fetal Development and Genetics - Page 316

Question 9: When describing the characteristics of the amniotic fluid to a pregnant


(see full question) woman, the nurse would include which information?
You selected: The amount gradually fluctuates during pregnancy.
Correct
Explanation: Amniotic fluid is alkaline. Amniotic fluid is composed of 98% water and 2%
organic matter. Amniotic fluid volume gradually fluctuates throughout
pregnancy. Sufficient amounts promote fetal movement to enhance
musculoskeletal development. (less)

Reference: Ricci, S.S., Kyle, T., & Carman, S. Maternity and Pediatric Nursing, 2nd ed.
Philadelphia: Wolters Kluwer Health/Lippincott Williams & Wilkins, 2013,
Chapter 10: Fetal Development and Genetics, p. 311.
Chapter 10: Fetal Development and Genetics - Page 311

Question 10: The fluid-filled, inner membrane sac surrounding the fetus is which
(see full question) structure?
You selected: endometrium
Incorrect
Correct response: amnion
Explanation: The fluid-filled, inner membrane sac surrounding the fetus is the amnion.
The chorion is the outer membrane surrounding the fetus. The
endometrium is the inner lining of the uterus. The decidua is the name
used for the endometrium during pregnancy. (less)

Reference: Ricci, S.S., Kyle, T., & Carman, S. Maternity and Pediatric Nursing, 2nd ed.
Philadelphia: Wolters Kluwer Health/Lippincott Williams & Wilkins, 2013,
Chapter 10: Fetal Development and Genetics, p. 311.
Chapter 10: Fetal Development and Genetics - Page 311
Answer Key
Question 1: A pregnant client and her husband have had a session with a genetic
(see full question) specialist. What is the role of the nurse after the client has seen a
specialist?
You selected: Review what has been discussed with the specialist.
Correct
Explanation: After the client has seen the specialist, the nurse should review what the
specialist has discussed with the family and clarify any doubts the couple
may have. The nurse shoul ... (more)

Reference: Ricci, S.S., Kyle, T., & Carman, S. Maternity and Pediatric Nursing, 2nd ed.
Philadelphia: Wolters Kluwer Health/Lippincott Williams & Wilkins, 2013,
Chapter 10: Fetal Development and Genetics, p. 328, 330.
Chapter 10: Fetal Development and Genetics - Page 328

Question 2: Which type of genetic test would be used to detect the possibility of Down
(see full question) syndrome?
You selected: chromosomal analysis
Correct
Explanation: Chromosomal analysis is part of the genetic testing for Down syndrome.
DNA analysis may be used in the detection of Huntington disease.
Hemoglobin electrophoresis may be used in ... (more)

Reference: Ricci, S.S., Kyle, T., & Carman, S. Maternity and Pediatric Nursing, 2nd ed.
Philadelphia: Wolters Kluwer Health/Lippincott Williams & Wilkins, 2013,
Chapter 10: Fetal Development and Genetics, p. 325.
Chapter 10: Fetal Development and Genetics - Page 325

Question 3: Smoking may be a cause of ectopic (tubal) pregnancy.


(see full question)
You selected: True
Correct

Question 4: A pregnant client is scheduled to undergo chorionic villi sampling (CVS) to


(see full question) rule out any birth defects. Ideally, when should this testing be completed?
You selected: 10 to 12 weeks of gestation
Correct
Explanation: Chorionic villus sampling (CVS) is typically performed between 10 to 12
weeks gestation. Sometimes it may be offered up to 14 weeks. The test is
not conducted before 10 weeks ... (more)

Reference: Ricci, S.S., Kyle, T., & Carman, S. Maternity and Pediatric Nursing, 2nd ed.
Philadelphia: Wolters Kluwer Health/Lippincott Williams & Wilkins, 2013,
Chapter 10: Fetal Development and Genetics, p. 329.
Chapter 10: Fetal Development and Genetics - Page 329

Question 5: A fetus is able to maintain blood circulation in utero by the presence of


(see full question) circulatory shunts. The ductus arteriosus in utero shunts blood from:
You selected: the pulmonary artery to the aorta.
Correct
Explanation: Because the fetal lungs are not inflated, blood must be diverted past them.
The ductus arteriosus helps to do this by shunting blood from the
pulmonary artery to the aorta. As the ... (more)
Reference: Ricci, S.S., Kyle, T., & Carman, S. Maternity and Pediatric Nursing, 2nd ed.
Philadelphia: Wolters Kluwer Health/Lippincott Williams & Wilkins, 2013,
Chapter 10: Fetal Development and Genetics, p. 315.
Chapter 10: Fetal Development and Genetics - Page 315

Question 6: A nurse is conducting an in-service program for a group of perinatal


(see full question) nurses. After teaching the group about autosomal dominant and recessive
inheritance patterns, the nurse determines that the group needs additional
teaching when they identify which condition as an example of an
autosomal dominant disorder?
You selected: Tay-Sachs disease
Correct
Explanation: Neurofibromatosis is an example of an autosomal dominant disorder.
Achondroplasia is an example of an autosomal dominant disorder.
Huntington disease is an example of an autosomal ... (more)

Reference: Ricci, S.S., Kyle, T., & Carman, S. Maternity and Pediatric Nursing, 2nd ed.
Philadelphia: Wolters Kluwer Health/Lippincott Williams & Wilkins, 2013,
Chapter 10: Fetal Development and Genetics, p. 322.
Chapter 10: Fetal Development and Genetics - Page 322

Question 7: The maternal serum alpha fetoprotein blood test is performed on pregnant
(see full question) women to screen for which condition?
You selected: fetal neural tube defects
Correct
Explanation: The maternal serum alpha fetoprotein blood test is performed on pregnant
women to screen for fetal neural tube defects. The 1-hour random glucose
tolerance test is used to screen f ... (more)

Reference: Ricci, S.S., Kyle, T., & Carman, S. Maternity and Pediatric Nursing, 2nd ed.
Philadelphia: Wolters Kluwer Health/Lippincott Williams & Wilkins, 2013,
Chapter 10: Fetal Development and Genetics, p. 329.
Chapter 10: Fetal Development and Genetics - Page 329

Question 8: After teaching a class on the stages of fetal development, the nurse
(see full question) determines that the teaching was successful when the group identifies
which stages? Select all that apply.
You selected: embryonic
preembryonic
fetal
Correct
Explanation: The three stages of fetal development are the preembryonic, embryonic,
and fetal stage. Placental and umbilical are not stages of fetal
development.

Question 9: A nursing student correctly identifies that a person's outward appearance


(see full question) or expression of genes is referred to as the:
You selected: phenotype.
Correct
Explanation: Alleles are two like genes. Phenotype refers to a person's outward
appearance or the expression of genes. Genotype refers to his or her
actual gene composition. Genome is the compl ... (more)

Reference: Ricci, S.S., Kyle, T., & Carman, S. Maternity and Pediatric Nursing, 2nd ed.
Philadelphia: Wolters Kluwer Health/Lippincott Williams & Wilkins, 2013,
Chapter 10: Fetal Development and Genetics, p. 320.
Chapter 10: Fetal Development and Genetics - Page 320

Question 10: A client who is 37 years of age presents to the health care clinic for her
(see full question) first prenatal checkup. Due to her advanced age, the nurse should prepare
to talk with the client about her increased risk for what complication?
You selected: preterm labor
Incorrect
Correct response: genetic disorders
Explanation: Women over the age of 35 are at increased risk of having a fetus with an
abnormal karyotype or other genetic disorders. Gestational diabetes, an
incompetent cervix, and preterm labor are risks for any pregnant woman.
(less)

Reference: Ricci, S.S., Kyle, T., & Carman, S. Maternity and Pediatric Nursing, 2nd ed.
Philadelphia: Wolters Kluwer Health/Lippincott Williams & Wilkins, 2013,
Chapter 10: Fetal Development and Genetics, p. 326.
Chapter 10: Fetal Development and Genetics - Page 326
Answer Key
Question 1: What are the two fetal membranes?
(see full question)
You selected: chorion and amnion
Correct
Explanation: The chorion and amnion are the two fetal membranes. The ectoderm,
mesoderm, and endoderm are layers in the developing blastocyst.

Reference: Ricci, S.S., Kyle, T., & Carman, S. Maternity and Pediatric Nursing, 2nd ed.
Philadelphia: Wolters Kluwer Health/Lippincott Williams & Wilkins, 2013,
Chapter 10: Fetal Development and Genetics, pp. 308-311.
Chapter 10: Fetal Development and Genetics - Page 308

Question 2: The nurse prepares a couple to have a karyotype performed. What


(see full question) describes a karyotype?
You selected: the dominant gene that will exert influence over a correspondingly located
recessive gene
Incorrect
Correct response: a visual presentation of the chromosome pattern of an individual
Explanation: A karyotype is a photograph of a person's chromosomes aligned in order.
Reference: Ricci, S.S., Kyle, T., & Carman, S. Maternity and Pediatric Nursing, 2nd ed.
Philadelphia: Wolters Kluwer Health/Lippincott Williams & Wilkins, 2013,
Chapter 10: Fetal Development and Genetics, p. 320.
Chapter 10: Fetal Development and Genetics - Page 320

Question 3: During a clinical rotation at a prenatal clinic, a client asks a nurse what
(see full question) causes certain birth defects. The nurse replies that they can be caused by
teratogens. What does the severity of the defects depend on? Select all
that apply.
You selected: when during development the conceptus is exposed to the teratogen
the particular teratogen to which the fetus is exposed
Correct
Explanation: A teratogen is a substance that causes birth defects. The severity of the
defect depends on when during development the conceptus is exposed to
the teratogen and the particular teratogenic agent to which the fetus is
exposed. (less)

Reference: Ricci, S.S., Kyle, T., & Carman, S. Maternity and Pediatric Nursing, 2nd ed.
Philadelphia: Wolters Kluwer Health/Lippincott Williams & Wilkins, 2013,
Chapter 10: Fetal Development and Genetics, p. 327.
Chapter 10: Fetal Development and Genetics - Page 327

Question 4: A fertilized ovum is known as which structure?


(see full question)
You selected: zygote
Correct
Explanation: A fertilized ovum is known as a zygote and is the beginning of potential
individual human development. The developing human organism is known
as an embryo from the time it implants ... (more)

Reference: Ricci, S.S., Kyle, T., & Carman, S. Maternity and Pediatric Nursing, 2nd ed.
Philadelphia: Wolters Kluwer Health/Lippincott Williams & Wilkins, 2013,
Chapter 10: Fetal Development and Genetics, p. 311.
Chapter 10: Fetal Development and Genetics - Page 311

Question 5: The nurse is caring for a child with Down syndrome (trisomy 21). This is an
(see full question) example of which type of inheritance?
You selected: Mendelian recessive
Incorrect
Correct response: chromosome nondisjunction
Explanation: Down syndrome occurs when an ovum or sperm cell does not divide
evenly, permitting an extra 21st chromosome to cross to a new cell.

Reference: Ricci, S.S., Kyle, T., & Carman, S. Maternity and Pediatric Nursing, 2nd ed.
Philadelphia: Wolters Kluwer Health/Lippincott Williams & Wilkins, 2013,
Chapter 10: Fetal Development and Genetics, p. 325.
Chapter 10: Fetal Development and Genetics - Page 325

Question 6: The nurse is developing a presentation for a community group of young


(see full question) adults discussing fetal development and pregnancy. The nurse would
identify that the sex of offspring is determined at the time of:
You selected: fertilization.
Correct
Explanation: Sex determination occurs at the time of fertilization. Meiosis refers to cell
division resulting in the formation of an ovum or sperm with half the
number of chromosomes. The morula develops after a series of four
cleavages following the formation of the zygote. Oogenesis refers to the
development of a mature ovum, which has half the number of
chromosomes. (less)

Question 7: After the nurse describes fetal circulation to a pregnant woman, the
(see full question) woman asks why her fetus has a different circulation pattern than hers.
When responding to the client, the nurse integrates understanding of what
information?
You selected: Fetal heart rates are rapid and circulation time is double that of adults.
Incorrect
Correct response: Fetal circulation carries highly oxygenated blood to vital areas first.
Explanation: Fetal circulation carries highly oxygenated blood to vital areas first while
shunting it away from less vital ones. Fetal blood is not thicker than that of
adults. Large volumes of oxygenated blood are not needed because the
placenta essentially takes over the functions of the lung and liver during
fetal life. Although fetal heart rates normally range from 120 to 160 beats
per minute, circulation time is not doubled. (less)

Question 8: A pregnant woman undergoing amniocentesis asks her nurse why the baby
(see full question) needs this fluid. What would be an accurate response from the nurse?
You selected: "Amniotic fluid cushions your baby to prevent injury."
Correct
Explanation: The amniotic fluid, kept inside the amnion, cushions the fetus against
injury, regulates temperature, and allows the fetus to move freely inside it,
which allows normal musculoskeletal development of the fetus. The
woman's blood supplies food toand carries wastes away fromthe fetus.
The placenta supplies the developing organism with food and oxygen; then
the umbilical cord connects the fetal blood vessels contained in the villi of
the placenta with those found within the fetal body. (less)

Reference: Ricci, S.S., Kyle, T., & Carman, S. Maternity and Pediatric Nursing, 2nd ed.
Philadelphia: Wolters Kluwer Health/Lippincott Williams & Wilkins, 2013,
Chapter 10: Fetal Development and Genetics, p. 311.
Chapter 10: Fetal Development and Genetics - Page 311

Question 9: A couple has just learned that their son will be born with Down syndrome.
(see full question) The nurse shows a lack of understanding when making which statement?
You selected: "I will alert your entire family about this so you don't have to."
Correct
Explanation: It is necessary to maintain confidentiality at all times, which prevents
health care providers from alerting family members about any inherited
characteristic unless the family member has given consent for the
information to be revealed. (less)

Reference: Ricci, S.S., Kyle, T., & Carman, S. Maternity and Pediatric Nursing, 2nd ed.
Philadelphia: Wolters Kluwer Health/Lippincott Williams & Wilkins, 2013,
Chapter 10: Fetal Development and Genetics, p. 327.
Chapter 10: Fetal Development and Genetics - Page 327
Question 10: A woman is to undergo chorionic villus sampling as part of a risk
(see full question) assessment for genetic disorders. What statement would the nurse include
when describing this test to the woman?
You selected: "A needle will be inserted directly into your fetus's umbilical vessel to
collect blood for testing."
Incorrect
Correct response: "A small piece of tissue from the fetal placenta will be removed and
analyzed."
Explanation: Percutaneous umbilical cord sampling involves the insertion of a needle
into the umbilical vessel. An amniocentesis involves the collection of
amniotic fluid from the amniotic sac. Fetal nuchal translucency involves the
use of intravaginal ultrasound to measure fluid collected in the
subcutaneous space between the skin and cervical spine of the fetus.
Chorionic villus sampling involves the removal of a small tissue specimen
from the fetal portion of the placenta. (less)

Reference: Ricci, S.S., Kyle, T., & Carman, S. Maternity and Pediatric Nursing, 2nd ed.
Philadelphia: Wolters Kluwer Health/Lippincott Williams & Wilkins, 2013,
Chapter 10: Fetal Development and Genetics, p. 329.
Chapter 10: Fetal Development and Genetics - Page 329

Question 11: During the development of the fetus, its chorionic villi eventually meet with
(see full question) an area of uterine tissue to form the placenta. Which statements
accurately describe a function of the placenta? Select all that apply.
You selected: It cushions the fetus against injury.
It carries waste away for excretion by the mother.
It produces hormones that help maintain the pregnancy.
Incorrect
Correct response: It slows the maternal immune response.
It produces hormones that help maintain the pregnancy.
It carries waste away for excretion by the mother.
Explanation: The placenta supplies the developing organism with food and oxygen,
carries waste away for excretion by the mother, slows the maternal
immune response so that the mother's body does not reject the fetal
tissues, and produces hormones that help maintain the pregnancy.
Wharton's jelly protects the umbilical cord and the foramen ovale permits
most of the blood to bypass the right ventricle. The amniotic fluid cushions
the fetus against injury. (less)

Reference: Ricci, S.S., Kyle, T., & Carman, S. Maternity and Pediatric Nursing, 2nd ed.
Philadelphia: Wolters Kluwer Health/Lippincott Williams & Wilkins, 2013,
Chapter 10: Fetal Development and Genetics, pp. 311-312.
Chapter 10: Fetal Development and Genetics - Page 311

Question 12: A man is heterozygous for cystic fibrosis (an autosomal recessive disorder).
(see full question) His partner, however, is homozygous for the healthy gene, meaning she is
not a carrier. What are the odds that their child will have the disease cystic
fibrosis?
You selected: 25%
Incorrect
Correct response: 0%
Explanation: If a man with the heterozygous genotype mated with a woman who had no
trait for cystic fibrosis, there would be a 50% chance a child born to the
couple would be completely disorder and carrier free, like the mother.
Likewise, there is a 50% chance their child will be heterozygous (a carrier),
like the father. There is no chance in this instance any of their children will
have the disorder. (less)

Question 13: A couple wants to start a family. They are concerned that their child will be
(see full question) at risk for cystic fibrosis because they each have a cousin with cystic
fibrosis. They are seeing a nurse practitioner for preconceptual counseling.
What would the nurse practitioner tell them about cystic fibrosis?
You selected: It is an autosomal recessive disorder.
Correct
Explanation: Cystic fibrosis is autosomal recessive. Nurses also consider other issues
when assessing the risk for genetic conditions in couples and families. For
example, when obtaining a preconception or prenatal family history, the
nurse asks if the prospective parents have common ancestors. This is
important to know because people who are related have more genes in
common than those who are unrelated, thus increasing their chance for
having children with autosomal recessive inherited condition such as cystic
fibrosis. Mitochondrial inheritance occurs with defects in energy conversion
and affects the nervous system, kidney, muscle, and liver. X-linked
inheritance, which has been inherited from a mutant allele of the mother,
affects males. Autosomal dominant is an X-linked dominant genetic
disease. (less)

Reference: Ricci, S.S., Kyle, T., & Carman, S. Maternity and Pediatric Nursing, 2nd ed.
Philadelphia: Wolters Kluwer Health/Lippincott Williams & Wilkins, 2013,
Chapter 10: Fetal Development and Genetics, pp. 322-323.
Chapter 10: Fetal Development and Genetics - Page 322

Question 14: Women having in vitro fertilization (IVF) can have both the egg and sperm
(see full question) examined for genetic disorders of single gene or chromosome concerns
before implantation.
You selected: True
Correct

Question 15: Cystic fibrosis is an example of which type of inheritance?


(see full question)
You selected: autosomal recessive
Correct
Explanation: Cystic fibrosis is an autosomal recessive inherited condition. Huntington
disease would be an example of an autosomal dominant inherited
condition. Hemophilia is an X-linked recessive inherited condition. Cleft lip
is a multifactorial inherited condition. (less)

Reference: Ricci, S.S., Kyle, T., & Carman, S. Maternity and Pediatric Nursing, 2nd ed.
Philadelphia: Wolters Kluwer Health/Lippincott Williams & Wilkins, 2013,
Chapter 10: Fetal Development and Genetics, pp. 322-323.
Chapter 10: Fetal Development and Genetics - Page 322

Question 16: When providing education on pre-conceptual considerations to a female


(see full question) client and her spouse, the nurse knows that teratogens during the
embryonic stage pose the greatest risk to the fetus. The nurse describes
this stage as occurring during which time frame?
You selected: weeks 2 to 8
Correct
Explanation: The embryonic stage produces the greatest risk of damaging effects to the
fetus because the cells are rapidly dividing and differentiating into specific
body structures. The embryonic stage of development begins at day 15
after conception and continues through week 8. Basic structures of all
major body organs and the main external features are completed during
this time period, including internal organs. Week 9 to birth is the fetal
stage; fertilization to the week 2 is the pre-embryonic stage. Weeks 2 to 8
are the time period known as the embryonic stage. (less)

Reference: Ricci, S.S., Kyle, T., & Carman, S. Maternity and Pediatric Nursing, 2nd ed.
Philadelphia: Wolters Kluwer Health/Lippincott Williams & Wilkins, 2013,
Chapter 10: Fetal Development and Genetics, p. 308.
Chapter 10: Fetal Development and Genetics - Page 308

Question 17: The fluid-filled, inner membrane sac surrounding the fetus is which
(see full question) structure?
You selected: amnion
Correct
Explanation: The fluid-filled, inner membrane sac surrounding the fetus is the amnion.
The chorion is the outer membrane surrounding the fetus. The
endometrium is the inner lining of the uterus. The decidua is the name
used for the endometrium during pregnancy. (less)
Reference: Ricci, S.S., Kyle, T., & Carman, S. Maternity and Pediatric Nursing, 2nd ed.
Philadelphia: Wolters Kluwer Health/Lippincott Williams & Wilkins, 2013,
Chapter 10: Fetal Development and Genetics, p. 311.
Chapter 10: Fetal Development and Genetics - Page 311

Question 18: A pregnant client in the first trimester asks the nurse about taking
(see full question) medications while she is pregnant. She tells the nurse that she heard that
it can be harmful to the fetus if medications are taken at certain times
during pregnancy. What is the best response by the nurse?
You selected: "Exposure to certain substances during the embryonic phase may be
harmful to the developing fetus."
Correct
Explanation: Exposure to a teratogen during the embryonic stage produces the greatest
damaging effects because cells are rapidly dividing and differentiating into
specific body structures.

Reference: Ricci, S.S., Kyle, T., & Carman, S. Maternity and Pediatric Nursing, 2nd ed.
Philadelphia: Wolters Kluwer Health/Lippincott Williams & Wilkins, 2013,
Chapter 10: Fetal Development and Genetics, p. 315.
Chapter 10: Fetal Development and Genetics - Page 315

Question 19: The fetus receives blood flow from the mother via the placenta and
(see full question) umbilical cord. What is the route of fetal circulation through the umbilical
cord?
You selected: The one umbilical vein carries oxygen rich blood to the fetus from the
placenta.
Correct
Explanation: There are two umbilical arteries and one umbilical vein. The arteries carry
waste from the fetus to the placenta; the vein carries oxygenated blood to
the fetus from the placenta.

Reference: Ricci, S.S., Kyle, T., & Carman, S. Maternity and Pediatric Nursing, 2nd ed.
Philadelphia: Wolters Kluwer Health/Lippincott Williams & Wilkins, 2013,
Chapter 10: Fetal Development and Genetics, p. 315.
Chapter 10: Fetal Development and Genetics - Page 315

Question 20: A woman is taking vaginal progesterone suppositories during her first
(see full question) trimester because her body does not produce enough of it naturally. She
asks the nurse what function this hormone has in her pregnancy. What
should the nurse explain is the primary function of progesterone?
You selected: maintains the endometrial lining of the uterus during pregnancy
Correct
Explanation: Progesterone is necessary to maintain the endometrial lining of the uterus
during pregnancy. It is human chorionic gonadotropin (hCG) that acts to
ensure the corpus luteum of the ovary continues to produce estrogen and
progesterone. Estrogen contributes to mammary gland development, and
human placental lactogen regulates maternal glucose, protein, and fat
levels. (less)

A newborn that has a surfactant deficiency will have which assessment noted on a physical exam?
a) grunting
b) hypertension
c) pink skin
regular respirations
d)
The nurse is caring for a large-for-gestational-age newborn (also known as macrosomia). What maternal condition is
the usual cause of this condition?
a) alcohol use
b) hypertension
c) celiac disease
d) Diabetes
Infants of drug-dependent women tend to be large for gestational age.
a) False
b) True
An 18-year-old client has given birth to a very-low-birth-weight preterm infant. Which intervention should the nurse
consider to prevent the newborn from losing body temperature?

a) Hold the newborn close, rocking gently.


b) Administer vitamin K to the newborn.
c) Provide isolette or radiant warmer care to the newborn.
d) Give the newborn a warm water bath.

When caring for a neonate of a mother with diabetes, which physiologic finding is most indicative of a hypoglycemic
episode?
a) jitteriness
b) hyperalert state
c) serum glucose level of 60 mg/dl
d) loud and forceful crying
The nurse weighs the new infant and calculates the child's measurements. The new mom asks, Did my baby grow
well? The doctor said he was LGA: What does that mean? What is the best explanation?

a) That means your baby is over the 90th percentile for weight.
b) That means your baby is average for gestational age.
c) That means your baby is in the 5th percentile for weight.
d) That means that your baby is lazy sometimes.
A woman who has given birth to a postterm newborn asks the nurse why her baby looks so thin with so little
muscle. The nurse integrates understanding about which concept when responding to the mother?
a
The newborn was exposed to an infection while in utero.
)
b
With postterm birth, the fetus uses stored nutrients to stay alive, and wasting occurs.
)
c
A postterm newborn has begun to break down red blood cells more quickly.
)
d
The newborn aspirated meconium, causing the wasted appearance.
)
A nursing student observes that the babies in the nursery are wrapped up warmly and are wearing knit caps. Which
explanation by the nursery staff would be correct?
a
"That's how we have always done it, and it seems to work out well."
)
b
"Newborns lose body heat easily and need to be kept warm until their body temperature stabilizes."
)
c
"The caps and blankets simulate the temperature of the mother's womb that they are used to."
)
d
"Studies show that newborns like the extra warmth."
)
A mother is concerned because her newborn daughter has lost 8 ounces within 3 days after birth. What response by
the nurse correctly addresses this concern?
a
This is a normal and expected finding.
)
b
This is not normal. Your baby needs to be checked for a possible illness.
)
c
Your baby is probably just dehydrated. You need to breastfeed her more often.
)
d
You need to give your baby formula since she has lost weight during breastfeeding.
)
A mother asks the nurse why her newborn is getting a Vitamin K injection in the birth room. The nurse explains that
the injection is necessary because:
a vitamin K aids in protein metabolism. Newborns have defective protein metabolism until 24 hours of
) life.
b
the mother was febrile at the time of birth and prophylactic Vitamin K is necessary.
)
c vitamin K is needed for coagulation, and the newborn does not produce vitamin K in the few days
) following birth.
d
newborns are prone to hypoglycemia, and Vitamin K helps maintain a steady blod glucose level.
)
When caring for a newborn several hours after birth, the nurse assesses his respiratory rate and counts it at 42
breaths per minute. Which intervention should be implemented?
a) Further assess the newborn for abnormal lung sounds.
b) Stimulate the newborn to cry.
c) Contact the pediatrician to report the newborn's tachypnea.
d) Nothing since this is a normal reading.
A neonate is born, and the nurse realizes that the infant is at risk for evaporative heat loss. Which intervention
would best prevent this from occurring?
a) Wrap the infant in a warm, dry blanket.
b) Bathe the infant immediately after birth.
c) Turn the birth room temperature up.
d) Place the infant in the mother's abdomen after birth.?

The heart rate of the newborn in the first few minutes after birth will be in which range?
a) 180 to 220 bpm
b) 120 to 130 bpm
c) 120 to 180 bpm
d) 80 to 120 bpm
nfants have a substance in their lungs, surfactant. What is the role of surfactant in the respiratory system?
a) It keeps alveoli from collapsing with breaths.
b) It allows oxygen to move in the lungs.
c) It expands the lungs with breaths.
d) It removes fluid from the lungs.
A nurse is explaining the benefits of breastfeeding to a client who has just given birth. Which statement correctly
explains the benefits of breastfeeding to this mother?

a) Mothers who breastfeed have increased breast size following nursing.


b) Breastfeeding provides more iron and calcium for the infant.
c) Breastfed infants gain weight faster than formula fed infants after 6 month of age.
d) Immunoglobulin IgA in breast milk boosts a newborns immune system.
A woman comes to the nursery and states "Mrs. Smith is ready for her baby. I will be glad to take the baby to her."
The woman is dressed in hospital scrub attire but has no name badge showing. What is the best response by the
nurse caring for the baby?
a) "Leave immediately! I'm calling security."
b) "I don't know you. Are you trying to take a baby?"
c) "You must be Mrs. Smith's sister. She said her sister is a nurse."
d) "May I see your identification, please?"
A nurse is discharge teaching with a group of new parents before they are discharged home with their infants. One
couple inquires as to why they need to place their new baby on its back to sleep. What is the nurse's bestresponse?
a
"Sleeping on the stomach is fine, too."
)
b
"It really isn't important how you place your baby for it to sleep as long as it is comfortable."
)
c
"Research has shown that placing an infant on its back to sleep reduces the risk for SIDS."
)
d
"Research has found that sleeping on their backs reduces the infants' risk of esophageal reflux."
)

A father asks the nurse what medication is in the baby's eyes and why it is needed. Which explanation is correct?
a
to destroy an infectious exudate of the vaginal canal
)
b
to prevent infection of the baby's eyes by bacteria which may have been in the vaginal canal
)
c
to prevent potentially harmful virus from invading the tear ducts
)
d
to prevent the baby's eyelids from sticking together to help see
)
An African American baby has discoloring which appears similar to bruising on his buttock after a normal vaginal
birth. This assessment should be documented as:
a) vascular nevi.
b) lanugo.
c) Mongolian spots.
d) bruising.
When caring for a newborn several hours after birth, the nurse assesses the newborn's respiratory rate. In a normal
newborn, this would be:
a) 16 to 20 breaths per minute.
b) 20 to 30 breaths per minute.
c) 30 to 60 breaths per minute.
d) 12 to 16 breaths per minute.
The infant has APGAR scores of 7 at 1 minute and 9 at 5 minutes. What is the indication of this assessment finding?
a) moderate difficulty and may need intervention
b) severe distress and absolute need of resuscitation
c) adjusting to extrauterine life
d) predicts fair neurologic future outcomes
The standard of care and recommendation by the Centers for Disease Control is to administer an immunization to
all newborns. Which immunization is recommended to be administered prior to discharge?
a) DTaP
b) HiB
c) hep B
d) Prevnar
On examination, the hands and feet of a 12-hour-old infant are cyanotic without other signs of distress. The nurse
should document this as:
a) potential for respiratory distress.
b) acrocyanosis.
c) poor oxygenation.
d) cold stress.
The Ballard scoring system evaluates newborns on which two factors?
a) physical maturity and neuromuscular maturity
b) body maturity and cranial nerve maturity
c) tone maturity and extremities maturity
d) skin maturity and reflex maturity
The AGPAR score is based on which 5 parameters?
a
heart rate, muscle tone, reflex irritability, respiratory effort, and color
)
b
heart rate, breaths per minute, irritability, reflexes, and color
)
c
heart rate, respiratory effort, temperature, tone, and color
)
d
heart rate, breaths per minute, irritability, tone, and color
)

The four essential components of labor are known as the four Ps. Which of the four Ps involves the pelvis?

a) passenger
b) psyche
c) passageway
d) powers
Which type of pelvis has a roomy, round inlet and is most favorable for vaginal birth?
a) gynecoid
b) anthropoid
c) platypelloid
d) android

A nurse is assisting a client who is in the first stage of labor. Which principle should the nurse keep in mind to help
make this client's labor and birth as natural as possible?
a
Women should be able to move about freely throughout labor.
)
b
Routine intravenous fluid should be implemented.
)
c
A woman should be allowed to assume a supine position.
)
d The support person's access to the client should be limited to prevent the client from becoming
) overwhelmed.
The RN in labor and birth documents the fetus as ROA. To what does this documentation refer for a fetus?
a) fetal size
b) fetal station
c) fetal attitude
d) fetal position
The initial descent of the fetus into the pelvis to zero station is which one of the cardinal movements of labor?
a) expulsion
b) flexion
c) engagement
d) extension
The nurse is measuring a contraction from the beginning of the increment to the end of the decrement for the same
contraction. The nurse would document this as which finding?
a) peak
b) frequency
c) duration
d) intensity
A nurse is coaching a woman during the second stage of labor. Which action should the nurse encourage the client
to do at this time?
a) Hold the breath while pushing during contractions.
b) Push with contractions and rest between them.
c) Pant while pushing.
d) Begin pushing as soon as the cervix has dilated to 8 cm.
The skull is the most important factor in relation to the labor and birth process. The fetal skull must be small enough
to travel through the bony pelvis. What feature of the fetal skull helps to make this passage possible?
a) caput succedaneum
b) vertex presentation
c) molding
d) cephalohematoma
What term is used to describe the position of the fetal long axis in relation to the long axis of the mother?
a) fetal position
b) fetal lie
c) fetal attitude
d) fetal presentation
The labor and delivery nurse has responsibility for monitoring fetal heart rate patterns. Which of the following
values would indicate fetal tachycardia?
a) > 160 bpm
b) > 120 bpm
c) > 80 bpm
d) > 100 bpm
The nurse is aware that cord compression is not continuous when variable decelerations occur and that
compression happens when which of the following takes place?
a) prematurity
b) The uterus relaxes between contractions.
c) fetal sleep
d) The uterus contracts and squeezes the cord against the fetus.
What is the normally accepted fetal heart rate range?
a) 110160 bpm
b) 120170 bpm
c) 100150 bpm
d) 90140 bpm
A client is in the first stage of labor, latent phase. Her membranes are intact, and her contractions are mild.
Considering the client's condition and phase of labor, the nurse knows that which aid will facilitate labor?
a) ambulation ad lib
b) bathroom privileges
c) complete bed rest
d) up in chair TID

The nurse in the emergency department is administering a nonsteroidal anti-inflammatory drug to a client who has
a migraine headache. The client is nauseated, so the nurse obtains an order to administer which NSAID drug IV?

Ketorolac tromethamine is the only NSAID that may be given IV.

Which of the following effects can occur as a result of the beta blocking?
Decrease in the plasma renin level
Explanation: The beta-blocking effect of labetalol results in a decrease in the plasma renin level. The
alpha-blocking action causes orthostatic hypotension. The beta-blocking action
prevents reflex tachycardia and elevations in blood pressure.
Question 3: A nurse should include which of the following information when educating the client's
(see full question) parents on the varicella vaccine? Select all that apply:
You selected: Provide the date for return for the next vaccination.
Discuss common adverse reactions.
Instruct the parents to bring immunization records to all visits.
Incorrect
Correct response: The risk of contracting vaccine-preventable diseases.
The benefits of immunization.
Provide the date for return for the next vaccination.
Discuss common adverse reactions.
Instruct the parents to bring immunization records to all visits.
Explanation: The following information should be included when educating the parents of a client
receiving a vaccination: the risk of contracting vaccine-preventable diseases, the
benefits of immunization, instruct the parents to bring immunization records to all
visits, provide the date for return for the next vaccination, discuss common adverse
reactions, and instruct parents to report any unusual or severe adverse reactions after
the administration of a vaccination.
Question 18: Ms. Catalin has been started on labetalol. Which of the following effects can occur as a
(see full question) result of the beta blocking?
Correct response: Decrease in the plasma renin level
Explanation: The beta-blocking effect of labetalol results in a decrease in the plasma renin level. The
alpha-blocking action causes orthostatic hypotension. The beta-blocking action
prevents reflex tachycardia and elevations in blood pressure. (less)

Question 9: Which of the following should the client be told about nasal drops? Select all that apply:
(see full question)
Correct response: Recline on a bed and hang your head over the edge.
After using the drops keep head down and rotate it from side to side.
Do not share the container with anyone.
Nasal burning or stinging may occur with the use of topical decongestants.
Explanation: A client should be counseled to administer nasal drops while reclined on a bed with
their head over the edge and after using the drops to keep the head dose and rotate it
from side to side. The client may experience some burning or stinging after the
administration of the nasal spray. The container should not be shared with anyone.
(less)
Question 11: A patient has been prescribed a decongestant drug for congestion associated with
(see full question) rhinitis. Which of the following possible side effects of the drug should the nurse
caution the patient about?
Correct response: Blurred vision
Explanation: The nurse should inform the patient that blurred vision is a possible side effect of
decongestant drugs. Additionally, an increased and not decreased pulse rate may also
be seen. Drowsiness is not seen with decongestant usage. Dryness of the nasal mucosa
and not the throat may be seen with decongestant drugs, which are used mostly as
topical sprays and drops.
Question 23: A client with asthma should always carry a rescue inhaler or quick-relief medication
(see full question) with them at all times. Which of the following are considered quick-relief medications?
Select all that apply:
Correct response: Metaproterenol (Alupent)
Albuterol (Proventil)
Explanation: Short-acting beta agonists (SABAs) are used as rescue treatment for asthma. B and D
are SABAs, A and E are long-acting beta agonists (LABAs). C is a cholinergic-blocking
drug.

Question 3: Which of the following exerts its effects by depressing the reabsorption of sodium in
(see full question) the kidney tubules, thereby increasing sodium and water excretion? (Choose one)
Correct response: Triamterene (Dyrenium)
Explanation: Triamterene (Dyrenium) is a potassium sparing diuretic that exerts its effect by
depressing the reabsorption of sodium in the kidney tubules, thereby increasing
sodium and water excretion. (less)
Question 3: Which of the following exerts its effects by depressing the reabsorption of sodium in
(see full question) the kidney tubules, thereby increasing sodium and water excretion? (Choose one)
Correct response: Triamterene (Dyrenium)
Explanation: Triamterene (Dyrenium) is a potassium sparing diuretic that exerts its effect by
depressing the reabsorption of sodium in the kidney tubules, thereby increasing
sodium and water excretion.
Which of the following are methods used to treat a drop in respiratory rate in a client receiving an opioid analgesic?
Select all that apply:
Stopping the opioid analgesic.
Administering naloxone (Narcan).
Explanation: Coaching the client to breathe or administering naloxone (in severe cases) are
methods used to treat a drop in respiratory rate in a client receiving an opioid
analgesic

A booster with haemophilus influenza B conjugate vaccine and hepatitis B surface antigen should be given at 15 to
18 months.
Answer Key
Question 1: A variety of complications can occur after a leg amputation. All of the following are
(see full question) possibilities in the immediate postoperative period, except?
You selected: Osteomyelitis
Correct
Explanation: Chronic osteomyelitis may occur after persistent infection in the late postoperative
period. Hematoma, hemorrhage, and infection are potential complications in the
immediate postoperative period. (less)

Reference: Hinkle, J.L., and Cheever, K.H. Brunner & Suddarth's Textbook of Medical-Surgical
Nursing, 13th ed. Philadelphia: Lippincott Williams & Wilkins, 2014, Chapter 43:
Management of Patients With Musculoskeletal Trauma, p. 1186.
Chapter 41: Musculoskeletal Care Modalities - Page 1186

Question 2: The client with a fractured femur is upset and agitated that skeletal traction will be
(see full question) necessary for 6 to 8 weeks. The client states, How can I stay like this for weeks? I
cant even move! Based on these statements, the nurse would identify which of
the following as the most appropriate nursing diagnosis?
You selected: Ineffective Coping related to prolonged immobility
Correct
Explanation: The client is displaying clinical manifestations of anxiety and ineffective coping.
Reference: Hinkle, J.L., and Cheever, K.H. Brunner & Suddarth's Textbook of Medical-Surgical
Nursing, 13th ed. Philadelphia: Lippincott Williams & Wilkins, 2014, Chapter 41:
Musculoskeletal Care Modalities, p. 1115.
Chapter 41: Musculoskeletal Care Modalities - Page 1115

Question 3: A patient has a fracture that is being treated with open rigid compression plate
(see full question) fixation devices. How will the progress of bone healing be monitored?
You selected: Serial x-rays
Correct
Explanation: Serial x-rays are used to monitor the progress of bone healing.
Reference: Hinkle, J.L., and Cheever, K.H. Brunner & Suddarth's Textbook of Medical-Surgical
Nursing, 13th ed. Philadelphia: Lippincott Williams & Wilkins, 2014, Chapter 40:
Assessment of Musculoskeletal Function, p. 1090.
Chapter 40: Assessment of Musculoskeletal Function - Page 1090

Question 4: Which area of the spinal column is subject to the greatest mechanical stress and
(see full question) degenerative changes?
You selected: Lower lumbar
Correct
Explanation: The lower lumbar disks, L4 to L5 and L5 to S1, are subject to the greatest
mechanical stress and greatest degenerative changes.

Reference: Hinkle, J.L., and Cheever, K.H. Brunner & Suddarth's Textbook of Medical-Surgical
Nursing, 13th ed. Philadelphia: Lippincott Williams & Wilkins, 2014, Chapter 42:
Management of Patients With Musculoskeletal Disorders, p. 1133.
Chapter 42: Management of Patients With Musculoskeletal Disorders - Page 1133

Question 5: The nurse is performing an assessment on an older adult patient and observes the
(see full question) patient has an increased forward curvature of the thoracic spine. What does the
nurse understand this common finding is known as?
You selected: Kyphosis
Correct
Explanation: Common deformities of the spine include kyphosis, which is an increased forward
curvature of the thoracic spine that causes a bowing or rounding of the back,
leading to a hunchback or slouching posture. The second deformity of the spine is
referred to as lordosis, or swayback, an exaggerated curvature of the lumbar
spine. A third deformity is scoliosis, which is a lateral curving deviation of the
spine (Fig. 40-4). Osteoporosis is abnormal excessive bone loss. (less)

Reference: Hinkle, J.L., and Cheever, K.H. Brunner & Suddarth's Textbook of Medical-Surgical
Nursing, 13th ed. Philadelphia: Lippincott Williams & Wilkins, 2014, Chapter 40:
Assessment of Musculoskeletal Function, p. 1095.
Chapter 40: Assessment of Musculoskeletal Function - Page 1095

Question 6: Which of the following orthopedic surgeries is done to correct and align a fracture
(see full question) after surgical dissection and exposure of the fracture?
You selected: Open reduction
Correct
Explanation: An open reduction is the correction and alignment of the fracture after surgical
dissection and exposure of the fracture. Arthrodesis is immobilizing fusion of a
joint. A joint arthroplasty or replacement is the replacement of joint surfaces with
metal or synthetic materials. A total joint arthroplasty is the replacement of both
the articular surfaces within a joint with metal or synthetic materials. (less)

Reference: Hinkle, J.L., and Cheever, K.H. Brunner & Suddarth's Textbook of Medical-Surgical
Nursing, 13th ed. Philadelphia: Lippincott Williams & Wilkins, 2014, Chapter 41:
Musculoskeletal Care Modalities, p. 1116.
Chapter 41: Musculoskeletal Care Modalities - Page 1116

Question 7: On a visit to the family physician, a client is diagnosed with a bunion on the lateral
(see full question) side of the great toe, at the metatarsophalangeal joint. Which statement should
the nurse include in the teaching session?
You selected: "Bunions may result from wearing shoes that are too big, causing friction when the
shoes slip back and forth."
Incorrect
Correct response: "Some bunions are congenital; others are caused by wearing shoes that are too
short or narrow."
Explanation: Bunions may be congenital or may be acquired by wearing shoes that are too
short or narrow, which increases pressure on the bursa at the
metatarsophalangeal joint. Acquired bunions can be prevented. Wearing shoes
that are too big may cause other types of foot trauma but not
bunions. Gout doesn't cause bunions. Although a client with gout may have pain in
the big toe, such pain doesn't result from a bunion. (less)

Reference: Hinkle, J.L., and Cheever, K.H. Brunner & Suddarth's Textbook of Medical-Surgical
Nursing, 13th ed. Philadelphia: Lippincott Williams & Wilkins, 2014, Chapter 42:
Management of Patients With Musculoskeletal Disorders, p. 1140.
Chapter 42: Management of Patients With Musculoskeletal Disorders - Page 1140

Question 8: A client experiences a musculoskeletal injury that involves the structure that
(see full question) connects a muscle to the bone. The nurse understands that this injury involves
which of the following?
You selected: Tendon
Correct
Explanation: Tendons are cordlike structures that attach muscles to the periosteum of the bone.
Ligaments consisting of fibrous tissue connect two adjacent, freely movable
bones. Cartilage is a firm dense type of connective tissue that reduces friction
between articular surfaces, absorbs shock, and reduces the stress on joint
surfaces. A joint is the junction between 2 or more bones. (less)

Reference: Hinkle, J.L., and Cheever, K.H. Brunner & Suddarth's Textbook of Medical-Surgical
Nursing, 13th ed. Philadelphia: Lippincott Williams & Wilkins, 2014, Chapter 40:
Assessment of Musculoskeletal Function, p. 1087.
Chapter 40: Assessment of Musculoskeletal Function - Page 1087

Question 9: Which of the following is the only selective estrogen receptor modulator approved
(see full question) for osteoporosis in post menopausal women?
You selected: Fosamax
Incorrect
Correct response: Raloxifene
Explanation: Raloxifene is the only selective estrogen receptor modulator (SERM) approved for
osteoporosis in post menopausal women as it does not increase the risk of breast
or uterine cancer, but it does come with an increased risk of thromboembolism.
Fosamax is a bisphosphonate. Forteo is a subcutaneously administered medication
that is given one daily for the treatment of osteoporosis. Denosumab has recently
been approved for treatment of postmenopausal women with osteoporosis who
are at risk for fractures. (less)

Reference: Hinkle, J.L., and Cheever, K.H. Brunner & Suddarth's Textbook of Medical-Surgical
Nursing, 13th ed. Philadelphia: Lippincott Williams & Wilkins, 2014, Chapter 42:
Management of Patients With Musculoskeletal Disorders, p. 1144.
Chapter 42: Management of Patients With Musculoskeletal Disorders - Page 1144

Question 10: The nurse recognizes that goal of treatment for metastatic bone cancer is to:
(see full question)
You selected: Promote pain relief and quality of life
Correct
Explanation: Treatment of metastatic bone cancer is palliative.
Reference: Hinkle, J.L., and Cheever, K.H. Brunner & Suddarth's Textbook of Medical-Surgical
Nursing, 13th ed. Philadelphia: Lippincott Williams & Wilkins, 2014, Chapter 42:
Management of Patients With Musculoskeletal Disorders, p. 1152.
Chapter 42: Management of Patients With Musculoskeletal Disorders - Page 1152

Question 11: Which of the following is an indicator of neurovascular compromise?


(see full question)
You selected: Capillary refill of more than 3 seconds
Correct
Explanation: Capillary refill of more than 3 seconds is an indicator of neurovascular
compromise. Other indicators include cool skin temperature, pale or cyanotic
color, weakness, paralysis, paresthesia, unrelenting pain, pain on passive stretch,
and absence of feeling. Cool skin temperature is an indicator of neurovascular
compromise. Unrelenting pain is an indicator of neurovascular compromise. Pain
on passive stretch is an indicator of neurovascular compromise. (less)

Reference: Hinkle, J.L., and Cheever, K.H. Brunner & Suddarth's Textbook of Medical-Surgical
Nursing, 13th ed. Philadelphia: Lippincott Williams & Wilkins, 2014, Chapter 40:
Assessment of Musculoskeletal Function, p. 1099.
Chapter 40: Assessment of Musculoskeletal Function - Page 1099

Question 12: The nurse is preparing a client for a hip replacement with the use of porous-coated
(see full question) cementless joint components. What does the nurse know is the benefit of this type
of component?
You selected: It prevents the client from developing infection related to the application of
cement in the joint spaces.
Incorrect
Correct response: It allows the bone to grow into the prosthesis and securely fix the joint
replacement in place.
Explanation: Porous-coated cementless joint components are used to allow the bone to grow
into the prosthesis and thus securely fix the joint replacement in place. The
prosthesis is not less expensive and cost is not a factor in reconstruction. The
client may still have a local or systemic reaction to the prostheses even if it does
not have cement. (less)

Reference: Hinkle, J.L., and Cheever, K.H. Brunner & Suddarth's Textbook of Medical-Surgical
Nursing, 13th ed. Philadelphia: Lippincott Williams & Wilkins, 2014, Chapter 41:
Musculoskeletal Care Modalities, p. 1117.
Chapter 41: Musculoskeletal Care Modalities - Page 1117

Question 13: Which assessment finding would cause the nurse to suspect compartment
(see full question) syndrome in the client following a bone biopsy?
You selected: Increased diameter of the calf
Correct
Explanation: Increasing diameter of the calf can be indicative of bleeding into the muscle. The
other findings are within normal limits.

Reference: Hinkle, J.L., and Cheever, K.H. Brunner & Suddarth's Textbook of Medical-Surgical
Nursing, 13th ed. Philadelphia: Lippincott Williams & Wilkins, 2014, Chapter 40:
Assessment of Musculoskeletal Function, p. 1098.
Chapter 40: Assessment of Musculoskeletal Function - Page 1098

Question 14: The nurse is planning discharge instructions for the client with osteomyelitis. What
(see full question) instructions should the nurse include in the discharge teaching?
You selected: You will receive IV antibiotics for 3 to 6 weeks.
Correct
Explanation: Treatment of osteomyelitis requires IV antibiotics for 3 to 6 weeks.
Reference: Hinkle, J.L., and Cheever, K.H. Brunner & Suddarth's Textbook of Medical-Surgical
Nursing, 13th ed. Philadelphia: Lippincott Williams & Wilkins, 2014, Chapter 42:
Management of Patients With Musculoskeletal Disorders, p. 1148.
Chapter 42: Management of Patients With Musculoskeletal Disorders - Page 1148

Question 15: What food can the nurse suggest to the client at risk for osteoporosis?
(see full question)
You selected: Chicken
Incorrect
Correct response: Broccoli
Explanation: Calcium is important for the prevention of osteoporosis. Broccoli is high in calcium.
Reference: Hinkle, J.L., and Cheever, K.H. Brunner & Suddarth's Textbook of Medical-Surgical
Nursing, 13th ed. Philadelphia: Lippincott Williams & Wilkins, 2014, Chapter 42:
Management of Patients With Musculoskeletal Disorders, p. 1141.
Chapter 42: Management of Patients With Musculoskeletal Disorders - Page 1141

Question 16: A nurse is caring for a client who recently underwent a total hip replacement. The
(see full question) nurse should:
You selected: limit hip flexion of the client's hip when he sits.
Correct
Explanation: The nurse should instruct the client to limit hip flexion to 90 degrees when he sits.
The nurse should supply an elevated toilet seat so that the client can sit without
having to flex his hip more than 90 degrees. The nurse should instruct the client
not to cross his legs to avoid dislodging or dislocating the prosthesis. The nurse
should caution the client against sitting in chairs that are too low or too soft; these
chairs increase flexion, which is undesirable. (less)

Reference: Hinkle, J.L., and Cheever, K.H. Brunner & Suddarth's Textbook of Medical-Surgical
Nursing, 13th ed. Philadelphia: Lippincott Williams & Wilkins, 2014, Chapter 41:
Musculoskeletal Care Modalities, p. 1119.
Chapter 41: Musculoskeletal Care Modalities - Page 1119

Question 17: A patient tells the nurse, I was working out and lifting weights and now that I
(see full question) have stopped, I am flabby and my muscles have gone! What is the best response
by the nurse?
You selected: While you are lifting weights, endorphins are released, creating increase in
muscle mass, but if the muscles are not used they will atrophy.
Incorrect
Correct response: Your muscles were in a state of hypertrophy from the weight lifting but it will
persist only if the exercise is continued.
Explanation: Muscles need to exercise to maintain function and strength. When a muscle
repeatedly develops maximum or close to maximum tension over a long time, as
in regular exercise with weights, the cross-sectional area of the muscle increases.
This enlargement, known as hypertrophy, results from an increase in the size of
individual muscle fibers without an increase in their number. Hypertrophy persists
only if the exercise is continued. (less)

Reference: Hinkle, J.L., and Cheever, K.H. Brunner & Suddarth's Textbook of Medical-Surgical
Nursing, 13th ed. Philadelphia: Lippincott Williams & Wilkins, 2014, Chapter 40:
Assessment of Musculoskeletal Function, p. 1093.
Chapter 40: Assessment of Musculoskeletal Function - Page 1093

Question 18: A patient who needs to increase her intake of calcium is advised to also increase
(see full question) her intake of another vitamin that supports calcium's absorption. Select that
vitamin.
You selected: Vitamin D
Correct
Explanation: To support the absorption of calcium from the gastrointestinal tract and increase
the amount of calcium in the blood, there needs to be sufficient active vitamin D.
Reference: Hinkle, J.L., and Cheever, K.H. Brunner & Suddarth's Textbook of Medical-Surgical
Nursing, 13th ed. Philadelphia: Lippincott Williams & Wilkins, 2014, Chapter 40:
Assessment of Musculoskeletal Function, p. 1089.
Chapter 40: Assessment of Musculoskeletal Function - Page 1089

Question 19: A client is brought to the emergency department by a softball team member
(see full question) whostates the client and another player ran into each other, and the client is
having severe pain in the right shoulder. What symptoms of a fractured clavicle
does the nurse recognize?
You selected: Right shoulder slopes downward and droops inward.
Correct
Explanation: The client with a fractured clavicle has restricted motion, and the affected
shoulder appears to slope downward and droop inward. The client will have pain,
not typically tingling and numbness in the right shoulder. Pain is not felt in the
unaffected shoulder. (less)

Reference: Hinkle, J.L., and Cheever, K.H. Brunner & Suddarth's Textbook of Medical-Surgical
Nursing, 13th ed. Philadelphia: Lippincott Williams & Wilkins, 2014, Chapter 43:
Management of Patients With Musculoskeletal Trauma, p. 1184.
Chapter 41: Musculoskeletal Care Modalities - Page 1184

Question 20: The nurse is caring for a client who lives alone and had a total knee replacement.
(see full question) An appropriate nursing diagnosis for the client is:
You selected: Risk for ineffective therapeutic regimen management
Correct
Explanation: The client without adequate support and resources is at risk for ineffective
therapeutic regimen management. A total knee replacement may be used to treat
avascular necrosis. While an orthopedic client is at risk for disturbed body image
and situational low self-esteem, there is no evidence that these exist for this
client. (less)

Reference: Hinkle, J.L., and Cheever, K.H. Brunner & Suddarth's Textbook of Medical-Surgical
Nursing, 13th ed. Philadelphia: Lippincott Williams & Wilkins, 2014, Chapter 41:
Musculoskeletal Care Modalities, p. 1128.
Chapter 41: Musculoskeletal Care Modalities - Page 1128
Answer Key
Question 1: An older adult client slipped on an area rug at home and fractured the left hip. The
(see full question) client is unable to have surgery immediately and is having severe pain. What
interventions should the nurse provide for the patient to minimize energy loss in
response to pain?
You selected: Avoid administering too much medication becausethe client is older.
Incorrect
Correct response: Administer prescribed analgesics around-the-clock.
Explanation: Pain associated with hip fracture is severe and must be carefully managed with
around-the-clock dosing of pain medication to minimize energy loss in response to
pain. The client may not request the medication even if they are in pain, and it
should be offered at the prescribed time. Give pain medication prior to providing
any type of care involved in moving the client. (less)

Reference: Hinkle, J.L., and Cheever, K.H. Brunner & Suddarth's Textbook of Medical-Surgical
Nursing, 13th ed. Philadelphia: Lippincott Williams & Wilkins, 2014, Chapter 43:
Management of Patients With Musculoskeletal Trauma, p. 1162.
Chapter 43: Management of Patients With Musculoskeletal Trauma - Page 1162

Question 2: A patient presents to the Emergency Department experiencing a severe anxiety


(see full question) attack and is hyperventilating. The nurse would expect the patients pH value to
be which of the following?
You selected: 7.50
Correct
Explanation: The patient is experiencing respiratory alkalosis. Respiratory alkalosis is a clinical
condition in which the arterial pH is greater than 7.45 and the PaCO 2 is less than
38 mm Hg. Respiratory alkalosis is always caused by hyperventilation, which
causes excessive blowing off of CO2 and, hence, a decrease in the plasma
carbonic acid concentration. Causes include extreme anxiety, hypoxemia, early
phase of salicylate intoxication, Gram-negative bacteremia, and inappropriate
ventilator settings. (less)
Reference: Hinkle, J.L., and Cheever, K.H. Brunner & Suddarth's Textbook of Medical-Surgical
Nursing, 13th ed., Philadelphia: Lippincott Williams & Wilkins, 2014, Chapter 13:
Fluid and Electrolytes: Balance and Disturbance, p. 270.
Chapter 13: Fluid and Electrolytes: Balance and Disturbance - Page 270

Question 3: Which of the following is a metabolic bone disease that is characterized by


(see full question) inadequate mineralization of bone?
You selected: Osteoporosis
Incorrect
Correct response: Osteomalacia
Explanation: Osteomalacia is a metabolic bone disease characterized by inadequate
mineralization of bone. Osteoporosis is characterized by reduction of total bone
mass and a change in bone structure that increases susceptibility to fracture.
Osteomyelitis is an infection of bone that comes from extension of soft tissue
infection, direct bone contamination, or hematogenous spread. Osteoarthritis (OA),
also known as degenerative joint disease, is the most common and frequently
disabling of the joint disorders. OA affects the articular cartilage, subchondral
bone, and synovium. (less)

Reference: Hinkle, J.L., and Cheever, K.H. Brunner & Suddarth's Textbook of Medical-Surgical
Nursing, 13th ed. Philadelphia: Lippincott Williams & Wilkins, 2014, Chapter 42:
Management of Patients With Musculoskeletal Disorders, p. 1146.
Chapter 42: Management of Patients With Musculoskeletal Disorders - Page 1146

Question 4: Which of the following is an inaccurate principle of traction?


(see full question)
You selected: Skeletal traction is interrupted to turn and reposition the patient.
Correct
Explanation: Skeletal traction is never interrupted. The weights are not removed unless
intermittent treatment is prescribed. The weights must hang freely with the
patient in good alignment in the center of the bed. (less)

Reference: Hinkle, J.L., and Cheever, K.H. Brunner & Suddarth's Textbook of Medical-Surgical
Nursing, 13th ed. Philadelphia: Lippincott Williams & Wilkins, 2014, Chapter 41:
Musculoskeletal Care Modalities, p. 1114.
Chapter 41: Musculoskeletal Care Modalities - Page 1114

Question 5: The nurse is caring for a client with multiple organ failure and in metabolic
(see full question) acidosis. Which pair of organs is responsible for regulatory processes and
compensation?
You selected: Lungs and kidney
Correct
Explanation: The lungs and kidneys facilitate the ratio of bicarbonate to carbonic acid. Carbon
dioxide is one of the components of carbonic acid. The lungs regulate carbonic
acid levels by releasing or conserving CO2 by increasing or decreasing the
respiratory rate. The kidneys assist in acidbase balance by retaining or excreting
bicarbonate ions. (less)

Reference: Hinkle, J.L., and Cheever, K.H. Brunner & Suddarth's Textbook of Medical-Surgical
Nursing, 13th ed. Philadelphia: Lippincott Williams & Wilkins, 2014, Chapter 13:
Fluid and Electrolytes: Balance and Disturbance, p. 267.
Chapter 13: Fluid and Electrolytes: Balance and Disturbance - Page 267

Question 6: Which type of fracture involves a break through only part of the cross-section of
(see full question) the bone?
You selected: Incomplete
Correct
Explanation: An incomplete fracture involves a break through only part of the cross-section of
the bone. A comminuted fracture is one that produces several bone fragments. An
open fracture is o ... (more)

Reference: Hinkle, J.L., and Cheever, K.H. Brunner & Suddarth's Textbook of Medical-Surgical
Nursing, 13th ed. Philadelphia: Lippincott Williams & Wilkins, 2014, Chapter 43:
Management of Patients With Musculoskeletal Trauma, p. 1160.
Chapter 43: Management of Patients With Musculoskeletal Trauma - Page 1160

Question 7: The nurse is caring for a client diagnosed with chronic obstructive pulmonary
(see full question) disease (COPD) and experiencing respiratory acidosis. The client asks what is
making the acidotic state. The nurse is most correct to identify which result of the
disease process that causes the rise in pH?
You selected: The lungs are unable to exchange oxygen and carbon dioxide.
Incorrect
Correct response: The lungs are not able to blow off carbon dioxide.
Explanation: In clients with chronic respiratory acidosis, the client is unable to blow off carbon
dioxide leaving in increased amount of hydrogen in the system. The increase in
hydrogen ions leads to acidosis. In COPD, the client is able to breathe in oxygen
and gas exchange can occur, it is the lungs ability to remove the carbon dioxide
from the system. Although individuals with COPD frequently have a history of
smoking, cilia is not the cause of the acidosis. (less)

Reference: Hinkle, J.L., and Cheever, K.H. Brunner & Suddarth's Textbook of Medical-Surgical
Nursing, 13th ed. Philadelphia: Lippincott Williams & Wilkins, 2014, Chapter 13:
Fluid and Electrolytes: Balance and Disturbance, p. 270.
Chapter 13: Fluid and Electrolytes: Balance and Disturbance - Page 270

Question 8: Which group is at the greatest risk for osteoporosis?


(see full question)
You selected: Caucasian women
Correct
Explanation: Small-framed, nonobese Caucasian women are at greatest risk for osteoporosis.
Asian women of slight build are at risk for low peak BMD. African American
women, who have a greater bone mass that Caucasian women, are less
susceptible to osteoporosis. Men have a greater peak bone mass and do not
experience sudden estrogen reduction. (less)

Reference: Hinkle, J.L., and Cheever, K.H. Brunner & Suddarth's Textbook of Medical-Surgical
Nursing, 13th ed. Philadelphia: Lippincott Williams & Wilkins, 2014, Chapter 42:
Management of Patients With Musculoskeletal Disorders, p. 1141.
Chapter 42: Management of Patients With Musculoskeletal Disorders - Page 1141

Question 9: A client is about to have a cast applied to his left arm. The nurse would alert the
(see full question) client to which of the following as the cast is applied?
You selected: Arm being moved to various positions
Incorrect
Correct response: Sensation of warmth or heat with application
Explanation: When a cast is applied, the client needs to be aware that he may feel a sensation
of warmth or heat due to the material being mixed with water. The client should
not feel an increase in pain during the application. The arm will be held in place to
ensure proper alignment during the application. The client should not feel
weakness in the extremity. This is more commonly experiences after a cast is
removed. (less)

Reference: Hinkle, J.L., and Cheever, K.H. Brunner & Suddarth's Textbook of Medical-Surgical
Nursing, 13th ed. Philadelphia: Lippincott Williams & Wilkins, 2014, Chapter 41:
Musculoskeletal Care Modalities, p. 1104.
Chapter 41: Musculoskeletal Care Modalities - Page 1104

Question 10: The patient presents to the emergency room with an open fracture of the femur.
(see full question) Which action would the nurse implement to prevent the most serious complication
of an open fracture?
You selected: Immobilize the joint to prevent movement of bone fragments.
Incorrect
Correct response: Cover the wound with a sterile dressing to prevent infection.
Explanation: The most important complication of an open fracture is infection. Therefore, the
wound is covered with a sterile dressing. No attempt is made to reduce the
fracture or apply pressure. (less)

Reference: Hinkle, J.L., and Cheever, K.H. Brunner & Suddarth's Textbook of Medical-Surgical
Nursing, 13th ed. Philadelphia: Lippincott Williams & Wilkins, 2014, Chapter 43:
Management of Patients With Musculoskeletal Trauma, p. 1162.
Chapter 43: Management of Patients With Musculoskeletal Trauma - Page 1162

Question 11: The nurse is caring for a patient undergoing alcohol withdrawal. Which of the
(see full question) following serum laboratory values should the nurse monitor most closely?
You selected: Potassium
Incorrect
Correct response: Magnesium
Explanation: Chronic alcohol abuse is a major cause of symptomatic hypomagnesemia in the
United States. The serum magnesium level should be measured at least every 2 or
3 days in patients undergoing alcohol withdrawal. The serum magnesium level
may be normal on admission but may decrease as a result of metabolic changes,
such as the intracellular shift of magnesium associated with IV glucose
administration. (less)

Reference: Hinkle, J.L., and Cheever, K.H. Brunner & Suddarth's Textbook of Medical-Surgical
Nursing, 13th ed., Philadelphia: Lippincott Williams & Wilkins, 2014, Chapter 13:
Fluid and Electrolytes: Balance and Disturbance, p. 262.
Chapter 13: Fluid and Electrolytes: Balance and Disturbance - Page 262

Question 12: A clients cast is removed. The client is worried because the skin appears mottled
(see full question) and is covered with a yellowish crust. What advice should the nurse give the client
to address the skin problem?
You selected: Consult a skin speciaqlist.
Incorrect
Correct response: Apply lotions and take warm baths or soaks.
Explanation: The client should be advised to apply lotions and take warm baths or soaks. This
will help in softening the skin and removing debris. The client usually sheds this
residue in a few days so the client need not consult a skin specialist. It is not
advisable to scrub the area vigorously. The client need not avoid exposure to
direct sunlight because the area is not photosensitive. (less)

Reference: Hinkle, J.L., and Cheever, K.H. Brunner & Suddarth's Textbook of Medical-Surgical
Nursing, 13th ed. Philadelphia: Lippincott Williams & Wilkins, 2014, Chapter 41:
Musculoskeletal Care Modalities, p. 1108.
Chapter 41: Musculoskeletal Care Modalities - Page 1108

Question 13: Which is a risk-lowering strategy for osteoporosis?


(see full question)
You selected: Smoking cessation
Correct
Explanation: Risk-lowering strategies include increased dietary calcium and vitamin D intake,
smoking cessation, alcohol and caffeine consumption in moderation, and outdoor
activity. Individual risk factors include low initial bone mass and increased age. A
lifestyle risk factor is a diet low in calcium and vitamin D. (less)

Reference: Hinkle, J.L., and Cheever, K.H. Brunner & Suddarth's Textbook of Medical-Surgical
Nursing, 13th ed., Philadelphia: Lippincott Williams & Wilkins, 2014, Chapter 42:
Management of Patients With Musculoskeletal Disorders, p. 1141.
Chapter 42: Management of Patients With Musculoskeletal Disorders - Page 1141

Question 14: Before seeing a newly assigned client with respiratory alkalosis, a nurse quickly
(see full question) reviews the client's medical history. Which condition is a predisposing factor
for respiratory alkalosis?
You selected: Extreme anxiety
Correct
Explanation: Extreme anxiety may lead to respiratory alkalosis by causing hyperventilation,
which results in excessive carbon dioxide (CO2) loss. Other conditions that may set
the stage for respiratory alkalosis include fever, heart failure, injury to the brain's
respiratory center, overventilation with a mechanical ventilator, pulmonary
embolism, and early salicylate intoxication. Type 1 diabetes may lead to diabetic
ketoacidosis; the deep, rapid respirations occurring in this disorder (Kussmaul's
respirations) don't cause excessive CO2 loss. Myasthenia
gravis and opioid overdose suppress the respiratory drive, causing CO2 retention,
not CO2 loss; this may lead to respiratory acidosis, not alkalosis. (less)

Reference: Hinkle, J.L., and Cheever, K.H. Brunner & Suddarth's Textbook of Medical-Surgical
Nursing, 13th ed. Philadelphia: Lippincott Williams & Wilkins, 2014, Chapter 13:
Fluid and Electrolytes: Balance and Disturbance, p. 270.
Chapter 13: Fluid and Electrolytes: Balance and Disturbance - Page 270

Question 15: There are a variety of problems that can become complications after a fracture.
(see full question) Which of the following is described as a condition that occurs from interruption of
the blood supply to the fracture fragments after which the bone tissue dies, most
commonly in the femoral head?
You selected: Avascular necrosis
Correct
Explanation: Avascular necrosis is described as a condition that occurs from interruption of the
blood supply to the fracture fragments after which the bone tissue dies, most
commonly in the femoral head. (less)

Reference: Hinkle, J.L., and Cheever, K.H. Brunner & Suddarth's Textbook of Medical-Surgical
Nursing, 13th ed. Philadelphia: Lippincott Williams & Wilkins, 2014, Chapter 43:
Management of Patients With Musculoskeletal Trauma, p. 1157.
Chapter 43: Management of Patients With Musculoskeletal Trauma - Page 1157

Question 16: The nurse is educating a patient with lower back pain on proper lifting techniques.
(see full question) The nurse would document what behavior as evidence the education was
effective?
You selected: The patient placed the load close to the body.
Correct
Explanation: Instructions for the patient with low back pain should include that when lifting, the
patient should avoid overreaching. The patient should also keep the load close to
the body, bend the knees and tighten the abdominal muscles, use a wide base of
support, and use a back brace to protect the back. Bending at the hips increases
the strain on the back muscles when lifting. (less)

Reference: Hinkle, J.L., and Cheever, K.H. Brunner & Suddarth's Textbook of Medical-Surgical
Nursing, 13th ed., Philadelphia: Lippincott Williams & Wilkins, 2014, Chapter 42:
Management of Patients With Musculoskeletal Disorders, p. 1134.
Chapter 42: Management of Patients With Musculoskeletal Disorders - Page 1134

Question 17: After undergoing surgery the previous day for a total knee replacement, a client
(see full question) states that he doesn't feel ready to ambulate yet. What should the nurse do?
You selected: Discuss the complications that the client's may experience if he doesn't cooperate
with the care plan.
Correct
Explanation: The nurse should discuss the care plan and its rationale with the client. Calling the
physician to report the client's noncompliance won't alter the client's degree of
participation and shouldn't be used to force the client to comply. Doing nothing
isn't acceptable. Although the client does have the right to make choices, it's the
nurse's responsibility to provide education to help the client make informed
decisions. Although the nurse should ultimately document the client's refusal, she
should first discuss the care plan with the client. (less)

Reference: Hinkle, J.L., and Cheever, K.H. Brunner & Suddarth's Textbook of Medical-Surgical
Nursing, 13th ed. Philadelphia: Lippincott Williams & Wilkins, 2014, Chapter 41:
Musculoskeletal Care Modalities, p. 1128.
Chapter 41: Musculoskeletal Care Modalities - Page 1128

Question 18: The nurse is caring for a client with an external fixator that requires pin care twice
(see full question) a day. The nurse observes that there is a new purulent drainage around one of the
pins. What intervention should the nurse anticipate doing?
You selected: Obtaining a culture
Correct
Explanation: A culture should be obtained if purulent drainage is present. Drainage should be
gently removed, not scrubbed. Iodine-based products interfere with tissue healing
and are not recommended for cleaning pin sites. Ointment should not be applied
to the pin site unless specifically ordered. (less)

Reference: Hinkle, J.L., and Cheever, K.H. Brunner & Suddarth's Textbook of Medical-Surgical
Nursing, 13th ed. Philadelphia: Lippincott Williams & Wilkins, 2014, Chapter 43:
Management of Patients With Musculoskeletal Trauma, p. 1187.
Chapter 41: Musculoskeletal Care Modalities - Page 1187

Question 19: The nurse is preparing a client for a surgical procedure that will allow visualization
(see full question) of the extent of joint damage of the knee for a client with rheumatoid arthritis and
also obtain a sample of synovial fluid. What procedure will the nurse prepare the
client for?
You selected: Arthroscopy
Correct
Explanation: Arthroscopic examination may be carried out to visualize the extent of joint
damage as well as to obtain a sample of synovial fluid. An open reduction would
be used for the treatment of a fracture. Needle aspiration will not allow
visualization of the joint damage but will allow obtaining the sample of synovial
fluid. Arthroplasty is the restructure of the joint surface after diagnosis is made.
(less)

Reference: Hinkle, J.L., and Cheever, K.H. Brunner & Suddarth's Textbook of Medical-Surgical
Nursing, 13th ed. Philadelphia: Lippincott Williams & Wilkins, 2014, Chapter 40:
Assessment of Musculoskeletal Function, p. 1100.
Chapter 42: Management of Patients With Musculoskeletal Disorders - Page 1100

Question 20: Which nursing intervention is essential in caring for a client with compartment
(see full question) syndrome?
You selected: Removing all external sources of pressure, such as clothing and jewelry
Correct
Explanation: Nursing measures should include removing all clothing, jewelry, and external
forms of pressure (such as dressings or casts) to prevent constriction and
additional tissue compromise. The extremity should be maintained at heart level
(further elevation may increase circulatory compromise, whereas a dependent
position may increase edema). A compression wrap, which increases tissue
pressure, could further damage the affected extremity. There is no indication that
diagnostic studies would require I.V. access in the affected extremity. (less)

Reference: Hinkle, J.L., and Cheever, K.H. Brunner & Suddarth's Textbook of Medical-Surgical
Nursing, 13th ed. Philadelphia: Lippincott Williams & Wilkins, 2014, Chapter 43:
Management of Patients With Musculoskeletal Trauma, p. 1165.
Chapter 43: Management of Patients With Musculoskeletal Trauma - Page 1165

A patient with severe hypervolemia is prescribed a loop diuretic. The nurse knows that this drug can cause a
significant loss of sodium and has to be carefully monitored. Which of the following drugs is most likely the one
that was prescribed?
You selected: Hydrodiuril
Incorrect
Correct response: Lasix
Explanation: Lasix is the only loop diuretic choice. The other choices are thiazide diuretics that
block sodium reabsorption.

Reference: Hinkle, J.L., and Cheever, K.H. Brunner & Suddarth's Textbook of Medical-Surgical
Nursing, 13th ed. Philadelphia: Lippincott Williams & Wilkins, 2014, Chapter 13:
Fluid and Electrolytes: Balance and Disturbance, p. 249.
Chapter 13: Fluid and Electrolytes: Balance and Disturbance - Page 249

Question 2: A client comes to the emergency department with status asthmaticus. His
(see full question) respiratory rate is 48 breaths/minute, and he is wheezing. An arterial blood gas
analysis reveals a pH of 7.52, a partial pressure of arterial carbon dioxide (PaCO 2)
of 30 mm Hg, PaO2 of 70 mm Hg, and bicarbonate (HCO3??') of 26 mEq/L. What
disorder is indicated by these findings?
You selected: Respiratory alkalosis
Correct
Explanation: Respiratory alkalosis results from alveolar hyperventilation. It's marked by a
decrease in PaCO2 to less than 35 mm Hg and an increase in blood pH over
7.45. Metabolic acidosis is ... (more)

Reference: Hinkle, J.L., and Cheever, K.H. Brunner & Suddarth's Textbook of Medical-Surgical
Nursing, 13th ed. Philadelphia: Lippincott Williams & Wilkins, 2014, Chapter 13:
Fluid and Electrolytes: Balance and Disturbance, p. 270.
Chapter 13: Fluid and Electrolytes: Balance and Disturbance - Page 270

Question 3: A priority nursing intervention for a client with hypervolemia involves which of the
(see full question) following?
You selected: Monitoring respiratory status for signs and symptoms of pulmonary complications.
Correct
Explanation: Hypervolemia, or fluid volume excess (FVE), refers to an isotonic expansion of the
extracellular fluid. Nursing interventions for FVE include measuring intake and
output, monitorin ... (more)

Reference: Hinkle, J.L., and Cheever, K.H. Brunner & Suddarth's Textbook of Medical-Surgical
Nursing, 13th ed. Philadelphia: Lippincott Williams & Wilkins, 2014, Chapter 13:
Fluid and Electrolytes: Balance and Disturbance, p. 249.
Chapter 13: Fluid and Electrolytes: Balance and Disturbance - Page 249

Question 4: Air embolism is a potential complication of IV therapy. The nurse should be alert to
(see full question) which clinical manifestation associated with air embolism?
You selected: Chest pain
Correct
Explanation: Manifestations of air embolism include dyspnea and cyanosis; hypotension; weak,
rapid pulse; loss of consciousness; and chest, shoulder, and low back pain.
Jaundice is not associat ... (more)

Reference: Hinkle, J.L., and Cheever, K.H. Brunner & Suddarth's Textbook of Medical-Surgical
Nursing, 13th ed. Philadelphia: Lippincott Williams & Wilkins, 2014, Chapter 13:
Fluid and Electrolytes: Balance and Disturbance, p. 280.
Chapter 13: Fluid and Electrolytes: Balance and Disturbance - Page 280

Question 5: A client has the following arterial blood gas (ABG) values: pH, 7.12; partial
(see full question) pressure of arterial carbon dioxide (PaCO 2), 40 mm Hg; and bicarbonate (HCO3),
15 mEq/L. These ABG values suggest which disorder?
You selected: Respiratory acidosis
Incorrect
Correct response: Metabolic acidosis
Explanation: This client's pH value is below normal, indicating acidosis. The HCO 3 value also is
below normal, reflecting an overwhelming accumulation of acids or excessive loss
of base, which suggests metabolic acidosis. The PaCO2 value is normal, indicating
absence of respiratory compensation. These ABG values eliminate respiratory
alkalosis, respiratory acidosis, and metabolic alkalosis. (less)

Reference: Hinkle, J.L., and Cheever, K.H. Brunner & Suddarth's Textbook of Medical-Surgical
Nursing, 13th ed. Philadelphia: Lippincott Williams & Wilkins, 2014, Chapter 13:
Fluid and Electrolytes: Balance and Disturbance, p. 268.
Chapter 13: Fluid and Electrolytes: Balance and Disturbance - Page 268

Question 6: A nurse can estimate serum osmolality at the bedside by using a formula. A
(see full question) patient who has a serum sodium level of 140 mEq/L would have a serum
osmolality of:
You selected: 280 mOsm/kg.
Correct
Explanation: Serum osmolality can be estimated by doubling the serum sodium or using the
formula: Na 2 = glucose/18 + BUN/3. Therefore, the nurse could estimate a
serum osmolality of 280 mOsm/k ... (more)

Reference: Hinkle, J.L., and Cheever, K.H. Brunner & Suddarth's Textbook of Medical-Surgical
Nursing, 13th ed. Philadelphia: Lippincott Williams & Wilkins, 2014, Chapter 13:
Fluid and Electrolytes: Balance and Disturbance, p. 241.
Chapter 13: Fluid and Electrolytes: Balance and Disturbance - Page 241

Question 7: A client with a suspected overdose of an unknown drug is admitted to the


(see full question) emergency department. Arterial blood gas values indicate respiratory acidosis.
What should the nurse do first?
You selected: Prepare to assist with ventilation.
Correct
Explanation: Respiratory acidosis is associated with hypoventilation; in this client,
hypoventilation suggests intake of a drug that has suppressed the brain's
respiratory center. Therefore, th ... (more)

Reference: Hinkle, J.L., and Cheever, K.H. Brunner & Suddarth's Textbook of Medical-Surgical
Nursing, 13th ed. Philadelphia: Lippincott Williams & Wilkins, 2014, Chapter 13:
Fluid and Electrolytes: Balance and Disturbance, p. 270.
Chapter 13: Fluid and Electrolytes: Balance and Disturbance - Page 270

Question 8: The nurse is analyzing the arterial blood gas (AGB) results of a patient diagnosed
(see full question) with severe pneumonia. Which of the following ABG results indicates respiratory
acidosis?
You selected: pH: 7.20, PaCO2: 65 mm Hg, HCO3: 26 mEq/L
Correct
Explanation: Respiratory acidosis is a clinical disorder in which the pH is less than 7.35 and the
PaCO2 is greater than 42 mm Hg and a compensatory increase in the plasma
HCO3 occurs. ... (more)

Reference: Hinkle, J.L., and Cheever, K.H. Brunner & Suddarth's Textbook of Medical-Surgical
Nursing, 13th ed., Philadelphia: Lippincott Williams & Wilkins, 2014, Chapter 13:
Fluid and Electrolytes: Balance and Disturbance, p. 269.
Chapter 13: Fluid and Electrolytes: Balance and Disturbance - Page 269

Question 9: Translocation is a term used to describe the general movement of fluid and
(see full question) chemicals within body fluids. In every clients body, fluid and electrolyte balance is
maintained through the process of translocation. What specific process allows
water to pass through a membrane from a dilute to a more concentrated area?
You selected: Osmosis
Correct
Explanation: Osmosis is the movement of water through a semi permeable membraneone
that allows some but not all substances in a solution to pass through from a
diluted area to a more co ... (more)

Reference: Hinkle, J.L., and Cheever, K.H. Brunner & Suddarth's Textbook of Medical-Surgical
Nursing, 13th ed. Philadelphia: Lippincott Williams & Wilkins, 2014, Chapter 13:
Fluid and Electrolytes: Balance and Disturbance, p. 237.
Chapter 13: Fluid and Electrolytes: Balance and Disturbance - Page 237

Question 10: Which of the following is considered an isotonic solution?


(see full question)
You selected: 0.9% normal saline
Correct
Explanation: An isotonic solution is 0.9% normal saline (NaCl). Dextran in NS is a colloid
solution, 0.45% normal saline is a hypotonic solution, and 3% NaCl is a hypertonic
solution.

Reference: Hinkle, J.L., and Cheever, K.H. Brunner & Suddarth's Textbook of Medical-Surgical
Nursing, 13th ed. Philadelphia: Lippincott Williams & Wilkins, 2014, Chapter 13:
Fluid and Electrolytes: Balance and Disturbance, p. 273.
Chapter 13: Fluid and Electrolytes: Balance and Disturbance - Page 273

Answer Key

Question 1: A client in the emergency department reports that he has


(see full question) been vomiting excessively for the past 2 days. His arterial blood gas analysis
shows a pH of 7.50, partial pressure of arterial carbon dioxide (PaCO 2) of 43 mm
Hg, partial pressure of arterial oxygen (PaO 2) of 75 mm Hg, and bicarbonate
(HCO3) of 42 mEq/L. Based on these findings, the nurse documents that the client
is experiencing which type of acid-base imbalance?
You selected: Metabolic alkalosis
Correct
Explanation: A pH over 7.45 with a HCO3 level over 26 mEq/L indicates metabolic alkalosis.
Metabolic alkalosis is always secondary to an underlying cause and is marked by
decreased amounts of acid or increased amounts of base HCO 3. The client isn't
experiencing respiratory alkalosis because the PaCO2 is normal. The client isn't
experiencing respiratory or metabolic acidosis because the pH is greater than
7.35. (less)

Reference: Hinkle, J.L., and Cheever, K.H. Brunner & Suddarth's Textbook of Medical-Surgical
Nursing, 13th ed. Philadelphia: Lippincott Williams & Wilkins, 2014, Chapter 13:
Fluid and Electrolytes: Balance and Disturbance, p. 269.
Chapter 13: Fluid and Electrolytes: Balance and Disturbance - Page 269

Question 2: Which set of arterial blood gas (ABG) results requires further investigation?
(see full question)
You selected: pH 7.49, PaCO2 30 mm Hg, PaO2 89 mm Hg, and HCO3 18 mEq/L
Correct
Explanation: The ABG results pH 7.49, PaCO2 30 mm Hg, PaO2 89 mm Hg, and HCO3 18 mEq/L
indicate respiratory alkalosis. The pH level is increased, and the HCO 3 and
PaCO2 levels are decreased. Normal values are pH 7.35 to 7.45; PaCO 2 35 to 45
mm Hg; HCO3 22 to 26 mEq/L. (less)

Reference: Hinkle, J.L., and Cheever, K.H. Brunner & Suddarth's Textbook of Medical-Surgical
Nursing, 13th ed. Philadelphia: Lippincott Williams & Wilkins, 2014, Chapter 13:
Fluid and Electrolytes: Balance and Disturbance, p. 270.
Chapter 13: Fluid and Electrolytes: Balance and Disturbance - Page 270

Question 3: When evaluating arterial blood gases (ABGs), which value is consistent with
(see full question) metabolic alkalosis?
You selected: HCO 21 mEq/L
Incorrect
Correct response: pH 7.48
Explanation: Metabolic alkalosis is a clinical disturbance characterized by a high pH and high
plasma bicarbonate concentration. The HCO value is below normal. The PaCO
value and the oxygen saturation level are within a normal range. (less)

Reference: Hinkle, J.L., and Cheever, K.H. Brunner & Suddarth's Textbook of Medical-Surgical
Nursing, 13th ed. Philadelphia: Lippincott Williams & Wilkins, 2014, Chapter 13:
Fluid and Electrolytes: Balance and Disturbance, p. 269.
Chapter 13: Fluid and Electrolytes: Balance and Disturbance - Page 269

Question 4: The nurse is caring for a patient with diabetes type I who is having severe
(see full question) vomiting and diarrhea. What condition that exhibits blood values with a low pH
and a low plasma bicarbonate concentration should the nurse assess for?
You selected: Metabolic acidosis
Correct
Explanation: Metabolic acidosis is a common clinical disturbance characterized by a low pH
(increased H+ concentration) and a low plasma bicarbonate concentration.
Metabolic alkalosis is a clinical disturbance characterized by a high pH (decreased
H+ concentration) and a high plasma bicarbonate concentration. Respiratory
acidosis is a clinical disorder in which the pH is less than 7.35 and the PaCO2 is
greater than 42 mm Hg and a compensatory increase in the plasma HCO3 occurs.
Respiratory alkalosis is a clinical condition in which the arterial pH is greater than
7.45 and the PaCO2 is less than 38 mm Hg. (less)

Reference: Hinkle, J.L., and Cheever, K.H. Brunner & Suddarth's Textbook of Medical-Surgical
Nursing, 13th ed. Philadelphia: Lippincott Williams & Wilkins, 2014, Chapter 13:
Fluid and Electrolytes: Balance and Disturbance, p. 268.
Chapter 13: Fluid and Electrolytes: Balance and Disturbance - Page 268

Question 5: A client hospitalized for treatment of a pulmonary embolism develops respiratory


(see full question) alkalosis. Which clinical findings commonly accompany respiratory alkalosis?
You selected: Light-headedness or paresthesia
Correct
Explanation: The client with respiratory alkalosis may complain of light-headedness
or paresthesia (numbness and tingling in the arms and legs). Nausea, vomiting,
abdominal pain, and diarrhea may accompany respiratory
acidosis. Hallucinations and tinnitus rarely are associated with respiratory alkalosis
or any other acid-base imbalance. (less)

Reference: Hinkle, J.L., and Cheever, K.H. Brunner & Suddarth's Textbook of Medical-Surgical
Nursing, 13th ed. Philadelphia: Lippincott Williams & Wilkins, 2014, Chapter 13:
Fluid and Electrolytes: Balance and Disturbance, p. 270.
Chapter 13: Fluid and Electrolytes: Balance and Disturbance - Page 270

Question 6: A 73-year-old male client was admitted to your hospital unit after 2days of
(see full question) vomiting and diarrhea. His wife became alarmed when he demonstrated confusion
and elevated temperature and reported dry mouth.You suspect the client is
experiencing which of the following conditions?
You selected: Dehydration
Correct
Explanation: Dehydration results when the volume of body fluid is significantly reduced in both
extracellular and intracellular compartments. In dehydration, all fluid
compartments have decreased volumes; in hypovolemia, only blood volume is low.
The most common fluid imbalance in older adults is dehydration. Hypervolemia is
caused by fluid intake that exceeds fluid loss, such as from excessive oral intake or
rapid IV infusion of fluid. Early signs of hypervolemia are weight gain, elevated BP,
and increased breathing effort. Hypercalcemia occurs when the serum calcium
level is higher than normal. Some of its signs include tingling in the extremities
and the area around the mouth (circumoral paresthesia) and muscle and
abdominal cramps. Hyperkalemia is an excess of potassium in the blood.
Symptoms include diarrhea, nausea, muscle weakness, paresthesias, and cardiac
dysrhythmias. (less)

Reference: Hinkle, J.L., and Cheever, K.H. Brunner & Suddarth's Textbook of Medical-Surgical
Nursing, 13th ed. Philadelphia: Lippincott Williams & Wilkins, 2014, Chapter 13:
Fluid and Electrolytes: Balance and Disturbance, p. 244.
Chapter 13: Fluid and Electrolytes: Balance and Disturbance - Page 244

Question 7: A client with pancreatic cancer has the following blood chemistry profile: Glucose,
(see full question) fasting: 204 mg/dl; blood urea nitrogen (BUN): 12 mg/dl; Creatinine: 0.9 mg/dl;
Sodium: 136 mEq/L; Potassium: 2.2 mEq/L; Chloride: 99 mEq/L; CO 2: 33 mEq/L.
Which result should the nurse identify as critical and report immediately?
You selected: Potassium
Correct
Explanation: The nurse should identify potassium: 2.2 mEq/L as critical because a normal
potassium level is 3.8 to 5.5 mEq/L. Severe hypokalemia can cause cardiac and
respiratory arrest, possibly leading to death. Hypokalemia also depresses the
release of insulin and results in glucose intolerance. The glucose level is above
normal (normal is 75 to 110 mg/dl) and the chloride level is a bit low (normal is
100 to 110 mEq/L). Although these levels should be reported, neither is life-
threatening. The BUN (normal is 8 to 26 mg/dl) and creatinine (normal is 0.8 to 1.4
mg/dl) are within normal range. (less)

Reference: Hinkle, J.L., and Cheever, K.H. Brunner & Suddarth's Textbook of Medical-Surgical
Nursing, 13th ed. Philadelphia: Lippincott Williams & Wilkins, 2014, Chapter 13:
Fluid and Electrolytes: Balance and Disturbance, p. 255.
Chapter 13: Fluid and Electrolytes: Balance and Disturbance - Page 255

Question 8: A clients potassium level is elevated. The nurse is reviewing the ECG tracing.
(see full question) Identify the area on the tracing where the nurse would expect to see peaks.
You selected: Your selection and the correct area, marked by the white box
Incorrect
Explanation: Potassium influences cardiac muscle activity. Alterations in potassium levels
change myocardial irritability and rhythm. Hyperkalemia is very dangerous;
cardiac arrest can occur. Cardiac effects of elevated serum potassium are
significant when the level is above 8 mEq/L. Hyperkalemia causes skeletal muscle
weakness and even paralysis, related to a depolarization block in t muscle.
Therefore, ventricular conduction is slowed. The earliest change that can be seen
are peaked, narrow T waves on the ECG. (less)

Reference: Hinkle, J.L., and Cheever, K.H. Brunner & Suddarth's Textbook of Medical-Surgical
Nursing, 13th ed. Philadelphia: Lippincott Williams & Wilkins, 2014, Chapter 13:
Fluid and Electrolytes: Balance and Disturbance, p. 255.
Chapter 13: Fluid and Electrolytes: Balance and Disturbance - Page 255

Question 9: Which of the following is a correct route of administration for potassium?


(see full question)
You selected: Oral
Correct
Explanation: Potassium may be administered through the oral route. Potassium is never
administered by IV push or intramuscularly to avoid replacing potassium too
quickly. Potassium is not administered subcutaneously. (less)

Reference: Hinkle, J.L., and Cheever, K.H. Brunner & Suddarth's Textbook of Medical-Surgical
Nursing, 13th ed. Philadelphia: Lippincott Williams & Wilkins, 2014, Chapter 13:
Fluid and Electrolytes: Balance and Disturbance, p. 257.
Chapter 13: Fluid and Electrolytes: Balance and Disturbance - Page 257

Question 10: Which of the following are the insensible mechanisms of fluid loss?
(see full question)
You selected: Breathing
Correct
Explanation: Loss of fluid from sweat or diaphoresis is referred to as insensible loss because it is
unnoticeable and immeasurable. Losses from urination and bowel elimination are
measurable.

Reference: Hinkle, J.L., and Cheever, K.H. Brunner & Suddarth's Textbook of Medical-Surgical
Nursing, 13th ed. Philadelphia: Lippincott Williams & Wilkins, 2014, Chapter 13:
Fluid and Electrolytes: Balance and Disturbance, p. 242.
Chapter 13: Fluid and Electrolytes: Balance and Disturbance - Page 242

Answer Key

Question 1: A client has had an exacerbation of ulcerative colitis with cramping and diarrhea
(see full question) persisting longer than 1 week. The nurse should assess the client for which
complication?
You selected: hypokalemia
Correct
Explanation: Excessive diarrhea causes significant depletion of the bodys stores of sodium and
potassium as well as fluid. The client should be closely monitored for hypokalemia
and hyponatremia. Ulcerative colitis does not place the client at risk for heart
failure, deep vein thrombosis, or hypocalcemia. (less)

Question 2: A nurse is caring for a woman receiving a lumbar epidural anesthetic block to
(see full question) control labor pain. What should the nurse do to prevent hypotension?
You selected: Ensure adequate hydration before the anesthetic is administered.
Correct
Explanation: Administration of an epidural anesthetic may lead to hypotension because
blocking the sympathetic fibers in the epidural space reduces peripheral
resistance. Administering fluids I.V. before the epidural anesthetic is given may
prevent hypotension. Ephedrine may be administered after an epidural block if a
woman becomes hypotensive and shows evidence of cardiovascular
decompensation. However, ephedrine isn't administered to prevent hypotension.
Oxygen is administered to a woman who becomes hypotensive, but it won't
prevent hypotension. Placing a pregnant woman in the supine position can
contribute to hypotension because of uterine pressure on the great vessels. (less)

Question 3: A client who has been taking furosemide has a serum potassium level of 3.2
(see full question) mEq/L. Which assessment findings by the nurse would confirm an electrolyte
imbalance?
You selected: Muscle weakness and a weak, irregular pulse
Correct
Explanation: The serum potassium level of 3.2 mEq/L is an indication of hypokalemia. Only 2%
of the potassium is found in the extracellular fluid, and it is primarily responsible
for neuromuscular activity. Muscle weakness and heart irregularities would be
evident with hypokalemia. Potassium deficit is caused by diarrhea. Tetany and
tremors are associated with hypokalemia. Headaches and poor tissue turgor are
associated with hyponatremia. (less)

Question 4: A client is receiving chemotherapy to treat breast cancer. Which assessment


(see full question) finding indicates a chemotherapy-inducedcomplication?
You selected: Serum potassium level of [2.6 mEq/L (2.6 mmol/L)}
Correct
Explanation: Chemotherapy commonly causes nausea and vomiting, which may lead to fluid
and electrolyte imbalances. Signs of fluid loss include a serum potassium level
below 3.5 mEq/L, decreased urine output (less than 40 ml/hour), and abnormally
low blood pressure. Urine output of 400 ml in 8 hours, serum sodium level of 142
mEq/L, and a blood pressure of 120/64 to 130/72 mm Hg aren't abnormal findings.
(less)

Question 5: A client has partial-thickness burns on both lower extremities and portions of the
(see full question) trunk. Which I.V. fluid does the nurse plan to administer first?
You selected: Dextrose 5% in water (D5W)
Incorrect
Correct response: Lactated Ringer's solution
Explanation: Lactated Ringer's solution replaces lost sodium and corrects metabolic acidosis,
both of which commonly occur following a burn. Albumin is used as adjunct
therapy, not as primary fluid replacement. D5W isn't given to burn clients during
the first 24 hours because it can cause pseudodiabetes. The client is hyperkalemic
as a result of the potassium shift from the intracellular space to the plasma, so
giving potassium would be detrimental. (less)

Question 6: The nurse determines that interventions for decreasing fluid retention have been
(see full question) effective when the nurse makes which assessment in child with nephrotic
syndrome?
You selected: decreased heart rate
Incorrect
Correct response: decreased abdominal girth
Explanation: Fluid accumulates in the abdomen and interstitial spaces owing to hydrostatic
pressure changes. Increased abdominal fluid is evidenced by an increase in
abdominal girth. Therefore, decreased abdominal girth is a sign of reduced fluid in
the third spaces and tissues. When fluid accumulates in the abdomen and
interstitial spaces, the child does not feel hungry and does not eat well. Although
increased caloric intake may indicate decreased intestinal edema, it is not the best
and most accurate indicator of fluid retention. Increased respiratory rate may be
an indication of increasing fluid in the abdomen (ascites) causing pressure on the
diaphragm. Heart rate usually stays in the normal range even with excessive fluid
volume. (less)

Question 7: A 31-year-old multigravid client at 39 weeks' gestation admitted to the hospital in


(see full question) active labor is receiving intravenous lactated Ringer's solution and a continuous
epidural anesthetic. During the first hour after administration of the anesthetic,
the nurse should monitor the client for:
You selected: hypotension.
Correct
Explanation: When a client receives an epidural anesthetic, sympathetic nerves are blocked
along with the pain nerves, possibly resulting in vasodilation and hypotension.
Other adverse effects include bladder distention, prolonged second stage of labor,
nausea and vomiting, pruritus, and delayed respiratory depression for up to 24
hours after administration. Diaphoresis and tremors are not usually associated
with the administration of epidural anesthesia. Headache, a common adverse
effect of many drugs, also is not associated with administration of epidural
anesthesia. (less)

Question 8: For a client with anorexia nervosa, which goal takes the highest priority?
(see full question)
You selected: The client will establish adequate daily nutritional intake.
Correct
Explanation: According to Maslow's hierarchy of needs, all humans must first meet basic
physiologic needs. Because a client with anorexia nervosa eats little or nothing,
the nurse must first plan to help the client meet this basic, immediate
physiological need. The nurse may give lesser priority to goals that address long-
term plans, self-perception, and potential complications. (less)

Question 9: Which finding would alert the nurse to suspect that a child with severe
(see full question) gastroenteritis who has been receiving intravenous therapy for the past several
hours may be developing circulatory overload?
You selected: auscultation of moist crackles
Correct
Explanation: An early sign of circulatory overload is moist rales or crackles heard when
auscultating over the chest wall. Elevated blood pressure, engorged neck veins, a
wide variation between fluid intake and output (with a higher intake than output),
shortness of breath, increased respiratory rate, dyspnea, and cyanosis occur later.
(less)

Question 10: A client has been experiencing abdominal cramps, diarrhea, and concentrated
(see full question) urine for the past 2 days. Which of the following would be included in a focused
assessment?
You selected: Signs of dehydration, including loss of weight; poor tissue turgor; and dry, cracked
mucous membranes
Correct
Explanation: When a client has abdominal cramps and diarrhea, there is a loss of extra fluids
from the body. Through a focused assessment, the nurse should assess for a fluid
volume deficit. This would be indicated by signs of dehydration and weight loss. A
focused assessment would usually indicate increased bowel sounds associated
with the cramping. Kidney suppression would not be associated with diarrhea
lasting 2 days; it might present with severe dehydration and hypovolemic shock.
There is a loss of bicarbonate through the diarrhea, which would result in
metabolic acidosis, not alkalosis. (less)

(see full question)A 160-pound patient, diagnosed with hypovolemia, is weighed every day. The health care provider
asked to be notified if the patient loses 1,000 mL of fluid in 24 hours. Choose the weight that would be consistent
with this amount of fluid loss.You selected:158 lbsCorrectExplanation:
A loss of 0.5 kg or 1 lb represents a fluid loss of about 500 mL. Therefore, a loss of 1,000 mL would be equivalent to
the loss of 2 lbs (160 2 = 158 lbs).

Reference:
Hinkle, J.L., and Cheever, K.H. Brunner & Suddarth's Textbook of Medical-Surgical Nursing, 13th ed. Philadelphia:
Lippincott Williams & Wilkins, 2014, Chapter 13: Fluid and Electrolytes: Balance and Disturbance, p. 247.
Chapter 13: Fluid and Electrolytes: Balance and Disturbance - Page 247

Question 2:
(see full question)A patient is diagnosed with hypocalcemia. The nurse advises the patient and his family to
immediately report the most characteristic manifestation. What is the most characteristic manifestation?You
selected:Tingling or twitching sensation in the fingersCorrectExplanation:
All the choices are signs and symptoms of hypocalcemia, but tetany is the most characteristic manifestation that
occurs when the calcium level is less than 4.4 mg/dL.

Reference:
Hinkle, J.L., and Cheever, K.H. Brunner & Suddarth's Textbook of Medical-Surgical Nursing, 13th ed. Philadelphia:
Lippincott Williams & Wilkins, 2014, Chapter 13: Fluid and Electrolytes: Balance and Disturbance, p. 259.
Chapter 13: Fluid and Electrolytes: Balance and Disturbance - Page 259

Question 3:
(see full question)A client with a suspected overdose of an unknown drug is admitted to the emergency
department. Arterial blood gas values indicate respiratory acidosis. What should the nurse do first?You
selected:Prepare to assist with ventilation.CorrectExplanation:
Respiratory acidosis is associated with hypoventilation; in this client, hypoventilation suggests intake of a drug that
has suppressed the brain's respiratory center. Therefore, the nurse should assume the client has respiratory
depression and should prepare to assist with ventilation. After the client's respiratory function has been stabilized,
the nurse can safely monitor the heart rhythm, prepare for gastric lavage, and obtain a urine specimen for drug
screening. (less)

Reference:
Hinkle, J.L., and Cheever, K.H. Brunner & Suddarth's Textbook of Medical-Surgical Nursing, 13th ed. Philadelphia:
Lippincott Williams & Wilkins, 2014, Chapter 13: Fluid and Electrolytes: Balance and Disturbance, p. 270.
Chapter 13: Fluid and Electrolytes: Balance and Disturbance - Page 270

Question 4:
(see full question)A patient is ordered to receive hypotonic IV solution to provide free water replacement. Which of
the following solutions will the nurse anticipate administering?You selected:0.45% NaClCorrectExplanation:
Half-strength saline (0.45%) is hypotonic. Hypotonic solutions are used to replace cellular fluid because it is
hypotonic compared with plasma. Another is to provide free water to excrete body wastes. At times, hypotonic
sodium solutions are used to treat hypernatremia and other hyperosmolar conditions. Lactated Ringers solution
and normal saline (0.9% NaCl) are isotonic. A solution that is 5% NaCl is hypertonic. (less)

Reference:
Hinkle, J.L., and Cheever, K.H. Brunner & Suddarth's Textbook of Medical-Surgical Nursing, 13 th ed., Philadelphia:
Lippincott Williams & Wilkins, 2014, Chapter 13: Fluid and Electrolytes: Balance and Disturbance, p. 273.
Chapter 13: Fluid and Electrolytes: Balance and Disturbance - Page 273
Question 5:
(see full question)Translocation is a term used to describe the general movement of fluid and chemicals within body
fluids. In every clients body, fluid and electrolyte balance is maintained through the process of translocation. What
specific process allows water to pass through a membrane from a dilute to a more concentrated area?You
selected:Active transportIncorrectCorrect response:OsmosisExplanation:
Osmosis is the movement of water through a semi permeable membraneone that allows some but not all
substances in a solution to pass through from a diluted area to a more concentrated area. Filtration promotes the
movement of fluid and some dissolved substances through a semi permeable membrane according to pressure
differences. This is the process of converting water into a vapor. Active transport requires the energy source ATP to
drive dissolved chemicals from an area of low concentration to an area of higher concentrationthe opposite of
passive diffusion. (less)

Reference:
Hinkle, J.L., and Cheever, K.H. Brunner & Suddarth's Textbook of Medical-Surgical Nursing, 13th ed. Philadelphia:
Lippincott Williams & Wilkins, 2014, Chapter 13: Fluid and Electrolytes: Balance and Disturbance, p. 237.
Chapter 13: Fluid and Electrolytes: Balance and Disturbance - Page 237

Question 6:
(see full question)A client presents with anorexia, nausea and vomiting, deep bone pain, and constipation. The
following are the client's laboratory values.

Na + 130 mEq/L
K + 4.6 mEq/L
Cl - 94 mEq/L
Mg ++ 2.8 mg/dL
Ca ++ 13 mg/dL

Which of the following alterations is consistent with the client's findings?


You selected:HypercalcemiaCorrectExplanation:
More than 99% of the bodys calcium is found in the skeletal system. Hypercalcemia (greater than 10.2 mg/dL) can
be a dangerous imbalance. The client presents with anorexia, nausea and vomiting, constipation, abdominal pain,
bone pain, and confusion. (less)

Reference:
Hinkle, J.L., and Cheever, K.H. Brunner & Suddarth's Textbook of Medical-Surgical Nursing, 13th ed. Philadelphia:
Lippincott Williams & Wilkins, 2014, Chapter 13: Fluid and Electrolytes: Balance and Disturbance, pp. 260-261.
Chapter 13: Fluid and Electrolytes: Balance and Disturbance - Page 260

Question 7:
(see full question)A 42-year-old client has chronic hypo natremia, which requires weekly blood labs to keep him
from lapsing into convulsions or a coma. What is the level of serum sodium below which convulsions or coma can
occur?You selected:135 mEq/LCorrectExplanation:
Normal serum concentration level ranges from 135 to145mEq/L. When the level dips below 135 mEq/L, there is
hypo natremia. Manifestations include mental confusion, muscular weakness, anorexia, restlessness, elevated body
temperature, tachycardia, nausea, vomiting, and personality changes. Convulsions or coma can occur, if the deficit
is severe. This level would indicate hyper natremia, which is serum sodium level above 145 mEq/L. Normal serum
concentration level ranges from 135 to 145 mEq/L. (less)

Reference:
Hinkle, J.L., and Cheever, K.H. Brunner & Suddarth's Textbook of Medical-Surgical Nursing, 13th ed. Philadelphia:
Lippincott Williams & Wilkins, 2014, Chapter 13: Fluid and Electrolytes: Balance and Disturbance, p. 251.
Chapter 13: Fluid and Electrolytes: Balance and Disturbance - Page 251

Question 8:
(see full question)A nurse is caring for a client with metastatic breast cancer who is extremely lethargic and very
slow to respond to stimuli. The laboratory report indicates a serum calcium level of 12.0 mg/dl, a serum potassium
level of 3.9 mEq/L, a serum chloride level of 101 mEq/L, and a serum sodium level of 140 mEq/L. Based on this
information, the nurse determines that the client's symptoms are most likely associated with which electrolyte
imbalance?You selected:HypercalcemiaCorrectExplanation:
The normal reference range for serum calcium is 9 to 11 mg/dl. A serum calcium level of 12 mg/dl clearly indicates
hypercalcemia. The client's other laboratory findings are within their normal ranges, so the client doesn't have
hypernatremia, hypochloremia, or hypokalemia. (less)

Reference:
Hinkle, J.L., and Cheever, K.H. Brunner & Suddarth's Textbook of Medical-Surgical Nursing, 13th ed. Philadelphia:
Lippincott Williams & Wilkins, 2014, Chapter 13: Fluid and Electrolytes: Balance and Disturbance, p. 260.
Chapter 13: Fluid and Electrolytes: Balance and Disturbance - Page 260

Question 9:
(see full question)When evaluating arterial blood gases (ABGs), which value is consistent with metabolic alkalosis?
You selected:pH 7.48CorrectExplanation:
Metabolic alkalosis is a clinical disturbance characterized by a high pH and high plasma bicarbonate concentration.
The HCO value is below normal. The PaCO value and the oxygen saturation level are within a normal range. (less)

Reference:
Hinkle, J.L., and Cheever, K.H. Brunner & Suddarth's Textbook of Medical-Surgical Nursing, 13th ed. Philadelphia:
Lippincott Williams & Wilkins, 2014, Chapter 13: Fluid and Electrolytes: Balance and Disturbance, p. 269.
Chapter 13: Fluid and Electrolytes: Balance and Disturbance - Page 269

Question 10:
(see full question)Treatment of FVE involves dietary restriction of sodium. Which of the following food choices would
be part of a low-sodium diet, mild restriction (2 to 3 g/day)?You selected:Three ounces of light or dark meat chicken,
1 cup of spaghetti and a garden saladCorrectExplanation:
Ham (1,400 mg Na for 3 oz) and bacon (155 mg Na/slice) are high in sodium as is tomato juice (660 mg Na/ cup)
and low fat cottage cheese (918 mg Na/cup). Packaged meals are high in sodium. (less)

Reference:
Hinkle, J.L., and Cheever, K.H. Brunner & Suddarth's Textbook of Medical-Surgical Nursing, 13th ed. Philadelphia:
Lippincott Williams & Wilkins, 2014, Chapter 13: Fluid and Electrolytes: Balance and Disturbance, p. 250.
Chapter 13: Fluid and Electrolytes: Balance and Disturbance - Page 250

Answer Key

Question 1: The nurse is analyzing the arterial blood gas (AGB) results of a patient diagnosed
(see full question) with severe pneumonia. Which of the following ABG results indicates respiratory
acidosis?
You selected: pH: 7.20, PaCO2: 65 mm Hg, HCO3: 26 mEq/L
Correct
Explanation: Respiratory acidosis is a clinical disorder in which the pH is less than 7.35 and the
PaCO2 is greater than 42 mm Hg and a compensatory increase in the plasma
HCO3 occurs. It may be either acute or chronic. The ABG of pH: 7.32, PaCO 2: 40
mm Hg, HCO3: 18 mEq/L indicates metabolic acidosis. The ABGs of pH: 7.50,
PaCO2: 30 mm Hg, and HCO3: 24 mEq/L indicate respiratory alkalosis. The ABGs of
pH 7.42, PaCO2: 45 mm Hg, and HCO3: 22 mEq/L indicate a normal result/no
imbalance. (less)

Reference: Hinkle, J.L., and Cheever, K.H. Brunner & Suddarth's Textbook of Medical-Surgical
Nursing, 13th ed., Philadelphia: Lippincott Williams & Wilkins, 2014, Chapter 13:
Fluid and Electrolytes: Balance and Disturbance, p. 269.
Chapter 13: Fluid and Electrolytes: Balance and Disturbance - Page 269

Question 2: The nurse is caring for four clients on a medical unit. The nurse is most correct to
(see full question) review which clients laboratory reports first for an electrolyte imbalance?
You selected: A 65-year-old with a myocardial infarction
Incorrect
Correct response: A 52-year-old with diarrhea
Explanation: Electrolytes are in both intracellular and extracellular water. Electrolyte deficiency
occurs from an inadequate intake of food, conditions that deplete water such as
nausea and vomiting, or disease processes that cause an excess of electrolyte
amounts. The 52-year-old with diarrhea would be the client most likely to have an
electrolyte imbalance. The orthopedic patients will not likely have an electrolyte
imbalance. Myocardial infarction patients will occasionally have electrolyte
imbalance, but this is the exception rather than the rule. (less)

Reference: Hinkle, J.L., and Cheever, K.H. Brunner & Suddarth's Textbook of Medical-Surgical
Nursing, 13th ed. Philadelphia: Lippincott Williams & Wilkins, 2014, Chapter 13:
Fluid and Electrolytes: Balance and Disturbance, p. 245.
Chapter 13: Fluid and Electrolytes: Balance and Disturbance - Page 245

Question 3: An elderly client takes 40 mg of Lasix twice a day. Which electrolyte imbalance is
(see full question) the most serious adverse effect of diuretic use?
You selected: Hyperkalemia
Incorrect
Correct response: Hypokalemia
Explanation: Hypokalemia (potassium level below 3.5 mEq/L) usually indicates a defict in total
potassium stores. Potassium-losing diuretics, such as loop diuretics, can induce
hypokalemia.

Reference: Hinkle, J.L., and Cheever, K.H. Brunner & Suddarth's Textbook of Medical-Surgical
Nursing, 13th ed. Philadelphia: Lippincott Williams & Wilkins, 2014, Chapter 13:
Fluid and Electrolytes: Balance and Disturbance, p. 255.
Chapter 13: Fluid and Electrolytes: Balance and Disturbance - Page 255

Question 4: Which intervention is most appropriate for a client with an arterial blood gas (ABG)
(see full question) of pH 7.5, a partial pressure of arterial carbon dioxide (PaCO 2) of 26 mm Hg,
oxygen (O2) saturation of 96%, bicarbonate (HCO3-) of 24 mEq/L, and a PaO2 of 94
mm Hg?
You selected: Administer ordered supplemental oxygen.
Incorrect
Correct response: Instruct the client to breathe into a paper bag.
Explanation: The ABG results reveal respiratory alkalosis. The best intervention to raise the
PaCO2 level would be to have the client breathe into a paper bag. Administering a
decongestant, offering fluids frequently, and administering supplemental oxygen
wouldn't raise the lowered PaCO2 level. (less)

Reference: Hinkle, J.L., and Cheever, K.H. Brunner & Suddarth's Textbook of Medical-Surgical
Nursing, 13th ed. Philadelphia: Lippincott Williams & Wilkins, 2014, Chapter 13:
Fluid and Electrolytes: Balance and Disturbance, p. 271.
Chapter 13: Fluid and Electrolytes: Balance and Disturbance - Page 271

Question 5: A client was admitted to your unit with a diagnosis of hypovolemia. When it is time
(see full question) to complete discharge teaching, which of the following will the nurse teach the
client and his family? Select all that apply.
You selected: Respond to thirst
Drink water as an inexpensive way to meet fluid needs.
Drink at least eight glasses of fluid each day.
Correct
Explanation: In addition, the nurse teaches clients who have a potential for hypovolemia and
their families to respond to thirst because it is an early indication of reduced fluid
volume; consume at least 8 to 10 (8 ounce) glasses of fluid each day and more
during hot, humid weather; drink water as an inexpensive means to meet fluid
requirements; and avoid beverages with alcohol and caffeine because they
increase urination and contribute to fluid deficits. (less)

Reference: Hinkle, J.L., and Cheever, K.H. Brunner & Suddarth's Textbook of Medical-Surgical
Nursing, 13th ed. Philadelphia: Lippincott Williams & Wilkins, 2014, Chapter 13:
Fluid and Electrolytes: Balance and Disturbance, pp. 245, 247, 249.
Chapter 13: Fluid and Electrolytes: Balance and Disturbance - Page 245

Question 6: A patient is being treated in the ICU 24 hours after having a radical neck
(see full question) dissection completed. The patients serum calcium level is 7.6 mg/dL. Which of the
following physical examination findings is consistent with this electrolyte
imbalance?
You selected: Muscle weakness
Incorrect
Correct response: Presence of Trousseaus sign
Explanation: A patient status post radical neck resection is prone to developing hypocalcemia.
Hypocalcemia is defined as a serum values lower than 8.6 mg/dL [2.15 mmol/L].
Signs and symptoms of hypocalcemia include: Chvosteks sign, which consists of
muscle twitching enervated by the facial nerve when the region that is about 2 cm
anterior to the earlobe, just below the zygomatic arch, is tapped, and a positive
Trousseaus sign can be elicited by inflating a blood pressure cuff on the upper arm
to about 20 mm Hg above systolic pressure; within 2 to 5 minutes, carpal spasm
(an adducted thumb, flexed wrist and metacarpophalangeal joints, and extended
interphalangeal joints with fingers together) will occur as ischemia of the ulnar
nerve develops. Slurred speech and muscle weakness are signs of hypercalcemia.
(less)

Reference: Hinkle, J.L., and Cheever, K.H. Brunner & Suddarth's Textbook of Medical-Surgical
Nursing, 13th ed., Philadelphia: Lippincott Williams & Wilkins, 2014, Chapter 13:
Fluid and Electrolytes: Balance and Disturbance, p. 259.
Chapter 13: Fluid and Electrolytes: Balance and Disturbance - Page 259

Question 7: A nurse is conducting an initial assessment on a client with possible tuberculosis.


(see full question) Which assessment finding indicates arisk factor for tuberculosis?
You selected: The client had a liver transplant 2 years ago.
Correct
Explanation: A history of immunocompromised status, such as that which occurs with liver
transplantation, places the client at a higher risk for contracting tuberculosis.
Other risk factors include inadequate health care, traveling to countries with high
rates of tuberculosis (such as southeastern Asia, Africa, and Latin America), being
a health care worker who performs procedures in which exposure to respiratory
secretions is likely, and being institutionalized. (less)

Reference: Smeltzer, S.C., and Bare, B. Brunner & Suddarth's Textbook of Medical-Surgical
Nursing, 12th ed. Philadelphia: Lippincott Williams & Wilkins, 2010, Chapter 14:
Fluid and Electrolytes: Balance and Disturbance, p. 567.
Chapter 13: Fluid and Electrolytes: Balance and Disturbance - Page 567

Question 8: Early signs of hypervolemia include


(see full question)
You selected: increased breathing effort and weight gain
Correct
Explanation: Early signs of hypervolemia are weight gain, elevated blood pressure, and
increased breathing effort. Eventually, fluid congestion in the lungs leads to moist
breath sounds. An earliest symptom of hypovolemia is thirst. (less)

Reference: Hinkle, J.L., and Cheever, K.H. Brunner & Suddarth's Textbook of Medical-Surgical
Nursing, 13th ed. Philadelphia: Lippincott Williams & Wilkins, 2014, Chapter 13:
Fluid and Electrolytes: Balance and Disturbance, p. 249.
Chapter 13: Fluid and Electrolytes: Balance and Disturbance - Page 249

Question 9: The physician has prescribed a hypotonic IV solution for a patient. Which IV
(see full question) solution should the nurse administer?
You selected: 0.45% sodium chloride
Correct
Explanation: Half-strength saline (0.45% sodium chloride) solution is frequently used as an IV
hypotonic solution.

Reference: Hinkle, J.L., and Cheever, K.H. Brunner & Suddarth's Textbook of Medical-Surgical
Nursing, 13th ed. Philadelphia: Lippincott Williams & Wilkins, 2014, Chapter 13:
Fluid and Electrolytes: Balance and Disturbance, p. 273.
Chapter 13: Fluid and Electrolytes: Balance and Disturbance - Page 273

Question 10: A patients serum sodium concentration is within the normal range. What should
(see full question) the nurse estimate the serum osmolality to be?
You selected: <136 mOsm/kg
Incorrect
Correct response: 275300 mOsm/kg
Explanation: In healthy adults, normal serum osmolality is 270 to 300 mOsm/kg (Crawford &
Harris, 2011c).
Reference: Hinkle, J.L., and Cheever, K.H. Brunner & Suddarth's Textbook of Medical-Surgical
Nursing, 13th ed. Philadelphia: Lippincott Williams & Wilkins, 2014, Chapter 13:
Fluid and Electrolytes: Balance and Disturbance, p. 241.
Chapter 13: Fluid and Electrolytes: Balance and Disturbance - Page 241
Answer Key

Question 1: Below which serum sodium level may convulsions or coma can occur?
(see full question)
You selected: 135 mEq/L
Correct
Explanation: Normal serum concentration level ranges from 135 to 145 mEq/L. When the level
dips below 135 mEq/L, there is hyponatremia. Manifestations of hyponatremia
include mental confusion, muscular weakness, anorexia, restlessness, elevated
body temperature, tachycardia, nausea, vomiting, and personality changes.
Convulsions or coma can occur if the deficit is severe. Values of 140, 142, and 145
mEq/L are within the normal range. (less)

Reference: Hinkle, J.L., and Cheever, K.H. Brunner & Suddarth's Textbook of Medical-Surgical
Nursing, 13th ed. Philadelphia: Lippincott Williams & Wilkins, 2014, Chapter 13:
Fluid and Electrolytes: Balance and Disturbance, p. 253.
Chapter 13: Fluid and Electrolytes: Balance and Disturbance - Page 253

Question 2: With which condition should the nurse expect that a decrease in serum osmolality
(see full question) will occur?
You selected: Diabetes insipidus
Incorrect
Correct response: Kidney failure
Explanation: Failure of the kidneys results in multiple fluid and electrolyte abnormalities.
Reference: Hinkle, J.L., and Cheever, K.H. Brunner & Suddarth's Textbook of Medical-Surgical
Nursing, 13th ed. Philadelphia: Lippincott Williams & Wilkins, 2014, Chapter 13:
Fluid and Electrolytes: Balance and Disturbance, p. 242.
Chapter 13: Fluid and Electrolytes: Balance and Disturbance - Page 242

Question 3: A client comes to the emergency department with status asthmaticus. His
(see full question) respiratory rate is 48 breaths/minute, and he is wheezing. An arterial blood gas
analysis reveals a pH of 7.52, a partial pressure of arterial carbon dioxide (PaCO 2)
of 30 mm Hg, PaO2 of 70 mm Hg, and bicarbonate (HCO3??') of 26 mEq/L. What
disorder is indicated by these findings?
You selected: Respiratory alkalosis
Correct
Explanation: Respiratory alkalosis results from alveolar hyperventilation. It's marked by a
decrease in PaCO2 to less than 35 mm Hg and an increase in blood pH over
7.45. Metabolic acidosis is marked by a decrease in HCO3? to less than 22 mEq/L,
and a decrease in blood pH to less than 7.35. In respiratory acidosis, the pH is less
than 7.35 and the PaCO2 is greater than 45 mm Hg. In metabolic alkalosis, the
HCO3? is greater than 26 mEq/L and the pH is greater than 7.45. (less)

Reference: Hinkle, J.L., and Cheever, K.H. Brunner & Suddarth's Textbook of Medical-Surgical
Nursing, 13th ed. Philadelphia: Lippincott Williams & Wilkins, 2014, Chapter 13:
Fluid and Electrolytes: Balance and Disturbance, p. 270.
Chapter 13: Fluid and Electrolytes: Balance and Disturbance - Page 270

Question 4: A client with pancreatic cancer has the following blood chemistry profile: Glucose,
(see full question) fasting: 204 mg/dl; blood urea nitrogen (BUN): 12 mg/dl; Creatinine: 0.9 mg/dl;
Sodium: 136 mEq/L; Potassium: 2.2 mEq/L; Chloride: 99 mEq/L; CO 2: 33 mEq/L.
Which result should the nurse identify as critical and report immediately?
You selected: Potassium
Correct
Explanation: The nurse should identify potassium: 2.2 mEq/L as critical because a normal
potassium level is 3.8 to 5.5 mEq/L. Severe hypokalemia can cause cardiac and
respiratory arrest, possibly leading to death. Hypokalemia also depresses the
release of insulin and results in glucose intolerance. The glucose level is above
normal (normal is 75 to 110 mg/dl) and the chloride level is a bit low (normal is
100 to 110 mEq/L). Although these levels should be reported, neither is life-
threatening. The BUN (normal is 8 to 26 mg/dl) and creatinine (normal is 0.8 to 1.4
mg/dl) are within normal range. (less)

Reference: Hinkle, J.L., and Cheever, K.H. Brunner & Suddarth's Textbook of Medical-Surgical
Nursing, 13th ed. Philadelphia: Lippincott Williams & Wilkins, 2014, Chapter 13:
Fluid and Electrolytes: Balance and Disturbance, p. 255.
Chapter 13: Fluid and Electrolytes: Balance and Disturbance - Page 255
Question 5: A client admitted with acute anxiety has the following arterial blood gas (ABG)
(see full question) values: pH, 7.55; partial pressure of arterial oxygen (PaO 2), 90 mm Hg; partial
pressure of arterial carbon dioxide (PaCO 2), 27 mm Hg; and bicarbonate (HCO3),
24 mEq/L. Based on these values, the nurse suspects:
You selected: respiratory alkalosis.
Correct
Explanation: This client's above-normal pH value indicates alkalosis. The below-normal
PaCO2 value indicates acid loss via hyperventilation; this type of acid loss occurs
only in respiratory alkalosis. These ABG values wouldn't occur in metabolic
acidosis, respiratory acidosis, or metabolic alkalosis. (less)

Reference: Hinkle, J.L., and Cheever, K.H. Brunner & Suddarth's Textbook of Medical-Surgical
Nursing, 13th ed. Philadelphia: Lippincott Williams & Wilkins, 2014, Chapter 13:
Fluid and Electrolytes: Balance and Disturbance, p. 270.
Chapter 13: Fluid and Electrolytes: Balance and Disturbance - Page 270

Question 6: A client in the emergency department reports that he has


(see full question) been vomiting excessively for the past 2 days. His arterial blood gas analysis
shows a pH of 7.50, partial pressure of arterial carbon dioxide (PaCO 2) of 43 mm
Hg, partial pressure of arterial oxygen (PaO 2) of 75 mm Hg, and bicarbonate
(HCO3) of 42 mEq/L. Based on these findings, the nurse documents that the client
is experiencing which type of acid-base imbalance?
You selected: Metabolic alkalosis
Correct
Explanation: A pH over 7.45 with a HCO3 level over 26 mEq/L indicates metabolic alkalosis.
Metabolic alkalosis is always secondary to an underlying cause and is marked by
decreased amounts of acid or increased amounts of base HCO 3. The client isn't
experiencing respiratory alkalosis because the PaCO2 is normal. The client isn't
experiencing respiratory or metabolic acidosis because the pH is greater than
7.35. (less)

Reference: Hinkle, J.L., and Cheever, K.H. Brunner & Suddarth's Textbook of Medical-Surgical
Nursing, 13th ed. Philadelphia: Lippincott Williams & Wilkins, 2014, Chapter 13:
Fluid and Electrolytes: Balance and Disturbance, p. 269.
Chapter 13: Fluid and Electrolytes: Balance and Disturbance - Page 269

Question 7: Which of the following electrolytes is a major cation in body fluid?


(see full question)
You selected: Potassium
Correct
Explanation: Potassium is a major cation that affects cardiac muscle functioning. Chloride is an
anion. Bicarbonate is an anion. Phosphate is an anion.

Reference: Hinkle, J.L., and Cheever, K.H. Brunner & Suddarth's Textbook of Medical-Surgical
Nursing, 13th ed. Philadelphia: Lippincott Williams & Wilkins, 2014, Chapter 13:
Fluid and Electrolytes: Balance and Disturbance, p. 238

Question 8: A nurse is reviewing a report of a client's routine urinalysis. Which value


(see full question) requires further investigation?
You selected: Urine pH of 3.0
Correct
Explanation: Normal urine pH is 4.5 to 8; therefore, a urine pH of 3.0 is abnormal and requires
further investigation. Urine specific gravity normally ranges from 1.010 to 1.025,
making this client's value normal. Normally, urine contains no protein, glucose,
ketones, bilirubin, bacteria, casts, or crystals. Red blood cells should measure 0 to
3 per high-power field; white blood cells, 0 to 4 per high-power field. Urine should
be clear, with color ranging from pale yellow to deep amber. (less)

Reference: Hinkle, J.L., and Cheever, K.H. Brunner & Suddarth's Textbook of Medical-Surgical
Nursing, 13th ed. Philadelphia: Lippincott Williams & Wilkins, 2014, Chapter 13:
Fluid and Electrolytes: Balance and Disturbance, p. 241.
Chapter 13: Fluid and Electrolytes: Balance and Disturbance - Page 241

Question 9: A nurse reviews the arterial blood gas (ABG) values of a client admitted
(see full question) with pneumonia: pH, 7.51; PaCO2, 28 mm Hg; PaO2, 70 mm Hg; and HCO3--, 24
mEq/L. What do these values indicate?
You selected: Respiratory alkalosis
Correct
Explanation: A client with pneumonia may hyperventilate in an effort to increase oxygen intake.
Hyperventilation leads to excess carbon dioxide (CO2) loss, which causes alkalosis
indicated by this client's elevated pH value. With respiratory alkalosis, the
kidneys' bicarbonate (HCO3) response is delayed, so the client's HCO3 level
remains normal. The below-normal value for the partial pressure of arterial carbon
dioxide (PaCO2) indicates CO2 loss and signals a respiratory component. Because
the HCO3 level is normal, this imbalance has no metabolic component. Therefore,
the client is experiencing respiratory alkalosis. (less)

Reference: Hinkle, J.L., and Cheever, K.H. Brunner & Suddarth's Textbook of Medical-Surgical
Nursing, 13th ed. Philadelphia: Lippincott Williams & Wilkins, 2014, Chapter 13:
Fluid and Electrolytes: Balance and Disturbance, p. 270.
Chapter 13: Fluid and Electrolytes: Balance and Disturbance - Page 270

Question 10: A nurse can estimate serum osmolality at the bedside by using a formula. A
(see full question) patient who has a serum sodium level of 140 mEq/L would have a serum
osmolality of:
You selected: 280 mOsm/kg.
Correct
Explanation: Serum osmolality can be estimated by doubling the serum sodium or using the
formula: Na 2 = glucose/18 + BUN/3. Therefore, the nurse could estimate a
serum osmolality of 280 mOsm/kg by doubling the serum sodium value of 140
mEq/L. (less)

Reference: Hinkle, J.L., and Cheever, K.H. Brunner & Suddarth's Textbook of Medical-Surgical
Nursing, 13th ed. Philadelphia: Lippincott Williams & Wilkins, 2014, Chapter 13:
Fluid and Electrolytes: Balance and Disturbance, p. 241.
Chapter 13: Fluid and Electrolytes: Balance and Disturbance - Page 241

Answer Key

Question 1: After suctioning a tracheostomy tube, the nurse assesses the client to determine
(see full question) the effectiveness of the suctioning. Which findings indicate that the airway is
now patent?
You selected: Effective breathing at a rate of 16 breaths/minute through the established airway
Correct
Explanation: Proper suctioning should produce a patent airway, as demonstrated by effective
breathing through the airway at a normal respiratory rate of 12 to 20
breaths/minute. The other options suggest ineffective suctioning. A respiratory
rate of 28 breaths/minute and accessory muscle use may indicate mild respiratory
distress. Increased pulse rate, rapid respirations, and cyanosis are signs
of hypoxia. Restlessness, pallor, increased pulse and respiratory rates, and
bubbling breath sounds indicate respiratory secretion accumulation. (less)

Reference: Hinkle, J.L., and Cheever, K.H. Brunner & Suddarth's Textbook of Medical-Surgical
Nursing, 13th ed. Philadelphia: Lippincott Williams & Wilkins, 2014, Chapter 21:
Respiratory Care Modalities, p. 515.
Chapter 21: Respiratory Care Modalities - Page 515

Question 2: A nurse has performed tracheal suctioning on a patient who experienced


(see full question) increasing dyspnea prior to a procedure. When applying the nursing process, how
can the nurse best evaluate the outcomes of this intervention?
You selected: Measure the patient's oxygen saturation.
Correct
Explanation: The patient's response to suctioning is usually determined by performing chest
auscultation and by measuring the patient's oxygen saturation. FET, incentive
spirometry, and percussion are not normally used as evaluative techniques. (less)

Reference: Hinkle, J.L., and Cheever, K.H. Brunner & Suddarth's Textbook of Medical-Surgical
Nursing, 13th ed., Philadelphia: Lippincott Williams & Wilkins, 2014, Chapter 21:
Respiratory Care Modalities, p. 510.
Chapter 21: Respiratory Care Modalities - Page 510

Question 3: What would the critical care nurse recognize as a condition that may indicate a
(see full question) patient's need to have a tracheostomy?
You selected: A patient requires permanent ventilation.
Correct
Explanation: A tracheostomy permits long-term use of mechanical ventilation to prevent
aspiration of oral and gastric secretions in the unconscious or paralyzed patient.
Indications for a tracheostomy do not include a respiratory rate of 10 breaths per
minute, symptoms of dyspnea, or respiratory acidosis. (less)

Reference: Hinkle, J.L., and Cheever, K.H. Brunner & Suddarth's Textbook of Medical-Surgical
Nursing, 13th ed., Philadelphia: Lippincott Williams & Wilkins, 2014, Chapter 21:
Respiratory Care Modalities, p. 506.
Chapter 21: Respiratory Care Modalities - Page 506

Question 4: A nurse is caring for a client who has a tracheostomy and temperature of 103 F
(see full question) (39.4 C). Which intervention will most likely lower the client's arterial blood
oxygen saturation?
You selected: Endotracheal suctioning
Correct
Explanation: Endotracheal suctioning removes secretions as well as gases from the airway and
lowers the arterial oxygen saturation (SaO2) level. Coughing and using
an incentive spirometer improve oxygenation and should raise or maintain oxygen
saturation. Because of superficial vasoconstriction, using a cooling blanket can
lower peripheral oxygen saturation readings, but SaO2 levels wouldn't be affected.
(less)

Reference: Hinkle, J.L., and Cheever, K.H. Brunner & Suddarth's Textbook of Medical-Surgical
Nursing, 13th ed. Philadelphia: Lippincott Williams & Wilkins, 2014, Chapter 21:
Respiratory Care Modalities, p. 530.
Chapter 21: Respiratory Care Modalities - Page 530

Question 5: A patient is exhibiting signs of a pneumothorax following tracheostomy. The


(see full question) surgeon inserts a chest tube into the anterior chest wall. What should the nurse
tell the family is the primary purpose of this chest tube?
You selected: To remove air from the pleural space
Correct
Explanation: Chest tubes and closed drainage systems are used to re-expand the lung involved
and to remove excess air, fluid, and blood. The primary purpose of a chest tube is
not to drain sputum secretions, monitor bleeding, or assist with mechanical
ventilation. (less)

Reference: Hinkle, J.L., and Cheever, K.H. Brunner & Suddarth's Textbook of Medical-Surgical
Nursing, 13th ed., Philadelphia: Lippincott Williams & Wilkins, 2014, Chapter 21:
Respiratory Care Modalities, p. 524.
Chapter 21: Respiratory Care Modalities - Page 524

Question 6: A nurse is assisting with a subclavian vein central line insertion when the client's
(see full question) oxygen saturation drops rapidly. He complains of shortness of breath and
becomes tachypneic. The nurse suspects the client has developed
apneumothorax. Further assessment findings supporting the presence of a
pneumothorax include:
You selected: diminished or absent breath sounds on the affected side.
Correct
Explanation: In the case of a pneumothorax, auscultating for breath sounds will reveal absent
or diminished breath sounds on the affected side. Paradoxical chest wall
movements occur in flail chest conditions. Tracheal deviation occurs in a tension
pneumothorax. Muffled or distant heart sounds occur in cardiac tamponade. (less)

Reference: Hinkle, J.L., and Cheever, K.H. Brunner & Suddarth's Textbook of Medical-Surgical
Nursing, 13th ed. Philadelphia: Lippincott Williams & Wilkins, 2014, Chapter 24:
Management of Patients With Chronic Pulmonary Disease, p. 630.
Chapter 24: Management of Patients With Chronic Pulmonary Disease - Page 630

Question 7: A nurse is caring for a patient with a subclavian central line who is receiving
(see full question) parenteral nutrition (PN). In preparing a care plan for this patient, what nursing
diagnosis should the nurse prioritize?
You selected: Risk for Infection Related to the Presence of a Subclavian Catheter
Correct
Explanation: The high glucose content of PN solutions makes the solutions an idea culture
media for bacterial and fungal growth, and the central venous access devices
provide a port of entry. Prevention of infection is consequently a high priority. The
patient will experience some inconveniences with regard to toileting, activity, and
sleep, but the infection risk is a priority over each of these. (less)

Reference: Hinkle, J.L., and Cheever, K.H. Brunner & Suddarth's Textbook of Medical-Surgical
Nursing, 13th ed., Philadelphia: Lippincott Williams & Wilkins, 2014, Chapter 45:
Digestive and Gastrointestinal Treatment Modalities, p. 1231.
Chapter 45: Digestive and Gastrointestinal Treatment Modalities - Page 1231

Question 8: The nurse is caring for a patient with a serum potassium level of 6.0 mEq/L. The
(see full question) patient is ordered to receive oral sodium polystyrene sulfonate (Kayexelate) and
furosemide (Lasix). What other orders should the nurse anticipate giving?
You selected: Discontinue the IV lactated Ringers solution.
Correct
Explanation: The lactated Ringers IV fluid is contributing to both the fluid volume excess and
the hyperkalemia. In addition to the volume of IV fluids contributing to the fluid
volume excess, lactated Ringers contains more sodium than daily requirements
and excess sodium worsens fluid volume excess. Lactated Ringers also contains
potassium, which would worsen the hyperkalemia. (less)

Reference: Hinkle, J.L., and Cheever, K.H. Brunner & Suddarth's Textbook of Medical-Surgical
Nursing, 13th ed., Philadelphia: Lippincott Williams & Wilkins, 2014, Chapter 13:
Fluid and Electrolytes: Balance and Disturbance, p. 273.
Chapter 13: Fluid and Electrolytes: Balance and Disturbance - Page 273

Question 9: The nurse is preparing to administer IV fluids for a patient with ketoacidosis who
(see full question) has a history of hypertension and congestive heart failure. What order for fluids
would the nurse anticipate infusing for this patient?
You selected: 0.45 normal saline
Correct
Explanation: Half-strength NS (0.45%) solution (also known as hypotonic saline solution) may
be used for rehydration of patients with hypertension or hypernatremia and those
at risk for heart failure. (less)

Reference: Hinkle, J.L., and Cheever, K.H. Brunner & Suddarth's Textbook of Medical-Surgical
Nursing, 13th ed. Philadelphia: Lippincott Williams & Wilkins, 2014, Chapter 51:
Assessment and Management of Patients With Diabetes, p. 1444.
Chapter 51: Assessment and Management of Patients With Diabetes - Page 1444

Question 10: A patient with end-stage liver disease has developed hypervolemia. What nursing
(see full question) interventions would be most appropriate when addressing the patient's fluid
volume excess? Select all that apply.
You selected: Administering diuretics
Implementing fluid restrictions
Enhancing patient positioning
Correct
Explanation: Administering diuretics, implementing fluid restrictions, and enhancing patient
positioning can optimize the management of fluid volume excess. Calcium channel
blockers and calorie ... (more)

Reference: Hinkle, J.L., and Cheever, K.H. Brunner & Suddarth's Textbook of Medical-Surgical
Nursing, 13th ed., Philadelphia: Lippincott Williams & Wilkins, 2014, Chapter 49:
Assessment and Management of Patients With Hepatic Disorders, p. 1377.
Chapter 49: Assessment and Management of Patients With Hepatic Disorders -
Page 1377

Answer Key

Question 1: A patient is ordered to receive hypotonic IV solution to provide free water


(see full question) replacement. Which of the following solutions will the nurse anticipate
administering?
You selected: 0.45% NaCl
Correct
Explanation: Half-strength saline (0.45%) is hypotonic. Hypotonic solutions are used to replace
cellular fluid because it is hypotonic compared with plasma. Another is to provide
free water to excrete body wastes. At times, hypotonic sodium solutions are used
to treat hypernatremia and other hyperosmolar conditions. Lactated Ringers
solution and normal saline (0.9% NaCl) are isotonic. A solution that is 5% NaCl is
hypertonic. (less)

Reference: Hinkle, J.L., and Cheever, K.H. Brunner & Suddarth's Textbook of Medical-Surgical
Nursing, 13th ed., Philadelphia: Lippincott Williams & Wilkins, 2014, Chapter 13:
Fluid and Electrolytes: Balance and Disturbance, p. 273.
Chapter 13: Fluid and Electrolytes: Balance and Disturbance - Page 273

Question 2: Which conditions leads to chronic respiratory acidosis in older adults?


(see full question)
You selected: Thoracic skeletal change
Correct
Explanation: Poor respiratory exchange as the result of chronic lung disease, inactivity, or
thoracic skeletal changes may lead to chronic respiratory acidosis. Decreased
renal function in older adults can cause an inability to concentrate urine and is
usually associated with fluid and electrolyte imbalance. A poor appetite, erratic
meal patterns, inability to prepare nutritious meals, or financial circumstances
may influence nutritional status, resulting in imbalances of electrolytes. Overuse
of sodium bicarbonate may lead to metabolic alkalosis. (less)

Reference: Hinkle, J.L., and Cheever, K.H. Brunner & Suddarth's Textbook of Medical-Surgical
Nursing, 13th ed. Philadelphia: Lippincott Williams & Wilkins, 2014, Chapter 13:
Fluid and Electrolytes: Balance and Disturbance, p. 270.
Chapter 13: Fluid and Electrolytes: Balance and Disturbance - Page 270

Question 3: A patients serum sodium concentration is within the normal range. What should
(see full question) the nurse estimate the serum osmolality to be?
You selected: 275300 mOsm/kg
Correct
Explanation: In healthy adults, normal serum osmolality is 270 to 300 mOsm/kg (Crawford &
Harris, 2011c).
Reference: Hinkle, J.L., and Cheever, K.H. Brunner & Suddarth's Textbook of Medical-Surgical
Nursing, 13th ed. Philadelphia: Lippincott Williams & Wilkins, 2014, Chapter 13:
Fluid and Electrolytes: Balance and Disturbance, p. 241.
Chapter 13: Fluid and Electrolytes: Balance and Disturbance - Page 241

Question 4: When evaluating arterial blood gases (ABGs), which value is consistent with
(see full question) metabolic alkalosis?
You selected: pH 7.48
Correct
Explanation: Metabolic alkalosis is a clinical disturbance characterized by a high pH and high
plasma bicarbonate concentration. The HCO value is below normal. The PaCO
value and the oxygen saturation level are within a normal range. (less)

Reference: Hinkle, J.L., and Cheever, K.H. Brunner & Suddarth's Textbook of Medical-Surgical
Nursing, 13th ed. Philadelphia: Lippincott Williams & Wilkins, 2014, Chapter 13:
Fluid and Electrolytes: Balance and Disturbance, p. 269.
Chapter 13: Fluid and Electrolytes: Balance and Disturbance - Page 269

Question 5: A physician orders an isotonic I.V. solution for a client. Which solution should the
(see full question) nurse plan to administer?
You selected: Lactated Ringer's solution
Correct
Explanation: Lactated Ringer's solution, with an osmolality of approximately 273 mOsm/L,
is isotonic. The nurse shouldn't give half-normal saline solution because
it's hypotonic, with an osmolality of 154 mOsm/L. Giving 5% dextrose and normal
saline solution (with an osmolality of 559 mOsm/L) or 10% dextrose in water (with
an osmolality of 505 mOsm/L) also would be incorrect because these solutions
are hypertonic. (less)

Reference: Hinkle, J.L., and Cheever, K.H. Brunner & Suddarth's Textbook of Medical-Surgical
Nursing, 13th ed. Philadelphia: Lippincott Williams & Wilkins, 2014, Chapter 13:
Fluid and Electrolytes: Balance and Disturbance, p. 248, 273.
Chapter 13: Fluid and Electrolytes: Balance and Disturbance - Page 248

Question 6: A 160-pound patient, diagnosed with hypovolemia, is weighed every day. The
(see full question) health care provider asked to be notified if the patient loses 1,000 mL of fluid in 24
hours. Choose the weight that would be consistent with this amount of fluid loss.
You selected: 158 lbs
Correct
Explanation: A loss of 0.5 kg or 1 lb represents a fluid loss of about 500 mL. Therefore, a loss of
1,000 mL would be equivalent to the loss of 2 lbs (160 2 = 158 lbs).

Reference: Hinkle, J.L., and Cheever, K.H. Brunner & Suddarth's Textbook of Medical-Surgical
Nursing, 13th ed. Philadelphia: Lippincott Williams & Wilkins, 2014, Chapter 13:
Fluid and Electrolytes: Balance and Disturbance, p. 247.
Chapter 13: Fluid and Electrolytes: Balance and Disturbance - Page 247

Question 7: A client presents with fatigue, nausea, vomiting, muscle weakness, and leg
(see full question) cramps. Laboratory values are as follows:

Na + 147 mEq/L
K + 3.0 mEq/L
Cl - 112 mEq/L
Mg ++ 2.3 mg/dL
Ca ++ 1.5 mg/dL

Which of the following is consistent with the client's findings?


You selected: Hypokalemia
Correct
Explanation: Potassium is the major intracellular electrolyte. Hypocalemia (below 3.5 mEq/L)
usually indicates a deficit in total potassium stores. Potassium deficiency can
result in derangements in physiology. Clinical signs include fatigue, anorexia,
nausea, vomiting, muscles weakness, leg cramps, decreased bowel motility, and
paresthesias. (less)

Reference: Hinkle, J.L., and Cheever, K.H. Brunner & Suddarth's Textbook of Medical-Surgical
Nursing, 13th ed. Philadelphia: Lippincott Williams & Wilkins, 2014, Chapter 13:
Fluid and Electrolytes: Balance and Disturbance, p. 255.
Chapter 13: Fluid and Electrolytes: Balance and Disturbance - Page 255

Question 8: A client was admitted to your unit with a diagnosis of hypovolemia. When it is time
(see full question) to complete discharge teaching, which of the following will the nurse teach the
client and his family? Select all that apply.
You selected: Drink at least eight glasses of fluid each day.
Respond to thirst
Drink water as an inexpensive way to meet fluid needs.
Correct
Explanation: In addition, the nurse teaches clients who have a potential for hypovolemia and
their families to respond to thirst because it is an early indication of reduced fluid
volume; consume at least 8 to 10 (8 ounce) glasses of fluid each day and more
during hot, humid weather; drink water as an inexpensive means to meet fluid
requirements; and avoid beverages with alcohol and caffeine because they
increase urination and contribute to fluid deficits. (less)

Reference: Hinkle, J.L., and Cheever, K.H. Brunner & Suddarth's Textbook of Medical-Surgical
Nursing, 13th ed. Philadelphia: Lippincott Williams & Wilkins, 2014, Chapter 13:
Fluid and Electrolytes: Balance and Disturbance, pp. 245, 247, 249.
Chapter 13: Fluid and Electrolytes: Balance and Disturbance - Page 245

Question 9: The nurse is caring for a client in heart failure with signs of hypervolemia. Which
(see full question) vital sign is indicative of the disease process?
You selected: Elevated blood pressure
Correct
Explanation: Indicative of hypervolemia is a bounding pulse and elevated blood pressure due to
the excess volume in the system. Respirations are not typically affected unless
there is fluid accumulation in the lungs. Temperature is not generally affected.
(less)

Reference: Hinkle, J.L., and Cheever, K.H. Brunner & Suddarth's Textbook of Medical-Surgical
Nursing, 13th ed. Philadelphia: Lippincott Williams & Wilkins, 2014, Chapter 13:
Fluid and Electrolytes: Balance and Disturbance, p. 246.
Chapter 13: Fluid and Electrolytes: Balance and Disturbance - Page 246
Question 10: A patient is diagnosed with hypocalcemia. The nurse advises the patient and his
(see full question) family to immediately report the most characteristic manifestation. What is the
most characteristic manifestation?
You selected: Tingling or twitching sensation in the fingers
Correct
Explanation: All the choices are signs and symptoms of hypocalcemia, but tetany is the most
characteristic manifestation that occurs when the calcium level is less than 4.4
mg/dL.

Reference: Hinkle, J.L., and Cheever, K.H. Brunner & Suddarth's Textbook of Medical-Surgical
Nursing, 13th ed. Philadelphia: Lippincott Williams & Wilkins, 2014, Chapter 13:
Fluid and Electrolytes: Balance and Disturbance, p. 259.
Chapter 13: Fluid and Electrolytes: Balance and Disturbance - Page 259

Question 11: A nurse is caring for a client admitted with a diagnosis of exacerbation of
(see full question) myasthenia gravis. Upon assessment of the client, the nurse notes the client has
severely depressed respirations. The nurse would expect to identify which acid-
base disturbance?
You selected: Respiratory acidosis
Correct
Explanation: Respiratory acidosis is always from inadequate excretion of CO 2 with inadequate
ventilation, resulting in elevated plasma CO2 concentrations. Respiratory acidosis
can occur in diseases that impair respiratory muscles such as myasthenia gravis.
(less)

Reference: Hinkle, J.L., and Cheever, K.H. Brunner & Suddarth's Textbook of Medical-Surgical
Nursing, 13th ed. Philadelphia: Lippincott Williams & Wilkins, 2014, Chapter 13:
Fluid and Electrolytes: Balance and Disturbance, p. 270.
Chapter 13: Fluid and Electrolytes: Balance and Disturbance - Page 270

Question 12: Which of the following arterial blood gas results would be consistent with
(see full question) metabolic alkalosis?
You selected: Serum bicarbonate of 28 mEq/L
Correct
Explanation: Evaluation of arterial blood gases reveals a pH greater than 7.45 and a serum
bicarbonate concentration greater than 26 mEq/L.

Reference: Hinkle, J.L., and Cheever, K.H. Brunner & Suddarth's Textbook of Medical-Surgical
Nursing, 13th ed. Philadelphia: Lippincott Williams & Wilkins, 2014, Chapter 13:
Fluid and Electrolytes: Balance and Disturbance, p. 269.
Chapter 13: Fluid and Electrolytes: Balance and Disturbance - Page 269

Question 13: With which condition should the nurse expect that a decrease in serum osmolality
(see full question) will occur?
You selected: Diabetes insipidus
Incorrect
Correct response: Kidney failure
Explanation: Failure of the kidneys results in multiple fluid and electrolyte abnormalities.
Reference: Hinkle, J.L., and Cheever, K.H. Brunner & Suddarth's Textbook of Medical-Surgical
Nursing, 13th ed. Philadelphia: Lippincott Williams & Wilkins, 2014, Chapter 13:
Fluid and Electrolytes: Balance and Disturbance, p. 242.
Chapter 13: Fluid and Electrolytes: Balance and Disturbance - Page 242

Question 14: A nurse reviews the results of an electrocardiogram (ECG) for a patient who is
(see full question) being assessed for hypokalemia. Which of the following would the nurse notice as
the most significant diagnostic indicator?
You selected: Elevated U wave
Correct
Explanation: An elevated U wave is specific for hypokalemia. Flat or inverted T waves may also
be present. The other tracings are consistent with hyperkalemia.

Reference: Hinkle, J.L., and Cheever, K.H. Brunner & Suddarth's Textbook of Medical-Surgical
Nursing, 13th ed. Philadelphia: Lippincott Williams & Wilkins, 2014, Chapter 13:
Fluid and Electrolytes: Balance and Disturbance, p. 255.
Chapter 13: Fluid and Electrolytes: Balance and Disturbance - Page 255
Question 15: A nurse is caring for an adult client with numerous draining wounds from
(see full question) gunshots. The client's pulse rate has increased from 100 to 130 beats per minute
over the last hour. The nurse should further assess the client for which of the
following?
You selected: Extracellular fluid volume deficit
Correct
Explanation: Fluid volume deficit (FVD) occurs when the loss extracellular fluid (ECF) volume
exceeds the intake of fluid. FVD results from loss of body fluids and occurs more
rapidly when coupled with decreaesd fluid intake. A cause of this loss is
hemorrhage. (less)

Reference: Hinkle, J.L., and Cheever, K.H. Brunner & Suddarth's Textbook of Medical-Surgical
Nursing, 13th ed. Philadelphia: Lippincott Williams & Wilkins, 2014, Chapter 13:
Fluid and Electrolytes: Balance and Disturbance, p. 245.
Chapter 13: Fluid and Electrolytes: Balance and Disturbance - Page 245

Question 16: Which nerve is implicated in the Chvosteks sign?


(see full question)
You selected: Facial
Correct
Explanation: Chvosteks sign consists of twitching of muscles supplied by the facial nerve when
the nerve is tapped about 2 cm anterior to the earlobe, just below the zygomatic
arch.

Reference: Hinkle, J.L., and Cheever, K.H. Brunner & Suddarth's Textbook of Medical-Surgical
Nursing, 13th ed. Philadelphia: Lippincott Williams & Wilkins, 2014, Chapter 13:
Fluid and Electrolytes: Balance and Disturbance, p. 259.
Chapter 13: Fluid and Electrolytes: Balance and Disturbance - Page 259

Question 17: The nurse is caring for a patient with a serum sodium level of 113 mEq/L. The
(see full question) nurse should monitor the patient for the development of which of the following?
You selected: Confusion
Correct
Explanation: Normal serum concentration level ranges from 135 to 145 mEq/L. Hyponatremia
exists when the serum level decreases below 135 mEq/L, there is. When the serum
sodium level decreases to less than 115 mEq/L (115 mmol/L), signs of increasing
intracranial pressure, such as lethargy, confusion, muscle twitching, focal
weakness, hemiparesis, papilledema, seizures, and death, may occur. General
manifestations of hyponatremia include poor skin turgor, dry mucosa, headache,
decreased saliva production, orthostatic fall in blood pressure, nausea, vomiting,
and abdominal cramping. Neurologic changes, including altered mental status,
status epilepticus, and coma, are probably related to cellular swelling and cerebral
edema associated with hyponatremia. Hallucinations are associated with
increased serum sodium levels. (less)

Reference: Hinkle, J.L., and Cheever, K.H. Brunner & Suddarth's Textbook of Medical-Surgical
Nursing, 13th ed., Philadelphia: Lippincott Williams & Wilkins, 2014, Chapter 13:
Fluid and Electrolytes: Balance and Disturbance, p. 251.
Chapter 13: Fluid and Electrolytes: Balance and Disturbance - Page 251

Question 18: A 77-year-old retired male client visits your general practice office twice monthly
(see full question) to maintain control of his congestive heart failure. He measures his weight daily
and phones it to your office for his medical record. In a 24-hour period, how much
fluid is this client retaining if his weight increases by 2 lb?
You selected: 500 ml
Incorrect
Correct response: 1L
Explanation: A 2-lb weight gain in 24 hours indicates that the client is retaining 1L of fluid.
Reference: Hinkle, J.L., and Cheever, K.H. Brunner & Suddarth's Textbook of Medical-Surgical
Nursing, 13th ed. Philadelphia: Lippincott Williams & Wilkins, 2014, Chapter 13:
Fluid and Electrolytes: Balance and Disturbance, p. 247.
Chapter 13: Fluid and Electrolytes: Balance and Disturbance - Page 247

Question 19: Before seeing a newly assigned client with respiratory alkalosis, a nurse quickly
(see full question) reviews the client's medical history. Which condition is a predisposing factor
for respiratory alkalosis?
You selected: Extreme anxiety
Correct
Explanation: Extreme anxiety may lead to respiratory alkalosis by causing hyperventilation,
which results in excessive carbon dioxide (CO2) loss. Other conditions that may set
the stage for respiratory alkalosis include fever, heart failure, injury to the brain's
respiratory center, overventilation with a mechanical ventilator, pulmonary
embolism, and early salicylate intoxication. Type 1 diabetes may lead to diabetic
ketoacidosis; the deep, rapid respirations occurring in this disorder (Kussmaul's
respirations) don't cause excessive CO2 loss. Myasthenia
gravis and opioid overdose suppress the respiratory drive, causing CO2 retention,
not CO2 loss; this may lead to respiratory acidosis, not alkalosis. (less)

Reference: Hinkle, J.L., and Cheever, K.H. Brunner & Suddarth's Textbook of Medical-Surgical
Nursing, 13th ed. Philadelphia: Lippincott Williams & Wilkins, 2014, Chapter 13:
Fluid and Electrolytes: Balance and Disturbance, p. 270.
Chapter 13: Fluid and Electrolytes: Balance and Disturbance - Page 270

Question 20: A patient is being treated in the ICU 24 hours after having a radical neck
(see full question) dissection completed. The patients serum calcium level is 7.6 mg/dL. Which of the
following physical examination findings is consistent with this electrolyte
imbalance?
You selected: Presence of Trousseaus sign
Correct
Explanation: A patient status post radical neck resection is prone to developing hypocalcemia.
Hypocalcemia is defined as a serum values lower than 8.6 mg/dL [2.15 mmol/L].
Signs and symptoms of hypocalcemia include: Chvosteks sign, which consists of
muscle twitching enervated by the facial nerve when the region that is about 2 cm
anterior to the earlobe, just below the zygomatic arch, is tapped, and a positive
Trousseaus sign can be elicited by inflating a blood pressure cuff on the upper arm
to about 20 mm Hg above systolic pressure; within 2 to 5 minutes, carpal spasm
(an adducted thumb, flexed wrist and metacarpophalangeal joints, and extended
interphalangeal joints with fingers together) will occur as ischemia of the ulnar
nerve develops. Slurred speech and muscle weakness are signs of hypercalcemia.
(less)

Reference: Hinkle, J.L., and Cheever, K.H. Brunner & Suddarth's Textbook of Medical-Surgical
Nursing, 13th ed., Philadelphia: Lippincott Williams & Wilkins, 2014, Chapter 13:
Fluid and Electrolytes: Balance and Disturbance, p. 259.
Chapter 13: Fluid and Electrolytes: Balance and Disturbance - Page 259

Answer Key

Question 1: A client on long-term mechanical ventilation becomes very frustrated when he


(see full question) tries to communicate. Which intervention should the nurse perform to assist the
client?
You selected: Ask the client to write, use a picture board, or spell words with an alphabet board.
Correct
Explanation: If the client uses an alternative method of communication, such as writing, using a
picture board, or spelling words on an alphabet board, he'll feel more in control
and be less frustrated. Assuring the client that everything will be all right offers
false reassurance and telling him not to be upset minimizes his feelings. Neither of
these methods helps the client to communicate. Family members are also likely to
encounter difficulty interpreting the wishes of a client with an endotracheal tube
or tracheostomy tube. Making them responsible for interpreting the client's
gestures may frustrate them. The client may be weaned off a mechanical
ventilator only when the physiologic parameters for weaning have been met.
(less)

Reference: Hinkle, J.L., and Cheever, K.H. Brunner & Suddarth's Textbook of Medical-Surgical
Nursing, 13th ed. Philadelphia: Lippincott Williams & Wilkins, 2014, Chapter 21:
Respiratory Care Modalities, p. 516.
Chapter 21: Respiratory Care Modalities - Page 516

Question 2: A nurse is caring for a client who recently underwent a tracheostomy.


(see full question) The first priority when caring for a client with a tracheostomy is:
You selected: keeping his airway patent.
Correct
Explanation: Maintaining a patent airway is the most basic and critical human need. Helping the
client communicate, encouraging him to perform ADLs, and preventing him from
developing an infect ... (more)

Reference: Hinkle, J.L., and Cheever, K.H. Brunner & Suddarth's Textbook of Medical-Surgical
Nursing, 13th ed. Philadelphia: Lippincott Williams & Wilkins, 2014, Chapter 21:
Respiratory Care Modalities, p. 520.
Chapter 21: Respiratory Care Modalities - Page 520

Question 3: What would the critical care nurse recognize as a condition that may indicate a
(see full question) patient's need to have a tracheostomy?
You selected: A patient requires permanent ventilation.
Correct
Explanation: A tracheostomy permits long-term use of mechanical ventilation to prevent
aspiration of oral and gastric secretions in the unconscious or paralyzed patient.
Indications for a trach ... (more)

Reference: Hinkle, J.L., and Cheever, K.H. Brunner & Suddarth's Textbook of Medical-Surgical
Nursing, 13th ed., Philadelphia: Lippincott Williams & Wilkins, 2014, Chapter 21:
Respiratory Care Modalities, p. 506.
Chapter 21: Respiratory Care Modalities - Page 506

Question 4: The nurse is preparing to perform tracheostomy care on a patient with a newly
(see full question) inserted tracheostomy tube. Which of the following actions, if preformed by the
nurse, indicates the need for further review of the procedure?
You selected: Places clean tracheostomy ties, and removes soiled ties after the new ties are in
place
Correct
Explanation: For a new tracheostomy, two people should assist with tie changes. The other
actions, if performed by the nurse during tracheostomy care, are correct.

Reference: Hinkle, J.L., and Cheever, K.H. Brunner & Suddarth's Textbook of Medical-Surgical
Nursing, 13th ed., Philadelphia: Lippincott Williams & Wilkins, 2014, Chapter 21:
Respiratory Care Modalities, p. 508.
Chapter 21: Respiratory Care Modalities - Page 508

Question 5: The nurse assesses that extravasation of a chemotherapy agent has occurred.
(see full question) What should the initial action of the nurse be?
You selected: Discontinue the infusion.
Correct
Explanation: If extravasation is suspected, the medication administration is stopped
immediately, and depending on the drug, the nurse may attempt to aspirate any
remaining drug from the extravasation site. The other actions listed may be
appropriate to perform, but should occur after discontinuing the infusion. (less)

Reference: Hinkle, J.L., and Cheever, K.H. Brunner & Suddarth's Textbook of Medical-Surgical
Nursing, 13th ed. Philadelphia: Lippincott Williams & Wilkins, 2014, Chapter 15:
Oncology: Nursing Management in Cancer Care, p. 335.
Chapter 15: Oncology: Nursing Management in Cancer Care - Page 335

Question 6: A client who is receiving IV fluid therapy has localized edema. What has caused
(see full question) this complication?
You selected: Fluid infiltration of the surrounding tissue
Correct
Explanation: If the venous access device fails to remain in the vein, fluid infiltrates the tissue,
causing localized edema. A thrombus, or clot, may form in the vein as a result of
trauma to the vein. If the blood clot breaks free and travels to the lungs, it will
cause a pulmonary embolism. Signs of infection are swelling, discomfort, and
redness or drainage from the site. One potentially fatal complication of IV fluid
therapy is the development of a bolus of air that reaches the lungs via the venous
system. (less)

Reference: Hinkle, J.L., and Cheever, K.H. Brunner & Suddarth's Textbook of Medical-Surgical
Nursing, 13th ed. Philadelphia: Lippincott Williams & Wilkins, 2014, Chapter 13:
Fluid and Electrolytes: Balance and Disturbance, p. 281.
http://thepoint.lww.com/vitalsource/display/9781469852744/page/281

Question 7: Air embolism is a potential complication of IV therapy. The nurse should be alert to
(see full question) which clinical manifestation associated with air embolism?
You selected: Chest pain
Correct
Explanation: Manifestations of air embolism include dyspnea and cyanosis; hypotension; weak,
rapid pulse; loss of consciousness; and chest, shoulder, and low back pain.
Jaundice is not associat ... (more)

Reference: Hinkle, J.L., and Cheever, K.H. Brunner & Suddarth's Textbook of Medical-Surgical
Nursing, 13th ed. Philadelphia: Lippincott Williams & Wilkins, 2014, Chapter 13:
Fluid and Electrolytes: Balance and Disturbance, p. 280.
Chapter 13: Fluid and Electrolytes: Balance and Disturbance - Page 280

Question 8: A patient receiving antiretroviral therapy is complaining of not urinating enough.


(see full question) What is the nurses best action?
You selected: Assess BUN and creatinine.
Correct
Explanation: Adverse effects associated with antiretroviral therapy include potential
nephrotoxicity. Assessing BUN and creatinine for patients who have decreased
urination is appropriate. The other answers will not assist the nurse in determining
the patients problem. Assessment of the problem should be undertaken before
interventions. (less)

Reference: Hinkle, J.L., and Cheever, K.H. Brunner & Suddarth's Textbook of Medical-Surgical
Nursing, 13th ed., Philadelphia: Lippincott Williams & Wilkins, 2014, Chapter 37:
Management of Patients With HIV Infection and AIDS, p. 1006.
Chapter 37: Management of Patients With HIV Infection and AIDS - Page 1006

Question 9: A 64-year-old male client needs an IV started. Which of the following nursing
(see full question) interventions would the nurse follow to prepare this client? Select all that apply.
You selected: Verify the identity of the client using multiple methods.
Give the client an idea of how much discomfort the procedure will cause.
Bring the IV equipment into the clients room first.
Explain how long the procedure is expected to take.
Incorrect
Correct response: Verify the identity of the client using multiple methods.
Explain how long the procedure is expected to take.
Give the client an idea of how much discomfort the procedure will cause.
Explanation: The process of preparing the client for IV therapy includes verifying the clients
identity, explaining the purpose of the IV, answering the clients questions, and
reviewing things like the time involved and level of discomfort. To reduce the
clients anxiety, it is best to review the purpose of the IV before bringing the
equipment to the clients room. (less)

Reference: Hinkle, J.L., and Cheever, K.H. Brunner & Suddarth's Textbook of Medical-Surgical
Nursing, 13th ed. Philadelphia: Lippincott Williams & Wilkins, 2014, Chapter 13:
Fluid and Electrolytes: Balance and Disturbance, p. 275.
http://thepoint.lww.com/vitalsource/display/9781469852744/page/275

Question 10: The client has just had a central line inserted for parenteral nutrition. The client is
(see full question) awaiting transport to the Radiology Department for catheter placement
verification. The client reports feeling anxious. Respirations are 28 breaths/minute.
The first action of the nurse is
You selected: Elevate the head of the bed.
Incorrect
Correct response: Auscultate lung sounds.
Explanation: Following placement of a central line, the client is at risk for a pneumothorax. The
client's report of anxiety and increased respiratory rate may be the first signs and
symptoms of a pneumothorax. The nurst first assesses the client by auscultating
lung sounds. Other actions include placing the client in Fowler's position and
consulting with the healthcare provider about findings. (less)

Reference: Hinkle, J.L., and Cheever, K.H. Brunner & Suddarth's Textbook of Medical-Surgical
Nursing, 13th ed. Philadelphia: Lippincott Williams & Wilkins, 2014, Chapter 45:
Digestive and Gastrointestinal Treatment Modalities, p. 1231.
Chapter 45: Digestive and Gastrointestinal Treatment Modalities - Page 1231
Question 1 See full question
Which strategy can help make the nurse a more effective teacher?
You Selected:
Including the client in the discussion
Correct response:
Including the client in the discussion
Explanation:
Question 2 See full question
A client with stage 1 Alzheimer's disease is diagnosed with terminal lung cancer. He asks the nurse what he should
do when he "reaches the end." How should the nurse respond?
You Selected:
"An advance directive will help to make sure that your wishes are carried out."
Correct response:
"An advance directive will help to make sure that your wishes are carried out."
Explanation:
Question 3 See full question
What is the advantage of using automated medication dispensing equipment?
You Selected:
It keeps a record of narcotic usage.
Correct response:
It keeps a record of narcotic usage.
Explanation:
Question 4 See full question
A child with spastic cerebral palsy receiving intrathecal baclofen therapy is admitted to the pediatric floor with
vomiting and dehydration. The family tells the nurse that they were scheduled to refill the baclofen pump today, but
had to cancel the appointment when the child became ill. The nurse should:
You Selected:
reschedule the pump refill for the day of discharge.
Correct response:
arrange for the pump to be refilled in the hospital.
Explanation:
Question 5 See full question
The client has returned to the surgery unit from the postanesthesia care unit (PACU). The clients respirations are
rapid and shallow, the pulse is 120 bpm, and the blood pressure is 88/52 mm Hg. The clients level of consciousness
is declining. The nurse should first:
You Selected:
call the health care provider (HCP).
Correct response:
call the rapid response team (RRT)/medical emergency team.
Explanation:
Question 6 See full question
The nurse manager has noticed a sharp increase in medication errors associated with IV antibiotic administration
over the past 2 months. The nurse manager should discuss the situation with each nurse involved and then:
You Selected:
ask them to attend in-service training for administration of IV medications.
Correct response:
ask them to attend in-service training for administration of IV medications.
Explanation:
Question 7 See full question
The nurse instructs the unlicensed assistive personnel (UAP) on how to care for a client with chest tubes that are
connected to water-seal drainage. The nurse should instruct the UAP to:
You Selected:
mark the time and amount of drainage on the collection container.
Correct response:
mark the time and amount of drainage on the collection container.
Explanation:
Question 8 See full question
While the nurse is caring for a primiparous client on the first postpartum day, the client asks, How is that woman
doing who lost her baby from prematurity? We were in labor together. Which response by the nurse would be most
appropriate?
You Selected:
Explain to the client that Nurses are not allowed to discuss other clients on the unit.
Correct response:
Explain to the client that Nurses are not allowed to discuss other clients on the unit.
Explanation:
Question 9 See full question
A nurse working in the emergency department receives an order from an orthopedic surgeon to obtain written
consent from a client for the surgical repair of a fractured forearm. The surgeon has not seen the client but has
reviewed the radiographs in the operating room between cases. Which of the following would be the most
appropriate response by the nurse to the surgeon?
You Selected:
Ill have the client sign, but you must explain the procedure before surgery.
Correct response:
It is your responsibility to obtain informed consent from the client.
Explanation:
Question 10 See full question
A client is discharged to a heart rehabilitation program. What lifestyle changes would be appropriate for the nurse
to review?
You Selected:
Ways to reduce the intake of unsaturated fats, regular participation in anaerobic burst training activity, and
increase in fluid intake
Correct response:
Reduced cholesterol levels, progressive activity levels, and coping strategies
Explanation:

Answer Key

Question 1 See full question


A client with terminal breast cancer is being cared for by a long-time friend who is a physician. The client has
identified her sister as the agent in her health care power of attorney. The client loses decision-making capacity,
and the sister tells the nurse, "A different physician will be caring for my sister now. I've dismissed her friend." In
response, the nurse should:

You Selected:
abide by the wishes of the sister who holds the durable power of attorney.

Correct response:
abide by the wishes of the sister who holds the durable power of attorney.

Explanation:

Question 2 See full question


A client is scheduled to undergo an exploratory laparoscopy. The registered nurse (RN) asks the licensed practical
nurse (LPN) to prepare the client for surgery. The RN must confirm that the LPN has specialized training before
delegating which task?

You Selected:
Initiating I.V. therapy, as ordered

Correct response:
Initiating I.V. therapy, as ordered
Explanation:

Question 3 See full question


The nurse is developing a community health education program about sexually transmitted infections. Which
information about women who acquire gonorrhea should be included?

You Selected:
Women with gonorrhea are usually asymptomatic.

Correct response:
Women with gonorrhea are usually asymptomatic.

Explanation:

Question 4 See full question


The nurse-manager on the oncology unit wants to address the issue of correct documentation of the effectiveness
of analgesia medication within 30 minutes after administration. What should the nurse-manager do first?

You Selected:
Complete a brief quality improvement study and chart audit to document the rate of adherence to the policy and
the pattern of documentation over shifts.

Correct response:
Complete a brief quality improvement study and chart audit to document the rate of adherence to the policy and
the pattern of documentation over shifts.

Explanation:

Question 5 See full question


After sustaining a stroke, a client is transferred to the rehabilitation unit. The medical-surgical nurse reviews the
client's residual neurological deficits with the rehabilitation nurse. Which neurological deficit places the client at the
greatest risk for skin breakdown?

You Selected:
Incontinence and right-sided hemiparesis

Correct response:
Incontinence and right-sided hemiparesis

Explanation:

Question 6 See full question


A child with meningococcal meningitis is being admitted to the pediatric unit. In preparation for the child's arrival,
the nurse should first?

You Selected:
institute droplet precautions.

Correct response:
institute droplet precautions.
Explanation:

Question 7 See full question


The nurse assigns an unlicensed assistive personnel (UAP) to the care of a client who has just returned from surgery
for repair of a fractured right wrist and application of an arm cast. The nurse should stress to the UAP the
importance of reporting:

You Selected:
the client cannot move the fingers on the right hand.

Correct response:
the client cannot move the fingers on the right hand.

Explanation:

Question 8 See full question


A client was brought to the hospital in an agitated state and admitted to a psychiatric unit for observation and
treatment. On admission, the client was found to be talking rapidly and folding and unfolding garments several
times while putting personal belongings away. The client is unable to settle down. Which nursing assessment would
have highest priority at this time?

You Selected:
Clients ability to care for self

Correct response:
Clients feelings of anxiety

Explanation:

Question 9 See full question


What would be important environmental assessments for the home care nurse to explore with a client who is being
discharged home?

You Selected:
Checking access to the home with a walker, access and safety measures in the bathroom, and access to food
preparation in the kitchen, and ensuring safety in the sleeping environment

Correct response:
Checking access to the home with a walker, access and safety measures in the bathroom, and access to food
preparation in the kitchen, and ensuring safety in the sleeping environment

Explanation:

Question 10 See full question


A client in a semiprivate room is diagnosed with pediculosis corpus. A nurse will initiate treatment after moving the
client to another room. The client's roommate asks the nurse for information about the client. What should the
nurse say?

You Selected:
"I'm sorry, but I can't share confidential information."

Correct response:
"I'm sorry, but I ca
Question 1 See full question
A physician orders ampicillin, 500 mg by mouth every 6 hours. This medication order is an example of:
You Selected:
a standing order.
Correct response:
a standing order.
Explanation:
Question 2 See full question
For a hospitalized client, the physician orders morphine, 4 mg I.V., every 2 hours as needed for pain. However, the
client refuses to take injections. Which nursing action is most appropriate?
You Selected:
Calling the physician to request an oral pain medication
Correct response:
Calling the physician to request an oral pain medication
Explanation:
Question 3 See full question
A physician orders naltrexone for a client participating in an outpatient drug and alcohol rehabilitation program.
Which action reflects the nurse's knowledge about this medication and the client's informed consent?
You Selected:
Discussing the health risks related to this medication
Correct response:
Discussing the health risks related to this medication
Explanation:
Question 4 See full question
In preparation for discharge, the nurse is reviewing information related to new dietary guidelines with the client.
This is an example of which step in discharge planning?
You Selected:
Providing client teaching.
Correct response:
Providing client teaching.
Explanation:
Question 5 See full question
Which of the following nursing actions would be most beneficial to a client and her husband who state they wish to
go through labor without the use of analgesics or anesthetic agents?
You Selected:
Provide information about the nature and availability of drugs for the client.
Correct response:
Act as an advocate for the couple and verbalize their wishes to nurses and physicians.
Explanation:
Improve your maste

Answer Key

Question 1 See full question


The following statement appears on a client's care plan: "Client will ambulate in the hall without assistance within 4
days." This statement is an example of:

You Selected:
a client outcome.

Correct response:
a client outcome.
Explanation:

Question 2 See full question


An adolescent admitted to the adolescent unit with pain caused by sickle cell crisis. Who should be consulted first
about this adolescent's care?

You Selected:
Pediatric pain specialist

Correct response:
Pediatric pain specialist

Explanation:

Question 3 See full question


The nurse is planning an education for new nurses on psychiatric units. Which topic should be given priority?

You Selected:
Breach of confidentiality

Correct response:
Breach of confidentiality

Explanation:

Question 4 See full question


The hospital is responding to a mass casualty disaster with adult and pediatric victims. After reallocating staff, the
charge nurse on the pediatric floor should:

You Selected:
review the census for clients that are candidates for early discharge.

Correct response:
review the census for clients that are candidates for early discharge.

Explanation:

Question 5 See full question


After receiving a report, the nurse is making out assignments. Which client would be appropriate to assign to
unlicensed assistive personnel?

You Selected:
an 8-month-old with pneumonia who will be discharged today

Correct response:
an 8-month-old with pneumonia who will be discharged today

Explanation:

Question 6 See full question


Which family should the nurse determine as most in need of follow-up?
You Selected:
a two-parent family with a foster child who has a history of caustic liquid ingestion

Correct response:
a single parent with a toddler who has third-degree burns over 20% of the body

Explanation:

Question 7 See full question


A client comes to the emergency department complaining of severe pain in the right flank, nausea, and vomiting.
The physician tentatively diagnoses right ureterolithiasis (renal calculi). When planning this client's care, the nurse
should assign the highest priority to which nursing diagnosis?

You Selected:
Acute pain

Correct response:
Acute pain

Explanation:

Question 8 See full question


A clients diagnosis of pneumonia requires treatment with antibiotics. The corresponding order in the clients chart
should be written as:

You Selected:
Moxifloxacin 400 mg daily.

Correct response:
Moxifloxacin 400 mg daily.

Explanation:

Question 9 See full question


A client with cancer-related pain has been prescribed a narcotic analgesic to be given around the clock. The client is
competent and has been actively involved in decisions regarding care. What should the nurse do if the client
refuses the next dose of analgesia?

You Selected:
Document the clients choice and re-assess pain in 1 hour.

Correct response:
Document the clients choice and re-assess pain in 1 hour.

Explanation:

Question 10 See full question


The nurse is documenting in the clients health record. Which information is most appropriate for the nurse to
record as objective data? Select all that apply.

You Selected:
Client ambulated to end of hallway.
Clients dressing is intact with scant amount of serous drainage.
Clients blood pressure is 120/80 mm Hg; pulse 76 bpm; respirations 14 breaths/min.

Correct response:
Clients blood pressure is 120/80 mm Hg; pulse 76 bpm; respirations 14 breaths/min.
Clients dressing is intact with scant amount of serous drainage.
Client ambulated to end of hallway.

Explanation:

Improve your mastery


TAKE A PRACTICE QUIZ

Answer Key

Question 1 See full question


As a nurse helps a client ambulate, the client says, "I had trouble sleeping last night." Which action should the
nurse take first?

You Selected:
Gathering more information about the client's sleep problem

Correct response:
Gathering more information about the client's sleep problem

Explanation:

Question 2 See full question


After instruction of a primigravid client at 8 weeks' gestation about measures to overcome early morning nausea
and vomiting, which client statement indicates the need for additional teaching?

You Selected:
"I will eat dry crackers or toast before arising in the morning."

Correct response:
"I will eat two large meals daily with frequent protein snacks."

Explanation:

Question 3 See full question


The nurse is preparing to administer a preoperative medication that includes a sedative to a client who is having
abdominal surgery. The nurse should first:

You Selected:
have the client empty the bladder.

Correct response:
have the client empty the bladder.
Explanation:

Question 4 See full question


A nurse is caring for an anorexic client with a nursing diagnosis of imbalanced nutrition: less than body
requirements related to dysfunctional eating patterns. Which interventions would be supportive for this client?
Select all that apply.

You Selected:
Allow the client to skip meals until the antidepressant levels are therapeutic.
Encourage the client to eat three substantial meals per day.
Provide small, frequent meals.

Correct response:
Provide small, frequent meals.
Monitor weight gain.
Encourage the client to keep a journal.
Monitor the client during meals and for 1 hour after meals.

Explanation:

Question 5 See full question


The client who is in Bucks traction is constipated. A plan of care that incorporates which breakfast would be most
helpful in reestablishing a normal bowel routine?

You Selected:
an orange, raisin bran and milk, and wheat toast with butter

Correct response:
an orange, raisin bran and milk, and wheat toast with butter

Explanation:

Question 6 See full question


A client receiving external radiation to the left thorax to treat lung cancer has a nursing diagnosis of Risk for
impaired skin integrity. Which intervention should be part of this client's care plan?

You Selected:
Applying talcum powder to the irradiated areas daily after bathing

Correct response:
Avoiding using deodorant soap on the irradiated areas

Explanation:

Question 7 See full question


An elderly client asks the nurse how to treat chronic constipation. What is the best recommendation the nurse can
make?

You Selected:
Take a stool softener such as docusate sodium daily.

Correct response:
Take a stool softener such as docusate sodium daily.
Explanation:

Question 8 See full question


While making a home visit to a multigravida 2 weeks after the birth of viable twins at 38 weeks gestation, the
nurse observes that the client looks pale, has dark circles around her eyes, and is breastfeeding one of the twins.
The clients apartment is clean, and nothing appears out of place. The client tells the nurse that she completed
three loads of laundry this morning. A priority need for this client is:

You Selected:
risk for imbalanced nutrition: Less than body requirements related to twin birth.

Correct response:
fatigue related to home maintenance and caring for twins.

Explanation:

Question 9 See full question


A nurse is caring for an infant who is to be administered an enema. What spiritually oriented interventions could the
nurse follow with newborns and infants?

You Selected:
Ask a child specialist to be present during treatment.

Correct response:
Encourage parents to be present during the treatment.

Explanation:

Question 10 See full question


Because of symptoms experienced after a cerebrovascular accident (CVA), the nurse discovers that a client needs
assistance using utensils while eating. What would the nurse do to support this activity of care?

You Selected:
Request that the clients food be pureed by dietary staff.

Correct response:
Encourage participation in the feeding process to the best of the client's abilities.

Explanation:
Question 1 See full question
A client with moderate Alzheimer's-related dementia is being prepared for discharge. What statement by the
caregiver demonstrates that discharge teaching about client safety has been effective?
You Selected:
"Someone should supervise him at all times."
Correct response:
"Someone should supervise him at all times."
Explanation:
Question 2 See full question
A client with stage II Alzheimer's disease is admitted to the short stay unit after cardiac catheterization that
involved a femoral puncture. The client is reminded to keep his leg straight. A knee immobilizer is applied, but the
client repeatedly attempts to remove it. The nurse is responsible for three other clients who underwent cardiac
catheterization. What's the best step the nurse can take?
You Selected:
Ask the staffing coordinator to assign a nursing assistant to sit with the client.
Correct response:
Ask the staffing coordinator to assign a nursing assistant to sit with the client.
Explanation:
Question 3 See full question
The nurse is caring for a neonate diagnosed with early onset sepsis and is being treated with intravenous
antibiotics. Which instructions will the nurse include in the parents teaching plan?
You Selected:
Wash hands thoroughly before touching the neonate.
Correct response:
Wash hands thoroughly before touching the neonate.
Explanation:
Question 4 See full question
When teaching parent workshops about measures to prevent lead poisoning in children, which preventive measure
should the nurse include as the most effective?
You Selected:
educating the public on common sources of lead
Correct response:
educating the public on common sources of lead
Explanation:
Question 5 See full question
Which client is at highest risk for developing a hospital-acquired infection?
You Selected:
A client with an indwelling urinary catheter
Correct response:
A client with an indwelling urinary catheter
Explanation:
Question 6 See full question
Safety concerns lead to the hospitalization of a client with a history of childhood sexual assault and dissociative
identity disorder. Which nursing interventions are most important? Select all that apply.
You Selected:
Support using a noteboook to continue communications with alters.
Allow time for processing feelings in a journal.
Provide anxiety management and rest.
Correct response:
Initiate precautions for suicide and self-mutilation.
Support using a noteboook to continue communications with alters.
Provide anxiety management and rest.
Allow time for processing feelings in a journal.
Explanation:
Question 7 See full question
A client has been placed in an isolation room, and family members have stated that access to the client seems
restricted. Which of the following actions would be appropriate for the nurse to take to address this situation? Select
all that apply.
You Selected:
A communication plan for the family and client
Acknowledgement of the family's concerns
A thorough explanation of the isolation procedures
Correct response:
A communication plan for the family and client
A thorough explanation of the isolation procedures
Acknowledgement of the family's concerns
Explanation:
Question 8 See full question
The nurse is caring for an immune compromised client with a fungal infection of the scalp. What recommendation
should the nurse make to prevent future problems?
You Selected:
Wash hair with a dandruff-preventing shampoo.
Correct response:
Avoid sharing combs and brushes.
Explanation:
Question 9 See full question
A nurse is caring for an infant who requires intravenous therapy. The nurse notes that the only available IV pump is
in a toddlers room. In which order should the nurse complete the following actions?

1. Remove pump from toddler's room


2. Clean the pump
3. Take pump into infants room
4. Use the pump
You Selected:
2, 1, 3, 4
Correct response:
1, 2, 3, 4
Explanation:
Question 10 See full question
A client is admitted to the emergency department with sneezing and coughing. The client is in the triage area,
waiting to be seen by a health care provider (HCP). To prevent spread of infection to others in the area and to the
health care staff, the nurse should:
You Selected:
give the client a surgical mask to wear.
Correct response:
give the client a surgical mask to wear.

Explanation: Question 1 See full question


A nurse notes that a client's I.V. insertion site is red, swollen, and warm to the touch. Which action should the nurse
take first?
You Selected:
Discontinue the I.V. infusion.
Correct response:
Discontinue the I.V. infusion.
Explanation:
Question 2 See full question
After having a total hip replacement, a client receives morphine sulfate by patient-controlled analgesia (PCA) pump.
The client says, "This pump doesn't help my pain at all." What should the nurse do in response to this statement?
You Selected:
Assess the client's understanding of the PCA pump.
Correct response:
Assess the client's understanding of the PCA pump.
Explanation:
Question 3 See full question
An unconscious client in the emergency department is given IV naloxone due to an overdose of heroin. Which
findings would indicate a therapeutic response to the naloxone? Select all that apply.
You Selected:
consciousness
increased respirations
Correct response:
increased respirations
consciousness
Explanation:
Question 4 See full question
A female client is treated for trichomoniasis with metronidazole. The nurse instructs the client that:
You Selected:
she should avoid alcohol during treatment and for 24 hours after completion of the drug.
Correct response:
she should avoid alcohol during treatment and for 24 hours after completion of the drug.
Explanation:
Question 5 See full question
An infant who weighs 7.5 kg is to receive ampicillin 25 mg/kg intrvenously every 6 hours. How many milligrams
would the nurse administer per dose? Record your answer using one decimal place.
Your Response:
187.5
Correct response:
187.5
Explanation:
Question 6 See full question
Which observation by the nurse indicates that the mother of a child receiving home IV nafcillin therapy requires
further teaching? The mother:
You Selected:
flushes the venous access site with heparin 20 minutes after giving the antibiotic.
Correct response:
flushes the venous access site with heparin 20 minutes after giving the antibiotic.
Explanation:
Question 7 See full question
What finding should the nurse interpret as indicating that a child is receiving too much IV fluid too rapidly?
You Selected:
moist crackles in the lung fields
Correct response:
moist crackles in the lung fields
Explanation:
Question 8 See full question
After 5 days of hospitalization, a client who is receiving morphine sulfate for pain control asks for pain medication
with increasing frequency and exhibits increased anxiety and restlessness. The vital signs are within normal ranges.
What is a possible cause of this behavior?
You Selected:
The client is addicted to the morphine.
Correct response:
The client has developed tolerance to the dose of morphine.
Explanation:
Question 9 See full question
The nurse has administered mannitol IV. Which is a priority assessment for the nurse to make after administering
this drug?
You Selected:
Monitor urine output.
Correct response:
Monitor urine output.
Explanation:
Question 10 See full question
An older adult who experienced a brief delirium realizes that the condition was caused by prescription medication
intoxication. Which of the following statements indicates the need for further education?
You Selected:
"I get medicines from three different doctors and they don't all know what I'm taking."
Correct response:
"I get medicines from three different doctors and they don't all know what I'm taking."
Explanation:
Question 1 See full question
A client's blood test results are: white blood cell (WBC) count is 1,000/l; hemoglobin (Hb) level, 14 g/dl; hematocrit
(HCT), 42%. Which goal is most important for this client?
You Selected:
Prevent infection
Correct response:
Prevent infection
Explanation:
Question 2 See full question
A client reports having pain in the casted left arm that is unrelieved by pain medication. The nurse assesses the
arm and notes that the fingers are swollen and difficult to separate. What should the nurse do first?
You Selected:
Call the health care provider (HCP) to report swelling and pain.
Correct response:
Call the health care provider (HCP) to report swelling and pain.
Explanation:
Question 3 See full question
A client with peripheral vascular disease has undergone a right femoral-popliteal bypass graft. The blood pressure
has decreased from 124/80 mm Hg to 88/62 mm Hg. What should the nurse assess first?
You Selected:
pedal pulses
Correct response:
pedal pulses
Explanation:
Question 4 See full question
A client has a herniated disk in the region of the third and fourth lumbar vertebrae. Which nursing assessment
finding most supports this diagnosis?
You Selected:
Severe lower back pain
Correct response:
Severe lower back pain
Explanation:
Question 5 See full question
During assessment of a small infant admitted with a diagnosis of meningitis, the infant becomes less responsive to
stimuli and exhibits bradycardia, slight hypertension, irregular respirations, and a temperature of 103.2 F (39.6 C).
The infants fontanel is more tense than at the last assessment. What should the nurse do first?
You Selected:
Administer an antipyretic.
Correct response:
Raise the head of the bed.
Explanation:
Question 6 See full question
An adolescent with type 1 diabetes is monitoring her blood glucose level at home. Which action indicates that the
client understands appropriate care management strategies for a blood glucose level of 250 mg/dL (13.9 mmol/L)?
The client will:
You Selected:
inject glucagon.
Correct response:
take insulin.
Explanation:
Question 7 See full question
A primigravid client at 41 weeks' gestation is admitted to the hospital's labor and birth unit in active labor. After 25
hours of labor with membranes ruptured for 24 hours, the client gives birth to a healthy neonate vaginally with a
midline episiotomy. Which problem should the nurse identify as the priority for the client?
You Selected:
sleep deprivation
Correct response:
risk for infection
Explanation:
Question 8 See full question
A 68-year-old client on day 2 after hip surgery has no cardiac history but reports having chest heaviness. The nurse
should first:
You Selected:
administer oxygen via nasal cannula.
Correct response:
inquire about the onset, duration, severity, and precipitating factors of the heaviness.
Explanation:
Question 9 See full question
A client is admitted to the hospital through the emergency department with chest pain. Which intervention is the
priority?
You Selected:
Assessing B-type natriuretic peptide levels
Correct response:
Assessing troponin 1 levels
Explanation:
Question 10 See full question
A nurse places electrodes on a collapsed individual who was visiting a hospitalized family member, the monitor
exhibits the following. Which interventions would the nurse do first?
You Selected:
Confirm the rhythm with a 12-lead ECG.
Correct response:
Assess the clients airway, breathing, and circulation.
Explanation:
Question 1 See full question
A 4-year-old boy is scheduled for a nephrectomy to remove a Wilms' tumor. Which intervention listed in the care
plan should the nurse question?
You Selected:
Palpate his abdomen to monitor tumor growth.
Correct response:
Palpate his abdomen to monitor tumor growth.
Explanation:
Question 2 See full question
A nurse needs to obtain a good monitor tracing on a client in labor The client lies in a supine position. Suddenly, she
complains of feeling light-headed and becomes diaphoretic. Which action should the nurse perform first?
You Selected:
Reposition the client to her left side.
Correct response:
Reposition the client to her left side.
Explanation:
Question 3 See full question
A client with stage II ovarian cancer undergoes a total abdominal hysterectomy and bilateral salpingo-
oophorectomy with tumor resection, omentectomy, appendectomy, and lymphadenectomy. During the second
postoperative day, which assessment finding requires immediate intervention?
You Selected:
Shallow breathing and increasing lethargy
Correct response:
Shallow breathing and increasing lethargy
Explanation:
Question 4 See full question
A 10-year-old child has blood glucose readings during a 24-hour period. Which reading requires the most immediate
intervention?
You Selected:
50 mg/dL (2.8 mmol/L)
Correct response:
50 mg/dL (2.8 mmol/L)
Explanation:
Question 5 See full question
A young child who has undergone a tonsillectomy refuses to let the nurse look at the tonsillar beds to check for
bleeding. To assess whether the child is bleeding from the tonsillar beds, which measure would be most
appropriate?
You Selected:
Observe for frequent swallowing.
Correct response:
Observe for frequent swallowing.
Explanation:
Question 6 See full question
A client undergoes a total hip replacement. Which statement made by the client indicates to the nurse that the
client requires further teaching?
You Selected:
"I don't know if I'll be able to get off that low toilet seat at home by myself."
Correct response:
"I don't know if I'll be able to get off that low toilet seat at home by myself."
Explanation:
Question 7 See full question
Twelve hours after a vaginal birth with epidural anesthesia, the nurse palpates the fundus of a primiparous client
and finds it to be firm, above the umbilicus, and deviated to the right. What should the nurse do next?
You Selected:
Encourage the client to ambulate to the bathroom and void.
Correct response:
Encourage the client to ambulate to the bathroom and void.
Explanation:
Question 8 See full question
The nurse is evaluating the clients risk for having a pressure sore. Which is the best indicator of risk for the clients
developing a pressure sore?
You Selected:
mobility status
Correct response:
mobility status
Explanation:
Question 9 See full question
A 12-year-old child is scheduled for surgery to repair a fractured tibia. One hour prior to surgery, the nurse assesses
that the child is febrile. What is the best action for the nurse to take?
You Selected:
Inform the surgeon.
Correct response:
Inform the surgeon.
Explanation:
Question 10 See full question
A client in the first stage of labor is being monitored using an external fetal monitor. After the nurse reviews the
monitoring strip from the clients chart (shown above), into which of the following positions would the nurse assist
the client?
You Selected:
Left lateral
Correct response:
Left lateral
Explanation:
Question 1 See full question
A client is receiving paroxetine 20 mg every morning. After taking the first three doses, the client tells the nurse
that the medication upsets his stomach. What instruction should the nurse give to the client?
You Selected:
"Take the medication with some food."
Correct response:
"Take the medication with some food."
Explanation:
Question 2 See full question
A client diagnosed with schizophrenia is being switched to risperidone long-acting injection. He is told that he will
remain on his oral dose of risperidone daily for approximately 1 month. The client says, "I did not have to take pills
when I was on fluphenazine shots in the past." The nurse should tell the client:
You Selected:
"Risperidone long-acting injection initially takes a little longer to reach the ideal blood level."
Correct response:
"Risperidone long-acting injection initially takes a little longer to reach the ideal blood level."
Explanation:
Question 3 See full question
A client receiving a blood transfusion begins to have chills and headache within the first 15 minutes of the
transfusion. The nurse should first:
You Selected:
discontinue the transfusion.
Correct response:
discontinue the transfusion.
Explanation:
Question 4 See full question
Griseofulvin was ordered to treat a child's ringworm of the scalp. The nurse instructs the parents to use the
medication for several weeks for which reason?
You Selected:
The growth of the causative organism into new cells is prevented with long-term use.
Correct response:
The growth of the causative organism into new cells is prevented with long-term use.
Explanation:
Question 5 See full question
A client will receive IV midazolam hydrochloride during surgery. Which finding indicates a therapeutic effect?
You Selected:
amnesia
Correct response:
amnesia
Explanation:
Question 6 See full question
Vasopressin is administered to the client with diabetes insipidus because it:
You Selected:
increases tubular reabsorption of water.
Correct response:
increases tubular reabsorption of water.
Explanation:
Question 7 See full question
A client is receiving total parenteral nutrition (TPN) solution. The nurse should assess a clients ability to metabolize
the TPN solution adequately by monitoring the client for which sign?
You Selected:
hyperglycemia
Correct response:
hyperglycemia
Explanation:
Question 8 See full question
During preparation for bowel surgery, a client receives an antibiotic to reduce intestinal bacteria. The nurse knows
that hypoprothrombinemia may occur as a result of antibiotic therapy. Which of the following vitamins would be
affected by this?
You Selected:
Vitamin K
Correct response:
Vitamin K
Explanation:
Question 9 See full question
A client who weighs 207 lb (94.1 kg) is to receive 1.5 mg/kg of gentamicin sulfate IV three times each day. How
many milligrams of medication should the nurse administer for each dose? Round to the nearest whole number.
Your Response:
19478.7
Correct response:
141
Explanation:
Question 10 See full question
A pediatric client is to receive an oral antibiotic dose. The clients meal tray is due to be delivered in 30 minutes.
Which of the following shows how the nurse should coordinate the antibiotic dose with the clients meal?
You Selected:
Wait to give the medication when the food arrives
Correct response:
Give the dose now before the tray arrives
Explanation:
Question 1 See full question
An adolescent is diagnosed with iron deficiency anemia. After emphasizing the importance of consuming dietary
iron, the nurse asks him to select iron-rich breakfast items from a sample menu. Which selection demonstrates
knowledge of dietary iron sources?
You Selected:
Ham and eggs
Correct response:
Ham and eggs
Explanation:
Question 2 See full question
For a client with anorexia nervosa, which goal takes the highest priority?
You Selected:
The client will establish adequate daily nutritional intake.
Correct response:
The client will establish adequate daily nutritional intake.
Explanation:
Question 3 See full question
The client has sore nares while a nasogastric (NG) tube is in place. Which nursing measure would be most
appropriate to help alleviate the client's discomfort?
You Selected:
Irrigate the tube with a cool solution.
Correct response:
Apply a water-soluble lubricant to the nares.
Explanation:
Question 4 See full question
To encourage adequate nutritional intake for a client with Alzheimer's disease, a nurse should:
You Selected:
stay with the client and encourage him to eat.
Correct response:
stay with the client and encourage him to eat.
Explanation:
Question 5 See full question
While making a home visit to a multigravida 2 weeks after the birth of viable twins at 38 weeks gestation, the
nurse observes that the client looks pale, has dark circles around her eyes, and is breastfeeding one of the twins.
The clients apartment is clean, and nothing appears out of place. The client tells the nurse that she completed
three loads of laundry this morning. A priority need for this client is:
You Selected:
possible anemia related to large volume of blood loss and twin birth.
Correct response:
fatigue related to home maintenance and caring for twins.
Explanation:
Question 6 See full question
A client with diabetes is explaining to the nurse how he cares for the feet at home. Which statement indicates the
client needs further instruction on how to care for the feet properly?
You Selected:
I inspect my feet once a week for cuts and redness.
Correct response:
I inspect my feet once a week for cuts and redness.
Explanation:
Question 7 See full question
The nurse is caring for a 4-year-old child who is experiencing pain. When evaluating the childs response to pain,
which of the following factors are most important for the nurse to assess? Select all that apply.
You Selected:
Past experiences
Parental presence
Communication skills
Correct response:
Parental presence
Personality traits
Past experiences
Communication skills
Explanation:
Question 8 See full question
The nurse will be assisting with the cast removal from the leg of a 5-year-old child. Which of the following is most
important for the nurse to warn the parents about before the procedure?
You Selected:
The child may be afraid of the cast cutter.
Correct response:
The child may be afraid of the cast cutter.
Explanation:
Question 9 See full question
Which of the following is a common method of evaluating the urine output for newborns, infants, and toddlers who
are not potty trained.
You Selected:
Weighing the diaper before and after micturition
Correct response:
Weighing the diaper before and after micturition
Explanation:
Question 10 See full question
A child is receiving amoxicillin for otitis media. Which action should the nurse recommend the mother do when the
child develops diarrhea?
You Selected:
Offer yogurt several times a day.
Correct response:
Offer yogurt several times a day.
Explanation:
Question 1 See full question
A nurse is working in a clinic where a family member's spouse is treated for a sexually transmitted disease. The
nurse is concerned about the risk to her family member. What is the most appropriate action for the nurse to take?
You Selected:
Encourage the client to speak with the family member about the diagnosis if he or she has not already done so.
Correct response:
Encourage the client to speak with the family member about the diagnosis if he or she has not already done so.
Explanation:
Question 2 See full question
A 14-year-old adolescent tells the nurse that she's in love with her 22-year-old neighbor and that they've had sex on
several occasions. She doesn't want her parents to know because she loves him and is afraid they'll be angry. What
is the nurse's best course of action?
You Selected:
Tell the adolescent that she won't say anything to her parents, but that she must tell the physician.
Correct response:
Tell the adolescent that the law requires her to report the sexual contact because of the age difference.
Explanation:
Question 3 See full question
Which document states that clients have a right to confidentiality of medical records and a right to privacy limiting
access to that information to healthcare providers associated with their care?
You Selected:
Medical privacy acts
Correct response:
Medical privacy acts
Explanation:
Question 4 See full question
A client has a reddened area over a bony prominence. The nurse finds an unlicensed nursing personnel (UAP)
massaging this area. The nurse should:
You Selected:
explain to the UAP that massage is effective because it improves blood flow to the area.
Correct response:
instruct the UAP that massage is contraindicated because it decreases blood flow to the area.
Explanation:
Question 5 See full question
The nurse assigns an unlicensed assistive personnel (UAP) to provide care for a client with peptic ulcer disease.
Concerned about possible ulcer perforation, the nurse should instruct the UAP to report to the nurse immediately if
the client has:
You Selected:
severe abdominal pain.
Correct response:
severe abdominal pain.
Explanation:
Question 6 See full question
A neonate is experiencing respiratory distress and is using a neonatal oxygen mask. An unlicensed assistive
personnel has positioned the oxygen mask as shown. The nurse is assessing the neonate and determines that the
mask:
You Selected:
is too small because it obstructs the nose.
Correct response:
is appropriate for the neonate.
Explanation:
Question 7 See full question
The client with acute mania has been admitted to the inpatient unit voluntarily. The nurse approaches the client
with medication to be taken orally as prescribed by the health care provider (HCP). The client states, "I do not need
that stuff." Which response by the nurse is best?
You Selected:
"I will get you some written information about the medication."
Correct response:
"The medication will help you feel calmer."
Explanation:
Question 8 See full question
During a home visit to an elderly client with mild dementia, the client's daughter reports that she has one major
problem with her mother. She says, "She sleeps most of the day and is up most of the night. I cannot get a decent
night's sleep anymore." Which suggestions should the nurse make to the daughter? Select all that apply.
You Selected:
Engage the client in simple, brief exercises or a short walk when she gets drowsy during the day.
Promote relaxation before bedtime with a warm bath or relaxing music.
Establish a set routine for rising, hygiene, meals, short rest periods, and bedtime.
Correct response:
Establish a set routine for rising, hygiene, meals, short rest periods, and bedtime.
Engage the client in simple, brief exercises or a short walk when she gets drowsy during the day.
Promote relaxation before bedtime with a warm bath or relaxing music.
Explanation:
Question 9 See full question
The nurse has received a change-of-shift report. The nurse should assess which client first?
You Selected:
a 36-year-old with chest tube due to spontaneous pneumothorax with current respiratory rate 18 breaths/min,
oxygen saturation 95% on oxygen at 2 L per nasal cannula
Correct response:
a 72-year-old admitted 2 days ago with a blood alcohol level of 0.08
Explanation:
Question 10 See full question
When assessing if a procedural risk to a client is justified, the ethical principle underlying the dilemma is known as
which of the following?
You Selected:
Nonmaleficence
Correct response:
Nonmaleficence
Explanation:

vQuestion 1 See full question


Which item in the care plan for a toddler with a seizure disorder should a nurse revise?
You Selected:
Arm restraints while asleep
Correct response:
Arm restraints while asleep
Explanation:
Question 2 See full question
A child, age 3, who tests positive for the human immunodeficiency virus (HIV) is placed in foster care. The foster
parents ask the nurse how to prevent HIV transmission to other family members. How should the nurse respond?
You Selected:
"Don't let the child share toys with other children."
Correct response:
"Wear gloves when you're likely to come into contact with the child's blood or body fluids."
Explanation:
Question 3 See full question
A nurse is assigned to care for a recently admitted client who has attempted suicide. What should the nurse do?
You Selected:
Search the client's belongings and his room carefully for items that could be used to attempt suicide.
Correct response:
Search the client's belongings and his room carefully for items that could be used to attempt suicide.
Explanation:
Question 4 See full question
When planning home care for a 3-year-old child with eczema, what should the nurse teach the mother to remove
from the child's environment at home?
You Selected:
metal toy trucks
Correct response:
stuffed animals
Explanation:
Question 5 See full question
While making rounds, the nurse enters a clients room and finds the client on the floor between the bed and the
bathroom. The nurse should first:
You Selected:
assess the clients current condition and vital signs.
Correct response:
assess the clients current condition and vital signs.
Explanation:
Question 6 See full question
A 26-year-old is being treated for delirium due to acute alcohol intoxication. The client is restless, does not want to
stay seated, and has a staggering gait. What should the nurse do first?
You Selected:
Provide one-to-one supervision of the client until detoxification treatment can begin.
Correct response:
Provide one-to-one supervision of the client until detoxification treatment can begin.
Explanation:
Question 7 See full question
A nurse assesses a client with psychotic symptoms and determines that the client likely poses a safety threat and
needs vest restraints. The client is adamantly opposed to this. What would be the best nursing action?
You Selected:
Restrain the client with vest restraints.
Correct response:
Contact the physician and obtain necessary orders.
Explanation:
Question 8 See full question
A school nurse is conducting a seminar for parents of preschool children on the prevention of head injuries. What is
the most appropriate information for the nurse to give the parents?
You Selected:
Children should always be supervised by an adult when playing.
Correct response:
Children should always wear helmets when riding bicycles.
Explanation:
Question 9 See full question
Using the Morse Fall Risk scale (see exhibit), the nurse should initiate highest fall risk precautions for which client?
You Selected:
a 27-year-old client with acute pancreatitis receiving morphine sulfate IV every 2 hours as needed for pain; no
significant medical history, smokes two packs of cigarettes per day; may be up independently; and has steady gait
Correct response:
a 62-year-old client with a history of Parkinsons disease, admitted for pneumonia and receiving IV antibiotics, who
has fallen at home but is able to ambulate with a cane and who during his hospitalization has gotten out of bed
without calling for assistance
Explanation:
Question 10 See full question
Which nursing action best addresses the outcome: The client will be free from falls?
You Selected:
Place emergency contacts telephone number in a prominent place.
Correct response:
Encourage use of

Answer Key

Question 1 See full question


Which guidelines define and regulate what the nurse may and may not do as a professional?

You Selected:
Nurse practice act

Correct response:
Nurse practice act

Explanation:

Question 2 See full question


Which measure included in the care plan for a client in the fourth stage of labor requires revision?

You Selected:
Have the client spend time with the neonate to initiate breast-feeding.

Correct response:
Obtain an order for catheterization to protect the bladder from trauma.

Explanation:

Question 3 See full question


A client with a history of alcohol abuse was admitted with bleeding esophageal varices. After several days of
treatment, the client is ready for discharge. The nurse enters the client's room to review discharge instructions with
the client when he tells the nurse that he wants help to quit drinking. How should the nurse respond?

You Selected:
"I'll notify your physician and call the social worker so she can discuss treatment options with you."

Correct response:
"I'll notify your physician and call the social worker so she can discuss treatment options with you."

Explanation:

Question 4 See full question


The nurse uses which part of the SBAR acronym when stating, I think the client is dry.

You Selected:
Assessment.

Correct response:
Assessment.
Explanation:

Question 5 See full question


The nurse and an unlicensed assistive personnel (UAP) are caring for clients in a birthing center. Which tasks should
the nurse delegate to the UAP? Select all that apply.

You Selected:
assisting an active labor client with breathing and relaxation
ambulating a postcesarean client to the bathroom

Correct response:
removing a Foley catheter from a preeclamptic client
assisting an active labor client with breathing and relaxation
ambulating a postcesarean client to the bathroom

Explanation:

Question 6 See full question


A 16-year-old primiparous client has decided to place her baby for adoption. The adoptive parents are on their way
to the hospital when the mother says, I want to see the baby one last time. The nurse should:

You Selected:
allow the client to see and hold the baby for as long as she desires.

Correct response:
allow the client to see and hold the baby for as long as she desires.

Explanation:

Question 7 See full question


Using the nursing process to make ethical decisions involves following several steps. Which step is the nurse
implementing when he or she reflects on the decision-making process and the role it will play in making future
decisions?

You Selected:
Evaluating.

Correct response:
Evaluating.

Explanation:

Question 8 See full question


When assessing if a procedural risk to a client is justified, the ethical principle underlying the dilemma is known as
which of the following?

You Selected:
Nonmaleficence

Correct response:
Nonmaleficence
Explanation:

Question 9 See full question


A nurse just received a shift report for a group of clients on the telemetry unit. Which client should the nurse assess
first?

You Selected:
The client with a history of heart failure who has bibasilar crackles and pitting edema in both feet

Correct response:
The client admitted with first-degree atrioventricular (AV) block whose cardiac monitor now reveals type II second-
degree AV block

Explanation:

Question 10 See full question


A nurse is caring for a 9-year-old child who is scheduled for surgery. The parents ask the nurse not to tell the child
about the surgery until leaving for the operating room. Which response best demonstrates the nurse's role in
supporting the child's rights?

You Selected:
I am legally obligated to inform your child of the surgery.

Correct response:
It is important to tell your child about the surgery in order to allow time for any questions to be answered.

Explanation:
Question 1 See full question
What information must a medication order include?
You Selected:
Physician's signature
Correct response:
Physician's signature
Explanation:
Question 2 See full question
A nurse is explaining medication benefits and adverse effects to a client with a history of psychosis. The client's
brother tells the nurse that she's wasting her time explaining things to the client. What information about informed
consent should the nurse use to respond to the brother's negative statement?
You Selected:
A third party must be present when a nurse informs clients about treatment options.
Correct response:
Informed consent is an important part of effective client care that helps accomplish treatment goals.
Explanation:
Question 3 See full question
A nurse is caring for a client with schizophrenia. Which outcome requires revising the client's care plan?
You Selected:
The client demonstrates the ability to meet his own self-care needs.
Correct response:
The client spends more time by himself.
Explanation:
Question 4 See full question
After the nurse informs the surgeon that a chest tube is malfunctioning, the health care provider asks the nurse to
reposition the tube and obtain a chest radiograph. The nurse should:
You Selected:
inform the surgeon this is not within the safe scope of practice.
Correct response:
inform the surgeon this is not within the safe scope of practice.
Explanation:
Question 5 See full question
The son of a dying patient is surprised at his mothers adamant request to meet with the hospital chaplain and has
taken the nurse aside and said, I dont think thats what she really wants. Shes never been a religious person in
the least. What is the nurses best action in this situation?
You Selected:
Contact the chaplain to arrange a visit with the patient.
Correct response:
Contact the chaplain to arrange a visit with the patient.
Explanation:
Question 6 See full question
The nurse is preparing for the admission of a client on a stretcher. In what position should the nurse place the bed?
You Selected:
Middle position.
Correct response:
Highest position.
Explanation:
Question 7 See full question
The nurse is beginning the shift and is planning care for 6 clients on the postpartum unit. Three of the clients have
immediate needs, and three of the clients are listed as stable. For the best utilization of time and client safety,
the nurse should make rounds on which client first?
You Selected:
a mother who had a spontaneous vaginal birth (SVB) and received carboprost 1 hour ago for increased bleeding
Correct response:
a mother who had a spontaneous vaginal birth (SVB) and received carboprost 1 hour ago for increased bleeding
Explanation:
Question 8 See full question
A pregnant client is seeking information from the nurse about a home birth with registered midwives. Which of the
following statements lets the nurse know that the client has considered the risks and benefits of using a midwife?
Select all that apply.
You Selected:
I understand the complications that could occur in a home birth setting.
I realize that I may need to be transferred to a hospital if complications develop.
I will develop a list of questions to use in interviewing potential midwives.
Correct response:
I will develop a list of questions to use in interviewing potential midwives.
I understand the complications that could occur in a home birth setting.
I realize that I may need to be transferred to a hospital if complications develop.
Explanation:
Question 9 See full question
A 16-year-old client is admitted to the emergency department following an accident. The client sustained a head
injury, is unconscious, and has compound fractures of the right tibia and fibula. No family members accompanied
the client and none can be reached by phone. The surgeon instructs the nurse to take the client to the operating
room immediately. Which of the following actions should the nurse take regarding informed consent?
You Selected:
Take the client to the operating room for surgery without informed consent.
Correct response:
Take the client to the operating room for surgery without informed consent.
Explanation:
Question 10 See full question
When the nurse enters a clients room to begin the initial assessment, the client is passionately kissing and
embracing his/her partner. What action should the nurse take?
You Selected:
Announce your presence by excusing yourself, and proceed to conduct the health history.
Correct response:
Do not disturb the couple, leave the room, and allow them privacy.
Explanation:
Question 1 See full question
The nurse-manager of a home health facility includes which item in the capital budget?
You Selected:
A $1,200 computer upgrade
Correct response:
A $1,200 computer upgrade
Explanation:
Question 2 See full question
Which concept is most important for a nurse to communicate to a client preparing to sign an informed consent for
electroconvulsive therapy (ECT)?
You Selected:
"You may experience a time of confusion after the treatment."
Correct response:
"You may experience a time of confusion after the treatment."
Explanation:
Question 3 See full question
The client has returned to the surgery unit from the postanesthesia care unit (PACU). The clients respirations are
rapid and shallow, the pulse is 120 bpm, and the blood pressure is 88/52 mm Hg. The clients level of consciousness
is declining. The nurse should first:
You Selected:
call the rapid response team (RRT)/medical emergency team.
Correct response:
call the rapid response team (RRT)/medical emergency team.
Explanation:
Question 4 See full question
A client has undergone a laparoscopic cholecystectomy. Which instruction should the nurse include in the discharge
teaching?
You Selected:
Report bile-colored drainage from any incision.
Correct response:
Report bile-colored drainage from any incision.
Explanation:
Question 5 See full question
A client is evaluated for severe pain in the right upper abdominal quadrant, which is accompanied by nausea and
vomiting. The physician diagnoses acute cholecystitis and cholelithiasis. For this client, which nursing diagnosis
takes top priority?
You Selected:
Imbalanced nutrition: Less than body requirements related to biliary inflammation
Correct response:
Acute pain related to biliary spasms
Explanation:
Question 6 See full question
A primigravid client gave birth vaginally 2 hours ago with no complications. As the nurse plans care for this
postpartum client, which postpartum goal would have the highest priority?
You Selected:
By the end of the shift, the client will describe a safe home environment.
Correct response:
The client will demonstrate self-care and infant care by the end of the shift.
Explanation:
Question 7 See full question
The nurse's unit council in the telemetry unit is responsible for performance improvement studies. What information
should they gather to study whether client education about resuming sexual activity after an acute myocardial
infarction (MI) is being taught?
You Selected:
The percentage of clients on the unit diagnosed with an acute MI who were taught about resuming sexual activity
Correct response:
The percentage of clients on the unit diagnosed with an acute MI who were taught about resuming sexual activity
Explanation:
Question 8 See full question
A nurse is serving on a task force to update the electronic health record. The task force should ensure that revisions
of the medical record will: (Select all that apply.)
You Selected:
have sufficient room for charting nurses' notes.
serve as a legal document.
be written so the client can understand what is written.
guide performance improvement.
Correct response:
aid in client care.
serve as a legal document.
facilitate data collection for clinical research.
guide performance improvement.
Explanation:
Question 9 See full question
A nurse on the pediatric unit is caring for a group of preschool children. Which situation takes highest priority?
You Selected:
A child admitted from the postanesthesia care unit who has a blood-saturated surgical dressing
Correct response:
A child who develops a fever during a blood transfusion
Explanation:
Question 10 See full question
A staff nurse on a busy pediatric unit would like to function effectively in the role of a leader. Which of the following
actions would the nurse employ to be a leader?
You Selected:
Encourage the staff to participate in the unit's decision-making process, and help the staff to improve their clinical
skills.
Correct response:
Encourage the staff to participate in the unit's decision-making process, and help the staff to improve their clinical
skills.
Explanation:
Question 1 See full question
A nurse-manager works for a nonprofit health care corporation whose revenues have significantly exceeded annual
expenses. The nurse-manager has been told to anticipate which action?
You Selected:
Receiving a portion of the revenue to improve client services on the unit
Correct response:
Receiving a portion of the revenue to improve client services on the unit
Explanation:
Question 2 See full question
During the planning step of the nursing process, the nurse:
You Selected:
establishes short- and long-term goals.
Correct response:
establishes short- and long-term goals.
Explanation:
Question 3 See full question
A nurse is prioritizing care for four new admissions to the inpatient psychiatric unit. Which client should the nurse
assess first?
You Selected:
A client with new-onset confusion and disorientation.
Correct response:
A client with new-onset confusion and disorientation.
Explanation:
Question 4 See full question
Accompanied by her partner, a client seeks admission to the labor and delivery area. She states that she's in labor
and says she attended the facility clinic for prenatal care. Which question should the nurse ask her first?
You Selected:
"What is your expected due date?"
Correct response:
"What is your expected due date?"
Explanation:
Question 5 See full question
A nurse caring for a client who had a stroke is using the unit's new computerized documentation system. The nurse
uses the information technology appropriately when she:
You Selected:
documents medications before administration.
Correct response:
documents medications after administration.
Explanation:
Question 6 See full question
A nurse on the gynecologic surgery unit observes a respiratory therapist (RT) take a medication cup with pills that
was sitting in the medication room. What course of action should the nurse take?
You Selected:
Report the situation to the nursing supervisor.
Correct response:
Report the situation to the nursing supervisor.
Explanation:
Question 7 See full question
The nurse is teaching a group of unlicensed personnel new to psychiatry about balance in a therapeutic milieu.
Which statement by a member of the group indicates the need for further teaching?
You Selected:
Controlling clients helps them feel more comfortable.
Correct response:
Controlling clients helps them feel more comfortable.
Explanation:
Question 8 See full question
The nurse notes that a placebo has been prescribed when a client requests pain medication. Which statement is
most accurate about the use of placebos in the clients plan of care?
You Selected:
The use of placebos violates the clients right to ethical care.
Correct response:
The use of placebos violates the clients right to ethical care.
Explanation:
Question 9 See full question
The nurse is caring for a child whose mother is deaf and untrusting of staff. She frequently cries at the bedside, but
refuses intervention from social work or the chaplain. Which issue is most important for the nurse to address with
the mother to promote a trusting relationship?
You Selected:
Communication barriers between the mother and staff
Correct response:
Communication barriers between the mother and staff
Explanation:
Question 10 See full question
A client is about to undergo cardiac catheterization for which he signed an informed consent. As the nurse enters
the room to administer sedation for the procedure, the client states, "I'm really worried about having this open
heart surgery." Based on this statement, how should the nurse proceed?
You Selected:
Withhold the medication and notify the physician immediately.
Correct response:
Withhold the medication and notify the physician immediately.

Explanation: Question 1 See full question


A client states that her "water broke." Which action requires the nurse to have specialialized training?
You Selected:
Conducting a bedside ultrasound for an amniotic fluid index
Correct response:
Conducting a bedside ultrasound for an amniotic fluid index
Explanation:
Question 2 See full question
A client with a history of alcohol abuse was admitted with bleeding esophageal varices. After several days of
treatment, the client is ready for discharge. The nurse enters the client's room to review discharge instructions with
the client when he tells the nurse that he wants help to quit drinking. How should the nurse respond?
You Selected:
"I'll notify your physician and call the social worker so she can discuss treatment options with you."
Correct response:
"I'll notify your physician and call the social worker so she can discuss treatment options with you."
Explanation:
Question 3 See full question
A client arrives at the emergency department after falling in the home. The nurse performing the assessment notes
the presence of pediculosis corpus. The client's skin and clothing are dirty. The client reports that his children work
and no one has time to assist him with his self-care activities. The nurse should:
You Selected:
Contact the nursing supervisor.
Correct response:
Contact the nursing supervisor.
Explanation:
Question 4 See full question
A client in a late stage of acquired immunodeficiency syndrome (AIDS) shows signs of AIDS-related dementia. Which
nursing diagnosis takes highest priority?
You Selected:
Risk for injury
Correct response:
Risk for injury
Explanation:
Question 5 See full question
During discharge teaching, a nurse is instructing a client about pneumonia. The client demonstrates his
understanding of relapse when he states that he must:
You Selected:
continue to take antibiotics for the entire 10 days.
Correct response:
continue to take antibiotics for the entire 10 days.
Explanation:
Question 6 See full question
A bedridden client is scheduled to receive subcutaneous injections of heparin at 8:00 a.m. and 8:00 p.m. each day.
The clients medication administration record would present these times as:
You Selected:
0800 and 2000
Correct response:
0800 and 2000
Explanation:
Question 7 See full question
A resident in a long-term care facility is scheduled to have a bath and is taken to the tub room by the nurse. The
resident does not want to be bathed and refuses to get undressed. Which of the following actions should the nurse
take?
You Selected:
Return the client to the room and document that the bath was not given.
Correct response:
Return the client to the room and document that the bath was not given.
Explanation:
Question 8 See full question
An anxious client asks the nurse for the results of recent blood work and wants to know what the results mean.
Which of the following responses by the nurse is the most appropriate?
You Selected:
I understand your concern. Ill call the physician to review the results with you.
Correct response:
I understand your concern. Ill call the physician to review the results with you.
Explanation:
Question 9 See full question
A nurse arriving for duty notes that a nursing assistant (or unregulated care provider [UCP]) has been assigned to a
complex client with treatments involving sterile technique. What is the responsibility of the nurse regarding the
assignment of the UCP?
You Selected:
Reassign the UCP to a client requiring basic tasks that the UCP has mastered.
Correct response:
Reassign the UCP to a client requiring basic tasks that the UCP has mastered.
Explanation:
Question 10 See full question
A nurse is caring for a client with type 2 diabetes who has had a myocardial infarction (MI) and is reporting nausea,
vomiting, dyspnea, and substernal chest pain. Which of the following is the priority intervention?
You Selected:
Control the pain and support breathing and oxygenation.
Correct response:
Control the pain and support breathing and oxygenation.
Explanation:
uestion 1 See full question
A client with end-stage pancreatic cancer has decided to terminate medical intervention. What should a nurse
anticipate when consulting with palliative care?
You Selected:
Referral for bereavement counseling
Correct response:
Referral for bereavement counseling
Explanation:
Question 2 See full question
A nurse is caring for an older adult client with advanced Parkinson's disease. Which client statement about advance
directives indicates a need for further instruction?
You Selected:
"I don't really need to sign anything. I'm depending on my physician to tell my family what to do if something bad
happens."
Correct response:
"I don't really need to sign anything. I'm depending on my physician to tell my family what to do if something bad
happens."
Explanation:
Question 3 See full question
A client with agnosia as a result of vascular dementia is staring at dinner and utensils without trying to eat. Which
intervention should the nurse attempt first?
You Selected:
Hand the fork to the client and say, "Use this fork to eat your green beans."
Correct response:
Hand the fork to the client and say, "Use this fork to eat your green beans."
Explanation:
Question 4 See full question
A client's left leg is in skeletal traction with a Thomas leg splint and Pearson attachment. Which intervention should
the nurse include in this client's care plan?
You Selected:
Teach the client how to prevent problems caused by immobility.
Correct response:
Teach the client how to prevent problems caused by immobility.
Explanation:
Question 5 See full question
A community nurse arrives at the home of a client. The client is in soiled clothes due to the inability to make it to
the bathroom in time. The nurse overhears the unregulated care provider (UCP) scolding the client for the soiled
clothes. What is the most appropriate response by the nurse to the UCP?
You Selected:
Your behavior in this situation is considered verbal abuse.
Correct response:
Your behavior in this situation is considered verbal abuse.
Explanation:
Answer Key

Question 1 See full question


The following statement appears on a client's care plan: "Client will ambulate in the hall without assistance within 4
days." This statement is an example of:

You Selected:
a client outcome.

Correct response:
a client outcome.

Explanation:

Question 2 See full question


The nurse is assisting with a bone marrow aspiration and biopsy. Place the tasks in the order in which the nurse
should perform them, from highest priority to least priority. All options must be used.

You Selected:
Verify the client has signed an informed consent.
Position the client in a side-lying position.
Clean the skin with an antiseptic solution.
Apply ice to the biopsy site.

Correct response:
Verify the client has signed an informed consent.
Position the client in a side-lying position.
Clean the skin with an antiseptic solution.
Apply ice to the biopsy site.

Explanation:

Question 3 See full question


A nurse is caring for a client with chest trauma. Which nursing diagnosis takes the highest priority?

You Selected:
Impaired gas exchange

Correct response:
Impaired gas exchange

Explanation:

Question 4 See full question


The nurse is educating parents of a child diagnosed with seasonal allergies. The nurse discusses therapeutic
management of the childs allergies and works with the parents to set goals that best support a quality childhood
experience. Which of the following goals is most important for the nurse to set with the parents?

You Selected:
Identifying ways to reduce the childs exposure to the allergens

Correct response:
Identifying ways to reduce the childs exposure to the allergens
Explanation:

Question 5 See full question


A client who is admitted to the adult unit of a mental health care facility with depression tells the nurse that he has
pedophilia. The nurse should:

You Selected:
be aware of personal opinions and views.

Correct response:
be aware of personal opinions and views.

Explanation:
Question 1 See full question
A client in a long-term care facility refuses to take his oral medications. The nurse threatens that she will apply
restraints and inject the medication if he doesn't take it orally. The nurse's statement constitutes which legal tort?
You Selected:
Assault
Correct response:
Assault
Explanation:
Question 2 See full question
While providing care for a hospitalized infant, a nurse is summoned to the phone. The caller requests information
about the infant's condition. The nurse should:
You Selected:
protect the infant's confidentiality by divulging no information to the caller.
Correct response:
determine the caller's identity before responding.
Explanation:
Question 3 See full question
A client being treated for complications of chronic obstructive pulmonary disease needs to be intubated. The client
has previously discussed his wish to not be intubated with his girlfriend of 5 years, whom he's designated as his
health care power of attorney. The client's children want their father to be intubated. A nurse caring for this client
knows that:
You Selected:
clients commonly confer health care power of attorney on someone who shares their personal values and beliefs.
Correct response:
clients commonly confer health care power of attorney on someone who shares their personal values and beliefs.
Explanation:
Question 4 See full question
The nurse-manager on the oncology unit wants to address the issue of correct documentation of the effectiveness
of analgesia medication within 30 minutes after administration. What should the nurse-manager do first?
You Selected:
Complete a brief quality improvement study and chart audit to document the rate of adherence to the policy and
the pattern of documentation over shifts.
Correct response:
Complete a brief quality improvement study and chart audit to document the rate of adherence to the policy and
the pattern of documentation over shifts.
Explanation:
Question 5 See full question
Which documentation tool will the nurse use to record the client's vital signs every 4 hours?
You Selected:
A graphic sheet.
Correct response:
A graphic sheet.
Explanation:
Question 1 See full question
The nurse-manager of an outpatient facility isn't satisfied with discharge planning policies and procedures. Knowing
other managers at similar facilities regarded as the "best" in the country, which steps should the nurse-manager
take as part of a continuous quality-improvement process?
You Selected:
Ask her staff nurses to investigate discharge policies and procedures at other outpatient facilities and recommend
changes.
Correct response:
Contact the nurse-managers at the best facilities and compare their discharge planning policies and procedures
with those of her facility.
Explanation:
Question 2 See full question
While reviewing the admission assessment of a client scheduled for colorectal surgery, the nurse discovers that the
client stopped taking medications to treat emphysema 3 months ago. What would be a priority in planning
collaborative care with the respiratory therapist?
You Selected:
Timely administration of breathing treatments.
Correct response:
Timely administration of breathing treatments.
Explanation:
Question 3 See full question
When cleaning the skin around an incision and drain site, what should the nurse do?
You Selected:
Clean the incision and drain site separately.
Correct response:
Clean the incision and drain site separately.
Explanation:
Question 4 See full question
A client is receiving fluid replacement with lactated Ringers after 40% of the body was burned 10 hours ago. The
assessment reveals temperature 97.1 F (36.2 C), heart rate 122 bpm, blood pressure 84/42 mm Hg, central
venous pressure (CVP) 2 mm Hg, and urine output 25 mL for the last 2 hours. The IV rate is currently at 375 mL/h.
Using the SBAR (Situation-Background-Assessment-Recommendation) technique for communication, the nurse calls
the health care provider (HCP) with a recommendation for:
You Selected:
furosemide
Correct response:
IV rate increase
Explanation:
Question 5 See full question
What role will the nurse have when admitting a client to a hospital for outpatient surgery that will result in
discharge the same day?
You Selected:
Provide detailed information on the procedure.
Correct response:
Complete regular admission procedures.
Explanation:
Question 1 See full question
A nurse is conducting a physical assessment on an adolescent who doesn't want her parents informed that she had
an abortion in the past. Which statement best describes the information security measures the nurse can
implement in this situation?
You Selected:
Respect the adolescent's wishes and maintain her confidentiality.
Correct response:
Respect the adolescent's wishes and maintain her confidentiality.
Explanation:
Question 2 See full question
A client with chronic obstructive pulmonary disease presents with respiratory acidosis and hypoxemia. He tells the
nurse that he doesn't want to be placed on a ventilator. What action should the nurse take?
You Selected:
Notify the physician immediately so he can determine client competency.
Correct response:
Notify the physician immediately so he can determine client competency.
Explanation:
Question 3 See full question
When a nurse tries to administer medication, the client refuses it, saying, "I don't have to take those pills if I don't
want to." What intervention by the nurse would have the highest priority?
You Selected:
Exploring how the client's feelings affect his/her decision to refuse medication
Correct response:
Exploring how the client's feelings affect his/her decision to refuse medication
Explanation:
Question 4 See full question
In many institutions, which of the following telephone or fax orders requires a signature within 24 hours by the
ordering physician or nurse practitioner?
You Selected:
Orders for diagnostic studies.
Correct response:
Orders for antibiotics.
Explanation:
Question 5 See full question
Which client's care may a registered nurse (RN) safely delegate to the nursing assistant?
You Selected:
A client who requires continuous pulse oximetry monitoring admitted with bronchitis.
Correct response:
A client requiring assistance ambulating, who was admitted with a history of seizures.
Explanation:
Question 1 See full question
A nurse has custody of a client's daily Kardex and care plan so she can give a change-of-shift report. After reporting
to the next shift, what steps should the nurse implement to maintain client confidentiality?
You Selected:
Shred the documents or place them in a container to protect confidentiality.
Correct response:
Shred the documents or place them in a container to protect confidentiality.
Explanation:
Question 2 See full question
A client diagnosed with gestational hypertension must have weekly blood pressure checks and urine testing at a
clinic. She does not have transportation. How can the nurse help this client be compliant with her care?
You Selected:
Ask the clinic case manager to speak with the client.
Correct response:
Ask the clinic case manager to speak with the client.
Explanation:
Question 3 See full question
A charge nurse assesses a group of staff nurses as competent individually but ineffective and nonproductive as a
team. How should the charge nurse address the staff nurses about her concerns?
You Selected:
Have the staff nurses express their feelings and emotions.
Correct response:
Have the staff nurses express their feelings and emotions.
Explanation:
Question 4 See full question
The nurse is giving care to an infant with a brain tumor. The nurse observes the infant arches the back (see figure).
The nurse should:
You Selected:
stroke the back to release the arching.
Correct response:
notify the health care provider (HCP).
Explanation:
Question 5 See full question
During a clients recent admission, family members report exhaustion and difficulty taking care of the dependent
client at home. The client's interests are best served by:
You Selected:
providing the caregivers with information on support groups for similar conditions.
Correct response:
calling a family conference and asking Social Services or Service Canada for assistance.
Explanation:
Question 1 See full question
A man with a 5-year history of multiple psychiatric admissions is brought to the emergency department by the
police. This client was found wandering the streets disheveled, shoeless, and confused. Based on his previous
medical records and current behavior, he is diagnosed with schizophrenia. The nurse should assign highest priority
to which nursing diagnosis?
You Selected:
Dressing or grooming self-care deficit
Correct response:
Risk for injury
Explanation:
Question 2 See full question
Which nursing diagnosis takes highest priority for a client with a compound fracture?
You Selected:
Risk for infection related to effects of trauma
Correct response:
Risk for infection related to effects of trauma
Explanation:
Question 3 See full question
When planning care for a client with a small-bowel obstruction, which of the following should the nurse consider to
be the primary goal?
You Selected:
Maintaining fluid balance
Correct response:
Maintaining fluid balance
Explanation:
Question 4 See full question
A nurse is using Dorothea Orem's general theory of nursing while caring for a client, which intervention is
appropriate?
You Selected:
Making the clients social environment comfortable
Correct response:
Providing discharge teaching about new medication
Explanation:
Question 5 See full question
Which client is the best candidate for a vaginal birth after a caesarean (VBAC)?
You Selected:
client who had a breech presentation in her last pregnancy, and this pregnancy is a vertex pregnancy
Correct response:
client who had a breech presentation in her last pregnancy, and this pregnancy is a vertex pregnancy
Explanation:
Question 1 See full question
A nurse is making assignments for the infant unit. The shift's team members include a licensed practical nurse
(LPN) with 10 years of experience, a registered nurse (RN) with 3 months of experience, and a client care assistant.
Which assignment is most appropriate for the LPN?
You Selected:
An infant being discharged to home following placement of a gastrostomy tube
Correct response:
An infant requiring abdominal dressing changes for a wound infection
Explanation:
Question 2 See full question
A multipara at 16 weeks gestation is diagnosed as having a fetus with probable anencephaly. The client is a devout
Baptist and has decided to continue the pregnancy and donate the neonatal organs after the death of the neonate.
The nurse should:
You Selected:
explore the nurses own feelings about the issues of anencephaly and organ donation.
Correct response:
explore the nurses own feelings about the issues of anencephaly and organ donation.
Explanation:
Question 3 See full question
Nurse researchers have proposed a study to examine the efficacy of a new wound care product. Which of the
following aspects of the methodology demonstrates that the nurses are attempting to maintain the ethical principle
of nonmaleficence?
You Selected:
The nurses have completed a literature review that suggests the new treatment may result in decreased wound
healing time.
Correct response:
The nurses are taking every responsible measure to ensure that no participants experience impaired wound
healing as a result of the study intervention.
Explanation:
Question 4 See full question
After completing a shift, a nurse realizes that documentation on a client was not completed before leaving the unit.
Which of the following actions by the nurse is most appropriate?
You Selected:
Call the unit, and dictate the entry to another nurse.
Correct response:
Enter the information tomorrow stating it is a late entry.
Explanation:
Question 5 See full question
A stable older adult client is comatose following a cerebral vascular accident. The primary healthcare provider
believes a gastrostomy tube should be placed for long-term nutrition. No family members have been located. Which
of the following should be done to obtain informed consent for the procedure?
You Selected:
The primary healthcare provider may act without consent to save the clients life.
Correct response:
The nurse should contact the person identified as the healthcare power of attorney.
Explanation:
Question 1 See full question
An elderly client who has been diagnosed with delusional disorder for many years is exhibiting early symptoms of
dementia. His daughter lives with him to help him manage daily activities, and he attends a day care program for
seniors during the week while she works. A nurse at the day care center hears him say, If my neighbor puts up a
fence, I will blow him away with my shotgun. He has never respected my property line, and I have had it! Which
action should the nurse take?
You Selected:
Report the comment to the neighbor, the daughter, and the police since there is the potential for a criminal act.
Correct response:
Report the comment to the neighbor, the daughter, and the police since there is the potential for a criminal act.
Explanation:
Question 2 See full question
A client reports having blurred vision after 4 days of taking haloperidol 1 mg twice a day, and benztropine 2 mg
twice a day. The nurse contacts the health care provider (HCP) to explain the situation, background, and
assessment and make a recommendation. Which information reported to the HCP is the assessment of the
situation?
You Selected:
"Mr. Roberts is reporting blurred vision since this morning."
Correct response:
"The higher dose of benztropine could be causing Mr. Roberts' blurred vision."
Explanation:
Question 3 See full question
The nurse is working in a newborn nursery and caring for several neonates. What precaution should be taken to
prevent an infant abduction?
You Selected:
Notify the hospital's security staff about anyone who appears unusual.
Correct response:
Notify the hospital's security staff about anyone who appears unusual.
Explanation:
Question 4 See full question
A stable older adult client is comatose following a cerebral vascular accident. The primary healthcare provider
believes a gastrostomy tube should be placed for long-term nutrition. No family members have been located. Which
of the following should be done to obtain informed consent for the procedure?
You Selected:
The nurse should contact the person identified as the healthcare power of attorney.
Correct response:
The nurse should contact the person identified as the healthcare power of attorney.
Explanation:
Question 5 See full question
The mother of an infant with a cleft lip asks when the repair will be scheduled. What is the nurses best response?
You Selected:
at 1 year of age
Correct response:
during the first 6 months of life
Explanation:
Question 1 See full question
The nurse at a substance abuse center answers the phone. A probation officer asks if a client is in treatment. The
nurse responds, "No, the client you're looking for isn't here." Which statement best describes the nurse's response?
You Selected:
Illegal, because she's withholding information from law enforcement agents
Correct response:
A violation of confidentiality because she informed the officer that the client wasn't there
Explanation:
Question 2 See full question
The nurse finds an unlicensed assistive personnel (UAP) massaging the reddened bony prominences of a client on
bed rest. The nurse should:
You Selected:
Instruct the UAP that massage is contraindicated because it decreases blood flow to the area.
Correct response:
Instruct the UAP that massage is contraindicated because it decreases blood flow to the area.
Explanation:
Question 3 See full question
A client with a history of asthma is admitted to the emergency department. The nurse notes that the client is
dyspneic, with a respiratory rate of 35 breaths/min, nasal flaring, and use of accessory muscles. Auscultation of the
lung fields reveals greatly diminished breath sounds. What should the nurse do first?
You Selected:
Administer bronchodilators as prescribed.
Correct response:
Administer bronchodilators as prescribed.
Explanation:
Question 4 See full question
Which client's care may a registered nurse (RN) safely delegate to the nursing assistant?
You Selected:
A client requiring assistance ambulating, who was admitted with a history of seizures.
Correct response:
A client requiring assistance ambulating, who was admitted with a history of seizures.
Explanation:
Question 5 See full question
A nurse is working with an unlicensed assistive personnel (UAP). Which clients should the nurse assign to the UAP?
Select all that apply.
You Selected:
Older adult client who had hip replacement surgery and needs to walk in the hall with a walker.
Adult client newly diagnosed with diabetes who is learning to administer insulin.
Adult client who had a hysterectomy 3 days ago and requires vital sign checks every 4 hours.
Correct response:
Older adult client who had hip replacement surgery and needs to walk in the hall with a walker.
Adult client who had a hysterectomy 3 days ago and requires vital sign checks every 4 hours.
Explanation:
Question 1 See full question
A transfusion of packed red blood cells has been ordered for a 1-year-old with a sickle cell anemia. The infant has a
25 gauge IV infusing dextrose with sodium and potassium. Using the situation, background, assessment,
recommendation (SBAR) method of communication, the nurse contacts the health care provider (HCP) and
recommends:
You Selected:
using the existing IV, but changing the fluids to normal saline for the transfusion.
Correct response:
using the existing IV, but changing the fluids to normal saline for the transfusion.
Explanation:
Question 2 See full question
Which nursing intervention is most appropriate for a client with multiple myeloma?
You Selected:
Balancing rest and activity
Correct response:
Preventing bone injury
Explanation:
Question 3 See full question
A nurse is providing inservice education for staff members about evidence collection after sexual assault. The
educational session is successful when staff members focus their initial care on which step?
You Selected:
Supporting the client's emotional status
Correct response:
Supporting the client's emotional status
Explanation:
Question 4 See full question
Nurses who provide care in a large, long-term care facility use charting by exception (CBE) as the preferred method
of documentation. This documentation method may have which of the following drawbacks?
You Selected:
Vulnerability to legal liability because the nurse's safe, routine care is not recorded.
Correct response:
Vulnerability to legal liability because the nurse's safe, routine care is not recorded.
Explanation:
Question 5 See full question
The parents of a healthy infant request information about advance directives. The nurse's best response is to:
You Selected:
inform the parents that advance directives are a legal document and need a notary.
Correct response:
ask open-ended questions to understand the parents' concerns.
Explanation:Question 1 See full question
A nurse-manager works for a nonprofit health care corporation whose revenues have significantly exceeded annual
expenses. The nurse-manager has been told to anticipate which action?

You Selected:
Receiving a portion of the revenue to improve client services on the unit

Correct response:
Receiving a portion of the revenue to improve client services on the unit

Explanation:

Question 2 See full question


When prioritizing a client's care plan based on Maslow's hierarchy of needs, a nurse's first priority would be:

You Selected:
administering pain medication.
Correct response:
administering pain medication.

Explanation:

Question 3 See full question


A nurse explains the guidelines for the unit's seclusion room to a client with an impulse control disorder. Which
client statement indicates that the nurse has adequately communicated the client's rights?

You Selected:
"Although I don't think I will, I can ask to go into seclusion, but I know you can make me go into the seclusion
room."

Correct response:
"Although I don't think I will, I can ask to go into seclusion, but I know you can make me go into the seclusion
room."

Explanation:

Question 4 See full question


A client is having elective surgery under general anesthesia. Who is responsible for obtaining the informed consent?

You Selected:
the surgeon

Correct response:
the surgeon

Explanation:

Question 5 See full question


The nurse assigns an unlicensed assistive personnel (UAP) to care for a client who has a newly applied long-leg
plaster cast. What should the nurse tell the UAP about proper care of the cast while it is drying?

You Selected:
Turn the client every 2 hours to promote even drying of the cast.

Correct response:
Turn the client every 2 hours to promote even drying of the cast.

Explanation:

Question 6 See full question


A nurse is caring for a client who is exhibiting signs and symptoms characteristic of a myocardial infarction (MI).
Which statement describes priorities the nurse should establish while performing the physical assessment?

You Selected:
Assess the client's level of pain, and administer prescribed analgesics.

Correct response:
Assess the client's level of pain, and administer prescribed analgesics.
Explanation:

Question 7 See full question


The nurse in the emergency department is triaging victims of an airplane crash. Prioritize the clients in the order in
which they should be treated from first to last. All options must be used.

You Selected:
q 14-year-old with a 2-inch (5.1-cm) laceration to the chin, history of asthma, respirations 26 breaths/min, audible
wheezing
a 22-year-old with a 2-inch (5.1-cm) laceration to the left temple, slightly confused
a 22-year-old female, 36 weeks pregnant with contractions every 10 to 15 minutes
a 75-year-old with a 2-inch (5.1-cm) laceration to the left forearm

Correct response:
q 14-year-old with a 2-inch (5.1-cm) laceration to the chin, history of asthma, respirations 26 breaths/min, audible
wheezing
a 22-year-old with a 2-inch (5.1-cm) laceration to the left temple, slightly confused
a 22-year-old female, 36 weeks pregnant with contractions every 10 to 15 minutes
a 75-year-old with a 2-inch (5.1-cm) laceration to the left forearm

Explanation:

Question 8 See full question


A client with a peritonsillar abscess has been hospitalized. Upon assessment, the nurse determines the following: a
temperature of 103 F (39.4 C), body chills, and leukocytosis. The client begins to have difficulty breathing. In what
order should the nurse perform the actions? All options must be used.

You Selected:
Open the airway.
Start an IV access site.
Call the health care provider (HCP).
Explain the situation to the family.

Correct response:
Open the airway.
Start an IV access site.
Call the health care provider (HCP).
Explain the situation to the family.

Explanation:

Question 9 See full question


The parents of a healthy infant request information about advance directives. The nurse's best response is to:

You Selected:
ask open-ended questions to understand the parents' concerns.

Correct response:
ask open-ended questions to understand the parents' concerns.

Explanation:

Question 10 See full question


The nurse at an outpatient surgical clinic witnesses client signatures. When obtaining signatures, which clients are
able to sign their own consent for a procedure/surgery? Select all that apply.
You Selected:
A 62-year-old with macular degeneration who is ordered a routine colonoscopy
A married 17-year-old who requires a cholecystectomy for relief of nausea and pain

Correct response:
A 62-year-old with macular degeneration who is ordered a routine colonoscopy
A married 17-year-old who requires a cholecystectomy for relief of nausea and pain

Explanation:

A bearded nurse needs to a)


assess the condition of a Powere
client with pulmonary d Air
tuberculosis (TB). What is Purifyin
the best respirator mask for g
this nurse? Respira
tor
a) Powered Air Purifying
Respirator
b) Volume controlled
respirator
c) N95 respirator
d) Cuirass respirator

2. A client for whom transmission-based b) Position the client so that


precautions are required is experiencing he or she can look out the
sensory deprivation. Which of the following window.
techniques could the nurse employ to assist c) Move the bed to various
this client? Select all that apply. places in the room or
periodically rearrange the
a) Communicate only using the intercom furnishings in the room
system. .d) Encourage activities that
b) Position the client so that he or she can the client can do
look out the window. independently such as
c) Move the bed to various places in the reading, working crossword
room or periodically rearrange the puzzles, playing solitaire,
furnishings in the room. and putting picture puzzles
d) Encourage activities that the client can do together.
independently such as reading, working
crossword puzzles, playing solitaire, and
putting picture puzzles together.
e) Position equipment that produces
monotonous sounds near the client so he or
she has a recognizable sound to hear.
3. A client has a draining wound that is b) Body-substance isolation
contaminated with Staphylococcus aureus.
The nurse should observe

a) Universal precautions
b) Body-substance isolation
c) Droplet precautions
d) Reverse precautions
4. A client in the emergency department 3ft
waiting room is showing signs of respiratory
symptoms. Approximately how much
distance from others should the nurse tell the
client to maintain? Fill in the blank with a
number.
5. A client is being admitted to the hospital with b) Wear a particulate air
a positive tuberculosis test and suspicious filter respirator during client
chest x-ray. Which of the following measures care.
by the nurse is appropriate?
a) Teach the client to dispose of tissues in a
special sealed device.
b) Wear a particulate air filter respirator
during client care.
c) Post infection control measures on the
room door, clearly identifying the disease.
d) Direct the client to provide a sputum
specimen at the public health department
within 6 months of discharge.
6. A client is experiencing generalized b) Prodromal period
weakness and body aches. In the progress of
infection, the client is in the

a) Acute period
b) Prodromal period
c) Convalescent period
d) Incubation period

7 A client is on contact precatuions. How frequently must the nurse ensure that c) Daily
. care items and bedside equipment for this client are cleaned?

a) Twice a day
b) Weekly
c) Daily
d) Hourly
8 A client with an infectious disease that requires airborne precautions must be a) Cover as much of the client's
. transported from his room to the radiology department. What measures body as possible during
would the nurse take to protect against spreading infection while overseeing transport.
the transport of this client? (Select all that apply.) b) Use the same PPE during
transport as the nurse would
a) Cover as much of the client's body as possible during transport. while caring for the client.
b) Use the same PPE during transport as the nurse would while caring for the c) Line the surface of the
client. wheelchair or stretcher with a
c) Line the surface of the wheelchair or stretcher with a clean sheet or bath clean sheet or bath blanket to
blanket to protect the surface from direct client contact. protect the surface from direct
d) Place a color-coded sticker on the client's hospital gown so that others will client contact.
know to keep their distance from the client. e) Make sure the client wears a
e) Make sure the client wears a mask or particulate air filter respirator. mask or particulate air filter
respirator.
9 A college-aged student has influenza. At what stage of the infection is the a) Prodromal stage
. student most infectious?

a) Prodromal stage
b) Convalescent period
c) Full stage of illness
d) Incubation period
1 For which of the following clients would the use of Standard Precautions alone b) An incontinent client in a
0 be appropriate? nursing home who has diarrhea
.
a) A child with chickenpox who is treated in the ER
b) An incontinent client in a nursing home who has diarrhea
c) A client with TB who needs medications administered
d) A client with diphtheria who needs pm care
1 An infection-control nurse is discussing needlestick injuries with a group of b) Recapping a needle
1 newly hired nurses. The infection control nurse informs the group that most
. needlestick injuries result from which of the following?

a) Needles left in the client's linen


b) Recapping a needle
c) Faulty needles and syringes
d) Full needle boxes
1 The latest CDC guidelines designate standard precautions for all substances a) Sweat
2 except which of the following?
.
a) Sweat
b) Blood
c) Vomitus
d) Urine

1 A lead nurse is removing her personal protective equipment after dressing the a) Handwashing before
3 infected wounds of a client. Which of the following is the highest priority nursing leaving the client's room.
. action?

a) Handwashing before leaving the client's room.


b) Make contact between two clean surfaces.
c) Make contact between two contaminated surfaces.
d) Remove the garments that are most contaminated.
1 Microorganisms present on the human body without host interference or d) Colonization
4 interaction refers to what?
.
a) Disease
b) Infection
c) Inflammation
d) Colonization
1 The nurse explains to the patient the first line of defense against infection is: c) intact skin and mucous
5 membranes.
. a) frequent hand washing with soap and water.
b) staying home when sick.
c) intact skin and mucous membranes.
d) early intervention with antibiotics.
e) low levels of normal flora.
1 A nurse has just finished a performing a procedure with a client that required 2)Gloves
6 personal protective equipment. After washing her hands, which sequence should 3)Mask
. the nurse use to remove equipment, to minimize the potential for disease 1)Eye Protection
transmission? 4)Gown or apron
1)Eye Protection
2)Gloves
3)Mask
4)Gown or apron
1 A nurse is caring for a child who is hospitalized for diphtheria. Which one of the d) Wear PPE when entering
7 following guidelines would be appropriate when caring for this client? the room for all interactions
. that may involve contact
a) Place client in private room that has monitored negative air pressure. with the client.
b) Use respiratory protection when entering the room of client with known or
suspected diphtheria.
c) Use a private room with the door closed.
d) Wear PPE when entering the room for all interactions that may involve contact
with the client.
1 A nurse is caring for a client in isolation. Which precaution should the nurse follow d) Discard used intravenous
8 when administering medication to a client in isolation? (IV) bags in the room.
.
a) Recap or detach needles from syringes.
b) Store all personal protective equipment (PPE) inside the room.
c) Use the washbasin outside the room.
d) Discard used intravenous (IV) bags in the room.
1 The nurse is caring for a client who has active tuberculosis (TB) and is in Airborne Place a surgical mask on
9 Precautions. The primary care provider orders a computed tomography (CT) the client and transport to
. examination of the chest. Which of the following actions by the nurse is the CT department at the
appropriate? specified time.
2 A nurse is caring for a client who has influenza and varicella. Which type of a) Airborne
0 transmission precautions should the nurse follow when caring for the client? b) Droplet
. Select all that apply. e) Contact

a) Airborne
b) Droplet
c) Basic
d) Standard
e) Contact
2 A nurse is caring for a client with rubella. What precautions should the nurse take a) Wear a mask when
1 when caring for this client? working within 3 feet of
. the client.
a) Wear a mask when working within 3 feet of the client.
b) Use a special high-filtration particulate respirator.
c) Wash hands with an antimicrobial agent or waterless antiseptic agent.
d) Change gloves after contact with the client's infective material.
2 A nurse is caring for a client with streptococcal pneumonia. The nurse has to initiate b) Droplet Precautions
2 precautions for the client. Based on this information, what type of precautions should
. the nurse initiate and review the procedures with staff members?

a) Airborne Precautions
b) Droplet Precautions
c) Contact Precautions
d) Protective Precautions
2 A nurse is donning gloves to care for a client. How often are gloves worn before they b) Once
3 should be discarded?
.
a) Gloves can be reworn indefinitely.
b) Once
c) 50 times
d) Twice
2 A nurse is following the CDC's guidelines for safe injection practices. Which of the a) Use a sterile, single-
4 following techniques would the nurse employ? Select all that apply. use, disposable syringe
. for each injection.
a) Use a sterile, single-use, disposable syringe for each injection. b) Follow aseptic
b) Follow aseptic technique. technique.
c) Use single-dose vials rather than multiple-dose vials when administering. c) Use single-dose vials
d) Prevent the contamination of injection equipment and medication. rather than multiple-
e) Ensure that all clients who need injections are kept on isolation precautions. dose vials when
administering.
d) Prevent the
contamination of
injection equipment and
medication.
2 A nurse is in charge of patient care for a patient who has MRSA. Which of the d) Wear gloves
5 following is an accurate guideline for using Transmission-Based Precautions when whenever entering the
. caring for this patient? patient's room.

a) Place the patient in a private room that has monitored negative air pressure.
b) Use respiratory protection when entering the room.
c) Keep visitors 3 feet from the patient.
d) Wear gloves whenever entering the patient's room.
2 A nurse is reviewing an adult client's chart and sees that the client is overdue for a 10 years
6 tetanus booster. What time must have elapsed for the client to require this shot?
.
2 A nurse is taking stock of the equipment in the room of an older adult client with d) Indwelling catheter
7 pneumonia who has been on parenteral nutrition for a long time. Which of the
. following equipment can transmit infection to older adult clients?

a) Specimen containers
b) Bath blanket
c) Face shields
d) Indwelling catheter

2 A nurse is working in a hospital in which a client has been admitted with pulmonary Airborne
8 tuberculosis. What type of precautions would this client require?
.
a) Airborne
b) Droplet
c) Reservoir
d) Contact
2 A nurse is working with a patient with an infectious disease that requires the nurse to a) Tuberculosis
9 wear a particulate air filter respirator. Which disease does the client likely have?
.
a) Tuberculosis
b) Impetigo
c) Influenza
d) Chickenpox
3 A nurse needs to send the blood and urine specimen of a client with acute diarrhea to c) Use sealed containers
0 the pathology laboratory. Which of the following precautions is of highest priority to in a plastic biohazard
. be taken by the nurse when collecting and delivering the specimens to the bag
laboratory?

a) Use a particulate air filter respirator


b) Use thoroughly washed gloves
c) Use sealed containers in a plastic biohazard bag
d) Use disposable cover gowns and goggles
3 Routine nasal and rectal swabbing of a newly admitted hospital client has come back c) "This means that this
1 positive for methicillin-resistant Staphylococcus aureus (MRSA), indicating that the organism in present on
. client is colonized with MRSA. The client is surprised at this finding, since he enjoys your skin, but it doesn't
generally robust health. What should the client's nurse teach him about this necessarily mean that
diagnostic finding? you will become sick."

a) "This finding becomes part of your medical record, but it is not a threat to the
health of yourself or others."
b) "You may not develop any symptoms, but you will likely be given a round of
antibiotics to eliminate these bacteria."
c) "This means that this organism in present on your skin, but it doesn't necessarily
mean that you will become sick."
d) "It's very fortunate that this was detected early, since this had the potential to
make you very sick."
3 To eliminate needlesticks as potential hazards to nurses, the nurse should Immediately deposit
2 uncapped needles into
. puncture-proof plastic
container
3 Two nurses are working together to double-bag some contaminated items from the d) Folds the top of the
3 room of a client in isolation. Which of the following is a role of the "clean" nurse? clean bag down on the
. outside to make a collar
or cuff
a) Touches only the inside of the clean bag
b) Places dirty items into a bag and closes the top
c) Places the contaminated bag in side the clean bag
d) Folds the top of the clean bag down on the outside to make a collar or cuff
3 What is the most common reason people contact healthcare providers? d) Infectious disease
4
. a) Sleeplessness
b) Pain
c) Anxiety
d) Infectious disease

A client comes to the c) Intact


emergency skin and
department with mucous
major burns over membranes
40% of his body. protect
Although all of the against
following are true, microbial
which one would invasion.
provide the rationale
for a nursing
diagnosis of Risk for
Infection?

a) Age, race, sex,


and hereditary
factors influence
susceptibility to
infection.
b) Stress may
adversely affect
normal defense
mechanisms.
c) Intact skin and
mucous membranes
protect against
microbial invasion.
d) White blood cells
provide resistance to
certain pathogens.

2. A client has an inguinal a) Surgical


hernia repair and later asepsis
develops a methicillin-
resistant
Staphylococcus aureus
infection. What is the
most important factor
to prevent this
infection?

a) Surgical asepsis
b) Decreased
antibiotics
c) Increased T cells
d) Increased vitamin C
3. A client suffers from b) Bacteria
bloody diarrhea after
eating contaminated
food at a local
restaurant. The client
has been infected with
a(an)

a) Virus
b) Bacteria
c) Protozoa
d) Fungi
4. A client with an upper d)
respiratory infection "Antibiotics
(common cold) tells the have no
nurse, "I am so angry effect on
with the nurse viruses."
practitioner because he
would not give me any
antibiotics." What
would be the most
accurate response by
the nurse?

a) "I know what you


mean; you need an
antibiotic."
b) "Let me talk to him
and see what we can
do."
c) "Why do you think
you need an
antibiotic?"
d) "Antibiotics have no
effect on viruses."
5 An experienced nurse is teaching a student c) Hand hygiene must
. nurse the proper use of hand hygiene. Which be performed after
of the following is an accurate guideline that contact with
should be discussed? inanimate objects
near the client.
a) The use of gloves eliminates the need for
hand hygiene.
b) The use of hand hygiene eliminates the
need for gloves.
c) Hand hygiene must be performed after
contact with inanimate objects near the client.
d) Hand lotions should not be used after hand
hygiene.
6 The following procedures have been ordered d) Urinary
. and implemented for a hospitalized client. catheterization
Which procedure carries the greatest risk for a
nosocomial infection?

a) Enema
b) Intramuscular injections
c) Heat lamp
d) Urinary catheterization
7 A home health nurse is completing a health "Have you had any
. history for a patient. What is one question that unusual symptoms
is important to ask to identify a latex allergy after blowing up
for this patient? balloons?"

a) "Have you ever had an allergic reaction to


shellfish or iodine?"
b) "Have you had any unusual symptoms after
blowing up balloons?"
c) "When you were a child, did you have
frequent infections?"
d) "Tell me what you use to wash your hands
after toileting."
8 How long should a healthcare worker scrub d) 15 seconds
. hands that are not visibly soiled for effective
hand hygiene?

a) 1 minute
b) 5 minutes
c) 30 seconds
d) 15 seconds

9. The nurse caring for clients at an outpatient clinic d) An 80-year-


determines which of the following clients is at old woman
greatest risk for infection?

a) A 2-year-old toddler
b) A 12-year-old girl
c) An 18-month-old infant
d) An 80-year-old woman
1 A nurse changing the linens of a patient bed is c) Keep hands
0. exposed to urine and performs hand hygiene. lower than
Which of the following is a guideline for performing elbows to allow
this skill properly following this patient encounter? water to flow
toward
a) Use an alcohol-based hand rub to fingertips.
decontaminate hands.
b) Remove all jewelry, including wedding bands
before handwashing.
c) Keep hands lower than elbows to allow water to
flow toward fingertips.
d) Pat dry with a paper towel, beginning with the
forearms and moving down to fingertips.
1 A nurse follows surgical asepsis techniques for d) Hold sterile
1. inserting an indwelling urinary catheter in a client. objects above
Which of the following is an accurate guideline for waist level to
using this technique? prevent
accidental
a) Open sterile packages so that the first edge of contamination.
the wrapper is directed toward you.
b) Consider the outside of the sterile package to be
sterile.
c) Consider the outer 3-inch edge of a sterile field
to be contaminated.
d) Hold sterile objects above waist level to prevent
accidental contamination.
1 A nurse has completed morning care for a client. c) Clean hands
2. There is no visible soiling on her hands. What type with an alcohol-
of technique is recommended by the CDC for hand based handrub.
hygiene?

a) Wash hands with soap and water.


b) Do not wash hands, apply clean gloves.
c) Clean hands with an alcohol-based handrub.
d) Wash hands with soap and water, follow with
handrub.

1 A nurse is educating a rural community group on how to c) Vectors


3. avoid contracting West Nile virus by using approved
insect repellant and wearing proper coverings when
outdoors. By what means is the pathogen involved in
West Nile virus transmitted?

a) Direct contact
b) Indirect contact
c) Vectors
d) Airborne route
1 A nurse is positioning a sterile drape to extend the b) Use
4. working area when performing a urinary catheterization. sterile
Which of the following is an appropriate technique for this gloves to
procedure? handle
the entire
a) When reaching over the drape do not allow clothing to drape
touch the drape. surface.
b) Use sterile gloves to handle the entire drape surface.
c) Touch only the outer two inches of the drape when not
wearing sterile gloves.
d) Fold the lower edges of the drape over the sterile-
gloved hands.
1 A nurse is preparing a sterile field and has removed the b) Facing
5. sterile drape from the outer wrapper. The nurse places away
the inner drape in the center of the work surface with the from the
outer flap facing in which direction? body

a) Angled to the left side


b) Facing away from the body
c) Toward the right side
d) Facing toward the body
1 A nurse is preparing to add a sterile solution to a sterile a) Position
6. container on a sterile field. After opening the container, it with the
which of the following would the nurse do with the cap? inside
facing up
on a flat
a) Position it with the inside facing up on a flat surface surface
b) Place it in the biohazard receptacle
c) Place it rim down on the corner of the sterile field
d) Hold it in the nondominant hand
1 The nurse is preparing to perform handwashing. Turn on the
7. Arrange the following steps in the correct order. faucet and adjust
force and
temperature of
the water.
Wet the hand and
wrist areas.
Apply soap
product.
Wash the palms
and back of the
hands for at least
15 seconds.
Pat hands dry
with a paper
towel.
Turn the faucet
off with a paper
towel.
1 A nurse is providing care to a client who has d) Over-
8. developed an infection due to Candida. The prescription of
infection is resistant to several medications. The antibiotics
client asks the nurse how he may have developed
this infection. When responding to the client, the
nurse would incorporate an understanding of
which of the following as contributing to the
organism's resistance?

a) Low intake of meat and poultry products


b) Less use of pasteurizing agents in milk
c) A decrease in the use of antibiotics in farming
d) Over-prescription of antibiotics
1 Nurses use medical asepsis in practice to reduce c) Clean the least
9. the number and transfer of pathogens. Which of soiled areas first
the following are principles of this practice? Select and then move to
all that apply. the more soiled
ones.
a) Shake out linens and patient clothing before e) Use personal
placing them back on the bed. grooming habits,
b) Move equipment close to you when brushing, such as
dusting, or scrubbing articles. shampooing hair
c) Clean the least soiled areas first and then move often, to prevent
to the more soiled ones. spreading
d) Carry soiled items close to the body to prevent microorganisms.
transfer of pathogens into the environment.
e) Use personal grooming habits, such as
shampooing hair often, to prevent spreading
microorganisms.
f) Place soiled bed linen or any other items on the
floor, instead of the bed or furniture.

2 A nurse working at a health care facility b) Client with


0. understands the need for providing aseptic care burn injuries
when caring for clients. Which of the following
clients is at greatest risk for infections?

a) Client who is pregnant


b) Client with burn injuries
c) Client with fracture
d) Client with fever
2 Surgical asepsis is defined as c) Absence of all
1. microorganisms
a) Absence of all virulent microorganisms
b) Use of handwashing, gowning, and gloving
c) Absence of all microorganisms
d) Slowed growth of microorganisms
2 The use of alcohol-based hand rubs for hand a) False
2. hygiene in healthcare facilities is approved by the
Centers for Disease Control (CDC), but The Joint
Commission (TJC) discourages its use.

a) False
b) True
2 What are the recommended cleansing agents for c) Antimicrobial
3. hand hygiene in any setting when the risk of products
infection is high?

a) Cold water
b) Liquid or bar hand soap
c) Antimicrobial products
d) Hot water
2 When caring for clients at the health care facility, a) Client with
4. the nurse knows that clients are susceptible to gastric tube
infections. Which of the following clients are at a feeding
greater risk for infection? Select all that apply. b) Client with an
IV catheter
a) Client with gastric tube feeding e) Client with an
b) Client with an IV catheter indwelling
c) Client with hypertension catheter
d) Client with fever and chills
e) Client with an indwelling catheter

PrepU Chp.5 Homeostasis, Adaptation, and Stress

Study online at quizlet.com/_1e84gz

1 A 65-year-old client has experienced the death of a d) Joining the


. parent and a family pet in one month. Which of the local garden
following is a coping mechanism that demonstrates club
adaptation?

a) Deciding to no longer attend his family reunion


picnic
b) Visiting a psychic
c) Buying a new car
d) Joining the local garden club
2 The children of a woman 60 years of age are c)
. distraught at her apparent lack of recovery following a Displacement
stroke several weeks earlier. The client's daughter has
frequently directed harsh criticism toward the nurses,
accusing them of a substandard effort in rehabilitating
her mother despite their best efforts. What defense
mechanism may the client's daughter be exhibiting?

a) Regression
b) Denial
c) Displacement
d) Sublimation
3 The client is a single mother of two children who c) Assisting
. attends college and works full time. She is seeing the the client to
college nurse due to a crying outburst in class. The identify the
first step of crisis intervention that the nurse employs reason for her
is what? outburst

a) Outlining several solutions to the crisis with the


client
b) Having the client select an acceptable solution to
her problem.
c) Assisting the client to identify the reason for her
outburst
d) Asking the client, "What would happen if you did
this solution?"
4 The client is experiencing stress due to family issues. b) "You will
. The nurse is teaching a client meditation to relieve her need a quiet
stress. The nurse states: place to
meditate."
a) "Perform these exercises for 5 minutes every day." d) "Focus on a
b) "You will need a quiet place to meditate." word that has
c) "Keep your eyes open, looking at a beautiful scene." meaning to
d) "Focus on a word that has meaning to you." you."
e) "Place yourself in a comfortable sitting position." e) "Place
yourself in a
comfortable
sitting
position."

5 Cold temperatures and loud noises are stressors to d) The


. one person but not another. Why does this occur? perception and
effects of
a) Although the perception is the same, the response stressors are
is individualized. highly
b) Both individuals will respond the same, depending individualized.
on the situation.
c) The internal environment of one person is more
selective.
d) The perception and effects of stressors are highly
individualized.
6 Family conflict around the care of a recently a) Clear
. hospitalized woman has escalated to the point that identification
crisis intervention may be required. What should this of the relevant
process begin with? problem

a) Clear identification of the relevant problem


b) Comparison of the family's situation to other
similar situations
c) Presentation of clear, achievable, and evidence-
based solutions
d) Careful and objective analysis of different proposed
options
7 A group of nursing students are reviewing information c) Adaptation
. about stress and coping. The students demonstrate
an understanding of the information when they
identify which of the following as the outcome of
coping?

a) Allostasis
b) Homeostasis
c) Adaptation
d) Compensation
8 A man has noticed bright red blood in his bowel d) Denial
. movements for over a month. He says to himself,
"Oh, it's just my hemorrhoids." What defense
mechanism is the man using?

a) Rationalization
b) Compensation
c) Repression
d) Denial

Question 1 See full question


A nurse-manager has decided to delegate responsibility for the review and revision of the surgical unit's client-
education materials. Which statement illustrates the best method of delegation?
You Selected:
Ask the two most proficient staff nurses to form a task force to review and revise client-education materials within
the next 6 weeks. Have these nurses solicit input from clients and staff members.
Correct response:
Ask the two most proficient staff nurses to form a task force to review and revise client-education materials within
the next 6 weeks. Have these nurses solicit input from clients and staff members.
Explanation:
Question 2 See full question
The nurse is caring for a group of clients on a medical-surgical nursing unit. Which task(s) could the nurse delegate
to unlicensed assistive personnel (UAP)? Select all that apply.
You Selected:
Obtain intake and outputs on a client experiencing heart failure.
Perform vital signs and oxygen saturation on a client returning from the catheterization lab.
Correct response:
Perform vital signs and oxygen saturation on a client returning from the catheterization lab.
Obtain intake and outputs on a client experiencing heart failure.
Explanation:
Question 3 See full question
The nurse has received a change of shift report on clients. Which client should the nurse assess first?
You Selected:
a client with asthma with respirations of 36 breaths/min whose wheezing has diminished
Correct response:
a client with asthma with respirations of 36 breaths/min whose wheezing has diminished
Explanation:
Question 4 See full question
A client newly diagnosed with deep vein thrombosis (DVT) of the left lower left extremity is on bed rest. The nurse
should instruct the unlicensed assistive personnel (UAP) providing routine morning care for the client to:
You Selected:
check that the legs are in a low, dependent position.
Correct response:
ensure that the lower extremity is elevated.
Explanation:
Question 5 See full question
A nurse hears a client state, I have had it with this marriage. It would be so much easier to just hire someone to kill
my husband! What action should the nurse take?
You Selected:
The nurse must start the process to warn the clients husband.
Correct response:
The nurse must start the process to warn the clients husband.
Explanation:
Question 1 See full question
Following an epidural and placement of internal monitors, a client's labor is augmented. Contractions are lasting
greater than 90 seconds and occurring every 1 minutes. The uterine resting tone is greater than 20 mm Hg with
an atypical fetal heart rate and pattern. Which action should the nurse take first?
You Selected:
Turn the client to her left side.
Correct response:
Turn off the oxytocin infusion.
Explanation:
Question 2 See full question
A widowed client who is receiving chemotherapy tells the nurse that he does not like to cook for himself. A
community resource for this client is:
You Selected:
a meal delivery service.
Correct response:
a meal delivery service.
Explanation:
Question 3 See full question
The nurse is teaching the client to self-administer insulin. Learning goals most likely will be attained when they are
established by the:
You Selected:
client, nurse, pharmacist, and health care provider so the client can participate in planning care with the entire
team.
Correct response:
client, nurse, pharmacist, and health care provider so the client can participate in planning care with the entire
team.
Explanation:
Question 4 See full question
A client in a long-term care facility has signed a form stating that he does not want to be resuscitated. He develops
an upper respiratory infection that progresses to pneumonia. His health rapidly deteriorates, and he is no longer
competent. During morning rounds, the nurse finds this client without vital signs. What should the nurse do next?
You Selected:
Go to the desk and review the clients chart to determine resuscitation status.
Correct response:
Notify the physician that the client has no vital signs.
Explanation:
Question 5 See full question
The nurse is caring for a client with severe diarrhea. The nurse recognizes that the client is at risk for developing
which of the following acid-base imbalances?
You Selected:
Metabolic acidosis
Correct response:
Metabolic acidosis
Explanation:
Question 1 See full question
A client is to be discharged from an alcohol rehabilitation program. What should the nurse emphasize in the
discharge plan as a priority?
You Selected:
follow-up care
Correct response:
follow-up care
Explanation:
Question 2 See full question
A client has renal colic due to renal lithiasis. What is the nurse's first priority in managing care for this client?
You Selected:
Administer an opioid analgesic as prescribed.
Correct response:
Administer an opioid analgesic as prescribed.
Explanation:
Question 3 See full question
The nurse is observing an unlicensed assistive personnel (UAP) give care to a client after gynecologic surgery. The
nurse should intervene if the UAP:
You Selected:
ambulates the client.
Correct response:
massages the clients legs.
Explanation:
Question 4 See full question
A client with a peritonsillar abscess has been hospitalized. Upon assessment, the nurse determines the following: a
temperature of 103 F (39.4 C), body chills, and leukocytosis. The client begins to have difficulty breathing. In what
order should the nurse perform the actions? All options must be used.
You Selected:
Open the airway.
Start an IV access site.
Explain the situation to the family.
Call the health care provider (HCP).
Correct response:
Open the airway.
Start an IV access site.
Call the health care provider (HCP).
Explain the situation to the family.
Explanation:
Question 5 See full question
After many years of advanced practice nursing, a nurse has recently enrolled in a nurse practitioner (NP) program.
This nurse has been attracted to the program by the potential after graduation to provide primary care for clients,
an opportunity that is most likely to exist in which of the following settings?
You Selected:
A long-term care facility.
Correct response:
A rural health center.
Explanation:
Question 1 See full question
As a client is being admitted to the facility, her husband asks the nurse why she must sign a statement confirming
that she has been told of her rights to communicate her wishes about life support and resuscitation. How should the
nurse respond?
You Selected:
"We make sure our clients know they have the right to specify advance directives and appoint someone to speak
for them."
Correct response:
"We make sure our clients know they have the right to specify advance directives and appoint someone to speak
for them."
Explanation:
Question 2 See full question
A client diagnosed with thyroid cancer signed a living will that states he doesn't want ventilatory support if his
condition deteriorates. As his condition worsens, the client states, "I changed my mind. I want everything done for
me." Which response by the nurse is best?
You Selected:
"What exactly do you mean by wanting 'everything' done for you?"
Correct response:
"What exactly do you mean by wanting 'everything' done for you?"
Explanation:
Question 3 See full question
A client tells the nurse that she has had sexual contact with someone whom she suspects has genital herpes. The
nurse should instruct the client to:
You Selected:
anticipate lesions within 25 to 30 days.
Correct response:
report any difficulty urinating.
Explanation:
Question 4 See full question
Which of the responsibilities related to the care of a client with a Foley catheter are appropriate for the nurse to
delegate to the unlicensed assistive personnel (UAP)? Select all that apply.
You Selected:
Provide Foley catheter and perineal care each shift.
Apply catheter-securing device to the client's leg.
Empty drainage bag, and record output at specified times.
Correct response:
Empty drainage bag, and record output at specified times.
Apply catheter-securing device to the client's leg.
Provide Foley catheter and perineal care each shift.
Ensure the urine drainage bag is below the level of the bladder at all times.
Explanation:
Question 5 See full question
Nurses who provide care in a large, long-term care facility use charting by exception (CBE) as the preferred method
of documentation. This documentation method may have which of the following drawbacks?
You Selected:
Failure to identify and record problems and associated interventions.
Correct response:
Vulnerability to legal liability because the nurse's safe, routine care is not recorded.
Explanation:
Question 1 See full question
Under which circumstance may a nurse communicate medical information without the client's consent?
You Selected:
wWhen treating the client with a sexually transmitted disease
Correct response:
wWhen treating the client with a sexually transmitted disease
Explanation:
Question 2 See full question
A surgeon is discussing surgery with a client diagnosed with colon cancer. The client is visibly shaken over the
possibility of a colostomy. Based on the client's response, the surgeon should collaborate with which health team
member?
You Selected:
Clinical educator
Correct response:
Enterostomal nurse
Explanation:

According to Maslow's hierarchy of c) Risk for


needs, which nursing diagnosis has body image
the lowest priority for a client disturbance
admitted to the intensive care unit
with a diagnosis of congestive
heart failure?

a) Ineffective coping
b) Impaired urinary elimination
c) Risk for body image disturbance
d) Ineffective airway clearance

2. After assessing a client, a nurse identifies the b) Actual


nursing diagnosis, "Ineffective Airway
Clearance related to thick tracheobronchial
secretions." The nurse would classify this
nursing diagnosis as which type?

a) Risk
b) Actual
c) Possible
d) Wellness
3. A client admitted for a surgical procedure tells b) Risk for
the nurse, "I am very worried because I am allergy
allergic to latex. I want to make sure that response
everyone knows this." In order to assure the related to
safety of the client, what nursing diagnosis latex allergy
would the nurse address?

a) Knowledge deficit related to surgical


procedure
b) Risk for allergy response related to latex
allergy
c) Risk for injury related to latex allergy
d) Anxiety related to surgical procedure
4. A client has just been taught about lowering b) Ask direct
cholesterol with diet and exercise. What is the questions
best way to evaluate that the client about the
understands the material? teaching plan.

a) Accept silence as client understanding.


b) Ask direct questions about the teaching
plan.
c) Allow the client to discuss personal issues.
d) Redirect conversation to the topic.
5. A client is being admitted from the emergency b) Ineffective
room with complaints of shortness of breath, airway
wheezing, and coughing. Which of the clearance
following would the nurse as an appropriate
nursing diagnosis?
a) Bronchial pneumonia
b) Ineffective airway clearance
c) Asthma attack
d) Acute dyspnea
6. The client's expected outcome is "The client c) Condition of
will maintain skin integrity by discharge." the skin over
Which of the following measures is best in bony
evaluating the outcome? prominences.

a) The client's ability to reposition self in bed.


b) Percent intake of a diet high in protein.
c) Condition of the skin over bony
prominences.
d) Pressure-relieving mattress on the bed.
7. A client with diabetes mellitus has been a) Ineffective
admitted to the hospital in diabetic health
ketoacidosis. During the admission maintenance
assessment of the client, the nurse learns that related to
the client is not following the prescribed client's denial
therapeutic regimen. The client states, "I don't of illness
really have diabetes. My doctor overreacts."
What is the most appropriate diagnosis for this
client's health problem?

a) Ineffective health maintenance related to


client's denial of illness
b) Risk for injury related to client's
mismanagement of disease
c) Ineffective coping related to client's inability
to manage the diabetic regimen
d) Risk for unstable blood glucose related to
client's reluctance to manage the diabetic
regimen
8. Implementation of the plan of care is most d) the nurse
successful when: includes family
members and
a) the nurse avoids further collecting of data other
until the evaluation phase. healthcare
b) the nurse takes on care and decision professionals.
making for the patient.
c) the nurse recognizes documentation will
occur during another phase.
d) the nurse includes family members and
other healthcare professionals.
9. The nurse develops long- b) Client returns home
term and short-term verbalizing an understanding
outcomes for a client of contributing factors,
admitted with asthma. medications, and signs and
Which of the following is an symptoms of an asthma
example of a long-term attack.
goal?

a) Within 72 hours of
admission, the client's
respiratory rate returns to
normal and retractions
disappear.
b) Client returns home
verbalizing an
understanding of
contributing factors,
medications, and signs and
symptoms of an asthma
attack.
c) By day 3 of
hospitalization, the client
verbalizes knowledge of
factors that exacerbate the
symptoms of asthma.
d) Within one hour of a
nebulizer treatment,
adventitious breath sounds
and cough are decreased.
10. The nurse has administered b) Document the
pain medication to a client effectiveness of the
with a fractured femur. One intervention.
hour later, the client reports
relief of pain. What is the
nurse's next action?

a) Plan to decrease the pain


medication next time.
b) Document the
effectiveness of the
intervention.
c) Instruct the client to use
imaging and slow breathing.
d) Instruct the client to wait
as long as possible to ask
for medication.
11. The nurse has instructed the c) Reassess the
client in self-catheterization, appropriateness of the
but the client is unable to method of instruction.
perform a return
demonstration. What is the
nurse's most appropriate
plan of action?

a) Revise the plan to include


the inclusion of a support
group.
b) Report the client's
inability to learn to the case
manager.
c) Reassess the
appropriateness of the
method of instruction.
d) Teach the content again
utilizing the same method.

1 The nurse is caring for a client admitted to the hospital for renal calculi. a) Assess for bladder
2. What is the best action to take first? distention.

a) Assess for bladder distention.


b) Force fluids by mouth.
c) Diet as tolerated.
d) Strain urine after each void.
1 The nurse is discussing diabetes mellitus with the family members of a c) Risk factors and
3. client recently diagnosed. In order to promote the health of the family prevention of diabetes
members, what would be the most important information for the nurse to mellitus
include?

a) Medications used to treat diabetes mellitus


b) The cellular metabolism of glucose
c) Risk factors and prevention of diabetes mellitus
d) The severity of the client's disease
1 A nurse is explaining the purpose of nursing diagnoses to a client. What c) "Nursing diagnoses are
4. would be the most appropriate statement for the nurse to make? used to guide the nurse in
selecting appropriate
a) "Nursing diagnoses are used to bill insurance for nursing care." nursing interventions."
b) "Nursing diagnoses are necessary to schedule the amount of nursing
care required by the client."
c) "Nursing diagnoses are used to guide the nurse in selecting appropriate
nursing interventions."
d) "Nursing diagnoses are necessary to validate the medical diagnosis."
1 The nurse is preparing a client with a bowel obstruction for emergency a) Inform the client what to
5. surgery. Of the following interventions, which has the highest priority? expect after the surgery.

a) Inform the client what to expect after the surgery.


b) Discuss discharge plans with the client.
c) Instruct the client and family in wound care.
d) Teach the client about dietary restrictions during recovery.
1 The nurse is preparing to administer a blood pressure medication to a b) Assess the client's blood
6. client. To ensure the client's safety, what is the priority action for the pressure to determine if the
nurse to take? medication is indicated.

a) Tell the client to report any side effects experienced.


b) Assess the client's blood pressure to determine if the medication is
indicated.
c) Determine the client's reaction to the medication in the past.
d) Ask the client to verbalize the purpose of the medication.
1 A nurse manager notes an increase in the frequency of client falls during a) Investigate the
7. the last month. To promote a positive working environment, how would circumstances that
the nurse manager most effectively deal with this problem? contributed to client falls

a) Investigate the circumstances that contributed to client falls


b) Reprimand the nursing personnel responsible for the clients when the
falls occurred
c) Institute a new policy on the prevention of client falls on the unit
d) Determine if client falls have increased on other nursing units in the
hospital
1 The nurse overhears two nursing students talking about nursing a) "Nursing interventions
8. interventions. Which statement by one of the nursing students indicates must be approved by other
further education is required? members of the health care
team."
a) "Nursing interventions must be approved by other members of the
health care team."
b) "Nursing interventions must be consistent with standards of care and
research findings."
c) "Nursing interventions must be compatible with other therapies
planned for the client."
d) "Nursing interventions must be culturally sensitive and individualized
for the client."

1 Nursing interventions for the client after prostate surgery include assisting the d) Revise the care plan to
9 client to ambulate to the bathroom. The nurse concludes that the client no allow the client to ambulate
. longer requires assistance. What is the nurse's best action? to the bathroom
independently.

a) Continue assisting the client to the bathroom to ensure the client's safety.
b) Instruct the client's family to assist the client to ambulate to the bathroom.
c) Consult with the physical therapist to determine the client's ability.
d) Revise the care plan to allow the client to ambulate to the bathroom
independently.
2 Of the following types of nursing diagnoses, which one is validated by the c) Actual nursing diagnosis
0 presence of major defining characteristics?
.
a) Wellness diagnosis
b) Possible nursing diagnosis
c) Actual nursing diagnosis
d) Risk nursing diagnosis
2 One hour after receiving pain medication, a postoperative client complains of b) Assess the client to
1 intense pain. What is the nurse's most appropriate first action? determine the cause of the
. pain.
a) Discuss the frequency of pain medication administration with the client.
b) Assess the client to determine the cause of the pain.
c) Assist the client to reposition and splint the incision.
d) Consult with the physician for additional pain medication.
2 A preceptor reviews the client outcomes written by a new nurse. Which outcome a) Within 3 days, client will
2 is the highest priority for the client with paranoid delusions? mingle in the day room
. without violence.
a) Within 3 days, client will mingle in the day room without violence.
b) Within 2 days, client will perform personal hygiene without reminders.
c) Client will verbalize side effects of antipsychotic medications within 24 hours.
d) Client will discuss delusions in therapy sessions before discharge.

PrepU Chp.38 End-of-Life Care


Study online at quizlet.com/_1e894r

1. According to the Harvard University Medical School committee, what function must be c) Brain function
irreversibly lost to define death?

a) Consciousness
b) Respiratory functions
c) Brain function
d) Reflexes
2. The admission department at a local hospital is registering an elderly man for an b) Advanced
outpatient test. The admissions nurse asks the man if he has an advanced directive. directives are
The man responds that he does not want to complete an advanced directive because limited only to
he does not want anyone controlling his finances. What would be appropriate healthcare
information for the nurse to share with this patient? instructions and
directives.
a) Advanced directives are not legal documents, so you have nothing to worry about.
b) Advanced directives are limited only to healthcare instructions and directives.
c) His finances cannot be managed without an advanced directive.
d) Advanced directives are implemented when you become incapacitated and then
you will use a living will to allow the state to manage your money.
3. An appropriate nursing diagnosis for the family of a client dying of cancer, whose d) Anticipatory
members have expressed sorrow over the forthcoming loss, would be ... grieving related
to loss of family
a) Potential for grieving related to loss of family member and sorrow member, as
b) Dysfunctional grieving related to the loss of family member, as manifested by evidenced by
behaviors indicating anxiety sorrow
c) Dysfunctional grieving related to future loss of family member, manifested by
family's developmental regression
d) Anticipatory grieving related to loss of family member, as evidenced by sorrow
4. The client is a young mother whose spouse died 3 months ago. The client is tearful b) Ineffective
and unkempt, eats a poor diet, and has lost 50 pounds since the death of the Coping related to
spouse. The client states, "I can't do this anymore." The nursing diagnosis best failure of
supported by these data is previously used
coping
a) Death Anxiety related to death of spouse. mechanisms.
b) Ineffective Coping related to failure of previously used coping mechanisms.
c) Ineffective Denial related to poor grief resolution.
d) Decisional Conflict related to inability to progress following spouse's death.
5. A client is diagnosed with a terminal illness. Who is usually responsible for deciding a) Physician
what, when, and how the client should be told?

a) Physician
b) Nurse
c) Clergy
d) Family
6. A client severely injured in a motor vehicle accident is rushed to the health care b) Breathing
facility with severe head injuries and profuse loss of blood. Which of the following becomes noisy
signs indicates approaching death?

a) Client is calm and peaceful


b) Breathing becomes noisy
c) Frequency of urination decreases
d) Arms and legs are warm to touch
7. A client states, "My children still need me. Why did I get cancer? I am only 30." This a) Anger
client is exhibiting which stage according to Kbler-Ross?

a) Anger
b) Acceptance
c) Denial
d) Bargaining
8. A dying patient is crying. She states, "I can't pray. I can't forgive myself." What would d) Spiritual
be an appropriate nursing diagnosis based on this data? Distress

a) Noncompliance
b) Knowledge Deficit
c) Low Self-Esteem
d) Spiritual Distress
9. The emergency department (ED) nurse accepts an unconscious client brought in by c) Obtain contact
ambulance. The client's family presents a durable power of attorney for health care information for the
for the client. Which of the following actions should the nurse take? person designated
to make decisions
a) Initiate active euthanasia for the client
b) Initiate a slow code in the case of cardiopulmonary or respiratory arrest
c) Obtain contact information for the person designated to make decisions for the
client
d) Communicate to other ED staff that there should be no attempts to resuscitate
the client
1 In the United States, what belief is the hospice movement based on? b) Meaningful
0. living during
a) Meaningful living during terminal illness is best supported in designated facilities. terminal illness is
b) Meaningful living during terminal illness is best supported in the home. best supported in
c) Meaningful living during terminal illness is meant to prolong physiologic dying. the home.
d) Meaningful living during terminal illness requires technologic interventions.
1 A man is diagnosed with terminal kidney failure. His wife demonstrates loss and b) Anticipatory loss
1. grief behaviors. What type of loss is the wife experiencing?

a) Maturational loss
b) Anticipatory loss
c) Bereavement
d) Dysfunctional grieving
1 "My father has been dead for over a year and my mother still can't talk about him a) "The inability to
2. without crying. Is that normal?" What is the best response by the nurse? talk about your
dad without crying,
a) "The inability to talk about your dad without crying, even after a year, is still even after a year,
considered normal." is still considered
b) "Everyone deals with loss differently. You just need to be patient with your normal."
mother."
c) "It is not normal. Your mother needs to see a therapist about her grief."
d) "Did your mother cry a lot before your father died?"

1 The nurse is assessing a client who was diagnosed with metastatic prostate cancer. c) Depression
3. The nurse notes that the client is exhibiting signs of loss, grief, and intense sadness.
Based upon this assessment data, the nurse will document that the client is in what
stage of death and dying?

a) Acceptance
b) Denial
c) Depression
d) Anger
1 The nurse is caring for a client who recently found out he has a terminal illness. The a) "Sometimes
4. nurse notes that the client is hostile and yelling. Which of the following statements by a person
the nurse shows that she has understanding of the Kbler-Ross emotional responses to returns to a
impending death? previous
stage."
a) "Sometimes a person returns to a previous stage."
b) "The duration of all stages is a few hours."
c) "Each stage of dying must be completed prior to moving to the next stage."
d) "The process is the same from person to person."
1 A nurse is caring for a terminally ill client who refuses to have food due to an inability c) Pulverize
5. to swallow solid food. Which of the following nursing interventions should the nurse food items
adopt to promote nutrition in the client?

a) Provide small servings


b) Provide nasogastric feed
c) Pulverize food items
d) Serve food attractively
1 The nurse is providing care to a group of terminally ill clients. The client who is most b) "Why did
6. likely experiencing the anger stage of grief is the one who states this have to
happen to me?"
a) "I don't care about anything. I have no energy."
b) "Why did this have to happen to me?"
c) "I just want to see my son have a family of his own."
d) "I do not believe I have this disease."
1 The psychiatrist is evaluating a client who has recently learned she has a terminal c) "Just let me
7. illness. Which of the following statements indicates to the psychiatrist that the client go on vacation
is in the Kbler-Ross stage of bargaining? with my wife;
then I'll be
a) "I waited years to see my grandchildren and now I won't see them." satisfied."
b) "I know that my family will be taken care of. I am at peace."
c) "Just let me go on vacation with my wife; then I'll be satisfied."
d) "Why is this happening to meI quit smoking."
1 A terminally ill client complains of excruciating pain in the back. Which of the d) Administer
8. following interventions should the nurse adopt to control pain in the client? narcotics

a) Provide skin care


b) Provide regular mouth care
c) Encourage family visits
d) Administer narcotics
1 A terminally ill client is being cared for at home and receiving hospice care. The b) Difficulty
9. hospice nurse is helping the family cope with the client's deteriorating condition, swallowing
educating them on the signs of approaching death. Which of the following would the
nurse include in this education plan?

a) Increased sensory stimulation


b) Difficulty swallowing
c) Decreased pain
d) Increased urinary output
2 A terminally ill client states to the nurse, "My situation is hopeless; I have no control a) Encourage
0. over anything." The nurse implements which of the following interventions to enable the client to
hope for the client? discuss his
feelings.
a) Encourage the client to discuss his feelings. b) Hold the
b) Hold the client's hand. client's hand.
c) Sit in a chair next to the client. c) Sit in a chair
d) State to the client, "We have explored all treatment options." next to the
e) Withhold information about disease progression. client.

2 What is the most important goal of care for the a) Providing a comfortable, dignified
1 dying client who is receiving comfort care? death
.
a) Providing a comfortable, dignified death
b) Using a feeding tube to provide nutrition
c) Ensuring family members are present at the
bedside
d) Identifying appropriate coping mechanisms

2 When preparing for palliative care with the a) "The goal of palliative care is to give
2 dying client, the nurse should provide the clients the best quality of life by the
. family with which explanation? aggressive management of symptoms."

a) "The goal of palliative care is to give clients


the best quality of life by the aggressive
management of symptoms."
b) "In palliative care, no attempts are to be
made to resuscitate a client whose breathing or
heart stops."
c) "Palliative care is the gradual withdrawal of
mechanical ventilation from a client with
terminal illness and poor prognosis."
d) "The client will have to go to an inpatient
hospice unit in order to receive palliative care."

2 Which of the following manifestations of grief d) Leaving the wife's room and
3 by the client who lost his wife three years belongings intact
. earlier is considered abnormal?

a) Showing a photograph of the decedent


b) Talking about his wife's absent-mindedness
c) Telling the nurse how his life has changed
d) Leaving the wife's room and belongings
intact

2 Which of the following phrases can do much to d) "Let me tell you about your illness."
4 instill hope in the dying patient?
.
a) "Everything will be fine, so don't worry."
b) "This is a hopeless situation."
c) "Nothing more can be done."
d) "Let me tell you about your illness."

2 Which of the following situations is most likely b) A client's death involves an


5 to warrant an autopsy? allegation of a medical error.
.
a) A palliative client dies unwitnessed during
the night.
b) A client's death involves an allegation of a
medical error.
c) A client dies after unsuccessful
cardiopulmonary resuscitation.
d) A client's death is attributed to an infectious
disease.

PrepU Chp.8 Client


Teaching

Das könnte Ihnen auch gefallen